Return Home Members Area Experts Area The best AskMe alternative!Answerway.com - You Have Questions? We have Answers! Answerway Information Contact Us Online Help
 Wednesday 8th May 2024 04:17:21 AM


 

Username:

Password:

or
Join Now!

 
These are answers that tomder55 has provided in Politicsrel=

Question/Answer
paraclete asked on 06/12/09 - 'Overrun by hordes of non-Whites and mongrels'

http://www.smh.com.au/world/overrun-by-hordes-of-nonwhites-and-mongrels-20090612-c5e6.html

Whilst it is a very offensive thought in a PC world, Von Brunn's outburst should give us all cause to think about the shape of our societies in a world run by UN decree.

In Australia we have had Indian students, quests in our country, demonstrating in the streets because another non-white non-indigenous group has made them the target of their criminal activity and the very real question I have is when did we invite either of these groups to come here? On the one hand we have allowed displaced persons to find a new home and what do they do, trash the place and try to convert it into the hell hole they came from, engaging in all manner of illegal activity, and we have another group who think that their particular type of political activism so favoured by their countrymen will be permitted here in our quite urban streets.

I don't want my country overrun by hordes of non-whites and mongrels. It's not a racist position but a position that peoples who don't integrate and take on the values of the society in which they live, arn't welcome, no matter how bad conditions are in their homeland. We have had it here with all manner of people who have imported their violent ideas and ideologies and who continue to fight their old wars on our streets whether it be Serbs, Croats, Tamils, Lebanese, Somalis, Thibetans and now Indians, we say please go home.

tomder55 answered on 06/13/09:

Having locust problems again ? The US has a simular problem . It stems from a change in the social contract .

We used to assimilate. The melting pot assumed that immigrants would adopt the society and we would absorb the best their culture has to offer.(mostly hard work ,morality and good food) .
But now the prevailing theory is a multi-cultural patchwork quilt of ethnic pockets each isolated from the other and celebrating the society they left ,not the country they have migrated to.

It will be the destruction of our country and no doubt yours.

paraclete rated this answer Excellent or Above Average Answer

Question/Answer
paraclete asked on 05/15/09 - What a difference a few thousand years makes?

I read recently where it is believed that the Pushtun people of Pakistan and Afghanistan are a lost tribe of Israel, a people displaced from Palistine by the Babylonians and Assyrians centuries ago. So the world watches and even applauds while the US and Pakistan battle these ideologically challenged people who truely think that their way is the right way. If the sort of campaign were being waged against jews in Palistine as is being waged in Pakistan then it would have world condemnation. But what is the difference here, it is religion. These people are on the wrong side of the religious divide, even though racially they may actually be jewish or should I even use the term semetic. It would seem the Pakistani campaign is anti-semetic. How difficult is it to discern where all of this fits today?

tomder55 answered on 05/20/09:

then again ;I wonder how the world would react if Israel took a lesson from Sri Lanka's handling of the Tamil Tigers this week .

I don't hear a peep of protest over the heavy handed way they dealt with the terrorists.

You use the words Pashtun . But we aren't fighting the Pashtun . We fight the TALIBAN who are predominantly Pashtun. Now you see the way the Taliban treat the people they control ? Their own people ? If they are Jewish then so was I after my circumcision.

paraclete rated this answer Excellent or Above Average Answer

Question/Answer
paraclete asked on 03/27/09 - Shades of 1975?

China bankrolling Kevin Rudd's stimulus plan by buying government bonds

By Steve Lewis

The Courier-Mail

March 27, 2009 12:00am


* China buying Australian Government bonds
* May make China our biggest lender
* Opposition says we're handcuffed to China

CHINA is secretly helping to bankroll Kevin Rudd's economic rescue plan as concerns grow over the relationship between the Communist superpower and the Labor Government.

The Courier-Mail can confirm that China is a significant investor in Australian government bonds -- used by Canberra to fund billions of dollars in emergency spending.

Market insiders believe China is buying 15 to 20 per cent of the $2 billion in Treasury securities being issued every week.

This would make China the single biggest lender to Australia, although details of who owns the bonds are cloaked in secrecy.

The program, authorised by Treasurer Wayne Swan, will leave Australia with a debt bill approaching $200 billion.

In response, the Opposition has raised concerns Australia could end up politically "handcuffed" to China as a result. China's appetite for Australian bonds comes just days after the Prime Minister secretly met China's fifth most powerful figure, Li Changchun, at the Lodge.

So not content with speaking Chinese the PM is getting into bed with them. The Labor Party is slow to learn. The Kemlani loans affair brought down the Labor government in 1975 and Australians have long memories, the Chinese loans affair could bring down the Labor government in 2011 it won't be Kevin in eleven

tomder55 answered on 04/07/09:

The problem isn't so much who funds massive increases in debt . It is the flawed theories of Keynes that thinks massive spending can get you out of recession.

labman rated this answer Excellent or Above Average Answer
paraclete rated this answer Excellent or Above Average Answer

Question/Answer
paraclete asked on 04/03/09 - He Just Couldn't Help Himself?

Barack Obama is on the nose for bowing the the Saudi King, particularly as he didn't show the same deference to the Queen(UK). Someone should instruct this "boy" on the correct protocol or keep the Obama's on a leash as Mr's Obama's snuggle with the Queen didn't go unnoticed. The American President is either the equal of any head of state or he is not, but he lowers the status of the US Presidency by forelock tugging with a foreign monarch, particularly a Muslim princeling.

The British people should rightly feel insulted by this turn of events and the American people feel demeened by them. The US is now a third class power. After eight years of Bush's gaffe's surely they could hope for better?

tomder55 answered on 04/07/09:

Presidents do not bow to Royalty PERIOD !!!!

He is a disgrace. The fact that he showed such deference to a Muslim leader makes one wonder about his loyalties. When he assumed the position King Abdula reportedly said "turn around so we can speak face to face ". He showed a complete lack of knowlege or interest in proper eticate . He brings with him Walmart like gifts for world leaders.

His 'suck up to the European and apologize to the world tour' is making me physically ill.


Here's a clue for you" Mr President "....

When President Bush was so called confrontational and showed genuine American leadership he was reelected . When he became more accomodationalist in his second term ;his party lost the White House.

He gave up on all key points at the G-20 . He insulted his own country in France . He got only token concessions at the NATO summit ;was smacked down by Sarkosy over Turkish admission into the EU . He made a rediculus speech about eliminating nukes as the NORKS launched a missile for a backdrop.

Then he goes to Turkey to affirm American dhimminitude to the umah.

labman rated this answer Excellent or Above Average Answer
paraclete rated this answer Excellent or Above Average Answer

Question/Answer
paraclete asked on 04/04/09 - Krudd is just an angry little man?

Evidence is emerging that our esteemed PM hasn't got it all together, that, in fact; the Krudd has spread to employee relations with few able to withstand his outbursts. I thought this idiot was a trained diplomat but his employee relations are far from diplomatic. Krudd is just a little Hitler, a small man in a big role and it is obviously too much for him. So much for his worldly aspirations, if he cannot master the local scene he won't get a gernsey at the UN

tomder55 answered on 04/07/09:

Why do you care what the UN thinks ?

As for Rudd he is your typical flaming lib . Nationalize what he can get away with (broadband infrastructure ) and hate the military (manifesting itself by bullying a flight attendant) . He appeases the Chinese as they hack into his computer .Spending Chinese money like crazy in a massive spending stimulus. He's joined at the hip with that idiot Al Gore on enviro-wacko theories. All while bleating about the dangers of the "Right" .

labman rated this answer Excellent or Above Average Answer
paraclete rated this answer Excellent or Above Average Answer

Question/Answer
Mathatmacoat asked on 03/02/09 - All is not well in Camelot

http://www.smh.com.au/world/talkback-king-leads-backlash-to-obama-20090302-8mew.html

Seems the gloss has gone off Obama's armour now that he is raising taxes and the barbarians represented by the republicans are out to get him. So much for his one hundred days in the sun. How dare he suggest that those who caused the crash bear some of the pain of fixing it?

tomder55 answered on 03/03/09:

Camelot ? we just elected Mordred . Camelot is going to be destroyed by this Fabian community organizer .

Barbarians are people who would tolerate the confiscation of over 50% of a persons wealth for the greater good of the state . That is servitude by any logic. What Obama is constructing rapidly is a feudal state .

Yeah ;populist mumbo-jumbo makes "the rich " easy targets . But the thirst of his ilk is insatiable . The middle class he champions will be his victims down the road. Just like what always happens when Marxist scumbags take over a country .

Mathatmacoat rated this answer Excellent or Above Average Answer

Question/Answer
Mathatmacoat asked on 03/02/09 - Why are people so bad in the USA?

It has been revealed that the US has the highest prison population in the world and the only other developed nation that comes even close is Russia.

http://www.kcl.ac.uk/depsta/law/research/icps/downloads/world-prison-pop-seventh.pdf

This begs the question; how can a country which claims to be the champion of freedom and human rights have 25% of the world's prisoners.

What is so bad about American lifestyle that it lands such a high percentage of the population in jail? Nor is the problem evident in the US close neighbours. For the US to have more prisoners than Russia it must be a very repressive regime indeed. One can only conclude that as people are pretty much the same everywhere it must be the socio-political system which creates the laws under which the people are imprisoned which is at fault.

Clean up your act america because we don't want this exported to us

tomder55 answered on 03/03/09:

We are experiencing an invasion that is the best explanation I can give. The U.S. Justice Department estimated that 270,000 illegal immigrants served jail time nationally in 2003.Roughly 17 percent of the prison population at the federal level are illegal aliens. That's a huge number since illegal aliens only account for about 3 percent of the total population.40% of the U.S. federal prison population is composed of non-citizens both here legally and illegally .

The other problem is systematic. Lawmakers trying to appear tough on crime passed mandatory sentencing laws for repeat crimes . In many cases that means minor crimes like marijuana possession endos up doing time if the person had done another crime previously. That clearly needs reform.

But the biggest problem here is the invasion from the South. Look at the situation on the border with Mexico . That area of Mexico is almost lawless with drug cartels battling the Mexican military for control . Much of that spills across the border and there are too mny people in this country that not only tolerate but also encourage the endless wave of illegala into the country.

labman rated this answer Excellent or Above Average Answer
Mathatmacoat rated this answer Excellent or Above Average Answer
paraclete rated this answer Excellent or Above Average Answer

Question/Answer
Mathatmacoat asked on 03/02/09 - Why are people so bad in the USA?

It has been revealed that the US has the highest prison population in the world and the only other developed nation that comes even close is Russia.

http://www.kcl.ac.uk/depsta/law/research/icps/downloads/world-prison-pop-seventh.pdf

This begs the question; how can a country which claims to be the champion of freedom and human rights have 25% of the world's prisoners.

What is so bad about American lifestyle that it lands such a high percentage of the population in jail? Nor is the problem evident in the US close neighbours. For the US to have more prisoners than Russia it must be a very repressive regime indeed. One can only conclude that as people are pretty much the same everywhere it must be the socio-political system which creates the laws under which the people are imprisoned which is at fault.

Clean up your act america because we don't want this exported to us

tomder55 answered on 03/03/09:

We are experiencing an invasion that is the best explanation I can give. The U.S. Justice Department estimated that 270,000 illegal immigrants served jail time nationally in 2003.Roughly 17 percent of the prison population at the federal level are illegal aliens. That's a huge number since illegal aliens only account for about 3 percent of the total population.40% of the U.S. federal prison population is composed of non-citizens both here legally and illegally .

The other problem is systematic. Lawmakers trying to appear tough on crime passed mandatory sentencing laws for repeat crimes . In many cases that means minor crimes like marijuana possession endos up doing time if the person had done another crime previously. That clearly needs reform.

But the biggest problem here is the invasion from the South. Look at the situation on the border with Mexico . That area of Mexico is almost lawless with drug cartels battling the Mexican military for control . Much of that spills across the border and there are too mny people in this country that not only tolerate but also encourage the endless wave of illegala into the country.

labman rated this answer Excellent or Above Average Answer
Mathatmacoat rated this answer Average Answer
paraclete rated this answer Excellent or Above Average Answer

Question/Answer
paraclete asked on 02/25/09 - It's time for America to lead again?

So Omama thinks that more of the same is a good recipe. The rest of us say it's time for America to get out of the way and let someoneelse lead for a change. Let's ask ourselves, where have they led us excepting down a dirty great hole?

They led us into a war in Iraq and where did that get us, I see devalued currency and high fuel prices and a ruptured economy.

They led us into a banking free for all which under the guise of providing homes for more people actually made more people destitute.

Now after years of sticking their head in the sand they want to lead us into an ecologically sustainable future. What I say is not on your nelly, we don't want solutions which are good for American business. So they invented solar energy, did they? They only deploy it when it suits them. We don't need a corn led recovery Mr. Obama, we need America to dismantle it's military industrial complex and find peaceful industries and stop selling arms to the world, Then we might believe you

tomder55 answered on 02/25/09:

the bloom coming off of hopenchange so quicky ?

Obama doesn't have the ability to lead a platoon up an ant hill so don't worry about him.

He's leading us down the path of subservience to an "international order " of unelected bureaucrats in the Hague and the halls of the UN .

But they will still need America's military might. It's just that our poor troops will don blue helmets and suddenly all interventions will be moral .

labman rated this answer Excellent or Above Average Answer
Mathatmacoat rated this answer Excellent or Above Average Answer
paraclete rated this answer Excellent or Above Average Answer

Question/Answer
Mathatmacoat asked on 02/02/09 - Neo-liberalism is dead?

It seems that yet another politician has decided that the debate is over. In his recent dissertation Mr Krudd has decided that neo-liberalism is dead

http://www.smh.com.au/news/opinion/gerard-henderson/rudd-neoliberal-with-the-facts/2009/02/02/1233423132293.html

What I have to say is it is going to take more than asperational comments from Rudd who stayed home from Davos to polish and deliver his oration to kill off rampant greed on Wall Street. I personally like the idea, I think floated by Obama, that no CEO should be paid more than the President, that would take down some US high flyers, and if applied in Australia would certainly send a few imported CEO's back where they came from, which considering the damage done here wouldn't be too soon.

One wonders what comes after neo-liberalism, If one listens to Rudd it is neo-interventalism, a system in which government intervenes to smooth out the bumps in the cycle. In fact, I think Keven would elimate the bumps in the cycle. Boring, Mr Rudd, Boring, and it smacks just a little of the controlled economies of the soviet era. I think you may have learned a little too much from your stay in China

So what should the post finacial debacle of the early twenty-first century look like?
A tightly regulated banking sector?
Free spending governments poring your money into "infrastructure" with every second worker in construction?
Monolithic national projects building railways thru nowhere to nowhere, thermal generation and solar generation in the vast interior, has the snowy project taught no one any lessons?
Social engineering programs like insulation of every house? every student with a computer and a volunteer job at the end of university? high speed broadband in every home? homes for the homeless? hospitals in federal hands? the nationalisation of the Murray-Darling river system?

tomder55 answered on 02/11/09:

I can go along with the stick it to the rich populist pablum . I'm going to keep my eye on the flood of high end properties that will plummet in value when the fat cats dump their houses on the market. That should help the Real Estate market !

I plan on seeing what Sotheby has on auction ;maybe one of those Mercedes can be picked up for the price of a new Chevy .

It won't do a thing to fix the economy but that's not what is at issue. Let's rally behind the populist demagogues of our times !! Stick it to them !!!! It won't help you or me . But we'll feel good about it . Just like Madame Defarge did as she knitted the names of her victims before they were guillotined !!!

Once their salaries are capped of course they will leave these jobs and move on to investment banks not affected like Lazard. We'll see what type of talent replaces them .

These banks will linger on like zombies ,sucking more life support from the gvt to stay afloat while the best people in the field go elsewhere. We won't let them die and we set up the scenario where the most talented managers go somewhere else.

But what will be left will be pure government bureaucracy bankers who will run their banks at least as well as Amtrack runs railroads and the Post Office runs mail delivery.(fyi they are nightmares of inefficiency)

No ;neo-liberalism is alive and well in the United States . What Obama and the Congress is doing goes far beyond neo-Keynsian theory.We need a new name for the excesses . Perhaps Hugo Chavez economics for the wholesale take over of the national economy is applicable .

Mathatmacoat rated this answer Excellent or Above Average Answer

Question/Answer
dublin40 asked on 01/16/09 - Harry Truman's U.S. army pension

We know that Pres. Truman was in the army during WW1. Why did he receive an army pension of $13.507.72 per year?

Thank you

tomder55 answered on 01/26/09:

it was the going rate ? he served ..he earned it.

The reason it came up was prior to Truman former Presidents did not get pensions for serving as President . Truman was living on his war time pension in 1958 and pleaded poverty .

Congress passed the President's pension and awarded Truman $25,000 annually ;benefits and a staff.

Today that perk has balloned of course. Their yearly salary pension is $191,000. Aside from that, each gets a staff that costs the taxpayer, $96,000 per president.

Former President Bill Clintoon has office space in Harlem that he rents for $516,000/year from tax money . President GHW Bush spends $69,000 a year on "equipment"
President Jimmy Carter spends $83,000 a year on "other services". Clintoon seems to spend the most across the board. His phone bill from 2006 cost taxpayers $104,000. We also pay for the satellite TV in his office, complete with eight separate receivers and all the movie channels that come with the "entertainment package".$1,800 per year.

Now we are adding another former President to the perpetual dole . I agree with the pension and some staffing ... The rest should be cut from the budget .

ChefGeno rated this answer Excellent or Above Average Answer
dublin40 rated this answer Average Answer

Question/Answer
paraclete asked on 12/24/08 - now they are calling him Kyoto Kev????

http://www.theaustralian.news.com.au/business/story/0,28124,24820880-36418,00.html

Some think it will be easy to reach a 60% reduction in emissions by 2050 and applaud Kev's modest targets of 5-15% reduction against 1990 by 2020. With enegry emissions growing at 1% a year just putting the brakes on and going for 0 growth will be great achievement let alone achieving a 1.5% reduction a year between now and 2050.

I'm waiting for Kyoto Kev to don the blue and red and with cape flying in the wind singlehandedly accomplish what no man has accomplished before, taking the Australian snout out of the energy trough

tomder55 answered on 12/26/08:

seems to me that a goal should be emission reductions without loss in economic output. no one addresses that . The US has acheived some real emission reductions in the last few months.But no one is praising it.

labman rated this answer Excellent or Above Average Answer
Mathatmacoat rated this answer Excellent or Above Average Answer
paraclete rated this answer Excellent or Above Average Answer

Question/Answer
paraclete asked on 12/24/08 - Now that's interesting?

apparently it has been discovered that you don't need dams to produce energy from flowing water. Using the principles of wind generation, underwater generators are now being installed in the Mississippi River. This is called hydrokenetic energy. Just shows; when you think about a problen you can find a solution
http://blog.wired.com/wiredscience/2008/12/hydrokinetic.html

tomder55 answered on 12/26/08:

"Verdant has spent more money on permitting their East River project that than they did on hardware,"....... Hydro Green's Stover hopes that his company's new unit will help shorten that regulatory process by generating environmental impact data that could ease concerns the turbines will disrupt river ecosystems and habitats.

This happens on all types of projects all the time.no matter how worthy they are . I'm delighted that they will buttheads with compatriot enviro-nut jobs .

wait until they find out that the spawning of the Mississippi River Catfish is disrupted by these turbines....

paraclete rated this answer Excellent or Above Average Answer

Question/Answer
Mathatmacoat asked on 12/17/08 - The slight of hand in emission statistics?

We cannot but wonder at why governments continue to quote emission reductions against 1990 benchmarks. In fact, this is slight of hand intended to conceal exactly how much pain will be inflicted on the average punter. Australia's declaration of a 5% reduction against 1990 benchmarks mean a reduction in average individual emissions of 34-41% it was revealed today. No modest reduction this but the most ambitious plan thus far, far exceeding any proposal in the EU or US to date.

What are we, a bunch of fools? More KRUDD and the greenies have the gaul to ask for more or they will migrate to Europe, well I say go. To paraphrase a well known phrase from Australian politics; well might we say God save the planet, because nothing will save the Barrier Reef. (Apologies to Gough Whitlam and the constitutional crisis of 1975). I see the Labor Party setting its-self up for another dismissal. If the opposition is true to its colours, this legislation is doomed. It is hardly the balanced approach suggested but the destruction of a successful economy for no reason

tomder55 answered on 12/19/08:

CNN Meteorologist: Manmade Global Warming Theory 'Arrogant'

“You know, to think that we could affect weather all that much is pretty arrogant,” Myers said. “Mother Nature is so big, the world is so big, the oceans are so big – I think we’re going to die from a lack of fresh water or we’re going to die from ocean acidification before we die from global warming, for sure.”



Myers is the second CNN meteorologist to challenge the global warming conventions common in the media. He also said trying to determine patterns occurring in the climate would be difficult based on such a short span.



“But this is like, you know you said – in your career – my career has been 22 years long,” Myers said. “That’s a good career in TV, but talking about climate – it’s like having a car for three days and saying, ‘This is a great car.’ Well, yeah – it was for three days, but maybe in days five, six and seven it won’t be so good. And that’s what we’re doing here.”



“We have 100 years worth of data, not millions of years that the world’s been around,” Myers continued.



Dr. Jay Lehr, an expert on environmental policy, told “Lou Dobbs Tonight” viewers you can detect subtle patterns over recorded history, but that dates back to the 13th Century.



“If we go back really, in recorded human history, in the 13th Century, we were probably 7 degrees Fahrenheit warmer than we are now and it was a very prosperous time for mankind,” Lehr said. “If go back to the Revolutionary War 300 years ago, it was very, very cold. We’ve been warming out of that cold spell from the Revolutionary War period and now we’re back into a cooling cycle.”



Lehr suggested the earth is presently entering a cooling cycle – a result of nature, not man.



Mathatmacoat rated this answer Excellent or Above Average Answer

Question/Answer
labman asked on 12/16/08 - Goverment in action

Lawmakers gather to discuss possible election to fill Obama's Senate seat — but instead vote to launch impeachment proceedings against Gov. Blagojevich |

tomder55 answered on 12/17/08:

At 1st I thought they had something .But I am becoming convinced that Fitgerald pulled the trigger too soon. I think if given a choice he would've waited until he had Guv Blago actually sealing a deal . But the Chitown Tribune ...acting like the responsible fifth column 4th estate they are ,was prepared to go public with a story they were holding onto since before the election. Fitz had no choice but the arrest him early ....probably blowing the case.

President-elect Obama conducted an intensive internal audit of the "Office of the President-elect " and cleared himself and his staff of any wrong-doing in the Pay-for-play-the-Chicagoway scandal .But since this is an ongoing investigation his team voted 1-0 to not release the results of the internal review of contacts with the Guv.We are assured that there were no "inappropriate" contacts between the Obama team and Blago;only appropriate contacts.

The Chitown Suntimes reports that that defense attorneys are getting calls from "victims" of Blago's pay-for-play schemes These "victims" are worried they are on the tapes and want to lawyer-up before Elliot Ness Fitzgerald comes knocking shotgun in arm.



Evidently this has been ongoing for years.

Question .. If Blago thought that the seat was gold worth mining now ;what was the seat worth the last time it was vacant in 2004 ? What was the quid pro quo to his partner in slime Tony Rezko and his protege of the Chicago way Barack Obama ?

For his part Blago has hired legendary"do anything to win" Ed Genson;previously defense lawyer for R.Kelly .


Does "anything to win" include leaking the details of Rahm Emanuel's negotiations ? I'd love to be a fly on the wall during discovery

labman rated this answer Excellent or Above Average Answer

Question/Answer
Mathatmacoat asked on 12/12/08 - How do you stimulate an economy?

Do you do it by plowing money into infrastructure projects with uncertain start dates, sounds like grandstanding to me! Do you do it by allowing business to defer tax payments? More grandstanding!
Do you do it by loaning money to institutions which continue to announce massive job cuts?

No you cannot stimulate an economy doing any of these things because they do nothing to provide certainty about the future.

What I ask is why can't politicians see that this isn't the time for grandstand gestures?

tomder55 answered on 12/12/08:

I agree . The moves made in Washington to address the economic down-turn have been revolting to me .

I think the Washington "bailouts " are the biggest upwards wealth transfers (theft ) in history . Suprisingly many of the Democrats who champion those unions and little people ...yada yada are supportive of these "bailouts ".Given their stated position ;as articulated nicely by their standard bearer -elect Barack Hussein Obama ,that the way to stimulate the economy is from the bottom-up ;it should suprise me that they have taken the lead in promoting this corporate welfare ...but it doesn't because I have known for years their hypocricy .
I would also note that it has been Republicans in Congress (those so called champions of the elites and robber barons ) who have been the only members of the Federal Government to attempt to put brakes and accountability into this madness.

labman rated this answer Excellent or Above Average Answer
Mathatmacoat rated this answer Excellent or Above Average Answer
paraclete rated this answer Excellent or Above Average Answer

Question/Answer
Mathatmacoat asked on 12/11/08 - God and bad economic policy?

when journalists write of government's bold new stimulus package, they look no further than the visible benefits and ignore the hidden costs. If a bridge it to be built, construction workers and steelmakers will no doubt benefit But there are hidden costs. In this new eeconomic environment we see a rush to promote infrastructure projects what are the consequences? The price of steel and construction wages will be higher, discouraging private investment. If the funds are to be borrowed in the marketplace, that will restrict funds available for private investment. And what will happen to unemployment when the infrastructure is completed?
The key to recovery is sustainable investment — not temporary jobs.
A further example of this was a stimulus package given to the solar industry by the Australian government created a boom in the industry, but change of government and change of policy has seen the industry going from boom to bust, where will the skilled workers be when the government again decides to turn on the tap as part of their new stimulus package, working on another infrastructure project?

Welfare Spending
If you give a man a fish he will have a single meal.
If you teach him how to fish, he will eat all his life.

~ Kuan-tzu, Taoist philospher (7th century BC).

comments?

tomder55 answered on 12/11/08:

if they let the productive people keep the fruits of their labor there would be instant recovery. Let the dead wood rot .no bailouts . there will be someone ready to pick up the pieces with a better way and the economy will be better off in the long run as a result.

labman rated this answer Excellent or Above Average Answer
Mathatmacoat rated this answer Excellent or Above Average Answer

Question/Answer
paraclete asked on 12/07/08 - Can we spend our way out of depression?

What is happening in the developed economies at the moment is compared with the Great Depression and the answer is suggested as a great spending spree on the part of both government and the people, but the problem isn't necessarily lack of demand, it's lack of finance, lack of confidence. Too many finance companies and banks have been burned by bad loans and as employment dries up so the availability of credit worthy borrowers dries up and the uncertainty grows so why lend to someone who may not have a job tomorrow?
In such an environment how can you spend your way out with infrastructure projects which take a long time to put in place? The stimulus is needed right now in the existing industries, not in industries which have to grow rapidly to meet a new demand.
The whole thing needs a radical change in thinking. To spend on consumer goods may actually mean supporting the job of someone in another country, not supporting the job of someone nearby.
So here is some radical thinking.
Governments should direct their spending to both fighting depression and fighting climate change by installing solar collectors on every building. I'm not talking about subsidising installation but actually owning the infrastructure so they can control what is actually done.
Governments should put every unemployed person immediately to work on community projects. Let's have the sense to realise that people need income and work not the soul destroying search for work in an economy that can't provide it. This may mean a great deal of concrete will get laid and many gardens planted but it is better. We need more white rocks and road verges mowed
Governments should immediately commence retraining programmes where trade and profession training is provided in existing educational institutions and no cost to the participant
so for the unemployed a system of national traineeships
two days work, two days training and one day to look for work

tomder55 answered on 12/11/08:

The current financial crisis is not—as some have said—a crisis of capitalism. It is in fact the opposite, a shattering demonstration that ill-considered government intervention in the private economy can have devastating consequences. The crisis has its roots in the U.S. government's efforts to increase homeownership, especially among minority and other underserved or low-income groups, and to do so through hidden financial subsidies rather than direct government expenditures. The story is an example, enlarged to an American scale, of the adverse results that flow from the misuse and manipulation of banking and credit by government. When this occurs in authoritarian regimes, we deride the outcome as a system of "policy loans" and note with an air of superiority that banks in these countries are weak, credit is limited, and financial crises are frequent. When the same thing happens in the United States, however, we blame "greedy" people, or poor regulation (or none), or credit default swaps, or anything else we can think of—except the government policies that got us into the disaster.



Read the rest here

paraclete rated this answer Excellent or Above Average Answer

Question/Answer
paraclete asked on 12/04/08 - Schools bans national anthem!

and they wonder why we don't like Muslims?
http://www.news.com.au/story/0,27574,24754060-1248,00.html
a school at the centre of a controversy to establish a new Muslim school has banned playing of the national anthem. Anywhere else in the world such people would be subject to an extreme reaction but they are permitted to conduct business as usual here. The values of these people are our values and therefore the establishment of their school is rightly opposed.

tomder55 answered on 12/06/08:

most schools in America already don't routinely play the National Anthem

such is the evil of political correctness.

labman rated this answer Excellent or Above Average Answer
paraclete rated this answer Excellent or Above Average Answer

Question/Answer
paraclete asked on 11/27/08 - So now we know the real reason?

Rolf Harris regrets the racist verse on Aborigines in Tie Me Kangaroo Down, Sport, the song that made him famous in Britain and launched the wobbleboard on an unsuspecting music world.

In Melbourne yesterday to plug a book of illustrations of the same name, the singer and painter said he had tried to erase the lines "Let me Abos go loose" and "They're of no further use" from all recordings over the years, with limited success.

"It was a mark of the times, done totally innocently with no realisation that you would offend at all … Just trying to create a fun song for a bunch of Aussies who were drinking themselves stupid on Swan Lager in London at the time," he said.

But half a century after penning the controversial lyrics, the London-based expatriate has not succumbed to political correctness. He blames traditional Aboriginal values for the dire living conditions in many indigenous communities.

"The attitude is that in their original way of life they would really wreck the surrounding countryside that they lived in and they would leave all the garbage and they would go walkabout to the next place," he said. "The traditional attitude is still there and I wish there was a simple solution but I'm not certain."

He has strong views about some Aborigines lamenting the conditions of their communities.

"You sit at home watching the television and you think to yourself, 'Get up off your arse and clean up the streets your bloody self' and 'Why would you expect somebody to come in and clean up your garbage which you've dumped everywhere?' But then you have to think to yourself that it's a different attitude to life."

Aboriginal children were never disciplined or expected to adhere to rules until adulthood, he said. "[Until] then they have a totally carefree life to do what they want and that quite often involves smashing everything that they have."

So this is the answer to our problems, let the abo's go loose, the're of no further use, blue?

tomder55 answered on 11/28/08:

actually I never made the connection to abo's; nor do I know what a didgeridoo is.

Mathatmacoat rated this answer Excellent or Above Average Answer
paraclete rated this answer Excellent or Above Average Answer

Question/Answer
paraclete asked on 11/19/08 - Them's fighin' words?

http://www.smh.com.au/news/us-election/alqaeda-number-two-ridicules-obama/2008/11/20/1226770577520.html

It seems Al Qaeda is on the offensive again, it appearently thinks being offensive to Obama will give it some brownie points. Al Qaeda thinks that Obama is what Black people in this country term a coconut, a person having questionable allegiences to his own race.

This is a strange move for an organisation which is yet to see how Obama will proceed in the war on terror. He certainly has promised to get tough on the Taliban and Al Qaeda in Afghanistan and to go beyond a war of words so prevelent amongst his predecessors. Why tweek the tail on the tiger and elicit a claw and fang response, unless he seeks to draw the US into the same killing field that destroyed the Russians.

It is strange that this missive didn't come directly from Osama bin Laden but from his number two. Could it be that Al Qaeda can no longer fake the tapes now that the US has a new leader?

tomder55 answered on 11/20/08:

Those hoping that a new president will bring respect and admiration for America are likely to be disappointed. The haters will always find reasons to despise us, no matter who's president.

al-Zawahri called President-elect Obama a "house slaves". (this is nothing new of course. Harry Belefonte called Colin Powell a "house slave" and the press hardly noticed ....but call the Obamamessiah a "house slave" and you cross that line !!!!)

Such vile language isn't surprisingcoming from an unhinged terrorist. al-Zawahri's one to talk. His terrorist group supports slavery in Africa.

But his words confirm that VP -elect Joe Biden's observation was exactly correct: Obama will be tested early, and often, by foreign leaders — allies and enemies alike.

That goes against what many people who supported Obama believed. They thought that the mantra of "change" would extend around the world, and that just by his election we could all have a transcendent Kumbayah moment, and the tenor of global affairs would change.

Well, it hasn't. Welcome to the real world, where America's wealth, power and freedom are the sources of both envy and spite.

When he takes office on Jan. 20, Obama will face a hostile world not very amenable to chants of "change" — from the Chi-coms keen to expand its military to challenge U.S. dominance on the high seas and in the global economy; to the resurgent Ruskies forging links with rogue regimes in Venezuela and Cuba, and using its energy supplies as a geopolitical weapon against the West; to a surprising renewal of piracy off the Horn of Africa; to an Iran seemingly determined to obtain a destabilizing nuclear weapon that it can brandish against its neighbors.

The question Obama should ask is :If our foes hate a strong America, just think how much more they will hate a weak one. Time for Obama to give up his childish fantasies . Right now he is unprepared to lead. He needs to learn in a hurry.



labman rated this answer Excellent or Above Average Answer
paraclete rated this answer Excellent or Above Average Answer

Question/Answer
paraclete asked on 11/19/08 - ol' Home week at the White House?

Seems Obama hasn't looked too far from the tree so far
with Holdings for AG and Clinton for State, will we see Powell for Defense, Bill Clinton as Ambassador at large and many Obama homies in other departments

tomder55 answered on 11/19/08:

I posted on this on the other site :

HRC as Sec State and other Clintonistas infiltrating Obama's administration

and

more of that Change stuff

labman rated this answer Excellent or Above Average Answer
paraclete rated this answer Excellent or Above Average Answer

Question/Answer
paraclete asked on 11/13/08 - "WHAT IF BEN LADEN WERE NEVER FOUND?"

http://news.smh.com.au/world/bin-laden-is-isolated-focused-on-his-own-security-cia-20081114-66ie.html

Not finding bin Laden has become the crutial element in the game of cat and mouse, sort of like a bizzaire where's willie? Not finding bin Laden didn't seem to do George Bush's political career any serious harm. After all he couldn't be less crediable. It remains to be seen whether not finding bin laden will do Barack Obama's career any harm.

It is suggested that finding bin Laden is the CIA's number one priority, so not finding bin Laden must have done the careers of these spooks serious harm. These people can find Taliban leaders and target them with drones with seeming monotonous regularity in the same terrain, yet bin Laden remains invisable. Not very crediable as the highest priority target in the world. Or perhaps he is some place else, watching but safe, hiding under a rock or in his grave. It's impossible to tell whether talk of him being among the Pustin is disinformation or reality.

What if he were never found? will legends arise of him being taken to heaven? It is imperative that bin Laden not be allowed to grow into some sort of Muslim mystic cult figure, he already presents too highly in a world where extremism is seen as a virtue

tomder55 answered on 11/18/08:

or in his grave. buried at Tora Bora

That's where I think he is .

The war against jihadistan was never against a single individual. The war against the jihadists has exposed the moral bankruptsy of their doctine of hate to the whole ummah.They are completely discredited ;especially in the Middle East where all they offered was repression. The fact that they are about to be chased out of their last viper nest by the combination of the coalition and the new Pakistan government ;and there is not nation left on earth to give them welcoming shelter is a testament to effectiveness of President Bush's policy .

labman rated this answer Excellent or Above Average Answer
paraclete rated this answer Excellent or Above Average Answer

Question/Answer
paraclete asked on 11/14/08 - SELF DECEPTION IS A TERRIABLE THING?

http://www.smh.com.au/news/world/many-years-ago-faith-changed-my-life-bush/2008/11/14/1226318871463.html

George Bush has alleged that america's purpose in Iraq and Afganistan was to protect Muslims. How decieved can you get and how stupid does he think the rest of us are? This is more of the insufferable arrogance that brought us WMD, The axis of evil,pre-emptive strikes and mission accomplished.

So now we are expected to believe that the war on terror was to protect muslims, not the USA. A war conducted so that Muslims could worship as they wished, as if anyone was stopping them except in George's unfathomiable mind. So George was protecting Muslims from the religious terrorism of Saddam Huessin, Mullah Omar and Slobadan Milosevic. Incrediable and how sad for the american people to be misrepresented in this way. When was George going to turn the war on those who stopped Christians worshipping as they wished, those self same Muslims he has been protecting?

tomder55 answered on 11/15/08:

1. In the Balkins were were protecting Muslims from ethnic cleansing

2. Afghanistan ;driving the Taliban from power enabled ordinary Afghanis to worship their faith the way they want to ;not the way the Taliban dictate.

3.Iraq : here is a direct quote from the war authorization bill :
Whereas Iraq persists in violating resolutions of the United Nations Security Council by continuing to engage in brutal repression of its civilian population thereby threatening international peace and security in the region

That was specifically about the Saddam repression of the Shia (a religious sect) and Kurds .
Since the war we have engaged terrorists who would perscute the people of Iraq ,be they Sunni ,Shia ,or Kurd .

labman rated this answer Excellent or Above Average Answer
paraclete rated this answer Excellent or Above Average Answer

Question/Answer
paraclete asked on 11/12/08 - Let's put the fear of God in them?

http://www.news.com.au/story/0,27574,24639985-421,00.html

Apparently the Mayor of Alice Springs, a central Australian city, thinks a month's stay by the riot squad should be sufficient to put fear into the hearts of some unruly youth who are making things unpleasant for the locals. One wonders where the local police are but then there are always motorists to prey on.

This is a new phase in dealing with disadvantaged youth and the aboriginal community and while we all
think strong measures are necessary to curb urban troublemaking, particularly from the aboriginal community who are the only group that behaves like this, surely a campaign of fear will not curb but escalate urban violence. What is probally needed is to round this lot up and send them back to their communities.

How do other communities deal with this, some constructive suggestions please?

tomder55 answered on 11/12/08:

How do other communities deal with this, some constructive suggestions please?

you have the space that the United States had in the early 19th century after the Lousiana purchase.

Back then Andrew Jackson decided that the best course of action was to create separation between the native population and the European settlers. His decision was to round them up and ship them out to territories beyond the Mississippi river. That resulted in such wonderful events as the forever infamous trail of tears .

In 1830 the Congress passed the "Indian Removal Act." Although many Americans were against the act, most notably Tennessee Congressman Davy Crockett, it passed anyway. President Jackson quickly signed the bill into law. The Cherokees attempted to fight removal legally by challenging it in the Supreme Court .The court ultimately decided that the only way the law was valid was if the Indians signed a treaty of removal. The Treaty of New Echota was was signed in 1835.4000 Cherokee died as a result of the forced removal from Georgia to Oklahoma.

You are wrong about the natives being the only unruly group of Aussies. I hear Muslim immigrant gangs are pretty unruly there also as well as the Aussie youth backlash .
http://www.breitbart.com/article.php?id=D8EEF3JO0&show_article=1

labman rated this answer Excellent or Above Average Answer
paraclete rated this answer Excellent or Above Average Answer

Question/Answer
paraclete asked on 10/26/08 - YEH RIGHT?

World leaders vowed today to overhaul the global financial system in the face of recession fears, but US President George Bush urged nations to "recommit" to free markets despite economic turmoil.

After a week of growing economic gloom and plunging stock markets, Asian and European leaders meeting in Beijing promised wide-ranging reforms, while UN Secretary General Ban Ki-moon also called for quick change.

"Leaders pledged to undertake effective and comprehensive reform of the international monetary and financial systems," the 40-member Asia Europe Meeting (ASEM) said in a statement released late yesterday.

"They agreed to take quickly appropriate initiatives in this respect, in consultation with all stakeholders and the relevant international financial institutions."

China's Premier Wen Jiabao called for more regulation of the world's financial system, saying after the summit: "We need to draw lessons from this crisis.

"We need financial innovation to serve the economy better. However, we need even more financial regulation to ensure financial safety."

Wen confirmed China's participation in a crucial summit in the United States on November 15 aimed at tackling the financial meltdown, without specifying which Chinese leader would attend the meeting of 20 industrialised and emerging powers.

The economic turmoil has led to growing criticism of US-style free market capitalism, with French President Nicolas Sarkozy earlier this week saying "the ideology of the dictatorship of the market... is dead".

But Bush, moving to set an agenda for the upcoming international economic summit, said today its participants must "recommit" to the principles of free enterprise and free trade.

So the reality is Bush doesn't get it, yeh Right, when has he ever got it? The American agenda has failed and failed spectacularly. so now the time is to regulate to take the cowboys out of the picture. Why doesn't he get it, the wild west is DEAD

tomder55 answered on 11/11/08:

in the US the collapse was from government social policy run amuck. The banks were mandated to issue credit to homebuyers that had well below the minimum standard of acceptable risk. The lending institutions in an effort to salvage a bad situation ,bundled these bad loans and traded them .But they were dealt a bad hand to begin with because programs to make home ownership available to people without the necessary resourse were imposed on the lending institutions.

So yes ; had free market principles applied to the home loan industry there would've been a recession (markets fluctate) ,but the financial bubble would not have burst .

labman rated this answer Excellent or Above Average Answer
paraclete rated this answer Excellent or Above Average Answer

Question/Answer
paraclete asked on 11/11/08 - Mass migration on a different scale?

http://www.smh.com.au/news/world/maldives-may-buy-part-of-australia/2008/11/10/1226165481956.html

Appearently the Maldives, a small muslim nation in the indian ocean has sufficient resources to buy themselves a new homeland, but why should they think of the continent of Australia, the dryest contentent on Earth. To go from perhaps the wettest and most cramped place to the dryest and most open seems a contrast to large to comtemplate, nor would there particularly militant form of Islam be welcomed in Australia, even in the vast deserts of its uninhabited west. There is no support infrastructure available for thgis nation to simply buy and migrate too!

I think this is the height of arrogance, that this country should simgle out another as a target for migration

tomder55 answered on 11/11/08:

Tell me about it. But as I always say ;a countries right to exist is soley dependent on it's ability to defend itself . A country without borders is not a country . What will Australia do to stem the migration ? Probably as much as the US has done to reduce the flow of the hordes crossing our southern border.

labman rated this answer Excellent or Above Average Answer
Mathatmacoat rated this answer Excellent or Above Average Answer
paraclete rated this answer Excellent or Above Average Answer

Question/Answer
paraclete asked on 09/16/08 - who must act?

We see the call for action because the artic ice is melting. The call now is for the world to act, as if the world could somehow instantly reverse the effects that have built up for a hundred years. There is a reality that there is one nation who must show leadership and act, but they are too busy counting their financial woes. That nation is the nation that has done more than any other to cause the problem in the first place. It is no good telling China and India they must act, certainly they need too, but the leadership must come from the USA. It is no good exporting the problem to China and India and then telling them they must curp their emissions as the price of doing business with the world, as if they could embrace the technology but somehow do it more efficiently. Yes, they can do it more efficiently by foregoing the technology and resorting to manual labour. a Billion hands making our garments. Utopia is upon them.

Stay the course is shouted about the war in Iraq, a war that has sapped the funding that could have been used. How about a new call, start the course.

My own nation is talking about 40 and 50% reductions by 2050 as if we have time to get it right, and trading schemes as if they somehow represent an answer, rather that a way of doing business as usual.
But I don't hear the US talking about how they will reduce emissions by 50%. Is it in their thinking at all to reduce emissions, or is it that they must wait for a political solution from a "new" administration, from an incoming messiah.

The reality is there will be no action, no way of saving the Artic and the world will morn the Polar Bear in the same way it morns the Dodo. I wonder will we morn the passing of the USA in the same way

tomder55 answered on 09/21/08:

Perhaps we will mourn the passing of the Polar Bear ;but I doubt it. The polar bear population today is around 25,000.Since the 1970s ;all while the world was warming ,polar bear numbers increased dramatically from around 5,000 to as many as 25,000 today (higher than at anytime in the 20th century). And historically, polar bears have thrived in temperatures even warmer than at present ;during the medieval warm period 1000 years ago and during the Holocene Climate Optimum between 5,000 and 9,000 years ago.

Polar bears have thrived during warmer climates because they are omnivores just like their cousin's the Brown and Black bears. Though Polar Bears eat seals more than any other food source at present, they have a varied diet when other foods are available including, fish, kelp, caribou, ducks, sea birds, beluga whale and musk ox and scavenged whale and walrus carcasses. In addition, Dr. Mitchell Taylor, a biologist with Nunavut Territorial government in Canada, pointed out in testimony to the U.S. Fish and Wildlife Service, that warming may be beneficial to bears since it creates better habitat for seals and would dramatically increase blueberry growth which bears eat when available.

Alaska's polar bear population is stable, and Canadian polar bear population has increased 25 percent from 12,000 to 15,000 during the past decade with 11 of Canada's 13 polar bear populations stable or increasing in number. Where polar bear weight and numbers are declining,too many bears competing for food, rather than arctic warming being the cause. That's right, the problem confronting polar bears may overpopulation not extinction!

As for the arctic ice ;why is the antarctic ice growing ? If global warming was causing Arctic ice shrinkage it would also cause reduction of southern ice.

Mary_Susan rated this answer Bad/Wrong Answer
paraclete rated this answer Excellent or Above Average Answer

Question/Answer
paraclete asked on 08/14/08 - Have you observed the other tradegy unfolding?

Have you noticed that Pakistan is unravelling. They are about to impeach the President, a man who originally took the position in a military coup.

This is a very dangerous scenario for the Pakistani people. It is highly unlikely the army will accept this. The remarks of the President regarding stability should be taken on board by the politicians, I don't think he is about to let them further destabilise the country. What do you think?

tomder55 answered on 08/14/08:

good point everyone has at least one eye on the Chinese fakery at the Olympics while the geopolitical world is facing many challenges.

Add to what you have talked about the ramping up of the Paki-India conflict over Kashmir and that situation is volatile . That is too bad because it looked not long ago that real progress was being made in that front.

Also their weakkneed response to Taliban and AQ forces establishing safe haven in their tribal areas and the displacement that is causing adds to the tension.

We should all pray that Musharraf retains the Presidency . WE don't want the jihadist extremist to take control.

labman rated this answer Excellent or Above Average Answer
Mary_Susan rated this answer Bad/Wrong Answer
paraclete rated this answer Excellent or Above Average Answer

Question/Answer
Mary_Susan asked on 08/13/08 - Another Bush Gigantic Failure-Russia

Yet, only a few months to go in a disastrous Presidency, and we see the results of Bush's blundering in foreign relations with the advent of Russia's bombing and invasion of Georgia.

Remember how Bush looked into Putin's eyes and saw a man he could work with??!! BWAH HA HA HA

Bush even has Condolezza as a top aid and Secretary of State...she, an academic EXPERT IN RUSSIA and they just ignored Russia.

I wonder if they are going to leave the problems to Dr. Strangelove-McCain?????

What a nightmare of a presidency!!!

NO MORE YEARS OF TRAGIC LEADERSHIP FROM REPUBS!!!

tomder55 answered on 08/14/08:

this is the same person who says we should engage and talk to our enemies . Well now you see what happens bwahaaahaaa . Your whole appeaser soft diplomacy approach bs has been exposed as the canard it is.

Bush yesterday announced a Trumanesque Berlin style air lift for Georgia and warned the Ruskies to not interfere . My hope is that the "aid" has quite a few SAMs anti-tank weapons and IED making kits .


CeeBee2 rated this answer Excellent or Above Average Answer
labman rated this answer Excellent or Above Average Answer
Mary_Susan rated this answer Bad/Wrong Answer
paraclete rated this answer Excellent or Above Average Answer

Question/Answer
HANK1 asked on 08/10/08 - Iraq & Afghanistan:


Something has been buggin' me for some years now. I would like to know if anyone in our government took the time to analyze the mind-set of the Muslims BEFORE we bombed IRAQ and AFGANISTAN. As we now know, the Muslims are destined to kill everyone who doesn't conform to their religion. We now know the Muslims knew about guerilla warfare. It seems to me that those powers-that-be were very naive about both.


Let me hear from you. Thanks.



HANK


tomder55 answered on 08/11/08:

I say let them worry about our mindset next time they decide to fly planes into our buildings.

Iraq is going well. I've yet to be convinced that Afghanistan is in our national interests beyond hunting the al-Qaeda leftovers. The dummycrats want us to invest alot more in that operation so we can be further bogged down like the Russians were.

Iraqis are stepping up to support their new freedom. It doesn't appear that the Afghanis are.

HANK1 rated this answer Excellent or Above Average Answer
labman rated this answer Excellent or Above Average Answer

Question/Answer
HANK1 asked on 08/09/08 - HELLO:

Good morning guys and gals. HANK is back. I've noticed that there's a lack of activity on this Board. Let's get the ball rollin', especially since there's so much going on in the political stream that's flowing sideways, upwards and downwards. No happy medium presently.

HANK

tomder55 answered on 08/11/08:

I have been spending most of my time elsewhere. The debates there are more even and interesting than here.

CeeBee2 rated this answer Excellent or Above Average Answer
HANK1 rated this answer Excellent or Above Average Answer

Question/Answer
paraclete asked on 07/26/08 - re you ignorant too?

It seems some of us just don't know what it is all about. Now this is completely unacceptable in a society with something a little short of 100% education. (the short is those who don't go to school, a compulsory exercise for the young)

http://www.smh.com.au/news/richard-glover/the-devils-in-the-detail/2008/07/25/1216492732872.html

thing is I know the answers but it seems some are just plain ignorant

tomder55 answered on 08/06/08:

Should we feel embarrassment about these gaps in our knowledge?

absolutely not !!! the fact is that every student leaves higher education with an understanding of less of the total accumulation of human knowlege than when they entered . It is just a fact that the rate of knowlege gained by humans has accelarated to a point where it outpaces our abilities to learn it . That is why there is such specialization .

labman rated this answer Excellent or Above Average Answer
paraclete rated this answer Excellent or Above Average Answer

Question/Answer
arcura asked on 07/03/08 - Did you know that there are massive volcanoes under the ice?

This might blow your mind the more you think about it.
http://www.foxnews.com/story/0,2933,374542,00.html

tomder55 answered on 07/07/08:

What the article fails to point out is what this volcanic activity may be having on the ice cap.

The Independent is reporting that scientist fear that there will be no ice on the North Pole this summer .The map they provide shows where the ice is thinnist appears to follow closely the Gakkel Ridge....the same ridge where the volcanic activity is taking place .


Here is the Independent article

Here is the map provided by the Independent ,and a map of the region :





labman rated this answer Excellent or Above Average Answer

Question/Answer
Mary_Susan asked on 07/03/08 - What is the *Real* Reason??

...that fundi/Ev Christians reject the climate crisis?

What is the reason?

tomder55 answered on 07/03/08:

I don't know why they don't but I reject it because there is no such thing as settled science. Once there is an orthodoxy like what is being claimed about climate change then it becomes a matter of faith .

The evidence is not "absolute " like the Goracle claims it is . How do I know ? Because there are plenty of their peers that say otherwise. Despite the Torquemada like effort to purge them;they have published their scientifically based evidence . And what they have published ;and the historical record confirms much of it, leads to a reasonable doubt that the planet is in danger ;or if it were ,that there is much we can do about it.

But that doesn't stop the high priest who lives in a house that consumes more carbon in a week than mine does in a year ;to advocate draconian social and life-style changes on the rest of us a “wrenching transformation of society”.

However the inconvenient truth is that a would-be central planner needs to get the masses on his side. To do that he adopted H.L. Mencken’s statement, “The whole aim of practical politics is to keep the populace alarmed by menacing it with an endless series of hobgoblins, all of them imaginary.” He lied that 98% of the scientific community agreed with him and his accomplices in the MSM regurgitated it.
Being an ifluential Clintonoid he was abole to reward the scientist that toed his line and punish those who didn't by refusing them Federal funding grants.
$50 billion has been spent promoting his theory (mostly tax money) and less that $1 billion to question it.
He pushed through the UN that would sap the US of it's wealth to support his ideas
What he did not count on is that Congress would see through Kyoto and outright reject it like they did before the Clintonoids left office. Now he blames Bush for killing it.

Now his movement is on the decline as more people have become aware of his Putsch attempt. Perhaps now rational climate policy can be adopted .

labman rated this answer Excellent or Above Average Answer
Mary_Susan rated this answer Bad/Wrong Answer
paraclete rated this answer Poor or Incomplete Answer

Question/Answer
arcura asked on 06/23/08 - Have you seen this site?

What to you personally think of it?
http://www.jonchristianryter.com/2008/080621.html

tomder55 answered on 06/23/08:

There is so much to go after Obama about it is not really necessary to bring this issue up. If someone declares themselves to be Christian I tend to take them at their word.

He has attended Christian churches for over 20 years that we know of and the only recorded times that he was presumably Muslim was during his earliest childhood .

The tape of Michelle has not been produced . I am beginning to doubt it's authenticity .It was revealed by a Hillary supporter and fueled on cable by a Hillary hack who tried to imply that Sean Hannity had the tape in his posession.

That being said ;Obama should speak more candidly about his African lineage and even more importantly ;his years working the streets of Chicago as a Daley operative .To listen to his latest campaign ad
his is an Horatio Alger story and that does not exactly match reality .

arcura rated this answer Excellent or Above Average Answer

Question/Answer
arcura asked on 05/17/08 - Do you believe this guy? I think I do.

He claims that the USA has more crude oil in the ground that the Arab counties all together do.
Watch and listen to what he says here.

http://video.google.com/videoplay?docid=3340274697167011147

tomder55 answered on 05/18/08:

I do not buy into conspiracy theory nonsense about the Rockefellers or big oil .The truth is that we do not know what our reserves truely are because the government refuses to allow exploratory drilling .

Certainly I BELIEVE that we have enough energy resources to be self sufficient . Really the question should be ;do we have enough CHEAP energy sources available .The answer to that is no . WE could not tap into existing reserves or find new sources that would bring the price of gasoline back to where we would prefer it to be.

The big benefit would be that the revenuse would not be going to thugs like Hugo Chavez and the Saudi clown princes.

So yes there is plenty of good reasons to take us in a new direction regarding energy independence. But don't look for cheap energy as being one of them.

arcura rated this answer Excellent or Above Average Answer
labman rated this answer Excellent or Above Average Answer
Mary_Susan rated this answer Excellent or Above Average Answer
paraclete rated this answer Excellent or Above Average Answer

Question/Answer
paraclete asked on 04/20/08 - let's talk about it?

Last weekend Australia underwent a historic grappling with the future and what momentous suggestions emerged from 1000 top minds getting together to find some new ideas and fresh paths.

Australia should become a republic.

overweight people should take the stairs

Children should eat fruit

Not exactly a fresh approach to the future and I fail to see how the decision making processes would change. So what I ask the experts here is, how does a stable parliamentary democracy benefit from becoming a republic?

http://www.smh.com.au/news/national/over-to-you-mr-rudd/2008/04/20/1208629731239.html

tomder55 answered on 04/21/08:

What I like about a republic is that local concerns are not so easily swept under the rug by the larger behometh national government .


I am perplexed to find one reason why even a ceremonial connection to the British crown is beneficial to the country . Your links are primarily historical true ;but it sorta flys in the face of my impression of Australia as fiercly independent . Perhaps I was wrong.

If I was an Aussie however I would be more concerned with Rudd's socialist polices and how they will bankrupt the country .

labman rated this answer Excellent or Above Average Answer
paraclete rated this answer Excellent or Above Average Answer

Question/Answer
HANK1 asked on 04/13/08 - Another Good Strategy ... but not for us:

Why couldn't the insurgents go into hibernation for six months? Our forces would pull out, thinking we won the war. Then the insurgents would clean up Iraq with carnage ... but not American carnage. And another thought: I readily believe that if we have a war with Iran, it will be Earth's FINAL WAR. Religion and politics will be the causes and involve the Jews, Islam and Christianity. When will this happen? In 2009!


Any comments?

Hank

tomder55 answered on 04/14/08:

in the Muslim war doctrine as outlined in their holy books that is called hudna .It is when you cease fire and truce because you are in a position of weakness .Then when you have regained the advantage you break the truce because in reality your word doesn't matter. This is actually in the their text when they describe the Treaty of Hudaybiyyah between Mohammed and the Quraysh tribe. He broke the treaty 2 years later.

We have seen this a number of times;most recently in Mooklie al-Sadr's temporary suspension of resistance during the Surge. This time however it was al-Maliki who went after him in Basra . al-Sadr was getting his butt handed to him so he again called for a cease fire arranged by his masters in Tehran. However this time no one is falling for the deception . The rooting out of his gang continues.

As for Iran ;we have been at war with them since 1979 . The seizing of the Embassy was an act of war ;and the Mullahs and their stooges like the Mahdi-hatter Ahamadjihad have used proxies to attack and kill US troops and citizens ever since. Tehran is vying for hegemony in the Gulf region and that is just not acceptable.

HANK1 rated this answer Excellent or Above Average Answer
purplewings rated this answer Excellent or Above Average Answer

Question/Answer
paraclete asked on 04/09/08 - Here's an interesting defense strategy

Australia has been told it must become strong enough to rip an arm off a gaint. Quiet literally this would mean the days of low taxation are over as fortress Australia become the norm.

Forces such as 400 strike aircraft and 40 submarines, just who are we going to take on

Is this a good policy in a developing world?

tomder55 answered on 04/09/08:

or you can continue to rely on the defense umbrella that the US provides .... or come under the protectorate of such emerging powers as China and India (heck I think you will need that type of force structure just to deal with Indonesia). I think Professor Babbage is very correct in his analysis. It is certain that Australia will never become a hegemon even in it's own corner of the world without a major increase in it's commitment to defense.

You are not alone . The EU is beginning to see the reality . They have beefed up their social nanny states on the back of US protection. But now when they are asked to do their share they hem and haw . The French added 1000 soldiers to the fight . whoop !! But they are not the worse. Other nations will only commit to non-combat roles.

labman rated this answer Excellent or Above Average Answer
paraclete rated this answer Excellent or Above Average Answer

Question/Answer
paraclete asked on 03/31/08 - will you be extinct in the Anthropocene age?

appearently climate change is advancing so radidly we have entered a human led extinction level event and the dawn of a new geological age; the Anthropocene. What typifies the dawn of a new age more often than not is an extinction level event, thus my question.

The odds on us surviving this extinction level event are falling every day

tomder55 answered on 04/03/08:

I guess Ted Turner is right . We are gonna be a bunch of cannibals.

http://www.ajc.com/metro/content/news/stories/2008/04/03/turner_0404.html

You know there was a period in Europe before the plague when the earth warmed and humans swarmed ? Then the plague came and Europe went into one of them cold spells.

http://www.newscientist.com/article/dn4464-early-farmers-warmed-earths-climate-.html


The secret agenda of the warming alarmists is to snuff out enough of us so that the resources left can be freely used by the lucky survivors.

labman rated this answer Excellent or Above Average Answer
paraclete rated this answer Excellent or Above Average Answer

Question/Answer
paraclete asked on 03/27/08 - Can you keep your junk in your own back yard?

perhaps it's time to ask if the US should be asked to launch its junk over its own lands and let its own population run the same risks as the rest of us

http://www.news.com.au/story/0,23599,23440301-2,00.html

tomder55 answered on 03/28/08:

quit crying you sound like chicken little

http://en.wikipedia.org/wiki/The_Sky_Is_Falling_(fable)

ladybugca rated this answer Excellent or Above Average Answer
paraclete rated this answer Excellent or Above Average Answer

Question/Answer
paraclete asked on 02/21/08 - What is the real significance of $100+ for oil?

The spot price for oil has risen to in excess of $100. What is the real significance of this latest development?

Is this just the result of the USD losing value as a result of low interets rates and poor economic conditions. You cannot have it both ways. Any government which shows its weakness internationally by using blatant stimulatory economic measures must pay in other ways.

tomder55 answered on 02/22/08:

no significance to the dollar value at all. the reason is supply is not meeting the new world wide demand and because EXXON and Venezuella is having a pissing contest because Chavez is an ass.

ladybugca rated this answer Excellent or Above Average Answer
paraclete rated this answer Excellent or Above Average Answer
powderpuff rated this answer Excellent or Above Average Answer
TTFNUAS rated this answer Excellent or Above Average Answer

Question/Answer
arcura asked on 02/14/08 - Are you for the Global Poverty act?

Can you believe that ANY American would sponsor or vote for a bill like this?

www.NewsWithViews.com
February 13, 2008
New Articles

Obama's Global Tax Proposal Up for Senate Vote
A nice-sounding bill called the “Global Poverty Act,” sponsored by Democratic presidential candidate and Senator Barack Obama, is up for a Senate vote on Thursday and could result in the imposition of a global tax on the United States. The bill, which has the support of many liberal religious groups, makes levels of U.S. foreign aid spending subservient to the dictates of the United Nations.........
http://www.newswithviews.com/Kincaid/cliff207.htm
by Cliff Kincaid

tomder55 answered on 02/14/08:

no and no again

Does the US taxpayer gets the right to deduct from the contribution the cost of maintaining the global peace, maintaining and operating the GPS system and enforcing the UN armistice on the Korean Peninsula among others? I am sure the answer is, "no".

Obama has made his position clear . The candidate who will not wear an American flag on his lapel but allows his offices in Texas to fly banners of Che Guevara is taking his revolution global.

arcura rated this answer Excellent or Above Average Answer
ladybugca rated this answer Excellent or Above Average Answer
Mary_Susan rated this answer Average Answer

Question/Answer
Mary_Susan asked on 01/11/08 - BUSH'S IDEA OF NAT.ID CARD, CALLED "REAL ID"

WASHINGTON (AP) - Residents of at least 17 states are suddenly stuck in the middle of a fight between the Bush administration and state governments over post-Sept. 11 security rules for driver's licenses - a dispute that, by May, could leave millions of people unable to use their licenses to board planes or enter federal buildings.

Homeland Security Secretary Michael Chertoff, who was unveiling final details of the REAL ID Act's rules on Friday, said that if states want their licenses to remain valid for air travel after May 2008, those states must seek a waiver indicating they want more time to comply with the legislation.

Chertoff, as he revealed final details of the REAL ID Act, said that in instances where a particular state doesn't seek a waiver, its residents will have to use a passport or a newly created federal passport card if they want to avoid a vigorous secondary screening at airport security.

``The last thing I want to do is punish citizens of a state who would love to have a REAL ID license but can't get one,'' Chertoff said. ``But in the end, the rule is the rule as passed by Congress.''

Chertoff spoke as he discussed the details of the administration's plan to improve security for driver's licenses in all 50 states - an effort delayed due to opposition from states worried about the cost and civil libertarians upset about what they believe are invasions of privacy.

Under the rules announced Friday, Americans born after Dec. 1, 1964, will have to get more secure driver's licenses in the next six years.

The Homeland Security Department has spent years crafting the final regulations for the REAL ID Act, a law designed to make it harder for terrorists, illegal immigrants and con artists to get government-issued identification. The effort once envisioned to take effect in 2008 has been pushed back in the hopes of winning over skeptical state officials.

To address some of those concerns, the government now plans to phase in a secure ID initiative that Congress approved in 2005. Now, DHS plans a key deadline in 2011 - when federal authorities hope all states will be in compliance - and then further measures to be enacted three years later.

To make the plan more appealing to cost-conscious states, federal authorities drastically reduced the expected cost from $14.6 billion to $3.9 billion, a 73 percent decline, said Homeland Security officials familiar with the plan.

The American Civil Liberties Union has fiercely objected to the effort, particularly the sharing of personal data among government agencies. The DHS and other officials say the only way to ensure an ID is safe is to check it against secure government data; critics such as the ACLU say that creates a system that is more likely to be infiltrated and have its personal data pilfered.

In its written objection to the law, the ACLU claims REAL ID amounts to the ``first-ever national identity card system,'' which ``would irreparably damage the fabric of American life.''

The Sept. 11 attacks were the main motivation for the changes.

The hijacker-pilot who flew into the Pentagon, Hani Hanjour, had four driver's licenses and ID cards from three states. The DHS, created in response to the attacks, has created a slogan for REAL ID: ``One driver, one license.''

By 2014, anyone seeking to board an airplane or enter a federal building would have to present a REAL ID-compliant driver's license, with the notable exception of those more than 50 years old, Homeland Security officials said.

The over-50 exemption was created to give states more time to get everyone new licenses, and officials say the risk of someone in that age group being a terrorist, illegal immigrant or con artist is much less. By 2017, even those over 50 must have a REAL ID-compliant card to board a plane.

So far, 17 states have passed legislation or resolutions objecting to the REAL ID Act's provisions, many due to concerns it will cost them too much to comply. The 17, according to the ACLU, are: Arizona, Colorado, Georgia, Hawaii, Idaho, Illinois, Maine, Missouri, Montana, Nebraska, Nevada, New Hampshire, North Dakota, Oklahoma, South Carolina, Tennessee and Washington state.

Among other details of the REAL ID plan:

The traditional driver's license photograph would be taken at the beginning of the application instead of the end so that if someone is rejected for failure to prove identity and citizenship, the applicant's photo would be kept on file and checked if that person tried to con the system again.

The cards will have three layers of security measures but will not contain microchips as some had expected. States will be able to choose from a menu which security measures they will put in their cards.

Over the next year, the government expects all states to begin checking both the Social Security numbers and immigration status of license applicants.

Most states already check Social Security numbers and about half check immigration status. Some, like New York, Virginia, North Carolina and California, have already implemented many of the security measures envisioned in REAL ID. In California, for example, officials expect the only major change to adopt the first phase would be to take the photograph at the beginning of the application process instead of the end.

After the Social Security and immigration status checks become nationwide practice, officials plan to move on to more expansive security checks, including state DMV offices checking with the State Department to verify those applicants who use passports to get a driver's license, verifying birth certificates and checking with other states to ensure an applicant doesn't have more than one license.

A few states have already signed written agreements indicating they plan to comply with REAL ID. Seventeen others, though, have passed legislation or resolutions objecting to it, often because of concerns about the cost of the extra security.

~~~~~~~~~~~~~~~~~~~~~~~~~~~~~~~~

What are your thoughts on this plan involving 50 states???

tomder55 answered on 01/15/08:

If there is to be a national id card (and I have no problem with that ) it should be issues by the national government . The responsibilty should not be on the states complying with standardized drivers licenses.

arcura rated this answer Excellent or Above Average Answer
Mary_Susan rated this answer Excellent or Above Average Answer

Question/Answer
paraclete asked on 12/05/07 - Who is the more dangerous US or Iran?

Not withstanding that Iran's leadership makes much belligerant noise, who is more dangerous. GWB has once agian been proven a liar on the subject of WMD



Bush: Nuke-less Iran remains dangerous Story Highlights
NEW: Iranian envoy says Iran has never sought nuclear weapons

Iran could transfer civilian nuclear work to military, President Bush says

U.S. intelligence estimate says Iran stopped nuclear arms work in 2003

U.S. national security adviser says U.S. policy toward Iran unchanged



WASHINGTON (CNN) -- Iran remains a danger to the world even though it stopped a program to develop a nuclear weapon four years ago, President Bush said Tuesday.


President Bush tells reporters Tuesday that Iran still poses a threat to the world.

"Iran was dangerous, Iran is dangerous and Iran will be dangerous if they have the knowledge necessary to make a nuclear weapon," Bush said, pointing out that Tehran continues to try to enrich uranium for civilian purposes and therefore develop technology that could be used for a weapon.

A declassified summary of a National Intelligence Estimate released by the U.S. government on Monday said Iran had stopped working toward a nuclear weapon in 2003 and is unlikely to be able to produce enough enriched uranium for a bomb until at least 2010.

Enriched uranium at low concentrations can be used to fuel nuclear power plants, but much higher concentrations are needed to yield a nuclear explosion.

The new estimate is less severe than a 2005 report that judged the Iranian leadership was "determined to develop nuclear weapons despite its international obligations and international pressure."

Earlier Tuesday, Iran ripped the Bush administration for rhetoric that came before Monday's release of the estimate.

"U.S. officials have so far inflicted ... damage on the Iranian nation by spreading lies against the country and by disturbing public opinion, therefore, they have to pay the price for their action," Iranian government spokesman Gholam-Hossein Elham is quoted as saying on the Web site of the official Islamic Republic News Agency.

In another posting, IRNA called the updated estimate "a necessary and positive step in Tehran-Washington relations, but undoubtedly is not sufficient."

"The U.S. administration should know that only admitting a mistake is not enough," the IRNA report said.

But Bush said Tuesday he saw the latest estimate on Iran as "a warning signal." Watch why Bush says he sees a danger in the report »

"What's to say they couldn't start another covert nuclear weapons program?" Bush asked.

The latest estimate shows "Iran needs to be taken seriously as a threat to peace," Bush said. See how the 2005 and 2007 estimates differ »



"We have good reason to continue to be concerned about Iran developing a nuclear weapon even after this most recent National Intelligence Estimate," he said. "In the words of the NIE, quote, Iranian entities are continuing to develop a range of technical capabilities that could be applied to producing nuclear weapons, if a decision is made to do so."

Hadley said U.S. policy toward Iran has not changed because of the new report.

"If we want to avoid a situation where we either have to accept Iran ... with a path to a nuclear weapon, or the possibility of having to use force to stop it, with all the connotations of World War III -- then we need to step up the diplomacy, step up the pressure, to get Iran to stop their so-called civilian uranium enrichment program," he said. "That's our policy going forward -- no change."

Britain on Tuesday also called for continued pressure on Iran.

"The report confirms that we were right to be worried about Iran seeking to develop nuclear weapons, and it also shows that the sanctions program and international pressure were having some effect," a spokesman for Prime Minister Gordon Brown said.

Israeli Defense Minister Ehud Barak echoed Bush's comments Tuesday.

"Iran is a main threat to the world and Israel," Barak said. "The entire world and the state of Israel should prepare to deal with this threat and thwart it."

Iran has insisted its nuclear program is strictly aimed at producing electricity, and the country has refused the U.N. Security Council's demand to halt its enrichment program.

Iran's ambassador to the International Atomic Energy Agency, Ali Asghar Soltanieh, said Tuesday that even the allegations that Tehran was pursuing a nuclear weapon up to 2003 were false.

"I categorically reject any allegation that Iran has had before, has now and will have [such a program] because a nuclear weapon is not in our defense decree," Soltanieh said.

"We are of the belief that a nuclear weapon would create a vulnerability and therefore we are and have and will be against nuclear weapons," he said.


The IAEA, the U.N. nuclear watchdog, has reported that Iran is cooperating with inspectors by providing access to declared nuclear material, documents and facilities. However, the agency also said Iran is withholding information in other areas, and as a result, the IAEA's knowledge about the status of the program is "diminishing."

Iran says its uranium enrichment work is allowed under the 1968 Non-Proliferation Treaty. The U.N. Security Council has passed two rounds of sanctions against Tehran, but Washington missed its goal of reaching consensus on tighter restrictions by the end of November, the State Department said last week.
++++++++++++++++++++++++++++++++++++++++++++++++++

When will the white House stop misleading the world about its intentions. The most dangerous nation in the world is the US and in the hands of a meglomaniac like Bush the danger is very real.

tomder55 answered on 12/07/07:

If I were to believe the new NIE would have to conclude that the Mullahs wet their pants after the Iraq invasion and halted their program just like Libya. But that is only if I were to believe the NIE that was authored by Valerie Plame cronies.

paraclete rated this answer Average Answer
Yiddishkeit rated this answer Excellent or Above Average Answer

Question/Answer
paraclete asked on 12/05/07 - A new world leader emerges?

While the Us has abdicated any leadership position on climate change it seems new leadership is emerging in unlikely places

I can unite the world on climate, says Rudd
Cynthia Banham Foreign Affairs Reporter
December 5, 2007



Garrett as support act: minister sidelined on global warming


AUSTRALIA will take on a highly ambitious and activist role on the international stage under the new Prime Minister, Kevin Rudd, who has unveiled a grand plan for uniting the world on climate change.

Heralding a significant shift in foreign policy from the Howard era, Mr Rudd - former diplomat and China expert - told the Herald yesterday he intended to use Australia's new position as a member of the Kyoto club to "bridge the gap" between developed and developing countries on future emissions controls.

In his first newspaper interview as Prime Minister, Mr Rudd admitted it was an enormous challenge but said Australia had a "national and international responsibility to the next generation" to do everything it could to counter the threat of climate change.

Mr Rudd will travel to Bali on Tuesday to join the UN conference on climate change, in what will be his first appearance on the world stage as Prime Minister.

While there, he revealed, he will also seek to enhance Australia's relationship with Indonesia when he meets its President, Susilo Bambang Yudhoyono.

The Climate Change Minister, Penny Wong, will play the key role as "principal negotiator" in Mr Rudd's vision for Australia as the world's new climate change broker, further underscoring her rapid rise in the new government.

"I fully recognise the difficulty of this because the distance between those two positions at present is enormous, but this is a gap which Australia in the past could not even hope to begin to bridge because we were not at the negotiating table at all in a substantive way," Mr Rudd said.

"We now are, and Senator Wong's brief, apart from arguing the Australian position, will be to do whatever is within her power and Australia's power to seek to bridge the gap between the positions of the developed and developing world on future emissions controls."

Having freshly ratified the Kyoto Protocol, Mr Rudd will personally hand Australia's instrument of ratification to the UN Secretary-General, Ban Ki-moon.

He will also meet the president of the World Bank, Robert Zoellick, and the former US vice-president, Al Gore. In his talks with Dr Yudhoyono, Mr Rudd said he would cover the "entire spectrum" of Australia's relationship with Indonesia, including the adequacy of security co-operation.

Terrorism would feature in these talks. As opposition leader, Mr Rudd nominated the rise of militant Islam as one of the biggest issues facing the world.

"My view is this is a relationship which our two governments can enhance, both at a level of security co-operation and economic engagement, as well as people-to-people contacts," Mr Rudd said.

"On the security policy engagement what I'd like to do with President Yudhoyono is quickly review the adequacy of our existing levels of security co-operation in the security, policing and intelligence areas, with a view to identifying any gaps which we can fill in the future in our common fight against terrorism."

After the Howard government's emphasis on the importance of bilateral relationships - especially with the US - Mr Rudd's grand plans for Australia in brokering a new global understanding on climate change signal a return to multilateralism and middle-power diplomacy as key features of the country's foreign policy.

It is significant that within days of assuming the prime ministership, Mr Rudd - who spent most of the first eight years of his parliamentary career as opposition spokesman on foreign affairs - has seized on climate change to showcase his new vision for Australia's role in international affairs.

He said he would use the opportunity in Bali to underline why Australia has altered its policy on climate change.

The former prime minister, John Howard, refused to ratify the protocol, along with the US.

"The reason for Australia changing its policy is our belief that the planet is now under increasingly significant threat and that therefore the requirement for global co-operation now is not optional, it is urgent," Mr Rudd said. He recognised the huge challenge he had set Senator Wong, and there was no guarantee of success.

"But I think we seize the fact that we have national and international responsibilities here to the next generation and that means using every ounce of Australia's national diplomatic energy to try and bridge that gap."

On the domestic front, Mr Rudd said it was still his Government's policy that there needed to be commitments from both developed and developing countries into the future on reducing emissions targets - despite his Environment Minister, Peter Garrett, contradicting this during the election campaign.

tomder55 answered on 12/07/07:

bwaa haa haaa haaa

He signed Kyoto fast enough and now he is backing off of enforcement .

http://www.news.com.au/heraldsun/story/0,21985,22883548-662,00.html

Another lefty smacked in the head by reality.


He will have the Aussies join the ranks of the rest of the phony world ;signing onto emission reductions they know damn well they won't achieve. But it will make him FEEL GOOD

paraclete rated this answer Poor or Incomplete Answer
Yiddishkeit rated this answer Excellent or Above Average Answer

Question/Answer
purplewings asked on 10/23/07 - Concerned about our government erosion.

I'm not just referring to the President being able to make new policy or ignore the wishes of the citizens, but to all politicians in general. In Michigan, we are in distress because of plant closings, corporate shut-downs, and lay offs with people losing their homes. At the same time, politicians in Lansing have overspent their budget leaving the state without proper funding for necessities such as police, prisons, schools, etc. Our elected officials have decided to rectify that by having another tax raise. Imagine losing jobs and homes to now being expected to pay for the state's failure to properly budget.

I recently read that when people stop paying attention or caring what the government does - it gives free rein for them to do whatever they like. It's hard to be interested when we are ignored and overburdened. How can we best get the government to do what they've been elected to do......serve us?

tomder55 answered on 10/24/07:

I always advise people to get active at the local level . Do what you can to improve your community and like minded action filters up .

I feel bad for the rust belt . The region grew due to heavy industrialization but we are living in the beginnings of the post industrial America .Like it or not those types of jobs are moving to a place where labor is cheaper. We can bemoan that fact or we can move forward and take steps to exploit the next great wave. When the automobile came there were alot of people who made and put shoes on horses who needed to find another way to make a living .

It is up to the community leaders to create an environment that will attract the new businesses that displace the old.

purplewings rated this answer Excellent or Above Average Answer

Question/Answer
MarySusan asked on 10/01/07 - REPUBLICAN PARTY COMMITTING SUICIDE?

"After convention speeches by the two Pats - Robertson and Buchanan - in 1992 helped elect Bill Clinton, organizers of the GOP's quadrennial gatherings effortlessly replaced Holy Roller hellfire with Happy Days hip hop.

In theory, political parties, whose function is to win first and govern later, are constantly evolving and adapting to changing demographics, issues and culture shifts.

But in practice in 2007, the Republican Party is diving for bottom. George Bush, the party's presidential candidates, and Republicans in Congress have set about destroying virtually everything they built.

They are defying all theories of rational self-interest, with behavior comparable to that of the Mets, that have in just 18 games thrown away a seemingly insurmountable advantage. Or, in the world of poker, behavior comparable to Mike "Full Tilt" Matusow, who has blown millions in stunning displays of ineptitude.

In fact, it is hard to find a match for the GOP's hodge-podge of manic stupidity: (Read more below)


The Supreme Court nomination of Harriett Miers; the mangling of New Orleans; the perseverating support of Rumsfeld and Gonzales; the insulation of Tom DeLay from ethics inquiries; the shunning of a presidential debate at Morgan State, a historically black college; the meticulous cultivation of corruption on Capitol Hill; the derisive treatment of such appointees as Paul O'Neil and Christine Todd Whitman turning them into attention-getting critics of the administration.

Nothing however, better exemplifies the compulsive irrationality that has taken over the Republican Party than its handling of the Hispanic electorate.

Latino voters, as Bush demonstrated in 2004, are by no means locked into the Democratic fold. On top of that, Republican strategists have been pounding for a decade the theme that Hispanics are crucial to the GOP future.

Ken Mehlman, who ran both Bush's 2004 campaign and the RNC, declared in a July 2006 speech (one of many on the subject) that as party chair,

"I know...that a Republican Party that does not reach out to Hispanics cannot win ... and a Republican Party that does not reach out to Hispanics does not deserve to win."

Similarly, Ed Gillespie, who ran the RNC before Mehlman, and who is now counselor to the president, laid it on the line in an April 2006 Wall Street Journal op-ed:

"The Republican Party cannot become an anti-immigration party. Our majority already rests too heavily on white voters, given that current demographic voting percentages will not allow us to hold our majority in the future. Between 2000 and 2004, President Bush increased his support in the Hispanic community by nine percentage points. Had he not, John Kerry would be president today.... Anti-immigration rhetoric is a political siren song, and Republicans must resist its lure by lashing ourselves to our party's twin masts of freedom and growth -- or our majority will crash on the shoals."

House and Senate Republicans have not only led the charge in killing immigration reform legislation, however, but their rhetoric has served to legitimize explicitly anti-immigrant and anti-Hispanic calls to action in city councils, on numerous web sites, on talk radio, and in public discourse generally.

Before the massive May 1, 2006, "A Day Without Immigrants" protests, Iowa Representative Steve King (R) declared:

"What would that May 1st look like without illegal immigration? There would be no one to smuggle across our southern border the heroin, marijuana, cocaine, and methamphetamines that plague the United States, reducing the U.S. supply of meth that day by 80%. The lives of 12 U.S. citizens would be saved who otherwise die a violent death at the hands of murderous illegal aliens each day. Another 13 Americans would survive who are otherwise killed each day by uninsured drunk driving illegals. Our hospital emergency rooms would not be flooded with everything from gunshot wounds, to anchor babies, to imported diseases to hangnails, giving American citizens the day off from standing in line behind illegals. Eight American children would not suffer the horror as a victim of a sex crime."

King's rhetoric undoubtedly appeals to many of his constituents - he gets re-elected by large margins - but some might suspect that the Iowa congressman is a Democratic plant. King's remarks, needless to say, are extensively covered in Spanish language media.

Conservative radio talk show host Neal Boortz does him one better.

"When we yank out the welcome mat, and they all start going back to Mexico, as a going away gift let's all give them a box of nuclear waste.... Tell 'em it'll heat tortillas."

Add to this the behavior of the current crop of Republican presidential candidates, all of whom know the importance of Latino voters in the general election.

In 2000 -- the last contested fight for the GOP nomination -- it would have been inconceivable that the candidates would have turned down a debate on Univision, the largest Spanish language television network in the country.

This year, with the exception of John McCain, they did.

Along similar lines, as recently as March 2006, Romney backed legislation that would have put millions of illegal immigrants on a "path to citizenship." Giuliani, in turn, was an outspoken supporter of immigrants, legal and illegal. Now, both of them have become unabashed critics of immigration reform, and treat as radioactive talk focused on immigrants' rights.

Of course, there could be a more subtle strategy at work here.

Perhaps Republican kingpins consider the best possible long-term strategy letting Democrats take over responsibility for the extraordinary mess Bush will leave behind. The next president will have to deal with Iraq, Iran with the bomb, biological and chemical threats, $8.98 trillion in national debt, global warming, rising gas prices, a Mideast on fire, overstretched troops, a legion of returning wounded soldiers, a country unprepared for its aging population, North Korea's supply of nuclear technology to Syria, a steadily eroding dollar, a surging China, and an exponential increase in the number of those who wish America ill.

Is this why the party plagued by weak allegiance on the part of female voters has three frontrunners for the nomination with a combined total of seven marriages and three trophy wives? And the fourth top candidate a Mormon at a time of anti-Mormon caricature (Under the Banner of Heaven, Big Love) facing major hurdles with the party's evangelical base?..." Thomas Edsall, blogging

~~~~~~~~~~~~~~~~~~~~~~~~~~~~~~~~~~~~~~~~~~~~

So, the Republicans are trying to lose so they can blame the horrific mess that Bush made in 8 years on the Democrats who they leave to clean up the mess???

tomder55 answered on 10/02/07:

The reason that Clinton won ;at least in 1992 is that Ross Perot syphoned off enough votes to swing the election to Clinton.

I will concede to some of the mistakes that Edsall throws out ;especially the no-shows at the debates ,but Edsall has to many factual errors to correct so I won't bother addressing them all.

The flaw in his rambling is the so called abandonment of the Hispanic vote. Pandering to the illegal population may be the Democrat expedient way ,but it is not right. There is not doubt that the Hispanic population is booming. Hispanics are the country’s largest minority But a growing population has little to do with voting strength.Hispanics were only 6 percent of the people who voted in the 2004 presidential election.One-third of Latinos in this country are under 18, and another one-third are not U.S. citizens.There are 27.1 million Hispanics 18 years and older in the U.S., but 40 percent of them are foreign nationals.

Take California .... Hispanics are 33 percent of the population but only 16 percent of the voters, Whites who are not Hispanic are now a minority in California but a clear majority of the voters, more than 70 percent.Elections are about who votes . Swings in party support are not decisive.

This also makes a false premise that Hispanics vote in a block. They do not . They represent too many nations to form a real consensus. Mexicans dominate in Texas, California, and New Mexico. Cubans dominate in Florida and are growing in New Jersey, Puerto Ricans, Dominicans, dominate in New York .

These folks all have different issues. A majority of Cubans vote Republican. Their primary concern is how candidates stand on issues involving Fidel Castro and Cuba.

Mexicans in Texas and California have voted Democratic in national elections. Their primary issue is immigration. But because the Mexican-American community is divided by citizens vs. non-citizens, those who came earlier and those who came later, those who are rich and those who are poor, monolithic voting cannot be assumed.

Puerto Ricans are U.S. citizens for whom immigration is no big deal. Their issues are as varied as the issues facing other native-born Americans.







Dark_Crow rated this answer Excellent or Above Average Answer
JimDandy rated this answer Excellent or Above Average Answer
MarySusan rated this answer Bad/Wrong Answer
tropicalstorm rated this answer Excellent or Above Average Answer

Question/Answer
Fritzella asked on 10/01/07 - RELIGIOUS RIGHT'S POLITICAL POWER IN SEVERE DECLINE

"WASHINGTON — Palm Sunday two years ago was a glorious day for Christian conservatives.

A president who'd proclaimed Jesus his favorite philosopher was racing back from vacation to sign a bill rushed through a compliant Congress at their bidding — a last-minute gamble to keep alive a severely brain-damaged woman in Florida.

That, however, was the peak of the Christian conservatives' political power.

Today, their nearly three-decade-long ascendance in the Republican Party is over. Their loyalties and priorities are in flux, the organizations that gave them political muscle are in disarray, the high-profile preachers who led them to influence through the 1980s and 1990s are being replaced by a new generation that's less interested in their agenda and their hold on politics and the 2008 Republican presidential nomination is in doubt.

"Less than four years after declarations that the Religious Right had taken over the Republican Party, these social conservatives seem almost powerless to influence its nomination process," said W. James Antle III, an editor at the American Spectator magazine who's written extensively about religious conservatives.

"They have the numbers. They have the capability. What they don't have is unity or any institutional leverage."

The Religious Right never had absolute power in the Republican Party. It never got the Republican president and Republican Congress to pursue a constitutional amendment banning abortion, for example....."

tomder55 answered on 10/01/07:

The Religious Right never had absolute power in the Republican Party.

That of course is the truth . In reality they have had about the same effect shaping policy in the Republican party as the Kossaks and code pinkos have had with the Democrats . In the end they are just another voting block to consider. Would the Kossaks not vote for Hillary in this general election even though sho polls near the bottom of their preference ? Of course not ! Simularily the so called religious right will hold their nose and vote for whichever Republican gets nominated . Would it be better for them to have Evita in the White House ?

Fritzella rated this answer Poor or Incomplete Answer

Question/Answer
tropicalstorm asked on 09/28/07 - How the heck..........

...does Hillary plan to pull off a $5,000.00 bond to every baby born and not expect to have to raise taxes or anything to cover it?????????

tomder55 answered on 09/29/07:

It's ging to come from the Social Security Trust Fund. Can I get mine + interest retroactive from 1955 ?

In the spirit of compromise that the Captn suggests will happen I propose that the plan should be revised to include only 'every child born in a marriage between a man and a woman who are U.S. Citizens at the time of the birth'. And why stop at $5000 ? Why not $50,000 ? It would instanty end poverty .....right ? I have a funny suspicion that it will work as well as the debit cards given to Katrina survivors.

So let me get this straight .Hillary wants government provided health care from cradle to grave.Now she also wants welfare from cradle to grave. Gotta love the progessive mind !

Ok enough fun ...seriously . There was a bipartisan proposal[Senate sponsors Senators Rick Santorum (R-PA), Jon Corzine (D-NJ), Charles Schumer (D-NY), and Jim DeMint (R-SC)] out for consideration a couple of years ago called the 'Americans Savings for Personal Investment, Retirement, and Education 'Act or ASPIRE Act.It proposes a much more modest version of Hillary's pandering .It offers $500 which would be invested toward the child's future education . At the age of 18 it could be cashed in for education, home ownership or retirement only .At the age of 30 ;the kid would need to pay back the $500 to the gvt. That way it is understood that this is not a gvt. give-away .It also pays for the next generation's $500 .

tropicalstorm rated this answer Excellent or Above Average Answer

Question/Answer
Fritzella asked on 09/26/07 - Republican Party Situation Dire for 2008

"Republican Party is in such bad shape heading into the 2008 congressional elections that many insiders are all but convinced things will get worse before they get better. The blame, some say, lies within the party itself.

Recent reports coming from the National Republican Congressional and Senatorial Committees paint a picture of institutions on the brink of bankruptcy and organizational disrepair. The Republican Party has ceded its traditional fundraising advantages to the Democrats. Recruiting conservative candidates to run for congressional office has proved exceedingly difficult. Even worse, several Republicans officials bemoaned, there is no short-term solutions on the horizon.

"In my lifetime you'd have to go back 30 years to find another time when the party committees were in such dire straits," Craig Shipley, president of the Republican public relations firm Shirley & Banister Public Affairs, told the Huffington Post. "The stimulus in the 70s was that the grassroots became disgusted with the abandonment of conservative principles and corruption of the party. The same thing is happening today. There is just a bad odor in the party right now."......--Sam Stein, Blogging

Edited for length.

~~~~~~~~~~~~~~~~~~~~~~~~~~~~~~~~~~~~~~~~~~~~~~~~~

tomder55 answered on 09/27/07:

yes the Republicans have issues . But then again ; when it comes time for 2008 election they will be able to point to the general ineffectiveness of the San Fran Nan and Harry Reid run Congress. The alternative that people voted for in 2006 does not appear to be able or willing to get their agenda enacted . Maybe after 2 years the electorate will realize that the Democrat promise was a trojan horse.

Democrats are getting money ;but lets face the facts , Obama is getting the most and he is going NOWHERE ......NOWHERE !!!! Does money really matter ? Fred Thompson is supposedly lagging in the money raising and he has defied conventional wisdom and caught up the the Republican front runners. Romney has more money available then any other Republican and he's going nowhere.

The Moveons and Kossaks poll all the time and Evita is on the bottom of their polls ;but she is going to win the nomination despite all the moonbat activists claims to "own the party" and the George Soros money.

Fritzella rated this answer Bad/Wrong Answer
JimDandy rated this answer Excellent or Above Average Answer
tropicalstorm rated this answer Excellent or Above Average Answer

Question/Answer
paraclete asked on 09/22/07 - The gaffe of the day?

Saddams dead, Nelsons dead, is Osama dead? they are all dead, so maybe that's why they can't find Osama?


Bush's Mandela death gaffe 'out there'

By staff writers

September 21, 2007 10:27pm
Article from: NEWS.com.au

NELSON Mandela is still very much alive despite a gaffe by US President George W. Bush, who alluded to the former South African leader's death in a speech yesterday.

"It's out there," said Achmat Dangor of the Nelson Mandela Foundation of Mr Bush's comment, which received worldwide media coverage.

"All we can do is reassure people, especially South Africans, that President Mandela is alive," he said.

In a speech defending his administration's Iraq policy, Mr Bush said former Iraqi leader Saddam Hussein's brutality had made it impossible to find a leader who could unite the country.

"I heard somebody say, 'Where's Mandela?'," he said.

"Well, Mandela's dead because Saddam Hussein killed all the Mandelas."

The bizarre gaffe was made in a press conference in Washington yesterday.

Mandela became South Africa's first black president in 1994. He won a Nobel Peace Prize for preaching racial harmony and guiding the nation into the post-apartheid era.

References to his death – Mandela is now 89 and increasingly frail – are seen as insensitive in South Africa.

With Reuters


++++++++++++++++++++++++++++++++++++++++++++++++++++

with thinking like this and a genius at the helm it's no wonder they can't solve problems, oh well George can solve it at the next OPEC meeting

tomder55 answered on 09/24/07:

More intentional sliming of the President by the Press. This is clearly taken out of content. If he would've replaced Mandela with something like "the founding fathers " he would've been clearer .But the intent of his comments is obvious to all but the dense and anyone who looks for any pretext ,no matter how valid it is ,to ridicule the President .

Here’s the quote in context, from the White House transcript:

Part of the reason why there is not this instant democracy in Iraq is because people are still recovering from Saddam Hussein’s brutal rule. I thought an interesting comment was made when somebody said to me, I heard somebody say, where’s Mandela? Well, Mandela is dead, because Saddam Hussein killed all the Mandelas. He was a brutal tyrant that divided people up and split families, and people are recovering from this. So there’s a psychological recovery that is taking place. And it’s hard work for them. And I understand it’s hard work for them. Having said that, I’m not going the give them a pass when it comes to the central government’s reconciliation efforts.

Yes if Saddam hadn't slaughtered all the Mandelas of the country [or future great leaders ]then yes it would've been easier to transition to a workable government .

Yes if someone had bumped off George Washington ,Thomas Jefferson and the other American founding fathers then it would be doubtful if our present gvt. would ever had taken off. And yes if someone had bumped off Nelson Mandela it is unlikely that the transition in South Africa would've been that successful .

I have to say the pattern also continues to this day with the road side bomb murder of Sheik Abdul-Sattar Abu Risha . Another reason why al-Qaeda,Syria, and Iran have to be defeated in Iraq.

Itsdb rated this answer Excellent or Above Average Answer
paraclete rated this answer Excellent or Above Average Answer

Question/Answer
paraclete asked on 09/13/07 - Ahmamadjihad is at it again

Ahmadinejad: Iran can help secure Iraq, Israel is 'cruel'
Iranian president urges U.S. and Britain to reconsider the invasion of Iraq


Iranian president says Israel "cannot continue its life"


(CNN) -- Iran wants "peace and friendship for all," the country's president said Wednesday while again denying Western assertions his nation is pursuing nuclear weapons and trying to destabilize Iraq.


Iranian President Mahmoud Ahmadinejad speaks at the Natanz nuclear facility in April.

But Mahmoud Ahmadinejad took a hard line against Israel, calling it "an invader" and saying it "cannot continue its life."

Asked if Iran had launched a proxy war in Iraq -- something the U.S. ambassador and top military commander there both asserted this week -- Ahmadinejad said the United States is merely seeking a scapegoat for its failing campaign in Iraq.

"Forces have come into Iraq and destroyed the security, and many people are killed," the Iranian president told Britain's ITN during an interview in the garden of the Iranian presidential palace in Tehran.

"And there are some claims that may seem very funny and ridiculous. Those who have lots of weaponry and warfare and thousands of soldiers -- if they are defeated, they blame others. There is no way to escape for peace."

Iranians do not believe in war and consider it a "last resort," he said.

He further claimed that Tehran is a friend of Iraq -- maintaining "good relationships" with the Sunni, Shiite and Kurdish factions -- and "if Iraq is not secure, we are the first country that would be damaged."

Don't Miss
Iran must pay $2.6 billion for attack on U.S. Marines
Iranian-American scholar back home after detention
Iran 'reaches nuclear target'
He added, "Responsible people should understand this: that Iran is against any sort of insecurity and attacks, and Iraq is able to defend themselves." Watch Ahmadinejad discuss issues affecting his country »

During the interview, Ahmadinejad struck a friendly tone toward Britain, saying he regretted that British soldiers have died in Iraq.

"We are sorry for your soldiers to be killed. We think peace should exist. Why should there be an invasion so that people will be killed?" he asked.

"We want friendship -- friendship to all. We love all nations and all human beings. Anyone who is killed, we are against it."

Ahmadinejad urged the United States and Britain to reconsider the invasion of Iraq. The two countries should "correct themselves," he said. If they don't, "the defeat would repeat."

The Islamic republic could help improve conditions in Iraq, but first coalition forces must leave, he said.

"We can help solve many problems in Iraq. We can help secure Iraq. We can help the attackers leave Iraq if the American government and British government correct themselves." he said.

Ahmadinejad has said in the past that Tehran would fill any power vacuum left by a withdrawal of coalition forces in Iraq.

The United States has cited the Iranian president's remarks as a reason to continue its efforts in Iraq.

As for allegations that Tehran is pursuing a nuclear weapon, Ahmadinejad said he resents the notion that Iran "has to obey whatever was put to us" and asked why there is no similar furor over American and British nuclear programs.

"Our bombs are dangerous, but American bombs are not dangerous?" he asked.

When the ITN interviewer asked if he could tour the Natanz nuclear facility in Iran, Ahmadinejad chuckled and asked him if he thought the United States or Britain would allow Iran to inspect their nuclear facilities.

"We do not need a bomb. We are against bombs, actually. There are many reasons we are against it," he said. "From a political point of view, it's not useful, we think."

The United Nations Security Council has so far imposed two rounds of limited sanctions against Iran for the country's refusal to suspend its uranium enrichment program.

Tehran has insisted the program is meant for peaceful energy production.

In regard to Israel, which Ahmadinejad has said should be politically "wiped off the map," the Iranian president said there is a way to deal with the Jewish state without violence.

Giving as an example the dissolution of the Soviet Union -- which he said came about "without war" -- Ahmadinejad suggested that "everything would be solved" if the Palestinian people were allowed to vote on their fate.


However, his hard-line rhetoric resurfaced when Ahmadinejad said Israel "cannot continue its life."

"Israel is an invader and is cruel, and it hasn't got a united public. All other nations are against it," he said. "We do not recognize them. They are attackers and illegal." E-mail to a friend
+++++++++++++++++++++++++++++++++++++++++++++++++++

What can this mean other than Iran intends to invade Iraq and attck Israel as soon a american forces leave Iraq?

tomder55 answered on 09/21/07:

The Mahdi hatter has spelled out a number of times what he intends. Like Hitler before him I suggest he is true to his word.

paraclete rated this answer Excellent or Above Average Answer

Question/Answer
paraclete asked on 09/16/07 - He told us what we already know

and have been saying?


Alan Greenspan says Iraq invasion motivated by oil

By Jeannine Aversa

September 17, 2007 06:40am
Article from: The Daily Telegraph


FORMER Federal Reserve Chairman Alan Greenspan, in his new book, says the US went to war in Iraq motivated largely by oil.

Mr Greenspan said: "I am saddened that it is politically inconvenient to acknowledge what everyone knows: the Iraq war is largely about oil."

His book also criticises US President George W. Bush for not responsibly handling the nation's spending and racking up big budget deficits.

A self-described "libertarian Republican," Mr Greenspan takes his own party to task for forsaking conservative principles that favour small government.

"My biggest frustration remained the president's unwillingness to wield his veto against out-of-control spending," Mr Greenspan wrote.

Mr Bush took office in 2001, the last time the Government produced a budget surplus.

Every year after that, the Government has been in the red. In 2004, the deficit swelled to a record $US413 billion ($A493.75 billion).

"The Republicans in Congress lost their way," Mr Greenspan wrote. "They swapped principle for power. They ended up with neither. They deserved to lose."

In 2006, voters put Democrats in charge of Congress for the first time in a dozen years.

Mr Greenspan's memoir, The Age of Turbulence: Adventures in a New World, is scheduled for release today. The book is a recollection of his life and his time as Fed chief.

Mr Greenspan, 81, ran the Fed for 18 1/2 years and was the second-longest serving chief. He served under four presidents, starting with his initial nomination by former President Ronald Reagan.

He says he began to write the book on February 1, 2006, the day his successor - Ben Bernanke -- took over.

The ex-Fed chief writes that he laments the loss of fiscal discipline.

"Deficits don't matter," to my chagrin, became part of Republicans' rhetoric."

Mr Greenspan long has argued that persistent budget deficits pose a danger to the economy over the long run.

Large projected surpluses were the basis for Mr Bush's $US1.35 trillion ($A1.61 trillion), 10-year tax cut approved in the summer of 2001.

Budget experts projected the Government would run a whopping $US5.6 trillion ($A6.69 trillion) worth of surpluses over the subsequent decade after the cuts.

Those surpluses, the basis for Mr Bush's campaign promises of a tax cut, never materialised.

tomder55 answered on 09/21/07:

you are aware of course that he has since revised his comments to take out the implication that it was Bush's reason for going to war and instead it was his own pesronal opinion. Further he clarified that he was in favor of the invasion on that pretext.

paraclete rated this answer Excellent or Above Average Answer

Question/Answer
paraclete asked on 08/16/07 - Deje vu or sensing the moment?

Hanson calls for halt to muslim immigration
August 16, 2007 - 9:14AM


Senate candidate Pauline Hanson has urged Federal Parliament to hold a moratorium on the number of Muslims entering Australia.

The right-wing former One Nation leader is seeking to register Pauline's United Australia Party in her bid for a political comeback by winning a Queensland senate seat in the upcoming federal election.

The 53-year-old former fish and chip shop owner, who won international notoriety during her brief spell as the independent MP for Oxley in the late 1990s, says she will targeting Muslims in her campaign.

"I want a moratorium put on the number of Muslims coming into Australia," Ms Hanson told the Nine network.

"People have a right to be very concerned about this because of the terrorist attacks that have happened throughout the world.

"I'm sick of these people coming out here and saying that our girls are like the meat market and the bible that is urinated on ... am I supposed to be tolerant?"

But Ms Hanson said she would have the support of Muslim women if they knew how oppressed they were.

"I think that if Muslim women realise how they have been treated I probably would get a lot of support," she said.

"Maybe we should look at the female genital mutilation that happens to young girls in this country ... if people want to live by these ways then go back to the Muslim countries."

Ms Hanson said immigration was not her only concern and she would campaign on other issues such as the privatisation of water.

She said many of the issues raised in her maiden speech to parliament in September 1996 had been adopted by the Howard government.

"Don't just say I'm simplistic and I don't know what I'm talking about. They said the same about the Aboriginal issue but the prime minister actually abolished ATSIC (Aboriginal and Torres Strait Islander Commission) and that was in my maiden speech."

Ms Hanson believes her campaign is winning strong support among the people of Queensland.

"I wouldn't say I've got no chance ... I've travelled around Queensland quite extensively over the past six months. There is tremendous support from people."

"They are wanting someone else to vote for ... so they are looking at me."

Ms Hanson was elected to parliament as an independent MP for the Queensland seat of Oxley at the 1996 election after being disendorsed as a Liberal candidate because of her strong views on race and immigration.

She failed to win the neighbouring seat of Blair in 1998, a senate seat in 2004 and a position in the NSW upper house in 2003.

AAP

tomder55 answered on 08/18/07:

Immigration has helped make countries such as the US and Australia what they are today. That immigration, however, was very different. Immigrants adopted the norms and customs of their new country. They changed to adapt to their new homeland. They didn’t expect their new homeland to change to adapt to them.

It is not just Australia and the US sleep walking . Given the fact that Europe is also being over-run it is safe to say that all of Western Civilization has their heads buried in the sand.

Mathatmacoat rated this answer Excellent or Above Average Answer
paraclete rated this answer Excellent or Above Average Answer

Question/Answer
pradeep asked on 07/24/07 - your ideal job

briefly describe your ideal job

tomder55 answered on 07/25/07:

I had an ideal job. I stood at the theater entrance and ripped tickets in half. Then after briefly running a carpet sweeper over the lobby floor I settled down and watched the show.

Next to that I think I would function well as a researchologist for perhaps a think tank. I would also love to be a professional student.Perhaps I would like getting paid for my on-line commentary and contributions if I could figure out a way to do it. I also would not mind being an antique buyer and seller. I would enjoy I think having a position with a professional baseball team .

Question/Answer
Dark_Crow asked on 07/22/07 - The sickest pervert here is whoever administrates this website.

.

tomder55 answered on 07/23/07:

agreed . I see the idiocy has spread to the Philosophy Board . It really is a shame . I notice that there are a corp of administrators on the new site.

Dark_Crow rated this answer Excellent or Above Average Answer
PrinceHassim rated this answer Excellent or Above Average Answer
tropicalstorm rated this answer Excellent or Above Average Answer

Question/Answer
paraclete asked on 07/22/07 - A different way of tacking the Iraq problem

with one small drawback.

Australian chief lashes US on Iraq


Tom Allard in Baghdad
July 23, 2007

AUSTRALIA'S Chief of Army, Lieutenant General Peter Leahy, has taken a swipe at the US military's strategy at the outset of the Iraq war, expressing disbelief that it has taken so long for commanders to realise the merits of engaging with the local population and winning their trust.

Visiting a counter-insurgency centre for excellence at Taji, just outside Baghdad, General Leahy was briefed by a US Army colonel, Manuel Diemer, on the new US strategy of schooling all unit commanders in the importance of developing a deep understanding of the culture of the communities in which they are conducting operations.

Colonel Diemer said the strategy meant combat units were now living among the population, doing more foot patrols, talking and interacting with the population well before they undertook any offensive operations.

This contrasts with the previous US practice of sending in forces with overwhelming firepower into trouble spots and then returning home to the relative comforts and security of a military base after blasting their way out of, or into, trouble.

The tactics have created huge resentment among ordinary Iraqis and helped fuel the insurgency. General Leahy expressed his strong support for the new strategy, but with an important, and thrice repeated, caveat. "I can't believe you guys weren't doing this two or three years ago," he said. Colonel Diemer concurred.

As he has travelled around the Middle East, General Leahy has emphasised to his troops how the nature of warfare has progressed from the Cold War principles of conventional warfare.

Rather than pitched battles between large militaries, soldiers were now working within and for communities.

"It's about protecting, supporting and persuading … and paving the way for reconstruction," he said.

Among soldiers anxious to be at the sharp end of a conflict such a message was not always warmly received, he said.

"Some of them think I'm a mongrel because I won't let them shoot the shit out of them [the insurgents]," he told Colonel Diemer.

In the two southern Iraqi provinces where Australians have been operating, they have achieved considerable success in developing links with tribal leaders and consulting widely before they undertake operations or reconstruction projects.

In Iraqi society, tribal affiliations are paramount, often trumping any loyalty to nation and even religious sect.

For Australian units patrolling in Iraq's south, a meeting with each village leader is essential.

Their efforts to reach out are usually met with traditional Iraqi hospitality, the slaughter of a goat and a long feast for the soldiers.

The only drawback, said the Australian commander in Iraq, Brigadier Gerard Fogarty, were the resulting bouts of diarrhoea many soldiers suffered.

Four Australians have been working with Colonel Diemer at the counter-insurgency centre since its inception eight months ago, a reflection that such policies have been an integral part of the Australian Army's doctrine for years, certainly well before the East Timor intervention in 1999.

tomder55 answered on 07/23/07:

I can't argue with results .

General Petraeus gets it . It would be a shame if the US feckless Congress did not let him complete his mission after they confirmed him for the job unanimously .

Citizen journalist Michael Totten is embedded in Iraq . He filed in his journal this interesting observation

http://www.michaeltotten.com/archives/001496.html

It’s a peculiar war. It is almost a not-war. Last July’s war in Northern Israel and Southern Lebanon was hundreds of times more violent and terrifying than this one. Explosions on both sides of the Lebanese-Israeli border were constant when I was there.

You’d think explosions and gunfire define Iraq if you look at this country from far away on the news. They do not. The media is a total distortion machine. Certain areas are still extremely violent, but the country as a whole is defined by heat, not war, at least in the summer. It is Iraq’s most singular characteristic. I dread going outside because it’s hot, not because I’m afraid I will get hurt.


Another citizen journalist Michael Yon has been reporting about Operation Arrowhead Ripper that began 1 month ago. He makes a telling point about how the military is doing the bulk of the diplomat corp's function.

http://www.michaelyon-online.com/wp/7-rules-1-oath.htm

Colonel Townsend’s staff had prepared a slideshow that started off with a draft of “7 Rules.” The final version of the 7 Rules were open to discussion and suggestions from those in attendance. The rules were followed by an Oath, also still in draft.

First Colonel Townsend reviewed the 7 Rules, presented here verbatim from the slides:

1) Protect your community from AQI, JAM and other terrorist militia.
2) Accept both peaceful Sunni, Shia and others.
3) Stay in your neighborhood/AO [area of operations] for your safety.
4) Take an oath of allegiance to the Constitution of Iraq.
5) Register with Iraqi Security Forces and Coalition Forces [biometrics for CF].
6) For your safety, wear a standard uniform and markings [an example was proposed].
7) Receive hiring preference for Iraqi Police and Army.

Then came the Oath, also presented here verbatim from the slides:

1) I will support and defend the Constitution of Iraq.
2) I will cooperate fully with the Iraqi government.
3) I will guard my neighborhood, community and city.
4) I will bear no arms outside my home without coordination of Iraqi Security Forces or Coalition Forces
5) I will bear no arms against the Government of Iraq, Iraqi Security Forces or Coalition Forces.
6) I will not support sectarian agendas.

After the proposal for the 7 Rules and the Oath were presented, the most interesting—fascinating, really—part of the meeting unfolded.

The Iraqi Army commanders and “Baqubah Guardians” then gave their input, and some of that input was as follows:

1) Protect your community from AQI, JAM and other terrorist militia.
Some attendees did not like that AQI and JAM were singled out, citing that this only validated those organizations, while not fully recognizing the problems from terrorist groups such as the Badr or IAI. Other attendees disagreed and thought the groups should be named, but finally it was decided to strike the names AQI and JAM.

2) Accept both peaceful Sunni, Shia and others.
After some intelligent discussion, the Iraqis wanted this changed to “Accept all peaceful Iraqi citizens without discrimination.”

3) Stay in your neighborhood/AO [area of operations] for your safety.
This needed clarification: Colonel Townsend was not saying they should not travel from their neighborhoods, but that they should not operate out of their neighborhoods, and the attendees agreed.

4) Take an oath of allegiance to the Constitution of Iraq.
Now it got interesting. One Iraqi said that even under the Saddam regime, bad as it was, the constitution still kept them together. He made no mention of the wars against the Kurds or Shia. But he went on to say that the current constitution tended to divide Iraq. No serious arguments were put forth on this today, but it was clear that fourth rule could lead to months or years of debate. After all, our own Constitution remains a work in progress, having been amended more than two dozen times. Each time that Americans bring this fact to forefront, it seems to assuage some of the “Constitutional-angst” among Iraqis, but that doesn’t change the fact that their government is about as solid as fog.

5) Register with Iraqi Security Forces and Coalition Forces [biometrics for CF].
The “biometrics” part of #5 was an issue partly because Coalition Forces do not share biometrics with the ISF, and so in fact we are asking Iraqis to submit to photographing, fingerprinting and retinal scans for our use. The Iraqis politely offered their consensus that this was not a good idea, and Colonel Townsend chuckled, saying even Americans wouldn’t go for that. [Can’t blame him for trying.]

6) For your safety, wear a standard uniform and markings [an example was proposed].
The uniform idea was fine with the Iraqis, especially so since we killed at least six of their militia members in the last 30 days. I saw our guys shoot four 1920s guys a few days ago on Sunday, killing two of them. The shooting was the fault of the 1920s guys: had they been wearing uniforms, they would be alive today. The Iraqis agreed that uniforms are a good idea.

7) Receive hiring preference for Iraqi Police and Army.
Point number seven received nods of approval.
On the Oath, the matter was more interesting:

1) I will support and defend the Constitution of Iraq.
Discussion around Point One of the Oath was similar to that around Point Four of the 7 Rules.

2) I will cooperate fully with the Iraqi government.
Point two received some pushback, but again, imagine asking all Americans to swear that “I will comply fully with the American Government.” It would be un-American to agree to that! And here in Iraq, if I were an Iraqi, I would never agree to “I will cooperate fully with the Government of Iraq.” What government? The one in Baghdad that refuses to send legal food shipments to Diyala Province? I saw this with my own eyes and videotaped officials in the “Iraqi government” refusing to help the Diyala Government, calling Diyala (verbatim) a “terrorist province.” Even though Diyala has been a province riddled by terrorists lately, that still doesn’t change the fact that people here went without food because of the government people in Baghdad they are now supposed to pledge allegiance to. No smart person was likely to sign that line.

The other points were subject to briefer discussion and easier agreement, although the easiest of all parts of the Oath was point Six—I will not support sectarian agendas. Every Iraqi in the room immediately was aboard on this one, and they even seemed enthusiastic about it.

I’ve saved an unmentioned point for last. The Iraqi flag appeared on some of the slides. But the graphic showed an Iraqi flag without the traditional words “God is Great.” This was clearly a potential flash point. In fact, one of the Iraqi interpreters nearly recused himself from the conversation. LT David Wallach, whose native tongue is Arabic, told me after the meeting that Saddam had put “God is Great” on the flags so that Iraqis would stop grinding the flags into the dirt with their feet. He said that Iraqis would never trample on anything that had those words written on it.

But other than the interpreter’s sudden jitters, I detected no overt emotion among the Iraqis. In fact, they were all calm, professional, and very polite. An Iraqi Colonel was generous enough to offer that he believed it to be just a mistake that “God is Great” was left off the flag that was used on the slides. But the Iraqis all agreed that nobody was going to sign anything that displayed an Iraqi flag without the phrase “God is Great.”

This might seem ominous to us. “Allah u Akbar!” are, after all, words that we have become accustomed to hearing when someone is doing something bad, like burning an American flag, or blowing up Americans. But these issues are more like the intense legal and media battles over the words “In God We Trust” on the money in our pockets, or the ongoing furor in some sectors over the phrase “One Nation, Under God, Indivisible . . . ” in the Pledge of Allegiance. (Not to mention the dust storms kicked up by the Pledge itself.)

Seeing “God is Great” written on the Iraqi flag might provoke some to protest “Why did we come here just to stand up a country who would write such things on their flag?” But I sat there in that meeting, which was completely civil and professional, and I thought about another flag, the one flying over South Carolina. Some people call that flag “heritage,” while others call it “hateful,” “painful” and “demeaning.” And today in that meeting, I thought about the descendants of slaves who are now top military commanders in the American Army, and in that moment I knew that Iraq could make it.


God bless Col Townsend and all who serve with him. He is doing the work of warrior and diplomat in the tradition of I guess General Douglas McArthur . While the Democrats organize slumber parties and the State Dept. plans grand bargain games that more resemble grand illusions , from the safety of Foggy Bottom (or in the case of Iraq the Green Zone )the Majors and Colonels in the field are showing the word how diplomacy should work ....from a position of strength.





paraclete rated this answer Excellent or Above Average Answer

Question/Answer
MarySusan asked on 07/22/07 - BUSH TO BE CENSURED--MY FINAL POST

WASHINGTON — "Liberal Democratic Sen. Russ Feingold said Sunday he wants Congress to censure President Bush for his management of the Iraq war and his "assault" against the Constitution.

Feingold, a prominent war critic, said he soon plans to offer two censure resolutions _ measures that would amount to a formal condemnation of the Republican president.

The first would seek to reprimand Bush for, as Feingold described it, getting the nation into war without adequate military preparation and for issuing misleading public statements. The resolution also would cite Vice President Dick Cheney and perhaps other administration officials.


The second measure would seek to censure Bush for what the Democrat called a continuous assault against the rule of law through such efforts as the warrantless surveillance program against suspected terrorists, Feingold said. It would also ask for a reprimand of Attorney General Alberto Gonzales and maybe others.

"This is an opportunity for people to say, let's at least reflect on the record that something terrible has happened here," said Feingold, D-Wis. "This administration has weakened America in a way that is frightful."

~~~~~~~~~~~~~~~~~~~~~~~~~~~~~~~~~~~~~~~~~~

Yeah, "somethng terrible" happened here in America....Shit for brains Christian voters electing a shit for brains President who fucked up everything he touched.

tomder55 answered on 07/23/07:

lol censure ? that's the best they can come up with ? Boy the Huffpos the kossaks and the moveon idiots are going to be disappointed .

Why don't they have another all night pajama party defeat-a-thon ? In the end ;the democrats will have squandered 2 years with their irrational Bush hatred instead of governance . It will be the shortest majority control in history .

Dark_Crow rated this answer Excellent or Above Average Answer
MarySusan rated this answer Excellent or Above Average Answer

Question/Answer
Choux... asked on 07/20/07 - Founding Fathers Would Have Disliked Bush

"I'm in Boston today, getting ready for my standup special tomorrow night live on HBO (last shameless plug, I promise), and walking around the city has made me remember: oh yeah, America started here. That's right, America was invented by liberal men in Boston and Philadelphia. Not that I don't love all of America, but rednecks who think they're the real America should read a history book once in a while. George Washington, Thomas Jefferson, Franklin, Madison -- the whole lot of them were well read, erudite, European thinking children of the enlightenment, and they would have had absolutely nothing in common and less to say to a cowboy simpleton like George Bush.

And speaking of who's a real American, was anyone as outraged as I was reading Robert Novak's little interview in the NY Times magazine on Sunday? Asked if in hindsight he would leave out the part of his 2003 column that identified Valerie Plame as a CIA operative, he said "I don't know. I thought journalistically it was justifiable. Nobody had told me -- and I still don't believe -- that it put anybody's life in danger. I don't think she was an important person in the CIA."

That really is quite an astounding quote, isn't it? How the hell would he know if it put anybody's life in danger? YOU'RE NOT IN THE CIA, BOB! They don't tell you any of their business! Considering the consequences of being wrong about such a hunch, is it really the patriotic thing to do? To sit in your office and just conjecture that this agent wasn't very important to the CIA? First, I think everyone who works at the CIA is important; and second, WHO THE HELL IS THIS MAN TO OUT PEOPLE IN THE DEADLY WORLD OF ESPIONAGE BECAUSE THAT'S WHAT HE "THINKS"?!

With patriots like that, I'm sure glad there are traitors like me and Michael Moore still living...."

~~~~~~~~~~~~~~~~~~~~~~~~~~~~~~~~~~~~~~~~~

Comments?

tomder55 answered on 07/21/07:

George Washington, Thomas Jefferson, , James Madison were not north east liberals either .Bill Maher needs to brush up on his history . All 3 of these founders were from Virginia.

I also guess by his idiot logic that the founders would've also hated Jimmy Carter and Bill Clinton because they were also simpletons from the South .

Plame may or may not have been an important person in the CIA .The point is that she was not a covert spy and had not been for a long time since Aldrich Ames outed her in 1994.She was an analyst staffer on the WMD desk . It was a legitimate question asking why the CIA would send a democrat hack to Niger to investigate such an important question. Everyone in Washington knew that Valerie Plame worked in the CIA . How did they know ? Because Joe Wilson loved to parade his trophy wife around town bragging about what she did . She was in 'Who's Who '.Novak used that to confirm the information he was given by Armitage (not Libby ....not Rove....not Cheney).

I wonder of Bill Maher is equally as outraged when the NY Slimes and the Washington Compost reveals national security secrets ? I kinda doubt it.

Choux... rated this answer Bad/Wrong Answer

Question/Answer
arcura asked on 07/20/07 - What do you think about Fred Dalton Thompson

for president.

look here to see how he stand on issues.

http://www.ontheissues.org/senate/Fred_Thompson.htm#Environment

tomder55 answered on 07/21/07:

I am leaning towards supporting either him or Rudy. My preference would be that both be on the ticket. I started seriously paying attention to him when he was publishing op-eds on the internet and making guest commentary on the Paul Harvey radio broadcasts . His is a sensible conservatism that has broad appeal .

So far he has taken all the shots that the very nervous Democrat campaign has thrown at him and they haven't damaged him at all. It is still early and I'll reserve my opinion until the NY primary or at least until he actually declares his candidacy. My belief is that once he does all the other candidates except perhaps Rudy will become 2nd tier candidates .

arcura rated this answer Excellent or Above Average Answer
Fritzella rated this answer Average Answer

Question/Answer
MarySusan asked on 07/18/07 - BUSH-his strategy for fighting AlQuaeda a Failure

WASHINGTON, July 17 — President Bush’s top counterterrorism advisers acknowledged Tuesday that the strategy for fighting Osama bin Laden’s leadership of Al Qaeda in Pakistan had failed, as the White House released a grim new intelligence assessment that has forced the administration to consider more aggressive measures inside Pakistan.


Pakistan’s president, Gen. Pervez Musharraf, in Turbat recently. His strategy in tribal areas has been criticized by President Bush’s advisers.

The intelligence report, the most formal assessment since the Sept. 11 attacks about the terrorist threat facing the United States, concludes that the United States is losing ground on a number of fronts in the fight against Al Qaeda, and describes the terrorist organization as having significantly strengthened over the past two years."

~~~~~~~~~~~~~~~~~~~~~~~~~~~~~~~~~~~~~~~~~~~~~

Bush's man Chertoff of Homeland Security said that there will probably be an attack on America this summer by alQuaeda.


Why did Bush and the Neo-Cons totally screw up all the wars:

War in Afghanistan
War against Iraq
War against Jihadists

We are now worse off than we were on 910, no doubt about it.

tomder55 answered on 07/19/07:

Pervez Musharraf discovered that it is futility to try to negotiate with jihadistan. He ceded N Waziristan and that backfired on him .So he is beginning to get tough on them again .....witness the recent siege on a Pakistani mosque.

The NIE report appears fanciful to me. But DC makes a good point . Why are the critics of Bush so invested in the defeat of America?

MarySusan rated this answer Bad/Wrong Answer
paraclete rated this answer Excellent or Above Average Answer

Question/Answer
Choux... asked on 07/19/07 - Bush will ask for more troops in Sept after "Surge"

The Bush Administration has indoctrinated many Americans of lower intelligence into believing all his lies and misinformation about the War on Jihadists and Iraq. (See poll results of questions on the War on Iraq.)

In September after the current "Surge", Bush plans to send in *more troops*. (Washington insider rumor)

How do you feel about this?

tomder55 answered on 07/19/07:

I would unleash the air force and the fleet . They appear to be under utilized in this conflict. Bush has said he will follow General Petraeus' recomendations . Washington insiders should take a permanent vacation or get a real job.

Choux... rated this answer Average Answer
Dark_Crow rated this answer Excellent or Above Average Answer
paraclete rated this answer Excellent or Above Average Answer

Question/Answer
Choux... asked on 07/19/07 - Joe Biden on Bush

Senator Biden appeared On PBS's The NewsHour With Jim Lehrer this evening, and spoke about President Bush and the war in Iraq. While he argued that neither the Democrats or the President have offered a sustainable political solution for Iraq, he also said,

"There's a famous justice, Oliver Wendell Holmes, who once said, 'Prejudice is like the pupil of the eye. The more light you shine on it, the more tightly it closes.' Well, for this president,information is like the pupil of the eye. The more information you give it, the more tightly he rejects it. He is living in an unrealistic fantasyland about the state of affairs on the ground in Iraq."

~~~~~~~~~~~~~~~~~~~~~~~~~~~~~~~~~~~~~


No one disagrees, do they?

tomder55 answered on 07/19/07:

Of all the serious Democrats I do have to admit that at least he has offered a practical alternative to debate. I don't agree with it ;his partition plan but at least it gives a framework for a rational discourse.

His ad hominem about President Bush is a cheap shot.

Choux... rated this answer Average Answer

Question/Answer
denberg asked on 07/18/07 - Can someone answer me a question on Clinton?

Do you remember a few years ago when Clinton was being questioned by Starr about Lewinksy, he answered, "It depends on what the meaning of 'is' is."

I was incredibly impressed by this at the time, how Clinton, under pressure, outwitted a bunch of prosecutors and law professors. However this was before I had a pc and I would like to read the entire transcript of that particular conversation (or at least the few paragraphs leading up to it). I've never forgotten Starr's fury at that exact moment - slamming his fist onto the table and letting out a snort of frustration. I want to read it again...

Many thanks.

tomder55 answered on 07/18/07:

yes but you have to find the text in the deposition yourself

http://www.cnn.com/icreport/report/volume3/volume375.html

denberg rated this answer Excellent or Above Average Answer

Question/Answer
MarySusan asked on 07/18/07 - Political Live Chat Tomorrow

"...tomorrow at 2 pm EDT, as the dust settles on the Senate showdown, HuffPost will host a live chat with MoveOn.org's Tom Matzzie, who is currently spearheading a multi-state, multi-media, multi-million dollar campaign focused on turning up the heat on Republicans who are obstructing an end to the war. To take part, please send your Iraq questions, along with your name and town, to livechat@huffingtonpost.com between now and the start of the chat. Then be sure to log on to HuffPost Wednesday at 2 o'clock Eastern for this timely and vital conversation..."

~~~~~~~~~~~~~~~~~~~~~~~~~~~~~~~~~~~~~~~~~~~~~~~~~~

Join me and sign up to be a participant on the live chat event on huffingtonpost tomorrow at 2. Start sending in your questions now so you will be considered!

Mary Sue

tomder55 answered on 07/18/07:

to save time the Huffpos and Kossaks should just go to the podium and speak for themselves instead of using their Democrat stooges to do their talking for them . Yesterday's stunt was one of the Senates historic low points.

MarySusan rated this answer Bad/Wrong Answer

Question/Answer
Mathatmacoat asked on 07/18/07 - Congratulations!

I want to congratulate the good people of this Board for their good manners and the lack of spam which seems to infest certain other places on answerway

tomder55 answered on 07/18/07:

if you want good manners and a lively debate go to askmehelpdesk

arcura rated this answer Excellent or Above Average Answer
Mathatmacoat rated this answer Excellent or Above Average Answer
paraclete rated this answer Excellent or Above Average Answer
Yiddishkeit rated this answer Excellent or Above Average Answer

Question/Answer
Mathatmacoat asked on 07/18/07 - Are you bored?

I am and although I seldom post nwes articles I couldn't resist this one.


Global Warming now world's most boring topic: report
Jim Schembri
July 18, 2007

Global warming and the debate over whether man-made carbon gas emissions are having a detrimental influence on climate change has been ranked as the most boring topic of conversation on earth, according to a new report.

The issue of global warming far out-performed other contenders for the title, such as the production of goat cheese, the musical genius of the artist formerly known as P Diddy and media speculation over the likely outcome of the upcoming federal election.

These topics still tracked strongly, according to the report, but global warming was identified as the topic most likely to prompt people into feigning heart attacks so as to avoid hearing the phrases "procrastination penalty", "precautionary principle" and "peer-reviewed analysis" ever again.

The study, conducted by a non-partisan think tank located somewhere between the small township of Tibooburra and the NSW border, identified global warming as the current topic of choice for people who want their dinner party to finish early.

According to the parents in the survey, global warming has now replaced the traditional bedtime story when it comes to putting children to sleep. The study found the topic was also being used instead of water cannon by riot police around the world to disperse crowds.

In a key finding, the survey revealed that the amount of damaging carbon dioxide released into the atmosphere as a result of discussing the global warming issue now exceeds the greenhouse gas emissions of northern China.

The survey also raised a number of important issues regarding the global warming debate.

Of those surveyed, 83 per cent said that while they understood both sides of the issue, they did not understand Al Gore.

Participants in the study were asked whether Gore's film An Inconvenient Truth had helped enlighten people to the importance of the global warming issue.

The standard response was that if the issue of global warming is as important and urgent to Gore as he keeps saying every time he is on Letterman, then why didn't he make the movie during the eight years he was vice-president of the United States, the second most powerful position in the world? Why did he wait until his political career was dead?

The issue was also raised as to why Gore personally came out to promote his film in Australia - a relatively insignificant market - and then make a big deal about all the carbon off-setting he had done to counter the pollution his trip had generated. Over 95 per cent of those who took part in the survey wanted to know why he didn't just do it all from his house via satellite.

Other key findings of the survey were:

* 89 per cent wanted to know how it was possible for humans to control the climate, given that they have enough trouble forecasting it;

* 96 per cent believe those who use the term "climate change denial" are attempting to equate it with "Holocaust denial"

* 100 per cent of these respondents also believe such people should receive lengthy prison terms for crimes against the English language;

* 79 per cent of the bands that took part in the Live Earth event did so because they feared the planet would be destroyed by global warming before they had a chance to receive free worldwide television exposure;

* 87 per cent only tuned in to watch the lead singer from Sneaky Sound System, who is hot;

* 92 per cent of those same people watched her on mute because they didn't want to hear that song again;

Of all the issues raised in the survey, most common was whether the global warming debate was all just an elaborate ruse designed to sell stuff.

The study highlighted how those who subscribe to the prophecy of global warming automatically commit themselves to purchasing a vast array of expensive products, whereas sceptics don't have to buy anything to support their point of view.

Over 98 per cent of people surveyed also predicted that the standard response from global warming proponents to that last statement would be: "yeah, it won't cost anything - except the future of your planet".

tomder55 answered on 07/18/07:

Gore's an idiot . He leaves a larger carbon footprint just in his personal residence then any 25 common folk.

The topic is not boring but legitimate debated is being stifled . Scientists rush to defend faulty conclusions becasue they know where their bread is buttered .

Anyway ;here in the states when we get bored about global warming there is always Posh Beckman or Paris Hilton to talk about .

arcura rated this answer Excellent or Above Average Answer
Mathatmacoat rated this answer Excellent or Above Average Answer
paraclete rated this answer Excellent or Above Average Answer

Question/Answer
paraclete asked on 07/15/07 - It this is so why wait until April?

Iraq PM: Country Can Manage Without U.S.
AP - Sat, 14 Jul 2007 23:20:12 -0400 (EDT)
By BASSEM MROUE

Prime Minister Nouri al-Maliki shrugged off U.S. doubts of his government's military and political progress on Saturday, saying Iraqi forces are capable and American troops can leave "any time they want."

One of his top aides, meanwhile, accused the United States of embarrassing the Iraqi government by violating human rights and treating his country like an "experiment in a U.S. lab."

Al-Maliki sought to display confidence at a time when pressure is mounting in Congress for a speedy withdrawal of U.S. forces. On Thursday, the House passed a measure calling for the U.S. to withdraw its troops by spring, hours after the White House reported mixed progress by the Iraqi government toward meeting 18 benchmarks.

During a press conference,
ADVERTISEMENT
al-Maliki shrugged off the progress report, saying that difficulty in enacting the reforms was "natural" given Iraq's turmoil.

"We are not talking about a government in a stable political environment but one in the shadow of huge challenges," al-Maliki said. "So when we talk about the presence of some negative points in the political process, that's fairly natural."

Al-Maliki said his government needs "time and effort" to enact the political reforms that Washington seeks -- "particularly since the political process is facing security, economic and services pressures, as well as regional and international interference."

But he said if necessary, Iraqi police and soldiers could fill the void left by the departure of coalition forces.

"We say in full confidence that we are able, God willing, to take the responsibility completely in running the security file if the international forces withdraw at any time they want," he said.

One of al-Maliki's close advisers, Shiite lawmaker Hassan al-Suneid, bristled over the American pressure, telling The Associated Press that "the situation looks as if it is an experiment in an American laboratory (judging) whether we succeed or fail."

He sharply criticized the U.S. military, saying it was committing human rights violations and embarrassing the Iraqi government through such tactics as building a wall around Baghdad's Sunni neighborhood of Azamiyah and launching repeated raids on suspected Shiite militiamen in the capital's slum of Sadr City.

He also criticized U.S. overtures to Sunni groups in Anbar and Diyala provinces, encouraging former insurgents to join the fight against al-Qaida in Iraq. "These are gangs of killers," he said.

tomder55 answered on 07/15/07:

"We are not talking about a government in a stable political environment but one in the shadow of huge challenges," al-Maliki said. "So when we talk about the presence of some negative points in the political process, that's fairly natural."

He's correct . The Democrats set benchmarks for themselves following their 2006 congressional victory . How many of them have they achieved ? Answer .....one ;they raised the minimum wage. I laugh at the American defeatocrats. They called the surge a failure before Gen.Petraeus had time to deploy all the troops .The General was overwhelmingly confirmed by Congress for this job. During his hearings he outlined his plan . He told them that he would make his first meaningful progress report in September. Yet they are already calling it a failure.

Why ? Because the Iraqi gvt. has met only half of the phony benchmarks that our Congress imposes on them . Yes it does appear to be an American laboratory experiment . Our Congress has taken 2 or 3 lengthy vacations since January and yet complain when an Iraqi Gvt. ;whose members risk their lives just by showing up,take a month off in the middle of 130 degree F summer.

He also criticized U.S. overtures to Sunni groups in Anbar and Diyala provinces, encouraging former insurgents to join the fight against al-Qaida in Iraq.

On that issue he is wrong .There has been tremendous progress in getting Sunni cooperation in defeating foreign jihadi who are responsible for most of the problems in the country .



Dark_Crow rated this answer Excellent or Above Average Answer
paraclete rated this answer Average Answer

Question/Answer
HANK1 asked on 07/12/07 - Cleaning House:



Let's bring all of our troops home from Afghanistan and Iraq. Let's round-up all the illegals in America and send them back to where they came from. Let's get health insurance for everyone in the United States. Bottom-line: Let the greatest country on Earth take care of itself and allow us to quit worrying about everyone in the World. We should be spending more money on self-preservation instead of pooping it away on lost causes.

At least that's one scenario! However, not necessarily mine! Any comments?

HANK

tomder55 answered on 07/15/07:

You know my response already ;the road out of Iraq goes through Demascus and Tehran . What Bobby is talking about in his posting is that Syria ;taking it's marching orders from the Mahdi -hatter ;has already advanced and dug in 3 miles inside Lebanon to prop up the Hezbollah . They in turn will attempt a take over of the Lebanese gvt. in the next couple of weeks .

The US has discovered Iranian missiles in Iraq aimed at US bases ,and has confirmed that advanced IEDs are being manufactured in Iran and smuggled into Iraq . They are responsible for hundreds of the US casualties.

The solution to the immigation issue proposed is not realistic . Close the border 1st ,and enforce the laws regarding employer hirings . Then I would streamline the visa system and increase the number of visas with preference given to the higher skilled professionals who wish to enter.

As I've said many times ,we do not have the luxury of retreating to fortress America protected by large oceans on the flank anymore. It doesn't take days to get here but hours. We are in a global economy and very dependent on it.

HANK1 rated this answer Excellent or Above Average Answer

Question/Answer
paraclete asked on 07/06/07 - The Gun Nuts are loose again!

or is it that Bob Katter is loose again?

You can tell it's an election year, the red necks are out in force.

Bob Katter calls for guns to fend off crocodile 'plague'

By Brian Williams

July 06, 2007 01:00am
Article from: The Courier-Mail

* MP says croc 'plague' spreading to new towns
* Says people should be armed and allowed to shoot
* Minister rejects 'gun-happy hillbillies' plan

ALL north Queensland residents and travellers should be armed with high-calibre weapons to protect themselves from crocodiles, according to maverick MP Bob Katter.

Mr Katter said croc numbers had reached plague proportions and the reptiles were being seen in places never before recorded.

"This is unprecedented in human history," Mr Katter said.

"They are knocking (ecosystems) out of gear ... and when you get them in those numbers, they are eating barra that have never been eaten before."

He said northerners were increasingly worried about the reptiles - which were almost exterminated by hunters in the 1960s - after sightings near beaches, boat ramps and swimming holes.

Croc sightings are reasonably common around Cairns and Townsville, where residents reported crocodiles off the town beach late last year.

Queensland Liberal Senator Ian Macdonald is also a longtime proponent of culling and northern MHR Warren Entsch has argued that trophy hunting might be a way of bringing revenue to remote communities.

Mr Katter said the state Government's crocodile management plan, which did not allow culling, was a joke.

"Should we have our kids eaten because of a decision by some absolute imbecilic public servant who's never been off the bitumen in Brisbane?" he said.

"People should be armed. What do they want us to do? Knock them on the head with a hammer?"

Environment Minister Lindy Nelson-Carr said Mr Katter's suggestions were ridiculous.

"The last thing Queensland needs is a bunch of gun-happy hillbillies on a crocodile shoot," she said.

Australia Zoo spokesman Wes Mannion, who captured rogue crocodiles with wildlife celebrity Steve Irwin, said there were many things in the world more dangerous than crocodiles.

"What about Mr Katter's car? Cars kill thousands of people each year," Mr Mannion said.

Environmental Protection Agency wildlife conservation branch director Rebecca Williams said there had been 17 attacks since 1985, and of those five were fatal.

Research by Queensland Parks and Wildlife scientist Mark Read has shown that about 60 per cent of people involved in incidents with crocodiles had been drinking.

tomder55 answered on 07/07/07:

this sounds very much like the problem e have here with deer and black bear. Conservation efforts were needed to avoid extinction but now they are not in danger .

The other problem is that human population leaving the cities is now encroaching on lands where these critters dwell. Of course human enounters with them are increasing.

Culling is indeed one way of dealing with the problem . On Fire Island NY the deer population got rediculously large. There was no longr a natural predator for them and they were becoming domesticated. The solution was to send in licensed hunters who kill em and grill em to shoot em. Once the population became more realistic the hunt stopped.

paraclete rated this answer Excellent or Above Average Answer

Question/Answer
Choux... asked on 07/02/07 - BRING THEM ON!!!!

Today is the fourth anniversary of the famous Bush expression named above:

"..anybody who wants to harm American troops will be found and brought to justice. There are some who feel like that if they attack us that we may decide to leave prematurely. They don't understand what they're talking about, if that's the case.

Let me finish. There are some who feel like -- that the conditions are such that they can attack us there. My answer is, bring them on!"


~~~~~~~~~~~~~~~~~~~~~~~~~~~~~~~~~~~~~~~~~~~~~


Comments?

tomder55 answered on 07/03/07:

and this is what he said last year :

“Saying `bring it on,’ kind of tough talk, you know, that sent the wrong signal to people,” the president said in answer to a question about mistakes he made in Iraq. “I learned some lessons about expressing myself maybe in a little more sophisticated manner — you know, `wanted dead or alive,’ that kind of talk.”

All during the Presidential campaign in 2004 a big to do was made over the fact that he would not publicly admit any mistakes. Now we know why . Moonbats will continue to gloat whether he admits them or not.But intellectual honesty is not a strength of the left.

Perhaps they bring up this non-issue because they cannot use news from the front to bolster their argument anymore ....so they dwell in the past.



Choux... rated this answer Bad/Wrong Answer
ETWolverine rated this answer Excellent or Above Average Answer

Question/Answer
Dark_Crow asked on 06/22/07 - Why do the European’s hate America?..............

I’m told it is because of capitalism; especially Germany, others say it is still a “Jewish” problem. That the moods of the rural populations reflect the mood of the thirties, and when the economy fails, and it will, there will be hell to pay.

Which brings me to the other question: Why don’t we pay attention to where our future lies, across the Pacific, and forget about the E.U. and especially Germany; where Democracy will soon end - That is, stop asking why certain people are not our friends (I already know, they have become Socialist.) .

tomder55 answered on 06/22/07:

Why do they hate us ? I liken it to those old spaghetti westerns. The weak townfolk would cower at the bad guys and beg the governor to send a sheriff .A Clint Eastwood type sheriff would come in and clean up the town and although they appreciated the efforts of the sheriff ;in fact they were just as afraid of him as they were of the bad guys. They long for those good ole days when they did not need to have a sheriff from outside to protect them .

You are correct in your assessement that our future is in the Asia ( I don't say Pacific because you also have to factor the Indian sub-Continent . I think our President understands this and that is why he negotiated the controversial pact with India over nuclear development .

If you don't get the Atlantic Magazine you should pick up this month's issue (July /August ) .It is a special China issue . The front page story : "Why China's rise is good for us " . I don't necessarily buy into the premise but I have not completed the issue yet .

Dark_Crow rated this answer Excellent or Above Average Answer
ETWolverine rated this answer Excellent or Above Average Answer

Question/Answer
paraclete asked on 06/22/07 - Now that's disgusting


Get set for a rude awakening

June 22, 2007 01:00am
Article from: The Daily Telegraph


AUSTRALIAN politics has been rocked this week by the shocking revelation, captured on film, that swearing occurs on building sites.

This follows horrific footage last week that some construction workers wear bad T-shirts and carry a bit of weight around the midriff.

Footage of a West Australian union official, forced by a federal government officer to leave an unsafe building site, responding with some choice expletives, has sent the political classes into a lather.

Hold the presses.

Building workers swear, they also curse, cuss and diss.

They do so in an industry where a worker dies, on average, every week.

Because lives and limbs are at stake tempers can flare, particularly when workers are told they can not take a stand on safety.

Before we watch the union official in question hoisted on a four-letter cross today, it's worth asking ourselves whether swearing in the Australian workplace is such an outrage.

Let's start on the cricket pitch.

For generations, Australia's representatives have used the well-aimed expletive as a way of unsettling and, yes, intimidating an opponent.

Back in 1932, at the height of bodyline, Australian players accused English captain Douglas Jardine of being a "bastard".

When Jardine complained at the close of play to Australian captain Victor Richardson, he turned to the dressing room and asked: "Which of you bastards called this bastard a bastard?"

Richardson's grandson Ian Chappell nurtured that tradition, pioneering what we now refer to as "sledging", which has turned into a fine art in recent years.

Matthew Hayden, Darren Lehmann and, of course, Shane Warne have all been pinged letting slip colourful adjectives.

What can we learn from our cricketers?

Probably that the best response to verbal abuse is humour.

When an Aussie cricketer asked Zimbabwean tail-end batsman Eddie Brandes why he was so fat, Brandes replied: "Because every time I sleep with your wife she gives me a biscuit."

Game, set and match to Eddie.

TV and radio recording studios are other workplaces where colourful language can sometimes be heard.

The doyen of talkback Alan Jones has an impressive back-catalogue of off-mike sprays, letting fly at everything from "f..king dust in the studio" to having to "catch a plane to raise f..king money for charity."

For former Today Tonight host Naomi Robson, things turned blue when there was an autocue problem.

"F..king drop it in at the last minute ... You should be able to read every f..king word, every comma. It should be very specific because if we've seen it before we've got a chance in our mind to go 'oh look they've f..ked up this, they've f..ked up that."'

As Naomi looks for a new career, maybe she should consider the building industry.

Which brings us to our politicians, whose workplace has the procedural equivalent of a swear-box, known as un-Parliamentary language.

It means that you can say basically anything you like about an opponent, as long as you then withdraw it from the record.

Things are not so simple outside Parliament.

When former Victorian premier Jeff Kennett and then Opposition front-bencher Andrew Peacock were caught on mobile phone discussing our Prime Minister's character they actually invented some brand new terms of abuse.

When media outlets attempted to air the tape the required beeps were so frequent it sounded like an old Roadrunner cartoon.

Interestingly, on becoming PM, John Howard rewarded Peacock for his creativity by appointing him to Australia's most sensitive diplomatic posting, ambassador to the US.

The final word on swearing should really go to the Howard Government.

When deciding the best way to market Australia abroad, what did they do?

They put a model in a bikini and paid her to swear.

Workers can rightly be asking today: "Where the bloody hell is the consistency?"

When did it become a crime to swear on a building sites?

And why is this front page news?

* Dave Noonan is national secretary CFMEU construction division

tomder55 answered on 06/22/07:

What ? With keeping the aboriginals from getting their daily brew I would think their plate was full !

If swearing scares off an Iranian sailor pirate I'm all for it.

paraclete rated this answer Excellent or Above Average Answer

Question/Answer
paraclete asked on 06/22/07 - A lesson for George and Condi

on how to deal with the Iranians


'Robust' Aussies fend off Iranians

June 22, 2007 08:38am
Article from: AAP



* Colourful language helps Aussies avoid capture
* Aussies react quickly, Brits caught at most vulnerable
* Video: 'You can't take us, we're Aussies'

AN Australian Navy boarding crew in the Gulf of Persia repelled an Iranian gunboat that threatened them a matter of weeks before 15 British sailors were captured in a similar incident, it was reported today.

The capture of the British crew in March developed into a major diplomatic incident before their release was negotiated.

But BBC reporter Frank Gardner, a security specialist, reported the Australians managed to avoid a similar incident - pointing their guns at the Iranians and used "colourful language" before a gunboat withdrew.

Escape

"What I've been told by several sources, military sources, (is that) there was a similar encounter, in this case between the Royal Australian Navy and Iranian gunboats, some months ago, or at least some months prior to the seizing of the British sailors," Gardner said on ABC radio.

"The Australians escaped capture by climbing back on board the ship they'd just searched. I'm told that they set up their weapons.

"No shots were exchanged but the Iranians backed off and the Australians were able to get helicoptered off that ship and they didn't get captured."

Robust attitude

He did not mention the name of the Australian ship.

Australians ships rotate through duties in the Gulf, chiefly searching ships.

"What I'm hearing is that it was a pretty robust attitude by the Australians," Gardner said.

"The words that somebody said to me was that they used pretty colourful language but I'm sure that alone didn't make the Iranians back off.

"They reacted, I'm told, incredibly quickly, whereas the Brits were caught at their most vulnerable moment climbing down off the ship (and) getting into their boats."

Gardner said the British should be embarrassed about the incident, but the issue was whether military intelligence had been passed on.

Cowards

"The point of this story is not that the Aussies were fantastically brave and the Brits were a bunch of cowards, although I'm sure some people will interpret (it that way)," he said.

"Lessons should have been drawn from what happened to the Australian crew."

He said he had not been able to find out whether the information on the Australian incident had been passed on to the British.

Prime Minister John Howard said today he was not in a position to confirm the report, but told Channel 7: "I'll be getting some further advice on it later this morning.

"The only thing I can say is that the people we have in the Gulf are engaged in very dangerous work and the RAN has done a fantastic job and a very courageous job.

"As to the particulars of that claim, I'm not advised."

tomder55 answered on 06/22/07:

This makes the British incident look even more rediculous than it was. If there was advanced notice that the Iranians were making this type of attempt then the Brits should've been prepared. Unfortunately in no way does the British navy resemble the navy of 'Britannia rules the waves' fame. It has been gutted and more reductions are in their future.

I like the Aussie "robust attitude ".Sounds like just what the doctor ordered.

ETWolverine rated this answer Excellent or Above Average Answer
paraclete rated this answer Excellent or Above Average Answer

Question/Answer
Mathatmacoat asked on 06/21/07 - Hey Mary Susan what happened to the politics board

since you turned up it has disappeared.

tomder55 answered on 06/21/07:

go to askme help desk it is a cite that has real moderators who care about the cite.

labman rated this answer Excellent or Above Average Answer
Mathatmacoat rated this answer Excellent or Above Average Answer
paraclete rated this answer Excellent or Above Average Answer
Toms777 rated this answer Excellent or Above Average Answer

Question/Answer
Dark_Crow asked on 06/16/07 - Mass. preserves gay marriage

The state legislature defeated a constitutional amendment to let the voters decide on a ban.
"We're proud of our state today, and we applaud the legislature for showing that Massachusetts is strongly behind fairness," (homosexuality)said Lee Swislow, executive director of Gay & Lesbian Advocates & Defenders.

What perversion next?

tomder55 answered on 06/16/07:

From strictly a political standpoint these state by state decisions are not tenable. Why ? Because the Constitution is very explicit on the issue. Article IV, Section 1 of the Constitution is commonly known as the Full Faith and Credit Clause.It states :

Full Faith and Credit shall be given in each State to the public Acts, Records, and judicial Proceedings of every other State.

And the Congress may by general Laws prescribe the Manner in which such Acts, Records and Proceedings shall be proved, and the Effect thereof.


Eventually this has to become a national issue. The clause stipulates that barring Congressional intervention ,(or Constitutional Amendment) ,the contracts exercised in Mass.(or San.Fran. or any place else that permits gay marriage) must be honored in Miss.or any other State that by statute prohibits it.

What perversion next ? Rick Santorum created alot of controversy with this comment :"In every society, the definition of marriage has not ever to my knowledge included homosexuality. That's not to pick on homosexuality. It's not, you know, man on child, man on dog, or whatever the case may be."........"If the Supreme Court says that you have the right to consensual [gay] sex within your home, then you have the right to bigamy, you have the right to polygamy, you have the right to incest, you have the right to adultery. You have the right to anything."







Dark_Crow rated this answer Excellent or Above Average Answer

Question/Answer
MarySusan asked on 06/14/07 - SITE OF FUTURE IMPROVED POLITICS BOARD!!

I plan on upgrading this Board by inviting new folks to participate, citizens who want a lively give and take about political issues that effect the lives of all decent citizens.

People who want to learn about the serious issues of our time, and people who want to share their knowledge (not propaganda and/or hate)with others for the betterment of all concerned.

STAY TUNED FOR FUTURE ANNOUNCEMENTS.

tomder55 answered on 06/14/07:

talking to yourself again ?

JacquelineA2006 rated this answer Excellent or Above Average Answer
MarySusan rated this answer Poor or Incomplete Answer
paraclete rated this answer Excellent or Above Average Answer

Question/Answer
CeeBee2 asked on 06/08/07 - Our depleted military--------------------

from the Internet:

Col. Andy Bacevich, America's foremost writer on military affairs, documents in the current issue of The American Conservative that Bush's insane war has depleted and exhausted the US Army and Marine Corps:

"Only a third of the regular Army's brigades qualify as combat-ready. In the reserve components, none meet that standard. When the last of the units reaches Baghdad as part of the president's strategy of escalation, the US will be left without a ready-to-deploy land force reserve."

"The stress of repeated combat tours is sapping the Army's lifeblood. Especially worrying is the accelerating exodus of experienced leaders. The service is currently short 3,000 commissioned officers. By next year, the number is projected to grow to 3,500. The Guard and reserves are in even worse shape. There the shortage amounts to 7,500 officers. Young West Pointers are bailing out of the Army at a rate not seen in three decades. In an effort to staunch the losses, that service has begun offering a $20,000 bonus to newly promoted captains who agree to stay on for an additional three years. Meanwhile, as more and more officers want out, fewer and fewer want in: ROTC scholarships go unfilled for a lack of qualified applicants."

*******************

One of my library volunteers was wooed by the Navy. Twenty-six y/o Barlaham is from Colombia and was promised a quicker route to citizenship, healthy paychecks, college, and was told about the benefits of the GI Bill if he signed up for the requisite four years. He did so, wanting a better life for himself, his wife, and his two stepdaughters. He had met with recruiters from all military branches and deliberately chose the Navy with the hope he won't end up getting killed in a conflict with a Middle Eastern country. He told me he would never have signed up with the Army or Marines.

tomder55 answered on 06/08/07:

Military Meets, Exceeds Recruiting Goals

By PAULINE JELINEK
The Associated Press
Tuesday, December 12, 2006; 11:11 PM

WASHINGTON -- Though Americans are increasingly pessimistic about the war in Iraq, the Pentagon said Tuesday it is having success enlisting new troops. The Navy and Air Force met their recruiting goals last month while the Army and Marine Corps exceeded theirs , the Defense Department announced.


Col. Andy Bacevich's opposition to the Iraq war is well known ;but I think part of his harshness is bitterness because he lost his son in the conflict . He may be a writer of military affairs but he is hardly the foremost and his is just one of many opinions by former officers that cover the whole spectrum of the debate. Frankly ;he sounds like Congressman Murhta and his hysterical 'the Army is broken 'screed. I know Sr. officers who would beg to differ .

CeeBee2 rated this answer Excellent or Above Average Answer

Question/Answer
CeeBee2 asked on 06/05/07 - More fun -- fencing our southern border...............

From Digger online --

Experts (And Digger) Say Virtual Fence Won't Work
By Digger

Experts -- and I -- agree that virtual fences don't work. They've been tried and failed. However where real fences have been tried, like in San Diego, they have been very effective. The cost differential is mild compared to the amount of money already thrown away on these experimental programs.

Washington Post

Since 1995, spending on border security has increased tenfold, from $1.2 billion to $12.7 billion, and the number of Border Patrol agents has more than doubled, from 5,000 to 12,319, according to the House Appropriations Committee. Yet the number of illegal immigrants in the United States has jumped from 5 million to more than 11 million.

In that same time no less than 2 "virtual fences" have been tried and failed. They obviously don't work and other than someone scratching someones business buddies back to the tune of $2.5 billion in taxpayer money, the latest "virtual fence", called the Secure Border Initiative Network (SBInet) awarded to Boeing, is doomed to failure as well.

The Department of Homeland Security and the former Immigration and Naturalization Service spent $429 million since 1998 on video and remote surveillance on the borders. But nearly half of 489 planned cameras were never installed, 60 percent of sensor alerts are never investigated, 90 percent of the rest are false alarms, and only 1 percent overall resulted in arrests, the Homeland Security inspector general reported in December.

That was really successful! And people are complaining about a few billion for a real fence.

Last week the House passed HR 6061 The Secure Fence Act and the Senate will be voting on HR 6061 which provides 700 miles of physical border fence next week.

"There has been a huge amount of money poured into the border . . . but the track record of the performance of these technologies is disappointing," said Doris Meissner, former INS commissioner.

Because they don't work and people keep proposing them because they don't want them to work. There are enemies in our midst that want these illegal aliens to keep pouring across. They make big money on them. Of course it's at the expense of the American taxpayer.

I'd say these "virtual fences" are "virtually" worthless.

tomder55 answered on 06/05/07:

I totally agree .Unless they help monitor solid walls they are useless and a virtual panacea .

To the broader point ;none of us here to my knowlege opposes legal immigration . If the country needs and expanded work force then by all means it should be easier and less cumbersone for immigrants to go through the process.

That being said ;back in 1986 a general amnesty was supposed to be coupled with all other guarantees to restrict the flow of illegals into the country. That bill was passed and no effort was made at the border control end of the bargain .12-15 million illegals later we are effectively being offered the same bargain again ..

Well ...fool me once.............

That is essentially the conservative opposition to the current plan being debated. WE just do not believe they take border security seriously . In a 9-11 world that is not acceptable.

CeeBee2 rated this answer Excellent or Above Average Answer

Question/Answer
CeeBee2 asked on 06/05/07 - Just for fun -- the more things change................

It's interesting that the Republicans had a different message when Clinton was President:

KY's Senior Senator Mitch McConnell said during a speech on the floor of the U.S. Senate
"Domestic terrorism is not a cause we have to fight or a project we need to fund. We are not interested in capturing bin Laden. Even though he has been offered to us. We are not the world's policemen. It's not our job to clean up other countries messes or arrest it's bad guys."

VERBATIM QUOTES FROM WHEN CLINTON WAS COMMITTING TROOPS TO BOSNIA:

"You can support the troops but not the president."--Rep Tom Delay (R-TX)

"Well, I just think it's a bad idea. What's going to happen is they're going to be over there for 10, 15, maybe 20 years."--Joe Scarborough (R-FL)

"Explain to the mothers and fathers of American servicemen that may come home in body bags why their son or daughter have to give up their life?"--Sean Hannity, Fox News, 4/6/99

"[The] President . . . is once again releasing American military might on a foreign country with an ill-defined objective and no exit strategy. He has yet to tell the Congress how much this operation will cost. And he has not informed our nation's armed forces about how long they will be away from home. These strikes do not make for a sound foreign policy."--Sen. Rick Santorum (R-PA)"

American foreign policy is now one huge big mystery. Simply put, the administration is trying to lead the world with a feel-good foreign policy."--Rep Tom Delay (R-TX)

"If we are going to commit American troops, we must be certain they have a clear mission, an achievable goal and an exit strategy."
--Karen Hughes, speaking on behalf of George W Bush

"I had doubts about the bombing campaign from the beginning . . I didn't think we had done enough in the diplomatic area."--Senator Trent Lott (R-MS)

"I cannot support a failed foreign policy. History teaches us that it is often easier to make war than peace. This administration is just learning that lesson right now. The President began this mission with very vague objectives and lots of unanswered questions. A month later, these questions are still unanswered. There are no clarified rules of engagement. There is no timetable. There is no legitimate definition of victory. There is no contingency plan for mission creep. There is no clear funding program. There is no agenda to bolster our over-extended military. There is no explanation defining what vital national interests are at stake. There was no strategic plan for war when the President started this thing, and there still is no plan today"--Rep Tom Delay (R-TX)

And the quote that wins the top prize:

"Victory means exit strategy, and it's important for the President to explain to us what the exit strategy is."-- Governor George W. Bush (R-TX)

tomder55 answered on 06/05/07:

good job Ceebee . As one who supported the Balkans campaigns I'd have to say that those quoted above were wrong then and generally right now. My guess is that they mistakenly reflected the realist impulse of the Republicans in the pre- 9-11 world.(which is still reflected in the attitudes of people like James Baker) You will recall that I have constantly compared President Bush to a Democrat President ;Harry Truman . Just this week there was another article comparing him to another Democrat war time President ..Woodrow Wilson . The comparisons are valid in the basic belief by all three that democracy can and should be exported .

I guess you've already heard all the Democrat quotes from the 1990s about the need for regime change in Iraq (for many of the same reasons btw that were used to justify the war against Serbia. ). Do you equally say that they were wrong ?

CeeBee2 rated this answer Excellent or Above Average Answer

Question/Answer
Itsdb asked on 06/05/07 - The latest on global warming...again

Get the tin-foil hats out again:

Agencies cut back efforts to monitor global warming from space

    The Bush administration is drastically scaling back efforts to measure global warming from space, just as President Bush tries to convince the world the United States is ready to take the lead in reducing greenhouse gases.

    A confidential report to the White House, obtained by The Associated Press, warns that U.S. scientists will soon lose much of their ability to monitor warming from space using a costly and problem-plagued satellite initiative begun more than a decade ago.

    Because of technology glitches and a near-doubling in the original $6.5 billion cost, the Defense Department has decided to downsize and launch four satellites paired into two orbits, instead of six satellites and three orbits.


    The satellites were intended to gather weather and climate data, replacing existing satellites as they come to the end of their useful lifetimes beginning in the next couple of years.

    The reduced system of four satellites will now focus on weather forecasting. Most of the climate instruments needed to collect more precise data over long periods are being eliminated.

    Instead, the Pentagon and two partners -- the National Oceanic and Atmospheric Administration and NASA -- will rely on European satellites for most of the climate data.

    "Unfortunately, the recent loss of climate sensors ... places the overall climate program in serious jeopardy," NOAA and NASA scientists told the White House in the Dec. 11 report obtained by the AP.

    They said they will face major gaps in data that can be collected only from satellites about ice caps and sheets, surface levels of seas and lakes, sizes of glaciers, surface radiation, water vapor, snow cover and atmospheric carbon dioxide.

    Rick Piltz, director of Climate Science Watch, a watchdog program of the Washington-based Government Accountability Project, called the situation a crisis.

    "We're going to start being blinded in our ability to observe the planet," said Piltz, whose group provided the AP with the previously undisclosed report. "It's criminal negligence, and the leaders in the climate science community are ringing the alarm bells on this crisis."


It's "criminal negligence" to quit wasting so much money on a "problem-plagued satellite initiative"? Here's an idea, let's make this thing work before we drive the screws deeper into the American people. Shouldn't it be criminal to give idiots like this a public forum?

The UN has issued its latest IPCC installment:



If global warming was doing things at a "chilling pace" wouldn't it be cooling? We're now back to the drastic sea rise scenario:



Surfing anyone?

tomder55 answered on 06/05/07:

I don't suppose that Piltz would consider launching his own sat.monitoring system. Perhaps he could lend his technical expertise to the NASA and Dept. Def. engineers who are tasked to making projects like this work.

As I recall;it took a manned mission to Hubble to give the telescope a pair of contact lenses for it to start sending back those beautiful pictures of deep space.

As I recall ;a couple of Mars probes crashed after months in transit to the planet;one of them because the engineers neglected to properly convert English to metric measurements .

As I recall we tragically lost 2 shuttles.One because we neglected to consider the possibilities of O rings being compromised by cold weather ;and one because we did not figure that foam coming off the fuel tank during lift off would damage heat shields.

As I recall NASA projects are started and cancelled all the time. Many are plagued by cost over-runs and are ill conceived in design or utility .

Are all these criminally negligent in nature ? Maybe Piltz should spread his moon-bat wings and take flight ;and monitor the global climate himself. No other data would convince him anyway.

Itsdb rated this answer Excellent or Above Average Answer

Question/Answer
Itsdb asked on 06/04/07 - Oops

Another setback in the Dems pledge to end corruption in congress...

    Grand jury indicts Rep. Jefferson in bribery investigation

    WASHINGTON - Rep. William Jefferson, D-La., was indicted Monday on federal charges of racketeering, soliciting bribes and money-laundering in a long-running bribery investigation into business deals he tried to broker in Africa.

    The indictment handed up in federal court in Alexandria., Va., Monday is 94 pages long and lists 16 alleged violations of federal law that could keep Jefferson in prison for up to 235 years, according to a Justice Department official who has seen the document.

    Among the charges listed in the indictment, said the official, are racketeering, soliciting bribes, wire fraud, money-laundering, obstruction of justice, conspiracy and violations of the Foreign Corrupt Practices Act. The official spoke on condition of anonymity because he was not authorized to discuss the case.

    Jefferson is accused of soliciting bribes for himself and his family, and also for bribing a Nigerian official.

    Almost two years ago, in August 2005, investigators raided Jefferson’s home in Louisiana and found $90,000 in cash stuffed into a box in his freezer.

    Jefferson, 63, whose Louisiana district includes New Orleans, has said little about the case publicly but has maintained his innocence. He was re-elected last year despite the looming investigation.

    Jefferson, in Louisiana on Monday, could not immediately be reached for comment.

    Two of Jefferson’s associates have already struck plea bargains with prosecutors and have been sentenced.

    Brett Pfeffer, a former congressional aide, admitted soliciting bribes on Jefferson’s behalf and was sentenced to eight years in prison.

    Another Jefferson associate, Louisville, Ky., telecommunications executive Vernon Jackson, pleaded guilty to paying between $400,000 and $1 million in bribes to Jefferson in exchange for his assistance securing business deals in Nigeria and other African nations. Jackson was sentenced to more than seven years in prison.

    Both Pfeffer and Jackson agreed to cooperate in the case against Jefferson in exchanges for their pleas.

    The impact of the case has stretched across continents and even roiled presidential politics in Nigeria. According to court records, Jefferson told associates that he needed cash to pay bribes to the country’s vice president, Atiku Abubakar.

    Abubakar denied the allegations, which figured prominently in that country’s presidential elections in April. Abubakar ran for the presidency and finished third.

    Court records indicate that Jefferson was videotape taking a $100,000 cash bribe from an FBI informant. Most of that money later turned up in a freezer in Jefferson’s home.


Well so what? It was just Bush overstepping boundaries again by conducting this raid in the first place, right? Maybe the Dems have contributed to ending the "culture of corruption" after all, since they no longer have "a taxpayer-subsidized sanctuary for crime."

Comments?

tomder55 answered on 06/05/07:

I wonder how much of the case was built because of the raid. Seems like having him on tape committing the crime and finding the evidence stashed in the freezer would be enough to indict. I had no doubt that the raid was constitutional but I would use that approach very cautiously .

When Jefferson is convicted it will represent the 2nd largest case of corruption ;2nd only to Duke Cunnigham. Seems like the culture of corruption is a broad enough street for both parties to travel on.....The one thing I have seen truely bi-partisan coming out of Congress.

Itsdb rated this answer Excellent or Above Average Answer

Question/Answer
kindj asked on 06/04/07 - I hate to say "I told you so..."

Dunno if anyone remembers (or if I even said it here), but I've maintained since day 1 that Putin is a bad guy, who wants nothing more than a return to the bad ol' days.

Well, here's another reason:


Updated: 8:15 a.m. CT June 4, 2007
MOSCOW - Russian President Vladimir Putin warned that Moscow could take “retaliatory steps” if Washington proceeds with plans to build a missile defense system for Europe, including possibly aiming nuclear weapons at targets on the continent.

Read the rest: http://www.msnbc.msn.com/id/19013495/

I don't like the guy. I don't trust the guy.

DK

tomder55 answered on 06/04/07:

That's because you did not look him in the eyes and see his soul . We did not see the good former KGB strongman as Bush did . He's got to keep up appearances on the home front. The homies don't consider him a good Euroweenie like we do . They think he's a "bad guy" who murders his opposition and is systematically reversing Russian democracy.

Our intent of course is to shield Europe from nuclear black mail by Islamo-nazi nukes in the future. We have offered to share the technology with Russia but Putin at least dismisses the idea of an Islamic threat.
How could Russian developed Iranian missiles possibly be a threat to Russia ?

I also think alot of this has to do with the upcoming G-8 summit. In the past he was able to get back up by his good buddies Chirac and Schroeder. But new leadership in France and Germany have changed the equation. Angela Merkel is much more wary of the Russians and Nicolas Sarkozy,has condemned human rights abuses by pro-Russian forces in Chechnya .


What was Rush's old line about Gorbachov ? The only difference between him and any other Russian leader of the past is that he's breathing.

kindj rated this answer Excellent or Above Average Answer

Question/Answer
kindj asked on 06/03/07 - Just heard on the news...

Fred Thompson is running! Just heard on NBC news.

Cool?

DK

tomder55 answered on 06/04/07:

yup ;he handed in his resignation to 'Law and Order' and to Paul Harvey so it looks like he's in .

The good thing is that at the grass-roots level his organization has been developing without his go-ahead . I agree with him that it is not too late. I think he could enter as late as November and still be a factor in the early primaries held in Feb. next year. (look for Al Gore to enter just after he wins the Nobel Peace Prize in the fall. )


The Wall Street Journal editorialized that Thompson's first big test of viability will be his ability to raise money quickly. They are skeptics about his ability to raise sufficient funds on the internet. But Howard Dean proved in 2004 that it could be done. The fact that he imploded after the fact had nothing to do with his fund raising abilities .

Thompson argues :"..the biggest problem we have today is what I believe is the disconnect between Washington, D.C., and the people of the United States. People are looking around at the pork barrel spending and the petty politics, the backbiting. The fighting over all things, large or small, is creating a cynicism among our people."
That cynicism is why Thompson believes that an unconventional run for the White House will have traction. To the skeptics Thompson says plainly :"it's too late to follow those rules even if I wanted to, and I don't want to."


I for one can't wait to see it. Item by item ;I agree with his positions more than any other candidate . Additionally he communicates them in a Linconesque or Will Rogers sorta down home common sense style that is easily understandable .

John Fund at the Wall Street Journal put it this way ; ...what Fred Thompson demonstrated to an enthusiastic Virginia Republican Party dinner Saturday is that he has gravitas, a presence and the ability to make people comfortable.


My only reservations is the question :Can he win in the general elections ?


Here is Thompson's most recent commentary on the Paul Harvey radio cast .As usual he is 100% correct.

Itsdb rated this answer Excellent or Above Average Answer
kindj rated this answer Excellent or Above Average Answer

Question/Answer
Itsdb asked on 06/01/07 - Global warming updates

In my paper today the AP painted Bush's announcement to press for "a global target for reducing greenhouse gases" as Bush doing a 180. Another AP article revealed (surprise) that environmental groups were quick to criticize Bush's plan.

    Friends of the Earth president Brent Blackwelder called the proposal "a complete charade. It is an attempt to make the Bush administration look like it takes global warming seriously without actually doing anything to curb emissions."

    National Environmental Trust president Philip Clapp said, "This is a transparent effort to divert attention from the president's refusal to accept any emissions reductions proposals at next week's G8 summit. After sitting out talks on global warming for years, the Bush administration doesn't have very much credibility with other governments on the issue."

    And, Daniel Weiss, climate strategy director for the liberal Center for American Progress, said the Bush administration has a "do-nothing" policy on global warming despite U.S. allies' best efforts to spur U.S. reductions.


Alrighty then, he hasn't done a thing yet and it's already a "complete charade" and a diversion - but at least he intends to bring China and India into the discussion.

What's more intersting is NASA Administrator Michael Griffin's interview with NPR yesterday. Greg Easterbrook recently criticized NASA in Wired magazine:

    Here are NASA's apparent current priorities: (1) Maintain a pointless space station. (2) Build a pointless Motel 6 on the moon. (3) Increase humanity's store of knowledge by studying the distant universe. (4) Keep money flowing to favored aerospace contractors and congressional districts.

    Only one priority of four correct! Worse, NASA's to-do list neglects the two things that are actually of tangible value to the taxpayers who foot its bills — research relevant to environmental policymaking and asteroid-strike protection.


Griffin responded in the interview:

    MR. INSKEEP: Well, I guess he is arguing on one level that - simply that there are other priorities out there. Are there priorities that you've had to cut in the last couple of years as you've reoriented toward the moon and other things that you've regretted having to cut?

    DR. GRIFFIN: Well, we have not cut any major priorities. Again, the regrets would always be that there are things left undone that we could do. But I think we've got the best 16-1/2-billion-dollar space program that we could have. But 16-1/2 billion dollars will buy many different kinds of space programs. The question is, in a democratic society, who gets to choose. Unfortunately for Greg, it's not him.

    MR. INSKEEP: One thing that's been mentioned that NASA is perhaps not spending as much money as it could on is studying climate change, global warming from space. Are you concerned about global warming?

    DR. GRIFFIN: I am aware that global warming - I am aware that global warming exists, I understand that the bulk of scientific evidence accumulated supports the claim that we've had about a one degree Centigrade rise in temperature over the last century to within an accuracy of about 20 percent.

    I'm also aware of recent findings that appear to have nailed down - pretty well nailed down the conclusion that much of that is manmade. Whether that is a long-term concern or not, I can't say.

    MR. INSKEEP: And I just wanted to make sure that I'm clear. Do you have any doubt that this is a problem that mankind has to wrestle with?

    DR. GRIFFIN: I have no doubt that global - that a trend of global warming exists. I am not sure that it is fair to say that it is a problem we must wrestle with. To assume that it is a problem is to assume that the state of earth's climate today is the optimal climate, the best climate that we could have or ever have had, and that we need to take steps to make sure that it doesn't change.

    First of all, I don't think it's within the power of human beings to assure that the climate does not change, as millions of years of history have shown. And second of all, I guess I would ask which human beings, where and when, are to be accorded the privilege of deciding that this particular climate that we have right here today, right now, is the best climate for all other human beings. I'm - I think that's a rather arrogant position for people to take.


    MR. INSKEEP: Is that thinking that informs you as you put together the budget - that something is happening, that it's worth studying, but you're not sure that you want to be battling it as an army might battle an enemy?

    DR. GRIFFIN: Nowhere in NASA's authorization, which of course governs what we do, is there anything at all telling us that we should take actions to effect climate change in either - in one way or another. We study global climate change - that is in our authorization. We think we do it rather well. I'm proud of that. But NASA is not an agency chartered to, quote, "battle climate change."


I love it...

tomder55 answered on 06/02/07:

Griffin's comments come after NASA scientist and chief source for Al Gore's screed James Hanson made 'chicken -little' comments that we had a decade to deal with global warming at most (a prediction he has been making every year for the past decade ).

As you know from the MSM coverage Hanson and some others were quick to slam Griffin .They convieniently ignore the scientists who rallied to Grifin's side.

It is fair to critique NASA's comittment to the ISS but that is hardly their only project or achievements ;and as you point out ,much of their work focuses on projects that will have a deep impact on the advancement of human knowlege of our own planet .(disclosure : that Northrop Grumman continues to receive contracts from NASA is in my interest ).

We have already posted data showing the Mars icecaps are melting

btw.. Reports from Neptune the furthest planet from the sun ;show that Neptune is warming also. As far as I know ,no one drives an SUV there and the report says there is no accumulation of greenhouse gases there .This further advances my favorite theory that the sun has alot to do with climate change.

As for President Bush's announcement .... well at least he is realistic enough to say that any comprehensive global policy to address global warming would not have a snowball's chance in hell to succeed if the worlds largest populated countries and emerging industrial nations are not included in the comprehensive plan.

Itsdb rated this answer Excellent or Above Average Answer

Question/Answer
ETWolverine asked on 06/01/07 - Follow-up of Steve's Global Peace Index post

I have now read the report. Thanks, Steve, for the link to the report.

Here's my take on it.

The Global Peace Index report put out by Visions of Humanity makes certain assumptions about what constitutes “peace” that are not necessarily true.

The report used 24 separate indicators of what constitutes peace, each quantified, translated to a 1-5 score and then given a weight within the overall index. The problem is that not all of the indicators used necessarily mean what the report assumes they mean. For instance, the report looks at the number of guns per 100,000 people as an indicator of “internal militarism” and “crime”, when in fact the opposite may be true… gun ownership may very well be a deterrent to both crime and internal militarism. Another example: the percentage of GDP used to support the military is considered an indicator of militarism, when in fact a strong military may be a deterrent to other countries making war against them. The number of population jailed per 100,000 might actually be an indicator of decreasing crime rather than an indicator of violence. And why is the indicator “Potential for terrorist acts” given the LOWEST weight of any indicator in the list? Terrorism would seem to me to be the single greatest threat to world peace in the modern era.

The indicators are as follows:

Indicator---------------------------------Weight (1-5)
Level of distrust in others--------------------4
Number of internal security
officers per 100,000 people------------------3
Number of homicides per 100,000 people---------4
Number of jailed population per 100,000--------3
Ease of access to weapons of minor destruction--3
Level of organized conflict (internal)---------5
Likelihood of violent demonstrations-----------3
Level of violent crime-------------------------4
Political instability--------------------------4
Respect for human rights-----------------------4
Volume of transfers of major weapons
as recipient per 100,000-----------------2
Potential for terrorist acts-------------------1
Number of deaths from organized conflict-------5
Military expenditures as a percentage of GDP---2
Number of armed services personnel
per 100,000 people---------------------------2
UN Deployments 2006-07
(percentage of total forces)-----------------4
Aggregate number of heavy weapons
per 100,000 people---------------------------3
Volume of transfers of major weapons
as supplier per 100,000----------------------2
Military capability/sophistication-------------2
Number of displaced people as a
percentage of population---------------------4
Relations with neighboring countries-----------5
Number of external and internal
conflicts fought: 2000-05--------------------5
Estimated number of deaths from
organized conflict (external)----------------5

Furthermore, the report tries to combine indicators of external war, internal war and crime into a single index. However, this ignores the fact that the reasons for each are different, and thus cannot be dealt with as a single item. Bringing an end to external war in a specific country will not necessarily end crime or internal conflict within that country. Lowering crime will not necessarily bring an end to war, either internal or external. To try to deal with all of these issues under the single banner of “peace” is naïve and shows a lack of understanding of what causes crime, war and internal conflict.

Finally, this report does nothing to state whether a specific war is or is not necessary. Nobody would rightly argue that defeating Hitler in WWII was a “bad” thing or that the war should not have been fought. Peace was not as desirable as victory against Hitler in that case. There have been other ‘righteous’ wars in the past as well… the war to free Kuwait from invasion by Iraq is a good example. Stating that armed conflict is bad without putting the conflict into context is naïve and not very useful to establishing true world peace.

For these reasons, I find the Global Peace Index report to be lacking in substantive information that can be used to bring peace. While it is an interesting attempt to quantify “peace” I believe that it fails in its goals of defining and quantifying peace and determining the indicators of peace.

What's your take?

Elliot

tomder55 answered on 06/01/07:

With gun ownership being a criteria the Swiss should be at the bottom of the list.

ya gotta love it. The report ranks Syria more peaceful than the US . They are one of the instigators of all that has happened in Lebanon in the last 2 years.

I consider this report as useful as those human rights reports that constantly rank the USA as a rogue tyranny because of the use of tasers .

What is my take ? Here is a recent news report from that most peaceful of nations Norway

“You can have peace, or you can have freedom. Don’t ever expect both at once.” —Robert A. Heinlein

ETWolverine rated this answer Excellent or Above Average Answer
Itsdb rated this answer Excellent or Above Average Answer

Question/Answer
Itsdb asked on 05/30/07 - Norway rated most peaceful nation

U.S. ranked 96th...

    A study has ranked Norway as the most peaceful country and Iraq as the least in a survey of 121 countries.

    The Global Peace Index, compiled by the Economist Intelligence Unit, looked at 24 factors to determine how peaceful each country was.

    It places the US at 96th on the list and the UK at 49th, while New Zealand ranks second and Japan fifth.

    The authors say it is the first attempt to produce such a wide-ranging league table of how peaceful countries are.

    Factors examined by the authors include levels of violence and organised crime within the country and military expenditure.

    The survey has been backed by the Dalai Lama, Archbishop Desmond Tutu, former US President Jimmy Carter and US economist Joseph Stiglitz, who are all Nobel prize laureates.

    It is also supported by Queen Noor of Jordan.

    'Wake-up call'

    Scandinavian and other European countries generally performed well in the survey.

    But Britain's ranking comes partly from its involvement in Iraq and other conflicts.

    The United States is 96th - between Yemen and Iran - again because of such things as its military spending, its involvement in Iraq, violent crime at home, and a high prison population.

    The survey also places Russia and Israel at the wrong end of the scale - 118th and 119th respectively.


    The brainchild of Steve Killelea, an Australian entrepreneur, the survey is meant to inform governments, international organisations, and campaign groups.

    Mr Killelea said: "This is a wake-up call for leaders around the globe. Countries need to become more peaceful to solve the major challenges that the world faces - from climate change to decreasing biodiversity.

    "There is also a strong case for the world becoming more peaceful and it is now crucial for world leaders and business to take a lead," he said.

    He added that the high positions of Germany, which ranked 12th, and Japan revealed that "there can be light at the end of what may seem at the moment like a very dark tunnel."

    The study is published just before the G8 summit of leading countries next week.

    The authors say they are trying to supplant what they call some "woolly" definitions of peace with a scientific approach, that includes levels of violent crime, political instability, and a country's relations with its neighbours.

    But questions have been raised over the way some of these factors are brought together.

    The authors themselves acknowledge that there is a lack of data in many countries.

    What impact the new survey will have is unclear. The authors also argue that some countries - like Japan - may benefit from sheltering under the US military umbrella.


At least we beat Iran.

tomder55 answered on 05/30/07:

Norway depends on The Conventional Forces in Europe (CFE) treaty for their defense and has a small contingent with an annual budget of around $5 billion for their combined military. Norway’s Armed Forces Command is under pressure from the Ministry of Defense to produce a report detailing how the defense forces can save money over the next two to three years. Guess the nanny-state is falling short on funds.

CFE limits the number of key offensive weapons like tanks, artillery and aircraft across the continent, including the territory close to the Norwegian border with Russia.Putin has announced a moratorium on observance of the treaty and threatened to withdraw altogether .

If I were a Norwegian I would have no confidence that the combined forces of Europe would come to Norways aid if Russia got aggressive. Norway and Russia do have disputes over rights of the Barents Sea and as global warming creates new land exploitation opportunities territorial issues will become more pronounced. Already Canada (ranked 8th ) has to concern about defending it's northern coastal waters when the Arctic Sea becomes navigitable .

Norway had stepped up to the plate after 9-11 and was not the passive whimp that my posting might indicate under minister Kristin Krohn Devold .The budget increased during her tenure . But since 2005 Anne-Grete Strom-Erichsen ,a politician of the Labour Party became minister and with her came this new passivity .

Itsdb rated this answer Excellent or Above Average Answer

Question/Answer
ETWolverine asked on 05/30/07 - Doing the Booing Americans Won't Do

Michelle Malkin

The United States government is on the verge of approving a mass amnesty to millions of illegal aliens — a plan pushed aggressively by meddling Mexican officials who reap billions of dollars in remittances (illegal aliens' earnings sent back to Mexico) without having to lift a finger to clean up their own country.

And the thanks we get? Internationally televised public humiliation.

On Monday night, the beautiful young woman who represented America in the Miss Universe pageant was booed and mocked as she competed on stage in Mexico City. Rachel Smith, 22, did her best to respond with grace and dignity during the Top Five finalists' interview segment as the audience disrupted the event.

As soon as co-host Vanessa Minnillo invited Miss USA to pick a judge's name from a bowl of index cards, widespread howls broke out at the mere mention of "USA." The verbal derision continued as judge Tony Romo asked Smith to pick one moment in her life she would relive.

Definitely not this one.

Smith soldiered through her answer, describing an educational trip to South Africa. Catcalls and whistles nearly drowned out Smith's reply until she wrapped up with " Buenos noches, Mexico. "

I wouldn't have been so polite.

None of Miss USA's fellow Americans participating in the interview segment — neither Minnillo, nor macho co-host Mario Lopez, nor the dashing Romo — came to Smith's defense. Instead, Minnillo pleaded briefly with the unruly mob: "Okay, una momento, por favor. " Lopez stood mute with a dumb grin on his dimpled face. Pathetic.

In fact, Smith was subjected to anti-American hatred throughout the week-long event. Last week, during the contestants' national costume fashion show, Smith smiled bravely as a rowdy outdoor crowd hissed and booed at her. According to pageant observers, no other contestants received such treatment.

Pitifully, Donald Trump and his Miss Universe officials are downplaying Smith's experience — ignoring the fact that the last time the pageant was held in Mexico, Miss USA was abused in similar fashion. 1993 Miss USA Kenya Moore was infamously heckled when chosen for the semi-finals that year.

Just a tiny minority of America-haters, right? How quickly we forget.

Do you remember what happened in Guadalajara in 2004 during an Olympics qualification soccer match between the U.S. and Mexico? The stadium erupted in boos during the playing of "The Star-Spangled Banner." Fans yelled "Osama! Osama!" as the U.S. was eliminated by Mexico.

The following year, in March 2005, Mexican soccer fans again cheered the al Qaeda mastermind's name at a World Cup qualifier. ESPN reported the audience again booed and whistled during the U.S. national anthem, and plastic bags filled with urine were reportedly tossed on American players.

One Mexican fan told the Christian Science Monitor: "'Every schoolboy knows about 1848. . . . When they robbed our territory,' referring to when Texas, California and New Mexico were annexed to the U.S. as part of a peace treaty ending the war between the two countries, 'that was the beginning.'"

This bitterness is long-standing, deep-seated and stoked by top Mexican government officials and elites. But pointing this reality out in the context of our crucial national debate over sovereignty, immigration, assimilation, border security and the rule of law will get you labeled a bigot. Our leaders have concluded that it is better to pander, hide, pull out a friendly Spanish phrase like Minnillo did, and pray that the hatred will go away by giving the pro-amnesty lobby its legislative goodie-bag.

Meanwhile, as Manhattan Institute fellow Heather Mac Donald points out, the White House continues to attack opponents of the Bush-Kennedy amnesty package as "nativists." Conservative columnist Linda Chavez accused amnesty critics of "not liking Mexicans." Department of Homeland Security Secretary Michael Chertoff suggested enforcement advocates wanted to "execute" illegal aliens. And Republican Sen. Lindsay Graham trashed immigration enforcement proponents as "bigots" in front of the ethnocentric, open-borders group La Raza.

Yeah, we're the nativists.

Next, they'll tell us the mob at the Miss Universe pageant was simply "doing the booing Americans won't do."

Will President Bush speak out against the treatment Miss USA received in Mexico? Will any amnesty peddler in Washington? Imagine if Miss Mexico were booed, heckled and subjected to chants of "USA, USA" if the pageant had been held here.

Smith can hold her head up high. Those who are selling out our country, on the other hand, should hang their heads in shame.

Michelle Malkin is author of "Unhinged: Exposing Liberals Gone Wild." Her e-mail address is malkinblog@gmail.com.

COPYRIGHT 2007 CREATORS SYNDICATE, INC.


---------------

Great points by Michelle Malkin. I have no use for the Miss Universe pageant in any case. But why should anyone be forced to go through the publicly humiliating experience that Rachel Smith went through? And where is the thanks from the Mexican people for our open borders policy, the free money they get from us, and the de-facto citizenship they eventually attain though illegal means? Mexico doesn't reciprocate with its illegal aliens... it throws them in jail (if they are lucky, that's all that happens to them) and send them back to their countries of origin. Why are we pandering to a country that clearly hates us?

Elliot

tomder55 answered on 05/30/07:

One Mexican fan told the Christian Science Monitor: "'Every schoolboy knows about 1848. . . . When they robbed our territory,' referring to when Texas, California and New Mexico were annexed to the U.S. as part of a peace treaty ending the war between the two countries, 'that was the beginning.'"



The political attitudes of the Mexican immigrant population is forged in this belief that the Treaty of Guadalupe Hildago was a land grab by the US.

Actually the Mexican-American War is called la invasion estadounidense (the United States Invasion),or la intervención norteamericana (The North American intervention )south of the border. I was unaware until recently how wide spread is the belief that Southwest United States is Mexican property; but Mexican immigrant groups like La Raza and "Movimiento Estudiantil Chicano de Aztlan."(MEChA ) believe in the concept of reconquista of the vaguely-defined Aztec homeland .

Although it is a justifiable argument that American expansionism (especially of the slave institution )played a major role in the war ;it is a fact that the US was trying to negotiate the purchase of places like California and the New Mexico territories before open conflict began .It was the Mexicans that initiated hostilities with the attack on an American patrol ;called the Thornton affair ,and the unsuccessful attack on Fort Brown . It is also true that there was existing disputes on the border of Texas ,and even the independence of Texas was in dispute South of the border .

The territory aquired by the treaty was a bargain for the price paid. We made up for it slightly with the Gadsen Purchase .

I of course agree that the behavior of the mob at the Miss Universe Contest is unacceptable . But so is the booing of the American National Anthem at hockey games in Canada. The difference appears to be that in Canada this is the reaction of juiced up hooligans whereas in Mexico it appears to be the unofficial official policy of the state to stoke these anti-gringo feelings.

ETWolverine rated this answer Excellent or Above Average Answer

Question/Answer
Itsdb asked on 05/25/07 - Bush Hails Carter as Best Former President

by Scott Ott

(2007-05-20) — Just a day after former President Jimmy Carter told reporters that the Bush administration is “the worst in history” the current White House resident called Mr. Carter “the best former president ever.”

“I know that President Carter and I have had our differences,” said Mr. Bush, “But I think most Americans will agree with me that he’s a terrific ex-president. Things have never been better since Jimmy Carter left office.”

Mr. Bush pointed to an array of improvements, including a stable growing economy, lower taxes, reduced inflation and unemployment and increased American strength and preparedness — all of which he associated with Mr. Carter’s years as a former president.

“I think history will judge President Carter’s post-White House tenure favorably,” said Mr. Bush, “As a former president, Mr. Carter has overseen the nation’s longest period of expansion and growth in opportunity.”

tomder55 answered on 05/25/07:

Very kind of of the President . Carter is of course directly reponsible for much of the world situation that the President has to deal with ;therefore under that cirumstances such flattery is uncommon but I suppose commendible .

The best line I heard so far was Carter saying what he said about President Bush was like John Wayne Gacy calling a shoplifter a danger to society.

Itsdb rated this answer Excellent or Above Average Answer

Question/Answer
Itsdb asked on 05/24/07 - "They Are a Threat to Your Children, David"

If you can't tell I'm bored and not very busy today so it's blog day at NRO. Check out this clip from Bush's press conference today.

I'm confident David Gregory didn't get it.

tomder55 answered on 05/25/07:

No Gregory did not get it . Time to dust off that photo you have of someone's self examination of their hemmoroids .

Honestly ,I don't think the President should give any more press conferences unless he has a major announcement to make. He should then limit it to the question at hand.

Everyone knows his position on Iraq. He will not change anyone's mind ,and it's too late to think he can influence them from the bully -pulpit. Only events on the ground can make any signignificant change . I could give his response to any question about Iraq almost verbadum before he says it. listen to the generals and not the politicians.....pause.....If they don't fight us there, they'll follow us here.....etc .He's not wrong...but he is not adding anything to the discussion so why bother ? Better to let Tony Snow regurgitate the talking points.

It was painful and alternately depressing and outrageous the way he handled questions about the immigration debate and Alberto Gonzalez.

He has made his decision to support Gonzalez I think mostly for political reasons .

To replace him ,a new AG would be subject to the rediculous humiliation that a Senate hearing by the majority party would bring upon the candidate. The candidate would probably have to agree to appoint an independent council to investigate wide ranging issues that the Dems. want to nail the President with . That would destroy the administration by tying it up in endless defense .

But the President has to realize that Gonzalez' handling of the Justice Dept. in general, and this firing of the attorneys in particular ,has not served the President well .Gonzalez to me is incompetent .The President should keep a low profile on this issue . The investigations have already been exhausted . The President by speaking only adds fuel to Shmucky's fire .

On immigration ;I completely disagree with his whole approach. He misrepresents the opposition and paints us as a mob who would hunt down 12 million illegals and put them on boxcars . He is sticking a poker in the eyes of many of his few remaining supporters . Does he not see that ?



Itsdb rated this answer Excellent or Above Average Answer

Question/Answer
Itsdb asked on 05/24/07 - The Science Distraction

Interesting take on global warming thanks to Iain Murray on Planet Gore at NRO...

Spiked's Josie Appleton really hits the nail on the head in her excellent review of a new book by noted alarmist Mark Lynas. For instance, in relation to all this talk about a "generational challenge":

    Global warming offers us the chance to experience what few generations have had the privilege of knowing. It is a thrill, no less. Global warming is our Cold War. And just as American strategists worried at the end of the Cold War about the loss of the Red opposition, so environmentalists have a kind of attachment to global warming.

    Of course, they talk about it being ‘inconvenient’, and they wouldn’t have wished it upon the world. Lynas says that his collapse scenarios are a ‘reluctant conclusion’; in his book Heat, George Monbiot says that it pains him greatly to conclude that people will have to stop flying. But the more that society defines itself in relation to global warming, the less willing it is to let go. Global warming is now not so much a problem to solve, as an issue around which to reorganise society. This is more Noah’s flood than Clean Air Act, and the lesson is in the sins of hubris and consumerism. Global warming is sent to show people that (in Lynas’ words) they are ‘wasting their lives commuting to work in cars’. His proposed solution – to ‘cut our need for energy by living less consumptive lifestyles’ – will apparently form the basis of a new and happier society.

    I have a question: if there was a ‘miracle energy cure’, would Lynas use it? I suspect that a straight ‘yes’ would not be the reply. Which is insane, really, because if global warming is a problem, it is only a techno-fix that could solve it. All the arm-twisting in the world is not going to stop India and China flying, a fact shown by recent figures showing a massive boom in air travel. Daily media guilt-mongering has not stopped British people from enjoying weekends in Budapest or Prague, and nor should it. Governments, we can hope, will still be elected in 2050, and while that is the case carbon rations would still be ‘politically unrealistic’. Unless we live under a dictatorship of some Global Commission for the Environment then energy use will continue to rise dramatically; the only question is whether this energy comes from fossil fuels or some other source. And if it needs to come from some other source, we need a techno-fix.

    Techno-fixes are not some airy-fairy notion, some leap of faith. This is otherwise known as innovation, the only way that environmental problems have ever been solved or new energy systems produced. I am not aware of a major environmental problem successfully tackled by the mass of people consciously and systematically abstaining from some or other desirable activity. The lesson of history is that techno-fixes happen, and they happen fast in societies that are looking for solutions. There were only three years between the first controlled nuclear chain reaction and the dropping of a bomb on Hiroshima in 1945; only five years separated a nuclear reaction successfully lighting a lightbulb in 1951 and the first nuclear power station in 1956.


Quite right. In her conclusion, Josie says something that we have been saying for years - that this isn't a scientific issue, but a political one informed by science and economics:

    We need a new school of thought in the global warming debate, which is founded not on scientific facts but on political critique. It is only this that can explain the way in which the issue is framed, or its hold over social life and public debate. Lynas’ books suggest the attraction of the global warming issue has little to do with environmental problems. Instead, global warming appears to provide answers to life’s big questions, offering a new kind of historic mission and a new structure for personal morality.

    Only global warming doesn’t really answer any of these big questions - it shuts them down, solving the problem of meaning by abolishing meaning itself. As we look forward to 2050, we could hope to find some more profound answers to the riddle of existence than that measured in the rise and fall of carbon atoms. We could also hope to find some more sensible (but, possibly, less dramatic) solutions to any environmental challenges we face.

    We need to strip drama from climatology, and add drama to our lives. The question of how we live should be subject to mass, passionate debate, and Geophysical Research Letters should be left in the basement of the Radcliffe Science Library for the consultation of specialists.


Quite right, too. This is the way to the solution demanded by both democracy and justice. Yet the follower of scientism and the green zealot alike finds it all too incovenient.
~~~~~~~~~~~~~~~~~~~~~~~~~~~~~~~~~~~~~~~~~~~~~~~~~~~~~~

Global warming is this generation's 'cold war' and that makes sense. Some people simply must have drama, scandal and conflict, some boogeyman at which to focus their energy and in order to find meaning and purpose in life it seems, and global warming is a perfect fit.

It offers all the drama they can stand, all the redemptive possibilities they can explore, an endless supply of self-congratulatory reinforcement and the chance to "reorganise society" in their mold. Who can resist that, and why should facts, reason, or people get in the way?

Steve

tomder55 answered on 05/25/07:


Cold war ??? LOL ....How fitting !!!

This is the time of the year when the Southern Hemisphere battles "global warming ".Wonder how they are fairing these days ?

Weather breaks record in South Africa.

Australian Ski Resort to open 15 days early


Elsewhere Western Canada is feeling the heat as summer rapidly approaches.

But maybe if you go to the Caribbean you can escape the cold. NOT.


Global warming is now not so much a problem to solve, as an issue around which to reorganise society


Bingo .

I am not aware of a major environmental problem successfully tackled by the mass of people consciously and systematically abstaining from some or other desirable activity.

But they could care less about the technological solutions . They want human reduction. Ask any of them and they will tell you that human populaton growth is what is really fueling the problem. Ideally we should be gatherers of berries living in caves while they live in their McMansions and fly in private Gulfstreams .(Let's pay mea culpa carbon credits to ourselves hehehe) .

Itsdb rated this answer Excellent or Above Average Answer

Question/Answer
Itsdb asked on 05/24/07 - Anyone hear about this?

I sure didn't...

British transfer authority over the Maysan Province to the Iraqi government:


Handshakes: Maysan is now yours . . . British Major General Jonathan Shaw and provincial governor Adel Maliki make it official.

But how was it covered? Or was it covered?

"A Small Battle in the Media War" [Stephen Spruiell]

    Michael Yon writes about the media coverage of the British transfer of authority over the Maysan Province to the Iraqi government:

    The transfer of authority did not even make the cut for news for most US publications and networks. Of those which included the story in their news reports, most mentioned it only as part of an overall report about the day’s activities in Iraq. Many of those included it in reports which were headlined or sandwiched with bad news about the violence in other parts of Iraq.

    The Washington Post’s “Bombers Defy Security Push, Killing at least 158 in Baghdad” briefly mentions the transfer in a sentence in the seventeenth paragraph. Likewise, The New York Times’ “Bombings Kill at Least 171 Iraqis in Baghdad” mentions the transfer of the province somewhere in the sixth paragraph.

    This general theme carried over in the UK media coverage as well. The Guardian offered “‘We’ll be in control by end of 2007’ say Maliki. [sic] In Baghdad, carnage continues.” The Independent headline blared “Hundreds killed on Baghdad’s day of bombs and blood.” Not to be outdone in the dramatic headline competition, The Mirror gave the world “BLOODIEST DAY: 191 dead and hundreds maimed as 5 bombs rock Baghdad.”

    The BBC article titled “Iraq troops to take over security” reported on statements made by Iraqi Prime Minister Nouri Maliki in the speech prepared for the Maysan ceremony, (delivered by Iraq’s National Security Minister) about the schedule for turning over additional provinces to the control of Iraqi security forces. But before describing the ceremony and without ever providing any details about the province or the Iraqi security forces who now control it, the article inventoried recent attacks and included a mention of the withdrawal of Sadr loyalists from the Iraqi parliament.

    On the day after the ceremony, the BBC mentioned it in the sixth paragraph of an article titled “Two UK soldiers killed in Iraq.” The next day it was mentioned again in a BBC article titled “UK soldiers killed in Iraq named,” although this time it was relegated to a mention in the nineteenth paragraph. No details about the ceremony were given in either article, both of which also referenced recent US and UK military casualties, civilian casualties and sectarian violence.

    Along with Alex Zavis’ “British Hand Over Province to Iraqi Control” in The Los Angeles Times, two other reporters wrote stories headlined with the transfer ceremony. The Telegraph’s Thomas Harding filed his “200 killed as province returns to Iraqi control” from Camp Sparrowhawk, although he didn’t get around to anything about the transfer ceremony until the 20th paragraph. James Hider’s piece in The Times, “British put the ‘Wild West’ back under control of Iraqis,” was the only other news story about the transfer that was actually about the transfer.

    While the journalists and all the rest headed back to their worlds in big helicopters, I flew deeper into Maysan desert with the Queen’s Royal Lancers where, in less than 24 hours, our patrol would be attacked with the largest array of bombs I ever encountered in Iraq, and where two UK soldiers would die.


But then why would the drive-by media want to report on a significant milestone in Iraq?

tomder55 answered on 05/24/07:

Sometimes the news is do good that they have no choice but to report it . That is the situation this week that Joe Klein at Time Mag . became confronted with .

There is good news from Iraq, believe it or not. It comes from the most unlikely place: Anbar province, home of the Sunni insurgency. ... This is a result of sheiks stepping up and opposing AQI [al-Qaeda in Iraq] and volunteering their young men to serve in the police and army units there." The success in Anbar has led sheiks in at least two other Sunni-dominated provinces, Nineveh and Salahaddin, to ask for similar alliances against the foreign fighters. And, as TIME's Bobby Ghosh has reported, an influential leader of the Sunni insurgency, Harith al-Dari, has turned against al-Qaeda as well. It is possible that al-Qaeda is being rejected like a mismatched liver transplant by the body of the Iraqi insurgency.

Military.com also reported that the US became allied with Sheikh al Rashawi . This appears to be a generational shift of power in the tribes that bodes well for our effort if we play our cards right . We have a new power shift in France and Germany ...why not in Sunni Iraq ? More on this here.

Omar of Iraq the Model blog reports that There hasn’t been any major security incidents in Baghdad since the attacks on three bridges in both its northern and southern suburbs on Friday May 11, more than a week ago. This doesn’t mean Baghdad is essentially calm: there are episodes -albeit minor and limited- still happening from time to time.

I assure him ;the legacy media reports every blast as an attack happening in Baghdad . Perhaps some of the gun fire he hears is coming from this???? Clearly the surge was a failure before it began.




Itsdb rated this answer Excellent or Above Average Answer

Question/Answer
Itsdb asked on 05/23/07 - So there is no war on terror?

John Edwards is sure to get a boost from the moonbattery after giving his speech to the Council on Foreign Relations today.

"We need a post-Bush, post-9/11, post-Iraq military that is mission focused on protecting Americans from 21st century threats, not misused for discredited ideological purposes...by framing this as a war, we have walked right into the trap the terrorists have set—that we are engaged in some kind of clash of civilizations and a war on Islam."

I'm certain he must have just missed something here.

tomder55 answered on 05/24/07:

His terror war is against a bad coiffer and split ends .

His attitude disqualifies him to be Commander in Chief.

Itsdb rated this answer Excellent or Above Average Answer

Question/Answer
Itsdb asked on 05/22/07 - Muslim survey

And I'm not believing what I heard...ABC Radio news reported that most Muslims in America believe suicide bombings can never be justified, but "thirteen percent do in some cases." They then played a clip of some Muslim dude telling us Americans can now rest easy in knowing American Muslims are Americans first and Muslims second. Well now, that's comforting.

First, thirteen percent of Muslims believing suicide bombings can be justified to me is a disturbing number. And considering there are roughly 2.35 million Muslims in this country, that would mean over 300,000 don't have a problem with suicide bombings. On top of that, twenty-six percent of young Muslims can justify suicide bombings.

Moreover:

    63% identified themselves as Democrats or as "leaning" toward the Democratic Party, although "On key social issues," Pew says, "Muslims in the U.S. are much more conservative than the general public. Most say that homosexuality is a way of life that should be discouraged, rather than accepted, by society. A large majority of Muslims (59%) also say that government should do more to protect
    morality in society."

    Only 25 percent consider the U.S. war on terrorism a sincere attempt to curtail international terror. Only 40 percent said they believe Arab men carried out the attacks of Sept. 11, 2001.

    By six to one, they say the U.S. was wrong to invade Iraq, while a third say the same about Afghanistan -- far deeper than the opposition expressed by the general U.S. public.


And ABC tells us don't worry, be happy. Pew themselves headline the report, "Muslim Americans: Middle Class and Mostly Mainstream."

    The first-ever, nationwide, random sample survey of Muslim Americans finds them to be largely assimilated, happy with their lives, and moderate with respect to many of the issues that have divided Muslims and Westerners around the world.


So American Muslims are more conservative on social issues than most Americans, but 63% identify with the party of abortion and gay rights - 71% having voted for John Kerry. 300,000 can find a way to justify suicide bombings and only forty percent believe Arabs had anything to do with 9/11 ... yet Muslims in America are mostly moderate and mainstream.

I don't know about you but I find it all quite disturbing on many levels.

tomder55 answered on 05/23/07:

If 1 % of the world's Muslim community is a jihadists that is a 10 million pool to draw homicide bombers from.10% is 100 million. I'd have to say that the jihadi are more active in their recruitment efforts so the poll numbers if answered honestly would most likely increase to numbers simular to polls taken in Indonesia ,Londonistan and other enclaves around the world. Percentages are nice and academic. Raw numbers are disturbing.

Itsdb rated this answer Excellent or Above Average Answer

Question/Answer
ETWolverine asked on 05/22/07 - My e-mail to President Bush

Mr. President,

I realize that you most likely will never see this message. However, I felt it was my duty to log my concerns with your immigration policies.

First, I believe that I should tell you a bit about myself so that you understand my perspective on immigration. I am a 38-year-old banker working for a mid-sized bank in New York. I am the son and grandson of immigrants from Poland, survivors of the Holocaust. My parents were born in German DP camps after the war and are naturalized citizens. (My mother still keeps her green card as a keepsake due to her pride at having come to this country.)

My grandparents worked very hard to become successful in this country. My father’s parents were candy-shop owners (after years of running pushcarts to scrape together the funds needed to buy the shop) and my mother’s parents were tailors. They came to this country with literally nothing in their pockets, and managed to become productive members of society. They managed to put their children through college, and all of them became professionals. My father is an attorney and a stock broker for a major Wall Street firm (coincidentally, he was once the youngest attorney to be admitted to argue before the Supreme Court, a record that has since been broken but that my father is still proud of to this day), and my mother manages real estate. My family made good on the American dream within a generation, and I am very thankful for the opportunities that the United States afforded my family. Thanks to the opportunities that this country affords us, I became the first member of my family born in this country, and with the support of my family I graduated Brooklyn College with a BA in economics. I married a girl from your own home state of Texas (Houston, to be precise) and we have two wonderful children, a home in New Jersey, and a solid middle-class income. My sister is a teacher living in Brooklyn, and my brother is an MD, a graduate of SUNY Downstate Medical School. I truly believe that my family is a text-book success story for immigration within the USA.

As you can see, Mr. President, I am hardly an opponent of immigration. I value immigration as an important source for “new blood” and new ideas into the pool of American resources. I try to keep in mind that some of the greatest acheivements of our great country have come from immigrants: Albert Einstein and Nikola Tesla, perhaps the two greatests scientists of their times, were both immigrants. Eli Weisel is an immigrant. And there have been many others as well… too many to name in such a letter. Immigration is important to our national identity and an important tool for remaining at the top of the economic and technological food chain.

However, your recently announced immigration bill concerns me greatly.

Mr. President, part of the reason that my family was so successful in this country was the fact that we became part of American culture. Another important factor is that we didn’t demand support from the government for what was really our own responsibility to provide to ourselves. And finally, and perhaps most important, is the fact that we caqme here legally after waiting on the appropriate lists and doing the appropriate paperwork. We did not sneak into the United States, we came here legally.

Mr. President, these are important factors to consider, and none of them are addressed in your bill.

While I do not necessarily support making English our national language (though it would be nice to see), I also don’t believe that accomodating other languages in government business supports the idea of joining American culture. Language is one of the most important factors in any culture, and not giving immigrants a reason to learn English by forcing them to conduct government business in English is a barrier to cultural acclimation. It sets immigrant apart from the rest of society, which in turn hurts American society as a whole. (Not to mention the economic costs of providing government assistance in multiple languages.) Your bill does not address this point at all.

Secondly, your bill has not appropriately addressed the costs of illegal immigration. The vast majority of illegal immigrants are low-skilled labor. The Heritage Foundation did a study in which they found that the average cost of a low-skilled labor family to the US Government is roughly $22,000 after taxes. Mr. President, with 11 million illegal immigrants in this country, the government is spending roughly $242 Billion per year on supporting low-skilled laborers from other countries. That equates to $2.4 trillion over a 10 year period. When we consider that your proposed national budget for 2008 is only $2.3 trillion, the cost of illegal immigration is one of great concern. This is potentially the biggest disaster to our economy since the Great Depression. We can barely afford to support our own low-skilled workers and their families. We do not need to be importing poverty from other countries to support. Instead of low-skilled laborer’s, we should be encouraging the immigration of high-skill employees and innovators… the likes of Eli Weisel, Nikola Tesla and Albert Einstein… people who will push this country forward economically, socially and culturally, not those who will hold us back.

Thirdly, your bill grants amnesty to an entire sub-class of law-breakers. I know that you object to the use of the word “amnesty” in reference to this bill, but that is exactly what this bill is: an amnesty bill. Mr. President, the American Heritage Dictionary defines amnesty as "A general pardon granted by a government, especially for political offenses." It is difficult for me to see how this bill is anything but amnesty. It grants a general pardon to a specific group of people for the illegal political offense of entering this country illegally. This is potentially a very big mistake, Mr. President. It makes US citizenship cheap. My grandparents worked very hard to become citizens, and they hold that citizenship dear because it was something worth working for. But your bill makes it cheap. It makes coming to this country illegally a method of gaining relatively quick citizenship.

Mr. President, one of your predecessors, President Carter, made the mistake of legalizing illegal immigration. The result was the Mariel Boatlift, a huge rise in crime, particularly drug crime and violent crime, and influx of convicted murderers and rapists to the United States, and a financial and economic burden that we are still paying to this day. I beg you to reconsider your current course of action. Legalizing 11 million illegal aliens in one felt swoop is an economic, social and criminal burden that this country cannot bear without serious and irreversible consequences.

I have supported you with regard to the War on Terror and the War in Iraq (which I consider to be one and the same), the USA Patriot Act, your tax relief plans, and your handling of judicial appointments, as well as on most other issues. But on this issue, Mr. President, I must log my strong disagreement.

I know that you have the good of this country at heart, and I know that your compassion for your fellow man is what drives you to support this bill. I respect your motivation. But this is not good for America or the American people. I urge you to reconsider this immigration bill.

With greatest respect from your constituent and supporter,

Elliot (my full name)

Edison, New Jersey

God Bless the President of the United States of America

------------

What do you think? Did I catch the high points?

tomder55 answered on 05/22/07:

Love it ! I could modify it slightly and it could be my family history.

I think the President has his compassionate side motivating him as well as some political warning he is hearing from people like Karl Rove and the "Fred Barnes wing" and the "what's good for business is good for the USA "Republicans like Lawrence Kudlow (who trashed the Heritage Foundation figures on his weekend radio show)and the Wall Street Journal .

Both Democrats and Republicans are convinced that they need to bring in a new population that will irreversably alter the face of the country so they can attract the future majority to their camps.They also falsely believe that only with a tremendous influx of new blood the patient called Social Security will die.

I am suspicious of the grand bargain because :

1. I do not believe the Democrats negotiate in good faith and that many of the provisons we were told about over the weekend are already diluted down as some have read the fine print and legaleze of the bill. It raises hackles the way it was secretely negotiated and the attempt to ram it through without debate .

2. We have seen before in 1986 that once such a grand bargain is in place it is easy to ignore key provisions of the law enforcement aspect . The governemnt has not made an honest attempt to enforce existing rules so why should we believe that the much more complex provisions of this bill will be enforced ?

3. They blatantly lied to us last year . The provisions for the wall was passed and signed knowing there would be no serious attempt to build it .

All 3 points combined now makes this a matter of trust .I do not believe the collective US government wants to solve the immigration problem just like I became convinced 2 years ago that they do not want to seriously address Social Security .

4. Proponents say that it is unrealistic to deport the 12 million illegals arleady here . By that same logic it is equally impossible to get them to voluntarily go back home, pay a fine, and then would be expected to re-apply in a “touch-back” manoeuvre. Few who have children born in the United States (who are anyway, by that fact, Americans) would be inclined to do that.

5. What is a $5000 fine to people who will be collecting the Earned Income Tax credit conceivably for another decade ?

6. The bill would create a permanent sub-class in the country that would be subject to virtual servitude . We have seen how well that works out in Europe. The bill would legalize depressed wages and foster exploitation of low-skilled workers. Even the AFL-CIO who are anxious to get new membership realize this and oppose the bill for that reason.

I'm sure there are more objection as I examine it further and the onion gets peeled back.I think the President is acting in what he thinks is good faith but if we had not objected to some of his good faith moves we would have Harriet Meyers and Alberto Gonzalez in SCOTUS instead of Roberts and Alito. WE need to keep up the pressure. Writing him is a good start . I think I will also . The last time I did so was to President Reagan to basically support a policy he held. I got a nice form letter back.

ETWolverine rated this answer Excellent or Above Average Answer
Itsdb rated this answer Excellent or Above Average Answer

Question/Answer
kindj asked on 05/21/07 - This is kinda interesting.....

What do you think? Could this be the launch pad for a viable third party, or at least for reformation of the existing two parties?

http://www.unity08.com/

tomder55 answered on 05/21/07:

I'm pretty sure that this group is actively stroking Michael Bloomberg to run on their ticket. He has pledged to spend $100 million of his own money in the effort if he decides to do it.

If Bloomy goes for it then he will have Sen. Chuck Hagel tag along as his VP selection. Michael Bloomberg is nominally a Republican ,and the party claims it want's a bipartisan ticket so I do not know how that would work out ;except that Bloomy has jumped ship before for political expediency.

At this point it is unclear to me which party would be damaged more by a 3rd party run .What is clear to me is that just like Perot's and Nadar's bids ,a 3rd party challenge could only be a spoiler . They haven't really offered a platform beyond an attempt to garner the votes of people disaffected with the current crop of politicians (not that I blame them)....but the names I have seen considered are the same old same old .(McCain paired up with Leiberman .... Gore /Wesley Clark ..etc )

My problem with the whole concept of a unity ticket is that there are real differences in the world view of the 2 major parties and I do not see how this will bridge the differences. McCain-Feingold comes closest to mind when I think of what bipartisanship brings in today's environment (or the more recent manifestation of it ..this new McCain and Ted Kennedy sponsored immigration legislation).

The American Thinker has an interesting article on the perils of a unity ticket .It uses the Lincoln-Johnson ticket as it's example.

kindj rated this answer Excellent or Above Average Answer

Question/Answer
HANK1 asked on 05/19/07 - Self-defense:



Is it time for all Americans to arm themselves?

HANK

tomder55 answered on 05/20/07:

Americans should generally be aware of self defense . The 2nd Amendment is a choice. You can arm yourself if you chose to do so . But it says nothing about ALL Americans doing so.

HANK1 rated this answer Excellent or Above Average Answer

Question/Answer
HANK1 asked on 05/19/07 - Vigilantes:



Could vigilanty societies help law enforcement clean up drug traffic?

HANK

tomder55 answered on 05/20/07:

How do you define vigilatism ? Neighborhood watch groups are beneficial . The Guardian Angels have done some good work .Both work in cooperation with the law enforcement agencies and are NOT ARMED .

I do not think armed vigilaties would be beneficial . We do not need militas and paramilitaries creating a law unto themselves. See my post above about Islamberg. I'm sure by their community's standards their own concepts of law and order is vigorously enforced .

HANK1 rated this answer Excellent or Above Average Answer

Question/Answer
HANK1 asked on 05/19/07 - A Frightening Scenario:



As we have been told, there are 100 million Hispanics in the United States. Then there are a great number of men/women in our prisons, a goodly population of Muslims frequenting all walks of life and only Heaven knows the number of those who make up other minority groups in America. Many countries hate our guts. A large number of our soldiers are fighting in Iraq and Afghanistan and a large number of our soldiers are stationed in foreign countries. Folks, I ask you: How would the United States ward off a revolution in America IF things heated up and those factions came together as an army of one?

HANK

tomder55 answered on 05/20/07:

Why do you think the founders included the 2nd Amendment ?
Back then there was no domestic tranquility . There were threats frrom indiginous populations and from various European nations that had outposts on our borders.

Hank ;don't buy into the Democrats talking points about our military being stretched . It is not true . Yes I would expand the military back to close to Cold War levels ,but that's to counter the reckless cutbacks of the 1990s.

But ,to your larger point ;I think I'll feed into your fears a little .In the United States al Qaeda, Hezbollah, Hamas, Islamic Jihad and the Muslim Brotherhood – to name but a few terrorist organizations – have set up regional headquarters in Boston, Chicago, New York, Dallas, Los Angeles, Minneapolis, Tampa, Washington DC and over 38 other cities around the country. They are not only raising, laundering and funneling money back to the Middle East to support their terrorist organizations, they are setting up jihadi training camps right here in the United States.

This week Paul Williams of the Candian Free Press wrote an expose about one small enclave situated in the previously quite Catskill Mts. in N.Y. It is called Islamberg (I kid you not).

Situated within a dense forest at the foothills of the Catskill Mountains on the outskirts of Hancock, New York, Islamberg is not an ideal place for a summer vacation unless, of course, you are an exponent of the Jihad or a fan of Osama bin Laden.

The 70 acre complex is surrounded with "No trespassing" signs; the rocky terrain is infested with rattlesnakes; and the woods are home to black bears, coyotes, wolves, and a few bobcats.

The entrance to the community is at the bottom of a very steep hill that is difficult to navigate even on a bright sunny day in May. The road, dubbed Muslim Lane, is unpaved and marred by deep crevices that have been created by torrential downpours. On a wintry day, few, save those with all terrain vehicles, could venture forth from the remote encampment.

A sentry post has been established at the base of the hill.

The sentry, at the time of this visit, is an African American dressed in Islamic garb - - a skull cap, a prayer shawl, and a loose fitting shalwat kameez. He instructs us to turn around and leave. "Our community is not open to visitors," he says.

Behind the sentry and across a small stream stand dozens of inhabitants of the compound - - the men wearing skull caps and loose fitting tunics, the women in full burqa. They appear ready to deal with any unauthorized intruders.

The hillside is blighted by rusty trailers that appear to be without power or running water and a number of outhouses. The scent of raw sewage is in the air.

The place is even off limits to the local undertaker who says that he has delivered bodies to the complex but has never been granted entrance. "They come and take the bodies from my hearse. They won't allow me to get past the sentry post. They say that they want to prepare the bodies for burial. But I never get the bodies back. I don't know what's going on there but I don't think it's legal."

On the other side of the hill where few dare to go is a tiny village replete with a make-shift learning center (dubbed the "International Quranic Open University"); a trailer converted into a Laundromat; a small, green community center; a small and rather squalid grocery store; a newly constructed majid; over forty clapboard homes; and scores of additional trailers.

It is home to hundreds - - all in Islamic attire, and all African-Americans. Most drive late model SUVs with license plates from Pennsylvania, New Jersey, New York, Ohio, South Carolina, and Tennessee. The locals say that some work as tollbooth operators for the New York State Thruway, while others are employed at a credit card processing center that maintains confidential financial records.

While buzzing with activity during the week, the place becomes a virtual hive on weekends. The guest includes arrivals from the inner cities of New York, New Jersey, and Pennsylvania and, occasionally, white-robed dignitaries in Ray-Bans from the Middle East.

Venturing into the complex last summer, Douglas Hagmann, an intrepid investigator and director of the Northeast Intelligence Service, came upon a military training area at the eastern perimeter of the property. The area was equipped with ropes hanging from tall trees, wooden fences for scaling, a make-shift obstacle course, and a firing range. Hagmann said that the range appeared to have been in regular use.

Islamberg is not as benign as a Buddhist monastery or a Carmelite convent. Nearly every weekend, neighbors hear sounds of gunfire. Some, including a combat veteran of the Vietnam War, have heard the bang of small explosives. None of the neighbors wished to be identified for fear of "retaliation." "We don't even dare to slow down when we drive by," one resident said. "They own the mountain and they know it and there is nothing we can do about it but move, and we can't even do that. Who wants to buy a property near that?"

The complex serves to scare the bejeesus out of the local residents. "If you go there, you better wear body armor," a customer at the Circle E Diner in Hancock said. "They have armed guards and if they shoot you, nobody will find your body."


At Cousins, a watering hole in nearby Deposit, a barfly, who didn't wish to be identified, said: "The place is dangerous. You can hear gunfire up there. I can't understand why the FBI won't shut it down."

Islamberg is a branch of Muslims of the Americas Inc., a tax-exempt organization formed in 1980 by Pakistani cleric Sheikh Mubarak Ali Gilani, who refers to himself as "the sixth Sultan Ul Faqr," Gilani, has been directly linked by court documents to Jamaat ul-Fuqra or "community of the impoverished," an organization that seeks to "purify" Islam through violence.

Though primarily based in Lahore, Pakistan, Jamaat ul-Fuqra has operational headquarters in New York and openly recruits through various social service organizations in the U.S., including the prison system. Members live in hamaats or compounds, such as Islamberg, where they agree to abide by the laws of Jamaat ul-Fuqra, which are considered to be above local, state and federal authority. Additional hamaats have been established in Hyattsville, Maryland; Red House, Virginia; Falls Church, Virginia; Macon, Georgia; York, South Carolina; Dover, Tennessee; Buena Vista, Colorado; Talihina, Oklahoma; Tulare Country, California; Commerce, California; and Onalaska, Washington. Others are being built, including an expansive facility in Sherman, Pennsylvania.

Before becoming a citizen of Islamberg or any of the other Fuqra compounds, the recruits - - primarily inner city black men who became converts in prison - - are compelled to sign an oath that reads: "I shall always hear and obey, and whenever given the command, I shall readily fight for Allah's sake."

In the past, thousands of members of the U.S. branches of Jamaat ul-Fuqra traveled to Pakistan for paramilitary training, but encampments, such as Islamberg, are now capable of providing book-camp training so raw recruits are no longer required to travel abroad amidst the increased scrutiny of post 9/11.

Over the years, numerous members of Jamaat ul-Fuqra have been convicted in US courts of such crimes as conspiracy to commit murder, firebombing, gun smuggling, and workers' compensation fraud. Others remain leading suspects in criminal cases throughout the country, including ten unsolved assassinations and seventeen fire-bombings between 1979 and 1990.

The criminal charges against the group and the criminal convictions are not things of the past. In 2001, a resident of a California compound was charged with first-degree murder in the shooting of a sheriff's deputy; another was charged with gun-smuggling' and twenty-four members of the Red House community were convicted of firearms violations.

By 2004 federal investigators uncovered evidence that linked both the DC "sniper killer" John Allen Muhammed and "Shoe Bomber" Richard Reid to the group and reports surfaced that Wall Street Journal reporter Daniel Pearl was captured and beheaded in the process of attempting to obtain an interview with Sheikh Gilani in Pakistan.

Even though Jamaat ul-Fuqra has been involved in terror attacks and sundry criminal activities, recruited thousands of members from federal and state penal systems, and appears to be operating paramilitary facilities for militant Muslims, it remains to be placed on the official US Terror Watch List. On the contrary, it continues to operate, flourish, and expand as a legitimate nonprofit, tax-deductible charity.


For perspective: the area is very rural, without too many rules or oversight. Yet it’s just a few hours drive from NYC, with fairly cheap land. As pointed out, there are many more little jihadist communites nestled in simular locations around the country where Islamist paramiltaries practice and train .We chastise other countries, including some who we consider allies, for allowing terrorist training camps to exist within their borders. This is the pot calling the kettle black.

The obvious question is : If most of the residents have criminal records then the use and ownership of firearms by felons is prohibited; Why haven't the authorities gone in and busted the community ?
They had no problem with the idea when they raided the Branch Davidians under far less pretext .


HANK1 rated this answer Excellent or Above Average Answer

Question/Answer
Itsdb asked on 05/18/07 - Life, liberty and the pursuit of happiness for apes?

I posted this on that other board.

This opinion column appeared in my paper...

By JONATHAN BALCOMBE
Physicians Committee for Responsible Medicine

    It's a civil rights case with a twist. In late April, an Austrian judge denied personhood status (Registration required) and legal guardianship for 26-year-old Matthias Pan, who was kidnapped as an infant in Sierra Leone after his mother was shot. Brought to Austria illegally, Pan was sold to a research laboratory where he lived alone in a cage and was experimented on for many years before finally being released to a sanctuary.

    In her concluding statement, the judge explained that she never doubted that Pan should be considered a person, but she did not want to set a precedent that might weaken the case of humans with legal guardians. Pan's legal team will appeal the decision.

    Matthias Pan is, of course, not human. He is a chimpanzee.

    Although many of us might share the judge's view that chimps should qualify for personhood, current legal systems in the United States, Austria and most other countries do not. As a biologist and animal behavior expert, I believe it is time for the U.S. legal system to address this serious ethical issue.

    Like all nonhuman animals, chimps qualify as nothing more than property. It is perfectly legal to chain a chimp to a stake or put her in a 5-cubic-foot cage and inject her with hepatitis or HIV.

    That it's legal doesn't make it ethical. The sort of thinking that established this injustice is that we're smarter than them. But is "bright-makes-right" any basis for a sound moral system?

    You may be surprised to learn that we are not as highly evolved as chimpanzees. A recent analysis of 14,000 genes found that 233 chimp genes, compared with only 154 human ones, have been changed by natural selection since we shared a common ancestor.

    Despite popular assumptions, we are not always smarter.
    In a test of spatial memory, the numbers one to nine flash in a randomly scattered array across a computer screen for just one second before being replaced by white squares. A human observer is unlikely to recall the locations of more than two numbers in sequence. A chimpanzee will almost always successfully point to the former locations of all nine digits in the correct sequence. The dynamics of chimp society require keen awareness of where other group members are, which probably accounts for their exceptional skill on such tests.

    Chimpanzees were thought to have poor face recognition until someone had the bright idea of testing them on chimp faces instead of humans. They recognize chimp faces at least as well as we recognize human faces.

    Discoveries like this expose the prejudices that regard chimps as mere shadows of humans. But does it even matter how smart they are? After all, we don't deny basic rights and privileges to people of lower intellect.

    Surely what matters is what an individual feels. It is apparent that chimps experience life essentially as we do. They are highly aware, and chimp expert Frans de Waal asserts that they are as socially sophisticated as humans.


    They imitate, nurture, deceive, sympathize and plan. They have a broad emotional range spanning from jubilation to grief. Their cultures include different forms of tool manufacture and use, self-medication and bartering.

    So should they be granted rights? Governments are beginning to say "yes." In 1999, New Zealand banned the use of great apes in harmful experiments. And this year, the Balearic Parliament of Spain approved a resolution to grant legal rights to great apes.

    Meanwhile, Matthias Pan awaits his fate, as do 1,300 chimpanzees languishing in U.S. laboratories and an unknown number in squalid carnivals and roadside zoos. The day that they are free will be a great one for all apes -- and a step forward for humanity.


OK, (holding back the laughter) I love animals, but come on, do supposedly educated people actually believe this nonsense?

What does that 2 percent gene difference amount to?

Does a chimp's memory skills in matching numbers mean "we are not always smarter?"

What relevance does "recognizing an upside-down face" have in this debate?

Do chimps "experience life essentially as we do," or are there significant differences?

Are chimps "as socially sophisticated as humans?"

What exactly is significant about " 233 chimp genes, compared with only 154 human ones, have been changed by natural selection since we shared a common ancestor?"

If chimps are "more highly evolved" why aren't they now humans? What's the matter, some of them didn't want to come along? I will agree that some humans are obviously not as highly evolved as or smarter than chimps. Balcombe seems to be a perfect example of that.

Comments? Answers to my questions?

tomder55 answered on 05/18/07:

Some more Peter Singer crap . Philosophers frequently have too much time on their hands. When all humans enjoy the rights mentioned above;including the un-born , I will duly consider the rights of other simians.

Oh ,I have no doubt that they are frequenty treated cruelly ,but simular laws that apply to the treatment of my pet should apply to them .There are different levels of intellect throughout the animal world .I would argue that my Border Collie is as smart as a chimp . Both can be trained to do tricks.

Ayn Rand said that the concept of rights only arise in a social context. If I am living all alone on an island, the principle of the Right to Life, Liberty and the pursuit of happiness would be meaningless.Predators and those I prey on for my survival would not honor that social contract.

When Pete Seger tells me how he ensures the wilderbeast's rights when a lion attacks or how he would protect mine from a predatory virus that I could be cured of with some more research on a chimp ,then I will buy into his loony philosophy.

Itsdb rated this answer Excellent or Above Average Answer

Question/Answer
Itsdb asked on 05/17/07 - The latest Democrat petition

Tell President Bush: Bring our troops back home
With our National Guard and Reservists bogged down in a civil war in Iraq, we're less secure here at home. Without the capability to respond to emergencies, we're more vulnerable to the next natural disaster.

Send a message to President Bush and tell him to bring our troops home, where we need them to keep America safe


Don't they mean bring them home where they can do no more harm, Abu Ghraib, Gitmo, Haditha ... surely they don't mean to do as their drive is called, Protect our states. If so, when are they going to crack down on illegal immigration?

tomder55 answered on 05/18/07:

I swear the Democrats would promote meteorologists to Generals. You would think a brush fire or a melting ice cap is a bigger threat to national security than jihadistan !

We saw yesterday that they are going to capitulate on illegal immigration . Their idea of border security is to "certify "it's safe ,and then arrest border patrol agents for doing their job.

It fits the M.O. I guess ..... Abu Ghraib, Gitmo, Haditha ,El Paso .......

Itsdb rated this answer Excellent or Above Average Answer

Question/Answer
tropicalstorm asked on 05/18/07 - illegal immigrants

WASHINGTON - In a striking reach across party lines, the White House and key lawmakers agreed Thursday on a sweeping immigration plan to grant legal status to millions of people in the country unlawfully.

Sealed after months of secretive bargaining, the deal mandates bolstered border security and a high-tech employment verification system to prevent illegal workers from getting jobs.

President Bush said the proposal would "help enforce our borders but equally importantly, it'll treat people with respect."

The compromise brought together an unlikely alliance of liberal Democrats such as Sen. Edward M. Kennedy of Massachusetts and conservative Republicans such as Sen. Jon Kyl (news, bio, voting record) of Arizona on an issue that carries heavy potential risks and rewards for all involved.

Senate Majority Leader Harry Reid (news, bio, voting record), D-Nev., said debate would begin on Monday, but he cautioned, "I don't know if the immigration legislation is going to bear fruit and we're going to be able to pass it."

Almost instantly, the plan brought vehement criticism from both sides of the immigration issue, including liberals who called it unfair and unworkable and conservatives who branded it an overly permissive "amnesty."

The proposal constitutes a far-reaching change in the immigration system that would admit future arrivals seeking to put down roots in the U.S. based on their skills, education levels and job experience, limiting the importance of family ties. A new class of guest workers would be allowed in temporarily, but only after the new security measures were in place — expected to take 18 months.

"This is a bill where people who live here in our country will be treated without amnesty but without animosity," Bush said.

Kennedy hailed it as "the best possible chance we will have in years to secure our borders and bring millions of people out of the shadows and into the sunshine of America."

Kyl said the measure wasn't perfect, "but it represents the best opportunity that we have in a bipartisan way to do something about this problem."

It was clear, however, that many Republicans and Democrats were deeply skeptical. Reid said it needed improvement.

"I have serious concerns about some aspects of this proposal, including the structure of the temporary worker program and undue limitations on family immigration," Reid said.

Conservatives on both sides of the Capitol derided the deal as "amnesty" for illegal immigrants, using a politically charged word that figured prominently in campaigns across the country last year.

"I don't care how you try to spin it, this is amnesty," said Sen. Jim DeMint (news, bio, voting record), R-S.C.

The proposed agreement would allow illegal immigrants to come forward and obtain a "Z visa" and — after paying fees and a $5,000 fine — ultimately get on track for permanent residency, which could take between eight and 13 years. Heads of households would have to return to their home countries first.

They could come forward right away to claim a probationary card that would let them live and work legally in the U.S., but could not begin the path to permanent residency or citizenship until border security improvements and the high-tech worker identification program were completed.

A new crop of low-skilled guest workers would have to return home after stints of two years. They could renew their visas twice, but would be required to leave for a year in between each time. If they wanted to stay in the U.S. permanently, they would have to apply under the point system for a limited pool of green cards.

The program drew fire from liberal groups that said it was unworkable. They had joined Democrats in pressing instead for guest workers to be permitted to stay and work indefinitely in the U.S., and ultimately earn the chance to stay.

"Without a clear path to permanent residence for a healthy share of the future temporary workers, we run the risk of reproducing the widespread illegality that this bill is designed to address," said Frank Sharry, the executive director of the National Immigration Forum.

Sen. Byron Dorgan (news, bio, voting record), D-N.D., said he would try to kill the temporary worker program because it would bring in a potentially unlimited stream of immigrants to compete with Americans for jobs and depress wages.

In perhaps the most hotly debated change, the proposed plan would shift from an immigration system primarily weighted toward family ties toward one with preferences for people with advanced degrees and sophisticated skills. Republicans have long sought such revisions, which they say are needed to end "chain migration" that harms the economy.

Family connections alone would no longer be enough to qualify for a green card — except for spouses and minor children of U.S. citizens. Strict new limits would apply to U.S. citizens seeking to bring foreign-born parents into the country.

The issue quickly became a subject of debate among presidential candidates in both parties, exposing divisions among Republicans.

Sen. John McCain (news, bio, voting record), R-Ariz., who led the charge last year to push through an immigration overhaul, called the deal "the first step" and urged moving it forward before the politics of 2008 made such action impossible.

"We all know that this issue can be caught up in extracurricular politics unless we move forward as quickly as possible," said McCain.

Mitt Romney, another Republican presidential hopeful, issued a statement calling the plan "the wrong approach," saying it conferred "a form of amnesty" on illegal immigrants. "That is unfair to the millions of people who have applied to legally immigrate to the U.S.," the former Massachusetts governor said.

Former Republican Sen. Fred Thompson of Tennessee, who is weighing a presidential bid, said the measure should be scrapped in favor of one that secures the border.

"With this bill, the American people are going to think they are being sold the same bill of goods as before on border security," Thompson said in a statement.

Democratic Sen. Barack Obama (news, bio, voting record) of Illinois said the compromise needs work.

"Without modifications, the proposed bill could devalue the importance of family reunification, replace the current group of undocumented immigrants with a new undocumented population consisting of guest workers who will overstay their visas, and potentially drive down wages of American workers," Obama said in a statement

tomder55 answered on 05/18/07:

'Secretive bargaining',back door deals; Cutting the public from the debate ,that's how a major policy change is dealt with in a democracy ? The clue is that whenever they resort to those tactics it means they are going to stick it where you don't want it.

I support President Bush but I have to tell you. After 2 terms in office his legislative initiatives besides tax cuts have been a disaster . Besides the effect of the tax cuts which admittedly has been wonderful;he can point to :

McCain-Feingold ,which has proven to be the nightmare it's critics predicted

No Child Left Behind ;which he essentially piggy-backed onto a Ted Kennedy initative

and now this lame Immigration sell out which is another Kennedy inspired fiasco.

I'll tell you how out of touch the President is on this issue. Even Robert Byrd called the deal amnesty that rewards "those who break our immigration laws." Even the socialist EU has realised that immigration must be controlled and is introducing much tougher laws .

I just love the crap where they say that the Feds. will certify that the border is sealed . I have worked with Federal agencies for years and I have to tell you that their idea of certified leaves alot to be desired. How can they claim they will close the border when they are arresting their own border agents for doing their jobs???????

They have been neglegent in enforcing existing laws. Why should we believe that they will enforce new ones ?

Major provisions of the bill are this :

CURRENT ILLEGAL IMMIGRANTS

-They could come forward immediately and receive probationary legal status.

-Bill creates a new four-year, renewable "Z" nonimmigrant visa for those present within the U.S. before Jan. 1, 2007.

-Nonimmigrants may adjust status to lawful permanent residence once they pay $5,000 in fees and fines and their head of household returns to their home country.

-People under age 30 who were brought to the U.S. as minors could receive their green cards after three years, rather than eight.

-Nonimmigrant farmworkers who can demonstrate they have worked 150 hours or three years in agriculture can apply for green cards.

-No green cards for nonimmigrants can be processed until "triggers" for border security and workplace enforcement have been met, estimated to take 18 months. Processing of green cards for nonimmigrants will begin after clearing the visa backlog, which takes eight years.


BORDER SECURITY

-Hire 18,000 new border patrol agents.

-Erect 200 miles of vehicle barriers and 370 miles of fencing along the U.S.-Mexico border.

-Erect 70 ground-based radar and camera towers along the southern border.

-Deploy four unmanned aerial vehicles and supporting systems.

-End the program in which illegal immigrants are released upon apprehension.

-Provide for detaining up to 27,500 aliens per day on an annual basis.

-Use secure and effective identification tools to prevent unauthorized work.

---

WORKPLACE ENFORCEMENT

-Require employers to electronically verify new employees to prove identity and work eligibility.

-Increase penalties for unlawful hiring, employment and record keeping violations.

---

GUEST WORKERS (requires border security measures to be in place first)

-Create a new temporary guest worker program with two-year "Y visas," initially capped at 400,000 per year with annual adjustments based on market fluctuations

-Workers could renew the Y visa up to three times, but would be required to return home for a year in between each time. Those bringing dependents could obtain only one, nonrenewable two-year visa.

-Families could accompany guest workers only if they could show proof of medical insurance and demonstrate that their wages were 150 percent above the poverty level.

---

FUTURE IMMIGRANTS

-Spouses and minor children of U.S. citizens and permanent residents would be eligible for green cards based purely on their family connections, but other relatives such as adult children and siblings would not.

-380,000 visas a year would be awarded based on a point system, with about 50 percent based on employment criteria, 25 percent based on education, 15 percent on English proficiency and 10 percent on family connections.

-Apply new limits to U.S. citizens seeking to bring foreign-born parents into the country.

-Visas for parents of U.S. citizens would be capped annually at 40,000 and those for spouses and children at 87,000.


Message to all those legal immigrants who have played by the rules all these years ...How stupid are you ?

tropicalstorm rated this answer Excellent or Above Average Answer

Question/Answer
Dark_Crow asked on 05/17/07 - Who the Hell is Ron Paul!...................

Why should we support Ron Paul?

One thing I’m aware of: He wants to end birth-right citizenship for the children of illegal immigrants.

The ‘mainstream’ media doesn't like him, so all the more reason to support him.

In general, he is simply the best candidate for white Americans to support. As a matter of fact, the black population should vote for him as well, due to his stance against illegal immigration. Of course they won’t in view of the inherent racism against all whites.

tomder55 answered on 05/18/07:

He is a libertarian . There is alot of issues I agree with him and alot I don't . He is weak in national defense issues so that disqualifies him in my book. His basic position is that if we retreated to fortress America and left them all alone they would not attack us. I think that is hopelessly naive .

His best answer in Tues. debate involved the question of what cuts he would make to reduce spending and he started reciting a litany of Executive Departments he would eliminate .(Education etc.)His views of a limited gvt. is his greatest strength but regarding national security it is also his biggest weakness.

I agree with the birth-right citizenship issue but I think it would require amending the Constitution.It certainly would be challenged on 14th Amendment grounds and SCOTUS has consitently ruled in favor of it for legal residents ,and has implied that it includes the children of illegals.

The author of the Fourteenth Amendment, Rep. John A Bingham (OH), said : "I find no fault with the introductory clause, which is simply declaratory of what is written in the Constitution, that every human being born within the jurisdiction of the United States of parents not owing allegiance to any foreign sovereignty is, in the language of your Constitution itself, a natural born citizen."

The introductory clause reads:

Section 1. All persons born or naturalized in the United States, and subject to the jurisdiction thereof, are citizens of the United States and of the State wherein they reside.

Some in Congress made an attept in 1997 to take up the issue but the effort failed after there was intense opposition by immigrants rights groups.

After yesterday's sell out by the Senate (both Parties) I think the mood of Congress is to capitulate on the issue of illegal immigration totally .

Dark_Crow rated this answer Excellent or Above Average Answer
Itsdb rated this answer Excellent or Above Average Answer

Question/Answer
paraclete asked on 05/17/07 - He's bad and mean, ..

.. and worse than George Bush?

Does this mean he must be doing something right?
'Howard is a war criminal



Australia is funding terrorism and Prime Minister John Howard is a war criminal, Zimbabwe's Information and Publicity Minister Sikhanyiso Ndlovu says.

Zimbabwe's latest attack on the Australian government comes after Mr Howard banned the one-day cricket team from touring the southern African nation this September because of the despotic regime of President Robert Mugabe.

"The Australian people should really stand against John Howard's gestapo tendencies and interference with other states. He wants to cause insecurity in our country and that we will not allow," Dr Ndlovu told ABC Radio today.

"He is the international gestapo and a criminal ... he is worse than anybody else, his actions in banning the cricket is just one example of being the gestapo," Dr Ndlovu said of Mr Howard.

Last week, Mr Howard said he did not want the team to tour Zimbabwe because Mugabe was a "grubby dictator".

Cricket Australia was faced with the possibility of paying a multi-million dollar fine to the International Cricket Council for its failure to play the three one-day matches, but the council decided to not to impose the fine.

Dr Ndlovu said Australia was financing people who were destabilising the Zimbabwean regime.

"You continue to finance your puppets in our country who don't love their country.

"They are also the ... monies that come to them are to cause violence, you know, terrorist activities, I've got a long list of their terrorist activities here," Dr Ndlovu said.

AAP

tomder55 answered on 05/17/07:

I would consider it a badge of honor to be verbally attacked by anyone in the criminally incompetetent thug regime of Zimbabwe .Under Robert Mugabe lead, the country has been reduced from food sufficiency and net food exporter, to widespread starvation, plus inflation that runs above 2,000 percent.Only 10 percent of its normal wheat crops have been planted this year.They are already suffering from a severe shortages in maize.Zimbabwe was once self sufficient in wheat and maize production. Under the authority of Mugabe the government began to confiscate large commercial farms owned by white people and it has been unable to operate them successfully.


Now those ass wipes at the UN and the rest of the African block of nations ;acting in defiance of the West ,the nations that supply billions for aide to the continent, further made a mockery of the institution by electing Zimbabwe, a disgracefully ruled country, to head the U.N. Committee on Sustainable Development.It would be comical if there wasn't so much human misery associated with it.

Dr Ndlovu said Australia was financing people who were destabilising the Zimbabwean regime.

I certainly hope so and I hope the US is assisting . It is the humanitarian thing to do .

Australia said Monday it would spend $15 million backing Mugabe's critics, just a day after banning a cricket tour.I admire John Howard like few politicians in this world .

ETWolverine rated this answer Excellent or Above Average Answer
labman rated this answer Excellent or Above Average Answer
paraclete rated this answer Excellent or Above Average Answer

Question/Answer
Itsdb asked on 05/16/07 - Pelosi's Nuclear Gambit

From The Corner blog at NRO...

Boehner's office just sent this out:

DEMOCRATS TO CHANGE 185 YEAR-OLD HOUSE RULE TO ALLOW TAX HIKES WITHOUT HAVING TO VOTE

    May 16, 2007

    In a stunning move, House Democrats today revealed they will attempt to rewrite House rules that have gone unchanged since 1822 in order to make it possible to increase taxes and government spending without having to vote and be held accountable. House Republican Leader John Boehner (R-OH) today vowed Republicans will use every available means to fight this unprecedented change.

    “This is an astonishing attempt by the majority leadership to duck accountability for tax-and-spend policies the American people do not want,” Boehner said. “The majority leadership is gutting House rules that have been in place for 185 years so they can raise taxes and increase government spending without a vote. House Republicans will use every tool available to fight this abuse of power.”

    Last November, House Democratic leaders promised the most open, ethical Congress in history:

    “[W]e promised the American people that we would have the most honest and most open government and we will.” (Nancy Pelosi press stakeout, December 6, 2006)

    “We intend to have a Rules Committee ... that gives opposition voices and alternative proposals the ability to be heard and considered on the floor of the House.” (Steny Hoyer in CongressDaily PM, December 5, 2006)


    The rules House Democrats are seeking to change have not been changed since 1822.

    Republicans have already achieved significant legislative successes on the House floor with 11 consecutive “motion-to-recommit” victories that exposed flaws and substantively improved weaknesses in underlying Democrat bills. But rather than living by the same rules which have guided the House of Representatives for 185 years, Democrats are proposing to change the rules in order to game the system and raise taxes and increase spending without a House vote. What are House Democrats afraid of?


Andy McCarthy responds:

    Any chance the mainstream media will refer to Pelosi's procedural maneuver as "The Nuclear Option"?

    I didn't think so. That's evidently only for right-wing maniacs who want to force an accountable vote on confirming Bush judges, not for right-thinking liberals who want to avoid an accountable vote on raising Americans' taxes. Good to get that straightened out.


The fox is guarding the hen house folks. Not six months into their "corruption sweep" the Democrats are pulling out all the stops and doing every single thing they've complained the Republicans were doing - and then some.

When will the drive-by media to get off their asses and start hammering the Democrats like they've hammered Bush and the Republicans? I'm just hoping the American people aren't so gullible as to fall for their dirty tricks like they fell for their Trojan horse campaign last year.

And maybe they're catching on ... it was just a few weeks ago the papers were reporting that congress' approval rating was higher than Bush's. Last I heard congress' approval rating was 29 percent - and yet they still preach of some alleged mandate from the voters. Apparently all of those voters wear tin foil hats...

tomder55 answered on 05/17/07:

When a bill is voted out of committee and is on its way to the floor for a vote, the minority can send it back to committee, and preventing the vote until it comes back out again. And then the minority can send it back again, etc. That's the part that's been in effect since 1822.

If Pelosi and the Democrats change that, how does that make it possible to "raise taxes and increase spending without a House vote"?On the surface it sounds like they will merely be able to send it up for a full House vote whether the minority likes it or not... but the vote still takes place.

Maybe there's something about the proposed changes the posting didn't mention?

Congressman Eric Cantor explains that this is actually about the PAYGO rules. So it begins to make some sense.

PAYGO is that ploy where revenue is generated automatically with every addtional spending initiative. So yes,conceivably taxes could be formula increased without an additional vote for the increase under that system.

It's a funny thing. When the Republicans contemplated changing Senate rules we heard cassandras at the NY Slimes calling it the end of democracy ;the republic ,the Constitution.Robert sheets Byrd was waving his pocket constitution and lecturing the Senate about tradition and comity .

Now that Pelosi is attempting a simular rule change that breaches the tradition of the House I'm sure the Slimes will be simularily outraged.

As for the open government business ;well the Republican maneuvers appear to be an attempt for openness and accountability .Certainly tax increases should be subject to an up or down vote where everyone is on the record.


That is also the big difference between the Republicans attempting to change the rules and the Democrats. When the Republicans contemplated the "nuclear option" they were attempting to get an honest "up or down vote" on judicial nominees. Pelosi is trying to prevent one on tax increases.

~~~~~~~~~~~~~~~~~~~~~~~~~~~~~~~~~~~~~~~~
btw ;scroll down the Corner some more and read Kate O'Beirne's postings about the slick move both Parties in the Senate are trying to make on immigration reform. Sounds like conservatives are to be sold out by the national Republican party again.

Itsdb rated this answer Excellent or Above Average Answer

Question/Answer
Itsdb asked on 05/15/07 - Just How Crazy are the Democrats?

The paranoid style in American liberalism.

By Jonah Goldberg

    Most fair-minded readers will no doubt take me at my word when I say that a majority of Democrats in this country are out of their gourds.

    But, on the off chance that a few cynics won’t take my word for it, I offer you data. Rasmussen Reports, the public opinion outfit, recently asked voters whether President Bush knew about the 9/11 attacks beforehand. The findings? Well, here’s how the research firm put it: “Democrats in America are evenly divided on the question of whether George W. Bush knew about the 9/11 terrorist attacks in advance. Thirty-five percent of Democrats believe he did know, 39 percent say he did not know and 26 percent are not sure.”

    So, one in three Democrats believe that Bush was in on it somehow, and a majority of Democrats either believe that Bush knew about the attacks in advance or can’t quite make up their minds.

    There are only three ways to respond to this finding: It’s absolutely true, in which case the paranoid style of American liberalism has reached a fevered crescendo. Or, option B, it’s not true, and we can stop paying attention to these kinds of polls. Or there’s option C — it’s a little of both.

    My vote is for C. But before we get there, we should work through the ramifications of A and B.

    We don’t know what kind of motive respondents had in mind for Bush, but the most common version has Bush craftily enabling a terror attack as a way to whip up support for his foreign policy without too many questions.

    The problem with rebutting this sort of allegation is that there are too many reasons why it’s so stupid. It’s like trying to explain to a 4-year-old why Superman isn’t real. You can spend all day talking about how kryptonite just wouldn’t work that way. Or you can just say, “It’s make-believe.”

    Similarly, why try to explain that it’s implausible that Bush was evil enough to let this happen — and clever enough to get away with it — yet incapable either morally or intellectually of doing it again? After all, if he’s such a villainous super-genius to have paved the way for 9/11 without getting caught, why stop there? Democrats constantly insinuate that Bush plays politics with terror warnings on the assumption that the higher the terror level, the more support Bush has. Well, a couple of more 9/11s and Dick Cheney will finally be able to get that shiny Bill of Rights shredder he always wanted.

    And, if Bush — whom Democrats insist is a moron — is clever enough to green-light one 9/11, why is Iraq such a blunder? Surely a James Bond villain like Bush would just plant some WMD?

    No, the right response to the Rosie O’Donnell wing of the Democratic Party is, “It’s just make-believe.” But if they really believe it, then liberals must stop calling themselves the “reality-based” party and stop objecting to the suggestion that they have a problem with being called anti-American. Because when 61 percent of Democrats polled consider it plausible or certain that the U.S. government would let this happen, well, “blame America first” doesn’t really begin to cover it, does it?

    So then there’s option B — the poll is just wrong. This is quite plausible. Indeed, the poll is surely partly wrong. Many Democrats are probably just saying that Bush is incompetent or that he failed to connect the dots or that they’re just answering the question in a fit of pique. I’m game for option B. But if we’re going to throw this poll away, liberals need to offer the same benefit of the doubt when it comes to data that are more convenient for them. For example, liberals have been dining out on polls showing that Fox News viewers, or Republicans generally, are more likely to believe that Saddam Hussein was involved in 9/11. Now, however flimsy, tendentious, equivocal, or sparse you may think the evidence that Hussein had a hand in 9/11 may be, it’s ironclad compared with the nugatory proof that Bush somehow permitted or condoned those attacks.

    And then there’s option C, which is most assuredly the reality. The poll is partly wrong or misleading, but it’s also partly right and accurate. So maybe it’s not one in three Democrats suffering from paranoid delusions. Maybe it’s only one in five, or one in 10. In other words, the problem isn’t as profound as the poll makes it sound. But that doesn’t mean the Democratic Party doesn’t have a serious problem.


Comments?

Steve
P.S. And for you conspiracy theorists that don't believe the fire in the World Trade Center couldn't possibly be hot enough to melt steel, I give you this reminder.

tomder55 answered on 05/16/07:

maybe it's not 1 in 3 Democrats suffering from paranoid delusions. Maybe it's only 1 in 5 , or 1 in 10. In other words, the problem isn't as profound as the poll makes it sound. But that doesn't mean the Democratic Party doesn't have a serious problem.

I would have to say that if the party had 1 in 100 who think this way then they have serious problems. If there are any sober Democrats left they have to insist that the leadership stands up to the Soros/Kossak axis and put them in their place. It won't be Edwards ;he panders to them .It won't be Obama ;he has noce rhetoric. But he's clueless . Who knows ? Maybe Hillary will use them in a Sista Soulja moment with them .

But the Demoncrats are likely to turn this around and say it is proof of Bush's credibility problem...or cite polls that show some Republicans believe Saddam responsible for 9-11.

ETWolverine rated this answer Excellent or Above Average Answer
Itsdb rated this answer Excellent or Above Average Answer

Question/Answer
kindj asked on 05/14/07 - Did you know.......................................

.......that trying to teach the concepts of chivalry (honor, dignity, selflessness, etc) to 7th grade wannabe gangsters is like trying to teach a damn pig to sing?

....sigh....

Where are we going, and why are we in this handbasket?


DK

tomder55 answered on 05/15/07:

My wife currently teaches 8th grade science in a challenging school district(to say the least). She does not need the job .I earn enough for our family ;and she also gave up a promising and lucrative career in the Pharmaceutical Industry right when she was poised to cash in and advance into upper management because she always dreamed about being a teacher .It is a passion that she persued ;quit her job,spend almost a decade going back to school .She got her Masters degree .

Was it worth it ? I don't know . She has touched and influenced a few of her students .She has the advantage of bringing the experiences of a practical science career into her lessons .She has above all maintained a disciplined classroom conducive to learning . She spends long hours after school grading and going the extra mile to be able to give the best lessons she can each day. She has also spent quite a bit of her own money getting basic supplies that the district seems to not be able to provide.

Her reward was given to her a couple of weeks ago. The Principle decided that she was not the type of teacher she wanted in the school. She was graciously given the option to resign before she was canned . This ;one year before she is tenure eligible.

Honor ,dignity ,selflessness are codes she lives by . Yet ,if there is not a comittment from the highest levels ...if society itself has rejected 'chivalry' as an antiquated concept ;I think it is too much for a lone teacher to carry the torch . I will advice her to toe the line next time (if she doesn't say screw it and just goes back to the industry where her abilities are appreciated ),until she at least gets the tenure protection.

Itsdb rated this answer Excellent or Above Average Answer
kindj rated this answer Excellent or Above Average Answer

Question/Answer
HANK1 asked on 05/11/07 - Don't you wish Harry Truman was President?



Why Harry S. Truman is the president we want now!

By Marshall Loeb, MarketWatch
Last Update: 4:59 PM ET May 11, 2007


NEW YORK (MarketWatch) -- The cover of the current issue of Newsweek magazine features the jaunty photo of a president from the U.S. past, with the headline: "Wanted: A New Truman."
To which we say, Amen!

Several times in these columns we have praised Harry S. Truman, the nation's 30th president (from 1945 to 1953). Now that he is being held up as the prototypical example of the president we presumably want, it's a good time to examine just who and what he was.

Harry Truman was cantankerous and controversial, but also honest, loyal, determined, far-sighted, fearless, and -- above all -- courageous. He made more decisions that changed the course of history than any other president. Had he not occupied the office, America's story would have been quite different, and not nearly as ennobling or successful!

* Just a couple of excerpts from Loeb's article.

What do you think?

HANK

tomder55 answered on 05/12/07:

Hank I have repeatedly made the comparison between President Bush and Truman. Both were there at the beginning of a new ideological existential struggle .Both became the leader in an unpopular war that was part of that larger confrontation. Both will have created the doctrine that will be the future on how to wage the conflict in the future. Both will have left the White House very unpopular because of their uncompromising ways .Both will ultimately be vindicated by history.

HANK1 rated this answer Excellent or Above Average Answer

Question/Answer
Dark_Crow asked on 05/11/07 - who in the hell is Duncan Hunter?..................

.

tomder55 answered on 05/12/07:

He's one of the anklebiting second -tier candidates in the Republican field . He is in the race, but has no chance to win. As you demonstrate,his biggest problem right now is name recognition.

He is a Congressman from San Diego. His big issue is border security .He is as close to ideologically pure conservative as anyone else in the field . But the litmus test for Republicans this year cannot be ideological as much as practical. Who has the best chance of beating Hillary is what should be on their minds.

Look for him to try to differentiate himself from the 7 other 2nd tier candidates in the next debate [scheduled this week in S.Carolina] and attempt to break out of the pack and join Guiliani ,McCain ,and Romney as contenders. It won't happen .

Dark_Crow rated this answer Excellent or Above Average Answer

Question/Answer
Itsdb asked on 05/11/07 - Islamist-Left Alliance A Growing Force

The MEMRI Report

By STEVEN STALINSKY
May 9, 2007

"Where else can you sit down in a single evening and listen to senior people from Hamas, Hezbollah, the Muslim Brotherhood, people from the revolutionary left and the antiwar movement from around the globe?" — British Trotskyite John Rees at the Cairo Anti-War Conference, April 2007

    Over the past year, multiple international conferences have featured leaders of the anti-global left and Islamist groups working together. Go to any anti-war or anti-globalization demonstration in the West and chances are you will see the flags of Hezbollah and Hamas waved by people wearing Che Guevara T-shirts. And at some of these meetings, members of such radical Islamist groups as the Muslim Brotherhood, Hamas, and Hezbollah have enjoyed starring roles.

    The roster of Islamist-left alliances quietly grows every day: Massachusetts Institute of Technology linguistics professor Noam Chomsky praises Hamas and denounces America on Hezbollah's Al-Manar television. London Mayor Ken Livingstone invites a leading Islamist, Sheikh Yosef Al-Qaradawi, who is known for supporting suicide attacks, to visit his city. Iranian President Ahmadinejad calls for a world without America even as he plays host to a Tehran peace conference attended by American Mennonites, Quakers, Episcopalians, Methodists, and leaders of the National Council of Churches.

    The key forum at this year's annual Cairo Anti-War Conference was titled "Bridge-building Between the Left and Islam," and focused on practical ways to increase cooperation. The aim of the conference sessions were described in one piece of literature as tackling "the challenges and prospects facing the international anti-war and pro-intifada movements" and planning "strategy and tactics for bridging the gap and uniting Islamist and leftist ranks in the face of U.S. imperialism and Zionism."

    The Arabic press has lauded this phenomenon. An article in Egypt's Al-Ahram Weekly praises what it refers to as "Arab activists taking the lead in the growing anti-war movement worldwide."

    Some Islamist issues on which the anti-global left appears to have found common ground are countering the international boycott of Hamas, calling for the boycott of Israel, supporting Iran against the threat of a U.S. attack (as well as supporting its "right" to develop nuclear weapons), supporting avowedly anti-American countries such as Cuba and Venezuela, and, above all, opposing the war in Iraq.

    Many leftist blogs and Web sites monitored by MEMRI have been increasingly open to working with Islamist groups on the goals they both share — most notably the desire for America to leave Iraq. British Trotskyite John Rees, a regular at the Cairo conference since it started in 2002, told Al-Ahram in April that at this year's gathering he had discussed in particular the "ongoing dynamic between the anti-war movement and Egypt's Muslim Brotherhood." Mr. Rees also believes that the Islamist-Left coalition is gaining strength, while what is known as the "coalition of the willing" has faltered.

    In addition, MEMRI researchers monitoring jihadi Web sites have recently found Islamists trying to influence American anti-war efforts. On the Islamist Al-Mohajroon Web site, someone with the username Al-Wathiq Billah instructs readers on how to infiltrate popular American Internet forums to distribute jihadist films and spread disinformation about the war.

    "There is no doubt, my brothers, that raiding American forums is among the most important means of obtaining victory in the fierce media war ... and of influencing the views of the weak-minded American who pays his taxes so they will go to the infidel American army. This American is an idiot and does not know where Iraq is ... Every electronic mujahid" must engage in this raiding, Mr. Billah writes.

    Mr. Billah advises his jihadist readers to "register yourself using a purely American name" and to "invent stories about American soldiers you have personally known (as classmates... or members in a club who played baseball and tennis with you) who were drafted to Iraq and then committed suicide while in service by hanging or shooting themselves." The writing should, he says, provoke "frustration and anger towards their government, which will ... render them hostile to Bush ... and his Republican Party, and make them feel they must vote to bring the troops back from Iraq as soon as possible."

    Such an emerging alliance can only be expected to play a negative role in the ongoing war on terror.


Now I understand why the left dismisses Islamic terrorism - they believe they're on the same side. LIke I said to tom earlier, the left has just spent too much time in this position ...

tomder55 answered on 05/12/07:

Noam Chomsky and Ayman al-Zawahiri ...perfect together! One group wants to abolish religion .The other wants to have it's cult spread across the planet.So naturally they are perfect allies.Chomsky praises Enlightenment Ideals (one of his favorite expressions)and then supports Hamas and Hezbollah and sees no contradiction in that .


Atlas Shrugs puts it this way :

It is interesting to hear people voice surprise that the left would join forces with misogynistic, oppressive Islam. But it makes perfect sense. Both are a dogma, both require group think, both are collectivist, and both are entirely anti-individual. Both are totalitarian in nature. Perfect marriage. And to those shocked that feminists would go along, that too is Lenin/Marxist in origin.

It's that ole Hitler -Stalin Axis of convenience all over again ,....the eneny of my enemy etc. [their common enemy ? America]What the moonbats don't understand is that just like the end of the Ribbentrop-Molotov pact ,the Islamo-nazis are bound to turn on the useful idiots somewhere down the road.

Itsdb rated this answer Excellent or Above Average Answer

Question/Answer
Itsdb asked on 05/11/07 - The Islamic Threat to Europe: By the Numbers

By KRISTOFFER LARSSON

    Some things interest the media, others don't. Since the fall of the USSR, the United States has sought another menace to designate as the ultimate evil, a world threat the Americans desperately need to take on. The 9/11 attacks gave them that enemy. And when the White House speaks, the media listens obediently.

    Over the last number of years the "Islamic threat" has become one of the favourite issues for media coverage. It's all over the news--Muslims leaders pronouncing threats against the countries participating in occupying Muslim land.

    While America is the Western country most succumbed to the fear of Islamism, things aren't much better in Europe. Its media is highly Americanised and thus eager to reiterate U.S. governmental positions towards the non-Western world. Islamic terrorism is subsequently a theme close to the hearts of European journalists as well.

    Following this, you might think the journalists would be beside themselves with joy when the European Police Office (Europol) releases its first report on terrorism in the EU. I can assure you they weren't. In fact, to my best knowledge, not a single Swedish paper or news-channel has paid any attention to it whatsoever. I haven't seen it receiving much attention in other EU countries either (kudos to the EUobserver for having the decency to report on it). The report is namely a grave disappointment for the anti-Islamic campaigners.

    There were 498 incidents in eleven EU countries last year labelled as "terrorist attacks." The Basque separatist group ETA did best (136 terrorist attacks) and was responsible for the only deadly attack, killing two in Madrid. The remaining 497 fortunately cost no human lives.

    How about the Islamic terrorists then? Considering the perpetual warnings in our daily papers, the findings in the Europol report is, to say the least, surprising. The truth is that Islamists only carried out one out of the 498 terrorist attacks in the European Union in 2006. Don't believe me? The entire report (pdf) is available on Europol's website. Had Islamic fundamentalists been behind a higher number of attacks-say 136-it would have been front page news at every big daily. One attack is simply too few--it won't do if the image of an "Islamic threat" is to live on.

    The Europol report devotes several pages to Islamist terrorism, despite the low number. Except for the one attack in Germany this group was responsible for (which, by the way, failed and resulted in no victims), also Denmark and the United Kingdom reported that Islamists plotted to carry out one attack in each country respectively (incidentally, all three countries are accessory to the illegal occupation of Iraq). However, since these plans in both cases were exposed before they were set to work, they were not included among the 498. Either way, even after taking these plots into account, the report proves the genuine magnitude of Islamic terrorism in Europe--it's not exactly a huge threat.

    If we look at the people arrested on suspicion of terrorism offences, the figures are rather disproportionate; about half of them arrested were Muslim. In plain English: Muslims are a group causing very little terrorism in Europe, while at the same time much more likely to be arrested on suspicion of it. The constant media coverage of Muslims being arrested creates the false image of a serious threat in order to benefit the imperialist world-view Washington wants us to adopt. Meanwhile the Americans and their accomplices are carrying out genocide in Iraq. Clearly, something needs to be done about the media.


I have no idea who Kristoffer Larsson is, but apparently he's a moonbat in denial. When was the last time you've noticed the media "listen obediently" when Bush speaks?

A few notes from the report he cites:

    Altogether 498 attacks were carried out in the EU in 2006.The vast majority of them resulted in limited material damage and were not intended to kill.


Contrast that with "The London airplane plot and the trolley bomb case of Germany targeted civilians and transportation infrastructure in Member States.

The report continues with "The radicalisation process of the suspects in these cases is reported to have been rapid. The weapon of choice of Islamist terrorists are Improvised Explosive Devices made with homemade explosives. The cases reported by the UK and Denmark involved the use of Triacetone Triperoxide (TATP), a highly volatile explosive the use of which requires a certain degree of expertise."

    Half of all the terrorism arrests were related to Islamist terrorism. France, Spain, Italy and the
    Netherlands had the highest number of arrests of Islamist terrorist suspects. The majority of the arrested suspects were born in Algeria,Morocco and Tunisia and had loose affiliations to North African terrorist groups, such as the Moroccan Islamic Combatant Group and the Salafist Group for Preaching and Combat.


The report also notes, "The frequency of video statements by members of the original al-Qaeda leadership and other Islamist terrorists shows a marked increase. The propaganda is of greater sophistication, of high quality and more professional. English is used more often, either in direct speech or in subtitles, allowing potential access to a wider audience than previous publications in Arabic. These facts may point to a coordinated global media offensive from Islamist terrorists.

Meanwhile, Americans are engaged in genocide in Iraq ... must be that complicit media failing to report this genocide in favor of reports of IED's and suicide bombings. As I've said before, deny at your own peril.

Steve

tomder55 answered on 05/12/07:

For a different perspective of the threat posed by Isamic migration to Europe read this :

What Does Muslim Immigration Cost Europe?
Fjordman - 5/10/2007
Do gang rapes boost GDP? Was that an offensive question, you say? Well, according to Sweden's finance minister Pär Nuder, more immigrants should be allowed into Sweden in order to safeguard the welfare system. However, in reality estimates indicate that immigration costs Sweden at least 40 to 50 billion Swedish kroner every year, probably several hundred billions, and has greatly contributed to bringing the Swedish welfare state to the brink of bankruptcy. An estimated cost of immigration of 225 billion Swedish kroner in 2004, which is not unlikely, would equal 17.5% of Sweden's tax income that year, a heavy burden in a country which already has some of the highest levels of taxation in the world.
At the same time, the number of rape charges in Sweden has quadrupled in just above twenty years. Rape cases involving children under the age of 15 are six times as common today as they were a generation ago. Resident aliens from Algeria, Libya, Morocco and Tunisia dominate the group of rape suspects. Lawyer Ann Christine Hjelm, who has investigated violent crimes in one court, found that 85 per cent of the convicted rapists were born on foreign soil or by foreign parents. Swedish politicians want to continue Muslim immigration because it boosts the economy, yet the evidence so far indicates that it mainly boosts the number of gang rapes. Meanwhile, research shows that fear of honor killings is a very real issue for many immigrant girls in Sweden. 100.000 young Swedish girls live as virtual prisoners of their own families.

An ever growing group of non-western immigrants in Norway is dependent on welfare. This was the conclusion of a study by Tyra Ekhaugen of the Frisch Centre for Economic Research and the University of Oslo. Ekhaugen's research contradicted the often heard assertion that Norway's labor market depends increasingly on immigrants. The study indicated quite the reverse. If the present evolution continues, immigration will increase the pressure on the welfare state rather than relieving it because many immigrants do not join the tax-paying part of the population. "Non-Western immigrants" in Norway are recipients of social security benefits ten times as frequently as native Norwegians. If we remember that "non-Western immigrants" include Chinese, Indians and other non-Muslims who are known for (and statistically proven to be) hard working, this speaks volumes of the heavy burden Muslims constitute on the welfare state.

Journalist Halvor Tjønn from newspaper Aftenposten, one of the few genuinely critical journalists in the country, in June 2006 cited a report from NHO, the Confederation of Norwegian Enterprise. NHO stated that the current immigration policies were a serious threat to the country's economy. Norway is the planet's third largest exporter of oil, next to Saudi Arabia and Russia. Yet according to NHO, there is a risk that much of the profit Norway earns from oil could be spent on paying welfare for a rapidly growing immigrant population. The most profitable immigration would be high-skilled workers who stay for period of limited duration, but at the same time not too brief. A Danish think tank has estimated that the net cost of immigration was up to 50 billion kroner every year, and those were cautious estimates. Denmark could thus save huge sums by stopping immigration from less developed countries. A study found that every other immigrant from the Third World -- especially from Muslim countries -- lacked the qualifications for even the most menial jobs on the organized Danish labor market.

Professor Kjetil Storesletten at the University of Oslo said that the net contribution of immigrants to the economy was probably negative in
Norway, too. "Admitting immigrants with low levels of education leads nowhere. We cannot continue the immigration policies we have followed until now," said Storesletten. In Norway, social benefits and salaries for low-skilled workers are among the highest in the world. At the same time, the salaries for highly skilled workers are comparatively lower and the taxes are high. This compressed salary structure is the result of decades of Socialist policies in Scandinavia. It leads to attracting people with lower skills and little education, but also makes the countries less attractive for researchers and scientists. Still, the agency that handles immigration to Norway, UDI, in 2005 thought that the country must make it more attractive for both skilled and unskilled workers to move to Norway. "We need more immigrants," claimed UDI chief Trygve Nordby. "Too few dare to say that we have a large need for non-professional workers as well," he said. UDI, in turn, should be able to have more flexibility in deciding cases, and process cases more quickly and efficiently. As it turned out later, the bureaucrats of UDI were in fact so "flexible" that they had been running their own, private immigration policies, and that the agency's liberal interpretation of asylum rules had "stretched the boundaries" of the law. UDI violated both the law and political directives when it granted residency permits to nearly 200 Iraqi Kurds in the fall of 2005. A commission that probed the controversial permits blasted the former head of UDI, Trygve Nordby, and his successor resigned. More than half of the social security benefits in the city of Oslo are spent of non-Western immigrants, a portion that has grown tremendously over the years. City council leader Erling Lae warns against prejudice and states that without immigrants, there would be "complete chaos" in Oslo. Meanwhile, Norwegian newspapers are worried that online debates are "flooded with racist comments" about Muslims. One of the racist comments they mentioned was this one: "A large number of Muslims out of their own, free will choose to live in parallel societies where they speak their mother tongue, watch foreign TV, despise the society they have moved to and pick spouses from their original homelands. The only contact they have with the natives is in the social security office."

"The debate is often colored by ignorance. It is not a human right to be heard at any given time. It requires a certain minimum of social intelligence to be allowed to participate in the public debate," says Esten O. Sæther, online editor of the Leftist, pro-Multicultural newspaper Dagbladet. Sæther warns that the newspapers may have to impose stricter censorship on their online discussion forums and the comments to online articles. However, Dagbladet seems to have little understanding of why so many people are fed up with Muslims and their demands. Neither have they admitted that they were wrong in their one-sided praise of the positive effects of Muslim immigration for decades, nor apologized for demonizing the right-wing Progress Party and its leader Carl I. Hagen throughout the 90s for their "racism" for questioning the real costs of this immigration.

On the contrary, although information about the costs of Muslim immigration has been available for several years, this was a non-issue
in Norwegian media prior to the national elections in 2005. Instead, they were focusing on "poverty" in Norway and the need for increased welfare spending. This in a country that is among the wealthiest in the world and which provides its citizens with the highest social security benefits in Europe. Norway's media coverage of the national elections revealed a desire for a Leftist government, said Election expert Aarebrot, a professor at the Institute for Comparative Politics at the University of Bergen. He argued that, with few exceptions, most of Norway's media leans to the left politically. "Most newspapers are what I would call politically correct. By politically correct in Norway today I mean slightly radical, urban and liberal," Aarebrot said. Nearly 70 percent of journalists vote Labor (Ap), Socialist Left (SV) or Red Electoral Alliance (RV) according to a poll, and this is reflected in the press, Aarebrot said.

Oslo will have a non-Western majority in a few decades, if the current trends continue. There are now several researchers who predict that in Norway, Sweden and Denmark, the native population and their descendants will become a minority in their own country within this century. The only question is when. Since the Islamic Jihad usually enters a much more aggressive and physical phase once the Muslim population reaches 10 – 20% of the total in any given area, this does not bode well for the future of the urban regions in Scandinavia. Will they turn out different from similar regions in Thailand, the Philippines or Nigeria?

Admittedly, part of the problem lies with the Western European welfare state system itself and cannot be blamed on the immigrants alone. Iranian Nima Sanandaji tells of his family's meeting with the Swedish system: "In Sweden my family encountered a political system that seemed very strange. The interpreter told us that Sweden is a country where the government will put a check into your mailbox each month if you don't work. She explained that there was no reason to get a job." "Although my mother got several jobs, we concluded that this really didn't improve our family's economy. During the sixteen years we have been in Sweden, my mother has in total worked less than one year."

However, part of the problem is also due to the mentality of some of the Muslims who move there, yet display no loyalty to their new countries. Immigrant men who divorce their wives according to secular law, but stay married to them according to sharia, Islamic law, represent an increasing problem in the city of Odense, Denmark, according to Erik Simonsen, deputy major in charge of administration. The result is a large number of "single" women who receive welfare support. From other countries, it is known that some Muslim men to do this trick with several women at the same time. Some observers blamed the Muslim riots in France in 2005, accurately described by writer Mark Steyn as the "first welfare funded Jihad in history," on polygamy practiced by Muslim men, paid for by French taxpayers. But also immigrants who are financially independent are cheating, says Simonsen. ඘ percent of the immigrant economy in Odense is a black market economy. That's a lot, and it cannot be tolerated, because the law is equal for all."

As one Muslim in Norway stated: "I worked in a Pakistani shop, but all of the work there is 'unofficial.' Neither the boss nor I pay taxes to Norwegian authorities. In addition to this, I receive 100% disability benefits and welfare. I have to be cunning to make as much money as possible, since this is my only objective with being in Norway." Undoubtedly, many Muslims view welfare money from the infidels as Jizya, the poll-tax non-Muslims according to the Koran are supposed to pay to Muslims as tribute and a sign of their inferior status and submission to Islamic rule. In Britain, one member of an Islamic group warned an undercover reporter against getting a job because it would be contributing to the kuffar (non-Muslim) system.


One of the reasons for the low participation in the work force among Muslims is a very high drop out rate from schools, especially for boys. Although Muslims themselves blame this on "poor integration efforts" and "marginalization," some of them are actively obstructing their kids' education, lest they become too much like the infidels. Thousands of Muslim children with Scandinavian citizenship are sent to Koran schools in Pakistan and other countries, to prevent them from becoming "too Westernized." When this practice was documented and criticized by the Norwegian organization Human Rights Service, Pakistanis in Norway had the galls to ask for a school for their children in Pakistan, funded by Norwegian taxpayers. They will probably get it.
According to the Copenhagen Post, in Denmark when the country's schools open their doors again after the summer holidays, not every student is there to enter. Some schools report one in every five students missing from the schoolrooms. Daily newspaper Berlingske Tidende reported that some children had not returned from vacation in their homelands with their parents. 'When students are kept away from school start, it's a big problem for their integration and can affect the children's educational level,' said Integration Minister Rikke Hvilshøj. She said a committee would discuss methods to get parents to send their children to school from day one. 'These are children who have problems beforehand following what's going on at school,' Hvilshøj said. 'Not being there when the school year starts won't make it any easier. We need to make parents understand that if their children are to get a good chance in Denmark, they need to make sure that they are there when school starts.' Author and journalist Rushy Rahsid said she had often gone on long vacations to Pakistan with her parents as a child. 'Families go on such long trips to give their children a healthy dose of religion, culture, traditions, and family,' she said. 'If you buy a very expensive ticket, you want to take full advantage of the trip and meet the entire family.'

Not that the problems always disappear when they actually attend school, either. Going to school is no child's game for many children in Copenhagen, where beatings, kicks, robberies, and threats have everyday occurrences. The perpetrators are normally their own age. Violence and threats pose such a big problem in high schools in Malmö, Sweden, that the local school board wants to install surveillance cameras and security guards in the buildings. The city's education director Matz Nilsson said unruly and aggressive students had become a more common sight in the high schools of Malmö, the home of some of Scandinavia's biggest and roughest slums.

In the spring of 2006, police were deployed at a Berlin school after teachers complained that they could not cope with their students' aggression and disrespect. A teacher who recently left the school told the Tagesspiegel newspaper that ethnic Arab pupils were bullying ethnic Turks, Germans and other nationalities. "School for them," said Petra Eggebrecht, former director of the Rütli school, "is simply a place to fight for peer recognition, where young criminals become idols." Young people are also easy targets for Islamist organisations. Outside the Rütli school, the children greeted visitors in Arabic.

When reporters went to school they were pelted with paving stones by masked youths from the schoolyard as the district's mayor stood helplessly at the entrance of the building. An increasing problem in German schools is that Arab male students often refuse to respect the authority of women teachers. Students at the Ruetli Hauptschule were not shy about expressing their views to reporters. "The German (students) brown nose us, pay for things for us and stuff like that, so that we don't smash in their faces." But there are also conflicts between Arab and Turkish students, mirrored in battles between the city's foreign-dominated youth gangs. Integration of foreign youths in Berlin is often poor. Even second and third generation children frequently do not speak fluent German and many fail to complete school - all of which leads to a high jobless rate among immigrant youths. White German families are moving out of districts like Neukoelln.

Muslim immigration has led to a spike in organized and violent crime, too. As early as 2002, Lars Hedegaard together with Dr. Daniel Pipes wrote about how Third-world immigrants - most of them Muslims from countries such as Turkey, Somalia, Pakistan, Lebanon and Iraq - constituted less than 5 percent of the population but consumed upwards of 40 percent of the welfare spending. What's worse, however, is that Muslims were only 4 percent of Denmark's 5.4 million people but made up a majority of the country's convicted rapists, an especially combustible issue given that practically all the female victims were non-Muslim. Similar, if lesser, disproportions were found in other crimes.

In 2005, it was reported that 82 percent of crimes in Copenhagen were committed by immigrants or descendants of immigrants, and the police pressed charges against second-generation immigrants five times as frequently per capita as against ethnic Danes. Doormen working in the Danish capital are now often armed with guns or clubs so as to be able to defend themselves against violent immigrant gangs. There is more violence than ever before and some immigrant gangs have even been known to seek out doormen at their private homes. Immigrant criminal gangs are much more active than previously believed, and their brutality has them surpassing biker crime gangs like the Hell's Angels or Bandidos, the State Police said. A Norwegian researcher warned that ethnic gangs could give Norway the kind of immigrant-related organized crime that had previously been unknown in Scandinavia. Arne Johannessen from the Norwegian Police Organization warned in 2003 that the cost of crime may have been doubled during the previous decade, in part due to immigration.

In Sweden, there has been a steep rise in all kinds of crime and violence at the same time as an unprecedented amount of immigration to the country. Of course, Swedish politicians would never dream of connecting the two. However, in one of those rare cases where a Swedish newspaper has actually told the truth, Aftonbladet revealed that 9 out of the 10 most criminal ethnic groups in Sweden were Muslims, a trend known from other European countries such as France, where Muslims make up 10% of the general population, but maybe 70% of the prison population. The number of people under the age of 18 who are serving sentences in juvenile detention centres in Sweden has risen sharply over the last five years.

Not that Western European prisons will scare immigrant children too much. The relaxed regime at a prison housing some of Holland's most dangerous criminals has caused controversy. Pictures of tattooed prisoners enjoying a "social evening" and beer embarrassed the authorities which are sensitive to claims abroad that they are soft on crime. One inmate at Esserheem serving 14 years for murder said: "We have a party every weekend in our own prison bar. We can enjoy wonderful snacks, ice cream, beers, play cards or billiards and listen to music. If the weather is warm we can play tennis." Another prisoner spoke of "perfect" fresh food and said: "We can buy everything in the prison shop including sushi and sake." In the Netherlands, the economic argument deployed by both leftwing Multiculturalists and free-market conservatives - that immigration revives aging populations, provides new labor resources, and generates entrepreneurial activity – "simply does not apply." There has been no overall economic benefit to population change since unskilled guest workers were invited to the Netherlands in the early 1970s. According to Paul Scheffer, a leading critic of Multiculturalism and professor of urban sociology at Amsterdam University, up to 60% of first-generation Turkish and Moroccan populations are unemployed.

Andrew Bostom, editor of The Legacy of Jihad, quotes the observations made by Dr. Muqtedar Khan, a much-ballyhooed Muslim moderate from the U.S., after a trip to Belgium. Even Khan admitted the largesse of Belgium's welfare state towards its Muslims: "…the welfare check was normally 70 percent to 80 percent of the salary. For those [Muslims] who were married with children, welfare provided comfortable living and with low property values, even those on welfare could actually own homes." American expatriate writer Bruce Bawer, living in Norway, has documented in his book While Europe Slept how Oslo imams preach brazenly that Muslims should expect these welfare benefits—and feel justified in supplementing them by stealing from stores—as a form of jizya extracted from their infidel "host" societies. As Bostom points, out, "such attitudes, whatever their origins, raise this larger basic question: why does the West continue to validate the raw, imbalanced bigotry that denies non-Muslims any access to Mecca and Medina—upon pain of imprisonment, torture, and death—while Muslims demand and are granted the ability to settle, with generous accommodations, within Europe or America?"

Indeed, some of the Muslims are actively involved in supporting Jihad activities, verbally or physically. Mullah Krekar is convicted of terrorism in Jordan, he has been suspected of links to the Madrid terrorists, and the CIA claims he has been ordering bombings in Iraq. He has lived off Norwegian welfare money in Oslo for years, and reputedly gets a lot of free taxi rides from Muslim cab drivers who think he's a great guy. Norway's most controversial refugee has lodged a threat against the country that has hosted him and his family for the past 14 years. Mullah Krekar continues to fight deportation back to Iraq, and calls any such order "an offense" that should be punished. "I defend my rights in their court just like Western people defend their rights. I am patient like they are patient. But if my patience runs out, I will react like Orientals do." Asked how "Orientals" react, Krekar said: "I don't want to comment on that." Krekar has also spoken positively about terrorist leader Osama bin Laden and the recently killed Iraqi al-Qaida leader Abu Musab al-Zarqawi.

In Britain, the police have been investigating allegations that the four suspected London bombers from July 2005 collected more than £500,000 in benefits payments. The suspects are supposed to have used multiple aliases and addresses. One, Mr Ibrahim, is said to have had six aliases. Some are also shown to have claimed several nationalities, ages and national insurance numbers while in Britain. Investigators believe that bogus names were used to make some benefit claims. Two are also alleged to have obtained asylum using bogus passports and false names and nationalities. Mr Ibrahim, alleged to be the Hackney bus bomber, is believed to have used two dates of birth, six aliases, two national insurance numbers and two addresses. Mr Osman apparently went under five names, variously claimed that he was Eritrean or Somali, and used four addresses in southwest London. Mr Omar, 24, who is linked with the attack on a Tube train near Warren
Street, had five aliases.

The London Times quoted a report that expressed fears that Britain was "sleepwalking its way to apartheid" due to increased ethnic segregation. The paper, which took into account the background of the four July 7 suicide bombers who attacked London, pointed to social and economic deprivation as key drivers in their conversion to terrorism, ignoring the numerous calls for violent Jihad in the Koran and the hadith. It found that the Muslim community was the most "disadvantaged" faith group in the country, with lower educational attainment and higher unemployment. The document found that the unemployment rate among Britain's 1.6m Muslims was more than three times that of the general population and was the highest of all faith groups. About half of all Muslims were economically inactive (52%). That was higher than any other religious group.

DP111, an articulate British commentator at such websites as Jihad Watch, Little Green Footballs and Fjordman blog, points out that as Muslim families are very large, a single wage earner will find it hard to support all. They will need to supplement this by getting considerable benefits from the state. Thus at a minimum, "some 80% of the Muslim population" subsists on welfare benefits: "A considerable amount of this money to Muslims from the British taxpayer, finds its way to finance the Jihad. The same scenario must hold in all European states that have a considerable Muslim population." "We are in the ridiculous position of sheltering and feeding a population that is hell bent on destroying us. Islam's people, from the very outset, were nothing but a collective to gain plunder and loot at the expense of other people's work. In the past, it was conducted by war, conquest and then pillage. Now it is conducted by immigration (invasion), begging or crime. Meanwhile Muslim nations are given huge loans (aid), which we and they know will never be repaid. Thus, from a purely economic point of view, Islam seems to be a collective of people who live by the ethos of "beg, borrow or steal." So why do we, the capitalist countries, who do not believe in offering anyone a free lunch, subsidise the most lazy yet aggressive bunch of people on God's planet, who are bent on subverting our democratic system? The nub is, how has it come about, that the natural progression of the most advanced civilisation on earth is towards stupidity?"

He also points out that the cost of Muslim immigration is much higher than just welfare. "One really needs to factor the loss of confidence in the markets, loss of new investment because of a fear of flying or the disenchantment with intrusive security. Then there is the increased cost of insurance on all businesses." All this affects the competitivess of business with nations that do not have a significant Islamic presence, such as Japan, Korea and Taiwan. We may be talking of million dollars or more per Muslim individual per year. "And to top it all, we give them huge amounts of aid, welfare benefits on a grand
scale, while we live in fear, and our freedoms compromised."

Former Muslim Ali Sina claims that even in the USA, which has a smaller Muslim population and less social security benefits than Western Europe, Muslims are a huge drain on the economy: "Because about 2 million Muslims live in America and among them there are some who are terrorists, Americans are forced to expend hundreds of billions of dollars on homeland security. I have no idea how much is the actual cost. Let us be conservative and say it costs only $200 billion dollars per year. I reality it could be many times more. Does anyone have any idea? With just $200 billion dollars, every Muslim, including their children cost the taxpayers $100,000 dollars per year. This is the real "contribution" of Muslims to America Mr. President. Once you add the cost of the real damage caused in terrorist attacks, such as to planes, buildings, etc. this cost will be much higher. And here we are only talking about monetary cost. Can anyone calculate the cost that the families of the victims of terrorism pay? What is true in America is true in anywhere governments are forced to expend money to increase the security."

Bat Ye'or in her book Eurabia - The Euro-Arab Axis describes how we are in the middle of a total transformation of Europe, which is the result of an intentional policy. In an interview with Israeli newspaper Haaretz, Bat Ye'or fears that Europe will become more and more Islamized until it becomes "a political satellite of the Arab and Muslim world." "The European leaders have decided on an alliance with the Arab world, through which they have committed to accept the Arab and Muslim approach toward the United States and Israel. This is not only with respect to foreign policy, but also on issues engaging European society from within, such as immigration, the integration of the immigrants and the idea that Islam is part of Europe."

Former French PM Charles de Gaulle, disappointed by the loss of the French colonies in Africa and the Middle East, as well as with France's waning influence in the international arena, in the 1960's decided to create a strategic alliance with the Arab and Muslim world to compete with the dominance of the United States and the Soviet Union. This alliance became the position of the European Community (pre-European Union) during the course of the 1970s, when an extensive European-Arab dialogue developed. However, this alliance, in the context of which Europe allowed the immigration of millions of Muslims to its territories and adopted an anti-Israeli and anti-American policy, will ultimately "transform Europe into a continent under the thumb of the Arab and Muslim world."

As Bat Ye'or points out, most of the workings of Eurabia are hidden from the public view, but sometimes we can catch glimpses of it if we know what to look for. Italian Leftist PM Romano Prodi, President of the European Commission, the Politburo of the EUSSR, from 1999 to 2004, wants more cooperation with Arab countries. He talks about a free trade zone with the Arab world, but this implies the four freedoms of the EU, which includes the free movement of people. This is made clear in a document from 2003. This fact, and the implications of it, is virtually never mentioned in European media. During the Sixth Euro-Med Ministerial Conference in Brussels, 2003, it is stated that this initiative offers the EU's neighboring partners "gradual integration into the expanded European internal market and the possibility of ultimately reaching the EU's four fundamental freedoms: free movement of goods, services, capital and people." Ministers were also expected to back the Commission's proposal to set up a Euro-Mediterranean Foundation for the Dialogue of Cultures, a Euro-Mediterranean Parliamentary Assembly and a reinforced European Investment Bank (EIB).

In June 2006, after his Leftist coalition government signalized that they would be much more lenient with Muslim immigration to Italy, Romano Prodi announced that "It's time to look south and relaunch a new policy of cooperation for the Mediterranean." He was thinking of the Euro-Mediterranean partnership, launched in 1994 in Barcelona. Prodi said he had raised the issue with Egyptian president Hosni Mubarak, and Libyan leader Muammar Gaddafi. The prime minister then explained that the Barcelona Process – which includes the creation of a free trade zone by 2010 - was no longer sufficient and a new different approach was needed. "The countries on the southern shores of the Mediterranean expect that from us" he added.

Muslim immigration is costing enormous sums and creating havoc across Western Europe, and EU leaders are working to give Muslims easier access to Europe – because Arabs expect this from them. Thank you for pointing this out, Mr. Prodi. And thank you for removing all doubts whether the European Union needs to be dismantled, and its treacherous and corrupt political elites removed from power.

>Fjordman is a noted blogger who wrote for the Fjordman Blog in the past. He has also been published on many other websites, including Gates of Vienna, which is the publication where this article originally appeared.

Itsdb rated this answer Excellent or Above Average Answer

Question/Answer
tropicalstorm asked on 05/10/07 - Howard Dean on the hurricane

Listen to
Jim Quinn of www.warroom.com
on XM satelite radio channel 165 6 a.m. to 9 a.m.

For follow ups on....

He says reliable source says

The Kansas Governor told Brownback that she
'could not allow an opportunity pass. I made sure not to blame anybody outside the whitehouse. With Bush's numbers you can't really blame me for that'.

Brownback replied he was 'disappointed in her'.

She said, 'you know me I wouldn't say that if I didn't have to but Howard Dean told me [to].' She tried explaining it away saying 'well it could happen' [that way someday]. 'I didn't think it was right to use it either but in this climate I had to' [to not disappoint Dean]

She also said they have more than enough National Guard and humvees.

QUINN SAID HIS SOURCE SAID 'HOWARD DEAN CALLED HER AT
5 AM AND SAID HE WOULD GET BACK TO HER WITH THE DETAILS' [of what he wanted her to say].
DICK DURBIN CALLED HER BACK WITH THE DETAILS.
Layhe is somehow involved too.

She (the governor) called Brownback's office and the call was forwarded to him personally on his cell phone.


tomder55 answered on 05/11/07:

you mean Howard Dean on the Tornado.

The report is now being linked by many blogs. Supposedly she went on a radio station and admitted the story is true. I would love to hear the audio.

This is nothing ....wait until they figure out a way to link tornados with global warming .An arsonist goes on a rampage in California and it's global warming . But at least the Governator ,who has disaster in a much larger geographic area to control is not belly aching about the lack of resources.....yet.

tropicalstorm rated this answer Excellent or Above Average Answer

Question/Answer
Itsdb asked on 05/07/07 - The cost of CFL's

How much money does it take to screw in a compact fluorescent lightbulb? About $4.28 for the bulb and labor — unless you break the bulb. Then you, like Brandy Bridges of Ellsworth, Maine, could be looking at a cost of about $2,004.28, which doesn’t include the costs of frayed nerves and risks to health.

Sound crazy? Perhaps no more than the stampede to ban the incandescent light bulb in favor of compact fluorescent lightbulbs (CFLs) — a move already either adopted or being considered in California, Canada, the European Union and Australia.

According to an April 12 article in The Ellsworth American, Bridges had the misfortune of breaking a CFL during installation in her daughter’s bedroom: It dropped and shattered on the carpeted floor.

Aware that CFLs contain potentially hazardous substances, Bridges called her local Home Depot for advice. The store told her that the CFL contained mercury and that she should call the Poison Control hotline, which in turn directed her to the Maine Department of Environmental Protection.

The DEP sent a specialist to Bridges’ house to test for mercury contamination. The specialist found mercury levels in the bedroom in excess of six times the state’s “safe” level for mercury contamination of 300 billionths of a gram per cubic meter.

The DEP specialist recommended that Bridges call an environmental cleanup firm, which reportedly gave her a “low-ball” estimate of $2,000 to clean up the room. The room then was sealed off with plastic and Bridges began “gathering finances” to pay for the $2,000 cleaning. Reportedly, her insurance company wouldn’t cover the cleanup costs because mercury is a pollutant.

Given that the replacement of incandescent bulbs with CFLs in the average U.S. household is touted as saving as much as $180 annually in energy costs — and assuming that Bridges doesn’t break any more CFLs — it will take her more than 11 years to recoup the cleanup costs in the form of energy savings.

Even if you don’t go for the full-scale panic of the $2,000 cleanup, the do-it-yourself approach is still somewhat intense, if not downright alarming.

Consider the procedure offered by the Maine DEP’s Web page entitled, “What if I accidentally break a fluorescent bulb in my home?”

Don’t vacuum bulb debris because a standard vacuum will spread mercury-containing dust throughout the area and contaminate the vacuum. Ventilate the area and reduce the temperature. Wear protective equipment like goggles, coveralls and a dust mask.

Collect the waste material into an airtight container. Pat the area with the sticky side of tape. Wipe with a damp cloth. Finally, check with local authorities to see where hazardous waste may be properly disposed.

The only step the Maine DEP left off was the final one: Hope that you did a good enough cleanup so that you, your family and pets aren’t poisoned by any mercury inadvertently dispersed or missed.

This, of course, assumes that people are even aware that breaking CFLs entails special cleanup procedures.

The potentially hazardous CFL is being pushed by companies such as Wal-Mart, which wants to sell 100 million CFLs at five times the cost of incandescent bulbs during 2007, and, surprisingly, environmentalists.

It’s quite odd that environmentalists have embraced the CFL, which cannot now and will not in the foreseeable future be made without mercury. Given that there are about 4 billion lightbulb sockets in American households, we’re looking at the possibility of creating billions of hazardous waste sites such as the Bridges’ bedroom.

Usually, environmentalists want hazardous materials out of, not in, our homes.

These are the same people who go berserk at the thought of mercury being emitted from power plants and the presence of mercury in seafood. Environmentalists have whipped up so much fear of mercury among the public that many local governments have even launched mercury thermometer exchange programs.

As the activist group Environmental Defense urges us to buy CFLs, it defines mercury on a separate part of its Web site as a “highly toxic heavy metal that can cause brain damage and learning disabilities in fetuses and children” and as “one of the most poisonous forms of pollution.”

Greenpeace also recommends CFLs while simultaneously bemoaning contamination caused by a mercury thermometer factory in India. But where are mercury-containing CFLs made? Not in the U.S., under strict environmental regulation. CFLs are made in India and China, where environmental standards are virtually non-existent.

And let’s not forget about the regulatory nightmare known as the Superfund law, the EPA regulatory program best known for requiring expensive but often needless cleanup of toxic waste sites, along with endless litigation over such cleanups.

We’ll eventually be disposing billions and billions of CFL mercury bombs. Much of the mercury from discarded and/or broken CFLs is bound to make its way into the environment and give rise to Superfund liability, which in the past has needlessly disrupted many lives, cost tens of billions of dollars and sent many businesses into bankruptcy.

As each CFL contains 5 milligrams of mercury, at the Maine “safety” standard of 300 nanograms per cubic meter, it would take 16,667 cubic meters of soil to “safely” contain all the mercury in a single CFL. While CFL vendors and environmentalists tout the energy cost savings of CFLs, they conveniently omit the personal and societal costs of CFL disposal.

Not only are CFLs much more expensive than incandescent bulbs and emit light that many regard as inferior to incandescent bulbs, they pose a nightmare if they break and require special disposal procedures. Should government (egged on by environmentalists and the Wal-Marts of the world) impose on us such higher costs, denial of lighting choice, disposal hassles and breakage risks in the name of saving a few dollars every year on the electric bill?

~~~~~~~~~~~~~~~~~~~~~~~~~~~~~~~~~~~~~~~~~~~~~~~~~~~~~~

I have some Tyvek suits, goggles, gloves and respirators cheap. Will they now be selling these alongside the CFL's in Home Depot? I wonder how many have already been dumped in landfills?

tomder55 answered on 05/08/07:

Once they perfect LED technology this will all be mute .They can be made so they don't break. CFLs are a stop gap temporary answer.
Let's not count out the incadecent bulb yet.
G.E. is working on the next generation of those also . They are not going to surrender the market without a fight.

Elliot is right . There is less mercury in a CFL then in your wrist watch. But there should indeed be provisions for proper disposal .I think in the rush to ban Edison's bulb the gvts .are forgetting this basic fact.

ETWolverine rated this answer Excellent or Above Average Answer
Itsdb rated this answer Excellent or Above Average Answer

Question/Answer
Itsdb asked on 05/07/07 - Breaking news?

Surely you've heard about the 'scandal' that is Fred Thompson playing a racist role on TV 19 years ago. Huffpo even had this 'scandal' link on their website as "breaking" news (thank you Google cached search) with minx' headline of "Fred Thompson's Campaign Ends In Racist Fireball: LAT Discovers Videotape of Him Using Anti-Semitic Smears, "Fondling" Mein Kampf.

How pathetic.

tomder55 answered on 05/08/07:

yeah and Alan Alda a right winger because he played a conservative Presidential candidate in 'The West Wing '. And Michael Douglas is a rabid unscrupled capitalist because he played the part in 'Wall Street' .

This is really lame stuff.

Sean Penn is an unstable gungo-ho militarist because he played the part in 'Taps' . And Carrol O'Conner is a biggoted conservative because it became his signature role in 'All in the Family '. and best of all ....Alec Baldwin is an American patriot hero because he played Lt. Col.James Doolittle in 'Pearl Harbor'.lololololol

Itsdb rated this answer Excellent or Above Average Answer

Question/Answer
ETWolverine asked on 05/07/07 - A Change in France? For the better?

A FRENCH RUDY
By FRED SIEGEL

May 7, 2007 -- IF Nicholas Sarkozy, the new president-elect of France, reminds you of Rudy Giuliani, there's a reason. Sarkozy really is a kind of French Giuliani. More, he's clearly learned some things from Rudy (the two have met) - and his success may have lessons for Giuliani in his bid to become U.S. president.

Most accounts simply note that Sarkozy is remarkably pro-American for a French politician - describing America as "the greatest democracy in the world."

But when you get into the details of his admiration, you start hearing echoes of Rudy:

* "France," Sarkozy quips, "is like the Anglo-Saxon countries when it comes to inequality and poverty - but without their social mobility and full employment."

* He also mocks French anti-Americans as people who "envy" America's "brilliant success."

* Speaking to 300,000 French citizens who've fled the stagnation of their homeland for jobs in London, Sarkozy committed blasphemy by the normal rules of French politics: He praised England's less regulated and more dynamic economy as a model for France.

Yes, the French elected a man who promises a "rupture with the past" - for the same reason New Yorkers reluctantly elected Giuliani in 1993: because conditions were bad enough to risk change.

Some 70 percent of the French think their deeply indebted and grossly over-taxed country is in perilous decline; books on the country's bleak future have become a small publishing industry in Paris. Like Giuliani in ྙ, Sarkozy bluntly presents himself as a turnaround artist who can redeem the promise of lost greatness by challenging the conventional political assumptions of the permanent government of civil servants, political insiders and over-mighty interest groups who all feed off of a bloated state.

Both men are hard-edged originals with bruising political styles, energetic and inner-directed - outsiders to their political establishments who attract both a devoted following and bitter hostility.

Above all else, each has a hard-to-categorize politics - one that capitalizes on popular resentment of insulated elites clinging to the outdated ideologies of the 1960s.

Giuliani as mayor mocked the "compassionate" liberalism that left masses of people trapped in welfare while providing guaranteed jobs and votes for Gotham's Democrats. Sarkozy similarly mocks the "egalitarianism" of the French civil service - who have near-total job security and fat pensions, even as their management has left French unemployment running double the American rate for 30 years.

The two men met in 2002, when Giuliani had been invited to France to provide advice on how to combat the rising crime rate and Sarkozy was serving as Interior minister. The Frenchman talked to the American about "broken windows" policing and New York's famed COMSTAT program, which provided a meaningful metric for policing. More recently, Sarkozy has been talking up New York-style welfare reform - requiring the able-bodied to take available jobs.

Just as Giuliani wanted to make New York, with its Francified bureaucracies, more like the rest of America, Sarkozy wants to make France more like the more market-oriented Anglo-American economies. Both are critics of multiculturalism - and neither accepts that crime or terrorism can be explained by social causes.

Each talks in a language foreign to the elites - emphazing personal responsibility and the importance of the work ethic. In his recent book, "Témoignage" ["Testimony"], Sarkozy takes aim at those on the French left who depict the rioting Muslim youth of the banlieues as victims of police brutality and French racism. In a riff that's nearly pure Giuliani, he points to the massive social spending in the banlieue - and notes that it seems to have sown far more resentment than good will. Rudy-like, he argues that the young rioters have to adjust to France - rather than the other way around.

The similarities go beyond policy and persona. Sarkozy ran not only against Socialist candidate Segolene Royal, but against criticism of his own aggressive political style. Facing Royal in a crucial debate just days before the election, he managed to constrain his combatative personality lest he be seen as too aggressive. He passed the test, can Giuliani?

Sarkozy also had to overcome the unpopularity of sitting President Jacques Chirac, a member of his own party. Placed on the defensive by the failures of his fellow Gaullist, Sarkozy carefully but convincingly called for reversing the economic policies associated with the incumbent - without mentioning Chirac by name.

Giuliani, who has offered himself up as the salvation of a sinking Republican Party, should be watching closely. If he wins the Republican nomination, he'll similarly have to thread the needle of distancing himself from President Bush's foreign-policy failings without too directly criticizing the president.

The French Rudy pulled it off. Will the American Sarkozy manage it, too?

Fred Siegel is a senior fellow at the Manhattan Institute; his books include "The Prince of the City: Giuliani, New York and the Genius of American Life."

------------------------

Does this bode well for the future relationship between the USA and France, and regarding France's place in the GWOT? I would certainly like to think so.

And can Giuliani learn the lessons of restraint or rhetoric that Sarkozy managed to learn? Sarkozy has learned from Rudy. Can Rudy learn from Sarkozy?

What's your opinion?

Elliot

tomder55 answered on 05/08/07:

I said it on another reply .... With Sarkozy in France and Angela Merkel in Germany ;if Hillary should win the US elections there will be more pro-American leaders in France and Germany than in the US.

Ya gotta love a guy who says his administration would put an end to the influence of France's May 1968 generation, which he said was responsible for the decline of morality and authority in France.

Royal said there would be violence if he won . Then she hired a couple hundred goons to throw rocks near the Bastille. The violence she predicted was comparable to that which France routinely gets on Bastille Day celebrations and just as juvenile.



I hope this is a new age in the American French relationships. Perhaps this spells and end to Gaulism.

Rudy Giuliani cleaned up NYC by making sure "minor" and major infractions of the law didn’t go unpunished.By doing so he transformed the city to the best in the country .Sarkozy likewise has taken a no-nonsense approach to law and order.

I would not be opposed to a Rudy nomination and do in fact favor him over the rest of existing field. But there is one issue he could definitely learn from Sarkozy.

Sarkozy breached politically correct taboos on immigration and national identity and still won. That is one lesson Rudy could learn . He still tends to pander to illegals. Remember ,he was one of the first mayors to invoke a sanctuary policy ....in fact he sued the Federal Gvt. over the issue. Perhaps he has modified his views since 9-11 ,but if so he has not said so publically in a forceful way. Of all the socially "moderate "positions that Rudy takes,that is the most disturbing one to me. How can he be a such a tough law and order guy and still support the sanctuary concept like Gavin Newsome ?




ETWolverine rated this answer Excellent or Above Average Answer
Itsdb rated this answer Excellent or Above Average Answer

Question/Answer
Itsdb asked on 05/07/07 - Is a wake-up call needed?

Steve and Cokie Roberts (whose current columns I can never seem to find online) wrote a column on why Bush the elder was right on Iraq and Bush the younger was wrong, and discuss the reasons for American dissatisfaction with the war in Iraq:

    Vietnam (and Korea) both reflected a broad national consensus that international communism directly threatened American interests and had to be contained. The rationales for invading Iraq - finding weapons of mass destruction, thwarting terrorists, creating democracy - had some initial appeal in the aftermath of Sept. 11. Those arguments never had the resonance of anticommunism, and support for the war effort plummeted when many of Bush's justifications proved false.

    "This lower tolerance for casualties" in Iraq," wrote (Ohio State University Professor John) Mueller, "is largely due to the fact that the American public places far less value on the stakes in Iraq than it did on those in Korea and Vietnam."


My first question is why? Why doesn't the prospect of Islamic terrorism resonate with the American people? It certainly did on 9-11, why did that change - (and I don't buy the "justifications proved false" line as the primary reason)?

Steve

tomder55 answered on 05/07/07:

Steve ,I think that the wrong lessons were learned from Vietnam. Remember the Powell Doctrine ? 58,000 dead must never happen again . Get in fast with overwhelming force....achieve a limited objective ...and leave fast .Your exit strategy is a primary concern almost to a point that your raison de guerre is secondary .That was demonstrated in Desert Storm. Clinton further refined it by the false lesson of the Kosovo campaign ;that you can fight a casualty free ,relatively painless war.

The Vietnam analogy is almost an invitation to 'realist'isolationism (which the Roberts appear to be endorsing ). Those of us who supported the war ,and still do are more inclined to invoke a Munich analogy . We realize that Saddam had to be stopped in 1990 or he could've moved on to Saudi Arabi. We knew after 9-11 that the status quo in Iraq was deteriorating and we most likely could not contain Saddam much longer . He was a threat because of his potential for mischief . After 9-11 that threat had to be taken just a little more seriously .

Yes we could've taken him out and left but that would have been that post-Vietnam /limited objective thinking . Believe me ;the people arguing for intervention in Sudan are not thinking of going in/kicking butt /and leaving . They are looking to transform that society .

Likewise , leaving Iraq without transforing the country would've just handed the country off to the next threat .That is why we stayed in Europe all those years . That is why we stayed in Japan ;and South Korea and continue to stay in the Balkans. I ask the Roberts ;was it really 'realist' to stand toe to toe with the Soviets ,and come almost to the brink of nuclear exchange more than once,over German sovereignty ? Gimme a break! We did it because we remembered the lessons of Munich .

Itsdb rated this answer Excellent or Above Average Answer

Question/Answer
tropicalstorm asked on 05/04/07 - HIllary sHillary

she accents a southern gospel persona
now she pitches her voice to sound like a farmer claiming she grow up on the farm working along side with illegal migrant Mexican workers babysitting their children
yet here is her biography
does it all add up or not?

Hillary Diane Rodham was born at Edgewater Hospital in Chicago, Illinois, and was raised in a Methodist family in Park Ridge, Illinois. Her father, Hugh Ellsworth Rodham, was a son of English immigrants and operated a small business in the textile industry. Her mother, Dorothy Emma Howell Rodham, was a homemaker. She has two younger brothers, Hugh and Tony.

As a child, Hillary Rodham was involved in many activities at church and at her public school in Park Ridge. She participated in a variety of sports and earned awards as a Brownie and Girl Scout.[2] She attended Maine East High School, where she had participated in student council, the debating team and the National Honor Society

A curmudgeon was the way one Hillary biographer, Norman King described Hugh while another, Roger Morris finds him guilty of the "psychological abuse of his children. Chief Petty Officer Hugh Rodham was a drill instructor who trained recruits in the Navy during World War II. Afterward he became a successful businessman in Chicago who moved his family to Park Ridge, an upper middle class suburb from a city apartment three years after Hillary was born. He was a regal presence in this family; Hillary says it was like the television sitcom, Father Know Best. But the humor was lacking according to Dorothy who said of Hillary, "She had to put up with him." Of course, Dorothy did too.
Hillary was Daddy Hugh's girl but what does this mean?
She was Hugh Rodham's victim who wanted his love and approval even as she tried to escape his stinginess, irascibility and perfectionism. The victim survived and was marked by an identification with the aggressor. Like Hugh the adult Hillary became irritable, demanding and the family breadwinner but that's getting ahead of her story. When she brought home a report card with all A's, Daddy replied that it must be an awfully easy school. We're not told what Dorothy Rodham said when she saw the grades maybe because this wasn't important or perhaps Mother Dorothy was also hard to please. It was Dorothy who said there was no room in the house for cowards when little Hillary ran home after an attack by an "obnoxious girl." Forced to confront her attacker, she won the battle and now had the respect of the neighborhood players, says biographer David Brock.


While Hillary's childhood is usually described as solidly middle class, Oppenheimer offers a grim portrait. Hugh Rodham may have driven a Cadillac and owned a home in a white-bread Chicago suburb, he writes, but he was a cheapskate who refused to take care of the place, and his drapery business was a one-man shop with walls stained brown from chewing-tobacco juice. Hillary has her brother Tony to thank for many of these details, since Tony told Oppenheimer about a cousin, Oscar Dowdy, who became the source for them. Dowdy also says that Hillary's mother was given to making anti-Jewish slurs (some about Hillary's grandmother's second husband Max Rosenberg).





tomder55 answered on 05/04/07:

The New Republic has this rather bizzare and not believable story that Hillary tried to enlist in the Marines when she was a law professor in Arkansas. According to the tale they spin ;the Marines turned her down because she was too old and wore coke bottle thick glasses.



Their point was that this is why Hillary has been so hawkish (snicker) . Well she may have been hawkish .....by all reports she can toss an end table lamp pretty far...but that was before the Kossacks and the defeatocrats got to her. In her rush to the left to appease the loons at the base of her party she has gone from a hawk to una paloma amarillo .

tropicalstorm rated this answer Excellent or Above Average Answer
Itsdb rated this answer Excellent or Above Average Answer

Question/Answer
Itsdb asked on 05/03/07 - Notes from the convention

The California Democratic Party Convention was just held in San Diego. Here are some items of interest thanks to the staff at the San Diego Union-Tribune...

From the Convention report card:

    Candidate: Mike Gravel
    Appropriate career: Shock jock.
    Convention presence: None.
    Followers: None.
    Speech: Nothing to main convention—spoke during Friday reception while delegates grappled over the free crab cakes and hummus.
    X-factor: Pronounces name with emphasis on last syllable. Sounds French to us. Luckily, support cannot get below zero.
    Grade: A. May have enhanced name recognition.

    Candidate: Chris Dodd
    Appropriate career: Senator. Dude really looks like a senator.
    Convention presence: Not much. Media showed scant interest.
    Followers: One. And we think he's on the payroll.
    Speech: Theme of "When I was young" more likely to generate guilty calls to grandparents than votes.
    X-factor: Never trust a candidate when you can't see his eyes for his eyebrows.
    Grade: C. Why is he running?

    Candidate: Dennis Kucinich
    Appropriate career: Cult leader.
    Convention presence: Ubiquitous. Kucinich and his foxy red-haired Viking of a wife appeared at several of the caucuses and spent all Saturday in San Diego. He didn't spend money on much besides signs. Aging hippies promoted him for free.
    Followers: Somewhat smelly.
    Speech: For Kucinich, speeches are performance art, a full-body dance in which he and his audience rise and fall together. For a few minutes, he owned the room, but then he stopped and the Dems returned to the fluorescent light of reality.
    X-factor: After listening to Kucinich speak for 10 minutes, listeners lose the power of reason and logic. They begin to sway back and forth, muttering the words "Yes, Master."
    Grade: A. The right fit for liberal Californians.


Sensory overload, Seen and heard at the convention

    • Quote of the weekend: "Let's all get up and exercise out lower extremities."—State chairman Art Torres.

    • The Kucinich hootenanny wrapped up in just an hour on Friday night. We've been to some hootenannies, we've worked at hootenannies, and, Mr. Kucinich, that was no hootenanny.

    • The Guerilla Media workshop on Friday lasted half an hour and consisted of Dems complaining that Harry Reid had been misquoted when he said we lost the war. Good thing they had a workshop.

    • The Gore-Obama ticket pushed by some dreamers on Friday sounds good to us, but who's the top and who's the bottom?

    • City Councilmember Toni Atkins was a delegate this year. She attended a "small breakfast in La Jolla"with House Speaker Nancy Pelosi. Atkins said Pelosi told a story about admonishing the president to his face for suggesting "that the first woman Speaker of the House of Representatives ‘just stay at home' in his comments about her trip to Syria.”

    • The Progressive Democrats of America offered impeach-mints to passers-by.

    • Does Dennis Kucinich remind anyone else of Mr. Sulu?

    • The Edwards campaign gets the award for the most arrogant theme song. After his speech, Edwards was played off with Foo Fighters singing, "There goes my hero. / Watch him as he goes.”

    • The Lyndon LaRouchians were out in force, but mainly they demonstrated why we need to put more money into our schools. One told CityBeat that "Abe Lincoln kicked the British ass in the civil War."Another argued that folk music is based on rock 'n' roll.

    • Gov. Bill Richardson, in his press conference, made the gaffe of the weekend, one-upping the LaRouchians by telling reporters that Byron White's U.S. Supreme Court stint didn't last past the ླྀs. White retired in 1993.

    • Democrats have bad hair, but the balding guy with the mullet really took the cake. The business in front closed, and the party in back should have ended long ago.

    • The convention crowd on Saturday wouldn't let Congresswoman Maxine Waters leave after her speech without coming back for an encore. She returned to the podium and shouted, "Not another nickel. Not another dime. Not another soldier. Not this time!”

    • There's never enough free food, but the shortage of free forks at the Friday reception was baffling. Luckily, neither Dems nor CityBeat reporters are above eating hummus with their fingers.

    • This guy at the Progressive Caucus held above his head a sign that read "What is depleted uranium?"for, like, an hour. Talk about a feat of stamina. We were impressed.


Raucous caucus

    • The first motion from the Children's Caucus was to have refreshments at the next meeting. The second was to have soft drinks.

    • The Environmental Caucus turned the lights off for a minute to save energy. Of course, the meeting had to be a minute longer, to finish its work. Also, if you want to get in with that environmentalist hottie you've been eyeing, bring up incandescent light bulbs. It seems to get them in a lather.

    • There was a three-way tie for most lively caucus: Progressives, the LGBT and the old folks in the Senior Caucus. Of course, they may have been loud just so they could hear each other.

    • Only one standing officer survived from the last meeting of the Senior Caucus to the current one.

    • The Progressives are the biggest, and their ranks may have been the most rank, which, trust us, is saying something. Our reporter had to change places several times to find a less aromatic seat.

    • The Business and Professional Caucus had the fewest members. Sure, they had a professional-looking easel with a giant pad, but that does not seem to have lured people from their actual paying jobs.

    • Everybody wants to get with the Labor Caucus. They have rules limiting who can speak, including a requirement that you be a member of the caucus. But they spent so much time recognizing officials in the room who were running for office that they didn't actually get anything done.

    • When members of the Chicano-Latino Caucus wanted to vote yea for a measure, they held up their membership cards, which were green. So, the chair would say, "Those in favor hold your green cards up!"which was awesome.


In pictures









Does that outfit remind you of anything? Maybe Chief George Earl in Demolition Man?



Maybe Hillary can tell us how to use those $#!! three seashells.

tomder55 answered on 05/04/07:

The seminar to attend was conducted by John Edwards who elaborated on his two Americas theme . The 2 Americas are those who pay $400 for a haircut and those who pay $20 .



Speaking of Kucinich's wife :



Yeah he looks a little like Sulu but more like Ed Grimley

Itsdb rated this answer Excellent or Above Average Answer

Question/Answer
kindj asked on 05/03/07 - An add-on to TS's PC post....

Please don't misunderstand the meaning of my story. I love the school district where I work, and I love who I work for and with. I'm not exaggerating when I say it's the best place I've ever worked--bar none.

Earlier this year, we were figuring out how to spend our department budget. We'd spent quite a lot of our allocation on needed supples and stuff, and were really doing quite fine. However, as you know, these budgets are funny--spend every dime this year, or you don't get as much next year. Seems irresponsible to me, but who am I......

In any event, one of the things that my dept. head was going to order were these metal rods that you use to pull down the screen for overheads and stuff. Well, for four of them, they were going to charge us like sixty dollars. I said, "BS! I'll go to Home Depot and make us some good wooden ones for less than ten bucks." OK, Mr. K., you go and do that.

So I do. I bring them back, and there was much rejoicing and celebrating over my frugality, brilliance, ingenuity, and raw talent.

Until one day...

The principal (who I admire and respect, remember) comes to me and very apologetically said that we couldn't have our pull-down-rod-things. I asked if it was because I made them, to which she said no. I asked if we were supposed to irresponsibly over-spend to get them from the "approved" vendor. Again, no.

Why couldn't we have them, you ask?

Because the district (read: spineless, insulated, denying-there's-any-gang, violence, or drug problem school board, who are elected officials who don't know sh!t about education anyway) said they could be used as weapons.

My reply to my boss?

"You're damn right they could!"

You see, while it wasn't their original intent, I did in fact select the one-inch diameter dowel rods for my little project with just that very idea in mind.

I asked Ms. Principal if she would forward a question to our school board on my--and all the teachers in the school's--behalf: "So let me get this straight: We've had 3 students on 3 different campuses threaten violence in the school on a mss scale JUST IN THE LAST WEEK ALONE. Before that, a darn serious and well thought-out plot was discovered that involved the outright murder of MY principal and a few selected teachers--yours truly among them. There is NOTHING to prevent a student or an adult from strolling onto any campus in the district with heavy firepower and doing as they please.

Yet I can't even have a STICK? Well, OK. I'll do as I'm told. You see, I abide by the rules and laws that are given to me, unlike the aforementioned folks. The only request I have for you, Mr. School Board Dude, is 'Do you REALLY think that your dumbass and ignorant rules will be ANY comfort to my widow and my fatherless children? My blood--and that of countless other teachers and students--will be on YOUR inept hands."

She wouldn't relay my question, but did say that there was going to be a special meeting on school security over the summer, and she wanted me to present my plan for security that I had drafted at the beginning of the year to them.

But A STICK?

Don't do a damn thing to keep punks with cheap pistols off the campus, that's bad enough. But take away anything that someone can use to defend the students--THAT ARE THEIR RESPONSIBILITY--and themselves.

Genius. Pure genius.

See what you get when the inmates run the asylum?

DK

tomder55 answered on 05/03/07:

I have a titanium bat my wife named 'thumper'. Could always rig a tempoary hook on it to pull down the overhead.

Itsdb rated this answer Excellent or Above Average Answer
kindj rated this answer Excellent or Above Average Answer

Question/Answer
kindj asked on 05/03/07 - George Bush RESIGNS!!

We all have our disagreements with President Bush. Immigration, U.S. Attorney firings, Iraq, Darfur, etc. are all hot topics these days. The following "speech" was written yesterday by an ordinary Maineiac. While satirical in nature, all satire must have a basis in fact to be effective. An excellent piece by a person who does not write for a living. Sent with the author's permission.
The speech George W. Bush SHOULD give:

Normally, I start these things out by saying "My Fellow Americans." Not doing it this time. If the polls are any indication, I don't know who more than half of you are anymore. I do know something terrible has happened, and that you're really not fellow Americans any longer.

I'll cut right to the chase here: I quit. Now before anyone gets all in a lather about me quitting to avoid impeachment, or to avoid prosecution or something, let me assure you: there's been no breaking of laws or impeachable offenses in this office.

The reason I'm quitting is simple. I'm fed up with you people.

I'm fed up because you have no understanding of what's really going on in the world. Or of what's going on in this once-great nation of ours. And the majority of you are too damned lazy to do your homework and figure it out.

Let's start local. You've been sold a bill of goods by politicians and the news media. Polls show that the majority of you think the economy is in the tank. And that's despite record numbers of homeowners including record numbers of MINORITY homeowners. And while we're mentioning minorities, I'll point out that minority business ownership is at an all-time high. Our unemployment rate is as low as it ever was during the Clinton Administration. I've mentioned all those things before, but it doesn't seem to have sunk in.

Despite the shock to our economy of 9/11, the stock market has rebounded to record levels and more Americans than ever are participating in these markets. Meanwhile, all you can do is whine about gas prices, and most of you are too damn stupid to realize that gas prices are high because there's increased demand in other parts of the world, and because a small handful of noisy idiots are more worried about polar bears and beachfront property than your economic security.

We face real threats in the world. Don't give me this "blood for oil" thing. If I was trading blood for oil I would've already seized Iraq's oil fields and let the rest of the country go to hell. And don't give me this 'Bush Lied People Died' crap either. If I was the liar you morons take me for, I could've easily had chemical weapons planted in Iraq so they could be 'discovered.' Instead, I owned up to the fact that the intelligence was faulty. Let me remind you that the rest of the world thought Saddam had the goods, same as me. Let me also remind you that regime change in Iraq was official US policy before I came into office. Some guy named 'Clinton' established that policy. Bet you didn't know that, did you?

You idiots need to understand that we face a unique enemy. Back during the cold war, there were two major competing political and economic models squaring off. We won that war, but we did so because fundamentally, the Communists wanted to survive, just as we do. We were simply able to outspend and out-tech them.

That's not the case this time. The soldiers of our new enemy don't care if they survive. In fact, they want to die. That'd be fine, as long as they weren't also committed to taking as many of you with them as they can. But they are. They want to kill you. And the bastards are all over the globe.

You should be grateful that they haven't gotten any more of us here in the United States since September 11. But you're not. That's because you've got no idea how hard a small number of intelligence, military, law enforcement and homeland security people have worked to make sure of that. When this whole mess started, I warned you that this would be a long and difficult fight. I'm disappointed how many of you people think a long and difficult fight amounts to a single season of 'Survivor'. Instead, you've grown impatient. You're incapable of seeing things through the long lens of history, the way our enemies do. You think that wars should last a few months, a few years, tops.

Making matters worse, you actively support those who help the enemy. Every time you buy the New York Times, every time you send a donation to a cut-and-run Democrat's political campaign, well, dammit, you might just as well Fedex a grenade launcher to a Jihadist. It amounts to the same thing.

In this day and age, it's easy enough to find the truth. It's all over the Internet. It just isn't on the pages of the New York Times or on NBC News. But even if it were, I doubt you'd be any smarter. Most of you would rather watch American Idol.

I could say more about your expectations that the government will always be there to bail you out, even if you're too stupid to leave a city that's below sea level and has a hurricane approaching. I could say more about your insane belief that government, not your own wallet, is where the money comes from. But I've come to the conclusion that were I to do so, it would sail right over your heads.

So I quit. I'm going back to Crawford. I've got an energy-efficient house down there (Al Gore could only dream) and the capability to be fully self-sufficient. No one ever heard of Crawford before I got elected, and as soon as I'm done here pretty much no one will ever hear of it again. Maybe I'll be lucky enough to die of old age before the last pillars of America fall.

Oh, and by the way, Cheney's quitting too. That means the smiling, vacuous nitwit Pelosi is your new President. You asked for it. Watch what she does carefully, because I still have a glimmer of hope that there're just enough of you remaining who are smart enough to turn this thing around in 2008.

So that's it. God bless what's left of America. Some of you know what I mean. The rest of you, * off.





--
Charles F. Hovey, Jr.
Managing Director
Molpus Woodlands Advisors
85 Eastern Avenue
Gloucester, MA 01930
978-282-8251
Fax: 978-282-1182
Corporate offices in Jackson, Mississippi
please see: www.molpus.com

tomder55 answered on 05/03/07:

although he would be perfectly justified to make such a speech he won't ...He cares too much for the country .He is a plain spoken patriot from Texas which means by nature he has a stuborn streak in him.

History will treat his presidency much kinder than his contemporaries are ;much like Harry Truman's .

I've said it before ; future Presidents will be fighting the war against jihadistan into the next generation no doubt .The Bush doctrine will still be the basic template that they will use to fight the war.

Itsdb rated this answer Excellent or Above Average Answer
kindj rated this answer Excellent or Above Average Answer

Question/Answer
ETWolverine asked on 05/02/07 - Global warming or just breaking wind?

Experts: Rice Farming Huge Source of Methane Emissions

Wednesday, May 02, 2007

Associated Press
BANGKOK, Thailand —

As delegates to a climate conference here debate how to reduce greenhouse gases, one of the problems — and a possible solution — lies in the rice fields that cover much of Thailand, the rest of Asia and beyond.

Methane emissions from flooded rice paddies contribute to global warming just as coal-fired power plants, automobile exhausts and other sources do with the carbon dioxide they spew into the atmosphere.

In fact, the report from the Intergovernmental Panel on Climate Change meeting this week in Bangkok concludes that rice production was a main cause of rising methane emissions in the 20th century. It calls for better controls.

• Click here to visit FOXNews.com's Natural Science Center.

"There is no other crop that is emitting such a large amount of greenhouse gases," said Reiner Wassmann, a climate change specialist at the International Rice Research Institute in the Philippines.

"Methane emissions are unique to rice," he said. "If Asian countries are exploring possibilities to reduce greenhouse gas, they have to look at rice production. I'm not saying it's the biggest source, but in Asia it's a source that cannot be neglected."

It's the bacteria that thrive in flooded paddies that produce methane, by decomposing manure used as fertilizer and other organic matter in the oxygen-free environment. The gas is emitted through the plants or directly into the atmosphere.

A molecule of methane is 21 times more potent than a molecule of carbon dioxide as a heat-trapping gas.

Although carbon dioxide is still the bigger problem, representing 70 percent of the warming potential in the atmosphere, rising levels of methane now account for 23 percent, reports the U.S. Environmental Protection Agency.

After decades of atmospheric buildup, methane — also emitted naturally from wetlands and from other manmade sources, such as landfills and cattle farming — has leveled off in the past few years.

Some scientists credit changes in rice production, and some also trace it to repairs in oil and gas storage facilities that can leak methane.

A 2005 study by U.S. scientists focused on China, which produces a third of the world's rice and where rice fields have shrunk by 24 million acres in the past decade as farmers shifted to other crops and abandoned marginal land.

The study also found that nitrogen-based fertilizer has replaced manure, and many Chinese farmers are using less water on their fields.

For Asians, modifying rice production might prove easier and cheaper than some of the other fixes proposed in the IPCC draft report, such as switching from coal to solar, wind power or other renewable energy sources.

But despite the recent leveling off, the EPA projects that global methane emissions will rise again, as rice fields expand with growing populations.

Wassmann said few countries have followed China's example, instead ignoring such solutions as periodically draining their fields or shifting to locations that need less water.

Scientists say such measures pose the same challenge for poor countries as proposals to introduce environmentally friendly tilling methods or capping methane from livestock manure: Farmers often lack the funds and know-how to shift away from techniques in use for generations.

"In the developing world, you really have to think first and foremost about providing population with food," said Pete Smith of the University of Aberdeen in Scotland, lead author of the IPCC report's section on agriculture. "You can't start thinking about climate mitigation if you have to feed your family."

Thailand, the world's largest rice exporter, shows both the promise and limitations of trying to make the industry greener.

Most large mills here burn leftover rice husks for power — a more climate friendly source than coal or oil — and are increasingly selling excess power back to the state.

"Instead of letting it rot in the fields and produce bad gas, we burn it and make use of it," said Rut Subniran, executive chairman of the Patum Rice Mill and Granary outside Bangkok. "This is good for the country because it can reduce our oil imports. It's good for the environment."

But a few miles away, impoverished rice farmers have largely ignored government calls to periodically drain their fields to help reduce methane emissions.

Busy harvesting the latest crop, some blamed tradition and habit, but others said draining the fields was just too costly.

"The government has told us how rice paddies release methane," said Adisak Wantayachiwa, who farms 28 acres north of Bangkok. But most farmers "don't want to pay the cost of draining their fields," he said. "They would just rather keep them flooded."

----------------

So... technology is not solely responsible for global warming. Agriculture, specifically rice paddies, which are a main source of a staple food product in most third world countries, are a major source of greenhouse gasses.

So, what are we to do. We can't use technology because it causes carbon dioxide, and we can't farm because it causes methane. Can't raise cows, pigs or chickens either... they are also major sources of methane production. We also have to cork our own butts, since we also fart methane. How will we live?

That is the idiocy of the whole eco-movement. They try to get us to change our lifestyles based on their biases, with no scientific proof to back up global warming or the greenhouse effect. But in order to comply with the restrictions they would have on us, we literally would have to kill ourselves off.

Between you and me, if it came down to a choice of possibly dying off in a few hundred thousand years or definitely dying off now, I'll choose the first option. But that's just me; I'm selfish that way.

Elliot

tomder55 answered on 05/03/07:

I can't take it anymore !!! An inconvenient tooth . They want us to switch from consuming live stock to grain ;....now eating the grain is no good ? I refuse to graze on switch grass ;besides..we have to conserve that for ethanol.

I think we should all look toward New Zealand to lead the way !! Maybe we can all buy fart credits .

ETWolverine rated this answer Excellent or Above Average Answer
Itsdb rated this answer Excellent or Above Average Answer

Question/Answer
tropicalstorm asked on 05/03/07 - hotels taking Bibles out

of drawers and replacing them with An Inconvenient Truth.

California taking mom and dad out of books because some kids now have one parent or two same sex parents therefore promoting heterosexual behavior.

Pittsburgh delays smoking ban largely because of lawsuit by Mitchels restuarant (which happens to be where the Lawyers and Police go for lunch during court)

What next besides the threat of a ban on my barbque grill?

tomder55 answered on 05/03/07:

they don't call him the Goracle for nothing.I knew it wouldn't be long before a religion developed around his cult ideas .

They will take my grill when they pry my spatula from my cold dead fingers.

tropicalstorm rated this answer Excellent or Above Average Answer

Question/Answer
Itsdb asked on 04/30/07 - Huh?

Sorry if this is old news to you, but did any of you hear Schmucky Schumer telling us what Harry Reid meant by saying the Iraq war is lost?

    "What Harry Reid is saying is that this war is lost -- in other words, a war where we mainly spend our time policing a civil war between Shiites and Sunnis. We are not going to solve that problem. . . . The war is not lost. And Harry Reid believes this -- we Democrats believe it. . . . So the bottom line is if the war continues on this path, if we continue to try to police and settle a civil war that's been going on for hundreds of years in Iraq, we can't win. But on the other hand, if we change the mission and have that mission focus on the more narrow goal of counterterrorism, we sure can win."


It's good that Democrats are on board with Bush and Petraeus' counterterrorism efforts in Iraq. Or are they? Who the heck knows what they're thinking, they seem to be straddling both sides of the fence as usual.

tomder55 answered on 04/30/07:

I give you the schmuckster from 16 months ago :

December 11, 2005

The Honorable George W. Bush
President of the United States
The White House
1600 Pennsylvania Avenue
Washington, DC 20500

Dear President Bush:

As you know, the training and deployment of Iraqi Security Forces (ISF) capable of defending Iraq from outside influences and policing against domestic insurgents is a key objective of our ongoing efforts in Operation Iraqi Freedom, and one that must be completed before we can end our military presence. Like you, I recognize that America must succeed in Iraq and in order to accomplish that goal I steadfastly support the establishment of a competent and capable ISF.

I am troubled by recent reports that the establishment of such forces is not moving forward as well as was hoped, and by the continuing inability to obtain a definitive assessment of the current status and abilities of the ISF. These matters lie at the very heart of the length and success of American military involvement in Iraq, and it is important for the American people to fully understand how much progress we have made in helping to establish the ISF.

I am deeply concerned that the lack of this information will make it increasingly difficult to chart a proper course in Iraq and may ultimately serve to undermine our chances of success. In order to give America the best chance for lasting victory in Iraq, I urge you to appoint a senior delegation of distinguished retired military leaders to travel to Iraq, assess the situation for themselves, and issue a public report of their findings.

As you have often said yourself, “As the Iraqis stand up, we will stand down.” Unfortunately, information available on the training and readiness of the ISF makes it difficult to predict when this will occur. For instance, in October it was revealed that out of over ninety ISF battalions, the number of fully independent, or “Category One” ISF battalions was revised down from three to one. The fact that two battalions were apparently downgraded would appear to be a significant step backward. In order to prevent any further regressions, it is important to have full and clear accounting of the situation on the ground and an explanation as to how this occurred.

There are also important and unanswered questions as to the actual size and cohesiveness of the ISF. As of mid-November the Department of Defense indicated that there were 211,700 members of the ISF, only about 65% of the number hoped to be reached by August of 2007. If we are to be successful in Iraq, we must clearly understand whether it is possible to complete training those already enlisted, as well as recruit and train the remaining personnel before the deadline.

Similarly, information from the GAO would indicate that there is a significant chance that possibly tens of thousands of those in the police component of the ISF may have deserted. Concerns also abound that the ISF is still 18-24 months away from operational independence and lack a sufficient command structure, and are a target for insurgent infiltration. Obviously, we need to get to the bottom of whether or not the ranks of the ISF are stable and cohesive before we can certify that they have been fully trained and are ready to assume independent responsibility for Iraq’s security.

I agree with you that the stakes in Iraq and that the United States must succeed in its work there. By providing an unvarnished analysis of ISF numbers, capability, and cohesion the delegation I am asking you to appoint could provide the American people with an honest assessment of our progress, inform the Administration and Congress of the best path to take forward, and allow us to move toward the shared goal of bringing our troops home and out of harms way. I urge you to appoint this delegation as soon as possible.

Sincerely,

Charles E. Schumer
United States Senate

~~~~~~~~~~~~~~~~~~~~~~~~~~~~~~~~~

Does he not see that many of concerns then are being addressed ? The Iraqi army is on its way to being an army, accepting training, showing up for training, giving their lives for their own country. But he may have missed that fact because he was too involved in his conflict of interest role in the recent Gonzales hearings .

Itsdb rated this answer Excellent or Above Average Answer

Question/Answer
Mathatmacoat asked on 04/29/07 - Now for something of absolutely Earth shattering importance!


Cricket World Cup

Adam Gilchrist


Farce ruins Cup defence

By Toby Forage
FOXSPORTS.com.au editor
April 29, 2007

AUSTRALIA won an historic third successive World Cup this morning, but not before celebrating prematurely as the tournament's troubled run ended in complete farce.

With three overs left in the match, reduced to 38 overs-a-side because of rain, Sri Lanka's batsmen were offered the light and accepted, sparking wild celebrations from the Australia players.

But after whoops of delight and much prancing around, Ricky Ponting's men were told the game wasn't over after all, and after a long discussion with umpires Aleem Dar and Steve Bucknor, as well as off-field officials, play continued in near darkness.

It was an appalling way to finish a tournament that has been pilloried for its excessive length and lack of depth, and the International Cricket Council will have much to ponder in the four years between now and the next tournament on the sub-continent

The biggest shame is that Australia historic moment, and a performance of sheer class from Adam Gilchrist, will be forgotten as a result of a quite ludicrous finale.

Even the presentation rostrum was sent back into the stands by the officials when the only result possible was an Australia victory.

Three overs in darkness and another wicket later, the game mercifully ended with Sri Lanka on 8-215, chasing Australia's imposing 4-281, losing by 53 runs in a match punctuated by rain delays and the chaos of the ilk Fawlty Towers scriptwriters would have been proud of.

Australia's total was set up by an awesome knock of 149 by Gilchrist, who racked up his runs in little more than two hours from just 104 balls to set a new individual high score record in the World Cup final, beating Ponting's record of four years ago of 140.

"It's been a while between drinks for my hundred, and really pleasing to do it on such an important day," Gilchrist said in the gloom as Australia celebrated in front of a crowd that was probably grateful for its bright yellow team colours, given the ridiculous gloom.

"It's an unbelievable feeling. The guys have worked so hard," he added, without making mention of the bizarre circumstances of victory.

Gilchrist, dropped on 31, and fellow left hander Matthew Hayden's stand of 172 was a World Cup final first-wicket record, surpassing the 129 shared by England's Mike Brearley and Geoff Boycott during West Indies' 92-run win at Lord's in 1979.

Gilchrist opened up in Chaminda Vaas's second over. He flicked the bowler's eighth ball for four over square leg, and next ball he drove him over long-on for six.

Vaas, after an expensive three-over spell costing 24 runs, was replaced by fellow quick Dilhara Fernando, retained despite conceding 45 runs in five overs during Tuesday's 81-run semi-final win over New Zealand.

Fernando, in his second over, dropped a low caught and bowled chance off Gilchrist's checked drive, with the keeper on 31 and Australia 0-47, and conceded 74 from his eight overs.

Next ball Gilchrist struck him for four through mid-wicket to bring up Australia's fifty. The ball after was lashed through long-on and Gilchrist immediately topped that with a six in the same area.

He completed a 43-ball fifty with two sixes and five fours.

Off spinner Tillakaratne Dilshan wasn't let off the hook, Gilchrist driving the bowler over his head for two superb straight sixes as he passed his previous best score this tournament, 59 not out against Bangladesh.

The 35 year old then saw Australia to 100 in just 102 balls by off driving Fernando for six.

Gilchrist swept Murali for a six that soared over mid-wicket before Sri Lanka skipper Mahela Jayawardene brought back trump card Malinga in a bid to break the stand. His first ball back was smashed for six over long-off by Hayden.

Gilchrist then struck Malinga to the same boundary to bring up a superb century in 72 balls with six sixes and eight fours.

Hayden, renowned as a power-hitter and the tournament's leading run-scorer, was still in the 30s.

Sri Lanka's chase began badly when Upul Tharanga edged a ball into Gilchrist's gloves behind the wicket after less than 10 minutes of the reply.

But Kumar Sangakkara and veteran left hander Sanath Jayasuriya gave Sri Lanka hope of a repeat of the 1996 final, when it beat Australia, with a partnership of 116 before Sangakkara was caught on 54 by Ponting off Brad Hogg's spin bowling.

When part-time bowler Michael Clarke clean bowled Jayasuriya for 63 with a short ball that didn't bounce, Sri Lanka's chase had faltered, and victory began to look inevitable.

After the farce of the end that wasn't, victory eventually was Australia's, and it's third straight World Cup in the bag.

Australia did not lose a single match at this World Cup, extending an extraordinary run of victories that goes back to 1999 and defeat to Pakistan in the group stage of that tournament, which it won to start the hat-trick of titles.

With Agence France-Presse

tomder55 answered on 04/30/07:

see;even people familiar with the game aren't sure of the rules and when the game "mercifully "ends .

What does " offered the light " mean ? Is that before or after tea break ?

Mathatmacoat rated this answer Excellent or Above Average Answer
paraclete rated this answer Excellent or Above Average Answer

Question/Answer
Itsdb asked on 04/27/07 - 2008 Democratic National Convention agenda

I was able, through my sly and cunning, to find the agenda for the upcoming 2008 Democratic National Convention. This is top secret information that I acquired through a link on the DNC’s website, along with other documents and security related issues. It said something like…”New York Times - go here.

For you, dear readers, The Agenda!

7:00 P.M. Opening flag burning.

7:15 P.M. Pledge of allegiance to U.N.

7:30 P.M. Ted Kennedy proposes a toast

7:30 till 8:00 P.M. Nonreligious prayer and worship. Jessie Jackson and Al Sharpton.

8:00 P.M. Ted Kennedy proposes a toast.

8:05 P.M. Ceremonial tree hugging.

8:15- 8:30 P.M. Gay Wedding Barney Frank Presiding.

8:30 P.M. Ted Kennedy proposes a toast.

8:35 P.M. Free Saddam Rally. Cindy Sheehan , Susan Sarandon.

9:00 P.M. Keynote speech. The proper etiquette for surrender French President Jacques Chirac.

9:15 P.M. Ted Kennedy proposes a toast.

9:20 P.M. Collection to benefit Osama Bin Laden kidney transplant fund

9:30 P.M. Unveiling of plan to free freedom fighters from Guantanamo Bay. Sean Penn.

9:40 P.M. Why I hate the Military, A short talk by William Jefferson Clinton

9:45 P.M. Ted Kennedy proposes a toast

9:50 P.M. Dan Rather presented Truth in Broadcasting award, presented by Michael Moore.

9:55 P.M., Ted Kennedy proposes a toast

10:00 P.M. How George bush and Donald Rumsfeld brought down the World Trade Center Towers, Howard Dean.

10:30 P.M. Nomination of Hillary Rodham Clinton by Mahmood Ahmadinejad.

11:00 P.M. Ted Kennedy proposes a toast

11:05 P.M. Al Gore reinvents Internet

11:15 P.M. Our Troops are War criminals , John Kerry

11:30 P.M.Coronation Of Mrs. Rodham Clinton

12:00 A.M. Ted Kennedy proposes a toast

12:05 A.M. Bill asks Ted to drive Hillary home

Also, I’ve acquired the manifesto. These are the 25 points of interest for the Democrats. And to think I’ve been a Republican all this time. Gosh, the Democrats are right.

1. Mandatory homosexuality

2. Drug-filled condoms in schools

3. Introduce the new Destruction of Marriage Act

4. Border fence replaced with free shuttle buses

5. Osama Bin Laden to be Secretary of State

6. Withdraw from Iraq, apologize, reinstate Hussein

7. English language banned from all Federal buildings

8. Math classes replaced by encounter groups

9. All taxes to be tripled

10. All fortunes over $250,000 to be confiscated

11. On-demand welfare

12. Tofurkey to be named official Thanksgiving dish

13. Freeways to be removed, replaced with light rail systems

14. Pledge of Allegiance in schools replaced with morning flag-burning

15. Stem cells allowed to be harvested from any child under the age of 8

16. Comatose people to be ground up and fed to poor

17. Quarterly mandatory abortion lottery

18. God to be mocked roundly

19. Dissolve Executive Branch: reassign responsibilities to UN

20. Jane Fonda to be appointed Secretary of Appeasement

21. Outlaw all firearms: previous owners assigned to anger management therapy

22. Texas returned to Mexico

23. Ban Christmas: replace with Celebrate our Monkey Ancestors Day

24. Carter added to Mount Rushmore

25. Modify USA’s motto to “Land of the French and the home of the brave”

tomder55 answered on 04/27/07:

free Saddam ? not while he's enjoying those virgins.

Itsdb rated this answer Excellent or Above Average Answer

Question/Answer
kindj asked on 04/26/07 - Oil wars? Hmmmm....I think not.

Crude Oil and Total Petroleum Imports Top 15 Countries
February 2007 Import Highlights: Released on April 17, 2007
Preliminary monthly data on the origins of crude oil imports in February 2007 has been released and it shows that four countries have each exported more than 1.10 million barrels per day to the United States. Including those countries, a total of five countries exported over 1.00 million barrels per day of crude oil to the United States (see table below). The top five exporting countries accounted for 72 percent of United States crude oil imports in February while the top ten sources accounted for approximately 89 percent of all U.S. crude oil imports. The top sources of US crude oil imports for February were Canada (1.838 million barrels per day), Mexico (1.358 million barrels per day), Saudi Arabia (1.185 million barrels per day), Venezuela (1.115 million barrels per day) and Nigeria (1.061 million barrels per day). The rest of the top ten sources, in order, were Angola (0.451 million barrels per day), Algeria (0.392 million barrels per day), Iraq (0.325 million barrels per day), Ecuador (0.178 million barrels per day), and Kuwait (0.158 million barrels per day). Total crude oil imports averaged 9.047 million barrels per day in February, which is a decrease of 1.145 million barrels per day from January 2007.

Canada remained the largest exporter of total petroleum in February, exporting 2.386 million barrels per day to the United States, which was a slight decrease from last month (2.470 thousand barrels per day). The second largest exporter of total petroleum was Mexico with 1.507 million barrels per day.

Crude Oil Imports (Top 15 Countries)
(Thousand Barrels per Day)
Country Feb-07 Jan-07 YTD 2007 Feb-06 Jan - Feb 2006

--------------------------------------------------------------------------------

CANADA 1,838 1,856 1,847 1,710 1,740
MEXICO 1,358 1,435 1,398 1,774 1,735
SAUDI ARABIA 1,185 1,559 1,382 1,418 1,375
VENEZUELA 1,115 955 1,031 1,178 1,204
NIGERIA 1,061 1,106 1,085 1,342 1,232
ANGOLA 451 553 504 464 441
ALGERIA 392 548 474 163 201
IRAQ 325 531 433 450 493
ECUADOR 178 269 226 222 302
KUWAIT 158 172 165 152 110
UNITED KINGDOM 137 61 97 82 58
BRAZIL 103 204 156 164 110
CHAD 87 70 78 77 76
COLOMBIA 73 137 106 126 148
LIBYA 63 9 35 58 48

Total Imports of Petroleum (Top 15 Countries)
(Thousand Barrels per Day)
Country Feb-07 Jan-07 YTD 2007 Feb-06 Jan - Feb 2006

--------------------------------------------------------------------------------

CANADA 2,386 2,470 2,430 2,262 2,287
MEXICO 1,507 1,566 1,538 1,878 1,835
VENEZUELA 1,353 1,195 1,270 1,475 1,508
SAUDI ARABIA 1,207 1,563 1,394 1,451 1,408
NIGERIA 1,102 1,136 1,120 1,377 1,277
ALGERIA 554 778 672 446 586
ANGOLA 464 574 522 478 454
IRAQ 325 531 433 450 493
VIRGIN ISLANDS 312 425 371 318 297
UNITED KINGDOM 257 194 224 205 196
RUSSIA 236 347 294 304 259
ECUADOR 185 272 231 234 311
KUWAIT 168 172 170 158 114
BRAZIL 150 250 203 203 152
TRINIDAD AND TOBAGO 135 121 128 62 102

Note: The data in the tables above exclude oil imports into the U.S. territories.

Seems to me that if the left wants to insist that we are waging war in Iraq over oil, then they need to step us and explain why--if oil is our motivation--why we haven't taken over Canada, Mexico, etc.

This was easy to find, it's here: http://www.eia.doe.gov/pub/oil_gas/petroleum/data_publications/company_level_imports/current/import.html

So why do we still get this erroneous babble from the left?

DK

tomder55 answered on 04/26/07:

Of course it would make sense that Canada and Mexico would be our primary sources of foreign oil. I'm not thrilled about the Venezuela total . Oil of course is a comodity sold in the international market so origin does not determine price ; but there are some countries I'd just as soon not deal with .

The fictional narrative that the war was about controlling Iraqi oil was always fanciful . Anyone who even investigates it superficially would realize that there has been no plunder of Iraqi oil. Instead ;we have spent precious human and monitary resource trying to rebuild the infrastructure of Iraq.

So far there has not been a single contract awarded to a US firm by the new Iraqi government .A Norwegian firm signed a deal with the Kurds.Lukoil is in the negotiation phase ;Lukoil is Russian and the Russians did nothing to support the effort.South Korean oil ministers signed a memorandum of understanding to cooperate in the development of Iraqi oil fields .China, India, Vietnam are all trying to take advantage of laws Iraq will soon pass. The Chief Executive Officer at Ivanhoe Energy;a Canadian firm , stated the company has signed a joint partnership agreement with Japanese Inpex Corp. to develop an oil field in Iraq.

If there is an oil grab going on ,we are too busy trying to defend and help secure the country to get on that gravy train.

kindj rated this answer Excellent or Above Average Answer

Question/Answer
curious98 asked on 04/26/07 - Pat Tillman and Jessica Lynch

For quite sometime I have been hammering on the head of some of our colleagues on these Q&A boards that Government Administrations all over feel no repulsion whatsoever about lying to us in the most deliberate and obnoxious way. I have read today this article which seems to show that I am not totally wrong:

Quote:

The searing congressional testimonies from the family of Pat Tillman and Jessica Lynch may sound the long overdue death knell for mainstream American public support for the Bush administration and its criminal war.

Tuesday, one unwilling and outraged war poster child and the agonized family members of the other stood before Congress, looked the Bush administration in the eye, and said, “Enough.”

Kevin Tillman, who served with his brother, accused the Bush administration of intentional deceit.
The accounts of the Tillmans, Lynch, and their fellow soldiers lay bare the fact that the Bush administration engaged in an elaborate and deliberate criminal cover-up of Tillman’s fratricide, as well as Lynch’s ordeal, to deceive the American people.

In what can only described as post-facto rape, the administration hijacked the images of Tillman and Lynch for war propaganda, fabricated elaborate pro-war fantasies around both, and then flooded the media with these pro-Bush, pro-war falsehoods.

The violation of Tillman will prove to be even more grotesque and appalling if investigations determine that Tillman’s disapproval of the war contributed in any way to his death.

An historic turning point has been reached. The American people who embraced the Pat Tillman/Jessica Lynch lie will “get” the Pat Tillman/Jessica Lynch truth, now that it has been honestly and heroically presented.

Now the tidal wave begins.

Piece by piece, the Bush administration’s criminal construct is finally being undone. The Bush administration, and its “war on terrorism” (in its present incarnation), will not recover from this mortal blow.

By Larry Chin
Online Journal Associate Editor

Apr 26, 2007, 00:40”

Unquote:

Your comments, please.

Curious98

PS:
BTW. The above case is making a lot of noise in Europe and we are wondering what the Bush Administration is going to say in their defence.

tomder55 answered on 04/26/07:

Claude ;

I would like to add that since the war began I have made a deliberate and ,I could say a rather exhaustive search to bring stories of legitimate acts of bravery ,sacrifice and heroism of the fighting forces of the Multinational force . Why is it so difficult ? Because the press has not given us these type of stories . They instead have concocted a tale that highlights abuse and mismanagement that they roll out on a seemingly daily basis .Progress is never acheived ; good is never done .The only acts of courage they deem worthy of coverage is of the injured soldier's fight to cope .

The situation has gotten so rediculous that the military has our soldiers wearing freaking video camera's on their helmets just to prove that they are not doing an atrocity in the field .



Perhaps if the press were acting responsibly then the military would not be tempted to embellish the narrative . Heck ;I'd be happy with some stright up honest reporting .

curious98 rated this answer Excellent or Above Average Answer
Itsdb rated this answer Excellent or Above Average Answer
kindj rated this answer Excellent or Above Average Answer

Question/Answer
curious98 asked on 04/26/07 - Pat Tillman and Jessica Lynch

For quite sometime I have been hammering on the head of some of our colleagues on these Q&A boards that Government Administrations all over feel no repulsion whatsoever about lying to us in the most deliberate and obnoxious way. I have read today this article which seems to show that I am not totally wrong:

Quote:

The searing congressional testimonies from the family of Pat Tillman and Jessica Lynch may sound the long overdue death knell for mainstream American public support for the Bush administration and its criminal war.

Tuesday, one unwilling and outraged war poster child and the agonized family members of the other stood before Congress, looked the Bush administration in the eye, and said, “Enough.”

Kevin Tillman, who served with his brother, accused the Bush administration of intentional deceit.
The accounts of the Tillmans, Lynch, and their fellow soldiers lay bare the fact that the Bush administration engaged in an elaborate and deliberate criminal cover-up of Tillman’s fratricide, as well as Lynch’s ordeal, to deceive the American people.

In what can only described as post-facto rape, the administration hijacked the images of Tillman and Lynch for war propaganda, fabricated elaborate pro-war fantasies around both, and then flooded the media with these pro-Bush, pro-war falsehoods.

The violation of Tillman will prove to be even more grotesque and appalling if investigations determine that Tillman’s disapproval of the war contributed in any way to his death.

An historic turning point has been reached. The American people who embraced the Pat Tillman/Jessica Lynch lie will “get” the Pat Tillman/Jessica Lynch truth, now that it has been honestly and heroically presented.

Now the tidal wave begins.

Piece by piece, the Bush administration’s criminal construct is finally being undone. The Bush administration, and its “war on terrorism” (in its present incarnation), will not recover from this mortal blow.

By Larry Chin
Online Journal Associate Editor

Apr 26, 2007, 00:40”

Unquote:

Your comments, please.

Curious98

PS:
BTW. The above case is making a lot of noise in Europe and we are wondering what the Bush Administration is going to say in their defence.

tomder55 answered on 04/26/07:

I'm glad you posted this ;I was preparing a post on the very subject already so I may just as well use it as my response.I do not condone the circumstances behind the Tillman cover-up ;and as you will see below ,there is more to the Jessica Lynch story than she testified about .


The Army's attempt to portray the death of Pat Tillman as a heroic sacrifice instead of a classic friendly -fire FUBAR is only partly related to the CYA (cover your ass)syndrome. In the Second World War, similar incidents were also rewritten to put a positive gloss upon them.

Take the case of Captain Colin Kelly, a U.S. Army B-17 pilot in the Philippines . When the Japanese navy staged an amphibious landing, Kelly and his squadron were sent out to attack the invasion fleet. The bombers attacked individually, and Kelly's was hit as it dropped its bombs on the ships below. Kelly was killed when his plane crashed near its base, after the rest of the crew bailed out. The story that was circulated in the press was Kelly's plane had been hit on its attack run, that he had ordered his crew to bail out, and that he had then crashed his plane into the Japanese battleship Haruna, for which action he was supposedly awarded the Congressional Medal of Honor.

All very nice, except: Kelly's plane was hit after dropping its bombs, which failed to hit anything; the battleship Haruna was several hundred miles away when Kelly dropped his bombs ; Kelly's plane had almost returned to its field when he ordered the crew to bail out; Kelly was killed trying to bring the plane in for an emergency landing; for which action he was awarded the Distinguished Service Cross. Other than that, the story in the press was absolutely true.

Now, Kelly was an authentic hero: he made an attack on the Japanese against heavy odds, and he saved most of his crew by bringing the plane close to its base. Why then, did the Army feel the need to embellish? Because, at the time, the war was going very badly, and the country needed heroes more than it needed the "truth."

One can find other similar examples, especially in the early days of World War II. General MacArthur, for instance, was awarded the Congressional Medal of Honor and not court-martialed for gross incompetence over his defense of the Philippines. Ernest King remained as Commander-in-Chief of the U.S. Navy, and not relieved for his inept handling of the U-boat crisis off the coasts of America between January and May 1942. The military grossly inflated the number of U-boats sunk in that time (fewer than ten) and hid the number of Allied ships sunk; medals were awarded on the flimsiest of evidence because the country needed heroes and good news when both were scarce.

There is a word for this sort of thing. It is "propaganda". Now, this word has come to take on bad connotations in our day, but in fact, propaganda is a necessary component of any war effort .Our press sixty years ago understood this when they were willing agents of U.S. and Allied propaganda. Of course, back then, the Press considered themselves to be Americans first and journalists second, so the kind of mindless "objectivity"(in their words ;I have a less flattering description for their actions) we see in reporting from Iraq and Afghanistan would have been unthinkable in the Philippines, North Africa, Italy or Normandy. The Press was on our side, and spoke possessively of "our troops." There was none of this "U.S. commanders claimed today....", but rather, "Today our troops smashed... "There was a very good reason for this.

Clausewitz wrote that modern war rests on three pillars: the state, the army and the people. If one of the pillars is weak or collapses, the war effort collapses with it. It is impossible to wage a war without controlling the information the public receives concerning the conduct of that war. War is a terrifying business; it is hard enough on men who have been trained to endure its rigor and who have become accustomed to its images. Civilians, without any standard by which to judge, will find the whole thing repulsive, and will turn away from it. The morale of the home front will suffer. This is particularly true when they attempt to judge the progress and leadership of war by the standards of the civilian world. Wars are not won by armies that are 90% effective over those that are 85% effective, but by those that are 15% effective over those that are 10% effective. This is due to the compounding effect of myriad minor errors that combine to make even the simplest thing difficult; it is what separates "real" war from theoretical war . From the press perspective; wars are always being run badly. Civilian morale suffers, which in turn undermines support for the state and the army, and the war effort collapses.

If this war has demonstrated anything, it is the impossibility of waging a sustained war in a liberal democracy in the presence of an unfettered press. The enemies of liberal democracy, of course, do not believe in, nor have to deal with, a free press and this gives them an dangerous advantage in the kind of war where will to victory is far more important than actual victories on the battlefield.

To further illustrate this point ,all you have to do is critically investigate the circumstances behind the fanciful reporting of the Jessica Lynch capture and rescue .
First ;Jessica Lynch was a hero just by serving her country whether she fired a shot or was knocked out immediately during the ambush that injured her. BUT ;the story of her shoot-out with Iraqi forces was not a product of the US military but of the US media. The US media created this recounting of her exploits from vague, unofficial statements by "undisclosed officials" .

But when the truth war revealed ,they started looking for someone to blame. Who else would they pin it on but the US military?

The fact is that the Washington Compost first reported the fiction about her shoot out with the Iraqis .

'She Was Fighting to the Death'


Details Emerging of W. Va. Soldier's Capture and Rescue


By Susan Schmidt and Vernon Loeb


Washington Post Staff Writers
Thursday, April 3, 2003; Page A01


Pfc. Jessica Lynch, rescued Tuesday from an Iraqi hospital, fought fiercely and shot several enemy soldiers after Iraqi forces ambushed the Army's 507th Ordnance Maintenance Company, firing her weapon until she ran out of ammunition, U.S. officials said yesterday.


Lynch, a 19-year-old supply clerk, continued firing at the Iraqis even after she sustained multiple gunshot wounds and watched several other soldiers in her unit die around her in fighting March 23, one official said. The ambush took place after a 507th convoy, supporting the advancing 3rd Infantry Division, took a wrong turn near the southern city of Nasiriyah.


"She was fighting to the death," the official said. "She did not want to be taken alive." Lynch was also stabbed when Iraqi forces closed in on her position, the official said, noting that initial intelligence reports indicated that she had been stabbed to death. No official gave any indication yesterday, however, that Lynch's wounds had been life-threatening


Several officials cautioned that the precise sequence of events is still being determined, and that further information will emerge as Lynch is debriefed. Reports thus far are based on battlefield intelligence, they said, which comes from monitored communications and from Iraqi sources in Nasiriyah whose reliability has yet to be assessed. Pentagon officials said they had heard "rumors" of Lynch's heroics but had no confirmation.


So there you have it .In the initial reporting ,the US Military was urging caution as they had not determined the facts yet . The Washington Compost sourced this story from one official that they couldn't or wouldn't even identify.

Why couldn't the Compost identify a military official praising a soldier?? Doesthat make sense to you ? Is that really a secret? This isn't a whistle blower or Bush Administration insider. It would more than likely be an officer if this person existed at all.

So why couldn't The Compost name the source? The answer is obvious; because the reporters don't even know who it was, or if the incident even occurred.

Jessica Lynch had the courage to go before the Congress to give her side of the story . However ,she is just as misinformed over the circumstances regarding the reporting of her case as most of the country and the world is . If the Washington Compost has a legitimate source for their original story then I would suggest that now is the time to come forward. They have let this fiction that it was the US Army trying to mythify Jessica Lynch linger far too long.




curious98 rated this answer Excellent or Above Average Answer
Itsdb rated this answer Excellent or Above Average Answer
kindj rated this answer Excellent or Above Average Answer

Question/Answer
tropicalstorm asked on 04/26/07 - new planet answer

the new planet they discovered that is 20 light years away may contain the answer for liberals they can take themselves and their PC life and live on that planet and leave us to destroy ourselves. Rosie O'Donell will weigh twice her current weight. I wonder what other wonders they would encounter when they get to their new planet????

tomder55 answered on 04/26/07:

Does the new planet appear to be cooling or warming ? A Class M planet .....beam me down Scotty !

You have a good idea .They can start over fresh, with an unspoiled planet, and implement their utopian nut case ideas there.I think we should call it Planet Kucinich .Send the Goricle first as a good will ambassador .

I actually like the Princes idea better. Strip-mine that place and really piss off the libs .

Itsdb rated this answer Excellent or Above Average Answer
tropicalstorm rated this answer Excellent or Above Average Answer

Question/Answer
PrinceHassim asked on 04/26/07 - New adventures for Bush ..................


................... The discovery of a new planet thought capable of sustaining life has had the White House abuzz, says a correspondent.

It has been mooted that there could be vast oil reserves a little over twenty light years away, and already

White House staffers are preparing plans to invade the planet and take over all availble fuel sources.

Karl Rove is understood to be collecvting a dossier on the putative inhabitants of the newly discovered orb, and has posited that they will have WMDs capable of blowing the earth back into the past in little over twenty years.

The denizens are not known to be friendly towards the Buish administration, so a pre-emptive strike is already on the table in the White House War Room.

Should we be worried? Should we join the National Guard?



tomder55 answered on 04/26/07:

I'll go !!! I think the discovery is exciting .If we can pick off Marvin the Martian...tanto mejor .

Frankly ;part of the reason for space exploration ;beyond the obvious persuit of *pure* science is to discover resources to exploit. We learned that from the Europeans .

Itsdb rated this answer Excellent or Above Average Answer
PrinceHassim rated this answer Excellent or Above Average Answer

Question/Answer
Itsdb asked on 04/24/07 - 'Kryptonite' discovered in mine

Kryptonite is no longer just the stuff of fiction feared by caped superheroes.

A new mineral matching its unique chemistry - as described in the film Superman Returns - has been identified in a mine in Serbia.

According to movie and comic-book storylines, kryptonite is supposed to sap Superman's powers whenever he is exposed to its large green crystals.

The real mineral is white and harmless, says Dr Chris Stanley, a mineralogist at London's Natural History Museum.

"I'm afraid it's not green and it doesn't glow either - although it will react to ultraviolet light by fluorescing a pinkish-orange," he told BBC News.

Rock heist

Researchers from mining group Rio Tinto discovered the unusual mineral and enlisted the help of Dr Stanley when they could not match it with anything known previously to science.

Once the London expert had unravelled the mineral's chemical make-up, he was shocked to discover this formula was already referenced in literature - albeit fictional literature.

"Towards the end of my research I searched the web using the mineral's chemical formula - sodium lithium boron silicate hydroxide - and was amazed to discover that same scientific name, written on a case of rock containing kryptonite stolen by Lex Luther from a museum in the film Superman Returns.

"The new mineral does not contain fluorine (which it does in the film) and is white rather than green but, in all other respects, the chemistry matches that for the rock containing kryptonite."

The mineral is relatively hard but is very small grained. Each individual crystal is less than five microns (millionths of a metre) across.
~~~~~~~~~~~~~~~~~~~~~~~~~~~~~~~~~~~~~~~~~~~~~~~~~~~~~~

Leave it to those Europeans though, "it will be formally named Jadarite" instead of kryptonite. Meanwhile, I hear Superman has been discovered also - and he's being prepped for his role as 'roving ambassador' on a mission to save the United States. Could this explain what happened to Captain America?

tomder55 answered on 04/24/07:

It was that florine reaction that was key! Too bad it wasn't blue .We could take on Bizzaro .White kryptonite kills plant life ...word to Al Gore.But it was used to kill Virus X so it can't be all bad .


Speaking of the roving ambassador. We were treated to some more Kentucky Fried Hillary at a gathering organized by Al Sharpton. Actually Hillary and Obama were both doing their best(worst) imitation of a Southern black accent but that is not really the point.

She was doing the 'women have to clean up the mess from our stupid white men 'screed and said "when I walk into the Oval Office in 2009 'aaammm afraid I'm going to lift up the rug and aammm going to see so much stuff under thair." This of course will be a departure from the previous Clinton terms when all the stains on the carpet to be cleaned were there for all to see.

Itsdb rated this answer Excellent or Above Average Answer

Question/Answer
tropicalstorm asked on 04/24/07 - mor-on

liberals
now New York is banning metal bats and
Milwaukee wants to start using plastic handcuffs on rowdy students. You know that will have the liberals screaming about psychological damage!

tomder55 answered on 04/24/07:

I want to use the metal bats on the rowdy students. There's a nice *ping* when they connect .

ETWolverine rated this answer Excellent or Above Average Answer
Itsdb rated this answer Excellent or Above Average Answer
tropicalstorm rated this answer Excellent or Above Average Answer

Question/Answer
kindj asked on 04/23/07 - Liberals are "compassionate?" Please!!!

I've been practicing restraint for months now, trying to rise above the filth that contaminates a good portion of this globe. However, I am about to jump into the fray with both feet, loose the figurative broadsword and mace, and take some figurative scalps.

The "Everyone Hates Christianity" board is out of control, as most of you know.

The same group that preaches "tolerance," "compassion," and "understanding" cannot live by those words they spout everytime the opportunity comes up. I'm thinking of one in particular. She claims to be an "intelligent" person and a compassionate one; however, her posts belie her position, and reveal her as nothing more than a bitter, hate-filled person who has no tolerance or love for anyone who doesn't think exactly like her. A couple of the others that folks seem to have problems with will usually give credit where credit is due, and agree to disagree, at least to a point.

But a simple post showing the severe storms in our area--just for the sake of passing along the info--were met with disparaging comments about God by one (to which I think I responded appropriately) and another.....well, never mind.

I think I'm about done with them, on second thought. Between the people who rabidly defend their version of the faith while at the same time attacking others are getting on my nerves as much as the ones running any Judeo-Christian belief into the ground.

I seem to remember something about not casting pearls before swine.....you think that applies to that board?

D "gettin' just plain pissed off" K

tomder55 answered on 04/24/07:

Most postings there I just ignore . I pick and choose my battles with the full knowlege that any insight I bring to the discussion will most likely be ignored out of hand. I let the personal insults roll off my back . Mostly I take it from the source.

The one expert you refer to has bitten the hand that feeds her and has turned on more than one expert who has shown concern for her condition (which she never seems to tire of telling to anyone who listens). I just don't bother and I find I often reply just to instigate the diatribe I am certain to receive .

Same thing with the expert with the cookie cutter god built in his own image . I can trigger alot of !!!!!! and horse hockey pucks with a timely reply to one of his invectives ....always good for a laugh .

I know in the eyes of some of the Christians there I'm a pagan and to the non-believers I'm a Christo-nazi. Between the whole group I figure I must be doing something right.

The way I figure it I could just go away;get pissed off and battle to no avail ,or just have some fun with it. Since I am here because it's fun then I choose option #3.

Itsdb rated this answer Excellent or Above Average Answer
kindj rated this answer Excellent or Above Average Answer

Question/Answer
Itsdb asked on 04/23/07 - Someone finally asks the question

after Hillary gave the answer...

Could a Vote for Hillary Be a Two-For-One Deal?

    It's something none of the other presidential contenders has to think about: If Sen. Hillary Clinton, D-N.Y., wins, she's the only candidate who would have a first gentleman — Bill Clinton — available to take on duties.

    And of course he's not just any guy; he's a former president.

    Campaigning in Iowa this weekend, Clinton said she couldn't think of "a better cheerleader for America" than her husband and wants him to help rebuild international friendships.

    The senator has been careful not to suggest that a vote for her is a two-for-one deal the way her husband did in 1992.

    But she has made it very clear that she will, as she says, put her high wattage husband to work as a sort of wandering ambassador of good will.

    "I love her very much and I think she would be a great president," Clinton said last week about his wife. "And all presidents need help. They need all the help they can get."

    It makes sense. When he was president, he loved to travel. With the development of his foundation, his globe-trotting has only intensified.

    "What makes Bill Clinton special is he wouldn't just [be] viewed as the the spouse of a president, he would be viewed as a…former world leader in his own right," said Wall Street Journal writer John Fund.

    Hillary Clinton is not talking about a formal post for the ex-president; family members are not allowed to be members of the Cabinet. But even an informal role could be tricky.

    "Bill Clinton has so much prominence and such charisma, he's in danger of overshadowing everyone around him," Fund said.

    Some wonder if there would be conflicts of interest. Since he left the White House, Clinton has raked in roughly $40 million for speaking engagements in 36 countries, including China, Colombia, Australia and Saudi Arabia.

    Last year, top officials in Dubai called him for advice on how to proceed with a deal to control U.S. ports, just as his wife was fighting against the very same deal.

tomder55 answered on 04/23/07:

The First Philanderer's world tour !! He's like the sailor looking for a port in every storm or was that a storm in every port. Certainly the idea of him being a cheerleader for the US is patently absurd. But I'm sure he'd do a representative job keeping the American end up .

Itsdb rated this answer Excellent or Above Average Answer

Question/Answer
Itsdb asked on 04/23/07 - How committed are you?

Since stopping global warming is all the talk now, how committed are you? Rocker Sheryl Crow makes the following suggestions:

    Although my ideas are in the earliest stages of development, they are, in my mind, worth investigating. One of my favorites is in the area of forest conservation which we heavily rely on for oxygen. I propose a limitation be put on how many squares of toilet paper can be used in any one sitting. Now, I don't want to rob any law-abiding American of his or her God-given rights, but I think we are an industrious enough people that we can make it work with only one square per restroom visit, except, of course, on those pesky occasions where 2 to 3 could be required. When presenting this idea to my younger brother, who's judgment I trust implicitly, he proposed taking it one step further. I believe his quote was, "how bout just washing the one square out."

    I also like the idea of not using paper napkins, which happen to be made from virgin wood and represent the height of wastefulness. I have designed a clothing line that has what's called a "dining sleeve." The sleeve is detachable and can be replaced with another "dining sleeve," after usage. The design will offer the "diner" the convenience of wiping his mouth on his sleeve rather than throwing out yet another barely used paper product. I think this idea could also translate quite well to those suffering with an annoying head cold.


Reportedly, Sheryl touched Karl Rove over the weekend and he objected, can she not figure out why?

Someone mentioned an Army directive on conserving toilet paper from the sixties this morning that might be helpful in deciding which way to vote:

    Take one square of toilet paper, fold it in half and then in half again so you have an equal square, tear off one corner to create a hole for your middle finger for traction.


OK, so who's in, one square of toilet paper per sitting?

Wearing snot on your sleeves, yes or no?

Bonus question, does anyone know what Sheryl's guitars are made of?

tomder55 answered on 04/23/07:

She should be happy to know that I frequently wipe my mouth with my sleeve ;wash my hands and wipe them on my shirt . I wonder how it would go over if she invited me to one of those fancy Brentwood gatherings so I can demonstrate for her eco-friendly swells. I bet I would be called (to quote Alec Baldwin) a pig .


bonus question ..... virgin forest wood and cat guts

Itsdb rated this answer Excellent or Above Average Answer

Question/Answer
paraclete asked on 04/23/07 - Muslim conversion has a secret agenda


Islamic group paying criminals to become Muslims

April 23, 2007 08:24am
Article from: The Daily Telegraph


AN Islamic group in Sydney's southwest has been paying the state's most dangerous criminals to become Muslims under the belief they could hatch a prison outbreak.

Authorities said yesterday an outside network from Bankstown was paying the prisoners - some of them murderers and rapists - to convert to Islam.

Intercepted calls and messages indicated the 12 converted prisoners thought the outside contacts could help them escape Goulburn's super max jail. The "Super Max Jihadists" have been organising for 18 months under ringleader Bassam Hamzy.

Meetings, martyrdom

Speaking in Arabic and English, the gang holds regular meetings and talk about martyrdom.

The converted prisoners came to the attention of authorities when money was moved between their bank accounts and outside contacts.

Hamzy, jailed for 21 years for murder, has been separated from the gang and sent to Lithgow jail where he is in isolation.

Conversion allowed

Corrective Services Commissioner Ron Woodham said the prisoners would still be allowed to practise Islam.

"We have known for quite some time there has been conversion not only in super max but in other correctional facilities," he said.

"It seemed innocent enough ... we later realised they were more organised than we realised."

Extreme high security

Hamzy, 28, has been declared extreme high security and will be allowed no outside contact as police conduct an investigation.

The other gang members have been ordered to speak only in English and have been banned from having visitors.

"Some of the people who have converted believe this outside network can assist them in an escape," Mr Woodham said.

A rift in the super max facility has since broken out between the gang and anti-Islamic prisoners such as contract killer Lindsay Rose, who is trying to re-convert the inmates.

Backpacker murder Ivan Milat has not joined the gang.

tomder55 answered on 04/23/07:

Our jails are also breeding grounds for jihadists . Something about radical Islamists and criminals is compatable.

paraclete rated this answer Excellent or Above Average Answer

Question/Answer
tropicalstorm asked on 04/22/07 - pc'ers at it again

Now they are complaining about Geico's caveman commerical's being insensitive to the dead. They
say just because cavemen are extinct doesn't give them the right to make fun of them.
WHAT NEXT????

tomder55 answered on 04/22/07:

What is the pc term anyway ? Cro-Magnon? Neanderthal? Neo sapien ? I guess the complaint is that it's so easy to offend that even a caveman commercial can do it.

If a cave man is so ubber-sensitive then what about other races (could they be code for other races )?????,

classes (the cave man orders roast duck with mango salsa at the restaurant...no doubt a subtle code for yuppies) ????,

genders(where are the female cavemen anyway ?)????? ,

or species (why do they use a cockney gecko ?) ????

all of whom have an ax of past discrimination to grind .

To the pc world the commericals may be making light of a very serious social problem. Remember ;we are living in a post-Imus world .My take on the ads however are a little different . To me they work because they poke fun at sterotyping and how we make fools of ourselves with presumptions.But maybe being a member of a group that gets descriminated against myself (angry white guy /christofascists;mackeral-smacking Romanist);perhaps I should be hyper-sensitive about the commercials also.

Best commercial :

One of the cavemen is on television being interviewed by a Bill OReilley type cable news anchorman. “How can it be offensive if it’s true?”, asks the presenter. “First of all, I’m not a hundred percent in love with your tone right now”, replies the caveman.

"Tone aside, historically, you guys have struggled to adapt”, retorts the commentator. “Right,” the caveman replies, “walking upright, discovering fire, inventing the wheel, laying the foundation for all mankind. You’re right. Good point. Sorry we couldn’t get that to you sooner.”

A female anchor comes onto the screen to say "someone woke up on the wrong side of the rock".



kindj rated this answer Excellent or Above Average Answer
labman rated this answer Excellent or Above Average Answer
paraclete rated this answer Excellent or Above Average Answer
tropicalstorm rated this answer Excellent or Above Average Answer

Question/Answer
Itsdb asked on 04/18/07 - Not so fast?

And I'm not speaking of Gov. Corzine...

    New turbulence in debate over effect of warming on hurricanes

    By Associated Press
    Tuesday, April 17, 2007 - Updated: 07:46 PM EST

    WASHINGTON - The debate over whether global warming affects hurricanes may be running into some unexpected turbulence.

    Many researchers believe warming is causing the storms to get stronger, while others aren’t so sure.

    Now, a new study raises the possibility that global warming might even make it harder for hurricanes to form.

    The findings, by Gabriel A. Vecchi of the National Oceanic and Atmospheric Administration and Brian J. Soden of the University of Miami, are reported in Wednesday’s issue of Geophysical Research Letters.

    Vecchi and Soden used 18 complex computer climate models to anticipate the effects of warming in the years 2001-2020 and 2018-2100.

    Included in the results were an increase in vertical wind shear over the tropical Atlantic and eastern Pacific oceans.

    Vertical wind shear is a difference in wind speed or direction at different altitudes. When a hurricane encounters vertical wind shear the hurricane can weaken when the heat of rising air dissipates over a larger area.

    On the other hand, warm water provides the energy that drives hurricanes, so warmer conditions should make the storms stronger.

    "We don’t know whether the change in shear will cancel out the increased potential from warming oceans, but the shear increase would tend to make the Atlantic and East Pacific less favorable to hurricanes," said Vecchi, of NOAA’s Geophysical Fluid Dynamics Laboratory in Princeton, N.J.

    "Which one of the two _ warming oceans or increasing shear _ will be the dominant factor? Will they cancel out? We and others are currently exploring those very questions, and we hope to have a better grasp on that answer in the near future," Vecchi said.

    "What we can say is that the magnitude of the shear change is large enough that it cannot be ignored," he added.

    Any decrease in strength or frequency of storms caused by shear would apply only if all else was equal, Vecchi said, "but all else is not equal, since the shear increase is being driven by global warming."

    Soden, of Miami’s Rosenstiel School for Marine and Atmospheric Science, added: "This study does not in any way undermine the widespread consensus in the scientific community about the reality of global warming."

    The massive destruction caused by Hurricanes Katrina and Rita in 2005 focused attention on tropical cyclones _ as these storms are also known _ and some well-known researchers suggested the warming seas were fueling stronger storms.

    Last year an El Nino _ a warming of the water in the tropical Pacific that can affect weather worldwide _ dampened the Atlantic hurricane season.


Gee, and here I thought all this global warming stuff was already figured out. Now we have "unexpected turbulence." Note the article says things such as:

"aren’t so sure"
"raises the possibility"
"On the other hand"
"should"
"We don’t know"
"which ... will be the dominant factor?"
"only if"
"researchers suggested"

And my favorite, "complex computer climate models"

Comments?

tomder55 answered on 04/18/07:

What do we have ;about a generation of seriously monitoring hurricane activity by satellite ? On what basis do they even create complex models from ? It's my opinion that if we hadn't built New Orleans in a sink hole that very few people in America would even remember the name Katrina . Quick ...what were the names of the other major hurricanes from that season ????

So this is what science is reduced to ..... If they are right they say ;just as we predicted global warming caused increased hurricane activity .But if it doesn't happen ;like last year ; then they say ...see global warming caused wind shear. They got all bases covered . Good job!!!

ETWolverine rated this answer Excellent or Above Average Answer
Itsdb rated this answer Excellent or Above Average Answer

Question/Answer
Itsdb asked on 04/18/07 - Iran-made arms seized in Afghanistan

WASHINGTON -- U.S. forces recently intercepted Iranian-made weapons intended for Taliban fighters in Afghanistan, the Pentagon's top general said Tuesday, suggesting wider Iranian war involvement in the region.

It appeared to be the first publicly disclosed instance of Iranian arms entering Afghanistan, although it was not immediately clear whether the weapons came directly from Iran or were shipped through a third party.

Gen. Peter Pace, chairman of the Joint Chiefs of Staff, said that unlike in Iraq, where U.S. officials say they are certain that arms are being supplied to insurgents by Iran's secretive Quds Force, the Iranian link in Afghanistan is murky.

"It is not as clear in Afghanistan which Iranian entity is responsible, but we have intercepted weapons in Afghanistan headed for the Taliban that were made in Iran," Pace told reporters over breakfast.

He said the weapons, including mortars and C-4 plastic explosives, were intercepted in Kandahar province within the past month. He did not describe the quantity of the materials or say whether it was the first time U.S. forces had found Iranian-made arms in Afghanistan.

Asked about Pace's remarks, a Pentagon spokesman, Army Col. Gary Keck, said he had not heard of previous instances of Iranian weaponry being found in Afghanistan but he was not certain this was the first time.

Iran has had an uneven relationship with Afghanistan over the years. During the wars of the past quarter-century -- the 1979-89 Soviet occupation, the subsequent civil war, Taliban rule starting in 1996 and the 2001 U.S.-led invasion -- millions of Afghans, particularly from the western provinces, took refuge in Iran.

In a statement responding to Pace's comments, the Paris-based National Council of Resistance of Iran, a coalition of Iranian opposition groups, said the Quds Force has been active in Afghanistan for years.

Mohammad Mohaddessin, chairman of the Iranian group's foreign affairs committee, said, "Export of fundamentalism and terrorism to neighboring and Islamic countries has been one of the pillars of the clerical regime's foreign policy -- something that the Iranian resistance has warned about for the past two decades."

Perhaps we need to export a little something to Iran?

tomder55 answered on 04/18/07:

Gee ;now why would Shia Iran be cooperating with the Sunni Taliban ? Maybe the deliniations are not as clear cut as some would have us believe ? Previously, based on captured Iranian documents in Irbil ,we learned that Iran also assisted Sunni groups, and the name at the top of that list was Al Qaida in Iraq. At the time the report came out, detractors quickly pointed out that a Shia state would not help a Sunni group, nor that a Sunni group who slaughters as many Shia as possible and considers all Shia to be apostates would want a Shia state’s help.

I will be interested to see the reaction by our dhimmicrats .They claim to support the war effort in Afghanistan . I wonder if they will now support us taking action against a nation that is arming our enemies there ? I kinda doubt it.

Itsdb rated this answer Excellent or Above Average Answer

Question/Answer
kindj asked on 04/17/07 - Concerning the Virginia Tech tragedy...

I've posted this before, but in light of the current tragedy I feel compelled to post it again. One small comment of my own follow the article.
~~~~~~~~~~~~~~~~~~~~~~~~~~~~~~~~~~~~~~~~~~~~~~~~~~~
On Sheep, Wolves, and Sheepdogs - Dave Grossman
By LTC (RET) Dave Grossman, author of "On Killing."
Honor never grows old, and honor rejoices the heart of age. It does so because honor is, finally, about defending those noble and worthy things that deserve defending, even if it comes at a high cost. In our time, that may mean social disapproval, public scorn, hardship, persecution, or as always,even death itself. The question remains: What is worth defending? What is worth dying for? What is worth living for? - William J. Bennett - in a lecture to the United States Naval Academy November 24, 1997

One Vietnam veteran, an old retired colonel, once said this to me:

"Most of the people in our society are sheep. They are kind, gentle, productive creatures who can only hurt one another by accident." This is true. Remember, the murder rate is six per 100,000 per year, and the aggravated assault rate is four per 1,000 per year. What this means is that the vast majority of Americans are not inclined to hurt one another. Some estimates say that two million Americans are victims of violent crimes every year, a tragic, staggering number, perhaps an all-time record rate of violent crime. But there are almost 300 million Americans, which means that the odds of being a victim of violent crime is considerably less than one in a hundred on any given year. Furthermore, since many violent crimes are committed by repeat offenders, the actual number of violent citizens is considerably less than two million.

Thus there is a paradox, and we must grasp both ends of the situation: We may well be in the most violent times in history, but violence is still remarkably rare. This is because most citizens are kind, decent people who are not capable of hurting each other, except by accident or under extreme provocation. They are sheep.

I mean nothing negative by calling them sheep. To me it is like the pretty, blue robin's egg. Inside it is soft and gooey but someday it will grow into something wonderful. But the egg cannot survive without its hard blue shell. Police officers, soldiers, and other warriors are like that shell, and someday the civilization they protect will grow into something wonderful.? For now, though, they need warriors to protect them from the predators.

"Then there are the wolves," the old war veteran said, "and the wolves feed on the sheep without mercy." Do you believe there are wolves out there who will feed on the flock without mercy? You better believe it. There are evil men in this world and they are capable of evil deeds. The moment you forget that or pretend it is not so, you become a sheep. There is no safety in denial.

"Then there are sheepdogs," he went on, "and I'm a sheepdog. I live to protect the flock and confront the wolf."

If you have no capacity for violence then you are a healthy productive citizen, a sheep. If you have a capacity for violence and no empathy for your fellow citizens, then you have defined an aggressive sociopath, a wolf. But what if you have a capacity for violence, and a deep love for your fellow citizens? What do you have then? A sheepdog, a warrior, someone who is walking the hero's path. Someone who can walk into the heart of darkness, into the universal human phobia, and walk out unscathed

Let me expand on this old soldier's excellent model of the sheep, wolves, and sheepdogs. We know that the sheep live in denial, that is what makes them sheep. They do not want to believe that there is evil in the world. They can accept the fact that fires can happen, which is why they want fire extinguishers, fire sprinklers, fire alarms and fire exits throughout their kids' schools.

But many of them are outraged at the idea of putting an armed police officer in their kid's school. Our children are thousands of times more likely to be killed or seriously injured by school violence than fire, but the sheep's only response to the possibility of violence is denial. The idea of someone coming to kill or harm their child is just too hard, and so they chose the path of denial.

The sheep generally do not like the sheepdog. He looks a lot like the wolf. He has fangs and the capacity for violence. The difference, though, is that the sheepdog must not, can not and will not ever harm the sheep. Any sheep dog who intentionally harms the lowliest little lamb will be punished and removed. The world cannot work any other way, at least not in a representative democracy or a republic such as ours.

Still, the sheepdog disturbs the sheep. He is a constant reminder that there are wolves in the land. They would prefer that he didn't tell them where to go, or give them traffic tickets, or stand at the ready in our airports in camouflage fatigues holding an M-16. The sheep would much rather have the sheepdog cash in his fangs, spray paint himself white, and go, "Baa."

Until the wolf shows up. Then the entire flock tries desperately to hide behind one lonely sheepdog.

The students, the victims, at Columbine High School were big, tough high school students, and under ordinary circumstances they would not have had the time of day for a police officer. They were not bad kids; they just had nothing to say to a cop. When the school was under attack, however, and SWAT teams were clearing the rooms and hallways, the officers had to physically peel those clinging, sobbing kids off of them. This is how the little lambs feel about their sheepdog when the wolf is at the door.

Look at what happened after September 11, 2001 when the wolf pounded hard on the door. Remember how America, more than ever before, felt differently about their law enforcement officers and military personnel? Remember how many times you heard the word hero?

Understand that there is nothing morally superior about being a sheepdog; it is just what you choose to be. Also understand that a sheepdog is a funny critter: He is always sniffing around out on the perimeter, checking the breeze, barking at things that go bump in the night, and yearning for a righteous battle. That is, the young sheepdogs yearn for a righteous battle. The old sheepdogs are a little older and wiser, but they move to the sound of the guns when needed right along with the young ones.

Here is how the sheep and the sheepdog think differently. The sheep pretend the wolf will never come, but the sheepdog lives for that day. After the attacks on September 11, 2001, most of the sheep, that is, most citizens in America said, "Thank God I wasn't on one of those planes." The sheepdogs, the warriors, said, "Dear God, I wish I could have been on one of those planes. Maybe I could have made a difference." When you are truly transformed into a warrior and have truly invested yourself into warriorhood, you want to be there. You want to be able to make a difference.

There is nothing morally superior about the sheepdog, the warrior, but he does have one real advantage. Only one. And that is that he is able to survive and thrive in an environment that destroys 98 percent of the population. There was research conducted a few years ago with individuals convicted of violent crimes. These cons were in prison for serious, predatory crimes of violence: assaults, murders and killing law enforcement officers. The vast majority said that they specifically targeted victims by body language: slumped walk, passive behavior and lack of awareness. They chose their victims like big cats do in Africa, when they select one out of the herd that is least able to protect itself.

Some people may be destined to be sheep and others might be genetically primed to be wolves or sheepdogs. But I believe that most people can choose which one they want to be, and I'm proud to say that more and more Americans are choosing to become sheepdogs.

Seven months after the attack on September 11, 2001, Todd Beamer was honored in his hometown of Cranbury, New Jersey. Todd, as you recall, was the man on Flight 93 over Pennsylvania who called on his cell phone to alert an operator from United Airlines about the hijacking. When he learned of the other three passenger planes that had been used as weapons, Todd dropped his phone and uttered the words, "Let's roll," which authorities believe was a signal to the other passengers to confront the terrorist hijackers. In one hour, a transformation occurred among the passengers - athletes, business people and parents. -- from sheep to sheepdogs and together they fought the wolves, ultimately saving an unknown number of lives on the ground.

Here is the point I like to emphasize, especially to the thousands of police officers and soldiers I speak to each year. In nature the sheep, real sheep, are born as sheep. Sheepdogs are born that way, and so are wolves. They didn't have a choice. But you are not a critter. As a human being, you can be whatever you want to be. It is a conscious, moral decision.

If you want to be a sheep, then you can be a sheep and that is okay, but you must understand the price you pay. When the wolf comes, you and your loved ones are going to die if there is not a sheepdog there to protect you. If you want to be a wolf, you can be one, but the sheepdogs are going to hunt you down and you will never have rest, safety, trust or love. But if you want to be a sheepdog and walk the warrior's path, then you must make a conscious and moral decision every day to dedicate, equip and prepare yourself to thrive in that toxic, corrosive moment when the wolf comes knocking at the door.

For example, many officers carry their weapons in church.? They are well concealed in ankle holsters, shoulder holsters or inside-the-belt holsters tucked into the small of their backs.? Anytime you go to some form of religious service, there is a very good chance that a police officer in your congregation is carrying. You will never know if there is such an individual in your place of worship, until the wolf appears to massacre you and your loved ones.

I was training a group of police officers in Texas, and during the break, one officer asked his friend if he carried his weapon in church. The other cop replied, "I will never be caught without my gun in church." I asked why he felt so strongly about this, and he told me about a cop he knew who was at a church massacre in Ft. Worth, Texas in 1999. In that incident, a mentally deranged individual came into the church and opened fire, gunning down fourteen people. He said that officer believed he could have saved every life that day if he had been carrying his gun. His own son was shot, and all he could do was throw himself on the boy's body and wait to die. That cop looked me in the eye and said, "Do you have any idea how hard it would be to live with yourself after that?"

Some individuals would be horrified if they knew this police officer was carrying a weapon in church. They might call him paranoid and would probably scorn him. Yet these same individuals would be enraged and would call for "heads to roll" if they found out that the airbags in their cars were defective, or that the fire extinguisher and fire sprinklers in their kids' school did not work. They can accept the fact that fires and traffic accidents can happen and that there must be safeguards against them.

Their only response to the wolf, though, is denial, and all too often their response to the sheepdog is scorn and disdain. But the sheepdog quietly asks himself, "Do you have and idea how hard it would be to live with yourself if your loved ones attacked and killed, and you had to stand there helplessly because you were unprepared for that day?"

It is denial that turns people into sheep. Sheep are psychologically destroyed by combat because their only defense is denial, which is counterproductive and destructive, resulting in fear, helplessness and horror when the wolf shows up.

Denial kills you twice. It kills you once, at your moment of truth when you are not physically prepared: you didn't bring your gun, you didn't train. Your only defense was wishful thinking. Hope is not a strategy. Denial kills you a second time because even if you do physically survive, you are psychologically shattered by your fear helplessness and horror at your moment of truth.

Gavin de Becker puts it like this in Fear Less, his superb post-9/11 book, which should be required reading for anyone trying to come to terms with our current world situation: "...denial can be seductive, but it has an insidious side effect. For all the peace of mind deniers think they get by saying it isn't so, the fall they take when faced with new violence is all the more unsettling."

Denial is a save-now-pay-later scheme, a contract written entirely in small print, for in the long run, the denying person knows the truth on some level.

And so the warrior must strive to confront denial in all aspects of his life, and prepare himself for the day when evil comes. If you are warrior who is legally authorized to carry a weapon and you step outside without that weapon, then you become a sheep, pretending that the bad man will not come today. No one can be "on" 24/7, for a lifetime. Everyone needs down time. But if you are authorized to carry a weapon, and you walk outside without it, just take a deep breath, and say this to yourself...

"Baa."

This business of being a sheep or a sheep dog is not a yes-no dichotomy. It is not an all-or-nothing, either-or choice. It is a matter of degrees, a continuum. On one end is an abject, head-in-the-sand-sheep and on the other end is the ultimate warrior. Few people exist completely on one end or the other. Most of us live somewhere in between. Since 9-11 almost everyone in America took a step up that continuum, away from denial. The sheep took a few steps toward accepting and appreciating their warriors, and the warriors started taking their job more seriously. The degree to which you move up that continuum, away from sheephood and denial, is the degree to which you and your loved ones will survive, physically and psychologically at your moment of truth.
~~~~~~~~~~~~~~~~~~~~~~~~~~~~~~~~~~~~~~~~~~~~~~~~~~~

So the only questions that remain are these:

1. What are you today, either by nature or by choice?

2. What will you be tomorrow, by choice?

As for me, I am damn proud to have been a sheepdog for years, and God willing, will continue to be one for years.

The predators in our world NEED us to be sheep, so they can control us. Nothing irritates them more than someone who REFUSES to be a victim.

In fact, I gotta say it's downright gratifying to see the look in a predator's eyes change from aggression to fear as he realizes that HE is now the HUNTED.

DK

tomder55 answered on 04/18/07:

glad you reposted it . I forgot to make a copy last time.

If only some trained instructors like yourself were permitted to carry I imagine these events would not be as terrible.

kindj rated this answer Excellent or Above Average Answer

Question/Answer
curious98 asked on 04/17/07 - Virginia massacre


I wish to express from here my most sincere condoleances to the families, relatives and friends of the victims of the shooting rampage yesterday at the Virginia Tech University, which is already qualified as the deadliest attack of its kind in the USA.
I know your Constitution, as per the 2nd Amendment, guarantees everyone the right to carry a weapon..
Last month, a friend of mine and his wife went to visit their son in Newark (N.J) and they flew directly from Barcelona to that city.
They had to go through a considerable amount of police controls at the Barcelona airport, which makes sense to avoid any problems while flying. But when they went through the police at the Newark airport they had to go through all kind of security controls, like pictures, fingerprints plus having their entire luggage opened, removing their shoes, etc.
Theoretically, at least, they were basically looking for small weapons of some kind… which makes sense… except when you start considering how easy is for anyone in your country to get as many weapons and ammunition as he/she/they can afford to pay for.
Any terrorist does not have to go to the trouble of trying to sneak any weapons in your country, when they can be simply bought there…
And it must be real simple, as otherwise how can it be explained that this South Korean fellow could avail himself of 2 guns and enough ammo., to accomplish his deadly task?
In view of all previous letal experiences would not it be about time to start reconsidering this 2nd Amendment?
Curious98

tomder55 answered on 04/17/07:

Why don't you wait for the facts to come out before you make presumptions ?

The criminals are not legally obtaining guns in this country. Law abiding citizens have the right to obtain guns and go through hoops of controls to get them including backround checks. I still say what happened could've been much less a tragedy if the school did not adopt a zero tolerance policy about students having guns on campus.

As to the last question ;I consider the right to defend myself a fundamental right of all humans . It not only protects us from vultures who would do us harm but more important guards our rights to defend ourselves from the tyrany of government . WE in America choose not to be sheeple .

CeeBee2 rated this answer Excellent or Above Average Answer
curious98 rated this answer Average Answer
ETWolverine rated this answer Excellent or Above Average Answer

Question/Answer
Itsdb asked on 04/16/07 - Bad advice?

Maybe Imus should have consulted some of these folks before talking to Al Sharpton...

1996 Pulitzer Prize finalist cartoonist Ted Rall, calls Condoleeza Rice Bush's "house nigga" in a cartoon:



In 2005, black, left-wing blogger Steve Gilliard posts a photo of Lt. Gov. Michael S. Steele as Sambo:



Also in 2005, USA Today publishes a doctored photo of Condoleeza Rice making her appear like something out of "The Omen."



To this day, Washington Monthly magazine has posts on Kevin Drum's "Political Animal" blog from 2005 calling Michelle Malkin a "cunt" and a "whore."

    Michelle Malkin is a cunt.
    Posted by: hostile on April 7, 2005 at 11:17 AM | PERMALINK

    Michelle Malkin is a cunt.

    Check your talking points, dude. The Official Left-wing Anti-Malkin punch-word is "whore."
    Posted by: Once a dem on April 7, 2005 at 11:21 AM


Drum believes it's important to allow such comments to stand, even though his "comment section might be full of trolls and their vitriol."

I guess it's just all in who you are and who you insult.

tomder55 answered on 04/17/07:

Emerge, (a defunct black news monthly) once put on its cover a caricature of a grinning Clarence Thomas dressed as a lawn jockey, holding a lantern . The headline ... "Uncle Thomas, Lawn Jockey for the Far Right." Inside the mag was a cartoon of a kneeling Thomas shining the shoes of Antonin Scalia.

Pat Oliphant depicts Condi Rice as a parrot with exagerated black physical features . One cartoon has Bush saying "How woodums wike to be secwetawy of state?" to which Rice responds, "Awwrk!! OK, chief. Anything you say, chief. You bet, chief! You're my hero, chief!"

It is a complete racial insult to someone who Oliphant couldn't clean her shoes by comparison .

Hillary once joked that Mahatma Gandhi used to run a gas station in St. Louis.Joe Biden once said "You CANNOT go into a 7-11 or a Dunkin Donuts without an Indian accent."

Imus did everyone a favor . His remarks opened the pandoras box and out popped the hypocrites for all to see.









Itsdb rated this answer Excellent or Above Average Answer

Question/Answer
ETWolverine asked on 04/16/07 - I finally figured out the cause of global warming.

I have done a study of this issue, and I have come to the conclusion that the Liberals are right. Global Warming is indeed a man-made occurance. Furthermore, it is the fault of the Federal Government of the United States, just as the Libs have been saying for years.

The cause of global warming?

Taxes.

After having done a study of the issue, I have come to the conclusion that there is a positive corallary between taxes and temperature records.

First of all, just as temperatures have been going up for years, taxes have been going up as well. So there is the first correlation.

But the evidence is much stronger than that. I have done a review of Federal tax receipts as a percentage of GDP for the period of 1946 - 2006. I have also done are review of temperature records to obtain the average temperatures for the month of April (tax month) in Albany, NY. While temperature records for Federal tax collections were complete, the temperature records for Albany were missing 9 years worth of information. Nevertheless, despite the incompleteness of the data, I continued my study. (After all, if the pseudo-scientists who make claims of global warming can do so with huge amounts of data lacking, so can I.)

My study led to the following conclusion. Over the past 60 years, tax receipts as a percentage of GDP have gone up by 190 basis points. If we eliminate the years for which we do not have temperature records, the increase in taxes as a percentage of GDP increases by 210 basis points. During the same period, temperatures for the month of April have increased an average of 0.03 degrees Celcius. This shows a clear correlation... taxes up, temperatures up.

Furthermore, there were 23 cases where both temperatures and percetage of GDP moved in the same direction. That is, when taxes as a percentage of GDP went up, the temperature went up, and when taxes as a percentage of GDP went down, temperatures went down. That's 23 out of 51 times when there was congruity between the movement of taxes as a percentage of GDP and temperatures in Albany. This shows a clear correlation between taxes and global warming.

Since US Federal taxes are a function of the US government, temperature change must also be a function of the US government. This would mean that the US government is at fault for Global warming, just as the Liberals have stated.

There are two clear solutions to global warming. The first would be to increase GDP while holding taxes at their current level. We need to increase the productivity of the United States so that taxes become a smaller percentage of GDP, thus driving environmental temperatures down. Of course, this would require an increase in our industrial performance and capacity. But since we have now proven that industrial emmissions aren't the real cause of global warming, that shouldn't be too much of a problem.

The second solution would be to hold GDP steady, but lower taxes. This too would result in taxes being a lower percentage of GDP. This might sound easier than the first solution... after all, it just takes a vote of Congress to make that happen. However, in reality, getting Congress to agree to lower taxes is never an easy task. It is easier to build thousands of new industrial plants than it is to get Congress to cut taxes. Nevertheless, it might be time for Congress to take the hard actions necessary to protect the world from tax-driven global warming. The US government has a responsibility to act.

Below are the data used to come to the above conclusions.

.......Taxes..%Chng..Albany..Chng in
Year...% GDP...GDP.. Temp....Temp.
1947...16.5...-1.1...6.4.....-0.7
1948...16.2...-0.3...8.5......2.1
1949...14.5...-1.7...9.1......0.6
1950...14.4...-0.1...6.3.....-2.8
1951...16.1....1.7...8.6......2.3
1952...19......2.9...10.2.....1.6
1953...18.7...-0.3...8.0.....-2.2
1954...18.5...-0.2...8.9......0.9
1955...16.6...-1.9...9.8......0.9
1956...17.5....0.9...5.8.....-4.0
1957...17.8....0.3...9.4......3.6
1958...17.3...-0.5...9.5......0.1
1959...16.1...-1.2...9.1.....-0.4
1960...17.9....1.8...9.8......0.7
1961...17.8...-0.1...6.8.....-3.0
1962...17.6...-0.2...8.7......1.9
1963...17.8....0.2...7.9.....-0.8
1964...17.6...-0.2...8.0......0.1
1965...17.0...-0.6...N/A
1966...17.4....0.4...N/A
1967...18.3....0.9...N/A
1968...17.7...-0.6...N/A
1969...19.7....2.0...N/A
1970...19.0...-0.7...N/A
1971...17.3...-1.7...N/A
1972...17.6....0.3...N/A
1973...17.7....0.1...9.2
1974...18.3....0.6...8.9.....-0.3
1975...17.9...-0.4...4.9.....-4.0
1976...17.2...-0.7...9.9..... 5.0
1977...18.0....0.8...8.6.....-1.3
1978...18.0....0.0...6.2.....-2.4
1979...18.5....0.5...7.7......1.5
1980...19.0....0.5...8.9......1.2
1981...19.6....0.6...9.1......0.2
1982...19.1...-0.5...7.2.....-1.9
1983...17.5...-1.6...8.0......0.8
1984...17.4...-0.1...8.8......0.8
1985...17.7....0.3...9.6......0.8
1986...17.4...-0.3...10.4.....0.8
1987...18.4....1.0...10.3....-0.1
1988...18.2...-0.2...8.0.....-2.3
1989...18.4....0.2...6.9.....-1.1
1990...18.0...-0.4...9.5......2.6
1991...17.8...-0.2...10.6.....1.1
1992...17.5...-0.3...7.0.....-3.6
1993...17.6....0.1...9.0......2.0
1994...18.1....0.5...9.0......0.0
1995...18.5....0.4...6.7.....-2.3
1996...18.9....0.4...7.9......1.2
1997...19.3....0.4...7.0.....-0.9
1998...20.0....0.7...9.5......2.5
1999...20.0....0.0...8.6.....-0.9
2000...20.9....0.9...7.4.....-1.2
2001...19.8...-1.1...8.7......1.3
2002...17.9...-1.9...9.9......1.2
2003...16.5...-1.4...7.0.....-2.9
2004...16.3...-0.2...9.3......2.3
2005...17.6....1.3...10.1.....0.8
2006...18.4....0.8...10.0....-0.1

Average Change 0.19..........0.03

Hey, it makes about as much sense as any argument the pseudo-scientific knuckleheaded environ-mental cases put out.

Elliot

tomder55 answered on 04/17/07:

When you put those numbers on a graph does it look like a hockey stick ? Elliot; I can understand doing all that statistical analysis for pay .... but is it also your hobby ?

All I know is that I have to plan my escape route tonight to get home from work . All the rivers around here are rising ,and already I had to take a serpentine route into work this morning to avoid closed roads. Now I hear that it may snow by the end of the week . As Sapph keeps on saying ;send some of that global warming my way. I am at least 2 weeks behind in my gardening work .

ETWolverine rated this answer Excellent or Above Average Answer

Question/Answer
paraclete asked on 04/17/07 - It appears the US needs l'ttle ol' us

to maintain its status as a "world power"?

Now that is bizzaire

Israel PM urges Australia Iraq stay
April 17, 2007 - 12:03PM

Australian and US troops have to leave Iraq some time, but should do so in a way which maintained the reputation of the US as a major world power, Israel's prime minister said today.

Ehud Olmert said he did not want to revisit whether the US should have gone into Iraq in the first place.

"You are there. Others are there. The question is how should America pull out and the other forces should pull out from Iraq," Mr Olmert told ABC radio from Jerusalem.

"I think they should pull out in a manner, which will maintain the prestige and the perception of America as a major power.

"Why? Because if the perception will be different then the possible consequences and ramifications will be very unpleasant for the moderate forces in our part of the world. And that is something we are worried about."

Mr Olmert said he was not saying that US and Australia and other forces should never leave Iraq.

"What I say is it is your decision to make. It is America's decision to make," he said.

"What I just tell you as someone that lives in this part of the world for so many years and has known a little bit about the dynamics of this part of the world, is that having known what you have invested for so many years, how much it has cost you.

"Just think of what will be the possible ramifications of a premature pullout on the standing of the forces which are essential for our interests and your interests and your security in this part of the world."

AAP


tomder55 answered on 04/17/07:

I would say that it has been the anglo-sphere that has been the guardians at the Gates of Vienna and have been most responsible for preventing the world from plunging into the abyss of a return to the dark ages.

"I think they should pull out in a manner, which will maintain the prestige and the perception of America as a major power.

"Why? Because if the perception will be different then the possible consequences and ramifications will be very unpleasant for the moderate forces in our part of the world. And that is something we are worried about."


Fogetting our prestige ;I think it would be the height of irresponsibly to leave Iraq without a suitable security arrangement alternative in place. I don't know or understand why Olmert would argue otherwise. An Iraq that is a terrorist haven is definitely not in Israel's interests .

paraclete rated this answer Excellent or Above Average Answer

Question/Answer
kindj asked on 04/13/07 - Concerning a previous question

The Navy's "directive" on t-shirts.

I seriously doubt the USN issued such a directive, but there are HUGE chunks of our population that would LOVE to see such an order given.

To them, I dedicate the following Ted Nugent song:

"Don't waste your time on me
I got my own direction
Watch me close, wait and see
I'm lookin' for perfection
I make up my own mind
And I'll leave you far behind
When the goin' gets tough
You can kiss my ass

I believe in animal rights
I let my dog hump on my shin
I can tolerate sexual choice
But not with the next of kin (uh-uh, boy)
I've heard it all before
I ain't gonna take any more
When the goin' gets tough
You can kiss my ass

Everybody!
Kiss my ass, (pucker up)
Kiss my ass
Kiss my ass
Kiss my ass, (c'mon babe)

I've heard it all before
And I ain't gonna take no more, no no
When the goin' gets tough
Kiss my ass!

I see the weenies with the dirty hair
Protestin' on the street
They condemn the clothes we wear
And the morality of what we eat, yeah
It's gotta be a fluke
They make me wanna puke
When the goin' gets tough
They can kiss my ass

Everybody gotta
Kiss my ass - C'mon gang bangers
Kiss my ass - Janet Reno
Kiss my ass - C'mon Billary
Kiss my ass - Callin' on Jesse Jackson
Kiss my ass - How about the IRS
Kiss my ass - Hey, Howard Stern
Kiss my ass - United Nations
Kiss my ass - All those Liberals
Kiss my ass - C'mon Sarah Brady
Kiss my ass - Oh, Courtney Love I've got your hole
Kiss my ass - Beavis, Butthead
Kiss my ass - How about Crips and Bloods"


To Elliot:

I LOVED that song! Got it saved on my classroom computer right now. Too bad I can't play it when students are around. As for me, if I were to ride back into the darkness today, I think I'd take this little ditty by Manowar with me:

CALL TO ARMS

When they see us they will run for their lives
To the end they will pay for their lies
So long did we wait, now we are home

Here once again there's a battle to fight
Gather together for the sound and the might
So long did we wait, now we are home

Now we will fight for the kingdom, fighting with steel
Kill all of them, their blood is our seal
Fight till the last of the enemy is dead
Ride through their blood that we gladly have shed

I now issue the call, are you ready to fight - yeah
Fight altogether as one for the right
To be free once again - tonight we will win

I can see by the look that you have in your eyes
You came here for metal, to fight and to die
Defenders of steel, now we are home

Fight for the kingdom bound for glory
Armed with a heart of steel
I swear by the brothers who stand before me
To no man shall I kneel
Their blood is upon my steel


Let the terrorists suck on THAT one for a while!

DK


tomder55 answered on 04/13/07:

Have heard the Nugent song before . Most of the sites I went on said the Navy Directive wasn't real ...but had enough of the truth in it to be believable .

kindj rated this answer Excellent or Above Average Answer
ETWolverine rated this answer Excellent or Above Average Answer
Itsdb rated this answer Excellent or Above Average Answer

Question/Answer
Itsdb asked on 04/13/07 - A Nappy Hair Affair

A Nappy Hair Affair is best known for sponsoring grass roots hair grooming sessions called Hair Days. These are gathering where sisters come together with others who understand their natural hair care needs and their choice to embrace a style more in keeping with their culture. Hair Days, which have fostered a spirit of bonding and support, have become so popular since I held the first one at my home in May, 1998 that they are being held in several U.S. cities. We also have a Hair Day network in other countries.

But A Nappy Hair Affair (ANHA) is about so much more than hair. It is about reclaiming and respecting our culture. We have been conditioned to hate one of our most unique characteristics-our hair in its most natural state. We have been conditioned to accept European standards of beauty and to reject our own. ANHA exists to cause a shift in such negative mindsets and promote a positive image of people of African descent. We do it through support, affirmation and education.

But Hair Days are our hallmark. These simple gatherings where sisters and brothers symbolically celebrate their culture by nurturing their natural hair have had powerful and healing effects.

One of our most devoted members is Alpha Thomas who is a breast cancer survivor. Her story of losing her locks while undergoing chemotherapy but not losing her spirit has been an inspiration for other women in our group. Alpha's locks have grown back with a vengeance, and I count on her as my unofficial counselor for other women who have come to me with similar challenges. We welcome you to become a part of our nappy nation where we are very much into celebrating who we really are.

Nappily,
Mosetta
~~~~~~~~~~~~~~~~~~~~~~~~~~~~~~~~~~~~~~~~~~~~~~~~~~~~~~~

I am not making this up. You can buy her book Nappyisms, and a CD, Love and Nappiness, and apparently someone has made a Nappy T-shirt, which appears to read "I'm nappy, happy and free."



I wonder if Mosetta will call the Rutgers gals and encourage them to celebrate their "nappiness?"

tomder55 answered on 04/13/07:

Then there is this rap group called 'Nappy Roots' . They have this self appreciating album called 'Watermelon, Chicken and Gritz'.Some of the tracs that celebrate the culture are : Hustla ;Ballin' on a Budget;and Ho Down .

Itsdb rated this answer Excellent or Above Average Answer

Question/Answer
Itsdb asked on 04/12/07 - Racist bile

I'm sure you've all heard about or seen the O'Reilly/Geraldo smackdown, and Joanne Ostrow,
Denver Post Television Critic has defined for us what "racist bile" is.

O'Reilly invited Ms. Ostrow to his program and she declined, so he sent his producer to talk to her. After asking her several times what "racist bile" O'Reilly had 'spewed' she finally boiled it down to this, he used the term "illegal alien" instead of "undocumented immigrant." There you have it, another phrase bites the dust - in spite of its accuracy, and in spite of the fact there is no indication of race in using the term.

Illegal - prohibited by law
Alien - foreign, owing allegiance to another country

Seems straightforward to me, but then what do I know, I'm apparently just a racist. And oh yea, I watched the video and read the transcript and the only one who used the term 'illegal alien(s)' was Geraldo - 8 times. One more episode of a liberal member of the media creating their own reality at the expense of the truth and someone else's reputation.

tomder55 answered on 04/12/07:

CAN YOU GET ME IN?
Words & Music by Bob Haworth
Copyright 2006 - Three Cats Music, BMI
All Rights Reserved

WELL, I WROTE MY CONGRESSMAN TO LET HIM KNOW
THAT I’M THINKIN’ ‘BOUT HEADIN’ DOWN TO MEXICO
AND I WAS HOPIN’ HE COULD PULL SOME STRINGS
TO GET ME IN
YA SEE, I DON’T WANNA BOTHER WITH LEGALITIES
NO PASSPORTS OR VISAS, IF YOU PLEASE
I’M JUST ASKIN’ FOR THE SAME DEAL THERE
THAT WE GIVE THEM

CHORUS:
AND I SAID,
“CAN YOU GET ME IN? CAN YOU GET ME IN?
I MIGHT STAY FOR AWHILE AND PRETEND I’M A CITIZEN
BUT I DON’T WANNA LEARN THEIR NATIVE TONGUE
AND I WON’T PAY TAXES – THAT’S NO FUN!
I JUST WANNA START A BRAND NEW LIFE
IF YOU CAN GET ME IN”

NOW I PLAN TO TAKE THE WHOLE FAMILY
ALL MY COUSINS, MY SIBLINGS AND EVEN AUNT BEA
WE’D ALL LIKE JOBS AND A REAL NICE PLACE TO LIVE
I WANT MY KIDS IN AN ENGLISH SPEAKING SCHOOL
I WANT FOOD STAMPS AND HEALTH CARE – I’M NO FOOL
‘CAUSE ALL THAT STUFF IS MY PREROGATIVE!

CHORUS: SO, CAN YOU GET ME IN? …etc.


WELL, LAW AND ORDER ACROSS THE BORDER
IT DON’T MEAN A THING TO ME
IT’S SO EASY WHEN THE WHEELS ARE GREASY
JUST OPEN THE GATE AND TOSS AWAY THE KEY

NOW ON THE 4TH OF JULY I PLAN TO MAKE SOME NOISE
MIGHT THROW A LITTLE PARTY FOR ME AND THE BOYS
WE’LL GO WAVIN’ OUR AMERICAN FLAG ALL OVER TOWN
WE’LL DRINK BUD AND COORS AND HAVE BARBECUE
TALK BASEBALL, APPLE PIE AND MOTHERHOOD TOO
AND WE DON’T WANT THE LOCAL POPULATION
TRYIN’ TO PUT US DOWN

CHORUS:
SO CAN YOU GET ME IN? CAN YOU GET ME IN?
I MIGHT STAY FOR AWHILE AND PRETEND I’M A CITIZEN
BUT I DON’T WANNA LEARN NO ESPANIOLE
PRESS “ONE” FOR ENGLISH, THAT’S ALL I KNOW
I JUST WANNA START A BRAND NEW LIFE
IF YOU CAN GET ME IN
/////////////////////////////////////

Geraldo MAY have had a point if this was the first time an innocent lost their life due to the lawless activity of an illegal alien .Sadly that is hardly the case . Local and State law enforcement look the other way because they claim it's a Federal concern.The Feds. hardly bother at all. Rivera can ignore the immigrant factor of the case all he wants to but that does not change the reality .Ramos would not have been driving drunk on American streets if laws were enforced . Instead he cynically claimed we "lure "them here . Well not really ,but the OReilley v. Geraldo shout out shows just how far from a serious debate we are really having on the issue .

Itsdb rated this answer Excellent or Above Average Answer

Question/Answer
otoka asked on 04/11/07 - theatre and performance

proscenium theatre

tomder55 answered on 04/11/07:

not sure if there is a question here.

That is the typical construct of a theater with the stage "framed " so the audience sits in front (the house) and from their vantage point are looking through to a room where the play is being performed . Most Broadway stages are set up this way.

ETWolverine rated this answer Excellent or Above Average Answer
otoka rated this answer Above Average Answer

Question/Answer
tropicalstorm asked on 04/11/07 - LOL

driving directions from New york to London

1. Head southwest on Broadway toward Warren St 0.2 mi
1 min

2. Turn left at Park Row 0.1 mi
1 min

3. Slight right at Frankfort St 0.3 mi
1 min

4. Turn left at Pearl St 56 ft

5. Turn right onto the F.D.R. Dr N ramp 0.4 mi
1 min

6. Merge onto FDR Dr N 7.7 mi
12 mins

7. Take exit 17 on the left for Triboro Bridge/Grand Central Pkwy toward I-278/Bruckner Expy 0.4 mi
2 mins

8. Merge onto Triborough Bridge
Partial toll road 0.4 mi
1 min

9. Merge onto I-278 E via the ramp to I-87 N/Bronx/Upstate N Y/New England 0.6 mi
1 min

10. Take exit 47 to merge onto Bruckner Expy/I-278 E toward New Haven 1.9 mi
2 mins

11. Take the I-278 E exit toward New Haven 0.3 mi

12. Merge onto Bruckner Expy 5.0 mi
6 mins

13. Continue on I-95 N
Partial toll road
Entering Connecticut 62.1 mi
1 hour 12 mins

14. Take exit 48 on the left to merge onto I-91 N toward Hartford 36.8 mi
37 mins

15. Take exit 29 for US-5 N/CT-15 toward I-84/E Hartford/Boston 0.4 mi

16. Merge onto CT-15 N 1.7 mi
2 mins

17. Merge onto I-84 E
Partial toll road
Entering Massachusetts 40.7 mi
38 mins

18. Take the exit onto I-90 E/Mass Pike/Massachusetts Turnpike toward N.H.-Maine/Boston
Partial toll road 56.0 mi
56 mins

19. Take exit 24 A-B-C on the left toward I-93 N/Concord NH/S Station/I-93 S/Quincy 0.4 mi
1 min

20. Merge onto Atlantic Ave 0.8 mi
3 mins

21. Turn right at Central St 0.1 mi

22. Turn right at Long Wharf 0.1 mi

23. Swim across the Atlantic Ocean 3,462 mi
29 days 0 hours

...
24. Slight right at E05 0.5 mi
2 mins

25. At the traffic circle, take the 2nd exit onto E05/Pont Vauban 0.1 mi

26. Turn right at E05 5.7 mi
10 mins

27. Take the exit onto A29/E44 toward Amiens
Toll road 27.8 mi
23 mins

28. Take the exit toward Dieppe/Amiens/Calais/A151/Rouen
Toll road 1.1 mi
1 min

29. Merge onto A29/E44
Toll road 22.6 mi
19 mins

30. Take the exit onto A28/E402 45.6 mi
37 mins

31. Take the exit onto A16/E402 toward Boulogne/Calais
Toll road 44.3 mi
38 mins

32. Take exit 29 toward Boulogne-Centre/Outreau/Le Portel 0.6 mi
1 min

33. Merge onto N416 1.1 mi
1 min

34. At the traffic circle, take the 1st exit onto N1 0.4 mi
1 min

35. At the traffic circle, take the 2nd exit and stay on N1 0.1 mi
1 min

36. At the traffic circle, take the 2nd exit and stay on N1 0.9 mi
2 mins

37. At the traffic circle, take the 1st exit 0.6 mi
1 min

38. Slight left at Rue Ferdinand Farjon 427 ft

39. At the traffic circle, take the 2nd exit 0.4 mi
1 min

40. Slight right at Dover - Boulougne-sur-Mer 30.1 mi
1 hour 50 mins

41. Continue on Dover - Boulogne-sur-Mer 0.2 mi

42. Continue on Eastern Service Rd 0.3 mi
2 mins

43. Turn right at E Ramp 0.4 mi
2 mins

44. Slight right at Dock Exit Rd 0.1 mi

45. At Eastern Docks Roundabout, take the 2nd exit onto A20 0.6 mi
2 mins

46. Slight left to stay on A20 0.3 mi

47. At Prince of Wales Roundabout, take the 2nd exit onto A20/Limekiln St 0.2 mi
1 min

48. At Limekiln Roundabout, take the 3rd exit onto A20 0.3 mi
1 min

49. At Western Heights Roundabout, take the 1st exit and stay on A20 7.0 mi
8 mins

50. Continue on M20 (signs for M20/London/Ashford) 49.7 mi
47 mins

51. Continue on A20 (signs for London (SE)/Lewisham) 9.7 mi
15 mins

52. At Clifton's Roundabout, take the 2nd exit and stay on A20 2.2 mi
6 mins

53. At the traffic circle, take the 2nd exit and stay on A20 1.3 mi
4 mins

54. Slight left at A2 0.7 mi
2 mins

55. Slight right at A2/Kender St 72 ft

56. Turn right at Kender St 0.3 mi
1 min

57. Turn left at A2 1.9 mi
5 mins

58. At Brick Layers Arms, take the 1st exit onto A201/New Kent Rd 0.6 mi
2 mins

59. At the traffic circle, take the 2nd exit onto A302/St George's Rd 0.4 mi
1 min

60. Turn left at A3203/Lambeth Rd 0.6 mi
3 mins

61. At the traffic circle, take the 2nd exit onto A3203 0.2 mi
1 min

62. At Horseferry Rd, take the 3rd exit onto A3212

COMPARED TO

Driving directions from Myrtle Beach, S.C. to San Diego, Ca

1. Head northeast on N Kings Hwy toward 5th Ave N 0.4 mi
1 min

2. Turn left at Main St 0.2 mi
1 min

3. Continue on US-501 14.1 mi
20 mins

...
4. Turn left at US-378 28.8 mi
42 mins

...
5. Slight right at SC-51 29.8 mi
44 mins

...
6. Continue on W Evans St/SC-S-21-31 0.2 mi
1 min

7. Turn left at W David H McLeod Blvd 1.6 mi
3 mins

8. Continue on I-20 W
Passing through Georgia
Entering Alabama 420 mi
6 hours 17 mins

...
9. Take exit 136 for I-459 toward Montgomery/Tuscaloosa/Gadsden 1.1 mi
1 min

10. Merge onto I-459 S 28.5 mi
25 mins

...
11. Take the I-20 W/I-59 S exit toward Tuscaloosa 1.2 mi
1 min

12. Merge onto I-20 W
Passing through Mississippi, Louisiana
Entering Texas 1,084 mi
15 hours 50 mins

...
13. Merge onto I-10 W
Passing through New Mexico
Entering Arizona 542 mi
7 hours 33 mins

...
14. Take exit 199 to merge onto I-8 W toward San Diego
Entering California 336 mi
4 hours 43 mins

...
15. Take the CA-125 S/CA-125 N exit toward CA-94 0.3 mi

16. Keep left at the fork to continue toward CA-125 S and merge onto CA-125 S 2.4 mi
3 mins

17. Continue on CA-94 W (signs for CA-94 W) 8.5 mi
8 mins

18. Exit onto F St 0.7 mi
3 mins

19. Turn right at 9th Ave 0.1 mi

courtousy Google

from shore to shore
62 steps across the ocean
19 across the USA

tomder55 answered on 04/11/07:

I don't mind the swim.It's the driving through the People's Republic of Massachusetts I onject to . Why can't I just start the swim off the tail end of Long Island N.Y. ?

ETWolverine rated this answer Excellent or Above Average Answer
Itsdb rated this answer Excellent or Above Average Answer
tropicalstorm rated this answer Excellent or Above Average Answer

Question/Answer
tropicalstorm asked on 04/11/07 - Did Nancy Pelosi violate the Logan Act?

Robert F. Turner’s opinion piece in today’s Wall Street Journal (page A10, subscription required for online access) contends that House Dhimmi, excuse us, House Speaker Nancy Pelosi may have committed a felony by conducting unauthorized diplomacy with the dictator of Syria. Given her habit of selecting people like John Murtha of Abscam fame, Alcee Hastings (impeached and removed from office for taking a bribe), and William “Cold Cash” Jefferson for important posts, this would be entirely consistent with her “most ethical Congress in history.”

Illegal Diplomacy
By ROBERT F. TURNER
April 6, 2007; Page A10

House Speaker Nancy Pelosi may well have committed a felony in traveling to Damascus this week, against the wishes of the president, to communicate on foreign-policy issues with Syrian President Bashar Assad.

…The “Logan Act” makes it a felony and provides for a prison sentence of up to three years for any American, “without authority of the United States,” to communicate with a foreign government in an effort to influence that government’s behavior on any “disputes or controversies with the United States.” Some background on this statute helps to understand why Ms. Pelosi may be in serious trouble.

…The U.S. is in the midst of two wars authorized by Congress. For Ms. Pelosi to flaunt the Constitution in these circumstances is not only shortsighted; it may well be a felony, as the Logan Act has been part of our criminal law for more than two centuries. Perhaps it is time to enforce the law.

http://www4.law.cornell.edu/uscode/html/uscode18/usc_sec_18_00000953—-000-.html

TITLE 18 > PART I > CHAPTER 45 > § 953

§ 953. Private correspondence with foreign governments

Any citizen of the United States, wherever he may be, who, without authority of the United States, directly or indirectly commences or carries on any correspondence or intercourse with any foreign government or any officer or agent thereof, with intent to influence the measures or conduct of any foreign government or of any officer or agent thereof, in relation to any disputes or controversies with the United States, or to defeat the measures of the United States, shall be fined under this title or imprisoned not more than three years, or both.

This section shall not abridge the right of a citizen to apply, himself or his agent, to any foreign government or the agents thereof for redress of any injury which he may have sustained from such government or any of its agents or subjects.

Meanwhile, that headscarf that Pelosi put on to show her second-class status as a woman and as a dhimmi (second-class citizen in a militant Islamic country) is very appropriate, and we think she should continue to wear it in Congress. It shows her acceptance of the demeaning status of a kafir female (worth one-quarter of a Muslim male under Saudi Arabian blood money compensation tables), and is on a par with an African-American addressing white people as “Massah” in the 21st century. The Speaker of the House has not only allegedly violated the Logan Act by conducting diplomacy with a hostile foreign power, she has engaged in behavior far more suitable for female Islamic property or for a slave than for a free American. Self-owning American men and women should therefore treat her with the respect that one accords an individual who is a slave by choice; that is, with total and unmitigated contempt.

Posted by Bill Levinson on
pelosi post )

tomder55 answered on 04/11/07:

yes she did ,but frankly ,politicians in Congress have been violating it for as long as I can remember.I am more critical of the trip over the mixed message it sent rather than any issues of legality .She is after-all the 3rd in line to the Presidency (God help us).

Newt Gingrich when he was speaker took a junket romp to China with John Boehner in 1997 .Gingrich did make pronouncements about how the US would respond if China attacked Taiwan .The US policy has been vague about Taiwan ,not wanting to definitely commit to the defense of the Island .The Clinonoids released a statement to the effect that Gingrich was speaking for himself . Gingrich of course now has been a vocal critic of the Pelosi trip.I think he has a short memory .

I think the Gonzales Justice Dept. should launch an investigation ;perhaps Pat Fitzgerald isn't doing anything now . Then President Bush should gracefully pardon Madam Meme from any wrongdoing and for the bad taste in headress. But before he does he should correct her on US policy . The road to Damascus isn't the road to peace .... The Road THROUGH Damasus is .

The Pelosi trip is old news . What I want to know is why Majority Leader Steny Hoyer went on a junket to Cairo to meet with terrorists ????? Secretary of State Condoleezza Rice has refused in the past to meet with the Muslim Brotherhood,because of their ties to Hamas. It appears that the Democrats are very comfortable hob-nobbing with murderous thugs .

tropicalstorm rated this answer Excellent or Above Average Answer
Itsdb rated this answer Excellent or Above Average Answer

Question/Answer
tropicalstorm asked on 04/10/07 - LOL remember Tomder's post

of the poplar bears "stranded" on the iceberg.
Rush says they can swim 60 miles so animal instinct would probably tell them SWIM. Land being within 60 miles MIGHT be another story though!

tomder55 answered on 04/10/07:

they've never had it better .In fact they have an abundance of food since the Alaska cold spell prevents > sea otters from swimming to safety .

tropicalstorm rated this answer Excellent or Above Average Answer

Question/Answer
curious98 asked on 04/10/07 - 4th anniversary of the fall of Baghdad


When on occasion of the 4th anniversary of the fall of Baghdad to the US forces hundreds of thousands of Iraqis in the city of Najaf gather for a big anti-US rally called by fiery cleric Moqtada al-Sadr, with the crowd burning US flags; while Baghdad has spent last Monday under curfew to avoid the risk of serious confrontations and while Rear Admiral Mark Fox told reporters yesterday “We acknowledge that while there have been substantial accomplishments in Iraq since 2003, but the past four years have also been very disappointing, frustrating and increasingly dangerous in many parts of Iraq”, is it not about time for Americans to start wondering what the heck are you doing there, risking every day the life of your soldiers, when the majority of the Iraqi population considers you as an invading force…?

Your mission is over. Saddam is no longer a menace. WMD have never existed. Whatever “democracy” that is possible in Iraq has already been established. Your forces cannot obviously cope with the latent civil war going on right now. And on top of everything a great part of Americans are yearning for a retreat of your troops…

Why is Bush so adamant to continue over there, come what may? What is he going to gain out of this stubburness?

Any comments?
Curious98

tomder55 answered on 04/10/07:

See Gen. Petraeus letter to the Iraqi's on the post below . I quote :

On this April 9th, some Iraqis reportedly may demonstrate against the coalition force presence in Iraq. That is their right in the new Iraq. It would only be fair, however, to note that they will be able to exercise that right because coalition forces liberated them from a tyrannical, barbaric regime that never would have permitted such freedom of expression.

Those who take to the streets should recall, moreover, that were it not for the actions of coalition forces in 2003 (and, to be sure, actions by Iraqi, as well as coalition, forces since then), they also would not have been able to celebrate the recent religious holidays as they did in such massive numbers. Nor would they have been able to select their leaders by free and democratic elections, vote on their constitution, or take at least the initial steps toward establishment of a government that is representative of, and responsive to, all Iraqis.


I'd like to point out that your figures on the number of protesters is WAY OFF . What you are quoting is what was expected ;not the turn out . Even al-AP and the NY Slimes only put the numbers at tens of thousands ".Reuters called it thousands .Rear Admiral Mark Fox who you quote to support your thesis noted that the Coalition closely monitored the protest, and put the number of demonstrators between 5,000 and 7,000.



Above is an aerial view of the protest .If there are hundreds of thousands there ,then they are standing 10 deep on top of each other .

Too bad the 3000 people who gathered calling for an end to sectarian violence did not garner the same media attention as the 1800+ articles that covered the Najaf demonstration . Wonder why that is so ?????????

The fat cleric Mookie al-sadr crawled out of his spider hole long enough to call on his supported to stage this protest . Big deal . It is my impression that whenever a Muslim cleric calls followers out to the street they are more than willing to put on a show. Just like they got whipped up to a frenzy over cartoons of Mohammed . I have no reason to believe that their calls for us to leave represent more than a small faction's desires . I also think it is of note that Fat Mookie was a no show for his own demonstration .

Yes ,much of our mission has been achieved . We could've just as easily walked away from Europe after WWII . Why not ? Germany was defeated . We however did not think it was in the best interest of the Europeans to be over-run by the dominant Soviet forces . You don't do you ? Do you think it is a good idea to allow the newly liberated Iraq to get gobbled up by their stronger neighbors ? Would you care for that to perhaps expand into a regional conflict ? Maybe you don't care .

As far as the American reaction . Well let's say that yes ;many of us would like it to end . Despite the European opinion of us as a bunch of Conservative bullies itching for a fight ,we really don't like sending our loved ones to hell on earth.(ironically the Muslims complain we are too liberal ) Most of us do not support leaving Iraq if the job is not finished. .I was in the Capital last weekend . The only protests that were going on in Washington DC was a handful of people who think circumcision is brutality .


WE never "yearn for retreat " That appears to be a Spanish trait .



curious98 rated this answer Above Average Answer

Question/Answer
HANK1 asked on 04/07/07 - Wouldn't This Be A Hoot:



In 2008:

President: Hillary Clinton

Vice - President: Nancy Pelosi

Secretary of State: Bill Clinton

Is it probable and/or possible?

HANK

tomder55 answered on 04/07/07:

President Evita is possible . I don't think she will pick Pelosi as a running mate . She probably would do well with Bill Richardson but she will more likely make some kind of deal with Obama.There is also rumors of a possible Evan Bayh VP run;but I think she may go with someone like Mark Warner of Virginia .

She would not pick Bill Clinton to serve in her cabinet but would use him instead as an advisor and special envoy . ohhh; who am I kidding .....he would be running the country . Does anyone really think she could do it herself ?

HANK1 rated this answer Excellent or Above Average Answer
Itsdb rated this answer Excellent or Above Average Answer

Question/Answer
tropicalstorm asked on 04/07/07 - A TWIST TO THE POLITICAN
KISSING THE BABY

tomder55 answered on 04/07/07:

The poor kid will need therapy for years with those nightmares of being man-handled by the hilderbeast .

Itsdb rated this answer Excellent or Above Average Answer
tropicalstorm rated this answer Excellent or Above Average Answer

Question/Answer
Itsdb asked on 04/07/07 - A global warming Easter

For the first in my life as far as I can recall, we're expecting a white Easter. And this just a couple of days after scientists predicted a return of the dustbowl and the water supply running out in 20 years for many people.

But hey, the good news is now that "desalination is the wave of the future," all that rising seawater will have somewhere to go.

And by the way, we've had a wetter than normal winter and spring thus far, local lake levels are rising and farmers are predicting record crops.

Happy global warming Easter.

tomder55 answered on 04/07/07:

as you well know you cannot point to specific regional weather patterns to make a case about global warming ....well ,that is unless you point to regional weather patterns to support the case that global warming is a fact .

I was in Washington DC over the weekend and enjoyed 80 degree warmth and those wonderful cherry blossoms . The last 2 days back in NY we woke up to below freezing mornings. (btw ..the only protesters in DC were a handful in front of the empty Capitol wanting a ban on circumcisions.)

Desalination is a good thing to develop. Certainly no one argues that humans will increasingly compete for potable water .Of course if they are planning to do it to keep the golf courses green then perhaps their priorities are skewed.

Itsdb rated this answer Excellent or Above Average Answer
labman rated this answer Excellent or Above Average Answer

Question/Answer
ETWolverine asked on 04/06/07 - Probably the best discussion of How Liberals Think...

...that I have ever heard. Check out Evan Sayet's speech to The Heritage Foundation.

http://www.youtube.com/watch?v=eaE98w1KZ-c

Its long... about 45 minutes... but worth the time.

Let me know what you guys think.

Elliot

tomder55 answered on 04/06/07:

"The modern liberal will invariably side with evil over good, wrong over right, and the behaviors that lead to failure over those that lead to success,"

"How could you possibly live in the freest nation in the history of the world and only see oppression? How could you live in the least imperialist power in human history and see us as the ultimate in imperialism? How can you live in the least bigoted nation in human history ... and see racism lurking in every dark shadow?"

Robert Fulghum book, "All I Really Need to Know I Learned in Kindergarten," "reads like the bible of modern liberalism and the playbook of Democratic Party policy."


Pretty hard hitting . Leave it to a reformed liberal to be the harshest critic . Former Bill Mahr writer ,wow ! He's looking for one of those pies in the face ....oh wait... they save that type of treatement for petite women like Ann Coulter .




We are beginning to gather a collection of conservative comedy commentators. Sayet ,Dennis Miller ,and Jackie Mason who's new book 'Schmucks!: Our Favorite Fakes, Frauds, Lowlifes, Liars, the Armed and Dangerous, and Good Guys Gone Bad' should be an instant best seller.

Check out Sayet's web site .He blasts Chris Matthews and the left media today .

ETWolverine rated this answer Excellent or Above Average Answer

Question/Answer
Itsdb asked on 04/05/07 - The Score

al-AP writes...

    TEHRAN, Iran - President Mahmoud Ahmadinejad defused a growing confrontation with Britain, announcing the surprise release of 15 captive British sailors Wednesday and then gleefully accepting the crew's thanks and handshakes in what he called an Easter gift.

    British Prime Minister Tony Blair expressed "profound relief" over the peaceful end to the 13-day crisis. "Throughout we have taken a measured approach - firm but calm, not negotiating, but not confronting either," Blair said in London, adding a message to the Iranian people that "we bear you no ill will."

    The announcement in Tehran was a breakthrough in a crisis that had escalated over nearly two weeks, raising oil prices and fears of military conflict in the volatile region. The move to release the sailors suggested that Iran's hard-line leadership decided it had shown its strength but did not want to push the standoff too far.

    Iran did not get the main thing it sought - a public apology for entering Iranian waters. Britain, which said its crew was in Iraqi waters when seized, insists it never offered a quid pro quo, either, instead relying on quiet diplomacy.

    Syria, Iran's close ally, said it played a role in winning the release. "Syria exercised a sort of quiet diplomacy to solve this problem and encourage dialogue between the two parties," Syrian Foreign Minister Walid al-Moallem said in Damascus.

    The announcement of the release came hours after U.S. House Speaker Nancy Pelosi met with President Bashar Assad in Damascus, trying to show that a U.S. dialogue with Syria - rejected by the Bush administration - could bring benefits for the Middle East. The British sailors were not part of their talks, and it was not clear if the release was timed to coincide with her visit.

    Iran's official news agency said the British crew was to leave Iran by plane at 8 a.m. today. By Wednesday evening they had still not been handed over to the British Embassy in Tehran and the embassy said it was not clear where they would spend the night. Britain's ambassador met with the sailors and confirmed they were in good health, Britain's Foreign Office said.

    Several British newspapers credited Blair's foreign policy adviser Nigel Sheinwald and Iranian chief negotiator Ali Larijani with laying the groundwork for an agreement during telephone contacts that began Tuesday night. Larijani had gone on British TV on Monday and signaled that Tehran was looking for a diplomatic solution.

    British officials were told to pay close attention to Ahmadinejad's press conference but were unsure the release would come until they heard his words, The Independent newspaper said.

    Ahmadinejad timed the announcement so as to make a dramatic splash, springing it halfway through a two-hour news conference.

    The president first gave a medal of honor to the commander of the Iranian coast guards who captured the Britons, and admonished London for sending a mother, Leading Seaman Faye Turney, on such a dangerous mission in the Persian Gulf.

    He said the British government was "not brave enough"
    to admit the crew had been in Iranian waters when it was captured.

    Ahmadinejad then declared that even though Iran had the right to put the Britons on trial, he had "pardoned" them to mark the March 30 birthday of the Prophet Muhammad and the coming Easter holiday.

    "This pardon is a gift to the British people," he said.


    After the news conference, Iranian television showed a beaming Ahmadinejad on the steps of the presidential palace shaking hands with the Britons - some towering over him. The men were decked out in business suits and Turney wore an Islamic head scarf.

    "Your people have been really kind to us, and we appreciate it very much," one of the British men told Ahmadinejad in English. Another male service member said: "We are grateful for your forgiveness."


    Ahmadinejad responded in Farsi, "You are welcome."


The score? al-AP/Iran/Ahmedinejad/Syria/Islamofascism - at least 5

US & British Foreign policy/British Marines/women's rights/GWOT - 0

The Mahdi Hatter looks like a god for 'towering' British Marines (and one in a scarf) "thanking" him for his Easter gift. Syria gets a triple play for Pelosi's visit, her idiotic misrepresentation of Israel's message and perfect timing for 'facilitating' the release of the hostages. And they all get a boost from al-AP's glowing account of the whole affair. Can it get any more pathetic?

tomder55 answered on 04/06/07:

I guess the rest of the story was over-looked . There are many angles to explore in this debacle but the key one I think is that this appears to be a great victory for Iran.

Jalal Sharafi had been captured during what was described as a botched raid by US forces on an Iranian counsulate in Irbil Iraq .The Independent claimed the raid was a failure, and that two other men, who went uncaptured, were the only meaningful targets.But that was not the truth .[The paper also incredibly implies that kidnapping is somehow a new tactic for the Iranians lol.]

US and Iraq had to this point denied that Sharafi had been detained ,but suddenly this week not only did we reverse course and admit it ;but also released the scum coincidently right before the release of the British sailors.

According to Eli Lake at the NY Sun ;The decision to release Jalal Sharafi on Tuesday was made at the White House, according to an administration official who asked to be anonymous because of the sensitivity of the information. The release took place over the objections of some commanders in the field. Mr. Sharafi, the second secretary of the Iranian Embassy in Baghdad, is believed by American military intelligence also to be a member of the lethal Quds Force, the terrorist-supporting organization whose members have been fair game for American soldiers and Iraqi allies since a change in the rules of engagement was issued in December.....

The release of Iranian nationals detained by America was one of the primary Iranian demands during the negotiations. "They think they won this round. They were able to take the hostages and suffer no consequences," an American official said.


Indeed they are correct in that opinion . They secured the release of one of their key asset in Iraq for the return of the British sailors caputed in Iraqi waters . Yes it's true as Lake points out that we still have a number of Iranians in custody ,but the Iranians now know we are willing to negotiate with pirates and terorists . The table has been set . Having successfully gained the release of Sharafi ,it is almost certain that they will make a play to secure the release of the rest of their assets .Charles Krauthammer claims they were granted access to the five Iranian "consular officials" detained.

The Iranians have found our soft spot ;the Brits who's only real concern at this point is how fast they can bolt and still appear to save face . I hate to say it ,but the whole performance of the Brits ;from the hotages themselve to Tony Blair was weak and lame. Contrast the prisoners compliance with those of former American POWs like John McCain who is crippled by the actions of his captors.

Four British soldiers were killed in a Basra roadside bombing by what the Guardian described as Shia "rogue militia" suspected of having links with Iran.

As Amir Taheri wrote :"President Bush’s decision to change the rules of engagement for US forces in Iraq with the new “surge” strategy, allowing Americans to kill or capture any Iranian perceived as a threat, made it more difficult for the mullahs to do an Abi Waqqas. As a result, the British, whose rules of engagement prevent them from fighting Iranians even in self-defence, were chosen as the softer target."

That the British rear-guard is coming under attack was predictable .It should be a lesson for those in our country who think it wise to give a date when we cut and run. But we have ourselves shown our vulnerability by complying with the demands of pirates/terrorists /kidnappers which I think damages our effort there .




Itsdb rated this answer Excellent or Above Average Answer

Question/Answer
tropicalstorm asked on 04/04/07 - need another invention here

Tomder inc
We need to invent a comment back recording device that goes DIRECTLY back to a REAL person ASAP!
Yesterday I was at the car wash and noticed that it went up one whole dollar since last summer. Needing the bird poop washed off my windshield I figured what the hey! So I started plopping my quarters in the machine. After EACH and every quarter the machine says, (in a Betty Boop voice) 'Stop, don't hit me. Vandalism is just wrong'. I wanted to comment back, 'YOU are the one vandalizing me charging a whole dollar more for you to annoy me!'
THEN each button I pushed presoak, wash, rinse
it said the same thing 'Stop, don't hit me. Vandalism is just wrong"
I had to have heard that at least 15 times in the whole 2 minute and 54 second amount of my car wash time.

cordially
Tomder inc

CRC
sapph

tomder55 answered on 04/05/07:

I bet more than one of them eventually get vandalized. Primarily the speaker gets smashed.

Heard this on the radio.....During one of those damn telemarketting calls the person at the receiving end had had enough and decided to teach them a lesson. When he received a call he asked the telemarketer Why do you want to speak to the deceased ?

He then went on to pretend that he was in the house because he was doing a homicide investigation and proceeded to grill the telemarketer ...asking how he knew the deceased and what was the nature of the business he had with him ? He got very aggressive with the line of questioning asking for his name. When the telemarketer refused he claimed the call was being traced.

tropicalstorm rated this answer Excellent or Above Average Answer

Question/Answer
tropicalstorm asked on 04/04/07 - conspiracy theory or no conspiracy

For -various- reasons I believe "THEY" knew it was coming.
ignored warning )

“I saw papers that show US knew al-Qa’ida would attack cities with aeroplanes” Whistleblower the White House wants to silence speaks to The Independent By Andrew Buncombe in Washington

02 April 2004

A former translator for the FBI with top-secret security clearance says she has provided information to the panel investigating the 11 September attacks which prove senior officials knew of al-Qa’ida’s plans to attack the US with aircraft months before the strikes happened.

She said the claim by the National Security Adviser, Condoleezza Rice, that there was no such information was “an outrageous lie”.

Sibel Edmonds said she spent more than three hours in a closed session with the commission’s investigators providing information that was circulating within the FBI in the spring and summer of 2001 suggesting that an attack using aircraft was just months away and the terrorists were in place. The Bush administration, meanwhile, has sought to silence her and has obtained a gagging order from a court by citing the rarely used “state secrets privilege”.

She told The Independent yesterday: “I gave [the commission] details of specific investigation files, the specific dates, specific target information, specific managers in charge of the investigation. I gave them everything so that they could go back and follow up. This is not hearsay. These are things that are documented. These things can be established very easily.”

She added: “There was general information about the time-frame, about methods to be used ­ but not specifically about how they would be used ­ and about people being in place and who was ordering these sorts of terror attacks. There were other cities that were mentioned. Major cities ­ with skyscrapers.”

The accusations from Mrs. Edmonds, 33, a Turkish-American who speaks Azerbaijani, Farsi, Turkish and English, will reignite the controversy over whether the administration ignored warnings about al-Qa’ida. That controversy was sparked most recently by Richard Clarke, a former counter-terrorism official, who has accused the administration of ignoring his warnings.

The issue ­ what the administration knew and when ­ is central to the investigation by the 9/11 Commission, which has been hearing testimony in public and private from government officials, intelligence officials and secret sources. Earlier this week, the White House made a U-turn when it said that Ms Rice would appear in public before the commission to answer questions. Mr. Bush and his deputy, Dick Cheney, will also be questioned in a closed-door session.

Mrs. Edmonds, 33, says she gave her evidence to the commission in a specially constructed “secure” room at its offices in Washington on 11 February. She was hired as a translator for the FBI’s Washington field office on 13 September 2001, just two days after the al-Qa’ida attacks. Her job was to translate documents and recordings from FBI wire-taps.

She said it was clear there was sufficient information during the spring and summer of 2001 to indicate terrorists were planning an attack. “Most of what I told the commission ­ 90 per cent of it ­ related to the investigations that I was involved in or just from working in the department. Two hundred translators side by side, you get to see and hear a lot of other things as well.”

“President Bush said they had no specific information about 11 September and that is accurate but only because he said 11 September,” she said. There was, however, general information about the use of airplanes and that an attack was just months away.

To try to refute Mr. Clarke’s accusations, Ms Rice said the administration did take steps to counter al-Qa’ida. But in an opinion piece in The Washington Post on 22 March, Ms Rice wrote: “Despite what some have suggested, we received no intelligence that terrorists were preparing to attack the homeland using airplanes as missiles, though some analysts speculated that terrorists might hijack planes to try and free US-held terrorists.”

Mrs. Edmonds said that by using the word “we”, Ms Rice told an “outrageous lie”. She said: “Rice says ‘we’ not ‘I’. That would include all people from the FBI, the CIA and DIA [Defence Intelligence Agency]. I am saying that is impossible.”

It is impossible at this stage to verify Mrs. Edmonds’ claims. However, some senior US senators testified to her credibility in 2002 when she went public with separate allegations relating to alleged incompetence and corruption within the FBI’s translation department.

tomder55 answered on 04/05/07:

She was fired from the FBI in April 2002 .Edmonds alleges she was fired after complaining to managers about shoddy wiretap translations and told them an interpreter with a relative at a foreign embassy might have compromised national security after the Sept. 11 terror attacks by passing information from an FBI wiretap to the target of an investigation.The Justice Department investigation into Edmonds' firing, which remains classified, determined that Edmonds never qualified for formal whistle-blower protection because she was a contract worker, not a full-time FBI employee.But FBI Director Robert Mueller did admit to the Senate that she was discharged at least partly for being a whistle-blower.

The translator she was complaining about is Melek Can Dickerson . According to her letter to Tom Kean , CoChair of the 9-11 Commission :

Melek Can Dickerson, a Turkish translator, was hired by the FBI after September 11, and was placed in charge of translating the most sensitive information related to terrorists and criminals under the bureau's investigation. Melek Can Dickerson was granted Top Secret Clearance, which can be granted only after conducting a thorough background investigation. Melek Can Dickerson used to work for a semi-legit organizations that were the FBI's targets of investigation. Melek Can Dickerson had on going relationships with two individuals who were FBI's targets of investigation. For months Melek Can Dickerson blocked all-important information related to these semi-legit organizations and the individuals she and her husband associated with. She stamped hundreds, if not thousands, of documents related to these targets as "Not Pertinent". Melek Can Dickerson attempted to prevent others from translating these documents important to the FBI's investigations and our fight against terrorism. Melek Can Dickerson, with the assistance of her direct supervisor, Mike Feghali, took hundreds of pages of top-secret sensitive intelligence documents outside the FBI to unknown recipients. Melek Can Dickerson, with the assistance of her direct supervisor, forged signatures on top-secret documents related to certain [September 11-related] detainees. After all these incidents were confirmed and reported to FBI management, Melek Can Dickerson was allowed to remain in her position, to continue the translation of sensitive intelligence received by the FBI, and to maintain her Top Secret Clearance. Apparently bureaucratic mid-level FBI management and administrators decided that it would not look good for the bureau if this security breach and espionage case was investigated and made public, especially after going through Robert Hanssen's case (FBI spy scandal). This case (Melek Can Dickerson) was confirmed by the Senate Judiciary Committee (please refer to Senator Leahy's and Grassley's letters dated June 19 and August 13, 2002, and Senator Grassley's statement on CBS 60 Minutes in October 2002, provided to your investigators in January-February 2004). This Dickerson incident received major coverage by the press (please refer to media background provided to your investigators in January-February 2004). According to [FBI] director [Robert] Mueller, the inspector general criticized the FBI for failing to adequately pursue this espionage report regarding Melek Can Dickerson (please refer to DOJ-IG report Re: Sibel Edmonds and FBI Translation, provided to you prior to the completion of your report). I provided your investigators with a detailed and specific account of this issue, the names of other witnesses willing to corroborate this, and additional documents (please refer to tape-recorded 3.5 hours' testimony by Sibel Edmonds, provided to your investigators on February 11, 2004).

Today, more than two years since the Dickerson incident was reported to the FBI, and more than two years since this information was confirmed by the United States Congress and reported by the press, these administrators in charge of FBI personnel security and language departments in the FBI remain in their positions and in charge of translation quality and translation departments' security. Melek Can Dickerson and several FBI targets of investigation hastily left the United States in 2002, and the case still remains uninvestigated criminally. Not only does the supervisor facilitating these criminal conducts remain in a supervisory position, he has been promoted to supervising Arabic-language units of the FBI's counter-terrorism and counter-intelligence investigations. Your report has omitted these significant incidents, has forgone any accountability whatsoever, and your recommendations have refrained from addressing this serious information security breach and highly likely espionage issue. This issue needs to be investigated and criminally prosecuted. The translation of our intelligence is being entrusted to individuals with loyalties to our enemies. Important "chit-chats" and "chatters" are being intentionally blocked. Why did your report choose to exclude this information and these serious issues despite the evidence and briefings you received? How can budget increases address and resolve this misconduct by mid-level bureaucratic management? How can the addition of a new bureaucratic layer, "intelligence czar", in its cocoon removed from the action lines, address and resolve this problem?


Now what I read into this is not a coverup of information as much as the fact that the FBI and all the intelligence agencies were poorly managed prior to 9-11 . That frankly has already been confirmed, and the appointment of an intelligenc czar's effectiveness may indeed be a debatable point.However ,it is an attempt to correct the deficiencies Edmonds points out. In such an atmosphere of over-all shoddy performance it should not be a suprise that when confronted by specific intelligence related to a possible attack on U.S. soil that the Bureau approached it like a bunch of key-stone cops. But here is an important point to consider .... Robert S. Mueller III became the director of the FBI September 4,2001 ...one week before 9-11 . That has to be considered when reading her specific charges related to 9-11 :

Over three years ago, more than four months prior to the September 11 terrorist attacks, in April 2001, a long-term FBI informant/asset who had been providing the bureau with information since 1990, provided two FBI agents and a translator with specific information regarding a terrorist attack being planned by Osama bin Laden. This asset/informant was previously a high-level intelligence officer in Iran in charge of intelligence from Afghanistan. Through his contacts in Afghanistan he received information that: 1) Osama Bin Laden was planning a major terrorist attack in the United States targeting four to five major cities, 2) the attack was going to involve airplanes, 3) some of the individuals in charge of carrying out this attack were already in place in the United States, 4) the attack was going to be carried out soon, in a few months. The agents who received this information reported it to their superior, Special Agent in Charge of Counter-terrorism Thomas Frields, at the FBI Washington Field Office, by filing 藾" forms, and the translator translated and documented this information. No action was taken by the special agent in charge, and after [September 11] the agents and the translators were told to "keep quiet" regarding this issue. The translator who was present during the session with the FBI informant, Mr Behrooz Sarshar, reported this incident to director Mueller in writing, and later to the Department of Justice inspector general. The press reported this incident, and in fact the report in the Chicago Tribune on July 21, 2004, stated that FBI officials had confirmed that this information was received in April 2001, and further, the Chicago Tribune quoted an aide to director Mueller that he (Mueller) was surprised that the commission never raised this particular issue with him during the hearing (please refer to Chicago Tribune article, dated July 21, 2004). Mr Sarshar reported this issue to your investigators on February 12, 2004, and provided them with specific dates, location, witness names, and the contact information for that particular Iranian asset and the two special agents who received the information (please refer to the tape-recorded testimony provided to your investigators during a 2.5 hours' testimony by Mr Sarshar on February 12, 2004). I provided your investigators with a detailed and specific account of this issue, the names of other witnesses, and documents I had seen (please refer to tape-recorded 3.5 hours' testimony by Sibel Edmonds, provided to your investigators on February 11, 2004). Mr Sarshar also provided the Department of Justice inspector general with specific information regarding this issue (please refer to DOJ-IG report Re: Sibel Edmonds and FBI Translation, provided to you prior to the completion of your report).

Mueller ,was on the job one week before 9-11. If he indeed have information handed to him about possible terrorist attacks he certainly did not have sufficient time to act on them. Before he took over Thomas J. Pickard was interm director . He took over after Clinton appointee Louis Freeh left office June 25, 2001.

As you can see ,the delay in transition due to the 2000 election recounts had a major role in the intelligence agencies fumbling the ball. As we have documented many times here ,the intelligence community was not up to the task during the Clinton years. I am very certain that enough raw intel. was available to come to the conclusion that there would likely be a major attack on the US by OBL. That hardly means there was a conspiracy inside the gvt. to allow it to happen .

Edmonds has also claimed that FBI translators of Middle-Eastern origin openly praised the 9-11 attacks .

"During my work with the bureau, I was seriously taken aback by what I heard and witnessed within the translation department," she said. "There were those who openly divided the fronts as 'Us' – the Middle-Easterners who shared certain views – and 'Them' – the Americans who were the outsiders [whose] arrogance was now 'leading to their own destruction.'"

Not long after the attacks, Edmonds said one translator said: "It is about time that they get a taste of what they have been giving to the rest of the Middle East."

She says the remark was made in front of the unit supervisor, also of Middle Eastern origin.

"These comments were neither rare nor made in a whisper," Edmonds said. "They were open and loud."

She says such attitudes call into question "the integrity and accuracy" of information Arabic translators are feeding agents.

Edmonds says agents who don't speak Arabic have no way of knowing whether the information they receive from translators is tainted.

"They simply have to trust the information given to them by translators," she said, "and based on that, decide to act or not act."


That makes open the possibility that information about al-Qaeda was being passed on to superiors by moles inside the agency .Another issue then is ;How can you act in a timely manner if you cannot trust the veracity of the inflrmation you receive ? Foreign translators are never to be trusted. But how many Americans speak the language ? If possible foreign translators should always be teamed with an American counterpart who has enough knowlege of the language to verify that the translation is likely accurate.

The dept. was in transition ;the information could not be trusted by her own admission. Even if the FBI had the correct information that al-Qaeda was planning an attack by planes ,the FBI was not permitted by law to contact the CIA to gather names of suspects. Ok then ..what to do ? Inform the public ? ...shut down the airlines ? There was nothing actionable to do without more detailed information.

Based on her being canned ,which was probably unjust ,she set up a web site just in time to try to influence the 2004 elections and has been playing this conspiracy card.There is no basis to conclude conspiracy however ;just incompetence in the construct of the intelligence agencies and the rules they were forced to work under ......but we knew that already .















tropicalstorm rated this answer Excellent or Above Average Answer

Question/Answer
tropicalstorm asked on 04/02/07 - john travolta
wants us to
conserve

here is his house complete with two double garages and room for his five planes.

travoltas planes and excuses )

Clocking up at least 30,000 flying miles in the past 12 months means he has produced an estimated 800 tons of carbon emissions – nearly 100 times the average Briton's tally.
Travolta, a Scientologist, claimed the solution to global warming could be found in outer space and blamed his hefty flying mileage on the nature of the movie business.



Think we should contribute to the cause???

tomder55 answered on 04/03/07:

in his defense he did admit that he may not be the best spokesperson on the issue. I guess the ability to understate is part of his fine acting skills . Not to worry ,John has the solution . He is a scientologist who has a deep held faith that aliens will provide us with the alternate fuels needed to solve the crisis.

tropicalstorm rated this answer Excellent or Above Average Answer

Question/Answer
Itsdb asked on 04/01/07 - Good news on Iraq

In the form of no news? For the first time I can remember since the Iraq war began, our paper had NO articles on bombings and troops killed today. There were 4 blurbs, Biden and Obama criticizing the war, the UN sending staff back to Baghdad, and an agreement to relocate Arabs that Saddam had moved to Kirkuk to force out Kurds.

Could the tide be turning in the media? Or is it just that global warming has overtaken Iraq as the cause of the day?

tomder55 answered on 04/02/07:

Taking a few days in DC to see cherry blossoms and the sites. Not much time for news or net. The big story here I guess was the Matthew Dowd noisy defection . Either he has a book coming soon or he is planning on hitching a ride on a Democat candidate's wagon is the way I read it .(he was a democrat...former Lloyd Benson staffer; before opportunity's fortune steered him Bush's way. ) Dowd in many ways is like David Gergen ,a man who has no real conviction and will work for whoever is in power and adjust their beliefs accordingly .

But the real question I have is this : how long has Dowd opposed the war and what did he do as a chief aid to the President in steering while he was opposed ? He was a major pollster/analysist for the President . The US pr on this war has stunk .The adminstration has been behind the curve in dealing with issues like Abu Ghraib or the Haditha Marines. How much of this was Dowd responsible for ? Has he been steering Bush away from the words that would communicate to the public ? Has he been altering his data or selecting phraese that would not resonate with the public ?

The news reported 6 soldiers killed in car road side bombings and of course the press here is gaga over Pelosi's gravitas as our new Sec.State . Meanwhile McCain is in Iraq saying the surge is working .The a**hole reporter for CNN Michael Ware was harrassed him and mocking him during a press conference.(this is the same joker who admiringly reported that he witnessed insurgents assembling chemical bombs [no wmd?] to use as terrorist weapons...he also is responsible for airing the sniper snuff video of a US soldier).

Iraqi security forces clashed with al-Qaeda near Syria and killed 21 .

Thosands of Iraqis in Basrah will have tap water for the first time in 25 years thanks to the MNF. In reality the US Corp of Engineers have over 1000 projects ongoing inside the country including a $235 million water treatment facility in Nasiriya. Mrs. Bush has sponsored 'Project Hope 'which builds schools in Iraq. Unfortunately our military has the worst PR dept in US history and apparently there have been moles inside the White House bent on undermining the effort.

Itsdb rated this answer Excellent or Above Average Answer

Question/Answer
PrinceHassim asked on 03/31/07 - Terry Jones speaks out

Call that humiliation?


No hoods. No electric shocks. No beatings. These Iranians clearly are a very uncivilised bunch

Terry Jones
Saturday March 31, 2007
The Guardian

I share the outrage expressed in the British press over the treatment of our naval personnel accused by Iran of illegally entering their waters. It is a disgrace. We would never dream of treating captives like this - allowing them to smoke cigarettes, for example, even though it has been proven that smoking kills. And as for compelling poor servicewoman Faye Turney to wear a black headscarf, and then allowing the picture to be posted around the world - have the Iranians no concept of civilised behaviour? For God's sake, what's wrong with putting a bag over her head? That's what we do with the Muslims we capture: we put bags over their heads, so it's hard to breathe. Then it's perfectly acceptable to take photographs of them and circulate them to the press because the captives can't be recognised and humiliated in the way these unfortunate British service people are.

It is also unacceptable that these British captives should be made to talk on television and say things that they may regret later. If the Iranians put duct tape over their mouths, like we do to our captives, they wouldn't be able to talk at all. Of course they'd probably find it even harder to breathe - especially with a bag over their head - but at least they wouldn't be humiliated.

And what's all this about allowing the captives to write letters home saying they are all right? It's time the Iranians fell into line with the rest of the civilised world: they should allow their captives the privacy of solitary confinement. That's one of the many privileges the US grants to its captives in Guantánamo Bay.

The true mark of a civilised country is that it doesn't rush into charging people whom it has arbitrarily arrested in places it's just invaded. The inmates of Guantánamo, for example, have been enjoying all the privacy they want for almost five years, and the first inmate has only just been charged. What a contrast to the disgraceful Iranian rush to parade their captives before the cameras!

What's more, it is clear that the Iranians are not giving their British prisoners any decent physical exercise. The US military make sure that their Iraqi captives enjoy PT. This takes the form of exciting "stress positions", which the captives are expected to hold for hours on end so as to improve their stomach and calf muscles. A common exercise is where they are made to stand on the balls of their feet and then squat so that their thighs are parallel to the ground. This creates intense pain and, finally, muscle failure. It's all good healthy fun and has the bonus that the captives will confess to anything to get out of it.

And this brings me to my final point. It is clear from her TV appearance that servicewoman Turney has been put under pressure. The newspapers have persuaded behavioural psychologists to examine the footage and they all conclude that she is "unhappy and stressed".

What is so appalling is the underhand way in which the Iranians have got her "unhappy and stressed". She shows no signs of electrocution or burn marks and there are no signs of beating on her face. This is unacceptable. If captives are to be put under duress, such as by forcing them into compromising sexual positions, or having electric shocks to their genitals, they should be photographed, as they were in Abu Ghraib. The photographs should then be circulated around the civilised world so that everyone can see exactly what has been going on.

As Stephen Glover pointed out in the Daily Mail, perhaps it would not be right to bomb Iran in retaliation for the humiliation of our servicemen, but clearly the Iranian people must be made to suffer - whether by beefing up sanctions, as the Mail suggests, or simply by getting President Bush to hurry up and invade, as he intends to anyway, and bring democracy and western values to the country, as he has in Iraq.

Terry Jones - film director, actor and Python a la Monty Python

www.terry-jones.net

tomder55 answered on 04/01/07:

The Sydney Morning Hreald reported that David Hicks showed up for his hearing overweight. So now the accusation is that the Americans tortured him by forcing him to over eat .

After all the hearings and stuff Hicks will be released after confessing to supporting efforts to murder innocent civilians. Many people in Iran will rot in jail for the rest of their lives for much less.

Terry Jones is very funny in a British sort of way . But , has he ever read the human rights record of Iran ?
Human rights have significantly deteriorated there in recent years (not that they were ever that great);despite the decision by the UN Human Rights Watch group to give up monitoring the country . I know that it is a tongue in cheek commentary but it is telling that his sympathy lies with jihadists and not with his nations sailors.

PrinceHassim rated this answer Excellent or Above Average Answer

Question/Answer
Itsdb asked on 03/31/07 - Fact-finding?

The leader of the US House of Representatives, Speaker Nancy Pelosi, in coming days will visit Syria, a country President George W. Bush has shunned as a state sponsor of terrorism, despite being asked by the administration not to go.

"In our view, it is not the right time to have these sort of high-profile visitors to Syria," State Department spokesman Sean McCormack told reporters Friday.

Pelosi will not be the first member of Congress in recent months to travel to Syria, but as House speaker she is the most senior.

"This is a country that is a state sponsor of terror, one that is trying to disrupt the Saniora government in Lebanon and one that is allowing foreign fighters to flow into Iraq from its borders," White House spokeswoman Dana Perino said.

Meanwhile, Nancy Pelosi, arrived in Israel on Friday on her second fact-finding trip to the Middle East since she took over in January.

Among those in her delegation is Rep. Keith Ellison, the first Muslim member of Congress. Others traveling with Pelosi include Rep. Tom Lantos, the Democratic chairman of the House Foreign Affairs Committee.

The group planned to meet with Prime Minister Ehud Olmert, and to travel to the West Bank to meet with Palestinian Authority President Mahmoud Abbas, said Ellison's spokesman, Rick Jauert.

The speaker plans to address the Israeli Knesset on Sunday in what will be her first address to a foreign government legislature and as the highest ranking American woman to speak before the Israeli parliament, according to Pelosi's office.

She is expected to discuss "America's commitment to Israel and the challenges facing the two nations in the Middle East," according to a statement.

~~~~~~~~~~~~~~~~~~~~~~~~~~~~~~~~~~~~~~~~~~~~~~~~~~~~~~

Just curious here, what 'facts' is Pelosi hoping to find - traveling to Syria with "the first Muslim" in congress no less? Yes, congressmen take these 'fact-finding' missions all the time, but is anyone besides me concerned this congress is trying to subvert the President's foreign policy?

tomder55 answered on 04/01/07:

The bad news is she is coming back.Finally has a real exuse for flying on a 747.

If you log onto the NY Post today there is a clever picture of Pelosi dressed as Miss Syria. This all comes in the wake of her refusal to allow Resolution 267(59 co-sponsors including 22 Democrats....a true test of her call for bipartisanship),calling for the immediate and unconditional release of the British sailors to get debated in the House ;basically telling the Brits to drop dead .One just has to wonder what she'll tell the Knesset !

What we have is the Speaker of the House ,the third in line to the President dissing our allies and paying homage to our enemies. The world turned upside down .

This is another in a growing line of Democrats who have travelled to Syria to assure them that a new boss is in town and that they have nothing to fear of the US (remember ...the real enemy is President Bush).Notice that fellow Democrat Elliot Engel ;author of the 'Syria Accountability and Lebaese Sovereignty Act' is NOT one of her entourage. I wonder why ? Ellison is there instead to show just how dhimmi we can become.

Details of the agenda reveal that she plans to make sure our surrender date is acceptable to Assad and the other terror masters ; to summarize the first quarter of shame of her reign and to get marching orders for the second;to formally apologize for the unfair treatment of Syrian musicians on that Northwest flight.

But all of this is seconday to her real reason for going .Let's face it ;it's Easter recess and every other Congress person went home .But if Pelosi went back to San Fran she would have to deal with those Code Pink loonies . So she's going to Syria instead .....her home away from home .

Itsdb rated this answer Excellent or Above Average Answer

Question/Answer
Itsdb asked on 03/30/07 - You gotta love her...

Fresh off her sparring match with the Donald, Rosie O'Donnell sides with Iran and talks up the 9/11 conspiracy. From The View yesterday...

    ROSIE O’DONNELL: Alright we’re going to take a break and come back and talk about the situation in Iran with the soldiers, the British soldiers who were in international waters. The British say they were in the right waters and the Iranians say no they were in our waters, and so it begins.

    JOY BEHAR: Do you think people are now clicking us off because you promoed that?

    O’DONNELL: Well, I don’t know, but we're going to talk about it and I'm sure it will make the news.

    O’DONNELL: Here’s the problem with what you just said, "us," because it's now Britain and the United States pretty much against the rest of the world. How did this happen?

    O’DONNELL: It's just it’s very hard in America when anyone from the mid-east has been so demonized that no matter what, it's impossible for some people to believe that the Iranians in any way could ever do anything ethical in any capacity. They are not people. They have somehow been dehumanized to the point where they’re not people who they’re just the enemy, the terrorists.

    MARCIA GAY HARDEN: That digresses from what the real issues are. You worded a war on terror, personally that is propaganda.

    O’DONNELL: Exactly, Marcia. Thank you.

    HARDEN: I don't like the wording of it.

    O’DONNELL: Right, because it makes people into evil and good.

    BEHAR: This guy Amanidajaja (sic), whatever his name is. He is a bad guy, he is a very bad guy. He stated explicitly he wants to wipe Israel off the map. This guy is a bad guy.

    O’DONNELL: I'm not saying he's a good guy and I want him over for breakfast. No I’m not. I’m saying that in America we are fed propaganda and if you want to know what's happening in the world go outside of the U.S. media because it's owned by four corporations one of them is this one. And you know what, go outside of the country to find out what's going on in our country because it's frightening. It’s frightening.

    HASSELBECK: So you think we're being brainwashed as a whole country? I think not. I think it’s a media

    O’DONNELL: Democracy is threatened in a way it hasn't been in 200 years and if America doesn't stand up we're in big trouble.

    HASSELBECK: Do you believe that the government had anything to do with the attack of 9/11? Do you believe in a conspiracy in terms of the attack of 9/11?

    O’DONNELL: No. But I do believe the first time in history that fire has ever melted steel. I do believe that it defies physics for the World Trade Center Tower Seven, building seven, which collapsed in on itself, it is impossible for a building to fall the way it fell without explosives being involved, World Trade Center Seven. World Trade Center one and Two got hit by plains. Seven, miraculously, for the first time in history, steel was melted by fire. It is physically impossible.

    HASSELBECK: And who do you think is responsible for that?

    O’DONNELL: I have no idea. But to say that we don't know it was imploded, that there was implosion in the demolition, is beyond ignorant. Look at the film. Get a physics expert here from Yale, from Harvard. Pick the school. It defies reason.


So according to Rosie, it's the US and Britian against the rest of the world, which means 'we' have no standing in the current Iranian/UK standoff, we've 'demonized' everyone in the mideast - they're not human - just terrorists ("Don't fear the terrorists. They’re mothers and fathers"). We would also have to go outside of the US to learn about all the 'frightening' things happening here (which might explain why she has "no idea" who attacked us on 9/11).

Rosie also accuses the British of instigating this whole hostage affair on her blog:

    False flag operations are covert operations conducted by governments, corporations, or other organizations, which are designed to appear as if they are being carried out by other entities.

    the british did it on purpose
    into iranian waters
    as
    US MILITARY BUILD UP ON THE IRANIAN BORDER

    we will be in iran
    before summer
    as planned

    come on people
    u have 2 c
    i know u can


Why give this woman any attention? She's dangerous, so here's hoping more people will 'click her off.' She could use some time off to work on juvenile poetry anyway.

tomder55 answered on 03/30/07:

There is not enough tin foil in the world to cover her big head.

With the exception of Liz Hasselback there is no reason to watch 'The View' at all. Laura Ingraham was having a field day ripping Rosie. She asked if Rosie had steel beams on her roof to support her when she hangs upside down.

I'd like her to explain how steel gets made if fire won't melt it. Maybe she should ask some of those physics expert from Yale, or Harvard what smelting means . We Americans are sure a bunch of brainwashed people . I wonder if ABC Network appreciates that accusation .

I just gotta wonder if Rosie thinks the mullahs would let her and her wife live in peace in Iran .

Itsdb rated this answer Excellent or Above Average Answer
kindj rated this answer Excellent or Above Average Answer

Question/Answer
paraclete asked on 03/29/07 - forest plan fools the greens?


Forest fund better than Kyoto, Howard says

March 29, 2007 10:04am
Article from: AAP


THE Government's plan to stop regional deforestation would cut greenhouse gas emissions more than signing the Kyoto Protocol, Prime Minister John Howard said today.

Mr Howard said the $200 million plan would address climate change by slowing the rate of trees being cut down in Indonesia.

"As everybody knows, if you can do that you will reduce greenhouse gas emissions," he said on ABC radio.

"In fact, 20 per cent of global greenhouse gas emissions come from clearing the world's forests and that is second only to emissions from burning fossil fuels to produce electricity.

"And it's more than all the world's emissions from transport. What this initiative will do, in a shorter period of time, is make greater contribution to reducing greenhouse gas emissions than, in fact, the Kyoto protocol."

Under the plan, Australia will form a global fund to fight illegal logging and forest destruction, principally in Indonesia.

It is designed to help developing countries start sustainable forest industries, plant new forests, stop illegal destruction of rainforests, provide monitoring of forest production and education in forest management, and help communities dependent on illegal rainforest timber find alternative jobs.

Environment Minister Malcolm Turnbull said reviving forests was the only way to materially reduce the amount of carbon dioxide in our atmosphere in the near term.

"We will be working with other countries, developed countries, developing countries, to stop the destruction of the world's remaining forests, to promote sustainable management of forests, to promote new planting of forests," he said today.

Labor treasury spokesman Wayne Swan said the idea had merit but Mr Howard could do more closer to home by signing the Kyoto agreement.

"I'm pleased he recognises the importance of that (but) why doesn't he recognise the importance of this country getting on board with a comprehensive and ambitious agenda to combat dangerous climate change at home and abroad," Mr Swan said.

"That's the whole point. If he can recognise it (climate change) in Indonesia why can't he recognise it at home."

Labor Deputy Leader Julia Gillard said Labor would look at the proposal but also challenged the government to effect change in Australia.

The Australian Greens called the Government hypocritical for discouraging logging in developing nations while condoning the same activity here.

Greens leader Bob Brown said Prime Minister John Howard thought he could simply write a cheque to improve his climate change credentials.

"It's a stunning piece of hypocrisy that he is putting $200 million into stopping forest burning in South-East Asia while he is authorising forest burning in southern Australia," Senator Brown said.

"Under his signature yesterday, 14 massive forest burns took place in Tasmania, sending thousands of tonnes of greenhouse gas into the atmosphere.

"Our Prime Minister is a forest fool."

tomder55 answered on 03/29/07:

I think a comprehensive plan of reforestation and controlled land use would bring greater results then the idiotic Kyoto accords .

I see the environmentalists are just a wacky there as here. Conservationists have long recognized the importance of controlled burns and even the benefits of controlled logging in the overall health of forests . Part of the problem that the US has is that undergrowth is permitted to grow unchecked by controlled fires .When they ignite instead we are left to combat uncontrolled fires . If they were serious,they would insist on legislation to selectively thin, selectively log, and selectively burn forests - especially those that are overgrown, old, and dry, and at greatest risk for wildfires . This would expand the logging industries of the countries involved ;create lots of jobs ;keep forests young and healthy;maximize the carbon-uptake efficiency of the forests in question .


Scientists at UC Irvine have discovered that cooling may occur in areas where charred trees in boreal forests expose more snow, which reflects sunlight into space which cancel out any greenhouse gas warming that may occure due to the fire. I know that does not apply to the regions that Howard is concerned about .But I imagine the natural progression of forest growth and destruction by fire that has occured since humans were cave dwellers is probably of little impact in either the temperature of the planet or the climate fluctuations.

paraclete rated this answer Excellent or Above Average Answer

Question/Answer
tropicalstorm asked on 03/28/07 - what to do!

with San Francisco's ban on plastic garbage bags
what are the alternatives for people walking their dogs
and obeying the law to clean up after Rover?
Now I guess you have to walk with the dog chain in one hand and a super dupper pooper scooper and something to bag it in in the other. A paper bag doesn't sound very good of an option.
I think we should come up with an invention here.

tomder55 answered on 03/28/07:

"If You're Gooiiiing To San Francisco...Be Sure To Carry A PooperScooper In Your Hand"

Despite the biodegradability of paper, it takes a hell of a lot more energy to process the wood pulp to make the paper, and the chemicals used aren't exactly environmentally friendly. Plastic is just a recyclable as paper . I don't get it.


I wonder if Gavin Newsom's body condom is made of recycled plastic ?

kindj rated this answer Excellent or Above Average Answer
tropicalstorm rated this answer Excellent or Above Average Answer

Question/Answer
tropicalstorm asked on 03/27/07 - vote for your favorite







tomder55 answered on 03/28/07:

The Evita /Ceasar Romero Joker simularity is scary .

Another one :



ETWolverine rated this answer Excellent or Above Average Answer
Itsdb rated this answer Excellent or Above Average Answer
tropicalstorm rated this answer Excellent or Above Average Answer

Question/Answer
Itsdb asked on 03/27/07 - An unbeatable team?

Hillary Clinton, who I've heard is the "smartest woman in the world," declared at her star-studded fundraiser over the weekened that "Bill is probably the most popular person in the world."

How in the world can anyone else compete with that tandem, especially since she's already been sworn in as Commander-in-Chief?

tomder55 answered on 03/27/07:

He is the most popular . He was at Elton John's birthday so you know it's true. Leading politicians compare themselves to him . He has something to say about all the popular trends from the TV show 24 to Britney Spears (I see a cigar in her future ) .He has recently campaigned to save TV Land channel. He donned spandex to raise money for Evita's campaign at a "spinning class" (sweating to the sleezy )

Sounds like the perfect candidate for his lifetime ambition .....Sec Gen. of the UN .
~~~~~~~~~~~~~~~~~~~~~~~~~~~~~~~~~~~~

Back when Bill Clinton and Hillary got married Bill told her, "There's one thing I want you to know. There's a box under my bed and I don't want you to look in it until I die."

Hillary agreed to this but, over the years, the curiosity got the better of her and she finally looked in it. She found three beer cans and 1.5 million dollars in cash.

When she asked Bill what the beer cans were for, he replied, "Well, those are for all the times I've cheated on you."

Hillary said, "Well, that's not bad after all these years and you being a politician and traveling and all."

She was about to leave, but then she said, "Hey, Bill, what about the 1.5 million dollars?"

Bill replied, "That's for all the times the box got full and I had to cash the cans in."

Itsdb rated this answer Excellent or Above Average Answer
ETWolverine rated this answer Excellent or Above Average Answer
labman rated this answer Excellent or Above Average Answer

Question/Answer
kindj asked on 03/26/07 - Viking's Home Journal

Viking's Home Journal



VIKINGS HOME JOURNAL
(Today's Journal for the modern Viking)

* Three fast and tasty village dog recipes for the working Viking
who doesn't have all day to cook.
* War wound stitchery-Don't throw away those severed body parts.
Needle point tips that can make that foot or arm good as new.
* Surprising reader's poll: 9 out of 10 Viking women are not
satisfied in bed. Find out what they really want.
* Burning pitch techniques that can really let you rain hell on your
neighbors!
* Surrounded by intellectuals-How one Viking escaped. By David-the-Saxon.
* Viking mid-life crisis-Is raping murdering and pillaging all there is ?
* Is your son a Pansy?-A candid article by Erick-the-Red which every father
should read.
* Don't let your Viking Tupperware party end in a blood bath-Do's and don'ts
for a successful evening.
* Detroit unveils the New 89 line of warships-Faster, sleeker, fewer
slaves in the galley!

AT YOUR VILLAGE NEWSSTANDS NOW !

tomder55 answered on 03/27/07:

The Norse god Thor decided to become a mortal for a while and went down to earth.

He met a beautiful girl and they spent the evening together.

In the morning Thor decided to reveal his identity to the woman.

"I'm Thor" he said.

"You're thor!????" she said, "Lithen buthter, I'm tho thor I can hardly thit down!"


ETWolverine rated this answer Excellent or Above Average Answer
kindj rated this answer Excellent or Above Average Answer

Question/Answer
paraclete asked on 03/27/07 - The answer to the SUV?

Trust the French to lead the way and come up with a real alternative to the urban blight of the SUV

Let them ride bikes: Parisians to hit roads
March 26, 2007

PARIS: On July 15, the day after Bastille Day, Parisians will wake up to discover thousands of low-cost rental bikes at hundreds of high-tech bicycle stations scattered throughout the city, an ambitious program to cut traffic, reduce pollution, improve parking and enhance the city's image as a greener, quieter, more relaxed place.

By the end of the year, there should be 20,600 bikes at 1450 stations - or about one station every 250 metres across the entire city. Based on experience elsewhere - particularly in Lyon, France's third-largest city - regular users of the bikes will ride them almost free.

"We think it could change Paris's image - make it quieter, less polluted, with a nicer atmosphere, a better way of life," said Jean-Luc Dumesnil, an aide to the Mayor, Bertrand Delanoe.

Anthonin Darbon, director of Cyclocity, which operates Lyon's program and won the contract to run the one in Paris, said 95 per cent of the roughly 20,000 daily bicycle rentals in Lyon are free because of their length.

Cyclocity is a subsidiary of the outdoor advertising behemoth JCDecaux. London, Dublin, Sydney and Melbourne are reportedly considering similar rental programs.

The Cyclocity concept evolved from utopian "bike-sharing" ideas tried in Europe in the 1960s, most famously in Amsterdam. But in the end, the bikes were stolen and became too beaten-up to ride.

JCDecaux developed a sturdier, less vandal-prone bike, along with a rental system to discourage theft: each rider must leave a credit card or refundable deposit of about €150 ($250). In Lyon, about 10 per cent of the bikes are stolen each year, but many are later recovered.

To encourage people to return bikes quickly, rental rates rise the longer the bikes are out. In Paris, for instance, renting a bike will be free for the first half-hour, €1 for the next, €2 for the third, and so on.

In a complex, 10-year public-private partnership deal, JCDecaux will provide all the bikes and build the pick-up/drop-off stations. Each will have racks connected to a centralised computer that can monitor each bike's condition and location. In exchange, Paris is giving the company exclusive control over 1628 city-owned billboards.

The Washington Post


tomder55 answered on 03/27/07:

I've been to Paris and it was quite a video game navigating the traffic. I expect that on a bike I'd be road kill.

Itsdb rated this answer Excellent or Above Average Answer
paraclete rated this answer Excellent or Above Average Answer

Question/Answer
PrinceHassim asked on 03/26/07 - Iraq.

I told you it was a civil war and ye believed me not!

Since al-Qaeda bombed one of the most important Shiite shrines in Iraq 13 months ago, tens of thousands of Iraqis have been killed and whole neighborhoods have undergone sectarian cleansing. The bombing caused the once-relatively quiescent Shiite community to rise up in a campaign of revenge.

The United Nations Assistance Mission in Iraq said 34,452 Iraqis died last year alone.

Perhaps the American war of independence was nothing but an insurgency. Mel Gibson fought in it in the patriot and he and his militia wore farm clothes not military uniform.

Being a prophet is a hard calling. Ho hum!

tomder55 answered on 03/26/07:

And who burned down the Reichstag ? Check your premise first and ask yourself ;could al-Qaeda possibly have had access to the mosque long enough to set precision munitions in place ? Just ask yourself... who benefits most ? More likely it was an inside job by the Iranian stooge al-Sadr to incite the reaction that it got.You will note that immediately after the explosion al-Sadr lashed out at the US and the Iraqi government ;not al-qaeda.

I won't get into a pissing contest over semantics . Call it a civil war if you choose.I don't think it is an inevidibility unless we retreat .Do you wish for defeat ? I think you should be up front about that .Forgetting the rational for the invasion and if you supported it ;why would you be rooting for a negative outcome ?

ETWolverine rated this answer Excellent or Above Average Answer
PrinceHassim rated this answer Excellent or Above Average Answer

Question/Answer
Itsdb asked on 03/23/07 - Remarks by the President on the Iraq War Emergency Supplemental

THE PRESIDENT: Today I'm joined here at the White House by veterans, family members of people serving in combat, family members of those who have sacrificed. I am honored that they have joined me here today.

Here in Washington, members of both parties recognize that our most solemn responsibility is to support our troops in the war on terror. Yet, today, a narrow majority in the House of Representatives advocated its responsibility by passing a war spending bill that has no chance of becoming law, and brings us no closer to getting our troops the resources they need to do their job.

The purpose of the emergency war spending bill I requested was to provide our troops with vital funding. Instead, Democrats in the House, in an act of political theater, voted to substitute their judgment for that of our military commanders on the ground in Iraq. They set rigid restrictions that will require an army of lawyers to interpret. They set an arbitrary date for withdrawal without regard for conditions on the ground. And they tacked on billions for pet projects that have nothing to do with winning the war on terror. This bill has too much pork, too many conditions and an artificial timetable for withdrawal.

As I have made clear for weeks, I will veto it if it comes to my desk. And because the vote in the House was so close, it is clear that my veto would be sustained. Today's action in the House does only one thing: it delays the delivering of vital resources for our troops. A narrow majority has decided to take this course, just as General Petraeus and his troops are carrying out a new strategy to help the Iraqis secure their capital city.

Amid the real challenges in Iraq, we're beginning to see some signs of progress. Yet, to score political points, the Democratic majority in the House has shown it is willing to undermine the gains our troops are making on the ground.

Democrats want to make clear that they oppose the war in Iraq. They have made their point. For some, that is not enough. These Democrats believe that the longer they can delay funding for our troops, the more likely they are to force me to accept restrictions on our commanders, an artificial timetable for withdrawal, and their pet spending projects. This is not going to happen. Our men and women in uniform need these emergency war funds. The Secretary of Defense has warned that if Congress does not approve the emergency funding for our troops by April the 15th, our men and women in uniform will face significant disruptions, and so would their families.

The Democrats have sent their message, now it's time to send their money. This is an important moment -- a decision for the new leaders in Congress. Our men in women in uniform should not have to worry that politicians in Washington will deny them the funds and the flexibility they need to win. Congress needs to send me a clean bill that I can sign without delay. I expect Congress to do its duty and to fund our troops, and so do the American people -- and so do the good men and women standing with me here today.

Thank you for your time.

~~~~~~~~~~~~~~~~~~~~~~~~~~~~~~~~~~~~~~~~~~~~~~~~~~~~~~

How many Democrats have complained that Bush did not supply our troops with adequate armor and other supplies? Recently?

Edward M. Kennedy and Chris Dodd

Senator Barbara A. Mikulski (D-Md.), Senator Ken Salazar (D-Colo.) and others

Harry Reid

Senator Jeff Bingaman (D-NM)

John Murtha

How many others? Pardon my French, but how the hell do Democrats reconcile slamming the President for not supplying adequate armor while either endorsing or voting to cut the funding they need to survive?

The President is absolutely correct, not only has the Democratic majority in the House ... shown it is willing to undermine the gains our troops are making on the ground, they're more than willing to cut their throats, too.

tomder55 answered on 03/24/07:

I think the vote demonstrated how weak the Democrat majority coalition really is. Pelosi had to use our tax money to bribe and arm twist her fellow Demoncrats to gain the slightest of majorities for this vote. I guess that promise to reform pork barrel legislation is off the table ,much like their other lies about reform.

I guess it will probably be lost on most people that Pelosi and Murtha treated funding for our troops as the equivalent of just another piece of highway legislation. The bill shows more concern of the needs of a handful of spinach farmers than to our troops.

Ironically this vote will do nothing to take the pressure off of Pelosi by the Code Pinkos (unless she cut an under the table deal with Maxine Waters and the "pull out now" crowd that isn't being reported ).They will still protest outside her home. But at least she will meet her mortage payment. The same cannot be said for the families of the troops

But anyway my real point is that if she has the mandate she claims ,she wouldn't have needed all the machinations and gyrations to get this bill passed. I have to wonder if she has exhausted all her political capital .If a deal was cut with Maxine Waters it will be revealed soon enough and then we'll see how blue the blue dogs really are .

Time for some Mitch-slapping .McConnell has to get the timetable and if possible the pork out of the Senate version of the bill.Otherwise it's time for the President to wield his veto pen. I'd personally rather see the Senate deal with it however. The Democrats will not be shy in levelling the charge that it was not them ,but the President ,who didn't support the troops by vetoing the funding legislation.

Itsdb rated this answer Excellent or Above Average Answer

Question/Answer
ETWolverine asked on 03/22/07 - Some prices of common products (approximations only)

Jack Daniels Bourbon =$94.90 per gallon

1792 Bourbon = $149.90 per gallon

Reposado Tequila = $139.90 per gallon

Balvinie Single Malt Scotch Whiskey = $179.90 per gallon

Crappy domestic merlo = $29.90 per gallon

Crappy domestic beer = $23.09 per gallon

Sparkling water = $18.90 per gallon

Pantene Pro V shampoo with conditioner = $41.50 per gallon

Listerine mouthwash = $26.52 per gallon

Toothpaste (cheap stuff) = $42.45 per gallon

Evian bottled water = $11.43 per gallon

Tropicana Orange Juice = $7.99 per gallon

Coca Cola = $6.20 per gallon

Gatorade sports drink = $14.56 per gallon

Pet Promise canned dog food = $16.25 per gallon


And the cost of gasoline at the pump averages about $2.57 per gallon (nationawide average as of 3/19/07).

What in the hell is the big issue with gas prices. Why is everyone in such an uproar over the high cost of gas or the profits that oil companies are making? Your shampoo maker is making more per gallon than your oil company is. Why isn't anyone talking about the high price of shampoo and the shampoo company profits that should be taken away from them?

Elliot

tomder55 answered on 03/23/07:

I think a better comparison would be what the price of gas should be if it had followed the inflation rate in the last 20 years .I believe that even with the spikes in the last year or two ,the inflation adjusted price in 1981 was more . Captn has a point . Petroleum products drive the economy and price fluctuations do have a greater ripple effect . All told ;I'd rather put the $35 fill up to get a magnum of Moet .Just imagine what I'd save if I did not have to fire up the heater the last couple of months . I could've had my fill of liquid refreshments .

ETWolverine rated this answer Excellent or Above Average Answer
Itsdb rated this answer Excellent or Above Average Answer

Question/Answer
Itsdb asked on 03/22/07 - GOP Scandal?

Followup to Sapph's youtube video...

As I noted on here post, ABC (and the SF Chronicle and others) took the bait and hinted at a GOP conspiracy:

    CLAIRE SHIPMAN: "Robin, the ultimate conspiracy theory, some Democrats think a Republican operative could be responsible because it not only makes Hillary Clinton look bad but Barack Obama look bad, since it's an attack ad."

    ROBIN ROBERTS: "Something to think about."


Wrong!

    Mystery creator of anti-Clinton ad steps forward
    By Jim Kuhnhenn, Associated Press

    WASHINGTON — The Internet video sensation that targeted Sen. Hillary Rodham Clinton now has rival Sen. Barack Obama on the spot.

    Heralded by many as the embodiment of Web-driven citizen activism, the mysterious YouTube ad now stands revealed as the work of a Democratic operative employed by a consulting firm with Obama links.

    "It's true ... yeah, it's me," said Philip de Vellis, a 33-year-old strategist with Blue State Digital, a Washington company that advises Democratic candidates and liberal groups.

    Blue State designed Obama's website, and one of the firm's founding members, Joe Rospars, took a leave from the company to work as Obama's director of new media.

    Obama, Blue State and de Vellis all say de Vellis acted on his own. De Vellis left the company on Wednesday. He said he resigned; Thomas Gensemer, the firm's managing director, said he was fired.

    The entire episode hangs a cloud over the Obama camp.

    Since he arrived on the national political scene, Obama has won convert after convert with a vow to rise above the bare-knuckle fray of politics.

    However tenuous, any link to the ad, with its Orwellian image of Clinton as Big Brother, raises questions the Obama camp would rather not face.

    In a statement, the Obama campaign said it "had no knowledge and had nothing to do with the creation of the ad."

    "Blue State Digital has separated ties with this individual and we have been assured he did no work on our campaign's account," it added.

    De Vellis, in a blog he wrote after he had been identified by Huffingtonpost.com, appeared to acknowledge the trouble he had brewed. "I support Senator Obama," he wrote. "I hope he wins the primary. (I recognize that this ad is not his style of politics)."

    It's not as if Obama's campaign is not willing to mix it up.

    Last month, Obama adviser Robert Gibbs referred to the infamous Lincoln Bedroom sleepovers of the Clinton era after the Clinton team demanded that Obama apologize for anti-Clinton remarks by Hollywood producer and Obama backer David Geffen.

    And this week, Obama consultant David Axelrod publicly challenged Clinton strategist Mark Penn over his characterization of Obama's stance on the war in Iraq.

    The unmasking of de Vellis also cracks the enticing image of the Internet as a freewheeling arena where average citizens engage in vigorous, often provocative, discourse.

    De Vellis said he acted like any techno-savvy, politically attuned Web surfer. He said he worked on a Sunday in his apartment, using his Mac computer and video editing software to alter an updated version of a classic Apple ad that aired during the Super Bowl in 1984.

    But the fact remains that de Vellis was a political professional. He had worked for Democratic Rep. Sherrod Brown in his successful campaign for U.S. Senate in Ohio. And he was working for a firm with political clients, including Obama.

    "Obviously some people are going to look at this and see that I'm working in politics and they'll think that it's kind of disingenuous or not genuine," de Vellis said in an interview with The Associated Press. "I still think that ordinary citizens can change politics. It could have been anyone else who could have made this ad."

    The ad portrayed Clinton on a huge television screen addressing an audience that sat in a trancelike state. A female athlete, running ahead of storm troopers, sprints into the auditorium and tosses a hammer at the screen, destroying Clinton's image. "On January 14th the Democratic primary will begin," the text states. "And you will see why 2008 isn't going to be like 1984." It signs off with "BarackObama.com"

    In the interview, and later in a blog written for the Huffington Post, de Vellis expressed pride in his creation, while acknowledging that his employers are "disappointed and angry at me, and deservedly so."

    "It changes the trajectory of my career," he said.


OK, who'll be first to come out and apologize for blaming this on Republican dirty tricks?

tomder55 answered on 03/22/07:

I doubt there will be and apology but if I was involved in any other campaign I'd hire the guy in a nano-second if the Obama camp is indeed dumping him.

Itsdb rated this answer Excellent or Above Average Answer

Question/Answer
tropicalstorm asked on 03/22/07 - Hybrid green car lie

Once I get more facts and sources I plan to post this on the Christianity board----oooowwwwwhh!

"The Prius costs an average of $3.25 per mile driven over a lifetime of 100,000 miles"
"The Hummer costs a more fiscal $1.95 per mile to put on the road over an expected lifetime of 300,000 mile"

green car’ is the source of some of the worst pollution in North America; it takes more combined energy per Prius to produce than a Hummer.


the Prius is partly driven by a battery which contains nickel. The nickel is mined and smelted at a plant in Sudbury, Ontario. This plant has caused so much environmental damage to the surrounding environment that NASA has used the ‘dead zone’ around the plant to test moon rovers. The area around the plant is devoid of any life for miles.
The plant is the source of all the nickel found in a Prius’ battery and Toyota purchases 1,000 tons annually. Dubbed the Superstack, the plague-factory has spread sulfur dioxide across northern Ontario, becoming every environmentalist’s nightmare.
“The acid rain around Sudbury was so bad it destroyed all the plants and the soil slid down off the hillside,” said Canadian Greenpeace energy-coordinator David Martin during an interview with Mail, a British-based newspaper.
The Prius is powered by not one, but two engines: a standard 76 horsepower, 1.5-liter gas engine found in most cars today and a battery- powered engine that deals out 67 horsepower and a whooping 295ft/lbs of torque, below 2000 revolutions per minute. Essentially, the Toyota Synergy Drive system, as it is so called, propels the car from a dead stop to up to 30mph. This is where the largest percent of gas is consumed. As any physics major can tell you, it takes more energy to get an object moving than to keep it moving. The battery is recharged through the braking system, as well as when the gasoline engine takes over anywhere north of 30mph. It seems like a great energy efficient and environmentally sound car, right?

The plant is the source of all the nickel found in a Prius’ battery and Toyota purchases 1,000 tons annually. Dubbed the Superstack, the plague-factory has spread sulfur dioxide across northern Ontario, becoming every environmentalist’s nightmare.
“The acid rain around Sudbury was so bad it destroyed all the plants and the soil slid down off the hillside,” said Canadian Greenpeace energy-coordinator David Martin during an interview with Mail, a British-based newspaper.
All of this would be bad enough in and of itself; however, the journey to make a hybrid doesn’t end there. The nickel produced by this disastrous plant is shipped via massive container ship to the largest nickel refinery in Europe. From there, the nickel hops over to China to produce ‘nickel foam.’ From there, it goes to Japan. Finally, the completed batteries are shipped to the United States, finalizing the around-the-world trip required to produce a single Prius battery. Are these not sounding less and less like environmentally sound cars and more like a farce?
Wait, I haven’t even got to the best part yet.
When you pool together all the combined energy it takes to drive and build a Toyota Prius, the flagship car of energy fanatics, it takes almost 50 percent more energy than a Hummer - the Prius’s arch nemesis.
Through a study by CNW Marketing called “Dust to Dust,” the total combined energy is taken from all the electrical, fuel, transportation, materials (metal, plastic, etc) and hundreds of other factors over the expected lifetime of a vehicle. The Prius costs an average of $3.25 per mile driven over a lifetime of 100,000 miles - the expected lifespan of the Hybrid.
The Hummer, on the other hand, costs a more fiscal $1.95 per mile to put on the road over an expected lifetime of 300,000 miles. That means the Hummer will last three times longer than a Prius and use less combined energy doing it.
SO, IF you are really an environmentalist - ditch the Prius. Instead, buy one of the most economical cars available - a Toyota Scion xB. The Scion only costs a paltry $0.48 per mile to put on the road. If you are still obsessed over gas mileage - buy a Chevy Aveo and fix that lead foot.
One last fun fact for you: it takes five years to offset the premium price of a Prius. Meaning, you have to wait 60 months to save any money over a non-hybrid car because of lower gas expenses.

green lie
)

tomder55 answered on 03/22/07:

Both vehicles are overhyped for different reasons .

The Belmont Club had a nice posting on this subject yesterday .

Wrechard asks this compelling question :

The sad fact about most of these environmental question is that it may require us to trade off one set of objectives against another. Maybe the "world" should decide which it values more. In the case of "Global Warming" for example, many of the policies designed to reduce "Greenhouse Gases" may exacerbate poverty in the Third World. How does one rank different goals -- such as for example reducing "greenhouse gases" and reducing hunger -- and combine them into a single policy?

I have noted in the past the the Prius is constantly down graded in MPG ratings . It turns out that on short drives the Prius battery works fine . But if you are a commuter who drives significant highway miles the mpg is actually reduced because the car depends more on the traditional combustion engine .

Another example of wrechard's point is in the case of incandescent light bulb vs. fluorescent lights. Fluorescent light bulbs contain mercury. The standard fluorescent lamp contains approximately 20 milligrams of mercury.improper disposal of fluorescent lamps can contaminate the environment. Over 800 million lamps are produced each year to replace 800 million lamps that are disposed. 1 gram of mercury is enough to contaminate a 2-acre pond,so there is enough mercury in those bulbs to contaminate 20 million acres of water.The manufacture of PCBs was discontinued for just this reason .

But as you see there is a movement across the country by State and local governments to mandate the use of fluorescents because they are more energy efficient than the standard bulb. But I have yet to hear any proposal by these same gvts. to mandate a recycling program to deal with the huge increase in these these bulbs and their proper disposal . Safe disposal requires storing the bulbs unbroken until they can be processed. Once they are cracked for any reason they discharge the toxins.

My opinion is that no one can convince me that there is a compelling need to own a Hummer. To me it's a luxury car and I think that they should be taxed differently than your standard car or light truck. It is a gas guzzler and a road hog ;especially in an urban setting .Edmunds lists the 5 year, 75,000 mile cost of a Prius to operate at $35,000. The Hummer H2 is $70,000. Consumer Reports has the Hummer has one of the worse reliability records ;the Prius ranks number 1 in customer satisfaction ( although I suspect that Prius drivers are trying to make a statement : look at me ...I am green!!)

Also I seriously doubt that most Hummers actually reach that magical 300,000 milage .I also doubt the life expectency of the Prius is only 100,000 . The warranty is 100,000 . So the comparision is misleading .Also I question the travel route of the battery (to China, Europe, the U.S., and Japan) .Toyota did not become a leading car seller by being inefficient. Most likely the US auto companies are going to follow Toyota's lead and introduce their own version of hybrid autos. Why ? Because there is a huge consumer demand for them. Yes I think hybrids in their present form are over-rated ,but I do think they are part of the solution towards a goal of energy independence.

As for the zinc factory . I suspect they do alot more than just make batteries for the Prius.But regardless I don't think anyone would disagree that smoke stack scrubbers should be employed and that should be mandated by law.

ETWolverine rated this answer Excellent or Above Average Answer
labman rated this answer Excellent or Above Average Answer
tropicalstorm rated this answer Excellent or Above Average Answer

Question/Answer
kindj asked on 03/21/07 - Sudden Jihad Syndrome

Perspectives: 'Sudden Jihad Syndrome' - A reason to carry firearms for self-defense
Larry Pratt - Guest Columnist
OneNewsNow.com
March 8, 2007


I have compiled (quite easily, I might add) a list of murders and attempted murders carried out by Muslims acting on the basis of what the founder of Islam taught them.

Convinced that they will go to heaven if they die killing infidels who Mohammed taught his followers to hate, some Muslims don't bother joining al Qaeda or some other organized band of thugs. They get so filled with hatred from hearing Islamic sermons and visiting jihadi web sites that they decide to become freelance Jihadis.

The condition is recognizable and has been labeled Sudden Jihad Syndrome (SJS) -- although some people are unable, or unwilling, to recognize SJS. Politically correct spokesmen for the FBI and other agencies seem to be under orders to issue a denial that a SJS-related murder could possibly have anything to do with Islam.

The rest of the population should come to grips with the fact that many in America are susceptible to SJS. The best antidote for many SJS-related acts of terrorism is a bullet fired from the gun of a prospective victim.

The following list is offered to show that SJS is a reality, and that rational Americans should be prepared to deliver the most effective known antidote – a bullet administered at the first sign of an outbreak of SJS.



March 1994 – Rashid Baz, a Muslim from Lebanon, opened fire on a van containing members of the Lubavitch Hassids in Brooklyn. One was killed.

February 1997 – Palestinian-born Ali Abu Kamal opened fire on the observation deck of the Empire State Building, killing a tourist and injuring six other people before committing suicide. His daughter, Linda Kamal, said in 2007 that the family is tired of lying and admitted that a letter had been found on her father's body explaining his Muslim hatred for Israel and America.

July 2002 – Egyptian-born Hesham Mohammed Hadayet walked into the Israeli Airlines El Al terminal at the Los Angeles airport and began shooting Jews. He killed two and injured another four. He was known to sympathize with al Qaeda.

September 2002 – Patrick Gott killed one and wounded another in the New Orleans airport. He had entered the terminal with a shotgun and his Koran.

October 2002 – John Mohammed and Lee Malvo killed 13 people in the Washington, DC area. Both were converts to Islam and had attended a jihad training camp in southwestern Virginia.

August 2003 – Mohammed Ali Alayed almost totally decapitated his erstwhile Jewish friend, Ariel Sellouk, following Alayed’s getting serious about his Islamic faith. He went to a mosque after killing Sellouk.

October 2005 – Joel Henry Hinrichs III, a convert to Islam, was an engineering student at Oklahoma University. His student career ended when a bomb he had strapped on himself went off prematurely outside a crowed stadium, killing only himself. Police subsequently cleared explosives from the apartment that Hinrichs had shared with Muslim students from Pakistan.

April 2006 – Muslim Ayhan Surucu was so angry when his sister started to wear make-up and date men in Berlin, Germany, that he put a gun to her head at a bus stop and killed her. Boys at a nearby school, attended mainly by the children of immigrant Muslim families, cheered and applauded when news of the murder reached them.

May 2006 – Mohammed Reza Taheri-azar, a University of North Carolina at Chapel Hill student, rented an SUV and drove it through a crowded part of the campus-- intentionally trying to hit people and wounding nine. In a series of letters to the student newspaper, he explained that he acted in obedience to Koranic dictates.

June 2006 – Michael Ford, a Muslim convert, walked into work at a Safeway warehouse in Denver and opened fire on his coworkers, killing one and injuring five. Relatives explained that he was being teased at work because he’s a Muslim and he could not take it anymore.

July 2006 – Naveed Alzal Haq, a Pakistani, walked into the Jewish Federation Center in downtown Seattle and shot six women-- killing one, and wounding five( one of whom was pregnant). He stated matter-of-factly: "I am a Muslim American angry at Israel."

August 2006 – Omeed Aziz Popal, a Muslim Afghan refugee, used his SUV as a weapon and ran down at least 14 people and a bicyclist in the San Francisco Bay area. He was targeting Jewish neighborhoods to terrorize.

January 2007 – A 22-year-old Muslim, Ismail Yassin Mohamed, stole a car in Minneapolis and rammed it into other cars before stealing a van and doing the same, injuring several drivers and pedestrians at crosswalks and on sidewalks before police caught up with him. Mohamed called himself a "terrorist."

February 2007 – Ibrihim Ahmed, a Nashville cab driver and Muslim, was enraged that two passengers did not agree with him about Islam. When they got out of the cab, he tried to run them down, striking one in a parking lot.

February 2007 – Sulejman Talovic, a Bosnian Muslim immigrant, went to a Salt Lake City mosque on a Friday night. Then he went to one of only two malls in the state which prohibits civilian carrying of concealed weapons. He killed five before an off-duty cop (not subject to ban) used a concealed firearm stopped his murder spree.

Cornell University did a study estimating that there are seven million Muslims in the U.S. If even a single percent of that population is motivated to go kill a few infidels for Allah, the country would be facing 70,000 murderers on the loose.

The examples that I have found of Islam being the driving force for murder show that the prospect of Sudden Jihad Syndrome should be reason to loosen the country’s concealed carry laws. The Utah Jihadi did not have a concealed carry permit, but he carried his weapons to the scene of the crime concealed anyway. Why should the laws make it harder for the rest of us to counter what criminals are already doing?

Put another way, our current restrictions on concealed carry on the books in most states facilitate murder by tying the hands of victims.
The law should be on our side, not on the side of the bad guys.

~~~~~~~~~~~~~~~~~~~~~~~~~~~~~~~~~~~~~~~~~~~~~~~~~~~~~~

Seems like an irrefutable argument to me.

How 'bout you?

DK

tomder55 answered on 03/21/07:

I'd have to say that besides the DC snipers , John Mohammed and Lee Malvo ,most of the cases were intentionally underplayed in the US press and mass media.

I'm with you on the conceal and carry laws . There was predictions of a return to the Wild Wild West after Fla. loosened it's gun laws and those fears have proven to be unfounded .Not in Florida or in any of the other 37 states with legalized concealed carry laws has the violent crime rate increased .Time after time it has been proven that guns in the hands of law abiding citizens is not an issue ;it saves lives . But even more basic is that there is a constituional guaranteed right to bear arms .

Itsdb rated this answer Excellent or Above Average Answer
kindj rated this answer Excellent or Above Average Answer

Question/Answer
tropicalstorm asked on 03/20/07 - is she hill=arious?????

Remake of an old super bowl commerical

big sister hillary )

http://www.youtube.com/watch?v=99zvYV4kiKI

or
http://www.youtube.com
type in hillary big sister or vote different

tomder55 answered on 03/20/07:

let's see ...Hillary spouting pablum and nonsense cliches ...and a desperate attempt to shut her up. Even though it is clearly a rip off of the 񓟠 " Apple ad and probably violates copyrights ,it is nonetheless a powerful message. It could end up being the most powerful attack ad since the 1964 anti-Goldwater ad where the girl picking daiseys gets nuked. It morphs everyone's fears of a Evita Presidency into a single image. The far right and the far left find common ground. It strips away all the warm colorful images she has tried to convey in her internet forays and leaves the essence unvanished .....Big Sister.

I'd bet that this ad gets more hits in less time than the combined forces of both campaigns and all their Madison Av. polish could generate . If the campaigns are not involved then anyone with a pc and a creative bone in their body can post ads that are campaign changing . This could represent a moment in history that will not be forgotten . For people who have been looking for a new pardigm from business as usual politics, the new era has arrived .This could easily make McCain-Feingold obsolete .......and it never would've been possible without the internet . No wonder Hillary talks so often of regulating the net !

Itsdb rated this answer Excellent or Above Average Answer
tropicalstorm rated this answer Excellent or Above Average Answer

Question/Answer
tropicalstorm asked on 03/19/07 - when will we hear the truth about dry runs

I keep hearing stories of dry runs like the other day the latest one I heard is a guy carrying metal bomb making devices in his rectum
and today the Pittsburgh airport found threatening letters

http://warroom.com/ )

ABOUT THOSE IMAMS
By RICHARD MINITER

THE notorious case of U.S. Airways Flight 300 gets stranger by the minute, as more facts emerge about why six traveling Muslim clerics were asked to deplane. A passenger on that flight - I'll call her "Pauline" - has inadvertently publicized some facts via a much-forwarded e-mail; she gave me more details in an interview this week. The airport police report confirms some of her claims and holds more revelations of its own. And U.S. Airways spokeswoman Andrea Rader also confirmed much of Pauline's account.

One detail that's escaped most reports is that other Muslim passengers were left undisturbed and later joined in a round of applause for the U.S. Airways crew. "It wasn't that they were Muslim," says Pauline. "It was all of the suspicious things they did." Sitting by Minneapolis-St. Paul's Airport Gate C9, she noticed one imam immediately. "He was pacing nervously, talking in Arabic," she said.

As the plane boarded, she said, no one refused to fly. The public prayers and an Arabic phone call triggered no alarms. But then a note from a passenger about suspicious movements of the imams got the crew's attention. To Pauline, everything seemed normal. Then the captain - in classic laconic pilot-style - announced there had been a "mix-up in our paperwork" and that the flight would be delayed.

In reality, the crew was waiting for the FBI and local police to arrive.

Contrary to press accounts that a single note from a passenger triggered the imams' removal, Captain John Howard Wood was weighing multiple factors.

* An Arabic speaker was seated near two of the imams in the plane's tail. That passenger pulled a flight attendant aside and, in a whisper, translated what the men were saying: invoking "bin Laden" and condemning America for "killing Saddam," according to police reports.

* An imam seated in first class asked for a seat-belt extender - the extra strap that obese people use because the standard belt is too short. According to both an on-duty and a deadheading flight attendant, he looked too thin to need one. A seat-belt extender can easily be used as a weapon - just wrap one end around your fist, and swing the heavy metal buckle.

* All six imams had boarded together, with the first-class passengers - even though only one of them had a first-class ticket. Three had one-way tickets. Between the six men, only one had checked a bag.

And, Pauline said, they spread out - just like the 9/11 hijackers. Two sat in first class, two in the middle and two back in the economy section, police reports show. Some, according to Rader, took seats not assigned to them.

* Finally, a gate attendant told the captain she was suspicious of the imams, according to police reports.

So the captain made his decision to delay the flight based on many complaints, not one. He also consulted a federal air marshal, a U.S. Airways ground-security coordinator and the airline's security office in Phoenix. All thought the imams were acting suspiciously, Rader told me.

One more odd thing went unnoticed at the time: The men prayed both at the gate and on the plane. Yet observant Muslims pray only once at sundown, not twice.

"It was almost as if they were intentionally trying to get kicked off the flight," Pauline said.

While the imams were soon released, Pauline is fuming: "We are the victims of these people. They need to be more sensitive to us. They were totally insensitive to us and then accused us of being insensitive to them."

The flight was delayed for some 31/2 hours. Bomb-sniffing dogs swept the plane, and every passenger got re-screened.

"I think it was either a foiled attempt to take over the plane or it was a publicity stunt to accuse us of being insensitive," Pauline told me. "It had to be to intimidate U.S. Airways to ease up on security."

So far, U.S. Airways refuses to be intimidated, even though the feds have launched an investigation. "We are absolutely backing this crew," Rader said.

Tucked away in the police report is this little gem: One imam had complained to a passenger that some nations don't follow sharia law and had said his job in Bakersfield, Calif., was a cover for "representing Muslims here in the U.S."

What are the imams really up to? Something more than praying, it seems.

Some argue that these Imans behaved this way in hopes that they would be deplaned. Once deplaned, they could yell discrimination. Others, like contributor to "The Aviation Nation", Annie Jacobsen, would argue that it is distinctly possible that they were practicing a dry run. That they may have wanted to see just how much they could get away with for a jihad attempt further on down the line. Annie makes a good argument for dry runs; the jihadists have been known to make several attempts on one target before they get it right. For example, Annie sites; "The recent dry run or probe on American Airlines flight 63 occurred on a flight that has already been saved once from a terrorist attack by the heroics of flight crew and passengers. If you recall, it was American Airlines flight 63 that "shoe bomber" Richard Reid tried to take down with explosives hidden in his shoes, in December of 2001". Click here to read her article

When it comes right down to it, I think that both arguments are quite right. While the Imans could have been hoping to make a cry of discrimination -- it doesn't negate the idea that, on it's own -- that, too, is a form of a dry run. It really is the perfect plan isn't it? Cry discrimination now on a "dry run" so that later, when they are fully prepared to execute another attack -- they are assured that their plan would go uninterrupted because we have become conditioned to be sensitive. We are learning that a Muslim passenger can do what the rest of us can not -- make those around us uncomfortable. Remember the story of the woman who became "claustrophobic" on the British flight? She had to be overcome by her fellow passengers and escorted off of the plane.

The reason she was escorted off the plane was because she made others on the plane uncomfortable by her behavior. That was o.k. because she was a white female. But, if an Arab-looking, loudly praying, Muslim who's behavior is questionable, should make you uncomfortable, you'd better just shake that feeling off as a discriminating thought.

While shaking it off -- we've once again made ourselves easy targets. Or, as "Pauline" would say; "we are the victims".

tomder55 answered on 03/19/07:

Snakes on a plane

The Immams are now suing the passangers . I think the law suits against both the airlines and the passangers should be vigorously contested .

This is terrorism's M.O. ....FEAR . If we are afraid to even report a perceived threat then our ability to fight terrorism is severely weakened .

The fact is that the Immam's actions caused this response. It was terrorist profiling not racial profiling . The reality is that no one is in a better position to change this perception than the Immams themselves .

Itsdb rated this answer Excellent or Above Average Answer
paraclete rated this answer Excellent or Above Average Answer
tropicalstorm rated this answer Excellent or Above Average Answer

Question/Answer
Itsdb asked on 03/16/07 - Iraqi leader thanks U.S. for assistance

An Iraqi leader told President Bush on Thursday that the latest security crackdown in his country is working, but cautioned it will take more time. He thanked the American people for supporting Iraq.

"We are not finished, but we are doing better than expected," said Vice President Adel Abdul-Mahdi, a leading politician in the powerful Shiite Supreme Council for the Islamic Revolution in Iraq.

Bush met with Abdul-Mahdi in the Oval Office for just under an hour. The meeting happened as House Democrats advanced legislation setting a timetable for a U.S. troop withdrawal despite a veto threat from the White House.

"I appreciate the progress that you're making," Bush told the Iraqi leader. "I know it's hard work. It's hard work to overcome distrust that has built up over the years because your country was ruled by a tyrant."
~~~~~~~~~~~~~~~~~~~~~~~~~~~~~~~~~~~~~~~~~~~~~~~~~~~~~~~

Ain't that a kick in the teeth, Democrats trying to pull the rug out from under the Iraqis just as they express gratitude...

tomder55 answered on 03/17/07:

Last month there was an assassination attempt against Adel Abdul Mahdi .The bombing was an inside job. Loyalists of al-Sadr bombed the Public Works Ministry .He received minor wounds. Twelve were killed and 42 wounded.

The assassination of Vice President Mahdi ,an economist ,would have serious political rammifications .He is an influencial politician, and is next in line to succeed President Talibani.The assassination of a prominent Shia politician would help promote Sadr's image as protector of the Shia, and allow him to return to Iraq from his self imposed exile in Iran unnopposed.

Sadr's grip on the Mahdi Army has slipped and that in itself is a positive development that is a direct result of the surge. We have further undermined his power by directly negotiating with the Mahdi militia in his absence.

The wife of Ayatollah Abdul Majid al-Khoei , the pro-U.S. Shia cleric who was executed just days after the liberation of Iraq, has openly accused Sadr of murdering her husband. He was knifed to death by a mob on the steps of the Imam Ali shrine in Najaf. Khoei's wife claims Sadr ordered the execution. Sadr was charged with the murder of Khoei by an Iraqi court in 2003, but the government has never executed the arrest warrant.

As far as the Dems. What more can be said about how wrong they are ? Their whole political future rests in the defeat of the US .What an unfortunate position to put oneself into.

Itsdb rated this answer Excellent or Above Average Answer

Question/Answer
Itsdb asked on 03/16/07 - Now this is creepy

Barack Obama Thinks John Edwards is 'Kind of Cute'
By Michael Amsterdam
Mar 15, 2007

    At least he didn't call him gay. Or worse drop the 'f' bomb. Barack Obama, one of the leading candidates for president from the Democratic side of the aisle has basically dubbed one of his rivals a 'cute.' Shockingly, he wasn't speaking of former First lady and frontrunner Hillary Clinton. According to a report from 'The Hill' Barack saved his praise for the man dubbed the 'Breck Girl' by liberal New York Times columnist Maureen Dowd.

    Elana Schor reports from the paper: Sen. Barack Obama (D-Ill.) slipped in a compliment — of sorts — about a fellow 2008 hopeful during his appearances on the Iowa stump last weekend. “I want to wait and hear what John Edwards has to say, he’s kind of good-looking,” Obama envisioned Iowa caucus-goers from the small town of Clinton telling themselves. During an appearance in West Burlington, Iowa, the phrase appeared again, this time with Edwards as “kind of cute.”

    Oh the humanity. Edwards recently made headlines by pulling out of a debate in Nevada because it was co-hosted by Fox News. Besides the Ann Coulter 'f' bomb, Edwards is fighting a perception that he is 'soft.' Was this an attempt by Obama to marginalize Edwards even further?

    The Hill notes that at least one Edwards supporter was nonplussed by the reference, coming as Obama stresses rising above petty politics and chafes at press attention to his own good looks. “Substance is what makes Edwards stand out from the pack,” the supporter said. “Amid the hype, he’s proven time and again to be the candidate with substantive ideas, and that’s what matters most to caucus-goers.”


I'm not sure which is more creepy, Obama calling Edwards cute, that the writer of this article being 'shocked' that he wasn't referring to Hillary - or that I might honestly have to consider him cuter than Evita.



tomder55 answered on 03/17/07:

Maybe Obama is secretly jealous of Edwards hair doo.

“Substance is what makes Edwards stand out from the pack,”
thanks for the laugh of the week. The only person of substance in the Democrat pack is Bill Richardson but I disagree too much with the positions he takes.

See where Bill Clinton is blasting the NY Slimes for not going after Obama hard enough ? While his wife waffles on a daily basis about what her Iraq position should be ,Clinton says tthat Obama was hesitant about taking a stand initially about the war at the outset and claimed that had Obama been in the Senate at the time he would've voted for the war .He is probably correct about that but of that we will never really know. In Clinton's view if Obama is going to make an issue out of the fact that he didn't vote for the war, then more attention should be paid to the fact that Obama didn't have to grapple with the intel that Senators had to deal with, and can't be certain what he would have done if he had. Curtis Sliwa was at a Hillary fundraiser and reported that Clinton was railing about the Slimes not giving Hillary a fair shake because she refuses to apologize for her vote authorizing the Iraq war and has not adequately taken Obama to task .

Imagine that.....the Slimes taking sides....hard to believe .

I think the real point is that all the Senators were privy to the most recent intel and armed with that ,they determined that overthrowing Saddam was justified .Clinton's view however is correct. The Slimes treats Obama with favor because they think his opposition is some kind of moral stand he takes while the truth is probably that given the intel ,Obama most likely would've been persuaded to vote in favor of the war. But ,if this is what the Clintonoids are clinging to then their campaign is in more danger than I thought. It's amazing that the Clinton machine is struggling against such a light weight.

Looks like the spectre of Obama's Islamic childhood will dog him throughout the campaign. The LA Slimes has an article that addresses this . Brit Humes at Fox suggests that the article contradicts Obama's accounts of his childhood.

Barack Obama's chief spokesman has been saying since January that the Democratic presidential candidate has never been a practicing Muslim. Now the Los Angeles Times is reporting that Obama was registered as a Muslim when he attended primary school in Indonesia as a boy. The Times quotes friends and teachers as saying Obama took Muslim religious classes in school and went to prayers at a local mosque.

The Obama campaign reacted to the story this morning by reiterating its position that the senator, quote, "has never been a Muslim, was not raised a Muslim, and is a committed Christian."




Itsdb rated this answer Excellent or Above Average Answer

Question/Answer
Itsdb asked on 03/16/07 - Diagnosis: Cheney

Sorry folks, not hogging the board just too much juicy stuff today :)

    By Charles Krauthammer
    Friday, March 16, 2007; Page A21

    "What is wrong with Dick Cheney?" asks Michelle Cottle in the inaugural issue of the newly relaunched New Republic. She then spends the next 1,900 words marshaling evidence suggesting that his cardiac disease has left him demented and mentally disordered.

    The charming part of this not-to-be-missed article (titled "Heart of Darkness," no less) is that it is framed as an exercise in compassion. Since Cottle knows that the only way for her New Republic readers to understand Cheney is that he is evil -- "next time you see Cheney behaving oddly, don't automatically assume that he's a bad man," she advises -- surely the generous thing for a liberal to do is write him off as simply nuts. In the wonderland of liberalism, Cottle is trying to make the case for Cheney by offering the insanity defense.

    She doesn't seem to understand that showing how circulatory problems can affect the brain proves nothing unless you first show the existence of a psychiatric disorder. Yet Cottle offers nothing in Cheney's presenting symptoms or behavior to justify a psychiatric diagnosis of any kind, let alone dementia.

    What behavior does she cite as evidence of Cheney's looniness?

    (a) Using a four-letter word in an exchange with Sen. Patrick Leahy. Good God, by that standard, I should long ago have been committed and the entire borough of Brooklyn quarantined.

    (b) "Shoot a man in the face and not bother to call your boss 'til the next day?" Another way of putting that is this: After a hunting accident, Cheney tried to get things in order before going public. Not the best decision, as I wrote at the time, but perfectly understandable. And if that is deranged, what do you say about a young Teddy Kennedy being far less forthcoming about something far more serious -- how he came to leave a dead woman at the bottom of a pond? I am passing no judgment. I am simply pointing out how surpassingly stupid it is to attribute such behavior to mental illness.

    (c) Longtime associate Brent Scowcroft quoted as saying, "Dick Cheney I don't know anymore." Well. After Sept. 11, 2001, Cheney adopted a view about fighting jihadism, America's new existential enemy, that differed radically from the "realist" foreign policy approach that he had shared a decade earlier with Scowcroft. That's a psychiatric symptom? By that standard, Saul of Tarsus, Arthur Vandenberg, Irving Kristol, Ronald Reagan -- to pick at random from a thousand such cases of men undergoing a profound change of worldview -- are psychiatric cases. Indeed, by that standard, Andrew Sullivan is stark raving mad. (Okay, perhaps not the best of counterexamples.)

    I, too, know Dick Cheney. And I know something about the effects of physical illness on mental functioning. In my younger days, writing in the Archives of General Psychiatry, I identified a psychiatric syndrome ("Secondary Mania," the title of the paper) that was associated entirely with organic (i.e., underlying physical) disorders. The British medical journal Lancet found this discovery notable enough to devote an editorial to it and to alert clinicians to look for its presenting symptoms.

    And as a former chief resident of the psychiatric consultation service at Massachusetts General Hospital -- my house staff and I were called in to diagnose and treat medical inpatients (many of them post-op, many with cardiac disease) who had developed psychiatric symptoms -- I know something about organically caused dementias. And I know pseudoscientific rubbish when I see it.

    I was at first inclined to pass off Cottle's piece as a weird put-on -- when people become particularly deranged about this administration, it's hard to tell -- but her earnest and lengthy piling on of medical research about dementia and cardiovascular disease suggests that she is quite serious.

    And supremely silly. Such silliness has a pedigree, mind you. It is in the great tradition of the 1964 poll of psychiatrists that found Barry Goldwater clinically paranoid. Goldwater having become over the years the liberals' favorite conservative (because of his libertarianism), nary a word is heard today about him being mentally ill or about that shameful election-year misuse of medical authority by the psychiatrists who responded to the poll. The disease they saw in Goldwater was, in fact, deviation from liberalism, which remains today so incomprehensible to some that it must be explained by resort to arterial plaques and cardiac ejection fractions.

    If there's a diagnosis to be made here, it is this: yet another case of the one other syndrome I have been credited with identifying, a condition that addles the brain of otherwise normal journalists and can strike without warning -- Bush Derangement Syndrome, Cheney Variant.


Ain't that a hoot? No, it's about as condescending, demeaning - and low - as one can get politically. Nothing like having a bunch of moonbats walking around now saying, "he can't help it, the poor thing is suffering so," and feigning compassion. I would post the Cottle column but I refuse to pay TNR a dime, but here is the preview:

    What is wrong with Dick Cheney? Since the earliest days of his vice presidency, people have been asking this question. At first, it was mostly out of partisan pique; but, increasingly, it's in troubled tones, as one of the most powerful men on the planet grows ever more rigid, belligerent, and just plain odd in both his public utterances ("Go f_ _ _ yourself," Senator Leahy) and private actions (shoot a man in the face and not bother to call your boss 'til the next day: What's up with that?). In October 2005, longtime Bush-Cheney pal Brent Scowcroft fanned the Dick-has-changed flames when he told The New Yorker, "I consider Cheney a good friend--I've known him for thirty years. But Dick Cheney I don't know anymore." By the following February, a Newsweek profile noted that speculation as to the causes of the vice president's "darkening persona" had become a favorite Beltway parlor game. ("Has he been transformed, warped, perhaps corrupted--by stress, wealth, aging, illness, the real terrors of the world or possibly some inner goblins?") Fast-forward a year, and Cheney can hardly open his mouth without setting off a fresh wave of buzz about whether he has finally gone 'round the bend. As Washington Post columnist Jim Hoagland recently asked, "Is the vice president losing his influence, or perhaps his mind?"


Personally I think Cheney is as lucid as ever, and the things he's said during his recent travels have been refreshing. At times I think he may be the only Republican in office left with a spine, and I know that's unacceptable to the left - no one dare challenge the left on anything, let alone tell them to go "f_ _ _ themselves."

I know, as a Christian I shouldn't think this way but I know a few more on the left that could use a good expletive thrown their way. And no I haven't lost my mind either - and I darn sure don't need their phony, condescending compassion.

tomder55 answered on 03/17/07:

I know someone who had a heart attack and needed multiple bypasses. This marked in my opinion this persons slow decline diagnosed as Alzheimers . I don't know Cheney but I see no evidence of any decline in facility from his public statements or the events of the last year. Honestly ;I have had some concerns about the President . I remember his speeches and press conferences of the past and he increasingly seems less able and comfortable to articulate his thoughts and positions.Even his prepared statements are plodding like it's an effort to complete them .

With both men I like their plain spoken honesty . When I think of some of the questions fools like Wolf Blitzer has asked him I think he could've made a good case in court if he had turned around and blasted some pepper into Blitzer's face.....well maybe not that far ....but to tell him to F~~~off.

Itsdb rated this answer Excellent or Above Average Answer

Question/Answer
JacquelineA2006 asked on 03/16/07 - John Naisbitt


I think somebody needs to read a book by John Naisbitt. I think you guys should read Mind Sets because he knows how to look at the trends.. and he is the one that wrote the book called Megatrends. He knows the ten or twelve mind sets to apply to things objectively and to look at the situation in a more transparent passion.

tomder55 answered on 03/17/07:

He has recently been on the lecture circuit and when the topic of global warming comes up he is like many of us here ;as DC says a charlatin .He says that Al Gore's movie is not called "An Inconvenient Conjecture" or "An Inconvenient Supposition" .It is called "An Inconvenient Truth", and, "truth" is a term associated with religious conviction. He wonders who gave Gore the sacred tablets.

He also predicted that Africa would be the next big human battle ground and that appears to be shaping up as an accurate assessment .The US Military recently reorganized and added a Africa as a seperate command area.

Naisbitt once worked in the Kennedy Adm. and was also a top executive at IBM and Eastman Kodak. There are many who say that in the current climate ,Kennedy would've been a Republican.

Naisbitt's suggestion that we process information without filtering it through the preconceived ideas imposed by PC is sound advice.

JacquelineA2006 rated this answer Excellent or Above Average Answer

Question/Answer
ETWolverine asked on 03/16/07 - A TRUE FEMALE LEADER

With all the media coverage such sub-par female leaders as Hillarious Rotten Clinton and Nancy "Stretch" Pelosi have been getting, I thought it might be nice to post this article about a REAL female leader... Lady Margaret Thatcher, the Iron Lady.



The Lady Is a Mensch
by Rabbi Irwin Katsof

Margaret Thatcher's lesson in what makes people truly great.


While working as a fundraiser for Aish HaTorah, I often moved among the rich and famous - most of them men - and learned a great deal about the drive for success, nerves of steel and risk-taking. But it was from a lady that I learned what it means to be a mensch -- a caring human being.

Larry King, with whom I had co-authored the book Powerful Prayers, introduced me to Lady Margaret Thatcher, and we invited her to receive the King David Award in recognition of her support for the Jewish people while she was the prime minister of Great Britain.

Why did she care so much for the Jews? In answer to that question, she told me a fascinating story.

During World War II, a Jewish family from Eastern Europe had somehow connected with her parents and requested that they temporarily house their daughter. The Jewish family could not get out, but they were able to arrange a visa and ticket for their only child. At the time, Lady Thatcher's family was very poor and had barely enough for themselves, but they took in this Jewish girl, who ended up staying with them the entire length of the war and sharing Lady Thatcher's room. Lady Thatcher said that her parents taught her that it was essential to help all people in need, but it was especially important to help the Jewish people, who had been so unfairly persecuted throughout history.

This was someone we very much wanted to honor.

CARING FOR THE LITTLE PEOPLE

After a year of back and forth discussions with her staff and attempting to co-ordinate her schedule with ours, she agreed to accept the King David Award at an elegant dinner at the Carlyle Hotel in New York to be held in March of 1997.

During the course of the evening, everyone wanted to have their picture taken with Lady Thatcher. She knew this would happen and she had her assistant call us before the evening to find out color of the wall coverings in the room, so that her outfit would not clash.

She was popular. All one hundred of the participating couples posed for a picture with her, one couple at a time. After they all filed through, you would think she'd had enough -- all the flash bulbs can be very irritating -- but then she requested that we ask the orchestra members if they too wanted their picture taken.

I was impressed. I had never seen such graciousness. I had worked with many celebrities and political leaders and some were quite haughty about the whole picture-taking experience. They limited the number of photos or they wouldn't do posed shots, or they insisted that no flash be used. Others were very accommodating but I have never had anyone suggest additional photos with staff.

After I arranged the photos for the violinist and the harp player, Lady Thatcher suggested that perhaps the chef of the hotel and his staff would also like their picture taken. I was amazed. I was also getting a bit concerned. I wasn't sure just how long this was going to continue. There is a big staff at the Carlyle -- we could be doing this all night. Luckily, it stopped after the chef. I was impressed though. She was a real lady who understood the importance of being good to "the little people."

IRON LADY

A few months later, she agreed to come on one of our missions to Israel and I witnessed the same behavior. She was accompanied by three Scotland Yard security personnel. She had an armored van and a chase car that followed the van. Her security detail came to Israel three days prior and cased each venue. They explained that for the first dinner -- at the Rockefeller Museum in Arab-populated East Jerusalem -- that everyone would need to be seated and then she would walk in and proceed straight to her seat and sit down. She would have her back to the wall with guards beside her to stop anyone from approaching her.

Well, I saw why she was known as the "Iron Lady." She had her own plans. She walked into the banquet room, and to the consternation of her Scotland Yard guards, she took the next 20 minutes to circle around each table and shake hands with each and every guest. People were so enthused and felt so touched by her personal warmth, lack of pompousness and genuine human caring.

When she came to our table, she stopped and kneeled down to meet my 5-year-old daughter Bracha eye-to-eye. She shook her little hand and had a short conversation. She asked Bracha her name and her age, and looked as if she was genuinely enjoying herself.

A dinner at the Knesset concluded the trip, and there were many important guests and speakers. Prime Minister Bibi Netanyahu spoke, Mayor of Jerusalem Ehud Olmert spoke, Senator Joe Biden spoke, New Jersey Governor Christine Whitman spoke. Of course, each speaker was introduced, with their many accomplishments listed. During this time, the dinner participants had been served the salad course only. When it came Lady Thatcher's turn to speak, she got up and said only a few words, announcing that she had just ripped up her prepared speech "to ribbons." Then she declared, "I have never lost an election and this is because I can read a crowd. This crowd is hungry. As a woman and mother, I say 'Serve dinner!'"

As she sat down to tremendous applause, I was reminded of the quality that God values above all in a great leader. Both Moses and David were shepherds who were chosen to lead the Jewish people because they showed compassion for small defenseless lambs. It was when each had demonstrated this trait that God said "You who tend little sheep with such mercy will be a compassionate leader for My people."

Lady Thatcher brought home this important lesson about what makes people truly great: It is their sensitivity to the people they don't have to care about or be nice to. That's when you see their true character.

--------------

There is nothing like a great leader, and in my opinion, Hillary and Nancy are nothing like a great leader. But Maggie Thatcher sure is. G-d bless her.

Elliot

tomder55 answered on 03/16/07:

It was Maggie who convinced GHW Bush to grow a pair and lead the coalition to retake Kuwait . She also sent the British fleet on a risky mission to free the people of the Falkland Islands when they were invaded by Argentina (It is only when you look now and see success that you say that it was good fortune. It was not. We lost 250 of our best young men. I felt every one.
).She also made a valiant attempt to move the British away from disasterous socialism (There can be no liberty unless there is economic liberty). But most of all she personified leadership

Being prime minister is a lonely job... you cannot lead from the crowd.

If you lead a country like Britain, a strong country, a country which has taken a lead in world affairs in good times and in bad, a country that is always reliable, then you have to have a touch of iron about you.


If you set out to be liked, you would be prepared to compromise on anything at any time, and you would achieve nothing.


and moral clarity

I am in politics because of the conflict between good and evil, and I believe that in the end good will triumph.

Of course it's the same old story. Truth usually is the same old story.


she would know how to respond to events today

I seem to smell the stench of appeasement in the air.


ETWolverine rated this answer Excellent or Above Average Answer

Question/Answer
Itsdb asked on 03/16/07 - E-Mail Indicates Rove Role in Firings

    E-mails released this week, including a set issued Thursday night by the Justice Department, appear to contradict the administration's assertion that Bush's staff had only limited involvement in the firings of eight U.S. attorneys, which Democrats have suggested were a politically motivated purge.

    Each new piece in the rapidly unfolding saga of how the prosecutors came to be dismissed has made it more difficult for the White House to insulate itself from the controversy.

    The latest e-mails between White House and Justice Department officials show that Rove inquired in early January 2005 about firing U.S. attorneys.

    The one-page document, which incorporates an e-mail exchange in January 2005, also indicates Gonzales was considering dismissing up to 20 percent of U.S. attorneys in the weeks before he took over the Justice Department.

    In the e-mails, Gonzales' top aide, Kyle Sampson, says that an across-the-board housecleaning "would certainly send ripples through the U.S. attorney community if we told folks they got one term only." But it concludes that "if Karl thinks there would be political will to do it, then so do I." ...

    he e-mails "show conclusively that Karl Rove was in the middle of this mess from the beginning," said Sen. Charles Schumer, D-N.Y. "Every time new information comes out, it proves that the White House was not telling the truth."

    Earlier Thursday, Rove said the controversy was being fueled by "superheated political rhetoric," adding that there was no similar uproar when President Clinton dismissed all 93 U.S. attorneys at the beginning of his first term.

    "We're at a point where people want to play politics with it. That's fine," Rove told students at Troy University in Alabama.


Here's my take on this 'outrage,' so what! Tim Russert comes out a hero in the Plame Affair for his impressive gaps in recall, yet the Bush administration is conspiring and get this - playing politics - over something he has complete legal authority over.

As I pointed out to Tom, US Code states simply Each United States attorney is subject to removal by the President. I've spent over a dozen years trying to understand government codes at work and I have yet to find anything as simple and straightforward as this one.

The president is to appoint US attorneys "with the advice and consent of the Senate" but is under no obligation to consult them on removal. If anyone is playing politics it is the Democrats. They have no basis to question Bush on this, and if it upsets them that a Senator might interefere with the president's appointments they need a Cheney to Leahy moment.

Democrats have had a long list of demands on this president to oust a member of his team, Rumsfeld, Bolton, Rove, Rove, Rove (who wasn't the leaker), Gonzales, Ashcroft, Condi - who else? Who the hell are they to whine about his administration firing some on his own? And when will the rest of the GOP get a spine?

tomder55 answered on 03/16/07:

The last question is the key one. I hope this gives a really foul taste in the mouth of the Repubicans who evidently forgot the implications of being the minority party . Kimberley Strassel of the Wall Street Journal makes some good observations today .

Their GOP brethren are no longer running Congress, and the Democrats who took over want blood. The administration made its first, costly, mistake by underestimating the opposition.

Democrats can't be accused of the same naivete. They understood on Nov. 8 that their real majority power would rest in investigations and the almighty subpoena.


She goes on to detail what was essentially a lie that Deputy Attorney General Paul McNulty made when testifying about the firing.

What it[the Bush Adm.] failed to consider was the new political landscape. A White House and Justice Department on their game, scanning the Schumer-Feinstein battlefield ahead, would have sent Mr. McNulty to the Hill with a very different script. The deputy AG would have laid out the president's absolute right to hire and fire, and pleasantly noted that while the eight attorneys were all fine people, the chief was making a change. He would also have declined to serve up any gory specifics of the administration's personnel decisions. If details had later leaked, the administration would have at least staked out a principled position, and an honest one at that.

What was really pissing me off was this attempt by Bush and the Republicans after the election to make an effort at bi-partisanship . Yeah ,Pelosi ,Reid et al were all playing lip service to the idea but you didn't have to be Nostradamus to see that they would stab at him once his guard was down.

In this affair and in his overall performance Alberto Gonzales has been a lame AG . and yes the Dems. will go after him also because he dared defend "torture " ,and they know that they are planning on continuing to chip away at Presidential powers regarding security and law enforcement ; but their eyes have always been on Rove because he is competent . Libby got convicted of a non-crime .Why shouldn't Rove be destroyed for a non-coverup? But for the life of me I can't figure out how an e-mail almost a year before the event took place could have had any relevence.


Itsdb rated this answer Excellent or Above Average Answer

Question/Answer
Dark_Crow asked on 03/15/07 - New philosophy forum......

http://www.philosophyforums.net/

It looks promising.

tomder55 answered on 03/16/07:

Went there this morning . I will probably sign up ;but I spend alot of time on this board and life goes on outside of cyberspace. Got my garden and good hiking opportunities coming soon (after today's global warming noreaster passes ).

Dark_Crow rated this answer Excellent or Above Average Answer
JacquelineA2006 rated this answer Excellent or Above Average Answer

Question/Answer
Itsdb asked on 03/15/07 - Shifting again - or just shifty?

In her speech to the DNC last month, Evita said "If we in Congress don't end this war before January of 2009, as president I will."

Now, in an interview with the NY Slimes she offers a "more nuanced position than the one she has provided at her campaign events, where she has backed the goal of “bringing the troops home.”

    Senator Hillary Rodham Clinton foresees a “remaining military as well as political mission” in Iraq, and says that if elected president, she would keep a reduced military force there to fight Al Qaeda, deter Iranian aggression, protect the Kurds and possibly support the Iraqi military.


Her "scaled-down American military force that she would maintain would stay off the streets in Baghdad and would no longer try to protect Iraqis from sectarian violence — even if it descended into ethnic cleansing."

Nothing like another good ethnic cleansing under a Clinton's watch, eh? I guess she's now almost in line with Michael Moore's "there isn’t much you can do ‘til the smoke settles and blood is mopped up" position - at least for today.

tomder55 answered on 03/16/07:

I can't keep track of all the contradictory and conflicting positions the Dems. are taking on the Iraq war. Sen. Mitch McConnell (who continues to impress me as minority leader ...much better than Frist) made a floor speech during the recent Senate debate where he detailed position shifts by Madame Defarge ;Harry Reid ; and Joe Biden in a short few months .He Mitch-slapped them !!

“It is unprecedented in the powers it would arrogate to the Congress in a time of war; it is a clear statement of retreat from the support that the Senate only recently gave to General David Petraeus; and its passage would be absolutely fatal to our mission in Iraq. “Previous resolutions proposed by the Democrats were a mere statement of opinion, or sentiment. This one has a binding quality. It would interfere with the President and General Petraeus’ operational authority to conduct the war in Iraq as he and his commanders see fit. It would substitute for their judgment the judgment of 535 members of Congress.

How dare he invoke the Constitution into the debate !!

Itsdb rated this answer Excellent or Above Average Answer

Question/Answer
Itsdb asked on 03/13/07 - Inconvenient news for the Goracle

Times disses Al film as convenient stretch of truth
By Herald staff
Tuesday, March 13, 2007 - Updated: 01:37 AM EST

The New York Times fires a shot today at Al Gore and his Academy Award-winning global warming film, “An Inconvenient Truth,” saying it involves “hype” and shoddy science.

“Hollywood has a thing for Al Gore and his three-alarm film . . . So do many environmentalists, who praise him as a visionary, and many scientists, who laud him for raising public awareness,” the Times reports. “But part of his scientific audience is uneasy . . . these scientists argue that some of Mr. Gore’s central points are exaggerated and erroneous.”

The Times quotes geologist Don J. Easterbrook, addressing the Geological Society of America: “I don’t want to pick on Al Gore. But there are a lot of inaccuracies . . . we have to temper that with real data.”

James E. Hansen of NASA’s Goddard Institute for Space Studies, a Gore adviser, told the Times, “Al does an exceptionally good job of seeing the forest for the trees,” but his work has “imperfections.” He singled out Gore’s dire prediction of more, deadlier hurricanes as exaggerated.

The Times cites a recent U.N. report’s prediction of a maximum 23-inch ocean rise this century, while Gore claims the ocean will rise 20 feet over an unspecified time, flooding entire cities.

Gore told The Times his movie made “the most important and salient points” about climate change,” but not necessarily “some nuances and distinctions.”

Some scientists recently have been publicly questioning the greenhouse-gas theory, saying evidence points to natural heating and cooling cycles. Gore has been demanding political action to cut emissions, but scientists also are divided on whether that would actually alter the warming trend.

Last week, Britain’s Channel 4 announced “The Great Global Warming Swindle,” a counter-documentary in which scientists dispute manmade global-warming theories and discuss professional pressures to go along with them.
~~~~~~~~~~~~~~~~~~~~~~~~~~~~~~~~~~~~~~~~~~~~~~~~~~~~~~

What's an exaggeration or imperfection here and there as long as you get the 'nuances' that lead to an Oscar?

tomder55 answered on 03/14/07:

the fallacy is the term concensus .unproven contentions that have a consensus cannot be science. How is their theory testable ?



When Gore found out that Mars was warming he said "It was bad enough when just the Earth was affected, but now it looks like the entire solar system may be in danger...We must act before it is too late."

On a positive note ;they did find out why the Arctic is melting





Where are the sinking PI evacuees, Mr Gore?


Whereabouts in New Zealand are the sinking Pacific Island evacuees, Mr Gore?

The New Zealand Super Fund has been challenged to ask Al Gore, when he visits the country next month, for the whereabouts of the Pacific Islanders that Gore says in his film have been evacuated to New Zealand because their islands are drowning.

This challenge has been issued by the New Zealand Climate Science Coalition, through Owen McShane, chairman of its policy panel. The coalition also wants to know who is paying for the visit. The challenge follows the announcement at the Labour Party conference that Al Gore is to visit New Zealand on 14 November for a private meeting with the Super Fund board, invited MPs and business leaders – but no media.

“In view of the announcement at the Labour Party conference and the linkage with the Super Fund, New Zealanders are entitled to know just who is meeting the cost of bringing to New Zealand a rabble rouser who has admitted to being liberal with the truth,” said Mr McShane. The Super Fund should be asked to explain how it can have faith in a man like Gore who made the following admission in an interview in the United States with David Roberts of Grist magazine (9 May 2006):

"Nobody is interested in solutions if they don't think there's a problem. Given that starting point, I believe it is appropriate to have an over-representation of factual presentations on how dangerous (global warming) is, as a predicate for opening up the audience to listen to what the solutions are, and how hopeful it is that we are going to solve this crisis."

“How much ‘over-representation’ can we expect from Mr Gore in New Zealand? He has been liberal with the truth about Pacific Islanders being evacuated to New Zealand because their islands are sinking; he has ignored the fact that Antartica is accumulating ice; and he still promotes the “hockey stick” curve that has been unmasked as scientific fraud; just how much else of his film and claims on global warming can we believe?” asked Mr McShane.

Itsdb rated this answer Excellent or Above Average Answer

Question/Answer
Itsdb asked on 03/13/07 - Darn those conservatives...

...they just won't go away.

By rights, American conservatism should be dead or on life support by now. -Gary Kamiya, in The Coulterization of the American right

First question, by rights? Why?

Kamiya also wrote, now that its ideology has been exposed as empty and its leadership incompetent and corrupt, free-floating hatred is the only thing it has to offer. Of course while informing us of this right-wing hatred he offered these kind words about conservatives:

    "vacuous bottom-feeders"

    "factually challenged right-wing hack"

    "pathetic figure"

    "marginally less creepy"

    "vulgar"

    "bigoted"

    "know-nothing"

    "slimy Gollum"

    "obsessive mean-spiritedness"

    "unhinged"

    "obsessive mean-spiritedness"

    "intellectual sterility"


And my favorite, "Christo-jihadists."

Is this just another example of what Elliot calls the George Carlin rule, "all of your stuff is $h!t and all of my $h!t is stuff?" Or is it something deeper, like hatred, intolerance and bigotry?

tomder55 answered on 03/14/07:

See where Hillary reconstituted the old "vast Right Wing Conspiracy "mantra into her stump speech ? Let them keep their rhetoric and blast Coulter . No one pays attention to their warblings ,that is why they have attempted to use more toxic language .The more they complain ,the more books Coulter sells .That is not the case when an idiot like Franken speaks.

....and yes Edwards is a faggot.

As for conservatism , Peggy Noonan advised after the 2006 election that we take a breath and have a dialogue about what has worked and what hasn't . I think that is sound advice so long as we don't get too much into naval gazing . There are real issues to confront that liberals have seized the initiative on ;and it's only their incompetence that prevents them from making even bigger gains . Competence appears to be an issue that will be a major factor in the election cycle .

Itsdb rated this answer Excellent or Above Average Answer

Question/Answer
Itsdb asked on 03/12/07 - Capitalism 3.0?

Long read but worth it...

Gore Funding Plan For "A New World Order"

If your employer began paying you 80 cents on the dollar, but, not to worry, the other 20 cents was going to support "good causes", thereby giving you value instead of capital, would you be pleased?

If not, you won't like what Al Gore has been quietly planning along with his Global Warming initiative. He and others are working to achieve that very thing and to bring it about in a manner which doesn't give you a vote in which values your dollars end up supporting.

Gore is quietly funding an assault on Capitalism as we know it, one that begins but doesn't end with Global Warming. That's only the model for what's coming next.

Sans news conferences and Oscar nods, it's a well-funded, grand design to re-shape, not just America, but the global economy in such a manner so as to inculcate Liberal values into the world's system of finance.

Defeated at the ballot box, it seems Gore has figured out that if he can follow, or perhaps even lead the money, he and other liberals can bring about the social and economic change they want, whether the middle class likes it, or not.

Gone unnoticed in the recent controversy over Gore's Generation Investment Management LLP (GIM) is this blurb below from the European partnership's web site:

    Dedicated to thought leadership on sustainability and capital markets -5% of our profitability is allocated to the Generation Foundation


The Generation Foundation is not your typical foundation involved in giving money to what it views as deserving causes. And the ramifications of its work should be of genuine concern for fiscal and social conservatives, civil libertarians and anyone who supports the free market system and the economics, or laisser-faire capitalism of Milton Friedman.

By their own admission, Gore and his well-heeled, high-minded brand of liberal thinkers believe Capitalism is either dead, or deserves to be. Here's the first quick look at a now Gore supported and funded, self-proclaimed New World Order as ruminated upon back in 2002.

    Above all, the man is wondering whether Americans need to pause for a moment, now that the millennium has turned and the market has crashed and corporate ethics seem like the quaint idea of a bygone era. And then, after this pause, the man wonders if we need to think long and hard about what we want and how money -- and, in particular, value -- figures into the arc of our lives.

    He has come up with an idea that admittedly might not solve any of our financial and social ills, but that he hopes might solve quite a few of them.

    The man's name is Jed Emerson. And his idea is called the "blended value" proposition.


Jed Emerson used to spend his afternoons walking into shooting galleries to dispense clean needles and condoms in San Francisco's Tenderloin district and now lecture(s) to the most powerful business leaders on the planet. (emphasis mine)

Those facts aren't inserted to disparage Emerson's work, but to highlight precisely the brand of social values Gore wants to build into our economic system so that they are realized though a non-democratic system controlled by wealth, as opposed to votes. And conservatism's good friend George Soros is linked to the work, as well.

Gore's foundation has employed the now well-credential-ed Emerson as their first senior research fellow.

    Jed Emerson is the first Generation Foundation Senior Fellow. Learn more about his body of work on Blended Value.

    Additional Link: Jed Emerson Wants To Change The World His goal: a kind of capitalist utopia in which sound business practices are rewarded, shareholders are empowered and our portfolios do more than just make money.


GIM is based in Europe, that's fitting, as Gore's plans for the American economy appear to be based more on socialist European values than America's. Actually, the effort is much larger than Gore and impossible to paint as anything other than the fiscal championing of an extreme Left wing ideology: Capitalism 3.0.

    In Capitalism 3.0, Peter Barnes redefines the debate about the costs and benefits of the operating system known as the free market. Despite clunky features, early versions of capitalism were somewhat successful. The current model, however, is packed with proprietary features that benefit a lucky few while threatening to crash the system for everyone else. Far from being "free," the market is accessible only to huge corporations that reap the benefits while passing the costs on to the consumer. Barnes maps out a better way. Drawn from his own career as a highly successful entrepreneur.


Really? But wait. I thought Capitalism 3.0 was the conceptualization of Gore's minion Emerson?

    Capitalism 3.0 February/March 2006
    by Jed Emerson and Sheila Bonini
    Originally published as part of the Blended Value Map and a chapter in the book series The Accountable Corporation


And who is Peter Barnes?

    Peter Barnes, founder of Working Assets and a board member at the Tomales Bay Institute's On The Commons


You might recall my post on Working Assets, the Leftist publishing company that plucked Glenn Greenwald out of the blogosphere to publish a hastily written book to bash Bush and oppose the Patriot Act. Their financial backing links them to the Phoenix Group, does the name George Soros ring a bell? It isn't me making these connections, it's the New York Times.

    To understand the financial connections that can now be documented, you'll also want to understand the Phoenix Group (PG), as reported on here in The Hill, and in depth through the New York Times, Wiring the Vast Left Wing Conspiracy, July 2004. Wealthy associates of the group have been propping up the Netroots movement, enjoying the cachet of a ground up grass roots movement that's actually financed and, I'd argue, controlled from the top down by big money, just as is most all contemporary politics. The Times piece is an absolute must read.

    Some of the key PG players, though far from all, are Howard Dean, George Soros, Simon Rosenberg, Andy Rappaport, and SEIU President Andy Stern.


Take a moment and process that. Some of the very same individuals, now well networked, can almost immediately launch a New York Times best seller written by an all but unreadable blogger, a book intended to attack Bush and defeat the Patriot Act; meanwhile the same individuals are drawing a road map for the US economy based upon a self-labeled Utopian neo-capitalist ideology. Scary? It should be, as the Right has not kept up.

This is not a battle to be won or lost tomorrow, or even next year. It's a battle conservatives and libertarians must understand and engage over the next decade if they wish to prevail.

Now that we understand who would design this new values-based economic system Gore and his Leftist buddies have in mind, it's important to understand the notion behind Blended Value. Building moral or social value into our economic system may sound nice, but whose values? Who gets to decide? Not you. The system only works provided the society marches in lockstep on difficult choices. In Gore's proposed system, decisions don't flow from the bottom up, as with democracy. They flow from the top down with the money.

If Gore has his way, the wealthiest of the world, those who invest and support corporations in the largest measure, get to decide how everyone gets to live, what causes are supported, or dropped. And he has the nerve to call it freedom? It's not even close.

Corporate valuation based upon social and moral judgments means one could devise a better mouse trap, one that every body wants, but if it doesn't measure up within some grand Utopian plan, it's corporate valuation will always be sub par, so it would not attract the type of investment required to launch. What happens to choice? You can't choose to buy, or not buy, that which never gets built.

And what of the cost of building these social and moral values into our system of economics? Given what we have seen from Gore and other elitists preaching Global Warming, while preparing to preach social values, do you think the seven and eight figure salaried elite are going to do the sacrificing? You can call social programs value, they still cost money in the end.

Man2 The result of such a system would be upward pressure on prices and downward pressure on wages as profitability declines. In Gore's developing system, the middle class will pay the price for the implementation of a left wing, liberal-socialist agenda - and they will never even be asked to vote on whether to do it, or not. Gore's efforts regarding Global Warming establish precisely that paradigm. There's no telling where they'll go next.

For maximum valuation, will a corporation need to plant trees? build abortion clinics? pass out hypodermics, as one of its authors once did? Whose to say? Not you or me, that much is clear.

In short, what globally thinking socialists like Soros, and now Gore, have not been able to achieve through America's ballot box, they would seek to impose on our economic system. If you think I'm crazy, take a look at one of Gore's tracts. Both the illustrations and the text remind one more of the Eastern Block than the freedom we've come to know as Americans.

    It’s time for capitalism to mutate again. We’re due. Here’s why: “Release 1.0” — the original model — created not only wealth, but also a blizzard of economic, social, and environmental costs. In succeeding in its mission, it also exploited the world’s resources and peoples as if there were no tomorrow.

    “Release 2.0” — evolving since the late 1960s — has been increasingly regulated and “civilized” as it has attempted to keep pace with increasing awareness of its costly “side effects.” But Release 2.0 has hit a plateau in its efforts to build wealth and at the same time make deposits in the bank of social value.

    Layering regulations over regulations, and social initiatives over more social initiatives, just isn’t going to result in the hoped-for economic, social, and environmental returns. The problem is that even the most forward-thinking corporations are still driven by a mindset that is obsolete.

    What’s needed is the next iteration of capitalism — a new model that stems from an understanding that our common goal should be to maximize our value potential. The model should be based on a common understanding of what value is (to our minds, it should be a blend of economic, environmental, and social factors). And, it should be implemented with the common understanding that maximizing value, regardless of whether one is the “customer of” or the “investor in” the entity, requires taking all three elements into account.

    Capitalism 3.0, as we’re calling it, represents an opportunity to break existing frameworks and create a model of accountability that addresses the realities of the world we’re living in.


Read the other publications if you wish, they make it very clear. And Global Warming is only the beginning, albeit a critically important one. If Gore has his way, if we alter our economics to address his over-heated fear of Global Warming, we will be walking on the very path Gore and Soros and whomever else wants, while putting money in their pockets to fund their agenda. It's a path to the ruination of a free America, not some cure.

As Friedman knew, in capitalism money is freedom. If Gore and his new found European friends succeed in controlling our money while selling us on the concept of greater value through support for social programs of their liking, not necessarily ours - it won't be freedom, or Capitalism 3.0, it'll be a sham akin to Socialism from which America may never recover.

The current Global Warming debate is only the beginning, not the end. Just ask Gore, after all, it's his plan. If you think it isn't critical to push back, or tread carefully around global warming, you may want to think again.
~~~~~~~~~~~~~~~~~~~~~~~~~~~~~~~~~~~~~~~~~~~~~~~~~~~~~~

We're already seeing significant appearances of this "blended value" concept with the recent corporate alliance, it's no longer just a Ben & Jerry thing. Public schools are pushing the global warming agenda on our kids, not to mention exploiting them. Climate change was the cover story in last week's Sports Illustrated:



With the help of some university professor, SI speculated that if it would have been one degree warmer, Vic Wertz' smash hit in game 1 of the 1954 world series would have glanced off of Willie Mays fingertips (and I wondered if it were 1 degree cooler would it have bounced off his wrist?).

The global warming scam is coming at us from every angle, in education, economics, politics - and golf courses, ski slopes and ballparks. What are you going to do?

tomder55 answered on 03/12/07:

How dare you question the musing of the Goracle !

Imagine that there is a new scientific theory that warns of an impending crisis, and points to a way out.

This theory quickly draws support from leading scientists, politicians and celebrities around the world. Research is funded by distinguished philanthropies, and carried out at prestigious universities. The crisis is reported frequently in the media. The science is taught in college and high school classrooms.

I don’t mean global warming. I’m talking about another theory, which rose to prominence a century ago.

Its supporters included Theodore Roosevelt, Woodrow Wilson, and Winston Churchill. It was approved by Supreme Court justices Oliver Wendell Holmes and Louis Brandeis, who ruled in its favor. The famous names who supported it included Alexander Graham Bell, inventor of the telephone; activist Margaret Sanger; botanist Luther Burbank; Leland Stanford, founder of Stanford University; the novelist H. G. Wells; the playwright George Bernard Shaw; and hundreds of others. Nobel Prize winners gave support. Research was backed by the Carnegie and Rockefeller Foundations. The Cold Springs Harbor Institute was built to carry out this research, but important work was also done at Harvard, Yale, Princeton, Stanford and Johns Hopkins. Legislation to address the crisis was passed in states from New York to California.

These efforts had the support of the National Academy of Sciences, the American Medical Association, and the National Research Council. It was said that if Jesus were alive, he would have supported this effort.

All in all, the research, legislation and molding of public opinion surrounding the theory went on for almost half a century. Those who opposed the theory were shouted down and called reactionary, blind to reality, or just plain ignorant. But in hindsight, what is surprising is that so few people objected.

Today, we know that this famous theory that gained so much support was actually pseudoscience. The crisis it claimed was nonexistent. And the actions taken in the name of theory were morally and criminally wrong. Ultimately, they led to the deaths of millions of people.

The theory was eugenics, and its history is so dreadful —- and, to those who were caught up in it, so embarrassing —- that it is now rarely discussed. But it is a story that should be well known to every citizen, so that its horrors are not repeated.

Michael Crichton


Did you know that during the Clinton days it was Enron that was the top promoter of the global warming fears and a big promoter of Kyoto. Why would an energy company take such a position ? Enron did not supply or generate energy;it was the middleman in the energy market.

If co2 emissions were to be curbed by treaty, it would take the form of a cap-and-trade system. Power plants would be given allowances of CO2 emissions, and if one came under the cap, it could sell its emission credits to a producer who was having trouble coming under the limits.
Enron figured it was positioned to be the broker. Basically, the cap-and-trade scheme was creating an artificial market, and Enron planned on cornering it.

So, when Bush rejected the Kyoto protocol, the administration stuck a stake in the very scheme that an Enron internal memo had declared, according to the Washington Post, would "do more to promote Enron's business than almost any other regulatory initiative outside of restructuring the energy and natural gas industries in Europe and the United States."

Evidently with Enron out of the picture there was an inconvienient void to be filled that Gore was more than willing to fill . No wonder he hasn't thrown his hat in the ring ....there are much more lucrative enterprises to persue.


Itsdb rated this answer Excellent or Above Average Answer

Question/Answer
Itsdb asked on 03/10/07 - Does it matter or not?

Tell me if you've heard this before?

    Why are you obsessed with Bill Clinton’s sex life?

    Clinton's private life has nothing to do with his public life.

    The man's sex life is none of our business.


Or any number of similar quotes. So why are the media and other moonbats scrutinizing Rudy and Newt's personal lives if it doesn't matter? I hear the NY Times has been on a mission against Rudy this week, and even though Gingrich isn't running for anything yet his "sordid" personal life is suddenly under scrutiny again.

al-AP is doing their hit jobs on Rudy this week, I'm sure hoping to break his stride and sway social conservatives from getting behind what is likely the Reps best chance. If Giuliani can't keep his family together, how will he keep the country together? It was good of al-AP to form that question in our minds wasn't it?

Steve and Cokie Roberts asked this:

    To much of America, Rudy Giuliani is a genuine hero, but not to his ex-wife or their children. So which images will have more power? America's Mayor or Broadway Rudy? The Faithful Leader or The Unfaithful Husband? The Able Manager or The Absent Dad?


I don't know, and frankly I don't care, but the Roberts seem certain the "smart political money" says Rudy's negatives will "prove fatal" before the election.

tomder55 answered on 03/12/07:

I honestly thought that the impeechment of Clinton for what he was accused of was wasted effort. There were plenty of good reasons to dump the chump ,but not the one they used. The whole Lewinsky/Paula Jones thing came from the exactl thing we always complain about ;prosecutors gone wild . The whole investgation was supposed to be centered around the White Water corruption .Indeed there were many "fall guys" who took a hit for the Clintoons including Susan McDougal and Jim Guy Tucker ,a sitting govenor . But the investigation strayed to matters of sex and lying to cover up infidelity .Clinton was punished for his lying during deposition but I never did think that it merited removal from office.

I think Rudy and Gingrich are wise to air out their dirty laundry now ,rather than wait. Gingrich ..... let's face the facts ;he was a complete hypocrite and I do think the revelation will negatively affect his chances . But I don't think the electorate really cares who was or wasn't divorced or how many times . The Democrats are delusional if they think they can peel away the conservative or religious base . Who exactly are they offering as an alternative ?

Rudy's chances rest on whether another candidate emerges that has all the attributes the Republican base is looking for . There have been no perfect candidates to date but everything else being equal ,I'd rather they pick someone who can win the general election because besides Richardson ,I don't see a Democrat who wouldn't be a disaster.

Itsdb rated this answer Excellent or Above Average Answer
JacquelineA2006 rated this answer Excellent or Above Average Answer

Question/Answer
tropicalstorm asked on 03/11/07 - a start anyway at disillusionment of Moore fans

USTIN, Texas - As documentary filmmakers, Debbie Melnyk and Rick Caine looked up to Michael Moore.

Then they tried to do a documentary of their own about him — and ran into the same sort of resistance Moore himself famously faces in his own films.

The result is "Manufacturing Dissent," which turns the camera on the confrontational documentarian and examines some of his methods. Among their revelations in the movie, which had its world premiere Saturday night at the South by Southwest film festival: That Moore actually did speak with then-General Motors chairman Roger Smith, the evasive subject of his 1989 debut "Roger & Me," but chose to withhold that footage from the final cut.

The husband-and-wife directors spent over two years making the movie, which follows Moore on his college tour promoting 2004's "Fahrenheit 9/11." The film shows Melnyk repeatedly approaching Moore for an interview and being rejected; members of Moore's team also kick the couple out of the audience at one of his speeches, saying they weren't allowed to be shooting there.

At their own premiere Saturday night, the Toronto-based filmmakers expected pro-Moore plants in the audience heckling or trying to otherwise sabotage the screening, but it turned out to be a tame affair.

"It went really well," Melnyk said. "People really liked the film and laughed at the right spots and got the movie and we're really happy about it."

Moore hasn't commented publicly on "Manufacturing Dissent" and Melnyk thinks he never will. He also hasn't responded to several calls and e-mails from The Associated Press.

"There's no point for Michael to respond to the film because then it gives it publicity," she said.

"(President) Bush didn't respond to `Fahrenheit 9/11,' and there's a reason for that," Caine added.

The two were and still are fans of all his movies — including the polarizing "Fahrenheit 9/11," which grossed over $119 million and won the Palme d'Or at the
Cannes Film Festival — and initially wanted to do a biography on him. They traveled to his childhood home of Davison, Mich., visited his high school and traced his early days in politics and journalism.

"The fact that he made documentaries entertaining was extremely influential and got all kinds of people out to see them," said Melnyk, whose previous films with Caine include 1998's "Junket Whore." "Let's face it, he made documentaries popular and that is great for all documentary filmmakers."

"All of these films — `Super Size Me,' `An Inconvenient Truth' — we've all been riding in his wake," said Caine. "There's a nonfiction film revolution going on and we're all beneficiaries of that. For that point alone, he's worth celebrating."

But after four months of unsuccessfully trying to sit down with Moore for an on-camera interview, they realized they needed to approach the subject from a different angle. They began looking at the process Moore employs in his films, and the deeper they dug, the more they began to question him.

The fact that Moore spoke with Smith, including a lengthy question-and-answer exchange during a May 1987 GM shareholders meeting, first was reported in a Premiere magazine article three years later. Transcripts of the discussion had been leaked to the magazine, and a clip of the meeting appeared in "Manufacturing Dissent." Moore also reportedly interviewed Smith on camera in January 1988 at the Waldorf Astoria hotel in New York.

Since then, in the years since "Roger & Me" put Moore on the map, those details seem to have been suppressed and forgotten.

"It was shocking, because to me that was the whole premise of `Roger & Me,'" Melnyk said.

She and Caine also had trouble finding people to talk on camera about Moore, partly because potential interview subjects assumed they were creating a right-wing attack piece; as self-proclaimed left-wingers, they weren't.

Despite what they've learned, the directors still appreciate Moore.

"We're a bit disappointed and disillusioned with Michael," Melnyk said, "but we are still very grateful to him for putting documentaries out there in a major way that people can go to a DVD store and they're right up there alongside dramatic features."

tomder55 answered on 03/12/07:

That's funny .'Roger and Me ' was the only Moore movie I liked . I was in fact looking forward to his next hit piece about the Pharmaceutical Industry .It is something I know about so it would be easier to separate the BS and dogma conventional wisdom from the truth .

He also avoided the guy who made "Michael Moore Hates America" which also examines Moore's methods. But the difference is apparently they did this film without any pre-conceived conclusions that they used as a template for their filming .

Apparently truthful documentaries do not get the airing they deserve ;but throw in some good ole fashion Bush bashing and you get a Canne's film festival award or an academy award.

Itsdb rated this answer Excellent or Above Average Answer
tropicalstorm rated this answer Excellent or Above Average Answer

Question/Answer
kindj asked on 03/10/07 - A great quote

"It appears we have appointed our worst generals to command forces, and our most gifted and brilliant to edit newspapers!




In fact, I discovered by reading newspapers that these editor/geniuses plainly saw all my strategic defects from the start, yet failed to inform me until it was too late.




Accordingly, I'm readily willing to yield my command to these obviously superior intellects, and I'll, in turn, do my best for the Cause by writing editorials - after the fact."



Robert E. Lee, 1863

tomder55 answered on 03/10/07:

Amazing .or he could've waited and become a revisionist historian .

I believe that Robert E. Lee was a great man -- able, intelligent,
well-motivated and moral, and much beloved by his army. He did
what he believed to be right. On the other hand, I have long been
uncomfortable with certain aspects of the Lee tradition. I suspect that
this discomfort has several sources. Without revealing too much about
myself, I acknowledge that a parochial grade school education may
have provoked in me a perverse skepticism of lives of the saints. Lincoln
scholar Don E. Fehrenbacher characterizes the "idealized Lincoln" as
"insufferably virtuous" -- a characterization that I can appreciate.

In the case of Lee my discomfort has been more specific. He was a master strategist and tactician but was so
committed to the offensive that he suffered grievous and irreplaceable
losses that progressively limited the viability of his army. He was mag-
nanimous toward the North but fought bitterly and aggressively against
it. He was kind and protective of his soldiers but regularly risked their
lives in daring offensive strokes, ordered the July 3 attack on Cemetery
Ridge, and continued the war long after a Southern victory was possi-
ble. He was conciliatory after the war yet categorical in his defense of
what the South had done and outspokenly critical of the North's post-
war treatment of the South.

This book is an effort to rationalize these conflicting pictures of Lee.


That was from the Preface of "Lee Considered: General Robert E. Lee and Civil War History" by historian Alan T.Nolan (1996).

Until I read it Lee was universally recognized as one of the worlds greatest Generals. It's a good thing ole Nolan came along to debunk that myth !From genious General ;who divided his outmanned army at Chancellorsville and routed the much larger army commanded by Joe Hooker by suprising him with a Battle of Cannae maneuver,to reckless commander who wasted his men's lives needlessly by being aggressive.



Itsdb rated this answer Excellent or Above Average Answer
kindj rated this answer Excellent or Above Average Answer

Question/Answer
Itsdb asked on 03/09/07 - Thompson to run?

WASHINGTON — Former Senate Majority Leader Howard Baker Jr. said today he is urging his friend Fred Thompson to run for president and encouraging others to convince him.

Baker, R-Tenn., in a telephone interview with The News Sentinel, said: "I'm planting the seed that he might run or might be convinced to run." Baker also was White House chief of staff to President Reagan and most recently served as U.S. ambassador to Japan.

Thompson, 64, an actor on the TV drama "Law & Order" also was a Republican senator from Tennessee for eight years through 2002.

Baker said he's talked to Thompson about running for president. "He doesn't confirm that he's interested and he also doesn't tell me to stop," Baker said.

Thompson could not be reached for comment.

Baker and Thompson go way back and both got national attention when Baker in 1973, as a senator, named Thompson as Republican counsel to the Senate Watergate Committee exploring the Nixon Administration's connections to the Watergate burglary and its cover-up.

Thompson also helped Baker in his 1980 presidential campaign when Ronald Reagan won the GOP nomination and the presidency.

A couple of potential complications in a Thompson race is that he is good friends with Sen. John McCain, R-Ariz., a presidential candidate. Thompson endorsed McCain for president in his 2000 bid.

Also, Thompson has two small children in his second marriage: ages 4 and younger than 1.

Baker said his wife, former Sen. Nancy Kassebaum Baker, R-Kan., already supports McCain for president and Baker said he too may back McCain if Thompson does not run.

Baker said he has not talked to any big money people about helping Thompson. "I'm just sending up some trial balloons so we'll see how that works."

~~~~~~~~~~~~~~~~~~~~~~~~~~~~~~~~~~~~~~~~~~~~~~~~~~~~~~

Hmmm, whaddya think? Another conservative actor, well spoken, well respected - even in Hollywood (for the moment)?

tomder55 answered on 03/09/07:

I heard that rumor last week . It would make the race interesting .

I am not at all happy with the process this cycle. The nominee will be decided before most of the country is even paying attention. The nominee will lock up enough delegates to clinch it by this time next year . That's 6 months before the conventions . Plenty of time for a nominee to have a major melt down . ...Then what ?

Conservatives do not appear to be thrilled with the current group of candidates and Thompson could fill that void if he answers the all important question ....can he win ?

Itsdb rated this answer Excellent or Above Average Answer

Question/Answer
Itsdb asked on 03/09/07 - Plame to testify

We've spent millions of dollars to get Rove/Cheney/Bush in an investigation with no conspiracy, no coverup, no administration leak, no charges filed, and now Plame is going to testify before a congressional panel on how the White House handled the disclosure of her 'secret' identity as a 'covert' agent.

Why in God's name do we need a congressional investigation now? Will they wonder why we wasted that much time, money and reputations on an investigation in which the prosecutor knew 3 months ahead of time the 'leaker' was a fierce bureaucratic opponent of the White House? Or why the judge refused to allow evidence of "the fallibility of the witness whose testimony was most decisive," namely Russert's statement "under oath that he did not know that one may not be accompanied by a lawyer to a grand jury hearing."

    The defense had tapes showing Russert saying on television three times that lawyers are barred from grand jury proceedings.


What important light is Plame going to shed on this event? I'm betting her "victimhood" will be played out for every ounce of sympathy that can be mustered. Shouldn't this whole affair be put rest now?

tomder55 answered on 03/09/07:

She's to testify in front of the House Oversight and Government Reform Committee run by Henry Waxman (remember Excon's warning about the guy . He is a pit Bull. ). The only purpose for these hearings is to gather evidence in an impeachment play . Fitzgerald is also slated to testify . Waxman also thinks he can bring down Karl Rove. Here is a letter Waxman sent to Rep.Davis when Waxman was ranking minority member last year . Here is Waxman's invitation to Fitzgerald. Here is Waxman's version of a chronology of events .

I doubt if she will be sworn in ;although I think the Repuublican members insist that EVERY witness who testify be sworn in.

Plame is there for the sympathy effect . No doubt the tears will flow as she describes how her life has been ruined .... Then off to another Vanity Fair photo session and a cocktail party or 2 . Everyone already knows that she recommended Wilson when the CIA wanted a flunky to go sip sweet tea by a Niger pool side .

Maybe she is there to testify about the Pay Plame Act to reward her for her steller contributions to the good of the country ??

Note that Joe Wilson is conspicuosly absent from the invitees . Maybe that's because the Senate has already found him to be an insufferable liar .

Byron York wrote on NRO blog :

If Valerie Plame Wilson testifies before Congress, she will likely be asked detailed questions about her job status. Judge Reggie Walton tried, unsuccessfully, to keep the issue out of the Libby trial. But as Henry Waxman pointed out, that was a narrow legal proceeding, and a congressional hearing would have a broader scope.

So Mrs. Wilson will likely be asked what years she operated under non-official cover. She will likely be asked about her return to the United States, about her maternity leave, and about her job description when she returned to work. She will likely be asked whether that job description could accurately be called covert. She will likely be asked whether her area of work had changed, or was changing, at the time of Robert Novak's July 14, 2003 column that mentioned her name. And she will likely be asked what, if any, damage to national security resulted from the publication of her name.



Valerie ....What do you think about the fact that the person who leaked your name, Richard Armitage, was given immunity by the prosecutor ? Is Richard Armitage named in your law suit ? Maybe she will reveal a block buster like the fact that she had already been outed a decade ago by Aldridge Ames ?

Itsdb rated this answer Excellent or Above Average Answer

Question/Answer
tropicalstorm asked on 03/08/07 - I want one to scrap!

Four F-14 fighter jets seized at two California airfields

03:20 PM MST on Wednesday, March 7, 2007

By SHARON McNARY
The Press-Enterprise

Federal authorities seized four F-14 Tomcat fighter jets from two air museums in Chino and a Victorville company on Tuesday, declaring that a Navy official improperly sold them.
Story continues below
Terry Pierson / The Press-Enterprise
William J. Hayes, an assistant special agent with the U.S. Immigration and Customs Enforcement agency, right, stands with an agency officer near one of the F-14 Tomcats seized at the Chino Airport.

Featured in the 1986 film "Top Gun," the Navy stopped using its fleet of F-14s last year.

Two Tomcats were seized at Yanks Air Museum, and a third at Planes of Fame, both located at the Chino Airport.

A Yanks employee, who showed the plane to an undercover investigator in 2005, bragged that he had rebuilt the F-14 from parts and that it still had its afterburners, which give a jet extra thrust, according to an affidavit prepared by an investigator with the U.S. Immigration and Customs Enforcement agency.

The fourth Tomcat, used as a prop on the TV show "JAG," about military lawyers, was seized at Aviation Warehouse, a business tenant of the Southern California Logistics Airport in Victorville.

The seizure was intended, in part, to prevent the non-functioning jets or their parts from being bought by U.S. companies acting on behalf of Iran, the only country whose air force still flies the F-14, the affidavit said.

The Inland owners of the jets are not suspected of supplying parts to Iran, according to a federal agent involved in the investigation.

A Navy official sold three of the planes to a scrap dealer, who sold them to a used aircraft dealer who sold them to Yanks, which sold one to Planes of Fame. The Navy official sold the fourth plane to a scrap dealer which resold it to Paramount Pictures, which sold it to the aircraft dealer, according to the affidavit.

The F-14 was introduced in 1972 as a replacement for the Navy's primary fighter, the F-4 Phantom.

The United States permitted the government of Iran to buy F-14s during the 1960s and 1970s under the regime of Mohammad Reza Pahlavi, the shah of Iran.

After the fall of the shah and the Iranian hostage crisis, the United States placed an embargo on the trade of parts for the plane, and a flourishing black market developed, the affidavit said.

The Pentagon halted sales of spare parts from its recently retired F-14 fighter jet fleet because of concerns they could be transferred to Iran, The Associated Press reported in February.

Investigators from the U.S. Immigration and Customs Enforcement agency, the Navy Criminal Investigative Service and the Defense Criminal Investigative Service seized the four planes, immigration and customs spokeswoman Virginia Kice said.

The agency has the authority to seize military and technical equipment to prevent it from falling into the hands of terrorists or countries hostile to the United States. The other two agencies have jurisdiction over the disposal of surplus military equipment.

"It is important that we eliminate the vulnerability that any of this technology could be transferred to those who would wish to harm us or our national security," said William Hayes, assistant special agent in charge of investigations for the immigration and customs agency in Orange County.

Headed for Scrap

The planes will be reduced to scrap, Hayes said.

The owners of the airplanes are not suspected of selling spare parts to Iran or any company representing Iran, Hayes said.

Investigators discovered the planes' existence in September 2005 during the probe of a Bakersfield company that is suspected of being a company secretly acting on behalf of the Iranian government.

Federal law requires military aircraft to be destroyed when they are sold as scrap for recycling, but an officer put in charge of disposing of the four planes at Point Mugu Naval Air Station failed to do so, the affidavit said.

Instead, the officer sold relatively intact planes to two scrap metal companies for $2,000 to $4,000 each in 1999, depositing the proceeds in a base "morale, welfare and recreation" account.

An investigation is continuing into the circumstances of those sales,

rest of story


page


tomder55 answered on 03/09/07:

The Tomcat is my all time favorite jet .





here was the patch I wore in Iran :



The Tomcat can still out perform most jets in the sky . But alas ;there is one better now ;the F 22 Raptor .





I don't think the Tomcat should be consigned to scrap . I think they should be donated to museums like the Grumman Memorial Park ;on the grounds that I spent many a summer day picnicing with mine and other Grumman employee families. But if they have to retire ....I just wish they would let them do it in style ....over the skys of Tehran

ETWolverine rated this answer Excellent or Above Average Answer
tropicalstorm rated this answer Excellent or Above Average Answer

Question/Answer
Itsdb asked on 03/08/07 - Speaking of the media...

As you are probably aware, I love the media. It used to be that what wasn't on the opinion pages at least appeared to be news, but al-AP doesn't even pretend to print news any more. Take for example this from today's paper (note that they declare the media as losers askide from Russert, though al-AP came through unscathed):

    CIA Leak Trial Winners and Losers

    By MICHAEL J. SNIFFEN, Associated Press Writer
    Wednesday, March 7, 2007

    (03-07) 13:37 PST WASHINGTON, (AP) --

    Convicted of four felonies, "Scooter" Libby has to lead the list of losers in the CIA leak trial, but he's got company. His boss Dick Cheney didn't fare so well. Neither did the news media.

    And the winners? A less obvious group with some surprises: Bloggers. A juror who knew trial witnesses personally. Even a Sunday talk show host who survived a brutal five-hour cross-examination.

    LOSERS

    _ I. Lewis "Scooter" Libby: Jurors saw him every day, heard him on eight hours of taped grand jury testimony and didn't believe him. Even should he win an appeal, his life could remain in limbo for years and he'll be forever known as the only guy tried in the CIA leak case.

    _ Dick Cheney: Portrayed as a thin-skinned Machiavelli who lurked behind the scenes while orchestrating a bungled response to criticism of the Iraq war.

    _ The Bush administration: Still can't get past the story of how it messed up intelligence about nonexistent weapons of mass destruction before the U.S.-led invasion of Iraq.


    _ Richard Armitage: His original leak of CIA operative Valerie Plame's job was exposed as far more calculated than the accidental afterthought he claimed. On tape, he blurted it out to Bob Woodward of The Washington Post five separate times in 66 expletive-laced seconds. And out of the blue he called in columnist Robert Novak to tell him — two years after Novak had given up trying to interview him.

    _ Big-time Washington reporters: Exposed as imperfect note-takers eager to shield their high-level official sources even when the sources were just spinning them. Woodward; Judith Miller, then with The New York Times; John Dickerson then with Time, and David Gregory of NBC appeared to shrug off exclusives dropped in their laps.

    _ Matthew Cooper: The former Time magazine reporter won't get work as a note-taker after defense attorneys picked apart his notes and described him interviewing Libby while sprawled on his bed after swimming at the country club. He also used off-the-record material to confirm a story.

    _ Courtroom 16 clock: Original equipment with the courthouse that opened in 1952. Hadn't worked for years, but was repaired once The New York Times reported that "time stands still" at the trial. Couldn't go the distance, was hours behind by trial's end.

    WINNERS

    _ Patrick Fitzgerald: Special prosecutor with an air of righteous indignation battled two of the nation's best criminal defense lawyers head-to-head and won.

    _ Denis Collins: Both sides thought the other would strike this juror who once worked for Woodward and was a neighbor of another witness, NBC's Tim Russert. In seven weeks, the ex-Washington Post reporter developed the hottest insider book proposal in circulation. Publishers: See the preview on Huffingtonpost.com.

    _ Karl Rove: President Bush's political adviser was spared prosecution, and the trial only added to his reputation as the powerful operative behind the curtain. Defense attorneys said the White House was willing to sacrifice Libby to save Rove.

    _ Tim Russert: Survived having the Bush team label his "Meet the Press" as the most pliable venue on Sunday TV and being coaxed early on by an FBI agent into spilling details of his talk with Libby. Stuck to his story during a grueling five-hour cross-examination, and jurors believed him over Libby.

    _ Ari Fleischer: Ex-White House press secretary leaked but got immunity, displayed a previously unknown self-deprecating wit that helped persuade jurors of his credibility and proved he could fend off high-priced defense lawyers as easily as he had White House reporters.

    _ Valerie Plame: Although the leak ended her CIA career, the wife of war critic Joseph Wilson seems to be doing just fine in the limelight with a book on the way, a movie in the works and a lawsuit pending against Bush officials.

    _ Don Imus: Defense attorneys praised the radio and MSNBC morning show host for asking NBC colleagues tough questions that prosecutors weren't asking. "The man in the cowboy hat" appeared in several of video clips in court.

    _ Reggie Walton: In his big-media debut, the presiding judge won admirers for his willingness to change his mind as he doggedly searched for the right path through innumerable legal thickets. Thanks to closed-circuit television, trial observers saw Walton in his element as he handled drug defendants with tough love during breaks in the Libby trial.

    _ Bloggers: They made headlines for taking seats on press row for the first time in a big trial and parlayed it into more Web readership.

    TOO SOON TO CALL

    Theodore Wells and William Jeffress: Displayed the full range of courtroom skills in a lost cause, but they still have another at-bat in the Court of Appeals.


They certainly didn't mince any moonbat-worthy words when it came to the administration.

Thin-skinned Machiavelli who lurked behind the scenes while orchestrating a bungled response...

Can't get past the story of how it messed up intelligence about nonexistent weapons of mass destruction...

I thought I was reading HuffPo. In declaring Fleischer a winner they insulted him - I guess they must have missed him on Letterman - and Rove the winner is still just "Bush's brain." And of course Fitzgerald and the wounded Plame are rock stars.

What exactly was accomplished with this time and money spent getting one guy for perjury for an investigation in which the prosecutor knew 3 months ahead of time who the leaker was?

tomder55 answered on 03/08/07:

where is Andrea Mithchell ? I think her testimony would've been the key to a Libby aquitial as it would've driven home the point of misstatements and lapses in memory . She said that it was "widely known" amongst reporters following the Wilson/Niger story that Wilson wife was with the CIA.She ,Russert ,and probalby Mattthews all knew in advance. Andrea Mitchell has since disavowed the statement, saying she was confused, or it was taken out of context, or she was just wrong. Here's one of her explanations :

“So clearly back in October of ’03, I screwed it up. . . . I was quite surprised to hear about [making the October 2003 statement] because it isn’t consistent with anything in my memory. I can’t find any notes that reflect this, this alleged knowledge, and so I was muddled on the timeline, that is all I can imagine.”.

The judge would not allow her to be questioned during the trial .


Tim Russert is a lying MF .He knew about Plame before the Novak column.
Judge Reggie Walton provided cover for Russert who would've been grilled on the stand during the defense phase when Walton asked for it . Russert was lying out of his ass when he said that he did not know about Plame before his conversation with Libby and he was lying out of his ass when he testified that he did not reveal sources . It was later learned that he fingered Libby to an FBI agent and this was a cornerstone of the prosecution's case.Russert’s interview with the FBI agent was buried in a footnote, allowing Russert to claim he had never revealed a source.Russert did talk to the FBI and they tried to conceal the conversation . He has no integrity .

Russert ,a lawyer himself lied under oath when he said he did not know that people testify before grand juries without their lawyers present. The matter was relevant because the government allowed Russert to give his testimony to the Grand Jury in his office, with his lawyer by his side. These facts would've come out with additional cross examination and the reasonable doubt would've been fortified .

Itsdb rated this answer Excellent or Above Average Answer

Question/Answer
Itsdb asked on 03/07/07 - That's all she wrote...

It's over, it's done, no need to campaign any more. Emily's List has already sworn Hillary in as president on our behalf.



Now that she's in charge perhaps she can do something about we selfish Americans, you know, the ones that led John Edwards to invoke the name of Jesus...

"I think that Jesus would be disappointed in our ignoring the plight of those around us who are suffering and our focus on our own selfish short-term needs...I think he would be appalled, actually."

Wow, the left has sure gotten religious all of a sudden, Hillary reading scripture, equating climate change with "tinkering with God's creation" and doing her best James Cleveland impersonation ... and now Edwards telling us what Jesus would think. I think maybe he might be appalled Edwards wasn't housing a couple dozen homeless at his estate paid for with his earnings as a personal injury lawyer...



A hat tip to tom on the Edwards pic...

tomder55 answered on 03/08/07:

It is clear to me that Edwards is doing his best version of Buddy Holly's fadeaway . He will be irrelelvent to the race by Labor day (which would be a prefectly ironic day for him to drop out of the contest ). I think the contest between Hillary and Obama is which one gets to sit on the back of the bus. I think both will be on the ticket .

Itsdb rated this answer Excellent or Above Average Answer

Question/Answer
tropicalstorm asked on 03/07/07 - ownapieceofamerica

anybody can now own a piece of America

pieceofamerica )

tomder55 answered on 03/08/07:

Well I own a bigger piece than that but I hear rumors that the adjacent property is slated to house a 400 foot high wind farm so I don't think the property will be of much more value than the little sliver the site offers . Maybe I should do a trade in now .

tropicalstorm rated this answer Excellent or Above Average Answer

Question/Answer
tropicalstorm asked on 03/07/07 - Please keep me updated

when the Artic villages need to turn their air conditioners on. Right now it is below zero there.
just curious what they call a heat wave.
would it be 70 degrees or 100 or what?
From what I looked up their average high is 70 degrees
but is that air conditioner weather to them?

tomder55 answered on 03/08/07:

I hear it is very rough on the doggies at the Idiotrod this year.

tropicalstorm rated this answer Excellent or Above Average Answer

Question/Answer
paraclete asked on 03/08/07 - A cost of climate change that can't be counted in dollars - survival, or can it?

In a week that has seen one group absolutely refute man's imput into climate change, isn't it time we stoped arguing the why's and started arguing the abatement of the effects? Who is going to make room for the displaced, look after the sick and increase food production to make up for losses? There will be no point Australia opening it's doors to the displaced of the Pacific rim when it's own agriculture will be devistated, or Europe opening it's doors to the displaced of Africa. America already has a migrant problem whether it admits it or not.

Tony McMichael
March 8, 2007

Doubts about the reality of human-induced climate change have largely dissipated, so now we must face the challenge.

The primary and urgent task is to reduce greenhouse gas emissions. However, since the climate change genie is already partly out of the bottle, we must also devise ways of adapting and lessening adverse effects.

Much of the early public debate about climate change focused on the need to keep the economy ticking over, protect infrastructure, and maintain tourism and recreational facilities. The real problem, though, goes much deeper. Climate change, if not constrained, is ultimately a biological threat. We have been slow to grasp this fundamental point.

Warming is affecting physical and biotic systems. Icesheets are melting faster than was expected just five years ago. Long-term drying is emerging in southern and western Africa, southern Europe, India and Australia. The seasonal cycles of birds, bugs, bears and buds are changing, and are getting out of kilter with one another. This evidence that climate change is disrupting many of Earth's life-support systems means that human health is also at risk.

Initially, the health risks will be greatest in - though not confined to - poorer and vulnerable populations. Many of these vulnerable populations are in tropical and subtropical regions.

In Australia, climate change (including greater weather variability) will cause more death, illness and injury from heatwaves, storms, floods and bushfires. It will influence the range and seasonality of various infectious diseases. For example, outbreaks of the mosquito-borne viral disease dengue will tend to extend southwards, near the coastlines.

Changes in climate will also impair various ecological processes that underpin our health. Crop yields, for example, will be affected by changes in soil moisture, pollinating insect activity, and temperature-sensitive photosynthesis. Recent research indicates that rice yields will decline with warmer temperatures. Such changes in local food production and, hence, prices will affect food choices, nutrition and health.

Meanwhile, the rising probability of population displacement and environmental refugee flows in much of the Asia-Pacific region, due to climatic and other large-scale environmental changes, will pose other risks to social stability, wellbeing and health.

The severity of this drought raises another worrying prospect. Much of rural southern Australia now seems destined for long-term drying. CSIRO modelling indicates that drought frequency is likely to increase over much of Australia this century.

This prospect of prolonged dry conditions casts a long shadow over the outlook for rural livelihoods and living conditions. Rural communities are likely to suffer a range of adverse effects on their health. Their specific health risks include increased exposures to extremes of heat, airborne dusts, and bushfire smoke.

Mental health may be affected. In the more vulnerable communities, where collective resources and resilience are low, there are well-recognised risks of anxiety, depression and suicide. Child health, too, will be jeopardised - especially in relation to emotional experiences, family tensions and the loss of community facilities for play and development.

There are also issues of fresh water and hygiene; food choices and nutrition; and the effect of economic stresses and community erosion on unhealthy behaviours (such as smoking, alcohol consumption and self-medication). For remote indigenous communities, there is the additional risk of loss of traditional plant and animal food species.

This all poses a major research challenge. We need to understand the health risks. We need to understand how different patterns of community response can modify the impacts on wellbeing and health. This will help us shape intervention strategies, social policies and health-care services.

Such interventions are needed more generally in Australia. We should be developing adaptive strategies to protect communities everywhere against the health risks posed by climate change. This discussion is already well advanced in Europe, Canada and, recently, the US.

Meanwhile, two things are clear. First, some degree of human-induced climate change has very likely already occurred. Second, more warming is in the pipeline from recent greenhouse gas emissions whose climatic effects have not yet been fully realised.

Yet the world community has not really understood the full biological and ecological import of this remarkable human-induced environmental change.

The UK Stern report highlighted the long-term risks from unabated climate change to our economic system. Awareness of the risks to species survival, ecological systems and human health should draw our attention to the more serious, fundamental prospect of damage to the world's life-support system.

Tony McMichael is the director of the National Centre for Epidemiology and Population Health at the Australian National University. This is an edited version of an ABC Radio National's Perspectives program and is based on a paper given yesterday at the National Rural Health Conference.

tomder55 answered on 03/08/07:

Chicago : 9th coldest February in 137 years; 4th snowiest since 1929

FEBRUARY SUMMARY: It was the coldest February since at least 1989 (18 years) and possibly 1979 for the nation as a whole, and the month is expected to rank between the 8th and 15th coldest in 113 years of national records. National precipitation trended up 134% over last year with snowfall up 60% over last year.

An analysis of the United Nations widely-touted 2007 IPCC Global Warming Summary for Policymakers by UK Lord Viscount Monckton has found 31 errors and exaggerations. Since Lord Monckton alerted the UN about its errors , the UN substantially rewrote and corrected the report, Monckton claims in his new analysis.

The UN's climate-change body, the Intergovernmental Panel on Climate Change, did not reply directly to Lord Monckton's criticism, but made many of the corrections nevertheless. "The tradition of elementary but serious scientific errors, of which the notorious `hockey-stick' graph of estimated global temperatures over the past 1,000 years is an example, is alive and well in the UN's 2007 report," Lord Monckton said.

"The UN has still not corrected or apologized for the `hockey-stick', by which it falsely abolished the mediaeval warm period, when temperatures were 2 or 3C warmer than today, and disaster failed to ensue. But it has been forced to correct several schoolboy howlers - though it has not had the honesty to announce publicly and clearly that it has done so," Monckton said. "The heavily-corrected version of the IPCC report has been furtively posted on the IPCC's website, www.ipcc.ch. There has been no public statement by the IPCC admitting to the errors," Monckton added.

The UN has been forced to halve its high-end estimate of the rise in sea-level to 2100, and it has also sharply reduced its estimate of our entire effect on the climate since 1750, according to Monckton. Monckton echoed UK Lord Nigel Lawson's call that the IPCC be disbanded. "It is too politicized and too incompetent to serve any useful purpose," Monckton said.

Monckton's new analysis also points out significant science errors in Al Gore's Oscar winning film "An Inconvenient Truth." "The IPCC's exaggerations and errors parallel those of Al Gore in his notorious sci-fi horror film An Inconvenient Truth, now being peddled to schoolchildren worldwide," Monckton said.

The growth of methane -- one of the most potent global warming gases -- has stalled after rapidly rising in the Earth's atmosphere for more than a century, Oregon scientists say. In the most detailed look at methane measurements, researchers at Portland State University and Oregon Health & Science University find that the buildup of methane in the atmosphere has been slowing for nearly a quarter of a century. And the Oregon scientists don't foresee methane emissions increasing again anytime soon because of human activities. "It's good news because you have one global-warming gas that's not increasing very rapidly, or at all," said Aslam Khalil, an atmospheric physicist at PSU.

Global warming has taken the place of Communism as an absurdity that the left will defend to the death regardless of the evidence showing its folly.

paraclete rated this answer Excellent or Above Average Answer

Question/Answer
tropicalstorm asked on 03/07/07 - someone on yahoo asked can my trailer court
do this too???

BOSTON (Reuters) - More than 30 Vermont towns passed resolutions on Tuesday seeking to impeach
President Bush, while at least 16 towns in the tiny New England state called on Washington to withdraw U.S. troops from
Iraq.

Known for picturesque autumn foliage, colonial inns, maple sugar and old-fashion dairy farms, Vermont is in the vanguard of a grass-roots protest movement to impeach Bush over his handling of the unpopular Iraq war.

"We're putting impeachment on the table," said James Leas, a Vermont lawyer who helped to draft the resolutions and is tracking the votes. "The people in all these towns are voting to get this process started and bring the troops home now."

The resolutions passed on Vermont's annual town meeting day -- a colonial era tradition where citizens debate issues of the day big and small -- are symbolic and cannot force Congress to impeach Bush, but they "may help instigate further discussions in the legislature," said state Rep. David Zuckerman.

"The president must be held accountable," said Zuckerman, a politician from Burlington, Vermont's largest city.

After casting votes on budgets and other routine items, citizens of 32 towns in Vermont backed a measure calling on the U.S. Congress to file articles of impeachment against Bush for misleading the nation on Iraq's weapons of mass destruction and for engaging in illegal wiretapping, among other charges.

Five Vermont towns passed similar resolutions last year.

The idea of impeaching Bush resides firmly outside the political mainstream.

The new Democratic-controlled Congress has steered clear of the subject, and Wisconsin Sen. Russell Feingold's call last year to censure Bush -- a step short of an impeachment -- found scant support on Capitol Hill, even among fellow Democrats.

Vermont's congressional delegation has shown no serious interest in the idea.

'SOLDIERS HOME NOW'

Sixteen Vermont towns passed a separate "soldiers home now" resolution calling on the White House, the U.S. Congress and Vermont's elected officials to withdraw troops from Iraq.

"The best way to support them is to bring each and every one of them home now and take good care of them when they get home," the resolution said.

It was unclear how many towns had put the resolutions to a vote, and the results of all the town meetings in the state of about 609,000 people may not be known for days.

Residents of Burlington were voting on a separate question calling for a new investigation into the September 11 attacks.

Voters were asked to circle "yes" or "no" to the question: "Shall Vermont's Congressional Delegation be advised to demand a new, thorough, and truly independent forensic investigation that fully addresses the many questions surrounding the tragic events of September 11, 2001?"

Doug Dunbebin, who gathered signatures to get the issue on the ballot, said questions linger about September 11, when hijacked plane attacks killed nearly 3,000 people at New York's World Trade Center, at the
Pentagon and in Pennsylvania.

A group known as Scholars for 9/11 Truth believes the events of that day were part of a conspiracy engineered by the U.S. government and that it took more than two planes to bring down the Twin Towers in New York.

Vermont's new U.S. representative, Peter Welch (news, bio, voting record), a Democrat, said there was no need for a further investigation.

tomder55 answered on 03/07/07:

Funny thing about impeachments . The Costitution clearly gives that power to Congress ;not to some local yokel town board.

Article 1, Section 2, Clause 5--The House of Representatives shall choose their speaker and other officers; and shall have the sole power of impeachment.

Article 1, Section 3, Clause 6--"The Senate shall have the sole power to try all impeachments.

Maybe the people of the good state of Vermont ought to pick up a copy and read it.

kindj rated this answer Excellent or Above Average Answer
tropicalstorm rated this answer Excellent or Above Average Answer

Question/Answer
tropicalstorm asked on 03/06/07 - SUICIDE?

MOSCOW - A journalist who fell to his death from a fifth-story window had received threats while gathering material for a report claiming Russia planned to provide sophisticated weapons to
Syria and
Iran, his newspaper said Tuesday.

Prosecutors have opened an inquest into the death of Ivan Safronov, a military affairs writer for the daily Kommersant who died Friday in what some media said could have been murder.

Kommersant reported that Safronov told his editors he would write a story about Russian plans to sell weapons to Iran and Syria via Belarus, but they said he had not yet submitted the article.

Kommersant said Safronov recently told colleagues that he had been warned he would face a criminal investigation on charges of revealing state secrets if he reported allegations that Russia had struck a deal to supply highly advanced Iskander missiles to Syria. If confirmed, such a contract would upset the balance of forces in the Mideast and likely anger
Israel and the United States.

Safronov did not say where the warning came from, according to Kommersant, but he had repeatedly been questioned in the past by the Federal Security Service or FSB, which suspected him of divulging state secrets in his reports. The FSB is the main successor agency to the KGB.

"Ivan Safronov said he was not going to write about it for a while because he was warned that it would create a huge international scandal and the FSB would launch a criminal case on charges of breaching state secrets," it said.

Russia's arms sales to such nations as Iran and Syria have been an irritant in its relations with the United States and have drawn criticism from Israel.

Ignoring U.S. and Israeli criticism, Russia has delivered 29 Tor-M1 mobile surface-to-air missile systems to Iran under a $700 million contract. Russian news reports have said repeatedly the Iranians were pushing Russia to sell much more potent, long-range S-300 air defense missile systems.

Kommersant reported that, before his trip to an international arms fair last month in the United Arab Emirates, Safronov said he would try to confirm indications that Russia planned to sell S-300 missiles to Iran and Su-30 fighter jets to Syria via Belarus. Safronov later called his editors from Abu Dhabi and said he had received the necessary confirmation from Russian officials who attended the exhibit, Kommersant reported.

Upon his return from the trip, it said, Safronov told colleagues that he also had learned about Russian plans to provide Syria with Iskander missiles, MiG-29 fighter jets and Pantsyr-S1 air defense systems.

The Iskander, a sophisticated surface-to-surface missile with a range of 175 miles, would give Syria the capability to strike targets in Israel with high precision. Israel has complained about past sales of anti-tank missiles to Syria, saying that some landed in the hands of the militant group Hezbollah.

A spokeswoman for the Rosoboronexport state arms trading monopoly refused to comment on Kommersant's report on the alleged weapons deals.

Russia has been plagued by attacks on reporters who seek to expose official corruption and other abuses. The problem was highlighted by the October killing of Anna Politkovskaya, an investigative reporter and a harsh critic of human rights abuses in
Chechnya.

A report Tuesday from the Brussels-based International News Safety Institute listed
Iraq, Russia and Colombia as the deadliest countries for journalists and their support staff. There were 138 deaths in Iraq over the past decade, 88 in Russia and 72 in Colombia, the report said.

___

tomder55 answered on 03/07/07:

Gives new meaning to the line "Don't kill the messenger " .This last decade has been rough on the profession with over 1000 reporters slain worldwide with Iraq and Russia topping the list as the deadliest countries for the profession.[88 journalists were killed in Russia. 136 journalists have been killed in Iraq.]

Safronov lived on the 3rd story but went up 2 flights so he could do his Peter Pan imitation from the 5th floor.I don't know. The KGB in the old days had much more imagination. When they poinsoned someone it could never be traced back to them.

But these activities have taken new turns that are disturbing . Alexander Litvinenko we all know was offed in London . But just recently a Putin critic,Paul Joyal , was shot and wounded in the groin (ouch)in front of his house in Prince George’s County Va.;Two days after a national television appearance on 'Dateline'questioning whether Putinwas behind the poisoning of Litvinenko. It has been attributed to a robbery attempt.(yeah right!).On Dateline, Joyal gave this assessment of the Kremlin’s special way of handling critics and journalists:
“A message has been communicated to anyone who wants to speak out against the Kremlin: ‘If you do, no matter who you are, where you are, we will find you, and we will silence you — in the most horrible way possible.’” and that includes getting shot in the nuts .

Other hits against Americans have included :

Paul Klebnikov ; an editor at Forbes magazine,was shot nine times outside his office in Moscow in 2004 and died in a hospital.

Yesterday it was reported that two American women are now in the hospital in Moscow having been poisoned by the colorless, odorless and highly radioactive Thallium.


As far as arms sales go ;I'm sure the Ruskies weren't thrilled when we were providing Stingers to the Afghans in the 1980s .

tropicalstorm rated this answer Excellent or Above Average Answer

Question/Answer
Itsdb asked on 03/06/07 - That reminds me...

Tom mentioned Jim Webb - did any of you read or hear about this?

Lies and damn lies
Cal Thomas

    In the words of Hollywood mogul David Geffen, "Everybody in politics lies…" But when some politicians tell lies that damage a person's character in the eyes of voters and ultimately lead to his defeat, those are damnable lies that need to be corrected.

    Last fall, about a month before the November election, the Associated Press ran a story that claimed Sen. George Allen, Virginia Republican, had failed to disclose stock options he had earned while serving as a director of Commonwealth Biotechnologies Inc. (CBI). The story suggested Allen might have violated ethics rules because the company, which is based in Richmond, had conducted business with the state when Allen was governor. Allen had served on CBI's board between his departure as governor and his election to the Senate.

    Allen reported the stock options in 2000, but he did not file subsequent reports because the price of CBI stock plunged, making the options worth less than he paid for them, denying him a profit.

    Last October, the Associated Press ran a story that said Allen had failed to report his CBI stock options and hinted at possible wrongdoing by Allen when he was governor because the company had done business with the state. This was all that Allen's challenger, now Sen. James Webb, and the Democratic Senatorial Campaign Committee headed by New York Sen. Chuck Schumer needed. They prepared an attack ad, alleging that Allen's stock options were worth $1.1 million and were not worthless, as he had claimed. The ad also made the connection between CBI and the state, charging Allen tried to "steer government contracts to a company that paid him in stock options." AP did not report anything about Allen trying to steer government contracts to the state, but Jim Webb "approved this message" anyway.

    An analysis of the negative ad by AP political writer Bob Lewis revealed its inaccuracies. One must conclude that, since the information was available to Webb and Schumer, the two deliberately used factual inaccuracies in the negative ad. But why let truth get in the way of an effective election strategy? The damage was done and since the ad fit nicely into the Democrats' theme of "the culture of corruption" in the Republican majority, the desired result was achieved. Allen lost the election by 9,000 votes.

    The Allen camp asked for a formal ruling by the Senate Ethics Committee and on Feb. 16, it came. In a letter to Allen, signed by committee chairman Barbara Boxer, California Democrat and committee vice chair John Cornyn, Texas Republican, Allen was exonerated of any wrongdoing: "The committee has determined that your ownership of CBI stock options did not constitute deferred compensation during the relevant reporting periods." Therefore, they said, Allen was not required to amend the reports.

    Allen made his share of mistakes during his re-election campaign, but this was not one of them. His opponent and Sen. Schumer, neither of whom has apologized or retracted their accusations, unfairly smeared him.

    In commenting on the Senate Ethics Committee letter and the incorrect negative ad that contributed to Allen's defeat, a Richmond Times Dispatch editorial asked a question familiar to many public figures who have been unfairly slimed, "So where does George Allen go to get his reputation back, never mind his job in the Senate?"

    Where, indeed? The AP printed a story on Feb. 21 correcting the errors in its earlier story that were used in the Allen attack ad, but it came nearly four months too late.

    This saga is important for a number of reasons. First, it cost a good man an important job. Second, it significantly contributed to a change in the balance of power in the Senate. Third, it again exposed an unholy alliance between liberal politicians and the leftist big media who are quick to attack someone whose policies and party they don't like, but rarely correct errors of their own making, or investigate bogus charges when they help the policies and party the media prefer.


Culture of corruption indeed. It's a real shame the American public failed to recognize where the real culture of corruption was...

tomder55 answered on 03/07/07:

I can't for the life of me undersatnd why anygood person would want to run for public office these days . It has become a blood sport where the losers reputations are forever smeared or in the case of Libby ,thrown into jail. The press being much less than observers to the process have increasingly become arbiters . If you lie or misrepresent the facts in the right way, at the right time, the damage is permanently done. Retractions do not have anywhere's near the same weight. You note that it took commentary by Cal Thomas and lesser read journals like NewsMax for this to get any coverage .The electorate for the most part work their asses off and become "infomed electorate " by watching a 25 minute segment of the evening news . Frankly they are easy to manipulate . What Allen brought to the table ultimately was not as relevent as the uttering of a single nonsensical word :"Macaca.""Macaca.""Macaca.""Macaca."
"Macaca.""Macaca.""Macaca.""Macaca."
Now we have Jim Webb instead....Iran's greatest ally next to Hafez al-Assad .

Itsdb rated this answer Excellent or Above Average Answer

Question/Answer
Itsdb asked on 03/06/07 - Refusing to move on?

Often when issues of race are mentioned here I've wondered when we can just move past all of the nonsense. Sure there is still racism but will it ever be eradicated? I don't think so, but I do believe if people will quit reminding us of their color and telling us how bigoted and racist we are then things will get better. I don't need anyone to tell me they're black, I can see it - and I don't care.

Bruce S. Gordon just resigned as president of the NAACP, "after clashing with the board over the group's mission."

Apparently Gordon wanted to "pull (them) into the post-civil rights period" according to Julian Bond. Bond of course "firmly rejected the idea."

"We're not post civil rights," he said. "The struggle continues."

Gordon had another vision, "We are going to be very outcome-oriented, very results-oriented," he said in July, "as opposed to activity and effort-oriented."

Who needs results? Bond insisted the NAACP is "resisting philosophical change. We're staying the course."

Doesn't Bond realize how much trouble that phrase got Bush into? I guess black leaders are still more interested in victimhood than solving problems. So apparently are the Democratic presidential frontrunners as they showed in their gratuitous church visit on Sunday.

Comments?

tomder55 answered on 03/06/07:

That is what makes the Obama campaign so intriguing . Did you catch any of his address this weekend ? He sounded at times like Bill Cosby .He recognized the contributions and paid tribute to the Civil Rights movement; while at the same time he said something was missing from the current generation.He could've pandered like Madame Defarge but instead he tried to expand the dialogue ....in fact it could've easily been made by a white candidate .



“I can’t says for certain that we have instilled that same sense of moral clarity and purpose in this generation. Sometimes I feel like we’ve lost it a little bit,”

“I don’t know who taught black kids that reading and writing and conjugating yo’ verbs was “acting white,” we gotta get over that mentality. "

"don't tell me that it doesn't have a little to do with the fact that we got too many daddies not acting like daddies. Don't think that fatherhood ends at conception."


If a Sista Soulja moment is required he already made it . It was an impressive performance.

Now I have a ton of questions about the guy ;he is still a blank slate in my book .But in a race between him and Evita he wins hands down .It would be wise not to underestimate him.

Itsdb rated this answer Excellent or Above Average Answer

Question/Answer
ETWolverine asked on 03/06/07 - Israel, The Bully.

Neighborhood Bully

Well, the neighborhood bully, he's just one man,
His enemies say he's on their land.
They got him outnumbered about a million to one,
He got no place to escape to, no place to run.
He's the neighborhood bully.

The neighborhood bully just lives to survive,
He's criticized and condemned for being alive.
He's not supposed to fight back, he's supposed to have thick skin,
He's supposed to lay down and die when his door is kicked in.
He's the neighborhood bully.

The neighborhood bully been driven out of every land,
He's wandered the earth an exiled man.
Seen his family scattered, his people hounded and torn,
He's always on trial for just being born.
He's the neighborhood bully.

Well, he knocked out a lynch mob, he was criticized,
Old women condemned him, said he should apologize.
Then he destroyed a bomb factory, nobody was glad.
The bombs were meant for him.
He was supposed to feel bad.
He's the neighborhood bully.

Well, the chances are against it and the odds are slim
That he'll live by the rules that the world makes for him,
'Cause there's a noose at his neck and a gun at his back
And a license to kill him is given out to every maniac.
He's the neighborhood bully.

He got no allies to really speak of.
What he gets he must pay for, he don't get it out of love.
He buys obsolete weapons and he won't be denied
But no one sends flesh and blood to fight by his side.
He's the neighborhood bully.

Well, he's surrounded by pacifists who all want peace,
They pray for it nightly that the bloodshed must cease.
Now, they wouldn't hurt a fly.
To hurt one they would weep.
They lay and they wait for this bully to fall asleep.
He's the neighborhood bully.

Every empire that's enslaved him is gone,
Egypt and Rome, even the great Babylon.
He's made a garden of paradise in the desert sand,
In bed with nobody, under no one's command.
He's the neighborhood bully.

Now his holiest books have been trampled upon,
No contract he signed was worth what it was written on.
He took the crumbs of the world and he turned it into wealth,
Took sickness and disease and he turned it into health.
He's the neighborhood bully.

What's anybody indebted to him for?
Nothin', they say.
He just likes to cause war.
Pride and prejudice and superstition indeed,
They wait for this bully like a dog waits to feed.
He's the neighborhood bully.

What has he done to wear so many scars?
Does he change the course of rivers?
Does he pollute the moon and stars?
Neighborhood bully, standing on the hill,
Running out the clock, time standing still,
Neighborhood bully.

By Robert Allen Zimmerman, AKA Bob Dylan

From the album "Infidels", Copyright © 1983 Special Rider Music

tomder55 answered on 03/06/07:

I think the neighhood bully should've adopted the Andy Jackson solution .The United States could not have survived and prospered with such a deadly enemy threatening it's borders .So he relocated the problem . The settlers built their communities in peace. It seems to me that the more Israel concedes the more existential danger it places itself into.

ETWolverine rated this answer Excellent or Above Average Answer

Question/Answer
tropicalstorm asked on 03/06/07 - Libby verdict

WASHINGTON - Former White House aide I. Lewis "Scooter" Libby was convicted Tuesday of obstruction, perjury and lying to the
FBI in an investigation into the leak of a
CIA operative's identity.
ADVERTISEMENT

Libby, the former chief of staff to Vice President
Dick Cheney, was accused of lying and obstructing the investigation into the 2003 leak of CIA operative
Valerie Plame's identity to reporters.

He was acquitted of one count of lying to the FBI.

Libby had little reaction to the verdict. He stood expressionless as the jury left the room. His lawyer, Theodore Wells, said they were "very disappointed" with the verdict.

The verdict was read on the 10th day of deliberations. Libby faces up to 30 yeaerrs in prison, though under federal sentencing guidelines likely will receive far less.

U.S. District Judge Reggie B. Walton ordered a pre-sentencing report be completed by May 15. Judges use such reports to help determine sentences.

Libby faced two counts of perjury, two counts of lying to the FBI and one count of obstruction of justice. Prosecutors said he discussed Plame's name with reporters and, fearing prosecution, made up a story to make those discussions seem innocuous.

Libby's defense team said he learned about Plame from Cheney, forgot about it, then learned it again a month later from NBC newsman Tim Russert. Anything he told reporters about Plame, Libby said, was just chatter and rumors, not official government information.

Special prosecutor Patrick Fitzgerald said that was a lie. But Libby's defense team had argued that it would be unfair to convict Libby in a case where so many witnesses changed their stories or had memory problems.

Wells said he would ask the court for a new trial by April 13. Such requests are common following criminal convictions.

"Despite our disappointment in the jurors' verdict, we believe in the American justice system and we believe in the jury system," Wells told reporters outside the federal courthouse. "We intend to file a motion for a new trial and if that is denied, we will appeal the conviction. We have every confidence that ultimately Mr. Libby will be exonerated.... We intend to keep fighting to establish his innocence."

Libby will be allowed to remain free while awaiting sentencing, which is set for June 5.

As the verdicts were read, Libby's wife choked out a sob and sank her head. Moments later, she embraced the defense attorneys.

The jury acquitted Libby of one count of lying to the FBI about his conversation with Time magazine reporter Matthew Cooper.

tomder55 answered on 03/06/07:

Always take the fifth . That's the lesson I take out of this and the Martha Stewart trial.Evidently reporters like that weasel Russert, whose own notes contradict their version of events are more believable to a jury than an administration employee .

The other lesson is don't go on trial in DC . The only jurors they could find that didn't have much pretrial knowledge of the case were too ignorant to deliberate and judge the facts of the case.Even today they were asking the judge what the charges agaisnt Libby were .Friday they asked the judge what "reasonable doubt " meant. I'm sure the jurors when questioned will prove that they had no clue about what they convicted him on .The MSM will of course egg them on and the headlines will be about outing Plame.Already that p.o.s.Harry Reid is sending out statements about manipulation of intelligence and demanding that Bush don't pardon Libby .


I think the punishment should fit the crime . The precedence for committing pergury to a grand jury is getting your license suspended (Clinton) . The precedence for theft of national security documents is community service and a $50,000 fine (Sandy Berger ).The precedence for stuffing your freezer full of bribe money is losing a seat on an important House of Representative comittee.(William Jefferson) This is the Martha Stewart verdict redux. Libby guilty ...unbelievable ...Two border agents are sent to prison on the word of a Mexican drug dealer who's granted immunity..even better .

Itsdb rated this answer Excellent or Above Average Answer
tropicalstorm rated this answer Excellent or Above Average Answer

Question/Answer
Itsdb asked on 03/05/07 - The makeover continues?

For the life of me I can't recall Hillary ever showing her religious side before, or maybe she did and I just missed it. Excerpts from yesterday's love fest in Selma:

    This is the day the Lord has made. Let us rejoice and be glad in it. And I want to begin by giving praise to the Almighty for the blessings he has bestowed upon us as a congregation, as a people, and as a nation. and I thank you so much, Reverend Armstrong, for welcoming me to this historic church.

    And I thank the First Baptist Church family for opening your hearts and your home to me and to so many visitors today. I have to confess that I did seek dispensation from Reverend Armstrong to come because you know, I'm a Methodist. And I'm in one of those mixed marriages.


She can't say those things can she? Mixed marriage? Is she referring to Baptist/Methodist or black/white, since Bill was "the first black president?" (On a side note, notice this remark by Clinton back on 2001, "And those of us who don't happen to fall in those categories are infidels and all of us are fair game.")

    I come here this morning as a sister in worship...


A sister in worship?

    How can we say everything is fine when we have an energy policy whose prices are too high, who make us dependent on foreign governments that do not wish us well, and when we face the real threat of climate change, which is tinkering with God's creation?


Wow, invoking "tinkering with God's creation" in relation to climate change? How in the world is she going to reconcile that one?

    We have to stay awake. We have a march to finish. On this floor today, let us say with one voice the words of James Cleveland's great freedom hymn, "I don't feel no ways tired/I come too far from where I started from/Nobody told me that the road would be easy/I don't believe he brought me this far to leave me."


Isn't the idea of Hillary quoting a song like this a bit patronizing?

    And we know -- we know -- we know, if we finish this march, what awaits us? St. Paul told us, in the letter to the Galatians, "Let us not grow weary in doing good, for in due seasons we shall reap, if we do not lose heart."


First black gospel, now the bible? OK, any takers on how many times she quotes scripture, uses the word worship, cites a James Cleveland song or mentions "God's creation" anywhere but at a church from here on out?

tomder55 answered on 03/05/07:

I said to Saphh this morning : Isn't it the liberals who always denounce the introducing of religion into politics ? Yet both Obama and Clinton did campaigning in churches this weekend;with Hillary doing a God-aweful imitation of Billy Graham.So much for that separation of church and state.

Drudge has the audio .He called it a Kentucky Fried Hillary . That would be :

1 skinny leg
1 fat thigh
1 left wing
No Breast.




AAAAAAhhh don' feel no ways taahrd .Ah come to faaaaaaaaaar Can't wait the hear her in a Brooklyn Synagogue . Maybe she can take Chinese lessons from Rosie

Imagine a Republican trying to pander to the Mex.-American vote sounding like Speedy Gonzales .

Itsdb rated this answer Excellent or Above Average Answer

Question/Answer
Itsdb asked on 03/05/07 - On Walter Reed Hospital

Let me first say I believe it is our duty to furnish first rate care for our veterans. On the other hand I have to question the timing and motive of WaPo's exposé on Walter Reed. That VA hospitals have had a history of mismanagement and poor conditions is not news. What is news is that since the deplorable conditions many were in in the 90's, the quality of care has seen a sharp increase.

A 2005 article in Washington Monthly notes most grievances are about access to the system, not about the quality of care received by those who get in.

The article also noted the VHA system received the National Committee for Quality Assurance's seal of approval, In every single category, the VHA system outperforms the highest rated non-VHA hospitals.

Since WaPo's article, bloggers and other critics have seized on another Bush 'failure.'

    Sen. Carl Levin, D-Mich., said the scandal is emblematic of the Bush administration.

    "I believe this disgraceful neglect has been the result of two things. One is a lack of accountability," said Levin, who is chairman of the Senate Armed Services Committee.

    Levin said the second factor "is the overoptimism which just symbolized this administration right from the beginning of this war, that the mission was accomplished."


Chucky Schumer wrote to SecDef Gates, “To think that men and women are serving their country in the most honorable and courageous way possible and all we give them is a dilapidated, rat-infested, run-down building to recover is a disgrace. My fear is that Walter Reed is just the tip of the iceberg, and merely highlights the pervasive and systemic mistreatment of our service members.”

He said on ABC's "This Week" yesterday, "I'm worried about if it's this bad at the outpatient facilities at Walter Reed, how is it in the rest of the country? Because Walter Reed is our crown jewel."

Crown Jewel, eh? So why is Walter Reed moving to Bethesda as part of the latest BRAC decisions?

    An Office of Integration was formed in November 2005 to oversee ... the merger of Walter Reed Army Medical Center and National Naval Medical Center by September 2011 into one tri-service military medical center located on the Bethesda campus. That facility will be called the Walter Reed National Military Medical Center, and it will be staffed by Air Force, Army, and Navy military personnel.


Sustandard conditions for our military heroes is unacceptable on any level, but since WRAMC will be moved completely to the Bethesda campus by 2011 aren't these comments by the left about dilapidated, run-down buildings a bit disengenuous ... particularly when they can't make up their minds on whether or not they give a damn about the troops? And, is a failure in any government-run operation a surprise?

tomder55 answered on 03/05/07:

I don't think that Walter Reed is part of the VA . It's a DOD facility . Bethesda is a first rate and as you pointed out ;Walter Reed was scheduled for shut down and consolidation with Bethesda .Note my link is from May of last year and this consolidation was part of BRAC .Why would the Compost be suprised that the facilities at Walter Reed were not first class ? But still, the conditions there were being ignored and someone needed to say it.

You hit the nail on it with the last sentence. We should all keep this in mind as we contemplate nationally managed health care for everyone. I'd say Walter Reed and the VA are symbolic of the problem .Military families have always known this truth(although the quality of the care varies depending on the location).

I don't know when the shoddy conditions began. All I know is that the primary care of the injured troops have been 1st class. The survival rate has never been better.

I don't think it is news that the VA has generally been a sloppily run institution and I'm sure the increases in clientelle due to the war against jihadistan has added pressure on them .

Where was Henry Waxman's grandstanding about this 10 years ago ? This did not suddenly happen within the last 6 years .Military hospitals were only one of the casualties suffered by the huge cutbacks in military spending under Clinton.

Itsdb rated this answer Excellent or Above Average Answer

Question/Answer
Itsdb asked on 03/03/07 - Clamping down on kids

Critics denounce Pizza Hut program that rewards young readers with free pizzas

By David Crary
ASSOCIATED PRESS

12:51 p.m. March 2, 2007

NEW YORK – You've read the book, now eat the pizza.

Since 1985, that's been the gist of Pizza Hut's Book It, an incentive program used by 50,000 schools nationwide to reward young readers with free pizzas. The program is now under attack by child-development experts who say it promotes bad eating habits and turns teachers into corporate promoters.

Book It, which reaches about 22 million children a year, “epitomizes everything that's wrong with corporate-sponsored programs in school,” said Susan Linn, a Harvard psychologist and co-founder of the Campaign for a Commercial-Free Childhood.

“In the name of education, it promotes junk food consumption to a captive audience ... and undermines parents by positioning family visits to Pizza Hut as an integral component of raising literate children,” Linn said.

This week, Linn's organization called on parents to end their schools' participation in the long-standing program.

Though some activists have previously questioned Book It, Linn said Friday that only after the recent upsurge of concern over child obesity and junk food did her group feel it could make headway with a formal protest campaign. She said many schools are trying to reduce students' access to soda, and contended that Book It should face similar scrutiny.

But the program – which has given away more than 200 million pizzas – has deep roots and many admirers at the highest levels of politics and education. It won a citation in 1988 from President Reagan, and its advisory board includes representatives of prominent education groups, including teachers unions and the American Library Association.

“We're really proud of the program,” said Leslie Tubbs, its director for the past five years. “We get hundreds of e-mails from alumni who praise it and say it helped them get started with reading.”

Dallas-based Pizza Hut says Book It is the nation's largest reading motivation program – conducted annually in about 925,000 elementary school classrooms from Oct. 1 through March 31. A two-month program is offered for preschoolers.

Participating teachers set a monthly reading goal for each student; those who meet the goal get a certificate they can redeem at Pizza Hut for a free Personal Pan Pizza. Families often accompany the winners, turning the event into a celebration that can boost business for the restaurant.

Teachers find the program an enjoyable way to build interest in reading, Tubbs said. “We're helping them to do their jobs,” she said.

At Strafford Elementary School in Strafford, Mo., the roughly 500 students collectively read 30,000 books a year with Book It's help, said principal Lucille Cogdill.

“I don't have any negative things at all to say about it,” Cogdill said. “I know there's concern about obesity, but Book It is not causing it, and the schools aren't causing it.”

Chris Carney, principal at Bennett Elementary School in Fort Lauderdale, Fla., also is a Book It fan, saying it encourages family togetherness and provides a tool for persuading children to try books instead of video games.

“I don't want to see kids gorging pizzas,” he said. “But the positive effects outweigh other effects.”

Among those campaigning against Book It is Alfie Kohn, an author whose 11 books on education and parenting include “Punished By Rewards, which questions the value of incentive programs.

“The more kids see books as a way to get pizza or some other prize, the less interest they'll have in reading itself,” Kohn, a former teacher, said in a telephone interview. “They tend to choose easier books to get through faster.”

Another critic of Book It and the broader phenomenon of corporate incursions into schools is Alex Molnar, director of the Commercialism in Education Research Unit at Arizona State University.

He described Book It as a “dreadful program” that puts pressure on parents to celebrate with their reward-winning children at Pizza Huts.

“This is corporate America using the schools as a crow bar to get inside the front doors of students' homes,” he said. “It's very hard for children whose parents who don't want to engage in this to not feel ostracized.”

Molnar acknowledged that Book It is well-regarded by many educators and politicians, but said it might be reevaluated in light of rising concerns about child obesity.

“To the extent that this program is correctly identified as part of the problem, then there's a chance of reducing its scope,” he said.
~~~~~~~~~~~~~~~~~~~~~~~~~~~~~~~~~~~~~~~~~~~~~~~~~~~~~~~

I'm sorry, but this just makes me mad. Why the %$#@ should anyone care if a kid gets rewarded with a pizza for reading books? These Nazis, the very ones that tell everyone else to mind their own business, want to dictate every facet of YOUR life. It's a wonder kids can have any fun any more or ever learn a *&%^ thing. Don't teach them about competition or winning, don't use red pencils for marking papers any more because it might damage their little psyches, pass them when they don't know the material - and take away their incentives.

Oh that's right, you can't teach them how to think because then they'll be able to recognize when they're being controlled, indoctrinated and used as pawns in this dangerous "progressive" game being played.

No?

tomder55 answered on 03/04/07:

It is absolutely un-American to attack pizza .You can cater a pizza to anyones desires. Although it originated in Italy a a peasants food it never obtained popularity until America adopted it and tinkered with the recipes.The ways to make it are unlimitted today .With the choices available it is the perfect American food .It almost replaces apple pie.

Pizza is a combination food .It takes ingredients from many of the food groups on the pyramid and combines them in one healthy and tasty recipe. A teacher could make a whole lesson out of dissecting a pizza and asking the students to catalogue the food groups used to make a pizza.The crust is from grain .... Carbohydrate for energy.The sauce is from veggies.Tomatos are high in Vitamin A for night vision ;antioxidants and the new hot nutritional item lycopene.
The cheese from dairy ;a good source of calcium for strong bones...very important in a young child's development .Any meat is from the meat or protein group which is necessary in developing strong muscles .

OK ;so it is often loaded with sodium and saturated fats the way places like Pizza Hut makes it but no one says it has to be eaten all at once.The biggest problem in America isn't what we eat ,but our portion control.

It is patently rediculous to condemn corporate influence in the public schools. The same people doing it are the loudest critics of how schools are funded today .Heck ,their new rock star candidate Barak Obama has made "Encourage private or corporate investment in public school programs".
one of his education goals I think we need to put MORE pressure on the corporate world to assist in the education of our students. These students are the future employees and any education they get in the childhood will be less on the job training needed. Corporations like J&J and Microsoft have contributed heavily to public education and that model should be encouraged for more American companies. In fact it’s surprising that there’s not more of an investment in education by corporations ;it happens to be a great reputation booster. For the life of me I cannot find a down side to it .


Itsdb rated this answer Excellent or Above Average Answer

Question/Answer
Itsdb asked on 03/02/07 - Is the American Nuremberg near?

As you may recall, the latest rage among the greens is to call for a "climate Nuremberg" for those "bastards" destroying the climate. Now we know for sure those "bastards" refer to the U.S.

    Arctic residents tell panel U.S. carbon emissions violate their human rights

    By George Gedda
    ASSOCIATED PRESS

    10:16 a.m. March 1, 2007

    WASHINGTON – Northern Canadians told an international commission Thursday that carbon emissions from the United States have contributed so much to global warming that they should be considered a human rights violation. One activist said temperatures have climbed so much that Arctic residents need air conditioners.

    The case was pressed by the Inuit community before the 34-nation Inter-American Commission on Human Rights. In a petition, the group asked the commission's assistance “in obtaining relief” from the impact of global warming, and makes specific reference to the United States as the country most responsible for the phenomenon.

    The commission, however, lacks the legal authority to compel the United States to take action.

    Sheila Watt-Cloutier, an Inuit activist, said the well-being of her people is under threat – and that the need for air conditioning is just one example of the spread of global warming.

    Climate change, she said is “destroying our right to life, health property and means of subsistence,” she said. “States that do not recognize these impacts and take action violate our human rights.”

    She said ice formations are much more likely to detach from land, and take unsuspecting hunters out to sea where they face an uncertain fate.

    Beyond that, she said hunters can no longer be sure of ice thickness and whether it is safe to travel.

    “Many hunters have been killed or seriously injured after falling through ice that was traditionally known to be safe,” she said.

    The United States did not respond to the Inuit claims before the commission, an arm of the Organization of American States. The Bush administration has said it is taking steps to reduce global warming, but domestic and international critics say it is not doing enough, given that the United States is the world's largest emitter of greenhouse gases.

    Scientists generally agree the Arctic is the first place on Earth to be affected by rising global temperatures. They say that unless developed nations such as the United States – responsible for one-fourth of world's greenhouse gases – do not dramatically reduce their emissions within the next 15 years, the Arctic ice likely will melt by the end of the century.

    The Inuit population hails from Canada, Russia and Greenland, as well as Alaska, where they are known as Eskimos. They have been trying to tell the world for more than a decade about the shifting winds and thinning ice that have damaged the hunting grounds the Northern peoples have used for thousands of years.

    Watt-Cloutier was nominated with former Vice President Al Gore for a Nobel Peace Prize for their work on climate change.


Well I guess if they can't defeat us militarily or economically they might as well get us for "crimes against humanity" for destroying the earth. I hear it gets upwards of 60 and occasionally 70 in the arctic interior - I'm sure that feels pretty warm to the Inuits. I even heard that Fort Yukon, Alaska - just north of the arctic circle - actually hit a high of 100°F once. On June 27, 1915.

This is sure to influence the Nobel committee to continue their trend of selecting ecologists and micro-credit pioneers instead of more traditional winners like terrorists. Why?

"This is clearly some of the most import conflict prevention work that is being done. Climate change could lead to enormous waves of refugees, the likes of which the world has never seen before," Heidi Soerensen, a Socialist Left MP who nominated Gore and Watt-Cloutier, told daily Aftenposten on Thursday.

"One hundred million climate refugees, major changes in drinking water supplies and a reduction in biological diversity ... will rapidly become a major security threat," co-nominator Boerge Brende, of the Conservative party, told the paper.


Climate Nuremberg, climate refugees, what's next?

tomder55 answered on 03/02/07:

Here is my answer and what will earn me a one star rating to the same question by Pericles on the Christianity board :

Between 1994 to 2004, carbon dioxide emissions from the U.S. increased by 12.9%, the same increase as that of France and a smaller increase than that of Canada, and China. U.S. emissions grew at a slower rate than the emissions from 143 of 213 countries (67%). The United States forests are carbon dioxide sinks, meaning they absorbs a significant amt of the carbo the US produces . For that matter ,so is the oceans of the world ("Picoplankton do some heavy lifting." Science, Vol. 315, Feb 9, 2007, pp. 777-778) Picoplanktons absorb and "sequester" huge amounts of carbon.

I think they should bring Al Gore up on charges . His electric bills resemble phone numbers (221,000 kilowatt hours last year in only One of his homes).Good thing he buys carbon offsets or we would really be in trouble !

ETWolverine rated this answer Excellent or Above Average Answer
Itsdb rated this answer Excellent or Above Average Answer

Question/Answer
Itsdb asked on 03/01/07 - Is there no shame?

I heard this PSA entitled The Gift (Windows Media File) on the radio at lunch. You hear these innocent little children saying things like...

    "I'm getting a catcher's mitt"

    "I'm getting ice skates"

    "I'm getting a jigsaw puzzle"


And then...

    "I'm getting dying coral reefs"

    "A blue bicycle"

    "A walkie-talkie"

    "I'm getting a severe drought"

    "A cool black skateboard"

    "I'm getting melted ice caps"
    "A killer heat wave"
    "A shrinking glacier"
    "I'm getting a devastating flood"


    Now wait for the voiceover...

      "Adults are generous. We're even giving kids global warming ... but it's not too late.


    Isn't that child exploitation? It isn't enough to hammer us with global warming stories every day? Do they have to take sweet sounding children, tease you with what sounds like a kid describing a birthday present and then hit you with "I'm getting dying coral reefs?"

    The media can't even tell the truth, they have to manipulate and scam - and then everyone wonders why some of us are skeptical. Take this for instance, the headline in the Daily Mail on February 1st, Global warming sees polar bears stranded on melting ice. They cling precariously to the top of what is left of the ice floe, their fragile grip the perfect symbol of the tragedy of global warming.



    A recap from a 2004 Woods Hole Oceanographic Institution expedition shows another picture of these bears with the caption, Mother polar bear and cub on interesting ice sculpture carved by waves.

    From "I'm getting a catcher's mitt" to "I'm getting a devastating flood," and from "Mother polar bear and cub on interesting ice sculpture carved by waves" to "Global warming sees polar bears stranded on melting ice." Is there no shame with these people?

    tomder55 answered on 03/02/07:

    this isn't new of course . there was that famous attack ad from LBJ in 1964 that had a nuke targetting a little girl picking flowers .

    The Vatican warns of exploiting children in advertising .


    Even today, some advertising is simply and deliberately untrue. Generally speaking, though, the problem of truth in advertising is somewhat more subtle: it is not that advertising says what is overtly false, but that it can distort the truth by implying things that are not so or withholding relevant facts. As Pope John Paul II points out, on both the individual and social levels, truth and freedom are inseparable; without truth as the basis, starting point and criterion of discernment, judgement, choice and action, there can be no authentic exercise of freedom. The Catechism of the Catholic Church, quoting the Second Vatican Council, insists that the content of communication be “true and — within the limits set by justice and charity — complete”; the content should, moreover, be communicated “honestly and properly.”

    To be sure, advertising, like other forms of expression, has its own conventions and forms of stylisation, and these must be taken into account when discussing truthfulness. People take for granted some rhetorical and symbolic exaggeration in advertising; within the limits of recognised and accepted practise, this can be allowable.

    But it is a fundamental principle that advertising may not deliberately seek to deceive, whether it does that by what it says, by what it implies, or by what it fails to say. The proper exercise of the right to information demands that the content of what is communicated be true and, within the limits set by justice and charity, complete. … Included here is the obligation to avoid any manipulation of truth for any reason.

    The Dignity of the Human Person

    There is an imperative requirement that advertising respect the human person, their right to make a responsible choice, their interior freedom; all these goods would be violated if man’s lower inclinations were to be exploited, or his capacity to reflect and decide compromised.

    These abuses are not merely hypothetical possibilities but realities in much advertising today. Advertising can violate the dignity of the human person both through its content — what is advertised, the manner in which it is advertised — and through the impact it seeks to make upon its audience. It can appeal to lust, vanity, envy and greed, and of techniques that manipulate and exploit human weakness. In such circumstances, advertisements readily become vehicles of a deformed outlook on life, on the family, on religion and on morality — an outlook that does not respect the true dignity and destiny of the human person.

    This problem is especially acute where particularly vulnerable groups or classes of persons are concerned: children and young people, the elderly, the poor, the culturally disadvantaged.

    Much advertising directed at children apparently tries to exploit their credulity and suggestibility, in the hope that they will put pressure on their parents to buy products of no real benefit to them. Advertising like this offends against the dignity and rights of both children and parents; it intrudes upon the parent-child relationship and seeks to manipulate it to its own base ends.

    Itsdb rated this answer Excellent or Above Average Answer
    labman rated this answer Excellent or Above Average Answer

Question/Answer
JacquelineA2006 asked on 03/01/07 - 2007.. part II


i was trying to post a follow-up question but it wasn't working on here for some reason. anyhow this is an example of one of the many responses i get from men on this topic..
----------
Whats wrong with a girl a having the car and offering to pick the guy the up?

It's a date and a 2 way thing...

If your both interested in each other meet half way... She drives the guy pays?

Next time it can be the other way around.
----------

Now, that is just stupid.

tomder55 answered on 03/02/07:

I would say that if a relationship is established then these deals could be made as the exception .

Maybe I have been married so long that I don't understand the dynamics but as long as I could get transportation I always picked up my dates at their homes ;showed up on time ,and when possible had some small conversation with her parents.I have worked since Middle school so I always had the money to pay.

As I got older if a date offered to pay ,or offered to pick me up, and we had been dating for a while ,then, as I said ,those exceptions were made. As a first time date ...never .

JacquelineA2006 rated this answer Excellent or Above Average Answer

Question/Answer
Itsdb asked on 03/01/07 - This and that...

NYC Urges People Not to Use N-Word

    Thursday March 1, 2007 4:46 AM
    By SARA KUGLER
    Associated Press Writer

    NEW YORK (AP) - New York declared the n-word off limits to all races Wednesday in a purely symbolic resolution prompted by the common, casual use of the slur in hip-hop music, comedy and street slang.

    ``People are using it out of context,'' said Leroy Comrie, a black city councilman who sponsored the unanimously passed measure. ``People are also denigrating themselves by using the word, and disrespecting their history.''

    New York's resolution is not binding and merely calls on residents to stop using the slur. Leaders of the nation's largest city also hope to set an example.

    Rudis Mata, 21, of New York said it was pointless to ban the word if city had no plans to enforce it, adding that he thinks it's a violation of free speech.

    ``I don't necessarily think people should ban the word, but it's a derogatory term and it shouldn't be used,'' he said. ``It's different from other curse words. It has a history.''

    Other municipalities have already passed similar measures in a debate that rose to a fever pitch late last year after ``Seinfeld'' actor Michael Richards spewed the word repeatedly at a comedy club in Los Angeles.

    Hip-hop pioneer Kurtis Blow Walker said when the resolution was proposed that blacks needed to stop using the word so ``we can elevate our minds to a better future.''

    Others argue that use of the word by blacks is empowering, that reclaiming a slur and giving it a new meaning takes away its punch. Oscar-winner Jamie Foxx, for example, said he would not stop using the word, and did not see anything inappropriate about blacks using it within their own circles.

    But in the uproar over Richards' outburst, black leaders including the Rev. Jesse Jackson and California Rep. Maxine Waters said it is impossible to paper over the epithet's origins and ugly history of humiliating blacks. They challenged the public and the entertainment industry to stop using the epithet.

    ``I forgive those young people who do not know their history, and I blame myself and my generation for not preparing you,'' Councilman Albert Vann said. ``But today we are going to know our history. We are not going to refer to ourselves by anything negative, the way the slave master referred to black people, using the n-word...''

    A group of black teenagers having lunch in a Dallas mall disagreed, saying the resolution won't affect their use of the word.

    ``We grew up saying it and it's what I say all the time,'' said 17-year-old Tiara Smith. ``It's not going to stop anybody from saying it.''


Everything goes back to Seinfeld. It took Kramer to get blacks (some anyway) to see it doesn't "empower" them?

'That's so gay': Is it OK?

    By LISA LEFF Associated Press Writer
    © 2007 The Associated Press

    SANTA ROSA, Calif. — When a few classmates razzed Rebekah Rice about her Mormon upbringing with questions such as, "Do you have 10 moms?" she shot back: "That's so gay."

    Those three words landed the high school freshman in the principal's office and resulted in a lawsuit that raises this question: When do playground insults used every day all over America cross the line into hate speech that must be stamped out?

    After Rice got a warning and a notation in her file, her parents sued, claiming officials at Santa Rosa's Maria Carillo High violated their daughter's First Amendment rights when they disciplined her for uttering a phrase "which enjoys widespread currency in youth culture," according to court documents.

    Testifying last week about the 2002 incident, Rice, now 18, said that when she uttered those words, she was not referring to anyone's sexual orientation. She said the phrase meant: "That's so stupid, that's so silly, that's so dumb."

    But school officials say they took a strict stand against the putdown after two boys were paid to beat up a gay student the year before.

    "The district has a statutory duty to protect gay students from harassment," the district's lawyers argued in a legal brief. "In furtherance of this goal, prohibition of the phrase `That's so gay' ... was a reasonable regulation..."

    Superior Court Judge Elaine Rushing plans to issue a ruling in the non-jury trial after final written arguments are submitted in April. Her gag order prevents the two sides from discussing the case.

    Derogatory terms for homosexuality have long been used as insults. But the landscape has become confusing in recent years as minority groups have tried to reclaim terms like "queer," "ghetto" and the n-word..."

    Eliza Byard, deputy executive director of the New York-based Gay, Lesbian and Straight Education Network, said nearly nine out of 10 gay students her organization surveyed in 2005 reported hearing "That's so gay" or "You're so gay" frequently.

    "It bothers them a lot," Byard said. "As odd or funny as the phrase sounds, imagine what it feels like to be in a setting where you consistently hear it used to describe something undesirable or stupid, and it also refers to you."

    She said it is OK to discipline students for using the phrase after efforts have been made to educate them.


Man is it ever confusing. All this "empowerment" by blacks "reclaiming" the "n-word" and "ghetto" and homosexuals reclaiming "queer," but nobody else can say it? Apparently there's no word yet on if the Santa Rosa school district considers asking a Mormon if she has 10 moms is offensive, hurtful or harrassing.

And finally, in al-AP's analysis of the decision by the Bush administration to join in talks between Iraq, Iran and Syria...

    Yet there were hints that the administration is testing Iranian, and domestic American, willingness to go further. In the diplomatic world, tough talk can sometimes be a cover for more conciliatory gestures.


Any of you recall the media, a Democrat or a liberal blogger say something like "tough talk can sometimes be a cover" whenever Bush has engaged in tough talk about Iran before? I can certainly recall many an instances of downplaying Ahmadinejad's "tough talk," but never Bush. Did they just figure this out or was it just never politically expedient to acknowledge sometimes the president has to talk tough until now?

tomder55 answered on 03/01/07:

Michael Richards is speechless.

I got a better idea ;let's ban symbolic laws and non-binding resolutions . It's amazing what passes for responsible governance these days .I wish I could symbolically pay my taxes .

I guess I'll now be able to walk the streets without hearing some boom box blasting gangsta rap . Good job city council !!!

Itsdb rated this answer Excellent or Above Average Answer

Question/Answer
tropicalstorm asked on 03/01/07 - whats is the sense?

On the radio they were talking about Germany is cutting back on I guess their equivalent to HUD Sect 8. They said everybody in bigger places had to move to smaller places. Then they ran out of smaller places. So they told everybody that didn't get to move they are not allowed to use some of the rooms in their house and they send the housing authority around to make sure their off limits rooms are kept empty.

tomder55 answered on 03/01/07:

boy is this stupid ! Gives new meaning to Lebensraum I guess.


The local housing authority in the eastern town of Loebau said on Friday the new regulations stipulate the tenants -- who all live on welfare -- now only qualify for smaller homes.

Because there is a shortage of smaller dwellings, the tenants are being allowed to stay, so long as the space they use does not exceed the new limit.

"The recipients are only allowed apartments of a certain size, but there aren't enough smaller apartments available," said Matthias Urbansky, head of the local housing authority.

"The people involved seem to be quite happy with the new set up," he said, noting that inspectors nevertheless make regular patrols to ensure the rooms standing empty are not being used.

Not everyone sees the sense of living in an apartment with off-limits areas.

"It feels stupid not being able to go into all the rooms of your apartment any more," one 49-year-old woman was quoted as saying in the Dresdner Morgenpost newspaper.


yeah stupid sums it up well . typical nanny state cradle to grave stupidity .They are paying brown shirts to inspect the empty spaces !! lolololololol "Achtung!! Getten sie houseplanten out of der dead spacen! Schnell!.....Erbarmlich, ich folgte nur Reihenfolgen "

Itsdb rated this answer Excellent or Above Average Answer
tropicalstorm rated this answer Excellent or Above Average Answer

Question/Answer
Itsdb asked on 02/28/07 - Fix the media?

The recent National Conference for Media Reform was held, where such notables as FCC Commissioner Michael J. Copps, Bill Moyers, and Jane "a truly powerful media is one that can stop a war, not start one" Fonda spoke.

Jane seems to have a different view of reality than I do...

    Eighty percent of expert guests on TV news shows come from far-right think tanks like the Leadership Institute. Knowing this, is it any surprise that it’s been so hard for us to frame the issues and set the terms of the debate?


I guess she's never seen Hardball, watched a Democratic party spokesman shout down a conservative, or read a paper lately. That type of nonsense was to be expected, but what got me is the NCMR's vision for a better media:

    We were reminded that media reform is crucial not just in creating better media but advancing every issue we care about: civil rights, education, the environment, the economy, health care, fair elections. Solving any of these problems will be impossible without fixing the media.


Is that your vision for a better media?

tomder55 answered on 03/01/07:

Obviously it was a loaded conference ;a typical moonbats on parade affair . Dennis Kucinich was there ;that about sums it up . It is his committee in the House that will soon hold hearings to reinstate the fairness doctrine.

Bottom line ;in a free market the liberals are having a difficult time competing in the arena of ideas so they are returning to the basics ;which is legislation restricting the 1st amendment . Kucinich' basic argument is a pile of sh*t..that the gvt. owns the airways so therefore it is right to make conditions on content. SCOTUS backed up that position in 1969

"A license permits broadcasting, but the licensee has no constitutional right to be the one who holds the license or to monopolize a... frequency to the exclusion of his fellow citizens. There is nothing in the First Amendment which prevents the Government from requiring a licensee to share his frequency with others.... It is the right of the viewers and listeners, not the right of the broadcasters, which is paramount."
Red Lion Broadcasting Co. v. FCC, 1969.

Before the Fairness Doctrine was repelled in 1987 there was no real conservative talk radio . There were some pioneers that had shows but they were the exception . Repealling the doctrine enabled Rush and other celebrity conservative hosts to do their shows without the rediculous requirement of being compelled to air alternate commentary . As a result there was a revival of AM radio that had taken a terrible beating when FM began to dominate in music radio.Lately the libs tried to get into the game with Airhead America .As I said ,they had real trouble competing in the market place. But ,they still control the major alphabet soup networks and publicly supported PBS .So their argument that their ideas aren't aired is a lie.

There MAY have been a case when there were only a few media outlets ,but with the explosion of cable and satellite options there are plenty of places where all views can be freely aired . The logic for the renewal of the Fairness Doctrine comes down to repression of certain speech .The result of a reinstituted fairness doctrine would not be fair at all. Controversial speech(read conservative speech ) would be stifled as the threat of random investigations and warnings discouraged broadcasters from airing what FCC bureaucrats claim is "unbalanced" views.Most stations rather than deal with that will just change format and eliminate political talk show formats . The bottom line would be censorship of ideas .

I have no doubt that if the Democrats retain the majority and gain the Presidency in 2008 that the doctrine will indeed be reinstated . That is what is at stake in 2008....the roll back of free speech.

Itsdb rated this answer Excellent or Above Average Answer

Question/Answer
Itsdb asked on 02/28/07 - You gotta love those moonbats

Several people have noted comments from yesterday's Huffington Post entry on the alleged "targeting" of Cheney (which at least have since been removed):

    "Cheney's spokeswoman said he was fine."

    "F**K" - by geoman77

    "So Cheney is personally responsible for the deaths of 14 innocent people...and then he waddles off to lunch!! What a piece of sh*t" - by fantanfanny

    "They missed! Too bad" - by DinahM

    "You can't kill pure evil. Like an exorcism you have to drive a stake through it." - by Caeser

    "If at first you don't succeed..." - by tented

    "Better luck next time." - by TDB

    "What a different world we would be living in today if they had succeeded." - by pakiman47

    "Dr Evil escapes again...damn" - by truthtopower01

    "Who'd have thought that Afghanistan would make such a valiant attempt to save the United States of America?" - by DumbFireman

    "Another 14 people dead, and their blood is literally on Dick's hands." - by micdago

    "All this naive talk about Cheney "dying" ... He's SATAN, for Christ's sake!! Even if the bomb did get him, he would just come back as another right wing war monger.." - by neoconcriminals

    "If Cheney was seized by the Iraqi government, tried, convicted, and hung by the neck until his freakin head popped off, then I wold say Iraq is making "remarkable progress." - by lornejl


I wonder if that's the kind of "meaningful dialogue" the left wants us to have with Iran?

tomder55 answered on 02/28/07:

Randi Rhodes said such clever things as :

"You can't kill the undead ...use garlic next time "

"don't they realize if they kill Cheney Bush becomes President ?"


Me ;I'm just wondering why the Pakistani ISI would target him ? My guess is that we will hear of a plane crash soon with some ISI officers on board ........oops

Itsdb rated this answer Excellent or Above Average Answer

Question/Answer
JacquelineA2006 asked on 02/28/07 - 2007..


This is not exactly political but still a good question to put up in my opinion. I would like to say i am 25 before continuing. You know it use to be like this.. the guy would pick the girl up and take her out and then take her back home after the date. Nowadays i run across men and these are all types that want you to drive out and meet them at the club you were planning to go dancing at or drive into the city and meet up somewhere and go from there. If a lady has to drive out and meet them then that is not a date! Why are these men like this..? Why do they not want to treat the opposite sex like a lady? I like your responses to the other topics we talk about and wanted to get your views on this.

tomder55 answered on 02/28/07:

They are losers ;ignore them .I'm old fashion in that regard I guess. Any guy who would not come to your house and pick you up,and pay for the date is not worthy of you . Keep high standards and expect your standards to be met .... Otherwise the man will ultimately disappoint you . I insist on meeting who my daughter dates . She's 20 now so it is becoming harder to enforce but my values are imbedded .

JacquelineA2006 rated this answer Excellent or Above Average Answer
_JacquelineA rated this answer Excellent or Above Average Answer

Question/Answer
rusty asked on 02/27/07 - The Final Inspection


The Marine stood and faced God,
Which must always come to pass.
He hoped his shoes were shining,
Just as brightly as his brass.

"Step forward now, Marine,
How shall I deal with you?
Have you always turned the other cheek?
To My Church have you been true?"

The soldier squared his shoulders and said,
"No, Lord, I guess I ain't.
Because those of us who carry guns,
Can't always be a saint.

I've had to work most Sundays,
And at times my talk was tough.
And sometimes I've been violent,
Because the world is awfully rough.

But, I never took a penny,
That wasn't mine to keep...
Though I worked a lot of overtime,
When the bills got just too steep.

And I never passed a cry for help,
Though at times I shook with fear.
And sometimes, God, forgive me,
I've wept unmanly tears.

I know I don't deserve a place,
Among the people here.
They never wanted me around,
Except to calm their fears.

If you've a place for me here, Lord,
It needn't be so grand.
I never expected or had too much,
But if you don't, I'll understand.

There was a silence all around the throne,
Where the saints had often trod.
As the Marine waited quietly,
For the judgment of his God.

"Step forward now, you Marine,
You've borne your burdens well.
Walk peacefully on Heaven's streets,
You've done your time in Hell."

~ Author Unknown ~

tomder55 answered on 02/27/07:

The poem was written by Sgt. Joshua Helterbran of the 224th Engineer Battalion. After he found it circulating around he wrote a part II which he did sign :

- Part II

I'm very saddened by America today,
when they take credit for what others say.

I wrote a poem because of problems in my past,
how was I to know that it was going to last.

It has been read by all and loved the same,
but indeed at the end there is no name.

The name is simple for those who know,
it's not Kilmer, Longfellow, Service, or Poe.

It's a soldier who has fought for his country so true,
He's proud of the ole Red, White and Blue.

You now know the poem the one and the same,
The Final Inspection is the name.

I wrote it because of the trials so true,
and of my buddies who died for country and you.

So take this poem, take it as you trod,
because in Heaven I'll see my God.

He will look at me and say don't be sad,
others read your poem and you made them glad.

Now step forward my son and look your best,
and come inside with all the rest.


by: Sgt Joshua Helterbran

ETWolverine rated this answer Excellent or Above Average Answer
rusty rated this answer Excellent or Above Average Answer

Question/Answer
rusty asked on 02/27/07 - Iraq


What are your opinions about the war in Iraq? Do you think that it should be wrapped up and the President should focus on our current problems over here?

tomder55 answered on 02/27/07:

I copied the relevent part of an email from my cousin serving in Iraq now :

"From our perspective we can't understand ;they confirm a
General who advocated the surge of troops and then want a resolution to
deny the general what he needs. Washington politics at its worst. I am
convinced now that if Congress did not cut off funds for the South
Vietnam Army, we could have had a different outcome.

Several things in the works as you can see from the news. Iraqis
really getting involved and know this is their last chance. You are
correct, our success/failure here will decide our civilization for at least
the next 50 years. We have a choice order or chaos. I much rather fight
here than on 5th AVE NYC or downtown Houston. Most folks don't think
in those extreme terms, but it is reality. "


I concure with him and the other experts who have also made this point.

I wish things would've gone smoother and easier but the enemy had a say in how the war has gone ..and yes ;as with all wars there have been mistakes in strategy as I was never shy in pointing out .

That has not changed the fact that removing Saddam Hussein was a necessary move in this greater war against jihadistan . I will not try to justify it here as I have done so often in other postings .

"It's the wrong war, in the wrong place, at the wrong time - but it's good that Saddam is gone." so say the Democrats .If they accept the premise that it was the wrong war then they also have to accept the premise that Saddam in possession of WMD was no big thing . That Saddam's material and other support to terrorist was acceptable . If it was ok for him ,a person who twice in the last 25 years attacked his neighbors unprovoked ,then by all means it should be ok for Iran and N.Korea to also obtain these weapons ,as well as Libya and other regimes who covet them .

Perhaps the results we face today were predictable ,perhaps not.Joe Lieberman pointed out however a very predictable outcome of a retreat now.

Many of the worst errors in Iraq arose precisely because the Bush administration best-cased what would happen after Saddam was overthrown. Now many opponents of the war are making the very same best-case mistake--assuming we can pull back in the midst of a critical battle with impunity, even arguing that our retreat will reduce the terrorism and sectarian violence in Iraq.

In fact, halting the current security operation at midpoint, as virtually all of the congressional proposals seek to do, would have devastating consequences. It would put thousands of American troops already deployed in the heart of Baghdad in even greater danger--forced to choose between trying to hold their position without the required reinforcements or, more likely, abandoning them outright. A precipitous pullout would leave a gaping security vacuum in its wake, which terrorists, insurgents, militias and Iran would rush to fill--probably resulting in a spiral of ethnic cleansing and slaughter on a scale as yet unseen in Iraq.

I appeal to my colleagues in Congress to step back and think carefully about what to do next. Instead of undermining Gen. Petraeus before he has been in Iraq for even a month, let us give him and his troops the time and support they need to succeed.


Since we are there it is our moral responsiblity to leave a stable situation before we go . Anything less will be the equivilent of GHW Bush inciting the Iraqis to revolt and then standing by as they were slaughtered .

Itsdb rated this answer Excellent or Above Average Answer
rusty rated this answer Excellent or Above Average Answer

Question/Answer
JacquelineA2006 asked on 02/27/07 - gun control


I could point you to another country that has a very low crime rate and 100 percent gun ownership: Switzerland, where gun ownership is required by law and everybody has a military grade weapon in their house.

In Rwanda, the 1994 genocide was carried out largely by way of machete, not guns.

It's worth noting that in the USA, crime rates are trending down over the past 15 years, even while it becomes increasingly easier to not only obtain guns, but to get concealed carry permits, in most states over that same time period.

There's evidence to support both sides, which must force us to the conclusion that the fundamental character of the society is more important than the ease of obtaining guns. Not that that should come as any big surprise, though I'm sure to some people it will anyways. :-p

tomder55 answered on 02/27/07:


From someone who is not a gun owner ....

An armed populace is a respectful one .

JacquelineA2006 rated this answer Excellent or Above Average Answer
tropicalstorm rated this answer Excellent or Above Average Answer

Question/Answer
paraclete asked on 02/25/07 - May God curse the terrorists?

We could hope the Muslims are at last getting the message, Islamic terrorism is a curse and it is the Muslims who are cursed.

Bomber slaughters 40 in attack at business collegeFrom correspondents in Baghdad
February 26, 2007 05:04am

A SUICIDE bomber wearing a vest packed with explosives killed 40 people in a Baghdad college overnight, a day after Prime Minister Nuri al-Maliki expressed optimism about a security crackdown in the capital.

Guards stopped the bomber in the reception lobby of the Baghdad Economy and Administration College but the man managed to blow himself up, police said.

Police put the death toll at 40, with 35 people wounded.

Most of the victims were students, witnesses said.

"May God curse the terrorists," shouted some students after the attack. Others sat on the ground outside weeping.

A string of car bombings and rocket salvos also hit Baghdad overnight as insurgents defied efforts by US and Iraqi security forces to stabilise the capital.

A professor said the college attack happened as students were leaving morning classes and arriving for afternoon lessons.

Others doing exams were wounded by flying glass that tore through their classroom, the professor said.

"There were bodies everywhere," said the professor, who declined to be identified.

The blast left large pools of blood in the college's reception area. Textbooks and pens lay scattered on the floor.

The college is part of nearby Mustansiriya University, which was hit by twin bomb attacks last month that killed 70 people, mainly students.

Insurgents have repeatedly attacked universities and colleges in Baghdad, trying to strike fear into the city's middle class.

Many college professors and intellectuals have also been killed.

Mr Maliki expressed optimism yesterday about the 10-day-old security plan, regarded as a last chance to reverse Iraq's descent into civil war, and said US and Iraqi forces had killed about 400 suspected militants since it began.

US military officers have said they expected an increase in the use of suicide vests after security forces set up more checkpoints on Baghdad's roads to search vehicles and try to prevent car bombs.

Among the attacks overnight, rockets and mortar bombs crashed into a market in a Shiite area in southern Baghdad and there were conflicting reports about casualties, police said.

One police source said 10 people were killed in the attack in the Abu Dsher area of Doura neighbourhood. Two other police sources said no more than three people had been wounded.

A car bomb also killed one person and wounded four in central Baghdad, not far from the Iranian embassy, police said.

Police said the diplomatic mission did not appear to have been the target. The embassy compound was not damaged.

US forces have set up joint security outposts with Iraqi forces around the city and the crackdown does appear to have reduced the number of bodies found tortured and shot in the city, the apparent victims of death squads.

A typical daily body count had been about 40 or 50 a day in recent months but since the start of the plan it has been between five and 20. However, US commanders say it will take months to judge the success of the offensive.

A fuel tanker rigged with explosives killed 45 people yesterday when it blew up near a Sunni mosque in restive western province of Anbar, after the mosque's imam had criticised al Qaeda militants at Friday prayers, police and residents said.

US President George W. Bush is sending 21,500 extra troops to Iraq to help with the clampdown in Baghdad. Most are heading for the capital although 4000 will be sent to Anbar, the most dangerous province in Iraq for American forces.
++++++++++++++++++++++++++++++++++++++++++++++++++++++
all of this leads me to believe the fight is not being won

tomder55 answered on 02/26/07:

this war is being fought world wide . Why don't you report about the insurgency in Thailand that has claimed over 2000 lives in the last couple of years that the Thai gvt admits they are losing ?

No one has found a way to attack terrorist havens without crossing borders. Against these threats in Iraq,Iran,Waziristan ,Afghanistan ,Indonesia , Sudan ,elsehwere how far will diplomacy and engagement go against shutting down the jihadi breeding sites? I am waiting for anyone to come up with a better plan than President Bush and all I hear is retreat and concede territory to jihadistan .

Itsdb rated this answer Excellent or Above Average Answer
paraclete rated this answer Excellent or Above Average Answer

Question/Answer
paraclete asked on 02/24/07 - Is Japanese militarism on the rise again?

Let's face it folks to we have more to fear from a resurgent Japan than from Iran or North Korea. What purpose can Japan have for spying on the rest of the world?


Japan completes set of spy satellites

From correspondents in Tokyo

February 24, 2007 06:31pm


JAPAN launched its fourth spy satellite today, improving its ability to monitor potential threats including North Korea, whose missile and nuclear tests have spooked the region.

An H-2A rocket, delayed three times by bad weather, finally lifted off from the southern island of Tanegashima, carrying a radar satellite that will join two optical satellites and another radar satellite already in operation.

With the full complement of four satellites, Japan will be able to monitor any point on Earth once a day, government officials have said.

Japan's spy satellite program was initiated after North Korea launched a ballistic missile in 1998 that flew over Japan.

The program was delayed in 2003 when a rocket carrying two satellites veered off course and had to be destroyed in a spectacular fireball.

North Korea ratcheted up regional tensions last year when it conducted a nuclear test in October after a salvo of missile tests in July.

In January, China destroyed one of its own satellites by firing a ballistic missile at it, in an experiment that sparked criticism around the world.

Japan's space scientists have long complained that the country's technical prowess has fallen behind because of a 1969 parliamentary resolution limiting the use of space to peaceful purposes.

Prime Minister Shinzo Abe's ruling Liberal Democratic Party is likely to soon submit a bill that would ease regulations and allow non-aggressive military use of space.

The rocket launched today was also carrying an experimental optical satellite, aimed at improving the level of detail obtained from the next generation of satellites.

At present, Japan's satellites can distinguish objects a metre or more in diameter, whereas US military satellites are said to be able to do so for items one-tenth as large.






Copyright 2007 News Limited. All times AEDT (GMT +11).

tomder55 answered on 02/24/07:

let me get this straight . Japan decides to put eyes in the sky after almost a decade of provovative and deliberate intimidation;including twice flying birds over the nation capable of nuclear tips without notifying them of the test or trajectory ......but Japan is being militaristic .


Been tipping that VB again the weekend ?



I would have to say the big loser on the recent 6 party framework of an agreement regarding the NORKS is probably Japan . Even though the Japanese pointed out how often the NORKS have come to one of the Japanese isles and abducted it's citizens ;the wording of the framework says that the US would begin the process of changing North Korea's status as a terrorist sponsor of terrorism .

labman rated this answer Excellent or Above Average Answer
paraclete rated this answer Poor or Incomplete Answer

Question/Answer
Itsdb asked on 02/22/07 - What is an attack?

White House Stands Behind Cheney’s Attacks On Murtha And Pelosi

Yesterday, Vice President Cheney attacked House Speaker Nancy Pelosi (D-CA) and Rep. John Murtha (D-PA) for supporting Iraq redeployment:

    CHENEY: I think, in fact, if we were to do what Speaker Pelosi and Congressman Murtha are suggesting, all we’ll do is validate the al Qaeda strategy. … I think that’s exactly the wrong course to go on. I think that’s the course of action that Speaker Pelosi and Jack Murtha support. I think it would be a huge mistake for the country.

    Q Is that policy that we hear from the Speaker of the House, Nancy Pelosi — from other Democrats, is that a policy of defeat?

    CHENEY: Yes.


CBS reports Pelosi's response thusly:

    Vice President Dick Cheney on Wednesday harshly criticized Democrats' attempts to thwart President Bush's troop buildup in Iraq, saying their approach would "validate the al Qaeda strategy." House Speaker Nancy Pelosi fired back that Cheney was questioning critics' patriotism.

    "I hope the president will repudiate and distance himself from the vice president's remarks," Pelosi said.


Pelosi went further, "You cannot say as the president of the United States, 'I welcome disagreement in a time of war,' and then have the vice president of the United States go out of the country and mischaracterize a position of the speaker of the House and in a manner that says that person in that position of authority is acting against the national security of our country."

Try as I might I can't quite figure out the left. Either that or I know them better than they know themselves. In this case I'd have to say they're way too sensitive (which would explain all their sensitivity issues), yet they're so hypocritical and condescending at the same time. They've demanded the "Bush regime" come clean for years and yet they have a serious aversion to the truth.

Seeing as how Pelosi and Murtha have made their intentions 'vaguely clear' as we've already demonstrated on this board, Cheney was simply telling the truth. So, naturally that offends Madame Armani Speaker.

Contrast Cheney's 'attack' with Bill Maher on Leno Tuesday:

    “Joe Scarborough did a whole week of panel discussions on whether he was an idiot. The people who were defending him were saying, 'well, he's just inarticulate.' But inarticulate doesn't explain foreign policy. I mean, it's not that complicated. The man is a rube. He is a dolt. He is a yokel on the world stage. He is a Gilligan who cannot find his ass with two hands. He is a vain half-wit who interrupts one incoherent sentence with another incoherent sentence.


Now THAT's an attack.

Steve

tomder55 answered on 02/24/07:

Itsdb rated this answer Excellent or Above Average Answer

Question/Answer
HANK1 asked on 02/22/07 - IRAN:



I just heard that Iran is going to expand its nuclear interests. Today is the deadline for them to end the testing et al. They didn't. Look out now!

HANK

tomder55 answered on 02/24/07:

Iran knows with Russia and China's help no meaningful sanctions will come out of the UN .So again the only question is how far will the US go .

Iran is betting that there is enough political opposition in the US to prevent any meaningful action on our part .


It's fleet week in the Persian Gulf


Israel of curse has a bigger national interest in the game Iran is playing .the Telegraph reported that Israel is seeking flyover Iraq rights but I think it is a deception. The Israelis would not reveal it if it were so and I think that the US would not want to deal with the reaction to an Israeli strike through US controlled sky.But it does make for good Sabre rattling and may partly explain the NY Slimes article that Steve cited .


HANK1 rated this answer Excellent or Above Average Answer

Question/Answer
rusty asked on 02/21/07 - hillary clinton


If by "the rest of the world" you are in fact referring only to the United Kingdom, then you would be right. Otherwise horrendously wrong. Stop watching Fox News for your daily news. It's not news. It's right-wing jingoistic tripe.

Thinking back on the whole build-up to the Iraq war, I can recall an overwhelming sense that the entire case being built was horsesh*t. And most other people that I know felt the same way. Indeed I recall news stations/papers reporting what Bush was upto, followed by the whole thing over Hans Blix being told to f*ck off with his UN inspectors in case they actually *didn't* find something. Which kind of underscores the hidden agenda that the argument of WMD was a lie, and that you, the American people, and Congress were lied to, and that American soldiers are now dying for a lie.

Quite frankly, I couldn't care who the last president was. It's the *current* President who started the current fiasco, and it's at his feet that the blame squarely lies. It's the *current* President who has led American kids to their deaths for a lie.

But since you want to get into obession about Iraq, I've seen footage of a meeting where Donald Rumsefeld sold weapons to Iraq during the Reagan administration. This footage included Rumsfeld shaking hands with .. shock horror ... Saddam Hussein.

The obsession with Iraq began with the Iranian revolution in the late 1970s.

tomder55 answered on 02/22/07:

since the question was a bout Hillary the topic of the previous adm.was relevent.

If you had read previous posts and snswers of mine you would know I was critical of the US realpolitk policies of previous administrations . It was immoral to cozy up to brutal homicidal dictators for the sake of temporary stability . My guess is that you approved the status quo .

There was no debate before the war that argued about the intel. The only question was if the US should wait longer for Saddam to comply with UN resolutions and the terms of the 1991 cease fire...both of which he was in violation. Hans Blix just a few months before OIF began gave a laundry list of examples where Saddam was non-compliant.

We now know that much of the opposition came from the same folks who were getting the biggest kick-back in the oil for food briberies. Of course they had a stake in the status quo .

rusty rated this answer Excellent or Above Average Answer

Question/Answer
ETWolverine asked on 02/22/07 - State of the US Economy

This article is from The Christian Science Monitor
February 21, 2007 edition -

My comments are in Italics.

http://www.csmonitor.com/2007/0221/p01s03-usec.html

    US deficit is shrinking, for now
    With the robust economy, tax revenues are pouring in. But rising costs lie ahead.
    By Mark Trumbull | Staff writer of The Christian Science Monitor

    Despite the ongoing costs of US military campaigns in Iraq and Afghanistan,


And exactly why was it necessary for them to cite the war in Iraq and Afghanistan in the opening sentence of an article about national debt? Seems like the CSM is trying to take potshots at Bush, even as they report on something good taking place.

    the outlook for the federal budget has grown substantially brighter.

    Tax revenues are rising much faster than spending,


Remember this sentence. I'll be referencing it later

    according to Treasury Department numbers released last week. The recent trend is strong enough that, were it to continue, the budget could move into surplus in barely a year, one economist calculates.


One year, huh? That would be about 5 years ahead of the best predictions of most economists two years ago... and MUCH better than the predictions of the anti-Bush anti-tax-cut doomsayers.

    Already, the federal deficit is shrinking toward about half the size that it has averaged since 1970, when analyzed as a percentage of gross domestic product.

    The shift reflects a strong economy, with higher incomes and corporate profits generating a bigger flow of tax revenue.


Again, notie the discussion of HIGHER TAX REVENUE. This is an important point that I will come back to later.

    In turn, the Treasury's progress could help the economy by buoying investor confidence in the nation's fiscal position.

    Although it is a welcome change, the improvement does little to stave off the long-run challenges to the nation's financial health, many economists say. Baby boomers are starting to retire, placing new demands on government. Costs for healthcare programs like Medicare are still projected to rise faster than overall inflation.


Isn't that the perfect argument for personal healthcare accounts? I would think so... but it seems that others think it means we need to be pouring more money into a failing (faile?) system that can't be fixed.

    "The picture is getting brighter," and if there's no recession over the next several years "there are going to continue to be some good strides made," says Mark McMullen, a senior economist at Moody's Economy.com in West Chester, Pa. But "it's unlikely that we're going to see a balanced budget anytime in the near or long term."


Why not? If the deficit is decreasing to the point where we will see the deficit disappear within the next couple of years, why would we not see a balanced budget? If tax revenues remain strong and continue to grow, the result SHOULD be a balanced budget. The only thing that could keep the budget from balancing would be spending increases. So is someone planning to say "We have too much money coming in, we should spend more"? Apparently so. The problem isn't tax revenue, its government expenditures.

    Some experts say the budget could achieve balance in the short run of the next few years. In unveiling its proposed budget this month, the Bush administration forecast black ink on the federal ledger in 2012. The nonpartisan Congressional Budget Office (CBO), in its recent annual outlook, also shows a surplus for that year.

    A year ago, the CBO's forecast for the 2007 fiscal year called for a deficit of $270 billion. In the annual outlook released last month, the 2007 gap is projected at $172 billion.

    "Right now, we're in some sense in a relatively good spot," says Jim Horney, a budget analyst at the Center for Budget and Policy Priorities, a liberal think tank in Washington. "We're in the sixth year of an economic expansion," a time when federal revenues often rise along with a growing economy.

    But both the CBO and the White House make important assumptions that are far from assured.


And what makes them unassured? They seem to have been born out until now, despite the doomsayers' negative predictions.

    The CBO's annual outlook assumes that President Bush's tax cuts phase out in 2010 as scheduled, thus adding new tax revenues.


Remember what I mentioned above about higher tax revenues NOW? Why would anyone predict that higher taxes would increase tax revenues when history has clearly shown the opposite to be true? As is the present case. Bush's tax cuts have resulted in higher tax revenue for the government. Why would phasing them out in 2010 increase tax revenue? If you do the opposite of what you are doing now, wouldn't you logically get the opposite result of what you are getting now?

    Mr. Bush's budget calls for the tax cuts to be made permanent, but foresees a surplus in 2012 thanks to a sharp fall in Iraq spending and robust productivity growth in the economy.


Why is this an unrealistic assumption? Why should we believe that the war in Iraq will continue to cost as much as it has, and that we won't see any economic benefit from Iraq? And why wouldn't there be robust productivity growth? Is the CSM aware of some economic indicators of which nobody else knows?

    But several issues are unsettled. Among them: How much will military operations in Iraq and elsewhere cost?


It doesn't really matter. If we are receiving more tax revenue, we can handle more expenditures for the war.

    Will Congress make some of the Bush tax cuts permanent?


That is a key question. I think we can be certain that the current Congress WON'T make them permanent. But after 2008? Who knows.

    Will Congress scale back the alternative minimum tax (AMT), which is poised to take a rising tax toll on middle-class Americans in the years ahead?


Another good question.

    The answers will have a big impact on the budget, and may not be resolved before a new president takes office in 2009.


But since Bush will be in office until then, and since he will keep his tax cuts in place until then, we can expect his economic growth projections to be met or exceeded until then.

    The long-term outlook remains sobering, all sides agree. The cost of Medicare, in particular, is slated to soar due to healthcare inflation and an aging population.


Again, can you say Personal Healthcare Accounts?

    Even the near-term outlook comes with an asterisk. When Bush took office in 2001, the CBO was forecasting a decade of budget surpluses totaling more than $5 trillion. Then came a recession, the terrorist attacks of 9/11, and enormous wartime spending. The Bush tax cuts helped to stimulate the economy, but at the cost of lower tax revenue.


Ahem.... remember the sentence that I told you to keep in mind above? "Tax revenues are rising much faster than spending..." Is it just me, or did this article just contradict itself badly? On one hand it says that tax revenues are rising faster than spending, on the other hand it says that tax revenues decreased as a result of the Bush tax cuts. Well, which is it?

    "We had three years where revenues went down," says Mr. Horney. "All that has happened is that we have ... caught up from the really bad decline that we had."


Actually, we far exceeded what tax revenues were before 9/11 and the recession. Horney doesn't know what he's talking about.

    Still, analysts say the recent budget gains are good news for the government and the economy.

    The budget deficit now stands at about 1.4 percent of the nation's GDP, well below the 2.3 percent that's been the norm since 1970, according to economist Michael Darda of MKM Partners in Greenwich, Conn. "At the current pace, the budget could move back into surplus as early as May 2008," Mr. Darda wrote in a report to clients last week.

    That isn't a forecast, but it shows how the nation's fiscal health is closely related to that of the overall economy.


Duh... ya think?

    A more stable budget outlook, in turn, has benefits for the economy.


No $h!t. I never would have guessed.

    The less money the government has to borrow to pay its bills, the more is left for investment in new goods and services. Alternatively, the nation will be less reliant on foreign lenders to fund that investment – debt that siphons away a portion of national wealth.

    "Unexpectedly strong revenue growth" has improved the outlook quite a bit, says Mr. McMullen.

    In the CBO projections, for example, the nation's public debt is forecast to fall from 37 percent of GDP in 2006 to 30.5 percent of GDP in 2012.

    In the longer run, the rise of entitlements such as Medicare could force difficult choices to keep that debt from rising again.


PERSONAL HEALTHCARE ACCOUNTS!!! How many times do I have to say it?

    Conservatives say it will be vital to contain costs. "If nothing changes in Washington then both revenues and spending will be higher," says Chris Edwards, a tax expert at the libertarian Cato Institute in Washington. "It'll hammer the economy," he says, as government takes a larger share of GDP.


Definitely true. While I love the Bush tax cuts, I am pissed at his lack of use of the veto pen to kill pork. And I'm pissed at the former Congress' Republican leadership for not holding spending down. We definitely need a change in Washington.

    Others say the answer will probably involve tax hikes as well as some reductions in promised entitlement benefits – and that a modest increase in taxes need not damage economic growth.


I don't really have a problem with the decreases in benefits. But if the Bush tax cuts have resulted in higher tax revenues, why would you then raise taxes and reverse that trend?

    Both sides agree on the need to tame medical inflation, if not on how to do it.


Have I mentioned Personal Healthcare Accounts? Cut the insurance companies out of the loop, and healthcare costs for the patient will drop, without decreasing payment to the doctors. We cut costs by cutting out the middleman... like Geico."

    "If we were able to reduce the growth of the cost of healthcare," says Horney, "that would definitely be good for the economy."


Again, a comment that falls into the "no $h!t" category.

----------------------

Good news, badly reported.

Hey maybe that should be the Christian Science Monitor's new slogan.

Elliot

tomder55 answered on 02/22/07:

"Give me a one-handed economist," is what President Harry Truman used to say, when his economic advisers' would say "On the one hand, Mr. President..... but on the other hand."

So the economy is booming and the deficit is reducing in spite of us being in a global war against jihadistan . I say it's Bush's fault ! Damn you George W. Bush!

When state budget surpluses are taken into account, the current US government deficit is closer to about 1% of GDP. This consolidate figure is a better measure of the countries fiscal health and is more accurate in terms of comparing to other countries.

If the GOP had any spine after the 2004 elections they could've forced through some meaninful reform in Social Security and the way we fund health care .AND Just imagine the progress that would've been had they acted just a little bit like fiscal conservatives and capped some of the excess spending and growth of government.

The twin issues of S.S. and medical entitlements will be an increasing burden on the budget in the years ahead and I am concerned that we will have a Democrat solution imposed .I consider the last 2 years a wasted opportunity in that regard.

Now that the Dems. control the purse strings the danger is that as the budget gets closer to being balance, Congress will have the urge to spend even more money. .......oh wait ....the dems are in control again ...that explains everything ....this must be the Clinton boom re:emerging .

ETWolverine rated this answer Excellent or Above Average Answer
Itsdb rated this answer Excellent or Above Average Answer
rusty rated this answer Excellent or Above Average Answer

Question/Answer
ETWolverine asked on 02/21/07 - Are Americans finally waking up?

From today's NY Post.

AMERICA SAYS LET'S WIN WAR
By ANDY SOLTIS


February 21, 2007 -- In a dramatic finding, a new poll shows a solid majority of Americans still wants to win the war in Iraq - and keep U.S. troops there until the Baghdad government can take over.

Strong majorities also say victory is vital to the War on Terror and that Americans should support President Bush even if they have concerns about the way the war is being handled, according to the survey conducted by Public Opinion Strategies.

The poll found that 57 percent of Americans supported "finishing the job in Iraq" - keeping U.S. troops there until the Iraqis can provide security on their own. Forty-one percent disagreed.

By 53 percent to 43 percent they also believe victory in Iraq over the insurgents is still possible.

Despite last November's electoral victories by anti-war Democrats, the survey found little support among voters for a quick pullout of U.S. forces.

Only 25 percent of those surveyed agreed with the statement, "I don't really care what happens in Iraq after the U.S. leaves, I just want the troops brought home." Seventy-four percent disagreed.

The survey was conducted before last week's House of Representatives resolution repudiating Bush's war policies.

But by 53 percent to 46 percent, Americans said Democrats are going too far, too fast in demanding troop withdrawals.

But the poll of 800 registered voters found Americans pessimistic about Iraq's fledgling democracy.

Only 34 percent felt it would become a stable democracy, compared with 60 percent who said it would not.

Among other key findings of the poll conducted Feb. 5-7:

* When given a choice of four policies, an immediate withdrawal of U.S. troops was the least popular (17 percent).

* The most popular option (32 percent) was a withdrawal timetable.

* The next most popular policy, favored by 27 percent, was expressed by the statement: "The Iraq war is the front line in the battle against terrorism and our troops should stay there and do whatever it takes to restore order until the Iraqis can govern and provide security to the country."

The fourth option, favored by 23 percent, was the statement: "While I don't agree that the U.S. should be in the war, our troops should stay there and do whatever it takes to restore order until the Iraqis can govern and restore security to their country."

With Post Wire Services

andy.soltis@nypost.com


-----------------------

Interesting, no? Americans are a lot smarter about our options in Iraq than the Dems give them credit for. 50% of Americans expect and WANT the troops to stay in Iraq for as long as it takes to get the job done and support strategies that include a long-term deployment in Iraq.. Fully 82% of Americans are against an immediate pullout option, and 50% are against ANY option that includes a pullout until the job is over. And 57% of Americans say that we should support Bush on Iraq even if we don't agree with every aspect of how the war is run. These are SOLID numbers that favor Bush's policies.

It would seem that Murtha, Pelosi, Kennedy and Kerry's strategy of demanding immediate withdrawal at all costs is a losing stance for the Dems. I expect Clinton to gravitate towards the idea of a "timetable" as time goes along. Obama is in a bad spot... the darling of the far left, he can't easily shift to the right without losing some of his constituency.

I expect an all-out scramble within the Dem hierarchy to reposition themselves in lieu of this poll.

Are any other news outlets reporting this poll?

What is your opinion of the poll?

Elliot

tomder55 answered on 02/21/07:

I am beginning to believe that the election was more an issue of Republicans sitting it out then the American people looking for "dramatic" changes .

Then again ....the trojan horse nature of the democrat campaign left alot of the electorate in the dark over which course the democrats would persue and how aggressively they would persue it. The Dems. would be wise I believe to shelve Murtha and shmucky Shumer and move a little more cautiously .

This week began the Chinese New Year and the Vietnam TET period. It is the week in history that Walter Cronkite decided ,despite what historians universally agree was a huge victory for American troops ,that the Vietnam was lost.

This weekend and last week the Republicans stood damn near unanimous in their support of the President's policies (forget the few defectors in both houses ) .The Republicans at least have heard what Americans are saying .

Shumer after the vote on Saturday declared “There will be resolution after resolution, amendment after amendment . . . just like in the days of Vietnam,” .....“The pressure will mount, the president will find he has no strategy, he will have to change his strategy and the vast majority of our troops will be taken out of harm’s way and come home.”


The ‘moderate’ Democrats who won the 2006 midterms are nowhere in sight. The soul searching question they have to ask is what is their stake in America's defeat in Iraq ?
Selling out in Vietnam cost the Democrats political viability for a decade or more .Still today the question can easily be asked :can they be trusted to lead American foreign policy ? They may get some quick satisfaction in defeating President Bush but in the long run it is a losing strategy for them.

ETWolverine rated this answer Excellent or Above Average Answer

Question/Answer
paraclete asked on 02/20/07 - The time has come, the Walrus said

Blair to announce Iraq troop withdrawal

February 21, 2007 - 9:24
British Prime Minister Tony Blair will announce a timetable for withdrawing British troops from Iraq, with 1,500 to leave in several weeks, the BBC reports.

(More to come.)

[img s http://www.cartoonstock.com/lowres/vsh0107l.jpg]

tomder55 answered on 02/21/07:

yup I agree .it's time to go . the US should follow the British lead. We have done all we can for these people and still the violence continues at an unprecedented rate of 117.5 per 100K .There are near daily killings and the demolished surroundings will likely never be repaired. The police are not helping and the bad guys seem to be running the place.And there is no end in sight.
We ought to pull out and cut off all aide. Our money is just being wasted there .The American public does not like the idea of its citizens being murdered while trying to help a group of people who do not appreciate the effort.

So yes ;I say pull out of New Orleans immediately !!!!

ETWolverine rated this answer Excellent or Above Average Answer
paraclete rated this answer Excellent or Above Average Answer

Question/Answer
Itsdb asked on 02/20/07 - Let the whining begin

Court: No habeas for Guantánamo captives
BY CAROL ROSENBERG
crosenberg@MiamiHerald.com

The federal appeals panel in Washington, D.C., sided 2-1 with the Bush administration today, upholding an act of Congress that stripped Guantánamo Bay captives of the right to challenge their detention in lower federal courts.

The U.S. Court of Appeals for the District of Columbia handed the White House a key victory as it moves forward with plans to hold war-crimes trials for at least three Guantánamo captives at the remote Navy base in southeast Cuba.

It also sets the stage for an early decision on whether to intervene by the U.S. Supreme Court, which has twice before sided with the detainees.

Currently, three captives who have never been charged with crimes -- a Yemeni, a Pakistani and a Chinese citizen of the Uighur minority -- are asking the justices to consider their unlawful detention lawsuits.

''Federal courts have no jurisdiction in these cases,'' declared Judge A. Raymond Randolph for himself and Judge David B. Sentelle. Two successive acts of Congress, they said, had sufficiently stripped detainees of traditional recourse to the writ of habeas corpus.

''The arguments are creative but not cogent. To accept them would be to defy the will of Congress,'' Randolph wrote for the two men, who were appointed to the court by Presidents Ronald Reagan and George H.W. Bush.

The Republican-led Congress twice passed legislation that removed so-called ''enemy combatants'' of their right to challenge their detention without charge in U.S. civilian courts -- the Detainee Treatment Act of 2005 and the Military Commissions Act of 2006...

Judge Judith W. Rogers, a Clinton appointee, dissented, saying the Pentagon had failed to create a fair substitute outside the federal courts at which captives held without charge can challenge their detention.

''While judgments of military necessity are entitled to deference by the courts and while temporary custody during wartime may be justified in order properly to process those who have been captured,'' she said, ``the executive has had ample opportunity during the past five years during which the detainees have been held at Guantánamo Bay to determine who is being held and for what reason.''

The court ruled even as members of the Democratic-led Congress are crafting new detainee legislation to restore the civilian courts' jurisdiction in such cases and to more narrowly define an enemy combatant.

In New York, the Center for Constitutional Rights, which filed suits on behalf of many of the prisoners, issued a condemnation of the ruling.

''This decision empowers the president to do whatever he wishes to prisoners without any legal limitation as long as he does it off shore,'' said Shayana Kadidal, managing attorney of the Center for Constitutional Rights Guantánamo Global Justice Initiative.

Added CCR executive director Vincent Warren: ``Habeas corpus is a right that was enshrined in the Magna Carta to prevent kings from indefinitely and arbitrarily detaining anyone they chose. The combined actions of the Bush administration, the previous Congress and two of the three judges today have taken us back 900 years and granted the right of kings to the president.''
~~~~~~~~~~~~~~~~~~~~~~~~~~~~~~~~~~~~~~~~~~~~~~~~~~~~~~

Yeah well, don't fret Mr. Warren ... I'm sure you can count on this congress to continue undermining the president and siding with terrorists.

tomder55 answered on 02/20/07:

Why do I suspect people accused of killing our soldiers can't use US courts to appeal only bothers the left? But they would all be jubilant if Libby is found guilty.

Arlen Specter must be having a canary .

Fahd al-Utaibi aka Naif Fahd Al Aseemi Al Utaibi was released from Gitmo May 2006 along with 14 others. He is currently on trial in Yemen for forging travel documents in order to join the jihad in Iraq.

But he's not alone :

Mullah Abdul Ghafar a former detainee was killed in battle in Afghanistan

Maulvi Abdul Ghaffar captured in Afghanistan in December 2001, was one of the twenty-three prisoners released from Camp Delta in January 2004. After his release, he joined the remnants of the Taliban and was killed in a gunfight on September 26, 2004.

Abdullah Mehsud , also captured in Afghanistan masterminded the kidnapping of two Chinese engineers in Pakistan's South Waziristan region as well as returning to his position as an Al-Qaeda field commander. Mehsud has also claimed responsibility for the bombing at Islamabad's Marriott Hotel in October 2004. The blast injured seven people, including a U.S. diplomat, two Italians and the Pakistani prime minister's chief security officer.

Airat Vakhitov and Rustam Akhmyarov , two Russian nationals captured in Afghanistan in December 2001 and released from Guantánamo in late 2002, were arrested by Russian authorities on August 30, 2005. The two former detainees were arrested in Moscow for allegedly preparing a series of attacks in Russia. According to authorities, Vakhitov was using a local human rights group as cover for his activities.

Slimane Hadj Abderrahmane
After his release Abderrahmane announced plans to travel to fight in Chechnya. He said he regarded the document he signed promising not to take part in terrorist activity as "toilet paper". Danish security officials talked him out his trip.He has said that Denmark's role in the Occupation of Iraq meant that he thought Danish leaders were legitimate military targets.

Itsdb rated this answer Excellent or Above Average Answer

Question/Answer
tropicalstorm asked on 02/20/07 - Do you really know Hillary?

"Where is the G-ddamn f**king flag? I want the G-ddamn f**king flag up every f**king morning at f**king sunrise."
(From the book "Inside The White House" by Ronald Kessler, p. 244 - Hillary to the staff at the Arkansas Governor's mansion on Labor Day, 1991)

"You sold out, you motherf**ker! You sold out."
From the book "Inside" by Joseph Califano, p. 213 - Hillary yelling at Democrat lawyer.

"It's been said, and I think it's accurate, that my husband was obsessed by terrorism in general and al-Qaida in particular."
Hillary telling a post-9/11 world what a great commander in chief her husband was; Dateline, NBC 4/16/2004.

"I have to admit that a good deal of what my husband and I have learned [about Islam] has come from our daughter."
(TruthInMedia.org 8/8/1999 - Hillary at a White House function, proudly tells some Muslim groups she is gaining a greater appreciation of Islam because Chelsea was then taking a class on the "religion of peace")

"F**k off! It's enough that I have to see you shit-kickers every day, I'm not going to talk to you too. Just do your G-ddamn job and keep your mouth shut."
(From the book "American Evita" by Christopher Anderson, p. 90 - Hillary to her State Trooper bodyguards after one of them greeted her with "Good morning."

"You f**king idiot."
(From the book "Crossfire" p. 84 - Hillary to a State Trooper who was driving her to an event.)

"If you want to remain on this detail, get your f**king ass over here and grab those bags."
(From the book "The First Partner" p. 259 - Hillary to a Secret Service Agent who was reluctant to carry her luggage because he wanted to keep his hands free in case of an incident.)

"Get f**ked! Get the f**k out of my way! Get out of my face!"
(From the book "Hillary's Scheme" p. 89 - Hillary's various comments to her Secret Service detail.)

"Stay the f**k back, stay the f**k away from me! Don't come within ten yards of me, or else! Just f**king do as I say, Okay?"
(From the book "Unlimited Access" by Gary Aldrige, p. 139 - Hillary to the Secret Service detail.)

"Many of you are well enough off that [President Bush's] tax cuts may have helped you. We're saying that for America to get back on track, we're probably going to cut that short and not give it to you. We're going to have to take things away from you on behalf of the common good."
Hillary to her fellow liberals in a fund raising speech in San Francisco ; SFGate.com 6/28/2004.)

"Why do I have to keep proving to people that I am not a liar?"
(From the book "The Survivor," by John Harris, p. 382 - Hillary in her 2000 Senate campaign)

"Where's the miserable c**ksucker?"
(From the book "The Truth About Hillary" by Edward Klein, p. 5 - Hillary shouting at a Secret Service officer)

"No matter what you think about the Iraq war, there is one thing we can all agree on for the next days - we have to salute the courage and bravery of those who are risking their lives to vote and those brave Iraqi and American soldiers fighting to protect their right to vote.
(Posted on Hillary's clinton.senate.gov web site 01/28/05)

"Put this on the ground! I left my sunglasses in the limo. I need those sunglasses. We need to go back!"
(From the book "Dereliction of Duty" p. 71-72 - Hillary to Marine One helicopter pilot to turn back while en route to Air Force One.)

"A right-wing network was after his presidency...including perverting the Constitution."
(To Barbara Walters about the Republicans who impeached her husband; 20/20, ABC 6/8/2003.)

"Son of a bitch."
(From the book "American Evita" by Christopher Anderson, p. 259 - Hillary's opinion of President George W. Bush when she found out he secretly visited Iraq just days before her highly publicized trip to Iraq)

"What are you doing inviting these people into my home? These people are our enemies! They are trying to destroy us!"
(From the book "The Survivor" by John Harris, p. 99 - Hillary's reaction to an aide, when she found out that some Republicans had been invited to the Clinton White House)

"I mean, you've got a conservative and right-wing press presence with really nothing on the other end of the political spectrum."
(C-Span, 1/19/1997 - Hillary complains about the mainstream media, which are all conservatives in her opinion)

"Come on Bill, put your dick up. You can't f**k her here."
(From the book "Inside The White House" by Ronald Kessler, p. 243 - Hillary to Gov. Clinton when she spots him talking with an attractive female at an Arkansas political rally)

”You know, I'm going to start thanking the woman who cleans the restroom in the building I work in. I'm going to start thinking of her as a human being”-Hillary Clinton
(From the book "The Case Against Hillary Clinton" by Peggy Noonan, p. 55)

"You show people what you're willing to fight for when you fight your friends."
(From the book "The Agenda" by Bob Woodward, ch. 14)

Being a [ Chicago ] Cubs fan prepares you for life—and Washington .
(Newsweek, April 18, 1994 p. 17)

"We are at a stage in history in which remolding society is one of the great challenges facing all of us in the West."
(From the book "I've Always Been A Yankee Fan" by Thomas D. Kuiper, p. 119 - During her 1993 commencement address at the University of Texas)

"The only way to make a difference is to acquire power" (p. 68).
(From the book "I've Always Been A Yankee Fan" by Thomas D. Kuiper, p. 68 - Hillary to a friend before starting law school.)

"We just can't trust the American people to make those types of choices.... Government has to make those choices for people"
(From the book "I've Always Been A Yankee Fan" by Thomas D. Kuiper, p. 20 - Hillary to Rep. Dennis Hastert in 1993 discussing her health care plan

tomder55 answered on 02/20/07:

Dick Morris is preparing a documentary on the real Hillary .Hillary's defenders are already calling it a "swift boating" .

JacquelineA2006 rated this answer Excellent or Above Average Answer
tropicalstorm rated this answer Excellent or Above Average Answer

Question/Answer
tropicalstorm asked on 02/20/07 - I just found out

my neighbors, son Matt Bowe, just got killed in Iraq.
He was a medic. I think he was in a police station or something.
HOW are they ever going to stop this nightmare?
They don't even want to admit that the terrorists have more sophisticated weapons to shoot down our planes now. If all our planes are being shot down how will we ever get enough troops over there to win a war?

I haven't even heard it on the news yet, they keep talking about Britney's shave and the fools on the hill. Why do people go hiking in the nasty weather?

tomder55 answered on 02/20/07:

I do not think that what I write will allay your fears or reduce your anxiety .It is hitting too close to home .

I think either in Iraq or some place else the war will continue for a long time. However ;that being said ,all is not what it appears to be (especially as reported ) .

I go to the blog site Iraq the Model frequently because the site is run and operated by 2 Iraqis who have lived in Baghdad from the beginning .It gives a great picture into daily life in Iraq, good and bad and today Mohamed tells us that Iraqi families are returning to their homes. Approximately 500 families.



Al-Sabah reports that yesterday alone 327 families returned home and that the scene of vans loaded with furniture of refugees leaving Baghdad is no more. There were times when the average was around 20 a day. The 327 figure brought the total to more than 500 families across Baghdad.

Al-Hurra TV aired a report on the story and interviewed some of the returning Baghdadis, one man said “those who returned earlier and saw the change in the situation called us and encouraged us to return, and I too will encourage the rest to come back”. The report showed those families asking the army to stay and not abandon their neighborhood, and showed the officer in charge giving his number to the locals so that they can contact him directly in case of emergency.

Looking at the relative increase in the number of attacks and their geographic extent one can expect the coming days to bring more escalation, but with the amount of power available for US and Iraqi troops I think the bad guys will not be able to achieve much.


The Iraqi's seem to feel the tide is turning, realistically it could be wishful thinking, but since they left originally because of fear, it leads me to believe they would not come back so soon if they aren't seeing something that our news media isn't bothering to show us.


Itsdb rated this answer Excellent or Above Average Answer
tropicalstorm rated this answer Excellent or Above Average Answer

Question/Answer
HANK1 asked on 02/18/07 - Iraqui Civilians:



Are ALL the Iraqui CIVILIANS armed? It seems to me that they should be killing some of the insurgents themselves.

HANK

tomder55 answered on 02/19/07:

I think that if I was a young and able Iraqi youth I would be signing up for the ISF. Many have at tremendous risk to themselves and their families .

I think it a little unfair to question the courage of these people . They lived under a brutal dictatorship for 3 decades. They came out en mass 3 times to vote even though they had no assurance of their personal safety . The jihadists threatened slaughter those days and still they voted and proudly waved their purple fingers in the air.

We don’t have to teach Iraqi people courage but it is in our interest and I believe our resonsibility to provide them with training and to stick around until they are capable of defending their freedom.

HANK1 rated this answer Excellent or Above Average Answer

Question/Answer
_JacquelineA asked on 02/19/07 - hilary clinton


how do you all feel about hilary clinton wanting to run for president?

she is extremely and ambitious and is going to run but i do not care for her.. i am not going to vote for her..

tomder55 answered on 02/19/07:

There are many reasons to be wary of a Hillary Clinton Presidency. I could go on for decades about Hillary Clinton and her questionable past and will do so in the course of this campaign . But today I will concentrate on only one issue ;since it is the issue that dominates the news these days ;the Iraq war.

Hillary ,along with just about everyone else was in favor of regime change in Iraq . The Clinton Adm. official policy regarding Iraq was regime change . It was a policy adopted by the Clinton Adm . and both houses of Congress from 1998 . She was as privy to the intelligence about Iraq as anyone in the Senate . There was no one more articulate in arguing that position than her.

When she made a speech before the war on the Senate floor she spoke in classic Clinton triangulation . She left herself enough wiggle room so if the operation got tough or failed she could defend her vote for the war resolution. She talked about giving the President the flexibilty to continue the process of ramping up the pressure on Saddam . Did she not know that she was voting for a war authorization ? Of course she did ! Did she know that once the President has the authority to invade that he would give the go ahead ? OF course she did ! We had already mobilized and our troops were poised to enter Iraq.

She is now being completely disengenuos to say that she expected the diplomatic process to continue .She is out on the campaign trail essentially saying 'I voted for the Iraq war before I voted against it '.She frequently cites her speech as proof that she really did not vote for the war .The anti war crowd is not buying it. Now that she must pander to the anti-war left wing of the Democrat party , she wants the US to retreat in 90 days . [this in direct response to Obama saying he wants us out by March 2008]And if Bush doesn’t comply? She’ll call on Congress to revoke the Authorization to Use Military Force .

The worst part about it is its nakedly political purposes. Hillary has come to regret her run to the center during her Senatorial career, and now wants to suck up to the International ANSWER set that runs the Left of the Democrats. In order to establish her bona fides and compete with Barack Obama’s demand for a phased withdrawal by March 2008, she just bumped the timetable up by nine months.

Clinton once said that an immediate U.S. withdrawal from Iraq would be “a big mistake” and was also opposed to setting a fixed date for troop withdrawal. Did she have a change of heart and opinion ? Hardly.This is clearly a two step dance around a political mine-field for her.

Is this leadership ? Is this what we expect from someone who will lead us ? Is this Presidential material ? You be the judge. She scares the hell out of me.



_JacquelineA rated this answer Excellent or Above Average Answer
labman rated this answer Excellent or Above Average Answer
Itsdb rated this answer Excellent or Above Average Answer
rusty rated this answer Above Average Answer

Question/Answer
HANK1 asked on 02/17/07 - IRAQ:

I have no doubts that the United States will win the war in Iraq by November, 2008. Since the Democrats have been pessimists and quite irritabale, will a Republican victory in Iraq quench any thoughts of them having a majority in the House and Senate in 2008? I think the Democratic Party will become somewhat obsolete in the minds of most Americans and be termed losers

HANK.


tomder55 answered on 02/18/07:

The question is the definition of winnimg. Insuregencies can be beaten but it is ususally a process that takes around a decade and it has to eventually include bringing the insurgents into the political process . That is for the Iraqi's to sort out.

Our obligation is to hand the country over to a government that can provide internal security . That can easily be done in the next 2 years.

Our longer obligations will be based on our national interests and in the terms of an alliance with the new Iraqi gvt. to help them defend themselves from external enemies. We have ,and continue to do this in Korea ;50 years after the cease fire;and in Europe ...well past the end of WWII and even over a decade since the collapse of the Soviet Union.

We are in a global war against jihadistan of which Iraq is but one battle. How we conduct this war will be the continuous debate of our time. It will be a long war ;at least as long as the cold war lasted (which claimed over 100,000 Americans in combat).It is my belief that this war will have to be waged equally with Democrats in control of the legislature and the executive .

No,I don't believe the Democrat party will become obsolete . My view is that there have been competing visions about the nature of the American experiment since the days of the Federalist Papers ;written by Alexander Hamilton,John Jay and James Madison ....and the Anti-Federalist papers written most likely by authors such as George Clinton ,Richard Henry Lee and Patrick Henry. Political parties have come and gone ,but the nature of the debate really hasn't changed much .

HANK1 rated this answer Excellent or Above Average Answer

Question/Answer
Itsdb asked on 02/16/07 - Is Black History Month Getting Diluted?

Black History Month: Come February, the now-familiar observance seems to inspire ever more - and ever more random - celebrations.

The players are both big and small. Multinational corporations mount billboard campaigns, while community centers hold fashion shows and tourist spots highlight their connection to black history.

But does saturation equal success?

While the concept of Black History Month has been widely embraced in pop culture, it means some of the nation's most bitter history also is getting watered down into cliches or irrelevance. Some events have no historical tie-in at all - they're merely topics of interest to African-Americans. And, perhaps unsurprisingly, black history is used as a kind of commercial brand, which can feel off-key.

"It has become very mainstream," said Sheri Parks, a professor of American Studies at the University of Maryland. "I do think it's been diluted. Some of this seems like an excuse to put things on sale."

At Drexel University in Philadelphia, February events range from panel discussions about affirmative action and self-segregation on campus to a black art sale and an African American Down-Home Soul Food Dinner.

In Maryland's Prince George's County, there's Black History Magic, African Jewelry Making and a Black History Cheerleading Show.

A new-age center in Oakland, Calif., offered Mindful Drumming for Opening Minds and Healing Hearts and the University of Cincinnati's United Black Student Association has planned an event about online privacy titled "Has Facebook gone too far?"

Is this black history?

Though well-intentioned, the events are probably not what historian Carter G. Woodson had in mind when he created Negro History Week in 1926. He taught for decades that blacks must know their past before they could envision a brighter future.

By 1976, his organization, now called the Association for the Study of African American Life and History, had turned the week into Black History Month.

"The resistance was tremendous all over the country," said Maurice Thornton, a historian at the State University of New York at Albany. "There was a countervailing group who were doing their best to erase black history from the general public."

They lost the battle.

This month, Thornton said he gave a black history speech at the local office of the Federal Bureau of Investigations. "They're reaching out and want to let the world know that they're not just the old folks who tap your phones like they did during the civil rights era," he said.

Each night this month, there are several black history television programs to choose from - from BET's "Tupac: Thug Angel" to "Inside the Actors Studio" featuring two-hour interviews with Diana Ross and Eddie Murphy.

President Bush marked the month by holding a ceremony honoring modern-day black heroes including a New York City construction worker who saved a man from an oncoming subway train and an Olympic skier who lost her leg.

Black History Month "does caricature itself at times," said Linda Symcox, author of "Whose History?: The Struggle for National Standards in American Classrooms," about revising American history to include minority groups. Though she believes the month is a good thing overall, she said some events cross the line.

"If I were an African-American, I would be offended by having the month of February be some kind of palliative," she said.
~~~~~~~~~~~~~~~~~~~~~~~~~~~~~~~~~~~~~~~~~~~~~~~~~~~~~~~

I have mixed emotions about Black History Month. If blacks feel a need to have it, ok I guess, but like all good American initiatives it appears to have become another excuse to make money and do meaningless or silly things. I mean really, "Mindful Drumming for Opening Minds" and "Tupac: Thug Angel?"

And then there's the inevitable question of what would Blacks think of a White History Month, with the inevitable answer "every month is white history month." Does it really matter? How are we ever going to get beyond racism when the 'persecuted' keep their race front and center?

And what does this writer want BHM to be? ...it means some of the nation's most bitter history also is getting watered down into cliches or irrelevance.

I get the impression - much the same as I do when it comes to the often inexplicable Democratic romance with blacks - that racial bitterness is an essential nutrient.

Comments?

tomder55 answered on 02/16/07:

I heard one black comedian on 'The Daily Show' comment about the practice of naming an influential black American every day that by the end of the 2nd week you would be recognizing the Waylan brothers .

He was much more practical . He said that all he wanted was some of the casinos that native-Americans get .

Itsdb rated this answer Excellent or Above Average Answer

Question/Answer
Itsdb asked on 02/16/07 - "Green" pirates

I came across this interesting story today:

    Japan urged to 'use Greenpeace'

    New Zealand's Conservation Minister Chris Carter said it is imperative the Nisshin Maru is moved to prevent any pollution of the pristine area.

    Japan has refused the offer, saying the ship - which has been disabled by a fire - is no threat to the environment.

    The Japanese whalers have had a number of confrontations at sea with anti-whaling activists in recent days.

    One crew member is still missing after the fire broke out on board the Nisshin Maru - an 8,000-tonne processing ship - just before daybreak on Thursday.


Good for Japan, it's no wonder they refused help from an American environmental group since another is down there stirring up crap...

    The Sea Shepherd ships Farley Mowat and the Robert Hunter have been engaged in a confrontation with the Japanese whaling factory ship Nisshin Maru since 0530 Hours this morning.

    When the Robert Hunter first approached the Nisshin Maru there were three hunter killer vessels with the mother ship. These vessels quickly fled northward. The Nisshin Maru fled east straight towards the Farley Mowat. At two miles from the Farley Mowat, the Nisshin Maru turned and fled back west again.

    There was a pod of whales in the area near the whalers. The Sea Shepherd crew is happy to report that these whales fled and are now safe from the Japanese harpoons.

    The Sea Shepherd crew has successfully delivered six liters of butyric acid onto the flensing deck of the Nisshin Maru. This "butter acid" is a nontoxic obnoxious smelling substance. The foul smell has cleared the flensing deck and stopped all work of cutting up whales.

    Sea Shepherd crew in Zodiacs have nailed plates to the drain outlets (near the waterline) on the Nisshin Maru that spill the blood of the whales from the flensing deck into the sea. This is backing up the blood onto the flensing decks. The plates are secured by Hilt nail guns that drive steel nails through solid steel.


    Sea Shepherd has relayed the coordinates to the Greenpeace vessel Esperanza. Despite Greenpeace's refusal to cooperate with the Sea Shepherd ships, the whales need every anti-whaling ship in the area. Greenpeace refuses to give Sea Shepherd their position so it is unknown how long it will take them to reach the area where the whaling fleet is operating.

    Sea Shepherd was able to outmaneuver the Japanese satellite tracking system by taking the ships south of the Balleny Islands through heavy ice. Thirty years of ice navigation opposing the Canadian seal slaughter has proven quite helpful. The Japanese satellite surveillance program cannot track a ship's wake through the ice. "We came in the back door out of the freezer so to speak," said Captain Paul Watson.

    The following message was delivered to the Japanese whaling fleet this morning:

    Nisshin Maru, this is Captain Paul Watson of the Sea Shepherd vessel Farley Mowat. Please be advised that you are killing whales in the Antarctic Whale Sanctuary. You are targeting endangered species of whales in violation of international conservation law. You are killing whales in violation of the IWC global moratorium on commercial whaling. Please cease and desist your illegal whaling operations and leave the Antarctic Whale Sanctuary. We are acting in accordance with the principles of the United Nations World Charter for Nature. The Charter authorizes non-governmental organizations and individuals to uphold international conservation law.

    The Sea Shepherd ships have been searching for the Japanese fleet for five weeks and have covered thousands of square miles of ocean.

    The crew of both Sea Shepherd ships are relieved that the Japanese whaling fleet has been found and is now under pursuit and engaged in confrontation with the Robert Hunter and the Farley Mowat. They can now do what they do best – save whales!


Regardless of your view on whaling, what the heck is wrong with these people? In addition to this report Sea Shepherd was also caught ramming another Japanese vessel on video. Eco-terrorists have used vandalism, arson and spiked trees to force their agenda - now we have eco-pirates? Especially eco-pirates who mission statement ends with "Sea Shepherd adheres to the utilization of non-violent principles in the course of all actions and has taken a standard against violence in the protection of the oceans."

Comments?

tomder55 answered on 02/16/07:

I think they should remember what happened to the 'Rainbow Warrior' .Honestly ;if I were captain of one of the vessels I'd be tempted to point my harpoons in their direction. I was once on a fishing party boat going for blue fish . A drunk kept on crossing the bow trying to get to the best fishing spots before our boat. Our captain threatened to gaff the clown ;but he opted instead to dump a bucket of chum on the guy .

I also think if the Japanese are signatories to the treaties mentioned they should comply.

Itsdb rated this answer Excellent or Above Average Answer
labman rated this answer Excellent or Above Average Answer

Question/Answer
Itsdb asked on 02/16/07 - Al-Sadr Backs Security Plan?

Iraqi President Jalal Talabani says radical Shi'ite cleric Muqtada al-Sadr is believed to have ordered senior officials of his Imam al-Mahdi Army militia to leave Iraq.

Talabani said he believes al-Sadr supports new security operations launched in Iraq this week, and told his officials to leave the country in order to facilitate the security plan.

The comments on February 15 came amid speculation that al-Sadr himself was in Iran.

Talabani said he had no information on al-Sadr's whereabouts.

Meanwhile, reports say the leader of Al-Qaeda in Iraq -- Abu Ayyub al-Masri -- was wounded in a clash north of Baghdad.

The AP news agency and Al-Arabiya Television quote the Iraqi Interior Ministry as saying an aide to al-Masri was killed in the clash near Balad, a major U.S. base about 80 kilometers north of Baghdad.

~~~~~~~~~~~~~~~~~~~~~~~~~~~~~~~~~~~~~~~~~~~~~~~~~~~~~~~

I don't believe for a minute Mookie supports the new plan. I think he's doing what all good Jihadists do when the tide turns against them, run and hide and then lie through their teeth. But hey, if the Mookie Militia wants to leave Iraq...you finish the sentence.

There is no confirmation yet as to the al-Masri report, but we can hope. Personally I think the surge is already showing signs of success, but you wouldn't know it by reading the news. Heck, they don't even diguise their analysis as news any more they just report it as fact. This morning's report:

    BAGHDAD - U.S. and Iraqi forces pushed deeper into Sunni Muslim militant strongholds in the Iraqi capital Thursday-where cars rigged with explosives greeted their advance - while British-led teams in southern Iraq used shipping containers to block suspected weapon-smuggling routes from Iran...

    The series of car bomb blasts, which killed at least seven civilians, touched all corners of Baghdad. But it did little to disrupt a security sweep seeking to weaken militia groups' ability to fight U.S.-allied forces - and each other - as Iraq slips further into factional bloodshed.

    The attacks, however, pointed to the critical struggle to gain the upper hand on Baghdad's streets. The Pentagon hopes its current campaign of arrests and arms seizures will persuade average Iraqis militiamen are losing ground. Yet each explosion is another reminder of the militants' resources and resolve.


Is there something in the left's water that poisons their minds until they're incapable of acknowledging a good thing in Iraq, or God forbid, rooting for a success there? It's a good thing some facts just can't be overlooked even by the liberal media or Democrats in congress, such as, The number of Iraqi civilians killed in Baghdad's sectarian violence fell drastically overnight, an Iraqi military official said Friday, crediting the joint U.S.-Iraqi security operation that began in force just days ago.

Could things be turning around? If they do, will the media grit their teeth and report it?

tomder55 answered on 02/16/07:

The aide killed was Abu Abdullah al-Majamiai long believed to be head of Masri's security detail. I had heard rumors that TaskForce 145 had been closing in on him since they captured a close aide of his last week . The capture of killing of Masri would be a major victory . He is a far more competent,organized ,and dangerous than Zarqawi(who was pretty much just a brute).

Yes the surge is already working even though we have yet to add one extra troop to the effort.All it took was a little unleashing the dogs . The question really becomes at this point can it succeed before the Democrats remove all pretext of reservation and defund the effort .

My biggest concern is that they see our success,panic and speed up their timetable of betrayal . Remember ;going into the summer of 1864 Lincoln and the Republicans were on the ropes.But all it took was military victories by Grant and Sherman to put down the copper-head Democrats ....defeatocrats even then .

As far as Mookie is concerned ,I touched on this yesterday .I believe that he is making a mistake going to Iran. It was the Iraqi Army ,not the Mahdi militia that made swiss cheese out of Masri's aide. It will be the Iraqi Army ;not Mookies that will have cleaned Baghdad of al-Qaeda vermin. The Iraqi populace will know who they can count on when things get tough....(well that's the plan anyway) .

Itsdb rated this answer Excellent or Above Average Answer

Question/Answer
paraclete asked on 02/16/07 - now that's an anti semitic racial comment?

Jewish deputies against Muslim quota idea


February 16, 2007 - 1:14PM
Latest related coverage

* Limit Muslim migration, Australia warned

The Jewish Board of Deputies has distanced itself from comments by an Israeli academic who says Australia should limit its number of Muslim immigrants.

Professor Raphael Israeli is quoted in the Australian Jewish News as saying that without such a migration cap Australia risks being swamped by Indonesians.

But Prof Israeli told the Herald his comments had been misunderstood.

"When the Muslim population gets to a critical mass you have problems," he said.

"That is the general rule - so if it applies everywhere, it applies in Australia."

But NSW Jewish Board of Deputies CEO Vic Alhadeff today distanced his organisation from the comments.

"The Jewish community dissociates itself from the comments by Israeli academic Raphael Israeli that Australia should limit the number of Muslim immigrants," he said in a statement.

Mr Alhadeff said the Jewish community did not believe racial or ethnic quotas were helpful.

"We do not believe in racial or ethnic quotas or stereotyping," he said.

"These comments do not reflect the position of the Jewish community and are unhelpful in the extreme.

"The Jewish community has a strong and proud record in fighting racism, and condemns all expressions of bigotry."

Prof Israeli, an expert on Islamic history at Hebrew University in Jerusalem, has been brought to Australia by the Shalom Institute of the University of NSW.

He said Muslim immigrants had a reputation for manipulating the values of western countries, taking advantage of their hospitality and tolerance.

"Greeks or Italians or Jews don't use violence," he told Fairfax.

"When there are large Muslim populations who are prepared to use violence, you are in trouble.

"If there is only one or two per cent they don't dare to do it - they don't have the backing of big communities.

"They know they are drowned in the environment of non-Muslims and are better behaved."

AAP

tomder55 answered on 02/16/07:

There are some working models to predict a likely outcome. The Scandanavian countries are quickly approaching dhimmitude .Oslo will have a non-Western Muslim majority in a few decades, if the current trends continue. Norway, Sweden and Denmark;the native population will become a minority within the century. The only question is when. Since Jihadistan enters a much more aggressive phase once the Muslim population reaches 10 – 20% of the total population, this does not bode well for the future . Will they turn out different from similar regions in Thailand,or the Philippines ? Will we see riots like Paris had in Sydney ?


We already know about the rapes that are justified by religious intolerances happening in Australia as well as the Scandinavian nations I mentioned .Didn't one rapist declare that the victim had no right to say know because she was not wearing a head scarf ? Sheik Faiz Mohammed gave a lecture in Sydney where he informed his audience that rape victims had no one to blame but themselves. Women who wore skimpy clothing, invited men to rape them.Islamic "scholar", Shahid Mehdi said the same thing in a lecture in Copenhagen.Egyptian Sheik Yusaf al-Qaradawi claimed female rape victims should be punished if they were dressed immodestly when they were raped. He added, “For her to be absolved from guilt, a raped woman must have shown good conduct.”

In Norway two out of three prosecutions for rape are immigrants with a non-Western background and 80 percent of the victims are Norwegian women.In Sweden , Gang rapes, involving Muslim immigrant males and native Swedish girls, have become commonplace. Time Mag. BRUCE CRUMLEY AND ADAM SMITH wrote an essay describing the outrageous treatment of women. It opens with a book called "In the hell of the tournantes" the last word is a slang term for gang rape.

Published last month, the book has shocked France with its graphic accounts of the attacks and Bellil's impassioned denunciation of the increasing violence and sexual abuse committed against young women in the banlieues. Since 1999, rapes within the banlieue have increased by 15% to 20% every year. Dedicated to the countless "sisters in this hell, so they'll know there's a way out," Bellil shows precisely how and why sex crimes are surging in the projects. "As children of immigrants, we receive a strict upbringing and are judged very harshly if we stray from it," says the Algerian-born Bellil, 29, who was raised in a non-practicing Muslim household. "From the moment a girl steps outside, guys think they have the right to pass judgment and treat us differently. In extreme cases, this leads to violence or aggression."

In Bellil's own case, it led to a horrific sequence of gang rapes, in which she was brutalized in fetid apartments and on the ground between filthy trash cans. When one attack was over, her assailants offered Bellil compensation in the form of breakfast and a 10-franc coin. Though the assaults occurred in the late 1980s, Bellil didn't speak up or press charges until three other girls attacked by the same gang appealed to her. Bellil decided to write about the experience now to call attention to the spate of banlieue gang rapes and the perverse attitudes toward sex that feed the crimes.

Reports of sexual assaults against women have risen across France, with court convictions for rape having soared by 61% between 1995 to 2000. But specialists and victims' groups say violence against women is especially acute in the banlieues because of cultural attitudes toward women. Banlieue males may adopt the lifestyles of other French youths — pop music, fast cars and pornography — but they also frequently embrace the traditional prejudices of their immigrant parents when it comes to women: any neighborhood girl who smokes, uses makeup or wears attractive clothes is a whore. Bellil's attackers targeted her because she dressed as she pleased, mixed with males and liked to dance — and had begun a romance with another teen. Owing to the fact that most rapes involve individuals known to victims, intimidation often suffices to ensure that charges are never lodged. "Victims know that they won't be protected by the police," says Bellil, "and that both they and their families will be threatened if they speak up."


See the future ? I bet Professor Raphael Israeli does .




paraclete rated this answer Excellent or Above Average Answer

Question/Answer
tropicalstorm asked on 02/16/07 - LAW OF THE LAND

Diss a 'gay'? Go to jail!
Activists warn Christians targeted under new 'hate crimes' proposal
Posted: February 15, 2007
1:00 a.m. Eastern

By Bob Unruh
© 2007 WorldNetDaily.com

Two Christians in Australia have been indicted for criticizing Islam, and another for criticizing Zionism. A filmmaker has been threatened with arrest for using the word "homosexual" rather than "gay." Now a German priest faces jail time for publicly criticizing abortionists, and in Holland, "fornicators" and "adulterers" are protected classes and cannot be criticized.

All courtesy of the concept of federal "hate crimes" legislation, which unless defeated soon could be mandatory in the United States, warns a rising chorus of critics.

"All that matters are the delicate feelings of members of federally protected groups," said Michael Marcavage, director of RepentAmerica.com "Truth is not allowed as evidence in hate crimes trials. … A homosexual can claim emotional damage from hearing Scripture that describes his lifestyle as an abomination. He can press charges against the pastor or broadcaster who merely reads the Bible in public. The 'hater' can be fined thousands of dollars and even imprisoned!"

All this, he noted, to attack incidents that according to the FBI's 2005 Uniform Crime Report make up on one-fifteenth of 1 percent of all crimes.
The language is in a new proposal pending in Congress, H.R. 254, or the David Ray Hate Crimes Prevention Act. That, according to Rev. Ted Pike, of the National Prayer Network, starts out with a federal police state enforcement of "anti-hate" laws but would, as it has in other parts of the world, "lead inexorably to the end of free speech."

tomder55 answered on 02/16/07:

Tim Hardaway don't stand a chance then .

tropicalstorm rated this answer Excellent or Above Average Answer

Question/Answer
HANK1 asked on 02/15/07 - FREEDOM:



Have the people in the United States lost their love for freedom?

HANK

tomder55 answered on 02/15/07:

Hank ,as you know ,to some extent the social contract is a surrender of some personal freedom in return for collective security. Our contract is written in a way that any liberty we forfeit is of our choosing and at any time we have the means to change the rules and reassert any freedom surrendered .

I agree with the other experts about complacency and sometimes I supposed our memory needs to be jarred .But it is a sign of success that we quickly go about our lives .

It is a funny thing that occures frequently at any town hall in the country . For months at a time meetings of town representatives have almost zero attendance .But let an unpopular zoning decision or perhaps revenue and spending decision be made and suddenly the board meetings are standing room only .One only has to conclude that at times when the public is not mobilized that there is a general contentment ,that the public does not perceive their essential liberties at risk.

HANK1 rated this answer Excellent or Above Average Answer

Question/Answer
Itsdb asked on 02/14/07 - Huh?

Representative Lynn Woolsey (D - CA) made an interesting quote today (and I'm quoting from memory here), "I've been working to bring our troops home since before we sent them."

Now THAT takes some effort. Woolsey introduced the Bring the Troops Home and Iraq Sovereignty Restoration Act of 2007 in January, which calls for a withdrawal in six months, accelerating the training of Iraqi security forces IN that six months, and other nonsense. Highlights (if you can call them that) can be found here.

Batten down the hatches folks, the American jihad is underway...

tomder55 answered on 02/15/07:

I turned the debate on and off yesterday between running the snow blower. I liked the way Pete King was directing the minority debate . As I've said before, the Republican House members are doing a much better job defending the surge than the weasle Senate Republicans did .

Lynn Woolsey is the same moonbat who introduced introduced legislation (H.Res. 68) that calls on President Bush “to eliminate weapons of mass destruction from United States.

As far as her 'get out now 'strategy ,it is a matter of cold sinister calculus . Woolsey realizes that, given enough time, Americans will recognize that the Dems don’t have a suitable plan for Iraq and may look to the Republicans again in 2008. By creating a bloodbath in Iraq now, which can be blamed on the Bush presidency, Woolsey hopes to create a clean slate for a total Democratic victory in 2008. Sixteen other Kossack Demoncrats also signed onto her legislation.

She is clearly acting outside of the consent of Madam Mimi ,and I don't think Pelosi will let the bill get to the floor. But then again....if Pelosi keeps taking council from Murky Murtha ..who knows? .
Despite the posturing , the Democrats will not cut off funding or force a troop withdrawal this year. If Iraq improves they will take credit. Otherwise they can still blame everything on President Bush.

Itsdb rated this answer Excellent or Above Average Answer

Question/Answer
Itsdb asked on 02/14/07 - Got any plans?

Why bother with a cruise, a weekend in the Hamptons or Disney Wolrd when you can attend one of these exciting events?

Emergency Summit to Impeach Bush for War Crimes -February 17 & 18 New York City

Ides of March Forum, RESTORING OUR RIGHTS: A Public Forum on Spying, Detention, and TortureUniversity of Virginia Law School Caplin Auditorium, Charlottesville, Va -March 15, 2007, 7 p.m.




Or how about committing An Act of Art to End the Iraq Occupation? -February 19th, check for locations.

And if you missed the Beach Impeach or the Mall Impeach, you can always host the Bush Chain Gang in your community. Or, just join the National Boycott to Impeach for Peace. Yeah that'll keep the economy humming along for all those poor, suffering Americans.

tomder55 answered on 02/15/07:

well I would attend the Emergency Summit but I have an important date with some fresh snow covered mountain trails and my little used snow shoes.

Itsdb rated this answer Excellent or Above Average Answer

Question/Answer
kindj asked on 02/14/07 - I gotta tell you, this guy looks interesting:

http://www.cnn.com/2007/POLITICS/02/13/romney.announce/index.html

What do y'all think?

tomder55 answered on 02/14/07:

He's off to quite a controversial start .He probably could've chosen a better place to launch than the Ford Museum .Iunderstand his theme of turning to good ole American ingenuity to solve the problems and issues of the day , and to make a call for a transformation of government. But, there is that specter of anti-semitism that hangs around anything related to Henry Ford. I think Al Dershowitz made too big a deal about it today . Romney is clearly a friend of the Jews and of Israel .But it will add grist to the millstone . His staff should've known better .


He will have to also face obstacles because of his own religion. It shouldn't be an issue but if you believe the polls less than 5 % of Americans polled said they would support a Mormon canidate and unfortuately some of the loudest opposition comes from the Republican base itself.

I'd like to get a copy of his announcement . I heard some of it and it sounded Reaganesque ,but unfortunately he is not the greatest speaker on the planet. Like with President Bush, I will probably find it useful to read his speaches rather than listen to them.

His big hurdle will be that to be a successful Mass. pol. you have to come out with socially liberal positions.He was therefore a strong supporter of gay rights,gun control ..all the Huffington talking points. His challenge will be to appeal to the Republican base and at the same time to not be labelled a flip-flopper . Rudy has simular issues but he will have an easier time of it because he is nationally recognized for his leadership skills.Romney hopes the country will focus on his management skills (very successful business man /savior of the 2002 Winter Olympic Games...a less successful govenor however )

I give him big plusses for his unwavering support of the war. He also indicates that President Bush was right in his selections to positions at SCOTUS and shares his judicial view. Also if you put all the issues on a scale he comes out at least as conservative as the the two top contenders.

Overall I consider him a dark horse to win the nomination . I think Rudy and McCain will slug it out for the nomination unless there is some real ground swell for someone like Duncan Hunter or Newt Gingrich .

kindj rated this answer Excellent or Above Average Answer

Question/Answer
tropicalstorm asked on 02/14/07 - 'nother media cover up

Utah shooter was Bosnian Muslim

tomder55 answered on 02/14/07:

you mean the terrorist who went on a shooting spree was a Muslim . Imagine that ! Solejman Talovic .....I guess his name was the first tip. Gotta wonder what the motive is .

The news broke from The Salt Lake Tribune yesterday so I imagine that it was pretty timely announcement .OF Course the major news services had other things on their mind ...like the biggest blizzard ever .....and what people were buying for Valentines Day to be concerned about the detail . My guess is that when the media wakes up the emphasis will be on gun control.

Honestly ,I actually prefer that they don't overplay this . The health of the economy rests with consumer confidence. I'd hate to see a rash of shopping mall attacks .Shopping malls are almost the equivelent to cathedrals in our secular world .

ETWolverine rated this answer Excellent or Above Average Answer
tropicalstorm rated this answer Excellent or Above Average Answer

Question/Answer
Itsdb asked on 02/13/07 - New honor for Carter

LOS ANGELES - Jimmy Carter became the second former U.S. president to win a Grammy Award when he was honored in the spoken-word category Sunday for the audio-book version of his bestseller “Our Endangered Values: America’s Moral Crisis.”

Carter, 82, shared the award in a rare tie with actress Ruby Dee and her late husband Ossie Davis for “With Ossie and Ruby: In This Life Together.”

Recent winners in spoken word category:

2006 - Jimmy Carter/Ruby Dee & Ossie Davis

2005 - Barack Obama, Dreams from my Father

2004 - Bill Clinton, My Life

2003 - Al Franken, Lies and the Lying Liars Who Tell Them: A Fair and Balanced Look at the Right

Is there a pattern here? Any bets on a repeat for Obama this year with The Audacity of Hope?

In 2006, Carter's book didn't make the top 100 at BarnesandNoble.com. It didn't make the top 50 at Amazon.com, where the Dog Whisperer took top honors. Carter must have been very dramatic in his oration...

tomder55 answered on 02/14/07:

it is however a big seller at insomnia and sleep disorder treatment centers nationwide.

I'm still waiting for dual awards to Al Gore ;Oscar (best fantasy) ,and the Nobel Peace Prize ...before he announces his candidacy .

Itsdb rated this answer Excellent or Above Average Answer

Question/Answer
tropicalstorm asked on 02/13/07 - what a laugh

Limbaugh is playing parodies of
Johnny cash ball of fire
Louey Armstrong what a beautiful world and
Arthur Brown Fire

for al gore

tomder55 answered on 02/13/07:

I think you mean Johnny Cash's "Ring of fire "


I fell in to a burning ring of fire
I went down,down,down
and the flames went higher.
And it burns,burns,burns
the ring of fire
the ring of fire


lol that's funny !!

Itsdb rated this answer Excellent or Above Average Answer
tropicalstorm rated this answer Excellent or Above Average Answer

Question/Answer
HANK1 asked on 02/13/07 - LINCOLN:



Lincoln was against the spread of slavery into the territories but was NOT an abolitionist. Why was he called the great emancipator and a champion of black freedom?

HANK

tomder55 answered on 02/13/07:

Hank

Lincoln was always personally opposed to slavery .But his paramount position was of a strict constitutionalist.Since slavery was written into the law he had no desire to oppose it's lawfulness . His morality was not the primary consideration . His paramount concern was the preservation of the union. Anyone who teaches you that Lincoln's primary concern was slavery is misrepresenting his position and anyone who tells you he could care less about slavery is also telling you a fallacy .

Bottom line on your question is he got it done. That is why he is so honored .

HANK1 rated this answer Excellent or Above Average Answer
Itsdb rated this answer Excellent or Above Average Answer
labman rated this answer Excellent or Above Average Answer

Question/Answer
Itsdb asked on 02/13/07 - Where's the outrage?

Something that's puzzled troubled me for some time is the inconsistency from the left on what constitutes an outrage.

For instance, tom noted how Diane Sawyer caved to Ahmadinejad by wearing a hijab and his refusal to shake her hand, thereby allowing the Mahdi Hatter to perpetuate the stereotype that women are inferior. Where's the outrage? Instead, ABC's headline is trumpeted on the Huffington Post, EXCLUSIVE: Iranian President Says He's Ready to Cooperate on Nukes...

Imagine if Bush had requested Sawyer wear a dress for an interview...

In much of the Islamic world women are second class citizens (see above), the "insurgents" in Iraq, the Janjaweed in Sudan, the Taliban and Islamic factions all over are fighting to establish Sharia law that would spread this oppression of women. Where is the outrage?

Almost daily for the past 3 years or so our paper prints a letter to the editor referring to Bush's "lies." In Sawyer's interview the Mahdi Hatter denied any involvement in Iraq, claimed he was against any conflict and bloodshed, and we are the only country whose activities are completely transparent. The man is lying through his teeth for the entire interview, where is the outrage?

Moonbat conspiracists have long warned of Bush's impending theocratic rule, yet radical Islam seeks to establish a worldwide Caliphate by force. Where is the outrage?

Radical Islam cites two other reasons for their jihad against the west, support for Israel and the immoral, materialistic western lifestyle. Radical Islam seeks to eliminate Israel and kill or subjugate every Jew. The very issues the left holds dear; abortion, gay marriage, feminism, tolerance, multiculturalism, legalized drugs, needle exchanges, 'comprehensive sex education for kindergarteners,' and on and on, are a primary reason given by virtually every Jihadist for their desire to kill us. Radical Islam would put an end to virtually every value the left holds dear if given the chance.

Where is the outrage?

tomder55 answered on 02/13/07:

actually when I made those observations they were semi-tongue in cheek ... observing it as IF I were a liberal feminist . My real position on it is that when I traveled and in life I try to show respect for cultural differences ...even while living in Iran(fortunately for me it was a time of westernizing then so there was not much of a culture shock) .This was also true wearing yarmulka in my friends wedding party ,observing the traditions at a seder ...or dancing in a circle drinking too much ouzo at a Greek wedding (or eating spicey rice krispies at Indian gatherings). The Muslims believe they are showing women respect when they insist they cover themselves . It is the beating them to a pulp if they don't comply that gets me going .

Sawyer missed to boat and so many opportunities to give a good hard hitting journalistic interview that it would take too long to critique it point by point . I mentioned some of the more glaring omissions but did not touch any of the the follow up questions she could've asked to nail his lies and distortions .

You are right on target with your last paragraph. If they wanted to clean up the US in their image they would have to start at Hollywood and San Francisco .

Itsdb rated this answer Excellent or Above Average Answer

Question/Answer
labman asked on 02/13/07 - "How can you run for Senate
when you've never held a real job?"

My wife's coworker sent this to his local paper:


There is a widely circulating opinion in the mainstream media that says that President Bush is doing a poor job
with international affairs in general. That opinion says that he is dealing improperly with Iraq specifically. By
such accounts we are to believe that our military is drowning in problems from which there is no resolution
except to quit and go home.

Below is a text with interesting comparisons. After that is a section purported to be from Democratic Senator
John Glenn. I have not researched the authenticity, but the text is worth reading regardless of who said or
wrote the words.

The message was forwarded to me. The original had many grammatical errors, so I made some corrections. I
rearranged or deleted some headings. The content is essentially the same as I received except for those.
_____________________________________________________________________________

There were 39 combat-related killings in Iraq in January.
In the fair city of Detroit there were 35 murders in the
month of January. That's just one American city,
about as deadly as the entire war-torn country of Iraq.

When some claim that President Bush shouldn't
have started this war, consider the following:

a. FDR led us into World War II.

b. Germany never attacked us; Japan did.
From 1941-1945, 450,000 lives were lost,
an average of 112,500 per year.

c. Truman finished that war and started one in Korea.
North Korea never attacked us.
From 1950-1953, 55,000 lives were lost, an average of 18,334 per year.
d. John F. Kennedy started the Vietnam "conflict" in 1962.
Vietnam never attacked us.

e. Johnson turned Vietnam into a quagmire.
From 1965-1975, 58,000 lives were lost, an average of 5,800 per year.

f. Clinton went to war in Bosnia without UN or French consent.
Bosnia never attacked us.
He was offered Osama bin Laden's head on a platter
three times by Sudan, and did nothing.
Osama has attacked us on multiple occasions.

g. In the years since terrorists attacked us , President Bush
has liberated two countries, crushed the Taliban, crippled
al-Qaeda, put nuclear inspectors in Libya , Iran , and North
Korea without firing a shot, and captured a terrorist who
slaughtered 300,000 of his own people.

The Democrats are complaining about how long the war is taking,
but it took less time to take Iraq than it took Janet Reno
to take the Branch Davidian compound.
That was a 51-day operation.

We've been looking for evidence for chemical weapons
in Iraq for less time than it took Hillary Clinton to find
the Rose Law Firm billing records.
It took less time for the 3rd Infantry Division and the
Marines to destroy the Medina Republican Guard
than it took Ted Kennedy to call the police after his
Oldsmobile sank at Chappaquiddick.

It took less time to take Iraq than it took
to count the votes in Florida !!!!
Our Commander-In-Chief is doing a GREAT JOB!
The Military morale is high!
The biased media hopes we are too ignorant
to realize the facts.



Some people still don't understand why military personnel
do what they do for a living. This exchange between
Senators John Glenn and Senator Howard Metzenbaum
is worth reading. Not only is it a pretty impressive
impromptu speech, but it's also a good example of one
man's explanation of why men and women in the armed
services do what they do for a living.

This IS a typical, though sad, example of what
some who have never served think of the military.

JOHN GLENN (ON THE SENATE FLOOR)
Mon, 26 Jan 2004 11:13

Senator Metzenbaum (speaking to Senator Glenn):
"How can you run for Senate
when you've never held a real job?"

Senator Glenn (D-Ohio):
"I served 23 years in the United States Marine Corps.
I served through two wars. I flew 149 missions.
My plane was hit by anti-aircraft fire on 12 different
occasions. I was in the space program. It wasn't my
checkbook, Howard; it was my life on the line. It was
not a nine-to-five job, where I took time off to take the
daily cash receipts to the bank."
"I ask you to go with me, as I went the other day,
to a veteran's hospital and look those men,
with their mangled bodies, in the eye, and tell them
they didn't hold a job!

You go with me to the Space Program at NASA,
and go, as I have gone, to the widows and orphans
of Ed White, Gus Grissom and Roger Chaffee,
and you look those kids in the eye and tell them
that their dads didn't hold a job.

You go with me on Memorial Day and you stand in
Arlington National Cemetery, where I have more friends
buried than I'd like to remember, and you watch
those waving flags.

You stand there, and you think about this nation,
and you tell me that those people didn't have a job?
What about you?"

For those who don't remember,
during WWII, Howard Metzenbaum was an attorney
representing the Communist Party in the USA .
NOW HE'S A SENATOR!!!

tomder55 answered on 02/13/07:

Metzenbaum is an idiot . NASA astronauts are some of the highest educated ,motivated people ,and risk takers on the planet(especially the orginal group of "the right stuff"fame.)More of them should seek leadership positions in the country . It would certainly give them an outlet for their post-mission blues ;maybe prevent the silliness that happened with Lisa Nowak.

labman rated this answer Excellent or Above Average Answer
ETWolverine rated this answer Excellent or Above Average Answer

Question/Answer
HANK1 asked on 02/13/07 - MORE STRAEGY:



A couple of weeks ago I proposed an old wild west strategy for our troops to use in Iraq. That's what they're doing now!

How about this if what they're doing now doesn't work: Drop leaflets on Baghdad and the surrounding area telling all of the civilians to vacate the city. Our troops would help them, of course. Give them two weeks. Then level the place. This would make it possible to free up our troops in Iraq so they can take on Iran after a hiatus of sorts. This would allow our planes, ships and the Israeli air force to join our cause.

Your comments, please!

HANK

tomder55 answered on 02/13/07:

Let's get a grip . The insurgency and even the militias are a small minority there .

We are trying to help them become a civilized member of the world nations . Destroying the homes of about 8 million innocents will not accomplish our goals .

All we are saying is give Petraeus a chance.

HANK1 rated this answer Excellent or Above Average Answer

Question/Answer
paraclete asked on 02/12/07 - whose interfering?

Howard - Obama stouse a journalistic beatup or Obama interfering in Australian politics.

Howard is undiplomatic but correct


Gerard Henderson
February 13, 2007
Other related coverage

* Howard blasts Obama
* Rudd attacks PM over Obama bashing
* Obama blasts Howard on Iraq
* Stars and swipes: Howard defiant
* PM not sorry for Obama attack

Ask a blunt question and, every now and then, there is an undiplomatic answer. Certainly this was the case with the inaugural Sunday program for this year. Laurie Oakes put it to John Howard that the policy on Iraq enunciated by the US Democratic Party presidential aspirant Barack Hussein Obama was basically in line with that of Kevin Rudd and Labor in Australia.

Obama says he has a plan to withdraw all American forces from Iraq by March 2008 - about eight months before the presidential election of that year. Interviewed on Meet the Press last Sunday, the Opposition Leader, Kevin Rudd, committed a Labor government to withdrawing most Australian troops in Iraq at the end of a designated six-month rotation.

If Labor wins the election, likely to be held between mid-October and the end of November, this would mean that the Australian Defence Force would probably exit Iraq sometime early next year. The only Australian troops remaining in Iraq would be those whose task is to defend the Australian embassy in Baghdad.

Howard viewed the Rudd interview on Meet the Press. He could have chosen to fudge his response to Oakes's question about Obama's plan. But Howard appeared to consciously choose not to do so. Instead he described Obama's position as "wrong" and said such a policy "would just encourage those who wanted to destabilise and destroy Iraq".

The Prime Minister added that al-Qaeda members in Iraq could "put a circle around 2008" in their diaries and "pray as many times as possible for a victory not only for Obama but also for the Democrats".

Howard's comments were undiplomatic. It is unusual for an Australian political leader to support, or oppose, a particular candidate or party in another nation's election. It may have been for the better if the statement had not been made.

Yet it is a fact that Howard's comment has had such a big impact outside of Australia precisely because he was essentially correct. A designated and unconditional US withdrawal from Iraq by March 2008 would amount to a defeat for the multinational force and a victory for the insurgency, along with al-Qaeda, in Iraq. This should be recognised by those who oppose the allied commitment in Iraq as well as those who support it.

Howard is frequently accused by his political opponents of being poll driven. Yet there is scant evidence for such a proposition. It is understandable why many Australians oppose him - along with why many Australians support him. However, like it or not, Howard believes in the positions he has taken, including the Iraq commitment.

It seems he responded to Oakes in the manner in which he did because he genuinely believes Obama's plan would be disastrous for Iraq in particular and for the war against Islamist terrorism in general.

It is unlikely that he would have proffered a similar response if he had been asked to comment about Hillary Clinton's position in Iraq, simply because she has not designated a date for an unconditional US withdrawal from Iraq.

If Howard was undiplomatic in commenting on US domestic policies then the same can be said for Obama's response. Obama told reporters in Iowa that if Howard's comments were anything other than "empty rhetoric" he would commit 20,000 additional troops to the war. This is rhetoric in itself.

Australia has about 1500 men and women in Iraq. On a comparative population basis, an Australian force of 21,500 in Iraq would equate to some 320,000 Americans - almost double the US deployment.

It's a long time to the 2008 US presidential election. At the moment Obama is a high profile Democrat contender, as is Clinton. Yet it is not going to be an easy road for the African-American candidate. His government experience is confined to just two years in the US Senate, where he has not been associated with many prominent legislative initiatives.

Also, unlike Clinton, Americans as yet do not know much about Obama.

In his books The Audacity of Hope and Dreams from My Father, Obama indicates that he was brought up a Muslim and converted to Christianity. He is now a member of the United Church of Christ. There is no doubt that Obama will have a lot of appeal to the left of the Democratic Party. So did Howard Dean, who imploded during the 2004 US primaries, despite much support from the left.

If Howard wins in late 2007 and Obama prevails in late 2008 it is likely that both men will find a political accommodation. Likewise, a Rudd government would probably get along well enough with Bush during the final year of his presidency in 2008.

The reason why the Australian-American alliance works so well is because it is in the national interest of both nations. Sure, Australia benefits from the security and intelligence which the alliance provides. But the US also benefits from the alliance, with respect to its security and intelligence requirements.

The verbal punch-up between Howard and Obama indicates just how important the issue that divides them is to each man. Obama has consistently opposed allied intervention in Iraq. He is on record as saying that Saddam Hussein should have been left alone in Iraq, since he believed that he was no threat to the US.

On the other hand, Howard sincerely believed that Saddam had weapons of mass destruction and that he was a threat to the West and some Muslim nations alike.

What is going on right now is an undiplomatic row. However, it is unlikely to have long-term outcomes - irrespective of which party governs in Canberra or Washington in a couple of years.

Gerard Henderson is executive director of the Sydney Institute.

tomder55 answered on 02/13/07:

What PM Howard probably knows is that Obama went to a "Muslim school " (but not a madrassa) in Indonesia as a young child from 1967 to 1971. How that influenced him is unknown . But more importantly he is 100% correct that announcing a firm time table for withdrawal is irresponsible and any candidate that proposes one should be automatically disqualified from leading a great nation .

Itsdb rated this answer Excellent or Above Average Answer
paraclete rated this answer Excellent or Above Average Answer

Question/Answer
Itsdb asked on 02/12/07 - The ABC All-Dictator tour

Fresh off her chat with Bashar Assad (and gushing over the "tall, quiet" dictator and his wife who "famously live in a modest home...with three children they drive to school themselves, protecting family dinners, even biking together through villages"), Diane Sawyer visited with Mahmoud I'm-in-a-jihad.

Aside from his warm greetings to the good American people, denial of involvement in Iraq, repudiation of all conflict and a Clintonesque evasive action ("I do not know what you mean by militia.", the Mahdi Hatter gave Diane a bit of a smackdown...

    Mahdi: "Well, are you here to solve the problem of the American government in Iraq?"

    Sawyer: "I'm hoping that you can help solve the problem in Iraq."

    Mahdi: "Well, these are some points that must be discussed at the diplomatic level. You're just a journalist."


I wonder why ABC left Diane's "I'm hoping that you can help..." line out of their transcript? But hey, didn't she look stunning in her hijab?

tomder55 answered on 02/13/07:

I Wonder why a nose clip wasn't necessary ?

I don't know .If I was a women reporter I would probably not don a hijab or do anything to imply that women are inferior . I would've walked out of the interview if he had refused a hand shake. He insulted Dhimmi Diane from the start and in return she sucked up to him .

I also noticed that she had her entrourage of professional protesters following wherever she went. She said they were chanting anti-American slogans but who could be sure unless you speak Farsi ?



When you heard death to America slogans, I think you know it yourself, it is not related in any way to American public."

lol, nice dodge!!Boy does she look threatened !!!!! Where is her protest placard ?

Our people have no problem with American public and we have a very friendly relationship and this friendship is so great, that I wrote a letter to the American government, the aviation sector of our country and we wanted to establish direct flights from Tehran to New York and we want to have free travel of citizens.

hmmmmmm ,he wants to fly planes into N.Y. I wonder why ?


I did not see the whole interview . Did she ask him if he had an Ipod like she did with Assad ? "You're a country music fan. Faith Hill? Shania Twain?" I know she did not press on issues like the Mahdi-hatter wanting to wipe Israel off the map; his bizzare religious views ,any questions on Iran's relationship with Hezbollah...how and why Iran seeks to support undermining the legitimate government in Lebanon through its directing of Hezbollah activities ....or press him on the Iran nuclear program. So it had to come down to human interest fluff. She should've at least taken the opportunity to do a cheap plug for 'Irish Spring'.

Here ;I'll frame the question in a way that is palatable for all those involved :

Sawyer : If your nuke program really is for peaceful purposes, why do you think warmongering neocon fascists like President Bush think you're intent on making nulear weapons?

She can be tough in an interview when she wants to be . She once asked "You've been compared to Saddam Hussein. Nero. To Torquemada, who was head of the Inquisition."

But she was interviewing Ken Starr at the time.

She has in the last year completed her tour of the new axis of evil (Syria replacing Iraq) . In N. Korea she said this about the gulag education system :

"a world away from the unruly individualism of any American school."One student proclaimed "We are the happiest children in the world!"

Itsdb rated this answer Excellent or Above Average Answer

Question/Answer
Itsdb asked on 02/12/07 - Now this is interesting...

...considering comments about the state of the U.S. from certain among us from down under.

    Halal beef costs State $2000

    Mark Oberhardt

    February 12, 2007 01:38pm
    Article from: NEWS.com.au

    THE state government has been ordered to pay a former Muslim prisoner $2000 in compensation because it did not feed him Halal meat.

    In the Supreme Court yesterday, Justice Ann Lyons dismissed an appeal by the state government against the decision of the Anti Discrimination Tribunal.

    Last May, the Tribunal found the government directly discriminated against Sharif Mahommed during periods of his incareceration at the Wolston Correctional Centre and Palen Creek Correctional Centre.

    The Tribunal further found the government had indirectly discrimininated against Mahommed also at Palen Creek.

    Mahommed was jailed in March 2000 and released in April 2005.

    As a Muslim Mahommed eats only Halal meat _ meat blessed and slaughtered by a Muslim slaughterman, cooked and stored in accordance with religious laws.

    The court heard Mahommed had been the first prisoner in a Queensalnd (sic) jail to request Halal meat.

    As a result of his efforts all Muslim prisoners are now provided wirth (sic) fresh Halal meat.

    The court was told during the first 10 months of his jail term Mahommed was fed general prison food and later he was given tinned Halal meat which he had not been able to consistently eat.

    It caused Mahommed to lose a substantial amount of weight.


    Mahommed lodged an appeal with the Anti Discrimination Tribunal which found in his favor on two complaints.

    The government was ordered to pay Mahommed $2000 as compensation.

    The state government appealed to the Supreme Court seeking the decision be quashed and be remitted to the Tribunal for rehearing.

    However, Justice Lyons found the government had not shown a basis for a successful appeal.

    She said the tribunal finding had been on a purely factual basis and no question of law had arisen from the findings.


Will the Aussies now be employing 'Muslim slaughtermen' in the prisons sinced the canned stuff isn't good enough? I'd be careful who you give sharp knives to...

tomder55 answered on 02/13/07:

send him to Club Gitmo . They serve Halal meals there 3 times a day .

Itsdb rated this answer Excellent or Above Average Answer

Question/Answer
tropicalstorm asked on 02/11/07 - Eternal embrace
for Valentines day




Couple still hugging 5,000 years on

Tue Feb 6, 1:28 PM ET

ROME (Reuters) - Call it the eternal embrace.


Archaeologists in Italy have discovered a couple buried 5,000 to 6,000 years ago, hugging each other.

"It's an extraordinary case," said Elena Menotti, who led the team on their dig near the northern city of Mantova.

"There has not been a double burial found in the Neolithic period, much less two people hugging -- and they really are hugging."

Menotti said she believed the two, almost certainly a man and a woman although that needs to be confirmed, died young because their teeth were mostly intact and not worn down.

"I must say that when we discovered it, we all became very excited. I've been doing this job for 25 years. I've done digs at Pompeii, all the famous sites," she told Reuters.

"But I've never been so moved because this is the discovery of something special."

A laboratory will now try to determine the couple's age at the time of death and how long they had been buried.

Couple still hugging 5,000 years on

Tue Feb 6, 1:28 PM ET

ROME (Reuters) - Call it the eternal embrace.

Archaeologists in Italy have discovered a couple buried 5,000 to 6,000 years ago, hugging each other.

"It's an extraordinary case," said Elena Menotti, who led the team on their dig near the northern city of Mantova.

"There has not been a double burial found in the Neolithic period, much less two people hugging -- and they really are hugging."

Menotti said she believed the two, almost certainly a man and a woman although that needs to be confirmed, died young because their teeth were mostly intact and not worn down.

"I must say that when we discovered it, we all became very excited. I've been doing this job for 25 years. I've done digs at Pompeii, all the famous sites," she told Reuters.

"But I've never been so moved because this is the discovery of something special."

A laboratory will now try to determine the couple's age at the time of death and how long they had been buried.

tomder55 answered on 02/12/07:

According to the opera ; Aida princess of the Ethiopians hid in the crypt that her lover Radames, Captain of the Egyptian Guard was condemned to be buried alive in .Radames says 'Morir si pura et bella'('to die so pure and lovely') to Aida.They bid farewell to earth and its sorrows, and await the Dawn.

tropicalstorm rated this answer Excellent or Above Average Answer

Question/Answer
tropicalstorm asked on 02/11/07 - I can't get it

I don't know what I am doing wrong.
I do it JUST like this

ETERNAL EMBRACE

[img s=http://us.i1.yimg.com/us.yimg.com/i/ww/news/2007/02/06/020607embracefull.jpg space ]


Couple still hugging 5,000 years on

Tue Feb 6, 1:28 PM ET

ROME (Reuters) - Call it the eternal embrace.

Archaeologists in Italy have discovered a couple buried 5,000 to 6,000 years ago, hugging each other.

"It's an extraordinary case," said Elena Menotti, who led the team on their dig near the northern city of Mantova.

"There has not been a double burial found in the Neolithic period, much less two people hugging -- and they really are hugging."

Menotti said she believed the two, almost certainly a man and a woman although that needs to be confirmed, died young because their teeth were mostly intact and not worn down.

"I must say that when we discovered it, we all became very excited. I've been doing this job for 25 years. I've done digs at Pompeii, all the famous sites," she told Reuters.

"But I've never been so moved because this is the discovery of something special."

A laboratory will now try to determine the couple's age at the time of death and how long they had been buried.

tomder55 answered on 02/11/07:

replace the [ with < ,the ] with > and don't type in space just leave a space



tropicalstorm rated this answer Excellent or Above Average Answer

Question/Answer
tropicalstorm asked on 02/10/07 - without too much trouble

can someone find a website for the stryker brigade with a picture of a Romeo 4.

I want to know how they are compared to the Abrams

My Lee J broke his wrist right before graduation. Now they are changing him to Fort Riley, Kansas and to a Romeo 4 and now he is not going to be going overseas in March.


Thanks much
worried mom


tomder55 answered on 02/11/07:

The Stryker is one of the more controversial vehicles the Army is using . The criticsm come from the fact that it is not a tracked vehicle .It uses wheels that the driver can make adjustments to the tire pressure to fit the terrain the vehicle is driving on for maximum speed and traction. (for more on the controversy see this )It also has a 'run-flat 'function if necessary .Iraq's many paved roads and very dry climate make a number of criticisms about wheeled vehicles less significant and they are quiet in comparison to tanks .Wheels give the Stryker the speed to engage in high-speed chases through traffic. Strykers can reach speeds in excess of 70 mph.

The Stryker is an armored infantry transport vehicle and not a tank.The armor does make it more survivable than a Humvee, the only vehicle so far the Stryker has been called upon to actually replace. The Stryker provides an improvement on troop mobility and protection.Reports from the field say the Stryker has turned out to be very effective in protecting troops against IED attacks .For Iraq additional "slat" armor was added which looks like a fence around the vehicle.This helps in RPG protection. The fuel tank is internal and it has an automatic fire extinguisher system. It is a more versatile vehicle .It can be armed as needed .There are 10 different variants of the Stryker depending on their mission .

Soldiers and officers who use Strykers defend them as very effective vehicles.

... in more than a dozen interviews, commanders, soldiers and mechanics who use the Stryker fleet daily in one of Iraq's most dangerous areas unanimously praised the vehicle. The defects outlined in the report were either wrong or relatively minor and did little to hamper the Stryker's effectiveness, they said.

"I would tell you that at least 100 soldiers' lives have been saved because of the Stryker," said Col. Robert B. Brown, commander of the 1st Brigade, 25th Infantry Division, Stryker Brigade Combat Team, which uses about 225 Strykers for combat operations throughout northern Iraq. "That's being conservative," he said.





ETWolverine rated this answer Excellent or Above Average Answer
tropicalstorm rated this answer Excellent or Above Average Answer

Question/Answer
tropicalstorm asked on 02/09/07 - NEW twist

Zsa Zsa's husband is claiming he had a long on going affair with Anna Marie, so he could be the father. The
medical examiner is refusing to release Anna Marie's DNA so Now Anna Marie's baby could possibly end up being the most famous whoz'urdaddy baby.

tomder55 answered on 02/10/07:

I think it is sad and shameful that so many people who hang on to unstable celebs like leeches never love them enough to let them know when they are barreling towards a cliff,and attempt to steer them away from the brink. "Prince" (his father bought the title )Frederic von Anhalt is a slimey slug. Last time we heard from him was when he was suing Pfizer claiming that Viagra made him impotent . If that's true then how can he have a long on going affair and be the father ? Besides Zsa Zsa is in a coma ! It never ceases to amaze me how low humans can go.

powderpuff rated this answer Excellent or Above Average Answer
tropicalstorm rated this answer Excellent or Above Average Answer

Question/Answer
tropicalstorm asked on 02/08/07 - Poor anna Nicole

As much as people picked on her she has had it hard lately and now they (ABC) say she just died

tomder55 answered on 02/09/07:

in the case of Smith ,she was being called "the next Marilyn Monroe" although talent wise they cannot compare ,it was strictly looks. Monroe was a singer /actress /model .Smith was a model/stripper who later became an actress ,,,sorta .Her reality tv show became a cult hit.

Both had their celebrity statues enhanced by who they married .

By all appearances it looks like her life ended just as tragically . Intitial reports are saying something about perscription drugs.

tropicalstorm rated this answer Excellent or Above Average Answer

Question/Answer
tropicalstorm asked on 02/09/07 - Vote against Rep/dem

Michael savage says if he gets enough votes he will consider running for President. Even if you do not like him it might be worth the vote to get him to run to get the rep's and dem's to see just how tired the American public is getting with the way things are. And we really can't get much worse can we?

He has almost a half a million votes already
You can't vote twice cause it won't go through a second time.

http://www.homestead.com/prosites-prs/index.html

tomder55 answered on 02/09/07:

the best thing about him running is that it would get Weiner nation off the air . He calls the Iraq war a play for oil .I don't see much difference between him and the Daily Kos crowd on certain issues and he's certainly as rude .His comments about handicapped really irk me. Although I agree with alot of his positions ,I find his rhetoric distastefully over the top .In fact he makes Ann Coulter sound tame.

tropicalstorm rated this answer Excellent or Above Average Answer

Question/Answer
tropicalstorm asked on 02/09/07 - What if?

Hilliary becomes President do you think the china, silverware, antiques and other things they stole from the Whitehouse will return with them?


(Read ETW's reply to my Pelosi post for how
extravagant and expensive the plane she wants is)

tomder55 answered on 02/09/07:

nahh they have already been peddled on Ebay .

tropicalstorm rated this answer Excellent or Above Average Answer

Question/Answer
Closer_To_The_Heart asked on 02/09/07 - FOR ENTERTAINMENT PURPOSES...ENJOY

Somebody from California apparently wrote the top part, but somebody from Texas came back and put them on their butt's at the bottom.

CALIFORNIA:

I can wear sandals all year long.

I go to the Beach - not "down to the shore."


Our chicks are WAYYYY hotter than yours. Well... Miami can hang.


I say "like" and "for sure" and "right on" and "dude" and "totally" and "peace out" and "chill" and "tight" and "bro" and I say them often.


I know what real cheese & avocados taste like.

Everyone smokes weed and its no big deal.

We'll roll up 40 deep when something goes down.

I live next door to Mexicans, but we call them American's!

All the porn you watch is made here, cause we're better and that's how it is.

I don't get snow days off because there's only snow in Mammoth, Tahoe, Shasta, and Big Bear.

I know 65 mph really means 100.

When someone cuts me off, they get the horn and the finger and high speed chase cuz we don't fuck around on the road.

The drinking age is 21 but everyone starts at 14 (legally 18 if you live close enough to the border).

My governor can kick your governors ass.

I can go out at midnight.


You judge people based on what area code they live in, and when asked where you're from, you give your area code.

I might get looked at funny by locals when I'm on vacation in their state, but when they find out I'm from California I turn into a Greek GOD.

We don't stop at stop signs... we do a " California roll" No cop no stop baby!

I can get fresh and REAL Mexican food 24 hours a day.

All the TV shows you "other" states watch get filmed here.

We're the Golden State. Not the Cheese State. Not the Garden State ...GOLDEN!!!

We have In-N-Out (Arizona and Vegas are lucky we share that with them).


I have the most representation in the House of Representatives, which means MY opinion means more than yours, which means I'm better than you [geez.... hahaha].

The best athletes come from here.

*******IF YOU'RE FROM CALIFORNIA, REPOST THIS*******
******IF YOU'RE NOT, GO SIT IN A CORNER AND CRY******

++++++++++++++++++++++++++++++++++++++++++++

TEXAS :

Ahem... So.. Um.. yeah... I read this, and thought I would reply...

Hey... California, listen up... Texas is where its at!

I too can wear sandals all year long... plus I can put on boots to stomp your toes and I won't even stick out.

You may be able to go to the "beach" instead of the "shore"... but can you go to the drive thru "Beer Barn?" What now surfer boy?

Your chicks aren't way hotter than ours... they are almost equal... and that's only due to silicone, saline, Botox, lasers and hair dye... We have the real ones and they can beat yours up.

We're taught to say "Yes Sir" and "Yes Ma'am" and respect our elders because of it. We also say "Howdy" and "fixin" and "Y'all" which are pretty much recognized right away anywhere in the world :) We're famous.

You may know what real cheese and avocados taste like... but I know what 100% Grade A Angus Beef tastes like. Who wants avocados and cheese when you can have steak and potatoes?

Ha Ha ... who do you think grows the weed and sells it to you?

Why roll 40 deep when something goes down if 5 corn fed country boys can get the job done...

I live next door to Americans, but we call them Mexicans.

About your Porn.... 3 words... "Debbie Does Dallas "... You can brag about it now, but we started it.

Why would you brag about not getting snow days off?

We're smart enough to know 65mph means 65, but our speed limit is 70.

When someone cuts me off, they get run over by my big ass truck, then I give them the finger and tell them to go back to California.

The drinking age is 21, but if you aren't chasin' the beer by 1 yr. old... you're behind.


Yeah, Well my governor became the President of the United States … yours isn't even eligible.

You can go out at midnight? That's nice, I haven't even come home by then.

Ok... you said, "You judge people based on what area code they live in, and when asked where you're from, you give your area code" and as hard as I try I have no idea what you're talking about... I think you're watching too much TV.

Yeah, you'll definitely get looked at funny when you come to visit but we have another name for you pretty boys, and its not Greek, its French.

Of course you don't stop at stop signs... none of you can drive.

You can pick up Real Mexican food 24 hours a day huh... well I can swing by home depot and pick up 24 Real Mexicans anytime of day. Can you say catering?

All the TV shows get filmed there... but where does your favorite poker game from? Texas Hold'em anyone? Besides, we've got Walker Texas Ranger. Chuck Norris knows where it's at! LOL. (I had to add something 'bout that! LMAO)

You can keep your golden state... We're the Lone Star State ...the one and only!! Not to mention we are the ONLY flag that can fly at the same level as the United States flag. Everyone else is beneath it.

Do I have to remind you about the drive thru Beer Barn again? Does In-N-Out serve alcohol? (Oh and did I mention Dr. Pepper was created in Texas?)

You guys have the best athletes huh?... Nine words... Lance Armstrong and The University of Texas at Austin.

Though I could mention MICHAEL JOHNSON - Olympic Sprinter, World record holder in 200m and 400m, 5 Olympic Gold metals, 9 time World Champion (born Dallas, Tx ).

Oh and remind me again who won the Rose Bowl between USC and Texas ????? I believe it was the LONGHORNS!!!!!!!!!!!!!!!!!!

Football is a religion, not a sport.

90% of football "movies" you guys are making are about Texas Football.

Texas is the only state that can still separate to become its own country. The only way California's gonna accomplish that is if another earthquake comes along and you guys sink into the ocean. Can you say Atlantis... HaHaHa.

Come on Texans Show Your Colors! Repost!

And as the Great Sam Houston once said " Texas could survive without the United States , but the United States could not survive without Texas !!"

tomder55 answered on 02/09/07:

I'm from NY ...you talkin to me ? F~~`off

real cheeze is mutsarella you don't like it ,get outta my restaurant . no one eats avacados here. What kinda pu**y worries about snow ? We love football here but none of that whimpy college and high school stuff ....and we don't do the wave . We don't live anywhere near Mexico but we let them mow our lawns. We had a mayor that is going to be President .The govenor of Kalifornia can't even take his wife he is so whipped.and we don't need your porn .we got Britany partying around town with no underwear on.

Closer_To_The_Heart rated this answer Excellent or Above Average Answer
ETWolverine rated this answer Excellent or Above Average Answer
Itsdb rated this answer Excellent or Above Average Answer
tropicalstorm rated this answer Excellent or Above Average Answer

Question/Answer
HANK1 asked on 02/09/07 - BAGHDAD & LOS ANGELES:


Both cities are about 81 square miles. BAGHDAD has the Tigres River and Los Angeles has the Pacific. The Iraq war has been going on for five years. Do you think our troops could 'take over' Los Angeles in five years?

HANK

tomder55 answered on 02/09/07:

If you armed the gangs of L.A. with the weaponry the insuregents have and the gangs were committed to a fight to the death ,I imagine L.A. would be a tough nut to crack given the restrictive rules of engagement that have been used to date. See this latest report from L.A. Looks to me like Mayor Antonio Villaraigosa is planning a new surge [political opponents are questioning the strategy and calling on the mayor to retreat to the suburbs of San Bernardino and let the 'Crips' and 'Bloods' insurgents fight it out .But where is the non-binding resolution to pull the police from these dangerous areas? ].

L.A. already spends about a billion dollars fighting gangs. The mayor has called for an INTERNATIONAL EFFORT to curb gangs in L.A. I'm suprised that the MSM hasn't started to refer to it as a civil war. Let's call for a commission to develop a comprehensive plan ....the L.A. Surrender Group !

HANK1 rated this answer Excellent or Above Average Answer
Itsdb rated this answer Excellent or Above Average Answer

Question/Answer
Itsdb asked on 02/08/07 - A message from John Kerry

As you know I'm not leading a campaign for the presidency in 2008. Instead I have chosen to campaign to end the war in Iraq and protect America.

Yesterday I stood up with a remarkable group of Iraq war veterans who are speaking out because they believe the best way to support the troops is to change a course that squanders their lives. When brave patriots suffer and die because of the incompetence of mere politicians, the only patriotic choice is to demand change.

These veterans offered a profile in courage.

The Senate this week provided a profile in politics -- Republicans blocking even a vote up or down, one way or another on a bi-partisan resolution opposing the Bush escalation.

This has to end.


Republicans refuse even to go on record over the Bush escalation. We need to escalate the pressure for a policy change.

That's why I am introducing legislation that will again set a firm one-year deadline for the redeployment of most American troops from Iraq...

~~~~~~~~~~~~~~~~~~~~~~~~~~~~~~~~~~~~~~~~~~~~~~~~~~~~~~

Let me get this straight. In Kerryland, ending the fight against terrorists is protecting America, and we have to stop blocking congress from discouraging our troops and encouraging the enemy. Is that what he's saying?

tomder55 answered on 02/09/07:

I see that even though he is not running ,his mouth still is ...why is this guy relevent anymore ?


I wonder if he lead these courageous vets in tossing someone else's medal away .(or was that ribbons ? ) "I bet they won't follow him in driving someone elses SUV;marrying someone elses wife ;inheriting someone elses money .

Were these those poor dumb people who couldn't get a good education ?





John Kerry walks into a bar...Bartender says, ‘Hey, Senator Kerry, why the long face?'


John Kerry, a Senate loner, endures the peculiar pariah status that his party reserves for its losing nominees.

“Who will the windsurfing caucus back now?” a columnist for The Boston Herald wisecracked. A cartoonist for The Times-Picayune of New Orleans mocked Mr. Kerry as saying, “Actually I was for another White House bid before I was against it.”



I like the Republican's idea. If you are opposed then go on the record and vote for a binding resolution. The Democrats know that is completely irresponsible ...even for them . But if they don't do something then Cindy Sheehad may shut down one of their press conferences again ,and the moonbats at Kos may howl.


John Edwards being free of the yoke of a responsible position is leading the whole contingent of Democrat hopefuls toward the left. They will make McGovern look like Barry Goldwater by 2008 .

You can see Kerry's *detailed* plan at seta deadline.com All it really amts. to is a rehash of the worse ideas of the Iraq surrender group .

Itsdb rated this answer Excellent or Above Average Answer

Question/Answer
Itsdb asked on 02/08/07 - I'm this has something to do with...

global warming.

Upstate New York towns buried under 6 feet of snow

    20 deaths now blamed on slick roads, cold
    Associated Press

    While the northern Plains and Northeast shiver in dangerously cold temperatures, the folks in upstate New York are keeping warm shoveling snow – lots of snow.

    Since Sunday, the small towns of Parish and Mexico have recorded more than 6 feet of snow, and forecasters with the National Weather Service say it isn’t over yet.

    Another 2 feet or more of heavy lake effect snow was expected Thursday for the communities along eastern Lake Ontario, and more squalls are likely through the weekend.

    “We’re just trying to keep up. It’s almost an unreal amount,” said Mayor Randy Bateman of Oswego, where 70 inches of snow had fallen by Thursday morning. “We catch up when it stops, but then it just comes again, even heavier.”

    Whiteout conditions – the snow has been falling at a rate of 5 inches an hour at times – forced state police to temporarily close Interstate 81 between Central Square and Pulaski, a stretch of about 15 miles.

    Travel advisories against unnecessary travel were posted for Oswego and its neighboring counties. Mexico officials renewed a snow emergency declaration, and many government offices were closed.

    Schools were closed for a fourth day in Oswego and Mexico.

    In West Virginia, where as much as 9 inches of snow has fallen, some schools that had been closed were able to reopen on Thursday, but in most of the state, classes were still delayed, and in a few counties, canceled. Officials had to call snowplow drivers out of retirement Wednesday to clear the roads.

    The weather also disrupted travelers, leaving some stranded overnight in airports in the Midwest after flights to the Northeast were disrupted.

    Temperatures in the Northeast were inching back up to something closer to normal for this time of year, but the upper Midwest and northern Plains still awoke to subzero temperatures Thursday – minus-12 in Minneapolis and 3 below zero in Chicago.

    The bitter cold and slippery roads have contributed to at least 20 deaths – five in Ohio, four in Illinois, four in Indiana, two in Kentucky, two in Michigan, and one each in Wisconsin, New York and Maryland, authorities said. Three of them died Tuesday when two SUVs crashed on a slick road in Kosciusko County, Ind. An autopsy Wednesday determined that an elderly woman found in a New York City building had died of hypothermia.

    In Oswego, a big concern was keeping the city’s 800 fire hydrants clear, said Fire Chief Ed Geers.

    “We’re just trying to keep on top of digging out the hydrants. When you get 5 feet of snow in 24 hours, it’s tough,” Geers said.


I take it that six feet of snow is not normal in upstate NY.

tomder55 answered on 02/09/07:

not all at once ...but it is not unusual for the Buffalo area to get a large amt. of lake effect accumulation .Lake effect snow can be fierce at times.





But the scientist have come up with a long range prediction for weather in my area (although there is hardly a concensus ) . The Alberta Clipper that has plunged our area in sub-freezing temps has had the bonus of keeping precipitation to the south . But it is expected to warm up . One forecaster yesterday called for heavy accumulation of snow on Tuesday . But Yahoo weather is only showing a light snow Tues.

One has to wonder how they know what will happen in 50 years ????????

Itsdb rated this answer Excellent or Above Average Answer

Question/Answer
HANK1 asked on 02/09/07 - COMPARISON:



Are the World War II kamikaze pilots equivalent to the suicide bombers in Iraq and elsewhere? Some weird stuff, huh?

HANK

tomder55 answered on 02/09/07:

not really . The kamikaze attacked military targets ..not wedding parties ...not shoppers in the market ... not pizza parlors... not children going to and from school . Suicide bombers is too kind an expression ...better to call them %^@"#*%ing homicide bombers !!!!!!!!

HANK1 rated this answer Excellent or Above Average Answer
Itsdb rated this answer Excellent or Above Average Answer

Question/Answer
Itsdb asked on 02/08/07 - The Troops Also Need to Support the American People

By William M. Arkin

    I've been mulling over an NBC Nightly News report from Iraq last Friday in which a number of soldiers expressed frustration with opposition to war in the United States.

    I'm sure the soldiers were expressing a majority opinion common amongst the ranks - that's why it is news - and I'm also sure no one in the military leadership or the administration put the soldiers up to expressing their views, nor steered NBC reporter Richard Engel to the story.

    I'm all for everyone expressing their opinion, even those who wear the uniform of the United States Army. But I also hope that military commanders took the soldiers aside after the story and explained to them why it wasn't for them to disapprove of the American people.

    Friday's NBC Nightly News included a story from my colleague and friend Richard Engel, who was embedded with an active duty Army infantry battalion from Fort Lewis, Washington.

    Engel relayed how "troops here say they are increasingly frustrated by American criticism of the war. Many take it personally, believing it is also criticism of what they've been fighting for."

    First up was 21 year old junior enlisted man Tyler Johnson, whom Engel said was frustrated about war skepticism and thinks that critics "should come over and see what it's like firsthand before criticizing."

    "You may support or say we support the troops, but, so you're not supporting what they do, what they're here sweating for, what we bleed for, what we die for. It just don't make sense to me," Johnson said.

    Next up was Staff Sergeant Manuel Sahagun, who is on his second tour in Iraq. He complained that "one thing I don't like is when people back home say they support the troops, but they don't support the war. If they're going to support us, support us all the way."

    Next was Specialist Peter Manna: "If they don't think we're doing a good job, everything that we've done here is all in vain," he said.

    These soldiers should be grateful that the American public, which by all polls overwhelmingly disapproves of the Iraq war and the President's handling of it, do still offer their support to them, and their respect.

    Through every Abu Ghraib and Haditha, through every rape and murder, the American public has indulged those in uniform, accepting that the incidents were the product of bad apples or even of some administration or command order.

    Sure, it is the junior enlisted men who go to jail. But even at anti-war protests, the focus is firmly on the White House and the policy. We don't see very many "baby killer" epithets being thrown around these days, no one in uniform is being spit upon.

    So, we pay the soldiers a decent wage, take care of their families, provide them with housing and medical care and vast social support systems and ship obscene amenities into the war zone for them, we support them in every possible way, and their attitude is that we should in addition roll over and play dead, defer to the military and the generals and let them fight their war, and give up our rights and responsibilities to speak up because they are above society?

    I can imagine some post-9/11 moment, when the American people say enough already with the wars against terrorism and those in the national security establishment feel these same frustrations. In my little parable, those in leadership positions shake their heads that the people don't get it, that they don't understand that the threat from terrorism, while difficult to defeat, demands commitment and sacrifice and is very real because it is so shadowy, that the very survival of the United States is at stake. Those Hoovers and Nixons will use these kids in uniform as their soldiers. If it weren't about the United States, I'd say the story would end with a military coup where those in the know, and those with fire in their bellies, would save the nation from the people.

    But it is the United States, and the recent NBC report is just an ugly reminder of the price we pay for a mercenary - oops sorry, volunteer - force that thinks it is doing the dirty work.

    The notion of dirty work is that, like laundry, it is something that has to be done but no one else wants to do it. But Iraq is not dirty work: it is not some necessary endeavor; the people just don't believe that anymore.

    I'll accept that the soldiers, in order to soldier on, have to believe that they are manning the parapet, and that's where their frustrations come in. I'll accept as well that they are young and naïve and are frustrated with their own lack of progress and the never changing situation in Iraq. Cut off from society and constantly told that everyone supports them, no wonder the debate back home confuses them.

    America needs to ponder what it is we really owe those in uniform.
    I don't believe America needs a draft though I imagine we'd be having a different discussion if we had one.


I feel like the guy that commented on this column who wrote, "what I would really like to say is...not (or should not be) printable."

So our "confused," "mercenary" troops who are fighting not only for us and the Iraqis but their own lives, that are rightfully frustrated with the obscene, disgusting rhetoric coming from the war critics just need to be taken aside and given a good talking to before going back to base to enjoy their "obscene amenities?"

Whatever, says Mr. Arkin, who is in his response to the critics of his column said "I intentionally chose to criticize the military and used the word (mercenary) to incite and call into question their presumption that the public had a duty to support them. The public has duties, but not to the American military."

The good news I suppose is this poor little victim is taking a recommended hiatus from his column ... while giving his critics the one-finger salute of course.

tomder55 answered on 02/08/07:

this Arkin jerk goes over the top ...even for an MSM moonbat. I would ask this AH when did the military personnel check in their right to free speech ? I actually wish it was the standard that some public servants who are working on my dime were required to keep their big fat mouths shut .

I blew my top with this line : "Through every Abu Ghraib and Haditha, through every rape and murder, the American public has indulged those in uniform, accepting that the incidents were the product of bad apples or even of some administration or command order."

INDULGE ????????

Are you out of your mind ?!!!!! WE have asked the military to carry the load since 9-11 and the American people have been asked in return to take their shoes off in the airport and to shop so that the economy would recover. We shop and they get shot at . Who's indulging whom ? We've had hundreds of thousands of troops rotate in and out of some of the scummiest places on this earth and through it all there has been a handful of them that could be deemed bad apples . How dare he indict the entire corp!!

"Sure, it is the junior enlisted men who go to jail. But even at anti-war protests, the focus is firmly on the White House and the policy. We don't see very many 'baby killer' epithets being thrown around these days, no one in uniform is being spit upon."

I wonder what Pfc. Joshua Sparling would say about that ? I wonder what Guardsman Alexander Powell would say ?
Principled opposition to American foreign policy is one thing, assaulting brave soldiers who are carrying it out is another.

"So, we pay the soldiers a decent wage," ..."take care of their families, provide them with housing and medical care and vast social support systems and ship obscene amenities into the war zone for them, we support them in every possible way, and their attitude is that we should in addition roll over and play dead, defer to the military and the generals and let them fight their war, and give up our rights and responsibilities to speak up because they are above society?"

I was a navy brat as a child ,my uncle spent his whole adult life until he retired in the navy . My brother was a marine and my cousin is a career officer in the Army . He is on his 3rd tour in Iraq dating back to 1990 .He has a wife and 2 young children and I assure you they struggle on the pittance that this clown claims that we are "taking care "of his family with . I am frankly amazed that he hasn't cashed in yet and taken a big buck defense contractor position. I can only conclude that he believes in what he is doing ;sacrificing for something bigger than himself . My guess is that the pfc ,given the amt. of hours worked in very dangerous places is most likely getting less than a Walmart bagger.

He dare ask what we owe these soldiers ? How about respect and a thank you for a start ! I call on NBC so sack his sorry ass now !

Itsdb rated this answer Excellent or Above Average Answer
kindj rated this answer Excellent or Above Average Answer

Question/Answer
tropicalstorm asked on 02/07/07 - pelosis demand or else

she says she will cut military funds
The Bush administration has agreed to provide House Speaker Nancy Pelosi with regular access to an Air Force passenger jet, but the two sides are negotiating whether she will get the big aircraft she wants and who she may take as passengers, according to congressional and administration sources.
A congressional source said that Rep. John P. Murtha, chairman of House Appropriations subcommittee on defense, which controls the Pentagon's spending, has telephoned administration officials to urge them to give the speaker what she wants.
The congressional source said Pentagon officials complained that Mr. Murtha, Pennsylvania Democrat, is accusing them of sexism for not immediately heeding her request.
Megan E. Grote, Mr. Murtha's press secretary, said, "Mr. Murtha absolutely never said anything about being 'sexist.' We have no further comment."
Meanwhile, Republican Conference Chairman Adam Putnam of Florida said Mrs. Pelosi's request represents "an arrogance of office that just defies common sense" and called it "a major deviation from the previous speaker."
Minority Whip Roy Blunt of Missouri called it a "flying Lincoln Bedroom," and Rep. Patrick T. McHenry, North Carolina Republican, labeled the speaker's plane "Pelosi One."
"This is a bullet point to a larger value -- Pelosi's abuse of power continues," Mr. McHenry said yesterday. "It began when the speaker denied minority rights to Republicans, continued with her 'TunaGate' scandal, and now she's exploiting America's armed forces and taxpayers for her own personal convenience."

The congressional source said government lawyers are trying to reconcile Mrs. Pelosi's request with Defense Department policy and congressional travel rules.
The Washington Times first reported last week that Mrs. Pelosi's staff was pressing the administration for access to Air Force aircraft. Sources said the request went beyond what was offered to former House Speaker J. Dennis Hastert, Illinois Republican.
Mr. Hastert used an Air Force commuter-type jet to travel to and from his district. Mr. Hastert gained the access for security reasons after the September 11, 2001, attacks. Previously, the House speaker, who is second in the line of succession to the presidency, used commercial flights for such trips.
Mrs. Pelosi wants a larger aircraft that can fly to her home district of San Francisco nonstop. She also wants to be able to ferry other members of the congressional delegation, family members and her staff.
The speaker's request is being handled by her chief counsel, Bernard Raimo, a veteran Democratic lawyer on Capitol Hill.
"Who she can take is being worked out, outside the Air Force," said Ed Gulick, an Air Force spokesman at the Pentagon.
He said the Air Force is studying what types of planes are available for long, cross-country flights. Currently, three planes assigned to the 89th Airlift Wing at Andrews Air Force Base can make such nonstop flights year-round -- the C-32, C-40B and C-37.
Such VIP planes are in high demand.
"She's effectively taking a bird out of the fleet," said a defense source. "It will most directly impact the House, because they're the heavy users of the large aircraft. Congress looks at that Andrews fleet as their Hertz rent-a-car."
The congressional source said the speaker's office requested an Air Force plane to take her to a weekend Democratic retreat in Williamsburg, but the Pentagon declined.
The source said Mr. Hastert on one occasion used an Air Force plane for such an event. The Air Force later determined it was a mistake, and such flights were not repeated.
The source said the Pentagon will likely give in to Mrs. Pelosi's requests for a large plane and travel entourage, given her and Mr. Murtha's power over defense spending.
Mr. Raimo argues that Mrs. Pelosi needs a military aircraft, as opposed to commercial flights, for security reasons.
The defense source, who asked not to be named, termed her request "carte blanche," saying she wanted a plane that could carry an entourage just like President Bush, who flies on Air Force One, and Vice President Dick Cheney, who also always flies on military planes.

tomder55 answered on 02/08/07:

Republicans are not immune from taking a hit on this issue . After 9/11, the Speaker of the House was given access to military transport for security reasons.

Dennis Hastert was the first Speaker given access to military planes for travel .He flew on a plane that seated 12 and was manned by five. But he usually flew to his district commercially and that appears to be the big difference here (the article above is mistaken on that account ).

Pelosi's argument is that because her district is further away ,she needs a larger plane because of refueling issues. Let her take a commercial plane for politicking in her district . That should not be the military's responsibility .

Further ;The Washington Times, which originally broke this story, reported that Pelosi was interested in securing the jet to provide flights for her relatives and other members of the California delegation as well. This stinks of the very abuses of power that Pelosi swore to clean up.

The military has a specific directives that cover this ( Directive 4515.12 ).

Requests should be in "primary interest to Department of Defense" and on an "economical basis" with "prudent utilization of Department of Defense transportation resources."Approval is to be granted when "necessary to carry out the duties and responsibilities of the Department of Defense."Travel should only be granted, "when the purpose of the travel is of the primary interest to, and bears a substantial relationship to the programs or activities of the Department of Defense."The directive also stipulates in the law that military provided travel service should not "compete with United States commercial sea, air or land transportation when that transport exists and its use it not inconsistent with the primary purpose of the travel."

It appears that most of the travel that Pelosi envisions is excessive and abusive and violates the directive. This directive was passed in 1974 and could have been modified since then ;but I doubt it. It sounds more like it was a courtesy granted by the President that needs to be reviewed.










ETWolverine rated this answer Excellent or Above Average Answer
tropicalstorm rated this answer Excellent or Above Average Answer

Question/Answer
ETWolverine asked on 02/07/07 - A MUST READ FOR ALL AMERICANS

Excerpts from a speech by Joe Leiberman to Congress. This is must-read material for anyone who questions the war in Iraq. THIS is the speech that Bush should have given.

New York Post

A BATTLE OF WILLS
By JOE LIEBERMAN

February 7, 2007 -- EDITOR'S NOTE: The following is adapted from Sen. Joseph Lieberman's remarks to the Senate Monday on the Warner-Levin resolution to condemn the president's new Iraq strategy.

OUR nation has reached a critical crossroad in the war in Iraq. A new course has been chosen; a new commander is in place - and a new strategy has begun to be put into action on the ground by our troops.

It is altogether proper that we debate our policy in Iraq. It should be a debate that is as serious as the situation in Iraq and that reflects the powers the Constitution gives to Congress in matters of war. But that is not the debate this resolution invites.

The resolution before us won't stop the new strategy from going forward. Instead, its sponsors say, it will send a message of rebuke from the Senate to the president, from one end of Pennsylvania Avenue to the other. But there is a world beyond Pennsylvania Avenue that is watching and listening.

What we say here is being heard in Baghdad by Iraqi moderates, trying to decide whether the Americans will stand with them.

We are being heard by our men and women in uniform, who will be interested to know if we support the plan they've begun to carry out.

We are being heard by the leaders of the thuggish regimes in Iran and Syria, and by al Qaeda terrorists, eager for evidence that America's will is breaking.

And we are being heard across America by our constituents, who are wondering if their Congress is capable of serious action, not just hollow posturing.

This resolution is not about taking responsibility. It is the opposite - a resolution of irresolution.

If you believe that Gen. David Petraeus and his new strategy have a reasonable chance of success, then resolve to support him and his troops through the difficult days ahead. If you believe that this new strategy is flawed or that our cause is hopeless, then vote to stop it. Vote to cut off funds. Vote for a binding timeline for U.S. withdrawal. Have the courage of your convictions to accept the consequences of your convictions. That would be a resolution.

The non-binding measure before us, by contrast, is an accumulation of ambiguities and inconsistencies: at once for the war but also against the war; pledging support to the troops in the field but also washing its hands of what they are doing.

Cynics may say this kind of thing happens all of the time in Congress. They're wrong: If it passed, this resolution would be unique in American legislative history.

I contacted the Library of Congress on this question last week; I was told that, never before, when American soldiers have been in harm's way, fighting and dying in a conflict that Congress had voted to authorize, has Congress turned around and passed a resolution like this, disapproving of a particular battlefield strategy.

We heard from Gen. Petraeus during his confirmation hearing that war is a battle of wills. Our enemies believe they're winning in Iraq today. They believe that they can outlast us; that, sooner or later, we'll tire of this grinding conflict and go home.

That's the lesson that Osama bin Laden took from our retreats from Lebanon and Somalia in the ྌs and ྖs. It is a belief at the core of the insurgency in Iraq, and of radical Islam worldwide. And this resolution - by codifying our disunity, by disavowing the mission our troops are about to undertake - confirms our enemies' belief in American weakness.

What does this resolution tell our soldiers? I know that everyone here supports our troops - but actions have consequences, often unintended. When we send a message of irresolution, it doesn't support our troops. When we renounce their mission, it doesn't support our troops.

Everyone here knows that the American people are frustrated about the lack of progress in Iraq. Everyone here shares that frustration. And as elected representatives of the people, everyone here feels pressure to give expression to that frustration.

But this challenge is one that every democracy in every long, difficult war has had to confront. Nearly a century and a half ago, an American president wrestled with just this problem. It was in the midst of a terrible war - a civil war - in which hundreds of thousands of Americans were fighting and dying to secure the freedom of millions long and cruelly denied it.

"We here highly resolve" - that was Lincoln's message at Gettysburg. It was a message of resolution, of conviction against adversity, of hope against despair, and of confidence in the cause of freedom, which is America's cause.

Today, in the depths of a terrible war, on the brink of a decisive battle for Baghdad, let us have a serious debate about where we stand and where we must go in Iraq. That's the debate we should have - but not the debate that this resolution would bring.
------------------------

Why aren't more people in a position to speak out saying this?

Elliot

tomder55 answered on 02/07/07:

Funny thing ...the only thing I heard on the news about ole Joe was that he wanted a war tax .[let me tweek his idea a little and call instead for war bonds]

I found this little exchange between Warner and Leiberman from the debate about the competing resolutions :

WARNER: I thank the presiding officer. My question to my good friend and colleague is as follows: this debate is well underway. The plans are being discussed. I just inquired at the desk, and the McCain resolution is not filed. Yet, I understood you to say it had been filed. Could you help clarify for the Senate the position on that.

LIEBERMAN: I’d be happy to very briefly. The resolution Senator McCain and I and others have has been prepared, and I gather it has been the subject of negotiation between Senator Reid and Senator McConnell.

WARNER: But it’s not part of the record, so –

LIEBERMAN: No, no, that’s right. The debate going on now is obviously on the —

WARNER: I’ve always been very carefully, I feel very strongly that the Senate should work its will on facts that are out in the open, and I have filed my resolutions one after another at the desk, so all senators can have the benefit. Is that a possibility that we could have the benefit of this resolution?

LIEBERMAN: My dear friend, it’s more than a possibility. It’s a promise.

WARNER: At what time might the promise be executed?

LIEBERMAN: Well, there are copies of it around now, and we’ll get you one. They were publicly distributed on Thursday of last week.

WARNER: I’d be glad to give you my copy, but I’d feel presumptuous for me to address it unless it is properly before the senate.

LIEBERMAN: I thank my friend. The difference, of course, is that ours is as nonbinding as yours, but ours is a statement of support to our troops and benchmarks to the Iraqis.

WARNER: Madam president, I’m clearly — what I’ve read is correct. But I assure you that I forcefully argue that ours is in support of the troops, and there is no suggestion that one is less patriotic than the other, if I may say to my dear friend.

LIEBERMAN: One is not less patriotic than the other, but actions have consequences, as I said during my remarks. For the Senate to take this unprecedented action on a nonbinding resolution to disavow, disapprove a mission that our troops are being asked to carry out right now cannot help their morale.


Joe consistently gets it .

Here is a section of a recent New Yorker Magazine article by Jeff Goldberg :

“A lot of Democrats are essentially pacifists and somewhat isolationist,” he told me. He had particular problems with Senator Edward Kennedy’s proposal to deny the President funding for a troop surge, and with an idea recently raised by the senior senator from Connecticut, Christopher Dodd, to cap the number of American soldiers in Iraq. Lieberman was not willing to say whether he would remain a Democrat if the Party cut off funding for the war. “That would be stunning to me,” he said. “And very hurtful. And I’d be deeply affected by it. Let’s put it that way.”

Lieberman’s Democratic colleagues know that if he switched parties they would lose their majority, and so they tend to indulge him, unless they are speaking to reporters off the record. Even when Lieberman defends Bush, which is often, his colleagues avoid criticizing him in public—except when it becomes a bit too much, as it did, apparently, for Hillary Clinton.

Lieberman, after reviewing Petraeus’s testimony, said, “You have also said that you fear that there would be disastrous consequences for Iraq, for the region, for the world economy, for the United States in the war on terrorism if we exit Iraq prematurely.”

“Correct, sir,” the General replied.

Lieberman asked what effect the resolution would have “on our enemies in Iraq.”

Petraeus said that, as a soldier, he had put himself “in harm’s way” to protect the right to free speech, but added, “A commander in such an endeavor would obviously like the enemy to feel that there’s no hope.”

Lieberman, fortified by this response, said, “A Senate-passed resolution of disapproval for this new strategy in Iraq would give the enemy some encouragement, some feeling that—well, some clear expression that the American people were divided.”

“That’s correct, sir,” Petraeus said.

In that case, Lieberman said, he would “make a plea” to his colleagues on Petraeus’s behalf to defeat it. “If, God forbid, you are unable to succeed, then there will be plenty of time for the resolutions of disapproval.”


As Lieberman spoke, Clinton’s mask of equanimity seemed to slip for a moment, until she could assimilate the idea that Lieberman had, in essence, accused the Democratic Party of encouraging America’s enemies.


Well yeah. How else can they possibly explain unanimously confirming Gen. Petreus to the position but not endorsing the mission he designed and is being assigned to undertake ?

ETWolverine rated this answer Excellent or Above Average Answer

Question/Answer
tropicalstorm asked on 02/07/07 - Newest resort

A guy in Washington (state) is now trying to get a petition up where married couples will have to have kids within three years or their spouse will not be entitled to benefits.

tomder55 answered on 02/07/07:

The idea of course is to respond to arguments that homosexual marriages are different because they are barren.Their point is to prove that traditional marriage is not solely about children ... and that if it is not about children, it is about discrimination .

But they miss the point .Any marriage between a man and a woman has the potential to produce a child.Any union between two human beings of the same sex has absolutely zero potential to create human life . That is the only real distinction needed to defend the definition of marriage . Everything else ,is distraction to this basic difference.

What we are getting State by State is a mish-mosh of incompatable laws regarding marriage. As I've noted before ,the 'full faith ' clause of the Constitution demands that contracts in one state be honored in all States . If a marriage contract in Mass. is deemed legal then every state must honor it. The gay marriage folks are building up a solid case to bring before the Supreme Court one of these days. I am sure that the only way to prevent a national recognition of gay marriage is for a Constitutional 'Defense of Marriage ' Amendment to be ratified

tropicalstorm rated this answer Excellent or Above Average Answer
Itsdb rated this answer Excellent or Above Average Answer
labman rated this answer Excellent or Above Average Answer

Question/Answer
tropicalstorm asked on 02/06/07 - Which would you guess

The super bowl ad of the two guys accidently kissing is being banned. Which group do you think wants it banned?

If you already know would that have been your guess?

The gays want it banned because it is homophobic.
NOTE The homophobics didn't have a problem with it
They didn't say they wanted it banned because it was offensive or anything! Can't people just chill and laugh any more without finding problems where they don't exist?

tomder55 answered on 02/07/07:

A man had box seat tickets for the Super Bowl.

As he sits down, a man comes down and asks if anyone is sitting in the
seat next to him. "No," He says. "The seat is empty."

"This is incredible," said the man. "Who in their right mind would have a
seat like this for the Super Bowl, the
biggest sporting event in the world, and not use it?"

The first man says, "Well, actually, the seat belongs to me. I was supposed
to come with my wife, but she passed
away. This is the first Super Bowl we haven't been to together since we
got married."

"Oh ... I'm sorry to hear that. That's terrible.But couldn't you find someone else, like a friend or relative, or even a
neighbor to take the seat?" The man shakes his head.

"No. They're all at the funeral."

tropicalstorm rated this answer Excellent or Above Average Answer
ETWolverine rated this answer Excellent or Above Average Answer
Itsdb rated this answer Excellent or Above Average Answer

Question/Answer
paraclete asked on 02/07/07 - when you should not ask google?

This gives a new meaning to being googled.

Send for the search party, Google gives a bum steer



Stephen Hutcheon
February 7, 2007 - 1:30PM

For most people, the journey from the Shelbourne Hotel at 200 Sussex Street to Google's Sydney headquarters across the road at 201 Sussex Street would be a 30-step, 30-second trip.

But according to Google's new mapping service, the recommended route would see you take a 10.4-kilometre scenic detour that involves crossing the Harbour Bridge twice.

Google calculates that this route would entail an off-peak travel time of 18 minutes. It doesn't tell you that you'd also be up for a $3 bridge toll.

Google Maps Australia, which was launched on Tuesday as a fully functional mapping and search service, purports to offer directions between any two addresses.

But in a quirk, all driving directions queried from points east, south and west of Google's headquarters will suggest the same cross harbour detour - one that involves using the tunnel or the bridge, driving up to Falcon Street in Crows Nest before coming back down across the bridge.

For an eight-kilometre drive from Coogee in Sydney's east to Google's headquarters - a trip that normally takes 15-20 minutes in off-peak conditions - the Google directions suggest a circuitous 17.2-kilometre, 29-minute journey that involves two harbour crossings.

The recommended route would also incur tolls of $4.50 for using the Eastern Distributor and a $3 toll on the southbound journey back across the Harbour Bridge.

There are similar roundabout directions suggested from points in Vaucluse, Kings Cross, Surry Hills, Haymarket, Glebe, Rozelle and many other suburbs.

The blame for the wayward directions appears to lie with MapData Sciences, the Sydney-based company that supplies the mapping data to Google.

It incorrectly locates 201 Sussex Street on the Western Distributor, one street south of where it should be.

A similarly inaccurate set of driving directions is given on the Telstra-run WhereIs.com mapping service, which uses the same data.

"Google Maps Australia is currently a beta product and we welcome feedback as we continue to enhance the product for users in Australia," Google's Carl Sjogreen, a senior product manager for the new service, said.

Beta products are effectively in a testing phase even though they might have been released for public use, as in the case of Google Maps Australia.

tomder55 answered on 02/07/07:

yeah ,the same is true of the Tomtom GPs system. I find I still have to pretty much know where I'm going because otherwise the route tends to be serpentine .

paraclete rated this answer Excellent or Above Average Answer

Question/Answer
Itsdb asked on 02/06/07 - Inconvenient Kyoto Truths

Was life better when a sheet of ice a mile thick covered Chicago? Was it worse when Greenland was so warm that Vikings farmed there?

GEORGE F. WILL
Newsweek

    Feb. 12, 2007 issue - Enough already. It is time to call some bluffs. John Kerry says that one reason America has become an "international pariah" is President Bush's decision to "walk away from global warming." Kerry's accusation is opaque, but it implies the usual complaint that Bush is insufficiently enthusiastic about the Kyoto Protocol's binding caps on emissions of greenhouse gases. Many senators and other experts in climate science say we must "do something" about global warming. Barack Obama says "the world" is watching to see "what action we take."

    Fine. President Bush should give the world something amusing to watch. He should demand that the Senate vote on the protocol.

    Climate Cassandras say the facts are clear and the case is closed. (Sen. Barbara Boxer: "We're not going to take a lot of time debating this anymore.") The consensus catechism about global warming has six tenets: 1. Global warming is happening. 2. It is our (humanity's, but especially America's) fault. 3. It will continue unless we mend our ways. 4. If it continues we are in grave danger. 5. We know how to slow or even reverse the warming. 6. The benefits from doing that will far exceed the costs.

    Only the first tenet is clearly true, and only in the sense that the Earth warmed about 0.7 degrees Celsius in the 20th century. We do not know the extent to which human activity caused this. The activity is economic growth, the wealth-creation that makes possible improved well-being—better nutrition, medicine, education, etc. How much reduction of such social goods are we willing to accept by slowing economic activity in order to (try to) regulate the planet's climate?

    We do not know how much we must change our economic activity to produce a particular reduction of warming. And we do not know whether warming is necessarily dangerous. Over the millennia, the planet has warmed and cooled for reasons that are unclear but clearly were unrelated to SUVs. Was life better when ice a mile thick covered Chicago? Was it worse when Greenland was so warm that Vikings farmed there? Are we sure the climate at this particular moment is exactly right, and that it must be preserved, no matter the cost?

    It could cost tens of trillions (in expenditures and foregone economic growth, here and in less-favored parts of the planet) to try to fine-tune the planet's temperature. We cannot know if these trillions would purchase benefits commensurate with the benefits that would have come from social wealth that was not produced.

    In 1997, when the Kyoto Protocol's essential provisions were known, a "sense of the Senate" resolution declared opposition to any agreement that would do what the protocol aims to do. The Senate warned against any agreement that would require significant reductions of greenhouse-gas emissions in the United States and other developed nations without mandating "specific scheduled commitments" on the part of the 129 "developing" countries, which include China, India, Brazil and South Korea—the second, fourth, 10th and 11th largest economies. Nothing Americans can do to reduce greenhouse-gas emissions will make a significant impact on the global climate while every 10 days China fires up a coal-fueled generating plant big enough to power San Diego. China will construct 2,200 new coal plants by 2030.

    The Senate's resolution expressed opposition to any agreement that "would result in serious harm to the economy of the United States," which the Senate correctly thought Kyoto would do. The Senate said any agreement should be accompanied by "a detailed explanation of any legislation or regulatory actions that may be required to implement" it, and an analysis of the agreement's "detailed financial costs and other impacts" on the U.S. economy.

    The president is now on the side of the angels, having promised to "confront" the challenge of climate change. The confronting is one reason for his fascination with new fuels. (Another reason, he says, is U.S. imports of oil from unstable nations. Our largest foreign source of oil is turbulent Canada. Our second largest is Mexico, which is experiencing turbulence because of the soaring cost of tortillas. They are made from corn, which is ... well, read on.)

    Ethanol produces just slightly more energy than it takes to manufacture it. But now that the government is rigging energy markets with mandates, tariffs and subsidies, ethanol production might consume half of next year's corn crop. The price of corn already has doubled in a year. Hence the tortilla turbulence south of the border. Forests will be felled (will fewer trees mean more global warming?) to clear land for growing corn, which requires fertilizer, the manufacture of which requires energy. Oh, my.

    President Clinton and his earnest vice president knew better than to seek ratification of Kyoto by a Senate that had passed its resolution of disapproval 95-0. Fifty-six of those 95 senators are still serving. Two of them are John Kerry and Barbara Boxer. That is an inconvenient truth.


Comments?

tomder55 answered on 02/06/07:

make them vote on it now ...great idea !




Actually I think that Kyoto is already dead and that Chirac gave us a hint about where the new international "concensus"will be regarding a remedy.....carbon taxes . Write that check out to the UN immediately . They have been looking for an uninterupted flow of revenue unimpeded by the whims of the US, who would demand unreasonable conditions on their funding like reform. That is what LOST (Law of the Sea Treaty )is all about...getting mining and drilling rights direct from the UN . And that is what this global warming concensus is all about. Gee! what a suprise !! All the scientists on the UN payroll are in agreement !!

The more I read about the corn to ethanol conversion the more of a scam it sounds like to me. I initially brought all the hype about it but I'm thinking it may be a panacea and a future boondoggle. I have enough problems with price supports for food items but I can see a little of the logic behind it . But if corn to ethanol was so efficient then I think we would already be there .

Closer_To_The_Heart rated this answer Excellent or Above Average Answer
Itsdb rated this answer Excellent or Above Average Answer
labman rated this answer Excellent or Above Average Answer
paraclete rated this answer Excellent or Above Average Answer

Question/Answer
Itsdb asked on 02/05/07 - Will global warming swamp new stadium?

Phillip Matier, Andrew Ross
Monday, February 5, 2007

    Everyone figures that nightmarish traffic jams are the worst problem the Oakland A's will face if they move to the bayfront wilds of Fremont. But their biggest challenge may be something they hadn't thought of -- global warming.

    The A's want to build their ballpark and mall village on low-lying land west of Interstate 880, less than half a mile from a tidal channel. With ocean levels expected to rise as the globe heats up, the high tides that churn up that channel could turn the A's ballpark into prime waterfront property -- or into soup.

    "You are talking about a meter rise of the sea level by the end of the century (around the bay),'' said Will Travis, executive director of the Bay Conservation and Development Commission, which helps regulate shoreline construction.

    Local effects of the rising waters were the subject of a global warming conference this past week at San Francisco's Yerba Buena Center, hosted by the city's Public Utilities Commission. San Francisco and Oakland airports will be under water if no protective steps are taken, and areas of Silicon Valley that now are near the bay could "look like the Lost City of Atlantis" by 2100, in Travis' words.

    It was widely reported that the Giants' AT&T Park could be vulnerable, but what hasn't been noticed is that the same rising tide could turn the A's Field of Dreams into Field and Stream.

    The team and its fans don't have to wait decades to notice the changes along the local shoreline, Travis says -- they're already starting.

    "It's a combination of a rising sea level, increased rain, high tide and wind -- and (the water) can come up unpredictably,'' Travis said.

    If you don't believe it, Travis suggests you check out the occasionally flooded parking lots of Candlestick Point or the public promenade behind the just-opened Pier 1 1/2 in San Francisco.

    Travis predicts "high value'' properties like the A's stadium will need to be protected by seawalls along the shore and lots of pumps.

    And if that doesn't work?

    Well, said Travis, "One of the things they could do is play water polo.''

    The A's seem unfazed by the warming warning, saying they wouldn't be proceeding with planning for a Fremont ballpark if they thought water was a worry.

    Besides, said team spokesman Jim Young, "a century is a long way off, and I won't be available for comment in a hundred years when it becomes a problem.''


How irresponsible. Why aren't the A's being more considerate of their children and grandchildren's futures? Maybe they just wanted to be like Monster Park (Candlestick) and enjoy a little flooding and muck of their own?

tomder55 answered on 02/06/07:

The general manager of television stations WVII and WFVX, ABC and Fox affiliates in Bangor Maine , Michael Palmer , has told his staff of that when “Bar Harbor is underwater, then we can do global warming stories.” “Until then,”...“No more.”

The email message Palmer sent to his staff during this last summer ended up in the hands of the New York Times. One of Palmer's former staff members confirmed the email message went out during the summer after one of the stations broadcasted a live report from a movie theater in Maine where Al Gore’s movie on global warming, "An Inconvenient Truth," was opening.

Mr. Palmer started his email message by saying "I was wondering where we should send the bill for the live shot Friday at the theater for the Al Gore commercial we aired." He then indicated that he wanted no more stories broadcast on global warming because it isn't local news, and has evolved from a scientific issue into a political one.




Itsdb rated this answer Excellent or Above Average Answer

Question/Answer
paraclete asked on 02/06/07 - Have we played along too long?

At the risk of being vilified by certain dizzens of this Board, I post this article for informed discussion. Is it time to say to Israel and the Arabs, enough is enough?

Denounced, but Jewish dissent grows
Antony Loewenstein
February 6, 2007

Other related coverage
British Jews split over support for Israel

In a recent essay published by the American Jewish Committee, reflecting widely shared attitudes, Jews who criticised Israel and its policies were accused of stirring anti-Semitism. The executive director of the committee said "those who oppose Israel's basic right to exist, whether Jew or gentile, must be confronted".

It is clear, however, that a growing number of concerned Jews in the United States and Britain are no longer staying silent in the face of Israeli policies in Palestine and Lebanon.

A new organisation has just been launched in Britain giving voice to such Jews, for example. Independent Jewish Voices includes prominent British figures such as the historian Eric Hobsbawm and the Nobel Prize-winning playwright Harold Pinter. Hobsbawn told the Independent: "It is important for non-Jews to know that there are Jews who do not agree with the apparent consensus within the Jewish community that the only good Jew is one who supports Israel."

For too long, Jews in many Western nations have shunned and intimidated fellow Jews who speak out against the illegal settlements or the cruelty of the 40-year occupation. However, uncritical allegiance to Israel by its "supporters" is arguably a greater cause of anti-Semitism than the dissent they seek to suppress.

The recent release of a book by the former US president Jimmy Carter, Palestine: Peace not Apartheid, has ignited similar debates in the US. He says occupied Palestine is comparable to apartheid South Africa. Carter told Newsweek "the plight of the Palestinians - the confiscation of their land, that they're being suppressed against voicing their disapproval of what's happening, the building of the wall that intrudes deep within their territory, and the complete separation of Israelis from the Palestinians" - is a guarantee of further bloodshed between Israel and the Palestinians.

His book, a bestseller, generated fierce discussion in the US. Carter wrote in the Los Angeles Times that for the past 30 years "I have witnessed and experienced the severe restraints of any free and balanced discussion of the facts". He blamed the Zionist lobby for its success in bullying politicians and the media into obedience.

Since the book's release, David Horowitz has called Carter a "Jew-hater, genocide-enabler and liar". Alan Dershowitz calls the book "biased" and "indecent". Deborah Lipstadt says Carter is giving comfort to anti-Semites. Some Jewish members of the Carter Centre's advisory board have resigned and rabbis of America's largest synagogue cancelled a scheduled appearance at the centre.

More measured responses have appeared from Israelis, such as the Knesset member Yossi Beilin who wrote in the newspaper Forward that Carter's words "are simply not as jarring to Israeli ears, which have grown used to such language, especially in respect to the occupation". Although he rejects Carter's claims of racism against the occupation - it is "rather a nationalist drive for the acquisition of land" - Beilin says Israel's path almost guarantees turning the Jewish state into an international "pariah". Indeed, soon after the conclusion of the recent Lebanon war, Ha'aretz admitted the existence of an "apartheid regime" in the territories.

Despite Israel's denials of expansionist policy, the Israeli peace group Peace Now says about 40 per cent of settlements have been built on private Palestinian land. Such moves are illegal and do not provide more security as claimed. Furthermore, in towns such as Hebron, hundreds of fundamentalist Jews are allowed to live freely while tens of thousands of Palestinians suffer daily indignities.

The controversy surrounding Carter's book is designed to avoid discussion of such matters. The smears against Carter are similar to the reception of two US academics, John Mearsheimer and Stephen Walt, who released a paper, The Israel Lobby, which was critical of that lobby in the US. It sparked a furore. The Zionist lobby has not realised playing the man no longer works.

While the settlements continue to expand, the Israeli Prime Minister, Ehud Olmert - who recently praised the Iraq war as bringing "stability" to the Middle East - has appointed a far-right extremist as deputy prime minister. Avigdor Lieberman has called for the bombing of Iran and Egypt and the murder of Arab Knesset members who talk to Hamas.

How can a democracy in the heart of the Middle East support a man who campaigns for the forced separation of Jews and Arab in Israel proper and the occupied territories?

Now that a growing number of concerned Jews are raising their voices publicly despite their community's pressure, there is a hopeful sign for more dialogue and thereby wider public understanding.

Antony Loewenstein is the author of My Israel Question, published by Melbourne University Publishing.

tomder55 answered on 02/06/07:

I guess England has their share of self-hating Jews also.

Jimmy Carter is being intellectually dishonest to compare what is going on in Israel to the apartheid regime in South Africa. Apartheid was absolute control over a majority of blacks by a small minority of whites. It was the opposite of democracy. In Israel majority rules. Arabs serve in the Knesset, on the Supreme Court and get to vote for their representatives, many of whom strongly oppose Israeli policies. It is inconcievable that Carter doesn't see the difference . Therefore there has to be something else sinister in his heart that motivates his disinformation .

I believe I posted on John Mearsheimer and Stephen Walt's toilet paper.It is endemic the level of hatred towards Jews that is undelying in their essay. The myth of the Jewish lobby in the US is about as valid as the myth being circulated around the ME that Jews grind up Arabs to make their bread.

What is worse is that it permeates throughout the US and is only occasionally addressed seriously.I heard Pat Buchanan over the weekend insist that Jews are over represented in the US Congress.Former US General and Democrat Presidential wanna-be Wesley Clark , commented recently that pressure for U.S. action against Iran’s nuclear weapons program was coming primarily from “New York money people.” Can you guess which religious/ethnic group he might be referring to? "You just have to read what's in the Israeli press. The Jewish community is divided but there is so much pressure being channeled from the New York money people to the office seekers."

Clark's comments are not simply "anti-Israel" but also anti-semetic .Are Jews now to be stigmatized without consequence as frustration over progress of world affairs seeks scapegoats ...much like what occured in Nazi Germany ?

Closer_To_The_Heart rated this answer Excellent or Above Average Answer
ETWolverine rated this answer Excellent or Above Average Answer
paraclete rated this answer Excellent or Above Average Answer

Question/Answer
kindj asked on 02/05/07 - Calling any and all Constitutional scholars!!!

First, read this oath of enlistment for the U.S. military:

"I, _____, do solemnly swear (or affirm) that I will support and defend the Constitution of the United States against all enemies, foreign and domestic; that I will bear true faith and allegiance to the same; and that I will obey the orders of the President of the United States and the orders of the officers appointed over me, according to regulations and the Uniform Code of Military Justice. So help me God."

Now, notice that defending the Constitution is placed before obey orders, etc. It seems to me that it is that way for a distinct purpose, but maybe not.

In any event, what is the obligation of our armed forces when the Constitution is under attack FROM WITHIN, when it's attacked by the CINC, or Congress, or the SC?

Does the soldier defend the Constitution, or does he obey the orders given by CINC (or whoever), even if those orders are AGAINST the Constitution?

Question 2: I saw no expiration date on that oath; nor did I see one on any separation paperwork when I left. While I know I'm not subject to recall (even if others are), it almost sounds like a "moral obligation" to do my part to ensure American safety. Of course, that's how I treat it ANYWAY, as do thousands of people who have sworn no such oath, but it did kinda get my brain to turning.

This is just a hypothetical "what if" kind of question. I'm not suggesting that we revolt against our government or anything. Just wondering at what point--if any--the soldier's duty to the Constitution outweighs his duty to superior officers, up to and including CINC.

D

tomder55 answered on 02/05/07:

Tough question .I don't think there is a right or wrong answer but the individual's conscience. Sen. Inouye admonished Ollie North during Iran /Contra that it was his duty to refuse to carry out illegal orders .But it became North's interpretation of the legality of his orders that dictated his actions and if he was found wrong he would've paid the price.

John Quincy Adams,said : “Our Constitution professedly rests upon the good sense and attachment of the people " . The founders envisioned an engaged populace. Oath or not;they saw everyone as ultimately being responsible for the preservation of the Constitution .

Mrs. Powell of Philadelphia asked Benjamin Franklin during the constitutional convention "Well, Doctor, what have we got, a republic or a monarchy?" Franklin responded, "A republic, if you can keep it." The deliberations at the convention were purpously held in secret so it is a matter of speculation what horse trading and sticking to principles actually occured behind closed doors(although inevidibly some of them were "leaked " for blatant political purposes).

Which Constitution are we talking about ? The original one or the one that through history by amendment has increased the powers of the Federal Gvt. ;all with the overwhelming consent of the governed mind you....remember how complicated a process it is to amend the Constitution .Yet we have it written in the Constitution that the federal government can tax your income . Gold and Silver were written in as the only base species for the monetary supply . That has obviously changed .So has the franchise that was limited to property owners ...and of course a black was 3/5 a person.

I think what is important to remember is that with or without the Constitution ,basic rights are guaranteed to the individual by the creator no matter what the Constitution and the law of the land says .But ...we pride ourselves as a society where the rule of law applies. 'Is a puzzlement' .

kindj rated this answer Excellent or Above Average Answer

Question/Answer
tropicalstorm asked on 02/05/07 - Can sHrillary do this

legally? She says the oil company has made its biggest profit last year and she wants to take their profit to use for 'smart energy'. Isn't she actually 'taking' their profit if she does?

I heard some things on the radio where democrats were actually admitting they will be raising taxes too.

tomder55 answered on 02/05/07:

John Edwards more than admitted that as President he would raise taxes. He said it yesterday on 'Meet the Press' .A man who made his fortune suing doctors and insurance companies, thereby driving up medical costs and driving doctors out of practice, now calls for public health care, and of course would have to raise our taxes to pay for it.In typical populist demogogery he claimed he would only tax the rich.
(I wonder if he remembers the last time a Democrat-ick Presidential candidate promised he would raise taxes...[Mondale lost the electoral vote in every state in the union except for his home state, Minnesota - which he won by fewer than 3,800 votes - and the District of Columbia. ])

Hillary's statement is an amazing admission and look into the Democrat-ick mind set . It never occures to them that they can't confiscate a companies profits ,or your personal wealth and property for that matter .She more resembles Hugo Chavez than an American politician.Chavez recently announced that he is confiscating the Venezuelan media, telecommunications companies, oil companies, banks etc. and placing them under his personal control.


President Bush has been proposing alternative energy legislation since his first administration. The difference is that the president's program does not include the theft of oil companies' profits .But last week he fell into the populist clap-trap when he went to Wall Street and scolded them for executive compensation . Between him and the Democrat-icks in Congress there will be wage floors ("living wages ") and wage ceilings that Americans will not be permitted to exceed.


At least the oil companies provide heat for our homes during these global warming chilly winters we are having .They provide the fuel for the engine of our economy . Shouldn't we start with profits from things that have no apparent value ?

Let's start with the profits she and Bill Clinton made on books ghost-written for them .Wouldn't they be better served increasing the teachers pay who will work in the schools that the profits from Oprah Winfrey's enterprises generates ?Maybe Hillary should give back the 100x investment returns on cattle futures she made and use it to stock food pantries around the country .

For that matter ,Forbes Mag. has identified many celebrities in the entertainment industry alone who's incomes exceed the compensation packages of most corporate executives . They are given these figures it is argued because they give people what they want . Do the oil companies do any less ?


Perhaps she is not aware that State and Federal taxes are already levied on the oil industry and represent a huge portion of the general gvt. revenues already .

But oil companies like any other business transfer the cost of taxes onto consumers. To put it into perspective ;I paid $1.97 this week for gas in N.J. Much of the cost covered State and Federal taxes the oil industry has to pay. I also bought a gallon of milk for $3.25 ...now get this .... the dairy industry is a beneficiary of price supports . That means my tax money is also going to the diary companies to prop up the price of the milk I buy .I am paying a higher price for a gallon of milk and paying even more to subsidize their industry . So who does Hillary think is screwing the American consumer more ?

Itsdb rated this answer Excellent or Above Average Answer
labman rated this answer Excellent or Above Average Answer
tropicalstorm rated this answer Excellent or Above Average Answer

Question/Answer
paraclete asked on 02/03/07 - some good arguments, but only arguments?

At last we have someone who can put the arguments suscintly
"The first is based on saying that scientists are wrong."
"The answer:
"To do nothing is to run a risk that we cannot afford to take,"

" even if the science was accepted, mankind could easily cope with rising temperatures."
The answer:
"That is an irresponsible position"

"the consequences of global warming is a remote, long-term problem"
The answer:
"indefensible from an ethical view".

So fellow campers (skeptics all) it's time to come out from under the covers, the sky is falling and there is no where to hide, not even inside your head.

We've wrecked the weather, Stern says



February 4, 2007


SIR Nicholas Stern, author of a major report on the economic impact of global warming, says the latest review of the scientific evidence by United Nations' experts has demolished the chief argument of so-called climate sceptics.

"I have heard three kinds of argument claiming that it is not necessary to combat climate change," Sir Nicholas told a conference in Paris on Friday.

"The first is based on saying that scientists are wrong. After the report of the IPCC [UN's Intergovernmental Panel on Climate Change] released today, this position is untenable," the former World Bank chief economist said.

The assessment by the IPCC said global warming was almost certainly caused by humans, and carbon pollution disgorged this century would disrupt the climate system for a thousand years.

"To do nothing is to run a risk that we cannot afford to take," said Sir Nicholas, as he called for "urgent, resolute and concerted" action in Europe and the international community as a whole to tackle the challenge.

He also rejected the argument that, even if the science was accepted, mankind could easily cope with rising temperatures.

"That is an irresponsible position, because it does not take into account the real risks linked to a very high rise in temperatures, for example in the case of a world where temperatures rise by five or six degrees," he said.

And those who dismissed the consequences of global warming as a remote, long-term problem were "indefensible from an ethical view".

In a report commissioned by the British Government last year Sir Nicholas warned that without urgent action the fallout of climate change could be on the scale of the two world wars and the Great Depression of the 1930s.

Singling out current and rising economic powerhouses the United States, China and India, he said the world must be prepared to pay now - in the form of green taxes or emissions trading schemes - to prevent economic disaster.

The IPCC, the UN's paramount scientific authority on global warming, predicted Earth's surface temperatures would rise between 1.8 and 4 degrees by 2100. It described this as a "best estimate" within a range of 1.1 to 6.4 degrees.

Meanwhile, lawyers said the IPCC report could trigger more lawsuits against big industrial emitters despite hurdles in pinning down blame for floods, droughts or rising seas.

"We're entering a new era," said Audley Sheppard, a partner at Clifford Chance law firm in Britain. He said major emitters of greenhouse gases could no longer argue they were unaware of the risks.

However, greenhouse gases, led by carbon dioxide from burning fossil fuels, mix into the atmosphere, which makes it hard to quantify who is to blame for emissions.
Source: The Sun-Herald

tomder55 answered on 02/04/07:

"global warming - no debate allowed".
Friday morning I awoke to a weather forecast of significant accumulations of snow. At the end of the day I was able to clear my drive way with a kitchen broom. But I guess we can now add 100 year weather forecasts to the list of things mankind has perfected.

I remember how silly I felt on Jan.1 2000 that I had purchased some extra cases of bottled water for the inevidible chaos that was going to follow Y2K.

BTW . We need to get Marvin the Martian to stop driving those SUVs.


Oh goody! My illudium Q-38
explosive space modulator

"Global warming" isn't really about science. Its agenda-driven by the folks who favor big government, income redistribution, dismantling our advanced technological civilization and running our lives for our own good.

Itsdb rated this answer Excellent or Above Average Answer
labman rated this answer Excellent or Above Average Answer
paraclete rated this answer Excellent or Above Average Answer

Question/Answer
Itsdb asked on 02/02/07 - What's a nuke or two...

in the hands of a few Jihadists?

Chirac's Iran blunder provides glimpse into nuclear Middle East
by Carole Landry

He may have taken back what he said but Jacques Chirac this week nevertheless became the first western head of state to break a taboo by recognising that Iran may become a nuclear-armed power.

By sketching out a scenario of Iran armed with a nuclear bomb and staring down Israel, the French president gave the world a glimpse of the new balance of power in the Middle East that western governments may be contemplating.

While such musings are commonplace in think tanks and conferences, they have a different resonance coming from the president of France, which has joined the United States and Europe in asserting that Iran will not be allowed to develop a bomb.

"Jacques Chirac said what many experts are saying in the world, even in the United States -- that a country that has the bomb doesn't use it and applies the rationale of deterrence," former foreign minister Hubert Vedrine commented.

Foreign policy expert Pascal Boniface agreed that Chirac "spoke as an expert and not as a head of state" at a time when the United Nations is seeking to put more pressure on Iran to give up its nuclear activities.

"In the official diplomatic discourse, these are things that are just not said," added Boniface of the Institute of International and Strategic Studies in Paris.

In an interview this week to three publications, Chirac minimised the threat posed by a nuclear-armed Iran, saying that Tehran would have to take into the account the fact that it would be "razed to the ground" if it launched a strike on Israel.

"Having one or perhaps a second bomb a little later, well that's not very dangerous," Chirac said in the interview to the New York Times, the Paris-based International Herald Tribune and the French weekly Nouvel Observateur.

"Where would Iran drop this bomb? On Israel?" he asked. "It would not have gone off 200 metres into the atmosphere before Tehran would be razed to the ground," Chirac was quoted as saying by the three publications.

The president retracted his comments, admitting that he had erred, and the Elysee issued a formal statement asserting that France's position was unchanged and that it considered a nuclear-armed Iran unacceptable.
~~~~~~~~~~~~~~~~~~~~~~~~~~~~~~~~~~~~~~~~~~~~~~~~~~~~~~~

Echoes of Reagan ("the bombing begins in five minutes") from Chirac? Sacre bleu! Of course it's not Chirac's fault he said those things, it's a “shameful campaign” by the American news media “using any excuse to engage in France-bashing.”

Perhaps Mr. Chirac and Mr. Biden should get together and start their own country?

tomder55 answered on 02/02/07:

He was partly responsible for the breakdown of the sanction regime against Iraq . It doesn't suprise me at all that he would try something simular with Iran. I say check his wallet....is it filled with Rial ? Chirac making a controversial comment is not necessarily news or a problem. But any cracks in the front against Iranian nuclear weapons (Great Britain, France, Germany, the United States) can only help to steel Iran's resolve. With the Iranian government continuing to defy the UN and the IAEA, comments like this can only exacerbate the problem.

His initial statement was bizzare in it's illogic......even for Chirac. Let's put aside for the moment that his statement went counter to his own ;or of France's stated policy regarding Iran. What he was signaling was a dynamic shift away from non-proliferation towards an extremely dangerous policy of MAD deterance in a region of the world where the word MAD (see below ) is more applicable to some of the leaders. I guess that scenario is easier to contemplate when it involves Tel Aviv and Tehran and not Paris.

"I should rather have paid attention to what I was saying and understood that perhaps I was on the record," Chirac said in the second interview on Tuesday , according to the New York Times.

true but perhaps his off the record reflections were much more revealing . He obviously is not concerned with the welfare of the State of Israel . Yes ......one or two Iranian bombs is about all it would take.

Regarding his comments that Israel could be a target of an Iranian weapon and that Israel would retaliate, Chirac said: "I don't think I spoke about Israel yesterday. Maybe I did so but I don't think so. I have no recollection of that."

I have a hard time believing Chirac was not aware if he was on or off the record. Because he's been involved in politics for decades, that argument seems incredulous.

~~~~~~~~~~~~~~~~~~~~~~~~~~~~~~~~~~~~~~~

Official Iranian radio has completed broadcasting a lengthy series on the imminent appearance of a messianic figure who will defeat Islam's enemies and impose Islamic Shiite rule over the entire world – even speculating on specific dates the so-called "Mahdi" will be revealed.

"Be joyous my heart, miracles of the Messiah will soon be here," reads a poem used to conclude the first broadcast. "The scent of breaths of the One we know comes from near. Grieve not of sorrow and melancholy, as assured I was … last night that a Savior will come, it's clear."





Itsdb rated this answer Excellent or Above Average Answer

Question/Answer
tropicalstorm asked on 02/02/07 - they realized they can't put parents in jail

They realized that if the child protective agency had to run to every single report on incidents of a child getting their hand swatted away from grabbing something or their diaper patted they would not have time or jail space to take care of the serious incidents.
They also realized that a parent going to jail for one year would do more psychological effect on the child than the little swat. They said they have sufficient child abuse laws for the real child abusers.
Someone was looking at things realistically

tomder55 answered on 02/02/07:

Duuuuhhhh .....ya think ? Gotta say ......give em enough time, even California moonbats have a chance to get it.

Lawmaker Retreats on Spanking Ban in California

(CNSNews.com) - Critics of a proposed ban on spanking in California claimed victory Thursday after the lawmakers who sponsored the measure failed to introduce it in the California State Assembly for the second week in a row. Assemblywoman Sally Lieber of San Jose proposed the measure, which would make spanking a child under the age of four a misdemeanor punishable by up to a year in jail, a $1,000 fine, or both. According to reports, Lieber was planning to introduce the bill no later than Thursday. When she didn't, pro-family groups, who had criticized the ban, claimed victory. "This home-invasion bill has been stopped cold by parents and grandparents who know that to love children is to discipline them and show them the way to live," said Randy Thomasson, president of Campaign for Children and Families (CCF). "Because so many people have spoken out, the Democrats in Sacramento realize that their liberal agenda is offending a whole lot of people," he added. "Sally Lieber mistakenly believes that parents who infrequently spank their children should be arrested, and she wrongly guesses that children who were spanked will respond by committing crimes," said Thomasson. "But the obvious truth is that most children who were appropriately spanked become law-abiding citizens, not criminals. Any elected official who supports a ban on spanking is attacking dads and moms and usurping their God-given responsibility to raise their own children.""

http://www.cnsnews.com/ViewFlash.asp?Page=/ThisHour/Archive/NTH20070201a.html

tropicalstorm rated this answer Excellent or Above Average Answer

Question/Answer
HANK1 asked on 02/02/07 - "SHIP OF FOOLS"



"The ship of fools is an old allegory, which has long been used in Western culture in literature and paintings. With a sense of self-criticism, it describes the world and its human inhabitants as a vessel whose deranged passengers neither know nor care where they are going." - Wikipedia

Well, guys and gals, perhaps it's time for HANK to take a hard line re: what's going on in our culture and elsewhere. How about an analogy? Let's call our White House a SHIP and call the politicians who haunt it EARTHLINGS who neither know nor care where WE, the citizens, are going. They're out for themselves. So, maybe our balance of power should be shifted from Washington to ALL State capitols. The United States exemplifies a FEDERAL republic whose central government is suppose to be restricted in power. Sorry! It's not! If each State became a REPUBLIC, a common sense REPUBLIC, said States would be ruled by very inclusive electorates. Reasoning: It's much easier for me to keep my eyes on the head of a pin instead of on a requiem. Smaller the better!

Would it be feasible to have 50 common sense REPUBLICS than what we have now? (Disclaimer: This is nothing but a hypothetical question)

HANK

tomder55 answered on 02/02/07:

Any student of American history understands how chaotic and ungovernable the infant nation was under a confederation. Both Alexander Hamilton (a strong central gvt. advocate ) AND James Madison (a Virgian states righter ) served in the early Congress and realized it was unworkable . They teamed up to promote a Constitutional Convention and co-authored 'The Federalist Papers ' to promote the Federal Reoublic we now have . For any serious student of American history and Constitutional law ,reading 'The Federalist Papers is a requisite .

A reading of the Constitution shows that individual states already have rights . The fact that those rights have to a degree eroded is not the fault of the framers .The States have chosen not to exercise their rights.

Amendment X :
The powers not delegated to the United States by the Constitution, nor prohibited by it to the states, are reserved to the states respectively, or to the people.

HANK1 rated this answer Excellent or Above Average Answer

Question/Answer
paraclete asked on 02/02/07 - The dirty tricks brigade of US business is at it again?


Reward offered to dispute climate report

From correspondents in London

February 02, 2007 12:48pm
Article from: Agence France-Presse

A RIGHT right-wing American think-tank is offering $US10,000 ($12,940) to scientists and economists to dispute a climate change report set to be released in Paris later today by the UN's top scientific panel.

The Guardian newspaper reported today that the American Enterprise Institute (AEI) sent letters to scientists in the United States, Britain and elsewhere offering the payments in exchange for articles emphasising the shortcoming of the UN's report.

AEI, which the paper said receives funding from oil giant ExxonMobil, also reportedly offered additional payments, and to reimburse travel expenses.

The UN report, due to be released today in Paris by the UN's Intergovernmental Panel for Climate Change (IPCC), is likely to give a bleak assessment of the damage to the future of the environment.

It is the culmination of four days of debate between more then 500 scientists at a closed-door meeting in Paris, who have been poring over the first review of the scientific evidence for global warming in six years.

AEI's letters characterise the IPCC report as "resistant to reasonable criticism and dissent and prone to summary conclusions that are poorly supported by the analytical work" and request articles that "thoughtfully explore the limitations of climate model outputs," The Guardian said.

Kenneth Green, the AEI visiting scholar who sent the letters, confirmed to The Guardian that the think-tank had approached scientists and analysts to pen essays that would be compiled into an independent review of the IPCC's report.

"Right now, the whole debate is polarised," Mr Green was quoted as saying by the newspaper.

"One group says that anyone with any doubts whatsoever are deniers and the other group is saying that anyone who wants to take action is alarmist. We don't think that approach has a lot of utility for intelligent policy."

+++++++++++++++++++++++++++++++++++++++++++
there can be no independence if you are paid to write the article

tomder55 answered on 02/02/07:

let me guess.you think the scientists who work for various gvt. agencies in the western world have free reign to independent study ? yeah right. If an environmentalist group pays scientists do you think they would be employed long if they offered a counter hypothesis to the one the organization champions?


"… those who are absolutely certain that the rise in temperatures is due solely to carbon dioxide have no scientific justification. It's pure guesswork."

- Henrik Svensmark, Director of the Centre for Sun-Climate Research, Danish National Space Centre

The UN sees the issue as a means to expand their power over the nations of the world .They long have advocated various international taxes so they would not be beholden to the contributions of the member states .With their track record why should anyone believe them ? ...because they paid scientists to be their mouth pieces ?

Your own positions on the Christianity board express a healthy scepticism toward the sciences. Why do you take their conclusions as gospel in this issue ?....especially when there is nowhere's near the concensus that they would like us to believe.

"There is a significant minority of genuine experts in the field who believe that the Armageddon scenario is grossly oversold, especially by climatologists in pursuit of government funding and research grants. Such dissidents are treated as if they were heretics within an established religion."
- Dominic Lawson, The Independent



http://www.geocraft.com/WVFossils/ice_ages.html

http://www.co2science.org/scripts/CO2ScienceB2C/Index.jsp

ETWolverine rated this answer Excellent or Above Average Answer
Itsdb rated this answer Excellent or Above Average Answer
paraclete rated this answer Excellent or Above Average Answer

Question/Answer
tropicalstorm asked on 02/01/07 - Ever notice?

Ever notice how all the looney liberal women ALL have that same shrill yell?
SHrillary, Jane Fonda, Cindy Sheehan, just to name a few
I cringe every time I hear the conservative radio talk show hosts play anything Cindy sheehan has to say,
Shrillary's 'PULL OUT NOW!' (was she yelling at Bill?)
and the one that screams, "Take your V and put it in a lock box NOW!" (Some sexual quote that I can not remember exactly how it goes).

WHY do they have to sound soooo hysterical and like they are mind controlled or what ever it is?

tomder55 answered on 02/01/07:

add Madame Speaker Mimi to the list
(I was wondering what code Pink actually meant when they chanted "Pull OUT NOW")
I forgot who I mentioned it to but there is a tidal wave of momentum for liberal women in this country that we were not prepared for .

Now there was a time when they
used to say
That behind every - "great man."
There had to be a - "great woman."
But in these times of change you
know
That it's no longer true.
So we're comin' out of the kitchen
'Cause there's somethin' we forgot
to say to you (we say)

Sisters are doin' it for themselves.
Standin' on their own two feet.
And ringin' on their own bells.
Sisters are doin' it for themselves.

Now this is a song to celebrate
The conscious liberation of the
female state!
Mothers - daughters and their
daughters too.
Woman to woman
We're singin' with you.
The "inferior sex" got a new exterior
We got doctors, lawyers, politicians
too.
Everybody - take a look around.
Can you see - can you see - can you
see
There's a woman right next to you.


( Aretha Franklin )




tropicalstorm rated this answer Excellent or Above Average Answer
Itsdb rated this answer Excellent or Above Average Answer

Question/Answer
paraclete asked on 02/01/07 - I'm doing my bit, now how about you?

Feeling totally vindicated, I will go on sequestrating Carbon but what are the rest of you going to do?


No time for never-never solutions
Mike Archer
February 1, 2007

It was suggested recently that if everyone on the planet started gorging themselves on fatty foods, the amount of carbon sequestered could reverse global warming as long as no one did a stitch of exercise other than to produce more butterball humans. It's a tasteless idea, but it does raise some important themes that bear thinking about as scientists gather for the latest diagnosis of the state of the Earth's climate.

It seems pretty clear that the Intergovernmental Panel on Climate Change will tell us the patient is far worse than we thought and that her condition is deteriorating far faster than we thought when it releases its latest report tomorrow.

Naysayers and sceptics can argue all they like about how much of this change is "natural" and how much is the result of human activity: the bottom line, in terms of treating the patient, is that the hotter she gets the less time we have to fix her up. Likewise, our options become more and more limited the longer we stand around like stunned mullets. We need to take action, now.

The trouble is that most of the major solutions being suggested to Australians are of the never-never kind. Whatever the relative merits of carbon sequestration and nuclear energy, for example, they will take decades to develop and decades more to have any serious impact on atmospheric carbon dioxide levels.

Worse still, these prescriptions carry an in-built assumption that we have the luxury of time in which to administer them. We don't.

More disturbingly, we now have plenty of evidence to suggest that swings in the global climate can happen faster than we previously believed. Much faster.

The US National Academy of Sciences' 2002 report Abrupt Climate Change: Inevitable Surprises noted, for example, that although general global warming and a glacial meltdown began about 15,000 years ago, the process came to an abrupt halt about 3000 years later in the span of a couple of decades.

Known as the Younger Dryas event, it featured a rapid, steep drop in global temperature and an abrupt return to full-on glacial conditions for about 500 years. It ended even more abruptly than it began, with a return to global warming that took perhaps as little as one decade.

The Hollywood sci-fi blockbuster The Day After Tomorrow began with a climatologist lecturing thick-headed US politicians about the vital message of the Younger Dryas event that abrupt climate change could happen again just as quickly, with awesome consequences.

For example, if the Greenland and Antarctica icesheets melt (which they are doing in spectacular fashion), sea levels could rise, as they have done many times in the past, by 100 metres. If that were to happen, forget the metre-in-a-century mantra, and forget half of Sydney, along with most of the world's coastal populations. Why climates swing so violently is less relevant than the consequences when they do.

As a palaeontologist and geologist who has studied the history of climate change and its effects on life, it's clear to me from Earth's fossil record that major swings in climate have had massive consequences for living things. Extinctions are the most common outcome.

In short, if we don't want these consequences, we don't have the luxury of time to dither. We must respond now.

I don't have all the answers, but I remind everyone of the 2001 Amsterdam Declaration on Global Change, put together by an eminent group of scientists from four international global change research programs. It pointed out that the dynamics of global systems "are characterised by critical thresholds and abrupt changes" and that "human activities could inadvertently trigger such changes with severe consequences for Earth's environment and inhabitants". Those changes could be irreversible and will be far less hospitable to human life.

The broader message here is that we shouldn't focus on climate change as the only threat looming on the horizon. We need to look as well to the other ways humans are increasingly modifying the planet for their own purposes and question whether we're at risk of crossing other thresholds that may lead, faster than expected, to ugly outcomes.

As the Amsterdam declaration noted, the planet behaves as a single, self-regulating system, with complex interactions and feedback between its component parts.

Humans are influencing environments in many ways, not just the atmosphere but the oceans, fresh water, biological systems and so on. All the signals coming back are that the way we live as a species is not sustainable.

While some might take comfort in the thought that "ugly" will not happen in their lifetime, new studies of thresholds and accelerating rates of change suggest these are problems that will challenge all generations now living on the planet.

The Prime Minister has rightly acknowledged that our way of managing the Murray-Darling Basin has passed its use-by date. That's a step in the right direction. Next, we all have to acknowledge that the same is true of our overall environmental management. We must invest now in environmentally friendly technologies, such as solar hydrogen to produce energy that won't cost the world.

Sooner or later, we're all going to have to cease our collective state of denial and accept that business and technology as usual is not an option. We simply can't keep gorging ourselves on the world's resources (even if 6 billion obese, inactive humans would sequester a lot of carbon). Civilisations exist by the grace of Earth, subject to change without notice. Let's hope we all realise that in time.

Mike Archer is dean of science at the University of NSW.

tomder55 answered on 02/01/07:

100 metres lol ...... here we have the dean of science of a major university quoting a Hollywood fantasy. Guess I'll have to link to all the science that says oceal levels are falling ....( oh this is getting tedious). But I will say that it may very well be that Australia is destined to permanent desertification . The Sahara was once a lush oasis . CLIMATE CHANGE HAPPENS

The tiny country of Tuvalu is not cooperating with global warming models. In the early 1990s, scientists warned that the Pacific coral atoll of nine islands - only 12 feet above sea level at its highest point - would vanish within decades, swamped by rising seas. Sea levels were supposedly rising at the rate of 1.5 inches per year.

However, new measurements show that sea levels have fallen 2.5 inches since that time. Similar sea-level declines have been recorded in Nauru and the Solomon Islands. (London Telegraph, 6 Aug 2000)

Also http://www.bigempire.com/sake/tuvalu1.html

"New perspectives for the future of the Maldives"
Nils-Axel Mörner, Michael Tooley, and Göran Possnert,
Global and Planetary Change, Vol. 40, Issues 1-2, Jan 2004, pp 177-182

http://stephenschneider.stanford.edu/Publications/PDF_Papers/MornerEtAl2004.pdf

From Climatic History of the Holocene, by James S. Aber
http://academic.emporia.edu/aberjame/ice/lec19/lec19.htm

Earth’s climate during the last 1,000 years

Medieval climatic optimum (AD 700-1200).
Medieval glaciation (AD 1200-1460).
Brief climatic improvement (AD 1460-1560).
Little Ice Age (AD 1560-1890).
Modern climatic optimum (AD 1890-2000).


Contrary to all the horror stories one hears about global warming-induced mass wastage of the Antarctic ice sheet leading to rising sea levels that gobble up coastal lowlands worldwide, the most recent decade of pertinent real-world data suggest that forces leading to just the opposite effect are apparently prevailing, even in the face of what climate alarmists typically describe as the greatest warming of the world in the past two millennia or more.

http://www.co2science.org/scripts/CO2ScienceB2C/articles/V9/N45/C2.jsp





Itsdb rated this answer Excellent or Above Average Answer
paraclete rated this answer Excellent or Above Average Answer

Question/Answer
Itsdb asked on 01/31/07 - A Biden Problem: Foot in Mouth

By JAKE TAPPER

WASHINGTON, Jan. 31, 2007 — Senator Joe Biden, D-Del., the loquacious chairman of the Senate Foreign Relations Committee who launched his presidential campaign today, may be experiencing an ailment not entirely unknown to him: foot in mouth disease.

Biden is taking some heat for comments he made to the New York Observer, in which he said of Sen. Barack Obama, D-Ill., a rival for the nomination: "I mean, you got the first mainstream African-American who is articulate and bright and clean and a nice-looking guy. I mean, that's a storybook, man."

Immediately the conservative media establishment — Rush Limbaugh, the Drudge Report, bloggers — publicly pounced. At Townhall.com, Mary Katherine Ham wrote: "A clean black man? The first black guy on the American political scene who can both shower regularly and speak properly? Is that really what Biden thinks? If a Republican had said this, we'd have a national outpouring of grief over the residual ignorance and racial insensitivity in our country, and the guy would be in sensitivity training until around about the time John Kerry is elected president."

Obama Responds

And notably, Obama himself didn't do much to knock the story down.

Asked about the comments at a press conference this afternoon, Obama said, "you'd have to ask Senator Clinton, uh, Senator Biden what he was thinking," initially stumbling by mentioning the name of the Democratic front-runner for the nomination, Sen. Hillary Clinton of New York. "I don't spend too much time worrying about what folks are talking about during a campaign season."

Asked if Biden meant to be complimentary, Obama said, "I'm not going to parse his words that carefully."

In a conference call with reporters about his presidential campaign, Biden acknowledged that he "was quoted accurately" in the New York Observer, but insists his comments are being misunderstood.

"Barack Obama is probably the most exciting candidate that either the Democratic or Republican party has produced at least since I've been around," Biden said. "He's fresh, he's new, he's insightful."

Biden said he regretted that "some have taken totally out of context my use of the word 'clean.'"

"My mother has an expression 'clean as a whistle, sharp as a tack,'" Biden said. "Look, the idea is, this guy is something brand new no one has seen before..."

~~~~~~~~~~~~~~~~~~~~~~~~~~~~~~~~~~~~~~~~~~~~~~~~~~~~~~~

It's a good thing I was pulling into the parking lot when I heard this or I would have had to pull over laughing. I really want to give Biden a pass on this - ok, so I really don't - come on, "the first mainstream African-American who is articulate and bright and clean and a nice-looking guy?"

What would Denzel say? Sidney Poitier? Bryant Gumbel? Lynn Swann? Colin Powell? Rod Paige? Alphonso Jackson? JC Watts? Michael Steele? Clarence Thomas? Alan Keyes, Emmitt Smith...

tomder55 answered on 02/01/07:

We should be thankful that Barack Obama is not like all those other inarticulate,dull ,dirty and ugly African- Americans. I bet he never realized how loaded a word "articulate" is in describing an African-American.


"You don't know my state," he said. "My state was a slave state. My state is a border state. My state has the eighth-largest black population in the country. My state is anything from a Northeast liberal state."

I guess in the state with the eighth largest black population' he never encountered an articulate ,clean ,good looking black man .

I think he should bring his point up next time he meets with Jesse Jackson and Al Sharpton.

For someone who built his entire political persona around the idea that he's bright, knowledgable and articulate, Sen. Joe Biden has proven to be remarkably oafish with regards to racial stereotypes. This is the same guy said "You cannot go to a 7-Eleven or a Dunkin' Donuts unless you have a slight Indian accent ."

No wonder Mark Levin calls Biden the dumbest man in the Senate .






Itsdb rated this answer Excellent or Above Average Answer

Question/Answer
kindj asked on 02/01/07 - You gotta be kidding me!

Franken for Senate?

http://www.msnbc.msn.com/id/16910222/

tomder55 answered on 02/01/07:

He's gotta do something . His airhead America rant radio show was a bust .He was a decent comedian at one time . This may be his best joke ever.

Can he win ?Don't forget Minn. was the state that gave us Govenor Jesse Ventura,Keith Ellison, and the infamous Paul Wellstone Memorial meltdown .

Give him credit ,he's gone far in this world by calling Rush Limbaugh and Bill O'Reilley a liar(did Bill win a Peabody or a Polk award ????) .

I personally thing that Sen.Norm Coleman will mop up the floor with Franken if Franken makes it out of the primaries .But if he does win ;he will fit in perfectly with the other misfits and clowns in the Senate.

kindj rated this answer Excellent or Above Average Answer

Question/Answer
tropicalstorm asked on 02/01/07 - looney toon in trouble

get the acme hack saw

Man arrested in Boston marketing ploy

By KEN MAGUIRE, Associated Press Writer 15 minutes ago

BOSTON - Several illuminated electronic devices planted at bridges and other spots in Boston threw a scare into the city Wednesday in what turned out to be a publicity campaign for a late-night cable cartoon. Most if not all of the devices depict a character giving the finger.
ADVERTISEMENT

Peter Berdovsky, 29, of Arlington, was arrested on one felony charge of placing a hoax device and one charge of disorderly conduct, state Attorney General Martha Coakley said later Wednesday. He had been hired to place the devices, she said.

Highways, bridges and a section of the Charles River were shut down and bomb squads were sent in before authorities declared the devices were harmless.

Turner Broadcasting, a division of Time Warner Inc. and parent of Cartoon Network, later said the devices were part of a promotion for the TV show "Aqua Teen Hunger Force," a surreal series about a talking milkshake, a box of fries and a meatball.

Authorities are investigating whether Turner and any other companies should be criminally charged, Coakley said. It wasn't immediately clear Wednesday who might have hired Berdovsky.

"We're not going to let this go without looking at the further roots of how this happened to cause the panic in this city," Coakley said at a news conference.

Those conducting the campaign should have known the devices could cause panic because they were placed in sensitive areas, she said. Turner did not notify officials of the publicity campaign until around 5 p.m., nearly four hours after the first calls came in about the devices, she and others said.

At least 14 of the devices were found, Coakley said.

"The packages in question are magnetic lights that pose no danger," Turner said in a statement.

It said the devices have been in place for two to three weeks in 10 cities: Boston; New York; Los Angeles; Chicago; Atlanta; Seattle; Portland, Ore.; Austin, Texas; San Francisco; and Philadelphia.

"We regret that they were mistakenly thought to pose any danger," the company said. As soon as the company realized the problem, it said, law enforcement officials were told of their locations in all 10 cities.

The marketing firm that put them up, Interference Inc., has been ordered to remove them immediately, said Phil Kent, Turner chairman.

"We apologize to the citizens of Boston that part of a marketing campaign was mistaken for a public danger," Kent said. "We appreciate the gravity of this situation and, like any responsible company would, are putting all necessary resources toward understanding the facts surrounding it as quickly as possible."

Interference Inc. had no immediate comment. A woman who answered the phone at the New York-based firm's offices Wednesday afternoon said the firm's CEO was out of town and would not be able to comment until Thursday.

There were no reports from police Wednesday of residents in the other nine cities spotting similar devices.

Homeland Security Department spokesman Russ Knocke praised Boston authorities for sharing their knowledge quickly with Washington officials and the public.

"Hoaxes are a tremendous burden on local law enforcement and counter-terrorism resources and there's absolutely no place for them in a post-9/11 world," Knocke said.

Authorities said some of the objects looked like circuit boards or had wires hanging from them.

The first device was found at a subway and bus station underneath Interstate 93, forcing the shutdown of the station and the highway.

Later, police said four calls, all around 1 p.m., reported devices at the Boston University Bridge and the Longfellow Bridge, both of which span the Charles River, at a Boston street corner and at the Tufts-New England Medical Center.

The package near the Boston University bridge was found attached to a structure beneath the span, authorities said.

Subway service across the Longfellow Bridge between Boston and Cambridge was briefly suspended, and Storrow Drive was closed as well. A similar device was found Wednesday evening just north of Fenway Park, police spokesman Eddy Chrispin said.

Wanda Higgins, a 47-year-old Weymouth resident and a nurse at Massachusetts General Hospital, heard about the threat as she watched television news coverage while preparing to leave work at 4 p.m.

"I saw the bomb squad guys carrying a paper bag with their bare hands," Higgins said. "I knew it couldn't be too serious."

Messages seeking additional comment from the Atlanta-based Cartoon Network were left with several publicists.

"Aqua Teen Hunger Force" is a cartoon with a cultish following that airs as part of the Adult Swim late-night block of programs for adults on the Cartoon Network. A feature length film based on the show is slated for release March 23.

The cartoon also includes two trouble-making, 1980s-graphic-like characters called "mooninites," named Ignignokt and Err — who were pictured on the suspicious devices. They are known for making the obscene hand gesture depicted on the devices.

tomder55 answered on 02/01/07:

told ya those LED lights were cutting edge .

They have been in place for two to three weeks in Boston, New York, Los Angeles, Chicago, Atlanta, Seattle, Portland, Austin, San Francisco, and Philadelphia. What took em so long to notice ?

truth be told...if those in charge had just gone on the web they would've known immediately that this was either a marketting ploy or electronic graffiti . The world is so stuck in the 1990s .



Terror alert lever code red

Itsdb rated this answer Excellent or Above Average Answer
tropicalstorm rated this answer Excellent or Above Average Answer

Question/Answer
Itsdb asked on 01/31/07 - Which is it?

Today on page 4A of our paper...

Contractor takes the heat for slow pace of post-Katrina program in Louisiana

MELINDA DESLATTE
Associated Press Writer

BATON ROUGE, La. — Nearly eight months after it was hired by the state, a consulting company in charge of dispensing billions in federal aid to people whose homes were damaged by Hurricanes Katrina and Rita has received 103,000 applications but handed out fewer than 400 grants (the print edition stated "fewer than 300 grants").

And now the company is getting much of the blame for the overall slow recovery of New Orleans and the rest of the Louisiana Gulf Coast.

Frustrated homeowners are bitterly criticizing Fairfax, Va.-based ICF International Inc., and state lawmakers are demanding Gov. Kathleen Blanco fire the company. But ICF is defending its handling of the aid program, saying it is a task of unprecedented proportions.

ICF was awarded a contract valued at up to $756 million in June to run the Road Home program, a $7.5 billion federally funded, state-administered program to compensate property owners whose houses were damaged or destroyed by the 2005 hurricanes. For ICF, the contract amounts to a potential 10 percent commission.

~~~~~~~~~~~~~~~~~~~~~~~~~~~~~~~~~~~~~~~~~~~~~~~~~~~~~~~
Congressional Black Caucus Wants Katrina Committee

WASHINGTON -- The Congressional Black Caucus has asked House Speaker Nancy Pelosi to form a new committee on Hurricane Katrina to focus more urgently on rebuilding the Gulf Coast, particularly New Orleans.

"The Bush administration has turned its back on our fellow Americans, the victims of the greatest disaster on American soil in our generation," CBC Chairwoman Carolyn Cheeks Kilpatrick, D-Mich., wrote in a letter to Pelosi. "How can we talk about reconstruction abroad when we cannot help our fellow Americans at home?"

Kilpatrick blasted President Bush for not mentioning Katrina in his State of the Union address last week and said forming a select House committee on the issue "offers the best hope for development."

A Pelosi spokesman, Drew Hammill, said Tuesday he had not seen the letter but that the office would look closely at the request.
~~~~~~~~~~~~~~~~~~~~~~~~~~~~~~~~~~~~~~~~~~~~~~~~~~~~~~

I and others have contended that Nagin and Blanco had more than their share of the blame for the disastrous handling of Katrina, and now we see a company hired by the state has only made grants on less than half a percent of the applications they've received.

And yet, we also learn again that it's Bush that has bungled the whole thing.

God sends a hurricane, people die - Bush's fault.

Nagin doesn't use school buses, people die - Bush's fault.

Blanco doesn't get her National Guard in place, people die - Bush's fault.

Blanco hires consulting firm to hand out grants, people still homeless - Bush's fault.

Idiots in New Orleans re-elect Nagin, murder rate rises, people homeless, others don't care to return, Saints lose NFC championship - Bush's fault.

That Bush sure has screwed everything up.

tomder55 answered on 01/31/07:

I think it would be much more efficient and speedier if they would just hand out debit cards .

Saints lose NFC championship yup that fumbling Bush did it again !

Itsdb rated this answer Excellent or Above Average Answer

Question/Answer
tropicalstorm asked on 01/31/07 - ban light bulbs

what a bright idea!

http://news.yahoo.com/s/nm/energy_california_lightbulbs_dc

while America is on a ban kick what are some things you think NEED banned and why?

Can we ban idiots?
how about dirty dishes?
or haunted cemetaries?

tomder55 answered on 01/31/07:

The "How Many Legislators Does it Take to Change a Lightbulb Act" lololol .They could save us all a ton of money just by not showing up for work .

So much for that ole easy bake oven . You'll wait a long time to bake a cake using a flourescent bulb.

I'm not in favor of the ban ,but I mostly use the energy saving bulbs .( you cannot use flourescents in all fixtures. They can't be used with a dimmer, and they cannot be used in recessed fixtures either.)
In the long run they have saved me money not only because they don't use as much electricity ,but to date they have not burned out . To me it was a no brainer to switch. I don't think they illuminate as much as standard bulbs either but it has not been a major issue .

But that is not the point . What is needed is public education rather than more laws. The retail business'around the country would cooperate in the effort and so would the utility companies. If they wanted some gvt action then why not offer rebates for the incentive ?

I do not think that fluorescent lightbulbs are the cutting edge of technology either . That would be LED lights The future technology that is rapidly approaching us is nano dot technology ....but you wouldn't expect a legislator to know that would ya ? What he want's to do is ban yesterday's technology and replace it with....well ... yesterday's technology .

You will be hearing more about nano-technology in the coming years . It is a new frontier in technology that is in it's infancy . Some of it is frightening ( it's applicability to future weapons ) but most of it promises many new breakthroughs that will positively impact the quality of human life .

what else to ban...Lets see .... Maybe restaurants and grocery stores should be banned from throwing food away that is perfectly good.

Itsdb rated this answer Excellent or Above Average Answer
tropicalstorm rated this answer Excellent or Above Average Answer

Question/Answer
paraclete asked on 01/31/07 - It's nice to know they have decided whose in and whose out?

Radical Islamist plan for world of pain
By Piers Akerman
January 30, 2007 12:00am

THE radical Islamists of Hizb ut-Tahrir (Party of Liberation) who met in Lakemba to cheer speakers calling for a global Islamic state, which somehow did not include Australia, raised a number of issues.

The first is about Australia's tolerance for such a call, for, as students of the group (which is banned in a number of nations) know, an Islamic state could not possibly condone many of the freedoms basic to our society.

Another issue is just as crucial: if Australia is not among the nations to be targeted and brought under the Islamist umbrella, exactly which nations does Hizb ut-Tahrir believe need an Islamist makeover?

If "humankind" can only escape destruction if it becomes subservient to the "divine order of Islam", and Australia is not to be subjected to the group's reconstructive process, wouldn't its members be better off going to a nation which would benefit from a more proscriptive regime?

The reality is of course that few Islamic nations want a bar of Hizb ut-Tahrir's proselytisers and they have to find helpful suckers in Western nations who will permit them to hold their "Islam-or-else" meetings.

As it is unimaginable that any Islamic nation, or any other nation for that matter, would host a conference of fundamentalist Christians calling for the restoration of Istanbul to Constantinople, why should any Western state play host to these plotters?

Are we so naive, so politically correct, that we feel compelled to host a party for people who wish to cut the throats of our metaphorical cousins, even if they promise that we are not (yet) in their sights?

One of the speakers at the conference, Palestinian Sheik Issam Amera, told the audience: "If two people are united and a third person comes along and tries to incite disunity . . . kill him. Muslims are not unique in doing so as most nations kill those charged with treason.

"The establishment of caliphate is an Islamic duty. The evidence for the duty for establishing (the) caliphate is confirmed in the Koran," he said.

The Koran also mentions supernatural bodies, but that doesn't make them real.

In fact, if one looks around the world and particularly at those states which nominate themselves as Islamic, one could draw the conclusion that followers of Islam are doomed to live in Third World poverty listening to hate-filled speeches from their religious leaders fuelled by envy of the comparatively successful nation of Israel, and with sufficient time on their hands to riot over almost nothing whenever someone gets to his feet in the local mosque.

Sunday's conference, though little different from others held in some Muslim communities lodged in Western societies, heard calls for the political destiny of the Muslim world to be in the hands of Muslims and for external forces to be rejected.

Judging by the manner in which members of Hamas and Hezbollah, both professing to be Islamic organisations, are currently kidnapping and killing each other, and considering the wars of attrition up to the current day waged between followers of the Shia and Sunni arms of Islam, any notion that Islamists could achieve a better record of governance than many Western nations, including Australia, is fanciful, no matter how vehement the calls for sacrifice and jihad may be.

Hizb ut-Tahrir clearly has no place in Australia, and it would be heartening if more Australian Muslims said this.

Unfortunately, as older Australians well know from the attempts to ban the Communist Party in the middle of the last century, it is awfully difficult to ban any group in this country.

Greens Senator Bob Brown, aided by the Left-inclined Fairfax press, is attempting to disrupt the activities of the Exclusive Brethren, a minor Christian sect which has angered him because it quoted from Green policies in a series of political advertisements, but has barely raised a whimper about the treatment of homosexuals in Muslim countries.

The NSW Government's recycled Police Minister John Watkins believes there is sufficient evidence to have Hizb ut-Tahrir banned but has failed to persuade his ALP colleagues in other states to join him in making such a recommendation to the Federal Government.

Federal Attorney-General Philip Ruddock has asked NSW to provide evidence that Hizb ut-Tahrir meets the definition of a terrorist group and has also called on federal agencies to review the comments made at its conference to determine whether its conduct crosses that threshold.

If Watson believes the body is breaking NSW laws, he is free to deal with it under state law.

Civil libertarians, usually highly visible and irritatingly voluble whenever Islam and ban are mentioned in the same sentence, are nowhere to be seen or heard.

Until firm evidence of illegal activity emerges, it would seem there is no recourse available beyond making the usual expressions of disgust and contempt that it deserves.

That of course, is fitting proof that Australia has a robust and admirable pluralist culture – just the sort of thing that gets up the noses of the Hiz ut-Tahrir – if indeed they have such organs behind their hijabs and scarfs.


tomder55 answered on 01/31/07:

"The establishment of caliphate is an Islamic duty. The evidence for the duty for establishing (the) caliphate is confirmed in the Koran," he said.

it is not that it's a rarity that the jihadists are so blatantly and arrogantly blunt ;it is just that in general the civilized world has not heard the clarion mustering them to the Gates of Vienna yet .Australia is one of the nations that have answered the call.

Itsdb rated this answer Excellent or Above Average Answer
paraclete rated this answer Excellent or Above Average Answer

Question/Answer
paraclete asked on 01/30/07 - Boy, am I glad I DON'T LIVE THERE

I left this place in 1979 and now I know why.


Climate change to 'devastate' Sydney

January 31, 2007 06:37am
Article from: The Daily Telegraph

Font size: + -

Send this article: Print Email

SYDNEY is looming as one of the world's major climate change casualties, with temperatures expected to soar 50 per cent higher than the average rise forecast for the entire planet.
For the first time, Australian scientists have charted in detail, the impacts on the nation's largest metropolis of man's insatiable demand for energy and burning of fossil fuels.

The Daily Telegraph today exclusively reveals the landmark CSIRO report commissioned by the State Government which - for the first time - specifically details the impact of climate change on NSW.

It paints a picture of a city baking under average temperatures almost 5C higher than now - which will kill 1300 people a year - and one battered by extreme winds and permanent drought.

NSW Premier Morris Iemma said the report's findings were alarming.

"This might sound like a doomsday scenario, but it is one we must confront," Mr Iemma said.

And it will put pressure on Prime Minister John Howard to commit to the same tough targets set by NSW - to reduce greenhouse gases 60 per cent by 2050.

Comparing today's climate, the CSIRO predicted Sydney would resemble the harsh dry conditions of the tiny village of Paterson, 150km northwest of the state capital, in less than 25 years.

By 2070, average temperatures will have soared by 4.8C - compared with 3C forecast for the planet by the International Panel for Climate Change this week.

In summer, maximum temperatures could rise by as much as 7C by 2070. But heat-related deaths will jump from 176 a year - the current annual average - to 1312 by 2050.

Our dams will be drained of water as the city plunges into a virtually permanent dry spell and evaporation rates increase by 24 per cent.

The frequency of droughts now average three every decade. By 2070 there will be only one year out of 10 that is free of drought.

The bleak assessment suggested Sydneysiders would have to reduce water consumption by 54 per cent for the city to remain sustainable within the next 20 years.

Extreme weather events, including 110m storm surges by 2100, will devastate the coastline as well as property.

Bushfire frequency will almost double, with rainfall expected to be reduced by up to 40 per cent.

The report will prompt calls for the creation of a national emissions trading scheme to be put back on the agenda despite Mr Howard's reluctance to sign up.

"The Commonwealth can no longer put its head in the sand on this issue. I have repeatedly asked the Prime Minister to show national leadership by convening a climate change summit," Mr Iemma said.

"I do not want my kids to ask me in 10 years time why I didn't do more to address the issue of climate change."

The CSIRO report warned that the city must work out how to adapt quickly, with the impacts of human-caused global warming now apparently inevitable.

"The future climate of Sydney is likely to be warmer and drier," the report says. "Such trends would also increase evaporation, heat waves, extreme winds and fire risk.

"Nevertheless, despite this trend towards drier conditions, the possibility of increases in extreme rainfall events remain.

"Although average changes in temperature, rainfall and evaporation will have long-term consequences for the catchment, the impacts of climate change are more likely to be felt through extreme weather events."

Climate change forecasts for the coastal zone put Narrabeen and Collaroy on the hot zone for storm-inspired sea surges of about 22m - which would inundate homes.

Freak surges of 110m would be catastrophic.

"Such increases in storm surge in conjunction with sea level rises, would increase the risk of coastal inundation," the CSIRO said.

++++++++++++++++++++++++++++++++++++++++++++++++++
I can't help feeling someone left the decimal points out of those storm surges but 5'C is a big increase in summer temperatures which can already hit 105'F, from what I read the Tsumami has nothing on this and if it happens here it happens everywhere?

tomder55 answered on 01/31/07:

I'm going to make a bumper sticker :CLIMATE CHANGE HAPPENS

the draconian measures the environmentalist would have us do would cause more human misery then it would fix. Carbon emissions have reduced hunger and poverty world wide and have raised the living standards of most of the world . Reasonable measures to reduce carbon emissions and alt. fuel development I'm all for (although I have had second doubts about the efficacy of ethanol production and large windmill farming ...I think smaller windmill farms servicing a localized market makes more sense where applicable ) . But even if the evidence is in about carbon emissions being a major factor ; the reductions needed to make any meaningful change would have an enormous negative impact. We would be better off planning to cope with a changed environment instead.
.........................................
as a bonus I will include my response to a climate change question yesterday on the Philosophy Board .I concure with Jim.McGinness' rating .

paraclete rated this answer Excellent or Above Average Answer

Question/Answer
tropicalstorm asked on 01/31/07 - why Katie couric's ratings LOW

Now the 'explanation' is gender bias of sexist America. People don't take her serious because she is a woman.

Here I thought it was because she only asks the fluff questions!

tomder55 answered on 01/31/07:

I tried for the 1st week to watch her show . I found I wanted to watch the NEWS instead so I switched back to ABC where Charles Gibson does a more traditional news broadcast if I'm even watching television at that time. But even that I can only stand for 15 minutes . The last 15 minutes of that time slot I switch on Fox to watch the Brit Hume roundtable discussions.

The biggest problem I had with the CBS evening news with Couric was that she was attempting to do the news in a semi-morning show format.But I give her credit for attempting a fresh approach

I do not think it is only her ;she actually understands the problem, but her solution is off . What the dinosaur media has yet to grasp is that there is a shrinking demographics of people who's sole source of news is network broadcasts . By the time they are reporting the news everyone knows about it through the internet ,24 hour cable news ,or the radio. If I were in charge of the networks I'd probably let the time slot go to expanded local broadcasting...maybe regional news magazine formats like 20/20 . That would satisfy any false sense of responsibility they have for public information broadcasting .

Closer_To_The_Heart rated this answer Excellent or Above Average Answer
ETWolverine rated this answer Excellent or Above Average Answer
Itsdb rated this answer Excellent or Above Average Answer
tropicalstorm rated this answer Excellent or Above Average Answer

Question/Answer
ETWolverine asked on 01/30/07 - Where is the war on terror?

Most members of the Democratic leadership have denounced the war in Iraq as not really part of the global war on terror. Nancy Pelosi in particular has argues that Iraq is not part of the war on terror, and others, like Harry Reid, Ted Kennedy, John Kerry and more have followed suit. (The fact that the majority of Muslim terrorists in the world are currently fighting in Iraq doesn't seem to sway them at all.) They claim that oth Shia and Sunni Muslims are being alienated by the war.

During the war between the terrorist organization Hezbollah and Israel this past summer, much of the same Democratic leadership called on Bush and Rice to negotiate a peace agreement between the two parties, saying that Israel fighting Hezbollah was not helpful to the global war on terrorism. They claim that not negotiating with Hezbollah alienates Shia Muslims.

For a while now, Democrats have criticized Bush for not regularizing diplomatic relations with the terrorist organization Hamas in the Palestinian Authority. They claim that not having open dialogue with Hamas is bad for the global war on terror because it alienates Palestinians.

Now we are hearing criticism of the posibility of open war with Iran, and criticism of Bush not negotiating with Iran. They claim that this is bad for the global war on terrorism, because it alienates Shia Muslims.

So the fighting in Iraq isn't good for the GWOT. Neither is fighting against Hezbollah, Hamas and Iran.

So where, exactly, is the appropriate place to fight the GWOT, according to the Democrats? Where do the Democrats, (who want us to believe that they support the GWOT but not the war in Iraq, and claim that they support the troops but not the war in Iraq), want us to fight the GWOT?

Just wondering.

Elliot

tomder55 answered on 01/30/07:

The GWOT is a misnomer so it is almost understandable that it's definition is an open book that can be filled in to mean whatever one want to believe.

The Democrats would like to limit it to the battle for Afghanistan ...(which I believe they only add to their hit list because much of the front has shifted away from it. Had we not engaged in Iraq I have no doubt that the loons who were sraying graffitti on the Capitol over the weekend would be protesting our continued deployment there )....and
the prewar strategy of combatting terrorism through law enforcement agencies exclusively .

One can only conclude they still ,in spite of all the evidence ,do not believe we are in a struggle for civilization with a transnational ideology hell bent on forcing us to "submit" .

ETWolverine rated this answer Excellent or Above Average Answer

Question/Answer
tropicalstorm asked on 01/29/07 - one more post on waste contributing to pollution

over the past half century many mechanics and so forth have invented cars that last forever (Tucker). Carboraters that will triple your gas mileage (the ones they have at Advanced Auto and so forth are inferior compared to the ones they don't want us to know about).
And back in the fifties and sixties (even since) several people have invented alternative fuels. My ex's grandfather invented a clean clear corn fuel in the fifties. But what happens to these inventions? NOBODY hears about them because the government pays them off to NOT market them. WHY? Because big business would suffer since people would not have to buy cars and gas as often.
This really applies to everything. If you notice the insulation in stoves and refridgeraters nowadays it is nothing compared to the old. I use to be able to buy brooms that lasted years, now the bristles start falling out within months or the handle swivels off and
taping or gluing it doesn't work.

tomder55 answered on 01/30/07:

I think consumers should not be left off the hook . If you sort your trash like I do for recyling you will notice that the bulk of the corrugated and plastics tossed are on needlessly excessive packaging components. That is consumer driven (well also gvt regulations contribute greatly to this problem) .Trust me ;manufacturers would prefer to not set up labor intensive and bottom line cutting packaging lines if not necessary.

I have been a supporter of alternate fuels . But for each success story like Brazil's sugar to ethanol conversion there is a dubious claim for the value of corn coversion . I am beginning to come to the conclusion that the hype is all ADM and other agri-business pressure to further add subsidies to products they market. More than one resource has claimed that it takes more energy to produce ethanol from corn than it's output of useful return . The costs of storage and transportation as well as pump conversion is rarely mentioned. Finally;no one has proven to me that an auto can run in the cold north on anything better than a 20%addition to petroleum . Ethanol at best would be just a part of the energy independence solution and would have virtually no effect on carbon emissions given the processing emissions inherent in the conversion .

As far as the auto industry goes ,nothing suprises me. In 1974 Bradford Snell , a staff attorney for the U.S. Senate antitrust subcommittee suggested that GM sabotaged energy-efficient Trolley systems in 45 cities around the country in order to sell more fuel-guzzling buses and autos. They also ,according to Snell, forced the railroads to replace electric locomotives with GM-built diesels by threatening to withhold lucrative auto shipments.According to Snell,around 1939, GM got together with Standard Oil of California (now Chevron), Firestone, and other auto-related firms to set up a holding company that bought up trolley lines, dismantled them, and replaced them with buses. [I have to confess a certain bias in this case . My grandfather for years was a trolley operator .When the buses came he learned how to drive them but ultimately it cost him his life . He crashed his bus into a poll after he had let off his last passanger of the day. ]

no I do not think that gvt. pays them not to market but industry does lobby and pay off the gvt.

Itsdb rated this answer Excellent or Above Average Answer
tropicalstorm rated this answer Excellent or Above Average Answer

Question/Answer
Itsdb asked on 01/29/07 - The 50 percent solution?

In his upcoming book 'positively American,' Chucky Schumer tells us how he figured out how Democrats can win the White House in 2008 by discussing it with his imaginary friends, Joe and Eileen Bailey.

How? His 50 percent solution, ൓ goals, which...could help define what Democrats stand for."

    "For example, Democrats should commit to increasing reading and math scores 50 percent by dramatically increasing federal involvement, and funding, in public schools. We should increase the number of college graduates by 50 percent. We should call for reducing illegal immigration by at least 50 percent and increasing legal immigration. We should cut our dependence on foreign oil by 50 percent, and reduce cancer mortality, abortions and childhood obesity each by 50 percent. We should increase our ability to fight terrorism by 50 percent."


I don't know what the other five goals are yet nor his plan to reach those goals - but hey, at least he's trying to come up with something Democrats stand for. I just wonder if he remembered John Edwards' claim in the 2004 race, "If we do the work that we can do in this country, the work that we will do when John Kerry is president, people like Christopher Reeve are going to walk, get up out of that wheelchair and walk again."?

Comments?

tomder55 answered on 01/29/07:

I'm waiting for the library to get my reserve for the book processed .I won't buy it .

I heard Shumer talking about the book on 'Meet the Press.

MR. RUSSERT: A, a, a press story in the New York Post described it this way, it was this headline, this is the Schumer book, “Schumer A Party Pooper; New Book: Dems ‘Lost Touch’ With Middle Class.”

“In a revealing new book on politics, Senator Charles Schumer comes out with guns blazing—not at President Bush, but at his own Democratic Party.

“Schumer ... rips his party for being in the clutches of special-interest groups for too long and for losing touch with the middle class.

“‘Washington Democrats too often took their cues from interest groups without considering the needs of the average person. ... Group identities around the country were less important, but those claiming to represent group interests in Washington were stronger than ever,’ Schumer wrote.”

Which special-interest groups?

SEN. SCHUMER: Well, all of them. Left, right and center. I bring up, in my book, talk when I got to Congress in 1980, for instance, and that was—that was about crime was ripping apart my district. I come to Washington, and I find out that the ACLU is writing crime legislation, has a veto over any piece of crime legislation. Now, they should be at the table. Their views should be considered.But our job, whether we’re Democrat or Republican, is not to just take what the interest groups want and just make it into legislation, it’s to balance their needs against others’. I believe in the environment, but there’s the issue of jobs. I believe in civil liberties, but there’s the issue of security. And what both parties have done, Tim, is forgotten the average middle class voter—yes, I call them the Baileys, but they could be anybody—and instead paid too much attention to interest groups.

The good news for us, Tim, and we talk about this in the book, people have an inclination to support the Democrats. Joe and Eileen Bailey, this middle class couple, they bought into Reagan Republicanism in 1980. They were in great shape. “Get the government off my back. Get it out of the way.” 2006, the world is different. We have terrorism, which we’ve talked about. Our—their kids in their schools have to compete against Chinese, kids in Chinese and Indian schools. We live a lot longer. Not only is that a problem for Medicare and Social Security, but how we live, getting married later, when and whether to have kids, all that leisure time. And for the first time, the Baileys are saying, “You know, I might need some government help.” The party that comes up with a platform that adjusts all these major changes caused by technology to the Baileys’ lives and, and, and helps them, doesn’t supplant them, realizes they’re doing OK, will be the majority party.



I have to say that I don't find to much to disagree with Schmucky this time except for this over reliance on the gvt. to solve our problems for us... Which I think is the fundamental difference. But who knows ? If the Bailey's of America prefer the nanny state that the Shumers of the world champion then the Democrats will be the majority .


Itsdb rated this answer Excellent or Above Average Answer

Question/Answer
Choux... asked on 01/28/07 - End War in Iraq by Jan 2009

AP Iowa -
"Hillary Rodham Clinton said Sunday that
President Bush should withdraw all U.S. troops from
Iraq before he leaves office, asserting it would be "the height of irresponsibility" to pass the war along to the next commander in chief.

"This was his decision to go to war with an ill-conceived plan and an incompetently executed strategy," the Democratic senator from New York said her in initial presidential campaign swing through Iowa.

"We expect him to extricate our country from this before he leaves office" in January 2009, the former first lady said."

~~~~~~~~~~~~~~~~~~~~~~~~~~~~~~~~~~~~~~~~~~~~~`

It was Bush's blundering that caused this mess in the MiddleEast...time for him to wind up the war

tomder55 answered on 01/29/07:

It is the height of irresponsiblity for Hillary to be spouting these sentiments off even if she believes them to be true.

One has to question her fitness to command . But I'm sure if she were commander in chief she would want the opposition party to at least make an appearance of supporting her during the war . It does send a signal to the jihadists that all they have to do is wait us out and hope that a new election cycle in America will radically alter our strategies.

"This was his decision to go to war

She voted along with 70 other Senators and the vast majority of the House to approve the war resolution. She can spin it any way she wants to but that is the fact .When she made the vote she knew EXACTLY what she was approving .

All these jump ship Senators are going to have egg all over their face if this plan succeeds. But if it fails they have to look in the mirror and ask themselves how their rhetoric contributed to it.

Choux... rated this answer Poor or Incomplete Answer
tropicalstorm rated this answer Excellent or Above Average Answer

Question/Answer
tropicalstorm asked on 01/29/07 - American vehicles SUV vs the death traps

Why do you think we Americans drive SUV's as opposed to the alternative little bread box size cars we have as an alternative?
From the way I see, our government did away with the medium size cars of the sixties because they said they were polluting our environment. I could never understand why they didn't just modify the cars to the standards they wanted instead of coming out with the little cars with no style. The main standards people tell me are emissions, lighter material than the steel and better gas mileage.
Anyway, they put us in these little cars where you can barely fit two adults and three kids. Then they say the kids up to seven years old or sixty pounds have to have car seats making even less room in the car. So solution, the SUV now a family of 5 or more can comfortable go out together without having to take two vehicles. Soccer moms can take more than three kids to soccer without a problem.


tomder55 answered on 01/29/07:

I for one used to like the sationwagon . For years I drove a Corolla and never felt threatened on the road. I now have an Outback because my commute got longer and I do not like to lose work time due to bad weather .I would only drive an SUV if I was hauling a large family because station wagons are no longer availabe.

Itsdb rated this answer Excellent or Above Average Answer
tropicalstorm rated this answer Excellent or Above Average Answer

Question/Answer
tropicalstorm asked on 01/28/07 - Art Bell reading the news

Hillary criticizes Bush for not running the war right; BUT blames herself for allowing it to happen.

tomder55 answered on 01/28/07:

Those spikes in the fence she sits on must be irritating .
Hindsight is 20/20 as they say ,and in the case of Hillary there's alot of hind sight to see.

but that goes for most of the 2nd guessers in Congress.

The war resolution was passed overwhelmingly . Tom Paine gave some of his most scathing critique to 'sunshine patriots'.Then again he also said we would hang together or hang seperately and by all appearances there are many in Congress who are more than willing to let us hang seperately .

ETWolverine rated this answer Excellent or Above Average Answer
tropicalstorm rated this answer Excellent or Above Average Answer

Question/Answer
Dark_Crow asked on 01/27/07 - Is it true that a penny saved is a penny earned?

My personal opinion is that it is not true literally, or figuratively. In fact I think the idiom is foolish in the context of economics, and goes to the heart of the difference between right thinking and wrong thinking because it excludes the concept of creating wealth and new sources of growth; that is, the difference between Nation Building i.e. development to foster social harmony and economic growth- and stagnation- the result of cutting and running in Iraq.

Comments?

tomder55 answered on 01/28/07:

If you put it in the bank ,it will be reinvested back into the economy in the form of a loan AND you will have still saved the penny. . If you throw it in a jar then it is wasted opportunity .

I for one make sure that all my pocket change gets spent. I do not allow it to accumulate .I annoy patrons waiting behind me while I count out exact change.

There is no utility for a penny that is not spent because it takes more resources to make it then it is worth. But spending it becomes a multiplier. If I buy a penny's worth of goods and services then 1 cent is spent. Then if the person I purchased from again uses the penny it is 2 cents worth of goods and services it has purchased ...and so on ....

I fully agree with your Iraq analogy. A good comparison was the Marshall Plan . Aiding and molding the post war Western Europe made good strategic and even better economic sense.

Dark_Crow rated this answer Excellent or Above Average Answer

Question/Answer
Itsdb asked on 01/27/07 - Troops Authorized to Kill Iranian Operatives in Iraq

So what were they supposed to do, invite them in for tea?

    Administration Strategy Stirs Concern Among Some Officials

    By Dafna Linzer
    Washington Post Staff Writer
    Friday, January 26, 2007; Page A01

    The Bush administration has authorized the U.S. military to kill or capture Iranian operatives inside Iraq as part of an aggressive new strategy to weaken Tehran's influence across the Middle East and compel it to give up its nuclear program, according to government and counterterrorism officials with direct knowledge of the effort.

    For more than a year, U.S. forces in Iraq have secretly detained dozens of suspected Iranian agents, holding them for three to four days at a time. The "catch and release" policy was designed to avoid escalating tensions with Iran and yet intimidate its emissaries. U.S. forces collected DNA samples from some of the Iranians without their knowledge, subjected others to retina scans, and fingerprinted and photographed all of them before letting them go.

    Last summer, however, senior administration officials decided that a more confrontational approach was necessary, as Iran's regional influence grew and U.S. efforts to isolate Tehran appeared to be failing. The country's nuclear work was advancing, U.S. allies were resisting robust sanctions against the Tehran government, and Iran was aggravating sectarian violence in Iraq.


The rest of the article is here.

The Washington Compost is just doing what they do best, trying to stir up crap. And so once again Bush has to defend himself on something that should be a no-brainer...pun intended. The Washington Compost apparently thinks it wrong to kill Iranians in Iraq as "there is no evidence the Iranians have directly attacked U.S. troops in Iraq, intelligence officials said."

Dingy Harry Reid said the president needs congressional approval for any program that could "escalate this conflict" with Iran. Reid said Bush should be engaged in direct diplomacy with Iran and other countries in the region to avoid a widening conflict, rather than "sending battle carrier groups" to sit off the Iranian coast.

For a bunch of people that think Bush is an idiot, isn't it odd that the Compost and the blithering idiots in congress can't figure out that having Iranians in Iraq is a bad thing? Haven't they all scoffed at the notion that Iran has been operating in Iraq? And now the Compost admits that "for three years, the Iranians have operated an embedding program there, offering operational training, intelligence and weaponry to several Shiite militias connected to the Iraqi government, to the insurgency and to the violence against Sunni factions" and doesn't bat an eye? I guess we should just continue to "play catch and release" or just pretend Iranians aren't there?

Makes me think our troops should just act as one does towards a little child holding his hands over his face and saying "you can't see me."

tomder55 answered on 01/28/07:

Beats the hell out of catch and release.

It was the compost in May that reported the presence of a large number of the Iranian Revolutionary Guard's Qods Force inside Iraq.
I have been told that their numbers exceed 30,000.

Coalition forces have suffered the consequences of Iran's military presence. U.S. and British officials contend that the IRGC has introduced into Iraq "shaped charge designs" -- powerful bombs that channel the force of an explosion into a narrow path. (Lebanese Hezbollah also has used such bombs effectively against Israeli tanks.) According to the British, at least 10 of their soldiers in southern Iraq have been killed since May 2005 by the combination of such explosives and remote triggering devices. Marine Gen. Peter Pace, chairman of the Joint Chiefs of Staff, noted in a March briefing with Defense Secretary Donald H. Rumsfeld that these makeshift bombs are "traceable back to Iran."

Strategic Forecasting Inc 'Stratfor',a pretty reliable intel. site (subscription only )reported this week that :

Prince Bandar bin Sultan … plans to visit Iran on Thursday to hold talks with senior officials, including his Iranian counterpart, Ali Larijani.

Bandar’s trip … is an indication that the two rival states are engaged in serious negotiations over Iraq. The driving force behind these talks has been Iran, which signaled last week that it is willing to work with Saudi Arabia to help stabilize Iraq and resolve other regional issues.

In fact, it appears Iran is using Saudi Arabia as a conduit to send messages to the United States

(Meanwhile) … a flurry of statements saying Iran is willing to cooperate with the International Atomic Energy Agency.

In another development, Ahmadinejad told Iraqi President Jalal Talabani on Wednesday that Tehran is “fully ready for any cooperation which will lead to security and peace in Iraq.” … Iranian officials said they are ready to negotiate a settlement with the United States on issues ranging from Afghanistan to Lebanon.

The Iranians are moving toward a conciliatory approach on all fronts, which has been made possible in part by what appears to be a reining in of Ahmadinejad and his ultraconservative faction …

The clerical regime realizes it has reached the end of the road … any additional defiance could be detrimental to its national security and foreign policy objectives. Another key factor is Khamenei’s failing health: The Iranians want to consolidate their position in Iraq and in the region before there is a change at the helm …


I of course am a skeptic wherever Iran is involved and realize that Stratfor has for some time been an advocate of back room negotiations with moderates in Iran . I do not believe that election setbacks for the Mahdi-hatter or the evident rise of Ali Akbar Hashemi Rafsanjani to the head of Iran’s 'Assembly of Experts' will signal a foreign policy shift of the Mullahs. They still intend to go full steam ahead with their nuke program ;and to harry our forces in Iraq. The so-called moderates and the hardliners in Iran have precisely the same ambitions for their nation;domination of the Middle East, development of nuclear power, and the downfall of America. [chapter 73, hadith no. 1298: ‘Verily, war is deception,’ ]

I think Stratfor neglected to mention the new tough stand that the US is taking ,and backing up with it's mobilization of the fleet ,and how that is changing the equation. Iranian influence may be at it's zenith right now and they want to exploit it by holding negotiations from a position of strength while they still have it.

We on the other hand need to continue to press Iran as hard as we can as fast as we can. Strategy Page explains where the Saudi interests lie.

Of course it’s nice what the Iranians and Saudis are saying. But it’s what they do that is important.




Itsdb rated this answer Excellent or Above Average Answer
labman rated this answer Excellent or Above Average Answer

Question/Answer
HANK1 asked on 01/27/07 - ERNIE PYLE:



How would the great World War II correspondent, Ernie Pyle, view the Iraqi War at this time?

HANK

tomder55 answered on 01/27/07:

For one thing Pyle would be with the troops and not reporting from the Green Zone. I hear a different narrative from embedded reporters than from the MSM wire services who rely on fictional accounts relayed from unreliable sources ;like APs single source for much of it' s disinformation ;Capt. Jamil Gholaiem Hussein of the Iraqi police force .

For a time there was speculation that he was in fact a fictional source but after 6 weeks of controversy AP managed to produce this officer. The question remains ;why was he the sole source of so much of their reporting . There is also questions about the veracity of much of the narrative he spun. Hussein, was the sources for an AP story in late November about the burning and shooting of six people during a sectarian attack at a Sunni mosque.

The U.S. military and the Iraqi Interior Ministry raised the doubts about Hussein in questioning the veracity of the AP's initial reporting on the incident, and the Iraqi ministry suggested that many news organization were giving a distorted, exaggerated picture of the conflict in Iraq. Internet bloggers accusing the AP of having made up Hussein's identity in order to disseminate false news about the war.

The ministry had initially denied Hussein's existence,its first search of records failed to turn up his full name. But Hussein has since been produced and arrested because Irqi police sign a pledge not to talk to reporters when they join the force.

The AP is of course declaring themselves vindicated but it doesn't change the fact that they have used a suspicious sole source for alot of their reporting . The story of the mosque burning they already admitted they botched.

Whether Jamil Hussein actually exists is really a secondary issue. The fact that the AP used a single source for dozens of inflammatory stories about atrocities in Iraq that still have yet to find any confirmation is almost as disturbing as making the source up.

When Ernie Pyle reported you could bank on the authenticity of the story because he was reporting as an eye witness to history directly from the battle field.

HANK1 rated this answer Excellent or Above Average Answer

Question/Answer
tropicalstorm asked on 01/26/07 - local talk show host claims

continuously that we are not fighting a war like past wars. We are fighting a highly technical war and so a lot less lives are lost than would be. But he sounds like he is parroting something he heard that sounded good and he can't back up a lot of things he says. You know how them hosts are they keep talking their talk and stiffle whay you want to say because they can't back it up.
Anyway, what kinds of things could he be referring too that are technically advanced from other wars?

tomder55 answered on 01/26/07:

I understand your concern . My cousin is a high ranking officer over there serving in his 3rd tour dating back to 1990, and the other day a copter was downed and the news reported that senior officers were casualties. I was in a semi-panic and spent alot of time researching and making calls after I heard the news.

Despite all the talk in the media , the casualty rate of our troops is low compared to simular wars we've been involved in. If I was on the ground I would want to be surrounded by an Abrams. There is a heavy bulkhead that seperates the crew from the fuel tanks for one thing .None of the crew need be exposed outside of the tank because it's periscopes monitor 360 degrees with day and night vision ;although on occasion they have been attacked while outside the hatches.

During operation Desert Storm 18 Abrams were disabled (only 9 permanently )during combat with some of the best Russian T-72s .There were NO COMBAT CASUALTIES in an Abrams (3 wounds as a result of some friendly fire by Hellfire missiles fired from AH-64 Apache attack helicopters ).

There were again NO CASUALTIES during OIF in 2003. Since the invasion there have been a few incidents where there were casualties (IEDs primarily ,one tank fell off a bridge. )Iraqis on occasion have gotten close enough to pop off an RPG and disable an Abrams tracks . Snipers occasionally fire on crew that is exposed above the turret hatches or near the tank.

The so called “Hezbollah IED”(made in Iran ) sends out an anti-tank like projectile upon detonation but they have been more effective against other armored vehicles .

The armor on the Abrams has been upgraded to deal with urban warfare .

The down side is that an IED is relatively cheap when compared to an Abrams tank. So the costs of losing one even if there are no casualties are greater .

A counterpoint to the talk show host's case is that the Abrams is more of a blunt instrument that has many high tech features. Experiences on the ground have shown the Abrams capable in all types of combat. Experiences in Iraq have shown the US Army that the Abrams tank will constitute a significant portion of the combat force well into this century, as there are no viable substitutes for it. So we will upgrade the tank for service for another twenty years rather than look for that high tech panacea. The high tech 'Future Combat System' that planners envision cannot provide with technology what the Abrams provides with sheer bulk.

tropicalstorm rated this answer Excellent or Above Average Answer

Question/Answer
Dark_Crow asked on 01/25/07 - Given that all human activity springs from two sources, which is the greater in man’-impulse or de

.

tomder55 answered on 01/25/07:

yes it can be argued that the cart was placed before the horse. I think that was mostly a blunder on our part that we are still trying to address. The people of Baghdad right now long for security and I don't franky think they care where it comes from . The war will be won or lost on that outcome.

Choux... rated this answer Excellent or Above Average Answer
Dark_Crow rated this answer Excellent or Above Average Answer

Question/Answer
tropicalstorm asked on 01/25/07 - why are the republican

radio talk show hosts still talking about Valerie not being covert? That is such old news and they are beating it to death.

tomder55 answered on 01/25/07:

The Libby trial began and I am not happy with the direction it is taking .

The trial is taking place in DC and it looks like Fitgerald was able to get alot of Bush haters on the jury (Dems out number Republicans 10:1 there ).

Libby's running a strange and dangerous defense so far. His lawyer is alleging that Libby is a fall guy for Carl Rove and for an administration that was in civil war with itself.

Richard Armitage has admitted to being the leaker, so Rove and Libby are both off the hook automatically except for the charge that Libby Lied . The charges against Libby are about obstruction of justice, not about leaking Plame's identity, so the defense's argument about Libby taking the fall for Rove seem very misguided. That really has nothing to do with the charges against him.The defense is trying to unload all the blame on the Bush Adm. to get Libby off, because if Libby is seen associated with the Bush Administration, he'll go down anyways with a jury of Bush haters .That is the only thing that makes sense.

But it is risky ,if Libby is going to point fingers at the administration he can kiss the possibility of a pardon goodby.

ETWolverine rated this answer Excellent or Above Average Answer
tropicalstorm rated this answer Excellent or Above Average Answer

Question/Answer
tropicalstorm asked on 01/25/07 - why do people dread jury duty?

Replies for IF people dread jury duty and really don't want to do it.

A. I stick to my own race, I wouldn't know
B. I am all for profiling
C. I am for the little guy, companies have their racket.
D. Let the criminal rot in jail; at least send them all to their own island.
E. No I am not KKK, but I can't relate to anything outside my own problems

tomder55 answered on 01/25/07:

I was dismissed twice ...both because I did not want to serve on the particular cases.

First time I told the defense lawyer during the selection period that if the DA took the time and effort to indict the defendent then he probably was guilty .

The second time I told the plaintiff's lawyer in a personal injury case that if I saw the sidewalk in a state of disrepair ,I would probably walk across it a little more carefully .

tropicalstorm rated this answer Excellent or Above Average Answer

Question/Answer
Choux... asked on 01/25/07 - State of the Union Address

".....I thought of Willy Loman as I watched George Bush deliver his State of the Union address. Here was a man like Willy who was absolutely confident of his own charm, a personality man who had nothing of substance to sell; a man who brings ruin to all around him as he clings to his fantasies of success. Only unlike Willy, George Bush is our Salesman of Death. He stood there delivering his tired spiel, unpacking his tawdry goods; the misbegotten war, while peddling terror and no taxes as if they were shiny new stock. He dragged out all the initiatives that he should have considered six years ago, which now seemed shopworn, threadbare, and counterfeit in his hands, new sources of energy, health care, and his disastrous No Child Left Behind and its destruction of our educational system. Never has America had a leader who is so incorruptible, because there is nothing in George Bush that could be corrupted. To corrupt someone implies that they begin with some virtue, and it was difficult to think of any virtue known to man possessed by this President.

George W. Bush had death to sell to the Congress and the American people, the death of our young soldiers to be sacrificed to his desperate need for another chance, another big score, all part of his fantasy of success, and his dread of failure. As even the Democrats in Congress bobbed up and down in response to his lies and banalities, I was a bit confused, and annoyed; then I realized that nobody was paying close attention to his words, this sign of deference may have been an effort to stay awake, like the snoozing John McCain, or the jumping up and down of Nancy Pelosi to keep her foot from falling asleep. I expected Laura, like the loyal Linda Loman, to shout out from the balcony, "Attention must be paid," but instead she was playing a game of three card Monte, undoubtedly taught to her by Rove himself, exploiting the heroism of an African American working man, one who never enjoyed any of the benefits of Bush's America, to distract from her husband's failures and lend George some of this hero's aura. Perhaps the real Linda Loman was Condi Rice whose face was a mask of tragedy. Medea or Medusa, take your pick, it was awful to behold in its desperation for Condi like Laura and the Cheneys the tragedy wasn't what they had done to America, but what they had lost for themselves, power, respect, and honor. Perhaps the material profits of war are not enough for some people. Sadly, one knows that George W. will never have a moment when he understands how he went wrong, and what a disaster he has brought down on the American family. The big difference between that great play and this President is that you could weep for Willy Loman but never for this salesman of death."

Part of an excellent blog by Sherman Yellen.
~~~~~~~~~~~~~~~~~~~~~~~~~~~~~~~~~~~~~~~~~~~~~~~~

I was specially touched by his reference to the twisted, angonized face of Condi Rice when she was captured on camera. Her face told it all about the Bush years, the Bush failures...most specially the failure of the Iraqi War.

tomder55 answered on 01/25/07:

Yeah I can see the simularity with Willie Lowman . Like Willie ,President Bush is trying hard to be a leader of a dysfunctional family and manage in a dysfunctional world . His dreams and visons are noble and worthy of the effort.

while peddling terror and no taxes as if they were shiny new stock.

but he did propose tax increases in his health care plan.


He dragged out all the initiatives that he should have considered six years ago, which now seemed shopworn, threadbare, and counterfeit in his hands, new sources of energy, health care, and his disastrous No Child Left Behind and its destruction of our educational system.

The health care initiative is relatively new but the President has consistently tried to get Congress to act on energy independence . NCLB has been law since his first year and has been revised and tweeked every year since. I'm suprised that Sherman Yellen is not aware since the nationalizing of the education system is a Democrat socialist concept that I for one have real issues with . I on the other hand still think that the Dept. of Education should be abolished.

For those of us who believe in free markets and education choice , the President unveiled two voucher like initiatives .
He would require persistently underperforming schools to offer "Promise Scholarships" that would allow low-income students to transfer to private schools or out-of-district public schools. Federal funds would follow the students to their new schools. He would also offer grants through an "Opportunity Scholarships Program" that would help local governments offer school choice programs that include private schools. Why would the Democrats oppose that ? I thought they were pro-choice . Oh ,that's right ...only when it comes to a womens's right to kill a baby.

I would have liked to see him sing his own praises about the state of the economy a little more . Jobs continue to boom. So do real incomes, productivity and profits.Unemployment today is only 4.5 percent. Federal, state and local tax collections are soaring through the roof. Budget deficits are plunging. Inflation-adjusted GDP is averaging more than 3 percent. Family wealth stands at a record of slightly more than $54 trillion. Total employment is at a record 146 million employed .With the exception of the foolish new Marxists in Central and South America ;old Europe ;and jihadistan ;the rest of the world is imitating the American model .As free enterprise has been unleashed around the world, government planning once again has been rejected except where noted and in the mind of John Edwards and Dennis Kucinich.


As far as Iraq goes ;it is a good thing that President Bush didn't invoke the memory of Neville Chamberlain . The Democrats would've bruised their palms clapping so hard .President Bush, noted that "the consequences of failure would be grievous and far-reaching," presented a similar plan for Iraq with "the goal of victory."
When he said that I saw the collective group of Democrats pull out their dictionary . Evidently the word voctory is not part of their lexicon. For the Democrats, it's about ending the war in Iraq, not winning it.



He accurately described America as "a nation at war" against a "totalitarian threat," a "wicked" enemy that wants "to kill Americans, kill democracy in the Middle East, and gain the weapons to kill on an even more horrific scale." Perhaps after all this time the Democrats still don't get it ?

Since we are describing facial expressions and physical quirks what was with Speaker Meme's blinking ? It looked like she was sending out Morse code to Harry Reid. With all those face lifts I was surprised she can still blink.

I do not like the whole pomp behind the SOTU address . The Constituion states in Art. 2 sec.3 He shall from time to time give to the Congress information of the state of the union, and recommend to their consideration such measures as he shall judge necessary and expedient. If it were me ;I would once a year submit a list of my legislative 'wish list 'to Congress and instead would make a televised address to the nation. I would only go to Capital Hill when absolutely necessary. or ..if I did make a SOTU Address it would be short and sweet full of generalities and broad visions. I would not waste my time articulating a laundry list of suggestions for the likes of James Webb to nit-pick over .

If submitting the SOTU requirements in written form was good enough for every President from Jefferson to Wilson ,it is good enough for me.

Choux... rated this answer Excellent or Above Average Answer
ETWolverine rated this answer Excellent or Above Average Answer
Itsdb rated this answer Excellent or Above Average Answer

Question/Answer
Itsdb asked on 01/24/07 - What does this mean?

Yesterday at Petraeus' nomination hearing, Evita said this to the general:

    "You wrote the book, General, but the policy is not by the book. And you are being asked to square the circle, to find a military solution to a political crisis. I, among others on this committee, have put forward ideas about disapproving the escalation not because we in any way embrace failure or defeat, but because we are trying to get the attention of our government and the government of Iraq."


What is that supposed to mean, besides the notion that Hillary thinks he's being asked to do the impossible?

Steve

tomder55 answered on 01/24/07:

This is actually no worse then John Warner's parsing . He said he authored a bill opposed to the 'surge' that he will not submit......yet, because the President requested for someone to come up with a better idea.

One sponsor, Senator John W. Warner, Republican of Virginia, said the measure is “not meant to be confrontational” but rather was an acceptance of the president’s invitation to come forward with alternative plans, if they have any.

What this turn coat has up his sleave is to get a resolution on the Senate floor opposing the President's plan that can also garner enough Republican votes to reach the 65 vote count which would send a much more powerful message than the Democrat proposed resolution that may get around 51-52 votes.

But then in the hearing he had the juevos to try to justify this betrayal to General Petreaus .

We're not a division here today of patriots who support the troops and those who are making statements and working on resolutions that could be translated as aiding and abetting the enemy. We're trying to exercise the fundamental responsibilities of our democracy and how this nation has two co-equal branches of the government, each bearing its own responsibilities

Evita I expect it from .

Itsdb rated this answer Excellent or Above Average Answer

Question/Answer
ETWolverine asked on 01/24/07 - The smirk

I watched the State of the Union address and the Democratic rebuttal last night.

Was it just me, or did Jim Web look like he was smirking during his rebuttal?

How many times has Bush been accused of smirking when giving a speech? Why should we not apply the same standard to Web?

Just wondering.

Elliot

tomder55 answered on 01/24/07:

He otherwise looked subdued ;like someone had sedated him lest he become rabid


As I look at Iraq, I recall the words of former general and soon-to-be President Dwight Eisenhower during the dark days of the Korean War, which had fallen into a bloody stalemate. "When comes the end?" asked the General who had commanded our forces in Europe during World War Two. And as soon as he became President, he brought the Korean War to an end.

I would ask the Senator with the vivid imagination what the hell is he's talking about ?? What's in place there iscalled a truce.There was never any resolution .

I guess the fact that US troops still are on the front line in Korea never occured to him. If Ike took the advice of today's Democrats in Korea we would've withdrawn over the horizon to Japan and watched the NORKS overtake the whole penninsula.

The rest of his response was tired old populist clap-trap that Dick Gephart used to spew and John Edwards still does.

ETWolverine rated this answer Excellent or Above Average Answer
Itsdb rated this answer Excellent or Above Average Answer

Question/Answer
paraclete asked on 01/23/07 - Have you got yours yet?

Congratulations on the purchase of your genuine Government Official (TM). With regular maintenance your Government Official (TM) should provide you with a lifetime of sweetheart deals, insider information, preferential legislation and other fine services. Before you begin using your product, we would appreciate it if you would take the time to fill out this customer service card. This information will not be sold to any other party, and will be used solely to aid us in better fulfilling your future needs in political influence.

1. Which of our fine products did you buy?

* __ President
* __ Vice-President
* __ Senator
* __ Congressman
* __ Governor
* __ Cabinet Secretary - Commerce
* __ Cabinet Secretary - Other
* __ Other Elected Official (please specify) _________________
* __ Other Appointed Official (please specify) _________________


2. How did you hear about your Government Official (TM)?

(Please check all that apply)

* __ TV ad
* __ Magazine/newspaper ad
* __ Shared jail cell with
* __ Former partner of
* __ Unindicted co-conspirator with
* __ Procured for
* __ Related to
* __ Recommended by lobbyist
* __ Recommended by organized crime figure
* __ Frequently mentioned in conspiracy theories (on Internet)
* __ Frequently mentioned in conspiracy theories (elsewhere)
* __ Spoke at fundraiser at my temple
* __ Solicited bribe from me
* __ Attempted to seduce me


3. How do you expect to use your Government Official (TM)?

(Please check all that apply)

* __ Obtain lucrative government contracts
* __ Have my prejudices turned into law
* __ Obtain diplomatic concessions
* __ Obtain trade concessions
* __ Have embargo lifted from own nation/ally
* __ Have embargo imposed on enemy/rival nation/religious infidels
* __ Obtain patronage job for self/spouse/mistress
* __ Forestall military action against self/allies
* __ Instigate military action against enemies/aggressors/future targets
* __ Impede criminal/civil investigation of self/associates/spouse
* __ Obtain pardon for self/associates/spouse
* __ Inflict punitive legislation on class enemies/rivals/hated ethnic groups
* __ Inflict punitive regulation on business competitors/environmental exploiters/capitalist pigs


4. What factors influenced your purchase?

(Please check all that apply)

* __ Performance of currently owned model
* __ Reputation
* __ Price
* __ Appearance
* __ Party affiliation
* __ Professed beliefs of Government Official (TM)
* __ Actual beliefs of Government Official (TM)
* __ Orders from boss/superior officer/foreign government
* __ Blackmail
* __ Celebrity endorsement


5. Is this product intended as a replacement for a currently owned Government Official (TM)? ______

If you answered "yes," please indicate your reason(s) for changing models.

* __ Excessive operating/maintenance costs.
* __ Needs have grown beyond capacity of current model.
* __ Defect in current model:
* __ Dead
* __ Senile
* __ Indicted
* __ Convicted
* __ Resigned in disgrace
* __ Switched parties/beliefs
* __ Outbribed by competing interest

Thank you for your valuable time. Always remember: when you choose a "Government Official (TM)," you have chosen the best politician money can buy.

tomder55 answered on 01/24/07:

typically circulated on K Street here in the US .But here there is a discount for bulk purchases.

Itsdb rated this answer Excellent or Above Average Answer
paraclete rated this answer Excellent or Above Average Answer

Question/Answer
paraclete asked on 01/24/07 - Who said that?

Make the Pie Higher

I think we all agree, the past is over.
This is still a dangerous world.
It's a world of madmen
And uncertainty
And potential mental losses.

Rarely is the question asked
Is our children learning?
Will the highways of the internet
Become more few?
How many hands have I shaked?

They misunderestimate me.
I am a pitbull on the pantleg of opportunity.
I know that the human being and the fish
Can coexist.

Families is where our nation finds hope
Where our wings take dream.
Put food on your family!
Knock down the tollbooth!
Vulcanize society!
Make the pie higher!
Make the pie higher!

tomder55 answered on 01/24/07:

'They misunderestimate me' is a dead give-away. My extemporaneous speech compared to my other communication skills is also a lot to be desired . Thankfully all my utterances aren't examined through a fine tooth comb.

Bush has the good sense to look at this weakness of his humorously to the point that he even gave kudos to 'Bushisms'. Maybe when he is out of office he can cash in on them like Yogi Berra has done.

If you are not familiar ,Berra is a famous former Baseball player who is known for his various malaprops. His tendency to use them elevated him to the level of a folk philosopher simular to Will Rogers ,and many of them have become part of the lexicon ...[example when you come to a cross in the road take it ]. I fully expect that 'Bushisms ' will be part of President Bush's legacy,and they will be recalled fondly.

ETWolverine rated this answer Excellent or Above Average Answer
paraclete rated this answer Excellent or Above Average Answer

Question/Answer
Itsdb asked on 01/23/07 - Ever heard anything like this?

This may be a bit long, but...

    "The puppeteers who pull Bush's strings are pulling the strings of the evangelicals, as well as the rest of the homophobic, xenophobic scum who listen to Rush and Sean, looking for somebody to be better than." -AlreadyRaptured

    "Bob...everything you wrote was valid..except the part where you opine that one day W could tell the truth..he lives in a constructed world..he IS Pinnochio and his nose reaches well into the innards of the neo-cons who pull his strings." -halsey

    "The thing about the Bush presidency is that it governs for the benefit of very narrow group of family and friends. These folks who are the power behind the Bush presidency don't give a rip about the rest of us or the future of the nation." -Carol Davidek-Waller

    "The agenda is to occupy Iraq as a staging area for invasion and occupation of other middle-eastern countries such as Iran and Syria. This is not news, and this is not a secret. This is the published plan of the PNAC, which pulls the strings on most of Bush the Fool's foreign policy." -Natureboy

    "What we know for sure is that Bush is beholden to the Christian right..." -Katha Pollitt

    "it’s not fiscal conservatives to whom Bush is beholden, but the religious right who seem to forgive all other sins against the movement this president makes as long as he pays lip service to their pet causes." -Derek Phillips


Get the point? Good...

    New Coalition Aims To Keep Democrats Loyal to Populist Issues

    By JIM KUHNHENN
    Associated Press
    January 23, 2007

    WASHINGTON — Looking to instill discipline among Democrats, a coalition of labor, trial lawyers, and liberal groups is launching lobbying and campaign organizations this week to keep Democratic lawmakers from straying on populist issues.

    Democrats who don't hew to this agenda could find themselves facing well-funded primary opponents — an aggressive strategy to counter moderate and conservative blocs within the party.

    The groups have organized as two entities — a lobbying wing called They Work For Us and a campaign arm called Working for Us PAC.

    "Our PAC will encourage Democrats to act like Democrats — and if they don't — they better get out of the way," Steve Rosenthal, one of the coalition's main organizers, wrote in a memorandum describing the organization.

    The new effort is emerging as House Democrats conclude the first stage of their legislative agenda — a two-week rush of legislation that had wide appeal within the party and even among Republicans.

    "Now, we move into issues like improving access to health care, fighting jobs moving overseas, protecting rights of workers to organize," Mr. Rosenthal, who is a leading labor political strategist, said in an interview. "It's really going to take a very strong effort for Democrats to keep people in line."

    Mr. Rosenthal founded America Coming Together, a political organization that mobilized Democratic voters in the 2004 presidential election.

    In addition to Mr. Rosenthal, the two groups are led by some of the most influential organizers in labor and liberal politics, including the secretary-treasurer of the Service Employees International Union, Anna Burger; the executive director of MoveOn.org Political Action, Eli Pariser, and a senior vice president at the American Association for Justice, formerly the American Trial Lawyers Association, Linda Lipsen.

    The lobbying arm is organized as a nonprofit organization and already has raised $200,000 toward a goal of more than $3 million for the next two years, Mr. Rosenthal said. It would concentrate on certain key congressional districts to pressure moderate or conservative Democrats to vote with the party's leadership.

    The labor wing of the Democratic Party has been especially frustrated by Democrats who have voted in favor of trade deals that union leaders said hurt American workers by driving jobs overseas. The new political action committee, Mr. Rosenthal said, would be ready to recruit more liberal Democrats to run against them.

    "If Democrats in Congress don't stand strong and fight for the goals we are supposed to share, then progressives need to agitate so they will, or work hard to replace them," said Donna Edwards, a member of the They Work for Us board who unsuccessfully challenged Democrat Al Wynn, a 14-year House veteran from Maryland, in last year's Democratic primary.

    Keeping the party unified won't be easy for the House speaker, Rep. Nancy Pelosi of California, as she moves ahead with the rest of the Democratic agenda. Many of the new Democratic lawmakers elected in November replaced incumbents in typically Republican districts. That could make it difficult for many to support some of the party's stances on economic and social issues.

    Mr. Rosenthal said the new coalition would not single out members with moderate to conservative constituencies. Instead, it will target lawmakers who coalition organizers believe are out of step with their voters.

    "We're not trying to force people to the left of their districts," he said. "We want to make sure that Democrats primarily represent their districts."


Let me get this straight, Bush is just a puppet whose strings are pulled by, take your pick; big business, insurance & drug companies, neocons, the religous right, PNAC, etc., but it's critical that someone pull the Democrats' strings? It's outrageous for Bush to listen to an evangelical but MoveOn, ACT, and co. can beat straying Democrats into submission for the liberal cause? How much have we heard about "the Bush regime" and its silencing dissent, or as one blogger put it about Republicans, "they will walk in lockstep right off the cliff, taking the country down with them?"

So much for bipartisanship and open minds, eh?

tomder55 answered on 01/23/07:

Well I think you nailed it .

This was the relevent part of the Moveon.org e-mail from Dec 9,2004

Dear MoveOn member,

Who will lead the Democratic Party? The answer may come as soon as this weekend, when the state Democratic Party leaders gather to discuss who should chair the Democratic National Committee (DNC) for the next four years.1 The election for chair is rarely competitive. But this year, with the race wide open, we have the chance to elect a leader who will reconnect the Democratic Party with its constituents -- us.

For years, the Party has been lead by elite Washington insiders who are closer to corporate lobbyists than they are to the Democratic base. But we can't afford four more years of leadership by a consulting class of professional election losers. In the last year, grassroots contributors like us gave more than $300 million to the Kerry campaign and the DNC, and proved that the Party doesn't need corporate cash to be competitive.Now it's our Party: we bought it, we own it, and we're going to take it back.


The Democrats are under the ownership of George Soros . He spent $18 million trying to influence the outcome of the 2004 election . It's not his fault that Kerry was incompetent . Front Page Mag. goes into greater details. The call the puppet masters of the Democrats the "shadow party ". Also it is informative to read about the Thunder Road Group .

Hillary is to be installed at their bidding .


Itsdb rated this answer Excellent or Above Average Answer

Question/Answer
Itsdb asked on 01/22/07 - Congratulations

I know this has nothing to do with politics but congratulations to da Bears and the Colts on their Super Bowl berths. I had hoped New Orleans could pull it off - shame on Sean Payton for calling 3 pass plays in a row in the third quarter instead of slamming out a few yards to get their kicker a bit closer, and losing all the momentum they had just gained - but I digress. It was good to see the Colts get over the first hump.

Congrats also to both coaches, although Dungy got the short shrift in my paper this morning with not one, but two articles heralding Lovie Smith's being the first black head coach to reach the big game - they'll play the game at the same time won't they? Also, if everyone wants us to get over racism (so maybe there is a bit of politics here), if it really isn't about "the color of a man's skin," why does everyone keep bringing it up?

Lastly, as if anyone else here will care, Parcells called it quits. What wannabe or has been coach will Jerry get next? Could he possibly talk Cowher into coming to town? If the Steelers hire Tomlin could that send Russ Grimm to Dallas and give the 'Boys another legend from a rivalry of old? Will Peyton win the big one?

tomder55 answered on 01/22/07:

When da Bears marched to a score right before half time and did not need to throw a pass once I knew the game was over .

As for the Colts ,I'm rooting for em . What was interesting is that all season the critique on Eli Manning was that his mechanics were flawed ;that he threw off of his back foot. But I saw brother Peyton throw from all over the place last night and still complete passes with pin point accuracy .I'd wager no one complains about his mechanics. He will take his flawed mechanics to Canton .

Parcells tried to get in line for the Giants GM job . They wisely said no thanks .True to form ;Parcells was on his 3 year plan with the Boys .Time for him to retire for good . I really liked him as a Giant coach ,but what has been revealed in the last decade is that he can win a Super Bowl with Bill Belichick on his staff and not without him .Meanwhile Belichick has had a nice run on his own without Parcells .

Itsdb rated this answer Excellent or Above Average Answer

Question/Answer
tropicalstorm asked on 01/22/07 - What went wrong?

Back in the sixties I was all for the within the next century there will be a woman President. Women will straighten out the mess men made through the century.
Yeah, go for it the world will be in a much better shape...A Leave it to Beaver mom as President!
Now the thoughts of a woman President is scary-oooh!
With woman like Hill-ar-i-ous, Nancy P, Nancy F, Barbra Boxer and Cindy Shehan it scares me to even be of the female race.
Excuse me while I go have a nervous breakdown at the mere thought!
It's soooo scary--oooh that.....I forgot the ?

tomder55 answered on 01/22/07:

my fantasy was an all Amazon army.




oh well ...

with Nancy convincing the country that it takes a woman to clean the house (I'm sure the feminists love the way she phrases that )and Madame Defarge Clinton trying to prove she can be a manly chief executive I'm sure the glass ceiling will soon be broken through (hey they already cracked the glass rotunda).

The down side of course will be that if they are as disasterous as I expect them to be ;womens progress will be set back a century .

tropicalstorm rated this answer Excellent or Above Average Answer
Itsdb rated this answer Excellent or Above Average Answer

Question/Answer
HANK1 asked on 01/21/07 - THE UNITED NATIONS:



What's this group of idiots doing these days?

HANK

tomder55 answered on 01/22/07:

Hank .

I will start with the good :
1. They passed sanctions against Iran in December but enforcement is limited to non-military means (economic). Iran defiant as ever announced this weekend that they will conduct missile exercises designed to close the Straits of Hormuz. So as ususal ,the UN passed resolutions without any teeth.

2.New Sec. Gen. Ban Ki-moon called Holocaust deniers 'misguided individuals'in a direct slap at the Mahdi-hatter.

3. Ban Ki-Moon, is floating a proposal to downgrade the Department of Disarmament Affairs .... And considering putting an American in charge of it...and it's pissing off peaceniks .


now for the rest :

1. UN "peace keepers" in Haiti are being accused of a massacre.

2.The UN mission in
Liberia is accused of sexual abuse.

3.Hamadoun Toure , the new head of the UN's International Telecommunications Union says that he does not plan to take over governance of the internet. yeah ,right !!I believe that !

4.Ban Ki-moon called for outside examination of all United Nations activities after a published report that the United Nations Development Program in North Korea was making unaccountable payments in hard currency to local staff members and the NORK government. This reeks of another Oil-for-food scheme.

The NORK scandal is of course only the tip of the iceberg with this corrupt UN agency .It has been complicit in aiding Uzbekistan's repressive Karimov regime,and Robert Mugabe's in Zimbabwe . The UNDP is evidently a dictator aid agency .

HANK1 rated this answer Excellent or Above Average Answer

Question/Answer
tropicalstorm asked on 01/21/07 - 2009-12

And Democrats are in control of EVERYTHING
A. Cut and run
B. Stay and do what bush woulda done if they hadn't tied his hands. Then act like it was THEIR plan.
C. Try to reason with the snakes to not bite.
D. Keep on doing the Kerry I am for the war/against the war.
E. Other and what?

tomder55 answered on 01/21/07:

please... you are giving me nightmares !

The answer is A

The example is in the conclusion of the Vietnam war. The Democrats did not particularily care about the genocides that happened in the wake of our retreat in both Vietnam and Cambodia. Their concience was clear so long as they had Richard Nixon to blame for the outcome.

tropicalstorm rated this answer Excellent or Above Average Answer

Question/Answer
tropicalstorm asked on 01/21/07 - Bush bashing

Like I've said before I am not a Bush-bot, but I just don't get the democrats they tie Bush's hands on the war then they say send more troops, then they say cut the funds, cut and run, then they 'say reason with them'. I swear they just say the opposite of whatever Bush wants just to make him look bad.

tomder55 answered on 01/21/07:

The problem with their latest take about negotiations is this ;why would you want to go into negotiations ...assuming they are desirable ..and weaken your bargaining position before you sit down at the table ? If they are looking for a negotiated settlement you would think that they would at least for appearance sake present a united front . Maybe they are just too stupid to understand how their irresponsibe rhetoric undemines the very position they advocate .

Itsdb rated this answer Excellent or Above Average Answer
labman rated this answer Excellent or Above Average Answer
tropicalstorm rated this answer Excellent or Above Average Answer

Question/Answer
HANK1 asked on 01/20/07 - HILLARY:



She threw her hat in the ring for President today over the Internet instead of going to the news media. She said she would end the war in Iraq if elected. Tell me -- why doesn't she tell us her formula for ending the war NOW so she can save lives? In two years, we might OWN Iraq.

HANK

tomder55 answered on 01/21/07:

Wasn't that announcement a shocker ?The MSM is hyperventilating in it's excitement.

Hank ,as you probably have noticed ,Hillary rarely puts herself into a position where she has to field questions from a journalist who may challenge her views. That is why she went virtual in her announcement and who on Monday she will have a virtual town hall like Q&A session. Look for her to break this pattern by going on 'The View' for a hard hitting interview with Rosie.

She said she was "starting a conversation with America" and also invited voters to voice their opinions on health care, Social Security, Medicare and the Iraqi war. In 1999, when she was running for Senator of New York, Hillary was traveling around the state on her "listening tour." Perhaps one of these day she will go on tour and honestly tell us exactly where she stands on the issues.She is the chamillion that all the other Trojan Horse Democrats emulate.

Maybe while we "chat" she can explain to us how the Rose Law Firm records and FBI files of 340 Republican officials ended up in her living quarters in the White House.

Hillary voted for the Iraq war .Up until very recently she hedged her bets and was a fence sitter . In the days before her announcement she travelled to Iraq and Afghanistan with her presumptive VP nominee Evan Bayh ,and evidently has had a metamorphoses .She has now joined the ranks of the retreat and defeat Democrats.

Dark_Crow rated this answer Excellent or Above Average Answer
HANK1 rated this answer Excellent or Above Average Answer
Itsdb rated this answer Excellent or Above Average Answer

Question/Answer
Itsdb asked on 01/20/07 - You're kidding me

Yesterday, Grandma Pelosi said "The president knows that because the troops are in harm's way, that we won't cut off the resources. That's why he's moving so quickly to put them in harm's way..."

Really? Does anyone here, even the most ardent critic, really believe Bush wants to put Americans in harm's way? Even so, even if any reasonably sane American believes Bush gets some thrill or is so obsessed with winning or proving he was right about Iraq that he would intentionally sacrifice more troops, should an American congressperson ever make such a pathetically outrageous statement?

tomder55 answered on 01/21/07:

Now that the Dems have picked the easy fruit off the bottom of the tree ,they will realize just how incompetent a leader Pelosi is . She pissed off John Dingle at the end of the week with her announcing her desire to create a global warming panel. Even Charles Rangel mouthed off against some of her tax scheming .

In case, there were any doubts that Democrats want to lose in Iraq, Pelosi just put an end to that debate.But as she spouts her borderline treason she at least looks preternaturally fresh .

Does granny feel any obligation to come up with an alternative plan ? I kinda doubt it; It's too much fun hurling barbs from the cheap seats. What we are getting from the Democrats ,and Speaker Pelosi in particular is over-heated rhetoric coupled with feckless action. They will continue along that path so long as the American people let them get away with it.

She is in power today because of one and only one reason ;the country's discontent on the way Iraq has gone down. Her problem is that the only Democrat alternative on the table is withdrawal,and it would be completely irreponsible for them to cut off the funding like they did in the mid 1970s to achieve it.But that is what they really want.

She has to construct a pr campaign to achieve this goal of withdrawal while at the same time putting the blame of the inevidible disasters that will follow on President Bush . She is setting the table with comments like the one you quote.

Itsdb rated this answer Excellent or Above Average Answer

Question/Answer
Choux... asked on 01/20/07 - THE AGE OF GLOBAL WARMING DENIAL IS OVER

"An unprecedented coalition of blue-chip US companies and environmental lobby groups will urge President Bush next week to get serious about global warming, calling for caps on carbon dioxide emissions that would cut greenhouse gases by 10-30 per cent over 15 years.

The group, called the US Climate Action Partnership, will unveil the details of its plan on the eve of President Bush's State of the Union speech on Tuesday. The companies involved include some of the old-fashioned pollution-generating industries normally associated with anti-environmental policies and politicians - the chemical giant DuPont, the bulldozer company Caterpillar, the aluminium producer Alcoa and the US subsidiary of BP.

They, and environmental lobby groups such as Environmental Defense and the Natural Resources Defense Council, said yesterday they will call for "swift federal action on reducing greenhouse gas emissions and speeding the adoption of climate-friendly technology".

The initiative was the latest of several indications of a big shift in US attitudes on global warming. The two-week-old new Democrat-led Congress has already generated a flurry of bills offering emissions-reduction targets. Nancy Pelosi, the new Speaker, is setting up a dedicated climate change committee in the House of Representatives with the power to recommend legislation.

Ms Pelosi has also promised a legislative package on energy independence, to be delivered by Indepedence Day on 4 July. Her enthusiasm is mirrored in the Senate by Barbara Boxer, the incoming chair of the Environment and Public Works Committee, who has called the fight against global warming her number-one priority.

The change in attitudes goes beyond the political arena. The star feature of the Detroit Auto Show last week was a plug-in hybrid vehicle being developed from General Motors.

The age of global warming denial, meanwhile, also appears to be drawing to a close. Exxon Mobil, the world's largest oil company, has cut its funding to groups who argue global warming is a hoax, and is now working to develop strategies it can accept for emissions reduction.

That's a huge change from just a few months ago, when Exxon Mobil's chief executive, Lee Raymond, arguably the world's most prominent global warming sceptic, was still at the helm, and the Senate Energy Committee was headed by the Oklahoma Republican James Inhofe, who made it his business to dismiss scientific opinion on climate change as a conspiracy.

The biggest hold-out against radical policy change is probably the Bush White House. Aides to the President have indicated his State of the Union speech will include some provisions on energy, notably championing the use of ethanol-based fuels. The administration remains opposed, however, to any mandatory caps on carbon dioxide emissions.

The White House is likely to come under increasing pressure to do something, however. One possible route has already been taken by Mr Bush's fellow Republican, California's Governor Arnold Schwarzenegger, who has endorsed a 25 per cent reduction in greenhouse gases in his state by the year 2020.

The Schwarzenegger plan does not operate on a rigid system of emissions caps, but rather offers incentives to companies who move faster than their competitors, who can "trade" their margin of emissions reduction with companies lagging behind. The "cap and trade" system contrasts with a bill championed by Senator Boxer, to mandate a reduction in emissions to 1990 levels by 2020." The Independent, UK
~~~~~~~~~~~~~~~~~~~~~~~~~~~~~~~~~~~~~~~~~~~~~~~~

The War on Iraq is lost.
Bush/Cheney have disgraced their offices.
The Republican party is in disgrace over ethical and sexual problems.
Moderate Americans have had it with religious extremists like Fawell and Robertson and all the loopy "religious" fanatics.

Ken Mehlman warned in a speech today that the Republican Party must *"clean up their act"* and bring in *minorities* or face being a permanent minority party, loosing the 2008 presidential election and Congressional seats.....far into the future.

SO WELCOME TO THE 21ST CENTURY, POLITICOS.

***THE FIGHT IS OVER...THE DEMOCRATS WIN***
















tomder55 answered on 01/20/07:

The companies in the group are General Electric, DuPont, Alcoa, Caterpillar, Duke Energy, PG&E, FPL Group, PNM Resources, BP, and Lehman Brothers.

I for one am happy that these companies want to take environmental initiatives. GE for one can finish cleaning up the Hudson River ,since they almost single handedly destroyed the river by dumping PCBs into it;and they have been under court order for a long time to remedy the situation.

I'm sure all these corporations already meet the goals they are proposing . lol
The whole group of them are Greenwashers [Greenwashing is a form of public relations propaganda which gives something the appearance of being environmentally friendly when it is, in fact, not. ]

What they are really looking for is tax breaks for meeting these goals. What they are looking for is some more of the same flawed pollution-credit schemes that have failed to make any difference under the Kyoto years. The availability of pollution credits tells the corporation that it's just great to pollute, as long as they can afford to pay off someone else who isn't using their allotted amounts of pollution.
I could get rich real quick . All I need to do is to set up a dummy corporation and fail to pollute. That will leave me plenty of carbon credits to sell to GE or Alcoa.

When a cause becomes political it is a good idea to examine the motives of the players. But it doesn't supprise me at all that polluters are asking the Federal Gvt. to clean up their mess.

What is actually amazing to me is that enviro-wacko organizations like Environmental Defense, Natural Resources Defense Council, Pew Center on Global Climate Change, and World ResourcesInstitute could be fooled by such a disengenuous attempt. If I was an environmental advocate I would be very suspicious of this marriage of convenience between some of the worse polluters in the country and what are apparently sell-out environmental groups.

I would also be very suspicious of the timing of this announcement ;days before the SOTU address by President Bush and rapid developments in the legislative front .These companies are represented by the biggest lobbiests and represent some of the largest sources of campaign contributions .

If I was an enviro-wacko I'd look closely at the fine print.But then again;if I was an eco-nut I would also be opposed to any measure that didn't shut down the world economy over night. I'd also have some negative things to say about biofuels, and nuclear energy even though they are clearly part of the remedy in reducing man made green house emissions.

~~~~~~~~~~~~~~~~~~~~~~~~~~~~~~~~~~~~~~~
the rest of your comments are irrelevent to the post so I will ignore them at this time.

Choux... rated this answer Poor or Incomplete Answer

Question/Answer
Itsdb asked on 01/19/07 - Space war heating up?

Russia first denied it as a rumor, but...

Three nations join China test protest
CHISAKI WATANABE
Associated Press

TOKYO - Britain, Japan and Australia joined the United States on Friday in voicing concern about the rising militarization of space after China successfully carried out a test of an anti-satellite weapon.

But Russia expressed skepticism about the Jan. 11 test, in which an old Chinese weather satellite was destroyed by a ballistic missile.

In Beijing, meanwhile, a foreign ministry spokesman said Friday he was unaware of such a test. The spokesman, Liu Jianchao, added that China was against the militarization of space.

In Washington, Gordin Johndroe said the United States had expressed its concern to China.

"The United States believes China's development and testing of such weapons is inconsistent with the spirit of cooperation that both countries aspire to in the civil space area," Johndroe said Thursday.

Analysts said China's weather satellites travel at about the same altitude as U.S. spy satellites, so the test represented an indirect threat to American defense systems.

Officials in Japan, Britain and Australia immediately demanded China explain its actions.

"We must use space peacefully," Japanese Prime Minister Shinzo Abe told reporters. "We are asking the Chinese government about the test."

Japanese Foreign Minister Taro Aso criticized Beijing for failing to give advance notice. He also said debris from the test could scatter in the atmosphere.

"We told China that we doubt if we could call this a peaceful use," Aso said at a news conference.

A spokesman for British Prime Minister Tony Blair said diplomats had complained to their counterparts in Beijing about the lack of consultation with the international community.

They also relayed their concern that debris from the test would strike other satellites orbiting the Earth, said Blair's official spokesman, who spoke on condition of anonymity in line with government policy.

Blair's spokesman said the British government did not believe China's test contravened international law.

"However, we believe that this development of technology and the manner in which this test was conducted is inconsistent with the spirit of China's statement to the U.N. and other bodies on the military use of space," the spokesman said.

Australian Foreign Minister Alexander Downer, who was in New York, said his government opposed the test and had called upon Beijing's ambassador to Australia for an explanation.

"Our concern about this is that to have a capacity to shoot down satellites in outer space is not consistent with ... the traditional Chinese position of opposition to the militarization of outer space," he told reporters.

"So we've asked the Chinese for an explanation as to what this may mean," Downer said, adding that so far Chinese officials, including the ambassador in Canberra, said they were not aware of the test.

Russian Defense Minister Sergei Ivanov voiced doubts about details of the test.

"I'm afraid that it didn't have an anti-satellite basis. And, maybe, it's good that it didn't," Sergei Ivanov said in televised remarks, adding that Russia was against militarization of space.

He did not elaborate and it was unclear whether he questioning the success of the test or its intent.

RIA Novosti news agency quoted retired Col. Gen. Leonid Ivashov, the former head of the Russian defense ministry's international military cooperation department, as saying that the Chinese weapon was modeled on the Soviet IS-1 missile designed to destroy satellites, developed in the 1970s.

Another Russian military expert, Maj. Gen. Vyacheslav Fateyev, criticized the Chinese test as "hooliganism," but added that "it shows that Beijing has a strong capability," ITAR-Tass said.

It also quoted Russian Lt. Gen. Leonid Sazhin as saying that the Chinese test was a response to the United States developing space-based weapons. "China tested anti-satellite weapons to ensure its security," Sazhin said, according to ITAR-Tass.

Russia and China have forged what they called "strategic partnership" after ending their Cold War rivalry, pledging commitment to a "multipolar world" - a term that highlights their opposition to perceived U.S. domination in global affairs.

South Korea has also conveyed its concerns to China, according to the Foreign Ministry.
~~~~~~~~~~~~~~~~~~~~~~~~~~~~~~~~~~~~~~~~~~~~~~~~~~~~~~

Comments?

tomder55 answered on 01/19/07:

oh I see . They shoot down a sattelite and it is Bush's fault for developing defensive systems that may save a US city from an incoming NORK nuclear missile .

Itsdb rated this answer Excellent or Above Average Answer

Question/Answer
paraclete asked on 01/18/07 - Being a Christian is not a good idea for a politician?

It appears that, if you are a politician, you need to be very careful which religious organisations you endorse. However fearless as ever, John Howard has demonstrated that the Muslim vote can do whatever it likes.

John Howard in religious hate row

By Gerard McManus and Michael Harvey

January 19, 2007 01:00am
Article from: Herald-Sun


JOHN Howard has been caught in a religious storm after taping a goodwill message for a fundamentalist Christian group accused of inciting anti-Islamic hatred.

Islamic leaders last night condemned the Prime Minister for appearing in a DVD message for Catch the Fire ministries.

The controversial movement has sponsored a major multi-denomination gathering in Melbourne on Australia Day.

But the PM's decision to back the event has been described as "dangerous".

Muslim community leaders said Mr Howard risked legitimising hateful anti-Islamic views.

"It sends quite a dangerous message to mainstream Australia that extremist views and hate speech is normal and justified," said former president of the Islamic Council of Victoria, Yasser Soliman.

The outcry comes as federal police launched an investigation into inflammatory comments by Sydney's Sheik Feiz Mohammed, which included a description of Jews as pigs and calls for children to die as "martyrs of faith".

The hateful comments appeared in DVDs of Sheik Feiz's lectures, being sold in Australia and the UK.

Acting Attorney-General Kevin Andrews said the Government was worried about a pattern of behaviour among outspoken Islamic leaders.

Mr Andrews described the comments as reprehensible and offensive.

Federal Opposition Leader Kevin Rudd called the sheik's rant obscene and demanded action.

"As I see it, Sheik Mohammed's comments add up to an incitement to terrorism," Mr Rudd said.

Catch the Fire has been at the centre of accusations two of its pastors exposed Muslims to hatred and ridicule.

Friday's Christian rally at Festival Hall is expected to attract up to 5000 people of various denominations.

Mr Howard is being promoted in flyers as delivering a keynote message which will be "personally directed to Catch the Fire ministries".

The rally will also say prayers against terrorism, for divine protection for Australia's armed forces and for the Government, according to organisers.

Pastor Danny Nalliah, one of two Catch the Fire ministers charged with breaking state vilification laws in 2002, last night refused to release the PM's message.

"I am not at liberty to say what the Prime Minister says in the message," Pastor Nalliah said.

"We asked him to speak on Australian values.

"This is a really big breakthrough to have all Christian denominations working together.

"This has been my heart and passion."

Last month the Victorian Court of Appeal threw out the charges brought in the Victorian Civil and Administrative Appeals Tribunal which had sought to jail Pastor Nalliah and Daniel Scot for allegedly inciting hatred, contempt, revulsion and ridicule of Muslims.

The pair have endured a long-running legal case, which has cost them about $150,000, but pledged to go to jail rather than retract their views.

Mr Soliman, a member of the PM's Muslim Community Reference Group, said Mr Howard should think twice about addressing the group.

"It is a dangerous thing for a senior politician to do," he said.

"I can't stop the Prime Minister addressing who he wants to, but he should be very cautious, especially with groups which have a history of toxic-hate speech."

Islamic Council of Victoria board member Waleed Aly described Catch the Fire as "spectacularly ignorant", claiming its members were in alliance with the far-Right League of Rights.

"If the PM wants to associate with these people, it's not a good look," Mr Aly said.

Other leading politicians who have addressed Catch the Fire ministries include Deputy Prime Minister Mark Vaile and Treasurer Peter Costello.

Mr Howard was travelling last night and a spokesman said he was unavailable to comment.

tomder55 answered on 01/19/07:

I wish I could be there . John Howard is my kind of leader . Imagine the gall of him inciting and inflaming by saying prayers against terrorism, for divine protection for Australia's armed forces and for the Government.....and speaking about Australian values . The nerve!! Obviously that qualifies as toxic-hate speech

paraclete rated this answer Excellent or Above Average Answer

Question/Answer
Itsdb asked on 01/18/07 - A plea from the darkness of Guantanamo Bay

GUANTANAMO BAY NAVAL BASE, Cuba - I am writing from the darkness of the U.S. detention camp at Guantanamo in the hope that I can make our voices heard by the world. My hand quivers as I hold the pen.

In January 2002, I was picked up in Pakistan, blindfolded, shackled, drugged and loaded onto a plane flown to Cuba. When we got off the plane in Guantanamo, we did not know where we were. They took us to Camp X-Ray and locked us in cages with two buckets - one empty and one filled with water. We were to urinate in one and wash in the other.

At Guantanamo, soldiers have assaulted me, placed me in solitary confinement, threatened to kill me, threatened to kill my daughter and told me I will stay in Cuba for the rest of my life. They have deprived me of sleep, forced me to listen to extremely loud music and shined intense lights in my face. They have placed me in cold rooms for hours without food, drink or the ability to go to the bathroom or wash for prayers. They have wrapped me in the Israeli flag and told me there is a holy war between the Cross and the Star of David on one hand and the Crescent on the other. They have beaten me unconscious.

What I write here is not what my imagination fancies or my insanity dictates. These are verifiable facts witnessed by other detainees, representatives of the Red Cross, interrogators and translators.

During the first few years at Guantanamo, I was interrogated many times. My interrogators told me that they wanted me to admit that I am from al-Qaida and that I was involved in the terrorist attacks on the United States. I told them that I have no connection to what they described. I am not a member of al-Qaida. I did not encourage anyone to go fight for al-Qaida.

Al-Qaida and Osama bin Laden have done nothing but kill and denigrate a religion. I never fought, and I never carried a weapon. I like the United States, and I am not an enemy. I have lived in the United States, and I wanted to become a citizen.

I know that the soldiers who did bad things to me represent themselves, not the United States. And I have to say that not all American soldiers stationed in Cuba tortured us or mistreated us. There were soldiers who treated us very humanely. Some even cried when they witnessed our dire conditions.

Once, in Camp Delta, a soldier apologized to me and offered me hot chocolate and cookies. When I thanked him, he said, "I do not need you to thank me." I include this because I do not want readers to think that I fault all Americans.

But, why, after five years, is there no conclusion to the situation at Guantanamo?

For how long will fathers, mothers, wives, siblings and children cry for their imprisoned loved ones? For how long will my daughter have to ask about my return? The answers can only be found with the fair-minded people of America.

I would rather die than stay here forever, and I have tried to commit suicide many times. The purpose of Guantanamo is to destroy people, and I have been destroyed. I am hopeless because our voices are not heard from the depths of the detention center.

If I die, please remember that there was a human being named Jumah at Guantanamo whose beliefs, dignity and humanity were abused. Please remember that there are hundreds of detainees at Guantanamo suffering the same misfortune. They have not been charged with any crimes. They have not been accused of taking any action against the United States.

Show the world the letters I gave you. Let the world read them. Let the world know the agony of the detainees in Cuba.

Jumah al-Dossari is a 33-year-old citizen of Bahrain. This column was excerpted from letters he wrote to his attorneys and published originally in The Los Angeles Times. Its contents have been deemed unclassified by the Department of Defense.
~~~~~~~~~~~~~~~~~~~~~~~~~~~~~~~~~~~~~~~~~~~~~~~~~~~~~~

Perhaps you've already read al-Dossari's account - seeing as how it's been circulated worldwide by virtually every media outlet and left-wing blogger. I can't speak on Mr. al-Dossari's guilt or innocence, but
why are we allowing this guy to hold the U.S. hostage from his prison cell?

Seems we have some things terribly backward here, we have a guy from Bahrain that says he's been to the U.S., likes the U.S. and wants to become a citizen, yet in his lengthy account doesn't seem to tell us why he was in Afhganistan, headed to Pakistan precisely when we were prosecuting a war there.

Then, we have the story of a black college student in North Carolina that's been so oppressed to be - as Kathleen Parker put it - "reduced by centuries of white-male oppression to stripping for food and tuition."

Before any charges were filed, "students produced a "wanted" poster with photos of team members and demonstrated with signs reading, "It's Sunday morning, time to confess." Not only that, a "Group of 88" Duke faculty members "ran an ad demanding that the lacrosse team players confess."

So we convict the rich white kids, demanding a confession, but exonerate the guy in Afghanistan. Does that make sense?

tomder55 answered on 01/18/07:

In my days out of control I would on ocassion go to a bar where women of all colors stripped . It was good money ;lots of tips . The men in the bar were also all colors of the rainbow coalition. So maybe it was just a male dominated culture.

Except

I also knew women who's ladies night out was to frequent male-stripper clubs . Those guys were of all color and they made some nice pocket change dancing .

So I think the notion that this women was a victim because she was a stripper is one of the lamest excuses I've ever heard for her completly criminal and irresponsible behavior. She and that idiot prosecutor have criminally defamed the reputations of these students and put them and their families through hell.

The Duke faculty members involved should be dismissed and sued by the families .Mike Nifong should be disbarred and spend some quality time in jail .He played the role of Al Sharpton (Tawana Brawley case ); as a provocateur and I have no doubt that he prompted the stripper and coached her many story revisions.

I often take vacation time on the Afghan-Pakistan border ;especially during times of pitched battle. Don't you ?

Itsdb rated this answer Excellent or Above Average Answer

Question/Answer
paraclete asked on 01/17/07 - Sacre Blu - est impossible, mon ami?


French close to kissing Britain

January 16, 2007 12:00
Article from: The Daily Telegraph


WOULD France have been better off under Queen Elizabeth II?

The revelation that the French government proposed a union of Britain and France in 1956 - even offering to accept the sovereignty of the British Queen - has left scholars on both sides of the Channel scratching their heads.

Newly discovered documents in Britain's National Archives show how former French Prime Minister Guy Mollet discussed the possibility of a merger between the two countries with British Prime Minister Sir Anthony Eden.

"I completely fell off my seat,'' said Richard Vinen, an expert in French history at King's College in London. "It's such a bizarre thing to propose.''

Eden rejected the idea of a union but was more favourable to a French proposal to join the Commonwealth, according to the documents - one of which said Mollet "had not thought there need be difficulty over France accepting the headship of her Majesty''.

While the two nations - separated by a thin body of water - have been bitter rivals since the Middle Ages, the two EU partners now concentrate on trading tourists rather than arrows.

What animosity remains has been relegated to name-calling, with the French and British reduced to froggies and rosbifs (roast beef) respectively.

But proposals for Anglo-French unity are not new.

Winston Churchill, in a last-ditch attempt to keep France on the side of the Allies in World War II, appealed for a full union of the two nations in June of 1940.

After the war, Ernest Bevin, Britain's foreign secretary, also toyed with the idea of a "Western Union'', a European - and African - bloc led by Britain and France.

The proposals all shared an element of desperation, said Kevin Ruane, a historian at Canterbury Christ Church University, England.

"It's so impracticable an idea that it has only been raised in extreme situations,'' he said.

Threatened by an Arab revolt in French Algeria and hobbled by instability at home, France was desperate to maintain its independence from both the Soviet Union and the United States, Ruane said.

Eden, who fought in France during World War I and spoke the language fluently, might have seemed particularly approachable to Mollet, a former English teacher.

But the former French leader's memoirs showed nothing about the proposal, said Francois Lafon, a history professor at La Sorbonne in Paris and a Mollet biographer.

Lafon suggested it was probably just a political tactic to press the British to firm up their role for the imminent attack on Egypt, whose leader, Gamal Abdel Nasser, the French accused of supporting the Algerian insurgents.

But even under the circumstances, the suggestion that France would accept the British Queen struck historians as bizarre.

Mollet was a socialist, and left-wing Frenchmen looked to the execution of French King Louis XVI as one of the crowning achievements of the French Revolution.

They would have been unlikely to welcome a foreign monarch with open arms. "It must have been some kind of eccentric gesture,'' Vinen said.

A year after Britain turned down France's proposed merger, the French joined the Common Market, the European Union's predecessor.

By the time Britain tried to join the group seven years later, Charles De Gaulle had largely revived France's international standing, even as Britain's economy continued to stagnate.

De Gaulle vetoed Britain's attempts to join the European Economic Community - twice.

"In retrospect, the irony of this was that the losers were the British,'' Vinen said. "Maybe we'd be in a better position being ruled by Charles de Gaulle in 1965 than Harold Wilson.''

Not all Frenchmen were so sure.

"Can you imagine?'' said Jose-Alain Fralon, author of Help, the English Are Invading!

"What would the English tabloids do if they could no longer tell stories about the froggies, and what about those French who blame everything on the English?''

The British, he added, are "our most dear enemies'' and "we would lose all of the saltiness in our relationship'' had the two countries merged.

Still, he said, the two peoples complement each other marvellously.

"Roast beef and frogs don't go together in the same dish. But frogs' legs as a starter and a good roast beef as the main dish - c'est merveilleux,'' he said.

The documents, which have been unclassified for over 20 years, were found by a BBC producer late last month.

tomder55 answered on 01/18/07:

While reading the article I was thinking that Charles de Gaulle would never've let that happen .He was committed to an independent France .

Although he did not run the 5th Republic until 1958 he was still very influential in French politics ;including the development of an independent French nuclear deterent.

He left NATO ,and only under a vision of French dominance on the continent did he agree to participation in the common market .France would lead a greater Europe into a power equal to the US and the Soviet Union(une troisieme suerpuissance). Having England participate would not be acceptable since English entry would've diluted France'influence.

Gaullism meant restoring France to it's "proper place" in the hierarchy of world powers ( rentrer dans le rang. .The merging of the 2 nations would've been a serious blow to the concept.

Itsdb rated this answer Excellent or Above Average Answer
paraclete rated this answer Excellent or Above Average Answer

Question/Answer
ETWolverine asked on 01/17/07 - Define "Lost"

For months now, liberals have been demanding that the Bush Administration define what they mean by "winning the war in Iraq". On any number of occaisions, conservatives have define "winning" as developing a strong, capable, democratic government in Iraq that is able to handle its own internal and external security issues, and which safeguards the liberties of all its citizens. There have been some inroads made on these goals, as well as some setbacks. And by that definition we have not won the war yet, but we have most definitely not lost yet either.

Yet there are a number of Democrats and a few Republicans who continue to claim that we have "lost the war in Iraq". Nancy Pelosi has said it. Dodd has said it. Harry Reid has said it. Others say it as well. On this board, we have seen Chou say it over and over again. Clearly the conservatives among us are using a different definition of "win" and "lose" than the liberals among us.

So could those among us who say that we have lost the war in Iraq please explain what they mean by "lost"? We are clearly defining things differently, so a clarification of what "win" and "lose" mean in the context of the war in Iraq would be helpful in clearing up the misunderstanding. How do you define "lose"?

Elliot

tomder55 answered on 01/17/07:

What is 'lost' ? Well this war was lost at the onset because it was a war of choice by a Republican . Had it been a Democrat who waged a war of choice to liberate oppressed people then it wouldn't much matter that the troops are still there 10 years later; it would be a great success. (Clinton /the Balkans)

I love that notion going around that the American people voted against the war in 2006 and at that point it was lost . That may be more accurate then we think. Yes ,it can be lost without defeat in battle .All it takes is a lack of resolve when things get difficult .The same people today would be begging for a negotiated settlement in the middle of the Battle of the Bulge or when the allies got bogged down in Normandy fighting in the hedgerows.

ETWolverine rated this answer Excellent or Above Average Answer

Question/Answer
Itsdb asked on 01/17/07 - What will we do?

Once again we're in an environmental dilemma, this time over cremation. What should we do, encourage cremation as being "more eco-friendly, since land is not used for burial," or should we discourage it because of "toxic emissions, especially mercury fumes from incinerating dental fillings?" Perhaps all of those aging baby boomers should have their silver fillings replaced so they can't wreck the environment when they're dead and gone?

Steve

tomder55 answered on 01/17/07:

I keep telling people I want to be fish food .I've eaten my share of sea food and turn around is fair .

I don't have any moral objection to cremation ;although I would hate to contribute to green house gases .

I do have one with expensive caskets .I think this space saving option is a good idea ; bury the body vertically instead of in the horizontal position and without that damn expensive casket .Then use the land for grazing . I bet we make excellent fertilizer since alot of us are full of .........

A man gets a telegram about his mother-in-law's death .It asks whether she should be cremated or buried . After some thought he reponds "Best take no chances....Burn her and then bury the ashes " .

ETWolverine rated this answer Excellent or Above Average Answer
Itsdb rated this answer Excellent or Above Average Answer
labman rated this answer Excellent or Above Average Answer

Question/Answer
Itsdb asked on 01/16/07 - What exactly is...

...a "fact finding mission" anyway? And why are so many potential presidential candidates running around all over the globe? What business do they have injecting themselves in US foregin policy?

NM governor Bill Richardson has recently supposedly negotiated a ceasefire in the Sudan and met with the Norks over their nukes. Hillary is on a "fact-finding" mission to Iraq, Pakistan and Afghanistan, calling for more troops to head off a "big spring offensive" by the Taliban and al Qaeda. Where will Obama go? Do any of these solo missions help further US interests?

tomder55 answered on 01/17/07:

SO in the same breath Hillary says that the nation is tottering ;and that she is encouraged about the direction the county is going . Here is some facts for Evita to digest .The "big spring offensive " she is talking about happens every year . The Taliban reminants come out of their mountian lair ;occupy a couple of mud hut villages ,set off a couple of car bombs ,and engage NATO forces . They lose a couple of thousand jihadists and then retreat back to Pakistan to lick their wounds .They are not an existential threat to the nation because the Afghans have lived under their rule before and did not like it .

The notable thing about Madame Defarge's trip was that she travelled with Sen. Evan Bayh . That may be revealing about who she envisions as her Veep.

Do any of these solo missions help further US interests? Yes in some cases they do .Many times they secretly have White House support and marching orders . There really is no prohibition of public officials from taking these trips /junkets. The utility should be measured in results ;Not in Kerry-like talking points about how many foreign leaders he has spoken to that are anti-Bush. It is not unreasonable for us to expect responsible behavior and advancement of American interest while elected leaders are overseas .

But ;in the case a Richardson ,a cynic could be led to believe that his motivations are more domestic politics than any desire to solve the Sudan Civil War. The biggest stumbling block for Govenors at the national level is their international relations experience.A success in the Sudan crisis would have been a high-profile win for Richardson. But unfortunately for him ,it aint happening .My advice to him would be to emphasis his local accomplishments .

Obama takes junkets to Springfield ,Il. where he delights in the local press' comparison of him and Abe Lincoln.When you have the MSM all agog why do you need to leave the states ?

Itsdb rated this answer Excellent or Above Average Answer

Question/Answer
kindj asked on 01/16/07 - Cab wars

Subject: Muslim cab wars in Dhimmiapolis - er, I mean, Minneapolis January 11, 2007


Muslim cab wars in Dhimmiapolis - er, I mean, Minneapolis January 11, 2007
By Greg Strange

By now you've probably heard about the Muslim cab wars in Minneapolis. If not, here's a quick synopsis. Seventy-five percent of the cab drivers servicing the Minneapolis International Airport are Somali Muslims and most of them have been refusing service to infidel passengers who engage in behaviors that aren't up to snuff when it comes to Islam.

For instance, let's say you're an average godless and contemptible infidel who just bought a bottle of Cabernet Sauvignon at the duty-free shop and
then you try to hail a cab with that devil's concoction in your possession. You're committing a grave sin against the Religion of Peace and can't
possibly expect an observant Muslim to be complicit in your sin, not to mention defile his taxi by transporting you and your iniquitous liquid.

So what are you supposed to do? Well, apparently you're supposed to show your multicultural tolerance by acquiescing to the primitive customs of the
Islamic cabbies and finding some other means of transportation that won't involve giving offense to a person of another culture or faith.

As another example, let's say you're an unsighted person who gets around with the help of a seeing-eye dog. Yep, that's right. Don't expect to defile
the purity of a Muslim taxi with your filthy beast. Refusing service in that kind of situation is a direct violation of the Americans with Disabilities
Act (ADA), but so far Islam trumps such infidel legislation.

Amazing, isn't it? In a world of political correctness gone mad, who could possibly mistreat the unsighted and get away with it? Muslims, that's who!
But imagine that the cab drivers, rather than being Muslims, were instead bible-thumping Christian fundamentalists who were refusing fares for the
exact same reasons. How long do you think they would get away with it before the ACLU would be all over them like white on rice and before Frank Rich,
New York times columnist extraordinaire, would write his umpteenth column about the Christian fundamentalist takeover of America? Half a day, maybe?

But for whatever unfathomable reason, the entire Western world, in its wretched multicultural misguidedness, seems to believe that it has to bend
over backwards to accommodate the primitive beliefs of Islamic religious fanatics. The question of our time, of the age, is, why? Why would self-proclaimed secular Western societies do this? Why, why, why?

It all stems from the cult of multiculturalism which basically says that any culture is as good as any other, so therefore all cultures must be
tolerated. Anything less would be intolerant and intolerance is evil. And since Islamic culture is just another culture that is as good as any other,
it must be tolerated in the name of multicultural tolerance, even if it is itself supremely intolerant and could eventually supplant the preexisting
culture of tolerance, which would, in effect, spell the end of all that cherished toleration.

It could be the ultimate paradox. The West commits cultural suicide in the name of tolerance and in so doing, leaves the world in the hands of its most
intolerant people. Brilliant!

Volumes are currently being filled with examples from all over the Western world of its groveling obsequiousness before Islam, all in the name of
extreme multiculturalism. It's everything from removing pork from hospital menus for fear of offending Muslims to banning the English national flag
from English prisons because it displays the cross of St. George, which was used by the Crusaders and is therefore deemed offensive by imprisoned Muslim criminals.

If you stop and think about it, given that Islamic law's top ten list of things to avoid includes not only alcohol and dogs, but infidels themselves,
it would seem that for observant Muslims there can only be one final and ultimate solution if there is ever to be a world that is pure. It's enough
to send a shiver down an infidel's back.

In the meantime, there may be a tiny smidgen of hope in Minnesota, at least for the moment, where it is still theoretically possible that Minneapolis
won't become Dhimmiapolis. The Metropolitan Airport Commission, which oversees policy at Minneapolis
International Airport, is going to conduct hearings to decide on a proposal that states that all cab drivers at the airport will be expected to carry
all passengers with alcohol and seeing-eye dogs. If there is any sanity left in the Western world, that proposal will pass.

That would no doubt upset the Somali cab drivers, but here's something for them to think about. If Islam was such a great thing, they could just get jobs at Mogadishu International instead of having to slink off to some infidel country to find a decent life. But Mogadishu International is almost never open for business because Somalia is a failed, war-torn Islamic state, and like so many other basket-case Islamic states, the only thing to be found there is misery, violence and hard times.

So Somalis, as well as other denizens of other failed Islamic states, fan out all over the globe seeking economic opportunity in non-Islamic countries, and in particular, Western countries since they tend to be the most prosperous. The amazing thing is that so many of them never seem to be able to put two and two together to figure out that if going to a non-Islamic country is the only way to get a decent life, then maybe Islam's not all it's cracked up to be.

So instead of adapting to their new environments, they continue right on with their primitive and irrational customs while their Western hosts, who
would never tolerate such nonsense from their own Christian populations, mouth vapid platitudes about tolerance and the splendor of diversity.

Maybe Minneapolis will draw the line at the mistreatment of blind people byIslamic primitives. On the other hand, Minneapolis was the chosen site of
the flying imams' mini-airport jihad, not to mention having just sent the first Muslim to the U.S. Congress. I guess you could say all bets are off
and there could still be a Dhimmiapolis in the future.

~~~~~~~~~~~~~~~~~~~~~~~~~~~~~~~~~~~~~~~~~~~~~~~~~~~~

So how far off is this author?

tomder55 answered on 01/17/07:

They have to follow the law or the rules of their employer or the airport regulations.

I see this no different in this than in the case of a pharmacy clerks who refuse to dispense morning after pills . Is there,or should there be a law that says they are required to do so ? In both cases I don't think so . It should be up to the pharmacy ownership ;and in whatever authority oversees the airport .

It seems that in the respect of an orderly process at the taxi pick up locations the airport would certainly be in their right ;and it would be reasonable to have a stipulation ,that any taxi that wished to pick up fares at the airport should be required to pick up anyone requesting the service.

Multiculturalism began to run amok when the concept shifted from Melting Pot to Mosaic. The melting pot absorbed some of the cultural identity of immigrants but the emphasis was on becoming "Americanized " . In the mosaic model a patchwork of seperate identities are quilted together . The weakness becomes a lack of a common identity .Multiculturalism feeds fragmentation.

In the case of the jihadist it becomes more complex. What do you do if a people wants to live in your society but don't embrace the values that society stands for?If you don't want to live in a free society , why did you move there in the first place?I'm all for retaining one's unique religious and/or ethnic identity. But if you're going to live in a country dominated by a culture other than your own, you need to face up to reality and make your peace with it. If you can't ,then leave .

E Pluribus Unum

ETWolverine rated this answer Excellent or Above Average Answer
kindj rated this answer Excellent or Above Average Answer

Question/Answer
paraclete asked on 01/16/07 - The madness of King George

perhaps relecting the problems of his eighteenth century counterpart has king George finally lost it?

January 13, 2007

It’s One Thing to Flirt With Madness, but When Madness Starts Flirting Back?


— Eric Martin @ 6:28 am

“I call it the madman theory, Bob. I want the North Vietnamese to believe that I’ve reached the point that I might do anything to stop the war. We’ll just slip the word to them that, ‘for God’s sake, you know Nixon is obsessed about communism. We can’t restrain him when he’s angry — and he has his hand on the nuclear button’ — and Ho Chi Minh himself will be in Paris in two days begging for peace.”

Richard Nixon to H.R. Haldeman, 1969 (via TCR)

It’s long been my contention that the vast majority of the Iran/Syria related bellicosity emanating from the White House over the past three years has been hollow saber rattling of one form or another. Roughly a year into the invasion of Iraq it became apparent to most observers (even in the White House) that our military options vis-a-vis Iran (and Syria to a lesser extent) were severely limited.

For one, our sizable and enduring military commitments in Iraq and Afghanistan have limited our ability to take similar action against Iran/Syria. Not to mention the fact that Iran has a wide range of retaliatory options available - a menu made broader by the convenient proximity of so many American personnel right next door in Iraq (as well as a largely sympathetic Shiite faction in a hegemonic position in Iraq). Taking action against Syria, though perhaps less problematic than with respect to Iran, would nevertheless open a pandora’s box of potential regional destabilizations that we might not be able to contain (the usurpers of Assad, for example, would be far more hostile to our interests).

With those limitations in mind, the repeated threats (both veiled and overt), and consistent maintenance of a generally hostile posture, can mostly be interpreted in two ways (with various power nodes within the administration probably pursuing varying strategies in this regard):

First, the bluster can be seen as an attempt to augment the perception in Iranian/Syrian eyes that we still have a military capability to be reckoned with. This would be useful in order to compel Iran/Syria to offer better terms and concessions at the negotiating table. The problem with this reading is that those in favor of opening talks with Iran/Syria have thus far been unable to convince the President of the wisdom of this course. So to the extent that some Bush administration officials have been seeking to bolster our hand in negotiations by assuming a - somewhat hollow - threatening posture, the next step in this two-part strategy remains elusive. This renders phase one utterly counterproductive.

Second, there are elements (prominent ones, supported by forces like Cheney) that actually want military confrontation with Iran/Syria. So some of the heated rhetoric and associated provocations are indicative of a legitimate strategy to spark a war. Still, despite these belligerent intentions, Bush has thus far resisted commencing the final countdown, so to speak. At the end of the day, most high ranking military officials (including new Defense Secretary Gates) counseling Bush are most likely reminding him, repeatedly, that such an widening of the conflict could lead to an unprecedented catastrophe.

Due to the simultaneous pursuit of these diverging strategies, all we have been left with is an incoherent and muddled policy that juts out in various directions in fits and spurts: full of hostile rhetoric and provocation, yet without any discernible means or will to follow either course to fruition or productive resolution. All bad cop, no good cop. All preparation, no realization.

That being said, the flirtation with madness has taken on some truly ominous shades as of late. This is either the result of masterful subterfuge undertaken as a prelude to negotiations (now, from a position of perceived strength), the actual build up to war or, in the alternative, still more outward manifestations of the near-paralytic internal divisions in the White House’s policy making apparatus.

Steve Clemons yesterday offered a chilling bit of information:

Washington intelligence, military and foreign policy circles are abuzz today with speculation that the President, yesterday or in recent days, sent a secret Executive Order to the Secretary of Defense and to the Director of the CIA to launch military operations against Syria and Iran.

The President may have started a new secret, informal war against Syria and Iran without the consent of Congress or any broad discussion with the country.

That rather bold prediction comes in the context of several other actions, warnings and rhetorical escalations that seem to lend support to the most dire reading of events. There was the seizure of Iranian officials in Southern Iraq weeks ago, the storming of the Iranian consulate (lesser diplomatic outpost?) and capture of five Iranian citizens in Kurdistan, an inflammatory speech by President Bush on Wednesday, the appointment of a Navy man (well versed in the use of air power) to head CENTCOM, and some other military moves that would normally complement the preparation for war. Steve Clemons, again, this time quoting Flynt Leverett:

The deployment of a second carrier strike group to the theater — confirmed in the speech — is clearly directed against Iran. Since, in contrast to previous U.S. air campaigns in the Gulf, military planners developing contingencies for striking target sets in Iran must assume that the United States would not be able to use land-based air assets in theater (because of political opposition in the region), they are surely positing a force posture of at least two, and possible three carrier strike groups to provide the necessary numbers and variety of tactical aircraft.

Similarly, the President’s announcement that additional Patriot batteries would go to the Gulf is clearly directed against Iran. We have previously deployed Patriot batteries to the region to deal with the Iraqi SCUD threat. Today, the only missile threat in the region for the Patriot to address is posed, at least theoretically, by Iran’s Shihab-3.

On top of that, Garanace Franke-Ruta passes along some unsettling rumors that argue that the extra troops in Iraq resulting from the surge will be tasked with the mission of protecting the vulnerable military supply lines that stretch through southern Iraq that would be targeted pursuant an eventual strike on Iran. More preparations for war?

I readily admit that at least part of my assessment of the situation, and conclusion that confrontation with Iran/Syria is not in the cards, is born out of hope and necessity: the results from such a widening of the conflict could be near-cataclysmic. As such, I have put faith in the notion that even President Bush must appreciate this fact and thus avoid launching such attacks - even if, at times, his state of indecisiveness and desire to confront Syria and Iran lead to a schizophrenic translation into policy.

Still, I am growing increasingly worried that Bush might just be foolish and desperate enough to do the unthinkable. There are certainly enough committed zealots in his inner circle that would counsel him to act so rashly. Or at the very least, blunder his way into a regional war.

Doing so under any circumstances, however, would truly signify the all out madness of King George.
+++++++++++++++++++++++++++++++++++++++++++++++++++++

so sports fans, what do you think, will King George do the unthinkable?

tomder55 answered on 01/16/07:

The assumption is that Bush is initiating hostilities instead of responding to existing conditions . You have read my recent posts on Iran ,so you know that I have detailed specific instances of interdiction of weapons and jihadist crossing both Iran and Syrian borders with the purpose of engaging the coalition forces in combat. Since both nations have done nothing to prevent it ,we can reasonably conclude that they are compliant .We are trying to stabilize Iraq and they are doing everything they can to distrupt our efforts ;including crossing the line into open acts of warfare that kill our forces . That to me is enough provocation .

To add to that ,the Mahdi-hatter has been upfront in his rhetoric about his plans .More than once he has threatened to close the Straits of Hormuz and to attack the Gulf States . He has also conducted a series of war games where he more than demonstrated his capability to disrupt shipping in the Gulf.

That type of blackmail we cannot allow to go unanswered . The cost to the global economy would be too high . Yes the Patriot Batteries and the fleet are there to protect our allies in the region .If the Mahdi-hatter wasn't saying it perhaps it would not be necessary .

Maybe it is only saber-rattling or as Def.Sec Robert Gates says “The Iranians clearly believe that we are tied down in Iraq, that they have the initiative, that they’re in a position to press us in many ways,” ....“We are simply trying to communicate to the region that we are going to be there for a long time.”

Perhaps it is only a prelude to negotiations as Martin speculates . The Mahdi-hatter ought to pray to the 12th Imam that is all it is . I'm tired of the concern over what they will do . Let them worry for a change over what we will do.

Martin may not have the most recent information btw. He thinks only the USS Ike and the U.S.S. John Stennis carrier battle groups will be deployed . He is mistaken .I hear from good sources that the USS Ronnie Reagan will be dispatched out of San Diego soon to join the armada....as well as an amphibious assault group led by the U.S.S. Boxer, which can land several thousand U.S. Marines .

Yes ,time to put the fear of Allah into their hearts . As Victor Davis Hanson wrote :I would not wish to fight the United States — either militarily, politically, or culturally. For every threat, our history teaches us that Americans offer not just a rejoinder, but the specter of a devastating answer of a magnitude almost inconceivable to those now chanting and threatening in the streets of the Middle East. Do they have any idea of what sort of dangerous people we really are? Do they understand the history of the names of those ships now off their coasts, like the USS Peleliu or Enterprise, or the pedigree of the 82nd or 101st Airborne?

I am tired of reading that Iran has a wide range of retaliatory options available . Let them worry about what we bring to the table. Let the negotiations begin !!!!




Itsdb rated this answer Excellent or Above Average Answer
paraclete rated this answer Excellent or Above Average Answer

Question/Answer
captainoutrageous asked on 01/14/07 - Satire

Just days after the Rev. Pat Robertson claimed on his 螌 Club" program that God warned him of "mass killings" in the United States late in 2007, God held a rare news conference today to deny having spoken to the controversial televangelist.

For the usually publicity-shy King of the Universe, the press conference held at the Chicago Airport Marriott signaled a sharp break with tradition.

But appearing before the press in His trademark flowing robes and white beard, and carrying what appeared to be a lightning bolt, God said that He decided to convene the extraordinary press briefing because "I had to set the record straight about this."

"I want to make it clear that at no time at the end of the year did I have any conversation with the Rev. Pat Robertson," the Supreme Being said. "Personally, I think the guy is delusional."

God then distributed His personal phone logs for the month of December to prove that He had in fact no contact with the Rev. Robertson.

"I don't make a habit of talking to TV personalities," God emphasized. "Although on New Year's Eve I did have a brief chat with Ryan Seacrest to wish him good luck."

Answering a reporter's question, God acknowledged that with war raging around the globe, 2006 had been a "difficult year" for the forces of goodness, but He remained upbeat, pointing to some of His accomplishments in the year just past.

"At least I got Judith Regan fired," He said.

Elsewhere, Britney Spears checked into a rehab center after being driven there by her 1-year-old son, Sean Preston."

Andy Borowitz, Seattle Times

tomder55 answered on 01/16/07:

and OJ is still on his self proclaimed mission ,searching every golf course in the country, looking for his wife's killer.

captainoutrageous rated this answer Excellent or Above Average Answer

Question/Answer
kindj asked on 01/15/07 - A little late, but a good read:

Made in the USA: Spoiled brats

--------------------------------------------------------------------------------
Posted: November 20, 2006
1:00 a.m. Eastern




The other day I was reading Newsweek magazine and came across some poll data I found rather hard to believe. It must be true given the source, right? The same magazine that employs Michael (Qurans in the toilets at Gitmo) Isikoff. Here I promised myself this week I would be nice and I start off in this way. Oh what a mean man I am.

The Newsweek poll alleges that 67 percent of Americans are unhappy with the direction the country is headed and 69 percent of the country is unhappy with the performance of the president. In essence 2/3s of the citizenry just ain't happy and want a change.

So being the knuckle dragger I am, I starting thinking, ''What we are so unhappy about?''

Is it that we have electricity and running water 24 hours a day, 7 days a week? Is our unhappiness the result of having air conditioning in the summer and heating in the winter? Could it be that 95.4 percent of these unhappy folks have a job? Maybe it is the ability to walk into a grocery store at any time and see more food in moments than Darfur has seen in the last year?

Maybe it is the ability to drive from the Pacific Ocean to the Atlantic Ocean without having to present identification papers as we move through each state? Or possibly the hundreds of clean and safe motels we would find along the way that can provide temporary shelter? I guess having thousands of restaurants with varying cuisine from around the world is just not good enough. Or could it be that when we wreck our car, emergency workers show up and provide services to help all involved. Whether you are rich or poor they treat your wounds and even, if necessary, send a helicopter to take you to the hospital.

Perhaps you are one of the 70 percent of Americans who own a home, you may be upset with knowing that in the unfortunate case of having a fire, a group of trained firefighters will appear in moments and use top notch equipment to extinguish the flames thus saving you, your family and your belongings. Or if, while at home watching one of your many flat screen TVs, a burglar or prowler intrudes; an officer equipped with a gun and a bullet-proof vest will come to defend you and your family against attack or loss. This all in the backdrop of a neighborhood free of bombs or militias raping and pillaging the residents. Neighborhoods where 90 percent of teenagers own cell phones and computers.

How about the complete religious, social and political freedoms we enjoy that are the envy of everyone in the world? Maybe that is what has 67 percent of you folks unhappy.

Fact is, we are the largest group of ungrateful, spoiled brats the world has ever seen. No wonder the world loves the U.S. yet has a great disdain for its citizens. They see us for what we are. The most blessed people in the world who do nothing but complain about what we don't have and what we hate about the country instead of thanking the good Lord we live here.

I know, I know. What about the president who took us into war and has no plan to get us out? The president who has a measly 31 percent approval rating? Is this the same president who guided the nation in the dark days after 9/11? The president that cut taxes to bring an economy out of recession? Could this be the same guy who has been called every name in the book for succeeding in keeping all the spoiled brats safe from terrorist attacks? The commander in chief of an all-volunteer army that is out there defending you and me?

Make no mistake about it. The troops in Iraq and Afghanistan have volunteered to serve, and in many cases have died for your freedom. There is currently no draft in this country. They didn't have to go. They are able to refuse to go and end up with either a ''general'' discharge, an ''other than honorable'' discharge or, worst case scenario, a ''dishonorable'' discharge after a few days in the brig.

So why then the flat out discontentment in the minds of 69 percent of Americans? Say what you want but I blame it on the media. If it bleeds it leads and they specialize in bad news. Everybody will watch a car crash with blood and guts. How many will watch kids selling lemonade at the corner? The media knows this and media outlets are for-profit corporations. They offer what sells. Just ask why they are going to allow a murderer like O.J. Simpson to write a book and do a TV special about how he didn't kill his wife but if he did … insane!

Stop buying the negative venom you are fed everyday by the media. Shut off the TV, burn Newsweek, and use the New York Times for the bottom of your bird cage. Then start being grateful for all we have as a country. There is exponentially more good than bad.

I close with one of my favorite quotes from B.C. Forbes in 1953:



''What have Americans to be thankful for? More than any other people on the earth, we enjoy complete religious freedom, political freedom, social freedom. Our liberties are sacredly safeguarded by the Constitution of the United States, 'the most wonderful work ever struck off at a given time by the brain and purpose of man.' Yes, we Americans of today have been bequeathed a noble heritage. Let us pray that we may hand it down unsullied to our children and theirs.''
I suggest this Thanksgiving we sit back and count our blessings for all we have. If we don't, what we have will be taken away. Then we will have to explain to future generations why we squandered such blessing and abundance. If we are not careful this generation will be known as the ''greediest and most ungrateful generation.'' A far cry from the proud Americans of the ''greatest generation'' who left us an untarnished legacy.

Craig R. Smith is an author, commentator and popular media guest because he instantly engages audiences with his common-sense analyses of local, national and global trends. Serving as CEO of Swiss America for nearly 25 years, Craig understands that Americans want solid answers to the tough questions and that real leadership begins with servanthood. Craig's most recent book is "Black Gold Stranglehold: The Myth of Scarcity and the Politics of Oil," which he co-authored withWND columnist Jerome R. Corsi. For media interviews please call Holly at 800-950-2428.

tomder55 answered on 01/15/07:

I can't agree more









kindj rated this answer Excellent or Above Average Answer
paraclete rated this answer Excellent or Above Average Answer

Question/Answer
Itsdb asked on 01/15/07 - The race is over

All those wannabe presidential candidates might as well stop and save their time and money, Ebony magazine has already declared a winner for 2008:



Will America elect a smoker? At least he admitted he DID inhale.

tomder55 answered on 01/15/07:

I think it is a fait accompli . I know this because I turned on 24 last night and saw that there was an Obama lookalike (sorta) playing the part of President.



When Hillary was hot then Geena Davis played her as President in a TV drama. But Hillary evidently is old news because the show did not last .

There was a rumor earlier today that he would announce today on MLK day . But that has since been refuted by his staff.

Today's headline : Obama...I Was Against War Funding Before I Was For It




When not smoking he's thinking about his next light up.

Itsdb rated this answer Excellent or Above Average Answer

Question/Answer
Choux... asked on 01/13/07 - BUSH IGNORES ONLY SUCCESS..REPEATS FAILURE PARADIGM

"George Bush's Presidency is not only a failure, but the seed of multi-generational disasters. Whether it is the disappearance of the middle class, the lost 8 years in stopping and reversing global warming, the disinvestment in the education and physical fitness of the next generation, the spiraling national debt, or wars and the reduction of trust in the United States, George Bush's legacy is already written, "years the locusts have eaten".

The policies that produced these disasters were animated by radical rightwing philosophy, sold to the American people through lies, dissembling and intimidation, and imposed through rigid discipline and attacking opponents. It is not an overstatement to suggest that, if these policies were really sound and popular, such tactics would be not only unwelcome but unnecessary.

Iraq is the largest of these catastrophes. Out of power, the radical rightwing bemoaned the concensus approach to foreign policy of using the military as a last resort, and when our national interest was at stake. Seizing power, they filled Bush's blank bonnet with visions of grandeur, (ab)using the US military as the primary instrument of policy to transform the world. Bush, a fellow war-avoider, swallowed their vision and their dismissive attitude toward diplomacy because the other side was unworthy, and because the real goal was overthrow anyhow.

This is all the more regrettable because Bush actually has a single success in his Presidency, Libya's abandoning its nuclear program in exchange for removal of sanctions and normalization of relations. The radical rightwing has not been vocal about this success, however, for a simple reason: it was achieved through diplomacy and compromise, not regime change.

Libya's Muammar Qaddafi is not exactly a lamb. If you were to read his rhetoric, e.g., "the streets of America will run red with blood" and "we consider ourselves at a state of war", you would be forgiven if you thought you were really listening to Iran's Ahmadinejad. Intelligence analysts identified a plant in a residential neighborhood that was a chemical munitions factory, believed to produce nerve gases.

And yet, the Administration talked to him, and compromised. They stopped insisting that Qaddafi turn over high level government officials involved in the PanAM 103 bombing as a condition to remove sanctions and in exchange for Qaddafi abandoning his nuclear program. Did we achieve the full measure of justice? No, we did not. But, the world was made a bit safer. It was a good compromise.

In Iran: Missed lessons from the Cuban Missile Crisis (1962), it was suggested that following the lesson of how that crisis was resolved, responding to a conciliatory suggestion while ignoring the more belligerent, would be a useful paradigm for exploring what might be achieved through talking, through engagement. It worked with Cuba in 1962, and George Bush made it work with Libya.

Bush could have touted his one success, and tried to repeat it. Instead, he preferred the neocon prescription of unending warfare for failure." Paul Abrams, blogging

~~~~~~~~~~~~~~~~~~~~~~~~~~~~~~~~~~~~~~~~~~~~~~~~~

Tomorrow: Go Bears!!!

tomder55 answered on 01/14/07:

The radical rightwing has not been vocal about this success, however, for a simple reason: it was achieved through diplomacy and compromise, not regime change.

We have frequently cited the turning of Qaddafi ,and the breaking up of the AQ Kahn network as successes in the war against jihadistan. In the case of Qaddafi he only considered seriously entering into negotiations after the invasion of Iraq. And in the case of the Kahn network we pressured Pakistan to cooperate...again afet we had flexed our muscle.

They stopped insisting that Qaddafi turn over high level government officials involved in the PanAM 103 bombing as a condition to remove sanctions and in exchange for Qaddafi abandoning his nuclear program.

Instead Libya was forced to pay $10 million to each family of the 270 people killed in the bombing of Pan Am Flight 103 and to turn two men held responsible for the attacks over to Scottish officials .One of the men, Al Amin Khalifa Fhimah, was acquitted. A court in The Hague found the other man, Abdelbaset Ali Mohmed Al Megrahi, guilty and sentenced him to life in prison. So far Libya has paid $8 million to each and the falimies are still asking for the other $2 million.

The popular talking points about Libya is that it has been transformed from a "rogue state", under international sanctions, to a partner of the West, and especially of the United States. Tripoli is now bustling with foreign businessmen; air links have been restored; Libyan assets have been unfrozen; foreign investment is pouring into the oil industry. The country as a whole is enjoying a building boom. All of this is true to a degree but Qaddafi is far from the boy scout and loyal ally that this view suggests .

Five Bulgarian nurses and a Palestinian doctor.arrived in Libya in February 1998 and went to work at the Al Fateh Children's Hospital in Benghazi, Libya's second city. Less than six months later, children at the hospital began testing positive for HIV.The nurses and the doctor were accused of deliberately inocculating 426 children with the virus. According to a Libyan intelligence report submitted to the court, the nurses "confessed "to the crime. One of the nurses, Kristiyana Vulcheva, allegedly "confessed "that she had been given vials of HIV-tainted blood by a British friend who was working for a subsidiary of Halliburton. Gaddafi charged that the nurses and the doctor had acted on orders from the CIA and Israel's Mossad.Lawyers for the nurses said that the confessions had been extracted under torture. They said the nurses had been given electric shocks. One nurse, Snezhana Dimitrova, said she had been suspended from a door jam with her hands manacled behind her back until her shoulders dislocated.

All 6 have been condemned to death. The families of the Libyan children have asked for Bulgaria to pay $10 million compensation for each sick or dead child (the same amt.that Libya had to pay for the Pan Am 103 murders). Gaddafi is attempting to extort from the international community a sum equivalent to the one he was forced to pay to the families of the Lockerbie victims.

But it gets even better. Libya offered yesterday to free the nurses and doctor in exchange for Abdelbaset Ali Mohmed Al Megrahiin ;the convicted bomber.

Missed lessons from the Cuban Missile Crisis (1962), it was suggested that following the lesson of how that crisis was resolved, responding to a conciliatory suggestion while ignoring the more belligerent, would be a useful paradigm for exploring what might be achieved through talking, through engagement. It worked with Cuba in 1962, and George Bush made it work with Libya.

Paul Abrams has a romantacised image of how the Cuban missile crisis went down. We were close to a nuclear exchange with a power of equal nuclear capablility .So from the outset the comparision is flawed. The Soviets pulled a fast one by putting nukes in Cuba and negotiations were held under the radar out side of public knowlege. We now know that the crisis ended when we agreed to remove nukes from Turkey in exchange for the removal of the Soviet nukes in Cuba. This little fact was withheld from the American people and our NATO allies with an outright lie by the Kennedy administration who worried about the exposure of the concession to blackmail . We also pledged to not invade Cuba and thus enslaved to population to the jack-boot of the Castro regime to this day.Some compromise to achieve status quo pre-crisis. If that's his view of successful negotiations he can have it.








Choux... rated this answer Above Average Answer

Question/Answer
arcura asked on 01/13/07 - Here is news with views to agree or disagree with.

Read it all. The guy writes well and makes some or bad points well depending on your opinion.

http://www.newswithviews.com/Taft/john35.htm

tomder55 answered on 01/14/07:

That news with views is some site. If I took 1/2 the things I just read on it .I would bunker down in my house ;arm myself to the teeth , stock my pantry with survival rations and join a revolution.

I would have to say however that most of the site was over the top alarmism. It is the libertarian version of tin foil hat paranoia.

Yes I think we should be ever vigilant to guard our feedoms. But ,I do not live under the fantasy that they are under constant assault by the folks we elect to protect them.

arcura rated this answer Excellent or Above Average Answer

Question/Answer
Itsdb asked on 01/12/07 - Hungry Mexicans, U.S. to blame

Everything's our fault isn't it?

Mexico leader in tortilla pledge

Mexico's President Felipe Calderon has pledged to intervene to tackle the soaring price of tortillas, the flat corn bread which is a local staple.

The price of tortillas, the main source of calories for many of Mexico's poor, rose by more than 10% last year.

Mr Calderon said the government would clamp down on speculators and search for cheaper providers of corn.

But he ruled out imposing subsidies or price controls, which were lifted in the late 1990s.

"We will take all the measures within reach of the federal government to avoid escalating prices," Mr Calderon said.

In the meantime Mr Calderon has told his agriculture secretary to import corn to ease the problem.

"I don't care if it's brought from thousands of kilometres away, the most important thing is that this [shortage] is not used as an excuse to raise prices," he said on Thursday.

Earlier this week, angry housewives heckled Mr Calderon at his public appearances, pleading with him to bring tortilla prices down.

"When there isn't enough money to buy meat, you do without," one woman in Mexico City, Bonifacia Ysidro, told the Associated press. "Tortillas you can't do without."

Ms Ysidro said she paid 25 pesos - about a sixth of her family's daily income - for enough tortillas to feed her family of six.

On Thursday, government officials from the Federal Competition Commission said that they were investigating claims that tortilla companies around the country were manipulating prices and restricting supplies to boost profits.

"If we detect monopolistic practices, we could impose fines of up to 70m pesos [$6.4m]," commission director Eduardo Perez Motta said in a statement.

Under the 1994 North American Free Trade Agreement, Mexico used to get cheap corn imports from the US, but Mexico's Economy Minister Eduardo Sojo said that with more US corn being diverted into ethanol production, supply was dwindling.

~~~~~~~~~~~~~~~~~~~~~~~~~~~~~~~~~~~~~~~~~~~~~~~~~~~~~~~

As a huge fan of tortillas I can certainly sympathize with how Mexicans are now paying a premium for tortillas, so what shall we do to help out?

We're kind of between a rock and a hard place here aren't we? If we curtail ethanol production so we can send more cheap corn to Mexico we'll be back to depending more on fossil fuels and a greater contribution to global warming. If we clamp down on illegal immigration there won't be enough money going back home to Mexico to buy tortillas. What can we do???

Btw, I prefer flour tortillas in most cases, heated on an open burner and slathered with butter. Sometimes sprinkled with cinnamon and sugar or spread with a little honey.

tomder55 answered on 01/13/07:

To further complicate the issue ,the American taxpayer shells out huge bucks to subsidize the corn market.($41.9 billion on corn subsidies from 1995 to 2004 ..much of it going to ethanol conversion ) That makes absolutely no sense. Obviously there are enough markets for American corn that it should not need price propping .

Across the world, government subsidies wreak havoc with farm economies. The effect on Mexico is obvious. The Mexican farmer due to NAFTA doesn't only have to compete against a much more efficient agri-business in the US (Cargill and Archer Daniels Midland are large exporters of US subsidized corn to Mexico),but also with the American taxpayer . This leads to declining prices of Mexican corn which lowers family incomes. They literally can't afford to feed their families. Then we wonder why they cross the US border in hordes.

If I was a Mexican I would be demanding a renegotiation of NAFTA. Just like we don't like dumping into our markets ,neither should they .

Itsdb rated this answer Excellent or Above Average Answer

Question/Answer
HANK1 asked on 01/13/07 - WISDOM:


Benjamin Franklin chaffed the hesitant signatories of the US Declaration of Independence with his famous observation that "we must all hang together, or assuredly we shall all hang separately."

This certainly applies to our presence in Iraq. Who says we can't learn from history?

HANK

tomder55 answered on 01/13/07:

Excellent observation .The old saying was that POLITCS STOPS AT THE WATER'S EDGE. I don't know if that was ever really true but it is a fine sentiment .

I was very angry at the tone of the Senate hearings and the unusually brutal grilling that Condi Rice was subject to from frankly both sides of the aisle. Sen. Boxer in particluar went way over the line when she suggested that because Condi Rice was single without children that she had no real stake in the outcome;implying that she did not care that members of other families are bearing the burden .

Boxer said

Who pays the price? I'm not going to pay a personal price. My kids are too old and my grandchild is too young. You're not going to pay a particular price, as I understand it, with an immediate family. So who pays the price? The American military and their families.

If you can't attack on substance, attack the person -- it's been a staple of liberal tactics for a long time . But Barbara Boxer's outrageous attack on Condi Rice hit a new low - even for Boxer. What does Condoleezza Rice's personal life have to do with the war in Iraq? Nothing.

The unmistakable message Boxer sent all Americans is that if you are single or childless, you have no stake in public policy -- that you're something less than fully American. Boxer owes an apology ,not just to Secretary Rice , but to the millions of other singles and adults in this nation who don't have children.

Unless you have a child of military age you have no stake in it. Interesting proposition isn't it ? Does Boxer know what it's like to be a black women growing up in the segregated South ? How can Boxer claim to speak of discrimination ;poverty and other pet issues she claims to champion.if she has not experienced it ? Did she experience racism ? What is her dog in that fight ? No she grew up privileged ;therefore she has no right to an opinion . You see how absurd you can get when you use her logic ?

But then again radicals use this tactic all the time. I am not allowed to have an opinion on "reproductive rights "because I am not a women. I should never argue that we should wage war because I did not serve in the military. I suppose I should have no opinion on law and order because I was never either a criminal ,a victim ,a cop ,or a judge.

I don't recall the fact that Att General Janet Reno was childless being raised when the issues of the Waco attack or Elian Gonzalez was raised .

Why isn't NOW condemning this blatant attack on feminism ? Rice is a seasoned diplomat, a renowned scholar and an advisor to two presidents.She is the first female black Secretary of State . She has served as National Security Adviser. Not bad for a single black women isn't it ? She should be praised for breaking through multiple glass ceilings .

Boxer should read read Condi Rice's bio. and see if she comes even close to walking in Rice's shoes .Does Senator Boxer really believe that Secretary Rice is not qualified to make high level decisions because she is a single, childless woman? Let me clue in Sen Boxer . I can be concerned with crime victims, AIDS sufferers, and families in Darfur; not because my uncle's a crime victim in Darfur with AIDS, but because they're people and have a soul and that means we should care about them.

Hank I am convinced that most of the garbage is due to the fact that the Senators are in front of cameras .Maybe more sober debate would be waged if all we were privy to was transcripts. But I am for open gvt. and I would prefer that we elected sober representatives instead.






HANK1 rated this answer Excellent or Above Average Answer

Question/Answer
Itsdb asked on 01/12/07 - Chinese facing shortage of wives

China will have 30 million more men of marriageable age than women by 2020, making it difficult for them to find wives, according to a national report.

The gender imbalance could lead to social instability, the report by the State Population and Family Planning Commission warned.

It found that around 118 boys were born to every 100 girls in 2005.

A traditional preference for boys, in a country with a one-child policy, is the root of the problem, the report says.

Abortions on female foetuses are believed to be widespread as couples, particularly in rural areas, hope for a son who will look after them in their old age.

There is also suspected under-reporting of female births.

'Embrace girls'

The report said the 118 to 100 ratio of newborn boys to girls had jumped from 110 to 100 recorded in 2000.

In some areas of southern China, such as Guangdong and Hainan, the figure was 130 boys to 100 girls in 2005.


map

China fears bachelor future

Nationwide this means there will be 30 million more men than women by 2020, making it difficult for those particularly with low income or little education to find a wife, the report said.

"The increasing difficulties men face finding wives may lead to social instability," the report said.

The report went on: "We need to develop a 'movement to embrace girls'... and effectively contain the trend towards greater gender imbalances..."
~~~~~~~~~~~~~~~~~~~~~~~~~~~~~~~~~~~~~~~~~~~~~~~~~~~~~~~

Tell me, who didn't see that coming? If Americans were to embrace gender-selection techniques, or gay marriage on a large scale would we eventually face similar problems?

tomder55 answered on 01/12/07:

Just what the world needs ,a 3.25 million man army looking for wives ! There's a recipe for an aggressive military if I've ever seen one . If they weren't wacking the Falun Gong women so they could harvest body parts maybe there would be more women available.

The book Bare Branches: The Security Implications of Asia's Surplus Male Population discusses the implications of this imbalance .According to the authors, history, biology, and sociology all suggest that these "surplus males" will generate high levels of crime and social disorder .They predict that China and India may build up their armed forces further just to create a safety valve.

Without the military ,many of them could seek to emigrate seeking mates or get into the mail order bride industry which impacts North Korea due to human traffickers smuggling N.Korean brides into China which I'm sure makes NORK men thrilled .The shortage of women in China may end up posing an existential threat to the Kim Jong mentally Il regime worse than anything US policy makers are planning .

But to further complicate the matter ,the culture in Taiwan is simular and to make up for the short-fall ,they import brides from the mainland . These women however are outnumbered by those smuggled into the country illegally, who, in exchange for $10,000, can legalize their status with marriages of convenience, then head for the brothels to earn real money. These bogus nuptials are difficult to detect since many Taiwanese men hop between marriages until they find a woman who can bear them a son.

The US ? we are equal opportunity abortionists. We don't make a distinction which gender of babies we snuff. The article you reference mentions China's aging population . WE have a simular one that is not as noticeable because we supplement our population with immigrants .

Itsdb rated this answer Excellent or Above Average Answer

Question/Answer
Itsdb asked on 01/11/07 - Darn that global warming again

Governor Directs State Government to Prepare for Extremely Cold Temperatures

By: Office of the Governor
Published: Jan 11, 2007 at 08:38

In anticipation of unusually cold temperatures forecast throughout much of California in the coming days, Gov. Schwarzenegger directed state agencies to activate the extreme temperature protocols established last year to assist the most vulnerable populations in the state and asked local governments and the people of California to take common sense precautions in the unseasonably cold weather.

"Northern and Central California will see night time temperatures drop into the teens and low 20s," said Gov. Schwarzenegger. "Because of the extreme cold, I have directed state government to spring into action to protect our most vulnerable communities. The state has made 11 National Guard armories available and will make additional facilities available, such as fairgrounds, should local governments deem it necessary."

Among the steps the state is taking are:

* Protecting Seniors and the Disabled: The Department of Health Services is making contact with its licensed facilities and local health departments to ensure that they are aware of the cold weather event and to inform them of protocols to protect the health and safety of the vulnerable populations in their care given the extreme weather. Additionally, the Department of Social Services is making contact with County Welfare Directors to ensure that they are aware of the cold weather event and ensure that In Home Supportive Service (IHSS) workers are aware of the event and the protocols to protect the health and safety of the vulnerable populations in their care given the extreme weather.

* Warming Centers: The California National Guard and the California Department of Food and Agriculture are working with local government officials to make armories and fair grounds available to supplement their sheltering needs. Currently, there are 11 California National Guard armories that have been opened as shelters from the cold in Gilroy, Sunnyvale, Santa Cruz, Merced, Sylmar, Ventura, Los Angeles (Federal Ave), Culver City, Santa Ana, Fullerton and Glendale. They are opened daily from 6 p.m. to 7 a.m. Below is a list with addresses to the facilities. An additional 16 locations in parts of the state expected to be hardest hit by the cold weather have been identified and put on standby should local governments request the assistance.

National Weather Service has reported to OES that temperatures will drop into the 20s and 30s Wednesday night into Thursday, followed by daytime temperatures in the mid 40s in most areas. On Friday and into the weekend, temperatures will drop further, reaching the high teens to low 20s in most areas at night, and continued daytime lows in the mid 40s.

~~~~~~~~~~~~~~~~~~~~~~~~~~~~~~~~~~~~~~~~~~~~~~~~~~~~~~~

You'd think all those grenhouse gases in California would keep those record lows away, wouldn't you? By the way, we're expecting a high of 20 here in the high plains of Texas on Monday, so all you people enjoying the record warmth think of us poor, cold Texans, Californians, and snow-packed Coloradoans this weekend.

tomder55 answered on 01/12/07:

Ahnold is RINO on steroids .Alot of Republicans in the State feel like suckers .He is starting his new term with the audacity and arrogance of a Hugo Chavez.He has even called California a modern nation state like Athens or Sparta .

"We are the modern equivalent of the ancient city-states of Athens and Sparta. California has the ideas of Athens and the power of Sparta," Schwarzenegger, who played Hercules in his first film role, told legislators at the capitol."Not only can we lead California into the future ... we can show the nation and the world how to get there."
"We can do this because we have the economic strength, the population, the technological force of a nation-state,"



Free of the burden of having to seek reelection and knowing he can never run for the Presidency ,he has finally been able to show his true colors ...and they are scary !

He has vowed to offer health insurance for every Californian; $43 billion in new public works projects on top of a record $42.7 billion approved by voters two months ago
He wants to reform the national Republican party into his image .He wants to overhaul political districts and ...single handedly solve global warming .

Last year, he backed pioneering legislation to cut the state's greenhouse gas emissions 25% by 2020 even as the administration of Republican President George Bush has taken a skeptical view about global warming.

On Tuesday, Schwarzenegger said he would sign an executive order mandating a reduction of the carbon content of transportation fuels by 10% by 2020.

"I propose that California be the first in the world to develop a low carbon fuel standard that leads us away from fossil fuels," he said."Let us blaze the way, for the US, for China and for the rest of the world."


In his view ,the national gvt. doesn't lead on issues of health care or the environment. That he reasons leaves a vaccume that he is more than willing to fill . He has the ego to do it ,and now he rules the most populous state . That gives him to rule the nation from the left coast and set an example for the rest of the country .

He is dangerous. Look for a movement to amend the constitution to make him eligible for the Presidency .


Itsdb rated this answer Excellent or Above Average Answer

Question/Answer
Itsdb asked on 01/11/07 - Polls, schmolls...

Speaking of polls, the left has begun an all out assault on Bush's plan for Iraq and fully intend on pushing the polls in their favor. On that front the Democrats have furnished a "PartyBuilder Letters to Editors tool," complete with a form letter for you to personalize, additional talking points and a distribution service to the papers of your choice.

The letter:

    *Subject: Won't work

    *Message: Last night, after much delay, President Bush announced his new strategy for the war in Iraq. Unfortunately, this so-called strategy is actually more of the same thing - only it commits more U.S. troops to a region being devoured by a civil war. It is unconscionable that more American soldiers will lose their lives in a last ditch effort that Bush knows won't work. There is no good reason to escalate the war, unless you're a President whose poll numbers are sagging. Bush needs to explain to the American people how his latest plan differs from his previous stay-the-course policies - all of which have failed. I wholeheartedly support the efforts of Democratic leaders to move towards bringing troops home, and giving responsibility to the Iraqi people for their future."


Talking points:

    # President Bush’s escalation plan is nothing but “stay the course” with more troops.

    # The President’s escalation plan has failed over and over. It is opposed by top military leaders, the Iraq Study Group, foreign policy experts, and the American people.

    # President Bush has ignored the bipartisan Iraq Study Group, which recommended we begin the phased redeployment of American troops and end our open-ended commitment to Iraq.

    # Congress should continue to exercise its Constitutional authority to hold the president accountable for a change of course that allows for our troops to come home.


So much for the new bipartisanship. I'm sorry - no I'm not - this is just plain BS, and I'm sick and #$@*&! tired of the left ramming this crap down our throats. They have no plan, other than to surrender and place every one of our lives at risk. This is what you democrat voters voted for, to throw the United States of America under the bus - and for all of you who've bemoaned the loss of Iraqi civilians - as the president said, the result for the Iraqi would be "mass killings on an unimaginable scale."

So go ahead, turn your backs on the Iraqis, turn your backs on America, turn your backs on your own safety and send those letters.

Steve

tomder55 answered on 01/12/07:

I wonder who will take the initiative and do a nexus search on which democrats over the last couple of years have been loudest about the need to increase troop levels . Do they think we forgot ? I knew already that the Iraq Surrender group would provide cover for the Dems.in particular and anyone in Congress who wanted to bailout . That is why it was commissioned .

You are right ;this is not about the Iraqi people . It is about our own security .

Itsdb rated this answer Excellent or Above Average Answer

Question/Answer
Choux... asked on 01/11/07 - FIRST POLL RESULTS ARE IN

"Americans broadly reject President Bush's plan for a surge of U.S. forces into Iraq, with substantial majorities dismissing his arguments that it'll end the war more quickly and increase the odds of victory, an ABC News/Washington Post poll finds.

Indeed, rather than Bush bolstering public confidence, the national survey, conducted after his address to the nation on his new Iraq strategy, finds that a new high -- 57 percent -- think the United States is losing the war. Just 29 percent think it's winning."


CBS:

"Fifty percent of those who saw the speech said they disapprove of the president's proposals, while 37 percent said they approve. Just one-third of those surveyed said they support Mr. Bush's call to send more than 20,000 additional troops to Iraq.

Following the speech, 68 percent of Americans -- the same number as prior to the speech -- said they were uneasy about the president's ability to make decisions about Iraq."

~~~~~~~~~~~~~~~~~~~~~~~~~~~~~~~~~~~~~~~~~~~~

Bush made no inroads into his failing public opinion about his ability to conduct his War of Adventurism in Iraq after his speech last night. There is no rescuing the low opinion of his Presidency by this move to add 20,000 more troops to the Baghdad area.

The Bush Presidency is sooooo over; a complete failure in almost all areas despite having a rubberstamp Congress until a few days ago.

I guess he will just have to up his psychiatric drug intake over the next two years.

tomder55 answered on 01/12/07:

I did not expect that this decision would be popular. Good thing leadership doesn't depend on opinion polls .His perfomance is much better than the wishy-washy Pelosi and Reid who indicated less than a month ago that they would support an increase in troop strength. They should just cede their seats to Michael Moore and Cindy Sheehad since they run the party anyway. I still look for the Democrat "leadership" to put an alternative on the table besides surrender and perhaps 'symbolic votes' against the initiative .

Many of the Conservatives and Republicans who have defected have done so because the war was not being prosecuted aggressively enough . I expect if Bush is true to his word about easing some of the ROEs ,they will return to the fold.

The one thing that Bush did not do was to adequately equate Iraq with the larger war against jihadistan . Too much of the speech was about the Iraqi people and not enough about why our being there makes us safer .Truman was able to convince the US that Turkey and Greece was worth defending after WWII from Soviet aggression . Bush needs to make a simular case . His communication difficulties prevents him from being truely great .But history will treat him much kinder than opinion polls.

Choux... rated this answer Above Average Answer
Itsdb rated this answer Excellent or Above Average Answer

Question/Answer
Itsdb asked on 01/11/07 - Who's insane?



That seems to be the consensus among the critics, that Bush is insane for doing the same over and over in Iraq and after last night's speech it's "the same old story." Let's compare the 'insanity.'

As I pointed out yesterday, the backlash against the US over Somalia has begun. Tom noted the world is now "calling for blue helmet rapists to intervene."

Back in 1992, Boutros Boutros-Ghali called for military intervention in Somalia - now get this - "after traditional U.N. peace-keeping efforts have failed" to control the situation.

The first UN peacekeeping mission was established in 1948 to monitor and keep the peace between Israel and the Arab states. Since then a number of UN peacekeeping missions have taken place there, with armed forces in the Sinai, the Golan Heights, Lebanon and such. That's worked well hasn't it?

In 1949 the UN established the United Nations Military Observer Group to supervise the India and Pakistan ceasefire, and 58 years and thousands of dead later there is still no resolution to the Kashmir conflict.

Korea, Bosnia, Rwanda, the Congo, Sierra Leone, East Timor anyone?

Who's insane, Bush for continuing to press forward in the war on terror in spite of the appeasers and cowards, with the Husseins gone, al-Zarqawi gone, the Taliban out of power, two new functioning albeit delicate democracies? Or the rest of the world for calling on the UN to "keep the peace" in Somalia and the Sudan and rein in Iran and North Korea? That same UN whose history of sanctions, 'deplorings' and 'interventions' (doing the same thing over and over again and expecting a different result) have done nothing to stop the loss of millions of lives?

tomder55 answered on 01/11/07:

yes that is definitely one of those insane notions.

another one that has bothered me for a long time and ultimately helped shape my current philosophy is the real-politik concept of propping up brutal dictators ,who by jack-booted meas suppresses their populations,to achieve the illusion of stability .

another one is the notion that limited warfare saves lives.

another one that I heard yesterday on the other board is that the illiterate and ignorant are not worthy of self determination.

Itsdb rated this answer Excellent or Above Average Answer

Question/Answer
Itsdb asked on 01/10/07 - Questions

Elliot asked this in my previous post and I thought I'd asked them here.

Why is Darfur the "right war" and Somalia the "wrong war"? Or Iraq, for that matter?

tomder55 answered on 01/11/07:

The answer is that whichever war we engage in is the wrong war and whichever one we sit out is the right one. A decade later everyone is critical of our non-action in Rhwanda .

Itsdb rated this answer Excellent or Above Average Answer
ETWolverine rated this answer Excellent or Above Average Answer

Question/Answer
Itsdb asked on 01/10/07 - That didn't take long

As I suspected yesterday, the critics are out over US air strikes in Somalia...

UN chief Ban Ki-moon said, "The secretary-general is concerned about the new dimension this kind of action could introduce to the conflict and the possible escalation of hostilities that may result." (Though I digress, I heard another great Dennis Miller, "I took the UN tour recently...even the guidebook is spinless.")

Italy's Foreign Minister Massimo D'Alema said Rome opposed "unilateral initiatives that could spark new tensions in an area that is already very destabilised".

Ken Menkhaus, a 'US Horn of Africa specialist' said, "Before this, it was just tacit support for Ethiopia. Now the US has fingerprints on the intervention and is going to be held more accountable...This has the potential for a backlash both in Somalia and the region."

Amadeu Altafaj, spokesman for EU Development Commissioner Louis Michel said, "Any incident of this kind is not helpful in the long term."

French Foreign Ministry spokesman Jean-Baptiste Mattei said the US attacks "complicate the situation in Somalia and could increase the tensions that are already strong in the country."

The African Union’s chief executive, Alpha Oumar Konare stressed "the need for all concerned actors to refrain from any action likely to complicate the current situation."

Richard Cornwell, analyst with South Africa’s Institute for Security Studies, said the strikes "certainly complicates matters" for an African peacekeeping mission:

    "Any sort of peacekeeping mission is fraught with difficulties if there is no political situation it can be hooked on to...Certainly it would be a matter of some surprise were the AU to be able to mount a sizeable or effective operation in Somalia within the next few months."


Norway said Washington's explanation was "not sufficient" and that "We support the fight against terrorism but we think that the best way to pursue the fight is in a court of law," according to Norwegian foreign ministry undersecretary Raymond Johansen.

al-AP's Tom Raum apparently thinks Bush is just playing politics:

    "Send in more troops, set goals for the Iraqi government and assure Americans it's better to wage war there than here. And now the U.S. military is back in Somalia, too, once again attacking suspected terrorist targets...

    As Bush outlines his new Iraq strategy, he may well mention the new U.S. airstrikes in Somalia that targeted Islamic extremists.

    He can cite the war on terrorism's multiple fronts. It fits in with his fight-them-abroad-not-at-home thesis. Administration allies suggest the U.S. withdrawal from Somalia in 1993 helped strengthen the al-Qaida terror network."


There you have it, Bush has screwed everything up again. He attacked a country that was not a threat without provocation, he didn't ask the UN (or Europe) and he's mucked up the effort to get a coalition together to clean things up. No word on any criticism of Ethiopa's involvement.

tomder55 answered on 01/11/07:

had we butted out the Ethiopians would've been able to completely destroy the jihadists completely under the radar. Now the world will bemoan the use of disproportionate force ...and already are calling for blue helmet rapists to intervene.

ETWolverine rated this answer Excellent or Above Average Answer
Itsdb rated this answer Excellent or Above Average Answer

Question/Answer
paraclete asked on 01/11/07 - Blow up the politicians - That's the Islamc way

Parliament 'target of rockets'
Email Print Normal font Large font January 10, 2007



A terror suspect who allegedly bought five stolen army rocket launchers said he would use them to blow up "the nuclear place" and Parliament House, a Sydney court has been told.

It is alleged the man, Mohammad Ali Elomar, was later seen near Sydney's Lucas Heights nuclear reactor, where an access lock for a gate to a nearby reservoir had recently been cut.

The claims were contained in a police statement tendered to Central Local Court, where Taha Abdul Rahman - the man accused of selling the rocket launchers - was today refused bail.

Abdul Rahman, 28, from Leumeah, in Sydney's south-west, faces 17 charges related to the possession and supply of seven rocket launchers allegedly stolen from the Australian Defence Force.

The statement did not detail how Abdul Rahman allegedly acquired the weapons.

But according to the statement, he sold five rocket launchers to Elomar through an associate, Adnan Darwiche, who was last year jailed for life for double murder.

Elomar, 41, of Bankstown, is one of nine Sydney men charged in late 2005 with conspiring to make explosives in preparation for a terrorist attack.

Abdul Rahman allegedly sold Darwiche a rocket launcher on September 30, 2003 for $15,000, and on October 9 sold him six more for $70,000.

Darwiche said Elomar had given him most of the $70,000 to buy five rocket launchers on his behalf, the statement said.

Prior to the purchase, Elomar allegedly commented: "Look what is happening overseas. It is a war against Muslims. We should do something about it over here."

According to the police statement, a source claimed Elomar was "elated he had received the rocket launchers".

"Elomar first joked that he would use them to blow up Parliament House and later more seriously said, `I am going to blow up the nuclear place'," the statement alleged.

On December 28, 2004, Elomar and two co-accused were allegedly seen near the Lucas Heights reactor, each later giving differing versions of their activities to police.

Elomar and the eight other terror suspects are still before the courts.

Darwiche was one of three men convicted of shooting dead two people at a Greenacre home in October 2003, during a feud with another family.

The court was told the killers dismissed using the rocket launcher on that occasion, as they believed the warhead could pass through the fibro house without exploding.

Abdul Rahman was arrested last week following a lengthy investigation by Strike Force Ridgecrop, established by NSW and federal police to probe the possession and supply of rocket launchers.

The police statement alleges that from 2001, he was involved in the supply of illegal handguns, explosives and rocket launchers to the criminal element in NSW.

Abdul Rahman allegedly said he made just $1,000 profit on each rocket launcher sold. Only one has so far been recovered.

Abdul Rahman's wife, Belinda Rahman, gave evidence at his bail hearing after defence barrister William Brewer argued his client needed to care for his two young daughters at home.

As the accused looked on via videolink, Mr Brewer urged that bail considerations not be "lost in a bit of hysteria" over the rocket launchers.

But prosecutor Wendy Abraham QC opposed bail, telling the court that five of the rocket launchers introduced into the community by Abdul Rahman "ended up with someone who is currently charged with terrorism offences".

"The bottom line is these are extremely powerful weapons designed to penetrate armour and concrete," she said.

Magistrate Allan Moore refused bail, remanding Abdul Rahman in custody "for the protection of the community at large".

He faces two counts of dishonestly receiving stolen property, seven counts of unauthorised possession of a prohibited weapon, seven counts of unauthorised supply of a prohibited weapon, and one count of possession of ammunition.

He will return to court on March 21.

AAP

tomder55 answered on 01/11/07:

thankfully he was captured .

He faces two counts of dishonestly receiving stolen property, seven counts of unauthorised possession of a prohibited weapon, seven counts of unauthorised supply of a prohibited weapon, and one count of possession of ammunition

is that it ? you gotta do better than that . Aren't there conspiracy to commit murder or terrorism laws ?

paraclete rated this answer Excellent or Above Average Answer

Question/Answer
paraclete asked on 01/11/07 - Blow up the politicians - That's the Islamc way

Parliament 'target of rockets'
Email Print Normal font Large font January 10, 2007



A terror suspect who allegedly bought five stolen army rocket launchers said he would use them to blow up "the nuclear place" and Parliament House, a Sydney court has been told.

It is alleged the man, Mohammad Ali Elomar, was later seen near Sydney's Lucas Heights nuclear reactor, where an access lock for a gate to a nearby reservoir had recently been cut.

The claims were contained in a police statement tendered to Central Local Court, where Taha Abdul Rahman - the man accused of selling the rocket launchers - was today refused bail.

Abdul Rahman, 28, from Leumeah, in Sydney's south-west, faces 17 charges related to the possession and supply of seven rocket launchers allegedly stolen from the Australian Defence Force.

The statement did not detail how Abdul Rahman allegedly acquired the weapons.

But according to the statement, he sold five rocket launchers to Elomar through an associate, Adnan Darwiche, who was last year jailed for life for double murder.

Elomar, 41, of Bankstown, is one of nine Sydney men charged in late 2005 with conspiring to make explosives in preparation for a terrorist attack.

Abdul Rahman allegedly sold Darwiche a rocket launcher on September 30, 2003 for $15,000, and on October 9 sold him six more for $70,000.

Darwiche said Elomar had given him most of the $70,000 to buy five rocket launchers on his behalf, the statement said.

Prior to the purchase, Elomar allegedly commented: "Look what is happening overseas. It is a war against Muslims. We should do something about it over here."

According to the police statement, a source claimed Elomar was "elated he had received the rocket launchers".

"Elomar first joked that he would use them to blow up Parliament House and later more seriously said, `I am going to blow up the nuclear place'," the statement alleged.

On December 28, 2004, Elomar and two co-accused were allegedly seen near the Lucas Heights reactor, each later giving differing versions of their activities to police.

Elomar and the eight other terror suspects are still before the courts.

Darwiche was one of three men convicted of shooting dead two people at a Greenacre home in October 2003, during a feud with another family.

The court was told the killers dismissed using the rocket launcher on that occasion, as they believed the warhead could pass through the fibro house without exploding.

Abdul Rahman was arrested last week following a lengthy investigation by Strike Force Ridgecrop, established by NSW and federal police to probe the possession and supply of rocket launchers.

The police statement alleges that from 2001, he was involved in the supply of illegal handguns, explosives and rocket launchers to the criminal element in NSW.

Abdul Rahman allegedly said he made just $1,000 profit on each rocket launcher sold. Only one has so far been recovered.

Abdul Rahman's wife, Belinda Rahman, gave evidence at his bail hearing after defence barrister William Brewer argued his client needed to care for his two young daughters at home.

As the accused looked on via videolink, Mr Brewer urged that bail considerations not be "lost in a bit of hysteria" over the rocket launchers.

But prosecutor Wendy Abraham QC opposed bail, telling the court that five of the rocket launchers introduced into the community by Abdul Rahman "ended up with someone who is currently charged with terrorism offences".

"The bottom line is these are extremely powerful weapons designed to penetrate armour and concrete," she said.

Magistrate Allan Moore refused bail, remanding Abdul Rahman in custody "for the protection of the community at large".

He faces two counts of dishonestly receiving stolen property, seven counts of unauthorised possession of a prohibited weapon, seven counts of unauthorised supply of a prohibited weapon, and one count of possession of ammunition.

He will return to court on March 21.

AAP

tomder55 answered on 01/11/07:

thankfully he was captured .

He faces two counts of dishonestly receiving stolen property, seven counts of unauthorised possession of a prohibited weapon, seven counts of unauthorised supply of a prohibited weapon, and one count of possession of ammunition

is that it ? you gotta do better than that . Aren't there conspiracy to commit murder or terrorism laws ?

paraclete rated this answer Excellent or Above Average Answer

Question/Answer
paraclete asked on 01/11/07 - The mouth from the south has struck again

Australia 'a convict nation of liars'

By Paul Carter

January 11, 2007 09:33pm
Article from: AAP


AUSTRALIA'S controversial mufti Sheik Taj al-Din al-Hilali has landed himself in hot water again, this time by saying Australian Muslims are more entitled to the country than those with a convict heritage.

Prime Minister John Howard said today he expected the mufti's comments to amuse Australians, however one Muslim leader was quick to apologise for the sheik.

Speaking in Arabic on Egyptian television Sheik al-Hilali said, according to a Seven Network translation, that white Australians arrived in the country shackled as convicts.

"We (Muslims) came as free people. We bought our own tickets. We are entitled to Australia more than they are,'' he said.

The mufti was on the Egyptian chat show explaining the controversy last year over his comments likening immodestly-dressed women to uncovered meat.

But according to the translation, he said the controversy was a white conspiracy aimed at terrorising Australian Muslims.

Mr Howard laughed off the spiritual leader's convict comments.

"I think it will bring a wry smile to the face of Australians who don't actually feel the least bit offended that many of our ancestors came here as convicts,'' Mr Howard said.

"It's almost a badge of honour for many Australians.''

But while the convict jibes might be forgiven by some, as they are when levelled by English cricket fans, the sheik's comments are expected to cause outrage in some quarters - especially the claim that white Australians "are the biggest liars''.

The mufti told Egytpian television that outrage over his controversial meat sermon was "a calculated conspiracy'', that started with him, "in order to bring the Islamic community to its knees''.

He also said "Australian law guarantees freedoms up to a crazy level'', when reportedly referring to anti-Muslim courts and the harsh sentencing of a Muslim gang rapist in Sydney.

Islamic Friendship Association president Keysar Trad today criticised and defended his close friend, saying some of his comments were "ill-advised''.

"I believe his intention was to indicate that we choose to be in Australia because we love Australia, because his Egyptian interviewers were asking him why he stays and puts up with the controversy here,'' Mr Trad said.

"He was defending Australia, but saying sometimes democracy fails, and the reaction to his comments put a lie to the democratic principle of free speech.

"But I, as a Muslim Australian, do feel the need to apologise for anyone who is offended by these comments.''

Mr Trad also questioned the accuracy of the translation, saying the mufti's opponents were waiting with malicious intent to misrepresent his comments.

"It's evident by the controversy that has erupted again that there are people out there watching every comment he makes,'' Mr Trad said.

+++++++++++++++++++++++++++++++++++++++++++++++++++++++
As a sixth generation Australia who's Irish forebares came to this country as free settlers in 1822. I am deeply offended by this upstart's comments. My people did the hard yards in this country and my grandfather lies in Ypres, France, he fell in the first world war defending freedom.

He's just pig meat. What I say is it is time for him to leave. What do you say?

tomder55 answered on 01/11/07:

Go for it Clete !! Pig meat is a good description. I don't know your geneology but I doubt if every Aussie of European ancestory came from the prison population as you have proved in this posting .Besides ,as I recall someone who had a debt was a worthy candidate of prison time back then ;and many people would be singing 'Jail House Rock 'if that were true today .

But even if your ancestors were ax murderers why would it be relevent today ? Do the sons really bear the sins of the fathers for over 6 generations ?

"Fathers shall not be put to death for their sons, nor shall sons be put to death for their fathers; everyone shall be put to death for his own sin." (Deuteronomy 24:16)

"The person who sins will die. The son will not bear the punishment for the father’s iniquity, nor will the father bear the punishment for the son’s iniquity; the righteousness of the righteous will be upon himself, and the wickedness of the wicked will be upon himself." (Ezekiel 18:20)


The moron is blessed with living in a great free nation and instead of becoming apart of it he would peel off a slice and make it into hades like the rest of the ummah has become .

paraclete rated this answer Excellent or Above Average Answer

Question/Answer
paraclete asked on 01/11/07 - FOI - the truth is - unavailable

The truth is out there: in Wellington not Canberra

When New Zealand passed the Official Information Act in 1983 it was greeted with hails of dismay by public service unions, lawyers and academics. Cartoonists depicted the act as a useless halfway compromise with the Official Secrets Act. The then prime minister Robert Muldoon described it as a 'nine day wonder'.

The Australian FoI Act was passed at the same time but was greeted with far greater enthusiasm. Then prime minister Bob Hawke boldly stated: ''Information about government operations is not, after all, some kind of 'favour' to be bestowed by a benevolent government or to be extorted from a reluctant bureaucracy. It is, quite simply, a public right.''

What a difference 23 years and some very different attitudes of our political masters can make.

In Canberra, where we were promised a new democratic right, we now have public officials who will fight all the way to the High Court to deny access to old policy documents.

We have a Canberra public service that appears, by actions and words, nervous about any level of transparency, and a cabinet that insists it needs to keep every bit of advice and discussion completely under wraps in order to function.

Like modern day Jeremiahs they preach of the harm that will befall Australian democracy and the Westminster system if journalists or others get their hands on policy documents. Peter Costello summed up the Government view in August last year, warning: ''We do have candid and fearless moments in the cabinet. This may surprise you but it does happen. We would be far less fearless and candid in the cabinet if we knew that the minutes were going to be released under FoI. That protection is very, very important to us.''

But look at New Zealand where the Official Information Act was treated as a joke. Their act grants access to so much cabinet Information that there are guidelines published on the internet on releasing it.

The New Zealand Cabinet Manual states: "There is no blanket exemption for any class of papers under the Official Information Act 1982. Cabinet and Cabinet committee records are therefore covered by the Official Information Act in the usual way, and every request for Cabinet records must be considered on its merits against the criteria in the Act."

It's impossible to imagine any senior Australian public servant adopting the advice of Marie Shroff, the New Zealand's secretary of cabinet and clerk of the Executive Council for 15 years, and now the Privacy Commissioner, who said:

"If I, as a civil servant, write a Cabinet paper which I expect to be sought for public release I am going to be extraordinarily careful to get my facts right, to avoid trespassing into politics, to give comprehensive reasons for and against a proposal, and to think very carefully about my recommendations. My advice will therefore be balanced, accurate and comprehensive."

It's just as inconceivable the Commonwealth Treasury would issue the following press statement put out by their New Zealand counterpart on June 1:

"In previous years the Treasury has received numerous requests under the Official Information Act for Budget related information. Last year, in anticipation of such requests we released a number of Budget 2005 documents onto our website shortly after Budget day.

"Following the success of the 2005 release, we have made available the following Budget 2006 documents, which are among the most frequently requested."

The site includes copies of cabinet papers, the budget strategy, briefing papers and other documents relating to the 2006 New Zealand budget.

According to the secretaries of Prime Minister and Cabinet and Treasury in Australia, and testimony accepted by the High Court in last year's landmark FoI decision in the McKinnon case, releasing this type of information would bring government to a standstill.

There are numerous examples of the New Zealand government publishing documents on the net that would have Australian ministers and senior public servants reaching for the smelling salts. From climate change to the economy to immigration, New Zealand routinely publishes material Australian bureaucrats would fight to the death to keep secret. And somehow, their government keeps functioning, and no one complains government suffers when the public gets to see more than the press release.

See examples of what's available in New Zealand here:
http://www.climatechange.govt.nz/resources/cabinet/index.html
http://www.treasury.govt.nz/iarfs/cbc05-276/default.asp
http://www.mfe.govt.nz/issues/organisms/law-changes/commission/cabinet-paper1.pdf
http://www.med.govt.nz/templates/MultipageDocumentTOC____22462.aspx
http://www.immigration.govt.nz/NR/rdonlyres/4B5AEDE8-0D6A-46A8-8CA2-531EC33299DF/0/EDC0419010.doc http://www.dia.govt.nz/diawebsite.nsf/wpg_URL/Legislative-Reviews-Local-Government-Act-Review-Local-Government-Cabinet-Papers?OpenDocument

- Rick Snell

SMH FoI Editor Matthew Moore is on leave.
Rick Snell lectures in law at the University of Tasmania where he specialises in FoI.
++++++++++++++++++++++++++++++++++++++++++++++++++
"If I, as a civil servant, write a Cabinet paper which I expect to be sought for public release I am going to be extraordinarily careful to get my facts right,
Now wouldn't it have been nice if this attitude permiated all political processes in the "Free" World?

tomder55 answered on 01/11/07:

If I was a public servant ,knowing every little scribble or unedited comment were to become part of the public record I would make an extra effort to destroy as much of it as I could before leaving office. Had Nixon destroyed his recordings or had never made them in the first place it is unlikely that he would've been forced from office.Watergate would've been no more relevent than the 'Tea Pot Dome Scandal '.

I like the loosening of FOI rules but I don't agree that it should be a blank check to forage through the national archives .

paraclete rated this answer Excellent or Above Average Answer

Question/Answer
Choux... asked on 01/11/07 - BUSH ISSUES FATWA ON SYRIA AND IRAN

"I truly believed that President George W. Bush was at least going to announce that Condi Rice was going on a shoe-buying tour of Tehran and Damascus, and I was also fully confident such a "diplomatic" trip would have been just another Karen Hughes-type PR campaign. If Bush were a sane person he would have taken the simple Karl Rovian step of an opening of negotiations, heeded that one tiny piece of the Baker-Hamilton study, and taken a little wind out of the war opposition's sails.(The mainstream media would have given him great press for it.)

Instead, Bush issued a fatwa against Iran and Syria. He is sending in an armed-to-the-teeth carrier group into the Persian Gulf and he is escalating the war in Iraq. Bush is pursuing Richard Nixon's Cambodia strategy: escalate and expand the war and try to "win" it. He'll probably ratchet up the air war too just as Nixon did.

The net-roots organization, the "World Can't Wait" (worldcantwait.org), which has been calling for Bush's removal from office for years, just got a shot in the arm with Bush's bellicose and insane speech tonight. Bush must be removed from office before he ignites a global catastrophe.

If Bush sparks a "Gulf of Tonkin incident" in the Persian Gulf, and gets us into a shooting war with Iran, or launches air strikes against Iran's nuclear facilities, he could easily create the conditions for the outbreak of a regional war, or even something we might call World War III.

Bush's speech tonight showed that he is a Neo-Con, right-wing Christian fundamentalist. He has not a clue about what is going on in Baghdad. He spoke of "neighborhoods" being secured and U.S. "check points" being set up. He talked of an attempt to block arms and other supplies coming from Iran. He will follow not one of his "Uncle Jim" Baker's suggestions; he's still rebelling against his daddy. Bush was speaking about Al-Maliki as if he is really going to move against the Mahdi Army of Moqtada al-Sadr. A thousands times, Bush showed in this speech tonight that he is completely out of touch with reality. Bush has drunk the Kool-Aid, there is no turning back. His messianic fantasies about the Middle East were on display for the all world to see.

Kudos to Keith Olberman for latching on to Bush's statements about the U.S. stance toward Iran and Syria; that was the most important revelation of the speech tonight: Bush appears to be threatening a regional war. Arianna Huffington deserves our gratitude for a clear and forceful performance on Joe Scarborough's show. And even Chris Matthews did an adequate job following Olberman's lead on the Iran and Syria issue. The Democrats in Congress must stop this madness before Bush leads the United States and the world into an even bigger bloodbath in the Middle East, one that could make Baghdad look like -- to quote neo-con Kenneth Adelman -- a "cake walk."" Joseph A. Palermo, Blogging
~~~~~~~~~~~~~~~~~~~~~~~~~~~~~~~~~~~~~~~~~~~~~~~

So....

tomder55 answered on 01/11/07:

Bush is pursuing Richard Nixon's Cambodia strategy: escalate and expand the war and try to "win" it. He'll probably ratchet up the air war too just as Nixon did.

Well I certainly hope so ! When Nixon ratchetted up the bombing campaign during operation LINEBACKER 1 , and mined Haiphong Harbor the N.Vietnamese were as I recall a little more willing to come to the negotiation table.

North Vietnam felt the full fury of U.S. airpower, with B-52 bombers, and other jet bombers delivering new laser guided bombs and other high tech ordnance. Fuel depots, rail lines, power plants, industrial centers, bridges and numerous other targets were slammed by accurate bombing by the U.S. LINEBACKER was considered a great success and it prompted one former aide to the Johnson administration to say " Linebacker had a greater impact in its first four months than Rolling Thunder had in four years." North Vietnam was sent reeling and sought to reopen peace negotiations, and as a result President Nixon ordered a bombing halt on October 23, 1972. Peace looked like it was close at hand at last.

I expect that eventually we will be sitting down with those nations ,but under better terms than what we have presently .Besides it is really no secret that informal exchanges are being made through third party negotiations .But in the meantime I would like to see consequences applied to Iran for every IED that is exploded in Iraq ,like an Iranian submarine or an Iranian air defense system destroyed. For every attack by Iran's proxies like Mookie al-Sadr there should be a reciprocal act.

In fact ,we may have already initiated the opening salvos .

Bush won't sparks a "Gulf of Tonkin incident" ....the Mahdi-Hatter might. The way I see it ;unless he starts a war with the U.S. in the next year ,domestic politics in Iran will remove him from power. Time is not on his side so in desperation I see him staging an incident .Bush knows this too . He has put sufficient force in the region to deal with Ahmamadjihad.

Choux... rated this answer Above Average Answer
Itsdb rated this answer Excellent or Above Average Answer

Question/Answer
Choux... asked on 01/11/07 - BUSH! SEND MORE TROOPS AND YOU GO, TOO!!!!!

1/10/07

Dear Mr. President,

Thanks for your address to the nation. It's good to know you still want to talk to us after how we behaved in November.

Listen, can I be frank? Sending in 20,000 more troops just ain't gonna do the job. That will only bring the troop level back up to what it was last year. And we were losing the war last year! We've already had over a million troops serve some time in Iraq since 2003. Another few thousand is simply not enough to find those weapons of mass destruction! Er, I mean... bringing those responsible for 9/11 to justice! Um, scratch that. Try this -- BRING DEMOCRACY TO THE MIDDLE EAST! YES!!!

You've got to show some courage, dude! You've got to win this one! C'mon, you got Saddam! You hung 'im high! I loved watching the video of that -- just like the old wild west! The bad guy wore black! The hangmen were as crazy as the hangee! Lynch mobs rule!!!

Look, I have to admit I feel very sorry for the predicament you're in. As Ricky Bobby said, "If you're not first, you're last." And you being humiliated in front of the whole world does NONE of us Americans any good.

Sir, listen to me. You have to send in MILLIONS of troops to Iraq, not thousands! The only way to lick this thing now is to flood Iraq with millions of us! I know that you're out of combat-ready soldiers -- so you have to look elsewhere! The only way you are going to beat a nation of 27 million -- Iraq -- is to send in at least 28 million! Here's how it would work:

The first 27 million Americans go in and kill one Iraqi each. That will quickly take care of any insurgency. The other one million of us will stay and rebuild the country. Simple.

Now, I know you're saying, where will I find 28 million Americans to go to Iraq? Here are some suggestions:

1. More than 62,000,000 Americans voted for you in the last election (the one that took place a year and half into a war we already knew we were losing). I am confident that at least a third of them would want to put their body where there vote was and sign up to volunteer. I know many of these people and, while we may disagree politically, I know that they don't believe someone else should have to go and fight their fight for them -- while they hide here in America.

2. Start a "Kill an Iraqi" Meet-Up group in cities across the country. I know this idea is so early-21st century, but I once went to a Lou Dobbs Meet-Up and, I swear, some of the best ideas happen after the third mojito. I'm sure you'll get another five million or so enlistees from this effort.

3. Send over all members of the mainstream media. After all, they were your collaborators in bringing us this war -- and many of them are already trained from having been "embedded!" If that doesn't bring the total to 28 million, then draft all viewers of the FOX News channel.

Mr. Bush, do not give up! Now is not the time to pull your punch! Don't be a weenie by sending in a few over-tired troops. Get your people behind you and YOU lead them in like a true commander in chief! Leave no conservative behind! Full speed ahead!

We promise to write. Go get 'em W!

Yours,

Michael Moore

~~~~~~~~~~~~~~~~~~~~~~~~~~~~~~~~~~~~~~~~~~~~~~~~

tomder55 answered on 01/11/07:

Michael Moore is too rediculous to respond to . But my theme today is that more troops by will not make a difference unless we take off the gloves and let them do their job.

Bing West, in an subscription-only article in this month's Atlantic, argues that America has messed things up in Iraq, not primarily for lack of committing resources, but by using them poorly.{too long to c/p the whole article so I will spare you and hit on some highlights }.

Here is an exchange with an angry Iraqi police officer who is being questioned by the bureaucracy which is obsessed with how one terrorist may have died under the Iraqi police commander's questioning. :

Tracy walked outside and escorted the compact and unsmiling police chief, Colonel Farouq, into his office.

"Every American is asking how one terrorist died," he said angrily. "We questioned him, and he died. That's all I say. He betrayed my police. [My police officers'] heads were tossed in the dirt in Baiji. And all you ask is how a terrorist died."

"We go by the law," Tracy said. "We have rules we follow."

"Rules? What about nine bodies without heads? What about my brother's body?" Farouq raged. "My mother complains I have lost the family because I help Americans." Farouq's younger brother had been killed in the ambush, his body mutilated.

"Baiji's a hundred kilometers from here," the battalion commander, Lieutenant Colonel James Donnellan, said. "I'll take a force there. You can come with me."

"When?" Farouq demanded to know.

"Higher has to coordinate," Donnellan said. "Two or three days."

"The bodies will be gone by then. You investigate a dead terrorist right away. But my brother has to wait," Farouq said. "Your rules? You won't see strong Iraqi police the American way for a hundred years."


We already have numerical advantage and clearly outspend our enemies. We have been fighting with our hands tied behind our backs with silly rules of engagement.... or as West puts it "American troops mockingly refer to arrests of insurgents as 'catch and release.'"

We have the numbers to get the job done . The open question is do we have the will. Given the election results and rhetoric from people like Michael Moore I have my doubts.




Choux... rated this answer Bad/Wrong Answer

Question/Answer
paraclete asked on 01/11/07 - Send them in, clown


January 11, 2007
Despite it being the great, avoidable tragedy of the decade, part of me has rather enjoyed the endless quagmire that Iraq has become. It gives me the quiet satisfaction of justice being done – a sensation, incidentally, wholly absent from Saddam’s barbaric execution. The sheer hideousness of the outcome so perfectly highlights the idiocy of the idea, the inadequacy of its justification and the ineptness of its implementation. It’s like a Greek tragedy, with an act of hubris leading to inexorable nemesis. It's not unlike Oedipus, except that instead of the protagonist killing his father, he killed “the man who tried to kill my dad”. That, and I don’t even want to think about George Jr gettin’ it on with Barbara Bush.
The other difference is that in Greek tragedy, the hubris leads to the death of the protagonist. Which would have been more apt than the death of 3,000 American troops, and countless civilians. Still, President Bush has suffered a political death, at any rate, and has become such a ‘lame duck’, as the American jargon has it, that I’m astonished Dick Cheney hasn’t tried to hunt him.

And yet today we learn that despite the massive slap in the face that was the Democrats’ seizure of both Houses of Congress, President Bush wants to send 21,000 more troops to Iraq. Before we get onto this, one thing is abundantly clear. There should be no more Australian troops, and the ones that are there should be brought back as fast as possible. There is clearly no justification left for our presence there other than the most obvious one – our Prime Minister’s need to ingratiate himself with the US. Given our reliance on the US alliance for our defence, this was perhaps unsurprising. But it has gone on more than long enough. We’ve done our bit, and it’s time we left Bush to clean up his own mess.

And this is why I’m sympathetic to the idea of sending more troops. The Democrats have sought to oppose it, and I can understand why – it should resonate well with American voters. But I'm not sure that's right. The fact is that this situation is entirely the fault of the President and the Congress, who approved it at the time. To abandon Iraq now would be even crueller to ordinary Iraqis than the invasion was in the first place. It’s clear that the domestic forces have little to no chance of fixing things, and the hornet’s nest that Bush so incautiously poked still needs to be quietened.

There seems to be a consensus that not enough troops were sent initially, a decision that has been sheeted home to Donald Rumsfeld. If so, surely more is a good idea? Especially as it's still so clear that the numbers on the ground now are inadequate to secure the country. Whether even 20,000 more will suffice is also impossible to ascertain. Let’s hope they send sufficient numbers for the environment to become safer, rather than even more dangerous due to the increased number of target.

I do feel for the troops. I know people who have fought in Iraq, and the prospect of losing more people like them merely to gratify the grandiose vanity of an insipid simpleton does not exactly appeal. But what other choice does Bush, or the US, really have? Trying to actually end this farce properly is surely a better option – and a nobler one, even – than just washing their hands of the whole business.

The so-called Powell Doctrine – of intervening reluctantly, with clear and realistic objectives and with such an overwhelming force that you can achieve them without many casualties – has never looked better than it does from here. Especially when contrasted with the Bush Doctrine, which seems to be to intervene ignorantly, with little preparation, and make an open-ended commitment without enough troops to realistically achieve it, relying instead on foolish optimism. (Perhaps the Iraqis are just taking a really long time to put together those garlands of flowers with which we were told they’d welcome their liberators?) We can only hope that there are other commanders as restrained and pragmatic as Colin Powell within the US Army. And, more importantly, that the administration listens to them this time.

I don’t know enough about the situation on the ground to know whether the proposed plan will help matters much. But let’s hope that, for the first time, the Americans actually know what they’re doing.

Dominic Knight
++++++++++++++++++++++++++++++++++++++++++++++++
so the question is, is the decision to send more troops the act of an insipid simpleton, as this article suggests?

tomder55 answered on 01/11/07:

The Powell doctrine when it all comes down the the basics was nothing more than what Clinton did..... A brief period of violence that leaves the underlying issue unresolved ;but for the moment you can strut like a cock because you showed them . The flaw in the Powell Doctrine was not the massive use of force;but the obsession with exit strategy over victory .It also was designed for limited objectives .

Here is my response to Fred on a simular question on the Christianity Board :

I understand that the President is difficult to listen to .I find it easier to read the transcripts the next day . Here is the speech . If before November, he had introduced this plan plus a serious plan to secure our borders (which he's still ignoring) Democrats would still be the minority party.

An additional five divisions may not sound like a lot. But if we also change the "rules of engagement"(ROE) that these troops are required to follow, that should be enough to quell the Sunni and Shiite militias.The ROEs in Iraq are so stringent, it's reasonable to wonder how we can even wage war at all ; especially against an enemy that recognizes no rules or morality in its own fighting.

Marine Capt. Rob Secher complained to his family that "any time an American fires a weapon there has to be an investigation into why there was an escalation of force." Secher was later killed in action.


Last year a U.S. drone spotted 100 Taliban leaders who had gathered for a funeral. All could have been taken out . Instead, ROEs kept us from doing so.How many missed chances like that have there been? How many innocent civilians and allied troops have died due to such silly formalities? They fight and then face hard time in Leavenworth. What nonsense !!!

This week's attack on al-Qaida in Somalia using AC-130 gunships and killing the leader responsible for the 1998 embassy bombing may have finally signaled that the President finally gets it. Time to take the gloves off and unleash the dogs. Play by Chicago rules (Sean Connery quote from the movie 'The Untouchables ' :

Malone: You said you wanted to get Capone. Do you really wanna get him? You see what I'm saying is, what are you prepared to do?

Ness: Anything and everything in my power.

Malone: And *then* what are you prepared to do? If you open the can on these worms you must be prepared to go all the way because they're not gonna give up the fight until one of you is dead.

Ness: How do you do it then?

Malone: You wanna know how you do it? Here's how, they pull a knife, you pull a gun. He sends one of yours to the hospital, you send one of his to the morgue. That's the Chicago way, and that's how you get Capone! Now do you want to do that? Are you ready to do that?

Ness: I have sworn to capture this man with all legal powers at my disposal and I will do so.

Malone: Well, the Lord hates a coward. Do you know what a blood oath is, Mr. Ness?

Ness: Yes.

Malone: Good, 'cause you just took one.


It looks like his plan went into effect before the speech began .Of course the NY Slimes is spinning this as "more trouble in the neighborhood" but in fact it is what it says it is: more dead bad-guys.



Still this will not be finished until we deal with Iran's "surge " in Iraq.




paraclete rated this answer Excellent or Above Average Answer

Question/Answer
Itsdb asked on 01/10/07 - Terrorists strike West Texas

***Breaking news***

By Karen Smith Welch

A tail-flicking terrorist cell knocked out power to Texas and New Mexico Xcel Energy customers 616 times in 2006.

Most were suicide missions.

"As far as I know, no squirrel ever survived the encounter with a power line," Xcel spokesman Wes Reeves said.

Amarillo customers found themselves the victims of 211 of those strikes last year.

Animalistic attacks on the power grid continued Tuesday when a suicide squirrel took out electric service to 4,564 customers in southwest Amarillo.

"The squirrel did not make it - one fatality," Reeves said.

This time, the perpetrator hit Xcel's Southwest 34th Avenue substation, shorting out the system and causing a 26-minute outage for neighborhoods within a mile in all directions, Reeves said.

"I'm always blaming my problems on a squirrel," he said. "It's kind of a utility spokesman's ongoing joke."

~~~~~~~~~~~~~~~~~~~~~~~~~~~~~~~~~~~~~~~~~~~~~~~~~~~~~~
Gotcha.

tomder55 answered on 01/10/07:

Good thing your Senator isn't Chuck Shumer . He would have an alarmist Sunday morning press conference where he would demand that Homeland security funding be made available to the entire State for the purpose of sinking the power grid underground ;and for the purpose of having the national guard patrol the overhead lines in the interim.

Itsdb rated this answer Excellent or Above Average Answer

Question/Answer
ETWolverine asked on 01/10/07 - A name for the Mahdi Hatter

In the prior post, Tom mentioned:

    We have to settle on a name for the Mahdi Hatter

    I kind of like Ahm~a~mad~jihad .I have been spelling it that way for a couple of weeks and nobody has seemed to notice.


I figure it's a topic worthy of its own string.

I actually did notice that Tom had been spelling it that way, and I approve.

I've actually been using "Ahmad-genocide", but I think Tom's is pretty good too.

Anyone else have an opinion? (What, am I nuts? Of course you all have opinions...)

Elliot

tomder55 answered on 01/10/07:

or it could be simplified to Ahminajihad

ETWolverine rated this answer Excellent or Above Average Answer
Itsdb rated this answer Excellent or Above Average Answer
Mathatmacoat rated this answer Excellent or Above Average Answer

Question/Answer
Itsdb asked on 01/09/07 - And you thought LA was bad...

Iran smog 'kills 3,600 in month'

Air pollution is estimated to have killed nearly 10,000 people in Tehran over a one-year period, including 3,600 in a month, Iranian officials say.

Most of the deaths were caused by heart attacks and respiratory illnesses brought on by smog, they said.

The scale of the problem led one senior official to say living in the Iranian capital was like "collective suicide".

Cheap fuel encourages car use in Iran, correspondents say, and many vehicles do not meet global emissions standards.

"It is a very serious and lethal crisis, a collective suicide," the director of Tehran's clean air committee, Mohammad Hadi Heydarzadeh, told an Iranian newspaper.

"A real revolution is needed to resolve this problem."

He said air quality had worsened and was linked to some 3,600 deaths in October. Many of the deaths were caused by heart attacks brought on by the air pollution.

New figures showed a sharp rise in pollution-related deaths in Iran, where 9,900 people died of pollution in the previous Iranian year (March 2005 to March 2006).

The latest assessments were based on World Bank figures which extrapolate mortality rates according to certain levels of pollution.

~~~~~~~~~~~~~~~~~~~~~~~~~~~~~~~~~~~~~~~~~~~~~~~~~~~~~~~

Everyone ready to “bow down to the greatness of the Iranian nation?” Maybe the Mahdi Hatter will buddy up with alGore to fix this? Come to think of it, the two of them do kind of resemble each other...







tomder55 answered on 01/09/07:

not gonna happen . Stinky ;who now has his own web site ...(click on the English translation to see his rants in a non-chickenscratch format)
is the source of the foul air . Actually I think he attacked NYC yesterday .



thanks for the link to this pix ;it's a classic .

Itsdb rated this answer Excellent or Above Average Answer

Question/Answer
Dark_Crow asked on 01/09/07 - What ever happened to patriotic reporters?............

http://www.opinionjournal.com/federation/feature/?id=110009203

First myth. Media technology had changed. Vietnam was the first war in which television was available to a mass audience, and, as both critics and admirers of TV unite in saying, television brings the war home in often unsettling graphic images. But the Second World War also brought the struggle home through Pathé and Movietone newsreels shown in thousands of theaters nationwide at a time when Americans went to the movies remarkably often. Moreover, television accounts between 1962 and 1968 were not critical of the American effort in Vietnam, and public support for the war then actually increased.
Second myth. The war in Vietnam was conducted without censorship. As a result, the press, with trivial exceptions, could report anything it wanted. Moreover, the absence of a formal declaration of war made it possible for several Americans, including important journalists, to travel to Hanoi, where they made statements about conditions there that often parroted the North Vietnamese party line. But the censorship rules in the Second World War and in Korea, jointly devised by the press and the government, aimed at precluding premature disclosure of military secrets, such as the location of specific combat units and plans for military attacks. The media problem in Vietnam was not the disclosure of secrets but the conveying of an attitude.
Third myth. The press did not report military matters with adequate intelligence and context because few, if any, journalists had any military training. But that has always been the case. One veteran reporter, S.L.A. Marshall, put the real difference this way: once upon a time, "the American correspondent . . . was an American first, a correspondent second." But in Vietnam, that attitude shifted. An older journalist in Vietnam, who had covered the Second World War, lamented the bitter divisions among the reporters in Saigon, where there were "two camps": "those who wanted to win the war and those who wanted to lose it." The new reporters filed exciting, irreverent copy, which made it to the front pages; the veteran reporters' copy ended up buried way in back.

In place of these three myths, we should consider three much more plausible explanations: the first is the weak and ambivalent political leadership that American presidents brought to Vietnam; the second is the existence in the country of a vocal radical movement; and the third is the change that has occurred in the control of media organizations.
First, Presidents Kennedy and Johnson both wanted to avoid losing Vietnam without waging a major war in Asia. Kennedy tried to deny that Americans were fighting. A cable that his administration sent in 1962 instructed diplomats and soldiers never to imply to reporters any "all-out U.S. involvement." Other messages stressed that "this is not a U.S. war." When David Halberstam of the New York Times wrote stories criticizing the South Vietnamese government, Kennedy tried to have him fired

tomder55 answered on 01/09/07:

Where is Ernie Pyle when you need him ?


I read recently that of 500 reporters on the ground in Iraq only 6 are currently embedded with combat units . The rest spend their time in the Green Zone hugging a drink in the Mahogany Hill lounge .Disconnected from the reality they are free to create their own preconceived one .

Somewhere along the line elite members of the dinosaur media came to equate the mantra of having a right to dissent as becaming a duty to dissent . They wake up in the morning and genuflect to the image of Bob Woodward ,and try to imagine themselves the next hot shots who will bring down the President .

What happened to Patriotic reporters ? Maybe they have trouble getting hired by the fifth column fourth estate . I know of at least 2 patriotic reporters ; Michael Yon,and Bill Roggio who have been to Iraq a number of times .Most of their stories are related via blogs although Yon occacionally is featured in MSM publications .You won't see their by-line in the NY Slimes.

arcura rated this answer Excellent or Above Average Answer
Dark_Crow rated this answer Excellent or Above Average Answer

Question/Answer
Itsdb asked on 01/09/07 - U.S. strike in Somalia targets 3 terrorists

By Josh Meyer
Tribune Newspapers: Los Angeles Times
Published January 9, 2007

WASHINGTON -- A U.S. Air Force Special Operations gunship struck a location in southern Somalia on Monday where three Al Qaeda operatives suspected in the bombings of U.S. embassies in Africa more than eight years ago were believed to be hiding, a U.S. defense official said.

U.S. military and counterterrorism officials said they did not yet know whether any of the three fugitives had been killed.

"It's not clear what the outcome is at this point," said the counterterrorism official, who spoke on condition of anonymity because the operation was classified.

U.S. officials have secretly been negotiating with Somali clans believed to have sheltered the three men, hoping to obtain information about their locations. It could not be determined Monday whether the air strike was based on information provided by the clans.

The U.S. AC-130 gunship that carried out the strike was based in Djibouti, just north of Somalia. The strike was first reported by CBS News and independently confirmed by the Los Angeles Times.

~~~~~~~~~~~~~~~~~~~~~~~~~~~~~~~~~~~~~~~~~~~~~~~~~~~~~~~

ABC reported earlier this morning that Fazul Abdullah Mohammed was believed to have been killed. What do you suppose the reaction's going to be from the left for such a brazen, unprovoked attack?

On a side note the BBC, as astute as ever, offered this photo of the AC-130 with the following caption:


The heavily-armed AC-130 gunship can fly at night

Wow, what a technological breakthrough, the AC-130 can fly at night!

tomder55 answered on 01/09/07:

sure was a stroke of genious adding enough fire-power to a Herc transport plane to 'area saturation 'a small city with a 40 mm cannon firing 120+ rounds/minute, as well as a 105 mm cannon,;and 3 gatling guns . It's pseudo carpet bombing !It is definitly not one of them "win hearts and minds "weapons .

btw ,not only CAN it be used at night ;it is PRIMARILY and TYPICALLY ONLY used at night because since it was designed as a cargo/troop transport it doesn't have the best maneuverability.

ETWolverine rated this answer Excellent or Above Average Answer
Itsdb rated this answer Excellent or Above Average Answer

Question/Answer
ETWolverine asked on 01/09/07 - A Brief Story About Boxers

JPost.com

January 8, 2007; 6:58:37 AM
Israel Stories: The holy underwear
Posted by JEREMY CARDASH |

Tsfat, city of mysticism, home to some of the most famous minds in Jewish history. A city surrounded by the graves of the greatest halachic authorities from the time of the mishna to the present. And of course, the city with the most alternative English spellings in all the land of Israel.

Our hotel, just outside Sfat (yes I know I spelled it differently), which shall remain nameless, is one of the area's better hotels. It is known for its almost utopian atmosphere; no kids, good food and a spa.

So with great excitement we started unpacking. I pulled open a drawer ready to pile in my clothes and jumped back, for there staring at me was a large pair of red knickers. The sort of knickers that make men lie when their wives ask ‘does my bum look big in these’ only to be answered by ‘no dear’ while thinking ‘even a hippo would lose herself in those, or ‘maybe we could rent a marquee for our upcoming simcha’ or ‘stand at the end of the garden so I can see all of you’. I could go on all night and alienate myself from all female-kind, but whoever owned those knickers was married to a good liar.

A call was made to housekeeping. “We have a large pair of red knickers in our room”.

“That’s very nice sir, how can we help?”

“They don’t belong to us can you get someone to remove them.”

“Just leave them outside the door and the maid will take them”.

“No, you send the maid to get them; there is no way I am leaving these outside our room!”

“Someone will be along soon."

Thought – wasn’t there a book called The Red Tent?

Ten minutes went passed and nothing. I decided to march to reception.

“Excuse me; there is a large pair of red knickers living in one of our drawers”.

“That’s very nice sir, how can we help?”

“Can someone come now and remove them.”

Muffled voices came from behind the desk, then arguments.

“Look I very rarely get to stay in Safed (yeh, yeh) and I would like this dealt with before the end of the Shabbat.”

Suddenly, and as if from nowhere, a maid appeared.

“You know, Zfat is very holy place,” said the maid as we walked back to my room, “red wards off evil eye.”

Well what can you say to that?

The following week I was at the Kotel (Wailing Wall), where I was approached by a bearded man offering red strings in return for charity.

“Wear this and it will bring you luck and ward off the evil eye”.

My wife looked at me with that "don’t say anything" look in her eye.

But who could resist?

“Sorry,” I said, “I prefer boxers.”
--------------------
Elliot

tomder55 answered on 01/09/07:

lol for a while I thought this was going to end up being a fat joke . That spelling Zfat threw me off I guess ...or was it the red tent reference ?

ETWolverine rated this answer Excellent or Above Average Answer

Question/Answer
Dark_Crow asked on 01/08/07 - a record $100 billion. ......................

“Since the attacks of September 11th, 2001, Congress has approved about $500 billion for Iraq, Afghanistan and other terrorism-fighting efforts.
The White House is working on its largest-ever appeal for more war funds - a record $100 billion. It will be submitted along with Mr Bush's February 5th budget.”
ireland.com/newspaper/breaking/2007/0108/breaking4.htm

Here is what I suppose: Suppose that if all this money had been spent to alleviate hunger and suffering in, say sub-Africa. Would we still have Terrorism against the U.S. or, would friends of the Terrorist rise up against them?

tomder55 answered on 01/09/07:

For the record ;the Washington Compost reported at the end of last month that Bush has tripled direct humanitarian aid to Africa since taking office to more than $4 billion a year from $1.4 billion in 2001; and intends to bring the level up to $9 billion by 2010.Also under his tenure direct trade with Africa has doubled .

According to the aricle ;Bush launched his $1.2 billion malaria initiative in June 2005 with the goal of reducing malaria-related deaths in 15 African countries by 50 percent. The disease kills more than 1 million people a year, most of them African children under age 5.

The malaria program complements the president's largest global health initiative, the $15 billion, five-year plan known as the President's Emergency Plan for AIDS Relief (PEPFAR). Under the program, about 800,000 Africans are receiving drugs that enable them to live longer with the disease and help to prevent mother-to-child transmission of the virus.

"We launched one of the most important initiatives in American history, as far as I'm concerned, and that is the Emergency Plan for AIDS Relief," Bush said at a recent conference on malaria.



This has come due to increase pressure from his religious constituency to do more for the region .But ,I am sure advisers like Condi Rice has made him aware of the geopolitical importance of the region .

From a PR standpoint ,what is missing from the formula is a lip biting, teary eyed pronouncement from the President constantly reminding us of his initiatives ...like the previous blowhard who occupied the White House was fond to do. So instead we get "Bush doesn't care about black people." by idiots like Kanye West

[But when you give to the needy, do not let your left hand know what your right hand is doing, so that your giving may be in secret. And your Father who sees in secret will reward you. Matthew 6:3-4]

Of course ,with all foreign assistance my biggest concern is that it probably will go down a sink hole due to the corruption in the governments we are dealing with and the inherent inefficieny in bureaucracies .$9 billions spent efficiently by well run private charitable organizations will do more than $100 billion handed over to tin-pot dictators .Without accountability too often the money is wasted . From 1970 to 2000 over $400 BILLION in aid has been sent to Africa. We have exactly NOTHING to show for it. Nor do the Africans.My own opinion on poverty is that the best way to eradicate it is to help people realize the benefits of a free market economy.

Regarding terrorism ,the article points out that Although some activists criticize Bush for not doing more to end the ongoing genocide in the Darfur region of Sudan, others credit him for playing a role in ending deadly conflicts in Liberia, the Congo and other parts of Sudan.

Eradicating poverty is a moral goal but Idoubt that it will be a decisive factor in reducing terrorism .I have not been convinced that there is a relationship between poverty and terrorism .Perhaps it is easier to recruit a foot soldier if you wave cold hard cash in their face . That was Saddam's formula in Palestine.15 of the 19 hijackers involved in the Sept. 11 attacks were of middle class or wealthier families from one of the wealthiest countries in the world (Saudi Arabia)their leader, Osama Bin Laden, a billionaire. Poverty does contribute to political instability ,but I rarely hear poverty mentioned in the various manifestos of terrorist organizations . Usually they espouse a specific political ideology or goal instead .Nasra Hassan, who wrote an essay based on encounters with extremist youth and their families in "An Arsenal of Believers," in the New Yorker in 2001. offers the 'experience of indignities suffered, of political humiliation, and of desperation borne out of a sense of futility,' as possible explanations of why some people turn to terror. In other words, it is politics, not economics.

A John F. Kennedy School of Government researcher has cast doubt on the widely held belief that terrorism stems from poverty, finding instead that terrorist violence is related to a nation's level of political freedom. Associate Professor of Public Policy Alberto Abadie concluded that "In the past, we heard people refer to the strong link between terrorism and poverty, but in fact when you look at the data, it's not there. This is true not only for events of international terrorism, as previous studies have shown, but perhaps more surprisingly also for the overall level of terrorism, both of domestic and of foreign origin" He instead finds that terrorism is more influenced by levels of political freedom in a country ;although he admits that the most repressive regimes have ways of dealing with internal terrorism that we would find distasteful.He also finds that countries where terrorists can find a geographic cover are more vulnerable .

According to A Hoover Institute paper A study by scholars Alan Krueger and Jitka Maleckova reached the conclusion that the terrorists are not poor people and do not come from poor societies. And ;a study of India has demonstrated that terrorism in the subcontinent has occurred in the most prosperous (Punjab) and most egalitarian (Kashmir, with a poverty ratio of 3.5 compared with the national average of 26 percent) regions and that, on the other hand, the poorest regions such as North Bihar have been free of terrorism. In the Arab countries (such as Egypt and Saudi Arabia, but also in North Africa), the terrorists originated not in the poorest and most neglected districts but hailed from places with concentrations of radical preachers. The backwardness, if any, was intellectual and cultural — not economic and social.


Dark_Crow rated this answer Excellent or Above Average Answer

Question/Answer
HANK1 asked on 01/07/07 - DEMOCRACY:



I'm beginning to wonder if a Democracy is the best form of government. Too much 'hurray for me and to hell with you' going on. Our Constitution was set up to have State and Local governments take care of 95% of our business, leaving the federal government with 5%. I wonder what would happen if we didn't have the Bill of Rights.

What's your take on this?

HANK

tomder55 answered on 01/08/07:

I would like to remind you that the confederates lost the civil war. Also in the debates prior to adopting the constitution ,the federalists clearly beat the anti-federalists . I would also remind you that such states rights advocates as Thomas Jefferson ,once ascended to the Presidency himself ;did as much as any President since to strengthen the role and powers of the Federal govenrment .

There is no doubt that the Bill of Rights were necessary add ons to the Constitution . The country would not have signed on to the constitution without a guarantee that they would be adopted ;nor would our form of gvt. survived without them.

The modern day definition of Democracies can be summed up this way ....they are political systems whose leaders are elected in competitive multi-party and multi-candidate processes in which opposition parties have a legitimate chance of attaining power or participating in power. This differs from the classic definition of democracy in many respects.

We are not a pure democracy ;we are a Federal Republic (but democratic as the modern definition defines it ). We are a Representative form of governemnt .We elect representatives to serve us and to represent our interests in Washington . It would be absurd to have 250 million people in a town hall setting casting individual votes on every decision made .As Alexander Hamilton said :"We are a Republican Government. Real liberty is never found in despotism or in the extremes of Democracy... It has been observed that a pure democracy if it were practicable would be the most perfect government. Experience has proved that no position is more false than this. The ancient democracies in which the people themselves deliberated never possessed one good feature of government. Their very character was tyranny; their figure deformity."

I happen to believe that our Representative form of gvt. is the best that humans with all our flaws has ever invented .... or as Churchill was fond of saying :"Democracy is the worst form of government except for all those others that have been tried."

There are those like Clete who do not trust human nature and they have a valid point. Our system demands vigilance of the electorate . But Clete argues that a benevelent dictatorship run by the right leader (and I assume he is refering to the Messiah who will reign on earth )is the best form of gvt. When that utopian reality comes to pass I will certainly sign on. But until that time ;I prefer to have my leaders accountable to the electorate who put them in their position of leadership .


HANK1 rated this answer Excellent or Above Average Answer
Itsdb rated this answer Excellent or Above Average Answer
paraclete rated this answer Excellent or Above Average Answer

Question/Answer
Choux... asked on 01/08/07 - CONSTITUTIONAL CRISIS LOOMS

by Paul Begala:

"On October 19 I debated Bob Novak at Emory University. The topic was "Civil Liberties in a Time of War." I kicked his ass, but that's not why I mention it. In the debate I predicted that, after the Democrats captured the Congress, Pres. Bush would provoke a Constitutional crisis by refusing to comply with congressional subpoenas.

Pres. Bush, I predicted, will effectively tell Speaker Pelosi, "You send the Capitol Police to enforce your subpoena. I'll send the 82d Airborne to resist them. Let's meet on the Mall and see who wins."

Novak said I was crazy. It's beginning to look like I was right.

The only reason George W. Bush would turn loose of White House Counsel Harriett Miers - who gazes upon our president with an adoration and veneration bordering on idolatry - is because he wants a war-time consigliere.

In a way that might make Harry F. Byrd proud, our president is about to embark on a policy of massive resistance. He will instruct his lawyers to delay, deny and refuse to comply with any effort by Congress to get to the bottom of official corruption - especially as the billions squandered or stolen in Mr. Bush's war. He'll try to run out the clock, then take his chances with his hand-picked right-wing judiciary. (Keep in mind the DC Circuit Court of Appeals, through which this dispute would flow, includes such Bush appointees as Brett Kavanaugh, a Ken Starr protégé whose work in the Bush White House was described by Henry Waxman as promoting "an imperial presidency." And the Supreme Court has such presidential suck-ups as John Roberts and Sam Alito.)

Thank God the American people - and Nancy Pelosi -- have given the responsibility of oversight we have constitutional heroes like Waxman and John Dingell. They are fair and tough. But even in their combined 83 years experience in Congress they have not seen a crowd that has more contempt for the Constitution than the Bush-Cheney team. I would not be surprised to learn that, in anticipation of receiving congressional subpoenas, the Bushies were having shredding parties that would make Ollie North and Fawn Hall blush.

Let's all watch to see who Bush appoints to replace Ms. Miers. If he chooses someone like David Addington, Vice President Cheney's chief counsel, we'll know Mr. Bush intends to shred yet another Article of the Constitution."

~~~~~~~~~~~~~~~~~~~~~~~~~~~~~~~~~~~~~~~~~~~~~~~~

Do you think that the Constitutional Crisis that looms on the horizon will take place or will Bush cooperate with the investigations of his malfeasance?

tomder55 answered on 01/08/07:

I believe there is a good chance that the President will invoke executive privilege for some subpoenas as well he should. This will not cause a Constitutional crisis ;that is just demogogery.

Executive privilege has been exercised many times in our life time by all the Presidents ,as far back as Washington .The Supreme Court has ruled over and over that when the President or the President's top aides communicate with one another about matters of national policy, they are presumptively privileged in those communications.The White House can't be made subject to the Congress or to the courts by having inquiries that would make the Prsident unable to meet with his top advisers. He couldn't conduct policy-making in the White House if every top aide to the President knew that his or her communications with the President or with each other could be revealed to the whole world at the drop of a hat. Nobody could make policy under that kind of circumstance.

But ,there have been times when the courts have ruled against executive privilege . Paul Begala ,being an aide to Bill Clinton should know this as well as anyone .Clinton was denied it when the courts ruled that Kenneth Starr's need to collect evidence in his obstruction of justice probe outweighed Clinton's interest in preserving the confidentiality of White House discussions. Did this provoke a constitutional crisis ? The difference with this is that in Clinton's case he was invoking it in a criminal investigation. In Bush's case ;it will be over matters of national security most likely and Bush will have greater justification to use it.

Now ;the Dems are talking about getting records and testimony from Dick Cheney over his energy task force meetings . In that case the White House has less of a case if it tries to invoke Executive Privilege . Clinton tried to use it when Congress was seeking information about Hillarycare hearings which were also conducted in secrecy and the courts ruled against the Clintons then (see the 1993 decision of the U.S. Court of Appeals for the D.C. Circuit in Association of American Physicians and Surgeons, Inc. v. Hillary Clinton. )and I suspect they would simularily rule against Cheney on the same principle .Although Hillary was not on the same level as the VP ;it can be argued that Cheney constitionally has no executive status in the Constitution beyond sitting around and waiting for the President to drop dead. His only other defined role is Legislative .

Choux... rated this answer Excellent or Above Average Answer
Itsdb rated this answer Excellent or Above Average Answer

Question/Answer
Choux... asked on 01/07/07 - "greenwashing" OIL LIARS ARE FINISHED!!!!

Web Exclusive
By Jerry Adler
Newsweek
Updated: 6:10 p.m. CT Jan 4, 2007

Jan. 4, 2007 - For more than three decades, the tobacco industry carried on a campaign of disinformation intended to mislead Americans about the health risks of smoking—a strategy that has been dubbed “manufacturing uncertainty” in the minds of consumers. And ever since global warming emerged as an environmental threat, there has been a well-funded public campaign to cast doubt on the scientific consensus about the danger of global warming and its source in fossil-fuel combustion. A report this week by the Union of Concerned Scientists finds a parallel between the efforts to whitewash tobacco and “greenwash” oil—and points the finger of responsibility at the world’s largest corporation, ExxonMobil.


Under its former chairman and CEO, Lee Raymond, who retired in 2005 as one of the best-paid corporate executives in history, ExxonMobil was well known for its hostility to government regulations on emissions of carbon dioxide. But, according to the report, the op-eds and position papers were only the visible tip of Exxon’s effort to fund a small group of researchers and an overlapping network of think tanks that could be relied on to spread the message that global warming was nothing to worry about—or at least, nothing the government could or should do anything about. Their frequently repeated call for “sound science” on global warming echoes the tobacco industry’s endless demand for more research on whether cigarettes really, truly, unquestionably cause cancer.

Of course, cigarette companies weren’t concerned just about future sales, but the billions of dollars in compensation they eventually had to … umm … cough up. ExxonMobil’s motivation, presumably, is to protect a fantastically lucrative market: its 2005 profits of $36 billion made it the most profitable corporation in history. But that very wealth puts them in a position both to shape and eventually dominate the postcarbon energy world, if they choose to do so. Ironically, as the report points out, the company and its shareholders will suffer if it gets left behind in the transition to less polluting forms of energy.

For its part, ExxonMobil—after promulgating, and then withdrawing 20 minutes later, a statement that called the report an “attempt to smear our name and confuse the discussion”—wants you to know that it now accepts some responsibility for global warming. Specifically, and in boldface, it admitted that “It is clear today that greenhouse gas emissions are one of the factors that contribute to climate change, and that the use of fossil fuels is a major source of these emissions.” That would seem, on the face of it, to contradict the assertions of some of its favored researchers in the ever-shrinking coterie of global-warming skeptics. The question, of course, is what specific policies ExxonMobil is willing to accept to curb those emissions. With a new Congress taking office, climate change is likely to be a much more salient issue this year than it has been for the last six—so ExxonMobil will have the chance to show if it means what it’s saying now."

~~~~~~~~~~~~~~~~~~~~~~~~~~~~~~~~~~~~~~~~~~~~~~

tomder55 answered on 01/08/07:

I would say off the bat that the energy companies like Exxon-Mobile have taken the lead in research of alternative fuel sources that are more environmentally friendly.

Of course little tid-bits of facts aren't good enough for the bullies who would rather wage a campaign of intimidation against the companies that prime the pump of the American economy. Last month Senators sent a letter to ExxonMobil CEO Rex Tillerson warning him against speaking anything other than the pc party lie on the subject of global warming .

The text of the letter authored by the idiots John D.(Jay) Rockefeller IV and Olympia Snowe (no amt. of global warming can melt her)is here .

Jay Rockefeller's personal wealth is derived from his families exploration of oil (great-grandson of oil tycoon John D. Rockefeller founder of Standard Oil that branched off to become Exxon-Mobile).Jay comes to us from coal-producing West Virginia, where people know something about carbon emissions .

The Senator wouldn't dare risk an ethics inquiry by threatening specific consequences if Tillerson declined this 'offer he can't refuse'. But in case he doesn't understand his company's jeopardy, the letter adds that "ExxonMobil and its partners in denial have manufactured controversy, sown doubt, and impeded progress with strategies all-too reminiscent of those used by the tobacco industry for so many years."

The Senator believes global warming is a fact, and therefore all debate about the issue must stop and ExxonMobil should "end its dangerous support of the [global warming] 'deniers.' " Not only that, the company "should repudiate its climate change denial campaign and make public its funding history." And in extra penance for being "one of the world's largest carbon emitters," Exxon should spend that money on "global remediation efforts."

They are so afraid of debate that they want Exxon to stop financing a minority band of scientists who are dissenters and can barely get their name or positions heard .But environmentalists have been wrong about almost every other apocalyptic claim they've made since I've been alive be it global famine, overpopulation, natural resource exhaustion, the evils of pesticides, global cooling,etc. It is useful to have a heathy debate by sceptics .Where is the harm in presenting alternative opinions based on sound research ? Science too often gets locked in a dogmatic acceptance of "facts " and too frequently those facts are just plain wrong.

But even worse is this inquistion by the pc. global warming crowd. This letter by the Senators is just plain intimidation. Has Jay Rockefeller become the next Tomas Torquemada ? To Exxon's credit ,it has indicated that it will resist this blatant intimidation and will continue to fund free-market science and reseach .

I for one wish that Rockefeller would in the interest of full disclosure be up front about how his policies would affect the living standard of Americans if implemented . But that aint happening because that would take integrity .





Choux... rated this answer Excellent or Above Average Answer
ETWolverine rated this answer Excellent or Above Average Answer

Question/Answer
HANK1 asked on 01/07/07 - PELOSI:



Pelosi just said on the radio that there may not be any funding for our troops if Bush sends 20,000 to 30,000 troops to Iraq. I guess she wants our guys and gals to go over there and fight with scraps. All she's doing is trying to please the public without thinking about the safety of those doing the fighting. DAMN -- I HATE POLITICIANS! Those idiots in Washington are just like the idiots in Hollywood. They're out of touch with REALITY! A better means to an end -- a cardinal rule needed to defeat competition on all levels.

HANK

tomder55 answered on 01/08/07:

this from the same folks who were crying about body armor and up-armored hummers.

Reality check; the Dems probably could attempt a defunding at the House level by stalling appropriations bills . This would be a huge political mistake and I think they know it. But they could bow to the pressure from the fringe wack jobs like DailyKos.

I found it amazing that the Dem. leadership had a press conference last week and Cindy Sheehad and her group of professional protesters were chanting slogans . The Democrat Leadership retreated to a closed room and left the mikes to the protesters. I would've had them evicted from the Capitol .

y best guess is the Dems will hold hearings but basically they will leave the war effort in the hands of the White House. They do not want to be " tainted " by the war going into the 2008 election cycle. Their attitiude will be "Bush started it ,let Bush finish it." The Dems will be content to sit on the sidelines and jeer from the peanut gallery . They really do not want the responsibilty of making the tough choices in the war against jihadistan.

They will be brutal in other means like tax increases and proposing rediculous increases in domestic spending . They will say "how can we fund these needed social spending increases and balance the budget if we are strapped with paying for the war unless we increase taxes ? " They will call that "paygo"pay as you go.

HANK1 rated this answer Excellent or Above Average Answer

Question/Answer
arcura asked on 01/06/07 - What do you think would be fair about this?

Frontier family fights to return home
Eviction erases a vestige of Montana's homestead history
By Michael Martinez
Chicago Tribune national correspondent

January 1, 2007

HIGHLAND MOUNTAINS, Mont. -- Since 1878, four generations of Vadis and Howard Stratton's family lived as homesteaders on this mountain, much like TV's "The Waltons," the clan says.

The family made electricity with an 1893 waterwheel, powered by a creek shooting downhill through a homemade pipe. Vadis Stratton and her late husband, Howard, who both didn't drink, smoke or cuss, raised seven kids here. Their home was a 1912 log cabin with a wood-burning stove near the original shed-sized 1878 dwelling. They cut timber for nearby Butte's mines and even prospected for gold. Right through last year, Vadis used a wringer washer.

It all made for a 19th Century homestead working until the present day.

Chester Arthur was president when the government invited covered-wagon pioneers like the Strattons' ancestors to settle the summits and valleys on the Continental Divide.

Now the government has ordered the Strattons to leave, evicting the last of the frontier families that had populated this mountain and its namesake hamlet of Highland City, a nearly vanished ghost town founded in the 1860s gold rush.

After winning a court ruling in 2005, the U.S. Forest Service declared 81-year-old Vadis Stratton and her son David, 42, illegal squatters in the Beaverhead-Deerlodge National Forest and forced the matriarch in August to move into a senior citizen home in Butte, an hour's drive away.

Stratton, a great-great-grandmother whose brown hair is without gray after nearly a lifetime in the mountains, says the government has wronged her.

"I feel like I was railroaded out of the place," Stratton said last month in her room at the Continental Gardens home in Butte. Though she had temporarily resided in Butte during her kids' schooling, it's the first time she hasn't lived full time on the mountain in 26 years.

Now HUD subsidizes her rent at $3,528 a year. She pays the remainder, $204 a month, from her $545 monthly Social Security check. Her son David is living in a trailer while he fixes up an old Butte house.



94-year-old paperwork at issue

At the heart of the Forest Service's case is its assertion that the Strattons' claimed 140-acre spread, with its historic 10-acre mill site, was never officially homesteaded and that two previous government authorizations to occupy the land, in the 1930s and ྂs, had expired.

Stratton's tale has become a cause celebre in Montana, a long-running feud between forest rangers and a family with a place in state history. She is asking to be able to die on the mountain, where her husband spent his last days before he died in 1994.

The family, which moved from California, claims it applied for a homestead in 1912 and in 1913. The 1912 application was withdrawn by the family because the local forest ranger said the land was reserved for an administrative center, which was never built, according to the family and the Forest Service. The service says the 1912 application by Fred Stratton, Howard's father, states that he had "not settled on land," which family members dispute.

The subsequent 1913 application, the family says, was submitted but never processed by the local Forest Service office--which a spokesman disputed.

"I don't know about that," Forest Service spokesman Jack de Golia said in an e-mail about the 1913 application.

The Forest Service granted the family permission to live on the mountain and operate a sawmill in 1939 and then two ore mills in 1979, but by 1980, milling operations ceased, terminating the family's tenancy on the mountain, according to the government.

By 2000, the Forest Service began informing Vadis and David Stratton that they would have to vacate the land.

In December 2005, U.S. District Court Judge Sam Haddon ruled that "the United States is entitled to possession of the property" and that the Forest Service "properly terminated the Strattons' occupancy."

Stratton and two of her sons, David and Mark, who represented themselves in court, are appealing, contending they were unlawfully denied a jury trial.

The government has owned this mountain land since the Louisiana Purchase of 1803 and established forest reserves here in the 1890s and early 1900s. In 1905, those reserves came under Forest Service control with the agency's creation, de Golia said.

Stratton's daughter, Alice Lester, 57, of Camarillo, Calif., said her family and forest rangers long enjoyed a gentlemen's agreement about the land.



History laced with gold

When Howard Stratton told rangers about submitting payment for another homestead application, "the Forest Service said, `Save your money, Howard. Nobody is going to ask you to leave here. Everybody knows the Strattons,'" Lester said.

"It was a gentlemen's handshake, and all that went by the wayside when these young people [with the Forest Service] came in, and some hot-doggers and young guns said, `We're gonna straighten things out and change the way things are.' And the gentlemen's handshake no longer existed," Lester said.

The family is part of Montana's gold mining history. In 1989, Stratton's husband and son Mark found a disk-shaped gold nugget weighing 27.495 troy ounces and big enough to cover the palm of an adult's hand.

Called the Highland Centennial Gold Nugget, it's believed to be the seventh-largest nugget ever found in Montana and the largest still in existence; it now sits behind glass in an alarm-rigged safe at the Montana Tech Mineral Museum in Butte.

Second-generation homesteader Fred Stratton found another renowned nugget: it weighed only an ounce but was almost a perfect miniature of the Venus de Milo statue, Vadis Stratton said. It was later displayed at the 1933-34 World's Fair in Chicago, family members said.

Over the years, the Stratton homestead served as a sanctuary for hikers and motorists lost in the woods--including rangers whose vehicles got stuck on the 7-mile dirt road that the family built, members said.

The family and Forest Service now accuse each other of acting in bad faith in seeking an arrangement for Vadis Stratton to continue living on the mountain.

"I believe the Forest Service has bent over backwards trying to accommodate the Strattons, only to be stonewalled," then-Forest Supervisor Thomas Reilly wrote in 2003 to the Butte-based Montana Standard newspaper. "The public interest is not served when private parties occupy public land without legal authorization."

The Strattons said they offered three mountain parcels they owned--and then all three parcels together--in exchange for keeping their historic spread. At one point, a Butte real estate agent even offered prime land in the Elkhorn Mountains on behalf of the family, Lester said. But the service rejected all the offers, she said.

All the Forest Service wanted, Stratton charged, was for her to leave--without anything immediately offered in return.

"Their idea of bending over backwards was forcing me to sign a quitclaim deed giving them everything," Stratton said.

For now, the 1913 log cabin is in shambles after vandals broke windows and tossed about clothes and magazines the family left behind. Lester blames the Forest Service for failing to care for the structure.

But de Golia said the appeal leaves unclear whether the agency is now the custodian of the structures.

"It's in limbo. It's ambiguous," de Golia said, adding that vandalism is a problem throughout the forest.

tomder55 answered on 01/07/07:

nothing fair about it at all. but that has been a negative about our park system for some time now. In NY ,whole communities like Doodletown were displaced by the forced purchase of property to make Harriman State Park .

arcura rated this answer Excellent or Above Average Answer

Question/Answer
arcura asked on 01/06/07 - Bush is soon to tell us what his new plan for Iraq is.

What kind of mess do you think he'll com up with this time?
Have you noticed that the messes Bush leaves where ever he wanders makes it look like he never was house broke?

tomder55 answered on 01/07/07:

I dispute the premise of the question . I think that a policy shift is needed but that is typically true of any ongoing policy .

I will say what I know and speculate on the rest.

It is clear that there is an ongoing shake up at the cabinet level of the White House and the General staff at the Pentagon and on the field . Rumsfeld is gone at SecDef ;replaced by Robert M. Gates .
John Negroponte, who has held numerous posts in the Bush White House, will step down as Director of National Intelligence to become the Deputy Secretary of State. This post has been vacant since Robert Zoellick resigned in July. According to some Washington insiders, Negroponte has not been effective at tackling the immense intelligence bureaucracy(DNI is the rediculous super intelligence monster that combined all the agencies of intel under one directorship at the suggestion of the 9-11 Commission .As was predictiable;this agency is too big and instead of making for swifter and more coordinated decisions it has become a joke), and he doesn’t want to be around in the spring when Senate Democrats set about legislating tighter congressional oversight of the terrorist-surveillance program. Retired Vice Adm. Mike McConnell, a career intelligence officer, will be nominated to replace Negroponte as DNI.

In Iraq, more changes are afoot. Army Lt. Gen. David Petraeus, lauded for his previous successes in training Iraqi troops, will replace Gen. George W. Casey, Jr., as ground commander in Iraq. The overall commander for U.S. military operations in the Middle East, Gen. John Abizaid, will be replaced by Navy Adm. William Fallon. Making the changeover complete, Ryan Crocker, currently the U.S. envoy to Pakistan, will replace Zalmay Khalilzad as ambassador to Iraq. Like the others, Khalilzad isn’t exactly being pushed out the door, as he will be nominated as U.S. ambassador to the United Nations, a post vacated by the stalwart but congressionally unconfirmed John Bolton.

With Saddam finally permanently hung out to dry, the Bush administration—so often accused of intransigence over the question of Iraq—is preparing to raise troop levels temporarily and provide economic aid to improve stability on the ground. The administration is considering a troop increase of 17,000 to 20,000, with most of the additional troops stationed in and around Baghdad. The mission has to do with using them in a security role to quell violence in Baghdad and the surrounding area.

I am not one who thinks that the absence of violence in Baghdad is a measure of our success. It is good fodder for the press and the anti-war crowd ,but ultimately irrelevent to our strategic goal so long as violence doesn't lead to the collapse of the gvt. or the economy . Latest reports out of Iraq says that the economy is improving every day .

The government there needs a shake up and a leader more committed to the state rather than his own sectarian interests .It is my hope that the surge will not soley be in support of the Maliki government . He has been a major disappointment in my book but yesterday may have been a turning point.
He said that Iraq's armed forces are set for an assault on Baghdad to take out militias and rogue security forces.He said that aided by multinational troops, the Iraqi forces "will hunt down all outlaws regardless of their sectarian and political affiliations".

I hope we spare more than a few thousand troops to take on the Mahdi Army of Muqtada al-Sadr ;and reserve at least one bullet for his forehead .So before any new offensive is initiated Maliki must understand that if al-Sadr gets in our cross hairs this time we will pull the trigger. And that goes for any mission . No more "letting Fallujah off the hook" ... All operations conducted to a conclusion.Our "goal" is to help Iraq stand on its own to be able to defend and sustain itself, that cannot be achieved by allowing fear of militias to determine how situations are handled.I do not know what was spoken about in the conversation between Maliki and President Bush for over two hours, but I can imagine that Maliki must have been made to understand that he has taken too long in his political game playing with groups such the Mahdi Army and his own militia.

The economic package would focus on relieving Iraq’s high rate of unemployment, providing short-term jobs for Iraqis and micro loans to Iraqi entrepreneurs. In addition, the Treasury Department is reviewing Iraq’s now-dormant state-owned industries, with the goal of reopening those deemed economically viable. Between an increased troop presence and increased economic opportunity, the Bush team hopes to drain support from Iraq’s militants, bringing the country sustainable stability sooner rather than later.


arcura rated this answer Excellent or Above Average Answer
HANK1 rated this answer Excellent or Above Average Answer

Question/Answer
HANK1 asked on 01/05/07 - WHAT'S THIS?



TAKE BACK AMERICA 2007

JUNE 18-20th, 2007 - WASHINGTON, DC

MARK YOUR CALENDARS - PLAN TO ATTEND THE YEAR'S LARGEST AND MOST IMPORTANT GATHERING OF PROGRESSIVE LEADERS, THINKERS AND ACTIVISTS.

Dear Supporter,

Last year at Take Back America 2006, we came together to lay plans and build alliances to deliver the big progressive victories of 2006. And after the Conference, we delivered! Congress is under new management, populist champions are commanding the spotlight, and the once-feared conservative movement has been shaken.

But have we finished the job of Taking Back America? Hardly. The hard work has just begun. We still lack affordable health care and education. Our nation remains "addicted to oil". Poverty continues to be pervasive. Our troops are mired in a civil war abroad that's destabilizing the globe. Conservatives haven't given up. And they want to keep the White House in 2008.

We mobilized and moved mountains last year. Let's do it again. Please save the date, so that you can be sure to join us for Take Back America 2007: June 18th - 20th at Washington DC's Washington Hilton Hotel.

Take Back America 2007 promises to provide an exhilarating forum to engage with progressive leaders, interact with progressive media, and organize with other passionate people committed to building on our successes and broadening the mandate for change. It will feature skills trainings, workshops -- and strategy sessions with the folks who defied the prognosticators and nay-sayers in 2006, so we'll have the tools to bring about real reform in 2007.

And, with several presidential candidates expected to participate, Take Back America 2007 will give you an early chance to assess who best can carry the progressive torch in 2008.

We hope you can make it, and look forward to seeing you in Washington this June 18th-20th! Be sure to save the date, and stay tuned for an announcement when our early-bird registration opens.

Regards,



Roger Hickey, Co-Director
Campaign for America's Future

****************

Did you participate in 2006?

HANK

tomder55 answered on 01/06/07:

Roger Hickey is a radical lefty . Take Back America is an organization which sole purpose is to assist the Democrats take over power in Washington.

HANK1 rated this answer Excellent or Above Average Answer

Question/Answer
Choux... asked on 01/05/07 - DEMS DRAFT BILL TO EXPAND ANTITERROR EFFORTS

Democrats Draft Bill to Expand Antiterror Efforts, Protect Civil Liberties

"House Democrats have reportedly prepared sweeping legislation, to be considered as early as next week, that would authorize billions of additional dollars to screen all cargo bound for the U.S., purchase new screening technology for airline passengers, and dramatically expand efforts to secure nuclear materials around the world. The bill also features provisions intended to ensure that new and existing initiatives do not infringe on Americans’ civil liberties".

~~~~~~~~~~~~~~~~~~~~~~~~~~~~~~~~~~~~~~~~~~~~~~~~~~

tomder55 answered on 01/06/07:

it is absolutely goofy to suggest that all cargo entering the country will be screened . This from the same folks who think it is perfectly acceptable for people to walk across the border unprocessed. Is Pelosi's 1st 100 hrs over yet ? It seems like 100 years already.

From what I understand all they did was take a list of all the 9-11 commission's recomendations and added them to the bill ;whether the suggestions were good ideas or not .

How much extra do you want to pay for the goods you purchase ?This sounds like a protectionism bill masked in security language.

Choux... rated this answer Poor or Incomplete Answer
ETWolverine rated this answer Excellent or Above Average Answer
labman rated this answer Excellent or Above Average Answer

Question/Answer
Choux... asked on 01/05/07 - IRAQ WAR LOST..WHY PUT 20,000 MORE IN HARM'S WAY?????

Senator Joseph Biden, chairman of the Senate Foreign Relations Committee, said yesterday that he believes top officials in the Bush administration have privately concluded they have lost Iraq and are simply trying to postpone disaster so the next president will "be the guy landing helicopters inside the Green Zone, taking people off the roof," in a chaotic withdrawal reminiscent of Vietnam.

"I have reached the tentative conclusion that a significant portion of this administration, maybe even including the vice president, believes Iraq is lost," Biden said. "They have no answer to deal with how badly they have screwed it up. I am not being facetious now. Therefore, the best thing to do is keep it from totally collapsing on your watch and hand it off to the next guy -- literally, not figuratively."

Biden gave the comments in an interview as he outlined an ambitious agenda for the committee, including holding four weeks of hearings focused on every aspect of U.S. policy in Iraq. The hearings will call top political, economic and intelligence experts; foreign diplomats; and former and current senior U.S. officials to examine the situation in Iraq and possible plans for dealing with it. Secretary of State Condoleezza Rice will probably testify next Thursday to defend the president's new plan, but at least eight other plans will be examined over several sessions of the committee.

Other witnesses invited for at least 10 days of hearings include former national security advisers and secretaries of state, including Brent Scowcroft, Samuel R. "Sandy" Berger, Zbigniew Brzezinski, Henry A. Kissinger, Madeleine K. Albright and George P. Shultz.

Biden expressed opposition to the president's plan for a "surge" of additional U.S. troops and said he has grave doubts about whether the Iraqi government has the will or the capacity to help implement a new approach. He said he hopes to use the hearings to "illuminate the alternatives available to this president" and to provide a platform for influencing Americans, especially Republican lawmakers.

"There is nothing a United States Senate can do to stop a president from conducting his war," Biden said. "The only thing that is going to change the president's mind, if he continues on a course that is counterproductive, is having his party walk away from his position."

~~~~~~~~~~~~~~~~~~~~~~~~~~~~~~~~~~~~~~~~~~~~~
Not to miss hearings on Bush's War of Adventurism on Iraq begin soon.

Why put 20,000 more US soldiers in harm's way?

tomder55 answered on 01/06/07:

this from the guy who's partition plan even the Iraq Surrender Group scoffed at . I look forward to his hearings.

He is wrong of course . WE know from history that Congress defunded Vietnam and there was not a damn thing that President Ford could do about it . In Ford's brief term ,he attempted vetos of Congress 66 times (so much for the lie the Dems and the press were spewing this week about the times being better because Ford was a moderate who they could work with ).But still he could not prevent Congress from ending the war.

It is evident that the Dems plan to stick it to the country with an aggressive domestic agenda ,and will be content to sit on the sidelines /holding hearings on Iraq ...but not doing anything or submitting an alternate plan ; for as long as Cindy Sheehad and the Kossaks permit them to do so.

Why add aditional troops ? because with an effective strategy and different rules of engagement they can succeed .

Choux... rated this answer Average Answer

Question/Answer
Itsdb asked on 01/05/07 - Pizza for Progressives

The Kossacks have apparently anointed John Edwards as their candidate for 2008 and have thus begun a "Pizza for Progressives: the Edwards Edition" fundraiser. This is where you are asked to forego your pizza/espresso/beer and hot dog for the week and donate the amount it would cost instead. I know everyone has their own idea of what a pizza should be but I gotta ask, would it be too hard to pass up a pizza that looks like this?



Looks to me like Kos' dog relieved himself on the pizza. So forgetting the fundraiser, how about a pizza war? This is more along the lines of my taste in pizza:


California Pizza Kitchen's 'Sicilian'

"A spicy marinara sauce with sweet Italian sausage, spicy Capicola ham, julienne salami, Fontina, Mozzarella and Parmesan cheeses. Topped with fresh oregano and basil."

A wonderful light, thin, crispy crust and flavors that send my taste buds to heaven. Something I imagine Reagan might have liked. And you? Come on you New Yorkers, I know this is a sensitive issue to you guys.

tomder55 answered on 01/05/07:


And here I would've thought that a campaign gimic for Edwards would've been to sacrifice the Wendy's Chili a side of fries and a medium Frosty.
He must love NY pizza . There's enough oil in every pie to satisfy a slip and fall lawyer .But as I remeber from 2004 ; he and Kerry are fond of shrimp vindallo, grilled diver sea scallops, and prosciutto pizza .

The pie you posted looks awefully good .

I used to swear by NY pie until I went to Kansas for a few years . Pizza Hut used to have those matza crust pies back then and I spent many a dinner at the all you can eat buffets they'd have . A couple of pitchers of cheap diluted brew to wash it down and my evening was complete. I don't know how it is in Tx. but in NY Pizza Hut has thickened their crusts and it just aint the same .

Usually when I buy pies these days it's simple with either 'shrooms or pepperoni. (and keep the papertowels handy to wipe up the excess oil .)

Itsdb rated this answer Excellent or Above Average Answer

Question/Answer
Itsdb asked on 01/04/07 - WONDERFUL HISTORIC DAY

Since user ended previous question without opportunity to comment (I applaud her openmindeness and willingness to discuss other views), allow me to congratulate Speaker Pelosi on her historic achievement. I wish her great success - and a speedy return home to care for her grandchildren.

tomder55 answered on 01/05/07:

I have vowed not to critque her term until the 1st 100 hours are over (but she has got to lose that nervous girly giggle).

Bob Novak had an interesting column yesterday that touched on an issue we raised during the campaign;the Trojan Horse Democrats . They will soon emerge and show their true face.

ETWolverine rated this answer Excellent or Above Average Answer
Itsdb rated this answer Excellent or Above Average Answer

Question/Answer
Itsdb asked on 01/03/07 - Photo Evidence of US Troops Commiting Atrocities

Received this shocking message in my email today...


Soldiers Force Children to See Saw Until They Give Up Information!


Soldier Forces Child to Hang By Fingertips - Just for Fun


Soldiers Force Children to Do Their Dirty Work


The Tickle Torture is Unbearable


Child Bites Soldier in Self Defense


Rations Shortage; Soldiers Begin Eating Children


Soldiers Lack Body Armor; Forced to Use Childen as Shields


Soldiers Vent Anger by Throwing Children Around


Iraqi Children Thankful Soldiers Didn't Open Fire On Soccer Game

tomder55 answered on 01/04/07:







Itsdb rated this answer Excellent or Above Average Answer

Question/Answer
Itsdb asked on 01/03/07 - Gay sheep experiments outrage campaigners

2nd January 2007 11:24am
Tony Grew

Scientists and gay rights activists have criticised experiments being conducted in the US that aim to turn gay sheep straight.

Oregon State University is conducting research into the hormonal balance in the brains of rams who are attracted to other rams.

The scientists claim their research is aimed only at reducing or eliminating gay sheep.

About one in ten rams mount other rams instead of ewes. This 10% cost farmers money, and scientists have been experimenting with changing the hormone levels in the brains of these ‘gay’ animals. So far the tests have been successful, with formerly ‘gay’ rams mounting ewes.

The biologist leading the experiments, Charles Roselli, said:

"In general, sexuality has been under-studied because of political concerns. People don’t want science looking into what determines sexuality.

"It’s a touchy issue. In fact, several studies have shown that people who believe homosexuality is biologically based are less homophobic than people who think that this orientation is acquired."

Roselli also said that possible applications of his research to humans were not a matter for him.

Udo Schuklenk, Professor of Bioethics at Glasgow Caledonian University, who has written to the researchers pressing them to stop, told The Times: "I don’t believe the motives of the study are homophobic, but their work brings the terrible possibility of exploitation by homophobic societies. Imagine this technology in the hands of Iran, for example.

"It is typical of the US to ignore the global context in which this is taking place."

The relatively simple process of altering the hormonal balance of the brain could easily be applied to humans, raising questions about the nature of homosexuality and the possibility that in future parents could screen unborn children for ‘gayness’ and also have their children treated to become heterosexual.

Critics of the experiments are led by tennis champion Martina Navratilova, who branded them homophobic and cruel, as well as deeply insulting to gay and lesbian people.

She wrote to the university in November, saying, "I respectfully ask that you pull the plug on this appalling and misguided research.

"Surely you can find a way to redirect the millions of public tax dollars that are being wasted on these experiments to a more fruitful venture—perhaps by funding a gay and lesbian community centre to foster dialogue and acceptance for people of all sexual preferences?"

Peter Tatchell, a gay rights campaigner, said: "These experiments echo Nazi research in the early 1940s which aimed at eradicating homosexuality.

"They stink of eugenics. There is a danger that extreme homophobic regimes may try to use these experimental results to change the orientation of gay people."


~~~~~~~~~~~~~~~~~~~~~~~~~~~~~~~~~~~~~~~~~~~~~~~~~~~~~~

I don't know whether to laugh or cry or just cry out laughing. I'm interested to know if the same gay advocates that object to this would do so if the objective was to encourage more gay sheep.

I'm sure these OSU researchers were doing exactly what they claimed, trying to help farmers by increasing the value of such 'gay' sheep. Little did they know they were doing the typical US thing of ignoring "the global context in which this is taking place," unlike the cloning of Dolly, heralded as "one of the most significant scientific breakthroughs" of the 90's (although if this research had also been conducted in Scotland I bet the Glasgow Caledonian professor would be singing a different tune).

But hey, if nothing else at least these OSU scientists have given us another reason to do something about Iran.

tomder55 answered on 01/04/07:

I guess those rams have not found the right Rachel yet .

I don't know why they would go this route rather than artificial insemination if they were just trying to increase the herd .


Wouldn't that be a hoot if all this testing revealed that homosexuality is a hormone imbalance ? Take a pill and cure the affliction ! Think about it .. Biology is the blame all for a whole slew of anti-social behaviors like sexual agression,schizophrenia,and hyperactivity. We make up names for afflicitions like ADHD and give em a pill so their behavior is managed ;to control their impulses and compulsions .At the same time the pc world says being homosexual is ok because they were "born that way" .

On the other hand ;the danger in all this is that business of the handicapped trying to biologically manipulate a fetus to ensure they are born with the same afflicition the parents have. I think it was you who posted about the deaf couple who wanted to birth a deaf baby. Would they then manipulate the fetus to ensure homosexuality ?

I see this as another case where science is treading on that razor edge . We do need to occasionally take pause and consider the ethical implications of our discoveries. Just because we can doesn't mean we should .

Itsdb rated this answer Excellent or Above Average Answer

Question/Answer
Choux... asked on 01/03/07 - 20,000 TROOP INCREASE FOR **POLITICAL** REASONS

“President Bush is expected to announce his new Iraq strategy in an address to the nation early next week.” According to the BBC, “The speech will reveal a plan to send more US troops to Iraq.”

Last night on NBC News, Jim Miklaszewski reported that the new strategy will be announced next Tuesday, and that an administration official “admitted to us today that this surge option is more of a political decision than a military one.”

Just weeks ago, CentCom commander Gen. John Abizaid told Congress “I met with every divisional commander, General Casey, the corps commander, General Dempsey, we all talked together. And I said, in your professional opinion, if we were to bring in more American Troops now, does it add considerably to our ability to achieve success in Iraq? And they all said no.”

~~~~~~~~~~~~~~~~~~~~~~~~~~~~~~~~~~~~~~~~~~~~

Would you want your child to die or be maimed for **Bush's Political reasons** in his War of Adventurism on Iraq?

tomder55 answered on 01/04/07:

I don't know how anyone gets an accurate read of the situation on the ground from the MSM . Currently there are a total of six reporters embedded with our military forces. The rest (more than 500) are running around the country seeking anti-American stories or hiding out in the Green Zone's 'Mahogany Hill'. Everything we get from our media in Iraq is negative.

Maybe the build up is for Iran.....hmmmm.. The USS John C. Stennis strike group will this month join the USS Dwight Eisenhower aircraft carrier group and USS Boxer strike force in the Persian Gulf...Deployment of the Stennis group puts a total of 16,000 US sailors in the region as well as another nuclear carrier and 7 escort warships, 10 air squadrons, 2 submarines and helicopters to support amphibious landings on enemy soil....more troops on the ground.....

I don't understand what "political decision " means . If politics entered into Bush's decision I am sure he would be announcing a draw down of troops .

btw. rumors have it that the number will be closer to 40,000 additional troops .No matter what the number I am more interested to see if he announces a change in the Rules of Engagement .That to me is the important information .Announcing a bump in troop strength is not strategy .We don't need additional troops to train Iraqi forces or to sit in the Green Zone sipping tea. If they are going there to put down the Mahdi para-military /militia /gang (whatever) then I am all for it.

I am tired of talk about exit strategy .Bush is right.He went to the Marine Commandant and asked for a victory strategy .Reports of Casey’s coming dismissal is further evidence of Bush’s resolve to pursue a proactive policy on Iraq and Iran rather than accept the defeat that the Iraq Surrender Group suggested .

We are not losing . We set out to topple Saddam;and did. We decided to govern Iraq while they wrote a constitution;we did. We decided to help them with their elections;they were successful each time. We said we would train an Iraqi army;they are up to 200,000 men and growing (and in many cases- not all, performing admirably). We said we would continue to stay until the Iraq gov't can run things and defend itself...and we will. Out of 5 objectives we have achieved 4 (#5 is ongoing).
1.end Saddam Hussein's regime
2.prevent the Iraqi WMD programs from falling into terrorist hands
3.end Saddam's support of Islamic terror
4.establish a new government in Iraq consisting of a constitution and an elected government.
5.establish a security infrastructure there that will suffice to maintain the government until it can take care of security themselves .

I have indicated to my daughter that I support whatever decision she takes. Actually I think she would look good in a Navy Uniform ;but that is her decision. y cousin was in Desert Storm and is in his 2nd tour in Iraq since OIF. No ,I do not wish to see him hurt or killed ,but wild horses couldn't prevent him from going .As I understand it ,he had options this time.

Choux... rated this answer Above Average Answer
ETWolverine rated this answer Excellent or Above Average Answer

Question/Answer
Itsdb asked on 01/02/07 - Book closed on America's villain

That was the headline of the following al-AP 'anaylsis' in my paper on Saturday:

    CALVIN WOODWARD
    Associated Press

    WASHINGTON - When U.S. leaders decided it was time to despise Saddam Hussein, he made the perfect villain.

    He was cocky and cunning. He looked dangerous and deranged standing at rallies firing a gun into the air, conduct unbecoming a head of government.

    He was Hitler Lite, or as the first President Bush put it, "Hitler revisited," lacking the endless armies, but close enough for U.S. purposes. He had a history of atrocities. His black mustache heightened the aura of menace.


    America's quarter-century entanglement with the Iraqi leader ended Friday at the gallows.

    His hanging closed the books on a man who dealt with and benefited from the United States, then defied it, then ran like a rabbit into a hole in the ground, reduced to his own army of one.

    Saddam's capture Dec. 13, 2003, was a rare day of triumph for the United States after the Iraq invasion. In contrast, his execution brought worries that violence would spike beyond its usual chaotic level.

    Minibus and car bombings Baghdad killed at least 68 people in the hours after his death. The toll was not unusually high and the attacks were not immediately tied to his execution.

    Saddam was vilified by the U.S. government probably more than any dictator since Adolf Hitler.

    And this is a country with a long and still-active tradition of personalizing its enemy, making conflicts less about competing interests than about specific madmen and loose cannons - Manuel Noriega, Slobodan Milosevic, Moammar Gadhafi, Fidel Castro, the wanted-dead-or-alive Osama bin Laden.

    While others cry "death to America," America assembles a rogues gallery.

    Colin Powell, writing in his memoirs about the lead-up to the first Gulf War, objected to the portrayal of Saddam as the "devil incarnate" by the elder President Bush and aides.

    "President Bush has taken to demonizing Saddam in public just as he had Manuel Noriega," said Powell, who was chairman of the Joint Chiefs of Staff in the Gulf War, then secretary of state for the Iraq war. He suggested U.S. officials "cool the rhetoric. Not that the charges were untrue, but the demonizing left me uneasy."

    Yet a decade later, in the words of the younger President Bush and aides, including Powell, the case for war was about "Saddam's chemical weapons business," "his weapons of mass destruction," "his terrorist associations," his "massive clandestine nuclear weapons program," "his evil mind."

    U.S. officials were never comfortable with Saddam but treated him as a useful counterweight to the hostile theocracy in Iran after the U.S.-supported shah fled the country in 1979.

    Iraq was at least a partly westernized and secular presence in a time of rising anti-American sentiment in the region, and had relations with the Soviets that Washington wanted to restrain.

    In the long war between Iran and Iraq, the Reagan administration helped Saddam get international loans, restored formal relations in 1984 and secretly provided Iraq with intelligence and military support.

    It sent Donald H. Rumsfeld, who had served in the Nixon and Ford administrations, on a tour in December 1983 that included a stop in Baghdad and meetings there with Saddam and his foreign minister, Tariq Aziz.

    Worried about Syria and oil supplies as well as Iran, Rumsfeld suggested relations between the two countries had "more similarities than differences," according to his report from the meetings. An equally accommodating Saddam suggested his part of the world had more in common culturally with Washington than Moscow.

    In his meeting with the foreign minister - but not Saddam - Rumsfeld parenthetically raised subjects that hindered the U.S. from doing more for Iraq in its war with Iran. Two decades later, with Rumsfeld as defense secretary, these subjects would be used to summon rage against the Iraqi leader.

    "I made clear that our efforts to assist were inhibited by certain things that made it difficult for us, citing the use of chemical weapons, possible escalation in the Gulf, and human rights," Rumsfeld wrote back then.

    By 1991, the United States was at war with Iraq, assembling a coalition to force Saddam to reverse his annexation of Kuwait. Saddam was the target of U.S. denunciation from then on, as a sponsor of terrorism, a seeker of weapons of mass destruction, and a ruthless murderer of Kurds, opponents of his rule and inconvenient family members.

    Left in power after his forces retreated from Kuwait, Saddam was a volcanic presence in U.S. affairs for another decade, capped but toxic. It was a time of convoluted sanctions, fitful weapons inspections and no-fly-zone confrontations.

    A fuzzy Iraqi TV picture captured the 1983 handshake between Saddam and Rumsfeld the envoy on a day when the two men agreed it was too bad a generation of Americans and Iraqis had grown up without knowing each other.

    The future would bring the next generations together on bloody streets in a conflict neither side imagined then. And the man who shook Saddam's hand would direct the costly war that drove him from power, into the hole and to the executioner.


Is that all it is, America is just busy "personalizing its enemy, making conflicts less about competing interests than about specific madmen and loose cannons?"

Does the media have a role in that, as in say ignoring the numerous reasons given for going to war with Iraq and the 12 years of unenforced UN resolutions in favor of focusing on one aspect?

Is it just me or is anyone else a little creeped out that an American 'journalist' could write about Saddam in such a dismissive, sarcastic manner?

tomder55 answered on 01/03/07:

It really is hard to believe that Colin Powell was Chair of Joint Chiefs of Staff at such a critical time in our history . The more I read ,the more I conclude that if he had not been overruled ,then Saddam would've controlled the oil fields of Iraq;Kuwait and Saudi Arabia. His idea of using massive force to achieve a limited objective ;and have an exit strategy after the clock runs out at 100 hrs. has pretty much been refutiated .

It also is pretty much a useless exercise to rehash the fact that at one time we had an alliance of convenience with Saddam . I concede that point . We could've stacked chemical weapons as high as the tower of babel there without any expectation that he would've used them on his population .


I like Cal Thomas's take on the Saddam hanging .

...in the end Saddam's execution wasn't about revenge. It was about justice. Many countries — from Britain, which has abolished capital punishment, to Russia, where a moratorium on capital punishment now exists, have halted executions because they believe, incorrectly, that doing so makes them more humane. It does precisely the opposite and sends the message that innocent human life has less value than the life of a killer.

As Gerald Ford said to Tom DeFrank ;(paraphrase ) the fact that Saddam was a murderous scum was enough justification for toppling him ...wmd was simply the murder weapon of choice.

Itsdb rated this answer Excellent or Above Average Answer

Question/Answer
Itsdb asked on 01/02/07 - Bush's last chance?

In more al-AP analysis...

    DEB RIECHMANN
    Associated Press

    CRAWFORD, Texas - Whatever the reasons for President Bush's lengthy deliberations on a new Iraq policy, they undoubtedly will serve two political purposes: Letting the grim milestone of 3,000 U.S. deaths in Iraq and the potential backlash from Saddam Hussein's execution pass before the public hears his new ideas.

    The execution of Saddam by his countrymen would help legitimize the U.S.-led invasion in 2003. Yet, if it incites more bloodshed, it would remind Americans that the situation is "grave and deteriorating," as the recent Iraq Study Group concluded.

    The American public has grown weary of the war and even though past wars have seen vastly higher casualties, a U.S. death toll topping 3,000 - which is approaching - would shine a spotlight on the human toll of U.S. involvement.

    For now, Bush has been able to fend off calls for withdrawal of U.S. troops. Yet if the situation in Iraq doesn't improve - and quickly - those calls could begin to drown out whatever new ideas he puts forth in the early weeks of the new year.

    Americans are a patient lot and likely will give Bush the time and backing he needs to take another shot at getting a U.S. policy in Iraq that works. And the new Democratically led Congress, which convenes on Jan. 4, probably won't block the commander in chief if he decides to briefly increase troop levels.

    "It is likely his last chance, however," said analyst Jon Alterman. "Republicans and Democrats alike will be looking for early signs that the president's policy isn't working, in which case they will quickly head for the exits. My sense is that this is taking a long time because they know it's their last shot."

    Dan Bartlett, counselor to the president, said Friday that neither the approaching 3,000th U.S. death in Iraq nor Saddam's execution is "dictating when" Bush's speech will be delivered. Those two events, though, will influence its reception by the American people.

    When Saddam was pulled from his hiding spot in a spider hole in December 2003, public opinion shifted in Bush's favor. But the former Iraqi leader's execution likely would have less inclination to sway public opinion now because Americans' views have hardened as the war has intensified.

    The White House viewed Saddam's execution as an "important milestone in the Iraqi people's efforts to replace the rule of a tyrant with the rule of law."

    Still, skeptics of the president's policy can argue that remains mired in violent turmoil. And those opposed to a surge in U.S. troops will use the 3,000th death as a reason to continue opposing one.

    "I think there was a time when the death of Saddam Hussein would have given Bush the kind of political capital he needs to call for an increase in troops and an expansion of the military effort there, but I think we're past that time," said Julian Zelizer, a political historian at Boston University.

    Bush is expected to deliver his speech - laying out his plan to improve security, assist the Iraqis in reaching a political reconciliation between warring sects and help with reconstruction - before his State of the Union address on Jan. 23.

    The president, who has been at his ranch in Crawford, Texas, this week, spent a windy, rainy Friday talking about Iraq on the phone with British Prime Minister Tony Blair and took time to ponder discussions he had on Thursday with Vice President Dick Cheney, Secretary of State Condoleezza Rice, Defense Secretary Robert Gates, National Security Adviser Stephen Hadley and Gen. Peter Pace, chairman of the Joint Chiefs of Staff.

    Military historian Frederick Kagan at the American Enterprise Institute, a conservative think tank in Washington, said Bush's speech - whenever it occurs - will overwhelm any event in Iraq.

    "We've reached a critical moment. The president is clearly contemplating a strategy that will be very different from what he has been doing," said Kagan, who is advocating a large surge in U.S. forces in Iraq. "That message is going to dominate the discussion. The American people want to know whether we're going to win this war, and they're going to listen very carefully to whatever the president says."

    Whether he likes it or not, the president's legacy is hinged to Iraq. Poor progress there helped sweep Democrats into power in both the House and the Senate in November's midterm elections. If the United States continues to be mired in a violent Iraq, the chances for Democrats to capture the White House in 2008 brighten as well.

    Bush's approval for handling Iraq was at 27 percent in early December, according to AP-Ipsos polling - his lowest approval rating yet in this area. Seventy-one percent disapproved of how he was managing the war. Moreover, almost two-thirds, or 63 percent, doubt that a stable, democratic government will be established in Iraq. That's up from 54 percent who felt that way in June.

    At this juncture, some political analysts think the timing of Bush's Iraqi speech is irrelevant. They say he has just one roll of the dice left on Iraq, and if his strategy does not help stabilize things there, it will be quickly overtaken by events.


After the seemingly thousands of times I've heard or read that Bush "rushed to war" or "failed to plan," it sure seemed curious to me that this al-AP writer would open her analysis with "Whatever the reasons for President Bush's lengthy deliberations." Why the rush now?

Are the twin political purposes of "Letting the grim milestone of 3,000 U.S. deaths in Iraq and the potential backlash from Saddam Hussein's execution pass before the public hears his new ideas" behind the 'delay?'

Is all of this irrelevent since the left isn't kidding around and is going to fight those Democrats for not getting us out of Iraq already?

tomder55 answered on 01/03/07:

I have generally agreed with Frederick Kagan's take on Iraq . He is a long time advocate of the troop surge option .

After the seemingly thousands of times I've heard or read that Bush "rushed to war" or "failed to plan," it sure seemed curious to me that this al-AP writer would open her analysis with "Whatever the reasons for President Bush's lengthy deliberations." Why the rush now?

That question is easy . The Iraq Surrender Group has spoken and has given cover to the cut and run crowd. Why should Bush seek other options when the mighty Baker has spoken ? Heck ;he told Bush the report wasn't a salad to pick through ;it's all or nothing .


Are the twin political purposes of "Letting the grim milestone of 3,000 U.S. deaths in Iraq and the potential backlash from Saddam Hussein's execution pass before the public hears his new ideas" behind the 'delay?'

No ;if Bush really governed by public opinion he would be planning for our troops to exit stage left for the nearest troop transport ship after the Nov. elections and the public "pulled a Spain" .


Is all of this irrelevent since the left isn't kidding around and is going to fight those Democrats for not getting us out of Iraq already?

More likely the Democrat leadership will go along because to oppose the President means they would have to offer a concrete alternative . So Evita will grudgingly give her support and will argue that quid pro quo bench marks be established. The rest of the Dems except Kucinich and the rest that cannot be hurt by openly opposing the plan will bray at the moon. Oh yeah ;and Joe Biden will continue to talk to himself about partition .







Itsdb rated this answer Excellent or Above Average Answer

Question/Answer
Choux... asked on 01/02/07 - SADAM'S BOTCHED EXECUTION , A MARTYR IS CREATED

I guess we are all familiar with the circumstances of the hanging of Saddam Hussein, and I imagine some here watched it on the tape created by someone holding up his cell phone and photographing it.

All I could think was it should have been Osama bin Laden being executed. Bin Laden and the Saudi terrorists who carried out the 911 attacks!! But, apparently Bush/Cheney and their powerful oil company execs have managed a deal of some sorts giving bin Laden a pass.

Here are the comments of Tom Brokaw on the Imus radio show:

BROKAW: "No, it’s — you know, as we portray ourselves around the world as the champions of democracy and the rule of law — first of all, that began to unravel in the eyes of a lot of people in that part of world with Abu Ghraib and the great cruelties and indignities that were imposed on people there. The debate goes on here about Guantanamo and about access to people’s private records. And then to say that we are going to install in Iraq a judicial system and a democratic form of government and have something that resembled the worst kind of nightmare out of the old American West. Not much dignity. He was, he was a god awful man and he did have a trial, but not have control of the execution, and to have it really just fuel more sectarian violence at a time when we are trying to dampen that is not helpful, which is an understatement.

IMUS: Well, I guess the New York Times reported and I was also talking to Richard about that the United States apparently unsuccessfully prevailed about Maliki to delay this.

BROKAW: Yep.

IMUS: I wonder, I wonder why he refused? I mean…

BROKAW: I honestly don’t know either. But Saddam Hussein who had disappeared, in effect, as some kind of a symbol over there, suddenly becomes a martyr. He was a terrible tyrant who was responsible for an untold number of deaths, you know, waged his own jihad against the Shiite in that country, especially in the south following Operation Desert Storm in the early 1990s, and now he’s able to stand up there with the hood off and invoke prayer and even invoke the Palestinians, and go out in the eyes of his people at least as a martyr.

IMUS: I mean, it’s difficult to imagine how this could have turned out worse.

BROKAW: No, it is pretty difficult to imagine, and it’s, you know, just as the military commanders and the political people who are trying to run the war think that they’ve got something quieted over in one front, it pops up in another."


Today we learn the Bush is removing Gen. Casey blaming him for the failures in prosecuting the War on Iraq...WHICH WE ALL KNOW WAS RUN BY RUMSFELD FROM THE PENTAGON. The feedback from Generals in the field was ignored by Bush and his administration per many news stories over the last year or so.


So now, the Sunnis because of the recorded taunting of Saddam at the gallows, have a martyr to unite behind eternally WITH ALL THEIR HEARTS in their civil war against the majority Shi'a.


Bush has proven himself to be totally incompetent.

IMPEACH NOW

tomder55 answered on 01/03/07:

I think I would've been screamin obscenities at him if I was one of the family members . Indeed I think that happens at American executions but the Iraqis forgot to ask people to turn off their cell phones. Too freakin bad if the Sunnis are upset .

Now imagine how the world community would've reacted if the US conducted the execution.Everyone says we cannot export our brand of democracy to the middle east and I agree. They have forged their own .

Saddam had a trial at great risks to those who conducted it. ;he was defended (even by that doofus Ramsey Clark ...a former Att. General of the US). I do not understand the beef.

Was the trial and execution any worse than the kangaroo court the world community assembled for Slobodan Milosevic ?...They dragged that on for so long that he died of old age waiting for the conclusion.

Itsdb rated this answer Excellent or Above Average Answer
Choux... rated this answer Excellent or Above Average Answer
labman rated this answer Excellent or Above Average Answer

Question/Answer
Itsdb asked on 01/02/07 - Would you repeat that?

As you all know, James Brown was laid to rest last week, in a funky way befitting the "Godfather of Soul." No disrespect to the man - may God rest his soul - but did anyone besides me find Sharpton's eulogy a little curious to say the least?

    In life and in death, James Brown should be remembered for his impact on music and on the world, not for the many people that surrounded him, the Rev. Al Sharpton said Friday in a passionate eulogy befitting the godfather of soul.

    "When he started singing, we were sitting in the back of the bus. When he stopped singing we were flying Lear jets"


Who is this 'we' he's talking about? Al, Michael and Jesse? Am I the only guy in the country that sees disrespect, insensitivity and a complete lack of judgment in that comment? How many of Al's constituency are "flying lear jets"?

tomder55 answered on 01/02/07:

James Brown raised Sharpton (sorta;shortly after Brown and Sharpton met,Brown’s son Teddy died in a car crash and he in effect became the surrogate son to Brown) so when he talks about the Lear jets ,he is speaking from personal experience and since he sees himself as the representative of black achievement then by extention he can use the word "we " . sounds bizzare I know ;but that is the closest meaning I can divine from his incohenence .

Itsdb rated this answer Excellent or Above Average Answer

Question/Answer
Choux... asked on 12/27/06 - METAPHOR FOR DEATH OF MODERN REPUBLICANISM

"Why do I say this? As many of you know who pay any attention to what comes out of my big mouth, I am not given to false sentiments. So if it were, say, Rush Limbaugh, I would most likely yawn.

No, I offer this small tribute because while I disagreed with Ford on many issues (Nixon, for example), when he led the GOP was a time when sane people were in charge of that party.
People you could do business with on issues like civil rights and clean water.

It was also a time of Republican foreign policy internationalism with an emphasis on diplomacy, in the wake of the dreadful mistake of Vietnam. Most importantly, a willingness to meet in the actual political middle, not some false center that Joe Lieberman invented in his patently dishonest noggin.

I can't remember where I saw this, so I am unable to link, but there was an analysis done that showed that Ford's judicial picks were actually to the left of Clinton's, (unlike current Republican judges, like THIS IDIOT). Ford supported the ERA, the right to choose in most instances and affirmative action. You get the point.

I didn't love the man, but I did respect him and do mourn his passing. For it is a *perfect metaphor for the death of moderate Republicanism*, which just as surely met its Maker on November 7th, 2006, at the hands of a band of corrupt and right-wing neoconservative pseudo-thinkers, retrograde fundamentalist science-haters and venal corporate-humpers." Cliff Schecter, Blogging

~~~~~~~~~~~~~~~~~~~~~~~~~~~~~~~~~~~~~~~~~~~~~~~~~

tomder55 answered on 01/02/07:

At the time I was a Democrat . I faulted Ford with the pardon and like many Democrats have seen the error in my view. It led me to make one of the worse political mistakes I've ever made....voting for Jimmy Carter in the first presidential election I voted in ;which I corrected in 1980 .

Ford was a (put it generously )moderate Republican . Truth be told ;had he not been President ,his legislative service was ordinary . He rose to be House Republican leader but that was as a result ;as the Congressional Quarterly put it he "built a reputation for being solid, dependable and loyal — a man more comfortable carrying out the programs of others than in initiating things on his own." His one notable achievement during those years was being seated on the 'Warren Commission'.

As it turns out ;the country lucked out that Spiro Agnew was booted (resigned ) due to corruption and Ford was appointed .

But besides him being a calming presence ,I did not think (nor do I now )his Presidency was a good one . But I do not want to get intio the details here on the day of his funeral . Suffice it to say he should be honored because Ford served his country with decency and honor and he deserves our respect.


ETWolverine rated this answer Excellent or Above Average Answer
Itsdb rated this answer Excellent or Above Average Answer
Choux... rated this answer Excellent or Above Average Answer

Question/Answer
Choux... asked on 12/26/06 - ARMED FORCES CONSIDERING MERCENARIES

WASHINGTON: "The armed forces, already struggling to meet recruiting goals, are considering expanding the number of noncitizens in the ranks including disputed proposals to open recruiting stations overseas and put more immigrants on a faster track to U.S. citizenship if they volunteer according to Pentagon officials.

Foreign citizens' serving in the U.S. military is a highly charged issue, which could expose the Pentagon to criticism that it is essentially using mercenaries to defend the country. Other analysts voice concern that a large contingent of noncitizens under arms could jeopardize national security or reflect badly on Americans' willingness to serve in uniform."

~~~~~~~~~~~~~~~~~~~~~~~~~~~~~~~~~~~~~~~~~~~~~

What do you think about the use of mercenaries in America's Wars?

tomder55 answered on 12/27/06:

First a couple of misconceptions must be corrected .

First ;The armed forces are not struggling to meet recruiting goals.

Second ;The US has always recruited foreign fighters and currently there are foreign soldiers in our services .The Philippines supplies hundreds of recruits each year, and have been doing so for decades.

7 percent of America's active fighting forces are immigrants with green cards.There were 28,000 immigrant soldiers five years ago; that number has climbed to 39,000 today, not counting the thousands of foreign contractors hired since 9/11. 59 immigrant casualties have been granted posthumous citizenship ;and a new rule allows their families to use the deceased as a sponsor for their own residency papers. Their service is steeped in pride and should be honored . Foreign volunteers tend to be exemplary in the line of duty, and units of mostly Hispanic fighters are doing some of the tough work in Iraq.
The problem is that currently there is no guarantee of citizenship even after they have served their time.I have no problem with granting citizenship to legal immigrants in exchange for serving in the U.S. military. It beats the hell out of granting citizenship to illegal aliens for successfully avoiding getting picked up while using someone else's social security number.

Americans have always feigned a degree of uncomfortability in the practice
(the Declaration of Independence warns of the dangers of mercenaries which of course did not stop the colonist from accepting help from many mercenaries themselves ..A good portion of the officers who trained and led the American army and navy were foreign mercenaries.); but that has never stopped us from filling the ranks with foreigners. 20 percent of Union Soldiers in the Civil War were immigrants .Many a Union unit was totally filled by recruits right off the boat .

The real change that everyone is talking about today is a new law that gave the Pentagon authority to bring immigrants to the United States if it determines it is vital to national security. So far, the Pentagon has not taken advantage of it but the possibility is on the table and it may go into effect if we go ahead and increase the overall strength of out armed services.

btw ;Elliot is right about the use of the various foreign legions and the very effective Gurkhas who fought for England for over 200 years . Also note that Swiss Guard have defended the Vatican for centuries (and don't underestimate their effectiveness ;only the very elite make it into the Swiss Guard .Many of the first "Green Berets" were Poles, Czechs, Romanians, etc that escaped the Iron Curtain, and hated the Soviets with a passion. We welcomed them with open arms for their native language skills, and their knowledge of their former country's customs and terrain. Perfect for behind enemy lines guerrilla warfare.Imagine an elite group of Muslim soldiers who despise what the ummah has become working with the armed forces in areas of intel gathering where we have a clear disadvantage .

Choux... rated this answer Excellent or Above Average Answer
ETWolverine rated this answer Excellent or Above Average Answer

Question/Answer
Choux... asked on 12/26/06 - ARMED FORCES CONSIDERING MERCENARIES

WASHINGTON: "The armed forces, already struggling to meet recruiting goals, are considering expanding the number of noncitizens in the ranks including disputed proposals to open recruiting stations overseas and put more immigrants on a faster track to U.S. citizenship if they volunteer according to Pentagon officials.

Foreign citizens' serving in the U.S. military is a highly charged issue, which could expose the Pentagon to criticism that it is essentially using mercenaries to defend the country. Other analysts voice concern that a large contingent of noncitizens under arms could jeopardize national security or reflect badly on Americans' willingness to serve in uniform."

~~~~~~~~~~~~~~~~~~~~~~~~~~~~~~~~~~~~~~~~~~~~~

What do you think about the use of mercenaries in America's Wars?

tomder55 answered on 12/27/06:

First a couple of misconceptions must be corrected .

First ;The armed forces are not struggling to meet recruiting goals.

Second ;The US has always recruited foreign fighters and currently there are foreign soldiers in our services .The Philippines supplies hundreds of recruits each year, and have been doing so for decades.

7 percent of America's active fighting forces are immigrants with green cards.There were 28,000 immigrant soldiers five years ago; that number has climbed to 39,000 today, not counting the thousands of foreign contractors hired since 9/11. 59 immigrant casualties have been granted posthumous citizenship ;and a new rule allows their families to use the deceased as a sponsor for their own residency papers. Their service is steeped in pride and should be honored . Foreign volunteers tend to be exemplary in the line of duty, and units of mostly Hispanic fighters are doing some of the tough work in Iraq.
The problem is that currently there is no guarantee of citizenship even after they have served their time.I have no problem with granting citizenship to legal immigrants in exchange for serving in the U.S. military. It beats the hell out of granting citizenship to illegal aliens for successfully avoiding getting picked up while using someone else's social security number.

Americans have always feigned a degree of uncomfortability in the practice
(the Declaration of Independence warns of the dangers of mercenaries which of course did not stop the colonist from accepting help from many mercenaries themselves ..A good portion of the officers who trained and led the American army and navy were foreign mercenaries.); but that has never stopped us from filling the ranks with foreigners. 20 percent of Union Soldiers in the Civil War were immigrants .Many a Union unit was totally filled by recruits right off the boat .

The real change that everyone is talking about today is a new law that gave the Pentagon authority to bring immigrants to the United States if it determines it is vital to national security. So far, the Pentagon has not taken advantage of it but the possibility is on the table and it may go into effect if we go ahead and increase the overall strength of out armed services.

btw ;Elliot is right about the use of the various foreign legions and the very effective Gurkhas who fought for England for over 200 years . Also note that Swiss Guard have defended the Vatican for centuries (and don't underestimate their effectiveness ;only ther very elite amke it into the Swiss Guard .Many of the first "Green Berets" were Poles, Czechs, Romanians, etc that escaped the Iron Curtain, and hated the Soviets with a passion. We welcomed them with open arms for their native language skills, and their knowledge of their former country's customs and terrain. Perfect for behind enemy lines guerrilla warfare.Imagine an elite group of Muslim soldiers who despise what the ummah has become working with the armed forces in areas of intel gathering where we have a clear disadvantage .

Choux... rated this answer Excellent or Above Average Answer
ETWolverine rated this answer Excellent or Above Average Answer

Question/Answer
Choux... asked on 12/26/06 - SADDAM HUSSEIN TO BE EXECUTE BY JAN 27, 2007

Things in Iraq are going to go real well January 27th going forward....

Anyone else get the shivers upon reading this newsbite?

tomder55 answered on 12/27/06:

Most of the Iraqi population will be pleased,and I expect celebrations when Saddam dances at the end of the hangman's noose.

I think the insurgency has moved on and did not expect Saddam to ever reclaim his former position . Seeing him cowering in a spider hole sorta ruined that Saladin image. But any Tikriti who had visions of his restoration will have that last bit of hope dashed .

Like a number of other events it represents a chance for Iraq to have a new beginning if they are bold enough to seize it . It is clear from the results of 3 seperate election that the people are open to it. They have the desire and have shown the courage .

Our role is to defeat foreign interventionist in the country whether they support the Sunni insurgency or they are the Iranian provocateurs who support and incite the Shia militias . Frankly I do not blame the shia too much for going on a retaliation spree but ultimately it does not help them secure the nation ,and there will be no benefit to them if the nation is not secure and free from foreign dominance once we leave .

ETWolverine rated this answer Excellent or Above Average Answer
ladybugca rated this answer Excellent or Above Average Answer
Choux... rated this answer Excellent or Above Average Answer

Question/Answer
paraclete asked on 12/27/06 - did Bush know, I wonder?

time will always defeat capability, and what George Bush doesn't have in Iraq is time


2006 a bloody year for terror

By Clive Williams

December 27, 2006 01:00am


THIS past year has been one of the bloodiest on record for terrorism, largely due to the ongoing carnage in Iraq. As many as 655,000 Iraqis have been killed during the war since 2003, a Johns Hopkins University study claims.

I define terrorism as "politically (including socially and religiously) motivated violence, mainly directed against non-combatants, intended to shock and terrify, to achieve a strategic outcome".

Terrorism may be perpetrated by individuals, groups or states. Historically, states have been far more efficient at killing civilians than individuals or groups.

In 2006 the war in Lebanon between Hezbollah and Israel accounted for 1183 Lebanese and 163 Israeli deaths. Ethnic cleansing in Darfur, Sudan, has resulted in at least 300,000 deaths since 2003.

The multi-faceted Afghanistan conflict accounted for at least 4000 Afghan deaths in 2006, while NATO/US forces there suffered nearly 200 deaths during the year.

The tit-for-tat terrorism, since September 2000, between the Palestinians and Israelis in Gaza and the West Bank has resulted in about 4400 Palestinian deaths and 1100 Israeli deaths. This year, the ratio of Palestinian deaths to Israeli deaths moved closer to 10 to one, notwithstanding the recent Hamas versus Fatah violence.

Numbers do not, of course, tell the full story. Terrorists and insurgents play for time and encourage their adversaries to kill civilians as it suits their purpose.

Every civilian that an "occupation" force kills in Iraq, Afghanistan, Palestine or Chechnya makes them more enemies.

Time will always beat capability. It is a truism that military forces and politicians don't yet seem ready to acknowledge, despite all the evidence.

In terms of operations by terrorist groups in the West, they are having less success against better prepared security services.

Seventeen Muslim prospective terrorists were arrested in Ontario, Canada, in June, and a plot to detonate an aircraft liquid explosive bomb in the UK was foiled in August.

There were also significant arrests throughout the year in Europe.

The greatest loss of life in one terrorist incident was the Indian Mumbai train bombing in July. The bombing killed 259 commuters, mainly Hindus.

Pakistan-based terrorists were responsible for the deadly attack.

It did not, however, achieve its intended purpose of disrupting the Indian economy or sparking sectarian violence.

The issue of terrorism by states is contentious and tends to get put in the too-hard basket. In the case of state terrorism (or war crimes), victor's justice prevailed, meaning the occasional sacrifice of low-level perpetrators, but no attempt made to deal with culpable senior policymakers.

Clearly, Russian state terrorism habits die hard, demonstrated by the spate of probable FSB-organised assassinations of those critical of President Putin and his cronies.

In Australia, terrorism organiser Faheem Khalid Lodhi was convicted of terrorism offences and sentenced in August to 20 years in jail. He was the first to be convicted under the new 2005 anti-terrorism legislation.

Melbourne terror suspect Jack Thomas faces a new trial based on admissions he made to The Age newspaper and ABC TV's Four Corners program.

More than 20 other alleged terrorists from Sydney and Melbourne have been remanded in custody to stand trial. David Hicks attracted increasing public sympathy and the Howard Government's case for leaving him to American "justice" seemed increasingly threadbare. So, what are the terrorism prospects for 2007?

US President George W. Bush will keep on floundering indecisively over Iraq. Whatever the US does, the insurgent and terrorist violence looks set to continue. NATO looks like staying the course in Afghanistan at least until Bush leaves office, but the prognosis is for the security situation there to continue to deteriorate.

Israel will continue to play for time and hope that no one challenges its gradual absorption of the West Bank.

The Government in Sudan will count on the West not being sufficiently motivated to prevent the Janjaweed ethnic cleansing of land-tilling tribesmen, Russia will continue its heavy-handed occupation of Chechnya and the low-level terrorism in southern Thailand and the Philippines will continue.

Strategically, there seems little likelihood of anyone addressing the root causes that terrorist groups exploit to gain new members or which motivate self-starter Muslim groups in the West to strike at their host societies for their "anti-Muslim" foreign policies.

In 2006, Australia and its close allies managed to stay on top of their domestic security situations. This involved a combination of luck and good management. There can, of course, be no guarantee that we will continue to be as lucky in 2007.

Clive Williams is a Visiting Fellow at the Australian National University and an Adjunct Professor at Macquarie University

tomder55 answered on 12/27/06:

I dispute his basic premise that Time will always beat capability. It is a truism that military forces and politicians don't yet seem ready to acknowledge, despite all the evidence
......because there are clear examples where insurgencies ,that have used terrorist tactics, have been defeated .

The US defeated an insurgency after the Spanish American war in the Phillipines .The US & Philippines defeated the Huks, in the 1960s The Brits defeated the Malayan communists .The Brits defeated the IRA .The Red Army Faction,Baader-Meinhof and the Red Brigades are gone. The US killed Che in
Bolivia, then helped defeat the communists in El Salvador. The US helped
destroy the insurgent Viet Cong in South Vietnam, but then cut off
ammunition to the South Vietnamese Army when it was being attacked by
the North Vietnamese Army's tank divisions in 1975. The US defeated various Indian tribes in the 1800s.The Chinese army defeated the large Tibetan insurgency
in the 1960s, after which the Tibetans emphasized the role of
non-violence.
Armies routinely defeat insurgencies. You just don't hear about those that don't exist any more.

I also dispute his contention that Israel is engaged in tit-for tat terrorism . 50 Palestinian-made Kassam rockets have been fired across the Gaza border since the latest version of "truce " began without any significant Israeli response. While the world demands parallel response by Israel or less;I think they could settle the issue by attacking Gaza with disproportional attacks . Besides ;if it is civilians who are doing the attacks then how could killing them in response be considered a terrorist act ?

His last couple of paragraphs expose him for being just another appeaser. If there is a root cause for jihadist violence that can be addressed by peaceful means go ahead and address it . I'd like to know what it is first because all I see is a doctrine/dogma of conquest as the root cause.

ETWolverine rated this answer Excellent or Above Average Answer
Itsdb rated this answer Excellent or Above Average Answer
paraclete rated this answer Excellent or Above Average Answer

Question/Answer
Choux... asked on 12/23/06 - PARDONS AND COMMUTATIONS

WASHINGTON (AP) -- "President Bush issued 16 pardons Thursday and commuted the sentence of an Iowa man convicted of drug charges.

Six of the federal offenses were drug crimes, while others included bank fraud, mail fraud, the acceptance of a kickback, a false statement on a loan application and conspiracy to defraud the government over taxes.

Seven of the 16 received no prison or jail time, instead getting probation or a reduction in their military pensions.

The longest sentence was nine years, for aiding cocaine distribution, followed by a six-year term for conspiracy to possess marijuana.

With this batch, Bush has issued 113 pardons and commuted three sentences in his nearly six years in the White House, according to spokesman Tony Fratto.

A pardon amounts to federal forgiveness for one's crime, while a commutation cuts short an existing prison term.

On the pardons list were:

# Charles James Allen of Winchester, Virginia, conspiracy to defraud the United States. A former federal employee, Allen was convicted in 1979 for approving payments to James Hilles Associates Inc., a Virginia firm, for office supplies that were never delivered. In return, Allen received car parts, a radio, a freezer and other gifts from the firm. He was sentenced to a year of custody to be served by 30 days in jail, 90 days in a work-release program, and the remaining period on parole.

# William Sidney Baldwin Sr. of Green Pond, South Carolina, conspiracy to possess marijuana. Sentenced October 27, 1981, to six years' imprisonment.

# Timothy Evans Barfield of Cary, North Carolina, aiding and abetting false statements on a Small Business Administration loan application. Sentenced July 17, 1989, to three years' probation, including 96 hours of community service.

# Clyde Philip Boudreaux of Thibodaux, Louisiana, borrowing money from enlisted men, accepting a non-interest-bearing loan from a government contractor and signing and swearing to a false affidavit. Sentenced December 2, 1975, to a Navy reprimand, loss of numbers on the promotion list and a $1,000 fine.

# Marie Georgette Ginette Briere of Gatineau, Quebec, possession of cocaine with intent to distribute. Sentenced July 9, 1982, to three years' imprisonment and three years' special parole.

# Dale C. Critz Jr., Savannah, Georgia, making a false statement. Sentenced July 13, 1989, to three years' probation.

# Mark Alan Eberwine of San Antonio, Texas, conspiracy to defraud the United States by impeding, impairing and obstructing the assessment of taxes by the Internal Revenue Service and making false declarations to the grand jury. Sentenced February 1, 1985, as amended April 23, 1986, to two years' imprisonment.

# Colin Earl Francis of Naugatuck, Connecticut, accepting a kickback. Sentenced May 7, 1993, to two years' probation and a $2,500 fine.

# George Thomas Harley of Albuquerque, New Mexico, aiding and abetting the distribution of cocaine. Sentenced November 30, 1984, to nine years' imprisonment and five years' special parole.

# Patricia Ann Hultman, of Kane, Pennsylvania, conspiracy to possess with intent to distribute and to distribute cocaine and other controlled substances. Sentenced October 28, 1985, to one year of imprisonment.

# Eric William Olson of Ojai, California, conspiracy to possess with intent to distribute, possession with intent to distribute, possession, and use of hashish. Sentenced February 21, 1984, by an Army general court-martial to confinement at hard labor for one year, reduction in pay grade, forfeiture of all pay and allowances and a bad conduct discharge.

# Thomas R. Reece of Cumming, Georgia, violating the Internal Revenue Code pertaining to alcohol. Sentenced May 2, 1969, to one year of imprisonment.

# Larry Gene Ross of Indio, California, making false statements in a bank loan application. Sentenced August 15, 1989, to four years' probation and $7,654.20 in restitution.

# Jearld David Swanner of Lexington, Oklahoma, making false statements in a bank loan application. Sentenced December 6, 1991, to three years' probation.

# James Walter Taylor of McCrory, Arkansas, bank fraud. Sentenced October 18, 1991, to 90 days in jail, followed by two years and nine months' probation.

# Janet Theone Upton of Salinas, California, mail fraud. Sentenced May 23, 1975, to two years' unsupervised probation.

Bush also commuted the sentence of Phillip Anthony Emmert of Washington, Iowa, whose case involved conspiracy to distribute methamphetamine.

Emmert was sentenced December 23, 1992, to 262 months' imprisonment (reduced on February 21, 1996) and five years' supervised release.

Bush directed that Emmert's sentence expire on this coming January 20, but left the supervised release intact.

Pardons are an end-of-the-year presidential tradition, and Bush was not expected to issue any more this year. He last issued pardons in August.

"Requests for executive clemency receive intense individualized consideration based upon an established set of objective criteria," spokesman Fratto said.

He said that after investigation by the Office of the Pardon Attorney at the Justice Department, most weight is given to these factors:

# the seriousness of the crime;

# how long ago it was committed;

# the acceptance of responsibility and showing remorse;

# post-conviction conduct and contributions to society;

# any specific compelling need for relief;

# official recommendations, including from the sentencing judge, the probation officer and the federal prosecutor.

The list did not include former White House aide I. Lewis "Scooter" Libby, charged in the CIA leak case with perjury and obstruction.

Libby, whose trial is scheduled to begin in January, is accused of lying to investigators about his conversations with reporters regarding outed CIA operative Valerie Plame, the wife of an Iraq war critic.

Bush remains among the stingiest of postwar presidents on pardons.

President Clinton issued 457 in eight years in office. Bush's father, George H. W. Bush, issued 77 in four years. President Reagan issued 406 in eight years, and President Carter issued 563 in four years.

Since World War II, the largest number of pardon and commutations -- 2,031 -- came from President Truman, who served 82 days short of eight years.

Copyright 2006 The Associated Press.

~~~~~~~~~~~~~~~~~~~~~~~~~~~~~~~~~~~~~~~~~~~~~~~~~


Does a President really need these powers??

tomder55 answered on 12/23/06:

The power to pardon is written in the Constitution (Article II, Section 2 ) Needed or not ,it was debated and included into the Constitution by the founders .The Framers wanted the power used for narrow purposes in times of war and rebellion as an inducement for a laying down of arms and national reconciliation. Alexander Hamilton argued in the Federalist Papers (No. 74) that "in seasons of insurrection or rebellion, there are often critical moments, when a well-timed offer of pardon to the insurgents or rebels may restore the tranquillity of the common wealth; and which, if suffered to pass unimproved, it may never be possible afterwards to recall." Anti-Federalists were very much opposed to pardons because it was reminicent of the abuses of the King.

At times it has been used the way the framers envisioned .Washington pardoned leaders of the Whiskey Rebellion,Andrew Johnson pardoned Confederate soldiers following the Civil War; Jimmy Carter pardoned draft dodger of the Vietnam War.

I do not see any particularily controversial pardons in the group above but they do go beyond what the founders intended ;but that is how the power has evolved .In my view it is done too often for political expediency or even more cynically for political favor (see Clinton's pardon of Marc Rich ).
The Courts however have interpreted the power as absolute.

Controversial would be Ford's pardon of Nixon . Bush Sr. pardon of Iran/Contra participants ;and many of Clinton's (especially the 140 that were issued on his last day of office)like his pardons of convicted FALN terrorists.

Since 1977, presidents have received about 600 pardon or clemency petitions a year and have granted around ten percent of these.I cannot tell you how often State Govenors have also exercised that power.

Choux... rated this answer Excellent or Above Average Answer

Question/Answer
Choux... asked on 12/22/06 - MITT ROMNEY TO ANNOUNCE

- "Massachusetts Gov. Mitt Romney is making plans for his campaign for the 2008 Republican presidential nomination in two phases early next month, a top adviser told The Associated Press on Friday.

The Massachusetts chief executive is expected to file paperwork as early as Jan. 2 with the Federal Election Commission, establishing a presidential campaign committee and permitting himself to begin raising money for his race on the first business day of the new year. Romney will leave office on Jan. 4.

As soon as the week of Jan. 8, Romney will hold a ceremony to officially declare his candidacy, said the adviser, a top aide who spoke on condition of anonymity in advance of the official filing".

~~~~~~~~~~~~~~~~~~~~~~~~~~~~~~~~~~~~~~~~~~~~~~~

Can the Mormon Church(and operatives) contribute unlimited amounts of money to support his run for president?

tomder55 answered on 12/23/06:

Romney is financing the early stages of his campaign with a multistate fund-raising operation that is allowing him to maximize legal donations.Since July 2004, Romney has set up affiliates of his political action committee, the 'Commonwealth PAC', in five states. By having donors spread their contributions across the various affiliates, Romney has been able to effectively evade the $5,000-per-donor annual contribution limit .Romney raised money quickly from relatively few contributors. By comparison ,McCain has to rely on a single Federal PAC .

It's a funny thing . I rarely hear his religion mentioned among conservatives . Oh there are plenty reasons to be a skeptic about him . He is a recent convert to many conservative issues. But it appears to me that I find more mention about his religion among the Huffpos.

Choux... rated this answer Excellent or Above Average Answer
Itsdb rated this answer Excellent or Above Average Answer
labman rated this answer Excellent or Above Average Answer

Question/Answer
ETWolverine asked on 12/22/06 - Genetic Engineering

DOCS' DESIGNER DEFECT BABY

By ANDY GELLER
December 22, 2006

If you have the right doctor and the right lab, you can have a baby with the right "designer disability."

That's right - you can create a baby with a birth defect.

For years, the medical world has been abuzz with talk of creating perfect designer babies through prenatal testing.

Now, some doctors are doing the opposite - making babies with made-to-order genetic defects, according to The Associated Press.

The procedure is an ethical minefield, but parents with disabilities like deafness or dwarfism say it just means making babies like themselves.

A recent Johns Hopkins survey of 137 American clinics that offer embryo screening found that four of them have done the procedure, which costs $15,000.

One of the nation's leading experts in reproductive medicine, Dr. Jamie Grifo of New York University, believes parents should be able to decide whether or not to have the procedure.

"Parents should certainly have a say in how they should be able to make decisions about how to conduct their reproductive lives, rather than leaving it up to some regulator or legislator," he said.

The procedure, part of a technique called preimplantation genetic diagnosis is done after eggs and sperm are mixed in a lab dish.

Before the embryo is implanted in the womb, a cell is removed to allow doctors to examine it for a genetic defect. In the past, such an embryo would be discarded.

But now, if the embryo contains a gene for, say, dwarfism or deafness, it is implanted in the womb.

Grifo does not believe the practice is widespread. "I'm the expert, and I've never had anyone ask for me to do it," he said.

The phrase "designer disability" was coined after two deaf lesbian psychotherapists used sperm from a deaf family friend to have two deaf children.

Cara Reynolds of Collingswood, N.J., considered having the procedure so she could have a dwarf baby.

In part, she felt she ought to be able to decide whether she could have a child that looks like her and her husband, Gibson, also a dwarf.

"You cannot tell me that I cannot have a child who's going to look like me. It's just unbelievably presumptuous, and they're playing God," she said.

But her decision was also based on the fact that the couple's newborn daughter died last year from a dwarfism-related disease called homozygous achondroplasia.

Dwarf couples have a 25 percent chance of having babies afflicted with the lethal condition.

"I'm looking to avoid a fatal condition in my next child," she said. "This is not a light decision. My child died in my husband's arms, and I don't want that to happen again."

But Reynolds ultimately decided against the procedure because her insurance didn't cover it and her age, 39, limited her chances for success. She now plans to adopt a dwarf baby. With Post Wire Services

----------------

This is dangerous moral and legal territory we are treading here. Creating "perfect" designer babies is problematic in its own right, though if it eliminates disabilities and genetic disease, I'd be willing to back it. But do doctors have the moral right to deliberately create babies with disabilities, who will suffer from those disabilities for their entire lives (even if they end up living "happy" lives), just because the parents want it for whatever reason? Do parents have a moral right to request it? Does the government have a moral right to get involved in regulating genetic testing for the purposes of "engineering" designer babies? How about religious laws... where do they stand on genetic engineering of either "perfect" babies or "designer disabilities"? What are the moral, legal and religious reprecussions of this technology?

Elliot

tomder55 answered on 12/23/06:

The most recent research has revealed that more genetic differences exists between people than was previously thought by the Human Genome project ;which would make genetic screening not as accurate as previously thought.Because of this it is difficult to say with any degree of reliabilty what "normal" is when looking at the human genetic bar code.

This makes the practice of making decisions about terminating pregnecies due to allegeded abnormalities frightening and the idea of eugenically manipulating a fetus doubly so.

Damn ;we had to crush the last society that thought it could produce the perfect human ! When are we going to learn ?

ETWolverine rated this answer Excellent or Above Average Answer

Question/Answer
Itsdb asked on 12/22/06 - Failed the litmus test?

A feud over Bush's pick
Warren Bell draws criticism as the Corporation for Public Broadcasting appointee.

By Matea Gold, Times Staff Writer
December 22, 2006

NEW YORK President Bush quietly appointed television sitcom producer Warren Bell to the board of the Corporation for Public Broadcasting this week, overriding opposition from public broadcasting advocates who fear the outspoken conservative will politicize the post.

Bell's nomination had been stalled since September because of concerns about his qualifications among several members of the Senate Commerce Committee, which must approve nominees to the board of the CPB, the private nonprofit that distributes federal funds to public television and radio stations.

But Bush was able to circumvent the need for Senate approval by naming Bell to the board Wednesday evening as a recess appointee. His term will last about a year, unless a permanent nominee for the seat is confirmed before then.

"There had not been action in the Senate on his nomination," said White House spokesman Tony Fratto. "The president felt the need to get it done."

Fratto said Bush believes Bell's 17 years of experience in network television gives him the right credentials for the post.

"He has innovative ideas on making public television more competitive with mainstream media and expressed a very strong commitment to improving CPB," he said.

Bell, executive producer of ABC's "According to Jim," said in an interview that he hopes to strengthen public broadcasting during his time on the board.

He added that the nomination process "has not been a lot of fun."

"The whole thing completely blindsided me," Bell said. "I don't think I was remotely aware of what a hot spot public broadcasting is politically."

Indeed, his appointment was condemned Thursday by some members of the Commerce Committee who said that Bell's lack of public broadcasting experience and his partisan writings for the conservative National Review made him an inappropriate choice.

"This appointment by the Bush administration makes it clear that they simply don't care about the integrity or quality of our public broadcasting system," said Sen. Frank R. Lautenberg (D-N.J.).

As a contributor to the online edition of the National Review, Bell has made no secret of his political views, writing in one piece that he is "thoroughly conservative in ways that strike horror into the hearts of my Hollywood colleagues."

His comments alarmed some public broadcasting advocates, who worry he would plunge the system back into the kind of rancorous debate that erupted last year when Kenneth Y. Tomlinson, then chairman of the CPB board, sought to promote more conservatives in the system. Tomlinson resigned last fall after an internal investigation concluded his actions broke federal law.

"The CPB has had some very substantial controversy attached to it recently, and I think especially because of that the president should have paid attention to some of the concerns in Congress," said Sen. Byron L. Dorgan (D-N.D.), who called the recess appointment "a real serious mistake."

Added Sen. Barbara Boxer (D-Calif.): "The American people made clear on Nov. 7 they wanted bipartisanship from their government, and President Bush once again chose to ignore the concerns of the Senate instead of choosing a consensus nominee."

But Bell, whose appointment is effective immediately, said he will separate his own politics from his role as a board member.

"I'm a comedy writer," he said. "I'm really not nearly a political person as I've been made out to be. I think once people meet me and speak to me and hear what I'm about, they'll see the concerns that were raised were greatly overblown.

"I'm not an ideologue and I'm certainly not Ken Tomlinson," Bell added. "I'm not on a crusade, except to maybe make PBS a really great network for people to watch."

After working for nearly two decades as a writer and producer for sitcoms such as "Life's Work," "Ellen" and "Coach," Bell said he's interested in helping reinvigorate PBS' lineup.

"I certainly think top-quality scripted programming might have a place on PBS the way it has on BBC," he said.

However, the veteran producer will have to contend with a development system vastly different from that of a commercial network. Local PBS stations rather than Hollywood studios produce the bulk of programming, and although CPB provides funding for some shows, it has little influence on what airs. PBS distributes a national slate of programs, which each station then independently decides how to schedule.

Bell's appointment was greeted tepidly by public television officials.

"It is our hope that Warren Bell will make positive contributions to CPB's mission of strong support for public broadcasting," said PBS spokeswoman Lea Sloan. "However, it is important to remember that PBS is a separate and independent organization that is governed by its own board of directors."

John Lawson, president of the Assn. of Public Television Stations, called Bell's nomination a victory of "inside-the-Beltway connections and ideology."

"We hope he will live up to his statements and make a positive contribution to the cause of noncommercial, public service media in the U.S.," Lawson added.

The sitcom producer said he has been stunned by the sharp criticism of his nomination, adding that it is unfair to judge him solely on his National Review columns.

"It was almost entirely people attacking me on the basis of having seen the corner of my shadow going down the hall," he said, saying that taking his comments out of context "makes it sound like I'm Rush Limbaugh with a meat cleaver."

"At one point during the confirmation process," Bell added, "I said, 'I need to go back to Hollywood, where people play fair.'"

~~~~~~~~~~~~~~~~~~~~~~~~~~~~~~~~~~~~~~~~~~~~~~~~~~~~~~

So many comments where does one begin? Our incoming Democrat majority is still confusing obstruction with bipartisanship I see.

17 years as a writer/producer for successful - and critically acclaimed as I recall - shows such as "Ellen" and the extremely funny "Coach" raises "concerns about his qualifications?"

What exactly is a 'consensus nominee' to Democrats anyway?

Is it even possible for a conservative to "make a positive contribution to the cause of noncommercial, public service media in the U.S?"

What constitutes "public service" in the media?

tomder55 answered on 12/23/06:

Funny ;every article I read on this mentioned him as a contributor to' National Review. 'I just did an archive search and found he has done a total of 2 minor articles for them this year.

I have to admit ,I do watch PBS ...wouldn't want to miss 'Antique Road Show'..besides ;According to Fairness and Accuracy in Media(FAIR) .PBS is already too conservative. lol

First order of business is to dump the chump Bill Moyers. Hey ;that would be doing him a favor ....freeing up his time to run for President .


Bush also used a recess appointment to select Mark McKinnon
(an adviser to John McCain )to be a member of the Broadcasting Board of Governors, a little-known but influential federal agency that oversees U.S. government-sponsored news outlets such as the Voice of America . Guess VOA will no longer be a tool of the fifth column fourth estate to propagandize for the enemy at our expense.

Itsdb rated this answer Excellent or Above Average Answer

Question/Answer
Itsdb asked on 12/21/06 - So confusing...

An organization called SaveDarfur is running a comercial saying, "President Bush, stop the genocide," and ran an ad that states:

    We have seen the haunting pictures, heard the cries of grieving mothers. Mr. President, you can end it tomorrow when you speak to the United Nations and the world. You need only stand up and say that the United States and its allies will take decisive action now to protect lives in Darfur. That the United States, and the rest of the world, will move now to deploy the UN force they've already approved. In America the support for action crosses partisan lines. John McCain and Barack Obama, Bill Clinton and Bob Dole support taking action now. Please, Mr. President. Beyond politics, beyond borders, beyond religion, there is the moral imperative to save lives. Mr. President, the world is waiting. The lives of two million people hang on your every word tomorrow.

    Please don't let them down.


Among its executive committe are include Amnesty International, NAACP, Citizens for Global Solutions and International Crisis Group (Working to prevent conflict worldwide), all groups critical of the war on terror and the war in Iraq.

Granted, I'm all for saving Darfur, but how do these people reconcile the fact they're demanding Bush send troops to Sudan to "stop the genocide" while being angry at Bush for doing something about the genocide and other atrocities perpetrated by Saddam Hussein?

My feeble mind doesn't understand.

Steve

tomder55 answered on 12/21/06:

What ? Sudan did not attack us ! What is the plan ? What is the exit strategy ? Is this a war of choice ? Do we have the International Community's support ? The UN ? Where are the blue helmets ? What about our allies ? If we go there does that mean we've taken our eyes off ,and dropped the ball in the war on terror ? I think we should have a much more nuanced approach don't you ? Maybe we should define genocide first (sorta like what is is ).

From American Thinker

It is ironic that the very people who claim to have the most concern for the plight of the innocent men, women, and children being massacred in the Darfur region of the Republic of Sudan are complicit in this humanitarian crisis.


On many levels this conflict represents a failure of the media and liberal ideology with its distain for decisive military action. The government of Sudan is backing the Arab Janjaweed rebels as they slaughter and displace thousands of non-Arab inhabitants of the Darfur region. This conflict represents a government annihilating its own population and an utter failure on the part of the United Nations to intervene. This is a military conflict that necessitates a militarily-imposed solution.


The problem with the solution is that it requires a significant investment of time and troops on the part of any nation or coalition to truly affect change. The United Nations has been shown to be truly impotent when it comes to affecting real change and it will not commit the necessary resources to militarily impose a stop to the on-going genocide in that country. It can pass resolution after resolution but it lacks the will to put any teeth behind its mandates. In times of crisis when the UN fails the world then looks to the United States to assist cleaning up the mess no other country can or will deal with, and herein is where the problem lies.


The US has a track record of attempting to deal with humanitarian crisis like the one in Darfur and in many cases does so quite successfully. The problem arises when the media become involved. In all armed conflicts there will be casualties, which, the media then seize upon and begin to mold perceptions that we are failing as a country. In no time the calls ring out for our capitulation and retreat from an otherwise successful engagement.


People concerned for Darfur in this country chide the US government to help put an end to the genocide but are unwilling to support military involvement. Our leaders have been burned too many times to commit troops into an unstable region against a government-sponsored rebel force. American support is present at the onset of such a campaign but as soon as the first soldier is killed the media start pointing fingers and wringing their hands in panic.


One needs only to look at the Tet offensive in Viet-Nam or the situation in Somalia. The Tet offensive was a stunning military victory for the US and was portrayed as a massive defeat. The media served to break the will of the American people to continue to fight, something the Vietcong could never have done on their own. In Somalia, our forces were wining on the ground and making a difference in ending the human suffering until the "Black Hawk Down" incident when we lost 18 brave Americans. The mission was successful, however, the media portrayal led to a shift in public support for the effort with a subsequent withdrawal of forces. Our withdraw then had the unfortunate effect of emboldening a certain Al-Qaeda leader and we all know what the result of that was.


Why should Darfur be any different? The liberals in this country send out cries of "Help the people in Darfur" and they hold rallies, sign petitions, and involve international courts and organizations to condemn and bring charges against the perpetrators, all the while the body count is racking up and there is no real hope insight. They call on our President to get involved and stop the genocide, but the only way to do that is to go in and take out the Janjaweed rebels and the government that is providing support and build a nation based on freedom and liberty.


Sound familiar? It should as it is the exact mission we are trying to accomplish in Iraq and it is all too evident how it is being portrayed in the media. Darfur is doomed because the very people calling for the United States to save the people of that region are the same ones willing to leave the people of Iraq to wither and die on the vine. What will happen if we do commit to stopping the atrocity in Darfur and something goes wrong? Or the whole process takes longer than one season of the hottest new fall TV show, whose head will the media call for? Will we get reports of an unbeatable rebel insurgency? How long until the natives get restless and call for an exit strategy in Darfur?


It is an absolute travesty that a region that truly needs our assistance, as we are the only nation on the face of the planet that can be of true assistance, will have to suffer through half measures and ineffective humanitarian solutions because the US media has so inaccurately portrayed successful events in American military history as losses that no politician will be willing to electively choose to engage in a conflict of that scope.


The conflict in Darfur is abhorrent and tragic; however, that tragedy is compounded by the fact that the United States has lost the will to act in the face of tyrants, dictators, and thugs, not because we are soft as a people, but because the media has so distorted the truth and sided with our enemies it has become almost impossible to win, even if we do.

Itsdb rated this answer Excellent or Above Average Answer
ETWolverine rated this answer Excellent or Above Average Answer
labman rated this answer Excellent or Above Average Answer

Question/Answer
paraclete asked on 12/20/06 - How easily we forget. So much for international largesse?

Tsunami aftermath littered with donors' broken promises

December 21, 2006

LONDON: About half of the billions of dollars donated to help the victims of the Indian Ocean tsunami two years ago has still not been spent, the BBC says.

According to figures obtained by the broadcaster from a database compiled by the United Nations Department for Aid and Development, several foreign governments have given only a small proportion - and some none - of the money they promised.

Of the $US6.7 billion pledged, about a 10th has yet to be delivered, and only $US3.4 billion has been spent thus far, according to the BBC.

Among the countries that came up with a fraction of what they had promised, China offered $US301 million to help Sri Lanka but has thus far delivered just $US1.28 million, the broadcaster said. Spain pledged $US60 million to Sri Lanka but came up with less than $US1 million, while France said it would give $US79 million but delivered just over $US1 million.

Kuwait had pledged slightly less than $US10 million to the Maldives but has yet to actually hand over any money.

The US has given about 38 per cent of the money it promised, while the European Commission owes about $US70 million, and Britain $US12 million.

Of the $US2.2 billion donated to the Red Cross worldwide in the aftermath of the disaster, about $US1.3 billion had not yet been spent, and of the 50,000 homes promised by the Red Cross, just 8000 had been finished, the BBC said.

The international director for the British Red Cross, Mathias Schmale, defended the organisation's performance. "It sounds bad, but I think it needs to be put into context," Mr Schmale said. "It will take time to spend this money in a responsible manner."

But the UN's special rapporteur on housing, Miloon Kothari, said: "It should really not take this long to build permanent housing."

Agence France-Presse

tomder55 answered on 12/20/06:

According to the Times Online , money sent to the Indonesian province of Aceh to help tsunami survivors is now being used to enforce sharia law.

International aid workers and Indonesian womens organisations are now expressing dismay that the flow of foreign cash for reconstruction has allowed the government to spend scarce money on a new bureaucracy and religious police to enforce puritan laws, such as the compulsory wearing of headscarves.

Some say there are more sharia police than regular police on the local government payroll and that many of them are aggressive young men.


Under those conditions why should I care that pledged money is not going there ? All my life I have seen people of good will donate time after time to see money that they thought was going towards a worthy goal gobbled up by corruption . Massive amounts of cash only serve to allow these corrupt regimes to keep and extend their grip on power.

We cannot simply refuse to help when people starve to death or when tsunamis strike, but we also cannot fund the imposition of shari'a and other kinds of oppression. We need to insist on controlling the delivery of aid to ensure that it reaches the real victims. When we have successfully done so, we have seen the benefits of our generosity reflected in improved lives . When we do not, we become accessories to the violence and abuse .



labman rated this answer Excellent or Above Average Answer
paraclete rated this answer Excellent or Above Average Answer

Question/Answer
paraclete asked on 12/20/06 - He only makes trouble for troublemakers?

Face to face with Iraq's most wanted

McGeough in Baghdad
December 20, 2006

A CONVOY of about 10 unmarked sedans filled with bodyguards arrives at a small, nondescript home in Sadr City. They quickly fan out to throw a cordon around the property. Inside, small talk on the merits of learning Arabic comes to an abrupt halt - the face of death has entered the room.

This is Abu Deraa. To the Americans and to elements of the Iraqi Government he is a mass murderer. But to millions of Shiites, his rampages across Baghdad and beyond are their salvation.

Revered and reviled for his orchestration of thousands of Sunni deaths in a sectarian war that is tearing Iraq apart, Abu Deraa has been dubbed by friends and foes as "the Shiites' Zarqawi". He may top the most-wanted lists for the US and some government officials, but Abu Deraa is held high by Shiites because one of their own now casts a shadow over the land that is as menacing as that of Abu Musab al-Zarqawi, the depraved Sunni terrorist leader who died in a US air strike in June.

Unlike Zarqawi, who used the internet to drive a vast publicity machine that always took credit for his deeds, Abu Deraa has carefully shielded himself, building mystique and a huge following through bloody and merciless terror. This week, however, after a month-long series of tense negotiations through intermediaries, the warlord agreed to meet the Herald in his first face-to-face interview with a foreign reporter.

Small and very quietly spoken, he smokes heavily as he sits cross-legged on a floor cushion. Unsurprisingly, he presents himself as a simple man who is proud to "fight" for his country. "I'll be the first and the last to defend my people and my creed from troublemakers," he says.

In Abu Deraa's book, there are two camps of troublemakers. First, the American-led coalition forces in Iraq, and then a violent and indiscriminating Sunni insurgency which is killing Shiite Iraqis en masse as it is cheered on by Sunni political and tribal leaders and a popular support base.

In our hour-long meeting in a private home in Sadr City, a Shiite stronghold in Baghdad, Abu Deraa says: "You hear how the Shia are dying. But I never act unless I'm sure and have proof from witnesses who swear that my targets have killed my Shia brothers - then I find a solution.

"Shiite families are weak when their sons have been killed - they cannot defend themselves, so they ask for my help."

Abu Deraa has become the most notorious leader of the Shiite death squads that have emerged this year, kidnapping, torturing and murdering their way through whole communities as a tit-for-tat campaign of sectarian cleansing forces thousands of Shiite and Sunni families to flee communities in which their Islamic sect is the minority.

Now he is being hunted to the death by US and Iraqi forces.

But confronted with an American claim that, like half-a-dozen others, he is a fighter who has broken away from the Shiite militia, the Mahdi Army, and now has "gone rogue" with his own killer squad, he insists: "I act only on orders from Najaf [the Mahdi Army base south of Baghdad], and there has been no breach with Moqtada al-Sadr [the leader of the Mahdi Army]."

Challenged on claims that he has ruthlessly killed thousands of Sunnis, his only answer is to invoke a simple Islamic prayer that is a part of the Arabic vernacular - "Inshallah", which means "God willing".

Asked about the Koran's prohibition on killing, Abu Deraa remains defiant: "I'm the first to say it is haram [forbidden] to spill good Iraqi blood, but when it comes to people who plant car bombs to kill our women and children, I'm ready for them."

It is a difficult interview. Abu Deraa is happy for others in the room to respond to questions on his behalf. His own answers come slowly after he ponders his thoughts. At times he seems distracted or agitated; at some moments, even dazed.

Our interview takes place just hours after gunmen masquerading as members of the Iraqi National Army abduct up to 30 civilians from the headquarters of the Red Crescent in central Baghdad. The brazen daylight raid is typical of what is usually assumed to be the work of Abu Deraa - but he refuses to go into the detail of specific operations.

Other such round-ups for which he is held responsible include the kidnapping of the Iraqi Olympic Committee, the mass abduction of about 150 staff and visitors from the Iraqi Education Ministry, and, last Friday, the disappearance of more than 30 people from the city's Sinak auto-repair strip. Usually Shiite victims of these mass abductions are released within 48 hours and, over time, the bodies of the abducted Sunnis are found dumped.

Questioned about a video circulating in Baghdad in which he is seen abducting and personally executing one of Saddam Hussein's lawyers - part of which was published by smh.com.au earlier this year - Abu Deraa is anything but contrite.

He sidesteps the question by urging death for all of the former dictator's legal team and by defending the killers of Khamis al-Obeidi: "They were good mujahideen. Al-Obeidi deserved to die he deserved more than death. None who defends Saddam are honourable men."

Asked if he was responsible for the hundreds of mutilated Sunni bodies recovered from the Al-Sadaa area, Abu Deraa responds first with what he says is a quote from Sadr: "The Sunnis are our brothers in good times and bad."

He goes on in defence of Shiite relations with Iraq's Christians and some Sunnis, before making a declaration that does not constitute a denial of the charge of mass and cold-blooded murder. "I only want the people who kill women and children," he says.

Despite repeated calls from Washington and other capitals for the Prime Minister, Nouri al-Maliki, to act against rampant militias and death squad leaders such as Abu Deraa, this warlord makes it perfectly clear that his work is far from complete.

Like most Shiites, he lumps Sunni insurgents and their supporters under the generic term "Takfiryeen", meaning those who would make Shiite outcasts from Islam.

Are there many Takfiryeen? He answers: "There are many, too many. There is no solution for Iraq - now it is in God's hands."

The power has failed and we're sitting in the dark when Abu Deraa decides the interview is over. He orders us out into the night - back through the tense streets of Sadr City and more than 20 Mahdi Army checkpoints before we arrive in downtown Baghdad.

tomder55 answered on 12/20/06:

aka Ismail al-Zerjawi ;The guy is a sadistic bastard murderer .Let's stop the pretext about anything noble in what he does . His MO is to drill holes in the head of his victims. I do not buy into him being 'disowned' by Muqtada al-Sadr either .More likely he is operating with al-Sadr's approval and support .al-Sadr does not want his name associated with the murders;just like his role in the homicide of Imam Abdul Majid al-Khoei...or for that matter ;the bombing of the golden dome al-Askariya shrine in Samarra.

There were rumors earlier this month that Abu Deraa had been killed but I have found no reliable confirmation and since Paul McGeough alleges to have interviewed him since then, the rumors are probably false.

paraclete rated this answer Excellent or Above Average Answer

Question/Answer
paraclete asked on 12/20/06 - Well that's a (tax) relief?

Iraq kickbacks tax deductible
December 20, 2006 - 12:57

A tax office investigation has found the $390 million AWB funnelled to Saddam Hussein's regime were not bribes and could rightly be claimed as tax deductions.

The Australian Taxation Office launched an inquiry in August after claims emerged at the inquiry into the Iraqi kickbacks scandal about how AWB treated the payments in its accounts.

Commissioner Terence Cole, QC, heard suggestions that AWB claimed as a tax deduction the kickbacks it funnelled to the Iraqi dictator's regime in breach of the United Nations' oil-for-food program.

Following the tax office investigation, AWB issued a short statement today saying it had been cleared of any wrongdoing.

"AWB has received written confirmation today from the Australian Taxation Office (ATO) stating that the ATO has finalised the AWB Group business audit for the years ended 30 September 2000 to 2004 inclusive in relation to payments under the oil-for-food programme," the company said.

"The ATO accepts that for the reasons set out in the Cole inquiry report payments made by AWB under the United Nations Oil-for-Food program do not constitute bribes to foreign public officials for the purposes of the Income Tax Assessment Act 1997."

AWB's former chief financial officer Paul Ingleby told the Cole inquiry earlier this year that the wheat exporter had claimed up to $300 million in kickbacks as a tax write-off.

Under questioning by counsel assisting the inquiry John Agius, SC, Mr Ingleby agreed that the payment of "trucking fees" - the kickbacks demanded by Saddam's regime - was treated by AWB as an expense and therefore a tax deduction.

An ATO spokeswoman said she could not comment on the inquiry into AWB's tax affairs.

"Under the (income tax and privacy) law we can't comment on individual taxpayers and that includes companies," she said.

"But companies or individuals are free to talk about it themselves."

It is not the only favourable decision by the ATO for AWB and its former executives.

Disgraced AWB figure Trevor Flugge escaped paying tax on almost $1 million in salary and allowances paid to him by an Australian government aid agency for his work in Iraq after the Howard Government shepherded a new tax law through Parliament late last year.

The law, exempting Mr Flugge and scores of other Australia aid workers in Iraq from Australian income tax, was cleared through Parliament just weeks after the Prime Minister announced the Cole inquiry into the AWB scandal last December.

As a result, Mr Flugge earned $978,776 tax free from AusAID after being hired by the Howard Government to work for Iraq's occupation government in April 2003.

Mr Flugge was severely criticised in the Cole report for being deeply involved in AWB's payment of almost $300 million in kickbacks to Saddam Hussein's regime under the UN's oil-for-food program before the war.

AAP with Marian Wilkinson
+++++++++++++++++++++++++++++++++++++++++++++++

It's nice to know that double taxation won't apply here, but I'm sure this rubs salt into the wounds of the US exporters who erronously claim that the AWB somehow cheated them out of the Iraqi market, afterall, they would have been trading with the enemy if they had been involved.

tomder55 answered on 12/20/06:

so in other words a kickback bribe to Saddam Hussein becomes a legal tax deduction. It was just a matter of the cost of doing business with Saddam.

Well that solves everything . They violated UN sanctions to the tune of $235 million ; but since bribing Saddam was not illegal in Iraq then AWB is exonerated . Somehow the logic of that argument escapes me.It is beyond comical for the Cole Commission to characterize the payments as "tariffs".

As Labor Party Treasury spokesman, Wayne Swan correctly points out :
'There is a loophole in the Tax Act which is big enough to drive wheat truck through and what we see today ... is the government giving AWB a pat on the back, a financial reward for their bribes,'

Glad to see you have no problem reimbursing AWB $90 million of your tax money for their scam .But my real question is : if all was up and up with the deal ,then why is the gvt. putting together a task force [headed by Peter Donaldson, a retired federal police officer who specialised in organised crime and worked as a chief investigator for Cole]to investigate possible criminal and corporate charges against former AWB executives who were involved in it ?

paraclete rated this answer Excellent or Above Average Answer

Question/Answer
paraclete asked on 12/20/06 - Speaking, of course, out of his generosity of Spirit?

This from the man who bulldozed the homes of hundreds of thousands of second class citizens, His people would do well to live as well as the Australian aboriginees, who, of course, are free to buy alcohol or not, being free citizens who have the same or better entitlements to welfare than the average citizen and the same rights as any other citizen. I wonder if all citizens of Zimbabwe are free to do as they please?

Mugabe scathing on Australia
16.12.2006. 14:18:53


Zimbabwean President Robert Mugabe launched a stinging attack against Australia at his ruling party's annual convention.

In a speech to ZANU-PF delegates, Mr Mugabe said Aboriginal Australians had been dehumanised and were dependent on alcohol.

Mr Mugabe addressed around 4,000 delegates at the convention at a school at Goromonzi, about 30 kilometres east of the capital Harare.

During the speech he also called for unity in the troubled southern African nation which is facing its worst economic crisis since independence from Britain in 1980.

Mr Mugabe, 82, decried rivalry and divisions in the ruling party over economic policies, corruption and factions allegedly vying to succeed him.

He used the example of Australia to poor scorn on the UK's colonial history, saying that the British had sent murderers and robbers there who had embarked on a campaign to destroy the native Aboriginal people.

"Just now the Aborigines in Australia are no people at all. They have been dehumanised. The Australians started a campaign of destroying them, eliminating them, and those they didn't eliminate they gave alcohol. And today they cannot do without alcohol. The few of them who are there, they are a sorry sight," Mr Mugabe said.

He also attacked Commonwealth Secretary-General Don McKinnon who said recently that Zimbabwe's political and economic crisis showed no signs of abating.

tomder55 answered on 12/20/06:

His land redistribution plan destroyed the countries agriculture base . Now Zimbabwe counts on the generosity of other nations to feed it's starving population . He has treated the majority population in his country far worse than Australia has treated it's neediest . Native Zimbabweans shutter at the word "gukurahundi" ....Originally an agriculturual term for spring rains that wash away chaff, it now refers to Mugabe's 5th column goon squad which has practiced it's own "cleansing ". Over 20,000 people have been "cleansed " .Mugabe's crimes were virtually ignored at the time as everyone focused on South Africa.

Since Australia is lobbying to get a criminal indictment of crimes against humanity against "Bob the bulldozer"I can see why he would have a beef against Australia.

Itsdb rated this answer Excellent or Above Average Answer
paraclete rated this answer Excellent or Above Average Answer

Question/Answer
Itsdb asked on 12/18/06 - I love science

...and reporters. In the most ridiculous item of the day:

Scientists Study Human Olfactory Ability

By MALCOLM RITTER
AP Science Writer

NEW YORK (AP) -- By studying blindfolded college students who crawled through grass to sniff out a chocolate-scented trail, scientists say they've found evidence of a human smelling ability that experts thought was impossible.

The study indicates the human brain compares information it gets from each nostril to get clues about where a smell is coming from. And it suggests dogs, mice and other mammals do the same thing, contrary to what most scientists have thought.

People compare signals from each ear to locate the source of a noise. But the prevailing notion has been that mammals can't follow the same strategy for smells, because their nostrils are too close together to get distinct signals.

"We debunked that," said Noam Sobel of the University of California, Berkeley, who reported the new results Sunday with graduate student Jess Porter and others on the Web site of the journal Nature Neuroscience. The work will appear in the journal's January issue.

The report isn't the first to suggest the two-nostril idea. But Sobel and colleagues have now "opened the doors for full consideration of it," said an expert familiar with the work, neuroscientist Charles Wysocki of the Monell Chemical Senses Center in Philadelphia.

The new paper reports five experiments. One tracked tiny particles used in theatrical fog to show that each human nostril really does sample a distinct region in space.

But most of the paper focuses on what a group of undergraduate psychology students could do on a patch of lawn on the Berkeley campus.

One outdoor experiment was designed to see if people could use just their noses to follow a 30-foot-long trail of chocolate scent, which traced a dogleg course through the grass.

The trail was created with scented twine. But the 32 participants were blindfolded and equipped with thick gloves, kneepads and elbow pads to make sure they couldn't see or feel it. They also wore earmuffs.

Before they began, they were shown a video of proper scent-tracking form, which requires putting the nose on the ground. "People don't really want to do that," Porter said.

Two-thirds of the participants succeeded in following the scent. But when they tried it again with their noses plugged, nobody could do it.

Another experiment found that people got better with practice. Yet another experiment, with 14 participants, found that the volunteers did better if they used two nostrils than if one nostril was taped shut. They succeeded 66 percent of the time with two nostrils, versus 36 percent with one nostril.

But did that really mean their brains were benefiting from two independent signals? Maybe both signals are the same, but the olfactory system just works better if it gets input from both nostrils. Or maybe the real explanation is that people take in less odor with one nostril than with two, giving a weaker signal.

To sort that out, researchers retested four of the participants who'd gone through the practice sessions. This time, the subjects wore devices over their nostrils that controlled the airflow into their noses.

One version of the device was basically an extension of a normal nose, with two holes to sniff through, each supplying air to one nostril. The other version had only one hole. It took in the same amount of air as the first version, but simulated the effect of having only one big nostril.

The participants were less successful and slower when they had the equivalent of one nostril. That supports the idea that people benefit from having two, researchers said.
~~~~~~~~~~~~~~~~~~~~~~~~~~~~~~~~~~~~~~~~~~~~~~~~~~~~~~~
Bwaaaahaahaaaaa! So it took an experiment to determine that humans can follow a scent - unless their nose is plugged - and two working nostrils are better than one. I wonder how much government grant money went into that one?

Here's some research of my own at no cost to anyone else to debunk "the prevailing notion...that mammals can't follow the same strategy for smells, because their nostrils are too close together to get distinct signals."

The nose of a 13-year-old male Caucasian human:


The nose of an explosive detecting German Shepherd:


Gee, they look roughly the same distance apart. And as for whether or not humans can sniff out a trail, I suppose researchers have missed stories like this. And they wonder why some of us don't jump on their global warming, evolution and when is a child a child bandwagons?

tomder55 answered on 12/18/06:

I hear you can track the Mahdi-hatter with both nostrils of your proboscis stuffed with super glue and stucco. University of California, Berkeley heh ? Normally the line they track with their noses is white .

Itsdb rated this answer Excellent or Above Average Answer

Question/Answer
Choux... asked on 12/17/06 - US COUNTERINSURGENCY MANUAL PRINTED ON WEB

"Our IraqSlogger editors are stunned that the Pentagon has released to the entire world and posted on the Web the U.S. militarys new 282-page counterinsurgency war-fighting manual.

This is the first post-9/11 war on terror-era U.S. military counterinsurgency manual the long-awaited doctrine meant in part to help turn the tide for U.S. forces in Iraq and Afghanistan.

The counterinsurgency field manual's cover reads in part, "Distribution Restriction: Approved for Public Release; Distribution is unlimited."

Why?

Now you and everyone, including Al Qaeda terrorists and insurgents, can read the entire 282-page manual.

It's posted on multiple military Web sites.

While the manual doesnt contain classified secrets, it contains an astounding amount of seemingly sensitive military doctrine, with subject headings including:

-- INTELLIGENCE, SURVEILLANCE, AND RECONNAISSANCE OPERATIONS

-- HUMAN INTELLIGENCE AND OPERATIONAL REPORTING

-- COUNTERINTELLIGENCE AND COUNTERRECONNAISSANCE

-- INTELLIGENCE COLLABORATION

-- INTELLIGENCE CELLS AND WORKING GROUPS

-- PROTECTING SOURCES

-- EXECUTING COUNTERINSURGENCY OPERATIONS

-- TARGETING

Should such sensitive and detailed information be dished up to the U.S.s enemies, especially via Pentagon Web sites?

In the manual's foreword, Lt. Generals David Petreaus and James Amos write in part, "With our Soldiers and Marines fighting insurgents in Afghanistan and Iraq, it is essential that we give them a manual that provides principles and guidelines for counterinsurgency operations."

How would a U.S. soldier or Marine now in Iraq or Afghanistan feel knowing the hot-off-the-presses counterinsurgency manual is available to the bad guys at the same time it is available to the good guys?

Granted, the Pentagon posts a lot on the Web, but it seems fair to ask whether this entire 282-page document (surely, there are additional classified components) should be made available to all.

Will the manual be of value to Al Qaeda terrorists, Iraqi insurgents, and the Taliban?

Would the U.S. think it had scored an intelligence coup if it got its hands on the insurgents' 282-page field manual?

How would the American people respond if asked in a poll whether the U.S. military's counterinsurgency manual should be shared with the insurgents?

"Your IraqSlogger team will keep an eye out on terrorist and insurgent Web sites to see whether they provide links to Pentagon Web sites providing the counterinsurgency manual -- or whether they go so far as to translate the manual into Arabic and other languages.

Stay tuned."

~~~~~~~~~~~~~~~~~~~~~~~~~~~~~~~~~~~~~~~~~~~~~~~


What the eff?

tomder55 answered on 12/17/06:

Maybe they wanted it released before it got leaked ?

Here is the link for 'FM 3-24/FMFM 3-24'. Read it yourself to determine if there is anything useful for the enemy . I kind of doubt it. The terrorists are concerned with our stupidity. Our failure to take common-sense measures due to political correctness,, our bizarre fascination with propagandizing against ourselves.The military doesn't need the manual as much as our MSM does so they can realize how their efforts harm us.

Consider that the MSM is reporting this as the military catching up....a good thing right ?Yet the same media in report after report is condemning Don Rumsfeld for his transformation doctrine.

The MSM demands a level of perfection by the military that the MSM could never itself perform at. The American people expect and accept a very high level of incompetent in their government but are surprised to discover frequently war does not work the way it is mapped out in the strategy room .Our foes count on us getting discouraged with it all and going home.

Counterinsurgency is a long process that is winnable .It does not require massive troop presence but it does require patience.That is why the "Generals" don't want to do it .They'd rather go in ;kick butt ,and have an "exit strategy " in place before the first shot is fired . Counterinsurgency works by making the local political structure strong enough to contain or break the Insurgency;and that takes time.We will lose because the American people do not have the patients for that .

Right now out of 18 provinces in Iraq only 2 are considered "not ready" for transition to Iraqi control. The two provinces are Anbar, the province along the Syrian border and Basrah along the border with Iran. (get the picture ? )In both those areas significant progress had been made prior to the election .But I think the American election was a serious setback in the war effort.

Choux... rated this answer Bad/Wrong Answer
ETWolverine rated this answer Excellent or Above Average Answer
Itsdb rated this answer Excellent or Above Average Answer

Question/Answer
Choux... asked on 12/16/06 - YOU GO TO POSTERITY WITH THE REPUTATION YOU HAVE

NOT THE REPUTATION YOU WISH YOU HAD.

"The Associated Press noted the departure of Donald Rumsfeld with a curious retrospective, quoting a biographer who suggests that he is a "tragic figure" because of his wasted "talent and promise." But Nixon, who called him a "ruthless little bastard," had Rummy's number from the start. His "talent" was as a political hit man, a vicious insider who would do whatever his bosses wanted.
He was and is a nasty person of shrewd but limited intellect, a bully and a braggart and a bullshit artist. Nobody will miss him.

Do Rumsfeld and his friends regret his public disgrace, which will follow him into the grave? Too bad. They should have thought about that earlier, when he was slandering generals and political opponents alike (for being right), enriching war profiteers, and making one egregious error in judgement after another. No wonder he won praise today from Dick Cheney, a man whose predictions on Iraq have been as accurate as his shooting.

The only thing that could change the way Rumsfeld is remembered is if more is revealed about his complicity in torture and other war crimes.

What should Rumsfeld be remembered for, if not Iraq? For selling contraceptives and sweeteners as a drug executive? No. The GOP wanted this war, and Rumsfeld gave it to them. They wanted lying and deception, and Rummy delivered. They wanted to cut costs when it came to protecting our soldiers, and to jack them up when it came to making Halliburton rich. Again, Rumsfeld came through.

The only form of combat at which Rumsfeld ever excelled was bureaucratic infighting. That, and not expertise or brains, is why Nixon named him to a Cabinet post. "I need a man who will be in there fighting," Nixon said on the White House tapes. "He's a ruthless little bastard ... He's tough enough that if he knows what I want, he isn't going to come in and try to sell me something."

Rumsfeld knew how to get things done - particularly things that advanced Rumsfeld's career. He was appointed to direct the Office of Economic Opportunity so that the GOP could run it into the ground, but he proved an aggressive and adept advocate for some of its programs. That wasn't out of idealism, but rather as a way to expand his own turf.

As Secretary of Defense, he increased the military budget during a period of dtente and reduced military need. Why? Because - again - he wanted more power.

His first private-sector job, as CEO of G. D. Searle, was well-suited to his talents. He cut underperforming divisions, per the corporate trends of the day. (Some business analysts believe this tactic, while good for short-term stock values, actually guts the long term worth of some companies while making employees suffer needlessly.)

His political skills came in especially handy at Searle's helm, since he was able to persuade the Reagan Administration to reverse government policy and permit the use of Searle's formerly-banned product, Aspartame.

(Rumsfeld continues to profit from the decisions of his political pals. During the bird-flu scare Bush allocated a billion dollars to purchase Tamiflu, which another Rumsfeld company developed. The result was a few more million dollars in value for Rummy's portfolio.)

But Iraq will remain the capstone of Rumsfeld's career. He treated the lives and welfare of our soldiers as cavalierly as he did the jobs of employees at those Searle divisions he closed down. His limitations, both intellectual and moral, made him the Republican Party's perfect instrument for the pursuit of this war. He was the creature of the Party that created and nurtured him, and an accurate reflection of it.

His most famous quote was not only flippant but dishonest, since it was used to conceal his own managerial incompetence, lack of proper planning, and indifference to the human cost of his actions. Let's not forget the question that prompted it, either, from a soldier serving in Iraq:

Army Spc. Thomas Wilson: Why do we soldiers have to dig through local landfills for pieces of scrap metal and compromised ballistic glass to up-armor our vehicles? And why don't we have those resources readily available to us?

Rumsfeld: It isn't a matter of money. It isn't a matter on the part of the army of desire. It's a matter of production and capability of doing it. As you know, ah, you go to war with the army you have--not the army you might want or wish to have at a later time.--You can have all the armor in the world on a tank and it can (still) be blown up...

Rumsfeld's press conferences were widely noted for his bullying, confusing, and often incoherent comments. What was less obvious to most press observers was that his elliptically-phrased aggression was an intentional strategy. He kept reporters confused, intidimated, and off-balance while showering the public with his muddled thinking, cynical manipulations, and flat-out lies.

The content of the AP piece is generally fair and balanced, although they turned to a Cato Institute scholar rather than one of his many progressive detractors for the observation that he will be remembered with a "dark epitaph."

But the AP's lamentation for the fact that his career "ended in ignominy" is a curious one. He will be remembered, if at all, for these qualities: callousness, libelous comments about those who disagreed with him, a hallucinogenic detachment from reality, smug refusal to consider other people's opinions, mental shallowness, and a sociopathic inability to take responsibility for his own actions.

Most of all, he will be remembered for that most destructive and personally unappealing combination of personality traits: arrogance and incompetence.

Given that record, what could be a more appropriate end to his career than "ignominy"? RK Eskrow, Blogger

~~~~~~~~~~~~~~~~~~~~~~~~~~~~~~~~~~~~~~~~~~~~~~~~~

tomder55 answered on 12/17/06:

Let's see ; Lincoln and his Sec War Stanton went to war with some raw recruits using Napoleanic tactics ;by the end of the war ,the Generals had been replaced with Generals who wanted to fight the war how Lincoln and Stanton wanted them to execute it. They learned to not submit to the will of the military establishment ,but rather imposed their will on the military to do the bidding of the civilian leadership . One imagines that they were strong willed and stubborn .I know fron the historical record that they alienated many of these generals ;the most celebrated of whom,George B. McClellan ,went so far as to challenge Lincoln for the Presidency in 1864.

I think it is a glowing tributes to President Bush ,Don Rumsfeld and the subordinates up and down the chain of command [that Rummy bullied] ,that 4 1/2 years after the attack on America Sept. 11,2001 ,that the critics can now mutter such nonsense that the threat must not have been as great as we mistakenly first believed.But then again ,the critics have to make that argument lest they are compelled to admit that the disasterous policies of the previous administration had put the nation in such danger .

But these are the same people who would just as soon see Pax Americana end. They no longer want the country to have the burden of defending the free world. They would just as soon see an emerging China become a counterweight to US strength (since by now even they realize that EU has no desire to assume their share of the burden).

Rumsfeld realizes that the US cannot afford to even appear to be weak. During his farewell address Friday he said :

Today, it should be clear that not only is weakness provocative,but the perception of weakness on our part can be provocative as well. ......A conclusion by our enemies that the United States lacks the will or the resolve to carry out missions that demand sacrifice and demand patience is every bit as dangerous as an imbalance of conventional military power

And Rumsfeld is right! Indeed ,this is what OBL has written :

"But your most disgraceful case was in Somalia; where- after vigorous propaganda about the power of the USA and its post cold war leadership of the new world order- you moved tens of thousands of international force, including twenty eight thousands American solders into Somalia. However, when tens of your solders were killed in minor battles and one American Pilot was dragged in the streets of Mogadishu you left the area carrying disappointment, humiliation, defeat and your dead with you. Clinton appeared in front of the whole world threatening and promising revenge , but these threats were merely a preparation for withdrawal. You have been disgraced by Allah and you withdrew; the extent of your impotence and weaknesses became very clear. It was a pleasure for the "heart" of every Muslim and a remedy to the "chests" of believing nations to see you defeated in the three Islamic cities of Beirut , Aden and Mogadishu. "

If this is what Bin Laden said after Somalia, just think what the terrorists will say, if we withdraw from Iraq before the job is done. This is where the Dems and bloggers like RK Eskrow want to take the US: back into Clintonoid weakness, which emboldens terrorists. I leave him to his opinion .I consider Rummy one of our finest Sec Def. ;and certainly one of the finest American public servant ever .Reagan's Star Wars efforts were totally ridiculed during his administration. Today, Democrats are trying to claim these efforts as their own and take credit for that which they tried to kill at the time. The same thing will happen to President Bush's war on terror and Rummy's military transformation program . When the results become more obvious these interlopers will try to claim success when in fact they are disgraceful in their opposition.












Choux... rated this answer Excellent or Above Average Answer
ETWolverine rated this answer Excellent or Above Average Answer
Itsdb rated this answer Excellent or Above Average Answer
labman rated this answer Excellent or Above Average Answer

Question/Answer
Itsdb asked on 12/15/06 - PC police at it again

Carolers told to stop singing at event

RIVERSIDE: After Sasha Cohen ice skates, a city staff member has a choir halted midsong.

12:17 AM PST on Thursday, December 14, 2006

By ROBERT P. MAYER and MARLENE TOSCANO
The Press-Enterprise

While an Olympic-medal-winning ice skater smiled and listened to Christmas carols, a Riverside city staff member silenced the singing group because she was afraid the skater would be offended because she is Jewish.

Sasha Cohen, the 2006 Olympic silver medalist and 2006 U.S. National Champion, had just finished her performance at the Riverside Outdoor Ice Skating Rink on the downtown pedestrian mall and had begun to sign autographs when the Rubidoux High School Madrigals began to sing a Christmas carol.

The choir had barely launched into "God Rest Ye Merry Gentleman" when a police officer and Michelle Baldwin, a city special-events employee, approached choir director Staci Della-Rocco and told her to stop the choir's performance.

Baldwin had contracted with the choir for the event.

Della-Rocco said she complied with the request "because a policeman told me to stop. I didn't want to have a big old huge scene in front of my kids," she said. "I figured I would deal with it later. I would give it some thought and deal with it later."

Amber Eyerly, with the New York-based PR firm that helped promote the event, said that Cohen didn't make the request.

Baldwin could not be reached for comment. City officials referred questions to Development Director Belinda J. Graham, who confirmed the incident.

"This request was simply made by a staff member who was attempting to be sensitive to the celebrity guest, without considering the wider implications...or consulting with her supervisor for guidance," Graham said in an e-mail.

Mayor Ron Loveridge said he was troubled by the incident.

"You kind of wish people do a little checking first. You certainly have my apology," he said, referring to the choir members. "It is unfortunate."

Steve Frasher, spokesman with the Riverside Police Department, said no report of the incident had been filed and that he could neither confirm nor deny that any officer was involved.

"I felt so bad for my kids and that whole situation," Della-Rocco said. "I mean, what kind of a lesson was that? A police officer and a city official telling my kids they can't sing Christmas music?"

Della-Rocco said the students were devastated.

"I just thought it was really rude," said Samantha Moore, a student in the choir. "Everyone basically thought it was really stupid."

Eyerly said she accompanied Cohen through most of the night and that the skater appeared to enjoy the carolers.

"The comical thing is that Sasha wished everyone a Merry Christmas," Eyerly said.

Reach Robert Mayer at 951-368-9455 or rmayer@PE.com

~~~~~~~~~~~~~~~~~~~~~~~~~~~~~~~~~~~~~~~~~~~~~~~~~~~~~~~

We now have city officials - and by some reports police - crashing Christmas programs to prevent someone from being offended by those nasty Christians. Now isn't that special?

I thought we were supposed to celebrating the different cultures in this society - all but the Christian culture? Is that how it works?

tomder55 answered on 12/16/06:

you could tell she was uncomfortable and offended by the way she was smiling at the carollers .Perhaps she wasn't offended ?Perhaps she had heard carols before and likes them ? What do you expect in Kalifornia ?

The NY slimes reported that she was 1/2 Jewish.I'd be willing to bet the other 1/2 is of a Christian heritage .She grew up on Long Island NY. where I can attest that Jews and Christians live side by side very comfortably . There is hardly a house that is not decorated in some festive holiday theme.We grew up singing carols and hanukkah songs during those mandatory winter programs that we hated to perform in .Nobody thought anything about it. It was a saner time.

Cohen's mother, Galina Cohen, said her daughter was "stunned" to learn of the crackdown and believes "Christmas carols are part of celebrating the holiday season."



This article about Cohen makes it very clear that she has gone just about as far as she can in competitive circuit and is instead branching off into celebrity persuits. I do not think she would be into alienating her audiences no matter who they were. Good for her .Nothing wrong with cashing in ;I'm in favor of it.

A possible solution to the Riverside PC squad's dilemna would be to sing some Kosher Christmas carols . Possible selections :

"Deck the Halls With Loaves of Challah"

"Oy Vey, Maria"

"Brisket Roasting on an Open Fire"

"Arrest ye merry carollers , you're in a danger zone.We cannot let you sing that song in front of Sasha Cohen."

Now if you'll excuse me ,I'll put on my copy of the Streisand ; Barry Manilow or Neil Diamond's Christmas CD.One of the best Christmas albums ever was Jewish Phil Spector's classic A Christmas Gift for You from Phil Spector


Perhaps I'll just listen to 'Hark the Herald Angels Sing' written by that Jewish composer Felix Mendelssohn.

ETWolverine rated this answer Excellent or Above Average Answer
Itsdb rated this answer Excellent or Above Average Answer

Question/Answer
drgade asked on 12/15/06 - Is this a thoughtful C & P?

You needn't waste your time answering that. I know it's good....with a little humor.

A New Direction

The Democrats new promise "A New Direction For
America - Vote Democratic"

The stock market is at a new all-time high and
America's 401K's are back.
A new direction from there means, what?

Unemployment is at 25 year lows.
A new direction from there means, what?

Oil prices are plummeting.
A new direction from there means, what?

Taxes are at 20 year lows.
A new direction from there means, what?

Federal tax revenues are at all-time highs.
A new direction from there means, what?

The Federal deficit is down almost 50%, just as
predicted over last year.
A new direction from there means. what?

Home valuations are up 200% over the past 3.5 years.
A new direction from there means, what?

Inflation is in check, hovering at 20 year lows.
A new direction from there means, what?

Not a single terrorist attack on US soil since
9/11/01.
A new direction from there means, what?

Osama bin Laden is living under a rock in a dark cave,
having not surfaced in years, if he's alive at all, while 95% of
Al Queda's top dogs are either dead or in custody, cooperating with
US Intel.
A new direction from there means, what?

Several major terrorist attacks already thwarted by US
and British Intel, including the recent planned attack involving
10 Jumbo Jets being exploded in mid-air over major US cities in order to
celebrate the anniversary of the 9/11/01 attacks.
A new direction from there means, what?

Just as President Bush foretold us on a number of
occasions, Iraq was to be made "ground zero" for the war on terrorism --
and just as President Bush said they would, terrorist cells from
all over the region are arriving from the shadows of their hiding places
and flooding into Iraq in order to get their faces blown off by US
Marines rather than boarding planes and heading to the United States to
wage war on us here.
A new direction from there means, what?

Now let me see, do I have this right? I can expect:

The economy to go South
Illegals to go North
Taxes to go Up
Employment to go Down
Terrorism to come In
Tax breaks to go Out
Social Security to go away
Health Care to go the same way gas prices have gone

*But what the heck !*
I can gain comfort by knowing that Nancy P, Hillory C,
John K, Edward K, Howard D, Harry R and Obama have worked hard to
create a comprehensive National Security Plan, Health Care
Plan, Immigration Reform Plan, Gay Rights Plan, Same Sex Marriage Plan,
Abortion On Demand Plan, Tolerance of Everyone and Everything Plan, How
to Return all Troops to the U.S. in The Next Six Months Plan, A Get
Tough Plan, adapted from the French Plan by the same name and a
How Everyone Can Become as Wealthy as We Are Plan. I forgot the No More
Katrina Storm Plan.

*Now I know why I feel good after the elections.*
*I am going to be able to sleep soooooo much better at
nights knowing these dedicated politicians are thinking of me and my
welfare.

tomder55 answered on 12/16/06:

thanks Doc . I'll remeber these in my daily tilting at windmills trying to convince moonbats the error of their ways.

drgade rated this answer Excellent or Above Average Answer

Question/Answer
paraclete asked on 12/15/06 - Perhaps Bush and Rice can explain this?

Oh, I know, it isn't happening!

Leave it all behind and you'll live

December 16, 2006

Like Bosnia before it, Baghdad is being carved into sectarian territories as thousands of families are forced to flee their homes and livelihoods, reports Paul McGeough.

There is a broken man behind the steel door in al-Salam City. Until early this year, Raad Chasep was a prince of the farming plains in the Euphrates Valley west of Baghdad. But today he and his family are paupers, displaced and distraught as a deepening civil war tears Iraq apart.

On the verge of tears, the 39-year-old describes the transition from their rural mansion to a two-room hovel in this Shiite quarter of Baghdad's north-west: "We had three reception rooms, nine bedrooms and seven bathrooms. We had our own meat, vegetables, milk but tonight we have no dinner."

It literally was a swap. A Shiite, Chasep recounts how a friend introduced him to a Sunni family that needed to flee predominantly Shiite al-Salam City. He describes a numbing meeting at which the exchange was formalised: "I felt as if I was giving away my life and everything that my father built up over 50 years."

The cash-free swap was part of a vicious campaign of sectarian cleansing in Iraq that is fast creating one of the biggest refugee and internal displacement crises in recent time. But much of the flight goes under the international radar because of the manner in which it happens.

The moneyed and professional classes simply pack up and leave. Since the US-led invasion in 2003, nearly 2 million Iraqis have fled to Syria, Jordan and Lebanon where their numbers and, in many cases their money, are now causing new political and social tensions. The United Nations estimates the departure rate from Iraq at 100,000 a month.

The internal displacement is more harrowing. Localised, it starts with unexplained murders in minority quarters, visits by masked men, wall messages and - for those who still don't get it - mortars and rockets lobbed from adjoining neighbourhoods.

Thousands or families are believed to be resorting to house swaps like Chasep's arrangement. It's all done informally, each party agreeing to reverse the exchange when, or if, peace and security allow.

"Maybe in 25 years' time," Chasep complains.

The mainly Shiite district of Hurriyah, on the capital's westside, is the latest to have been near fully cleansed of Sunnis. Last Saturday militiamen of the Shiite Mahdi Army stormed into what locals say was the last pocket of Sunni homes in the area.

Amid a five-hour battle, about 100 Sunni families packed meagre belongings into a convoy which assembled outside the fortified Muhaimin Mosque. Refusing entreaties from the Iraqi Government and offers of protection from the Iraqi National Army, they waited until dusk and slipped out of the district under cover of curfew.

Under Saddam Hussein, an estimated 4000 Sunni families lived in Hurriyah, five kilometres from the sprawling, bunker-like Green Zone - the seat of Iraqi and US government and military power in Baghdad. Today there is said to be none.

As retaliatory attacks on Shiites in adjoining neighbourhoods were stepped up on Sunday, the newly displaced Sunnis from Hurriyah - some of them hooded and armed - marched in a Sunni quarter of the city in protest against inaction by the predominantly Shiite Government and its security forces.

Omar Abdul-Sattar, a spokesman for the Sunni Arab Iraqi Islamic Party, told local television that in the past five months more than 300 Sunni families have been driven from Hurriyah as more than 300 Sunnis have been killed or wounded, five Sunni mosques have been torched and homes and businesses destroyed.

On Monday, a hapless Defence Ministry spokesman was wheeled out to defend the Government's security efforts.

But Mohammed al-Askari seemed to make the Sunnis' point when he told reporters: "We can't deny the presence of the outlaws in Hurriyah who have managed to intimidate residents and force some out of their houses. But Hurriyah isn't the only area where this is happening in Baghdad. It's going on in other neighbourhoods, too, and all Iraqis are being targeted, not only one sect."

Ironically, Hurriyah means "freedom".

It was to Hurriyah that Raad Chasep and his wife and three children fled when they were driven out of Abu Ghraib, left at the mercy of a swap deal and charity from the Shiite mosques of al-Salam City.

In the social order of deeply tribal Iraqi, there are few who stand taller than men of the land, so Chasep's fall on hard times is especially humiliating. Previously a non-smoker, he now lights the next cigarette from the last as he explains: "We lived like kings I used to give orders. On the farm I would kill a sheep for a visitor like you, but now I can hardly afford to give you tea.

"This is so shameful for me; dignity is so important for an Arab. My honour is my home, my money and my reputation - I've lost it all. What is courage and bravery when you have nothing? Farmers don't cry, but now I do because I fear I will have to turn to crime to provide for my family.

"Sometimes, I just want to die."

He recalls a simple life among Sunnis he believed were "friends and brothers" on the Euphrates. The first interruption was the disappearance of a good friend and then, in quick succession, the murder of Ishmail Hamid Ali, his psychiatrist uncle, and of one of his cousins.

"At first I thought that these were revenge killings become of some old disputes, but then the faces of our neighbours changed," he says. "Wall messages ordered us out and then a masked man - whose eyes I recognised as my best friend - came and said that we had to leave in just the clothes we were wearing."

Chasep got his family out, thinking he would be able to return to Abu Ghraib to liquidate his animals and other assets for ready cash. But within days he took a phone call telling him everything had been stripped from his farm and home.

Chasep's family is one of tens of thousands that have been forced to move as mosaic-like, Iraq becomes like Bosnia of the 1990s. Violence is being used to carve out sectarian territories which, in turn, become launch pads for assaults on minorities in neighbouring districts. There is no good or right side; the Sunnis are as brutal as the Shiites and Iraq's significant Christian minority often is caught in the middle.

Going across the city, district by district, to explain the forced flight of Sunnis and Shiites from areas in which they were the minority, a well-informed local concludes: "The last Sunni in Shaab was the school guard, but the Mahdi Army killed him."

Chasep makes the reverse point about Abu Ghraib: "The only Shiites left in Abu Ghraib are the dead and our homes are taken by the Sunnis."

Under Saddam, Sunnis rarely accounted for more that 70 per cent of any local community. But the well-informed local rattles off the districts in which they now account for 95 to 100 per cent. He does the same for the Shiite enclaves; before the invasion they were anywhere between 40 and 75 per cent Shiite; now their numbers range from 70 to 95 per cent.

Simplistically, the division of the city is often described around the notion of the Tigris River as a new Berlin Wall.

But it is more complex. While the west bank, which is called the Karkh, is predominantly Sunni, there are significant Shiite enclaves and the weekend clean-out of Hurriyah is seen as a bid for a purely Shiite corridor that would run east-west across the whole north of metropolitan Baghdad.

The east bank, or al-Rusafa, is dominated by Sadr City, a Shiite stronghold of about 2.5 million that also is home to the Mahdi Army militia headed by the virulently anti-US cleric, Moqtada al-Sadr. More than 200 Shiites died in a series of co-ordinated bombings in Sadr City late last month.

A particular crisis point is right on the Tigris, just north of the city centre, where Shiite Khadamiya is on the predominantly Karkh side and Sunni Adhamiyah is on the mainly Shiite al-Rusafa bank of the river. Explosive tension between the two has forced the close of a bridge link.

But the very location on the western side of the Shiites' gold-domed al-Khadum Mosque and of the Sunnis' revered Abu Hanifa Mosque on the eastern side generates fierce fighting.

This week, the north-east of the city became a new flashpoint. The Mahdi Army blamed Sunnis for a series of car bombings which killed at least 15 people in the predominantly Shiite areas of Kamaliya, al-Obeidi and Baghdad Jadidah on Wednesday.

Claiming the rockets were fired from an apartment building housing Sunni Palestinian refugees in the mixed Baladiyat quarter, the Mahdi Army attacked with mortar fire, reportedly killing five Palestinians.

Like rural flashpoints around the city, much of Baghdad is policed by unofficial militia checkpoints and road blocks, at which Sunnis and Shiites alike risk kidnapping or death on the strength of the sectarian links revealed in their identity papers.

Unlike other Iraqis, Shiites and Sunnis alike, Chasep does not fall back on any romantic memories of life during the dictatorship of Saddam.

But at the same time he fears worse to come: "It is a disaster - Iraq is over, it has been destroyed. We're strangers in our own land. There will be civil war the sectarian fires are burning like never before in Iraq's history."

At first he disowns ideas of revenge, insisting: "Not all Sunnis are bad; not all Shiites are good." But as the conversation turns to the activities of the more notorious warlords now running rampant in Baghdad, the sectarian imperative kicks in: "We can't have peace with the Sunnis. Of course, they must die."

The entrenched tit-for-tat nature of the violence is not lost on him as Chasep reveals the reason for the flight of the owners of the home in which he is now lives.

As his wife, Miasa, makes tea, he takes the Herald to the other room to show the message that told the previous Sunni occupants of the home that time was up for them in al-Salam City - the walls are pockmarked by gunfire from the Mahdi Army and, just around the corner from the house, are the shot-up remains of their family car.

tomder55 answered on 12/15/06:

Are you saying that Yugoslavia was better off under the jack booted iron curtain rule by Marshall Tito ? Perhaps the Indian sub-continent should still be under British rule because when they were no longer in power there was a big displacement .

It can be argued that in transition we should've handed Iraq over to the expatriots like Chalabi so that the Iraqi people would've had time to develop the institutions needed for democracy to flourish. Bush takes his lumps for not forseeing the vaccum that the Iranian client al Sadr has attempted to fill.

Islamism for a long time was the only opposition to ME dictatorships because we in the West ,in the guise of short sighted realism ,propped up the dictators and the hereditary monarchies and did not support democratic forces in the region. Continuing to support dicatorships only strengthened the Islamist movement and increased hostility of us .

I have no doubt that in the long run given the support they need ,the Iraqis will reject Islamism and tribalism. Lets say in a worse case scenario that Islamism becomes the ruling class in Iraq for a while . The next step and last step would be performance and results ;and that is where they will fall flat on their face. The performance of Islamists is not winning them new supporters in Iraq .If anything most of the people are on the sideline waiting to see how things pan out. They would've been more supportive us if we had been more assertive but that is also water under the bridge and a fair critique of our policy there to date .

Ultimately the Islamists cannot provide to the people what they need and desire and just like in Iran ,real democratic movements will emerge in opposition .

The Balkans went through some real rough times after the fall of the iron curtain . My mom for one was a frequent visitor to the area around Sarajevo and witnessed much of the destruction .
You will recall the terrible civil war that ended with the Dayton Accords . They forged a gvt. out of multiethnicity that has today a three-member multiethnic presidency . I think Iraq just like Bosnia will forge their own brand of democracy and will be better off for it .

paraclete rated this answer Excellent or Above Average Answer

Question/Answer
ETWolverine asked on 12/14/06 - Are we responsible for global warming on other planets?

SUV's On Jupiter?
Are humans responsible for climate change on the outer reaches of the solar system, or is it the sun?

Paul Joseph Watson
Prison Planet
Thursday, November 16, 2006

Kofi Annan today slammed global warming skeptics as being "out of step" and "out of time," but how will altering human activity halt climate change when the evidence clearly indicates that the sun itself and not SUV's is heating up the entire solar system?

"The U.N. chief lamented "a frightening lack of leadership" in fashioning next steps to reduce global emissions. "Let us start being more politically courageous," he urged the hundreds of delegates from some 180 member nations of the 1992 U.N. climate treaty," reports Forbes.

But how do we square the fact that almost every planet in our solar system is simultaneously undergoing temperature change and volatile weather patterns. Does this not suggest that global warming is a natural cycle as a result of the evolving nature of the sun? Can Al Gore fill me in on this one?

- Space.com: Global Warming on Pluto Puzzles Scientists
In what is largely a reversal of an August announcement, astronomers today said Pluto is undergoing global warming in its thin atmosphere even as it moves farther from the Sun on its long, odd-shaped orbit.

- Space.com: New Storm on Jupiter Hints at Climate Change
The latest images could provide evidence that Jupiter is in the midst of a global change that can modify temperatures by as much as 10 degrees Fahrenheit on different parts of the globe.

- Current Science & Technology Center: Global Warming on Mars?
A study of the ice caps on Mars may show that the red planet is experiencing a warming trend. If both Mars and Earth are experiencing global warming, then perhaps there is a larger phenomenon going on in the Solar System that is causing their global climates to change.

- United Press International: NASA looks at a monster storm on Saturn
NASA says its Cassini spacecraft has found a hurricane-like storm at Saturn's South Pole, nearly 5,000 miles across -- or two-thirds Earth's diameter.

- Science Agogo: Global Warming Detected on Triton
There may not be much industrial pollution on Neptune's largest moon, but things are hotting up nonetheless. "At least since 1989, Triton has been undergoing a period of global warming," confirms astronomer James Elliot, professor of Earth, Atmospheric and Planetary Sciences at Massachusetts Institute of Technology. "Percentage-wise, it's a very large increase."

- Associated Press: Study says sun getting hotter
Solar radiation reaching the Earth is 0.036 percent warmer than it was in 1986, when the current solar cycle was beginning, a researcher reports in a study to be published Friday in the journal Science. The finding is based on an analysis of satellites that measure the temperature of sunlight.

- London Telegraph: The truth about global warming - it's the Sun that's to blame
Global warming has finally been explained: the Earth is getting hotter because the Sun is burning more brightly than at any time during the past 1,000 years, according to new research.

The simple fact is that throughout the ages the earth has swung wildly between a warm, wet, stable climate, to a cold, dry and windy one - long before the first fossil fuel was burned. The changes we are now witnessing are a walk in the park compared to the battering that our planet has taken in the past.

This is not a defense of the oil cartels or the Neo-Con wreckers, who would have every motivation to ignore global warming whether it is man-made or not.

Nor is it a blanket denial of the fact that the earth is getting very gradually hotter, but how do we reconcile global warming taking place at the farthest reaches of the solar system with the contention that it is caused by human activity? Have our exhaust fumes left earth's atmosphere and slipped through a black hole to Triton?

The assertion that global warming is man made is so oppressively enforced upon popular opinion, especially in Europe, that expressing a scintilla of doubt is akin to holocaust denial in some cases. Such is the insipid brainwashing that has taken place via television, newspapers and exalted talking heads - global warming skeptics are forced to wear the metaphoric yellow star and only discuss their doubts in hushed tones and conciliatory frameworks, or be cat-called, harangued and jeered by an army of do-gooders who righteously believe they are rescuing mother earth by recycling a wine bottle or putting their paper in a separate trash can.

Fearmongering about an imminent climate doomsday also hogs news coverage and important environmental issues like GM food, mad scientist chimera cloning and the usurpation and abuse of corporations like Monsanto flies under the radar.

Global warming is cited as an excuse to meter out further control and surveillance over our daily lives, RFID chips on our trash cans, GPS satellite tacking and taxation by the mile, as well as a global tax at the gas pump.

The extremist wing of the environmentalist movement, characterized by people like Dr. Erik Pianka, advocate the mass culling of humanity via plagues and state sanctioned bio-terrorism, in order to "save" the earth from the disease of humanity. Nazi-like genocial population control measures and the environmental establishment have always held a close alliance.

The orthodox organized religion of global warming and its disastrous consequences for our freedom of speech, freedom of mobility and our right to remain outside of the system, needs to be questioned on the foundational basis that the phenomenon is solar-system wide and it is mainly caused by the natural evolution of the sun and not human activity.

------------

I still question whether global warming is indeed taking place at all. I have been researching at this issue for several years now, looking at global mean temperatures, actual temperatures in specific locations at specific dates, number of heating or cooling degree days per year, and to date I have not found any sort of pattern or trend either upward or downward in global temperatures. And I've been trying.

But assuming there is indeed global warming (a point I am not ready to conceed), the article above shows that it has absolutely NOTHING to do with human activity, industrialization or any other such nonsense.

Elliot

tomder55 answered on 12/14/06:

The Sun warms the planet ? Who would've thunk it ?

Kofi Annan today slammed global warming skeptics as being "out of step" and "out of time

Apparently Kofi doesn't have the time to read his own organization's reports . The U.N.'s Intergovernmental Panel on Climate Change (IPCC) draft report to be released early next year reduces its overall estimate of the human impact on global warming by one-fourth, and halves its predictions for rises in sea-level by 2100.

The IPCC's new figures are attributed to "a refinement due to better data on how climate works."

The panel's report "says that the overall human effect on global warming since the industrial revolution is less than had been thought, due to the unexpected levels of cooling caused by aerosol sprays, which reflect heat from the sun," the paper said.

Furthermore, "large amounts of heat have been absorbed by the oceans, masking the warming effect."

Does this mean Al Gore won't get his Academy Award now?

Sen. Boxer is already saying "see ....elections have consequences !"



ETWolverine rated this answer Excellent or Above Average Answer
Itsdb rated this answer Excellent or Above Average Answer

Question/Answer
HANK1 asked on 12/14/06 - NATIONAL DEBT:



Why do some American citizens worry about our national debt when the Feds own 70% of the real estate in the United States?

HANK

tomder55 answered on 12/14/06:

excellent question ;the assets more than cover the debt. however the issue is repaying the debt. the more your budget is used to service debt the less is available for other services the electorate demands .

HANK1 rated this answer Excellent or Above Average Answer
Itsdb rated this answer Excellent or Above Average Answer

Question/Answer
Itsdb asked on 12/14/06 - Today's Hero: Judge James Robertson

Federal District Judge James Robertson ruled today that the Guantanamo military tribunals are illegal as constituted.

That was November 2004. I wonder if they'll feel the same way about this Clinton appointee today?

Judge sides with Bush administration on terror law provisions

The Associated Press
Wednesday, December 13, 2006
WASHINGTON

A federal judge upheld the Bush administration's new terrorism law Wednesday, agreeing that Guantanamo Bay detainees have no right to challenge their imprisonment in U.S. courts.

The ruling by U.S. District Judge James Robertson is the first to address the new Military Commissions Act and gives a legal victory to the Bush administration at a time when it has been fending off criticism of the law from Democrats and libertarians.

Robertson rejected a legal challenge by Salim Ahmed Hamdan, a former driver for Osama bin Laden whose case prompted the Supreme Court to strike down the Bush administration's policy on detainees last year.

Following Hamdan's victory, Bush asked for and got a new law that established military commissions to try enemy combatants and stripped them of the right to seek their freedom in U.S. courts.

Hamdan's case was sent back before Robertson, a nominee of former President Bill Clinton who was a prominent civil rights advocate as an attorney in private practice.

Though Robertson originally sided with Hamdan, he said he no longer had jurisdiction to hear Hamdan's case because Congress clearly intended to keep such disputes out of federal courts. He said foreigners being held in overseas military prisons do not have the right to challenge their detention, a right people inside the country normally enjoy.

The ruling does not affect the fate of hundreds of detainees whose cases are awaiting a ruling by a Washington appeals court, which is reviewing two precedent-setting detainee cases challenging the new law.

The Justice Department did not immediately have a comment on Robertson's ruling. Messages seeking comment were left with Hamdan's lawyer, Neal Katyal.

tomder55 answered on 12/14/06:

Remember when the law was passed and Excon [and the tinfoil hat crowd] was fearful that we were going to round up legals and put them in secret prisons near area 51 ? Well even if there was a chance of that ,Judge Robertson tossed that part of the law out in his ruling .

He is also a hero to the folks who want us to change the currency so that the blind can tell the difference between the various bills.

I still suspect that ultimately this case will be decided in SCOTUS.

I know who is really unhappy about this ;the Aussies who want this guy David Hicks to get a trial . I say we should just hand him over to them and let them deal with him .

The decision is here

Key wording in the ruling : "Petitioner has never entered the United States and accordingly does not enjoy the implied protection that accompanies presence on American soil."

Itsdb rated this answer Excellent or Above Average Answer

Question/Answer
ETWolverine asked on 12/13/06 - Sen. Johnson Hospitalized for Possible Stroke

Wednesday, December 13, 2006
>Fox News

WASHINGTON South Dakota Democratic Sen. Tim Johnson was admitted Wednesday to George Washington University hospital in Washington, D.C., for symptoms aides say indicate a stroke.

"As this stage, he is undergoing a comprehensive evaluation by the stroke team. Further details will be forthcoming when more is known," said a statement released from his office.

Johnson, who turns 60 on Monday, is already a cancer survivor, having had his prostate gland removed in March 2004. According to news reports, he is also deaf in his left ear as a result of surgery to remove a benign tumor from his eardrum after returning from the Vietnam War.

While the extent of his illness isn't known yet, Washington political watchers are already noting that a Johnson replacement in the Senate would be made by Republican Gov. Michael Rounds.

With the 110th Congress split 49 Democrats, 49 Republicans and two independents that align themselves with the Democrats, majority control of the Senate could shift in such a case.


Johnson was first elected to the Senate in 1996, and defeated John Thune in his 2002 re-election by 524 votes. Thune then joined him in the Senate after defeating former Sen. Tom Daschle in 2004.

-------------

This story is being confirmed by CNN and Drudge. Others are starting to pick this up too, so I'm pretty sure its real.

The Senate may still be up for grabs, folks. Not much detail yet, but this is an interesting development. And for all those who thought that the Dems had a "clean sweep" in the House and Senate, it looks like the final page of the 2006 election is yet to be written.

Elliot

tomder55 answered on 12/14/06:

I agree our prayers should be for a speedy recovery . Your "callousness "does not even come close the the contemptable way the MSM has been all over this story .It was the MSM that jumped on the political implications so now it is fair game to discuss ,even though it would've been preferable to wait a few days to show a modicum of concern for his health and his families welfare .It would not suprise me at all that they are laying the groundwork now to shame Republicans into appointing another Democrat to take Johnson's seat.

I would not be comfortable getting a seat in this manner.Tim Johnson is one of the few good democrats in the Senate, along with Ben Nelson , Kent Conrad ,possibly Mark Pryor and definitely Joe Leiberman(who I would be more than happy to see jump parties ). Tim Johnson was one of only four democrats to vote to confirm Samuel Alito.Losing his vote loses one of the Dems. rationale ties to reality.

There is a history here ;Democrat Govenor of Ga. Roy Barnes, appointed Zell Miller to a U.S. Senate seat following the death of Republican Sen. Paul Coverdell in July 2000.When Republican Senator Heinz died ,the Governor of PA was democrat Bob Casey, Sr. and he appointed a democrat Harris Wofford to replace Heinz. Also after the 2000 election the Democrats had no qualms about Jim Jeffords switching parties and giving them the majority in the Senate ;and the Republicans equally embraced Sen . Richard Shelby (Al) when he switched parties in 1994 .

Another thing to consider is that there are a few RINOs who wouldn't suprise me if they were to jump ship and change party loyalty . Chafee is out ;but Snowe ,Collins and Specter come to mind.

We also owe Johnson a bit of gratitude . He beat Thune to gain his seat ,and in return Thune ran and defeated Daschle .

There is no guarantee that he would give up his seat even if incapacitated . Senator Karl Mundt was incapacitated by a stroke in 1969 and kept his seat for three years.All the Republicans did was strip him from any committee assignment . Simularly you will recall the caning of Sen. Charles Sumner by Congressman Preston Brooks in 1856 . Sumner spent 3 1/2 years recovering but retained his seat and served 14 more years after .Jesse Helms and Strom Thurman were wheeled into the Senate for years ,and Robert Byrd today is barely breathing .

I am trying to find out the specifics of South Dakota law. Some States make it a requirement for a replacement to be of the same party .

Here are some other examples :

Paul Wellstone (D) - replaced by an Independent (same party as Gov. Ventura)

John Chaffee (R)- Replaced by his Son, Lincoln Chafee. (By a Republican Governor).

Quentin Northrup Burdick (D)- Replaced bu his Wife (by a Democratic Governor).

Scoop Henry Jackson (D) - Replaced by Republican Dan Evans, appointed by a Republican Governor.

James Allen (D) - Replaced by his wife, appointed by a Democratic Governor

Hubert Humphery (D) - Replaced by his wife, appointed by a Democratic Governor

Robert Kennedy (D) - Replaced by a Republican, appointed by a Republican Governor.








ETWolverine rated this answer Excellent or Above Average Answer
labman rated this answer Excellent or Above Average Answer

Question/Answer
HANK1 asked on 12/13/06 - What? No golf course?



HOUSTON (MarketWatch) -- Jeffrey Skilling reported Wednesday to a low-security prison in Waseca, Minnesota, where he could be spending the first of many Christmases for his crimes as a top executive of Enron Corp. (End of quote)

Skilling has access to a basketball court and a ping pong table. Ain't that a hoot? The prison grounds used to be a college campus.

HANK

tomder55 answered on 12/14/06:

sounds reasonable . prisoners in higher security prisons have access to simular accomodations including weight equipment to help them get ripped muscles (the better to overwhelm the guards ).

HANK1 rated this answer Excellent or Above Average Answer
Itsdb rated this answer Excellent or Above Average Answer

Question/Answer
paraclete asked on 12/14/06 - Now that's virgin on the rediculous?


Banned for a George Bush T-shirt

By Mark Dunn

December 14, 2006 12:00am
Article from: Herald-Sun

AN Australian was barred from a London-Melbourne flight unless he removed a T-shirt depicting George Bush as the world's number one terrorist.

Allen Jasson was also prevented from catching a connecting flight within Australia later the same day unless he removed the offending T-shirt.

Mr Jasson says Qantas and Virgin Blue were engaging in censorship but the airlines say the T-shirt was a security issue and could affect the sensitivities of other passengers.

"The woman at the security check-in (at Heathrow) just said to me, 'You are not wearing that'," Mr Jasson, 55, said yesterday.

Mr Jasson, who lives in London and was flying to Australia to visit family on December 2, said he was first told he would need to turn the T-shirt inside-out before he would be allowed to board the Qantas flight.

"I told her I had the right to express my opinion," he said.

"She called other security and other people got involved. Ultimately, they said it was a security issue . . . in light of the present situation."

After a prolonged argument about freedom of speech and expression, Mr Jasson said a Qantas gate manager said he could not fly at all unless he wore another T-shirt.

Mr Jasson said his clothing had already been checked in and he was forced to buy a new T-shirt this time with London Underground written on it coincidentally the site of a terrorist attack last year.

"I felt I had made my point and caved in," Mr Jasson said.

But after arriving in Australia, Mr Jasson said he put his Bush T-shirt back on and was again banned from boarding a connecting flight this time a Virgin Blue plane from Adelaide to Melbourne.

"It was argued other passengers could be offended," Mr Jasson said.

"I said it was most offensive that I would be prevented from expressing my political views."

Mr Jasson said the T-shirt often sparked comment from people in the street.

A Virgin Blue spokeswoman said the airline had a policy to ban offensive clothing and bare feet. "Most people use common sense and don't go out of their way to offend people," she said.

tomder55 answered on 12/14/06:

I wonder if he would've been asked to remove the shirt if it had a pix of OBL



or perhaps a Hezbollah flag with a fist holding an AK47 ?


ETWolverine rated this answer Excellent or Above Average Answer
Itsdb rated this answer Excellent or Above Average Answer
paraclete rated this answer Excellent or Above Average Answer

Question/Answer
ETWolverine asked on 12/13/06 - Does she have any substance at all?

In contrast with the last post about Jeane Kirkpatrick, we are now saddled with Nancy Pelosi as Speaker of the House. Soes Pelosi have ANY substance whatsoever? Please note the single common thread that runs through every interview, speech and story written about her.

    I'm a mother of five. I have five grandchildren. And I always say: Think of a lioness. Think of a mother bear. You come anywhere near our cubs, you're dead. And so, in terms of any threat to our country, people have to know we'll be there to preemptively strike.
    ---Nancy Pelosi Interview on NewsHour with Jim Lehrer, Thursday, March 30, 2006

    I have five grandchildren. My husband always says that he'd like to time my speeches to see how long it takes for me to mention my grandchildren. He knows it's going to be there. It's just a question of how soon. They live in Texas and Arizona, which is wonderful for them but a little far for us. My husband and I prayed for grandchildren; we not only prayed, we begged. It was not a pretty sight. But we forgot to pray that they live in California.
    ---A Conversation With Rep. Nancy Pelosi (D-Calif.), House Minority Leader Sunday, August 27, 2006; Page W08


    And if all goes as expected, in less than 100 hours my sixth grandchild will be born.
    I want my grandchild to be born into an America where government is for and by the people. I want my grandchild to be born into an America that rewards and values hard work. I want my grandchild to be born into an America where you are not labeled a terrorist coddler when you honor the Constitution.
    I want my grandchild to be born into an America where if the U.S. Central Command judges the situation in Iraq to be near chaos, with "violence at all-time high, spreading geographically", if the top intelligence agencies tell you that the war in Iraq is inspiring the very terrorism it was purported to prevent, and if four highly respected military newspapers say of the Secretary of Defense that "his strategy has failed, and his ability to lead is compromised...[he] must go" that you fire your Secretary of Defense and change the course.
    I want my grandchild to be born into an America whose government honors its duty for accountability and oversight.
    I want my grandchild to be born into an America that inspires innovation, that leads with dignity and diplomacy, that rejects fear mongering, and whose leaders start each day remembering that the Constitution begins with the simple but revolutionary phrase "We the people," which announced to the world that here, the people rule.
    ---The Huffington Post, One Hundred Hours, Nancy Pelosi blogging, 11/7/06

    "Last week I celebrated by birthday and my grandchildren - ages 4 and 6 - called to sing 'Happy Birthday.' And the surprise, the real gift, was that they sang it in Hebrew."
    ---Jewish Telegraphic Agency November 8, 2006

    I am a mother of five. We have five children and five grandchildren. We are expecting our sixth grandchild in October. And we certainly appreciate the value of family. We see family in our community as a source of strength, and a source of comfort to people. What constitutes that family is an individual and personal decision, but for all, it is a place where people find love, comfort, and support.
    ---Speech on House Floor regarding gay marriage, July 18, 2006

    I'm a mom. I have five children, and I have five grandchildren. I always say to people, 'Think lioness.' This is how Democrats are. You threaten our children -- and that's American -- you threaten our country, you're dead. You're dead."
    --- New Yorker interview, May 30, 2006

    I have five children, five grandchildren; I try to abide by all the teachings of the church in relationship to family. I think my family speaks very clearly to that.
    ---National Catholic Reporter Online interview by Joe Feuerherd, 1/22/03

    As a mother, I know the sacred and blessed choice of life. That decision does not belong to the politicians. It is ours to make - as women, with our families, our physicians, and our faith.
    As a grandmother, I want my children and grandchildren to grow up in a society that respects their rights and values their health. The right to plan for healthy families. The right to privacy. The right to choose. These are fundamental American rights to be preserved for all time, for all generations.
    I am a mother of five, a grandmother of five, and a devout Roman Catholic who supports a women's right to choose.
    ---March for Women's Lives Rally Sunday, April 25, 2004

    Well, I appreciate your saying that and I think one of my first acts as -- post-election, will be to become a grandmother for the sixth time. We're anxiously awaiting the birth of our grandchild, who is due the first week in November, so a good omen. We get ready for our new grandbaby as we get ready for a new Congress.
    ---Interview with Wolf Blitzer on CNNs Situation Room, 11/6/06.


Okay, I get it. She's a grandmother, and therefore knows what's best for her grandkids. (Her kids must love it when she comes to visit... I wonder if she tells her children what they are "doing wrong" in raising her grandkids.) How does that qualify her to be the third in line for the Presidency?

And how does Pelosi "making decisions solely on the basis of being a grandmother" differ from Bush "making decisions based solely on his religious beliefs", as he has been accused of? It seems to me that Pelosi's grandchildren are her religion of choice, as opposed to Bush's Christianity. What's the real difference? I don't happen to agree with that assessment of Bush, but even assuming its true, how is Pelosi any different?

Just wondering.

Elliot

tomder55 answered on 12/13/06:

I wonder if she grasps the inherent sexism in this constant reminder that she is a grandma . We do not refer to male congressmen as grandfathers .Although it may just be a deliberate attempt to seperate her from her San Fran roots . As we know they are not exactly family friendly there ,so it would just be smart politics ;a pre election campaign stategy to soften her image .She also played "the lib goes to church card "resulting in that sound bite that Laura Ingraham plays in the into. to her show :"as we say in church 'let there be peace on earth and let it begin with us'".

From her 60 Minutes fluff piece with Lesley Stahl (early Nov. ):

As the kids got older, Pelosi threw herself into state politics, eventually becoming chairman of the California Democratic party. She didnt run for Congress until she was 46, and now at 66, as shes poised to go down in the history books, what Nancy Pelosi wants you to know is that when it comes to her real goal in life: shes just like any other woman her age.

"It's great. It's fabulous. It was my goal in life and now I've achieved it. I'm a grandmother," Pelosi says laughing.

Ask Nancy Pelosi to describe herself and the first thing out of her mouth is that shes a mother of five and a grandmother of five.

"When I asked your daughter Christine how you rule, she said you were motherly," Stahl tells Pelosi.

"I guess it depends on your definition of motherly. If motherly means well have order in the house, yes," she replies.


later on when commenting about the war she said this bit of brilliance :

"Do you not think that the war in Iraq now, today, is the war on terror?" Stahl asks.

"No. The war on terror is the war in Afghanistan ," Pelosi says.

"But you don't think that the terrorists have moved into Iraq now?" Stahl continues.

"They have," Pelosi agrees. "The jihadists in Iraq. But that doesn't mean we stay there. They'll stay there as long as we're there."

Asked what she would say to Republicans, who have said that Pelosi and the Democrats do not understand the serious nature of the threat, the congresswoman says, "I, as a mother and a grandmother, 14 years on the intelligence committee. Don't tell me I have any underestimation of what the threat is to our country.
(?????????huh ??????)So, if you want to justify your failed policy by saying we don't understand the threat, clearly you didn't understand the situation you got us into."

After her boffo performance on 60 Minutes the Democrats decided that a better strategy would be to make her disappear for a few weeks before she again displayed her brilliant grasp of geo-politics. But these comments don't come from ignorance ,rather they come from true conviction ;a belief in the truism that if only we would leave them alone ,our enemies would leave us alone. (blame America first )I don't understand why she doesn't take that logic to the next level and postulate that if only we were to leave Afghanistan so would the jihadists .By extention she should be calling for us to leave there too.



ETWolverine rated this answer Excellent or Above Average Answer

Question/Answer
HANK1 asked on 12/12/06 - DISGUSTING QUESTION:


I heard on the news this a.m. that 60 Iraqi laborers were killed and 220 were injured by a suicide bomber. That did it for me! Tell me this:

Before we went into Iraq, did anyone conduct an in depth study re: the mentality and religion of the Muslims? I'll have some comments after you answer this question! I'm damn mad!

HANK

tomder55 answered on 12/12/06:

Yes it is horrible and best represents what we are fighting against. Like Steve pointed out ;what is the difference if they blow up the WTC ;with thousands of people who's only fault that day was trying to provide for their families; or if the lure day laborours into a central area and murder them in Baghdad ?

Here is the Reuters report about the massacre .

Interior Ministry sources said 236 people were wounded in the Baghdad blast after the suicide bomber lured a crowd of day labourers to his vehicle with the promise of work. They said 70 people were killed.

The 7 a.m. bombing took place in Tayaran Square, a popular gathering point for construction workers who frequent the cafes and street vendors while waiting for the chance of some work. Many of the workers who gather in the area are poor Shi'ites.

"A driver with a pickup truck stopped and asked for labourers. When they gathered around the car it exploded," said a witness, who was helping a stumbling survivor with a blood- stained bandage covering his head.

"They were poor labourers looking for work. The poor are supposed to be protected by the government," he said.

"HORRIBLE MASSACRE"

Calling the attack a "horrible massacre", Maliki blamed it on Saddam Hussein sympathisers and Sunni Islamist al Qaeda.

"These terrorist groups are trying to spread chaos by killing and fuelling sectarian strife," he said in a statement.

Iraq is gripped by violence between majority Shi'ites and Sunni Arabs dominant under Saddam but now the backbone of the insurgency. Tens of thousands have been killed in what many Iraqis fear is a slide towards all-out civil war.


This has been a pattern for some time now since Zarqwai detailed his strategy to OBL of fomenting a civil war by repeatedly targetting Shias. Security is not easy to provide given the determination of the rejectionists and the opportunity it provides flamers like al-Sadr to exploit the situation .

I dare say that even in the US if a determined group decided to attack "day labourers "standing on main street USA looking for work, and were willing to kill themselves in the effort ,that there would be little that could be done to prevent it .

I also dare to say that it is signals of American lack of steadfastness that prevents those in Iraq who would otherwise fully embrace the life line we've tossed them to fully commit to a free Iraq future .

It is a vicious cycle that is exasperated by the fickless of our comittment. Neither side can count on us being honest brokers because neither side can be certain we will be there after a couple more years . But they know that the Iranians and Syrians (who instigate the violence on both sides )will be there after the last US helicopter leaves. I can't say at this point I blame them . Bush tried to signal that our support for their future was a longstanding commitment but they can read the tea leaves of American domestic politics as well as you and I.


ETWolverine rated this answer Excellent or Above Average Answer
HANK1 rated this answer Excellent or Above Average Answer
Itsdb rated this answer Excellent or Above Average Answer

Question/Answer
Itsdb asked on 12/11/06 - And another sign America is rolling downhill...

4-year-old suspended after hugging teacher's aide

Associated Press

WACO, Texas School administrators gave a 4-year-old student an in-school suspension for inappropriately touching a teacher's aide after the pre-kindergartner hugged the woman.

A letter from La Vega school district administrators to the student's parents said that the boy was involved in "inappropriate physical behavior interpreted as sexual contact and/or sexual harassment" after he hugged the woman and he "rubbed his face in the chest of (the) female employee" on Nov. 10.

DaMarcus Blackwell, the father of the boy who attends La Vega Primary School, said he filed a complaint with the district. He said that his son doesn't understand why he was punished.

"When I got that letter, my world flipped," Blackwell said in a story in Sunday's editions of the Waco Tribune-Herald.

La Vega school district officials said student privacy laws prevented them from commenting.

After Blackwell filed a complaint, a subsequent letter from the district said the offense had been changed to "inappropriate physical contact" and removed references of sexual contact or sexual harassment from the boy's file.

Administrators said the district's student handbook contains no specific guidelines referring to contact between teachers and students but does state that inappropriate physical contact will result in a discipline referral.

The La Vega school district, which has five schools, covers about 30 miles around Waco.


Hmmm, either we now have 4 year old sexual perverts, we have a teacher's aide that is overly sensitive, or sexual harassment laws and rules have gotten way out of hand - or all three?

I say either the kid is innocent or he's a biproduct of Planned Parenthood's "comprehensive sex education" for preschoolers. What do you say?

tomder55 answered on 12/12/06:

good thing the kid wasn't packing any heat ;he really would've been in trouble then. he may have even been suspended.

Itsdb rated this answer Excellent or Above Average Answer

Question/Answer
Itsdb asked on 12/11/06 - Will they ever get it right?

Seems I've heard this before...

Agriculture is major factor in causing global warming

A recent report by the United Nations Food and Agriculture Organization Livestock's Long Shadow - Environmental Issues and Options, warns of the dire environmental consequences of the world's growing meat and dairy production.

According to the report, animal agriculture uses 30% of the Earth's land surface for pasture and feed crop production. It is the driving force in worldwide deforestation and wildlife habitat destruction, with 70% of the irreplaceable Amazon rain forest turned into pasture.

Eventually, pastures are degraded into desert through overgrazing, compaction and erosion. Animal agriculture contributes more pollution to our waterways than all other human activities combined.

Principal sources are animal wastes, as well as soil particles, minerals, organic debris, fertilizers, and pesticides from feed cropland. Most of the world's water supplies are used for irrigating animal feed crops.

Animal agriculture is also a key source of manmade greenhouse gases responsible for global warming.

Sixty-five percent of nitrous oxide, the most damaging of these, is emitted by animal waste, according to the FAO report, and 37% of methane comes from cattle's digestive process. Operation of farm machinery and transport trucks account for 9% of carbon-dioxide emissions.

We don't have to wait for Earth Day to help save our planet. We can start with the next trip to the grocery store.


Ok, before I get to the problem I have to question this:

    Animal agriculture is also a key source of manmade greenhouse gases responsible for global warming.


So cow patties are now manmade?

Well, the solutions are obvious, and when I say "we" I mean Americans.

First we must enact an immediate ban on bovine intercourse and eat more beef to drive down the supply.

Second, once the beef supply has dwindled to a trickle all dairy products will be moved to the controlled substance list. Any remaining nonessential cows will be sent to predominantly Hindu countries.

Third, we must all then take a vow of poverty as the growth in meat and dairy product consumption is due to increased prosperity.

Fourth, rodeos will be restricted to using mechanical bulls and stick horses.

Fifth, the World Cow Chip Throwing Contest will switch from using cow chips to more environmentally friendly tofu patties.

Sixth, an extensive cow chip harvesting campaign will be enacted with cow chip collection centers being set up all across the nation. The collected cow chips will then be used as fuel for the UN building. The benefit of using them here is obvious - there is a ready supply of BS once the cow chip inventory is exhausted.

Oh, and this Texas ranch must be an example of how "pastures are degraded into desert."

tomder55 answered on 12/12/06:

PITAs having a good day . There is a perverse logic in this argument . Consuming animals is a secondary source of food regarding energy consumption . It takes energy to grow the cattle feed ,and then additional energy to slaughter and prepare the steaks. If we would just graze it would be a much more efficient process. The wackos will not be happy until we go back to our roots;living in caves ;forging for berries and eating insects for our protein source.

Itsdb rated this answer Excellent or Above Average Answer

Question/Answer
Choux... asked on 12/10/06 - IT'S GLOBAL TERRORISM, STUPID!

"When the Iraq Study Group's report was unveiled this week, it was like the opening of a blockbuster movie, with reporters counting down the minutes until it was released. But now that all the hoopla has subsided, all we are left with is a Washington inside job: a report written by Washington insiders, for Washington insiders, who share the same mindset that led us into the misguided war in Iraq.

The Iraq Study Group essentially sees Iraq the same way that most of official Washington does - as the be-all and end-all of our foreign and national security policy. Nothing could be further from the truth. Any decisions about our Iraq policy must be guided by our top national security priority: defeating the global terrorist networks operating in countries around the world. We cannot look at Iraq in isolation; we need to also be looking at Somalia and Afghanistan and the many other places around the world where we face grave and growing threats.

The report has some good recommendations, including its call for the U.S. to step up diplomatic efforts with countries like Iran and Syria. But many of its recommendations perpetuate the Iraq-centric policies that have failed so miserably. They fail to correct the course that the American people rejected at the polls in November.

The recommendation that we embed our best troops in the Iraqi army, for example, might seem like a good idea in isolation, but what about our critical effort to fight a resurgent Taliban and al Qaeda in Afghanistan, the country that was the staging ground for the 9/11 attack? Our ongoing efforts in Iraq are straining our military and limiting our capacity to effectively pursue the fight against terrorist networks around the world.

By redeploying our troops from Iraq, we can pursue a new national security strategy that will make our country safer. We can finish the job in Afghanistan. We can scrap the "transformational diplomacy" this Administration has used to offend, push away, and ultimately alienate so many of our friends and allies, and replace it with an aggressive, multilateral approach that would leverage the strength of our friends to defeat our common enemies.

And we can repair and infuse new capabilities and strength into our armed forces. By freeing up our Special Forces assets and redeploying our military power from Iraq, we will be better positioned to handle global threats and future contingencies. Our current state of readiness is unacceptable and must be repaired. Our National Guard, too, must be capable of responding to natural disasters and future contingencies.

The way to win a war against global terrorist networks is not to keep over 140,000 American troops in Iraq indefinitely. We will weaken, not strengthen, our national security by continuing to pour a disproportionate level of our military and intelligence and fiscal resources into Iraq.

Unfortunately, while the Iraq Study Group's report recognizes that the Administration's policy is not working, it doesn't correct the myopic focus on Iraq that has so dangerously weakened our national security. In the end, this report is a regrettable example of 'official Washington' missing the point. The growing threats we face in places like Afghanistan and Somalia are every bit as important to our national security as Iraq. Until Congress and the Administration recognize that, we will only perpetuate the deeply misguided policies that got us into Iraq in the first place." Senator Russ Feingold, [D. Wisc.]

~~~~~~~~~~~~~~~~~~~~~~~~~~~~~~~~~~~~~~~~~~~~~~~~~

tomder55 answered on 12/11/06:

Glad to see that Feingold agrees with me that Iraq has to be viewed in the context of a larger war agaisnt jihadistan . He considers it a distraction ;I consider it integral . He thinks the war is ,in his words ,defeating the global terrorist networks operating in countries around the world. I agree but also think it is imperative that we also engage the nations that support the jihadists and radical Islam (neither side of the aisle is yet willing to acknowledge this basic fact adequately ).

The report has some good recommendations, including its call for the U.S. to step up diplomatic efforts with countries like Iran and Syria.

I on the other hand think that "external approach," a "reinvigorated diplomatic effort," a "New Diplomatic Offensive" or whatever the Iraq Surrender Group suggests
have been and will continue to be a waste of time. Why do they reason that Iraqi Sunnis would stop shooting Shiites and American troops if the United States successfully pressured Israel to give the Golan Heights to Syria ? Do they really think that removal of U.S. objections to Iran's nuclear program would persuade the Shiite militias to stop shooting ? They are supposed to be the realists but see nothing but fantasy in their suggestions regarding diplomacy .In its narration of the facts, the report acknowledges that Iran and Syria have been promoting violence in Iraq. But all they recommend is tossing more carrots toward them . As we know by the futile efforts of the EU-3 ,carrots have not prevented the Mahdi-hatter from increasing the pace of his uranium enrichment.... nor has it prevented IEDs from having 'made in Iran'stickers all over them .

We are and have been in a shooting war with Iran and Syria for some time now . Why not face the facts ? The road out of Baghdad leads through Tehran and Damascus.

Feingold thinks that Afghanistan will reconstitute as a base of al Qaeda . I think they have moved on to greener pastures . The Taliban can be dealt with by the NATO forces deployed and the Karzai gvt. extending it's control of tribal regions . al-Qaeda is not sitting around in the wilderness of Afghanistan and the tribal regions of Pakistan waiting for the eventuality that they can set up base camps of operations there . Feingold is flat out wrong. The theater of operations has shifted ,but he at least recognized that Somalia is part of that shift.

Our current state of readiness is unacceptable and must be repaired. Our National Guard, too, must be capable of responding to natural disasters and future contingencies.

It is laughable that the Demoncrats can complain about our military being strained given the gutting they gave the military in the 1990s when ,in Feingold's words ,the threat was "emerging" . I am sure in light of his comments he will have no problem with introducing huge increases in expenditures to increase the size of the voluntary force and the upgrade their equipment. No one is suggesting we keep over 140,000 American troops in Iraq indefinitely. That is pure demogogery on his part .

Choux... rated this answer Bad/Wrong Answer
ETWolverine rated this answer Excellent or Above Average Answer
labman rated this answer Excellent or Above Average Answer

Question/Answer
paraclete asked on 12/09/06 - Look out America, Pauline is coming


Please explain, US tells Pauline

By Edmund Burke

December 10, 2006 12:00am
Article from: The Sunday Mail (Qld)


PAULINE Hanson is furious with immigration policy again - but this time she's on the outside looking in.

The former One Nation leader is battling US red tape as she seeks to become a refugee from the hot Australian Christmas.

"I had planned a holiday in Manhattan but because of the time I spent in prison there is a problem with the visa," she said.

"I have to go down to Sydney to the American consulate to show them documents that show the case was quashed.

"I was furious when I found out - absolutely furious. I might even have to have my fingerprints taken before they will let me in," she said.

Ms Hanson was looking forward to a white Christmas. "I want to enjoy that real Christmas feeling, you don't really get it in Australia."

Ms Hanson was jailed in 2003 for electoral fraud and spent 73 days in prison before her conviction was overturned on 6 November, 2006.

She attempted a comeback in Sydney in 2004, standing as an independent for the Senate, but failed.

But last week the former fish shop owner, who has just finished writing her memoirs, again signalled a possible comeback to politics.

"My book is out in March and I will probably make a final decision on my future in February," she said at her 60ha farm west of Ipswich.

If she ran it would be as a Queensland independent but "you never know, a wonderful man could come and take me away from all this".

Ms Hanson, 52, is in a long-term relationship with Irish construction worker Seamus Doheny, 49.

"He hates the limelight, but at the end of the day he lives in Sydney and I live up here and if that doesn't change I am not going to put my life on hold for him," she said.

The former federal MP for Oxley said she started thinking about a comeback when Sydney radio station poll showed that 99 per cent of respondents wanted her to return.

North Queensland MP Bob Katter said he would not rule out taking Ms Hanson into his alliance of Independent MPs.

"But she has said that she wants to be her own boss and the last thing I want to do is pick a fight with Pauline Hanson."

tomder55 answered on 12/10/06:

I am for immigration control but am not a nativist like she appears to be . Ironic that she would complain about entry policies of the US that she apparently supports for Australia.

That being said;I think the US should allow her to pay a visit this month.A New York Christmas should be experienced at least once by everyone. The hours of pleasure sitting in a car trying to navigate the blocks around the Rockefeller Center Christmas tree is worth the price of admission.You really feel the holiday spirt jostling with the crowd at FAO Shwartz or Macy's at Herald Square trying to purchase the 'Tickle me Elmo'.She should stand on line for hours outdoor when thre is a -20 degree wind chill waiting to see the Radio City Christmas Show. Perhaps there might even be a white Christmas although that's not a guarantee .....global warming and all that .


She should extend her vacation a week so she can jam he butt into Time Square on New Years Eve with a million other idiots . Possibly she could even get close enough to actually see the ball drop at midnight.

Tell her that New Yorkers avoid Manhattan like the plague this time of year .

paraclete rated this answer Excellent or Above Average Answer

Question/Answer
HANK1 asked on 12/09/06 - WHO ARE THE HUMBUGS:

Writing for WIRED NEWS: David Hambling: 02:00 AM Dec, 05, 2006

"The crowd is getting ugly. Soldiers roll up in a Hummer. Suddenly, the whole right half of your body is screaming in agony. You feel like you've been dipped in molten lava. You almost faint from shock and pain, but instead you stumble backwards -- and then start running. To your surprise, everyone else is running too. In a few seconds, the street is completely empty.

You've just been hit with a new nonlethal weapon that has been certified for use in Iraq -- even though critics argue there may be unforeseen effects.

According to documents obtained for Wired News under federal sunshine laws, the Air Force's Active Denial System, or ADS, has been certified safe after lengthy tests by military scientists in the lab and in war games.

The ADS shoots a beam of millimeters waves, which are longer in wavelength than x-rays but shorter than microwaves -- 94 GHz (= 3 mm wavelength) compared to 2.45 GHz (= 12 cm wavelength) in a standard microwave oven.

The longer waves are thought to limit the effects of the radiation. If used properly, ADS will produce no lasting adverse affects, the military argues.

Documents acquired for Wired News using the Freedom of Information Act claim that most of the radiation (83 percent) is instantly absorbed by the top layer of the skin, heating it rapidly.

The beam produces what experimenters call the "Goodbye effect," or "prompt and highly motivated escape behavior." In human tests, most subjects reached their pain threshold within 3 seconds, and none of the subjects could endure more than 5 seconds.

"It will repel you," one test subject said. "If hit by the beam, you will move out of it -- reflexively and quickly. You for sure will not be eager to experience it again."

But while subjects may feel like they have sustained serious burns, the documents claim effects are not long-lasting. At most, "some volunteers who tolerate the heat may experience prolonged redness or even small blisters," the Air Force experiments concluded.

The reports describe an elaborate series of investigations involving human subjects.

The volunteers were military personnel: active, reserve or retired, who volunteered for the tests. They were unpaid, but the subjects would "benefit from direct knowledge that an effective nonlethal weapon system could soon be in the inventory," said one report. The tests ranged from simple exposure in the laboratory to elaborate war games involving hundreds of participants.

The military simulated crowd control situations, rescuing helicopter crews in a Black Hawk Down setting and urban assaults. More unusual tests involved alcohol, attack dogs and maze-like obstacle courses.

In more than 10,000 exposures, there were six cases of blistering and one instance of second-degree burns in a laboratory accident, the documents claim.

The ADS was developed in complete secrecy for 10 years at a cost of $40 million. Its existence was revealed in 2001 by news reports, but most details of ADS human testing remain classified. There has been no independent checking of the military's claims.

The ADS technology is ready to deploy, and the Army requested ADS-armed Strykers for Iraq last year. But the military is well aware that any adverse publicity could finish the program, and it does not want to risk distressed victims wailing about evil new weapons on CNN.

This may mean yet more rounds of testing for the ADS."

Could this weapon end the war in Iraq? Who are THE persons saying "NO" to this strategy NOW?

HANK

tomder55 answered on 12/10/06:

The humbugs are the ususal suspectsalthough I pause when I consider the implications of domestic crowd control (although this technology appears on surface less damaging than a billy club ,water cannon ,or tear gas ).The use of this weapon on peaceful protesters would be unacceptable.

However it appears to me that wearing a layer of tin foil can repel this new technology (lol).Under perfect atmospheric conditions it seems to be straight forward;aim the beam and cook the suspect. It is less clear how it would work on a foggy or rainy day.

The other concern is that the radio frequency could be modified to make it lethal . Even at the does given ,it is possible that eye damage can result since the eyes are also external organs.

ETWolverine rated this answer Excellent or Above Average Answer
HANK1 rated this answer Excellent or Above Average Answer

Question/Answer
paraclete asked on 12/08/06 - Is Rice the next victim of failure in Iraq?

Dueling Views Pit Baker Against Rice


By DAVID E. SANGER
Published: December 8, 2006

WASHINGTON, Dec. 7 Many of the blistering critiques of the Bush administration contained in the Iraq Study Groups report boil down to this: the differing worldviews of Baker versus Rice.



Former Secretary of State James A. Baker III was the architect of the new diplomatic offensive in the Middle East that the commission recommended Wednesday as one of its main prescriptions for extracting the country from the mess in Iraq. Ever since, he has been talking on television, to Congress and to Iraqis and foreign diplomats about how he would conduct American foreign policy differently. Very differently.

At a midday meeting with reporters on Thursday, Mr. Baker insisted that the study group had rejected looking backward. But he then proceeded to make a passionate argument for a course of action he believed Condoleezza Rice, the current secretary of state, should be pursuing while carefully never mentioning Ms. Rice by name.

The United States should engage Iran, Mr. Baker contended, if only to reveal its rejectionist attitude; it should try to flip the Syrians; and it should begin a renewed quest for peace between Israel and the Palestinians that, he maintained, would help convince Arab moderates that America was not all about invasions and regime change.

Meanwhile, Ms. Rice remained publicly silent, sitting across town in the office that Mr. Baker gave up 14 years ago. She has yet to say anything about the public tutorial being conducted by the man who first knew her when she was a mid-level Soviet expert on the National Security Council. She has not responded to Mr. Bakers argument, delivered in a tone that drips with isnt-this-obvious, that America has to be willing to talk to its adversaries (a premise Ms. Rice has questioned if the conditions are not right), or his dismissal of the administrations early argument that the way to peace in the Middle East was through quick, decisive victory in Baghdad.

Aides to the 52-year-old Ms. Rice say she is acutely aware that there is little percentage in getting into a public argument with Mr. Baker, the 76-year-old architect of the first Bush administrations Middle East policy. But Thursday, as President Bush gently pushed back against some of Mr. Bakers recommendations, Ms. Rices aides and allies were offering a private defense, saying that she already has a coherent, effective strategy for the region.

She has advocated deepening the isolation of Syria, because she believes much of the rest of the Arab world condemns its efforts to topple Lebanons government, they said; and in seeking to isolate Iran, they said, she hopes to capitalize on the fears of nations like Saudi Arabia, Egypt and Jordan that Iran seeks to dominate the region, with the option of wielding a nuclear weapon.

Ms. Rice makes no apology for the premium she has placed on promoting democracy in the Middle East, even though that is an idea that Mr. Baker and his commission conspicuously ignored in spelling out their recommendations. I dont think that the road to democracy in Iraq is at all utopian, she said in April.

It is plenty utopian to Mr. Baker, who has made clear his view that the quest is entirely ill-suited to the realities of striking a political deal that may keep Sunnis and Shiites from killing each other, and that may extract American forces from Iraq.

Mr. Baker said nothing on Thursday about looking for Jeffersonian democrats in Iraq; he would be happy with few good Iraqi nationalists who can keep the country from splintering apart.

They start from completely different places, said Dennis Ross, the Middle East negotiator who worked for Mr. Baker years ago and left the State Department early in the Bush administration. Baker approaches everything with a negotiators mindset. That doesnt mean every negotiation leads to a deal, but you engage your adversaries and use your leverage to change their behavior. This administration has never had a negotiators mind-set. It divides the world into friends and foes, and the foes are incorrigible and not redeemable. There has been more of an instinct toward regime change than to changing regime behavior.

To some degree, the Bush administration has softened that approach in its second term, and Ms. Rices aides contend that much of what is recommended in the Baker report, including a regional group to support the country, is already under way.

Mr. Bush himself seems uncertain how to handle his always-uncomfortable relationship with his fathers friend. It was Mr. Baker who in 2000 ran the strategy for winning the Florida recount, but he has also made little secret in private that he regards the administration as a bunch of diplomatic go-cart racers, more interested in speed than strategy and prone to ruinous crashes.

The administration has sent out word that it regards Mr. Bakers recommendations as more than a little anachronistic, better suited to the Middle East of 1991 than to the one they are confronting and to some degree have created in 2006 three years after the Iraq invasion. It is a criticism that angers Mr. Baker, members of the study group say.

Iran and Syria illustrate the differing approaches of Mr. Baker and Ms. Rice. If you can flip the Syrians you will cure Israels Hezbollah problem, Mr. Baker said Thursday, noting that Syria is the transit point for arms shipments to Hezbollah. He said Syrian officials told him that they do have the ability to convince Hamas to acknowledge Israels right to exist, and added, If we accomplish that, that would give the Ehud Olmert a negotiating partner.

Ms. Rices allies argue that if it were all that simple, the Syrian problem would have been solved long ago. Stephen J. Hadley, national security adviser and Ms. Rices former deputy, said recently that the problem isnt one of communication, its one of cooperation. Now that Mr. Baker has taken his differences public, the mystery is this: is he speaking for Mr. Bushs father? We never figured that out, said one fellow member of the panel. There was always this implication that there was a tremendous amount of frustration from the old man about what was happening. But Jim was always very careful.

The elder Mr. Bush was careful, too. Asked if he wanted to offer his insights to the panel, he declined.

tomder55 answered on 12/09/06:

Funny ;Condi used to be called a protoge of Baker and his discredited school of realism .Condi should be more visible. It is a mistake to leave this false impression that daddy Bush and the adults have taken over .

Actually in recent months I've been critical that too often she retreats to her past performances in the GHW Bush administration .She has not gone far enough in reforming the State Dept. and weeding out the careerists who equate endless shuttle diplomacy (and pray at the alter of Palestinian Statehood )as job security .Frankly ;it wouldn't suprise me if Rice quitely behind the scenes supported the statements about Palestine in the report .It's as if instead of reform ,Condi has been assimilated into the Foggy Bottom Borg***.


This Baker report is like so much toilet paper. Bush will hopefully thank the commission and ignore their rediculous suggestions that we should parley with the enemy and seek terms of surrender.As Winston Churchill said :

Many new arrangements are being contrived as a result of the hard experience through which we have passed and the many mistakes which no doubt we have made - for success is the result of making many mistakes and learning from experience. If the lull is to end, if the storm is to renew itself, we will be ready, will will not flinch, we can take it again.

We ask no favours of the enemy. We seek from them no compunction. On the contrary, if tonight our people were asked to cast their vote whether a convention should be entered into to stop the bombing of cities, the overwhelming majority would cry, "No, we will mete out to them the measure, and more than the measure, that they have meted out to us." The people with one voice would say: "You have committed every crime under the sun. Where you have been the least resisted there you have been the most brutal. It was you who began the indiscriminate bombing. We will have no truce or parley with you, or the grisly gang who work your wicked will. You do your worst - and we will do our best." Perhaps it may be our turn soon; perhaps it may be our turn now.

[Winston Churchill, 14 July 1941]


~~~~~~~~~~~~~~~~~~~~~~~~~~~~~~~~~~~~~~~~~~
note : "Foggy Bottom" is the Washington district where the State Dept building is located.Frequently the State Dept. is called 'Foggy Bottom' which seems amusingly appropriate.

"Borg" is a fictional alien race in Star Trek who go around the universe assimilating other alien species.

The bureaucracy at Foggy Bottom is entrenched ,and too often conducts business counter to the policies of the administration in power. Rice has a mandate to reform the agency but I am concerned that she has developed too much of that diplomat bureaucratic mind set in her current position. She was better as National Security Adviser in my opinion.

paraclete rated this answer Excellent or Above Average Answer

Question/Answer
ETWolverine asked on 12/08/06 - The ISG report

Well, I finally read the report. It took me two days to do it, but then again, wading through that much crap takes time, and I left my hip-boots home. But I have now finished reading it.

Twice.

I didn't want to make any comments until I was sure that I understood the report thoroughly. Now I do.

I feel like I need a shower.

When we really get down to the meat of this report, the crux is the recommendations. Below are the 79 recommendations made by the ISG, along with my comments.

--------

RECOMMENDATION 1:
The United States, working with the Iraqi government, should launch the comprehensive New Diplomatic Offensive to deal with the problems of Iraq and of the region. This new diplomatic offensive should be launched before December 31, 2006.

(A diplomatic "offensive", eh? Sounds interesting. We've been trying to make diplomatic moves in Iraq for 3 years, but a "new offesive" sounds good. What will it consist of?)

RECOMMENDATION 2:
The goals of the diplomatic offensive as it relates to regional players should be to:
i. Support the unity and territorial integrity of Iraq.
ii. Stop destabilizing interventions and actions by Iraqs neighbors.
iii. Secure Iraqs borders, including the use of joint patrols with neighboring countries.
iv. Prevent the expansion of the instability and conflict beyond Iraqs borders.
v. Promote economic assistance, commerce, trade, political support, and, if possible, military assistance for the Iraqi government from non-neighboring Muslim nations.
vi. Energize countries to support national political reconciliation in Iraq.
vii. Validate Iraqs legitimacy by resuming diplomatic relations, where appropriate, and reestablishing embassies in Baghdad.
viii. Assist Iraq in establishing active working embassies in key capitals in the region (for example, in Riyadh, Saudi Arabia).
ix. Help Iraq reach a mutually acceptable agreement on Kirkuk.
x. Assist the Iraqi government in achieving certain security, political, and economic milestones, including better performance on issues such as national reconciliation, equitable distribution of oil revenues, and the dismantling of militias.


(Alright, now I'm stumped. How do the goals of this proposed "new diplomatic offensive" differ from what Bush and the State Department have been trying to do till now? Which one of these is the "new" part?)

RECOMMENDATION 3:
As a complement to the diplomatic offensive, and in addition to the Support Group discussed below, the United States and the Iraqi government should support the holding of a conference or meeting in Baghdad of the Organization of the Islamic Conference or the Arab League both to assist the Iraqi government in promoting national reconciliation in Iraq and to reestablish their diplomatic presence in Iraq.

(So this committee, in its brilliance, has determined that the best way to solve the problems of Iraq is to set up another committee... which will debate, hem and haw, pat each other on the back for their brilliance, and generally get nothing done as most committees do. Why didn't I think of that?)


RECOMMENDATION 4:
As an instrument of the New Diplomatic Offensive, an Iraq International Support Group should be organized immediately following the launch of the New Diplomatic Offensive.

(Ok. "Immediately" sounds good. I guess accomplishing nothing quickly is better than accomplishing nothing slowly.)

RECOMMENDATION 5:
The Support Group should consist of Iraq and all the states bordering Iraq, including Iran and Syria; the key regional states, including Egypt and the Gulf States; the five permanent members of the United Nations Security Council; the European Union; and, of course, Iraq itself.

(Wow. Iraq should be part of the Support Group TWICE. I guess that's a pretty good suggestion since it is their country that the Iraq Support Group would be making decisions on. Does this mean that they get twice the vote in this committee as all the other members?

And Iran and Syria? I had to read this part over again because I wasn't sure I had read it correctly. Aren't Iran and Syria the countries that have been supporting and fomenting the insurgency and the sectarian violence? Why in the hell would ANYONE suggest making them part of the group that will help make major decisions about Iraq? Isn't that like sitting down with the Mafia and asking their help in stopping organized crime?)

Other countriesfor instance, Germany, Japan and South Koreathat might be willing to contribute to resolving political, diplomatic, and security problems affecting
Iraq could also become members

(Germany? Isn't this the same Germany that has been obstructive about the war in Iraq and the larger global war on terror since day one?)

RECOMMENDATION 6:
The New Diplomatic Offensive and the work of the Support Group should be carried out with urgency, and should be conducted by and organized a the level of foreign minister or above. The Secretary of State, if not the President, should lead the U.S. effort. That effort should be both bilateral and multilateral, as circumstance require.

(Urgency? Again, I guess a quick rush to nowhere is better than a slow rush to nowhere. And I'm glad to see that the Support Group would consist of high-ranking diplomats who will accomplish nothing rather than low-level flunkies who will accomplish nothing.)

RECOMMENDATION 7:
The Support Group should call on the participation of the office of the United Nations Secretary- General in its work. The United Nations Secretary-General should designate a Special Envoy as his representative.

(Ooooh, the UN. Now we're talking about a body with more experience than any other in accomplishin nothing. They'll make a great addition to the non-resolution of this problem. Great idea.)

RECOMMENDATION 8:
The Support Group, as part of the New Diplomatic Offensive, should develop specific approaches to neighboring countries that take into account the interests, perspectives, and potential contributions as suggested above.

(Another terrific idea. Kowtow to other nations on behalf of Iraq... especially Iran and Syria. That will go a long way toward helping this Support Group accomplish nothing, but will cost Iraq and the USA more than if we hadn't kowtowed.)


RECOMMENDATION 9:
Under the aegis of the New Diplomatic Offensive and the Support Group, the United States should engage directly with Iran and Syria in order to try to obtain their commitment to constructive policies toward Iraq and other regional issues. In engaging Syria and Iran, the United States should consider incentives, as well as disincentives, in seeking constructive results.

(I think I missed something here. Didn't the Baker commission ask Iran and Syria if they would like to work with the USA to help solve the problems in Iraq? Didn't they turn the Baker commission down cold? That's what Jim Baker said yesterday at a press conference. So why would we engage in diplomacy with nations that have directly turned down the idea of diplomacy and are aggressive nations who are fomenting the problems in the first place. Baker's reasoning, when asked at the press conference yesterday, was that if we ask them and they turn us down, we can hold them up to international scrutiny. Big whoop! How does that help Iraq?)

RECOMMENDATION 10:
The issue of Irans nuclear programs should continue to be dealt with by the United Nations Security Council and its five permanent members (i.e., the United States, United Kingdom, France, Russia, and China) plus Germany.

(Yeah, because they have been so effective and successful in dealing with it until now. And all those nations have been SOOO cooperative and helpful.)


RECOMMENDATION 11:
Diplomatic efforts within the Support Group should seek to persuade Iran that it should take specific steps to improve the situation in Iraq.

(OOOOH, that'll convince 'em. Talk about "specific steps" instead of "general steps" that Iran should refuse to take.)

RECOMMENDATION 12:
The United States and the Support Group should encourage and persuade Syria of the merit of such contributions as the following:
Syria can control its border with Iraq to the maximum extent possible and work together with Iraqis on joint patrols on the border. Doing so will help stem the flow of funding, insurgents, and terrorists in and out of Iraq.
Syria can establish hotlines to exchange information with the Iraqis.
Syria can increase its political and economic cooperation with Iraq.

(Great idea. Since Syria is directly responsible for arming and funding the insurgents, asking them to pretty please stop will definitely be an effective deterent. Isn't this a little like asking the fox to guard the hen-house?)

RECOMMENDATION 13:
There must be a renewed and sustained commitment by the United States to a comprehensive Arab-Israeli peace on all fronts: Lebanon and Syria, and President Bushs June 2002 commitment to a two-state solution for Israel and Palestine.

(First of all, how is this any different from what Bush has been doing all along?

Second, why is Israel being linked to Iraq? Is Israel's conflicts with Hamas and Hizbollah in any way influencing the insurgents in Iraq or the Iraqi people? Not that I have seen. This just seems to be a way of bribing Iraq's neighbors to get involved in the Support Group. But why is Israel being used as the bribe? What will Israel get in return?)

RECOMMENDATION 14:
This effort should includeas soon as possiblethe unconditional calling and holding of meetings, under the auspices of the United States or the Quartet (i.e., the United States, Russia, European Union, and the United Nations), between Israel and Lebanon and Syria on the one hand, and Israel and Palestinians (who acknowledge Israels right to exist) on the other. The purpose of these meetings would be to negotiate peace as was done at the Madrid Conference in 1991, and on two separate tracks one Syrian/Lebanese, and the other Palestinian.

(Again, this is just a way of bribing OTHER nations to be part of the Iraq Support Group. This report is suggesting that the USA throw its staunchest ally in the Middle East to the wolves in exchange for support from nations that are not trustworthy and have never supported the USA before. It's a rediculous recommendations.)

RECOMMENDATION 15:
Concerning Syria, some elements of that negotiated peace should be:
Syrias full adherence to UN Security Council Resolution 1701 of August 2006, which provides the framework for Lebanon to regain sovereign control over its territory.
Syrias full cooperation with all investigations into political assassinations in Lebanon, especially those of Rafik Hariri and Pierre Gemayel.
A verifiable cessation of Syrian aid to Hezbollah and the use of Syrian territory for transshipment of Iranian weapons and aid to Hezbollah. (This step would do much to solve Israels problem with Hezbollah.)
Syrias use of its influence with Hamas and Hezbollah for the release of the captured Israeli Defense Force soldiers.
A verifiable cessation of Syrian efforts to undermine the democratically elected government of Lebanon.
A verifiable cessation of arms shipments from or transiting through Syria for Hamas and other radical Palestinian groups.
A Syrian commitment to help obtain from Hamas an acknowledgment of Israels right to exist.
Greater Syrian efforts to seal its border with Iraq.

(Ohh, now that we have repeated what we want from Syria... for about the hundreth time... they'll be sure to take it to heart this time.)

RECOMMENDATION 16:
In exchange for these actions and in the context of a full and secure peace agreement, the Israelis should return the Golan Heights, with a U.S. security guarantee for Israel that could include an international force on the border, including U.S. troops if requested by both parties.

(Like I said, Baker is ready to throw Israel to the wolves as a bribe... if Israel was bombed all the way from within Lebanon's current borders, how much more will they have to take if Hizbollah gets control of the Golan Heights?)

RECOMMENDATION 17:
Concerning the Palestinian issue, elements of that negotiated peace should include:
Adherence to UN Security Council Resolutions 242 and 338 and to the principle of land for peace, which are the only bases for achieving peace.
Strong support for Palestinian President Mahmoud Abbas and the Palestinian Authority to take the lead in preparing the way for negotiations with Israel.
A major effort to move from the current hostilities by consolidating the cease-fire reached between the Palestinians and the Israelis in November 2006.
Support for a Palestinian national unity government.
Sustainable negotiations leading to a final peace settlement along the lines of President Bushs two-state solution, which would address the key final status issues of borders, settlements, Jerusalem, the right of return, and the end of conflict.

(Again, the first hundred times these concepts were discussed in failed peace talks must have not been enough. This time both parties will be sure to go along with it.)

RECOMMENDATION 18:
It is critical for the United States to provide additional political, economic, and military support for Afghanistan, including resources that might become available as combat forces are moved from Iraq.

(Gee, I never would have thought of that. What geniuses these committee members are. We actually need to support Afghanistan in the long term? Who would have guessed?)

RECOMMENDATION 19:
The President and the leadership of his national security team should remain in close and frequent contact with the Iraqi leadership. These contacts must convey a clear message: there must be action by the Iraqi government to make substantial progress toward the achievement of milestones. In public diplomacy, the President should convey as much detail as possible about the substance of these exchanges in order to keep the American people, the Iraqi people, and the countries in the region well informed.

(Yeah, I guess the fact that Bush said as much to Maliki several times now and the idea of milestones was rejected by Maliki doesn't mean anything. Maliki will be sure to accept milestones if we bring up the subjkect again.)

RECOMMENDATION 20:
If the Iraqi government demonstrates political will and makes substantial progress toward the achievement of milestones on national reconciliation, security ,and governance, the United States should make clear its willingness to continue training, assistance, and support for Iraqs security forces, and to continue political, military, and economic support for the Iraqi government. As Iraq becomes more capable of governing, defending, and sustaining itself, the U.S. military and civilian presence in Iraq can be reduced.

(If the Iraqi government does what they are supposed to do, we'll continue to help them? Gee, that's another concept that I wouldn't have thought of. Where do these geniuses come up with these gems?)

RECOMMENDATION 21:
If the Iraqi government does not make substantial progress toward the achievement of milestones on national reconciliation, security, and governance the United States should reduce its political, military, or economic support for the Iraqi government.

(The carrot AND the stick? What a novel approach.)

RECOMMENDATION 22:
The President should state that the United States does not seek permanent military bases in Iraq. If the Iraqi government were to request a temporary base or bases, then the U.S. government could consider that request as it would in the case of any other government.

(Uh... yeah. Right. Whatever. Permanent bases, temporary bases, who the hell cares. It doesn't talk to the real issues in Iraq which are all tied to the insurgency.)

RECOMMENDATION 23:
The President should restate that the United States does not seek to control Iraqs oil.

(Yeah, because the fact that we haven't grabbed Iraqi oil until now even when oil prices went up to $3/gallon hasn't convinced the Iraqis of that fact yet. Those who already believe it don't need to be convinced, and those who don't believe it are convinced that Bush is an evil oil barron anyway, and nothing he does or says will convince them.)

RECOMMENDATION 24:
The contemplated completion dates of the end of 2006 or early 2007 for some milestones may not be realistic. These should be completed by the first quarter of 2007.

(So they are proposing milestones that they admitt can't be reached by the Iraqis. WHY?!?!?!)

RECOMMENDATION 25:
These milestones are a good start. The United States should consult closely with the Iraqi government and develop additional milestones in three areas: national reconciliation, security, and improving government services affecting the daily lives of Iraqis. As with the current milestones, these additional milestones should be tied to calendar dates to the fullest extent possible.

(I have no problem with the concept of milestones. I think its a good idea. But if the Iraqis aren't going to accept the milestones, what good will that do? Should that be a reason to pull out our troops? And how will pulling out our troops help the Iraqi government reach those milestones? Won't the lack of security HINDER rather than help the Iraqi government in reaching those milestones?)

RECOMMENDATION 26:
Constitution review. Review of the constitution is essential to national reconciliation and should be pursued on an urgent basis. The United Nations has expertise in this field, and should play a role in this process.

(This is already on the table. Nothing new here.)

RECOMMENDATION 27:
De-Baathification. Political reconciliation requires the reintegration of Baathists and Arab nationalists into national life, with the leading figures of Saddam Husseins regime excluded. The United States should encourage the return of qualified Iraqi professionalsSunni or Shia, nationalist or ex-Baathist, Kurd or Turkmen or Christian or Arabinto the government.

(De-Baathification and reintegration of former Baathists into society. Why didn't Bush think of that? Oh yeah... he did. Again, nothing new here.)

RECOMMENDATION 28:
Oil revenue sharing. Oil revenues should accrue to the central government and be shared on the basis of population. No formula that gives control over revenues from future fields to the regions or gives control of oil fields to the regions is compatible with national reconciliation.

(This idea has been on the table for 5 years now. Again, the committee shows its brilliance in coming up with new ideas.)

RECOMMENDATION 29:
Provincial elections. Provincial elections should be held at the earliest possible date. Under the constitution, new provincial elections should have been held already. They are necessary to restore representative government.

(I agree with this. But until the security situation can be cleared up, there can't be provincial elections that will be truly representative. Too many people will be intimidated to vote their consciences. Security has to come BEFORE any vote can take place.)

RECOMMENDATION 30:
Kirkuk. Given the very dangerous situation in Kirkuk, international arbitration is necessary to avert communal violence. Kirkuks mix of Kurdish, Arab, and Turkmen populations could make it a powder keg. A referendum on the future of Kirkuk (as required by the Iraqi Constitution before the end of 2007) would be explosive and should be delayed. This issue should be placed on the agenda of the International Iraq Support Group as part of the New Diplomatic Offensive.

(Yeah, because the Support Group will be so effective at finding a solution for Kirkuk. The commission's idea is brilliant... take a national powderkeg issue and turn it into an international powderkeg issue.)

RECOMMENDATION 31:
Amnesty. Amnesty proposals must be far-reaching. Any successful effort at national reconciliation must involve those in the government finding ways and means to reconcile with former bitter enemies.

(This idea has been floated and rejected by both our government and the Iraqi government. Killers must be brought to justice. Otherwise the next group of killers to come along will think that they can get away with it too. More appeasement from Baker and Co.)

RECOMMENDATION 32:
Minorities. The rights of women and the rights of all minority communities in Iraq, including Turkmen, Chaldeans, Assyrians, Yazidis, Sabeans, and Armenians, must be protected.

(Really? Minorities need legal protections of their rights as well? I never would have guessed.)

RECOMMENDATION 33:
Civil society. The Iraqi government should stop using the process of registering nongovernmental organizations as a tool for politicizing or stopping their activities. Registration should be solely an administrative act, not an occasion for government censorship and interference.

(Hmmm... is the commission intimating that an Arab govenment might be overstepping its legal and moral bounds? Naw...)

RECOMMENDATION 34:
The question of the future U.S. force presence must be on the table for discussion as the national reconciliation dialogue takes place. Its inclusion will increase the likelihood of participation by insurgents and militia leaders, and thereby increase the possibilities for success.

(Rediculous. Inclusion of the future of US force presence in Iraq as part of the talks will INCREASE INSURGENT RESISTANCE. If the terrorists think that blowing stuff up will result in US forces being pulled out of Iraq, they'll keep blowing stuff up. They need to understand that we are not going away no matter how hard they try to make us. We will leave when WE are and the Iraqi government are ready for us to do so, and not one minute earlier.)

RECOMMENDATION 35:
The United States must make active efforts to engage all parties in Iraq, with the exception of al Qaeda. The United States must find a way to talk to Grand Ayatollah Sistani, Moqtada al-Sadr, and militia and insurgent leaders.

(This idea is about as brilliant as the idea of bringing Iran and Syria into the Support Group. If we ask them to pretty please stop sending terrorists and insurgents to blow up civillians, I'm sure Mookie and Sistani will agree this time.)

RECOMMENDATION 36:
The United States should encourage dialogue between sectarian communities, as outlined in the New Diplomatic Offensive above. It should press religious leaders inside and outside Iraq to speak out on behalf of peace and reconciliation.

(Another set of ideas that nobody else has come up with before. What original thinkers there are on this committee.)

RECOMMENDATION 37:
Iraqi amnesty proposals must not be undercut in Washington by either the executive or the legislative branch.

(A broad amnesty idea is dead, as it should be. Amnesty may be granted on a case by case basis, but a blanket amnesty is a BAD IDEA. As for the idea being undercut by those within our government, when was the last time that Congress DIDN'T try to undercut the President? And now that the incoming Congress is Democat, I expect much more undercutting of the President to take place. So in plain parlace, "Don't bet on it.")

RECOMMENDATION 38:
The United States should support the presence of neutral international experts as advisors to the Iraqi government on the processes of disarmament, demobilization, and reintegration.

(Ooooh! Another committee. Great idea. Why do there need to be "expert's on disarmament"? How hard is it to put all your guns in a big pile and have them loaded onto a truck to be melted down to scrap? For this we need "experts"?)

RECOMMENDATION 39:
The United States should provide financial and technical support and establish a single office in Iraq to coordinate assistance to the Iraqi government and its expert advisors to aid a program to disarm, demobilize, and reintegrate militia members.

(Oooohhhh! This new committee of experts hit the big time... they get their own office and everything.)

RECOMMENDATION 40:
The United States should not make an open-ended commitment to keep large numbers of American troops deployed in Iraq.

(Nor should they make a deadline for pullout of the troops. Just as an "open-ended committment" is a disincentive to the Iraqi government to take responsibility for their own problems, a shedule for pullout is a disincentive to terrorists to stop blowing stuff up.)

RECOMMENDATION 41:
The United States must make it clear to the Iraqi government that the United States could carry out its plans, including planned redeployments, even if Iraq does not implement its planned changes. Americas other security needs and the future of our military cannot be made hostage to the actions or inactions of the Iraqi government.

(I agree in general with this recommendation. So that makes it... what... one out of 41 so far?)

RECOMMENDATION 42:
We should seek to complete the training and equipping mission by the first quarter of 2008 as stated by General George Casey on October 24, 2006.

(Uh... yeah... about that... its already being done. How does this constitute a new direction in Iraq?)

RECOMMENDATION 43:
Military priorities in Iraq must change, with the highest priority given to the training, equipping, advising, and support mission and to counterterrorism operations.

(I disagree strongly with this. Yes, training should be a high priority item. But the highest priority item should ALWAYS be to find and kill the insurgents. THAT is what the military is really there for, and we haven't utilized their abilities in that area. We've been too concerned with training Iraqis and not enough with killing the enemy.)

RECOMMENDATION 44:
The most highly qualified U.S. officers and military personnel should be assigned to the imbedded teams, and American teams should be present with Iraqi units down to the company level. The U.S. military should establish suitable career-enhancing incentives for these officers and personnel.

(Career-enhancing incentives? What the hell? A soldier goes where he is told to go, kills who he is told to kill, and doesn't make military decisions based on personal career enhancement. If he does, he needs to be somewhere OTHER than in combat. It is pretty clear from this staement alone that the guys who wrote this report haven't got a f#(&ing clue about how the modern US military works or should work. They are thinking of the Vietnam era when soldiers were conscripts and needed incentives to do their jobs. THESE soldiers are volunteers. The modern military is a whole different animal than the military of 40 years ago.

RECOMMENDATION 45:
The United States should support more and better equipment for the Iraqi Army by encouraging the Iraqi government to accelerate its Foreign Military Sales requests and, as American combat brigades move out of Iraq, by leaving behind some American equipment for Iraqi forces.

(As I understand it, the US military is already leaving stuff behind for the Iraqi military. Nothing new here either.)

RECOMMENDATION 46:
The new Secretary of Defense should make every effort to build healthy civil-military relations ,by creating an environment in which the senior military feel free to offer independent advice not only to the civilian leadership in the Pentagon but also to the President and the National Security Council, as envisioned in the Goldwater- Nichols legislation.

(The SecDef should encourage truthful opinions from his generals. He needs honest information to make good decisions. But he should DISCOURAGE those who he disagrees with from going public with their disagreements. Just because your senior disagreed with your opinion and made a different decision than you would have is not a reason to go to the press with an exclusive interview about how terrile the SecDef is, or to be a source of "leaks" to the newspapers.)

RECOMMENDATION 47:
As redeployment proceeds, the Pentagon leadership should emphasize training and education programs for the forces that have returned to the continental United States in order to reset the force and restore the U.S. military to a high level of readiness for global contingencies.

(Gee, because the military hasn't already thought of that by itself. That is SOP for the military in ALL conflicts... when the troops get home, they get some downtime, and slowly rebuild to full readiness levels through training.)

RECOMMENDATION 48:
As equipment returns to the United States, Congress should appropriate sufficient funds to restore the equipment to full functionality over the next five years.

(You mean that soldiers should have fully functional equipment at all times? And that Congress should fund it? Ya don't say...)

RECOMMENDATION 49:
The administration, in full consultation with the relevant committees of Congress, should assess the full future budgetary impact of the war in Iraq and its potential impact on the future readiness of the force, the ability to recruit and retain high-quality personnel, needed investments in procurement and in research and development, and the budgets of other U.S. government agencies involved in the stability and reconstruction effort.

(You mean the government is supposed to consider the effect of war on the military? Another brilliant idea that nobody has ever thought of.)

RECOMMENDATION 50:
The entire Iraqi National Police should be transferred to the Ministry of Defense, where the police commando units will become part of the new Iraqi Army.

(Not sure I agree with this idea. If the entire police and military force is under the single command of the Ministry of Defence, doesn't that open the possibility of a coup by the Minister of Defence? Shouldn't the military be controlled by the Ministry of Defense and the police by the Ministry of the Interior in order to keep a balancce of powers? The same way that our federal police, Secret Service, military and intelligence departments are controlled out of different Departments. Just wondering.)

RECOMMENDATION 51:
The entire Iraqi Border Police should be transferred to the Ministry of Defense, which would have total responsibility for border control and external security.

(Border control is legitimately a responsibility of the Defense Ministry. I have no issue with this idea. That's 2 out of 51.)

RECOMMENDATION 52:
The Iraqi Police Service should be given greater responsibility to conduct criminal investigations and should expand its cooperation with other elements in the Iraqi judicial system in order to better control crime and protect Iraqi civilians.

(Already being worked on. But a good idea, even if they were a little late.)

RECOMMENDATION 53:
The Iraqi Ministry of the Interior should undergo a process of organizational transformation, including efforts to expand the capability and reach of the current major crime unit (or Criminal Investigation Division) and to exert more authority over local police forces. The sole authority to pay police salaries and disburse financial support to local police should be transferred to the Ministry
of the Interior.

(And how is the Interior Ministry supposed to do that if we are suggesting that the police be transferred to the Ministry of Defense? This recommendation contradicts #50.)

RECOMMENDATION 54:
The Iraqi Ministry of the Interior should proceed with current efforts to identify, register, and control the Facilities Protection Service.

(In other words, they should continue what they are already doing. Good idea. Glad they thought of it.)

RECOMMENDATION 55:
The U.S. Department of Defense should continue its mission to train the Iraqi National Police and the Iraqi Border Police, which should be placed within the Iraqi Ministry of Defense.

(In other words, they should continue what they are already doing. Good idea. Glad they thought of it.)

RECOMMENDATION 56:
The U.S. Department of Justice should direct the training mission of the police forces remaining under the Ministry of the Interior.

(Actually not a bad idea. Justice is generally more familiar with crime-stopping than the military is. The two functions are different and require different specialties. This is Justice's specialty. The ISG is now 3 for 56.)

RECOMMENDATION 57:
Just as U.S. military training teams are imbedded within Iraqi Army units, the current practice of imbedding U.S. police trainers should be expanded and the numbers of civilian training officers increased so that teams can cover all levels of the Iraqi Police Service, including local police stations. These trainers should be obtained from among experienced civilian police executives and supervisors from around the world. These officers would replace the military police personnel currently assigned to training teams.

(Again a good idea. Cops are better at training people to be cops than soldiers are. 4 for 57.)

RECOMMENDATION 58:
The FBI should expand its investigative and forensic training and facilities within Iraq, to include coverage of terrorism as well as criminal activity.

(Basically the same idea as #57. I still agree, but they don't get another point for saying essentially the same thing twice.)

RECOMMENDATION 59:
The Iraqi government should provide funds to expand and upgrade communications equipment and motor vehicles for the Iraqi Police Service.

(Iraqi cops should have cars? What a novel idea.)

RECOMMENDATION 60:
The U.S. Department of Justice should lead the work of organizational transformation in the Ministry of the Interior. This approach must involve Iraqi officials, starting at senior levels and moving down, to create a strategic plan and work out standard administrative procedures, codes of conduct, and operational measures that Iraqis will accept and use. These plans must be drawn up in partnership.

(How does this differ from what Bush has been pushing for the past 3 years? It doesn't.)

RECOMMENDATION 61:
Programs led by the U.S. Department of Justice to establish courts; to train judges, prosecutors, and investigators; and to create institutions and practices to fight corruption must be strongly supported and funded. New and refurbished courthouses with improved physical security, secure housing for judges and judicial staff, witness protection facilities, and a new Iraqi Marshals Service are essential parts of a secure and functioning system of justice.

(This is #57 again... get the US Dept of Justice to train the Iraqis. They don't get extra points for saying it a third time either.)

RECOMMENDATION 62:
As soon as possible, the U.S. government should provide technical assistance to the Iraqi government to prepare a draft oil law that defines the rights of regional and local governments and creates a fiscal and legal framework for investment. Legal clarity is essential to attract investment.
The U.S. government should encourage the Iraqi government to accelerate contracting for the comprehensive well work-overs in the southern fields needed to increase production, but the United States should no longer fund such infrastructure projects.
The U.S. military should work with the Iraqi military and with private security forces to protect oil infrastructure and contractors. Protective measures could include a program to improve pipeline security by paying local tribes solely on the basis of throughput (rather than fixed amounts).
Metering should be implemented at both ends of the supply line. This step would immediately improve accountability in the oil sector.
In conjunction with the International Monetary Fund, the U.S. government should press Iraq to continue reducing subsidies in the energy sector, instead of providing grant assistance. Until Iraqis pay market prices for oil products, drastic fuel shortages will remain.

(Again, how does this differ from what Bush has been pushing for until now?)

RECOMMENDATION 63:
The United States should encourage investment in Iraqs oil sector by the international community and by international energy companies.
The United States should assist Iraqi leaders to reorganize the national oil industry as a commercial enterprise, in order to enhance efficiency, transparency, and accountability.
To combat corruption, the U.S. government should urge the Iraqi government to post all oil contracts, volumes, and prices on the Web so that Iraqis and outside observers can track exports and export revenues.
The United States should support the World Banks efforts to ensure that best practices are used in contracting. This support involves providing Iraqi officials with contracting templates and training them in contracting, auditing, and reviewing audits.
The United States should provide technical assistance to the Ministry of Oil for enhancing maintenance, improving the payments process, managing cash flows, contracting and auditing, and updating professional training programs for management and technical personnel

(Ditto.)

RECOMMENDATION 64:
U.S. economic assistance should be increased to a level of $5 billion per year rather than being permitted to decline. The President needs to ask for the necessary resources and must work hard to win the support of Congress. Capacity building and job creation, including reliance on the Commanders Emergency Response Program, should be U.S. priorities. Economic assistance should be provided on a nonsectarian basis.

(Congress is gonna love this one. The Rep Congress tried to shortchange funding in Iraq in 2006 and 2007. The Dems in Congress were looking forward to an excuse to defund Iraq. I happen to agree with this idea, though. That's 5 for 64.)

RECOMMENDATION 65:
An essential part of reconstruction efforts in Iraq should be greater involvement by and with international partners, who should do more than just contribute money. They should also actively participate in the design and construction of projects.

(You have got to be kidding me. Let Iran and Syria and every other Arab country anywhere near Iraq's oil... and not expect the oil to just simply dissappear into thin air--- or rather into foreign countries' oil reserves? Why in the hell would we trust any other country to be involved with Iraq's oil infrastructure?)

RECOMMENDATION 66:
The United States should take the lead in funding assistance requests from the United Nations High Commissioner for Refugees, and other humanitarian agencies.

(To "lead" implies that someone will follow. That won't happen if the UN is involved. If we want to solve problems of refugees and humanitarian aid, we need to keep the child rapists from the UN as far away from Iraq as possible.)

RECOMMENDATION 67:
The President should create a Senior Advisor for Economic Reconstruction in Iraq.

(Another Senior Advisor to head another comittee? Whooppeee!!! We're just running over with committees.)

RECOMMENDATION 68:
The Chief of Mission in Iraq should have the authority to spend significant funds through a program structured along the lines of the Commanders Emergency Response Program, and should have the authority to rescind funding from programs and projects in which the government of Iraq is not demonstrating effective partnership.

(Doesn't the Cheif of Mission already have that authority? I believe he does.)

RECOMMENDATION 69:
The authority of the Special Inspector General for Iraq Reconstruction should be renewed for the duration of assistance programs in Iraq.

(In other words, they should continue what they are already doing. Good idea. Glad they thought of it.)

RECOMMENDATION 70:
A more flexible security assistance program for Iraq, breaking down the barriers to effective interagency cooperation, should be authorized and implemented.

(What... no invisible wall? Agencies should work together toward a common goal? Share information and resources? What a terrific idea. I wonder if that applies to intelligence agencies back here in the USA too.)

RECOMMENDATION 71:
Authority to merge U.S. funds with those from international donors and Iraqi participants on behalf of assistance projects should be provided.

(More pooling of resources for the common goal? My, how smart.)

RECOMMENDATION 72:
Costs for the war in Iraq should be included in the Presidents annual budget request, starting in FY 2008: the war is in its fourth year, and the normal budget process should not be circumvented. Funding requests for the war in Iraq should be presented clearly to Congress and the American people. Congress must carry out its constitutional responsibility to review budget requests for the war in Iraq carefully and to conduct oversight.

(I saw copies of the 2006 and 2007 budget proposals by Bush. Costs of the Iraq war were clearly stated in the budget in the military expenditures section. It had its own account line and everything. So again, this is more of what is already being done.)

RECOMMENDATION 73:
The Secretary of State, the Secretary of Defense, and the Director of National Intelligence should accord the highest possible priority to professional language proficiency and cultural training, in general and specifically for U.S. officers and personnel about to be assigned to Iraq.

(There's a shortage of Arabic-language and culture specialists in the government's employ. Unless the government is willing to splurge on copies of the Rossetta Stone Arabic Langauge software, how do Baker and Co. expect that to be accomplished? Personally, I think the Rossetta Stone software is a good idea.)

RECOMMENDATION 74:
In the short term, if not enough civilians volunteer to fill key positions in Iraq, civilian agencies must fill those positions with directed assignments. Steps should be taken to mitigate familial or financial hardships posed by directed assignments, including tax exclusions similar to those authorized for U.S. military personnel serving in Iraq.

(How does this differ from what we are doing now? If a position needs to be filled in Iraq, volunteers are sought. If there aren't enough volunteers, people are assigned to those positions anyway. I agree that we should be forgiving taxes for military and government personnel in Iraq. Frankly, I think that ALL military salaries everywhere should be tax free, regardless of assignment. I also think that mortgage payments --- principal and interest--- and electric and water bills should be automatically frozen while military or govenment personnell are assigned overseas to decrease familial hardship.)

RECOMMENDATION 75:
For the longer term, the United States government needs to improve how its constituent agenciesDefense, State, Agency for International Development, Treasury, Justice, the intelligence community, and others respond to a complex stability operation like that represented by this decades Iraq and Afghanistan wars and the previous decades operations in the Balkans. They need to train for, and conduct, joint operations across agency boundaries, following the Goldwater-Nichols model that has proved so successful in the U.S. armed services.

(So the various departments should work together in the future and learn to respond better to emergencies? Another brilliant new idea.)

RECOMMENDATION 76:
The State Department should train personnel to carry out civilian tasks associated with a complex stability operation outside of the traditional embassy setting. It should establish a Foreign Service Reserve Corps with personnel and expertise to provide surge capacity for such an operation. Other key civilian agencies, including Treasury, Justice, and Agriculture, need to create similar technical assistance capabilities.

(A foreign service reserve corps is actually a pretty good idea... a pool of people that can be called upon for foreign diplomatic service and other functions --- such as managing reconstruction efforts --- that are called up only when special needs arise. They would go through periodic diplomatic training and testing to maintain their status as reserve diplomats. I actually like this idea. 6 for 76.)

RECOMMENDATION 77:
The Director of National Intelligence and the Secretary of Defense should devote significantly greater analytic resources to the task of understanding the threats and sources of violence in Iraq.

(Uhhhh... the threat is that a bomb will explode. The source of the threat is insurgents who plant bombs. What's to understand?)

RECOMMENDATION 78:
The Director of National Intelligence and the Secretary of Defense should also institute immediate changes in the collection of data about violence and the sources of violence in Iraq to provide a more accurate picture of events on the ground.

(Again... the bombs explode: that is the nature of the violence. The source of the violence is the guys who plant the bombs. What data do we need to collect? The number of people being killed? For what purpose? Scorekeeping? If you stop the violence by killing the terrorists, there won't be a need to collect data on terrorist attacks.)

RECOMMENDATION 79:
The CIA should provide additional personnel in Iraq to develop and train an effective intelligence service and to build a counterterrorism intelligence center that will facilitate intelligence-led counterterrorism efforts.

(Again, how does this differ from #57? That's the fourth time this idea is being stated.)

--------------

What I see here is a document that is so full of fluff that sounds nice and looks good but has no real substance to it that it couls have only come from a committee. This document was an attempt to get everyone at the table to agree, not a hard-hitting effort to actually solve problems in Iraq. Aside from 5 or 6 good ideas, there is absolutely NOTHING in the document worthy of the 80+ pages it was printed on.

Not once is the idea of ramping up military operations to catch or kill terrorists even discussed. It doesn't say to increase force levels. It doesn't say to DECREASE force levels either. It only says "Don't make an open-ended committment of force in Iraq."

This entire document is one giant form of "Keep on doing what you have been doing, but do more of it, and do it with those who hate your guts anyway... and try to get other departments in teh US government, the UN and other Arab countries to do it too."

That's a solution? That's a change of direction?

What a f@(&ing waste of time and money.

Elliot

tomder55 answered on 12/08/06:

lol I heard moonbat Randi Rhodes go off on a rant about #s 62 & 63. She claims that is proof positive that the whole Iraq gambit was about plundering oil.

My cousin who is a senior officer is going back to Iraq in January . As I understand it he was asking to be imbedded with the Iraqis. Nothing in the mission to Iraq is detrimental to a soldiers career. He would be insulted by the implication . This will be his second tour during OIF and he also served in Desert Storm .

I think the only concensus reached so far is to screw Israel.The Israeli-Palestinian question has nothing to do with our problems in Iraq, and the ISG report presents no evidence to the contrary . Yet without any other negotiations ,they added quid pro quo, the Golan Heights and the "Right of Return " for Palestinians back into the equation (as well as the sell out of the Cedar Revolution in Lebanon).So in other words a Republican (Baker ) and a Democrat (Jimmy Carter ) are on the same page. Concensus is reached ! Long Live Realpolitik !! Since they could not find any answers (besides victory which evidently is off the table) ;they resorted to that well worn mantra and myth that all would be well in the world if only Israel did not exist .



Adherence to UN Security Council Resolutions 242 and 338 and to the principle of land for peace, which are the only bases for achieving peace.

Resolution 242 dates to 1967 and Resolution 338 dates to 1973 .Perhaps we can excuse the whole group of geezers but that is NOT US policy . May I remind them :


First, the United States remains committed to my vision and to its implementation as described in the roadmap. The United States will do its utmost to prevent any attempt by anyone to impose any other plan. Under the roadmap, Palestinians must undertake an immediate cessation of armed activity and all acts of violence against Israelis anywhere, and all official Palestinian institutions must end incitement against Israel. The Palestinian leadership must act decisively against terror, including sustained, targeted, and effective operations to stop terrorism and dismantle terrorist capabilities and infrastructure. Palestinians must undertake a comprehensive and fundamental political reform that includes a strong parliamentary democracy and an empowered prime minister.

Second, there will be no security for Israelis or Palestinians until they and all states, in the region and beyond, join together to fight terrorism and dismantle terrorist organizations. The United States reiterates its steadfast commitment to Israel's security, including secure, defensible borders, and to preserve and strengthen Israel's capability to deter and defend itself, by itself, against any threat or possible combination of threats.

Third, Israel will retain its right to defend itself against terrorism, including to take actions against terrorist organizations. The United States will lead efforts, working together with Jordan, Egypt, and others in the international community, to build the capacity and will of Palestinian institutions to fight terrorism, dismantle terrorist organizations, and prevent the areas from which Israel has withdrawn from posing a threat that would have to be addressed by any other means.
[Letter President Bush to Ariel Sharon ]

Don't have time to continue but you know where I stand on this .

ETWolverine rated this answer Excellent or Above Average Answer
Itsdb rated this answer Excellent or Above Average Answer

Question/Answer
Itsdb asked on 12/08/06 - Another sign America is rolling downhill

Dec. 7, 2006, 7:13PM

By DAVID CRARY
Associated Press

NEW YORK A month after distribution began, the over-the-counter version of the morning-after pill is now available at pharmacies nationwide. Planned Parenthood celebrated Wednesday with a free giveaway of the emergency contraceptive, while critics insisted that Plan B's accessibility will soon be a cause for regret.

Plan B was the focus of bitter debate for years before the Food and Drug Administration, after repeated delays, declared in August that customers 18 and older should be able to buy it in pharmacies without a prescription.

The manufacturer, Barr Pharmaceuticals, then needed to develop new packaging for the over-the-counter version; it announced the start of national distribution on Nov. 6.

Plan B, a high dose of a drug found in many regular birth-control pills, can lower the chance of pregnancy by up to 89 percent if taken within 72 hours of unprotected sex. Girls 17 and younger still need a prescription to buy Plan B, though an older person male or female could buy it over the counter on a teenage girl's behalf.

Supporters of Plan B had sought over-the-counter approval on the premise that wider availability would reduce the number of unplanned pregnancies and abortions.

Critics of the drug challenge those claims, arguing that Plan B will promote promiscuity and unsafe sex; they warn that men might coerce underage partners into using it. Some critics also consider the pill tantamount to abortion, although it differs from the abortion pill RU-486 and has no effect on women who are already pregnant.

Carol Cox, a Barr Pharmaceuticals spokeswoman, said the Plan B distribution went smoothly, but she would not specify how many pills were issued. She said the company does not expect detailed sales information until February. The cost varies. Kate Horle, with Planned Parenthood in Colorado, said prices there range from $30 at her group's clinics to $61 at some supermarkets.

While some independent pharmacies are not stocking Plan B because of moral objections or perceived lack of demand, the pill is widely available. Chains such as CVS, Rite-Aid and Walgreens pledge to ensure that customers can buy Plan B onsite even if an employee declines to provide service for reasons of conscience. Planned Parenthood centers also sell the pill.

On Wednesday, in celebration of Plan B's expanded availability, the pills were given away free at more than 350 Planned Parenthood centers in 30 states.

Planned Parenthood's vice president for medical affairs, Dr. Vanessa Cullins, urged women to back up their regular birth control by keeping emergency contraception in their medicine cabinet.

Some critics spoke of pushing state legislation to curtail access to it. "The battle is not over," said Wendy Wright, president of Concerned Women for America. Four states Arkansas, Georgia, Mississippi and South Dakota allow pharmacists to opt out of providing emergency contraceptives.

~~~~~~~~~~~~~~~~~~~~~~~~~~~~~~~~~~~~~~~~~~~~~~~~~~~~~~

Regardless of your view on the subject, isn't it a little sad that the nation's largest provider of abortions would 'celebrate' with a giveaway, so EC could "be in every womans medicine cabinet?" How did we get to the point that life has such little value, and I still want to know how those 'progressives' can whine about terrorist's rights and war casualites while abortion is their holy grail of issues?

And on that note, I would encourage every 'pro-choice' individual to watch In the Womb: Animals on the National Geographic Channel this Sunday night. Let me know if while watching that baby dolphin swim in its mother's womb if you go "awww, how cute." Then tell me why.

tomder55 answered on 12/08/06:

Plan B being "kill the babies" ?

and for the convience of having "safe sex" ,all you have to do is live with these small inconvienences :

Nausea
Abdominal pain
Fatigue
Headache
Dizziness
menstrual changes.
Breast tenderness
Vomiting
Diarrhea

The morning after pill is used to induce an abortion. It doesn't matter whether its the day after,or the month after .It is poison . It induces abortion in any woman carrying a child at any stage of development.

Pro-life advocates said Iowa Planned Parenthood is empowering rapists to force the women they sexually abuse to use the drug so they can cover up their crimes.

"I think the fact they are handing this out to men is outrageous," Rachel Cunningham of the Iowa Family Policy Center told the Des Moines Register newspaper.

"I don't think I've ever seen a campaign that would better enable abusive men to hide their crimes," she added.

Cunningham said the Plan B drug makes it easier for someone to hide a sexually abusive relationship or one instance of rape or incest.

For that reason alone it should be at best be dispensed by perscription only . Who's to say that a pregnant women wouldn't be slipped one of these pill out of spite ? I can conjure up hundereds of nightmare scenarios.

Me ;I'm for defunding Planned Parenthood until they can convince me that they are not complicit in the cover up of statutory rapes .

Itsdb rated this answer Excellent or Above Average Answer

Question/Answer
paraclete asked on 12/07/06 - The peril of Pauline, HELLO boys! I'M BACK?

PAULINE HANSON, WHO 10 YEARS AGO FLAGSHIPPED THE MOVEMENT OF AUSTRALIAN POLITICS TO THE RIGHT IS BACK AND LOADED FOR ANYTHING THAT MOVES
Perils of Pauline out of Africa

By Nick Butterly

December 07, 2006 12:12pm
Article from: AAP

UP to a third of Black South Africans coming into Australia may have TB and some have AIDS, according to One Nation founder Pauline Hanson.

Flagging a political comeback at next year's federal election, Ms Hanson also said she was angry Australians were unable to swim in public baths because of the sensitivities of Muslims.

"You don't bend over backwards appeasing their demands because the politicians want their vote ... unless you change this you will have social incohesion which is already happening in this country," she said on Southern Cross radio.

"If they don't want it, then they can go back where they came from."

Related Blog: What do you think about Hanson's plan?

Ms Hanson said she was committed to running as an independent in Queensland at the next election, though she had not made up her mind if she would run for the lower house or the Senate.

She said she decided to make a political comeback after members of the public had urged her to do so.

"I have still got a lot of issues and I have still got a lot of concerns like a lot of Australians and I think that I would like to voice my opinions and have a say on behalf of the Australian people, to represent them," she said.

Ms Hanson said federal parliamentary secretary for immigration Andrew Robb had admitted on television that the Government was letting in many "black South Africans" with health issues.

"Andrew Robb actually admitted that a few months ago, it was on TV," she said.

"He indicated there was around about 37 per cent of black South Africans that were coming in with ongoing health issues."

Ms Hanson said she had been told by a hospital worker of an African woman with AIDS who had given birth to a child with AIDS.

"There's increasing numbers of TB (tuberculosis) and they have picked up ... it could be almost one third that actually carries TB," she said.

"I have got children, grandchildren ... I would hate to see anyone who may be subjected to catching TB or AIDS."

Questioned on her concerns about Muslims, Ms Hanson said she was angry that Australians were no longer able to sing Christmas carols in schools or swim in public swimming pools.

"What people are getting all upset about is the fact that you can't sing Christmas carols in schools because it upsets a certain amount of people," she said.

"You can't actually swim in baths because a certain amount of people want their privacy to swim in those baths, swimming baths and pools that have been paid for by the taxpayers over the years and yet they are closed to everyone to use at the same time."

Politicians were quick to rebuff Ms Hanson's comments.

Australian Democrats senator Andrew Bartlett , who faces a fight to retain his Queensland seat next year, said: "Once again Pauline Hanson opens her mouth and spreads offensive urban myths that bear no resemblance to reality.

"She brings no evidence to back up her claims. They are nothing more than hearsay.

"Her musings don't further debate or address community concerns, they only cause fear and division and they must be challenged."

Queensland Premier Peter Beattie, who has earned the ire of Ms Hanson for rejecting her bid to be compensated for wrongful imprisonment, said although he disagreed with her ideas he supported her democratic right to run.

Mr Beattie said disillusionment in Queensland with the Nationals would boost the cause of Ms Hanson and other right wing political movements such as Bob Katter's "The Beast".

"It's really going to be an interesting time in the next federal election," Mr Beattie said.

"I think the heart of it is disillusionment with conservative politics in the bush and the regions."

Ms Hanson said she would make an official announcement about her comeback next year. She said she would give herself plenty of time to campaign.

Ms Hanson was jailed in 2003 for fraudulently registering the One Nation party and spent 73 days in prison before her conviction was overturned.

She split from her former party and has spent the past two years writing her autobiography.
++++++++++++++++++++++++++++++++++++++++++++++++++++++

Pauline is a colourful figure and not afraid to say what most of us are thinking, who's country is this anyway. It doesn't belong to the Lebs, it doesn't belong to the Abo's, it doesn't belong to the kiwi's, it doesn't belong to the yank's, it it belongs to those who have worked hard to preserve a particular way of life and a lifestyle which is the envy of the world.

tomder55 answered on 12/07/06:

Headline of the day

Australia: Mosque to get police guard for bikini rally

The march is organized by Christine Hawkins, a "Melbourne grandmother." It is intended to protest comments by an Australian imam who explained rape on the ground that women in Australia walk around like "uncovered meat."

YOU GO GIRL !!! I want photos !

ETWolverine rated this answer Excellent or Above Average Answer
Itsdb rated this answer Excellent or Above Average Answer
paraclete rated this answer Excellent or Above Average Answer

Question/Answer
HANK1 asked on 12/06/06 - Cowboys and Indians:



Why don't our soldiers in Iraq take a lesson from the Indians who roamed the plains during the old Wild West days? They surrounded wagon trains. No escape was possible. Why don't our soldiers use the same stategy in Iraq. Surround Baghdad, synchronize their watches and then start moving towards the center of that hell hole? The old squeeze play. Our planes and choppers could protect their 'backs' and take care of any trouble that may arise north, south, east and west. Since Bush is sending 30,000 more troops to the area, I think this strategy might work.

HANK

tomder55 answered on 12/07/06:

Bush met Monday with Shiite cleric Abdul Aziz al-Hakim .Al-Hakin said that he "vehemently" opposes any regional or international effort to solve Iraq's problems that goes around the unity government in Baghdad. I agree. Take that James Baker !!! He urged Bush to step up military attacks against insurgents and al-Qaeda terrorists . Again I agree. But while doing that ,I would tell him either neutralize Mookie al-Sadr and his Mahdi brigade or we will do it .Defeating the enemy is the path to peace.

HANK1 rated this answer Excellent or Above Average Answer
Itsdb rated this answer Excellent or Above Average Answer

Question/Answer
Choux... asked on 12/06/06 - Report of Iraq Study Group

"The Iraq Study Group called the situation in Iraq "grave and deteriorating" Wednesday and recommended a radically different approach from President Bush's current policy, including the withdrawal of most U.S. combat troops by early 2008.

In delivering its report to Bush and Congress, the bipartisan panel listed 79 recommendations for change in Iraq strategy, including direct talks with Iran and Syria as part of a "diplomatic offensive."

All 10 members of the panel, chaired by former Secretary of State James Baker, a Republican, and former Democratic Rep. Lee Hamilton of Indiana, met with Bush at the White House to present the bound report. (View the complete report -- PDF)

The Bush administration has repeatedly rejected calls to seek help from Iran and Syria.

But the report states that "Iraq's neighbors and key states in and outside the region should form a support group" to help Iraq achieve long-term security and political reconciliation -- "neither of which it can sustain on its own."

"If we don't talk to them, we don't see much progress being made," Hamilton said. "You can't look at this part of the world and pick and choose which countries you're going to deal with."

The panel, which was chartered by Congress, warns of dire consequences, both at home and abroad, if the U.S. fails to take action.

"If the situation continues to deteriorate, the consequences could be severe. A slide toward chaos could trigger the collapse of Iraq's government and a humanitarian catastrophe," the report says.

"Neighboring countries could intervene. Sunni-Shia clashes could spread. Al Qaeda could win a propaganda victory and expand its base of operations. The global standing of the United States could be diminished. Americans could become more polarized."

On the military front, the report suggests, "By the first quarter of 2008, subject to unexpected developments in the security situation on the ground, all combat brigades not necessary for force protection could be out of Iraq."

It adds: "At that time, U.S. combat forces in Iraq could be deployed only in units embedded with Iraqi forces, in rapid-reaction and special operations teams and in training, equipping, advising, force protection and search and rescue."

The co-chairs said they took "a pragmatic approach" to determining the best course for Iraq and determined the solution was not a military, political or economic one, but rather a combination of the three.

"We no longer can afford to stay the course," Baker said. "If we do what we recommend in this report, it will certainly improve our chances for success."

Hamilton echoed his colleague's sentiments, saying the Iraqi people are "suffering great hardship" and their lives must be improved....."

~~~~~~~~~~~~~~~~~~~~~~~~~~~~~~~~~~~~~~~~~~~~~~~

tomder55 answered on 12/07/06:

I hearby procaim this the report by the 'Iraq Surrender Group '(ISG).Our enemies both inside and outside the country know they have a winning strategy in defeating us. Just inflict 75-100 casualities per month on our forces, which is easy to do, and we will surrender. We no longer have reason to mock the French. Just look at the front cover of the NY Post today for my view of the whole thing .America must have a death wish.

I imagine Baker et al being around during the Revolution. We'd still be singing 'God Save The King '. Not too many people know this but James Baker is a partner in Baker, Botts law firm which is representing Saudi Arabia in the lawsuit filed against the kingdom by the relatives of the victims of the September 11 attacks.Interesting that we would determine our future ME policy by the recomendations of a man with such and obvious conflict of interest.I think Bush should ban almost all of his father's lackeys from the White House.

If you look at the membership of the ISG ,some of them like Meese and Sandy O'Conner are tolerable .But where is the military person in the whole group ? There isn't any . Of the whole group only Chuck Robb had the balls to venture outside of the Green Zone to get a take from the soldier in the field during their very brief stay in the country .The group never once stepped foot in the one region in Iraq where things are actually going well; in the northern region of Iraqi Kurdistan.

It is disgusting to me that the course of the war will be determined by what Leon Panetta and Vernon Jordon (last seen finding a job for Monica Lewinsky in NY)thinks .Didn't Lee Hamilton already do enough damage to the country with the 9-11 Commission's flawed report ? To this whole group of septuagenarians I say retire already ;take your gvt. pensions and leave us alone ! You try to help but collectively you are doing more harm than good .

How do these idiots reconcile No country in the region wants a chaotic Iraq, ...It is clear to the Iraq Study Group members that all of Iraqs neighbors are anxious about the situation in Iraq, ....They favor a unified Iraq that is strong enough to maintain its territorial integrity, but not so powerful as to threaten its neighbors

Meanwhile at the same time they recognizing that Iraqs most murderous militias, the Badr brigade, is a client of Iran ? What drug induced that contradiction ? RECOMMENDATION 10: The issue of Irans nuclear programs should continue to be dealt with by the United Nations Security Council. Yeah ;that's worked so well.And think about it ...the only progress the UN made was when John Bolton put a cattle prod to their a** ! The ISG's suggestion that Iran and Syria could be "helpful allies" is rediculous .It is reminiscent of Neville Chamberlain's capitulation to Hitler.


Peace in our Time.

What in his past makes Baker think that he can influence Syria in any way ? Baker traveled to Syria for meetings with the late President Hafez el-Assad and other top officials in Sept. 1990, Jan. 1991, Mar. 1991, Apr. 1991 (twice), May 1991, Jul. 1991, Sept. 1991 (twice), Oct. 1991 and Jul. 1992 (twice)in futile efforts to have them cooperate in the ME peace efforts .

The report also by implication puts on the table the issue of "right of return"."Sustainable negotiations leading to a final peace settlement along the lines of President Bush's two-state solution, which would address the key final status issues of borders, settlements, Jerusalem, the right of return and the end of conflict." The bogus "right of return " has never been part of Bush's policy;the road map ;or either of the Oslo accords .It has no place in this document .If the Palestinians have the so called right of return then the Mexicans most surely have the same right to reconquer the American SouthWest .

If I was a leader of Israel and Bush actually attempted to implement the part of the plan that calls for Israel to surrender the Golan Heights I would be tempted to break off diplomatic relations with the US.Baker wants to convene a summit of regional nations dubbed 'Madrid-2' ,and he would exclude Israel from participating .See anything wrong with that ? Madrid-2 would be promoted as a forum to discuss Iraq's future, but actually focus on Arab demands for Israel to withdraw from territories captured in the 1967 war. As Baker sees this, the conference would provide a unique opportunity for the United States to strike a deal without Jewish pressure . Appoint an anti-semite to such a position and that is what you get.

Victor Davis Hanson said it best :

Do we really believe that Iran and Syria fear chaos in the region, when chaos alone gives them a reprieve from U.S., U.N., and European scrutiny about serial assassination, terrorist promotion, and nuclear acquisition? For all the Arabist solidarity rhetoric, compared to the frightening specter of a democratic and prosperous Iraq on their borders, a returned Golan Heights, or Israeli-free West Bank means little to Syria or Iran. Did we learn after the unilateral Israeli withdrawal from Lebanon and Gaza, or the pre-9/11 American sudden flight from Lebanon and Somalia, that perceptions are far more important in the Middle East than even land?


Incoming SecDef Gates was part of this commission so I think that Bush will adopt significant portions of the groups proposals . God help us ! .

Choux... rated this answer Average Answer
Itsdb rated this answer Excellent or Above Average Answer

Question/Answer
Choux... asked on 12/06/06 - BUSH'S HEARTLESS POLICY OPPOSED BY DEMS.

"Senator Barbara Boxer (D-Calif.), incoming chairwoman of the Environment and Public Works Committee, says that she plans to back Sen. Hillary Clintons plan to provide federally-funded healthcare for workers who became sick after spending time at Ground Zero in the days immediately following the September 11 attacks. The Republican Congress and White House have resisted pleas to provide such care for years."

~~~~~~~~~~~~~~~~~~~~~~~~~~~~~~~~~~~~~~~~~~~~~~~~

tomder55 answered on 12/07/06:

I agree with Evita . Not enough has been done by either Congress or the White House.

This from the NY Daily News today :

No sooner had Japan's planes left the smoke-filled skies over Pearl Harbor 65 years ago today than Americans began to fight back. Among them were brigades of ordinary people - civilians - who joined in recovering the dead and salvaging the sunken fleet.
It was difficult work performed under dangerous conditions. Much of the effort took place underwater, with divers patching holes in bombed-out hulls and bringing live explosives to the surface. Trapped in many of the ships were flammable gasoline vapors, toxic hydrogen sulfide gas and waterlogged bodies. Oil coated everything. A diver was killed when his air hoses were severed.

As the war moved across the Pacific, more civilians stepped up in service to the nation. And America recognized that it had obligations to these citizen responders. Secretary of War Henry Stimson so stated in a letter to Congress dated July 2, 1942:

"Many cases of personal injuries and deaths of employees of government contractors already have occurred, particularly at Pearl Harbor, Wake Island and in the Philippines. It is believed that relief for such injured persons and dependents of those sustaining death should be afforded by the Congress."

Five months later, Congress passed the War Hazards Compensation Act, providing the wounded with health care and reimbursement for lost wages, and blunting the pain of fatalities with family death benefits. The law was made retroactive to Dec. 7, 1941.

Hardly a single federal lawmaker holding office today would find fault with the decision to do right by those who had served and suffered for their country. Now, after another unprovoked attack on America, Washington must tend to the needs of another group who answered the call for their country, performing difficult, dangerous work amid jagged metal and toxic substances: the forgotten victims of 9/11.

The White House and Congress must stand behind the principle that in a time of war, the United States, as a nation, has a duty to ease the tolls borne by people who have put the national interest above their own. And the U.S. must do so in this instance by guaranteeing medical care for the more than 40,000 firefighters, police officers, construction workers, volunteers and others who served at Ground Zero.

The afflictions are concentrated naturally where the act of war was perpetrated, but the illnesses extend across the country because thousands of Americans rallied from at least 35 states to do their part. Citizens from California, Florida, Texas, Oregon and points in between labored shoulder to shoulder with citizens from the metropolitan area in the poisonous cloud that was released by the collapse of the World Trade Center. Falsely assured, like New Yorkers, that the air was safe, they are just as sick as the local responders. And, dispersed to home, they are perhaps even more forgotten.

Dorothy Hall, 70, a Red Cross volunteer from Burns, Tenn., worked for six weeks at Ground Zero and now suffers from interstitial lung disease - the same type of ailment that killed Trade Center responders Detective James Zadroga, Officer James Godbee, Firefighter Stephen Johnson and AT&T Wireless technician Mark DeBiase.

Nancy Hachmeister, 50, a search-and-rescue dog handler from Bountiful, Utah, spent 10 days with her German shepherd Ivey scouring The Pile for signs of life and human remains. Ever since, Hachmeister has been plagued by sinus infections and a persistent cough, sometimes hacking so violently that she vomits. Ivey died last year of cancer at the age of 9.

Hachmeister wants Washington, including Health and Human Services Secretary Michael Leavitt, a former Utah governor, to dedicate research dollars toward 9/11-related diseases. "He needs to know there is a definite problem, and not just for the New Yorkers who were there," Hachmeister said. "They need to be aware and do what they can as far as putting money into research to figure out what's going on here."

The focus on Leavitt is spot-on. It has been three months since the secretary - caught in the glare of the fifth anniversary of the terror attack - assigned his "A-team" to formulate what, if anything, his agency would do for the forgotten victims of 9/11. Since then, Leavitt and his aides have relapsed into the torpor that has become a hallmark of their record.

Slow to recognize that debilitating asthmas, potentially fatal lung-scarring diseases, chronic coughs and other respiratory illnesses are prevalent among Trade Center responders, they have provided only piecemeal aid - and only under determined prodding by Sen. Hillary Clinton and Reps. Carolyn Maloney and Vito Fossella.

It was only in October that HHS finally delivered federal funding to actually provide care to stricken rescue and recovery workers - $40 million that will allow specialists at the Fire Department and Mount Sinai Medical Center's World Trade Center clinics to treat, rather than simply screen and monitor, the ill. The money was, of course, appreciated, but at present rates of spending it will likely run out next summer.

More money will be crucial because Mount Sinai projects that its patient census will grow from 2,000 today to 13,000. The Fire Department also will expend its funding, and responders who are scattered across the country require care - and their unaddressed needs are beyond dispute.

In Menlo Park, Calif., Frank Fraone, 46, copes with permanently damaged lungs after working at The Pile 12 hours a day for 12 days as a member of a FEMA Urban Search and Rescue team. In Kittery, Maine, ironworker Bob Glancy, 51, developed Reactive Airways Distress Syndrome, a form of asthma, after he cut Trade Center steel for a month. In Huron, Ohio, the Rev. Stephen Petrovich, 55, an Eastern Orthodox priest, lives with shortness of breath and coughs up a half cup of phlegm a day, conditions that began after he spent 11 days at Ground Zero's morgue.

As Dr. Robin Herbert, director of Mount Sinai Medical Center's World Trade Center Medical Monitoring Program Data and Coordination Center, put it in September when a study showed that almost 70% of responders in her program had new or worsening lung conditions: "There should no longer be any doubt about the health effects of the World Trade Center. Our patients are sick and will need ongoing health monitoring and treatment for the rest of their lives."

These facts should be well known to Leavitt because they are well known to Dr. John Howard, director of the National Institute of Occupational Safety and Health, who served as the secretary's 9/11 health coordinator for seven months before that "A-team" took over and promptly receded into the mists. In light of this abdication, Congress must act.

Clinton, Maloney and Fossella must continue to lead the way toward providing first-class, long-term medical care to Trade Center responders. They have urged President Bush to include a line for funding in the federal budget, and Clinton has pressed the Senate to approve a five-year $1.9 billion appropriation. Her long efforts bore fruit Tuesday: California Sen. Barbara Boxer, who will head the Senate's environment committee when the Democratic majority takes over in January, got foursquare behind Clinton's proposal.

Referring to the Ground Zero workers, Boxer said: "We definitely owe them the help to get well, yes, because they were down there because we were attacked."

Precisely.

Full-scale congressional hearings are the next order of business in compelling Leavitt to fulfill the responsibility of designing and implementing a complex, fully robust health care program. One that covers a broad range of defined illnesses; provides outpatient and inpatient services; takes care of diagnostic tests and medications; treats patients without regard to health insurance; monitors for the anticipated arrival of cancers linked to exposure to Ground Zero carcinogens for years to come, and serves patients across the country through affiliated clinics staffed by the most knowledgeable specialists. Mount Sinai's program could well be the model.

There is clear precedent for a congressional inquiry of this nature. It is to be found in the history of what happened after an earlier generation of Americans stepped to the fore in response to an act of war. On Sept. 17, 1942, Subcommittee No. 1 of the House Judiciary Committee convened to delve into the national obligation to the thousands of civilians who joined the cause of World War II, starting at Pearl Harbor.

In that zone of devastation, brigades of men in the employ of contractors helped raise five sunken ships and recover weapons and equipment from two more. Of the 5,000 dives during the two-year salvage effort, half were performed by civilians.

"The Navy was fortunate indeed to once again have the personnel and the experience of the Pacific Bridge Co.," wrote retired Vice Adm. Homer Wallin in "Pearl Harbor: Why, How, Fleet Salvage and Final Appraisal," a ship-by-ship account of the salvage effort published by the Navy in 1968. "Without them the work could not have been done."

But as civilians, the responders were not entitled to military benefits when they were injured, disabled or killed. At the same time, they were excluded from workers' compensation, which covered only employment in their home states. Appeals for help rose as the battle zone expanded.

Subcommittee No. 1, chaired by Rep. Emmanuel Celler, who represented Brooklyn in Congress for half a century, confronted the question of duty in testimony by Lt. Col. Reese Hill of the War Department's Quartermaster Corps. As Rep. Earl Michener of Michigan put it, "Colonel, we appreciate your problem exactly and I am sure we all want to help, but you can see where the burden falls. It comes back to Congress."

And, indeed, it did. After a dissection of benefit regulations and a discussion of who should bear the cost of aiding the wounded, captured and killed, Hill responded, "It is a responsibility which, in my personal opinion, should be borne by the people as a whole, since it is a part of the war activity."

True then, true today, true timelessly.

Choux... rated this answer Bad/Wrong Answer
Itsdb rated this answer Excellent or Above Average Answer

Question/Answer
Choux... asked on 12/06/06 - RIGHT WING SUPREME COURT IN ACTION

"The Supreme Court on Tuesday rejected, in an 8-to-1 ruling, the Bush administration's aggressive use of immigration laws to automatically expel legal immigrants for minor drug crimes, a decision that could spare thousands from being deported. Since 1996, more than 12 million legal immigrants have been subject to mandatory deportation if they are found guilty of drug trafficking; four years ago, the government expanded the reach of this law to include simple drug possession, which ordinarily carries only a one-year sentence."

~~~~~~~~~~~~~~~~~~~~~~~~~~~~~~~~~~~~~~~~~~~~~~~

tomder55 answered on 12/07/06:

In some countries possession of minor amts of illegal drugs will get you hanged . GO figure . This immigrant doesn't get to stay in the U.S. based on this ruling; he only gets the right to ask the Immigration officials to let him stay. They can still say no.

A legal alien convicted of any crime may be deported, but the Immigration authorities generally have discretion to deport or not, unless the crime is an "aggravated felony," . This case ( a target=_new target=_new target=_new href=http://docket.medill.northwestern.edu/archives/003525.php >Lopez v Gonzalez ) involved small-scale drug possession which would have been a misdemeanor (not a felony at all, much less an "aggravated" one) if prosecuted in federal court. But because the defendant was prosecuted in state court in a state which treated all drug possession as a felony, the Justice Department argued that he was guilty of an "aggravated felony"....a result which would have meant that aliens committing the same crime would sometimes be subject to mandatory deportation and sometimes not, depending on which court they got prosecuted in.

This was simply an interpretation of an ambiguous statute. Congress can change it if it is unhappy with the result. Clearly by the 8-1 ruling there was a problem with the way the Justice Dept was interpreting the law .

This case is relatively minor . I am more intersted in how the court decides the 'Parents Involved in Community Schls. V. Seattle Sch. Dist.'
and the 'Meredith v. Jefferson County Board of Ed.' cases about racial preferences in public schools.In each system, students are offered a choice of schools but can be denied admission based on their race if enrolling at a particular school would upset the racial balance. The question the court will have to decide is if Affirmative action has become a way of perpetuating discrimination rather than overcoming it. Already there are massive demonstrations by leftist racists outside the Court trying to intimidate the justices .




Choux... rated this answer Above Average Answer

Question/Answer
Choux... asked on 12/05/06 - At Hearing, Gates, We are Not Winning Iraq War

WASHINGTON, Dec. 5 "President Bushs nominee to be defense secretary said today that the United States was not winning the war in Iraq, and that an American failure there could help to ignite a regional conflagration in the Middle East.

Robert M. Gates, who will succeed Donald H. Rumsfeld as the Pentagons chief if he is confirmed as expected, also told senators that the United States went to war in Iraq without enough troops, as some generals said at the outset of the conflict.

The statements about the situation in Iraq came during exchanges with Senators Carl Levin of Michigan, the panels ranking Democrat and soon to be chairman, and John McCain, Republican of Arizona, during Mr. Gatess confirmation hearing before the Senate Armed Services Committee.

Mr. Gates, do you believe that we are currently winning in Iraq? Mr. Levin asked.

No, sir, Mr. Gates replied, going on to agree with the senator that a political settlement is needed to end the blood-letting, and that the United States needed to convey a sense of urgency to the Iraqis about reaching an accord.

Mr. Levin said Mr. Gatess remarks amounted to a necessary, refreshing breath of reality.

Senator McCain pursued the point about victory being elusive. We are not winning the war in Iraq, is that correct? the senator asked.

That is my view, yes, senator, Mr. Gates replied.

And therefore the status quo is not acceptable? Mr. McCain pressed.

That is correct, sir, Mr. Gates said.

He added that the United States is not losing the war, either.

His assessment came minutes after Senator John W. Warner, the Virginia Republican who heads the committee, said he believed that the United States was drifting sideways in Iraq, and that the American people are demanding change.

Mr. Gates said there clearly were insufficient troops in Iraq after the initial invasion. While he said that he envisions a dramatically smaller number of United States troops there, he said an American presence would be required for a long time.

Developments in Iraq in the next year or two will shape the future of the entire Middle East, Mr. Gates said in describing the possibility of a regional conflagration arising out of the Iraq bloodshed..."

~~~~~~~~~~~~~~~~~~~~~~~~~~~~~~~~~~~~~~~~~~~~~~~~~

I agree, a "refreshing breath of REALILTY".

tomder55 answered on 12/06/06:

Gates also said We're not losing the war in Iraq either...Gates more or less said we're not winning, nor losing the war in Iraq (what do you expect from a "realist" ).But leave it to the MSM to cherry pick a single sentence out of hours of testimony . If Gates had said outright that we were winning in Iraq, you can bet that the MSM wouldn't even have reported the story. But they love the idea of us losing in Iraq so much, that they give ample attention to it.

I happen to agree with frick . The whole confirmation process is a joke and in the current climate it behoves a candidate to tell the committee exactly what it wants to hear .He is also right in pointing out that Gates is a life long spook and they see the world differently than the military does. The Pentagon may rue the day that Rummy left .

Mr. Gates said there clearly were insufficient troops in Iraq after the initial invasion. While he said that he envisions a dramatically smaller number of United States troops there, he said an American presence would be required for a long time.

He also said that he is very open to the possibility of and the necessity of an increase in the end-strength of the Army."But that again doesn't fit in with the MSM credo so they under report that . Gen Zinni;the hero of the MSM comes out and warns against cutting and running in Iraq and even recommends sending more troops but that also got little air time.

Of more concern to me is his statement that we should not attack Syria and that he does not see the Congressional authorization of 2001 as applicable to an attack on Iran.He also bizzarely stated that Iran cannot militarily attack us ,demonstrating his lack of knowlege of history .Thankfully he will not be in the position to make the decision on Iran and Syria.

I will give him kudos about his Taiwan position.




Choux... rated this answer Excellent or Above Average Answer
Itsdb rated this answer Excellent or Above Average Answer
labman rated this answer Excellent or Above Average Answer

Question/Answer
Choux... asked on 12/05/06 - BUSH SHOWS HIS CALLOUSNESS

"At a recent White House reception, President Bush asked Sen.-elect Jim Webb (D-VA), Hows your boy? referring to Webbs son Jimmy, who is serving in Iraq. Webb answered, Id like to get them out of Iraq, Mr. President, to which Bush responded, Thats not what I asked you. Webb then replied, Thats between me and my boy, Mr. President.

The right wing has been attacking Webb for his reaction to Bushs question. Last night, Fox News host Bill OReilly said Webb was rude, inappropriate, and disrespectful, because Bush was merely trying to extend a nice gesture. The National Reviews Corner called him classless and conservative columnist George Will labeled him a boor.

But according to Rep. Jim Moran (D-VA), Bush was **told that Webbs son had a recent brush with death** in Iraq and was warned to be extra sensitive when talking to the Sen.-elect. ThinkProgress yesterday spoke with Morans office and confirmed the congressmans statement, first reported by hcc in VA:

Not only did Bush **know about it, he was *specifically briefed* on the incident before meeting with Webb**, and was cautioned to be extra sensitive in speaking with Webb about his son....."

~~~~~~~~~~~~~~~~~~~~~~~~~~~~~~~~~~~~~~~~~~~~

tomder55 answered on 12/06/06:

I'm sure Webb is the toast of the Kossacks .
They publicy express a desire to punch the President in the face on numberous postings .I'm sure Webb will whine when the Secret Service pays him a visit .

I think Bush should've avoided the moron since he was showing his rudeness by not participating on the reception line. But Bush ,being compassionate tried to reach out ,and show concern for Webb's son; a civil and caring question, as one parent to another. Web being the crude classless impolite bumpkin that he is ,as demonstrated in his graphic novel where he conjured up thoughts of performing oral sex on a son , brushed off the President ;again showing how unready for prime time he is .He is a pompous poser .He fancies himself an Andrew Jackson, but he's more like the mayor from Dukes of Hazzard.

He showed that he wanted everyone to know about his offensive behavior to the president because he was the source for the news article. He is a blowhard who will fit right in with the likes of Harry Reid , Evita Clinton, Howard Dean, Al Gore, Ted Kennedy ,Schmuck Shumer ,Carl Levin ,Joe Biden ,Dickie Turbin Durbin, and Jean F. Kerry. The democrats are setting a new record ;imploding before they are even officially seated .


The people of Va. will regret this choice and I predict that soon we will find out the true circumstances about him getting booted out of the Reagan presidency from his short tenure as Sec. Navy .

Choux... rated this answer Bad/Wrong Answer
ETWolverine rated this answer Excellent or Above Average Answer
Itsdb rated this answer Excellent or Above Average Answer

Question/Answer
Itsdb asked on 12/05/06 - On Bolton

I didn't realize anyone was still listening to John Kerry...

    Sen. John Kerry, D-Mass, said Bolton's departure could be a turning point for the administration.

    "With the Middle East on the verge of chaos and the nuclear threats from Iran and North Korea increasing, we need a United Nations ambassador who has the full support of Congress and can help rally the international community to tackle the serious threats we face," Kerry said. He said it was an opportunity for Bush to nominate an ambassador "who enjoys the support necessary to unite our country and the world and who can put results ahead of ideology."


I thought Rumsfled's departure was supposed to be a turning point. What kind of turning point is Kerry talking about? What kind of results is Kerry looking for during this time of chaos and nuclear threat that would have made Bolton the wrong man for the job? Who could he possibly have in mind that "enjoys the support necessary to unite our country and the world and who can put results ahead of ideology?" Bill Clinton, king of the world, maybe?

tomder55 answered on 12/05/06:


Kerry is too stupid to be taken seriously anymore . Let him go on Leno and make jokes about our troops .Bolton had the support in the Senate to pass a floor vote. He was held up in committee by the Dems and that idiot Lincoln Chafee .

The Wall Street Journal (no link :subscription)opens it's commentary this way :

If one measure of a man is his enemies, then John Bolton must have done something right during his six years in the Bush Administration. Yesterday's announcement that Mr. Bolton will resign as U.S. Ambassador to the United Nations when his recess appointment ends with this Congress was no doubt cheered in Tehran, Damascus, Caracas, Pyongyang, and Christopher Dodd's Senate office.


This was my comments on the expert forum :

stupid Demoncrats don't recognize that he was the best person for the UN post . No one in the country has a better handle on issues of proliferation ;which just happens to be the biggest issue facing the UN today . All the Dems talk of the coveted power of oversight is baloney .During a critical point in history the United States is without representation in the UN, because the Senate refused to perform its duty to advise and consent the president .Instead they stone-walled this appointment for petty politics .

As the Wall Street Journal said today :

It is no small diplomatic irony that, as an Undersecretary at the State Department in President Bush's first term, Mr. Bolton organized the Proliferation Security Initiative that is the one effective multilateral restraint against the spread of WMD. And it does not speak well for the American political system that it can't find a place in government at this dangerous moment in history for one of the world's leading anti-proliferation experts.

He would have been confirmed if a vote was allowed to go forward to the Senate floor . He was bottled up in the Senate Committee by a straight partisan vote. 58 senators had pledged to confirm Bolton .


I say turn Turtle Bay into the next Trump condo project .I think that Bush ought to leave the ambassador's seat at the U.N. vacant to make a statement about how useless we think the UN really is.

As for Bolton ;I think Bush should immediately place him as an undersecretary at the State Dept. to keep Condi Rice on her toes .

Itsdb rated this answer Excellent or Above Average Answer

Question/Answer
HANK1 asked on 12/04/06 - WOW!


Was the Russian spy who died of radiation poisoning a Muslim? Was he embalmed?

HANK

tomder55 answered on 12/04/06:

He did indeed convent to Islam and request a Muslim burial .

http://www.dnaindia.com/report.asp?NewsID=1067697

what is really scary is that polonium-210 is used to make nuclear triggers.It is highly toxic but Polonium 210 has a half-life of roughly 140 days . The Ruskies weaponized it and knew to give him just enough to die slowly (and give them plenty of time to leave the scene.)The short 1/2 life means that time is short to trace the material .

ON the day Alexander Litvinenko was poisoned, the Italian defence consultant Mario Scaramella handed him documents that named a private Russian security agency called 'Dignity and Honour'(headed by Colonel Valentin Velichko, a former KGB officer, it offers for hire ex-KGB spies including Spetsnaz-trained killers ) as a possible threat to his life.Most people see them as an extension of Putins secret service.

The Italian newspaper 'Corriere della Sera' gave the name of the killer as Igor Vlasov .He is a 46-year-old expert in covert operations ,belonged to Russia's Spetsnaz special forces, the paper states and was fingered in the document Litvinenko received.

Litvinenko's theory was that Putin was using terrorism as a means to crack down on terrorism . I do not know the merit of this .The revelation of his Muslim conversion adds more doubt to his claim. But one thing is sure ;the Russians have the poisioning of opponents down to a science .Just ask Ukraine's President Viktor Yushchenko (poisoned with dioxin during the Orange Revolution )

HANK1 rated this answer Excellent or Above Average Answer

Question/Answer
paraclete asked on 12/01/06 - Africa must protect itself from the crime of all time?

I have never been a Kofi Annan fan, but this piece of over the top crap demonstrates why the UN has been going nowhere in recent years.

I note he singles out the United States as the instrument of climate change rather than recognising his new friends China or his old friend Russia
as a chief cuase of climate change

Nigeria: Climate Change and Africa



Daily Trust (Abuja)

EDITORIAL
December 1, 2006
Posted to the web December 1, 2006


Kofi Annan, Africa's outgoing Secretary General of the United Nations, on 15th November, 2006, told a UN conference meeting in Nairobi, Kenya, for re-negotiating the Kyoto Protocol on "climate change", that those leaders (like President Bush and John Howard of Australia) who are not strong enough to stand up to industrialists in their econonmies and demand new technological inventions from them in order to cut down emissions of gasses which reduce carbon dioxide in the atmosphere, are suffering from "frightening lack of leadership". Such leaders do not, he argued, wish to face the terrible reality that change in the global climate is an "all-encompassing threat" to the earth and human life.

Annan strong talk came after his hands were strengthened by an official report by Sir Michael Stern released in Britain prior to the Nairobi meeting attended by over 100 cabinet ministers. The report linked harmful effects of climate change to devastating losses to the global economy. This slant in the report tilted the balance away from views which focus only Africa being the helpless main loser from "global warming". It allows focus, for example, on the combination of massive pollution in China's cities (from emissions from factories burning coal for power) with new destructive incidences of hurricanes, floods in some parts of the country, and fierce droughts in other regions. Consequently, China's economy is reported to have lost between 3 to 6 per cent of its gross national product. Drawing attention to similar losses in North America and Europe make it easy to mobilise public opinion globally for governments to take required measures.


It is, however, important to put focus on Africa. A significant element in the Kyoto Protocol is the provision that industrial economies should, by 2012, reduce their harmful gas emissions to pre-1990 levels.

Africa comes nowhere near China, India, Brazil and Mexico in contributing to pollution, yet it is already suffering most from droughts, devastating floods (like what hit Somalia and Kenya soon after the climate change conferees left Nairobi). Hurricanes have already decimated thousands of livestock, economic tree- crops, and arable crops.

African economies are weak, debt-depleted and lacking in technological capacity for both harnessing nature and fighting nature's wrath. This vulnerability is often exploited by the devise of luring African countries individually into making bilateral deals with main climate villains like the United States and Australia, from a position of weakness in isolation, instead of steadfastly fighting for multilateral treaties which are mandatory within the umbrella of United Nations treaties.

Multilateral initiatives pushed by African countries should insist on adding to international law the notion of a 'climate crime' which is punishable as derivative economic acts of war against other people's lives, economies and social systems. Factories in the United States whose industrial emissions cumulatively cause devastating droughts in Africa; hurricanes which destroy huge investments in infrastructures (in climatically vulnerable countries like Bangladesh and Jamaica), should be seen as committing climate crimes against humanity, and be made culpable for the crime.



West Africa
Nigeria



The establishment of an 'International Climate Court of Justice and Reparations' to try such willful economic warfare must be fought for immediately.

Hollywood movies made at the peak of the Cold War suggested that 'Climate' was already being seen as a potential military weapon by the super powers. What has been slow is the emergence of a legal arsenal for matching the effects of this new frontier of force.

Africa should push for its growth as a means of self defence.

tomder55 answered on 12/02/06:

I think it took real leadership to recognized the fraud that Kyoto was and to not blindly go along with conventional wisdom on it's false assumptions and rediculously dangerous remedy.

China's economy loses more because they are not quite free market yet .Their rise came from technology theft and manipulation of their currency. Their econoomy is very much a Potamkin Village that becomes more evident the further away you go from the urban centers.His linking any economic struggle to environmental issues is a straw man and a huge stretch of logic.

Drawing attention to similar losses in North America and Europe make it easy to mobilise public opinion globally for governments to take required measures. ummmm the US GDP increased at an annual rate of 2.2 percent in the third quarter of 2006.Real gross national product increased 1.9 percent in the third quarter, and is showing sustainable growth .

Africa should use it's multi lateral clout as much as there is to make free trade zones inside the continent and as leverage in trade deals. I see nothing wrong with that suggestion.

The idea of climate crimes against humanity is ludicrous. But it illustrates the dangerous dictatorial tendencies of the whole organization Annan has run and plundered from for years.

He is the one who should be brought up on charges of crimes against humanity . The UN has overseen and overlooked so much human misery in his tenure;much of which occured in Africa . While he spews his utter nonsense about climate ,the UN has yet to approve a peace mission to the Sudan . When evidence of the slaughter there was presented ,the UN debated the meaning of the word "genocide", even as the genocide was occuring right under their noses.

When they manage to send in peace keepers ,the soldiers rape and plunder the people they are there to help.The United Nations has investigated 319 peacekeeping personnel for sexual abuse since 2004. The reporting of a single US soldiers misdeeds gets covered around the world and becomes the object of world wide condemnation . Where is the outrage over this ? The blue helmet has become black and blue through self-inflicted wounds,Jane Holl Lute, a senior U.N. peacekeeping official who heads a U.N. task force on sexual exploitation, told a congressional committee investigating allegations that U.N. personnel participated in rape, prostitution and pedophilia in Congo. Weve had a problem probably since the inception of peacekeeping - problems of this kind of exploitation of vulnerable populations,...

The travesty here isnt that the UN is failing to prevent every crime before its occurred, its that the suspects are free to act with near-impunity.

I will not miss Annan and I would throw a party the day the UN disbands.


Dark_Crow rated this answer Excellent or Above Average Answer
labman rated this answer Excellent or Above Average Answer
paraclete rated this answer Excellent or Above Average Answer

Question/Answer
Choux... asked on 12/01/06 - HOW AMERICA GOT TO THIS TERRIBLE PLACE

Barry Yourgrau says the following while blogging::


"I think the answer lies in a kind of perfect storm of overlapping agendas: Bush's class bias; his personal shallowness, ineptness, and pathological narcissism (public reality reduced to the feelings in his gut) (how such a gutless man always invokes his gut, eh?)--all armor-plated and abetted by his distorted and demented misconstruction of religion. Add Cheney's Strangelovian monomania of President-as-Tyrant, along with the Neo-Con project, which Rove has hitched onto for his political purposes; and a media that comprehends only celebrity and entertainment. A media which finds war and its quasi-fascist (sorry, the q-f word applies, I think) flagflapping just the ratings/ad-selling ticket."


That seems like a good assessment to me.

tomder55 answered on 12/01/06:

lol it never ceases to amaze me how the moonbats convince themselves that the MSM is a right wing instrument. It defies logic and reason !! LOLOLOLOL The NY Slimes flag waving ???? ohhhhhh ...I can't take it !!!ROFLMAO !!!

I'd be willing to bet this Barry fella has never picked up any literature or read anything in the PNAC ,the AEI ,or the writings of Leo Strauss, Irving Kristol,Norman Podhoretz , David Horowitz,or Natan Sharansky ;and has no clue what the 'neo-con project' is . I've no doubt that he has brought into the nonsense that neo-con is a euphemism for a zionist conspiracy . Here's a clue for him . We reject appeasment and souless realpolitik.

Bush ? Despite his flaws he was the best leader for his time .

Choux... rated this answer Bad/Wrong Answer
ETWolverine rated this answer Excellent or Above Average Answer
labman rated this answer Excellent or Above Average Answer

Question/Answer
Choux... asked on 12/01/06 - PEACEFUL COUP UNDERWAY IN BAGHDAD

Tom Hayden in a special to the Huffington Post::

"A peaceful coup is being attempted in Baghdad, seeking to replace Nouri al-Maliki with a coalition between the Sunni political leader Saleh al-Mutlak and the Shiite insurgent leader Moqtada al-Sadr.

In the background are calls from Iraq's leading Shiite and Sunni clerics for an American withdrawal timetable.

Al-Mutlak, an ex-Baathist who heads the Iraqi Front for National Dialogue has eleven seats in parliament which, combined with Sadr's twenty percent bloc, is enough to destabilize or even bring down the regime of al-Maliki.

As reported last week in the Huntington Post, secret efforts to strike a deal with the Sunni nationalist resistance have been underway for months. Ex-Baathists like Mutlak, Sunnis in the Muslim Scholars Association, and in particular the revered Sunni cleric Harith al-Dhari, are strongly supportive of a political settlement based on a US withdrawal timetable. But the sudden move by al-Sadr's Shiite bloc, which pulled out of the Baghdad government over al-Maliki's meeting with Bush, provides the anti-occupation coalition with significant, perhaps decisive, power, if they choose to bring down al-Maliki's shaky coalition.

US commanders make no secret of their desire to crush al-Sadr's Mahdi Army - indeed they are waging a war of attrition - but they will be frustrated if the new coalition takes hold. Mainstream media has reported that the US has hoped to cajole the Sunnis to align with al-Maliki against al-Sadr, a scenario that seemingly is being rejected and reversed. Instead, al-Sadr's bloc is demanding a US timeline for withdrawal.

CNN' Nick Robertson featured an interview today [Thursday morning] with al-Mutlak in Baghdad, describing the unfolding transition plan as having been months in the making. It appeared that a threatened al-Maliki would have to join the call for US withdrawal, or face the possibility of being replaced by an interim government. Wolf Blitzer described the al-Maliki government as "teetering." [Earlier this year, 104 Iraqi parliamentarians, over forty percent of its membership, signed a resolution calling for an American withdrawal timetable; it was tabled under American pressure.]

Any of these scenarios would seem intolerable to the Bush Administration. But how would it respond to a demand from a reconstituted Baghdad government for a withdrawal timetable? Send more American troops into Sadr City? Facing a request from Baghdad for withdrawal, American domestic demand for a pullout could become overwhelming, even for Bush.

This week's immediate outcome cannot be predicted, depending as it does on al-Maliki's response, the US embassy's role, and above all, the determination of al-Sadr to forge a coalition with al-Mutlak across the sectarian divides.

However, al-Sadr is a well-known Arab Shiite often at odds with more pro-Irani Shiite parties like that Abd al-Aziz al-Hakim, and has been a critic of the "political quietism" of the elderly Grand Ayatollah Ali al-Sistani. His base is the Shiite urban underclass, centered in Sadr City slum. His forces fought in collaboration with the Sunnis during the American siege of Falluja in 2004, and rose against the American forces on two other occasions in 2003 and 2004. They have sent 100,000 people into the streets demanding US withdrawal, and on one occasion collected one million signatures door to door on a withdrawal petition. [for more information, see Ahmed Hashim's Insurgency and Counter-Insurgency in Iraq, Cornell, 2006]

IF INTERESTED, READ ENTIRE ARTICLE AT HUFFINGTONPOST DOT COM. IT IS LONG.


~~~~~~~~~~~~~~~~~~~~~~~~~~~~~~~~~~~~~~~~~~~~~~~

I wonder how the fluid, violent situation in Iraq will effect Bush's foreign policy in the next few months?

Will soldiers want to go there under the increase of force proposed by Bush and McCain?

tomder55 answered on 12/01/06:

sorta blows that long held myth that Sunnis and Shia cannot and will not act together in matters of mutual interest . I can't tell you how many times I heard that tired old mantra that Saddam and OBL would not work in cooperation because of the different sects they belonged to .

That being said ;I think Tom (of SDS and Chicago 7 fame) Hayden is confusing alliance shifts in a Parlimentary system that routinely occure in that form of democracy ;frequently resulting in a change in the leadership of the government ,as a "coup". I guess he has difficulties in shaking free from his revolutionary past .

It very well may be that there is a rift in the Maliki-al Sadr block that captured the majority of the seats .I do not think that is a bad thing at all. If the Sunni block could get a piece of real representation then perhaps they would abandon their rediculous ;futile ;suicidal ,"insurgency ".
It actually may be too late for the Sunni's to salvage their political position.

The irony of their position is that they started the insurgency to return to power and resist occupation .They now pray every day that the US doesn't leave .The reality is that the Sunnis brought this situation on themselves and they have only themselves to blame. I have no symmpathy whatsoever. The US in my opinion would probably be better off trying to help the Shia and Kurds consolidate their positions and absorbing the militias into a national force .(the sooner the militias are not controlled by warlords the better off everyone will be)
and saying to hell with the Sunnis .Drop some condolences cards down on Anbar Province for good measure ."Too bad ...you lose A**holes ! "


However ;as long as al-Sadr lives he wastes valuable oxygen .US commanders make no secret of their desire to crush al-Sadr's Mahdi Army - indeed they are waging a war of attrition - but they will be frustrated if the new coalition takes hold. No Sh*t! Of course we want the rediclous evil twin of the Stay Puft Marshmallow Man from Ghost Busters put down .But I don't see how this new alleged alliance with al-Mutlak could possibly increase al-Sadr's political position. He already has the best of both worlds . The Shia coalition cannot run the country without his support and he also has a free hand at ethnically cleansing his areas of influence ...with the govenments under the table support. You will recall that it was the Maliki gvt. that told the US to take down road blocks in Sadr City recently .

Personally I would not shed a tear if the Maliki gvt. goes down . People forget that he was a jihadists in the Dawa Party in Syria before the war and indeed does have too close a relationship with Tehran for my liking .I have no doubt that Dawa and Hezbollah are connected at the hip. United States Ambassador to Iraq, Zalmay Khalilzad thinks he is independent from Iran . He is mistaken .After the war Maliki was part of the disasterous Bremer DeBaathification program of the interim government ;which he administered enthusiastically .Of course he could prove me wrong by seriously cracking down on al-Sadr ;but he won't .

I wonder how the fluid, violent situation in Iraq will effect Bush's foreign policy in the next few months?

I don't think that will be the biggest influence on US foreign policy in the coming months .As Iran continues to ramp up it's lunatic leaders plans ,the US response will dominate . I think alot will be resolved in Iraq in the next year[ which I'm sure the Democrats will be more than happy to take credit for but in fact their efforts at undemining our policy will be irrelevent to the outcome ]. The aim of the war was to empty Iraq of al queda ;the terror supporter Saddam Hussein ;And create a situation in which it was unlikely for them to gain power.
The people who hit the WTC and the Pentagon were sunni ,not shia.
So if al queda is decimated and has no more support in Iraq ,and the Baathists of Saddam can no longer threaten the area then all that is left is to help stabilize the nation .btw ....much of the violence that the MSM loves to report on is Sunnis purging foreign al-qaeda jihadists from their territory . As for Saddam ;all that remains is the hangin .

Democratization is ideal but if it ultimately turns into a bridge too far then the war will still have been a success .Besides ;the seeds of democracy that were planted I believe are taking root .

Will soldiers want to go there under the increase of force proposed by Bush and McCain? of course they would !!! ;especially if we unleashed the dog.

Choux... rated this answer Excellent or Above Average Answer

Question/Answer
Itsdb asked on 11/30/06 - Hearings on Bush's Crimes Will Unite Nation

By David Swanson

An astonishing 44 percent of Americans do not want President Bush impeached (Newsweek), 36 percent approve of the job he's doing (AP-Ipsos), 33 percent support the Iraq War (CNN). What, you may ask, is the matter with these people? Well, primarily this: they get their news by watching television and occasionally glancing at a magazine or newspaper. They don't know the things you know if you listen to progressive radio or search out news on the internet or read the ends of articles that begin on page 18.

What could change their minds? Well, picture this. Close your eyes and imagine it, and I think you will find it truly beautiful: for months the celebrity stories and corporate video news releases and pseudo-journalistic sycophants who saturate American living rooms with the stench of apathy and satisfaction will be displaced. In their stead will be an endless dramatic parade of the overwhelming evidence of crimes and abuses of power committed by Bush, Cheney, and their immediate subordinates. Picture heated interrogations and recitation of evidence all morning, followed by endless commentary and chatter all afternoon. For months. Picture every man, woman, and child in America knowing in intimate detail the Downing Street Memos, the purpose of Cheney's energy meetings, the unconstitutionality of signing statements, and the history of habeas corpus from 1215 to 2006. OK, maybe not the whole history of habeas corpus, but what it is and who took it away.

What you're picturing is a shift of political power that will make the recent election look like a warm-up. What you're picturing is the key to passing legislation that will not be vetoed or signing statemented. What you are picturing is impeachment by Spring. And what can make it happen are Congressional "investigations."

I put "investigations" in quotation marks because the primary purpose of these "investigations" will be to communicate what is already public knowledge to the third of the public that's never heard of it. Much more may be uncovered, but no more is needed to completely remake American politics if people are only informed of it.

The Democrats, and any Republicans who want to stay in Congress, should "investigate" the illegal spying programs criminal creations to which Bush has already confessed, but also the myriad other crimes for which he has not as proudly taken credit.

Imagine the public learning about the al Jazeera memo in which Bush advocated bombing a television station, and the White House memo in which Bush proposed painting planes with UN colors and flying them low over Iraq in hopes of getting them shot at and helping to start the war that he was publicly claiming he wanted to avoid. Think about that for a minute. Everyone's television will show them what went on at that meeting, and then what Bush and Blair said at the press conference that followed. Then members of the media will interview the reporters who were so laughably lied to that day about their feelings of being used. Then other members of the media comment on that commentary. And so forth. Just as if an attractive white girl had been lost on a beach.

Imagine the American public hearing from the authors and participants in the various Downing Street memos and meetings. Imagine this: What if everyone, even the guy who bags your groceries and the woman who delivers your mail, even your father in law, knew what was in the memo from early 2002 that let Bush know there was almost no chance Iraq had tried to buy uranium from Niger? What if Doug Feith's and Richard Perle's careers were as well known as Oprah's, and the Project for a New American Century was a household word? What if the whole collection of evidence that the war was based on lies was reviewed in televised hearings? What if Yoo Memos were the raw material for late night talk show jokes?

And what if there were hearings on the permanent U.S. military bases being built in Iraq? What if the companies building them, the Iraqis living near them, and the officials approving them were questioned? Would Americans be pleased to learn of such an enormous expense made without Congressional approval or media acknowledgment? We you and I know that the United States has been building permanent bases in Iraq. But what if everyone knew about it in excruciating detail and it was the topic of endless chatter on their TVs?

Rep. John Dingell has already committed to investigating war profiteering and Cheney's energy task force. He should be encouraged in that, as should the many other incoming committee chair people who will do the needed "investigations." And we should encourage Nancy Pelosi to name Rush Holt chair of the Intelligence Committee, because he can be expected to do investigations that we need.

Henry Waxman, Dennis Kucinich, Diane Watson, Jerrold Nadler, John Conyers, Charles Rangel, Bennie Thompson, Bob Filner, William Delahunt, Marty Meehan: they're all chairing committees that can bring Bush and Cheney down and restore the rule of law in this nation if they only choose to do so. The same is true in the Senate for John Rockefeller, Jeff Bingaman, Joseph Biden, Patrick Leahy, Daniel Akaka, and Carl Levin who has already pledged to "investigate" extraordinary renditions. Imagine if Americans knew what extraordinary renditions were! Imagine if videos like this one put out this week by the ACLU were shown on every news show on network and cable.

What can make that happen? Congressional "investigations." Thus far the corporate media is still treating the Democrats as the meaningless minority party. But don't imagine for a minute that the media will not show Republican leaders testifying under oath in hearings on the Hill. The ratings will be too high to resist. And that third of the country that disagrees with us will be quickly brought around. Bush will finally become the uniter he has always said he is.

~~~~~~~~~~~~~~~~~~~~~~~~~~~~~~~~~~~~~~~~~~~~~~~~~~~~~~

We told you so but you wouldn't listen. This is the Democrat base, a group of moonbats so consumed with hatred for Bush they actually believe stirring up every bit of crap they possibly can will "unite" the nation. I got news for ya buddy, if you do you had better have yourself some of that body armor "that Bush failed to provide the troops" cuz' you're gonna need it.

I say he put "investigations" in quotation marks only because he isn't interested in the whole truth - just the 'truth' as he has envisioned. He has admitted what I've said on these pages many times, the left's driving purpose in life is to gain power and "bring Bush and Cheney down." The hell with everything else.

I guess I need a question, so to be an "aggressive progressive" as Mr. Swanson is, does one have to check their brain at the door?

Steve

By the way Mr. Swanson, how did I come across your drivel?

tomder55 answered on 12/01/06:


I'll wager the 36 % that support Bush are the more informed populace for the very reasons he mentions . We cannot trust the MSM as a single source of information so we communicate amongst ourselves and dig up the facts on the net ourselves . It takes vigilance and most people will not take the extra effort so when UP reports for 1/2 a year the 'eye witness accounts' of a fictional source most of the populace does not learn of the truth because the MSM (which relies on UP and Reuters to do most of their grunt work )will bury the inconvient truth .

The moonbats however take a phony story (like the "secret FEMA prisons" ) from alternet and spread it all over the blogsphere .{my guess is that if 600 prisons were being constructed by Halliburton that it wouldn't take long for it to be a front page story in the NY Slimes }

Let Waxman investigate . Honestly ;if he uncovered contractor fraud it would be a good thing . Harry Truman did that while in Congress during the early days of WWII and it helped the war effort .

In fact ;I hope they spend the next two years doing nothing but tying up Congress in endless hearings .First ...what a show !!! Better than watching the fattys lose weight or whatever other reality show they concoct. Second ...they won't have time for their other mischief.

Itsdb rated this answer Excellent or Above Average Answer
labman rated this answer Excellent or Above Average Answer

Question/Answer
ETWolverine asked on 11/30/06 - What the current state of tort law has brought us.

Or is it the current state of stupidity of the American people that is causing this? I'm betting its a combination of both.

Consumer Labels

The following are REAL warning labels on real products, that were put there by companies afraid of being sued frivolously by complete morons.

1. On Sears hairdryer:

"Do not use while sleeping."
(But..., that's the only time I have to work on my hair)

2. On a bar of Dial soap:

"Directions: Use like regular soap."
(And that would be how. . . ?)

3. On some Swanson frozen dinners:

"Serving suggestions: Defrost."
(But it's "just" a suggestion)

4. On Tesco's Tiramisu dessert (printed on bottom of box):

"Do not turn upside down."
(Oops, too late!)

5. On Marks & Spencer Bread Pudding:

"Product will be hot after heating."
(Hmm . . . .)

6. On packaging for a Rowenta iron:

"Do not iron clothes on body."
(But wouldn't this save even more time?)

7. On Boot's Children's Cough Medicine:

"Do not drive a car or operate machinery after taking this medication."
(We could do a lot to reduce the rate of construction accidents if we could just get those 5-year-olds with head colds off those forklifts.)

8. On Nytol Sleep Aid:

"Warning: May cause drowsiness."
(One would hope)

9. On most brands of Christmas lights:

"For indoor or outdoor use only."
(As opposed to underwater?)

10. On a Japanese food processor:

"Not to be used for the other use."
(I gotta admit, I'm curious.)

11. On Sainsbury's peanuts:

"Warning: Contains nuts."
(NEWS FLASH)

12. On a child's Superman costume:

"Wearing of this garment does not enable you to fly."
(I don't blame the company, I blame parents for this one.)

13. On a Swedish chain saw:

"Do not attempt to stop chain with your hands."
(Was there a chance of this happening somewhere? . . .)

14. On a bottle of Palmolive Dishwashing liquid:

"Do not use on food."
(Hey, Mom, we're out of syrup! It's OK, honey, just grab the Palmolive!)


I long ago came to the conclusion that "common sense" isn't all that common. This just proves it one more time.

Elliot

tomder55 answered on 12/01/06:

"Recycled flush water unsafe for drinking." -- On a toilet at a public sports facility in Ann Arbor, Michigan.


"This product not intended for use as a dental drill." -- On an electric rotary tool.


"Do not drive with sunshield in place." -- On a cardboard sunshield that keeps the sun off the dashboard.

"May irritate eyes." -- On a can of self-defense pepper spray.

"Do not use orally." -- On a toilet bowl cleaning brush.

"Please keep out of children." -- On a butcher knife.

"Do not use for drying pets." -- In the manual for a microwave oven.

"Warning: has been found to cause cancer in laboratory mice." -- On a box of rat poison.

"Beware! To touch these wires is instant death. Anyone found doing so will be prosecuted." -- On a sign at a railroad station.



ETWolverine rated this answer Excellent or Above Average Answer

Question/Answer
kindj asked on 11/30/06 - Now THIS is the way to do business!

With the "other" now in total power, this will be the way to go, as it will be as easy as buttering bread!


TO: Honorable Secretary of Agriculture
Washington, D.C

Dear Sir,

I have been evacuated from New Orleans because the flood took my old trailer and beat up car. I thought I might go into business to supplement my welfare check.

My friend over at Wells, Iowa received a check for $1,000 from the Government for not raising hogs. Right now I'm getting extra help from the government and Red Cross while I'm displaced but when that stops I want to go into the "not-raising-hogs" business.

What I want to know is, in your opinion, what is the best kind of farm not to raise hogs on, and what is the best breed of hogs not to raise? I want to be sure tha t I approach this endeavor in keeping with all governmental policies. I would prefer not to raise razorbacks, but if that is not a good breed not to raise, then I will just as gladly not raise Yorkshires or Durocs.
As I see it, the hardest part of this program will be in keeping an accurate inventory of how many hogs I haven't raised.

My friend, Peterson, is very joyful about the future of the business. He has been raising hogs for twenty years or so, and the best he ever made on them was $422 in 1968, until this year when he got your check for $1000 for not raising hogs. If I get $1000 for not raising 50 hogs, will I get $2000 for not raising 100 hogs? I plan to operate on a small scale at first, holding
myself down to about 4000 hogs not raised, which will mean about $80,000 the first year. Then I can afford an airplane.

Now another thing, these hogs I will not raise will not eat 100,000 bushels of corn. I understand that you also pay farmers for not raising corn and wheat. Will I qualify for payments for not raising wheat and corn not to feed the 4000 hogs I am not going to raise?

Also, I am considering the "not milking cows" business, so send me any information you have on that too. In view of these circumstances, you understand that I will be totally unemployed and plan to file for unemployment and food
stamps.

Be assured you will have my vote in the coming election.

Patriotically Yours,
Ima Taker

tomder55 answered on 12/01/06:

Them city-folks and we-ums,
Are perty much alike.
(Though they ain't used to livin' in the sticks)
We don't like stone or cement,
(But we is in agreement,)
When we gets down to talkin' politics....

The country's in the very best of hands,
The best of haaaaaaaaaaaaaands,
The best of hands.

The Treasury says the national debt is climbin' to the sky.
And government expenditures have never been so high.
It makes a fella get a gleam of pride within his eye,
To see how our economy expands.
The country's in the very best of hands!

The country's in the very best of hands,
The best of haaaaaaaaaaaaaands,
The best of hands.



You oughta hear the Senate when they drawin' up a bill.
"Whereases" and "to whits" are crowded in each codicil.
Such legal terminology would give your heart a thrill,
There's phrases there that no one understands.
The country's in the very best of hands!

The building boom they say is getting bigger every day,
And when I ask a fella, "How can everybody pay?"
He comes up with an answer that makes everything OK:
Supplies are gettin' greater than demands.
The country's in the very best of hands!

Don't you believe them Congressmen and Senators are dumb.
When they run into problems that is tough to overcome,
They just declares a thing they calls a moratorium,
The upper and the lower house disbands.
The country's in the very best of hands!

Us voters is connected to the nominee.
The nominee's connected to the Treasury.
When he ain't connected to the Treasury,
He sits around on his thigh bones.

They sits around in this place they got,
This big Congressional parking lot,
Just sits around on they you-know-whats!
Up there they calls 'em they thigh-bones.

Them bones, them bones, gonna rise again,
Gonna exercise the franchise again
While the budget doubles in size again,
When they gets up off-a their thigh-bones.
The country's in the very best of hands,
The best of haaaaaaaaaaaaaands,
The best of hands.

"The farm bill should be eighty-nine percent of parity."
Another fella recomends it should be ninety-three.
But eighty, ninety-five percent, who care's about degree,
It's parity that no-one understands.

The country's in the very best of hands!

Them GOPs and Democrats, each hates the other one.
They's always criticizing how the country should be run.
But neither tells the public what the other's one and done.
As long as no-one knows where no-one stands,
The country's in the very best of hands!

They sits around in this place they're at,
Where folks in Congress has always sat.
Just sits around on their excess fat!
Up there they calls 'em they thigh-bones.

They sits around 'til they starts to snore,
Jumps up and hollers "I has the floor!"
Then sits right down where they sat before.
Up there they calls 'em they thigh-bones.

Them bones, them bones, gonna rise again
So dignified and so wise again.
While the budget doubles in size again.
When they gets up off of their thigh-bones.
The country's in the very best of hands,
The best of haaaaaaaaaaaaaands,
The best of hands.

The money that they taxes us is known as revenues,
They compounds the collateral, subtracts the residues
Don't worry 'bout the principal, or interest that accrues,
They're shippin' all that stuff to foreign lands.

The country's in the very best of haaa-aaaaaaa-aaands!

[from Lil Abner]


ETWolverine rated this answer Excellent or Above Average Answer
Itsdb rated this answer Excellent or Above Average Answer
kindj rated this answer Excellent or Above Average Answer

Question/Answer
Itsdb asked on 11/30/06 - That darn global warming

Hurricane season ends quietly

By JESSICA GRESKO Associated Press Writer
2006 The Associated Press

MIAMI The mild 2006 Atlantic hurricane season draws to a close Thursday without a single hurricane striking the United States - a stark contrast to the record-breaking 2005 season that killed more than 1,500 people and left thousands homeless along the Gulf Coast.

Nine named storms and five hurricanes formed this season, and just two of the hurricanes were considered major. That is considered a near-normal season and well short of the rough season government scientists had forecast.

"We got a much-welcome break after a lot of the coast had been compromised in the last several years, but this is a one-season type break," said Gerry Bell, lead seasonal hurricane forecaster for the National Oceanic and Atmospheric Administration.

In May, scientists predicted 13 to 16 named storms and eight to 10 hurricanes, with four to six of them major.

The 2005 hurricane season was the busiest on record, with 28 named storms, including 15 hurricanes, four of which hit the United States, including Katrina and Rita.

Bell urged people not to become complacent about the next season, which starts June 1. Forecasters say the Atlantic is still in an active hurricane period that began in 1995 and could last another decade or more.

This year, a warm-water trend known as El Nino developed more quickly than expected in the Pacific, squashing the formation of storms in the Atlantic and creating crosswinds that can rip hurricanes apart. At the same time, upper-level air currents pushed most hurricanes out to sea, away from the U.S. mainland.

Only two storms, Tropical Storms Alberto and Ernesto, hit the U.S. mainland in 2006. Neither caused significant damage.

The season effectively ended with Hurricane Isaac, the last named storm, which dissipated Oct. 2.

~~~~~~~~~~~~~~~~~~~~~~~~~~~~~~~~~~~~~~~~~~~~~~~~~~~~~~~

It was just April when experts predicted 2006 will have about nine hurricanes (average is 5.9); 17 named storms (average is 9.6), 85 named storm days (average is 49.1); 45 hurricane days (average is 24.5); 5 intense (Category 3-4-5) hurricanes (average is 2.3) and 13 intense hurricane days (average is 5.0).

The probability of U.S. major hurricane landfall is estimated to be about 55 percent above average.


I guess they missed that one. Btw, here in Amarillo, TX we have 7 inches of snow on the ground, it's about 18 degrees, the wind chill last night was 15 below zero. it was predicted we would receive a 'light dusting.'

With all those incorrect predictions, especially the 'sky is falling' predictions on the 2006 hurricane season, what makes any of these global warming 'experts' think they can predict this alleged threat?

tomder55 answered on 11/30/06:

spreading some more inconvienient truth again ? all you need to have to be a good meteorologist is a dart board and a handsome presence in front of a green screen .

We are expected to make it up to 60 degrees here today ;but word.;.....the hawk is on the way . Your Cowboys will be bringin in the chill to the Meadowlands . Let's see how Tomo throws the ball with a swirling 60 mph wind .

Itsdb rated this answer Excellent or Above Average Answer

Question/Answer
kindj asked on 11/29/06 - December 7, 2008, began inauspiciously.

At 0753 at Pearl Harbor in Hawaii, the attack that had triggered America's entry into World War II, sixty-seven years before, was ceremoniously commemorated, an honor guard, taps, a 21-gun salute, the bugle's notes and the rifles' crack drifting across the bay to the USS Arizona memorial, where Admiral Arthur Peterson, USN Ret., laid a wreath in memory of the sailors sleeping below, one of whom was his own grandfather.

On the West coast it was 1053, and in Washington D.C. it was one fifty-three in the afternoon, 1353 military
time.

In 2006 America, tired of War in Iraq, had elected Democrats to modest majorities in both houses of Congress. Representative Nancy Pelosi became Speaker of the House, third in line for the presidency. In the spring of 2007, on a narrow, party-line vote, Congress, led by Senators John Kerry and Ted Kennedy
and Barbara Boxer refused to authorize spending to continue the war in Iraq, and set September 30, 2007, as the deadline for complete withdrawal of American troops.

President Bush spoke to the country, to the American forces in Iraq, to those who had been there, and to
the Iraqi people, to apologize for the short-sightedness and irresponsibility of the American congress and the tragedy he believed would follow after leaving task of nurturing a representative and stable government in Iraq half done, his voice choked, tears running down his stoic face, a betrayal of emotion for which he was resoundingly criticized and denounced in much of America's media.

The level of violence across Iraq immediately subsided, as the Americans began preparations to redeploy back to the States. Mahmoud Ahmadinejad praised the new Congress for its clear vision and sound judgment. America's Democrats rejoiced and congratulated themselves for bringing peace with honor
and ending the illegal war based on lies that George Bush had begun only to enrich his friends in the
military-industrial complex, and promised to retake the Presidency in 2008.

"The failure of many Americans, including many of the leading Democrats in Congress, and some Republicans, to fully appreciate the persistent, long-term threat posed to America's liberties and survival, and to the future of Liberal Democracies everywhere, by an Islamic Resistance Movement that envisions a world dominated and defined by an Islamic Caliphate of religious totalitarianism, and which will fight any
war, make any sacrifice, suffer any hardship, and pay any price to achieve it, may prove to be the kind of
blunder upon which the fate of America turns, and falls."

At 1000 on September 30, 2007, precisely on schedule, the last C-5A Galaxy carrying the last company of
American combat troops in Iraq had roared down the Baghdad runway and lifted into the air. Only a few
hundred American technical and military advisers and political liaisons remained in-country.

The Galaxy's wheels had scarcely retracted when Iraq erupted in the real civil war many had feared and
foreseen, and which many others had predicted would not happen if only the American imperialists left
Iraq. Sunni militias, Shia militias, and Al Qaeda militias ravaged and savaged the country, killing
hundreds of thousands of Iraqis known or suspected to have collaborated with the Americans, killing Shias
for being Shias, Sunnis for being Sunnis, Americans for being Americans, and anyone else who happened to be in the wrong place at the wrong time.

By noon, not one of the American advisers and liaisons left behind remained alive. Many had been beheaded as they screamed. Most of their bodies were dumped in the river and never seen again. In the next thirty days more than a million Iraqis died. The General Assembly of the United Nations voted to condemn the violence, and recessed for lunch and martinis. In America, there was no political will to redeploy back to Iraq. And after a few months of rabid bloodletting, the situation in Iraq calmed to a tense simmer of sporadic violence and political jockeying, punctuated by the occasional assassination, while several million refugees fled the country. Only Kurdistan, in the north, which had thrown up a line of its Peshmurga fighters to keep the southern violence away, remained stable and at relative peace.

In the spring of 2008 America began its quadrennial circus of a national election, and in November elected
a Democrat, the Junior Senator from New York, Hillary Rodham Clinton, as it next president, to the surprise
of few. Her running mate, to the surprise of many, was San Francisco Mayor Gavin Newsom, whose intelligence, charisma, and reputation as an indefatigable campaigner for gay marriage and the homeless of San Francisco helped solidify Clinton's support among liberal Democrats who only grudgingly forgave her for
not openly opposing the Iraq war sooner, and the Clinton-Newsom ticket went to the top with a narrow 50.2% lead over Republican John McCain's 49.8% of the popular vote, despite, or perhaps because of,
Clinton's and Newsom's lack of foreign policy and military experience.

America, or a slim voting majority of it, felt it had had all the war it ever wanted to see, and Hillary had
led her party to a glorious (if narrow) victory with the unambiguous slogan: "Clinton & Newsom: No More
War." Crowds at every whistle stop had cheered and chanted, No more war! No more war! No more war! At
victory parties George Bush, Dick Cheney, Karl Rove, Donald Rumsfeld and Condoleezza Rice were hung and sometimes burned in effigy, enthusiastic crowds chanted "No more war!" many times more, and local
bands cranked up the theme from the first Clinton electoral victory, "Don't stop thinking about tomorrow...yesterday's gone, yesterday's gone...," and indeed, it was.

President Bush had been a very lame duck since the 2006 election, and with a Democratic Congress could do little but veto most of the bills it sent him. The Democrats couldn't override his vetoes, so for nearly two years almost nothing important had been accomplished by anyone on the Hill or in the White House. After the 2008 election it was transition time, flocks and herds of thoroughly demoralized Republican staff began leaving Washington in search of greener pastures, Congress adjourned for the Holidays, Democrats came house hunting, and Clinton and Newsom began the briefings they would get from a fully cooperative Bush administration on the state of the nation and the state of the world they would inherit and have to cope with for the next four years, or eight, and in those last weeks of November both Hillary and Gavin seemed to age rather quickly. The exhilaration of the campaign was over, and the weight of a tumultuous world began to settle on their shoulders.

Back in early October, 2006, North Korean President (for life) Kim Jong Il had announced the detonation of
a nuclear bomb deep in a tunnel in the stony mountains of North Korea. The seismic signature had been small, and American intelligence at first doubted whether it had been a nuclear explosion at all. Traces of
radioactive emissions were detected a few days later, and the intelligence estimate revised to conclude that
it had been a failed test that produced perhaps only 10% or less of the expected yield, only 0.5 to 1.5
kilotons, not the 20 kilotons, at least, that Western intelligence had anticipated.


Kim Jong Il gloated. The deception had worked. The Americans were thinking in terms of long range
intercontinental ballistic missiles with huge warheads that they could shoot out of the sky with their
sophisticated billion-dollar anti-missile defense systems. He was thinking in terms of small warheads
carried by small, medium range cruise missiles that could be launched from many places, and infiltrated
close enough to slip in under the radar and hit America's coastal cities.

On the evening of December 6, 2008, a junior analyst in the National Security Agency was going over routine satellite photo production of ship movements in the Atlantic and Pacific within a thousand miles of the US coasts. Late in the shift he thought he saw something through a haze of fatigue and caffeine, and called a supervisor over to talk.

"Look," he said, photos up on several computer screens, more printed out and spread across his desk,
"See? These boats, not big ships, fishing boats, yachts, they've been moving in along shipping lanes
for several days, across from the South Pacific toward the West coast, up from the South Atlantic toward the
east. Nothing very unusual, they're all small and slow, and scattered up and down the oceans, it seems,
but if you look at the times and courses..." and he pulled out a chart he had plotted, "They're approaching so they will all arrive at about the same time, or all be about the same distance off the coast at about the same time...," he trailed off.

The supervisor looked a bit quizzical. "Coincidence? Probably. You need more sleep. Too much fun in the
night, eh? Let me know if you see something we can do something with." And walked away.

At 0723 Hawaii time on the 67th Anniversary of the Pearl Harbor attack three old fishing trawlers, about
100 miles apart, and each about 300 miles off the east coast, launched six small cruise missiles from launch
tubes that could be dismantled and stored in the holds under ice, or fish, and set up in less than an hour.
The missiles were launched at precisely one minute intervals. As soon as each boat had launched its pair,
the skeleton crew began to abandon ship into a fast rubber inflatable. The captain was last off, and just
before going overboard started the timer on the scuttling charges. Fifteen minutes later and ten miles
away, each crew was going up the nets into a small freighter or tanker of Moroccan or Liberian registry,
where each man was issued new identification as ship's crew. The rubber inflatables were shot and sunk, and just about then charges in the bilges of each of the three trawlers blew the hulls out, and they sank with
no one on board and no distress signals in less than two minutes.
The missiles had been built in a joint operation by North Korea and Iran, and tested in Iran, so they would not have to overfly any other country. The small nuclear warheads had only been tested deep underground. The GPS guidance and detonating systems had worked perfectly, after a few corrections. They flew fifty feet above sea level, and 500 feet above ground level on the last leg of the trip, using computers and terrain data modified from open market technology and flight directors, autopilots, adapted from commercial aviation units. They would adjust speed to arrive on target at specific times and altitudes, and detonate upon reaching the programmed GPS coordinates. They were not as adaptable and intelligent as American cruise missiles, but they did not need to be. Not for this mission.

They were small, less than twenty feet long, and only 18 inches in diameter, powered by small, quiet, fuel-efficient, high-bypass turbofans, and painted in a mottled light blue and light gray ghost camouflage.
Cruising at 600 knots, just below the speed of sound, they were nearly impossible to see or hear. They came in under the radar until they reached the coast. After that they were lost in the ground clutter. Nobody saw it coming.

At precisely 0753, Hawaii time, 1353 in the District of Columbia, sixty-seven years to the minute after the
Pearl Harbor attack began, the first of six missiles to hit the Washington area exploded in a huge white
burst of nuclear fire just 500 feet above the White House, which disappeared in a mist of powdered plaster and stone, concrete and steel. President Bush and President-Elect Clinton had been meeting with Condoleezza Rice and Mrs. Clinton's national security adviser, reviewing the latest National Security Estimate, when they instantaneously turned into a plasma of the atomic elements that had once been human beings. No trace remained.

Alarms immediately began going off all over Washington, and precisely one minute later the second
missile exploded just as it struck the Capital dome, instantly turning thousands of tons of granite that had one moment before been the nation's center of government into thousands of tons of granite shrapnel that shredded several square miles of Washington like a leviathan Claymore mine. At precisely one minute
intervals, four more 3 kiloton nuclear weapons exploded at an altitude of 500 feet AGL above the Pentagon, the CIA headquarters, the NSA headquarters, the FBI headquarters, all of which were fully staffed in the middle of the day. In five minutes, the government of the United States of America was decapitated, and a quarter million of the people who made the place run were dead, or dying, or had simply disappeared.

Also at 1353 Eastern time, a missile had blown off just above the New York Stock Exchange, in New York
City, and thousands of years of collective financial knowledge and experience evaporated in the nuclear
flame. In one minute intervals, others had hit the financial centers of Boston and Baltimore, and the
Naval base at Norfolk, Virginia.

Simultaneously, within the same 10-minute window of hell, nuclear tipped cruise missiles devastated the
largest intermodel shipping facility on the West coast at San Pedro harbor, exploded just above the Library
Tower in central Los Angeles, and short circuited the computer technology ghetto of Silicon Valley in Santa
Clara County, big time. One exploded ten feet away from the top of the Bank of America Building in San
Francisco and set much of the east slope of the city ablaze. Another giant fireball flared among the phalanx of office towers along the Capitol Mall in Sacramento, instantly obliterating Arnold Schwarzenegger and the state government of California, the largest state economy in the US, the seventh largest economy in the world. Two ripped open the heart of Portland, Oregon, one shattered the financial district of Seattle, and the last one turned the Microsoft campus into a pillar of fire and smoke, wiping from the face of history, in a second, the IT giant that had revolutionized global communications.

It was 0803, Hawaii time. Ten minutes.

Three million Americans dead. And not a trace of the assault fleet remained on the surface of any ocean.

Vice-President Elect Gavin Newsom was in his bedroom at home in Pacific Heights, his window overlooking the Golden Gate and the Marin bluffs. He thought he heard an oddly loud crack of thunder and saw a flash
reflected on the hills across the inlet, but it was a clear day and nothing else seemed out of place. He
continued packing for the return trip to Washington, his second since the election, to continue his transition briefings and begin organizing his staff. His nomination as Hillary's running mate had come as a huge surprise, and he was elated.

Someone rapped on the door, loudly, twice, and without waiting for a reply the senior Secret Service officer
on his detail opened it and stepped quickly in. "Come with me, now," he said. Gavin was startled. "I need to
finish packing," he replied. "No time, sir. Something has happened. Very big.
No details yet. We have to get you out of here, NOW! RIGHT NOW! GO! GO! GO!" He grabbed Newsom's arm, swung him around, and pushed him out the door, where two other Secret Service agents flanked him down the stairs and out to a running black Suburban waiting in the garage. They pushed him into the back seat, jumped in, and the driver gunned the engine, out the drive, down the street, tires squealing. Nobody spoke until they were headed over the Bridge, northbound at seventy-five miles an hour, weaving through the traffic which wasn't yet the gridlock it would soon become.

"What the hell's going on?" he finally demanded.

"Okay. This is what I know," the officer said. "The US has apparently sustained multiple nuclear attacks in
the last fifteen minutes, including Washington D.C. and San Francisco. Financial district. We're not sure
how many, at least ten, maybe twenty. Lots of dead. Got the White House, the Capital, the Pentagon. Our
job is to get you on an airplane at the nearest functioning airport, that'll be Novato, and get you to a safe place.."

"Where?" Newsom asked. Things were moving way too fast now.

"Don't know yet. We'll get orders."

The Air Force Learjet had been airborne for two minutes when a cell phone buzzed, and the Secret
Service captain answered it and handed it off to the Vice President Elect. "It's Mr. Cheney, sir," he said.

"Gavin?" Dick Cheney asked. "Yes, sir," Newsom replied, subdued, for the events of the last hour had
sobered up his elated mood considerably.

"Okay, Gavin. I don't know what you know, so I'll tell you what I can. There have been approximately 20 nuclear strikes on government and financial targets in the US, about an hour ago. No real damage estimate
yet, except that it's awful. A hundred times 9/11, maybe a thousand times. I happened to be at the Air
Force Academy in Colorado Springs, and have moved into Cheyenne Mountain to set up a temporary HQ, until we get things sorted out. As you know Cheyenne was vacated by NORAD a few years ago, so we have plenty of space. You will be flown here, nonstop."

"I know you haven't a lot of national and international experience." Cheney had thought of saying that Newsom had none, but Newsom would be too painfully aware of that. He didn't need reminding. "The President is missing and presumed dead. So is Mrs. Clinton. So you may become the next president, in
about six weeks. I don't know. he Constitution says the Vice President succeeds a president who is dead or
disabled, but it doesn't say what happens if the President Elect dies before being inaugurated. I suppose the Court will have to answer that, if we can cobble one together by then. In the meantime, I will assume you will be inaugurated. You'll have a steep learning curve, a real steep curve. All presidents do, under the best of circumstances, and these are not the best of circumstances."

The next day a hard winter storm roared down the West coast from Alaska, pelting rescue workers in bombed out city centers with hard, cold rain, that did not let up for a week. People alive but injured or trapped in the wreckage died of hypothermia before they were found. Two days later, a cold front out of Canada brought heavy snow to the Northeast. Millions were already without electricity, and in a week of subzero weather hundreds of thousands more died. More than four million, altogether. More than one of every one hundred Americans.

Al Qaeda had picked December 7 because it was the anniversary of the attack on Pearl Harbor, and
because, just before Christmas, the Infidel holiday, it would destroy the Christmas shopping season so
important to so many retailers, driving another nail into the national economy of the Great Satan. And it
would destroy the festive spirit of the season for millions of Americans, perhaps for all. The perfect
psyop. Psychological warfare. And the weather forecasters had predicted severe winter storms on both
coasts during the week immediately after disaster.

Al Qaeda leaders had calculated, correctly, that by turning up the violence in Iraq during the weeks
before the 2006 election it could achieve an anti-war Democratic Congress that would vote to end America's
wars in the Middle East, and then by turning down the violence in Iraq after the election of an anti-war
Democratic Congress, it could lull America into a false sense of safety and security in anticipation of
the "peace in our time" that America's new ruling party had promised would follow from what Al Qaeda
perceived, correctly, as America's retreat before the unstoppable determination of the Islamic Resistance
Movement, the Jihad. America did not call it that, of course. The Americans thought they were just ending a
bad and illegal war ginned up by George W. Bush to depose Saddam Hussein who had proven not to have WMDs after all, the ones the Americans had never found, the ones buried in Syria. Al Qaeda saw more clearly. It was a capitulation, a de facto surrender of the Middle East to the coming Islamic Caliphate that would someday rule the world. The martyrs of Islam had beaten the Great Satan to its knees. In time they
would cut off its head.

By Christmas, the American economy had imploded. Inflation soared, unemployment soared, businesses
closed, cities that had suffered direct hits became ghost towns. Tax revenues evaporated, leaving state
governments without funds to pay unemployment benefits or teachers' salaries. With the New York Stock
Exchange gone, stock trading ended, and values plummeted. Retirement assets and pension funds
disappeared in a wink. Nobody knew what to expect. Real estate crashed, and major banks filed for
bankruptcy. With the collapse of the American economy, the largest on earth, the most productive country on earth, with just 5% of the global population producing one third of the global economic output, the rest of
the global economy fell into chaos. Oil shipments stopped, food shipments stopped, and in that winter
millions of people in third world countries starved to death.

The America era was over.

"In the spring of 1941, Nazi Germany was poised to dominate the earth. France, the low countries, Norway,
Denmark, Austria, Czechoslovakia, Yugoslavia, Greece, and much of Poland had been overrun by the Germans. All of Europe, save neutral Sweden and Switzerland, was in the hands of Hitler's friends and allies: dictators or monarchs who ruled fascist Italy, Vichy France, Franco's Spain, Portugal, the Balkan
countries, Finland, and above all the Soviet Union."

"A single German division under General Erwin Rommel, sent to rescue beleaguered Italians in Libya, drove
Britain's Middle Eastern armies flying and threatened the Suez lifeline; while in Iraq a coup d'etat by the
pro-German Rashid Ali cut the land road to India. In Asia, Germany's ally, Japan, was coiled to strike,
ready to take Southeast Asia and invade India. No need to involve the United States; by seizing the Indies,
Japan could break the American embargo and obtain all the oil needed for the Axis Powers to pursue their war aims.

"Hitler should have sent the bulk of his armies to serve under Rommel, who would have done what Alexander did and Bonaparte failed to do: He would have taken the Middle East and led his armies to India. There he would have linked up with the Japanese. Europe, Asia, and Africa, would have belonged to the coalition of dictators and militarists."

"The Nazi-Soviet-Japanese alliance commanded armed forces and resources that utterly dwarfed the military resources that the holdouts, Britain (with its empire), and the United States, could field. The
English-speaking countries would have been isolated in a hostile world and would have had no realistic option but to make their peace with the enemy, retaining some autonomy for a time, perhaps, but doomed ultimately to succumb. Nazi Germany, as leader of the coalition, would have ruled the world."

"Only Hitler's astonishing blunder in betraying and invading his Soviet ally kept it from happening." -
David Frompkin, Professor of International Relations and History, Boston University, writing in What If:
Eminent Historians Imagine What Might Have Been
(Putnam 1999) pp. 308,
309.

History is made, wars are won and lost, cultures and nations and civilizations come and go, rise and fall,
as much by blunders as by victories.

The failure of many Americans, including many of the leading Democrats in Congress, and some Republicans, to fully appreciate the persistent, long-term threat posed to America's liberties and survival, and to the future of Liberal Democracies everywhere, by an Islamic Resistance Movement that envisions a world dominated and defined by an Islamic Caliphate of religious totalitarianism, and which will fight any
war, make any sacrifice, suffer any hardship, and pay any price to achieve it, may prove to be the kind of
blunder upon which the fate of America turns, and falls.

Raymond S. Kraft is an attorney and writer in northern California.

tomder55 answered on 11/30/06:

Dennis

The only change I would make would be to place an aerosol based bioweapon in the warheads, rather than nuclear.

With the assasination of Alexander Litvinenko using aerosol based polonium-210 ,a nuclear agent ,your scenario is more chilling and plausable.

My first thought along this line came weeks ago during the test itself .In my scenario the Iranians ,who were clearly there at the test detonations as well as the successful launches of the intermediate range missiles ;and who most likely financed the project, obtain nukes and delivery systems from the NORKS and use them to shut down the Straits of Hormuz and perhaps target an American carrier. The Mahdi-hatter Ahmamadjihad would not really care about retaliation since he thinks he is ushering in the return of the 12th Imam who was last seen falling down a well I believe.

kindj rated this answer Excellent or Above Average Answer

Question/Answer
paraclete asked on 11/29/06 - Now it's not PC for terrorists to be Muslim?


Correctness has gone too far

By Karen Brooks

November 29, 2006 01:00am
Article from: The Courier-Mail


IN HER opening address at the Brisbane Writers Festival this year, British author Lionel Shriver (winner of the Orange Prize for fiction) made the point that political correctness has become so tired that "it's now correct to despair of political correctness".

We've seen ridiculous examples of the so-called "PC brigade" at work lately. From renaming "fairy" penguins to demanding that cheerleaders "cover up", never mind the dulling-down of Australia's favourite summer sport to the point where it's just not cricket.

Perhaps the most disturbing example of PC at work, however, centres on author John Dale's newest book, Army of the Pure.

Commissioned by leading publisher Scholastic Australia to produce a ripping action-adventure yarn that would have kids (boys particularly), turning the pages with anticipation, Dale more than fulfilled his brief.

The book has been described as "almost flawless".

It's about a group of kids in a soccer club who stumble upon a terrorist plot to blow up the Lucas Heights nuclear reactor in Sydney's south, and are chased by an extremist group who also happen to be Arabic-speaking.

The tale resonates in a world where nightly TV bulletins and newspapers regularly report on terrorism, religious schisms and global politics.

But now librarians and bookshops have refused to stock the book, so it has been withdrawn. Dale is appalled at the decision, which is based solely on what has been described as the "Muslim issue".

Andrew Berkhut, general manager at Scholastic Australia, said: "The reality is if the gatekeepers won't support it, it can't be published."

Dale's literary agent, Lyn Tranter, has called the decision not to stock the book as "gutless" and described it as "censorship by salesmen".

What on this divided, angry, diverse and wonderful Earth is going on here? Are we so prissy and precious, so "PC" that we can't abide marketing a work of fiction because it dares to imagine bad (and good) characters who are Muslim?

Do I detect an overwhelming odour of hypocrisy here?

We live in a society where on the one hand, political correctness is enshrined as "sensitivity" and hauled out and fashioned to suit any occasion, while on the other, portrayals of Islam, Muslims and anybody from the Middle East are unremittingly stereotypical, narrow, often taken out of context and used to incite hatred.

Rebuffing a work of fiction on the basis of a "Muslim issue" is offensive. It somehow implies that the representations within the book are more than imaginative, and that, yet again, a few characters stand in for all. Ironically, those who protest against the content are doing exactly what they foolishly believe they're preventing: stereotyping and reducing a diverse people to cliches.

But the negative response to Dale's book has even wider implications for all creative artists.

Holding an imaginative mirror up to society and culture has happened since Homer first chanted his verses of gods, war, hubris and homecoming. It's called verisimilitude.

A misuse of political correctness has meant that an author's right to freedom of expression in the creative domain has been quashed.

In The Republic, Plato argued that poets should be banished from his ideal state. He believed that people confused poetic representations with the "truth".

Likewise, Russian author Leo Tolstoy wanted to banish artists who aroused morally questionable feelings. His list of exiles included Sophocles, Shakespeare and Beethoven.

Ironically, it's because of the great artists, writers and thinkers (including Plato and Tolstoy), that we're able to embrace works that challenge us, that dare to speak the unspeakable, portray what should not be shown, and that stand up to the status quo, forcing us to feel, to question "truth" and slowly rethink our positions.

These works don't simply litter the streets of cultural memories; they're the stones and bones of our past. They're the foundation of the future.

Whereas journalism ideally works in binarisms, presenting "both sides of the story", creative artists are not so obliged. We rely on them to plunge us into the lives and psyches of different characters: good, bad and, yes, Muslim too. Without artistic licence, we stifle the creative impulse, curb imaginative expression and invite Orwell's thought police into our communities and, worse, our heads.

Our Government has made "Muslim" an "issue", not a kids' book that imagines adventure, heroism, friendship and loyalty and happens to portray heroic and villainous Muslim characters, among others.

Whether we agree or disagree with the content and characters our artists create is irrelevant: don't let being afraid (disguised as PC sensitivity) generate censorship.

tomder55 answered on 11/30/06:

PC drives me nuts . It has been many years since a Muslim terrorist was even the villian in an American movies. Even the great Tom Clancy's novel ;'The Sum of all Fears ',about Muslims smuggling a nuke into the US to blow up the city that the SuperBowl was playing in ;was changed in the movie form to have neo-nazi Americans as the villians .

paraclete rated this answer Excellent or Above Average Answer

Question/Answer
Choux... asked on 11/29/06 - Atheist Comments on American Politics

Sam Harris comments:

"While the religious divisions in our world are self-evident, many people still imagine that religious conflict is always caused by a lack of education, by poverty, or by politics. Yet the September 11th hijackers were college-educated, middle-class, and had no discernible experience of political oppression. They did, however, spend a remarkable amount of time at their local mosques talking about the depravity of infidels and about the pleasures that await martyrs in Paradise.

How many more architects and mechanical engineers must hit the wall at 400 miles an hour before we admit to ourselves that jihadist violence is not merely a matter of education, poverty, or politics? The truth, astonishingly enough, is that in the year 2006 a person can have sufficient intellectual and material resources to build a nuclear bomb and still believe that he will get 72 virgins in Paradise. Western secularists, liberals, and moderates have been very slow to understand this. The cause of their confusion is simple: They don't know what it is like to really believe in God.

The United States now stands alone in the developed world as a country that conducts its national discourse under the shadow of religious literalism. Eighty-three percent of the U.S. population believes that Jesus literally rose from the dead; 53% believe that the universe is 6,000 years old. This is embarrassing. Add to this comedy of false certainties the fact that 44% of Americans are confident that Jesus will return to Earth sometime in the next 50 years and you will glimpse the terrible liability of this sort of thinking.

Nearly half of the American population is eagerly anticipating the end of the world. This dewy-eyed nihilism provides absolutely no incentive to build a sustainable civilization. Many of these people are lunatics, but they are not the lunatic fringe. Some of them can actually get Karl Rove on the phone whenever they want.

While Muslim extremists now fly planes into our buildings, saw the heads off journalists and aid-workers, and riot by the tens of thousands over cartoons, several recent polls reveal that atheists are now the most reviled minority in the United States. A majority of Americans say they would refuse to vote for an atheist even if he were a "well-qualified candidate" from their own political party. Atheism, therefore, is a perfect impediment to holding elected office in this country (while being a woman, black, Muslim, Jewish, or gay is not). Most Americans also say that of all the unsavory alternatives on offer, they would be least likely to allow their child to marry an atheist. These declarations of prejudice might be enough to make some atheists angry. But they are not what makes me angry.

As an atheist, I am angry that we live in a society in which the plain truth cannot be spoken without offending 90% of the population. The plain truth is this: There is no good reason to believe in a personal God; there is no good reason to believe that the Bible, the Koran, or any other book was dictated by an omniscient being; we do not, in any important sense, get our morality from religion; the Bible and the Koran are not, even remotely, the best sources of guidance we have for living in the 21st century; and the belief in God and in the divine provenance of scripture is getting a lot of people killed unnecessarily."

~~~~~~~~~~~~~~~~~~~~~~~~~~~~~~~~~~~~~~~~~~~~~

Interesting points. Anyone wish to comment on what he said?

Those usual suspects who deposit hate, propaganda and lies as their answer....get one star....**I don't read your bs in answers or posts for that matter.

All I have to do is read the first sentence to detect same. :)


Serious answers welcome.

tomder55 answered on 11/30/06:

Sayyid Qutb ;the intellectual guru of the modern day jihadist was fortunate enough to receive some prime US education from some of our finest institutions of secular progressive learning. And what an education it was !! He learned all about America's materialism , triviality, restrictions on divorce, sexual permissiveness ,indulgences and the overall lack of a moral base from these liberal universities .It was lessons he took to heart;completely rejected ,and that laid the foundation for his radical intepretation of Islam. He went back to Egypt and joined the Muslim Brotherhood.

the American girl is well acquainted with her body's seductive capacity. She knows it lies in the face, and in expressive eyes, and thirsty lips. She knows seductiveness lies in the round breasts, the full buttocks, and in the shapely thighs, sleek legs -- and she shows all this and does not hide it.....


Jazz is his preferred music, and it is created by Negroes to satisfy their love of noise and to whet their sexual desires .

(from his writing "The America I Saw")

He came back from our secular institutions of learning a radicalized man. Now keep in mind ;this was in the early 1950s BEFORE the "sexual revolution" . Clearly he saw the path towards moral decay that was being advocated by our secular institutions he studied in .

Sam Harris is incorrect . It is not the religious aspect of American life that pisses off the radical Islamist ;it is the permissiveness that atheists and secular progressives embrace that they condemn. When we become the 'Dhimmi States Americastan'their first order of business will be to go after the excesses that the secularists /atheists embrace.

Choux... rated this answer Bad/Wrong Answer
ETWolverine rated this answer Excellent or Above Average Answer
Itsdb rated this answer Excellent or Above Average Answer
kindj rated this answer Excellent or Above Average Answer
labman rated this answer Excellent or Above Average Answer

Question/Answer
Itsdb asked on 11/29/06 - Supreme Court takes up global warming for first time

WASHINGTON - "Frustrated by Bush administration inaction on global warming, states and environmentalists urged the Supreme Court Wednesday to declare greenhouse gases to be air pollutants that the government must regulate.

The courts first case on the politically charged topic showed an apparent split between its liberal and conservative justices, with Anthony Kennedy potentially the decisive vote in determining whether the administration must abandon its refusal to treat carbon dioxide and other greenhouse gases as air pollutants that imperil public health.

Justice Samuel Alito, who with Chief Justice John Roberts seemed most skeptical of the states position, said that even in the best of circumstances, the reduction in greenhouse gases would be relatively small.

Justice David Souter indicated that every little bit would help. They dont have to show that it will stop global warming. Their point is that will reduce the degree of global warming and likely reduce the degree of loss, he said.

The case involves whether the Environmental Protection Agency must regulate emissions of greenhouse gases from new vehicles under a provision of the Clean Air Act. When a decision comes sometime before July, it could have a significant ripple effect that could extend to power plants as well as states efforts to impose more stringent regulations on car tailpipe emissions.

Many scientists believe that greenhouse gases, flowing into the atmosphere at an unprecedented rate, are leading to a warming of the Earth, rising sea levels and other marked ecological changes.

Carbon dioxide, the principal greenhouse gas, is produced when fossil fuels such as oil and natural gas are burned. One way to reduce those emissions is to have more fuel-efficient cars.

We own property, 200 miles of coastline, that were losing, Massachusetts assistant attorney general James Milkey said on behalf of 12 states and 13 environmental groups that sued EPA.

Deputy Solicitor General Gregory Garre, representing the Bush administration, cautioned justices that EPA regulation could have a significant economic impact on the United States because 85 percent of the U.S. economy is tied to sources of greenhouse gas emissions.

Garre also argued that the EPA was right not to act given the substantial scientific uncertainty surrounding global climate change.

Roberts pointed out that regulating carbon dioxide emissions from new vehicles addresses just one aspect of an issue of global dimensions.

The argument by those pushing for EPA action on vehicle emissions might or might not be valid, but it assumes everything else is going to remain constant, Roberts observed.

Whether Roberts was correct, Congress is expected to become more involved next year in the debate on global warming because newly empowered Democrats have promised to give the issue a thorough airing..."

Well, now that we have the Democrats in office all our worries will be over...

tomder55 answered on 11/30/06:

Congress has never given the EPA regulatory authority over greenhouse gases...[period....end of discussion....right? ].... so on the surface this should be a no brainer .They have a mandate to enforce relevant statutory provisions about soot, smog and acid rain that affects local and regional or even national concerns ;not global concerns or climate .But Twelve states, three cities and numerous groups say the Clean Air Act plainly requires the agency to do so .They say The Clean Air Act requires EPA to regulate any "air pollutant" its administrator finds could "reasonably be anticipated to endanger public health or welfare." There in is the justification for them taking it to the Supremes.

The scary part of all of this is that again it will be the single vote of Justice Anthony Kennedy that will decide what could be a monumental decision. The great debate about global warming could now be permanently be decided on how justice Kennedy is feeling on the day of the decision. (what is his schedule for the upcoming DC cocktail party circuit? ). He may very well decree that we can only breath 1/2 as often as we are now doing .

In favor of the feds regulating CO2 :Ginsburg, Souter("Every little bit helps." ...what an idiot !! ), Stevens, Breyer

Against : Alito, Stevens, Thomas, Scalia

Deciding vote : Anthony Kennedy (uugh )


Things are not looking good . Kennedy in oral arguments (most of the arguments were about "standing " which requires the plaintiff to show "injury " ) brought up that obscure case about sulferous emissions that to any rational person would appear irrelevent to the case since there is really no debate that sulfuric acid is a pollutant .

Judge Ginsberg listened attentively to the
oral arguments



Look for Arnie to immediately propose punative regulations and hefty fines on businesses in Kalifornia once SCOTUS finds in favor of regulating CO2 (he already attempted it this year on vehicle emissions) . I don't doubt that Eliot Spitzer in NY won't be far behind.

Itsdb rated this answer Excellent or Above Average Answer

Question/Answer
arcura asked on 11/29/06 - The NEW economics according to Nancy Pelosi....

Published: October 22, 2006

Nancy Pelosi condemed the new record highs of the stock market as "just another example of Bush policies helping the rich get richer".

"First Bush cut taxes for the rich and the economy has rebounded with new record low unemployment rates, which only means wealthy employers are getting even wealthier at the expense of the underpaid working class".


She went on to say "Despite the billions of dollars being spent in Iraq our economy is still strong and government tax revenues are at all time highs. What this really means is that business is exploiting the war effort and working Americans just to put money in their own pockets".

When questioned about recent stock market highs she responded "Only the rich benefit from these record highs. Working Americans, welfare recipients, the unemployed and minorities are not sharing in these
obscene record highs".

"There is no question these windfall profits and income created by the Bush administration need to be taxed at 100% rate and those dollars redistributed to the poor and working class".

"Profits from the stock market do not reward the hard work of our working class who, by their hard work, are responsible for generating these corporate profits that create stock market profits for the rich.
We in congress will need to address this issue to either tax these profits or to control the stock market to prevent this unearned income to flow to the rich".

When asked about the fact that over 80% of all Americans have investments in mutual funds, retirement funds, 401K's, and the stock market she replied "That may be true, but probably only 5% account for
90% of all these investment dollars. That's just more "trickle down" economics claiming that if a corporation is successful that everyone from the CEO to the floor sweeper benefit from higher wages and job security which is ridiculous". How much of this "trickle down" ever get to the unemployed and minorites in our county? None, and that's the tragedy of these stock market highs."

"We democrats are going to address this issue after the election when we take control of the congress. We will return to the 60% to 80% tax rates on the rich and we will be able to take at least 30% of all currrent
lower Federal Income Tax taxpayers off the roles and increase government income substantially. We need to work toward the goal of equalizing income in our country and at the same time limiting the amount the rich can invest."

When asked how these new tax dollars would be spent, she replied "We need to raise the st andard of living of our poor, unemployed and minorities. For example, we have an estimated 12 million ILLEGAL immigrents in our country who need our help along with millions of
unemployed minorities. Stock market windfall profits taxes could go a long ways to guarantee these people the standard of living they would like to have as Americans"
Quesrion: Does she frighten you as much as she does me?
She is going to tax me to give my money to illegal people who are have a very high rate of visicous crimes.

tomder55 answered on 11/29/06:

Fred ; I tried to warn you that the moderate face that the Dems were showing in the last elections was the face of a Trojan Horse ....Inside was lurking extreme socialist liberalism. Do you really expect anything less from the San Francisco demoncrats machine?

She is also pretty clueless about the stock market. Frankly ;with people being empowered to manage their own retirement funds through the ever increasing use of 401-Ks and IRA funds ,the investor class has increased to include most working people .A rise in the stock market lifts all boats . She really ought to honor Milton Friedman by reading some of his economic theories .

arcura rated this answer Excellent or Above Average Answer
ETWolverine rated this answer Excellent or Above Average Answer

Question/Answer
Itsdb asked on 11/27/06 - Cut and Run, the Only Brave Thing to Do

An addendum to my reply to Choux...

Sunday, November 26th, 2006

Friends,

Tomorrow marks the day that we will have been in Iraq longer than we were in all of World War II.

That's right. We were able to defeat all of Nazi Germany, Mussolini, and the entire Japanese empire in LESS time than it's taken the world's only superpower to secure the road from the airport to downtown Baghdad.

And we haven't even done THAT. After 1,347 days, in the same time it took us to took us to sweep across North Africa, storm the beaches of Italy, conquer the South Pacific, and liberate all of Western Europe, we cannot, after over 3 and 1/2 years, even take over a single highway and protect ourselves from a homemade device of two tin cans placed in a pothole. No wonder the cab fare from the airport into Baghdad is now running around $35,000 for the 25-minute ride. And that doesn't even include a friggin' helmet.

Is this utter failure the fault of our troops? Hardly. That's because no amount of troops or choppers or democracy shot out of the barrel of a gun is ever going to "win" the war in Iraq. It is a lost war, lost because it never had a right to be won, lost because it was started by men who have never been to war, men who hide behind others sent to fight and die.

Let's listen to what the Iraqi people are saying, according to a recent poll conducted by the University of Maryland:

** 71% of all Iraqis now want the U.S. out of Iraq.

** 61% of all Iraqis SUPPORT insurgent attacks on U.S. troops.

Yes, the vast majority of Iraqi citizens believe that our soldiers should be killed and maimed! So what the hell are we still doing there? Talk about not getting the hint.

There are many ways to liberate a country. Usually the residents of that country rise up and liberate themselves. That's how we did it. You can also do it through nonviolent, mass civil disobedience. That's how India did it. You can get the world to boycott a regime until they are so ostracized they capitulate. That's how South Africa did it. Or you can just wait them out and, sooner or later, the king's legions simply leave (sometimes just because they're too cold). That's how Canada did it.

The one way that DOESN'T work is to invade a country and tell the people, "We are here to liberate you!" -- when they have done NOTHING to liberate themselves. Where were all the suicide bombers when Saddam was oppressing them? Where were the insurgents planting bombs along the roadside as the evildoer Saddam's convoy passed them by? I guess ol' Saddam was a cruel despot -- but not cruel enough for thousands to risk their necks. "Oh no, Mike, they couldn't do that! Saddam would have had them killed!" Really? You don't think King George had any of the colonial insurgents killed? You don't think Patrick Henry or Tom Paine were afraid? That didn't stop them. When tens of thousands aren't willing to shed their own blood to remove a dictator, that should be the first clue that they aren't going to be willing participants when you decide you're going to do the liberating for them.

A country can HELP another people overthrow a tyrant (that's what the French did for us in our revolution), but after you help them, you leave. Immediately. The French didn't stay and tell us how to set up our government. They didn't say, "we're not leaving because we want your natural resources." They left us to our own devices and it took us six years before we had an election. And then we had a bloody civil war. That's what happens, and history is full of these examples. The French didn't say, "Oh, we better stay in America, otherwise they're going to kill each other over that slavery issue!"

The only way a war of liberation has a chance of succeeding is if the oppressed people being liberated have their own citizens behind them -- and a group of Washingtons, Jeffersons, Franklins, Ghandis and Mandellas leading them. Where are these beacons of liberty in Iraq? This is a joke and it's been a joke since the beginning. Yes, the joke's been on us, but with 655,000 Iraqis now dead as a result of our invasion (source: Johns Hopkins University), I guess the cruel joke is on them. At least they've been liberated, permanently.

So I don't want to hear another word about sending more troops (wake up, America, John McCain is bonkers), or "redeploying" them, or waiting four months to begin the "phase-out." There is only one solution and it is this: Leave. Now. Start tonight. Get out of there as fast as we can. As much as people of good heart and conscience don't want to believe this, as much as it kills us to accept defeat, there is nothing we can do to undo the damage we have done. What's happened has happened. If you were to drive drunk down the road and you killed a child, there would be nothing you could do to bring that child back to life. If you invade and destroy a country, plunging it into a civil war, there isn't much you can do 'til the smoke settles and blood is mopped up. Then maybe you can atone for the atrocity you have committed and help the living come back to a better life.

The Soviet Union got out of Afghanistan in 36 weeks. They did so and suffered hardly any losses as they left. They realized the mistake they had made and removed their troops. A civil war ensued. The bad guys won. Later, we overthrew the bad guys and everybody lived happily ever after. See! It all works out in the end!

The responsibility to end this war now falls upon the Democrats. Congress controls the purse strings and the Constitution says only Congress can declare war. Mr. Reid and Ms. Pelosi now hold the power to put an end to this madness. Failure to do so will bring the wrath of the voters. We aren't kidding around, Democrats, and if you don't believe us, just go ahead and continue this war another month. We will fight you harder than we did the Republicans. The opening page of my website has a photo of Nancy Pelosi and Harry Reid, each made up by a collage of photos of the American soldiers who have died in Bush's War. But it is now about to become the Bush/Democratic Party War unless swift action is taken.

This is what we demand:

1. Bring the troops home now. Not six months from now. NOW. Quit looking for a way to win. We can't win. We've lost. Sometimes you lose. This is one of those times. Be brave and admit it.

2. Apologize to our soldiers and make amends. Tell them we are sorry they were used to fight a war that had NOTHING to do with our national security. We must commit to taking care of them so that they suffer as little as possible. The mentally and physically maimed must get the best care and significant financial compensation. The families of the deceased deserve the biggest apology and they must be taken care of for the rest of their lives.

3. We must atone for the atrocity we have perpetuated on the people of Iraq. There are few evils worse than waging a war based on a lie, invading another country because you want what they have buried under the ground. Now many more will die. Their blood is on our hands, regardless for whom we voted. If you pay taxes, you have contributed to the three billion dollars a week now being spent to drive Iraq into the hellhole it's become. When the civil war is over, we will have to help rebuild Iraq. We can receive no redemption until we have atoned.

In closing, there is one final thing I know. We Americans are better than what has been done in our name. A majority of us were upset and angry after 9/11 and we lost our minds. We didn't think straight and we never looked at a map. Because we are kept stupid through our pathetic education system and our lazy media, we knew nothing of history. We didn't know that WE were the ones funding and arming Saddam for many years, including those when he massacred the Kurds. He was our guy. We didn't know what a Sunni or a Shiite was, never even heard the words. Eighty percent of our young adults (according to National Geographic) were not able to find Iraq on the map. Our leaders played off our stupidity, manipulated us with lies, and scared us to death.

But at our core we are a good people. We may be slow learners, but that "Mission Accomplished" banner struck us as odd, and soon we began to ask some questions. Then we began to get smart. By this past November 7th, we got mad and tried to right our wrongs. The majority now know the truth. The majority now feel a deep sadness and guilt and a hope that somehow we can make make it all right again.

Unfortunately, we can't. So we will accept the consequences of our actions and do our best to be there should the Iraqi people ever dare to seek our help in the future. We ask for their forgiveness.

We demand the Democrats listen to us and get out of Iraq now.

Yours,

Michael Moore
www.michaelmoore.com
mmflint@aol.com

~~~~~~~~~~~~~~~~~~~~~~~~~~~~~~~~~~~~~~~~~~~~~~~~~~~~~~

I replied to the previous post prior to reading this email from Moore in which I said, "That is exactly what the left wants us to do, give up..."

    Moore: As much as people of good heart and conscience don't want to believe this, as much as it kills us to accept defeat...


"cut our losses..."

    Moore: Bring the troops home now. Not six months from now. NOW. Quit looking for a way to win. We can't win. We've lost. Sometimes you lose. This is one of those times. Be brave and admit it.


"add a good dose of self-chastisement..."

    Moore: Apologize to our soldiers and make amends. Tell them we are sorry they were used to fight a war that had NOTHING to do with our national security...The families of the deceased deserve the biggest apology and they must be taken care of for the rest of their lives...We must atone for the atrocity we have perpetuated on the people of Iraq. There are few evils worse than waging a war based on a lie, invading another country because you want what they have buried under the ground...The majority now feel a deep sadness and guilt and a hope that somehow we can make make it all right again. Unfortunately, we can't. So we will accept the consequences of our actions and do our best to be there should the Iraqi people ever dare to seek our help in the future. We ask for their forgiveness."


"mind our own business"

    Moore: We aren't kidding around, Democrats, and if you don't believe us, just go ahead and continue this war another month. We will fight you harder than we did the Republicans


"and opening the door to the full fledged civil war in Iraq they've bemoaned for the past year."

    Moore: When the civil war is over, we will have to help rebuild Iraq.


So many questions on this pathetic tirade where does one start? How about, is it not rather callous and hypocritical to repeatedly throw tantrums over an alleged 髧,000' dead Iraqis and then resign oneself - in a rather cavalier way - to civil war?

And then we had a bloody civil war. That's what happens.

On his Afghanistan remarks, is Moore saying after the bad guys win in Iraq it will then be ok to come in and defeat the bad guys so everyone can live happily ever after?

Can anyone explain this mindset?

tomder55 answered on 11/27/06:

perhaps Moore has forgotten how democracy was brought to modern day Japan and Germany ? I also guess he doesn't see the built in contradiction with his Afghanistan comparison.

Anyone who compares the two wars in terms of times is showing their stupidity.As a point of accuracy ;although there is a fixed date in history when WWII ended in truth it cannot really be said to have ended until the Berlin Wall came down ....as a matter of fact ,our occupation of Germany has not really ceased ;has it? or if he want some more quibbling then how about setting the date of the end of WWII as 1955 ;the official date of West German sovereignty ? (that would make our involvement in the war in Europe lasting 13 years ) .We did not have a formal peace treaty with Japan until 1951. The papers then published VE and VJ day headlines simular to the mission accomplished banners .

Since each war is different and is fought differently his comparison is meaningless. Had we fought WWII the same way we fight this pc war ,we'd still be fighting WWII (or speaking German). If he argued that the politicians have not done their job to rally the nation to defeat jihadistan then I would heartly agree.

A better comparison would be the US involvement with the Phillipines after the Spanish American war. We fought jihadists there also .It was a brutal messy affair that lasted 14 years . We lost 4200 in that span but the net result is that we are not dealing with a jihadist state in the Phillipines today .

Itsdb rated this answer Excellent or Above Average Answer

Question/Answer
Choux... asked on 11/26/06 - Question for Tom and Elliot

Excerpted from the transcript of ABC's This Week with George Stephanopoulos:

STEPHANOPOULOS: Is it a civil war in Iraq right now?

KING ABDULLAH: Well, George, the difficulty that we're tackling with here is, we're juggling with the strong potential of three civil wars in the region, whether it's the (1.)Palestinians, that of (2.)Lebanon or of (3.)Iraq...

... And we could possibly imagine going into 2007 and having three civil wars on our hands. And therefore, it is time that we really take a strong step forward as part of the international community and make sure we avert the Middle East from a tremendous crisis that I fear, and I see could possibly happen in 2007.

~~~~~~~~~~~~~~~~~~~~~~~~~~~~~~~~~~~~~~~~~~~~~

I have a serious question based on this quote.

Was it one of the goals of the Bush Administration and the NeoCons to have the Middle East go up for grabs, as it were? A secondary alternative to the naive quest to install "Democracy" in Iraq and watch it flower and spread to their neighbors?

Choux


PS-I'm not altogether sure my work is done here in view of the situation in the middle east.

tomder55 answered on 11/27/06:

What I am seeing is that finally there is an admission that what we are engaged in is more than just war in Iraq but a wider struggle against the increased influence of the jihadists that has been growing for the last 30-40 years (perhaps even longer if you lookk at the history of the Muslim Brotherhood).

He cites 3 possible trouble spots next year ; Palestine,Lebanon and Iraq. The question I guess is do you look at them as single isolated events or is there something that connects these 3 places ? When I look at the Hamas/ PLA struggle I see the hand of Tehran. When I look at the coming 2nd war in Lebanon between Hezzbollah and Israel I see Tehran . When I look at the emerging civil war in Lebanon ,I see the hand of Syria;working in close alliance with Tehran .

Finally; when I see the current troubles in Iraq ,I can trace them back to a single event ;the bombing of the golden dome Mosque by (I have strong evidence to this speculation )Shiite militias working under the direction of Tehran ;specifically the Mahdi Brigade. The truth is that so long as the Shia restrained from retaliation from the well publicised strategy of al-Qaeda to foment civil war by attacking Shia civilians ,then the coalition forces could concentrate on weeding out the foreign jihadists and bringing the Sunnis into the political process. Tehran saw that it was working, but a stable Iraq democracy is not in their best interests. So their surrogate first bombed the mosque and then instigated the current tit-for-tat gangland style slayings .

But not to worry ;James Baker and his band of realists are on the scene to rescue the ummah from the folly of actually offering them an alternative to living under the jack boot of dictators and hereditary monarchs on one hand and Taliban like theocracy on the other . Their solution to the problem is "engaging with Iran and Syria ".

As Michael Young of the UK Times put it .

In recent weeks the idea that the United States and the UK should engage Syria, but also Iran, to stabilise Iraq has been all the rage. On Tuesday, in an east Beirut suburb, Lebanons industry minister, Pierre Gemayel, showed what the cost of engagement might be. The scion of a prominent Christian political family was assassinated in broad daylight. This was the latest in a series of killings and bomb attacks that the UN investigator looking into the murder of the late Prime Minister, Rafiq Hariri, has determined are linked. ...

If political realism is about interests, then realists must prove that a country that has ignored successive UN resolutions demanding Syrian non-interference in Lebanon could somehow be a force for stability in Iraq, to which it has funnelled hundreds of foreign fighters. Engaging Mr Assad over Iraq will mean the gradual return of Syrian hegemony over Lebanon, since neither the US nor the UK will be in a position to deny Syria in Lebanon while asking favours in Iraq.


I have to agree with them on one point . There will be no solution in Iraq now until we "engage " Iran and Syria. My definition of engagement and theirs is not congruent.We must "convince " Syria and Iran to leave Iraq alone or they face defeat .That does not necessarily mean military defeat although I would not take it off the table....if they can be defeated diplomatically and/or economically that would be fine.If you cut off the head of the snake ,it doesn't take long for the snake to perish .

Unfortunately I no longer believe there is political will in this country to do what is necessary. Pehaps that is George Bush's biggest mistake ;misjudging our will to see this conflict through.

Regarding your question about democracy ;have you looked at the history of Lebanon ? It was a thriving democracy before their first civil war. Beirut was the "Paris of the Middle East " and many considered the country the Switzerland of the Middle East as well because it was the banking capital of the region .The biggest cause of the civil war was the changing demographics of the nation (and you know what I mean by that as jihadists have been using the same playbook since Mohammed) .The war was exasperated and extended due to interference by Syria and the PLO . The country has never recovered and although there was a brief hope that the democracy could be revived after the Hariri assassination and the Cedar Revolution that resulted from the popular outrage over it; ultimately it is doomed to failure because the international community does not support democracy in Lebanaon ;and I dare say anywhere else in the Middle East .

So it is not really a question of "is democracy possible ?" History has shown in Lebanon (and Turkey ,Indonesia and other nations )that democracy is possible in the Islamic world . It is not a question of desire either ,both the Lebanese and Iraqis demonstrated through the risk of going to the ballot box that they want representative government. But their democracies are fledging because their nations are weak are under assault by nations who do not want democracy there ;and have been given very little support by the international community in their effort.

Let's not delude ourselves ,the fabulous Baker boys and Dr. Henry Kissinger could care less how the people in the Middle East live .Kissinger ,arguing in the International Herald Tribune suggests that we create a "regional structure" that will set the Sunni nations in opposition to the Shia to keep Iran in check ;a classical realist thinking dichotomy . It was this rational I will remind you that dictated our policy regarding the Iran /Iraq war that resulted in millions of deaths . In Kissinger's world however this is called stabilty . In classical realist thinking he is imaginging the chess pieces in place ..... Gulf States scared out of their minds over events in Lebanon and Iraq can be manipulated to play into the long festering schism in the Islamic world . What more could you dream of then Muslims killing Muslims ? The Iraqis and Lebananese people ?... Just more grist. They will learn to despise the realists proclamation that nations don't have friends ;only interests .

Of course ,since the Iranian bomb becomes an inevidibilty in the Kissinger scenario then by extention the Sunni nations would also have to be simularily armed so we could go back to a mini-version of the good old days of MAD .The fact that Israel is then the target of both sides is of little consequence since their existence is a fly in the ointment of the grand strategy .

The short answer to your question (as I could write all day on this issue ) is that :

1. Bush's strategy (naive or not )was to offer the democratic alternative

2. The realists on the other hand would like nothing better than to have the ummah face off and split along traditional religious schism .

But that was never a goal stated or otherwise by "the neocons" . They were very clear in all their publications that they wanted the ME safe for democracy .Perhaps it was too Wilsonian in it's idealism but I can argue that throughout most of America's existence thatidealism has been the prime motivator of our foreign policy .

Choux... rated this answer Excellent or Above Average Answer
IQGuru rated this answer Excellent or Above Average Answer

Question/Answer
paraclete asked on 11/24/06 - Even the troops say it is wrong and all smoke and mirrors?



Iraq a moral blunder: war hero

By Patrick Walters

November 25, 2006 12:00am
Article from: The Australian


THE former SAS officer who devised and executed the Iraq war plan for Australia's special forces says that the nation's involvement has been a strategic and moral blunder.

Peter Tinley, who was decorated for his military service in Afghanistan and Iraq, has broken ranks to condemn the Howard Government over its handling of the war and has called for an immediate withdrawal of Australian troops.

"It was a cynical use of the Australian Defence Force by the Government," the ex-SAS operations officer told The Weekend Australian yesterday.

"This war duped the Australian Defence Force and the Australian people in terms of thinking it was in some way legitimate."

As the lead tactical planner for Australia's special forces in the US in late 2002, Mr Tinley was in a unique position to observe intelligence on Iraq's weapons of mass destruction program and the coalition's military preparations in the lead-up to the war.

Mr Tinley, 44, who retired from the army last year after a distinguished 25-year career, said the US-led coalition had been naive in its thinking about what it could achieve after a quick military invasion of Iraq.

"They never had enough troops to fully lock down the major centres and infrastructure or the borders," he said.

In Iraq in 2003, Mr Tinley served as deputy commander for the 550-strong joint special forces task group that took control of western Iraq.

Part of his command was 1 SAS Squadron, which was awarded a US Meritorious Unit citation for its "sustained gallantry", contributing to a comprehensive success for coalition forces in Iraq.

He served 17 years with the elite SAS regiment, leaving the army as a major last year. In 2003 he was appointed a member of the Order of Australia (AM) for "dynamic leadership and consistent professional excellence".

His comments came as Baghdad experienced its deadliest day of sectarian violence since the coalition's March 2003 invasion, with 160 killed and 250 injured by five powerful car bombs in the Shia district of Sadr City.

In recent weeks, British Prime Minister Tony Blair has conceded Iraq has become a "disaster", while the Iraq quagmire contributed to the swing against US President George W.Bush in this month's congressional elections.

Britain has set a tentative timetable this week for withdrawing some of its troops, while the US and coalition forces consider options to end the conflict, which could include a short-term lift in troop numbers.

John Howard said yesterday that despite all Iraq's problems, he still believed he had made the right decision to take Australia to war in 2003.

"Everybody back in 2003, including Kim Beazley and particularly Kevin Rudd and even (French President) Jacques Chirac, were all saying Iraq had weapons of mass destruction," the Prime Minister said.

He said Australia had not agreed to take on any new responsibilities in Iraq and any changes to Australia's 750-strong military presence would depend on a possible withdrawal of British forces.

During war planning with US and British special forces at Fort Campbell, Kentucky, in 2002, Mr Tinley says he never saw any hard intelligence that Saddam Hussein's regime possessed weapons of mass destruction.

"When I pressed them (US intelligence) for more specific imagery or information regarding locations or likely locations of WMD they confessed, off the record, that there had not been any tangible sighting of any WMD or WMD enabling equipment for some years," he said.

"It was all shadows and inferenced conversations between Iraqis. There was an overwhelming desire for all of the planning staff to simply believe that the Iraqis had learned how to conceal their WMD assets away from the US (surveillance) assets."

Coalition special forces troops were charged with hunting down Scud missiles and Saddam's suspected WMD arsenals, operating from just west of Baghdad all the way through to the Jordanian border, and between the Syrian and Saudi frontiers.

After the initial invasion, the search for WMD became something of a "standing joke" with neither coalition troops nor the Iraq Survey Group turning up anything of consequence.

"The notion that pre-emption is a legitimate strategy in the face of such unconvincing intelligence is a betrayal of the Australian way," he said.

Mr Tinley told The Weekend Australian he was now speaking out having expected people "far more capable and more senior than me" to have expressed serious reservations about Australia's involvement in Iraq.

"During our preparations for this war I remember hearing (ex-defence chief) General Peter Gration's misgivings and assumed he did not possess all the information that our Prime Minister did," he said.

"I now reflect on his commentary with a completely different view and am saddened that other prominent people in our society didn't speak louder at the time and aren't continuing to speak out in light of what we now know."

He said the Government had broken a moral contract with its defence force in sending it to an "immoral war".

The Government's stance on Iraq and later on issues such as the Tampa had gradually allowed fear to become a motivating factor in the electorate, he said.

Mr Tinley said the Howard Government had failed to be honest with Australians about Iraq and "you can't separate the sentiment of the defence force from that of the people".

He advocates an immediate pullout of Australia's 500-strong task force in southern Iraq but accepts that security forces must be kept to guard the embassy in Baghdad. "Our 500 troops are in the south-west of Iraq under British tactical command while our US partners are doing all the heavy lifting in the remainder of the country," he said.

A more meaningful contribution could be through providing defence and security force training in a safer neighbouring country, such as Kuwait. "This is no slur on our soldiers. (Brigadier) Mick Moon and his men have been doing a fantastic job."

tomder55 answered on 11/25/06:

What a hypocrite ! What is his belly ache ;that things did not go smoothly . He had access to the intel before the invasion . Where was his objections then ?

He reminds me of Colin Powell. Powell made the most convincing speech before the invasion of the necessity to end Saddam's reign of terror. Then when things got messy ,suddenly we hear that he was in fact a reluctant warrior and was allegedly councelling against the invasion . So who am I to believe ;the Powell who spoke persuasivly on the floor of the UN or the new side of the chamilion ?

The same is true of this Tinley .As the lead tactical planner for Australia's special forces in the US in late 2002, Mr Tinley was in a unique position to observe intelligence on Iraq's weapons of mass destruction program and the coalition's military preparations in the lead-up to the war. So where were his public reservations then ? This is nothing more than a 'cover your a** when things get hot'.

The unimpeachable truth was that everyone in the world thought Saddam was still harboring WMD and retaining the ability to reconstitue his programs once the UN sanction regime collapsed . We have found more than enough proof of both allegations to justify his removal on that fact alone.

paraclete rated this answer Excellent or Above Average Answer

Question/Answer
paraclete asked on 11/23/06 - A case of thanksgiving

as you give thanks you are not fighting terrorism at home, are you giving thanks for this, the consequence of carrying that fight to foreign shores?

Worst Iraq blast since war

Iraqis walk past the site of a car bomb explosion in the Sadr City district of Baghdad.

Iraqis walk past the site of a car bomb explosion in the Sadr City district of Baghdad.
Photo: AP
November 24, 2006 - 7:09AM

In the worst attack on Baghdad since the war to unseat Saddam Hussein, insurgents killed at least 152 people and wounded 236 in a series of car bombings in the Shi'ite district of Sadr City, security and medical sources said.

The attacks prompted the interior ministry to announce an indefinite curfew in the capital, effective from 8pm today (0400 AEDT Friday) .

Wounded clogged the hospitals of Sadr City, with dozens lying bleeding in the corridors as overworked staff struggled to tend to the casualties.

"Of those killed, 88 bodies are in the Imam Ali hospital and 55 in Sadr City hospital," a medic said, saying many of the bodies were burned beyond recognition.

A police official said other bodies were taken to hospitals outside the district.

Hospital security forces kept at bay hundreds of relatives struggling to see the dead and wounded.

The bloodiest bomb exploded in a crowded market in the Hay neighbourhood, targeting stores selling religious CDs, as well as electronics outlets selling mobile phones.

In the increasingly bitter sectarian war gripping the capital, crowded markets in Shi'ite neighbourhoods and villages have been popular targets for the bombers of the Sunni-led insurgency.

In the market, the twisted frame of a car carrying the explosives sat amid the wreckage of the shops, while everywhere the ground was littered with pools of blood and debris from the stores.

As ambulances rushed to attend to the wounded, pillars of black smoke billowed over the stricken neighbourhood and inhabitants collected the bits of flesh left by the dead.

Interior ministry spokesman Brigadier General Abdel Karim Khalaf told state television Al-Iraqiya that police believe a total of eight bombs were planted in Sadr City.

"Four of them exploded, one we have located and also arrested the driver of the car, but three are still to be detected," Khalaf said, without elaborating.

He also said 10 rounds slammed into Sadr City after the bombings and were expected to have caused casualties.

Minutes before the blasts about 100 masked gunmen attacked the Shi'ite-controlled health ministry clashing with guards and Iraqi soldiers, Deputy Health Minister Hakim al-Zamili said.

"First a series of mortars were fired at the building from the nearby Al-Fadhel neighbourhood, and then about 100 masked gunmen holding machine guns attacked the building," said Zamili.

"About 2,000 employees are trapped in the building. I am also in the building," he added. "The gunmen came in civilian cars and pick-up trucks and started shooting at the building and wounded a number of employees."

The attack was eventually dispersed, and only five people were wounded, he added.

On Sunday, gunmen kidnapped Deputy Health minister Ammar al-Assafar from his home in Baghdad's Adhamiyah district. Zamili himself escaped an assassination bid on Monday, but two of his guards were killed in the ambush.

The attack on the ministry mirrors a November 14 raid on the Sunni-controlled ministry of higher education, in which at least 150 people were kidnapped.

In the aftermath of the attacks in Sadr City and on the ministry, some 13 mortar rounds rained down on Adhamiyah, a Sunni neighbourhood, wounding 10 people.

Explosions continued to reverberate through Baghdad well into the night.

Sadr City, the impoverished district of followers loyal to radical Shi'ite cleric Moqtada al-Sadr, was also the site of an early morning incident involving American soldiers.

"US troops fired on a minibus carrying workers and killed a number of them in Al-Falah street at 6am," Imam Abdel Zahra al-Suwaidi from the Sadr movement said.

A medic at the Sadr City hospital said four people were killed and eight wounded, including two women.

But the US military said Iraqi forces fired on the vehicle during a raid to detain the leader of a kidnapping cell.

"A vehicle displaying hostile intent was identified as an immediate threat to Iraqi forces. Iraqi forces fired on the vehicle to neutralise the threat," the military said without mentioning civilian casualties.

US forces have regularly raided Sadr City to hunt for leaders of kidnapping cells alleged to be militiamen loyal to Sadr.

On Wednesday, a UN report said Iraq's sectarian conflict killed at least 3,709 people in October, the highest monthly death toll since the 2003 US-led invasion.

The figures, from data provided by the health ministry and morgues, compared with a previous high of 3,590 in July, which the United Nations at the time called "unprecedented."

The report came as US President George W. Bush and Iraqi Prime Minister Nuri al-Maliki prepared to meet in Jordan next Wednesday.

Meanwhile, the US military said three more soldiers were killed in Iraq, bringing its losses since the invasion to 2,866, according to Pentagon figures.

Police also recovered eight bodies near the central city of Diwaniyah, while 12 people were reported killed in the city of Baquba.

US Secretary of the Army Francis Harvey arrived in Baghdad today to visit troops on the Thanksgiving holiday, a military spokeswoman said.

AFP

tomder55 answered on 11/25/06:

your hatred of America is irrational . When you look in the mirror you see us. We have just been more successful.

Perhaps you bemoaned the fate of the Iraqis as much when Saddam was waging a one sided civil war agaisnt them and slaughtering them by the thousands. Or when Saddam invaded neighboring countries that resulted in the deaths of millions.

The Iraqis have for the first time a chance at self rule. It is there for them if they want it. But you are the type who says that Marshal Tito with his brutal jackbooted tactics was good for the Balkans ...and no doubt that since Russian democracy was so messy that it is better that Putin is now restoring the former KGB tactics of control .

Do you plan on posting every incident of violence out of Iraq ? My response will be the same.

ETWolverine rated this answer Excellent or Above Average Answer
paraclete rated this answer Bad/Wrong Answer

Question/Answer
ETWolverine asked on 11/21/06 - Neologisms and other humor

The new business vocabulary:

404: Someone who is clueless. From the World Wide Web error message ̚ Not Found" meaning that the requested document could not be located. Example: "Don't bother asking him, he's 404, man."

Adminisphere: The rarefied organizational layers beginning just above the rank and file. Decdisions that fall from the adminisphere are often profoundly inappropriate or irrellevant to the problems they were designed to solve.

assmosis: The process by which some people seem to absorb success and advancement by kissing up to the boss. You will all be measured on this at some point in your career.

blamestorming: Sitting around in a group discussing why a deadline was missed or a project failed and who was responsible. This one will be particularly valuable to those of you who have projects going right now.

chainsaw consultant: An outside expert bought in to reduce the employee head count, leaving the brass with clean hands.

clm: Short lingo for "career limiting move". Used among miroserfs to describe ill-advised activity. Trashing your boss while she is within earshot is a serious clm. (Related to clb, "career limiting behavior")

dilberted: to be exploited and oppressed by your boss. Derived from the experiences of Dilbert, the geek-in-hell comic strip character. Example: "I've been dilberted again. The old man revised the specs for the fourth time this week."

flight risk: Used to describe employees who are suspected of planning to leave the company or department soon.

ohnosecond: The infinitesimal amount of time it takes to realize that you've just made a big mistake. (See also clm.)

percussive maintenance: The fine art of whacking the crap out of an electronic device to get it working again. (Sometime use on people as well.)

salmon day: The experience of spending an entire day swimming upstream only to die in the end. We've had these before--- and will again.

seagull manager: A manager who flies in, makes a lot of noise, poops all over everything then leaves. Another word for consultant.


The Lost Balloonist
A man is flying in a hot air balloon and realizes he is lost. He reduces height and spots a man down below. He lowers the balloon further and shouts, "Excuse me, can you help me? I promised my friend I would meet him half an hour ago, but I don't know where I am."

The man below says,"Yes. You are in a hot air balloon, hovering approximately 30 feet above this field. You are between 40 and 41 degrees N. latitude, and between 58 and 59 degrees W. longitude."

"You must be an engineer" says the balloonist.

"I am," replies the man. "How did you know?"

"Well," says the balloonist, "everything you have told me is technically correct, but I have no idea what to make of your information, and the fact is I am still lost."

The man below says "You must be a manager."

"I am," replies the balloonist, "but how did you know?"

"Well," says the man, "you don't know where you are, or where you are going to. You have made a promise which you have no idea how to keep, and you expect me to solve your problem. The fact is you are in the exact same position you were in before we met, but now it is somehow my fault."




And here are some funny signs:

On an electricians truck: Let us remove your shorts.

On a Maternity Room door: Push. Push. Push.

At an Optometrists Office: If you don't see what you're looking for, you've come to the right place.

At a road-side rest stop: Eat here, get gas.

Enjoy your day.

Elliot

tomder55 answered on 11/21/06:

we have' prairie dogs '.When something is happening in the office they stick their heads above the cubicles to see what's going on .And of course we have our share of 'ignoranus'(A person who's both stupid and an a**hole)

ETWolverine rated this answer Excellent or Above Average Answer

Question/Answer
paraclete asked on 11/21/06 - I find this imcomprehensible, what about you?

Low food security is the new hunger
November 21, 2006 - 10:56AM

The US government has tweaked its terminology in referring to the nearly 11 million Americans who face a constant struggle with hunger to refer to them as people with "very low food security".

According to a report released this month by the US Department of Agriculture, roughly 35 million Americans had difficulty feeding themselves in 2005 and of those some 10.8 million went hungry.

But unlike last year's report on hunger in America, which labelled families who don't get enough to eat as having "food insecurity with hunger", this year's report referred to them as having "very low food security".

The change in terminology has angered groups that fight hunger who say it is aimed at hiding a stark reality.

"There is very widespread feeling that it was a mistake to water down the language," Jim Weill, director of the Washington-based Food Research and Action Centre, a non-profit organisation, said.

"There are 35 million people in this country who are struggling with hunger, no matter what you call it," he added. "And there is no way ultimately to obscure the fact that we're an incredibly wealthy country with 35 million people who are struggling with hunger."

US officials have defended the change, saying it is based on a recommendation from the National Academies, which advises the government on science and medical issues.

AFP

tomder55 answered on 11/21/06:

I wonder how many of them are simply on the Cindy Sheehad diet ?

Orwellian language was around long before Orwell identified it . The 10.8 million represents roughly 3.6% of the population of the country . Given the continuos influx of immigrants (legal and illegal)who regularily enter the country in impoverished state it is actually a credit that the figure is so low.

Anyway ;the term 'food security' is not something that the USDA invented . It happens to be the term in favor by world organizations. The UN's Food and Agriculture Organization (FAO) has been using the terminology for a long time(at least as early as the World Food Summit held in Rome in 1996 ). 'The Rome Declaration on World Food Security' was one of the documents that came from the meeting .

Based on the current accepted definitions ,the USDA published it's findings. But these NGO types will ridicule it not because of the data but because the new terminology ,that is already accepted world wide ,was adopted by Bush instead of Clinton.

The problem with the NGOs criticism and the press coverage is that it does not help a single hungry person get fed. It only serves to undermine real efforts to eradicate hunger .

ETWolverine rated this answer Excellent or Above Average Answer
paraclete rated this answer Excellent or Above Average Answer

Question/Answer
paraclete asked on 11/19/06 - The Idea would be wildly unpopular with who?

Only the US and Australia

US pours scorn on international greenhouse tax proposal


Peter Hartcher Political Editor in Hanoi
November 20, 2006

THE US Secretary of State, Condoleezza Rice, has described as unacceptable a French proposal to tax the imports of countries that refuse to sign the Kyoto Protocol.

In the sharpest divide yet between the two main global approaches to dealing with climate change, Dr Rice said the idea would be "wildly unpopular" and predicted it would never be implemented.

The Prime Minister, John Howard, has claimed strong and unanimous support for his greenhouse policies from the summit of 21 Asia-Pacific Economic Co-operation forum leaders in Hanoi.

Mr Howard's position, which is identical to that of the US, stresses what he called yesterday "a balanced approach" that addresses climate change and energy security, not sacrificing economic growth in the interests of environmental responsibility.

He said he had the vocal support of the US President, George Bush, the Canadian Prime Minister, Stephen Harper, and the Singaporean Prime Minister, Lee Hsien Loong, at the summit yesterday, and that no dissenting voices were raised.

Dr Rice's position is the first high-level US response to the French proposal last week. It marks a new level of tension between the European proscriptive approach to global warming and the US and Australian emphasis on voluntary, technology-based solutions.

Portraying the idea as anti-growth, Dr Rice told the Herald: "I don't think that would be a particularly useful or acceptable proposal in a world economy that is highly dependent on economic growth in the US and, increasingly, on economic growth in China."

The French Prime Minister, Dominique de Villepin, proposed the tax as a Europe-wide measure to penalise what he called "environmental dumping".

It would be imposed on countries that did not agree to the carbon emissions limits decided for the Kyoto Protocol's next phase, from 2012.

"We have decided to reinforce the principle that the polluter pays," Mr de Villepin said. His proposed tax would hit exports from Australia if Canberra stayed outside the Kyoto Protocol.

Mr de Villepin's proposal, and Dr Rice's response, shows the potential for trade wars in the differing policy responses to global warming.

Mr Howard said he had spoken to the other Asia-Pacific leaders on the central importance of technological solutions, the desirability of nuclear power and the importance of the AP6 group, a kind of counter-Kyoto group that embraces Australia, China, India, Japan, South Korea and the US and rejects proscriptive approaches.

"I reminded them to think about what their economies were like 30 years ago" and how they had been revolutionised by information technology.

"Think what technology can do to transform the environmental consequences of the use of fossil fuels."

When

tomder55 answered on 11/20/06:

A tariff by any other name is still a tariff. I have not seen where restricting trade benefits anyone .Dominique de Villepin is a first class hypocrite . An examination of his motives demonstrates that his motives is protectionism of French industry and not environmental concerns.

He and Chirac sees the Kyoto Protocol as the first component of authentic global governance. Angry that we dare refuse to surrender our national sovereignty France is now pressing for punitive carbon taxes on imports into any of EU member nation from countries that refused to sign it. Any product imported into an EU country under this scheme could be assessed a special import tax on the energy consumed in the course of producing the imported product before they would be allowed to be sold to that countrys consumers. Meanwhile any products exported from that same country to the United States or Australia (or China and India ? Does he really relish a trade war with the indusrial giants ? ) would receive an indirect subsidy from the home country in the form of an energy cost rebate, on the theory that the exporter had incurred higher costs in complying with the Kyoto Protocol. The net effect would be to punish the United States and Australia with trade reprisals for exercising sovereign rights not to join a treaty with which it disagrees.

The United Nations is even worse .It has been advocating such a global carbon tax for years (paid to the UN of course ).

I would remind him that when the EU instituted the their carbon trading schemes in 2004 ;the Emissions Trading System (ETS), France (the country so comitted to carbon emission reductions )ended up with a 19 million ton surplus of carbon permits, without a noticeable reduction in emissions.

Almost nothing is made without the use energy . What this dufus is proposing is a tariff /tax on everything. But if he worded it that way he would be laughed off the planet.And who would be most affected by these taxes on everything ? .....the folks with the least ability to pay them . Good plan Dominique !!!!

paraclete rated this answer Excellent or Above Average Answer

Question/Answer
paraclete asked on 11/19/06 - Is environmentalism an anti-social religion?

Does environmentalism trap the thirn world in poverty?

The real question is what do you put first people or the environment.

Make poverty history: first by getting rid of the greens

Miranda Devine
November 19, 2006

AT U2's Sydney concerts last week, Bono urged the audience to text their names to a Make Poverty History phone number. Later he flashed the names on a big screen and sent a thank you text to all those mobile phones in Telstra Stadium. As an act of charity it doesn't come much easier, unless you count wearing wristbands.

This is not to sneer at Bono for raising consciousness of the world's poor, or his audience for making a gesture.

But as protesters and green activists gather in Melbourne this weekend to lay the usual blame for poverty on the greed of developed nations, a powerful new documentary shines light on a different villain.

Mine Your Own Business, which opens this week, shows that the "powerful group telling the world's poor how to live, how to work, even how to think" are not the world leaders gathered in Melbourne. They're not even wealthy multinational corporations, but wealthy multinational environment groups such as Greenpeace.

"Upper-class Western environmentalists" are the greatest enemy of the world's poor, says the documentary's maker, self-described left-wing journalist Phelim McAleer, from Northern Ireland.

He shows how environmental groups opposed to change and economic growth are trying to keep the developing world poor. "Poor but happy", is how they see it.

Posted to Romania by The Financial Times in 2000, McAleer covered the Greenpeace campaign to prevent the opening of a goldmine in the Transylvanian mountains. It changed his views on environmental activism.

What he found in Rosia Montana was an impoverished village, with 75 per cent unemployment, little sanitation or running water and people desperate for jobs. It had been a mining town since Roman times but the last state-owned mine was closing and a Canadian company, Gabriel Resources, wanted to take over. It had promised to provide jobs, rebuild infrastructure and clean up pollution from old mines.

Early on McAleer acknowledges his film was part-funded by Gabriel Resources but says he retained editorial control.

He interviews Francoise Heidebroek, a Belgian green activist who says villagers are better off being farmers and riding horses. But as the villagers explain, nothing grows except potatoes, and at minus 25 degrees they prefer cars and indoor toilets.

Gheorghe Lucian, an unemployed miner, tells McAleer: "People have no food to eat. They don't have money for clothes I know what I need - a job."

McAleer took Lucian to similar projects around the world, and interviewed activists such as Mark Fenn, World Wide Fund for Nature's American representative in southern Madagascar, who opposes a Rio Tinto mine in the impoverished fishing village of Fort Daupin, which would create 2000 jobs. "The quaintness, the small-town feeling will change," Fenn says.

Fenn insists that Lucian doesn't really understand poverty. "How do we perceive who's rich, who's poor " Fenn says. "I could put you with a family and you count how many times in a day that family smile Then I put you with a family well off, in New York or London, and you count how many times people smile and measure stress Then you tell me who is rich and who is poor."

Underlining the hypocrisy, Fenn shows McAleer the luxury house he is building and catamaran he bought for $US30,000 ($39,000) - "a good price". As McAleer says, the average salary in the village is less than $US100 a month.

But, "the indicators of wellbeing aren't housing, nutrition, health, education", says Fenn, although he sends his own children to school in South Africa.

The villagers tell McAleer the opposite. One says she wants her children to become "a midwife, a doctor, or an engineer".

It's the same story in Chile where activists have halted a goldmine in the Andes. A young man tells McAleer: "I'm not asking for much, just a normal job."

McAleer shows how progressives oppose progress and have become part of an "authoritarian world order", telling people in the developing world how they must live. He hopes his film will show well-meaning Westerners the consequences of their blind faith in the new "religion" of environmentalism.

tomder55 answered on 11/20/06:

I'm not sure that Bono fits into the catagory described here . He seems to have his head screwed on right . He is commited to his goals and unlike alot of his contemporaries understands and appreceiates the under reported work that people like John Howard and President Bush contribute towards the goals of poverty elimination and eradiaction of diseases like AIDS.

But to the larger issue unfortunately I have to concure. Often well intentioned environmental gestures have unintended consequences.

The greatest example of this I guess would be the banning of DDT. Oh sure it probably wasn't the safest chemical to be spraying but the explosion in mosquito born ailments since the ban is telling . In retrospect the banning of DDT appears to be an over reaction that has caused more harm than good.

ETWolverine rated this answer Excellent or Above Average Answer
paraclete rated this answer Excellent or Above Average Answer

Question/Answer
paraclete asked on 11/20/06 - WARNING!..WARNING! MAD Scientist loose?

NASA looks at plan to blot out Sun
November 20, 2006

THE idea seems like something out of a Superman comic: a machine or missile shoots tonnes of particles into the atmosphere that would block the Sun's rays, cool down the overheated Earth, and reverse global warming.

But at the weekend scientists gathered in a closed session organised by NASA and Stanford University to discuss researching such a strategy. The idea is called geo-engineering: using technology to tinker with the Earth's delicate climate balance.

Ken Caldeira, a climate scientist at the Carnegie Institution's Department of Global Ecology at Stanford University, said his modelling showed the idea worked. "We found that if you blocked 20 per cent of the sunlight over the Arctic Ocean it would be enough to restore sea ice," he said.

The Boston Globe

tomder55 answered on 11/20/06:

Then again , they cloud seed with silver iodide in an attempt to enhance percipitation. And the tin foil hat crowd say that Russia China and the US have developed High-frequency Active Aural Research Programs (HAARP)to manipulate climates. They even claim that Katrina was a result of such manipulation .

Scientists are realizing that the earths weather is controlled by Sun's natural Electromagnetic Radiation reaching the earth. The Suns Radiations and Ultraviolet Rays have to cross the ionosphere to reach the earth.

There are indications that the solar radiations and flares are directly responsible for planetary weather changes.
The ionosphere acts as a filter for the solar radiations to reach the earth. If one can manipulate and control the filter, it becomes a potential source of massive weather modification. That is what the computer simulation models have found. Controlling the ionosphere potentially allows weather control.

I know that there is concerns about the unintended consequences of Artifical Weather Manipulation (AWM) or that it could easily be converted into just another weapon but there are plenty of upsides to this research .

Forgetting Earth;if there is any long term goal of humans colonizing other planets then they will most likely need some kind of weather manipulating skills ,and if this "global warming " business is almost past the point of no return like the Gores of the world claim;then AWM may be the answer for survival don't you think ?

There are many private research initiatives already (80 that we knew of in 2005).The research will be done if NASA does it or someone else does. There are many useful applications of research that was done in the last century that at the time seemed science fiction and the efforts of a 'mad scientist'. I guess that is the price they pay for being visionary.

paraclete rated this answer Excellent or Above Average Answer

Question/Answer
paraclete asked on 11/17/06 - A question on Climate Change?

Does this happen very fifteen hundred years? Every Century? Every Millenium?

Siberian heatwave brings chilling warning


Adrian Blomfield in Moscow
November 18, 2006

SIBERIA is basking in its warmest November for 70 years, putting its permafrost, wildlife and even the human population at risk.

Russian scientists warned on Thursday that southern Siberia, already known as one of the fastest warming regions on the planet, is facing grave consequences as a result of the unnaturally temperate start to its typically harsh winter.

November is normally a month when silence swathes the vast evergreen forests as migratory birds depart for warmer climes and resident mammals settle down to hibernate.

This year, though, the forests are alive with uneasy sound. Bears and badgers have yet to hibernate, while hares, whose coats have changed from grey to white in anticipation of snow, have become easy prey.

Even the plants seem confused. For the first time in memory, dandelions and raspberries have bloomed in several parts.

Some areas are recording highs of 12 degrees, with temperatures across southern Siberia seven to 10 degrees warmer than normal.

A 984,000-square-kilometre expanse of permafrost has started to melt, releasing into the atmosphere large quantities of methane and carbon dioxide, the main greenhouse gases that cause global warming.

Insomniac bears are roaming the forests of south-western Siberia, scaring local people, as the weather stays too warm for the animals to fall into their usual winter slumber.

The furry mammals escape harsh winters by going to sleep in October-November for about six months, but in the snowless Kemerovo region where the weather is unseasonably warm, bears have no desire yet to hibernate.

If snow does not fall for another month, there is a risk that some bears and badgers could starve to death. "The longer these animals stay awake the less fat they accumulated over the summer to prepare for hibernation will be left," said Anatoly Lobanchuk, head of the veterinary department in the southern Siberian region of Kemerovo.

The situation is of even greater concern in northern Siberia. Ice packs are failing to form, forcing polar bears, seals and walruses to remain on land, disrupting breeding cycles.

Telegraph, London; Reuters

tomder55 answered on 11/18/06:

Convetional wisdom I guess rushes to the findings in the Stern report released this week ,while studies I cited are quickly buried and marginalized. I do not know if the Siberian pattern is cyclical or not .I will leave that to the scientists to debate.

I do know there is plenty of evidence that areas in the arctic were once lush and tropical. Erik the Red's Viking colonists fattened their cattle on the lush pastures of Greenland ,so we know that significant climate changes have occured even as late as recorded human history. Was Erik et al driving SUVs ?

Northern Europe enjoyed an almost Mediterranean climate right up to the early 14th Century prior to the Little Ice Age . On the other hand ;the Thames would freeze in the winter for about 3 centuries (17th, 18th and early 19th Centuries )to such an extent that fairs and markets were routinely conducted on the ice. Here in NY it was possible to walk across NY Harbor and from Manhattan to Brooklyn. Was it human activity that changed that ?

Weather patterns from the last century and the beginning of the present one indicate that the climate is indeed shifting again, but whether this is the fault of our gas-guzzling, carbon-belching lifestyle or part of the natural rhythm of our planet is up for debate.

Unfortunatley it appears that one side of the debate is destined to be ridiculed and dismissed as heracy. It's science doing it's best to immitate the worse instincts of religion if you ask me.

I support the reduction of fuel emissions .I think we should be funding research into sustainable wind, water and solar power production.There is a logical reason to do so without trying to bombard me with junk science and scare mongering .

ETWolverine rated this answer Excellent or Above Average Answer
paraclete rated this answer Excellent or Above Average Answer

Question/Answer
Mathatmacoat asked on 11/16/06 - Towards unbiased reporting on climate change

since a recent post indicated a highly biased position, those with interest in this subject might like to visit

www.worldviewofglobalwarming.org

where evidence is portrayed giving you the opportunity to make up your own mind

tomder55 answered on 11/17/06:

nice photos

Many scientists encourage the mapping of established interests from across the political spectrum onto science and then use science as a proxy for political battle over these interests. That my friend skews the results of the reasearch bady and is very much in conflict with the basic established 'scientific method' of research . As you know ,that is the argument that many on the Christianity Board uses to refute Darwinism.

I see Al Gore (the Cindy Sheehan of Global Warming) jetted into Aussie the other day(can't quite figure out why he didn't use an outrigger canoe and paddle to avoid spewing pollutants into the skies of Australia ) and called our nations the "Bonnie and Clyde's of global warming" for daring not to sign onto a treaty that none of the signatories complies with. Canada signed onto Kyoto. So far they have spent billions and haven't even come close to meeting the standards and now admit they can't meet them. Meanwhile the US has done a better job of cleaning up than Canada has and is not a signatory .

He said it was significant that Howard had in recent days acknowledged a "damaging increase" in carbon dioxide emissions and thrown his weight behind a global carbon trading scheme (couldn't have said it better myself)to reduce greenhouse gases.

He also said this absurdity :

"I sincerely believe if Australia joined the rest of the world community in the Kyoto process, then the pressure on Bush would be enormous, just enormous,"

Reality check fat Albert . The US Senate rejected Kyoto 99-0 BEFORE there was a President Bush .

ETWolverine rated this answer Excellent or Above Average Answer
Itsdb rated this answer Excellent or Above Average Answer
Mathatmacoat rated this answer Excellent or Above Average Answer
paraclete rated this answer Excellent or Above Average Answer

Question/Answer
labman asked on 11/16/06 - Does anybody else find this ludicrous?

Do you think Israel would quit threatening them if they declared Israel had the right to exist and had no intention of attacking Isreal or helping others to?

UNITED NATIONS (Reuters) - Iran, whose president has vowed to wipe Israel off the map, complained to the United Nations on Wednesday that the Jewish state was repeatedly threatening to bomb it.

The threats were "matters of extreme gravity" and the U.N. Security Council should condemn them and demand that Israel "cease and desist immediately from the threat of the use of force against members of the United Nations," Iranian U.N. Ambassador Javad Zarif said.



His comments, which came in a letter to U.N. Secretary-General Kofi Annan dated November 10 and circulated at the United Nations on Wednesday, prompted a quick rejoinder from John Bolton, the U.S. ambassador to the United Nations.



"I would say this is perhaps an example of the Iranians trying to learn 'chutzpah,"' Bolton told reporters, using a Yiddish word for unmitigated gall or outrageously arrogant behavior.

Fool story at http://news.yahoo.com/s/nm/20061115/ts_nm/iran_israel_un_dc_2

tomder55 answered on 11/16/06:

"It's 1938 and Iran is Germany. And Iran is racing to arm itself with atomic bombs," ...."Believe him and stop him," .... "This is what we must do. Everything else pales before this."
(Benjamin Netanyahu in a speach to the annual United Jewish Communities General Assembly )


And we just elected a Chamberlain congress

ETWolverine rated this answer Excellent or Above Average Answer
Itsdb rated this answer Excellent or Above Average Answer
labman rated this answer Excellent or Above Average Answer

Question/Answer
paraclete asked on 11/14/06 - Could this be the death of democracy in Australia?

In a landmark High Court decision, the High Court of Australia threw out a century of progress and signed the death warrant of the Australian States which form the Commonwealth of Australia

The States are dead

By Tim Dunlop
Tuesday, November 14, 2006 at 01:01pm


No matter what you think of the IR laws that were the basis of the High Court case decided today, the ruling is a blow to anyone who believes in the Australian Federation and the liberal principle that government power should be as dispersed as possible. Todays decision, granted by a 5-2 majority of the High Court, is yet another way in which political power in Australia is being centralised in the Federal Government. So while I have no opinion on the specifics of the legal argumenttheres no arguing with a 5-2 majority on that scorethe net effect is not something Im particularly thrilled about.

And when I say that power is being centralised in Federal Government, I mean the Federal Government, not just the Howard Government. As much as his boosters are loathe to admit it, one day there will be no Howard Government, there will be no Coalition Government, there will be a Labor Government in Canberra, and Mr Howards actions, which have precipitated this court case in the first place, have just handed that future Labor Government an enormous amount of power.

The issue, then, ultimately goes way beyond party politics.

To illustrate that, it is worth noting that two the dissenting Justices, Kirby and Callinan, are what you might call ideological opposites, but this issue has united them. Justice Kirby makes the point:

615 ...this is such an important case for the content of constitutional power in Australia. The majority concludes that not a single one of the myriad constitutional arguments of the States succeeds. Truly, this reveals the apogee of federal constitutional power and a profound weakness in the legal checks and balances which the founders sought to provide to the Australian Commonwealth.

Justice Callinan noted that, The act in its present form is well beyond, and in contradiction of, what was intended and expressed in the constitution by the founders.

I well remember all those Coalition politicians at the time of the Referendum on Australia becoming a Republic endlessly spouting the mantra that, The States created the Commonwealth; the Commonwealth didnt create the States, a mantra designed to stave off what they thought of as the centralising, anti-State tendencies of becoming a Republic. It is more than a little ironic that the Prime Minister who killed that referendum dead, partly on the back of the states rights argument, has just been the birthing partner to a decision that arguably does more harm to states rights than a Republic ever would have. Having helped God save the Queen, he has just helped hand a future Labor Government enormous power.

Beyond that, well, the ironies continue. Given that Labor is traditionally the party that wants to take power away from the States, as well being the ones keen to fight the next election in large part on Mr Howards draconian new industrial laws, I cant imagine that they are too upset about todays decision.

tomder55 answered on 11/14/06:

I do not have an opinion on the Federal system in Australia . I think what the people laid out in the constitution is what the system should be . I do have a strong negative opinion regarding unelected oligarchs deciding how the Australian system should be designed. Such fundamental changes in structure should be decided by the people ;not life time judges . In the US we have the amendment process which ultimately makes the people the final arbiter . Although it is rarely used ,it is our check on an out of control judiciary (since our legislators and executives long ago surrendered their obligation to exercise their check on the courts)

paraclete rated this answer Excellent or Above Average Answer

Question/Answer
Itsdb asked on 11/14/06 - A Liberal's Pledge to Disheartened Conservatives

November 14th, 2006

To My Conservative Brothers and Sisters,

I know you are dismayed and disheartened at the results of last week's election. You're worried that the country is heading toward a very bad place you don't want it to go. Your 12-year Republican Revolution has ended with so much yet to do, so many promises left unfulfilled. You are in a funk, and I understand.

Well, cheer up, my friends! Do not despair. I have good news for you. I, and the millions of others who are now in charge with our Democratic Congress, have a pledge we would like to make to you, a list of promises that we offer you because we value you as our fellow Americans. You deserve to know what we plan to do with our newfound power -- and, to be specific, what we will do to you and for you.

Thus, here is our Liberal's Pledge to Disheartened Conservatives:

Dear Conservatives and Republicans,

I, and my fellow signatories, hereby make these promises to you:

    1. We will always respect you for your conservative beliefs. We will never, ever, call you "unpatriotic" simply because you disagree with us. In fact, we encourage you to dissent and disagree with us.

    2. We will let you marry whomever you want, even when some of us consider your behavior to be "different" or "immoral." Who you marry is none of our business. Love and be in love -- it's a wonderful gift.

    3. We will not spend your grandchildren's money on our personal whims or to enrich our friends. It's your checkbook, too, and we will balance it for you.

    4. When we soon bring our sons and daughters home from Iraq, we will bring your sons and daughters home, too. They deserve to live. We promise never to send your kids off to war based on either a mistake or a lie.

    5. When we make America the last Western democracy to have universal health coverage, and all Americans are able to get help when they fall ill, we promise that you, too, will be able to see a doctor, regardless of your ability to pay. And when stem cell research delivers treatments and cures for diseases that affect you and your loved ones, we'll make sure those advances are available to you and your family, too.

    6. Even though you have opposed environmental regulation, when we clean up our air and water, we, the Democratic majority, will let you, too, breathe the cleaner air and drink the purer water.

    7. Should a mass murderer ever kill 3,000 people on our soil, we will devote every single resource to tracking him down and bringing him to justice. Immediately. We will protect you.

    8. We will never stick our nose in your bedroom or your womb. What you do there as consenting adults is your business. We will continue to count your age from the moment you were born, not the moment you were conceived.

    9. We will not take away your hunting guns. If you need an automatic weapon or a handgun to kill a bird or a deer, then you really aren't much of a hunter and you should, perhaps, pick up another sport. We will make our streets and schools as free as we can from these weapons and we will protect your children just as we would protect ours.

    10. When we raise the minimum wage, we will pay you -- and your employees -- that new wage, too. When women are finally paid what men make, we will pay conservative women that wage, too.

    11. We will respect your religious beliefs, even when you don't put those beliefs into practice. In fact, we will actively seek to promote your most radical religious beliefs ("Blessed are the poor," "Blessed are the peacemakers," "Love your enemies," "It is easier for a camel to go through the eye of a needle than for a rich man to enter the kingdom of God," and "Whatever you did for one of the least of these brothers of mine, you did for me."). We will let people in other countries know that God doesn't just bless America, he blesses everyone. We will discourage religious intolerance and fanaticism -- starting with the fanaticism here at home, thus setting a good example for the rest of the world.

    12. We will not tolerate politicians who are corrupt and who are bought and paid for by the rich. We will go after any elected leader who puts him or herself ahead of the people. And we promise you we will go after the corrupt politicians on our side FIRST. If we fail to do this, we need you to call us on it. Simply because we are in power does not give us the right to turn our heads the other way when our party goes astray. Please perform this important duty as the loyal opposition.


I promise all of the above to you because this is your country, too. You are every bit as American as we are. We are all in this together. We sink or swim as one. Thank you for your years of service to this country and for giving us the opportunity to see if we can make things a bit better for our 300 million fellow Americans -- and for the rest of the world.

Signed,

Michael Moore
mmflint@aol.com
(Click here to sign the pledge)

~~~~~~~~~~~~~~~~~~~~~~~~~~~~~~~~~~~~~~~~~~~~~~~~~~~~~~~

Isn't that comforting (or mildly translated, what a jerk)?

Steve

tomder55 answered on 11/14/06:

Michael Moore and his ilk in the blogsphere were marginalized with the Lamont loss. The Demoncrats did not win because of his influence . They ran by fielding blue dog Democrats who in some cases could teach the Republicans some lessons in conservatism.These rookies will be so disallusioned in 2 years that they will change parties.

I can guarantee that Moore will no longer have a prominent seat at the Demoncrat conventions .The Demons.said to the "progressives" on the internet like Moore and Kos "thanks....don't let the door hit you on the way out" .

ETWolverine rated this answer Excellent or Above Average Answer
Itsdb rated this answer Excellent or Above Average Answer
labman rated this answer Excellent or Above Average Answer

Question/Answer
captainoutrageous asked on 11/14/06 - Govern, Don't Gloat

I found the following of interest and thought I would share it:

Govern, Dont Gloat

By LEON E. PANETTA
Published: November 12, 2006
Seaside, Calif.

Paul Weston
WE govern our democracy either by leadership or by crisis. Last Tuesday, the American people sent a clear message that they are sick and tired of government by crisis. They elected Democrats to the House and Senate not to prolong gridlock, but to govern.

There are those who believe that the best political strategy for 2008 is for the Democrats to continue to confront President Bush and seal his fate as a failed president. The danger, however, is that if the Democrats become nothing more than a party of obstruction, it will be only a matter of time before they too will lose the trust of the American people. The lesson of this election is that the public will no longer tolerate incompetence and gridlock, whether it comes from the Republicans or the Democrats.

Twelve years ago, President Clinton suffered a similar defeat when Republicans captured both houses of Congress. As chief of staff to the president at the time, I was asked to comment on the implications of that midterm election for the president and the future of the nation. My response was that the real question was whether a party that had been a minority in Congress was now prepared to work with the president to govern the nation.

Today it is fair to ask the same question of the Democrats.

In 1994, the Republicans decided they would directly challenge the president with their Contract With America. The result was the shutdown of the federal government. Badly damaged by the public outrage over such antics, Speaker of the House Newt Gingrich decided that Republicans had to work with the president if they were to survive. This led to a period of cooperation that produced, among other achievements, a balanced federal budget and welfare reform. The Republicans held their majority in 1996.

In the wake of this election, the Democrats and the president face the same choice: gridlock or cooperation?

While both sides are speaking the words of reconciliation, nothing will really change until they can trust each other. After six years of partisan trench warfare, that will not be easy. It begins with a cease-fire on the rhetoric of cheap shots and ultimatums. Karl Rove and other political consultants need to take a long vacation. Both sides need to speak honestly with each other and be willing to compromise.

The legislative work can begin on areas where there is likely consensus: immigration reform, lobbying and ethics reform, and education with the reauthorization of No Child Left Behind.

If that works, Congress and the administration can move on to negotiate tougher issues like establishing long-term budget discipline, expanding energy alternatives, fixing the prescription drug benefit and increasing the minimum wage.

And, finally, on the war in Iraq, despite the bitter differences, both the Democrats and the president face the same brutal reality. We need a new strategy to stabilize Iraq so that our troops can begin to come home without leaving a disaster behind. The president took an important step by replacing Secretary of Defense Donald Rumsfeld with Robert Gates. The Iraq Study Group led by James Baker and Lee Hamilton, of which I am a member, will soon make its recommendations, which we hope will provide the beginning of a unified strategy.

Democracy needs the trust of the people. But that can only occur if our elected leaders can trust one another. That is not only good politics, it just happens to be good for the country.

Leon E. Panetta is a former Democratic representative, director of the office of management and budget, and White House chief of staff.

tomder55 answered on 11/14/06:

Just yesterday it was reported that incoming House Speaker Nancy Pelosi is supporting Rep. John Murtha for majority leader. This is a sign that Pelosi, despite all her talk of moving to the center and reaching out to conservatives, will govern from the left. It is a direct assault on the moderate wing of the Democrat Party and a deliberate break with the second-ranking Democrat in the House, Rep. Steny Hoyer .

The next test for whether Pelosi will govern from the left or the center will be if she appoints Alcee Hastings ,the impeached former federal judge, to chair the Intelligence Committee. No national security supporter will be comfortable with Hastings' having oversight of the nation's secrets, but the pressure on Pelosi to appease the Black Caucus is immense.

I am seeing other signs that they indeed will spend the next 2 years attempting a coup de grace on President Bush . Henry Waxman is salivating at holding inquisitions ,as are Dingal and Conyers .I do not think Pelosi can reign them in .

Former Clinton financial guru Robert Rubin and the Democrat Party's leading economic spokesman is calling for tax increases . That appears to be their answer for budget discipline.

Just days after winning an election where they promised they would not increase taxes he said :

"You cannot solve the nation's fiscal problems without increased revenues," .... "I think if you were to increase taxes right now, you would have probably about zero negative effect on the economy." The economics and politics here are worth parsing.


The idea of 'paygo' is being floated which on the surface is not bad. If you introduce new spending it has to be countered with either budget reductions in other areas or increased revenues to cover the cost. The problem is that I cannot not find too many areas where budget reductions can be agreed upon so the standard Democrat response seems to be tax increases. Secretly that is what Charles Rangal is pushing for also .

I on the otherhand think it should be a priority for Congress to eliminate the confiscatory AMT(alternate minimum tax ) and the marriage penalty . Both sides agree that the Middle Class needs more tax relief . There is 2 no brainers if you ask me .

It is interesting that he lowers expectations on the minimum wage . As I read it ,that is going to be done in the 1st 100 hours if I believe Pelosi's rhetoric. So what we will have is higher minimum wages and open borders . Wonder who the employer will hire ?

I wish that both sides could get together and reform Social Security . That 1000 lb. gorilla is not going away and every year makes it a tougher nut to crack . Panetta does not even mention it but is quick to suggest new entitlements like perscription drugs.

I am very concerned about the Iraq Study Group's recomendations .I do not like what I am reading coming out of the staged leaks . With Gates in as SECDEF I think whatever their policy recommendation is ,will be adopted not just as a policy option but ;like the 9-11 Commission Report ; a fait accomli ,with Panetta and other members of the group holding court in the media ,grading the level of adoption of the policy .

Even worse ; many of the Democrats will hold court with George McGovern to discuss policy options . He recently wrote a book arguing the case that the best option is a quick exit from Iraq ;to be complete by June. Others who have

captainoutrageous rated this answer Excellent or Above Average Answer

Question/Answer
arcura asked on 11/13/06 - Have you seen the movie "United 93"? This guy, no ...

Subject: Pilot's blog Date: Fri, 7 Jul 2006 16:31:32 +0000
This is the response from a retired Delta pilot in response to questions about whether he was planning to see "United 93."
I haven't seen the movie, yet, but I intend to when I get the chance. Retirement has made me busier than ever, and I haven't had the chance to see many movies lately.
As a Delta B-767 captain myself at the time of the attacks on 9/11 I was in crew rest in Orlando that morning. I had just turned on the TV in my hotel room only to see the World Trade Center tower on fire, and then saw the second airplane hit the other tower. My immediate reaction was
"Terrorists...we're at war", followed by the realization that we airline crew-members had all dodged a bullet; it could have bee any one of us flying those planes. As soon as the news stations flashed the first pictures of the terrorists I knew just how close and personal the bullet I dodged was. There, on the screen for all to see, was a man who had sat in my jump seat the previous July. His name was Mohammad Atta, the leader of the terrorist hijackers.
Atta had boarded my flight from Baltimore to Atlanta on July 26, 2001 wearing an American Airlines first officer uniform. He had he corresponding AA company ID identifying him as a pilot, not to mention the required FAA pilot license and medical certificate that he was required to how me as proof of his aircrew status for access to my jump seat.

An airline pilot riding a cockpit jump seat is a long established protocol among the airlines of the world, a courtesy extended by the management and captains of one airline to pilots and flight attendants of other airlines in recognition of their aircrew status. My admission of Mohammad Atta to my cockpit jumpseat that day was merely a routine exercise of this protocol.
Something seemed a bit different about this jumpseat rider, though, because in my usual course of conversation with him as we reached cruise altitude he avoided all my questions about his personal life and focused very intently upon the cockpit instruments and our operation of the aircraft. I asked him what he flew at American and he said, "These", but he asked incessant questions about how we did this or why we did that. I said, "This is a 767. They all operate the same way." But he said, "No, we operate them differently at American." That seemed very strange, because I knew better. I asked him about his background, and he admitted he was from Saudi Arabia. I asked him when he came over to this country and he said "A couple of years ago." to which I asked, "Are you a US citizen? He said no.
I also found that very strange because I know that in order to have an Airline Transport Pilot rating, the rating required to be an airline captain, one has to be a US citizen, and knowing the US airlines and their hiring processes as I do, I found it hard to believe that American Airlines would hire a non-US citizen who couldn't upgrade to captain when the time came. He said, "The rules have changed." which I also knew to be untrue. Besides, he was just, shall I say, "Creepy"? My copilot and I were both glad to get rid of this guy when we got to Atlanta.
There was nothing to indicate, though, that he was anything other than who or what he said he was, because he had the documentation to prove who he was. In retrospect, we now know his uniform was stolen and his documents were forged. Information later came to light as to how this was done.
It seems that Mohammad Atta and his cronies had possibly stolen pilot uniforms and credentials from hotel rooms during the previous year.
We had many security alerts at the airline to watch out for our personal items in hotel rooms because these were mysteriously disappearing, but nobody knew why. Atta and his men used these to make dry runs prior to their actual hijackings on 9/11. How do I know? I called the FBI as soon as I saw his face on the TV that day, and the agent on the other end of the line took my information and told me I'd hear back from them when all the dust settled. A few weeks later I got a letter from the Bureau saying that my call was one of at least half a dozen calls that day from other pilots who had had the same experience. Flights were being selected at random to make test runs for accessing the cockpit. It seems we had all dodged bullets.
Over the years my attitude towards the War Against Terrorism and the wars in Afghanistan and Iraq have been known to be on the red neck, warmongering, rah-rah-shoot-em-up side of things. I've been known to lose my patience with those who say the war in Iraq or anywhere else in the Muslim world is wrong, or who say we shouldn't become involved in that area of the world for political correctness reasons. Maybe it's because I dodged the bullet so closely back in 2001 that I feel this way. I have very little patience for political rhetoric or debate against this war because for a couple of hours back in July 2001, when I was engaged in conversation with a major perpetrator in this war, I came so close to being one of its victims that I can think in no other terms.

I don't mind admitting that one of the reasons I retired early from Delta last May, other than to protect my disappearing company retirement, was because it became harder and harder for me to go to work every day knowing that the war wasn't being taken seriously by the general public.
The worst offenders were the Liberal detractors to the present administration, and right or wrong, this administration is at least taking the bull by the horns and fighting our enemies, which is something concrete that I can appreciate. Nobody was taking this war seriously, and it seems everyone found fault with the US government rather than with those who attacked us. I found that ncomprehensible.
I also found myself being scrutinized by TSA screeners more and more every day when I went to work, and suffered the humiliating indignity of being identified about half the time for body searches in front of the general flying public who looked at the entire process as being ludicrous. "They don't even trust their own pilots!" accompanied by an unbelieving snicker was the usual response. Here I was, a retired USAF officer who had been entrusted to fly nuclear weapons around the world, who had been granted a Top Secret clearance and had been on missions over the course of 21 years in the military that I still can't talk about without fear of prosecution by the DoD, who was being scanned by a flunky TSA screener looking for any sign of a pen knife or nail file on my person.
It wasn't until six months after my retirement when my wife and I flew to Key West, FL last November that I was finally able to rid myself of the visage of Mohammad Atta sitting behind me on my jumpseat, watching my every action in the cockpit and willing to slit my throat at the slightest provocation. I missed being a headline by a mere 47 days, and could very well have been among the aircrew casualties on 9/11 had one of my flights on my monthly schedule been a transcontinental flight from Boston or New York to the west coast on the 11th of September. Very few people know that, while only four airliners crashed that day, four more were targeted, and two of them were Delta flights. The only reason these four weren't involved is because they either had minor maintenance problems which delayed them at the gate or they were scheduled to depart after the FAA decided to ground all flights. Theirs are the pilots and flight attendants who REALLY dodged the bullet that day, and my faith in a higher power is restored as a result.
I will see United 93 when I get the chance, and I will probably enjoy the movie for its realness and historical significance, but forgive me if I do not embrace the Muslim world for the rest of my life. The Islamic world is no friend of the West, and although we may be able to get along with their governments in the future, the stated goal of Islam is world conquest through Jihad and it is the extremist Jihadists, backed and funded by "friendly" Muslim governments, whom we have to fear the most. We must have a presence in the Middle East, and we must have friends in the Middle East, even if we have to fight wars to get them. Only someone who has dodged a bullet can fully appreciate that fact.
Best to all, Pat Gilmore

Editor's Note: For some reason which is beyond me, some people do not want to believe this. Perhaps they do not want to believe that Jihadist terrorism actually exists, because it someone doesn't believe it yet, they never will. Capt. Gilmore himself posted this comment, in our comments below, but I will put it here for all to see:

I assure you this letter is true. As to the fact that I wrote that a holder of an Airline Transport Pi lot&nb sp;rating (ATP) must be a US citizen, I admit that I was mistaken here. I had always assumed so, because that's what I had heard, so I looked up the requirements for an ATP just now.
There is nothing that says that US citizenship is required. Okay, I'll bite the bullet on that one. I received my ATP back in 1975 and now that I think of it I do not remember having to prove my citizenship.
However, the rest of the story is true. As for my airline career, I worked for Western Airlines (who merged with Delta in 1987), Jet America Airlines (who was bought by Alaska Airlines in 1988), and Delta Airlines, as well as a few "fly by night" cargo airlines during my furlough period from Western from 1981 - 1985. I also flew in Vietnam as a transport pilot and retired from the USAF Reserve in 1991 after the Gulf War. I have 21,500+ flight hours in T-41, T-37, T-38, C-141/L-300, CE-500, CV-440, MD-80/82, B-727, B-737, B-757, and B-767 aircraft, all logged between 1970 and 2005 when I retired from Delta.
Trust me, folks, this was real. I must admit I am quite surprised that my letter made it this far on the internet. The letter was nothing more than an innocent reply to a group of friends, one of whom sent me a similar letter from another Delta pilot who had been flying the morning of 9/11 and who had experienced the flying that day for himself. His letter had detailed his thoughts as he viewed the movie "United 93", and he also told in detail how he had been diverted to Knoxville when the FAA shut down the airspace. My friend had asked me if I had known of any other similar experiences, so I wrote him what I had encountered myself a few months before. This was my letter to him.
Another retired Delta captain contacted me yesterday after reading this blog and related an experience his wife had on a flight from Portland, OR to Atlanta in August 2001, just a week or so after my experience with Atta. She was riding on a company pass and seated in First Class. A person of Middle Eastern" descent had sought permission to sit on the cockpit jumpseat, but was denied access by the captain because he did not have an FAA Medical certificate. She said he ranted and raved because he couldn't ride the cockpit jump seat, even though there were three empty seats in First Class, which the captain offered him. What pilot in his right mind would refuse a Fir st Class seat over a cramped cockpit jump seat? He stormed off the aircraft and they left him at the gate. You see, mine wasn't the only experience leading up to 9/11.
Delta Airlines Corporate Security even contacted me a few days ago to ask if I had, indeed written this letter. I wrote them back that I had.
They were worried that someone was using my name without my knowledge. I assured them I was the author.
Keep the faith, and don't let the bastards get you down.
Pat Gilmore

tomder55 answered on 11/13/06:

I saw it .From an information viewpoint it was not as well done as the documentary about Flight 93 done by Discovery Channel .That was more informative and in my opinion better acted .If you get a chance to see it,that would be the version I recommend .

As to Pat Gilmore's larger point ,I agree.We are not yet on a war footing necessary to prosecute this war seriously and the American people ,as indicated by the elections last week,have already tired of it. They long for those good old days of the 1990s when the war was treated as a half-hearted law enforcement exercise and incidents of acts of terrorism fell withing the range of an acceptable nuisance to be tolerated .

arcura rated this answer Excellent or Above Average Answer

Question/Answer
kindj asked on 11/10/06 - What a coward!

v



v



v


v

v
v
V
V
V
V
V
V
V



(in other words, see previous question on board)

tomder55 answered on 11/11/06:

I see this as another chapter in 'Groundhog Day'. The alarm clock will ring and a new choux comment or post will appear in one manifestations or another.

Itsdb rated this answer Excellent or Above Average Answer
JesseJamesDupree rated this answer Excellent or Above Average Answer
kindj rated this answer Excellent or Above Average Answer
nikki6 rated this answer Excellent or Above Average Answer

Question/Answer
ETWolverine asked on 11/10/06 - Now that the Dems will control both the House and the Senate...

...I guess we can expect an improvement in all the problems they have complained about, through the implementation of their policies.

I guess we can expect a pullout from Iraq, which will, of course, result in the terrorists no longer hating us and trying to destroy us, an end to sectarian violence in Iraq, and the establishment of a strong, capable Democratic Iraqi government.

I guess we can expect higher taxes, which will, of course, eliminate the budget deficit, increase emplyment, put more money in the pockets of the poor, save the social security system, keep inflation low and keep the economy healthy.

I guess we can expect "windfall profits" to be taxed, which will, of course, keep the price of oil and medicines low and affordable to the poor and middle classes and the retired.

I guess we can expect open borders and illegal immigrants being granted the rights of legal citizens, which will, of course, stop the flow of illegal immigrants into the United States, keep illegals from taking American jobs, keep medical and education costs low, assure us that only legal citizens have the right to vote, and secure our borders against incursions by terrorists.

Or not.

Just how these policies will work remains to be seen. As does America's reactions to them.

Elliot

tomder55 answered on 11/10/06:

When the Iraqi's (especially all those who have bet their life by throwing their lot with us) see the last helicoper leaving and they get on those overcrowded refugee boats to relocate will we know how many jihadists hitch a ride ? We are in full mode planning our retreat now .But the illusion is that all will be well if we just leave them their turf and isolate the ummah. Jihadistan is on the march . They have probed our defenses and resolve and have in a short 6 years created a breach. The only thing that stands in the way of their victory is the remaining 2 years of a tenacious President who by all appearances is mortally wounded. The question is ; can he get on his horse one more time like El Cid and push back the horde ?

ETWolverine rated this answer Excellent or Above Average Answer
webguy rated this answer Excellent or Above Average Answer

Question/Answer
Choux... asked on 11/09/06 - GATES INHERITS FROM RUMMY MILITARY WITH COWED LEADERSHIP

WASHINGTON, Nov. 8 "If confirmed as defense secretary, Robert M. Gates would take control of a military whose ground forces are stretched and strained by a costly and bloody war and whose officers yearn to give unvarnished military advice without fear of reprisal.

Addressing their needs, hearing their views and gaining their trust are widely viewed as crucial first steps toward any change Mr. Gates has in mind for prosecuting the war in Iraq.

Task No. 1 is to generate the strategy for victory in Iraq, said a senior officer who served under Donald H. Rumsfeld, the outgoing defense secretary. A critical enabler of that, in my view, is getting the right information from the right people. One source one source of right information is the senior uniformed military who have to be empowered to speak the truth.

It may not be easy for Mr. Gates to repair the strained communications with the uniformed military, said another officer, who recalled that sessions with Mr. Rumsfeld have been nicknamed the wire-brush treatment because of his brusque style of questioning. Like others, the officer spoke on condition of anonymity out of military tradition.

Mr. Gates is a member of the independent panel reviewing the United States strategy on Iraq for President Bush, and his influence on that groups recommendations can hardly be diminished by his selection to run the Pentagon. Little is known about his views on recommendations by the panel, which is led by other heavy hitters: former Secretary of State James A. Baker III and former Representative Lee H. Hamilton.

The group has been to Iraq and already heard from the military. But that was before the announcement that Mr. Rumsfeld would be leaving.

Mr. Gates inherits the best fighting force in the world, but what he also inherits is a cowed leadership, said another recently retired senior officer.

One Air Force general said that he thought the first month of Mr. Gatess tenure would make or break his stint at the Pentagon, and that he should start with candid sessions with the joint chiefs.

Senator Carl Levin of Michigan, the ranking Democrat on the Senate Armed Services Committee, said Mr. Gates struck him as somebody who will listen, particularly to members of the uniformed services.

And I think, in that respect, he said, he will be a very pleasant change from Secretary Rumsfeld.

The chairman of the Joint Chiefs of Staff, Gen. Peter Pace, and his deputy, Adm. Edmund P. Giambastiani Jr., have been criticized by some in uniform as siding with Mr. Rumsfeld on critical issues. The question is whether the two senior officers took to Mr. Rumsfeld the arguments of other officers, and whether the senior officer corps learned to censor itself.

Inside the Pentagon, the belief is this current chairman and this current vice chairman were selected for their ability to get along with Rumsfeld, said a senior civilian at the Department of the Army.

At the same time, Mr. Gates will probably move to correct an unintended side effect of Mr. Rumsfelds management of the armed services, as the Army and Air Force have already moved outside traditional budget channels to make their case for more money directly to the White Houses Office of Management and Budget, breaking decades of agreed rules even as Pentagon spending grows to historic levels.

Just as important as improving trust between the Pentagons civilian leaders and the senior officers is mending relations between senior officers and the militarys next generation of generals and admirals some of whom see their elders as not standing up to Mr. Rumsfeld and his inner circle of political appointees when they disagreed with planning for the war in Iraq and the counterinsurgency effort.

Course corrections for Iraq are certainly anticipated, but officials predicted that Mr. Rumsfelds push for future military transformation would become a secondary priority as Mr. Gates deals with the challenges that threaten to overwhelm both the military and its budget.

Gates will focus less on transformation and more on understanding the world around us, one Pentagon official said. We all agree that needs to happen.

If confirmed, Mr. Gates might be able to shift intelligence assets to help the military mission in Iraq.

Senior military officers and Pentagon civilians, speaking Wednesday after President Bush announced the resignation of Mr. Rumsfeld, said Mr. Gatess long experience in intelligence was well suited for him to lead the militarys global campaign against terrorism.

Mr. Gates will face a Congress, especially a Democratic-controlled House, that is less hostile to him than it might have been to Mr. Rumsfeld had he continued....".

tomder55 answered on 11/10/06:

The greatest success of the Rummy tenure is the transformation program and much of it is already in place .As far as the tranformation ;it is a done deal....mission accomplished .

The resignation of Rummy signals the continued long road of the country to dhimminitude. Had he resigned and been replaced by a like minded SECDEF then I would feel better about it . But Gates (a member of the Baker Commision )is one of daddy Bush/James Baker's lackey retreads of the real-politik school of foreign policy who would rather accept a jack booted dictator running nations in the middle east (so long as he is our bastard )for the temporary promise of stability than dare offer the people an option of liberty .

I would remind anyone who is looking into the backround of Gates to look at 3 key aspects of his past .

1. He was a close advisor to Bush I during Operation Desert Storm . He was part of the crowd that also included James Baker and Colin Powell that encouraged the shia and kurds to revolt and then stood by ;not giving them support ,watching Saddam slaughter 100s of thousands of them .

He spent most of his professional life at the CIA before retiring and becoming the President of Texas A&M University. Robert Gates certainly knows the intelligence end of military affairs, but his expertise ends there. Moreover, his management skills as DCI weren't particularly impressive, and as an analyst, he was part of a CIA team that consistently got it wrong on their assessments of the former Soviet Union.

2. He was deeply involved in Iran Contra . Like it or not ,Iran Contra was an illegal operation . Carl Levin at the time of Gates confirmation hearing for Director CIA slammed him and did not vote for his confirmation based on his role . Look for that to be overlooked . Since Gates and Baker are crafting an elaborate backdoor exit from Iraq which when you strip away the smoke and mirrors resembles the worse of the Democrat plans no doubt the Iran Contra connection will be brushed off.

3. Gates wrote in an op-ed in 1994 that a NORK nuke was an inevidibility . Typical realist clap-trap .

Look for the Bush adm. to now adopt the Baker commission recommendations lock stock and barrel whether they are good ideas or not. The elections, Rumsfeld's resignation and Gates' nomination signal to friend and foe alike that the political establishment is trying to negotiate the terms of our departure from Iraq. It's only a matter of time now.The enemy knowing this will do whatever is necessary to ramp up the pressure to hasten our retreat . We have put our troops in greater danger this week.

Best to start eating pita bread and hummus ;and fit your women with the latest style in black chadora fashion designed in Tehran.

(I will gladly change my tune btw if they suprise me and prosecute every aspect of the war against jihadistan more aggressively)

Rumsfeld is serving his President by taking the fall for Iraq;a problem for America that likely would not have occurred had Rumsfeld's original post-war plan in 2003 been followed. Rumsfeld and Jay Garner, the retired general he appointed for this task, planned a very brief U.S. occupation and a swift turnover to an Iraqi provisional government. Regrettably, the Powell run State Dept staff and Viceroy L. Paul Bremer intervened and got the President's ear. After that the mission began to resemble a bridge too far ;a noble effort that could not out run the forces both external and internal ;foreign and domestic that planned to undermine it .

When the dung hits the fan next year in Iran we will all wish Rummy was there .

Itsdb rated this answer Excellent or Above Average Answer
Choux... rated this answer Bad/Wrong Answer

Question/Answer
Choux... asked on 11/09/06 - BUSH, SAME OLD RHETORICAL GARBAGE

"Yesterday I heard the same old rhetorical garbage in the President's press conference. America remains a nation at war, but the enemy that attacked us on September 11th is not the enemy we are fighting in Iraq. As of Tuesday, the President can no longer mask an Iraqi civil war as part of the real war
on terrorism.

This election was a referendum on the President's disastrous course in Iraq, and the American people clearly had ENOUGH.

I look forward to working closely with Robert Gates in the coming years. A fresh new face is what's best for this Administration. A fresh new policy is what's best for America." Representative John Murtha


!!!!!!!!!!!!!!!!!!!!!!!!!!!!!!!!!!!!!!!!

Thank you Representative Murtha for starting the movement that led to glorious defeat of the Radical Right wing Republicans on Tuesday.

Keep on keeping on representing us decent people.

tomder55 answered on 11/09/06:

According to polls in the Mideast during this years election, terrorists supported a win for the liberal Democrat party because Democrats bitterly attacked President Bush for the global war on terror, especially in Iraq.

Every Democrat in Congress ran against Bush and his war in Iraq. That resonated well with jihadist terrorists, who used the Democrat campaign propaganda in their own Mideast newspapers, radio and television outlets.



Choux... rated this answer Bad/Wrong Answer
Itsdb rated this answer Excellent or Above Average Answer
labman rated this answer Excellent or Above Average Answer

Question/Answer
excon asked on 11/09/06 - Henry Waxman


Hello wingers without a wing:

Heres a story for ya. I joined the Beverly Hills Young Democrats in 1967. I joined, not because I was a Democrat, but because I wanted to screw rich Jewish chicks. It worked. I managed to screw 8 out of the 10 chicks on the steering committee of the BHYD. Henry Waxman, our president, the nice Jewish boy that he was, hadnt screwed a single one.

But, thats not the point of this story. The point is, Henry asked me to succeed him. He would support my candidacy for president, if I would support him in his run for Tony Belinsons seat in the California state house - and I did.

I became the president of the most powerful young democratic club, in a state where the adult party was in shambles. We were the only cohesive organization in the state, and all the candidate's came to us. We put money in candidates hands, and workers on the ground. Henry won.

In 1969, if I hadnt been caught in Mexico with a load of pot in my plane, I was going to be a police commissioner under the new LA Mayor, Thom Bradley, whom I helped elect. The Wolverine will get a kick out of that. I certainly chuckle when I think about it.

A few years later, Henry defeated Thom Reese for congress and hes been there ever since. I went to visit him in 1992. He wouldnt see me.

Anyway, hes a powerful dude these days. How powerful is he? Ill betcha tom will know.

excon

tomder55 answered on 11/09/06:

Waxman has been a particular thorn to me for a long time . Every regulation intended to throw a monkey wrench into the nutritional supplement industry is intiated from his office .He claims to be some sorta of consumer advocate but he is in fact the kommisar of the food police.

But that is not what you are talking about.

Henry Waxman intends to be a very busy man as chair of the House Government Reform Committee. He intends to make full use of his "oversight" authority to bully and extract as much political blood from the Republicans in an effort to pull a coup de gras in the leadup to the 2008 elections . He ,John Dingell (Energy and Commerce Committee ...look to see hearings on Cheney energy taskforce ) and John Conyers (House Judiciary Committee) intend to tie up the executive in endless hearings in the hope of damaging the effectiveness of the executive . I fully expect a Constitutional crisis when Cheney and Bush cite executive privilege to ignore one or more of their subpoenas.

Waxman plans extensive hearings on what he calls profiteering by contractors engaged in rebuilding efforts in Iraq and Katrina. Waxman also promises to go after what he considers to be the obscene profits made in the oil and pharmaceutical industry (word to stock holders ....sell now ).Waxman is a pitbull and is very good at what he does. He is the business community's worse nightmare.

excon rated this answer Excellent or Above Average Answer
Itsdb rated this answer Excellent or Above Average Answer

Question/Answer
ETWolverine asked on 11/09/06 - Well, that didn't take long...

Charlie Rangle is just his old self... and the true face of the Democratic Party.

RANGEL 'MEANS' TO IRK VP
By IAN BISHOP Post Correspondent

November 9, 2006 -- Rep. Charles Rangel, the incoming chairman of the Ways and Means Committee, revealed yesterday that he's got his eye on Capitol Hill office space now held by the man he recently called a "son of a bitch" - Vice President Dick Cheney.

"Mr. Cheney enjoys an office on the second floor of the House of Representatives that historically has been designated for the Ways and Means Committee chairman," explained Rangel, who vaulted to the top slot of the tax-writing panel - one of the most powerful in Congress - when Democrats rolled over the GOP to take control of the House.

"I talked to [future House Speaker] Nancy Pelosi about it this morning," a giddy Rangel crowed during a news conference at his Harlem office.

"I'm trying to find some way to be gentle as I restore the dignity of that office," chuckled Rangel. "You gotta go, you gotta go."


Megan McGinn, a spokeswoman for Cheney, said: "We have not seen Congressman Rangel's comments, but it has always been our understanding that the vice president's office in the House of Representatives was on a temporary loan. As the president said this afternoon, we look forward to working with the new Congress on issues confronting this country."

In Rangel's debut news conference as Ways and Means chairman-in-waiting, he dismissed GOP claims that he's bent on tinkering with Bush's tax cuts before they expire in 2010 - or set on letting them lapse in four years.

"At my age, 76, I don't buy green bananas. Bush won't be around in 2010. We don't know what the economy is going to look like in 2010, so let's get on and see what we can do," he said.

And he dismissed the Republicans' campaign talking point that he'd suspend military funding to halt the war in Iraq.

"I want to make it clear that no member of Congress is prepared to cut funds that would put our brave men and women in jeopardy. What stops a war are the American people," he said.

"The American people have given Democrats a great opportunity to provide leadership. I am not certain whether they were in love with us, but one thing certain is that they were not satisfied with the Republican leadership."

Despite his jab at Cheney, Rangel vowed to work across party lines to tackle issues such as Social Security. "The only way that we can solve these problems is by working with the Republicans," he said. "The president's going to have to decide whether he wants to be a lame-duck president."

"If he wants to reach across the aisle, we too would like to show that the country wanted bipartisanship," Rangel added.

Still, he couldn't resist taking a shot at Bush's immigration-reform plan, calling a guest-worker program "as close to slavery as you can legally get."

Rangel is getting the last laugh in a nasty war of words with Cheney.

Down the closing campaign stretch, Cheney ridiculed Rangel as an irresponsible tax-lover who wouldn't extend a "single one" of the tax cuts.

Rangel blew his stack when Cheney told Fox News Channel, "Charlie doesn't understand how the economy works."

"He's such a real son of a bitch, he just enjoys a confrontation," Rangel snapped to The Post.

ian.bishop@nypost.com

I have a feeling that this is exactly what the country needs before the 2008 Presidential election... a two-year taste of what the Dems are really all about.

Elliot

tomder55 answered on 11/09/06:

I would be interested to see if the VEEP has historically been given an office in the House of Reps wing . I know as President of the Senate he should get one but is it common practice in the House ? and if it is;is it something the majority party grants for Veeps of their own party ?

Rangel is as ass . He denies it but I know he is itching to pull the plug on the funding of the war.


---------------------------------
Another casualty because it appears that the Senate will also go Dummycrat will be John Bolton at UN. Joe Biden will be head of the Senate Foreign Relations Committee and will block Bolton's confirmation . Before the election there was talk of Bush re-submitting Bolton's nomination. Now I doubt that's likely . (damn ) This after his brilliant working of the UN over the NORKS . A defiant Bush would nominate him anyway and then wait for another opportunity to recess appoint him or perhaps hire him in the State Dept and have him defacto- work in the U.N.

ETWolverine rated this answer Excellent or Above Average Answer

Question/Answer
arcura asked on 11/08/06 - Is this another step in the Muslims taking over?

MINNEAPOLIS, Minnesota (CNN) -- In a political first, a Muslim has been elected to serve in the U.S. Congress.
Keith Ellison, a Minnesota state legislator and lawyer, reached the political milestone by defeating two other candidates in Minnesota's 5th Congressional District, which covers the Minneapolis area.
His victory was part of the Democratic wave that seized control of the House of Representatives from the Republicans.
Ellison won 56 percent of the vote, defeating Republican Alan Fine and the Independence Party's Tammy Lee, both of whom garnered 21 percent of the vote. A Green Party candidate received 2 percent.
With 99 percent of the precincts reporting, Ellison received 135,519 votes, Fine 51,896, and Lee, 51,250.
Ellison is also the first African-American from Minnesota to be elected to the U.S. House. He ran on the Democratic-Farmer-Labor ticket in a district that is heavily liberal.
Members of that party, a uniquely Minnesotan movement, describe the DFL as the state chapter of the Democratic Party.
Ellison's winning platform
Ellison's views reflect Democratic ideals and discontent. He is opposed to the war in Iraq and on his Web site, he has called "for an immediate withdrawal of U.S. troops from Iraq."
"I opposed the war before it began. I was against this war once it started and I am the only candidate calling for an immediate withdrawal of troops."
His religious message is one of inclusiveness.
Regarding his Muslim faith, he said, "people draw strength and moral courage from a variety of religious traditions."
"Mine have come from both Catholicism and Islam. I was raised Catholic and later became a Muslim while attending Wayne State University. I am inspired by the Quran's message of an encompassing divine love, and a deep faith guides my life every day."
Ellison's position on the Israeli-Palestinian issue is supportive of the two-state solution and the road map to peace process. He has been critical of the Hamas movement.
"Peace is necessary for both Israeli and Palestinian people, and I wholeheartedly support peace movements in Israel and throughout the region," he said in a statement on his Web site.
He was endorsed by the Twin Cities newspaper, the American Jewish World, which said, "In Ellison, we have a moderate Muslim who extends his hand in friendship to the Jewish community and supports the security of the State of Israel."
Ellison is pro-choice and pro-labor, and supports "universal single payer health care" -- long popular stances among liberals.
The seat Ellison won had been held by Rep. Martin Olav Sabo, the longtime Democratic incumbent, whose retirement sparked a wide-open race. Sabo won 70 percent of the vote for the House seat in 2004.

tomder55 answered on 11/09/06:

I don't believe he ran on a muslim agenda just a liberal one . There were other ominous signs that we are well on our way toward dhimminitude but this was not one of them.

arcura rated this answer Excellent or Above Average Answer

Question/Answer
excon asked on 11/08/06 - Bwa ha ha ha -VICTORY


Hello outsiders:

I tol ya.

excon

tomder55 answered on 11/08/06:

The Senate is still an open question . Even with a one vote majority we need to remember that Cheney has a vote .

It is important to note that it was not a defeat for conservatism; it was a defeat for Republicanism, or at least, what Republicanism has come to represent. On the other board I identified the 'trojan horse strategy 'of the Democrats . What I did not add there was that the current crop of Republicans did not lose by promising a limited gvt. ;they instead tried to hold onto a big gvt.that they helped grow.

Bush has again proven that supply side tax cuts can grow revenues (how many more times does that need to be proven ? Kennedy ,Reagan ,and now Bush have increased tax revenues by cutting taxes) But deficits do grow at a rate equal to the growth of the gvt. spending; and when fingers are in the cookie jar they get dirty .

It is my view that the Democrats will not be equal to the task . They will squander the next 2 years doing investigations. But the Republicans cannot go into 2008 thinking they can run against Pelosi and Hillary and think that will be enough. They need to differentiate themselves from the Democrats . Their overall performance leads too many people like Hank to conclude there is not a dimes worth of difference.

excon rated this answer Excellent or Above Average Answer
Itsdb rated this answer Excellent or Above Average Answer

Question/Answer
HANK1 asked on 11/07/06 - SHOULD I VOTE?


Since there's been so much corruption on all levels of government and corporations, why should I vote? This has been going on since the days of Al Capone. A virgin in politics will eventually turn into a gnome after he/she is in office a year or so. De-ja-vu anyone? Don't politicians have the ability to change their colors, their long sticky tongues, and their eyes which can be moved independently of each other? Doesn't this make a politician two-eyed? How about saying one thing and doing another? Why should I vote?

Your HONEST opinion, please!

HANK

tomder55 answered on 11/08/06:

too late for me to comment now . My question to anyone is ;if the issue was corruption (and the exit polls indicated that) then in all honesty do you really think the Democrats are the answer ? Look at the record of NJs Robert Menendez ;or Harry Reid ;or the new Speaker of the House .

I think it is the civic duty of every eligible citizen to vote. If you don't like the names on the ballot then bring in a write-in.

ETWolverine rated this answer Excellent or Above Average Answer
HANK1 rated this answer Excellent or Above Average Answer

Question/Answer
paraclete asked on 11/05/06 - How Bush can win once again!

We didn't tell him earlier for the obvious reasons

Bush: woo the alien abductee vote!
Posted by Evan Maloney on Saturday, November 04, 06 at 11:45 pm

According to a study conducted in 2002 by the Roper Center for Public Opinion one in five Americans believes in Alien abductions, while 3.7 million Americans have been abducted by aliens. A potential slice of the swing-voting population in the USA.

My advice to George W. Bush is this: woo the alien abductee vote before the mid-terms, offer free counselling to any person of voting age who has been probed by an alien in the middle of the night and theres a good chance that both the Senate and House of Reps will remain in your parties culpable, I mean, capable hands. (Karl Roves response to this would probably be: Waste a time: any American who has been abducted by aliens is without a doubt already a hard-core Republican voter.")

Yes, there are some questions, too. Does anyone know if there were any UFO sighting before H. G. Wells wrote War of the Worlds? (excluding things like Apollo riding his fiery chariot across the sky) or did people just start to see UFOs after they had first been imagined by writers?

Secondly, how else might George W Bush woo the voters before the mid-terms?

tomder55 answered on 11/05/06:

sorry ;the tin foil hat moonbats have the alien abduction vote locked up .But it makes sense in a way to give it an effort .That would counter the "alien" vote that the Democrats court; but I don't think it will be enough to counter the crypt vote that traditionally goes Democrat,or their suppress the military vote efforts.

Secondly, how else might George W Bush woo the voters before the mid-terms?

Bush has been actively campaigning around the nation all week for those Republican candidates that some claimed did not want his support .

paraclete rated this answer Excellent or Above Average Answer

Question/Answer
Choux... asked on 11/04/06 - US Soldiers Committing Suicide in Iraq

Follow-Up:

"The US army has sent mental health specialists to Iraq to determine why so many soldiers are committing suicide there, a US media report said.

Eleven US soldiers and three Marines have killed themselves in the past seven months in Iraq, an annualised rate of 17 suicides per 100,000 soldiers.

The usual rate of army suicides is 13 per 100,000 soldiers, the report in the USA Today newspaper said.

A dozen other army deaths being investigated in Iraq could include suicides, and the US Navy is also investigating one possible suicide, it said.

"The number of suicides has caused the army to be concerned," said Lieutenant Colonel Elspeth Cameron Ritchie, an army psychiatrist helping investigate the deaths.

"Is there something different going on in Iraq that we really need to pay attention to?"

Most of the suicides have occurred since May 1, when major combat operations were declared over.

Depression, harsh and dangerous living conditions, a long deployment and the accessibility of weapons could contribute to the problem, experts said.

The army has sent 478 soldiers home from Iraq for mental-health reasons, the daily said."

~~~~~~~~~~~~~~~~~~~~~~~~~~~~~~~~~~~~~~~~~~~~

The much publicized story of the suicide of the female soldier can be viewed by a simple web search.

tomder55 answered on 11/05/06:

when they see reports from home of this Vietnam like mentality by the 1960s retreads I'm sure that depresses them .

Choux... rated this answer Bad/Wrong Answer

Question/Answer
Choux... asked on 11/04/06 - WORST POLITICAL ENVIRON. FOR REPUBLICANS SINCE WATERGATE

It's the worst political environment for Republican candidates since Watergate," said Glen Bolger, a Republican pollster working in many of the top races this year.

Joe Gaylord, who was the political lieutenant to Newt Gingrich when Mr. Gingrich led the Republican takeover of the House in 1994, said that based on polling he had seen in recent weeks, he expected his party to lose from 25 seats to 30 seats Tuesday. That general assessment was repeatedly echoed in interviews by Republicans close to the White House and the Republican National Committee.


~~~~~~~~~~~~~~~~~~~~~~~~~~~~~~~~~~~~~~~~~~~~~

It will be an interesting election night Tuesday seeing if the Republican bums bet thrown out.

tomder55 answered on 11/05/06:

The average gain in the House after a midterm election for the party out of power has been about 25 seats so all he is doing is citing averages. Rove is predicting that, at worst, Republicans will lose only 8 to 10 seats and he is pretty good at this stuff.

Mid term elections generally do not favor the party in power. As an example ;the 1994 mid-term elections brought a 54 seat swing when the Republicans captured the House for the first time since 1954 . The also added nine Senate seats . One of the best features of that election was the ousting of the corrupt Dan Rostenkowski.

In 1886 the Senate turned over to the Democrats (they already had a majority in the House) and Reagan's last 2 years were governed by a split gvt.

I see a shift of about 15-20 seats in the House which would give the Democrats an unworkable majority . The real race to me is in the Senate . I think it is a tossup right now . I also see the Senate as where the real danger lie for Republicans.

Without majorities in both houses they will never be able to advance their agenda ;whatever that is.But if they take both houses then they could sneak budget cut for the military and other tax increases into appropriations bills that Bush would have a hard time vetoing . They would also block all the fine work that Bush has been doing in realigning the judiciary.

If the Democrats were running on a platform loudly proclaiming what they would do once elected then perhaps they could pull off a 1994 type victory . But besides endless hearings on Bush and tax increases I don't know what their agenda is .They cant come out and say were going to roll back tax cuts ;cut and run from the war against jihadistan, and impeach Bush, yet that is precisely what they are likely to do should the electorate hand them the keys.

In the close states, they may hate the President but they will still vote for the Republican representative who has helped the district most or whom they like.
In this gerrymandered American political world, that means that it is unlikely that the Repubublicans will lose as many seats as the pundits suggest .


Choux... rated this answer Average Answer
labman rated this answer Excellent or Above Average Answer

Question/Answer
HANK1 asked on 11/04/06 - HURRAY-HURRAY:


Voting Time:

"While walking down the street one day, a U.S. senator is tragically hit by a truck and dies.

His soul arrives in Heaven and is met by St. Peter at the entrance.

"Welcome to Heaven," says St. Peter. "Before you settle in, it seems there is a problem. We seldom see a high official around these parts, you see, so we're not sure what to do with you."

"No problem, just let me in."

"Well, I'd like to, but I have orders from higher up. What we'll do is have you spend one day in Hell and one in Heaven. Then you can choose where to spend eternity."

"Really, I've made up my mind. I want to be in Heaven," says the senator.

"I'm sorry, but we have our rules."

And with that, St. Peter escorts him to the elevator and he goes down.

The doors open and he finds himself in the middle of a green golf course. In the distance is a clubhouse and standing in front of it are all his friends and other politicians who had worked with him. Everyone is very happy and in evening dress. They run to greet him, shake his hand and reminisce about the good times they had while getting rich at the expense of the people.

They play a friendly game of golf and then dine on lobster, caviar and champagne. Also present is the devil, who really is a very friendly guy. He has a good time dancing and telling jokes. They are having such a good time that before he realizes it, it is time to go. Everyone gives him a hearty farewell and waves while the elevator rises.

The elevator goes up, up, up and the door reopens on Heaven where St. Peter is waiting for him.

"Now it's time to visit Heaven."

So, 24 hours pass with the senator joining a group of contented souls moving from cloud to cloud, playing the harp and singing. They have a good time and before he realizes it, the 24 hours have gone by and St. Peter returns.

"Well, then, you've spent a day in Hell and another in Heaven. Now, choose your eternity."

The senator reflects for a minute, then he answers.

"Well, I would never have said it before. I mean Heaven has been delightful, but I think I
would be better off in Hell."

So, St. Peter escorts him to the elevator and he goes down, down, down to Hell.

The doors of the elevator open and he's in the middle of a barren land covered with waste and garbage. He sees all his friends, dressed in rags, picking up the trash and putting it in black bags, as more trash falls from above. The devil comes over to him and puts his arm around his shoulder.

"I don't understand," stammers the senator. "Yesterday, I was here and there was a golf course and a clubhouse and we ate lobster and caviar, drank champagne, danced and had a great time. Now, it's just a wasteland full of garbage and my friends look miserable. What happened"?

The devil looks at him, smiles and says, "Yesterday, we were campaigning. Today, you voted."

Source: In an e-mail I received today.

HANK


tomder55 answered on 11/05/06:

good one . two weeks ago my STAR Property Tax Rebate arrived in the mail.Propitious timing if you ask me.

HANK1 rated this answer Excellent or Above Average Answer

Question/Answer
paraclete asked on 11/03/06 - Such largese is hitherto unknown?

In a bold attempt to address the issue of reconciliation, whatever that is, the Government of the Soveriegn state of New South Wales, with a population of some 6 million people, 140,000 of them of indigenous extraction, or abstraction, according to circumstance, has decide to enter the area to address it's growing indigenous problems of riot, affray, general disorder and discontent. I am absolutely staggered by the sums of money put on the table here. These grants should be able to buy every indigenous person half a sausage at the next barbie, that is if someone provides the barbie.


Grants pool for NSW reconciliation.
4, 2006 - 10:30AM

The NSW government today announced $40,000 funding in small grants for programs to promote reconciliation between Aboriginal and main stream communities.

It is the first time the government has made all its funding available at once, in a bid to make it easier for community groups to apply for the grants, which are administered by the NSW Reconciliation Council.

Minister for Aboriginal Affairs Milton Orkopoulos said the NSW government over three years had previously handed out 37 grants, ranging from $500 to $5000 for reconciliation activities or events.

"By minimising red-tape, we hope more community groups will be aware of these funds and be able to utilise them more effectively," Mr Orkopoulos said.

Previous projects include the First Contact Memorial on Oxley Hill, overlooking Bowral in the Southern Highlands, and last month's Wollumbin Festival in the Tweed Valley, northern NSW.

Mr Orkopoulos today attended the opening of the NSW Reconciliation Council AGM, which runs until tomorrow at Darlington Public School in inner-city Darlington.

AAP

tomder55 answered on 11/04/06:

40 thousand ? Enough to make some p.r. bulletin bill boards and perhaps a radio ad. Reconciliation (whatever that is )is high on the agenda I see. Somewhere is Australia someone will erect a statue to Steve Irwin that will cost more than that .Why don't they just give them season tickets for the NSW SpeedBlitz Blues ? That will at least keep the native sedated .

paraclete rated this answer Excellent or Above Average Answer

Question/Answer
paraclete asked on 11/04/06 - I was wondering where the war is?

Police seize scores of guns, ammunition

November 4, 2006 - 2:03PM

About 80 guns and 35,000 rounds of ammunition have been seized by police from a house in Sydney's south-west.

Officers found a "strong room" containing 65 unregistered rifles of various ages and calibre, some of which were banned weapons, during a search of the Sutton Forest property yesterday, police said.

Police also seized 11 registered rifles, two registered hand guns, around 35,000 rounds of ammunition and a large quantity of firearms parts.

A 66-year-old Sutton Forest man is helping police with their inquiries.

tomder55 answered on 11/04/06:

Not really enough information to draw any conclusions .Is there a reason that the man's name is withheld ? Does it start with the name Mohammed ?

Choux is right ;stories like this are common here . What are the constitutioan rights of an Aussie to own guns? I for one consider it to be an important right although I do not own any myself. I do think that registration requirements are reasonable .But there are guns that are declared illegal to own in some parts of the US that really should not be .So collectors become law breakers.

paraclete rated this answer Excellent or Above Average Answer

Question/Answer
Choux... asked on 11/03/06 - Sen Joe Biden's Plan

Labperson said that the Democrats have no plan to end the war in Iraq. What follows is Senator Joe Biden's Five Point Plan. (In addition, Senator Murtha has a plan).

Establish three regions in Iraq -- one each for Shiite Muslims, Sunni Muslims and Kurds -- all governed by a central Baghdad-based government.

Provide the Sunni region with a 20 percent share of oil revenues, giving the Sunnis more resources than they would otherwise have and strengthening their interest in a stable Iraq. Increased stability, in turn, would attract more foreign investment, he said, which also would benefit Shiites and Kurds.

Tie increased U.S. reconstruction assistance to the protection of rights for minorities and women.

Propose a regional security conference, convened by the United Nations, that would draw a pledge from Iraq's neighbors to respect its boundaries and work cooperatively.

Withdraw most U.S. troops by 2008, leaving a force of about 20,000 to help maintain security.

~~~~~~~~~~~~~~~~~~~~~~~~~~~~~~~~~~~~~~~~~~~~~~~~


I don't necessarily support Biden's plan.

Have a lovely evening, Choux

tomder55 answered on 11/03/06:

there are 40 something Democrat Senators each with their own plan . The same goes for the Congressional Democrats . One wants immediate withdrawal ;one wants phased ;one wants redeploy to Kuwait ;one wants redeploy to Kurdistan ;one wants redeploy to Okinawa where they will be available over the horizon for rapid response 5000 miles away from the front line.

PARTITION AND RUN or maybe a better slogan would be "balkanize and beat it"

Some of Biden's plan is worthy of a good debate at least .(the oil revenue sharing I think will eventually be negotiated by the Iraqi gvt. as part of a federal plan even though 20% is probably too generous given that the Sunni population doesn't often get their fingers dirty working the oil fields)

I personally think that a partition would probably be a disaster .First no Iraqi leader has proposed such a solution and the Iraqi people have not indicated they support such a plan . Would he impose it on them ?

Counter-terrorism expert Anthony Cordesmann of the Center for Strategic and International Studies said of the Biden plan that splitting Iraq "along ethnic and sectarian lines would mean massive relocations of people and enormous upheaval in Iraqs major cities where all three groups reside.

Kurdistan would come under direct attack by Iran and Turkey who do not want to see Kurdish territory expanded. The Shia regions would immediately become a Lebanon like satellite of Tehran and the Sunni area would become a stateless enclave for terrorists or possibly become consumed by Syria. He makes no proviso for Baghdad which in itself is a divided city among the ethnics so it is doubtful that in his scenario a central authority would exist at all .

Propose a regional security conference, convened by the United Nations, that would draw a pledge from Iraq's neighbors to respect its boundaries and work cooperatively.

These partition borders and terms would be agreed to and enforced by an unspecified future agreement signed by unidentified leaders of each group and neighboring nations, an agreement produced under an international or United Nations umbrella.(his words)

All I can say to that is bwaaahaaahaaahaaa .If the neighbors were inclinded to respect Iraq territorial integrity then most of the conflict would already be resolved. A UN umbrella ? Note how effective the peace force in Lebanon is working . The German contingent and the Israelis are close to coming to blows .


Withdraw most U.S. troops by 2008, leaving a force of about 20,000 to help maintain security

Wasn't he one of the carps who were complaining that there were not enough American troops ? Under the current plans as I have seen them reported, 50,000 seems a reasonable number of troops that will be rotated out of Iraq next year. If the Iraqi army is at 75% strength by next summer, and everything else goes well, 50,000 US troops should be able to leave in the summer rotation and not be replaced.So all he is doing is restating the current administration plans and calling them his own. Biden is trying to take credit for the policy already laid out by the President which is 'as Iraqis step up Americans will stand down'.But then again he has a history of plagerizing (Biden quit the presidential race in 1988 when someone discovered he had stolen the words of a British politician. ).





Choux... rated this answer Average Answer
labman rated this answer Excellent or Above Average Answer

Question/Answer
paraclete asked on 11/02/06 - BIG BROTHER IS WATCHING? IS HE WATCHING you TOO?

UK a 'Big Brother' society
3.11.2006. 09:40:44

Britain has been labelled a 'Big Brother' society for invading personal privacy (AAP)

A new report warns Britain is becoming a "surveillance society", with cameras, credit card analysis and travel movements used to track people's lives by the minute.

The 140-page document, produced by academic group the Surveillance Studies Network, warns that people's lives will be monitored even more in the next decade by the government, the public sector, employers and big business.

Britain is ranked bottom of the democratic Western world and alongside Russia for its record on protecting individual privacy in a table published by Privacy International, a human rights watchdog.

There are up to 4.2 million CCTV (closed-circuit television) cameras about one for every 14 people or nearly 10 percent of those around the world with ever-more sophisticated technology.

Every person is caught on camera about 300 times each day.

Despite opposition from civil liberties groups, the government is pushing ahead with plans to introduce biometric identity cards, arguing it will improve internal security and curb illegal immigration.

This week, Prime Minister Tony Blair said he wants to increase the police's DNA database to cover even people released without charge.

Keeping a close eye

As international data protection and privacy commissioners met in London, Britain's information commissioner Richard Thomas said the report was a "clear signal" the country was becoming a surveillance society.

"It's not just cameras in the street and things like that. It's technology monitoring our movements and activities," he told BBC radio.

Using mobile phones, credit cards, the Internet and even driving now left an "electronic footprint", he said, while organisations increasingly shared information.

Mr Thomas whose role is to promote public access to official information and protect personal data insisted the authors of the report, which he commissioned, were not scaremongering by painting a "sinister, Orwellian picture".

Instead, it was the start of a necessary debate about what should be the limits of technology, he added.

"We've got to say: 'Where do we want the lines to be drawn? How much do we want to have surveillance changing the nature of society?" he said, accepting some uses may help in issues like counter-terrorism or serious crime.

"We've got to stand back and see where technology is taking us and make sure we are happy."

Closer surveillance looms

The report suggests that by 2016 shoppers will be scanned as they enter stores through tags in their clothes, with information matched to details on loyalty cards to recognise shopping habits.

Cars linked to satellite navigation systems will allow police to monitor speeds and journeys more closely, while workers will be subject to biometric and psychometric tests to weed out unsuitable candidates or health risks, it adds.

Care was needed to prevent creating a climate of suspicion and mistrust, it added.

Britain's Department for Constitutional Affairs accepted there was a balance to be struck between sharing information responsibly and respecting citizens' rights.

The Association of Chief Police Officers of England and Wales agreed, saying safeguards existed already against abusing surveillance and its use had to be proportionate.


SOURCE: AFP

tomder55 answered on 11/03/06:

Well of course the assumption is that there is a right to privacy . At most in the US it is an extrapolated reading of the Constitution that gives any individual privacy . A good argument can be made for the privacy in your home but it is less clear in my view that we are entitled to have that extended into the public square .

labman rated this answer Excellent or Above Average Answer
paraclete rated this answer Excellent or Above Average Answer

Question/Answer
Choux... asked on 11/02/06 - SOLDIERS COMMITTING SUICIDE

A breaking story: the cause of death for some soldiers has been falsified like in the case of the football player turned soldier, Tillman, killed by friendly fire at the beginning of the war in Afghanistan.

The story that effected me the most was the suicide of a soldier in Iraq who was in the interrogation unit, and couldn't stand it anymore and ended her life.

tomder55 answered on 11/03/06:

Army specialist Alyssa Peterson, 27, a Flagstaff, Ariz., native serving with C Company, 311th Military Intelligence BN, 101st Airborne. Peterson was an Arabic-speaking interrogator assigned to the prison at our air base in troubled Tal-Afar in northwestern Iraq. According to official records, she died on Sept. 15, 2003, from a non-hostile weapons discharge.She was the third American woman killed in Iraq, so her death drew wide press attention.

FOIA information has revealed that she did object to interrogation techniques so she was reassigned out of the interrogation unit to guard duty and suicide prevention . She did shoot herself with her service rifle.

I am disturbed by the way the military handled this ;the Tilman case and the embellishing of the Jessica Lynch story .To me there is enough heroism in the forces that fabrications are not needed . Also it is not unusual for soldiers to encounter a battlefield in their mind that leads to suicide .Suicides should be honestly reported .Friendly fire is as old as warfare . They should not have tried to cover that up either .

But perhaps if the press was a little more diligent in reporting the heroism in the field then the military would not feel the need to play P.R. games that are almost surely going to be revealed as false .

Choux... rated this answer Average Answer
labman rated this answer Excellent or Above Average Answer

Question/Answer
Choux... asked on 11/02/06 - INTERESTING RACE

Keith Ellison, the Democratic candidate running for Congress in Minnesotas fifth district, is a Muslim. If he wins, he will be the first Muslim to be elected to Congress.

~~~~~~~~~~~~~~~~~~~~~~~~~~~~~~~~~~~~~~~~~~~~~~~

There are quite a few interesting races this year. This is one election result I'll be checking out next Tuesday night.

tomder55 answered on 11/03/06:

Ellison will also become the first "person of color" elected to federal office from Minnesota .He is the Democrat nominee in an overwhelming Democrat district so I can assume he will be taking his oath of office with his hand on the Koran.(the Democrat-Farmer-Labor coalition has held the seat since 1963).

I went to his website and he toes the radical Democrat progressive agenda across the board.

He has written defenses of Louis Farrakhan against charges of anti-Semitism and a a 1990 article suggested the creation of a seperatist state for blacks.Ellison has said he was never a member of the Nation of Islam, but that he worked with it to organize the Minnesota contingent to the Million Man March .When he decided to become a politico he backed off his endorsements of Farrakhan and made overtures to the Jewish community .That got him and endorsement from ' The American Jewish World', a local Twin Cites newspaper.

Ethics charges against him include Campaign finance violations;Late payment of taxes and there was a mini-controversy over his getting his license suspended .(he cannot recall how many times that has happened due to his being a scoff-law (mostly parking tickets). There is also a sexual charge against him that to me is questionable and I'm willing to give him the benefit of the doubt until the proof otherwise is in . It involves harassment . He has a restraining order against the woman ;he denies any relationship with her . It appears to be a woman with an ax to grind because she did not get a promotion.

Choux... rated this answer Average Answer

Question/Answer
paraclete asked on 11/01/06 - A balanced view on multiculturism and immigration from a "liberal"?


Silver lining to Hilaly furore

By Chris Hurford

November 02, 2006 12:00am
Article from: The Australian


THE Australian people have rightly been up in arms over Sheik Taj al-Din al-Hilaly's outrageous remarks about women and jihadists. But for all the fretting and wailing, there could be a positive side to the past week's events.

They may encourage political leaders to toughen our settlement policies and redefine multiculturalism. For too long, some uninformed commentators have preached diversity and tolerance at the expense of integration and social cohesion. That must change.

But first to Hilaly. The provocative sheik was clearly an unsuitable immigrant to this country. He came here in 1982 on a visitor's visa, which did not require an interview. A Liberal minister allowed him to stay on an extension to a temporary visa. But Hilaly should have been sent back to Egypt, where he could have applied for a visa. If proper procedures, including an interview with immigration officials, were enforced, Hilaly would probably have been denied entry.

In 1983 Bob Hawke was elected prime minister and my Labor predecessor wrongly allowed Hilaly another extension. But when I became the minister in 1985, I decided not to approve Hilaly's application for permanent residence or to renew his temporary visa. After all, he had a lengthy history of inflaming divisions in his community. He had made little effort to settle here, including by improving his ability to speak the English language. And he had persisted in offending, for instance, Jewish Australians in his sermons, in which he chose to get involved in the Middle East conflict, one of a number of old-world discords we discouraged from being imported into our society.

In the past week, columnists and politicians have speculated about who was right and who was wrong, and have sought to drive wedges between me and former colleagues. Paul Keating and Leo McLeay, the argument goes, undid all my good work and let their petty political interests override the national interest. To be fair, my former colleagues merely acted as any member of parliament would in the circumstances. Looking after one's constituents by introducing them to the minister is hardly a sin in public life. I do not know whether they went to Hawke behind my back. Nor do I know who made the decision to grant temporary visa extensions to Hilaly after I left the portfolio in 1987, before one successor unwisely granted Hilaly permanent residence in 1990. By then I had long gone to New York as consul-general.

What I do know is that Hawke removed my excellent head of department, Bill McKinnon, sending him to New Zealand as high commissioner a couple of months before moving me into another portfolio in March 1987. Hawke did not consult me about the McKinnon move. He told me that he wanted me in a more senior portfolio, community services, being vacated by Labor's deputy leader in the Senate, Don Grimes.

I believe the reason for these moves, and for the mistakes made because of them, are found in the then prevalent conventional wisdom that so-called ethnic leaders were complaining about the settlement policies I was pursuing and McKinnon was implementing.

The accepted wisdom was generated by a false belief that there were votes in paying homage to self-chosen ethnic leaders and continuing to muddy the real meaning of multiculturalism. My intuition told me they were wrong. And the vote in the republican referendum of 1999, in which significant groups of ethnic minorities supported the constitutional monarchy, (regrettably) confirmed that intuition: that ethnic leaders, with their personal agendas, were not representative of the vast majority of immigrants, who merely yearn to make a contribution to an Australian culture that they respect. But that was then. What to do now?

Well, for starters, we are in dire need of better settlement policies. That word settlement is jargon that describes policies devoted to integrating migrants into our society. It is very important that we, too, are happy about their settling here. After all, we need migrants to help address our economic and defence vulnerabilities.

We've made some awful settlement mistakes over the years.

One of the biggest was settling migrants in those enormous camps that spawned many of the ghettos in our mainland capital cities. After I took over the immigration portfolio, we closed many oversized camps. But the damage had been done. Some of that damage can be seen in the western Sydney area of Lakemba. There are too many in that Muslim community with inadequate education and training, and too many of them are underemployed or unemployed.

Another mistake is a more recent one: a development in the 20 years since I stopped minding that difficult portfolio. There has been a retreat from interviewing toughly and with good judgment those from overseas who apply to come here; but we must choose only those who are assessed as likely to integrate well. Furthermore, we have retreated from sending home more readily those who do not make the grade before being given permanent residence. They and we would be better off if that tougher approach were reinstated.

One of the reasons for the damaging retreat from applying the old toughness and good judgment has been the disgraceful outsourcing of so much of the administration to private-sector immigration agents. Since my day, this sadly has been adopted by Labor and Liberals alike. This policy is not only very unfair to poorer applicants, who cannot afford the large fees, but abandons so many of the necessary checks that need to be made to ensure that only people who are suitable come here.

Our leaders also need to define multiculturalism more appropriately. Of course, many of us want to feel a warm inner glow when considering our achievement of settling people with the cultures of 140 separate nations. That multicultural settlement has been aided by government programs aimed at helping newcomers to recognise that we respect their cultures and want them to feel at home here while pursuing chosen aspects of their former way of life, provided their contribution to our culture conforms with our core Western values.

By these measures they have settled better and more quickly, and have learned English more readily. Alas, some, particularly in the academic class, have gone over the top and converted the adjective multicultural into a noun, multiculturalism. They have left the impression that separate development of these cultures should be an objective of policy. But does separate development ring a bell with you? South African apartheid, perhaps? This has never been the objective of our policy, nor should it be. We are not, nor should we be, a nation of many cultures. We are a multiracial nation that strongly celebrates core Western cultural values of liberal democracy.

If the Hilaly episode helps us to toughen our settlement policies and turns us to developing a cohesion in our one-Australian culture, then there has been a silver lining to this dark cloud. A solution to the Lakemba problem will result only if we recognise our mistakes of the past. We also need to do a better job of encouraging Muslim integration into our and way of life.

Chris Hurford was a federal Labor MP for the seat of Adelaide from 1969 to 1987 and immigration minister in the Hawke Labor government from 1985 to 1987.
+++++++++++++++++++++++++++++++++++++++++++++++++++++++++++

I'm sure you can see parallels in this in the immigration problems of many countries today. Let's hope the lessons can be learned without repeating the mistakes of the past?

tomder55 answered on 11/02/06:

Immigrants have in the past settled in ghettos near urban centers because that is where the jobs were; in the industrial era .

Typically there is assimilation . I think in the US this has been easier to accomplish because American culture has been evolutionary. Part of the exchange for assimilation was that we would adopt the best of the culture of the immigrants.

I'm not sure that formula works as well in established cultures that are centuries old.That is why the influx of immigrants into old Europe is causing such problems . They want the immigrants to be their as a work force but they also want them to remain isolated outside the culture .

Certainly I agree that immigrants should be vetted before entry and favor that approach here. It is up to each country to determine how many and which immigrants they can reasonably absorb.

ETWolverine rated this answer Excellent or Above Average Answer
paraclete rated this answer Excellent or Above Average Answer

Question/Answer
paraclete asked on 11/02/06 - It seems Superman ain't dead yet

he's just hiding out in Australia?


Mystery man in daring rescue

By Lincoln Archer

November 02, 2006 02:30pm
Article from: NEWS.com.au

THE hunt is on for a mystery motorist who jumped onto a moving semi-trailer and slammed on the handbrake after seeing the vehicle swerving across the road with the driver unconscious at the wheel.

The man was driving along a main road outside Cessnock in New South Wales at about 6.50am (AEDT) when he saw the truck driver slumped over the wheel of his semi, which was heading in the opposite direction, the NSW ambulance service has said.

The driver immediately did a U-turn, then drove past the truck and parked ahead of it. He then ran towards the out-of-control semi and leapt aboard.

"He climbed into the cab, pushed the driver aside and pulled on the handbrake," an ambulance spokesman has said.

The man called an ambulance and waited with the ailing truckie, a middle-aged diabetic who had fallen unconscious because of low blood sugar levels, until it arrived.

The spokesman has said the driver's action probably saved the truckie's life. "It was a huge semi so who knows what could have happened," he has said.

Without medical attention, an injection of glucose into a muscle, the truckie may have lapsed into a potentially fatal diabetic coma.

But despite his heroics, the Good Samaritan's identity remains unknown.

"He told the paramedics what had happened, then drove off. He didn't stick around," the spokesman has said.

It is believed the man is a miner and that he may have driven straight to work after his daring deeds.

"A lot of people are trying to contact him," the spokesman has said.

The driver was treated at the scene and had partially regained consciousness by the time he reached Cessnock Hospital.

However a hospital spokeswoman later said the man is in a serious condition in a "high dependency" unit, which she has described as "one step away from intensive care".

The ambulance service has said it is possible the truckie will not remember the actions of his mystery saviour.

He has said diabetics who have lapsed into unconsciousness are often "a little vague and disorientated" when they wake up.

He has said that even though ambulance officers had probably told him what had happened as he was regaining consciousness, "I doubt if he would remember anything about it".

tomder55 answered on 11/02/06:

Woke up this morning, what did I see
A big black cloud hanging over me
I switched on the radio and nearly dropped dead
The news was so bad that I fell out of bed
There was a gas strike, oil strike, lorry strike, bread strike
Got to be a Superman to survive
Gas bills, rent bills, tax bills, phone bills
I'm such a wreck but I'm staying alive

[Look in the paper, what do I see,
Robbery, violence, insanity.]

Hey girl we've got to get out of this place
There's got to be something better than this
I need you, but I hate to see you this way
If I were Superman then we'd fly away
I'd really like to change the world
And save it from the mess it's in
I'm too weak, I'm so thin
I'd like to fly but I can't even swim

Superman Superman I want to fly like Superman
Superman Superman wish I could fly like Superman
Superman Superman wish I could fly like Superman
Superman Superman I want to be like Superman
Superman Superman I want to fly like Superman
(Kinks)

ETWolverine rated this answer Excellent or Above Average Answer
excon rated this answer Excellent or Above Average Answer
paraclete rated this answer Excellent or Above Average Answer

Question/Answer
CeeBee2 asked on 11/02/06 - Brand new book about the rescue dogs of 9/11 --

I just cataloged:

Dog heroes of September 11th: a tribute to America's search and rescue dogs
by Nona Kilgore Bauer, The National Disaster Search Dog Foundation

List Price: $29.95
Hardcover: 232 pages
Publisher: Kennel Club Books
ISBN: 1593789998

It's a "coffee table" type book with lots of beautiful photos of the rescue dogs plus text about each dog's role and efforts during the rescue efforts - a terrific Christmas gift for an animal lover.

tomder55 answered on 11/02/06:

great ! I'll have my daughter reserve it for me . It's great having a librarian in the family. Don't have to deal with late fees anymore. If you can't wait there is a slide show preview here . One of the first dogs they showed is a border collie that looks like mine after a couple of days in the woods .

CeeBee2 rated this answer Excellent or Above Average Answer

Question/Answer
excon asked on 11/01/06 - 9/11


Hello:

Did 9/11 scare you personally? Do you fear for your safety, or your family's safety?

excon

tomder55 answered on 11/01/06:

At the time it did . My wife and daughter were on the observation floor of the WTC a couple of weeks before the attack and my daughter had flown in a commercial airliner for the first time without the family the weekend before the attack .

I don't know ....perhaps the experience was different for those who do not live in the Tri-State NY area . I attended memorials to firefighters murdered that day who lived in my little town .My wife had many students whos fathers or family members were killed . So for us ;yes the attack was personal.

Am I afraid ? No but I am concerned for the safety of my family . I have no reason to believe that jihadistan shot it's wad in one day . There most likely will be other attacks and I do not wish to have it become so common that we shrug it off and ,clean the broken glass ,bury the dead and go on with our lives like Israel has had to endure.

excon rated this answer Excellent or Above Average Answer
labman rated this answer Excellent or Above Average Answer

Question/Answer
ETWolverine asked on 11/01/06 - Are the Democrats experiencing a Meltdown?

This is a question that was asked of the Republicans back in 2004 due to internal party disagreements, but I have yet to hear it asked of the Democrats. Yet there seems to be much more public evidence of a Dem meltdown than a Republican one.

For instance: Kerry's insult of the troops in Iraq:

    "You know, education, if you make the most of it, if you study hard and do your homework, and you make an effort to be smart, uh, you can do well. If you don't, you get stuck in Iraq."


Then Kerry tried to defend his remarks by saying:

    "If anyone thinks a veteran would criticize the more than 140,000 heroes serving in Iraq and not the president who got us stuck there, they're crazy."


Oh really?

Let's see, is there any veteran who has ever criticized and insulted heroes in uniform as they were fighting a war?

    "[American soldiers] raped, cut off ears, cut off heads, taped wires from portable telephones to human genitals and turned up the power, cut off limbs, blown up bodies, randomly shot at civilians, razed villages in fashion reminiscent of Genghis Khan, shot cattle and dogs for fun, poisoned food stocks and generally ravaged the countryside of South Vietnam."


Oh.. yeah... that was John Kerry too, in his testimony to the Senate back in 1971. And it turned out to be false. So are we to believe that he didn't mean to insult the troops that time either?

Then there's Kerry's attacks at Bush (who isn't a candidate, I will remind you), claiming that Bush, not Kerry, should apologize to the troops. For what? For calling them the greatest military in the world and publicly stating that they can do any job they are given? And Kerry's attacks at Bush over Katerina (an issue that has nothing to do with Kerry's insults of the soldiers in Iraq).

--------------

Then there's the other, less-reported insult from the Dems: Charlie Rangle calling Dick Cheney a "son of a bitch" in an interview.

On Monday, Dick Cheney, in an interview on CNBC news, questioned Charlie Rangle's tax policies... an issue that is open for debate because Charlie is planning on becoming the Chairman of the House Ways and Means Committee. He said that Rangle would not continue a single one of President Bush's tax cuts, and that he believes that would be bad for the economy. "I don't think the stock market would like it." He also said that "CHarlie Rangle doesn't understand how the economy works." Every one of these statements are legitimate political issues. You can agree or disagree with them, but they are completely above board.

Rangle's response?

"He's such a real son of a bitch, he just enjoys confrontation," Rangle told the NY Post. Rangle then described himself as "warm and personable", while suggesting that Cheney may need "rehab" for "whatever deficit he may have suffered". And as if such personal attacks weren't enough, he insulted Cheney over the hunting-accident that took place earlier this year. When asked if he thought Cheney was mentally ill, Rangle replied "I don't think he's shot anyone in the face lately, so I'll give him the benefit of the doubt."

So rather than answer the questuions of tax policy that Cheney brought up, Rangle resorted to insults. I still haven't seen a response to the actual tax issues from Rangle's camp.

---------

Then there's Wolf Blitzer's attempt to have a "gotcha" moment against Lynne Cheney on Friday... Cheney was there to be interviewed with regard to a new children's book she has written. Instead, Blitzer tried to make it a political interview. It backfired on him BADLY, when Lynne Cheney gave him a verbal emasculation. Here's the transcript of the segment in question:

    L. CHENEY: I watched your program last night and I was troubled.

    BLITZER: All right. Well, that was probably the purpose, to get people to think, to get people to discuss these issues because a lot of conservatives and ...

    L. CHENEY: Well, all right, Wolf. I'm here to talk about my book, but if you want to talk about distortion ...

    BLITZER: We'll talk about your book.

    L. CHENEY: Well, right, but what is CNN doing running terrorist tape of terrorists shooting Americans? I mean, I thought Duncan Hunter ask you a very good question and you didn't answer it. Do you want us to win?

    BLITZER: The answer, of course, is we want the United States to win. We are Americans. There's no doubt about that. Do you think we want terrorists to win?

    L. CHENEY: Then why are you running terrorist propaganda?

    BLITZER: With all due respect -- with all due respect, this is not terrorist propaganda.

    L. CHENEY: Oh, Wolf.

    BLITZER: This is reporting the news which is what we do. We're not partisan.

    L. CHENEY: Where did you get the film?

    BLITZER: We got the film -- look, this is an issue that has been widely discussed. This is an issue that we have reported on extensively. We make no apologies for showing that. That was a very carefully considered decision, why we did that, and I think -- and I think -- that if you're ...

    L. CHENEY: Well, I think it's shocking.

    BLITZER: ...a serious journalist, you want to report the news. Sometimes the news is good, sometimes the news isn't so good but ...

    L. CHENEY: But, Wolf, there's a difference between the news and terrorist propaganda. Why do you give the terrorists the floor?

    BLITZER: And if you put it in context, that's what news is. We said it was propaganda. We didn't distort where we got it. We didn't distort anything about it. We gave it the context.

    Let's talk about another issue in the news
    , then we'll get to the book. This -- the Democrats are now complaining bitterly in this Virginia race, George Allen using novels -- novels -- that Jim Webb, his Democratic challenger, has written in which there are sexual references, and they're making a big deal out of this. I want you to listen to what Jim Webb said today in responding to this very sharp attack from George Allen.

    L. CHENEY: Now, do you promise, Wolf, that we're going to talk about my book?

    BLITZER: I do promise.

    L. CHENEY: Because this seems to me a mighty long trip around the merry-go-round.


----------

Then there's Rosie O'Donnell's ad-hominem comments regarding Condi Rice on The View:

    Joy Behar: A black female Republican can win, is my opinion, in this country.

    Babara Walters: Then youre talking about Condoleeza.

    Behar: Condoleeza could possibly run.

    Rosie ODonnell: I dont think she could win, because I think shes like that person on Scooby Doo who unzips themself and then its Dick Cheneys evil twin brother is inside of her. Thats what I think.

    Behar: Im not sure about that, because I think she has her own opinions. And I dont know if thats a hundred percent that shes not following orders right now, but if she was in the position, she would change.

    ODonnell: Ooh, Scooby, I think that scary man with the ring is actually the ghost!


-----------

So... what we see here is a growing pattern of Dems and Libs not having any responses to the comments of Republicans, and instead insulting or attacking their opponents... with little effect other than to make themselves look bad. It seems to be a party-wide meltdown... resorting to insults when logic fails them, which is fairly often. And I haven't even mentioned any older comments by Dems that liken our soldier to the Soviet Gulag or Pol Pot (Dick Durbin), that praise the "charitable work" of terrorist leader OBL without mentioning the humanitarian actions of our own troops all over the world (Patty Murray), etc.

So, is the Democratic Party suffering from a meltdown? Or do meltdowns only apply to Republicans?

Elliot

tomder55 answered on 11/01/06:

Haven't seen the View but what I hear is that everyone on the show save Walters is fairly sick of Rosie's incoherent rants.

You left out the part where Blitzer tried to equate the porn in James Webb's novels with Lynn Cheney's fiction books .

Rangel is on the record as saying that he cannot think of any of the Bush tax cuts worth salvaging and extending .Honestly if Rangel ever came through with some of his alt. plans it would not be terrible. He claims to be an advocate of PAYGO (although I suspect that he is more inclined to leave the growth of gvt. unchecked and would opt for the tax increases to satisfy PAYGO) and discretionary spending caps but if Congress is undisciplined run by the Republicans imagine how worse it will be run by the Dems.! .He claims to want to fix the Alt.Min Tax problem (which would suit me fine because due to it my wife's income is disproportionatly taxed)but the question always becomes :who do the Dems think are the middle class ? My guess is that his rhetoric runs centrist during the campaign season but his true feathers will emerge after the election .Let's hope that if the Republicans manage to hold onto the House that Rangel is true to his word .

His attacks on Cheney are way out of line but that follows insults to the President by both Pelosi and Reid so I think that is typical of them and the press reduces the impact by shortening the news cycle on comments like that from the Dems.Funny thing about it is that overall ,he was kinder to Hugo Chavez. I would've phrased it differently . I would've called Cheney




"you magnificant Bastard !"

ETWolverine rated this answer Excellent or Above Average Answer

Question/Answer
excon asked on 11/01/06 - New Direction


Hello politicos:

Time to put it on the line. I predict a Democratic landslide. They will take over BOTH houses of congress.

I also predict, that Santorum will be chosen the new Defense Secretary. Nahhh. I'm joking.... At least, I hope I am.

excon

tomder55 answered on 11/01/06:

you may be right about Congress although I think it will be with un-workable majorities which will mean that the Dems. can hold their impeachment inquiries officially instead of in cheap motel rooms that only C-Span covers .They will not be able to do much else. Bush will discover the veto pen .

As for Santorum ;yes he will probably lose but as David Brooks says ;he will be missed :

Political Theater and the Real Rick Santorum

By DAVID BROOKS (NY Slimes subscription required )
October 29, 2006

Every poll suggests that Rick Santorum will lose his race to return to the U.S. Senate. Thats probably good news in Pennsylvanias bobo suburbs, where folks regard Santorum as an ideological misfit and a social blight. But its certainly bad for poor people around the world.

For there has been at least one constant in Washington over the past 12 years: almost every time a serious piece of antipoverty legislation surfaces in Congress, Rick Santorum is there playing a leadership role.

In the mid-1990s, he was a floor manager for welfare reform, the most successful piece of domestic legislation of the past 10 years. He then helped found the Renewal Alliance to help charitable groups with funding and parents with flextime legislation.

More recently, he has pushed through a stream of legislation to help the underprivileged, often with Democratic partners. With Dick Durbin and Joe Biden, Santorum has sponsored a series of laws to fight global AIDS and offer third world debt relief. With Chuck Schumer and Harold Ford, hes pushed to offer savings accounts to children from low-income families. With John Kerry, hes proposed homeownership tax credits. With Chris Dodd, he backed legislation authorizing $860 million for autism research. With Joe Lieberman he pushed legislation to reward savings by low-income families.

In addition, hes issued a torrent of proposals, many of which have become law: efforts to fight tuberculosis; to provide assistance to orphans and vulnerable children in developing countries; to provide housing for people with AIDS; to increase funding for Social Services Block Grants and organizations like Healthy Start and the Childrens Aid Society; to finance community health centers; to combat genocide in Sudan.

I could fill this column, if not this entire page, with a list of ideas, proposals and laws Santorum has poured out over the past dozen years. Its hard to think of another politician who has been so active and so productive on these issues.

Like many people who admire his output, I disagree with Santorum on key matters like immigration, abortion, gay marriage. Im often put off by his unnecessarily slashing style and his culture war rhetoric.

But government is ultimately not about the theater or the light shows of public controversy, its about legislation and results. And the substance of Santorums work is impressive. Bono, who has worked closely with him over the years, got it right: I would suggest that Rick Santorum has a kind of Tourettes disease; he will always say the most unpopular thing. But on our issues, he has been a defender of the most vulnerable.

Santorum doesnt have the jocular manner of most politicians. His colleagues eyes can glaze over as he lectures them on the need to, say, devote a week of Senate floor time to poverty. Hes not the most social member of the club. Many politicians praise family values and seem to spend as little time as possible with their own families, but Santorum is at home almost constantly. And there is sometimes a humorlessness to his missionary zeal.

But no one can doubt his rigor. Jonathan Rauch of The National Journal wrote the smartest review of Santorums book, It Takes a Family. Rauch noted that while Goldwaterite conservatives see the individual as the essential unit of society, Santorum sees the family as the essential unit.

Rauch observed, Where Goldwater denounced collectivism as the enemy of the individual, Santorum denounces individualism as the enemy of the family. That belief has led Santorum in interesting and sometimes problematical directions, but the argument itself is a serious one. His discussion of the philosopher Alasdair MacIntyre, for example, is as sophisticated as anything in Barack Obamas recent book. If Santorum were pro-choice, hed be a media star and a campus hero.

The bottom line is this: If serious antipoverty work is going to be done, its going to emerge from a coalition of liberals and religious conservatives. Without Santorum, thats less likely to happen. If senators are going to be honestly appraised, its going to require commentators who can look beyond the theater of public controversy and at least pretend to care about actual legislation. Santorum has never gotten a fair shake from the media.

And so after Election Day, the underprivileged will probably have lost one of their least cuddly but most effective champions.


ETWolverine rated this answer Excellent or Above Average Answer
excon rated this answer Excellent or Above Average Answer

Question/Answer
HANK1 asked on 10/31/06 - KERRY, THE FOOL!



Kerry laid an egg yesterday when he spoke about education. Bottom-line: He said something like if you're not educated, you'll end up in Iraq. Boy, that guy is a piece of work.

Any comment?

HANK

tomder55 answered on 11/01/06:

Kerry began his career disparaging American troops and his disdain and contempt for the military has been evident throughout his public life. It is no suprise that this Boston Brahmin who was pampered all his life and struggled through his education reviles the lower classes .They think they are entitled to political, cultural and intellectual leadership.

HANK1 rated this answer Excellent or Above Average Answer
labman rated this answer Excellent or Above Average Answer

Question/Answer
HANK1 asked on 10/31/06 - SCANDEL:

John McCain comes across as being the guardian angel for the American people. I had a sizeable CD in the Lincoln Savings & Loan Assn. in 1989. Check this:

"In 1989, the Lincoln Savings and Loan Association of Irvine, Calif., collapsed. Lincoln's chairman, Charles H. Keating Jr., was faulted for the thrift's failure. Keating, however, told the House Banking Committee that the FHLBB and its former chief Edwin J. Gray were pursuing a vendetta against him. Gray testified that several U.S. senators had approached him and requested that he ease off on the Lincoln investigation. It came out that these senators had been beneficiaries of $1.3 million (collective total) in campaign contributions from Keating.

This allegation set off a series of investigations by the California government, the United States Department of Justice, and the Senate Ethics Committee. The ethics committee's investigation focused on five senators: Alan MacG. Cranston (D-Calif.); Dennis W. DeConcini (D-Ariz.); JOHN H. GLENN JR. (D-Ohio); JOHN S. McCAIN III (R-Ariz.); and Donald W. Riegle Jr. (D-Mich), who became known as the Keating Five."

Source: Wikepedia

I was able to recover and secure my principle because I didn't have an identure! Many retirees in Arizona and elsewhere really to it on the chin! Keating was sentenced to spend some time in the slammer. Ain't that a HOOT?

Any comments?

HANK

tomder55 answered on 11/01/06:

His role in the Keating 5 will certainly be raised as a campaign issue. There are other reasons why I am suspicious of a McCain candidacy .I think there are many better Republicans who could be the standard bearer in 2008 but there is a good possibility that he and Hillary are unstoppable speeding trains that are headed for a collision and the nation awaits on the precipice to see which one will push it over the edge .

Given the choice between the two ,who would you vote for ? The Choice ?.... Who are you most against ..... Aint that a sad commentary ?

ETWolverine rated this answer Excellent or Above Average Answer
excon rated this answer Excellent or Above Average Answer
HANK1 rated this answer Excellent or Above Average Answer

Question/Answer
Itsdb asked on 10/31/06 - Those poor Brits...

...but on the other hand, maybe it will keep algore away from the U.S.

    Brits sound alarm over warming
    Blair government hires Al Gore to advise on disaster that report says could damage the worlds economy.

    By THOMAS WAGNER Associated Press Writer

    LONDON Unchecked global warming will devastate the world economy on the scale of the world wars and the Great Depression, a British government report said Monday, as the country launched a bid to convince doubters that environmentalism and economic growth can coincide.

    Britain hired former Vice President Al Gore, who has emerged as a powerful environmental spokesman since his defeat in the 2000 presidential election, to advise the government on climate change a clear indication of Prime Minister Tony Blairs dissatisfaction with current U.S. policy.

    Blair, President Bushs top ally in the Iraq war, said unabated climate change would eventually cost the world between 5 percent and 20 percent of global gross domestic product each year. He called for bold and decisive action to cut carbon emissions and stem the worst of the temperature rise.

    It is not in doubt that, if the science is right, the consequences for our planet are literally disastrous, he said. This disaster is not set to happen in some science fiction future many years ahead, but in our lifetime.

    The report emphasized that global warming can only be fought with the cooperation of major countries such as the United States and China, and represents a huge contrast to the Bush administrations wait-and-see global warming policies.

    Sir Nicholas Stern, the senior government economist who wrote the report, said that acting now to cut greenhouse gas emissions would cost about 1 percent of global GDP each year. He recommended a low-carbon global economy through measures including taxation, regulation of greenhouse gas emissions and carbon trading.

    Bush kept America out of the Kyoto international treaty to reduce greenhouse gases, saying the pact would harm the U.S. economy. The international agreement was reached in Kyoto, Japan, in 1997 and expires in 2012.

    Blair made his displeasure with U.S. environmental policy clear when he signed an agreement this year with California Gov. Arnold Schwarzenegger to develop new technologies to combat the problem. The measure imposed the first emissions cap in the United States on utilities, refineries and manufacturing plants in a bid to curb the gases that scientists blame for warming the Earth.

    The prime minister and the report also said that no matter what Britain, the United States and Japan do, the battle against global warming cannot succeed without deciding when and how to control the greenhouse gas emissions by such fast-industrializing giants as China and India.

    The report said warming could have effects such as melting glaciers, rising sea levels, declining crop yields, drinking water shortages, higher death tolls from malnutrition and heat stress, and widespread outbreaks of malaria and dengue fever.


Is the science right? Will algore run for Prime Minister or will he shoot for World President? Or will he eventually realize his hot air is a significant contributor to global warming?



I just love that picture...

tomder55 answered on 10/31/06:

I made comments about Gore in my replies to Paraclete

Basically it is very disturbing that a former Veep (and Clinton is doing this also ) is acting as an agent for a foreign country . It is K Street on steroids if you ask me.

Itsdb rated this answer Excellent or Above Average Answer

Question/Answer
Itsdb asked on 10/31/06 - Kerry: Go to college or get stuck in Iraq

Kerry must not have gotten the memo to mask his liberalism while campaigning...

    Posted by Warner Todd Huston on October 31, 2006 - 09:39.

    The Pasadena Star News yesterday reported on a rally for failing California Democratic gubernatorial candidate, Phil Angelides, at which John Kerry warned students in the audience that if they didn't get an education they would have no other alternative than to be forced into the US Armed Forces.

    "Kerry then told the students that if they were able to navigate the education system, they could get comfortable jobs - 'If you don't, you get stuck in Iraq,' he said to a mixture of laughter and gasps."

    Obviously, Kerry feels that all our soldiers are uneducated louts with no other opportunities.

    So, how many news outlets do you think will bother to cover this gasp inducing claim from former Democratic Party presidential candidate, John F. Kerry?

    Let's see if this is picked up by anyone. It's been a day already and only Drudge and Neil Boortz seems to have caught it thus far.


What Kerry said is typical, no surprise. He should be called to the carpet for one more slam at the finest, smartest, toughest military in the world he once compared to 'terrorists,' but what gets me is his audience laughed. How pathetic, and another reason why Democrats can't be trusted to run this country.

~~~~~~~~~~~~~~~~~~~~~~~~~~~~~~~~~~~~~~~~~~~~~~~~~~~~~~

Here's how the San Diego CBS affiliate reported it:

    The race for governor comes to San Diego Tuesday. Democrat Phil Angelides will be stumping in America's Finest City.

    Monday night, Angelides received support from Senator John Kerry in Pasadena.

    "Phil's life in American politics has been characterized by a desire to make government work for people," said Senator Kerry.

    Senator Kerry won California's electoral votes in 2004. Angelides is clearly hoping some of that popularity will rub off as he continues to trail in the polls.


Guess they missed the rest of his speech.

~~~~~~~~~~~~~~~~~~~~~~~~~~~~~~~~~~~~~~~~~~~~~~~~~~~~~~

McCain laid into Kerry, and Kerry repsonded this way:

    A Kerry adviser said that Kerry was referring to the general intellectual abilities of Pres. Bush and not U.S. troops. The quotation, the adviser admits, came out slightly "mangled."

    Late 10/30, Kerry issued a response to criticism from WH press. sec. Tony Snow: "If anyone thinks a veteran would criticize the more than 140,000 heroes serving in Iraq and not the president who got us stuck there, they're crazy. This is the classic G.O.P. playbook. I'm sick and tired of these despicable Republican attacks that always seem to come from those who never can be found to serve in war, but love to attack those who did."

    Kerry aide David Wade: "When will John McCain ask his new Republican best friends George Bush and Dick Cheney to apologize for misleading America into war and giving us a Katrina foreign policy that has betrayed our ideals, killed and maimed our soldiers, and widened the terrorist threat instead of defeating it? John McCain's neoconservative pals are afraid to debate veterans who live and breathe the concerns of our troops, not the empty slogans of an Administration that sent our brave troops to war without body armor."


Yeah, that's it, it's Bush's fault Kerry can't keep his mouth shut and it's "despicable Republican attacks" to criticize him for demeaning our men and women in uniform. Go ahead and vote Democrat, this is the kind of idiot you're gonna get.

tomder55 answered on 10/31/06:

it is reflective of the elitist attitude of the left ......what Mark Levin calls the Mercedes Marxists .

If that is not the fact then I call on Ned Lamont; Bob Casey Jr. ;Benjamin Cardin and all other Democrat's who are now running for leadership positions in Congress to repudiate Kerry's scurillous characterization.

"You know, education, if you make the most of it, if you study hard and do your homework, and you make an effort to be smart, uh, you can do well. If you don't, you get stuck in Iraq." John Kerry


As Paul Simon said "Everything looks worse in black and white."


It is just too bad for Little Lord John Forbes Flaunt-le-Kerry that the facts get into the way of a good populist screed.

1)A growing number of Non Commissioned Officers, Limited Duty Officers and Chief Warrant Officers have or are obtaining college degress while in uniform which is adding formal education to a massive pool of experenced professionals that shape and model jr. military members.

2)Commands that once depended heavily on officers at the division level, today can additional depend on College Educated Officer-In Charge(E-7 and Above)and give to the Flag and Commanding Officers a greater ability to assign officers as needed by the command and when needed, thus putting more officers in the field.

3)Military Members from Iraq to Kosovo are taking part in online and distance learning college courses, while still meeting the mission and tasking requirements.

4)Flag and Commanding Officers are encourging jr. enlisted members to take courses offered by colleges located close to their bases. These Commanding Officers also encourge those Non-Commissioned Officers and Officers with degress to provide support services and encourgement to those Jr. members who are in college and the jr. members earn promotion points on their military evals yearly.

5)Today Military is techno based from UAV to Smart Bombs ,military members today have more general education than their World War 2, Korean and Vietnam counterpart combine. Boot Camps have moved away from shaping the physical prowlness of member to classroom based training, members today spend up to 75% of their boot camp training in a classroom setting.

6)A new military member of today arrives at his unit with education-based training from boot camp to tech school, which in turn cuts down on individial training in the field and adds to the unit as a whole

7)A E-7 and above with more than 20 years of military service today in some cases has just as much, if not more formal college education than recently commissioned officers.

8)During the Presidency of Ronald Reagan policies were put in place that encourged enlisted military members to obtain formal college educations and was part of the transformation and build up of our forces.

9)A majority of first time enlisted military members in uniform today have at least 10 college credit hours thru formal classroom training or college level courses.

10)Formal classroom based training has in the last 20 years cut maintain repair times in half and saved the American Taxpayer billions in repair cost and maintainence upkeep.



I meant what I said before
I had to say that I didn't mean it
like I said it.



Itsdb rated this answer Excellent or Above Average Answer

Question/Answer
arcura asked on 10/31/06 - Is there any GOOD news about progress in Iraq?

Watch this video.
http://www.glennbeck.com/realstory/iraq-video.shtml

tomder55 answered on 10/31/06:

can't link to video but yes there is .

The 3rd Battalion, 1st Brigade of the 7th Iraqi Army Division, commanded by Colonel Kareem, officially assumed responsibility for operations in Ramadi . This continues the trend of Iraqis"standing up" .

I admit that recently it is hard to find encourging news . The jihadists waged a Tet like offensive during the "HOLY MONTH " of Ramadan aimed at influencing the November American elections . Their collaborators in the Press were practically popping the champage in their glee when the news broke that over 100 Americans were killed this month.The NY Slimes made their cover story a photo expose of funerals at Arlington.

It is useful to look at what we have accomplished there. Here are some of them .Saddam Hussein was removed from power and is on trial for genocidal atrocities . His homicidal sadistic offspring were offed and the reign of the family has come to an end. A rogue State sponsor of terrorism has been CONFIRMED disarmed from WMD.

The marshes have been restored and are rehabilitiating from one of the biggest environmental man made disasters in the region. (Saddam was also responsible for the torching of the Kuwaiti oil wells in 1990 ;another major environmental calamity )
The Iraqi economy is growing ;it's currency is stable and convertable . Dozens of newspapers and other media has opened that are not state sponsored or controlled . The Kurds finally have a secure homeland .

Elections have been held thrice . A freely elected assembly runs the central gvt. (clumsily for sure but they never did this type of thing before ).The people freely approved a constitution

Abu Musab al-Zarqawi was killed as was and is a whole generation of al-Qaeda jihadists .

I could go on but you get the idea. All we have to do is disprove the jihadist's theory (expressed in publication by both Saddam Hussein and OBL )that the US is a paper tiger that is unwilling to wage a prolonged protracted conflict ;that the US gets war weary and will stay until the last helicopter retreats .

We have given them reason to believe this in the past in the desert of Iran,in Beirut ,in Mogodishu ,and in Vietnam. The question is ;will history repeat?



arcura rated this answer Excellent or Above Average Answer
ETWolverine rated this answer Excellent or Above Average Answer

Question/Answer
paraclete asked on 10/31/06 - A storm in a teacup, perhaps?


World 'furious' over Aussie climate inaction

October 31, 2006 10:16am
Article from: AAP


THE rest of the world was furious at Australia's refusal to sign the Kyoto Protocol, the Greens said today.

A major report released in Britain overnight warns of economic and climatic disaster if urgent action is not taken to tackle global warming.

The Stern Review on the Economics of Climate Change said action now would save about $US2.5 trillion ($3.25 trillion) compared with doing nothing, and would help avert catastrophe.

The report's author, British government Economic Service head and former World Bank chief economist Sir Nicholas Stern, said the Kyoto Protocol should be seen as a first step towards global emissions trading.

However, the federal Government says there is no need for Australia to sign the protocol.

Greens senator Christine Milne said this worried the global community.

"The rest of the world community is, in fact, privately furious and despairing about Australia," Senator Milne said.

She said the Government just didn't understand the ramifications of global warming.

"When you come to climate change in the last five minutes as the Howard Government has done, I guess it is pretty difficult to absorb what those of us who have been studying for 20 years understand."

"It means the Great Barrier Reef dead; it means huge desert areas across the world, including expanded deserts in Australia; it's a massive sea level rise," she said.

Senator Milne said Sir Nicholas wanted to give Australia room to change its position on climate change.

"What Sir Nicholas Stern is doing is trying to create the space for Australia to change its position without losing face," she said.

But Australia had been given too much space already.

"What Sir Nicholas Stern doesn't understand is that appeasing Australia just encourages bad behaviour."

Greens leader Bob Brown it was a disgrace that Australia was not acting on climate change.

"Australia should be leading the world on climate change technology - we've got the best solar powered technology in the world. But it's all being exported to China and to Germany and elsewhere because this Government won't get behind it."

++++++++++++++++++++++++++++++++++++++++++++++++

Now I would have thought that if we have the technology and we were exporting it to the real polluters, this was a good thing, but appearently not. I think we are expected to turn the country into one vast solar cell, but who will turn the lights on at sundown?

tomder55 answered on 10/31/06:

An editorial in Investors.com details the Brit scheme to add a confiscatory "eco-tax" .

The Irish Times details how Al Gore is now a paid agent for the Brits . His job is to convince the American public that action must be taken urgently to combat the "disastrous" threat of global warming.

I am not sure that Gore can be any more an advocate than he already has been ,but this news is galling that a former Vice President of the country would shill himself to a foreign country to advance their interests in our country .


But the best story about global warming did not come from either Australia or England . It came from Bangor ,Maine.

According to the NY Slimes :

Michael Palmer, the general manager of television stations WVII and WFVX, ABC and Fox affiliates in Bangor, has told his joint staff of nine men and women that when Bar Harbor is underwater, then we can do global warming stories.

Until then, he added. No more.

Mr. Palmer laid out his policy in an e-mail message sent out during the summer. A copy was sent to The New York Times. Mr. Palmer did not respond to a phone message left with an employee of the stations nor to an e-mail message. But a former staff member confirmed the e-mail message that went out during the summer after the stations broadcast a live report from a movie theater in Maine where Al Gores movie on global warming, An Inconvenient Truth, was opening.

Mr. Palmer began his e-mail message: I was wondering where we should send the bill for the live shot Friday at the theater for the Al Gore commercial we aired.

Mr. Palmer said he wanted no more stories broadcast on global warming because: a) we do local news, b) the issue evolved from hard science into hard politics and c) despite what you may have heard from the mainstream media, this science is far from conclusive. Mr. Palmer said in his e-mail message to his operations manager and two women who served as a news anchor and a reporter that he placed global warming stories in the same category as the killer African bee scare from the 1970s or, more recently, the Y2K scare when everyones computer was going to self-destruct.


Then again Bar Harbor is one of my favorite vacation destinations and I'd hate to see it underwater . I doubt if Palmer will be getting an invitation to the Society of Environmental Journalists any time soon .


paraclete rated this answer Excellent or Above Average Answer

Question/Answer
paraclete asked on 10/31/06 - A storm in a teacup, perhaps?


World 'furious' over Aussie climate inaction

October 31, 2006 10:16am
Article from: AAP


THE rest of the world was furious at Australia's refusal to sign the Kyoto Protocol, the Greens said today.

A major report released in Britain overnight warns of economic and climatic disaster if urgent action is not taken to tackle global warming.

The Stern Review on the Economics of Climate Change said action now would save about $US2.5 trillion ($3.25 trillion) compared with doing nothing, and would help avert catastrophe.

The report's author, British government Economic Service head and former World Bank chief economist Sir Nicholas Stern, said the Kyoto Protocol should be seen as a first step towards global emissions trading.

However, the federal Government says there is no need for Australia to sign the protocol.

Greens senator Christine Milne said this worried the global community.

"The rest of the world community is, in fact, privately furious and despairing about Australia," Senator Milne said.

She said the Government just didn't understand the ramifications of global warming.

"When you come to climate change in the last five minutes as the Howard Government has done, I guess it is pretty difficult to absorb what those of us who have been studying for 20 years understand."

"It means the Great Barrier Reef dead; it means huge desert areas across the world, including expanded deserts in Australia; it's a massive sea level rise," she said.

Senator Milne said Sir Nicholas wanted to give Australia room to change its position on climate change.

"What Sir Nicholas Stern is doing is trying to create the space for Australia to change its position without losing face," she said.

But Australia had been given too much space already.

"What Sir Nicholas Stern doesn't understand is that appeasing Australia just encourages bad behaviour."

Greens leader Bob Brown it was a disgrace that Australia was not acting on climate change.

"Australia should be leading the world on climate change technology - we've got the best solar powered technology in the world. But it's all being exported to China and to Germany and elsewhere because this Government won't get behind it."

++++++++++++++++++++++++++++++++++++++++++++++++

Now I would have thought that if we have the technology and we were exporting it to the real polluters, this was a good thing, but appearently not. I think we are expected to turn the country into one vast solar cell, but who will turn the lights on at sundown?

tomder55 answered on 10/31/06:

I'd have to say that very few raised an eye-brow when Australia did not sign . The protocol is flawed beyond any usefulness. I guess given that Australia is a semi-isolated island continent ;that makes it the perfect laboratory for the greenies .To hell with the fact that Australia in general has been as responsible as any nation ;and that the critics hail from nations whose records are less than environmentally friendly ;that most signatories are not keeping up their end anyway ;and that they could give a rat's a** about your life style.

Al Gore flys in more private planes ;SUVs and lives in mansions that you and I have ever dreamed about .Yet the Brits just hired him to try to convince us in the States that it would be a good idea to implement a global warming tax. They could care less about the environment . They are more interested in picking our pockets .

Itsdb rated this answer Excellent or Above Average Answer
labman rated this answer Excellent or Above Average Answer
paraclete rated this answer Excellent or Above Average Answer

Question/Answer
Itsdb asked on 10/28/06 - Take two

Since some didn't like my quoting MIchelle Malkin quoting the NY Times, here it is from the NY Times itself:

    Banking Data: A Mea Culpa

    Since the job of public editor requires me to probe and question the published work and wisdom of Times journalists, theres a special responsibility for me to acknowledge my own flawed assessments.

    My July 2 column strongly supported The Timess decision to publish its June 23 article on a once-secret banking-data surveillance program. After pondering for several months, I have decided I was off base. There were reasons to publish the controversial article, but they were slightly outweighed by two factors to which I gave too little emphasis. While its a close call now, as it was then, I dont think the article should have been published.

    Those two factors are really what bring me to this corrective commentary: the apparent legality of the program in the United States, and the absence of any evidence that anyones private data had actually been misused. I had mentioned both as being part of the most substantial argument against running the story, but that reference was relegated to the bottom of my column.

    The source of the data, as my column noted, was the Society for Worldwide Interbank Financial Telecommunication, or Swift. That Belgium-based consortium said it had honored administrative subpoenas from the American government because it has a subsidiary in this country.

    I havent found any evidence in the intervening months that the surveillance program was illegal under United States laws. Although data-protection authorities in Europe have complained that the formerly secret program violated their rules on privacy, there have been no Times reports of legal action being taken. Data-protection rules are often stricter in Europe than in America, and have been a frequent source of friction.

    Also, there still havent been any abuses of private data linked to the program, which apparently has continued to function. That, plus the legality issue, has left me wondering what harm actually was avoided when The Times and two other newspapers disclosed the program. The lack of appropriate oversight to catch any abuses in the absence of media attention was a key reason I originally supported publication. I think, however, that I gave it too much weight.

    In addition, I became embarrassed by the how-secret-is-it issue, although that isnt a cause of my altered conclusion. My original support for the article rested heavily on the fact that so many people already knew about the program that serious terrorists also must have been aware of it. But critical, and clever, readers were quick to point to a contradiction: the Times article and headline had both emphasized that a secret program was being exposed. (If one sentence down in the article had acknowledged that a number of people were probably aware of the program, both the newsroom and I would have been better able to address that wave of criticism.)

    What kept me from seeing these matters more clearly earlier in what admittedly was a close call? I fear I allowed the vicious criticism of The Times by the Bush administration to trigger my instinctive affinity for the underdog and enduring faith in a free press two traits that I warned readers about in my first column.


But will they do it again?

tomder55 answered on 10/29/06:

meanwhile an important data collecting technique has been severly compromised. well done NY Slimes.....certainly serving the public interest aren't ya ?

excon rated this answer Excellent or Above Average Answer
Itsdb rated this answer Excellent or Above Average Answer
labman rated this answer Excellent or Above Average Answer

Question/Answer
Itsdb asked on 10/28/06 - Oops, my bad...

Michelle Malkin points out the NY Times was mistaken in leaking the classified terrorist financing program:

"I don't think the article should have been published. . . . I haven't found any evidence in the intervening months that the surveillance program was illegal. . . . The lack of appropriate oversight to catch any abuses in the absence of media attention was a key reason I originally supported publication. I think, however, that I gave it too much weight."

At least they admit the mistake. Any bets they'll learn from this?

tomder55 answered on 10/29/06:

Wish I knew which page they published this mea culpa on. I'm sure it wasn't page one .

The real point of course is that they published BEFORE they did their homework in an effort to extract as much political gain for the Dems as possible.

excon rated this answer Excellent or Above Average Answer
Itsdb rated this answer Excellent or Above Average Answer

Question/Answer
excon asked on 10/28/06 - Country Club - Gitmo


Hello wrongwingers:

I'm sorry. I was wrong. They've installed a swimming pool for the inmates at Gitmo. Because we're so nice, after the've taken a "dunk", they're more than willing to tell us anything we want to know.

Bush is soooo wonderful... I really love it when he tells us the truth, like he just did. We don't torture. I believe it now - specially after news about the pool.

excon

tomder55 answered on 10/29/06:

? I dunno to many of the scum in Gitmo I'm sure taking a bath is .

Cheney was responding to a radio interviewer from North Dakota station WDAY who asked whether water boarding, which involves simulated drowning, was a "no-brainer" if the information it yielded would save American lives.

Seems like a no brainer to me . Our enemy would only dunk our heads after they severed it from the rest of our body .

I hear the big lie constantly that aggressive interrogation doesn't yield useful intel but The U.S. military's escape and evasion schools( SERE ) teach that anyone will break and give up secrets if they are subject to this technique long enough . And it is no shame to break.Every military air crew has been subject to waterboarding as part of their training .

excon rated this answer Excellent or Above Average Answer

Question/Answer
CeeBee2 asked on 10/24/06 - Is this why we're in Iraq?

A good reason?

or, if the link doesn't work --

http://alternet.org/story/43045/

tomder55 answered on 10/26/06:

If you are into conspiracy theories then I recommend a new book about Cheney called "Vice: Dick Cheney and the Hijacking of the American Presidency " by Lou Dubose and Jack Bernstein. They trace the evil conspiracy concocted by Cheney and the "Bush cabal" back to Cheney's heart attacks which accordingly screwed up his mind. The cabal was organized in the final days of the Ford Administration .They kicked out Nelson Rockefeller and demoted Henry Kissinger and arranged for the ousting of Richard Colby at the CIA and James Schlesinger from Defense Sec. and installed GHW Bush as the CIA head.
Cheney went under radar relativiely and temporarily filled a spot in Congress .Then the cabal waited until Reagan training the next generation (Wolfowitz; Gingrich and the Oedipal GW ).

During Reagan the cabal went to work by promoting GHW Bush to VP where he secretly managed 'Iran Contra 'and other nefarious enterprises. By the time GHW Bush became president all the ducks were in a row to being their plan to take over the Persian Gulf oil fields .

I love good fiction as much as the next guy.


I remember after 9-11 the tin-foil hat crowd was going on about the plot to take over Afghanistan and to contruct an oil pipeline from the Caspian through Afghanistan and Pakistan to the Indian Ocean and that the Taliban was in the way .That was supposedly why the cabal let their agent OBL blow up the WTC to give the cabal an excuse to remove the Taliban and install their puppet Karzai . Amazing how often these schemes by real the rulers of the world fail huh ?

CeeBee2 rated this answer Excellent or Above Average Answer

Question/Answer
Itsdb asked on 10/24/06 - Zogby

In the latest Zogby poll, 10/10-10/16 (post-Foleygate):

Missouri Senate - Talent* (R) 50% McCaskill (D) 47%

Virginia Senate - Allen* (R) 50% Webb (D) 47%
(on 9/5 Webb led 50% to 43% for Allen)

Pennsylvania Senate - Santorum* (R) 44% Casey Jr. (D) 52%

Ohio Senate - Brown (D) 49% DeWine* (R) 45%
(DeWine was down 45% to 37% in July)

New Jersey Senate - Kean Jr. (R) 47% Menendez* (D) 45%
(Kean has climbed 10 points since 9/5)

*Incumbent

Still close, but the GOP is in the lead in 3 of John Kerry's 4 must-win senate races, DeWine is narrowing and Kean is ahead of the Democrat incumbent. And oh yeah, Lieberman is still ahead of Lamont.

tomder55 answered on 10/25/06:

maybe Michael J Fox will make the difference ? In Maryland ;another close race, He did his stem cell ad in favor of Democrat Cardin even though Cardin is the candidate who when given a choice actually voted against stem cell research . He is ahead of Steele by an average of 7.6% and Steele has been making up ground rapidly .

Dewine is making real progress but I think that Republican politics in Ohio is so bad that I wonder if he can pull it off.

The race that disappoints me is the Penn. race . Casey Jr. is running on name recognition alone .He is a political lightweight but will unseat one of the best Senators.

I also hope that Allen survives .It will be a testament if he does given the total focus on defeating him being done by the moonbat bloggers and the Washington Compost . Notice how they have jumped ship on Lamont .

The NJ race also has a name recognition factor on the Republican side .Tom Kean has managed to keep himself in the lime-light since his Govenorship. Tom Kean Jr.has been very supportive of the President and has not tried to "run away"from him .It is a joke that Menendez (the un-elected Senator ...he was appointed by the Govenor Jon Corzine to fill the seat made vacant by Corzine's resignation from the Senate to serve as Governor . );one of the most corrupt politicians in the most corrupt county ;Hudson Countythe bastion of dirty Democratic politics;in a corrupt state,is even close in this election .

It is a great soap opera like read . Menendez allegedly left his wife to live with his lover Kay LiCausi,who was employed at KL Strategies, Inc. as a staffer .Through her relationship with him she climbed to a well paid lobbyist according to the NY Slimes .


"This woman starts out as a midlevel staffer and then, all of a sudden, she's the greatest lobbyist on the East Coast?" said Bobby Jackson, the publisher of a small newspaper in Jersey City, who supported Glenn Cunningham, the former mayor of Jersey City, a political opponent of Mr. Menendez.
[...]
The tale of her swift success, however, is complicated by the widespread belief among elected officials and political consultants in Hudson County and former members of Mr. Menendez's staff that she and the congressman had a romantic relationship.


The relationship has extended to lucrative deals for KL with Menendez's PAC ;New Millennium PAC ...$45,000 in media consulting fees between January 2003 and September 2003.Menendez's own expeditures show that KL Strategies received $61,493 in fundraising consulting fees in the 2003-2004 election cycle.

This is just one of the many charges of corruption being leveled against him but NJ being what it is will probably return him to the Senate . Then when he is frog marched out in and orange jump suit ,the Govenor can again appoint his replacement as stomach turns .

Itsdb rated this answer Excellent or Above Average Answer

Question/Answer
paraclete asked on 10/24/06 - When you are top of the food chain, life is tough, but someone has to do it?

Poor environment rating for Australia, report

October 24, 2006 - 6:34PM

Australia has been lambasted in a new environment report that ranks it in the top ten countries living unsustainably.

Conservation group WWF-Australia says if the rest of the world copied Australians' lifestyles more than three planets would be needed to provide the resources and absorb the waste.

Australians still have one of the biggest so-called ecological footprints in the world, and produce more greenhouse gases per person than most other countries on the planet.

The Living Planet Report 2006, the international WWF's biennial statement on the state of the natural world, says that on current projections, humanity will be using two planets worth of natural resources by 2050.

It says Australia's ecological footprint - the amount of land and water area a human population uses to support its lifestyle - is currently at 6.6 global hectares per person per year.

That ranks Australia behind countries such as the United Arab Emirates, the United States and Kuwait, but above the United Kingdom, Russia, China and Japan.

The report identifies Australia as having the sixth biggest ecological footprint out of the 147 countries studied.

"The report confirms why it is that we are experiencing the kinds of problems we are right now, such as critical water shortages, the unprecedented decline of species, stressed fisheries and land degradation," WWF-Australia chief executive Greg Bourne said in a statement.

"If the rest of the world led the kind of lifestyles we do here in Australia, we would require three and a half planets to provide the resources we use and to absorb the waste we create."

The report says carbon dioxide continues to be the single largest component, accounting for about 51 per cent of Australia's ecological footprint.

"Cutting carbon dioxide emissions and setting targets for greenhouse gas reductions are essential if Australia is to reduce its ecological footprint to sustainable levels," Mr Bourne said.

The report comes as the federal government is set to announce major allocations of funding towards alternative energy technologies, principally for clean coal.

WWF called on the government to set a greenhouse gas emission reduction target of 30 per cent by 2030, which would put Australia on the path for a 60 per cent reduction by 2050, as recommended by the world's leading climate scientists.

The group also called for an end to land clearing in Australia, which it said was the number one threat to biodiversity, and for the implementation of a system of ecological accounting along the same lines as Australia's national accounts.

AAP

So which two planets will we use up first, Mars, Titan, Venus, ????????????????????????????????

tomder55 answered on 10/24/06:

think Im sophisticated
cos Im living my life like a good homosapien
But all around me everybodys multiplying
Till theyre walking round like flies man
So Im no better than the animals sitting in their cages
In the zoo man
cos compared to the flowers and the birds and the trees
I am an ape man
I think Im so educated and Im so civilized
cos Im a strict vegetarian
But with the over-population and inflation and starvation
And the crazy politicians
I dont feel safe in this world no more
I dont want to die in a nuclear war
I want to sail away to a distant shore and make like an ape man
Im an ape man, Im an ape ape man
Im an ape man Im a king kong man Im ape ape man
Im an ape man
cos compared to the sun that sits in the sky
Compared to the clouds as they roll by
Compared to the bugs and the spiders and flies
I am an ape man
In mans evolution he has created the cities and
The motor traffic rumble, but give me half a chance
And Id be taking off my clothes and living in the jungle
cos the only time that I feel at ease
Is swinging up and down in a coconut tree
Oh what a life of luxury to be like an ape man
Im an ape, Im an ape ape man, Im an ape man
Im a king kong man, Im a voo-doo man
Im an ape man
I look out my window, but I cant see the sky
cos the air pollution is fogging up my eyes
I want to get out of this city alive
And make like an ape man

paraclete rated this answer Excellent or Above Average Answer

Question/Answer
excon asked on 10/24/06 - Are we in trouble???


Hello wingers:

Karl Rove is very good at politics, because he knows how to reduce politics down to a few words, and then he hammers them. It works.

However, Rove is not very good at governing, because you cant reduce governing down to a few words like stay the course, or cut and run. It doesnt work.

Iraq is a failure. Even OReilly said it last night. He used the word lost. Its, lab, Wolverine, tom, you guys know it too. Yes, we shouldnt have gone in. But, once we were there, we could have won. We absolutely could have.

Now, all those terrible things that you said would happen if we leave Iraq, are gonna happen because we are gonna leave Iraq.

Did the failure happen because of the media? Or did it happen because the war wasnt prosecuted properly? Who is to blame - the NY Times, or Bush?

excon

tomder55 answered on 10/24/06:

As opposed to his opponents simple word slogans like "War for oil", "Bush lied" .

I got one more that I suggest they drop;"Religion of Peace"....maybe it can be changed to "snakes on a plane ".

Stay the course never meant using the same strategy over and over, never changing.Over the weekend the New York Slimes reported that the Bush administration is considering a timetable for the mission in Iraq. The insinuation was that said timetable represented a reversal of previous Bush administration policy which strongly resisted any sort of specific timetable for Iraq.

So the Bush administration responds to that insinuation by pointing out that a change in tactics doesnt mean that were deviating from the overall goal in Iraq, which is to stay there until the mission is complete.

First they complains about no plan, but then when Bush tells them over and over again what his plan is they accuse him of being unweilding, not willing to adapt strategy. Then when he says that the strategy is being tweaked they say HEY! I thought you said you were staying the course!

I guess it is Rove's and Bush's fault for keeping it simple so it was easier to explain .Apparently that concept was too complex to understand .
So as Tony Snow said, the MSM keeps miscategorizing it so they had to learn to stop using it.

Maybe I'll be the last man standing by the policy but I do not think it a failure yet;not by a long shot(I must have missed this on ABC World News Tonight).The coalition passed two southern provinces Muthanna and Dhi Qar over to Iraqi control and are planning to hand over the other 16 eventually . It is reasonable to have a dialogue with the Iraqi Gvt. to develop a schedule of the take-over .These discussions have been on going since the government of Nouri al-Maliki took office,but suddenly it is being catagorized as a time-table . What bunk ;in no case is a specific deadline or day for each milestone! This is the same course that Bush had initially set out; where the U.S. forces would be reduced when Iraq was ready to defend themselves (as they stand up we'll stand down ).




I still say the exit strategy out of Iraq is a road to Tehran and Damascus. Looks like the Bush Adm. is beginning to finally state the obvious regarding these neighbors.

I still look at the war as a theater in a wider war rather than an isolated war .

excon rated this answer Excellent or Above Average Answer
Itsdb rated this answer Excellent or Above Average Answer

Question/Answer
excon asked on 10/22/06 - Cut & Run


Hello Wrongwingers:

I finally learned what stay the course means. It means cut and run. I didn't know that. Did you?

excon

tomder55 answered on 10/23/06:

I assure you that is not the plan or a plan. When and iff the moonbat get control of the US government they will also realize that is not an option.

However there is an apparent growing school of thought that says we could use the threat of withdrawal as leverage against the various factions in Iraq. The caveat being that the leverage is gone the moment you announce a timetable .

We have the right strategy

1)birthing through three elections already ;and probably more to come ;an autonomous representative government

2) training an army subject to that civil government

3)pledging support until it can protect its own constitutional government.

Yes that would mean gradual reduction of troops and less of a coalition footprint in Iraqi affairs ;if that is 'cut and run' then so be it .

excon rated this answer Excellent or Above Average Answer

Question/Answer
paraclete asked on 10/23/06 - How much is too much?

Howard's Pacific demands

The Prime Minister, John Howard, is in the South Pacific talking tough: if the region's impoverished countries want Australian aid their governments must govern better and stamp out corruption.

The main targets of Mr Howard's barbs are the Solomon Islands and Papua New Guinea both are at odds with Canberra over the Julian Moti affair.

The Solomon Islands Prime Minister, Manasseh Sogavare, is angry that his country's police commissioner, Shane Castles, (who was loaned by the Australian Federal Police) ordered a police raid on his office on Friday.

The raid was meant to find evidence that could link Mr Sogavare to Mr Moti's flight to Honiara on a PNG Defence Force aircraft this month.
By flying out of PNG secretly Mr Moti escaped proceedings in Port Moresby that could have seen him extradited to Australia to face child sex allegations.

Mr Sogavare who has already expelled Australia's high commissioner has labelled the raid as provocative. And there are rumblings that the Honiara governnment might pull the plug on the Australian-led Regional Assistance Mission in Solomon Islands.

Critics say that while Australia's aid is welcomed, it should show more respect for Solomons' sovereignty.

Others, though, fear the island nation will again descend into chaos if RAMSI goes.

Another scenario suggests that if Australian aid is pulled from parts of the region, countries such as China and Taiwan will step in with fast bucks to buy diplomatic support for their rival causes. But do they have genuine regard for the long-term development of the problem-ridden islands?

Is Mr Howard right to lay down strict conditions on Australian aid? Or, as some Pacific leaders suggest, is he being too heavy-handed?

tomder55 answered on 10/23/06:

I'm sure that Australia being the preeminent player in the region is contributing the lion's share to RAMSI .

A quick look at it shows that RAMSI is doing some positive things according to it's web site :

Law and order has been restored, and guns are no longer used by criminals.
Government revenue has almost tripled, from SI$258 million in 2002, to SI$688 million in 2006.
Seven recruit classes of new police officers and 70 new prison officers have been recruited and trained.
Over one thousand public servants have received training in the re-established Institute of Public Administration and Management.
The economy has been recovering strongly. New sectors are starting to open up or previous sectors are being re-established. For instance, tourist boats are visiting many parts of the country, including the Shortlands, Gizo, the Marovo Lagoon, the Florida Islands, Santa Ana, Lata, Utupua and Tikopia.


RAMSI is key to keeping the Soloman Is. from desending into lawless chaos . The Aussie contriblution is critical . I think with that leverage it is right that Howard insist on conditions .

As for China ;the whole region is in danger of falling into their sphere .

paraclete rated this answer Excellent or Above Average Answer

Question/Answer
captainoutrageous asked on 10/22/06 - Schroeder: Bush's faith raised suspicion

By MELISSA EDDY, Associated Press Writer
Sat Oct 21, 7:06 PM ET

BERLIN - Ex-Chancellor Gerhard Schroeder, whose second term was marked by vehement opposition to the war in Iraq, described in an advance copy of his memoirs how he was suspicious of President Bush's constant references to his Christian faith.

In an excerpt of his book, "Decisions: My Life in Politics" published in the German weekly Der Spiegel Saturday, Schroeder discusses the key political choices that marked his seven-year term in office, including the decision to call early elections and his split with Bush over the Iraq war.

"I am anything but anti-American," Schroeder told Spiegel in an interview to accompany the excerpt of the more than 500-page book that goes on sale Thursday.

In it Schroeder, who led the Social Democrats to power in 1998, recalls the tears in his eyes as he watched television footage of people jumping from the burning World Trade Center on Sept. 11.

He knew Germany would have to react.

"It was important to me that Germany fulfill its requirements as an ally" of the U.S., he wrote. "It was also fully clear to me that this could also mean the German army's participation in an American military mission."

Several months later, during Bush's 2002 visit to Berlin, Schroeder wrote he was surprised at what he described as Bush's "exceptionally mild" speech to the German parliament.

While meetings with Bush at that time were friendly, Schroeder said he could not reconcile himself with the feeling that religion was the driving force behind many of Bush's political decisions.

"What bothered me, and in a certain way made me suspicious despite the relaxed atmosphere, was again and again in our discussions how much this president described himself as 'God-fearing,'" Schroeder wrote, adding he is a firm believer in the separation of church and state.

Schroeder accused some elements in U.S. as being hypocritical when it comes to secularism in government.

"We rightly criticize that in most Islamic states, the role of religion for society and the character of the rule of law are not clearly separated," Schroeder wrote. "But we fail to recognize that in the USA, the Christian fundamentalists and their interpretation of the Bible have similar tendencies."

tomder55 answered on 10/23/06:

He may very well have been uncomfortable with Bush and his references to faith but that is not the reason that Germany did not support the Iraq war. It had much more to do with Germany's trade relations with Saddam that amounted in the area of $350 million annually, and another $1 billion is reportedly sold through third parties even though there were UN imposed sanctions .($1.35 billion in annual trade with Iraq, directly or indirectly.)

Saddam Hussein had ordered Iraqi domestic businesses to show preference to German companies as a reward for Germany's "firm positive stand in rejecting the launching of a military attack against Iraq."In 2002, DaimlerChrysler as an example was awarded over $13 million in contracts for German trucks and spare parts.

Furthermre Germany was owed billions by Iraq in foreign debt and was interested in keeping Saddam in power for no other reason than to insure Iraq makes good on the debt.

captainoutrageous rated this answer Excellent or Above Average Answer
Itsdb rated this answer Excellent or Above Average Answer

Question/Answer
paraclete asked on 10/22/06 - what are they afraid of?

well so much for an FTA, the Americans are so afraid of Australian inguenity that they have banned an Aistralian food

There's no accounting for taste

October 21, 2006 12:00am
Article from: The Sunday Times

THE US has banned Vegemite, even to the point of searching Australians for jars of the spread when they enter the country.

The bizarre crackdown was prompted because Vegemite has been deemed illegal under US food laws.

The great Aussie icon - faithfully carried around the world by travellers from downunder - contains folate, which under a technicality, America allows to be added only to breads and cereals.

Australian expatriates in the US said enforcement of the ban had been gradually stepped up and was now ruining lifelong traditions of Vegemite on toast for breakfast.

Kraft spokeswoman Joanna Scott said: "The (US) Food and Drug Administration doesn't allow the import of Vegemite simply because the recipe does have the addition of folic acid.''

The US was "a minor market'' for Vegemite, she said.

tomder55 answered on 10/22/06:

Went to both the FDA and US Customs sites and found no restrictions regarding vegemite.

I can tell you from working in the supplement industry that our FDA is hostile to most of the market. They appear to often be agents of the drug industry where finding cures is the premium rather than prevention . Vegemite can contain up to 600% the vitamin content of most other allowable foods .


The article and simular ones I've read say it is because of folates which can only be an additives in breads and cereals here (where it is a REQUIRED additive because so many Americans consumed foods with insufficient folate levels ) .But that is a cop out . Folate is not an additive in vegemite but is naturally occuring .

A deficiency of folate, vitamin B12 or vitamin B6 may increase blood levels of homocysteine, and folate supplementation has been shown to decrease homocysteine levels .Evidence supports a role for supplementing folic acid for lowering homocysteine levels.Some evidence also associates low blood levels of folate with a greater risk of cancer .

Of course there is a risk in consuming too much folates because of it's interaction with vit. B12 and can contribute to anemia. However ,that is in megadoses that probably exceed the levels found in vegemite. Generally it is very hard to OD on B vitamins because they are water soluable .Your body flushes any excess rahter than storing them like it does with oil based vitamins like D and E .

I can find no justification for this except to tell you that overall there has been a tightening of imports since 9-11 . I have experienced delays in delivry of materials due to them being tied up in customs and FDA inspections . But as I said. I found nothing published that indicates there are restrictions of vegemite import.

ETWolverine rated this answer Excellent or Above Average Answer
Itsdb rated this answer Excellent or Above Average Answer
paraclete rated this answer Excellent or Above Average Answer

Question/Answer
paraclete asked on 10/20/06 - Climate change good news for camels?

All right for some: the deserts that have taken our rain


As the sun emerges after a day of rain, Michael McIlvenny sends a weather balloon up from the Giles meteorological station on the edge of the Gibson Desert.


James Woodford
October 21, 2006

IF THE rain is not falling in Sydney's catchments and throughout southern Australia, where has it gone?

The answer, says the acting head of the Bureau of Meteorology's National Climate Centre, David Jones, is north-west and Central Australia, where residents are finding climate change may have a wetter flipside.

Most dramatic is the desert outpost of Giles, which sits on the edge of the Tanami Desert near the junction of South Australia, Western Australian and the Northern Territory. In 50 years the remote weather station, home to five people, has seen its rainfall double - from a yearly average of about 150 millimetres to around 300.

If current trends continue, ecological changes will begin to follow - greener for the desert and the Kimberley, but browner for southern Australia.

Because Giles is one of the driest spots in the continent, a doubling of rainfall has not yet had a visible impact, says the officer in charge of the weather station there, Michael McIlvenny.

But Dr Peter Kendrick, Pilbara-based regional ecologist with the West Australian Department of Environment and Conservation, said doubling rainfall has a "huge impact" in such an ecosystem, given that desert fauna and flora are tuned to respond rapidly to episodic rainfall.

Dr Jones says he already believes the extra rainfall in some other less arid areas has given agriculture and grazing a valuable buffer against degradation.

But in southern Australia, the colour of climate change seems to be brown.

Dr Michael Raupach, a scientist with the CSIRO Division of Marine and Atmospheric Research, and chairman of the Global Carbon Project, has recently made some frightening observations from satellite photography.

He and his team have discovered large swathes of the continent are becoming visibly less green. "Depending on the area, we are finding parts of the continent that are more than 50 per cent less green," he said. "This means a browning of the continent. The trend started in the late 1990s and since then has been going on in a ratchet fashion, with jumps in browning occurring in drought years."

What makes this finding so alarming is that if the drought does not ease then the logical conclusion of the current trend is a massive death of vegetation, huge bushfires and the release of vast volumes of carbon, further feeding climate change.

Normally, Dr Raupach said, forests have the chance to recover through flooding rains between droughts, but the low-rainfall conditions of the past decade have been relentless.

While sporadic recovery of greenness occurred in places, nowhere has vegetation climbed back to what it normally would be between droughts. Worst affected seem to be south-west Western Australia and almost the entire Murray-Darling Basin, ecosystems, already fragile because of land degradation.

"It's almost literally true that it keeps me awake at night," Dr Raupach said.

The weather bureau's Dr Jones says "superficially, the rainfall shift to the north-west of the continent doesn't make a lot of sense". This is because theoretically the entire nation has been in the grip of El Nino for much of the last decade.

Perhaps the huge release of aerosols into the atmosphere by Asian nations could be a factor in the increased rain, he said.

One thing that is certain is that the Australian climate has shifted dramatically in the past half century. In a vast band of the continent between the Nullarbor coast and the Kimberley there has been an average annual increase in rainfall of between 100 and 200 millimetres.

"Around Broome and Wyndham, rainfall has increased by 300 millimetres particularly in summer and autumn," Dr Jones said.

On the other hand, Sydney's annual rainfall has decreased by between 100 and 200 millimetres a year and in Mackay by as much as 300 millimetres a year compared to the 1950s.

Weather systems known as north-west cloud bands used to travel across the continent from monsoonal troughs in the Kimberley, bringing the kind of rain to southern Australia which filled dams and caused floods.

"In the last few years to a decade these north-west cloud bands have almost disappeared. The linkage to the tropics has broken down," Dr Jones said.

"Since 1950, since global temperatures have increased along with aerosols and ozone, all of a sudden we have seen rainfall trends that are very distinct.

"One would be naive to put these trends down to natural variations. They're very large and a number are consistent with what we see from climate change computer models."

The drying of southern Australia has attracted the most attention until now, he said.

"What we are seeing in the rest of Australia is just as dramatic, it's just that it's positive. People don't seem to notice climate change when it's beneficial to humans."

Dr Kendrick said with greater rainfall, vegetation would increase in arid areas. There would be changes in fauna. The desert mouse had extended its range from the central deserts to the west Pilbara, and camel numbers were increasing.

tomder55 answered on 10/21/06:

I was under the impression that Aussie was one of the most urbanized nations on the planet and that the US image of the rugged outback nation is largely a myth. So ;if most of Australia lives in the cities then the largest concern in my view would be securing pottable water there.

As the Israelis and the US has proven ,water can be transported to make a desert bloom . Much of the American agriculture is done in former arid regions.

Humans do need to adapt because climate change does occure . The Sahara was once lush but it turned to sand long before humans started carbon-based fuel emissions .

paraclete rated this answer Excellent or Above Average Answer

Question/Answer
Itsdb asked on 10/20/06 - If you're in New York...

...why bother with Iraq for Sale when you've got the theatre? I heard about the Bush Bashing Festival now playing:

Bush Wars

    Told through 16 musical parodies and dozens of costume changes, 'Bush Wars' sinks its talons into everything from Dick Cheney literally in bed with the oil companies to the Supreme Court's "right hand turn" and from "Republican training school" to George W. in a soft shoe number with his bosom buddy, Jesus! The new material includes a hilarious take on the immigration guest workers' program, wire tapping, the re-writing of the Constitution and voter fraud! No one is spared during the ninety minutes of non-stop music and comedy. The musical takes a funny but insightful look at how America's current government has undermined the Geneva Convention, science (evolution and the environment), Social Security, personal security, religious freedom, personal freedom, pretzels, government secrets, the courts, the brain dead and yes, Democracy itself! TheaterSource


Bush is Bad: Impeachment Edition

    'Bush is Bad: Impeachment Edition' is a new edition of the hilarious, cathartic and unabashedly partisan musical revue by Joshua Rosenblum. 'Bush is Bad,' described as a cross between 'Forbidden Broadway' and 'The Daily Show,' offers catchy tunes, wickedly funny lyrics and scathing impersonations of the president and his dissembling gang of conspirators. TheaterSource


And the 5 star, Dumbya's Rapture

    A political satire that dramatizes the real and true history of the U.S. from 2001 to 2006, 'Dumbya's Rapture' tells the uproarious tale of one president, two wars and thousands of dead around the world. TheaterSource


Can't those libs make up their minds? One day thousands of dead around the world is a Bush war crime, the next it's "uproarious."

Steve
P.S. I think I'll stick to football this weekend.

tomder55 answered on 10/21/06:

this week is as close to peak leaf change season as wewill get and it looks like today will be relatively clear. I plan on taking my family apple picking and perhaps an afternoon hike.It is a good weekend for having the Giants playing on Monday night since I doubt I will be indoors much.

Yes NY inspite of it being too politically blue is a nice place to live. Hopefully one of these days we can get conservative leadership so I can continue to afford living here .Yesterday day Evita debated against what's-his-name....oh yeah ,John Spencer ,the Republican candidate for Senate .He said he was the only one on the podium who actually wanted to be a NY Senator. She admitted that she is considering running for President (duhhh ya think?)

NY theater has been pretty lame .There haven't been any new Andrew Loyd Webber plays in a while and Disney has maxed out in converting their carttons into theater productions . So Broadway is reduced to revivals ,or Vampire musicals that flop. Off Broadway has always been a counter-culture so it is not suprising that in the East Village ,where tin foil lines the roof tops ,that a Bash Bush festival would prosper .

Itsdb rated this answer Excellent or Above Average Answer

Question/Answer
MarySusan asked on 10/20/06 - INSIDER RUMORS ABOUT IRAQ

Insider information from an army general is that Bush is getting ready to overthrow the elected government of Iraq and put in a new government.


Comments...

tomder55 answered on 10/21/06:

where did you get this rubbish ? Alternet ,The Peoples Voice.org or HuffPo ?Time to dust off that tin-foil hat .

Even the Washington Compost said the rumor of a US backed coup were erroneous .

U.S. officials have received reports that a prominent Sunni politician, Saleh al-Mutlak, visited Arab capitals over the summer and promoted the idea of a national salvation government, suggesting, erroneously, that it would have American support.

I have no doubt that the new gvt, is under threat of a counter-revolution by elements of the former Baathist regime. I think that once Saddam is permanently disposed of that threat will diminish . But yes ;it is the nature of M.E. politics that political disputes are often settled by coup. That is why our commitment to them cannot waver .

excon rated this answer Excellent or Above Average Answer
Itsdb rated this answer Excellent or Above Average Answer
JesseJamesDupree rated this answer Excellent or Above Average Answer
MarySusan rated this answer Average Answer

Question/Answer
MarySusan asked on 10/20/06 - SHIITE MILITIA SIEZES AMARAH, WITHDRAWS

Shiite Militia Briefly Seizes Amarah
By CHRISTOPHER BODEEN

"BAGHDAD, Iraq (AP) - The Shiite militia run by the anti-American cleric Muqtada al-Sadr briefly seized control of the southern Iraqi city of Amarah on Friday in one of the boldest acts of defiance yet by the country's powerful, unofficial armies.

The militiamen later withdrew from the streets after Iraqi soldiers and mediators arrived, lifting their siege of police headquarters under a temporary truce negotiated with an al-Sadr envoy. It was not clear whether the cleric knew about his militia's planned takeover in advance.

British military spokesman Maj. Charlie Burbridge said 600 Iraqi army soldiers had retaken control of the city, but not before the 25 gunmen and police were killed in violence that began Thursday night. The Iraqi army dispatched two companies to Amarah, a city of 750,000, from Basra, the south's largest city.

``They've applied a solution and at the moment it's holding,'' Burbridge said. ``At the moment, it's tense but calm,'' he said.

Britain had 500 soldiers on standby if called for, Burbridge said, saying British military authorities were ``confident that they've (Iraqi security forces) responded as best as they can.''

Mahdi Army fighters had stormed three main police stations Friday morning, residents said, planting explosives that flattened the buildings in Amarah, a city just 30 miles from the Iranian border that was under British command until August, when it was returned to Iraqi government control.

About 800 black-clad militiamen with Kalashnikov rifles and rocket-propelled grenades were patrolling in commandeered police vehicles, witnesses said. Other fighters set up roadblocks on routes into the city and sound trucks circulated telling residents to stay indoors.

AP Television News footage showed thick, black smoke billowing from behind the barricades of a police station in Amarah. Much of the smoke came from fires set to vehicles that were parked within the compound. Hooded gunmen roamed the streets, some of whom seemed to be directing the others, while a stream of gunshots could be heard in the background.

Some streets were entirely deserted except for the gunmen, but on others children ran around, pointing out the source of gunfire, and a couple of bicyclists stopped to look at the smoke that enveloped the police station.

The events in Amarah - involving a dispute between the Mahdi Army and local security forces believed controlled by the rival Badr Brigade militia - highlight the threat of wider violence between rival Shiite factions, who have entrenched themselves among the majority Shiite population and are blamed for killings of rival Sunnis.

Shiite militia violence, mainly against the country's Sunni minority, has ravaged Iraq since February when a Shiite holy place in Samara was blown up. The violence has been on the increase, but this is the first recent fighting that has pitted Shiites against one another on such a scale.

Prime Minister Nouri Al-Maliki, a former Shiite activist, won the top government post last spring thanks in part to the support of al-Sadr, who controls 30 of the 275 seats in the national parliament and five Cabinet posts.

In a sign of al-Sadr's influence, al-Maliki this week ordered the release of one of the young cleric's top lieutenants, Sheik Mazen al-Sa'edi, who was arrested by U.S. troops in Baghdad for alleged links to sectarian death squads.

The fighting in Amarah came just days after al-Maliki met with al-Sadr at the cleric's headquarters in the holy city of Najaf to enlist support for reining in sectarian violence and building political stability.

The timing seemed to indicate al-Sadr and other Mahdi Army commanders did not have full control over individual units, lending weight to the theory that many such militia groups were acting on their own and carrying out local agendas.

Mahdi Army militiamen have long enjoyed a free rein in Amarah, the provincial capital of the southern province of Maysan. Militiamen in Amarah often summon local government officials for meetings at their offices. They roam the city with their weapons, manipulate the local police and set up checkpoints at will.

Since British troops left Amarah in August, residents say the militia has been involved in a series of killings, including slayings of merchants suspected of selling alcohol and women alleged to have engaged in behavior deemed immoral by militiamen.

Fighting broke out Thursday after Qassim al-Tamimi, the provincial head of police intelligence and a leading member of the rival Shiite Badr Brigade militia, was killed by a roadside bomb. In retaliation, his family kidnapped the teenage brother of the Mahdi Army commander in Amarah, Sheik Fadel al-Bahadli, to demand the hand-over of al-Tamimi's killers.

Amarah, a major population center in the resource-rich yet impoverished south, is a traditional center of Shiite defiance to successive Iraqi regimes. Its famed marshlands were drained by former dictator Saddam Hussein during the 1990s in reprisal for the city's role in the Shiite uprising that blazed through the region after the 1991 Gulf War.

The showdown between the Mahdi and Badr militias has the potential to develop into an all-out conflict between the heavily armed groups and their political sponsors, both with large blocs in parliament and backers of al-Maliki's ruling coalition. It also could shatter the unity of Iraq's majority Shiites at a time when an enduring Sunni insurgency shows no signs of abating.

The U.N. refugee agency said at least 914,000 Iraqis have fled their homes since the U.S.-led invasion in 2003, more than a third in response to the sectarian bloodshed this year.

Associated Press writer Hamza Hendawi contributed to this report.


10/20/06 13:50 Copyright The Associated Press.

~~~~~~~~~~~~~~~~~~~~~~~~~~~~~~~~~~~~~~~~~~~~~`

It is reported today that Bush is considering making changed to his conduct(non-conduct) of the war.

Remember, he said it woul be up to his successor to end the war.

Remember, it costs 8 Billion A MONTH for losing this war of adventurism by the Bush Crime Family.

Remember, American soldiers and individuals including innocent Iraqi women and children are dying every day for NOTHING, FOR FAILURE.

tomder55 answered on 10/21/06:

well yeah that is the news that the MSM and all the cut and runners would tend to key on . (although I've never really understood why rival muslim militias killing each other was bad news ....as far as I'm concerned the Mehdi Army and the Badr Brigades can fight to the last man standing ).It doesn't suprise me that they are reporting this 'Ramadan Offensive' with the same glee that they reported the Tet Offensive.

What you have here is a militia temporarily seizing a small town . Please tell me which town has any of the insurgency been able the take and hold?

Muqtada al-Sadr circulated a written statement Oct. 13 ordering his Mehdi army not to participate in sectarian violence in Iraq.So either that was a big lie ,or he does not have the control of his militia that every one thinks he has.

But I think the bigger news that is being under reported is the meeting of religious leaders from both Shia and Sunni sects taking place in Mecca in the Al-Safa Palace over looking "the Grand Mosque in Makkah" (no I didn't say Macacca).

The meeting is aimed at adopting a formal reconciliation document that calls for ending sectarian violence in Iraq. Iraqi Shiite and Sunni religious leaders have been in Makkah for most of the week trying to find a way to halt sectarian violence .A formal endorsement of the 'Makkah Document',drafted at a preparatory meeting of the Iraqi Sunni and Shiite leaders held in Jeddah last week ,is expected during the meeting.The Makkah document is calling for adopting some practical measures to end sectarian violence, and these included formation of committees in various Iraqi regions and organizing seminars of Sunni and Shiite religious leaders and scholars.


The document has received full approval and endorsement from Iraqs key leadership, most notably from Shiite leader Ali Al-Sistani, Iraqi Prime Minister Nuri Al-Maliki and even that scumbag Moqtada al-Sadr.

What ....you didn't hear about this ? well...I'll be.....
~~~~~~~~~~~~~~~~~~~~~~~~~~~~~~~~~~~~~~~~~

It is reported today that Bush is considering making changed to his conduct(non-conduct) of the war.

That is correct 'Operation Together Forward' that was supposed to stem the spiraling violence in Baghdad , has failed to achieve its objectives.'Together Forward' had a chance of success if Iraqi leaders who exercise influence over Shiite death squads and militias, had done their part to rein in the violence. Instead, politicians loyal to al-Sadr failed to support the operation . The biggest mistake made in Iraq was not destroying his milita and letting him live when he was cornered by our troops .

I think the statement by Bush and earlier by U.S. Army spokesman Maj. Gen. William Caldwell ;as well as the British non-action at Amarah sends a clear signal to Iraqi leaders that the coalition is finished fighting its battles. Bush has said 'when they stand up we will stand down ,and that I believe is the next phase of the war.

Just one more consideration . It is my well informed belief that the increased activity of Shia "militias" is the direct result of Iranian influence.With critical U.S. congressional elections approaching, Tehran is well aware of its ability to influence the election results by worsening the position of U.S. forces through its Shiite militant proxies in Iraq. Iran knows the United States is in the middle of revising its strategy in Iraq and has been encouraged by comments made by James Baker. Baker has strongly advocated engaging Iran in direct negotiations over Iraq;lwhich stinks of appeasment .I think a suitable exit strategy would be to redeploy to Tehran.

The more our politicians jump on this Baker bandwagon ,the weaker our position becomes. Too often we have signaled to the Iraqi gvt. through statements by Baker,our fifth column fourth estate ,and our "loyal" opposition party leadership that we will abandon our commitment to them . This brings up vivid memories to them of the decision by GHW Bush to incite a revolt of the Shia against Saddam that he did not back up. Now we have one of the key members of that administration suggesting we do the same. Baker needs to clarify that what he means by negotiations with the Mahdi-hatter does not include stroking him but of a strong armed variety .


ETWolverine rated this answer Excellent or Above Average Answer
JesseJamesDupree rated this answer Excellent or Above Average Answer
MarySusan rated this answer Poor or Incomplete Answer

Question/Answer
ETWolverine asked on 10/20/06 - IRAQ: UNITING AGAINST THE JIHADIS

By AMIR TAHERI

October 20, 2006 -- TALK to Iraqis these days, and you'll likely hear one thing: What are the Americans and Brits up to? The worry is that the U.S. and U.K. political mainstreams now regard the Iraq project as a disaster, with cut-and-run, or whistle-and-walk-away, the only options.

Most Iraqis regard the toppling of Saddam Hussein, the dismantling of his machinery of war and oppression and the introduction of pluralist politics to Iraq as an historic success. The issue is how to consolidate that victory, not to snatch defeat from its jaw. Those challenging this historic victory are enemies of both the Western democracies and the Iraqi people.

Iraq today is the central battlefield in the global war between two mutually exclusive visions of the future. Yet the jihadists now know they can't win on that battlefield. After three years of near-daily killings, often in the most horrible manner imaginable, they've failed to alter Iraq's political agenda. Nor have they won control of any territory or even broadened their constituency.

The jihadists have suffered thousands of casualties, with many more captured by Coalition forces and the new Iraqi army and police. Despite more than 120 suicide operations, and countless attacks on civilian targets, the jihadists have been on the defensive since they lost their chief base at Fallujah last year. Their strategic weakness: They can't translate their killings into political gains inside Iraq.

They kill teachers and children, but schools stay open. They kill doctors and patients, but hospitals still function. They kill civil servants, but the ministries are crawling back into operation. They kidnap and murder foreign businessmen, but more keep coming. They massacre volunteers for the new army and police, but the lines of those wishing to join grow longer.

They blow up pipelines and kill oil workers, but oil still flows. They kill judges and lawyers, but Iraq's new courts keep on working. They machine-gun buses carrying foreign pilgrims, but the pilgrims come back in growing numbers. They kill newspaper boys, but newspapers still get delivered every day.


Since liberation, an estimated 45,000 Iraqis have been killed, largely by insurgents and terrorists. Yet there are few signs that a majority of Iraqis are prepared to raise the white flag of surrender.

Recent events highlight the growing isolation of the jihadists and their Saddamite allies:

* A tribal alliance has joined together all Arab Sunni clans of western Iraq in a united front to "chase al Qaeda out of Iraq."

This was partly a response to a split inside al Qaeda's Iraq branch. Members of the terror group recently published an appeal to Osama bin Laden to dismiss Abu-Ayyub al-Masri, the group's " commander" in Iraq. Why? Al-Masri, an Egyptian terrorist, has tried to push the group's violence to new depths of perfidy by planting mines in primary schools and hospitals and organizing rackets against shopkeepers, both Shiite and Sunni. The statement calling for his dismissal calls him a "deviant" - a label that indicates the willingness of some al Qaeda members to liquidate him if he's not replaced.

* Iraq's National Assembly gave near-unanimous approval to a new plan for peace and reconciliation. Backed by all ethnic and religious communities through their political parties, the plan furthers the marginalization of the jihadists and Saddamites.

Under the plan, the different ethnic and religious groups would come to one another's help whenever needed in the battle against the insurgents. This would end a de facto situation in which Arab Sunni areas have been no-go areas for Shi'ite and Kurdish forces and vice-versa.

More, the plan envisages the creation of a unified information office to harmonize the sermons delivered at mosques, regardless of their affiliations. The idea is to use the mosque as a forum for a unified and democratic Iraq, rather than a hub of sectarian agitation.

* A third event is set to take place in Mecca next week at the end of the Muslim fasting month of Ramadan. This will bring together prominent Sunni and Shiite clerics from Iraq and eight other Muslim countries to discuss and approve a declaration demanding an end to sectarian feuds in Iraq. An initiative of the Organization of the Islamic Conference, the gathering reflects growing impatience with the jihadists throughout the Muslim world.


The proposed draft categorically states that bloodshed motivated by sectarian considerations is haram (forbidden) - and that its perpetrators are waging war on Islam as a whole. (Iraq's Grand Ayatollah Ali-Muhammad Husseini Sistani wants the gathering to go further by labeling as haram any incitement to sectarian hatred.)

The Mecca gathering represents the first major effort by Sunnis and Shiites toward mutual recognition as acceptable versions of the same faith since 1947. It is strongly supported by the Al-Azhar seminary in Cairo, the Council of Ulema in Mecca and Medina, the Shi'ite seminaries of Najaf (Iraq) and Qom (Iran) and all five associations of Iraq's Sunni clerics.

*

Despite the dramatic increase in terrorist attacks in recent weeks, new Iraq is holding its own because Iraqi morale is holding.

That morale, however, is under constant attack from two sources. The first is the part of the international (especially pan-Arab) media that depicts Iraq as a wayward train racing ahead with no light at the end of a dark tunnel.

The second threat to Iraqi morale is by far the most serious. It concerns uncertainty about the commitment of the United States and its allies to new Iraq.

Just as Rome was not built in a day, creating a pluralist democracy on the ruins of one of the nastiest of Arab tyrannies takes time. It took the United States and its allies 10 years to hand over the government of post-war Austria to Austrians. In Bosnia, the United States and its allies are now scheduled to hand over the reins of government to the Bosnians themselves - after a decade. In Iraq, the handover came just two years after liberation.

Iraqis are puzzled when they hear prominent Americans speaking of carving Iraq into three or more mini-states, as if Iraq were a blank sheet on which anyone could draw whatever he wanted.

The key to the future of Iraq lies in the United States. The Iraqis will not run away in the face of jihadism and Ba'athism. Is the same true of the Americans and their allies?

Amir Taheri is a member of Benador Associates.

-------------

I wish I had Taheri's talent for writing. He says exactly what I have been trying to say for 5 years now, but he says it much more clearly than I do.

Elliot

tomder55 answered on 10/20/06:

Thanks Amir ;that was refreshing and being Iranian he is more familiar with the region then most commentators .

I also agree with the sentiment stated by Irq PM Nouri al-Maliki who said yesterday that he hopes Saddam Hussein's trial will be short and that Hussein will be sentenced to death.

Definitely, with his execution, those betting on returning to power under the banner of Saddam and the Baath (party) will lose.

As long as he lives the Baathists "keep hope alive". Until Saddam is gone, he will create hopes of return for those who supported and profited by his leadership. Once gone they may initially try outrage but they will adjust quickly to the death of Saddam and move on. The new gvt. cannot feel free from a counter revolution until he is dead.

I read the blogs coming out of Baghdad . Omar and Mohammed have been publishing one called 'Iraq the Model ' since the war began . The wrote this recently :

There's no going back thirty years to the days of Saddam an there's no going back a thousand yeas to the days of the Caliphs.
It's over

We have accepted the rough road and the outcome will not be in the benefit of the criminals. The war is tough, painful and hard but I have no doubt of the outcome that will mean the end for the supporters of tyranny and extremism.

Surrendering is much closer to them than it is to us and history will remember with pride those who sacrificed for the freedom of Iraq
Maybe I will not live long to see that day but my children will certainly see it.

Sorry whiners, losers and pessimists. I only know to accept a challenge when I face one and I recognize only victory as an end.


When I see this is only strengthens resolve .

and here is what they said about that horribly distorted Lancet Report :

Among the things I cannot accept is exploiting the suffering of people to make gains that are not the least related to easing the suffering of those people. Im talking here about those researchers who used the transparency and open doors of the new Iraq to come and count the drops of blood we shed.

Human flesh is abundant and all they have to do is call this hospital or that office to get the count of casualties, even more they can knock on doors and ask us one by one and we would answer because weve got nothing to be ashamed of.

We believe in what were struggling for and we are proud of our sacrifices.

I wonder if that research team was willing to go to North Korea or Libya and I think they wouldnt have the guts to dare ask Saddam to let them in and investigate deaths under his regime.

No, they wouldve shit their pants the moment they set foot in Iraq and they would find themselves surrounded by the Mukhabarat men counting their breaths. However, maybe they would have the chance to receive a gift from the tyrant in exchange for painting a rosy picture about his rule.

They shamelessly made an auction of our blood, and it didnt make a difference if the blood was shed by a bomb or a bullet or a heart attack because the bigger the count the more useful it becomes to attack this or that policy in a political race and the more useful it becomes in cheerleading for murderous tyrannical regimes.

When the statistics announced by hospitals and military here, or even by the UN, did not satisfy their lust for more deaths, they resorted to mathematics to get a fake number that satisfies their sadistic urges.

When I read the report I can only feel apathy and inhumanity from those who did the count towards the victims and towards our suffering as a whole. I can tell they were so pleased when the equations their twisted minds designed led to those numbers and nothing can convince me that they did their so called research out of compassion or care.

To me their motives are clear, all they want is to prove that our struggle for freedom was the wrong thing to do. And they shamelessly use lies to do thiswhen they did not find the death they wanted to see on the ground, they faked it on paper! They disgust me

This fake research is an insult to every man, woman and child who lost their lives.
Behind every drop of blood is a noble story of sacrifice for a just cause that is struggling for living safe in freedom and prosperity.

Let those fools know that nothing will stop us from walking this road and nothing will stop our friends and allies from helping us reach safe shores. Theres simply no going back even if it cost us more and their fake statistics will not frighten usour sacrifices, like I said, make us proud because our bloods are not digits in those ugly papers. Our sacrifices are paving the way for future generations to live the better life we couldnt live.


like you said .I wish I had the talent to express myself in this manner .




ETWolverine rated this answer Excellent or Above Average Answer
Itsdb rated this answer Excellent or Above Average Answer

Question/Answer
excon asked on 10/19/06 - The Darkest of day's are upon us.


Hello rightless people:

To continue regarding our discussion about your rights that were stolen from you like, a thief in the night. Unbelievably, Im the only one who noticed.

You say that the recent tribunal law only applies to aliens, that it DOESNT apply to Americans. To that, I say, BUNK!

As it stands now, the president or his secretary of defense or their designates, can declare YOU to be an enemy combatant, and wisk you off to some CIA prison in Budapest.

Yes, you are an American held illegally. But, without the right to challenge your imprisonment (habeas corpus), who are you gonna complain to?

As it stands NOW, we are at the mercy of the president. I know, I know, you say we can trust him. To that, I say BUNK!

Previous to the dark day that just passed, we didnt HAVE to trust him. We didnt remain free because hes a benevolent guy. We remained free because we HAD rights. Now we dont!

Tell me Im wrong. But I am NOT.

excon

tomder55 answered on 10/20/06:

The MCA, in section 7, states the following: No court, justice, or judge shall have jurisdiction to hear or consider an application for a writ of habeas corpus filed by or on behalf of an alien detained by the United States who has been determined by the United States to have been properly detained as an enemy combatant or is awaiting such determination.
Clearly this does not do anything weaken the constitutional guarantee of habeas corpus for U.S. citizens, who are the only people afforded the protections of the constitution. It says nothing about denying habeas corpus to citizens.



yeah ;I've heard the shrill warnings from Howard Beale ......ooops I mean Keith Olberman and some constitutional scholar he had on claiming that no one was safe . They are wrong. [Olberman has been making some decent ratings with his not so Murrow-ish "Special Comment" segments. I half expect him to start leaning out windows to shout them out .]

If you want to find fault ,then point at the Supreme Court which clearly overstepped it's Constitutional mandate when it decided the Hamdan case .The majority ignored / misread the court's World War II precedents /stare decisis in cases like 'Johnson v. Eisentrager'.

The writ of habeas corpus had never before been interpreted to benefit enemy prisoners . If WWII had lasted 20 years we would've held onto captured Germans and Japanese that long .We have released Gitmo detainees already and have had confirmed cases of some of them returning to fight /kill our soldiers again .

I understand the different nature of this war but the Constitution clearly gives the President and the Congress the jurisdiction on how to conduct it. The Constitution gives Congress the authority to determine the jurisdiction of federal courts , and also declares that habeas corpus can be suspended "in Cases of Rebellion or Invasion" when "the public Safety may require it."

What is unusual this time ,and what is making these Constitutional Scholars have hissy fits is that Congress and the President did not take the court's power grab lying down. They told the courts to get out of the war on terror, stripped them of habeas jurisdiction over alien enemy combatants, and said there was nothing wrong with the military commissions.

But here is anotherway to look at the law .....Until the law was signed the detainees at Guantanamo Bay didn't have an avenue to prove their innocence. But with President Bush's signature that has changed. The White House says that it expects these trials to begin within a month or two.

Do you know why there was no fuss and feathers over the bill while Congress debated it ? Because the MSM was so orgasmic over the fact that some "moderate Republicans " were making an issue over interrogation techniques.Too bad because the press missed something monumental . The Congress and the President have given themselves and future leaders ,no matter which party they are from ,a blueprint on how to wage this war in the future .

excon rated this answer Excellent or Above Average Answer

Question/Answer
paraclete asked on 10/19/06 - When Climate Change becomes a political plus?

Finally, the penny drops


October 20, 2006

A hot Adelaide day helped change the Government's views on climate, writes Peter Hartcher.

ALEXANDER DOWNER, like a good local member, made an appearance between the flower displays and the dog judging at the Port Elliott Show two weeks ago in his electorate near Adelaide.

"It was a bloody hot day, 33 degrees and a north wind - often the Port Elliott Show is cold and rainy," recalls Downer. "And a bunch of people, not just farmers, were saying: 'Maybe there is something in this climate change thing."'

The Foreign Affairs Minister had taken a closer interest in climate change in recent years, but that day crystallised his thinking. He phoned one of his Liberal colleagues, Greg Hunt, who has spent years advocating the need to act on global warming.

"It's time," Downer told him.

For the slight and scholarly Hunt, it was a sweet moment. "He'd been working on me for a long time," confesses Downer. Only a few weeks earlier the pair had thrashed their way through a robust argument on the subject.

Hunt now believes that the conversion of the Howard Government is, effectively, complete and that Downer has been pivotal.

The Government, disdainful of the Kyoto Protocol on greenhouse emissions and long sceptical of the case for any action to address global warming, is positioning itself seriously to address the matter.

It is playing catch-up with its constituents. Sixty-eight per cent of Australians believe global warming - or global heating, as the Greens call it - is a "critical threat" to Australia's vital interests over the next 10 years, ahead of Islamic fundamentalism, according to the Lowy Institute poll published this month. Many major companies around the world are far advanced in working on the problem.

The drought, already very serious, will become dire as it slowly strangles the artery of south-eastern Australia, the Murray-Darling, in the year ahead, and entire towns and industries are rendered insolvent and obsolete.

So the Government will need to act with the zeal of a convert. But a great deal of work is going on unseen within the Government. While Labor continues to beat the Coalition with the lash of Kyoto, the Government is working to blindside Labor with a new agenda in the year to the next election.

Downer likens Labor's use of Kyoto to the witch-hunt practice of dunking women in water. If a government has not ratified the treaty, it's a witch. "I don't even believe in witches," he says.

He wants Australia, which has promised to meet its Kyoto obligations even though it refuses to ratify the treaty, to exceed the ambitions of Kyoto in cutting greenhouse gases.

Hunt, the federal parliamentary secretary on the environment, believes the Government has had two Eureka moments in the past 15 months. First was the realisation that the world needed a mechanism for concerted global action that would include the biggest greenhouse polluters. The Government ruled out the Kyoto Protocol because it had been rejected by the US and because it excluded the big new belchers, China and India.

But last year the Government helped craft a new international grouping, the six Asia-Pacific nations known as the AP-6. It embraces Australia, China, India, Japan, South Korea and the US, which together account for half of all global emissions. Second was a much more recent acceptance that Australia needed to take more dramatic and urgent steps to cut its own emissions.

"I think the cabinet was converted when Downer changed his mind, and he convinced the Prime Minister with help from Ian Campbell [the Environment Minister]," says Hunt. "And the Prime Minister moved a couple of other doubters when he moved."

Only one sceptic appears to remain in the cabinet room, and that's the Industry Minister, Ian Macfarlane.

Will the Government's agenda be all about nuclear power? So far, this has been the mainstay of the Prime Minister's response.

"I believe very strongly that nuclear power is part of the response to global warming," said Howard this week. "It is clean green."

But Howard has given the task of examining the future of Australia's nuclear sector to a panel chaired by the former Telstra boss Ziggy Switkowski. The companies that own Australia's uranium industry, BHP Billiton and Rio Tinto, have made the obvious point to the inquiry that it is not economic for Australia to process uranium, nor to develop a nuclear power industry.

Yesterday Switkowski, who is expecting to report to the Government in a month, said: "Australia is blessed with a couple of things - very low-cost electricity because of access to coal and gas, and has many centuries of coal supply available. Any comparison will be unfavourable for every alternative source in the absence of an explicit cost for carbon."

In other words, nuclear power will not be economic in Australia unless the Government makes a dramatic policy change. An "explicit cost for carbon" means the Government would need to charge a carbon tax, or introduce an emissions trading system - or provide vast subsidies to nuclear power plants.

If the Government takes any of these courses to make nuclear power viable, it will be a major change for Australia. But the Government is examining a great deal more than just the nuclear option.

The head of the Co-operative Research Centre for Greenhouses Gas Technologies, Dr Peter Cook, believes that new carbon capture technologies will allow Australia to cut its greenhouse emissions in half. This and other subjects are receiving a lot of attention inside the bureaucracy. Several proposals will be going to the cabinet in the months ahead. After making the journey from atheism to agnosticism, the Government now has religion on global warming. Thank God.

Peter Hartcher is the Herald's political editor.

tomder55 answered on 10/20/06:

I take it then that the environmentalists there have embraced nuclear as a viable clean alternative ? Talk about a conversion ! Our greenies don't want nukes ;or even windmills that interfere with their views of the sea from their spatial summer mansions .

I am all in favor of carbon emission reductions ;with or without the scientific communities chicken little, cassandra clarion calls.So long as the technology is there then it makes sense to reduce questionable emissions.

Itsdb rated this answer Excellent or Above Average Answer
paraclete rated this answer Excellent or Above Average Answer

Question/Answer
excon asked on 10/19/06 - Rules vs People


Hello:

I finally figured out WHY people join one party or the other. I get, that all of us want pretty much the same thing.

However, Republicans think the problems are caused because we havent written the rules exactly right, and the Democrats think the problems arent the rules, but whether people will obey them.

Thats why people like the Wolverine who, even though he disagrees with a law, will obey it absolutely, because its written. Then there are guys like me, who question the morality of the law, and make decisions to obey or not, based on that.

Neither position works for us as a country.

I had a business partner who was designing a job for an underling. As designed, the job had no redeeming qualities, and couldnt produce any satisfaction for the worker. I said, if the job isnt satisfying, well have trouble filling it. My partner said, whats that got to do with it? Heres the job. Heres the pay. Do it. He's a Republican.

Republicans think that all they have to do is write a law, and their work is done. People WILL obey, because they wrote it. Democrats worry so much about how a law will be perceived that they dont write laws that matter.

Maybe the solution is, that we have enough laws right now, and they should take a vacation.

excon

tomder55 answered on 10/19/06:

I'm not Republican but if it means anything ;I routinely violate the speed limits (mostly out of necessity ...I would get run off the highway if I adhered to the speed limits ). makes me a real desperado I guess.

I think that laws that are not enforceable or ones that the gvt. chooses not to enforce are a waste and should be reevaluated . I think your internet gambling law is one that should be looked at because I do not believe that the motive of passing the law is public safety as much as protecting State gambling revenues.

ETWolverine rated this answer Excellent or Above Average Answer
excon rated this answer Excellent or Above Average Answer

Question/Answer
paraclete asked on 10/19/06 - A revolutionary thought, the leader is not the sole repositary of widsom?

Iraq a 'catastrophic blunder'


The Iraq war has been a disaster and has substantially increased the terrorist threat Mr Howard said it would reduce

David Braithwaite
October 19, 2006 - 1:40PM

The war in Iraq has been a "catastrophic blunder" that has substantially increased the terrorist threat to Australia, one of the nation's most distinguished former diplomats said today.

Richard Woolcott, a retired foreign affairs chief who advised seven prime ministers, launched a sweeping attack on the federal government, saying that Australian democracy was not functioning as it should.

Mr Woolcott made the comments during a speech at the University of Newcastle's annual Human Rights and Social Justice lecture this afternoon.

He branded the Iraq war a "disaster", saying the Prime Minister seemed unable to admit the obvious.

"The Iraq war has been a disaster and has substantially increased the terrorist threat Mr Howard said it would reduce," he said.

"The aim of foreign and defence policy is to make Australia secure - ironically some of our policies have placed Australians at greater risk."

Mr Woolcott called on the government to come up with an exit strategy.

"The United States, the United Kingdom and Australia, having made such a catastrophic foreign and security policy blunder, are now trapped in a dilemma of their own making," he said.

He warned a precipitious withdrawal from Iraq could cause more chaos, however, staying the course would "only continue the bloodshed, energise the terrorist and Jihadists, inlcuding in our own region..."

Mr Woolcott's criticism of the war followed recent comments from Australia's former defence chief General Peter Cosgrove that it had boosted global terrorism and Britain's top soldier Sir Richard Dannatt, who called for the recall of his troops from Iraq.

Mr Woolcott said human rights suffered in a climate of war and fear.

"In 2006 our established ideals of decency, fairness, tolerance,
justice and truth in government are under challenge," he said.

Australia's democracy was not functioning as it should, he said.

"I believe it is affected by hubris, the arrogance that comes from 10 years in power, the politics of fear, nurtured by the so-called 'war on terror' and latent racism," he said.

"The government has also suffered from a lack of the important qualities of patience and humility.

"This is impacting adversely on the wider community, including in the areas of human rights and social justice."

Mr Woolcott said his service to four Liberal and three Labor prime ministers proved the objectivity to his remarks, but from "personal experience" he expected to be attacked.

"The present government tends to treat its critics - even those who have served it in the past - as virtual enemies rather than as possibly useful channels to community opinion," he said.

Calling on Australia to look past the economy and calculated distractions like the "cultural wars", Mr Woolcott said its citizens needed to address other issues in society.

"Obscuring the truth, discrediting individuals who do not agree with particular policies..., the myth ... that the Prime Minister is the sole repository of wise judgements and sound decision making, combined with a compliant public service and a strangely apathetic and detached wider community are all factors, which threaten the health of Australian democracy," he said.

+++++++++++++++++++++++++++++++++++++++++++++++++++++

so how do we undo this catastrophic blunder of uninspired leadership?

tomder55 answered on 10/19/06:

ZZZZZZZZZZZZZZZZZZ - huh? excuse me the mono-drone of the antiwar crowd that I've heard non-stop since before the war began put me to sleep .

paraclete rated this answer Excellent or Above Average Answer

Question/Answer
paraclete asked on 10/18/06 - Just what we needed is a new weapon?

Mysterious wounds point to Israel's use of new weapon

Rory McCarthy in Gaza City
October 19, 2006

DOCTORS in Gaza have reported previously unseen injuries from Israeli weapons that cause severe burning and leave deep internal wounds, often resulting in amputations or death.

The injuries were first seen in July, when Israel attacked Gaza after Palestinian militants captured an Israeli soldier.

"Bodies arrived severely fragmented, melted and disfigured," said Jumaa Saqa'a, a doctor at the Shifa hospital in Gaza City.

"We found internal burning of organs, while externally there were minute pieces of shrapnel. When we opened many of the injured people we found dusting on their internal organs."

It is not clear whether the injuries come from a new weapon. The Israeli military denied reports that the injuries came from a Dense Inert Metal Explosive (Dime), an experimental weapon.

In Gaza, Dr Saqa'a said the small pieces of shrapnel found in patients' bodies did not show up under X-ray. "We are used to seeing shrapnel penetrate the body making localised damage. Now we didn't see shrapnel, but we found the destruction," he said.

The doctors also found that patients who were stabilised after one or two days suddenly died. "The patient dies without any apparent scientific cause," Dr Saqa'a said. Photographs of some of the dead from Shifa hospital showed bodies that had been melted and blackened beyond recognition. In several cases doctors amputated badly burnt limbs.

At the Kamal Odwan hospital in Beit Lahiya, the deputy director, Saied Jouda, said he believed patients admitted even in recent days still showed signs of unusual injuries. The health ministry in Gaza reported that these injuries came from an "unprecedented type of projectile" and also noted severe burning and badly damaged internal organs.

Tissue samples from patients in Gaza were given to journalists from the Italian television channel RAI. In a documentary shown last week, the channel said the injuries appeared similar to the effects of Dime.

An Italian laboratory that analysed the samples reportedly said its results were compatible with the hypothesis that a Dime weapon was involved.

The weapon is still in the early stages of development in the US. It has a carbon-fibre casing and contains fine tungsten particles rather than metal shrapnel. It causes a very powerful blast, but with a much more limited radius than other explosives.

The Israeli military denies the use of Dime weapons.

"The defence establishment is investing considerable effort to develop weaponry in order to minimise the risk of injury to innocent civilians. With regard to allegations of the use of Dime weaponry, the Israel Defence Forces deny the possession or use of such weapons," it said.

"In addition it should be emphasised that the IDF only uses weapons in accordance with international law." Some Israeli military experts have also dismissed the suggestion that a Dime weapon is involved.

The Guardian

tomder55 answered on 10/19/06:

Pehaps the Israeli's developed something simular to DIME on their own . They are very capable you know.

What makes this explosive different from others is that it is a very intense bomb with a very small impact, an impact of only several yards. It uses tungsten and carbon.
It is an enormously hot intensive explosion.
It is a weapon that allows you to hit your target and only your target.

So what's the big deal over this small explosive?When terrorists hide amidst civilians, armies need to hit their enemy without hitting innocent civilians don't they ? If this weapon is beyond the development stage I say get it into the hands of our troops ASAP .

ETWolverine rated this answer Excellent or Above Average Answer
paraclete rated this answer Excellent or Above Average Answer

Question/Answer
Itsdb asked on 10/18/06 - The only question?

The columnist I love to despise most, Molly Ivins, in her column today quoted an article by "conservative in good standing" Richard J. Whalen, who spoke with the 'dissenting generals'.

Whalen quotes one general as saying "The only question is whether a war serves the national interest."

1) Is that "the only question?"

2) Does the war in Iraq serve our national interest, and if so, how?

Steve
P.S. Even though I love to despise her, I do wish Molly the best in her battle against breast cancer.

tomder55 answered on 10/18/06:

Wars should serve the national interest and I have explained in the body of work I've done here why I still think the Iraq war was the right decision .

I have a grave concern with this comment in her article.
"The dissenting retired generals are bent on making Iraq this nation's last strategically failed war -- that is, one doggedly waged by civilian officials largely to avoid personal accountability for their bad decisions...

What exactly are they suggesting here ? A coup like the CIA attempted ? The ending of our long standing Constitutional control of the military by the elected civilians ? Is that what Irvins wants ?

She goes on : During Vietnam, senior soldiers kept quiet. But after it ended, officers, including Colin Powell, "vowed it would never happen again." But Defense Secretary Donald Rumsfeld and the other civilians in charge overruled the military minds and ignored the possible consequences.

The false assumption being made here is that because they are senior officers that they will make the right decisions . We know from history that military blunders are as prevalent if not more so on the battle field level as they are at the political level . Lincoln fired many of his commanders .Many of his generals despised him.Gen. George McClellan left his command after Lincoln fired him a second time to run against Lincoln in 1864 . Truman fired Gen. MacArthur .

She is also wrong that commanders in Vietnam did not speak up . General LeMay was very outspoken about the effort ;but since his crique does not mesh with her talking points (he was very hawkish)it must've slipped her mind.

Finally she looks for comparisons with Vietnam. I'll give her one from one of her kindred at the NY Slimes. Tom Friedman who wrote in a column (sorry no link ) called 'Barney and Baghdad' :

But while there may be no single hand coordinating the upsurge in violence in Iraq, enough people seem to be deliberately stoking the fires there before our election that the parallel with Tet is not inappropriate. The jihadists want to sow so much havoc that Bush supporters will be defeated in the midterms and the president will face a revolt from his own party, as well as from Democrats, if he does not begin a pullout from Iraq.

The jihadists follow our politics much more closely than people realize. A friend at the Pentagon just sent me a post by the Global Islamic Media Front carried by the jihadist Web site Ana al-Muslim on Aug. 11. It begins: The people of jihad need to carry out a media war that is parallel to the military war and exert all possible efforts to wage it successfully. This is because we can observe the effect that the media have on nations to make them either support or reject an issue.

It then explains that for jihadist videos of attacks on Americans to have the biggest impact, Some persons will be needed who are proficient in the use of computer graphics including Photoshop, 3D Studio Max, or other programs that the people of jihad will need to design video clips about the operations.

Finally, the Web site suggests that jihadists flood e-mail and video of their operations to chat rooms, television channels, and to famous U.S. authors who have public e-mail addresses such as Friedman, Chomsky, Fukuyama, Huntington and others. This is the first time Ive ever been on the same mailing list with Noam Chomsky.


To his last point I say I doubt it . But yes I think the upsurge in violence there is a coordinated effort to influence our politics. Too bad there are so many 5TH Columnists like Irvins so willing to take the bait.







Itsdb rated this answer Excellent or Above Average Answer

Question/Answer
excon asked on 10/17/06 - Enemy Combatant


Hello Fascists:

How long do you think it will take Bush to declare drug users enemy combatants? I say less than 2 years.

excon

tomder55 answered on 10/18/06:

If the drug user is an American on American soil and is not waging jihad against the US and is only breaking US drug laws then they are protected under Constitutional due process . What we have here is a new right being conferred on terrorists courtesy of the Supreme Court which again exceeded it's authority and granted rights that never existed before . I refer you to a summary of the JOHNSON v. EISENTRAGER, 339 U.S. 763 (1950)
ruling by the primarily lauded Roosevelt Supreme Court to find the precedent/ * Stare Decisis * whatever that the Court should've refered to when making their decision on the issue of unlawful enemy combatants .

Modern American law has come a long way since the time when outbreak of war made every enemy national an outlaw, subject to both public and private slaughter, cruelty and plunder. But even by the most magnanimous view, our law does not abolish inherent distinctions recognized throughout the civilized world between citizens and aliens, nor between aliens of friendly and of enemy allegiance, nor between resident enemy aliens who have submitted themselves to our laws and non-resident enemy aliens who at all times have remained with, and adhered to, enemy governments.

But, in extending constitutional protections beyond the citizenry, the Court has been at pains to point out that it was the alien's presence within its territorial jurisdiction that gave the Judiciary power to act.


If this [Fifth] Amendment invests enemy aliens in unlawful hostile action against us with immunity from military trial, it puts them in a more protected position than our own soldiers.

We hold that the Constitution does not confer a right of personal security or an immunity from military trial and punishment upon an alien enemy engaged in the hostile service of a government at war with the United States.

It is not for us to say whether these prisoners were or were not guilty of a war crime, or whether if we were to retry the case we would agree to the findings of fact or the application of the laws of war made by the Military Commission. The petition shows that these prisoners were formally accused of violating the laws of war and fully informed of particulars of these charges.


See also the decision in EX PARTE QUIRIN, 317 U.S. 1 (1942)

the law of war draws a distinction between the armed forces and the peaceful populations of belligerent nations and also between those who are lawful and unlawful combatants. Lawful combatants are subject to capture and detention as prisoners of war by opposing military forces. Unlawful combatants are likewise subject to capture and detention, but in addition they are subject to trial and punishment by military tribunals for acts which render their belligerency unlawful.

Instead the imperial court again broke new ground ;ignored the Constitution of the United States and the verbadum text of the Geneva Convention when they confered constitutional rights on unlawful enemey combatants.

The law that Bush passed yesterday is a weak law that complies with the Court's recent decisions and still the slip and fall lawyers and the self proclaimed libertarians find fault in it because they will not be satisfied until grunts on the battle field are pulling out text of Miranda rights to be recited to captured enemies .

excon rated this answer Excellent or Above Average Answer
Itsdb rated this answer Excellent or Above Average Answer

Question/Answer
Itsdb asked on 10/16/06 - It's been confirmed...

...Bush is inspired by Satan.

    According to the Iranian media, Mr Ahmadinejad said he had inspirational links to God, and went on to say that if you were a true believer, God would show you miracles.

    Then the Iranian president said Mr Bush was similar to him.

    According to Mr Ahmadinejad, the US president also receives inspiration - but it is from Satan.

    He repeated: "Satan inspires Mr Bush."


First Chavez, now Ahmedinajad - what will those poor suckers at TheocracyWatch do now that these world 'leaders' have set the record straight?

tomder55 answered on 10/17/06:

can you imaging Chavez sitting in as the UN Security Council President ? The UN will decide soon if Venezuela gets a seat there ,and the chair is a rotating Presidency . No doubt if he gets in his turn will coincide with some Iranian aggression ;or perhaps the Beijing Olympics .

what will those poor suckers at TheocracyWatch do now that these world 'leaders' have set the record straight?

No doubt they will be reading "Tempting Faith: An Inside Story of Political Seduction" by David Kuo who says the Bush Administration is not moving fast enough to create a theocracy .

Itsdb rated this answer Excellent or Above Average Answer
paraclete rated this answer Excellent or Above Average Answer

Question/Answer
ETWolverine asked on 10/17/06 - I'm BAAAAAaaaaaack!!!

Been on vacation for the past 10 days. Now I come back, and look at how the fecal matter has connected with the rotary air impellers. (Translation: "how the $h!t has hit the fan".)

1) Harry Reid, the guy who was all over the MSM when the Abramoff story broke turns out to be one of the better examples of political corruption after all. How many different "questionable" items have come to light now about his political finances? 10? 12? I've lost count.

2) Treason has become punishable by 2 1/2 years in prison. I've seen J-walkers get more time than Lynne Stewart will serve. And the wench is actually going to appeal her sentence to try to get her law-license back. So to all who say that terrorism has to be handled by the criminal-justice system rather than the military system, I give you Lynne Stewart. Screw that: just line the scumbags up against a wall, tie electrodes to their privates, and throw the switch. Then when you have drained then of all the information they have, shoot them behind the ear and bury them in unmarked graves. Because the criminal-justice system is clearly not designed to deal with terrorists.

3) It turns out that Bush was right again... when he named the Axis of Evil, he was right on the money. The biggest threats to Democracy in 2002 were indeed Iraq, Iran and North Korea.

For those who argue that Bush invaded the "wrong country at the wrong time", and that by eliminating the Hussein regime we "empowered" Iran, I say BS. If we had instead invaded Iran and eliminated the Iranian Mullocracy, those same people would be arguing that Saddam was the real threat, and we had empowered Iraq. And if we had instead invaded North Korea, people would have argued that we were ignoring the "real" threats of Iran and Iraq and "empowering" South Korea. Well guess what: no matter what we had done, a power vacuum would have developed, and SOMEONE would have stepped up to fill that vacuum just as Ahmadinejad has done in Iran. That is the nature of power... whoever is able to do so grabs as much as he is able, and Ahmadinejad has just been the most vocal grabber to date in the Middle East, and Kim Jong Il has been the power-grabbing guy in Asia. It could just as easily have been any other leader. And so the argument that "Bush's ineptness and lack of forward-thinking destabilized the region" is pure BS. The region is perpetually unstable, power-grabs are the norm, and the only way to deal with them is to have the stronger military and the cojones to use it. Ahmadinejad and Jong Il are betting that the American people don't have the cojones. And if the Dems win Congress, they'll be right.

4) The Dow Jones Industrial Average hit several new highs, the unemployment rate hit new lows, and the economy is chugging along quite comfortably. The media has completely ignored this. They have played up the national debt, but they've ignored every other economic indicator in existence. Again. Or is that "still"?

5) The MSM has played up the Foley-Page scandal, but has essentially ignored the Harry Reid scandals (plural). They have called for everyone and their best friend who had any connection to Foley in the past 20 years to be strung up by their testicles. This is the same MSM that has repeatedly defended Clinton fo having sex with an underage intern in the Oval Office.

This is also the same MSM that has largly ignored the fact that former Rep. Gerry Studds (D-MA) who died this weekend was a child molestor who actually had sex with several Pages while in office (as opposed to just e-mailing and IMing them about sex). They lionized him as a gay-rights activist who was Congress' first openly gay member, but ignored the fact that he lured underage Congressional Pages into having sex with him.

Yeah, I'm back and better than ever... and more pissed off at the stupidity and hypocracy of the left than ever before.

Alright, libs... come and get some.

Elliot

tomder55 answered on 10/17/06:

figured you were on vacation . glad your back . I was next going to post on Lynne Stewart and her being a runner for the terrorists . Stewart was arrested along with Mohamed Yousry, an Arabic interpreter, and Ahmed Abdel Sattar. Sattar was sentenced for 24 years for the same offense !!!

The interesting thing to me is how much George Soros contributed to her defense . The other thing is that the judge is a Clintoon appointee . Need another good reason to vote Republicans in the Senate ? Imagine Judge Koeltl on the Supreme Court ! .

ETWolverine rated this answer Excellent or Above Average Answer
Itsdb rated this answer Excellent or Above Average Answer

Question/Answer
paraclete asked on 10/16/06 - The stench of a Mission Accomplished?

A blind eye to genocide

By Phillip Adams

October 17, 2006 08:51am
Article from: The Australian



Why is the media downplaying one of the world's great tragedies, asks Phillip Adams.

Into the valley of Death/ Rode the six hundred - Alfred, Lord Tennyson

IF the foolhardy Charge of the Light Brigade could so move Tennyson in 1854, where's a patriotic poet for the great fiasco of 2006? Six hundred soldiers in the valley of death? How about more than 600,000 dead civilians in Iraq? Instead of one of Victoria's resplendent regiments being sent to its doom by an incompetent general, 655,000 Iraqis are condemned to death by an incompetent US President.

Using methodologies employed to measure death tolls in epidemics endorsed by scholars from the US Centres for Disease Control the Johns Hopkins Bloomberg School of Public Health calculate that 655,000 Iraqi civilians have died prematurely, about 600,000 by violence, since the US-British-Australian invasion in 2003.

They admit to a statistical margin of error. A low of 426,369. A high of 793,363. These figures do not include Iraq's military dead or those of the so-called coalition of the willing. The survey shows the prewar mortality rate was 5.5 people per 1000 a year. It is now 19.8.

Total civilian victims? Let us allow a ratio of five or 10 injured for every death. Either figure means millions more casualties. Theirs not to reason why, theirs but to do and die in chaos unleashed by the neo-con conmen in Washington. And, yes, in Canberra. With Bob Woodward's devastating indictment of a dysfunctional administration in the news, remember the revelations in his previous book: that John Howard played a crucial role in urging George W. Bush to invade. Now the chickens are coming home to roost for the chicken hawks who were planning this war long before the Boeings hit the World Trade Centre's twin towers.

Three thousand Americans die on 9/11 and an incoherent Bush blames Baghdad. About 3000 US troops have died in Iraq, thus doubling the US death toll. Tens of thousands of young Americans have been shipped home to military hospitals, maimed or maddened. And the forces unleashed by the US in Bush's unforgivable mixture of vengeance and pre-emption continue to slaughter their fellow Iraqis. No wonder even former defence chief Peter Cosgrove admits the war has boosted global terrorism.

Surprise, surprise. Bush and Howard ridicule the Johns Hopkins figures though the coalition has tabled none of its own. Joining in Washington's efforts to obfuscate, the Iraqi Government bars the central morgue and health ministry from releasing any details of the mounting toll. The lies and disinformation that got us into the war continue and the Iraqis pay an intolerable price for the wish-fulfilment fantasies of armchair generals and neo-cons.

Almost as shocking as the body count is the response to it. It was inevitable that George and John would brush the figures aside, but why did the media underplay it? I had expected to see front-page headlines 蝟,000 die in Iraq" but had to search the pages of most papers to find the story, usually on page 10 or 11.

It was an also-ran on the television news. If it made the cut at all, it got the briefest mention late in the bulletins.

Both the ABC and the usually reliable SBS downplayed one of the world's greatest tragedies.

With Iraq flushing his presidency down the toilet and the US mid-term elections just days away, Bush is in deeper denial than ever. But why the wider world? Are we suffering from Iraq overkill? Compassion fatigue? Boredom? Principally, I suspect, the overwhelming feeling is of shame and deep embarrassment.

The coalition is no longer willing; it's time for cutting and running. The recent concession by Britain's army chief Richard Dannatt that the occupation of Iraq is stoking bloodshed worldwide and that British troops should be withdrawn merely confirms this.

Yet many in the pro-war lobby remain in thrall to its fatal attractions. Thus the otherwise admirable Christopher Hitchens seems willing to sacrifice more Iraqi lives for his ideals, while the Quadrant regiment, with the honourable exceptions of Owen Harries and Tom Switzer, are still defending the indefensible.

Theirs not to make reply, theirs not to reason why. On into the valley of other people's deaths. Ignoring the examples of many distinguished and disenchanted US conservatives Francis Fukuyama and William F. Buckley Jr come to mind most of Australia's right-wingers are still marching behind Howard. It's as if those great clouds of Bush's political flatulence were Chanel No.5. Iraq's responsibility for 9/11, the links between Osama bin Laden and Saddam Hussein, the Baathists' support for international terrorism, the weapons of mass destruction in their silos, the inevitability of a triumphant secular democracy in Baghdad and a subsequent domino effect throughout the Middle East. And the stench of 655,000 Iraqi corpses won't change anything. There is no price in other lives they're not willing to pay.

tomder55 answered on 10/17/06:

I doubt the numbers that the Johns Hopkins Bloomberg School are true . They seem to be flawed in that they were probably lied to and they probably did not have access to the entire country to take their sampling .

At this point here are two main conflicts going on inside the nation-state called Iraq.
One is the foreign invasion of insurgents who do the bombing that attacks our military and Iraqi infrastructure.The other is the increasing local sectarian violence. I am more than willing to be responsible for deaths that occured during the invasion and our troops fight against the various insurgencies but not for the deaths that are occuring in Iraqi v Iraqi fighting .

As you know these things are rarely pretty . The US would've had the same type of violence post-revolution if the loyalists didn't have the good sense to get on the fastest horses they could find and migrate to Canada. We have alreay talked about the violence post British occupation of the Indian sub-continent.

Winning the peace is in the end, an Iraqi problem demanding an Iraqi solution. We should convey to the Iraqis elected that we will stay as long as necessary to aid in stabilizing the nation and to defend them against intervention from the Mahdi-hatter and Syria et-al ;but it is up to them to provide the blue print for a stable democratic nation . If it means partition into a loose confederation fine. I have never said that a free Iraq should resemble the USA .

paraclete rated this answer Excellent or Above Average Answer

Question/Answer
paraclete asked on 10/15/06 - On the slippery slope to nowhere?

Unholy trinity set to drag us into the abyss

Ian Dunlop
October 16, 2006

Scorched earth

We are about to experience the convergence of three of the great issues confronting humanity. Climate change, the peaking of oil supply and water shortage are coming together in a manner which will profoundly alter our way of life, our institutions and our ability to prosper on this planet. Each is a major issue, but their convergence has received minimal attention.

Population is the main driver. In the 60 years since World War II, the world population has grown at an unprecedented rate, from 2.5 billion to 6.5billion today, with 9 billion forecast by 2050. That growth has triggered insatiable demand for natural resources, notably water, oil and other fossil fuels. Exponential economic growth in a finite world hitting physical limits is not a new idea; we have experienced limits at a local level, but we have either side-stepped them or found short-term solutions, becoming overly confident that any global limits could be similarly circumvented.

Today, just as the bulk of the world's population is about to step on to the growth escalator, global limits emerge that are real and imminent. The weight of scientific evidence points to the fact the globe cannot support its present population, let alone an additional 2.5 billion, unless we embrace change.

Climate change, peak oil, water shortage and population are contributing to a "tragedy of the commons", whereby free access and unrestricted demand for a finite resource doom the resource through over-exploitation. The benefits of exploitation accrue to individuals, whereas the costs are borne by all.

Examples at local level abound, include overfishing and interrupting river flows for farming and irrigation. One mark of a mature society is that equitable solutions are found to the "commons" dilemma, and we have been relatively successful in doing this at local level. However as these issues become national and global, solutions become harder. For climate change, peak oil and water, the ultimate "commons" is the earth's atmosphere which we have been using as a garbage dump for carbon and other emissions.

As Aristotle said: "What is common to the greatest number has the least care bestowed upon it. Everyone thinks chiefly of his own, hardly at all of the common interest." In an underpopulated world this may not matter, but in our overpopulated world it is disastrous.

Solutions require that we move beyond narrow national self-interest, take a global view and place our society and economy on a genuinely sustainable footing. Sustainability, "meeting the needs of the present without compromising the ability of future generations to meet their own needs", encompasses the entire basis upon which global society operates, not just the environment. It requires realigning our ethical framework, moving away from the winner-take-all individualism which has created so many of the "commons" problems, to a more co-operative individualism, where managing the global and local "commons" is paramount.

Rather than the negative, focusing on supposed job cuts and fear of change, we should focus on the positive: we have a unique opportunity to set humanity on a new course, built around an ethical renaissance and sustainable societies. Undoubtedly there will be pain in the short term as conventional politics, economics and business models are turned on their head. However, the tools and technologies to solve these problems are available, the cost is less than we have been led to believe, and the benefits greater. Further, change can be achieved rapidly given the right impetus.

The missing ingredients for change are acceptance of the problem, the collective will for action and genuine long-term vision and leadership. Given the dominance of short-term pragmatism in our political and corporate cultures, it is likely our leaders will continue to procrastinate and not rise to the challenge. The pressure for change must come from the community at large, where it is building toward a "tipping point" which will force a fundamental realignment of political and corporate attitudes.

Historically, this has rarely happened without a crisis. Fortunately the trinity are about to trigger that crisis with a prolonged period of "creative destruction" which will radically transform society and economy whether we like it or not. Our stark choice is either to embrace the tipping point bearing down upon us, seizing the opportunity to build a sustainable future, or fudge the issue, try to muddle through in the time-honoured manner and increasingly lose the ability to control our own affairs.

For Australia, along with many other countries, water is the priority. Resolving the water crisis will be the first test of whether we can combine long-term vision and principled leadership with the need to take the hard decisions quickly enough to stave off impending disaster. If so, it will stand us in good stead to tackle the even greater tasks ahead.

Formerly an oil, gas and coal industry executive, Ian Dunlop chaired the Australian Coal Association in 1987-88 and chaired the Experts Group on Emissions Trading of the Australian Greenhouse Office in 1999-2000.

tomder55 answered on 10/16/06:

2 factors to consider from the past .

Humans were migratory and if the climate or conditions for prosperous living were not there ;humans closed shop and moved . If Aussie cannot sustain it's population with it's water supply then perhaps so many people should not live there . That is true also btw in the American Southwest and places like Florida .The geographic population shift to the desert areas in this country make little sense to me .

As I have said on another posting ,nature has a way of culling the herd. If the population becomes unsustainable all the fancy global solutions that one-worlders advocate will be for naught. Nature will take it's course and all the science just delays the inevidible .

What Ian Dunlop is suggesting is utopianism that runs completely contrary to human nature .There always have and always will be competition for resources . The last drop of water will be fought over .He would serve his cause better if he made a case for population control in areas that cannot sustain itself .

paraclete rated this answer Excellent or Above Average Answer

Question/Answer
excon asked on 10/14/06 - I'm a criminal, all right!!


Hello Law Abiding Citizens:

I'm not one. I'm a felonious poker player. Should I be ashamed??? Maybe if I killed somebody.... but for placing a bet???? Nahhh. There must be something wrong with me..

Maybe that's why people kill other people a lot more now, than they ever did. Making stupid laws demean the not so stupid laws....

excon (forever)

tomder55 answered on 10/15/06:

First thing I would do is to end this silly practice of adding riders on legislation that deal with a completely different topic. The bill in question is a port security bill. The President is put in a position to either sign the bill including the rider or to veto it completely in which case the charge of "The Republican President is weak on port security " is splashed all over the headlines. I would give the President line item veto powers.


In the days following the passage of the bill, some of the best run and regulated poker sites like the ones run from England announced they were closing their rooms to U.S. play. To continue to play now you probably have to go to sites with less accountability to regulators .(PokerStars ;the exception,takes the position that poker is a game of skill so the US laws do not apply.)

The first part of the bill makes it a crime punishable by up to five years in jail for an Internet site to either accept any type of payment for Internet gambling not permitted or licensed by a state or to even provide information on how to fund an Internet account. Good luck trying to find some of these sites . As I said ,the well run sites have already shut down US operations . The seedier sites are virtual and could be run from literally anywhere .

The second part requires the government to impose regulations on financial institutions to identify and block financial transactions to these sites.But,unless there are international laws forcing compliance then how could they enforce these laws from institutions in the Carribean or other places ? Poker sites from parts unknown will continue to flourish and smart entrepreneurs will quickly find the holes in the government regulations to service needs of the on line poker patrons. Most likely we will discover that they are organized crime or terrorist run sites. (PokerStars is registered in Costa Rica and processes payments through subsidiaries in the Isle of Man and Cyprus.PokerStars is owned by the Israeli Scheinberg family)

I expect that this law will be revesed in the near future. I bet on it in fact.


excon rated this answer Excellent or Above Average Answer
Itsdb rated this answer Excellent or Above Average Answer
nikki6 rated this answer Excellent or Above Average Answer

Question/Answer
jackreade asked on 10/13/06 - Story on Reid "ABSOLUTE CRAP"

by kos
Wed Oct 11, 2006 at 02:58:29 PM PDT

"The AP's John Solomon, the go-to guy at the Associated Press for any anti-Democratic efforts, and this piece is absolute crap. The crux of the claim:

Senate Democratic Leader Harry Reid collected a $1.1 million windfall on a Las Vegas land sale even though he hadn't personally owned the property for three years, property deeds show.

Actually, he did own that land. It just so happened that three years ago, he transfered the property from his own personal name to that of an LLC.

It'd be kind of like me selling Daily Kos, and someone claiming I reaped a windfall from it because I "sold it three years ago". I didn't. Daily Kos became an LLC. As did Reid's piece of land.

And btw, this was all disclosed to the ethics committee. The place were things got sloppy is that Reid continued to disclose ownership of the land as a personal asset rather than ownership in the LLC which owned the land. But that's it. Fact is, the LLC had no other assets other than this piece of land, and Reid disclosed ownership of the piece of land.

Solomon is either being dishonest or an idiot. But watch the wingers and GOP try to gain traction off this story to divert from their coddling of a sexual predator.

Update: Note that there is no charge or evidence or anything that would suggest that Reid used his position of authority to boost the value of the land. That would be troublesome. As it is in NC-11 where Rep. Charles Taylor (R) used earmarks to line his pockets:

New Report Shows Taylor's Earmarks Benefit Land he Owns. According to a new report by the Wall Street Journal, Charles Taylor, a wealthy businessman and banker, was able to get millions of dollars in earmarks for his district to improve land where Taylor owns thousands of acres and where he has even developed. The report shows that Taylor owns at least 14,000 acres of prime land in his district, some of which is near the main highway in Maggie Valley which, last year, received $11.4 million in federal dollars. Taylor's companies own thousands of acres near the highway and had already developed a subdivision called Maggie Valley Leisure Estates. Another earmark last year sent $4.8 million to widen a highway through timber tracts that Taylor's companies own. He also got millions for a loan for long-time contributors and millions more for improvements to a park that sits directly in front of his flagship bank in the district.

Or, let's shoot even higher:

House Speaker Dennis Hastert denied Thursday that he pushed for federal funding for a proposed highway in northeastern Illinois so he and his wife could reap about $1.8 million from land deals near their home in Kendall County.

The Sunlight Foundation, a newly created group whose declared aim is to inform the public about what members of Congress do, has accused Hastert of not divulging connections between the $207 million earmark he won for a highway, the Prairie Parkway, and an investment he and his wife made in nearby land.

There are others -- Reps. Jerry Lewis, Richard Pombo, and Gary Miller for starters. This isn't a case of "everyone does it". It's a case of projection -- accusing Reid of doing something Republicans have made a habit of doing....".

~~~~~~~~~~~~~~~~~~~~~~~~~~~~~~~~~~~~~~~~~~~~~~~~

Rebuttal to the lying right wing post a few questions down.

tomder55 answered on 10/14/06:

"ooops...did I forget to mention the 700K I made on some property I didn't own...silly me!"

What a lie ! Reid is a principal in the LLC .He sold it to himself . It would take me hours to document all the cases where Reid has used his position to enrich himself and his sons and son-in-law. Fortunately the LA Times saved me the time and trouble .

This isn't just this one instance as bad as it is,but a pattern of abuse. He happens to be Senator of a State where 95% of the land is Federally owned and he has been in these types of schemes from the get-go. (or should I say Reid and his Mormon Mafia sons and son-in-law OWN NEVADA!!!)

As for Hastert and the other back benchers Kos mentions ,if they did something wrong I have no problem in booting them also . But in exchange all the corrupt Dems have to go like Reid ,William Jefferson and Nancy Pelosi.

I Want Reid and Pelosi frog-marched out !

Itsdb rated this answer Excellent or Above Average Answer
jackreade rated this answer Bad/Wrong Answer
labman rated this answer Excellent or Above Average Answer

Question/Answer
Itsdb asked on 10/12/06 - Global 'warning' no. 2

California AG Puts Climate Skeptics on Trial

By Steven Milloy
August 1, 2006

California Attorney General Bill Lockyer is apparently trying to position California as a leader in the movement to silence scientific debate.

The State of California has filed a request in federal court to force auto makers to disclose all documents and communications between the companies and the so-called climate skeptics. California accuses the climate skeptics of playing a major role in spreading disinformation about global warming.

The underlying litigation is a lawsuit by General Motors, DaimlerChrysler Corp., and the Association of Automobile Manufacturers against the state of California challenging the states greenhouse gas emissions limits for new cars, light-duty trucks and sports utility vehicles (Central Valley Chrysler-Jeep Inc. v. Catherine Witherspoon, No. 04-6663).

California has been joined in the lawsuit by environmental activist groups including, the Sierra Club, Natural Resources Defense Council and Environmental Defense.

In a pre-trial discovery motion, California and the environmental groups asked for:

    All DOCUMENTS relating to both GLOBAL WARMING and to any of the following individuals: S. Fred Singer, James Glassman, David Legates, Richard Lindzen, Patrick J. Michaels, Thomas Gale Moore, Robert C. Balling, Jr., Sherwood B. Idso, Craig D. Idso, Keith E. Idso, Sallie Baliunas, Paul Reiter, Chris Homer [sic], Ross McKitrick, Julian Morris, Frederick Seitz, Willie Soon, and Steven Milloy, including but not limited to:

    1. All DOCUMENTS relating to any communications between YOU and these individuals, and

    2. All DOCUMENTS relating to YOUR relationship (or the relationship of any automobile manufacturer or association of automobile manufacturers) with any of them, including but not limited to payments directly or indirectly from YOU or any other automobile manufacturer or association of automobile manufacturer to any of them.


The state then goes on to quote from Ross Gelbspans book entitled, The Heat Is On:

    Ever since climate change took center stage at the 1992 UN Conference on Environment and Development (UNCED) in Rio de Janeiro, Pat Michaels and Robert Balling, together with Sherwood Idso, S. Fred Singer, Richard S. Lindzen, and a few other high-profile greenhouse skeptics have proven extraordinarily adept at draining the issue of all sense of crisis. They have made frequent pronouncements on radio and television programs, including a number of appearances by some of them on the Rush Limbaugh show; their interviews, columns, and letters have appeared in newspapers ranging from local weeklies to The Washington Post and The Wall Street Journal. In the process they have helped create a broad public belief that the question of climate change is hopelessly mired in unknowns.

    The tiny group of dissenting scientists have been given prominent public visibility and congressional influence out of all proportion to their standing in the scientific community on the issue of global warming. They have used this platform to pound widely amplified drumbeats of doubt about climate change. These doubts are repeated by virtually every climate-related story in every news-papers and every TV and radio news outlet in the country.

    By keeping the discussion focused on whether there really is a problem, these dozen or so dissidentscontradicting the consensus view held by 2,500 of the worlds top climate scientistshave until now prevented discussion about how to address the problem.


California then asserts that:

    As set forth above, Defendants are entitled to review the documents most likely to contain internal dissent at the manufacturers and the most likely such documents are those dealing with the tactics of entities like the GCC and individuals like the climate skeptics.


The automakers responded by stating that:

    The so-called climate skeptics are not on trial in this case, and the court should resist defendants attempt to put them on trial. Nor does this case require the court definitively to resolve questions regarding GLOBAL WARMING writ large. At most, as Plaintiffs have stated before and will state again at the risk of redundancy, the only relevant issue in this case with respect to global warming is the much narrower issue of what impact, if any, the A.B. 1493 Regulations will have on global warming. To adjudicate this issue, the court will need to assess the greenhouse gas reductions that the A.B. 1493 Regulations will cause and then compare these reductions to the proffered experts view about how much this level of reduction will affect the global climate. In the context of this battle-of-experts, Defendants attempt to plumb the plaintiffs files for documents regarding Defendants hit-list of climate skeptics is beside the point.


There are at least three points to make here.

First, California and the global warming lobby doesnt like what the skeptics have to say and, by virtue of this sort of intimidation, is apparently out not only to silence the skeptics but to make sure that no one dare support the skeptics lest supporters be implicated as aiding and abetting thought-crimes against California-approved, politically-correct global warming science.

Next, I wonder whether Attorney General Lockyer disclosed to the judge that Gelbspan is a rather dubious character for example, he misrepresented himself as a Pulitizer Prize winner on the jacket of his book, entitled The Heat Is On. Gelbspan never won a Pulitzer, nor was he ever even nominated. Click for more on Gelbspan

Finally, AG Lockyer has a track record of trying to silence scientific debate. In 2001, for example, the pro-gun control Lockyer gagged California state experts who opposed Lockyers dubious plans for pre-sale ballistics fingerprinting.

The so-called climate skeptics are all that stand between junk science-based global warming alarmism and higher energy prices, reduced economic growth and increased Green political power.

Support your favorite skeptic or prepare for the consequences.

~~~~~~~~~~~~~~~~~~~~~~~~~~~~~~~~~~~~~~~~~~~~~~~~~~~~~~~

Well then, let's just get algore on the stand, too.



Still think Bush is taking your rights away? The left is trying their darndest to silence scientific debate. Who needs real science when you have preconceptual science?

Go ahead, vote for a liberal, see what you get.

tomder55 answered on 10/12/06:

now I'm getting scared . this is chilling

Itsdb rated this answer Excellent or Above Average Answer
paraclete rated this answer Excellent or Above Average Answer

Question/Answer
Itsdb asked on 10/12/06 - Global 'warning'

Last week I came across an article climing we had entered "ecological debt," meaining rising consumption of natural resources means that humans began "eating the planet" on 9 October. In looking for more info on this drivel I came across something even more interesting - and truly alarming on the 'global warming' front.

David Roberts, blogging in the environmental magazine Gristmill - a magazine that Al Gore and Billy Moyers granted interviews to - advocates Nuremberg-like trials for what algore calls "global warming deniers."

    Check out this startling excerpt from George Monbiot's new book Heat.

    It's about the climate-change "denial industry," which most of you are probably familiar with. What you may not know about is the peculiar role of the tobacco industry in the whole mess. I've read about this stuff for years and even I was surprised by some of the details.

    When we've finally gotten serious about global warming, when the impacts are really hitting us and we're in a full worldwide scramble to minimize the damage, we should have war crimes trials for these bastards -- some sort of climate Nuremberg.


You people want more liberals running this country? This is what you're going to eventually get if you put the Republicans out to pasture. You think Bush is taking away your rights? You ain't seen nothin' compared to what's going to happen if the moonbats take control...

tomder55 answered on 10/12/06:




Guilty of Heresy



Michael Crichton

Imagine that there is a new scientific theory that warns of an impending crisis, and points to a way out.

This theory quickly draws support from leading scientists, politicians and celebrities around the world. Research is funded by distinguished philanthropies, and carried out at prestigious universities. The crisis is reported frequently in the media. The science is taught in college and high school classrooms.

I don't mean global warming. I'm talking about another theory, which rose to prominence a century ago.

Its supporters included Theodore Roosevelt, Woodrow Wilson, and Winston Churchill. It was approved by Supreme Court justices Oliver Wendell Holmes and Louis Brandeis, who ruled in its favor. The famous names who supported it included Alexander Graham Bell, inventor of the telephone; activist Margaret Sanger; botanist Luther Burbank; Leland Stanford, founder of Stanford University; the novelist H. G. Wells; the playwright George Bernard Shaw; and hundreds of others. Nobel Prize winners gave support. Research was backed by the Carnegie and Rockefeller Foundations. The Cold Springs Harbor Institute was built to carry out this research, but important work was also done at Harvard, Yale, Princeton, Stanford and Johns Hopkins. Legislation to address the crisis was passed in states from New York to California.

These efforts had the support of the National Academy of Sciences, the American Medical Association, and the National Research Council. It was said that if Jesus were alive, he would have supported this effort.

All in all, the research, legislation and molding of public opinion surrounding the theory went on for almost half a century. Those who opposed the theory were shouted down and called reactionary, blind to reality, or just plain ignorant. But in hindsight, what is surprising is that so few people objected.

Today, we know that this famous theory that gained so much support was actually pseudoscience. The crisis it claimed was nonexistent. And the actions taken in the name of theory were morally and criminally wrong. Ultimately, they led to the deaths of millions of people.

The theory was eugenics, and its history is so dreadful --- and, to those who were caught up in it, so embarrassing --- that it is now rarely discussed. But it is a story that should be well know to every citizen, so that its horrors are not repeated.

The theory of eugenics postulated a crisis of the gene pool leading to the deterioration of the human race. The best human beings were not breeding as rapidly as the inferior ones --- the foreigners, immigrants, Jews, degenerates, the unfit, and the "feeble minded." Francis Galton, a respected British scientist, first speculated about this area, but his ideas were taken far beyond anything he intended. They were adopted by science-minded Americans, as well as those who had no interest in science but who were worried about the immigration of inferior races early in the twentieth century --- "dangerous human pests" who represented "the rising tide of imbeciles" and who were polluting the best of the human race.

The eugenicists and the immigrationists joined forces to put a stop to this. The plan was to identify individuals who were feeble-minded --- Jews were agreed to be largely feeble-minded, but so were many foreigners, as well as blacks --- and stop them from breeding by isolation in institutions or by sterilization.

As Margaret Sanger said, "Fostering the good-for-nothing at the expense of the good is an extreme cruelty there is not greater curse to posterity than that of bequeathing them an increasing population of imbeciles." She spoke of the burden of caring for "this dead weight of human waste."

Such views were widely shared. H.G. Wells spoke against "ill-trained swarms of inferior citizens." Theodore Roosevelt said that "Society has no business to permit degenerates to reproduce their kind." Luther Burbank" "Stop permitting criminals and weaklings to reproduce." George Bernard Shaw said that only eugenics could save mankind.

There was overt racism in this movement, exemplified by texts such as "The Rising Tide of Color Against White World Supremacy" by American author Lothrop Stoddard. But, at the time, racism was considered an unremarkable aspect of the effort to attain a marvelous goal --- the improvement of humankind in the future. It was this avant-garde notion that attracted the most liberal and progressive minds of a generation. California was one of twenty-nine American states to pass laws allowing sterilization, but it proved the most-forward-looking and enthusiastic --- more sterilizations were carried out in California than anywhere else in America.

Eugenics research was funded by the Carnegie Foundation, and later by the Rockefeller Foundation. The latter was so enthusiastic that even after the center of the eugenics effort moved to Germany, and involved the gassing of individuals from mental institutions, the Rockefeller Foundation continued to finance German researchers at a very high level. (The foundation was quiet about it, but they were still funding research in 1939, only months before the onset of World War II.)

Since the 1920s, American eugenicists had been jealous because the Germans had taken leadership of the movement away from them. The Germans were admirably progressive. They set up ordinary-looking houses where "mental defectives" were brought and interviewed one at a time, before being led into a back room, which was, in fact, a gas chamber. There, they were gassed with carbon monoxide, and their bodies disposed of in a crematorium located on the property.

Eventually, this program was expanded into a vast network of concentration camps located near railroad lines, enabling the efficient transport and of killing ten million undesirables.

After World War II, nobody was a eugenicist, and nobody had ever been a eugenicist. Biographers of the celebrated and the powerful did not dwell on the attractions of this philosophy to their subjects, and sometimes did not mention it at all. Eugenics ceased to be a subject for college classrooms, although some argue that its ideas continue to have currency in disguised form.

But in retrospect, three points stand out. First, despite the construction of Cold Springs Harbor Laboratory, despite the efforts of universities and the pleadings of lawyers, there was no scientific basis for eugenics. In fact, nobody at that time knew what a gene really was. The movement was able to proceed because it employed vague terms never rigorously defined. "Feeble-mindedness" could mean anything from poverty to illiteracy to epilepsy. Similarly, there was no clear definition of "degenerate" or "unfit."

Second, the eugenics movement was really a social program masquerading as a scientific one. What drove it was concern about immigration and racism and undesirable people moving into one's neighborhood or country. Once again, vague terminology helped conceal what was really going on.

Third, and most distressing, the scientific establishment in both the United States and Germany did not mount any sustained protest. Quite the contrary. In Germany scientists quickly fell into line with the program. Modern German researchers have gone back to review Nazi documents from the 1930s. They expected to find directives telling scientists what research should be done. But none were necessary. In the words of Ute Deichman, "Scientists, including those who were not members of the [Nazi] party, helped to get funding for their work through their modified behavior and direct cooperation with the state." Deichman speaks of the "active role of scientists themselves in regard to Nazi race policy where [research] was aimed at confirming the racial doctrine no external pressure can be documented." German scientists adjusted their research interests to the new policies. And those few who did not adjust disappeared.

A second example of politicized science is quite different in character, but it exemplifies the hazard of government ideology controlling the work of science, and of uncritical media promoting false concepts. Trofim Denisovich Lysenko was a self-promoting peasant who, it was said, "solved the problem of fertilizing the fields without fertilizers and minerals." In 1928 he claimed to have invented a procedure called vernalization, by which seeds were moistened and chilled to enhance the later growth of crops.

Lysenko's methods never faced a rigorous test, but his claim that his treated seeds passed on their characteristics to the next generation represented a revival of Lamarckian ideas at a time when the rest of the world was embracing Mendelian genetics. Josef Stalin was drawn to Lamarckian ideas, which implied a future unbounded by hereditary constraints; he also wanted improved agricultural production. Lysenko promised both, and became the darling of a Soviet media that was on the lookout for stories about clever peasants who had developed revolutionary procedures.

Lysenko was portrayed as a genius, and he milked his celebrity for all it was worth. He was especially skillful at denouncing this opponents. He used questionnaires from farmers to prove that vernalization increased crop yields, and thus avoided any direct tests. Carried on a wave of state-sponsored enthusiasm, his rise was rapid. By 1937, he was a member of the Supreme Soviet.

By then, Lysenko and his theories dominated Russian biology. The result was famines that killed millions, and purges that sent hundreds of dissenting Soviet scientists to the gulags or the firing squads. Lysenko was aggressive in attacking genetics, which was finally banned as "bourgeois pseudoscience" in 1948. There was never any basis for Lysenko's ideas, yet he controlled Soviet research for thirty years. Lysenkoism ended in the 1960s, but Russian biology still has not entirely recovered from that era.

Now we are engaged in a great new theory that once again has drawn the support of politicians, scientists, and celebrities around the world. Once again, the theory is promoted by major foundations. Once again, the research is carried out at prestigious universities. Once again, legislation is passed and social programs are urged in its name. Once again, critics are few and harshly dealt with.

Once again, the measures being urged have little basis in fact or science. Once again, groups with other agendas are hiding behind a movement that appears high-minded. Once again, claims of moral superiority are used to justify extreme actions. Once again, the fact that some people are hurt is shrugged off because an abstract cause is said to be greater than any human consequences. Once again, vague terms like sustainability and generational justice --- terms that have no agreed definition --- are employed in the service of a new crisis.

I am not arguing that global warming is the same as eugenics. But the similarities are not superficial. And I do claim that open and frank discussion of the data, and of the issues, is being suppressed. Leading scientific journals have taken strong editorial positions of the side of global warming, which, I argue, they have no business doing. Under the circumstances, any scientist who has doubts understands clearly that they will be wise to mute their expression.

One proof of this suppression is the fact that so many of the outspoken critics of global warming are retired professors. These individuals are not longer seeking grants, and no longer have to face colleagues whose grant applications and career advancement may be jeopardized by their criticisms.

In science, the old men are usually wrong. But in politics, the old men are wise, counsel caution, and in the end are often right.

The past history of human belief is a cautionary tale. We have killed thousands of our fellow human beings because we believed they had signed a contract with the devil, and had become witches. We still kill more than a thousand people each year for witchcraft. In my view, there is only one hope for humankind to emerge from what Carl Sagan called "the demon-haunted world" of our past. That hope is science.

But as Alston Chase put it, "when the search for truth is confused with political advocacy, the pursuit of knowledge is reduced to the quest for power."

That is the danger we now face. And this is why the intermixing of science and politics is a bad combination, with a bad history. We must remember the history, and be certain that what we present to the world as knowledge is disinterested and honest.
(Micheal Crichton Excerpted from 'State of Fear')

Itsdb rated this answer Excellent or Above Average Answer
labman rated this answer Excellent or Above Average Answer
paraclete rated this answer Excellent or Above Average Answer

Question/Answer
Itsdb asked on 10/11/06 - Culture of corruption?

Fresh on the heels of Foleygate in the Democrats' "Culture of corruption" campaign we have this...

AP Exclusive: Reid got $1M in land sale
JOHN SOLOMON and KATHLEEN HENNESSEY
Associated Press

WASHINGTON - Senate Democratic Leader Harry Reid collected a $1.1 million windfall on a Las Vegas land sale even though he hadn't personally owned the property for three years, property deeds show.

In the process, Reid did not disclose to Congress an earlier sale in which he transferred his land to a company created by a friend and took a financial stake in that company, according to records and interviews.

The Nevada Democrat's deal was engineered by Jay Brown, a longtime friend and former casino lawyer whose name surfaced in a major political bribery trial this summer and in other prior organized crime investigations. He's never been charged with wrongdoing - except for a 1981 federal securities complaint that was settled out of court.

Land deeds obtained by The Associated Press during a review of Reid's business dealings show:

    _The deal began in 1998 when Reid bought undeveloped residential property on Las Vegas' booming outskirts for about $400,000. Reid bought one lot outright, and a second parcel jointly with Brown. One of the sellers was a developer who was benefiting from a government land swap that Reid supported. The seller never talked to Reid.

    _In 2001, Reid sold the land for the same price to a limited liability corporation created by Brown. The senator didn't disclose the sale on his annual public ethics report or tell Congress he had any stake in Brown's company. He continued to report to Congress that he personally owned the land.

    _After getting local officials to rezone the property for a shopping center, Brown's company sold the land in 2004 to other developers and Reid took $1.1 million of the proceeds, nearly tripling the senator's investment. Reid reported it to Congress as a personal land sale.


The complex dealings allowed Reid to transfer ownership, legal liability and some tax consequences to Brown's company without public knowledge, but still collect a seven-figure payoff nearly three years later.

Reid hung up the phone when questioned about the deal during an AP interview last week.


The senator's aides said no money changed hands in 2001 and that Reid instead got an ownership stake in Brown's company equal to the value of his land. Reid continued to pay taxes on the land and didn't disclose the deal because he considered it a "technical transfer," they said.

They also said they have no documents proving Reid's stake in the company because it was an informal understanding between friends.

The 1998 purchase "was a normal business transaction at market prices," Reid spokesman Jim Manley said. "There were several legal steps associated with the investment during those years that did not alter Senator Reid's actual ownership interest in the land."

Senate ethics rules require lawmakers to disclose on their annual ethics report all transactions involving investment properties - regardless of profit or loss - and to report any ownership stake in companies.

Kent Cooper, who oversaw government disclosure reports for federal candidates for two decades in the Federal Election Commission, said Reid's failure to report the 2001 sale and his ties to Brown's company violated Senate rules.

"This is very, very clear," Cooper said. "Whether you make a profit or a loss you've got to put that transaction down so the public, voters, can see exactly what kind of money is moving to or from a member of Congress."

"It is especially disconcerting when you have a member of the leadership, of either party, not putting in the effort to make sure this is a complete and accurate report," said Cooper. "That says something to other members. It says something to the Ethics Committee."

Other parts of the deal - such as the informal handling of property taxes - raise questions about possible gifts or income reportable to Congress and the IRS, ethics experts said.

Stanley Brand, former Democratic chief counsel of the House, said Reid should have disclosed the 2001 sale and that his omission fits a larger culture in Congress where lawmakers aren't following or enforcing their own rules.

"It's like everything else we've seen in last two years. If it is not enforced, people think it's not enforced and they get lax and sloppy," Brand said.

SALE HIDDEN FROM CONGRESS

Reid and his wife, Landra, personally signed the deeds selling their full interest in the property to Brown's company, Patrick Lane LLC, for the same $400,000 they paid in 1998, records show.

Despite the sale, Reid continued to report on his public ethics reports that he personally owned the land until it was sold again in 2004. His disclosure forms to Congress do not mention an interest in Patrick Lane or the company's role in the 2004 sale.

AP first learned of the transaction from a former Reid aide who expressed concern the deal hadn't been properly reported.

Reid isn't listed anywhere on Patrick Lane's corporate filings with Nevada, even though the land he sold accounted for three-quarters of the company's assets. Brown is listed as the company's manager. Reid's office said Nevada law didn't require Reid to be mentioned in the filings.

"We have been friends for over 35 years. We didn't need a written agreement between us," Brown said.

The informalities didn't stop there.

PROPERTY TAXES LOOSELY HANDLED

Brown sometimes paid a share of the local property taxes on the lot Reid owned outright between 1998 and 2001, while Reid sometimes paid more than his share of taxes on the second parcel they co-owned.

And the two men continued to pay the property taxes from their personal checking accounts even after the land was sold to Patrick Lane in 2001, records show.

Brown said Reid first approached him in 1997 about land purchases and the two men considered the two lots a single investment.

"During the years of ownership, there may have been occasions that he advanced the property taxes, or that I advanced the property taxes," Brown said. "The bottom line is that between ourselves we always settled up and each of us paid our respective percentages."

Ultimately, Reid paid about 74 percent of the property taxes, slightly less than his actual 75.1 ownership stake, according to canceled checks kept at the local assessor's office. One year, the property tax payments were delinquent and resulted in a small penalty, the records show.

Ethics experts said such informality raises questions about whether any of Brown's tax payments amounted to a benefit for Reid. "It might be a gift," Cooper said.

Brand said the IRS might view the handling of the land taxes as undisclosed income to Reid but it was unlikely to prompt an investigation. "If someone is paying a liability you owe, there may be some income imputed. But at that level, it's pretty small dollars," he said.

FEDERAL LAND SWAPS

Nevada land deeds show Reid and his wife first bought the property in January 1998 in a proposed subdivision created partly with federal lands transferred by the Interior Department to private developers.

Reid's two lots were never owned by the government, but the piece of land joining Reid's property to the street corner - a key to the shopping center deal - came from the government in 1994.

One of the sellers was Fred Lessman, a vice president of land acquisition at Perma-Bilt Homes.

Around the time of the 1998 sale, Lessman and his company were completing a complicated federal land transfer that also involved an Arizona-based developer named Del Webb Corp.

In the deal, Del Webb and Perma-Bilt purchased environmentally sensitive lands in the Lake Tahoe area, transferred them to the government and then got in exchange several pieces of valuable Las Vegas land.

Lessman was personally involved, writing a March 1997 letter to Interior lobbying for the deal. "This exchange has been through many trials and tribulations ... we do not need to create any more stumbling blocks," Lessman wrote.

For years, Reid also had been encouraging Interior to make land swaps on behalf of Del Webb, where one of his former aides worked.

In 1994, Reid wrote a letter with other Nevada lawmakers on behalf of Del Webb, and then met personally with a top federal land official in Nevada. That official claimed in media reports he felt pressured by the senator. Reid denied any pressure.

The next year, Reid collected $18,000 in political donations from Del Webb's political action committee and employees. Del Webb's efforts to get federal land dragged on.

In December 1996, Reid wrote a second letter on behalf of Del Webb, urging Interior to answer the company's concerns. The deal came together in summer and fall 1997, with Perma-Bilt joining in.

In January 1998 - just days before he bought his land - Reid applauded the Lake Tahoe land transfers, saying they would create the "gateway to paradise."

~~~~~~~~~~~~~~~~~~~~~~~~~~~~~~~~~~~~~~~~~~~~~~~~~~~~~~

"Gateway to paradise" indeed. Isn't it great to know the point man for the Senate democrats' plan to take back congress from the GOP "culture of corruption" is such a standup guy?

Steve

tomder55 answered on 10/12/06:


Although I eventually did find a version of this story in the NY Slimes ,it certainly did not make the front page . Supposedly a Slimes reporter called to talk to Reid and he asked him the wrong question and Reid hung up on him.So the Slimes had no choice but to cover the story .The Slimes was more than happy to point out however that this is nothing more than a desperate smear job by the Republicans .

Mr. Reids spokesman, Jim Manley, suggested that Republican Party strategists were behind the accusations of potential ethics violations in the land deal.

Desperate to get their scandals off the front page, Mr. Manley said, the Republican Party is once again trying to smear Democrats. The fact is Senator Reid owned the land from 1998 to 2004, and he fully disclosed that fact.


The Washinton Compost to it's credit has it as a front page story ;but spins it to suggest that it is just an oversight , a technical error that Reid may need to correct, rather than a case of corruption .

Reid offered to amend his disclosure statement. Well that ought to take care of it !! If one of these days I failed to disclose my income to the IRS I will be sure to dismiss it by saying that I'll be happy to amend my return .Let's see now ;Taxes on $700,000....35% of $700,000 = $245,000 . No wonder Harry wants to tax the rich .If the income of the rich is sufficiently hidden then no skin off his a**. I want to see his tax returns now!

I will be interested if they devote as much time to this story as they have about George Allen's zero asset stock holding in an instance where Allen loses money on the deal if he ever cashed in.

As the malefactor in this instance ,as opposed to someone who showed poor judgement like Hastert ,I think it is time that Harry Reid step down not only as Senate Minority leader,but should also surrender his Senate seat . I also think the Senate should immediately commence a non-partisan investigation led by independent council Patrick Fitgerald. We're talking about some serious Senate ethics violations, are we not?What legislation did he hump through or ear-mark as a rider to facilitated this windfall deal?I want to see what is in his e-mails and text messages !!!! Resign now Harry !!Do it for your party, do it for the country, but most of all,do it for the children.Either that or get frog-marched out of the Senate ! What did the swimmer,Evita ,Chucky Shumer and Dicky Turbin know and when did they know it ?

But Reid will not step down ,nor will he be forced to . The MSM and the DemonRATS will circle the wagons . There will be no more follow up stories lasting more than a couple of days ...forget about over a week of continuous non-stop coverage .And the Republican's in the Senate don't have the onions to persue it ....in fact I expect Specter ,McCain ,Snowe et al to come to Reid's defense (especially McCain who is quite familiar with using his position to acquire personal wealth ).

Itsdb rated this answer Excellent or Above Average Answer

Question/Answer
HANK1 asked on 10/11/06 - NATIONAL DEBT:



Can our national debt be satisfied by printing $$$ et al at our Denver mint and/or elsewhere?

HANK

tomder55 answered on 10/11/06:

Not without causing inflation. Germany tried it post WWI and it created a hyper-inflation rendering it's paper currency worthless. You have it a little backward. Printed money purchases to debt. The debt doesn't go away; it gets financed because investors pay cash for bonds (essentially IOUs ). It has to be repaid with interest .If the US were to inflates the currency to pay off its debt by essentially printing 4 trillion dollars , prices will rise so that the dollars the creditor receives are worth a lot less than the dollars they originally lent out. Then who would want to purchase future bonds ?

excon rated this answer Excellent or Above Average Answer
HANK1 rated this answer Excellent or Above Average Answer

Question/Answer
HANK1 asked on 10/11/06 - HERE OR THERE:



President Bush made a statement at his news conference this a.m. that went something like this:

If we don't beat them (Muslims/insurgents) over there, we'll have to beat them here (United States).

I agree 100%. How about you?

HANK

tomder55 answered on 10/11/06:

Hank of course I agree

But didn't you just argue last week that we should withdraw to fortress America and raise the draw bridge behind us ?

There is absolutely no precedent for the proposition that when we back down, terrorists will, too. And now is no time to pursue a foreign policy based on fear, as some would have us do. Bin Laden himself has told us that it is precisely our fearnot a fearsome responsethat emboldens the terrorists most. And if anyone is truly acting out of fear these days, it may be terrorists. It was Zarqawi himself who wrote in an intercepted letter to an al-Qaida associate that, democracy in Iraq is coming. That, he said, will mean suffocation for the terrorists. Victory in the war against terrorism is about more than killing and capturing terrorists and dismantling their networksas important as these activities remain. Victory in the war against terrorism also requires supporting those who want freedom. The Iraqi people have demonstrated time and again, braving terrorist bombs and bullets to vote, that they want liberty for themselves and their children.Rep. Duncan Hunter

HANK1 rated this answer Excellent or Above Average Answer

Question/Answer
Itsdb asked on 10/10/06 - Let's hear it for Alaskans

Alaska villages reject Venezuela oil

By JEANNETTE J. LEE
Associated Press Writer

ANCHORAGE, Alaska (AP) -- In Alaska's native villages, the punishing winter cold is already coming through the walls of the lightly insulated plywood homes, many of the villagers are desperately poor, and heating-oil prices are among the highest in the nation.

And yet a few villages are refusing free heating oil from Venezuela, on the patriotic principle that no foreigner has the right to call their president "the devil."

The heating oil is being offered by the petroleum company controlled by Venezuelan President Hugo Chavez, President Bush's nemesis. While scores of Alaska's Eskimo and Indian villages say they have no choice but to accept, others would rather suffer.

"As a citizen of this country, you can have your own opinion of our president and our country. But I don't want a foreigner coming in here and bashing us," said Justine Gunderson, administrator for the tribal council in the Aleut village of Nelson Lagoon. "Even though we're in economically dire straits, it was the right choice to make."

Nelson Lagoon residents pay more than $5 a gallon for oil - or at least $300 a month per household - to heat their homes along the wind-swept coast of the Bering Sea, where temperatures can dip to minus-15. About one-quarter of the 70 villagers are looking for work, in part because Alaska's salmon fishing industry has been hit hard by competition from fish farms.

The donation to Alaska's native villages has focused attention on the rampant poverty and high fuel prices in a state that is otherwise awash in oil - and oil profits. In 2005, 86 percent of the Alaska's general fund, or $2.8 billion, came from oil from the North Slope.

The Aleutian Pribilof Islands Association, a native nonprofit organization that would have handled the heating oil donation on behalf of 291 households in Nelson Lagoon, Atka, St. Paul and St. George, rejected the offer because of the insults Chavez has hurled at Bush.

Chavez called Bush "the devil" in a speech to the United Nations last month. He has also called the president a terrorist and denounced the war in Iraq.

Dimitri Philemonof, president and chief executive of the association, said accepting the aid would be "compromising ourselves." "I think we have some duty to our country, and I think it's loyalty," he said.

Over the past two years, Citgo, the Venezuelan government's Texas-based oil subsidiary, has given millions of gallons of discounted heating oil to the poor in several states and cities - including New York, Connecticut, Vermont, Rhode Island, Massachusetts and Maine - in what is widely seen as an effort by Chavez to embarrass and irritate the U.S. government and make himself look good.

Maine Gov. John Baldacci, who approved an agreement last winter to buy discounted oil, said he had no plans this year to seek a similar arrangement. In Boston, a City Council member wants a landmark Citgo sign near Fenway Park taken down and replaced with an American flag. In Florida, a lawmaker asked the state to cancel Citgo's exclusive contract to sell fuel at turnpike service stations.

About 150 native villages in Alaska have accepted money for heating oil from Citgo. The oil company does not operate in Alaska, so instead of sending oil, it is donating about $5.3 million to native nonprofit organizations to buy 100 gallons this winter for each of more than 12,000 households.

"When you have a dire need and it is a matter of survival for your people, it doesn't matter where, what country, the gift or donation comes from," said Virginia Commack, an elder in the arctic village of Ambler, an impoverished Eskimo community of 280 where residents are paying $7.25 a gallon for fuel.

For years, Alaska natives have accused the state and federal governments of sending too little money to their tiny, far-flung communities, where fuel and grocery prices are bloated by the high costs of delivery by plane and barge.

An editorial last month in the Anchorage Daily News bashed the Legislature's rejection in March of an $8.8 million state supplement to a federal program that helps poor Alaskans with home heating costs.

"It's embarrassing that residents in a state with so much oil wealth should be looking to a foreign nation for help," the newspaper said. "It's hard to blame villagers for accepting the gift."

A spokesman for Gov. Frank Murkowski, John Manly, said the governor believes Chavez's donation is a ploy to undermine Americans' faith in their government. But he said it is up to each village to make its own decision.

"It seems like a very strange irony that we produce the oil and yet every year there seems to be a chronic problem in getting the fuel to people that need it," Manly said.

Joan Eddy, principal and teacher at Nelson Lagoon's school, said most buildings in town were erected 30 to 40 years ago, which makes them pretty old, considering how they get battered by the constant 20-25 mph wind coming off the ocean. Their heating systems are aging, too.

She noted the fuel barge is late arriving this year, and said residents are turning on their furnaces for only a few hours in the morning and at night.

"We're conserving as much as we can because we are concerned. It looks like it's going to be a snowy winter and cold," she said.

tomder55 answered on 10/11/06:

The Citgo station by me is beginning to grow cobb-webs on it's pump handles .

Itsdb rated this answer Excellent or Above Average Answer

Question/Answer
excon asked on 10/10/06 - Hastert coverup - His staff did it!

Hello:

Hastert: Well, I dunno if my staff covered up. They don't tell me anything. They'll be under oath pretty soon and then maybe we'll learn the truth.

Huh?

And, this is the guy who is running things for the Republicans............

Omigawd are we in trouble?

excon

tomder55 answered on 10/11/06:

my position hasn't changed. He should remove himself from consideration as Speaker of the House once the next Congress is seated in January with a Republican majority .All the Republican leadership positions should be open .The current leadership should make way for the next generation .

But if the Dem's take the House then a Rep. stalking pages will be the least of our concerns .

excon rated this answer Excellent or Above Average Answer
labman rated this answer Excellent or Above Average Answer

Question/Answer
excon asked on 10/11/06 - N. Korea


Hello:

Bush: Well, NOT talking to the N. Koreans didn't work, so I'm going to stay the course.

Huh?

You guy's really ought to do something with that guy.

excon

tomder55 answered on 10/11/06:

You really are not understanding what is happening here . This is not a failure of Bush policy but it is a set back for the Chinese and even more it is an indictment of the failure of mutilateral agreements like the NNPT and watchdog agencies like the IAEA. Yes it can be said that now there is a state of confusion regarding where to go from here but that is because no alternative beyond the Bush doctrine or stay the course multilaterlism has ever been suggested. The multilateral option is a failure and no one that I can see is pushing hard for the Bush Doctrine .Given time I suspect the world will be begging us to act however .

Why is it a Chinese failure ? Because primarily NORK is a sattelite state of theirs . It was the Chinese who proposed the 6 party format after Bush ramped up the pressure and word leaked that the US was ready to bomb.China even considered a preemptive invasion of their own against Kim to pressure him to the table for talks that ultimately dragged on and went nowhere. Kim Jong Il brazenly let his country become the first to walk away from the NPT and expel inspectors from the International Atomic Energy Agency.

In response, the South Koreans and Chinese not only urged more sympathy and understanding but gave North Korea more aid and money. Even after Kim Jong Il walked out the talks last autumn, President Hu Jintao responded by signing commercial deals with Pyongyang. Chinese companies negotiated a dozen 50-year concessions to exploit North Koreas oil and gas deposits, re-open iron ore and gold mines and repair ports, harbors, power stations and railroads. This only inspired South Korean companies to push their own government harder to develop ties with the North.

The folly of these actions will cost China dear in many ways. East Asias security framework is based on a deal which has now fallen apart. The United States prevented South Korea, Taiwan and Japan from acquiring their own nuclear deterrents and removed its own nuclear weapons from the region. In response China committed itself to preventing North Korea from becoming a nuclear power.

Beijing has become, as Mao might have put it, another paper tiger and the price will be a much more assertive Japan determined to ensure its own security.Somthing the Chinese rightfully dread.

Instead of saying that this has weakend US positions like the Dems. mistakenly contend ,the truth is that it strengthens Bush's position. He can now from a position of strength persuade Russia, France and China to agree to forceful measures against Iran, a far greater threat to international peace than North Korea. He can begin pushing through a drastic overhaul of the NPT; as he had tried to do in the past . Otherwise everyone will have to live with a future where any state, even one as poor and isolated as North Korea, can go nuclear and build its own arsenal of missiles. From there it is just a short step to imagining a world where Al Qaeda or some other crazed fanatic can take over another Afghanistan - and threaten the world. The actions of crazy Kim have legitimized everything Bush has been saying about rogue states with WMD in their arsenal .


excon rated this answer Excellent or Above Average Answer
labman rated this answer Excellent or Above Average Answer

Question/Answer
jackreade asked on 10/10/06 - Prepare for War Order

A prepare for war order was issued to the commanders of the various vessels now chugging to the Staits of Hormuz, and it is reported that these commanders are concerned because Congress has not issued a Declaration of War on Iran.

Is a War on Iran Rove's October surprise?

tomder55 answered on 10/10/06:

It would be foolish for them to cruise into a war zone without directives and ROEs .This is what I know ;The USS Ike was refitted at Norfolk and is at sea for the first time in 6 years . It is accompanied by the ususal number of support ships and subs.The ship conducted sea trials until October 2005, and its crew and air wing have been training for the deployment ever since.It is to be deployed in the Indian Ocean region.

It is going to relieve USS Enterprise which is scheduled to return home next month .I have not heard any information about an extended tour for Enterprise. This is not an increase in strenght as moonbat Randi Rhodes was bleating this afternoon.

Since the end of August, the Enterprise has launched 300 sorties and dropped about 90 precision bombs and rockets in support of NATO forces in Afghanistan. I would presume that the mission of Ike is simular but if the Mahdi-Hatter does something provocative the Ike will certainly be in the right area to react.

I will however remind you that back in August I posted that the Mahdi-hatter was angry with Hezbollah because they initiated a war with Israel ahead of his timeline. It is the Mahdi-hatter that has a plan to provoke and attack and I think it is generally agreed by those who analyse it that his plan was to initiate something in late fall. Iran thinks that the cloud cover over their mountian terrain will inhibit American attacks on their nuke facilites.At the time I said that the various war games Iran was conducting was in preparation for a future conflict coming sooner rather than later. I mentioned the messianic visions of the Mahdi-hatter and how he sees this year as the year of return of the 12th Imam .

The end of this month more or less coincides with the time when the impotent UNSC is supposed to issue it's next hand slap at the regime for their defiance. The Mahdi -hatter has already warned to UN not to impose sanctions. The Washington Times reported that the United States is confident that Russia and China will join it in pushing for U.N. sanctions against Iran.YNet News reported that Condoleezza Rice is expected to pressure allied nations to instruct their nations banks to cut off all ties with bodies contributing to Irans nuclear program.Dow Jones Newswires reported that EU diplomats are privately admitting that the negotiations with the Iranians are dead.

It is my view that given the rise in tensions it wouold be foolish of the US not to increase the force strength in the region. It is my understanding that many "moderate" Arab states will quitely cheerlead in the backround if we gave Iran a bloody nose.

Yes the US military prepares for war ,it would be neglegent not to especially given what we already know about Iran's preparations.

" America was expectant of the crisis that fell on December 7, 1941; and yet America remained reluctant to say out loud that war was unavoidable, inevitable, already under way -- the nation holding back as if the obvious war plans in Berlin and Tokyo were going to vanish like a lightning storm." (John Batchelor )








excon rated this answer Excellent or Above Average Answer
Itsdb rated this answer Excellent or Above Average Answer
jackreade rated this answer Excellent or Above Average Answer

Question/Answer
excon asked on 10/10/06 - Good news! The Republicans are going to lose control.


Hello people:

The bad news is the Democrats are going get it.

excon

PS> Question - what to do, what to do???

tomder55 answered on 10/10/06:

both parties are pretty lame .That much we agree on.

excon rated this answer Excellent or Above Average Answer

Question/Answer
excon asked on 10/10/06 - YES, it IS Bushs' fault


Hello Bushwackies:

Once apon a time there were weapons inspectors in N. Korea. Then George called them, along with Iraq and Iran, the axis of evil. George then invaded one of them.

N. Korea then kicked out the inspectors and started building a weapon.

I dunno. I don't like N. Korea. But, I think if I were running the show over there, I woulda done the same thing.

And, he was right, wasn't he? Not Bush - he's as wrong as you get. KIM is the one who was right, cause we're not going to be doin any invading real soon.

Doncha just hate it, when evil dudes are right, and YOUR president is wrong?

excon

tomder55 answered on 10/10/06:

Perhaps there were 2 SUB-KILOTONNE explosions that may or may not be nuclear .But I only have found one source reporting this so for now lets go with the one blast.

The small size of the detonations has led to speculation that the North Korean tests are really "faked" nukes using large quantities of convention munitions. Another possiblilty is that he secured one of the suitcase nukes that were on the open market at the beginning of the 1990s and decided to detonate it as a ruse to make us think his program is further along than it really is .

The Center for Non-Proliferation Studies confirms that suitcase nukes would be about a kilotonne in strength or less .

* Small size (60x40x20 cm) and relatively light weight (probably upward of 30 kg). These parameters are generally consistent with available information about Soviet 152-mm artillery shells, as well as with the U.S. SADM. * Low yield (less than 1 kt, maybe as low as 0.1 kt). *

Or I suppose that if I was to stretch it to the absurd then I might believe that his program is so successful that he was able to advance his program to a point where he was able to miniturize his arsenal already down to an efficient 'low yield' tactical nuclear weapon without any prior testing in a shorter period of time than anyone has done prior.

I don't buy into any of it . If this wasn't a dud then it was either a deception or the equivelent of a dirty bomb. Nothing that should change the strategic equation on our part. He is no more or less a threat to us then he was before ;but to his neighbors he has crossed the line .

You know how much he craves attention . He is a little jealous of all the attention we are paying to the Mahdi-Hatter .He hopes that this move gets enough attention that he can force us into one-on-one negotiations. If this is a nuke ,then he also hopes to sell it to the highest bidder in the M.E. which is now Iran since we've taken Saddam out of the market.



I'm so Ronery / So ronery / So ronery and sadry arone / There's no one / Just me onry / Sitting on my rittle throne / I work very hard to be number one guy / but, stiwr there's no one to right up my rife / Seems rike no one takes me serirousry / And so, I'm ronery / A rittle ronery / Poor rittle me / There's no one I can rerate to / Feewr rike a biwd in a cage / It's kinda siwry / but, not reawry / because, it's fiwring my body with rage / I'm the smartest, most crever, most physicawry fit / but, none of the women seem to give a sh*t / Maybe someday, they'wr awr notice me / And untiwr then, I'wr be ronery / Yeah, a rittle ronery / Poor rittle me...

excon rated this answer Excellent or Above Average Answer

Question/Answer
jackreade asked on 10/09/06 - 2001 Bush Withdraws US from Anti-Ballistic Missle Treaty

In 2001, Bush withdrew America from the Anti-Ballistic Missle Treaty.

Now, we have N Korea launching missles and conducting an underground nuclear weapons test using plutonium obtained since Bush Presidency.

Bush started a war of adventurism in Iraq.

We have Russia and Iran and others forming an alliance to oppose America's position as the sole remaining superpower, a hyperpower, as it were. The New Cold War.

There are rumors of a plan to invade Iran.



NEVER ENDING WAR IS OBVIOUSLY HIS SICK PLAN.


Who benefits from this plan? I know do you??

tomder55 answered on 10/10/06:

The ABM was a cold war relic . You will be happy when we have the capability of knocking N.Korean missiles out of the sky . As soon as possible Japan ,Korea ,and Taiwan should be armed to counter the threat from the loon running the gulag /concentration camp ,N.Korea .You are on the wrong side on this issue .

jackreade rated this answer Excellent or Above Average Answer

Question/Answer
excon asked on 10/09/06 - Bush failed!! - Bush failed!! - Bush failed!!!


Hello Bushies:

In the final analsys, North Korea is the biggest failure in an administration that knows nothing else BUT failure. The Koreans DO have WMD's!!!!!!!

It's a failure that endangers the lives of your family and our country. What does that say for family values?

excon

tomder55 answered on 10/09/06:

such is the danger in putting all your eggs into the "multilateral negotiations " option that the world community seems to love so much . The same mistake is playing itself out regarding Iran and would've been repeated if Iraq had been dealt with simularily .The sanction programs were a failure and once that happened Saddam would've been on it's way .

Since the world seems so intent on letting rogue regimes obtain nukes then let's just scrap anti-proliferation efforts and hope every madman on the planet buys into M.A.D. Think that likely .

Japan ,Taiwan and South Korea, it's time for you to go nuclear.

But we can be tranquil and thankful and proud For man's been endowed with a mushroom-shaped cloud And we know for certain that some lucky day Someone will set the spark off and we will all be blown away

excon rated this answer Excellent or Above Average Answer

Question/Answer
paraclete asked on 10/08/06 - on a hiding to nothing?

NATO chief warns Afghans may switch allegiance to Taliban
POSTED: 3:45 p.m. EDT, October 8, 2006

KABUL, Afghanistan (CNN) -- The commander of NATO forces in Afghanistan said Sunday he "would understand" if many Afghans switched their allegiance back to the Taliban due to the failure of international forces to deliver needed improvements.

Speaking a day after the fifth anniversary of the start of the U.S.-led war that toppled the Taliban regime, British Gen. David Richards, commander of NATO's International Security Assistance Force, also repeated his call for more troops.

"By this time next year I would understand if a lot of Afghans, down in the south in particular, said to us all, 'Listen, you're failing year after year at delivering the improvements which you have promised to us. And if you don't do something about it,' that 70 percent or so will start saying, 'Come on, we'd rather have the Taliban.'"

The Taliban imposed strict Islamic law on Afghanistan after it came to power in 1996. Human rights groups worldwide accused the Taliban of cruelty and depriving citizens of human rights and religious freedoms. The regime also sheltered al Qaeda's leadership, including Osama bin Laden.

Since ousting the Taliban, U.S. and international forces have faced complaints that badly needed basic services are not being provided to large parts of Afghanistan. Soldiers have continued to battle insurgents, including remnants of the Taliban.

U.S. forces have helped build the new Afghan military and steadily hand over power authority to NATO forces. Last week, authority in the east was transferred from the U.S.-led coalition to ISAF at a ceremony in the capital, Kabul.

ISAF has more than 30,000 troops serving in Afghanistan, more than a third of them U.S. forces.

Richards said NATO needs more troops "because we need to now exploit the advantages -- the more favorable situation we are in the process of achieving ... Everyone is aware I've asked for this reserve force."

"We're at a tipping point," he added. With greater effort and more financial flexibility "next year could be much better," he said.

tomder55 answered on 10/09/06:

This seems to contradict a report from Brigadier Ed Butler, Commander of British Forces in Afghanistan.

A senior British commander has said the Taliban has been "practically defeated" in parts of southern Afghanistan. Brigadier Ed Butler, Commander of British Forces in Afghanistan, called the conditions his troops had been fighting in "austere" but said there were signs of optimism.

He said: "We would acknowledge it has been pretty bloody and conditions are austere but what has been under-estimated by the Taliban is the sheer resilience, resolve and courage of the British Armed Forces. "It is important to note that we have practically defeated the Taliban in northern Helmand for this year. "People are turning towards institutions of government for the first time in 30 years and reconstruction and development projects have already started".
He added: "I remain very humbled by the morale of my force. It remains sky-high. "Plenty of people are quite fatigued after six months of very hard fighting but they have done an outstanding job and they really have made a difference to the ordinary people in Afghanistan."

labman rated this answer Excellent or Above Average Answer
paraclete rated this answer Excellent or Above Average Answer

Question/Answer
paraclete asked on 10/08/06 - It's just not cricket, old chap?

Al-Qaeda 'plotted Ashes attack'

By Andrew Ramsey and Simon Kearney

October 09, 2006 01:00am
Article from: The Australian

AL-QAEDA plotted to murder the entire Australian cricket team in their change rooms during last year's Ashes tour of Britain using sarin nerve gas sprayed by the men who bombed the London Underground.

A friend of one of the four bombers who killed 52 people when they bombed trains and buses in the British capital on July 7 last year told The Sunday Times newspaper that the al-Qaeda cell was initially ordered to kill the England and Australian cricket teams during the Edgbaston Test in Birmingham.

The claim was made by a family friend of bus bomber Hasib Hussain, who killed 13 people in London's Tavistock Square. According to the 32-year-old friend, whose family has links to a terrorist training camp in Kotli in northern Kashmir, the bombers were instructed to get jobs as stewards at the Edgbaston cricket ground and to spray sarin gas inside the changing rooms.

The second Test between England and Australia began in Edgbaston on August 4 last year.

The friend - whose real name was not published - said the attack may have been called off and the Tube bombings planned instead because one of the bombers, Shehzad Tanweer, was a cricket fan.

British and Australian authorities were unable to confirm the claims last night, but The Australian understands agencies in both countries were launching investigations to check the information. The man said he would pass his information to police.

A Cricket Australia spokesman said team management had worked closely with security chiefs and the London Metropolitan police in the aftermath of the London bombings and had not heard any reports that the teams were targeted.

Terror expert Rohan Gunaratna said some of the information supplied by the newspaper's source was incorrect.

The source said that Mohammed Sidique Kahn and Tanweer were told of the plan at the camp near Kotli in northern Kashmir, but Dr Gunaratna said the men had spent time only in another camp at Malakand.

However, Dr Gunaratna said al-Qaeda had considered attacking sporting venues in Europe and had tried to develop the skills to use nerve agents such as sarin gas, but had been unable to achieve that capability.

Former ASIO protective security boss Michael Roach said such an attack was feasible and prominent sporting teams should be careful about enclosed spaces.

He said that because sarin gas had been used successfully in the 1995 Aum Shinrikyo attack on the Tokyo subway that killed 12 people, al-Qaeda would consider it a potential weapon. "The fact is it has been used before with success," he said.

The management of prominent sporting teams should search and guard areas where the players gather, Mr Roach said.

Sarin is an odourless nerve agent that kills swiftly. It was first produced in 1938 in Germany as a pesticide and was believed to have been used in the Iran-Iraq war in the 1980s.

Australia's limited overs cricket squad is in India for the Champions Trophy.

Officials briefed players about the story last night.

The Australian team was playing a one-day international against England in Leeds on the day the terrorists struck London.

At the time, team management and the Australian Cricketers Association liaised closely with CA and security officials over safety concerns for the rest of the three-month tour.

A bomb scare was triggered at the Australians' London hotel later that week when a bus was abandoned outside the hotel's foyer in Kensington High Street.

And the team received an email threat sent to CA during the fourth Test of that series in Trent Bridge, Nottingham.

Details of the threat were passed to Scotland Yard and a 32-year-old man from Stoke-on-Trent was charged with "making allegations with threats to kill".

tomder55 answered on 10/09/06:

Since it is a well known fact that the 9-11 murderers would frequent American strip bars I can then assume they are safe from attack also .

Where did they possibly obtain sarin gas from ? Hmmmmmm.

labman rated this answer Excellent or Above Average Answer
paraclete rated this answer Excellent or Above Average Answer

Question/Answer
jackreade asked on 10/09/06 - THE END OF THE REPUBLICAN REVOLUTION

Time Magazine's cover story is about how the Foley scandal marks the end of the Republican Revolution that started with Ronald Reagan. (Full story there)





The End of a Revolution
Sex, lies and power games are just the latest symptoms of a Republican Party that has strayed from its ideals
By KAREN TUMULTY
"
We no ability to fight back and get our message out."

That quiet admission may have been the most damning one yet in the unfolding scandal surrounding Florida Congressman Mark Foley: holding on to power has become not just the means but also the end for the onetime reformers who in 1994 unseated a calcified and corrupted Democratic majority. Washington scandals, it seems, have been following a Moore's law of their own, coming at a faster clip every time there is a shift in control. It took 40 years for the House Democrats to exhaust their goodwill. It may take only 12 years for the Republicans to get there.

1994-2006 Republican Revolution RIP

~~~~~~~~~~~~~~~~~~~~~~~~~~~~~~~~~~~~~~~~~~~~~~~

tomder55 answered on 10/09/06:

The NY Slimes says today that the Republican base sees this as a personal sexual matter ... a matter of personal morality, not institutional dysfunction. Where have I heard that argument before ? Hint :did I see someone post that Clinton was doing Monica a favor ?

This is Foleys lifestyle, said Ron Gwaltney, a home builder, as he waited with his family outside a Christian rock concert last Thursday in Norfolk. He tried to keep it quiet from his family and his voters. He is responsible for what he did. He is paying a price for what he did. I am not sure how much farther it needs to go.......


All said the question of broader responsibility had quickly devolved into a storm of partisan charges and countercharges. And all insisted the episode would have little impact on their intentions to vote.


If it is Time's contention that the scandal will sway the Reagan Democrats .The answer is that they have for the most part returned to the Democrats anyway . If they think it sways the swing voters like the 'security moms' ...yes it probably does ,but they can be courted in future elections . See my response to excon yesterday for further elaboration.

excon rated this answer Excellent or Above Average Answer
jackreade rated this answer Average Answer
labman rated this answer Excellent or Above Average Answer

Question/Answer
CeeBee2 asked on 10/08/06 - Animated map - shows the history of the

Middle East in 90 seconds:

MapsOfConquest

tomder55 answered on 10/08/06:

Yes strategically the ME has always been the cross road of the world .Empires needed to contol it and that was long before oil played a key part in the equation. The Muslim conquest of the ME blocked the trade routes of the Europeans to the far East . They compensated for it by trying to find a direct sea route travelling east. So it can be said that the conquest of the ME helped in the discovery of the Western Hemisphere by Europeans sooner than it would've happened othrwise.

CeeBee2 rated this answer Excellent or Above Average Answer

Question/Answer
CeeBee2 asked on 10/08/06 - Please!!!! Save the kittens!!!!!...................

Save a kitten

tomder55 answered on 10/08/06:

the funny thing is that the things chasing the cats resemble Howard Dean and Ted Kennedy .

CeeBee2 rated this answer Excellent or Above Average Answer

Question/Answer
arcura asked on 10/07/06 - What dod you thing will happen if N. Korea tests????

Shots Fired Along Korean Border as Tensions Mount Over Nuke Tests
Saturday , October 07, 2006

SEOUL, South Korea Tensions mounted over North Korea's threat to test its first atomic bomb, with shots ringing out Saturday along the border with South Korea and Japan warning of harsh sanctions if Pyongyang goes nuclear.
With a possible test expected as early as Sunday, the U.N. Security Council issued a stern statement Friday urging the country to abandon its nuclear ambitions and warning of unspecified consequences if the isolated, communist regime doesn't comply.
Jittery nations have warned a test would unravel regional security and possibly trigger an arms race.
CountryWatch: North Korea
A midday incursion Saturday by North Korean troops into the southern side of the no-man's-land separating North and South Korea only stoked the unease.
South Korean soldiers rattled off 40 warning shots at the five communist troops who crossed the center line of the Demilitarized Zone, the inter-Korean buffer.
It was unclear whether the North Korean advance was intended as a provocation, or was an attempt to go fishing at a nearby stream, an official at South Korea's Joint Chiefs of Staff said on condition of anonymity, citing official policy. No one was hurt, and the North Koreans retreated.
While such border skirmishes are not unheard of, they are relatively rare. Saturday's incursion was only the second this year, the official said.
Meanwhile, world powers were stepping up diplomatic efforts to avert a nuclear test. Japanese Prime Minister Shinzo Abe was to visit Beijing on Sunday for talks with Chinese President Hu Jintao and then proceed to Seoul for talks with South Korean President Roh Moo-hyun the following day.
A State Department spokesman, Kurtis Cooper, said Saturday the United States was concerned about North Korea's threat to test its first atomic bomb and that the department was closely monitoring the high tensions.
Also Saturday, South Korea's nuclear envoy announced he will visit Beijing on Monday for two days of talks with Chinese officials about the threatened nuclear test.
In a separate statement from Tokyo, Japan's Foreign Ministry said it was prepared to push for punitive measures at the United Nations if the North goes ahead with the test.
"If North Korea conducts a nuclear weapons test despite the concerns expressed by international society, the Security Council must adopt a resolution outlining severely punitive measures," the ministry said.
Japan plans to step up economic sanctions against North Korea, tighten trade restrictions and freeze additional North Korea-linked bank accounts should a nuclear test be carried out, Japan's Nihon Keizai newspaper reported.
The U.N. statement adopted Friday expressed "deep concern" over North Korea's announcement Tuesday that it is planning a test.
The council acted amid speculation that a nuclear test could come on Sunday, the anniversary of North Korean leader Kim Jong Il's appointment as head of the Korean Workers' Party in 1997.
Japanese Vice Foreign Minister Shotaro Yachi told Japan's TV Asahi: "Based on the development so far, it would be best to view that a test is possible this weekend."
The U.N. statement also urged North Korea to return to six-nation negotiations aimed at persuading the country to abandon its nuclear ambitions in exchange for security guarantees and badly needed economic aid.
Those talks, which involve the United States, China, Japan, Russia and North and South Korea, have been stalled since late last year, when North Korea boycotted the negotiations in response to American economic sanctions.
A North Korea expert in China, the North's closest ally, said only the removal of the sanctions could dissuade the North.
"North Korea has already made a decision to carry out a test," said Li Dunqiu of China's State Council Development Research Center, a Cabinet-level think tank. But "if the U.S. removes sanctions ... then tensions can be eased. Otherwise launching a nuclear test is unavoidable for North Korea."
The United States imposed economic restrictions on North Korea last year to punish it for alleged counterfeiting and money laundering.
North Korea said Tuesday it decided to act in the face of what it claimed was "the U.S. extreme threat of a nuclear war," but gave no date for the test. Washington has repeatedly said it has no intention of invading North Korea

tomder55 answered on 10/08/06:

Poor Kim feeling neglected and unloved again ? Gee maybe the UN will act tough and send a stern letter of disapointment in their conduct .This is all bluster .Kim thinks that holding direct talks with the US gives him some prestige . The US has decided to take the world's advice and the treat the N.Korean issue in a multilateral regional way .But now the US gets criticized for not acting unilaterally .Go figure.

I think that enough of his neighbors have warned him not to test and he will graciously comply . Truth is ,he dare not risk a test that could likely fail. He is better off letting the world think he has an arsenal of precision nukes and delivery rockets.


I was sent from pranet Xiron to conquer the Earf /
I had a twiffic pran - I thought it would work /
I tried to get the Earfrings all to kill each other, y'see /
But it all went wrong and now I must decree /
You are worthress Arec Bardwin /
You are worthress Arec Bardwin /
You have faiwred in every way /
and now my stock in you has fawren /
Your career is stawrin' /
and you're worthress Arec Bardwin

arcura rated this answer Excellent or Above Average Answer

Question/Answer
jackreade asked on 10/07/06 - I Just Voted Absentee Ballot

I thought this would be interesting for the Politics Board...I just voted Absentee Ballot for Illinois, DuPage County, Downers Grove 058, and the following Proposition appeared on the ballot....and surprised me totally:

"Shall the United States Government immediately begin an orderly and rapid withdrawal of all its military personnel from Iraq, begining with the National Guard and reserves?"

I'd like to add that I live in one of the most heavily Republican Counties in the country. Many Republican candidated run unopposed.

Again, I was surprised. I wonder if anyone knows if something like this will appear on your ballot?

tomder55 answered on 10/08/06:

This is obviously a non-binding proposition . The President does have the authority to mobilize and use the National Guard. I have not seen it on the ballot before but many of the local townships have passed simular meaningless resolutions stating their opposition to the war here in the bluest of the blue states . I for one am more concerned that they take steps to curb the rediculous growth of property taxes which increasingly threaten to force me to move from the area . I told them in meetings that should be their concern instead of passing these grandstanding meaningless resolutions.

jackreade rated this answer Excellent or Above Average Answer

Question/Answer
excon asked on 10/07/06 - Hasert? Is he good? Or did he cover for a babyraper?


Hello Christian conservatives:

In the Foley scandal, you believe that the Republican leadership acted responsibly, or covered up for a pedophile.

I believe the evidence is clear. They covered up his activities for political purposes, just like the Catholic Bishops did.

However, my question is, do you think the Christian right is going to buy Hasert's story? Do you? Why? Because you can't believe that the party of virtue would do that? Don't you think Catholics thought that too?

excon

tomder55 answered on 10/08/06:

I am not concerned about the Chrisitian Right . I am more concerned from a political point of view with what was identified in the last election as the 'Security Moms' .

They were the swing vote in 2004 according to polster David Winston .They make up 11-14 % of the electorate;married women with children,who's vote swung on pretty much a single event ...the hostage crisis at that school in Beslan in September, 2004 . All the other issues tied into the one issue of how best to protect their children.The trend has manifested itself in increased concealed-weapons-permit applications among women; the rise of national-security-focused Web logs published by hard-hitting female "war bloggers" and an upsurge in political activism by women on core homeland-defense issues, such as immigration enforcement.
Married mothers said in interviews as recently as this summer that they remain concerned about national security and the ability of Democrats to keep them safe from terrorist strikes.But even then they were less confident in the ability of the Republicans to protect their children .At the time intangibles like gas prices and other bread and butter issues were in play and the President's campaign efforts in September had done a real good job towards securing their vote in November .

I haven't seen any recent polls involving their vote but I cannot imagine this tangible issue that is easily understandable to security moms is going to help the GOP this year. The troika of Hastert and Reynolds and Shimkus should've realized the implication that this revelation would be .Red flags are raised when a 50 something homosexual is trying to befriend 16 year olds . Wheter they saw the more explicit text messages or not they should've taken action to deal with Foley when they found out about it. Their neglegence showed an intollerable lack of leadership and judgement .

For that reason I think they would best serve their party by announcing that next term all leadership positions in the Republican caucus ,be they the majority or minority ,will be open for contest without them running .

I came to that conclusion when I saw the smoking gun evidence that he knew about Foley before he claims he did . Scott Palmer ;Hastert's chief of staff met with Florida Republican at the Capitol to discuss complaints about Foley's behavior toward pages back in November 2005 .Another one time Hastert staffer Kirk Fordham,appealed to Palmer in 2003 or earlier to intervene, after Fordham's own efforts to stop Foley's behavior had failed. Fordham said Foley and Palmer met within days to discuss the allegations. Now it is possible that Hastert was not told of this by his close aide Palmer but very unlikely ;They do not only work together but share a D.C. apartment .

Apparently the GOP strategy is to circle the wagons and try to move past this as quickly as possible .They hope that Hastert's "acceptance of responsibilty " and opening up an ethics committee investigation will be enough to turn the tide. But it is clear to me that the MSM trickle of daily revelations will continue probably right up to election day. Accepting responsibility is not enough. He should announce he will not seek to become Speaker in the next session .

It's not like he is indespensible. I do not consider his performace all that great anyway. As you recall I had alot of questions of his judgement back when he was making the phoney constitutional point about the FBI seizing evidence against William Jefferson. He has been an ineffective leader, one who has failed to communicate broad policy and philosophical goals for the House GOP unlike previous GOP Speakers Gingrich and Armey. The Republican message of fiscal restraint has obviously been lost since he assumed the chair and he got nowhere in ushering in what was supposed to be a Bush triumph after the election ;needed Social Security reform. He rarely counters the Democrats when they attack policy and has not done a thing to tout the economic achievements of this administration;which every Republican should be shouting from the rooftops . .We have not sniffed tax and spending reform even though they are consistently part of the GOP message.

David Drier, Mike Pence, and John Shadegg are all waiting in the wings and would make fine Republican leaders .It is not like the party is dependent on his leadership ;and with these Foley revelations his leadership is a determent .I look for him to make the announcement that he will step aside ...the sooner the better .




excon rated this answer Excellent or Above Average Answer

Question/Answer
Itsdb asked on 10/06/06 - FBI Says Soros CREW Sat On Foley Emails Since April

FBI Says Soros CREW Sat On Foley Emails Since April

By Dan Eggen
Friday, October 6, 2006; Page A04

The watchdog group that first provided the FBI with suspicious e-mails from then-Rep. Mark Foley (R-Fla.) said yesterday that FBI and Justice Department officials are attempting to cover up their inaction in the case by making false claims about the group....

CREW held a news conference Monday to announce that in July it had provided the FBI suspicious e-mails between Foley and a former House page. The group criticized the bureau for not taking more aggressive action and asked Justice Department Inspector General Glenn A. Fine to investigate the FBI's handling of the case.

Law enforcement officials said then that the e-mails did not provide enough evidence of a possible crime to warrant a full investigation. In the e-mails, Foley praises the physical attributes of one page and asks another teenager for his picture.

In subsequent days, unidentified Justice and FBI officials told reporters that the e-mails provided by CREW were heavily redacted and that the group refused to provide unedited versions to the FBI. One law enforcement official -- speaking on the condition of anonymity to discuss an ongoing investigation -- also told The Washington Post the FBI believed that CREW may have received the e-mails as early as April and that the group refused to tell the FBI how they were obtained.

Melanie Sloan, CREW's executive director, said copies of the original e-mails she sent to an FBI agent show those assertions to be wrong. Sloan said the agent called to confirm receipt of the e-mails and to ask if one of the parties was Foley.

Sloan said the group sent unedited e-mails to the FBI because "we wanted them to commence an investigation. We're sort of outraged that they're saying anything differently." The group has asked Fine's office to look into the FBI's assertions.

Justice Department spokesman Brian Roehrkasse declined to comment on that issue but defended the FBI's handling of the original e-mails: "The e-mails, while inappropriate, did not contain a criminal predicate to allow the FBI to move forward in an investigation."

~~~~~~~~~~~~~~~~~~~~~~~~~~~~~~~~~~~~~~~~~~~~~~~~~~~~~~~

Nothing unexpected here...a Soros funded radical left 'watchdog' group gets Foley emails(.pdf) in March, redacts them heavily before sending them to the FBI in July - under the pretense of being OUTRAGED at doing nothing to protect the children, and then accuses the FBI of having "fabricated and disseminated a cover-up story."

Oh, and as tom mentioned the emails that CREW sent didn't match the ones posted on Stop Sex Predators, an UNKNOWN site until a couple of weeks ago when it reported on some shocking emails. Nothing questionable about any of that is there?

tomder55 answered on 10/07/06:

Their plot is unravelling daily . The FBI is claiming that CREW had the e-mails as early as April and that when they handed them to the FBI the e-mails were so heavily redacted the FBI could nothing with them .If CREW was so worried about the well being of these Pages why wait until July to release them to authorities if they had them in April ?Why did they not hand them over to the Hosue Ethics Committee then ? CREW mentioned on CNN their source was a Hill Staffer. And how long did the Hill Staffer have these emails? Why is CREW obstructing the investigation?CREW may become more and more a fixture of the FBI investigation going forward .

Eleven House Republicans told Democratic leaders they want them to appear before the House Ethics Committee to answer questions about what they may have known -- and failed to disclose -- about Foley's communications .

We fully agree that anyone with knowledge of Foleys activities, who then attempted to conceal such activities, should be held accountable. Today, the bipartisan Ethics Committee announced that they will be conducting a complete investigation of the facts surrounding the case.

We support this decision and also believe that the seriousness of this goes beyond partisan politics and hope that you will join us in demonstrating full cooperation from your Members and and political operatives as this investigation continues to unfold.

Just as it must be determined whether any Republican Members or political operatives were aware of and attempted to conceal Mr. Foleys activities, it must also be determined whether any Democrat Members or political operatives were aware of, and attempted to conceal these same activities.

Therefore, we respectfully ask that you appear, under oath, before the House Ethics Committee.


I say make sure Nancy Pelosi is sworn in !!!!! What did she know and when did she know it ??? I want her to submit to a polygraph test !!!!! I want to know her prior contact with CREW .Why does she oppose an investigation led by Louis Freeh ? Why not him ? He was Clintoon's FBI man . What is she hiding ?

Same goes for Democratic Congressional Campaign Committee Chairman Rahm Emanuel ,that Clintonista slug ....and while they are at it drag Chucky Shumer before the Senate Ethics committee and find out why his staffers were dumpster diving to find personal financial papers of Michael Steele.Drag Soros in and put him under oath ! And while questioning him about CREW ask him if he is paying proper taxes on his Quantum Fund .
They want bi-partisanship and by God I say give it to them !! Put Brian Ross under oath also .Why did he run with th estory months after he found out about it ? Robin Katsoros, mother of Christopher Katsoros a former Pelosi page, ;did you offer a book deal to the pages ? Swear her in also !

As the saying goes ;the Dems. swatted a hornets nest .

Itsdb rated this answer Excellent or Above Average Answer
labman rated this answer Excellent or Above Average Answer

Question/Answer
HANK1 asked on 10/06/06 - IF I WAS PRESIDENT ...



1. I'd bring all of our guys and gals home from Iraq no later than December 1st. (As of this evening, some 2,737 of our troops have been killed in a war that's going nowhere. This does NOT include Afghanistan)

2. I'd bring almost all of our guys and gals home from Afghanistan no later than December 1st.

3. I'd suggest to all private citizens to vote out all the trash in Congress on November 7th.

4. I'd send ALL illegals back to where they came from.

5. I'd commend Wal-Mart for taking the lead in making prescription drugs (generic) affordable for all citizens. (I sure wish Sam was still with us!)

6. I'd author a Constitution of Morality for ALL local, State and Federal employees. This applies to all Congressman as well.

7. I'd increase the minimum wage to at least $9.40 an hour.

8. I'd tell Iran to go to Hell.

9. I'd tell North Korea to go to Hell.

10. I'd consider isolation for our Country after all of the above materialized.

That's it! I'm mad as hell about many things that are going on at this time in our small World. If I'm not making any sense, tell me! Even the homeless need a program to get back on their feet. That can be #11.

HANK






tomder55 answered on 10/07/06:

as I've told you many times we cannot solve our problems by retreating to fortress America . We tried it before and we got drageed into world affairs and it cost us much more for waiting. That takes care of point 1 and 2

I have no problem with 3 as long as you mean ALL the trash

4 cannot be done. We have to secure the border (or make it as secure as we can) and then concurently find a way to absorb and intigrate who is here, and to figure a more sensible way to get workers here legally . If the jobs were not here they would not be coming in .

5. I love Walmart but you are misreading what they are doing in this case . Number one it is political. The left has been bashing Walmart for a long time and all indications were that they were going to make Walmart a campaign issue. Go to many towns in America and you will find a well-organised group of activists whose primary motive for existence lies in preventing Walmart from building a store within in their town so that their poorer citizens can access cheap goods and entry-level jobs requiring few skills.They are mostly the lefties

"Walmart the campaign issue "fell apart when Walmart made their announcement that they would cut the drug prices ;so far exclusively in the Tampa Fla.area , where many of their patrons are already on a new Fed. guaranteed drug entitlement program. Retired people are an enormously powerful voting block.Bush was suckered into signing that rediculous entitlement by pressure from them and Walmart will now use them to diffuse some of the political pressure they have had come down on them .

That's fine with me . Let's see Walmart go national with this plan . Progressives hate Walmart, because the company is living proof that capitalism, not socialism, delivers affordable goods to the poor. Walmart helps keeps inflation down in this country, by forcing vendors to find ways to keep prices low in order to have access to Walmart's huge market.

But Hank ,in your fortress America Walmart would not have access to foreign supplies where drugs are produced cheaper (or most of the products they sell for that matter ).What happens when the only way they can make their plan affordable is by buying cheaper drugs produced in China ? What happens to your America first rhetoric then ?

6.Most public officials have ethics rules already .

7.Already market forces in my area has increased the real min.wage to almost that point . I certainly cannot hire entry level workers at that min wage . They would not show up to apply. The workers who would be hurt by that are the teens doing part time work . Maybe in some places Walmart workers would get an increase also but Walmart makes a profit by keeping tight margins . What happens then to your cheap drugs ?

8 and 9 I have no problem with but what does in mean ? Does it mean taking action or is it just empty rhetoric? Neither country is going away . They are an issue because they are a threat.

10 . see my comments above about isolation

HANK1 rated this answer Excellent or Above Average Answer

Question/Answer
HANK1 asked on 10/05/06 - DUMB! DUMB! DUMB!



Why do you think Foley thought he could away with his hanky panky in an election year? The average prison sentence is 15 years for a pedophile. Is Foley a full-pledged pedophile? I wonder how many MORE paper trails originate in Washington re: Skeletons in one's closet? What a bunch of clowns. President Bush sure has alot to contend with on a daily basis. Only a strong, strong person could handle this crap mentally. Sometimes I think Bush is running this country by himself. If he's getting help, who from?

HANK

tomder55 answered on 10/06/06:

He's a sick ephebophile . In DC had he consumated an act of sex with the page in question he would've been covered because the age of consent in DC is 16 .

Gee with all this discovered moral outrage by the Dems. you would think they would be leading a national campaign to increase the age of consent to at least 18 as they have to allow homosexual scout masters in the Boy Scouts ! but I digress

The irony is that Foley was one of the leading advocates in the House to strenghten internet predator laws . If he did anything criminal(which I doubt )it is due to some law he ushered in and championed . Was it good cover as cynics say ;or was he emotionally and psychologically torn by his compulsions ?

CeeBee2 rated this answer Excellent or Above Average Answer
HANK1 rated this answer Excellent or Above Average Answer

Question/Answer
MarySusan asked on 10/05/06 - "The Daily Show" is substantive news...

"Jon Stewart may joke about how his lead-in is puppets, but anyone who has ever watched "The Daily Show" knows it's a misnomer to call it fake news: It may be a fake newscast, but the news it reports and comments upon night after night is all too real. And now it's official: A study by the University of Indiana has found that "The Daily Show" is as substantive as network news.

This, as said above, is not news to anyone who watches the show; on the contrary, its viewers are highly educted and the quality roster of guests (John McCain, Helen Thomas, Thomas Ricks, Ken Mehlman, Pervez Musharraf and, okay, Samuel L. Jackson) makes it compelling viewing for news junkies as well. Indeed, in addition to covering the latest news, they often seize on less-reported news, with the added bonus of providing context to ongoing issues (nary a show goes by when Stewart does not make reference to the "Mess O'Potamia" in Iraq; also, they are all over the Foley scandal, obviously, but have frankly given more airtime this week to the recent rollback of detainee habeas corpus rights than I have seen elsewhere).

There's no question that the coverage is substantive (even, if as study-leader Professor Julia R. Fox cautions, both network news programs and "The Daily Show" are ratings-driven). But what the study does not mention is not only how the Daily Show now makes news (Stewart's Musharraf interview was picked up everywhere), but it often picks up news that has gone virtually unreported anywhere else, liike this shocking C-SPAN footage of House Judiciary Committee chair James Sensenbrenner cutting short a Congressional hearing on the Patriot Act in June 2005, actually turning off the microphone mid-sentence, gavelling out of the meeting and leaving the room. It was a stunning stunning abuse of power, and the MSM missed it (per Google, with a paltry234 hits). That's not only real news, it's real news that everyone else missed. :How's that for substance?

Eat the Press

~~~~~~~~~~~~~~~~~~~~~~~~~~~~~~~~~~~~~~~~~~~~~~~


This is the best news show on TV. I like the description...it is a fake news show, but not fake news. That sums it up.

Do you enjoy this news show?

tomder55 answered on 10/06/06:

I never miss it or the Colbert Report. Stewart is a genious in that he realizes that news has become just another entertainment option .For real substantive news you have to go beyond the print,television and radio medium and dig on your own in the net. I can deal with Stewart because I know he does not mask his program as serious journalism even though the content is as relevent and researched better than the reporting on CBS Evening News with Katie Couric.

excon rated this answer Excellent or Above Average Answer
MarySusan rated this answer Average Answer

Question/Answer
MarySusan asked on 10/06/06 - THREE MORE PAGES COME FORWARD

Brian Ross, Rhonda Schwartz & Maddy Sauer Report:

"Three more former congressional pages have come forward to reveal what they call "sexual approaches" over the Internet from former Congressman Mark Foley.

The pages served in the classes of 1998, 2000 and 2002. They independently approached ABC News after the Foley resignation through the Brian Ross & the Investigative Team's tip line on ABCNews.com. None wanted their names used because of the sensitive nature of the communications.

"I was seventeen years old and just returned to [my home state] when Foley began to e-mail me, asking if I had ever seen my page roommates naked and how big their penises were," said the page in the 2002 class.

The former page also said Foley told him that if he happened to be in Washington, D.C., he could stay at Foley's home if he "would engage in oral sex" with Foley.

The page told ABC News he was interviewed this week by FBI agents who had a six-page list of questions about Foley and the exchanges.

The second page who talked with ABC News, a graduate of the 2000 page class, says Foley actually visited the old page dorm and offered rides to events in his BMW.

"His e-mails developed into sexually explicit conversations, and he asked me for photographs of my erect penis," the former page said.

The page said Foley maintained e-mail contact with him even after he started college and arranged a sexual liaison after the page had turned 18.

The third page interviewed by ABC News, a graduate of the 1998 page class, said Foley's instant messages began while he was a senior in high school.

"Foley would say he was sitting in his boxers and ask what I was wearing," the page said.

"It became more weird, and I stopped responding," the page said.

All three pages described similar instant message and e-mail patterns, with remarkably similar escalations of provocative questions.

"He didn't want to talk about politics," the page said. "He wanted to talk about sex or my penis," the page said...".

~~~~~~~~~~~~~~~~~~~~~~~~~~~~~~~~~~~~~~~~~~~~~~~~~

I don't think this story is going to go away before the election....Republican Foley's conduct is revealed to be more and more predatory against young boys as more and more information comes out.

The Republican leadership is revealed as being more and more complicit in a cover-up for their star fund raiser.

tomder55 answered on 10/06/06:

yawn . This story has already crested and is going away. Foley is already out . That means the Republicans are already more accountable than the Dems ever have been .

Now it was revealed that at least one of the pages knew Foley was a sick-0 and baited him on and was looking for possibly a book deal. Not quite the "innocent children " they are being portrayed as .It was also revealed that many of these IMs happened after the page in question was over 18 .The page in question has now hired a high priced lawyer ....why ? A blackmail scheme gone south perhaps ?

We now know that some of the messages were altered . Is this another Rathergate ?

Nancy Pelosi, The Screecher of The House will not accept Lois Freeh as an investigator in this case . Why ? What is she hiding ?

Two of Clinton's smartest operatives have warned the Dems not to overplay their hand on this . Lanny Davis, former Clinton White House special counsel says about the Foley affair: "It's all about a private behavior situation, and I think democrats have to be real careful not to try to turn this into a political partisan issue." George Stephanopoulos agrees, saying, "Most voters who hear about this are just disgusted and they will be more disgusted if it is being thrown around for purely political reasons." But the House Dems. are too stupid to realize they have taken this as far as it will go.

excon rated this answer Excellent or Above Average Answer
Itsdb rated this answer Excellent or Above Average Answer
labman rated this answer Excellent or Above Average Answer
MarySusan rated this answer Bad/Wrong Answer

Question/Answer
excon asked on 10/05/06 - Why the double standard??? Indeed!


Hello wingers:

So, when Clinton engages in sex and covers it up, it's not the sex that pisses you off, it's the cover up... Ok....

But, when a Republican engages in sex, and covers it up, you wonder why getting laid is causing such a stir.

Are you really that dumb?

excon

tomder55 answered on 10/05/06:

actually it was the sex that made me angry but I have said many times that he was impeached for the wrong reason . should've been impeached for the selling of national security and military secrets for campaign funds .

But lets compare Hastert and Clinton for a moment .

Clintoon twice as an executive pardoned (once as a Guv and once as Prez) to predators who actually had sex with underage teens (children if you will ).The most famous one was while he was President . U.S. Rep. Mel Reynolds was serving a seven-year sentence for convictions on 15 counts of wire fraud, bank fraud, and other corruption, and for having sex with a 16-year-old campaign worker. Clinton slipped that pardon in on his last day in office .

Hastert told Mark Foley to leave ,and all we know for sure is that he was nurturing a relationship for some future date of consumation . The most I've seen on any of the Foley text messages is suggestive language ;or do you have anything that suggests Foley took the relationship further ?

So by official action Clintoon pardoned 2 sex offenders who were in jail for having sex with underaged teens and Hastert forced a deviant to leave office over much less. You don't see the difference ?

excon rated this answer Excellent or Above Average Answer
labman rated this answer Excellent or Above Average Answer

Question/Answer
MarySusan asked on 10/05/06 - LIVE BY THE SWORD?

"Is there anything worse, in the minds of most Americans than being a sexual predator to underage kids? Why it's right up there with rape and murder. Finding out who in Congress knew about Foley, when did they know it, and why didn't they do anything about it is as important to the nation as finding out who was behind 9/11.

Are our children worth less?

There's only one way to get this information. Send Foley, Hastert, Boehner, and the rest of them to Guantanamo and torture the information out of them.

I mean, that's this administration's policy, isn't it? Why restrict it to Muslims? Why not just expand it to all involved in anti-social activities, especially one as serious as this.

That we'd have to build a double-wide waterboard for Hastert is beside the point.

Habeas Corpus? Pfaff! If President Bush can waive it for Muslims, he can waive it for those who prey on our children, right?

It's the policy of the administration that Bush has the power to choose whom to torture. It's only logical thing to do, and the only way to get the truth out of these Republican Congressional leaders.

Living by the sword makes for tough payback."

Tom D'Antoni, blogging

tomder55 answered on 10/05/06:

I am sure that Tom D'Antoni would be equally outraged if Mark Foley were a Democrat. I know the MSM would be . Back when Dummycrat Gerry Studds was porking a page the Washington Compost described it as "a brief consenting homosexual relationship" and slammed detractors for conmducting a "witch hunt" against gays: "New England witch trials belong to the past, or so it is thought. This summer on Cape Cod, the reputation of Rep Gerry Studds was burned at the stake by a large number of his constituents determined to torch the congressman for his private life."The difference is that Republicans resign, while Dems defy the world when they get caught.

As we all know this has nothing to do about concern for "the children " and everything to do with October Suprise politics. The Dems and trhe MSM are throwing all the pasta on the wall and hoping some of it will stick .

So far they revisited the revolt of the Generals gambit ; the leak of selected parts of the NIE gambit ; the Bob Woodward hit book gambit .He doesn't even deny that the timing of the publication was politically motivated to affect the election ("Simon & Schuster and my bosses at the Washington Post said the only real obligation here is to tell it before the election.
"That's what we're doing. People can judge for themselves."
http://www.editorandpublisher.com/eandp/search/article_display.jsp?vnu_content_id=1003189931 ) ;and now the smear the whole Republican Party with the actions on one Rep. gambit .

I predict more attempts by them because they cannot or will not articulate what they bring to the table ;why voters should vote for them ;what they stand for .

ETWolverine rated this answer Excellent or Above Average Answer
Itsdb rated this answer Excellent or Above Average Answer
Judgment_Day rated this answer Excellent or Above Average Answer
labman rated this answer Excellent or Above Average Answer
MarySusan rated this answer Bad/Wrong Answer

Question/Answer
Itsdb asked on 10/04/06 - Fat terrorists

Rich diet making many Guantanamo detainees fat

By Michael Melia, Associated Press | October 4, 2006

SAN JUAN -- A high-calorie diet combined with life in the cell block -- almost around the clock in some cases is making detainees at Guantanamo Bay fat.

Meals totaling a whopping 4,200 calories per day are brought to their cells, well above the 2,000 to 3,000 calories recommended for weight maintenance by US government dietary guidelines. And some inmates are eating everything on the menu.

One detainee has almost doubled in weight, to 410 pounds, said Navy Commander Robert Durand, spokesman for the detention facilities at Guantanamo, a Navy base in southeast Cuba.

Human rights groups attribute the weight gain to lack of exercise. They cite accounts of released detainees who complained they were allowed to exercise fewer than three times a week outside their small cells.

But Durand said detainees are simply served a wide variety of food and are expected to choose what appeals to them.

``The detainees are advised that they are offered more food than necessary, to provide choice and variety, and that consuming all the food they are offered will result in weight gain," he said.

Most of the prisoners at Guantanamo were slightly underweight when they arrived. Since then, they've gained an average of 20 pounds, and most are now ``normal to mildly overweight or mildly obese," according to the most recent measurements, he said.

The meals include meats prepared according to Islamic guidelines, along with fresh bread, vegetables and yogurt.

With nearly all detainees fasting in the daytime during the Muslim holy month of Ramadan, authorities have arranged for a post-sunset meal and a midnight meal. Traditional desserts and honey also are served during the Ramadan observances.

Even two detainees who have been on a hunger strike for more than a year are at ``100 percent ideal body weight," from nutrients fed through tubes inserted in their noses, Durand said.

The calorie intake at Guantanamo is well above the norm for federal inmates in the United States, who receive about 2,900 calories a day, said US Bureau of Prisons spokesman Michael Truman.

He said weight gain in the civilian system is not widespread and that most inmates ``keep themselves in pretty good shape."

~~~~~~~~~~~~~~~~~~~~~~~~~~~~~~~~~~~~~~~~~~~~~~~~~~~~~~

Instead of discussing the absurd, why human rights groups are concerned that Gitmo detainees are too well fed, I have some absurd observations as to why fat terrorists are a good thing - seeing as how the left is doing all they can to get these guys released.

Fat terrorists are a good thing because:

1) Fat terrorists can't outrun pursuit

2) Potential hostages can outrun fat terrorists

3) Fat terrorists can't squeeze in and out of their cave

4) Fat terrorists are easier to spot by drone and satellite

5) A fat suicide bomber might absorb more of the blast, leading to fewer casualties and less collateral damage

6) Fat terrorists can't crawl under a car to plant explosives

7) Fat terrorists are at risk for heart disease, diabetes, stroke, osteoarthritis, and sleep apnea - things that slow them down and contribute to a shorter life span

8) Fat terrorists have to work harder to hide behind human shields

9) You can only get so many fat terrorists in a 1985 Toyota Tercel

10) It's easier to chase down a fat terrorist in the aisle of a 747

Any more?

tomder55 answered on 10/04/06:

yes I had heard that Halal is quite fattening .

Typical of human rights groups complaining they are being too well fed . the food is high in fat and cholesterol and it's tortureous to feed them such fattening foods.

This should shut up the scientist who warn that obesity is a greater threat than terrorism ;now they have metastized into a single threat .


Fat terrorists resemble their sympathisers like Michael Moore.

The bigger they are the harder they fall.

Itsdb rated this answer Excellent or Above Average Answer

Question/Answer
excon asked on 10/03/06 - Foley loves Poley

Hello wrongwingers:

What's even worse than protecting potential child molestors among its ranks (especially from the party that sold itself as the champion of conservative social values), are the record-breaking deficits.

They've partnered up with every special-interest lobbyist with a checkbook, and then used OUR checkbook to reward him!

They've out Demmed the Dems..... Who's the loser?? I don't see anybody, so it must be US!

excon

tomder55 answered on 10/03/06:

you mean National Debt. The deficits have been decreasing but the National Debt still grows. Through August, the cumulative deficit is $304.305 billion this year and was $354.123 billion during the same period last year.

The reported deficit does not include the monies stolen from Social Security and other trust funds. However ;the Republicans did have a realistic plan to reform the Social Security system had every not thought it was an untouchable golden calf.

I cannot argue with your critique of the spending pattern of the current Congress or the President's lack of the use of the veto pen .However when I go to the polls next month I have to ask myself who is likely to spend more ? I am not comforted by the Democrats like possibly future Ways and Means Chair Charles Rangal declaring that he means to raise taxes as soon as possible after he gets the reigns .

Dark_Crow rated this answer Excellent or Above Average Answer
excon rated this answer Excellent or Above Average Answer
labman rated this answer Excellent or Above Average Answer

Question/Answer
MarySusan asked on 10/02/06 - A COUPLE OF FOLY'S E-MAILS

"In addition to explicit sexual language, former Congressman Mark Foley's Internet messages also include repeated efforts to get the underage recipient to rendezvous with him at night.

"I would drive a few miles for a hot stud like you," Foley said in one message obtained by ABC News.

The FBI says it has opened a "preliminary investigation" of Foley's e-mails. Federal law enforcement officials say attempts by Foley to meet in person could constitute the necessary evidence for a federal charge of "soliciting for sex" with a minor on the Internet.

In another message, Foley, using the screen name Maf54, appears to describe having been together with the teen in San Diego.

Maf54: I miss you lots since san diego.
Teen: ya I cant wait til dc
Maf54: :)
Teen: did you pick a night for dinner
Maf54: not yetbut likely Friday
Teen: okill plan for Friday then
Maf54: that will be fun

The messages also show the teen is, at times, uncomfortable with Foley's aggressive approach.

Maf54: I want to see you
Teen: Like I said not til febthen we will go to dinner
Maf54: and then what happens
Teen: we eatwe drinkwho knowshang outlate into the night
Maf54: and
Teen: I dunno
Maf54: dunno what
Teen: hmmm I have the feeling that you are fishing hereim not sure what I would be comfortable withwell see

Foley resigned Friday after ABC News questioned him about the Internet messages.

He says he has checked into a rehabilitation facility to deal with alcohol and behavioral issues."

~~~~~~~~~~~~~~~~~~~~~~~~~~~~~~~~~~~~~~~~~~~~~~~~

So, "The Family Values Party" covers up this vile Foley's advances to young men, this pedophile freak was on a committee to protect missing and abused children.

The whole country is watching.

tomder55 answered on 10/03/06:

Perhaps Hastert believed the Democrats wouldn't out Foley because many of their own people were secretly homosexual and making pages 'boy toys 'also ? But as your last posting on this subject proves ,you hold the Republicans to a higher standard . So do I . I'm glad Foly resigned and I certainly have no problem with dumping slime from either party although I have nightmares of the clueless Nancy Pelosi being 2nd in line to the Presidency .Too bad they both can't lose.

If Republicans had jumped on Foley when all that was known about the emails was that they were "overly friendly" it would have been spun as homophobic Republicans out to get a harmless gay man. Who truly believes that the militant gay lobby didn't enter into the thinking of the Republican leadership who "knew" there was something there but didn't know exactly what when the question of what to do about Foley's emails first came up.

Too bad when the Democrats are caught they can't find that sanctimonious rage that they have displayed this week. Again ;Rep. Gerry Studds admitted to an affair with an underage page and told everyone to mind their own business . He was censured but not expelled by the Democrat majority and was reelected to his seat 5 times later . And let's not forget the swimmer who was driving drunk in his car with a staffer at the time of his plunge. He is still in the Senate .

A couple of cases that I failed to mention in my last response . Bill Clinton's(better put some ice on that lip honey) rape of Juonita Broderic. Rep. Gary Condit's "romantic" relationship and possible kidnapping/ murder of intern Chandra Levy ;while he was also having an affair with flight attendant, Anne Marie Smith.Condit tried to regain his seat but fortunatly the good people of his district thought otherwise.

CeeBee2 rated this answer Excellent or Above Average Answer
Itsdb rated this answer Excellent or Above Average Answer
labman rated this answer Excellent or Above Average Answer
MarySusan rated this answer Bad/Wrong Answer

Question/Answer
paraclete asked on 10/02/06 - So now there is a new way to be politically correct?

I wonder what his politically correct term for Jew was before he discovered he is one?

The fall of an all-American boy



Michael Gawenda
October 2, 2006

A few weeks ago, the New York Jewish newspaper The Forward published a front-page article in which it provided evidence to suggest that Etty Allen was Jewish, the daughter of Sephardic Jews. Indeed the Lumbruso family, of which she was a member, according to The Forward, was one of Tunisia's most distinguished pre-World War II Jewish families, having produced scholars, historians and even a chief rabbi or two of Tunis.

Tunisia was occupied by German forces for six months from November 1942 and Etty's father, Felix, was rounded up along with other Jews and sent to a labour camp.

Etty Lumbruso and her family migrated to the US after the war and Etty's American story is one of transformation - the sort of story Americans love - for she ended up married to a legendary football coach, George Allen, and their son, also George and himself a high school and college football star, was, until recently, a leading contender seeking the Republican Party's presidential nomination for 2008.

No longer. Allen, the senator from Virginia, who six weeks ago had an unassailable lead in his re-election bid against his Democratic opponent in the November 7 poll, is fighting for his political life.

Allen's apparent political demise - those presidential ambitions are now dead and buried - is a truly American story, and like so many American stories, it is hard to decide whether it is a tragedy or a farce. Perhaps it is both.

Despite the fact that he was raised in southern California, Allen throughout his political career - and even while he was a student at the University of Virginia - affected a populist, good ole boy identity, complete with a particular liking for the Confederate flag, chewing tobacco and multicoloured cowboy boots.

But six weeks ago, Allen's re-election campaign began to unravel. While speaing at an election rally in rural Virginia, he turned to a young man of Indian descent - dark-skinned! - who Allen knew was filming him for the campaign committee of his opponent.

"That fella here, over here in the yellow shirt, macaca or whatever his name is, he's with my opponent," he said. "So welcome to macaca here. Welcome to America and the real world of Virginia."

Within hours, the "macaca" slur was all over the internet. It made the front pages of virtually every American newspaper. The video was shown over and over again on cable news shows.

Allen refused to apologise. He said he had no idea what macaca meant. No one seemed to know what it meant except that it seemed to be a racial slur.

Allen apologised. Again and again. But he continued to say he had no idea where macaca had come from. Then came The Forward story about Etty Allen and her distinguished Jewish ancestry. The newspaper had contacted Allen to ask him whether he knew about his mother's background. Allen's campaign team decided to ignore the request.

Two weeks later, during a debate with his opponent, Jim Webb, a local television reporter asked Allen about The Forward story. Allen was stunned and furious. His mother, he said, was French-Italian "with some Spanish blood as well".

"I was raised, as she was raised, a Christian. Are you casting aspersions?"

Turns out that Allen lied. Etty Allen, after The Forward story was published, told her son that she had been raised a Jew and that when she came to America, she decided to shed her Jewish identity in order to save her children from the suffering her family had experienced. And because her husband, who was planning a football coaching career, thought the fact of her Jewishness might be a problem.

Allen meanwhile had become a man who had no idea who he actually was. He told reporters that he was proud of his newly discovered Jewish background but went on to say that he had just had a ham sandwich for lunch and that his mother made great pork chops. He said he had reassured her that he loved her more than ever. As for macaca, it turns out to be an African monkey and a word used by Italian-French North Africans before the war to well the rest is obvious.

America is a country where personal transformation is a defining element of personal and national identity, the great melting pot in which people from all sorts of backgrounds become as American as apple pie - or slices of pepperoni pizza. Last week, some of Allen's former college football teammates came forward to say that he often referred to black people as "niggers", something that he has vigorously denied. But few people believe him.

George Allen's rise and fall is a cautionary American tale of the down side of personal transformation: the past can come back to destroy you.

Michael Gawenda is the Herald's Washington correspondent.

tomder55 answered on 10/02/06:

both issues were already dealt with here . this was my posting on his Jewish heritage .Allen was not lying . He had to confront his mother to find out the truth . The fact is that she feared the very type of smearing he is now getting if the truth about his Jewish ancestory was revealed . Sadly ,she has a point.

I note that Michael Gawenda fails to mention that Allen's opponent Webb while in the military would drive a car through Watts hurling racial slurs at African Americans while pointing his gun at them.

labman rated this answer Excellent or Above Average Answer
paraclete rated this answer Excellent or Above Average Answer

Question/Answer
MarySusan asked on 10/02/06 - RICE WARNED OF ATTACK IN JULY 2001

WASHINGTON, Oct. 2 Members of the Sept. 11 commission said today that they were alarmed that they were told nothing about a White House meeting in July 2001 at which George J. Tenet, then the director of central intelligence, is reported to have warned Condoleezza Rice, then the national security adviser, about an imminent Al Qaeda attack and failed to persuade her to take action.

Details of the previously undisclosed meeting on July 10, 2001, two months before the Sept. 11 terror attacks, were first reported last week in a new book by the journalist Bob Woodward......"

~~~~~~~~~~~~~~~~~~~~~~~~~~~~~~~~~~~~~~~~~~~~~~~~~


Lots of pants on fire in the White House....liars, the lot of them.

tomder55 answered on 10/02/06:

Here is the essence of Woodward's claim :
George Tenent and Cofer Black are hearing a lot of intelligence "noise". They have "anxiety". Tenent has been losing sleep (according to Woodward's mysterious source).He did not know when, where or how, but Tenet felt there was too much noise in the intelligence systems. They go to Condi Rice with what they admit is "voodoo" intelligence to buttress their warning about bin Laden. Strangely, Condi's attention is not fully captured by this noise, sleeplessness, anxiety, and voodoo, and she doesn't "act". They are frustrated. They had wanted her to "do something". Condi suggests they do some more digging.

The entire section is one big "cover my ass" account by Tenet and Black;both of whom were Clinton holdovers, and both of whom let Bin Laden slip away 8 times before Bush ever took office.

Here is what I imagin the conversation went down :

Tenant: "I got a bad feeling about this boss. Things are too quiet."

Rice:"Have you any more to go on?"

Tenant: "Well...I'm not sleeping very well at night and I feel anxious a lot. I've found it's never wise to ignore your premonitions."



Not suprisingly Woodward did not get Condi's side of the story. Not that it matters .Woodward was completely discredited when he had that imaginary death bed interview with Reagan's CIA director Casey .Casey could not speak or sit up in his hospital bed as Woodward described it. Plus Casey's room was guarded 24/7 by guards and his wife and daughter were at his bedside and all said Woodward was lying about sneaking into Casey's room. When pressed on the Mike Douglas Show, Woodward couldn't even described the furnature inside Casey's hospital room where this death bed interview allegedly took place!

labman rated this answer Excellent or Above Average Answer
MarySusan rated this answer Poor or Incomplete Answer

Question/Answer
paraclete asked on 10/02/06 - Incurious George

Woodward's book damns Bush team


Michael Gawenda
October 2, 2006


THE White House is bracing for a week of damage control in the wake of the publication of a new book by the Watergate journalist Bob Woodward in which the Bush Administration is described as dysfunctional and faction-ridden, and President George Bush as "intellectually incurious".

State of Denial, of which almost a million copies were rushed to American bookshops at the weekend, claims Mr Bush's then chief-of-staff tried to convince him to fire the Defence Secretary, Donald Rumsfeld, at least twice in the last two years, the second time with the support of the first lady, Laura Bush.

It also reveals that the 82-year-old Henry Kissinger, secretary of state during the Nixon administration, has become an adviser to the President. Dr Kissinger, who has long argued that the Vietnam War was lost because of a lack of political will, has confirmed he has had regular meetings with Mr Bush, "just me and the President".

Several recent books have revealed how the White House has ignored advice from senior military officers and State Department officials that more troops were needed to defeat the growing insurgency in Iraq, warnings that came within weeks of Mr Bush declaring victory.

State of Denial confirms all of this and goes further, revealing how Mr Bush has never sought to question virtually any of the officials who had issued these warnings, instead relying on advice from senior commanders in the military who were loath to tell the President hard truths.

Mr Bush emerges from the book as a man of little self-doubt, who constantly tells his aides that as commander-in-chief, his job is to exude confidence in his decisions. He is, according to Woodward, a man of deep faith who prays regularly for guidance and believes his prayers are answered.

While Woodward's reporting that the then secretary of state, Colin Powell, and Mr Rumsfeld could not stand one another is not new, his assertion that Mr Powell's successor, Condoleezza Rice, urged Mr Bush to sack Mr Rumsfeld is. By the time she urged the sacking, Dr Rice and Mr Rumsfeld were hardly talking. At the urging of the Vice-President, Dick Cheney, Mr Bush refused to sack Mr Rumsfeld.

A number of Administration officials, including Andrew Card, who was the White House chief of staff until March this year, urged Mr Bush to sack Mr Rumsfeld, as did Mrs Bush, though the White House has emphatically denied this.

The book also reveals that both of Mr Bush's parents have become increasingly concerned about the war in Iraq.

At one stage, at a function, his mother, Barbara, approached David Boren, a former Democratic Party senator who had worked closely with her husband when Mr Bush senior was CIA director in the 1970s. She asked him to be candid about the war.

"Are we right to be worried about this Iraq thing?" Mrs Bush asked

"Yes, very worried," he said.

"Do you think it's a mistake?"

"Yes, ma'am. I think it's a huge mistake if we go in "

"Well, his father is certainly worried and is losing sleep over it. He's up at night worried."

"Why doesn't he talk to him?"

"He doesn't think he should unless he's asked."

Woodward suggests that President Bush never asked his father for advice about whether going to war in Iraq was a good idea, and has not talked to him at any great length about Iraq since the war started.

Indeed, Woodward's description of the relationship of President Bush with his father suggested that the President may love his father but does not respect him very much as a politician.

At one stage, according to Woodward, the Republican Party presidential hopeful John McCain was asked whether President Bush had ever asked him for his views on Iraq.

"No, no, he hasn't," Mr McCain says. "As a matter of fact, he's not intellectually curious.

"But one of the things he did say one time was 'I don't want to be like my father. I want to be like Ronald Reagan'."

There are some minor but revealing revelations in the book.

Mr Bush and his senior adviser, Karl Rove, for instance, enjoy fart jokes, which they tell each other sometimes in the middle of Oval Office meetings.

Just for the record and because I'm curious

incurious
adjective
incurious; showing absence of intellectual inquisitiveness or natural curiosity; "strangely incurious about the cause of the political upheaval surrounding them"

So to be intellectually incurious have a complete absence of any interest.

Which begs the question, how could you be a politician and be intellectually incurious? Just in it for the rush I guess.

tomder55 answered on 10/02/06:

What Woodward assumes is a "deteriorating situation" may, through the eyes of an experienced Middle-East analyst, be a normal series of events occurring as predicted -- no 'red flag,' no reason to call in Congressional leaders. What Woodward sees as an eminent civil war may be, through those experienced eyes, an expected escalation of violence that is being 'handled' as well as it can be; again no 'red flag.'

A President does not make decisions in a vacuum, he has intelligence estimates and many advisors. What does Bob Woodward have besides anonymous sources and . . . oh yes . . . a book contract?

Woodward shows a complete lack of historical perspective. I can illustrate many examples of the period before the end of a conflict being the most chaotic and deadly . Just ask the Ruskies .The Soviets sustained 20,00025,000 dead in Berlin and 81,000 for the entire operation. Another 280,000 were reported wounded or sick . The Germans sustained as many as 450,000 killed, wounded or missing, civilians included.

I will reserve some critique on the book until I've read it . From the exerpts I have read I tend to agree with the slamming of Viceroy Bremer . Rumsfeld should've paid more attention to what Jay Garner was telling him about what I think were boneheaded decisions by Bremer .But generally as I have posted ,Woodward is doing his own brand of damage control . His last book wasn't a hit piece on Bush and he withheld information about Plame to his main patron ,the Washington Compost. He is just re:establishing his credentials with the moonbats.

paraclete rated this answer Excellent or Above Average Answer

Question/Answer
Judgment_Day asked on 10/01/06 - Just a notice.....

Now, these are the senators who voted to give illegal aliens Social Security benefits. Regardless of political party, these politicians need to be defeated in 2006, 2008 or 2010, whenever they come up for office. They are grouped by
home state. If a state is not listed, there was no voting representative.

Alaska: Stevens (R)
Arizona: McCain (R)
Arkansas: Lincoln (D) Pryor (D)
California: Boxer (D) Feinstein (D)
Colorado: Salazar (D)
Connecticut: Dodd (D) Lieberman (D)
Delaware: Biden (D) Carper (D)
Florida: Martinez (R)
Hawaii: Akaka (D) Inouye (D)
Illinois: Durbin (D) Obama (D)
Indiana: Bayh (D) Lugar (R)
Iowa: Harkin (D)
Kansas: Brownback (R)
Louisiana: Landrieu (D)
Maryland: Mikulski (D) Sarbanes (D)
Massachusetts: Kennedy (D) Kerry (D)
Montana: Baucus (D)
Nebraska: Hagel (R)
Nevada: Reid (D)
New Jersey: Lautenberg (D) Menendez (D)
New Mexico: Bingaman (D)
New York: Clinton (D) Schumer (D)
North Dakota: Dorgan (D)
Ohio: DeWine (R) Voinovich(R)
Oregon: Wyden (D)
Pennsylvania: Specter (R)
Rhode Island: Chafee (R) Reed (D)
South Carolina: Graham (R)
South Dakota: Johnson (D)
Vermont: Jeffords (I) Leahy (D)
Washington: Cantwell (D) Murray(D)
West Virginia: Rockefeller (D), by Not Voting
Wisconsin: Feingold (D) Kohl (D)

tomder55 answered on 10/02/06:

For my part I never have and never will vote for Chucky Shumer or Evita Clinton.

excon rated this answer Excellent or Above Average Answer
Judgment_Day rated this answer Excellent or Above Average Answer

Question/Answer
MarySusan asked on 10/01/06 - FOLEY HITS ON HIGH SCHOOL BOY

GOP COVERS UP.

"So maybe some of you are shocked by the growing scandal over the cyber-trolling by Representative Mark Foley and the fact that Representative Denny Hastert and NRCC Chair Tom Reynolds, among others, have been fully aware that this man was hitting on high school boys for the past year.

You shouldn't be.

This is, very simply, emblematic of the current state of the GOP, a once proud party of Lincoln and Teddy Roosevelt that has devolved into an elected membership comprised of racists, criminals, self-haters and perverts. Am I exaggerating? I think not.

I'll leave the Neo-Confederates such as George Allen and the Abramoffers such as Bob Ney for another column. Let's just talk about the latter two categories. Former Representative Ed Schrock of Virginia, who sponsored anti-gay legislation in the House, was caught in 2004 making calls to a homosexual phone-sex operation. He then resigned without comment much like Foley. Hate yourself much Ed?

See Mayor Jim West of Spokane for a very similar story to the one currently bedeviling poor Mark Foley, who was still Chairing the House Caucus on Missing and Exploited children as of last week.

You want to find adultery among these miscreant moralizers? Why go no further then Pennsylvania Representative Don Sherwood, who thought it wasn't good enough to just cheat on his wife with a women in her 20s (he's in his 60s), but decided to try and choke her for extra credit. Or Representative Steve LaTourette of Ohio, who didn't only cheat on his wife of 28 years with an aide, but set her up with a nice little shop that lobbied a committee on which he sat (Transportation). The piles of money she made was helpful though, as it most likely helped pay his rent as she was living with him.

Divorce you say. Ok, well here's a math problem for you. What do you get when you add the divorces of Newt Gingrich + Rush Limbaugh + Bob Barr? A hint, you're almost in double digits.

Senate Republican Leader-to-be, Mitch McConnell, has no relationship with his first wife and daughters (he runs a helluva partisan machine in Kentucky and Washington, however). Senate Majority Leader Bill Frist? He dumped his fiancee for another woman on the eve of his marriage...."Cliff Schecter, blogging.

~~~~~~~~~~~~~~~~~~~~~~~~~~~~~~~~~~~~~~~~~~~~~~

My favorite Republican sexualcapade is the homosexual prostitute who visited the White House over 600 times, was it. He also sat in on White House briefings getting in with a press pass.

Sounds like a Rove "pimp" on the press corps!

I wonder who needed a homosexual prostitute?

tomder55 answered on 10/02/06:

seems to me that the term hypocracy can be flung to both sides of the aisle ...unless of course the Dems concede they are openly the party of sleeze perversion and corruption. Or do you admit that Republicans are held to a higher standard ?

I guess the biggest difference here is that the Republicans forced him to resign . Meanwhile Barney Franks ;who had a brothel running out of his apartment gets reelected term after term .

Meanwhile Democrat Rep. William Jefferson will be reeelcted even though he had $90,000 in bribe money stuffed in his freezer.

Meanwhile the Dems. invent charges of racism against Sen, George Allen while they revere a former KKK leader and reward him with essentially a lifetime position in the Senate .

I do not recall an outrageous cry of indignation by the Dems. when their own homosexual Congressman Gerry Eastman Studds admitted to having relations with a 17 year old Congressional page in 1983.As the House read their censure of him, Studds turned his back and ignored them. Later, at a press conference with the former page standing beside him, the two stated that what had happened between them was nobody's business but their own.Studds was re-elected five more terms after the censure.

I have many questions about the Foley case and am happy that low life is out . Ironically if he did commit a crime ,it is one that he helped usher in ; one about internet predators. Ironically the age of consent in D.C. is 16 . So had he actually had an affair with these interns then he would face ethics issues in Congress but probably no legal issues. But the bill he helped make law has the consentual age at 18 .

Besides his blackberry IM's did any of these past pages report any inappropriate conduct from Foley ? So far it's only alleged he engaged in sexual banter (if you read ABC's alleged transcript) with a *former* page online and sent some *benign* IMs. Who is this former page ? If he is over 18 then why has he not been identified ? Why did he and/or the press wait until a month before the elections to reveal this ? From what I have read ,the page was interviewed about this a year or more ago by the St. Petersburg Times.

You would think the Dems. would be championing someone like Mark Foley .Foley is one of the most pro-gay Republicans in Congress. He has a long, consistent record of supporting gay issues like domestic-partnership benefits, anti-discrimination legislation, and AIDS funding. His office maintains a policy of nondiscrimination on the basis of sexual orientation.

Foley deserved to leave Congress in disgrace but if a question exists about the Republican leadership for covering up the activity of one of their own then the Democrats and the press ,who also knew and withheld the information until they could make the most political advantage from it, are equally to blame for allowing Foley at least an extra year to prey on young pages.

Itsdb rated this answer Excellent or Above Average Answer
MarySusan rated this answer Poor or Incomplete Answer

Question/Answer
CeeBee2 asked on 09/29/06 - 2nd try - Newsweek cover for all but the U.S.

(Sorry - had forgotten to remove the quote marks in the tagging)

Newsweek Cover Stories

tomder55 answered on 09/29/06:

I'm sure now that NATO is taking command of operations things will be much better. (guffaw "caveats" placed on forces from Germany, Norway, Belgium and other nations variously keep them away from the south, away from heavy combat zones, or forbidding them from going out at night.).

Maybe you did not hear about that since the press was more concerned if the Presidents of Pakistan and Afghanistan didn't shake hands at a press conference (I wonder if a handshake is even common in their shared cultures? ).What did not make much news coming out of their dinner meeting with President Bush is that Musharraf and Karzai agreed to hold a grand loya jirga of all Pashtun tribal elders in an attempt to coordinate efforts to resolve the Taliban problem.

During spring and summer offensives over 1000 Taliban were killed . Now as fall approaches and the weather is less condusive to operations they will retreat to their "safe havens" and lick their wounds .In the spring they will come back to sustain more casualties.

paraclete rated this answer Excellent or Above Average Answer
ETWolverine rated this answer Excellent or Above Average Answer
labman rated this answer Excellent or Above Average Answer
CeeBee2 rated this answer Excellent or Above Average Answer

Question/Answer
Itsdb asked on 09/28/06 - House Approves Bill on Terror Detainees

By ANNE PLUMMER FLAHERTY Associated Press Writer
2006 The Associated Press

WASHINGTON The House approved legislation Wednesday giving the Bush administration authority to interrogate and prosecute terrorism detainees, moving President Bush to the edge of a pre-election victory with a key piece of his anti-terror plan.

The mostly party-line 253-168 vote in the Republican-run House prompted bitter charges afterward by House Speaker Dennis Hastert, R-Ill., that opposition Democrats were coddling terrorists, perhaps foreshadowing campaign attack ads to come. Democrats responded that the GOP leader was trying to provoke fear...

The legislation would establish a military court system to prosecute terror suspects, a response to the Supreme Court ruling in June that Congress' blessing was necessary. While the bill would grant defendants more legal rights than they had under the administration's old system, it nevertheless would eliminate rights usually granted in civilian and military courts.

The measure also provides extensive definitions of war crimes such as torture, rape and biological experiments _ but gives Bush broad authority to decide which other techniques U.S. interrogators can legally use. The provisions are intended to protect CIA interrogators from being prosecuted for war crimes.

For nearly two weeks, the GOP has been embarrassed as the White House and rebellious Republican senators have fought publicly over whether Bush's plan would give him too much authority. But they struck a compromise last Thursday, and Republicans are hoping approval will bolster their effort to cast themselves as strong on national security, a marquee issue this election year...

...House Democratic leader Nancy Pelosi said Democrats feared the House-passed measure could endanger U.S. soldiers by encouraging other countries to limit the rights of captured American troops. She said the bill would be vulnerable to being overturned by the Supreme Court.

"Speaker Hastert's false and inflammatory rhetoric is yet another desperate attempt to mislead the American people and provoke fear," said Pelosi, D-Calif., adding that Democrats "have an unshakable commitment to catching, convicting and punishing terrorists who attack Americans."

~~~~~~~~~~~~~~~~~~~~~~~~~~~~~~~~~~~~~~~~~~~~~~~~~~~~~~

"Democrats responded that the GOP leader was trying to provoke fear." What are the Democrats doing, instilling confidence?

    Kucinich: "This bill is everything we don't believe in."

    Levin: The bill would be "used by our terrorist enemies as evidence of U.S. hypocrisy when it comes to proclamations of human rights."

    Leahy: This is un-American, this is unconstitutional, this is contrary to American interests, this is not what a great and good and powerful nation should be doing.

    Hoyer: When our moral standing is eroded, our international credibility is diminished as well.

    Tauscher: "Keep in mind, the president's original plan has not given us the ability to prosecute anyone, because they got it wrong...and because they blew it, and are about to blow it again, we're still not going to be able to bring the 9/11 perpetrators to justice."

    Pelosi: Its been five years since 9/11, yet not one person who has been directly responsible for 9/11 has been prosecuted and punished. Theres something wrong with this picture. And this bill today, because it does violence to the Constitution of the United States, also could produce convictions that may well be overturned...

    This bill does not help us achieve the goal of bringing anyone to trial. It is badly flawed. It threatens the safety of our troops, our ability to prosecute terrorists effectively, our ability to protect the American people, and to honor our oath of office to protect and defend the Constitution...

    Perhaps most distressing, this bill could very well boomerang on us putting American troops in danger...


Isn't that 'provoking fear'?

Pelosi said Democrats "have an unshakable commitment to catching, convicting and punishing terrorists who attack Americans." Is that the Democrat plan for protecting us I've been hearing about, punishing those who attack us? What's their plan for preventing those attacks?

tomder55 answered on 09/28/06:

I just got finished listening to Karl Levin and Harry Reid making the same talking points. I would like any of these Senators to give me an example where our troops were treated in a manner consistent with their fairy-tale version of war in the last 50 years.

How may I ask can our troops have been treated any worse than McCain was in the Hanoi Hilton ? Can anyone account for Navy Lt. Robert J. Dwyer ? How about Michael Scott Speicher ? They were all captured during Desert Storm . Everyone knows Speicher was a POW but he never returned .Where is he or his remains ? Paul Gallagher ?Early in Desert Storm Coalition POWs began being showed on Iraqi television and gave coerced "peace"
statements.What methods did Saddam's thugs use to get these statements ?

How about the USS Pueblo ? They were starved and regularly tortured while in North Korean custody. This treatment became worse when the North Koreans realized that crewmen were secretly giving them the finger in staged propaganda photos. Speaking of Korea ;how were American POWs treated in that conflict ? Of the 7,190 who were captured approximately 3000 died in captivity, a mortality of 43%, largely of starvation .The ones that survived were subject to mass indoctrination in anti-American propaganda to regurgitate anti-American propaganda in signed statements and public broadcasts.Any attempts by officers to take command, forcing men to cooperate in their common interest, were broken up by Communist guards, unless the officers were willing to become collaborators.US authorities documented 66 US personnel kept against their will by Communist forces after the end of war.

As we know ,with the exception of Jessica Lynch who's captures didn't wait around long enough to brutalize her too much , Americans captured in this war ....well we are likely to find their headless charred bodies . I wonder how better they would 've been treated if prisoners at Gitmo weren't getting lemon-glazed chicken.

ETWolverine rated this answer Excellent or Above Average Answer
Itsdb rated this answer Excellent or Above Average Answer

Question/Answer
ETWolverine asked on 09/28/06 - TFAa

Trans-Fatty Asses.

The NY Board of Health has begun the process of making the use of Trans-Fatty Acids illegal for use in restaurants. They unanimously passed a proposal banning the use of TFAs in restaurants in New York, and the proposal will be brought before lawmakers shortly.

All Hail the Nanny State of New York.

The basic concepts of free choice and personal responsibility are becoming endangered species in New York. And the majority of New Yorkers are (rather stupidly) going along with it. First came the Bloomberg smoking ban. Then Bloomberg's personal attack against gun ownership. Then there is the little-known "diabetes database" which is an effort to track diabetics and make sure they take their meds --- whether they want to or not. (Never mind that Diabetes isn't a communicable disease, and the only person hurt by not taking their meds is the diabedic.) And NYers seem to support these ideas.

What happened to the idea of taking responsibility for our own decisions. If I want to eat fatty fries or a Krispy Kreme doughnut, that's my choice. If I want to drink in a bar where smoking is permitted, that's my choice. If I refuse to take my medications for my non-communicable disease, that's MY CHOICE. And I have to live with the consequences of that choice. But why are so many people in favor of government regulation of these choices. (Notably, these are most often the same people who want the government to "stay out of their bedrooms" and "off their bodies" on the abortion and gay rights issues.)

What really gets me is that this ban on TFAs will have virtually no effect on the health of Americans in general or NYers in specific. It turns out that only about 2% of average caloric intake is from TFAs. And it doesn't address the issue of overall caloric intake, which is the real cause of obesity and a much greater contributor to heart disease than TFAs.

In fact, I suspect that if people believe that their fast food is healthier because of the removal of TFAs, they will actually eat more of it, and as a result they will get fatter and more prone to heart disease. Eliminating one specific (rather low-quantity) ingredient from our diets is NOT going to change our overall health. All it will do is make food taste different.

So in fact, the ban itself, while it might sound good in theory, is a useless jesture that serves only to increase the power of the Nanny State over the public... without any real benefit to the public.

So how do we fight this movement toward Nanny-Statehood? How do we get people to start taking responsibility for themselves instead of relying on corporations' warning lables and government agencies' regulations to keep them safe? How do we stop this trend toward Socialism and Big Government, and litigation/punishment of corporations for giving people what they want?

What's next? A ban on sugar? Meatless burgers? A soda prohibition?

Elliot

tomder55 answered on 09/28/06:

trans fats are bad news and should be avoided . but if I'm going to order a couple of BK Enormous Omelet Sandwich's every day then I don't suppose my transfat intake would matter because my food habit/addiction will kill me anyway.

Many chains and brands are just following the marketting trends and making their processed foods healthier sans the transfats . The thing is that it is not really needed to make the krispykreme donut creamy and as the public is educated on the dangers of transfat intake then they have been making the decision to restrict consumption on their own .So giving the consumer the choice of a healthier product is plain good business.

NY State is rediculous. We will be the next state to sue for the purpose of padding the coffers while we claim to be looking out for the folks ....joining the idiot running Kalifornia who will put the final nail in the domestic auto industries coffin . As the Nanny says :"that's not acceptable !"

ETWolverine rated this answer Excellent or Above Average Answer
Itsdb rated this answer Excellent or Above Average Answer
paraclete rated this answer Excellent or Above Average Answer

Question/Answer
excon asked on 09/27/06 - Circular logic


Hello Wingers:

Here's how Bush says he's winning in Iraq.

Factually, Iraq didn't have any terrorists to begin with. Now there are plenty. Bush captured about 15,000 of 'em so far. And, the more he creates, the more he captures - giving credence to his claim that he's winning the war.

excon

PS> That's pronounced cir-QUE-ler in Bushspeak.

tomder55 answered on 09/28/06:

funny thing ;If I listen to what al Qaeda says in their version of the NIE I would believe that they are losing . The letter throws our conventional wisdom on it's head . It is Zarqawi who is upbraided for losing the "hearts and minds" of Iraqis; it is Zarqawi who is criticized for not reaching out Iraqi allies. It is Zarqawi who is blamed for plunging al-Qaeda into "weakness" in Iraq. It is Iraq as viewed, not through the pages of the New York Slimes, but through the prism of al-Qaeda.

But all this is old news .Iraq has morphed from even the ancient NIE (it has been a long time since it was published in the spring) . Mookie al-sadr is the current threat there ;and as we know .... UIran calls his shots.

excon rated this answer Excellent or Above Average Answer
paraclete rated this answer Excellent or Above Average Answer

Question/Answer
paraclete asked on 09/26/06 - Fighting the war you can afford?

Bush releases Iraq intel papers


September 27, 2006 - 8:26AM

A declassified government intelligence report says the war in Iraq has become a "cause celebre" for Islamic extremists, breeding deep resentment of the U.S. that is likely to get worse before it gets better.

In the bleak report, released on President Bush's orders, the nation's most veteran analysts conclude that despite serious damage to the leadership of al-Qaida, the threat from Islamic extremists has spread both in numbers and in geographic reach.

"If this trend continues, threats to U.S. interests at home and abroad will become more diverse, leading to increasing attacks worldwide," the document says. "The confluence of shared purpose and dispersed actors will make it harder to find and undermine jihadist groups."

Bush ordered a declassified version of the classified report released after several days of criticism sparked by portions that were leaked. Asked about those Tuesday, Bush said critics who believe the Iraq war has worsened terrorism are naive and mistaken.

The intelligence assessment, completed in April, has stirred a heated election-season argument over the course of U.S. national security in the years following the 2003 U.S.-led invasion of Iraq.

Bush and his top advisers had said the broad assessment on global terrorism supported their arguments that the world is safer. But more than three pages of stark judgments warning about the spread of terrorism contrasted with the administration's glass-half-full declarations.

The report said:

- The increased role of Iraqis in opposing al-Qaida in Iraq might lead the terror group's veteran foreign fighters to focus their efforts outside the country.

- While Iran and Syria are the most active state sponsors of terror, many other countries will be unable to prevent their resources from being exploited by terrorists.

- The underlying factors that are fueling the spread of the extremist Muslim movement outweigh its vulnerabilities. These factors are entrenched grievances and a slow pace of reform in home countries, rising anti-U.S. sentiment and the Iraq war.

- Groups "of all stripes" will increasingly use the Internet to communicate, train, recruit and obtain support.

The assessment also lays out weaknesses of the movement that analysts say must be exploited if its spread is to be slowed. For instance, they note that extremists want to see the establishment of strict Islamic governments in the Arab world - a development they say would be unpopular with most Muslims.

"Exposing the religious and political straitjacket that is implied by the jihadists' propaganda would help to divide them from the audiences they seek to persuade," the report says.

It also argues that the loss of key leaders - Osama bin Laden, Ayman al-Zawahri and Abu Musab al-Zarqawi - in "rapid succession" would probably cause the group to fracture.

Al-Zarqawi was killed in June, but the top two al-Qaida leaders have remained elusive for years.

tomder55 answered on 09/27/06:

Please read the question I just posted . The NIE is neither an endorsement or a critique of the policy . It lays out a balanced assessment of raw data . But the parts that have4 been declassified can be found here . Cherry pick to your heart's content.

paraclete rated this answer Excellent or Above Average Answer

Question/Answer
twelfth_imam asked on 09/26/06 - Free Press? Is the US media under the control of the White House?

Newsweek features 'Losing Afghanistan' in international edition, celebrity photographer in U.S.

Muriel Kane - Raw Story research director
Published: Monday September 25, 2006

The United States edition of the October 2, 2006 issue of Newsweek features a radically different cover story from its International counterparts, RAW STORY has learned.

The cover of International editions, aimed at Europe, Asia, and Latin America, displays in large letters the title "LOSING AFGHANISTAN," along with an arresting photograph of an armed jihadi.

The cover of the United States edition, in contrast, is dedicated to celebrity photographer Annie Leibovitz and is demurely captioned "My Life in Pictures."

The International cover story begins:

"You don't have to drive very far from Kabul these days to find the Taliban. In Ghazni province's Andar district, just over a two-hour trip from the capital on the main southern highway, a thin young man, dressed in brown and wearing a white prayer cap, stands by the roadside waiting for two NEWSWEEK correspondents. It is midday on the central Afghan plains, far from the jihadist-infested mountains to the east and west. Without speaking, the sentinel guides his visitors along a sandy horse trail toward a mud-brick village within sight of the highway. As they get closer a young Taliban fighter carrying a walkie-talkie and an AK-47 rifle pops out from behind a tree. He is manning an improvised explosive device, he explains, in case Afghan or U.S. troops try to enter the village."

The United Story cover story begins:

"Annie Leibovitz is tired and nursing a cold, and she' s just flown back to New York on the red-eye from Los Angeles, where she spent two days shooting Angelina Jolie for Vogue. Like so many of her photo sessions, there was nothing simple about it. 'I talked with Angelina before the shoot,' says Leibovitz, who's famous for her preparation. 'She felt like she was coming back from having the baby and she felt very sexy and ready to go.' ... There were 50 people on the set, and racks of clothes from the New York spring collections to be tried and styled."

The story aimed at the United States then goes on to discuss the difficulties Leibovitz had in photographing Tom Cruise and Katie Holmes' infant. The International story continues with difficulties of a very different kind:

"In Ghazni and in six provinces to the south, and in other hot spots to the east, Karzai's government barely exists outside district towns. Hard-core Taliban forces have filled the void by infiltrating from the relatively lawless tribal areas of Pakistan where they had fled at the end of 2001. Once back inside Afghanistan these committed jihadist commanders and fighters, aided by key sympathizers who had remained behind, have raised hundreds, if not thousands, of new, local recruits, many for pay. They feed on the people's disillusion with the lack of economic progress, equity and stability that Karzai's government, NATO, Washington and the international community had promised.

"NATO officials say the Taliban seems to be flush with cash, thanks to the guerrillas' alliance with prosperous opium traffickers. The fighters are paid more than $5 a daygood money in Afghanistan, and at least twice what the new Afghan National Army's 30,000 soldiers receive."

=====

What / who is controlling what Americans are allowed to read in the land of the free?

Sensible answers please.


tomder55 answered on 09/27/06:

It is really not news that the weekly news magazines have different covers that cater to the region they are selling to. But the content I doubt is decided by the same marketting concerns .

Certainly with investigative reporters like Michael Isikoff on their staff who writes one hit piece after another against the Bush administration ,you really can't seriously believe that they are being censored ?

And clearly the article you refer to that is the cover of the International edition is a main story in the American addition .

twelfth_imam rated this answer Excellent or Above Average Answer

Question/Answer
paraclete asked on 09/26/06 - I'm just quaking in my boots

So an Islamist shouts and we are all supposed to be afraid, well I not afraid, I say why waste the expense of bringing him here just get the job done now. Out of his own words he has condemned himself?

Shooting suspect warns Australia over extradition
Sarah Smiles, and Ed O'Loughlin in Beirut
September 26, 2006

AN AL-QAEDA supporter about to be extradited from Lebanon has warned that Australia "will suffer" if he is deported.

Speaking from a jail in Beirut, Saleh Jamal, who professes admiration for Osama bin Laden, said Australia was an illegitimate state that should be ruled by Muslims. He is wanted in NSW after Lakemba police station was shot up in 1998.

NSW police officers had been preparing to collect him from Beirut - where he had served a jail sentence for a terrorism-related offence - in July when Israeli military aircraft bombed Beirut's international airport.

With the Hezbollah-Israeli conflict over and the airport open, NSW police say they will go ahead with the extradition.

It is understood Jamal will be handed to Australian police at the prison where he is held and taken to the airport. He will be put on a flight to Dubai, from where he will be flown to Australia.

The Lebanese prosecutor's office said that "in principle" he would be deported within 10 days, once Australian security officials arrived to collect him from Roumieh prison, near Beirut.

Other Lebanese security sources said Jamal could be extradited as soon as today.

Interviewed in prison this month, Jamal railed against the Australian Government and said the country would "suffer the consequences" if he was extradited. "I wouldn't mind going back to Australia. They're the ones that will suffer the consequences, not me."

But he conceded: "I've got 125 years probably waiting for me."

Surrounded by six armed guards, Jamal expressed bitterness that he had been left behind in jail during the war.

"I heard on the news that 25,000 Australians [in Lebanon] were contacted to be evacuated. They never came to see me. At least they could have asked if I needed a bullet-proof jacket."

Jamal, from Sydney, fled from Australia to Lebanon on a false passport in early 2004, while awaiting trial over the Lakemba police station shooting. He was arrested in Beirut soon after, and was convicted in Beirut's Military Court of planning subversive attacks against the state.

Beirut's Supreme Military Court cut his sentence in April but the authorities kept him detained at Australia's request, a Lebanese military court official said. There is no extradition treaty between Lebanon and Australia, but a spokeswoman for NSW police said it had received "formal written advice" from Lebanese authorities approving his extradition.

Lebanese authorities accused Jamal of being linked to a bombing in the Syrian capital, Damascus, in 2004. He was also recorded as having visited the Ain el-Helweh Palestinian camp in south Lebanon, a stronghold of Sunni Muslim extremists.

The Jordanian-born Jamal denied any involvement in the Damascus bombing and said he visited Ain el-Helweh to get forged Palestinian refugee travel documents so that he could escape to Europe. He denied involvement in the Lakemba attack.

tomder55 answered on 09/26/06:

yeah do you really want him in your prison system infecting the place and recruiting more into the jihad ? I say if he is in a Lebanese jail let him linger there . I'm sure Aussie prisons are closer to Club Gitmo then the Lebanese prisons are .

Itsdb rated this answer Excellent or Above Average Answer
paraclete rated this answer Excellent or Above Average Answer

Question/Answer
twelfth_imam asked on 09/25/06 - Will the US still be in Iraq in 2050?

Army considers more combat units for Iraq

By Rowan Scarborough
THE WASHINGTON TIMES
September 25, 2006

The Army is studying whether to add more combat units to the rotation plan for Iraq and is considering accelerating the deployments for some brigades to meet a top commander's decision to keep more than 140,000 troops in the country through at least the spring of 2007, Pentagon officials say.
Rather than planning for a big drawdown of 30,000 Army soldiers and Marines this year to a level of 100,000, as field commanders had expected, the two services are now trying to figure out how to keep the equivalent of two extra divisions, or 40,000 troops, in Iraq.
Army Gen. John Abizaid, the top commander in the region, said last week he needed to maintain the higher-than-expected level because of increased sectarian violence in greater Baghdad between warring Sunni and Shi'ite Muslims.
The Army has met Gen. Abizaid's immediate need for more forces by delaying the departure of a Stryker armored vehicle brigade to Alaska and by calling in a fast-reaction brigade combat team from Kuwait. But a longer-term solution may require the Army to look at adding more units to the rotation mix.
"It may accelerate the pace of deployments, or it may mean looking at calling up additional units," said a Pentagon official who asked not to be named.
That option may become reality in November, when the Pentagon is expected to identify units that will go to Iraq next year. Currently, Army units deploy for about a year, then spend one year at their home base before going back to Iraq or Afghanistan. The Marine Corps, which patrols restive Anbar Province west of Baghdad, rotates two expeditionary forces every seven months.
The Army is facing more demand for troops at a time when military analysts say it is nearly stressed to the breaking point.
Non-deployed combat brigades are experiencing low readiness ratings due mostly to a lack of usable weapons and equipment. The wear and tear in Iraq is ruining M1A1 tanks, Bradley fighting vehicles, Humvee vehicles and other equipment at such a fast pace that the Army has neither the money nor the industrial base to replace them. A unit's functioning weapons systems are left in Iraq for use by replacement soldiers, leaving the stateside brigade well below the highest combat rating, according to Army officials and retired officers.
Gen. Peter Schoomaker, Army chief of staff, this summer asked Congress for nearly $50 billion over three years to replace broken equipment in a process known as "resetting" the force.
"We have inadequate Army and Marine Corps combat power to sustain this level of deployment," said retired Army Gen. Barry R. McCaffrey, a highly decorated Vietnam combatant who led the 24th Infantry Division in Desert Storm.
Gen. McCaffrey, who has traveled to Iraq in his role as a West Point professor, said the Army needs an immediate infusion of 80,000 new soldiers added to the active force of about 500,000.
"Not since World War II have we asked the Army and Marine Corps to operate at such a high intensity of deployments with such an under-resourced force," Gen. McCaffrey said.
Defense Secretary Donald H. Rumsfeld has rejected calls from Gen. McCaffrey and other retired generals to increase what is referred to as the authorized end strength approved by Congress each year. Instead, he opted for authorizing the Army a temporary increase of 482,000 to 512,000 active-duty soldiers.
Paul Boyce, an Army spokesman at the Pentagon, said, "The Army has been planning rotation of forces through the year 2007 and is fully prepared to maintain the commitment to press the fight against the enemy while also giving the soldiers as much predictability on future missions as possible. We will meet any need the American public asks."
Gen. McCaffrey said he considers the ongoing battle for Baghdad a "tipping point" which would decide the war's outcome. He applauded Gen. Abizaid for upping the force level.
"It clarified the situation," he said. "Abizaid wants to win. Ambassador [Zalmay] Khalilzad wants to win. What they are saying is [Prime Minister Nouri al-Maliki's] government is in a fragile state ... I think what they are saying is we can't allow this to come apart on us."

tomder55 answered on 09/25/06:

today the Iraqi gvt announced a new committee being formed to make amendments to their constitution which everyone agrees is needed.

I happen to agree that the overall size of the US military is not sufficient for our long term committment to the war on jihadistan ;Iraq being just a theater . I think as you point out the concensus among the military is clear on this subject .Researchers at conservative think tanks like the American Enterprise Institute and the Heritage Foundation also agree .I have cited Frank Gaffney's 'War Footing ' many times to illustrate what the country needs to do.

Todays Army is at roughly 1/2 million .I think it has to grow by 250,000 and that would be a minimum if you consider other possible theatres of deployment depending on how the geo-politics of the world turns .

The military would like to continue it's transformational plan but they should not be asked to choose between that or additional troops . The country has the resources for both.Recruiting and training takes time so there is a danger in procrastination .The sooner the better in my book. Rumsfeld if given a choice has argued for the transformational program . Rumsfeld is a great Secretary of Defense, one of the best I've ever seen, but he's dead wrong on this issue.And it's a bogus excuse that we can't do it with volunteers; our volunteer force was almost twice the size 15 years ago . What we are witnessing is the shortsightedness of the demobilization following the end of the cold war so we could collect that phoney "peace dividend" .Some of it was justified of course but I think we trimmed back way too much in the Clintoid feel good days.

It is not true that we do not have the troops needed to successfully gain our objectives in Iraq but it is also true that it will require a longer stay than was anticipated . Then again .....how long have our troops been in harms way patrolling the DMZ on the Korean peninsula ?

twelfth_imam rated this answer Excellent or Above Average Answer

Question/Answer
ETWolverine asked on 09/25/06 - I am probably stepping over the PC line here...

but this is too good to pass up.

The Hebrew word for "monkey" is "kof".
The Hebrew word for "cloud" is "anan"

Does that make Kofi Anan "The Monkey of the Clouds"?

Hey, I didn't make up the language. Don't shoot the messenger.

Elliot

tomder55 answered on 09/25/06:




ETWolverine rated this answer Excellent or Above Average Answer
Itsdb rated this answer Excellent or Above Average Answer

Question/Answer
excon asked on 09/24/06 - Terrorisim....


Hello again, wrongwingers:

Our intelligence services say the war in Iraq is causing terrorisim to SPREAD, instead of DECLINE!

What's up with that? Don't those guys watch FOX news?

excon

tomder55 answered on 09/25/06:

I would have to say off the bat that there has been significantly fewer terror attacks on the US since Bush engaged the jihadists .I bet the report also asserts that had we left Iraq alone Saddam would voluntarily have opened his doors to inspectors and all those attacks on embassies and navy ships would have stopped.Fighting back creates more terrorists has been the anti-war mantra since 9-11 as the Arizona 9-11 memorial illustrates . You dont win battles of terrorism with more battles.

My first question is :Is Karen DeYoung at the Washington Compost accurately reporting the content of the NIE report ,and if in an unlikely event that she is; is there are solid basis for the conclusion ?

The points raised are :

terrorism is becoming more decentralized

successful recruting of terrorists is on the rise

terrorists are using the Iraq war as the centerpiece of their recruiting campaigns

the sitation in Iraq has worsened the U.S. position with respect to fighting terrorism



It is of course an undeniable fact that terrorism and recruitment into the worldwide jihad was on the rise long before the Iraq invasion . DeYoung cites John Negroponte as saying "My colleagues and I still view the global jihadist terrorist movement, which emerged from the Afghan-Soviet conflict in the 1980s but is today inspired and led by al-Qaeda, as the preeminent threat to our citizens, homeland interests and friends." She also cites a statement by CIA Director Michael Hayden that "threats to the U.S. at home and abroad will become more diverse and that could lead to increasing attacks worldwide." Both statements leads me to believe they are linking the first two points but not the last two.

I think in all likelyhood that recruitment is on the rise ;a trend as I said started long before Iraq. I will even conceed that the jihadists are using the Iraq war as part of their recruitment propoganda . Not that they needed it . They were doing very well before citing our support for Israel ;our being stationed in Saudia Arabia. No doubt they have made some hay by citing our 'crusade' in Afghanistan which would be the focus of the anti-war meme at this point had we not given them the option to protest Iraq. Is the NIE saying that the jihadists would not have an effective recruitment campaign ongoing had we not invaded Iraq ? I'd like to see that case documented and debated .

When I can ,I hope to read the document or whatever part of it gets declassified as I'm sure DeYoung has not .All she is doing is relating the spin of the document that the leaker provided her.[the report was complete in April but the leaker decided to wait until the height of the election campaign ....timing is everything..the White House says that the stories in the Post are "not representative of the complete document."]

I would be interested to see if the NIE mentions that the one meglomaniac who was very active in supporting terrorism AND had experience in the utilization of wmd to terrorize citizens is now out of business. I would have thought that the starving 500,000 Iraqis to death in the 90's would have been a much more effective recruiting tape.
( but wait ....if I listen to the meme ;which I don't...then I have to believe that the same intelligence network that could not predict the rise and growing threat of al-qaeda ;and consistently ,along with the rest of the world's intel networks got the wmd issue of Iraq COMPLETELY wrong .... that their assessments are now rock solid indesputable . )

I will also be interested to see if they speculate as to how the effect our withdrawal from Iraq would affect the jihadist recruiting . As I understand the facts ,our retreats from Lebanon ,Somalia ,and the Ruskie defeat in Afghanistan enboldened the jihadists .But perhaps out intel agencies now buy into the OBL mantra that if we would just leave then all would be well in the world . We could sit down with the jihadists at the campfire cooking S'mores and sing kum-by-ya. We would no longer be the crusading evil infidels.

excon rated this answer Excellent or Above Average Answer
Itsdb rated this answer Excellent or Above Average Answer

Question/Answer
paraclete asked on 09/24/06 - After Afganistan there was Iraq, and now there is Lebanon?

Have you aksed yourself why it is these tinpot little islamic states give rise to terrorist movements and ultimastely to direct confrontation with the world?

I think I can tell you, it's called the United Nations which allows itself to be dictated to


Half a million rally in support of Hezbollah


September 24, 2006

HEZBOLLAH leader Sayyed Hassan Nasrallah rejected international calls to disarm his Lebanese guerillas and told 500,000 supporters at a "victory" rally they still had more than 20,000 rockets after a month of war with Israel.

Speaking to a sea of followers at a "divine victory" rally in south Beirut on Friday, Nasrallah said Hezbollah had emerged stronger from a conflict in which Israel declared it had destroyed most of the Shiite Muslim group's arsenal.

In his first public appearance since the war broke out in July, Nasrallah told hundreds of thousands of cheering supporters that Hezbollah had "more than 20,000 rockets" and that it had "recovered all its organisational and military capabilities . . . it is stronger than it was before July 12".

Nasrallah's estimate of Hezbollah's retained arsenal is five times greater than the total number it fired into Israel during the war, and higher than any previous figure he has given.

He also warned the United Nations peacekeepers not to seek confrontation with Hezbollah.

"Your mission is not to spy on Hezbollah or to disarm the resistance," he told a crowd packed into the Shiite Muslim suburbs which were heavily bombed in the 34-day war.

He said measures to stop Hezbollah rearming, including international forces patrolling the Lebanese coast and tighter security on the border with Lebanon, would have little impact.

The huge turnout in a country of 4 million was a gesture of defiance to Israel but also marked a challenge to the US-backed Lebanese Government of Prime Minister Fouad Siniora.

Nasrallah called for a change of government in Lebanon and slammed Arab leaders for failing to defend the Lebanese people.

"The building of an able, just and strong state starts first with the establishment of a national unity government. This is our new project that we will work for with all our force in the coming stage."

The rally had been expected to coincide with the final withdrawal of Israeli troops from the south, but Israel's army chief said on Wednesday the pullout might take a few more days.

Israeli forces have been gradually leaving territory they captured in fighting that began after Hezbollah guerillas seized two Israeli soldiers in a cross-border raid.

Israel and Hezbollah have both declared themselves victors in the war which killed nearly 1200 people in Lebanon, mostly civilians, and 157 Israelis, mainly soldiers.

Thousands had walked to the rally from Shiite villages in south Lebanon battered by Israel's bombardment and invasion.

The Beirut crowds carried pictures of Nasrallah and yellow Hezbollah flags bearing the message "Here we are Nasrallah".

Many wore yellow T-shirts and chanted pro-Hezbollah slogans. Some said they were there not only to celebrate but also simply to see the charismatic Nasrallah.

Associated Press

tomder55 answered on 09/24/06:

He also warned the United Nations peacekeepers not to seek confrontation with Hezbollah.

that is the one thing we can be sure of that will not happen.

Hugo Chavez in his rant at the UN this week suggested that the UN should be located somewhere else . It was the one thing he said that I can agree completly with. It should be located at one of it's success stories. Might I suggest Kigali ?

paraclete rated this answer Excellent or Above Average Answer

Question/Answer
paraclete asked on 09/23/06 - Now wouldn't that be a turn up for the books?

and it couldn't do the Replublicans any harm in the election either

French investigate report of bin Laden's 'death'

September 23, 2006 - 8:25PM
Latest related coverage

* Pakistani officials doubt Osama death report

France's Defence Ministry said on Saturday it could not confirm a newspaper report quoting French secret services as saying al-Qaeda leader Osama bin Laden had died but would launch an inquiry into the leak of secret papers.

The Defence Ministry issued the statement after the French regional newspaper L'Est Republicain said Saudi Arabia was convinced bin Laden had died of typhoid in Pakistan last month.

"The information published this morning in the L'Est Republicain newspaper relating to the supposed death of Osama bin Laden cannot be confirmed," the Defence Ministry said.

"The Defence Minister (Michele Alliot-Marie) has asked that an inquiry be carried out to determine the origin of the leak that can be punished by criminal charges."

A US intelligence official said the report should be treated with caution and a senior Pakistan government official said Islamabad had not received any information from any foreign government that would corroborate the story.

The Saudi Arabian Interior Ministry spokesman was not available for comment and a Western diplomat in Riyadh said the diplomatic community had no such information.

"If anyone was in the picture, I doubt it would be Saudi intelligence. Even if Saudi Arabia had information, they'd pass it on to the United States, not France. It doesn't ring true," the diplomat said.

The French newspaper printed what it said was a copy of the report dated September 21 and said it was passed on to President Jacques Chirac and Prime Minister Dominique de Villepin on the same day.

"According to a usually reliable source, the Saudi services are now convinced that Osama bin Laden is dead," the document said.

"The information gathered by the Saudis indicates that the head of al Qaeda was a victim while he was in Pakistan on August 23, 2006, of a very serious case of typhoid which led to a partial paralysis of his internal organs."

The report, which was stamped with a "confidential defence" label and the initials of the French secret service, said Saudi Arabia first heard the information on September 4 and that it was waiting for more details before making an official announcement.

The US intelligence official, who declined to be named, said bin Laden had long been rumoured to be suffering from kidney ailments and is reported to have received dialysis.

"We have believed him to be in declining health for some time and there have been other rumours of his demise."

He said bin Laden had "minimal operational involvement at this time" in al-Qaeda.

Saudi-born bin Laden was based in Afghanistan until the Taliban government there was overthrown by US-backed forces in 2001. Since then, US and Pakistani officials have regularly said they believe he is hiding somewhere on the rugged border between Afghanistan and Pakistan.

His last videotaped message released was in late 2004, but several low quality audio tapes have been released this year.

Senior US intelligence figures have said bin Laden remains a significant inspirational figure but his death or capture could have a substantial impact in the war on terrorism.

They note, however, that the death of al Qaeda's leader in Iraq, Abu Musab al-Zarqawi, in June, has failed to lead to any letup in the violence there.

Reuters

tomder55 answered on 09/24/06:

I think his carcass resides in the rubble of some cave in Tora Bora. But if this report is true then it can only be a good thing. Movements rarely die with the death of the founder . Bin Laden's version of Mohammed's worldwide jihad will not die with him.

However ,if he was not blown to a pink mist then at least he did not die a martyr for his delusions. A more fitting symbol of where he would lead the world cannot exist than him succumbing to the trots because he denied himself ,and would deny the world of modernity and advancements like basic plumbing and waste disposal .

excon rated this answer Excellent or Above Average Answer
paraclete rated this answer Excellent or Above Average Answer

Question/Answer
HANK1 asked on 09/22/06 - ILLEGALS:



I've always contended that U. S. corporations run the country and not Congress. I heard this a.m. that 56% of new employment this year has gone to ILLEGALS. This fact knocked out many chances for school kids, both High School and College, to get jobs this past Summer and those on Unemployment. This also effected some real LAZY PEOPLE who decided to do a day's work. Some employers pay cash to these employees to protect their identity. By doing this, they dodge paying State and Federal taxes.

This is slave labor. This is also criminal activity because the employees are in this Country illegally and the employers are defrauding our government, i.e. You and I.

What can we do about it? (Please don't mention voting or writing letters. Congressmen have large wastebaskets)

* PLease let me know if I am wrong about any aspect of my post. Thanks.

HANK

tomder55 answered on 09/24/06:

Politicians are not, as a class, outstandingly evil or insane. For the most part theyre just ridiculous people

It is no bad thing that our politicians are fools. We mortals all famously are. And the theory of democracy is that we can rule ourselves. If exceptionally wise and able men were required to run our democratic system, wed have a lot of explaining to do to the other fools around the world, from Zimbabwe to North Korea, upon whom we are always urging democratic institutions. Anyway, the history of kingdoms, oligarchies, and dictatorships indicates that ordinary fools do a pretty good job in politics, comparatively. (P. J. O'Rourke )

HANK1 rated this answer Excellent or Above Average Answer

Question/Answer
paraclete asked on 09/21/06 - Diplomacy, the american way or?

Pakistan: U.S. threatened to bomb us back to Stone Age
POSTED: 7:44 p.m. EDT, September 21, 2006

WASHINGTON (AP) -- President Pervez Musharraf of Pakistan says the United States threatened to bomb his country back to the Stone Age after the 9/11 attacks if he did not help America's war on terror.

Musharraf says the threat was delivered by Richard Armitage, then the deputy secretary of state, to Musharraf's intelligence director, the Pakistani leader told CBS-TV's ඄ Minutes."

"The intelligence director told me that (Armitage) said, 'Be prepared to be bombed. Be prepared to go back to the Stone Age,' " Musharraf said in the interview to be shown Sunday on the CBS television network.

It was insulting, Musharraf said. "I think it was a very rude remark," he told reporter Steve Kroft.

But, Musharraf said he reacted responsibly. "One has to think and take actions in the interests of the nation and that is what I did," he said.

The White House and State Department declined to comment on the conversation.

Armitage told CNN on Thursday that he never threatened to bomb Pakistan, wouldn't say such a thing and didn't have the authority to do it. Armitage said he did have a tough message for Pakistan, saying the Muslim nation was either "with us or against us," according to CNN. Armitage said he didn't know how his message was recounted so differently to Musharraf.

In a speech in January 2002, four months after the attacks on the World Trade Center and the Pentagon, Musharraf gave a speech in which he clearly came down on the side of reform at home and opposition to Islamic fundamentalism.

Pakistan to this day is considered an ally of the United States in the struggle with militant groups. Sometimes, however, Pakistan appears reluctant to go after the Taliban, which controlled neighboring Afghanistan until 2001 and has intensified its insurgency in the southern part of the country in recent months.

Musharraf is scheduled to meet on Friday at the White House with President Bush and then see Bush again next week in a three-way meeting with President Hamid Karzai of Afghanistan.

Musharraf told ඄ Minutes" that Armitage's message was delivered with demands that he turn over Pakistan's border posts and bases for the U.S. military to use in the war against the Taliban in Afghanistan. Some demands were "ludicrous," such as a demand he suppress domestic expression of support for terrorism against the United States.

"If somebody is expressing views, we cannot curb the expression of views," Musharraf said.

tomder55 answered on 09/22/06:

Since Afghanistan is land locked we needed the cooperation of a number of countries to launch an invasion . At a minimum we needed overflight rights . There was no choice but for Pakistan to cooperate .

I do not know how much arm twisting was done . I do not particularily care . A case can be made that the Pakistani ISI and parts of the military infrastructure has and continues to support at very least the Taliban if not al-Qaeda .

Khalid Shaikh Mohammed ,the central planner(along with his nephew his nephew Ramzi Yousef) of both the WTC attack in 1993 and the 9-11 attacks was born in Baluchistan and operated out of Pakistan as well as Kandahar ,Afghanistan.

Omar Sheikh ;on the instructions of General Mahmoud Ahmed, the then head of Pakistan's ISI wired $100,000 before the 9/11 attacks to Mohammed Atta, the lead hijacker.When Ahmed was exposed by the Wall Street Journal as having sent the money to the hijackers, he was forced to "retire" by Musharraf; a show demonstrating his willingness to weed out radical elements from his government in support of the US .

The Times of India reported that The US authorities sought his removal after confirming the fact that $100,000 were wired to WTC hijacker Mohammed Atta from Pakistan by Omar Sheikh at the instance of Gen Mahmud.

Agence France-Presse picked up the story, reporting that A highly-placed government source told AFP that the damning link between the general and the transfer of funds to Atta was part of evidence which India has officially sent to the US.

The final report of the 9/11 Commission assured us that To date, the U.S. government has not been able to determine the origin of the money used for the 9/11 attacks.

ummmmmm.....yeah right.

Daniel Pearl was investigating the Pakistani connection in Karachi when he was kidnapped and beheaded .

When Bush declared that countries were either for or against us clearly he had nations like Pakistan and perhaps Saudi Arabi in mind. Musharraf as I have written before has to walk a tight-rope .He does not have complete control over all his gvt. The ISI has developed, since the early 1980s, into a parallel structure, a state within a state, with staff and informers estimated by some at 150,000. It wields enormous power over all aspects of government.

Musharraf has for the most part been a partner in the war ;and perhaps there has been some great rewards for doing so ;but it has not enhanced his life span.

paraclete rated this answer Excellent or Above Average Answer

Question/Answer
paraclete asked on 09/21/06 - Well that's alright then

Let's all bow down before the great Sir Richard. (I think I'll just go over to the corner and eat worms)

Branson urges Australia to sign Kyoto


September 22, 2006 - 8:29AM

British Businessman Sir Richard Branson says he will pressure the Australian government to sign the Kyoto agreement.

Sir Richard's Virgin Group has made a 10-year, $US3 billion ($A3.9 billion) commitment to renewable energy initiatives.

In an announcement in New York, Sir Richard committed Virgin to a green future, pledging that future dividends and proceeds from the sale of assets, including shares from Virgin's airline and train operations, would be invested in renewable energy initiatives.

These initiatives would be within Virgin's business and also involve further investments in new biofuel research and development production, distribution and other projects to tackle emissions related to global warming.

The tycoon told ABC radio the Australian government was dragging the chain on global warming.

He said he would move to make sure that next time around Australia would "get its act together" and sign the environmental agreement.

"The world has a very serious problem - it's ill and it's going to get iller unless we do something radical," Sir Richard said.

"I have got children and one day they will have grandchildren and I want to make sure the grow up in a similar world to the world that my parents and grandparents bequeathed myself and the rest of us."

Sir Richard said the Virgin group was effectively taking on the oil and coal companies though this pledge.

"We need everybody to join in to do their bit to tackle global warming," he said.

"People can do it at their homes by turning off their airconditioning units, they can try to get smaller cars, they can try to get hybrid cars they can try to persuade their local petrol station to supply ethanol ... there's so much that the individual can do, if we can get a global movement going then I think we can reverse this problem.

"What we have effectively got is a fire burning around the world which is getting stronger and stronger every year, and we have go to put that fire out," he said.

Sir Richard said he would make sure not a penny of the money was wasted.

"I think if you happen to be lucky enough in your lifetime to become a successful entrepreneur, extreme wealth goes with it and therefore extreme responsibility goes with it," he said.

"Capitalism has it's faults and one of it's faults is that a lot of wealth ends up in the hands of very few people, and those people have the responsibility to use that wealth constructively and for the benefit of society."

2006 AAP

Somehow I don't think Richard gets out of his air conditioned office at 30,000 feet very often or perhaps he has used his rocket plane to take up a commanding position in geostationary orbit above the Earth from which lofty eminence he pulls the strings of his puppets here on Earth. Don't you just love meglomaniacs?

tomder55 answered on 09/22/06:

In the best of all worlds Koyoto will cost hundreds of TRILLIONs of dollars for an admitted negligable impact.

How will his planes fly ? In the US we call Branson and his ilk 'limosine liberals' .

My favorite one is Bobby Kennedy Jr. and the whole Kennedy clan (which now includes Ahhhnold and his suing all the auto companies to fill Kalifornia's state coffers ).Bobby Jr is a hypocrite who rants ceaselessly about the environment but flys around in a private jet .

The Kennedys own Arctic Royalty Trust, which leases out land for oil drilling in five states. Much of the land was accumulated by convincing poor rural black farmers to give away their mineral rights, not knowing what it meant. The Kennedys set the operation up as a Royalty Trust to avoid paying income and corporate taxes on the profits. Family members who receive royalties including fat Teddy ;Bobby Jr, and Joe Jr.


The Kennedy clan has a huge compound on Cape Cod Island . There was a proposal to make 130 offshore windturbines that would've supplied the island and the surronding region with affordable ,clean ,renewable energy . The the shallow shoals of Nantucket Sound are perfect for such a project . But it would 've sacrificed some aesthetics ;it would've ruined the Kennedy's view .

So Bobby Kennedy Jr.(the heretic) found a new cause ;no longer the environmental attorney/activist ,he became a property rights defender !!! He will defend the property rights of the rich and famous with the same vigor that he helps indigenous tribes in Latin America and Canada successfully negotiate treaties protecting traditional homelands. How egalitarian can you get ?!!

'Greenpeace', 'Natural Resources Defense Council' (where Robert Kennedy works), 'World Wildlife Fund', 'Public Interest Research Group 'and 'Friends of the Earth' all support the project .But Bobby Kennedy Jr., for one, said he found "zero" irony in the fact that he had devoted himself to environmental advocacy and yet opposed the wind project on Cape Cod.

"There are appropriate places for everything," he said in a telephone interview. "You would not want a wind farm in Yosemite, and you wouldn't want one in Central Park.".

He loves wind energy but not in his backyard . Perhaps he would find less to object to if they were located in the Bronx.


Itsdb rated this answer Excellent or Above Average Answer
paraclete rated this answer Excellent or Above Average Answer

Question/Answer
paraclete asked on 09/21/06 - After a revisionist address from the Mad Mahdi let's have some true revisionism?

Who needs values? Let's have slaves, straight roads


Paul Munroe
September 21, 2006

WHILE John Howard and Kim Beazley make us reflect on Australian values, is it worth considering alternatives? We are being exhorted to embrace the values of "mateship" and "a fair go" and the ethos of a society which frankly ceased to exist after Robert Menzies.

Perhaps some lateral thinking is required. If we want to take up the values of a bygone age, there has to be a strong case for us to return to values espoused by another bygone society, the Romans.

Take roads. These guys really knew how to build roads. Straight as a die and built to last. In contrast, the Pacific Highway is a disgrace. For much of its length it's a winding, deathtrap of a semi-tarred goat track.

Let's rebuild it the Roman way: an eight lane, straight line superhighway between Sydney and Brisbane.

Of course, the Romans could put down these highly effective transport links in no time at all because of an inexhaustible supply of cheap labour: slaves. Let's reintroduce slavery as well.

There are plenty of lost souls in Centrelink offices looking for a challenge in life. Let's give them one. With a few 16-hour days being put in by our newly enabled, low-cost, lash-motivated workforce, Sydneysiders will be enjoying long weekends in Byron by Easter.

Taking that a step further, Centrelink staff will be quickly joining the ranks of the idle, so let's enslave them too. If we run low on a workforce, we can easily "recruit" more in the Roman way. We simply invade and conquer nearby lands and enslave the locals; New Zealand will do for a start.

A welcome side effect is that they'll no longer be able to thump us at rugby three times a year.

Roman entertainment has lots to commend it too. Let's put bums on the seats of the empty amphitheatres at Homebush and bring back the days of the Colosseum. We loved Russell Crowe as Maximus in Gladiator; let's watch him put George Piggins to the sword for real.

Roman religion has much to commend it too. Single deity modes of worship seem to be a cause of conflict and a reason to hate (nice one, Pope Benedict). Let's take the Roman view, where there are lots of gods and you simply choose the one you like best, erect a small shrine in the garden shed and go there and worship.

There are other reasons to love the Romans. Wearing a toga is pretty much an invitation to party these days. Moreover, the appointment of a horse as a senator has to be an improvement on much of the National Party.

Of course, there will be a few drawbacks. Our political masters handing over power to their children is a worry. Lead plumbing has a few unwelcome side effects, too.

Finally, does anyone understand the Roman numbering system? I suppose if the FII and the MIV are straight, fast and uncongested, will anyone care?
=============================================

all I can say about this is, the thinking is too small. There are large numbers of unemployed in Iraq, right, of course, right, SO! why can't they be used to recreate the Garden of Eden, the marshes should be about the right place, just a few well placed trees and displaced arabs and you have the job done and all for the small price of 500 Billion Dollars (US) Cheap?

tomder55 answered on 09/21/06:

so if you extract the good from an era gone by it stands to reason the by extention all the wrong of that time is incorporated .a little convoluted thinking if you ask me . Heck ;let's just scrap Western Culture .It was built on colonial imperialism and exploitation (as an Aussie Paul Munroe should see the irony ) .All the values then by extention are compromised because of the hypocracy .

I note that this was posted in a section called 'The Heckler ' where I guess the readership can post emails to the publishers and fulfill their vanity when it is seen in print. I see no real substance to this except a lame attempt at humor perhaps .Paul Munroe ought to consider reducing the amt of Fosters he consumes .

paraclete rated this answer Excellent or Above Average Answer

Question/Answer
ETWolverine asked on 09/20/06 - Truth, Justice... and the comics.

From Malard Filmore over the past two days:

9/19/06




And 9/20/06



Can you think of anything more funny than the truth? These two cartoons capture the essence of the two greatest problems in international relations today.

Comments?

tomder55 answered on 09/20/06:

I was listening to Chavez ramble on at the UN . Can't wait to go home and see some video of his stand up comic act. He was bring the house down .Jezze ...Clarabel the Clown is on the podium !I think at one point he held up a Chomsky book saying that everyone should read it because it describes how America works .

The UN has become a showcase for all the nut job pin headed dictators around the world. what a disgrace that NYC has to host these morons instead of rounding them all up and locking them up at Rikers Island !

Just found the Yahoo news flash about it

and here is the pix of him holding up the Chomsky book.



the comics certainly do depict real life .Note the person behind him who can't stop laughing !

ETWolverine rated this answer Excellent or Above Average Answer
Itsdb rated this answer Excellent or Above Average Answer

Question/Answer
ETWolverine asked on 09/19/06 - The Religion of Peace

I have a few questions about the Islam, the so-called "religion of peace".

The Muslim world has been protesting for the past few days over the comments made by Pope Benedict XVI regarding Islam being a nation founded on violence. These protests have been extremely violent, and have resulted in a number of deaths, including the murders of Christians, destruction of churches, threats of global war and assassination of the Pope, burning of embassies, etc.

My questions are:

1) Doesn't this violent style of protest prove the Popes point about Islam being a violent religion?

2) I have heard several people say that violence and terrorism doesn't really represent all Muslims, and that nothing could ever represent all of Islam because there are so many differing oppinions within Islam. Yet, it has become clear to me that Islam shows great unity in its protests of the Pope. Whether the protests are violent or not, the Muslim world is pretty well united in condemning the Pope's remarks. So why are they unable to create the same unity to condemn terrorism?

3) Why is it that even when a few Muslim groups condemn violence, they do so with half-hearted condemnations, excuses, and with blame for others for causing the violence? Why can't they just issue a blanket condemnation of violence as a method of promoting Islam, without any caveats, excuses or blame? Something simple and straightforward like "We condemn violence by Muslims against anyone." No blaming America or Israel for the anger on the Muslim street that causes the terrorism. No excuses that they were offended by some offhand remark or ation. Just a blanket condemnation of violence by Muslims against anyone, no matter who.

4) In Israel, there is a strong and active "peace-now" movement. In the USA, the peace movement is quite vocal (perhaps too vocal). Australia's peace movement has been very active over the past several years. Europe seems to be made up of nothing but the peace movement. The UK opposition political parties seem to be part of the peace movement. But where is the Muslim peace movement? Why are Islamic countries the only ones that don't have an active peace movement agitating for world peace? Why is the so-called "religion of peace" so lacking in peace activists and peace-protestors? If violence is not supported by the majority of Muslims and doesn't represent the beliefs of the majority of Muslims, why aren't the majority of Muslims protesting in the streets (peacefully) against the terrorists and the rioters? Should we take their silence as tacit support of the violence, disinterest in the issues, or simply fear of reprecussions? And if it is fear of reprecussions, doesn't that mean that Islam has been hijacked by the jihadists, and really is no longer a religion of peace, but of violence and fear?

These are just a few questions based on my observations of the "religion of peace".

Elliot

tomder55 answered on 09/19/06:

To judge if his words are being interpreted correctly they should be read fully . Here is his address.

He correctly points out that there is a contradiction between Mohammed writing that There is no compulsion in religion.;and the instruction to wage jihad .

He uses a dialogue between Emperor Manuel II of Byzantine and a Persian scholar to extact the quote he used for illustration. Manual was one of the last Emperors before the empire was completely overrun by the Ottoman Turks .

He essentially makes a plea for civilty ,dialogue and reason (obvious when speaking about Islam he's zero for three ).But looking at his larger message it is a smack down on modern Christianity as well and it's dehellenization as he calls it,in which Western secularization has chipped away the fusion of faith and reason, placing them in completely separate spheres.

In this sense theology rightly belongs in the university and within the wide-ranging dialogue of sciences, not merely as a historical discipline and one of the human sciences, but precisely as theology, as inquiry into the rationality of faith. Only thus do we become capable of that genuine dialogue of cultures and religions so urgently needed today.

Without the right balance between them humans are condemned to the "pathologies and life-threatening diseases associated with religion and reason".

Only by studying faith with reason will we find solutions to the problems of our time is his message ;a message that evidently was missed by even the most learned Muslim based on the reactions he is getting .The reaction to a critique of a violent religion is more violence instead of taking the opportunity to refute the charge. Case closed.This convergence has obviously a long ways to go in Islam.

Hope the people of Venezuella are paying close attention to their religious leader being burned in effigy ;and keeping in mind that their dictator is building an alliance with the jihadists masters.

You are right. The day that Islam rises up to condemn and reject the terrorism of the jihadists which has caused thousands of deaths in the ummah with the same vehemence they do when over-reacting to the words of the Pope or silly cartoons ,then an attempt at dialogue and improving relations can begin. Until such time we can dispel with the myth of the moderate Muslim majority .



ETWolverine rated this answer Excellent or Above Average Answer
Itsdb rated this answer Excellent or Above Average Answer

Question/Answer
paraclete asked on 09/18/06 - Yeh like that's likely?

Having shown it's true colours, Islam is on a collision course with the west

vow to 'conquer Rome' in Pope backlash

From correspondents in Basra, Iraq

September 19, 2006 03:25am
Article from: Agence France-Presse


POPE Benedict XVI's apology for remarks seen as critical of Islam failed to quell anger in the Muslim world overnight as Iraqis burned him in effigy and al-Qaeda in Iraq vowed to "smash the cross".

Despite appeals for calm from Islamic and Western leaders, protests were held from Indonesia to Iraq over the Pope's citing of a medieval text last week that criticised some teachings of the Prophet Mohammed as evil and inhuman.

The leader of the world's 1.1 billion Roman Catholics yesterday said he was deeply sorry for the offence caused by his remarks and the Vatican launched a diplomatic offensive to explain to Muslim countries his position on Islam.

A handful of Muslim groups welcomed the 79-year-old Pope's apology but it failed to stem the tide of anger in many Muslim nations.

Mohammed Habib of Egypt's opposition Muslim Brotherhood said they considered the apology a retraction of the Pope's statement, but some Egyptian lawmakers demanded diplomatic ties with the Vatican be suspended.

The powerful All India Muslim Personal Law Board based in the northern city of Lucknow called for an end to protests against the Vatican but demonstrations were held elsewhere.

In Jakarta, some 100 hardliners rallied outside the Holy See's mission in the Indonesian capital, waving a banner depicting the Vatican as an axis of Satan.

Some 150 protesters from a youth party marched through the Pakistani Kashmiri capital Muzaffarabad chanting Death to Pope and burned him in effigy.

The Pope was also burned in effigy in this southern Iraqi port city where hundreds of Iraqis staged a demonstration today and called for an apology.

The 500 protesters, followers of Ayatollah Mahmud al-Hassani, a mystical Shiite Muslim cleric, also burned German and American flags and called for the Pope to be tried in an international court.

Al-Qaeda in Iraq warned in an internet statement it would wage jihad, or holy war, until the West is defeated.

We say to the servant of the cross (the Pope): wait for defeat. We say to infidels and tyrants: wait for what will afflict you. We continue our jihad, said the statement attributed to the Mujahideen consultative council.

We will smash the cross, it added, and conquer Rome.

tomder55 answered on 09/19/06:

The last Pope had to courageously confront the ideology of evil in his time and he did it at great risk.It almost killed him .He had a fundamental role in the victory of truth over the lies of totalitarian communism .He never wavered and didn't pull punches in his critique.

I see no reason why Pope Benedict should waver. He expressed regret that a bunch of diaper heads got offended by him pointing out a basic contradiction in their faith. He called on dialogue and reason("In the beginning was the word and the word was with God and the word was God" John 1:1... THE LOGOS OF REASON ).as being the great mediator and instead gets burned in effigy .He has nothing more to apologize for. He should say "I rest my case" .

We have been asking for leadership to speak honestly and with clarity about jihad and dhimmitude . Well now we have one.

paraclete rated this answer Excellent or Above Average Answer

Question/Answer
paraclete asked on 09/15/06 - on political relevance?

and just so you don't think american politics is the only game in town

Into the embrace of the unholy trinity
Hugh Mackay
September 15, 2006

John Howard is a political colossus. If he's not quite beyond politics, he's certainly risen above the party-political struggle. If he's not quite a statesman, he's become our quasi-president.
We know he's no visionary: his two big ideas for Australia - to introduce a GST, and to nobble the trade unions - have been implemented. There are no grand infrastructure plans on the horizon: the massive budget surpluses that could have gone into education, health or the environment have been squandered in tax cuts.
Given that the economy is relatively robust, it's as though Howard is preparing a different legacy: does he wish to be remembered not only as the great economic manager, but as the leader who helped clarify, and even redefine, what we Australians stand for? (Or is his pursuit of the values question a clever example of the politics of distraction, like Margaret Thatcher's infamous claim that "the things that unite us are greater than the things that divide us"?)
So what values does Howard personify? What is his values legacy likely to be? These are reasonable questions to ask of a leader who so obviously enjoys talking about such matters and who has cheerfully injected "un-Australian" into our vocabulary.
It has long been taken for granted that the holy trinity of Australian values is precisely the same as that enshrined in the French republic: liberty (expressed by us as the "fair go"); equality (once called "egalitarianism" here, in those heady days when we thought it was a dream that might come true); fraternity (given an idiomatic spin here as "mateship", perhaps the PM's official favourite).
But values are meaningless as slogans. "Watch my lips" is always a distraction: what we should be watching are the actions that express people's real values. Part of Howard's skill has been to sound as if he continues to espouse our traditional values while actually reinforcing a quite different set.
The core value in contemporary Australia, powerfully reinforced by spoken and unspoken messages from the PM, is materialism.
We are in the full flowering of the capitalist era and have been thoroughly seduced by the idea that wealth is the measure of our worth. Consumerism is rampant. Interest rates are our index of happiness. A nation of shareholders is no longer just a gleam in the prime ministerial eye. The US economy - ruthless and competitive, a place where rich people are proud of their wealth - is presented to us an example of what we could become if only we tried harder.
In the lexicon of politics, "society" has long since become "the economy". In the 'burbs, the search for the perfect bathroom tile to top off our renovations has nudged aside our interest in the health of our democracy. In academia, financial considerations are paramount. In business, the share price is king, even if a little moral queasiness has to be suppressed to achieve the result we aspire to.
In the distribution of wealth, we have steadily widened the gap between top and bottom. A new spirit of entitlement is emerging among the rich. Materialism strongly implies competitiveness; "survival of the fittest" is its mantra. (Egalitarianism was a hopelessly romantic idea, anyway - wasn't it?)
Second in Howard's trio comes pragmatism. If you need to bend the truth to suit the circumstances, so be it. But once you're committed, never yield. Tough it out; tough it out. Principles - whether involving human rights, ministerial propriety or care of the environment - are properly tempered by the shifting pressures of realpolitik. If expediency demands it, promises can be dismissed as "non-core".
Win at all costs; the end justifies the means; tactics before strategy. This is the language of pragmatism and it has become the language of Australian politics. Its seductive cadences can also be heard in sport, business and the Aussie backyard.
And the third of the trinity of Howard-endorsed values? Nationalism - described by Albert Einstein as an "infantile sickness" and by the English poet Richard Adlington as "a silly cock crowing on its own dunghill". Nationalism flourishes for all kinds of reasons, mostly to do with insecurity or triumphalism, or both. In the Howard era, the fires of nationalism have been fuelled by a rich blend of hubris and fear ("look how wonderful we are; look how threatened we are").
Our country - and the US - right or wrong. Tough on asylum-seekers: "We will decide who comes here and under what circumstances they come." Tough on dissent; dismissive of "mushy" multiculturalism; big on the "mainstream". (Howard-style nationalism permits an apology to Vietnam vets for not having been grateful enough to them, but not to Aborigines for dispossession and dislocation.)
Materialism, pragmatism, nationalism. They are not Howard's inventions, of course. To paraphrase Carl Jung, describing another leader in another era: "Howard is the loudspeaker that magnifies the inaudible whispers of the Australian soul until they can be heard in the Australian's unconscious ear."
Perhaps Howard already understood this when he said, all those years ago, "the times will suit me".
Hugh Mackay is a social researcher and commentator.

tomder55 answered on 09/15/06:

the massive budget surpluses that could have gone into education, health or the environment have been squandered in tax cuts.

Hugh Mackay's slant becomes apparent eary in this article. So the Aussies were over taxed and instead of fixing that unfairness he thinks that bloating the gvt. further is how to handle a windfall. My take is that the gvt. should only collect taxes it absolutely needs. If deficits are bad then surpluses are an outrage that needs immediate correction .

Please don't tell me you agree with Mackay's last couple of paragraphs about multi-culturalism . Tough on asylum-seekers: "We will decide who comes here and under what circumstances they come."
Well duhh ... yeah . I guess the idea of nationalism and borders is an anachronism to an era gone by to the likes of Mackay but in my view they are more important than ever.

It has long been taken for granted that the holy trinity of Australian values is precisely the same as that enshrined in the French republic: liberty (expressed by us as the "fair go"); equality (once called "egalitarianism" here, in those heady days when we thought it was a dream that might come true); fraternity (given an idiomatic spin here as "mateship", perhaps the PM's official favourite).

Clearly the French Revolution should be emulated ..It worked out so well afterall.Grand ideas ;virtuous proclamations ended in a reign of terror. What began as a revolution against a broke and corrupt royalty ended in the establishment of a dictatorship.

It appears to me that Howard reflects the views and attitudes I've heard from most Aussies I have known . He would not have retained his place for as long as he has if not. I think that Mackay should emigrate to old Europe where his views in practice are on display in the failed and economically weak nanny-states of the continent. Even there it appears that a backlash against surrendering sovereignty to the greater EU is in process. I think the French Revolution died a long time ago except in the minds of the hopelessly romantic.

paraclete rated this answer Excellent or Above Average Answer

Question/Answer
Itsdb asked on 09/13/06 - Torture!!!

At a Secret Interrogation, Dispute Flared Over Tactics
By DAVID JOHNSTON

WASHINGTON, Sept. 9 Abu Zubaydah, the first Osama bin Laden henchman captured by the United States after the terrorist attacks of Sept. 11, 2001, was bloodied and feverish when a C.I.A. security team delivered him to a secret safe house in Thailand for interrogation in the early spring of 2002. Bullet fragments had ripped through his abdomen and groin during a firefight in Pakistan several days earlier when he had been captured.

The events that unfolded at the safe house over the next few weeks proved to be fateful for the Bush administration. Within days, Mr. Zubaydah was being subjected to coercive interrogation techniques he was stripped, held in an icy room and jarred by earsplittingly loud music the genesis of practices later adopted by some within the military, and widely used by the Central Intelligence Agency in handling prominent terrorism suspects at secret overseas prisons...

Some of the officials who were interviewed for this article were briefed on the events as they occurred. Others were provided with accounts of the interrogation later...

By all accounts, Mr. Zubaydahs condition was rapidly deteriorating when he arrived in Thailand. Soon after his capture, Mr. Zubaydah nearly died of his infected wounds. At one point, he was covertly rushed to a hospital after C.I.A. medical officers warned that he might not survive if he did not receive more extensive medical treatment...

According to accounts by three former intelligence officials, the C.I.A. understood that the legal foundation for its role had been spelled out in a sweeping classified directive signed by Mr. Bush on Sept. 17, 2001. The directive, known as a memorandum of notification, authorized the C.I.A. for the first time to capture, detain and interrogate terrorism suspects, providing the foundation for what became its secret prison system.

After Mr. Zubaydahs capture, a C.I.A. interrogation team was dispatched from the agencys counterterrorism center to take the lead in his questioning, former law enforcement and intelligence officials said, and F.B.I. agents were withdrawn. The group included an agency consultant schooled in the harsher interrogation procedures to which American special forces are subjected in their training. Three former intelligence officials said the techniques had been drawn up on the basis of legal guidance from the Justice Department, but were not yet supported by a formal legal opinion.

In Thailand, the new C.I.A. team concluded that under standard questioning Mr. Zubaydah was revealing only a small fraction of what he knew, and decided that more aggressive techniques were warranted.

At times, Mr. Zubaydah, still weak from his wounds, was stripped and placed in a cell without a bunk or blankets. He stood or lay on the bare floor, sometimes with air-conditioning adjusted so that, one official said, Mr. Zubaydah seemed to turn blue. At other times, the interrogators piped in deafening blasts of music by groups like the Red Hot Chili Peppers. Sometimes, the interrogator would use simpler techniques, entering his cell to ask him to confess.

You know what I want, the interrogator would say to him, according to one officials account, departing leaving Mr. Zubaydah to brood over his answer...

~~~~~~~~~~~~~~~~~~~~~~~~~~~~~~~~~~~~~~~~~~~~~~~~~~~~~~~

First things first, the Times claims Bush's "directive, known as a memorandum of notification, authorized the C.I.A. for the first time to capture, detain and interrogate terrorism suspects."

Their memory is convenient...

    The first time I proposed a snatch, in 1993, the White House Counsel, Lloyd Cutler, demanded a meeting with the President to explain how it violated international law. Clinton had seemed to be siding with Cutler until Al Gore belatedly joined the meeting, having just flown overnight from South Africa. Clinton recapped the arguments on both sides for Gore: Lloyd says this. Dick says that. Gore laughed and said, 'That's a no-brainer. Of course it's a violation of international law, that's why it's a covert action. The guy is a terrorist. Go grab his ass.


Seems Al Gore didn't have a problem with snatching terrorists.

Now for the torture part, if someone blasted the Red Hot Chili Peppers into my room I'd confess anything. This is what everyone is so upset about?

tomder55 answered on 09/14/06:

now if you were talking Dipsy Chix ......



Torture just isn't what it used to be.



last paragraph sums it up :

As the president has made clear, the fact of the matter is that Abu Zubaydah was defiant and evasive until the approved procedures were used, one government official said. He soon began to provide information on key Al Qaeda operators to help us find and capture those responsible for the 9/11 attacks.

This official added, When you are concerned that a hard-core terrorist has information about an imminent threat that could put innocent lives at risk, rapport-building and stroking arent the top things on your agenda.



In other words the CIA and FBI played' good cop bad cop' .

FLASHBACK !

Zubaydah interrogation paying dividends
Bin Laden aide revealed 'dirty bomb' threat

Itsdb rated this answer Excellent or Above Average Answer

Question/Answer
Itsdb asked on 09/12/06 - Under fire, Democrat retreats

By Christina Bellantoni
THE WASHINGTON TIMES
Published September 12, 2006

The U.S. Senate campaign of Missouri Democrat Claire McCaskill has been under sharp criticism since she accused President Bush of letting poor blacks in Louisiana die during Hurricane Katrina.

Mrs. McCaskill, the state auditor, is attempting to link Republican Sen. Jim Talent with the president as she tries to unseat the incumbent.

"George Bush let people die on rooftops in New Orleans because they were poor and because they were black," she told a group of Democratic state legislators last week.

The comments, made as she outlined Mr. Talent's efforts to attract minority voters, were first reported by Pub Def Weekly, a St. Louis-based blog.

"In a spirited voice, she told them that she would do everything she could to make clear to every Democratic voter that 'George Bush has no better friend than Jim Talent,'" wrote Pub Def Weekly's Antonio D. French.

Republicans, including Senate Majority Leader Bill Frist of Tennessee, immediately called upon Mrs. McCaskill to apologize for the remarks.

"It is totally unconscionable to suggest that the Coast Guard helicopter crews who worked hours long past regulation were intentionally leaving poor, black Americans to die on their rooftops," Mr. Frist said on his blog.

Sen. Elizabeth Dole of North Carolina, who leads the Senate Republican campaign arm, issued a similar statement: "These outrageous comments have no place in public discourse."

But Mrs. McCaskill told KMOX radio in Missouri the statement's context as reported by Pub Def Weekly is "not exactly correct." She said she was acknowledging the sentiment of many Americans, and national tragedies are the time when people need government the most.

She characterized the Bush response to Katrina as "gross incompetence that turned tragic because so many of the people in New Orleans didn't have the resources to help themselves."

"Clearly, the Bush administration needs to accept blame for the tragedy in Katrina," she said.

Mr. Talent has been running television ads that laud his work with Democrats and portray him as an independent thinker. They don't mention Mr. Bush, who raised about $600,000 for Mr. Talent at a private Kansas City fundraiser Friday.

Former President Bill Clinton, meanwhile, campaigned for Mrs. McCaskill and Missouri Democrats at a weekend fundraiser in St. Louis that raised more than $1 million.

"You can vote for someone who will perpetuate the control of the narrow strip of the Republican Party that's been running America for six years," Mr. Clinton told attendees. "Or you can vote for someone who would be a proud successor in the Senate to Harry Truman: Claire McCaskill."
~~~~~~~~~~~~~~~~~~~~~~~~~~~~~~~~~~~~~~~~~~~~~~~~~~~~~~~

I love it. Keep it up Dems, you're doing the Republicans more good than you can imagine just by opening your mouths. Seriously, do the Democrats have anything to offer? Seems to me the only thing they have to offer is "it's all Bush's fault." Are they really that clueless?

tomder55 answered on 09/13/06:

the moonbats are really on parade this election cycle. Bob Bowman was nominated for Fla. 15th District and he is a 9-11 denier .

Itsdb rated this answer Excellent or Above Average Answer

Question/Answer
Itsdb asked on 09/12/06 - Bush assassination film applauded at festival

12/09/2006 - 17:21:55 A controversial British drama about the fictional assassination of US President George W Bush has been applauded following its premiere at the Toronto Film Festival in Canada.

A controversial British drama about the fictional assassination of US President George W Bush has been applauded following its premiere at the Toronto Film Festival in Canada.

Death of a President, which shows Bush shot dead in Chicago, Illinois in 2007, was recently slammed as "irresponsible" and "horrible" by American politicians.

However, director Gabriel Range claims critics are judging the film unfairly as they have yet to see it.

He says: "I think the film makes it clear it would really be a horrific event. I really don't think that anyone would get the idea of assassinating Bush from this film."

He adds: "It is using the lens of the future to look at the present. It is about issues that have affected us all in the last five years. It is a film about America today.

"The central conceit of the film was that it is a drama, but told in the style of what we hope is a fairly authentic, classic, retrospective documentary. Clearly, if we had told a retrospective documentary with a fictional president, it would have undermined and undercut that central idea."

~~~~~~~~~~~~~~~~~~~~~~~~~~~~~~~~~~~~~~~~~~~~~~~~~~~~~~

"Using the lens of the future to look at the present...It is a film about America today...told in the style of what we hope is a fairly authentic, classic, retrospective documentary."

Come again?

"Clearly, if we had told a retrospective documentary with a fictional president, it would have undermined and undercut that central idea."

What exactly is the central idea?

tomder55 answered on 09/13/06:

I discussed the idea with my daughter the other day about people living in an alternate reality . Documdramas loosely based on someone's perception of reality are common . In some cases ;like Michel Moore they inject themselves into the story ;they build a premise and then work on that adding half truths and distrotions to fortify their case to a point where the gullible believe for example that the WTC collapsed because of timed detonations and that the Pentagon was hit by a cruise missile ,or even that JFK was killed by a magic bullet. The news becomes fictionalized and fiction becomes 'reality based' entertainment .News anchors become personalities who play themselves in block buster movies. Children all over log onto the internet and become whoever they want to be.

Who's the President on TV ? Not George Bush ;it's Josiah Bartlett ;and Dick Cheney becomes a fictional character in 'Death of a President ' . The movie works for the moonbats because it feeds into all their preconceptions and evidently their yearnings . According to the synopsis :

In October 2007, President of the United States George W. Bush is assassinated by a hidden shooter on his way out of a Chicago hotel, in front of which an anti-war rally is being held. A man of Syrian origins, Jamal Abu Zikri, becomes the number one suspect. Three years later, a documentary filmmaker presents a movie about what the United States has become since the assassination..

The Syrian Jamal Abu Zikri possibly sympathetic to Jihad was initially suspected as the assassin. Dick Cheney, after being elevated to the position of President, uses the possible al-Qaeda connection of the suspect to push his own agenda. He calls for a Patriot Act 3, suspends most civil liberties and for military action against Syria. An already grotesque world situation keeps growing tesquer.

Eventually, the movie revealed that the perpetrator is a black American, a father of a soldier who had died on duty in Iraq. The assassin blames Bush for the death of his son. He shoots the President, then himself. The killer's suicide note reads: "There's no honor in standing for an immoral country. George Bush killed our David and I can't forgive him." So it turns out that the assassination was entirely personal.








Itsdb rated this answer Excellent or Above Average Answer

Question/Answer
HANK1 asked on 09/12/06 - HEWLETT - PACKARD:



"When you think of your own personal assets, chances are your home, car, and savings and investments come to mind. But what about your Social Security number (SSN), telephone records and your bank and credit card account numbers? To people known as pretexters, that information is a personal asset, too.

Pretexting is the practice of getting your personal information under false pretenses. Pretexters sell your information to people who may use it to get credit in your name, steal your assets, or to investigate or sue you. Pretexting is against the law."

Source: Federal Trade Commission

Are you following this story?

HANK

tomder55 answered on 09/13/06:

Patricia Dunn, chairman of HP is stepping down over the issue. She issued a sorta generic blanket apology for unspecified "certain inappropriate techniques". Congress and the FBI are launching investigations of HP's investigation so that may have been the only comments her lawyers permitted her to say . Much better would've been an admission that she screwed up royally.

Here is a decent link to explain what happened .

Apparently, it all began when she became angry after a media outlet quoted an anonymous HP source who reported that a meeting of directors was held at a posh spa in southern California.

She hired private detectives to find out who was talking to the press.

It's not a good idea for directors to talk to the press about company business but it's not illegal unless it involves confidential, fiduciary board discussions or knowledge.

By telling the press the board met at a posh spa, the director was disclosing to shareholders that perhaps board venues should be less expensive.

So what did Ms. Dunn do?

She went out and spent more shareholder money hiring detectives to find the director who told shareholders their money was being wasted on fancy meeting venues.

The detectives used questionable, possibly illegal, means to unearth her director-culprit. And they found him: George Keyworth II, a highly respected former science advisor to President Ronald Reagan and director of the physics division at the Los Alamos National Laboratory.

In the spring, she confronted him in front of the board and demanded his resignation but he refused. Then the board banned his re-election as a director. Another director, famous venture capitalist Tim Perkins, resigned in protest.............The FBI said yesterday it was looking at whether Ms. Dunn's detectives illegally wiretapped individuals and illegally hacked into their computers.

Ms. Dunn said the investigators she hired impersonated board members and journalists to acquire their phone records.


CLearly the use of 'pretexting' is illegal but it is unclear that Dunn aouthorized it .However keep in mind that a company has the perfect right to snoop on internal communications of all kinds.But in this case AT&T has already confirmed that Tom Perkins' personal telephone account records were pretexted. Honestly ;Tom Perkins is the wrong cat top be screwing around with !






ETWolverine rated this answer Excellent or Above Average Answer
HANK1 rated this answer Excellent or Above Average Answer

Question/Answer
paraclete asked on 09/13/06 - What's this, a secret war?

diggers' secret battles revealed

By Ian McPhedran

September 13, 2006 01:00am
Article from: The Daily Telegraph


DURING nine days of fierce fighting in southern Afghanistan, Australian special forces killed more than 150 Taliban and al-Qaeda fighters, and suffered just six men wounded.

In the most intense battles since the Vietnam War, Diggers from the Special Forces Task Group used superior weapons and overwhelming airborne fire support from USAF AC-130 Hercules Spectre gunships.

Codenamed Operation Perth, the hardest fighting took place in July during search-and-destroy missions in the Chora district, about 40km northeast of the Australian base at Tarin Khowt, in southern Afghanistan.

Despite secrecy surrounding the 12-month special forces deployment, The Daily Telegraph can reveal previously classified details of the Diggers' campaign.

During the year-long operation the three rotations of the task group have sustained 11 casualties, including several men seriously wounded.

One commando had part of his jaw blown off, another was shot in the buttocks and an SAS specialist was hit in the abdomen. Amazingly, the round missed his vital organs.

In one action, six commandos, including the company sergeant major, who sustained leg injuries, were wounded by an enemy rocket-propelled grenade.

Several men are to be awarded gallantry medals.

The Daily Telegraph can also reveal that at the height of the battle, three AC-130 Spectre aircraft ran out of ammunition.

"That was the first time for some of the crews that they have bled the magazines dry," a special forces source told The Daily Telegraph.

The task group includes a commando platoon of 50 men from the Sydney-based 4RAR and 40 SAS troopers from the unit's No.3 squadron. The 100 support soldiers include chemical weapons experts from the Incident Response Regiment.

The commandos and SAS troopers are angry that the task group will not be replaced when it leaves later this month.

"It's not right to pull out. We shouldn't just go there for a shoot 'em up and then come home," one soldier said.
===================================================

I just want to say, with forces like these, the Muslims don't stand a chance.

tomder55 answered on 09/13/06:

this happens every spring and summer. the Taliban come out of their hide-aways in Pakistan and the remote villages and the press dutifully report that they are resurgent. Then they engage coalition forces and 100s of them get killed at a time . They then go back to their lair and lick their wounds until the next spring . Eventually the national army will be strong enough to take over those operations .

Established countries all over the world fight simular insurgencies routinely . Close to you the Philippines are a prime example where the MILF is bothersome but hardly an existential threat to the country .

paraclete rated this answer Excellent or Above Average Answer

Question/Answer
Itsdb asked on 09/12/06 - Less Promise for Democrats in N.Y.

By RAYMOND HERNANDEZ

In a year when Democrats hope to take control of the House of Representatives, New York would appear to be fertile ground for toppling Republican incumbents. Democrats have a statewide edge in enrollment, and a popular incumbent, Senator Hillary Rodham Clinton, is at the top of the partys ticket.

In fact, just a few months ago, Democrats envisioned significant gains in New York, perhaps picking up as many as four seats, possibly even five. But that goal now seems increasingly remote, and there is an emerging consensus among political analysts that the partys best chance for capturing a Republican seat is the battle to succeed Representative Sherwood L. Boehlert, one of the most liberal Republicans in Congress, who is retiring.

At the same time, the Democratic Congressional Campaign Committee plans to spend roughly $50 million on advertisements for races around the country, according to Republican estimates. But none of that money has been set aside for New York races, except for Mr. Boehlerts seat in the 24th District in the Utica area, according to Democrats involved in the races.

The shifting local fortunes for Democrats could have serious political implications beyond New York. The party needs 15 seats to take control of the House. Even one victory in New York would be an important step toward that goal, giving the Democrats a cushion if they lose elsewhere in the nation.

The situation in New York is particularly surprising given the states reputation as a Democratic bastion. National and state party officials have spent months trying to create buzz around those races. But Republican incumbents, in New York and elsewhere, have been trying to shift the focus of the races away from hostility toward the Bush administration to more local concerns, like the potential loss of federal aid to their districts if they lose veteran congressmen.

Representative John E. Sweeney, one of the Republican incumbents, said the situation in New York demonstrated the drawbacks of the Democratic effort to present the midterm elections as a national referendum on President Bush and the policies of Republican leaders in Washington. Congressional races are local, he said. There can be superseding events like Watergate. But those are rare. These races really are a referendum on the people running.

In addition and perhaps most important the incumbents in New York are benefiting from being in legislative districts drawn to keep the Republican incumbents in place.

The Democrats inability to gain traction can be measured in the fund-raising disparity between them and Republicans, and is reflected in interviews with strategists in both parties and independent analysts. The national party assesses the strengths of a campaign according to several factors, including the ability of candidates to raise money on their own and their standing in polls...

In the meantime, he (Charlie Rangel) said, the unusually high level of public unhappiness with the direction of the nation works in favor of Democratic challengers, as long as they have enough money to keep their campaigns running. It is just as important as money, he added...

Independent analysts say that several factors make New York an especially difficult place for challengers. The state has 29 House seats, and 9 are held by Republicans, most in upstate areas where districts have been drawn to give Republicans a significant edge, countering increasing Democratic enrollment in the state...

Democrats say there is plenty of time for challengers to close any gaps. To that end, one prominent liberal group, MoveOn.org Political Action, is running advertisements attacking Mr. Sweeney and John R. Kuhl Jr., a first-term Republican from the Corning area, as part of a national campaign to help Democratic challengers who are in so-called second-tier races: contests that have the potential to become competitive but are not considered competitive yet...

(edited for length)

~~~~~~~~~~~~~~~~~~~~~~~~~~~~~~~~~~~~~~~~~~~~~~~~~~~~~~~

Shifting fortunes for Democrats? Gee, this fall was going to be doom and gloom for Republicans, what happened? Is it because politics are local, Republicans are better at strategizing, have more money, redistricting? All of the above? Is it just that Democrats have nothing better to offer besides "public unhappiness"? Or will it still be doomsday for Republicans?

tomder55 answered on 09/12/06:

One of my college projects that I spend a few all nighters on was a statistical analysis of voting patterns of the NY legislature. The pattern was pretty consistent with what was considered common knowledge ;that NY was in fact 3 large voting blocks divided between NY city ;the NYC suburbs especially Eastern Long Island and the upstate regions .Although NY City had a greater influence because of it's larger overall population ,voting patterns in the other 2 areas were more common with heartland 'red state ' patterns with pockets in and around some of the other state cities .

The Sweeney race (appropriately being contested in Saratoga) is a good one to watch if someone wants to gage Democrat chances in taking the House. He was in trouble earlier in the summer and Moveon.org had been smelling blood .Kirsten Gillibrand his opponent is keying in on Sweeney's support of the Iraq war . Going into Labor Day Sweeney was ahead by 19 points . They have the moonbats but Sweeney has Rudy . I like his chances .

Funny about Rangal complaining of redistricting shaping the race .I was in a district where the Rep. was an absolute national treasure but he resigned after the Democrats redrawing of the districts eliminated his completely .

Itsdb rated this answer Excellent or Above Average Answer

Question/Answer
HANK1 asked on 09/12/06 - A FRIGHTENING THOUGHT:



Do you think Islam will recruit white Americans (Caucasians) to do its dirty work for large amounts of $$$? As most of us know, some people will sell their souls for 'a bottle of cheap Port wine.'

HANK

tomder55 answered on 09/12/06:

there is always the unresolved issue about John Doe # 2 during the Oklahoma City Bombing that has been documented by Jayna Davis . Her investigative reports sheds light on some credible evidence of an Iraqi connection to the Oklahoma City Bombing .

SHE LOCATED two dozen witnesses who identified eight specific Middle Eastern men, the majority of whom were ex-Iraqi soldiers, who were seen with McVeigh and Nichols. (Read the whole link)

The answer is that already American Caucasians have been recruited .Have you forgotten Johnny Jihad ?

HANK1 rated this answer Excellent or Above Average Answer

Question/Answer
paraclete asked on 09/10/06 - There is no kudos in this, George?

Bin Laden hunt 'gone cold'

From correspondents in Washington

September 10, 2006 08:52pm
Article from: Agence France-Presse


FIVE years after the Sptember 11, 2001, attacks, the trail of their chief mastermind, Al-Qaeda founder Osama bin Laden, has gone cold, The Washington Post has reported on its website.

The newspaper said US commandos, whose job is to capture or kill bin Laden, have not received a credible lead in more than two years.

Nothing from the vast US intelligence world - no tips from informants, no snippets from electronic intercepts, no points on any satellite image - has led them anywhere near the Al-Qaeda leader, the report said, citing unnamed US and Pakistani officials.

"The handful of assets we have have given us nothing close to real-time intelligence" that could have led to his capture, The Post quotes one of the officials as saying.

Bin Laden is believed to be hiding in the northern reaches of the autonomous tribal region along the Afghanistan-Pakistan border.

Following a request from President George W. Bush to "flood the zone," the CIA has sharply increased the number of intelligence officers and assets devoted to his pursuit, the newspaper reported.

The problem, former and current counterterrorism officials say, is that no one is certain where the "zone" is.

"Here you've got a guy who's gone off the net and is hiding in some of the most formidable terrain in one of the most remote parts of the world surrounded by people he trusts implicitly," the Post quoted T. McCreary, a spokesman for the National Counterterrorism Center, as saying. "And he stays off the net and is probably not mobile. That's an extremely difficult problem."



so the way to confound the intelligence of supposedly the best intelligence service of the world is use your intelligence and just do nothing except issue a video now and again showing your old family snaps. One has to ask, surely it's possible to get tipped off by al jazerra when a video is dropped off and follow the courier, how hard can it be? The reality is, that, with all the high tech, the intelligence services can't think low tech any more. Even the unabomber couldn't hide out this long

tomder55 answered on 09/11/06:

no snippets from electronic intercepts, no points on any satellite image ...even OBL is not stupid enough to brouadcast where he is . But if he does not take advantage of electronic communication with his org. then it is not likely he is very relevent anymore . He must be leading a life worthy of a sheik who is the leader of an international jihad . wonder when the last time is he had a hot shower? He must smell like the goats he sleeps with.

Itsdb rated this answer Excellent or Above Average Answer
paraclete rated this answer Excellent or Above Average Answer

Question/Answer
paraclete asked on 09/10/06 - Let's all do it the Aussie way?

The revolution starts with this man

Michael Duffy
September 9, 2006
Page 1 of 2 | Single page

TODAY a man named Gary Sturgess will tell Peter Debnam and other Liberal state opposition leaders how they might win office. He believes they should catch a huge reform wave that has swept through government in Britain, and to a lesser extent America. It's a wave that has improved public services for millions of people in both those countries, but so far has hardly reached our shores.

In recent years (to mix metaphors) the opposition leaders have been all at sea, grasping at straws. Sturgess is offering them what is potentially a new ocean liner.

Sturgess, who ran the NSW Cabinet Office when Nick Greiner was premier, was one of the most innovative public servants this state has seen. He now lives in London and heads the Serco Institute. This develops ideas for Serco Group, a large company that manages public services, such as railways and prisons, in over three dozen countries. It's at the forefront of the opening of the public sector to private organisations that Sturgess will talk about today. He hasn't released his speech, but its description of what he calls the "public service economy" should reflect a talk he gave in Canada this year.

Few Australians are aware of the extraordinary changes that have occurred in Britain's government under Tony Blair. These have involved not new policy goals but improved ways of achieving existing goals. This is because the days when political parties had major differences over policies have ended.

Across Britain's public service, the emphasis has moved from delivering services to ensuring that services are delivered, and delivered to a higher standard than before. Often this involves paying private companies or voluntary organisations to do things once done by a cumbersome public sector.

Much more emphasis is now placed on measuring outcomes. Where once government paid for good intentions, now it increasingly demands results. Often these are monitored by new, independent assessment bodies. Private organisations that deliver well make a profit, and often, thanks to competition, still save the government money. Those that fail are dropped and replaced with new ones. There is a constant process of innovation and change unknown in the traditional public service.

tomder55 answered on 09/11/06:

Demanding results and proper accounting from public spending ? how Conservative can they get ?

ETWolverine rated this answer Excellent or Above Average Answer
paraclete rated this answer Excellent or Above Average Answer

Question/Answer
Mathatmacoat asked on 09/11/06 - hey, where is everybody?

where have you all gone, are you all hiding in case he does it again?

tomder55 answered on 09/11/06:

I for one have a life on weekends that does not include sitting in front of a computer...especially during peak hiking seasons . Yesterday I hiked up to a mountain that has a panoramic view of the NYC skyline . After 9-11 there was a large American flag that flew there . I have a great picture of it but have not been able to transfer it to the web for sharing . Anyway ,yesterday was a day just like 9/11 2001 was here ;warm and clear sky. We stopped for a while for a little reflection then sat down on the moss covered ground for a quick lunch before we resumed our hike .

No I do not fear OBL .He is more a symbol now than a real threat . He of course is trying to scare us again but consider this ... he is using 5+ year old video footage to try and do it . He has succeeded in creating his world wide califate in a remote mountainous region of Pakistan that is of so little value that Pakistan doesn't consider it worth defending . Gone are the days when OBL can produce quality video ,instead we get fuzzy audio from him weeks after he records it because that is how long ot takes him to get a runner from his lair to someone from al-Jazzera. He needed a State sponsor to continue his plans and that has been denied to him.

What he has become is a Che Guevara-like symbol of Jihadistan . I ,like President Bush see very little value in promoting that image .That is why you hear the President say he hardly ever thinks of OBL anymore .Why elevate someone who sleeps with the goats ?

Mathatmacoat rated this answer Excellent or Above Average Answer

Question/Answer
Itsdb asked on 09/09/06 - Kudos to the president.

I saw Gasoline prices at $2.49 per gallon today, 54 cents per gallon less than last month. Good job Mr. President.

tomder55 answered on 09/10/06:

I guess if he gets blamed for higher priced gas he should by logic be applauded when the gas prices fall. Unless the Mahdi-hatter does something stupid then prices are likely to fall much further before the elections ....How Rovian those commodity prices be !

Don't worry ;the Dems will just shift their talking points slightly . Remember that as the temperature starts to cool down the emphasis will shift to home heating and as demand increase ;price increases in natural gas is sure to follow.How Deanian of them ! It's almost like Chucky Schumer planned it !

ETWolverine rated this answer Excellent or Above Average Answer
Itsdb rated this answer Excellent or Above Average Answer

Question/Answer
HANK1 asked on 09/08/06 - OUR FRIEND, TONY BLAIR:



Tony will 'take a hike' next May ... if not before. I'd like to see him venture to America and take up residency here. Then run for President when the law permits. Could this happen?

HANK

tomder55 answered on 09/09/06:

I don't think it is unfair to say the president of the United States should be a native-born citizen. Often times allegiance is driven by your birth.The last thing we need is a 'Manchurian Candidate'(or in Schwarzenegger's case a chamilion ) .

"It is well known that the Founding Fathers were mindful in the extreme of foreign influences, and the dangers therein to the Republic. While experience has shown that a native-born Chief Executive is not necessarily immune to foreign influence, the odds are certainly more favorable if the president is an American plain and simple, who has never been, and is not at the time of taking office, anything else." [Balint Vazsyoni, director of the Center for the American Founding in testimony to the House Judiciary Committee's Subcommittee on the Constitution was held on July 24, 2000 to review amending the Constitution to allow foreign-born citizens to become president.]

For instance, if a native-born Cuban became president and we invaded Cuba to topple Castro wouldn't the president's loyalty and motives be questioned ? The Founding Fathers, in writing the Constitution, made sure that question need never be asked.

There were good reasons that the founders put the restriction of foreign born persons from becoming President (although it has been argued that some of them had vindictive reasons for preventing Alexander Hamilton from ever becoming President ...Hamilton was a native of Barbados ).In 1772 Poland had been partitioned among Austria, Prussia and Russia after agents of those countries bribed Polish nobles to elect a disloyal king.

It happens to be the only restriction that I know of to foreign born citizens regarding elected office. WE have govenors and Legislators all over the country who are foreign born. Rep. Lincoln Diaz-Balart, who is constitutionally prohibited from ever holding the presidency, has stated, "I don't know of any other country where the electoral process is so open to foreign-born citizens."

I would welcome Tony Blair and John Howard into the country with open arms but would not favor their attempt to become President . Rather ;I would give them high cabinet positions in the Executive like Henry Kissinger had or other prominent foreign born citizens like Madeleine Albright (Czechoslovakia) and Christian Herter (France) who were secretaries of State or Retired general John Shalikashvili, a former chairman of the Joint Chiefs of Staff(born in Poland).

HANK1 rated this answer Excellent or Above Average Answer
paraclete rated this answer Excellent or Above Average Answer

Question/Answer
HANK1 asked on 09/08/06 - MULLAHS:



Do the mullahs possess the means to destroy Israel and America?

HANK

tomder55 answered on 09/09/06:

Hank ; I always have in the back of my mind the possibilty that they may have already acquired one or more nuclear weapons on the open market. As you know the Soviet stock is not all accounted for and certainly they have the cash to have purchased them. In 1992 Russia was designated as the successor to the Soviet Union, which was authorized to have nuclear weapons by NTP. On the other hand Belarus, Kazakhstan, and Ukraine, where nuclear weapons had been deployed, became non-nuclear states. Thus all nuclear weapons out of Russia should have been transferred to Russia and put under the Russian control. But confusion in Russia lasted a long time and it made Russian managing ability questionable. Rumors were in the air that some nuclear weapons of former Soviet Union were missing and a part of them passed into Iran, Iraq, and Pakistan.

Another factor is that we do not know to what extent the AQ Kahn network was able to go in achieving the 'Islamic Bomb' . We now know that the Iranians had in their possession Chinese blueprints for the making of bombs .So it is not a case of them having to develop them from scratch .

Again ;they would not have to develop an enrichment cycle bomb grade plutonium and uranium was available on the open market. We know of 15 to 20 seizures of weapons-usable plutonium and highly-enriched uranium (HEW) have been recorded internationally , and policymakers must contemplate the possibility that ... as with other illegally traded commodities...what was seized is only a small fraction of what has been circulated through smuggling channels.Recorded cases include Russian managers of top secret defense plants offered plutonium for sale to visiting foreign scientists in the 1990s. Military officers stole HEU fuel from a submarine base in Murmansk. Agents of the Russian Foreign Intelligence Service reportedly masterminded the delivery of almost a pound of plutonium oxide from Moscow to Munich in August 1994. In a 1998 incident, the Russian Federal Security Service reportedly foiled an attempt by "staff members" of a major nuclear weapons plant in Chelyabinsk to steal some 18.5 kilograms of weapons-usable material ;enough to fashion a single nuclear weapon.

Less well documented are reports from Russia that large quantities of plutonium and HEU were removed from nuclear labs in the early 1990s (whether the material was exported or remains in Russia is anyone's guess), that scores of "suitcase-sized" nuclear weapons are missing from storage, and that certain labs are engaged in criminally-brokered schemes to enrich uranium and sell the weapons-grade product to "Middle Eastern states". If any such reports are true, serious nuclear smuggling from Russia are no longer a threat, but a fact. [The Bulletin of Atomic Scientists reports that Iran has imported unsafeguarded enriched uranium from an unknown country and calutron magnet cores from West Germany.
David Albright and Mark Hibbs, The Bulletin of Atomic Scientists, March 1992, pp. 9-11. ]


We also know the N.Koreans have the ability to produce that ,and we know they are more than willing to sell it for hard currency


The Times of UK reported last week that THE British government was warned more than two years ago that Iran had illegally acquired a missile system capable of carrying nuclear warheads.

So it is possible that Iran not only has already acquired nuclear weapons ;but also the means of delivery.

We know the N.Koreans have the ability to produce that ,and we know they are more than willing to sell it for hard currency .

It does not make sense to me that they would've ratchetted up their level of aggressivness unless that had an ace in their sleeve.

We know the N.Koreans have the ability to produce that ,and we know they are more than willing to sell it for hard currency .

So to answer your question ;yes I believe they are already an existential threat to Israel .Currently they are not but Iran plays chess and chess is a game where you think your moves many plays in advance .

other references :

http://www.library.cornell.edu/colldev/mideast/iranbmba.htm

http://www.iran-press-service.com/articles_2002/Jun_2002/iran_has_nuke_6602.htm

http://www.jpost.com/servlet/Satellite?cid=1138622510390&pagename=JPost%2FJPArticle%2FShowFull

Iran acquires nuclear weapons "ready for immediate use" from CIS countries, according to the German weekly Focus. The weapons are reportedly "an atom bomb which can be dropped from an airplane and a launcher for missiles with nuclear warheads."
"Iran Reportedly Acquires Nuclear Arms from CIS," Central Eurasia, 28 January 1993, p. 6.

In autumn 1991 Nursultan Nazarbayev, the president of Khazakhstan, sold three nuclear warheads to the Iranians. The Iranians wanted to use them as a prototype for their own bomb manufacturing. The price was said to have been 7.5 billion USD. Whether this amount is true or just the fantasies of a less paid government official, I cannot verify. The amount was to cover all bribes and kick-offs and military protection during transport. Every country involved had demanded their fair share of the deal.

Anyway, the warheads were removed from a military depot somewhere in Kazakhstan and transported by train down to Makhachkala in Daghestan. Here they were reloaded onto huge trucks and then taken through the Caucasian region and into Turkey. In the city of Dogubeyazit the Iranians met the convoy and took over. The three vehicles were then driven by Iranian drivers down to the border post Bazargan, where they entered Iranian territory.

The warheads were brought down to Teheran and parked in the military campus Lavizan. Here they were seen by a soldier who later defected to Israel and told the story to the Israeli intelligence services who at that time were unable to verify the matter further. Various rumours have been circulating ever since. Some stories say two bombs, some say four. The correct number, however, is three.


http://www.frontpagemag.com/Articles/ReadArticle.asp?ID=24019


ETWolverine rated this answer Excellent or Above Average Answer
HANK1 rated this answer Excellent or Above Average Answer
paraclete rated this answer Excellent or Above Average Answer

Question/Answer
Itsdb asked on 09/08/06 - Armitage Says He Was Source of CIA Leak

He Says He Did Not Know Covert Status

By R. Jeffrey Smith
Washington Post Staff Writer
Friday, September 8, 2006; Page A03

Former Deputy Secretary of State Richard L. Armitage said yesterday that he believes he was the initial source for a 2003 newspaper column by Robert D. Novak that disclosed the CIA's previously secret employment of Valerie Plame, the wife of a prominent critic of the U.S. war in Iraq.

Armitage said that he learned about Plame's employment from a State Department memo that did not mention her covert status, and that he had no knowledge of it at the time. In 40 years of reading classified materials, Armitage said in a telephone interview, "I have never seen in a memo . . . a covert agent's name."

Novak's disclosure of Plame's CIA employment ultimately led to the appointment of a special counsel to investigate the leak. In October of last year, a grand jury indicted vice presidential chief of staff I. Lewis "Scooter" Libby on charges of lying to investigators about his conversations with reporters about Plame, forcing his resignation and embarrassing the White House.

But Armitage, who said he testified about his actions to a grand jury three times, was not charged for making the disclosure, a circumstance he attributes to his candor in speaking with investigators about his action. He turned over his computers and never even hired an attorney, Armitage said, because "I did not need an attorney to tell me to tell the truth."

The confirmation of Armitage's role has provoked criticism of both him and the special counsel, Patrick J. Fitzgerald, who learned of it shortly after his appointment in 2003. Some have questioned why Armitage waited so long to speak up about it, and why Fitzgerald spent two years appearing to chase a question that had already been answered.

Armitage said yesterday that he did not disclose his role before now because Fitzgerald had asked him not to. But word of his role eventually began to circulate, and on Tuesday, Armitage said, he asked Fitzgerald to be freed of that promise. Fitzgerald agreed.

Armitage said that he was asked, during the course of the investigation, whether he had discussed the leak in advance with other senior administration officials, and that he replied: "Hell, no."

Instead, he said, he divulged Plame's name in an offhand way at the end of his conversation with Novak, whom he had not previously met. Novak had asked him why former ambassador Joseph C. Wilson IV -- Plame's husband -- had been tasked to probe Iraq's alleged interest in acquiring nuclear materials.

"Novak asked me, 'Hey, why did the CIA send Mr. Wilson to Niger?' I said, 'I don't know, but I think his wife worked out there,' " Armitage said.

Novak asked because Wilson at the time was accusing the White House of deliberately distorting the intelligence it had received on Iraq's nuclear program to justify its invasion of the country, an allegation that infuriated Vice President Cheney and other top officials.

But Armitage said he did not realize that he was a source for Novak's subsequent column naming Plame until October 2003, when Novak identified one of his sources as someone who was not a "partisan gunslinger." That mention provoked Armitage to tell then-Secretary of State Colin L. Powell, who told the State Department's top lawyer, who in turn arranged for Armitage to speak with the Justice Department.

Armitage also acknowledged making a similar offhand remark about Plame earlier in 2003 to Washington Post Assistant Managing Editor Bob Woodward, who was researching a book about the decision to invade Iraq. Armitage said he deeply regrets embarrassing Powell, the State Department, his friends and family, and the Wilsons.

~~~~~~~~~~~~~~~~~~~~~~~~~~~~~~~~~~~~~~~~~~~~~~~~~~~~~~

Unbelievable. Yeah, it's past time for Armitage to come clean but this Post piece is comical.

he believes he was the initial source

he divulged Plame's name in an offhand way

he did not realize that he was a source

Armitage also acknowledged making a similar offhand remark

Armitage said he deeply regrets embarrassing Powell, the State Department, his friends and family, and the Wilsons


Since when did the Post put on kid gloves over Plamegate? What about the rest of the 'regret' he expressed?

"It was a terrible error on my part. There wasn't a day when I didn't feel like I had let down the president, the secretary of state, my colleagues, my family and the Wilsons. I value my ability to keep state secrets. This was bad, and I really felt badly about this."

So the Post only thinks "Powell, the State Department, his friends and family, and the Wilsons" deserve a mea culpa from the guy that started a feeding frenzy on Bush, Rove and Cheney when Armitage acknowledged his failure to the president first? Un-be-lievable.

Oh, and of course, the Bush 'scandal' isn't over yet...

David Corn:



~~~~~~~~~~~~~~~~~~~~~~~~~~~~~~~~~~~~~~~~~~~~~~~~~~~~~~
NY Times:

    The revelation tells us something important. But, unfortunately, it is not the answer to the central question in the investigation whether there was an organized attempt by the White House to use Mrs. Wilson to discredit or punish her husband, Joseph Wilson. A former diplomat, Mr. Wilson debunked the claim that Saddam Hussein tried to buy uranium from Niger to make nuclear weapons.


Comments?

tomder55 answered on 09/08/06:

first a factual error that needs correcting Wilson did not debunk anything except the notion that he could be a reliable asset to the intelligence community .

I heard Armitage on the CBS news being interviewed by Katie Couric . When asked if he had gotten a lawyer he said no because he did not need anyone to tell him to tell the truth . More likely however ,the reason was that he interviewed with Fitzgerald early in the investigation and it was Fitzgerald who told Armitage to be quiet about his role . So here you have the prosecutor being told all he needed to know about the case; but still he persued a very expensive investigation of entrapment .....Why ? THAT IS THE CENTRAL QUESTION NY SLIMES !!!!

Itsdb rated this answer Excellent or Above Average Answer

Question/Answer
excon asked on 09/08/06 - Rights

Hello:

Are the rights you enjoy given to you by a government or are they inalienable? In other words, is your right to be represented at trial given to you? Or do you own it by virtue of your birth?

Me??? I believe Thomas Jefferson. I believe our founding documents. I believe in the USA. I believe what this nation stands for.

You?

excon

tomder55 answered on 09/08/06:

I do not think that the right to be represented in a trial is an inalienable right . Clearly Jefferson did not include that in his general statement about inalienable rights in the Declaration. Even when you read down to the text of specific grievences you find that the founders objected to not having trial by jury ...not the fact that they were not have represented .

The concept of inalienable rights came from the concept of natural rights. But even the founders understood that unfettered it would lead to anarchy. So what was the solution ? ..... the social contract ceding right to an authority who have that position by the consent of the governed for the purpose of maintaining order and mutual protection .

Classic liberal thinkers, for example, accepted the death penalty and incarceration as necessary elements of government even though it in theory violates the indiviuals liberty and inalienable right to life .

A Constitution does not necessarily guarantee inalienable rights ...it is a social contract and is by definition limited to those with whom the contract is bound .

Terrorist scum from the ME have limited rights under our Constitution and I think the concept of tribunal commissions more than satisfy them .



excon rated this answer Excellent or Above Average Answer

Question/Answer
paraclete asked on 09/08/06 - maybe George could learn something from him?

Honest John a straight-talker
By Mark Steyn
September 08, 2006 12:00am

JOHN Howard was quoted approvingly on a US radio show last week. Big deal, you say. He's a prime minister; what does he care if some rinky-dink talk-jockey recycles a couple of sound bites?

Well, the radio host in question was Rush Limbaugh, and Rush has more listeners than there are Australians. That's to say, about 25 million or so listeners, which is more than the number of Australians in Australia and Lebanon combined.
Why would gazillions of American radio listeners appreciate a line from Howard? Because he says things that none of their own leaders ever quite say.

Last week it was the stuff about Muslim immigrants needing to learn English and making sure they're cool with this equal-rights-for-women business.

The soi-disant arrogant Texas cowboy rarely shoots from the lip like that. Instead, he says things such as: "Freedom is the desire of every human heart."

Look, I'm a supporter of the Bush doctrine to spread liberty throughout the Muslim world, but I support it on hard-headed grounds of national security.

You only have to watch a couple of minutes of the lads in Gaza and southern Lebanon on the telly every night to realise freedom comes pretty low down on the list of their hearts' desires.

So, when the US President insists on reprising the line week in week out, he begins to sound utopian, if not utterly deluded. American conservatives would appreciate a rationale less hermetically sealed from reality.

By contrast, the Prime Minister's rhetoric meets what the law used to regard as the "reasonable man" test.

When Howard refers to blokes "raving on about jihad" and the way that those so inclined are "utterly antagonistic" to a free society, he's merely stating the obvious in a way that other Western leaders can't quite bring themselves to do.

His words align with reality, and one can't underestimate the value of that.

The other day, on a flight from Malaga to Manchester, a bunch of holidaying Brits mutinied and demanded the removal of two suspicious "Asian" passengers in "heavy clothes" and "checking their watches".

The evicted passengers appear to be blameless, but the other travellers had spent the days since the Heathrow arrests listening to British government ministers trotting out the usual hooey about how the improved security procedures would be impeccably non-discriminatory and they seem to have concluded, reasonably enough, that although the new rules may prevent your toothpaste, Diet Coke and gel-filled bra (to name three now prohibited items) from boarding, they were unlikely to stop the mad bombers getting on.

In other words, the more the gulf widens between the Government's multiculti PC pap and the obvious truth, the more the state risks de-legitimising itself in the eyes of the citizenry.

Tony Blair has a good pitch when he's surveying the distant horizon and the big picture and doing his Tone of Arabia routine, but he hasn't yet managed to find a line on the homegrown jihad that resonates with his electorate.

If I ran the speechwriting departments in the White House and Downing Street, Howard's bloke's-eye view would be the working template.

As someone who's been citing Canberra's finest across the US long before Limbaugh and the other Aussie-come-latelys jumped in, I like to think of myself as a kind of honorary cultural attache, like Dame Edna's friend Sir Les Patterson, but with less stained trousers.

I'm aware, after my trip to Australia last month, that various local lefties think I'm as nutty in this respect as Steve Irwin when he hailed Howard as "the greatest leader in the world". Perhaps it takes a croc hunter to appreciate a crock hunter: a politician with a keen eye for fashionable baloney and a willingness to wrestle it to the ground.

Still, I do think it's worth considering why, of the three doughty warriors of the Anglosphere, Howard has managed to avoid the traps that have ensnared George W. Bush and Blair.

For example, while Australia has some of the sweetest republicans in the Commonwealth, the Prime Minister's cultural conservatism strikes me as well grounded: in a time of rapid international and economic change, you have to prioritise, you can't put everything up for grabs, unto the flag and the crown.

The day after the London bombings, Blair said that the terrorists would not be allowed to "change our country or our way of life".

Of course not. That's his job, from accelerating European integration to his "reform" of the House of Lords. The British Prime Minister has turned the upper chamber into a house of cronies, the Islamists would like to make it a house of imams.

But once you accept the idea of tearing up a thousand years of history, the rest is largely a difference of degree. After a decade of modish vandalism, Blair has abandoned a lot of his sillier novelties because he's belatedly understood the dangers that arise when your citizens start to feel unmoored from their past. Howard didn't need to learn that on the job.

One should be cautious about comparisons between any nation and even its closest allies. Australia, it was pointed out to me on my recent foray, has compulsory voting, unlike the US, where turnout is 50 per cent, give or take, and much of the experts' energy is expended in trying to figure out ways to make sure the opposition's voters stay home.

In theory, Australia's system, by requiring parties to attract the votes of the allegedly less partisan centre, ought to tend towards a more moderate politics.

Yet, among the governments of the main English-speaking nations, the Howard ministry is the least wishy-washily centrist: on jihad, on education, on immigration.

In the US, Republicans are meant to be the daddy party but Bush's riffs on Islam ("religion of peace") and illegal immigration ("family values don't stop at the Rio Grande") are almost all mommy talk and despised as intellectually dishonest by many conservatives.

So how does Howard, with a 100 per cent turnout and all those supposed moderates to woo, get away with the daddy talk? Australians are not ostentatiously right-wing or even terribly conservative.

But it seems that when you toss the entire electorate into the voting booth, there's a big market among the not especially partisan for a party that disdains political correctness.

Alexander Downer's contempt for "lowest common denominator multilateralism" isn't especially right-wing or left-wing: outside the ABC studios and universities and assorted ethnic grievance-mongers, it's an unexceptional observation.

So Australia has, if not quite publicly, suspended the absurd deference to postmodern sovereignty that characterises the UN era. By comparison with Washington, it's honest about and comfortable with a modest, qualified neo-imperialism throughout the Pacific's "arc of instability". The Americans could learn a lot from the policy as well as from the Aussies' ease with it.

Obviously Australia is, in one sense, a small, distant nation and thus has a freer hand on Iraq than the US and on the wider jihad than Britain, which is in danger of turning into Somalia with chip shops.

And, if I'm honest, there are certain aspects of Australian life that I find problematic, from gun laws to a still over-regulated economic environment.

But, granted those and a few other caveats, Australia's is the only Western government on top of the three big challenges facing the developed world: not just the jihad but the more basic issue of civilisational confidence (hence the history summit) and the structural weaknesses of ageing Western democracies: Peter Costello's call for "one for mum, one for dad, one for Australia" is better put than any British minister would dare (though the fecund Blair certainly leads by example).

Just as the advantage of federalism is the local experimentation it allows, so on everything from basic post-9/11 temperament to regional military interventions the present Aussie Government is a kind of useful pilot scheme for the rest of the Anglosphere.

I only wish the ghastly, intellectually barren British Conservatives would learn a thing or two from it.

As for my own nation, I've left Canada out of this discussion but I'm modestly encouraged by small signs of Australianisation.

Our new Prime Minister was in London recently and a couple of local Tories told me how impressed they were: "Splendid chap, this new man of yours, Stephen Howard."

Close enough. When a Canadian PM gets mistaken for John Howard's cousin, that's higher praise than we've had in decades.

Mark Steyn, a Canadian columnist, is a regular contributor to The Australian's opinion page.

tomder55 answered on 09/08/06:

I have nothing but the highest regard for John Howard and yes Bush could learn alot from him. It does help alot that Aussies borders are much better controllable .

Steyn is also right about Canada ;they are slowly seeing the light and abandoning some of the extreme Euro-nanny state notions that stifles individual initiative.

When he leaves office Aussie will lose a treasure . My invitation to him to come here and shake up the US Congress still holds.

paraclete rated this answer Excellent or Above Average Answer

Question/Answer
paraclete asked on 09/08/06 - You have to get OBL before he kills you all

OBL is not the benign leader of Al Qaeda hiding out and neutralised, he is just as dangerous as ever

Osama shown with 9/11 attackers
By staff writers and wires
September 08, 2006 09:12am

A VIDEO tape showing al-Qaeda leader Osama bin Laden apparently meeting the perpetrators of the September 11 attacks has been aired for the first time today.

Arab television channel Al-Jazeera broadcast the tape just days ahead of the fifth anniversary of the deadly strikes on US cities and as the US Senate agreed to fund a new intelligence unit to hunt down bin Laden.

Al-Jazeera said the footage documented the "daily life" of al-Qaeda operatives as they trained and prepared for the attacks in the mountains of Afghanistan.

In one scene bin Laden is seen greeting a fighter against the backdrop of a mountain.

The video is said to feature two of the 19 Islamist militants that took part in the 9/11 attacks, Saudi nationals Hamza el-Ramdi and Wael el-Shemari.

They speak of the situation faced by Muslims in Bosnia and Chechnya.

Fifteen of the 19 attackers on September 11 were Saudis, and Al-Jazeera said it had only aired a few minutes of a document that lasted about an hour-and-a-half.

The footage also shows hand-to-hand combat practice between people wearing masks over their heads.

As the film went to air, the US Senate unanimously approved an additional $US200 million to this year's defence budget to fund an intelligence unit that would seek to hunt down bin Laden.

The measure, approved by a vote of 96 to 0, also requires the US Defence Department to report to Congress every three months about progress made toward apprehending bin Laden, the mastermind of the September 11, 2001 terror attacks.

The legislation was prepared by Democrats Kent Conrad and Byron Dorgan as an amendment to the 2007 Defence Appropriations Bill being debated in the Senate this week.

"Osama bin Laden, the head of al-Qaeda, planned, financed and organised a terrorist operation that killed thousands of Americans. It has now been more than 1800 days since those attacks, and this man is still on the loose," said Mr Conrad.

"The Senate agrees that it is chief among our priorities in the war on terror to bring the mastermind behind September 11 to the justice that a mass murderer deserves," he said.

"Our amendment makes certain that bringing Osama bin Laden to justice will be one of our country's most important priorities, and that he is pursued with real energy and with focus, clarity and a specific set of goals," said Mr Dorgan.

Al-Jazeera today also broadcast a recording attributed to the head of al-Qaeda in Iraq, Abu Hamza al-Muhajer, in which he said he was sure of victory against US-led forces in the country.

The recording was also posted on an Islamist Internet site, but its authenticity could not be immediately established.

In the internet statement, Muhajer urges Sunni Muslims to kill at least one US citizen within the next two weeks.

"Oh followers of (Taliban leader) Mullah Mohammed Omar, oh sons of (Al-Qaeda leader) Osama bin Laden, oh disciples of (slain Al-Qaeda in Iraq leader) Abu Mussab al-Zarqawi ... I urge each of you to kill at least one American within a period not exceeding 15 days," he says.

"I do not doubt for an instant victory (against US-led forces in Iraq)."

Muhajer calls President George W. Bush a "liar" and a "dog".

With Reuters and AFP

tomder55 answered on 09/08/06:

whooop ; they release a 5 year old video of them training . Let me see some recent video of an al-qaeda camp in operation . don't think you'll find it .

paraclete rated this answer Excellent or Above Average Answer

Question/Answer
paraclete asked on 09/07/06 - Now I have an alternative theory

It's multinationals trying to kill us off and steal our land, what's your theory on the crisis?

Everything makes you fat - except food


By David Southwell
Thursday, September 07, 2006 at 12:32pm


Everything makes you fat.

This would seem the inevitable conclusion coming out of the international conference on obesity being held in Sydney this week.

McMansions make you fat. Not fidgeting makes you fat.

Cheap food makes the world fat, excepting those who are starving, of course.

Bad playgrounds make kids fat along with TV ads.

And finally scientific confirmation of what many of us have always insisted big bones make you fat (or in my case just big boned).

Weight gain, it appears, is an extremely complex carbohydrate business with no end of mitigating factors.

I also have long cherished theories about what makes me and possibly you fat. It could be aliens aiming a big fat ray at the earth to make us too roly-poly to resist invasion. Alternatively physicists are bound to soon discover free floating fat molecules in the air that bond themselves to unsuspecting humans, such as myself, probably as a result of global warming or something.

I also blame gyms for having too many mirrors, joggers for associating weight loss with 70s headbands and fridges for being too easy to open.

However for a moment lets humour that lunatic fringe who insist that weight gain essentially and simply comes from eating too much fattening food and exercising too little.

No one to blame but ourselves? Thats not so easy to stomach.

tomder55 answered on 09/07/06:

the nanny state is close to finding solutions to the fatty-gate . Her thighness Hillary Clintoon is on the case (http://www.senate.gov/~clinton/issues/health/index.cfm?topic=obesity) and in Detroit if the Big Mac don't kill you the tax on it will .(http://money.cnn.com/2005/05/09/news/economy/fastfood_tax/)

paraclete rated this answer Excellent or Above Average Answer

Question/Answer
excon asked on 09/07/06 - Inhumane and degrading


Hello:

Well, I'm not very smart. I didn't graduate from Harvard. I didn't graduate from High School. I been in the slam. I live in the underbelly of society.

Yet, I know what inhumane and degrading treatment is. How come the president don't know?

excon

tomder55 answered on 09/07/06:

please codify it . that is what he is asking . Otherwise it sounds like the Potter Stewart's definition of pornography .

excon rated this answer Excellent or Above Average Answer

Question/Answer
MarySusan asked on 09/06/06 - Answers, Finally

President Musharaff is negotiating with bin Laden so the Osama can have his own area of safety and comfort in wilderness Pakistan, safe from attack from domestic and foreign powers. :D Well, it is all clear now, in the past, Bush closed the FBI bin Laden office, and also stated that he didn't think much about bin Laden any more when asked. I think we can safely conclude that Bush hasn't been looking for bin Laden since Tora Bora, or shortly thereafter.

Also, Valerie Plame it has been revealed was working for the ***WHITE HOUSE*** as an undercover agent assigned to find weapons of mass destruction in Iraq. She found none, they ruined her career.

Today, Bush announced that we indeed have secret prisons and that the prisoners will be brought to Gitmo for trial. Gitmo recently underwent construction which included an execution chamber addition. This is in response to a Supreme Court decision that Bush couldn't hold people for an indeterminant time withoug a trial.

I feel much better now knowing the truth about this stuff. All the lies were really infuriating me.

I wonder if he is going to come clean about the War on Iraq?

tomder55 answered on 09/07/06:

correction ,according to the recent book Plame was working for the CIA on Iraq WMD . She was outed by Richard Armitage second to Colin Powell at the State Dept. ;Armitage was no friend of the Bush Iraq policy and it turns out that there was no conspiracy .It was revealed that she was outed because Armitage has a big mouth and loves to be a gossip monger .

as for your other points I do not know all the details about the Pakistani arangement with the Taliban remnants (not OBL) .I do not believe the speculation so I will post on it when I have more details .

The secret prisons were the worse kept secret in the world .Bush also indicated that he wants the ability to continue to hold captured terrorists at undisclosed locations for the purpose of interogation .Perhaps we have squeezed all we can out of Khalid Sheik Muhammed (KSM)and all that's left is a tribunal and to hang his fat ass from the highest yardarm like the murderous pirate he is . I do not understand why you lament his loss of "rights" .

Yesterday Bush also asked Congress to pass long neglected legislation to spell out rules under which the goals of interrogation program may be pursued.

So today, I'm asking Congress to pass legislation that will clarify the rules for our personnel fighting the war on terror. First, I'm asking Congress to list the specific, recognizable offenses that would be considered crimes under the War Crimes Act -- so our personnel can know clearly what is prohibited in the handling of terrorist enemies. Second, I'm asking that Congress make explicit that by following the standards of the Detainee Treatment Act our personnel are fulfilling America's obligations under Common Article Three of the Geneva Conventions. Third, I'm asking that Congress make it clear that captured terrorists cannot use the Geneva Conventions as a basis to sue our personnel in courts -- in U.S. courts. The men and women who protect us should not have to fear lawsuits filed by terrorists because they're doing their jobs.

Like I have said ,Congress has not done it's share in the war on terror and have stood on the sideline as either cheerleaders or hecklers to decisions the President has had to make with very little input from them beside passing blanket war resolutions that authorize the President to use any necessary means to prosecute the war . The courts have done even worse .They have confired rights unheard of before for illegal combatants .Nowhere in the Geneva convention do you see anything that says the courts have a right to intervene or interpret them . That should be done by diplomacy .

Between them and the UN terrorist organizations have been given nation-state rights without any of the concuring nation-state responsibilities .Both have determined that Geneva rights are blanket rights wheter the nation or group is a signatory and have no intention of acting within it's constraints .

From a political point of view it could not have come at a better time . Now let's watch the Dummycrats argue for the rights of terrorists in a national debate right before the elections !!! BRILLIANT !!!!!!

Andrew Sullivan understands and he was rabid over it yesterday :"This is the Rove gambit: make this election a choice between legalizing torture or enabling the murderers of 9/11 to escape justice. "

bwaahaaahaaa haaa !! Of course his characterization of it is a little off. It's NOT a "choice between legalizing torture or enabling the murderers of 9/11 to escape justice". It's a choice between specifying what kinds of interrogation are allowable under our system and letting the murderers of 9/11 escape justice. It's a choice of weighing public risk against our values and reaching some compromise we can live with. Just keep track of the votes so people will remember who voted for what.Maybe if we treat all the al-Qaeda according to Geneva and interrogate no one worth a damn it may impress them so much they'll call the whole Jihad thingy off.

From my point of view the Geneva Conventions no longer serve their stated purpose. A new convention needs to be convened to determine what the status and "rights "of non-state actors are when they wage war . Gee ;perhaps we can invite OBL and representatives of Hamas ,Hezbollah and all the other pirate scum of the world and get their input ! Or maybe we should just ask the NY Slimes to craft a bill of rights for terrorists for us to sign onto .

If buttercups buzz'd after the bee,

If boats were on land, churches on sea,

If ponies rode men and if grass ate the cows,

And cats should be chased into holes by the mouse,

If the mamas sold their babies
To the gypsies for half a crown;

If summer were spring and the other way round,

Then all the world would be upside down.


'The World Turned Upside Down '







MarySusan rated this answer Excellent or Above Average Answer

Question/Answer
ETWolverine asked on 09/06/06 - An interesting analysis.

GLOVES OFF ON IRAN
By JOHN PODHORETZ

September 6, 2006 -- GEORGE W. Bush just delivered what may be the most important speech of his presidency since he went before the United Nations on Sept. 12, 2002, and declared his intention to seek regime change in Iraq.

The time has come, the president all but said yesterday, to take the gloves off with Iran.

"The world's free nations will not allow Iran to develop a nuclear weapon," he said flatly. He prefaced those words by saying that efforts were being made to find a diplomatic solution to the problem. Nonetheless, Bush has now said in the strongest sentence he has yet spoken on the matter that Iran will not go nuclear. He is unconditional about it.

In a carefully crafted speech, Bush laid out the parallels between the extremists of al Qaeda - Sunni Muslims - and the Shia extremists led by Iran. While they both use fiery rhetoric that may be easy to dismiss in certain quarters as an Islamic cultural affect, they are also uncommonly specific about their strategies and goals to achieve their aims.

Using captured documents, he showed how Osama bin Laden and the head of al Qaeda in Iraq have laid out with great precision their strategy to weaken and exhaust the United States and the free nations of the world - a strategy that is having some effect after three-plus hard years fighting in Iraq.

When discussing bin Laden's writings, Bush compared them to those of Lenin and Hitler a decade before they took power. The president pointed out: "History teaches that underestimating the words of evil and ambitious men is a terrible mistake." Then, almost immediately, he jumped from bin Laden to Iran's president, Mahmoud Ahmadinejad.

"Iran's leaders," he said, "have also declared their absolute hostility to America. Last October, Iran's president declared in a speech that some people ask - in his words - 'whether a world without the United States and Zionism can be achieved. I say that this goal is achievable.' Less than three months ago, Iran's president declared to America and other Western powers: 'Open your eyes and see the fate of Pharaoh. If you do not abandon the path of falsehood, your doomed destiny will be annihilation.' "

Bush continued: "Less than two months ago, [Ahmadinejad] warned: 'The anger of Muslims may reach an explosion point soon. If such a day comes, America and the West should know that the waves of the blast will not remain within the boundaries of our region.' He also delivered this message to the American people: 'If you would like to have good relations with the Iranian nation in the future, bow down before the greatness of the Iranian nation and surrender. If you don't accept to do this, the Iranian nation will force you to surrender and bow down.'"

Bush wants the world to understand that he sees the nation of Iran as different only in degree from bin Laden and the terrorists in Iraq, not different in kind. We are to take Ahmadinejad's rhetoric seriously. We are not to dismiss his threats as flowery rabble-rousing but as honest statements of intent.

And if you do that, then the conclusion is inescapable that the world must do everything it can to prevent Iran from joining the nuclear club. "Armed with nuclear weapons," Bush said, Islamic extremists "would blackmail the free world, and spread their ideologies of hate, and raise a mortal threat to the American people. If we allow them to do this, if we retreat from Iraq, if we don't uphold our duty to support those who are desirous to live in liberty, 50 years from now history will look back on our time with unforgiving clarity, and demand to know why we did not act. I'm not going to allow this to happen - and no future American president can allow it either."

So there it is. A week after Iran declared its intention to continue uranium enrichment, the president of the United States has said in no uncertain terms that it will be stopped - that the failure to stop it would lead history to judge him, us and the world in the harshest possible terms.

Like most people, I've presumed for the past few years that our commitment in Iraq and the extreme difficulty of targeting the proper sites had basically foreclosed a serious military option in Iran. Certainly the hesitant and cautious behavior of Secretary of State Condoleezza Rice in the past few months suggested as much.

Now it seems to me that, barring a miraculous change of heart on the part of the Iranian regime, a military strike is all but inevitable. Bush himself will view his own presidency as a failure if he doesn't act.

So act he will.

jpodhoretz@gmail.com
-----------------------------

Do you think we are headed in the direction of military action in Iran, or is this just rhetorric. Remember, we are talking about GWB... not exactly a guy who is afraid to use the military.

I happen to agree with Podhoretz' analysis. Bush tends to mean what he says and say what he means. If he says that "The world's free nations will not allow Iran to develop a nuclear weapon," the I am willing to guess that he means it.

What is your opinion?

tomder55 answered on 09/06/06:

I think we are already in a shooting war with Iran and it is rediculous to pretend otherwise .

Let's forget the history of the last 20 years and just deal with events in the last couple of years . First and foremost the Iranians have actively funded and supplied various groups in Iraq that have attacked and killed American forces . The pipeline of supplies is the strategic modern equivalent of the Ho Chi Min Trail and our efforts in Iraq cannot succeed unless it is shut down .

June ;after Zarqawai was offed it looked to me that our efforts in Iraq were finally bearing fruit . There was a unity gvt. in place .He had been betrayed by his own patrons .His goals of fomenting an Iraqi Civil war was not being realized as the Shia population was not responding to the provoactions by al-Qaeda The Sunnis had gone to the polls and were looking for a political solution .The US was playing honest broker between the ethnic groups .There was hope of US troop reductions as signs of stabilty were being realized and the Iraqi Army was growing more proficient.

The key to success was in Iran .As things looked there would've been a neutral Iraq that would no longer be the existential threat to Iran that the Saddam gvt was . It would've been shia majority that included Sunnis and Kurds. It appeared to be a done deal .

But Iran wants more. They are not satisfied with a neutral Iraq ;they want to dominate it . Abdel Aziz al-Hakim, the head of the Supreme Council for Islamic Revolution in Iraq traveled to Tehran and received new marching orders . Mookie al-Sadr's militia commenced an offensive.Shiite death squads attacked Sunni populations, and Sunni retaliated . From nearly having a political accommodation, the situation in Iraq deteriorated .It can no longer be spun any other way .The Iranians did not trust the concept of a neutral Iraq and they think that the US is weak so why should they settle for it when the alternative is to turn Iraq into a satellite state . They will use their proxies in Iraq to tie us down . They calculate ,(mistakenly I believe... although they may be correct when they gage the American domestic politics),that the US will not be in a position or have the will to attack them .

The Iranians are striking while the iron is hot and what's at stake is the geo-politics of the Persian Gulf and possibly the whole ummah . They through their proxies have created the recent cycle of violence that appears to be spinning out of control .They have shown that the newly elected gvt. unable to control the security situation ..not even in the capital city .With US troop help that situation may change but until the full Iraqi Army can stabilize the country then the situation will not change . We have taken troops from the provinces and when we do the security in those regions also becomes compromised. This gives an appearance of impotency that is not lost on anyone in the country who is sitting on the fence waiting to decide who's lot they will side with . The Iranians at this point are in control of the tempo and inititative . All we do is react.

If Iraq were to split up as some have advocated naively as a compromise solution then the Iranians know that they can dominate southern Iraq and will have added to their assets the Basra oil fields and will have attained a direct land rout to attack Kuwait and Saudi Arabia .

If you took the US military out of the region there is no doubt that Iran has the most powerful military . Without us there they could easily overwhelm all the gulf states . I am pretty sure that sometime in the near future they will incite the shia populations of the gulf states to attack their oil assets .

The shia believe their time has come . Perceptions being what they are ;the Hezzies in Lebanon were prepared for war and the Israeli's weren't . Short of a complete victory by Israel ,they have been able to claim victory by breaking the aura of Israeli invincibilty . This will further embolden the Shia in the region . They would also obtain this if the US prematurely withdrew from Iraq and the nation remained whole but an Iranian satellite .

But we cannot achieve any other situation with the current status quo . The fact is that the US is at war with Iran but we have not committed to it and I'm not sure we will . If the Mahdi-hatter wasn't such a nut job he could let this simmer without boiling over until the US gets completely sick of it's commitment .The reason this is so is because it will take a major military effort and a national commitement to wage this war and frankly there are too many people here who would rather we be in a pre-9/11 world and have not come to grips with what it will take to defeat jihadistan .

ETWolverine rated this answer Excellent or Above Average Answer
HANK1 rated this answer Excellent or Above Average Answer
Itsdb rated this answer Excellent or Above Average Answer

Question/Answer
paraclete asked on 09/06/06 - Let's get to the heart of the problem?

Palestinians on the verge of civil war
Paul McGeough, Chief Herald Correspondent in Gaza City
September 6, 2006

The power fails. Abu Annis stabs at the keys of his mobile phone, sparking a glimmer of light as he and six hardcore fighters for Islamic Jihad talk about life on the Gaza Strip - bombs, death and the daily grind.

Anxious that the late-night movement of such a group of men might be detected by the Israeli surveillance drones which are overhead constantly, they straggled in one at a time to this cinder-block home in Gaza's Jabaliya City. All defer to Abu Annis who, at just 22, is their appointed spokesman.

The conversation begins as most do in the Occupied Palestinian Territories - on the mind-numbing minutiae of the conflict with Israel. But the stern-faced young man demands light so that nothing will be lost when he addresses an ominous new twist in the turmoil of the Middle East - the threat of a Palestinian on Palestinian civil war.

The Palestinians have been isolated by the world since early this year when they elected a government controlled by Hamas, an Islamist movement with ties to outcast regimes in Iran and Syria. Now, in Gaza and on the West Bank, many of them cower in their homes as Hamas gunmen clash almost daily with loyalists of the regime they defeated.

It is their worst nightmare. Caught between the secularists of the late Yassar Arafat's failed Fatah movement and the fundamentalists of Hamas, they fear that the dream of their own separate state might shrivel on the scorched earth of the on-going clash between radical Islam and the West.

Amid the distractions of the Iraq war and, more recently, the new Lebanon war, little attention has been given to overlapping efforts of Washington, Europe and Israel to weaken or destroy an Arab rarity - the democratically elected Hamas government of Prime Minister Ismail Haniya.

Since late June, when militiamen from Hamas and other factions tunnelled under the Gaza-Israel border to capture an Israeli soldier, Gaza has been subjected to unrelenting Israeli retaliation - aerial and tank bombardments and border skirmishing have killed almost 200; more than 30 Hamas Cabinet ministers and MPs have been snatched in Israeli raids on the West Bank; and border closures have made a virtual prison of Gaza.

Earlier in the year, Washington, Brussels and Tel Aviv paralysed the working of the new government - and the local economy - when they cut vital funds transfers to back their demands that Hamas abandon its refusal to recognise the state of Israel and that it renounce violence.

And because Hamas is radical and Islamist - with friends in Tehran and Damascus - other US-friendly but anti-democratic Arab regimes sit on their hands as Palestine burns.

This is how the stage is set.

The popularly elected president Mahmoud Abbas is from Arafat's long-dominant Fatah faction, a secular nationalist movement that traditionally had had a mortgage on Palestinian power and which, over the years, has committed itself to a negotiated peace settlement with Israel. Despite the swing to Islam in the election, Abbas retains control of Palestinian negotiations with Israel and the world.

More importantly, Abbas commands the loyalty of much of the 35,000-strong Palestinian security forces, almost exclusively Fatah loyalists, who have taken to launching armed attacks on government buildings because they have not been paid for six months as a result of the global funding freeze.

Abbas also has seized control of much of the government's finances and communications networks.

Embittered by defeat, Fatah refused an invitation from Hamas to join a government of national unity in the aftermath of the January poll, prompting the International Crisis Group to conclude: "It's a question of power, pure and simple. [For Fatah,] allowing their main rivals to exercise it is inconceivable and too great a threat to their own positions".

Some in Fatah advocate military confrontation with Hamas - soaring prices for black-market weapons suggest that both sides are stockpiling arms and each is blaming the other for attempted and successful assassination and abduction of key figures from the two camps.

Despite its legendary corruption and abuse of power in office, Fatah is unable to accept its electoral defeat. The inexperienced Hamas, on the other hand, did not expect to win government and has been slow to grasp the levers of power decisively but, in response to Abbas's grip on the security forces Hamas has started setting up its own parallel forces. The tension is nerve-tingling.

At the mobile-phone-lit meeting at Jabaliya, the seven from Islamic Jihad set out their own unambiguous position. They are fighting to the death for the liberation of all of historic Palestine - "this is a religious war against the Jews" - but for them the showdown between Hamas and Fatah is an unnecessary distraction.

The spokesman Abu Annis steeples his fingers as he explains: "The different militias run a joint operations room to coordinate attacks.

"We are trying to hold things together. The political leaders work all the time to stop Hamas and Fatah fighting each other and we work on heightening an atmosphere of war to make them focus on Israel as the enemy, instead of each other".

Among the ruins of Netzarim, utterly destroyed by the Israelis last year when they abandon all their fortified settlements in Gaza, the Herald chanced upon the 30-something brothers Rubin and Khalid Khadoura, who were picnicking with their families in the cool shade of a spreading tree.

As the children fossicked in the rubble that had been Israeli homes and workshops, the brothers complained about the new siege and Rubin had frank words of caution for the Hamas leadership: "If I was the Prime Minister, I'd be thinking about the plight of the people without salaries because Hamas will never change the ideology of the people unless we have a good life."

But this was leavened with criticism of the international community and contempt for Israel: "They have made a prison of Gaza with their checkpoints and border controls that block food getting in and our fruit and vegetables going to markets. What's wrong with our democracy that they think they can arrest our leaders? Jimmy Carter vouched for the fairness of the vote, but our democracy is not acceptable to the world - is it?"

Amid another pile of rubble, not far away in Beit Hanoon, 65-year-old Mohammad Hussein affirms his commitment after what he claims has been almost 30 years as a resistance agnostic.

The new fire in his belly is caused by an Israeli air-strike that pulverised a four-story building in which his children lived and from which he ran one of the biggest supermarkets in the area.

"I worked for 22 years in Saudi Arabia for the money to build it," he says. "The Israelis call at one o'clock in the morning on my 17-year-old son Hussein's mobile, saying everyone must be out of the building in 15 minutes.

"'You must leave immediately', they tell him. When Hussein asks if he can be sure the call is real, the voice tells him to go outside, to look up and he will see the F-16 circling.

"Minutes later two missiles hit one side of the house. Neighbours rushed in, trying to help us get stuff out of the rest of the building but the Israelis call again, saying to get the people away because there will be another strike - six minutes later two more missiles knock down the rest of the building."

Surrounded by several of his sons, the old man bounces his one-year-old grandson Osama on his knee. Shouting to be heard over each other, all argue that such attacks only drive them into the arms of the militias.

"They oblige us to join the resistance," he says, before he delivers the Palestinian distillation of decades of failed diplomacy and ineffectual war:

"We have made agreements with Israel, but it doesn't respect them ... they don't want peace, they just want to drive us from our land. We have had six years of fighting in this latest round because they would give nothing in the previous six years. Do you really think things would be any different if they could bring Fatah back?"

Then Ribhi, the father of little Osama, demands silence because he has something to say: "You see this one-year-old? He will grow up to a bomber in Tel Aviv if the Israelis keep killing our people. He is my boy, but he is not as priceless as Jerusalem."

The grandfather gets up to leave, closing the family's case as he wanders back into the rubble: "We have all travelled. My sons study in Greece and we know what a good life is. But we all come back here because we don't forget where our roots are; we return to where we want to die." As he goes, he jabs a finger at his collapsed building: "For us? This is nothing - we'll build again".

To another house on a rise overlooking the Mediterranean north of Gaza City - a palatial residence that has not been bombed.

It is the home of Dr Nabil Shaath, who was foreign minister in the Fatah-led former government. Here, the swimming pool, the manicured gardens and the Asian household staff are read by ordinary Palestinians as proof of the rampant corruption that caused voters to turn against Fatah.

A pistol-packing bodyguard hovers as a seemingly contrite Shaath canvasses his party's options: "We've made many mistakes. We were angry after the election, so we refused to be a part of a Hamas government ... but now they need partners and we might be able to help.

"But we can't make the government that Bush wants - that is not how democracy works. The US dictated that there had to be an election, but it can't dictate the outcome".

The former minister acknowledges - but denies - allegations that Fatah has been instrumental in US-Israeli efforts to bring down Hamas. He insists: "Not true. Our president is a good friend of the US, but he has not been clubbing together with Washington and the Israelis ... if fact, the Israelis knife him in the back almost daily. Our interest is in destroying the occupation - not Hamas.

"But these stupid sanctions and the jailing of their leaders are helping Hamas - it's an excuse for their failure as a government. Hamas doesn't suffer one bit - it gets funds from Iran and the Arab world and they make sure that their own people don't suffer."

Shaath warns of dire consequences. The American campaign, he says, is destroying civil society, governance and any remaining Palestinian support for the peace process. He seems to have one-year-old Osama in mind as he finishes: "They are radicalising people and creating a governless society - just as they have done in Lebanon and Iraq."

Despite Israel's round-up of Palestinian officials on the West Bank, Hamas' Minister for Refugee Affairs, Professor Atef Odwan, tools around Gaza in a beaten-up Volkswagen Passat.

The presence of just a single bodyguard supports the view of many Palestinians that the Israelis have snatched none of Hamas' Gaza-based ministers or MP because they see the whole strip as an effective prison. Odwan explains: "They control our borders, the sea and the air, so why would they bother coming back in to face hatred and resistance? So I can move around. "

In his spartan office, the minister sets out the various elements of an emerging Hamas compromise that remains unacceptable to Israel and the key international players: "We have offered a 30-year truce in exchange for a Palestinian state within the 1967 borders, freedom for Palestinians held by the Israelis and complete sovereignty."

He readily acknowledges the glaring omission - there is no explicit recognition of Israel. He adds: "Its ཿ borders is de facto recognition, isn't it?"

So what does the Islamist professor make of democracy when Washington is leading a charge that has paralysed the Hamas government? "We have discovered that the Americans are liars; the Europeans too - they are not the democracy lovers they claim to be. The all just push their own interests."

On the sidelines, there is a loud 'we told you so' from Islamic Jihad.

As four great speakers bounce thumping jihadi anthems off the drab walls of Jabaliya City, young men with AK-47s slung over their black-shirted shoulders gather for the funeral of a fighter who was killed in a border clash with Israeli forces the previous night.

Hundreds assemble in a big street tent in which the 20-year-old Mohammad Al-Nedar's family receive condolences. Overseeing it all, Islamic Jihad political leader Khalid Al-Batsh eyes the boy's father and explains: "We don't leave the martyr's family alone. He won't have time to think of his loss - people will keep coming for three days and slowly he will forget his sadness".

As meaty dates and bitter Arabic coffee are passed around, Al Batsh commiserates with Hamas for the pressure it has come under: "They were pushed into a corner after the election and ordered to give immediate answers, but Hamas can't deal that way. The Americans and the rest won't even let Hamas prove that it accepts democracy ... but the reality is that Israel will give nothing.

"The worry now is after their debacle in Lebanon, the Israelis will be looking for a quick victory in Gaza to prove to their people that they are not as weak as they were revealed to be in Lebanon. And the US will push for superficial movement in the peace process as a gift to those heroes in Riyadh, Cairo and Amman ..." - he is talking about Washington's key Arab allies - "for keeping their mouths shut during the destruction of south Lebanon."

Dr Ibrahim Ibrach, a political analyst at Gaza's Al Azhar University, comes at the crisis from a different angle: "This place wasn't ready for a vote - how do you have proper elections when the country is occupied and instead of political parties, we have militias?

"And now Israel wants the internal Palestinian friction - it doesn't want to give Mahmoud Abbas a peace deal and it doesn't want Hamas to falter completely."

The Herald recounts an exchange with medical officials at Gaza's Shifa Hospital - they spoke of Palestinians crying as they stood in ATM queues to withdraw the last funds from their bank accounts and of rising complaints that the electoral endorsement of Hamas had been a case of 'putting too many eggs in the one basket'.

Ibrach agrees that the current crisis has eroded some of Hamas' popular support. But he makes a bold prediction: "They remain strong and if an election was held tomorrow, Hamas would probably win ... because the Fatah factions are fighting among themselves and the Americans and the Israelis are not going to give Abbas any kind of winning peace deal.

"A government of national unity might work. But if Hamas fails - or if it is made to fail - Palestinian voters will not rush back to Fatah's corruption and its failed peace efforts. The voters will be looking for someone else - that's why the risk of civil war is so real".

In May, all sides in the Palestinian equation seized on help from an unlikely quarter - Israel's jails. Five revered Palestinians inmates representing the key factions drew up their own plan for national reconciliation and demanded that it be adopted.

Intended more to bring Palestinians together than to appease Washington, it called for an independent state on all of the land seized by Israel in 1967, a right for the millions of Palestinian refugees around the world to return and for resistance by all means within the Israeli-occupied territories - not in Israel proper.

The prisoners' document acknowledged the role of the Palestinian Liberation Organisation, a bastion of Fatah power, but it argued that Hamas has to be brought into the PLO. Importantly, it urged the formation of a coalition government of Hamas and Fatah.

Abbas grasped the document and ran, proclaiming that if Hamas did not accept it, he would put it to a referendum within 40 days. It was a bold gamble - then president was banking on Hamas acquiescing or being seen by voters as the party-wreckers - and it didn't work.

Amidst the squabbling, the Hamas signatories to the prisoners' document withdrew their signatures. Abbas's objective was to see Hamas ousted, but as haggling continued it was overshadowed by renewed violence.

When Israel assassinated a key Hamas figure on June 9 and blame for the death of seven members of a single Palestinian family in a strike on a Gaza beach on the same day was laid at the Israelis' door, Hamas announced that its military wing was abandoning a ceasefire to which it had pretty well held for 16 months.

Hamas was playing hard ball. It had demonstrated that it could put its missiles on hold, and now it was sending a message through one of its senior MPs who told an International Crisis Group researcher: "The alternative to our government is a resumption of suicide attacks".

Finally, in the last week of June, virtually all the factions signed off on acceptance of the prisoners' initiative - but hardly anyone noticed because on the same day Hamas and two other militias mounted a combined operation in which the capture of Israeli Corporal Gilad Shalit brought down the wrath of Israel on all of Gaza.

Corporal Gilad remains a prisoner. And under constant Israeli fire, it is virtually impossible for the Palestinian leadership to deal with the immediate internal political challenge - to agree on the shape of a unity government. More difficult still, will be finding sufficient common ground between Hamas and Fatah for the two to work together, even in the short term.

Hamas believes it can retreat underground and still command broad Palestinian support - a poll in July found that almost 80 per cent of Palestinians supported the June 25 abduction of the Israeli soldier, a reflection of their deep bitterness over the detention of thousands of Palestinians by Israel.

Fatah might win back power. But if it is seen to have done so with foreign or Israeli support, it might well be a Pyrrhic victory ... particularly given the widespread acceptance in Gaza of evidence that Fatah officials urged European governments to tighten the screws on Hamas by withholding aid.

Both sides are counting and cursing - Tel Aviv says about a dozen Israelis have died and more than 300 Qassam rockets have lobbed into its territory since June 1. The Palestinians say more than 9000 Israeli missiles into its territory and almost 200 dead Palestinians.

Instead of joining forces to fight Israel, the Palestinian factions have turned on each other, with the president and his Fatah backers trying to undermine Hamas and Hamas accusing them of treason because of their support for the international effort to isolate the Hamas government. These days, their separate rallies condemn each other as often as they condemn Israel.

When the Herald interviewed Ghazi Hamad, the Hamas government's official spokesman, he claimed the local factions had seen the light and that they could sort out their differences. But a clearer indication of his thoughts appeared just days later in the newspaper Al Ayyam.

Billed as self-criticism, but directed at Fatah and some of the lesser factions, he was scathing: "Gaza is suffering under the yoke of anarchy and the swords of thugs. [After so much optimism when Israel pulled out a year ago] life became a nightmare and an intolerable burden".

Against the backdrop of Hamas' recently abandoned ceasefire, he admonished the others for the death and injuries inflicted by Israel's retaliatory attacks: "We've all be attacked by the bacteria of stupidity. Please have mercy on Gaza. Have mercy on us from your demagogy, chaos, guns, thugs [and] infighting. Let Gaza breath a bit - let it live.

""When you walk around in Gaza, you cannot help but avert your eyes from what you see: indescribable anarchy, policemen that nobody cares about, youth proudly carrying weapons, mourning tents set up in the middle of main streets ... Gaza has turned into a garbage dump, there is a stench ...

"The government cannot do anything, the opposition [Fatah] looks on from the sidelines, engaged in internal bickering, and the president has no power... We are walking aimlessly ... We applauded the elections ... but there has been a great step backwards. We spoke of national consensus, [but] it turned out to be like a leaf blowing in the wind..."

With all sides on a hair-trigger, the only identifiable circuit breaker is fear of the consequences of internal war - but despite local anger at the new levels of hardship, Hamas is judged to be the more popular and stronger and to be perceived by most Palestinians as the victim of foreign interference.

Fatah might regain power, but beset by its own internal factional and generational wars, the party's resurrection would do little to break the new hand of radical Islam in the Palestinian equation.

Like Hezbollah in Lebanon, Hamas is probably here to stay. Much is made of its refusal to explicitly recognise Israel, but local and international players can hardly be shocked - Washington's autocratic friends in Riyadh still refuse to do so; likewise Rabat; and Amman and Cairo both stonewalled on recognition until they negotiated their respective peace treaties with Tel Aviv.

After watching Israel's destruction of Lebanon, the Beirut-based Arab commentator Rami Khouri, upbraided the Americans and the Israelis for their obsessional interest in symptoms rather than root causes.

Noting that the Middle East crisis predated the arrival of both Hezbollah and Hamas on the scene, he writes: "Every tough issue in this region - Lebanon, Iraq, Syria, Iran, terrorism, radicalism, armed resistance groups - is somehow linked to the consequences of the festering Israeli-Palestinian conflict.

"The politicians and government leaders who dominate the region, or engage it from Western capitals, all look like rank amateurs or intemperate brutes as they flail at symptoms instead of grappling with the core issue that has seen this region spin off into ever greater circles of violence since the 1970s."

tomder55 answered on 09/06/06:

"Occupied Palestinian Territories " ...no bias in his word selection at all .For the record Paul McGeough ,you are living in the past ;the Israelis disengaged from Gaza .Gaza is in complete control of competing gangs that call themselves political movements .

It is their worst nightmare. Caught between the secularists of the late Yassar Arafat's failed Fatah movement and the fundamentalists of Hamas, they fear that the dream of their own separate state might shrivel on the scorched earth of the on-going clash between radical Islam and the West.

The majority of the people voted for these competing gangs .

the secularists of the late Yassar Arafat's failed Fatah movement

bwaaahaaahaaahaa !!!!! If they are secularist then Choux is an ayatollah.

Since late June, when militiamen from Hamas and other factions tunnelled under the Gaza-Israel border to capture an Israeli soldier, Gaza has been subjected to unrelenting Israeli retaliation - aerial and tank bombardments and border skirmishing have killed almost 200; more than 30 Hamas Cabinet ministers and MPs have been snatched in Israeli raids on the West Bank; and border closures have made a virtual prison of Gaza.

have they considered that if only they would return the kidnapped soldier and stop shelling Israel from Gaza (a point that must've slipped McGeough's pinhead )

Earlier in the year, Washington, Brussels and Tel Aviv paralysed the working of the new government - and the local economy - when they cut vital funds transfers to back their demands that Hamas abandon its refusal to recognise the state of Israel and that it renounce violence.

and does McGeough consider it unreasonable to attach these basic conditions for continuing aid ? "we will destroy you but first you must give us the funds to purchase the bombs "

Abbas and Fatah committed itself to a negotiated peace settlement with Israel " ?????? must be talking about a different Fatah then I remember .

More importantly, Abbas commands the loyalty of much of the 35,000-strong Palestinian security forces, almost exclusively Fatah loyalists, who have taken to launching armed attacks on government buildings because they have not been paid for six months as a result of the global funding freeze

pardon me if I don't shed a tear . They had years as leaders of the Palestinians to build a working economy . Why doesn't McGeough ask where all the millions of funds in aid sent over these many years is ? Lining the pockets of Fatah leadership is the correct answer . Abbas also has seized control of much of the government's finances and communications networks..... of that I have no doubt .

Jimmy Carter vouched for the fairness of the vote oh stop your killing me ROFLMAO!!! Carter also vouched for the fairness of the Venezuela elections ;and every other despots effort to legitimize their position by sham elections .

sorry ;can't deal with this anymore lets put it this way . It would take alot of toilet paper to wipe up all the pablum in this article .




paraclete rated this answer Excellent or Above Average Answer

Question/Answer
paraclete asked on 09/06/06 - Not to be alarmist, but, is he trying to tell you something?

Bush renews 9/11 national emergency

From correspondents in Washington

September 06, 2006 10:32am
Article from: Agence France-Presse

US President George W Bush has renewed the national emergency he declared days after the September 11, 2001, attacks, warning "the terrorist threat continues".

The White House announced the move one day befor Mr Bush was to give the third in a series of speeches on the war on terrorism ahead of November US legislative elections expected to be overshadowed by the unpopular Iraq war.

"Because the terrorist threat continues, the national emergency declared on September 14, 2001, and the measures adopted to deal with that emergency must continue in effect beyond September 14, 2006," Mr Bush said.

"Therefore, I am continuing in effect for an additional year the national emergency I declared on September 14, 2001, with respect to the terrorist threat."
======================================================
Interesting that he has made the move a couple of days after Al Qaeda made a call for America to convert to Islam.

tomder55 answered on 09/06/06:

not at all . it was due to expire so he renewed it another year. what has it meant to the average American besides increased security procedures at airports and an occasional road check ? Absolutely nothing .

ETWolverine rated this answer Excellent or Above Average Answer
Itsdb rated this answer Excellent or Above Average Answer
paraclete rated this answer Excellent or Above Average Answer

Question/Answer
Itsdb asked on 09/05/06 - No worries...

...the latest al-Qaeda was just a demonstration of the softer side of terrorism reports al-AP.

Latest al-Qaida message seen as PR bid

By SALAH NASRAWI Associated Press Writer
2006 The Associated Press

CAIRO, Egypt The new al-Qaida video featuring an American calling for his countrymen to convert to Islam raised fears it signaled an imminent attack, but experts in the region said Sunday it is more likely a bid to soften the terror group's image.

Adam Yehiye Gadahn, a 28-year-old American who the FBI believes attended al-Qaida training camps in Pakistan and served as an al-Qaida translator, also urged U.S. soldiers to switch sides in the Iraq and Afghan wars.

He appeared in a 48-minute video that was posted Saturday on an Islamic militant Web site along with footage of al-Qaida's No. 2 leader, Ayman al-Zawahri, who gave only a brief introduction to Gadahn while also calling on Americans to convert to Islam.

There have been widespread reports that some Muslim religious figures strongly criticized al-Qaida chief Osama bin Laden over the Sept. 11 attacks, saying he failed to follow directives in the Quran that require potential victims be warned that conversion to Islam could save them.

The criticism led to speculation after Gadahn's appearance that the Saturday video meant a warning was being issued and a new attack was imminent.


But experts discounted those fears.

"This is not a warning for an attack. It is rather a speech aimed at winning the Americans' sympathy and understanding," said Gamal Sultan, editor of the Islamic magazine Al Manar.

Columnist Mishari al-Thaydi of the London-based newspaper Asharq Al Awsat agreed, saying al-Qaida is trying to portray itself as a group with a religious mission, not a terrorist movement.

"They have always been accused of lacking a program, that they are just a bunch of zealots," al-Thaydi said. "People accuse them of forgetting the essence of Islam _ conversion of nonbelievers."

"By using this American," he added, "al-Zawahri is saying that he is a preacher and not a terrorist. He wants to take back the initiative which has been lost in the midst of terror."

Hani el-Sibaei, a former Egyptian militant who fought with al-Zawahri in Afghanistan and now lives in exile in London, said Gadahn's American roots make him the perfect spokesman.

"Al-Qaida uses him because he speaks their language and can convey the message better than others," said el-Sibaei.

It was the second time Gadahn appeared in a video with al-Zawahri. In a July 7 message marking the first anniversary of the terror attack on London commuters, Gadahn appeared briefly, saying no Muslim should "shed tears" for Westerners killed by al-Qaida attacks.

Some reports have identified Gadahn as al-Qaida's "propaganda chief," responsible for putting together the slick videos made by the group's al-Sahab media operation.

Thomas Hegghammer, a research fellow at the Norwegian Defense Research Establishment who studies militant Islamic Internet sites and videos, said Gadahn "has been instrumental in the English-language productions" of al-Qaida.

"He has appeared on a few videos speaking himself, but you've also seen a few videos with speeches by bin Laden and al-Zawahri with subtitles in perfect English," Hegghammer said.

He said Gadahn was likely one of a very few Westerners in a senior position in al-Qaida _ if not the only one. "There are certainly Western converts who have played a role in militancy, but none who are that high up."

Hegghammer said Gadahn is "a remnant of the original al-Qaida" that has fragmented since the Sept. 11 attack into a loose network of cells.

"And I don't think there are many like him. That's why we've seen him many times," Hegghammer said. "The obvious advantage (of his appearances) is it makes headlines in the West. ... He is an asset to them, definitely. My guess is the Americans are extremely keen on capturing him."

~~~~~~~~~~~~~~~~~~~~~~~~~~~~~~~~~~~~~~~~~~~~~~~~~~~~~~~

>>he failed to follow directives in the Quran that require potential victims be warned that conversion to Islam could save them.<<

And I thought Islam didn't teach violence. In spite of their effort to spin the softer side of al-Qaeda, this is a rather profound admission by al-AP. The "essence of Islam _ conversion of nonbelievers," apparently focuses on potential victims.

I reckon it should be comforting to know these Muslim "experts discounted those fears" of an imminent terrorist attack on those potential victims.

Comments? Come on now, tell me how Islam is a religion of peace again...

tomder55 answered on 09/05/06:

I must've missed the part about converting when I read OBLs 1998 fatwa :

The ruling to kill the Americans and their allies -- civilians and military -- is an individual duty for every Muslim who can do it in any country in which it is possible to do it, in order to liberate the al-Aqsa Mosque and the holy mosque [Mecca] from their grip, and in order for their armies to move out of all the lands of Islam, defeated and unable to threaten any Muslim. This is in accordance with the words of Almighty God, "and fight the pagans all together as they fight you all together," and "fight them until there is no more tumult or oppression, and there prevail justice and faith in God."

This is in addition to the words of Almighty God: "And why should ye not fight in the cause of God and of those who, being weak, are ill-treated (and oppressed)? -- women and children, whose cry is: 'Our Lord, rescue us from this town, whose people are oppressors; and raise for us from thee one who will help!'"

We -- with God's help -- call on every Muslim who believes in God and wishes to be rewarded to comply with God's order to kill the Americans and plunder their money wherever and whenever they find it. We also call on Muslim ulema, leaders, youths, and soldiers to launch the raid on Satan's U.S. troops and the devil's supporters allying with them, and to displace those who are behind them so that they may learn a lesson.


Itsdb rated this answer Excellent or Above Average Answer

Question/Answer
paraclete asked on 09/04/06 - Sad to relate,a great conservationist has gone?

Well no doubt wildlife everywhere and particularly crocodiles in Northern Australia breathed a siegh of relief today when it was learned one of their own had had the last say. Crikey, it's not fair you know, Steve was a great advocate of wildlife protection but he went as he lived, doing what he liked to do.

Steve Irwin killed by stingray


It was quite rare for someone to die from contact with a stingray ... Stingrays were dangerous if provoked



David Williams
September 4, 2006 - 4:55PM

Television personality and environmentalist Steve Irwin has died after being stung by a stingray while filming off north Queensland.

Known worldwide as the Crocodile Hunter, the 44-year-old was famous for his enthusiasm for wildlife and his catchcry "Crikey!"

The Queensland Police Service issued a statement saying Irwin collapsed after being stung at Batt Reef, Low Isles, off Port Douglas about 11am. He had been filming a documentary.

"Steve was hit by a stingray in the chest," said local diving operator Steve Edmondson, whose Poseidon boats were out on the Great Barrier Reef when the accident occured.

"He probably died from a cardiac arrest from the injury," he said.

Police said that, after the attack, Irwin's crew called for medical treatment at 11am and the Queensland Rescue Helicopter responded with a doctor and paramedic on board.

Puncture wound

Irwin had a puncture wound to the left side of his chest and was pronounced dead at the scene.

Police said Irwin's family had been advised and Irwin's body was being flown to Cairns.

It is believed his American-born wife Terri was trekking on Cradle Mountain in Tasmania when the accident happened.

Police in Tasmania say she had been told.

The Irwins have two children, a daughter, Bindi Sue, 8, and a son, Robert Clarence, usually known as Bob, 3.

When asked if he had ever heard of anyone dying from a stingray barb, Matthew Hurley, general manager of Quicksilver Group, whose company has taken tours to Low Isles for 26 years, said: "No, definitely not.

"We've never heard of or been involved with anything like that."

Ross Coleman, acting director at at University of Sydney Institute of Marine Science, told smh.com.au it was "quite rare" for someone to die from contact with a stingray and he couldn't recall hearing of another incident.

Stingrays were "dangerous if provoked", he said.

"As a recreational diving instructor you hear of people getting injured by standing on them ... but they rarely die."

'The zoo will go on'

Irwin's wife Terri would not close down the zoo, predicted Jim Dalrymple, whose local irrigation firm helped maintain the water supplies to Irwin's Australia Zoo in Beerwah on Queensland's Sunshine Coast.

The zoo is the biggest local employer with 550 staff, Mr Dalrymple said.

"I managed an irrigation business in Beerwah and had occasions where I served Steve personally.

tomder55 answered on 09/05/06:

how cool is it to go out doing something you love doing !! at the end of the day his success was because he was an adventurer as well as a wild life lover who pushed toe envelope whenever possible .

It worked ;he reached an audience that tunes into extreme sports and survivor reality shows. While he was having a ball he was also reaching them/educating them . A generation of youth learned to appreciate wildlife watching his show.

I am remined of the movie 'Second Hand Lions '. To summarize the plot a boy is thrust into the care of his adventurous aging uncles. He comes to realize that tales of their exploits in their youth that seem more real in Arabian Nights are true .Age has not tempered their enthusiasm for life . They die as a result of their life style but not before they have lived ;and live their life they did ...to the fullest .

That is how I will remember Steve Irwin .

paraclete rated this answer Excellent or Above Average Answer

Question/Answer
excon asked on 09/05/06 - Sailboats


Hello:

I bought a pool table for my boat. However, my balls dont stay still. How do I keep my balls from moving about?

excon

tomder55 answered on 09/05/06:

Steve sorta beat me to it . I say put them in a tight rack ......better to break em that way .

excon rated this answer Excellent or Above Average Answer
paraclete rated this answer Excellent or Above Average Answer

Question/Answer
paraclete asked on 09/04/06 - Better solutions needed now.

The pace of climate change is taking us by suprise. I'm suprised by the duration and severity of the current drought, now in its fifth year. Even drought hardy native plants are succumbing to the lack of moisture. If this keeps up in 30 years I will be living in a desert and so will a much larger proportion of the Earth's population. The time has come to stop protecting the overpaid energy/oil company and car industry executives and make radical changes to the way we use the Earth's resources. I'm pleased people like big Arnie are couragious enough to get on with the job.

Scientists 'surprised' on climate change

By Evan Schwarten

September 04, 2006 04:27pm
Article from: AAP


AUSTRALIA'S rapid climate change had caught scientists by surprise, a leading water expert said today.

Professor Peter Cullen, from the National Water Commission, said experts had expected the changes, which have left much of the country suffering drought conditions, but thought they would take much longer to take effect.

"I don't think any of us expected the climate change we have experienced over the last five years. I was expecting climate change but I was expecting it to take 30 years," he said.

Prof Cullen said Australia was drying out quickly and with water restrictions already in place in many areas, governments needed to consider all available options, such as recycling and desalination, to prevent an impending water crisis.

"We've got to look at the full range of options, I don't think we can afford to be doctrinaire and throw anything out," he said.

Prof Cullen said putting up the price of water was another option.

"I wouldn't be surprised to see water prices double in Australia in the next couple of years, we are paying about $1 a kilolitre, in most Australian cities," he said.

"In Germany they are paying $11 a kilolitre so we are very underpriced in terms of some other communities."

Prof Cullen was speaking at Brisbane's International River Symposium alongside University of Queensland Professor Paul Greenwood and the World Wildlife Fund's Dr Stuart Blanch.

Dr Blanch said new dams should only be considered a last resort, urging governments to fund the installation of rainwater tanks on private properties and investigate water recycling initiatives.

He hit out at the Queensland Government's plan to construct a massive dam on the Mary River on the Sunshine Coast, saying it was a bad choice of location.

The river's catchment area did not receive enough rainfall to consistently fill a dam, he said.

"I think panic has set in with the bureaucrats, government and water engineers and they are jumping to big dams, big pipelines without doing the hard work and seeing how far they can push recycling."

"What's the point in building big dams if they remain half full? It's much smarter to move to recycling."

tomder55 answered on 09/05/06:

what are they waiting for ? I say build the dam ;dig more resevoirs to capture rain run off when it does occure and contruct desalination plants .Also your sewer and wastewater treatment plants should recharge your rivers with clean potable water . The technology is there .

Climates change and population growth taxes the ecology . Clean water will be the geopolitical challenge of the future and the future is now .

paraclete rated this answer Excellent or Above Average Answer

Question/Answer
MarySusan asked on 09/05/06 - Rummy More Chamberlain than Churchill

Frank Rich on Rumsfeld: "More Chamberlain Than Churchill"

New York Times | Mother Jones | Posted Sunday September 3, 2006 at 08:24 PM

"Frank Rich turns a scathing eye on Donald Rumsfeld this week, writing an op-ed that Bush won't read about a guy Bush won't fire. Still, it needs to be said. Rich magnanimously declared that Rummy had outdone his previous gems of "stuff happens" and "you go to war with the Army you have" with his latest comparison of Iraq war critics to appeasers of the Nazis in the 1930s. Rich reminds Rumsfeld that he has a long memory, aided and abetted by the Internet:

Here's how brazen Mr. Rumsfeld was when he invoked Hitler's appeasers to score his cheap points: Since Hitler was photographed warmly shaking Neville Chamberlain's hand at Munich in 1938, the only image that comes close to matching it in ***epochal obsequiousness*** is the December 1983 photograph of Mr. Rumsfeld himself in Baghdad, warmly shaking the hand of Saddam Hussein in full fascist regalia. Is the defense secretary so self-deluded that he thought no one would remember a picture so easily Googled on the Web? Or worse, is he just too shameless to care?

Rich notes that Rumsfeld was singularly lacking in tough talk back in that 1983 visit...."

!!!!!!!!!!!!!!!!!!!!!!!!!!!!!!!!!!!!!!!!!!!!


I think we have all seen that picture of Rummy shaking Saddam's hand....look it up if your memory fails.

How long do you think Rumsfeld has left on the job?

tomder55 answered on 09/05/06:

as usual Frank Rich distorts history .It would be too much time to give him a history lesson but suffice it to say that Chamberlain and Hitler met just months before Hitler invaded Czechoslovakia.They agreed that Hitler could annex parts of the country with the allied powers consent .

Rumsfeld's meeting with Saddam was not an appeasement . They met December 20, 1983. Rumsfeld assigned as a special envoy for Ronald Reagan ...7 years before Desert Storm and our enmity with Iraq .This visit was PART of a tour of regional capitals including Baghdad . They discussed finding an alternate route for Iraqi oil and issues regarding our shared enmity with Iran and Syria. Rumsfeld made it clear to Saddam that our direct support to Iraq was limited because of Saddam's use of chemical weapons ,their human rights abuses ,and a concern of a regional escalation .

Rumsfeld returned to Baghdad March 1984. By this time, the U.S. had publicly condemned Iraq's chemical weapons use, stating, "The United States has concluded that the available evidence substantiates Iran's charges that Iraq used chemical weapons".Briefings for Rumsfeld's meetings noted that atmospherics in Iraq had deteriorated since his December visit because of Iraqi military reverses and because "bilateral relations were sharply set back by our March 5 condemnation of Iraq for CW use , despite our repeated warnings that this issue would emerge sooner or later" .

On March 3, the State Department intervened to prevent a U.S. company from shipping 22,000 pounds of phosphorous fluoride, a chemical weapons precursor, to Iraq.

The answer to Mark Rich is that the US in the early 1980s practiced a realpolitik strategy in the ME ;a policy I'm sure Frank Rich supports to this day . He is distorting history to suggest otherwise. As you know the one true goal of realpolitik is stability for stabilities sake. The US considered a possible Iranian victory would disrupt that stability .

For the record if one were looking for an 'appeasement '
of Iraq then we should look no further than US Ambassador April Gillespie telling Saddam that "We have no opinion on the Arab-Arab conflicts, like your border disagreement with Kuwait." Which Saddam interpreted as a green light for him to invade Kuwait .

As for your question ; I think that Rumsfeld has done a fine job .There is no reason to replace him except for the purpose of political expediency . Then again ; if Frank Rich and co.find him a burr in their saddle are looking for Rummy's scalp then maybe Bush should continue to retain his services just to annoy them . Rumsfeld has been a servant of this country since 1954 .

US Congress 1962-1969 ...

From 1969 to 1970, he served as Director of the Office of Economic Opportunity and Assistant to the President.

From 1971 to 1972, he was Counsellor to the President and Director of the Economic Stabilization Program.

In 1973, he left Washington, DC, to serve as U.S. Ambassador to the North Atlantic Treaty Organization (NATO) in Brussels, Belgium (1973-1974).

In August 1974, he was called back to Washington, DC, to serve as Chairman of the transition to the Presidency of Gerald R. Ford.

He then became Chief of Staff of the White House and a member of the President's Cabinet (1974-1975).

He served as Secretary of Defense, the youngest in the country's history (1975-1977) ...and again since 2001.
Even when he was in the provate sector he still served the contry in various commissions ...including :

Member of the President's General Advisory Committee on Arms Control (1982 - 1986)

Special Presidential Envoy on the Law of the Sea Treaty (1982 - 1983)

Senior Advisor to the President's Panel on Strategic Systems (1983 - 1984)

Member of the U.S. Joint Advisory Commission on U.S./Japan Relations (1983 - 1984)

Special Presidential Envoy to the Middle East (1983 - 1984)

Member of the National Commission on Public Service (1987 - 1990)

Member of the National Economic Commission (1988 - 1989)

Member of the Board of Visitors of the National Defense University (1988 - 1992)

Member of the Commission on U.S./Japan Relations (1989 - 1991)

Member of the U.S. Trade Deficit Review Commission (1999 - 2000)

member of the National Academy of Public Administration and a member of the boards of trustees of the Gerald R. Ford Foundation, the Hoover Institution at Stanford University, and the National Park Foundation, and as Chairman of the Eisenhower Exchange Fellowships, Inc.

What is Frank Rich's resume besides professional heckler from the peanut gallery ?




ETWolverine rated this answer Excellent or Above Average Answer
MarySusan rated this answer Excellent or Above Average Answer

Question/Answer
paraclete asked on 09/03/06 - Life would be better under Saddam?

Leading Shiite says militancy is winning


Something to smile about ... Iraqi soldiers celebrate after formally taking over Abu Ghraib prison from US forces. The notorious jail is empty.


Amit Paley in Baghdad and Julian Barnes in Washington
September 4, 2006

AS NEW figures show the numbers of civilian deaths in Iraq spiralling sharply upwards, the nation's most influential moderate Shiite leader has abandoned attempts to restrain his followers, admitting there is nothing he can do to prevent the country sliding towards civil war.

At the same time, a coalition of 300 mainly Sunni tribal leaders has demanded the release of Saddam Hussein so that he can reclaim the presidency. They also called for armed resistance against US-led foreign forces.

The Grand Ayatollah Ali al-Sistani is reported to be angry and disappointed that Shiites are ignoring his calls for calm and are switching their allegiance in their thousands to more militant groups which promise protection from Sunni violence and revenge for attacks.

"I will not be a political leader any more," he told his aides. "I am only happy to receive questions about religious matters."

The clan chieftains calling on Saturday for the restoration of Saddam were mostly Sunni Arabs and included the head of the 1.5 million-member Obeidi tribe. They said they planned to hold rallies in Sunni cities throughout the country to insist that Saddam be freed and that the charges against him and his co-defendants are dropped.

"If the demand is not carried out, we will lead a general, sweeping and popular uprising," said Sheik Wassfy al-Assy, a brother of the chief of the Obeidi tribe.

In a dismal assessment, a new Pentagon report has revealed that the numbers of attacks and civilian casualties in Iraq have risen sharply in recent months as sectarian violence has engulfed larger areas of the country. Deaths have risen by 1000 a month.

The quarterly report shows that the number of attacks over the past four months increased by 15 per cent and the number of Iraqi casualties rose by 51 per cent to more than 3000 violent deaths a month.

Over the longer term, the surge is even more grim. Weekly attacks have doubled from about 400 in the northern spring of 2004 to nearly 800 in recent weeks. The number of daily casualties has increased from fewer than 30 a day in 2004 to more than 110 a day in recent weeks.

The Pentagon report says Iraq is not engaged in a civil war, but acknowledges that Iraqi civilians are increasingly worried about such a conflict. It reports that Iraqis are optimistic about the future, but cautions that the positive outlook is eroding.

The US President, George Bush, is trying to shore up sagging public support for the Iraq war in advance of elections, but by putting hard numbers to the perception that Iraq is increasingly chaotic, the Pentagon report stands to further undermine support for the Administration's strategy in Iraq.

* The second most senior figure in al-Qaeda in Iraq was arrested a few days ago, the Iraqi Government has said. He was named as Hamed Juma Faris al-Suaidi, also known as Abu Humam or Abu Rana, and is said to be the deputy to Abu Ayyub al-Masri, who took over the Sunni Islamist insurgent group after US forces killed Abu Musab al-Zarqawi in June.

* In a new video, al-Qaeda calls on non-Muslims, especially Americans, to convert to Islam or suffer the consequences. Al-Qaeda's second-in-command, Ayman al-Zawahiri, introduced the speaker, who was identified as "Azzam the American", also known as Adam Yahiye Gadahn, an Islamic convert from California who is wanted for questioning by the FBI.

Los Angeles Times, The Washington Post

tomder55 answered on 09/04/06:

yeah life was very good to certain Sunni under Saddam's rule .

But I do not see the news about Sistani as positive at all. Without his leadership the Shia flock to idiots like al-Sadr. (I still say not destroying him is the biggest mistake we made in Iraq). Sistani has called on Al-Maliki to use his mandate by election to disarm the militias and restore order . That is a good sign because the gvt. and coalition can with his approval take their gloves off and deal with the Shia.

paraclete rated this answer Excellent or Above Average Answer

Question/Answer
Dark_Crow asked on 09/03/06 - the Koran............................................

Does the Koran expressly say that the Holy Land is given by the Lord to the Jews for their dominion?

tomder55 answered on 09/04/06:

depends on who you ask .

First it clearly says yes :

"Pharoah sought to scare them [the Israelites] out of the land [of Israel]: but We [Allah] drowned him [Pharoah] together with all who were with him. Then We [Allah] said to the Israelites: 'Dwell in this land [the Land of Israel]. When the promise of the hereafter [End of Days] comes to be fulfilled, We [Allah] shall assemble you [the Israelites] all together [in the Land of Israel]."
("Night Journey," chapter 17:100-104)

The Koran recognizes "Allah"'s instructions to Moses to conquer the Holy Land :

"And [remember] when Moses said to his people: 'O my people, call in remembrance the favour of God unto you, when he produced prophets among you, made you kings, and gave to you what He had not given to any other among the peoples. O my people, enter the Holy Land which God has assigned unto you, and turn not back ignominiously, for then will ye be overthrown, to your own ruin.'" (Koran 5:20-21)

But the Jews were disobedient :

Indignity is put over them wherever they may be, except when under a covenant (of protection) from God, and from men; they have drawn on themselves the Wrath of God, and destruction is put over them. This is because they disbelieved in the Ayats (evidences) of Allah and killed the Prophets without right. This is because they disobeyed (God) and used to transgress beyond bounds (in God's disobedience, crimes and sins)" (Koran 3:112)


As you know ,Mohammed flew to Jersualem on a horse with a human head to meet the prophets in Paradise so the kindered of Ishmael think they have claim to the land over the kindered of Isaac.


Dark_Crow rated this answer Excellent or Above Average Answer
paraclete rated this answer Excellent or Above Average Answer

Question/Answer
Itsdb asked on 09/03/06 - Convert, or else

Sat Sep 2, 2006 9:06 PM BST

By Heba Kandil

DUBAI (Reuters) - Al Qaeda called on U.S. President George W. Bush and non-Muslims especially in the United States to convert to Islam and abandon their 'misguided' ways or else suffer the consequences, according to a video posted on a Web site on Saturday.

The speaker was identified as Azzam the American, also known as Adam Yahiye Gadahn -- an Islamic convert from California wanted for questioning by the FBI and who U.S. authorities believe to be involved in a "propaganda" campaign for al Qaeda.

"If the Zionist crusader missionaries of hate and counter-Islam consultants like ... the crusader and chief George W. Bush were to abandon their unbelief and repent and enter into the light of Islam and turn their swords against the enemies of God, it would be accepted of them and they would be our brothers in Islam," Gadahn said in English.

"To Americans and the rest of Christendom we say, either repent (your) misguided ways and enter into the light of truth or keep your poison to yourself and suffer the consequences in this world and the next."


Al Qaeda's second-in-command Ayman al-Zawahri made a brief statement at the start of the tape urging viewers to listen carefully to the message, entitled: "An Invitation to Islam".

"Our brother Azzam the American is speaking to you out of pity for the fate that awaits (unbelievers) and as someone who wants to lift his people out of darkness and into the light," Zawahri said.

Zawahri, like Osama bin Laden and other leaders of al Qaeda -- the group that masterminded the September 11 attacks on the United States -- is thought to be hiding in Pakistan or Afghanistan.

Zawahri and Gadahn appeared to be speaking from different places, as Zawahri spoke in front of a black background.

The tape was dated September 2006 and appeared to have been recorded recently as Gadahn referred to Israel's war on Lebanon.

Gadahn appeared in the video dressed in a white turban and seated in front of a computer and books.

"But whatever you do don't attempt to spread your misery and misguidance to our lands," he said. The video carried Arabic subtitles of his English message.

FAITH AND JIHAD

Gadahn recited verses from the Muslim holy book the Koran in Arabic, then translated them into English and said Muslims needed to boost their faith to expel their countries' rulers.

"Muslims don't need democracy to rid themselves of their home grown despots and tyrants. What they do need is their Islamic faith, the sprit of jihad and the lifting of foreign troops and interference from their necks," he said, adding that God did not recognise a separation of religion and state.

"Those who think that democracy is synonymous with freedom are either people who haven't experienced life in America or Americans who haven't lived abroad."

Zawahri last appeared in a video in August in which he said that some leaders of Egypt's Gama'a Islamiya have joined al Qaeda. Gama'a Islamiya later denied his statement.

~~~~~~~~~~~~~~~~~~~~~~~~~~~~~~~~~~~~~~~~~~~~~~~~~~~~~~

He aslo reportedly said "Decide today, because today could be your last day."

Can anyone say "I told you so"? I can...

tomder55 answered on 09/04/06:

Maybe al-Qaeda will buy out MTV . They have become proficient at making videos and little else lately .

Fight in the name of Allah and in the way of Allah. Fight against those who disbelieve in Allah. Make a holy warWhen you meet your enemies who are polytheists, invite them to three courses of action. If they respond to any one of these, you also accept it and withhold yourself from doing them any harm. Invite them to (accept) Islam; if they respond to you, accept it from them and desist from fighting against them.If they refuse to accept Islam, demand from them the Jizya [the tax on non-Muslims specified in Quran 9:29]. If they agree to pay, accept it from them and hold off your hands. If they refuse to pay the tax, seek Allahs help and fight them. (Sahih Muslim 4294 ).

Reading the 'Hadith' is like reading 'Mein Kampf' ,or your football opponent's playbook.

Itsdb rated this answer Excellent or Above Average Answer

Question/Answer
excon asked on 09/01/06 - Priorities

Hello:

Lets say that you have cancer. You also have an ingrown toenail. Your medical budget is fixed. You spend %45 of your money on cancer treatment, and you spend %55 on the toenail.

Is that good?

excon

PS> For those not medically inclined, pretend cancer is Osama or Akminijad, and the ingrown toenail is Saddam.

tomder55 answered on 09/02/06:

As you know I do not seperate them any morethat the allied decision in WWII to take out Germany before Japan ,or to first attack Africa before continental Europe.

I will cut an paste an analysis of the al-Qaeda /Osama threat by Sratfor after the airline plot was broken up .

"First, while there obviously remains a threat from those not only sympathetic to al Qaeda, but actually participating in planning with those in the al Qaeda apex leadership, their ability to launch successful attacks outside of the Middle East is severely degraded.

Second, if the cell truly does have 50 people and 21 have already been detained, then al Qaeda might have lost its ability to operate below the radar of Western or at least U.K. intelligence agencies. Al Qaedas defining characteristic has always been its ability to maintain operational security. If that has been compromised, then al Qaedas importance as a force has diminished greatly.

Third, though further attacks could occur, it appears al Qaeda has lost the ability to alter the political decision-making of its targets. The Sept. 11 attack changed the world. The Madrid train attacks changed a government. This failed airliner attack only succeeded in closing an airport temporarily.

Fourth, the vanguard of militant Islam appears to have passed from Sunni/Wahhabi al Qaeda to Shiite Iran and Hezbollah. It is Iran that is shaping Western policies on the Middle East, and Hezbollah who is directly engaged with Israel. Al Qaeda, in contrast, appears unable to do significantly more than issue snazzy videos."


I think it says it all; Osama has been neutralized as an existential threat and should no longer be our priority .(see my response to Mary Sue on this same subject for more detail)

The real issue is whether Saddam should have been our priority and that indeed is adebatable point. However military operations against Iraq were ongoing for 13 years . Critics argue that he was contained ,but the facts get in the way of that argument especially in light of the revelations about the Oil for Food scheme. The sanction regime was collapsing and all the intelligence of the Western World had determined him to be the greatest threat in the region .

Iran at that same time did not have the Mahdi-hatter running the show ;Iran's President was Mohammad Khatami and the concensus of the West mistakenly considered him to be a reformer .Certainly Colin Powell and others in the State Dept. were practically salivating at the prospects of an open Iran under his leadership .He praised their 'open and free elections' and offered up only token sanctions on the regime as the nuclear clock ticked away and the threat they now pose grew.

We are about 2 years late in dealing with Iran and Syria. They have given us sufficient cause to deal with them militarily in that their finance and other direct aid to what is mistakenly refered to as the insurgency has cost us in Iraq. Like any war priorities and stratagies shift. It is easy in hindsight to say that we should've dealt with Iran first because at present they appear to be the bigger threat . But imagine the mischief Saddam could've caused on our flank while we were "bogged down " dealing with Iran. Heck ;just containing him cost us a fortune. We had at least 1 fleet flying sorties non-stop for 13 years .We had thousands of troops sitting in Saudia Arabia ;in Turkey who's mission was to be ready to strike at Saddam again. As I'm sure you are aware ,keeping troops at that state of readiness is costly .

Also remember that until recently therewas a concensus by almost everyone that the Iran regime was a straw house just waiting for the democratic movements to blow it away .I can say pretty confidently now that any such opposition group has been crushed and are no longer a factor .But that occured under Khatami (the' reformist President')and has not been a realistic factor in years. It is only now that the Iranian threat is apparent .

Most Americans according to polls saw the threat that Saddam posed and gave overwhelming support for the initiative. Until recently they did not see it in Iran ,and even if they do I do not think they could be mustered to deal with it .But I think knocking Iran off the chess board would also greatly help our effort in Iraq.

excon rated this answer Excellent or Above Average Answer

Question/Answer
Itsdb asked on 09/01/06 - Hezbollah's 'Victory'

By Charles Krauthammer
Friday, September 1, 2006; A21

"We did not think, even 1 percent, that the capture would lead to a war at this time and of this magnitude. You ask me, if I had known on July 11 . . . that the operation would lead to such a war, would I do it? I say no, absolutely not."

-- Hasan Nasrallah,

Hezbollah leader, Aug. 27

So much for the "strategic and historic victory" Nasrallah had claimed less than two weeks earlier. What real victor declares that, had he known, he would not have started the war that ended in triumph?

Nasrallah's admission, vastly underplayed in the West, makes clear what the Lebanese already knew. Hezbollah may have won the propaganda war, but on the ground it lost. Badly.

True, under the inept and indecisive leadership of Ehud Olmert, Israel did miss the opportunity to militarily destroy Hezbollah and make it a non-factor in Israel's security, Lebanon's politics and Iran's foreign policy. Nonetheless, Hezbollah was seriously hurt. It lost hundreds of its best fighters. A deeply entrenched infrastructure on Israel's border is in ruins. The great hero has had to go so deep into hiding that Nasrallah has been called "the underground mullah."

Most important, Hezbollah's political gains within Lebanon during the war have proved illusory. As the dust settles, the Lebanese are furious at Hezbollah for provoking a war that brought them nothing but devastation -- and then crowing about victory amid the ruins.

The Western media were once again taken in by the mystique of the "Arab street." The mob came out to cheer Hezbollah for raining rockets on Israel -- surprise! -- and the Arab governments that had initially criticized Hezbollah went conveniently silent. Now that the mob has gone home, Hezbollah is under renewed attack -- in newspapers in Saudi Arabia, Kuwait and Egypt, as well as by many Lebanese, including influential Shiite academics and clan leaders. The Arabs know where their interests lie. And they do not lie with a Shiite militia that fights for Iran.

Even before the devastation, Hezbollah in the last election garnered only about 20 percent of the vote, hardly a mandate. Hezbollah has guns, however, and that is the source of its power. But now even that is threatened. Hence Nasrallah's admission. He knows that Lebanon, however weak its army, has a deep desire to disarm him and that the arrival of Europeans in force, however weak their mandate, will make impossible the rebuilding of the vast Maginot Line he spent six years constructing.

Which is why the expected Round Two will, in fact, not happen. Hezbollah is in no position, either militarily or politically, for another round. Nasrallah's admission that the war was a mistake is an implicit pledge not to repeat it, lest he be completely finished as a Lebanese political figure.

The Lebanese know that Israel bombed easy-to-repair airport runways when it could have destroyed the new airport terminal and set Lebanon back 10 years. The Lebanese know that Israel attacked the Hezbollah TV towers when it could have pulverized Beirut's power grid, a billion-dollar reconstruction. The Lebanese know that the next time, Israel's leadership will hardly be as hesitant and restrained. Hezbollah dares not risk that next time.

Even more important is the shift once again in the internal Lebanese balance of power. With Nasrallah weakened, the other major factions are closing in around him. Even his major Christian ally, Michel Aoun, has called for Hezbollah's disarmament. The March 14 democratic movement has regained the upper hand and, with outside help, could marginalize Hezbollah.

In a country this weak, outsiders can be decisive. A strong European presence in the south, serious U.S. training and equipment for the Lebanese army, and relentless pressure at the United Nations can tip the balance. We should be especially aggressive at the United Nations in pursuing the investigation of Syria for the murder of Rafiq Hariri and in implementing resolutions mandating the disarmament of Hezbollah.

It was just 18 months ago that the democrats of the March 14 movement expelled Syria from Lebanon and rose to power, marking the apogee of the U.S. democratization project in the region. Nasrallah's temporary rise during the just-finished war marked that project's nadir. Nasrallah's crowing added to the general despair in Washington about a rising "Shiite crescent" stretching from Tehran to Beirut.

In fact, Hezbollah was seriously set back, as was Iran. In the Middle East, however, promising moments pass quickly. This one needs to be seized. We must pretend that Security Council Resolution 1701 was meant to be implemented and exert unrelieved pressure on behalf of those Lebanese -- a large majority -- who want to do the implementing.

~~~~~~~~~~~~~~~~~~~~~~~~~~~~~~~~~~~~~~~~~~~~~~~~~~~~~~~

Where is the media on this? I hadn't heard a word about Nasrallah's mea culpa until I read this column today. They were quick to jump on the Hezbollah bandwagon, following the propagandists and their staged photo ops, where are they now that Lebanon could use a little saturation coverage on such a monumental revelation?

tomder55 answered on 09/01/06:

So in other words the kidnapping of the soldiers was a bluff that Israel answered and raised. Calling the bluff of tyranny has proven to be a better strategy than appeasement..... again.

I wonder if the Lebanese understand what this misadventure has brought on them .They now must make a choice if they want their country dominated by Iran ,Syria and the Hezzies or do they want the benefits that could come from their Cedar Revolution ? It really is their choice. The Hezzies will be rearmed only if the Lebanese gvt permits it . It is not up to UNIFIL to prevent it .Even a beefed up UN presence will be a useless force .Suprisingly Krauthammer puts more faith into their mission than I do .

He is right about the damage to the infrastructure. Israel was restrained . Bombing tarmacs instead of terminals ;bombing easily repairable bridges .The lights never went out in Beirut and they were never without potable water.

They severely degraded Hezbollah's ability to attack Israel and if what I hear is true ,the Iranians are none too pleased with the timing of the war . The story goes that Iran wanted coordinated attacks at a later date in the fall . Iran had planned on giving a final answer to the world on Aug. 22 about the resolution 1697 . Instead he initiated war games. I think he tempered his response quite a bit based on the Hezbollah showing. He went from boasting that 'Iran must be wiped off the map ' to his August 26 declaration the Iran is no threat to Israel .

However ;I think Israel missed a golden opportunity to drive a stake into the heart of an enemy that deserved an even greater pummeling than they received. For that Olmert and Condi Rice I find responsible.

Frequently the 1st few reports from any battle are incorrect . What is remarkable about the MSM is their ability to mold a report into their preconceived meme and not deviate from it when the facts disprove it .


ETWolverine rated this answer Excellent or Above Average Answer
Itsdb rated this answer Excellent or Above Average Answer

Question/Answer
MarySusan asked on 08/30/06 - ISLAMO-THEISM

The Bush Propaganda and Noise Machine had a tough decision naming the religious terrorists. They are Islamo-Theists but, of course, that moniker was a no-starter. There was no way that the Bush Christian Republican Theist Party could use the word Islamo-Theists; it would remind every American that Bush was running a Theist Party of his own. A Theist Party that was attempting to turn the American Government into a virtual Dictatorship, a Fascist-Corporate State.

The solution, the same as all the Propaganda put out by the Bush Crime Family....call your opponents WHAT YOU ARE! Usurp the word.

That is the genesis of the incorrect term Islamofascists.

CORRECTLY, THEY ARE ISLAMO-THEISTS.

tomder55 answered on 09/01/06:

This is a very weak attempt to place a moral equivalence between American Christian conservatives and the jihadists . Your argument falls apart in the face of the evidence . We are now entering what is called "bombing season " in Indonesia where the jihadists have been killing for years and forcing conversion on non-believers . Jihadists have left a swath of destruction inside and outside the ummah . There is no comparable Christian political movement like that .But you know that already ;this posting was a flame to stimulate the reaction it received .

MarySusan rated this answer Average Answer

Question/Answer
paraclete asked on 09/01/06 - and now with the threats

Islamic leader warns of more riots


September 1, 2006 - 3:10PM

The chairman of the government's Islamic advisory committee has warned of more Cronulla-style riots unless the prime minister tones down his rhetoric on Muslim migrants.

John Howard sparked a backlash in some sections of Australia's Muslim community by saying on talk-back radio yesterday that Muslim migrants needed to conform to Australian values by learning English and treating women with respect.

But the head of the government's new muslim advisory committee, Dr Ameer Ali, warned of trouble unless the prime minister backed down.

"We have already witnessed one incident in Sydney recently in Cronulla, I don't want these scenes to be repeated because when you antagonise the younger generation, younger group, they are bound to react," Dr Ali told Macquarie Radio.

But Mr Howard today stood by his comments.

"I don't apologise," he told reporters.

"I think they are missing the point and the point is that I don't care and the Australian people don't care where people come from.

"There's a small section of the Islamic population which is unwilling to integrate and I have said generally all migrants ... they have to integrate."

AAP

Once again Muslims fail to recognise what is being said to them. Their culture is not acceptable and will not be tolerated in its extreme forms. For some time I have been critised for expressing this opinion, but I am not out of step with my culture, it is the Muslims who are out of step with community expectations. Once again Muslims demonstrate that they represent a fifth column, they it is they who are the threat and they do this by making threats of violence, instead of accepting the norms of the society in which they live and modifying thier behaviour. I say the time for tolerance is over and the time for integration has arrived, no matter where these muslim populations exist outside of their homelands

tomder55 answered on 09/01/06:

is that your final solution answer ?

Howard of course is correct ,but for greater effect he should publish his comments in cartoon form .

I do not criticize you for your opinion ;just that you rarely think it is worth backing up with force. It's all well and good to have such strong opinions but when it's time for action you so frequently condemn to point of the spear .

paraclete rated this answer Excellent or Above Average Answer

Question/Answer
HANK1 asked on 09/01/06 - JUST WONDERING ...



... if our troops in the Middle East have been trained in guerilla warfare.

HANK

tomder55 answered on 09/01/06:

I would say so . They have demonstrated their proficiency over and over in the last 3 years .Tell me the battle they have lost . Perhaps we have sometimes been slow to recognize a shifting battle field but once recognized our adaption has been swift . The under-reported news recently has been the drop in violence in Baghdad > since "Operation Forward Together" began.

HANK1 rated this answer Excellent or Above Average Answer

Question/Answer
MarySusan asked on 08/30/06 - ISLAMO-THEISM

The Bush Propaganda and Noise Machine had a tough decision naming the religious terrorists. They are Islamo-Theists but, of course, that moniker was a no-starter. There was no way that the Bush Christian Republican Theist Party could use the word Islamo-Theists; it would remind every American that Bush was running a Theist Party of his own. A Theist Party that was attempting to turn the American Government into a virtual Dictatorship, a Fascist-Corporate State.

The solution, the same as all the Propaganda put out by the Bush Crime Family....call your opponents WHAT YOU ARE! Usurp the word.

That is the genesis of the incorrect term Islamofascists.

CORRECTLY, THEY ARE ISLAMO-THEISTS.

tomder55 answered on 08/31/06:

as you know I abandoned using Islamo-Nazi or fascist a long time ago . I reluctantly have to give the whole religion the benefit of the doubt and also from a clear strategic impact it is better to isolate the hardcore than to make an enemy with billions of people.

The IRA was not called Catholic-terrorists and Mussolini was not a Christian-fascists .

I have opted for a variation of jihadists so I can isolate the enemy to the vermin who are radical Islamist expansionists . Fascism and Nazism however are accurate depictions of their methods.

ETWolverine rated this answer Excellent or Above Average Answer
Itsdb rated this answer Excellent or Above Average Answer
kindj rated this answer Excellent or Above Average Answer
MarySusan rated this answer Excellent or Above Average Answer

Question/Answer
paraclete asked on 08/30/06 - Sometimes, dispite our best intentions, idiots get elected?

AUSTRALIAN Democrats MP Sandra Kanck was set to use the protection of parliamentary privilege today to detail ways of committing suicide.

Ms Kanck was to move a motion in the South Australian Parliament challenging Federal Government laws which make it illegal to distribute such information by electronic means.

Once she delivers her speech she expects it to be posted on Parliament's website, bringing the SA Parliament into direct conflict with the Commonwealth's Suicide Material Related Offences Act.

Ms Kanck said the federal laws undermined two fundamental human rights, the right to free speech and the right to die with dignity.

"I cannot, in good conscience, allow the attack on human rights to go unchallenged," she said.

"The effect of this odious legislation will be to force desperate people to commit suicide by the most appalling of means.

"As a consequence some won't succeed and will be left in awful agony.

"Others will die in grotesque ways that psychologically scar those who find the body."

Ms Kanck rejected suggestions her action might prompt more young people to access the information on suicide in a bid to take their own lives.



I sometimes think it's a great pity politicians wern't forced to practice what they preach?

tomder55 answered on 08/30/06:

I'm sure she proposes it in the best of liberal compassionate intentions.

Itsdb rated this answer Excellent or Above Average Answer
paraclete rated this answer Excellent or Above Average Answer

Question/Answer
paraclete asked on 08/29/06 - Why are americans so anxious to export their form of democracy?

One in eight Americans in poverty: poll


August 30, 2006 - 7:39AM

In the world's biggest economy, one in eight Americans and almost one in four blacks lived in poverty last year, the US Census Bureau said, both ratios virtually unchanged from 2004.

The survey also showed 15.9 per cent of the population, or 46.6 million, had no health insurance, up from 15.6 per cent in 2004 and an increase for a fifth consecutive year, even as the economy grew at a 3.2 per cent clip.

It was the first year since President George W Bush took office in 2001 that the poverty rate did not increase. As in past years, the figures showed poverty especially concentrated among blacks and Hispanics.

In all, some 37 million Americans, or 12.6 per cent, lived below the poverty line, defined as having an annual income around $US10,000 ($A13,200) for an individual or $US20,000 ($A26,395) for a family of four.

The total showed a decrease of 90,000 from the 2004 figure, which Census Bureau officials said was "statistically insignificant."

The last time poverty declined was in 2000, the final year of Bill Clinton's presidency, when it fell to 11.3 per cent.

The stagnant poverty picture drew attention from Democrats and others who said not enough is being done to help the nation's poor.

"Far too many American families who work hard and play by the rules still wind up living in poverty," said Republican George Miller of California, the top Democrat on the House Education and Workforce Committee.

Around a quarter of blacks and 21.8 per cent of Hispanics were living in poverty. Among whites, the rate edged down to 8.3 per cent from 8.7 per cent in 2004.

"Among African Americans the problem correlates primarily to the inner-city and single mothers," said Michael Tanner of CATO Institute, a free-market think tank in Washington.

He noted that blacks also suffer disproportionately from poor education and lower quality jobs.

Black median income, at $US30,858 ($A40,720), was only 61 per cent of the median for whites.

Some 17.6 per cent of children under 18 and one in five of those under six were in poverty, higher than for any other age group.

Major cities with the highest proportions of poor people included Cleveland with 32.4 per cent and Detroit with 31.4 per cent under the poverty line.

2006 Reuters,

tomder55 answered on 08/30/06:

I mentioned to Hank yesterday that poverty ;a term that is constantly redefined needs to be looked at in historical standards and relative to how the rest of the world defines 'poverty ' . In America today poverty means means only one color tv ;and owning a used car as opposed to cardboard shacks and bloated stomachs from the malnutrition as much of the rest of the world defines it .Here is the article I linked to illustrate my point :

"As a group, America's poor are far from being chronically undernourished. The average consumption of protein, vitamins, and minerals is virtually the same for poor and middle-class children and, in most cases, is well above recommended norms. Poor children actually consume more meat than do higher-income children and have average protein intakes 100 percent above recommended levels. Most poor children today are, in fact, supernourished and grow up to be, on average, one inch taller and 10 pounds heavier that the GIs who stormed the beaches of Normandy in World War II."




ETWolverine rated this answer Excellent or Above Average Answer
paraclete rated this answer Excellent or Above Average Answer

Question/Answer
MarySusan asked on 08/29/06 - Alternate Thinking About Terrorists

There is one theory that 911 was Bin Laden's plan to scare America in to removing their troops and bases from the Islamic Holy Land.....there are now only 300 American soldiers there, and Bush said in a televised appearance some time ago when asked about Osama, "I dont' think about him any more"...we closed our Bin Laden office in the FBI....Bin Laden's mission accomplished?

AlQuaeda is splintered into various groups, the centralized leadership decimated...Bin Laden and the good Dr from Egypt making occasional television appearances.

We see the results of the Lebanon War. Israel was severely beaten because they couldn't FINISH OFF SUCH A PRIMITIVE, fighting force with sophisticated air power and weapons, an army of trained soldiers, a superior intelligence force.

I see no way to defeat terrorism by force and violence. Lebanon, Iraq, Afghanista has proven that.

Time for some serious thinking about American foreign policy. Note: nothing in the world will solve the violence problems between Jews and Palestineans. The reason, they are *both correct* in their reasons for the existence or non-existence of Israel.


Terrorism of all kinds has always been with us, and it will go on into the future, hopefully at a lesser pace, so time for creative approaches and super espionage and intelligence and police work to lessen the death.

Any comments?

tomder55 answered on 08/30/06:

Here's my take; OBL regrets the day he authorized the attack on the WTC . I'd say the last 5 years have been tough on the old boy .

After 9-11 ;his patron's the Taliban were dispersed and scattered into the hills . He no longer has a country that he can claim is his base of support . Today he hides out in at best a lawless region in Pakistan awaiting his inevidible turn . He goes to sleep at night every day knowing that he is a hunted man by the most powerful nation in the history of the world;and with the full understanding that the U.S. used only a small fraction of it's vast military might to bring him to this point.

In 2001 al-Qaeda was a relatively centralized organization, with a planning hub, a propaganda hub, a leadership team, all within a defined geographic area. All that is gone, because we destroyed it.There are no more al-Qaeda training camps pumping out jihadists .His troops by the thousands have been arrested, detained, or killed. So have many members of the crucial al-Qaeda leadership circle around bin Laden and Ayman al-Zawahiri.The al-Qaida that was centralized had a state, a base and limitless resources is gone. This is an important achievement.

The circle of people he can trust is shrinking and not being replaced .If anyone thinks that he is still central in 'command and control' they are mistaken .

Bush is right . Bin-Laden beyond being a mythical Che Guevera type figure to the jihadi is irrelevent.

Increased intelligence efforts by the United States and its allies have made it extremely difficult to execute transactions of any type(communications, travel, money transfers)have become more dangerous for the jihadists. Where bin Ladens central leadership team could once wire money around the world using normal bank networks, it now must rely on couriers with vests full of cash.Where bin Ladens network could once use satellite phones and the Internet for communication, it now has to avoid most forms of electronic communication, which leave an electronic trail back to the user. Bin Laden and al-Zawahiri now send information out through videotapes(a combined total of maybe 2 dozen in 5 years)and via operatives in Internet chat rooms. A poor substitute for direct communications. Training and operations have been decentralized, raising the risk of fragmentation and loss of unity. Jihadists everywhere face the threat of capture or martyrdom. To make matters worse for them ;All measures show that its brutal tactics (beheadings ,car bombs to incite sectarian violence )have cost it support in the Arab and Islamic world for the most part .No one shed a tear for the death of Zarqawi.

Yes jihadist terrorist organizations have fragmented and that poses a different set of challenges but without the command and control ;and the State backing and resourses these groups are hardly the existential threat that al-Qaeda became. Yes they can blow up trains but I do not see another 9-11 attack coming from small fragmented cells operating out of Miami .

As I've mentioned many times ,we are far from being in a war footing in this country .The increases in security have been at worse a minor inconvienence... Far from the wailings of the Cassandras who think their basic rights are being trampled. They should at very least acknowlege that it would've been much more difficult to break up the airline plot in England if the British and Pakistani investigators were handcuffed with the Constitutional restraints imposed on our law enforcement and intel organizations .Pehaps the plot would've succeeded. Instead the British apparently had the current plotters under surveillance for a sustained period. A recent analysis from Stratfor (no link subscription required ) pointed out the following observations :

"First, while there obviously remains a threat from those not only sympathetic to al Qaeda, but actually participating in planning with those in the al Qaeda apex leadership, their ability to launch successful attacks outside of the Middle East is severely degraded.

Second, if the cell truly does have 50 people and 21 have already been detained, then al Qaeda might have lost its ability to operate below the radar of Western or at least U.K. intelligence agencies. Al Qaedas defining characteristic has always been its ability to maintain operational security. If that has been compromised, then al Qaedas importance as a force has diminished greatly.

Third, though further attacks could occur, it appears al Qaeda has lost the ability to alter the political decision-making of its targets. The Sept. 11 attack changed the world. The Madrid train attacks changed a government. This failed airliner attack only succeeded in closing an airport temporarily.

Fourth, the vanguard of militant Islam appears to have passed from Sunni/Wahhabi al Qaeda to Shiite Iran and Hezbollah. It is Iran that is shaping Western policies on the Middle East, and Hezbollah who is directly engaged with Israel. Al Qaeda, in contrast, appears unable to do significantly more than issue snazzy videos."


I agree with the last sentence that it's time for creative approaches and super espionage and intelligence and police work to lessen the death. Indeed in the last 5 years that is what the administration has attempted .If some of the measures that heve been taken prove in fact to be unconstitutional as the many critics parrot then they are easily fixable. I have argued that Congress need be a partner in the war against jihadistan .Passing war resolutions and then sitting on their hands and being Monday morning quarterbacks ;or yelling obscenities from the bleacher seats is not in my view responsible governing .

Dark_Crow rated this answer Excellent or Above Average Answer
MarySusan rated this answer Excellent or Above Average Answer

Question/Answer
Dark_Crow asked on 08/29/06 - President Mahmoud Ahmadinejad challenged President Bush..

President Mahmoud Ahmadinejad challenged President Bush to a televised debate on world issues today. Of course I would like to see such a debate but another thing he said was that the veto right in the UN Security Council was a problem in that, it is an insult to the dignity, independence, freedom and sovereignty of the nations that do not enjoy that right. Do you suppose that that is part of why the UN fails so often?

tomder55 answered on 08/29/06:

Perhaps he was emboldened by the lovefest that Mike Wallace showed him.


If it were up to me we would not be in the UN .We should instead be in an organization where the security of free nations is guaranteed by common defense .The majority of the nations in the UN are run by despots and have interests that are anathema to ours .

The security council like the overall body of the UN has been an ineffective institution (the one exception being the time that the Soviets foolishly walked out of the debate about sending UN troops to Korea ;a mistake never repeated ) .

I think some of the NGO work the UN does is fine and salvagable but otherwise I'm sure Trump could find better use for the property at Turtle Bay.

Dark_Crow rated this answer Excellent or Above Average Answer
ETWolverine rated this answer Excellent or Above Average Answer
Itsdb rated this answer Excellent or Above Average Answer

Question/Answer
paraclete asked on 08/28/06 - now we are persecuting terrorists?

Curfew order for Jack Thomas


August 28, 2006 - 7:22PM


The Federal Government tonight defended the move to impose Australia's first control order on freed terror suspect Jack Thomas, saying the community needs to be protected.

The control order, the first granted under tough new anti-terror laws, was slapped on Mr Thomas this morning as he holidayed with his family in Victoria's South Gippsland, forcing him to quickly return to Melbourne.

Mr Thomas has been a free man since August 18, when the Victorian Court of Appeal quashed convictions for receiving funds from al-Qaeda and holding a false passport, and overturned a five-year jail sentence.

Mr Thomas, 33, was neither present nor represented when the Federal Magistrates Court in Canberra yesterday granted the control order, requested by federal police with the approval of Attorney General Philip Ruddock.

The court said because Mr Thomas had trained with al-Qaeda, the interim order was "reasonably necessary" to protect the public and prevent a terrorist act.

Under the order, the Melbourne father-of-three is confined to his house between midnight and 5am, must report to police three days a week, is banned from leaving Australia without permission, and is restricted in what phones he can use.

Mr Thomas is also banned from any contact with members of banned terrorist groups.

Jack Thomas' lawyer Rob Stary said his client would challenge the order at a directions hearing set down for later this week.

"We will be challenging it. We will be vigorously challenging it," Mr Stary said.

But if confirmed at a hearing due on Friday, the order will be in force for 12 months.

Jack Thomas' brother Les Thomas said the order amounted to the continuing persecution of his brother by the government and was nothing more than a political stunt.

"We just didn't expect them to stoop this low," he said.

"Obviously, the decision to quash my brother's convictions and make him a free man were a setback to the Australian Federal Police and the attorney-general's office, whose claims of Jack being some kind of terrorist sleeper were thrown out by a jury," Mr Thomas told AAP.

"There have been assorted claims made throughout the trial that have been proven to be false, yet the government is trying to save face in this case and score propaganda points.

"Fear is extraordinarily high in the community and people are frightened for all kinds of reasons.

"We are going to stand by Jack throughout this as we always have and see it through."

tomder55 answered on 08/28/06:

I see Aussie has their share of judicial oligarchs also.

I would opt for deportation . Send him to jihadistan where he longs to be ....and they say Gitmo is not needed

ETWolverine rated this answer Excellent or Above Average Answer
Itsdb rated this answer Excellent or Above Average Answer
paraclete rated this answer Excellent or Above Average Answer

Question/Answer
Itsdb asked on 08/26/06 - Chinese police end funeral striptease acts

Performances meant to increase attendance in order to honour the deceased

Official hotline set up for 'funeral misdeeds'
Aug. 23, 2006. 12:04 PM
REUTERS

BEIJING Striptease send-offs at funerals may become a thing of the past in east China after five people were arrested for organising the intimate farewells, state media reported today.

Police swooped last week after two groups of strippers gave "obscene performances" at a farmer's funeral in Donghai County, Jiangsu province, Xinhua news agency said.

The disrobing served a higher purpose, the report noted.

"Striptease used to be a common practice at funerals in Donghai's rural areas to allure viewers," it said. "Local villagers believe that the more people who attend the funeral, the more the dead person is honoured".

Wealthy families often employed two troupes of performers to attract a crowd. Two hundred showed up at last week's funeral.

Five strippers were detained and local officials "issued notices concerning funeral management", Xinhua said.

Now village officials must submit plans for funerals within 12 hours after a villager dies. And residents can report "funeral misdeeds" on a hotline, the report said.

~~~~~~~~~~~~~~~~~~~~~~~~~~~~~~~~~~~~~~~~~~~~~~~~~~~~~~

Now THAT'S a sendoff...

tomder55 answered on 08/27/06:

like Clete said ;it's getting so you just can't have fun at funerals anymore. All we have to do here is give attendees a days pay or have the Big Texan cater it .

Itsdb rated this answer Excellent or Above Average Answer

Question/Answer
Itsdb asked on 08/26/06 - Taxpayers paying for soldiers' breast, nose jobs

Associated Press
Aug. 25, 2006 07:10 AM

CANBERRA, Australia - Australian soldiers are indulging in expensive cosmetic surgery - including breast enlargements, nose jobs and face lifts - at taxpayers' expense, according to media reports Friday.

Official Australian Defense Force policy states that personnel can undergo plastic surgery at public expense for medical or psychological reasons that threaten their ability to work.

An army cook underwent a nose reduction operation Wednesday, while female service personnel have had breast enlargements after claiming that depression and poor confidence were hurting their work, News Ltd. newspapers reported.

Defense Minister Brendan Nelson said Friday he has ordered an investigation.

Cosmetic surgery consultant Pamela Noon told the newspapers her business has performed surgeries on six military personnel in the past year.

"While a feature might affect somebody's self-confidence, I can't see how it would help their ability to protect the country," Noon was quoted as saying.

~~~~~~~~~~~~~~~~~~~~~~~~~~~~~~~~~~~~~~~~~~~~~~~~~~~~~~

Breast enlargements? Interesting way to beef up your body armor...

tomder55 answered on 08/27/06:

kevlar padded ? when you go to war you go with the breasts you have ,not the ones you wish you had .(paraphrase Rummy)

Itsdb rated this answer Excellent or Above Average Answer
paraclete rated this answer Excellent or Above Average Answer

Question/Answer
paraclete asked on 08/25/06 - Their man in Islamabad

20.08. 2006

BUSH,BLAIR: THEIR MAN IN ISLAMABAD
INTERNATIONAL TERRORISM MONITOR: PAPER NO.109
By B.Raman

A hilarious novel by Graham Greene titled "Our Man in Havana" became a best-seller in the 1970s. It was about a mediocre officer of the MI-6, Britain's external intelligence agency, posted to Havana as a punishment for failing to produce any worthwhile intelligence in his career. One day he sends to his headquarters a sensational report, which he claims to have obtained from a mole, about the arrival in Cuba of a highly lethal Soviet missile for use against the US.

2. The MI-6 and the CIA examine the report. There is excitement in both the agencies over this intelligence coup. They inform their respective political leaders. The MI-6's man in Havana is flooded with encomiums.The more the MI-6 asks him for further details of the missile, the more he gets from his mole.

3. One day, the excitement in the MI-6 breaks the ceiling when they receive from their man what he claimed was a copy of the diagram of the missile.The UK Defence Department, the Pentagon and the political leaders of the two countries are informed. The British and American analysts are mystified.The missile, going by the diagram, looks like no other missile the USSR was known to have produced before.Studies are ordered as to how to counter it.

4. One British analyst has a vague feeling that he had seen a similar diagram somewhere before, but he cannot recall when and where.One day the vacuum cleaner in his house goes out of order.He opens it. Hey presto, he finds inside a diagram of the vacuum cleaner. He realises that what their man in Havana had sent as the diagram of a new Soviet missile, was actually the diagram of a vacuum cleaner.

5.There is utter consternation in the MI-6 headquarters. They call their man to London and question him. He admits that he never had a mole in the Cuban security set-up and that he had fabricated all his reports. He got the idea about the new missile while repairing his vacuum cleaner one day.

6.The chief of the MI-6 and his officers ask him to wait outside while they discuss his cheating.The senior officers advise the chief not to admit to the Prime Minister and the CIA that there was no such missile and that their man had made an ass of them.It would destroy the organisation's credibility and that of the chief.

7.They decide to request their man to apply for premature retirement and recommend to the Government that his request be accepted despite his outstanding work. They also decide to recommend him for knighthood for his outstanding performance in Havana. He remains on the records of the MI-6 one of the greatest intelligence operatives produced by the British intelligence.

8. One is reminded of the MI-6's Man in Havana as one watches with amazement the encomiums being showered on President Gen. Pervez Musharraf of Pakistan as a stalwart ally in the war against terrorism by President George Bush and Prime Minister Tony Blair despite an avalanche of evidence regarding his duplicity. What is the evidence available against Musharraf so far:


* His reluctance to hand over Omar Sheikh to the Americans for questioning regarding the kidnapping and beheading of Daniel Pearl, the US journalist.
* His continued refusal to hand over A.Q.Khan, the Pakistani nuclear scientist, to the US for interrogation on his links with Iran,Libya,North Korea, Syria, Iraq and Al Qaeda.
* His non-co-operation in the hunt for Osama bin Laden, his No.2 Ayman al-Zawahiri and other remnants of Al Qaeda, who are now operating from Waziristan in Pakistani territory.
* His reluctance to act against Mulla Mohammed Omar, the Amir, and other leaders and cadres of the Taliban, who are killing Americans, British, Canadians, Afghans and others from their sanctuaries in Pakistani territory.
* His refusal to act against the Lashkar-e-Toiba (LET) and its mother organisation the Jamaat-ud-Dawa (JUD) despite the LET's global ramifications and its links with Al Qaeda.
* His indignant denials of Indian and Afghan allegations regarding the jihadi terrorist infrastructure in Pakistani territory, which continues to encourage terrorism in India and Afghanistan.
* His making a deal with the Taliban and Al Qaeda remnants in Waziristan under which they have agreed to observe a cease-fire inside Waziristan in return for Musharraf's closing his eyes to their raids into Afghan territory

9. And, so on and so on and so on. In spite of all this, Mr.Bush and Mr.Blair keep showering praise on Their Man in Islamabad. Their praise shows no sign of stopping despite new evidence of the General's duplicity regarding the alleged plot to blow up 10 US-bound aircraft, the discovery of which was announced dramatically by the British police on August 10, 2006.

10.Musharraf and his officials proclaimed that it was Pakistan, which discovered the plot and alerted the British about it on August 9. They projected Rashid Rauf, a British citizen of Pakistani origin, as the chief co-ordinator of the plot on behalf of the Al Qaeda in Afghanistan. What strip-tease they have been playing about Rashid Rauf!


* They said he was arrested while crossing into Pakistan from Afghanistan a week before the British announcement.
* Sections of the Pakistani media reported that he was actually arrested in Bahawalpur in southern Punjab on August 8. He had acquired an expensive house there and married the sister-in-law (wife's sister) of Maulana Masood Azhar, the Amir of the Jaish-e-Mohammad (JEM), which was designated by the US as a Foreign Terrorist Organisation in December,2001.
* After the publication of the report of his arrest in Bahawalpur, the Pakistani officials changed their version. They said they had actually arrested an associate of Rashid Rauf while crossing over into Pakistan from Afghanistan and he led them to Rashid in Bahawalpur. They have not given the name of this associate.
* They said that the entire plot was conceived by the No.3 of Al Qaeda who, according to them, is based in Afghanistan, but they could not give his name except to say he was close to No.2 Zawahiri.
* Then, they said it was actually a son-in-law of Zawahiri, who conceived the plot and tried to use Rashid to have it executed. They gave the name of the so-called son-in-law. When it was pointed out to them that this son-in-law was reported by them earlier this year to have been killed in an American air raid in the Bajaur tribal agency, they have gone silent. Musharraf has advised his agencies not to give any more briefings to the media.

11.Musharraf has suddenly become a stickler for the law. In the past, the Pakistani authorities had informally handed over to the Americans without following the due process of the law Mir Aimal Kansi, Ramzi Yousef, Abu Zubaidah, Ramzi Binalshib, Khalid Sheikh Mohammad, Abu Faraj al-Libi and many others without informing their courts about their arrests.Abu Faraj was handed over despite the fact that he was the principal accused in the case relating to the plot to kill Musharraf in December,2003.

12.In the case of Rashid Rauf, they are following the entire procedure as laid down in the law. They informed a court of his arrest. They produced him before a magistrate and obtained his remand in police custody for interrogation. They have reportedly requested the British for a formal written application for handing him over so that they can put it up to the Magistrate for orders. A British police team is waiting in Islamabad patiently for an opportunity to question him.

13. Any police would have been anxious to question him as urgently as possible in order to neutralise any other threat before it materialises, but not the British. It is now 10 days since the plot was discovered, but the British are yet to interrogate the so-called principal co-ordinator of it. They are showing remarkable patience.It is like a clip in slow motion from a Charlie Chaplin movie. The whole case relating to Rashid is moving at a pace which would make the proverbial snail look a great sprinter.

14. Rashid Rauf may well go down in history as the terrorist, whom nobody wanted to interrogate. The Pakistanis don't want to interrogate him too much lest their duplicity be exposed.The British and the Americans don't want to be in a hurry to interrogate lest their own gullibility be exposed.Moreover, there is a great danger if it comes out that they again let themselves be taken for a ride by Musharraf.Not only will their credibility be in ruins, but they may even face claims for damages from airline companies and passengers, who incurred losses amounting to billions of dollars as a result of the drama staged by the British police.

15. The only way of avoiding all this is to persist with the drama and to go on showering encomiums and lollipops on Musharraf. It would be dangerous to admit that he was a trickster, who took them for a ride. Better to let him go down in history as the world's greatest warrior against terrorism and as the hero of the discovery of a plot to blow up 10 US-bound planes.

16. They sink or swim with Their Man in Islamabad.

(The writer is Additional Secretary (retd), Cabinet Secretariat, Govt. of India, New Delhi, and, presently, Director, Institute For Topical Studies, Chennai. E-mail: itschen36@gmail.com )


Copyright South Asia Analysis Group

tomder55 answered on 08/26/06:

I have to agree with labman. We cannot take on all dragons at once. After 9-11 Bush did alot of arm twisting in Pakistan to get Paki cooperation .Without their cooperation it would've been almost impossible to take out the Taliban and al-Qaeda in Afghanistan.

Yes I understand he is duplicitous but he understands that he is backing the right side and is doing it at extreme political risk . There are many in his country that would love to see him disappear including some in the hisghest ranks of his gvt. ;his military ;his security and intelligence services. Cooperating with us is a sign of courage . He should be given some slack if he sometimes straddles the fence.

India and Pakistan have a history that includes almost continuos warfare . I ask B.Raman if he would prefer Pakistan to be taken over by the jihadists ? Musharaf is the only barrier against that possibility .

labman rated this answer Excellent or Above Average Answer
paraclete rated this answer Excellent or Above Average Answer

Question/Answer
paraclete asked on 08/25/06 - Another US intelligence failure?

When will Bush get his team together?

US spies slammed for Iran failures

Dafna Linzer in Washington
August 25, 2006


A KEY House of Representatives committee has issued a stinging attack on US intelligence on Iran, saying the CIA and other agencies lack the ability to acquire essential information necessary to make judgements on Tehran's nuclear program or even its ties to terrorism.

The 29-page report, principally written by a Republican staff member on the House of Representatives' intelligence committee, fully backs the White House position that Iran is moving forward with a nuclear weapons program and poses a significant danger. But it attacks intelligence agencies for not providing enough direct evidence to support that claim.

Little evidence has been found to tie Iran to al-Qaeda and to the recent fighting between Israel and Hezbollah in southern Lebanon, says the report, which relies exclusively on public documents. Its authors did not interview intelligence officials.

But it warns the agencies to avoid the mistakes made over weapons of mass destruction before the Iraq war, saying Iran could easily be engaged in "a denial and deception campaign to exaggerate progress on its nuclear program as Saddam Hussein apparently did concerning his WMD programs".

"We want to avoid another 'slam dunk'," the committee chairman, Peter Hoekstra said, explaining why the staff report was made public. "We think it's important for the American people to understand the kinds of pressures that we are facing and to increase the American public's understanding of Iran as a threat."

The then CIA director, George Tenet, had called prewar intelligence on banned weapons a "slam dunk", but no such arms were ever found.

The report comes as the Bush Administration scrambles for leverage in its bid to force Iran to halt its nuclear program. Some Republicans privately oppose President George Bush's policy of potential engagement with Iran and believe it is crumbling in the face of European reluctance to impose strict measures on Tehran.

A spokesman for the intelligence groups disclosed that the report's principal author was Frederick Fleitz, a former CIA officer who had been a special assistant to John Bolton, a former undersecretary of state at the State Department. Mr Bolton, now US ambassador to the United Nations, had previously influenced the crafting of a tough policy that rejected talks with Tehran.

Coinciding with the report's release, policymakers expressed anger that spy agencies were playing down intelligence - including from the Israeli Government - of extensive contacts between Hezbollah and members of Iran's Revolutionary Guard. "The people in the community are unwilling to make judgement calls and don't know how to link anything together," one senior US official said.

"When they say there is 'no evidence,' you have to ask them what they mean? What is the meaning of the term 'evidence'?"

A separate report from the Royal Institute of International Affairs in London meanwhile has concluded that Iran has replaced the US as the most influential foreign power inside Iraq. It said the turmoil unleashed by the invasion of Iraq gave Tehran the chance to fill the void left by Saddam Hussein's downfall.

The Washington Post, The New York Times

This lack of ability is what has plunged the world into war once, it must not be allowed to do it again!

tomder55 answered on 08/25/06:

as in the intel related to Iraq ;the CIA is hardly the single analytical intel agency in the world and it appears to be true here ,like in Iraq ,that there is a broad concensus about the stage of development of the Iranian program.

I think that this report that the slimes and the Compost has willingly mislead their readers . The report that Peter Hoekstra's committee issued was not a caution that we are overestimating Iran's progress but that it is possible we are underestimating it ...that we learned too late of the growing threat .

Read this report in the slimes :The new report, from the House Intelligence Committee, led by Representative Peter Hoekstra, Republican of Michigan, portrayed Iran as a growing threat and criticized American spy agencies for cautious assessments about Irans weapons programs. Intelligence community managers and analysts must provide their best analytical judgments about Iranian W.M.D. programs and not shy away from provocative conclusions or bury disagreements in consensus assessments, the report said, using the abbreviation for weapons of mass destruction like nuclear arms.

Some policy makers also said they were displeased that American spy agencies were playing down intelligence reports including some from the Israeli government of extensive contacts recently between Hezbollah and members of Irans Revolutionary Guard. The people in the community are unwilling to make judgment calls and dont know how to link anything together, one senior United States official said.

Were not in a court of law, he said. When they say there is no evidence, you have to ask them what they mean, what is the meaning of the term evidence?

Itsdb rated this answer Excellent or Above Average Answer
paraclete rated this answer Excellent or Above Average Answer

Question/Answer
Itsdb asked on 08/24/06 - The NY Times has it right

Well, sort of...

A Matter of Appearances

Published: August 24, 2006

When Judge Anna Diggs Taylor was given the job of deciding whether the Bush administrations wiretapping program was unconstitutional, she certainly understood that she would be ruling on one of the most politically charged cases in recent history. So it would have been prudent for her to disclose any activity that might conceivably raise questions about her ability to be impartial. Regrettably, it was left to a conservative group, Judicial Watch, to point out her role as a trustee to a foundation that had given grants to a branch of the American Civil Liberties Union, a plaintiff in the case.

The foundation in question the Community Foundation for Southeastern Michigan is a large charity that gives out grants to a broad range of organizations engaged in community activity, including some regularly involved in litigation. The $125,000 in grant money directed to the state A.C.L.U. office over several years was for educational programs concerning issues unrelated to the wiretapping case, like racial profiling. While the judge clearly erred in not disclosing this involvement, it wouldnt seem, based on the known facts, to rise to the level of a conflict of interest reasonably requiring that she recuse herself from hearing the case under existing ethics rules.

Judge Taylors role at a grant-making foundation whose list of beneficiaries includes groups that regularly litigate in the courts is still disquieting and, even worse, it is not all that unusual for a member of the judiciary. The most important lesson here may be the wisdom of re-examining the sort of outside activities that are appropriate for sitting federal judges.

tomder55 answered on 08/25/06:

May be the first time in US history that the judge was also the plaintiff.
Judge Taylor is a law unto herself .As long as were appreciating irony, lets consider the irony of emphasizing the importance of holding one branch of the federal government, the executive, to the strict limits of the rule of law while sitting in another branch of the federal government, the judiciary, and blithely ignoring your own obligations.

It appears that conflicts of interest and shady politics run in the family . From Gateway Pundit blog :"She worked as the Legislative Assistant/Detroit Office Manager to liberal U. S. Rep. Charles C. Diggs, Jr., (D-MI) from 1967-1970- who was her first husband. His district included downtown Detroit and some of the city's poorest neighborhoods. He was the first chairman of the Congressional black Caucus. He also had trouble with the law: He resigned from Congress in 1980, two years after being convicted of 29 counts of operating a payroll kickback scheme in his office. In 1978, he was stripped of his committee and subcommittee chairmanships. He also was censured by the House. Anna Diggs Taylor was married to Rep. Charles C. Diggs, Jr. Anna Diggs Taylor later married S. Martin Taylor. He also was in democratic politics and helped Coleman Young in his 1973 campaign and Jimmy Carter in his 1976 victory. She became a Federal Judge in 1979. So, in case you are wondering... Our national security was just decided by a liberal judge closely associated (married) to a scandal-plagued Democratic Representative and America's most liberal and worst president, Jimmy Carter."

Gotta wonder how many other liberal judges are card carrying members of the ACLU and contribute to the group ? It probably happens routinely but in her case as an officer of an organization that is a major benefactor and to have presided over a lawsuit the Michigan chapter brought is over the top . But I doubt any sanctions will be levied against her.

This case did nothing to settle the legal questions on the merit of the NSA program .Her opinion is so flawed in it's sloppy writing and disconnect from any logical legal foundation that it has to be overturned.Even legal analysts who agree with her decision are baffled at her reasoning as how she came to it.

But I do think that the issue of the constitutionality of the program needs to be resolved and quickly . I do think we need a concensus in this country about how to proceed in the war against jihadistan because it will exist beyong this and the next administration and probably for the rest of my lifetime .



Itsdb rated this answer Excellent or Above Average Answer

Question/Answer
Bradd asked on 08/24/06 - John Birch Society

Is this organization still influential within the right-wing now that Communism (at least the Soviet flavor) has collapsed?

Is it still anti-Communist?

I remember it claiming that Eisenhower was a "conscious member of the Communist conspiracy" and other odd notions, including what appeared to be a close association with the Christian fundamentalists.

I don't hear much about it these days.

tomder55 answered on 08/25/06:

I would have to say that any ties they may have had with the right wing except possibly the Pat Buchannans of the world have been severed although many members still claim to support the GOP in some manner. They still belly ache about globalization. They voted as a group last year to impeach GW Bush over pretty much the same issues that the Kossacks rant about .

Bradd rated this answer Excellent or Above Average Answer

Question/Answer
Coup_de_Grace asked on 08/23/06 - French Annoy Violent Fascists....even today

"Another thing that got on my nerves was the loathsome cult for France which the big press, even then, carried on. A man couldn't help feeling ashamed to be a German when he saw these saccharine hymns of praise to the 'great cultural nation.' This wretched licking of France's boots more than once made me throw down one of these 'world newspapers.' And on such occasions I sometimes picked up the Volksblatt, which, to be sure, seemed to me much smaller, but in these matters somewhat more appetizing. I was not in agreement with the sharp anti-Semitic tone, but from time to time I read arguments which gave me some food for thought.

At all events, these occasions slowly made me acquainted with the man and the movement, which in those days guided Vienna's destinies: Dr. Karl Lueger I and the Christian Social Party.

When I arrived in Vienna, I was hostile to both of them.

The man and the movement seemed 'reactionary' in my eyes.

My common sense of justice, however, forced me to change this judgment in proportion as I had occasion to become acquainted with the man and his work; and slowly my fair judgment turned to unconcealed admiration. Today, more than ever, I regard this man as the greatest German mayor of all times.

How many of my basic principles were upset by this change in my attitude toward the Christian Social movement!

My views with regard to anti-Semitism thus succumbed to the passage of time, and this was my greatest transformation of all.

(Volume I, Chapter 2)MEIN KAMPF by Adolph Hitler

~~~~~~~~~~~~~~~~~~~~~~~~~~~~~~~~~~~~~~~~~~

tomder55 answered on 08/24/06:

do you think the recent actions of the French have been honorable or duplicitous ?

Coup_de_Grace rated this answer Poor or Incomplete Answer
ETWolverine rated this answer Excellent or Above Average Answer
Itsdb rated this answer Excellent or Above Average Answer

Question/Answer
paraclete asked on 08/23/06 - A stiff upper lip old chap and don't mind those Paki wallahs, eh?

US stumped by front page cricket news


August 23, 2006 - 3:05PM

Darrell Hair may not score a goodwill ambassador job in Pakistan anytime soon, but the Australian umpire has achieved something rare for cricket - front page news in the US.

Cricket seldom rates a mention in the US media.

Americans can't get over the fact a game can last five days, with breaks for "tea", and end in a draw.

Shane Warne may have made saucy headlines in Australia and England when he was caught in his Playboy underpants with two bimbos earlier this year, but thankfully for Americans, the story did not get a run in the US.

Shane who?

When the Ashes gets underway in Australia in November, Aussie and English expats in the US will be lucky to find a mention in American newspapers' sports pages.

But this week's ball tampering incident in London involving Hair and the Pakistan and English teams struck a chord with American editors.

Today's Los Angeles Times ran the story on its front page, relegating a killer bomb blast in Moscow, Saddam Hussein's genocide trial, the deaths of four US troops in Iraq and the Lebanon-Israeli conflict to its latter pages.

The New York Times also ran the cricket story.

"When a match becomes a scandal, that's just not cricket," the LA Times' headline read on the front page.

The US media was fascinated how the "genteel" game of cricket could get so ugly.

"An outsider - someone from the moon, say, or the United States - might imagine that this was merely a folly of the silly season in a game followed only by erudite aficionados of leg-byes, googlies and silly-mid-ons, to mention but a few of crickets more esoteric terms," the NY Times wrote.

"But that is not what this conflict turned out to be."

AAP

tomder55 answered on 08/23/06:

do cricket players use steroids ?

I sat though about 2 hours of a cricket match in the Bahamas . Got a good tan and a nice snooze.

paraclete rated this answer Excellent or Above Average Answer

Question/Answer
ETWolverine asked on 08/23/06 - The French Double-Cross

From National Review online.

August 22, 2006, 6:05 a.m.

Youll Never Confuse George W. Bush for a Frenchman
The worlds sole responsible power.

By Rich Lowry


The United States is not just the worlds sole superpower, it is the worlds only responsible power.

Consider the recent action related to a peacekeeping force taking control of southern Lebanon from Hezbollah. France initially agreed with the United States on a United Nations resolution creating an international force that would operate with robust rules of engagement to confront the terrorist guerrilla group. When the Arabs balked, France insisted that the rules of engagement be made considerably vaguer. Since France was going to lead the force, the U.S. deferred to Paris, which has subsequently said that it will contribute only 200 combat engineers to the force because ... the rules on engagement are so vague.

This is a spectacularly bald-faced diplomatic double-cross that makes one wonder if Secretary of State Condi Rice was wearing a Kick Me sign when she voted for the resolution in New York. It sinks any hope of a lasting settlement in southern Lebanon and further undermines the credibility of the United Nations. But the French dont care. They were able to serve their political purposes in the Middle East by triangulating between the United States and the Arabs. Consequences be damned.

Say what you will about the efficacy or delicacy of U.S. foreign policy, this is cynicism, bad faith, and rank selfishness of which America is almost incapable as a world power.
Indeed, in our willingness serially to believe the unreliable assurances of the French, we are an innocent abroad. First, they snookered us three years ago into believing that they wouldnt kick up trouble for us at the U.N. in the event Saddam Hussein didnt fully comply with his disarmament obligations. Now, we have been played the fool in Lebanon.

The root of our seeming navet is the earnest desire to deal with world problems. Saddam was a menace, but France and Germany were content to play diplomatic and political games at our expense. Southern Lebanon is, as we have seen in recent weeks, a deep source of instability in the region. The U.S. wanted to craft a long-term solution, but since we werent going to send troops ourselves, we needed a partner. Enter: France. Exit: any chance of a real settlement.

Civilization simply lacks backbone without the United States in the lead. Everyone agrees that a nuclear North Korea is a danger, but Russia and China play the role of enablers. Everyone thinks the same about Iran, but Europe is willing only to dither. Everyone thinks Iraq descending into chaos would be a disaster, but only the U.S. is pouring major resources into preventing it (granted, its our baby). Everyone supports the Afghan war, and NATO is actually pitching in there, but the Taliban is emboldened on the assumption that our European allies wont have the same commitment to doing the job that we do.

This is not to say that the U.S. is flawless. Our mistakes, however, tend to be the products of an excess of zeal and idealism. We dont do coldblooded calculation well. Some of this is the product of being a superpower dishonest diplomatic ploys are beneath us. Some of it is the nature of our democracy, which values openness and honesty.

Paranoid critics charge that we are in Iraq to control its oil. The French could have pulled off such a self-serving maneuver clothed in idealism, but we are in Iraq for exactly the achingly innocent reasons we say. We are spending and bleeding there trying to plant a liberal democracy in the hardscrabble soil of Mesopotamia.

When President Bush is gone, conservative foreign policy will change. But it wont be a change the foreign-policy establishment likes. It wont be toward a lets-talk-even-more-to-the-French multilateralism as represented by Nebraskas tiresome Republican Sen. Chuck Hagel. It will be something more selfish and hardheaded, something more French in its motivation Bush without the soft touches. Then, the world will miss the earnest do-gooding United States of old.

Rich Lowry is author of Legacy: Paying the Price for the Clinton Years.

2006 by King Features Syndicate


---------------

Comments? Is the rest of the world as calculating as Lowry says? Do we lack that cut-throat calculation in the world of international politics? Is that idealism our great weakness?

Elliot

tomder55 answered on 08/23/06:

In my answer to Clete's Lebanon question I float a theory I've heard that the French were at best useful idiots in the negotiations of 1701 (a role they are well suited for) .

I think Lowery has an interesting point . Mark Levin's current blog entitled "Doctrinal Issues
"
is very disturbing in that he thinks that Bush is losing the energy or desire to see his doctrine through.

For the sake of my own sanity I prefer the theory that 1701 was an elaborate deception .

ETWolverine rated this answer Excellent or Above Average Answer
Itsdb rated this answer Excellent or Above Average Answer
paraclete rated this answer Excellent or Above Average Answer

Question/Answer
paraclete asked on 08/23/06 - wither goes the lebanon?

Everybody got a bloody nose, except Iran and Syria


August 23, 2006

Israel's Lebanon adventure failed, but Hezbollah will not be seeking a return bout just yet, writes Tom Teepen.

SO WHO won in Lebanon? Israel? Hezbollah? Right question. Wrong time. Try again in a couple of years or in several. It will be a long time before this nasty little war's dust, swirled by the Middle East's ever contrary and battering winds, settles.

An even better question is rather longer than the terse one above: who won? Who lost? Hezbollah? Israel? Lebanon, Syria, Iran, the US? Moderate Islam or Jihadist Islam? Shiite Islam or Sunni?

In this short run, Hezbollah is crowing and the region's radicals quickly formed a chorale of concurrence. Hezbollah did hold off Israel, although an Israel that pulled up short rather than fully committing. Hezbollah's cadres showed themselves larger, better armed and more determined than just about anyone had expected.

And Hezbollah's Sheik Hassan Nasrallah has emerged as the new poster boy for anti-Israel zealots.

Iran and Syria, Hezbollah's enablers, armers and eggers-on, gained regional stature, with Iran provisionally the area's go-to guy, a status that will be cemented if Iran goes nuclear. Pan-Arabism flopped 30 years ago and Arab nationalism never filled the void. Now supranational Islamism may. Hope that the widening Sunni-Shiite split bars that.

The Iraq war, which George Bush and his people still tout as the front line against terrorism, once again showed itself to be only an ugly sideshow sucking up US treasure and lives in the forlorn service of a sectarian civil war.

Israel failed in its strategic objectives of disarming Hezbollah and implicitly cautioning Iran against adventurism. And in failing, Israel lost its intimidating reputation for military invincibility, which had been a security deterrent in its own right. That just about guarantees that some fool in the region will soon give Israel a chance to re-establish that reputation.

Israel's legendary intelligence, this time askew, brought a microscopic review on itself. Its Prime Minister, Ehud Olmert, stands on political quicksand. One "victory" Hezbollah may well have won for Islam and the Arab world is the election of a harder-line, more aggressive successor to Olmert.

Although giddy at the moment, Hezbollah is bloody as well. It survived but took huge losses in arms and personnel. Another similarly rash project is unlikely to appeal to Hezbollah's leadership any time soon.

Much will depend upon what the Lebanese make of the set-to in the long run. Right now they are furious with Israel, but they also know that Hezbollah brought this grief upon them, and an eager and skilled mercantile people cannot happily imagine the Islamist state Hezbollah and Iran intend for them. The key question then will be whether the West, with nothing less than its way of life at issue, will rally to the Lebanese if they send up flares.

Cox News Service

tomder55 answered on 08/23/06:

As you know I was as critical of Bush and Rice as I've ever been ,and I still have not gotten over it . However ;just for fun ,I'll float another theory that I recently heard .

Suppose it was apparent to them that Res. 1701 was bogus from the get-go and in fact Condi allowed the French to contruct this farce .

Why would they participate in this charade ? Well it became apparent early on that things were not going well for Israel and that Olmert had badly botched the war . (for proof of this see the IDF reservists who are now picketting in Jerusalem ..read of reports of IDF troops deploying and then having to scavenge for food because someone forgot to supply MREs;of empty storehouses for whole units ..read about the many tank troops who were supposedly killed as the command waffled about their instructions ,so they sat immobile like sitting ducks while the Hezzys took pot shots at their tanks ) .

As 1701 is collapsing right in front of our eyes it is not clear that Condi and Bush are doing anything to attempt to salvage it .Meanwhile IAEF picks off convoys in Bekaa with a wink and a nod and a proclaimation from Condi that Israel has a right to defend itself. It becomes increasingly clear that 1701 was designed to whither on the vine and die. Nothing else makes sense. Meanwhile as John Batchelor says the sides reload. When the fight resumes the IDF will have gotten a needed breather. Israel will have completely mobilized for a larger regional war .It will be easy for them to retake the territory that they have ceded . You will see a better prepared IDF and hopefull a war time leadership emerge in Israel.

I could be wrong but nothing else that has happened in the last month makes any sense to me .




paraclete rated this answer Excellent or Above Average Answer

Question/Answer
Coup_de_Grace asked on 08/21/06 - Mexican Plan to Annex American Southwest

PAT BUCHANAN: "The nature and character of the invasion is far different than anything that used to happen. 58% of the Mexican people in one survey indicated they believed that the American Southwest belonged actually to Mexico. It was stolen from them. It belongs to them and I think that the Mexican government has a direct program basically to push its poor, unemployed, and uneducated into the United States for a variety of purposes. And one of them, in my judgment in which I believe I documented it in the book is an attempt at the reconquista they call it, the reannexation of the seven days states of the American Southwest, link linguistically, ethnically and culturally to become as much a part of Mexico is they are a part of America. And I think that is well underway.

Filed under: Immigration

Pat Buchanan on a radio talk show.

~~~~~~~~~~~~~~~~~~~~~~~~~~~~~~~~~~~~~~~~~~~

Waddaya think?

tomder55 answered on 08/22/06:

I think that reconquista has been an unofficial defacto policy for some time now .
In the US it manifests itself with groups like MEChA "Movimiento Estudiantil Chicano de Aztlan." or "Chicano Student Movement of Aztlan."MEChA is an Hispanic separatist organization that encourages anti-American activities and civil disobedience. The radical members of MEChA who refer to themselves as "Mechistas," romanticize Mexican claims to the "lost Territories" of the Southwestern United States -- a Chicano country called Aztlan. In its national constitution, MEChA calls for self-determination by its members to liberate Aztlan. MEChA's national constitution starts out: "Chicano and Chicana students of Aztln must take upon themselves the responsibilities to promote Chicanismo within the community, politicizing our Raza(race) with an emphasis on indigenous consciousness to continue the struggle for the self-determination of the Chicano people for the purpose of liberating Aztln."Aztlan being the combined Mexico with the US SouthWest as a single nation. Voz Fronteriza, a Chicano student publication referred to Border Patrol officers killed in the line of duty as "pigs (migra)" trying to defend "the false frontier."

Academia Semillos del Pueblo (ASDP), a charter school partially funded by the Los Angeles Unified School District apparently advocates reconquista as part of it's curriculum with classes like class is called, A People's history of Expansion and Conflict, which includes, A thematic survey of American politics, society, culture and political economy; Emphasis throughout on the nations the U.S. usurped, invaded and dominated".The school received a founding grant from National Council of La Raza (The Council of The Race). The school's principal, Marcos Aguilar is a Mexican revolutionary radical who led a group of Latino students to seize a faculty lounge at UCLA in 1993 and set it on fire, causing $50,000 - $100,000 in damages.


Sorta put a steak in the heart of the now romantic image of the US being the big melting pot that absorbs and assimilates it's immigrants .We will balkanize under the Buchannan scenario because the wave of immigrants from a single nation is coming in faster than they can be assimilated . We have had waves of immigrants in the past of course but he argues that in between there were "time outs" as he calls them where the flow slowed . To further complicate the formual ,their physical proximity to Mexico gives Chicanos the option of life in both countries , in two places and in two cultures, something earlier immigrants never had. I often tell my employees that the first time my grandparents returned to Italy was when they retired and went on a Perillo Tour .What we do not need here is a situation like Canada has in Quebec where they are semi-autonomous and there is always a threat of union disolution.



ETWolverine rated this answer Excellent or Above Average Answer
Coup_de_Grace rated this answer Excellent or Above Average Answer
Itsdb rated this answer Excellent or Above Average Answer

Question/Answer
Mathatmacoat asked on 08/21/06 - Is this the stuff that wars are made of?

This has big implications, if some countries grow then some might shrink.

Bid to extend maritime boundaries
From correspondents in New York
August 21, 2006 01:24pm
Article from: Agence France-Presse

BRITAIN, France, Ireland and Spain are to launch a bid to extend their territorial waters at a UN conference on ocean rights that opens today, thus grabbing precious natural resources.

In a joint submission to the UN Commission on the Limits of the Continental Shelf, the four states have applied to extend their territorial waters in the Bay of Biscay and the Celtic Sea beyond the current legal limit of 200 nautical miles.

The commission meets at United Nations headquarters in New York from Monday to September 15.

The four countries are submitting scientific proof that their continental shelf, the submerged prolongation of land territory, stretches beyond 200 nautical miles and that their territorial waters should therefore be extended.

And the stakes are high.

Under the UN Convention on the Law of the Sea, which came into force in 1994, each country benefits from an exclusive economic zone (EEZ) within the 200-mile boundary from their shoreline.

This means each state has exclusive rights over all natural resources within their territorial waters, including fish, oil, gas, minerals and other precious reserves found in the sea and the ocean's subsoil.

A clause in the convention stipulates that the exclusive economic zone can be extended to up to 350 nautical miles if countries can scientifically prove that their continental shelf extends beyond the 200-mile boundary.

France has the biggest exclusive economic zone in the world after the United States, which has not signed the Convention on the Law of the Sea.

The submission by Britain, France, Ireland and Spain concerns a "small" zone of 80,000sq km, according to Walter Roest of the French Research Institute for Exploitation of the Sea (Ifremer) who will be among those presenting scientific research to back up the request.

The commission is charged with examining the validity of the four countries' request, based on scientific criteria.

This includes proving that the area is "a natural prolongation of the continental shelf, based on its shape, composition of the bedrock and shallow depth that extends far out to sea, for example off the coast of Brittany," in western France, Mr Roest told AFP in Paris.

"We have filed a joint submission to establish the limits of our shared shelf and then we will agree among ourselves how to divide up the maritime area," he said.

Mr Roest said he expected the UN body to rule on the scientific evidence within a year.

The scientist added that France would also file other requests concerning zones surrounding its overseas territories of the Kerguelen Islands in the Indian Ocean, New Caledonia in the Pacific and French Guiana in Latin America.

France, whose exclusive economic zone currently covers 10.2 million square kilometres, hopes to gain an extra one million square kilometres, Roest said, adding that Paris had to move fast as all submissions must be presented by 2009.

"Today, the commission is not yet overloaded but about 60 requests could be submitted by coastal states by 2009," he said.

New Zealand, Brazil and Australia are currently preparing submissions of their own.

Many countries will be scrambling to capture as much area as possible as the economic rewards could be huge.

Countries will have exclusive rights to exploit "hydrocarbons, minerals, living species on the ocean floor and bacteria for use in biotechnology," Roest explained.

Fishing rights though will remain limited to within 200 nautical miles from the coast, he added.

tomder55 answered on 08/21/06:

China has been doing it without any UN approval for almost a decade now . As You may recall I posted about The Law Of The Sea treaty (LOST a perfect acronym )a while ago . It was a treaty that President Reagan undertanding the dangers refused to sign . But President GHW Bush and Clintoon went out of their ways to get the treaty signed by ushering in some superficial revisions .

The biggest flaw in the treaty involves mining rights . It greatly expands a countries territorial claim and it creates a nightmare multinational bureaucracy that sounds like an excerpt from George Orwell's ��": At its center is the International Seabed Authority.'(ISA) The Authority '(as it calls itself) supervises a mining subsidiary called the 'Enterprise', ruled by an Assembly, Council, and various commissions and committees. Mining approval would be highly politicized.Companies that are allowed to mine would owe substantial fees to 'the Authority '(taxes to the UN ...how precious !!! ) and be required to do surveys for the Enterprise.

This new super "authority " will give the UN unprecendented power to regulate seven-tenths of the world's surface area, levy international taxes, impose production quotas (for deep-sea mining, oil production, etc.), govern ocean research and exploration, and create a multinational court to render and enforce its judgments. Some even aspire to giving the U.N. some of the US warships so it can have "blue hulls" to go along with its "blue helmets" to ensure that their edicts are obeyed.

Brazil, China, India, Malaysia, North Korea, Pakistan, and others have been making ocean claims deemed excessive by others under the protective umbrella of the treaty . China is using its own unique interpretation of the treaty to justify its increasing control over the South China Sea. The PRC creates and fortifies man-made islands near that sea's rich oil and mineral deposits, then asserts that LOST entitles it to exclusive economic control of the waters within a 200 nautical-mile radius ... including waters and choke points transited by Japanese and American oil tankers en route to and from the Persian Gulf.

Mathatmacoat rated this answer Excellent or Above Average Answer

Question/Answer
Coup_de_Grace asked on 08/18/06 - Iraq Forced to Import Oil

AP "Iraqi officials announced plans to double the amount of money spent to import fuel to combat the country's worst oil and gasoline shortages in years. Much of the fuel crisis is due to insurgent attacks on convoys and on Iraq's fragile pipeline network, Oil Ministry officials said."

~~~~~~~~~~~~~~~~~~~~~~~~~~~~~~~~~~~~~~~~~~~~~~~~

I remember that Bush sold his plan for occupation by saying that rebuilding of Iraq and the new democratic government would be paid for by money from the export of Iraqi oil.

No one can deny that the War on Iraq has been a total disaster judging by RESULTS.

Shiite Iran is totally empowered after a Hezbollah victory against Israel.

Comments?

tomder55 answered on 08/20/06:

actually it was my understanding that Iraq is swapping crude oil for refined petroleum. With the price of oil today the money should be flowing into Iraq . However ;the combinations of 2 decades of neglect of the oil infrastructure ;the insurgencies sabotage and attacks on it which discourage investment in the industry ,and most importantly the huge increase in demand by the Iraqi consumer has left production goals behind demand.

As of Aug. 14 Iraq's crude oil production is back at 2.5 million barrels/day after the pipeline that takes oil to Turkey was repaired. Iraq's Oil Minister Hussain al-Shahristani said plans are under way to raise Iraq's oil production to between 2.9 million and 3 million barrels /day (b/d) by the end of the current year, and raise it by 500,000 b/d annually by 2010 ;all through the domestic oil industry ..any foreign investments will increase the output.

I am hearing reports that Iran is actually angry at Hezbollah and Nasrallah . They depleted the missile supply that it took Iran years to build up with little to show for it ;and it is not likely that the stock can be resupplied .Also they are not pleased that Nasrallah emerged from the war as a rock hero in the Muslim world. Iran did not even get mentioned in the Cease-fire language as if their contribution to the conflict was irrelevent. They are also angry at the timing of the attack on Israel. They wanted the attack on Israel with missiles delayed until it better suited their time line (October or November ).

Iran is in no way 'empowered' however you have your finger on their desires. I am concerned about the coming weeks . They have something planned and I think it has something to do with the announcement of a possible N Korean nuclear test .


Coup_de_Grace rated this answer Excellent or Above Average Answer
Itsdb rated this answer Excellent or Above Average Answer

Question/Answer
Coup_de_Grace asked on 08/18/06 - Gitmo Revamp has an Execution Chamber

There is an execution chamber in the update of the Gitmo detention camp in Cuba.

~~~~~~~~~~~~~~~~~~~~~~~~~~~~~~~~~~~~~~~~~~~~

Comments?

tomder55 answered on 08/20/06:

Let's assume that all the hand wringing and all the judiciary maneouvers are FINALLY Exhausted and tribunals are ultimately approved under existing provisions of the UMCJ (which as I recall even SCOTUS had no problem with in their Hamdan v. Rumsfeld decision[ the court found no real problems with the tribunal rules, only that those rules need to be voted into law by Congress. ]).

I think the military in fact thinks that is exactly what will happen . Tribunals /and sentencing would be the military's responsibility . Guess what ...there are provisions in the UMCJ for the death penalty and I have no doubt that some of these 'detainees' are being held for reasons where that sentancing is more than justified.

The Camp Delta renovations being considered plans for this eventuality . A new court house is being designed and built . A more permanent housing facility needs to be built to imprison for longer periods of time those detainees who the tribunals deem deserve longer confinement .An execution chamber would also be needed if death penalties are imposed.

Army Col. Frederic Borch III has been designated the chief prosecutor and Air Force Col. Will Gunn as chief defense lawyer for the proposed trbunals. The Pentagon has listed 18 war crimes and eight other offenses that could be tried, including terrorist acts, and has issued rules for the tribunals. The cases would be decided by a panel of three to seven military officers who act as both judge and jury. Convictions could be handed down by a majority vote; a decision to sentence a defendant to death would have to be unanimous.


Coup_de_Grace rated this answer Excellent or Above Average Answer
Itsdb rated this answer Excellent or Above Average Answer
MicroGlyphics rated this answer Excellent or Above Average Answer

Question/Answer
Itsdb asked on 08/17/06 - Interpreting the news...

HARTFORD, Conn. - Ned Lamont, whose anti-war campaign rattled the political landscape by toppling Sen. Joe Lieberman in Connecticut's Democratic primary, is gaining support among voters - but Lieberman still has an edge, according to a poll released Thursday.

The Quinnipiac University poll has Lieberman leading Lamont among registered voters 49 percent to 38 percent. Republican Alan Schlesinger gets support from 4 percent. Among likely voters, Lieberman was supported by 53 percent, compared to Lamont's 41 percent and Schlesinger's 4 percent.

Lieberman, a nationally known centrist who has been criticized by many Democrats for supporting the war in Iraq and a perceived closeness to President Bush, lost the Aug. 8 Democratic primary by 10,000 votes. Political pundits say the primary was evidence of voters' frustration with the war and predict it could have national political ramifications.

Lieberman's advantage in the general election comes from broad support among unaffiliated and Republican voters. Fifty-three percent of likely voters polled said he deserves to be re-elected, and nearly half doubted that Lamont, a political novice who founded a company that wires college campuses for cable television, has enough experience to be senator.

"Senator Lieberman's support among Republicans is nothing short of amazing. It more than offsets what he has lost among Democrats," poll director Douglas Schwartz said. "As long as Lieberman maintains this kind of support among Republicans, while holding a significant number of Democratic votes, the veteran senator will be hard to beat."

Lamont, however, is improving since a July 20 Quinnipiac poll. In that survey of registered voters, he trailed Lieberman 51 percent to 27 percent with Schlesinger getting 9 percent. The latest poll quizzed both registered voters and voters likely to cast ballots; the July 20 poll only questioned registered voters.

Top state and national Democrats, including Sens. John Kerry, Ted Kennedy, Chris Dodd, Hillary Clinton and Frank Lautenberg, abandoned Lieberman after the primary and are endorsing Lamont. Former Sen. John Edwards, the 2004 candidate for vice president, was to campaign for Lamont on Thursday.

Some Senate Republicans, meanwhile, are throwing their support behind Lieberman instead of Schlesinger, who has been dogged by revelations of that he was sued by two New Jersey casinos for gambling debts, and that he gambled at a Connecticut casino under a false name in the 1990s while a state legislator.

Messages seeking comment were left with the three campaigns early Thursday.

The telephone poll was conducted between Aug. 10 to 14. Quinnipiac surveyed 1,319 registered voters and the poll has a sampling error margin of plus or minus 2.5 percentage points. Among the 1,083 likely voters, the margin of error is plus or minus 3 percentage points.

~~~~~~~~~~~~~~~~~~~~~~~~~~~~~~~~~~~~~~~~~~~~~~~~~~~~~~~

You'd think Lamont was about to pull away with from the pack according to this interpretation. In July Lieberman had 51 percent of registered voters. Today he has 49 percent of registered voters so he lost maybe 2% of those who may or may not vote.

Yet among likely voters his lead increases from 11 percent of registered voters to 12 percent of likely voters, while Schlesinger is in retreat. That apparently adds up to "Lamont is gaining support" while Lieberman still has "an edge."

If I remember right, there was a 7 percent turnout in the Connecticut primary, with Lamont winning by 4 percent. How did Michael Moore put it? Oh yeah, a "resounding defeat" for Lieberman. Lamont has all the big guns out now, Kennedy, Kerry, Hillary, etc., not to mention Michael Moore and the moonbat society trumpeting this "resounding defeat" by 4 percent of Democrats, yet a 12 point lead among likely voters is "an edge"?

Like I said before, Lieberman is going to win.

Comments?

tomder55 answered on 08/18/06:

Predictably the Demon-crats have threatened to penalize Lieberman for daring to challenge as an independent .

Assuming he wins and the nightmare of a Dem. takeover of the Senate occured then Joe would be in line based on his seniority to take over as chairman of the Homeland Security and Governmental Affairs Committee.... the committees that would presumaby take the lead in any investigations about President Bush .

That presents a quandry to the Dems . On the one hand they would like to strip him of his majority if he wins .But on the other hand ;if they do that and let's say that with Joe they obtained a 51 seat majority ;do they dare alienate him and risk him caucusing with the Republicans ? He could conceivably decide which party runs the Senate .

So go ahead Dums. piss him off .Take the Kossak's advice .I dare ya .Do you feel lucky ?

Itsdb rated this answer Excellent or Above Average Answer

Question/Answer
Coup_de_Grace asked on 08/17/06 - In a Nutshell

From today's Judge in landmark court decision::

"The Presidential Oath of Office is set forth in the Constitution and requires him to swear or affirm that he "will, to the best of my ability, preserve, protect and defend the Constitution of the United States."

The Government appears to argue here that, pursuant to the penumbra of Constitutional language in Article II, and particularly because the President is designated Commander in Chief of the Army and Navy, he has been granted the inherent power to violate not only the laws of the Congress but the First and Fourth Amendments of the Constitution, itself.

We must first note that the Office of the Chief Executive has itself been created, with its powers, by the Constitution. There are no hereditary Kings in America and no powers not created by the Constitution. So all "inherent powers" must derive from that Constitution."

~~~~~~~~~~~~~~~~~~~~~~~~~~~~~~~~~~~~~~~~~

There are no powers not created by the Constitution.

tomder55 answered on 08/18/06:

Major cases like this are never decided on the District court level. It's going to the SCOTUS eventually which will decide for the President. No District court judge can tell a President what to do, particularly in national security issues.So her nonsense about ceasing immediately is already old news . Both sides agree that the 6th Court of Appeals will hear the case in the fall.


Decision is here .Read it if you wish ;it is only 44 pages of political screed that more resembes a Daily Kos rant ,disguised as judicial ruling .


Our constitution was drafted by founders and ratified by a people who still held in vivid memory the image of King George III and his General Warrants.

There are no hereditary Kings in America and no powers not created by the Constitution.

She makes an extremely weak case on the 'standing' of the plaintiffs and then goes on to admit that if she does not make this weak case there would be no basis for her ruling .

Although this court is persuaded that Plaintiffs have alleged sufficient injury to establish standing, it is important to note that if the court were to deny standing based on the unsubstantiated minor distinctions drawn by Defendants, the Presidents action in warrantless wiretapping, in contravention of FISA, Title III, and the First and Fourth Amendments, would be immunized from judicial scrutiny.

Her argument as to the standing of those who brought the suit was "Well, if we don't rule that these journalists and other activist plaintifs have standing, who the hell else is going to stand up to Bush?" Even a Clintoon appointees would've realized the case this group of moonbats brought to the table had no standing .Leave it to a Carter judge ! Every Circut Court that has heard NSA realted cases have upheld the NSA's position,even the 9th Circus.

All this ruling really does is give the Democrats their talking points through most of the 2006 election season. No ...nothing political in the timing of this at all !!






Coup_de_Grace rated this answer Excellent or Above Average Answer
Itsdb rated this answer Excellent or Above Average Answer

Question/Answer
MicroGlyphics asked on 08/17/06 - Federal judge orders end to wiretap program: Says governments listening in without warrant is uncons

DETROIT - A federal judge ruled Thursday that the government's warrantless wiretapping program is unconstitutional and ordered an immediate halt to it.

U.S. District Judge Anna Diggs Taylor in Detroit became the first judge to strike down the National Security Agency's program, which she says violates the rights to free speech and privacy.

The American Civil Liberties Union filed the lawsuit on behalf of journalists, scholars and lawyers who say the program has made it difficult for them to do their jobs.

The government argued that the program is well within the president's authority, but said proving that would require revealing state secrets.

The ACLU said the state-secrets argument was irrelevant because the Bush administration already had publicly revealed enough information about the program for Taylor to rule.

Ref: http://www.msnbc.msn.com/id/14393611/

Everybody except the Bush Cabal recognised this, but they insisted on defying the law until a judge ruled it so, and of course there is no indication that the regime will care to stop these illegal acts.

The part I like is that the government argued that they were in the right except that to prove it they "would require revealing state secrets." O! Please. State Secrets? This entire organisation is a secret. It is such a public embarassment, that I am sure it will be expunged from future textbooks. If left intact, it will serve only as a cautionary tale.

Does anybody feel the court was wrongBush attorneys notwithstanding?

tomder55 answered on 08/17/06:

But of course it is unconstitutional to restrict terrorists from blowing up planes ?What was Bush thinking ??

The ACLU filed this case in Detroit knowing there was a moonbat in black robes there to hear the case and rubberstamp the outcome they desired . Why did they not file in DC district where the NSA is or in Virginia ? Because they know they would've been laughed out of court . Suprised they did not go to San Francisco wearing flowers in their hair . At least there they could've predetermined the appellate court decision also with the 9th Circus Court of Appeals .

The equivant to this judicial sleigh of hands would be if some religious group filed a law suit in Alabama saying gay marriage was a constitutional violation . they could say it is based on the "penumbras " of the 14th Amendment ....why not ? everyone else finds things not enumenrated it it ??? Yeah the judge may rule in their favor but ultimately the ruling would be overturned .

Some of the Pakistani Britons just arrested in the al-Qaida plot to blow up U.S. airliners had placed calls to several U.S. cities, including Washington and New York according to Pat Roberts, head of the Senate Intelligence Committee. The NSA spy program turned up evidence of the sleeper cells . Roberts said "They're not calling the United States simply to be calling the United States,".... "I'll just leave it at that."

I find this amazing . You who make a case that the war against Jihadistan should be a law enforcement exercise would tie the hands of the very investigators who you claim should be taking the lead in foghting the war . Do you understand how that plot in Britain was broken up ? How many civil rights do you think the Pakis used when they interrogated ? Read about it in The Guardian . Now we are not talking about torture mind you ;just data mining phone calls to determine a pattern of incoming calls from al Qaeda camps or known terrorists overseees. You need to get some perspective here.

The 9-11 commission filed a huge report on how our intel services were hampered by the likes of Clintonoids like Gorelick and her wall. Don't tell me that you want to handcuff intel and expect them to protect you .

ETWolverine rated this answer Excellent or Above Average Answer
excon rated this answer Excellent or Above Average Answer
Itsdb rated this answer Excellent or Above Average Answer
MicroGlyphics rated this answer Excellent or Above Average Answer
sissypants rated this answer Excellent or Above Average Answer

Question/Answer
jackreade asked on 08/16/06 - Investigation of 77 TV Stations for using Fake News

(AP) The Federal Communications Commission has mailed letters to the owners of 77 television stations inquiring about their use of video news releases, a type of programming critics refer to as "fake news."

Video news releases are packaged news stories that usually employ actors to portray reporters who are paid by commercial or government groups.

The letters were sparked by allegations that television stations have been airing the videos as part of their news programs without telling viewers who paid for them.

FCC Commissioner Jonathan Adelstein said Tuesday the letters ask station managers for information regarding agreements between the stations and the creators of the news releases. The FCC also asked whether there was any "consideration" given to the stations in return for airing the material.

"You can't tell any more the difference between what's propaganda and what's news," Adelstein said."

~~~~~~~~~~~~~~~~~~~~~~~~~~~~~~~~~~~~~~~~~~~~`

Comments?

tomder55 answered on 08/17/06:

I wonder if they included Dan Rather and 60 Minutes ;or Reuters and their fake photos ;or the NY Slimes and Jason Blair's fake reporting ?

Frankly I have to wonder what or why is it the FCC's jurisdiction .If a station decides that something is news then what business is it of the FCCs how the news is packaged and produced ? If my local yokel station gets a release from the military and maybe we have a base in the area and they decide it is newsworthy then it is there business. If they are being paid to air it then they should give full disclosure of course .

Same holds true with the local gvt. They send out press releases all the time that the press picks up . At least in the 2 cases I mentioned the news is the truth . To me fake news is when it is a fabrication not when you don't give full disclosure to the source.

Don't think this only happens with gvt. either . Advocacy groups of all stripes vye for the media influence . How often do you hear a story spinning information in favor of environmentalist concerns or labor unions . Where did these local stations get this info .......from hard investigation ??? bwahaaahaaahaa. They read a press release from PETA or NEA or AARP ,retype it and bingo some talking head says it on air .

jackreade rated this answer Average Answer

Question/Answer
Itsdb asked on 08/16/06 - "Facts don't justify wholesale murder"

That was a comment on my answer to Elliot's previous question.

So, for those who need reminding again...

"Iraq is a terrifying place to live. People are in constant fear of being denounced as opponents of the
regime. They are encouraged to report on the activities of family and neighbours. The security services can strike at any time. Arbitrary arrests and killings are commonplace. Between three and four million Iraqis, about 15% of the population, have fled their homeland rather than live under Saddam Husseins regime.

These grave violations of human rights are not the work of a number of overzealous individuals but the deliberate policy of the regime. Fear is Saddams chosen method for staying in power."

That was the opening lines of "SADDAM HUSSEIN: crimes and human rights abuses", by the Foreign & Commonwealth Office in London, 2002. Sources of this report:

Sources: US Committee for Refugees Report 2002, Human Rights Watch Country Report, International Alliance for Justice News Service 12/9/2002, Amnesty International Report Victims of Systematic Repression, British Governments own sources

Selections from the report:

"On 19 April 2002, the UN Commission on Human Rights passed a resolution drawing attention to the systematic, widespread and extremely grave violations of human rights and of international humanitarian law by the Government of Iraq, resulting in an all-pervasive repression and oppression sustained by broad-based discrimination and widespread terror.

"Torture is systematic in Iraq. The most senior figures in the regime are personally involved."

Udayy Saddam Hussein:

"Saddams elder son. He has been frequently accused of serial rape and murder of young women. He maintained a private torture chamber, known as al-Ghurfa al-Hamra (the Red Room), disguised as an electricity installation, in a building on the banks of the Tigris.

He personally executed dissidents in Basra during the uprising that followed the Gulf War in March 1991. In one infamous incident of mass torture, Udayy Hussein ordered the national football team to be caned on the soles of their feet after losing a World Cup qualifying match."

Qusayy Saddam Hussein:

"Saddams younger son. As head of the Iraqi internal security agencies, he has permitted and encouraged the endemic use of torture, including rape and the threat of rape, in Iraq."

"Under Saddam Husseins regime women lack even the basic right to life. A 1990 decree allows male relatives to kill a female relative in the name of honour without any punishment. Women have been tortured, ill-treated and in some cases summarily executed too, according to Amnesty International."

The Mahjar prison...normal occupancy of the Mahjar is 600-700 people. Thirty of the cells are underground and thirty other cells used to be dog kennels. Prisoners are beaten twice a day and the women regularly raped by their guards. They receive no medical treatment, but some prisoners have survived up to a year in the Mahjar. Two large oil storage tanks each with a capacity of 36,000 litres have been built close to the Mahjar. The tanks are full of petrol and are connected by pipes to the prison buildings in the Mahjar. The prison authorities have instructions to set light to the petrol and destroy the "Mahjar" in an emergency.

The Sijn Al-Tarbut (the casket prison)...in Baghdad. The prisoners here are kept in rows of rectangular steel boxes, as found in mortuaries, until they either confess to their crimes or die. There are around 100-150 boxes which are opened for half an hour a day to allow the prisoners some light and air. The prisoners receive only liquids.

The Qurtiyya (the can) prison...consists of 50-60 metal boxes the size of old tea chests in which detainees are locked under the same conditions as the Sijn Al-Tarbut. Each box has a tap for water and a floor made of mesh to allow the detainees to defecate."

Instructions for dealing with demonstrations:

    1. All officers, deputies and NCOs to report to their bases with all their weapons immediately upon hearing of a demonstration, in order to receive instructions.

    2. All of those responsible for the self defence of the directorate to remain at post without leaving their place of duty under the supervision of officer in charge.

    3. In the event of a hostile demonstration, these groups will be contained by closing all access routes and by taking control of all high points overlooking them.

    4. After taking the above measures and containing the hostile elements, armed force will be used in accordance with central instructions to kill 95% of them, and to leave 5% for interrogation.

    5. If the force comes under hostile fire from other directions and it is possible that there are saboteur elements in the vicinity to protect the demonstration, the force will return fire intensively.

    6. An emergency force will be prepared to reinforce the primary force and to defend sensitive sites.

    7. The technical unit will, when authorised, use technical means as instructed under the supervision of the officer of the unit and the security representative Tahir Mahmud Ahmad.

    (Editorial note: technical unit and technical means are euphemisms for chemical weapons.) (Steve's note...shooting fish in a barrel)


"In 1984, 4,000 political prisoners were executed at a single prison, the Abu Ghraib. An estimated 2,500 prisoners were executed between 1997 and 1999 in a further prison cleansing campaign. In February 2000, 64 male prisoners were executed at Abu Ghraib, followed in March by a further 58, all of whom had previously been held in solitary confinement. In October 2001, 23 political prisoners, mainly Shia Muslims, were executed at Abu Ghraib.

Between 1993 and 1998 around 3,000 prisoners from the Mahjar prison were executed in an execution area called the Hadiqa (garden) near to the prison."

"Documents captured by the Kurds during the Gulf War and handed over to the nongovernmental organisation Human Rights Watch provided much information about Saddams persecution of the Kurds. They detail the arrest and execution in 1983 of 8,000 Kurdish males aged 13 and upwards.

Amnesty International in 1985 drew attention to reports of hundreds more dead and missing, including the disappearance of 300 Kurdish children arrested in Sulaimaniya, of whom some were tortured and three died in custody.

In 1988, Iraqi government forces systematically razed Kurdish villages and killed civilians. Amnesty International estimates that over 100,000 Kurds were killed or disappeared during 1987-1988, in an operation known as the Anfal campaigns...The campaign included the use of chemical weapons. According to Human Rights Watch, a single attack on the Kurdish town of Halabja killed up to 5,000 civilians and injured some 10,000 more."

"The UN Special Rapporteur reports claims by Kurdish opposition sources that 94,000 individuals have been expelled from their homes since 1991."

"More than 100 Shia clerics have disappeared since the 1991 uprising...In early 1999, during a peaceful demonstration in response to the Iraqi regimes murder of the most senior Shia cleric in Iraq, Grand Ayatollah Sayyid Mohammed Sadiq al-Sadr, security forces fired into the crowd of protestors, killing hundreds of civilians, including women and children. Security forces were also involved in efforts to break-up Shia Friday prayers in Baghdad and other cities. Large numbers of Shia were rounded up, imprisoned without trial and tortured. In May 2001, two more Shia clerics were executed in Baghdad for publicly accusing the regime of the Grand Ayatollahs murder.

In response to attacks on government buildings and officials in southern Iraq during 1999, the Iraqi army and militia forces destroyed entire Shia villages in the south.

During the 1990s, Saddam pursued a policy of draining the marshes area of southern Iraq so forcing the population to relocate to urban areas where it was less able to offer assistance to antiregime elements and could be controlled more effectively by the regimes security forces. As an UN Environment Programme report put it The collapse of Marsh Arab society, a distinct indigenous people that has inhabited the marshlands for millennia, adds a human dimension to this environmental disaster. Around 40,000 of the estimated half-million Marsh Arabs are now living in refugee camps in Iran, while the rest are internally displaced within Iraq. A 5,000-year-old culture, heir to the ancient Sumerians and Babylonians, is in serious jeopardy of coming to an abrupt end."

Iraq invaded Kuwait on 2 August 1990. Iraqi forces committed robbery, raped Kuwaitis and expatriates, and carried out summary executions. Amnesty International documented many other abuses during the occupation of Kuwait.

Iraq denied access to the Red Cross, which has a mandate to provide protection and assistance to
civilians affected by international armed conflict.

As Iraq tried to impose its own identity on the occupied territory, Kuwaiti civilians were arrested
for crimes such as wearing beards
. People were dragged from their homes and held in improvised
detention centres. In findings based on a large number of interviews, Amnesty International listed 38 methods of torture used by the Iraqi occupiers, including beatings, breaking of limbs, extracting finger and toenails, inserting bottle necks into the rectum, and subjecting detainees to mock executions.

More than 600 Kuwaiti and third country nationals remain unaccounted for. The British Government believe some were still alive in 1998. Iraq refuses to comply with its UN obligation to account for the missing. It has failed to provide sufficient information to close more than three of the 600 or so files.

In an attempt to deter military action to expel it from Kuwait, the Iraqi regime took hostage several hundred foreign nationals (including children) in Iraq and Kuwait, and prevented thousands more from leaving. Worse still, hostages were held as human shields at a number of strategic military and civilian sites, many in inhumane conditions.

At the end of the Gulf War, the Iraqi army fleeing Kuwait set fire to some 1,160 Kuwaiti oil wells, with serious environmental consequences."

"The following methods of torture have all been reported to international human rights groups, such
as Amnesty International and Human Rights Watch, by the victims of torture or their families.

    Eye gouging: Amnesty International reported the case of a Kurdish businessman in Baghdad who was executed in 1997. When his family retrieved his body, the eyes had been gouged out and the empty eye
    sockets stuffed with paper.

    Piercing of hands with electric drill: A common method of torture for political detainees. Amnesty International reported one victim who then had acid poured into his open wounds.

    Suspension from the ceiling: Victims are blindfolded, stripped and suspended for hours by their wrists, often with their hands tied behind their backs. This causes dislocation of shoulders and tearing of muscles and ligaments.

    Electric shock: A common torture method. Shocks are applied to various parts of the body, including the genitals, ears, tongue and fingers.

    Sexual abuse: Victims, particularly women, have been raped and sexually abused, including reports of broken bottles being forced into the victims anus.

    "Falaqa": Victims are forced to lie face down and are then beaten on the soles of their feet with a cable, often losing consciousness.

    Other physical torture: Extinguishing cigarettes on various parts of the body, extraction of fingernails and toenails and beatings with canes, whips, hose pipes and metal rods are common.

    Mock executions: Victims are told that they are to be executed by firing squad and a mock execution is staged. Victims are hooded and brought before a firing squad, who then fire blank rounds.

    Acid baths: David Scheffer, US Ambassador-at-Large for War Crimes, reported that photographic evidence showed that Iraq had used acid baths during the invasion of Kuwait. Victims were hung by their wrists and gradually lowered into the acid.


There's some facts for you...and this is one of the milder reports. But I guess if Saddam was no threat to us then why bother, right?

Steve

tomder55 answered on 08/16/06:

I got facts ; Saddam waged a 30 year "civil war " of repression on the Shia and the Kurd populations . While that was happening and the death toll rose to countless numbers (a minimum of over 300,000 and more likely exceeding a million) slaughtered in his ethnic cleansings )only a handful of human rights orgs ever gave a rats ass.

What is happening now is militias funded and supported by Iran are vying to be the new Saddam and Baathists in the country . It is a wonder in fact that both the Shia and the Kurds have not rose in and effort to slaughter the Sunni population in retaliation .If not for the cool guidance of Supreme Ayatollah Sistani I believe they would . But instead ;he has agreed that a democratic Iraq is the destiny of the nation.

The ELECTED gvt. of Maliki has a tough go of it to stabilize the country and establish it's authority . That is never an easy task . Even in our own American history it could not be argued that stability came easy . The first gvt. failed completely. When Washington assumed the Presidency of the 2nd gvt. he had to don his military uniform and was prepared to strike at the Shay's who were rebelling in the outer provinces .We had to wage another war against Britain because they still threatened us 30 years after they had been defeated .

When the elected gvt. of Iraq took power tey became our ally in the fight against the jihadists of Tehran and Syria.

ETWolverine rated this answer Excellent or Above Average Answer
Itsdb rated this answer Excellent or Above Average Answer

Question/Answer
ETWolverine asked on 08/15/06 - For those who need to be reminded.

A few days ago, one of this board's regular contributors asked "Why Iraq?"

Well, I decided to go back and look at the justifications for the war in Iraq and see if they still hold up to close scrutiny.

So... here are the justifications used by the Bush administration. I think that every single one of them still holds up to close scrutiny. And Bush's message has not changed one iota since the beginning.

There were 7 main reasons given by the Bush Administration and Bush himself starting in a White House document called "A Decade of Deception". They are:

- Saddam Hussein's Defiance of United Nations Resolutions
- Saddam Hussein's Development of Weapons of Mass Destruction
- Saddam Hussein's Repression of the Iraqi People
- Saddam Hussein's Support for International Terrorism
- Saddam Hussein's Refusal to Account for Gulf War Prisoners
- Saddam Hussein's Refusal to Return Stolen Property
- Saddam Hussein's Efforts to Circumvent Economic Sanctions

Those are the exact same reasons that Bush has used ever since that day. He has not changed his reasons for the war in Iraq at any time.

Here are some quotes from President Bush during various speeches and appearances since then.

Our second goal is to prevent regimes that sponsor terror from threatening America or our friends and allies with weapons of mass destruction. Some of these regimes have been pretty quiet since September the 11th. But we know their true nature. North Korea is a regime arming with missiles and weapons of mass destruction, while starving its citizens.

Iran aggressively pursues these weapons and exports terror, while an unelected few repress the Iranian people's hope for freedom.

Iraq continues to flaunt its hostility toward America and to support terror. The Iraqi regime has plotted to develop anthrax, and nerve gas, and nuclear weapons for over a decade. This is a regime that has already used poison gas to murder thousands of its own citizens -- leaving the bodies of mothers huddled over their dead children. This is a regime that agreed to international inspections -- then kicked out the inspectors. This is a regime that has something to hide from the civilized world.

States like these, and their terrorist allies, constitute an axis of evil, arming to threaten the peace of the world. By seeking weapons of mass destruction, these regimes pose a grave and growing danger. They could provide these arms to terrorists, giving them the means to match their hatred. They could attack our allies or attempt to blackmail the United States. In any of these cases, the price of indifference would be catastrophic.

We will work closely with our coalition to deny terrorists and their state sponsors the materials, technology, and expertise to make and deliver weapons of mass destruction. We will develop and deploy effective missile defenses to protect America and our allies from sudden attack. And all nations should know: America will do what is necessary to ensure our nation's security.


--- The President's State of the Union Address, The United States Capitol, Washington, D.C., January 29, 2002.


I am highly doubtful that he'll [Saddam Hussein] meet our demands. I hope he does, but I'm highly doubtful. The reason I'm doubtful is he's had 11 years to meet the demands. For 11 long years he has basically told the United Nations and the world he doesn't care.

---Remarks by the President in Meeting with Central African Leaders, September 13, 2002.


The Italian Prime Minister joins other concerned world leaders who have called on the world to act. Among them, Prime Minister Blair of Great Britain, Prime Minister Aznar of Spain, President Kwasniewski of Poland. These leaders have reached the same conclusion I have -- that Saddam Hussein has made the case against himself.

He has broken every pledge he made to the United Nations and the world since his invasion of Kuwait was rolled back in 1991. Sixteen times the United Nations Security Council has passed resolutions designed to ensure that Iraq does not pose a threat to international peace and security. Saddam Hussein has violated every one of these 16 resolutions -- not once, but many times.

Saddam Hussein's regime continues to support terrorist groups and to oppress its civilian population. It refuses to account for missing Gulf War personnel, or to end illicit trade outside the U.N.'s oil-for-food program. And although the regime agreed in 1991 to destroy and stop developing all weapons of mass destruction and long-range missiles, it has broken every aspect of this fundamental pledge.


--- Radio Address by the President to the Nation, September 14, 2002


Eleven years ago, as a condition for ending the Persian Gulf War, the Iraqi regime was required to destroy its weapons of mass destruction, to cease all development of such weapons, and to stop all support for terrorist groups. The Iraqi regime has violated all of those obligations. It possesses and produces chemical and biological weapons. It is seeking nuclear weapons. It has given shelter and support to terrorism, and practices terror against its own people. The entire world has witnessed Iraq's eleven-year history of defiance, deception and bad faith.

---Remarks by the President on Iraq, Cincinnati Museum Center - Cincinnati Union Terminal, Cincinnati, Ohio, October 7, 2002


The dictator of Iraq is a student of Stalin, using murder as a tool of terror and control, within his own cabinet, within his own army, and even within his own family.

On Saddam Hussein's orders, opponents have been decapitated, wives and mothers of political opponents have been systematically raped as a method of intimidation, and political prisoners have been forced to watch their own children being tortured.

America believes that all people are entitled to hope and human rights, to the non-negotiable demands of human dignity. People everywhere prefer freedom to slavery; prosperity to squalor; self-government to the rule of terror and torture. America is a friend to the people of Iraq. Our demands are directed only at the regime that enslaves them and threatens us. When these demands are met, the first and greatest benefit will come to Iraqi men, women and children. The oppression of Kurds, Assyrians, Turkomans, Shi'a, Sunnis and others will be lifted. The long captivity of Iraq will end, and an era of new hope will begin.


---Remarks by the President on Iraq, Cincinnati Museum Center - Cincinnati Union Terminal, Cincinnati, Ohio, October 7, 2002


Q Sir, why should we be more worried about Saddam Hussein, who has no nuclear weapons, than Kim Chong-il, who is unstable and does have nuclear weapons?

THE PRESIDENT: Well, first of all, I think it's important to remember that Saddam Hussein was close to having a nuclear weapon. We don't know whether or not he has a nuclear weapon. We do expect him to disarm his weapons of mass destruction, that's what we expect.

Secondly, the international community has been trying to resolve the situation in Iraq through diplomacy for 11 years. And for 11 years, Saddam Hussein has defied the international community. And now we've brought the world together to send a clear signal: we expect him to disarm, to get rid of his weapons of mass destruction. The first step in determining whether or not he will do that was discouraging. His declaration was short. And the international community recognized that, that he wasn't forthcoming.

Again, I hope this Iraq situation will be resolved peacefully. One of my New Year's resolutions is to work to deal with these situations in a way so that they're resolved peacefully. But thus far, it appears that, first look, that Saddam Hussein hasn't heard the message.


---President Discusses Iraq and North Korea with Reporters, The Coffee Station, Crawford, Texas December 31, 2002


Our nation and the world must learn the lessons of the Korean Peninsula and not allow an even greater threat to rise up in Iraq. A brutal dictator, with a history of reckless aggression, with ties to terrorism, with great potential wealth, will not be permitted to dominate a vital region and threaten the United States. (Applause.)

Twelve years ago, Saddam Hussein faced the prospect of being the last casualty in a war he had started and lost. To spare himself, he agreed to disarm of all weapons of mass destruction. For the next 12 years, he systematically violated that agreement.


--- Excerpts from the State of the Union regarding Iraq, January 28, 2003



The dictator of Iraq is not disarming. To the contrary; he is deceiving. From intelligence sources we know, for instance, that thousands of Iraqi security personnel are at work hiding documents and materials from the U.N. inspectors, sanitizing inspection sites and monitoring the inspectors themselves. Iraqi officials accompany the inspectors in order to intimidate witnesses.

Iraq is blocking U-2 surveillance flights requested by the United Nations. Iraqi intelligence officers are posing as the scientists inspectors are supposed to interview. Real scientists have been coached by Iraqi officials on what to say. Intelligence sources indicate that Saddam Hussein has ordered that scientists who cooperate with U.N. inspectors in disarming Iraq will be killed, along with their families.

Year after year, Saddam Hussein has gone to elaborate lengths, spent enormous sums, taken great risks to build and keep weapons of mass destruction. But why? The only possible explanation, the only possible use he could have for those weapons, is to dominate, intimidate, or attack.

With nuclear arms or a full arsenal of chemical and biological weapons, Saddam Hussein could resume his ambitions of conquest in the Middle East and create deadly havoc in that region. And this Congress and the America people must recognize another threat. Evidence from intelligence sources, secret communications, and statements by people now in custody reveal that Saddam Hussein aids and protects terrorists, including members of al Qaeda. Secretly, and without fingerprints, he could provide one of his hidden weapons to terrorists, or help them develop their own.


--- Excerpts from the State of the Union regarding Iraq, January 28, 2003



Iraqi refugees tell us how forced confessions are obtained -- by torturing children while their parents are made to watch. International human rights groups have catalogued other methods used in the torture chambers of Iraq: electric shock, burning with hot irons, dripping acid on the skin, mutilation with electric drills, cutting out tongues, and rape. If this is not evil, then evil has no meaning.

--- Excerpts from the State of the Union regarding Iraq, January 28, 2003


We know that the Iraqis still are not volunteering information. Then when they do, what they are giving is often partial and misleading. We know that when confronted with facts, the Iraqis still are changing their story to explain those facts, but not enough to give us the truth.

---Secretary Powell's Remarks at U.N. Security Council Meeting, March 7, 2003


Iraq's talented people, rich culture, and tremendous potential have been hijacked by Saddam Hussein. His brutal regime has reduced a country with a long and proud history to an international pariah that oppresses its citizens, started two wars of aggression against its neighbors, and still poses a grave threat to the security of its region and the world.

Saddam's defiance of United Nations Security Council resolutions demanding the disarmament of his nuclear, chemical, biological, and long-range missile capacity has led to sanctions on Iraq and has undermined the authority of the U.N. For 12 years, the international community has tried to persuade him to disarm and thereby avoid military conflict, most recently through the unanimous adoption of UNSCR 1441. The responsibility is his. If Saddam refuses even now to cooperate fully with the United Nations, he brings on himself the serious consequences foreseen in UNSCR 1441 and previous resolutions.


---Statement of the Atlantic Summit: A Vision for Iraq and the Iraqi People, March 16, 2003


The dictator of Iraq and his weapons of mass destruction are a threat to the security of free nations. He is a danger to his neighbors. He's a sponsor of terrorism. He's an obstacle to progress in the Middle East. For decades he has been the cruel, cruel oppressor of the Iraq people.

On this very day 15 years ago, Saddam Hussein launched a chemical weapons attack on the Iraqi village of Halabja. With a single order the Iraqi regime killed thousands of men and women and children, without mercy or without shame. Saddam Hussein has proven he is capable of any crime. We must not permit his crimes to reach across the world.

Saddam Hussein has a history of mass murder. He possesses the weapons of mass murder. He agrees -- he agreed to disarm Iraq of these weapons as a condition for ending the Gulf War over a decade ago. The United Nations Security Council, in Resolution 1441, has declared Iraq in material breach of its longstanding obligations, demanding once again Iraq's full and immediate disarmament, and promised serious consequences if the regime refused to comply. That resolution was passed unanimously and its logic is inescapable; the Iraqi regime will disarm itself, or the Iraqi regime will be disarmed by force. And the regime has not disarmed itself.


---Press Availability with President Bush, Prime Minister Blair, President Aznar, and Prime Minister Barroso, The Azores, Portugal, March 16, 2003


Good afternoon. Yesterday, December the 13th, at around 8:30 p.m. Baghdad time, United States military forces captured Saddam Hussein alive. He was found near a farmhouse outside the city of Tikrit, in a swift raid conducted without casualties. And now the former dictator of Iraq will face the justice he denied to millions.

The capture of this man was crucial to the rise of a free Iraq. It marks the end of the road for him, and for all who bullied and killed in his name. For the Baathist holdouts largely responsible for the current violence, there will be no return to the corrupt power and privilege they once held. For the vast majority of Iraqi citizens who wish to live as free men and women, this event brings further assurance that the torture chambers and the secret police are gone forever.

And this afternoon, I have a message for the Iraqi people: You will not have to fear the rule of Saddam Hussein ever again. All Iraqis who take the side of freedom have taken the winning side. The goals of our coalition are the same as your goals -- sovereignty for your country, dignity for your great culture, and for every Iraqi citizen, the opportunity for a better life.


--- Remarks by the President on the Capture of Saddam Hussein, The Cabinet Room, December 14, 2003


The United States and our allies have ended terror regimes in Afghanistan and Iraq. All regimes are on notice that supporting terror is not a viable strategy for the long term.

September 11 made clear our enemies' goals and provided painful experience of how far they are willing to go to achieve them.

We must face our worst nightmare: The possibility of sudden, secret attack by chemical, biological, radiological, or nuclear weapons and the coming together of the terrorist threat with weapons of mass destruction.

Lasting peace and long-term security are only possible through the advance of prosperity, liberty, and human dignity.

The stakes could not be higher. If the Middle East is to leave behind stagnation, tyranny, and violence for export, then freedom must flourish in every corner of the region.


---From remarks by NSA Rice, Louisville, KY, 3/8/04


The fall of Saddam Hussein removed a source of instability and intimidation from the heart of the Middle East. All of Iraq's neighbors, including Jordan, are safer now. And the emergence of a peaceful, prosperous, and free Iraq will contribute to Jordan's security and prosperity.

---Remarks by President Bush and His Majesty King Abdullah II of the Hashemite Kingdom of Jordan in a Press Availability, The Rose Garden, May 6, 2004


To win this war, we are confronting regimes with ties to terror that arm to threaten the peace. We will remove threats before they arrive, instead of waiting for the next attack, the next catastrophe. That is one of the lessons of September the 11th we must never forget. Saddam Hussein's regime posed a threat to the American people, and people around the world. Iraq was a country in which millions of people lived in fear, and many thousands disappeared into mass graves. This was a regime that tortured children in front of their parents. This was a regime that invaded its neighbors. This is a regime that had used chemical weapons before. It had used weapons not only against countries in its neighborhood, but against its own citizens. This is a regime which gave cash rewards to families of suicide bombers. This is a regime that sheltered terrorist groups. This is a regime that hated America.

And so we saw a threat, and it was a real threat. And that's why I went to the United Nations. The administration looked at the intelligence, saw a threat, and remembered the facts and saw a threat. The Congress, members of both political parties, looked at the intelligence. They saw a threat. The members of the United Nations Security Council looked at the intelligence and saw a threat, and voted unanimously to send the message to Mr. Saddam Hussein, disarm or face serious consequences. As usual, he ignored the demands of the free world. So I had a choice to make -- either to trust the word of a madman, or defend America. Given that choice, I will defend America every time.


---Remarks by the President to the Military Personnel, Fort Lewis, Washington, June 18, 2004.


The Iraqi people are emerging from decades of tyranny and oppression. Under the regime of Saddam Hussein, the Shia and Kurds were brutally oppressed, and the vast majority of Sunni Arabs were also denied their basic rights, while senior regime officials enjoyed the privileges of unchecked power. The challenge facing Iraqis today is to put this past behind them, and come together to build a new Iraq that includes all of its people.

---President Addresses Nation, Discusses Iraq, War on Terror, Fort Bragg, North Carolina, June 28, 2005


President Talabani has dedicated his life to the cause of liberty in Iraq. As a lawyer, a journalist, and a political leader in Northern Iraq, he stood up to a brutal dictator, because he believes that every Iraqi deserves the be free. The dictator destroyed Kurdish villages, ordered poison gas attacks on a Kurdish city, and violently repressed other religious and ethnic groups. For President Talibani and his fellow citizens, the day Saddam was removed from power was a day of deliverance. And America will always be proud that we led the armies of liberation.

---President Welcomes President Talabani of Iraq to the White House, The East Room, September 13, 2005


Listen, thank you all for coming. It's been my honor to visit with folks who know firsthand the brutality of Saddam Hussein. These are folks who have suffered, one way or the other, because the tyrant was a law unto himself, and was willing to deny people basic human rights. The stories here are compelling stories. They're stories of sadness and stories of bravery.

In the course of our discussion, we were also able to talk about what a contrast it is between a society which was willing to jail people, torture people and beat people and kill people, to a society that is beginning to understand the fruits of democracy and freedom.


--- President Meets with Victims of Saddam Hussein, Discusses Progress, The Roosevelt Room, January 18, 2006.


The point that I am making is that for all the talk about Bush having changed his reasons for the war in Iraq, or not making it clear why we went into Iraq, I have been able to show you just a few examples where he was extremely clear what we went to Iraq for and why. The message never changed, and the reasons never changed.

The reasons we are in Iraq are because Saddam Hussein was a threat to the USA, who was developing WMDs, who had not complied with the UN for 12 years, who had lied to inspectors about the whereabouts and status of hsi WMDs and long-range weapons, and who oppressed his people brutally. And we are there because Saddam Hussein was a terrorist supporter. And finally, we are there because by fighting the terrorists there, we aren't fighting them here.

Bush didn't lie, he didn't change his messsage, and he didn't waiver. The reasons that we are in Iraq are spelled out clearly, and have been for nearly 4 years.

So for all those who ask "Why Iraq", now you know.

Elliot

tomder55 answered on 08/16/06:

That's right he hasn't wavered at all .That is a fine attribute of great leaders . Lincoln never folded in spite of the difficulties and setbacks and Truman remained determined despite falling poll numbers that guaranteed he would not win reelection if he attempted it .

The big divide here I believe ;and it was recently reinforced by a reply to one of my postings by excon;is those who see Iraq as a single war as opposed to those like me who see Iraq as a theater of a larger war . If a simular question was asked in WWII it might have been asked :why Germany before Japan ? Germany never attacked us .They would've gone further ...why Morrocco ? why Sicily ? etc .

Bush correctly identified 3 members of the 'Axis of Evil ' (I would've added Syria also but as it turns out they follow marching orders from Tehran anyway). Saddam was not contained in spite of the arguments to the contrary . He had to be dealt with and strategically he posed the greatest threat . Once he was removed the threat shifted from him to the other 2 . N. Korea's role has been adequately demonstrated in the last couple of months but the geopolitical realities makes them difficult to deal with at this time . Other major players like Saudi Arabia and Pakistan have for the moment been neutralized and have shown a willingness to cooperate (reluctantly possibly but useful nonetheless). I can guarantee that the naysayers would be saying "why Iran ?" if the decision had been made to confront them first .

ETWolverine rated this answer Excellent or Above Average Answer

Question/Answer
aime-cinema asked on 08/15/06 - Jean Marie Le Pen

If Jean-Marie Le Pen wins the May 2007 French election, what is he planning to do with France?

tomder55 answered on 08/16/06:

I do not think he can win . His effect may be able to skim enough protest votes away from Nicolas Sarkozy unfortunately and as a result the French may elect that buffoon Segolene Royal. {however she does have some attributes to bring to the table }




Who knows about Le Pen ? He may be in jail soon anyway over his comments about the Nazi Occupation of France not being so bad .

But if the race comes down to who looks better in a bathing suit then Royal wins in a run away

ETWolverine rated this answer Excellent or Above Average Answer

Question/Answer
paraclete asked on 08/15/06 - It seems it's a victory for both sides?

Perhaps victory needs the perspective of several thousand miles distance?

Bush: 'Hezbollah suffered a defeat'
President calls Lebanon a front in 'global war on terrorism'

Monday, August 14, 2006; Posted: 8:48 p.m. EDT (00:48 GMT)

President Bush blamed the monthlong conflict on Hezbollah, Iran and Syria.
Image:


WASHINGTON (CNN) -- President Bush declared Lebanon a front in the "global war on terrorism" Monday, equating the Israeli battle against Lebanon's Hezbollah guerrillas to the U.S.-led wars in Afghanistan and Iraq.

Bush said Hezbollah and its supporters in Iran and Syria were responsible for the 34-day war, and called that conflict "part of a broader struggle between freedom and terror." (Watch Bush blame Hezbollah, Iran and Syria for the crisis -- 2:06)

Bush said the U.N. resolution that took effect early Monday was an "important step that will help bring an end to the violence."

He said the conflict was a win for his administration's policy of encouraging democracy in the Middle East and a defeat for Hezbollah, discounting a claim of victory issued by the Shiite Muslim militia's leader earlier Monday. (Watch the president declare Hezbollah the loser -- :40)

"Hezbollah suffered a defeat in this crisis," Bush said during a news conference at the State Department.

"There's going to be a new power in the south of Lebanon," he said, referring to the U.N. force that will assist the Lebanese army in taking control of the area.

"How can you claim victory when you were a state within a state in southern Lebanon, and now you're going to be replaced by an international force?" he said.

Speaking after a day of meetings with Pentagon and State Department officials, Bush said the leaders of armed groups must choose between armed conflict and democracy.

He warned Iran against meddling in both Lebanon and Iraq, where U.S. troops are battling a persistent insurgency and trying to stave off a Sunni-Shiite civil war more than three years after the 2003 invasion.

"In both these countries, Iran is backing armed groups in the hope of stopping democracy from taking hold," he said.

"The message of this administration is clear: America will stay on the offense against al Qaeda. Iran must stop its support for terror," he said. "The leaders of these armed groups must make a choice: If they want to participate in the political life of their countries, they must disarm."

The president defended the U.S. role in settling the Israel-Lebanon conflict, saying Secretary of State Condoleezza Rice deserved "great credit" for the cease-fire agreement.

The administration resisted international calls for an immediate cease-fire, which it said would not have addressed the underlying causes of the conflict.

"We want peace," he said. "We're not interested in process. We want results."

Bush laid the blame for the conflict -- in which more than 1,000 people died -- on Hezbollah, Iran and Syria.

"America recognizes that civilians in Lebanon and Israel have suffered from the current violence, and we recognize that responsibility for this suffering lies with Hezbollah," Bush said.

"Responsibility for the suffering of the Lebanese people also lies with Hezbollah's state sponsors, Iran and Syria."

Bush said Iran "provides Hezbollah with financial support, weapons and training."

"Iran has made clear that it seeks the destruction of Israel," he said. "We can only imagine how much more dangerous this conflict would be if Iran had the nuclear weapon it seeks."

The United States maintains that Iran is pursuing nuclear weapons. The Iranian government says its nuclear program is intended solely for peaceful purposes.

=====================================================

Now to the important question. Since Lebanon is now the front line for the war on terror, will the US be invading anytime soon? or is GWB going to let someoneelse do the dieing this time?

tomder55 answered on 08/15/06:

From what I understand the grand French plan is to have Malaysian troops as the tip of the spear representing the blue helmets .

I do not share the President's spin on the situation . I would love to see the overmatched Lebanese Army assume their proper role but I do not see them asserting themselves even with French and Malaysian support.

Already the Israeli's are intercepting trucks carrying rockets to the terrorists . I'm sure quite a bit of the resupply is making it through since Olmert approved the push to the Litani and the encirclement of the Hezbollah fixed positions too late .

Had they followed the advice of the IDF command then they would've made the push to the Litani earlier ;performed a Canne encirclement ;secured the land routes to Tyre; and effectively cut Hezbollah off from reinforcement . Then they could've picked off their dug in positions piecemeal .

He warned Iran against meddling in both Lebanon and Iraq

The reality is that they have meddled in Iraq and Lebanon from the git-go.


Bush laid the blame for the conflict -- in which more than 1,000 people died -- on Hezbollah, Iran and Syria.

of course he is 100% correct about that .

Regarding your question ;I would not mind it so long as we took the road to Damascus . As far as I am concerned there are 241 Marines and other military personnel's deaths to avenge still . But I have to agree with Elliot. A decapitation of command and control is the way to go and that can be found ultimately in Tehran .

paraclete rated this answer Excellent or Above Average Answer

Question/Answer
jackreade asked on 08/14/06 - What Do You Think of This Assessment?

"In the wake of the thwarted plot to explode bombs on flight from Britain to the U.S., several commentators echoed Pres. Bush's slogan that fighting terrorism is going to be a long war, the defining struggle of this generation. No one mentioned striving for a long peace with the Arab world, which will also take a generation.
The roots of terrorism are not insane. Real conditions gave rise to a huge disaffected segment of Muslims, mostly young and male. From northern Africa across the entire Middle East to Pakistan, there has been a population boom without much hope that any child will receive adequate education, except in the Koran, or adequate work. The governments are either militaristic or dominated by reactionary royal families.

In other words, terrorism grew out of poverty, ignorance, and a sense of hopelessness about the future. Hating America and Israel is also complex, but these other factors made a huge contribution. Therefore, since Islam isn't going to change, and since this new wave of the dispossessed isn't going away, the West has two choices. We can keep supporting the conditions that inflame radical Islam, or we can move positively to bring the Muslim world into a global alliance.

Terrorism is global because national boundaries don't hold back anger and hopelessness anymore. Unlike Communism, terrorism can't be contained. To win a long peace will be difficult because of the deep roots of the problem. But we make it more difficult by being militaristic, by supporting reactionary regimes, by making oil the centerpiece of our foreign policy in the Middle East, by engendering bogeyman hatred of Arabs, and by refusing to see that peace is achievable without annihilating the enemy. The administration's attitude that only total victory is acceptable amounts to saying that every radicalized Muslim male must be killed or neutralized. The war between Israel and Hezbollah is quickly dismantling that fantasy.

The time is dark right now, and a new scare like the airline bombing plot galvanizes fear and anger. But retaliation isn't going to work in the long run. Our only alternative is the long peace, and we have to hope that this idea begins to take root soon, or else we will be trapped in an endless cycle of attack and response just as we are today." Deepak Chopra

~~~~~~~~~~~~~~~~~~~~~~~~~~~~~~~~~~~~~~~~~~~~~~~`


My opinion==Islam is the problem. Islam must voluntarily change first.

What is your opinion?

tomder55 answered on 08/15/06:

Typical of Chopra putting the cart before the horse . They have to be defeated before "striving for a long peace " .

What do I think ? I do not think that religions are the first to change . I think modernization comes to societies first and religions react lest they lose their relevence . The Islamoafascists are mostly anti-modernists and they will beat down their societies rather than let them enter the 21st century. That is why most of the casualties are Muslim on Muslim . They oppose us because unless we assume our roles of the subservient dhimmi they lose . Unless they dominate the world then modernity will overtakes them.

jackreade rated this answer Excellent or Above Average Answer

Question/Answer
jackreade asked on 08/14/06 - "I See Dead People"

Chris Matthews's comment after viewing Joe Lieberman's new political commercial. (It was a real unfortunate piece of work; no way to go out)

~~~~~~~~~~~~~~~~~~~~~~~~~~~~~~~~~~~~~~~~~~~~~

Has anyone else seen this commercial?

tomder55 answered on 08/15/06:

What is unfortunate is that a candidate has to make a justification in making an independent run as if is is something bad . Right now I look for candidates to come up with a good reason to be affiliated with either major party .Perhaps Ned Lamont ;the country club liberal(oh wait ...he quit his elite country club when he declared his candidacy ); should be asked what rational he has in being a Democrat besides pure political opportunism .

I admire him but I will not lament for Lieberman if he loses. Perhaps after 3 terms ;18 years he has grown stale(as are many of the 'bones' in the Senate ). The Conn. Senators will caucus with the Democrats no matter who wins.

ETWolverine rated this answer Excellent or Above Average Answer
jackreade rated this answer Poor or Incomplete Answer

Question/Answer
jackreade asked on 08/13/06 - Sore Loserman Doesn't Get It

From Sunday Talk Show::

"FEINGOLD: Well, I like Joe Lieberman, but I support Ned Lamont, because Joe is showing with that regrettable statement that he doesnt get it. He doesnt get it. The fact is that we were attacked on 9/11 by Al Qaeda and its affiliates and its sympathizers, not by Saddam Hussein. And unfortunately Senator Lieberman has supported the Bush Administrations disastrous strategic approach of getting us stuck in Iraq instead of focusing on those who attacked us. I mean, look at the places that have been attacked: India, Morocco, Turkey, Afghanistan, Indonesia, Somalia, Spain, Great Britain. What does this have to do with Iraq? And Senator Lieberman is stuck on that point. Ned Lamont and I believe that we should refocus on those who attacked us on 9/11 and not simply try to cover our tracks because this was such a very poor decision in terms of the overall battle against the terrorists who attacked us."

~~~~~~~~~~~~~~~~~~~~~~~~~~~~~~~~~~~~~~~~~~~~~~~

Succinct and true.

Comments?

tomder55 answered on 08/14/06:

I think the Dems should purge all their Senators and Reps who voted for the war resolution . 29 Democratic senators and 42 Democrat Representatives voted for the war resolution .This way the take over of the Democrat party by the moonbats will be complete .

jackreade rated this answer Excellent or Above Average Answer

Question/Answer
jackreade asked on 08/13/06 - Bush Caught Lying

From the President's Radio Address, August 12, 2006:

This plot is further evidence that the terrorists we face are sophisticated, and constantly changing their tactics... We're dealing with a new enemy that uses **new means of attack** and new methods to communicate.

From the New York Times, Aug. 12, 2006:

In 1995, a plot to bomb 12 American jumbo jets over the Pacific with a liquid explosive was discovered when the bomb makers accidentally set fire to their laboratory in Manila.

From the New York Times editorial, Aug. 12, 2006 :

The most frightening thing about the foiled plot to use liquid explosives to blow up airplanes over the Atlantic is that both the government and the aviation industry have been aware of the liquid bomb threat for years but have done little to prepare for it.

From the Associated Press:

As the British terror plot was unfolding, the Bush administration quietly tried to take away $6 million that was supposed to be spent this year developing new explosives detection technology....Rep. Martin Sabo, D-Minn., who joined Republicans to block the administration's recent diversion of explosives detection money, said research and development is crucial to thwarting future attacks, and there is bipartisan agreement that Homeland Security has fallen short. ''They clearly have been given lots of resources that they haven't been using,'' Sabo said."


Lie in *****
Comments?

tomder55 answered on 08/14/06:

maybe we should reconvene the 9-11
Commission so we can have some more posturing and posing by the 2nd guessers.

jackreade rated this answer Excellent or Above Average Answer

Question/Answer
jackreade asked on 08/13/06 - Iraq War is the Number One Issue

"The war in Iraq is the No. 1 issue in the country today. Americans are no longer willing to accept the human suffering or the financial toll of a war that has lasted for 3 years with no end in sight.

The numbers speak for themselves. We've lost almost 2,600 Americans.
More than 19,000 have been wounded, 46 percent of them so badly they couldn't return to their units. Tens of thousands of innocent Iraqis have been killed in sectarian violence. The U.S. is spending $11 million every hour - $8 billion every month. Yet the administration refuses to budge from its open-ended, stay-the-course policy. I know it and the American people know it: We need to redeploy our troops to the periphery and refocus on the real war against terrorism. It's long past the time for us to change direction." Murtha

~~~~~~~~~~~~~~~~~~~~~~~~~~~~~~~~~~~~~~~~~~~~~~~

Can't argue with this, can we Sen Loserman?

tomder55 answered on 08/14/06:

Murtha is beginning to sound like a broken record .

His opponent released this response :

TRANSLATING MURTHA

Wednesday, August 09, 2006

(MONONGAHELA, August 9) -- Washington County Commissioner and Pennsylvania 12th district Republican Congressional nominee Diana Irey -- responding to today's post by Jack Murtha on the left-wing Hollywood blog "Huffington Post" -- today released the following statement:

"Today, Jack Murtha further confirmed that he has turned his back on his constituents in southwestern Pennsylvania, in favor of his liberal Democratic allies in San Francisco and Hollywood. His post on the Hollywood blog 'Huffington Post' sounds so reasonable -- unless, that is, you understand how to translate 'Murthaspeak' into 'English.'

"Allow me to demonstrate:

"When Jack Murtha says, 'The war in Iraq is the No. 1 issue in the country today. Americans are no longer willing to accept the human suffering or the financial toll of a war that has lasted for 3 years with no end in sight,' what he means is 'Defending ourselves from terrorists is the number one issue in the country today, but I can't say that out loud - because, given my outrageous comments and my demand for immediate withdrawal/surrender, my constituents might just redeploy me out of the Congress. So instead, I'll use the phrase 'war in Iraq,' because after a time, EVERY war becomes somewhat unpopular.

"When Jack Murtha says, 'The numbers speak for themselves,' what he means is, 'But letting numbers speak for themselves doesn't serve my political agenda, so let me recount them for you again -- but I'll make sure to include only the numbers that will depress you: body counts and casualties. I won't say a word about the 25 million Iraqis who now enjoy freedom they never knew before, and I won't say a word about the millions of children who are now going to school in Iraq, and I won't say a word about the millions of Iraqis who are working hard every day to establish a vibrant democracy in the heart of the Middle East.'

"When Jack Murtha says, 'Yet the administration refuses to budge from its open-ended, stay-the-course policy,' what he means is, 'Darn it, I wish I had the President's intestinal fortitude -- when this guy makes up his mind to accomplish something important, he won't let anything stand in his way -- totally unlike me. I mean, I pushed Ronald Reagan to withdraw U.S. troops from Beirut the moment the going got tough, and I pushed Bill Clinton to withdraw U.S. troops from Somalia the moment the going got tough. And I know all we got in return for our withdrawal was a "Why, thank you, stupid Americans, now we'll go kill some more of your soldiers" from al-Qaeda, but, darn it, how was I to know?'

"And when Jack Murtha says, 'I know it and the American people know it: We need to redeploy our troops to the periphery and refocus on the real war against terrorism. It's long past the time for us to change direction,' what he really means is, 'I hope to goodness no one remembers how I stuck my foot in my mouth on "Meet the Press" when I said we could redeploy to Okinawa. Maybe if I keep saying "real war on terrorism" loud enough and long enough, they'll overlook the fact that I'm really calling for America to raise the white flag.'"



Paid for by Diana Irey for Congress

www.irey.com








GO DIANA !!!

jackreade rated this answer Excellent or Above Average Answer

Question/Answer
jackreade asked on 08/13/06 - What Do You Make of This?

LONDON - "NBC News has learned that U.S. and British authorities had a significant disagreement over when to move in on the suspects in the alleged plot to bring down trans-Atlantic airliners bound for the United States.

A senior British official knowledgeable about the case said British police were planning to continue to run surveillance for at least another week to try to obtain more evidence, while American officials pressured them to arrest the suspects sooner. The official spoke on condition of anonymity due to the sensitivity of the case.

In contrast to previous reports, the official suggested an attack was not imminent, saying the suspects had not yet purchased any airline tickets. In fact, some did not even have passports.


The source did say, however, that police believe one U.K.-based suspect was ready to conduct a "dry run." British authorities had wanted to let him go forward with part of the plan, but the Americans balked.

At the White House, a top aide to President Bush denied the account.

"There was unprecedented cooperation and coordination between the U.S., the U.K. and Pakistani officials throughout the case," said Frances Townsend, Assistant to the President for Homeland Security and Counterterrorism, "and we worked together to protect our citizens from harm while ensuring that we gathered as much info as possible to bring the plotters to justice. There was no disagreement between U.S. and U.K. officials."

Another U.S. official, however, acknowledges there was disagreement over timing. Analysts say that in recent years, American security officials have become edgier than the British in such cases because of missed opportunities leading up to 9/11.

Aside from the timing issue, there was excellent cooperation between the British and the Americans, officials told NBC....."

~~~~~~~~~~~~~~~~~~~~~~~~~~~~~~~~~~~~~~~~~~~~~~~~

tomder55 answered on 08/14/06:

If this is true then I can only speculate . Perhaps they were fearful of the 4th estate taking on their role of 5th column and leaking the details before arrests were made .

From Meet the Press yesterday David Gregory interviewing Michael Chertoff "

GREGORY: Let me ask you about our coordination with British authorities. What worked here?

CHERTOFF: Well, what worked is deep relationships; trust -- the fact that we did not have leaks prior to the takedown. You know, that's one of the critical lessons out of this whole thing. The British trusted us with very sensitive information, and they were able to do it because they were confident we weren't going to leak. That's why leaks are so pernicious. Not only do they actually reveal secrets, but they undercut the basis of trust, which is the foundation of our whole international effort.


George Stephanopoulos however speculated that this report is a fantasy of the blogsphere in an exchange with George Will."I'm sure somewhere out in that fog of paranoia we call the blogosphere, there are all kinds of people writing that the British police ... and the Bush administration timed this all to counter" the political fallout from Sen. Joseph Lieberman's loss in the August 8 Connecticut Democratic primary to anti-Iraq war candidate Ned Lamont. Stephanopoulos responded, "[T]here certainly is," and noted, "There was one report that said the United States ... rushed this -- the Brits wanted to wait." But Stephanopoulos failed to mention that this report had come from NBC News, not the blogsphere.


Another U.S. official, however, acknowledges there was disagreement over timing. Analysts say that in recent years, American security officials have become edgier than the British in such cases because of missed opportunities leading up to 9/11.

There is certainly some validity to this reasoning . Imagine the reaction had we acted too late .

Apparently the Brits were skittish about arresting the ring leader Rashid Rauf and did not until the US pressured them by threatening to "render " him .

Another interesting point to consider is that the Brits enjoy fewer civil liberties than Americans enjoy. British officials can wiretap without a court order. Under Blair, the British government extended police powers so that authorities can arrest and detain terrorism suspects for up to 28 days -(Blair had asked for 90 days ) without having to press charges. One wonders how the plot would've been uncovered without the less restrictive intel. gathering techniques that the Paki's and the Brits use . Simular methods would probably set the perps free here .

Itsdb rated this answer Excellent or Above Average Answer
jackreade rated this answer Excellent or Above Average Answer

Question/Answer
paraclete asked on 08/14/06 - At Last ! Someone has it right?

Face up to your problem, Muslims told
Annabel Crabb in London and agencies
August 14, 2006

LONDON'S most influential former police chief has rounded on Britain's Muslims, blaming them for the terrorist networks in the country.

"When will the Muslim community in this country accept an absolute, undeniable, total truth: that Islamic terrorism is their problem?" wrote John Stevens, former commissioner of the Metropolitan Police, in a Sunday newspaper.

In an inflammatory opinion column, he called on Muslims to "stop the denial, endless fudging and constant wailing that somehow it is everyone else's problem and, if Islamic terrorism exists at all, they are somehow the main victims".

Lord Stevens, whose continuing responsibilities in Britain include the inquiry into Princess Diana's death, also defended "racial profiling" at airports and other security hotspots, saying resources were being wasted on searching everybody out of a sense of fairness or delicacy.

"I'm a white, 62-year-old, suit-wearing ex-cop - I fly often, but do I really fit the profile of a suicide bomber?" Lord Stevens, who was commissioner of the Metropolitan Police until last year, wrote in the News of the World.

His comments clash noticeably with a speech delivered last week - before the terrorist arrests - by the Assistant Police Commissioner, Tarique Ghaffur, who said racial profiling methods had "discriminated" against the Muslim community and added to racial tensions in Britain.

Lord Stevens's intervention is also certain to increase fears among the leaders of Britain's Muslims, a group of whom wrote an open letter to the Prime Minister, Tony Blair, on Saturday imploring him to reduce tensions by altering his political stance on the Middle East.

"Of course, there'll be instant squealings that this is racism. It's not," Lord Stevens insisted.

"It's exactly the same as recognising that, during the Northern Ireland troubles that left thousands dead, the IRA were totally based in the Catholic community and the UVF in the Protestant."

Other reports yesterday, meanwhile, said Britain's "leader" of al-Qaeda had been captured in last week's raids. The unnamed man, who was one of the 24 people seized under anti-terrorism laws last Thursday, was described by The Sunday Times as "suspected not only of masterminding the foiled plot to bring down up to nine trans-Atlantic airliners, but also of involvement in other planned atrocities over the past few years".

Pakistani police allege that Rashid Rauf, a 26-year-old who they arrested last week, was acting as an agent there for the British-based terrorist network, organising cash support and training participants in the plan to blow up aircraft flying to the US in mid-air.

British intelligence and security agencies have been loath to claim a direct al-Qaeda involvement, although the US Secretary of Homeland Security, Michael Chertoff, readily expressed an opinion on the day of the arrests last week that the alleged plot bore the terrorist group's "hallmarks".

The Daily Telegraph in London reported that five of the suspects had learned bomb-making techniques in al-Qaeda training camps in Pakistan, and that they had recorded "martyrdom videos", which were to be released by al-Qaeda in the immediate aftermath of the attacks.

Pakistani officials quoted by the newspaper said some of the suspects visited the region at the same time as the London bombers Mohammed Sidique Khan and Shehzad Tanweer.

The US President, George Bush, in a scheduled address on Saturday told the American people that proponents of the foiled bombing plot shared a "totalitarian ideology" with Lebanon's Hezbollah.

Twenty-two suspects - chiefly young, British-born men of Pakistani descent - are still being questioned in British police stations. One has been released without charge, while the 24th is due for a hearing on Monday to decide whether police can continue their questioning.

As security precautions continued to cause hold-ups at London's Heathrow Airport, British Airways and the low-cost operator Ryanair said airport authorities were failing to cope with the crisis.

The chief executive of British Airways, Willie Walsh, said the airline had cancelled a quarter of its short-distance flights, and some planes had been taking off half-empty because passengers were being held up in queues for security searches.

=====================================================

Glad to see that someone is willing to put political correctness aside and identify the current wave of terrorism as a Muslim problem

tomder55 answered on 08/14/06:

Bush took baby steps in that direction last week by calling them Islamic fascists . The American Muslim community was of course outraged . The Council on American-Islamic Relations (CAIR) chairman, Parvez Ahmed, has written President Bush complaining that his comment about Islamic fascists "contributes to a rising level of hostility to Islam and the American-Muslim community."

Actually,Bush is doing just the opposite. By recognizing the totalitarian Islamist political movement that is seeking to subject the world's population to its ideology, the President is finally clarifying that we are not fighting "terrorism." We are fighting identifiable enemies who adhere to literal interpretations of Jihad as war against non-believers .

Muslims are now givin a choice to believe in this radical philosophy or to oppose it which is John Stevens point also . His words bring clarity to this war .

Itsdb rated this answer Excellent or Above Average Answer
paraclete rated this answer Excellent or Above Average Answer

Question/Answer
paraclete asked on 08/14/06 - Here's a novel solution?

Everyone wants an easy out and a solution to the current terrorist success in disrupting world air traffic. Well, here is the solution. Saves a lot of time. No Muslims allowed on aircraft. So all that has to be done is for passengers to be accredited by the local church, synagogue, temple and so forth, no accreditaion, no flight, it's money making and think how it will solve the problem of low attendence. A true win-win situation. There may even be a few conversions from Islam to more reasonable religions.

For those who say it's too harsh, Muslims can only travel on Muslim owned airlines. If they want to bomb those they will just be killing their own. Nothing new in that.

tomder55 answered on 08/14/06:

I offered up that possibility in my response to excon's posting about the naked ailrline.

I think it is interesting that Western travelers are howling protest over the new restrictions . Israels El Al has had tough passanger screening for years .

Itsdb rated this answer Excellent or Above Average Answer
paraclete rated this answer Excellent or Above Average Answer

Question/Answer
HANK1 asked on 08/11/06 - AMAZING GRACE:



It's very hard to look into the soul of a person but perhaps it's much easier to appraise the thinking of a barbarian. I just heard over the news that Israel has AGREED to the cease fire terms laid down by the United Nations. It's 5:40 p.m. 8/11/06 where I live. I'll predict that the Hezbollah will NOT agree to the cease fire and continue to do what they do best -- kill, kill kill. Tomorrow just might be a very hard day for our Israeli friends.

HANK

tomder55 answered on 08/12/06:

Have to agree with you there .Because it is in Hezbollah's best interest to accept the cease fire to buy time to rearm before they attack Israel again I have no doubt the dopes will not agree to it .This would fall right into the Muslim deception called hudna .

"if Muslims are weak, a truce may be made for ten years if necessary, for the Prophet (Allah bless him and give him peace) made a truce with the Quraysh for that long, as is related by Abu Dawud" ('Umdat as-Salik, o9.16).

Mohammed used the time to strengthen his forces and then defeated the Quraysh .


What I do not understand is why Olmert is so willing to agree to this . You will notice thar somehow the impass at the UN was only resolved after the Israeli's announced and mobilized for a major offensive that would've cut off the terrorists from their supply of rockets and hit them hard in the Bekka Valley .Suddenly the French found new motivation to settle.

Secretary Rice said to Wolf Blitzer about the International Force: "it has a mandate that will allow it to defend itself and to defend that mandate. But it's never been the expectation that this force is going to disarm Hezbollah. That will have to be done by the Lebanese."

I thought the Israeli goal and the UN's for that matter was the disarming of the terrorists . Why would they accept less given the fact that Hezbollah blitzed Israel with over 3,000 rockets in the last month ? The Lebanese have neither the means or apparently the desire to disarm them .

Sure would make me sleep good at night knowing that the French were there to prevent Hezbollah from launching rockets at my house.

I expect when all is said and done there will be a vote of no confidence that will remove Olmert and Netanyahu will become PM.

HANK1 rated this answer Excellent or Above Average Answer

Question/Answer
excon asked on 08/11/06 - Multi-National Force to protect whom???

Hello:

If Israel accepts UN sponsored peace keepers on its northern border, its my bet that the intent of the force will be to prevent Israel from defending themselves rather than preventing Hezbollah from re-arming.

WHEN push comes to shove, this peace keeping force, will, in my view, aim its guns south. The placement of such a force, will be the first loss for the IDF.

Therefore, I suggest that Israel has painted itself into a corner. A corner not too different than the corner we find ourselves in, in Iraq. IF Israel would have invaded Southern Lebanon with everything they had, they would have won in 10 days. They didnt, and theyll lose.

Just like we didnt in Iraq, and are losing. The problem is that Iran is going to be the victor, and that spells very big trouble for the entire world.

How did things go so wrong?

excon

tomder55 answered on 08/11/06:

I think that they could've run this operation better . I think Olmert is still trying to preserve his ante-bellum domestic agenda and has not yet realized that the minute the soldiers were kidnapped by Hamas and Hezbollah all the political calculations changed .

I also think they fell in love with the Clintonoid formula for warfare ,rely heavily on air power to compel the enemy into submission. (see the change in command that has brought about a change in tactics )It may have backfired on them somewhat although this battle is still only a month old ;ages compared to the lightning victories we are used to by the Israelis but not unusual in combat with a month being necessary to prepare the battle field .I do not see why a time table need be set .

I have no confidence in a peace keeping force either and Hezbollah's survival will be perceived as a victory ;much like how Arafat shot guns in the air celebrating his survival when he was cornered in Beirut and allowed to go into exile. Strange world indeed .

My bigger problem is the growing disagreement inside the White House over the future course .It all started I believe with the ill-advised trip by Condi into the region. Since then I think she has some idea that she can craft a regional grand- compromise ala Henry Kissinger . This would involve Syria and Iran because she believes she can tie this into a deal with Iran over their nukes. Seems she is having a hard time shedding her realpolitik (accomodation for temporary stability )roots .

excon rated this answer Excellent or Above Average Answer
Itsdb rated this answer Excellent or Above Average Answer

Question/Answer
excon asked on 08/10/06 - Waddya think?


Hello:

I'm gonna start a new airline. It's gonna be called Naked Airlines. Our motto is gonna be: No clothes - No bombs.

I'm sellin stock. Any takers?

excon

tomder55 answered on 08/11/06:

I'm actually a little encouraged by this . Think about it ;it is from the same script that the jihadists have been trying since the mid 90s (Project Bojinka ). They pulled it off once on 9-11 but basically the target and the methods have not changed much ....blow up planes ...target Washington DC and NY . We know what to expect and it is not likely that the passengers will allow the Richard Reid's of the world to light a match to their shoes anymore .No one will ever take a plane again using a box cutter .

Another thing positive is that the cell was tracked and broken before the event which means that terror cells are increasingly finding it difficult to operate in the West below the radar and their ability to successfully stage attacks has been severely degraded .

If someone wants to make a lot of money, they should start operating a no Muslims allowed airline.Only alcoholic beverages allowed !! (nah that wouldn't work ..the devout hijackers of 9-11 parties in Las Vegas before the attack)


Thanks but most people I do not wish to see naked ;but if everyone was issues airline supplied pajamas we could probably fly in comfort .Besides you could carry c-4 in your body as easily as a mule carries coke .Are you gonna add the deep cavity search as a flight bonus ?

excon rated this answer Excellent or Above Average Answer
Itsdb rated this answer Excellent or Above Average Answer

Question/Answer
jackreade asked on 08/10/06 - Sore Loserman

An incumbent, well-liked Senator and Bush war supporter cannot even get re-nominated in his party's primary election, and the interest was so high in voting against the war by voting against Loserman, that the turnout was higher than previous turnouts for presidential-senatorial elections! Oh, oh.

Crucial election only 3 minths away. Oh, oh.

tomder55 answered on 08/11/06:

it would've been interesting to see how the contest would've turned had the news of the Brit breaking up the terror plot had come a week earlier .

I'll admit what happened is a rarity . It happened in NY when a well respected Senator Jacob Javitz lost his seat to Al (Senator Pothole ) DAmato. In that case Javitz was somehow labeled as out of touch but it was forces inside the State that dethroned him .

In this case you had a motivated minority in the Democrat Party (mostly from outside the State ) that is waging their version of a putsch (although with all their braggadiccio it is hardly a secret ).They found their useful idiot in the name of Ned Lamont ;a person who they would otherwise consider irrelevent except for the fact that he is rich enough to fund his own campaign and he tows their version of the party line . I think he will go down big time when he has to defend his position in front of the wider Connecticut electorate .

It is sad to see were the Democrat party is headed toward but it justifies my decision to leave the party. As an opponent of the party I view with satisfaction their carnivorous ways but I do not think it is good for the country that so early in the conflict against jihadistan there is no unity of purpose . Lieberman is one of the exceptions and it would be a sad day if he were defeated .

Itsdb rated this answer Excellent or Above Average Answer
jackreade rated this answer Excellent or Above Average Answer

Question/Answer
HANK1 asked on 08/10/06 - Good Goin', Brits:



Foiled Plot Brings New Security, Delays
LONDON (AP) - British authorities said Thursday they thwarted a terrorist plot to simultaneously blow up several aircraft heading to the U.S. using explosives smuggled in carry-on luggage.

My favorite people has always been the English. They never let me down. Allow me to express my appreciation.

HANK

tomder55 answered on 08/10/06:

with all due respect to our allies across the pond; if they were not so tolerant of radical Islamists who run rampant in London then there would not be so many plots to discover and take down .groups that are not tolerated in many Muslim capitals set up shop in London and run their operations out of the city in the open . They run recruting and propoganda media freely there .

suggested reading Londonistan by Melanie Phillips demonstrates that the appeasment of Neville Chamberlain is alive and well in the British gvt. Phillips cites what she calls Britains dirty little secret: During the 1990s, Islamist radicals were given free rein in Britain as part of a gentlemens agreement. This agreement said that if the Brits left them alone, the Islamists would not attack their hosts. This turned Britain into the hub of al-Qaeda in Europe.Thirteen percent of British Muslims regard the July 7 bombers as martyrs, and between seven and 16 percent think suicide attacks on British targets can be justified.

The Brits like us fall prey to phony multi-culturalism and political correctness . Once Tony Blair leave I doubt the level of cooperation that we have shared in the war against jihadistan will be there.

HANK1 rated this answer Excellent or Above Average Answer
Itsdb rated this answer Excellent or Above Average Answer
jackreade rated this answer Excellent or Above Average Answer

Question/Answer
Itsdb asked on 08/10/06 - Once again, France is being...French

If you'll remember it was just a few days ago that "The United States and France reached agreement Saturday on a draft resolution that calls for an end to the fighting in Lebanon and the eventual deployment of a U.N.-mandated peacekeeping force."

By now I'm sure you've heard that France reneged on this agreement. Now they're considering their own plan:

    Speaking in Toulon on Tuesday, French president Jacques Chirac said he still hopes the US will back an initial US-France draft.

    If we dont manage it, there will obviously be a debate in the security council, Chirac told reporters.

    Everyone will present their position clearly, including, of course, France with its own resolution.

    Chiracs comments reflect widespread frustration at the UNs failure to overcome deadlock following France and Americas draft ceasefire resolution last weekend.

    Following Lebanese and Arab league objections to the draft, Paris is calling for changes to appease Arab concerns.


Anyone have Chirac's address so I can send him a gift?

tomder55 answered on 08/10/06:

It appears that the security council now consists of 5 permanent members ;10 elected members ;and the 22 nations of the Arab League .I just did not realize they had veto power before .

Chirac ? I'm sure he has visions of burning cars in his dreams .Dhimmi is as dhimmi does .

Jacques Chirac Prsident de la Rpublique franaise

lyse Palace 55, rue du Faubourg Saint Honor

Paris; France

but these days he may be at the summer residence at Fort de Breganon, in southeastern France.

Itsdb rated this answer Excellent or Above Average Answer

Question/Answer
ETWolverine asked on 08/09/06 - Leiberman Losses Primary

Joe Lieberman has lost the Democratic primary in CT. He ha vowed to un as an independant, as according to AP, he has already filed to run as an independant in the November election. The final returns from yesterday's primary shows Lamont beating Lieberman 52%-48%.

My questions:

1) Does a 52%-48% defeat of Lieberman count as a "referendum on the war in Iraq"? Even among Democrats (Republicans didn't vote in the Lieberman v. Lamont primary)?

2) Does the fact that Leiberman was able to come back from a 13-point deficit (in Gallup polls) in the past week mean anything?

3) Will Republican Alan Schlessinger be a factor in the race? He has been trailing both Lieberman and Lamont in the polls.

4) Is there a chance that Schlessinger will win because of a split-vote between Lamont and Lieberman?

5) Running as an Independant, does Lieberman have a shot at keeping his seat in the Senate? Can he garner enough Republican votes to beat Lamont's Democrat majority?

Elliot

tomder55 answered on 08/10/06:

1)not really ,when all is said and done ,only 7% of the Conn. voters came out . The fact that the Kossaks and the moveon crowd invested so much into the race ,and the spread was only 4 points is telling . Let the moonbats make Iraq the issue ;it may help them in the short term this year but they will lose the 2008 contest badly if they try to replay 1972 .

2)it is very significant because as I said ,the tin foil hat crowd put all their eggs in this race and given another week Lieberman would've prevailed.

btw. From what I understand the Kossaks are looking on Ebay to buy up some of those old Sore Loserman posters from 2000 but had Lieberman won it I'm sure they would've looked for a recount and a court challenge of the results .

3) Not at all. I think most Republicans will cross over and support Lieberman because Lamont is a bumpkin and would be an embarrasment for the State in the Senate .Schlessinger reminds me of what's his face running against Hillary (oh yeah ...former Yonkers Mayor John Spencer ...right...actually right now I am supporting and like "KT" McFarland alot ,and hope she can at least give Evita a fight of it )

Lieberman still holds some prestigious positions in the minority and unless the Kossaks get their way and pressure Harry Reid to expel him from the important committees .(I went to the Daily Kos blog site yesterday and found a posting advocating just that )then Lieberman will still be an influential Senator and one of the few left who can reach out across the chasm of political differences in this country

4) Not a chance.

5) I think he can pull it off . The money is a major handicap however .I think secretly the word is going out for Republicans to fund Lieberman unofficially but you know he is not going to get a dime from Schmuck Shumer . There will be organized Republican and Democrats for Lieberman but the Dems who do so will be taking great risks in doing so . I think Lamont has a small base and will not rise much above it but will get the dopes who vote party line across the board . Lieberman on the other hand can concentrate on the independent voters ;by far the largest voting block in the state .

ETWolverine rated this answer Excellent or Above Average Answer
Itsdb rated this answer Excellent or Above Average Answer

Question/Answer
Itsdb asked on 08/07/06 - Who'll be first?

So Mel Gibson got drunk and said some stupid anti-Semitic things and the media and left-wing bloggers had a field day with it. Last week the Huffington Post had column after column blistering Mel.

Madonna stages a mock crucifixion - in Rome - after the Vatican condemned warned her, "To crucify herself during the concert in the city of Popes and martyrs is an act of open hostility."

Who'll be the first to take Madonna to task for her pre-meditated hostility? Any of you Mel critics here or out there in the liberal media and blogosphere willing to take a shot at Madonna as well?

Steve

tomder55 answered on 08/09/06:

Did you see the Lanny Davis editorial in yesterday's Wall Street Journal ?

Here you have a Clintonista having an epiphany after doing some campaigning for Lieberman and realizing that "The far right does not have a monopoly on bigotry and hatred and sanctimony.

He quotes a poster at Huffington who's comments would've elicited a howl of protest if a right winger would've said them ...Good men, Daniel Webster and Faust would attest, sell their souls to the Devil. Is selling your soul to a god any worse? Leiberman cannot escape the religious bond he represents. Hell, his wife's name is Haggadah or Muffeletta or Diaspora or something you eat at Passover .

Note that the Huff and Puffers were slow to attack one of their own when making sober anti-semetic postings but the rants of a drunk are condemnable .

Ned Lamont shared his victory podium last night with Jesse (hymetown) Jackson and Al (Jews are "diamond merchants" )Sharpton so the face of antisemitism also wears dem tin-foil hats .




Itsdb rated this answer Excellent or Above Average Answer

Question/Answer
excon asked on 08/09/06 - Iraqi civil war


Hello pundits:

Help me out here. Lets see, al-Qaida is Sunni. Hezbollah is Shiite. They hate each other. For whatever reason, in Iraq Sunnis have made it their mission to slaughter Shiites, and Shiite death squads are returning fire.

And, just exactly what is wrong with letting these two enemies of ours destroy each other?

excon

tomder55 answered on 08/09/06:

I see nothing wrong with it if that were the case . As an example ;after WWII there was some pretty vicious revenge going on in France against those who were collaborators and members of the Vichy gvt. Our troops basically stood by and let it happen .

However we are not there to mediate sectarian tensions as your question implies . We are there to help provide security for the elected Gvt. of Iraq until such time that they can do it on their own.

There is an elected gvt. and there are foreign infiltrators, and militias with fascist ambitions, who don't like that fact, and are thus waging terror campaigns to destabilize the government. They incite and exploit sectarian tensions .Without them ,this whole "civil war" nonsense goes away . Strip away the terrorist's PR the legacy media ,and the Democrat's talking points and what you are left with is democratic elected govt. vs terrorism. We support the gvt.

excon rated this answer Excellent or Above Average Answer
MicroGlyphics rated this answer Excellent or Above Average Answer

Question/Answer
jackreade asked on 08/07/06 - 񓟠"

"America leads the world in advertising techniques, and now we'll need every ounce of Madison Avenue's skill to sell a difficult product. That product is victory. From the beginning we were told that victory was the only acceptable outcome in Iraq, and now selling that message has become twice as difficult in Lebanon.

Insurgents an terrorists aren't giving up. The Islamic world celebrates their existence. At this moment the most popular figure among Muslims everywhere is Sheik Hassan Nasrallah, the defiant Hezbollah leader who stands as tall as Osama bin Laden and has proved just as indestructible.

The reality of victory isn't even the point anymore. President Bush, in his surprise visit to Baghdad six weeks ago, announced a new plan to secure people's safety in Baghdad. He showed his total support for the al-Maliki government, which pledged to end the terror of Shi'ite militias on the street. A program of amnesty and national unification made headlines.

None of it has happened. But instead of saying so, which would be realistic, the image has to shift. The sales job needs tweaking. It's sad when image can't match reality. But isn't that the point in all wars? The home front must be sold the inevitability of victory and the impossibility of defeat. War-makers are frighteningly willing to sacrifice civilian lives while fiercely defending their own posturing. Thus Israel, with our backing, proclaimed that its Lebanon campaign could only end in the total destruction and disarming of Hezbollah. From the beginning some voices said this goal was impossible, and so it is proving. The image of victory was duly modified to lesser goals as things began to go contrary to plan. Israel next wanted a 15-mile safe zone in southern Lebanon, then a one-mile zone, then an international peacekeeping force. The reality is that there's nothing left to sell but the illusion that they will win.

Wars are places where illusions go to die. A great many died after the fall of Saigon in the Vietnam fiasco, but after thirty years a new crop sprouted again. Installing democracy by force in Iraq is an illusion; deep sectarian hatred is the reality. A government of national unity is an illusion; the U.S. putting a Shi'ite sectarian president in power is the reality. Iraqi security forces are an illusion; armed thugs dressed in police uniforms to make it easier to kidnap and slaughter innocent people is a reality.

These days I think I'm like most people, exhausted from criticizing the Bush war policy. All I really want now is an honest admission, first to all Americans and then to the world, that we've stirred up far more than we can handle. Let's stop fighting over WMDs and distorted evidence and yellow cake uranium. A real crisis faces the world on an order of magnitude no one ever anticipated. Every demon has flown out of Pandora's box, and trying to market the illusion that we're winning feels like a page from George Orwell. The citizens in 񓟠" were trapped in a world where war never ended, yet they woke up every morning to the cheerful news of impending victory that was just around the corner." Deepak Chopra

~~~~~~~~~~~~~~~~~~~~~~~~~~~~~~~~~~~~~~~~~~

Have all you right wing guys finally come to your senses?? :)

tomder55 answered on 08/08/06:

Typical defeatism .Chopra can talk about advertising and selling to the American people but the truth is that he would never be convinced .

Here is my take on the recent developments ....Iran is behind both the Hezbollah attack on Israel and the upsurge in violence in Baghdad .

Iran was so afraid of the direction that the ME has turned with the Cedar Revolution in Lebanon and the Iraqi elections and seating of a unity gvt. that it accelerated it's own timetable and has now intitiated hostilities ahead of it's chosen timetable.

Optimally they would've waited until they had a nuke in hand but events in the ummah were threatening to overtake them . Strong secure nations in Iraq and Lebanon are not in the mad mullahs interests so in both cases they have moved to stir up sectarian differences . The violence in both countries(Lebanon and Iraq) weakens the unity that is emerging.

There will be no end to this until a decisive confrontation with the Iran of the Madhi-hatter . When they get nukes it will be too late and I do not now think that there is time to wait for popular revolution in the country . Part of the calculation in their aggression is that Bush is weakened ;that the US midterm elections will further weaken him ;the Olmert does not have the resolve of a Sharon ;and the the West has the spine of a jelly fish . With talk of half measures by the like of Chopra Ahmadinejad may have a point.

.... we've stirred up far more than we can handle ...We stirred up nothing . Chopra's delusion is that by doing nothing everything was kumbaya before .Nothing could be farthur from the truth.

Itsdb rated this answer Excellent or Above Average Answer
jackreade rated this answer Excellent or Above Average Answer

Question/Answer
excon asked on 08/07/06 - Iraq and the War on Terror


Dear Neocons:

You say the wars are one in the same, and you say we're winning. Here's why they're not, and here's why we're not winning.

When Prime Minister Nouri al-Maliki addressed Congress last month, he declared Iraq to be on "the front line" of the war on terror, and proclaimed Iraqis to be America's "allies in the war on terror." But he also pointedly failed to condemn Hezbollah terrorism or, it seems safe to presume, to consider Hezbollah a terrorist group. Can the United States and Mr. al-Maliki really be talking about the same "terror" war?

Neo-cons never ask such a question, maybe because it leads to this one: Does propping up in Iraq what amounts to a proto-Shariah state that is reflexively anti-Israel if not reflexively pro-Hezbollah constitute victory in the "war on terror"? Call me crazy, but I don't think so.

excon

tomder55 answered on 08/08/06:

and the alternative was leaving a dictator in power who was paying families of homicide bombers to strap on vests full of explosives to kill Israelis ;who launched an unprovoked barrage of scud missiles into Israel in 1990 .

al-Maliki has shown great courage even to align himself with the US . While he was away in July a Coup plot was uncovered (which is of course a traditional Batthist method when they don't like the staus quo). Give him some slack . We cannot expect him to toe the line and give the same suport to Israel that we do . I'd be happy enough for Arab nations to hate Israel and leave her alone .

excon rated this answer Excellent or Above Average Answer

Question/Answer
paraclete asked on 08/07/06 - Has a solution to the fuel crisis already been found?

This points to an interesting but little known fact, trials are underway for a radical solution to the fuel crisis, yet few know about it. Why?

Tuckey's fuel solution: hydrogen
August 7, 2006 - 1:07PM

The answer to high petrol prices is not ethanol or cutting the petrol excise but a radical hydrogen-based technology being trialled in Perth buses, a Liberal backbencher says.

West Australian Liberal Wilson Tuckey today said the hydrogen solution was the way forward, as politicians debated the petrol price crisis.

Several ideas have been put forward, including greater use of bio-fuels like ethanol, a cut to the petrol excise and better public transport.

But Mr Tuckey is the first to suggest hydrogen as the answer.

He said the CSIRO had developed a device the size of a small domestic microwave oven that runs on mains power or a solar panel to extract enough hydrogen from water to power a family car for up to 150 km per day.

Three buses in Perth are currently running on hydrogen, Mr Tuckey said.

"Furthermore, the BMW company has already produced ordinary motor cars running on hydrogen," he said.

He said parliament's refusal to invest in the technology was outrageous.

"What enrages me is that this parliament, because Labor is no better than us, is refusing to make the investment in a kit that would transfer the ordinary car to a hydrogen car."

AAP


So how come this is hiding in a backwoods trial? Is it because this solution is home grown and doesn't make profits for oil companies, or is it because it's application to the Northern Hemisphere and lack of sunlight is limited?

tomder55 answered on 08/07/06:

besides Elliot's excellent answer I would add that an issue is also raised in the question ;where does the hydrogen come from . It is my understanding that it is extracted from fossil fuels and that it takes more energy to produce than if the fossil fuel was used as the source of driving the vehicle . Virtually all hydrogen today is produced from fossil fuels in processes that generate significant quantities of greenhouse gases and generating it from renewables is not viable today .

While I was in Maine I saw a possible short term solution . LL Bean was sponsoring free bus routes throughout the island that Acadia National Park is on . The Island Explorer busses are propane-powered shuttles that saves many unnecessary auto trips throughout the area . If this concept of corporate sponsored mass transit were to catch on then vehicular emmisions would drastically decrease and fuel usage would drop dramatically .

paraclete rated this answer Excellent or Above Average Answer

Question/Answer
ETWolverine asked on 08/02/06 - Questions regarding the Qana air raid.

Now that a couple of days have passed since the Israeli air raid in Qana by Israel that allegedly killed over 50 people including 35 children, what have we learned about the incident?

What I have learned is that there are more questions than answers about the incident, and that things may not be as they appear. For instance:

1) There seems to be a time discrepancy between when the air raid took place and when the building actually fell. Reports now being seen by the public state that the building fell 6-8 hours after it was hit by Israeli missiles, and the news goups on hand only started reporting the story after the building fell. Why the time discrepancy? And why were there still people in the building 6-8 hours later when the building fell, which is what caused all the deaths.

2) The "rescue worker" holding up baby corpses for the cameras has been identified as the same "rescue worker" who held up dead baby corpses for the cameras back in 1996 when Israeli missiles killed 100 in another air raid in Qana. Is this just a coincidence?

3) If the collapse of the building is the cause of death of the dead women and children in Qana, why were the corpses exhibiting a state of advanced rigor mortis... as if they had been dead for days, not hours?

4) Why did photos show a sparkling-clean pacifier on the corpse of a baby who covered head-to-toe in dust?

5) Where were all the men? All the corpses found were women and children. Where are the corpses of the men? If the building targeted by Israel was merely a civillian residential building, why were there no men in the building? Were all the husbands and fathers at work during the nightime hours? Why were there no men in the building at the time it collapsed, 8 hours after the bombs fell, if there were so many women and children in the building? Why no men?

None of this is proof that the Qana incident was staged, and I'm not actually sure whether it was or not. But it does raise some interesting questions. And given the fact that the Islamofascists have a history of making up massacres that never occured in order to get media sympathy (anybody remember the Jenin Massacre that never happened?), it behooves us to ask questions and verify information before jumping to conclusions about these events.

Elliot

tomder55 answered on 08/07/06:

see my posting today about the Reuter's photographer . apparently it is the same 'person 'which begs the question how is he getting around all over the country to take these pictures ? My guess is that Adnan Hajj is a bogus name for a group of photographers who are pooling pictures and manipulating them on Photoshop .

ETWolverine rated this answer Excellent or Above Average Answer

Question/Answer
jackreade asked on 08/06/06 - Bob Novak-Guiliani Will Run in 2008

"Massachusetts Gov. Mitt Romney, trailing in national polls of Republican voters, has the lead in organizing early primary states for the 2008 presidential run, in the opinion of neutral GOP politicians.

Romney is particularly strong in Iowa, where caucuses begin the presidential hunt. A win there would swell Romney's now anemic identification among Republican voters.

A footnote: A report in this column that Rudy Giuliani intends to run for president has been confirmed by one of the former New York mayor's closest Republican friends. He said Giuliani definitely is running.

Read entire column here.

~~~~~~~~~~~~~~~~~~~~~~~~~~~~~~~~~~~~~~~~~~~~~~~~`

GOP running Guiliani in 2008?

Comments?

tomder55 answered on 08/07/06:

I think it has been clear for some time that Rudy would run . There are strange campaign finance laws that prevent many candidates from officially declaring their intentions . Both he and Romney are good choices but I think they both carry baggage that will hurt them in the GOP primaries .

Way too early to tell but I'm pretty sure a dark horse no one is considering a top tier candidate will emerge as the flag bearer .

jackreade rated this answer Excellent or Above Average Answer

Question/Answer
jackreade asked on 07/31/06 - Romney Apologized for TarBaby Remark

BOSTON -- "Gov. Mitt Romney has apologized for referring to the troubled Big Dig construction project as a "tar baby" during a fundraiser with Iowa Republicans, saying he didn't know anyone would be offended by the term some consider a racial epithet.

In a speech Saturday, Romney, a Republican considering a run for president in 2008, acknowledged he took a big political risk in taking control of the project after a fatal tunnel ceiling collapse, but said inaction would have been even worse.

"The best thing politically would be to stay as far away from that tar baby as I can," he told a crowd of about 100 supporters in Ames, Iowa.

Black leaders were outraged at his use of the term, which dates to the 19th century Uncle Remus stories, referring to a doll made of tar that traps Br'er Rabbit. It has come to be known as a way of describing a sticky mess, and has been used as a derogatory term for a black person."

~~~~~~~~~~~~~~~~~~~~~~~~~~~~~~~~~~~~~~~~~~~~~~~~

This is the end of the tarbaby discussion.

tomder55 answered on 08/06/06:

people who get offended by the term in the context it was used are ignorant (willfully ?) of American literature . He had nothing to apologize for since he said nothing wrong and it was clearly not said to be racist .


Mel Gibson on the other hand has clear issues he needs to resolve .

jackreade rated this answer Excellent or Above Average Answer

Question/Answer
ETWolverine asked on 08/04/06 - A terrific article.

Jonathan Zimmerman: What would Lincoln do? -- Trading our liberties for security

01:00 AM EDT on Saturday, July 29, 2006

NEW YORK

LIKE MOST of my friends and colleagues, I'm outraged by President Bush's assault on basic civil liberties in the so-called War on Terror. We invoke Thomas Jefferson on the rights of man, James Madison on checks and balances, and, most of all, Benjamin Franklin on the dangers of compromising these values: "Those who would give up an essential liberty for temporary security deserve neither liberty nor security."

But here are two words that you'll never hear us say: Abraham Lincoln.

That's because Lincoln's wartime decisions raise the really tough issue that most Democrats continue to evade: When should we give up some liberties in the name of security? And unless we can frame an answer, we don't deserve to win Congress in November or the White House in 2008.

Consider Lincoln's predicament in April 1861, at the outset of the Civil War. Eleven slaveholding states had seceded; four "border states" -- Maryland, Kentucky, Missouri, and Delaware -- remained in the Union but all still practiced slavery.

To win the war, Lincoln had to make sure that these states did not also secede. Together, they would have added 45 percent to the white population of the Confederate States of America. Even more important, they would have nearly doubled the Confederacy's capacity to make guns, ammunition, and the other tools of war.

The most critical state was Maryland, of course, because it bounded the District of Columbia on three sides. On the fourth side lay Virginia, which had already left the Union. If Maryland seceded too, Lincoln would find his national capital surrounded by the enemy.

And he couldn't have that. There were clear pockets of secessionist sentiment in Maryland's biggest city, Baltimore, where many houses flew Confederate flags after the war began. So rather than risk losing the city -- and, quite possibly, the war -- Lincoln sent Army officials into Baltimore to arrest alleged secessionists and jail them at Fort McHenry. (The prisoners included a grandson of Francis Scott Key, who had written "The Star Spangled Banner" while the fort was under British fire, in 1814.)

A few months later, as the Maryland legislature was preparing to vote on secession, Lincoln had 31 of the lawmakers imprisoned on suspicion of Confederate sympathies. They stayed in jail until the next state election, to ensure that pro-Union candidates won.

No charges. No evidence. No trial.

Sound familiar?

Then, as now, the president's enemies mounted constitutional challenges to his actions. One of the people imprisoned in Baltimore, John Merryman, sued for his freedom in federal circuit court. The senior judge was none other than Chief Justice Roger Taney, a Marylander and author of the infamous Dred Scott decision. Taney ruled that Lincoln had no right to jail Merryman without cause, because the Constitution gave Congress -- not the president -- exclusive power to suspend basic liberties in times of war.

Lincoln's response? Go to hell. His primary job, he said, was to win the war, and he needed every possible weapon to do so. He refused to obey Taney's opinion, which would have freed hundreds of Confederate partisans. Who knows what they would have done if they'd been let loose?

That should sound familiar, too. Indeed, almost everything President Bush has done in the "War on Terror" echoes Lincoln's actions during the War Between the States. In the name of national security, the Bush administration has jailed suspected terrorists without showing cause. It has denied them the right to counsel and other basic liberties. It has conducted warrantless eavesdrops on phone calls and e-mails. And it has insisted that the White House -- not Congress -- has the right to do all of this, on its own.

As in the Civil War, meanwhile, the Supreme Court has sought to rein in the president. Most recently, it ruled that the White House could not establish secret military commissions without congressional authority. It's still not clear how the president -- or Congress -- will respond.

But here's what is clear: Benjamin Franklin was wrong. And Abraham Lincoln was right.

There are times when dangers are so immediate -- and so terrifying -- that we do need to sacrifice some freedoms to stop them. And the Civil War was one of those times.

Is the "War on Terror" another? Not yet. Whatever the threat of Islamic terrorism, it doesn't come close to the peril that the Confederates posed to the Union in 1861. Until President Bush can explain exactly why we need his extra-legal measures, we should all stand in opposition to them.

At the same time, though, liberals like myself need to start thinking -- and talking -- about when we, too, would give up some liberties to save the Union. A rash of suicide bombers' striking several American cities at the same time? A "dirty bomb" or nuclear attack? A smallpox or anthrax attack?

You might reply that our liberties define our nation: If we abandon them, we give up on America itself. But Abraham Lincoln said otherwise, and lucky for us. By sacrificing a bit of freedom for suspected Confederate sympathizers, he helped win freedom for nearly 4 million enslaved African-Americans.

I think it was worth it. And I bet you do, too.

Until we Democrats can specify when and how we'd take the same harsh measures that Lincoln did, we don't deserve to sit under his mantle. Or to run the country.

Jonathan Zimmerman, who teaches history and education at New York University, is the author of Innocents Abroad: American Teachers in the American Century, to be published this fall by Harvard University Press.

Source: http://www.projo.com/opinion/contributors/content/projo_20060729_29zimm.1596f33.html

--------------

What do you think? Comments, please.

Elliot

tomder55 answered on 08/06/06:

just wait until a shopping mall is attacked then I think the average American will begin to understand .Most Americans consider 9-11 a a singular event ,an abberation .They do not understand that it is some of the executive decisions made that have prevented attacks on our soil .

I agree with the author that Bush has failed to explain exactly why we need his extra-legal measures on a regular basis and has not done a sufficient job to utilize the bully pulpit to support his side of the national debate .It has been one of the most frustrating aspects of his presidency to me .Early on ,the color coding of the threat alerts was rediculous and did not convey the seriousness of the issue . When alert levels were raised and nothing happened ;instead of it showing the effectiveness of measures like the Patriot Act , a complaceny settled in .

We were not put into a war footing but instead were told that we should just go about our routine . There was no family in the country during our major wars that were not impacted in the past and that may be the biggest difference between then and now .

ETWolverine rated this answer Excellent or Above Average Answer

Question/Answer
paraclete asked on 07/28/06 - Perhaps the US arn't as expert as they would like to think?

This certainly beggs the question, just how well trained are american forces? Is it shoot first and ask questions later?


Aussie veto stopped war crime
From:
By Cameron Stewart

July 29, 2006


AUSTRALIA intervened to stop key US military strikes against Saddam Hussein's regime in Iraq, fearing they might constitute a war crime.
Major General Maurie McNarn, then a brigadier and commander of Australian forces in Iraq, on several occasions played a "red card" against the American plans, which included hits on individuals. His objections drew anger from some senior US military figures.

In one instance, Major General McNarn vetoed a US plan to drop a range of huge non-precision bombs on Baghdad, causing one angry US Air Force general to call the Australian a "pencil dick".

However, US military command accepted Major General McNarn's objection and the US plans were scrapped.

The revelation of how Australia actively and successfully used its veto power in the 2003 invasion of Iraq is contained in a new book on the US-Australian alliance, The Partnership, by The Weekend Australian's foreign editor, Greg Sheridan.

The book reveals that Australia, as a member of the so-called coalition of the willing in Iraq, was given a power known as a "red card" that allowed Major General McNarn to veto US military actions, including individual targets and the types of weapons used.

Australia's proactive use of the veto power - on strategic, military and ethical grounds - helped the Americans produce a more effective and ethical targeting policy during the war.

The book reveals that Major General McNarn - now the head of the Defence Intelligence Organisation - delivered a "great shock" to the US when he first used the red card and then put his objections to the proposed US military strike in writing.

"Shit," exclaimed one American when he saw the document. "What if this leaks?" Major General McNarn replied that if the US did not take the illegal action, it would not matter.

As coalition forces prepared plans to take Baghdad, Major General McNarn vetoed three of five proposed US Air Force weapon systems - mostly huge bombs - on the grounds that they were not accurate for a radius of less than 16m and, as a result, were unsuitable for use in a built-up area.

One another occasion, Australia, along with fellow coalition partner Britain, successfully whittled down a list of proposed individuals the US considered legitimate targets.

The book also reveals that before the war, which started in March 2003, Australia made repeated efforts to get the US to focus on post-conflict planning in a more coherent way.

The lack of early US planning for the post-war phase in Iraq is seen to have contributed substantially to the violent disorder now being experienced there.

Australia also argued for the US to try to involve the UN as much as possible after the war. However, in a frank conversation with Foreign Minister Alexander Downer on April 1, 2003, US President George W.Bush said the US would get the blame for destroying Iraq and he did not want others coming to rebuild it.

"The UN can't manage a damn thing," Mr Bush told Mr Downer, recalling his visit to Kosovo, where the President found the UN personnel to be "a bunch of drunks".

The book also reveals that immediately after the fall of Baghdad, Mr Downer told Paul Bremer, head of the Coalition Provisional Authority in Baghdad, that the coalition should leave as soon as it could, while Iraq was in a decent state.

Since then, the Howard Government has argued it would be wrong to "cut and run" from Iraq and says Australian troops will remain there for as long as they are needed.

The book also reveals how close and frank the bilateral relationship became in the months leading up to and during the war in Iraq.

It includes an account of a conversation between Mr Downer and Mr Bush in April 2003 in which the President likened North Korea's erratic leader, Kim Jong-il, to "a child who throws his food on the floor and expects all the adults to rush over and pick it up".

At the same meeting, Mr Bush warned Mr Downer that Australia was likely to suffer casualties on the ground in Iraq, but he expressed unqualified admiration for the "brave, skilled fighters" of Australia's elite SAS.

tomder55 answered on 07/29/06:

Guess our allies weren't "poodles "after all.

Read about the give and take and the sometimes heated exchanges between members of the British ,American ,French and Russian allies during WWII. Nothing new here .Targetting in many cases is strategic and allies often have different strategic interests .

I see nothing in the article taht suggests the Aussies prevented war crimes.

Bush as it turned out was 100 % correct about the UN .When their base of opertions was hit by the Iraqis ;they couldn't exit fast enough.

Itsdb rated this answer Excellent or Above Average Answer
paraclete rated this answer Excellent or Above Average Answer

Question/Answer
Itsdb asked on 07/28/06 - Hezbollah says it has fired a new rocket

BEIRUT, Lebanon Hezbollah said it fired a new rocket, called Khaibar-1, striking near the Israeli town of Afula, south of Haifa. Israeli authorities reported that five rockets hit fields outside Afula, causing no casualties.

The strike came two days after Hezbollah leader Sheik Hassan Nasrallah said in a televised speech that Hezbollah would start a new phase in the battle striking beyond the Israeli city of Haifa, which has been hit several times in lethal rocket fire.

The area around Afula, 30 miles south of the Israeli-Lebanese border area, has been struck before, but Israeli security officials said Friday's attacks were the southernmost so far.

~~~~~~~~~~~~~~~~~~~~~~~~~~~~~~~~~~~~~~~~~~~~~~~~~~~~~~~

How fitting. Hezbollah has a new rocket named after a Jewish town Mohammed conquered that refused to accept Islam. Is 30 miles into Israel taking it to a new level - or does Nasrallahlamadingdong have something else up his Jubba?

tomder55 answered on 07/28/06:

never heard of it . They must've added a couple of Roman Candles to a Katyusha .

Is 30 miles into Israel taking it to a new level ? maybe but I quoted a souce to Elliot that claims the next red line will be when Zelzal-2 missiles on their mobile launchers cross over into Lebanon from Syria and strike at Tel Aviv . That will compel the Israelis to open a new front agaist Syria . I think this would be a bad move on Syria's part but they are the puppets and the mad mullahs in Tehran pull the strings . I'm sure Ahmaddamadingdong will willingly fight to the last Syrian.

Itsdb rated this answer Excellent or Above Average Answer
ETWolverine rated this answer Excellent or Above Average Answer

Question/Answer
jackreade asked on 07/27/06 - Call for All Out War

So, now, alQuaeda leadership has called for all out Holy War on Israel by ALL MUSLIMS.

If this edict is a complete bust(as I think it will be), could this be considered the total defeat of Islamofascists/terrorists on our part?

tomder55 answered on 07/28/06:

get to answer 2 questions with the same answer yippeeee !!! (see above Rolcam's question )

bottom line ; the mad mullahs of Tehran if not checked will have nukes and delivery systems that will reach from one end of the ummah to the other . Iran wants to make itself the regional hegemon and so far only the US and Israel stand in it's way .

Another reason the reaction has been muted ; the Arab justification ,or more correctly stated reason /rational whatever for attacking Israel had been the issue of occupation . (they convieniently forget 1967 and 1973 in that narrative but what the hell ) But since 2000 Israel has not been an occupier in Lebanon and in fact was well on the way to a pull back to almost pre-1967 boundries . Still Israel has been attacked . For the moderate Arab it is hard to find justicication in the agressions of Hezbollah and Hamas.

Finally getting to the crux of your question ... the untold battle being waged in the ME is between the forces of extremism and moderation /modernization .The forces of extemism had been marching forward unchecked until Operation Iraqi Freedom . The mad mullahs and the al-Qaedas realize that if they don't conduct their battle of the bulge now that they will lose out once the people taste the benefits that joining the 21st century brings .Democracy and all that will naturally follow but not if the forces of extremism win this war (this attack on Israel is not the main event ).

Preparing for my vacation next week . On the way home tonight I am going to stop at the Winery and see if any Israeli wines are available . While I hike in the woods and sit on the beach /pool I will thank God for the blessing of living in the USA . If I was to look for a symbol of this conflict it is not the bombed out buildings in Beirut and Haifa ;it is the empty beaches and resort hotels in Haifa and Tyre that come to mind. How different things could be .

jackreade rated this answer Excellent or Above Average Answer

Question/Answer
ROLCAM asked on 07/28/06 - Very strange bed partners !!

The Muslim civil war.

Is the Sunni-Shia divide in the Middle East now deeper than the antagonism between Israel and the Arabs? You might think so given the response of some Arab governments to Hizbollah's decision to attack Israel. Even as Israeli bombs fell on Beirut and Tyre, Saudi Arabia, perhaps the most conservative Arab Muslim state of all, openly condemned the actions of the Shia Hizbollah in instigating conflict with Israel. Never before in the history of the Arab-Israeli conflict has a state that considers itself a leader of the Arab Muslim peoples backed Israel so openly.

What is going on ??

tomder55 answered on 07/28/06:

bottom line ; the mad mullahs of Tehran if not checked will have nukes and delivery systems that will reach from one end of the ummah to the other . Iran wants to make itself the regional hegemon and so far only the US and Israel stand in it's way .

Another reason the reaction has been muted ; the Arab justification ,or more correctly stated reason /rational whatever for attacking Israel had been the issue of occupation . (they convieniently forget 1967 and 1973 in that narrative but what the hell ) But since 2000 Israel has not been an occupier in Lebanon and in fact was well on the way to a pull back to almost pre-1967 boundries . Still Israel has been attacked . For the moderate Arab it is hard to find justicication in the agressions of Hezbollah and Hamas.

Finally getting to the crux of your question ... the untold battle being waged in the ME between the forces of extremism and moderation and modernization .The forces of extemism had been marching forward unchecked until Operation Iraqi Freedom . The mad mullahs and the al-Qaedas realize that if they don't conduct their battle of the4 bulge now that they will lose out once the people taste the benefits that joining the 21st century brings .Democracy and all that will naturally follow but not if the forces of extremism win this war (this attack on Israel is not the main event ).

Itsdb rated this answer Excellent or Above Average Answer
ROLCAM rated this answer Excellent or Above Average Answer

Question/Answer
jackreade asked on 07/26/06 - Middle-East, I'm so Confused

My latest thoughts.

It seems to me that perhaps the Neo-Cons who Middle-East had as their ultimate goal----> Sunnis fighting Shi'a; in other words, Muslims killing Muslims.

I don't think this is too far fetched. OK, I'm upset about War.

Is this idea too far fetched??
Condi's face and words seem to back me up. That word "sustainable".

tomder55 answered on 07/27/06:

Let's assume Saddam was still sitting happily in Baghdad and examine this though counter-factual history. How would the Iraqi tyrant react to one of his principal adversaries in Iran -- the Duelfer Report notes that Saddam's WMD programs were primarily due to these fears -- now rapidly developing a nuclear weapons program. Would he become more compliant with the Anglo-American efforts to persuade him to conform to UN resolutions, or would he embark on an expansive program of his own, not to be outdone by his Persian rivals? The latter is plainly the more plausible outcome. Saddam saw himself as the guardian not only of the Arab Middle East but of the Muslim world as well.

Instead of American forces sitting on Iran's borders, the United States would likely be in the untenable position of facing a WMD arms race between Baghdad and Tehran. Emboldened by the Security Council's failure to act on Iraq, both Hussein and Iranian President Mahmoud Ahmadinejad would emerge to present a problem in the Middle East that makes the current situation appear fortunate. Iraq would still be providing gifts of $20,000 to the families of Palestinian suicide bombers, exacerbating the current Arab-Israeli conflict. Abu Musab al-Zarqawi would have left his safe haven in Iraq to return to Afghanistan -- he was harbored by and provided medical care from Saddam's regime prior to the invasion of Iraq after being injured in Afghanistan -- to lead an al-Qaeda mujahideen there. Washington would be finding itself to be in state of diplomatic overextension.

Diplomacy has never been considered the Bush Administration's strong suit, and with the oil-for-food program enduring -- that is if sanctions had not been abandoned by now -- it is difficult to imagine how any alternative could have been more successful. Up to this point diplomacy has failed in both Iran and North Korea, and contrary to claims that President Bush has admitted to mistakes by engaging in multilateral diplomatic endeavors with North Korea, United States efforts to establish the six-party talks preceded the invasion of Iraq.


MIKE ALLEN AND ROMESH RATNESAR assert in their Time magazine piece that "in the span of four years, the Administration has been forced to rethink the doctrine with which it hoped to remake the world as the strategy's ineffectiveness is exposed by the very policies it prescribed." The authors proceed to state that the difficult occupation in Iraq "may have emboldened [Tehran and Pyongyang] in their quest to obtain nuclear weapons." This fallacy is largely contradicted by the fact that in April 2001, nearly two years before the initial invasion of Iraq, the International Atomic Energy Agency (IAEA) reported that North Korea "probably has one or two nuclear bombs." It is also no secret that Iran has maintained its own long-standing nuclear program from well before the Iraq invasion.

C&P from Did Bush Overreach?
By Robert T. McLean

I would add this brilliant observation by Jon1667 to a related question on Philosophy Board :

I was simply ILLUSRATING (not comparing, illustrating) how it is often a good thing, and not a bad thing, to attack tyranny despite the short-term consequences, since not doing so would have been worse in the long run. It was a good thing to have deposed Saddam, not only because that regime was intolerable, but because it has indeed shaken up things in the area. What is going on now will ultimately be a good thing, since, except for Israel, the Mideast has been a festering sore, which has produced only two things: oil and murder.


Damn I wish I had said that !!!

jackreade rated this answer Excellent or Above Average Answer

Question/Answer
dimwit asked on 07/27/06 - Moral Equivilence


Hello wingnuts, and especially Roland:

It is true that both Lebanese and Israeli civilians are all victims of this cruel war. But a moral universe where the efforts to defend a people against obliteration are considered as equally worthy of lamentation as that of those who wish to destroy them is one that raises moral obtuseness to the point of amorality. That's AMORALITY!

Comments?

dim

tomder55 answered on 07/27/06:

excellent observation . it demonstrates clearly the difference between weak moral eqivalent thinking and right and wrong .

dimwit rated this answer Excellent or Above Average Answer
excon rated this answer Excellent or Above Average Answer

Question/Answer
paraclete asked on 07/27/06 - Could he be right ~ is the Israeli military in control?

'Rogue elements' behind UN attack
From: AAP
July 27, 2006
ROGUE elements in the Israeli military were behind a rocket attack on a United Nations compound that killed four UN observers, former Australian deputy prime minister Tim Fischer said today.

Four UN observers were killed in the Israeli bombardment on their position in the south Lebanon border town of Khiam early yesterday (AEST).

Mr Fischer, who gained combat experience while serving with the Australian Army in the Vietnam War, said Israel and Hezbollah guerrillas in Lebanon were to blame for the increasingly bloody cross border conflict.

He said the Israeli response to Hizbollah left Israel open to criticism.

"I blame both sides. The Katushya rockets (fired by Hezbollah), their first salvo in Nazareth killed two Arab Israelis," Mr Fischer told ABC radio.

"Clearly there are rogue elements on both sides of the border. There is a rogue element in the Israeli military machine and it's about time the world called a spade a spade and recognised that that element has been there for several years and continues.


"And the evidence unfortunately is going to unfold further in the next 24 hours."

When asked if his judgment on the course of the fatal UN rocket attack was premature, Mr Fischer said: "Of course they (the Israeli government) will investigate it and I look forward to that and of course you will hold final judgment but as prima facie evidence 10 phone calls - 10 calls - were made from that clearly marked UN compound post.

"And why might you be suspicious? Because there's form. In 1996 the Fijians, the poor old Fijians, maintaining a UN compound in exactly the same area took a hit which killed 104 people sheltering in the UN compound and that has become known as the Qana massacre.

"Israel has a right to defend itself, but is being counter productive in the way it is defending itself."

tomder55 answered on 07/27/06:

so he is confirming it is a rocket attack and not an air strike ?


Major Paeta Hess-von Kruedener ,a Canadian who was killed there sent an e-mail to his former commander, a retired major-general, Lewis MacKenzie, in which he wrote that Hezbollah fighters were "all over" the U.N. position, Mr. MacKenzie said. Hezbollah troops, not the United Nations, were Israel's target.

"What he was telling us was Hezbollah fighters were all over his position and the IDF were targeting them, and that's a favorite trick by people who don't have representation in the U.N. They use the U.N. as shields knowing that they cannot be punished for it."

Jihadist cowards are well known for using innocents and neutrals as their shield so this report is consistant with their M.O.

Here is a picture of a UN flag flying side by side with a Hezbollah flag in Lebanon



Nuff said ????

ETWolverine rated this answer Excellent or Above Average Answer
Itsdb rated this answer Excellent or Above Average Answer
paraclete rated this answer Excellent or Above Average Answer

Question/Answer
ETWolverine asked on 07/26/06 - More News from the War in Lebanon

HEZ'S IRANIAN CACHE STASH

By URI DAN, with Post Wire Services

July 26, 2006 -- JERUSALEM - Israel claimed victory in Hezbollah's southern "capital" yesterday after a battle in Lebanon that uncovered Iranian-made weapons and electronic equipment and left 150 guerrillas dead.

Israeli forces found "war rooms" equipped with Iranian surveillance and eavesdropping gear in Bint Jbail, the main Hezbollah stronghold just inside the border.

"The town is completely controlled by us," an Israeli colonel said.

Caches of weapons were also found in Bint Jbail, Israeli officials said.

Mahmoud Komati, deputy chief of Hezbollah's political arm, refused to acknowledge the group was firing Iranian-made missiles into Israel.

"We don't deny nor confirm. We believe where the weapons come from is irrelevant," he said, adding that Hezbollah has weapons made in France, Russia, China and the United States.

"Some of our fighters carry M16s. So you think we buy them from America?" he asked.

The death toll from the two-week war continued to mount, including six Lebanese killed by a bomb in Nebatiyeh and a 15-year-old Arab girl killed by a Hezbollah rocket in northern Israel. At least three U.N. observers were also killed, with one other feared dead. They were from Austria, Canada, China and Finland, U.N. and Lebanese officials said.

U.N. Secretary-General Kofi Annan called on Israel to investigate the "apparently deliberate" deadly attack against the world body's observation post.

Israeli U.N. Ambassador Dan Gillerman expressed "deep regret" for what happened, but called Annan's accusations "premature and erroneous."

Gillerman said he was "shocked and deeply distressed by the hasty statement of the secretary-general insinuating that Israel has deliberately targeted the U.N. post." A spokesman for the Israeli Foreign Ministry said the incident would be investigated.

Among the yesterday's developments:

* A senior Hezbollah official said the group was surprised at the fierce Israeli reaction to the guerrillas' kidnapping of two soldiers.

"The truth is - let me say this clearly - we didn't even expect [this] response . . . that [Israel] would exploit this operation for this big war against us," said Komati, the Hezbollah spokesman.


* Saudi Arabia's King Abdullah and Iranian President Mahmoud Ahmadinejad said the offensive could ignite a wider Mideast war - and Syria was on its highest state of alert in 15 years.

* Israeli Defense Minister Amir Peretz said his forces will control only a "security strip" in southern Lebanon and hold it until an international force can replace them.

Peretz said anyone who enters the strip could be fired upon. The strip is expected to range from two to six miles deep, north of the border.

* Other military officials said they did not plan to push deeper into Lebanon.

"The intention is to deal with the Hezbollah infrastructure that is within reach," Col. Hemi Livni, who commands troops in the western sector of southern Lebanon, told Israel Army Radio. "That means in southern Lebanon, not going beyond that."

* Secretary of State Condoleezza Rice was told during her visit to Israel on Monday that a peacekeeping force "like in Bosnia" was needed, sources said.

Hezbollah leader Sheik Hassan Nasrallah took aim at Rice's peace proposals when he said in a televised speech that his terror group would not accept any "humiliating" conditions for a cease-fire.

He also said the war was entering a "new period" and threatened to fire missiles deeper into Israel.


* New fighting erupted yesterday near the town of Marun a-Ras, which Israel thought had been conquered on Sunday.

Abu Jaafar, commander of Hezbollah's central sector on the border, and four gunmen were killed and several soldiers from an Israeli tank unit were wounded, officials said.

Israel is now poised to knock out Hezbollah launching sites near the coastal city of Tyre, from which 12 Syrian-made rockets were fired at Haifa yesterday.

At least 66 Israelis were reported wounded from the attacks yesterday. A 76-year-old Israeli suffered a fatal heart attack as he hurried to an air-raid shelter in Haifa.

At least 418 people in Lebanon and 42 in Israel have been killed since the fighting began on July 12, when Hezbollah raided an Israeli patrol south of the border.


uri.dan@nypost.com

------------------

And now my comments:

1) We now have proof positive of Iranian and Syrian support for Hizbollah, with Hizbollah using Iranian spy equipment and Syrian missiles. I think that there is no longer any question of Iran's involvement in this whole mess.

2) A senior Hezbollah official said the group was surprised at the fierce Israeli reaction to the guerrillas' kidnapping of two soldiers.

"The truth is - let me say this clearly - we didn't even expect [this] response . . . that [Israel] would exploit this operation for this big war against us," said Komati, the Hezbollah spokesman.


In other words, they were expecting Israel to react as it has for the past decade. Hizbollah was clearly trying to goad Israel into a limited response that they could use against Israel. They just got more than they bargained for. But that statement makes clear the fact that the kidnappings were planned long in advance, taking into consideration Israel's probable reactions based on their history of limited response. Which means that this was merely to be one incident of many to come.

Well, to paraphrase Ronald Reagan: They counted on Israel to be passive. They counted wrong.

3) Hizbollah continues to reject any cease-fire proposals that actually make them cease firing. Israel is on board with the idea of a cease-fire that includes thei soldiers being returned, a satisfactory buffer zone, and a multi-national peacekeeping force patrolling the buffer zone. Hizbollah rejects the very idea of a cease-fire against Israel as humiliating. (Well, if I were getting my ass kicked up between my ears like Hizbollah in confrontation after confrontation, I'd be humiliated too.) I think that this fact alone should be the clearest indicator of who the aggressive party is, and who really wants peace.

Elliot

tomder55 answered on 07/26/06:

While the world press and the UN meme is that if the US does not include Syria and Iran into a cease-fire negotiation that no agreement can be achieved ; the Syrians have issued an ultimatum to the US. They threaten that attacks by Hezbollah will intensify and will penetrate deeper into Israel ;with perhaps direct targetting of Tel Aviv and Jerusalem unless they are included in the negotiations .

Their terms for a cease-fire according to John Batchelor is that America force Israel to withdraw from Lebanon, to cease tactical strikes on Hezbollah bases and weaponry in Lebanon, to cease tactical strikes on Hamas in Gaza, and to make territorial concessions that would mean that Israel could no longer defend either Jerusalem or Tel Aviv.

He also alluded to speculation that Condi Rice has already offered Syria ,through their puppet in Lebanon Prime Minister Siniora; to grant Syria a role in a new Lebanon ;an offer to drop the Hairiri assasination investigation ;to grant immunity to chief terrorists Nasrallah of Hezbollah and Meshal of Hamas ,and to just ignore the compelling evidence of the Iranian connection .

SAY IT AINT SO CONDI!!!


ETWolverine rated this answer Excellent or Above Average Answer

Question/Answer
ROLCAM asked on 07/26/06 - Waking up from another bad dream!!

"Obviously, the violence in Baghdad is still terrible, and therefore there needs to be more troops."

PRESIDENT BUSH


tomder55 answered on 07/26/06:

I don't object to that but I do have a problem with Iraq PM Maliki addressing a joint session of Congress today . He is the PM but I am concerned he is just a little too close to al-Sadr for my liking .I may be wrong about him ,that remains to be seen by the actions the Iraq gvt. takes in the next weeks regarding al-Sadr's militia violence . US troops are needed for more security in Baghdad but I hope they are not dragged into taking sides in the sectarian violence.

ROLCAM rated this answer Excellent or Above Average Answer

Question/Answer
paraclete asked on 07/25/06 - A wake up call to the UN?

Coming close after UN critism of Israel, is Israel telling the UN it can no longer just be an observer.

UN deaths 'deeply regrettable'
From: Agence France-Presse From correspondents in Tyre, Lebanon
July 26, 2006

Hit ... a huge plume of smoke rises above Khiam / AP ISRAEL said today it regretted the deaths of four UN military observers in southern Lebanon and would investigate the air strike that killed them.

"Israel sincerely regrets the tragic death of the U.N. personnel in south Lebanon," Israeli Foreign Ministry spokesman Mark Regev said.

"We do not target UN personnel and, since the beginning of this conflict, we have made a consistent effort to ensure the safety of all members of (the UN peacekeeping force). This tragic event will be thoroughly investigated," Mr Regev said.

A Chinese observer was among the four UN personnel killed in an Israeli air strike on Lebanon, Chinese state media said today.

Xinhua news agency named the observer as Du Zhaoyu and said his body had been recovered.

Earlier UN Secretary General Kofi Annan said he was "shocked" at Israel's "apparently deliberate targeting" of the UN post in Lebanon.


Advertisement:
Mr Annan described the strike as a "co-ordinated artillery and aerial attack on a long established and clearly marked UN post."

He said it took place "despite personal assurances given to me by Prime Minister Ehud Olmert that UN positions would be spared Israeli fire."

"Furthermore, General Alain Pelligrini, the UN Force Commander in south Lebanon, had been in repeated contact with Israeli officers throughout the day on Tuesday, stressing the need to protect that particular UN position from attack.

"I call on the Government of Israel to conduct a full investigation into this very disturbing incident and demand that any further attack on UN positions and personnel must stop.

"The names and nationalities of those killed are being withheld pending notification of their families. I extend sincere condolences to the families of our fallen peacekeepers."

Earlier it had been announced by US envoy to the UN John Bolton that an Israeli air attack on a UN post in south Lebanon this morning had left four UN observers dead.

Ambassadors from the permanent members of the UN Security Council were told about the dead from the reported Israeli attack during a meeting to discuss Iran, Mr Bolton said.

"It is something we take seriously,'' Bolton said. ``We are going to focus on this incident, see what we can find out about it.''


The UN said rescue efforts at the UN house and shelter in Lebanon border town of Khiam had been mounted.

"A rescue team from UNIFIL was sent to this place but they did not succeed for the moment to rescue the people under the rubble," said Milos Strugar, spokesman of the UN Interim Forces in Lebanon (UNIFIL).

"There were 14 other Israeli firings close to this position by the Israeli side and the firing continued during the UNIFIL rescue."

The hilltop town of Khiam, close to the Israeli border sheltered an infamous prison during Israel's 22-year occupation of south Lebanon to 2000, but is now a Hezbollah stronghold.

An AFP correspondent across the border in the town of Metulla in the far north of Israel saw a heavy bombardment under way of the Khiam area.

Meanwhile, Israeli troops entered the border town of Bint Jbeil, a Hizbollah military stronghold, for the first time overnight as they pushed deeper into Lebanese territory.

The Israeli army, which has been encountering fierce resistance in its war on Hizbollah despite its clear military superiority, entered the border town following a day of fierce fighting with the Lebanese Shiite guerrillas.

"The Israeli army is in Bint Jbeil. Some sporadic fighting is ongoing inside and around the town," Mr Strugar said.

Asked whether the Israeli troops had captured the town, he said: "It is difficult to know what is really happening at this time.

"The civilian population in Bint Jbeil is caught in the crossfire."

tomder55 answered on 07/26/06:

UNIFIL is running a kind of ambulance service on the Lebanese side of the border. That is not its official mission; it has failed in its official mission . UNIFIL is able to run convoys in an area where the Hezbollah are shifting squads around while the IDF doing its best to kill the terrorists . Yet until July 26 the UNIFIL had not suffered any fatalities from IDF fire but the danger of friendly fire remains a reality in a war zone.

I really have to ask again .Their mission in Lebanon was obviously a failure . Why have they remained ? Kofi Annnan should be questioned why his peace-keepers set up shop so close to the Hezbollah terrorists ;so close that it was a miracle they haven't been hit until now (especially their convoys that look from the sky like . They've been camping together in solidarity for 6 years . It was his decisionm to keep them there after hostilities commenced . Their fate was in his hands.Just another example where his judgement and competence should be scrutinized if you ask me .

Why is the UN repairing roads that Israel has destroyed in Hezbollah controlled areas ?

Koffi says, "This [was a] coordinated artillery and aerial attack on a long established and clearly marked UN post," and describes the attack as the result of "apparently deliberate targeting" He has been blatantly hostile to the Israeli counterattack and has demanded a cease-fire from the moment things went downhill for Hezbollah. That he would accuse Israel of murder fits the pattern. Would he intentionally endanger human beings to undermine and halt the Israeli advance into Lebanon? Im thinking now of Oil for Food. Certainly UNIFIL has been complicit in Hezbollah attacks before .In this case they are actively acting as human shields for terrorists .

paraclete rated this answer Excellent or Above Average Answer

Question/Answer
paraclete asked on 07/25/06 - A wake up call to the UN?

Coming close after UN critism of Israel, is Israel telling the UN it can no longer just be an observer.

UN deaths 'deeply regrettable'
From: Agence France-Presse From correspondents in Tyre, Lebanon
July 26, 2006

Hit ... a huge plume of smoke rises above Khiam / AP ISRAEL said today it regretted the deaths of four UN military observers in southern Lebanon and would investigate the air strike that killed them.

"Israel sincerely regrets the tragic death of the U.N. personnel in south Lebanon," Israeli Foreign Ministry spokesman Mark Regev said.

"We do not target UN personnel and, since the beginning of this conflict, we have made a consistent effort to ensure the safety of all members of (the UN peacekeeping force). This tragic event will be thoroughly investigated," Mr Regev said.

A Chinese observer was among the four UN personnel killed in an Israeli air strike on Lebanon, Chinese state media said today.

Xinhua news agency named the observer as Du Zhaoyu and said his body had been recovered.

Earlier UN Secretary General Kofi Annan said he was "shocked" at Israel's "apparently deliberate targeting" of the UN post in Lebanon.


Advertisement:
Mr Annan described the strike as a "co-ordinated artillery and aerial attack on a long established and clearly marked UN post."

He said it took place "despite personal assurances given to me by Prime Minister Ehud Olmert that UN positions would be spared Israeli fire."

"Furthermore, General Alain Pelligrini, the UN Force Commander in south Lebanon, had been in repeated contact with Israeli officers throughout the day on Tuesday, stressing the need to protect that particular UN position from attack.

"I call on the Government of Israel to conduct a full investigation into this very disturbing incident and demand that any further attack on UN positions and personnel must stop.

"The names and nationalities of those killed are being withheld pending notification of their families. I extend sincere condolences to the families of our fallen peacekeepers."

Earlier it had been announced by US envoy to the UN John Bolton that an Israeli air attack on a UN post in south Lebanon this morning had left four UN observers dead.

Ambassadors from the permanent members of the UN Security Council were told about the dead from the reported Israeli attack during a meeting to discuss Iran, Mr Bolton said.

"It is something we take seriously,'' Bolton said. ``We are going to focus on this incident, see what we can find out about it.''


The UN said rescue efforts at the UN house and shelter in Lebanon border town of Khiam had been mounted.

"A rescue team from UNIFIL was sent to this place but they did not succeed for the moment to rescue the people under the rubble," said Milos Strugar, spokesman of the UN Interim Forces in Lebanon (UNIFIL).

"There were 14 other Israeli firings close to this position by the Israeli side and the firing continued during the UNIFIL rescue."

The hilltop town of Khiam, close to the Israeli border sheltered an infamous prison during Israel's 22-year occupation of south Lebanon to 2000, but is now a Hezbollah stronghold.

An AFP correspondent across the border in the town of Metulla in the far north of Israel saw a heavy bombardment under way of the Khiam area.

Meanwhile, Israeli troops entered the border town of Bint Jbeil, a Hizbollah military stronghold, for the first time overnight as they pushed deeper into Lebanese territory.

The Israeli army, which has been encountering fierce resistance in its war on Hizbollah despite its clear military superiority, entered the border town following a day of fierce fighting with the Lebanese Shiite guerrillas.

"The Israeli army is in Bint Jbeil. Some sporadic fighting is ongoing inside and around the town," Mr Strugar said.

Asked whether the Israeli troops had captured the town, he said: "It is difficult to know what is really happening at this time.

"The civilian population in Bint Jbeil is caught in the crossfire."

tomder55 answered on 07/26/06:

there was no aerial bombing it was totally artillery . The terrorists were using the UN compound as a safe haven .

The UN has sat on their asses their for 6 years doing nothing about the terrorist buildup and occasional attack on Israel . Why don't they just leave as they did in Iraq when things got tough ?

paraclete rated this answer Excellent or Above Average Answer

Question/Answer
ETWolverine asked on 07/25/06 - Israeli wines

Tom,

If you are looking for some good Israeli wines from the northern part of Israel, there are a few that I can recommend.

Golan Wineries, located in the Golan Heights distributes a number of really good wines under a number of different lables including "Yarden", "Gamla" and "Golan".

I highly recommend these:

Semi-Sweet White
Yarden's Johannisberg Riesling
Gamla's White Riesling

Dry White
Yarden's Chardonnay and Sauvignon Blanc
Gamla's Chardonney

Desert Wines
Golan's Moscato

Dry Red (My favorites)
Yarden's Pinot Noir, Merlot and Cabernet Sauvignon
Gamla's Pinot Noir and Merlot
Golan's Cabernet Sauvignon (relatively cheap but good)

Sparkling Wines
Yarden's Blanc de Blanc

Buying these wines would certainly help the economy of some of the most hard-hit areas of Israel in the north. And they are good wines too. Yarden's reds, in particular, tend to be major prize winners at competitions.

Elliot

tomder55 answered on 07/26/06:

Before the weekend I intend to purchase some good Israeli wines. I have yet to read or hear a report on the economic impact of these unprovoked attacks on Israel. Every day I am getting a little angrier. I just have to wonder who would be crying for a cease fire if Israel was on the ropes ?

ETWolverine rated this answer Excellent or Above Average Answer

Question/Answer
dimwit asked on 07/25/06 - Second Lebanon Question


Hello:

My simplistic view of the world has raised a concern. Hopefully, the Wolverine, and others will set me straight.

I used to think of the US as the most dominant force on the planet. Then we lost in Vietnam. Then we lost in Lebanon in 82. Were losing in Iraq, and nobody has ever controlled Afghanistan.

My point is, I think the Islamists have the Americans number. Contrarily, and notwithstanding huge and longstanding lapses, I think the Israelis have ALWAYS had the Arabs number.

My question, and my concern is, has that equation changed?

dum

tomder55 answered on 07/25/06:

It is true that jihadists got a false impression of the fighting capabilities of the US forces prior to OIF . However it is doubtfull they still cling to these illusions . They thought that US forces were not capable of taking on the enemy in close quarters fighting. They have been proven wrong over and over again in Iraq.

The IDF is methodically reducing an entrenched enemy . It remains to be seen if Hezbollah terrorists die the heroic martyrs they claim to be or slink back into the landscape. I expect that IDF forces will be bathing in the Litani River shortly .

dimwit rated this answer Excellent or Above Average Answer
Judgment_Day rated this answer Excellent or Above Average Answer

Question/Answer
ROLCAM asked on 07/25/06 - No peace without justice !!

I have at last come across an article which has been written by a level-headed journalist.

I would like you to read it, here it is in its entirety.

It certainly does not hold back any punches.

**************************************************

George Bush, in all his wisdom, informed the world the other day that "the root of the problem lies with Hizbollah". Wow! The oracle has spoken. The great geographer who, prior to his election to the post of the most powerful man in the world, could barely name a dozen of the world's capitals thinks that his proclamations will be readily swallowed.

The problem is far more deep rooted in the injustices of the Palestinian problem , of which Hizbollah, together with Hamas, Islamic Jihad, Fatah and all the other "terrorist" groups are all products. Only last week, right wing Israelis commemorated the bombing of the King David Hotel in Jerusalem in which 92 persons lost their lives. This was the work of the Irgun svai lumi, another "terrorist" group who had Menachem Begin, a future Prime Minister, among its leaders.

Among those celebrating the King David Hotel massacre was Benjamin Netanjahu, another former PM who asked those present to distinguish between "terrorists" and "freedom fighters". Two weights and two measures, you may ask. Any student of modern Middle East history will know too well.

Of course, Israel has the right to defend its territory and the lives of its citizens from the indiscriminate shelling of a fanatical paramilitary group, and, failing control by the weak central Lebanese government in Beirut, has no alternative but to cross the border and fight them in legitimate self defence. But talk of overkill. Need Israeli planes have pounded Beirut International Airport and the country's road and bridge infrastructure? And hit "targets" in central and east Beirut? All this while declaring its readiness to befriend the Lebanese people. Something is terribly wrong with Israel's actions, possibly emerging from the country's immense and justified sense of insecurity.

Meanwhile, like the US, I am ashamed to say that Europe sits back waffling about a ceasefire and a peacekeeping force. The Lebanese people, already ravaged by so much sorrow and devastation, can only crouch in a corner watching their cities, so painstakingly rebuilt after the civil wars, crumble around them. The townsfolk of northern Israel and Haifa for the first time experience the absolute terror of bombardment from the sky. Who knows - as awful as this may sound - the trauma of these days may push younger generations towards a fairer and faster peace process.

America wants to give Israel the time to clean up Hizbollah. How very nave. After Afghanistan and Iraq, the Americans should have more than learnt their lesson. These people don't and won't go away. Like an ugly wart which is treated superficially, they will re-emerge and be the cause of more pain and instability.

They will only fade away into oblivion when there is no longer scope for them, when murderous recruits will be hard to come by, and this can only happen when the moderate, peace-loving majority of their compatriots, oppressed within their land or taking refuge outside it, will be satisfied that justice has been done.

Back to the drawing board folks, with a stronger European intervention and a US that is ready to impose peace by tightening its purse strings.

Throwing in a couple of Edward Saids with the Albrights and Kissingers of this world may make for a more even playing field. It will be painful, especially for Israel, but I fear it's their only hope. What comes after Hizbollah will only be worse.

rolcam.







tomder55 answered on 07/25/06:

I guess it doesn't matter that the Brits were given almost 1/2 hour warning that the King David was going to be bombed . Sufficient time to evacuate the building and it's surrounds.

As I mentioned in another posting leave Israel alone in peace and I'm sure they would be glad to respond proportionally .

ETWolverine rated this answer Excellent or Above Average Answer
Itsdb rated this answer Excellent or Above Average Answer
powderpuff rated this answer Excellent or Above Average Answer
ROLCAM rated this answer Average Answer

Question/Answer
dimwit asked on 07/25/06 - Lebanon

Hello:

Everybody wants a peackeeping force installed between Israel and Hezbollah, but nobody wants THEIR soldiers in it. Hmmmm.

Could it be that they realize that Hezbollah will attack them too? I think it is!

dimdude

tomder55 answered on 07/25/06:

actually UN blue helmets have figured a way to live side by side with Hezbollah. All they needed to do was turn a blind eye to the introduction of 10,000 rockets into the theater since 2000 and the construction of an intricate complex of bunkers and tunnels ....as well as the occasional cross border raid.

I heard that the Ruskies volunteered to put their troops on the buffer but I think that was just posturing ,a feeble attempt to demonstrate their relevence .

The Turks could be honest brokers but it would conjure up memories of the Ottomans.

France is saying that talk of a force is premature . I agree ;Hezbollah's capabilities have not nearly been degraded enough to consider it . Peace-keepers are not needed as much as war makers .Israel says that any new international force must be combat mission willing and able . I agree ,any peace keeping force without muscle would likely be held hostage and be ineffective .That being said;France ..being one of the big loud mouth nations when the issue of multiilateralism is raised should be taking the lead in volunteering if you asked me.
NATO nations say they are overstretched by their operation in Afghanistan (lol)Anyway ;NATO is ineffective without US participation as the Balkans proved . Thankfully New Zealand has signed on to the idea.

I think the real issue comes down to this : is there any neutral position in this war against jihadistan ? I don't think so. Jihadists need to be defeated . Either you are with them or against them .

dimwit rated this answer Excellent or Above Average Answer

Question/Answer
ETWolverine asked on 07/25/06 - The new Flip Flop

HEZBOLLAH 'COWARDS' TO BLAME FOR CIVILIAN SLAUGHTER: U.N. BIG
By URI DAN, with Post Wire Services

July 25, 2006 -- JERUSALEM - The top U.N. humanitarian official yesterday charged that Hezbollah is fighting like cowards and causing the deaths of hundreds of innocent women and children in Lebanon.

In scathing comments, U.N. humanitarian chief Jan Egeland accused Hezbollah of "cowardly blending" among civilians and said he was appalled that the group was proud that more innocents had died than its own fighters.

"Consistently, from the Hezbollah heartland, my message was that Hezbollah must stop this cowardly blending . . . among women and children," Egeland said at Larnaca airport in Cyprus after visiting Lebanon to coordinate an international aid effort.

"I heard they were proud because they lost very few fighters and that it was the civilians bearing the brunt of this. I don't think anyone should be proud of having many more children and women dead than armed men," Egeland added.

The terrorist group has built bunkers and tunnels near the Israeli border to shelter weapons and fighters, and its members easily blend in among civilians.

"We need a cessation of hostilities because this is a war where civilians are paying the price," said Egeland, who was heading to Israel next.

Egeland had previously called Israel's offensive "disproportionate" and "a violation of international humanitarian law."

Early this morning, Israel took control of the town of Bint Jbail in southern Lebanon following hours of intense fighting, Israeli radio reported. And the army said the fighting wasn't over yet.

Dozens of Hezbollah fighters were killed in the besieged terrorist stronghold, the Israeli military said.

Four Israeli soldiers also died - two in a helicopter crash and two in the fighting at Bint Jbail, known as "the capital of the [Hezbollah] resistance" and is believed to hold the group's largest arsenal in the area.

Israel has already taken Maroun al-Ras, another Hezbollah stronghold in southern Lebanon.

In other developments:

* Hezbollah guerrillas fired 80 more rockets into Israel, wounding 13 people, despite the steady push of Israeli ground troops north of the border.

* Hezbollah leader Sheik Hassan Nasrallah claimed he told "some of the main political leaders" in Lebanon he planned the July 12 kidnapping of Israeli soldiers, which triggered the border war.

"And nobody said to me 'You are not allowed to abduct Israeli soldiers,' " he told al-Jazeera TV.

* Four U.N. peacekeepers were wounded, one seriously, in the crossfire between Israel and Hezbollah in South Lebanon, U.N. officials said.

* Israeli diplomats around the world have been warned to beware of terrorist attacks because Hezbollah activated "sleeper cells" abroad. Missions and other diplomatic offices have been put on high alert, sources said.

* The White House said declaring a truce would not be enforceable now.

"I think the notion that you have a cease-fire at this point is unenforceable and does not really get us to the point we need to be at," presidential spokesman Tony Snow said in Washington.

* Hamas and other groups in Gaza are reportedly ready to accept a cease-fire deal that would include the release of Israeli soldier, Gilad Shalit, who was kidnapped on June 25.

The Palestinian Agriculture Minister told the Israeli newspaper Haaretz that Shalit would be returned if Israel guarantees the future release of Palestinian prisoners.

. In Lebanon, Israelis attacked 20 more missile launchers yesterday, but there are many left, officials said.

In the fighting at Bint Jbail, an estimated 100 to 200 Hezbollah holdouts were under siege by about 6,000 Israeli soldiers. At least 20 Israelis were wounded.

Much of Bint Jbail's population of 30,000 had fled. But the outnumbered holdouts put up ferocious resistance, Israeli officials said.

Some 300 Americans and 100 Europeans are believed to be trapped in villages south of Tyre, said a German official involved in evacuation efforts.

About 11,700 Americans have fled Lebanon since the conflict began, the State Department said.

-----------------------

Yesterday Egelund said that Israel's actions are illegal. Today, he says that Hizbollah are cowards, and they are happy about civillian casualties. Is this a flip flop? Did Egelund not know what was going on before he shot his mouth off yesterday? Does this flip-flop constitute a change in the UN's position? Does it matter?

We also hear that members of the Lebanese government knew about the kidnappings and missile attacks in advance... but nobody told Nasrallah that kidnapping soldiers is a bad thing, and the poor, stupid, rag-head couldn't figure it out on his own.

And in other news...

-------------------

CONDI SERVES RICE SURPRISE
By DEBORAH ORIN

July 25, 2006 -- Secretary of State Condoleezza Rice yesterday launched a Mideast shuttle diplomacy mission with a surprise stop in shell-shocked Lebanon, where she spelled out her terms for a cease-fire.

Rice's rules - backed up by the White House - stress that any cease-fire must be part of a broader deal that cracks down on how Hezbollah terrorists use southern Lebanon as a base to attack Israel.

"The situation on the border cannot return to what it was before July 12," she firmly told Lebanese Parliament Speaker Nabih Berri, a Hezbollah ally, in Beirut.

It was July 12 when Hezbollah crossed the border and kidnapped two Israeli soldiers, touching off the war that has devastated Lebanon, which is caught in the crossfire.

Rice's proposal was rejected by Berri, who demanded a cease-fire followed by a prisoner swap for the two Israeli soldiers, and then broader talks.

But Lebanese Prime Minister Fuad Saniora's office suggested he was more receptive and discussed Rice's ideas and "ways of developing them."

Rice also offered sympathy plus $30 million in humanitarian aid but rejected Lebanon's plea for an instant cease-fire that leaves Hezbollah in place - she said any cease-fire must be "sustainable."

That means a buffer zone of around 20 miles - longer than the range of the Katyusha rockets that Hezbollah fires across the border into Israel, she said.

Rice's plan, based on U.N. resolution 1559, also calls for an international force deployed along the Israeli-Lebanese border to keep Hezbollah from using it as a military staging area.

Rice later flew to Israel. Today she'll also meet marginalized Palestinian President Mahmoud Abbas before flying to Rome to meet other leaders and try to spearhead a solution.

"Any peace is going to have to be based on enduring principles and not on temporary solutions," Rice said after meeting with Israeli Foreign Minister Tzipi Livni. "The free world is facing a threat, the goal of Hezbollah is to set the world aflame and we will not let them succeed."

A central strategy for Rice and President Bush is to get other Arab states, like Egypt and Jordan, to pressure Syria to stop backing and arming Hezbollah.

The theory is that those Arab states are also worried about Hezbollah because of its links to the non-Arab state of Iran with its radical Shiite mullah leaders.

Rice's message to Lebanon's fragile government is that removing Hezbollah would be a big step toward finally giving it real control over its entire territory.

She also stressed that Lebanon's government - which claims it can't control Hezbollah and includes Hezbollah sympathizers like Berri - must sign any cease-fire and take responsibility for it.

"If there is a cessation of hostilities, the government of Lebanon is going to have to be the party," she said.

"Let's treat the government of Lebanon as the sovereign government that it is."

Saniora warmly greeted Rice with kisses on both cheeks as she arrived on a heavily armed helicopter from Cyprus because Israeli bombing has closed Beirut's airport.

Saniora also told Rice that Israel's bombardment has set his country "backwards 50 years" as he pleaded for an immediate cease-fire, his aides said.

Assistant Secretary of State David Welch said Berri claimed a prisoner exchange would fix other problems but "that is not what we think."

Hezbollah kidnapped the soldiers in hopes of a prisoner exchange - and Hezbollah chief Hassan Nasrullah told Al Jazeera TV that he alerted the Lebanese government before kidnapping the soldiers.

Back in Washington, White House press secretary Tony Snow stressed that Rice speaks for the president: "There's no give on this. The United States believes in a sustainable cease-fire."

He said that means Hezbollah must return the two kidnapped Israeli soldiers and remove the rockets that have been shelling Israel.

"The sustainable cease-fire is one that is not going to enable Hezbollah to declare victory," Snow added, making clear that a cease-fire that leaves Hezbollah in place would do just that.

"You've got to keep in mind, the aggressor in this case is not Israel, it's Hezbollah," Snow added. "Hezbollah crossed over into Israeli territory and kidnapped two soldiers."

deborah.orin@nypost.com

-----------------

It seems that the Lebanese Parliment's speaker, Nabih Berri is a Hizbollah sympathizer and ally. It also seems that he is prepared to reject any cease-fire negotiations that don't include a prisoner exchange (presumably in the ratio of 100-1).

Here's my prisoner exchange plan: in exchange for releasing the Israeli kidnapees, Israeli will grant you your life, and that of your countrymen. Since Lebanon's total population is approximately 3,800,000, I'm sure that an exchange ratio of 1,900,000-1 will satisfy even Mr. Berri.

What say you?

Elliot

tomder55 answered on 07/25/06:

maybe Egeland heard the same type of criticism as when he flapped his gums over the charge that the US was stingy in providing releif aid.

I wonder how fast everyone would be crying for a cease-fire if Israel was being beaten down. I am not in favor of Condi being there right now and I am not in favor of a cease-fire until the point of Tehran's spear is sufficiently blunted .

If the world is looking for a proportional proposition here it is : Don't attack Israel ;don't kidnap her soldiers ;don't fire rockets full of ball bearings into her cities ;leave Israel alone to live in peace and in return Israel will let you live in peace (with an emphasis on live). Other than that Israel should use 'Chicago rules' .

"He pulls a knife on you, you pull a gun. He sends one of yours to the hospital, you send one of his to the morgue. That's the Chicago way."

Sean Connery to Kevin Costner (Eliot Ness) in "The Untouchables"

ETWolverine rated this answer Excellent or Above Average Answer
Itsdb rated this answer Excellent or Above Average Answer
jackreade rated this answer Excellent or Above Average Answer
powderpuff rated this answer Excellent or Above Average Answer

Question/Answer
paraclete asked on 07/25/06 - Multilateralism like multicultirism is dead?

World trade talks collapse

From: Agence France-Presse
By Jonathan Fowler in Geneva

July 25, 2006


A FIVE-year global effort to liberalise world trade has collapsed in acrimony, driving a stake into lofty ambitions to free global commerce and help developing nations to climb out of poverty.
The collapse of talks among six key players from the 149-nation World Trade Organisation drove its chief Pascal Lamy to recommend an indefinite freeze in the faltering negotiations.

Mr Lamy later said it was necessary to clear the air and ensure that there was "no ambiguity" about the lack of political will for a deal among key members of the WTO, which sets the rules of global commerce.

"The end of a time-out can only come when members are ready to play ball," he said.

Brazilian Foreign Minister Celso Amorim said that trading nations could face a long wait before negotiations can resume, because of unresolved differences on farm trade which have dogged the talks.

"I don't believe that it will be a matter of weeks. It will certainly be a matter of months, and not maybe very few months," he said.


Indian trade minister Kamal Nath had earlier commented: "The round is not dead. But it's definitely between intensive care and the crematorium."

The failure by Australia, Brazil, the European Union, India, Japan and the United States to settle their disagreements on trade concessions - despite getting close - was the latest in a series of setbacks to attempts to conclude WTO's Doha Round talks.

"We were in a range that, while it was not small, was reachable," Mr Amorim said.

The Doha Round, which started in the Qatari capital in 2001, was billed as a move to complete the unfinished business of previous decades of reforms to make trade fairer, particularly in agricultural goods.

Launched just two months after the September 11 attacks in the US, it was also seen as a way to rebuild confidence in a world shaken by terrorism and as a once-in-a-generation chance to reduce stark divisions among rich and poor nations.

Experts have regularly warned that WTO members may shift their focus back to bilateral trade deals and regional trade accords, which can create a complex web of conflicting interests.

The multilateral WTO talks were seen by many as giving developing countries their best shot of a decent deal, because most poor nations tend to be at a disadvantage in bilateral trade negotiations with rich, powerful players.

"We've missed a very important opportunity to prove that multilateralism works, and that by working together - which is not easy to do - the nations of this world can solve some of the serious problems we face," Mr Lamy said.

The Brazilian and Indian-led G20 grouping of developing countries has increasingly flexed its muscle against the rich during the Doha Round negotiations.

Brussels and Washington had previously agreed that they wanted more concessions from the developing world on trade in services and manufactured goods in return for freeing up farm trade.

The latest failed talks, however, pitted the US against the EU and powerful emerging nations.

Washington found itself under pressure to offer deeper cuts in its contested farm subsidies but countered that its partners should themselves open markets by slashing customs duties on imported agricultural goods.

"The United States remains committed to a robust, ambitious and balanced round. Unfortunately our trading partners were more interested in the loopholes than in market access," US Trade Representative Susan Schwab said.

EU Trade Commissioner Peter Mandelson hit back, accusing Washington of failing to match moves by other WTO nations.

"The United States was unwilling or unable to show any flexibility on the issue of farm subsidies," Mr Mandelson said.

Critics say that Washington's subsidies skew trade in favour of US agribusiness.

But deeper cuts would have been difficult for Washington to sell domestically ahead of crucial mid-term elections in November, unless US negotiators could show they had obtained more from their counterparts.

Mr Nath blamed the uncompromising US stance.

"It's very clear that the EU made a movement (on customs duties). Everyone put something on the table except for one country, who said: 'We can't see anything on the table'," Mr Nath said.

Negotiators had hoped to hammer out a deal before it became even tougher to reach an agreement.

In July 2007, the White House is to lose its fast track authority for trade deals, allowing politicians to recover their right to pick apart international trade treaties, potentially creating new obstacles for any future talks at the WTO.

The Doha Round was meant to be completed with a trade deal in 2004. That target was later shifted to December 2006 because of persistent breakdowns.

tomder55 answered on 07/25/06:

I think it is unfortunate that the talks have stalled . I'd prefer a world wide trade mechanism than a collection of bilateral agreements and a multitude of trading blocks . Also consider the fact that agreements in the past have tinkered on the brink of collapse before a settlement was achieved .

I disagree with the author's reasoning that it is the intractible US position that is causing the stall of progress. There are many issues of contention besides farm subsidies (for the record I would gut US Agri-subsidies if I had my way...why should consumers pay more for propped up produce that can be purchased cheaper from an imported source). But ,the US did offer generous cuts in our subsidies( 60%) in exchange for EU concessions and it was the EU that balked .

In many ways the US has trade disadvantages . We are getting ripped for the agri-supports but emerging industrial nations get a pass for their protectionism of their industries ;Brazil and India come to mind . Another issue is the exchange rates of certain Asian currencies .

There is still time but it is running short . The US right now has a free-trade friendly President . Congress gave him a virtual blank check to negotiate this round with a Trade Promotion Authority that says Bush can come back with any trade agreement and Congress will not debate the particulars but would only vote it up or down . This authorization expires next June. After that ;trust me ;there are alot more representatives in Congress who lean toward protectionism .If the world continues to stall ,and Bush does not come back with a trade agreement before next June then it is probable that Doha process will collapse.

paraclete rated this answer Excellent or Above Average Answer

Question/Answer
ROLCAM asked on 07/24/06 - UN official: Israel action illegal !!


THE United Nations' top humanitarian official yesterday accused Israel of violating international law, as at least ten more civilians died on both sides of the Lebanese border and diplomatic efforts to end the conflict intensified.

Do you consider it illegal ?

tomder55 answered on 07/24/06:

I consider the UN a joke . The history of the U.N. is a history of failure especially in areas of global peace and security.All you need as an example of the UN is where they arguably have claimed recent success......East Timor .Somalia, Rwanda, Sierra Leone, Liberia, Congo where peacekeepers reportedly raped the local girls, and Darfur are examples of typical U.N. peacekeeping failures .

In 2000, blue helmets videotaped Hezbollah kidnapping three Israeli soldiers . The video could have been useful in rescuing the soldiers. But, for eight months, the U.N. troops angrily denied even having the tape. When forced to admit they did, they refused to release it because that might compromise their "neutrality."

The U.N. Commission on Human Rights is a protective shield for tyrants.The world's worst offenders on human rights are more likely to be members of the UNCHR than to be condemned by it.

Jan Egeland is the same a**h*le in the UN who said the US was stingy when it came to giving humanitarian aid . Has the schmuck visited Haifa in the last week ? So much for an imparital analysis.War crimes should be charged to Hezbollah for hiding among civilians if any are charged .

ETWolverine rated this answer Excellent or Above Average Answer
Itsdb rated this answer Excellent or Above Average Answer
jackreade rated this answer Excellent or Above Average Answer
powderpuff rated this answer Excellent or Above Average Answer
ROLCAM rated this answer Excellent or Above Average Answer

Question/Answer
ROLCAM asked on 07/24/06 - Back to the OLD DAYS :

Please see a back to the old days maxim:-
DIVIDE AND CONQUER.

see:- http://www.nytimes.com/2006/07/23/washington/23diplo.html?th&emc=th

Any reason why the U S A should be putting
their NOSE where it is not really required?

tomder55 answered on 07/24/06:

First everyone complains that the US is not doing enough . Now you say we are putting our noses where they don't belong. Which is it ?

As to the point of the slimes article : There is a chance Bashar al-Assad could be peeled away from his Iranian alliance because he is a sniveling coward . If possible then Iran would be pretty much isolated from any Arab support .In the Arab world Iran would stand alone .

The thugs in Tehran would still have N.Korean weapons to purchase and would still have the support of nations who are hostage to their oil supply...and of course Russia which cannot decide which is more important ...combatting Jihadistan expansion or creating a check to so-called American hegemony. It must thrill the Ruskies that Ahmadamadingdong is making overtures with Turkmenistan and other former Soviet muslim satellites countries or for that matter that the loony mullahs will have nukes and delivery systems that can hit main land Russia.


.

Itsdb rated this answer Excellent or Above Average Answer
ROLCAM rated this answer Excellent or Above Average Answer

Question/Answer
ROLCAM asked on 07/23/06 - ISRAEL'S RESPONSES !!

Comments about the latest happenings :-

EXAMPLE 1.

"If I punch you in the face, you have no right to shoot me with a revolver."

eXAMPLE 2.

"It was not a case of an eye for an eye or a tooth for a tooth. In this case we have ten eyes for an eye and ten teeth for a tooth."

rolcam

tomder55 answered on 07/23/06:

A proportional response to a barrage of rockets fired indescriminently at a population center would be to indescriminently fire the same number of rockets at innocent civilans . Israel has been very careful about who and where they target . You ought to give this issue up . Your arguemnt is not logical.

Esculapiusiam rated this answer Excellent or Above Average Answer
imatease rated this answer Excellent or Above Average Answer
Itsdb rated this answer Excellent or Above Average Answer
powderpuff rated this answer Excellent or Above Average Answer
ROLCAM rated this answer Bad/Wrong Answer

Question/Answer
ROLCAM asked on 07/23/06 - Lebanon: pity the nation !!


There seems to be no end in sight to the war in Lebanon. The situation seems to be getting worse by the day. I have yet to meet any level-headed analyst or observer who believes that Israel's response was not disproportionate.

The whole issue is intolerable.

rolcam

tomder55 answered on 07/23/06:

I imagine you would think differenty if the Kassam rockets were leveling your house .You would more likely demand that your gvt. do anything necessary to make sure you are NEVER threatened in that manner again . I laugh at the term disproportionate.The theory behind it is preposterous . Australian history is littered with massacres against Aboriginal people .Were they fought against in a proportional manner ?

dimwit rated this answer Excellent or Above Average Answer
Itsdb rated this answer Excellent or Above Average Answer
jackreade rated this answer Excellent or Above Average Answer
margie rated this answer Excellent or Above Average Answer
ROLCAM rated this answer Bad/Wrong Answer

Question/Answer
ROLCAM asked on 07/23/06 - Your View ??

Are Israel's military tactics justified?

tomder55 answered on 07/23/06:

no ;they are far too restrained

dimwit rated this answer Excellent or Above Average Answer
ETWolverine rated this answer Excellent or Above Average Answer
Itsdb rated this answer Excellent or Above Average Answer
margie rated this answer Excellent or Above Average Answer
paraclete rated this answer Excellent or Above Average Answer
powderpuff rated this answer Excellent or Above Average Answer
ROLCAM rated this answer Average Answer

Question/Answer
jackreade asked on 07/22/06 - Afghanistan Close to Anarchy

"The most senior British military commander in Afghanistan yesterday described the situation in the country as "close to anarchy" with feuding foreign agencies and unethical private security companies compounding problems caused by local corruption.

The stark warning came from Lieutenant General David Richards, head of Nato's international security force in Afghanistan, who warned that western forces there were short of equipment and were "running out of time" if they were going to meet the expectations of the Afghan people.

The assumption within Nato countries had been that the environment in Afghanistan after the defeat of the Taliban in 2002 would be benign, Gen Richards said. "That is clearly not the case," he said yesterday. He referred to disputes between tribes crossing the border with Pakistan, and divisions between religious and secular factions cynically manipulated by "anarcho-warlords".

Corrupt local officials were fuelling the problem and Nato's provincial reconstruction teams in Afghanistan were sending out conflicting signals, Gen Richards told a conference at the Royal United Services Institute in London. "The situation is close to anarchy," he said, referring in particular to what he called "the lack of unity between different agencies".

He described "poorly regulated private security companies" as unethical and "all too ready to discharge firearms". Nato forces in Afghanistan were short of equipment, notably aircraft, but also of medical evacuation systems and life-saving equipment.

Officials said later that France and Turkey had sent troops to Kabul but without any helicopters to support them.

Gen Richards will also take command of the 4,500-strong British brigade in Helmand province at the heart of the hostile, poppy-growing south of the country when it comes under Nato's overall authority. He said yesterday that Nato "could not afford not to succeed" in its attempt to bring long-term stability to Afghanistan and build up the country's national army and security forces. He described the mission as a watershed for Nato, taking on "land combat operations for the first time in its history".

The picture Gen Richards painted yesterday contrasted markedly with optimistic comments by ministers when they agreed earlier this month to send reinforcements to southern Afghanistan at the request of British commanders there. Many of those will be engineers with the task of appealing to Afghan "hearts and minds" by repairing the infrastructure, including irrigation systems.

Gen Richards said yesterday that was a priority. How to eradicate opium poppies - an issue repeatedly highlighted by ministers - was a problem that could only be tackled later....." guardian dot com dot uk

~~~~~~~~~~~~~~~~~~~~~~~~~~~~~~~~~~~~~~~~~~~~~


Comments???

tomder55 answered on 07/23/06:

With all due respect to our allies in NATO ;they appear to think they are there as 'peace keepers ' instead of war fighters . There is still an active enemy there to engage. But that being said Afghanistan is far form being in a state of anarchy.The Taliban has been forming into groups large enough to attack a town like they have done every spring and summer since we invaded. These groups are being caught in the open and slaughtered.When they have had successes they have been quicky reversed.They demonstrate no capability to control anything, and merely threaten and intimidate innocent Afghan civilians, then run away.
The Taliban is increasingly dependent on outsiders (from areas inside Pakistan). The people of Afghanistan won't willingly join them.

Itsdb rated this answer Excellent or Above Average Answer
jackreade rated this answer Excellent or Above Average Answer

Question/Answer
jackreade asked on 07/22/06 - Our War with Iran being fought through Surrogates

"Ted Koppel has one of his semi-regular op-eds today in the New York Times, except it is less op-ed than a reported piece based on a conversation Koppel had with a senior Jordanian intelligence official, who warned Koppel about Iran's growing power in the Middle East. "The United States is already at war with Iran," Koppel begins. "But for the time being the battle is being fought through surrogates."

Koppel goes on to note that "over the past couple of months alone, he told me, Hamas has received more than $300 million in cash, provided by Iran and funneled through Syria" and "the more than 12,000 missiles and rockets...in Hezbollah's arsenal were largely provided by Iran." Here's the important passage:

When Sheik Qaouk talked about Israel and Hezbollah, his organization's ambitions were not framed in purely defensive terms. There is only harmony between Hezbollah's endgame and the more provocative statements made over the past year by Mahmoud Ahmadinejad, Iran's president. Both foresee the elimination of the Jewish state.

Are the Israelis over-reacting in Lebanon? Perhaps they simply perceive their enemies' intentions with greater clarity than most. It is not the Lebanese who make the Israelis nervous, nor even Hezbollah. It is the puppet-masters in Tehran capitalizing on every opportunity that democratic reform presents. In the Palestinian territories, in Lebanon, in Egypt, should President Hosni Mubarak be so incautious as to hold a free election, it is the Islamists who benefit the most.

But Washington's greatest gift to the Iranians lies next door in Iraq......and so on...." Article on the Huffington Post Blog.

~~~~~~~~~~~~~~~~~~~~~~~~~~~~~~~~~~~~~~~~~~~~~~~~


It is not WWIII, it is a War with Iran???

Comments....

tomder55 answered on 07/23/06:

we have been in a shooting war with Iranian and Syrian regulars since we entered Iraq .Nothing surrogate about it . Koppel is correct in pointing out that the recent attacks on Israel are directed from Tehran and serve their strategic interests .

it is very much a global war. Just ask the Indian commuters of Bollywood .

jackreade rated this answer Excellent or Above Average Answer

Question/Answer
ROLCAM asked on 07/22/06 - I AM VERY DISAPPOINTED !!

This statement is very disappointing:-

QUOTATION OF THE DAY N Y TIMES. July, 22 2006.

"What I wont do is go to some place and try to get a cease-fire that I know isnt going to last."
CONDOLEEZZA RICE, the secretary of state, on the fighting in Lebanon.

Are you disappointed??

tomder55 answered on 07/23/06:

In other posts I've argued that Condi should not go at all . I instead would send Chairman of the Joint Chiefs of Staff ,General Peter Pace. Let him set up a tent and have all the interested parties come to him and he could tell them precisely what the US considers a solution to the crisis .

Itsdb rated this answer Excellent or Above Average Answer
ROLCAM rated this answer Excellent or Above Average Answer

Question/Answer
ROLCAM asked on 07/22/06 - THE BIG BULLY !!

This is exactly what I meant about the BIG BULLY !!

SEE:- http://www.nytimes.com/2006/07/22/world/middleeast/22military.html?_r=1&th&emc=th&oref=slogin

Honest members should judge for themselves.

rolcam.

tomder55 answered on 07/23/06:

As long as the Israelis are taking out Islamic terrorists, we should be doing everything in our power to assist in their noble efforts.I hope they are big a** bunker buster bombs ;And maybe a little napalm for the hard to reach places.We are probably delivering them in C-5s ;I'd prefer we deliver them in B-2s if you know what I mean.

This is what Bush meant when asking Israel to exercise restraint. Restraint from our point of view is precision bombing vs. carpet bombing. When you ask for Israel to exercise restraint, you mean you want Israel to roll over and die.

Guess when the US shipped massive armaments to England and Russia in WWII we are also being BIG BULLIES .

Guess we were acting the big bully when the US and Australia took back the Solomon Islands from the Japanese in WWII; and that our first objective in the Pacific Theater was the defense of Australia. When the Japanese made an attempt from the sea to take Port Moresby the US successfully countered the attempt during the battle of the Coral Sea.The total US casualties just to take Guadalcanal in the Solomons was 6000 in a 6 month campaign. The constant need to reinforce Guadalcanal weakened the Japanese effort in other theatres, leading to successful Australian counteroffensive in New Guinea, which culminated in the capture of Buna and Gona .

The New York Slimes cannot be stupid enough to think that Condi's agenda is rigid . Diplomacy will take her wherever she thinks will do the most good .

Itsdb rated this answer Excellent or Above Average Answer
ROLCAM rated this answer Bad/Wrong Answer

Question/Answer
paraclete asked on 07/22/06 - how do you kill an idea?

In order to prevail over Hezbollah, Israel must not only kill the terrorists, it must replace the idea that Hezbollah is good, an idea they have entrenched in Lebanon.

Immoveable foes are playing for keeps


All-out attack south Beirut's Beir al-Abed residential district has taken a pounding from Israeli bombardment.
Photo: AFP
July 22, 2006

The Israelis are destroying Lebanon's cities, but can they destroy Hezbollah and its supporters, asks Paul McGeough in Beirut.

AS IF by necessity, life in the Lebanese capital goes on in a terrified sort of way - drive past a school and it might not seem like a refugee camp; see all heads turn to the sky as Israeli jets growl overhead, and the pavement crowds could be responding to the first drops of a surprise shower of rain.

The Lebanese Prime Minister, Fouad Siniora, lamented that his country "has been torn to shreds". But it is not till you nose into the once-crowded slums of southern Beirut that utter desolation and destruction come to meet you like ghosts from all of this region's wars.

Huge slabs of freeway overpasses are missing - like slices gone from a cake. And Israeli bomber pilots, who fly so high they can't be seen, use United States-supplied guidance systems to gouge massive craters in the centre of key intersections.

But at times in the first 10 days of this war, Israel's targeting has been bizarre. On Tuesday it bombed two water-drilling rigs in Achrafieh, a Christian enclave - presumably they were mistaken for missile-launchers. A fire engine outside a hospital in the southern suburbs suffered the same fate.

There seemed to be an open verdict on a spectacular strike in Bourj al-Barajneh, deep in the city's south, on Wednesday night, when the Israelis dropped more than 20 tonnes of munitions on what they claimed was a Hezbollah bunker. But locals insisted the raw concrete hulk was a mosque under construction.

Edgy Hezbollah street wardens relented and allowed the Herald to investigate - despite its collapsed state, arches shaped into the concrete and two great halls on what would have been the first and second floors suggested the mound of rubble might have been a mosque; but it was impossible to tell what the basement might have been used for because it had been disemboweled by the massive force of several bunker-busters.

The Israelis' bomb delivery has an unnerving, through-the-letterbox precision. Like last week's attack on Hezbollah's nearby headquarters, the mosque-bunker was demolished with minimal damage to nearby high-rise apartment blocks.

But the utter destruction of block after block in the Beir el-Abed quarter of south Beirut was no accident - this had to be part of what the Israelis boast is considered strategic planning and target verification. It was deliberate.

Much of the imagery in this war has been stand-alone targets - a building, a bridge, a truck. But on Thursday, Hezbollah officials eased their stranglehold on southern Beirut to take foreign reporters into a desolate moonscape that 10 days ago was home to thousands of impoverished families.

Now, street after street, block after block on which dozens of 12- and 15-storey apartment buildings had stood have been smashed - either pan-caked or teetering. Some were stripped of whole facades to reveal the mangled interiors of their owners' lives.

High on a fractured apartment block, a clothes dryer defied gravity as it hung on a ledge. Lower down, a fridge, gas cylinders, plastic chairs and bedding poured into the street in a tangle of rubble.

Shredded bedsheets and clothing were snarled in a mess of exposed steel reinforcing; a building which had housed a street-level supermarket still smouldered; boxed shirts from a tailor's shop were strewn on the pavement.

The only sign of life was an angry 40-ish man. Pointing to the flattened building in which he had lived, he yelled: "We will build and rebuild for the sake of [Hezbollah leader] Hassan Nasrallah. If he asks for our homes, our wives and our children, we will sacrifice them."

The entire neighbourhood has fled, joining an internal refugee flood which United Nations officials estimate to be a humanitarian crisis for as many as half a million civilians.

The media's Hezbollah guide, Hussein Naboulsi, said: "The people who lived here left really early - those who didn't are under the rubble." Asked about Israeli claims that Hezbollah sheltered its forces and facilities in civilian communities, he snapped: "The military is on the border. This has nothing to do with the kidnapped soldiers. We have kidnapped them in the past and Israel has negotiated directly for their release. This war is a plot against Lebanon and revenge on Hezbollah because we are the only force that has defeated them."

In the shifting sands of Middle East politics, Hezbollah is a government within a government in Lebanon. In the southern border region and in southern Beirut, it provides community services and names streets after its heroes - like Iran's Ayatollah Khomeini or Hadi, Nasrallah's son who was killed in battle.

It runs a TV station, which was off the air for only an hour after an Israeli strike on its bunker. And it runs an army that killed more than 1000 Israelis during 18 years of resisting the Israeli occupation of south Lebanon.

But with Hezbollah as a pawn of Iran and Syria, Israel's declaration that it will turn Lebanon's clock back 20 years - which amounts to a promise of revisiting the carnage of its own civil war - and Hezbollah's determination to fight to the end, this has become a frightening clash in a part of the world which is at its most unstable in more than a decade.

Despite UN and European demands for a ceasefire, the US and Britain have stepped back, giving Israel time to attempt to achieve a growing and implacably held list of objectives by delaying any diplomatic intervention. With the US Secretary of State, Condoleezza Rice, not due in the region till late next week, it could be another fortnight or more before Israel might back off - if at all.

AT THE heart of this crisis is a struggle for the regional balance of power between the US-backed Israel and Iran. On another layer is the fear of the predominantly Sunni Arab leaders that they are being sidelined by a new "Shiite crescent" from Beirut to Tehran.

Despite challenges to their credibility from public anger in their respective countries over the Israeli campaign in Lebanon, their fears would have only risen this week, when Iraq's new Shiite Prime Minister, Nuri al-Maliki, led a chorus of Iraqi condemnation of Israel.

And while Maliki's declaration might have been troubling for the Arab leaders, he had taken a deliberate step into the Iranian tent, which amounted to a huge rebuff for the US which, since its 2003 invasion of Iraq, has banked on Baghdad being its newest best friend in the region.

As the Sunni Arab leaders seem to side with Israel, they are being mocked at home. In Cairo, Al Dustoor, an opposition weekly newspaper, lampooned President Hosni Mubarak, contrasting him with Hezbollah leader Hassan Nasrallah. Contrasting the 1997 death in battle against the Israelis of Nasrallah's son with Mubarak's nepotism in positioning his son to replace him, Al Dustoor headlined its critique: "The difference between a leader who offers his son as a martyr and a leader who offers his son as a successor!"

In the Arab-language Al Hayat, the Saudi commentator Dawood al-Shiryan wrote: "Hezbollah's step and that taken by Hamas before it lacks political wisdom. But to insist on calling the resistance to account for this mistake now that Israel's violent response has been launched has created a political reality that is difficult to describe."

The American understanding of the regional dynamic also is under attack. UN chief Kofi Annan's deputy, Mark Malloch Brown, ventured this week: "The Middle East is littered with the results of people believing there are military solutions to political problems in the region."

By the rules of guerilla war, Hezbollah doesn't have to win to win this contest; it only has to remain standing. But Israel does have to win - hence its determination to exterminate Hezbollah. That means a US-sanctioned continuation - or escalation - even though its inability to staunch the ferocity of Hezbollah's attacks raised the prospect of its forces being lured into an ugly ground war in Lebanon.

Despite battling on two fronts - Gaza and Lebanon - Israelis overwhelmingly support their Government's belief that they can, or must, destroy Hezbollah. On the other hand, Hezbollah is confident that international condemnation of the civilian death and destruction in Lebanon will force a retreat into diplomacy.

If Hezbollah lives to fight another day, it will have won the day, not only in the eyes of its own regional supporters, who will cheer its prowess in withstanding the Israeli onslaught, but also and unacceptably in the eyes of the Israeli military command.

Israel will therefore escalate its operations before it allows the diplomatic process to take over. And therein lies a post-September 11, 2001, challenge that is rooted in a neo-conservative global outlook - as Washington's UN envoy, John Bolton, said this week: "It is very hard to understand how you have a ceasefire with a terrorist organisation like Hezbollah."

Critiques this week echoed warnings in 2003 that the invasion of Iraq risked a dangerous distraction from the Israel-Palestine crisis. That distraction now threatens US efforts to reshape the region - not just by creating a democratic beachhead in Iraq, but also by blunting Iran's nuclear ambitions.

A lone critical voice in Israel was the Middle East Media Research Institute's director, Yigal Carmon. In the Haaretz newspaper, he said: "We are witnessing a most serious failure of our leaders. They allowed the state of Israel to fall into the Iranian trap by responding to Iran's provocation [through Hezbollah's abduction of the two soldiers].

"It was intended to disrupt the discussions of the G8 that were supposed to form an international consensus against the Iranian nuclear program. A responsible leadership would have delayed the response by several weeks, and not played into the Iranians' hands. The public is not stupid - it would have understood that a threat to 4 million people as a result of the Iranian nuclear program is more serious than the killing of soldiers in the north and the kidnapping of their comrades."

In a web briefing on the crisis, Anthony Cordesman, of the Centre for Strategic and International Studies in Washington, cast a spectre of deep gloom: "The regional impacts will be deeply negative Israeli attacks on Lebanon will further alienate Arabs and Muslims throughout the world.

"The same Arab and Muslim satellite TV stations and other media that portray Israel as 'guilty' and Hezbollah and Hamas as 'freedom fighters', will be showing US and British forces in Iraq as attacking Iraqis and guilty of crimes and atrocities.

"For many ordinary Arabs and Muslims, there will be a clear linkage between the Israeli-Palestinian conflict, Lebanon conflict, war in Iraq, war in Afghanistan and the war on terrorism. They will see at least the US, and perhaps the West as a whole, as anti-Arab and anti-Muslim."

Warning that the voices of anger would drown out those of peace, his best prediction was more of the same-old, same-old, rather than a regional explosion: "[It's] going to be a festering war of attrition where half-measures to resolve the conflict have no lasting meaning. It will be a further darkening of the security situation in the entire Middle East, new divisions in the West, and a dangerous spill-over of this conflict into the US and British effort to bring stability to Iraq and every other aspect of the war on terrorism."

That's all big-picture stuff. But in the little-picture world of Beir el-Abed in south Beirut this week, anger and resentment seemed to confirm it all.

When the Herald chanced upon a local in the city refuge, 45-year-old Hala Adeib was bent on revenge. Recalling killings and wars right back to the 1947 founding of Israel, she declared: "God is with us. Let them blow up our buildings - it only makes us stronger. I have raised two children alone and I'm willing to give them to the resistance. They have been raised on the principles of Hezbollah."

tomder55 answered on 07/22/06:

Any success that Hezbollah has in Lebanon comes from them providing basic services that the weak Lebanese Gvt. fails to do . But the support for them is being overplayed in this article and in most Western press. Where are the mass demonstrations in the ummah that should've followed this if the street was behind Hezbollah? They worked themselves into a frezy over a couple of cartoons but nothing while Israel bombs "innocents " .

paraclete rated this answer Excellent or Above Average Answer

Question/Answer
Itsdb asked on 07/21/06 - Ahmadinejad letter to Merkel...

"It's rather weird"

Iran leader asks Merkel for help on Zionism -German official

20 Jul 2006 17:37:49 GMT
Source: Reuters
By Louis Charbonneau

BERLIN, July 20 (Reuters) - A letter written by Iranian President Mahmoud Ahmadinejad to German Chancellor Angela Merkel asks her to help solve the Palestinian problem and deal with Zionism, a German government official said on Thursday.

"There's nothing about the nuclear issue (in the letter)," the official told Reuters on condition of anonymity due to the extreme sensitivity of the issue for the German government.

"It's all related to Germany and how we have to find a solution to the Palestinian problems and Zionism and so on. It's rather weird," the official, who has seen the letter, said.

Iranian students news agency said on Wednesday that Ahmadinejad had written to Merkel, but until Thursday officials had not spoken about the contents.

Zionism is a political movement that supports a homeland for the Jewish people in Palestine, now the state of Israel. The fate of Palestinian Arab refugees is one of the world's largest and most long-lasting refugee problems.

Berlin's relations with Ahmadinejad have been complicated by his denial of the Holocaust, in which Germany's Nazi regime killed six million Jews, and his call for Israel to be wiped off the map.

Holocaust denial is a crime in Germany punishable with up to five years in prison.

"It's extremely touchy (for the German government)," said the official, adding that the government did not yet know if or how it would respond. "There are a lot of propaganda phrases about Israel and the Jews inside."

In May Ahmadinejad wrote U.S. President George W. Bush an 18-page letter discussing religious values, history and international relations.

In it, he took swipes at Israel and at the United States.

He sharply criticized Bush on many fronts, implying that the invasions of Afghanistan and Iraq, abuses of detainees in U.S. prisons in Guantanamo Bay and Abu Ghraib -- and his staunch support for Israel -- were somehow inconsistent with Bush's Christian beliefs.

But the letter to Merkel was different and was not confrontational in tone, the official said.

"It's not negative like Ahmadinejad's letter to Bush. He is not criticising Germany," he said. "It's basically about how we have to work together and solve the problems of the world together."

In February, Merkel compared Ahmadinejad's statements and stance to Adolf Hitler's rise to power when he and his Nazi party began threatening to exterminate European Jewry.

"Remember that in 1933 many people said it was just rhetoric," Merkel said.

The German official said it was interesting that the letter did not discuss Iran's nuclear standoff with the United States, European Union and other countries.

Iran is facing possible action at the U.N. Security Council over suspicions that it is developing nuclear arms. Tehran denies the charge, saying it is working on nuclear fuel only to run power stations."

~~~~~~~~~~~~~~~~~~~~~~~~~~~~~~~~~~~~~~~~~~~~~~~~~~~~~~~

I think 'rather weird' is an understatement.

Comments?

tomder55 answered on 07/21/06:

remember . Mohammed gave his minions specific instructions to warn their adversary and offer them a way out before they strike at them. I would see this letter in the same context as the one he sent Bush offering us a way out if only we would willing adopt our role as dhimmi .

Perhaps the Mahdi Hatter thinks Merkel is a kindred of Adolph but so far I have been impressed with her .

"Remember that in 1933 many people said it was just rhetoric," Merkel said.

exactly

Itsdb rated this answer Excellent or Above Average Answer

Question/Answer
ROLCAM asked on 07/21/06 - Is this really realistic ??

jackread one of our esteemed members suggests in
one of his posts:-

" Wipe them off the face of the earth".

He is actually referring to both of the warring parties.

Who is going to do the wiping off?

Annihilate each other?

Some mighty other party/parties to do it for them?

Is this really realistic??

rolcam.

tomder55 answered on 07/21/06:

sometimes total victory is the only option

ROLCAM rated this answer Excellent or Above Average Answer

Question/Answer
ROLCAM asked on 07/21/06 - All that is over now.??


For 2 soldiers captured and 29 Israeli's dead, 309 persons in Lebanon has to die.
For 1 soldier captured, more than 60 Palestinians has to die.

Compared with their former stance this violence is excessively disproportionate.

rolcam.

tomder55 answered on 07/21/06:

so what you are saying is that if a gvt. finances a terrorist and the terrorist goes to a pizza parlor and blows himself up with 20 innocents that the proportional response would be for Israel to attack and kill 20 of their citizens ?


The equivalent would be the Normandy Invasion in WWII . To prep. the battle field so that the French could be liberated we sometimes had to bomb German targets inside densely populated civilian areas .


For the record. Not one Israeli bomb is directly targetting ANY civilian . The fact that civilians are being killed in Lebanon can be attributed directly to the tactics of the terrorists who hide and conduct operations behind the skirts and the cover of the civilian population. Hezbollah is holding the Lebanese people hostage .

dimwit rated this answer Excellent or Above Average Answer
ETWolverine rated this answer Excellent or Above Average Answer
Itsdb rated this answer Excellent or Above Average Answer
margie rated this answer Excellent or Above Average Answer
ROLCAM rated this answer Excellent or Above Average Answer

Question/Answer
jackreade asked on 07/20/06 - "Survivor Lebanon"

"Bush greenlights Israeli bombing of Lebanon with 25,000 Americans in-country and no plan for evacuation. Just like what happened when Katrina struck New Orleans, once again Amercans are left on their own while Republican government pursues a hidden ideological agenda and protects the interests of a select few." Dave Johnson

~~~~~~~~~~~~~~~~~~~~~~~~~~~~~~~~~~~~~~~~~~~~~~~~~~


Comments?

tomder55 answered on 07/21/06:

The assumption in the comment is that Israel can do nothing in it's self interest without getting the ok from the US . That assumption is patently false. The attack Israel came as a suprise to everyone nad Israel had to act and act fast and hard to the provocation .

Of the 25,000 Americans there only around 8000 have requested evacuation . Consider the logistics. Most other expatriots from other countries are evacuationg through land routes via the Syrian border . That is obviously not an option . A sea and air evacuation is the only method avialable . That takes time and is extremely dangerous.

Just like those living in Lebanon today ,we were informed by the Statye Dept.when we lived in Iran that we were living in a dangerous place and there were no guarantees for our safety . We took the risk anyway for the experience and a good paying job. We had our own evacuation plans in place as a contingency in addition to the eventuality that the American Embassy could not help us.
(it would've reqired a lenghty walk to the Turkish border .) For that matter using Johnson's comparison the residence of N.O. knew that in he event of a hurricane they may need to take some independent action to provide for their own safety .

This again illustrates the diffence between self relying conservatism and putting faith in the nanny-state liberalism.

Dave Johnson ;who ever the hell he is ;has not thought this through or is so blinded by Bush hatred that he is not thinking clearly . That must make him a Kossack.

ETWolverine rated this answer Excellent or Above Average Answer
Itsdb rated this answer Excellent or Above Average Answer
jackreade rated this answer Average Answer

Question/Answer
sissypants asked on 07/19/06 - veto

what is your opinion on stem cell research?

tomder55 answered on 07/20/06:

I have a problem with the fact that there are 10s of thousands of these embryo's stored unused at fertility clinics . What is going on with that ? Creating these embroyos without using them in an attempt to create a human is the most immoral aspect of this whole debate . After that ,it is a rationalization that "since they are sitting around unused they might as well be used for research " .Very weak if you ask me . They should not have been created in the first place . Where I see this leading to is the farming of embryos for the purpose of research and body part manufacturing .

The rest of my moral objections I will leave for another board .

The President's veto of this does not end stem cell research in this country .It is not illegal . All it did was prevent Federal funding of the research . If it is as valuable and promising as everyone claims it is then private enterprise will be more than able to fund and profit from the research .

From what I understand it ,embryonic stem cells are a pain in the butt to work with because the scientists do not know which direction they want to grow into . Adult stem cells are more stable and thus more promising . adult stem cells have been dramatically successful in improving some 72 medical conditions, including various forms of leukemia, juvenile arthritis, corneal regeneration, spinal cord injury, sickle cell anemia and liver cirrhosis.
The total number of benefits to human patients using embryonic stem cells is zero.

If adult stem cells can meet the need of curing these illnesses that eveyone wants cured then why even get into this morally questionable science ?

For that matter there is an abundance of stem cells in umbilical cord blood,and amniotic fluid ;and these are throw away parts anyway .

From a politics point of view ,I want my tax money going to the most promising research and that is in adult stem cells .

I am one who is very pro-science research . But I do not like being sold a bill of goods . For all the trumpetting recently of the birthday celebration of 'Dolly ',science is awefully quiet about the fate of the kindered of Dolly . How many failures have been tossed away ? How many revelations of bogus research have been recently revealed ? Tends to make me a skeptic and less willing to support such research . I believe that many of the claims are pie in the sky wishful thinking .

sissypants rated this answer Excellent or Above Average Answer

Question/Answer
ROLCAM asked on 07/20/06 - FALSE PROPORTIONALITY !!

The actual situation is much worse than as reported in mainstream newspapers. Here are some facts:

Israel's ongoing siege of the Gaza Strip is producing a dire shortage of water, food, medicine, and electricity - a form of collective punishment.

Israel is deliberately destroying the civilian infrastructure in Gaza - a form of collective punishment.

Israel destroyed Gaza's only power plant with the result that in Gaza's main hospitals, infant incubators, dialysis, and oxygen machines stopped running - a form of collective punishment.

Israeli fighter pilots are causing supersonic booms that terrorise the population - a form of collective punishment.

Children are psychologically harmed by the systematic, around the clock use of sonic booms - resulting in panic attacks, sleep disturbances, bedwetting and other detrimental effects - a form of collective punishment.

Since the bombing of the power station, treatment plants cannot pump and treat sewage in Gaza - a form of collective punishment.

Sonic booms are leading to spontaneous abortions (miscarriages) and premature births - a form of collective punishment.

Israeli Prime Minister Ehud Olmert's tactic is to "apply pressure" to the civilian population of Gaza. Collective punishment violates the Hague Convention as well as the Fourth Geneva Convention and constitutes a war crime.

"No general penalty, pecuniary or otherwise, shall be inflicted upon the population on account of the acts of individuals for which they cannot be regarded as jointly and severally responsible" - article 50 of the Hague Convention.

"No protected person may be punished for an offence he or she has not personally committed" - article 33, Fourth Geneva Convention.

These war crimes are being committed with weapons supplied by the United States, including fighter planes manufactured by Lockheed Martin and Boeing.

The use of US-made weapons to target civilians violates the US Arms Export Control Act as well as the Geneva Conventions.

The sonic boom raids are a clear violation of the Convention on the Rights of the Child, to which Israel is a signatory.

"In accordance with their obligations under international humanitarian law to protect the civilian population in armed conflicts, states parties shall take all feasible measures to ensure protection and care of children who are affected by an armed conflict" - article 38, Convention on the Rights of the Child.

Israel has a responsibility to protect its population from armed attack. Equally all civilians - whether in Palestine, Israel, or Lebanon - warrant protection.

tomder55 answered on 07/20/06:

I just love it when the world holds Israel to standards that no other warring country adheres to let alone the terrorists organization they are currently battling . What does your precious international laws say about kidnapping soldiers ?

In one case you have Hezzbollah ,using rockets designed by the Ruskies to saturate a battle field without discrimination ,on civilian targets . On the other you have Israel using precision bombing on military targets that happen to be hidden in civilian areas .

I leave you to judge the comparitive morality but I would suggest in both cases it is Hezbollah that is being immoral ;for targetting civilians ,and for putting their own non-combatants at risk .

ETWolverine rated this answer Excellent or Above Average Answer
Itsdb rated this answer Excellent or Above Average Answer
ROLCAM rated this answer Excellent or Above Average Answer

Question/Answer
jackreade asked on 07/20/06 - Iraq Reality verses Bush Administration Fantasy

"After the last three years, I didn't think I could be surprised by the level of cluelessness exhibited by the Bush administration when it comes to Iraq. Then I picked up this morning's New York Times.

But before we get to the jaw-dropping money quote that leaves no doubt the Bushies continue to view Iraq through zealots' eyes, let's start with a bracing shot of reality: the United Nations report on Iraq [pdf] released yesterday.
It paints a devastating -- and wrenchingly specific -- portrait of a country in bloody chaos.

First the numbers: 14,338 civilians killed in the first half of 2006. And, according to the UN report, civilian carnage is on "an upward trend," with more than 5,800 deaths and 5,700 injuries in the last two months. Indeed, on average, more than 100 Iraqi civilians were killed per day in June -- the highest monthly total since U.S. forces took control of Baghdad.

The report also puts a human face on those numbers and on the rampant violence raging in the country. It offers examples of homosexuals who have been targeted by militias and death squads because of their sexual orientation. And it's not just gays facing intolerance. The UN cites the case of an Iraqi tennis coach and two of his players who were gunned down in Baghdad because... they were wearing shorts! Forget the fashion police; we're talking fashion assassins. Others were targeted because their hair styles or facial hair didn't conform to the rules of the extremists now in control on both sides of the sectarian divide.

And according to the report, women have lost many of the freedoms they used to enjoy. In parts of Baghdad, they "are now prevented from going to the markets alone." They've also been warned not to drive cars and have been harassed for wearing pants. What's more, a new regulation dictates that women wishing to apply for a passport or travel abroad must be accompanied by their husbands or another male member of their family. And not wearing a headscarf can now mean being targeted for attack -- all the more troubling for the tens of thousands of non-Muslim Iraqi women.

The report also details kidnappings and acts of violence directed at children, including the chilling tale of a 12 year old boy named Osama who was kidnapped, raped, and grotesquely murdered -- hanged by his own clothing -- even though his family paid a $30,000 ransom.

This report is dripping in blood and suffering -- a stark reminder of the turmoil our failed invasion of Iraq has wrought. A point driven home yet again by Tuesday's suicide bombing in Kufa that saw at least 57 killed and 105 wounded when a man, on the pretext of offering work, drew a crowd of day laborers to his vehicle then blew them all up.

Which brings us back to the Bush administration's refusal to allow facts to interfere with its Iraq delusions. The latest proof of this deadly disconnect from reality comes courtesy of U.S. energy secretary Sam Bodman, and can be found buried near the end of the New York Times' story on the UN report.

According to the Times, Bodman, who is in Baghdad meeting with Iraq's oil and electricity ministers, "had a rosy view of progress here since his last visit in 2003."

Here is what Bodman told the Times "in an interview in the fortified Green Zone": "The situation seems far more stable than when I was here two or three years ago. The security seems better, people are more relaxed. There is optimism, at least among the people I talked to."

Raising the question: Just who the hell was he talking to? "People are more relaxed"? "There is optimism"? "The security seems better"? What country was he describing? Surely not the one he was sitting in.

Security in the meeting rooms of the garrisoned Green Zone may be better than it was in 2003, but the rest of Iraq is descending into what one Sunni leader described as "nothing less than an undeclared civil war."

"God knows what comes next," read a statement released by the Iraqi Islamic Party in reaction to the escalating violence. The group urged the nation's leaders "to lead Iraq out of this dark tunnel."

But while those in the midst of the mayhem see a dark tunnel, those in the Bush administration continue to see nothing but blue skies. Even as suicide bombs explode and the 2006 civilian death toll races toward 15,000, for the Bushies it's all relaxation and optimism.

Unbelievable. And sickening. And clueless, clueless, clueless." by Arianna Huffington, blogging

~~~~~~~~~~~~~~~~~~~~~~~~~~~~~~~~~~~~~~~~~~~~~~~~

It's always a bad idea to live in a fantasy world.

Comments?

tomder55 answered on 07/20/06:

I doubt if Arianna has spent much time in Iraq recently . So she is reporting from her own tin-foil hat perspective .Perhaps she should read reports from people who have actually been there like Jack Kelly National Security Writer for the Pittsburg Post-Gazette and syndicated columnist .

Also I do not trust the civilian casualty figures . Part of the problem of fighting "irregulars " is that since they are not uniformed it is hard to distinguish between them and civilians.

There are days I am discouraged about the news and progress from the theater and times I am optimistic. That happens in any war . I could put it into historical perspective but I doubt you would be convinced .

ETWolverine rated this answer Excellent or Above Average Answer
Itsdb rated this answer Excellent or Above Average Answer
jackreade rated this answer Bad/Wrong Answer

Question/Answer
jackreade asked on 07/19/06 - Bush Gropes Two Women at G8 Converence

Time for the President to go on some really strong meds. He groped a female oboe player, then the next day, groped THE CHANCELLOR OF GERMANY.

tomder55 answered on 07/20/06:

what a laugh . the male leaders of the summit were all over each other with hugs /arms around the necks /grasping and groping like jocks in the gym. But he gives a quick neck rub that temporarily startles Merkel and all of a sudden he is a sexual pervert .

I saw the Jon Stewart piece on it ... very amusing . My day is not complete until I watch The Daily Show and Colbert Report reruns (they are on too late to watch the first broadcasting but they are re-aired at more resonable times the next day )

CeeBee rated this answer Excellent or Above Average Answer
Itsdb rated this answer Excellent or Above Average Answer
jackreade rated this answer Above Average Answer
labman rated this answer Excellent or Above Average Answer

Question/Answer
paraclete asked on 07/19/06 - And they still say it's not a CIVIL WAR?

How many bodies do you need before a conflict becomes a civil war?

6000 civilians killed in May-June, says UN

From: Reuters From correspondents in Baghdad
July 19, 2006

ABOUT 6000 Iraqi civilians were killed in the last two months alone as casualties continued an "upward trend", the United Nations said.

The estimate, compiled with data from Baghdad morgue and the Health Ministry, is the latest attempt to give some statistical expression to the daily bloodshed in Baghdad and elsewhere.
It was part of a bi-monthly UN report on human rights in Iraq.

In a country with barely functioning public institutions, where relatives routinely remove the bodies of victims for burial with little legal formality, any estimates of mortality from the violence are highly approximate.

But a surge in sectarian killing in the past few months has been seen in rising trends in those statistics that exist.

Baghdad morgue officials have said they took in 1595 bodies in June, 1375 in May and 1155 in April. Of those, about 80 per cent were victims of violent deaths, they said.

In December, in the first and only official US estimate of the cost of the war in Iraqi lives, US President George W. Bush said some 30,000 Iraqis had been killed since the 2003 invasion.

A website that compiles media reports of casualties now estimates civilian deaths alone at around 40,000. By no means all deaths have been reported in the media.

The UN noted Iraqi health officials have said a recent media reckoning of 50,000 dead is probably an underestimate.

The UN report included all the morgue entries for May and June in its total of violent deaths along with 1294 deaths recorded in May by the Health Ministry and 1554 in June.

That gave a total cited in the report of 5818 deaths in the two months, the "overwhelming majority" of them in Baghdad.

Health Ministry figures incorporated in a monthly report issued by the Interior Ministry, however, show a lower level of civilian deaths. According to that report, 935 civilians were killed in violence in May and 889 in June.

The UN report cited a Health Ministry figure of 6826 civilians killed in the first six months of 2006 and a morgue total of 14,338 bodies taken in in Baghdad in the same period.

tomder55 answered on 07/19/06:

Calling it civil war is a matter of semantics . I think 'sectarian violence 'is more acccurate at this time . That happens all over the globe and it is not called civil war . Thousands of Indians both Muslim and Hindu have died from sectarian violence over the years but no one has called that civil war. The truth is that it's NOT "civil war". Not yet, at least.There are no organized military forces trying to seize and hold territorial objectives and/or political control.

Was it more moral for the UN to sanction Iraq after the first Gulf War ? Over a million children it is estimated died as a result of the sanctions . Perhaps it would've been better if Saddam was removed then . But wait ? That wasn't done because we needed Saddam's jack-boots to provide stability so the country wouldn't devolve into sectarian violence. right ? It doesn't matter that it was one way ethnic cleaning he maintained ;that hundreds of thousands of Kurds and Shia were slaughtered to keep the illusion of stability .

I of course think the sanctions would've worked if the top level at the UN had not been corrupted by violating the sanctions thus prolonging them ;the same UN that is suddenly concerned about what it wrought.

Itsdb rated this answer Excellent or Above Average Answer
paraclete rated this answer Excellent or Above Average Answer

Question/Answer
paraclete asked on 07/19/06 - Itb seems you can't save some people from their government?

when all its about is saving face

Tsunami warnings 'not sent'
From: By Cindy Wockner in Central Java
July 19, 2006

AP INDONESIAN officials failed to pass on expert warnings that the deadly tsunami was on its way, a government minister confessed early this morning.

Jakarta was told up to 45 minutes before tragedy struck that deadly waves would likely follow Monday's undersea earthquake.

In a bizarrely frank admission, Science and Technology Minister Kusmayanto Kadiman said alerts were sent from the Pacific Tsunami Warning Centre and Japan's Meteorological Agency after the quake, but "we did not announce them".

"If it (the tsunami) did not occur, what would have happened?" he said to reporters in Jakarta, apparently suggesting efforts to avoid an unnecessary mass panic. He did not elaborate.

With just 45 minutes' notice, but without an automated system to pass warnings on to villagers via loudspeakers or mobile phone text messages, it is unlikely a large-scale evacuation could have been achieved.

However, the failure will cause anger among shaken survivors and grieving relatives of the more than 340 dead.

Yesterday - before Mr Kadiman's admission - three Australians living in the idyllic West Java coastal village of Batu Keras told how the only warning they had was a roar like the sound of screaming jet engines and a fishing boat flying through the air.

Similar tales were being told up and down the stretch of West Java's coastline that fell victim to the notorious "Pacific Ring of Fire" which has claimed so many lives in the past two years.

Former Darwin bank manager Lyal Mackintosh, 59, was reading a book at his home 100m from the beach when he heard a roaring noise.

"If you have ever been outside an airport terminal when a 747 starts its engines, that's what it sounded like," he said.

"It was a roar. I just jumped on my motorbike and took off."

He caught a glimpse of the biggest of the waves to wash over the beachfront, and said it was about 4m high.

The Bay Surf Shop, for which Mr Mackintosh imports surfboards from Australia, was almost demolished by a fishing boat.

Fellow Australians, Andrew Warmbrunn, of Geelong and Grahame Malligan, of Sydney, were sitting at home chatting when they felt the quake.

"About 40 minutes after that I heard a noise and I looked through the palm trees and saw a boat heading up into the air," Mr Warmbrunn said.

"It started off a little bit quiet and then we looked up and it was an aeroplane at full roar. I ran inside the house and yelled 'tsunami' to my wife, grabbed my son and wife and ran."

Mr Warmbrunn did not even look over his shoulder to see the wave.

Mr Warmbrunn, 34, has been living in the village for the past year, finishing a Newcastle University thesis on the fishing industry and local economy.

With 171 of the village's 300 boats now destroyed, Mr Warmbrunn is not sure what will become of his thesis.

Sydney man Grahame Malligan, 45, fared a little better. The two fishing boats which his Indonesian wife owns were damaged but not destroyed.

tomder55 answered on 07/19/06:

if this had been the US Bush would've been accused of causing the Tsunami and the MSM would've made it headline news for 3 months .

Itsdb rated this answer Excellent or Above Average Answer
paraclete rated this answer Excellent or Above Average Answer

Question/Answer
paraclete asked on 07/18/06 - Eyes wide shut on the issue of the century?

Climate change has even US conservatives worried, but here the hip pocket still rules, writes Elizabeth Farrelly.

Australia is unusual among First World countries in combining a relatively educated populace, an extraordinarily fragile environment and a crude mining mentality. It's not a good mix. Indeed, as Jared Diamond pointed out in Collapse, our ruthless extension of the mining mind-set from minerals to renewable resources such as soil, fisheries and forests has only intensified our continental fragility.

Yet we go on exploiting our land rather than our intelligence, global warming or no, and choosing our leaders accordingly.

This is the mystery. Polls show we worry about climate change, but we vote from the hip pocket. John Howard, the polls tell us, makes us feel safe. But we blind ourselves to the yawning chasm between feeling safe and being safe. Ask the ostrich.

Howard is right to berate the states for their pathetic record on environmental initiatives, but wrong to attack their push for carbon trading (worth $13 billion worldwide last year). He is right to suggest Australia could become an energy superpower but it is reprehensible of him to focus the strategy on grubby old non-renewables such as coal, oil and uranium. Right to press the climate-change button, however tentatively; wrong to offer the nuclear solution.

Climate change has become a moral issue. Maybe the moral issue. If, as is arguable, morality is no more (or less) than a herd survival code, we might reasonably see all wars as the discordant death rattles of opposing fundamentalisms, soon to be replaced by some clean new enviro-religion. This new faith will make sacraments of rainwater, commandments of cycling and recycling, and prophets of well, there's the rub.

In Australia, where governments quail before moral issues, the vacuum is filling with an unlikely alliance of business and philanthropic lobby groups. The Australian Business Roundtable on Climate Change argued in April that a 60 per cent cut in Australia's emissions is compatible with strong economic growth. Westpac's chief executive officer, David Morgan, known for lampooning emissions proposals as Mein Kampf and seeing carbon trading as a European conspiracy, notes that "the next president of the United States [is expected] to initiate urgent action on climate change".

Now at last, the Climate Institute of Australia, has launched its Top Ten Tipping Points on Climate Change. Headed by the Australia Institute's Clive Hamilton, the institute is intelligent, purposeful, well placed. Never mind that the best it can do in the profit, sorry, prophet department is Bob Carr, whom you will remember as the man who turned a decade-long opportunity to green NSW into a filthy enviro-mess.

Any church is more than the sum of its saints, and there are bigger issues at stake. As Tipping Point says, we are entering the "oh shit" phase of global warming. Pretty much everyone is taking it seriously except us.

Our colleagues in climate crime are vanishing faster than the ice caps. Britain may be underachieving on its emissions targets, but business there is pushing Tony Blair for stronger regulations. In the US, where the writer Elizabeth Kolbert argues the need for an "environmental Churchill", an obstructionist Bush White House is nevertheless ringed by cities, states, Congress and the courts, plus a few inner-Republican colleagues, determined to make change.

Last year, California's Governor, Arnold Schwarzenegger, launched a plan to cut state emissions by 80 per cent by 2050. "The debate is over," he said. "The science is in. The time to act is now." Right-wing evangelical leaders of 30 million people marched on Capitol Hill, urging leadership on climate change. Since then, 238 US mayors have pledged to "meet or beat" Kyoto; the House of Representatives Appropriations Committee has supported emissions caps and the Supreme Court has agreed to decide whether CO 2 regulation should be mandatory.

The climate-change climate is changing, fast, and Australia is being left out in the warm. Ostrichised. It's a leadership thing, and it is dangerous - politically, morally, existentially. More, it's bad finance, with renewables - in which Australia could so easily excel - reaping $74 billion globally in 2005.

The US developer-turned-greenmeister and World Green Building Council president, David Gottfried, argued in Sydney earlier this year: "Green is the new black. All the big businesses are in the game, and it's for the benefit of everybody." Gottfried had one small goal in Australia: a tax credit for five-star green buildings. The Howard Government's Sustainable Cities report noted last August the "need for the Australian government to assume a leadership role". Yet the opposite is happening. Said Gottfried: "I don't see your Federal Government involved they're not at the table."

We've just had the hottest year on record. Atmospheric CO 2 is at its highest for 650,000 years. The seas are rising, the ice melting. Most scientists believe we have underestimated the impact. Yet we do nothing. The clever country, if we ever were that, has succumbed to waste, greed and denial. This is not just laziness. It's officially required, as the ABC's Four Corners demonstrated in February, documenting government pressure on CSIRO scientists to zip up on climate.

Yet we in Australia no longer care if our politicians lie. We don't mind if they peddle influence at $5000 a pop. In fact, we like it. At state and federal levels we consistently choose leaders who offer feelgood delusions and lugubrious denials over the truth of survival. Even David Attenborough, a long-time climate sceptic, has finally come round. As ever, he goes to the heart: "How could I look my grandchildren in the eye and say I knew about this and I did nothing?"

Elizabeth Farrelly writes on planning, architecture and aesthetics for the Herald.

tomder55 answered on 07/19/06:

if Arnie says then it must be so : "Man has caused da global wamingk " He is asking for draconian powers so he can roll back California emmissions to 1990s levels by 2020 . Oh wow ! how ambitious! I guess he will have to hide his Govenator's SUV's and Hummers in the mansion and wear dark glasses to avoid detection as he is driven around in them .He extensively travels in private jets and it is estimated that he is out of State 1 in every 5 days since he was inaugurated .

Of course what he doesn't say is that he is just going to drive the energy industry out of State or off-shore . The result will be no regional emission reductions but a California loss in revenue from an important sector of the economy. It will drive up the cost of energy in California making it less attractive for business to set up shop.

There are much more creative ways to deal with global warming if indeed it becomes an issue. I'm particularilly fond of planting more trees since I think that deforestation is the primary culprit if there is a human cause for it .

Itsdb rated this answer Excellent or Above Average Answer
paraclete rated this answer Excellent or Above Average Answer

Question/Answer
paraclete asked on 07/18/06 - Boy George has a solution for everything?


Bush's solution: 'stop doing this s---'



A microphone picked up an unaware George W Bush saying that Syria should press Hezbollah to "stop doing this shit" and that his secretary of state may go to the Middle East soon.

The US president was talking privately to British Prime Minister Tony Blair during a lunch at the Group of Eight (G8) summit in St Petersburg about an upsurge of violence in the Middle East.

Neither immediately realised a microphone was transmitting their candid thoughts on that and other issues.

"I think Condi (Secretary of State Condoleezza Rice) is going to go (to the Middle East) pretty soon," Bush said.

Blair replied: "Right, that's all that matters, it will take some time to get that together".

Rice headed back to the United States after the G8 summit closed yesterday and will decide when to make her Middle East trip, a State Department spokeswoman said.

Blair added: "See, if she (Rice) goes out she's got to succeed as it were, where as I can just go out and talk".

Bush replied: "See, the irony is what they need to do is get Syria to get Hezbollah to stop doing this shit and it's over".

Blair eventually noticed the microphone and hastily switched it off but not before the conversation had reached news media.

When Bush learned the microphones were on, he asked what he had said, and after being shown a transcript "he rolled his eyes and laughed," White House spokesman Tony Snow said aboard Air Force One returning to Washington.

While his language was salty, the message from Bush was what it had been throughout the summit - that Syria is supporting Hezbollah guerrillas in southern Lebanon and should force them to stop shelling Israel and return abducted Israeli soldiers.

US officials believe that if Hezbollah did so, Israel's military strikes on Lebanon might stop.

Bush also seemed to complain about UN Secretary-General Kofi Annan wanting an immediate ceasefire to stop the violence between Israel and Hezbollah.

"I don't like the sequence of it," Bush said.

"His attitude is basically ceasefire and everything else happens."

Blair said: "I think the thing that is really difficult is you can't stop this unless you get this international presence agreed".

A G8 statement yesterday suggested the UN Security Council should consider an international security and monitoring presence on the Lebanon-Israel border, an idea Blair is pushing.

Later, Bush said he felt like telling Annan to telephone Syrian President Bashar al-Assad "and make something happen".

"We're not blaming Israel and we're not blaming the Lebanese government," he said.

Snow said Bush was reiterating his view and Hezbollah should return the soldiers and stop firing rockets before talking about a ceasefire.

"He likes Kofi Annan and he is not only happy to work with him but has been supportive from the very start of the UN mission to the region," Snow said.

The United States would wait and see what the UN recommends about a stabilisation force, he said.

"I think it's awfully premature to be talking about US troops," Snow said.

"Let's just wait and see what the UN has to recommend ... and the means by which they hope to achieve it."

The discussions were not about "armies marching in and trying to fight Hezbollah," Snow said.

"What they're looking for is a stabilisation force to try to be able to support the Lebanese army. The Lebanese army clearly is not capable at this point of keeping peace in southern Lebanon."

Reuters

tomder55 answered on 07/18/06:

What makes anyone think they really did not know he mike was hot ? I don't think he gave a $#*& whether there was an open microphone nearby or not.Maybe they wanted some plain honest talk to be heard . Would have been more fun if he'd repeated Reagan's "The bombing begins in five minutes"or maybe saying "Iran's going to get a nuke faster than they expected " .

The media here seemed fixated yesterday on this non-issue. The funny thing is that some of their favorite Presidents like Truman and LBJ used "salty language" and no one ever felt the need to plaster it on the front page ;in fact it was considered a badge of honor .

As for Condi traveling to the war zone ; well lets say I think that the frame work for a settlement should be in place before she went. Shuttle diplomacy is all well and good but rarely is that attempted at the beginning of the diplomatic process. There has to be alot of back channel communications first .

I see French Prime Minister Dominique de Villepin arrived in Beirut to lend his support for Hezbollah .

ETWolverine rated this answer Excellent or Above Average Answer
Itsdb rated this answer Excellent or Above Average Answer
Judgment_Day rated this answer Excellent or Above Average Answer
paraclete rated this answer Excellent or Above Average Answer

Question/Answer
Erewhon asked on 07/17/06 - Ten more years ... or longer?

Military leaders foresee Iraq exit in 2016
By Rowan Scarborough
THE WASHINGTON TIMES
Published July 17, 2006

U.S. war commanders think some level of American forces will be needed in Iraq until 2016 and those forces will receive continued support from the vast majority of Iraqis.

At the tactical level, the U.S. is getting better at detecting deadly improvised explosive devices (IEDs), especially using unmanned spy planes. But the enemy is growing more sophisticated. A raid on an IED factory earlier this year netted two bomb-makers who hold master's degrees in chemistry and physics -- from U.S. colleges.

These were among the points made by Iraq war commanders at a closed-door conference last spring at Fort Carson, Colo., home to the 7th Infantry Division. Maj. Gen. Robert W. Mixon Jr., the division's commander, invited scores of retired generals and admirals in the Fort Carson area to hear the commanders and give them feedback.

Lt. Col. David Johnson, division spokesman, said the session was the second held this year at Fort Carson. A third is planned for the fall.

"The whole point is to share knowledge of what is going on in the Army today and to share ideas in an open forum," Col. Johnson said. The Fort Carson-area retired community has "a lot of knowledge and a lot of experience, and we wanted to tap into that," he said.

The seminar is just one example of how the Army is constantly re-examining how it conducts the war on terror in Afghanistan, Iraq and worldwide.

Some say the military has a near-obsession with scrutinizing each and every mission and listing things that could have been done better. At Fort Leavenworth, Kan., the Center for Army Lessons Learned collects volumes of after-action reports and commanders' insights and turns them into "lessons learned" reports distributed throughout the Army.

Out in the field, commanders learn lessons on the spot. When Brig. Gen. Kurt Cichowski, chief of staff for strategy at the U.S. Iraq command, was asked earlier this month by reporters how the security crackdown in Baghdad was going, he answered, "I will tell you that there's an evaluation that is going on right now about the entire operation that has started, and those are the kinds of lessons learned that we hope to tease out of what has happened in order to improve it for the future."

At Fort Carson, among the featured combat veterans was Col. H.R. McMaster, whose 3rd Armored Cavalry Regiment gained fame by liberating the northern town of Tal Afar from foreign terrorists and Iraqi insurgents. The town's mayor, Najim Abdullah Abid al-Jibouri, penned an open letter in February thanking the American troops for his people's liberty. The mayor visited Fort Carson in May to personally thank the soldiers and their families.

One retired officer attendee made notes and e-mailed his minutes of the session to other officers. The notes say there was general agreement on one issue: the "mainstream media" largely ignores progress. A commander said an embedded reporter filed a generally positive story on the operation in Tal Afar, only to see his stateside editors gut it and apply a negative spin.

In fact, editors have grown increasingly resistant to embedding reporters with combat units, something they demanded be done before the invasion in March 2003. The purported reason: They think contact with U.S. service members hurts the reporters' objectivity.

"They come to see the world through the eyes of the troops," said the retired officer's e-mail. Now, newspapers and magazine rely heavily on Iraqi stringers who telephone in reports from various combat scenes.

"We are clearly winning the fight against the insurgents, but we are losing the public relations battle, both in the war zone and in the States," said the e-mail.

Insurgent infiltration of the Iraqi Security Forces is also a big problem. A Green Beret caught a police lieutenant directing by telephone the placement of an IED so it would damage a coalition convoy.

Copyright 2006 News World Communications, Inc. All rights reserved.

===

Is withdrawal by 2016 too optimistic?

tomder55 answered on 07/18/06:

One retired officer attendee made notes and e-mailed his minutes of the session to other officers. The notes say there was general agreement on one issue: the "mainstream media" largely ignores progress. A commander said an embedded reporter filed a generally positive story on the operation in Tal Afar, only to see his stateside editors gut it and apply a negative spin.

In fact, editors have grown increasingly resistant to embedding reporters with combat units, something they demanded be done before the invasion in March 2003. The purported reason: They think contact with U.S. service members hurts the reporters' objectivity.

"They come to see the world through the eyes of the troops," said the retired officer's e-mail. Now, newspapers and magazine rely heavily on Iraqi stringers who telephone in reports from various combat scenes.


sorry for the long cut and paste but those couple of paragraphs have been my biggest rub with the coverage . It is certainly true that reporters traveling with the troops send to the States a completely different and I think more accurate accounting of what is happening on the ground . Reporters filing reports from 'mahogony hill' (the bar at the hotel in the Green Zone)are only sending out reports from 2nd hand sources .

I have surmised for some time now that the US and the new Iraqi gvt. will come to a mutual defense treaty when all is said and done. This would be in line with what we currently have with many of the Gulf Emerates like Qatar and Kuwait and what we had previously with Saudi Arabia .

Even when the internal security issues have been resolved ,I do not think the Iraqi's will be capable of defending themselves from hostile neighbors for the foreseeable future. It would be mutually beneficial and I think that those who are calling for complete withdrawal are kidding themselves .

Complete withdrawal from where ? Iraq? The Gulf region ? the Middle East ? the world ? It aint happening in spite of murky Murtha's bleating that we should "strategically re-deploy somewhere over the rainbow horizon (to Okinawa was the last stopping ground I believe).

Erewhon rated this answer Excellent or Above Average Answer

Question/Answer
jackreade asked on 07/17/06 - EDGAR

Cheney's secret service name is Edgar. I thought, yeah, as in Edgar Allan Poe the American writer of dark stories...what a perfect name. But NO! it stands for Edgar Bergen, you all remember Edgar Bergen and Charlie McCarthy!! lolol...

The secret service knows the score.



jack

tomder55 answered on 07/18/06:

and here I thought they were using the name 'DUCK' or 'Dead Eye ' . I hear Clintoon's was 'ZIPPERS'

jackreade rated this answer Excellent or Above Average Answer
purplewings rated this answer Excellent or Above Average Answer

Question/Answer
HANK1 asked on 07/17/06 - SELECTIVE SERVICE:



Will Bush reinstate the draft this year?

HANK

tomder55 answered on 07/17/06:

Bush cannot reinstate the draft . It has to be done by Congress.

My response ;as of right now the military is exceeding it's recruitment goals . But never say never .

Erewhon rated this answer Excellent or Above Average Answer
HANK1 rated this answer Excellent or Above Average Answer
Itsdb rated this answer Excellent or Above Average Answer

Question/Answer
dimwit asked on 07/16/06 - Victory or Retaliation?


Hello:

The War on Terror could be conducted to produce either of those results. Which one would you opt for? Do you think there's a difference? Which one SHOULD we be waging? Which one ARE we waging?

Could you say the Dems would wage one kind of war, intending to produce one of those results, while the Republicans would wage the other intending to produce the exact opposite result?

Could one side say they were attempting to achieve victory, while secretly enjoying the enemys demise? Could the other side openly enjoy the enemys misery, and call for more, while pretending it seeks only victory? Could one side say everything and do nothing, while secretly enjoying OUR demise?

Finally, is retribution wrong? Should it be evenly parsed out, or should you hit back with everything you've got?

dim

tomder55 answered on 07/17/06:

This week has more than illustrated the folly of proportionality . In an age of aysmmetrical assaults the idea that a superior power would handcuff itself with a proprtional response plays into the enemies hands . Indeed it is the very restraint in response that gets us our greatest condemnation. Had we carpet bombed Fallujah instead of going door to door and detaining terrorists in Abu Ghraib then we would've come under far less condemnation . Think that's not true ? Look at how the Ruskies leveled Grosny without so much as a vote of condemnation from the UN.

"The natural aim of military operations is the enemy's overthrow. . . . Since both belligerents hold that view, it would follow that military operations could not be suspended . . . until one or other side were finally defeated." -von Clausewitz

dimwit rated this answer Excellent or Above Average Answer

Question/Answer
kindj asked on 07/16/06 - Three short ones

The last one is perhaps the most timely....

The US Navy welcomed the latest member of its fleet today.

The USS William Jefferson Clinton CVS1 set sail today from its home port of Vancouver, BC.

The ship is the first of its kind in the Navy and is a standing legacy to President Clinton and his foresight in military budget cuts. The ship is constructed nearly entirely from recycled aluminum and is completely solar powered with a top speed of 5 knots. It boasts an arsenal comprised of one F14 Tomcat or F18 Hornet aircraft, which although they cannot be launched or captured on the 100 foot flight deck, form a very menacing presence.

As a standing order there are no firearms allowed on board. The 20 person crew is completely diversified and includes members of all races, creeds, sex, and sexual orientation.

The ship's purpose is not defined so much as a unit of national defense in fact in times of conflict its orders are to remain in hiding in Canada, but will be used extensively for social experimentation and whatever worthless jobs the ex-commander in chief and his wife can think of.

It is largely rumored that the ship will also be the set for the upcoming season of MTV's "The Real World."
~~~~~~~~~~~~~~~~~~~~~~~~~~~~~~~~~~~~~~~~~~~~~~~~~~~~~~~~~~~~~~~~~~~~~~~~~


Two US Marines are listening to the radio in Iraq.

"American soldiers," coos a soft female voice, "Your so-called national Leaders have lied to you. You are needlessly risking your lives to wage A useless, unjust, illegal, and unwinnable war. Now is the time to return home to your loved ones, while you are still alive. If you foolishly insist on remaining where you are not wanted, the brave resistance fighters will have no choice but to kill you and add your name to the long ever-increasing casualty list of this insane war. So why risk never seeing your loved ones again for a so-called president who has repeatedly lied and deceived you at every opportunity? Why should you be sacrificed so that US corporations can enjoy fatter profits? The only wise thing to do is return home now, while you are still drawing breath, before you return zippered into a bodybag."

"What's this?" sneers one Marine. "An Islamo-terrorist version of Tokyo Rose?"

"No," answers the other. "It's just CNN!"
~~~~~~~~~~~~~~~~~~~~~~~~~~~~~~~~~~~~~~~~~~~~~~~~~~~~~~~~~~~~~~~~~~~~~~~~~


The Israeli Ambassador at the U.N. began, "Ladies and gentlemen before I commence with my speech, I want to relay an old Passover story to all of you ...

"When Moses was leading the Jews out of Egypt toward the Promised Land, he had to go through the nearly endless Sinai desert. The people became thirsty and needed water. So Moses struck the side of a mountain with his staff and a pond appeared with crystal clean, cool water. The people rejoiced and drank to their hearts' content.

"Moses wished to cleanse his whole body, so he went over to the other side of the pond, took all of his clothes off and dove into the cool waters. Only when Moses came out of the water, he discovered that all his clothes had been stolen. 'And,' he said, 'I have reasons to believe that the Palestinians stole my clothes.'"

The Palestinian delegate, hearing this accusation, jumps from his seat and screams out, "This is a travesty. It is widely known that there were no Palestinians there at that time!"

"And with that in mind," said the Israeli Ambassador, "let me now begin my speech."

tomder55 answered on 07/17/06:

and the USS Clinton has a curious cigar shaped hull.

kindj rated this answer Excellent or Above Average Answer

Question/Answer
jackreade asked on 07/17/06 - Republican Party in Shambles

Today on a talking heads show, Newt Gingrich said we are in WWIII.

Condi Rice says that democracy is just around the corner in the Middle East.

So Neo-Nazi Board members and rational thinkers, what is it?

tomder55 answered on 07/17/06:

from one rational thinker to another . Newt Gingrich is a student of history . He traces events for over 58 years to make his case that we are in WWIII . I cannot dispute his logic. When we shed the cloak of P.C. and acknowledge the reality that this war pits the free world against those who embrace a radical Islamic ideology, then perhaps we will come to a unified response .There are still to many people in denial in this country and the rest of the West .

I was a little put off by Condi this week calling for restrain from Israel especially since I think their response was restrained . Imagine a missle barrage aimed at Chicago ,Detroit and other northern border cities by terrorists working out of Canada . How would we respond ? The only pass I will give her is that it is her job to use diplomat-speak .Gotta hope she was not sincere . I think a better statement would've been no statement .Poorly timed dialogue is often worse than no talk at all.

Lebanon once looked like a potential Bush administration success story.When Codi visited Lebanon she visited with Emile Lahoud the very person against whom the Cedar Revolution had taken place. Her meeting with him implied a degree of legitimacy to his Presidency .A serious effort was not undertaken to disarm Hezbollah ;not by the Lebanese Gvt. and not by pressure from our State Dept .

Dialoge for dialogue's sake is a waste of time . I thought she would be more results oriented .The plague of diplomacy has been a love affair with the process instead of results .Condi should get out of the State Dept. and back to national security . She was sent to Foggy Bottom to reform the agency but it may be beyond reform and it is swallowing her.

Deomcracy is just around the corner ;but turning the corner will take some time yet .

Itsdb rated this answer Excellent or Above Average Answer
jackreade rated this answer Excellent or Above Average Answer

Question/Answer
paraclete asked on 07/17/06 - The Global War?

Chaos rules in this borderless war
July 17, 2006

The renewed fighting between Israel and Hezbollah is detail in a much bigger picture, writes Paul Sheehan.


When historians narrate the beginnings of the third global war, a war already under way with more than 200,000 killed, they may choose the moment on October 12, 2000, when a small fishing skiff sailed up to an American destroyer, the USS Cole, at anchor off Aden harbour in Yemen.

As the skiff approached, the two Arabs on board smiled and waved at the sailors on deck. Then the two men stood to attention.

In the next instant, the Cole was gutted by an enormous bomb. It liquefied the bombers, killed or wounded 56 sailors, and disabled a heavily armoured state-of-the-art warship. Other suicide bombs had exploded before, and many more since, but the attack on the Cole was the first frontal assault on the US military by al-Qaeda, and the emergence of al-Qaeda globalised and modernised the cause of jihad.

War and murder have been carried out in the name of Allah in Thailand, Bali, Sumatra, the Philippines, Nigeria, Algeria, Somalia, Yemen, Saudi Arabia, Iran, Iraq, Israel, Gaza, the West Bank, Egypt, Afghanistan, Lebanon, Pakistan, India, Bosnia, Albania, Kenya, Tanzania, France, the Netherlands, Britain, Spain, Denmark, Russia, the United States and Sudan, where mass murder and mass rape have been the tools of cultural war.

What makes this global war different from the First and Second World Wars is that there are tens of thousands of combatants who actually want to die, and in the process kill as many non-believers as possible. As the bombs, missiles and rockets have been exploding in Lebanon, Israel and Gaza, medievalists who are key drivers in this cultural struggle have been ecstatic. You can hear it in the rhetoric of Hezbollah's spiritual leader, Hassan Nasrallah, and Iran's President, Mahmoud Ahmadinejad.

Most disturbingly, jihad is being driven by three separate, distinct and often competing strands of Islam: Sunni, financed by the oil-powered Wahabist fundamentalists of Saudi Arabia, and dominated by the ideology of al-Qaeda and Osama bin Laden; Shiite, an extension of the theocracy of Iran, and highly active in Iraq, Lebanon, Afghanistan and the Palestinian territories; and Pakistani Muslim nationalism, the wellspring of jihad in Kashmir, support for the Taliban, and terrorist attacks in India and Britain, with its large Pakistani emigre community.

What these distinct mutations of Islam have in common is an appetite for war and murder and sexual oppression. It is no coincidence that the governments in all three of these wellsprings of jihad have weapons of mass destruction. Pakistan has the first "Islamic bomb", acquired through a campaign of theft, stealth and illegality. Saudi Arabia and Iran have a different kind of bomb - oil in immense and strategic quantities.

Iran, the prototype of the modern Islamic theocracy (which also wants the nuclear bomb to match and trump Israel's nuclear option), is stronger today than it was a week ago. It already wields disproportionate power in the Middle East through its proxies helping to tie down 150,000 US troops in Iraq, which is 60 per cent Shiite. Now, thanks to its proxies in Lebanon, Israel has been goaded into an attack on a democratic neighbour.

At the weekend, the foreign ministers from 18 Arab League nations held an emergency meeting in Cairo after which the Secretary-General of the League, Amr Moussa, declared that the Middle East peace process was "dead".

Lebanon's Prime Minister, Fouad Siniora, said that Israel's "war machine" had turned his country into a "disaster zone".

Another victory for the provocateurs of Islamic fundamentalism. Every time chaos has engulfed the Middle East, militant Islam has emerged with greater power. Creating chaos is thus the modus operandi of jihadists.

Lebanon's newly reborn democracy and stability, after 20 years of civil war, has been the greatest act of national reconstruction in the Arab world, an enormous achievement. The Government in Beirut relies on a detente between former enemies, Christians, Sunnis, Shias and Druze, some 18 separate factions.

No one in this fragile democracy is willing or able to disarm the Hezbollah militia dominant in the Shiite south of Lebanon. Far more important has been the multibillion-dollar rebuilding of the economy.

The United Nations may have passed Resolution 1559 calling for the disarming of Lebanon's militias, but who in Lebanon would be willing to go into that hornet's nest? Who would be willing to plunge the nation into another civil war? By blaming the Lebanese Government for Hezbollah's actions, Israel has demanded the impossible from its neighbour.

Hezbollah has been thriving on chaos since it began in 1982 in response to Israel's invasion. When it launched its first major suicide attack in the region on April 18, 1983, killing 63 people at the US embassy in Beirut, it created the template for the borderless war we are part of, whether we want to be or not.

The only glimmer of good news at the weekend was the criticism of Hezbollah by the governments of Saudi Arabia, Egypt and Jordan. And to put the current mayhem in perspective, the Cold War was waged for more than 50 years between democratic capitalism and totalitarianism, with nuclear weapons massed on either side. It was not a cold war for the tens of millions of people who died in purges within China and the Soviet Union, and in wars or civil wars in Vietnam, Korea, Latin America and Africa.

The Cold War ended with no nuclear weapons being used. But compared with medievalists waging jihad, the communist powers of the Soviet Union and China were prudent, rational players. In today's global struggle, the objective evidence is overwhelming that where militant Islam goes, bloodshed follows. In a whole range of different settings, for many adherents of Islam the Koran is not a book of peace but a call to war.

Is that the way you see it?
Where Islam goes, bloodshed follows?

tomder55 answered on 07/17/06:

What Sheehan is assuming is that the GWOT is a failure and the battle against extremism is now a clash of civilizations. His so called glimmer of good news is in fact a significant progress. So I am not willing to concede his thesis yet .

Israel has been goaded into an attack on a democratic neighbour.

Nonsense ;again it is blame the victim. Israeli response has been measured and it has been Hezbollah who has day by day raised the stakes. Israeli limited goals is to cripple the terrorist org. and have the Lebanese Army replace them at the Israeli border . (in this regard their goals match the statement that came out of G8 ).

And of course Iranian President Ahmadinejad keeps on daring Israel to attack Syria.(He will fight Israel to the last Lebanese and Syrian) That will not happen I think until Tel Aviv is hit. When that happens then the war will escalate (and Israel will have already prepared the battle field ) but it will still not be the clash of cultures that Sheehan is saying already exists as long as there are significant "moderate "rulers who are looking forward to the 21st century instead of the 8th .

paraclete rated this answer Excellent or Above Average Answer

Question/Answer
Fritzella asked on 07/15/06 - Kicking Open a Hornet's Nest

"Sometime in the fall of 2002, I likened a U.S. invasion of Iraq to "kicking open a hornets' nest." I predicted that, if the Iraqis decided to fight in the cities, our casualties would be between five and ten thousand U.S. troops at least. Now, U.S. casualties exceed 20,000.

But the "hornets' nest" I predicted was not just an interminable and
intractable U.S. occupation of Iraq. It was wider war in the Middle East. The larger hornets' nest is now swarming.

By our justified overthrow of the Taliban in Afghanistan, though unsuccessful decapitation of al Qaeda, we removed a thorn in Iran's side. By removing Saddam Hussein, we removed a thorn in Iran's other side.

But, inadvertantly and ignorantly, we empowered Iran to undertake a major intervention on behalf of the Shiite majority in Iraq. In response to our insistence that Iran not develop any nuclear capability, Iran and Syria have emboldened Hezbullah in Lebanon to energize Israel's formidable military and Hamas to do the same.

The U.S. is fighting a two front war with Afghanistan and Iraqi insurgents. Israel is fighting a two front war with Lebanon and the Palestinians.

Wouldn't you think this would be exactly the time when the nation's wisemen, those neoconservative idealists who saw the great American empire imposing democracy on the Middle East at the point of a bayonet, who secretely envisioned Iraq as our military base in the region, to be heard from? Of course, I mean Paul Wolfowitz, Richard Perle, Douglas Feith, Steven Cambone, and so many triumphalist others so present on the talk shows in early 2003. Haven't seen much of them recently.

Richard Cheney and Donald Rumsfeld are left to manage the disaster. You don't hear either one of them linking their arrogant decisions four years ago to the disaster unfolding in the Middle East.

We have some lessons in democracy to be learned here at home. Democracy does not work without accountability. Today there is no accountability in American democracy.

On the other hand, perhaps there will be in 2006 and 2008...if the Democrats recapture conviction and courage." Gary Hart

~~~~~~~~~~~~~~~~~~~~~~~~~~~~~~~~~~~~~~~~~~~~~~~~


What about the 125,000 or so troops in Iraq who are potentially trapped in a potential mounting hornet's nest? Surrounded.

tomder55 answered on 07/15/06:

What has Gary Hart been sniffing (besides Donna Rice ) ? Does he think the neocons started the war against jihadistan ? lol

By our justified overthrow of the Taliban in Afghanistan, though unsuccessful decapitation of al Qaeda, we removed a thorn in Iran's side.


He keeps up this false myth that Shia and Sunni never cooperate . That somehow al -qaeda was a threat to Iran ? He forgets that the 9-11 commission determined that al-Qaeda (a so called Sunni terrorist group) and Hezbolla (puppets of the "Shia" Iranian regime ) worked side by side in the bombing of the Kobar Towers in 1996.In relation to Iran, commission investigators said intelligence "showed far greater potential for collaboration between Hezbollah and al Qaeda than many had previously thought." Tom Kean, also said in a television appearance that "there were a lot more [al Qaeda ]active contacts, frankly, with Iran and with Pakistan than there were with Iraq."
The original U.S. indictment of bin Laden, filed in 1998, said al Qaeda "forged alliances .. with the government of Iran and its associated terrorist group Hezbollah for the purpose of working together against their perceived common enemies in the West, particularly the United States." The 9-11 report also says that several years before the Khobar attack, "bin Laden's representatives and Iranian officials had discussed putting aside Shia-Sunni divisions to cooperate against the common enemy." A group of al Qaeda representatives then traveled to Iran and to Hezbollah training camps in Lebanon for "training in explosives, intelligence and security."

So let's put this phony premise to rest once and for all .Shia and Sunni cooperate when it serves their purpose and in their case the cooperation was in creating a world wide califate .al Qaeda and Iran cooperated ;al Qaeda and Iraq cooperated ,and Iran and Syria (Baathist Sunni ) are alligned today .

Hmmmm there were neo-cons back then too ....maybe they provoked the attack?

What about the 125,000 or so troops in Iraq who are potentially trapped in a potential mounting hornet's nest? Surrounded.

You mean the US troops that are forward deployed into the heart of the ummah ready to take the fight wherever it leads to . You want to talk about a "thorn in Iran's side" ? How about US troops surronding Iran FROM Iraq ;Afghanistan; the Persian Gulf ,the Indian Ocean ,and (currently )US ally Pakistan ? Who's surrounded again ?



Fritzella rated this answer Above Average Answer
Itsdb rated this answer Excellent or Above Average Answer
powderpuff rated this answer Excellent or Above Average Answer

Question/Answer
Fritzella asked on 07/15/06 - Wesley Clark Says

"I am a proud member of the Democratic Party, and I believe it is our partys responsibility to support the will of the Democratic primary voters in Connecticut. I personally look forward to supporting the candidate CT voters elect as the Democratic nominee. Though, as an aside, I must say I find it ironic that Senator Lieberman is now planning a potential run as an independent after he continually questioned my loyalty to the Democratic Party during the 2004 presidential primary".


NaNaNa Naaa
NaNaNa Naaa
Hey Hey Hey

Good-bye!!!

Lieberman

Anyone else have any comments about Lieberman?

tomder55 answered on 07/15/06:

I think it is imperetive for Lieberman to win the primary for the sake of the Democrats . If Ned Lamont wins then it will be open season on moderate Democrats and the take over of the party by the moonbat Moveon.org and the Cindy Sheehad /Michael Moore radical wing will be complete and any shot the Dummycrats have of taking back the Congress will be destroyed for another decade.

That alone is almost reason enough for me to support Lamont but Joe Lieberman embodies everything that is good in American politics and it would be a damn shame if he was not sent back to the Senate. He is a senior member of the Senate and has a huge clout. He votes party line more than 95% of the time .What more can the Dems. ask for ? He takes the position the Democrats used to take about foreign policy before McGovern and company destroyed the party.

In that regard former President Bill Clintoon is correct :

"If we allow our differences over what to do now in Iraq to divide us instead of focusing on replacing Republicans in Congress; that's the nuttiest strategy I ever heard in my life,"

I wonder if the moonbats would've opposed Clintton's reelection if we had been at war in Iraq ? It was he who championed the regime change direction we eventuall took . Ask him about troop withdrawal and he sounds like a Republican .Clinton said Lieberman is right to oppose a fixed timetable for withdrawing troops from Iraq, saying, "Why send a signal about when you're going to leave to people trying to keep Iraq divided? Would you make any political deals if you knew you could just hang around and maybe get what you want?"


Wesley Clark is a bitter old fool . When Gen. MacArthur said "Old soldiers never die; they just fade away."clearly he had the likes of Clark in mind. It is debatable if he was a sound General but I will give him the benefit of the doubt . However he is completely over his head when he wades into national politics. He should do himself a favor....write a couple more books /join the lecture circuit /make a few million ...even become a taking head if he wants ....or better yet ...go fishing /watch the Yanks beat the White Sox .....retire already !!

captainoutrageous rated this answer Excellent or Above Average Answer
Fritzella rated this answer Excellent or Above Average Answer
labman rated this answer Excellent or Above Average Answer

Question/Answer
Fritzella asked on 07/13/06 - Middle-East Slipping into All Our War

Is this a good thing?


Fritzie

tomder55 answered on 07/14/06:

The NY Sun has an op ed piece that starts "the war on Iran has begun". It also says: "Years from now, the kidnapping of Corporal Gilad Shalit will be regarded like the assassination of Archduke Ferdinand."

It was inevidable . All you have to do is look at it from the Iranian perspective to find the logic. This week was a deadline by the UN that Iran will fail to meet. Also the G8 is meeting and I'm sure that Iranian defiance will be a major topic (although the idiot Chirac would like to make the meeting a referendum on raising airline taxes so more medicine could be donated to Africa).

Through a series of provocations Hamas and Hezbollah have forced Israel to respond militarily . The usual actors ar parroting the line(unfortunately including Bush [bring back the cowboy PLEASE]....and Rice) that Israel should respond in a "measured way " .But Israel cannot do a measured response anymore . Yesterday's attack on Haifa eliminated that possibilty . It is the equivalent of attacking Chicago . Israel is now forced to go deep into Lebanon and secure Southern Lebanon against future missle attacks on it's major cities . The next missle may have some of Iraq's 'non-existant ' chemical weapons on it . I think the Israeli's have to secure the Bekaa Valley, the seat of Hezbollah power ,and the storage place of much of their munitions .


Meanwhile the delusional clown that runs Iran ;Ahmadinejad ( or is that Ahma Dahma Ding Dong ? ); will proclaim to the world ,and specifically to the ummah ,that "see ..... Israel is the threat we say she is .....that is why we are justified to continue our nuke program ".The Gaza and northern Israel attacks were planned for quite a while, which means that Iran wanted this war, this way. It isnt just a target of opportunity or a sudden impulse; its part of a strategic decision .

The problem is that it makes no sense . If he pushes the envelope too hard he is going down . The only way it makes sense is if you believe his rhetoric about the 12th Immam who will arrive at the last hour to save the Iranians from apocalypse. As I often say ,when someone like him says he's going to do something you best believe he will try .

Look for the puppet ruler of Syria ,Assad ,to provoke Israel into opening a third front soon.He doesn't want to but Iran calls the shots. We may get to see which is the better missle ;the Shahab 3 or the Jherico . I'm placing my bets on the Jherico.

ETWolverine rated this answer Excellent or Above Average Answer
Fritzella rated this answer Excellent or Above Average Answer
Itsdb rated this answer Excellent or Above Average Answer
purplewings rated this answer Excellent or Above Average Answer

Question/Answer
Erewhon asked on 07/13/06 - Reflections on War, Detention and Rights ... ... ... ...


Reflections on War, Detention and Rights

By ADAM LIPTAK
Published: July 13, 2006


Mamdouh Habib, an Australian who says he was tortured in Egypt before being sent to Guantnamo Bay, Cuba, for intense interrogation and indefinite detention, had reason to be wary when a man claiming to be his lawyer came to see him in the fall of 2004.

The lawyer, Joseph Margulies, had anticipated the reasonable fears of a client who for years had been tricked, disoriented, humiliated and worse. He had a letter of introduction from Mr. Habibs wife, Maha. But American interrogators had once falsely told Mr. Habib that his wife was dead, and Mr. Margulies feared that his client would think the letter a forgery or the product of coercion.

As backup, Mr. Margulies came armed with a few memories Mrs. Habib had shared with him about her husband: the location of their first date, the first gift he gave her, and the people who looked after their youngest son when their oldest boy was ill.

There is no reason to repeat those private reminiscences here, Mr. Margulies writes in Guantnamo and the Abuse of Presidential Power, but suffice it to say, they worked. When I shared that information with Mamdouh, he began to cry.

The detention centre at Guantnamo Bay, created in early 2002 to hold suspected terrorists, is still home to about 450 prisoners. Mr. Margulies filed suit on behalf of four of them, including Mr. Habib, just months after the first Guantnamo camp was built. For the next two years the Bush administration refused to let the four men know about the suit, much less meet with their lawyer.

It was in that case, Rasul v. Bush, that the Supreme Court in June 2004 landed the first body blow to the Bush administrations assertion that it has the unilateral power to designate people as terrorists and then hold them forever without charges. Mr. Marguliess meeting with Mr. Habib followed the Rasul decision.

Last month, in a kind of sequel to Rasul, the Supreme Court said the administrations plans for trying Guantnamo prisoners using secret evidence offended both military justice and international law. The administration and Congress are at work recasting those plans, and the Pentagon announced this week that it will comply with an important provision of the Geneva Conventions, the one prohibiting outrages upon personal dignity, in particular humiliating and degrading treatment.

Mr. Habib was lucky in his lawyer, as Mr. Margulies is a resourceful advocate, a serious and sober legal analyst and a fine, sometimes luminous writer. In his new book Mr. Margulies weaves together a history of wartime interrogation, a consideration of the legal standards that apply to it and an assessment of the toll that Guantnamo has taken on the men and boys held there, and on the nations reputation and values.

The books title, with its dry allusion to the separation of powers, does not do it justice. Guantnamo and the Abuse of Presidential Power represents the best account yet of what Mr. Margulies calls a human rights debacle that will eventually take its place alongside other wartime misadventures, including the internment of Japanese-Americans during World War II, the prosecutions under the Espionage and Sedition Acts during World War I, and the suspension of the writ of habeas corpus during the Civil War.

The first problem in considering Guantnamo is one of metaphor. It is a prison, certainly, but not one meant to mete out punishment for past crimes. It is a kind of prisoner-of-war camp too, a way to incapacitate supposed combatants for the duration of hostilities so that they cannot return to the field of battle. But here the hostilities the so-called war on terror may last forever. And the battlefield is the globe.

Most crucially, Guantnamo is an interrogation chamber. To be effective, administration strategists said, it should operate outside the American legal system, without the risk, Mr. Margulies writes, of interference by courts and counsel into the delicate relationship between interrogators and prisoners. And to be more effective yet, they went on, the prisoners had to be denied the protections of the Geneva Conventions.

Until Guantnamo, the United States had an excellent reputation for the humane treatment of captured combatants. During World War II, for instance, Mr. Margulies writes, when more than 400,000 German, Italian and Japanese prisoners of war were held in the United States, their captors followed the Geneva Conventions with an almost compulsive regard. Because the conventions require that prisoners be afforded the same living conditions as their guards, for instance, American camp commanders ordered their own soldiers to sleep in tents until barracks for the prisoners were completed.

The Guantnamo prisoners, by contrast, were made to endure stress positions, extreme temperatures, sleep deprivation, blaring music, strobe lights, religious insults and sexual humiliation. Three prisoners there recently committed suicide.

In Oath Betrayed Dr. Steven H. Miles, an expert on medical ethics, collects evidence that armed forces physicians, nurses and medics had been passive and active partners in the systemic neglect and abuse of prisoners at Guantnamo, in Iraq and in Afghanistan.

In his short, passionate and disjointed book, made up mostly of information from raw documents, reports and news accounts, Dr. Miles allows outrage to substitute for analysis. Still, he collects countless examples of medical complicity in abuse that is all the more disturbing for the lack of any notable protest. Doctors have, Dr. Miles writes, certified prisoners as healthy enough to withstand harsh treatment, monitored them during interrogations and concealed evidence of their mistreatment.

Enough practitioners complied when they should have resisted, or kept quiet when they should have spoken out, Dr. Miles writes, to allow abusive interrogational practices and a neglectful prison environment to operate largely without medical opposition or disclosure.

Lawyers were also slow to rise to the challenge of Guantnamo. In the early days the establishment bar and even some of the major civil rights groups held their fire, leaving it to Mr. Margulies and a handful of other lawyers notably those of the Center for Constitutional Rights in New York, Thomas B. Wilner of Shearman & Sterling in Washington and Clive Stafford Smith in New Orleans to file the most important American lawsuits since the Sept. 11 attacks. In an aside on page 158 of his book, Mr. Margulies notes that he was not paid for his work on the Rasul case.

Inside the government, though, the situation was more complicated. Lawyers in the military and the State Department fought an honourable if largely losing battle to try to preserve the Geneva Conventions.

In 2002 Defense Secretary Donald H. Rumsfeld described those held at Guantnamo as among the most dangerous, best trained, vicious killers on the face of the earth. But a recent study prepared at the Seton Hall University School of Law shows that just 8 percent of the detainees were even said by the government to be Al Qaeda fighters.

More than 300 Guantnamo detainees have been released or transferred, Mr. Habib among them. The United States government never charged him with a crime, and he is back in Australia, a free man.

---

GUANTNAMO AND THE ABUSE OF PRESIDENTIAL POWER, By Joseph Margulies
322 pages. Simon & Schuster. $25.

OATH BETRAYED - Torture, Medical Complicity, and the War on Terror,
By Steven H. Miles, M.D.

220 pages. Random House. $23.95.

===

Where now?


tomder55 answered on 07/13/06:

Joseph Margulies has a problem with Lincoln suspending habeus corpus in the middle of a Civil War ? That about says it all. No wonder the NY Slimes book critic praises him.

Let's talk about Mamdouh Habib. When he returned to Aussie his passport was taken away .Foreign Minister Alexander Downer says it is because of his terrorist ties . Habib tried to appeal the decision but soon dropped it due to the overwhelming evidence against him. Downer made the decisioon based on information that ASIO provided . He was released from GITMO not because the US couldn't prove allegations about him . He was released into the custody of the Aussie Gvt. with assurances that ASIO would monitor his activities and ensure he did not present a security threat. That was because the Aussie libs were in a tizzy over his detainment .Now they have to deal with the scum.

Downer has also looked into the allegations that Habib was sent to Egypt for torture and could find no evidence of it .

Interesting to note that Habib is a serial accuser .


Why is everybody always picking on me?

Erewhon rated this answer Excellent or Above Average Answer
Itsdb rated this answer Excellent or Above Average Answer

Question/Answer
kindj asked on 07/11/06 - Feel the pride!

http://www.wtv-zone.com/Mary/THISWILLMAKEYOUPROUD.HTML

tomder55 answered on 07/13/06:

was so busy emailing this to everyone on my list that I almost forget to thank you for it . I had heard of his heroism but the pictorial that accompanies it is priceless.

ETWolverine rated this answer Excellent or Above Average Answer
kindj rated this answer Excellent or Above Average Answer

Question/Answer
Erewhon asked on 07/11/06 - Will Bush ever get real?

Another Mission Accomplished
Published: July 11, 2006

The release of the White House midsession budget review
This is proof, if anyone still needs it, that this administration is desperate for something to boast about. On Mr. Bushs watch, triple-digit budget surpluses have turned into annual triple-digit budget deficits. Theres no information in the midsession report to alter that utterly dispiriting fact. Yes, the report is expected to project that this years deficit will be somewhat less gargantuan than last years probably somewhere between $280 billion and $300 billion, versus a $318 billion shortfall in 2005. Thats not much to crow about.

But Mr. Bush is likely to gloat, anyway. Earlier this year, the administration conveniently projected a highly inflated deficit of $423 billion. With that as a starting point, the actual results can be spun to look as if theyre worth cheering.

The razzle-dazzle wont end there. As he did in his remarks on Saturday, Mr. Bush is sure to use todays event to credit tax cuts for a projected surge in tax revenue. The Treasury is expected to take in about $250 billion more in 2006 than in 2005, for a total take of $2.4 trillion. Devoid of context, the number looks impressive.

In fact, it is $100 billion less than the $2.5 trillion revenue estimate the administration touted when it set out in 2001 to sell its policy of never-ending tax cuts. Even with this years bigger haul, real revenue growth during the Bush years will be abysmal, averaging about 0.3 percent per capita, versus an average of nearly 10 percent in all previous post-World War II business cycles. That might be excusable if the recent revenue improvements could reasonably be expected to continue. They cannot. Much of the increase in tax receipts is from corporate profits, high-income investors and super high-earning executives, sources that are just as unpredictable as the financial markets to which theyre inevitably linked.

So, the revenue surge is neither a sign that the tax cuts are working nor of sustainable economic growth. A growing number of economists, most prominently from the Congressional Budget Office, point out that upsurges in revenue are also the result of growing income inequality in the United States, an observation that is consistent with mounting evidence of a rapidly widening gap between the rich and everyone else. As corporations and high- income Americans claim ever more of the economic pie, revenues rise, even if theres no increase in overall economic growth.

If Mr. Bush looked behind his headline numbers, he, too, could see that the rich are getting richer while the rest are, at best, only holding ground [and the poorest have been losing ground for nine years!]. It would make sense to use some of the windfall revenue to enact policies and programs that tilt against growing inequality. Unfortunately, hes flogging more tax cuts that will deepen the divide.

===

Will Bush ever get real?


tomder55 answered on 07/12/06:

Since the tax cut were passed in May 2003, GDP has expanded by more than 20%, or roughly $2.2 trillion. In the three years before the tax cut, GDP in the US grew just 10.4%.

The tax revenue trends suggest that government statistics are underestimating economic growth. People and companies do not pay taxes on income or profits they do not earn.


Erewhon rated this answer Excellent or Above Average Answer

Question/Answer
Erewhon asked on 07/12/06 - Bush Takes a Step Back


Terror and Power: Bush Takes a Step Back
By SCOTT SHANE
Published: July 12, 2006

WASHINGTON, July 11 From the outset, President Bush declared that the battle against Al Qaeda would be a war like no other, fought by new rules against new enemies not entitled to the old protections afforded to either prisoners of war or criminal defendants.

But the White House acknowledgment on Tuesday that a key clause of the Geneva Conventions applies to Qaeda detainees, as a recent Supreme Court ruling affirmed, is only the latest step in the gradual erosion of the administrations aggressive legal stance.

The administrations initial position emerged in 2002 only after a fierce internal legal debate, and it has been revised in the face of international opinion, Congressional curbs and Supreme Court rulings. Two central ideas of the war on terror that the president could fight it exclusively on the basis of his constitutional powers and that terrorist suspects had few, if any, rights have been modified repeatedly.

Scholars debated the meaning of a Defense Department memo made public on Tuesday that declared that the clause in the Geneva Conventions, Common Article 3, applies as a matter of law to the conflict with Al Qaeda.

Administration officials suggested that the memo only restated what was already policy that detainees must be treated humanely. But what was undeniable was that the presidents executive order of Feb. 7, 2002, declared that Article 3 did not apply to Al Qaeda or to Taliban detainees, and that the newly released memo, written by Deputy Defense Secretary Gordon R. England, said it did.

After the Pentagon released the memo, the White House confirmed that it had formally withdrawn part of the 2002 order and accepted that Article 3 now applied to Qaeda detainees. That article prohibits humiliating and degrading treatment of prisoners and requires trials affording all the judicial guarantees which are recognized as indispensable by civilized peoples.

This is an important course correction, and there are political ramifications to it, said Scott L. Silliman, an expert on the law of war at Duke University. Top defense officials never really clarified when Geneva applied and when it didnt, he said.

Richard H. Kohn, a military historian at the University of North Carolina, said the administration might have anticipated that it would have to adjust its policies, formed under immense pressure after the Sept. 11, 2001, terrorist attacks.

They were going to reach as far as possible to prosecute this war, and if they were forced to scale back, theyd scale back, Mr. Kohn said. Almost from the beginning, the administration has had to back away and fuzz up the issues.

If there has been a retreat, it may partly reflect a change in the perceived threat from Al Qaeda since the disorienting days after Sept. 11. As months, then years, passed without a new attack in the United States, the toughest measures seemed steadily less justifiable.

As time passed, and no more buildings were blowing up, it was no longer an emergency, and the rules had to be renegotiated, said Dennis E. Showalter, a professor of history at Colorado College.

In retrospect, all the contradictions that have emerged in the last four years were present in embryo in the 2002 presidential order.

The order began by noting that our recent extensive discussions had shown that deciding how Geneva rules would apply to Qaeda prisoners involves complex legal questions. It said that the conventions protections did not apply to terror suspects, but also that our values as a nation nonetheless call for us to treat detainees humanely, including those who are not legally entitled to such treatment.

In 2003, the administration decided that Article 3 would be applied to all prisoners captured in Iraq even non-Iraqi members of Al Qaeda. But the May 2004 revelations of abuse of prisoners at Abu Ghraib showed that the policy had not always been followed, and in response, the Defense Department repeatedly whittled down the list of approved interrogation techniques.

In 2004, the Justice Department reversed course as well, formally withdrawing a 2002 opinion asserting that nothing short of treatment resulting in organ failure was banned as torture.

In late 2005, the administration was forced to accept legislation proposed by Senator John McCain, Republican of Arizona, to ban cruel, inhuman or degrading treatment of prisoners held by the United States anywhere in the world.

In the meantime, the Supreme Court was knocking down some of the administrations key assertions of presidential power in the battle against terror.

In Rasul v. Bush in 2004, the court ruled that American courts had the authority to decide whether foreign terror suspects held at Guantnamo Bay, Cuba, had been rightfully detained. And on June 29, in Hamdan v. Rumsfeld, the court rejected the administrations rules for military commissions set up to try Guantnamo detainees, saying it had failed to seek Congressional approval and had fallen short of the standards set by law and the Geneva Conventions.

It was the Hamdan ruling that prompted Mr. Englands memo. It is my understanding, he wrote, that all current Defense Department rules were already in compliance with Article 3.

But Mr. Englands wording suggested that after all the policy adjustment since 2002, he was not certain everyone was operating from the same playbook: I request that you promptly review all relevant directives, regulations, policies, practices and procedures under your purview to ensure that they comply with the standard of Common Article 3.

Mr. Englands uncertainty was not surprising, Mr. Silliman said. Mixed messages over exactly which rules applied where, and which Geneva protections were to be honored and which ignored, were at the root of prisoner abuse scandals from Guantnamo to Iraq to Afghanistan, he said.

Its clear when you look at Abu Ghraib and everything else that there was a tremendous amount of confusion, Mr. Silliman said.

Even as legal experts parsed Mr. Englands memo, confusion lingered. The American Civil Liberties Union welcomed the memo as a first big step toward ending four years of lawlessness on detainee issues. But it also noted that in testimony Tuesday, other administration officials suggested that Congress simply adopt as law the proposed military commissions in exactly the form that civil libertarians say falls far short of Article 3.

That skepticism was shared by Martin S. Lederman, a former Justice Department official now at the Georgetown University law school.

The administration has fought tooth and nail for four years to say Common Article 3 does not apply to Al Qaeda, Mr. Lederman said. Having lost that fight, Im afraid theyre now saying, Never mind, weve been in compliance with Article 3 all along.

===

Well, well, well!

Well?


tomder55 answered on 07/12/06:

here is essentially my response to a simular question on another forum :

The President caved in . I don't like it and I don't like the Supreme's decision . They basically rewrote the Geneva Convention and gave terrorists and their organizations not covered under convention provisions the same rights as nation-state signatories .

The black robed oligarchs gave the President an out . Congress was more than willing to write laws for the terms of confinement and "court martial " that complied with existing standards . The Court (Stevens ) wrote that if Congress passed the legislation they could live with it . But Bush surrendered before they could and announced yesterday that (I guess) the courts interpretation of the Geneva treaty now is the standard (even though they completely distorted Geneva Article 3 which provides only a very basic right of humane treatment and was meant to apply to irregulars in non-international conflicts, like guerrillas in a civil war.
I guess they found their definition in the "emanations from penumbras ")
.................

here is my take . They were caught on the battle field out of uniform that makes them subject to "competent " tribunals and under Geneva terms subect to possible capital punishment . Let the tribunals begin.

Here is my favorite expert on SCOTUS issues;Mark Levin's take on this decision by Bush :

What the courts, Congress and, alas, the administration have now done by applying the Geneva Conventions protections to unlawful enemy combatants is to ensure that more and more warfare will involve terrorism since the rules of prisoner treatment apply to everybody, regardless of their behavior which means there are no rules of war. The result will be more horrific attacks on citizens, more wanton slaughter, and more terrorism. Like anything else if you reward certain behavior, you get more of it. We are now rewarding terrorism by extending lawful protections to its practitioners.

Erewhon rated this answer Excellent or Above Average Answer
ETWolverine rated this answer Excellent or Above Average Answer

Question/Answer
paraclete asked on 07/10/06 - If you ever wanted a reason to get the troops out of Iraq?

this is it! This is not how you win the hearts and minds of the people, this is not how you introduce the benefits of democracy! and for all those who might want to mitigate, there is no excuse.

Rape and murder: five more US soldiers charged
July 10, 2006 - 1:22AM


Steven Green, seen here in a March 2005 photo, is charged with killing and raping an Iraqi woman and killing her family.
Photo: AP

Five US soldiers were charged in a rape and multiple murder case that has outraged Iraqis, as documents obtained by Reuters on Sunday showed the rape victim was a minor aged just 14, and not over 20 as US officials say.

Days after former private Steven Green was charged as a civilian in a US court with rape and four murders, four serving soldiers were charged with the same offences, the US military said in statement that did not name the troops.

Another soldier, apparently a sixth member of Green's former unit in the 502nd Infantry Regiment, was charged on Saturday with dereliction of duty for not reporting the crime in March.

All five were charged with conspiring with Green, who is accused by US prosecutors of going with three others to a house near the checkpoint they were manning outside Mahmudiya, near Baghdad, and of killing a couple and their two daughters.

Those court documents gave the raped daughter's estimated age as 25, though US military officials in Iraq say their documents have her as 20.
Her identity card and a copy of her death certificate, however, show she was just 14.

Local officials and relatives had said she was 15 or 16.

Abeer Qasim Hamza al-Janabi was born on August 19, 1991 in Baghdad, according to the identity card, provided to Reuters by a relative. Issued in 1993, it features a photograph of her at 18 months, wide-eyed and with a lick of dark hair over her brow.

A copy of her death certificate, dated March 13, gives the same birth date.
She was found at home by a relative on March 12 and had died from "gunshot wounds to the head, with burns", that document, signed by doctor Wael Habib and a registrar, asserts.

With five Americans now facing the death penalty in the case, the fact the rape victim was a minor could be a factor in sentencing in the event of any convictions. Abeer's sister Hadeel was just six when she died of "several gunshot wounds".

The killers tried to burn the bodies and house to cover their tracks, relatives and local officials have said.

Iraqi Prime Minister Nuri al-Maliki, balancing a dependence on US firepower with a need to show Iraqis he is in charge, has voiced frustration with a mounting number of cases against Americans and wants a review of their immunity from Iraqi law.

Since revelations in March of a US probe into whether Marines killed 24 people at Haditha, Mahmudiya is the fifth case of serious crime being investigated by the military. In all, 16 troops have been charged with murder in the past month or so - as many as in the previous three years of the war.

Officers say generals are cracking down to try to curb harm to civilians that have turned Iraqis against the troops. One said a report submitted on Friday to the top general in Iraq should see action against Marine commanders who failed to act on evidence troops might have killed civilians at Haditha on November 19.

Green, 21, has since been discharged from the army due to a "personality disorder". The case came to light during stress counselling for a soldier last month following the kidnap and killing of two other men from the same unit near Mahmudiya.

A soldier cited in US court documents as the first witness told investigators that Green and three others drank alcohol and discussed rape. They then told the soldier to keep watch on the radio as they set off for the house, some in civilian clothes.

Two soldiers who said they went to the house accused Green of killing the parents and child before he and the other soldier in the home raped the woman. Green then shot her too, they said.

A sixth unidentified soldier is mentioned in court papers as discussing the case later with the first witness at their base.

The military said: "The five soldiers were charged in connection with their alleged participation in the rape and murder of a young Iraqi woman and three members of her family.

"The fifth soldier was charged with dereliction of duty for his failure to report the rape and murder ... but is not alleged to have been a direct participant in the rape and killings."

Reuters

tomder55 answered on 07/10/06:

I don't see the connection . A couple of criminal soldiers will have the full weight of the law come down on them . Their actions do not reflect the conduct of the coalition forces . What has this got to do with wheter we should continue our commitment to the new Iraqi gvt? none .The troops should be given a punishment in line with how we deal with rapists and murderers in America -- if they are found guilty.


Maliki's request about the immunity law , Order 17 ,that Viceroy Bremer set up during the CPA is reasonable . This article of course is spinning it against the troops when primarily his concern is about private contractors who also have immunity and probably should not .

The CPA set up the order when there was no effective Iraqi gvt. Now there is one and no doubt the terms and conditions of our presence there I'm sure will be renegotiated ;just like our military presence in other countries are subject to .

paraclete rated this answer Excellent or Above Average Answer

Question/Answer
Judgment_Day asked on 07/09/06 - SPORTS PORK SPENDING

By Patrick Hruby
Page 2


Having split the atom and put a man on the moon, the federal government has funded its most ambitious project yet.


Specifically, immortalizing the likes of Danny Biasone.


Haven't heard of Biasone, the father of the 24-second shot clock? Then you probably haven't heard of the Greater Syracuse Sports Hall of Fame, either.


Which, come to think of it, wouldn't be a total shock.



No offense to Danny Biasone -- but does Greater Syracuse really need a Sports Hall of Fame? Dedicated to the proposition that Syracuse, N.Y., is a world sports mecca, the Hall counts Biasone among its inductees and is notable for at least two reasons: (1) a dedication to the proposition that Syracuse is a world sports mecca; and (2) it does not include Jim Brown.


Beyond that, one other factor separates the Syracuse Hall from its peers in the field of petty regional homage, fellow cultural meccas such as the Alabama Health Care Hall of Fame and the National Corvette Museum.


Try $75,000. Of your money.


Two years ago, Congress tacked a grant for the Syracuse Hall onto a larger appropriations bill covering Veterans Affairs and Housing and Urban Development.


And no, it wasn't an accounting mistake.


"It wasn't a great deal of money," says Dan Gage, a spokesman for Rep. James T. Walsh (R-N.Y.), the congressman responsible for the funding. "There are similar projects all across the country. This was no different from what is being done elsewhere."


True enough. According to the nonpartisan watchdog group Citizens Against Government Waste, Congress spends more than $25 billion annually on pork-barrel projects -- budget-greasing, constituent-pleasing oinkers like the National Cattle Congress ($250,000) and the money-down-a-flushless-drain Waterfree Urinal Conservation Initiative ($1 million).


The really upsetting part? Some -- too much -- of that largesse goes to sports.


Think $1 million for the First Tee youth golf program. Nearly $73,000 to build a miniature red-brick replica of Oriole Park at Camden Yards. A cool $35,000 for the Alabama Sports Hall of Fame, which also doesn't honor Jim Brown.


Just this year, the same spendthrift legislative body that once signed off on $640 Pentagon toilet seats and a $50 million indoor rain forest -- in Iowa, of all places -- approved $50,000 for the Capitol Hill baseball and softball league.


Pay taxes? Thanks in advance. But don't expect a free league T-shirt.


"It's incredibly ridiculous what we'll spend money on," says Rep. Jeff Flake (R-Ariz.), a vocal critic of excessive federal spending. "Pork is just out of control."


Is it ever. Gaze upon your W-2s, America. And despair. The Pennsylvania Hunting and Fishing Museum gets $100,000. A program encouraging Iowa residents to exercise and eat healthy food gets $200,000.


Why are Flake's distinguished colleagues so eager to spend the people's moolah on stuff like the U.S. Soccer Foundation ($950,000)? Simple. Pork paves the road to reelection. Quid pro quo.


Rep. Walsh's district hosts the Syracuse Hall. The Richard Steele Boxing Club -- recently awarded a $100,000 federal earmark -- is located in Henderson, Nev., home state to Senate appropriator Harry Reid (D-Nev.).


Citizens Against Government Waste calls Sen. Robert Byrd (D-W.V.) "the King of Pork," a fitting title for a man with more than 30 federally funded projects to his name. (No, really -- they're named after him). The Monarch of White Meat is also the longest-serving senator in American history.


This is not a coincidence.


"I had an opponent in my last election that said, 'We need to vote Flake out because he doesn't bring home the bacon,' " Flake says. "You don't want your neighboring congressman's press release to be longer than yours."


Heaven forbid. Which is why many members of Congress view pork projects the way the rest of us view screaming infants: irritating and cringe-inducing when they belong to someone else, cute n' cuddly when they're your own.


Take the mini-Camden Yards. Located in Aberdeen, Md., the ballpark is being built by the Cal Ripken Sr. Foundation and will serve as the centerpiece of a baseball program focused on at-risk youth.


"A program like this will save a lot of kids that could have gone the other way," says Rep. Dutch Ruppersberger (D-Md.), who helped secure funding for the stadium. "I find it really offensive that you're attempting to help underprivileged children, and it's being called pork."


Ruppersberger has a point: like many pork projects, the Ripken stadium sounds worthy. But worthiness isn't the problem. Financial solvency is the problem.


Between an estimated deficit of $300 billion and escalating bills for Hurricane Katrina reconstruction and the war in Iraq, the federal government simply can't afford to fund character education through youth-league baseball. Not without borrowing even more cash.


Speaking of money and Iraq: Last summer, Ruppersberger praised the staff at the U.S. Army proving ground in Aberdeen for creating Humvee armor that helps protect soldiers, the same front-line troops the federal government has struggled to reimburse for self-purchased body armor.


Rep. Jeff Flake is going to battle over sports pork.Do the math. That $73,000 Congress spent on Ripken Stadium? It could have purchased 48 armor-plated vests at $1,500 a pop. Every dollar spent on sports pork is one less dollar spent on something more important.


Worse still, many pork recipients don't really need the money.


Remember the First Tee? The organization boasts a $5 million operating budget and a trustee list that reads like a Who's Who of CEOs. Congress still forked over $1 million in 2004. Last year, the Tiger Woods Foundation received a $100,000 grant toward the construction of an education center in California, despite the foundation's reported net assets of $32.6 million and a namesake who annually earns $50 million-plus.


From 2002 to last year, the Baseball Hall of Fame received a total of $1.57 million to produce once-a-year educational broadcasts covering topics such as women in baseball and the physics of pitching. $1.57 million. That's less than one percent of the New York Yankees' payroll, and not even equal to Alex Rodriguez's salary for a single month of the season.


"Why do they need the American taxpayer?" asks David Williams, policy vice president for Citizens Against Government Waste. "Baseball is flush with money."


Outraged yet? You should be. Yet before you blame Congress, take a long look in the mirror. Representatives carry out the people's will. And when the people are all too willing to let their duly elected officials drop $400,000 on the University of Rhode Island's Institute of International Sport and $990,000 on a California soccer center, all without a single peep of protest -- well, we get the bloated federal budgets we deserve.


More than that, we get budgets we demand.. Just ask John Rathburn, president of the Syracuse Hall. As a taxpayer, would he have voted to fund the Alabama Sports Hall of Fame?


"If I thought it would benefit the community," he says, "I would say yes."


Yes. The three-letter word that makes sports pork possible, the three-letter obscenity that makes Flake want to cry. And laugh. And cry some more, an overstuffed appropriations bill in hand.


After all, Rathburn's sentiment is hardly unique: two years ago, government dollars went to the Country Music Hall of Fame ($250,000), the Rock and Roll Hall of Fame ($350,000), even the Paper Industry Hall of Fame ($70,000).


"The National Cowgirls Hall of Fame was in there, for crying out loud," Flake says, sighing. "I'd like to nominate some of my colleagues for the National Pork Barrel Hall of Fame."


Of course, there's just one problem with Flake's modest proposal. Congress would probably fund it.


Patrick Hruby is a columnist for Page 2.


I'm an advocate of sports. Sports entertainment permits me, as the fan, to have a hobby and some lighthearted competition between friends. OK sometimes I get a little over board when my favorite baseball teams lose, but it's all in fun. I came across this article on ESPN and I think it has valid points concerning sports pork spending. In fact I'd much rather cut the pork and shoulder financial responsibility for our soldiers welfare abroad. Comments?

tomder55 answered on 07/09/06:

How could Syracuse have a sports hallof fame and not honor Jim Brown .He was the best football player of his time and that wasn't even his best sport .No one has ever come close to his ability on the Lacrosse field when he played both football and lacrosse at Syracuse. He was a first-team All-American in both .Besides that he was a great college basketball player and he lettered in track in the discus, high jump and sprints.

He completed his senior year in lacrosse beating arch-rival Army in the final game to secure a 10-0 season for Syracuse. Prior to the game he had competed in a track meet against Colgate where Brown scored 13 points in 3 events . He did not have time to change his clothes between events so he just threw on his lacrosse jersey over his track shorts ;grabbed his stick and ran out of the locker room just in tim to start the game.

He played one more game that season scoring 5 goals in one half against the nations best players in the North -South All star game.

If any athelete deserves recognition in Syracue it is Jim Brown .

.........................

Robert Byrd is the king of pork in the Senate and he makes no bones about it .

"They call me 'The Pork King,' they don't know how much I enjoy it."

He has has more than 30 public works named after him ;and the narcissistic Senator convinced the taxpayers of W.Vaginia to shell out funds to erect a statue of him in theState Capitol ;violating state law whick prohibits that until the pol is out of office and dead for 50 years. In his time he has secured over a billion dollars in pork spending for wasteful projects.

Has any of this helped lift W. Va. from it's poverty ? Nope ;it still ranks 49th in per-capita gross state product .So under his care-taking the state has become a defacto ward of the Federal Government .

Hope they add these little tid-bits to the statue honorong this former KKK grand pooh-bah [or was that "Exalted Cyclops"?].

........

I don't see an end to it . At the beginning of the year I thought that ear-mark reform would be part of the legislative achievments this year .But now congress is in recess and when they come back they will be in campaign mode. They have shelved many of the important issues that they debated this session .

Congress was given the constitutional power of the purse string . The reason that ear-mark spending has proliferated is because it is a fast track for re-election . In other words ;the electorate see more advantage in sending back to congress someone who is a wasteful spender than to hold them accountable for the practice. Representatives are measured by their ability to 'bring home the bacon'. The electorate buys ear-marks in exchange for votes. I know this to be a fact. I have been on committees on how to best divvy up 'Community Development Block Grant 'funds.The block grant program was created in the 1970s to help spur community economic revitalization and to improve a community's stock of housing for lower-income families. It provides financial grants directly to communities in accordance with a formula based on population, housing needs, and the number of low-income families in the community.
Never have I heard anyone say that 'we really do not need the funds this year'. It was always discussions on creative ways to get an even greater slice of the pie .

As bad as the system was ;at least the allocated funds were determined by local community priorities. I do not think any one would've considered it a priority for us to spend funds on local sports recognition but I would not put that past my Rep. in Congress who often times will make these funds available after a series of meetings in a luxury box at a Washington area arena.

Revelations of such wasteful spending is so bizzare that it becomes comical ,but instead of laughing about it which lets the Congress off the hook ,we should be sufficintly outraged to demand change. I do not see that happening either .

Judgment_Day rated this answer Excellent or Above Average Answer

Question/Answer
Erewhon asked on 07/08/06 - Welcoming or Worrying?

Hate Groups Are Infiltrating the Military, Group Asserts

By JOHN KIFNER
Published: July 7, 2006

A decade after the Pentagon declared a zero-tolerance policy for racist hate groups, recruiting shortfalls caused by the war in Iraq have allowed "large numbers of neo-Nazis and skinhead extremists" to infiltrate the military, according to a watchdog organization.

The Southern Poverty Law Center, which tracks racist and right-wing militia groups, estimated that the numbers could run into the thousands, citing interviews with Defense Department investigators and reports and postings on racist Web sites and magazines.

"We've got Aryan Nations graffiti in Baghdad," the group quoted a Defense Department investigator as saying in a report to be posted today on its Web site, www.splcenter.org. "That's a problem."

A Defense Department spokeswoman said officials there could not comment on the report because they had not yet seen it.

The center called on Defense Secretary Donald H. Rumsfeld to appoint a task force to study the problem, declare a new zero tolerance policy and strictly enforce it.

The report said that neo-Nazi groups like the National Alliance, whose founder, William Pierce, wrote "The Turner Diaries," the novel that was the inspiration and blueprint for Timothy J. McVeigh's bombing of the Oklahoma City federal building, sought to enroll followers in the Army to get training for a race war.

The groups are being abetted, the report said, by pressure on recruiters, particularly for the Army, to meet quotas that are more difficult to reach because of the growing unpopularity of the war in Iraq.

The report quotes Scott Barfield, a Defense Department investigator, saying, "Recruiters are knowingly allowing neo-Nazis and white supremacists to join the armed forces, and commanders don't remove them from the military even after we positively identify them as extremists or gang members."

Mr. Barfield said Army recruiters struggled last year to meet goals. "They don't want to make a big deal again about neo-Nazis in the military," he said, "because then parents who are already worried about their kids signing up and dying in Iraq are going to be even more reluctant about their kids enlisting if they feel they'll be exposed to gangs and white supremacists."

The 1996 crackdown on extremists came after revelations that Mr. McVeigh had espoused far-right ideas when he was in the Army and recruited two fellow soldiers to aid his bomb plot. Those revelations were followed by a furor that developed when three white paratroopers were convicted of the random slaying of a black couple in order to win tattoos and 19 others were discharged for participating in neo-Nazi activities.

The defense secretary at the time, William Perry, said the rules were meant to leave no room for racist and extremist activities within the military. But the report said Mr. Barfield, who is based at Fort Lewis, Wash., had said that he had provided evidence on 320 extremists there in the past year, but that only two had been discharged. He also said there was an online network of neo-Nazis.

"They're communicating with each other about weapons, about recruiting, about keeping their identities secret, about organizing within the military," he said. "Several of these individuals have since been deployed to combat missions in Iraq."

The report cited accounts by neo-Nazis of their infiltration of the military, including a discussion on the white supremacist Web site Stormfront. "There are others among you in the forces," one participant wrote. "You are never alone."

An article in the National Alliance magazine Resistance urged skinheads to join the Army and insist on being assigned to light infantry units.

The Southern Poverty Law Center identified the author as Steven Barry, who it said was a former Special Forces officer who was the alliance's "military unit coordinator."

"Light infantry is your branch of choice because the coming race war and the ethnic cleansing to follow will be very much an infantryman's war," he wrote. "It will be house-to-house, neighborhood-by-neighborhood until your town or city is cleared and the alien races are driven into the countryside where they can be hunted down and 'cleansed.' "

He concluded: "As a professional soldier, my goal is to fill the ranks of the United States Army with skinheads. As street brawlers, you will be useless in the coming race war. As trained infantrymen, you will join the ranks of the Aryan warrior brotherhood."

===

If this is true, would you welcome it or worry about it?


tomder55 answered on 07/08/06:

The 1996 crackdown on extremists came after revelations that Mr. McVeigh had espoused far-right ideas when he was in the Army and recruited two fellow soldiers to aid his bomb plot. Those revelations were followed by a furor that developed when three white paratroopers were convicted of the random slaying of a black couple in order to win tattoos and 19 others were discharged for participating in neo-Nazi activities.


In the 90's the administration was looking for skinheads and found 3;while thousands of gang members went unchecked.

The military is a microcosum of America inevitably affected by all the problems of
society at large ; including the spread of
gang-related crime and violence.I have also read that Crips and Bloods and Chicago's 'Folk Gangsters' have a presence in the military . A few thousand of over a million personnel is what we are talking about here .You're bound to get some riff raff , that does not mean the Army is being over-run

"There are people who want to become Soldiers because they want something better," he said. "That's a fact, and people do join gangs, but they also leave that behind in some instances to serve."
Too bad no one bothers to publish stories on how many gang-banger's lives were transformed and saved when they chose the alternative of joining the military .The military has historically been a viable and valuable option for many young people who have gotten off to a bad start .

Where there has been criminal gang related incidents they are treated and investgated as crime.

Erewhon rated this answer Excellent or Above Average Answer
Judgment_Day rated this answer Excellent or Above Average Answer
kindj rated this answer Excellent or Above Average Answer

Question/Answer
Itsdb asked on 07/07/06 - Forget sanctions, show films

Michael Moore is putting on his second annual Traverse City Film Festival, complete with a Salute to Iranian Cinema. As he puts it, "a sort of "Let's get to know them first this time!" effort." No word on whether there will be subtitles or if you need to brush up on your Farsi.

The choices:

    Men at Work, Directed By: Mani Haghighi

    Four men from Tehran are on their way to the mountains for a weekend ski trip. As they round a curve, they encounter a boulder that sits on the edge of a cliff. Together they decide that the boulder must be pushed off that cliff. And, for the next 80 minutes, that's what we see them try to do in this funny, poignant allegory by the acclaimed Iranian director Mani Haghighi. No matter what they try to do, the rock won't budge. And neither will they. This is a great, small film that has many big things to say. We are proud to welcome from Tehran the director, Mani Haghighi, as part of our Salute to Iranian Cinema. Mr. Haghighi will speak after the film. Not Rated

    President Mir Qanbar, Directed By: Mohammed Shirvani

    A poor, 74-year-old man by the name of Mir Qanbar decides that anyone can grow up to be president of Iran. So he declares his candidacy and sets off on the campaign trail with his mule and his loyal friend, Seifollah, to the many remote, poverty-stricken villages of his district. He has a cart and a bicycle and a megaphone. He promises everyone he meets that he will clean up the government and represent the little guy. Director Mohammed Shirvani has brilliantly captured the quiet dignity and determination of Mir Qanbar in this exquisite documentary about a man to whom democracy is not just a word or a promise. Not Rated


And my personal choice...

    Iron Island, Directed By: Mohammad Rasoulof

    In this astonishing film directed by Mohammad Rasoulof, a huge abandoned oil tanker sits a few miles off the coast of Iran. Captain Nemat, played by Ali Nasirian, establishes his own society on the ship and others join him as they create a floating city. With their own money, power, clothing factory, and jobs, they are self sufficient, but Nemat rules with an iron fist. His absolute power is tested when two young lovers, under increasing pressure from outside influences, defy his authority. This allegory for contemporary Iranian society is funny one moment and brutal the next. Part of our Salute to Iranian Cinema. Not Rated


Personally I think Men at Work might be pretty funny, President Mir Qanbar may be moving, but Iron Island may not have the desired effect.

To make this work Iran needs to reciprocate, any suggestions on American films to show in Iran? Oh, that's right, Iran has banned American films.

tomder55 answered on 07/08/06:

During my time in Tehran I went to the movies frequently . As I recall their favorites were Clint Eastwood spaghetti westerns. My choice would be 'High
Plains Drifter '.The surreal setting ;a town with a populace that desperately wants change but is afraid to take matters into their own hand seems symbolic of Iran society today .

'Men at Work' I saw that scenario played out a few times on the Iranian streets. Let's say the boulder was a stalled car in the middle of the road instead . The people in the car would get out ;open the hood and stare into the engine. Many more passers-by would gather around also staring into the engine ;begin a debate that sometimes led to fisticuffs. No one would think that with all the people gathered that they had the muscle to move the car to the side of the road so others could pass by.

I'm curious why Moore did not include such films as 'Our Times' by Iranian female director Rakhshan Bani-Etemad . It is a story about a women who cannot get her name on the Presidential Ballot and also cannot find a home for her mother and daughter because single women are discriminated against in modern Iran. Another movie that deals with the treatment of women in Iran is 'The Circle ' by Iranian director Jafar Panahi . Not suprising;this film is also banned in Iran.

Itsdb rated this answer Excellent or Above Average Answer
purplewings rated this answer Excellent or Above Average Answer

Question/Answer
Itsdb asked on 07/07/06 - US foils 'New York tunnel plot'

US authorities say they have disrupted the early stages of a plot to attack New York City's mass transit system.

The alleged plot was discovered during routine monitoring of internet chatrooms used by extremist groups...

Democratic Senator Charles Schumer of New York said this was one instance "where intelligence was on top of its game and discovered the plot when it was just in the talking phase".

A number of plots targeting subways, tunnels and other New York City landmarks have come to light since the 2001 attacks on the World Trade Center."

The NY Times headline tomorrow?

Bush administration secretly monitoring internet conversations
Program monitors chat rooms

The US government, without the knowledge of private American citizens, has engaged for years in a secret effort to eavesdrop on private internet conversations...

tomder55 answered on 07/07/06:

How long before the MSM claims that the plot "wasn't that serious"? .

Truth told however their scheme would've killed alot of motorists but I doubt it would've breeched the tunnel walls and even if it did ;the Holland tunnel is dug into a solid layer of bedrock ;it doesn't actually sit on the bottom of the Hudson .Had it flloded the tunnel it would not have flooded the Wall Street district because that sits above sea level .

A number of plots targeting subways, tunnels and other New York City landmarks have come to light since the 2001 attacks on the World Trade Center."

ummm Michael Chertoff did you hear that ? let me repeat it :

A number of plots targeting subways, tunnels and other New York City landmarks have come to light since the 2001 attacks on the World Trade Center."

Itsdb rated this answer Excellent or Above Average Answer

Question/Answer
.Choux. asked on 07/06/06 - Disaster Everywhere One Looks, Thanks Bush Crime Family!

"From deteriorating security in Afghanistan and Somalia to mayhem in the Middle East, confrontation with Iran and eroding relations with Russia, the White House suddenly sees crisis in every direction.

North Korea's long-range missile test Tuesday, although unsuccessful, was another reminder of the bleak foreign policy landscape that faces President Bush even outside of Iraq. Few foreign policy experts foresee the reclusive Stalinist state giving up the nuclear weapons it appears to have acquired, making it another in a long list of world problems that threaten to cloud the closing years of the Bush administration, according to foreign policy experts in both parties.

"I am hard-pressed to think of any other moment in modern times where there have been so many challenges facing this country simultaneously," said Richard N. Haass, a **former senior Bush administration official** who heads the Council on Foreign Relations. "The danger is that Mr. Bush will hand over a White House to a successor that will face a far messier world, with far fewer resources left to cope with it."
Washington Post dot com.

&&&&&&&&&&&&&&&&&&&&&&&&&&&&&&&&&&&&&&&&&&&&&&&&&&


What a mess Bush has created in foreign affairs!

Let's see, perhaps things are better at home...

illegal immigration, they are flooding over our southern border

federal deficit, Oy!

government spending, plan D Medicare, what a fiasco!

Price of gasoline, sky high!

pollution of all kinds, *cough*

approval of the president, about 30%

on and on and on....

tomder55 answered on 07/07/06:

I agree with the experts who identified the foreign policy issues he inherited as being challenging no matter who was President .

Few foreign policy experts foresee the reclusive Stalinist state giving up the nuclear weapons it appears to have acquired , making it another in a long list of world problems that threaten to cloud the closing years of the Bush administration, according to foreign policy experts in both parties.

If indeed they have acquired nukes you can thank the slick one and his staff of appeasers ;who's kewpie doll tap dance that was a substitute for honest diplomacy around the issue of N Korea's proliferation ;was the reason they were able to get nukes in the first place . btw ;you failed to point out that one of the chief policy advisors to Clinton {whilst Bill Clinton was otherwise engaged with Monica Lewinski and others in sex parlors },Madeleine Albright ,was also quoted in this hit piece from the Compost .







Did you also note that a number of Clintoon advisers were suggesting that we shoot the TD-2 down on the launch pad . So who was suggesting the use of the military option last week .... Bush or the Clintonistas ?

Richard N. Haass was a close associate with Colin Powell in the White House . He was also in GHW Bush's adm. .I would consider him part of the 'realist' school of foreign policy and as you know I am not a big fan of stability-for-stabilitie's-sake (although there are times it is temporarily useful).It does not suprise me that he thinks the US has 'fewer resources 'for dealing with global challenges . I see the opposite . We now have a battle hardened military who know how to fight jihadistan in their territory . Challenges in the Far East if they come to military confrontaion will have the Navy as the lead force .I think USPACOM and Admiral William J. Fallon are up to the challenge .

It just never ceases to amaze me . Reagan stood down the Soviet Union with 10s of thousands capable nukes pointed in our direction but N.Korea fires off a dud TD-2 and it's panic-ville. During the 1980s our military was built to take on confrontations in 2 1/2 theaters of operations simulataniously . Even with the draconian cuts in the 1990s, (the so called 'peace dividend 'that the Clintonoids used along with huge tax increases to temporarily balance the budget) ,the military is still more than capable of dealing with the end game in Afghanistan and Iraq as well as being a tool of foreign policy elswhere on the globe if needed .

I won't get into a point by point discussion on the domestic issues .Some of the items you list I agree with and others I do not think the performance of the executive is particularily relevent .Suffice it to say that I think he did a hell of a job bringing us out of the economic shock from the combined tech-stock bubble burst and the economic effects of 9-11 . I also like the direction he is steering the courts toward . Other domestic issues I have been less than pleased with the results .

.Choux. rated this answer Average Answer
Itsdb rated this answer Excellent or Above Average Answer
kindj rated this answer Excellent or Above Average Answer
labman rated this answer Excellent or Above Average Answer

Question/Answer
Itsdb asked on 07/06/06 - If you hadn't heard...

Since it was an issue last week, I thought it only fair to point out that "Rush Limbaugh will not face charges in Palm Beach County for the bottle of Viagra found in his luggage that was prescribed in his psychologist's name, prosecutors said Wednesday."

The best take on it? Judi McLeod at Canada Free Press said, Now the drive-by media has a reason to hate Rush Limbaugh. Fearful of news that is not politically correct, they went out on a run to take down Rush.

Wait `till it hits home that it took a Rush Limbaugh to give the backbiting boys of the mainstream media a sure cure for their advanced case of Erectile Dysfunction.


Ouch! LOL :)

tomder55 answered on 07/06/06:

He said that he will cancel all his international travel plans until the agreed upon probation period ends because he does not want to get set up by customs personnel again . Besides having to deal with a gloating agent evidently the news was leaked to the press before he left the airport .

Meanwhile all the predictable jokes were said ;by far the best by Robin Williams :

I mean, why do you need Viagra? Its a hard thing when hes a big prck already.?"

Where was he staying in the Caribbean? Club Medicated?

"Rush was in the Caribbean to go fly-fishing as he pretended to use his nether regions as a rod and reel. "

ETWolverine rated this answer Excellent or Above Average Answer
powderpuff rated this answer Excellent or Above Average Answer
Itsdb rated this answer Excellent or Above Average Answer

Question/Answer
Erewhon asked on 07/05/06 - Why is the federal government suddenly trying to block state efforts to protect public health throug

Bullies Along the Potomac
By NINA MENDELSON
Published: July 5, 2006

WITH conservatives controlling the White House and Congress, one might expect states' rights to be firmly in vogue. But several recent federal actions have seriously weakened the states' efforts to protect health and the environment.

In March, for example, the House of Representatives passed, and the Senate is considering, the benign-sounding "National Uniformity for Food Act," which would bar states from addressing food-borne hazards, leaving food safety solely to the Food and Drug Administration. Thirty-nine state attorneys general signed a letter opposing the act.

The proposed law would keep California from applying its highly effective Proposition 65 a law meant to protect people from carcinogens to foods. This year, the state used Proposition 65 to stop Pepsi from selling soda bottles with labels that contain lead. The federal proposal would also keep Michigan and Connecticut from requiring labels for sulfite-containing dried fruit, which can cause serious allergic reactions.

This act is no anomaly. A report last month from Representative Henry Waxman, Democrat of California, shows that since 2001 Congress has enacted 27 laws that pre-empt state authority in areas from air pollution to consumer protection.

The Bush administration is following Congress's lead but in a quieter way that is likely to undermine states even more. Its strategy to block California limits on greenhouse-gas emissions from cars (which 10 other states plan to adopt) is a case in point. Since 1967, California has been allowed to set its own automotive pollution limits, subject to limited review by the Environmental Protection Agency. But this spring, the Department of Transportation stepped in by inserting into its new fuel-economy standards for light trucks a statement that the exclusive federal authority to set fuel-economy standards bars California's emissions limits, because car manufacturers might comply with emission limits by increasing fuel efficiency. The car industry will surely use this to reinforce its position in court that California cannot set greenhouse-gas limits.

The F.D.A., for its part, is moving to limit states' authority over prescription drug labels. Buried in the preamble to the agency's new labeling standards meant to make drug labels easier to read is a statement that state agencies and courts cannot require any safety information beyond what the F.D.A. requires. GlaxoSmithKline has already cited the F.D.A. statement in court, arguing that the company is not responsible for failing to disclose that Paxil increases suicide risk.

The F.D.A. has also moved faster than Congress to oppose California's food safety efforts. The agency tried to block a lawsuit brought by California's attorney general against tuna manufacturers for failing to place warnings on tuna cans about the dangers of mercury. Because of the risks to fetal development and children, the F.D.A. itself has recommended limits on tuna consumption, but it has stopped short of requiring labels. In the California case, at the tuna companies' request, the agency wrote a letter to the attorney general accusing California of interfering with the F.D.A.'s wish "to avoid overexposing consumers to warnings" of tuna's risks. In May, the judge ruled that the state could not require any extra warnings.

Federal environmental and health rules have historically provided a floor of minimum protection. States, for their part, have led the way on countless matters, from requiring health insurers to cover mammograms to stringently regulating mercury emissions from power plants.

So why is the federal government suddenly trying to block state efforts to protect public health through bureaucratic actions largely outside the public view? The unfortunate result is that big businesses' revenues are being shielded, while protections for consumers and the environment are being stripped away.

Nina Mendelson, a professor at the University of Michigan Law School, is a scholar at the Center for Progressive Reform.


===

Q: Why is the federal government suddenly trying to block state efforts to protect public health through bureaucratic actions largely outside the public view?


tomder55 answered on 07/06/06:

I would say that there is a long history in this country ;dating back to Teddy Roosevelt and the muck raking era of early industrialization; of Federal efforts to standardize issues regarding consumer protection . This is not meant to dilute protections but to enhance protections . Interstate commerce increased and the advent of a mobile populations made the effort ever more imperitive .

A recent example before the hated GWB became President is the 'The Food Quality Protection Act of 1996'which mandates a single, health-based standard for pesticides in foods; provides special protections for infants and children; expedites approval of safer pesticides, and creates incentives for the development of effective crop protection tools for American farmers.

It is not just in foods ,but also in medicines ;banking ;labor relations and laws ,and handicap accessability among others where Federal standards have been set.

Normally the Democrats are gung-ho for more Federal meddling in issues like this . I don't buy their arguement that safety will be compromised .If they think that California standards are better then they sould work the make that the national standard . If there is a public health concern about food quality in California it should be the same concern for the other 49 states .A scientifically based, consistent standards will apply nationwide; food cannot be considered safe in one state and unsafe in another.


From an industry point of view the new law would make standardized labelling which would be a benefit because the food made for one state would be compliant in all the states and the need for customized formulating and labelling would be eliminated ;a cost savings anyway you look at it .

It will benefit both consumers and businesses I think because the standardization will replaces a patchwork quilt of state regulations governing food safety and warning labels with a single, national standard. I already work in an industry (health products )where labelling is standardized . It is a logical thing to do and it helps educate consumers about what they use .

The law provides for making food safety regulation for packaged food consistent across all 50 states and under the jurisdiction of the Food and Drug Administration (FDA).
Provides consumers with a single set of consistent, science-based food safety regulations.
Curbs regulatory burdens on all businesses by setting a single set of national guidelines.
Extends to packaged food the same consumer protections as in other areas of federal regulation including meat, poultry, nutrition labeling, and drugs and medical devices.
Establishes a procedure requiring the FDA to evaluate state regulations different than its own, and consider applying that states standards nationally.

Appears to be a win win for both consumer and industry to me.


Erewhon rated this answer Excellent or Above Average Answer

Question/Answer
paraclete asked on 07/05/06 - Paranoia runs high as Kim defies us all?

U.S. officials: North Korea tests long-range missile

Wednesday, July 5, 2006; Posted: 12:03 a.m. EDT (04:03 GMT)

WASHINGTON (CNN) -- North Korea test-fired a long-range missile and five shorter-range rockets early Wednesday, but the closely watched long-range test failed within a minute, U.S. officials said.

The tests began shortly after 3:30 a.m. local time (2:30 p.m. Tuesday ET) and lasted for about five hours.

The Taepodong-2 missile, which some analysts believed capable of hitting the western United States, failed after about 40 seconds, U.S. officials said.

The U.N. Security Council planned to meet Wednesday morning to discuss North Korea's actions.

North Korean Foreign Ministry officials confirmed the tests Wednesday to reporters for two Japanese broadcasters, NHK and TV Tokyo.

U.S. National Security Adviser Stephen Hadley described the missile launches as "provocative behavior," but said they posted no immediate threat to the United States.

Washington dispatched Christopher Hill, its top negotiator in the six-party talks with the two Koreas, Japan, China and Russia, to consult with U.S. allies in Asia after the tests, Hadley said.

Hill has been the top U.S. negotiator in the six-party talks aimed at convincing North Korea to give up its nuclear weapons program.

A statement from the White House said the United States "strongly condemns" the launches and North Korea's "unwillingness to heed calls for restraint from the international community."

"We are consulting with international partners on next steps," the statement said.

"This provocative act violates a standing moratorium on missile tests to which the North had previously committed."

The United States and Japan had urged Pyongyang to stick with the moratorium on long-range missile tests it declared in 1999, after it fired a Taepodong-1 missile over Japan in 1998.

"We can now examine what the launches tell us about the intentions of North Korea," Hadley told reporters.

Washington and North Korea's Asian neighbors have been trying to convince North Korea to give up its nuclear program since 2002. Analysts called the tests an effort by North Korea to redirect attention to those talks.

"North Korea's point here is that they have capabilities, growing capabilities, and that they should be taken in a very serious way," said Wendy Sherman, a former State Department official who held talks with North Korean leader Kim Jong-Il during the Clinton administration.

Watching preparations
Intelligence agencies around the region had been watching preparations for the long-range test, but the shorter-range missiles were launched from a different site. At least four of those missiles were variants of the Soviet-era Scud series, with ranges estimated from about 100 to over 600 miles.

The Taepodong-2 landed about 200 miles west of Japan in the Sea of Japan, a U.S. military source said.

A spokesman for South Korean President Roh Moo-hyun said after a National Security Council meeting Wednesday that North Korea must take responsibility for events resulting from its firing of the missiles.

Shinzo Abe, Japan's chief Cabinet secretary, said the test was a source of "grave concern."

Abe said Japan, which provides an extensive amount of food aid to North Korea, would respond to the tests with a strong protest. Japan has previously suggested it would withhold some of that aid or limit trade with Pyongyang if North Korea conducted a test.

A Japanese foreign ministry press official, Akira Chiba, told CNN that Japan was studying "stern measures" and these would be announced shortly.

Jim Walsh, a national security analyst at the Massachusetts Institute of Technology, said the intent of the test appeared to be aimed at drawing attention back to North Korean demands in the six-party talks. But Walsh said the tests "do not represent an immediate military threat to the United States."

'Difficult technology'
"It's very difficult technology. They very clearly have not mastered it," he said. "Most estimates are they will not master it for another 10 years."

A senior State Department official said a response would be coordinated among the remaining members of the talks, with Japan likely to take the lead. But the Bush administration does not want to "overplay" the tests, the official said.

At the United Nations, U.S. Ambassador John Bolton said he was "urgently consulting" with other members of the 15-nation Security Council.

President Bush met with Hadley, Defense Secretary Donald Rumsfeld and Secretary of State Condoleezza Rice as the tests were going on, a senior administration official said. But Bush will go ahead with plans to watch Independence Day fireworks and hold a gathering at the White House for his 60th birthday, the official said.

State Department officials said Tuesday that fuel trucks had departed the site where the Taepodong-2 had been set on a launch pad, indicating that a test may have been near.

And on Monday, North Korea's state-run media accused the United States of harassing it and vowed to respond to any pre-emptive attack "with a relentless annihilating strike and a nuclear war with a mighty nuclear deterrent." (Watch why North Korea is talking about annihilating the U.S. -- 2:04)

The White House has dismissed that threat as "hypothetical." (Full story)

But the U.S. Northern Command increased security measures at its Cheyenne Mountain Air Force Station in Colorado Springs, Colo., a few weeks ago, a military official confirmed Tuesday.

The base is the seat of the North American Aerospace Defense Command, and some of its command-and-control operations might have been used if the United States attempted to use its ballistic missile interceptors -- which have a mixed record of success -- to shoot down a potential Taepodong-2 test.

Rick Lehner, a spokesman for the U.S. Missile Defense Agency, told CNN that two interceptor missiles were activated at Vandenberg Air Force Base in California in anticipation of the test and could have been fired by controllers at NORAD. Lehner said nine other interceptors were activated at Fort Greely, Alaska.

CNN's David Ensor, Barbara Starr, Kyra Phillips, Elise Labott, Justine Redman, Atika Shubert, Sohn Jie-Ae, Stan Wilson and Ed Henry contributed to this report

tomder55 answered on 07/05/06:

hmmm ....just about the same time that the Shuttle was launched yesterday .

I do not think it a coincidence either that this happened on the same day that the Iranian puppets in Gaza demonstrated that they could launch a better grade Qassam rocket that landed on a school house in Ashkelon .This escalation by both the Palestinians (with Iran backing ) and by N Korea was a coordinated event and should not be seen as separate .

Itsdb rated this answer Excellent or Above Average Answer
paraclete rated this answer Excellent or Above Average Answer

Question/Answer
paraclete asked on 07/05/06 - It's all too hard, and guess who has won after all?

CIA gives up on bin Laden
From: Agence France-Presse
From correspondents in Washington

July 05, 2006


THE CIA has closed a unit tasked with capturing Osama bin Laden and his top deputies, the New York Times has reported.
Members of the unit, formed in 1996, have been reassigned to other duties, although hunting top Al-Qaeda leaders remains a priority, officials close to US intelligence agencies told the daily.

"There are still people who wake up every day with the job of trying to find bin Laden," one official, who spoke on condition of anonymity, was quoted as saying.

The decision was taken because US officials believe Al-Qaeda is no longer as "hierarchical" as it once was, and amid growing threats from Al-Qaeda-inspired groups acting independent of bin Laden and his number two, Ayman al-Zawahiri, the Times said.

"It reflects a belief that the agency can better deal with high-level threats by focusing on regional trends rather than on specific organisations or individuals," the daily wrote.


Bin Laden claimed responsibility for ordering the September 11, 2001 attacks on the United States that killed close to 3000 people.

tomder55 answered on 07/05/06:

probably a wise move . We know where he is but it would be a diplomatic nightmare at this point to go in and get him. He is contained and although he still has mythical influence on al-Qaeda I think they are generally correct in their assessment that the organization is fractured and de-centralized . Don't worry ,he will not die of old age .

Judgment_Day rated this answer Excellent or Above Average Answer
labman rated this answer Excellent or Above Average Answer
paraclete rated this answer Excellent or Above Average Answer

Question/Answer
.Choux. asked on 07/04/06 - Real Victims of Faux News are the Radical Right Who Believe Them

"When I was covering the war in Iraq, we reporters would sometimes tune to Fox News and watch, mystified, as it purported to describe how Iraqis loved Americans. Such coverage (backed by delusional Journal editorials baffling to anyone who was actually in Iraq) misled conservatives about Iraq from the beginning. In retrospect, the real victims of Fox News weren't the liberals it attacked but the conservatives who believed it.

Historically, we in the press have done more damage to our nation by withholding secret information than by publishing it. One example was this newspaper's withholding details of the plans for the Bay of Pigs invasion. President Kennedy himself suggested that the U.S. would have been better served if The Times had published the full story and derailed the invasion.

Then there were the C.I.A. abuses that journalists kept mum about until they spilled over and prompted the Church Committee investigation in the 1970's. And there are secrets we should have found, but didn't: in the run-up to the Iraq war, the press particularly this newspaper was too credulous about claims that Iraq possessed large amounts of W.M.D.

In each of these cases, we were too compliant. We failed in our watchdog role..." Nicholas Christoff NY Times

~~~~~~~~~~~~~~~~~~~~~~~~~~~~~~~~~~~~~~~~~~~~~



Comments?

tomder55 answered on 07/05/06:

I hear the Slimes hired Tokyo Rose as their new public relations manager .

It is beyond comical that Christoff is so naive as to believe that Fox news is the sole source of information for Conservatives . I'm sure he thinks that because he believes the rest of the country flocks to his rag as the single source of 'news fit to print' .

It is eually laughable that he thinks that the damage done by the Church Commitee was good for the country .

From 1789 to 1974 Presidents as commanders in chief were given wide latitude to conduct clandestine operations they believed were in the national interest. President Washington, in his first annual message to Congress requested a Contingency Fund. Washington was given $40,000, not chump change in the early 1790s. He was not required to report how he spent this money, he merely had to divulge the amount of money spent, without revealing to whom or for what reasons it had been spent. Jefferson, Madison, Andrew Jackson, and Lincoln, all authorized clandestine operations out of this fund, and did not report the details to Congress. This pattern persisted until the mid-1970s with little or no change, other than the increasing size and bureaucratization of the nations intelligence services .

The real aberration was the Church Commitee when the United States granted Congress the greatest control over intelligence matters of any Western nation, and overturned the system which had prevailed in the United States since the Founding.

The damage done to the CIA was extensive. Congress for the 1st time got line item budgetary control over the CIA. As you know Congress loves to pork barrel and they had a field day with the CIA budget ;forcing expensive but unnecessary toys on the agency as it substituted manpower in the field for high-tech gadgets .

Leaks about CIA activities by Congressional members and staff began to explode and in very few cases has the offending party been disciplined. As a perfect example ;Senator Robert Torricelli of New Jersey led the charge in the 1990s to prevent the CIA from hiring unsavory characters. Torricelli defended State Department employee Robert Nuccio, who leaked classified material dealing with CIA operations in Guatemala to Torricelli, who in turn held a press conference and revealed the information to the media. It was these revelations that led to congressional restrictions on the ability of agents in the field to deal with bad people.

They turned the CIA into cautious bureaucrats who avoid the risks that come with taking action, who fill out every form in triplicate......just in time for 9-11 .

This is the results that Christoff champions ? What a suprise !

Since the Slimes story broke banks have reported an outflow of deposits from the accounts being monitored to other banks centered in Europe. Muslim banks are disfavored by terrorists because the Koran prohibits paying interest on deposits.

"Training, feeding, housing and arming terrorists requires money...Tracking this money through our banking system is one tool we've used to try to prevent attacks on Americans. The effectiveness of this tool has been undermined by the Times article. So, it looks like an elevated level of risk is something else we'll have to get used to, courtesy of the New York Times."John W. Snow Treasury Sec.

.Choux. rated this answer Above Average Answer
Itsdb rated this answer Excellent or Above Average Answer

Question/Answer
katiy asked on 07/04/06 - Check your facts

When the Senate voted to go to war, the majority at that time was Democrat.

tomder55 answered on 07/04/06:

in 2005 the Congress weighed in (Detainee Treatment act )on this issue and not only affrimed the Bush policy but also told SCOTUS specifically they had no business in the issue. Under the DTA, the only court that is authorized to hear appeals brought by captured terrorists is the United States Court of Appeals for the District of Columbia which had ruled in favor of the federal government. If the Supreme Court lacked jurisdiction to hear this case, then why should the President, the Congress, and the American people obey its ruling? Why should we allow five unelected, unaccountable, and unrepresentative judges decide one of the most important issues of the day ? The Supreme Courts actual authority is only as deep as the willingness of the other branches of government and the American people to obey its commands. At some point, we have to say, enough is enough. I think we have reached that point .

ETWolverine rated this answer Excellent or Above Average Answer
katiy rated this answer Excellent or Above Average Answer

Question/Answer
Erewhon asked on 07/03/06 - I'd like your opibnions - without foul language - on this piece:

July 1 - 2, 2006
What's to Stop Him?
Bush's Assaults on Freedom

By PAUL CRAIG ROBERTS

On June 29 the US Supreme Court in a 5-3 decision ruled that President Bush's effort to railroad tortured Guantanamo Bay detainees in kangaroo courts "violates both US law and the Geneva Conventions."

Better late than never, but it sure took a long time for the checks and balances to call a halt to the illegal and unconstitutional behavior of the executive.

The Legal Times quotes David Remes, a partner in the law firm of Covington & Burling: "At the broadest level, the Court has rejected the basic legal theory of the Bush administration since 9/11--that the president has the inherent power to do whatever he wants in the name of fighting terrorism without accountability to Congress or the courts."

Perhaps the Court's ruling has more far reaching implications. In finding Bush in violation of the Geneva Conventions, the ruling may have created a prima facie case for charges to be filed against Bush as a war criminal.

Many readers have concluded that Bush assumed the war criminal's mantle when he illegally invaded Iraq under false pretenses. The US itself established the Nuremberg standard that it is a war crime to launch a war of aggression. This was the charge that the chief US prosecutor brought against German leaders at the Nuremberg trials.

The importance of the Supreme Court's decision, however, is that a legal decision by America's highest court has ruled Bush to be in violation of the Geneva Conventions.

There are many reasons to impeach Bush. His flagrant disregard for international law, US civil liberties, the separation of powers, public opinion and human rights associate Bush with the worst tyrants of the 20th century. It is true that Bush has not yet been able to subvert all the institutions that constrain his executive power, but he and his band of Federalist Society lawyers have been working around the clock to eliminate the constraints that the US Constitution and international law place on executive power.

Republicans are "outraged" that "liberal judges" have prevented Bush from "protecting us from terrorists." In the US Senate, Majority Leader Bill Frist said that Republicans will propose legislation to enable Bush to get around the Supreme Court's decision. Senator Arlen Spector (R, PA) already had a bill ready. What sense does it make to talk about "liberal opposition" when liberal Republicans like Spector are falling all over themselves to kow-tow to Bush.

Americans are going to have to decide which is the greater threat: terrorists or the Republican Party's determination to shred American civil liberties and the separation of powers in the name of executive power and the "war on terror."

The rest of the world has already reached a decision. A Harris Poll recently conducted for the Financial Times found that the populations of our European allies--Britain, France, Italy and Spain--view the United States as the greatest threat to global stability.

A Pew Foundation survey released the same week found that 60 percent of the British believe that Bush has made the world less safe and that 79 percent of the Spanish oppose Bush's war on terror.

Republicans and conservatives equate civil liberties with homosexual marriage, abortion, racial quotas, flag burning, banning of school prayer, and crime resulting from a lax punishment of criminals. This is partly the fault of the ACLU and leftwingers, who go to extremes to make a point. But it is also the fault of conservatives, who believe that their government is incapable of evil deeds.

In their dangerous and ill-founded belief, conservatives are in total opposition to the Founding Fathers, who went to the trouble of writing the Constitution and the Bill of Rights in order to protect us from our government. Most conservatives believe that they do not need constitutional protections, because they "are not doing anything wrong." Conservatives have come to this absurd conclusion despite the Republicans' decision to sell out the Bill of Rights for the sake of temporary power.

A number of important books have recently been published that decry America's decaying virtue. In Lawless World, the distinguished British jurist, Philippe Sands, documents the destruction by George Bush and Tony Blair of the system of international law put in place by Franklin D. Roosevelt and Winston Churchill. In The Peace of Illusions, Christopher Layne documents the American drive for global hegemony that threatens the world with war and destruction. Americans are enjoying a sense of power with little appreciation of where it is leading them.

Congress has collapsed in the face of Bush's refusal to abide by statutory law and his "signing statements" by which Bush asserts his independence of US law. Bush has done what he can to turn the Supreme Court into a rubber stamp of his unaccountable power by placing John Roberts and Samual Alito on the bench. Though much diminished by these appointments, the Court found the strength to rise up in opposition to Bush's budding tyranny.

Amazingly, on the very same day in England, where our individual rights originated, the High Court struck down Tony Blair's "anti-terrorism" laws as illegal breaches of the human rights of suspects. As with the Bush regime, the Blair regime tried to justify its illegality on the grounds of "protecting the public," but a far larger percentage of the British population than the American understands that the erosion of civil liberty is a greater threat to their safety than terrorists.

Thus, in the two lands most associated with civil liberties, courts have struck down the tyrannical acts of the corrupt executive. Perhaps the fact that courts have reaffirmed the rule of law will give hope and renewed strength to the friends of liberty to withstand the assaults on freedom that are the hallmarks of the Bush and Blair regimes. On the other hand the two tyrants might ignore the courts as they have statutory law.

What's to stop them?

Paul Craig Roberts was Assistant Secretary of the Treasury in the Reagan administration. He was Associate Editor of the Wall Street Journal editorial page and Contributing Editor of National Review. He is coauthor of The Tyranny of Good Intentions.He can be reached at: paulcraigroberts@yahoo.com

===

Reasoned opinions please. No foul language!

tomder55 answered on 07/04/06:

hard not to use foul languange but here I go.

I concure with the other experts who take exception to the loaded biased wording of this article . The author is delusional that Bush will be impeached and that he will ever be subject to trial in the Hague.But he can live in his fantasy world if he wishes to . Not going to spend much time dealing with his fantasies. (for the record he is not a liberal ,he comes from the Pat Buchannan wing of the Republicans ..the so called paleo-conservatives.They oppose most of what the Republicans do today including any of our foreign commitments war or otherwise ;NAFTA ;WTO :UN etc. They would have the US retreat to fortress America and burn ALL the bridges behind them. If asked to be honest most of their anamous is targetted at our support for Israel. Yes the whole group is anti-semetic )

Geneva conventions were convened and signed so war could be fought in a specific agreed upon manner. The whole point of a convention is to establish mutually beneficial conditions and assurances for both sides of a conflict . As an example ;It is lawful for a NATION to have troops get in a uniform and fire upon your opponent who is also in uniform firing back at you. It is not lawful to strap a homicide belt on a teenage boy and have him detonate it in a pizza parlor ...at least by Geneva definitions it isn't .That would be an example of an illegal combatant .

SCOTUS in it's decision last week trashed the principles of Geneva and has now for the first time confered sovereignty on al-Qaida and its affiliates;terorist organizations all. . But even more than that by definition Justice Stevens in la-la jihad land has also given Geneva status to an nonState non Signatory organization that never the less violates evey premise and paragraph of the conventions every time it takes up arms ,flys planes into civilian targets ,beheads a captive ,blows up a car bomb . They get all the benefits with none of the obligations. We're bound, they're not.

Even stranger ;Stevens and SCOTUS apparently has brought into the jihadists vision of the world . Al Qaeda are the warriors of the 'ummah without borders'headed by God Himself. .They are the the 'Dar al Salaam' (House of Peace),we are the 'Dar al Harb' (House of War).

What SCOTUS actually did was sever the last thread of justification for the Geneva Conventions . What Bush had done was find a way within the principles and language of Geneva to rationally deal with illegal combatants. The only salvation now for the convention language will be the loop hole that the Court left ... That Congress in it's wisdom will create a statute to effectively deal with the Gitm detainees esentially as he has done so already .

Meanwhile I expect that most nations dealing with the war against global jihad will pretty much scrap or ignore the treaty also and will not come under anywheres near the consternation that Bush has .

Erewhon rated this answer Excellent or Above Average Answer

Question/Answer
paraclete asked on 07/03/06 - Is okay to be homosexual but it's not okay to be female?

The confusion within the american branch of the anglican church becomes more obvious the longer it goes. This is what comes of departing from biblical principles, you don't know when to jump, and how high.

More Dioceses Reject New Female Leader

By Alan Cooperman
Washington Post Staff Writer
Monday, July 3, 2006; Page A01

To visit Episcopal parishes across her huge but sparsely populated Nevada diocese, Bishop Katharine Jefferts Schori pilots a small airplane. She often bumps down on tiny airstrips, but wherever her single-engine Cessna 172 lands, she is welcome.
That's about to change.
On June 18, the Episcopal Church's General Convention elected Jefferts Schori to a nine-year term as the denomination's presiding bishop, making her the first woman to head any branch of the Anglican Communion, the worldwide family of churches descended from the Church of England.
Although she will not take up her new role until November, six U.S. dioceses already have rejected her authority, and that number is rising. Many church leaders expect that by the time she takes office, about five more, for a total of 10 percent of the nation's 111 Episcopal dioceses, will have joined the rejectionist camp.
Moreover, conservative Anglicans overseas have made no secret of their hope that the archbishop of Canterbury, the spiritual leader of the Anglican Communion, will not invite Jefferts Schori to the next gathering of the heads of the 38 constituent churches in 2008.
Gender is only part of the reason that some conservatives in the church are unhappy about her election. Jefferts Schori, 52, is also firmly planted in the U.S. church's dominant liberal wing. Three years ago, she voted with the majority of Episcopal bishops to approve the New Hampshire Diocese's election of V. Gene Robinson, the first openly gay bishop in the Anglican Communion. She has allowed the blessing of same-sex couples in her Nevada diocese.
Most recently, she irritated some conservatives by speaking about "Mother Jesus" in a sermon.
Trained as a scientist as well as a theologian, she entered the priesthood relatively late in life, 12 years ago, after an initial career as an oceanographer specializing in octopuses and squids. Her husband is a retired professor of theoretical mathematics, and they have a daughter serving in the Air Force.
The Rev. Ian T. Douglas, a professor at the Episcopal Divinity School in Cambridge, Mass., said Jefferts Schori edged out six other candidates for presiding bishop because she is not only "whip smart" but also "very methodical, clear and measured" in her thinking.
To those who accuse her of heresy for referring to a female Jesus, she responds with a typically learned disquisition on medieval mystics and saints who used similar language, including Julian of Norwich and St. Teresa of Avila. "I was trying to say that the work of the cross was in some ways like giving birth to a new creation," she said. "That is straight-down-the-middle orthodox theology."
Yet she acknowledged that she likes to shake people up a bit.
"All language is metaphorical, and if we insist that particular words have only one meaning and the way we understand those words is the only possible interpretation, we have elevated that text to an idol," she said in a telephone interview. "I'm encouraging people to look beyond their favorite understandings." Jefferts Schori's "all language is metaphorical" approach is a giant red flag to traditionalists at home and abroad who believe that the Episcopal Church is heading toward schism because it has departed from the plain words of the Bible.
"The incoming presiding bishop has made her positions very clear -- that she is committed to the new agenda, committed to same-sex blessings, committed to having same-sex partners in the leadership in the church -- which means she is also not committed to the faith as delivered to the saints," said Bishop Robert W. Duncan of Pittsburgh.
Pittsburgh was among the first dioceses to reject Jefferts Schori's authority, along with South Carolina; central Florida; San Joaquin, Calif.; Fort Worth; and Springfield, Ill.
Her election may also hasten the departure of individual congregations. Two large congregations in Northern Virginia, the Falls Church and Truro Church, announced last week that they will go through ൰ days of discernment" this fall to consider their status.
"We prayed and hoped that the General Convention would really turn around and change direction, but obviously it didn't," said the Rev. Martyn Minns, who is retiring as Truro Church's rector and has been named a bishop in the conservative Anglican Church of Nigeria.
Virginia Bishop Peter J. Lee said he hopes to persuade both congregations to stay. He declined to say whether he would fight to keep control of their buildings and property if they left the denomination -- one of the main disincentives for congregations to break away.
Duncan, of Pittsburgh, said none of the dioceses that have spurned Jefferts Schori are quitting the Episcopal Church. Instead, they are asking Archbishop of Canterbury Rowan Williams for "alternative primatial oversight" -- the naming of a conservative primate from some other country to oversee them temporarily, he said.
Jefferts Schori called the request "an anxious response" that was "quite predictable." "Most of the bishops who protested have been protesting for years about the presence of ordained women in the church," she said.
Thirty years after the Episcopal Church began ordaining women, three U.S. dioceses -- Fort Worth, San Joaquin and Quincy, Ill. -- refuse to allow female priests. Elsewhere in the Anglican Communion, change has come even more slowly: 13 of the communion's 38 churches have no female priests, and besides the United States, the only countries with female bishops are Canada and New Zealand.
Raised as a Roman Catholic in Oregon, Jefferts Schori switched to the Episcopal Church with her parents when she was 9. But her deeper turn to faith came at 22, when a close friend died in a plane crash.
At the time, she was taking a graduate course in the philosophy of science and "reading Heisenberg and Bohr and Einstein and the great physicists who talk about mystery," she recalled. "Both things were, I think, a great nudge to send me off looking for spiritual answers."
In her study of marine invertebrates, she said, she saw "the great wonder and variety of creation." And when federal research funds began to dry up in the 1980s, three members of her congregation in Corvallis, Ore., suggested she become a priest.
Cathy Roskam, the suffagran bishop of New York and a friend of Jefferts Schori, said women hold 3 percent of the leadership positions in the Anglican Communion.
"Many women feel that were we represented even close to the percentage we have in the pews, we would not be having these divisions over human sexuality," she said of the hierarchy. "Of course, women differ over sexuality. We just wouldn't be dividing over it."
Jefferts Schori agreed. The message of her election, she said, is not that Episcopalians don't care what other Anglicans think, but "that we're more interested in feeding hungry people and relieving suffering than we are in arguing about what gender someone is or what sexual orientation someone has."

tomder55 answered on 07/03/06:

biblical principles ?

ok then there is no sin for being a women so I have no problem with the idea of women in the clergy or the heirarchy . I could not say the same for homosexuals .

paraclete rated this answer Excellent or Above Average Answer

Question/Answer
Erewhon asked on 07/02/06 - Bush's Anti-Media Attack seen openly for what it is ....


Bush's present attack on the NYT is patently politically motivated to please the Repuiblican right and give a little solace to his own Republican critics in the run-up to mid term elections.

If he wants to attack the NYT for running a story about Bush & Co. tracking the bank accounts of his fellow Americans, then he is free to do so - this is the Land of the Free, and even the president gets freedom of speech, right?

But why os why does he absolve the LA Times and the Wall Street Journal - both Republican conservative rags - from any culpability when they ran the same story? The LA Times was just a couple of hours away from reaching its independent decision to run the story - which it did run - and the NYT pipped it at the post, hitting the streets first.

Does this show Bush's desperation, his favouritism, his need to attack any liberal media for reporting the truth?

Where is his sense of moral outrage at the LAT and the WSJ?

Curious minds want to know.



tomder55 answered on 07/03/06:

I don't know the motivations for the LA slimes but from what I understand the Wall Street Journal only decided to go with the story when it became clear that the NY Slimes would publish first . Als othe editorial board of the Wall Street Journal was opposed to brealing the story and has been very critical of the Slimes for doing so. In fact they explained themselves very well in this editorial . I will highlight the key parts :

We should make clear that the News and Editorial sections of the Journal are separate, with different editors. The Journal story on Treasury's antiterror methods was a product of the News department, and these columns had no say in the decision to publish. We have reported the story ourselves, however, and the facts are that the Times's decision was notably different from the Journal's.
According to Tony Fratto, Treasury's Assistant Secretary for Public Affairs, he first contacted the Times some two months ago. He had heard Times reporters were asking questions about the highly classified program involving Swift, an international banking consortium that has cooperated with the U.S. to follow the money making its way to the likes of al Qaeda or Hezbollah. Mr. Fratto went on to ask the Times not to publish such a story on grounds that it would damage this useful terror-tracking method.

Sometime later, Secretary John Snow invited Times Executive Editor Bill Keller to his Treasury office to deliver the same message. Later still, Mr. Fratto says, Tom Kean and Lee Hamilton, the leaders of the 9/11 Commission, made the same request of Mr. Keller. Democratic Congressman John Murtha and Director of National Intelligence John Negroponte also urged the newspaper not to publish the story.

The Times decided to publish anyway,
letting Mr. Fratto know about its decision a week ago Wednesday. The Times agreed to delay publishing by a day to give Mr. Fratto a chance to bring the appropriate Treasury official home from overseas. Based on his own discussions with Times reporters and editors, Mr. Fratto says he believed "they had about 80% of the story, but they had about 30% of it wrong." So the Administration decided that, in the interest of telling a more complete and accurate story, they would declassify a series of talking points about the program. They discussed those with the Times the next day, June 22.

Around the same time, Treasury contacted Journal reporter Glenn Simpson to offer him the same declassified information. Mr. Simpson has been working the terror finance beat for some time, including asking questions about the operations of Swift, and it is a common practice in Washington for government officials to disclose a story that is going to become public anyway to more than one reporter. Our guess is that Treasury also felt Mr. Simpson would write a straighter story than the Times, which was pushing a violation-of-privacy angle; on our reading of the two June 23 stories, he did.


We recount all this because more than a few commentators have tried to link the Journal and Times at the hip. On the left, the motive is to help shield the Times from political criticism. On the right, the goal is to tar everyone in the "mainstream media." But anyone who understands how publishing decisions are made knows that different newspapers make up their minds differently.
Some argue that the Journal should have still declined to run the antiterror story. However, at no point did Treasury officials tell us not to publish the information. And while Journal editors knew the Times was about to publish the story, Treasury officials did not tell our editors they had urged the Times not to publish. What Journal editors did know is that they had senior government officials providing news they didn't mind seeing in print. If this was a "leak," it was entirely authorized.

Would the Journal have published the story had we discovered it as the Times did, and had the Administration asked us not to? Speaking for the editorial columns, our answer is probably not. Mr. Keller's argument that the terrorists surely knew about the Swift monitoring is his own leap of faith. The terror financiers might have known the U.S. could track money from the U.S., but they might not have known the U.S. could follow the money from, say, Saudi Arabia. The first thing an al Qaeda financier would have done when the story broke is check if his bank was part of Swift.

Just as dubious is the defense in a Times editorial this week that "The Swift story bears no resemblance to security breaches, like disclosure of troop locations, that would clearly compromise the immediate safety of specific individuals." In this asymmetric war against terrorists, intelligence and financial tracking are the equivalent of troop movements. They are America's main weapons.

The Times itself said as much in a typically hectoring September 24, 2001, editorial "Finances of Terror": "Much more is needed, including stricter regulations, the recruitment of specialized investigators and greater cooperation with foreign banking authorities." Isn't the latter precisely what the Swift operation is?

Whether the Journal News department would agree with us in this or other cases, we can't say. We do know, however, that Journal editors have withheld stories at the government's request in the past, notably during the Gulf War when they learned that a European company that had sold defense equipment to Iraq was secretly helping the Pentagon. Readers have to decide for themselves, based on our day-to-day work, whether they think Journal editors are making the correct publishing judgments.





Which brings us back to the New York Times. We suspect that the Times has tried to use the Journal as its political heatshield precisely because it knows our editors have more credibility on these matters.
As Alexander Bickel wrote, the relationship between government and the press in the free society is an inevitable and essential contest. The government needs a certain amount of secrecy to function, especially on national security, and the press in its watchdog role tries to discover what it can. The government can't expect total secrecy, Bickel writes, "but the game similarly calls on the press to consider the responsibilities that its position implies. Not everything is fit to print."
The obligation of the press is to take the government seriously when it makes a request not to publish. Is the motive mainly political? How important are the national security concerns? And how do those concerns balance against the public's right to know?

The problem with the Times is that millions of Americans no longer believe that its editors would make those calculations in anything close to good faith. We certainly don't. On issue after issue, it has become clear that the Times believes the U.S. is not really at war, and in any case the Bush Administration lacks the legitimacy to wage it.

So, for example, it promulgates a double standard on "leaks," deploring them in the case of Valerie Plame and demanding a special counsel when the leaker was presumably someone in the White House and the journalist a conservative columnist. But then it hails as heroic and public-spirited the leak to the Times itself that revealed the National Security Agency's al Qaeda wiretaps.

Mr. Keller's open letter explaining his decision to expose the Treasury program all but admits that he did so because he doesn't agree with, or believe, the Bush Administration. "Since September 11, 2001, our government has launched broad and secret anti-terror monitoring programs without seeking authorizing legislation and without fully briefing the Congress," he writes, and "some officials who have been involved in these programs have spoken to the Times about their discomfort over the legality of the government's actions and over the adequacy of oversight." Since the Treasury story broke, as it happens, no one but Congressman Ed Markey and a few cranks have even objected to the program, much less claimed illegality.

Perhaps Mr. Keller has been listening to his boss, Times Publisher Arthur Sulzberger Jr., who in a recent commencement address apologized to the graduates because his generation "had seen the horrors and futility of war and smelled the stench of corruption in government.

"Our children, we vowed, would never know that. So, well, sorry. It wasn't supposed to be this way," the publisher continued. "You weren't supposed to be graduating into an America fighting a misbegotten war in a foreign land. You weren't supposed to be graduating into a world where we are still fighting for fundamental human rights," and so on. Forgive us if we conclude that a newspaper led by someone who speaks this way to college seniors has as a major goal not winning the war on terror but obstructing it.






In all of this, Mr. Sulzberger and the Times are reminiscent of a publisher from an earlier era, Colonel Robert McCormick of the Chicago Tribune. In the 1930s and into World War II, the Tribune was implacable in its opposition to FDR and his conduct of the war. During the war itself, his newspaper also exposed secrets, including one story after the victory at Midway in 1942 that essentially disclosed that the U.S. had broken Japanese codes. The government considered, but decided against, prosecuting McCormick's paper under the Espionage Act of 1917.
That was a wise decision, and not only because it would have drawn more attention to the Tribune "scoop." Once a government starts indicting reporters for publishing stories, there will be no drawing any lines against such prosecutions, and we will be well down the road to an Official Secrets Act that will let government dictate coverage.

The current political clamor is nonetheless a warning to the press about the path the Times is walking. Already, its partisan demand for a special counsel in the Plame case has led to a reporter going to jail and to defeats in court over protecting sources. Now the politicians are talking about Espionage Act prosecutions. All of which is cause for the rest of us in the media to recognize, heeding Alexander Bickel, that sometimes all the news is not fit to print.

Erewhon rated this answer Excellent or Above Average Answer

Question/Answer
Erewhon asked on 07/02/06 - Is O'Reilly O'Really O'Right? ...................

Fox News host Bill O'Reilly again suggested that Iraq should be run as it was under Saddam Hussein, stating: "Saddam was able to control Iraq, as you know, and defeat insurgencies against him. The new Iraqi government can do the same, but it needs to get much tougher." O'Reilly also declared that the American Civil Liberties Union, the BBC, and Air America Radio "are helping the terrorists."

==

Is he right?

tomder55 answered on 07/03/06:

I think you are intentionally mischaracterizing his comments about the new Iraqi gvt. and comparing them to one of the most brutal regimes of the 20th century . He most certainly is not advocating a return to Baathist methods and you know that too.

The organizations you mention and some of the legacy media indeed help the terrorists .

Erewhon rated this answer Excellent or Above Average Answer

Question/Answer
Erewhon asked on 07/02/06 - USSC in turmoil ...................................................... .............................

In Chief Justice John G. Roberts Jr.'s first term, the Supreme Court was scene to both momentous decisions and battles postponed.

By LINDA GREENHOUSE
Published: July 2, 2006

WASHINGTON, July 1 As the dust settled on a consequential Supreme Court term, the first in 11 years with a change in membership and the first in two decades with a new chief justice, one question that lingered was whether it was now the Roberts court, in fact as well as in name.

The answer: not yet.

Chief Justice John G. Roberts Jr. was clearly in charge, presiding over the court with grace, wit and meticulous preparation. But he was not in control.

In the court's most significant nonunanimous cases, Chief Justice Roberts was in dissent almost as often as he was in the majority. His goal of inspiring the court to speak softly and unanimously seemed a distant aspiration as important cases failed to produce majority opinions and members of the court, including occasionally the chief justice himself, gave voice to their frustration and pique with colleagues who did not see things their way.

The term's closing weeks were particularly ragged. The court issued no decision in a major patent case that had drawn intense interest from the business community, announcing two months after the argument, over the dissents of three justices, that the case had been "improvidently granted" they should not have agreed to decide it in the first place.

So if it wasn't yet the Roberts court, what exactly was it?

Perhaps it was the Kennedy court, based on the frequency with which Justice Anthony M. Kennedy cast the deciding vote in important cases.

Or perhaps it was more accurately seen as the Stevens court, reflecting the ability of John Paul Stevens, the senior associate justice in tenure as well as in age, to deliver a majority in the case for which the term will go down in history, the decision on military commissions that rejected the Bush administration's view of open-ended presidential authority.

Chief Justice Roberts did not participate in that case because he had ruled on it a year earlier as an appeals court judge. Based on his vote to uphold the administration's position then, he almost certainly would have joined Justices Antonin Scalia, Clarence Thomas and Samuel A. Alito Jr., the newest member of the court, in dissent.

If none of these labels Roberts court, Kennedy court, Stevens court seem to fit precisely, it is probably because what the Supreme Court really was in its 2005-6 term was a court in transition.

For the justices, it was a time of testing, of battles joined and battles, for the moment, postponed.

The term's early period of unanimity, during which cases on such contentious subjects as abortion and federalism were dispatched quickly, with narrowly phrased opinions, reflected agreement not on the underlying legal principles but rather on the desirability of moving on without getting bogged down in a fruitless search for common ground. This was especially so in the term's early months, when Justice Sandra Day O'Connor was still sitting but was counting the days until a new justice could take her place.

Once Justice O'Connor retired in late January, after Justice Alito's confirmation, and as the court moved into the heart of the term, some of the court's early inhibitions seemed to fall away. Yet when its most conservative members reached out aggressively to test the boundaries of consensus in the term's major environmental case, Justice Kennedy unexpectedly pushed back and left them well short of their goal.

In that case, Chief Justice Roberts along with Justices Alito, Scalia and Thomas tried to cut back on federal regulators' expansive view of their authority under the Clean Water Act to define wetlands.

Justice Kennedy also deserted the conservatives in a redistricting case from Texas when he found a violation of the Voting Rights Act in the dismantling of a Congressional district that had previously had a Mexican-American majority. The action of the Republican-led Texas Legislature had deprived the Latinos of the ability to elect the candidate of their choice, Justice Kennedy said, leaving Chief Justice Roberts to complain in dissent, "It is a sordid business, this divvying us up by race."

Nonetheless, there was little doubt that in its transition, the court was becoming more conservative. A statistical analysis by Jason Harrow on the Scotusblog Web site showed that Justice Alito voted with the conservative justices 15 percent more often than Justice O'Connor had.

A separate analysis, by the Supreme Court Institute at Georgetown University Law Center, showed that Justice Alito and Chief Justice Roberts had the highest agreement rate of any two justices in the court's nonunanimous cases, 88 percent, slightly higher than the agreement rate between Justice O'Connor and Justice David H. Souter in the first half of the term, 87.5 percent.

Chief Justice Roberts agreed with Justice Scalia in 77.5 percent of the nonunanimous cases and with Justice Stevens, arguably the court's most liberal member, only 35 percent of the time. The least agreement between any pair of justices was between Justices Alito and Stevens, 23.1 percent.

The court decided 69 cases with signed opinions in the term that began on Oct. 3 and ended on June 29. Nearly half were decided without dissent, a greater number than usual, although not dramatically so. Sixteen cases were decided by five-justice majorities, either 5 to 4 or 5 to 3, a proportion very close to the 10-year average.

One measure of the court's shift to the right is in dissenting votes. In the previous term, the justice who dissented least often was Stephen G. Breyer, who dissented in 10 of the term's 74 decisions. But this term, he had the second-highest number of dissents, 16; Justice Stevens had the most, 19. Justice Thomas and Justices Ruth Bader Ginsburg and Souter were also frequent dissenters. Of those who served the full term, Chief Justice Roberts had the fewest dissents, seven. Justice Kennedy had the second fewest, with nine.
===

Comments?


tomder55 answered on 07/03/06:

a couple of comments

1. Stevens got his historic /momentus /historic decision that will forever etch his name in the history books as the author of the legislation that propelled the Imperial SCOTUS into having the powers to determine the execution of war wheter the Constitution gives them the authority or not ...an unbelievable power grab that the court will most likely get away with . He should be proud of himself . Now that he has his signature legislation isn't it time for the old goat to move on and retire ?

2. What is it about Reagan appointees that they think they have to swing ?

3. The Borking of Bork is proving to be the most disasterous play that the Senate has ever done .

The Court is in transition but there is more work to be done . Bush has done a excellent job in placing competent conservatives in the lower courts but he needs more time and one more SCOTUS candidate (or his Republican successor) to shape the court in his vision. Look for the next nominee to be treated very roughly by the Democrats in the Senate and most likely it will require a nuclear option to break their phony fillibuster.

Erewhon rated this answer Excellent or Above Average Answer

Question/Answer
jackreade asked on 07/01/06 - Freedom of the Press Anathma to Terrorist Ideologists

"....the virulent hatred espoused by terrorists, judging by their literature, is directed not just against our people and our buildings. It is also aimed at our values, at our freedoms and at our faith in the self-government of an informed electorate. If the freedom of the press makes some Americans uneasy, it is anathema to the ideologists of terror.

Thirty-five years ago yesterday, in the Supreme Court ruling that stopped the government from suppressing the secret Vietnam War history called the Pentagon Papers, Justice Hugo Black wrote: "The government's power to censor the press was abolished so that the press would remain ***forever free*** to censure the government. The press was protected so that it could bare the secrets of the government and inform the people." New York Times

~~~~~~~~~~~~~~~~~~~~~~~~~~~~~~~~~~~~~~~~~~~~~~~~~~~

tomder55 answered on 07/02/06:

nice piece of CYA from Keller and Baquet but they don't get off the hook that easily and yes I understand they are just itching for this show down with the administration . They can cloak their defenses with rationals about absolutes in the 1st amendment like this is some college debatin club ,but they have seriously undermined our ability to track the terrorists they speak of . When the jihadists defeat us I'm sure they will argue about their absolute rights just before the blade slices their necks.

I read this , and it didn't explain at all why they published . Just a lot of self-praise. Basically, we did it because we could. Does anyone doubt that if this was June 1944, the Slimes and others would print details of the D-Day invasion?

Terrorists and their financial backers assumed the US was tracking their transactions to try to get at their MONEY. The SWIFT program was not designed to get their MONEY, it was designed to get THEM. To trace the movements of the money in order to find the terrorists who ultimately receive the money. We were deliberately not going after the money, which made the terrorists think they were safe. We were not even getting the information from SWIFT in realtime, but after the fact, so it wasn't useful for us to get at the money, but it was still useful to track down the terrorists in the field who ultimately received the money. Hambali was caught using this program precisely because we never moved to seize the money, we USED the money to find HIM and arrest him. Now they know we are doing this and they will simply stop using any longterm bank accounts to move their money so by the time we get the data from SWIFT it won't be useful in finding them. The sh*theads at the Slimes think this doesn't matter, but it does. It's a very serious setback in our ability to stealthily track down these guys without them seeing us coming.

The answer to the question they pose :When Do We Publish a Secret? is straighforward ...When it's been declassified and released by the proper authorities.

Ellsburg would've been jailed for revealing the Pentagon
Papers to the press if he had not gone underground first .

jackreade rated this answer Excellent or Above Average Answer

Question/Answer
Itsdb asked on 07/01/06 - A pattern of treason?

Incident is only latest for terrorist-tipping Times

By Michelle Malkin

The New York Times (proudly publishing all the secrets unfit to spill since 9/11) and their reckless anonymous sources (come out, come out, you cowards) tipped off terrorists to America's efforts to track their financial activities.

Guess what? It isn't the first time blabbermouth journalists have jeopardized terror financing investigations since September 11, according to the government.

I remind you of the case of the Treason Times, the Holy Land Foundation, and the Global Relief Foundation. As the New York Post reported last September, the Justice Department charged that "a veteran New York Times foreign correspondent warned an alleged terror-funding Islamic charity that the FBI was about to raid its office potentially endangering the lives of federal agents." Times reporter Philip Shenon was accused of blowing the cover on a December 14, 2001 raid of the Global Relief Foundation.

"It has been conclusively established that Global Relief Foundation learned of the search from reporter Philip Shenon of The New York Times," U.S. attorney Patrick Fitzgerald wrote in an Aug. 7, 2002 , letter to the Times' legal department.

Shenon's phone tip to the Muslim charity (which occurred one day before the FBI searched the foundation's offices), Fitzgerald said, "seriously compromised the integrity of the investigation and potentially endangered the safety of federal law-enforcement personnel." The Global Relief Foundation wasn't some beneficent neighborhood charity sending shoes and Muslim Barbie dolls to poor kids overseas. It was designated a terror financing organization in October 2002 by the Treasury Department, which reported that GRF "has connections to, has provided support for, and has provided assistance to Usama Bin Ladin, the al Qaida Network, and other known terrorist groups."

The Muslim charity had "received funding from individuals associated with al Qaida. GRF officials have had extensive contacts with a close associate of Usama Bin Ladin, who has been convicted in a U.S. court for his role in the 1998 bombings of the U.S. embassies in Kenya and Tanzania." Moreover, the Treasury Department said, "GRF members have dealt with officials of the Taliban, while the Taliban was subject to international sanctions."

Shenon's then-colleague, Judith Miller, had placed a similar call to another Muslim terrorist front financier, the Holy Land Foundation, a few weeks before Shenon's call to the GRF. She was supposedly asking for "comment" on an impending freeze of their assets. According to Fitzgerald in court papers, Miller allegedly also warned them that "government action was imminent." The FBI raided the Holy Land Foundation's offices the day after Miller's article was published in the Times.

The Times' reporters surprise, surprise--refuse to cooperate with investigators trying to identify the leakers. The government is appealing a ruling protecting the loose-lipped reporters' phone records. Which side are they on? Actions speak louder than words.

Oh, and while they continue to sabotage terror financing investigations, the blabbermouths of the Times should be reminded as the conservative bloggers Bill Keller despises so much are doing of their own call in the immediate aftermath of 9/11 for vigorous counterterrorism measures to stop the bankrolling of terror:

"The Bush administration is preparing new laws to help track terrorists through their money-laundering activity and is readying an executive order freezing the assets of known terrorists. Much more is needed, including stricter regulations, the recruitment of specialized investigators and greater cooperation with foreign banking authorities. There must also be closer coordination among America's law enforcement, national security and financial regulatory agencies."

"Much more is needed?" Right. And when the Bush administration came through, the Times stabbed them, and us, in the backs. The lesson is clear. When terror strikes, don't believe a word the know-it-all Times prints. They are opportunistic hindsight hypocrites who endanger us all.

~~~~~~~~~~~~~~~~~~~~~~~~~~~~~~~~~~~~~~~~~~~~~~~~~~~~~~

Comments?

tomder55 answered on 07/01/06:

At the time of Miller's incarceration I thought that it was this case that Fitzgerald was mining information about .On December 3, 2001, Miller telephoned officials with the Holy Land Foundation and asks them to comment about a government raid on the charity planned for the next day. Two days later Miller writes in the Slimes that President Bush is about to announce that the US is freezing the assets of Holy Land and two other financial groups for supporting Hamas.On December 13 Philip Shenon telephones officials at the Global Relief Foundation and asks them to comment about an imminent government crackdown on that charity. Global Relief employees start destroying documents.

Fitzgerald just happens to be investigating the charities.He had been wiretapping Global Relief and another charity in hopes of learning evidence of criminal activity, but after the Slimes leaks he changes plans and carries out a hastily arranged raid on the charity on the 14th. It has not yet been determined who in the gvt. leaked the info. to the Slimes but it was clear that Miller and Shenon undermined an investigation into these Muslim charities .

The 9-11 commission in fact concluded that EVERY raid on Muslim charities after 9-11 was compromised by "press leaks" to the charities .

Itsdb rated this answer Excellent or Above Average Answer

Question/Answer
Erewhon asked on 06/30/06 - Bush's plug is pulled but ehe USSC!



In a stinging rebuke of President George Bush's high handed and illegal tactics in the war on terrorism, the mighty US Supreme Court yesterday struck down as unlawful Bush's military tribunal system set up to try Guantanamo prisoners.

By a 5-3 vote, the country's highest court declared that the tribunals, which Bush created in the aftermath of the 11 September attacks, violated the Geneva Conventions and US military rules.

"We conclude that the military commission convened to try Salim Ahmed Hamdan [Osama bin Laden's former chauffeur] lacks power to proceed because its structure and procedures violate the international agreement that covers treatment of prisoners of war, as well as the Uniform Code of Military Justice," Justice John Paul Stevens wrote in the judgment.

Stevens said the military commissions were not authorised by any act of the US Congress but added that Mr Bush was free to seek the necessary authority (that he already thinks, erroneously, that he holds in his prwesidential paws!)

The decision was a stinging blow for the administration in a case brought by Hamdan, who was Osama bin Laden's driver in Afghanistan. Hamdan, one of hundreds of foreign terrorist suspects at the US naval base in Guantanamo Bay, Cuba, was captured in November 2001.

It prompted immediate calls for the US authorities to swiftly release prisoners or allow fair trials for all the detainees, who include eight British residents.

Kate Allen, the UK director of Amnesty International said: "This ruling should now lead to the US administration ending the scandal of holding Guantanamo prisoners in defiance of international human rights law. The Guantanamo Bay prison camp should be closed and all detainees allowed fair trials or released to safe countries."

Jose-Luis Diaz, a spokesman for the United Nations High Commissioner for Human Rights, Louise Arbour, said the decision also appeared to vindicate "the need for vigilance" in protecting rights, including those of terrorist suspects.

The US currently holds about 450 detainees at the facility. The Pentagon says 136 of these detainees have been approved for release or transfer, but cannot be released because they are either too dangerous, unwanted by their home country or are at risk of being tortured if returned to their countries. LOL!!!

Ten detainees have had charges filed - they were due to be tried by the military tribunals now ruled unlawful.

The ruling, on the last day of the court's 2005-06 term, followed the deaths of three Guantanamo prisoners this month and increased calls for Mr Bush to close the prison camp. US treatment of inmates at Guantanamo and in Iraq and Afghanistan has drawn international criticism.

Within the past few months, Peter Hain, the Northern Ireland Minister, and Lord Goldsmith, the Attorney General, have called for its closure. Tony Blair, the Prime Minister, has only said that the camp is an "anomaly" that should be dealt with "sooner or later".

One of Hamdan's military lawyers, Lieutenant Commander Charles Swift, praised the court action. "All we wanted was a fair trial," he said outside the Supreme Court. "Yes, it is a rebuke for the process... It means we can't be scared out of who we are."

Justice Stevens said in his 73-page opinion: "The rules specified for Hamdan's trial are illegal."

He said the system has to incorporate even "the barest of those trial protections that have been recognised by customary international law".

The tribunals, ruled Stevens, failed to provide even the most fundamental protection under US military rules - the right for a defendant to be present at a hearing.



===

Bush is shown to be the tyrant he is. I told you he would be found out and I am vindicated by no less a body than the US Supreme Court!

tomder55 answered on 06/30/06:

I used to think the court could never out do itself with the silly Roe V Wade ruling. But now in less than a year it has out done itself with both the Kelo decision and now the Hamdan ruling .The Court had no authority to hear the case, because a statute clearly deprived it of jurisdiction (Detainee Treatment Act of 2005 ).

no court, justice or judge shall have jurisdiction to hear or consider applications on behalf of Guantanamo detainees.

But as I've noted before ,the unelected branch has a history of usurping power it is not constitutionally endowed with. I read much of Justice Stevens ruling and it amounts to : We dont need no stinking jurisdiction. But as to the merits of his decision ...well I've responded on this on jacks posting here and on Dark Crow's posting on the Philosophy Board . There is no merit . The President is the Commander in Chief during war . The Congress has the role of war declaration and funding ONLY ,and the US courts have no constitutional role in war . As I mentioned in one of them (Crow's I think) even when the President makes a terrible decision like the internment of Japanese-Americans ;the courts have conceeded that the President had the authority . They had nothing to say when Roosevelt had 8 German prisoners executed on his authority .

The few times that the courts have interfered in the President's constitional authority to wage war did not turn out well for them. Chief Justice Roger Taney tried to stop Lincoln from suspending habeas corpus and Lincoln basically ignored him . Who do you remember Lincoln or Taney ?

I think what Bush should do is defy the courts. When the Specter bill gets bogged down in endless Congressional wrangling and debate (their typical gyrations when they really do not want to act )the President should start the tribunals and let's see what force the court can muster to stop him . When he is accused of lawlessness he should explain that the Supreme Courts decision in Hamdan is not law because the Court lacked jurisdiction to make it. (don't worry he won't do it even though it is perfectly within his authority )

Erewhon rated this answer Excellent or Above Average Answer
Itsdb rated this answer Excellent or Above Average Answer
purplewings rated this answer Excellent or Above Average Answer

Question/Answer
ROLCAM asked on 06/30/06 - Good News !!

Hamas takes 'historic' step. 6/29/2006.

HAMAS yesterday took its biggest step yet towards recognising Israel by agreeing to a programme that calls for establishing a Palestinian state in the West Bank and Gaza Strip alongside the Jewish state, Palestinian officials said.

What are your latest views ??

tomder55 answered on 06/30/06:

did they declare that before or after they tunneled under the wall ;kidnapped and detained an Israeli soldier and murdered a student ? Guess they had a change of heart after they were staring down the barrels of 3 Israeli Armored Divisions .

I have not heard one Hamas leader utter the proposal .Specifically I have not heard that from Khaled Mash'al the Hamas leader who calls the shots out of Damascus Syria at the pleasure of the Iranians. Since Israel is in the process of rounding up Hamas MPs then it should be no issue for them to speak of their desire for normalcy face to face .

paraclete rated this answer Excellent or Above Average Answer
ROLCAM rated this answer Excellent or Above Average Answer

Question/Answer
ROLCAM asked on 06/30/06 - More good news !!

Chimps and apes to get same rights as humans.

SPAIN is not the first country that springs to mind as a land of animal lovers - it is better known for bullfighting and the large number of stray cats and dogs on its streets.

Is this a sound political decision ?

tomder55 answered on 06/30/06:

good point about the bull fighting . The bigger battle of bull is obviously being waged in the Spanish Parliment .

Is is a sound decision ? I don't think so but is is emblematic of what you can expect from a nanny-state . I am almost sure that the great apes are better served and more humanely treated in the worse run Spanish zoo than any facility the the gvt . could conceive .

Itsdb rated this answer Excellent or Above Average Answer
ROLCAM rated this answer Excellent or Above Average Answer

Question/Answer
paraclete asked on 06/30/06 - so now the US thinks it's laws apply to Australia

AWB to face $1bn lawsuit
From:
By Caroline Overington

June 30, 2006


US and Canadian wheat farmers are preparing a $1 billion damages claim against Australian wheat exporter AWB, using racketeering laws designed to hobble the mafia. The farmers will claim they suffered lost income because AWB was paying illegal kickbacks to Saddam Hussein's regime in Iraq.

If it succeeds, the action would cripple AWB, a company mostly owned by Australian wheat farmers.

The action will be led by Atlanta lawyer Roderick E. Edmond and his Australian colleague, former Adelaide crown prosecutor and human rights lawyer, Michael Hourigan.

The lawyers plan to use the Racketeer Influenced and Corrupt Organisations Act, passed by the US Congress in 1970 to eliminate organised crime. The aim of RICO was to destroy the mafia and, more recently, terrorist organisations, but American lawyers have used the act for civil actions since the 1980s.

The law covers bribery, kickbacks and extortion. It applies only when there is a pattern of criminal activity, rather than a "one-off crime".

AWB is accused of funnelling $290 million to Saddam's regime over four years, from mid-1999 until the US-led invasion ended the UN's oil-for-food program in March 2003.

Under the RICO law, any person who succeeds in establishing a claim can automatically receive three times their actual damages, plus costs.

The action will be taken on behalf of a handful of farmers in the first instance but thousands of US and Canadian wheat farmers could join the classaction.

The lawyers would have to prove that AWB's action in paying kickbacks to Saddam's regime was criminal and that the financial damage they suffered was a direct result of AWB kickbacks.

AWB has admitted making the payments to a Jordanian trucking company, Alia. But executives claim the company thought the payments were for legitimate transport costs and did not know Alia was a front for Saddam.

Mr Edmond is a former US army captain turned private attorney who used the RICO Act in 2004 to launch a class action on behalf of Iraqis tortured by US forces at Abu Ghraib prison in Baghdad.

Mr Edmond has also acted for the widow of one of the US postal workers who died after anthrax was put in the mail in late 2001.

Mr Hourigan is an Adelaide-born human rights lawyer and UN crimes investigator. He travelled to Baghdad in 2004, found many of the victims of torture and their families, and videotaped the evidence.

In 1996, Mr Hourigan was asked to lead a UN team investigating the 1994 genocide in Rwanda. The investigation inspired the 1998 BBC documentary When Good Men Do Nothing.

The legal action comes just weeks after prominent US congressman Norm Coleman was savaged by colleagues for refusing to take action against AWB.

Mr Coleman is head of the US Senate's permanent investigations sub-committee. He planned a probe into AWB's wheat sales to Iraq in 2004, but it was scrapped after the Howard Government sent its then US ambassador, Michael Thawley, to lobby on behalf of AWB. Mr Thawley told Mr Coleman the claims were baseless and malicious.

Mr Coleman, a Republican and strong supporter of US President George W. Bush, has been accused of dropping the probe to protect John Howard, an ally in the Iraq war.

He denies the claim, saying he did not pursue an investigation into AWB's wheat sales because the UN was already conducting its own probe. A UN inquiry into the scandal, known as the Volcker report, found that AWB was the biggest single supplier of illicit funds to Saddam's regime under the oil-for-food program.

Last year, the Howard Government established the Cole inquiry to investigate the scandal and it, too, has uncovered evidence that some executives at AWB were aware that Saddam was corrupting the program.

Australia's trade with Iraq, worth $800 million a year, has been severely damaged by the scandal, which has also prompted calls for AWB to be stripped of its monopoly over Australian wheat exports.

Iraq is refusing to deal with AWB. It recently agreed to buy wheat from a new consortium, Wheat Australia, for a very cheap price.

AWB is now refusing to supply Wheat Australia with any more wheat for future deals.

tomder55 answered on 06/30/06:

if the trial is to be held in Aussie then only Aussie laws should apply . But I will say that AWB was a Saddam enabler . If the sanctions were strictly adhered to they would've been successful and the need for war could've been averted . If I were you I would not be championing those scoundrels.

Itsdb rated this answer Excellent or Above Average Answer
paraclete rated this answer Excellent or Above Average Answer

Question/Answer
jackreade asked on 06/29/06 - Putin does a 180

"Today's New York Times carries the following headline: "Putin orders death for killers of Russians in Iraq."

The story tells of Putin's decision to have Russian military intelligence target the terrorists who killed four of its embassy employees in Iraq. This is the same Putin and the same Russia that has repeatedly criticized Israel for its targeted killing of terrorists, even "ticking bomb" terrorists who are planning imminent attacks.

The Russian Foreign Minister condemns Israel's killing of Abdel Aziz al-Rantissi, who was the head of the terrorist organization Hamas and who had pulled the trigger on numerous terrorist attacks. In fact, Putin invited Hamas official to Moscow as his state guests.

According to the BBC article Rantissi Killing: World Reaction, "Russia has repeatedly stressed the unacceptability of extrajudicial settling of scores and 'targeted killings'."

Except, it seems, when its own citizens are murdered by terrorists -- then it is fine to do what it condemns Israel for doing.

The rest of the world is no different: condemning Israel for what they themselves do with impunity.

The time has common to end this double standard.

(I know this posting will stimulate the usual anti-Israel, anti-Semitic, and anti-Dershowitz fulminations, along with some thoughtful responses. The knee-jerk reaction to anything I write about Israel simply confirms my point about the double standard.)

~~~~~~~~~~~~~~~~~~~~~~~~~~~~~~~~~~~~~~~~~~~~~~~


Nice to see Putin come to his senses. :)

tomder55 answered on 06/30/06:

if he want to address the terror masters then he should be eyeing Tehran . The genious of the Israeli method of targetted assasinations is that it is directed at the leadership ;not the henchmen.

jackreade rated this answer Excellent or Above Average Answer

Question/Answer
jackreade asked on 06/30/06 - So much for the Republican Supreme Court Strategy

uSpreme Court Rejects Guantanamo Trials
President Says He'll Seek Congressional Approval for Tribunals
By TERENCE HUNT, AP

WASHINGTON (June 29) - "After a Supreme Court decision overruling war crimes trials for Guantanamo Bay detainees, President Bush suggested Thursday he would seek Congress' approval to proceed with trying terrorism suspects before military tribunals.

"To the extent that there is latitude to work with the Congress to determine whether or not the military tribunals will be an avenue in which to give people their day in court, we will do so," he said. "The American people need to know that the ruling, as I understand it, won't cause killers to be put out on the street."

Bush said little more, saying he had received only a "drive-by briefing" on the ruling just out earlier Thursday morning.

The Supreme Court decided that Bush's proposed trials for certain detainees at the controversial U.S. prison in Cuba were illegal under U.S. law and international Geneva conventions. A separate opinion, written by Justice Stephen Breyer, appeared to invite Bush to go to Congress to seek the authority to change that, and Bush's short answer indicated that is his intention.

The president declined to say whether the decision would prompt him to more quickly follow through on his promise to close the prison, as many world leaders and human rights groups have urged.


White House spokesman Tony Snow said later that Bush still wants to close the Guantanamo Bay facility once the administration can determine what to do with the prisoners, and he said the Supreme Court decision does not affect that."

~~~~~~~~~~~~~~~~~~~~~~~~~~~~~~~~~~~~~~~~~~~~~~~~


Comments...

tomder55 answered on 06/30/06:

Roberts was recused because he was in the appealate court that made the original decision so it would've been a 5-4 ruling . Evidently Kennedy considers himself the new Sandee Day . I would say that Bush has 2 more years to add another justice to the court .

In direct response to this outrageous decision which strips the President of his constitutionally mandated war-making powers Senator Arlen Specter introduced the 'Unprivileged Combatant Act of 2006.' Note the following provisions of the bill :

Section 301: Findings: This title is in direct response to the United States Supreme Courts ruling in Rasul v. Bush.

Section 302: Definition Section: Defines primary terms in the bill such as field tribunal, classification tribunal, military commission, and unprivileged combatant.

Section 303: Authorizing Military Commissions: Authorizes the president to establish military commissions for the trial of individuals for offenses provided in this title


The important aspect of this court ruling that will be explored in the days ahead is the apparent recognition of quasi-and non-state entities that are now given equal consideration to nationalities .If the Geneva Convention now applies to a non-state that is a non-signatory does it not then apply to ALL non-states that are non-signatories ? Amazing that al -Qaeda a terrorist organization has now been granted the status of 'state ' without all the correlating responsibilities of a state .They do not have to adhere to Geneva provisions ,only the state that are forced to deal with them .

The answer on how to proceed in the war has also been revealed thanks to the Supremes ....privatize it . Instead of using US assets we should organize and finance an army of uninhibited by rules of engagement ..un-uniformed mercenaries ,who do not recognize mundane nuiances like borders to attack al -Qaeda inside their sanctuaries in Pakistan.

But in a more civilized time privateers were considered pirates and were dealt with appropriately .In the eyes of the Supremes al-qaeda is a nation-state ,not pirates.

This ruling is the death-knell of the Geneva accords . The Geneva Convention relating to the treatment of prisoners of war provides in Article 4, that "

"A. Prisoners of war ... are persons belonging to one of the following categories, who have fallen into the power of the enemy:


ŕ. Members of the armed forces of a Party to the conflict as well as members of militias or volunteer corps forming part of such armed forces.


Ŗ. Members of other militias and members of other volunteer corps, including those of organized resistance movements, belonging to a Party to the conflict and operating in or outside their own territory, even if this terrirory is occupied, provided that such militias or volunteer corps, including such organized resistance movements, fulfill the following conditions:


"(a) That of being commanded by a person responsible for his subordinates;

(b) That of having a fixed distinctive sign recognizable at a distance;

(c) That of carrying arms openly;

(d) That of conducting their operations in accordance with the laws and customs of war."


As Mark Levin explains :The purpose of this language is to make clear that NOT every combatant is covered by this treaty i.e., that in order to receive the Convention's protections, combatants must accept and comply with basic rules of war.

The Bush administration correctly interpreted the status of the detainees .By all the logic of Geneva they were illegal combatants .The Geneva Conventions is and was intended as a Legal Contract between two warring parties.Why should they care about the Geneva Convention when its a one sided contract that even if disregard we must still honor ? Each side should only honor what the other side honors.

I foresee a day when we become very selective on who we take as a prisoner in battle . Dead men tell no tales .

jackreade rated this answer Excellent or Above Average Answer

Question/Answer
jackreade asked on 06/29/06 - Gerrymandering

Taking a move from the Republican play sheet, states that are likely to go to Democratic governers(NY, NJ, ILL, NJ) in November are likely to call for redistricting, draw new boundaries for congressional districts in order to increase the number of Democratic representatives by perhaps as many as 40 representatives.

Texas Republicans are already in the process of planning to redistrict for the benefit of the Republican party.

Apparently, there is nothing in the Constitution about when redistricting can occur. Redistricting can occur at any time, any number of times.

Comments?

tomder55 answered on 06/30/06:

Doesn't matter who is govenor ;it's the majority of the State Legislature that is critical .
I already lost an outstanding congressman in my district to democrat gerrymandering after the 2000 census ;didn't matter in the least that Pataki was guv. . My comment . That's the way the game is played . Always has been . Just because most states redistrict after the census doesn't mean that formula is carved in stone. The Supreme Court's only caveat was that you could not gerrymander for racial reasons.

jackreade rated this answer Excellent or Above Average Answer

Question/Answer
HANK1 asked on 06/29/06 - HERE IT IS:



www.holidays.net/independence/story.htm

HANK

tomder55 answered on 06/29/06:

thanks for the link Hank . What can I say ;the founders were a special generation .

And for the support of this Declaration, with a firm reliance on the protection of divine Providence, we mutually pledge to each other our Lives, our Fortunes and our sacred Honor.

They risked it all.

HANK1 rated this answer Excellent or Above Average Answer

Question/Answer
kindj asked on 06/29/06 - This seems outrageous to me

OK, I'll be the first to admit that I'm not as up on the recent events in Israel as I should be. However, in reading a story today about Israel's mission to recover their captured soldier, I came across this quote:

>>We have no government, we have nothing. They have all been taken, said Saeb Erekat, an ally of the moderate Palestinian president, Mahmoud Abbas. This is absolutely unacceptable and we demand their release immediately.<<

The MODERATE Abbas?

Excuse me?

So he's only moderately a terrorist?

Am I wrong here?

On another related note: The US (as a whole) could learn a few things about duty, honor, and loyalty from the Israelis. This mission to recover their captured brother simply glows with honor, and sends all the testosterone I've lost from years at a desk in the "real world" rushing all at once back into my system. Seems kinda silly, but after reading about it last night, I laced up my boots and went on a quick 5 mile run. Well, kinda quick.

OK, not as quick as it once would have been.

And I was sucking wind by mile 3.

But damn, it was inspiring to read!

Those are the guys I want on MY side should things go to hell.

But really, Abbas a moderate? WTF?

DK

tomder55 answered on 06/29/06:

isn't it amazing ? I guess he never actually picked up a gun ? Abbas was one of Arafat's top lackies .He funded the raid on the Israeli Olympians in Munich.He is head of head of Fatah but is also deeply rooted in the Al Aqsa Martyr's Brigades .

From his own lips :

"Our first priority is to lift the economic and political siege, then to end the occupation of our land once and for all, and to establish our independent Palestinian state."

He is also a holocost denier . He wrote a book in 1983 entitled "The Other Side: The Secret Relationship between Nazism and the Zionist Movement. "


This whole label of "moderate " comes with the attempt (in vain I think ) to find a 'partner in peace'. That is why the Israelis tolerated Arafat for so long ,and why they now find ways to smuggle arms to Abbas (so he can combat the Hamas factions).


you are so right about the Israeli troops . I was openly cheering when I heard the strength of their incursion and the fact that last night their air force buzzed Assad's somerhome setting off a sonic boom and probably soiling his pants .

kindj rated this answer Excellent or Above Average Answer

Question/Answer
arcura asked on 06/29/06 - What do you think about this? Are we waking up?

Classification: UNCLASSIFIED

When WW-III Started****1979

This is not very long, but very informative You have to read the catalogue of events in this brief piece. Then, ask yourself how anyone can take the position that all we have to do is bring our troops home from Iraq, sit back, reset the snooze alarm, go back to sleep, and no one will ever bother us again. In case you missed it, World War III began in November 1979... that alarm has been ringing for years

US Navy Captain Ouimette is the Executive Officer at Naval Air Station, Pensacola, Florida. Here is a copy of the speech he gave last month. It is an accurate account of why we are in so much trouble today and why this action is so necessary.

AMERICA NEEDS TO WAKE UP!

That's what we think we heard on the 11th of September 2001 (When more than 3,000 Americans were killed -AD) and maybe it was, but I think it should have been "Get Out of Bed!" In fact, I think the alarm clock has been buzzing since 1979 and we have continued to hit the snooze button and roll over for a few more minutes of peaceful sleep since then.

It was a cool fall day in November 1979 in a country going through a religious and political upheaval when a group of Iranian students attacked and seized the American Embassy in Tehran. This seizure was an outright attack on American soil; it was an attack that held the world's most powerful country hostage and paralyzed a Presidency. The attack on this sovereign U. S. embassy set the stage for events to follow for the next 25 years.

America was still reeling from the aftermath of the Vietnam experience and had a serious threat from the Soviet Union when then, President Carter, had to do something. He chose to conduct a clandestine raid in the desert. The ill-fated mission ended in ruin, but stood as a symbol of America's inability to deal with terrorism.

America's military had been decimated and down sized/right sized since the end of the Vietnam War. A poorly trained, poorly equipped and poorly organized military was called on to execute a complex mission that was doomed from the start.

Shortly after the Tehran experience, Americans began to be kidnapped and killed throughout the Middle East. America could do little to protect her citizens living and working abroad. The attacks against US soil continued.

In April of 1983 a large vehicle packed with high explosives was driven into the US Embassy compound in Beirut When it explodes, it kills 63 people. The alarm went off again and America hit the Snooze Button once more.

Then just six short months later in 1983 a ! large truck heavily laden down with over 2500 pounds of TNT smashed through the main gate of the US Marine Corps headquarters in Beirut and 241 US servicemen are killed. America mourns her dead and hit the Snooze Button once more.

Two months later in December 1983, another truck loaded with explosives is driven into the US Embassy in Kuwait, and America continues her slumber.

The following year, in September 1984, another van was driven into the gate of the US Embassy in Beirut and America slept.

Soon the terrorism spreads to Europe. In April 1985 a bomb explodes in a restaurant frequented by US soldiers in Madrid.

Then in August 1985 a Volkswagen loaded with explosives is driven into the main gate of the US Air Force Base at Rhein-Main, 22 are killed and the snooze alarm is buzzing louder and louder as US interests are continually attacked.

Fifty-nine days later in 1985 a cruise ship, the Achille Lauro is hijacked and we watched as an American in a wheelchair is singled out of the passenger list and executed.

The terrorists then shift their tactics to bombing civilian airliners when they bomb TWA Flight 840 in April of 1986 that killed 4 and the most tragic bombing, Pan Am Flight 103 over Lockerbie, Scotland in1988, killing 259.

Clinton treated these terrorist acts as crimes; in fact we are still trying to bring these people to trial. These are acts of war.

The wake up alarm is getting louder and louder.

The terrorists decide to bring the fight to America. In January 1993, two CIA agents are shot and killed as they enter CIA headquarters in Langley, Virginia.

The following month, February 1993, a group of terrorists are arrested after a rented van packed with explosives is driven into the underground parking garage of the World Trade Center in New York City. Six people are killed and over 1000 are injured. Still this is a crime and not an act of war? The Snooze alarm is! depressed again.

Then in November 1995 a car bomb explodes at a US military complex in Riyadh, Saudi Arabia killing seven service men and women.

A few months later in June of 1996, another truck bomb explodes only 35 yards from the US military compound in Dhahran, Saudi Arabia. It destroys the Khobar Towers, a US Air Force barracks, killing 19 and injuring over 500. The terrorists are getting braver and smarter as they see that America does not respond decisively.

They move to coordinate their attacks in a simultaneous attack on two US embassies in Kenya and Tanzania.. These attacks were planned with precision. They kill 224. America responds with cruise missile attacks and goes back to sleep.

The USS Cole was docked in the port of Aden, Yemen for refueling on 12 October 2000, when a small craft pulled along side the ship and exploded killing 17 US Navy Sailors. Attacking a US War Ship is an act of war, but we sent the FBI to investigate the crime and went back to sleep.

And of course you know the events of 11 September 2001. Most Americans think this was the first attack against US soil or in America. How wrong they are. America has been under a constant attack since 1979 and we chose to hit the snooze alarm and roll over and! go back to sleep.

In the news lately we have seen lots of finger pointing from every high officials in government over what they knew and what they didn't know. But if you've read the papers and paid a little attention I think you can see exactly what they knew. You don't have to be in the FBI or CIA or on the National Security Council to see the pattern that has been developing since 1979.

I think we have been in a war for the past 25 years and it will continue until we as a people decide enough is enough. America needs to "Get out of Bed" and act decisively now. America has been changed forever.. We have to be ready to pay the price and make the sacrifice to ensure our way of life continues. We cannot afford to keep hitting the snooze button again and again and roll over an! d go back to sleep.

After the attack on Pearl Harbor, Admiral Yamamoto said "... it seems all we have done is awakened a sleeping giant." This is the message we need to disseminate to terrorists around the world.

This is not a political thing to be hashed over in an election year this is an AMERICAN thing. This is about our Freedom and the Freedom of our children in years to come.

If you believe in this please forward it to as many people as you can especially to the young people and all those who dozed off in history class and who seem so quick to protest such a necessary military action. If you don't believe it, just delete it and go back to sleep.

SFC Tafoya, Sean M.
Support Ops Class V NCOIC
HHD, LTF 548
LSA Anaconda, Iraq

tomder55 answered on 06/29/06:

I think he has a valid point but I would just comment that it is better to reference in the context of the continued jihad against the world initiated by Mohammed . This is just the latest chapter in this Odyssey .

arcura rated this answer Excellent or Above Average Answer
ETWolverine rated this answer Excellent or Above Average Answer
labman rated this answer Excellent or Above Average Answer

Question/Answer
jackreade asked on 06/28/06 - End of Iraq War and Occupation at Hand ?

"BAGHDAD, Iraq (AP) - Eleven Sunni insurgent groups have offered an immediate halt to all attacks - including those on American troops - if the United States agrees to withdraw foreign forces from Iraq in two years, insurgent and government officials told The Associated Press on Wednesday.

Withdrawal is the centerpiece of a set of demands from the groups, which operate north of Baghdad in the heavily Sunni Arab provinces of Salahuddin and Diyala. Although much of the fighting has been to the west, those provinces are increasingly violent and attacks there have crippled oil and commerce routes.

The groups who've made contact have largely shunned attacks on Iraqi civilians, focusing instead on the U.S.-led coalition forces. Their offer coincides with Prime Minister Nouri al-Maliki's decision to reach out to the Sunni insurgency with a reconciliation plan that includes an amnesty for fighters.

The Islamic Army in Iraq, Muhammad Army and the Mujahedeen Shura Council - the umbrella group that covers eight militant groups including al-Qaida in Iraq - were not party to any offers to the government".

^^^^^^^^^^^^^^^^^^^^^^^^^^^^^^^^^^^^^^^^^^^^^^

This action may just be the puzzle piece toward progress in getting Iraq back on its feet and under their new government?

What do you think?

tomder55 answered on 06/29/06:

Interesting to note that the Iraqi insurgency's timetable for an American withdrawal is longer than the Democratic plan in Congress. It is an important start in the negotiation process that will bring the insurgency back into the fold .

My favorite Iraqi blogger Mohammed wrote :

So far, everybody in Iraq feels good about Maliki's plan and expressed their hopes for it to meet success and ease the suffering of the Iraqi people; everybody except for the Sadrists and the association of Muslim scholars who both criticized the plan and said it wasn't acceptable and expected it to fail. The question is do they are expecting it to fail only because they think it is not framed in a workable way or because they wish for it to fail? I'm afraid the latter is the likely answer.

What a mistake it was to let Mookie Sadr off the hook when we had him at the battle in the cemetary near the Imam Ali Shrine in Najaf.

Maliki knows that the hardest core (Sadrites;the groups controlled by Iran ;some of the hard core Baathists who know that they commited horrible crimes against Iraqis while they were in control ;the hard core criminals that Sadddam released from jail during the invasion) are unlikely to accept his plan . He is working with the seven groups he is most likely to convince to lay down arms. After that the Mahdi Army et-al can be brought into the fold by other means .

Ultimately I think it will be a success because for the insurgency the alternative is to feel the wrath of the Shia who unfettered will take their revenge .Ask them if they would prefer that or sitting in Abu Ghraib with womens underware on their heads .

Americas role in this end game will be critical ;if we are forced out too soon by a political change in this country then all bets will be off as far as the ultimate success of this process.

ETWolverine rated this answer Excellent or Above Average Answer
Itsdb rated this answer Excellent or Above Average Answer
jackreade rated this answer Excellent or Above Average Answer

Question/Answer
Itsdb asked on 06/27/06 - Keller explains

Excerpts from "Letter From Bill Keller on The Times's Banking Records Report"

    Some of the incoming mail quotes the angry words of conservative bloggers and TV or radio pundits who say that drawing attention to the government's anti-terror measures is unpatriotic and dangerous. (I could ask, if that's the case, why they are drawing so much attention to the story themselves by yelling about it on the airwaves and the Internet.)...

    ...It's not our job to pass judgment on whether this program is legal or effective, but the story cites strong arguments from proponents that this is the case. While some experts familiar with the program have doubts about its legality, which has never been tested in the courts, and while some bank officials worry that a temporary program has taken on an air of permanence, we cited considerable evidence that the program helps catch and prosecute financers of terror, and we have not identified any serious abuses of privacy so far. A reasonable person, informed about this program, might well decide to applaud it. That said, we hesitate to preempt the role of legislators and courts, and ultimately the electorate, which cannot consider a program if they don't know about it.

    We weighed most heavily the Administration's concern that describing this program would endanger it. The central argument we heard from officials at senior levels was that international bankers would stop cooperating, would resist, if this program saw the light of day. We don't know what the banking consortium will do, but we found this argument puzzling. First, the bankers provide this information under the authority of a subpoena, which imposes a legal obligation. Second, if, as the Administration says, the program is legal, highly effective, and well protected against invasion of privacy, the bankers should have little trouble defending it. The Bush Administration and America itself may be unpopular in Europe these days, but policing the byways of international terror seems to have pretty strong support everywhere. And while it is too early to tell, the initial signs are that our article is not generating a banker backlash against the program.

    By the way, we heard similar arguments against publishing last year's reporting on the NSA eavesdropping program. We were told then that our article would mean the death of that program. We were told that telecommunications companies would — if the public knew what they were doing — withdraw their cooperation. To the best of my knowledge, that has not happened. While our coverage has led to much public debate and new congressional oversight, to the best of our knowledge the eavesdropping program continues to operate much as it did before. Members of Congress have proposed to amend the law to put the eavesdropping program on a firm legal footing. And the man who presided over it and defended it was handily confirmed for promotion as the head of the CIA.

    A secondary argument against publishing the banking story was that publication would lead terrorists to change tactics. But that argument was made in a half-hearted way. It has been widely reported — indeed, trumpeted by the Treasury Department — that the U.S. makes every effort to track international financing of terror. Terror financiers know this, which is why they have already moved as much as they can to cruder methods. But they also continue to use the international banking system, because it is immeasurably more efficient than toting suitcases of cash.

    I can appreciate that other conscientious people could have gone through the process I've outlined above and come to a different conclusion. But nobody should think that we made this decision casually, with any animus toward the current Administration, or without fully weighing the issues.


Does anyone actually believe the Times printed the story with no animus toward the administration - particularly when in his opening remarks he disparages the "angry words of conservative bloggers and TV or radio pundits"? It's not their job to pass judgment by the way...

The Times apparently feels it is their responsibility to make sure the public has a say in every intelligence program, since they "cannot consider a program if they don't know about it." Aparrently warrants and subpoenas aren't enough any more, we must also wait for public approval before gathering information that may prevent an attack or catch a terrorist.

Wizbang appropriately summed the letter up this way:

    Dear Reader:

    1) We have no reason to believe the program was illegal in any way.

    2) We have every reason to believe it was effective at catching terrorists.

    3) We ran the story anyway, screw you.

    Bill Keller

tomder55 answered on 06/28/06:

we cited considerable evidence that the program helps catch and prosecute financers of terror, and we have not identified any serious abuses of privacy so far. A reasonable person, informed about this program, might well decide to applaud it.

but not the Slimes.

That said, we hesitate to preempt the role of legislators and courts, and ultimately the electorate, which cannot consider a program if they don't know about it.

ummmm that's why it is called covert YOU ARE NOT SUPPOSED TO KNOW ABOUT IT !!!!!

And while it is too early to tell, the initial signs are that our article is not generating a banker backlash against the program.

so you think moron !

I want to see this idiot frog-marched .




Itsdb rated this answer Excellent or Above Average Answer

Question/Answer
Itsdb asked on 06/28/06 - 'We're About to Enter the ླྀs Again'

By Randy Hall
CNSNews.com Staff Writer/Editor
June 28, 2006

(CNSNews.com) - America is about to revisit one of the most turbulent decades in its history, Democratic National Committee Chairman Howard Dean told a religious conference in Washington, D.C., on Tuesday. "We're about to enter the ླྀs again," Dean said, but he was not referring to the Vietnam War or racial tensions.

Dean said he is looking for "the age of enlightenment led by religious figures who want to greet Americans with a moral, uplifting vision."

"The problem is when we hit that ླྀs spot again, which I am optimistic we're about to hit, we have to make sure that we don't make the same mistakes," Dean added.

Anger over the Vietnam War and the country's escalating racial tensions made the late 1960s one of the most painful eras in American history. Republican Richard Nixon was elected president in 1968, following the assassinations of Martin Luther King, Jr., and Sen. Robert Kennedy, as well as the riot-marred Democratic National Convention in Chicago.

Later in his speech Tuesday, Dean appeared to backtrack. "I'm not asking to go back to the ླྀs; we made some mistakes in the ླྀs," he said. "If you look at how we did public housing, we essentially created ghettoes for poor people" instead of using today's method of mixed-income housing.

Another mistake Democrats made in the ླྀs, Dean acknowledged, was that "we did give things away for free, and that's a huge mistake because that does create a culture of dependence, and that's not good for anybody, either," he noted, a reference to the Great Society welfare programs created by Democratic President Lyndon Johnson in the mid-1960s.

"Those mistakes were not the downfall of our program," Dean added. "They helped a lot more people than they hurt. But we can do better and we will do better and our time is coming."

Alternating between references to the "McCarthy era" of the 1950s, which he accused the Bush administration of reviving, the decade of the 1960s and the current era, Dean explained that he was "looking to go back to the same moral principles of the ཮s and ླྀs."

That was a time that stressed "everybody's in it together," he said. "We know that no one person can succeed unless everybody else succeeds."

Dean's comments Tuesday came at a religious gathering convened in the nation's capital to discuss ways of eliminating poverty. After stating that America "is about as divided as it has been probably since the Civil War," Dean declared that "we need to come together around moral principles, and I'm talking about moral principles like making sure no child goes to bed hungry at night."

"I'm talking about moral principles like making sure everybody in America has health insurance just like 36 other countries in the world," he added. "This is a moral nation, and we want it to be a moral nation again."

As one method of accomplishing that goal, the DNC chairman called on Congress "to raise the minimum wage until we have a living wage in this country." He dismissed criticism of a minimum wage hike as "economists' mumbo-jumbo."

"We're simply asking to give the people who are working for minimum wage the same raise that Congress has had every year for the last 20 years," he said.

Dean also stated that the Democratic Party helped give people "the opportunity to become middle class" during the 1960s.

"I do think that empowering people to help themselves is what we should be doing in the 21st century," he added, stating that the Democratic Party now emphasizes the value of work.

"If you work hard, you ought to be able to support your family," the DNC chairman noted, and "in America, you need the opportunity to work hard, and that means some level of support from government -- no handouts, but some level of support so that you really do have a genuine opportunity to contribute to the country."

The DNC chairman pointed to President Bush's tax cuts as a major obstacle to what he called "tax fairness." He also criticized the Republican Congress for being "the biggest 'big government' government we've ever had," though he did make at least one positive comment about the GOP.

"How about if I'm a wild-eyed radical liberal who is willing to say the conservatives had some good ideas?" Dean told his audience. "But let's go back and make what we wanted to work, using some of their ideas to make sure that the mistakes don't get made again," he added.

"It's nice to see that Howard Dean's hostility to the religious community ends when people of faith vote Democrat," Republican National Committee spokesman Josh Holmes told Cybercast News Service.

Holmes added he was not surprised that "Howard Dean's political perspective is derived from a 1960s counterculture view of the world. What is surprising -- and disturbing -- is that he can urge a massive expansion of government and denounce the Democrat mistake of creating a 'culture of dependence' in the same speech."

"He may want to revisit that mistake to update his talking points and the Democrat policy manual," Holmes said.

Before leaving Tuesday's conference, the DNC chairman thanked those in attendance for giving him "a big lift."

"I came in the wrong door when I first got here," Dean said. "I came in the back, and everybody was talking about praising the Lord, and I thought, 'I am home. Finally, a group of people who want to praise the Lord and help their fellow man just like Jesus did and just like Jesus taught.' Thank you so much for doing that for me."

~~~~~~~~~~~~~~~~~~~~~~~~~~~~~~~~~~~~~~~~~~~~~~~~~~~~~~~

Pardon me, but - BWAAAAAA HAAAA HAAAAAaaaaaaaaaaaa!

So much to discuss I don't know where to start...anyone care to take a stab at it?

tomder55 answered on 06/28/06:



Wrap that tin foil hat a little tighter . The US electorate has repudiated the 'progressive 'message for about 38 years now. Glad to see the moonbats are seizing control of the Democrat party again . Should be a fun ride . It was amazing to me that Dean who overloaded in the 2004 primaries was made the party spokesperson .But you could tell that they were stuck in the 1960s when they carted out that 1960s retread John Kerry as their standard bearer and Teddy Kennedy as the party patrioch .

Itsdb rated this answer Excellent or Above Average Answer

Question/Answer
Itsdb asked on 06/28/06 - Where exactly...

...does the left stand on anything?

"It's time to bring them home." -John Murtha

"Last month, I introduced Senate Joint Resolution 36 which calls for the withdrawal of our combat troops from Iraq by the end of this year." -John Kerry

"We say bring the troops home now!" -Cindy Sheehan

Finally, Republicans have their own cut'n'run plan

    By Molly Ivins
    Fort Worth Star-Telegram
    Salt Lake Tribune

    AUSTIN, Texas - And then along comes Cut'n'Run Casey. We spend all last week listening to cut'n'run Democrats talking about their cut'n'run strategy for Iraq, and the only issue is whether they want to cut'n'run by the end of this year or to cut'n'run by the end of next year, and oh, by the way, did I mention that Republicans had been choreographed to refer to the Democrats' plans as cut'n'run?

    As Vice President Dick (''Last Throes'') Cheney said Thursday, redeployment of our troops would be ''the worst possible thing we could do. . . . No matter how you carve it - you can call it anything you want - but basically it is packing it in, going home, persuading and convincing and validating the theory that the Americans don't have the stomach for this fight.''

    Then right in the middle of Cut'n'Run Week, the top American commander in Iraq, Gen. George W. Casey Jr., held a classified briefing at the Pentagon and revealed his plan to reduce the 14 combat brigades now in Iraq to five or six. And here's the best part: Rather than wait 'til the end of this year or, heaven forefend, next year, Casey wants to start moving those troops out in September, just before whatever it is that happens in early November. They don't call him George W. Jr. for nothing.

    One has to admit, the party never ends with the Bush administration. The only question about Cut'n'Run Week is whether they meant to punctuate a weeklong festival of referring to Democrats as the party of ''retreat'' and ''the white flag'' with this rather abrupt announcement of their own cut'n'run program. Was it an error of timing?

    I say no. I say Karl Rove doesn't make timing mistakes. This administration thoroughly believes the media and the people have a collective recollection of no more than one day. Five days of cut'n'run, one day off and BAM, you get your own cut'n'run plan out there.


"Instead of offering real strategies for success, Republicans continue to play politics with this war. When it comes to Iraq, the only schedule that matters to Republicans is the U.S. election schedule" -Nancy Pelosi

So do they want the troops home or not - or just not in time to give Republicans an election boost? What kind of chutzpah does it take to demand Bush bring the troops home - so long as it's either immediately or after the election? Sounds to me like more evidence that the left is more concerned with power than the lives of our military heroes.

Steve

tomder55 answered on 06/28/06:

As I suspected these calls for troop withdrawal by the Democrats is the real political ploy .Of that you are 100% .correct

Bush has made no secret that [paraphrase] as the Iraqis stand up we will stand down. Perhaps the mission is winding down if General Casey is publicly offering a schedule for a phase down . It seems to me that most of our goals in the country are well on their way to being met. If the Iraqis feel that they can provide for their own internal security then the presence for a large American force is not really justified (barring possible hostilities with Iraq's neighbors).Clearly the conditions on the ground is the determining factor but the Dems. will play the politics for all it's worth (they have nothing else to offer).

The Dems. spin it and say :
Levin :Frankly, it's one of the worst-kept secrets in this town that there is going to be reductions in our forces, redeployments in our forces, before the election....

WALLACE: Now, this is twice now you have linked this to the election, so let me ask you, do you think the decision to pull troops out is a military decision or a political decision?

LEVIN: It should be a military decision. General Casey at the Pentagon a few days ago said he believes there will be fairly substantial troop reductions this year. Of course, when we say military decisions, ultimately, it should be a civilian decision.

But it shouldn't be a political decision, but it is going to be with this administration. It's as clear as your face, which is mighty clear, that before this election, this November, there's going to be troop reductions in Iraq, and the president will then claim some kind of progress or victory.


In your face Levin ! Yes it shouldn't be about the politics .

Itsdb rated this answer Excellent or Above Average Answer

Question/Answer
Itsdb asked on 06/27/06 - Oh how "Times" change

From Powerline, a September 24, 2001 editorial in the NY Times:

    Organizing the hijacking of the planes that crashed into the World Trade Center and the Pentagon took significant sums of money. The cost of these plots suggests that putting Osama bin Laden and other international terrorists out of business will require more than diplomatic coalitions and military action. Washington and its allies must also disable the financial networks used by terrorists.

    The Bush administration is preparing new laws to help track terrorists through their money-laundering activity and is readying an executive order freezing the assets
    of known terrorists. Much more is needed, including stricter regulations, the recruitment of specialized investigators and greater cooperation with foreign banking authorities. There must also must be closer coordination among America's law enforcement, national security and financial regulatory agencies.

    Osama bin Laden originally rose to prominence because his inherited fortune allowed him to bankroll Arab volunteers fighting Soviet forces in Afghanistan. Since then, he has acquired funds from a panoply of Islamic charities and illegal and legal businesses, including export-import and commodity trading firms, and is estimated to have as much as $300 million at his disposal.

    Some of these businesses move funds through major commercial banks that lack the procedures to monitor such transactions properly. Locally, terrorists can utilize tiny unregulated storefront financial centers, including what are known as hawala banks, which people in South Asian immigrant communities in the United States and other Western countries use to transfer money abroad. Though some smaller financial transactions are likely to slip through undetected even after new rules are in place, much of the financing needed for major attacks could dry up.

    Washington should revive international efforts begun during the Clinton administration to pressure countries with dangerously loose banking regulations to adopt and enforce stricter rules. These need to be accompanied by strong sanctions against doing business with financial institutions based in these nations. The Bush administration initially opposed such measures. But after the events of Sept. 11, it appears ready to embrace them.

    The Treasury Department also needs new domestic legal weapons to crack down on money laundering by terrorists. The new laws should mandate the identification of all account owners, prohibit transactions with "shell banks" that have no physical premises and require closer monitoring of accounts coming from countries with lax banking laws. Prosecutors, meanwhile, should be able to freeze more easily the assets of suspected terrorists. The Senate Banking Committee plans to hold hearings this week on a bill providing for such measures. It should be approved and signed into law by President Bush.

    New regulations requiring money service businesses like the hawala banks to register and imposing criminal penalties on those that do not are scheduled to come into force late next year. The effective date should be moved up to this fall, and rules should be strictly enforced the moment they take effect. If America is going to wage a new kind of war against terrorism, it must act on all fronts, including the financial one.


I guess they forgot?

Steve

tomder55 answered on 06/28/06:

What irked me was Alberto Gonzalez saying I think it's premature to call for a prosecution of The New York Times . Perhaps so ;but it is not too soon to haul some NY Slimes reportes into his office and demand disclosure of their sources .It is not to soon to assign a prosecutor to the case ( Patrick Fitzgerald isn't doing anything right now ..is he ?). If they don't cooperate then size them up for their orange jump-suit ala Judith Miller.

The White House has been slow to deal with these leaks .Forget the press ;they won't even go after known leakers like Mary McCarthy (at least she got canned for her actions ) No wonder the Slimes thinks it can act with impunity .

It doesn't take rocket science to understand that a terrorist event costs plenty of money to carry off. I do not even think that all of OBLs wealth could've accomplished 9-11 without state backing .

Powerline also has a letter written by Lieutenant Tom Cotton that is well worth reading :

Dear Messrs. Keller, Lichtblau & Risen:

Congratulations on disclosing our government's highly classified anti-terrorist-financing program (June 23). I apologize for not writing sooner. But I am a lieutenant in the United States Army and I spent the last four days patrolling one of the more dangerous areas in Iraq. (Alas, operational security and common sense prevent me from even revealing this unclassified location in a private medium like email.)

Unfortunately, as I supervised my soldiers late one night, I heard a booming explosion several miles away. I learned a few hours later that a powerful roadside bomb killed one soldier and severely injured another from my 130-man company. I deeply hope that we can find and kill or capture the terrorists responsible for that bomb. But, of course, these terrorists do not spring from the soil like Plato's guardians. No, they require financing to obtain mortars and artillery shells, priming explosives, wiring and circuitry, not to mention for training and payments to locals willing to emplace bombs in exchange for a few months' salary. As your story states, the program was legal, briefed to Congress, supported in the government and financial industry, and very successful.

Not anymore. You may think you have done a public service, but you have gravely endangered the lives of my soldiers and all other soldiers and innocent Iraqis here. Next time I hear that familiar explosion -- or next time I feel it -- I will wonder whether we could have stopped that bomb had you not instructed terrorists how to evade our financial surveillance.

And, by the way, having graduated from Harvard Law and practiced with a federal appellate judge and two Washington law firms before becoming an infantry officer, I am well-versed in the espionage laws relevant to this story and others -- laws you have plainly violated. I hope that my colleagues at the Department of Justice match the courage of my soldiers here and prosecute you and your newspaper to the fullest extent of the law. By the time we return home, maybe you will be in your rightful place: not at the Pulitzer announcements, but behind bars.

Very truly yours,

Tom Cotton
Baghdad, Iraq

Itsdb rated this answer Excellent or Above Average Answer

Question/Answer
jackreade asked on 06/28/06 - Bush Falls into AlQuaeda Trap

From ABCNews dot com:

Al Qaeda Strategic Vision: Engage the U.S. Overseas, Not at Home

June 27, 2006 10:09 AM

"Westpoint Al Qaeda's strategic vision involves challenging the United States and its allies overseas using small- to medium-scale attacks, according to an online book available on extremist websites that has become the seminal jihadi textbook. The first English translation of the text is being circulated this week among DOD and government policy circles.

The translation is being released by the Combating Terrorism Center at West Point. As ABC News reported last month, the Center has been translating thousands of declassified insurgent and extremist documents that were seized in Iraq and Afghanistan.

Abu Bakr Naji, an al Qaeda insider and author of the book, "The Management of Savagery," believes that the 9/11 attacks accomplished what they needed to by forcing the U.S. to commit their military overseas. He says 9/11 forced the U.S. to fall into the "trap" of overextending their military and that "it began to become clear to the American administration that it was being drained."

"The focus is on mid- to small-range targets in the region and not go after big symbolic targets like the Twin Towers," says Will McCants, a fellow at the Combating Terrorism Center at West Point, who translated the 268-page document.

McCants describes Naji as a highly placed, well-informed insider whose book lays out the big strategic vision of al Qaeda.

McCants believes that Naji is very concerned that a large-scale attack, such as the aborted chemical attack that would have targeted New York City subways in early 2003, would alienate al Qaeda's constituency. "Naji is wary of initiating that sort of attack because right now he feels al Qaeda has the upper-hand in the public relations battle," said McCants.

"Naji believes the way you really hurt empires is to make them commit their military far from their base of operations," according to McCants."

@@@@@@@@@@@@@@@@@@@@@@@@@@@@@@@@@@@@@@@@@@@@@@@2


Did Bush play into the hands of the Al Quaeda master plan?

Is the way to hurt empires is to make them commit their military far from their base of operations? Sounds like the terrorists are taking a lesson from the Roman Empire. Why would any comparison be true today in the age of airplanes, and technology? Are they full of it?

tomder55 answered on 06/28/06:

it certainly would fit into alot of people's preconceptions to compare the US under an 'Emperor' Bush to the fall of the Romans . The Romans of course had an extremely long run as the sole super-power .

As you can tell by all these attacks in the US on small to midsized targets that this strategy is working exceedingly well for al Qaeda . They have been pummelled in Iraq so drawing us into battle in the umah is working well for them too.Zarqawi was the equivalent to them what Stonewall Jackson was to the South ...General Lee could never find an adequate replacement

As for the chemical attack on the NY subways ;well there are a couple of theories . First : Ayman Al-Zawahiri gave instructions to wait until after the 2004 elections because if Kerry won we probably would've substantially retreated from the war against jihadistan . Second : it was determined that there would not be enough victims in such an attack .

McCants believes that Naji is very concerned that a large-scale attack, such as the aborted chemical attack that would have targeted New York City subways in early 2003, would alienate al Qaeda's constituency. "Naji is wary of initiating that sort of attack because right now he feels al Qaeda has the upper-hand in the public relations battle," said McCants.

al-qaeda is losing the battle of hearts and minds in Iraq and elsewhere because of their barbaric tactics . But if they could carry out an attack that kills thousands of Americans they would do so again with the blessing of their leadership .

I am sure that al-Qaeda excels in long term strategic thinking but they have not demonstrated an ability to have a particularly long planning cycle . The fact that we have broken up cells in this country and their apparent lack to stike inside leads me to believe that our overall strategy has been sound. When one also factors in the large number of senior al Qaeda planners who have been captured or killed since 9/11, it is clear that the organization is under enormous pressure.

How much longer could al Qaeda wait before activating any sleeper cells it might have? Logic would argue that any sleeper operatives still out in the cold either must be getting exceedingly nervous at this point or they do not exist. If they do exist, the ability to remain hidden so long after 9/11 implies that they possess a degree of professionalism on par with that of the KGB and far exceeding anything exhibited by al Qaeda operatives so far .

ETWolverine rated this answer Excellent or Above Average Answer
Itsdb rated this answer Excellent or Above Average Answer
jackreade rated this answer Excellent or Above Average Answer

Question/Answer
jackreade asked on 06/28/06 - For Board Member, excon

Today, it was revealed that Republican radio mouthpiece, Rush Limprod was detained for bringing *Viagra* into the country without a prescription in his name.

What is an unmarried man doing with a sexual enhancement product. Isn't the Republican Party the party of high Christian morals? Isn't it still a sin to have sex in any other way except with one's spouse?

I imagine the right wing Christians here will rush(pun intended)to condemn Mr Limprod.

NOte: He confessed to being addicted to hillbilly heroin(Oxycontin) and is involved in a court case related to that addiction.

$$$$$$$$$$$$$$$$$$$$$$$$$$$$$$$$$$$$$$$$$$$$$$$$


OK, line up and give him heck!

tomder55 answered on 06/28/06:

Talk about ad hominem ! 55 year old man uses Viagra. Wow. NY Slimes exposes legal classified anti-terrorist program. Zzzzz.



I will not condemn or defend him . Just wanted to clarify that a settlement was reached in his other case which all but clears him of the charges pending he behaves . I do not know if popping the blue pill constitutes misbehavior in the eyes of the law in Palm Beach but it is not illegal under Florida law for a physician to prescribe medication in a third party's name if all parties are aware and the doctor documents it correctly .Doctors give "sample packs" of medication out all of the time to patients to try. No story here ..sorry .

Rush was released without charges in this Viagra case but I imagine that the crusading DA would love to tie this to a possible violation of the deal .

ETWolverine rated this answer Excellent or Above Average Answer
Itsdb rated this answer Excellent or Above Average Answer
jackreade rated this answer Excellent or Above Average Answer

Question/Answer
CeeBee2 asked on 06/27/06 - Maybe this is the problem????

Article Abstract:
Progressives have fallen into a trap. Emboldened by President Bush's plummeting approval ratings, progressives increasingly point to Bush's "failures" and label him and his administration as incompetent. Self-satisfying as this criticism may be, it misses the bigger point. Bush's disasters Katrina, the Iraq War, the budget deficit are not so much a testament to his incompetence or a failure of execution. Rather, they are the natural, even inevitable result of his conservative governing philosophy. It is conservatism itself, carried out according to plan, that is at fault.

Read the entire article at:

Bush is not an incompetent

What do you think?

tomder55 answered on 06/27/06:

Sum up the argument this way : Since Bush can no longer run ,it is not in the oppositions best interest to lay the blame for what they perceive as failures on Bush because it then would be logical to conclude that if you found a more competent conservative than Bush you could conceivably get better results . Better at this point to place these alleged failures on the underlying flawed political philosophy that is the premise in the decisions that have led to these "failures ".In other words it is in fact the conservative successes that has the progressives in a panic .

But where they continue to fall into this trap is that this continues to help them define their alternative . Are they saying that the progressive philosophy would make for a more competent administering of the executive branch ? How so ? What is it about progressive that would make them better managers ? What exactly is their alternative ? They shy away from giving details . They have consistently failed to excite the electorate with their alternative and with good reason .

The last 2 Democrat Presidents ran from the middle .They did not offer a progressive alternative just better management . The candidates who have run from the left or even the moderate left have been pummelled at the polls.

I would love to hear it however . George Lakoff, Marc Ettlinger and Sam Ferguson have it right in this regard . Let's put the cards on the table . Let's hear liberals shout proudly they are liberal and make their case. Then let's see which side of the debate the American people really think is most competent for running the country .

CeeBee2 rated this answer Excellent or Above Average Answer
Itsdb rated this answer Excellent or Above Average Answer

Question/Answer
ETWolverine asked on 06/26/06 - Making The Connection

From the Weekly Standard "Scrapbook", 7/3/06.

"When we went to Beirut, I said to President Reagan, 'Get out.' Now, the other day we were doing a debate, and they said, 'Well Beirut was a different situation. We cut and run.' We didn't cut and run. President Reagan made the decision to change direction because he knew we couldn't win it. Even in Somalia, President Clinton made the decision, 'We have to, we have to change direction.'... We need to change direction. We can't win a war like this.... At some point you got to reassess it like Reagan did in Beirut, like Clinton did in Somalia. You just have to say, 'Okay, it's time to change direction.'"

--- Rep. John Murtha, unrging a U.S. withdrawal from Iraq, "Meet the Press," June 18, 2006.


"We have seen in the last decade the decline of the American government and the weaknesses of the American soldier, who is ready to wage cold wars and unprepared to fight long wars. This was proven in Beirut when the Marines fled after two explosions. It also proves they can run in less than 24 hours, and this was also repeated in Somalia... After a few blows, [the Americans] ran in defeat.... They forgot about being the world leader and the leader of the new world order. [They] left, dragging their corpses and their shameful defeat."

--- Osama bin Laden, ABC News interview, May 28, 1998.

Anyone get the point yet? It was the very cases that Murtha points to as examples of "reassessment" and "changing direction" that led OBL to believe that we had become weak, and that we could be defeated on 9-11. And now Murtha wants us to make the same mistake again, which will only lead to more 9-11s.

And people wonder why I say that Murtha is anti-soldier. The actions that he is proposing are going to get more soldiers killed. The very fact that he is proposing them at all emboldens the enemy and is getting soldiers killed.

If Murtha really cared fo the soldiers, he'd shut the hell up and let them get on with the job of winning the war. But he doesn't. Instead he claims that they don't have the ability to win the war (despite the fact that they keep proving him wrong) and keeps accusing them of war crimes without any proof of what actually occured. THAT is anti-military, anti-soldier rhetorric, and it is endangering the troops.

Elliot

tomder55 answered on 06/27/06:

His "reassessment" and "changing direction" always sounds like "RUN AWAY !!! " to me .

He is not pro-military he is pro-military procurement... but mostly if it benefits his district . His brother , Robert Murtha's lobbying firm represents 10 companies that received more than $20 million from last year's defense spending bill. "Clients of the lobbying firm KSA Consulting -- whose top officials also include former congressional aide Carmen V. Scialabba, who worked for Rep. Murtha as a congressional aide for 27 years -- received a total of $20.8 million from the bill," . Murtha is the top recipient of defense industry dollars this year . It is in defense spending that he got his reputation as a hawk ;that and the impeneterable shield he has surrounded himself with by claiming a veteran's immunity from criticsm or even critical scrutiny .

ETWolverine rated this answer Excellent or Above Average Answer

Question/Answer
kindj asked on 06/26/06 - From the Christianity board

This cracks me up. I felt the need to copy it over here for the benefit of all.

~~~~~~~~~~~~~~~~~~~~~~~~~~~~~~~~~~~~~~~~~~~~~~~~~~~~
Christians can't morally endorse cruel and unusual punishment unless they want to morph Jesus into an "eye for an eye" advocate.

"I celebrated inwardly when it was revealed that a single hold-out juror prevented the so-called twentieth hijacker from 9/11, Zacarias Moussaoui , from receiving the death penalty. This juror gave no reason, but I hope it was conscience pure and simple. The U.S. has isolated itself among First World countries by allowing the death penalty--123 countries have abolished it completely, or in practice never use it, a few permitting it under extreme circumstances.
Of the 50 countries that newly abolished the death penalty since 1985, only 4 have reinstated it. Why aren't more people chilled by the fact that in 2004, 97% of executions took place in China, Iran, Viet Nam, and the U.S.?

Execution amounts to cruel and unusual punishment by the world's prevailing standards. A current case before the Supreme Court is testing that proposition here. Yet somehow the American public feels undisturbed by this issue. Few if any politicians dare to run on the wrong side. In this case "wrong" means humane and rational. Why do we kill criminals? ***The right wing surely can't hide behind morality, unless they want to warp Jesus into an eye-for-an-eye advocate.***

No, the death penalty is almost entirely irrational. It has little if any deterrent effect. Tragic mistakes have been made in its application. The very fact that inmates must wait on death row for years, even decades, is cruel enough. How many times do they die in their own minds before the actual event?

The landscape of cruelty in America has become more and more disturbing recently. Guantanamo is a global disgrace, yet one hears feeble outcries over it here, especially in Congress. Abu Ghraib has led to minimal repercussions, and rumors of CIA torture centers in Eastern Europe sound all too plausible. The fact that the tide of cruelty has crept up gradually is no excuse.

Having escaped death, Moussaoui now faces doing time in a "super max" prison in Florence, Colorado, where Terry Nichols (co-conspirator with Timothy McVeigh) and Ted Kaczynski (the Unibomber) already endure conditions that frequently induce psychosis. A super max prison is an antiseptic hell where inmates sit in isolation for 23 hours a day, being allowed out of their cells for only an hour's exercise. They have no human contact, no television, no library except for a collection of law books (access to legal information is mandated by the courts). In some of these facilities, which have grown extremely popular in recent decades, the cells are lit up 24 hours a day under surveillance cameras.

Under what possible moral scheme can a civilized country consider this anything but barbaric? Our prisons are called penitentiaries (from the root word 'penance') because over two hundred years ago it was felt that an enlightened society must move beyond Old Testament revenge for wrong-doing. Now we have slipped back across that moral boundary, and the saddest thing, in this boom time for building more prisons, locking away more non-violent criminals, and handing down maximum sentences, is that we have learned to condone cruelty almost as if it didn't exist. As if it was a good thing."===Deepak Chopra, Blogging

~~~~~~~~~~~~~~~~~~~~~~~~~~~~~~~~~~~~~~~~~~~~~~~~~~~

Now, is it just me, or does the signature line really say all that needs to be said?

tomder55 answered on 06/27/06:

His mind is a whirl of incoherent disconnected contradictory philosophies but he convinces his groupies to believe he makes sense when he speaks .He must really serve some good kool-aid .These philosophies include but are not limited to French Existentialism, postmodernism, relativism, drug induced hippie-era Anarcho-Marxism, animals-are-people-too environmentalism, inhumane Humanism, Gandhi-lite and Noam Chomskyism . I would love to see how his peace rather than victory speeches would fly at a jihadist rally .I could hear it now "if you were to just adjust your attitudes all your problems would disappear " . I remember in the pre-war phase of Iraq he suggested that a solution to the crisis would be to build a Disney World in the M.E. I wonder if he takes off his rose colored glasses and tin-foil hat when he discusses the inequities and poverty in the Indian sub-Continent ?

kindj rated this answer Excellent or Above Average Answer

Question/Answer
kindj asked on 06/26/06 - OK, gonna give it a shot

Here is an example of the lovely West Texas springtime weather:


tomder55 answered on 06/26/06:

in NY we have been living in the tropics for the last week . I decided yesterday that so long as there was no lightning it was safe for me to mow the yard ;it was beginning to look like Jungle Habitat .

maybe it is jsut the resolution . When I was in Kansas a storm like that would have a more purplish hue .

I bet all that rain beats brush fires .

Itsdb rated this answer Excellent or Above Average Answer
kindj rated this answer Excellent or Above Average Answer

Question/Answer
jackreade asked on 06/24/06 - Found the W'sMD? Guess we can go home.

"We now have found stockpiles." - Rick Santorum Wednesday, June 21, 2006 - two days ago

"It turns out there were weapons of mass destruction in Iraq after all. Up to 500 canisters of degraded chemical weaponry that, according to David Kay who headed the U.S. weapons-hunting team in Iraq from 2003 until early 2004, is about as hazardous as what the average American household has under their sink.
They can cause burns but are unlikely to kill anyone. The canisters pre-date the first Gulf War.

This didn't stop master politician Rick Santorum from trotting out the evidence. Proof that there were Weapons of Mass Destruction in Iraq after all. I applaud Rick Santorum on two points. First, for acknowledging that the reason we went to Iraq was to search for weapons of mass destruction. It's nice to see the Senator, who has been such a supporter of the war, ***admit to the original pretext that would turn Iraq into a ^breeding ground^ for terrorists and cost 2,500 American soldiers their lives***. It would be easy to forget with the constant drumbeat out of Washington about Democracy being on the march, and bringing freedom to the Middle East, the original reason for the invasion. A lot of people think we went to Iraq to make life better for the Iraqi people. Santorum has the courage to remind us that's ***not why*** we are there.

The administration is on the same page. "They are weapons of mass destruction. They are harmful to human beings. And they have been found," Donald Rumsfeld said. Weapons of Mass Destruction can now be reduced to things that burn. The bar is lowering.

The other great thing about Rick Santorum, engaged in a heated Senate race he is destined to lose, reaching his fundamentalist arms around these old, leaky, degraded canisters, is the Republican policy, under the Bush administration, of **stretching the un-truth, of manipulating intelligence for political ends**. It's like 2002 all over again. It's uranium in Niger, it's the Downing Street memo, it's a capability to launch a nuclear attack in less than an hour, it's tubes on a boat, it's Colin Powell's slide show to the United Nations, it's weapons-of-mass-destruction-related-activities. Here now, in plain view, a loony Republican Senator reducing the war in Iraq to these old, probably lost canisters.

"Remember," he tells us, "Saddam Hussein killed 5,000 of his own people with just three of these canisters."

We, of course, have killed at least 30,000 of Saddam's people, and probably many, many more, in our search for those elusive uranium enriched aluminum tubes. And when we didn't find those we said we were spreading Democracy. Now that we have failed on that front it's good to see the argument returning to its roots. ***Now that we've found the weapons of mass destruction we can go home***.

To quote Rudolph Giuliani at the Republican National Convention which I had the pleasure to attend. "Weapons of mass destruction? Saddam Hussein was himself a weapon of mass destruction." The man farted nitrogen. ****The catastrophe is ours.****



- Stephen Elliott is the author of Looking Forward To It Or How I Learned To Stop Worrying And Love The American Political Process

$$$$$$$$$$$$$$$$$$$$$$$$$$$$$$$$$$$$$$$$$$$$$$$$

tomder55 answered on 06/24/06:

lets have a reality show where they open a couple of these shells in an enclosed room where Stephen Elliot is seated . Have him take a deep breath and say (sniff sniff )" this is old gas ...no probelmo "

jackreade rated this answer Average Answer
powderpuff rated this answer Excellent or Above Average Answer

Question/Answer
HANK1 asked on 06/23/06 - THE MICE WILL PLAY:

Ever seen "The Fleecing Of America" on NBC Nightly News? Those politicians are stealing us blind. After they're exposed, there are no follow-ups re: final dispositions of this hanky panky. Are they repremanded in some way? Perhaps a slap on the wrist? What can the average American citizen do about giving these Devils their due?

* Please don't tell me we can vote them out of office. If they leave the White House, more mice will take their place and do the same thing.

HANK

tomder55 answered on 06/24/06:

Hank ,only the Congress can spend money . Oh the President signs the legislation into law and is equally responsible but if there is a correction to be done it has to start at Congress. I have a few thoughts :

1. An Amendment that provides for term limits . The American people deemed it worthy to limit the term of the President ;Why not Congress ?Part of the corruption stems from the constant state of campaigning being done . The House of Reps serves a 2 year term . That means that someone like Jack Murtha who has been there since 1974 has campaigned for his job an incredible 16 times .Obviously he has to keep his campaign coffers full at all times. That leads to corruption .In his case he is one of the top beneficararies of "earmark " legislation .Yes ,a Congress person has to spend alot of time making sure that Federal dollars are pumped into their district . That leads to alot of the items that are reported on in the show you mention . Even in a case like Murtha where he was caught on tape during ABSCAM negotiating the dollar amt. of a bribe during the sting (he was an unindicted co-conspirator ),the people in his district have continued to send him back to the House year after year after year after year ....

2.Earmark reform . These are pork barrel spending packages that get added into unrelated legislation by the congress. A member may not like it but they vote it in anyway because ...well let's say a bridge to no-where is attached to a Vets Adm. funding bill . If they vote against the bill because of the bridge then it is reported that they voted against Vets benefits . The answer is to have congress vote on the record for every stipulation of a bill. Then it would be less likely that the bridge would be funded.

But then again maybe not.Let's say that member does not like the bridge but also wants congress to approve a highway extention to run by a hotel his brother owns . He will vote for the bridge funding because he wants the sponsor of the bridge to vote for the highway.

3. Presidential line item veto. Ultimately the President has to sign a bill into law. If he had the ability to remove the bridge and the highway extention from the bill then he could still sign a bill for Vets Adm funding without the added pork.

4. Remove the revolving door. Too many people alternate between gvt. service and being lobbiest . Set a 2 year restriction from gvt. service to working as a lobbiest.

these are just a few ways to reduce the lunacy but hate to say it because you said you do not want to hear it...they are ultimately accountable to the people who elect them in .

HANK1 rated this answer Excellent or Above Average Answer

Question/Answer
ETWolverine asked on 06/23/06 - WMDs revisited.

Let's see now. What could possibly point to the fact that Saddam really did have WMDs?

1) 500 TONS (that's 1 million pounds) of yellowcake uranium was found in Iraq.

2) 500 sarin and mustard gas filled weapons were found in Iraq.

3) The Dalfour report, which every liberal points to over the fact that it says that Saddam didn't have "stockpiles" of WMDs, clearly also states that Saddam had WMD programs in place, including an anthrax production facility that could produce anthrax-filled weapons within 4 weeks of releasing the UN sanctions. (Interesting how that part of the Dalfour report is completely ignored by the left.)

4) We have documents and audio tapes that prove that Saddam and his administration were deliberately hiding WMDs and lying about their existance and in what amounts to the UN inspection teams.

5) If Saddam didn't have such WMD programs, why were two of his cabinet officials nicknamed "Chemical Ali" and "Dr. Germ" by Iraqi officials?

6) Let's not forget that Saddam had used WMDs against Iran and the Kurds before as well, so claiming that he never had WMDs is just purely rediculous.

C'mon, lefties. The perponderance of "in-your-face" evidence is huge. When are you guys going to admitt that Bush was right all along?

Elliot

tomder55 answered on 06/23/06:

Imagine if we had never invaded Iraq and some of those 500 shells was used on a NYC subway attack.Would the liberals excuse Bush by saying they were old shells, and that Saddam basically had come clean ?

here is some more "old news " for the moonbats to consider . The only reason that the IED attack was not a success was because the attackers did not realize the shell was binary and that chemical reaction is caused by the spin of the shell .

Read former spook's blog (Back Story
)for some more informed assessment of the discovery .

While a number of bloggers, including Captain's Quarters and Powerline, have done an excellent job in tracking yesterday's announcement (and the underwhelming media response), there is a back story that must be told. It's a story of brueaucratic inepitude, apparent political and personal agendas, and the efforts of a few courageous individuals to get the truth out.

The story begins in April of this year, when a team of intelligence analysts, assigned to the Army's National Ground Intelligence Center (NGIC) published an exhaustive report on the continued recovery of chemical weapons in Iraq. Their report clearly noted that the weapons were clearly manufactured before the first Gulf War. However, the NGIC analysts also observed that some of the weapons remained in good condition (suggesting an Iraqi effort to preserve them), and posed a potential threat to coalition forces, if they fell into the hands of insurgents. From what I'm told, the report contained a full listing of all chemical weapons discovered in Iraq since the fall of Saddam, cut-away diagrams of the weapons, locations where they were found, and their potential lethality if employed by terrorists.

Obviously, the NGIC report ran against the conventional wisdom that "Iraq had no WMD" after the U.S.-led invasion, and (to its credit), the organization published the report, which was posted on INTELINK (the intelligence community's classified intranet) in April of this year. In that forum, the report could be easily accessed by anyone with access to the system, the proper security clearance, and a valid need-to-know. From an analytical standpoint, the team at NGIC did their job, and they deserve tremendous credit for publishing their report. That's what analysts are supposed to do--tell the truth, and let the chips fall where they may, even if their findings run contrary to popular assumptions and political agendas.

Shortly after the NGIC item was posted on INTELINK, Senator Santorum learned of its existence, and began pressing the Army for more information, and declassification of the report's key findings. At this juncture, however, political agendas and bureaucratic tail-covering became a factor. A GOP source sent me a copy of Senator Santorum's letter, requesting information on chemical weapons in Iraq, back in April. Amazing (or, perhaps not-so-amazingly), both NGIC and the Army's Intelligence and Security Command (INSCOM) ignored Santorum's request. Normally, DOD agencies are supposed to respond to a request from a member of Congress within 48 hours; the Army ignored Santorum's request for more than a month. In fact, Santorum and Hoekstra didn't get their information until the Intelligence Committee chairman obtained a copy of the NGIC report and reportedly "hit the ceiling." After that, the Director of National Intelligence, Ambassador John Negroponte, agreed to declassify portions of the report, which were released yesterday.

Why did the Army ignore Senator Santorum's initial request? That's an issue that the INSCOM commander, Major General John D. Freitas III, may be asked to explain the next time he's on the Hill. The same holds true for the NGIC Commander, an Army Colonel. But beyond the DOD's efforts to "slow-roll" Senator Santorum and Chairman Hoekstra, there's the larger issue of why the Defense Department and Intelligence Community "sat" on this information. Sources tell me that there is no evidence of the NGIC report making its way into high-visibility intelligence products, such as the daily update for the Chairman of the Joint Chiefs of Staff, the CIA's flagship National Intelligence Daily (NID), or the Presidential Daily Brief (PDB), now handled by Negroponte's staff. Additionally, there was no effort to inform key members of Congress on this issue, until they began demanding answers. Congressman Hoekstra has every right to be pissed; the Chairman of the House Intelligence Committee should not learn about the discovery of WMD in Iraq via an "under-the-table" copy of an Army report that was published almost two months earlier.

As a young intelligence officer, I was drilled that important information should make its way up the chain of command as soon as possible. Apparently, things have changed since I left the business. Information that contradicts prevailing judgements can be ignored, or simply buried on an intelligence website--let the customer find out on his own. If members of Congress want information, simply delay your response as long as possible, and provide data only when someone with enough horsepower (in this case, the HPSCI chairman) demands answers. Then, provide only a fraction of what they ask for.

If all this sounds vaguely familiar, it should. Such tactics have been part-and-parcel of how the intel community does business for decades. It's the sort of behavior that has created barriers between various intelligence agencies, and generated lingering suspicion and distrust between the community, the Congress, and (ultimately) the American people. More than a year into his tenure as DNI, Negroponte's intelligence community is still operating a lot like its predecessor. The American people have a right to know that we've been uncovering WMD in Iraq--just as they were led to believe that none still existed. Withholding that information is inexcusable; intel bureaucrats were apparently uncomfortable with the revelation that they had been wrong on Iraqi WMD, not once, not twice, but a total of three times.

The MSM--if it ever gets around to this story--will likely claim that Santorum and Hoekstra are playing politics with intelligence. This blog has been critical of Congress playing fast-and-loose with intel information in the past, but that doesn't appear to be the case this time. Santorum and Hoekstra played by the rules, made their requests through proper channels, and only released declassified portions of the document, with the approval of the DNI. Compare that to the antics of Vermont Senator Pat Leahy--who was booted from the Senate intel committee for leaking classified information--and you'll see that Santorum and Hoekstra were models of patience and decorum.

Kudos to the NGIC team for publishing this discovery, and to the members of Congress--Santorum, Hoekstra (and Pennsylvania Congressman Curt Weldon)--who pushed for its public release. Our elected officials should demand answers on why this important data never made its way up the chain of command, and why their requests for information were apparently stone-walled by the Pentagon and the intel community.


The big story here is that again elements of the intel. community are working to undermine Adminstration policy . The fact that Saddam had WMD is not a suprise to thinking persons. It is not a suprise either that they are beggining to turn up ,the amts. unaccounted for were too large to hide forever. That the moonbats dismiss this as important is pure cynical politic . But the danger of rogue elements in the intel depts is my big concern .

ETWolverine rated this answer Excellent or Above Average Answer
labman rated this answer Excellent or Above Average Answer
Itsdb rated this answer Excellent or Above Average Answer

Question/Answer
jackreade asked on 06/23/06 - Karzai-Reassess Plan for Afghanistan!

"Afghanistan's President Hamid Karzai has urged the international community to reassess how it is fighting the Taleban and their allies.

He said he was not surprised that so many people were being killed in southern Afghanistan.

His comments came as the US military said four more of its soldiers had been killed in north-eastern Afghanistan.

Meanwhile, al-Qaeda number two Ayman al-Zawahiri has called on Afghans to fight foreign forces in their country.

'Change in approach'

Speaking in Kabul, President Karzai said improving local government and strengthening the police and army was the way in which to tackle the problem of terrorism.


"I have systematically, consistently and on a daily basis warned the international community of what was developing in Afghanistan... and of a change in approach by the international community in this regard."

The BBC's Alastair Leithead in Kabul says Mr Karzai's criticism is a sign of his growing frustration at the worsening security situation in Afghanistan.

Without mentioning Pakistan by name, Mr Karzai also made strong references to where the international forces should be turning their attention to cut off the insurgent source of funding, training and ideology, our correspondent adds.

Pakistan has been widely blamed for not doing enough to stop insurgents crossing into Afghanistan and even for complicity - something Pakistan denies.


Coalition casualties since 2001
United States: 310
Britain: 8
Other nations: 83

The American soldiers were killed after US-led coalition forces attacked "enemy extremists" in the Kamdesh district of Nuristan Province, a coalition statement said.

Fighting has intensified in Afghanistan in recent months. Hundreds of suspected Taleban militants have been killed.

Attacks blamed on the Taleban and their allies, including suicide bombings, have also targeted US and other foreign troops.

'Stand as one'

In a three-minute video that emerged on Thursday, Osama bin Laden's deputy, Ayman al-Zawahiri called on Afghans to stand up against the "infidel forces that are invading Muslim lands".


"Muslim brothers in Afghanistan, and especially in Kabul, stand as one with the mujahideen (Islamic fighters) so that the invading forces might be expelled," he said.

Mr Karzai responded by describing the al-Qaeda deputy as an "enemy of the Afghan people" and blamed him for causing Afghans misery over the years.....

%%%%%%%%%%%%%%%%%%%%%%%%%%%%%%%%%%%%%%%%%%%



The Occupations are falling apart, and we only get "stay the Course" out of Bush and his Crime Family. Stay the Course? No new ideas or plans for the two wars the Bush administration fought and are being lost to insurgents. A Republican dominated government incompetent by any prudent person's judgement.

tomder55 answered on 06/23/06:

Let's put it this way ;Afghanistan ,like Iraq is pretty much surrounded by neighbors who would like nothing more than to see democracy fail there . think it important for the US to honor it's commitments to free nations .

Karzai knows the only way to make major headway against the Taliban led jihadist is to eliminate the Taliban in Pakistan, where the militant group is based. This posture has led to heightened tensions between Islamabad and Kabul in recent months.


I think it a positive development today that Pres. Musharraf of Pakistan called for more cooperation between the two nations in the war on terror . But in return he wants a power a sharing deal arranged between Karzai and the pro-Pakistan Taliban factions . If that were to happen then just like in Iraq I'm sure that Pakistan would turn on the al Qaida foreign jihadists who are living in sanctuary there .Karzi will most likely resist this overture even though some of the Taliban have become more moderate and should probably be considered .The US should "suggest" to him that it may not be the worse thing to make this accomodation .

If such an arrangement could be made then probably Musharrif would "over-look" a US led incursion into the border area between Pakistan and Afghanisan for the purpose of cleaning out the al -qaeda presence there .

You wanted new ideas and plans ? There it is . If I can get this info then you can be sure that the Bush Adm knows it too. Well gee .Why didn't they just say so ? DUH . Any democrat who has to ask that question does not deserve a leadership role .

ETWolverine rated this answer Excellent or Above Average Answer
jackreade rated this answer Poor or Incomplete Answer

Question/Answer
jackreade asked on 06/23/06 - Read This and Get P*ssed-Off

From the website baltmoresun dot com.

"WASHINGTON // Nearly five years after the Sept. 11 attacks, the Departments of Justice and Homeland Security continue to clash over who is in charge of coordinating and vetting information on terrorism. As a result, state and local authorities continue to get conflicting or incomplete information - sometimes none at all - on threats inside the United States, officials say.

The feud over control of the information caused federal agencies last week to miss a White House deadline for outlining how it should be distributed to state and local authorities, intelligence and counterterrorism officials said yesterday.

The absence of a federal game plan is causing "confusion at the state level," said Col. Ken Bouche, who heads the information management division of the Illinois State Police. "The longer we wait ... the more leads we miss."

Under federal law, the Departments of Justice and Homeland Security are considered the main repositories for information about terrorism.

In part because it has a long-established system for sharing information through joint task forces, the Justice Department was at first "somewhat resistant" to the notion of a broader plan, said a counterterrorism official familiar with the issue. Eventually, officials at Justice agreed that Homeland Security had an important role to play and that a plan was needed to incorporate the department.

Meanwhile, though Homeland Security officials were well-intentioned, they consistently took the position that the legislation that created the department gave it sole responsibility for information sharing, the counterterrorism official said.

In a December memo, President Bush directed Attorney General Alberto R. Gonzales and Homeland Security Secretary Michael Chertoff to provide a solution by June 14.

But White House spokeswoman Dana Perino said yesterday that the president still had not received their proposal and left open when she expected it to arrive.

"They reached some consensus on broad aspects of the framework, and they are still working," she said. "We expect to see some additional progress soon."


%%%%%%%%%%%%%%%%%%%%%%%%%%%%%%%%%%%%%%%%%%%%%%%%


So, the heads of two of Bush's agencies are having a TURF WAR??????????

So much for the Bush Crime Family protecting American soil from terrorist attacks. They are in a TURF WAR.

Gambling with the lives of American's everywhere.

tomder55 answered on 06/23/06:

I thought that was why the office of DNI headed by John Negroponte was established to make decisions on 'turf issues'. But as you know ,the Federal Burro~cracy has a lot of career geeks stalking the hallways of their Depts with the mind set that they can out-last specific administrations .They are ususally very protective of their "turf " and are reluctant to embrace change . They will be purged just like the CIA needs purging of the Clintonoids .

jackreade rated this answer Poor or Incomplete Answer

Question/Answer
jackreade asked on 06/22/06 - Santorum outed as Liar about his WMD's Claims



From CROOKS AND LIARS BLOG SITE::::

"Santorum debunked by Colmes over WMD's

Santorum debunked over WMD's by **FOX NEWS***

Santorum showed up to do his thing with Peter Hoekstra on H&C and it took one phone call by Jim Angle of FOX News to debunk Santorum's WMD claims today. That's pretty embarrassing when the Dick Cheney network undermines him. Hannity was all excited and tried to say that WMD's were only "a part of the reason we went Iraq." (See quote.)

If that is the document that's classified, isn't little Ricky breaking about a gazillion different federal laws by exposing them? I've taken the precaution of blackening it a bit. Of course, I'm no attorney, but I believe this is the law.

Santorum: I'll show you the classified documents right here...

Colmes: It's Alan Colmes. Senator, the Iraq Survey Group, uhh, let me go to the Duelfer Report-says Iraq did not have the weapons our intelligence believed were there. And Jim Angle who reported this for Fox News-quotes a defense official who says these were pre-1991 weapons that could not have been fired as designed because they already been degraded.

And the official went on to say that they are-these are not the WMD's this country and the rest of the world believed Iraq had-and not the WMD's for which this country went to war. So the chest beating that the Republicans are doing tonight thinking this is a justification is not confirmed by the defense department.

Santorum: Well, ahh, I'd like to know who that is. The fact of the matter is I'll wait and see what the actual Defense Department formally says or more importantly what the administration formally says. This report...

&&&&&&&&&&&&&&&&&&&&&&&&&&&&&&&&&&&&&&&&&&&&&&&&&&

Santorum called liar by Fox News over WMD remarks.

My comment? bwahah haha hahahahahahah

tomder55 answered on 06/23/06:

Alan Colmes is the token liberal foil on Fox . As for the WMD claim he makes that these were not the WMD they were looking for ....b.s. We were looking for unaccounted for WMD . I think 500 shells of chemical weapons is a significant find in light of the fact that it only took about 15 shells to kill 5000 Kurds . I know the moonbat image associated with WMD stockpiles is probably a hill of diabolical looking devices in a gleaming underground facility like the doomsday machines in a James Bond movie. That doesn't mean the weapons couldn't have killed people and just imagine the havoc they could cause as IEDs or sold on the market to terrorists .

The entire point of the UN Security Council resolutions was to strip Saddam of that capability ;and he obviously retained it, and lied about it.I can only assume that the fact that the Iraqis "missed" 500 older chemical shells that were in storage is due to the fact that they had a lot of newer WMDs they had to worry about disposing of. If those were the only ones they had you can bet they would know where they were.

My question is the timing of the declassification . If Bush wanted to use the info for political advantage it could've been released closer to discovery ;in time for the 2004 election.Of course the existence of such devices would remain classified if there was uncertainty of other stockpiles or hidden assets because we would not want to let the cat out of the bag until there was a full accounting .Who knows how many more of those shells are out in the thousands of weapons dumps?

I think the reason why the info. was held onto has something to do with the pending Iran crisis . The info.that is open source already implicates Russia ,China and France (especially Russia helped move a large qty. of them out of Iraq into Syria. )Our strategy so far in dealing with Iran is to have the Security Council's cooperation . For that we need Russia China and France on the same page .

Itsdb rated this answer Excellent or Above Average Answer
ETWolverine rated this answer Excellent or Above Average Answer
jackreade rated this answer Average Answer

Question/Answer
purplewings asked on 06/22/06 - Amnesty USA once again......

AIR TORTURE - you know what it is.

But not enough people living in the United States do. They don't realize that the U.S. government is hiding the truth and transporting people to countries like Syria, Jordan and Egypt, where they are tortured in our names.

Like you, most people oppose torture. And it is critical that we get their attention right now when we have an opportunity to end this practice for good. Air Torture is the stark and shocking campaign we need to focus attention on this critical issue. You can help us get the word out. Your gift will allow us to publish a full-page ad in the New York Times that will increase public awareness around this issue. Click here.

Right now, our country's values rest on a terrifyingly slippery slope.

Just last month, Senator Dianne Feinstein (D-Calif.) asked the new CIA director nominee, Michael Hayden, whether "waterboarding" - simulating drowning by strapping a person to a board, tipping the board, and pouring water over a cloth on his mouth and nose or submersing him in water-is an acceptable interrogation
technique. General Hayden responded, "Let me defer that to a closed session, and I would be happy to discuss it in some detail."

People living in the United States must take action now to spread the truth about extraordinary rendition. Click here to help us spread our message about grounding Air Torture.

Here's the plan. With your help and the help of thousands of others:

1. Amnesty International will launch a media campaign, including a full-page ad in the New York Times, that will draw public attention to the practice of extraordinary rendition. We will also reach out to journalists and bloggers to spread the word about Air Torture around the country.

2. Amnesty activists will spread our message through protests at airports around the world. Be on the lookout for Air Torture "pilots" and "flight attendants!"

3. Amnesty International will educate Congress and let our leaders know that we demand that they ground Air Torture.
XXXXXXXXXXXXXXXXXXXXXXXXXXXXXXXXXXXXXXXXXX
Now, if our war prisoners are transported to other countries, are their guards American or not? If it is their own countrymen torturing them, is America to blame? Where should we hold these people?

Simply asking about waterboarding, does not mean it actually happens - but if so, is this equal to having a giant rat put into a pillowcase and tied over your head as has been done in interrogation by other countries?

How do these people think we're supposed to find out where our killers are hiding? Do they think all we have to do is ask them?

And I'd like to point out here that once again, the bottom line is about money. Send your donation......

Do these people not recognize we are at war and trying to save people's lives by getting rid of the murderers?

tomder55 answered on 06/22/06:

Renditions fall into that gray area that I am not completely comfortable with and would rather not know about . I am sure there is a rough game being played by rough people to keep us safe from the rough people who would like us dead .

This of course began during the Clintoon reign ( (PDD 39) and the press and Amnesty Int. were conspicuous in their silence back then. I am not sure how often it has happened but I sense it doesn't have much space in our play book even though the Europeans claim we've used it literally thousands of times . I know of one or 2 cases where the Italian courts (issued arrest warrants against 13 American intelligence operatives, charging that they kidnapped a radical Islamic cleric )and Spanish courts have alleged CIA renditions have taken place on their jurisdictions (use of an airport on the island of Mallorca ).

purplewings rated this answer Excellent or Above Average Answer

Question/Answer
paraclete asked on 06/22/06 - I think this might be all bull?

Gas-friendly cows an emission possible to cut global warming

Richard Macey
June 22, 2006

SUCCESS in the search for a bull that eats and burps less is tipped to curb the nation's greenhouse emissions by more than half a million tonnes over the next 25 years.

Scientists from NSW's Department of Primary Industries have been working for the past 15 years to find a way to breed more efficient beef cattle.

When cattle munch a meal, explained one researcher, Robert Herd, much of the food is converted by fermentation into methane gas. "Ten per cent of the energy eaten is just burped off," he said, adding that feed was the biggest cost in raising beef cattle.

His team found that certain cattle can eat significantly less food, and thus belch less gas, but somehow grow as much steak as regular animals.

They also discovered the cattle carry a protein that could be used as a genetic flag to signal the special talent.

After a decade of research, the scientists came up with a blood analysis that has been developed into a commercial test for selecting bulls able to breed the most food-efficient cows and steers.

Although it has been developed to cut farming costs, the scientists now believe the burp-reduced cattle will also help fight global warming, because methane is also a greenhouse gas, many times more powerful than carbon dioxide.

Another member of the research team, Andrew Alford, has calculated that even by the most conservative estimates, the cattle should reduce Australia's methane emissions by 568,100 tonnes over 25 years - a 3 per cent reduction in the gas belched by beef herds.

If methane-efficient cattle were used by all farmers, the reduction could be up to 16 per cent. Mr Alford said that if a carbon trading program was introduced, the reduced belching could even be worth money to farmers - about $2.16 per cow each year.

"The estimated 568,100 tonnes of methane abated over the 25 years could, on current values for carbon trading, imply an annual return of $5 million across the national beef industry."

Australia's livestock is blamed for about 12.3 per cent of national greenhouse emissions. Stopping 568,100 tonnes of methane from entering the atmosphere, the scientists said, is equal to blocking 11.9 million tonnes of carbon dioxide.

The leader of the department's methane research effort, Roger Hegarty, said it may be possible to develop other methane-efficient animals, including sheep.

He said that if a tax was ever levied on greenhouse emissions, curbing livestock belching would be crucial for farmers.

Dr Hegarty estimated 95 per cent of methane from beef cattle was belched. The rest, he said, was "flatulence".

tomder55 answered on 06/22/06:

I thought it was cattle flatulence that was the culprit . See ;it's just like I've been saying ....GMO will save the planet

Erewhon rated this answer Excellent or Above Average Answer
paraclete rated this answer Excellent or Above Average Answer

Question/Answer
paraclete asked on 06/22/06 - Paranoia grips the US?

It's remarkable how paranoid you can be when you see threats at every turn

US gun owners target UN for July 4 conspiracy
Email Print Normal font Large font June 22, 2006 - 8:44AM

Americans mistakenly worried the United Nations is plotting to take away their guns on July 4 -- US Independence Day -- are flooding the world body with angry letters and postcards, the chairman of a UN conference on the illegal small arms trade says.

"I myself have received over 100,000 letters from the US public, criticising me personally, saying, 'You are having this conference on the 4th of July, you are not going to get our guns on that day,"' said Prasad Kariyawasam, Sri Lanka's UN ambassador.

"That is a total misconception as far as we are concerned," Kariyawasam told reporters ahead of the two-week meeting opening on Monday.

For one, July 4 is a holiday at UN headquarters and the world body's staff will be watching a fireworks display from the UN lawn rather than attending any meetings, he said.

For another, the UN conference will look only at illegal arms and "does not in any way address legal possession," a matter left to national governments to regulate rather than the United Nations, he added.

The campaign is largely the work of the US National Rifle Association, whose executive vice president, Wayne LaPierre, warns on an NRA web site of a July 4 plot "to finalise a UN treaty that would strip all citizens of all nations of their right to self-protection."

Kariyawasam said, "The UN conference will not negotiate any treaty to prohibit citizens of any country from possessing firearms or to interfere with the legal trade in small arms and light weapons."

LaPierre, who also uses the site to pitch his new book, "The Global War on Your Guns," asks NRA members to send letters to Kariyawasam and UN Secretary-General Kofi Annan warning that "the American people will never let you take away the rights that our 4th of July holiday represents."

The group also asks members to write to John Bolton, the US ambassador to the United Nations, urging him to "ensure the defeat of this treaty." Bolton's office confirmed he had received tens of thousands of cards from concerned Americans.

"We understand their concerns and will work during the conference to communicate their concerns," Bolton spokesman Richard Grenell said.

At the same time, 1 million people around the world -- symbolising the number of people killed by guns since the last UN small arms conference in 2001 -- have signed a petition backing stronger controls on arms deals in a campaign organised by Oxfam International, Amnesty International and the International Action Network on Small Arms.

The June 26-July 7 UN conference was called to review a 2001 UN action plan aimed at stemming the illegal global trade in small arms, which, as defined by the United Nations, range from pistols and grenades to mortars and shoulder-fired anti-aircraft and anti-tank missiles.

The action program set out broad guidelines for national and global measures to track arms sales, promote better management of government arms stockpiles and encourage the destruction of illicit arms.

Reuters

tomder55 answered on 06/22/06:

I don't own a gun but if I did I would say that the UN can pry my gun from my cold dead fingers. I don't need a gun because people like kindj are covering my back .

Itsdb rated this answer Excellent or Above Average Answer
paraclete rated this answer Excellent or Above Average Answer

Question/Answer
Judgment_Day asked on 06/22/06 - Alright US Citizens And Legal Residents...

Ted Kennedy was screaming his lungs out again and this time it was for a national minimum wage increase. I think it already was shot down by some of his miserly cohorts. I was with Ted on this one all the way. It's been at least nine year since the last federal increase. Too bad it will not happen. In recent posts we addressed tax-cuts, limitations to welfare programs, and reduced funding for such programs as the arts...on balance I believe the one natural resource that's not a just a figure is humans, our families. Some States were so embarrassed by our federal governments lack of response to increase the wage for almost a decade that they chose to increase the minimum wage statewide on their own.

Meanwhile....

WASHINGTON -- Despite record low approval ratings, House lawmakers Tuesday embraced a $3,300 pay raise that will increase their salaries to $168,500.

The 2 percent cost-of-living raise would be the seventh straight for members of the House and Senate.

Lawmakers easily squelched a bid by Rep. Jim Matheson, D-Utah, to get a direct vote to block the COLA, which is automatically awarded unless lawmakers vote to block it.

In the early days of GOP control of Congress, lawmakers routinely denied themselves the annual COLA. Last year, the Senate voted 92-6 to deny the raise but quietly surrendered the position in House-Senate talks.


As part of an ethics reform bill in 1989, Congress gave up their ability to accept pay for speeches and made annual cost-of-living pay increases automatic unless the lawmakers voted otherwise.

The pay issue has been linked to the annual Transportation and Treasury Department spending bill because that measure stipulates that civil servants get raises of 2.7 percent, the same as military personnel will receive. Under a complicated formula, the increase translates to 2 percent for members of Congress.

Like last year, Matheson led a quixotic drive to block the raise. He was the only member to speak on the topic.

"I do not think that it is appropriate to let this bill go through without an up or down vote on whether or not Congress should have an increase in its own pay," Matheson said.

But by a 249-167 vote, the House rejected Matheson's procedural attempt to get a direct vote on the pay raise.

The pay raise would also apply to the vice president - who is president of the Senate - congressional leaders and Supreme Court justices.

This year, Vice President Cheney, House Speaker Dennis Hastert and Chief Justice William Rehnquist receive $212,100. Associate justices receive $203,000. House and Senate party leaders get $183,500.

President George W. Bush's salary of $400,000 is unaffected by the legislation.



tomder55 answered on 06/22/06:

Most areas of the US the minimum wage has been rendered obsolete by market forces. You can't hire anyone in the NY area legally or otherwise at the Federal Minimum wage .You would be laughed at if you tried. Nationwide a hike in the minimum wage would affect about 6 % of the workforce and about 1/2 of these are part timers or student workers .I do not think that an adjustment would have any real impact on poverty .The numbers that Kennedy and cohorts are talking about would not lift your low end workers into any real degree of self sufficiency .(in NY that would require a family of 4 to make around $35,000/yr. )

I personally think that the claim that it would negatively impact business is over stated at least during times of economic prosperity ;but it may actually have a negative impact on unemployment when jobs are harder to find . I think Kennedy and co are thinking small . What would serve the workers better would be to expand training incentives so that these low rung workers could begin to climb the ladder of advancement . Minimum wage laws in my view just make the 1st rung harder to reach .

ETWolverine rated this answer Excellent or Above Average Answer
Judgment_Day rated this answer Above Average Answer

Question/Answer
paraclete asked on 06/21/06 - How do you deal with corruption in Iraq?

You send them a message.

Iraqi minister's guard killed by Aussies


June 22, 2006 - 5:43AM
Iraq's trade minister lashed out at Australia today after escorts guarding an embassy delegation that visited him at his Baghdad office shot dead one of his own guards and wounded several others.

"They are trampling on the dignity and sovereignty of Iraqis," Abdel Falah al-Sudani, a member of parliament's dominant Shi'ite bloc, said on state television.

"We demand an explanation from the Australian government for this intentional and unwarranted criminal aggression against members of our protection force. It should also compensate the family of the martyr and the wounded," Sudani said.

An interior ministry source told AFP gunmen with the Australian delegation had opened fire at the plainclothes bodyguards they encountered as they left the minister's compound in Baghdad's western Harthiya neighbourhood.

"They thought the bodyguards had nothing to do with the Iraqi police forces guarding the compound," the source said.

The Department of Foreign Affairs and Trade in Canberra said the shooting was under investigation.

"We are aware of an incident in Baghdad and it is currently being investigated," ministry spokesman Matt Adamson said in Canberra.

Reuters footage showed the Iraqi bodyguards' sports utility vehicle crashed into a pole, its windscreen peppered with bullet holes.

Earlier, a spokesman for the Iraqi Trade Ministry said Sudani had called for the suspension of trade ties with Australia over the incident.

"Immediately, the minister called on the Iraqi government to suspend all kinds of trade relations with the Australian government until it gives an explanation and pays compensations for the families of the killed and wounded people," Mohamed Hannon said.

It is not known why the delegation was in Baghdad, but Australia has been working hard to win back one of its most important wheat markets after the AWB kickback scandal saw Iraq suspend dealings with monopoly exporter AWB Ltd in February.

A 2005 UN report alleged AWB was one of more than 2,000 firms that had paid kickbacks worth $US1.8 billion ($A2.4 billion) to Saddam Hussein's government through the UN-managed "oil-for-food" account. If proved, AWB would have broken UN sanctions against Iraq.

AFP/AP/Reuters

tomder55 answered on 06/22/06:

just the price that has to be paid for Austrian Eggs and Italian Wax .I think the League of Nations should investigate .

paraclete rated this answer Excellent or Above Average Answer

Question/Answer
ETWolverine asked on 06/22/06 - About those WMDs...

Senator Rick Santorum (R-PA) announced yesterday that the Iraq WMDs that Bush said exist actually do exist.

Santoum cited an unclassified report fom John Negroponte to the House Permanent Select Committee on Intelligence (of which Santorum is a member) which details unclassified information regarding the existence of pre-war Iraqi WMDs.

The report states

Since 2003 Coalition forces have recovered approximately 500 weapons munitions which contain degraded mustard or sarin nerve agent.

Despite many efforts to locate and destroy Iraqs pre-Gulf War chemical munitions, filled and unfilled pre-Gulf War chemical munitions are assessed to still exist.

Pre-Gulf War Iraqi chemical weapons could be sold on the black market. Use of these weapons by terrorists or insurgent groups would have implications for Coalition forces in Iraq. The possibility of use outside Iraq cannot be ruled out.

The most likely munitions remaining are sarin and mustard-filled projectiles.

The purity of the agent inside the munitions depends on many factors, including the manufacturing process, potential additives, and environmental storage conditions. While agents degrade over time, chemical warfare agents remain hazardous and potentially lethal.

It has been reported in open press that insurgents and Iraqi groups desire to acquire and use chemical weapons.

Read the report here.

I have a few questions about this.

1) Why was this not picked up as a major story by the press today? Even the NY Post missed this story. I would think that the confirmation of Bush's assertions regarding the possible existence of Iraqi WMDs would be a major story and the lead-in for most of the media. Why isn't it?

2) Bush was accused of lying when everyone thought the WMDs didn't exist. They didn't accuse him of faulty intelligence or of making a mistake. They said he lied. Now that we know that Bush didn't lie and that WMDs DID exist, does that make those who accused him of lying out to be liars themselves? Or are they just simply mistaken?

3) Will anyone in the media or from the Democratic party apologize to Bush for calling him a liar? Or do they only demand apologies from Republicans?

This is definitely not a good month to be a Democrat.

- Al Zarqawi killed.
- Hundreds of arrests and killings of insurgents and insurgent leaders.
- Iraqi government takes office.
- Republicans pull the rug out from under the Dems feet on a vote for pulling out of Iraq.
- Bush's poll numbers are showing significant improvement.
- And now, some of the WMDs have been found.

Some people just can't catch a break. They are called "Democrats".

Elliot

tomder55 answered on 06/22/06:

I was going to post this if no one else did . Would add that on Tues. nights John Batchelor show he and John Loftus were discussing that former inspector Dave Gaubatz had interviewed with the NY Times and that they were going to publish something significant about the WMD he thinks is in a flooded storage facility in Nasaryia. Evidently his report of this finding went to Centcom and was loaded into their computer in Saudia Arabia . Then the Saudis evicited us and in the process of moving the report was erased . He did get confirmation however that satellites show the cites have not been touched and are oly awaiting someone to drain and dig them up . Supposedly the US has a renewed interest in the sites .

As for the Santorum revelation the press is quick to point out that this is old news or discredited news or bogus news ;take your pick . Their big thing is that these are old weapons .DUH ? No one ever said that Saddam was manufacturing fresh stock ;just that he still had stockpiles left over that was hidden from inspectors . Alan Colmes last night dismissed the finding because the weapons were probably degraded . That would be true of biological weapons but what the shells contained are Sarin and mustard gas . I find this claim by him rediculous and a quick search reveals that although Sarin does degrade mustard gas is quite stable .


Artillery shells filled with Mustard 33. Iraq declared that 550 shells filled with mustard had been "lost" shortly after the Gulf War. To date, no evidence of the missing munitions has been found. Iraq claimed that the chemical warfare agents filled into these weapons would be degraded a long time ago and, therefore, there would be no need for their accounting. However, a dozen mustard-filled shells were recovered at a former CW storage facility in the period 1997-1998. The chemical sampling of these munitions, in April 1998, revealed that the mustard was still of the highest quality. After seven years, the purity of mustard ranged between 94 and 97%. Thus, Iraq has to account for these munitions which would be ready for combat use. The resolution of this specific issue would also increase confidence in accepting Iraqs other declarations on losses of chemical weapons which it has not been possible to verify.

What Santorum was citing last night was recently declassified documents so it is insulting for the MSM and the critics to harp on this being old news .

Rep. Peter Hoekstra had the best line to Colmes : If theyre so confident theyre degraded, maybe we can drop it off at a liberals house, see if theyll store it in their garage.


ETWolverine rated this answer Excellent or Above Average Answer
Itsdb rated this answer Excellent or Above Average Answer

Question/Answer
Itsdb asked on 06/21/06 - Another setback...

Yesterday I commented that the sacrifice that Pfc. Tucker and Pfc. Menchaca made would be exploited. This morning's headline story telling of their barbaric murder warned, "The discovery of the bodies dealt a new setback to U.S. efforts to seize the momentum against al-Qaida in Iraq after killing its leader, Abu Musab al-Zarqawi, in a June 7 airstrike."

Dealt a new setback, eh? What about other news from yesterday that wasn't in my paper this morning?

    COALITION FORCES DETAIN SENIOR AL-QAIDA IN IRAQ NETWORK MEMBER

    Release Date: 6/20/2006
    Release Number: 06-06-02PE
    Description: BAGHDAD Coalition forces detained a senior al-Qaida in Iraq network member and three suspected terrorists during coordinated raids southwest of Baqubah June 19.

    The terrorist is reportedly a senior al-Qaida cell leader throughout central Iraq, north of Baghdad. Hes known to be involved in facilitating foreign terrorists throughout central Iraq, and is suspected of having ties to previous attacks on Coalition and Iraqi forces.

    Coalition forces secured multiple buildings and detained the known terrorist plus three suspected terrorists without incident. Troops found an AK-47 with several magazines of ammunition and destroyed them all on site.

    Several women and children were present at the raid sites. None were harmed and all were returned to their homes once the troops ensured the area was secure.


    COALITION FORCES KILL 15 TERRORISTS, DETAIN THREE OTEHRS

    Release Date: 6/20/2006
    Release Number: 06-06-02PE
    Description: BAGHDAD Coalition forces killed 15 terrorists and detained three other suspects during simultaneous raids north of Baqubah June 20.

    Coalition forces came under immediate small arms fire from a rooftop upon arrival to the objective area. The ground force returned fire, killing nine armed terrorists on the rooftop, and an additional two armed terrorists who were identified firing on Coalition forces from next to the building, were killed by Coalition aircraft supporting fire.

    Following this initial contact, Coalition forces found 10 AK-47 assault rifles, one shotgun, one pistol and a crate of explosives.

    One supporting aircraft hit utility wires as they were engaging the armed terrorists. The aircraft was damaged and forced to make a controlled landing. There were no injuries to the crew and the ground force immediately secured the site.

    Three armed suspects were then killed by another Coalition aircraft as they attempted to attack the downed aircraft.

    After securing the aircraft, Coalition forces moved to assault the building that several terrorists had fled to following the first contact. One terrorist was killed by a Coalition sniper as he attempted to engage the troops from the nearby rooftop.

    The force cleared the buildings, detaining the three terrorists. The captured individuals, who fled earlier, were found hiding amidst nine women. None of the women were injured. One of the detained terrorists was wounded at the initial target building after he engaged Coalition forces.

    The raids were targeting individuals associated with a suspected senior al-Qaida in Iraq network member targeted in previous Coalition operations.

    The downed aircraft was recovered prior to all forces returning to base, and all weapons and explosives were destroyed on-site.


No doubt the loss of these two heroic men was tragic, but it's pathetic that our own media has to exploit their deaths as a "setback" while our troops are over there kicking some terrorist a** in return.

Steve

tomder55 answered on 06/22/06:

I think we had about 8000 troops on the seach and rescue mission who are now free to do some search and destroy .


My brother told me that according to the Marine manual , there may come a time that you will be killed from your own weapon but a marine should make sure that he is only beaten to death with it ;the chamber should be emptied first .

Mark Levin's monologue on Tues was a must hear and fortunately he published it on his blog :

Its time to reorient our thinking toward victory. In fact, its time to not only celebrate the Greatest Generation, as the media has characterized the World War II generation, but to emulate it in many respects.

For the past few years, weve watched the Senate debate, the Congress adopt, and then the president sign legislation that would confer constitutional rights on unlawful enemy combatants captured on the battlefield and detained at Guantanamo Bay, while we watch as our Marines are accused of war crimes at Haditha without the benefit of any due-process rights. (And notice, not a word from John McCain, Lindsey Graham, or Chuck Hagel.) Weve watched as self-labeled human-rights groups have demanded that the Geneva Conventions be applied to terrorists, even though theyre applicable only to those who honor the rules of war. Weve watched as the ACLU and Julius and Ethel Rosenbergs offspring have brought lawsuits before activist courts challenging the conditions of the detained terrorists. Weve watched as virtually every intelligence gathering technique is attacked as a civil-liberties and constitutional violation, from the Patriot Act and the NSA intercept program to data-mining and interrogating the enemy. Weve watched as the Supreme Court and now lower federal courts have intervened in the presidents constitutional commander-in-chief duties, substituting their policy preferences for his despite their lack of information, experience, or competence. Weve watched as the media have used every opportunity to undermine our war effort with flat-out false reporting (the phony story about flushing of a Koran down a toilet at Gitmo), the exploitation of Abu Ghraib (with overkill coverage), the promotion of irresponsible antiwar voices (such as Cindy Sheehan and Michael Berg), and the support of antiwar politicians like John Murtha (who went from a relative unknown to an overnight media sensation because of his shrill and irresponsible antiwar allegations). And weve watched as the media have splashed some of our nations most important war secrets across their front pages, and then give themselves awards for aiding and abetting the enemy.

No branch of government is acting as they acted during World War II. Rather than undermining the presidents leadership, Congress should be looking for ways to contribute to the winning this war. Theyre few and far between. Rather than intervening in war-making decisions, the courts should acknowledge their limitations during war as past courts have. And even the administration appears tentative about using more military might to destroy this enemy, a hesitation which rarely entered FDR or Trumans thinking. And, of course, whereas the media in the 1940s focused their reportage on the evil that was the enemy, much of todays media view George Bush as the real problem.

Meanwhile, two kidnapped U.S. soldiers were apparently brutally tortured and murdered today. And the question I pose to those who rightly honor the Greatest Generation is this: What would our country have done 60 years ago in response to this war crime? How would our political and military leadership have acted? By all accounts, they would have demanded severe retaliation and retribution. And by that I dont mean bringing the perpetrators to justice, as if were talking about some law-enforcement response to a white-collar crime. No, Im talking about a military response of such devastation that the enemy fears the consequences of future kidnappings and executions of our men and women in uniform. And thats whats missing in this war the enemy does not fear us (at least not enough) and defeatism (rather than victory) is being preached from Capitol Hill and the news and editorial pages.


Itsdb rated this answer Excellent or Above Average Answer

Question/Answer
Itsdb asked on 06/21/06 - On Iraq, Kerry Again Leaves Democrats Fuming

From today's NY Times...

By KATE ZERNIKE

WASHINGTON, June 20 - When Senator John Kerry was their presidential nominee in 2004, Democrats fervently wished he would express himself firmly about the Iraq war.

Mr. Kerry has found his resolve. But it has not made his fellow Democrats any happier. They fear the latest evolution of Mr. Kerry's views on Iraq may now complicate their hopes of taking back a majority in Congress in 2006.

As the Senate prepared for what promises to be a sharp debate starting on Wednesday about whether to begin pulling troops from Iraq, the Democratic leadership wants its members to rally behind a proposal that calls for some troops to move out by the end of this year but does not set a fixed date for complete withdrawal. Mr. Kerry has insisted on setting a date, for American combat troops to pull out in 12 months, saying anything less is too cautious.

In drawing up a schedule for the Wednesday session, the Democratic leadership has arranged for its plan to be debated first, pushing Mr. Kerry and his proposal into the evening, too late for the nightly television news, to starve it of some attention.

Senate Democrats have been loath to express their opinions publicly, determined to emphasize a united front. But interviews suggest a frustration with Mr. Kerry, never popular among the caucus, and still unpopular among many Democrats for failing to defeat a president they considered vulnerable. Privately, some of his Democratic peers complain that he is too focused on the next presidential campaign...

...Mr. Kerry's insistence on pushing ahead with his own plan has left the Democrats divided, and open to renewed Republican accusations that they are indecisive and weak - the same ridicule that Republicans heaped on Mr. Kerry in 2004, when his "I was for it before I was against it" statement about a vote on money for the war became a punch line...

...Some Democrats felt Mr. Kerry allowed Republicans to embarrass them in a vote last week, when the Republicans embraced Mr. Kerry's proposal, certain it would be defeated and allow them to declare themselves the party of unity and strength...


In a telephone interview on Tuesday, Mr. Kerry characterized his statement as the position of strength, and said Democrats had made a mistake not to take a firm stand.

"The Democrats need to be strong and stand up with a clear articulation about how we make the United States stronger," he said. "As far as I'm concerned, we should go right at Karl Rove and his phony tough talk that is calculated purely for the election and not for a successful strategy in Iraq..."

...Senator Charles E. Schumer of New York, chairman of the Democrats' effort to take control of the Senate, minimized the effect the vote would have in November.

"The public and the voters are looking at how George Bush handles Iraq," he said. "They know that he got us in there, they know he's the commander in chief, and they don't believe he has figured out a strategy that will show light at the end of the tunnel. That is the overriding issue in the election, far more than what Democrats are doing."

~~~~~~~~~~~~~~~~~~~~~~~~~~~~~~~~~~~~~~~~~~~~~~~~~~~~~~

The rest can be read here.

So much for Democrat unity. We have the moonbat wing that's not happy with Hillary, and Kerry - who has been running for president for 2 years - has those in power fuming at him. Another indication that Al Gore will be the man in 2008? Could this be part of a plan by some to have Al ride in on his white horse to save the day?

tomder55 answered on 06/22/06:

as I've said before Gore has history in his favor .

Caught some of Kerry's act on C-Span last night . He had an amendment that would fix a set date for withdrawal (ANOTHER flip flop ). It is of course lunacy and first Joe Lieberman and then John Warner took him to task over it . He was even using that Murtha non-sense "over the rainbow horizon " which from what I gather now means Okinawa ;even though the Japs are kicking us off the island .

Itsdb rated this answer Excellent or Above Average Answer

Question/Answer
Judgment_Day asked on 06/21/06 - AMERICANS THAT WORK FOR A LIVING....

...AND that includes working legally and earning an honest crime-free living. OK I think most people want the personal immediate relief of tax-cuts. However beyond tax relief discussions (which is a moot point for me personally since I've never received much if any relief from any of the presidents including Kennedy, LBJ, Nixon, Ford, Carter, Reagan, Bush Sr., Clinton or GW)....I would like to propose that our United States should stop promoting hand-outs. I'm a proponent for having a president that would fight for cutting back on some programs and tightening qualifications. I would especially like to see time limits placed on how long an individual could benefit from the welfare system. Besides the hand-outs to our own society we should also consider all the money allocated via budget spending that will eventually come out of our pockets for rebuilding Iraq and other foreign infrastructure.


*I do think we should have funds raised (even if by taxation) and placed in budget for disaster relief, etc... So just to be clarify I believe some programs are good and necessary.




Comments?

tomder55 answered on 06/22/06:

I agree with what you say about domestic spending . I think even for entitlements the future is means testing (hate to say it because a significant sum of my money has already gone to SS. but since they refuse to reform the flawed program I think we are going to be left with the bill ) .

I generally agree with what you say about foreign aid .Disaster relief is imperative as you point out ( wish we would get some credit for it once in a while ) and aid to specific nations should be contingent on need and if it serves our foreign policy interests . However ,maybe you should reconsider the issue of aid to Iraq and keep in mind the Marshall plan as a model .

At the end of WWII there was no certainly that Western Europe wouldn't fall into the sphere of the Communist .France and Italy had large communist parties ;and by 1947 both Greece and Turkey were threatened by expanding communist insurgencies. It was largely American assistance that tipped the balance and kept these nations in the Western sphere . In 1948 the Marshall Plan was introduced (imagine that ...3 years after the war there was still chaos and no one was asking us to redeploy over the horizon ).It was a major financial commitment to rebuild Europe (and thuse create new markets for all the jobs the returning GIs would fill). We also through NATO committed to the defense of Western Europe .

I think a simular commitment rationally applied in the M.E. would help in the transformation we seek in the region (and if that don't work there is always the glass desert option )

Judgment_Day rated this answer Excellent or Above Average Answer

Question/Answer
ETWolverine asked on 06/21/06 - Article on the US economy.

The Wonder of Voodoo Economics
Bush 43 gets it.

By Rich Lowry

Who says you cant cut taxes, increase spending, and reduce the federal budget deficit all at the same time? Thats what the Bush administration has managed to do. Two decades after then-presidential candidate George H.W. Bush characterized Ronald Reagans idea that tax cuts would spur revenue-generating economic growth as voodoo economics, the witch doctor is again at work.

When President Bush pledged in 2004 to cut the deficit in half by 2009, critics guffawed. The Boston Globe headlined a story, Bushs plan to halve federal deficit seen as unlikely; higher spending, lower taxes dont mix, analysts say. Fanciful, laughable and all spin, said the critics.

Well, it turns out that 2009 might be coming early this year. The 2004 deficit had been projected to hit $521 billion, or 4.5 percent of gross domestic product. Bushs goal was to cut it to 2.25 percent of GDP by 2009not exactly as stirring a national goal as putting a man on the moon, but one that was nonetheless pronounced unattainable. This year, the deficit could go as low as $300 billion, right around the 2009 goal of 2.5 percent of GDP.

The key to the reduction is revenue growth, which has been stoked by economic growth. Government revenues are up 12.9 percent in the first eight months of this year over the same eight-month period last yearwithout any tax increases. When individuals, investors, and corporations have more cash in a growing economy, they send more to the federal government in tax payments.

This, despiteor, more accurately, because ofa couple of rounds of Bush tax cuts that were supposed to have been fiscally ruinous. The Bush tax reductions played some role in the economic expansion and therefore are responsible, partly, for the increased revenues. This doesnt mean that tax cuts pay for themselves, as their most fervent advocates say. But they certainly can offset some of their own cost.

In 1999, the Congressional Budget Office was projecting 2006 total federal revenues of nearly $2.4 trillion, prior to anyone foreseeing Bushs tax cuts. This year, revenue could go as high as nearly $2.4 trillion, even after those tax cuts. In January 2003, prior to Bushs second round of tax cuts in that year, the CBO guessed revenues would be close to $2.4 trillion this yearagain, in the ballpark of where they could be this year.

According to Brian Riedl of the Heritage Foundation, if annual spending increases in the Bush years had been limited to the rate of the Clinton years, roughly 3.3 percent, there would be a federal surplus now. Instead, spending has been growing at 8 percent a year. That demonstrates that the formula for deficit reduction from the 1990smoderate-spending restraint coupled with higher-than-expected growth-generated revenueswould work again today, if only someone could manage the moderate-spending restraint.

Another similarity from the 1990s is that the revenue surge is driven by high-end earners and corporations. Liberals always rue it when the rich get richer, but when they dont, the federal fisc tends to be ruined because they are the ones who pay most of the taxes. The deficit climbed unexpectedly in the early Bush years and is declining unexpectedly now, not because the projections for economic growth were wildly off, but because the kind of people who pay the most taxes took a bath early in the decade and are recovering now. Almost 47 percent of income taxes are paid by those making more than $200,000 a year, and they are thriving again. A chunk of the current revenue surge is also from corporate income taxes, which are up 30 percent over last year.

There are limits to voodoo. Todays fiscal improvements will be overwhelmed by the exploding costs of entitlements just over the horizon. In light of that, we should be maintaining a high-growth, low-tax economy to reap all the benefits of growth, but dutifully restraining entitlements. Thats not sorcery, but just good sense.

Rich Lowry is author of Legacy: Paying the Price for the Clinton Years.
(c) 2006 King Features Syndicate.

-----------------

Hmmmm... tax cuts paying for themselves through increased economic activity that generates higher tax revenue from lower tax rates? Budget deficits that are reduced through tax cuts? Where have I heard that before?

Nah, it would never work... just like it didn't work for Reagan... except that it did. Twice, now.

Elliot

tomder55 answered on 06/21/06:

It also worked in the 1960s when Kennedy's tax cuts were implemented (posthumously in Feb.1964 ...by 1966 revenues grew over 17%). Thrice.

"Budget deficits are not caused by wild-eyed spenders, but by slow economic growth and periodic recession.... in short, it is a paradoxical truth that tax-rates are too high today and tax revenues are too low, and the soundest way to raise the revenues in the long run is to cut the rates."

John F. Kennedy, speech to the Economic Club of New York. December 14, 1962.


Just imagine what would happen if the Republicans were not spending like drunken demoncrats ;Back in Black ! The Republicans are not moving fast enough on earmark reform and that continues to make them vulnerable .

Lowery is right about entitlements . Imaging if Bush were able to move Congress to reform Social Security and relieve us of that and other future unfunded liabilities .

ETWolverine rated this answer Excellent or Above Average Answer

Question/Answer
paraclete asked on 06/21/06 - The Empire declared?

In a grotesque parody of the plot in Star Wars, John Howard has moved to impose his imperial stamp on government in Australia. The "I am the Senate" syndrome is evident in these moves.


Howard pulls Senate into line
From:
By Steve Lewis

June 21, 2006

John Howard / File
Mr Howard ... 'The changes won't entrench government power.'

EVERY Senate committee will have a Coalition chairman under an audacious Howard Government plan to centralise executive power.

The Government's proposal will gut the Senate inquiry structure, allowing it to avoid embarrassing scrutiny of legislation and political controversies such as the children overboard scandal.

Announced last night by Finance Minister Nick Minchin, the plan represents the most assertive use of the Government's Senate majority since it gained control a year ago.

Opposition Leader Kim Beazley said today the Government was damaging democracy with the plan.

"The very fabric of our democracy, by all these measures, is being continually undermined," Mr Beazley told the Nine Network.

"When we were in office (we) existed with a situation of a high level of Senate accountability," Mr Beazley said.

Opposition senators described the senate plan as "arrogant and high-handed", complaining that it would further sideline the Senate as a house of review.

Under the changes, the Government will have to first approve any matter or piece of legislation being referred to Senate committees.

The Government will also be able to ensure every chairman is from the Coalition and can stop embarrassing witnesses from appearing before Senate inquiries.

In a letter sent to party leaders, Senator Minchin said the changes were designed to "achieve greater efficiencies and effectiveness".

This would be done by collapsing the present 16 committees into a system of eight or 10.

But while a number of committees at present are chaired by non-Coalition senators, the Government now wants to head each of the new committees, reflecting its one-seat majority in the Senate.

John Howard has previously said the Government would be "modest, even humble" in how it used its Senate majority, which was unexpectedly gained at the 2004 election.

Last night, the Prime Minister rejected claims it would entrench government power and allow the Coalition to avoid scrutiny.

But Labor's Senate leader Chris Evans said the move was a "nail in the coffin of the Senate's capacity to hold this Government to account".

The changes to the committee structure meant the "end to the powerful system that brought revelations about children overboard and failings in military justice", he said.

Greens leader Bob Brown said the changes meant all committees would be "controlled from the Prime Minister's office" and that Parliament was "being sidelined by executive power".

Mr Beazlaey also today attacked the Government's proposed changes to electoral laws under which the electoral roll close would close on the day an election was called.


"It (the Government) is chopping kids out of a chance to vote at the next election," Mr Beazley said

tomder55 answered on 06/21/06:

I thought that the reason political parties try to achieve a majority is to use it to advance their agenda. Maybe in Australia it is different but here ;when Democrats are the majority they control the agenda as do the Republicans . My suggestion to the minority parties would be ... try to win an election . There must be a reason that Howard keeps winning that has more to do with the people's preferences more than some trumped up belly aching about procedural changes .

ETWolverine rated this answer Excellent or Above Average Answer
paraclete rated this answer Excellent or Above Average Answer

Question/Answer
purplewings asked on 06/21/06 - Questions and responses with Amnesty Int. Psychologist

I just found this on the web. It's the announcement I received in my email inviting questions in regards to torture and abuse, if anyone wants to read it.

http://www.amnestyusa.org/askamnesty/live/display.php?topic=65

tomder55 answered on 06/21/06:

booo hooo ; they get a shrink answering questions about the psycological effect of the terroists detention ? I see they cherry picked the questions and none that disputes their logic is present .

ETWolverine rated this answer Excellent or Above Average Answer
purplewings rated this answer Excellent or Above Average Answer

Question/Answer
jackreade asked on 06/19/06 - 2 American Prisoners of War

Al Quaeda is now holding 2 American soldiers prisoners of war. I wonder what will happen to them? I'm thinking that the barbarians will decapitate them on Arab television because of what happened to Zarqawi last week.

What are the rules for handling prisoners of war?

tomder55 answered on 06/20/06:

They were killed .

There are no rules ....only America need comply with any written rules ;no one else . This was true in Korea ;Vietnam throughout the Middle East wherever Americans have been captured . Yeah ;we are so much better... but we will perish from the hands of Mordred .

ETWolverine rated this answer Excellent or Above Average Answer
jackreade rated this answer Above Average Answer

Question/Answer
Itsdb asked on 06/19/06 - Bush the environmentalist

BushCo wants you to know: Caring about the environment is for pinko terrorist idiots

Bush is the worst environmental president in the nation's history. Period. The proofs are irrefutable, and the list of his administration's sinister assaults on the pale blue dot we all call home is painful and tragic and punishable in the afterlife by seven billion years of listening to Lynne Cheney being scraped across a chalkboard.

No natural resource has been left unmolested: From forest management to air quality to water pollution to emissions standards to land management to industrial farming to reduced controls on heavy polluters to global warming to nuclear waste to our energy policy, BushCo has made atrociously efficient progress in decimating, in just three short years, 30 years of staunch environmental protections.

Dubya, by way of his industry cronies, has initiating more than 200 major rollbacks of America's most significant environmental laws. Hey, it's nothing new: As governor of Texas, Bush made his state No. 1 in thick smudgy black air and water pollution that makes babies gag and eyes water and cancer cells flourish. He had the ugliest enviro record of any governor in the state's history.

~~~~~~~~~~~~~~~~~~~~~~~~~~~~~~~~~~~~~~~~~~~~~~~~~~~~~~

Sound familiar? Last week tomder cited a NY Times op-ed praising Bush for designating the Northwestern Hawaiian Islands a national monument. I noted that I hadn't seen anything in our local paper on this. They finally reported on it today. More praise for Bush? Ha! From the title to the last paragraph, nothing but skepticism. Newest national monument is scenic, but few can visit read the headline in our paper.

    New National Monument a Wonder Few Can See
    By TARA GODVIN , 06.18.2006, 02:17 PM

    The newest national monument boasts crystalline blue water, unspoiled islands with white sand beaches and vast reefs teeming with marine life, including 7,000 species found nowhere else on Earth.

    But unlike Mount Rushmore or the Statue of Liberty, the Northwestern Hawaiian Islands will never be a place visitors can see by just packing the kids into the car for a week.

    And Hawaiians themselves are unsure how much access they will have.

    Remoteness is one factor, as the islands are scattered across 1,400 miles of the Pacific. No public flights have landed at the sole airport, on Midway Atoll, since 2002, and cruise ships make only occasional stops.

    Federal authorities also have long put strict limits on who can set foot in the area to protect its endangered monk seals, nesting green sea turtles and other rare species, along with some 14 million nesting seabirds.

    President Bush created the vast marine sanctuary last week.

    "It is a place to maintain biodiversity and to maintain basically the nurseries of the Pacific," said Conrad C. Lautenbacher, head of the National Oceanic and Atmospheric Administration, which will manage nearly all of the protected area.

    The few people familiar with the riches held by the string of islands - Hawaiians who revere the area, researchers and a handful of fishermen - are waiting to find out how the area's new status will affect their access to the area.

    William Aila, who has been fighting to protect the area since 1986, was pleased the president provided the maximum protection for the area.

    "For Hawaiians, it's really a reconnection and taking responsibility for these islands to the north of us, what we consider our elder islands," he said.

    Aila is a Hawaiian activist, fisherman and Democratic candidate for governor, as well as a member of the Coral Reef Ecosystem Reserve Council, which had been advising federal authorities in what had been a multiyear process to make the islands a marine sanctuary.

    He said commercial fishing doesn't belong in the islands and the president was right to put a five-year phase-out on the eight or nine permits in effect for the area.

    But Aila said he is concerned about what traditional Hawaiian fishermen will be allowed to do in the area. According to the president's proclamation: "Any monument resource harvested from the monument will be consumed in the monument."

    Hawaiian oral histories tell of a long tradition of bringing fish back from the islands to share with family, he said, and feathers molted annually by the red-tailed tropicbird are needed to restore historic Hawaiian capes held in the Bishop Museum in Honolulu.

    Kitty Simonds, executive director of the Western Pacific Regional Fishery Management Council, said her organization is concerned about the commercial fishing phase-out.

    "It looks good to the rest of the world. But as far as I'm concerned, it was an easy declaration because no one lives there," she said. "So all you have are the few fishermen who would like to continue their livelihoods there.

    "What happened to the American dream?" Simonds said.

    Scientists also want to know more about the declaration's effects.

    "We don't know the details of what this designation will mean for the research. But we're hopeful that we'll be able to continue a robust research project up there," said Malia Rivera, just returned last weekend from a three-week research trip to the islands with the University of Hawaii's Hawaii Institute of Marine Biology.


So that's the best we can get out of the AP on a grand, historic environmental move by Bush? How might this have read had say, Al Gore made this move?

tomder55 answered on 06/19/06:

gee remoteness heh ? That never stops them from bemoaning the possibilty of drilling in ANWAR ; a national site that for most will never be a place visitors can see .

I'd say this has been a good month for Hawaii . First the Akaka Bill stalls in the Senate ;then the stealth USS Hawaii nuke attack sub was commissioned ,and now this . Yes ,a very good month .

Itsdb rated this answer Excellent or Above Average Answer

Question/Answer
paraclete asked on 06/18/06 - It's enough to make a president paranoid(if he wasn't already?)

Julian Borger
June 18, 2006

Are world leaders trying to tell George Bush something, asks Julian Borger in Washington.

A BRAIDED leather whip, a sniper rifle, six jars of fertiliser and a copy of The Worst-Case Scenario Survival Handbook were among the presents foreign leaders have given George Bush. They are clearly trying to tell him something.

The inventory of official gifts from 2004, published by the State Department, reads like the wish list of the sort of paranoid survivalist who holes up in his log cabin to await Armageddon.

The President received a startling array of weapons, including daggers and a machete from Gabon. The braided whip was from the Hungarian Prime Minister.

The Worst-Case Scenario Survival Handbook, a gift from the Sultan of Brunei, has tips on how to use some of these weapons in a tight spot. It also explains how to wrestle an alligator, escape from a mountain lion and take a punch.

But the small arsenal of guns presented by Jordan's King Abdullah, including a $10,000 sniper rifle, would presumably render much of that advice unnecessary. The king also gave Mr Bush six jars of "various fertilisers" on a rotating wooden stand.

According to the Jordanian embassy, the jars contained neither manure nor the sort of chemicals that can be turned into bombs but rather an array of volcanic soils found around the country.

In each instance listed by the State Department, acceptance of the gift is justified by the phrase "non-acceptance would cause embarrassment to donor and US Government".

But acceptance clearly has its own embarrassments. Mr Bush received a vocabulary-expanding game called Forgotten English, from Brunei.

Meanwhile, it is hard to imagine Donald Rumsfeld summoning much enthusiasm for an aromatherapy gift set from the Jordanians. That gift was passed on to the government department that disposes of unwanted presents.

If the top members of the Bush Administration met to compare gifts, Donald Rumsfeld would no doubt have been looking enviously over the President's shoulder at some of his weapons, or at the special edition of The Art of War Dick Cheney got from the Chinese Vice-President.

But Mr Cheney also received presents for his fun-loving side: a "Happy Day" clock from the Swiss President, gold silk pillows, scented candles and a pottery incense burner (the Jordanians again).

It is apparent that a lot of the foreign dignitaries do not do much research before buying gifts. Mr Bush, a reformed drunk, was given a cellarful of wine in 2004.

Officials are only allowed to keep gifts worth under $US305 ($A413) after they leave office. Others are consigned to libraries or archives, where they are occasionally displayed to show the America's warm ties with the rest of the world.

Despite standing shoulder to shoulder in 2004, Mr Bush got nothing from Tony Blair, for Christmas or his birthday.

GUARDIAN

tomder55 answered on 06/19/06:

I would've thought Julian Borger must be the new reporter in town ;why else would he get such a lame assignment to comment on gifts Bush gets ? But reviewing his body of work it is revealed that he is just another Bush hating hack . The Guardian is full of them . The Smoking Gun provides the complete list published by the Federal Register and the list Borger cherry-picks is just a small number of the gifts . The US is of course open enough to reveal such details . I wonder what John Howard has stashed ?

paraclete rated this answer Excellent or Above Average Answer

Question/Answer
paraclete asked on 06/17/06 - If you live by the sea it's time to move, by the way I did years ago

Thawing icy plains a threat as rot sets in

Janet Wilson in Los Angeles
June 17, 2006

ANCIENT woolly mammoth bones and grasslands locked in a 1 million-square-kilometre stretch of Siberian permafrost are starting to thaw, with the potential to unleash billions of tonnes of carbon and accelerate global warming, Russian and American scientists have concluded.

"It's like taking food out of your freezer leave it on your counter for a few days and it rots," said a University of Florida botany professor, Ted Schuur, describing the process in which bacteria convert decaying animal and plant matter in the soil into carbon dioxide, methane and other harmful greenhouse gases.

The study, published in yesterday's issue of the journal Science, concluded that while other global warming researchers were factoring carbon reserves in the ocean, and in current soils and vegetation on the earth into their calculations, they had overlooked vast amounts of carbon trapped in permafrost in the northern plains of Siberia and central Alaska.

If all the permafrost thawed and was released as heat-trapping carbon dioxide, it could nearly double the 730 billion tonnes of carbon now in the atmosphere, the scientists said.

What was most surprising, they said, was the size and depth of the terrain that they found could be affected, at an average of 25 metres deep and containing about 500 billion tonnes of carbon.

"It's like finding a new continent under the earth," said the lead author, Sergey Zimov. He said the vast, carbon-rich area had been buried over many millenniums by a unique layer of windborne "loess" dust that covered bones of mammoth, bison, sabre-toothed tiger and abundant grasses they fed on, then froze about 10,000 years ago into permafrost.

The research team also found that carbon stored over tens of thousands of years could bubble up from thawed soil in as little as 100 years.

Los Angeles Times

tomder55 answered on 06/17/06:

run for the hills !!!! it's the attack of the wooly mammoth bones !!! sounds like some cheap 1950s Sci Fi movie . I am getting a bumper sticker for my car : CLIMATE CHANGE HAPPENS . (It will replace my old and peeling sitcker :CLINTON HAPPENS .)

Millions of years ago, when the Arctic was warm and wet and tundra didn't exist, lush forests stretched all the way to the North Pole. In fact many archeologists are going to the Arctic to examine the well preserved remnants of the forest that used to be there .

The 'Palaeocene/Eocene Thermal Maximum' occured 55 million years ago . The Earth released the equivalent of a giant fart into the atmosphere consisting of green house gasses. That warmed the earth by 5C .The surface waters of the Arctic Ocean were ice-free and as warm as 18C .But the sudden increase in greenhouse gasses boosted them to 24C and the waters suddenly filled with a tropical algae .The water changed from salty to fresh, and the ocean became covered with a thick layer of freshwater fern, called Azolla. This growth of fern, is linked to the later drops in temperature in the area. This has been confirmed from data collected from core samples taken in the area .

All this happened before humans were emitting Co2 into the atmosphere.You would agree that this is part of the natural cycle the planet goes through ;right ?


Itsdb rated this answer Excellent or Above Average Answer
paraclete rated this answer Excellent or Above Average Answer

Question/Answer
paraclete asked on 06/17/06 - What happened to the dream of a liberal democracy

George Washington:

As Mankind becomes more liberal, they will be more apt to allow that all those who conduct themselves as worthy members of the community are equally entitled to the protections of civil government. I hope ever to see America among the foremost nations of justice and liberality.

tomder55 answered on 06/17/06:

America is among the foremost nations of justice and liberality . Note there is nothing in his quote advocating a liberal democracy ;especially as it is defined today as a nanny state .

In Washington's time they compromised and said that for census electoral purposes a black was 3/5th a person ;they were slaves and had no right to vote . Women were disenfranchised also.We have gone through civil rights movements and civil rights are mainstream in this country . When looked at in that content ,Washington's wish is being fulfilled in this country . All those who conduct themselves as worthy members of the community are equally entitled to the protections of civil government .Tell me who isn't .

paraclete rated this answer Excellent or Above Average Answer
purplewings rated this answer Excellent or Above Average Answer

Question/Answer
Itsdb asked on 06/16/06 - Let the hand wringing begin

House passes politically charged Iraq resolution

By Vicki Allen

WASHINGTON (Reuters) - In a vote charged with election-year politics, the U.S. House of Representatives on Friday passed a symbolic resolution that wrapped the Iraq conflict into the war on terrorism and rejected a deadline for U.S. troop withdrawal.

The House voted 256-153 for the resolution that sparked two days of emotional debate as Republicans sought to depict Democrats as weak on terrorism while Democrats decried President George W. Bush's policies that they said led to chaos in Iraq and detracted from the fight against al Qaeda.

"Will we fight or will we retreat? That's the question that's posed to us," said House Majority Leader John Boehner, an Ohio Republican. "Defeating repressive radical terrorists and their allies is our defining task of the 21st century."

But in impassioned debate, Rep. John Murtha, a Pennsylvania Democrat, erupted in anger at Republicans who talked about continuing the fight in Iraq.

Murtha, a Vietnam veteran and defense hawk who rocked the Congress last year when he turned against the war, said it was "easy to stay in an air-conditioned office and say I'm going to stay the course." He added, "That's why I get so upset when they stand here sanctimoniously and say we're fighting this thing. It's the troops that are doing the fighting."

Reuters 2006. All Rights Reserved.

~~~~~~~~~~~~~~~~~~~~~~~~~~~~~~~~~~~~~~~~~~~~~~~~~~~~~~

Senate Rejects U.S. Troop Pullout in Iraq

By LIZ SIDOTI

WASHINGTON - Congress plunged into divisive election-year debate on the Iraq war Thursday as the U.S. military death toll reached 2,500. The Senate soundly rejected a call to withdraw combat troops by year's end, and House Republicans laid the groundwork for their own vote.

In a move Democrats criticized as gamesmanship, Senate Republicans brought up the withdrawal measure and quickly dispatched it for now on a 93-6 vote.

The proposal would have allowed "only forces that are critical to completing the mission of standing up Iraqi security forces" to remain in Iraq in 2007.

~~~~~~~~~~~~~~~~~~~~~~~~~~~~~~~~~~~~~~~~~~~~~~~~~~~~~~

Comments?

tomder55 answered on 06/16/06:

Here are the 6 moonbats in the Senate

Kerry ,Boxer ,Byrd ,Feingold ,Harkin ,Kennedy

as if that's a suprise .

This is funny because Kerry was pontificating saying that he was going to craft legislation that would have a US pull-out by the end of 2006 . But Mitch McConnell beat him to the punch .

The House vote was more Party line but none of them are going to be major players in the 2008 election. The rest of the Senate Democrat Presidential hopefulls are still sitting on the fence ;at least Hillary is out front in being opposed to a time-table .

Hey Murtha ! Take that vote "over the horizon"!! I wonder how prominent Murtha's name was on Zarqawi's Thumb-drive ?

ETWolverine rated this answer Excellent or Above Average Answer
Itsdb rated this answer Excellent or Above Average Answer

Question/Answer
ETWolverine asked on 06/16/06 - Looking for information

Does anyone have a website that shows the new document (translated) that was released by the Iraqi government that details the problems al Qaeda is having? The one that was found at Zarqawi's (not so) safe house? The NY Post reported on the document today and Ralph Peters had a nice op-ed piece on it. I was hoping to get a copy of the document for my records so that I could reference it later.

Elliot

tomder55 answered on 06/16/06:

Got it thanks to Centcom

.........................................

Page 1 of 4

A glance at the reality of Baghdad in light of the latest events (sectarian turmoil)

It has been proven that the Shiites have a power and influence in Baghdad that cannot be taken lightly, particularly when the power of the Ministries of Interior and Defense is given to them, compared with the power of the mujahidin in Baghdad. During a military confrontation, they will be in a better position because they represent the power of the state along with the power of the popular militias. Most of the mujahidin power lies in surprise attacks (hit and run) or setting up explosive charges and booby traps. This is a different matter than a battle with organized forces that possess machinery and suitable communications networks. Thus, what is fixed in the minds of the Shiite and Sunni population is that the Shiites are stronger in Baghdad and closer to controlling it while the mujahidin (who represent the backbone of the Sunni people) are not considered more than a daily annoyance to the Shiite government. The only power the mujahidin have is what they have already demonstrated in hunting down drifted patrols and taking sniper shots at those patrol members who stray far from their patrols, or planting booby traps among the citizens and hiding among them in the hope that the explosions will injure an American or members of the government. In other words, these activities could be understood as hitting the scared and the hiding ones, which is an image that requires a concerted effort to change, as well as Allahs wisdom.
The strength of the brothers in Baghdad is built mainly on booby trapped cars, and most of the mujahidin groups in Baghdad are generally groups of assassin without any organized military capabilities.
There is a clear absence of organization among the groups of the brothers in Baghdad, whether at the leadership level in Baghdad, the brigade leaders, or their groups therein. Coordination among them is very difficult, which appears clearly when the group undertake a join operations
The policy followed by the brothers in Baghdad is a media oriented policy without a clear comprehensive plan to capture an area or an enemy center. Other word, the significance of the strategy of their work is to show in the media that the American and the government do not control the situation and there is resistance against them. This policy dragged us to the type of operations that are attracted to the media, and we go to the streets from time to time for more possible noisy operations which follow the same direction.
This direction has large positive effects; however, being preoccupied with it alone delays more important operations such as taking control of some areas, preserving it and assuming power in Baghdad (for example, taking control of a university, a hospital, or a Sunni religious site).



Page 2 of 4

At the same time, the Americans and the Government were able to absorb our painful blows, sustain them, compensate their losses with new replacements, and follow strategic plans which allowed them in the past few years to take control of Baghdad as well as other areas one after the other. That is why every year is worse than the previous year as far as the Mujahidins control and influence over Baghdad.

The role that the Islamic party and the Islamic Scholars Committee play in numbing the Sunni people through the media is a dangerous role. It has been proven from the course of the events that the American investment in the Party and the Committee were not in vain. In spite of the gravity of the events, they were able to calm down the Sunni people, justify the enemy deeds, and give the enemy the opportunity to do more work without any recourse and supervision. This situation stemmed from two matters:
n First, their media power is presented by their special radio and TV stations as the sole Sunni information source, coupled with our weak media which is confined mainly to the Internet, without a flyer or newspaper to present these events.

n Second, in the course of their control of the majority of the speakers at mosques who convert right into wrong and wrong into right, and present Islam in a sinful manner and sins in a Muslim manner. At the same time we did not have any positive impact or benefits from our operations.

The mujahidin do not have any stored weapons and ammunition in their possession in Baghdad, particularly rockets, such as C5K Katyosha or bomber or mortars which we realized their importance and shortage in Baghdad. That was due to lack of check and balance, and proper follow-ups.

The National Guard status is frequently raised and whether they belong to the Sunnis or Shiites. Too much talk is around whether we belong to them or not, or should we strike and kill their men or not?
It is believed that this matter serves the Americans very well. I believe that the Committee and the Party are pushing this issue because they want to have an influence, similar to the Mujahidins. When and if a Sunni units from the National Guard are formed, and begin to compete with the mujahidin and squeeze them, we will have a problem; we either let them go beyond the limits or fight them and risk inciting the Sunnis against us through the Partys and the Committees channels.



Page 3 of 4

I believe that we should not allow this situation to exist at all, and we should bury it before it surfaces and reject any suggestion to that effect.

(Salah), the military commander of Baghdad (he used to be the commander of the Rassafah County and still is) is a courageous young man with a good determination but he has little and simple experience in the military field and does not have a clear vision about the current stage and how to deal with it Most of his work at al-Rassafah County is to take cars to the Jubur Arab Tribes, convert them into booby traps and take them back inside Baghdad for explosion. And the more booby trap cars he makes, the more success he has. This alone is not a work plan and we do not benefit from it in the medium range let alone the long range.


(Salah): The current commander of Northern al-Karkh (Abu-Huda) is very concerned because of his deteriorating security situation caused by being pursued by the Americans, since they have his picture and voice print. Therefore, his movement is very restricted and he is unable to do anything here. We should remove him from Baghdad to a location where he can work easier; otherwise he is closer to become totally ineffective. I know nothing about his past military experience or organizational skills.

(Salah): Northern al-Karkh groups are estimated at 40 mujahid, so is the Southern Karkh. They could double that number if necessary. Al-Rassafah groups in general is estimated at 30 mujahidin as I was informed by the commander of al-Rassafah. These are very small numbers compared to the tens of thousands of the enemy troops. How can we increase these numbers?
Page 4 of 4

Blank

End of Document/Translation

ETWolverine rated this answer Excellent or Above Average Answer

Question/Answer
ROLCAM asked on 06/16/06 - Where do you stand?

International pressure to close Camp Delta and either release or try the detainees in a court of law is mounting. National protection versus the rights of individuals - where do you stand?

tomder55 answered on 06/16/06:

is the same international pressure being applied to the Chinese to close the death camps that house the Falun Gong ?

or


are we too busy forming business partnerships ?

Itsdb rated this answer Excellent or Above Average Answer
purplewings rated this answer Excellent or Above Average Answer
ROLCAM rated this answer Excellent or Above Average Answer

Question/Answer
Itsdb asked on 06/15/06 - Arctic dips as global waters rise

Arctic sea level has been falling by a little over 2mm a year - a movement that sets the region against the global trend of rising waters.

A Dutch-UK team made the discovery after analysing radar altimetry data gathered by Europe's ERS-2 satellite.

It is well known that the world's oceans do not share a uniform height; but even so, the scientists are somewhat puzzled by their results.

Global sea level is expected to keep on climbing as the Earth's climate warms.

To find the Arctic out of step, even temporarily, emphasises the great need for more research in the region, the team says.

"We have high confidence in the results; it's now down to the geophysics community to explain them," said Dr Remko Scharroo, from consultants Altimetrics LLC, who led the study.

Next year has been designated International Polar Year, and major oceanographic expeditions are planned to take research vessels into the northern region to sample its icy waters.

"This may provide clues as to what is causing the changes we're seeing," explained co-researcher Dr Seymour Laxon, from University College London (UCL). "I think it's a true statement to say the Arctic Ocean is the least well understood body of water out there."

The recent trend could be linked to changes in the temperature and salinity (saltiness) of Arctic waters. This would have to be investigated, he said.

~~~~~~~~~~~~~~~~~~~~~~~~~~~~~~~~~~~~~~~~~~~~~~~~~~~~~~

Ok you smart people, help them out here.

tomder55 answered on 06/16/06:

there used to be this land bridge across what is now the Bearing Sea . This enabled the American Indian to migrate from Asia to the Western Hemisphere. Somehow the land bridge disappeared due to rising sea levels . What human activity caused this ? Was it the smoke from the teepees emiting carbon into the atmosphere ? Was it the deforestation by the natives ? I quote : Over the course of history, humans have often destroyed forests. Settlers often burned trees down to have land to plant crops, and the Native Americans burned forests to develop grasslands full of large game animals.

This is human activity that predates industrialization by centuries . I humbly submit that the continued deforestion of places like the Amazon delta is having a much greater impact than the burning of fossil fuels .

What does one of the most brilliant minds on the planet say about it ? Well Stephen Hawkings suggests that we just move . I would think however that if we were to colonize other planets in a large scale basis we would most probably need to learn how to manipulate the environment to make it habitable to a large number of humans . I believe that there is no better laboratory to work in than the one that God gave us ;this planet .

But what do I know ? I am not Stephen Hawkings .

ETWolverine rated this answer Excellent or Above Average Answer
Itsdb rated this answer Excellent or Above Average Answer
paraclete rated this answer Excellent or Above Average Answer

Question/Answer
kindj asked on 06/15/06 - I'll stir the pot a little more...

I realize that this may very well give a certain person an aneurysm, but I'll proceed anyway. Read at your own risk.

So during a staff meeting yesterday, it was announced by the boss that I would be leaving the agency (something he and I had already been over--this was merely the "public announcement"). He told everyone that my last day would be July 14th, to which I added, "Which is Bastille Day, of course." Little tounge-in-cheek jab that I knew no one would get. My boss, a Hispanic, asked with a sneer, "What the hell is Bastille Day?" I explained about the storming of the Bastille prison in France that housed political prisoners and was sort of a symbol of the oppressive monarchy of France, and how that event kicked off the French Revolution. I added that for me, though, it was a good day to justify buying an expensive bottle of Beaujolais, which I love.

His response to me was, "Why the hell would we care about some French holiday? This ain't France, you know."

To which I replied, "It ain't Mexico, either, but we sure do a lot of celebrating on the 5th of May and the 16th of September."

Him: "Well, there isn't any French people in America."

Me: "Go to Southeast Texas, Louisiana, Mississippi, Alabama, Florida, Maine, Minnisota, etc. and say that. There's French aplenty there."

Him: "Well, not as many as there are Mexicans, so why should I care?"

Me: "You should care because of the nature of the US Constitution, which ensures that a minority shall not be oppressed by the majority. Which is the same part of the Constitution you fall back on when defending LULAC or the National Council on La Raza."

Him: "Well, no one is denegrating the French people in America."

Me: "You just did, when you snubbed a very important holiday for them."

He turned red with rage and walked off.

Don't know if I'll actually make it to the 14th or not, but damn was that satisfying, to use his own arguments against him.

No question, just another story from your local rabble-rouser, who still likes to wear his IDF t-shirt whenever he's near the University.

DK

tomder55 answered on 06/15/06:

Isn't Sep. 16 Mexican Independence Day . Celebrating their rebellion against Joseph Bonaparte ;Napolean's( ruler of FRANCE )brother ? Isn't Cinco de Mayo the day they smashed the FRENCH army ?

Maybe you will get an extended vacation before the new teaching gig.

Itsdb rated this answer Excellent or Above Average Answer
kindj rated this answer Excellent or Above Average Answer

Question/Answer
Itsdb asked on 06/14/06 - Horror of horrors!

This just can't be, I am shocked and OUTRAGED!

Court asked to quash Canada law

Three Arab men have asked the Canadian Supreme Court to overturn measures that allow foreign-born terror suspects to be held for years without charge.

The men argue that detaining them indefinitely or deporting them back to their homelands to face possible torture violates the constitution.

Canadian intelligence services accuse the three of having ties to al-Qaeda.

The case comes amid an intense security debate in Canada after 17 people were held over an alleged bomb plot.

The three-day hearing began on Tuesday with the men's lawyers asking the Supreme Court to quash the special measures because they circumvent normal judicial process and are thus unconstitutional.

They also challenged the conditions of the men's detention and the government's refusal to disclose the evidence against them.

"Cases should be heard fully and publicly by an independent and impartial court," said Joanne Doyon, who represents one of the men, Adil Charkaoui.

"I am asking for the same rights as any Canadian, as any human being," said Mr Charkaoui, who together with Mohamed Harkat and Hassan Almrei is challenging the legislation.

"They cage us like animals...threaten us with deportation to countries where we would face torture and certain death."

National security

The men have been accused of having links to Osama Bin Laden's al-Qaeda network.

The government says Mr Charkaoui trained at an al-Qaeda camp in Afghanistan. It says Mr Almrei arrived on a false passport and belonged to a forgery ring linked to a terror group, while a lower court judge has found that Mr Harkat lied about having ties to al-Qaeda.

The men say they are innocent and say they fear being tortured or killed if forced to return to their home countries, Algeria, Morocco and Syria.

The Canadian government decided that while the men challenged their deportation in the courts, it was too great a risk to allow them to be at liberty in Canada. They were, however, free to leave Canada at any time.

The security certificate programme has been enshrined in Canadian law since 1978.

It allows the federal government to detain or deport immigrants without charge and without providing evidence to their lawyers.


The government says a certificate is only issued when there is a need to use sensitive information that needs to be protected for reasons of national security or for the safety of any person.

Since the 11 September 2001 attacks in the US, the measures have been used to jail five terror suspects, including the three appellants.

Mr Charkaoui was subsequently released on strict bail conditions.

Federal lawyers are arguing that the security measures and the exceptional secrecy in these cases are essential to safeguard intelligence sources.

The Supreme Court is expected to take several months to issue a ruling.



Adil Charkaoui walks past a number of protest signs outside the Ottawa court

~~~~~~~~~~~~~~~~~~~~~~~~~~~~~~~~~~~~~~~~~~~~~~~~~~~~~~

Call Human Rights Watch! Call the ACLU! Call Amnesty International! Call the UN! Call Ramsey Clark! How dare Canada hold terror suspects "without charge and without providing evidence to their lawyers"! Who do they think they are, the Bush administration?

tomder55 answered on 06/15/06:

see this way with the law no one can complain the due process is not being confired on them . I guess habeus corpus is not such a big concern North of the border . am beginning to appreciate Canada better lately ;especially since Stephen Harper became PM .

Itsdb rated this answer Excellent or Above Average Answer
purplewings rated this answer Excellent or Above Average Answer

Question/Answer
jackreade asked on 06/15/06 - GOP Ignoring Hillary Opponent

ALBANY, N.Y. (AP) - John Spencer is bitterly attacking the national Republican Party for not rallying behind his bid to oust Sen. Hillary Rodham Clinton, complaining the situation reminds him of the combat patrols he went on in Vietnam.

"In Vietnam, one of the most frightening duties any soldier had was walking point. You could hit booby traps or ambushes, but you always knew your buddies were behind you," the former Yonkers mayor says in a radio advertisement previewed Wednesday for The Associated Press.

"Running for U.S. Senate against Hillary Clinton is a lot like walking point. You find out who your buddies are real fast."

Spencer is trailing badly in the polls and in fundraising, and the national Republican Party has offered no financial help. He says in the ad that the national GOP is "afraid" to back him.

In a direct challenge to the party, Spencer says: "Don't send your checks to Washington. Get behind me."

The 60-second spot is to begin airing Thursday.

Brian Nick, a spokesman for the GOP National Senatorial Campaign Committee, declined to comment on the ad.

~~~~~~~~~~~~~~~~~~~~~~~~~~~~~~~~~~~~~~~~~~~~~~~

Does this mean that the Republicans consider Hillary unbeatable?

tomder55 answered on 06/15/06:

In NY she is . I am suprised that Spencer did not realize that he was being put out as so much fodder because the Republicans had to put some one ....anyone on the ballot. If she was beatable then Rudy ,Pataki ;or even Pirro would've taken her on . Pirro decided that she had a better shot at going after Spitzer's vacated Att Gen . slot (Spitzer will be the next Govenor .The Republicans are barely contesting that slot also). Pataki and Rudy have ideas about 2008 and the rest of the NY Republican org. is so weak that they were not even considered . Shumer won reelection by over 70% and Hillary will probably top that .

Clinton has raised more than $40 million since she was elected and will spend it freely ;not so much to beat Spencer ,but to nationalize her campaign. The Kossacks can boo and hiss her all they want to but by the time the 2008 primaries roll around she will be in a position to bury them with campaign spending . Only Gore's or Kerry's private monies will be able to counter her .

Spencer was Mayor of Yonkers and anyone from NY knows that Yonkers is a problem city and he was a practitioner of nepotism . Plus his marital problems are at least the equal of the Clintoons. He has never denied that while he was mayor, he had a long affair with his chief of staff while he was married, and had two children with her before they were eventually married . I would've probably supported his Republican opponent KT McFarland but she decided to bow out of the race this week .

Spencer will likely run an attack dog campaign against Hillary . His attack on the National Republcan party covers his eventual defeat but it also distances him from the National Republicans and that I guess he figures is a good thing.

ETWolverine rated this answer Excellent or Above Average Answer
jackreade rated this answer Excellent or Above Average Answer

Question/Answer
purplewings asked on 06/14/06 - Need help? There is/was a free lunch for some.

Reading this article almost made me sick to my stomach as I wonder how I will pay for my car repair of this morning and my home insurance due later this month along with the rest of my necessities. I'm trying to help my son get through college and my work week was cut down to 4 days, while the pension fund from GM notified me they had made a calculation error over the past year and would be taking the money back over a 6 month period......this knocks my check from $700 to $100 a month. Isn't there a government group to help me? :(

http://articles.news.aol.com/news/article.adp?id=20060613200009990001&ncid=NWS00010000000001


FEMA debit cards were sometimes used to pay for vacations, season football tickets and even a sex change operation, according to an audit.

Prison inmates, a supposed victim who used a New Orleans cemetery for a home address, and a person who spent 70 days at a Hawaiian hotel all were able to wrongly get taxpayer help, according to evidence that gives a new black eye to the nation's disaster relief agency.

Federal investigators even informed Congress that one man apparently used FEMA assistance money for a sex change operation.

Agents from the Government Accountability Office, the investigative arm of Congress, went undercover to expose the ease of receiving disaster expense checks from the Federal Emergency Management Agency.

The GAO concluded that as much as 16 percent of the billions of dollars in FEMA help to individuals after the two hurricanes was unwarranted.

The findings are detailed in testimony, obtained by The Associated Press, that is to be delivered at a hearing Wednesday by the House Homeland Security subcommittee on investigations.

To dramatize the problem, GAO provided lawmakers with a copy of a $2,358 U.S. Treasury check for rental assistance that an undercover agent got using a bogus address. The money was paid even after FEMA learned from its inspector that the undercover applicant did not live at the address.

"This is an assault on the American taxpayer," said Rep. Michael McCaul, R-Texas, chairman of the subcommittee that will conduct the hearing."Prosecutors
from the federal level down should be looking at prosecuting these crimes and putting the criminals who committed them in jail for a long time."

FEMA spokesman Aaron Walker said Tuesday that the agency, already criticized for a poor response to Katrina, makes its highest priority during a disaster
"to get help quickly to those in desperate need of our assistance."

"Even as we put victims first, we take very seriously our responsibility to be outstanding stewards of taxpayer dollars, and we are careful to make sure that funds are distributed appropriately," he said.

FEMA said it has identified more than 1,500 cases of potential fraud after Katrina and Rita and has referred those cases to the Homeland Security inspector
general. The agency said it has identified $16.8 million in improperly awarded disaster relief money and has started efforts to collect the money.

The GAO said it was 95 percent confident that improper and potentially fraudulent payments were much higher - between $600 million and $1.4 billion.

The investigative agency said it found people lodged in hotels often were paid twice, since FEMA gave them individual rental assistance and paid hotels directly. FEMA paid California hotels $8,000 to house one individual - the same person who received three rental assistance payments for both disasters.

In another instance, FEMA paid an individual $2,358 in rental assistance, while at the same time paying about $8,000 for the same person to stay 70 nights at more than $100 per night in a Hawaii hotel.

FEMA also could not establish that 750 debit cards worth $1.5 million even went to Katrina victims, the auditors said.

How do you feel about this? Is there any recourse?

tomder55 answered on 06/14/06:

Sorry to say that this is just a representative sample of the type of abuse that happens as government expands and entitlements grow . This will of coure be attributed to Bush mismanagement but it is an endemic problem no matter who is in charge . When someone comes up to you and says 'I'm from the government, and I'm here to help.' ,run the other way .

If you go to a local food pantry you will notice that they give out food to the needy. They never give out cash for the needy to buy groceries . Wonder why ? Because they know there is no guarantee that the cash dolled out will be used for it's intended purpose .

Blame the abuser .In this case I almost have to give FEMA a pass . The money was thrown out in a hurried and haphazard way because of all of the idiots who believed that disaster response was the sole responsibility of FEMA and the Federal Gvt. The pressure that built up, as the media hysterically reported(Geraldo Rivera :someone do something !!!!!), forced them to put out the money as quickly as they could with no regard to accounting.FEMA was never supposed to be first responders ;they were supposed to assist locals who had the primary responsibilites .

purplewings rated this answer Excellent or Above Average Answer

Question/Answer
ETWolverine asked on 06/14/06 - That's twice.

This is the second time that Bush has managed to sneak to Iraq for a face-to-face with the people on the ground there. This is the second time that he has managed to pull off this sort of coup and thumb his nose at the terrorists.

This has been a really bad month for the left-wing mainstream media. First, the Iraqi government finally took office. Then Zarqawi was killed. Since then, there have been a number of under-reported but high-profile captures or killings of terrorist leaders based on information gathered from Zarqawi's hideout. And finally, Bush pulls off his SECOND sneak trip to Iraq. All of these facts have managed to knock the MSM's "main" story of the Haditha "massacre" right off the front page.

Boy, it must suck being an MSM talking head these days. Don't they just hate it when things go right for Bush?

Elliot

tomder55 answered on 06/14/06:

don't forget Bilbray's victory in the Cal. 50th district where the dems learned that their 'culture of corruption 'campaign will get them nowhere and that the immigration issue has some teeth for the Republicans (if they don't blow it ) .

In contrast the Dems have been sucking up to the moonbat bloggers ; and this week to the "mainstream" Moonbats ;where KERRY DID ANOTHER FLIP FLOP . "It was wrong and I was wrong to vote for that Iraqi war resolution." ,and Hillary was booed by the faithful .

ETWolverine rated this answer Excellent or Above Average Answer
Itsdb rated this answer Excellent or Above Average Answer

Question/Answer
ETWolverine asked on 06/14/06 - For Jackreade,

From today's NY Post.

MURTHA'S RUSH TO JUDGMENT
By ILARIO PANTANO

June 14, 2006 -- EDITOR'S NOTE: Ilario Pantano wrote the following letter to The Washington Post (which printed it), but we believe it's of interest to New York Post readers, too.


A year ago I was charged with two counts of premeditated murder and with other war crimes related to my service in Iraq. My wife and mother sat in a Camp Lejeune courtroom for five days while prosecutors painted me as a monster; then autopsy evidence blew their case out of the water, and the Marine Corps dropped all charges against me ["Marine Officer Cleared in Killing of Two Iraqis," Washington Post news story, May 27, 2005].

So I know something about rushing to judgment, which is why I am so disturbed by the remarks of Rep. John P. Murtha (D-Pa.) regarding the Haditha incident ["Death Toll Rises in Haditha Attack, GOP Leader Says," news story, May 20]. Mr. Murtha said, "Our troops overreacted because of the pressure on them, and they killed innocent civilians in cold blood."
In the United States, we have a civil and military court system that relies on an investigatory and judicial process to make determinations based on evidence. The system is not served by such grand pronouncements of horror and guilt without the accuser even having read the investigative report.

Mr. Murtha's position is particularly suspect when he is quoted by news services as saying that the strain of deployment "has caused them [the Marines] to crack in situations like this." Not only is he certain of the Marines' guilt but he claims to know the cause, which he conveniently attributes to a policy he opposes.

Members of the U.S. military serving in Iraq need more than Mr. Murtha's pseudo-sympathy. They need leaders to stand with them even in the hardest of times. Let the courts decide if these Marines are guilty. They haven't even been charged with a crime yet, so it is premature to presume their guilt - unless that presumption is tied to a political motive.

Ilario Pantano
Jacksonville, N.C.


And that is why I say that Murtha is anti-soldier and makes his anti-soldier statements based on political ambitions.

tomder55 answered on 06/14/06:

the only way Murtha can call himself pro-military is that he supports defense spending that is earmarked for his district . GO DIANA IREY !!!!!!!

ETWolverine rated this answer Excellent or Above Average Answer

Question/Answer
ETWolverine asked on 06/14/06 - Where is Amnesty International? Where is the ACLU?

Did you know that there are 7 young soldiers currently being held in jail without charges being filed against them? Did you know that they are behind bars fo 23 hours a day in 8x8 cells, and can only see their families through a plexiglass window and wearing hand and leg shackles? These men are the ones being investigated for the Haditha "massacre" which nobody is sure even happened. The investigation is still ongoing.

So where is the ACLU demanding the civil rights of THESE prisoners? Where is Amnesty International to demand that the cruelty against these prisoners be stopped? Where is the outcy from the left of "cuel and unusual punishment" and "torture of prisoners"? Where is John McCain to decry the treatment of these prisoners? Where is John Murtha's alleged support for the military soldiers, rather than his assumption of guilt before the facts are even in?

Oh, I forgot. Those rules don't apply to soldiers or those who support and accomplish GWB's policies. None of that liberal "rights of the accused" stuff for them. That only applies to criminals and terrorists, not soldiers who fight and bleed and die for their country.

Elliot

tomder55 answered on 06/14/06:

Steve referenced a blog site called Sweetness & Light that has been calling Time Mag to the mat over their apparently flawed reporting of the incident .

The Guardian last year reported on the environment that our Marines were operating in. Suffice it to say that if this had been WWII ,we would've fire bombed the whole city and be done with it .We shouldn't ask our troops to enter those places unless we have the will to take out the fighters and subdue those who support them . I have been told of patrols driving through the streets and they see children playing outside ;and when they return ,the children and all the residence have retreated inside their homes . That is when the troops know they will come under attack ;probably from an IED that had been previously placed where the children had been playing . Are we supposed to believe then that the residents are not the enemy also ?

That being said ;and this is where my opinion is torn on this ; the incident happened Nov. of last year ann inspite of the braying MSM ,the military has had a running investigation ongoing for a long time . The military has more information then they are revealing and if they deem it that thezse Marines need to be confined in the manner that they are ....well I sure as hell don't like it ..but I should give the Marine Corp. the benefit of the doubt also .




ETWolverine rated this answer Excellent or Above Average Answer

Question/Answer
Itsdb asked on 06/13/06 - The Real Iraq

Long, but a must read.

By Amir Taheri

Spending time in the United States after a tour of Iraq can be a disorienting experience these days. Within hours of arriving here, as I can attest from a recent visit, one is confronted with an image of Iraq that is unrecognizable. It is created in several overlapping ways: through television footage showing the charred remains of vehicles used in suicide attacks, surrounded by wailing women in black and grim-looking men carrying coffins; by armchair strategists and political gurus predicting further doom or pontificating about how the war should have been fought in the first place; by authors of instant-history books making their rounds to dissect the various fundamental mistakes committed by the Bush administration; and by reporters, cocooned in hotels in Baghdad, explaining the carnage and chaos in the streets as signs of the countrys impending or undeclared civil war. Add to all this the days alleged scandal or revelationan outed CIA operative, a reportedly doctored intelligence report, a leaked pessimistic assessmentand it is no wonder the American public registers disillusion with Iraq and everyone who embroiled the U.S. in its troubles.

It would be hard indeed for the average interested citizen to find out on his own just how grossly this image distorts the realities of present-day Iraq. Part of the problem, faced by even the most well-meaning news organizations, is the difficulty of covering so large and complex a subject; naturally, in such circumstances, sensational items rise to the top. But even ostensibly more objective efforts, like the Brookings Institutions much-cited Iraq Index with its constantly updated array of security, economic, and public-opinion indicators, tell us little about the actual feel of the country on the ground.

To make matters worse, many of the newsmen, pundits, and commentators on whom American viewers and readers rely to describe the situation have been contaminated by the increasing bitterness of American politics. Clearly there are those in the media and the think tanks who wish the Iraq enterprise to end in tragedy, as a just comeuppance for George W. Bush. Others, prompted by noble sentiment, so abhor the idea of war that they would banish it from human discourse before admitting that, in some circumstances, military power can be used in support of a good cause. But whatever the reason, the half-truths and outright misinformation that now function as conventional wisdom have gravely disserved the American people.

For someone like myself who has spent considerable time in Iraqa country I first visited in 1968current reality there is, nevertheless, very different from this conventional wisdom, and so are the prospects for Iraqs future. It helps to know where to look, what sources to trust, and how to evaluate the present moment against the background of Iraqi and Middle Eastern history.

Since my first encounter with Iraq almost 40 years ago, I have relied on several broad measures of social and economic health to assess the countrys condition. Through good times and bad, these signs have proved remarkably accurateas accurate, that is, as is possible in human affairs. For some time now, all have been pointing in an unequivocally positive direction.

The first sign is refugees. When things have been truly desperate in Iraqin 1959, 1969, 1971, 1973, 1980, 1988, and 1990long queues of Iraqis have formed at the Turkish and Iranian frontiers, hoping to escape. In 1973, for example, when Saddam Hussein decided to expel all those whose ancestors had not been Ottoman citizens before Iraqs creation as a state, some 1.2 million Iraqis left their homes in the space of just six weeks. This was not the temporary exile of a small group of middle-class professionals and intellectuals, which is a common enough phenomenon in most Arab countries. Rather, it was a departure en masse, affecting people both in small villages and in big cities, and it was a scene regularly repeated under Saddam Hussein.

Since the toppling of Saddam in 2003, this is one highly damaging image we have not seen on our television setsand we can be sure that we would be seeing it if it were there to be shown. To the contrary, Iraqis, far from fleeing, have been returning home. By the end of 2005, in the most conservative estimate, the number of returnees topped the 1.2-million mark. Many of the camps set up for fleeing Iraqis in Turkey, Iran, and Saudi Arabia since 1959 have now closed down. The oldest such center, at Ashrafiayh in southwest Iran, was formally shut when its last Iraqi guests returned home in 2004.

A second dependable sign likewise concerns human movement, but of a different kind. This is the flow of religious pilgrims to the Shiite shrines in Karbala and Najaf. Whenever things start to go badly in Iraq, this stream is reduced to a trickle and then it dries up completely. From 1991 (when Saddam Hussein massacred Shiites involved in a revolt against him) to 2003, there were scarcely any pilgrims to these cities. Since Saddams fall, they have been flooded with visitors. In 2005, the holy sites received an estimated 12 million pilgrims, making them the most visited spots in the entire Muslim world, ahead of both Mecca and Medina.

Over 3,000 Iraqi clerics have also returned from exile, and Shiite seminaries, which just a few years ago held no more than a few dozen pupils, now boast over 15,000 from 40 different countries. This is because Najaf, the oldest center of Shiite scholarship, is once again able to offer an alternative to Qom, the Iranian holy city where a radical and highly politicized version of Shiism is taught. Those wishing to pursue the study of more traditional and quietist forms of Shiism now go to Iraq where, unlike in Iran, the seminaries are not controlled by the government and its secret police.

A third sign, this one of the hard economic variety, is the value of the Iraqi dinar, especially as compared with the regions other major currencies. In the final years of Saddam Husseins rule, the Iraqi dinar was in free fall; after 1995, it was no longer even traded in Iran and Kuwait. By contrast, the new dinar, introduced early in 2004, is doing well against both the Kuwaiti dinar and the Iranian rial, having risen by 17 percent against the former and by 23 percent against the latter. Although it is still impossible to fix its value against a basket of international currencies, the new Iraqi dinar has done well against the U.S. dollar, increasing in value by almost 18 percent between August 2004 and August 2005. The overwhelming majority of Iraqis, and millions of Iranians and Kuwaitis, now treat it as a safe and solid medium of exchange.

My fourth time-tested sign is the level of activity by small and medium-sized businesses. In the past, whenever things have gone downhill in Iraq, large numbers of such enterprises have simply closed down, with the countrys most capable entrepreneurs decamping to Jordan, Syria, Saudi Arabia, the Persian Gulf states, Turkey, Iran, and even Europe and North America. Since liberation, however, Iraq has witnessed a private-sector boom, especially among small and medium-sized businesses.

According to the International Monetary Fund (IMF) and the World Bank, as well as numerous private studies, the Iraqi economy has been doing better than any other in the region. The countrys gross domestic product rose to almost $90 billion in 2004 (the latest year for which figures are available), more than double the output for 2003, and its real growth rate, as estimated by the IMF, was 52.3 per cent. In that same period, exports increased by more than $3 billion, while the inflation rate fell to 25.4 percent, down from 70 percent in 2002. The unemployment rate was halved, from 60 percent to 30 percent.

Related to this is the level of agricultural activity. Between 1991 and 2003, the countrys farm sector experienced unprecedented decline, in the end leaving almost the entire nation dependent on rations distributed by the United Nations under Oil-for-Food. In the past two years, by contrast, Iraqi agriculture has undergone an equally unprecedented revival. Iraq now exports foodstuffs to neighboring countries, something that has not happened since the 1950s. Much of the upturn is due to smallholders who, shaking off the collectivist system imposed by the Baathists, have retaken control of land that was confiscated decades ago by the state.

Finally, one of the surest indices of the health of Iraqi society has always been its readiness to talk to the outside world. Iraqis are a verbalizing people; when they fall silent, life is incontrovertibly becoming hard for them. There have been times, indeed, when one could find scarcely a single Iraqi, whether in Iraq or abroad, prepared to express an opinion on anything remotely political. This is what Kanan Makiya meant when he described Saddam Husseins regime as a republic of fear.

Today, again by way of dramatic contrast, Iraqis are voluble to a fault. Talk radio, television talk-shows, and Internet blogs are all the rage, while heated debate is the order of the day in shops, tea-houses, bazaars, mosques, offices, and private homes. A catharsis is how Luay Abdulilah, the Iraqi short-story writer and diarist, describes it. This is one way of taking revenge against decades of deadly silence. Moreover, a vast network of independent media has emerged in Iraq, including over 100 privately-owned newspapers and magazines and more than two dozen radio and television stations. To anyone familiar with the state of the media in the Arab world, it is a truism that Iraq today is the place where freedom of expression is most effectively exercised.

That an experienced observer of Iraq with a sense of history can point to so many positive factors in the countrys present condition will not do much, of course, to sway the more determined critics of the U.S. intervention there. They might even agree that the images fed to the American public show only part of the picture, and that the news from Iraq is not uniformly bad. But the root of their opposition runs deeper, to political fundamentals.

Their critique can be summarized in the aphorism that democracy cannot be imposed by force. It is a view that can be found among the more sophisticated elements on the Left and, increasingly, among dissenters on the Right, from Senator Chuck Hagel of Nebraska to the ex-neoconservative Francis Fukuyama. As Senator Hagel puts it, You cannot in my opinion just impose a democratic form of government on a country with no history and no culture and no tradition of democracy.

I would tend to agree. But is Iraq such a place? In point of fact, before the 1958 pro-Soviet military coup detat that established a leftist dictatorship, Iraq did have its modest but nevertheless significant share of democratic history, culture, and tradition. The country came into being through a popular referendum held in 1921. A constitutional monarchy modeled on the United Kingdom, it had a bicameral parliament, several political parties (including the Baath and the Communists), and periodic elections that led to changes of policy and government. At the time, Iraq also enjoyed the freest press in the Arab world, plus the widest space for debate and dissent in the Muslim Middle East.

To be sure, Baghdad in those days was no Westminster, and, as the 1958 coup proved, Iraqi democracy was fragile. But every serious student of contemporary Iraq knows that substantial segments of the population, from all ethnic and religious communities, had more than a taste of the modern worlds democratic aspirations. As evidence, one need only consult the immense literary and artistic production of Iraqis both before and after the 1958 coup. Under successor dictatorial regimes, it is true, the conviction took hold that democratic principles had no future in Iraqa conviction that was responsible in large part for driving almost five million Iraqis, a quarter of the population, into exile between 1958 and 2003, just as the opposite conviction is attracting so many of them and their children back to Iraq today.

A related argument used to condemn Iraqs democratic prospects is that it is an artificial country, one that can be held together only by a dictator. But did any nation-state fall from the heavens wholly made? All are to some extent artificial creations, and the U.S. is preeminently so. The truth is that Iraqone of the 53 founding countries of the United Nationsis older than a majority of that organizations current 198 member states. Within the Arab League, and setting aside Oman and Yemen, none of the 22 members is older. Two-thirds of the 122 countries regarded as democracies by Freedom House came into being after Iraqs appearance on the map.

Critics of the democratic project in Iraq also claim that, because it is a multi-ethnic and multi-confessional state, the country is doomed to despotism, civil war, or disintegration. But the same could be said of virtually all Middle Eastern states, most of which are neither multi-ethnic nor multi-confessional. More important, all Iraqis, regardless of their ethnic, linguistic, and sectarian differences, share a sense of national identityuruqa (Iraqi-ness)that has developed over the past eight decades. A unified, federal state may still come to grief in Iraqhistory is not written in advancebut even should a divorce become inevitable at some point, a democratic Iraq would be in a better position to manage it.

What all of this demonstrates is that, contrary to received opinion, Operation Iraqi Freedom was not an attempt to impose democracy by force. Rather, it was an effort to use force to remove impediments to democratization, primarily by deposing a tyrant who had utterly suppressed a well-established aspect of the countrys identity. It may take years before we know for certain whether or not post-liberation Iraq has definitely chosen democracy. But one thing is certain: without the use of force to remove the Baathist regime, the people of Iraq would not have had the opportunity even to contemplate a democratic future.

Assessing the progress of that democratic project is no simple matter. But, by any reasonable standard, Iraqis have made extraordinary strides. In a series of municipal polls and two general elections in the past three years, up to 70 percent of eligible Iraqis have voted. This new orientation is supported by more than 60 political parties and organizations, the first genuinely free-trade unions in the Arab world, a growing number of professional associations acting independently of the state, and more than 400 nongovernmental organizations representing diverse segments of civil society. A new constitution, written by Iraqis representing the full spectrum of political, ethnic, and religious sensibilities was overwhelmingly approved by the electorate in a referendum last October.

Iraqs new democratic reality is also reflected in the vocabulary of politics used at every level of society. Many new wordsaccountability, transparency, pluralism, dissenthave entered political discourse in Iraq for the first time. More remarkably, perhaps, all parties and personalities currently engaged in the democratic process have committed themselves to the principle that power should be sought, won, and lost only through free and fair elections.

These democratic achievements are especially impressive when set side by side with the declared aims of the enemies of the new Iraq, who have put up a determined fight against it. Since the countrys liberation, the jihadists and residual Baathists have killed an estimated 23,000 Iraqis, mostly civilians, in scores of random attacks and suicide operations. Indirectly, they have caused the death of thousands more, by sabotaging water and electricity services and by provoking sectarian revenge attacks.

But they have failed to translate their talent for mayhem and murder into political success. Their campaign has not succeeded in appreciably slowing down, let alone stopping, the countrys democratization. Indeed, at each step along the way, the jihadists and Baathists have seen their self-declared objectives thwarted.

After the invasion, they tried at first to prevent the formation of a Governing Council, the expression of Iraqs continued existence as a sovereign nation-state. They managed to murder several members of the council, including its president in 2003, but failed to prevent its formation or to keep it from performing its task in the interim period. The next aim of the insurgents was to stop municipal elections. Their message was simple: candidates and voters would be killed. But, once again, they failed: thousands of men and women came forward as candidates and more than 1.5 million Iraqis voted in the localities where elections were held.

The insurgency made similar threats in the lead-up to the first general election, and the result was the same. Despite killing 36 candidates and 148 voters, they failed to derail the balloting, in which the number of voters rose to more than 8 million. Nor could the insurgency prevent the writing of the new democratic constitution, despite a campaign of assassination against its drafters. The text was ready in time and was submitted to and approved by a referendum, exactly as planned. The number of voters rose yet again, to more than 9 million.

What of relations among the Shiites, Sunnis, and Kurdsthe focus of so much attention of late? For almost three years, the insurgency worked hard to keep the Arab Sunni community, which accounts for some 15 percent of the population, out of the political process. But that campaign collapsed when millions of Sunnis turned out to vote in the constitutional referendum and in the second general election, which saw almost 11 million Iraqis go to the polls. As I write, ,b>all political parties representing the Arab Sunni minority have joined the political process and have strong representation in the new parliament. With the convening of that parliament, and the nomination in April of a new prime minister and a three-man presidential council, the way is open for the formation of a broad-based government of national unity to lead Iraq over the next four years.

As for the insurgencys effort to foment sectarian violencea strategy first launched in earnest toward the end of 2005this too has run aground. The hope here was to provoke a full-scale war between the Arab Sunni minority and the Arab Shiites who account for some 60 percent of the population. The new strategy, like the ones previously tried, has certainly produced many deaths. But despite countless cases of sectarian killings by so-called militias, there is still no sign that the Shiites as a whole will acquiesce in the role assigned them by the insurgency and organize a concerted campaign of nationwide retaliation.

Finally, despite the impression created by relentlessly dire reporting in the West, the insurgency has proved unable to shut down essential government services. Hundreds of teachers and schoolchildren have been killed in incidents including the beheading of two teachers in their classrooms this April and horrific suicide attacks against school buses. But by September 2004, most schools across Iraq and virtually all universities were open and functioning. By September 2005, more than 8.5 million Iraqi children and young people were attending school or universityan all-time record in the nations history.

A similar story applies to Iraqs clinics and hospitals. Between October 2003 and January 2006, more than 80 medical doctors and over 400 nurses and medical auxiliaries were murdered by the insurgents. The jihadists also raided several hospitals, killing ordinary patients in their beds. But, once again, they failed in their objectives. By January 2006, all of Iraqs 600 state-owned hospitals and clinics were in full operation, along with dozens of new ones set up by the private sector since liberation.

Another of the insurgencys strategic goals was to bring the Iraqi oil industry to a halt and to disrupt the export of crude. Since July 2003, Iraqs oil infrastructure has been the target of more than 3,000 attacks and attempts at sabotage. But once more the insurgency has failed to achieve its goals. Iraq has resumed its membership in the Organization of Petroleum Exporting Countries (OPEC) and has returned to world markets as a major oil exporter. According to projections, by the end of 2006 it will be producing its full OPEC quota of 2.8 million barrels a day.

The Baathist remnant and its jihadist allies resemble a gambler who wins a heap of chips at a roulette table only to discover that he cannot exchange them for real money at the front desk. The enemies of the new Iraq have succeeded in ruining the lives of tens of thousands of Iraqis, but over the past three years they have advanced their overarching goals, such as they are, very little. Instead, they have been militarily contained and politically defeated again and again, and the beneficiary has been Iraqi democracy.

None of this means that the new Iraq is out of the woods. Far from it. Democratic success still requires a great deal of patience, determination, and luck. The U.S.-led coalition, its allies, and partners have achieved most of their major political objectives, but that achievement remains under threat and could be endangered if the U.S., for whatever reason, should decide to snatch a defeat from the jaws of victory.

The current mandate of the U.S.-led coalition runs out at the end of this year, and it is unlikely that Washington and its allies will want to maintain their military presence at current levels. In the past few months, more than half of the 103 bases used by the coalition have been transferred to the new Iraqi army. The best guess is that the number of U.S. and coalition troops could be cut from 140,000 to 25,000 or 30,000 by the end of 2007.

One might wonder why, if the military mission has been so successful, the U.S. still needs to maintain a military presence in Iraq for at least another two years. There are three reasons for this.

The first is to discourage Iraqs predatory neighbors, notably Iran and Syria, which might wish to pursue their own agendas against the new government in Baghdad. Iran has already revived some claims under the Treaties of Erzerum (1846), according to which Tehran would enjoy a droit de regard over Shiite shrines in Iraq. In Syria, some in that countrys ruling circles have invoked the possibility of annexing the area known as Jazirah, the so-called Sunni triangle, in the name of Arab unity. For its part, Turkey is making noises about the Treaty of Lausanne (1923), which gave it a claim to the oilfields of northern Iraq. All of these pretensions need to be rebuffed.

The second reason for extending Americas military presence is political. The U.S. is acting as an arbiter among Iraqs various ethnic and religious communities and political factions. It is, in a sense, a traffic cop, giving Iraqis a green or red light when and if needed. It is important that the U.S. continue performing this role for the first year or two of the newly elected parliament and government.

Finally, the U.S. and its allies have a key role to play in training and testing Iraqs new army and police. Impressive success has already been achieved in that field. Nevertheless, the new Iraqi army needs at least another year or two before it will have developed adequate logistical capacities and learned to organize and conduct operations involving its various branches.

But will the U.S. stay the course? Many are betting against it. The Baathists and jihadists, their prior efforts to derail Iraqi democracy having come to naught, have now pinned their hopes on creating enough chaos and death to persuade Washington of the futility of its endeavors. In this, they have the tacit support not only of local Arab and Muslim despots rightly fearful of the democratic genie but of all those in the West whose own incessant theme has been the certainty of American failure. Among Bush-haters in the U.S., just as among anti-Americans around the world, predictions of civil war in Iraq, of spreading regional hostilities, and of a revived global terrorism are not about to cease any time soon.

But more sober observers should understand the real balance sheet in Iraq. Democracy is succeeding. Moreover, thanks to its success in Iraq, there are stirrings elsewhere in the region. Beyond the much-publicized electoral concessions wrung from authoritarian rulers in Egypt and Saudi Arabia, there is a new democratic discourse to be heard. Nationalism and pan-Arabism, yesterdays hollow rallying cries, have given way to a big idea of a very different kind. Debate and dissent are in the air where there was none beforea development owing, in significant measure, to the U.S. campaign in Iraq and the brilliant if still checkered Iraqi response.

The stakes, in short, could not be higher. This is all the more reason to celebrate, to build on, and to consolidate what has already been accomplished. Instead of railing against the Bush administration, Americas elites would do better, and incidentally display greater self-respect, to direct their wrath where it properly belongs: at those violent and unrestrained enemies of democracy in Iraq who are, in truth, the enemies of democracy in America as well, and of everything America has ever stood for.

Is Iraq a quagmire, a disaster, a failure? Certainly not; none of the above. Of all the adjectives used by skeptics and critics to describe todays Iraq, the only one that has a ring of truth is messy. Yes, the situation in Iraq today is messy. Births always are. Since when is that a reason to declare a baby unworthy of life?

Amir Taheri, formerly the executive editor of Kayhan, Irans largest daily newspaper, is the author of ten books and a frequent contributor to numerous publications in the Middle East and Europe. His work appears regularly in the New York Post.

~~~~~~~~~~~~~~~~~~~~~~~~~~~~~~~~~~~~~~~~~~~~~~~~~~~~~~~

Are you paying attention Mr. Murtha?

tomder55 answered on 06/13/06:

as you know by how many times I have referenced his article about the last helicopter that I am a regualar reader of Taheri.

I see the US assistance to Iraq continuing past 2008 in some form or another (probably bases like we had/have in Germany ) .Even a Democrat President will have to realize that reality ;if nothing else than the help defend an ally against aggressive neighbors as we still do around the world .If it is not in the form of a UN mandate then it will be in the form of an alliance .

quietist forms of Shiism Is this the form where they preach love and acceptance of Jews and Christians?

Itsdb rated this answer Excellent or Above Average Answer

Question/Answer
Itsdb asked on 06/13/06 - Where is HRW?

Suspects complain of torture

Chris Wattie, Melissa Leong and Tom Blackwell
National Post

Tuesday, June 13, 2006

BRAMPTON - Many of the 17 men accused of plotting a terrorist attack on Toronto claim they are being tortured at the provincial jail where they have been held since their arrest last weekend.

One of the accused interrupted a brief court appearance yesterday by shouting accusations at the justice of the peace hearing the case.

"They tortured us, straight up!" Yasin Mohamed, 24, said during an outburst in Brampton court, holding up both arms, which were manacled to two of the other accused. "These wounds speak for themselves."

There were no visible marks or bruises on Mr. Mohamed's arms or on any of the other 14 men who appeared in court yesterday. Two others accused in the high-profile case are scheduled to appear later this month.

A spokesman for the Maplehurst Correctional Complex, the provincial facility west of Toronto where the accused are being held, said there was no evidence any of the men had been mistreated and none had filed complaints.

Still, the Ministry of Community Safety and Correctional Services is looking into the allegations aired in court yesterday, said Julia Noonan, a ministry spokeswoman.

"Any kind of violence is not tolerated in our institutions, be it staff on inmates or inmates on each other ... and appropriate action is taken if it takes place, including contacting police," Ms. Noonan said.

Justice of the peace Keith Currie has imposed a publication ban on all evidence, court orders or reasons for those orders presented at yesterday's appearance, which set dates for future bail and other hearings for the men.

However, lawyers for the accused said outside court their clients are being subjected to what one called "cruel and unusual punishment," including being held in segregation, being roughly handled by guards, being regularly woken and not having lights in their cells turned off at night.

"Clearly leaving the light on 24 hours a day and waking them up every half hour for the last 10 days constitutes torture,"
said Rocco Galati, lawyer for Ahmad Mustafa Ghany.

David Kolinsky, who represents Zakaria Amara, said his client is locked in a tiny concrete cell with no windows. Food is delivered through a small slit and the lights are kept on at all times, he added.

He said the federal court has previously deemed such treatment as "cruel and unusual punishment" and claimed it is known to cause depression and suicide.

"I don't think there's any basis to keep him in solitary confinement. It's normally a form of punishment for people who misbehave and are violent towards other offenders."

Mr. Kolinsky told a crowd of reporters outside the suburban Brampton courthouse that his client has told him he was abused by a guard at the Maplehurst jail.

He said Mr. Amara was being searched by a guard when he laughed because he was ticklish. According to his client, the guard then pinned him on the ground, drilled his finger into his cheek, flicked him in the eye and said: "Is this funny?" Mr. Kolinsky said.

Mr. Amara's face is still sore and his thumbs are still numb from plastic restraints that were placed on his hands, Mr. Kolinsky added.

Defence lawyers also ,b>complained their clients are given only five minutes to eat their meals or their food is taken away, and they are forced to keep their eyes on the floor and are led around bent over at 90-degree angles by guards.

The accused are being held in segregation cells and are only allowed 20 minutes exercise a day, their lawyers said.

Arif Raza said his client Saad Khalid is not being ill-treated, but has been "collaterally damaged" by hearing the guards "screaming and shouting" at other inmates.

"You hear about in fascist countries where prisoners are deliberately tortured so that others can hear ... so the fear is instilled in the minds of the others," Mr. Raza said outside court.

"I'm quite sure that's not what is happening, but it is possible that the same impact would result on the minds of the other inmates."

Ms. Noonan said dim night lights are kept on in all cells in all of the province's jails for security reasons.

None of the inmates were woken deliberately during the night, but regular patrols through Maplehurst may have inadvertently disturbed their sleep, said the spokeswoman.

She also denied allegations that the inmates were given only five minutes to eat their food, saying the minimum is 30 minutes and they usually get more. They can also buy food from the jail canteen and eat it "at their leisure."

Ms. Noonan did confirm that some of the accused may have been moved around with their torsos leaning forward at a 90-degree angle, calling it a common method to manage high-security inmates wearing leg and arm restraints.

"Sometimes when individuals come into custody, if they've never been in custody before, they may be surprised. It's not like being at home," she said. "If individuals are dealing with severe charges, we have to deal with them accordingly."

Don Ford, a spokesman for the union that represents guards at Maplehurst, also said such high-profile prisoners are generally treated with kid gloves.

"I can't see any circumstances in this case that would lead someone to take justice into their own hands or anything of that sort.... The policy would be completely hands-off," Mr. Ford said.

He said he hopes the ministry investigates the matter promptly and thoroughly.

"We don't need this kind of a cloud hanging over our members. We don't want the public thinking we have a bunch of blood-thirsty vigilantes working at the Maplehurst Correctional Centre."

The men were led into court shackled and handcuffed together in groups of three, four or five at a time. A group of family and friends filled one corner of the courtroom and waved to some of the accused as they were led into the dock.

Police arrested the 17 men, all from the Toronto area, 10 days ago and charged them under federal anti-terrorism legislation with participating in a terrorist group, receiving training at a terrorist facility, recruiting for a terrorist organization, importing weapons and preparing a bomb.

Sources familiar with the case have told the National Post that the group was plotting to build and set off two or more powerful truck bombs in downtown Toronto and to charge into a public place firing weapons into a crowd.

The Post has learned that the group was planning to bomb the Toronto headquarters of the Canadian Security Intelligence Service and the Toronto Stock Exchange, using three tonnes of ammonium nitrate mixed with diesel fuel.

National Post 2006

~~~~~~~~~~~~~~~~~~~~~~~~~~~~~~~~~~~~~~~~~~~~~~~~~~~~~~

Where is the media? Where is the Daily Kos? Where is the UN? Call the Red Cross. I demand a full accounting of the torture of these Canadian insurgents.

tomder55 answered on 06/13/06:

I see they have studied the al -qaeda manual Specifically lesson 8

IF AN INDICTMENT IS ISSUED AND THE TRIAL, BEGINS, THE BROTHER HAS TO PAY ATTENTION TO THE FOLLOWING:

1 . At the beginning of the trial, once more the brothers must insist on proving that torture was inflicted on them by State Security [investigators ]before the judge.

2. Complain [to the court] of mistreatment while in prison.

3. Make arrangements for the brother s defense with the attorney, whether he was retained by the brother s family or court-appointed.

4. The brother has to do his best to know the names of the state security officers, who participated in his torture and mention their names to the judge.[These names may be obtained from brothers who had to deal with those officers in previous cases.]

5. Some brothers may tell and may be lured by the state security investigators to testify against the brothers [i.e. affirmation witness ], either by not keeping them together in the same prison during the trials, or by letting them talk to the media. In this case, they have to be treated gently, and should be offered good advice, good treatment, and pray that God may guide them.

6. During the trial, the court has to be notified of any mistreatment of the brothers inside the prison.

7. It is possible to resort to a hunger strike, but it is a tactic that can either succeed or fail.

8. Take advantage of visits to communicate with brothers outside prison and exchange information that may be helpful to them in their work outside prison [according to what occurred during the investigations]. The importance of mastering the art of hiding messages is self evident here.

-When the brothers are transported from and to the prison [on their way to the court] they should shout Islamic slogans out loud from inside the prison cars to impress upon the people and their family the need to support Islam.

-Inside the prison, the brother should not accept any work that may belittle or demean him or his brothers, such as the cleaning of the prison bathrooms or hallways.

-The brothers should create an Islamic program for themselves inside the prison, as well as recreational and educational ones, etc.

-The brother in prison should be a role model in selflessness. Brothers should also pay attention to each others needs and should help each other and unite vis a vis the prison officers.

-The brothers must take advantage of their presence in prison for obeying and worshipping [God] and memorizing the Qora an, etc. This is in addition to all guidelines and procedures that were contained in the lesson on interrogation and investigation. Lastly, each of us has to understand that we don t achieve victory against our enemies through these actions and security procedures. Rather, victory is achieved by obeying Almighty and Glorious God and because of their many sins. Every brother has to be careful so as not to commit sins and everyone of us has to do his best in obeying Almighty God, Who said in his Holy Book: We will, without doubt. help Our messengers and those who believe (both)in this world s life and the one Day when the Witnesses will stand forth. May God guide us.



To this pure Muslim youth, the believer, the mujahid (fighter) for Gods sake. I present this modest effort as a contribution from me to pave the way that will lead to Almighty God and to establish a caliphate along the lines of the prophet.

The prophet, peace be upon him, said according to what was related by Imam Ahmed: Let the prophecy that God wants be in you, yet God may remove it if He so wills, and then there will be a Caliphate according to the prophets path [instruction], if God so wills it. He will also remove that [the Caliphate] if He so wills, and you will have a disobedient king if God so wills it. Once again, if God so wills, He will remove him [the disobedient king], and you will have an oppressive lung. [Finally], if God so wills, He will remove him [the oppressive king], and you will have a Caliphate according to the prophet s path [instruction]. He then became silent.

THE IMPORTANCE OF TEAM WORK:

1 .Team work is the only translation of Gods command, as well as that of the prophet, to unite and not to disunite. Almighty God says, And hold fast, all together, by the Rope which Allah (stretches out for you)
[bwahaaaahaaa how fitting ], and be not divided among yourselves. In Sahih Muslim, it was reported by Abu Horairah, may Allah look kindly upon him, that the prophet, may Allahs peace and greetings be upon him, said: Allah approves three [things] for you and disapproves three [things]: He approves that you worship him, that you do not disbelieve in Him, and that you hold fast, all together, by the Rope which Allah, and be not divided among yourselves. He disapproves of three: gossip, asking too much [for help], and squandering money.
2. Abandoning team work for individual and haphazard work means disobeying that orders of God and the prophet and falling victim to disunity.

3. Team work is-conducive to cooperation in righteousness and piety.

4. Upholding religion, which God has ordered us by His saying, Uphold religion, will necessarily require an all out confrontation against all our enemies, who want to recreate darkness. In addition, it is imperative to stand against darkness in all arenas: the media, education, [religious] guidance, and counseling, as well as others. This will make it necessary for us to move on numerous fields so as to enable the Islamic movement to confront ignorance and achieve victory against it in the battle to uphold religion. All these vital goals can not be adequately achieved without organized team work. Therefore, team work becomes a necessity, in accordance with the fundamental rule, Duty cannot be accomplished without it, and it is a requirement. This way, team work is achieved through mustering and organizing the ranks, while putting the Amir (the Prince) before them, and the right man in the right place, making plans for action, organizing work, and obtaining facets of power......

.........................................
If you find your opposing team's playbook for a football game you should be able to beat them handily . These clowns were following al -qaeda's play book when they were planning their attack . The way to conduct themselves while in custody is there too .It should not come as a suprise that these tactics are tried . What is silly is how the MSM ;Amnesty International, and Human Rights Watch,enablers buy into this horse excrement .

ETWolverine rated this answer Excellent or Above Average Answer
Itsdb rated this answer Excellent or Above Average Answer
Judgment_Day rated this answer Excellent or Above Average Answer

Question/Answer
jackreade asked on 06/12/06 - Asymetrical Warfare

When a hostage commits suicide, it is called asymetrical warfare. Guantanamo has been home for what, after *five years* of captivity - no charges made - not released, fully amounts to *hostage* status. There are plans to enlarge Gitmo.



Let's not lose out American Democratic values.

tomder55 answered on 06/13/06:

Now you are calling detainees captured on the Afghan battle field hostages ? I declare this week a "it sucks to be you " week for jihadists . I'm supposed to be concerned about their despair ? Probably depressed by the news about their friend Zarqawi. And hoping to share in his 72 Helen Thomas look a likes .

Want to blame anyone for the delay ? Blame the courts . They should've thrown out the Hamdan v. Rumsfeld case a long time ago.(it will be decided this month by the Supreme Court but it took way too long to get there )The only way this case is found for the defendant is if the courts consider the many amicus curiae briefs submitted that sourced foreign laws . Military Tribunals ARE Constitutional .

jackreade rated this answer Average Answer

Question/Answer
Itsdb asked on 06/13/06 - Leak of name shows small-minded motive

By Jim Wright

FORT WORTH - As in the sordid revelations that brought down the Nixon administration in 1974, one of the saddest aspects of the secret White House calls that outed special agent Valerie Plame Wilson is the sheer pettiness of the scheme.

Now that top white House political adviser Karl Rove himself has been identified as the one who spoke to Time magazine, identifying Joseph Wilsons wife as a secret CIA agent
, the stupid venture takes on a new dimension of apparent officialdom.

It wasnt just some crackpot, low-level zealot who took it on himself to embarrass Wilson and his wife. It was deliberate policy! And for what? Why would anyone of Roves level consider it worth risking serious censure for violating an unambiguous law? Just to punish, hurt, disadvantage or discredit two civil servants?

Was it worth it? Was Wilsons professional conclusion that Saddam Hussein never tried to buy yellowcake uranium from Niger so damning that it required this kind of heavy-handed retaliation against Wilsons family?

One other question: Is it even possible that Rove was unaware of the law prohibiting the knowing public disclosure of a covert U.S. agent?


That law is not unclear. Under penalty of up to 10 years in prison and a $50,000 fine, it forbids anyone with authorized access to classified information from intentionally disclosing the identity of covert agents.

Congress passed that law in 1982 after the violent death of an undercover agent whod been fingered to our Cold War opposition by a disgruntled American who maliciously published the agents name and location, along with those of several other secret U.S. operatives.

Nobody thinks the White House would wish any such lamentable fate for Valerie Wilson (or Plame, if you prefer her professional name).

Rove reportedly says he didnt speak her actual name to the Time reporter, referring to her simply as Wilsons wife.

Yet it wasnt just an accidental slip of the tongue that blew this secret agents cover. It was a deliberate plan concocted by a person or persons in the White House to try to embarrass or discredit the former ambassador.

Calls went out (whether from Rove or other functionaries) not only to Time but also to columnist Bob Novak, reporter Judith Miller of The New York Times and who knows what other scoop-hungry journalists, all planting the story that the ambassadors wife was an undercover CIA agent.

Surely the White House wasnt trying to get Valerie Wilson shot. But the leakers did not blanch at destroying her usefulness to the country and thus ruining her career.

Soon after everyone began awakening to the truth that Iraq had no WMDs (our initial justification for the pre-emptive U.S. invasion), the ambassadors official investigation had revealed that Saddams reported nuclear intrigue with Niger was only another exaggerated scare.

This whole flap was, on its face, a rather clumsy effort to sully and discredit the reputations of Wilson and his wife.

And to what end? A blunt warning to others in government that, if they value their careers, theyd better check their independent judgment at the door and get with the program? (When we say there are WMDs, then find WMDs! Are you on the team or not?)

All this is painfully reminiscent of the Nixon days. The Enemies List. The tawdry break-ins at the Democratic headquarters, and at a private psychiatrists office to scoop up personal files of a man who opposed the Vietnam War. The use of federal agencies - the FBI and the IRS - to hound and harass private citizens who disagreed with the administration. No, this one leaking incident doesnt rise to that level. Some say, Tempest in a teapot, or Small potatoes.

Still, President Bush would do himself and the country a favor if hed insist that everyone in the White House read the transcript of the 1973 House Judiciary Committee debate on the Nixon administrations dirty tricks.

It is a recitation of petty things that, taken together, dealt with grand principle. The principle is that the powers of government are a sacred trust, never to be employed to punish or embarrass ones domestic political opponents.

The very pettiness of the deed itself is, and should be, an embarrassment to us all.

Jim Wright is a former speaker of the U.S. House of Representatives. He wrote this essay for the Fort Worth Star-Telegram.

~~~~~~~~~~~~~~~~~~~~~~~~~~~~~~~~~~~~~~~~~~~~~~~~~~~~~~~

I've been waiting since July of last year to post Wright's drivel...and this:

Rove Won't Be Charged in CIA Leak Case

By JOHN SOLOMON , 06.13.2006, 07:35 AM

Top White House aide Karl Rove has been told by prosecutors he won't be charged with any crimes in the investigation into the leak of a CIA officer's identity, his lawyer said Tuesday.

Attorney Robert Luskin said that special prosecutor Patrick Fitzgerald informed him of the decision on Monday, ending months of speculation about the fate of one of President Bush's closest advisers. Rove testified five times before a grand jury.

Fitzgerald has already secured a criminal indictment against Vice President Dick Cheney's former chief of staff, I. Lewis "Scooter" Libby.

"On June 12, 2006, Special Counsel Patrick Fitzgerald formally advised us that he does not anticipate seeking charges against Karl Rove," Luskin said in a statement.

"In deference to the pending case, we will not make any further public statements about the subject matter of the investigation," Luskin said. "We believe the special counsel's decision should put an end to the baseless speculation about Mr. Rove's conduct."

Fitzgerald has been investigating whether senior administration officials intentionally leaked the identity of CIA undercover operative Valerie Plame in retribution because her husband, former Ambassador Joseph Wilson, sharply criticized the administration's pursuit of war in Iraq.

Rove, who most recently appeared before a grand jury in April, has admitted he spoke with columnist Robert Novak and Time magazine reporter Matt Cooper in the days before they published Plame's name in July 2003.

Rove, however, did not originally tell prosecutors about his conversation with Cooper, only revealing it after his lawyer discovered a White House e-mail that referred to it.

Fitzgerald was investigating whether Rove lied or obstructed justice in failing to initially disclose the conversation. The presidential aide blamed a faulty memory and sought to testify before the grand jury after finding the e-mail to correct his testimony.

The threat of indictment had hung over Rove, the man President Bush dubbed "the architect" of his re-election, even as Rove was focusing on the arduous task of halting Bush's popularity spiral and keeping Democrats from capturing the House or Senate in November elections.

Fitzgerald's investigation has been underway since the start of the 2004 election, and the decision not to indict Rove is certain to cheer Republicans concerned about Bush's low approval ratings and the prospects of a difficult 2006 congressional election.

"The fact is this, I thought it was wrong when you had people like Howard Dean and (Sen.) Harry Reid presuming that he was guilty,"
Republican Party Chairman Ken Mehlman told Fox News Channel's "Fox and Friends" show Tuesday morning.

Democrats, on the other hand, had no reason to cheer the development.

"Good news for the White House, not so good news for America," Dean, the Democratic Party chairman, said Tuesday on NBC's "Today" show.

Rove has been at Bush's sides since his days as Texas governor and was the architect of Bush's two presidential election victory. A political strategist, Rove assumed new policy responsibilities inside the White House in 2005 as deputy chief of staff.

However, as part of the shake-up brought by new White House chief of staff Joshua Bolton, Rove shed those policymaking duties earlier this year to return to full time politics.

Fitzgerald's case against Libby is moving toward trial, as the two sides work through pretrial issues such as access to classified documents.

Libby, 55, was charged last October with lying to the FBI and a federal grand jury about how he learned and when he subsequently told three reporters about CIA officer Valerie Plame. He faces five counts of perjury, false statements and obstruction of justice.

Plame's identity was exposed eight days after her husband, Bush administration critic and former U.S. Ambassador Joseph Wilson, alleged that the U.S. government had manipulated prewar intelligence to exaggerate an Iraqi nuclear threat.

With Rove's fate now decided, other unfinished business in Fitzgerald's probe focuses on the source who provided Washington Post reporter Bob Woodwind information about Plame.

Woodwind says his source, who he has not publicly identified, provided the information about Wilson's wife, several weeks before Novak learned of Plame's identity. The Post reporter, who never wrote a story, was interviewed by Fitzgerald late last year.

~~~~~~~~~~~~~~~~~~~~~~~~~~~~~~~~~~~~~~~~~~~~~~~~~~~~~~

Let the spin begin.

tomder55 answered on 06/13/06:

you mean THE Jim Wright ? The first Speaker of the House to reign in disgrace over ethics charges ? That's all I 've got to say on him .

Is it Fitzmas yet ?


No Frog March today !

Wonder how this will affect Plame's book deal ?

Does this mean that Patrick Fitzgerald finally packs up his tent and goes home ? I'd like an accounting about how much was spent since the Libby indictment on this crusade of his . Indeed when will he announce that he is dropping the charges of Libby ?

Separated at birth Patrick Fitzgerald and D.A. Mike Nifong ....add Ronnie Earle and you get triplets ...add Elliot Spitzer and ...well you see where tis is headed ;prosecutors run wild .

I wonder what moonbat Jason Leopold at "truth out" thinks today ? Did Jason really have sources? If so, who were they?

Just heard Howard Dean bemoaning the injustice of it all . I wonder ...if they start committing suicide will it be a form of asymmetical warfare ?

Itsdb rated this answer Excellent or Above Average Answer

Question/Answer
jackreade asked on 06/12/06 - Murtha-Perfect Time To Get Out of Iraq

WASHINGTON - "Encouraged by the death of a top terrorist leader and continued progress on the formation of a new government in
Iraq,
President Bush is gathering his top military and civilian war advisers to plan the U.S. role in the country's future.

The president planned two days of meetings at the mountainous Camp David presidential retreat starting Monday, with national security advisers on hand and top commanders in Iraq connected by videoconference.

White House officials have said announcements of force reductions are not expected. Yet the top U.S. commander in Baghdad predicted on the eve of the meeting that coalition troops will gradually move out of the country in the coming months.

Gen. George Casey said he thinks it will be possible to withdraw *some* of the 130,000 U.S. forces in the months ahead ***as long as Iraq's government and security forces make progress***.

Casey would not say whether he plans to advise Bush on a troop reduction plan Monday. But the general hinted the time soon may come for such a recommendation.

"I was waiting until we got a government seated before I gave the president another recommendation so we have some sense of what we've got," Casey said Sunday on CBS' "Face the Nation."

Iraq's prime minister, Nouri al-Maliki, took office last month and appointed the key final ministers last week.

Bush has said the new government marks a new chapter in the U.S. relationship with Iraq. ***With Republicans worried about losing control of Congress in November's midterm elections and most Americans saying they would like some troops to come home, Bush is under pressure***. Only a third of respondents to an Associated Press-Ipsos poll in early June supported Bush's handling of the situation an all-time low.

But he has been careful not to signal any troop reductions yet, continuing to say he will make those decisions when commanders in the field advise him to do so.

Rep. John Murtha , D-Pa., a critic of U.S. involvement in Iraq, said Monday that now is "a perfect time" for a troop withdrawal. "People want a change in this country ... a change in direction, and I hope the president hears that and I hope the Iraqis ask us to leave," Murtha said on ABC's "Good Morning America."

"Even this attack on Zarqawi happened from the air," Murtha said. "There's no real need for us to be inside the country."

~~~~~~~~~~~~~~~~~~~~~~~~~~~~~~~~~~~~~~~~~~~~~~`

Murtha's right. Zarqawi was killed from the air, not on land. Americans don't want to see soldierd die and get maimed ad infinitum.....and will vote against those who disagree.



tomder55 answered on 06/13/06:

I wonder what his fee is for going on 'Hardball' ???? Seems like he is on every other day .

I addressed this last week . Dopey Murtha is prepared to give Zarqawi in death what he could not achieve in life ;driving the US out of Iraq. Murthaa says that he was killed from the sky ? For someone who claims to be this military expert he is suprisingly ignorant about the process. Even ABC News paid tribute to the work on the ground that Task Force 145 did to make the assault successful . But Murtha evidently comes from the school of thought that electronic intel is sufficient ;the same type of flawed thinking that left our guard down 0n 9-11 .God forbid that on a day of triumph he give recognition to our troops . He is the last to give them any credit but the first to find the podium when he is condemning their conduct .


The President met at Camp David to meet with his advisers ;AND other experts to consider policy options . I am sure that ONE of the options will be GRADUAL troops reductions as the situation on the ground continues to progress. I am sure when that happens Murtha will take all credit for it .

One of the advisers who was present yesterday was Frederick Kagan of the American Enterprise Institute .He argues ;and I think it makes alot of sense and the President would be wise to listen ;that we need to make one or two final offensives before we consider withdrawal . We have been reluctant to do so because of domestic politics .....not in Iraq ,but here at home. We have instead been 'hunkering down 'which does no one any good . This offensive should be modeled after Sec. Rice's Clear ;Hold ;Build strategy . Of all the "exit strategies " I've read or heard about her's is the soundest and it beats the hell out of' cut and run 'Murtha's .

Eight million Iraqis voted for their interim national government , almost 10 million participated in the referendum on their new constitution and even more than that voted in the elections last year for a full-term government . Every time the Iraqis have been given the chance since Saddam was overthrown, they have voted for self-government and hope over the violence and hatred the terrorists offer them.None of this would have happened without the coalition forces led by the U.S. And almost all of the progress in Iraq and throughout the Middle East will be lost if those forces are withdrawn faster than the Iraqi military is capable of securing the country. In 1990 we brought their hopes for change up and then left them to suffer the wrath of a vengeful tyranny . Let's not repeat that mistake again .

jackreade rated this answer Poor or Incomplete Answer

Question/Answer
Itsdb asked on 06/12/06 - New Homeland Security Chief?



"A black bear got more than it bargained for after straying into a family garden in the US state of New Jersey.

The unwelcome intruder was forced up a tree - twice - by the family pet, a tabby cat called Jack..."

Maybe we should get Jack down on the southern border?

tomder55 answered on 06/12/06:

This weekend two swans had me in a standoff and blocked the trail I was on . They were protecting their two remaining off spring and were motivated . I decided to concede and blazed a new trail rather than to test the thesis that they couldn't hurt me .

Itsdb rated this answer Excellent or Above Average Answer
powderpuff rated this answer Excellent or Above Average Answer

Question/Answer
Itsdb asked on 06/12/06 - New Homeland Security Chief?



"A black bear got more than it bargained for after straying into a family garden in the US state of New Jersey.

The unwelcome intruder was forced up a tree - twice - by the family pet, a tabby cat called Jack..."

Maybe we should get Jack down on the southern border?

tomder55 answered on 06/12/06:

I knew that cats had to have some kind of purpose besides chow mein

Itsdb rated this answer Excellent or Above Average Answer

Question/Answer
Erewhon asked on 06/12/06 - Is this man mad?

Guantanamo Bay suicides 'an act of war'
ANDREW SELSKY AND JENNIFER LOVEN

* Suicides bring renewed condemnation of US base in Cuba
* US authorities describe the deaths as 'asymmetric warfare against us'
* Pressure sure to increase to close facility

Key quote
"They have no regard for human life, neither ours nor their own. I believe this was not an act of desperation but an act of asymmetric warfare against us"
-- Navy Rear-Admiral Harry Harris, base commander


THREE Guantanamo Bay detainees hanged themselves using nooses made of bedsheets and clothes, the commander of the facility confirmed yesterday, describing the suicides as "an act of asymmetric warfare" against the United States.

The suicides, which US military officials said were co-ordinated, have triggered further condemnation of the isolated detention centre in Cuba, which holds some 460 men on suspicion of links to al-Qaeda and the Taleban. Only ten have been charged with crimes and there has been growing international pressure on the US to close the prison.

Two men from Saudi Arabia and one from Yemen were found dead shortly after midnight on Saturday in separate cells, said the US Southern Command, which has jurisdiction over the prison. Attempts to revive the men were unsuccessful.

"They hung themselves with fabricated nooses made out of clothes and bed sheets," Navy Rear-Admiral Harry Harris, the base commander, said. "They have no regard for human life, neither ours nor their own. I believe this was not an act of desperation but an act of asymmetric warfare against us."

Military officials said the men had been held in Guantanamo Bay for about four years. The Saudi authorities identified their dead citizens as Mani bin Shaman bin Turki al Habradi and Yasser Talal Abdullah Yahya al Zahrani. The identity of the Yemeni is not yet known.

All three detainees had engaged in a hunger strike in protest at their indefinite imprisonment and had been force-fed before giving up their protest.

One of the detainees was a mid or high-level al-Qaeda operative, while another had been captured in Afghanistan and participated in a riot at a prison there, Rear-Admiral Harris said. The third belonged to a splinter group.

Detainees have not been allowed to know about classified evidence of allegations against them and thus are unable to respond.

"They're determined, intelligent, committed and they continue to do everything they can to become martyrs in the jihad," said General John Craddock, commander of the Miami-based Southern Command. All three men left suicide notes, Gen Craddock said. He refused to detail the contents of the contents.

Pentagon officials said the three men were in Camp 1, the highest-security area at Guantanamo, and that none of them had tried to commit suicide before. To help prevent more suicides, guards will now give bed sheets to detainees only when they go to bed and remove them after they wake up in the morning, Harris said.

George Bush, the US president expressed "serious concern" over the suicides and directed his administration to reach out diplomatically while it investigates the incident.

Tony Blair, the Prime Minister, has taken a cautious line on Guantanamo Bay, calling it an "anomaly" but not demanding its immediate closure.

Some of his ministers take a harder line, however, with Harriet Harman, the constitutional affairs minister and possible contender for the Labour deputy leadership, yesterday calling for immediate action over the prison.

"If it is perfectly legal and there is nothing going wrong there, why don't they have it in America and then the American court system can supervise it?" she said.

"It is in a legal no-man's land. Either it should be moved to America or it should be closed."

Guantanamo officials have reported 41 unsuccessful suicide attempts by 25 detainees since the US began taking prisoners to the base in January 2002. Defence lawyers contend the number of suicide attempts is higher.
Another embarrassment for Bush

THE three suicides at Guantanamo Bay have achieved a victory in death that they failed to find in life: their actions embarrass the US and seem likely to increase the pressure on Washington to close the facility.

The rights and wrongs of the situation scarcely matter: the US has been tried and found wanting in the court of public opinion. For Washington there is one simple question: are the advantages of Guantanamo Bay now outweighed by the international opprobrium the prison attracts?

It would be out of character for the Bush administration to change its mind because of pressure from the international community, but the camp gives the US's critics a stick with which to beat Washington.

President Bush has said: "We'd like it [Guantanamo] to be empty. Trouble is, there are some that, if put out on the streets, would create grave harm to American citizens and other citizens of the world." (Does it have to be Guantanamo or nowhere? Is Bush that short of imagination?)

And the view among many ordinary Americans is that these three inmates have set an ideal example for the remaining 460 or so prisoners incarcerated on Cuba. Only human rights lobby groups and hand-wringing liberals are much concerned by the fate of prisoners whose living conditions are, in most respects, little worse than those in ordinary American jails.

The difference is that those prisoners have been tried and convicted of specific crimes.

American officials will protest that the suicides are no more than a political protest. (Thery aslready have!) But no matter how grisly it may be, as publicity stunts go, those dead inmates have the whip hand. That this should be the case is perhaps the biggest defeat and embarrassment the US has suffered in the post-9/11 world.

Its stubbornness and indifference to the opinion of the rest of the world have helped foster a situation in which, grotesquely, terrorists have become the sympathetic victims.

That, more than anything else, has been Washington's most disastrous defeat in the war on terrorism. This weekends' deaths at Guantanamo Bay seem likely, alas, to confirm that.

ALEX MASSIE IN WASHINGTON

===

Is the Admiral completely MAD?




tomder55 answered on 06/12/06:

this is pretty much the same post that Brian made over the weekend so I will cut and paste my repsonse :

It is an act of war ;in this case the psyc-ops war .Bill O'Reilly was there on Fri doing interviews. I wonder if the timing has anything to do with getting press attention ? Nahhh;couldn't be .

Is this any different than if they had strapped a homicide belt on themselves ? Not really ; the apologists are always finding a rational for that too....despair I believe has been tossed around as an excuse before .ummmm perhaps despair that they are not free to murder many thousands of innocent American civilians.

But I guess in this pc world Rear Adm Harry Harris is being too honest for his own good . However ;I don't have to be pc. I think it is a terrible abuse of these prisoners ...hanging by bed sheets when there was probably perfectly good American rope available . I think we should issue them a cyanide capsule with their gormet meals . If Bush wants to close Gitmo that would probably be the quickest way .

The jihadists are dropping like flies this week . The press should be reporting it as good news . Shiite happens .

...........

Herman Goering swallowed a suicide cap, where was the protests from that injustice?

Lets remember that many US pilots faced long years of incarceration in Vietcong and North Vietnamese prisons in far worse conditions than Gitmo . I don't recall reading that any of them attempted suicide.I agree with Bush that it is probably time to start figuring out who can be sent home and who cannot.If you want to blame someone for the delay than point your finger at the courts. If I were Bush I would just order the military tribunals to proceed .It is my guess that very few of them will fall into the catagory of 'being in the wrong place at the wrong time .' What were 2 Saudis and a Yemeni doing in Afghanistan anyway ?


Erewhon rated this answer Excellent or Above Average Answer
Itsdb rated this answer Excellent or Above Average Answer
labman rated this answer Excellent or Above Average Answer

Question/Answer
HANK1 asked on 06/11/06 - IMMIGRANT DISEASES:



Does the influx of illegal aliens have serious hidden medical consequences?

tomder55 answered on 06/12/06:

yes it does . diseases that had been eradicated in this country are making a comeback .The New England Journal of Medicine (3-11-99) reported that "more than 600,000 immigrants enter the United States each year from countries where intestinal parasites are endemic.Malaria was eradicated in the United States in the early part of the 20th century, but now is reemerging as a serious health problem. Dr. Jane Zucker of the Centers for Disease Control cites immigration as a contributing factor in the recent outbreaks of mosquito-borne transmission in the densely populated areas of New Jersey, New York and Texas. The Centers for Disease Control has declared that the United States has been free from indigenous measles since 1998, and the only cases come in with immigrants.

Processing them at places like Ellis Island before entry gave us a chance in the past to screen out those with contagins .

HANK1 rated this answer Excellent or Above Average Answer
paraclete rated this answer Excellent or Above Average Answer

Question/Answer
Mathatmacoat asked on 06/11/06 - Now, that's progress?

Oklahoma legalises execution of repeat child molesters

June 11, 2006

The governor of the US state of Oklahoma has signed a bill allowing for the execution of repeat child molesters, a move that some critics argue is unconstitutional.

The US Supreme Court ruled in 1977 that the death penalty can only be applied in murder cases, but the author of the Oklahoma bill said it is time to challenge that ruling.

"Predators with multiple convictions for child molestation have proven they will continue to prey on Oklahoma's children until they are stopped," state senator Jay Gumm said in a statement.

"The crime of child molestation ripples throughout the life of a child, robbing them of their innocence and sense of safety," the Democrat lawmaker added.

"We need to send the message as a state, that if you repeatedly prey on our children we will find you, prosecute you, convict you and execute you."

The Oklahoma law makes people convicted of rape and other sex crimes more than once against children younger than 14 years of age eligible for execution.

Oklahoma is the fifth state to allow the death penalty for sex crimes against children.

The bill's signing by Democrat governor Brad Henry yesterday came a day after his Republican counterpart in South Carolina endorsed similar legislation in honour of a girl murdered last year by a registered sex offender.

South Carolina governor Mark Sanford said in a statement that the law, called Jessie's Law for the victim, would "be an incredibly powerful deterrent to offenders that have already been released.

"Jessie's Law is about sending a very clear message that there are some lines you do not cross," Sanford said.

The other states with similar legislation are Louisiana, Florida and Montana. Louisiana has one inmate on death row for the crime, but the case has not yet made its way through the appellate process.

In 1977, the US Supreme Court ruled that the death penalty could not be imposed on someone convicted for the rape of an adult woman.

Some critics warned that making child molesting a capital offence could further endanger children.

"There's an incentive not to leave the victim alive," warned Colin Garrett, the Death Penalty Resource Counsel for the National Association of Criminal Defence Lawyers.

Another concern is that the severity of the punishment will prevent some victims -- or their families -- from reporting the abuse because most children are molested by family members, he added.

There are 38 US states which currently allow for the death penalty and 1,026 people have been executed since it was reinstated in 1976, according to the Death Penalty Information Centre.

More than 120 people in the United States have been released from death row since 1973 because evidence surfaced of their innocence.

The number of death penalties imposed has declined dramatically in recent years, to 125 in 2004 from 276 in 1999, according to the Death Penalty Information Centre. There were 3,370 people on death row in the United States on April 1.

AFP

tomder55 answered on 06/11/06:

Similar statutes already are in place in Florida, Louisiana and Montana, and South Carolina's governor is expected to sign a similar law soon. I see nothing in your post that shows the slighest concern for the victims lke Jessica Lunsford ,who the law was named for ;who's life was cut short before she had a chance to live it by one of these repeat child molesters. Let them challenge the constitutionality of the law . Instead of 'cruel and unusual ,I call it mercy killing .

I see nothing wrong with the idea of death penalty to someone who has served notice on society that they are predators. Repeat offenders will not rehabilitate . Just get rid of them before another child becomes a victim . Give them a speedy trial ,and appeals process then just get rid of them .

I got an idea ! Let the judge or the lawyer that is responsible for releasing them take these lowlifes into their own homes and assume the responsibility of them. Maybe they will have some little children or their grandchildren that can entertain them during their stay there.

To answer your question . Death penalty for repeat offenders is progress.

http://www.kansascity.com/mld/kansascity/14790683.htm


Mathatmacoat rated this answer Excellent or Above Average Answer
paraclete rated this answer Excellent or Above Average Answer
purplewings rated this answer Excellent or Above Average Answer

Question/Answer
paraclete asked on 06/11/06 - If you can't win ~

then suicide, that'll fix em?


Inmate suicides 'an act of war'
From: Reuters

June 11, 2006


THE commander of the US Joint Task Force Guantanamo, Rear Admiral Harry Harris has described the overnight suicide of three inmates "as an act of war".

Three foreign prisoners were found dead overnight after hanging themselves with clothing and bedsheets in the first deaths at the US naval base at Guantanamo Bay, Cuba, since the prison opened in January 2002, US defence officials said.

The military said two Saudis and one Yemeni were found unresponsive and not breathing in their cells by guards and that attempts to resuscitate the detainees failed.

They were pronounced dead by a physician at Guantanamo, which holds just under 500 foreigners captured mainly in the US war against the Taliban and al Qaeda in Afghanistan.

The suicides have thrown a spotlight on the camp that has drawn widespread criticism against the Bush administration from foreign countries, including some allies, and human rights advocates.

Navy Rear Admiral Harry Harris, commander of Guantanamo, described the suicides were an act of warfare.

"They are smart. They are creative, they are committed. They have no regard for life, neither ours nor their own. I believe this was not an act of desperation, but an act of asymmetrical warfare waged against us," Rear Adm. Harris said.

The suicides threw a fresh spotlight on the camp that has drawn widespread criticism against the Bush administration from foreign countries, including some allies, and human rights advocates.

Facing indefinite detention with none of the rights afforded formal prisoners of war or criminal suspects in the US justice system, dozens of the detainees have undertaken hunger strikes and attempted suicide.

Guantanamo has been one of string of issues that have undermined support abroad for Washington's war on terrorism, declared after the Sept. 11 attacks. The deaths come as President George W. Bush faces growing public doubt about the war in Iraq.

The US military said the bodies were being treated "with the utmost respect." An investigation has begun, it said.

A White House spokesman said Bush expressed serious concern on Saturday when he was told about the three suicides.

Spokesman Tony Snow said Bush, who is at Camp David this weekend, was told of the deaths by Secretary of State Condoleezza Rice.

"He expressed serious concern," Snow said, adding that US officials made a round of telephone calls to notify American allies abroad.

Bush has said he would like to close the detention center and spoke of Guantanamo on Friday at a joint news conference with Danish Prime Minister Anders Fogh Rasmussen, who raised concerns about it with the US president.

"We'd like it to be empty," Bush said. "And we're now in the process of working with countries to repatriate people."

Ken Roth, head of Human Rights Watch in New York, said the suicides at Guantanamo likely were driven by despair.

"Sadly suicides like these are entirely predictable when people are held outside the law with no end in sight. They despair of spending the rest of their lives detained at the whim of their jailer with no sense of when it would end," he said.

tomder55 answered on 06/11/06:

It is an act of war ;in this case the psy-ops war .Bill O'Reilly was there on Fri doing interviews. I wonder if the timing has anything to do with getting press attention ? Nahhh;couldn't be .

Is this any different than if they had strapped a homicide belt on themselves ? Not really ; the apologists are always finding a rational for that too....despair I believe has been tossed around as an excuse before .ummmm perhaps despair that they are not free to murder many thousands of innocent American civilians.

But I guess in this pc world Rear Adm Harry Harris is being too honest for his own good . However ;I don't have to be pc. I think it is a terrible abuse of these prisoners ...hanging by bed sheets when there was probably perfectly good American rope available . I think we should issue them a cyanide capsule with their gormet meals . If Bush wants to close Gitmo that would probably be the quickest way .

The jihadists are dropping like flies this week . The press should be reporting it as good news . Shiite happens .

paraclete rated this answer Poor or Incomplete Answer
purplewings rated this answer Excellent or Above Average Answer

Question/Answer
Mathatmacoat asked on 06/10/06 - americans half baked about border security?

Help keep Texas safe from illegal immigrants

June 9, 2006 - 12:20PM

The governor of Texas wants to turn all the world into a virtual posse.

Rick Perry has announced a $US5 million plan to install hundreds of night-vision cameras on private land along the Mexican border and put the live video on the Internet, so that anyone with a computer who spots illegal immigrants trying to slip across can report it on a toll-free hot line.

"I look at this as not different from the neighborhood watches we have had in our communities for years and years," Perry said last week.

Some say it is a dangerous idea and a waste of money.

"This is just one of those half-baked ideas that people dream up to save money but have no practical applications," said Jim Harrington, director of the Texas Civil Rights Project in Austin. "We would be far better off to invest that money in Mexican small towns along the border so people wouldn't have to emigrate."

The plan marks a political about-face for Perry, a Republican seeking re-election, who has long argued that security along the state's 1930km border with Mexico is strictly a federal responsibility.

This week, he said cuts in federal homeland security funding, a rise in reports of border violence and the crossing of Mexican soldiers into Texas about two years ago have demonstrated that "Texas cannot wait for Washington, DC, to act."

Under the plan, announced on the eve of the state GOP convention, cameras and other equipment would be supplied to willing landowners and placed along some of the most remote reaches of the border. The live video would be made available to law enforcement and anyone else with an internet connection.

Viewers would be able to call day or night to report anything that looks like trespassing, drug smuggling or something else suspicious.

The governor plans to pay for it all with grant money the state already has, and wants the first cameras in place within 30 days.

The Border Patrol already has lots of its own surveillance cameras along the border, but the images are not made available to the public.

Border Patrol Chief David Aguilar did not comment directly on the governor's plan Wednesday, but said: "We are looking forward to the opportunity to sit down and discuss it with him to ensure that whatever is done will be aligned with the efforts of the Border Patrol."

Agency officials did not immediately return calls for comment Thursday.

Luis Figueroa, an attorney with the Mexican American Legal Defence and Educational Fund, warned that the cameras could lead to racial profiling and vigilanteism.

"This leaves the door open to anyone who has a vindictive state of mind or a racial motive," Figueroa said. "Anyone down there could easily be mistaken and falsely accused of something they didn't do."

Harrington said letting the public watch what is essentially a law enforcement search could be illegal.

And T.J. Bonner, president of the union that represents nearly all Border Patrol agents, said the plan could further strain the overworked agency.

"At first blush, it sounds like just another crazy idea that is going to overwhelm the capabilities of the federal government to be able to respond to the number of calls coming in and to the number of reports," Bonner said. "But there is a silver lining: It might just make legislators aware."

Bonner said it won't take smugglers long to figure out where the cameras are.

Connie Hair, a spokeswoman for the Minuteman organisation, which patrols the border against illegal immigrants, said access to the video should be restricted to trained volunteers and law enforcement officials, to prevent smugglers from using the equipment to adjust their routes.

But the governor said it will be hard to know where the cameras are just by watching the live internet video. And if the smugglers do figure it out, the equipment can easily be moved. "This isn't our first rodeo," he said.

AP

tomder55 answered on 06/10/06:

The govenor was being negligent when he said that border security was strictly a Federal problem . That reeks of the same attitude that the elected of Lousiana and New Orleans took prior to Katrina. As govenor of the State it is his duty to deal with security within Texas borders . This ploy with cameras is his attempt to do nothing while appearing to do something . No ;giving money to Mexican border towns is not the answer either ;Having a State police patrolling the border or the Texas Natioanl Guard would be more effoective OR putting cameras on a border wall ...now your talking ! When he or Bush talk about "virtual fences" then that is code for them not taking the idea of border security seriously .

There are alot of issues with this govenor and the conservative base .He has done squat to this point on border security .He is a tax and spend politician and he has failed to pass a cap on property appraisal increases.His wife, Rebecca Flores was a united farmworkers organizer so you know that he is pretty symapthetic towards a liberal immigration policy anyway .

Texas Republicans had a convention in Alamo City this week . Their concensus is that they want a PHYSICAL Barrier not a virtual one .

Many of them are also peeved over Perry's plan to build a trans-Texas corridor which will no doubt open many eminenent domain issues. I have not studied the issue fully but I have seen other highway projects like 'The Big Dig ' in Boston turn into big buck boondoggles . It appears that it will be so expansive as to make many small Texas towns drive over country .

How does this connect to the border issue ? Well evidently part of this super-highway will go through Mexico and connect with Mexican ports .Part of the deal would be to allow Mexican trucks to cross the border on the highway with minimal electronic inspection. So what you have in Texas is the government of the state giving lip service to border securtiy while at the same time promoting a highway system that would make the border even less secure.

ETWolverine rated this answer Excellent or Above Average Answer
Mathatmacoat rated this answer Excellent or Above Average Answer

Question/Answer
Itsdb asked on 06/09/06 - 'Time' for a Haditha hoax?

From the Sweetness & Light website:

Time correspondent Tim McGirk, who broke the Haditha story, said that in the weeks before publication, he had lobbied editors to use the word massacre in the March 27 story.

That was a battle I lost, Mr. McGirk said by phone May 30 from Jerusalem, where he is currently based. I think the editors felt massacre was too heavy of a word. They didnt want to use it; they felt there was some justification for what had happened.

I think it was definitely a massacre, Mr. McGirk said.

~~~~~~~~~~~~~~~~~~~~~~~~~~~~~~~~~~~~~~~~~~~~~~~~~~~~~~~

Times Corrections About Haditha

This is a now all too familiar pattern with our one party media.

The following "corrections" have been added to the very bottom of Times two blockbuster exclusives on Haditha in their current online versions:

    Collateral Damage or Civilian Massacre in Haditha?

    Last November, U.S. Marines killed 15 Iraqi civilians in their homes. Was it self-defense, an accident or cold-blooded revenge? A Time exclusive

    By TIM MCGIRK / BAGHDAD

    Sunday, Mar. 19, 2006

    In the original version of this story, TIME reported that "a day after the incident, a Haditha journalism student videotaped the scene at the local morgue and at the homes where the killings had occurred. The video was obtained by the Hammurabi Human Rights Group, which cooperates with the internationally respected Human Rights Watch, and has been shared with TIME." In fact, Human Rights Watch has no ties or association with the Hammurabi Human Rights Group. TIME regrets the error.

    http://www.time.com/time/world/article/0,8599,1174649-1,00.html

    One Morning in Haditha

    U.S. Marines killed 15 Iraqi civilians in their homes last November. Was it self-defense, an accident or cold-blooded revenge?

    By TIM MCGIRK/ BAGHDAD

    Mar. 27, 2006

    In the original version of this story, TIME reported that "a day after the incident, a Haditha journalism student videotaped the scene at the local morgue and at the homes where the killings had occurred. The video was obtained by the Hammurabi Human Rights Group, which cooperates with the internationally respected Human Rights Watch, and has been shared with TIME." In fact, Human Rights Watch has no ties or association with the Hammurabi Human Rights Group. TIME regrets the error.
    http://time-proxy.yaga.com/time/archive/preview/0,10987,1174682, 00.html


In fact, Time had originally reported that it was Human Rights Watch who had provided the tape. They then retracted that and claimed that it came from Hammurabi which works with Human Rights Watch. And now they have backed off even that.

Note that even now Time still does not correct the intentionally false portrayal of the source of the videotape that they gave in all of their original stories and interviews.

Times source, Thaer Thabit al-Hadithi, is not a "young man." He is not a "budding journalism student."

And al-Haditha is not separate and apart from the Hammurabi Human Rights Group. Nor is he a man who wanted to remain anonymous because he feared for his safety.

Al-Haditha is 43 years old. He "created" Hammurabi 16 months ago. (Before that he worked directly under the head of Hadithas hospital, Dr. Walid al-Obeidi, who pronounced that all the victims had been shot at close range.)

In fact, al-Haditha is one of Hammurabis only two members. He serves as its "Secretary General" while the only other member, Abdul-Rahman al-Mashhadani, performs as its "Chairman.")

Al-Haditha is the one and only person behind this tape. He made it. And he sat on it for four months before turning it over to Time magazine.

But it looks like Time did not consider these mundane facts about the maker of this tape compelling enough. So they made up additional romantic details and invented the involvement of the "internationally respected Human Rights Watch" to burnish the videos provenance.

Its something Time does on a regular basis.

Here is another "correction" that is now buried at the bottom of another Time Haditha story from last month. It is by Matthew Cooper of Plame/Rove notoriety.

    The Haditha Scandals Other Casualty

    With the Pentagon completing its probe into whether U.S. forces massacred civilians one November morning in Western Iraq, the damage to Americas image abroad could take a further hit

    By MATTHEW COOPER/WASHINGTON

    Posted Friday, May. 26, 2006

    In the original version of this story, TIME reported that "one of the most damning pieces of evidence investigators have in their possession, John Sifton of Human Rights Watch told Times Tim McGirk, is a photo, taken by a Marine with his cell phone that shows Iraqis kneeling and thus posing no threat before they were shot." While Sifton did tell TIME that there was photographic evidence, taken by Marines, he had only heard about the specific content of the photos from reports done by NBC, and had no firsthand knowledge. TIME regrets the error.

    http://www.time.com/time/world/article/0,8599,1198843,00.html


Never mind that now "one of the most damning pieces of evidence" has already taken on the mantle of historical fact. Time regrets the error.

So much so that they once again buried the correction at the bottom of its online archive of the story which few will revisit.

Clearly Time thinks very highly of the Soros-funded (and viscerally anti-American) Human Rights Watch. They use every opportunity to cite HRW to bolster their claims, even if they have to make things up.

Apparently Time believes invoking "the internationally respected Human Rights Watch" gives their questionably sourced facts credibility

And so what it if they arent involved?

Time can always sneak in a retraction later when nobody is looking, once the story has "gotten legs."

~~~~~~~~~~~~~~~~~~~~~~~~~~~~~~~~~~~~~~~~~~~~~~~~~~~~~~

Why Did Rights Group Sit On Its Haditha Story?

The man Time Magazine first described as a "budding journalism student" Thaer Thabit al-Hadithi is now portrayed as an "Iraq Investigator" by the DNCs Associated Press.

But excerpts from the APs article raises some questions about him and his groups only other member, Abdul-Rahman al-Mashhadani:

    Iraq Investigator Tells AP About Haditha

    June 7, 2006

    BAGHDAD, Iraq A small group of U.S. Marines alleged to have killed up to two dozen Iraqi civilians conducted a house-to-house hunt that stretched over three hours, while other Marines in Haditha did not intervene, according to an Iraqi human rights investigator.

    The Associated Press interview of the activist is the most detailed account yet of Iraqi accusations that Marines went on a rampage after a comrade was killed by a bomb. Two separate U.S. military investigations of the incident are under way.

    Thaer al-Hadithi, a member and spokesman for the Hammurabi human rights association, a Sunni Muslim group, recounted with the help of a satellite map when and where Iraqi civilians cowered and sometimes died.

    The case, which came to public attention two months ago because of a video released by the Hammurabi group, is threatening to further weaken popular support for the Iraq war in the United States and has tarnished the militarys image in Iraq and around the world

    Hammurabi chairman Abdul-Rahman al-Mashhadani told the AP on Tuesday that his group was investigating other violations of Iraqi civil rights by Western forces in the mainly Sunni Arab provinces of Anbar and Salaheddin to the west and north of Baghdad. He said the group also was looking into violations by Iraqi security forces, militias and tribal clans.

    "We are also against terrorism," he said

    Al-Hadithi did not attend any of the meetings between victims families and the U.S. military, but he based his account of what the Marines officer said on briefings from Hammurabis Haditha representative and conversations with the families.

    Al-Mashhadani, Hammurabis chairman, who lectures on economics at Baghdads al-Mustansiriyah University, said the organization was publicizing the Haditha incident to make sure its not repeated


But back in March when this story first was reported by Time Magazine we were told by Time reporter Aparisim Ghosh (via Democracy Now!) that Thaer Thabit (aka Thabet) al-Hadithi did not want his name known out of concern for his personal safety:

    AMY GOODMAN: And then, this Haditha journalism student, who is this student?

    APARISIM GHOSH: Wed rather not say, for his own protection, but hes a young local man. Its not uncommon in Iraq for young people to have video cameras and cameras, and theres so much going on in their lives that they have plenty to shoot.

    AMY GOODMAN: And you got a hold of this, or Hammurabi Human Rights got a hold of this.

    APARISIM GHOSH: He brought the tape to Hammurabi Human Rights, which is a local human rights group, and they brought it to us once they found out that we were inquiring about this.


Why start a human rights group if you want to remain anonymous? And why did Time pretend their source was young? Why did they pretend he had no involvement with Hammurabi? (When in fact he is its founder.)

But that is just the start of the many questionable aspects of Thabits accounts.

Bear in mind that this "budding journalism student" waited until the next day to videotape this alleged atrocity, which supposedly happened on his very doorstep.

Note that this same "budding journalism student" and self-proclaimed human rights watcher did not bother to turn over his video to a media outlet or a real human rights group from November 2005 until March 2006. A four month delay.

Thats how eager they were to make sure such a crime is never again repeated.

Now look at Thabits partner at Hammurabi, Abdul-Rahman al-Mashhadani.

Al-Mashhadani is described in a December 15, 2005 article from the Institute For War And Peace as an election monitor. In fact, he expressed great satisfaction with the turnout in the Iraqi elections:

    Abdul-Rahman al-Mashhadani, an election monitor in Baghdad with the non-governmental organisation Hamurabi, said no major violations occurred in Baghdad. He expected 90 per cent turnout in Sunni Arab areas.


(Which is quite ironic, given that only 150 people out of 90,000 dared to risk their lives to vote in the earlier October referendum in Haditha.)

But if Abdul-Rahman al-Mashhadani was talking to the media in December about the election turnout, why didnt he tell them then about this alleged atrocity at Haditha that had just occurred three weeks before?

Wasnt that the purpose in starting the Hammurabi group?

It just doesnt make sense.

~~~~~~~~~~~~~~~~~~~~~~~~~~~~~~~~~~~~~~~~~~~~~~~~~~~~~~

By the way, Time's 'Haditha' reporter Tim McGirk is also known for his Thanksgiving with the Taliban.

Comments?

tomder55 answered on 06/10/06:

This is the same McGirk that gave us the fake wedding massacre of 2002 .

some more at American Thinker

The truth will be revealed .

Right now I find it disturbing reports that the Marines who have not even been charged yet are in shackles at Fort Pendelton .Detainees at Gitmo are allowed to play soccer when not in their cells but our Marines are shackled while outside and while inside they are in solitary confinement. I am not certain of the veracity of this report because Michael Savage has been running with it but if true it is a disgrace .

Itsdb rated this answer Excellent or Above Average Answer

Question/Answer
jackreade asked on 06/10/06 - Murtha Running in the Fall

"WASHINGTON (CNN) -- Rep. John Murtha, an outspoken opponent of the war in Iraq, announced Friday that he would run for majority leader if the Democrats take over the House in the fall elections.

"If we prevail as I hope and know we will, and return to the majority this next Congress, I have decided to run for the open seat of the majority leader," Murtha said in a letter to his Democratic colleagues."

~~~~~~~~~~~~~~~~~~~~~~~~~~~~~~~~~~~~~~~~~~~~~~~

This is very good news. As we all know, today one of the disgraced members of the Bush Crime Family, the unapologetic Tom Delay left the House of Representatives. It will be good for America to have a real man, a strong honest man interested in running for this important job.

Democrats everywhere will be energized to get out the independent vote as well as the Democratic Base vote.....including *Christian* African Americans, in November.

Many thanks to Al Gore for his fine film which has put the issue of climate in the face of voters.

Many thanks to Ann Coulter for disgracing herself on television promoting her latest screed the purpose of which is to convince average Americans that it is *OK to hate*.

The tide has turned. Americans are against the Bush Crime Family and Bush's War of aggression on Iraq, war solicited by LIES and conducted with too few soldiers to win.

Aren't you happy the tide has turned?

tomder55 answered on 06/10/06:


And here I thought he was next in line of suscession for the vacancy left by the death of Zarqawi.

It certainly doesn't suprise me that someone would challenge the current democrat leadership positions. I am suprised no one has announced a challenge to Pelosi for the #1 spot. His opponent would be Minority Whip Steny Hoyer.Tip for Hoyer ..Do not question his patriotism ...that is taboo !! No freedom of speach where that subject is concerned. Murtha isn't either ambitious enough or smart enough to do this himself.Murtha is a lap dog of Nancy Pelosi who favors a fast withdrawal from Iraq. Hoyer is more inclined to tough it out in Iraq, and has a long history of friction with Pelosi.This is Pelosi's doing. She plays Murtha like a drum, and he's just dumb enough to trot out and do whatever she says. I've got a feeling he's just screwed any chances he may have had for ANY leadership position.Adds a senior Democratic aide: "A lot of members are very angry that Jack Murtha has decided to blow up the caucus and declare a leadership race when we are the most unified we've been in years. We're really focused on taking back the house and should not be distracted with a leadership race. It's going to be a huge diversion."


If I were Murtha ,I would be more concerned with winning reelection in the Fall first.



Honest man ? You must be talking about a different Jack Murtha. As I have pointed out before ,when you talk of culture of corruption ,Murhta is a poster boy .Had he remained an anonymous back bencher then what is being revealed about him during this campaign probably would've fallen under the radar . But , all he has accompished as an Congressman for over 30 years is to compile an impressive list of pork going to his district .If Murtha were a powerful Republican legislator, the media would probably be all over this story.

A former aide from John Murtha's office, Carmen Scialabba, is a top official at KSA Consulting, where Kit Murtha (his brother ) is a senior partner. KSA has directly lobbied Murtha's office on behalf of defense clients that directly benefited from the $417-billion 2004 Defense bill. Murtha's subcommittee staff helps write Defense appropriations bills and oversees the earmark requests forwarded by Democrats. The contracts for KSA clients in the bill were entirely earmarks. Most of KSA's defense clients hired the firm only after Kit Murtha became a senior partner in 2002.

John Murtha is the top House recipient of campaign contributions from the defense industry for the past three years. . Murtha had received over $200,000 from defense firms in the 2006 election cycle, surpassing the next highest recipient by over $60,000.

Kit Murtha's has been lobbying for defense firms since at least 1986.In 1994 Westinghouse made Kit Murtha its director of state and local government affairs, in which role he would also lobby the Pennsylvania congressional delegation in Washington. At that time, John Murtha chaired the defense appropriations subcommittee.

He is also no stranger to corruption in the past . He was an unindicted co-conspirator in the Abscam scandal.When the House ethics committee cleared Murtha in 1981, The House ethics committee's lead counsel, E. Barrett Prettyman Jr.,resigned. When asked if he had resigned because of the committee's Murtha vote, he said that would be "a logical conclusion." The Congressman Rep. Don Bailey (D-Pa.)who defended him at the hearings later wrote this letter to Murhta which states that his opinion about Murtha's involvement in Abscam had dramatically changed by 2002.

"I was, to be honest, critical about how you misled me about Abscam where you convinced me you had voluntarily told federal agents about the offer of money to you," ...."I learned later, after I had successfully defeated the ethics charges against you, that you had merely manipulated the system to cooperate with federal agents to avoid prosecution,"

An ethically suspect member of Congress, with close, personal connections to lobbyists whose clients are benefited by his committee? Why hasn't the MSM run with this ? Could it be their love affair with his Bush bashing and his virulent anti-war position ? Nah ! When asked about his corrupt legacy Murtha responds :"Questions about my record are clearly an attempt to distract attention from the real issue, which is that our brave men and women in uniform are dying and being injured every day in the middle of a civil war that can be resolved only by the Iraqis themselves."


As Steve posted yesterday there is now reasonable speculation that The Hadita charges are hoaxes . Murhta has been drooling over them from the get -go .He convicted the marines in the court of public opinion before any real proof was established .This could be his Dan Rather moment. As Paul Hackett said "With one broad stroke, he's recklessly indicted all those Marines ... I don't know if he's gotten addicted to the microphones and the cameras. For him to continue to foam at the mouth, it's irresponsible, it's stupid, it's wrong."

ETWolverine rated this answer Excellent or Above Average Answer
jackreade rated this answer Average Answer

Question/Answer
Itsdb asked on 06/08/06 - Bolton and Deputy UN Sec-Gen go at it...

...and the "drive by media" takes a shot.

"Deputy Secretary General of the United Nations Mark Malloch Brown yesterday delivered remarkably candid remarks before a conference sponsored by our two organizations in which he called for greater U.S. leadership in strengthening the United Nations leadership which is clearly lacking," reads the opening remarks of a statement by former Clinton Chief of Staff John Podesta and Richard C. Leone

Ambassador Bolton offered a 'candid' response to Brown's speech, calling it "a very, very grave mistake."

Podesta and Leone's statement responds:

    "In typical fashion, Bolton responded with a threat: To have the Deputy Secretary General criticize the United States in such a manner can only do harm to the United Nations...Thoughtful and objective readers will agree that Mr. Malloch Browns remarks do not merit the bomb-throwing invective typical of our nations current diplomat to the United Nations; indeed, John Bolton sadly proves Mr. Malloch Browns point.


The Washington Post chimes in with:

    "Ambassador John Bolton called on Secretary-General Kofi Annan to ``repudiate'' the speech given by his top aide, Deputy Secretary-General Mark Malloch Brown, or live with ``adverse'' consequences."


Ignoring that Brown's speech calls Annan "arguably the UNs best-ever Secretary-General" and praises the effectiveness and "bargain bin price" of UN peacekeeping efforts (except of course those "tragic mistakes in Rwanda, Somalia and Yugoslavia" he mentions), he insults the American people by implying we just don't have any idea about the good the UN does because of "Rush Limbaugh and Fox News."

There is much more if you wish to do your own "thoughtful and objective" study, but let's see if I have this right.

According to Brown, the UN is both effective and cost effective, but we're too stupid to know that because all we hear is Rush and all we watch is Fox News.

Bolton's response that Brown's speech was "a very, very grave mistake" to criticize the US that way, warns of the potential harm to the UN and calls on Annan to repudiate the speech, that Annan "needs to make it clear that these remarks did not represent his opinion about the United States" is a a "bomb-throwing invective."

Brown's speech was "sincere and constructive critique."

"To have the Deputy Secretary General criticize the United States in such a manner can only do harm to the United Nations" is a threat.

Many of us despise the UN because of Rush and Fox News.

Lastly, "My hope is that he looks at the potential adverse effect that these intemperate remarks would have on the organization and repudiate it," is best reported as a threat to Kofi Annan telling him to repudiate it "or live with ``adverse'' consequences."

Have I got this all right?

tomder55 answered on 06/09/06:

Brown's comments attacking the American people was a disgrace and he should be called to the carpet over it . Also our vaunted MSM should've been all over his case but instead almost acquiesced with his view by their silence .

It appears that Brown was speaking to the choir based on the speaker list at the conference sponsored by the ' Security and Peace Initiative' :

Madeleine Albright, former U.S. Secretary of State during the Clintoon reign

Mark Malloch Brown, U.N. Deputy Secretary-General

Jim Leach, U.S. Congressman from Iowa ;consistently one of the most liberal Republicans in the House. He voted against the 2002 Iraq War Resolution and favors abortion rights. He was the only House Republican to vote against the 2003 tax cut. His district is considered Iowa's most Democratic district.

Richard C. Leone, President, The Century Foundation author of "The War on Our Freedoms: Civil Liberties in the Age of Terrorism"

John Podesta ,another Clintonista ..former Chief of Staff.

George Soros,the anti-American Founder and Chairman, Open Society Institute ...enough said .

If you can stomach his full comments they can be found here

A quick summary
1. The UN is indepensible
2. If the UN was a mistake, it was an American mistake. Truman intended to exercise power through the UN. Give us the power you promised
3.Kofi Annan is the best UN Secretary General ever. If you haven't noticed it's because his achievements have been cheapened.
4.It's true the UN was a catastrophe during the 1990s, during a compliant US administration I won't mention because this is not a political speech. And I'll omit the fact that one of the principal persons in charge of Peacekeeping was Kofi Annan. But he's in charge now. Things are different.
4.Peace keeping works best when there is already peace .It doesn't work as well when there is no peace .
5.despite the fact that the UN is doing such a good job it is failing. Why? because America isn't giving us the money. Isn't giving us the political support. But mostly it isn't giving us the money.
6.Although the human rights commission was completely discreditied the US should support the NEW Human Rights commission .
7.This one I have to dod verbadum it is too funny :
More broadly, Americans complain about the UNs bureaucracy, weak decision-making, the lack of accountable modern management structures and the political divisions of the General Assembly here in New York. And my response is, "guilty on all counts."

But why?

In significant part because the U.S. has not stuck with its projectits professed wish to have a strong, effective United Nationsin a systematic way.


So it is US fault that they are corrupt ,weak and inefficent . He goes on to make the comments that you refer to ;that the UN is getting bad press from Fox and Rush . He then accuses the US of extremism....why ? Because we will not fund a new UN building .We can see this even on apparently non-controversial issues such as renovating the dilapidated UN headquarters in New York. He uses the metaphor to compare the building to the state of the UN ;caused by US neglect .If only the US would give more money to the UN all would be well in the world . And why should we do so ? Because WE NEED THE UN . You can't make this stuff up .

Boil it all down to this . Brown was making an appeal to the left wing of American politics to do something. I'm sure Goerge Soros was attentive even if Middle America wasn't .


ETWolverine rated this answer Excellent or Above Average Answer
Itsdb rated this answer Excellent or Above Average Answer

Question/Answer
jackreade asked on 06/08/06 - It's Alright Ma I'm Only Bleeding

"Darkness at the break of noon
Shadows even the silver spoon
The handmade blade, the child's balloon
Eclipses both the sun and moon
To understand you know too soon
There is no sense in trying.

Pointed threats, they bluff with scorn
Suicide remarks are torn
From the fool's gold mouthpiece
The hollow horn plays wasted words
Proves to warn
That he not busy being born
Is busy dying.

Temptation's page flies out the door
You follow, find yourself at war
Watch waterfalls of pity roar
You feel to moan but unlike before
You discover
That you'd just be
One more person crying.

So don't fear if you hear
A foreign sound to your ear
It's alright, Ma, I'm only sighing.

As some warn victory, some downfall
Private reasons great or small
Can be seen in the eyes of those that call
To make all that should be killed to crawl
While others say don't hate nothing at all
Except hatred.

Disillusioned words like bullets bark
As human gods aim for their mark
Made everything from toy guns that spark
To flesh-colored Christs that glow in the dark
It's easy to see without looking too far
That not much
Is really sacred.

While preachers preach of evil fates
Teachers teach that knowledge waits
Can lead to hundred-dollar plates
Goodness hides behind its gates
But even the president of the United States
Sometimes must have
To stand naked.

An' though the rules of the road have been lodged
It's only people's games that you got to dodge
And it's alright, Ma, I can make it.

Advertising signs that con you
Into thinking you're the one
That can do what's never been done
That can win what's never been won
Meantime life outside goes on
All around you.

You lose yourself, you reappear
You suddenly find you got nothing to fear
Alone you stand with nobody near
When a trembling distant voice, unclear
Startles your sleeping ears to hear
That somebody thinks
They really found you.

A question in your nerves is lit
Yet you know there is no answer fit to satisfy
Insure you not to quit
To keep it in your mind and not fergit
That it is not he or she or them or it
That you belong to.

Although the masters make the rules
For the wise men and the fools
I got nothing, Ma, to live up to.

For them that must obey authority
That they do not respect in any degree
Who despise their jobs, their destinies
Speak jealously of them that are free
Cultivate their flowers to be
Nothing more than something
They invest in.

While some on principles baptized
To strict party platform ties
Social clubs in drag disguise
Outsiders they can freely criticize
Tell nothing except who to idolize
And then say God bless him.

While one who sings with his tongue on fire
Gargles in the rat race choir
Bent out of shape from society's pliers
Cares not to come up any higher
But rather get you down in the hole
That he's in.

But I mean no harm nor put fault
On anyone that lives in a vault
But it's alright, Ma, if I can't please him.

Old lady judges watch people in pairs
Limited in sex, they dare
To push fake morals, insult and stare
While money doesn't talk, it swears
Obscenity, who really cares
Propaganda, all is phony.

While them that defend what they cannot see
With a killer's pride, security
It blows the minds most bitterly
For them that think death's honesty
Won't fall upon them naturally
Life sometimes
Must get lonely.

My eyes collide head-on with stuffed graveyards
False gods, I scuff
At pettiness which plays so rough
Walk upside-down inside handcuffs
Kick my legs to crash it off
Say okay, I have had enough
What else can you show me?

And if my thought-dreams could be seen
They'd probably put my head in a guillotine
But it's alright, Ma, it's life, and life only."

Bob Dylan
Copyright 1965; renewed 1993 Special Rider Music

Columbia Records

~~~~~~~~~~~~~~~~~~~~~~~~~~~~~~~~~~~~~~~~~~~

Comments?

tomder55 answered on 06/09/06:

its a wonder Dylan never committed suicide . Can you show me one song where he celebrates life ? Maybe during his brief forray into Christianity in the late 1970s but damn ..he should lighten up and at least celebrate how fortunate his life has been .

jackreade rated this answer Poor or Incomplete Answer
purplewings rated this answer Excellent or Above Average Answer

Question/Answer
jackreade asked on 06/09/06 - Timetable

"Representative John P. Murtha, a Pennsylvania Democrat and former marine who has become a fierce critic of the Iraq war, said now that a "real thorn" in the side of the Americans has been removed, Iraqi forces were trained and a government was in place, the Bush administration should compose a timetable for the withdrawal of American troops.

"We cannot win this," he said in an interview on CNN. "It is a civil war they are involved in. Al Qaeda is a small part of this."

He added, "We have Sunnis fighting Shiites and the Americans are caught in between."

The insurgency and violence in Iraq is fueled by a complicated fabric of foreign fighters, Saddam Hussein loyalists and other groups, while most recently, militias have been blamed for sectarian strife."

~~~~~~~~~~~~~~~~~~~~~~~~~~~~~~~~~~~~~~~~~~~~~~~

American soldiers caught up in a civil war?
Do you agree with Murtha that a timetable is needed?

tomder55 answered on 06/09/06:

lol Murtha doesn't have a clue . Zarqwai's stated goal is the withdrawal of American troops ,and the day he is wacked what does Murtha say ? Time to go . Bwaaahhaaahaaahaaa!!

No I would NEVER publish a time table regardless of wheter we have one or not . Murhta with his so called understanding of the military should understand that rational .

He is so caught up in the politics of it that he cannot give credit on a day when the military has certainly earned it ;but he has no problem being in the lead when it comes to condemning them . He is a disgrace .

ETWolverine rated this answer Excellent or Above Average Answer
Itsdb rated this answer Excellent or Above Average Answer
jackreade rated this answer Excellent or Above Average Answer
LTgolf rated this answer Excellent or Above Average Answer

Question/Answer
CeeBee2 asked on 06/07/06 - The Making of a Terrorist: Recruitment, Training, and

Root Causes - a new 3 volume set I'm going to catalog after I post this. Was published by Praeger Security International in Westport, CT and edited by James J.F. Forest, Director of Terrorism Studies and Assistant Professor of PoliSci at the U.S. Military Academy at West Point. Each volume is about one of the topics mentioned in the subtitle; each volume has a terrific index. You might be interested in asking for it at your local public library. The set appears to cover every question about/aspect of terrorism anyone could ever think of.

tomder55 answered on 06/08/06:

Here is the Wilson Quartely review .

added them to my list of must reads .

I also recommend "'War Footing' 10 Steps America Must take to prevail in the war for the free world" ;edited by Frank Gaffney Jr.[Center for Security Policy: Founder and President] with contributions from many security experts .

CeeBee2 rated this answer Excellent or Above Average Answer

Question/Answer
excon asked on 06/07/06 - Gay Marriage Amendment goes down


Hello wrongwingers:

Hellllllooooo!!!! In this country, we don't vote on civil rights.!!!! IF we vote on civil rights, its to GIVE civil rights to a group who has previously been denied those rights. Thats what weve ALWAYS done! Our Constitution is a document of INCLUSION. YOU, for the first time in history, want to CONSITUTIONALIZE discrimination.

Who, among you, don't get that simple civics lesson? Who among you want to spin this into a states rights question?

Fortunately, it's not gonna happen - not in my great country. And, those among you who DON'T get it - your time is passing.

excon

tomder55 answered on 06/08/06:

Good idea . Let's float an amendment that gives gay marriage official sanction and let's see how far it gets !!

I for one want DOMA passed as an amendment for the purpose of protecting federalism from judicial tryanny . I was not in favor of the amendment introduced. As it turned out it was not even a good political ploy because Frist is so lame at ushering bills through our Roman Senate .

excon rated this answer Excellent or Above Average Answer

Question/Answer
jackreade asked on 06/07/06 - Will You do Despicable Things Just For Money?

"NEW YORK (AP) - The group of outspoken 9/11 widows who pushed for the commission to investigate the attacks are ``self-obsessed'' and act ``as if the terrorist attacks happened only to them,'' conservative author Ann Coulter charges in her new book.

Coulter appeared on NBC's ``Today'' show on Tuesday and reiterated her stance, saying the women used their grief ``to make a political point.''

In her book, Coulter said, ``I've never seen people enjoying their husbands' deaths so much.''

The women are Kristen Breitweiser, Lorie Van Auken, Mindy Kleinberg and Patty Casazza of New Jersey. Coulter refers to them as the ``Witches of East Brunswick,'' the New Jersey town where two of them live.

``Having my husband burn alive in a building brought me no joy,'' Van Auken told the Daily News in Wednesday's editions in response to Coulter.

``She sounds like a very disturbed, unraveled person,'' Breitweiser said."

~~~~~~~~~~~~~~~~~~~~~~~~~~~~~~~~~~~~~~~~~~~~~~~~


Would you do or say anything despicable in order to make money???? Ann Coulter would. :)










tomder55 answered on 06/08/06:

I said on another post that I thought she was a flame thrower.She is Michael Savage with long, blonde hair and high heels. She is purposefully outrageous to attract attention so she sells books and gets TV appearences and speaking engagements.
Her point about the widows has some merit but certainly I would not phrase it the way she did . If I was a lib I would be as upset about it as conservatives should be when a NY State official jokes about shooting the President between the eyes likeNY state Comptroller Alan Hevesi did .( btw no one has asked him to resign of this . )

Personally I think the families of the 9-11 victims overall have too much influence. All you have to see is the lack of progress in the rebuilding of the WTC site to realize something is really wrong. The women in question have made some inflamatory statements of their own but they get a pass much like Cindy Sheehad because of their victimhood .Sorry ,they thrust themselves into the political arena using their victimhood as a prop .They picked this fight counting on their victim status to protect them from any response.It doesn't work that way . They hardly represent the majority of the families of 9-11 victims ,yet whenever someone like Chris Matthews needs an anti-Bush quote from them ,they are there for the calling .

In a simular vein ;I also think Tom Kean and Lee Hamilton should pack up their bags and go home with a thanks for your service and opinions .The 9-11 Commision fulfilled their mandate by making recommendations . That did not mean necessarily that they would be adopted completely or that theirs was the only reforms worthy of consideration . Some of what the commission suggested was wrong in my view .

jackreade rated this answer Average Answer

Question/Answer
jackreade asked on 06/07/06 - Copy of May 6 Cable from Ambassador Khalilzad to Condi

"Crime in Iraq is rated by the U.S. State Department as critical and will continue to get worse for the foreseeable future," the embassy in Baghdad reports in the cable, which was addressed to Secretary of State Condoleezza Rice .

"Crime, terrorism, and warfare are a significant threat in all parts of Iraq. Active military operations are ongoing. The Department of State continues to strongly warn U.S. citizens against travel to Iraq, which remains very dangerous. ****Remnants of the former regime, transnational terrorists, criminal elements and numerous insurgent groups remain active.***

"Attacks against military and civilian targets continue throughout the country, including inside the international zone. These attacks have resulted in deaths and injuries of American citizens. Planned and random killings are common as are kidnappings for ransom and political reasons."

"Overall security in Iraq is worsening," the embassy reports....." edited for length only.

~~~~~~~~~~~~~~~~~~~~~~~~~~~~~~~~~~~~~~~~~~~~~~~

Three years after the invasion of Iraq, and this is the truth about what is going on inside Iraq.

How can any sane person argue that the Iraq War has NOT been a collossal failure? A failure, in part, because of poor or no planning for after the highly tooted, "ahock and awe", the failure in not putting many more boots on the ground......

tomder55 answered on 06/08/06:

Ever read any of the human rights reports of Iraq before OIF ? Talk about disasters ! The difference of course is that the violence now is criminal and not state sanctioned.

oh ;and yes ..there is hope for a better future for the vast majority of Iraqis .

ETWolverine rated this answer Excellent or Above Average Answer
jackreade rated this answer Average Answer

Question/Answer
paraclete asked on 06/08/06 - Are we on the eve of WWIII?



WND Exclusive FROM WND'S JERUSALEM BUREAU
'We're on the eve of World War III'
Ex-Mossad chief urges West to unite, warns of Muslims imposing ideology
Posted: March 28, 2006
1:00 a.m. Eastern


2006 WorldNetDaily.com

JERUSALEM Global civilization is on the verge of "World War III," a massive conflict in which the Islamic world will attempt to impose its ideology on Western nations, according to Meir Amit, a former director of Israel's Mossad intelligence agency.

Amit, one of the most esteemed figures in the international defense establishment, warned Islamic nations and global Islamist groups will continue launching "all kinds of attacks" against Western states. He urged the international community to immediately unite and coordinate a strategy to fight against the "Islamic war."

"We are on the eve of war with the Islamic world, which will wage a war and all kinds of actions and attacks against the Western world. We already noticed the terrorists in the world hit Spain, England, France. I call it World War III. You must look at it from this angle and treat it wider, not as a problem of terrorism here and there," said Amit, speaking during an exclusive interview with WND's Aaron Klein and ABC Radio's John Batchelor broadcast on Batchelor's national program, for which Klein serves as a co-host. (Listen to the Amit interview.)

Amit served as Mossad chief from 1963 to 1968. He directed some of the most notorious Mossad operations during that time and pioneered many of the tactics currently used by intelligence agencies worldwide. The subject of multiple books and movies, Amit is routinely described as a "living legend." Now in his mid-80s, Amit serves as chairman of Israel's Center for Special Studies.

The former intelligence chief referenced recent terror attacks against Israel, Europe and the United States; Iran's alleged nuclear ambitions; the insurgency in Iraq and Afghanistan; and worldwide Muslim riots.

"It looks to me like it is a kind of coordinated or contemplated problem to somehow impose the Islamic idea all over the world," Amit said.

Israel is routinely attacked by Palestinian terror groups. Since December 2000, 993 Israelis have been killed. Spain in March 2004 was struck by a series of coordinated bombings on its commuter train system, killing 192 people. London was rocked last July by bombings on its transportation system. France has been the scene of violent Muslim riots and attacks. And on Sept. 11, 2001, 2,986 people were killed when the U.S. was hit with coordinated terror attacks.

Violent Muslim riots erupted last month in the West Bank, Syria and Lebanon after cartoon images of Muhammad were printed in a Danish newspaper. The riots spread across the Middle East and throughout Europe.

At least 40 people were killed yesterday in a blast north of Mosul in Iraq. Iran and Syria have been accused of aiding the insurgency there and in Afghanistan against U.S. and European troops.

Amit urged Western nations to "unite and work together. Unfortunately, the world is not uniting. China and Russia are problems. This should be taken into consideration."

Both China and Russia have been aiding Iran's nuclear program, which Tehran claims is intended for peaceful purposes only. Russia last month received a delegation of Hamas leaders, and pledged to maintain diplomatic relations with the terror group in spite of efforts by the U.S. and Israel to isolate the newly elected Hamas-led Palestinian government.

Amit said Iran currently poses the most serious threat to the international community.

"The Iranians [are] financing terrorists in Israel and sending money," Amit said. "This is [my country's] immediate problem. But I think the most serious problem is Iran developing nuclear power."

Amit said Israel should not lead a military attack against Iran's suspected nuclear facilities, instead urging support for the course of diplomacy and sanctions.

"The problem of [Iranian] nuclear armaments is not an Israeli problem; it is a worldwide problem. Your question refers to what Israel can do. It shouldn't do anything by itself. It should maybe throw the idea that this is a world problem and all the Western world should unite, join hands and work together," said Amit.

"I am not sure whether a military operation would be the best solution. At least not the first solution. But you can put sanctions on Iran."

With regard to his warnings of a new world war, Amit clarified he was not advocating the international community take measures against all Arab countries:

"I know very well the Arab world. I have many friends in Arab world leaders. Not all of them think the same. They are also split in different groups. ... Although I think they will wage an Islamic war against the Western world, we must take into account they are not one piece. Somehow we must learn the differences between different sections and parts of the Arab world."

Related special offers:

"The Politically Incorrect Guide to Islam (and the Crusades)"

"Mohammed: The Ugly Truth About the Founder of the World's Most Violent Religion"

tomder55 answered on 06/08/06:

WWIII has been underway for a while. We did not realize we were fighting it ;we thought we were dealing with criminal elements .

There is also a battle for the soul of Islam as Amit says ;and I will attempt to simplify the complex relationships .Prior to 9-11 there were jihadists threatening dictators throughout the ummah ;and dictators suppressing democratic reformists so they could claim to be the only alternative to the jihadists . We are betting that by introducing and supporting the democratic reformists that they can realize a third way . Both the jihadists and the dictators have a stake in opposing our effort.

Amit is correct that Iran is the next focal point .Not only because they finance the terror war on Israel but also because they influence and support along with al-Qaeda The Society of the Muslim Brothers in Egypt and Jordan . Yes it is true ;shia and sunni are not adverse to cooperating when they have a common goal ;and a world wide Califate is to them a worthy goal .

paraclete rated this answer Excellent or Above Average Answer

Question/Answer
purplewings asked on 06/08/06 - Notorious Beheader - Deader 'n a doorknob.

Terror Leader Al-Zarqawi Dies in U.S. Air Strike
By PATRICK QUINN, AP

BAGHDAD, Iraq (June 8) - Abu Musab al-Zarqawi, al-Qaida's leader in Iraq who led a bloody campaign of suicide bombings and kidnappings, has been killed in an air strike, U.S. and Iraqi officials said Thursday, adding his identity was confirmed by fingerprints and a first-hand look at his face. It was a major victory in the U.S.-led war in Iraq and the broader war on terror.





Will he receive 70 virgins in his heaven? Is it right for us to celebrate his demise? Should we declare a holiday?

tomder55 answered on 06/08/06:

what gave him away was that video of him with someone showing him how to fire an AK-47.

He will need 70 for from what I hear he is in 70 pieces.

ETWolverine rated this answer Excellent or Above Average Answer
Itsdb rated this answer Excellent or Above Average Answer
LTgolf rated this answer Excellent or Above Average Answer
purplewings rated this answer Excellent or Above Average Answer

Question/Answer
ETWolverine asked on 06/07/06 - An interesting political science excersize.

Hello all.

A few of you may have figured out from my screenname that I'm a comic book collector (alright I'm a geek, but I don't give a $h!t).

This summer, Marvel Comics is having an event called "Civil War" that will be running through several of the titles. Here's the concept:

The series will center upon a newly enacted Super-Human Registration Act, an act which splits notable superheroes within the Marvel Universe. This will result in two super-powered factions forming, and will build to the titular Civil War, into which themes from current events will be weaved, although writer Mark Millar has noted "The political allegory is only for those that are politically aware. Kids are going to read it and just see a big superhero fight."


According to the New York Times article on the story, "The story opens with a reckless fight between the New Warriors, filming a reality television show, and a cadre of villains. The battle goes awry for the heroes, resulting in villain Nitro creating an explosion that takes out all but one of the New Warriors, a local school, and the surrounding neighborhood. This event crystallizes a government movement to register all super-powered beings as living weapons of mass destruction.The subsequent Registration Act will divide the heroes into two camps, one led by Captain America, the other by Iron Man." Iron Man takes the side of supporting the Super-Human Registration Act, and Captain America will be against. Some people have already taken sides as well. In New Avengers: Illuminati, Mister Fantastic took Iron Man's side on the bill, but Doctor Strange was against it. Black Bolt's alliegence is still inconclusive, due to his inability to communicate vocally. Marvel has said that the Fantastic Four will be divided. All that is confirmed is that husband and wife will be divided and that Invisible Woman will be among those who are against the bill. Along the way, Marvel will unveil its version of Guantnamo Bay, enemy combatants, embedded reporters and more. The question at the heart of the series is a fundamental one: 'Would you give up your civil liberties to feel safer in the world?' "

The above was taken from a Wikipedia article on the Civil War storyline, citations eliminated.

So, here's the question: do you support the Superhuman Registration Act (SRA), or are you against it?

There are several arguments both for and against the SRA.

Pro:
1) Right now, there are too many people who are untrained in police activities who are making mistakes that could get themselves or others killed. The SRA calls for uniform training and oversight of anyone who wants to be a "costumed vigilante" so that they don't make those kinds of mistakes and risk hurting people... as the New Warriors did in their search for ratings for their reality TV show.

2) Cops and soldiers are trained and regulated. Why shouldn't superheroes be similarly trained and regulated if they are doing the job of cops and soldiers.

3) The rules would only apply to those who are actual superheroes. Those who have superpowers but who do not wear costumes and fight crime need not register (at least according to my understanding of the SRA). So there is no issue of profiling or rules that apply to a specific class or group of people. People can choose whether they wish to participate or not, but if they choose not to, any costumed superheroing they do would be illegal... the same as if you owned a gun illegally without registering it.

4) Government resouces and superheo talent could be properly pooled to help get the job of stopping supervillians done more efficiently. If superheroes had the NSA's electronic spying ability and the CIA and FBI's intelligence gathering capabilities at their beck and call, they could stop supervillians before they put their plots in motion.

5) Superheroes who sign in under the SRA will get government benefits and salaries like any government employee, which is good for the superheroes, especially those who are not well-off like Spiderman and Luke Cage.

Anti:
1) It is tradition fo costumed heroes to remain anonymous, even to the government. (This is the weakest of all the anti-SRA arguments in my opinion.)

2) As Captain America puts it, how long will it be before the government starts deciding who the badguys are? The point of the superhero is to be above political considerations of government agencies.

3) If all superheroes are registered, who will do the "black" stuff that the govenment can't acknowledge doing? The type of stuff that Wolverine does on his own? If all the heroes work for the government, disavowing them is not an option.

4) The SRA restricts the individual liberties of the heroes.

5) Although the identities of the heroes would be kept top secret, if someone got their hands on that information, it could endanger the families and friends of the heroes.

6) Working for the government would actually make their jobs harder: has there ever been a project that the government didn't complicate with red tape and politics and committee meetings?

And I am sure that there are other arguments on both sides of the issue that I haven't thought of or read yet.

What is your opinion on the matter? Is the SRA a good thing or a bad thing. The superheroes are split down the middle on the issue, and it is going to result, as the storyline's name makes clear, in a civil war among the superheroes.

As the tagline for the series says: "Which side will you choose?"

Elliot

tomder55 answered on 06/07/06:

I may be flip flopping on this one . I just read that part of the Super-Human Registration provision would be for them to reveal their alter-egos .That would be totally unacceptable .(Amazing Spiderman #529/30/31 ...He testifies before Congress )

ETWolverine rated this answer Excellent or Above Average Answer

Question/Answer
ETWolverine asked on 06/07/06 - An interesting political science excersize.

Hello all.

A few of you may have figured out from my screenname that I'm a comic book collector (alright I'm a geek, but I don't give a $h!t).

This summer, Marvel Comics is having an event called "Civil War" that will be running through several of the titles. Here's the concept:

The series will center upon a newly enacted Super-Human Registration Act, an act which splits notable superheroes within the Marvel Universe. This will result in two super-powered factions forming, and will build to the titular Civil War, into which themes from current events will be weaved, although writer Mark Millar has noted "The political allegory is only for those that are politically aware. Kids are going to read it and just see a big superhero fight."


According to the New York Times article on the story, "The story opens with a reckless fight between the New Warriors, filming a reality television show, and a cadre of villains. The battle goes awry for the heroes, resulting in villain Nitro creating an explosion that takes out all but one of the New Warriors, a local school, and the surrounding neighborhood. This event crystallizes a government movement to register all super-powered beings as living weapons of mass destruction.The subsequent Registration Act will divide the heroes into two camps, one led by Captain America, the other by Iron Man." Iron Man takes the side of supporting the Super-Human Registration Act, and Captain America will be against. Some people have already taken sides as well. In New Avengers: Illuminati, Mister Fantastic took Iron Man's side on the bill, but Doctor Strange was against it. Black Bolt's alliegence is still inconclusive, due to his inability to communicate vocally. Marvel has said that the Fantastic Four will be divided. All that is confirmed is that husband and wife will be divided and that Invisible Woman will be among those who are against the bill. Along the way, Marvel will unveil its version of Guantnamo Bay, enemy combatants, embedded reporters and more. The question at the heart of the series is a fundamental one: 'Would you give up your civil liberties to feel safer in the world?' "

The above was taken from a Wikipedia article on the Civil War storyline, citations eliminated.

So, here's the question: do you support the Superhuman Registration Act (SRA), or are you against it?

There are several arguments both for and against the SRA.

Pro:
1) Right now, there are too many people who are untrained in police activities who are making mistakes that could get themselves or others killed. The SRA calls for uniform training and oversight of anyone who wants to be a "costumed vigilante" so that they don't make those kinds of mistakes and risk hurting people... as the New Warriors did in their search for ratings for their reality TV show.

2) Cops and soldiers are trained and regulated. Why shouldn't superheroes be similarly trained and regulated if they are doing the job of cops and soldiers.

3) The rules would only apply to those who are actual superheroes. Those who have superpowers but who do not wear costumes and fight crime need not register (at least according to my understanding of the SRA). So there is no issue of profiling or rules that apply to a specific class or group of people. People can choose whether they wish to participate or not, but if they choose not to, any costumed superheroing they do would be illegal... the same as if you owned a gun illegally without registering it.

4) Government resouces and superheo talent could be properly pooled to help get the job of stopping supervillians done more efficiently. If superheroes had the NSA's electronic spying ability and the CIA and FBI's intelligence gathering capabilities at their beck and call, they could stop supervillians before they put their plots in motion.

5) Superheroes who sign in under the SRA will get government benefits and salaries like any government employee, which is good for the superheroes, especially those who are not well-off like Spiderman and Luke Cage.

Anti:
1) It is tradition fo costumed heroes to remain anonymous, even to the government. (This is the weakest of all the anti-SRA arguments in my opinion.)

2) As Captain America puts it, how long will it be before the government starts deciding who the badguys are? The point of the superhero is to be above political considerations of government agencies.

3) If all superheroes are registered, who will do the "black" stuff that the govenment can't acknowledge doing? The type of stuff that Wolverine does on his own? If all the heroes work for the government, disavowing them is not an option.

4) The SRA restricts the individual liberties of the heroes.

5) Although the identities of the heroes would be kept top secret, if someone got their hands on that information, it could endanger the families and friends of the heroes.

6) Working for the government would actually make their jobs harder: has there ever been a project that the government didn't complicate with red tape and politics and committee meetings?

And I am sure that there are other arguments on both sides of the issue that I haven't thought of or read yet.

What is your opinion on the matter? Is the SRA a good thing or a bad thing. The superheroes are split down the middle on the issue, and it is going to result, as the storyline's name makes clear, in a civil war among the superheroes.

As the tagline for the series says: "Which side will you choose?"

Elliot

tomder55 answered on 06/07/06:

you know which side Marvel is coming down on if Captain America is opposed .Which side is Hulk on ? That's my side !!! I haven't kept up .Is he still in Fantastic Four or Avengers ?

My own opinion : Are doctors registered ?Lawyers? Law Enforcement personel ? Even black ops forces are registered somewhere deep in the gvt. roles. Why not Super Heroes ? It is not like they have a history of proven loyalties . They change sides as often as WWE wrestlers. Heck ;1/2 the time you don't even know if a super hero isn't a super villian ;and sometimes like Spiderman they are perceived as villians even as they do good work (no wonder he is FOR registration ...it would help clarify any misconceptions about his status ) . What are they ;Vigilantes or mercenaries ready to fight for the highest bidder ? With registration at least you get a chance of knowing who is on your side .

ETWolverine rated this answer Excellent or Above Average Answer

Question/Answer
jackreade asked on 06/06/06 - Combating Racism

Some real ugly stuff is going on here, so I have to speak up.

I have known many fine middle class black people, professionals and CHRISTIANS.....many black working class people, CHRISTIANS, all kinds of African Americans.

I have known poor black people and poor white people....it's about surviving for poor people no matter what color.....


jack

tomder55 answered on 06/06/06:

it's about surviving for poor people no matter what color.....


I think a bigger problem is the lack of realization that in this country you can change your circumstances . Many people have... and if not for themsleves ,then they certainly don't confine their off-spring to their reality .

Itsdb rated this answer Excellent or Above Average Answer
jackreade rated this answer Poor or Incomplete Answer
kindj rated this answer Excellent or Above Average Answer
purplewings rated this answer Excellent or Above Average Answer
sissypants rated this answer Excellent or Above Average Answer

Question/Answer
kindj asked on 06/06/06 - Got time for a small rant?

So I'm sitting in my office, recovering (slowly) from a long night of tending to my sick wife, who was up all hours of the night with some stomach thing. My secretary beeps me to tell me my 9 o'clock appointment is here. Stifling the urge to swear out loud, I push my personal fatigue and concerns aside to tend to the business at hand.

In walks a young "gentleman" (a term I use very loosely), who happens to be African-American. I use that term simply because it's the current PC term of the hour. Whether or not the young man has ever even been to Africa is unknown to me, though after three minutes of conversation with him, I would be profoundly shocked if he could even locate Africa on a globe. But I digress. What really caught my attention was his T-shirt, which stated boldly with very well-done background graphics: "If you see the cops, warn a brother."

It took every ounce of self-control I possessed to not lay into this obviously misguided young man. By wearing statements such as these, aren't members of the black community advertising themselves as someone who NEEDS to be warned if the police are around? If I, a white American (oh, I could say "Irish-American," but never having lived in or even visited Ireland, that would be just silly) wore such a shirt or made such a statement, I would be immediately crucified by virtually everyone for being "racist." If I were conversing with this young man about the missing baby from our town (who has been found and is safe and healthy, thank God), and said something like "it was probably some black person that took her," would I not be labeled a racist? Sure I would, and rightfully so.

So is it just me, or does it seem like certain ethnic groups are their own worst enemies when it comes to race issues? In the circles where I live and have lived, which include military, blue collar, farming and ranching, law enforcement, faith communities, the "for profit" world, education, and institutes of higher learning, most of the white people I've ever talked to say that race is pretty much irrelevant to them, it's the quality of the person and their actions that matter. WHich sounds to me an awful lot like what Dr. King said in his most famous speech.

I personally think Dr. King would literally vomit if he saw how a great many black people were conducting themselves today while at the same time praising him and his actions, and continually shooting themselves in the foot.

Love me, hate me, disregard me. I don't care. That's what I have to say for the moment.

DK

tomder55 answered on 06/06/06:

Bill Cosby for one has been traveling around the country at his own expense in a series of discussions/lectures that he calls "Call outs" .His message has been the same throughout: social responsibility and careful attention to the behavior of young people by responsible adults."I have a problem with people sitting there and saying God and Jesus will find a way," Cosby said. He also said he had a problem "with churches who allow drug dealers to set up two blocks away." He also called on block women graduating from college to lead the way . He has emphasized the need for proper parenting and education as self-help answers for low-income, urban families.To those who say that he is preaching to the choir during these stops Cosby says ;'so does the preacher every Sunday '.


He more than any other black leader is carrying on the legacy of MLK .

CeeBee2 rated this answer Excellent or Above Average Answer
Itsdb rated this answer Excellent or Above Average Answer
kindj rated this answer Excellent or Above Average Answer
powderpuff rated this answer Excellent or Above Average Answer
purplewings rated this answer Excellent or Above Average Answer
sissypants rated this answer Excellent or Above Average Answer

Question/Answer
quixotic_Choux asked on 06/05/06 - Political Theater not Leadership

"Irony is hardly what the architects of the coming weeks of gay-bashing have in mind. Their stated goal is to draw a bright line between Us and Other, between God's faithful and Satan's secularists, between moral absolutes that must not bend, and moral relativists who must be bent.

The problem, though, is that there's hardly anyone in the audience who takes this staged witch hunt at face value.

The White House, and Republican Senators, aren't dewey-eyed. They know this is political theater. They fully embrace the cynical motivation behind it: an attempt to turn the nation's attention to the homo horror show. Iraq, energy, health care, global warming -- who wouldn't be delighted to avoid accountability for a disgraceful record of neglect by playing pin the tail on the pansy?

And yet most members of Bush's own base also know the score. They know this isn't the real culture war; it's a culture war game. They rightly see this amendment hoopla as a charade, an attempt to pander them back into the fold, a half-hearted bit of pre-election showmanship aimed at diverting them from their anger about deficits, spending, corruption and incompetence.

Younger Americans, too, are largely immune from this spectacle. Most people under 40 think love-and-let-love is the right reaction to same sex marriage, and they don't see the problem with gay couples getting the same civil rights that married spouses have.

So it turns out that there's practically nobody left in the audience for these pious solons to play to. Everyone's hip to the con; everyone's backstage, behind the curtain, eye-to-eye with Oz. Rather than putting on a grand national affirmation of traditional values, our own Americn Oberammergau, the Republican right is instead producing the ultimate postmodern wink-wink. The only people these posturers are really playing to are themselves.

And, of course, to homophobes, whose anger this phony passion play will undoubtedly arouse. It may be only a culture war game to preening pomos like Rove and Frist, but to its targets, it's no game. That's not just stage blood on Washington's hands." by Marty Kaplan, blogger

~~~~~~~~~~~~~~~~~~~~~~~~~~~~~~~~~~~~~~~~~~~~~~~~

Do you think Bush's political game will get out the right wing homophobes in November and lead to some or many Republican victories?

If a homosexual(s) is/are killed by riled up extremists; are those in the Bush Administration who approved this approach, guilty of a terrible sin?

tomder55 answered on 06/06/06:

To describe as pandering to radical those who wish to preserve the man-woman-based definition of marriage known to every civilization is to stand the word on its head. It is beyond intellectually dishonest to describe those who favor preserving the definition of marriage as "radical" rather than to so describe those who wish to change the gender-based definition of marriage for the first time in history. Even if you support same-sex marriage, you should at least have the honesty to admit that it is you who favors something radical. Go ahead ;put it up to the ballot . Across the board State by State an initiative supporting gay marriage gets overwhelmingly defeated .

This is the constitutional way to go about this issue. Too bad that we can't trust that the law of the land (DOMA ) will not be oveturned by the oligarchs in black robes or there would be no talk of a marriage amendment . I support a federalist approach . Too bad the judiciary doesn't .

In my view Michael Greve of the American Enterprise Institute has a much better plan . He says that the DOMA should be submitted as the Constitutional Amendment with an adenum that the definition of marriage shall be determined by each state legislature, or, by a statewide referendum. And it could be written in such a way to make sure that a court cannot interpret the law so as to require a state to legalize same-sex marriage. Imagine all this extra effort just to protect ourselves from judicial tyranny .

I got an idea . If the homosexual advocates are so confident that they are right in their position then why not propose an amendment that guarantees their so called right to marriage ?

quixotic_Choux rated this answer Poor or Incomplete Answer

Question/Answer
paraclete asked on 06/04/06 - Is the UN complicit in murder?

Cover-up claims after UN order


June 4, 2006 - 5:03PM


* Street gangs defy foreign troops
* Nelson warns against Timor becoming failed state
* Downer issues warning on Alkatiri resignation
* Rebel leader's wife 'flees to Perth'
* UN must 'commit more to East Timor'

The United Nations has ordered staff in East Timor not to cooperate with Australian Federal Police investigating the massacre of 12 unarmed Timorese officers by renegade soldiers, prompting allegations of a cover-up.

A letter from the UN's deputy representative in Timor, Pakistani General Anis Bajwa, had been circulated to all staff, including employees evacuated to Australia, directing them not to assist AFP detectives investigating the worst atrocity since the violence of 1999.

A copy of the letter had been passed to Australia's Embassy in Dili, outraged diplomats and AFP sources confirmed to AAP.

However a UN spokeswoman denied the existence of the letter.

"There was no letter and the United Nations is perfectly willing to cooperate with an investigation as soon as we are requested to by the East Timorese government," UN spokeswoman in Dili Donna Cusumano said.

Australian Foreign Minister Alexander Downer yesterday met senior UN officials and East Timorese President Xanana Gusmao to urge full cooperation with the investigation into the murders, which took place on May 25 outside the UN police headquarters.

Mr Downer urged the need for a "thorough investigation" by the AFP.

A senior adviser to East Timorese Foreign Minister Jose Ramos Horta today told AAP his country's government had pledged a full investigation, handing the job to the AFP to ensure no-one interfered or hid any facts.

The massacre of the East Timorese officers occurred after renegade soldiers opened fire on the police as they left their headquarters under a truce brokered by the UN's police commander in Dili, Saif Malik, also from Pakistan.

Malik ignored advice that to lead unarmed Timorese police past the guns of the soldiers would lead to a massacre.

"He was told by all his advisers not to take them out there, but he would not listen," sources close to the investigation said.

"He kept insisting the presence of the UN could protect them."

Instead the police were escorted out behind a blue UN flag and got less than 100 metres from their headquarters when two soldiers opened fire, killing 12 officers and wounding at least 20, including UN police protectors from the Philippines.

The soldiers executed several wounded police at point-blank range, firing shotguns into their heads, sources within the AFP said.

"It looks like the UN or Malik or someone wants to bury it all now," one investigator said.

Investigators are studying photographs apparently taken during the shootings.

Junior inspector Felner Cortereal was shot three times but survived the massacre, and was recovering today at Dili's main hospital.

His wife, Madalena Sanches, was shot in the back and stomach, and almost died while being evacuated to Darwin.

Cortereal confirmed UN police assured the terrified Timorese officers they would be safe as they emerged to find three soldiers with "big guns" facing them to the left and right.

"They (the UN) said: 'Just relax, they won't shoot, because we have negotiated with them,'" he said.

"They said it had worked in other countries.

"Maybe they believed that principle, but we were not confident."

The death toll could have been even higher, he said, except the soldiers had fired wildly, spraying shots in all directions.

Cortereal said the atrocity should not be pushed aside to protect the UN's reputation.

"For me now, we must get to the roots of this, find out why it happened," he said.

AAP

tomder55 answered on 06/04/06:

Such is the peril when you rely on blue helmets for peace-keeping . Recall all the boasting at the UN about E.Timor being a model of a successful UN intervention . But like most of their efforts ,it was a Potamkin Village.I have heard many stories recalled by the IDF that they knew when Hezzbolla was going to launch an attack because the UN peace-keepers would get into their SUVs and evacuate the area prior to the attack.As often happens the UN troops are armed but lack the will to use them .

There is no charge against the UN that suprises me .

I share Australian Defense Minister Brendan Nelson's concern that E.Timor could become a failed State and a terrorist haven .Being as close as the island is to Australia makes it a matter of national importance to you ;just like having Cuba a threat 90 miles off the US is a threat to us.Nelson, said a broader coalition could be built if the East Timorese government asked for it.I say however that a coalition should be built whether they ask for it or not. The US would willingly help if we were asked ;at least Rummy indicated that at the Singapore Summit .So far our assistance has been modest and there are not sufficient Australian forces there to guard a rice factory let alone maintain order .

Australia urged the United Nations to take a greater role. I say tell the blue helmets to stay home. Just like during the Tsunami in 2004 ;they are always a day late .

paraclete rated this answer Excellent or Above Average Answer

Question/Answer
jackreade asked on 06/03/06 - Hillary Easily a Loser for Reelection

"When I launched my primary Senate challenge against Hillary Clinton, I always believed that we could rock the political establishment IF the race focused on the issues. And this week proved my point.

John Zogby polled New York State voters and found that if the race came down to a choice between Hillary Clinton and an unnamed anti-war candidate, 38 percent of the voters would choose the incumbent, 32 percent would pick the anti-war candidate and the rest of the voters said they werent sure or would vote for someone else.
So, two-thirds of the voters are ready to vote against the incumbent because of her support for the war.

This is not a surprise. My opponent is the best political example of the worst about celebrity culture: poll ratings based on name recognition and celebrity even as people know very little about where she stands, particularly on the war. I cant tell you how many voters I have talked to in the past six months who have said, no, she cant be for the war (yes), she couldnt possibly support the death penalty (she does), she cant really believe NAFTA was a good thing (yup), she cant be for making it a crime to burn the flag because thats a First Amendment right (sorry, but yes) and it cant be that Rupert Murdoch is holding a fundraiser for her (want a ticket?)."

by Jonathan Tasini, blogging.

~~~~~~~~~~~~~~~~~~~~~~~~~~~~~~~~~~~~~~~~~

Hillary can easily go down in flames because of her stand on the Iraq War in November.

I will be glad to see her out of the picture, will you?

tomder55 answered on 06/03/06:

Her thighness will not have to work up a sweat to win her reelection bid . The Republicans are likely to put on the ballot to contest her "to be named later " . Such is the pathetic state of the NY Republican party .

The posting is more appropriate if she were to attempt a run at the Presidency . I can only hope that the Democrats try to find the most anti-war candidate available . But more likely they will come to their senses and pick someone who will argue that he/she can fight the war better .Of course the flaw in that thinking is that they will not be able to come up with a course /strategy much different and certaily no better than the current Bush doctrine .

jackreade rated this answer Excellent or Above Average Answer
ETWolverine rated this answer Excellent or Above Average Answer

Question/Answer
paraclete asked on 06/03/06 - The dirty war?

Iraqi Assails U.S. for Strikes on Civilians


By RICHARD A. OPPEL Jr.
Published: June 2, 2006

BAGHDAD, Iraq, June 1 Prime Minister Nuri Kamal al-Maliki lashed out at the American military on Thursday, denouncing what he characterized as habitual attacks by troops against Iraqi civilians.


As outrage over reports that American marines killed 24 Iraqis in the town of Haditha last year continued to shake the new government, the country's senior leaders said that they would demand that American officials turn over their investigative files on the killings and that the Iraqi government would conduct its own inquiry.

In his comments, Mr. Maliki said violence against civilians had become a "daily phenomenon" by many troops in the American-led coalition who "do not respect the Iraqi people."

"They crush them with their vehicles and kill them just on suspicion," he said. "This is completely unacceptable." Attacks on civilians will play a role in future decisions on how long to ask American forces to remain in Iraq, the prime minister added.

The denunciation was an unusual declaration for a government that remains desperately dependent on American forces to keep some form of order in the country amid a resilient Sunni Arab insurgency in the west, widespread sectarian violence in Baghdad, and deadly feuding among Shiite militias that increasingly control the south.

It was also a sign of the growing pressure on Mr. Maliki, whose governing coalition includes Sunni Arabs who were enraged by news of the killings in Haditha, a city deep in Sunni-dominated Anbar Province. At the same time, he is being pushed by the Americans to resolve the quarreling within his fragile coalition that has left him unable to fill cabinet posts for the Ministries of Defense and the Interior, the two top security jobs in the country.

Military and Congressional officials have said they believe that an investigation into the deaths of two dozen Iraqis in Haditha on Nov. 19 will show that a group of marines shot and killed civilians without justification or provocation. Survivors in Haditha say the troops shot men, women and children in the head and chest at close range.

For the second day in a row, President Bush spoke directly about the furor surrounding the case. "Obviously, the allegations are very troubling for me and equally troubling for our military, especially the Marine Corps," President Bush said Thursday, in response to a question from a reporter after a meeting of his cabinet. Referring to the chairman of the Joint Chiefs of Staff, Gen. Peter Pace, he added, "I've spoken to General Pace about this issue quite a few times."

Investigators are examining the role of senior commanders in the aftermath of the Haditha killings, and trying to determine how high up the chain of command culpability may rest.

Marine officials said Thursday that Maj. Gen. Stephen T. Johnson, who was the top Marine Corps commander in Iraq during the Haditha killings, had been set to be promoted to become the service's senior officer in charge of personnel, a three-star position.

General Johnson is widely respected by the Marine Corps' senior leadership, yet officials said it was unlikely that the Pentagon would put him up for promotion until the Haditha investigations were concluded.

The Washington Post reported Thursday that a parallel investigation into whether the killings were covered up has concluded that some officers reported false information and that superiors failed to adequately scrutinize the reports about the two dozen deaths.

The newspaper said that the inquiry had determined that Staff Sgt. Frank Wuterich, a squad leader present at Haditha, made a false statement when he reported that a roadside bombing had killed 15 civilians. The inquiry also said that an intelligence unit that later visited the site failed to highlight that civilians had gunshot wounds.

In Baghdad, senior Iraqi officials demanded an apology and explanation about Haditha from the United States and vowed their own inquiry.

"We in the ministers' cabinet condemned this crime and demanded that coalition forces show the reasons behind this massacre," Deputy Prime Minister Salam al-Zubaie, one of the most powerful Sunni Arabs in the new government, said in an interview.

It seems the US has lost the war for the hearts and minds of the People in Iraq?

tomder55 answered on 06/03/06:

well good . I think they would not be doing their jobs as Iraqi leaders if they did not take this posture. I think we ought to cooperate with them in any investigations as well as conduct thorough investigations on our own .

Your closing comment/question is not based on any of the facts presented in the article nor do you support it with any additional facts .I for one am more concerned about the hearts and minds of the American people .Two major military reports will come out soon on the Haditha incident, and no one will deny that justice need be served if that is required. But the atmosphere around this event is going to get uncontrollably manic, and that will feed the dark, retreat to fortress America sentiments already poisoning the country's mood. This will not bode well because the jihadists will not give up so easily . Also all the pet humanitarian missions will also be abandoned or dismissed before being attempted . The military is very correct when they say that 99.9% serve admirably . It is a testiment to Americas lack of will that the 0.1% can break it so easily . The Iraqi people know better .

paraclete rated this answer Excellent or Above Average Answer
excon rated this answer Excellent or Above Average Answer

Question/Answer
jackreade asked on 06/02/06 - Sexual Bombshell Hits Washington

It has been announced today from two reliable sources that George Bush is having a sexual affair with Condoleeza Rice.

I'm trying to remember, when was the last time a sitting president had a mistress. John F. Kennedy?

tomder55 answered on 06/03/06:

here is the slander piece of crap :

June 2, 2006 -- A White House source, speaking on background, vehemently denied to WMR that there are marital problems between President Bush and First Lady Laura Bush over a reported extramarital affair between Mr. Bush and Secretary of State Condoleezza Rice. However, two mainstream media sources have confirmed that their sources also have reported an ongoing affair between Mr. Bush and Rice.

The mainstream media is hamstrung in reporting stories about Bush's personal life. For example, in 2001, the media highlighted Bush's comments about his passing out from choking on a pretzel while watching a football game in the White House. In reality, Bush, who claims he gave up drinking years ago, passed out from being inebriated. Washington's movers and shakers knew the story about Bush's drinking but the media studiously avoided it.
.....................................
Not only does the coward not identify his sources but then this bogus source will not identifiy the secondary source. This reoprt is not worthy of the National Inquirer. In this country it is hard for a public person to sue for libel but if ever there was a case for it ...here it is .

jackreade rated this answer Excellent or Above Average Answer
purplewings rated this answer Excellent or Above Average Answer

Question/Answer
excon asked on 06/02/06 - Queer Marriage


Hello wingers:

(1) Do you think an amendment to the Constitution to ban gay marriage is a good thing? No need to say why. If you say yes, I know why. Just as Ill know why you dont support it. (2) Do you think it will actually become part of the Constitution? (3) Is this an attempt to better our country or to pander to the religious right? (4) If it has no chance to become law, why do you think it is being proposed? (5) Do you know how an amendment becomes part of the Constitution?

excon

tomder55 answered on 06/02/06:

the whole idea of amending a constitution is to change it . Therefore if an amendment to ban hand guns garnered enough support at the Federal level to get ratification then bingo ... handguns are now banned no matter how you read the 2nd amendment . Want proof ? Amendments were written into the constitution to both ban alcohol and to repeal the ban. When the ban was in place it was a constitutional ban .Simularily the 14th amendment could be repealed (no I do not advocate that ).

Now you may argue that the Georgia ban violated in some measure that I am unaware of the Federal Constitution but by the very amendment process it was impossible to be against the Georgia constitution . The judge who made that ruling is plain wrong .

How can such a restriction be against the equal protection clause ? If it is ,then so is bans on polygamy ,and incest and possibly age limits to marriage . Marriage by definition is restrictive and even those arguing to include same sex marriages are limiting the scope of marriage .

excon rated this answer Excellent or Above Average Answer

Question/Answer
excon asked on 06/02/06 - Queer Marriage


Hello wingers:

(1) Do you think an amendment to the Constitution to ban gay marriage is a good thing? No need to say why. If you say yes, I know why. Just as Ill know why you dont support it. (2) Do you think it will actually become part of the Constitution? (3) Is this an attempt to better our country or to pander to the religious right? (4) If it has no chance to become law, why do you think it is being proposed? (5) Do you know how an amendment becomes part of the Constitution?

excon

tomder55 answered on 06/02/06:

I'll answer the last first . yes I know how an amendment becomes law . It is not intended to be an easy process .

I do not favor a marriage amendment and yes it is as political as the 'equal rights amendment ' which btw would've mandated same sex marriage recognition. That amendment still lingers in the system even though it supposedly had a deadline for ratification. It's status is that it needs only 3 more State Legislaures to ratify it before it becomes the law of the land . That is the problem with amendments proposals . They can linger in the process for years . The 27th amendment was ratified 200 years after it was proposed .

I think amending the Constitution should be reserved for issues that cannot be resolved through the federal system.The President did bring up the 'full faith and credit clause ' when he argued in favor of the amendment and that I agree could be an un-resolvable constitutional issue . The one argument against that proposition is that States can refuse to recognize a marriage if the marriage violates a strong public policy of the state (the "public policy exception" ). It has historically been used against under-aged marriages ;poligamy ;and incestuous marriages .The precedent is there for a State to not recognize a same-sex marriage regardless of the 'full faith and credit' clause.

The same -sex marriage proponents have brought this situation to where it currently is . The reason opponents are going the amendment route is because there is a lack of trust in the courts . In the example mention above ,Federal courts have interpreted the U.S. Constitution to place some limits on states' ability to restrict access to marriage. Recently a federal judge struck down an amendment to Nebraskas constitution that prohibited the state from granting legal protections to any same-sex relationship similar to marriage (http://www.nebar.com/pdfs/DCOpinPDFs/4-03cv3155.pdf ) . Again the imperial court overstepped it's authority by denying the state it's constitutional right to regulate marriage .That is why many feel that an amendment is the only real protection that traditional marriage has .

excon rated this answer Excellent or Above Average Answer

Question/Answer
kindj asked on 06/01/06 - Marines at Haditha--by someone who was there

A reporter's shock at the Haditha allegations
By Arwa Damon
CNN


Wednesday, May 31, 2006; Posted: 9:02 p.m. EDT (01:02 GMT)


BAGHDAD, Iraq (CNN) -- It actually took me a while to put all the pieces together -- that I know these guys, the U.S. Marines at the heart of the alleged massacre of Iraqi civilians in Haditha.

I don't know why it didn't register with me until now. It was only after scrolling through the tapes that we shot in Haditha last fall, and I found footage of some of the officers that had been relieved of their command, that it hit me.

I know the Marines that were operating in western al Anbar, from Husayba all the way to Haditha. I went on countless operations in 2005 up and down the Euphrates River Valley. I was pinned on rooftops with them in Ubeydi for hours taking incoming fire, and I've seen them not fire a shot back because they did not have positive identification on a target.

I saw their horror when they thought that they finally had identified their target, fired a tank round that went through a wall and into a house filled with civilians. They then rushed to help the wounded -- remarkably no one was killed.

I was with them in Husayba as they went house to house in an area where insurgents would booby-trap doors, or lie in wait behind closed doors with an AK-47, basically on suicide missions, just waiting for the Marines to come through and open fire. There were civilians in the city as well, and the Marines were always keenly aware of that fact. How they didn't fire at shadows, not knowing what was waiting in each house, I don't know. But they didn't.

And I was with them in Haditha, a month before the alleged killings last November of some 24 Iraqi civilians.

I'm told that investigators now strongly suspect a rampage by a small number of Marines who snapped after one of their own was killed by a roadside bomb.

Haditha was full of IEDs. It seemed they were everywhere, like a minefield. In fact, the number of times that we were told that we were standing right on top of an IED minutes before it was found turned into a dark joke between my CNN team and me.

In fact, when we initially left to link up with the company that we were meant to be embedded with, the Humvee that I was in was hit by an IED. Another 2 inches and we would have been killed. Thankfully, no one was injured.

We missed the beginning of the operation, and ended up entering Haditha that evening. The city was empty of insurgents, or they had gone into hiding as they so often do, blending with the civilian population, waiting for U.S. and Iraqi forces to sweep through and then popping up again.

But this time, after this operation, the Marines and the Iraqi Army were not going to pull out, they were going to set up fixed bases.

Now, all these months later, while watching the tapes, I found a walk and talk with one of the company commanders that was relieved of his duty as a result of the Haditha probe.

After being hit by an IED, his men were searching the area and found a massive weapons cache in a mosque. Although it wasn't his company that we were embedded with, the Marines had taken me to the mosque so we could get footage of the cache.

And so began the e-mails and phone calls between myself and my two other CNN crew members, Jennifer Eccleston and Gabe Ramirez: Do you remember when we were talking with the battalion commander and his intel guy right outside the school and then half an hour later they found an IED in that spot? Do you remember when we were sitting chatting with them at the school? And all the other "do you remember whens."

There was also -- can you believe it? -- the allegations of the Haditha probe.


A little dose of reality to mix with the specualtion and rhetoric. Wait to pass judgement, please.
~~~~~~~~~~~~~~~~~~~~~~~~~~~~~~~~~~~~~~~~~~~~~~~~~~~

Words of wisdom from a CNN reporter? I do believe I might faint....

DK

tomder55 answered on 06/02/06:

Marine 2nd Lt. Ilario Pantano was charged with murder for killing two jihadists who had just fled a bomb-making house south of Baghdad. After the press had its fun, the Marines dropped all charges, finding him guilty only of defending his men.

A Marine charged with shooting an apparently injured and unarmed Iraqi in a Fallujah mosque that had been used as a sanctuary and an armory for jihadists did not face court-martial. The Marine was found to be justified in perceiving a threat and using deadly force.

Seven British soldiers were charged with murder stemming from a May, 2003 incident in Ferkah, Iraq. The trial collapsed in November 2005 after it became clear that many of the key Iraqi witnesses were lying in order to gain blood money .
Three women admitted lying about being assaulted by British soldiers .One witness at the court martial, Samira Rishek, a Marsh-Arab who had claimed to have been brutally beaten by the soldiers while she was pregnant, admitted to the court it was a wicked lie". The court heard Rishek, along with other witnesses, was paid $100 a day to give evidence at the trial and that she only agreed to give evidence after being told she would be paid.


The case against the British troops fell apart after 29 months and the Brits spent $18 million in the investigation. During the interval the press had an open field day and the presumption of guilt was the rule .


Pantano spoke out about Murthas rush to judgment in the Haditha case in a letter to the editor of the Washington Compost . Members of the U.S. military serving in Iraq need more than Mr. Murtha's pseudo-sympathy. They need leaders to stand with them even in the hardest of times. Let the courts decide if these Marines are guilty. They haven't even been charged with a crime yet, so it is premature to presume their guilt -- unless that presumption is tied to a political motive.


Well said !Safa Younis Salim is the sole survivor of this incident and so far I have heard 4 different versions of her account.


This is what I know . The Marine investigators have been collecting forensic evidence. There is plenty of it remaining . Bullets that went through bodies embedded into walls are being collected . The rifling marks of the bullets will tell which gun was used ,and the dna will tell which body the bullet passed through . Each Marine is assigned a specific gun so if they were involved in the incident then the evidence will point directly to them.

The Compost also says that a drone was video taping but it is not clear that it captured the incident on tape . They also say there is an audio of radio message traffic between the Marines and a command center that according to a defense lawyer confirms that the Marines came under small fire attack as thety claim.

Last month, the Marines relieved of command the battalion commander, Lt. Col. Jeffrey R. Chessani, as well as McConnell and another company commander, citing a lack of confidence in their leadership.
The suspension of a promotion to the top Marine General in Iraq at the time ;Maj. Gen. Stephen Johnson ;leads me to believe that if there was a coverup that it went probably at least to battalion level maybe higher . But I have no facts to back that up ;just speculation .

kindj rated this answer Excellent or Above Average Answer

Question/Answer
paraclete asked on 06/02/06 - Let's see how I can do on this one?

1) Bush intentionally misled Americans to convince them to go to war in Iraq.

True.

2) Bush pressured intelligence agencies to bias their judgments.

True

3) Promoting democracy in the Middle East is a postwar rationalization.

True

4) Saddam's WMD's were the only justification for war with Iraq.

True

5) Because no WMD's were found, Saddam posed no threat.

True

6) Saddam's WMD's have been accounted for.

False.

7) The war in Iraq was all about oil.

True.

8) The US and the CIA 'created' Osama bin Laden.

True

9) "George Bush doesn't care about black people".

True

10) Liberals are more tolerant than conservatives.

False

Generally speaking this is what the world believes with or without justification.

tomder55 answered on 06/02/06:

lets see (if I was a moonbat I would say )

11) Australia went to E. Timor for humanitarian reasons .

FALSE they are concerned about oil rights in the sea between E.Timor and Austraila .

12) Australians are being greeted as liberators in East Timor.

FALSE .Gangs are hurling rocks from sling shots at the Australian liberators

13 ) East Timor was better off under the Indonesian dictatorship

TRUE Since they were "liberated " they have a failed state with competing violent factions that have infiltrated the security apparatus and institutions .

what the heck .who cares if the facts speak of a different truth . it's perceptions that matter ...right ?

paraclete rated this answer Excellent or Above Average Answer

Question/Answer
Itsdb asked on 06/01/06 - Another pop quiz

True or false again - and please explain your answer.

1) Bush intentionally misled Americans to convince them to go to war in Iraq.

2) Bush pressured intelligence agencies to bias their judgments.

3) Promoting democracy in the Middle East is a postwar rationalization.

4) Saddam's WMD's were the only justification for war with Iraq.

5) Because no WMD's were found, Saddam posed no threat.

6) Saddam's WMD's have been accounted for.

7) The war in Iraq was all about oil.

8) The US and the CIA 'created' Osama bin Laden.

9) "George Bush doesn't care about black people".

10) Liberals are more tolerant than conservatives.

tomder55 answered on 06/01/06:

1. nonsense . why would he do that if once we are there the whole cover would be blown ?

2. yeah that's the ticket . he has proven how much control he has over the intel. agencies by how many times they go whining to the press .

3.not true. for those who complain that this was a neo-con coup ;the PNC was an advocate for regime change to promote democracy throughout the 1990s . Promoting democracy has been a keynote strategy in the war against jihadistan since Bush detailed it long before OIF .

4. there was a litany of reasons given to go to war stated repeatedly by both the Administration AND in the war power resolution that Congress passed . Did they not read the bill they passed ?

5. because Saddam was removed from power and is on trial for Mass murders he is no longer a threat.

6. not definitely . Recent released documents and translations of tapes Saddam made clearly point to a transfer of WMD out of the country conducted with the help of the Russian Spetsnaz.

7.obviously . you can tell bty the $70 + per barrel we spend on a barrel today .

8.OBL was born into a rich family with close ties to the Saudi royals . The charge that we funded OBL when we funded the mujahideen is a patent destortion of the facts . Yes we funneled aid to Afghan fighters but not to Arab jihadists . They were funded by fellow arabs .Former CIA official Milt Bearden, who ran the Agency's Afghan operation in the late 1980s, says, "The CIA did not recruit Arabs," as there was no need to do so. There were hundreds of thousands of Afghans all too willing to fight, and the Arabs who did come for jihad were "very disruptive . . . the Afghans thought they were a pain in the ass." Ayman al-Zawahiri, confirmed that the "Afghan Arabs" did not receive any U.S. funding during the war in Afghanistan in his book "Knights Under the Prophet's Banner" .THE US NEVER HAD A RELATIONSHIP WITH OBL OR AL-QAEDA !!! PERIOD !!

After Iraq invaded Kuwait in 1990, bin Laden offered to help defend Saudi Arabia (with 12,000 armed men) but was rebuffed by the Saudi government. The presence of US troops in Saudia Arabia was his chief justification for war against the US .

9. too rediculous a statement to acknowledge .

10. ditto

ETWolverine rated this answer Excellent or Above Average Answer
Itsdb rated this answer Excellent or Above Average Answer

Question/Answer
ETWolverine asked on 06/01/06 - I'm angry about this one...

The Department of Homeland Security slashed NY's terrorism security budget by 40%, while increasing the funding to such "high risk" areas as Jacksonville, FL, St. Louis, MO, Milwaukee, WI, Louisville, KY, and Omaha, NE. Their official reasoning in doing so was the lack of landmarks in NYC, as compared to the "higher risk" areas.

Huh?

Let's see here:

City: Jacksonville, FL,
Major Landmarks: Alltell Stadium, home of the Jacksonville Jaguars. (Yeah, there's a popular winning team for you.)

City: St. Louis, MO,
Major Landmarks: Gateway Arch (We have two of them over every McDonalds.)

City: Milwaukee, WI,
Major Landmarks: a bunch of beer breweries (and not much else).

City: Louisville, KY,
Major Landmarks: Churchill Downs Racetrack (and aside from the Kentuky Derby, the Kentuky Oaks and Breeders Cup meets, what exactly do they do with this "major landmark" the rest of the year?)

City: Omaha, NE.
Major Landmarks: Offut Air Force Base (Are they seriously telling us that a military base needs outside funding to provide security?)

And then there's New York:
Major Landmarks:
George Washington Bridge
Brooklyn Bridge
Manhattan Bridge
Verrazano-Narrows Bridge
Brooklyn Battery Tunnel
Holland Tunnel
Lincoln Tunnel
Penn Station
Grand Central Station/Grand Central Terminal
The MTA Subway System
Liberty Island/Statue of Liberty
Ellis Island
United Nations complex
New York Stock Exchange
American Stock Exchange
NASDAQ
Governor's Island
Lincoln Center
MOMA
Metropolitan Museum of Art
Guggenheim Museum
American Museum of Natural History/Hayden Planetarium
Whitney Museum of American Art
Intrepid Sea-Air-Space Museum
Saint Patrick's Cathedral
the Cathedral of St. John the Divine
Riverside Church
Temple Emanu-El (wouldn't that be a juicy target for Muslim terrorists)
Jewish Museum (another juicy target for Islamic terrorism)
Central Park
Rockefeller Center
Times Square
Herald Square
The Empire State Building
The Freedom Tower rebuilding site/World Trade Center/ World Trade Center Memorial
Yankee Stadium
Shea Stadium
Arthur Ashe Stadium
South Street Seaport
Carnegie Hall
Gracie Mansion
New York Public Library
Jacob K. Javits Convention Center

And many, many more.

The idea of cutting the security/anti-terrorism funding of a major city that has been the victim of two major terrorist attacks in the past 15 years is just rediculous, considering how target-rich this city is... with mostly "soft" targets. And to compound it with even more stupidity by increasing the funding to cities that have no serious targets of note, and state that NYC doesn't have any targets... that's just going overboard on the stupidity.

I don't agree with Chuck Schumer very often, but I agree with him on this one... this situation is just shockingly stupid. What the hell could Chertoff be thinking?

Elliot

tomder55 answered on 06/01/06:

Washington DC is also seeing a reduction in DHS spending.Amazing ; The two cities hit on 9-11 and most likely to ge thit again !

I have to admit ;I am getting pretty tired of Chertoff's act . He has not been effective as DHS Chief in a number of cases including Katrina response and the border issue .

The whole idea of the executive dept. having control of the allocations was to avoid the very type of pork barrel politics that seems to be in play here.

Not only in NYC and DC where this funding cut is completely unacceptable but also in other major centers like San Diego where the border issue plays a major role ,and there is also a huge Navy base ,funding cuts will be draconian .Texas also took a big hit which makes me really wonder about DHS commitment to border security.

I guess they figured that the big bulls-eye pained on a NYC street map by OBL has been replaced by smaller ones in places like Moose Breath Montana .

ETWolverine rated this answer Excellent or Above Average Answer
powderpuff rated this answer Excellent or Above Average Answer

Question/Answer
Itsdb asked on 05/31/06 - Poor baby...

Ted Rall has an opinion piece today entitled The 10,000th Haditha. Ignoring some of his more offensive statements for the moment, he said:

    Those of us who raised our voices against this war from the start, having fruitlessly complained about stories of battlefield abuse reported by the European media, are suffering from marginalization fatigue.


Am I supposed to feel sympathy for poor Ted, the guy who after the death of Ronald Reagan claimed Reagan is "turning crispy brown right about now"?

Whose fault is it that people like Ted might be "suffering from marginalization fatigue"? Is it Generalissimo El Busho's fault, or could the fact that Rall is selling "Hey Bush: "Get Out of Al Gore's House!" postcards six years after the election be an indication of something?

tomder55 answered on 06/01/06:

There is a huge disconnect here if he thinks the media has suppressed anti-war coverage and news . Why is this story not a bigger story in the Middle East ? True or not (and at this point I believe most of the reporting as accurate ),it has to be weighed increasingly against the barbarity of the jihadists . Michelle Malkin has a pretty good column on Hadith that also speaks to morons like Rall .

There are countless numbers of anti-war zealots on the American Left rooting for failure. They believe the worst about the troops. They've blindly embraced frauds who've lied about their military service and lied about wartime atrocities. They've allied themselves with socialist kooks and coddled murderous dictators. They are looking for any excuse to pull out, abandon military operations and reconstruction, and impeach the president.
They insist on giving suspected foreign terrorists more benefit of the doubt than our own men and women in uniform. And that, I know, I am not willing to do.

I will wait. I will pray. And I will remind you that while the murder of civilians is and remains an anomaly in American military history, it is the jihadists' way of life.


Yesterday it was reported that Ahmed al-Dabash ,a terrorist ring -leader who confessed to over 100 beheadings was captured . That barely made it through the news cycle as a major story . I guess by Ralls logic then it is because the public has become marginalize fatigued whatever that means . That of course does not prevent the MSM from reporting every single car bomb incident .Saddam has been officially charged for the Hallabjah atrocitiy (5,000 Kurds snuffed by WMD )but the press hardly mentions it .

Today also the Adm. announced that the troops in Iraq will receive Core Values Training which makes me believe that the Adm. thinks there is truth to what is being reported about Hadith . While I do think that what the press is reporting about Haditha has merit (the killings AND the subsequent cover-up) I am not ready to make a blanket comdemnation like Jack Murtha and Ted Rall are .


So far in Iraq, three American soldiers have been convicted of murder. Pvt. Federico Daniel Merida of the North Carolina National Guard received 25 years in prison after being convicted in the shooting death of a 17-year-old Iraqi soldier with whom he had consensual sex.

Army Staff Sgt. Johnny Horne of Wilson, N.C., pleaded guilty to unpremeditated murder in the execution of a severely wounded Iraqi teenager during fighting in Baghdad and received three years.

Staff Sgt. Cardenas J. Alban was convicted of murder in the same case and given a one-year sentence.

But not all charges turn out to be true .

Second Lt. Erick J. Anderson was simularily intially charged with murder in Iraq only to later have the charges dismissed.

2nd Lt. Ilario Pantano was cleared of charges in the death of two Iraqi civilians

Staff Sgt. Shane Werst was aquitted of charges he killed an unarmed Iraqi.

They are innocent but will they ever be able to recover their good names ?

Murders during war do happen but like Michelle Malkin says ;in the case of the US military it is the exception .





Itsdb rated this answer Excellent or Above Average Answer

Question/Answer
jackreade asked on 06/01/06 - ***POP QUIZ***

This is a True of False Quiz constructed by Paul Cummins. Ready, Begin:rue or false:

1. Iraq's reconstruction -- as promised before the U.S. invasion -- has been paid for with Iraq's oil reserves...

2. Iraq's weapons of mass destruction posed an imminent mushroom-cloud threat to the U.S.A...

3. The U.S. invasion of Iraq was greeted by cheering Iraqis...

4. Lucrative no-competitive, no-bid contracts are a responsible way to do business...

5. Torture is an effective way to gain essential information and win international admiration...

6. Tax cuts for the affluent are an effective way to fund wartime expenses...

7. Expenses and debts can be increased indefinitely without worrying about revenue...

8. It is the responsibility of tomorrow's children to pay for the debts of today's adults...

9. The president of the U.S. should not feel constrained by the Constitution if he feels that it need not apply to certain situations that he feels warrant circumventing the Constitution...

10. To combat terrorism, the president of the U.S. has the sole and unquestionable power to seize an American citizen on U.S. soil, send him off to prison and hold him there without evidence or charge indefinitely...

11. Spending $186 million dollars a day in Iraq is a good investment for the future of the U.S.

12. Global warming is just a theory which we don't need to take seriously...

13. Increased pollution and climate changing emissions are not sufficient reasons to restrict the profits of coal-producing plants...

14. Nuclear proliferation is not a danger the U.S. needs to be very concerned about...

15. Corporate CEOs have gotten so good they warrant their current $450:1 ratio to workers pay -- up from 43.1 thirty years ago.

16. The U.S. decision to attack Iraq was justified because God told George W. Bush to do so.

How did you score? How does America?

~~~~~~~~~~~~~~~~~~~~~~~~~~~~~~~~~~~~~~~~~~~~~~~~~

What was your score?

tomder55 answered on 06/01/06:

1. I recall that Colin Powell made reference to Iraqi oil revenue being held in trust "in accordance with international law". Perhaps some officials believed that they could recoup funds that way but as early as Jan.2003 Powell put that idea to rest .

2. Yes they did when you listen to the context that people like Condi Rice were refering to the mushroom cloud metaphor . The Duelfer Report stated in fact that 'Iraq was a more dangerous place than we realized'

3.yes it was . it was the occupation that was problematic . We wasted a year under Viceroy Bremer . We should've gone with the original Pentagon idea to let the INC set up the provisional government and we made a mistake by only partially de-Baathifiying . We are still greeted for the most part by the Iraqis but they are worried about our committment to stay there. Many of them are hesitant to back us because if we leave before a working gvt. can provide security they will be in greater risk.

4.No contract bids are SOP .They were before Clinton gave tons of them to Halliburton and before Bush did .

5. I could care less about international admiration . Torture is effective but we do not employ it unless you consider lap dances from female interrogators as torture .

6. I would prefer floating bonds myself but tax cuts have proven their effectiveness to stimulate the economy . Our problems are with spending other than military .

7.no but since most disciplinesd consumers who do not abuse the system have a reasonable debt . That enables them to do things like purchase houses (otherwise known as tangible assets )that they would otherwise not be able to do . It is likewise not unreasonable for the nation to have debt . The country certainly has sufficient tangible assets to offset the debt . Heck the gvt. owns most of the land west of the Mississippi ! .

8. that's the way the system works . No one complained about generational transfer of wealth during the Social Security reform debates .

9. I do not know of one case where this has happened .

10. according the the war resolution that Congress passed he does . However ,the courts ruled that prisoners held either domestically or for that matter on the battle field are entitled to due process . So now US soldiers need to read Miranda rights to terrorists and jihadists .

11. definitely . trust me we will spend much more than that before we win the war against jihadistan . did you total up how much it cost to win the Cold War ?

12. yes it is a theory ;one we should take seriously and weigh it against all the facts . Should we adopt policies aimed at reducing Co2 emmisions ? definetly .Should we adopt bone headed mandates like Kyoto ? No way !

13. You think it more likely for them to adopt 21st. century technology to clean their emmissions if they are taking a hit in their profits ? What you need to do is prove to them that running clean coal technology will increase their profits .

14. of course it is . why do you think we have taken such an agressive posture with Iran ;N.Korea, Iraq (yes the one WMD we did find in great quantity was yellow cake uranium ...probably from Niger )? Why do you think we pressured Libya to abandon their WMD program ? Why do you think we broke up the AQ Kahn network ? Just ask the gulf states what they think of the prospect of an Iranian bomb.

15. that of course is entirely up to the stock holders .

16. some moonbats may believe this but no one serious does .

ETWolverine rated this answer Excellent or Above Average Answer
Itsdb rated this answer Excellent or Above Average Answer
jackreade rated this answer Average Answer
LTgolf rated this answer Excellent or Above Average Answer

Question/Answer
Itsdb asked on 05/30/06 - The Comeback Kid

As Democrats worry about their 2008 chances, out of the wilderness comes a stranger to save them. Wait a minute. Thats no stranger. thats . . . Al Gore!?!

Seems several of us are betting on Gore in ང, including me. What say you? Forget the President and Secretary-General Clinton pair possibility, what about another Clinton-Gore ticket, president and vice president of the world?

tomder55 answered on 05/31/06:

The election of 1824 was a close and disputed election which went to the House of Reps. JQ Adams ultimately won but his opponent Andrew Jackson later became President .

In the election of 1888 Grover Cleveland the incumbent faced Republican challenger Benjamin Harrison. Cleveland won the popular vote but lost the electoral vote. Harrison became President Cleveland won a rematch four years later.

and the eery comparison :

In 1960 sitting VP Nixon lost a close and controversial election to JFK by slightly over 100,000 votes nationwide .Illinois was the disputed state(Kennedy won the State by )by 8,000 votes )and in all probability the Daley machine massaged the results ;but Nixon instead of contesting the vote conceded . 8 years later Nixon won the Presidency .

History is on Gore's side . I hate to write praise of him and I won't ,but ,his reivention has been pretty impressive. After his initial feel sorry for himself period he has not only found a theme that resonates that he leading on (I do not agree with him but he is in the forefront of the debate ). He has also figured a way to make environmentalism an economic debate where he takes the side that it is economically friendly to be green .

Among those who share this vision is new Tresury Sec. Henry Paulson who is chaiman of the Board of Directors of the Nature Conservancy.He recently donated $100 million in stock to conservation. He also led Goldman, Sachs in a green direction, despite skepticism from some of his shareholders.

Gore founded ' Generation Investment Management 'in 2004 with former Goldman Sachs exec. David Blood. A London firm ;they put money from institutions and rich people into companies that are going green or make products that help others be more environmentally or socially sound.(BTW .also in the cabal of Goldman Sachs people of influence are new White Hosue Chief of Staff Joshua Bolten and Govenor of NJ Jon Corzine ...they are taking over !!!)

Gore was also a leading advocate of the internet .No he did not invent it , the military did ,but again it was one of the issues that he was and is ahead of the pack on .He has made a fortune for himself on Google stock (and has turned Google into a left wing advocacy org. but that is another issue). How much ? He could drop $50 million into his campaign and not feel the effect .In 2003 Gore took a seat on the board of Apple Computer; today he has 60,000 options .He also helped launch Current TV which is gaining a nich with young viewers . One of the features is that viewers can submit pod broadcasts between 5 seconds and 15 minutes to air on the network .

He commands up the $150,000 per lecture and his new documentary was timed for release just at the time for the campaign season to begin.

Hillary is increasingly being panned by the Democrats for her pro-Iraq war stance. The moonbats are looking for an alternative and Gore is emerging at the right time .

Itsdb rated this answer Excellent or Above Average Answer

Question/Answer
Itsdb asked on 05/30/06 - Secretary-General Bill Clinton

From an LA Times editorial on May 28th:

    The U.N. needs Bill more than the U.S. needs Hillary.
    May 28, 2006

    "THE BEST THING HILLARY RODHAM CLINTON could do for humanity is not run for president. Nothing against her personally, mind you; it's just that her aspirations could get in the way of her husband's worthier ones.

    In our continuing quest to find an appropriate job for our favorite ex-president a year and a half ago we suggested he become chairman of the Democratic Party we now offer an even better suggestion. This time, it's a post he has coveted. Not long after leaving office in 2001, Clinton reportedly told an aide that his dream job would be secretary-general of the United Nations. That's our dream too."


~~~~~~~~~~~~~~~~~~~~~~~~~~~~~~~~~~~~~~~~~~~~~~~~~~~~~~

Talk about surreal...

It's interesting that the Times would make note that "the U.N.'s reputation has been tattered by peacekeeper sex scandals," and they want Bill Clinton to fix that? LOL.

    If the Security Council members were truly inspired to pick the right man for the job, and if Hillary Clinton's candidacy were the only obstacle standing between her husband and global leadership (granted, a big if), we'd like to think she'd do the right thing and put her presidential aspirations on hold. The world needs Bill more than the U.S. needs Hillary.


What the world needs now is Bill, sweet Bill
He's the only thing that there's just too little of...

tomder55 answered on 05/31/06:

yeah ;a job that offers virtual immunity .God help us . You think Coffee Anon. is bad wait until you see Bubba's act on the world stage . women of the world ...tighten up your chastity belts !!

I do not see how this prevents Madame Defarge from running for President . That would make it ideal I would think from their point of view .Bubba having his fingers on a world governing body and Evita having her fingers on the only nation that has the finances and the military to impose the UN's will .

SCARY MOVIE V

ETWolverine rated this answer Excellent or Above Average Answer
Itsdb rated this answer Excellent or Above Average Answer

Question/Answer
Itsdb asked on 05/30/06 - Murtha

"This is the kind of war you have to win the hearts and minds of the people," Rep. John Murtha, D-Pa., said Sunday. "And we're set back every time something like this happens. This is worse than Abu Ghraib."

If these Marines did what was alleged, it's indefensible. With that disclaimer, how does a congressman traipsing around the country trying these Marines in the court of public opinion achieve achieve his stated goal of winning the hearts and minds of the Iraqi people? Does anyone believe Murtha gives a hoot about the Iraqi people? Do yo believe Murtha is interested in justice or is he just enjoying his new celebrity status?

tomder55 answered on 05/30/06:

Murtha is less concerned with winning the hearts and minds of the Iraqis then he is for winning the campaign donations of 'code pink' and 'answer'. Of course there are set backs as al-jazzera and other jihadistan publications quote him verbadum .

You would think that an ex-marine would have the decency to wait until inquiries are fininished before he condemns these troops whether the allegations are true or not;before they have a chance to present their side . Previously he painted them as cold blooded murderers and then absolved them because of they were over-reacting due to the extreme pressure they are under (Blame Bush). Well by definition if they were over-reacting then it was not pre-meditated and not cold blooded .

I still say he has a hidden motive in all this . Murtha is beefing up his anti-war credibility, only so he can cry "they're questioning my patriotism/trying to silence me", when he gets indicted for influence peddling. Perhaps now we understand why he was perceived a hawk ...it was true as far as military procurement is concerned . Who knew his motive was self-interest ? What he has is a long history of supporting troops by voting for procurements while at the same time opposing any action they are deployed in .

ETWolverine rated this answer Excellent or Above Average Answer
Itsdb rated this answer Excellent or Above Average Answer

Question/Answer
excon asked on 05/27/06 - My dead horse ain't so dead


Hello drugwarriors:

Upon further review, the recent study by Dr. Donald Tashkin of UCLA goes much further in suggesting that THC inhibits the growth of cancer cells, than I had previously gathered.

As a drugwarrior, he said of the study, "Our major hypothesis, was that heavy, long-term use of marijuana will increase the risk of lung and upper-airways cancers." However, Tashkin concluded, "we failed to observe a positive association of marijuana use and lung, and upper-airway cancers."

He goes on, it would be difficult to extract from these data the conclusion that marijuana is protective against lung cancer. But that is not an unreasonable hypothesis."

This is big. THC could be the cure for cancer!!! If you were diagnosed with inoperable cancer, would you try pot?

excon

tomder55 answered on 05/30/06:

the reason whole plants are almost never approved as perscritions is that it is hard to show which compound or combinations of compounds in the plant are effective . How does anyone know that it is the THC that is causing the effect being observed ? This research is in it's infancy and it should not be approved without a whole lot more research ;especially given the liability issues related to marketting even approved and effective products .

I doubt that marijuana will ever be approved as a plant that you light up and smoke . It is fact that there are too many other possible carcinogens inhaled . Perhaps an oral ingestable will eventually find it's way on the market with controlled doses but doobies ...I don't think so .

excon rated this answer Excellent or Above Average Answer
sissypants rated this answer Excellent or Above Average Answer

Question/Answer
Itsdb asked on 05/30/06 - Sen. Reid defends taking free tickets to boxing matches


Harry Reid With hands on heart calling on Republicans to clean up their act with lobbyists
AP Photo/J. SCOTT APPLEWHITE

Tuesday, May 30, 2006
John Solomon
Associated Press

Washington -- Senate Democratic Leader Harry Reid accepted free ringside tickets from the Nevada Athletic Commission to three professional boxing matches while that state agency was trying to influence him on federal regulation of boxing.

Reid, Democrat of Nevada, took the free seats for Las Vegas fights between 2003 and 2005 as he was pressing legislation to increase government oversight of the sport, including the creation of a federal boxing commission that Nevada's agency feared might usurp its authority.

He defended the gifts, saying they would never influence his position on the bill and he was simply trying to learn how his legislation might affect an important home state industry. "Anyone from Nevada would say I'm glad he is there taking care of the state's No. 1 businesses," he said.

"I love the fights anyways, so it wasn't like being punished," added the senator, a former boxer and boxing judge.

Senate ethics rules generally allow lawmakers to accept gifts from federal, state or local governments, but specifically warn against taking such gifts -- particularly on multiple occasions -- when they might be connected to efforts to influence official actions.

"Senators and Senate staff should be wary of accepting any gift where it appears that the gift is motivated by a desire to reward, influence, or elicit favorable official action," the Senate ethics manual states. It cites the 1990s example of an Oregon lawmaker who took gifts for personal use from a South Carolina state university and its president while that school was trying to influence his official actions.

Several ethics experts said Reid should have paid for the tickets, which were close to the ring and worth between several hundred and several thousand dollars each, to avoid the appearance he was being influenced by gifts.

Two sena tors who joined Reid for fights with the complimentary tickets took markedly different steps.

Sen. John McCain, Republican of Arizona, insisted on paying $1,400 for the tickets he shared with Reid for a 2004 championship fight. Sen. John Ensign, Republican of Nevada, accepted free tickets to another fight with Reid but already had recused himself from Reid's federal boxing legislation because his father was an executive for a Las Vegas hotel that hosts fights.

In an interview Thursday in his Capitol office, Reid broadly defended his decisions to accept the tickets and to take several actions benefiting disgraced lobbyist Jack Abramoff's clients and partners as they donated to him.

"I'm not Goodie two shoes. I just feel these events are nothing I did wrong," Reid said.

Reid had separate meetings in June 2003 in his Senate offices with two Abramoff tribal clients and Edward Ayoob, a former staffer who went to work lobbying with Abramoff.

The meetings occurred over a five-day span in which Ayoob also threw a fund-raiser for Reid at the firm where Ayoob and Abramoff worked that netted numerous donations from Abramoff's partners, firm and clients.

Reid said he viewed the two official meetings and the fund-raiser as a single event.

One of the tribes, the Saginaw Chippewa of Michigan, donated $9,000 to Reid at the fund-raiser and the next morning met briefly with Reid and Ayoob at Reid's office to discuss federal programs. Reid and the tribal chairman posed for a picture.

~~~~~~~~~~~~~~~~~~~~~~~~~~~~~~~~~~~~~~~~~~~~~~~~~~~~~~

That's the way to punch at those corrupt Republicans Harry. Things aren't looking too good for the dems election strategy this year, what do you think they're new plan will be - or will they change it at all?

tomder55 answered on 05/30/06:

What do ring side seats in Vegas go for these days ?.....$1400 (that McCain paid ? )I bet that wasn't even scalper prices .What's his reasoning again ? It was a fact finding mission ? I wonder why this is different than a Tom Delay golfing junket ? Who gets payed off in Vegas stays in Vegas .


That Democrat culture of corruption rears it's ugly head again . Reid can't help himself he has such a passion for the sport it clouds his judgement. As PBS gushes about Reid :

He has said that growing up among miners taught him how to settle things with a fight if need be. He was a middleweight boxer in high school and has admitted to getting "called out" a time or two for an old-fashioned fist fight. I guess he is in good company with Sen. Boxer .

Ahhh ,the sweet science !!

The Senate ethics manualstates :

"Senators and Senate staff should be wary of accepting any gift where it appears that the gift is motivated by a desire to reward, influence or elicit favorable official action," ..."Repeatedly taking gifts which the Gifts Rule otherwise permits to be accepted may, nonetheless, reflect discredit upon the institution, and should be avoided,"

But in this case as well as all his dealings with Abramoff clients it will be argued there was no quid pro quo .I wonder if Rep. Jefferson will now be able to argue that the $100,000 in his freezer would in no way influence his decisions .

Itsdb rated this answer Excellent or Above Average Answer
katiy rated this answer Excellent or Above Average Answer

Question/Answer
jocase asked on 05/29/06 - Enron Executives Pardoned?

After the Enron executives are sentenced, will they be pardoned by the outgoing President Bush?
You heard it here first!

What do you think?

tomder55 answered on 05/30/06:

I don't think so . It was the Bush Justice Dept. that aggressively persued their prosecution . I would also point out that not only Enron ,but also Adephia and Worldcom happened during the lawless 1990s under Clinton's watch and that Enron curried alot of favors from the Clintoon's in exchange for campaign contributions . Like I mentioned in my response to Choux about this issue ;Kenny boy traveled so often with Ron Brown on official gvt. business that when Brown's plane crashed ,Enron stock started to free fall drop until Enron put out a press release that Lay had not been on the plane .

I hope Bush does not pull a Clinton . Notable is that Denis Rich brought her husband's pardon with big donations to the DNC and the Clintoon library .

In another case Clinton pardoned criminals from New Square NY after the Hasidic community voted 1359-10 for Hillary for NY Senator . Every other Hasidic enclave overwhelmingly voted for her opponent Rick Lazio. Evidently no quid pro quo was proven.

Here is a list of his pardons :

http://www.usdoj.gov/opa/pardonchartlst.htm

He granted clemency to 16 Puerto Rican FALN terrorists.President Clinton did not follow formal pardon procedures. He skipped the Department of Justice and attorneys. The FBI did not conduct any background checks and the FALN did not even execute a formal request. These facts, coupled with the Department of Justices 1996 denial of their clemency, make Clintons motives highly questionable.

It has become a tradition of sorts for Presidents to grant pardons before they leave office . I guarantee that no matter who Bush pardons it will spark outrageous indignation .

Itsdb rated this answer Excellent or Above Average Answer
jocase rated this answer Excellent or Above Average Answer
purplewings rated this answer Excellent or Above Average Answer

Question/Answer
jackreade asked on 05/28/06 - Reply from NSA

This is a letter Mr. Shafranovich received in response to his request for a copy of any information the NSA has collected on him from the telephone monitoing system. From his web site:

Mr. Yakov Shafranovich
XXXXXXXXXXXXXXXXXXX
XXXXXXXXXXXXXXXXXXX

Dear Mr. Shafranovich:

This responds to your Freedom of Information Act (FOIA) request of 11 May 2006, which was received by this office on 12 May 2006, for a list of all phone records collected by the NSA from telecommunications companies under your home and cell phone number and any other information this Agency may maintain on you. Please refer to the case number at the top of the page when contacting us about your request. There are no assessable fees for this request. Your request has been processed under the provisions of the FOIA.

Because of the classified nature of the National Security Agencys efforts to prevent and protect against terrorist attacks, the fact of whether or not any specific technique or method or activity is employed in that effort is exempt from release pursuant to the exemption provisions of the FOIA.

We can neither confirm nor deny the existence of records responsive to your request. The fact of the existence or non-existence of responsive records is a currently and properly classified matter in accordance with Executive Order 12958, as amended. Thus, your request is denied pursuant to the first exemption of the FOIA, which provides that the FOIA does not apply to matters that are specifically authorized under criteria established by an Executive Order to be kept secret in the interest of national defense or foreign relations and are properly classified pursuant to such Executive Order.

Moreover, the third exemption of the FOIA provides for the withholding of information specifically protected from disclosure by statute. Thus, your request is also denied because the fact of the existence or non-existence of the information is exempted from disclosure pursuant to the third exemption. The specific statutes applicable in this case are Title 18 U.S. Code 798; Title 50 U.S. Code 403-1(i); and Section 6, Public Law 86-36 (50 U.S. Code 402 note).

As your request is being denied, you are hereby advised of this Agencys appeal procedures. Any person denied access to information may file an appeal to the NSA/CSS Freedom of Information Act Appeal Authority. The appeal must be postmarked no later than 60 calendar days of the date of the initial denial letter. The appeal shall be in writing addressed to the NSA/CSS FOIA Appeal Authority (DC34), National Security Agency, 9800 Savage Road STE 6248, Fort George G. Meade, MD 20755-6248. The appeal shall reference the adverse determination and shall contain, in sufficient detail and particularity, the grounds upon which the requester believes that the determination is unwarranted. The NSA/CSS FOJA Appeal Authority will endeavor to respond to the appeal within 20 working days after receipt, absent any unusual circumstances.

If we have misinterpreted your request and you have been affiliated with the NSA in some way as an employee, applicant, or visitor and are looking for records related to those activities, you may submit a signed Privacy Act request to seek that type of information. If you provide a Social Security number, it will assist us with the search for responsive records.

Sincerely,

LOUIS F. GILES
Director of Policy

I haven't yet decided whether to appeal.

~~~~~~~~~~~~~~~~~~~~~~~~~~~~~~~~~~~~~~~~~~~~~~~~~


I thought it was interesting and definitely the real meaning of surreal.

tomder55 answered on 05/29/06:

not surreal at all . in fact ,no one in gvt. has confirmed or denied what USA Today has reported nor should they . This is ;if true ; a classified national security operation . If the operation is true as reported ;and there is no indication to me that it is since many of the major telecom companies have made blanket denials ;then there is nothing illegal in the operation ,and in fact was probably very beneficial in the war against Islamonazis and was probably instrumental in avoiding another attack on domestic USA.

excon rated this answer Excellent or Above Average Answer
jackreade rated this answer Excellent or Above Average Answer

Question/Answer
excon asked on 05/28/06 - Old felonious excon


Hello nannystaters:

As many of you know, I participate in on-line poker. As of June 7, my poker playing activities will become a felony. The head of the state gambling commission, however, said that it is unlikely that individual gamblers will be targeted. Yeah right!

(1) Do you believe the head gambling cop? (2) Why would they spend the time to make gambling a felony if they had no intention of enforcing it? (3) Isn't making law that they don't enforce and have no intention of enforcing, a problem? (4) Should I quit? (5) Do you want your cops spending time doing this kind of enforcement?

excon

PS> NO, I ain't gonna quit. And YES, because of that decision, I know the Wolverine thinks I should go to jail. (6) Do you?

tomder55 answered on 05/29/06:

They can't go after those who run the games because they are based off-shore in other countries. If they want to enforce it they have to go after the players. (unless they let the UN take over the internet then you will be tired in the Hague ).But with it being done out of country users are more suceptible to fraud without redress. Better to maker it legal and regulated inside the country .

Hello ? What are they thinking ? PartyPoker.com had more than a million people play for money on its site last year, the vast majority of them Americans.The firm went public on the London Stock Exchange last summer. The IPO, which valued PartyPoker at $10 billion, was Britain's biggest in 2005. Why are we outsourcing this industry ? It doesn't makes sense.

With States across the country running numbers rackets I think they would find it hard to justify preventing gamblers from playing skill games like poker and sports betting ....games they actually have a chance of winning at .Plus they promote lotteries everywhere - TV, radio, etc. And generally the people who lose the most money on lotteries are the really poor people who are on welfare and can least afford it. I call lotteries the poor person's tax.

excon rated this answer Excellent or Above Average Answer
powderpuff rated this answer Excellent or Above Average Answer

Question/Answer
quixotic_Choux asked on 05/25/06 - Kenny Boy Pontificates

Ken Lay was found guilty today of conspiracy and faces a a 10 to 15 year prison sentence. We all know of the collapse of Enron.

"I firmly believe I'm innocent of the charges against me," Lay said following the hearing. "We believe that God in fact is in control and indeed he does work all things for good for those who love the lord."

bwahaha haha haha

~~~~~~~~~~~~~~~~~~~~~~~~~~~~~~~~~~~~~~~~~~~~~~~~


How long can religious extremists go supporting those who give lip service to God while stealing everything they come in contact blind???

tomder55 answered on 05/27/06:

Time for some splainin the facts .The dirty deeds at Enron happened under the Clintonoid(which rhymes with hemorroid ) watch . It is the Bush justice dept. that is bringing the Enron principles to justice. comprende ?

Kenny boy was such a favorite at the Clintoon white house that when Ron Brown's plane crashed ENRON stock started to plummet because of rumors of Lay being on the jet with Brown.'In 1992, Enron donated $100,000 to Clinton's inauguration, and Ken Lay, stayed at the White House 11 times.'

NewsMax summed up the corrupt relationship nicely :

Enron executives traveled with Secretary of Commerce Ron Brown in 1994 on trade missions to Russia, India, Indonesia and China, cutting U.S. taxpayer-financed deals in each country. In fact, Brown paid a great deal of attention to Enron. Indonesia was pressed by Clinton's secretary of commerce to accept Enron deals laced with corruption.

Enron's most twisted activities involve a 1994 trade trip to Indonesia with then-Commerce Secretary Brown. Immediately after traveling to Indonesia, Brown personally sought approval for Enron electric power plants sponsored by U.S. funding. The documents show that the Clinton administration knew the deals were also filled with kickbacks for Indonesian president Suharto.


read the rest . I only pasted a small part of the link .

-ON JULY 5, 1995, Enron Corporation donated $100,000 to the Democratic National Committee. Six days later, Enron executives were on a trade mission with Commerce Secretary Mickey Kantor to Bosnia and Croatia. With Kantor's support, Enron signed a $100 million contract to build a 150-megawatt power plant.

-According to a study by the Center for Public Integrity, Enron, U.S. West, GTE, McDonnell Douglas, and Fluor donated a combined $563,000 to the Democratic party during 1993 and 1994 and received $2.6 billion in foreign contracts secured with government help. The Globe found that during the first Clinton term, 27 firms had donated $2.3 million to the Democrats and received nearly $5.5 billion in federal support.


-Ken Lay was a close friend of
Mack McLarty, Clinton's first chief of staff.

-In his 1993 disclosure statement, Robert Rubin listed Enron as one of the firms with which he had had "significant
contact" while at Goldman Sachs. Enron was
represented by the law firm of Akin, Gump, Strauss,Hauer & Feld, the firm where Clinton advisers Robert Strauss and Vernon Jordan worked.

In other words ;what everyone claims is a special relationship between this Admnistration and Halliburton was in fact the case between Bubba and Enron .

To sum it up ....Enrongate is a democrat scandal.



quixotic_Choux rated this answer Poor or Incomplete Answer

Question/Answer
Itsdb asked on 05/25/06 - A.C.L.U. May Block Criticism by Its Board

By STEPHANIE STROM
Published: May 24, 2006

The American Civil Liberties Union is weighing new standards that would discourage its board members from publicly criticizing the organization's policies and internal administration.

"Where an individual director disagrees with a board position on matters of civil liberties policy, the director should refrain from publicly highlighting the fact of such disagreement," the committee that compiled the standards wrote in its proposals.

"Directors should remember that there is always a material prospect that public airing of the disagreement will affect the A.C.L.U. adversely in terms of public support and fund-raising," the proposals state.

Given the organization's longtime commitment to defending free speech, some former board members were shocked by the proposals.

Nat Hentoff, a writer and former A.C.L.U. board member, was incredulous. "You sure that didn't come out of Dick Cheney's office?" he asked.

"For the national board to consider promulgating a gag order on its members I can't think of anything more contrary to the reason the A.C.L.U. exists," Mr. Hentoff added.

The proposals say that "a director may publicly disagree with an A.C.L.U. policy position, but may not criticize the A.C.L.U. board or staff." But Wendy Kaminer, a board member and a public critic of some decisions made by the organization's leadership, said that was a distinction without a difference.

"If you disagree with a policy position," she said, "you are implicitly criticizing the judgment of whoever adopted the position, board or staff."

Anthony D. Romero, the A.C.L.U.'s executive director, said that he had not yet read the proposals and that it would be premature to discuss them before the board reviews them at its June meeting.

Mr. Romero said it was not unusual for the A.C.L.U. to grapple with conflicting issues involving civil liberties. "Take hate speech," he said. "While believing in free speech, we do not believe in or condone speech that attacks minorities."

Lawrence A. Hamermesh, chairman of the committee, which was formed to define rights and responsibilities of board members, also said it was too early to discuss the proposals, as did Alison Steiner, a committee member who filed a dissent against some recommendations.

In a background report, the committee wrote that "its proposed guidelines are more in the nature of a statement of best practices" that could be used to help new board members "understand and conform to the board's shared understanding of the responsibilities of its members."

But some former board members and A.C.L.U. supporters said the proposals were an effort to stifle dissent.

"It sets up a framework for punitive action," said Muriel Morisey, a law professor at Temple University who served on the board for four years until 2004.

Susan Herman, a Brooklyn Law School professor who serves on the board, said board members and others were jumping to conclusions.

"No one is arguing that board members have no right to disagree or express their own point of view," Ms. Herman said. "Many of us simply think that in exercising that right, board members should also consider their fiduciary duty to the A.C.L.U. and its process ideals."

When the committee was formed last year, its mission was to set standards on when board members could be suspended or ousted.

The board had just rejected a proposal to remove Ms. Kaminer and Michael Meyers, another board member, because the two had publicly criticized Mr. Romero and the board for decisions that they contended violated A.C.L.U. principles and policies, including signing a grant agreement requiring the group to check its employees against government terrorist watch lists a position it later reversed and the use of sophisticated data-mining techniques to recruit members.

Mr. Meyers lost his bid for re-election to the board last year, but Ms. Kaminer has continued to speak out. Last month, she was quoted in The New York Sun as criticizing the group's endorsement of legislation to regulate advertising done by counseling centers run by anti-abortion groups. The bill would prohibit such centers from running advertisements suggesting that they provide abortion services when they actually try to persuade women to continue their pregnancies.

Ms. Kaminer and another board member, John C. Brittain, charged that the proposal threatened free speech. "I find it quite appalling that the A.C.L.U. is actively supporting this," Ms. Kaminer told The Sun.

The uproar their comments produced at the April board meeting illustrates how contentious the issue of directors' publicly airing dissent with policies and procedures has become at the organization.

Some directors lamented that Ms. Kaminer and Mr. Brittain had shared their disagreement with the paper, and Mr. Romero angrily denounced Ms. Kaminer. "I got frustrated and lost my temper," he said yesterday. "In retrospect, that was a mistake."

At the meeting, Mr. Romero did not denounce Mr. Brittain. But board members said he had demanded that Ms. Steiner step outside the meeting room, where he chastised her for the look on her face when he was criticizing Ms. Kaminer.

"Anthony went on to say that because I was Wendy's 'friend' and did not appear ready to join him in 'getting rid of her,' (by, among other things, lobbying her affiliate to remove her as its representative) I was no better than she was, and then stormed off angrily," Ms. Steiner wrote in an e-mail message to the board.

Later in the meeting, Mr. Romero asked another board member, David F. Kennison, to step outside after Mr. Kennison apologized for failing to object to Mr. Romero's attack on Ms. Kaminer.

Mr. Kennison reported in an e-mail message that Mr. Romero "told me that he would 'never' apologize to the target of his outburst and that his evaluation of her performance as a member of this board was justified by information he had been accumulating in a 'thick file on her.' "

When Mr. Kennison asked whether Mr. Romero intended to start such a file on him, "he asked me what made me think that he didn't already have a file on me," Mr. Kennison wrote.

Mr. Romero said Mr. Kennison had provoked him. "I do not have a file on Wendy," he said.

In a telephone interview, Mr. Kennison said his biggest concern was the relationship between the board and the A.C.L.U. staff.

"I think of the board as the brain and the staff as the fang and the claws," he said, "and the brain should govern the fangs and claws rather than the other way around."

~~~~~~~~~~~~~~~~~~~~~~~~~~~~~~~~~~~~~~~~~~~~~~~~~~~~~~

You know, this would be funny if it didn't emphatically highlight the hypocrisy of it all. So what do you think? A little or a lot hypocritical? Totally justified? Or it all depends on whose interest is at stake?

Steve

tomder55 answered on 05/26/06:

first they collected a data base on their membership ;now they are censoring their members .throw in a dose of pc ... they are well on their way towards creating a model for their unstated goal ....liberal dictatorship ..

Itsdb rated this answer Excellent or Above Average Answer

Question/Answer
arcura asked on 05/25/06 - Is it true that Latinos fill jobs that no one else wants?

Rising black-Latino clash on jobs
By Daniel B. Wood, Staff writer of The Christian Science Monitor Thu May 25, 4:00 AM ET
LOS ANGELES - From where Johnny Blair Vaughn sits outside Lucy Florence Coffee House in the heart of Los Angeles's black community, he can feel the temperature rising over immigration.
The biggest reason, says the father of seven, is jobs.
"If you drive across this city, you will see 99 percent of all construction is being done by Hispanics.... You will see no African-American males on these sites, and that is a big change," says Mr. Vaughn, who has worked in construction for two decades. His two oldest boys, in their early 20s, have been turned down so many times for jobs - as framers, roofers, cement layers - that they no longer apply, he says.
While Los Angeles is ground zero for black-Hispanic friction these days, echoes of Vaughn's words are rising throughout urban black America as Congress labors over immigration reform. In cities where almost half of the young black men are unemployed, a debate is raging over whether Latinos - undocumented and not - are elbowing aside blacks for jobs in stores, restaurants, hotels, manufacturing plants, and elsewhere.
Hispanics and blacks tend to gravitate to the same inner-city areas and low-skill labor markets - and the result is a clash over jobs that require less skill and less education, experts say.
"In this era of mass immigration, no group has benefited less or been harmed more than the African-American population," says Vernon Briggs, a Cornell University professor who researches immigration policy and the American labor force.
Yet a precise relationship between the presence of immigrants and the loss of black jobs has not been clearly proven in research. Rather, the influx of legal and illegal immigrants has been so massive that it has affected the internal migration of native-born Americans to the point where "economists have given up trying to prove a one-to-one-displacement," says Dr. Briggs.
2004 statistics show that manual labor jobs in agriculture one third of them were filled by Latinos, two thirds by USA citizens.
In the construction trades one fourth of the manual labor jobs were filled by Latinos, three fourths by USA citizens. That ratio appears to be changing as more workers willing to work for lower wages are flooding the job market.

tomder55 answered on 05/25/06:

I do not know if Latinos do or not but I know for a fact that illegal aliens dod not fill postions that no one else wants.

I linked yesterday to and article in Real Clear Politics yesterday by Tom Sowell that debunks some of the myths that have become conventional wisdom ; including this :

How many times have we heard that illegal aliens are taking "jobs that Americans won't do"? Just what specifically are those jobs?

Even in occupations where illegals are concentrated, such as agriculture, cleaning, construction, and food preparation, the great majority of the work is still being done by people who are not illegal aliens.

The highest concentration of illegals is in agriculture, where they are 24 percent of the people employed. That means three-quarters of the people are not illegal aliens. But when will the glib phrase-mongers stop telling us that the illegals are simply taking "jobs that Americans won't do"?


As far as the clash of races that your link implies . The black population needs to listen a little more to Bill Cosby and less to Jesse Jackson .A large part of it is historical discrimination of course but before the cry of racism gets to loud ,why would would there be a preference to discriminate against American blacks over hispanics ?

You've got to crawl before you can walk. And if you didn't want to apply yourself and get an education, well, you need to take that lesser paying job that immigrants do.But if an employer can get away with exploiting the immegrant who is illegal then they will hire them over the black laborer because they do not have to comply with minimum labor standards .All the more reason to restrict the flow of illegals into the country .

But which politicians are the black laborers likely to support ? The ones who pander to the open border advocates . Go figure .

arcura rated this answer Excellent or Above Average Answer

Question/Answer
ROLCAM asked on 05/25/06 - The Gaucho Laird !!

An article worth reading:-

http://heritage.scotsman.com/greatscots.cfm?id=1785672005

rolcam.

tomder55 answered on 05/25/06:

sounds like me . my one forray into being a landlord was a disaster.

ROLCAM rated this answer Excellent or Above Average Answer

Question/Answer
Fritzella asked on 05/24/06 - Waiving SEC Rules


"Intelligence Czar Can Waive SEC Rules
Now, the White House's top spymaster can cite national security to exempt businesses from reporting requirements


President George W. Bush has bestowed on his intelligence czar, John Negroponte, broad authority, in the name of national security, to excuse publicly traded companies from their usual accounting and securities-disclosure obligations. Notice of the development came in a brief entry in the Federal Register, dated May 5, 2006, that was opaque to the untrained eye.

Unbeknownst to almost all of Washington and the financial world, Bush and every other President since Jimmy Carter have had the authority to exempt companies working on certain top-secret defense projects from portions of the 1934 Securities Exchange Act. Administration officials told BusinessWeek that they believe this is the first time a President has ever delegated the authority to someone outside the Oval Office. It couldn't be immediately determined whether any company has received a waiver under this provision.

The timing of Bush's move is intriguing. On the same day the President signed the memo, Porter Goss resigned as director of the Central Intelligence Agency amid criticism of ineffectiveness and poor morale at the agency. Only six days later, on May 11, USA Today reported that the National Security Agency had obtained millions of calling records of ordinary citizens provided by three major U.S. phone companies. Negroponte oversees both the CIA and NSA in his role as the administration's top intelligence official.

FEW ANSWERS. White House spokeswoman Dana M. Perino said the timing of the May 5 Presidential memo had no significance. "There was nothing specific that prompted this memo," Perino said.

In addition to refusing to explain why Bush decided to delegate this authority to Negroponte, the White House declined to say whether Bush or any other President has ever exercised the authority and allowed a company to avoid standard securities disclosure and accounting requirements. The White House wouldn't comment on whether Negroponte has granted such a waiver, and BusinessWeek so far hasn't identified any companies affected by the provision. Negroponte's office did not respond to requests for comment.

Securities-law experts said they were unfamiliar with the May 5 memo and the underlying Presidential authority at issue. John C. Coffee, a securities-law professor at Columbia University, speculated that defense contractors might want to use such an exemption to mask secret assignments for the Pentagon or CIA. "What you might hide is investments: You've spent umpteen million dollars that comes out of your working capital to build a plant in Iraq," which the government wants to keep secret. "That's the kind of scenario that would be plausible," Coffee said.

AUTHORITY GRANTED. William McLucas, the Securities & Exchange Commission's former enforcement chief, suggested that the ability to conceal financial information in the name of national security could lead some companies "to play fast and loose with their numbers." McLucas, a partner at the law firm Wilmer Cutler Pickering Hale & Dorr in Washington, added: "It could be that you have a bunch of books and records out there that no one knows about."

The memo Bush signed on May 5, which was published seven days later in the Federal Register, had the unrevealing title "Assignment of Function Relating to Granting of Authority for Issuance of Certain Directives: Memorandum for the Director of National Intelligence." In the document, Bush addressed Negroponte, saying: "I hereby assign to you the function of the President under section 13(b)(3)(A) of the Securities Exchange Act of 1934, as amended."

A trip to the statute books showed that the amended version of the 1934 act states that "with respect to matters concerning the national security of the United States," the President or the head of an Executive Branch agency may exempt companies from certain critical legal obligations. These obligations include keeping accurate "books, records, and accounts" and maintaining "a system of internal accounting controls sufficient" to ensure the propriety of financial transactions and the preparation of financial statements in compliance with "generally accepted accounting principles."
From Businessweek online.

What's really going on here?

tomder55 answered on 05/25/06:

I don't know for sure but the last paragraph spells it out that it is written into the SEC laws way back in 1934 . That would've been a beloved Democrat President and Congress that enacted the provision . I have no idea if it has ever been implemented but I suspect it probably was during the depression and WWII.

Before you critique Bush remember that it was he who signed into law the Sarbanes-Oxley Act which tightened up requirements for corporate accounting practices .

Fritzella rated this answer Excellent or Above Average Answer
Itsdb rated this answer Excellent or Above Average Answer

Question/Answer
excon asked on 05/24/06 - Oil


Hello:

A lot of folks can't understand how we came to have an oil shortage here in our country.

Well, there's a very simple answer. Nobody bothered to check the oil. We just didn't know we were getting low.

The reasons for that is purely geographical. You see, our OIL is located in Alaska, California,
Oklahoma, Texas, Utah and Wyoming.

Our DIPSTICKS, however, are located in Washington, D.C.

excon

tomder55 answered on 05/25/06:

LOL true .

excon rated this answer Excellent or Above Average Answer

Question/Answer
Fritzella asked on 05/24/06 - Constitutional Crisis

"WASHINGTON (AP) - The FBI's raid on a Democrat's office rippled through Capitol Hill Wednesday, with majority Republicans demanding that the agency surrender documents and other items its agents seized under what lawmakers said were unconstitutional circumstances.

``We think those materials ought to be returned,'' said House Speaker Dennis Hastert, adding that the FBI agents involved ``ought to be frozen out of that (case) just for the sake of the constitutional aspects of it.''

The Saturday night search of Rep. William Jefferson's office on Capitol Hill brought Democrats and Republicans together in rare election-year accord, with both parties protesting agency conduct they said violated the Constitution's separation of powers doctrine.

``Not anyone here is above the law,'' Pelosi told reporters Tuesday. But, she added, ``I think you've seen abuse of power of the executive branch over this weekend.''

A day earlier, Hastert, R-Ill., complained personally to President Bush about raid. Other House officials have predicted that the case would bring all three branches together at the Supreme Court for a constitutional showdown." From my ISP homepage

~~~~~~~~~~~~~~~~~~~~~~~~~~~~~~~~~~~~~~~~~~~~~~~`

So, in the dark of night, the Executive Branch ordered a raid on a Congressman's office, the Congress of the United States of America.

Did you know that Congressional Representatives are the only branch of government that is elected directly by the people????

So a Congressman is a scum....some Presidents and some Senators are much bigger scum.

Now, we have the Republican and Democratic Representatives up in arms about Bush's actions UNDER COVER OF DARKNESS.

The Bush madness is escalating....

tomder55 answered on 05/25/06:

interesting . I note that they are not nearly as concerned about separation-of-powers issues when they supoena documents from the vice presidents energy-task-force or from personal attorneys of the President during confirmation hearings .Miguel Estrada withdrew his nomination over the issue and he would've been an excellent circut court judge.

I actually think there is alot of merit in the congress' complaint but it is not as cut and dry as they would have us believe. The speech and debate clause of the constitution (Article I, Section 6, Clause 1....The purpose of the clause is to prevent the arrest and prosecution of unpopular legislators based on their political views.) does not give them immunity like UN officials appear to have.

Here is the clause :

Section 6. The Senators and Representatives shall receive a compensation for their services, to be ascertained by law, and paid out of the treasury of the United States. They shall in all cases, except treason, felony and breach of the peace, be privileged from arrest during their attendance at the session of their respective Houses, and in going to and returning from the same; and for any speech or debate in either House, they shall not be questioned in any other place.


As >Findlaw explains much better than I could :

''The immunities of the Speech or Debate Clause were not written into the Constitution simply for the personal or private benefit of Members of Congress, but to protect the integrity of the legislative process by insuring the independence of individual legislators.''

The protection of this clause is not limited to words spoken in debate. ''Committee reports, resolutions, and the act of voting are equally covered, as are 'things generally done in a session of the House by one of its members in relation to the business before it.''' 385 Thus, so long as legislators are ''acting in the sphere of legitimate legislative activity,'' they are ''protected not only from the consequence of litigation's results but also from the burden of defending themselves.


I think it is safe to say that taking bribes of cash ;wrapping it up in tin foil and hiding it in your freezer does not fall under the protection of 'legitimate legislative activity'.You will recall of course that in 1994 Daniel Rostenkowski tried to invoke the clause to protect him from his shady dealings and the courts rejected his claim to immunity .

In Jefferson's case the FBI didnt just rush in and ransack Jeffersons office. First, they subpoenaed the records. Jefferson and his lawyers ignored it for months. They then obtained a 95-page search warrant from a judge, who issued it. The criticism should tehn also be aimed not only at the executive , but the judiciary as well. They should be pleased ...there was judicial review .

Of note : in the search warrent is a section entitled GOVERNMENT EFFORTS TO EXHAUST ALL LESSER INTRUSIVE APPROACHES TO OBTAINING RELEVANT DOCUMENTS AND RECORDS LOCATED IN THE WASHINGTON, D.C. CONGRESSIONAL OFFICE OF WILLIAM J. JEFFERSON.

Here are the safeguards that the FBI was required to use during the search :

The government has exhausted all other reasonable methods to obtain these records in a timely manner short of requesting this search warrant. A member of Congressman Jefferson's staff has indicated to law enforcement agents that records relevant to the investigation remain in Congressman Jefferson's Capitol Hill office, which the government has been unable to obtain to date. Left with no other method, the government is proceeding in this fashion.


Then the warrant describes a set of extraordinary procedures involving not only prosecutors but also the Court, by which Congressional privileges will be respected:


To ensure the prosecution team does not inadvertently review any potentially politically sensitive, non-responsive items in the office, or information that may fall within the purview of the Speech or Debate Clause privilege, or any other pertinent privilege, the physical search of the office will be conducted by special agents from the Federal Bureau of Investigation who have had no substantive role in the investigation.

The non-case agents will remove from the office those paper records determined to be responsive [to a detailed list of things to be search for included at the end of the warrant request]Other than as required to determine responsiveness, the non-case agents will not disclose to anyone any politically sensitive and non-responsive items inadvertently seen by the non-case agents during the course of the search of the office and will attest in writing to their compliance with this procedure.

Before giving any paper records seized from the office to the prosecution team, the non-case agents will deliver the seized paper records to the designated Filter Team.Prior to their appointment, the Filter Team will have had no role or connection to the investigation in this matter and their subsequent roles in the investigation will be confined to their duties and responsibilities in connection with these special procedures.

The Filter Team will review the paper records seized from the office to validate that they are responsiveAny paper records seized from the office that are determined by the Filter Team to be unresponsivewill be promptly returned to the office

Paper records validated by the Filter Team as responsivewill undergo a second level of review by the Filter Team. The Filter Team will review the responsive records to determine if they may fall within the purview of the Speech or Debate Clause privilege or any other pertinent privilege

For those paper records determined by the Filter Team as potentially within the purview of the Speech or Debate Clause privilege, or any other pertinent privilege, the Filter Team shall provide a log of those potentially privileged paper records to counsel for Congressman Jefferson. The log shall identify the record by date, recipient, sender and subject matter.The Filter Team shall not provide the log or copies of the potentially privileged paper records to the prosecution team, unless otherwise ordered by the Court

The Filter Team shall then request the District Court to review the potentially privileged paper records in order for the Court to name a final determination whether they contain privileged information, unless counsel for Congressman Jefferson consents to the production to the prosecution team of certain of the potentially privileged paper records.


Basically what Jefferson was doing was hiding pertinent records requested by the justice dept. inside his office because he assumed that his office was a safe haven for for him to conduct his criminal activities.


The Congress could easily fix this problem . They could expel Jefferson for his criminal activity . Pelosi could show some leadership and insist that the democrats adopt the same criteria for removal that the Republicans used to remove Tom Delay ,and Duke Cunningham ;especially if Pelosi is so concerned about the democrat culture of corruption . Of course she did ask politely that he remove himself from the 'ways and means 'committee which he respectfully declined to do .


Hastert may be feeling the heat himself so he is motivated in this issue .




ETWolverine rated this answer Excellent or Above Average Answer
Fritzella rated this answer Excellent or Above Average Answer

Question/Answer
Fritzella asked on 05/23/06 - Republican Party Imploding

"President Bush's nationally televised address on immigration Monday night was intended as a grand gesture to revive his collapsing presidency, but instead he has plunged the Republican Party into a political centrifuge that is breaking it down into its raw elements, which are colliding into each other, triggering explosions of unexpected and ever greater magnitude.

The nativist Republican base is at the throat of the business community. The Republican House of Representatives, in the grip of the far right, is at war with the Republican Senate. The evangelical religious right is paralyzed while the Roman Catholic Church has emerged as a mobilizing force behind the mass demonstrations of millions of Hispanic immigrants. Every effort Bush makes to hold a nonexistent Republican center is generating an opposing effect within his party." Sidney Blumenthal -- Salon dot com

~~~~~~~~~~~~~~~~~~~~~~~~~~~~~~~~~~~~~~~~~~~~~~~

tomder55 answered on 05/24/06:

we need to at least replace all the aborted by the boomers if we are to fund their retirements .

Bush should've led by enforcing the existing laws .But he didn't so yes ;now the Republican coalition is in danger of disintegrating .

Fritzella rated this answer Poor or Incomplete Answer
Itsdb rated this answer Excellent or Above Average Answer

Question/Answer
excon asked on 05/24/06 - Pot is GOOD for you


Hello drugwarriors:

Dr. Donald Tashkin, a professor at the David Gefin School of Medicine at the University of California, conducted a study funded by the National Institute on Drug Abuse. It says that even heavy marijuana smokers were NO more likely to develop lung, head or neck cancer than non users, in contrast with tobacco users.

The finding are a surprise (to them) because marijuana smoke has some of the same cancer causing substances as tobacco smoke, often in higher concentrations. One possible explanation, Tashkin said, is that THC, a key ingredient in marijuana, may inhibit tumor growth.

Funded by the National Institute on Drug Abuse, Taskin states that his study was intended to confirm our suspicions that marijuana might be a risk factor for lung, head and neck cancer. We havent been able to confirm that.

Hmmmm, whoda thunk it?

excon

PS> The one thing they didnt study is whether white women, under the influence of marijuana, fall uncontrollably, under the spell of and into the clutches of dark skinned males.

tomder55 answered on 05/24/06:

I am pleased that Tashkin is continuing his great research on the drug. He is probably the leading researcher on marijuana in the country .So his work has to be taken seriously .

Still ,other findings by him have resulted in data that should concern pot smokers regarding their respiratory health .

He has reported on the pulmonary consequences of habitual marijuana use, which include symptoms of chronic bronchitis, an increased frequency of acute chest illnesses, a heightened risk of pulmonary infection, and an apparently increased risk of disease in both the upper airway and the lungs. Smoking produces significant damage to the cilia in the lining of the airways. "The damage to the ciliated cells in the lining of the airways caused by smoking tobacco, and/or marijuana weakens the ability of the lungs to remove inhaled particles, making the lungs more vulnerable to infection," says Dr. Tashkin.

At first, the cilia-covered cells that sweep soot out of the lungs begin to die off, replaced by mucous-producing cells and other tissue cells that proliferate far beyond normal. In more than half the volunteers who smoke either marijuana or tobacco, the cells eventually became highly abnormal and skin-like in texture, a condition that doctors consider precancerous.

"The cells are disorganized," Tashkin said. "It doesn't mean they're going to turn into cancer, but it's on the way to cancer. These changes were as apparent in the marijuana-only smokers and were as common as in the tobacco-only smokers."



ETWolverine rated this answer Excellent or Above Average Answer
excon rated this answer Excellent or Above Average Answer

Question/Answer
Itsdb asked on 05/24/06 - Nailed it...

tomder55 answered on 05/24/06:

We will impose sanctions !!! but < wink > keep the oil fowing ... what genocide ? ..I see nuthsink ...

Itsdb rated this answer Excellent or Above Average Answer

Question/Answer
excon asked on 05/23/06 - Solution to Gitmo


Hello:

Here's a solution to Gitmo that I would approve of. Even the Wolverine might like it.

Just open the doors. Let Castro deal with them.

excon

PS> Viva La Marielitos

tomder55 answered on 05/24/06:

yeah ;they'd love it in a Castro prison .But I think it more likely he would hold a ticky tape parade around Plaza de la Revolucion ,have Hugo Chavez as his special guest, and then set them on a boat destined to Miami .

I know some of the Marielitas ;they were not all criminals .

excon rated this answer Excellent or Above Average Answer

Question/Answer
Itsdb asked on 05/23/06 - What the...?

On May 18th, before Cindy Sheehan, Code Pink and others of the moonbat society tried to deliver their Don't Attack Iran petition to Bush at the 'crime scene' (the White House) and protested at Rumsfeld's house, she gave a speech. Curious as to what Cindy had to say I watched some of the video of her speech (painful as it was). In her speech she said the following:

    Peace is not an absence of conflict, it's solving conflict nonviolently. We'll always have conflict in the world but killing to solve that conflict is barbaric. And we will never have a 'matriotic society' until we hold our leaders accountable.


Is this some new feminist movement, a 'matriotic society?' You can, according to Cindy, be a male or female Matriot, but I think I'll pass.

Guys, should we take some time off and let women run the world for a few days? How do you think it would go? I guess these 'matriots' think they can nurture al Qaeda into submission.

Comments?

tomder55 answered on 05/24/06:

Cindy Sheehad ..your 15 minutes of fame was over 6 months ago .She's put a lot of milage on her grieving matriarch act . I see she's taken her travelin road act to Aussie this week .Matriotism hmmmm. By her own words Matriotism is the opposite of patriotism. That about tells all .



I admire President Chavez for his strength to resist the United States,





Cindy Sheehad at her book signing :



Itsdb rated this answer Excellent or Above Average Answer

Question/Answer
Fritzella asked on 05/23/06 - Senator Dodd to run for President(D) in 2008

WASHINGTON (CNN) -- U.S. Sen. Christopher Dodd said that he is eyeing a presidential bid in 2008, joining a packed field of Democrats considering a White House run.

The Connecticut Democrat outlined his decision in Tuesday editions of The Hartford Courant, his state's largest newspaper.

"This is the right time for me," he told the paper. "This is the right thing to do."

Dodd, who turns 62 Saturday, is serving his fifth term in the Senate."

~~~~~~~~~~~~~~~~~~~~~~~~~~~~~~~~~~~~~~~~~~~~~~~~

I thought he was dead.

As I remember, he is a Moderate Democrat.

Any comments?

tomder55 answered on 05/24/06:

You must be thinking of his dad who was a strong anti-communist Democrat .Dodd opposed Ronald Reagan's anti-communist efforts, calling them "folly, pure and simple," and saying Mr. Reagan was waging "a conflict that can't be won." It was.
Moderate democrat is a relative term .I consider Lieberman a moderate democrat but the radicals in the party are trying to find a primary candidate to oppose him .

If you look at Dodd's voting record you see that he primarily tows the Democrat line with a decidely left of center view in particular to Latin America. He gushes over Castro;where he has a long term goal of normalizing relations with the dictator .Before that he was a big supporter of the Sandinistas .


Fritzella rated this answer Excellent or Above Average Answer

Question/Answer
Erewhon asked on 05/23/06 - Why Ameica needs Immigrant Labour ...

A Job Americans Won't Do, Even at $34 an Hour
Some landscape firms rebut claims that higher pay, not immigration reform, is needed.
By David Streitfeld, Times Staff Writer
May 18, 2006

Cyndi Smallwood is looking for a few strong men for her landscaping company. Guys with no fear of a hot sun, who can shovel dirt all day long. She'll pay as much as $34 an hour.

She can't find them.

ADVERTISEMENT

Maybe potential employees don't know about her tiny Riverside firm. Maybe the problem is Southern California's solid economy and low unemployment rate. Or maybe manual labor is something that many Americans couldn't dream of doing.

"I'm baffled why more people do not apply," Smallwood says.

President Bush is not. In his speech to the nation Monday night, he referred to "jobs Americans are not doing," echoing a point he has been making for years. To fill these spurned jobs and keep the economy humming, Bush says, the U.S. needs a guest worker program.

Otherwise, the logic goes, fruit will rot in the fields, offices will overflow with trash and lawns and parks will revert to desert.

Countering that view, opponents of a guest worker program say that Americans would find the jobs more enticing if there wasn't foreign competition to swell the labor pool and push wages down.

Smallwood is ambivalent on immigration reform, saying demands for immediate citizenship by those who entered the country illegally are offensive. But without a guest worker program, she says, her company probably will not survive.

"To get workers, you have to steal them from other companies," the 54-year-old entrepreneur says.

Even that has been unproductive recently. She'd ideally like to add eight employees by the end of the year to her current staff of 12.

The lawn and landscape business in California is heavily Latino, with an abundance of illegal immigrants. In a study of Los Angeles County's "off-the-books" labor force, the Economic Roundtable, a nonprofit research organization, estimated that a quarter of the landscape workers were undocumented. That leaves the companies vulnerable to crackdowns, which has them agitating for guest workers.

At Smallwood's company, Diversified Landscape Management, there's one white employee, an engineer. The other employees are Latino and, as far as Smallwood can tell, all in the country legally. Her employees need driver's licenses and the ability to move through freeway checkpoints near the border, which tend to eliminate any with fake papers.

Thirty years ago, those in the landscape industry say, white crews were common. Now, says Jim Newtson, a San Diego contractor, "if you see a white guy, you do a double-take, like when you saw an interracial couple back in the 1960s."

Managers in the business explain it as a cultural shift, saying that native-born, middle-class Americans of all races and ethnic backgrounds tend to look down on manual labor. That leaves immigrants to do the work.

"The people I grew up with 40 years ago expected to work hard physically," says Bob Wade of Wade Landscape in Laguna Beach.

"This is a pretty pampered little town. The kids don't expect to work hard," Wade says. "A lot don't expect to work at all. They just float."

Wade fired one employee three times, the last time for going to look at girls on the beach instead of spraying weeds. The employee his son now works in the restaurant industry.

Larger economic forces come into play too. Orange County, for example, consistently has the lowest jobless rate in the state. Although that could be a draw for laborers in states with high unemployment, the high housing prices in the county act as a brake on that sort of migration.

Smallwood grew up doing manual labor. The daughter of a sharecropper in Mississippi, she had to pick her share of cotton from age 6. "I wouldn't do that again for any price," she says.

When she moved to California, she worked as a property manager, then developed a lawn-care business, which she sold in 1998. The death of her only child, Michael, from a drug overdose two years later drew her outside to her own garden. "I watered, fertilized, planted and pruned, determined that nothing else was going to die on me," she says.

===

Will you take the job?

tomder55 answered on 05/23/06:

$34 /hr for a 40hr week translates to $70,720 annually . She is not advertising her offer enough or she is full of tostidos .

Elliot got it right ;she has an agenda .

Erewhon rated this answer Excellent or Above Average Answer
ETWolverine rated this answer Excellent or Above Average Answer
Itsdb rated this answer Excellent or Above Average Answer

Question/Answer
jackreade asked on 05/23/06 - Containment

"An old word is gaining new currency in Washington: containment. You may be hearing a lot more of it as the Bush administration hunkers down for its final two years. Containment of Iraq's low-level civil war, which shows every sign of persisting for years despite the new government inaugurated this week. Containment of Iran's nuclear power, which may lead to a missile defense system in Europe. Containment of the Islamism revived by Hamas and Hizbullah, by the Sunni suicide bombers in Iraq, as well as by the "Shiite Crescent"--as Jordan's King Abdullah once called it--running from Iran through Southern Iraq and into the Gulf.

During the cold war, containment doctrine was based on the premise that the Soviet Union was a powerful force that was going to be around for a long time to come. Containment's chief author, George Kennan, concluded that the best Washington could do was to keep the Soviet bloc penned up in its sphere of influence until it expired of its own internal problems (though Kennan later despaired that containment had become too militarily focused, culminating in Vietnam). The policy was carefully laid out in NSC-68, the basic blueprint for containment, in the spring of 1950. Forty years later, the policy succeeded." Michael Hirsh, Newsweek

~~~~~~~~~~~~~~~~~~~~~~~~~~~~~~~~~~~~~~~~~~~~~~~~

So, back to containment, a new Cold War?

It seems to me that the Islamic countries are stronger than ever after the invasion of Iraq.

tomder55 answered on 05/23/06:

Containment cost the US over 100,000 combat deaths during the cold war and did not succeed until the Soviet Union was out spent militarily . It was based on a premise that there would be a rational leader in the Kremlin at all times and even then we came to the brink of nuclear exchange more than once. The most notable of course was the Cuban missle crisis but there were other incidents that are not as prominent in the history books . I am far from convinced that Ahmadinejad is rational and I know that Kim Jong Il is not .

But in either case they are satellite nations to the real adversary in the new geo-political struggle; the nation that has it's fingerprints on all the proliferation of nuclear technology around the world ;the nation that has hegomic desires in the Pacific.

As far as Iraq is concerned we tried containment for 12 years after Desert Storm and the policy was on the brink of failure before OIF. Our 'friends and allies ' made sure that it would fail .

Itsdb rated this answer Excellent or Above Average Answer
jackreade rated this answer Excellent or Above Average Answer

Question/Answer
jackreade asked on 05/22/06 - Merciary Army?

NEW YORK "Little known to the American public, there are some 50,000 private contractors in Iraq, providing support for the U.S. military, among other activities. So why not go all the way, hints Ted Koppel in a New York Times op-ed on Monday, and form a real "mercenary army"?

Such a move involving what he calls "latter-day Hessians" would represent, he writes, "the inevitable response of a market economy to a host of seemingly intractable public policy and security problems."

The issue is raised by our "over-extended military" and inability of the United Nations to form adequate peace forces. Meanwhile, Americans business interests grow ever more active abroad in dangerous spots.

"Just as the all-volunteer military relieved the government of much of the political pressure that had accompanied the draft, so a rent-a-force, harnessing the privilege of every putative warrior to hire himself out for more than he could ever make in the direct service of Uncle Sam, might relieve us of an array of current political pressures," Koppel explains, tongue possibly in cheek."

~~~~~~~~~~~~~~~~~~~~~~~~~~~~~~~~~~~~~~~~~~~~~~~~~

What about a merciary army?

tomder55 answered on 05/22/06:

Eleanor Cliff the detestible pundit on "The McLaughlin Group" already beat Koppel to the punch by suggesting last year that our all volunteer military was already a mercenary force.Of course she couched her comments by appearing to support the concept of pay increases for the military .

"But I think what we're coming to grips with is the fact that we actually have a mercenary Army," ..."And it doesn't have a nice ring to it. We call it 'volunteers,' but we're basically paying people to serve their country. And if you're going to pay people and have a mercenary Army, you're going to have to pay the market rate. And so the bounties are going up -- more money for tuition, higher enlistment bonuses -- and I think it's appropriate."

So any soldier who receives compensation is a mercenary ? What the dope failed to grasp was that even when we had draftees they were paid for their service . I bet her takehome pay is much more than any careerist's in the services .

So Koppel left ABC to become a pundit for the Slimes? Guess that job at al-Jazerra was filled ? I bet he is relieved he won't be recounting the names of the dead mercenaries anymore . I do not know what facts Koppel has been looking at recently because his editorial is only available on line for paid subscritions and I would not waste a dime on that rag ; but here are some facts for Ted .

In the last seven months, the U.S. Army has met or exceeded all of its recruiting goals. In that time, over 160,000 people have enlisted, or re-enlisted. The total strength of the active duty and reserve forces are 1.2 million men and women, all of them volunteers.

Except for a few months in 2004-5, the military has been able to maintain its strength, despite wartime conditions. The biggest problem has not been casualties but the disruption to family life caused by so many troops getting sent to combat zones. This discouraged re-enlistments in reserve units, although mainly among the non-combat troops. In combat units, re-enlistments were at record levels.

Faced with that problem the military has for years now hired civilian contractors to provide services to the military that used to be filled by rank and file . He is not saying anything new or interesting except for the fact that his tongue-in-cheek sarcasm sells well with the moonbat brigade .Actually what is particularly telling is how well recruiting is going in a exploding economy.



When will the NY Slimes publically announce that they have officially changed their format from serious news to tabloid ?


http://www.whitehouse.gov/news/releases/2006/05/print/20060511.html

http://www.defenselink.mil/news/May2006/20060510_5080.html

http://www.defenselink.mil/releases/2006/nr20060510-13008.html



ETWolverine rated this answer Excellent or Above Average Answer
Itsdb rated this answer Excellent or Above Average Answer
jackreade rated this answer Average Answer

Question/Answer
excon asked on 05/21/06 - Dead Soldiers


Hello citizens:

In Vietnam, when they reported the American dead, as a feel good measure (because it was probably a lie), they at least reported a decisively larger number of enemy dead. Why arent they doing that in Iraq?

excon

tomder55 answered on 05/22/06:

it's perceptions . the human cost of war and the prospect of collateral damage must figure centrally in any decision to go to war when the press is willing to exploit the explosion of a single car bomb. but you see it is only perceptions when regarding US military action . It took years for the press to pay attention to the massacres that occure daily in Sudan and other places in the world where total war is waged like the Russian detruction of the city of Grozny .

So the Penatgon has no choice but to wage a perception war by trying to manage the information and shape the coverage of American operations abroad. They have largely been unsuccessful in this effort as the media has had an agenda to 'get Bush' .

How do I know this is so ? Well thousands of Serbs died during the seventy-eight day NATO air campaign in 1999 and the press barely reported on it .

I don't know ;perhaps it only matters if there are troops on the ground . We flew thousands of sortees against Iraq in the periods between Iraq wars and dropped thousands of tons of arsenal including cruise missles .I do not recall casualties being high on the worry list of the press then.

excon rated this answer Excellent or Above Average Answer

Question/Answer
Erewhon asked on 05/22/06 - Why did Bush lie?

NYT - May 21, 2006
Editorial
An Immigration Bottom Line

This week starts the endgame for immigration reform in the Senate. Months of debate have come down to this: whether the comprehensive solution at the core of the Senate bill will survive the hostile attentions of those who do not want real reform at all. A brace of amendments has already warped and weakened the bill though not fatally, thanks to a bipartisan coalition that has fended off repeated attempts at sabotage. But there is still a danger that any legislation will be further compromised or even gutted to conform with the House's deplorable bill.

A good immigration bill must honor the nation's values and be sensible enough to work. It must not violate the hopes of deserving people who want to work toward citizenship. It must not create a servant class of "guest workers" shackled to their employers and forbidden to aspire to permanent legal status. It must give newcomers equal treatment under the law and respect their rights of due process. It must impose rigorous enforcement of labor laws, so unscrupulous employers cannot exploit illegal workers. And it must clear the existing backlogs of millions seeking to enter the country legally, so that illegal immigrants do not win an unfair place in line.

'Amnesty' and the Mythical Middle Ground. The Senate is the only hope for real reform this year because the House has already chosen its plan. It wants to wall off Mexico, turn 11 million or so illegal immigrants into an Ohio-size nation of felons, and then pick them off through arrests, deportation and an atmosphere of focused hostility until they all go home, abandoning their families and jobs.

That spirit of wishful hunkering has infected the Senate, where Democrats and moderate Republicans have had to struggle against the obstinacy of those who join their counterparts in the House in seeing immigration entirely as a pest-control problem. President Bush has aligned himself with the thoughtful reformers, but in a slippery way. "There's some people in our party who think, you know, deportation will work," Mr. Bush said on Thursday. "There are people in the other party that want to have automatic amnesty. As I said in my speech, I've found a good middle ground."

But nobody favoring the Senate bill wants automatic amnesty. It imposes a long and difficult path to citizenship. Illegal immigrants must have a clean record and a job, speak English and pay a big fine. That is what the president wants, though he tries not to say it. His mildness has only validated the efforts of those who cling to the enforcement-only delusion, and who have tried so hard to strip the Senate bill of any meaningful paths to citizenship.

Mr. Bush should have joined the debate far earlier and more assertively, insisting that the "middle ground" lies nowhere near those who refuse any accommodation and favor mass deportations.

The Border Fixation. An immigration solution cannot be focused only on the border. We've tried that. Border enforcement has swelled in the last 20 years, with no visible effect. Mr. Bush's plan to send National Guard troops was seen on both sides, rightly, as a ploy to placate the xenophobes. It would be good to expand the Border Patrol. But the best help we can give it is to enforce workplace rules, ease the pressure for visas and restore law and order in a comprehensive way.

The Enforcement Gap. The value of illegal immigrants to many employers is their fearful willingness to work for low pay in bad conditions. People who are secure in their status will stand up against abuses, leading to better treatment for all. Workplace enforcement is one tactic. Employers who risk real punishment will be less likely to flout the rules. But guest worker programs without the citizenship option are also an invitation to worker abuse, and a shameful abdication of America's values. Mr. Bush has taken this path. Congress must not.

Fairness and Workability. The current bill divides the 11 million to 12 million illegal immigrants into three groups. Those who arrived less than two years ago would have to go home. Those who have been here for two to five years would be treated as guest workers, and would have to leave and re-enter the country to keep that status. The rest would be able to seek citizenship.

Will this cumbersome bureaucratic solution work? It depends on the willingness of the two-to-five-year group to step forward. For immigration reform to succeed it must lure people out of the shadows a goal that may be fatally compromised by the punitive hoops the bill erects.

Another profound shortcoming of the bill is its harsh criminal-justice provisions. It greatly expands the types of immigration-related offenses that constitute "aggravated felonies" and thus grounds for detention and deportation. People who use false passports to flee persecution, for example, might be ensnared. The bill increases penalties and the risk of deportation for minor infractions, like failing to file a change of address form. It removes judges' and immigration officers' discretion to weigh individual circumstances, adding toughness at the cost of fairness and decency.

The Xenophobia Problem. The Senate's debate has laid bare a hostility to immigrants that is depressing in its spitefulness and vigor. From Senator James Inhofe's amendment declaring English the national language to one from Jon Kyl that would have barred low-skilled guest workers from seeking permanent status to another from John Ensign that would have denied Social Security credit for work done before an immigrant is legalized, the debate has been littered with attempts to stifle, stymie or blow up the process.

The bipartisan coalition pursuing thoughtfulness over such simplistic hostility has proved sturdy so far. The senators who have fashioned the consensus for comprehensive reform must stick together, or the possibility of a solution this year will die, along with the hopes of millions.
============

Why did Bush lie about "the other party"?

Curious minds want to know!


tomder55 answered on 05/22/06:

he did not lie about the other party ...there are others in the demoncrats who do want automatic amnesty . The NY Slimes is playing semantics games again .

Laying out their own principles for revamping the nation's immigration laws in response to what House Minority Leader Nancy Pelosi called President Bush's "political ploy," Democrats went beyond Mr. Bush's plan for a temporary-worker program and called for a system of "earned legalization" for illegal aliens.
At a Capitol Hill press conference, Democrats proposed allowing illegal immigrants who have worked in the United States for a yet-to-be-determined minimum period of time to stay here and be granted permanent legal residency, creating a "pathway" to eventual citizenship.


Now this comes from the democrat leader in the House of Reps. What she is proposing is amnesty by an other name.

In the Senate the appointed Sen. from NJ Robert Menendez is a vocal advocate of amnesty although to be politically correct he says that because it is not "blanket" amnesty he proposes then it is not amnesty . That is the same semantics that the Slimes uses .

Erewhon rated this answer Excellent or Above Average Answer

Question/Answer
CeeBee2 asked on 05/21/06 - Ethnic cleansing in Iraq?

ZNet | Iraq

Iraq Is Disintegrating

As ethnic cleansing takes hold across central Iraq, there is an exodus of people fleeing for their lives as sectarian assassins and death squads hunt them down.

by Patrick Cockburn ; May 20, 2006

Khanaqin, North-East Iraq. The state of Iraq now resembles Bosnia at the height of the fighting in the 1990s when each community fled to places where its members were a majority and were able to defend themselves. "Be gone by evening prayers or we will kill you," warned one of four men who called at the house of Leila Mohammed, a pregnant mother of three children in the city of Baquba, in Diyala province north-east of Baghdad. He offered chocolate to one of her children to try to find out the names of the men in the family.

Mrs Mohammed is a Kurd and a Shia in Baquba, which has a majority of Sunni Arabs. Her husband, Ahmed, who traded fruit in the local market, said: "They threatened the Kurds and the Shia and told them to get out. Later I went back to try to get our furniture but there was too much shooting and I was trapped in our house. I came away with nothing." He and his wife now live with nine other relatives in a three-room hovel in Khanaqin.

The same pattern of intimidation, flight and death is being repeated in mixed provinces all over Iraq. By now Iraqis do not have to be reminded of the consequences of ignoring threats.

In Baquba, with a population of 350,000, gunmen last week ordered people off a bus, separated the men from the women and shot dead 11 of them. Not far away police found the mutilated body of a kidnapped six-year-old boy for whom a ransom had already been paid.

The sectarian warfare in Baghdad is sparsely reported but the provinces around the capital are now so dangerous for reporters that they seldom, if ever, go there, except as embeds with US troops. Two months ago in Mosul, I met an Iraqi army captain from Diyala who said Sunni and Shia were slaughtering each other in his home province. "Whoever is in a minority runs," he said. "If forces are more equal they fight it out."

It was impossible to travel to Baquba, the capital of Diyala, from Baghdad without extreme danger of being killed on the road. But I thought that if I took the road from Kurdistan leading south, kept close to the Iranian border and stayed in Kurdish-controlled territory I could reach Khanaqin, a town of 75,000 people in eastern Diyala. If what the army captain said about the killings and mass flight was true then there were bound to be refugees who had reached there.

I thought it was too dangerous to go beyond the town into the Arab part of Diyala province, once famous for its fruit, since it is largely under insurgent control. But, as I had hoped, it was possible to talk to Kurds who had sought refuge in Khanaqin over the past month.

Salam Hussein Rostam, a police lieutenant in charge of registering and investigating people arriving in terror from all over Iraq, gestured to an enormous file of paper beside him. "I've received 200 families recently, most of them in the last week," he said. This means that about one thousand people have sought refuge in one small town. Lt Rostam said that the refugees were coming from all over Iraq. In some cases they had left not because they were threatened with death but because they were fired from their jobs for belonging to the wrong community. "I know of two health workers from Baghdad who were sacked simply because they were Kurds and not Shia," he said.

This was probably because the Health Ministry in Baghdad is controlled by the party of Muqtada al-Sadr, the Shia cleric.

The flight of the middle class started about six months after the invasion in 2003 as it became clear Iraq was becoming more, not less, violent. They moved to Jordan, Syria and Egypt. The suicide bombing campaign was largely directed against Shias who only began to retaliate after they had taken over the government in May last year. Interior Ministry forces arrested, tortured and killed Sunnis.

But a decisive step towards sectarian civil war took place when the Shia Al-Askari shrine in Samarra was blown up on 22 February this year. Some 1,300 Sunni were killed in retaliation.

Kadm Darwish Ali, a policeman from Baquba and now also a refugee, said: "Everything got worse after Samarra. I had been threatened with death before but now I felt every time I appeared in the street I was likely to die."

Every community has its atrocity stories. The cousin of a friend was a Sunni Arab who worked in the wholly Shia district of Qadamiyah in west Baghdad. One day last month he disappeared. Three days later his body was discovered on a rubbish dump in another Shia district. "His face was so badly mutilated," said my friend, that "we only knew it was him from a wart on his arm."

Since the destruction of the mosque in Samarra sectarian warfare has broken out in every Iraqi city where there is a mixed population. In many cases the minority is too small to stand and fight. Sunnis have been fleeing Basra after a series of killings. Christians are being eliminated in Mosul in the north. Shias are being killed or driven out of cities and towns north of Baghdad such as Baquba or Samarra itself.

Dujail, 40 miles north of Baghdad, is the Shia village where Saddam Hussein carrying out a judicial massacre, killing 148 people after an attempt to assassinate him in 1982. He is on trial for the killings. The villagers are now paying a terrible price for giving evidence at his trial.

In the past few months Sunni insurgents have been stopping them at an improvised checkpoint on the road to Baghdad. Masked gunmen glance at their identity cards and if under place of birth is written "Dujail" they kill them. So far 20 villagers have been murdered and 20 have disappeared.

*************************
Now what should the U.S. do?

tomder55 answered on 05/21/06:

news

The toll in Sao Paulo rose to 170 on Thursday and growing .Human Rights Watch said some of the of police may have been summary s. This has harldy made the news ;but a drive by that claimed 3 in Iraq made page after page of copy on Google.

I only bring that up because such is hardly the exception in the world . I could've used a number of African countries as an example or even France post liberation after the fall of the Vichy government where we had many more occupying troops there then we ever will have in Iraq.After the liberation, France was swept with a wave of assassinations.And speaking of former Yugoslavia;after WWII ;The Partisans executed many Ustashe and Chetniks, as well as their collaborators. One of the best documented incidents was the Bleiburg massacre where 50,000 disarmed soldiers and 30,000 civilians were ed .

The Phillipines is one of the largest democracies in the world even though they too have a thirsty insurgency. The insurgency in Iraq is defeated except for pockets of squads and ists .

There is no civil war in Iraq anymore than there is one in Jordan, Saudi Arabia or Egypt. These countries also have Islamo- ts ing people. Earlier this month Iraqi tribal leaders met in Kazimiyah mosque to sign an honor compact to denounce and reject ism and sectarian . Bet you did not hear that on the evening news .

The ists and squads are trying to frighten Iraqis and the Americans away by their hideously brutal and often indiscriminate attacks, but they promise the Iraqi people only shed and barbarism. The coalition, on the other hand, is helping the vast mass of the Iraqi people to build a better future.

Of course other headlines like the swearing in of the unity gvt. was not mentioned anywhere in the article above. Despite the nay sayers this gvt. is universally recognized as the legitimate gvt. of Iraq.

The Secretary-General wishes the new Government every success in confronting the enormous challenges facing Iraq, a spokesman for Mr. Annan said in a statement released in New York. He hopes that the process of forming a broad-based and inclusive government will be completed as soon as possible, so that it will be able to quickly address the crucial issues of national reconciliation, security, the rule of law, respect for human rights, reconstruction and development.

The Secretary-General also paid tribute to the courage and determination that the Iraqi people have shown despite the ongoing , urging them to seize the opportunity offered by the successful establishment of a broad-based and inclusive Government, and come together to support it and build the foundations of a united, peaceful and prosperous Iraq.

He reiterated the UN's commitment to fully support the new Government and the people of Iraq, a pledge echoed by his Special Representative, Ashraf Qazi.


That is not to say that it is yet a stable gvt. but as I mentioned above that is hardly a rarity in the world today .There are still a great many security issues to resolve. The US will have an important role in this of course but ultimately the responsibility of success or failure is in the Iraqis hands.That's what "sovereignty" is all about .

Yesterday should be a day of celebration but the media of course would not allow it .According to the Brookings Institution ;On an index of political freedom for countries in the Middle East, Iraq now ranks fourth, just below Israel, Lebanon, and Morocco.

We liberated Iraq from one of the worst mass inging dictators in our time.But even those who were opposed to the invasion of Iraq should recognise that this is a whole new battle between the values of a liberal civil society and nihilism. What should the US do ?

Let every nation know, whether it wishes us well or ill, that we shall pay any price, bear any burden, meet any hardship, support any friend, oppose any foe, in order to assure the survival and the success of liberty.
JFK












CeeBee2 rated this answer Excellent or Above Average Answer
ETWolverine rated this answer Excellent or Above Average Answer
jackreade rated this answer Excellent or Above Average Answer

Question/Answer
excon asked on 05/19/06 - The Prince of Pot


Hello Drugwarriors:

Marc Emery, a Canadian citizen, is facing extradition to the United States, as a drug kingpin. He faces the death penalty if the DEA gets their hands on him. Or, if they decide to be nice to him, he'll serve 27 years in the slam because of mandatory minimums.

In Canada, however, no one has ever been sentenced to jail for selling seeds, and only two people have ever been fined. Marc in 1996 and 1998; and Ian Hunter, fined $200 in the year 2000.

Oh yeah, he only sells seeds. Hes made no money personally, and he gave all the profits from his seed business to the cause of legalizing marijuana. Hes really a pot activist - not a drug dealer. He lives in an apartment and drives a leased car.

Should we kill him?

excon

tomder55 answered on 05/19/06:

Nope let him serve some time. Personally I do not think he will be extradited . He has a claim in Canadian court that the arrest and extradition request is politically motivated . He is also claiming that extradition would constitute cruel and unusual punishment [Section Twelve of the Canadian Charter of Rights and Freedoms...I think it is safe to say that getting wacked for selling pot seeds constitutes 'cruel and unusual' ].He is also asserting that although it is a crime in Canada ,it is not enforced so therefore it is not REALLY an offense . Hey ...let the Canadians keep him .
Politically it is stupid . We do not need a Free Emery movement in this country like we have with that cop killer Mumia Abu-Jamal .

excon rated this answer Excellent or Above Average Answer

Question/Answer
labman asked on 05/19/06 - Priorities?

Selected headlines from Reuters: Top Stories on My Yahoo:

US urged to close Guantanamo

N.Korea may be preparing missile launch: reports

Iran now enriching home processed uranium: source

Rebel attack downs key Colombia oil pipeline: army

Three senior Taliban captured in Afghan clashes

Egypt rejects US criticism over Nour case

Saddam Hussein novel hits stores in Japan

In the top story the UN is saying we are violating international law. I doubt Guantanamo is a pleasent place, but I wonder if it needs to be a high priority for the UN today. Does it violate international law to hijack an airliner filled with civilians and crash it into a building filled with more civilians?

tomder55 answered on 05/19/06:

Where are these human rights lawyers while UN workers systematically rape 8 year old girls around the globe?

There is an intersting coincidence to all this . There was a mini-riot at Gitmo when detainees tried to prevent guards from saving the lives of other detainees who allegedly tried to commit suicide .Story here Yeah that's right .Our guards risked their lives trying to save the lives of these POS. !

I would also add that President Bush left the door opened to what will become even more pressure to close Gitmo down by declaring that he would very much like to end Guantanamoduring a German television interview.[mistake one] He said he had to wait until a Supreme Court Decision as to whether the people need to have a fair trial in a civilian court or in a military court [mistake two] . He should proceed with tribunals and not wait for a court decision. The reason they have been detained so long in limbo is because all these challenges to his authority to have the military conduct them. Starting with the Moussaoui case the judiciary has taken over the war and enemy trying to destroy the country are being given full due process. What's next ? Reading Miranda rights on the battle field ?

Even worse

Washington has been soliciting advice on how to rehabilitate detainees who might otherwise rejoin the jihad if freed. One of those consulted, Rohan Gunaratna, the author of Inside Al-Qaeda, said: The Americans are now seriously thinking of rehabilitation. ARRRRGH.... you gotta be kidding !!! Theyre jihadists and when they are released they will continue to be so . What are they planning on doing ? Brain wash them into denying their faith ? Yeah that always works !!

labman rated this answer Excellent or Above Average Answer

Question/Answer
excon asked on 05/19/06 - I support the ENTIRE Bill of Rights


Hello:

Lets say that a hurricane just devastated your city. People are on street with guns. Lawlessness prevails. When they finally decide to come to work, the cops first job is to confiscate all the guns, in order to make the city safe.

If you have a gun, (1) will you give it up? (2) If you do give yours up, will that make you feel safer? (3) Will you actually BE safer? (4) If you now dont own a gun, in times of lawlessness, would you rather have or not have a gun? (5) If youd rather not have a gun when all those around you do, and there are no cops around to protect you, please tell me why.

excon

tomder55 answered on 05/19/06:

I'm with you on this . I do not know if it is true that the N.O. police went door to door or not but the only ones who should've been disarmed were the ones using them in a crime .

excon rated this answer Excellent or Above Average Answer

Question/Answer
Itsdb asked on 05/19/06 - Moussaoui's sentence

tomder55 answered on 05/19/06:

He sure went from "I WON" to begging for a re-trial in a hurry .


perhaps this is why :

Itsdb rated this answer Excellent or Above Average Answer

Question/Answer
Itsdb asked on 05/19/06 - BRAVE NEW SCHOOLS

Jesus with erection' ignites outrage

Student newspaper publishes drawings in response to Muhammad 'toons
Posted: April 26, 2006
5:00 p.m. Eastern

2006 WorldNetDaily.com

A Catholic activist organization has written to Oregon's governor and state lawmakers to protest a University of Oregon student newspaper for having published cartoons showing Jesus Christ naked and with an erection.

In its March edition, the Insurgent, an "alternative" student paper on the Eugene, Ore., campus printed 12 hand-drawn cartoons of Jesus as a response to rival paper the Commentator having published the controversial cartoons of Muhammad originally published in Europe that sparked Muslim riots worldwide. The Insurgent claimed it published the drawings to "provoke dialogue."

William Donohue, president of the Catholic League, said the university's president, Dave Frohnmayer, had been unresponsive to complaints about the drawings, so he had written to the governor, every state legislators and the chancellor of the Oregon University System, among others.

Student newspaper publishes drawings in response to "The March edition of the Insurgent ... was one of the most obscene assaults on Christianity I have ever seen," Donohue said in a statement. "To make sure that the persons I wrote to understand how vile this attack was, I sent a photocopy of the two most offensive graphics: one was a depiction of a naked Jesus on the cross with an erection; the other, titled 'Resurrection,' showed a naked Jesus kissing another naked man, both sporting erections."

Donohue also says there were other depictions of Jesus on the cross that were "so gratuitously offensive that only the most depraved would defend them." He also noted the paper published two commentaries attacking Catholicism.

"That all of this appeared in a student newspaper, during Lent, on the campus of a state institution, makes one wonder what is going on at the University of Oregon," added Donohue.

While not describing the more sexual drawings, the main student newspaper at the university, the Oregon Daily Emerald, also criticized the Insurgent.

"The Insurgent editorial indicates a desire to show Americans why the original cartoons were so offensive to the Muslim world," wrote the editor of the Emerald. "According to the editorial, 'What is "not a big deal" in the US (sic) is apparently a humongous big deal to others. Why should we assume it would not be?'

"However, printing home-grown cartoons depicting Jesus on a cross/pogo stick or Jesus on a cross/hangliding apparatus are not inflammatory in the same manner as the anti-Islam cartoons, and therefore fail to produce the intended empathy from Christians to Muslims."

Added the paper: "Unlike the Danish cartoons, the Insurgent drawings seem intended to simply incite controversy for controversy's sake rather than making specific social commentaries."

A spokesman for Frohnmayer contacted WorldNetDaily after press time to say that the university president had posted a statement regarding the controversy surrounding the cartoons:

"I share your concern about the offensive nature of the content contained within the publication.

"I understand why it may seem as if the University should have prevented publication or should take some action against those responsible for the publication. The Student Insurgent is not owned, controlled or published by the University of Oregon and is funded with student fees. Therefore, the University cannot exercise editorial control over its content.

"The best response to offensive speech often is more speech. ... I am strongly opposed to speech that makes individuals feel that they or their beliefs are unwelcome or belittled, and I can assure you I will use all permissible means to respond to publications such as the recent Insurgent."

~~~~~~~~~~~~~~~~~~~~~~~~~~~~~~~~~~~~~~~~~~~~~~~~~~~~~~~

Last year, UO's Programs Finance Committee decided to de-fund a conservative campus magazine because a transgendered student was offended - by rejecting the magazine's 21 year old mission statement.

On Tuesday, UO ruled against the students grievance filed against The Insurgent, saying it "did not practice discrimination" while affirming the statute on contributing to the "physical and cultural development of students." Any bets on whether or not UO's PFC will try to de-fund The Insurgent?

Compared to the Mohammed cartoons what's the point of this? What possible good might The Insurgent think would come out of publishing these images?

How does this aid in the "cultural and physical development of students" as required by UO's statutes on using student fees? Comments?

tomder55 answered on 05/19/06:


This was when ? Last Month ? Funny ;I did not see any Christian rioting over this .No one got a close Nick Berg like shave over this ? I have to question the editors penis fixation however.






"I understand why it may seem as if the University should have prevented publication or should take some action against those responsible for the publication. The Student Insurgent is not owned, controlled or published by the University of Oregon and is funded with student fees. Therefore, the University cannot exercise editorial control over its content.

"The best response to offensive speech often is more speech. ... I am strongly opposed to speech that makes individuals feel that they or their beliefs are unwelcome or belittled, and I can assure you I will use all permissible means to respond to publications such as the recent Insurgent."


I wonder what his reaction would be if the "Insurgent " was a neo-Nazi publication ?

Itsdb rated this answer Excellent or Above Average Answer
paraclete rated this answer Bad/Wrong Answer

Question/Answer
jackreade asked on 05/18/06 - Faux News Now Racist TV Network

Faux news is now officially a racist network. The radical right is desperate to hold together the coalition of the radical Christians, traditional Republicans, and angry white guys---a coalition which is breaking down rapidly. Dobson of Focus on the Family issued a warning to Bush-none of the religious rights pet issues have been taken care of as promised, so don't expect votes from his group in the fall elections.

Gibson, a talking head on Faux news, gave an op-ed piece begging Americans to reproduce more rapidly because now half the babies born in America are of Hispanic(minority) heritage. (Yeah, white people, get it on!) It won't be very long(a few decades) BEFORE IN AMERICA WHITE PEOPLE WILL BE A MINORITY.

Several other talking heads have echoed Gibson's worries by repeating overtly race baiting comments and fear tactics toward the racist element in the Republican Party.

jack

tomder55 answered on 05/19/06:

Too late Shelby Steele absolves us from the quilt trip being layed down on us white guys .

ETWolverine rated this answer Excellent or Above Average Answer
Itsdb rated this answer Excellent or Above Average Answer
jackreade rated this answer Poor or Incomplete Answer

Question/Answer
paraclete asked on 05/19/06 - MI LI IS NOT IN IRAQ, IS IT?

Proving that nothing has changed in the military psyche in 30 years, the US now has another Mi Li on it's hands. Now come on tell me the buck doesn't stop at the top on this one? And you wonder why I think the attitude of the people is the same as the administration.

US soldiers massacred Iraqis: congressman


By Drew Brown in Washington
May 19, 2006
Related coverage

* War a mistake: Italy plans to pull out troops

A PENTAGON report will show the killing of more than a dozen Iraqi civilians in November was deliberate and far worse than first reported, a member of the US Congress has said.

"There was no firefight. There was no IED [improvised explosive device] that killed those innocent people," John Murtha, a Democrat and a former US marines colonel, told a news conference.

"Our troops overreacted because of the pressure on them. And they killed innocent civilians in cold blood," he said. "That is what the [US investigators'] report is going to tell." The comments were the first on the record by a US official over the November 19 incident, which is still being investigated by US military authorities.

Mr Murtha is a member of Congress's defence appropriations subcommittee and an opponent of the Bush Administration's Iraq policies.

Mr Murtha said he had not read the report but had learnt about its findings from military commanders and other sources.

"There is an ongoing investigation," said Lieutenant-Colonel Sean Gibson, a US marines spokesman at Central Command headquarters. "Any comment at this time would be inappropriate." Colonel Gibson and a Pentagon spokeswoman, Cheryl Irwin, said the military had yet to decide what, if any action, might be taken against the marines.

Three commanders whose troops were involved in the incident were relieved of their duties in April. The marines did not link their dismissals to the incident, saying only that their superior, General Richard Natonski, had lost confidence in their ability to command. All three were given staff jobs in the military.

The US military in Iraq initially reported that one marine and 15 Iraqi civilians travelling in a bus were killed by a roadside bomb in the western Iraq insurgent stronghold of Haditha. It said eight insurgents were killed in an ensuing firefight.

But Lieutenant-General Peter Chiarelli, the ground commander of coalition forces in Iraq, ordered an investigation on February 14 after a reporter with Time magazine told US authorities of claims the marines had killed innocent civilians.

After CNN broke the news of the initial investigation in March, military officials told Knight Ridder the civilians were killed not in the initial blast but were apparently caught in the crossfire of a subsequent gun battle as 12 to 15 marines fought insurgents from house to house over the next five hours. At the time, officials said four of the civilians killed were women and five were children.

Subsequent reporting from Haditha revealed a different account of events, with survivors describing marines breaking down the door of a house and shooting the occupants.

Mr Murtha said nothing indicated the Iraqis killed in the incident were at fault.

"One man was killed with an IED," he said, referring to a marine killed by the bomb.

"After that, they actually went into the houses and killed women and children."

Knight Ridder

tomder55 answered on 05/19/06:

Time magazine goes to Iraq, interviews supposed relatives of dead civilians who they claim were killed without provocation by American Marines after an IED attack on a US Marine convoy in or near their village.Good enough for Jack Murtha . Hang EM!!! When Matthews brought up the My Lai massacre, Murtha agreed that this case was like My Lai. But this case is not like My Lai, and Murtha knows it. Nothing these Marines are accused of resembles the murders at My Lai.Question for Murtha: If the Marines over-reacted then the killings were notpre-meditated so how could this be cold blooded murder ?

You would think he would have the sense and decency to wait until the final report had been released before potentially ruining the lives and careers of the accused? Apparently not.It is frankly sickening to watch him rush to the podium with glee to make his dispicable pronouncements .Murhta has the brass to convict the Marines and then ,to score political points ,he becomes a psychologist and declares them 'non compus mentis' by reason of "combat stress". The discredited Murhta has now assigned himself to be judge ,jury and executioner while he conducts his trial in the court of the MSM(Chris Matthews is his court jester ) . As a lawmaker he should be ashamed of himself by this presumption of guilt. His past military record does not give him a pass at this attempt at cheap political gain[Murtha has had his campaign coffers supplied with the Code Pink/ANSWER crowd and now he feels he has to slander Marines, before they have a chance to present their side. ].

Congress should censure him for his reckless and irreponsible rants regardless of the outcome of the investigation and Court Martial of these Marines.

Frankly ;I think they need to look a little closer into John Murtha's real motives .It appears to me that Murtha is beefing up his anti-war credibility, only so he can cry "they're questioning my patriotism/trying to silence me", when he gets indicted for influence peddling. Perhaps now we understand why he was perceived a hawk ...it was true as far as military procurement is concerned . Who new his motive was self-interest ?

People of Pennsylvania ;isn't it time to vote this POS out of office ? Vote for Diana Irey in the November election.

ETWolverine rated this answer Excellent or Above Average Answer
excon rated this answer Excellent or Above Average Answer
Itsdb rated this answer Excellent or Above Average Answer
paraclete rated this answer Average Answer

Question/Answer
jackreade asked on 05/18/06 - Faux News Now Racist TV Network

Faux news is now officially a racist network. The radical right is desperate to hold together the coalition of the radical Christians, traditional Republicans, and angry white guys---a coalition which is breaking down rapidly. Dobson of Focus on the Family issued a warning to Bush-none of the religious rights pet issues have been taken care of as promised, so don't expect votes from his group in the fall elections.

Gibson, a talking head on Faux news, gave an op-ed piece begging Americans to reproduce more rapidly because now half the babies born in America are of Hispanic(minority) heritage. (Yeah, white people, get it on!) It won't be very long(a few decades) BEFORE IN AMERICA WHITE PEOPLE WILL BE A MINORITY.

Several other talking heads have echoed Gibson's worries by repeating overtly race baiting comments and fear tactics toward the racist element in the Republican Party.

jack

tomder55 answered on 05/19/06:

Guess I'll just have to switch to Communist News Network (CNN) or MSLSD .

ETWolverine rated this answer Excellent or Above Average Answer
Itsdb rated this answer Excellent or Above Average Answer
jackreade rated this answer Bad/Wrong Answer

Question/Answer
jackreade asked on 05/17/06 - Ward Churchill Controversary Update

This is long, but you don't have to read it all to get the point.

"The University of Colorado's report on the investigation of Ward Churchill's alleged scholarly misconduct has just been released. Churchill was found guilty of deliberate false assertions, misrepresentation of sources, and plagiarism. One member of the committee suggested that Churchill be fired from his tenured position; two more urged a five-year suspension without pay; two urged a two-year suspension without pay.

Here's a sample of one of the more interesting forms of misconduct, related to what some people call "sock-puppetry": Churchill is found guilty of passing off others' work as his own (plagiarism), but also of passing off his own work as others'. The latter is faulted as a general departure from "established standards regarding author names on publications" (p. 89); but it's also more specifically, and more seriously, faulted because Churchill then used the work published under another's name "as apparently independent authority for claims that he makes in his own later scholarship" (p. 89). This "permits the author to create the false appearance that his claims are supported by other scholars when, in fact, he is the only source for such claims" (p. 90). Here's an example, from pp. 23-24 (some paragraph breaks and emphasis added):

Footnotes 63 and 64 of his "Perversions of Justice," in Struggle for the Land (1993 edition), contain basically three sources to support the claims regarding the General Allotment Act of 1887. All appear to the reader to be reputable, independent third-party sources.

First, Professor Churchill cites directly to the originally enacted version of the General Allotment Act of 1887 .... [But n]ot only is his statement unsupported by his source, but also more significantly, he did not follow the referencing convention that a lawyer or historian citing a lengthy statute for a particular detail normally would follow, which is to pinpoint the precise section number of the multi-section statute that supported his claim. As one will see throughout this report, this general reference to an apparent independent source in its entirety constitutes an unconventional referencing style frequently employed by Professor Churchill to create the appearance of independent support for his claims, while simultaneously discouraging or, at least, making far more difficult, any effort by other researchers to check his claims by failing to pinpoint the precise location of his claimed support in an otherwise lengthy work. Standing alone, this referencing failure might constitute some level of sloppiness, but certainly would not constitute research misconduct.

When it is combined with a pattern of other misconduct reflected in this and other allegations, however, the Committee is left with a firm impression, by a preponderance of the evidence, that it constituted part of a deliberate research stratagem to create the appearance of independent verifiable support for claims that could not be supported through existing primary and secondary sources. To put it most simply, it was part of a pattern and consistent research stratagem to cloak extreme, unsupportable, propaganda-like claims of fact that support Professor Churchill's legal and political claims with the aura of authentic scholarly research by referencing apparently (but not actually) supportive independent third-party sources. The next problem discussed with these two footnotes makes this stratagem far clearer.

The other two apparently independent third-party sources cited in footnotes 63 and 64 are essays published in the same volume, The State of Native America, one under the name of a person named Rebecca Robbins and the other under the name of M. Annette Jaimes, the editor of the volume. Since both essays do contain statements of the type that Professor Churchill claims, that might have put an end to the matter of research misconduct regarding this allegation, except for the fact that in response to the separate allegation that he had plagiarized the Robbins essay in another later published piece, Professor Churchill said in Submission E that he had in fact ghostwritten both the Robbins and the Jaimes essays, in full.... [This] constitutes a serious problem of research misconduct. The initial support for the disputed statement involved three independent sources. As already noted, the Act does not expressly provide what Professor Churchill claims and therefore can provide no support for his claims whatsoever. The two other apparently independent third-party sources, the Robbins and Jaimes essays, turn out not to be independent sources at all but, rather, to have been ghostwritten in their entirety by Professor Churchill. This action provided him with apparent independent sources that he could and did in fact cite to support otherwise insupportable claims of legal and historical fact. In short, when one carefully dissects the Churchill claim quoted in the original allegation, the three apparently independent third-party sources dissolve into one source (the Act) that clearly does not expressly support his claim, and two other sources (the Robbins and Jaimes chapters) that he wrote himself.

Although Professor Churchill purported to offer his claims as supported by research, based on independent sources, it turns out that the claims not only cannot be supported but that he has misrepresented the independent nature of his sources employed to buttress the unsupportable details of his conclusions....

Incidentally, while the Churchill report generally seems very thoughtful and scholarly, it does have a small error, included in the discussion of whether the circumstances in which the charges were brought -- public condemnation of Churchill for his description of the victims of the World Trade Center attacks as "little Eichmanns" -- affect the propriety of the investigation. The report states (p. 4):

To use an analogy, a motorist who is stopped and ticketed for speeding because the police officer was offended by the contents of her bumper sticker, and who otherwise would have been sent away with a warning, is still guilty of speeding, even if the officer's motive for punishing the speeder was the offense taken to the speeder's exercise of her right to free speech. No court would consider the improper motive of the police officer to constitute a defense to speeding, however protected by legal free speech guarantees the contents of the bumper sticker might be.

In fact, the First Amendment rule, as set forth in Wayte v. U.S., 470 U.S. 598 (1985), is:

"Selectivity in the enforcement of criminal laws is . . . subject to constitutional constraints." In particular, the decision to prosecute may not be "'deliberately based upon an unjustifiable standard such as race, religion, or other arbitrary classification,'" including the exercise of protected statutory and constitutional rights [such as free speech].

Even prosecution of people who are guilty of a nonspeech crime might thus violate the First Amendment if the government deliberately selected them for prosecution because of their constitutionally protected expression (though I should note that this is a very tough claim to prove).

Nonetheless, whatever may be the rule for criminal prosecutions triggered by the policeman's own hostility to the target's speech, such a rule need not be applied here. This isn't a criminal prosecution, but the university's decision whether to keep someone on its faculty; it need not keep a dishonest scholar on board, even if the complaints about the scholar were motivated partly by the complainers' hostility to the scholar's viewpoints. And as best I can tell, there's little reason to think that the University wouldn't have investigated Churchill had he been accused of the same misconduct but had expressed diferrent views. These are serious charges, and my guess is that most universities would indeed look into alleged multiple falsification of evidence and plagiarism by their faculty members.

There was a connection between Churchill's politics and the investigation, but it seems to me much more attenuated than in the bumper sticker context. Churchill first attracted public notice because of his "little Eichmanns" comment. This led people to scrutinize his work, and past critics of his to repeat their criticisms. This in turn yielded the large body of accusations, large enough that the University had to take notice (in a way that it didn't seem to have done when at least one of the accusations had been separately brought to its notice some years before). So the better analogy is if someone had caused a lot of controversy by his bumper sticker; this caused a lot of people to notice him, and in the process to notice that he was speeding; they in turn complained to the police officer; and the police officer gave him a speeding ticket. There, I think there's no problem under Wayte; the government official (the police officer) wasn't making the enforcement decision based on the bumper sticker, though the people who complained to the officer -- private parties who have no viewpoint-neutrality obligation under the First Amendment -- were motivated by the bumper sticker.

As the report points out, "public figures who choose to speak out on controversial matters of public concern naturally attract more controversy and attention to their background and work than scholars quietly writing about more esoteric matters that are not the subject of political debate" (p. 4) (emphasis added). That seems to me to be exactly what happened here. Unfortunately for Ward Churchill, it turns out that his scholarship couldn't bear the attention that his statements prompted." by Eugene Volokh, blogger

~~~~~~~~~~~~~~~~~~~~~~~~~~~~~~~~~~~~~~~~~~~~~~~~~~~

So, he is a liar and a scoundrel, and that fact is proven. Good News.

tomder55 answered on 05/18/06:

I'll cut and past from Volokh Conspiracy blog because I can't say it any better .

Though only one member of the Churchill investigative committee recommended that Churchill be fired -- two others recommended a five-year unpaid suspension, and two more recommended a two-year unpaid suspension -- it seems to me that this one member was right.

As best I can tell, from what press accounts I've read and from the Report itself, Churchill hasn't shown any contrition. His falsification, fabrication, and plagiarism (in the Committee's words), which the Committee quite plausibly found to be deliberate, are substantial.

And these are falsehoods in his published work, which can readily be checked. How can his future students be confident that things he says in class are accurate? (Yes, we try to instill skepticism in our students, but they still rightly expect that they can count on our factual assertions, rather than double-checking every word.) How can his colleagues, and Colorado taxpayers, be confident that his students are learning things accurately? His work has been cited by over 100 times in law reviews alone, and law isn't even his main field; I assume that quite a few scholars are now wondering whether their reliance on his work led their own work to be in error. How can other scholars, and his other readers, ever rely on anything he says?

It seems to me that keeping him on the faculty would be a substantial disservice to Colorado students, Colorado taxpayers, and the academic fields in which he works. I hope that in its sympathy for a colleague, and its desire to avoid hassle or even litigation, the University doesn't lose sight of that.




jackreade rated this answer Excellent or Above Average Answer

Question/Answer
Itsdb asked on 05/17/06 - Mexico Threatens U.S. With Lawsuits

CIUDAD JUAREZ, Mexico (AP) -- Mexico warned Tuesday it would file lawsuits in U.S. courts if National Guard troops detain migrants on the border and some officials said they fear the crackdown will force illegal crossers into more perilous areas to avoid detection.

President Bush announced Monday that he will send 6,000 National Guard troops to the 2,000-mile (3,200-kilometer) U.S.-Mexico border, but said the troops will provide intelligence and surveillance support to U.S. Border Patrol agents and will not catch and detain illegal immigrants.

"If there is a real wave of rights abuses, if we see the National Guard starting to directly participate in detaining people ... we would immediately start filing lawsuits through our consulates," Foreign Secretary Luis Ernesto Derbez said in an interview with a Mexico City radio station.

Mexican officials worry the crackdown will lead to immigrant deaths. Since the U.S. toughened security at crossing spots in Texas and California in 1994, immigrants have flooded Arizona's hard-to-patrol desert and deaths have increased.

Immigrant support groups estimate 500 people died trying to cross the border in 2005. The Border Patrol reported 473 deaths as of September 30.

In Ciudad Juarez, Mexico, Julieta Nunez Gonzalez, the local representative of Mexico's National Immigration Institute, said Tuesday she will ask the Mexican government to send a protection force, Grupo Beta, to remote sections of the border.

Sending the National Guard "will not stop the flow of migrants. To the contrary, it will probably go up," as people try to get into the U.S. with hopes of applying for a possible amnesty program, Nunez said.

Waiting to cross in Ciudad Juarez was Juan Canche, 36, who traveled 2,000 kilometers (1,243 miles) to the border from the southern Mexican town of Izamal, where he had left his wife, five children and mother.

"Even with a lot of guards and soldiers in place, we have to jump that puddle," said Canche, referring to the drought-stricken Rio Grande, dividing Ciudad Juarez and El Paso, Texas. "My family is hungry and there is no work in my land. I have to risk it."

Mexican newspapers Tuesday characterized the National Guard plan as a hardening of the U.S. position, and some criticized President Vicente Fox for not taking a stronger stand, though Fox called Bush on Sunday to express his concerns.

Fox's spokesman, Ruben Aguilar, said Tuesday that Mexico accepted Bush's statement that the Guard troops didn't imply a militarization of the area, and that Mexico remained "optimistic" that the U.S. Senate would approve an immigration reform "in the interests of both countries."

He noted Bush expressed support for the legalization of some immigrants and the implementation of a guest worker program.

"This is definitely not a militarization," said Aguilar, who also dismissed as "absolutely false" rumors that Mexico would send its own troops to the border in response.

Critics have accused Bush of using the plan to win support for immigration reform from U.S. conservatives, who are more interested in tightening border security.

Bush said it was a stopgap measure while the Border Patrol builds up its resources to more effectively secure the border.

Presidential hopeful Felipe Calderon of Fox's National Action Party issued a statement that the military presence would endanger migrants without stopping them.

"These measures have been proven mistaken. They increase the social and human costs for migrants and only benefit criminal groups that make money on the hopes and suffering of those looking for an opportunity," Calderon said.

Salvadoran President Tony Saca said he was worried that there could be an increase in abuses against immigrants because National Guard troops are trained to handle natural disasters and wars.

Along the border in Nuevo Laredo, across from Laredo, Texas, Honduran Antonio Auriel said he was determined to make it into the United States.

"Soldiers on the border? That won't stop me," he said. "I'll swim the river and jump the wall. I'm going to arrive in the United States."

~~~~~~~~~~~~~~~~~~~~~~~~~~~~~~~~~~~~~~~~~~~~~~~~~~~~~~~

What, they don't like the idea that their military, which aids and abets illegals and smugglers, facing US troops? Any bets on whether or not the ACLU will take up their cause?

tomder55 answered on 05/18/06:

Their military is worse that that they also enable and assist the drug cartels.

Leftist presidential candidate Andres Manuel Lopez Obrador made political hay on the issue the other day and said something that needed to be said .Lopez Obrador said that illegal immigration to the United States was "Mexico's disgrace," caused by the government's failure to create enough jobs. He correctly points out that without the safety valve that our "open border " provides it is likely there will be violent revolts against the gvt.

He is right . Mexico has all the resources needed to be a prosperous nation . Fox And friends have run a business friendly economy but it has not benefited the people . Perhaps with a closed border the Mexicans will finally have to deal with their social problems .

Itsdb rated this answer Excellent or Above Average Answer

Question/Answer
jackreade asked on 05/17/06 - "...hug the tarbaby.."

Yesterday in his first press conference, Tony Snow used the word "tarbaby" **TWICE**...he's "not going to hug the tarbaby". Well, well well, how interesting that he would dig up a pejorative work for dark-skinned people, a word that sent a shiver through me---tarbaby has the same code meaning as the word n*gg*r has overt offensive meaning. It is that offensive. He used it twice.

But, ya know, I(jack) was "born and bred in a briar patch"* and I'll never stop fighting against racists, OF ANY KIND, by gum.

jack




*Credit to Uncle Remus stories, where tarbaby comes from, too.

tomder55 answered on 05/18/06:

yeah popularized by that racist Walt Disney in the racist film 'Song of the South'and Joel Chandler Harris was obvious a racist when he wrote the Uncle Remus children stories ;and Toni Morrison won a Nobel prize for a novel tittled 'Tar Baby'.That was racist also .

If he used it in a racist context someone had better explain how.


Our language makes us all susceptible to being suckered by the votaries of division and derision. The English tongue features more words and nuances than any other. Bigots regularly appropriate everyday words for vile uses. But do their abuses mean that nobody can use such locutions as "chink in the armor," "a nip in the air," "spic 'n' span" or "cheese and crackers?"

who wrote the above ? .....Tony Snow

ETWolverine rated this answer Excellent or Above Average Answer
excon rated this answer Excellent or Above Average Answer
jackreade rated this answer Bad/Wrong Answer

Question/Answer
jackreade asked on 05/16/06 - Guard Too Expensive To Use for Boarder Work-Chertoff

"In December of 2005, Fox News talking head Bill OReilly floated an unlikely even brash idea to the Homeland Security secretary to seal off the porous southwest border.

Why dont you put the National Guard on the border to back up the border patrol and stop the bleeding, and then start to increase the Border Patrol, the high-tech and all of that? OReilly asked.

Michael Chertoff, in those relatively calmer days before mass pro-immigration rallies, heated immigration reform politics in the Senate and cellar-dwelling opinion polls for President Bush, dismissed the idea out of hand.

Well, the National Guard is really, first of all, [ONE]*not trained* for that mission, Chertoff told OReilly. I mean, the fact of the matter is the border is a special place. There are special challenges that are faced there.

Chertoff added that that it would take a [TWO]*huge amount of National Guard troops*, that they would need new training. But couldnt the National Guard pull it off, OReilly asked?

I think it would be a [THREE]*horribly over-expensive* and very difficult way to manage this problem, Chertoff said. Unless you would be prepared to leave those people in the National Guard day and night for month after month after month, you would eventually have to come to grips with the challenge in a more comprehensive way.

~~~~~~~~~~~~~~~~~~~~~~~~~~~~~~~~~~~~~~~~~~~~~~~~~~

tomder55 answered on 05/17/06:

Had Bush increased the size of the border patrol like Congress authorized him to do then the guard would not be necessary.Look at it this way ... the National Guard going to the border is needed because they are going to do the job the Mexicans won't do .And if they get some target practice in by picking off a few coyotes ? tanto mejor

jackreade rated this answer Excellent or Above Average Answer

Question/Answer
Erewhon asked on 05/16/06 - What Conservatives are really up to!


Professor Galbraith said,

"The modern conservative is engaged
in one of man's oldest exercises
in moral philosophy; that is,
the search for a superior
moral justification for selfishness."


How is this demonstrated by the Bush administration?




tomder55 answered on 05/17/06:

You are correct in that capitalism enshrines self-interest. No;it is not evil when one act in one's own benefit.It is a good thing to be self reliant, to be proud of your achievements.

Even the act of giving to charity is done in selfish motive...feel good about yourself ..pride ..promise of eternal reward . Sometimes being selfish is the most selfless thing you can do.Altruism is acting for the benefit of other people. Sacrificing a portion of ones income by donating to charity is defined as an altruistic act. Religion teaches that is a good thing ;it helps you obtain salvation .But to be motivated into aiding others due to a promise of the greatest compensation possible for doing so, is actually a self-serving act.

Erewhon rated this answer Excellent or Above Average Answer
Itsdb rated this answer Excellent or Above Average Answer

Question/Answer
powderpuff asked on 05/16/06 - Immigration to the USA

Trying to figure out why we have such a problem with illegal immigration I did a google search and found this:

http://www.usimmigrationsupport.org/greencardlottery.html

Does this mean that the US lets 50,000 people legally immigrate here yearly? Only 50,000?

"The United States Government issues 50,000 permanent resident cards (Green Cards) every year through the Diversity Immigrant Visa Program, commonly known as the Green Card Lottery. Applicants are selected randomly by a computer-generated drawing." "The process of completing and submitting a DV Lottery application can be confusing." "It is a misconception to believe that winning the Green Card Lottery automatically grants the selected applicant U.S. residence, a Green Card or United States Citizenship. There are several additional forms and documents that must be submitted to the U.S. government before the applicant receives permanent residence in the United States."

"Every year 100,000 winning notification letters (Notice of Approval) are mailed to the applicants selected through the electronic drawing. The "Notice of Approval" does not guarantee that the applicant and his/her family will receive a Green Card. The Notice of Approval means that the applicant has been selected to continue the application process."

If the US only allows 50,000 to immigrate yearly, I think this could be part of our problem. I'm fairly sure there are many more than 50,000 people who want to immigrate here every year.

Why only 50,000? Is 50,000 a good cut off point? I see lots of space left here so I think we have the room.

tomder55 answered on 05/17/06:

The captain got the facts right ,and in truth it is clear that we could still absorb more .

There is a dirth of H-1B visas issued ;the high skilled market .It's kinda sick actually . Many of the graduates of our university system are not permitted to work here afterwords. Many of the worlds best scientists and engineers live in the United States, but many others dont and the number of engineering degrees awarded in our country is down 20 percent since 1985. If current trends persist, more than 90 percent of the worlds engineers will live in Asia as early as 2010.More than half of the masters and doctoral degrees at U.S. universities in these skills are awarded to foreign nationals. It makes no sense to train foreign engineers and scientists here, and then send them home to compete against U.S. companies. The H-1B visa quota for 2006 was reached two months before the year even began.

These high skilled /high waged immigrants would help pay for the social system strain caused by the increase in low waged immigrants .They in turn would be the future middle class of this country ;as all previous immigrant waves have created .

No one here has argued against immigration . All that has been asked for is to end the tide of illegal immigration .

powderpuff rated this answer Excellent or Above Average Answer
Itsdb rated this answer Excellent or Above Average Answer
sissypants rated this answer Excellent or Above Average Answer

Question/Answer
Erewhon asked on 05/16/06 - Bush, Cheney Drop Huge Cake On Iraq, Crush Power Plant

In celebration of the third anniversary of the invasion of Iraq, President Bush and Vice President Cheney dropped an enormous three-tiered anniversary cake on central Iraq, accidentally crushing the only working power plant in the area.

"Everything's just great in Iraq," said Cheney, who was so thrilled with the progress of the country that he was "planning to winter there someday."

"Things are so good now," said President Bush, "just imagine how fantastic it will be when they have a McDonald's on every corner."

Mohammed dar al Salim, a former baker whose shop had been destroyed by looters a year ago, agreed. "The future is certainly bright," he told reporters. "It's the present that worries me."

Fifty people who were killed yesterday as a result of the civil war could not be reached for comment.

===

Comments?

tomder55 answered on 05/16/06:

Tom Burka's pretty good .

Erewhon rated this answer Excellent or Above Average Answer

Question/Answer
Erewhon asked on 05/16/06 - Another White House shake-up on the way!

In Major White House Shakeup, Bush To Replace Rove and Cheney With Rove And Cheney

Rumsfeld to Stay On

President Bush today admitted that he would be making major changes to White House staff in an attempt to address flagging poll numbers and "fatigue." Bush focused on Karl Rove and Dick Cheney, who Americans have indicated are at the heart of some of the deepest and worst missteps the administration has taken, among them the faltering response to Hurricane Katrina and the shooting of people in the face.

Rove and Cheney are to step down by week's end, after which they will be reappointed to their current positions. "I am not afraid to make tough calls, admit mistakes, and correct them," said Bush. "This is a White House which is committed to growth and change."

Bush pooh-poohed suggestions that the shakeup was purely cosmetic, and more of a public relations gambit than a substantive makeover. "This Dick Cheney is not the same Dick Cheney who took office with me in 2001. For one thing, I understand he's had some implants."

He also suggested that Karl Rove had been chastened and reborn in the wake of criticism and by the stigma of having to give up his office, if only for two days. Rove himself said, "I'm an idiot one day and a genius the next." Bush said that Rove had been "half-right," but would not elaborate other than to say that "this time out," the White House would employ him every other day.

Bush refused to replace Secretary of Defense Donald Rumsfeld, either with Rumsfeld or another person, despite calls for his resignation in the face of his poor handling of the Iraq war and the Abu Ghraib scandal. "I don't think anyone could have foreseen that the Iraq war would have cost this much money or caused any loss of life."

==

Comments?

tomder55 answered on 05/16/06:

better yet ;Cheney steps down.... replace him with Condi (setting up a Condi 2008 run ) Shift Rummy to State to institute aggressive reforms in the State Dept ,and replace Rummy at Dept Defense with Cheney (already a former Def. Sec.) .

Erewhon rated this answer Excellent or Above Average Answer

Question/Answer
jackreade asked on 05/15/06 - National Guard to shoot Mexicans??

Are the National Guard guys deployed to the US Mexico boarder going to shoot the illegals?

Or, is this just to mollify the radical right who have threatened to stay home on election day because Bush hasn't implemented amy of their favorite causes....gay marriage amendment, repeal abortion amendment, immigration, etc....

Hey, why don't Cubans have to go back. oh yeah, Bush buying Cuban-Hispanic votes in Florida. Yeah, yeah, that's right.

tomder55 answered on 05/16/06:

lets fill the Rio Grande with piranas and sharks and if the illegals cross it then we will consider asylum . The immigration policy for Cubans has been in place for years .In 1985 a quota was established for the first time.Cuban undocumented entrants have always had special status: While entrants from other countries have been required to demonstrate that they are fleeing political persecution to be granted refugee status, it was officially assumed that anyone arriving in the United States from Cuba was a bona fide refugee and treated as such . Evidently both parties cater to that "Cuban-Hispanic votes in Florida " .

The answer is no ;the guard will not be 'shooting 'at illegals anymore than the border patrol already has the authority to do so.

ETWolverine rated this answer Excellent or Above Average Answer
Itsdb rated this answer Excellent or Above Average Answer
jackreade rated this answer Excellent or Above Average Answer

Question/Answer
arcura asked on 05/15/06 - Do you argree with this by David Brownlow?????????

THE U.S. DEPARTMENT OF OFFENSE - PROPPING UP THE EMPIRE
By David Brownlow
May 14, 2006
NewsWithViews.com
Many are unaware that at $500 billion a year, we Americans squander as much on our military as the rest of the world does on theirs - combined! This includes some of the (other) really cutthroat regimes, like China, Russia, and Iran. There is no one else even close.
Could it be possible that American defense needs are so unique that it takes $500 billion just to protect our country? No, that is not possible. Defending a country is relatively cheap. Building an empire is not. Somewhere along the way, we seem to have confused our military with a Department of Offense.
Without a doubt, we have created the most fearsome and ferocious military machine the world has ever known. Unfortunately, instead of being used as a defensive force, our military has become the global enforcement arm for the madmen who have seized control of our country - who wield this power like spoiled kids as they strut around the world, barking out intimidations and threats to any who refuse to bow down before them.
It is time to take away their toys.
Propping up the Empire requires a massive amount of firepower - which has left us with weapons systems that defy all logic. The new F-22 radar-evading fighter, at $280 million a copy, is a perfect example of how crazy things have gotten. Two hundred and eighty million bucks for an airplane, when a top of the line F-16 sells for about $20 million, is completely nuts. There will be no need for radar-evading fighters as soon as we start minding our own business - and quit trying to sneak around in places we do not belong.
The F-22 is just one example of the insanity. There are dozens of others. We can be confident of this - if we keep building the weapons of war, our leaders will keep finding ways to use them.
Even our Navy, which we have every right to maintain, has become an empire building force. Does it really take fourteen carrier battle groups just to protect our coastline? No, of course not. The same goes for most of the other 450 war ships in our fleet. Unless there is a foreign invading force lurking out there that we are not aware of, we could get by with a lot fewer ships. We just need to keep our sailors at home patrolling our own shores, instead of wandering around the world patrolling everybody elses.
Our active troop strength is also completely out of whack with our defense needs. Why do we need 1.4 million soldiers in uniform? They are certainly not being used to protect our borders. Maybe that is because nearly 400,000 of them being used to protect somebody elses borders!
On top of the 140,000 we have getting shot up in Iraq, there are over 200,000 American soldiers stationed in 140 of the worlds 192 countries. This includes 66,000 in Germany, 35,000 in Japan, and 30,000 in Korea. We beat those guys over 50 years ago, so what are we doing still hanging around in their countries? We even have soldiers stranded in some of the worlds most obscure places 1,300 in Djibouti, 400 in Quatar, and 2,000 in Serbia, to name a few. It is unbelievable.
It is time to bring every one of them home!
Here is an interesting fact; if we add up the annual military spending for the entire western hemisphere, minus the U.S., it comes to less than $25 billion a year. How could it possibly take America twenty times that amount to defend ourselves? Well for starters, our neighbors tend to mind their own business and keep their armies out of other peoples countries. Canada, with a larger landmass to protect than we do, finds a way to defend their country for about $8 billion a year. Brazil, with a population of 186 million and 5,000 miles of coastline to protect, manages to defend themselves for only $15 billion a year. Mexico does it for only $4 billion.
Our military spending is spiraling out of control not because our defensive needs are so much greater than anyone elses it is because our offensive needs are so much greater.
What is the alternative to this huge military machine we have unleashed on the world? We could start minding our own business, and quit causing trouble in every corner of the planet. If we did that, a motivated, highly trained and well-armed (well regulated) militia could replace almost every one of our full time soldiers in the Army and Marine Corps. We just need to reset our priorities - and commit to keeping our men out of foreign entanglements.
By simply obeying the law, and getting out of the empire building business, we could dramatically trim back the size and scope of our Department of Offense while making the "homeland," and the world, a much safer place.

tomder55 answered on 05/15/06:

Since the 1950s the spending on the military relative to the GDP has steadily decreased . In 2003 it was 3.7 % of GDP . During the final years of the Cold War it was over 6%. During the height of the Cold War it was around 10% . I would say that our war against jihadistan (which is in no way an empire building exercise ) is at least as critical to our countries future as the COld War was .

In 1960 ;the military budget consumed slightly over 70% of the discretionary spending .Today it is below 50%.


Do I agree with him . Yes when the issue is abortion . No when he is talking about the role of the military and our foreign policy .No ;he does not understand the war against jihadistan and what is at stake. He is one of those who think the US can retreat from the world and that would solve all our problems . No when he spouts crazy Michael Moore tin-foil hat theories about someone setting off detonations inside the WTC on 9-11 .No when he talks conspiracy non-sense about the Federal Reserve.No when he talks about us butting out of the Israel -Palestinian issue .No on the war on drugs .


Yes when he talks about the excesses of the judiciary .Yes when he talks about the 2nd Amendment. Yes when he talks about Social Security .Yes when he says to either reform the UN or get out .Yes when he talks about streamlining or eliminating many Fed Agencies .Yes on repealing certain regulations that impede business job creation .?Yes on allowing capitalism to work. Yes on non-military spending .Yes on the environment .Yes on hate crimes.

arcura rated this answer Excellent or Above Average Answer

Question/Answer
excon asked on 05/14/06 - The Future


Hello again donthaveanythingtohiders:

If I trusted the government to use the data they are collecting as they say they are, then I would have NO problem with the collection. However, I dont trust the government. Plus, I dont trust anyone who does.

Come take a journey with me into the future. Thats something politicians cant/dont do, but you can.

Lets say youre a member of PETA. Im not exactly sure what PETA is (and I dont want to know), thats why Im using it as an example. Right now, membership in PETA is perfectly legal. Lets say that, in the future, membership in PETA becomes highly unpopular. Or lets say that some leaders of PETA commit a violent act in the name of PETA. As a response, the government would like to talk to, and maybe even charge, all the members of PETA for being terrorists. But they dont know how they could possibly find them.

Then somebody says, well, we HAVE this database..............

Lets say, they come out with a super laxative, and some up tight politician ('pun intended', who has an old laxative factory in his district), decides that crapping on demand is un-American. He manages to pass a law making possession of the superlaxative illegal, and wants to round up illicit eliminators. YOU happen to have tried it, ONCE, and you are IN a database.

Then somebody from SEA (you figure out what that means), says well, we HAVE this database......

excon

tomder55 answered on 05/15/06:

As I have said many times ,the final arbiter of what is constitutional is the people . The laws the executive used in this case have been passed by Congress ;signed by the executive ;and reviewed by the judiciary .

Is membership in PETA some quarded secret ? What if PETA did become terrorists ? They are close to it now.

I don't understand this flap . The people who objected to the NSA terrorist monitoring program at least had the debatable argument that it required FISA approval .There is no stipulation for that in this data-mining program at all.

If you look far enough into the USA Today article you will find this :

Paul Butler, a former U.S. prosecutor who specialized in terrorism crimes, said FISA approval generally isnt necessary for government data-mining operations. FISA does not prohibit the government from doing data mining, said Butler, now a partner with the law firm Akin Gump Strauss Hauer & Feld in Washington, D.C.

The caveat, he said, is that personal identifiers such as names, Social Security numbers and street addresses cant be included as part of the search. That requires an additional level of probable cause, he said.


Data -mining is a valuable tool that does not violate any privacy you think is guaranteed in the bill of rights. Don't believe me ? Ask the ACLU

excon rated this answer Excellent or Above Average Answer
Itsdb rated this answer Excellent or Above Average Answer

Question/Answer
Erewhon asked on 05/13/06 - Bush's erratic behaviour explained? GOP said to be "very worried!"



All the presidents' minds
By ALEX MASSIE IN WASHINGTON

IT IS as demanding and stressful a job as any, making extraordinary demands upon the 42 men who have held the post.

Lyndon Johnson complained that being president of the United States was an "unrelenting business" in which work was "always there to be done", to the point where "it became a question of how much the physical constitution could take".

Now, researchers writing in the Journal of Nervous and Mental Disease have concluded that half the presidents in US history suffered from mental illness.

Three professors of psychiatry at the prestigious Duke University in North Carolina analysed presidential biographies and other historical records before concluding that

18 of the 37 presidents who served between 1776 and 1974 suffered from psychiatric disorders.

Of those 18, at least ten suffered psychiatric problems while in office which, more often than not, may have affected their performance in the job. The sole exception cited by the Duke psychiatrists was Teddy Roosevelt, whose bipolar disorder seems not to have hampered his muscular approach to the presidency.

The pattern of poor mental health runs from the founding fathers to Richard Nixon. John Adams, the second president, suffered from depression while his successor, Thomas Jefferson, was hampered by social phobia.

Depression was the most common diagnosis, occurring in 24 per cent of cases, followed by anxiety (8 per cent), bipolar disorder (8 per cent) and alcoholism (8 per cent). Other depressives included Dwight Eisenhower, Rutherford Hayes, James Madison, James Garfield and John Quincy Adams.

Of Woodrow Wilson, the researchers noted: "The development of paranoia and other mental changes, which could have amplified his rigidity of character, perhaps prevented him from taking advantage of his opportunities as president of the world's most powerful country after the First World War."

Wilson's predecessor, William Taft, was accused by one biographer of losing interest in the presidency. His administration was characterised as one of "drift, drift, drift - little attempted, nothing done". The professors say Taft "coped with the stress of the presidency by overeating to the point of massive obesity, and obstructive sleep apnoea meant that he probably could not give full attention to the job".

However they note that "no national calamities appear to have occurred due to presidential mental illness".

Although the research further dents the already tarnished notion of presidential omnipotence, the researchers argue that publicising the psychiatric problems suffered by presidents is beneficial. "Presidents are seen to be human, and if so many of them have a major psychiatric disorder, it could at least lessen the long-standing stigma toward mental illness," they argue.

Michael Kazin, professor of history at Georgetown University in Washington, said: "There are certain aspects of any powerful politician's career which make it quite rational for a president to be mentally troubled."

===

Should sitting presidents be subject to monthly psychiatric evaluation to ensure that they are sane and not behaving strangely, as the present incumbent is doing?



tomder55 answered on 05/14/06:

they join the ranks with the loons of the royal families in Brit history .

Erewhon rated this answer Excellent or Above Average Answer

Question/Answer
jackreade asked on 05/13/06 - Spying Goes Farther than Terrorist Surveillance Program

"CongressDaily reports that former NSA staffer Russell Tice will testify to the Senate Armed Services Committee next week that not only do employees at the agency believe the activities they are being asked to perform are unlawful, but that what has been disclosed so far is only the tip of the iceberg. Tice will tell Congress that former NSA head Gen. Michael Hayden, Bushs nominee to be the next CIA director, oversaw more illegal activity that has yet to be disclosed:

A former intelligence officer for the National Security Agency said Thursday he plans to tell Senate staffers next week that unlawful activity occurred at the agency under the supervision of Gen. Michael Hayden beyond what has been publicly reported, while hinting that it might have involved the illegal use of space-based satellites and systems to spy on U.S. citizens.

[Tice] said he plans to tell the committee staffers the NSA conducted illegal and unconstitutional surveillance of U.S. citizens while he was there with the knowledge of Hayden. I think the people I talk to next week are going to be shocked when I tell them what I have to tell them. Its pretty hard to believe, Tice said. I hope that theyll clean up the abuses and have some oversight into these programs, which doesnt exist right now.

Tice said his information is **different from the Terrorist Surveillance Program** that Bush acknowledged in December and from news accounts this week that the NSA has been secretly collecting phone call records of millions of Americans. Its an angle that you havent heard about yet, he said. He would not discuss with a reporter the details of his allegations, saying doing so would compromise classified information and put him at risk of going to jail. He said he will not confirm or deny if his allegations involve the illegal use of space systems and satellites".

~~~~~~~~~~~~~~~~~~~~~~~~~~~~~~~~~~~~~~~~~~~~~~~

tomder55 answered on 05/13/06:

Would this be the same Tice who was stripped of his security clearance after he reported his suspicions that a former colleague at the Defense Intelligence Agency was a spy ? The matter was dismissed by the DIA, but Tice pressed it .He was determined to be unsuitable to perform his duties and was put on administrative leave. He was later assigned to unload furniture from trucks at an NSA warehouse and also served an eight-month tour of duty in the NSA motor pool during an evaluation period. He was finally fired after his doctor's diagnosis him of psychotic paranoia..He was fired for apparently violating his security clearance by taking unauthorized peeks into the background of the female Asian employee he thought was a Chinese spy. In fact, Tice was involved in high-tech stalking of the woman.

He became this patriotic whistle blower after he was fired .He spilled out his guts to James Risen of the NY Slimes and the story of the NSA foreign intercepts program was disclosed at a later date to coincide with the release of Risen's book "State of War" and the congressional vote on the extension of the Patriot Act. Two days after the book was released Tice decided to apply for federal whistleblower protection and has since joined the National Security Whistleblowers Coalition.



What this article you quote fails to mention is that the new information Tice is refering to is the LEGAL Clinton approved 'Echelon 'Program that was uncovered in 2000.

jackreade rated this answer Poor or Incomplete Answer

Question/Answer
jackreade asked on 05/13/06 - Political Book Review

"In The Greatest Story Ever Sold: The Decline and Fall of Truth from 9/11 to Katrina (September, Penguin Press), a scathing rebuke of the current administrations definition of truth, New York Times columnist Frank Rich examines the propaganda misinformation of the Bush era.

Though it was a sequence of events that led to Richs frustration, he notes the war in Iraq as the epitome of all of the administrations shortcomings: Placing a higher priority on partisan politics than the nations welfare, lazy and poor planning, public relations as a substitute for policy, arrogance, unilateralism, an inability to admit or correct mistakes: the same themes recur again and again, he says.


Rich does, however, reserve a measure of respect for the method behind the madness, while shunning the insanity itself. Beyond the White Houses policy, he says, is also a fascinating narrative: the fictional story they rolled out, quite brilliantly at times, that sold the nation on a war against an enemy that did not attack us on 9/11. The Democrats tardy, timid and laughably inarticulate response has perturbed him, but not nearly as much as the actions of the Republican administration: The White House . . . sold a war of choice to the American people on fictitious grounds and with disastrous results that will continue to play out on many fronts for years to come, he says."

~~~~~~~~~~~~~~~~~~~~~~~~~~~~~~~~~~~~~~~~~~~~~~~

He pretty much nails it. I look forward to reading it.

tomder55 answered on 05/13/06:

I see they timed the release perfectly to have maximum impact during the election cycle.

This is the same a**hole who slammed the film 'Flight 93'the film that honors the heroic passangers of the flight ;who's actions prevented the plane from striking another target , as being 'exploitive' .

Rich, who was once the most feared theatre critic in America when he was initially on the Slimes ,was able to close a Broadway show with one of his reviews .The powers that be on Broadway demanded Rich be removed from his critic position at the Slimes.They hated Rich. He was destroying the NY theater community and its revenues.

He has now taken his panning style into political commentary . I admit that he is a terrific writer and does his best when his work is dripping in sarcasm . However , he has become a mouth piece for the aging boomers who see America through the prism of the Vietnam era.... Iraq and the Bush Adm. last hurrah to take those fading tie-dyed tee shirts and faded jeans out of their closets ,and chant anti-war slogans ,before they go to that retirement community in Fla. where they can play a round of golf in between changes of Depends.

I'm sure the book will sell fairly well .He is always best when he is preaching to the choir. When the book is released I'm sure that I will have alot to say about his version of truth. Rich, Paul Krugman and Maureen Dowd (I can't get a date therefore men hate intelligent women)are the three stooges of the NY Slimes.But although all three are very talented writers ,the bottom line is all they do is translate the DNC talking point faxes into pseudo-intellectual prose for the limosine liberals to digest and regurgitate .

excon rated this answer Excellent or Above Average Answer
jackreade rated this answer Poor or Incomplete Answer

Question/Answer
paraclete asked on 05/13/06 - Warning, American in imminent danger

America is in danger of another attack by terrorists.

John Howard was in Washington on 9/11, he was in Washington before the attack on Iraq and look, he's doing it again.

Howard goes to Washington


May 13, 2006 - 10:20AM

Prime Minister John Howard has played down opinion polls showing flagging support for US President George W Bush as he arrived in the United States for a top level official visit.

Mr Howard and his wife Janette will spend five days at Blair House, the official residence near the White House in Washington, as guests of Mr Bush.

The prime minister was greeted by dignitaries including Blair House manager Randy Bumgardner and spoke briefly to waiting journalists before moving inside the historic building.

He declined to say if he would be giving the president any advice on how to battle through his poll problems.

"I'm a guest of his and let's not get into that," Mr Howard told reporters.

"Anybody who's been in politics a long time goes through periods of challenge and also great success.

"He's been a very successful person. To be elected twice as president of the United States is an astonishing achievement."

Mr Bush's personal popularity rating has dropped to about 30 per cent as public opinion turns against the war in Iraq.

Mr Howard and Mr Bush will discuss the future of troop deployments in Iraq and Afghanistan, Iran's nuclear program, the nuclear Non-Proliferation Treaty and the growing importance of China and India in formal talks on Tuesday.

The two leaders will meet informally on Sunday afternoon when Mr Howard will plant two saplings from the White House garden in the grounds of the Australian embassy.

The trees are a gift from the president to mark the relationship between the two countries.

President Bush will also host a black-tie dinner in the White House on Tuesday night - only the seventh such honour he has accorded to a visiting leader during his two terms of office.

Mr Howard said the Australia-US friendship was strong.

"I do have a very close relationship with President Bush," he said.

"I like him and we think along similar lines on a lot of issues ... but ... the relationship goes beyond the personal relationship between two heads of government."

The alliance would become more important in coming years, Mr Howard said.

"Our economies could get closer together and our world view, although it will vary on some occasions, on some issues, will still be very similar with the rise of China and India in the Asia-Pacific region, clearly the challenge of handling Iran in an intelligent sensible way, the growing importance of the potential of nuclear energy," he said.

"All of these are the sorts of things that we'll be talking about and they're the sorts of things that an American president and an Australian prime minister should talk about on a regular basis."

It is Mr Howard's second visit to the United States within a year, with last July's trip to Washington, London and Baghdad costing the taxpayer $616,000.

The prime minister and his wife will also visit Chicago, Ottawa and Dublin in this two-week, three-country tour.

AAP

tomder55 answered on 05/13/06:

John Howard is a world treasure. It is proper that we honor him .Too bad he will not be addressing a joint session of Congress ;knock some sense into them . He has been a staunch ally .But if he is here for 5 days he should go someplace else than the capital city . I now formally invite him to spend a couple of days in my small town ;a reality check from the bizarre world of the beltway.

paraclete rated this answer Excellent or Above Average Answer

Question/Answer
ETWolverine asked on 05/12/06 - A proposed response to Ahmadinejad's letter.

The following is my proposal of how Bush should respond to the rediculous letter sent by Mahmoud Ahmadinejad.

---------------------

The White House
Oval Office
From The Desk of George W. Bush

Dear President Ahmadinejad:

I was delighted to read your recent letter. In fact, my whole Administration was smiling for days afterwards mostly because they couldnt stop laughing. But I felt that, given your attempt at diplomatic outreach, your letter deserves an appropriate response.

First of all, I find it interesting that you quote the teachings of Jesus Christ, considering that you have publicly stated on any number of occasions that you would like nothing better than to destroy anyone who doesnt follow the Islamic religion, and specifically your particular branch of Shia. This, of course, includes the USA, Israel, and most of Europe.

You question my putting troops in harms way in Iraq, but ignore the fact that your government has been putting its troops in harms way against the Iraqi military for decades. Your government saw Saddam Husseins Iraq as a threat to your sovereignty, reacted to that perceived threat with military action, and sent troops to fight against the Iraqi military. The only difference between your countries actions and mine is that we beat the Iraqi military in a matter of weeks, whereas your soldiers simply died in combat, with no real resolution to the conflict.

You question why I would send troops against Saddam Husseins regime based on WMDs that you claim did not exist. But didnt Saddam Hussein gas your troops with chemical weapons during the 1980s? Dont those constitute weapons of mass destruction? How can you, as the leader of the Iranian people who suffered most of all from Saddam Husseins illegal weapons programs, ask why I would send troops to stop those illegal weapons from being used or given to others who would use them against civilians.

Next, you ask about the existence of Israel and question the events of the Holocaust. I wonder how anyone who claims to be a teacher could possibly wonder whether the Holocaust took place or not. Most people claiming to be teachers know how to read, and the amount of evidence available to the public to read is huge. The Holocaust is by far the most documented crime in history, with witnesses from both the victims and the perpetrators having given testimony on the subject. But, of course, actually studying the subject of the holocaust would take an ability to read, and given the lack of literacy I found in your letter, I question your ability to read at all, much less understand the intricacies of researching historical fact.

Furthermore, the Jews did not steal the land of Israel from the Palestinian people. They WERE the Palestinian people. They were the ones called Palestinians prior to 1948. The ones who you now refer to as Palestinians were actually Jordanians. It wasnt until the 1950s that the term Palestinian was coined to refer to the indigenous Jordanians living there. Furthermore, the land had belonged to the Jewish people for approximately 3,200 years before it was claimed by the Jordanians. They had lived there all that time in an unbroken string, with different governments, regimes and rulers over them. They didnt steal anything from the so-called Palestinian people.

Finally, it should be noted that Israel did not simply come into existence. The establishment of the Jewish State of Israel was voted on in the United Nations by representatives of all its member nations. The Muslim community decided that they did not like this turn of events and promptly attacked Israel trying to steal the land that rightfully belonged to them. That attack failed, as did all subsequent attacks. Those subsequent attacks by the Muslim community directly resulted in the increase in Israels size to its post 1967 borders. And if Israel had actually annexed the West Bank and Gaza Strip into the sovereign borders of Israel and expel the Muslims from the area, none of the violence that has followed since would have occurred. But Moshe Dayan was overly sensitive in his dealing with the Muslim world, and chose to instead to invite them back to live in peace and harmony alongside the Jews. The West Bank and Gaza, instead of becoming part of Israel, were labeled occupied territories, and eventually illegally occupied territories, though there was nothing illegal about it.

I find it interesting that you quote Jesus Christ with such facility, but ignore the fact that Jesus was a Jew who lived and died in the Land of Israel, further proving that the land belonged to the Jews.

You also questioned the treatment of POWs being held in Guantanimo Bay and Abu Ghraib in your letter. However, I must point out that Iran is not exactly known for the soft-handedness of the treatment of its prisoners. For that matter, your countrys entire human rights record is a giant mess. According to the U.S. Department of State Country Reports on Human Rights:

Prison conditions in the country [Iran] were poor. Many prisoners were held in solitary confinement or denied adequate food or medical care to force confessions. After its 2003 visit, the UN Working Group on Arbitrary Detentions reported that "for the first time since its establishment, [the working group] has been confronted with a strategy of widespread use of solitary confinement for its own sake and not for traditional disciplinary purposes." The working group described Sector 209 of Evin prison as a "prison within a prison," designed for the "systematic, large-scale use of absolute solitary confinement, frequently for long periods."

By contrast, CBS News Rosa Hwang did a story about Camp Delta in Guantanimo Bay in which she said the following:

What we saw at Camp Delta seemed hardly a gulag. The prisoners appeared to be well fed and kept in quarters typical of any medium- to maximum-security U.S. prison. The cells are sparse, yet neat. The guard forces are serious, yet professional.

The detainees seemed to spend most of their time battling the oppressive heat, dust and bugs, as opposed to battling allegedly abusive guards For the most part, they regarded us with mild curiosity.


On the topic of supposed secret black prisons run by the USA, the State Departments report says this about Iran:

The UNSR reported that much of the prisoner abuse occurred in unofficial detention centers run by unofficial intelligence services and the military. The UN Working Group on Arbitrary Detention raised this issue with the country's Article 90 parliamentary commission during its 2003 visit, generating a commission inquiry that reportedly confirmed the existence of numerous unofficial prisons.

Furthermore, I would point out to you that the actual existence of any such black prisons has been questioned recently in the media, as opposed to your secret prisons which have been confirmed to exist.

In your letter, you asked why the international community is so afraid of Iran developing nuclear technology. The answer to that, Mr. President, can be found in your own rhetoric. No other nation that capable of developing nuclear technology has threatened to destroy a sovereign nation that its President happens to disagree with. No nation besides Iran has offered to sell its nuclear secrets to the Sudanese government, one of the most corrupt and brutal governments in the entire world. No other nation with developing nuclear ambitions has declared a holy war against the rest of the worlds religions. We are worried about you, Mr. President, and the rhetoric of hate that you continue to spout. When people who hate as much as you do get their hands on big bombs, they tend to go off, and we would rather not have any nuclear weapons go off any time soon.

And finally, you argued that Democracy and capitalism are dead, but that the law of G-d continues to survive and thrive. I happen to agree with you that the Law of G-d is still alive and well. That Law is a Law of love, charity, responsibility and human decency. Unfortunately, that is not the Law that you practice. The Law that you practice is one of hatred, bigotry, oppression, torture, and the suppression of the human spirit of growth. Those are very different values from the ones that I learned in Sunday School. Furthermore, if Democracy is dead, why is it that so many people are trying to get into the United States, both legally and illegally. I may differ with some lawmakers on how to handle the matter of illegal immigration, but the fact is that people are coming to the United States in droves to experience our way of life. Irans immigration rate is -0.48%. Thats negative point five percent. People are trying to get OUT of Iran. Iran has 40% of its population living below the poverty line, 11% unemployment and 16% inflation. I think that a comparison of the USAs economic strength and Democratic way of life verses the Iranian theocratic system speaks for itself.

Despite our disagreements, I am happy that you decided to open communications between our two countries. I hope that this will be only the first step in an open dialogue and that we can move forward together in the spirit of friendship and international cooperation. In fact, in the spirit of friendship, Vice President Cheney has asked me to extend a personal and open invitation to you to join him quail hunting some afternoon.

With deepest respect,



George W. Bush
President of the Unites States of America
---------------------

What do you think?

Elliot

tomder55 answered on 05/12/06:

to me the most interesting thing about his letter is that Ahmadinejad must have one helluva tin foil hat . When discussing 9-11 he stole a page from the moonbat Michael Moore crowd.

September eleven was not a simple operation. Could it be planned and executed without coordination with intelligence and security services or their extensive infiltration? Of course this is just an educated guess. Why have the various aspects of the attacks been kept secret? Why are we not told who botched their responsibilities? And, why arent those responsible and the guilty parties identified and put on trial?

He is correct of course that 9-11 was also carried out by an experienced intelligence service ;just not the one he is implying .Laurie Mylroie has shown how the first World Trade Center attacks were linked to Saddams intelligence service, and how the 9-11 attacks required the same coordination and operational skill that only the intelligence agencies of established states can deliver . OBL was involved of course ,but he did not act alone and no President Ahmadinejad ;it was not the Mossad or the CIA .

Here is a translation of Mahmoud Ahmadinejad's letter slightly unfiltered .

Dear Infidel Crusader Zionist sock-puppet Saudi-lackey depoiler of Mesopotamia woman-touching pigdog fiendish (293 words excised) Shah-licking son of a toads offal: I trust this finds you well. I have much on my mind, and have taken the pen to unburden my breast. I have enclosed a self-addressed stamped envelope should you wish to reply.

(429 words concerning Jewish penetration of the Postal System excised)

. . . Do you not realize you are beaten, as a donkey is beaten, but knoweth not his donkeyhood is cursed? Your comics have turned against you in your own lair, and mock you without mercy. We have seen the videos of the Meal of the Correspondents, and we know how your left regards the men of the laugh as prophets and seers. It is only a matter of time before Johnny Carson (applause be upon him) returns from occlusion to request that you, Mr. President, take the Slauson cutoff, get out of your car, and cut off your Slauson, Hi-yo, salaam. And a third part of the Slauson shall be stained with the tears of the womenfolk, and (9323 words excised)

. . . Our people glow with pride over our nuclear efforts, sometimes literally. I repeat that the enrichment is for peaceful purposes only, and we seek only peace, and peace is our goal, and there is nothing more we love than peace. Except death. Sorry; forgot. Death is definitely number one. In third place of things we love, well, there were those nice ice-cream desserts they had at this little place in Tehran. When I was Mayor I had them brought in on Fridays. Good times, good times. But once I found a hair.

(2356 words excised concerning Jewish penetration of the Iranian Dessert-Industrial complex)

... Na na na nah, nah, everythings underground! And your Congressors cut funding for the nuclear bombs which permit the busting of the bunker. Na na na! I do a taunting dance and cock my hips mockingly! In sudden seriousness, please to be thank them for this, although we lost a days work in the labs due to the celebration. I even permitted the drinking of whiskey, and decreed that the suppliers of alcohol be only lightly killed. (549 repetitions of na na na na excised)

. . . and if you had the problem I have with razors you would know why my beard seems so tentative at times; if I may speak with you man to anointed hastener of the Apocalypse, how do you get such a smooth shave? A hot towel? Perhaps the Five-Blade Razor of which we have heard muttered rumors? Personally, I use an exfoliating agent which (8343 words excised)

. . . and Jack Bauer will not be able to save you this time, my friend. If there is an attack on our country we will double our aid to the Iraqi patriots, double our funding to Hezbollah and its female auxiliary wing Sisboombah, and double again our attempts to secrete through your borders weapons both chemical and biological.

Ah er, reduce everything I said in the previous paragraph by half. We will START doing those things. Yes, that is the thing that is the ticket: start. We will also use our fearsome weapons of unspeakable lethality to destroy your planes before they are even built, let alone launched. We can sink your mighty aircraft carriers by shouting in unison, so great is our national will.

. . . Seriously, when I came to the UN and you didnt even send a fruit basket, it hurt. Did you not see how well I was received? Did you not see the light of God that surrounded me when I spoke, how no one blinked as I related our message, how doves came out of my mouth and the pants of all were filled with flowers. Did you not note how the exact number of letters I spoke divided by the sum (in Euros) we paid the Chinese engineers was the winning lottery number the following week? Including the Powerball? And you seek to confound my work to bring back the Messiah and bring the world once more into the arms of Islam? Including all penguins?

What are you, nuts?

Sincerely and Death to America,

Mahmoud, descendant of Xerxes, 34th degree Mason, personal valet of the hidden Imam, and not just a member of the Hair Club for Men Im also the President! Death to America.




ETWolverine rated this answer Excellent or Above Average Answer

Question/Answer
excon asked on 05/12/06 - Hamas


Hello Laydownforislamers:

Do you think your tax dollars should go to feed Hamas led hungry Palestinians? If yes, do you think any of that money will be spent on weapons that will be used on Israel, our ally? Or maybe even, God forbid, us?

Against the backdrop of Ahmadinejad's letter this week and his constant threats against the US and its allies in recent weeks and months, it is clear that Iran perceives itself as being in a state of active war against the US. It is also a fact that Hamas is now an official client of Tehran.

Indeed, even before Hamas subordinated itself to Tehran, the movement was in a declared state of war against America. On December 17, 2001, Hamas published a joint declaration with the Islamic Jihad in which it declared, "Americans are the enemies of the Palestinian people," and Americans "are a target for future attacks."

The Bush administration just pledged $10 million in medical assistance to Hamas. Every penny of ours that is transferred in "direct aid" to the Palestinians is money that will prevent Hamas from failing.

Do I feel for starving children around the world? Sure, but I feel less about the children of our enemy. You?

excon

tomder55 answered on 05/12/06:

In principle I have no problem with the food aid package . But I would send food and not cash. This smells like oil-for food . Israel has been in a state of war with them for years and still they released $55 million in tax and customs revenues "for humanitarian needs such as medicines and health needs.(translate ...kassam rockets )" . What both nations are doing is bowing to international pressure.

excon rated this answer Excellent or Above Average Answer
Itsdb rated this answer Excellent or Above Average Answer

Question/Answer
excon asked on 05/12/06 - NSA and Bush


Hello goingdownthedrainers:

"The privacy of all Americans is fiercely protected in all our activities".

Bwa, ha ha ha ha ha ha. Chuckle, chuckle.... Oh, boy.

That's kinda like "Mission Accomplished" isn't it?

I know. You don'thaveanythingtohiders are just fine with it. But in fact, you don'thaveanythingtohiders, are really a bunch of havealottohiders, if say a secretly acquired video of one's bedroom activities all of a sudden appeared on the internet.

Or, it could be true, that donthaveanythingtohiders, never screw, cause its dirty.

excon

tomder55 answered on 05/12/06:

I did not realize that getting my phone # gave them the ability to watch me in my bedroom . Gee aint technology great ? But analyzing phone call traffic to look for clusters of callers associated with al Qaeda is a very good thing to do, exploits our technology and poses absolutely no invasion of privacy.

Is what the gvt doing lawful ? Congress and President Clinton in 1994 thought so

excon rated this answer Excellent or Above Average Answer
Itsdb rated this answer Excellent or Above Average Answer

Question/Answer
jackreade asked on 05/11/06 - Freedom or Security?

What do you choose?

If you choose security, you will be happy to see that Big Brother is firmly in place as we see in the news today that the government can monitor all our telephone activity. They have the largest data base in the history of the world, and you're in it.

If you choose freedom, wake up.

The Islamic terrorists want America to make a choice between freedom and security. That is part of their plans.

~~~~~~~~~~~~~~~~~~~~~~~~~~~~~~~~~~~~~~~~~~~~~~~~~~

tomder55 answered on 05/12/06:

I just can't bring myself to get all worked up about the fact that the gvt. knows I called the local pizza shop last Friday .If they care about it that would concern me .It's obvious that what the NSA does with this vast amount of data is to run it through computers, looking for suspicious patterns, especially involving known or suspected terrorist phone numbers. Assuming that there are 200 million adult Americans, each of whom places or receives ten phone calls a day (a conservative estimate), it would require an army of 35,000 full-time NSA employees to pay a total of one second of attention to each call.

Are you as outraged at the invasion of privacy that you are subjected to by banks ,credit card companies and internet/telephone marketers who regularly peddle your personal information on the open market ?

Itsdb rated this answer Excellent or Above Average Answer
jackreade rated this answer Poor or Incomplete Answer

Question/Answer
jackreade asked on 05/12/06 - America is now a Plutocracy

AMERICA IS NOW... A PLUTOCRACY


The BushCrimeFamily, Cheney, NeoCon PLUTOCRACY. Government by the Millionaire Class

It was just all too easy to subvert our Republic.

tomder55 answered on 05/12/06:

then there are Clintonistas who got rich after they became President .Or maybe in 2004 we could've elected Kerry ,the wealthiest member of the Senate .Or perhaps we have watched 2 generations of the Kennedy clan abuse power. Tell you what ....Why Don't you run ?

jackreade rated this answer Poor or Incomplete Answer

Question/Answer
fredg asked on 05/11/06 - Tax Rebates

Hi,
According to the National News Networks, the President of the US, some Democrats, and Most Republicans, are now going to extend the "Tax Rebates" to Americans!
This is trying to replace the National News with something "good", while forgetting about the other things, such as Iraq, $8,000,000,000,000 (YES, Trillion) National Debt of the United States!
These tax rebates would be as follows: forgive me if I'm off a few dollars:
annual income of less than $100,000 = rebate of from $46 to $400 for the year.
Annual income of over $100,000 = rebate of around $3,000.
Annual income of One Million = around $4,000.
Annual income over One Million = $41,000 approx.
The rich get richer, and the poor get poorer. For most of use, it means a little extra gas money, but not too many gallons worth!

If you are earning less than $100,000 per year (which is 95% of Americans!); you will probably get back $46 for the year!

This ploy by the Republics is to get re-elected in November in the United States.
Question: Since the President's approval rating is now down to about 31%, and the Republicans' ratings are below 25%, do you think this ploy will work??
fredg

tomder55 answered on 05/11/06:

I think it is silly to give the rebates . They did more for the avg. American this week by extending the tax cuts. Rebates are temporary, one-time events that boost economic growth less than permanent tax cuts.The fiscally responsible, pro-growth policy choice is permanent tax rate reductions combined with spending restraint.They took care of the first part of the equation with the tax cuts now it is time for them to restrain themselves from the pork tray .

excon rated this answer Excellent or Above Average Answer
fredg rated this answer Excellent or Above Average Answer

Question/Answer
Erewhon asked on 05/11/06 - With all the flair of George Bush, Scotland gives a luncheon for a man who didn't attend!

Bill Clinton missed out on the meal itself, but after his round of golf gave an 80-minute speech on his vision for the world in the 21st century.

No such thing as a free lunch when taxpayer helps pick up tab for Bill
RHIANNON EDWARD

* Taxpayers foot the bill for Clinton visit to Scotland
* Lunch bill comes when Executive claims to have 'no money available'
* Bill Clinton did not actually attend meal as he was playing golf

Key quote "That money would have been far better spent in Malawi, rather than on a booze-up for members of the Executive. They have lost all sense of priority. If people want to have lunch with Bill Clinton, fine, but not at a total abuse of taxpayers' money." - Alex Neil, SNP MSP for Central Scotland

Story in full
BILL Clinton, the former president of the United States, flew into Scotland yesterday to speak at a gala lunch subsidised by tens of thousands of pounds of taxpayers' money.
Advertisement: dating.scotsman.com 20% off

Although the event was organised by three Glasgow businessmen, the Executive paid out 20,000 to support the event. Government agencies, including the Scottish Environment Protection Agency (SEPA), and other public bodies took tables at a cost of 5,000 each.

Among the politicians who enjoyed a lunch of Scottish lobster, rack of lamb and Drambuie ice-cream, washed down with an elegant 2003 Crozes Hermitage, were Tom McCabe, the finance minister, and Tavish Scott, the Liberal Democrat transport minister. Nicola Sturgeon, the deputy leader of the Scottish National Party, was also at the event in Glasgow's Thistle Hotel.

Glasgow council taxpayers might have been surprised to learn the authority thought it needed to buy a table, which was headed by Steven Purcell, the council leader.

Alex Neil, SNP MSP for Central Scotland and convener of the enterprise committee, said if the Executive wanted to help charities there were more efficient ways of doing it. "That money would have been far better spent in Malawi, rather than on a booze-up for members of the Executive. They have lost all sense of priority. If people want to have lunch with Bill Clinton, fine, but not at a total abuse of taxpayers' money."

Mr Neil said that only Jack McConnell, the First Minister, and another member of Cabinet were needed to represent Scotland. "It annoys me that we are working with charities struggling to get 10,000 and the Executive says there is no money available - and then they waste 20,000 on an event like this."

Bill Aitken, the Conservative chief whip, said:

"I am all for promoting Scotland, but this clearly has not been value for money. The fact that Mr Clinton turned up so late indicates that he did not consider the event important and this money could have been better spent."

Mr Clinton did not attend the meal, but chose instead to play golf at Prestwick. It was only after shooting an 89 on the famous Ayrshire links that he headed to Glasgow to wow his audience with an 80-minute speech.

Despite Mr Clinton's absence from the meal itself, the chance to hear the former president speak was enough to open the cheque books of people from a wide range of businesses. Royal Bank of Scotland took three of the 50 tables, each one costing 5,000. The HSBC took one, as did Dunfermline Building Society.

They were packed in alongside the hairdresser Charlie Miller, the businesswoman Michelle Mone, and Barry Ferguson, the captain of Rangers Football Club. Sir Tom Hunter, the entrepreneur and philanthropist, the former footballer Kenny Dalglish and Willie Haughey, the tycoon, were also present.

Mr Clinton set out his vision of the world in the 21st century, using the forces of globalisation for good and harnessing the full potential of clean energy.

He also gave public support to his old friend, Tony Blair. Mr Clinton described the "third way" as the "best political philosophy" for a modern country.

He said: "Whatever the political problems the government are in, the UK is way better off than it would have been had it not been governed the way it has for the last ten years."

An Executive spokesman said it had spent 20,000 on branding the event and taking one table for international media. He said "This is a great opportunity to promote Scotland on the international stage so we think this is money well spent."

ProjectScotland, the Executive body to recruit youngsters into volunteering and Determined to Succeed, the Executive body to boost education standards, also had a table each, which they paid for.

Glasgow City Council said: "We are actively engaged in raising the city's profile on a national and international stage. Whether it be attracting further commercial investment and jobs, or showcasing our dramatic transformation and regeneration, it is crucial that Glasgow plays a part in events such as this."

===

Is there something wrong with giving a luncheon for a man who was not going to sit down and eat it?


tomder55 answered on 05/11/06:

yes there is but Bubba wasn't raised that way .

it's not like our former Prez to miss a meal . that heart attack must've been a wake up call. Me Iwould not have missed that meal but then again I only get and 89 on a 9 hole course .

Mr Clinton described the "third way" as the "best political philosophy" for a modern country.

lol if that aint the very definition of triangulation I don't know what is !!!

Erewhon rated this answer Excellent or Above Average Answer

Question/Answer
quixotic_Choux asked on 05/10/06 - "Eve of Destruction"

"The eastern world, it is exploding
Violence flarin, bullets loadin
Youre old enough to kill, but not for votin
You dont believe in war, but whats that gun youre totin
And even the Jordan River has bodies floatin

But you tell me
Over and over and over again, my friend
Ah, you dont believe
Were on the eve
of destruction.

Dont you understand what Im tryin to say
Cant you feel the fears Im feelin today?
If the button is pushed, theres no runnin away
Therell be no one to save, with the world in a grave
[Take a look around ya boy, it's bound to scare ya boy]

And you tell me
Over and over and over again, my friend
Ah, you dont believe
Were on the eve
of destruction.

Yeah, my bloods so mad feels like coagulatin
Im sitting here just contemplatin
I cant twist the truth, it knows no regulation.
Handful of senators dont pass legislation
And marches alone cant bring integration
When human respect is disintegratin
This whole crazy world is just too frustratin

And you tell me
Over and over and over again, my friend
Ah, you dont believe
Were on the eve
of destruction.

Think of all the hate there is in Red China
Then take a look around to Selma, Alabama
You may leave here for 4 days in space
But when you return, its the same old place
The poundin of the drums, the pride and disgrace
You can bury your dead, but dont leave a trace
Hate your next-door neighbor, but dont forget to say grace
And tell me over and over and over and over again, my friend
You dont believe
Were on the eve
Of destruction
Mm, no no, you dont believe
Were on the eve
of destruction.

Song by Barry McGuire

~~~~~~~~~~~~~~~~~~~~~~~~~~~~~~~~~~~~~~~~~~~~~~

tomder55 answered on 05/11/06:


American Girls and American Guys
We'll always stand up and salute
We'll always recognize
When we see Old Glory Flying
There's a lot of men dead
So we can sleep in peace at night
When we lay down our head

My daddy served in the army
Where he lost his right eye
But he flew a flag out in our yard
Until the day that he died
He wanted my mother, my brother, my sister and me
To grow up and live happy
In the land of the free.

Now this nation that I love
Has fallen under attack
A mighty sucker punch came flyin' in
From somewhere in the back
Soon as we could see clearly
Through our big black eye
Man, we lit up your world
Like the 4th of July

Hey Uncle Sam
Put your name at the top of his list
And the Statue of Liberty
Started shakin' her fist
And the eagle will fly
Man, it's gonna be hell
When you hear Mother Freedom
Start ringin' her bell
And it feels like the whole wide world is raining down on you
Brought to you Courtesy of the Red White and Blue

Justice will be served
And the battle will rage
This big dog will fight
When you rattle his cage
And you'll be sorry that you messed with
The U.S. of A.
'Cause we'll put a boot in your a**
It's the American way

Hey Uncle Sam
Put your name at the top of his list
And the Statue of Liberty
Started shakin' her fist
And the eagle will fly
Man, it's gonna be hell
When you hear Mother Freedom
Start ringin' her bell
And it feels like the whole wide world is raining down on you
Brought to you Courtesy of the Red White and Blue

song by Toby Keith



ETWolverine rated this answer Excellent or Above Average Answer
quixotic_Choux rated this answer Excellent or Above Average Answer

Question/Answer
jackreade asked on 05/10/06 - Beware of Christian Nuclear Jihad

"The "Divine Strake" bomb test scheduled for the US nuclear test site in Nevada has been delayed, at least temporarily, due to a lawsuit filed by nearby residents who feared it would send radioactive material their way. It's already sent something else: a message to the Islamic world. We're telling them that our God is greater than theirs, because he's given us the Bomb. And we're warning them that we're planning to use it.

"Divine Strake" will use non-nuclear explosives to simulate a nuclear 'bunker-buster' attack, creating a mushroom cloud that may rise 10,000 feet (nearly two miles) into the air. Its stated purpose is to enhance our ability to use nuclear bombs to destroy underground facilities. And where have we claimed it will be necessary to do that? The Islamic Republic of Iran, of course.

This explosion is very possibly a violation of nuclear test-ban treaty provisions, despite the use of non-nuclear explosives, because it is being used to design nuclear weaponry. But testing is not its only purpose. The test is intended to send a threat - not just to Ahmadinejad, but to the entire Muslim world. In all likelihood, the name is part of the communication.
.
So far, the only people who have responded in fear are residents of nearby areas. They have sued to stop the test because they're afraid it will scatter radioactive materials from old nuclear tests into the air and onto their communities.

Why the name? "Strake" is an obscure term from nautical construction, which the American Heritage Dictionary defines as "a single continuous line of planking or metal plating extending on a vessel's hull from stem to stern." It's meaningless in this context, and serves as a place-marker for the adjective: "divine." [or "typo" for Strike?, hence, Divine Strike-jackreade's comment]

Is the name part of a psychological operation against Iran, and Muslims in general? It's very likely. The widely-publicized burning of an insurgent's body by American soldiers was a classic psy-ops move, carefully planned by military intelligence (Hayden's people). So are the Quran desecrations and other anti-Islamic gestures used by the torture artists in the American Gulag.

The flap over Bush's use of the word "crusade" to describe the Afghanistan invasion, and the Islamic objection to the term "Operation Enduring Freedom" (only God is ever-enduring for Muslims) made the impact of language crystal-clear to US leaders. It's impossible to imagine that Pentagon leaders wouldn't realize the message they're sending by using a word like "divine" to describe a gigantic bomb.

So what's the message? It's simple: Beware of the Christian Nuclear Jihad." RJ Eskow, Blogger

~~~~~~~~~~~~~~~~~~~~~~~~~~~~~~~~~~~~~~~~~~~~~~~~~~~


Do you think that threats of great violence and psy ops, and perhaps, the real bombing of Iran will have any kind of effect Bush and Admin desire?

tomder55 answered on 05/11/06:

personally I'd be sending them blunter messages .


Why is it assumed that the test is to simulate a nuclear bunker buster. Perhaps the military is trying to find a non-nuclear approach to dealing with hardened underground bunkers/facilities .

I find nothing wrong with the delay . Let the gvt. comply with all local ordinances . From what I have read ,the blast will not be at a former nuclear test area ,but let it go through all possible review first . A two to three week delay is not important .

jackreade rated this answer Above Average Answer

Question/Answer
paraclete asked on 05/10/06 - and it takes James Bond to work it out?

INVASION USA
Osama's exploits south of border
Al-Qaida in league with Mexican radicals in plot to penetrate U.S., says MI6 report
Posted: May 10, 2006
1:00 a.m. Eastern

Editor's note: The following story is adapted from Joseph Farah's G2 Bulletin, the premium, online intelligence newsletter published by the founder of WND.

By Gordon Thomas
2006 WorldNetDaily.com

LONDON Britain's secret intelligence service, MI6, has established the first proof al-Qaida is playing a major role in the new Cold War between North and South America with Osama bin Laden's terror network seeing itself in league with Mexican subversives in infiltrating the U.S. border.

The evidence emerged as Venezuelan President Hugo Chavez swash-buckled into London after scoring a win in yet another venomous battle with Washington for influence and economic advantage across the Latin American continent.

Chavez is in London to meet the capital's anti-Bush mayor, Ken Livingstone, and other prominent British opponents of the war in Iraq. His arrival coincides with the downward spiral politically of Prime Minister Tony Blair largely over his continued support for Bush.

Downing Street will monitor the Chavez visit closely not least because he controls the western hemisphere's largest supply of oil reserves. As oil prices soar, Chavez has used the extra profits to reinforce his position with his electorate. He said last week he would seek "indefinite" re-election beyond the constitutional limit of 2014.

Chavez, a 51 year-old paratrooper, descended on London this week and was boosted by the knowledge that his rapidly expanding clout in Southern America could soon see a dramatic shift of power after elections in Peru, Nicaragua and Mexico.

This would result in a standoff between Western oil companies worried about rising oil prices and South American oil producers' new-found enthusiasm for threatening foreign companies with a further hike.

In the words of a MI6 memo, the situation "is a new and dangerous threat to stability that is also being exploited by al-Qaida."

Details of al-Qaeda's penetration into Latin America emerged from documents discovered during recent anti-terrorist operations in Pakistan to try and locate Osama bin Laden.

The documents included evidence that al-Qaida has established links with the Colombian terror group, FARC, and the Shining Path, SL, in Peru. They also reveal al-Qaida's links with thousands of Muslim students in the Dominican Republic.

Another Pakistani document shows the links between al-Qaida and Mexico's Popular Revolutionary Army, EPR. The documents reveal that al-Qaida sees EPR as collaborators in attacks in Mexico on foreign targets "especially those of the United States and Britain." It also says that EPR can play a key role in allowing al-Qaida operatives to enter the United States through the busiest land crossing in the world Tijuana.

Another document reveals that along Peru's border with Chile "a large Arab community is providing substantial sums of money for al-Qaida."

But the closest links al-Qaida has are with Venezuela. Exploiting Chavez's latest tirade against the Bush administration, al-Qaida is firmly entrenched in the country.

Before flying to London, Chavez said: "The axis of evil is Washington and its allies around the world who go about threatening, invading and murdering. We are forming the axis of good."

The godfather of that axis is Fidel Castro, Cuba's leader for 45 years. But in support is Evo Morales, the president of Bolivia who last week promised: "I am going to be a nightmare for Washington."

In coming presidential elections the candidates are Nicaragua's Daniel Ortega, Peru's Ollanta Humala and Andres Lopez of Mexico. The Mexican populist likes to see himself as a mirror image of Chavez and has labelled the country's outgoing president, Vincente Fox, "a puppy of Bush."

The documents discovered in Pakistan have become of prime concern to MI6 given Britain's substantial holdings in Latin America. These could be seriously damaged by what one MI6 officer called "Chavez and his rogue's gallery of sinister wannabees and corrupt opportunists."

Chavez has so far spectacularly avoided Washington's efforts to curb his ambitions. He has warned Secretary of State, Condoleezza Rice, "I sting those who rattle me."

It is over threats like that MI6 analysts try to decipher how far Chavez will allow al-Qaida to be his sting master.

Already MI6 say that Venezuela is now one of the main conduits for trafficking drugs to Europe and al-Qaida is a major player.

From Venezuela the drugs are taken by high-speed ocean-going boats to Africa's West Sahara. The cargoes are run ashore north of the town of Dakhia and trucked overland through Morocco into southern Spain. From there they are smuggled into France, Germany and Britain.

Deep inside their headquarters overlooking the River Thames, the MI6 analysts work in a room that is accessed by a swipe card, the codes of which change regularly.

The room houses the Terrorist Attack Assessment Center. Inside its computer-lined walls and state-of-the-art communications, analysts sit at workstations around the clock. TAAC is directly linked to the Pentagon and the CIA. Both have their versions of TAAC.

The MI6 department regularly updates its director general, John Scarlett. He is the quintessential English spymaster. In his customized suits and hand-stitched cotton shirts, he has a touch of the James Bond about his sartorial elegance.

He is taking a close interest in the documents that indicate how al-Qaida sees Latin America as a continent where it can expand its activities.

MI6 analysts have established that the documents are the work of Ayman al-Zawahiri, a founder member of al-Qaida and accepted by Western intelligence services as its prime strategist next to bin Laden.

Al-Zawahiri studied in Paris and London to become a recognized authority in behavioral psychology. After graduating from Cairo University he traveled widely.

An MI6 file confirms a Mossad profile of the heavily bearded psychiatrist that he is arrogant and takes an obsessive pleasure watching film of the attacks of Sept. 11, 2001 when he first emerged from the shadows to sit alongside bin Laden.

Both MI6 and Mossad believe al-Zawahiri made several visits to Latin America during the last decade.

As Joseph Farah's G2 Bulletin first reported in 2003, Pentagon officials have confirmed human smuggling rings in Latin America are attempting to sneak al-Qaida operatives into the U.S.

Before the U.S.-led coalition attacked Iraq, the U.S. State Department offered congressional testimony that both al-Qaida and the Shiite terrorist group Hezbollah were taking firm hold in "America's backyard."

Mark F. Wong, the State Department's acting coordinator for counterterrorism, told the House International Relations Committee about the threat posed by both groups in Latin America.

Yet, then the matter seems to have been dropped perhaps for diplomatic reasons, perhaps for political reasons.

But in 2003, G2 Bulletin reported authorities in Silvio Pettirosi International Airport in Asuncion, the Paraguayan capital, reported the arrival of a growing number of visitors carrying European passports, but undoubtedly appearing to be more Middle Eastern than anything else.

Some of these "Europeans" could not even speak the language of their so-called mother land.

There was very little doubt most of these visitors went on to find their way to the triple border region where Argentina, Brazil and Paraguay meet. This region, often described as a lawless area, is nicknamed by some intelligence station agents as "The Muslim Triangle meeting zone."

Intelligence experts have been warning since the late 1990s they had noticed a tendency among Islamic terrorists to operate from Paraguay, a landlocked country in the heart of South America, with a territory slightly smaller than California, and with geographic extremes perfect for hiding illegal activities.

G2 Bulletin reported in 2003 the terrorists using Argentina are organized in active cells around the country with safe houses in neighboring Paraguay. An Argentinean document seen by G2B describes part of the drug-smuggling trail, as well as that of weapons and people. These elaborate trails run through a web of border crossings pointing also to the complex cooperation between various "smuggling experts." These belong to jihadi organizations such as al-Qaida, joining forces with local drug lords, developing and oiling their smuggling mechanism all the way to Mexico aiming ultimately to hit the U.S.

The Argentinean intelligence service assessment, privy among others, to European and Middle Eastern agencies, has reached a significant and grave conclusion, according to G2 Bulletin. It claims since 9/11 and the partial success in the war against terrorism, mainly in the Middle East, Afghanistan and Central Asia, the jihadi pendulum is tilting more and more toward South America. The reason terrorist cells in Paraguay, whether active or dormant, can continue to grow and flourish, is because of widespread corruption in South America.

The lawlessness and disorder in Paraguay, enabled operatives of such terrorist groups as al-Qaida, Hezbollah, Islamic Jihad and Hamas to feel safe, even in the heart of Asuncion. These organizations, and probably more, turned Paraguay into a logistical base, as one local journalist told G2 Bulletin: "It's easy. At this stage our country is not engulfed in a civil war or guerrilla campaign and, therefore, security forces are more prone to financial kickbacks."

The terrorists even get some official support in Latin America, according to some sources. As WorldNetDaily reported, a Venezuelan military defector claims Venezuela's Chavez developed ties to terrorist groups such as al-Qaida even providing it with $1 million in cash after Sept. 11, 2001.

Air Force Maj. Juan Diaz Castillo, who was Chavez's pilot, told WorldNetDaily through an interpreter that "the American people should awaken and be aware of the enemy they have just three hours' flight from the United States."

Diaz said he was part of an operation in which Chavez gave $1 million to al-Qaida for relocation costs, shortly after the Sept. 11, 2001, terrorist attacks on the United States.

London-based intelligence expert Gordon Thomas is the author of "Gideon's Spies: The Secret History of the Mossad"and a regular contributor to Joseph Farah's G2 Bulletin. This report includes background reports from previous G2 Bulletin dispatches.

Related offers: Get "Secrets of the Invasion," the May issue of Whistleblower magazine, which dissects the real reasons for Washington's tolerance of a porous border and millions of illegal aliens living and working inside America.

tomder55 answered on 05/10/06:

al-Qaeda just wants to come here to do the jobs Americans won't do .

the open borders crowd says there is no threat from the Southern border even though it is an indisputable fact that the most dangerous street gang operating in the United States today ;MS 13 ;is a militia that has infiltrated from el- Salvador through Mexico. Their ties to al-qaeda has already been reported on (the American intel. agencies of course find no link).(additional link here )

paraclete rated this answer Excellent or Above Average Answer

Question/Answer
Itsdb asked on 05/09/06 - Rhetoric? No.

Yesterday I posted the following:

>>the economy is booming, unemployment is extremely low, the stock market is soaring, millions of jobs have been added and we've had no more terrorist attacks here.<<

If you'll notice in the comments my assertions were challenged, and I love a good challenge. So, on to the challenge...

>>The economy is booming...<<

    The combination of strong growth and low inflation has been as good as anything seen since the 1960s, with the sole exception of the explosive growth of the late 1990s. The economy is on track to grow by about 3.5 per cent for the second consecutive year - close to the level economists view as the speed limit before inflation starts to rise. The core inflation rate has remained remarkably subdued despite the upward pressures ex-pected from surging oil prices. Wage growth may have struggled to keep pace with inflation, but soaring house prices have more than made up for this shortfall for the two-thirds of the population that own their own homes.


>>unemployment is extremely low<<

    U.S. employers added an unexpectedly strong 211,000 jobs in March and the jobless rate slipped to a 4-1/2-year low, according to a government report on Friday that underlined a relatively vigorous labor market.

    (It remained at 4.7% for April)


>>(those dot com guys are all working at fast food places now - yeah, they're working)<<

    I'll leave that one for someone else to explain.


>>the stock market is soaring????<<

    Dow Inches Closer to All-Time High

    NEW YORK, May 8 -- Stocks finished a quiet session little changed Monday as investors' anticipation of the Federal Reserve's decision on interest rates muted their reaction to lower oil prices and a trio of acquisitions.

    The Dow Jones industrial average rose 6.80, or 0.1 percent, to 11,584.54, its highest close in more than six years. The Dow is 138 points from its all-time high close of 11,722.98, reached Jan. 14, 2000.

    The Standard & Poor's 500-stock index fell 1.10, or 0.1 percent, to 1324.66, and the Nasdaq composite index gained 2.42, or 0.1 percent, to 2344.99.

    With no new reports to offer clues about the economy, investors traded cautiously ahead of the Fed's latest move on interest rates when policymakers meet Wednesday.

    Investors have been impressed by the economy's strength despite surging energy costs and the Fed's mission to stifle inflation by gradually raising short-term interest rates. Enthusiasm over solid corporate earnings growth -- companies in the S&P 500 have averaged double-digit gains for 15 consecutive quarters -- have carried stocks higher so far this year.


>>Where do you live???<<

    Amarillo, Texas.

    "Nothing but sunshine and blue sky."

    That's the bright picture Keller Williams Realtor David Grimes paints of Amarillo's current and future housing market...The developers are developing them as fast as they can," he said. "There's just not enough lots right now. and that's a good problem to have."

    ~~~~~~~~~~~~~~~~~~~~~~~~~~~~~~~~~~~~~~~~~~~~~~~

    "Building permits over the past 15 years have generally shown a steady increase with 2004 and 2005 being banner years. Building permits issued in 2004 totaled $358 million in construction value and, in 2005, totaled $454.8 million in construction value."

    ~~~~~~~~~~~~~~~~~~~~~~~~~~~~~~~~~~~~~~~~~~~~~~~

    Amarillo's boom isn't over yet.

    Experts predict a rosy future as large commercial projects continue to boost the area's economy.

    "When you look at Bell Helicopter, the new Ben E. Keith Distribution Center and medical center construction, those are the anchors for everything else," said Cary Finney, city of Amarillo Building Official. "Those are the projects that will mean more jobs."

    And more jobs will help the service industry, including retail, hotels, restaurants and other businesses.

    "Right now, we're seeing a healthy growth," Finney said.

    In 2005, the city of Amarillo granted $259 million worth of permits for commercial construction, renovation and roofing. City-issued commercial building permits for new construction last year is more than double the figure for 2004.

    "I think the development will continue," said commercial investor J. Gaut of J. Gaut & Associates. "We have a lot going for us in Amarillo - favorable climate, good cost of living, and plenty of available land."


>>Oh and the terrorists are all in the MidEast killing our soldiers and civilians from various countries.<<

    Like I said, we've had no more terrorist attacks here. Is that to be disputed? And apparently ol' bin Laden is going to direct his fight toward international peacekeepers in Sudan. Kind of speaks to the cowardly, evil nature of a terrorist to target innocent civilians and peacekeepers in a country as ravaged as Sudan wouldn't you say?

tomder55 answered on 05/09/06:

As Larry Kudlow repeatedly says ;the economic boom is the greatest story never told .

Todays economy may be the greatest story never told. Its an American boom, spurred by lower tax rates, huge profits, big productivity, plentiful jobs, and an ongoing free-market capitalist resiliency. Its also a global boom, marked by a spread of free-market capitalism like weve never seen before.

Recent data on production, retail sales, and employment are stronger than expected. The latest durable-goods report shows huge gains in orders for big-ticket items like airplanes, transportation, metals, machinery, and computers even cars and parts. These orders suggest that the economic boom will continue as far as the eye can see. And theres more: The backlog of unfilled orders, the best leading-indicator of business activity, gained 12 percent at an annual rate in the first quarter. With this kind of real-world corporate activity in the pipeline, highly profitable businesses will be doing a lot of hiring in the months ahead in order to expand plant and equipment capacity. Just what the doctor ordered.


also here

Even the NY Slimes had to admit the economy is robust .

Itsdb rated this answer Excellent or Above Average Answer

Question/Answer
jackreade asked on 05/08/06 - Prosecuting the Press??

"Never once in the history of the United States has the national government criminally prosecuted the press for publishing information the government would rather keep secret. In recent weeks, however, the Bush administration and its advocates, including Attorney General Alberto Gonzalez, have repeatedly threatened to prosecute the New York Times and the Washington Post for publishing their Pulitzer Prize-winning exposs of the administration's secret prisons in Eastern Europe and secret NSA surveillance of American citizens.

Specifically, the President and some of his supporters have threatened to prosecute reporters and publishers for violating a provision of the 1917 Espionage Act, which provides in part that "whoever having unauthorized possession . . . of information relating to the national defense, which information the possessor has reason to believe could be used to the injury of the United States . . . willfully communicates . . . the same to any person not entitled to receive it . . . is guilty of an offense punishable by 10 years in prison."

For at least three reasons, such threats are largely empty ones. First, this provision was never intended to reach the press. When the Espionage Act of 1917 was initially proposed by President Woodrow Wilson, it included a section that would expressly have made it a crime for the press to publish information that the President had declared to be "of such character that it is or might be useful to the enemy." Congress overwhelmingly rejected this proposal, with members of both parties characterizing it as "un-American" and as "an instrument of tyranny." The provision of the 1917 Act invoked by the Attorney General Gonzalez was directed at enemy spies, not at reporters and newspapers attempting to inform the American people about the activities of their government. Unfortunately, the Bush administration appears not to know the difference.

Second, if the section of the 1917 Act applied to journalists, it would unquestionably violate the First Amendment. Laws regulating speech must be precisely tailored to prohibit only speech that may constitutionally be proscribed. This requirement addresses the concern that overbroad laws - laws that are not narrowly crafted - will chill the willingness of individuals to speak freely because of a fear that their expression might be unlawful. Not surprisingly, because the 1917 Act was drafted before the Supreme Court had ever interpreted the First Amendment, it does not incorporate any of the safeguards the Court has since held the First Amendment requires. For example, the Espionage Act provision is not limited only to publications that pose a "clear and present danger." For this reason, any prosecution of the press under this section would be dismissed out-of-hand because the statute itself is unconstitutional.

Third, if Congress today enacted legislation incorporating the requirements of the First Amendment, it could not reach the exposs published by the New York Times and the Washington Post, for they were clearly protected by the First Amendment. Under existing law, such a statute would have to be limited to publications that (a) do not disclose information of legitimate and important public interest and (b) pose a clear and present danger of serious harm to the national security. The exposs of the Bush administration's secret prisons and secret electronic surveillance of American citizens clearly concerned matters of legitimate and important public interest, and the administration has made no showing that these disclosures created a clear and present danger of serious harm to the national security. Thus, under a properly drawn statute these disclosures could not constitutionally be punished.

I do not mean to suggest, of course, that the government has no interest in keeping military secrets or that it may never punish the press for disclosing classified information. To the contrary, the government may take many steps to keep such information secret, including (in appropriate circumstances) firing and even criminally prosecuting public employees who unlawfully leak such information. Moreover, in narrowly-defined circumstances, the government may prosecute the press for disclosing classified national security information. Such a prosecution might be consistent with the First Amendment, for example, if a newspaper reveals that the government has secretly broken an important al Qaeda code, where this disclosure causes al Qaeda to change its code. But the government can never punish the press for publishing information of legitimate and important public concern, and especially not when the information reveals possible government wrongdoing, as was true in both the secret prison and NSA situations. Such revelations are essential to effective self-governance and they are at the very core of the First Amendment.

Although the continuing threats of the Bush administration are largely bluster, they must nonetheless be taken seriously. They represent further steps in this administration's relentless campaign to intimidate and control the press, and to keep the American people in the dark. This, in itself, poses a clear and present danger to our democracy."
by Geoffrey R. Stone

~~~~~~~~~~~~~~~~~~~~~~~~~~~~~~~~~~~~~~~~~~~~~~~~~~~


Comments?

tomder55 answered on 05/09/06:

Your author is mistaken on his premise.

Although the 1st amendment has endured there has been instances in American history when the First Amendment has been set aside. There have been sedition acts and wartime set asides of the 'freedom of press' .

The Sedition Act of 1798, signed by President John Adams, allowed a provision that made it a criminal act to speak or write maliciously of the president or of Congress, which was defined as with the intent to defame or to bring either into contempt or disrepute.The act expired in 1800 and was not renewed.

Wartime censorship has been used to protect national security interests. Abraham Lincoln used this type of censorship during the Civil War. The First Amendment was considered secondary to the preservation of the nation. This included opening mail and censoring anti-Union newspapers.

Your author is not correct about the Espionage Act of 1917 .During WWI the Espionage Act of 1917 made it a crime to write or say anything that might encourage disloyalty or interfere with drafting of servicemen .Subversive books were taken off the shelves in stores and libraries. A Federal Censorship Board was assigned to regulate such activities.

Publications which the Wilson Administration determined were guilty of violating the 1917 Espionage Act "were subject to being deprived of mailing privilege, a blow to most periodicals," according to Sidney Kobre's Development of American Journalism book. A section of the Espionage Act allowed the Postmaster General to declare all letters, circulars, newspapers, pamphlet books and other materials that violated the Act to be unmailable.

As a result of the Espionage Act, about 75 newspapers either lost their mailing privileges or were pressured to print nothing more about World War I between June 1917 and May 1918.


The laws were ruled to be compliant with the United States Constitution in the United States Supreme Court case Schenck v. United States, 249 U.S. 47 (1919).

Justice Holmes speaking for a unanimous Court, concluded ..."The question in every case is whether the words used are used in such circumstances and are of such a nature as to create a clear and present danger that they will bring about the substantive evils that Congress has a right to prevent." During wartime, utterances tolerable in peacetime can be punished.

Another Sedition Act, passed in 1918, considered speaking disloyal or abusive language about the flag, Constitution and government a criminal act.

Similar activities took place during World War II. It was considered an illegal activity to advocate violent overthrow of the government at any level, or to say, do or write anything that might encourage insubordination among the military or to encourage disloyalty.

During the 1930s, the Federal Communications Commission began its policy of requiring broadcast station owners who engage in airing editorial opinions on a controversial issue to offer time on the air to any opposing opinion. The FCC argued that it was not violating First Amendment rights of broadcasters because the airwaves are a limited resource and belong to the public. Thus the right of public access to the airwaves must be protected. This practice, known as the Fairness Doctrine, was scrapped in 1987 and there is talk of it being re-introduced again.I would oppose that since currently there are many on air means for expressing opinions .






ETWolverine rated this answer Excellent or Above Average Answer
Itsdb rated this answer Excellent or Above Average Answer
jackreade rated this answer Average Answer

Question/Answer
excon asked on 05/08/06 - Poll


Hello wingdudes:

Are you still one of the 33% who still support Bush?

I've taken the liberty of answering for you. Tell me if I'm wrong.

Its: yes
gade: yes
Wolverine: of course
labdude: who cares
kindj: I don't think so
tom: Him either anymore
Pdub: probably
Hank: yes
ladybug: dunno

Of course, I've left out any of you who I believe never did support him. Although, some of you might now. Choux? CeeBee? Clete? Beezle? Ben? Ronnie? Anyone?

excon

tomder55 answered on 05/08/06:

If an election were held today between Bush and any other Demoncrat I would vote for Bush .

Do I support him ;mostly yes . I am disappointed that Social Security reform was squashed with barely a fight . That was a fight worth taking to the mat .
I think he only has half the tax equation right. He cut rates and the Laffer curve is working ,and tax revenues are high .However ,when it comes to spending and the expansion of the government the only conclusion I can come to is that he is not conservative. His threats of veto now are almost laughable and the only hope I have that the "emergency "supplemental pork bill will not pass is if the Republican members of the House of Reps. gets a sudden reality check and realize that that pork laden piece of garbage is political suicide. I read an arguement this weekend that by cutting taxes you make the cost of gvt. less expensive and more attractive to the public ...therefore the public demands more of the gvt. It takes discipline to equally cut un-necessary programs . If the size of gvt. continues to increase no amt. of additional revenue will be enough to feed the beast .

I support most of what he has done in the war against Jihadistan except that he has not connected energy security with the war effort. I think the Bush Doctrine will be the model used by future Presidents on how to execute this war. I think short of total war ,the best way to transform the ummah is through democratic reforms .I do think that something comparable to the Marshall Plan also needs to be introduced .The military has done everything asked of them . The civilian equivalent agencies in gvt.(particularily the State Dept. ) have not lived up to their end.

There are many things that he is doing that I have argued the full impact of their initiation will be realized after he is out of office . He will leave a lasting positive impact on the judiciary .His intel reorganization is desperately needed ,and although I have mixed feelings about the adding on extra layers of the intel. bureaucracy ,the centralization of intel. analysis is long over-due .

The short answer is that I still fall within the 35% and that I do not think public opinion polls particularily relevent . A leader who decides based on the daily popularity data as if this were the last 5 minutes of 'American Idol 'is someone I could not support anyway. The model I keep using for basis of comparison is Harry Truman and Bush still compares favorably to him in my view.

ETWolverine rated this answer Excellent or Above Average Answer
Itsdb rated this answer Excellent or Above Average Answer
excon rated this answer Excellent or Above Average Answer

Question/Answer
Erewhon asked on 05/08/06 - Does Germany show the way for the USA?



Germany's jobless urged to take work in asparagus harvest
ALLAN HALL IN BERLIN

EUROPE'S largest legal annual migration is under way with university professors joining roadsweepers and the jobless to pour into Germany from eastern Europe to pick asparagus.

People come in their tens of thousands from Poland and the Czech Republic for two-months of plucking that which Germans love to see on their dinner tables, but of whose harvest they want no part.
Living.scotsman.com MPU

Once again the German government is throwing money at the nation's five million jobless, trying to persuade and cajole them into working for a change. But asparagus-picking is one job they refuse to do.

The asparagus spring harvest is again stirring a heated controversy over the use of seasonal workers from abroad who some complain are taking away jobs from unemployed Germans.

The return of about 300,000 Poles on work permits this season, and several thousand Czechs, has become a national controversy as many people wonder why some of Germany's jobless can't do the work.

Using a carrot-and-stick strategy, the German labour office has launched a campaign to fill at least 10 per cent of seasonal harvest jobs with Germans on the dole. According to Edelgard Woythe, the head of the local employment office in Potsdam - the capital of Brandenburg where 50,000 unemployed could theoretically do the job - the government is trying everything to get Germans into the fields.

"We've put in enormous efforts to convince people to work in the fields," she said. "We've set up training sessions where people can learn how to pick asparagus."

Despite an extra bonus of 25 (18) a day and free transport to the fields, among Brandenburg's army of jobless only 154 have so far taken on harvesting jobs.

Germans can earn between 1,400-3,000 during the six-week asparagus season.

If that's not enough of an incentive, some of the unemployed may be made to learn the hard way - through cuts in their benefits.

===

How would this go down in the USA? Should feckless Americans who would rather be on the dole than pick the nation's lettuce have their handouts cut if they refuse to take the jobs available?



tomder55 answered on 05/08/06:

We already went through this ;one of the few things that Clintoon had right (see workfare .) We have reformed to a large extent the obvious flaws in the social welfare state and although it is uneven because of our Federal System (meaning some bleeding heart states like NY are more "generous" ) overall it is a much better system than the nanny European states ;but I kinda like Angela Merkel and I think she has what it takes to eventually initiate the economic reforms necessary to reduce the high unemployment rates there .

Erewhon rated this answer Excellent or Above Average Answer
Itsdb rated this answer Excellent or Above Average Answer

Question/Answer
kindj asked on 05/07/06 - Charitable Cheney

Charitable Dick Cheney, media's best-kept secret
JOHN REINIERS


Let Vice President Cheney unload a hail of buckshot - and it makes mainstream media headlines as a defining moment is his failed vice presidency.


Let him file his federal tax return and it is reported by CBS News that "Cheney tops Bush in the battle of the bucks."


Let him donate what was the largest amount of bucks in history to charity by any public servant, and you guessed it - nary a headline.


But then again it was a paltry $6.87 million, more than three-quarters of the reported income of the Cheneys.


Read this again: The Cheneys gave $6.87 million to charity in 2005.
A small story perhaps, but come on - doesn't a multimillion dollar contribution to charities by a vice president deserve special recognition? Frankly, I was astonished when I first read this and thought it was a typo because it was buried in a column that leads off with President Bush's tax return - which wasn't even newsworthy - just the typical annual report on the tax returns of the president and vice president. As a matter of fact, the AP headline read "Cheney's income 10 times the Bushes'. And the L.A. Times reported: "Bush pays taxes, Cheney awaits refund,"


I could go on with other headlines, but you get the point. Not one headline in the mainstream media that Cheney gave $6.87 million to charity. The "refund" headline by the L.A. Times is laughable. The reason he's getting a refund is because he overpaid in estimating his taxes and had too much withheld. Another paper spun the AP story by saying not only did Cheney make ten times as much as Bush, but "He is looking for a $1.9 million refund." What gall.


Another equally compelling headline would have been when a former vice president's tax return - Al Gore's - reported a paltry $367 in charitable contributions in 1997. Of course this item never made the headlines either - given the bias of the mainstream media.


The Cheney's income was largely the result of his exercising stock options from his stint at Halliburton, some deferred compensation and royalties from three books written by Mrs. Cheney.


Of interest, the AP story referred to Cheney's adjusted gross income as "largely padded" with income he received by exercising stock options that had been set aside for charity. Here's a guy that sets up a gift arrangement for charity with Halliburton when he took office in 2001 and the AP elects to describe his return as being "padded" -- this was income earmarked for charity in 2001. Why the use of such a pejorative term? (Like padding an expense account.)


The Washington Post couldn't resist referring to Halliburton as "a large military contractor in Iraq," as if Iraq had something to do with this story. And so as to belittle this astounding donation, the Post said "the Cheneys appear to have taken advantage of a special tax break." Hey, anybody who gives three quarters of what they've earned to charity deserves a tax break.


The majority of Americans do not realize how devastatingly effective the media is in shaping attitudes. They can and will destroy the reputations of those they oppose. What is so alarming is that the media mistakes their limitations for high standards.

John Reiniers is a columnist for Hernando Today. He lives in Spring Hill.

tomder55 answered on 05/07/06:


What ? That can't be ! As Reiniers wrote ;Here was the headline in the L.A. Slimes :

Bush Pays Taxes, Cheney Awaits Refund


Compare the Cheneys $6.87 million donation to Al Gores $387 in 1997

Another under-reported fact about his charitible donations;he gave $1.3 million to Capital Partners for Education which provides scholarships for low-income students to attend private high schools.

kindj rated this answer Excellent or Above Average Answer

Question/Answer
quixotic_Choux asked on 05/07/06 - Oil Company Profits

A number of years ago, oil company profits were set by government and oil company agreement and legislation at no more than 10% of the cost of a barrel of oil. So, if oil sells at $30.00 a barrel, the oil company can take a $3.00 profit maximum.

Currently oil trades at $72.00 a barrel with the expectation of $100.00 by the end of the year. $10.00 a barrel profit maximum.

Some expect the $200.00 barrel price within two years. Profit would be $20.00 a barrel.


The windfall profits of exxon come from this situation; it has nothing to do with exxon DOING ANYTHING.

~~~~~~~~~~~~~~~~~~~~~~~~~~~~~~~~~~~~~~~~~~~~~~~~


Should exxon and other companies be allowed to keep the massive profits due to no action on their part, in fact, the four manor oil companies refuse to build more refineries???????

tomder55 answered on 05/07/06:

No ;I think they should invest some of their profits in exploration to discover new oil fields (November 2005, oil imports cost the US $24 billion, and amounted to more than one-third of the trade deficit.), to create more efficient mixes of petroleum and other alternate domestic sources,to add bio-diesal pumps in every station ,and to build more refining capacity that at least meets current domestic demand.

Exxon/Mobile would like to do most of these same things also but it is regulations and domestic opposition that prevents them from doing so. They DO NOT refuse to build more refineries . They are prevented from doing so based on environmental regualtions and the NIMBY attitude of most Americans which got so bad that last year during post-Katrina President Bush resorted to making the suggestion to build refineries on closed military bases .In 1981, the US had 324 refineries with a total capacity of 18.6 million barrels per day. Today, there are just 132 oil refineries with a capacity of 16.8 million b.p.d. meanwhile America demands 21 million barrels of oil per day(up from 18.5 million barrels per day in Jan.2002).

I don't understand why these companies are held to a different standard. When most industries have record profits they are highlighted for their accomlishment on Wall Street.Besides ,the oil industries profits over time were not as lucrative as those in banking, pharmaceutical and many other industries. 2004 Exxon Mobil earned more money -- $25.33 billion -- than any other company on the Fortune 500 list of largest corporations;But if measured by it's gross profit margin, it ranked No. 127.Most financial institutions, such as commercial banks, are routinely more profitable than Exxon Mobil . Exxon Mobil's gross margin of 9.8 cents of profit for every dollar of revenue pales in comparison to Citigroup's 15.7 cents in 2004. By percentage of total revenue, banking is consistently the most profitable industry in America, followed closely by the drug industry(Merck made 25.3 cents for every dollar ).

They sell a product. There is very high demand for that product and a finite known supply. Shouldn't Exxon Mobil be able to charge whatever the market will bear? What is puzzling is not the companies making big profits, but this attitude of entitlement . If we are truly bothered by the oil companies huge profits, we would trade down to smaller cars, push for more and better public transportation options, and stop buying 5,000 square foot Mcmansions for small families .

The fact is that many people who know believe that we have already reached "peak oil". By that I mean that 750 billion barrels have been produced; 1000 billion barrels are known in the ground
;1000 billion barrels are estimated undiscovered .About Half of all that can be produced has been produced according to peak oil theories.

We cannot drill out of this problem but it would help in the short term ;if nothing else than only for energy security reasons. The long term solutions must be addressed and until they are ;then frankly ...we should pay at the pump the value of the product . It's not like we are paying the highest prices at the pump in the world even though currently we consume the most.

excon rated this answer Excellent or Above Average Answer
quixotic_Choux rated this answer Average Answer

Question/Answer
quixotic_Choux asked on 05/06/06 - Iraq War - Foreign Policy Disaster

".....But the war did not go well for long. Though Secretary of State Condoleezza Rice now seems to have replaced Secretary of Defense Donald Rumsfeld as chief cheerleader for the war, many now see the invasion of Iraq as an historic disaster on par with Napoleon's invasion of Moscow in 1812 and the Athenian invasion of Sicily in 415 BCE. Both ended empires that had seemed invincible. Tom Ricks reported in The Washington Post April 30, that military leaders are considering whether, "the surest -- and perhaps now the only -- way to bring stability to Iraq is to divide the country into three pieces." The alternative view is not to press on to victory, Ricks says, but to withdraw and allow a civil war to settle the question.

America has lost more than a division's worth of brave soldiers to the war, with over 2,400 killed and 17,500 maimed. Our national debt increases by over $2 billion every week to pay for the war. America's international reputation is at its lowest point in history. Even our closest allies mistrust our motives, question our vision and are saddened by our abandonment of shared values. It is not that they resent American leadership; they just do not want this kind of leadership.

The danger of nuclear terrorism has also grown as the ideology of al Qaeda has spread like wildfire throughout the Muslim world. But our programs to secure and eliminate the highly-enriched uranium and plutonium scattered in stockpiles in dozens of countries have not kept pace. If Osama bin Laden can get his hands on these materials, his group can almost certainly build, deliver and detonate a bomb that can destroy any American city. Without this material he is powerless to do so. Yet we spend only $1 billion on year these programs. We spend this much every 4 days in Iraq.


Perhaps the most disheartening is that our senior government officials have not acknowledged these failures or given the slightest indication that they are working on correctives. On the contrary, the 2006 National Security Strategy of the United States repeats the emphasis on preemptive war, this time focusing on Iran rather than Iraq. As faux news anchor Stephen Colbert said in his mocking tribute to President Bush at the White House Correspondents dinner, "When the president decides something on Monday, he still believes it on Wednesday - no matter what happened Tuesday."

The administration is strategically exhausted. Its only solution to the problem of Iran is to repeat the Iraq playbook. The speeches, the refusal to negotiate directly with Iran, the unnerving presence of Iranian exiles whispering sweet promises in Washington, the framing of the issue as one of the "credibility of the Security Council" are all straight out of the campaign that successfully fooled a majority of the nation, convincing them that Iraq was an urgent threat and somehow linked to September 11.

Thus, it falls to those of us outside of the governing circles to detail the failures, to forge new strategies and champion a new course. Some are already doing just that; more are needed. Most importantly, we must expose fully the mistakes of this strategy and of those who developed it so that America does not lurch into an unnecessary war. Not again".....Shortened from an article by Joe Cinicincione.

~~~~~~~~~~~~~~~~~~~~~~~~~~~~~~~~~~~~~~~~~~~~~~~~~~


Blunder after horrific blunder, and now today, his hand picked replacement to head the CIA, Goss, had to "resign" hastily leaving a seriously damaged agency...after only 19 months.

tomder55 answered on 05/06/06:

Joseph Cirincione ,proliferation "expert",good friend of Sean Penn ? That Joseph Cirincione ? Of Course an August 2002 report from his very own hand at the Carnegie Endowment for International Peace said Iraq "almost certainly does have large numbers of chemical weapons and some biological weapons." The same Cirincione who argued against invasion because Saddam might use his WMD ?

"One could argue further that it would be a failure if a military action prompted an otherwise contained Saddam Hussein to unleash chemical or biological weapons against Israel and/or U.S. forces, prompting Israel and/or the United States to use nuclear weapons in response."

and of course he was wrong even in this statement as Saddam was hardly contained ...as the Oil for Food scandal continues to reveal. Anyway ;I'll be kind .I seriously disagree with most of the satements of this expert who's non-proliferation efforts have been a series of coddlings of dictators be it Saddam ,Kim Jong Il ,or now the mullahs in Iran.


As for Potter Goss' resignation ,yes I am concerned about it . But thre is only speculation so far as to the reason.

The 3 biggies so far are
- it was a conflict with Negroponte

-he was worn down by CIA rebels

-he was connected to the Cunningham scandal or knew someone on his staff who was [Watergate-gate...Hookergate ??]

I tend to believe that it had something to do with Negroponte and the DNI but not what the speculation is .

In a speech in San Antonio last week, Negroponte's top deputy, Michael Hayden, declared that an office largely under Negroponte's control the National Counterterrorism Center, or NCTC was now in charge of dictating the role other agencies will play in terror analysis. ... In the speech, Hayden also said Negroponte's office would be in charge of "liaison" relationships with foreign intelligence services long the treasured turf of the CIA which have historically produced much of the most important intelligence, according to a former senior CIA official. ... CIA supporters are upset about what they see as the neutering of an agency that helped win the Cold War and worry that it will undermine its human spy responsibilities, of which the CIA is still in charge. "It's a huge thing going on. It's a huge drama and nobody's picking up on it," the former CIA official said of the DNI's realignment of CIA responsibilities. "CIA feels quite friendless right now. We're seeing more pieces of it just keep being moved to the door."

well booo-hooo ! Maybe if the CIA had prevented 9-11 ,maybe they would not feel so friendless. Do they not think that maybe if they had been a little more effective that the Defense Dept. would not be looking to strengthen their own intel. capability ? If the CIA got gutted or even dismantled I do not think we would be worse off.

Potter Goss was never considered to be a long term solution .In the short time that Goss was there he had a big impact .No matter how the media spins this as disarray ,Goss accomplished what he set out to do. Porter Goss was obviously ready to retire but was talked into going to the CIA and taking care of business. He chased the deadwood and some of the rogue elements out of Dodge . He nabbed a leaker (and probably numerous others), and he allowed Congress to diminish the CIAs leadership through integration with the DNI.

The comparisons of Bush and Truman just keep on coming .Harry Truman when his approval ratings were in the 20% range was creating the CIA, NSA, and Joint Chiefs of Staff while the left was complaining that he was building a police state and body bags were flowing back from Korea. Trumans work won the cold war decades after he was gone. Bush is basically re-organizing and modernizing the infrastructure that Truman built to reflect the modern reality .And like Truman the benefits will be realized after his term is finished.





excon rated this answer Excellent or Above Average Answer
quixotic_Choux rated this answer Poor or Incomplete Answer

Question/Answer
ETWolverine asked on 05/05/06 - Aztlan

With all that we have been hearing about illegal immigrants demanding their rights, the mainstream meadia has been largely ignoring the "Aztlan" crowd... the Mexican-rights groups that demand the return of Mexican lands to Mexico, lands such as Texas, California, Nevada, and Utah, and parts of Arizona, New Mexico, Colorado, and Wyoming. They too have had a very large presence in the various demonstrations, with slogans like "Mexico to the Mexicans" and "Get off Our Land", and other stuff.

I suggest that we put this out to the Aztlan crowd: we'll give back all the lands ceeded to the US by the Treaty of Guadalupe Hidalgo in exactly the same condition that it was when we took it. We'll take every structure, every piece of machinery, every company, every hospital, every school, every job, with us when we leave. We will leave you with exactly what you had when you lost the war... an empty desert.

What say you? How excited are you to get a barren wasteland that you will have to build from scratch in order to get the least little income from? When you take that empty piece of land that is more burden than asset, will you then recognize that it was American ingenuity and sweat and determination that built everything that has ever existed there since 1848?

Be careful what you wish for, you might just get it.

I am sick of people demanding that America give up what it has worked for and earned to those who have not. American know-how built this country, made it the richest and most free country in the world for those who obey its laws. That know-how and determination has made many other countries richer as well, and provided goods and services thoughout the world. I am sick of people who have benefited from our generosity and ability putting us down as "imperialist pigs" or "usurpers" or any other thing.

Sorry for the rant. I just needed to vent.

Comments are appreciated.

Elliot

tomder55 answered on 05/05/06:

Samuel P. Huntington, chairman of Harvard University's Academy for International and Area Studies, warned in "The Hispanic Challenge," :


Demographically, socially, and culturally, the reconquista (re-conquest) of the Southwest United States by Mexican immigrants is well underway. No other immigrant group in U.S. history has asserted or could assert a historical claim to U.S. territory. Mexicans and Mexican Americans can and do make that claim ...


Yes ;notice how the reconquistadors get little press . How prominent are they ? Well at least in California members of MEChA are in the highest levels of government (Cruz Bustamante;Antonio Villaraigosa;Gil Cedillo).MEChA's national constitution calls for the "liberation of our land."


The Mexicans were on the losing side of the conflict and did not have the best negotiating position at the time . Still the United States shelled out $15 million in exchange for the land (1.36 million km ) agreed to take over $3.25 million in debts Mexico owed to American citizens. Not alot of money by today's standards but it is the same amount paid for the Louisiana territory purchase(2,144,476 km2) for less land .So The Treaty of Guadalupe Hidalgo WAS NOT A LAND GRAB ! .Another comparison ; we purchased Alaska(1,600,000 km) from the Russians for $7.2 million dollars ;which is the equivalent to $1.67 billion in 2006 dollars . Do the math . What we have here is fair compensation to Mexico for the purchase of undeveloped land. Yes ,it was part of a peace treaty to a war that Mexico was the loser in ,but it cannot be considered conqurered territory either .

I suggest that we put this out to the Aztlan crowd: we'll give back all the lands ceeded to the US by the Treaty of Guadalupe Hidalgo in exactly the same condition that it was when we took it. We'll take every structure, every piece of machinery, every company, every hospital, every school, every job, with us when we leave. We will leave you with exactly what you had when you lost the war... an empty desert.

Indeed ! I also propose that solution to Bolivia's Evo Morales . There is no way I would cede a company I owned in a foreign country to the country without blowing it up first .


ETWolverine rated this answer Excellent or Above Average Answer
Itsdb rated this answer Excellent or Above Average Answer

Question/Answer
Erewhon asked on 05/04/06 - Yeah for sensible Laura Bush! She is more tolerant than her husband. Why is that?

Bush's Spanish "not that good"

WASHINGTON (Reuters) - The White House on Thursday disputed an account of President George W. Bush singing the U.S. national anthem in Spanish during the 2000 presidential campaign, saying his Spanish is not that good.

Critics have accused Bush of hypocrisy for opposing a Spanish language version of the anthem.

They pointed to a book called "American Dynasty" by Kevin Phillips, who wrote that Bush "would drop in at Hispanic festivals and parties, sometimes joining in singing 'The Star-Spangled Banner' in Spanish."

White House spokesman Scott McClellan said the assertion did not ring true to him because, "The president speaks Spanish, but not that well."

"I'm saying that not only was that suggestion absurd, but that he couldn't possibly sing the national anthem in Spanish. He's not that good with his Spanish," McClellan said.

Bush, a former governor of Texas, sprinkles his speeches with Spanish phrases, as he did during both his presidential campaigns, to show kinship with Hispanics.

But last week, he said he thought the national anthem should be sung in English, after the "Star-Spangled Banner," or "Nuestro Himno," made its debut with a new Latin beat and Spanish lyrics.

Bush's wife, Laura, appeared to disagree.

"I don't think there's anything wrong with singing it in Spanish," said told CNN in an interview on Wednesday.

She said she thought it should be sung in English, but pointed out that,

"We are a nation of immigrants. We are a nation of many, many languages, because immigrants come and bring their languages."



===

Why do you think Laura is more tolerant that george? Could it be because she is a womand and therefore more nurturing than her right wing conservative husband who has to placate those further to the right of him?


Or is she just plain stupid?

tomder55 answered on 05/04/06:

Condi Rice doesn't think it's a big deal either . With all due respect to both ;I disagree. I think not so much the language as the wording in "Nuestro Himno" makes it the 'Star Mangled Banner '.A remix to be released in June will contain several lines that condemn U.S. immigration laws. "These kids have no parents, cause all of these mean laws ... let's not start a war with all these hard workers, they can't help where they were born."


Erewhon rated this answer Excellent or Above Average Answer
Itsdb rated this answer Excellent or Above Average Answer
kindj rated this answer Excellent or Above Average Answer

Question/Answer
ETWolverine asked on 05/04/06 - Random thoughts...

Who was the first person to look at a cow and say, "I think I'll squeeze these dangly things here and drink what comes out"?

Who was the first person to say, "See that chicken over there ... I'm gonna eat the first thing that comes out if its butt"?

Isn't Disney World just a people trap operated by a mouse?

If electricity comes from electrons, does morality come from morons?

Do illiterate people get the full effect of Alphabet soup?

Can you get cornered in a round room?

Why do we wash behind our ears? Who really looks there?

Why don't the hairs on your arms get split ends?

If an atheist has to go to court, do they make him swear on the Bible?

Why is it illegal to park in a handicapped parking space but its ok to use a handicapped toilet?

In that song, she'll be coming around the mountain, who is she?

How come we say 'It's colder than hell outside' when isn't it realistically always colder than hell since hell is supposed to be fire and brimstone?

Why is it that if something says, "do not eat" on the packaging it becomes extra tempting to eat?

Why are people so scared of mice, yet we all love Mickey Mouse?

Wouldn't it be smart to make the sticky stuff on envelopes taste like chocolate?

Elliot

tomder55 answered on 05/04/06:

I can answer who "she " is ..sorta .

The song was orginally a spiritual with the title 'When the Chariot Comes' .


O, who will drive the chariot When she comes? O, who will drive the chariot When she comes? O, who will drive the chariot, O, who will drive the chariot, O, who will drive the chariot When she comes?

King Jesus, he'll be driver when she comes, When she comes . . . .

She'll be loaded with bright Angels When she comes . . . .

She will neither rock nor totter, When she comes . . . .

She will run so level and steady, When she comes . . . .

She will take us to the portals, When she comes . . . .


The words were changed to the more familiar verses in the Appalachians .

Anyway the "she in the song is the chariot .

For the more familiar version ,others have suggested that the "she " is a union organizer Mary Harris Jones .;the famous 'Mother Jones' who was going to come around the mountain to do some union organizing ....and still more modern bawdy versions of the song say that "she " is a lover .

ETWolverine rated this answer Excellent or Above Average Answer
Itsdb rated this answer Excellent or Above Average Answer

Question/Answer
Coup_de_Grace asked on 05/03/06 - Truth about Kennedy Assassination

I understand after a conversation with a co-worker that next week there will be a television documentary about the Kennedy assassination with new information that has been covered up since the very beginning, including information withheld from the Warren Report.

Does anyone on this board know when and what time this program will air(need time zone to convert, too)?

Do you feel that there was more to the assassination than what was explained to the American people?



Susan

Benjamin and Susan Grace

tomder55 answered on 05/04/06:

I used to think that there was some grand conspiracy ,and it probalby would not take much to convince me again because I still don't understand why Jack Ruby offed Oswald ;but I watched an ABC documentary that was pretty convincing .They opined that Oswald acted alone .They used modern ballistics experts using full-scale models of the cars, dummies, lasers and computers in the blocked-off Dalley Plaza . There was nothing odd, nothing unusual about the pattern of Oswald's bullets;including the so -called magic bullet . They also determined that any theoretical shots fired from the grassy knoll, the overpass, or any of the other popular conspiracy theory locations (my favorite is the storm drain )could not possibly have hit the President the way he was hit.

ETWolverine rated this answer Excellent or Above Average Answer
Coup_de_Grace rated this answer Excellent or Above Average Answer

Question/Answer
quixotic_Choux asked on 05/03/06 - Legalizing Drugs for Personal Use

"MEXICO CITY (AP) - Mexican President Vicente Fox will sign into law a measure that decriminalizes the possession of small amounts of marijuana, cocaine, heroin and other drugs for personal use, his spokesman said Tuesday.

Spokesman Ruben Aguilar defended the law, which was approved Friday by Mexico's Senate, despite criticism in the United States that it could increase casual drug use.

``The president is going to sign this law,'' said Aguilar, who called the legislation ``a better tool ... that allows better action and better coordination in the fight against drug dealing.''

``The government believes that this law represents progress, because it established the minimum quantities that a citizen can carry for personal use,'' Aguilar said.

~~~~~~~~~~~~~~~~~~~~~~~~~~~~~~~~~~~~~~~~~~~~~~~~~~




Any effect on the USA, do you think?

tomder55 answered on 05/03/06:

maybe there will be a flow of illegal immigrants heading south ?


For the failed state of Mexico ,this is the contemporary equivalent of 'bread and circuses' .During the reign of Emperor Claudius there was about a one to one ratio of work days and holidays where public funded games were played for the amusement of the masses .The games formed a barrier for autocracy against revolution. The games occupied the time of these people, provided a safety valve for their passions, distorted their instincts and diverted their activity.

The gvt. of Mexico appears incapable of setting up a situation in the country where commerce thrives . The country is loaded in resources and has the population that for all appearances seems more than willing to work hard and provide for their families. But the gvt is as corrupt as the day is long . If they did not have the safety valve of the US border ,and now apparently legal drugs ,then perhaps the people would wake up and realize that they have a suitable working constitution and all that is lacking in their country is responsible leadership.

excon rated this answer Excellent or Above Average Answer
Itsdb rated this answer Excellent or Above Average Answer
quixotic_Choux rated this answer Excellent or Above Average Answer

Question/Answer
Itsdb asked on 05/02/06 - Worth the read...

White Guilt and the Western Past
Why is America so delicate with the enemy?

BY SHELBY STEELE
Tuesday, May 2, 2006 12:01 a.m. EDT

There is something rather odd in the way America has come to fight its wars since World War II.

For one thing, it is now unimaginable that we would use anything approaching the full measure of our military power (the nuclear option aside) in the wars we fight. And this seems only reasonable given the relative weakness of our Third World enemies in Vietnam and in the Middle East. But the fact is that we lost in Vietnam, and today, despite our vast power, we are only slogging along--if admirably--in Iraq against a hit-and-run insurgency that cannot stop us even as we seem unable to stop it. Yet no one--including, very likely, the insurgents themselves--believes that America lacks the raw power to defeat this insurgency if it wants to. So clearly it is America that determines the scale of this war. It is America, in fact, that fights so as to make a little room for an insurgency.

Certainly since Vietnam, America has increasingly practiced a policy of minimalism and restraint in war. And now this unacknowledged policy, which always makes a space for the enemy, has us in another long and rather passionless war against a weak enemy.

Why this new minimalism in war?

It began, I believe, in a late-20th-century event that transformed the world more profoundly than the collapse of communism: the world-wide collapse of white supremacy as a source of moral authority, political legitimacy and even sovereignty. This idea had organized the entire world, divided up its resources, imposed the nation-state system across the globe, and delivered the majority of the world's population into servitude and oppression. After World War II, revolutions across the globe, from India to Algeria and from Indonesia to the American civil rights revolution, defeated the authority inherent in white supremacy, if not the idea itself. And this defeat exacted a price: the West was left stigmatized by its sins. Today, the white West--like Germany after the Nazi defeat--lives in a kind of secular penitence in which the slightest echo of past sins brings down withering condemnation. There is now a cloud over white skin where there once was unquestioned authority.

I call this white guilt not because it is a guilt of conscience but because people stigmatized with moral crimes--here racism and imperialism--lack moral authority and so act guiltily whether they feel guilt or not.

They struggle, above all else, to dissociate themselves from the past sins they are stigmatized with. When they behave in ways that invoke the memory of those sins, they must labor to prove that they have not relapsed into their group's former sinfulness. So when America--the greatest embodiment of Western power--goes to war in Third World Iraq, it must also labor to dissociate that action from the great Western sin of imperialism. Thus, in Iraq we are in two wars, one against an insurgency and another against the past--two fronts, two victories to win, one military, the other a victory of dissociation.

The collapse of white supremacy--and the resulting white guilt--introduced a new mechanism of power into the world: stigmatization with the evil of the Western past. And this stigmatization is power because it affects the terms of legitimacy for Western nations and for their actions in the world. In Iraq, America is fighting as much for the legitimacy of its war effort as for victory in war. In fact, legitimacy may be the more important goal. If a military victory makes us look like an imperialist nation bent on occupying and raping the resources of a poor brown nation, then victory would mean less because it would have no legitimacy. Europe would scorn. Conversely, if America suffered a military loss in Iraq but in so doing dispelled the imperialist stigma, the loss would be seen as a necessary sacrifice made to restore our nation's legitimacy. Europe's halls of internationalism would suddenly open to us.

Because dissociation from the racist and imperialist stigma is so tied to legitimacy in this age of white guilt, America's act of going to war can have legitimacy only if it seems to be an act of social work--something that uplifts and transforms the poor brown nation (thus dissociating us from the white exploitations of old). So our war effort in Iraq is shrouded in a new language of social work in which democracy is cast as an instrument of social transformation bringing new institutions, new relations between men and women, new ideas of individual autonomy, new and more open forms of education, new ways of overcoming poverty--war as the Great Society.

This does not mean that President Bush is insincere in his desire to bring democracy to Iraq, nor is it to say that democracy won't ultimately be socially transformative in Iraq. It's just that today the United States cannot go to war in the Third World simply to defeat a dangerous enemy.

White guilt makes our Third World enemies into colored victims, people whose problems--even the tyrannies they live under--were created by the historical disruptions and injustices of the white West. We must "understand" and pity our enemy even as we fight him. And, though Islamic extremism is one of the most pernicious forms of evil opportunism that has ever existed, we have felt compelled to fight it with an almost managerial minimalism that shows us to be beyond the passions of war--and thus well dissociated from the avariciousness of the white supremacist past.

Anti-Americanism, whether in Europe or on the American left, works by the mechanism of white guilt. It stigmatizes America with all the imperialistic and racist ugliness of the white Western past so that America becomes a kind of straw man, a construct of Western sin. (The Abu Ghraib and Guantanamo prisons were the focus of such stigmatization campaigns.) Once the stigma is in place, one need only be anti-American in order to be "good," in order to have an automatic moral legitimacy and power in relation to America. (People as seemingly disparate as President Jacques Chirac and the Rev. Al Sharpton are devoted pursuers of the moral high ground to be had in anti-Americanism.) This formula is the most dependable source of power for today's international left. Virtue and power by mere anti-Americanism. And it is all the more appealing since, unlike real virtues, it requires no sacrifice or effort--only outrage at every slight echo of the imperialist past.

Today words like "power" and "victory" are so stigmatized with Western sin that, in many quarters, it is politically incorrect even to utter them. For the West, "might" can never be right. And victory, when won by the West against a Third World enemy, is always oppression. But, in reality, military victory is also the victory of one idea and the defeat of another. Only American victory in Iraq defeats the idea of Islamic extremism. But in today's atmosphere of Western contrition, it is impolitic to say so.

America and the broader West are now going through a rather tender era, a time when Western societies have very little defense against the moral accusations that come from their own left wings and from those vast stretches of nonwhite humanity that were once so disregarded.

Europeans are utterly confounded by the swelling Muslim populations in their midst. America has run from its own mounting immigration problem for decades, and even today, after finally taking up the issue, our government seems entirely flummoxed. White guilt is a vacuum of moral authority visited on the present by the shames of the past. In the abstract it seems a slight thing, almost irrelevant, an unconvincing proposition. Yet a society as enormously powerful as America lacks the authority to ask its most brilliant, wealthy and superbly educated minority students to compete freely for college admission with poor whites who lack all these things. Just can't do it.

Whether the problem is race relations, education, immigration or war, white guilt imposes so much minimalism and restraint that our worst problems tend to linger and deepen. Our leaders work within a double bind. If they do what is truly necessary to solve a problem--win a war, fix immigration--they lose legitimacy.

To maintain their legitimacy, they practice the minimalism that makes problems linger. What but minimalism is left when you are running from stigmatization as a "unilateralist cowboy"? And where is the will to truly regulate the southern border when those who ask for this are slimed as bigots? This is how white guilt defines what is possible in America. You go at a problem until you meet stigmatization, then you retreat into minimalism.

Possibly white guilt's worst effect is that it does not permit whites--and nonwhites--to appreciate something extraordinary: the fact that whites in America, and even elsewhere in the West, have achieved a truly remarkable moral transformation. One is forbidden to speak thus, but it is simply true. There are no serious advocates of white supremacy in America today, because whites see this idea as morally repugnant. If there is still the odd white bigot out there surviving past his time, there are millions of whites who only feel goodwill toward minorities.

This is a fact that must be integrated into our public life--absorbed as new history--so that America can once again feel the moral authority to seriously tackle its most profound problems. Then, if we decide to go to war, it can be with enough ferocity to win.

Mr. Steele, a research fellow at the Hoover Institution at Stanford University, is author, most recently, of "White Guilt: How Blacks and Whites Together Destroyed the Promise of the Civil Rights Era," published this week by HarperCollins.

~~~~~~~~~~~~~~~~~~~~~~~~~~~~~~~~~~~~~~~~~~~~~~~~~~~~~~

Comments?

tomder55 answered on 05/03/06:

white guilt or western guilt...white self loathing or western self loathing ? The Ruskies nearly leveled Grozny and reduced much of the rest of Chechnya to ashes, killing tens of thousands of civilians in the mid-90s .Do you recall the press making it a headline event ? Nope . Instead we were fixated with creative humidores .Then again ;the Ruskies are still dealing with a low level Chechen insurgency so maybe they did not go far enough .

After the early part of the 20th Century I think Western society became appalled at the prospect of total war with of course the Europeans have taken it to the extreme . But even in the US there is a large population that does not have the stomach for the fight at hand.

I do not think it a matter of guilt as much as intimidation. The sins of the whites are no worse than the sins of other races .We just love the self flagulation .

Itsdb rated this answer Excellent or Above Average Answer

Question/Answer
quixotic_Choux asked on 05/01/06 - "Immigration Demostration Day"

I'm curious, why isn't the Bush right wing spin machine out there today attacking the illegal aliens????

I think I know the answer, what say you???

tomder55 answered on 05/02/06:

I boycotted taco bell yesterday . I think the more they demonstrate the more hardened the American people's position will be .Clearly no meaningful solution to the problem can be addressed without dealing with border security first . After that I am willing to negotiate a reasonable solution to the problem . It is doable ;but not with open borders .

ETWolverine rated this answer Excellent or Above Average Answer
quixotic_Choux rated this answer Excellent or Above Average Answer

Question/Answer
quixotic_Choux asked on 05/01/06 - Marbury vs. Madison

"Marbury v. Madison, 5 U.S. (1 Cranch) 137 (1803)[1], is a landmark case in United States law, the basis for the exercise of judicial review of Federal statutes by the United States Supreme Court as a constitutional power.

The Court ruled that it had the power to declare void a statute which it considered repugnant to the United States Constitution. Chief Justice John Marshall presided over the case, and used the case to legally establish the right of the judiciaryand in particular, the Supreme Court to determine the constitutionality of the actions of coequal branches of government and thus laid the basis for the current power of the Supreme Court." Cut and Paste from Wikipedia

~~~~~~~~~~~~~~~~~~~~~~~~~~~~~~~~~~~~~~~~~~~~~~~~~~

Finally, something memorized in American History class becomes relevant. (just kidding)

See previous question posted by jack about Bush stating he didn't have to obey laws *he didn't agree with*, thus usurping the power of the judiciary as delineated in Marbury vs Madison and marginalizing Congressional powers---giving the Executive Branch unprecidented and unconstitutional powers.

Shall we just let Bush toss Marbury vs Madison into the dustbin of history along with the Constitution??I thought right wingers loved strict interpretation of the Constitution. Where's the outrage???

tomder55 answered on 05/02/06:

There are many who make the sound arguement that Marbury was a coup . Congress constitutionally has a set of rigid defined powers while the founders left the Executive and Judicial not so defined . Those are left open while Congress must point to specific authority .


Without getting into the specifics ;Justice Marshall knew that his branch was weak ;but used the ruling to invent powers to the judiciary that they did not have defined Constitutionally .The irony of it is that if you read the court decision it is very narrow in scope.It ONLY declared unconstitutional a power which Congress sought to grant to the Court itself, and it was found to conflict with Article III of the Constitution, which governs the Federal courts. It was NOT considered the precident that it is today .No Federal law was struck down after it until the Dred Scott decision in 1857 .(the decision was made in 1803 )

Jefferson objected: "To consider the judges as the ultimate arbiters of all constitutional questions [is] a very dangerous doctrine indeed and one which would place us under the despotism of an oligarchy,"

Clearly the court WAS NOT intended to be the final arbiter of what is constitutional .(as I said to my bar tender at the speakeasy the other day ...); At the end of the day ;it is the American people who decide what is and is not Constitutional .The court was not explicitly given this right to the ultimate determination about the Constitutionality of Laws by the Constitution. It was a right claimed by the Supreme Court in Marbury vs Madison. It has been left in place because neither the excutive branch and the legislative branch have challenged that right.

Itsdb rated this answer Excellent or Above Average Answer
quixotic_Choux rated this answer Excellent or Above Average Answer

Question/Answer
quixotic_Choux asked on 05/01/06 - "Mission Accomplished"

Today is the third anniversary of Bush's embarasssing speech before the banner MISSION ACCOMPLISHED.

What was he thinking; how dumb can you get? Is he now waiting for the rapture?

Our soldiers have been dying in Iraq longer than we were in Korea. There is no end in sight.


Well, was invading Iraq worth it now that we see that Iran is the greater threat, BY FAR???

tomder55 answered on 05/02/06:

I can answer what he was thinking and yes in hindsight the stunt was a blunder and he has had to live with the egg on his face but there was a real reason why it was done .

General Franks who was in charge of the military phase of the operation ;a phase that accomplised a three-week capture of Baghdad (Phase III); advised the President to announce that major combat operations were completed . Franks wrote in his book that he was led to believe that there would be an infusion of international troops for Phase IV (the infamous CPA led first by Jay Garner and later by Viceroy Bremer with it's $18.6 billion reconstruction money that Congress appropriated remained unspent). Of course the international help was not forthcoming .

As for invading Iraq ;Iran ..whatever . I have a feeling that many people today if we were living during WWII would be asking why we invaded Italy first before Germany . After all ;we were not attacked by Italy were we ?

Itsdb rated this answer Excellent or Above Average Answer
purplewings rated this answer Excellent or Above Average Answer
quixotic_Choux rated this answer Excellent or Above Average Answer

Question/Answer
paraclete asked on 05/02/06 - Look who's irrelevant now!

Never put your head in a paper tiger's mouth


May 2, 2006

History has taught us that revolutionaries should be taken seriously, writes Gerard Henderson.

SOME Englishmen still go out in the midday sun. Others just do lunch. It seems that the English-born journalist and author Robert Fisk had elected to take the second option when he was interviewed live from Beirut on ABC TV's Lateline last Wednesday. At the conclusion of the program, presenter Tony Jones described the interview as "rather unusual". You can say that again.

Fisk's considered position on the West and the Middle East is set out at length in a 1300-page book, The Great War for Civilisation: The Conquest of the Middle East (Fourth Estate, 2005). Yet those who saw Fisk's 11-minute interview with Jones may have got a clearer sense of his essential thesis compared with those who have waded through his recent tome. It was as if the commentator was somewhat relaxed on the job (so to speak) and, consequently, spoke with greater candour than would usually be expected from The Independent's man in Beirut.

Fisk began by saying that the likes of the al-Qaeda leader, Osama bin Laden, and the al-Qaeda in Iraq supremo, Abu Musab al-Zarqawi, "don't actually matter". According to his view, they are "part of the bestialisation of those people we want to hate". He said "the organisation which bin Laden has created exists, so the individuals per se don't really matter much anymore".

One of the lessons of history is that revolutionaries should be taken seriously, since they usually do, or attempt to do, what they say they intend to do. This is true of Vladimir Lenin and the Bolsheviks, Adolf Hitler and the Nazis and more besides.

Bin Laden said he would attack the West, and has done so on many occasions. Zarqawi said he would wage an insurgency against the UN-approved government in Iraq and its Western allies (including the US, Britain and Australia). At present, Zarqawi and his fellow terrorists are waging war on soldiers and civilians in Iraq.

Yet, Fisk maintains, neither man actually matters anymore. Why? Well, according to this view, bin Laden and Zarqawi are essentially creations of, wait for it, the West. They are mere figures "to be hated and to be bestialised in front of the television screens" and depicted by Western TV producers as "the latest mad lunatic, the latest fanatic, the latest terrorist whom we have to be concerned about". It was as if bin Laden and Zarqawi had not chosen to send out their own video and audio revolutionary messages with the expressed intention they be reported on Western TV. But they have.

When Jones implied that Fisk might be underestimating the significance of Zarqawi's most recent revolutionary message, Fisk responded by positing the suggestion that "these people [are] being put out before us as caricatures to hate" by George Bush. So it's all Bush's fault, apparently. When Jones suggested that bin Laden was a problem, Fisk responded: "It's a problem for you, isn't it?" The response was in the affirmative. Whereupon Fisk quickly backtracked and acknowledged that it was a problem for him as well.

A similar confusion emerged when Fisk maintained the West is "helping to create the creatures of evil". Fisk gesticulated inverted commas when saying the word "evil" - implying that he did not necessarily believe the murderer Zarqawi was necessarily evil. Zarqawi's Iraqi civilian victims, and their families, would hold a different view, no doubt. When Jones argued that the West just could not ignore al-Qaeda in Iraq, Fisk again backtracked, declaring: "No; absolutely not; you're right."

There followed a Fisk "look, look, look, look" interjection and it was soon good night from Jones. And it was still lunch time in Lebanon. The acclaimed author of The Great War for Civilisation signed off by implying that the "injustice in the Muslim world" is due primarily to "Westerners".

In his recent book, Fisk describes war as "the total failure of the human spirit" and comments that he knows of an editor "who has wearied of hearing" him say this. The author then asks: "How many editors have first-hand experience of war?" Fair enough. But how many journalists have first-hand experience of government? For example, the fact is Britain and its allies had few options in 1939. It's difficult to see how a determination to stand up to Hitler can be equated with a total failure of the human spirit.

Fisk's position is much the same about World War I, in which his father took part. He argues that William Fisk fought "in the trenches of France because of a shot fired in a city he'd never heard of called Sarajevo". This is simplistic at best. William Fisk found himself in a trench on the Western Front because Germany invaded Belgium and France. The elected politicians of the day had to make a decision whether to resist German aggression or not. Journalists are not required to make such decisions.

Fisk stands in the tradition of the alienated Western intellectual. He has many fellow-travellers. The playwright Harold Pinter used the occasion of his 2005 Nobel Prize for literature address to question whether the West ever has any "moral sensibility". Pinter's discomfort with the US and Britain is so intense that he went so far as to support Slobodan Milosevic in Serbia.

Then there is John Pilger, who told the Green Left Weekly (November 3, 2004) that "while we abhor and condemn the continuing loss of innocent life in Iraq, we have no choice but to support the resistance".

There is a plausible case against the US/Britain/Australia policy in the Middle East in general - and Iraq in particular. But blaming the West for virtually all the problems and injustices in the Middle East is a cop-out. Especially since Arabs, Muslim and Christian alike are the principal victims of the attacks by radical Islamists. In ignoring this in his rather unusual, albeit brutally honest Lateline interview, Fisk demonstrated that he was, well, out to lunch.

Gerard Henderson is executive director of the Sydney Institute.

tomder55 answered on 05/02/06:

and Hitler was a figment of our imagination also ...... yes ...that's right we created him and we have created Ahmadinejad. Zarqawi not evil ??? Tell that to Nick Berg.

That's ok ;there are enough moonbats out there that will digest his written pablum for him to make the NY Slimes best seller's list .

Itsdb rated this answer Excellent or Above Average Answer
paraclete rated this answer Excellent or Above Average Answer

Question/Answer
Erewhon asked on 05/01/06 - The Union of Scotland with England ...

Union of Scotland with England
BRENDAN O'BRIEN

THE ACT of Union marrying Scotland and England, providing for one parliament to administer the two nations, was passed in January 1707 and came into legal effect in May of that year.
However, the two nations' courtship was anything but smooth.

For centuries English kings failed to unite the two countries by conquest from Edward I through Henry VIII's 'rough wooing' of Scotland in 1542. James VI of Scotland and the I of England in 1603 also failed to unite the countries under the crown. His son, Charles I, faired no better before his execution, but attempts at a union continued under Cromwell during the Interregnum.

Both countries had very different motivations for union before 1707. The English wanted to secure a Protestant monarch and passed the Act of Settlement in 1700 to that effect. But the Scots jeopardised the English succession by passing an Act of Security making it their business to choose who would be Scottish monarch. The English also wanted to end the Auld Alliance between Scotland and France that curtailed many of their imperial ambitions.
Resources

Parcel o' Rogues

For the Scots, the issues were financial. The economy had been bankrupted by the failed Darien expedition of 1698 when almost the entire country had invested in a scheme to secure a colony on the Panamanian peninsula controlling trade between the Atlantic and Pacific. A trade war between the two countries followed.

Provisions in the 1707 Act established a trade, customs and political union. The Scots secured extremely favourable conditions on tax (the Scots would raise only 1/40th of revenue) at the expense of under-representation in the British parliament (a twelfth of seats in the Commons, and a handful in the Lords, for a country which was a sixth of the total population).

The Scottish people did not want a union and rioted but their nobles were more easily swayed. Many Scots nobles who partook in the negotiations were bribed, one according to some accounts for the shockingly small sum of 11, leading to Burns's famous depiction of them, summing up the popular view, as a Parcel o' Rogues.

Parcel o' Rogues

Fareweel tae all oor Scottish fame
Fareweel oor ancient glory
Fareweel even tae oor Scottish name
Sae famed in martial story
Now Sark runs tae the Solway sand
Tweed runs tae the ocean
To mark where England's province stand
Such a parcel of rogues in a nation

What force or guile could ne'er subdue
Through many warlike ages
Is wrought now by a coward few
For hireling traitor's wages
The English steel we could disdain
Secure in valour's station
But English gold has been oor bane
Such a parcel of rogues in a nation

Oh would that ere I saw the day
That treason thus should sell us
My auld grey heid was laid in clay
Wi' Bruce and loyal Wallace
But pith an' power tae my last hour
I'll mak' this declaration
We're bought and sold for English gold
Such a parcel of rogues in a nation


====

Isn't history fascinating?




tomder55 answered on 05/01/06:

certainly is. thanks for that .

btw . a fair comparable situation would be for Mexico to be incorporated into the United States as a state or territory . But current trends show that what is really happening is a Reconquista ,or to carve out Southwest United States and create Aztlan .

Erewhon rated this answer Excellent or Above Average Answer

Question/Answer
excon asked on 04/29/06 - Good News on the Drug Front


Hello:

Well, I feel right at home with you rightwingers today. Me, and you're dude, Limpfellow, have both felt the wrath of the DEA.

The difference is, I hate 'em. He loves 'em. So, how could anyone love an agency who's 5 year long witch hunt finally screwed him over big time? Frankly, I kinda like it. No, I hate the DEA, but when they bust a guy like the Limpone, I think it's all worth it.

Plus, Mexico is legalizing possession for small amounts of drugs including heroine and LSD. Put that in your pipe, DEA.

I think I'll illegally immigrate to Mexico.

excon

tomder55 answered on 05/01/06:

I don't have much to say on this . Rush is not my favorite radio personality ;partly because of the take he took on drug addiction .I know enough addicts of drugs legal and illegal ,as well as other social weaknesses to know that a strict law enforcement solution is not forthcoming or compassionate .But that is not really the issue .Perhaps he has tempered his opinions as a result of his addictions .

The DA blustered 2 years ago and denied Rush the settlement that he evidently gave Rush . Instead ;he boasted back then when he claimed that medical records "indicate evidence that would support in excess of 10 felony counts for violations." They also suggested that Rush was involved in money laundering . Evidently there was nothing there ;nor was Rush's violations as serious as the prosecutor let on .

Yes ,Rush was able to afford the best lawyers money can buy . But ;a prosecutor grandstanding is still a prosecutor grandstanding ,and makes me very suspicious if justice is in their agenda ;let alone their vocabulary . The question is not if Rush was able to tilt the scales in his favor with his money as much as if he became a target simply because he was Rush .

excon rated this answer Excellent or Above Average Answer
sissypants rated this answer Excellent or Above Average Answer

Question/Answer
paraclete asked on 05/01/06 - How's this for military incompetence - Rumsfeld should go !

Read this and weep all you who have an interest in seeing the conflict in Iraq over.

Not content with leaveing bin Laden at large the US military has done it again. Total incompetence all the way to the top, or is it diliberate?

http://www.theage.com.au/articles/2006/04/30/1146335608444.html?from=top5

tomder55 answered on 05/01/06:

sorry ,can't take seriously what Mike Scheuer says (aka " anonymous" author of Imperial Hubris: Why the West is Losing the War on Terror which he penned while employed at the CIA ,with his superior's approval .....The CIA is past the point of any usefulness .It should be disolved and intelligence ops should be conducted like they were before the creation of the CIA ...by the military ).

Scheuer as the so called' head of the agency's Osama bin Laden unit'is in fact an OBL fan . He wrote in the book (and my jaw dropped when Iread it ) :

"For nearly a decade now, bin Laden has demonstrated patience, brilliant planning, managerial expertise, sound strategic and tactical sense, admirable character traits , eloquence, and focused, limited war aims. He has never, to my knowledge, behaved or spoken in a way that could be described as "irrational in the extreme."

here's more :
"The strength of his personality and message is likely to lead to an enduring legacy that will long survice his own departure from the scene."

"Osama bin Laden appears to be a geniunely pious Muslim; a devoted family man; a talented, focused, and patient insurgent commander; a frank and eloquent speaker; a successful businessman; and an individual of conviction, intellectual honestly, compassion, humility, and physical bravery
.

Remember, this was written after several large-scale attacks on innocents and revised after the 9/11 attacks. Saying that OBL has some nice traits is like looking back at Hitler and saying,' Yes, but he was a fine painter and was kind to animals'.

Instead of hunting him down ,it looks like he was an OBL groupie. While he complains that Zarqawi was not offed , it is well known that we had multiple chances to get OBL that we passed up on . Where was Scheuer then ? Why was it that every time we targetted OBL ,the intel. was wrong ?

The guy is a moonbat . He goes on to draw parallels between Osama's first Declaration of war in 1996 and his second in 1998, a "Jihad against Jews and Crusaders," with the US Declaration of Independence.

But instead of being portrayed as the loon that he is ,he is on CBS network as some kind of terrorism expert commentator where he is free to spout anti-Israel conspircacy theories




He told Four Corners that during 2002, the Bush Administration received detailed intelligence about Zarqawi's training camp in Iraqi Kurdistan.

so much for the anti-war denial that Saddam had no relationship with al-Qaeda .

paraclete rated this answer Excellent or Above Average Answer

Question/Answer
excon asked on 04/30/06 - Gas - I've got some


Hello Righty's:

You say it's a supply and demand problem and then throw up your hands, as though there isn't anything to be done about it. I agree about the supply and demand problem, however I suggest that there are lots of things we can do. It was, after all, politics that affected the supply and demand problem in the first place.

For example, we got a scare in the 70's. It may have been the first time any of realized that oil is finite. We made a few political changes, but then the crisis was over and it was back to cheap gas. Again, we pretended that it wouldn't run out.

Well, it's gonna run out, and the closer it gets to that point, the more expensive it's gonna get. That's the REAL supply and demand problem we have.

Because we stuck our head in the sand in the past (what were we doing - looking for oil?), we allowed some two bit tyrants to get a lot power over us. They still have that power.

What can we do? Take our head out of the sand. Drill? Sure. Nuclear? Of course. Conserve? Makes sense to me. Ethanol? Yes. Hemp (NOT marijuana)could play a very bit roll (but out head is still in the sand). Are there other things we can do? I'll bet there is: solar, wind, geothermal, hydrogen, fusion....... ad infinitem....

Uhhh, leadership on this issue is critical. I see none.

excon

PS> I address the righty's because you're in charge. But the lefty's got nothing either.

tomder55 answered on 05/01/06:

I do not disagree with anything you said except that as of last year the US did take baby-steps towards a comprehensive plan. Is it enough ...No ;but it is the closest thing we have had that could be called leadership since Jimmy Carter on the issue .

A decade ago the US made the initial step towards ethanol conversion by giving credits to agri-businesses that made ethanol . They also slapped a defacto tarrif on ethanol imports . The US auto manufactures created engines capable of the conversion . And then the movement died .

Last year Congress instituted mandatory E-10 % summer blend . The recent spike is due to the fact that all the stations have to empty and scrub their tanks before the new blend can be filled . Then there is a shortage of domestic ethanol in spite of the tax credits ...and imported ethanol is artificially expensive .

Combine that with uncertainty over the oil supply due to Iran and the Islamic 'rebels ' in Nigeria who are disrupting about 25 % of the prefered light crude supply that comes from Nigeria and well ...it's a mess .

You know where I stand . I consider conversion to domestic supplies of energy a national security priority . We had 30 years we squandered but that is in the past . We are in a situation now where our short sightedness means that effectively we are funding both sides of WWIII .

excon rated this answer Excellent or Above Average Answer
Itsdb rated this answer Excellent or Above Average Answer

Question/Answer
fredg asked on 04/30/06 - Bush ploy for ratings?

Bush and a well known "look alike" appeared at a function, shown by CNN Headline news. The "look alike" was the same person from the Jay Leno show, who is very, very good at imitating Pres. Bush.
Is this just another ploy by the Bush Administration to have him appear as "just one of the little people"?
Looking for better Presidential ratings?
fredg

tomder55 answered on 05/01/06:

Geeze ;he can't even have some fun at a dinner without someone looking for sinister ulterior motives . lighten up Fred .

ETWolverine rated this answer Excellent or Above Average Answer
fredg rated this answer Excellent or Above Average Answer

Question/Answer
excon asked on 04/30/06 - Bomb, bomb, bomb - bomb, bomb Iran - to the tune of "Barbara Ann"


Hello asleepatheswitchers:

Heres what to do with Iran. Mad. It worked with the commies. Look, I dont think were gonna prevent them from getting a bomb. Frankly, Im not sure they dont already have one. What? You trust the intelligence??? No, I dont think they made one, but I sure think they coulda bought one.

Let em get it. Then we tell em that if you use it, well blow YOU off the map and all your Muslim neighbors too. The only problem with that, is that were dealing with religious fanatics who think that being blown off the map is a GOOD thing.

excon

PS> How come I always have the solutions to the worlds problems?

tomder55 answered on 05/01/06:

you have them recruting thousands of suicide -homicide bombers and you think that MAD would be a deterent ? I thought you understood the nature of the enemy better than that.

excon rated this answer Excellent or Above Average Answer

Question/Answer
jackreade asked on 05/01/06 - "Failed States"

Book Review cut and paste from Amazon dot com follows:

"Forget Iraq and Sudan--America is the foremost failed state, argues the latest polemic from America's most controversial Left intellectual. Chomsky (Imperial Ambitions) contends the U.S. government wallows in lawless military aggression (the Iraq war is merely the latest example); ignores public opinion on everything from global warming to social spending and foreign policy; and jeopardizes domestic security by under-funding homeland defense in favor of tax cuts for the rich and by provoking hatred and instability abroad that may lead to terrorist blowback or nuclear conflict. Ranging haphazardly from the Seminole War forward, Chomsky's jeremiad views American interventionism as a pageant of imperialist power-plays motivated by crass business interests. Disdaining euphemisms, he denounces American "terror" and "war crimes," castigates the public-bamboozling "government-media propaganda campaign" and floats comparisons to Mongols and Nazis. Chomsky's fans will love it, but even mainstream critics are catching up to the substance of his take on Bush Administration policies; meanwhile his uncompromising moral sensibility, icy logic and withering sarcasm remain in a class by themselves. Required reading for every thoughtful citizen."
Copyright Reed Business Information, a division of Reed Elsevier Inc. All rights reserved.

~~~~~~~~~~~~~~~~~~~~~~~~~~~~~~~~~~~~~~~~~~~~~~~~~~~


Are your fingers all a-tingle waiting to make an ad hominum attack on Chomsky?? Forget it!

How about dealing with his ideas described in this book review?

tomder55 answered on 05/01/06:

I have nothing to say beyond contempt for Chomsky. I do not put any credance to his anti-American bashes . Simply put ;the US is a democratic form of government and in spite of his wishes that the "majority "wants different policies on global warming , social spending , foreign policy; ,domestic security ,tax policy .ect; The government is elected by the people and serves at the people's pleasure .

How dare he compare the US to the Nazi's !As a so called man of education he must know better ,and either he is a propagandist as bad as he claims the gvt is ;or he is completely delusional.

Itsdb rated this answer Excellent or Above Average Answer
jackreade rated this answer Excellent or Above Average Answer

Question/Answer
CeeBee2 asked on 04/29/06 - Here's the English translation of the Spanish

version of the National Anthem:

'NUESTRO HIMNO' [Our Anthem]

Verse 1:
The day is breaking, do you see it? In the light of the dawn?
What we so acclaimed at nightfall?
Its stars, its stripes,
flew yesterday
In the fierce battle
in a sign of victory,
The glow of battle, in step with liberty
At night they said: "It's being defended!"

Chorus:
Oh say!
The voice of your starry beauty
is still unfolding
Over the land of the free
The sacred flag?

Verse 2:
Its stars, its stripes,
Freedom, we are equal
We are brothers, in our anthem.
In the fierce combat in a sign of victory
The glow of battle, in step with liberty
My people keep fighting
It's time to break the chains
At night they said: "It's being defended!"
Oh say! Your starry beauty is still unfolding.

AP

tomder55 answered on 04/30/06:

I'd have to say that is a loose translation of the English lyrics. Jose can you si?

A remix to be released in June will contain several lines in English that condemn U.S. immigration laws. Among them: "These kids have no parents, cause all of these mean laws ... let's not start a war with all these hard workers, they can't help where they were born."

CeeBee2 rated this answer Excellent or Above Average Answer

Question/Answer
Erewhon asked on 04/29/06 - The original words to the tune of the American national anthem ...


As a public service, I post the original words to the tune of the national anthem. Older readers will recognise it as an English drinking song!

I
To Anacreon in Heaven, where he fat in full glee,
A few fons [sons] of Harmony fent [sent] a petition,
That He their Infpirer and Patron would be;
When this anfwer arrived from the Jolly Old Grecian
"Voice, Fiddle, and Flute,
"no longer be mute,
"I'll lend you my Name and infpire you to boot,
"And, befides, I'll infruct you like me to entwine
"The Myrtle of Venus with Bacchus's Vine.

II
The news through OLYMPUS immediately flew;
When OLD THUNDER pretended to give himfelf Airs
"If thefe mortals are fuffer'd their Scheme to perfue,
"The Devil a Goddefs will ftay above the Stairs.
"Hark, already they cry,
"In tranfports of Joy,
"Away to the Sons of ANACREON we'll fly,
"And there, with good Fellows, we'll learn to entwine
"The Myrtle of VENUS with BUCCUS'S Vine.

III
"The YELLOW-HAIRED GOD and his nine fufty Maids
"From Helicon's Banks will incontinent flee,
"IDALIA will boaft but of tenantlefs Shades,
"And the bi-forked Hill a mere Desart will be
"My Thunder, no fear on't,
"Shall foon do it's Errand,
" and, dam'me! I'll fwinge the Ringleaders, I warrant,
"I'll trim the young Dogs, for thus daring to twine
"The Myrtle of VENUS with BACCUS'S Vine.

IV
APOLLO rose up; and faid, "Pr'ythee ne'er quarrel,
"Good King of the Gods, with my Vot'ries below:
"Your Thunder is ufelefs." - then, fhewing his Laurel,
Cry'd, "Sic evitabile fulmen, you know!
"then over each Head
"My Laurels I'll fpread;
"So my Sons from your Crackers no Mifchief fhall dread,
"Whilst fnug in their Club-Room, they jovially twine
"The Myrtle of VENUS with BACCUS'S Vine.

V
Next MOMUS got up, with his rifible Phiz [face],
And fwore with APOLLO he'd cheerfully join
"The full Tide of Harmony ftill shall be his,
"But the Song, and the Catch, & the Laugh fhall be mine
"Then, JOVE, be not jealous
Of thefe honeft Fellows.
Cry'd JOVE, "We relent, fince the Truth you now tell us;
"And fwear, by OLD STYX, that they long fhall entwine
"The Myrtle of VENUS with BACCUS'S Vine.

VI
Ye fons of ANACREON, then, join Hand in Hand;
Preferve Unanimity, Friendfhip, and Love!
'Tis your's to fupport what's fo happily plann'd;
You've the Sanction of Gods, and the FIAT of Jove.
While thus we agree
Our Toaft let it be.
May our club flourish happy, united and free!
And long may the Sons of ANACREON intwine
The Myrtle of VENUS with BACCUS'S Vine.


Jolly good sun [fun], what!

So, it is not the first time that the words of your NA have been changed.

If you want to change the words of "The East Is Red," "Deutchland Uber Allers," the "Marsellaise," or "God Save the Queen," go right ahead.



tomder55 answered on 04/30/06:

The Star-Spangled Banner was officially made the national anthem by Congress only in 1931 and I have heard versions of the song where lyrics have been added . That is all besides the point . Ceebee is correct in the language of the song being the issue.

Erewhon rated this answer Excellent or Above Average Answer

Question/Answer
Erewhon asked on 04/29/06 - It's Traditional to aggiornamentise The NA! Looky here, folks, it's not scripture!



Star-Mangled Banner
A look at some controversial, and botched, renditions of our national anthem

by John Gettings

Controversial Singings of the National Anthem
"Oh say, can you see, by the dawn's early night . . ."
Since World War II the Star-Spangled Banner has become a permanent fixture at sports events in the United States. And over the last 30 years artists have gradually made non-traditional renditions of the song commonplace.
Frequency doesn't necessarily breed acceptance, however. While America may seem less surprised by new renditions of the national anthem, it still considers the song's performance a litmus test for patriotism.


Jose Feliciano
Oct. 7, 1968
Puerto Rican blind singer/guitarist Jose Feliciano stunned the crowd at Tiger Stadium in Detroit, and the rest of America, when he strummed a slow, bluesy rendition of the national anthem before Game 5 of the World Series between Detroit and St. Louis. The 23-year-old's performance was the first nontraditional version seen by mainstream America, and it is generally considered the Lexington and Concord of Star-Spangled Banner controversies. The fiery response from Vietnam-weary America was not surprising, considering the tumultuous year for American patriotism. Good or bad, however, Feliciano's performance opened the door for the countless interpretations of the Star-Spangled Banner we hear today.


Jimi Hendrix
Aug. 17, 1969
It wasn't a sports event, but it was controversial. During the final set of the historic Woodstock music festival Jimi Hendrix let loose with a rendition of the Star-Spangled Banner on electric guitar that's been called everything from the most important political rock statement of the 1960s, to an afterthought caught in one of Hendrix's worst performances. It was his first gig since the breakup of the Jimi Hendrix Experience and all but 10% of the festival's 400,000 concert goers stayed for his Monday morning set. But there was no question the performance was controversial. Even today, music scholars can't agree on what message, if any, Hendrix's screaming guitar and ballistic feedback was trying to deliver.
[Note: Hendrix's entire Woodstock set, including the Star-Spangled Banner, can be heard on the 1999 MCA release Jimi Hendrix Live at Woodstock.]


Marvin Gaye
A little more than a week before Motown legend Marvin Gaye picked up two Grammy Awards for his classic "Sexual Healing," he performed the national anthem before the 1983 NBA All-Star Game at The Forum in Inglewood, Calif. Accompanied by a drum machine, Gaye's interpretation added elements of soul and funk to the national anthem. Gaye, who, coincidently, also sang the anthem during the same World Series as Jose Feliciano in 1968, was scrutinized for his performance, but the fallout didn't compare to that of Feliciano's rendition. The NBA players were most receptive. Especially Gaye fan Julius Erving, who loved the performance and went on to win game MVP honors.
[Note: Gaye's performance is the first track of the 1996 Polygram compilation NBA at 50: Musical Celebration.]


Roseanne Barr
July 25, 1990
The poster child for Star-Spangled Banner controversy, Barr (whose last name and reputation were still intact at this time) tried to add her own brand of humor to the singing of the national anthem before a baseball game in San Diego. After screeching through an off-key version of the song she added some clichd baseball humor by spitting and grabbing her crotch. The popular sit-com comedian immediately became public enemy number one. After hearing a tape of Barr, President George Bush (Sr.) called it "disgusting" and "a disgrace."


Aerosmith
May 27, 2001
Singing on Memorial Day before the start of the Indianapolis 500, Steven Tyler, lead singer of the rock group Aerosmith, angered veterans by changing the last line of the song. Instead of singing "home of the brave," Tyler sings "home of the Indianapolis 500." He apologized the next day, releasing the following statement: "I got in trouble my whole life for having a big mouth. I'm very proud to be an American and live in the home of the brave."


And then there are the unforgettable performances where we watched confident artists painfully struggle with this very difficult song. Here are two examples:


Robert Goulet
May 25, 1965
Although he was born in the United States, Robert Goulet moved to Canada when he was 14 years old and had never sang the Star-Spangled Banner in public before May 25, 1965. That night, moments before the much-anticipated rematch of boxing heavyweight champion Muhammad Ali and Sonny Liston in Lewiston, Maine, Goulet began, "Oh say, can you see, by the dawn's early night..." The bout lasted less than one round, and the disappointing fight didn't provide a big enough shadow for Goulet's performance to hide behind. Although he's done it without incident hundreds of times since, Goulet says he is always asked about his infamous flub.


Carl Lewis
Jan. 21, 1993
There's no question nine-time Olympic track-and-field gold medallist Carl Lewis can carry a batonas he did many times for U.S. Olympic relay teamsbut how about a tune? Before a Chicago BullsNew Jersey Nets basketball game, in front of a sellout crowd in East Rutherford, N.J., Lewis orchestrated the musical equivalent of a train wreck. Later explaining that he was hoarse from participating in inaugural events at the White House the day before, Lewis faltered during the lyric "rockets red glare," and then mid-song told the fans, "I'll make up for it." He never did.

===

Ronnie [Erewhon] = reporting from the Home of the Brays in the Land of the Free!



tomder55 answered on 04/30/06:

If it means anything to you ,my preference would be that the national anthem be 'God Bless America ' with the OFFICIAL version being an interpretation of the great Kate Smith's version because it is the version that is sung with the most pride .Other versions like Celion Dion's tend to sound more like a lament or a funeral song.

On 9-11 in a show of national unity ,the US Senate sang it on the Capitol steps ,not the National Anthem.

It is not derived from some bawdy British saloon song but was composed by an American (yes he was the son of an immigrant ....,(his family had come to America from Russia when Irving was a small boy);Irvin Berlin. Originally penned and composed during WWI while Berlin was stationed at Camp Yaphank on Long Island ,NY.; it was shelved by Berlin's superiors as being too jingoistic.

Kate Smith 20 years later ,looking for a song to commerate the 20th Anniversay of the end of WWI.After trying to compose a new song he remembered that he had one filed gathering dust .

Smith intruduced the new song to the nation Nov. 10,1938 and it was so powerful that it electrified the nation.She recorded it for RCA Victor on March 21, 1939.

The lyrics were inserted into the Congressional Record, and there was a movement to make the song our national anthem but Kate Smith herself testified before Congress asking them to keep the 'Star Spangled Banner 'as the anthem.But if it is not the official anthem ,in other ways it is the nation's adopted one .

Erewhon rated this answer Excellent or Above Average Answer

Question/Answer
Erewhon asked on 04/29/06 - Just to prove how far the Scots have come since the old days ....



Why Scotland can't Handel the BBC's new World Cup anthem
GERRI PEEV AND JONATHAN LESSWARE

THE BBC was accused of "insensitivity and tactlessness" last night after choosing to mark the World Cup this summer with a piece of music that celebrates the man responsible for perhaps the most infamous military massacre in Scottish history - the battle of Culloden.

See the Conquering Hero Comes, by Handel, will accompany the national broadcaster's coverage of the football tournament from Germany. While the music frequently and innocently accompanied victorious football teams across the UK in the first half of the 20th century, for some Scots it has much darker connotations.

Handel composed the score as a tribute to the Duke of Cumberland, more vividly remembered as Butcher Cumberland, who led government forces to victory over the heavily outnumbered Jacobites at Culloden.

Last night, politicians and historians criticised the choice of a song which celebrated the massacre. Rob Gibson, the Scottish Nationalist Party's Highland and Islands Member [of the] Scottish Parliament, said: "How can they possibly encourage people to support the England team when we are exposed to symbols of oppression like this?

"As far as I'm concerned the BBC often causes offence with its insensitivity to Scottish history. At a time when multiculturalism is being celebrated, I can't understand how they can be so insensitive. It's an anglocentric view they have of the world and of music."

The MSP, who has written a number of books on Scottish history, including a recent one on the clearances, said the BBC should be making more programmes on history and how these events shaped the world.

Ted Cowan, Professor of Scottish history at Glasgow University, described the choice of song as tactless, but said it was typical of the outdated sentiments that often surface in football. "I think it's pretty tactless to revive something like that," he said.

Prof Cowan said Culloden had been misrepresented in history as a battle of nations. He added: "Culloden was not actually a battle between Scotland and England, there were Scots and English on both sides. But the authorities in London hijacked the victory and portrayed it as beating the rebellious Scots."

About 1,000 of the 5,000 troops loyal to Charles Edward Stuart, Bonnie Prince Charlie, were slaughtered by the 9,000-strong troops of William Augustus, the Duke of Cumberland at Culloden, in just 40 minutes, on 16 April, 1746. The battle, which took place on moorland near Inverness, forever shattered the Jacobite dream of installing a Stuart on the British throne. A further 1,000 Highlanders who fled the battlefield died in the subsequent weeks.

Angus MacNeil, the SNP's culture spokesman at Westminster and Western Isles MP, said he regretted the fact that the BBC's move could give "posthumous publicity" to Cumberland, "a man who should be forgotten in a way Bonnie Prince Charlie will never be".

While Culloden is still regarded as an infamous event by many in Scotland, it was celebrated extensively at the time, by both the English and some lowland, Protestant Scots happy at the defeat of the Jacobites, who were mainly Catholics and Episcopalians.

The London government encouraged those celebrations, and Handel, a German who lived most of his life in England, composed See the Conquering Hero Comes as his contribution.

His classical anthem has now been adapted by Andrew Davis, and the modern rendition, performed by the BBC Concert Orchestra and BBC Singers, has been entitled Sports Prepare.

This article: http://news.scotsman.com/index.cfm?id=641642006

Share a personal tale about your family's colourful Scots heritage" www.heritage.scotsman.com

2006 Scotsman.com | con

===

Is it obvious that distance has not lent enchantment to the scene?



tomder55 answered on 04/30/06:

it is also obvious that as Scottish nationalism is encouraged ever further the dream of unity on the island is realized . One common language ,and a shared national identity ;that is how nations are sustained.

Erewhon rated this answer Excellent or Above Average Answer

Question/Answer
Erewhon asked on 04/29/06 - Sr. Presidente. Tenemos su permiso de morir en espaol, o debemos morir en ingls?

Bush thinks national anthem should be sung in English
BY MARK SILVA
Chicago Tribune

WASHINGTON - President Bush said Friday that "The Star Spangled Banner" should be sung in English and that the national anthem would not hold the same value when sung in Spanish.

"I think the national anthem ought to be sung in English," Bush said, in response to a question at a Rose Garden news conference. "People who want to be a citizen of this country ought to learn English, and they ought to learn to sing the national anthem in English."

===

Is it OK if Americans who speak Spanish die in Spanish in Iraq and Afghanistan, or would Senor Bush prefer they do that in English too?

tomder55 answered on 04/29/06:

Most countries of the world have an official language for conducting gvt.and commerce . Canada I guess sings their anthem bi-lingually English and French ,but then again there is always the underlying danger of union disolution in Canada ;curiously in the province where the language difference is most pronounced . Other than that I can think of no other example where a national anthem is bi-lingual .

Erewhon rated this answer Excellent or Above Average Answer

Question/Answer
kindj asked on 04/27/06 - Their hypocrisy knows no bounds

Mexicos Immigration Law: Lets Try it Here at Home - By J. Michael Waller

Mexico has a radical idea for a rational immigration policy that most Americans would love. However, Mexican officials havent been sharing that idea with us as they press for our Congress to adopt the McCain-Kennedy immigration reform bill. Thats too bad, because Mexico, which annually deports more illegal aliens than the United States does, has much to teach us about how it handles the immigration issue. Under Mexican law, it is a felony to be an illegal alien in Mexico. At a time when the Supreme Court and many politicians seek to bring American law in line with foreign legal norms, its noteworthy that nobody has argued that the US look at how Mexico deals with immigration and what it might teach us about how best to solve our illegal immigration problem. To prevent tourists from taking advantage of their immigration system, Mexico has a single, streamlined law that ensures that foreign visitors and immigrants are:

in the country legally;
have the means to sustain themselves economically;
not destined to be burdens on society;
of economic and social benefit to society;
of good character and have no criminal records; and
contributors to the general well-being of the nation.


The law also ensures that:

immigration authorities have a record of each foreign visitor;
foreign visitors do not violate their visa status;
foreign visitors are banned from interfering in the countrys internal politics;
foreign visitors who enter under false pretenses are imprisoned or deported;
foreign visitors violating the terms of their entry are imprisoned or deported;
those who aid in illegal immigration will be sent to prison.


Who could disagree with such a law? It makes perfect sense. The Mexican constitution strictly defines the rights of citizens and the denial of many fundamental rights to non-citizens, illegal and illegal. Under the constitution, the Ley General de Poblacin, or General Law on Population, spells out specifically the countrys immigration policy. It is an interesting law and one that should cause us all to ask, Why is our great southern neighbor pushing us to water down our own immigration laws and policies, when its own immigration restrictions are the toughest on the continent? If a felony is a crime punishable by more than one year in prison, then Mexican law makes it a felony to be an illegal alien in Mexico.

Yet, if the United States adopted such statutes, Mexico no doubt would denounce it as a manifestation of American racism and bigotry.

tomder55 answered on 04/28/06:

Guess that ruins my retirement plans of crossing the border and squatting on someones property ;hoisting up old Glory ;exploiting the social services while I catch some sun and some Tequila in Cancun.

Monday is May Day and the illegals are planning a wild cat strike .A Day Without Illegal Aliens. This is supposed to frighten you.

This act is supposed to drive American business to it's knees and force us to see the fallacy of our position.

But ;it will do nothing to hurt American employers who play by the rules. Only the underground economy will take a hit and I could care less. The thing to do is to drive around town and see which businesses shut down . Then report them to the IRS.

It should be a national shopping and eat out day in an effort to support the businesses that play by the rules .



kindj rated this answer Excellent or Above Average Answer

Question/Answer
Erewhon asked on 04/28/06 - Foreign countries with adjoining borders can be good neighbours ...


In a year's time, on 1 May, 2007, the Union of the English and Scottish Parliaments will be three hundred years old.

The longevity of this close political association between two of Europe's ancient nations is remarkable.

It shows what can be forged between discrete countries having a common border when the will to build rather than tear down is present.

Could this fine example of symbiotic union be used in other countries?



tomder55 answered on 04/28/06:

perhaps in some future date when there is a more common language and heritage then the borders will not be a factor . You are correct that the evolution of the Island took several centuries and some of the transformation was not pretty . Remember that Scotland was ruled by the English monarch for over 100 years before 1707 unification of Great Britain . Relations between England ,Scotland and Whales were not always congenial . There were periods of warfare ,and rebellion andit was only in 1998 that the Scotish Parliment reconvened for the first time since 1707 . Let's not pretend that England and Scottland have been happy neighbors sitting by the camp fire eating s'mores and singing Kum-bai-Ya .

Erewhon rated this answer Excellent or Above Average Answer

Question/Answer
excon asked on 04/27/06 - Messages


Hello rightdingers (I mean rightWingers):

You guys have a problem with distinguishing the message from the messenger. As a matter of fact, you have a BIG problem with messages, in and of themselves. You guys love to say what message does this send? I guess you think people are real malleable. Maybe some are.

I love my country. I love it because its worthy of my love - not because someone told me to. Do you love it here because you just read a clever bumper sticker? Maybe some of you do.

Im a good person because thats the way I choose to live my life - not because someone told me to. Are you a good person because someone told you to be? Maybe some of you are.

Do you think our society changes depending on the current message? Or perhaps the LOUDER message (and Im not talking about fads)? I dont! I know some of you do.

If my government behaved, it wouldnt matter much what the messengers say, because the truth speaks for itself. However, if it doesnt behave, I want to know about it. Thats the presss job. That is their ONLY job. Thats why they get special dispensation in the form of the First Amendment.

Now, you and I can argue about whether we should render prisoners to foreign countries for questioning. But there should be no argument about the press telling us about it. I know, some of you think they should only report good news.

excon

tomder55 answered on 04/28/06:

I don't ask for only good news .I ask for balance and unfortunately it is not there . If the Washington Compost and the NY Slimes advertised themselves as a source of biased journalism then I could live with it . I rarely take on the 'Daily Kos' because I know he is a moonbat with an agenda . I would never consider him the source of record.

Dana Priest of the Washington Post won her Pulitzer Prize for reporting on probably erroneous and inaccurate assertions that the U.S. was running secret prisoner camps in former East block European nations . [yes this was reported but did not get nearly the play that the initial report generated ;most Americans do not know that the EU does not think the report credible].

Investigations into reports that US agents shipped prisoners through European airports to secret detention centers have produced no evidence of illegal CIA activities, the European Union's antiterrorism coordinator said yesterday.

The investigations also have not turned up any proof of secret renditions of terror suspects on EU territory, Gijs de Vries told a European Parliament committee investigating the allegations


Will Dana Priest have that Pulitzer prize revoked? Probably not .

Now ; for weeks every time that Tom Delay's name was mentioned the press salivated . Big scandal ! Republican culture of corruption !! . But,did you hear that a high ranking Democrat was forced to step down from the House ethics committee ? Maybe ;but if you weren't paying attention you missed it ....his name is Alan Mollohan (D.W.Va.)The charge ? He steered millions of dollars to nonprofit groups in his district with much of the money going to organizations run by people who contribute to his campaign. Any different from the allegations leveled at Delay ? Well ;actually yeah. When Delay played his shell games with campaign money there was no law against it . It became illegal after the fact.

Me ; I read the stories and spend alot of time determining if the report is credible. Many times they are not . When Dan Rather and co. at 60 Minutes fabricated a story about the President during the election were we supposed to take it at face value just because such an esteeemed reporter as Rather said so ? I don't think so.

To sum it up . I am not looking to attack the messenger as much as to find out the truth. Unfortunately I have found that the messenger cannot be trusted .





excon rated this answer Excellent or Above Average Answer

Question/Answer
Erewhon asked on 04/27/06 - Series of Setbacks Threatens Morale of Religious Right .........


Series of Setbacks Threatens Morale of Religious Right
Wednesday, April 26, 2006
By Kelley Beaucar Vlahos



Some Question Whether GOP Plan For Iraq Is Enough

WASHINGTON More than a year after "values voters" propelled President Bush to a second term in office, many religious conservatives say they are starting to feel undervalued, an emotion that could spell danger for congressional Republicans ahead of a contentious midterm election.

You can cut it with a knife, thats how upset they are, said Richard Viguerie, a long-time member of the social conservative movement, which is largely evangelical and considered to be the base of last year's presidential victory.

Among the disappointments cited are increased spending under the Republican-controlled White House and Congress, and a lack of focus on domestic issues dear to this voting bloc. Recent Capitol Hill scandals shadowing some of the "religious rights" brightest stars and a lost battle to save Terri Schiavo have also threatened the morale and strength of this political lobby, say leaders.

I definitely think there are morale problems and waning enthusiasm, said Gary Bauer, head of the Campaign for Working Families, a conservative political action committee.

Part of it, I think is were in a second term (presidency) and theres been no major progress on things that the base really cares about, said Bauer, who ran for the GOP presidential nomination in 2000. There is sort of this simmering frustration out there that 'man, they want our votes on Election Day, but they are going to fight on 50 other issues before they get to our issues.'

Conservatives, however, do acknowledge two major morale boosts over the last year the confirmations of Supreme Court Chief Justice John Roberts and Associate Justice Samuel Alito.

Any disappointment was mitigated by John Roberts and Justice Alito. That is a huge thing for social conservatives, said Dr. Richard Land, president of the Southern Baptist Convention's Ethics & Religious Liberty Commission. The president has fulfilled no campaign promises more faithfully than only nominating strict constructionalists and non-activist judges.

Still, Viguerie said the simmering frustration has heated up to the boiling point for many conservatives, some of whom are privately suggesting that the prospect of a Democratic win in November aint all that bad," and that a divided government would scale back growth and purge the Republican Party of its non-conservatives.

In 2004, 78 percent of evangelical voters, who made up 23 percent of the electorate, voted for Bush, according to exit polling. Evangelicals identified themselves as Republicans, 48 percent the largest numbers in recent history compared to 23 percent who called themselves Democrats.

If the last year has angered these voters, it hasnt stopped them from making noise, as evidenced by a recent summit of evangelical Christians and conservative Jewish activists in Washington, D.C. The summit produced a Values Voters Contract with Congress, demanding legislation that would address issues like keeping God in the Pledge of Allegiance, banning cloning and getting a constitutional amendment enshrining marriage as a union between one man and one woman.

In terms of this bloc of voters it isnt so much that they are complacent, it's more that they are hesitant because they havent seen the results that they were promised, said William Greene, head of RightMarch.com, which participated in the March summit organized by Texas-based Vision America. RightsMarch.com also recently announced a new event with the Minutemen Project to highlight voter anger at unchecked illegal immigration.

Scandals Rattle the Base

While conservatives say they are disappointed by the inability of Congress to achieve their agenda, others suggest that recent Capitol Hill scandals have also taken their toll.

Rep. Tom DeLay, R-Texas, a major supporter of conservative issues, announced earlier this month that he was resigning from Congress before the November election because he thought his legal problems were interfering with the Republican agenda in the House.

The former majority leader has been indicted on money laundering and conspiracy charges relating to the election of Republican Texas state representatives in 2002. Two of his former aides, Michael Scanlon and Tony Rudy, have pleaded guilty to conspiracy charges in a federal bribery and fraud investigation involving convicted Washington lobbyist Jack Abramoff.

Former Christian Coalition head Ralph Reed and major grassroots organizer Focus on the Family have also been connected to Abramoff in Senate hearings and in news reports.

Marvin Olasky, a professor at the University of Texas and editor of the Christian WORLD magazine who helped create the faith-based initiative in the White House, has been very critical of Reed, who reportedly took $4 million from Abramoff to launch an anti-gambling campaign that would ultimately help Abramoffs Indian casino clients fend off a competing tribe's bid for a casino.

Olasky said that Reed has damaged the reputation of evangelical leaders and the movement. This internal stuff it zaps the energy of the movement, he said. Thats why transparency and honesty are so important.

Reed has responded to Olaskys criticisms on his Web site. I was not hired (by Abramoff) to lobby. We mailed direct mail letters and aired radio ads opposing illegal casinos, he wrote. Had I known then what I know now, I would not have undertaken the work ... I regret any difficultly it has cause the pro-family community.

Focus on the Family chief Dr. James Dobson flatly denies his group engaged in its own anti-gambling campaign at Reed's urging as part of his work with Abramoff. The group was using its muscle to fight illegal gambling period, say officials there.

Paul Hetrick, spokesman for Focus on the Family, said the virility of the grassroots organization has not been affected by the story.

Were not going to be the least bit intimidated, restricted or constrained by any external factors, said Hetrick.

Elsewhere, some conservatives suggest that fight in 2005 to keep Terri Schiavo alive in Florida, which drew Congressional Republicans into a failed attempt to legislate the situation, might have given them a black eye with the public. Comments from television evangelist Pat Robertson, who recently called for the assassination of Venezuelan President Hugo Chavez, also have not won widespread support.

Greene said he doesn't agree with those claims.

"I don't think it hurts the cause in terms of the issues the support is there no matter what people like Pat Robertson says," said Greene. As for Schiavo, "the folks that really form the base of the Republican Party feel just as strongly that she shouldnt be starved to death and that hasn't changed."

Land said he expects that values voters who are looking forward to a debate this summer over a federal marriage amendment and stem cell research will again be a strong force in the 2006 midterm election.

I dont see any big dip in morale, Land said. I think social conservatives are looking forward to 2006 and to the 2008 campaign. They feel the wind at their backs.

=====

Should the GOP be concerned?

tomder55 answered on 04/28/06:

yes the GOP should be concerned . It will be difficult for the Democrats to take Congress from the Republicans but it is not an impossiblilty . I said yesterday that Tony Snow was brought in to make the administation's case to the base. The Republicans need a solid turnout of the base at the polls . Then they can concentrate on capturing enough of the middle to hold on with only a few seats lost .

But I would not be concerned about the base . Bottom line ...they will hold their nose and vote for the Republican because the prospect of Senate Majority Leader Reid and Speaker of the House Pelosi is worse .

Erewhon rated this answer Excellent or Above Average Answer

Question/Answer
kindj asked on 04/26/06 - The other side of the SecDef coin

This source seems like he ought to be fairly reliable, given his resume (see bottom). While reading, keep in mind comments that I made in another post concerning "new strategies for new wars."

~~~~~~~~~~~~~~~~~~~~~~~~~~~~~~~~~~~~~~~~~~~~~

Jed Babbin: Keep the Big Dog running
PDF | Email
Jed Babbin, The Examiner
Apr 25, 2006 7:00 AM (10 hrs ago)

WASHINGTON - Everyone is saying that Secretary of Defense Donald Rumsfelds days are numbered, thanks in part to increasing calls by some former generals for Rumsfelds resignation.


But Rumsfeld was hired by George W. Bush to do precisely what he has done to the consternation of the generals who are now coming out to complain about him.

When President Bush brought Rumsfeld back to the Pentagon, the president told him to shake up the Pentagon, to transform it from the Cold War structure and culture that it was stuck in to a new force with strategies that could respond to the post-Cold War world.

Months before Sept. 11, as Rumsfeld began the transformation of the Pentagon, he ran into contumacious obstructionism from the army and its then-Chief of Staff Gen. Eric Shinseki. Shinseki dug his heels in and refused to change much of anything about the Army. Shinseki went as far as to go behind Rumsfelds back to the Senate where his political mentor (and long-time family friend, Sen. Dan Inouye of Hawaii) and others backed his play.

But for the political cover Sen. Inouye gave Shinseki, he might have been fired then and there. Civilian control of the military means people such as Shinseki cannot be allowed to play the back-channel political games he played again and again. Shinseki stayed, and the Army went on to spend billions on the Stryker armored vehicle, a Cold War style peacekeeping vehicle that is too big and too heavy to be moved by a C-130 tactical airlifter without being partially disassembled.

And then came Sept. 11. The Secretary of Defense became the secretary of war and the transformation he had brought to the Pentagon had to be continued under fire. Still, the Army resisted.

Shinseki balked at striking at the Taliban. For the record, our forces slashed into the Taliban around Oct. 5, 2001, less than a month after Sept 11. But aside from Rangers and Army Special Forces the Army stayed home. Shinseki wanted at least six months to assemble and move an enormous Soviet-like force into Afghanistan and the president wasnt having any of it. This is why Shinseki retired in 2003 with a festering grudge against Rumsfeld.

And then Rumsfeld did the unthinkable. Instead of replacing Shinseki with one of his like-minded underlings, Rumsfeld looked for someone who would fight. Gen. Peter Schoomaker, a Special Forces vet, was brought out of retirement to transform the Army in the middle of a war. And he did it. But in the process Rumsfeld, Schoomaker and his team shook up a lot of people.

Of the six who have called for Rumsfelds firing, all came to rank and prestige in the Clinton days, what some Pentagon wags now call the Great Period of Neglect. It was the era of Blackhawk Down, of Shinseki ordering the army to wear black berets and buying them from China and of Gen. Anthony Zinni, then commander of CENTCOM, becoming addicted to stability in the Middle East, entranced by the Arab leaders hed come to know well. Stability meant leaving Saddam alone, so Zinni spoke often against the Iraq war before it began. Stability now means leaving Iran to pursue its nuclear weapons program undisturbed.

President Bush has made it clear that Rumsfeld has his confidence and that, in his judgment, its best for America that Rumsfeld stays. This will only result, sooner rather than later, in another political exercise and thats all the generals revolt is to remove him. Mr. Bushs opponents see Rumsfeld as vulnerable. They cant rid themselves of George W. Bush, but they can damage him by damaging Rumsfeld.

Rumsfeld is the Big Dog, and those whose feathers he has ruffled in the Pentagon, the press and Congress are the poodles who chase after him. They should follow the principle one Southern gent often reminds me of: If you cant run with the big dog, youd better go sit on the porch.

Jed Babbin is a former deputy under-secretary of defense and the author of Inside the Asylum: Why the UN and Old Europe are Worse than You Think and (with Edward Timperlake) Showdown: Why China Wants War with the United States. He is also a contributing editor at FamilySecurityMatters.org.
Examiner
~~~~~~~~~~~~~~~~~~~~~~~~~~~~~~~~~~~~~~~~~~~~~~~

Kinda flies in the face of what the MSM's told us, doesn't it?

DK

tomder55 answered on 04/27/06:

I have read Babbin's work in the Spectator for a couple of years now . Also recommend his book 'Inside the Asylum: Why the United Nations and Old Europe Are Worse Than You Think'.He is a regualr guest on the 'John Batchelor' show which I tape and listen to daily at work.

No one should be surprised that Secretary Rumsfeld has met resistance in his transformation project. Plenty of entrenched positions and feifdoms have been disrupted inside Dept. Def. In early 2000, Former Clinton CENTCOM commander, Anthony Zinni told Congress "Iraq remains the most significant near-term threat to U.S. interests in the Arabian Gulf region," adding,

"Iraq probably is continuing clandestine nuclear research, [and] retains stocks of chemical and biological munitions

Even if Baghdad reversed its course and surrendered all WMD capabilities, it retains scientific, technical, and industrial infrastructure to replace agents and munitions within weeks or months."
I guess Saddam ceased being a threat when Bush became President .

kindj rated this answer Excellent or Above Average Answer

Question/Answer
jackreade asked on 04/26/06 - Words of the New Press Secretary

Copy and Paste follows:

"The following are excerpts from a column that the new White House Press Secretary, Tony Snow, wrote for the Washington Times on October 26, 1989."

"Post-Woodstock Americans have destroyed the old taboo about discussing sex but they haven't disturbed the taboo's essence, which is to avoid mentioning any of the important stuff. More words about sex probably have been printed since 1980 than in the previous history of mankind, but those words haven't added a pamphlet's worth of new truth.

...most of this verbiage and imagery concern nothing more than technique. There's no mention of sex's *weird* (?) emotional impact. There's no mention of passion. There's no mention of the fact that sex is *fun only when you do it right* (HUH?).

This brings in the subject of relations between men and women, which is more complicated and *inscrutable* than the relations between interlocking body organs. If you want to make sense of real sex, you have to think about people's feelings, including the unexpressed and ineffable emotions that often give rise to sex. You need to admit that sex produces bewilderment, wonderment, fascination, confusion, elation, depression - virtually every imaginable emotion. Most of all, you need to screw up the courage to talk about love, if only to expose how shallow, pitiful, vicious and desperate loveless sex can be.

...Most studies now confirm that sex education has helped encourage sexual activity among youth. That's probably because the classes have focused on technique without addressing matters of the soul. If we really want to encourage people to behave sensibly when they're naked, why not dwell on the scary (and funny) aspects of sex - the late-night headaches, the early-morning heartaches, the vulnerability, the hilarity, the dangers of passions requited and unrequited?">>>Tony Snow *asterisks mine*


~~~~~~~~~~~~~~~~~~~~~~~~~~~~~~~~~~~~~~~~~~~~~~~~~~~~
~~~~~~~~~~~~~~~~~~~~~~~~~~~~~~~~~~~~~~~~~~~~~~~~~~~~
"Great. We'll finally have press secretary who can explain how Bush is screwing the country. Was it fun? Did he do it right?">>Anon
~~~~~~~~~~~~~~~~~~~~~~~~~~~~~~~~~~~~~~~~~~~~~~~~~~~~~
~~~~~~~~~~~~~~~~~~~~~~~~~~~~~~~~~~~~~~~~~~~~~~~~~~~~~


So, the above boxed in remark is one comment by an anonymous internet user on Huffington Post dot com, what do you think of the Tony Snow appointment?

Also, bonus question, do you think he knows anything, anything at all, about adult sex?

tomder55 answered on 04/27/06:

Tony Snow will be a fine press sec. In the coming days you will be able to find tons of quotes by him . He has been an opinionated commentator on radio ,television ,print ,and as a talk show host so I'm sure that there is plenty of fodder.I note that this one goes back to 1989 .I'm sure there are more recent comments to dissect.

Snow won numerous awards during his print career, including citations from the Society of Professional Journalists, The Associated Press and Gannett. His careers also includes service as an advocate for the Mentally Ill and Developmentally Disabled and as a teacher in Kenya and Cincinnati -- in subjects ranging from East African Geography to calculus. He is an avid musician (he plays guitars, alto sax, tenor sax, flute and alto flute).

As a opinionated journalist he has frequently taken the Bush adm to task and I'm sure you will see cherry picked quotes from him about the President when he was critical of the President's performance. Snow, in an Associated Press interview Wednesday, said: "It's public record. I've written some critical stuff. When you're a columnist, you're going to criticize and you're going to praise."

He is exactly what the President needs.Given his substantial background in politics and broadcasting, it's hard to imagine a better possible pick. That may be what has Democrats so up in arms over his selection.

Before he took the job on he had to get medical clearance because he is a colon cancer patient .He had most if not all of his colon removed last year . That hasn't stopped some moonbat lefty bloggers from taking cheap nasty swipes at him :

Snow has had a recent bout with colon cancer- (not uncommon for clench-sphinctered Republicans) which accounts for his hollow-eyed, narrow-foreheaded, Frankenstein look lately.

I guess it goes with the territory .
........................................
bonus question :do you think he knows anything, anything at all, about adult sex?

Who ? Tony Snow who is a father of 3 ;or George Bush who is a father of 2 ?





Itsdb rated this answer Excellent or Above Average Answer
jackreade rated this answer Excellent or Above Average Answer

Question/Answer
jackreade asked on 04/25/06 - Bush's Answer To High Gas Prices

From Yahoo news(how appropriate)

"Bush on Tuesday ordered a temporary suspension of environmental rules for gasoline, making it easier for refiners to meet demand and ***possibly*** dampen prices at the pump. He also halted for the summer the purchase of crude oil for the government's emergency reserve.

The moves came as political pressure intensified on Bush to do something about gasoline prices that are expected to stay high throughout the summer".

tomder55 answered on 04/26/06:

His move on the strategic reserve is like spit in the ocean . Tampering with the strategic reserve is meaningless ;nonetheless ;I agree that it is senseless to top it off when prices are high . Much better to fill it like he did when prices were lower. I would not release a drop of it to manipulate prices . It is there to provide a cushion for serious disruptions in supply .Besides ;it would be ineffective .When Clinton tapped the reserves in the late 90s it brought down pump prices by only 4 cents a gallon, and only for a week or two.


His talk about windfall profits is as demogogic as Chucky Shumer's rants. The price of oil is set in the world market . We have created both the supply and the demand problems and that is where the issue needs to be addressed. Don't blame the oil companies .No one sheds a tear for them when they were losing money when the price of a barrel was low.
Besides if you are looking for windfall profits it is in the tax revenue that the national and state gvts . collect from the sale of oil .Sadly ;both House Speaker Dennis Hastert and Senate Majority Leader Bill Frist have also urged the president to order a federal investigation into price gouging .You would think that people had learned their lessons about price controls during the 1970s. Price controls didn't stop the cost of gasoline from rising. They just changed how we paid for them. Instead of prices rising until the amount people wanted equaled the amount available, chronic shortages of gasoline had Americans waiting in lines for hours. Yet, the supposedly permanent shortages disappeared instantly as soon as price controls were removed. Go ahead ;force them to open their books.
They will show Chuckie Shumer that they are making a decent margin, but nothing compared to what the government is making off the "poor" gasoline consumer. They will also focus on the reasons they have become mostly transporters and refiners and not producers.... environmentalist obstructionism.


As far as easing the environmental controls all I have to say is kudos ! These custom made mixtures are great if you have the refining capacity to deal with them ;but we do not. We have doubled our demand in the last 30 years .Go ahead and ask EXXON if they would mind investing some of their "windfall profits" in new refineries ...of course they would . What is preventing it ? NIMBYism is the culprit.

At the same time we have half the refining capacity we had then. The refineries have to shut down and retool for each boutique blend. It is no accident that the recent price hikes coincided with refineries retooling for the conversion.Through the government limitation of America's energy production, we have caused the outsourcing of our energy needs to foreign countries.No new refineries ;no new drilling at either ANWR or on the continental shelf off either coast ;no building of nuclear plants . We are reaping what we have sow'n.

jackreade rated this answer Excellent or Above Average Answer

Question/Answer
jackreade asked on 04/25/06 - Nuclear Weapons Test, Nevada, June 2

Is this test to determine how effective small nuclear weapons might be in an invasion situation, say, such as Iran?

tomder55 answered on 04/26/06:

the test will not be nuclear ,but high yield conventional arsenal (700 tons ). Officials said the test, code-named Divine Strake, is part of research to "determine the potential for future non-nuclear concepts" -- such as high-energy weapons or the simultaneous use of multiple conventional bombs to destroy deeply buried and fortified military targets. In other words it is to see if a conventional strike on fortified underground facilities can destroy the target as well as a nuclear tipped bunker buster .

The confusion is the fault of James Tegnelia, director of the Pentagon's Defense Threat Reduction Agency who described the test as a massive blast that would create a "mushroom cloud" of dust over Nevada. to defense reporters . Officials in Nevada became rightly concerned .State regulators have raised questions about pollution and hazardous materials but have been satisfied that all standards will be met .The closest underground nuclear test was conducted 1 1/2 miles from the site so there is no chance that fallout from past nuclear tests will go airborne.

Perhaps the test will also be used to determine the lowest possible nuclear yield to destroy hardened underground targets . Can't take the nuclear option off the table.

jackreade rated this answer Excellent or Above Average Answer

Question/Answer
jackreade asked on 04/25/06 - Grounds for Impeachment-Brzezinski

"In an op-ed titled Do Not Attack Iran, former National Security Adviser Zbigniew Brzezinski today makes the case against launching an air strike on Iranian nuclear facilities. First on his list:

In the absence of an imminent threat (with the Iranians at least several years away from having a nuclear arsenal), the attack would be a unilateral act of war. If undertaken without formal Congressional declaration, it would be unconstitutional and merit the impeachment of the President.

Most strikingly, Brzezinski wonders whether the Bush administrations current strategy is actually designed to deliberately encourag[e] greater Iranian intransigence and undercut chances of reaching a diplomatic solution:

How else to explain the current U.S. negotiating stance: the United States is refusing to participate in the on-going negotiations with Iran but insists on dealing only through proxies. That stands in sharp contrast with the simultaneous negotiations with North Korea, in which the United States is actively engaged.

At the same time, the United States is allocating funds for the destabilization of the Iranian regime and is reportedly injecting Special Forces teams into Iran to stir up non-Iranian ethnic minorities in order to fragment the Iranian state (in the name of democratization!).

Brzezinski is the latest in a long line of national security experts and others advising against a military strike in Iran. (Read our full list HERE.) But Brzezinski also makes a strong proactive case on Iran, calling for the Bush administration to sober up, to think strategically, with a historic perspective and with Americas national interest primarily in mind.

~~~~~~~~~~~~~~~~~~~~~~~~~~~~~~~~~~~~~~~~~~~~~~~~~~

tomder55 answered on 04/26/06:

1st ;it is not a given that the US will strike at Iran .If I were to guess;it is more likely that Iran will take preemptive action on their own .So if the US does not strike Brzezinski can say :you heard it here first .

2nd not only is there an immenent threat (besides the fact that he is only guessing at the timeline. I happen to think that they are far more advanced than we realize and that is why their posture and tone has become more belligerent.);but we have in fact been in a shooting war with Iran since we entered Iraq through their surrogates and Iranian intelligence forces .

3rd in fact we have been in a state of undeclared war against Iran since the 1979 hostage incident .We have not made them answer to direct attacks on our military in Beirut ,and in Saudi Arabia .The only time we fought back was during the Iraq-Iran war when we had a pretty significant naval battle against them.

He is the prototypical pre-9-11 strategist .As Jimmy C's National Security Adviser Brzezinski came up with the failed "Green Belt Strategy "

That strategy was based on the assumption that the United States and its allies were unable to contain the Soviet Union, then expanding its zone of influence into Africa, the Indian Ocean region and, through left-leaning regimes, in Latin America. To counter that expanding threat, Brzezinski envisaged the creation of a string of Islamic allies that, for religious and political reasons, would prefer the United States against the "godless" Soviet empire.

The second stage in Brzezinski's grand strategy was to incite the Muslim peoples of the Soviet Union to revolt against Moscow and thus frustrate its global schemes.

The Bzrezinski strategy had been partly inspired by Helene Carrere d'Encausse, who, in her book "The Fragmented Empire," predicted the disintegration of the Soviet Union as a result of revolts by Muslim minorities.

When the Islamic revolution started in Iran, the Carter administration saw it as the confirmation of its assumption that only Islamists could muster enough popular support to provide an alternative to both the existing regime and the pro-Soviet leftist movements.

The Carter administration went out of its way to support the new regime in Tehran. A ban imposed on the sale of arms and materiel to Iran, imposed in 1978, was lifted, and a 1954 presidential "finding" by Dwight Eisenhower was dusted off to reaffirm Washington's commitment to defending Iran against Soviet or other threats.

Also to symbolize support for the mullahs, President Carter initially rejected a visa application for the exiled shah to travel to New York for medical treatment.

Just weeks after the mullahs' regime was formed, Brzezinski traveled to Morocco to meet Mehdi Bazargan, Ayatollah Khomeini's first prime minister. At the meeting, Brzezinski invited the new Iranian regime to enter into a strategic partnership with the United States. Bazargan, concerned that the Iranian left might bid for power against the still wobbly regime of the mullahs, was "ecstatic" about the American offer.

The embassy raid came just days after the Brzezinski-Bazargan meeting in Morocco and, by all accounts, took Khomeini by surprise. It is now clear that leftist groups opposed to rapprochement with the United States had inspired the raid.

Khomeini saw it as a leftist ploy to undermine his authority. He was also concerned about the possibility of the United States taking strong military and political action against his still fragile regime.

Deciding to hedge his bets, the ayatollah played a double game for several days, waiting to gauge the American reaction.

According to his late son Ahmad, who had been asked to coordinate with the embassy-raiders, the ayatollah feared "thunder and lightning" from Washington. But what came, instead, was a series of bland statements by Carter and his aides pleading for the release of the hostages on humanitarian grounds.

Carter's envoy to the United Nations, a certain Andrew Young, described Khomeini as "a 20th-century saint," and begged the ayatollah to show "magnanimity and compassion."

Carter went further by sending a letter to Khomeini.

Written in longhand, it was an appeal from "one believer to a man of God."

Carter's syrupy prose must have amused Khomeini, who preferred a minimalist style with such phrases as "we shall cut off America's hands."

As days passed, with the U.S. diplomats paraded in front of TV cameras blindfolded and threatened with execution, it became increasingly clear that there would be no "thunder and lightning" from Washington. By the end of the first week of the drama (which was to last for 444 days, ending as Ronald Reagan entered the White House), Khomeini's view of America had changed.

Ahmad Khomeini's memoirs echo the surprise that his father, the ayatollah, showed, as the Carter administration behaved "like a headless chicken."

What especially surprised Khomeini was that Cater and his aides, notably Secretary of State Cyrus Vance, rather than condemning the seizure and the treatment of the hostages as a barbarous act, appeared apologetic for unspecified mistakes supposedly committed by the United States and asked for forgiveness and magnanimity.

Once he had concluded that America would not take any meaningful action against his regime, Khomeini took over control of the hostage enterprise and used it to prop up his "anti-imperialist" credentials while outflanking the left.

The surprising show of weakness from Washington also encouraged the mullahs and the hostage-holders to come up with a fresh demand each day. Started as a revolutionary gesture, the episode soon led to a demand for the United States to capture and hand over the shah for trial. When signals came that Washington might actually consider doing so, other demands were advanced. The United States was asked to apologize to Muslim peoples everywhere and, in effect, change its foreign policy to please the ayatollah.

Matters worsened when a military mission to rescue the hostages ended in tragedy in the Iranian desert. The force dispatched by Carter fled under the cover of night, leaving behind the charred bodies of eight of their comrades.

In his memoirs, Ahmad nicely captures the mood of his father, who had expected the Americans to do "something serious," such as threatening to block Iran's oil exports or even firing a few missiles at the ayatollah's neighborhood.

But not only did none of that happen, the Carter administration was plunged into internal feuds as Vance resigned in protest of the rescue attempt.

It was then that Khomeini coined his notorious phrase, "America cannot do a damn thing."

He also ordered that the slogan "Death to America" be inscribed in all official buildings and vehicles. The U.S. flag was to be painted at the entrance of airports, railway stations, ministries, factories, schools, hotels and bazaars so that the faithful could trample it under their feet every day.

The slogan "America cannot do a damn thing" became the basis of all strategies worked out by Islamist militant groups, including those opposed to Khomeini.

That slogan was tested and proved right for almost a quarter of a century. Between Nov. 4, 1979, and 9/11, a total of 671 Americans were held hostage for varying lengths of time in several Muslim countries. Nearly 1,000 Americans were killed, including 241 Marines blown up while sleeping in Beirut in 1983.

For 22 years the United States, under presidents from both parties, behaved in exactly the way that Khomeini predicted. It took countless successive blows, including the 1993 bombing of the World Trade Center in New York, without decisive retaliation. That attitude invited, indeed encouraged, more attacks.


His new position on Iran is earily simular to his old position on Iran .In other words;a failed policy .

jackreade rated this answer Excellent or Above Average Answer

Question/Answer
Erewhon asked on 04/25/06 - "Growing" .......



Growing ... for those who did not believe that the criticism against Rummy was growing, here is proof:


Crisis building in White House over Iraq war
Apr. 25, 2006. 01:00 AM
RICHARD GWYN

A week ago, it was the generals. Now it's the colonels and majors and captains. Moreover, these officers are in uniform and have none of the security from retribution of the generals who had all already retired.

In a front-page story Sunday, The New York Times described an "extraordinary debate" now going on among younger American officers "in military academies, in the armed services staff colleges, and even in command posts and mess halls in Iraq."

This debate is about the war in Iraq, about the tactics and prospects of the American forces there, and, most particularly, about Defence Secretary Donald Rumsfeld, already the target of stinging criticism by a half dozen recently-retired senior generals, most of whom had served in Iraq.

The names of these junior officers have all been withheld by the Times. If ever identified, they would be court-martialed. So readers have to take it on faith that the paper has described their opinions accurately.

Richard Gwyn's column appears Tuesdays and Fridays. gwynR@sympatico.ca.

====

Growing ...

Monday, April 24, 2006

Younger army officers also want Rumsfeld to step down

By Khalid Hasan

WASHINGTON: The dump Rumsfeld movement is gathering steam, with younger officers in military academies, armed services staff colleges and even command posts and mess halls calling for the ouster of the defence secretary.

[...]

Simply put, the question is whether the focus should be, as Rumsfeld believes, on a lean high-tech force with an eye toward possible opponents like China, or on troop-heavy counterinsurgency missions more suited to hunting terrorists, with spies and boots on the ground.

Courtesy http://www.DailyTimes.com.pk

===

Growing ...

WASHINGTON The revolt by retired generals who publicly criticized Defense Secretary Donald Rumsfeld has opened an extraordinary debate among younger officers, in military academies, in the armed services' staff colleges and even in command posts and mess halls in Iraq.

Junior and midlevel officers are discussing whether the war plans for Iraq reflected unvarnished military advice, whether the retired generals should have spoken out, whether active-duty generals will feel free to state their views in private sessions with the civilian leaders and, most divisive of all, whether Rumsfeld should resign.

In recent weeks, military correspondents of The New York Times discussed these issues with dozens of younger officers and cadets in classrooms and with combat units in the field, as well as in informal conversations at the Pentagon and in e-mail exchanges and telephone calls.

To protect their careers, the officers were granted anonymity so they could speak frankly about the debates they have had and have heard. The stances that emerged are anything but uniform, although all seem colored by deep concern over the quality of civil-military relations and the way ahead in Iraq.

The discussions often flare with anger, particularly among many midlevel officers who have served in Iraq and Afghanistan and face the prospect of additional tours of duty.

"This is about the moral bankruptcy of general officers who lived through the Vietnam era yet refused to advise our civilian leadership properly," said an army major in the Special Forces who has served two combat tours. "I can only hope that my generation does better someday."

An army major who is an intelligence specialist said: "The history I will take away from this is that the current crop of generals failed to stand up and say, 'We cannot do this mission.' They confused the cultural can-do attitude with their responsibilities as leaders to delay the start of the war until we had an adequate force. I think the backlash against the general officers will be seen in the resignation of officers" who might otherwise have stayed in uniform for more years.

An army colonel enrolled in a Defense Department university said that an informal poll among his classmates indicated that about 25 percent believed that Rumsfeld should resign, while 75 percent believed that he should remain.

But of the second group, two-thirds thought he should acknowledge errors that were made and "show that he is not the intolerant and inflexible person some paint him to be," the colonel said.

Many officers who blame Rumsfeld are not faulting President George W. Bush - in contrast to the situation in the 1960s, when both President Lyndon Johnson and Defense Secretary Robert McNamara drew criticism over Vietnam from the officer corps. (McNamara, like Rumsfeld, was also resented from the outset for his attempts to reshape the military.)

But some are criticizing both Bush and Rumsfeld, along with the military leadership, like the army major in the Special Forces.

"I believe that a large number of officers hate Rumsfeld as much as I do, and would like to see him go," he said.

"The army, however, went gently into that good night of Iraq without saying a word," he added, summarizing conversations with other officers. "For that reason, most of us know that we have to share the burden of responsibility for this tragedy. And at the end of the day, it wasn't Rumsfeld who sent us to war, it was the president.

"Officers know better than anyone else that the buck stops at the top. I think we are too deep into this for Rumsfeld's resignation to mean much."

He added: "But this is all academic. Most officers would acknowledge that we cannot leave Iraq, regardless of their thoughts on the invasion. We destroyed the internal security of that state, so now we have to restore it. Otherwise, we will just return later, when it is even more terrible."

The debates are fueled by the desire to mete out blame for the situation in Iraq, a drawn-out war that has taken many military lives and has no clear end in sight.

A midgrade officer who has served two tours in Iraq said that a number of his cohorts were angered last month when Secretary of State Condoleezza Rice said that "tactical errors, a thousand of them, I am sure," had been made in Iraq.

"We have not lost a single tactical engagement on the ground in Iraq," the officer said, noting that the definition of tactical missions is specific movements against an enemy target. "The mistakes have all been at the strategic and political levels."

Many officers said that a crisis of leadership extended to serious questions about the top generals' commitment to sustain a seasoned officer corps that was being deployed on repeated tours to the long-term counterinsurgency operations in Iraq and Afghanistan while the rest of the government did not appear to be on the same wartime footing.

Said one army officer with experience in Iraq, "We are forced to develop innovative ways to convince, coerce and cajole officers to stay in, to support a war effort of national-level importance that is being done without a defensewide, governmentwide or nationwide commitment of resources."

Another army major who served in Iraq said a fresh round of debates about the future of the U.S. military had also broken out. The question is whether the focus should be, as Rumsfeld believes, on a lean high-tech force with an eye toward possible opponents like China, or on troop-heavy counterinsurgency missions more suited to hunting terrorists, with spies and boots on the ground.

"I feel conflicted by this debate, and I think a lot of my colleagues are also conflicted," said an army colonel completing a year of work at one of the military's advanced schools. He expressed discomfort at the public airing of criticism of Rumsfeld and the Iraq war planning by retired generals. But he said his classmates were also aware of how the Rumsfeld Pentagon quashed dissenting views that many argued were proved correct.

====

GROWING ...


BILL JACOBS

ALARM bells were ringing at Whitehall today after it emerged that United States Defence Secretary Donald Rumsfeld has drawn up plans to step up the fight against terrorism around the globe.

Pentagon leaks suggest he has approved an ambitious strategy to take the fight against dealers of death beyond recognised war zones such as Iraq and Afghanistan.

A key element is a significantly expanded role for the military - in particular US special forces such as the American army's Green Berets, immortalised by the John Wayne film of that name.

At a time when British generals are already concerned about the Hollywood "shoulder holster" attitude to peace-keeping and tackling insurgents in Iraq, they could be forgiven for letting a shiver run down their spines.

The 200-odd groups of elite Special Operations Troops dotted at US embassies around the world will now be entitled to go into action without the approval of the relevant ambassadors in a manner reminiscent of the famous rugby dictum of "getting your retaliation in first".

The big fear is that the US will try to draw the British SAS into these clandestine activities putting UK citizens and soldiers at greater risk. And with Tony Blair increasingly gung-ho as his time in 10 Downing Street draws to a close, the less aggressive members of his Cabinet, such as Foreign Secretary Jack Straw, must be worried.

Mr Rumsfeld is just one of a posse of US "hawks" on foreign policy and military action now on the inside track in the White House. He, Vice President Dick Cheney and former defence adviser Paul Wolfowitz were around the administration of George Bush's father when he took over as president from Ronald Reagan. They were considered peripheral and slightly mad figures, but George Bush junior brought them to the heart of US policy making and, since 9/11, they have played a considerable role in the military adventures in Afghanistan and Iraq.

The fear is that they have given the go-ahead for further expeditions. The biggest concern is that they have Iran in their sights. Now, there is an argument to say that it would be far easier to deal with the current extreme Iranian prime minister if Iraq had been left alone and Saddam Hussein was still in power. But, following the toppling of the Baghdad Butcher, any action to stop Tehran developing an atom bomb would set the Middle East alight.

With Hamas in charge of the Palestinian authority, the stakes are now fearsomely high. New Israeli Prime Minister Ehud Olmert is also in aggressive mood as he tries to prove his tough guy credentials against the Palestinian terrorists who regularly send suicide bombers into Israel's towns and cities.

Mr Blair in his monthly press conference yesterday was keen to stress that Iran was not Iraq - but then again neither was Iraq until President Bush got it into his head that ousting Saddam was the answer to international terrorism.

The Prime Minister was also keen to imply there were no divisions between him and Mr Straw, who described using nuclear-tipped or other such missiles to take out Iran's atomic plants as "nuts".

However, he was keen to stress that a tough message had to be sent out to the new regime and the ayatollahs that back them. The worrying thing is that we've been here before.

Over the Iraqi invasion Mr Blair was talking tough while Mr Straw desperately tried to stitch up a UN resolution to bring the rest of Europe and at least some of the Middle East on board for the invasion.

This was in the face of strong opposition from Mr Bush and guess who? Mr Cheney and Mr Rumsfeld.

===

From: The COUNCIL ON FOREIGN RELATIONS
A nonpartisan resource for information and analysis

Gelb: Rumsfelds Resignation Should Have Been Accepted a Year Ago

Interviewee: Leslie H. Gelb, President Emeritus and Board Senior Fellow
Interviewer: Bernard Gwertzman, Consulting Editor

April 19, 2006

Leslie H. GelbLeslie H. Gelb, president emeritus of CFR and a former Pentagon and State Department official in the Johnson and Carter administrations, says the public criticism of Secretary of Defense Donald M. Rumsfeld by some retired senior military officers is due to their unhappiness "that they didn't speak up earlier, while they were on the job."

"In good part, they were telling us the reason they didn't speak up, and the reason they think their colleagues didn't speak out against the Rumsfeld decisions, is that Rumsfeld was intimidating them and making it impossible for them to say their piece," Gelb, a senior board fellow, says. He says that it would have been better for the country if Rumsfeld's resignation had been accepted a year ago.

"He had become a serious lightning rod; it was hard both for Democrats and a number of Republicans to work with him; and, inside the Pentagon, the poisonous atmosphere had begun to develop. And here we are, in two wars in Iraq and Afghanistan, and you can't have a secretary of defense under so much fire [still] being able to do his job at the same time."

You and I have been observing developments in the political-military sphere for many years. I can't remember anything similar to this current "revolt of the retired generals" against Defense Secretary Rumsfeld. Can you?

The history of anything like this is quite short. Back in the Truman administration there was something called the "revolt of the admirals" but that was essentially over the Navy budget. Then you have to jump forward many, many years to the Vietnam War. There was an enormous amount of grumbling and criticism by the military of the civilian leadership, particularly of Defense Secretary Robert McNamara [who served under Presidents John F. Kennedy and Lyndon B. Johnson].

What was the criticism about?

The criticism was over McNamara's strategy and the White House's strategy of how to fight the war. This is the exact same thing that's come up now with respect to Iraq. McNamara, McGeorge Bundywho was national security adviser for Presidents Kennedy and Johnson [1961-66]and others wanted to follow a gradual approach to the war, signaling to Hanoi with each new increment of American power that we could do more down the line. They said do it gradually, both as a punishment and a warning of more to come.

This was also, as I recall, to keep China out of the war. Right?

Well, the main thing to keep China out of the war was not to invade North Vietnam itself, or to bomb Haiphong Harbor, where a lot of Chinese equipment came into North Vietnam. The Air Force and the Navy were criticizing McNamara very harshlybut privatelyover that strategy. They wanted a more all-out air bombing campaign that wouldn't allow the North Vietnamese time to adjust. As unhappy as they wereand they were extremely unhappythey never went public with it by name, only by leak.

You mean they didn't publicly speak out, but they leaked their dissatisfaction to the press?

That's right. No serving or retired generals stood up to criticize McNamara's handling of the war, even though they were doing that privately. Essentially they made their unhappiness known through press leaks.

Now you were in the Pentagon in those days, among other things putting together what later became known as the "Pentagon Papers." Do you think the criticism was warranted? If there had been a heavier bombing would it have made much of a difference?

I don't know that it would have made a difference in the ultimate outcome of the war, because I think the North Vietnamese and their allies in the South, the Vietcong, were going to fight as long as necessary to drive out any foreign power. The force of nationalism was that strong. But I think it would have made a difference in the military campaign. I think what the military was saying was militarily correctthat we could have done much more damage and made it much more difficult for the North Vietnamese to adjust with a more all-out bombing campaign.

Now, of course we did mine Haiphong harbor, but that was in the Nixon administration when most of the U.S. troops had already left.

That's correct. The last incident I can think of took place in the Carter administration, where one of the senior army generals in Iraq, General John Singlaub, publicly attacked President Jimmy Carter while serving, over Carter's decision to withdraw some U.S. troops from South Korea. And Carter fired him. And as far as I can remember, that's it.

What do you think is the impetus driving these generals?

Well, all of them either served in the war in Iraq or were intimately involved in the planning of that war. So these were people who all knew what they were talking about. Now what's the motivation? I think, in part, they all are unhappy that they didn't speak up earlier while they were on the job. In good part, they were telling us the reason they didn't speak up, and the reason they think their colleagues didn't speak out against the Rumsfeld decisions, is that Rumsfeld was intimidating them and making it impossible for them to say their piece. And while none of them pointed directly to the fate of General Eric Shinseki, the chief of staff of the Army, they had that very much in mind. You'll remember, Shinseki is the one who told Congress in early 2003 it would take at least 300,000 troops to safely garrison Iraq after a military victory. But he got fired and nobody came to his defense.

And Secretary of the Army Thomas White was fired for backing him, right?

That's exactly right. People have forgotten that, but Army Secretary White did the same thing. So the others looked at it, and said, well, it just doesn't work, so they'd go along and try to adjust to the system. And additionally, as you know, these guys do not like to criticize the civilian sphere, because they believe it would have a terrible effect on the morale of the serving troops.

So now you have this incredible situation where active duty generals are being called on to back the secretary of defense against their former colleagues.

Yes. This is a very tricky situation, because people go to Iraq, like I did a year ago, governors and senators, and they talk to these people, these generals. And by and large, the generals say we're making progress, and you don't hear much criticism from them. But those of us who know many of them personally, over the years, hear two stories: the official story, and then a much more pessimistic one privately.

So it causes great confusion in the minds of the American people, because the generals are saying very positive things for publication, and a number of them are being much more cautious, and even critical, in private.

I was fascinated in reading Cobra II, the new book by Michael R. Gordon, the chief military correspondent of the New York Times, and retired Marine Lt. Gen. Bernard E. Trainor, where they quote a number of the generals or colonels who are critical of the planning for the war on the record, which is unusual, I thought.

Yes, very. It shows the degree of their frustration, because all the military planning over the course of a dozen years for Iraqand that's how long the planning has been going on thereshowed you need at least 300,000 troops on the ground in order to provide basic security. And almost all of it also showed that you needed to keep as much of the Baathist-dominated army intact, after military victory, to support those 300,000 troops. But what we had was about 125,000, and virtually no Iraqi army. So, you know, this was at leastat least400,000 or 500,000 troops short of what the planning said was necessary.

Put yourself in the mind of Rumsfeld. Now, you've dealt with Rumsfeld for years.

Yes, decades.

Why was he so stubborn on keeping the troop level on such a minimum level?

You know, there are two explanations, a good one and a bad one. I don't know which is true, or if both are true. The good one is, he really believed he could do the job with 125,000 troops or so, and he could keep the necessary security thereafter, and essentially get out. He believed his own propaganda about Iraqis welcoming us with open arms, and that we wouldn't need to fight after we got rid of Saddam. So, you know, that's the benign, positive explanation.

The bad explanation is that he and [former Deputy Defense Secretary Paul] Wolfowitz and others feared that if the American people were told that you needed 300,000 plus troops for an extended period in Iraq, in order to do the job, that there would be opposition to the war. And they were interested in making sure that opposition was de minimus, and they were willing to take the risk on the ground in Iraq.

It's ironic, because at the time the war was launched, I don't think the public knew the difference between 125,000 or 325,000, so long as the job got done.

Well, I'm not sure, and I don't think that was their opinion. They were worried about public reaction to a large-scale and extended troop involvement. The same thing on paying for the war. You'll remember that Wolfowitz said, so far as reconstruction was concerned, it would be paid for out of Iraqi oil revenues. Well, at that very time he was saying that, all of us who had been involved in post-war planningand we had a study underway at the Council at the timeknew from the oil companies that that was inconceivable, and that oil production would go down below pre-war levels, and that it would be many years before production would come up to pre-war levels, let alone be able to pay for economic reconstruction. If you remember, Wolfowitz said it might cost us $1.5 billion. It's over $200 billion now.

Well, it's quite clear that the president is committed not to let Rumsfeld resign. Do you think he would have been better served with Rumsfeld out of there?

I think both politically and in terms of the decision-making process, it probably would have been best to accept his resignation a year ago. He had become a serious lightning rod; it was hard both for Democrats and a number of Republicans to work with him; and inside the Pentagon, the poisonous atmosphere had begun to develop. And here we are, we're in two wars in Iraq and Afghanistan, and you can't have a secretary of defense under so much fire [still] being able to do his job at the same time.

What is your opinion of him, putting aside the criticism? If you were writing an essay about Rumsfeld, what would you say? Has be been a lousy secretary of defense?

I think he did a pretty good job with the continuing process of reforming the military. That is, he was pushing them in the correct direction. It wasn't done in a very cooperative spirit, but he did push that process. I'm not sure if he correctly tackled the strategic problems that are facing the United States today. If you look at his statements that pertained to the budget, they tend to be more ideological than they are serious policy statements. And, you know, in terms of the handling of both the Iraq and Afghan wars, I think history will showand the histories that have been written already showthat there were more minuses, far more minuses, than plusses.

Do you think, really in hindsight, if there had been, say, 350,000 troops in Iraq, that that would have made a difference?

I know it's very contentious, whether things would have worked out differently even if we had the proper number of troops. But I do believe that if we had had over 300,000 troops there, within the first two months after getting rid of Saddam, and had kept the Iraqi army intact except for the senior officer corps, that we would have been able to establish security, get economic projects underway and completed, and that we would have been able to pass that country on to the Iraqis, in relative peace, within two years or so. I do believe that.

Now in the book My Year in Iraq by L. Paul Bremer, who was head of the Coalition Provisional Authority (CPA), says the Iraqi army had already disintegrated and his decree disbanding it officially was just a formality.

He can state anything he wants. If the word had gone out that there were junior officers reconstituting the force, that they were going to get paid $200 a month, they would have been back there.

Well, in fact the book by Trainor and Gordon states the U.S. military was already recruiting Iraqi officers when they were ordered to stop by Bremer.

They didn't have a hard time getting volunteers.

Right. So how's this thing going to end?

I think the generals, on balance, did a courageous thing. They spoke up and they broke that wall of silence that had been protecting President Bush and Secretary Rumsfeld, who were making it look as if the military were really happy with their decisions. That had always been their defense. They said they gave the military whatever the military wanted. Well, that just wasn't correct. So that story has been largely shattered. And even though you haven't had more than six generals come forward to support it, and even though you haven't had resignations of active duty generals, I think the public understands that these people who have spoken out represent only the tip of the iceberg. It will make them think even harder and more critically about where the president is leading us in Iraq.

===


IRAQ: NEW HOPE, SAME VIOLENCE

After months of political infighting, Iraq has a democratically elected prime-minister designate. But it also has an unbowed insurgency, sectarian bloodshed, a moribund economy, and increasingly, a superpower-led [US led] occupying army that seems unsure of what to try next.

====

And so it grows ...While this will certainly be distasteful to some, it would be folly to ignore it!





tomder55 answered on 04/25/06:

Here is the link to the NY Slimes article. Note the twisting and turning of anonymous sources as reported in the Slimes :

"One Army colonel enrolled in a Defense Department university said an informal poll among his classmates indicated that about 25 percent believed that Mr. Rumsfeld should resign, and 75 percent believed that he should remain."

Here is how the quote should read :

"One Army colonel(Unnamed) enrolled in a Defense Department university (Un-specified)said an informal poll (Read water cooler talk) among his classmates indicated that about 25 percent believed that Mr. Rumsfeld should resign, and 75 percent(Read I talked to four guys and one says he is getting out because of Rumsfeld) believed that he should remain."

This just in - Four un-named JrROTC cadets at a Kansas high school demand Rumsfeld's resignation.


Meanwhile Hillary Rotten is going to guarantee that this non-story contines to get play . But the big story ....the treason of a major CIA player ,Mary McCarthy ,has been buried .

Erewhon rated this answer Excellent or Above Average Answer
Itsdb rated this answer Excellent or Above Average Answer
drgade rated this answer Excellent or Above Average Answer

Question/Answer
Erewhon asked on 04/23/06 - "The Bush Era is at an end" --- The Scotsman



GEORGE Bush is not yet halfway through his second presidential term, but power is visibly draining from the White House. Republicans who were once his cheerleaders now barely mention his name: elections are coming, and it is time for them to ditch the president.

The gains from Bush's spectacular presidential victory just 18 months ago have entirely vanished. Like Tony Blair, he has seen his personal authority buried in the quagmire of Iraq and found the political world waiting impatiently for his departure.

But in Britain, we at least know that the power leaving 10 Downing Street is going towards Gordon Brown. In America, it is obvious that Bush is the loser - but it is by no means clear to whom power is gravitating towards.

The Democrats, as a party, are optimistic. The Iraq war is finally catching up with Bush and his party: for the first time since the 11 September attacks, the Republicans are now seen as the weaker of the two parties on national security.

A remarkable 30% of Americans now want immediate withdrawal from Iraq, twice the level of two years ago. This is not Democrat policy, but it indicates irritation with Bush and is an encouraging sign for this leaderless party ahead of the November elections.

So the Democrats are planning a summer of patriotic protest. One leaked battle plan shows they will lay on anti-Bush rallies using serving soldiers outside military barracks. The Republicans' only defensive strategy is to distance themselves from Bush.

Meanwhile, the Hillary Clinton machine grows stronger. The New York senator has assembled a formidable team which already looks superior to the White House, drawing on reconvened heavyweights from the Bill Clinton era.

Names like Ann Lewis, a renowned strategist, pollster Mark Penn and Terry McAuliffe, a fundraiser and likely UK ambassador under Mrs Clinton, are becoming central figures around the "Friends of Hillary" group which is preparing for power.

She is a divisive figure, hated in the Bible Belt - but then Bush was hated just as much in the urban areas. And she has made inroads into rural areas of New York State which have similar dynamics to the dozen or so swing states which decide US elections.

Every speech Mrs Clinton makes has one eye on wavering Republican voters, and there are few in Washington who would not describe her as the overwhelming favourite to be the Democrats' candidate for president. There is less agreement on who she would face.

Journalists fantasise that it will be Condoleezza Rice. To have an all-woman presidency race is such an alluring prospect that some have convinced themselves it is true - but the smarter money says Condi will run as a vice-president, if she stands at all.

The reason, often forgotten in Britain, is that she is not a politician and has never stood for elected office. She is an academic who became an adviser and who was then appointed Secretary of State - when she denies ambitions for the presidency, she is being sincere.

While Rudy Giuliani is the most popular figure, the former New York mayor is pro-gay rights and pro-choice on abortion. He may well win more votes in America, but may have trouble getting the Republican nomination for presidency: if he tries.

So the favourite to fight Hillary is Senator John McCain, who would stand as a 72-year-old Vietnam veteran and a far less divisive figure than Bush. He polls well amongst Democrats and has spent much time joining forces with them to forge legislation.

When he ran for president in 2000 he was easily eclipsed by Bush - but as he remarked on Friday, "six years ago, nobody knew me". Now, he is popular on both sides and speaks about Iraq with an eloquence that Bush's supporters wish the president had.

McCain's liability on one issue may still explode on to the political scene, however: immigration. Bush wants to grant amnesty to the 11 million illegal immigrants in America, mostly Hispanics, believing they will repay the Republicans with votes.

Some 70% of Americans are against it. Conservatives believe lawbreakers are being rewarded, and low-paid American workers fear Bush is colluding with employers to keep wages down by giving citizenship to dirt-poor Latinos.

It says much about Bush's lack of power that it took McCain to deliver the legislation in a compromise the senator negotiated last week. If McCain's plan succeeds, it will make him a target for protest amongst those whose votes he will depend upon for the presidential nomination.

The dark horse Republican candidate is Mitt Romney, governor of Massachusetts - a Republican elected to run one of the most left-wing states in America. He is a Mormon, which many Bushies consider apostasy, but has a proven ability to win votes.

On his fundraising tours (lately to McCain's home state of Arizona) he has performed well on the stump - cracking jokes about his religion and drawing laughter and admiration from the type of people who would, in another era, have burnt him at the stake.

Impressively, Romney has brokered a scheme for universal health provision in Massachusetts using an innovative mix of personal payments, business charges and state subsidy. Even Mrs Clinton backs the plan: it could be a template for the whole of America.

So the scene is set for a McCain v Romney battle for the Republican nomination - taking on Mrs Clinton and her formidable team. Bush is left an increasingly lonely and irrelevant figure, from whom aspirational Republican politicians recoil.

You can see it on the cars of Washington. The "Kerry-Edwards" stickers are hopelessly outdated, yet are still stuck to bumpers because their owners have not been bothered to remove them. But the name "Bush" has been whipped off every vehicle in Washington.

The Democrats sense an end to their long wilderness.

If they win back the chamber in November, they will seek to humiliate Bush and exact revenge for the impeachment of Bill Clinton, who could soon be back at the White House, in the First Lady's wing.

Power seldom lives at one address in Washington. It slides between Congress, the Supreme Court and the White House - where it had stayed for six years. But no more. The presidency may have two more years to run, but the Bush era is already over.

===

Agree / disagree? Why?

tomder55 answered on 04/24/06:

There is alot here .

I think bigger than Iraq ;Bush's polling numbers are tied to oil prices ;something completely out of his control.

As Sharansky noted ,Bush is willing to fight for something even if he stands against popular opinion. He lost alot of his political capital pushing for his needed Social Security reforms . He also bucked popular opinion with his stand on the DPW issue ,and the position he takes on illegal immigrants ,both of whick has hurt him with his base. The Republicans however would be making a terrible mistake if they distance themselves from Bush in the coming campaign.... In fact ;that would be the thing that would guarantee a Democrat takeover of Congress.

I have no doubt that Evita Clinton will win the Democrat nomination . Madame Defarge is becoming quite adept at the Clintonista triangulation . Note how this weekend she called for a fence on the Mexican border ....Amazing in light of the fact that she recently made claims that by enforcing tougher immigration laws, we are in effect transgressing against Jesus Christ. She cannot win the general election in my view so the Democrats would be wise to reconsider before it is too late . Her war chest is enormous ,and it doesn't look like she needs to spend a dime to win reelection for her Senate seat ..such is the pathetic state of the Republican party in N.Y.

I would love to see Condi run ,but I have to take her at her word when she says she has no interest in the job. Last I heard ,she would like to get out of governement and be commissioner of the NFL .She will be a success whatever she does.

I have very mixed feelings about McCain . I would have a hard time giving him my full support . There are things about McCain's past that have been swept under the rug that I am sure the Dems. would love to bring up .I like Rudy Guilliani alot but I have to question if he has that national appeal . I do not think he has gone through the scrutiny of a national campaign so there are some parts of his personal life that have not gone under intense scrutiny . I do not know enough yet about Romney except that if he and Hillary are the finalists then national health care will be a big topic in the election .

I do not agree that these are the only viable Republican candidates . George Allen is impressive and I think he will have a large appeal to Republican primary voters. I also think that among Democrats Al Gore still has appeal (God knows why ... But his version of 'Scary Movie' is due out this month and it is sure to be a hit) .


I do agree that if the Democrats win the House they will squander the next 2 years on vendetta which will almost guarantee a Republican win in 2008. In fact ;beside "KILL BUSH vol.2" ..I do not know of any other serious agenda the Democrats have .

Erewhon rated this answer Excellent or Above Average Answer
Itsdb rated this answer Excellent or Above Average Answer

Question/Answer
paraclete asked on 04/24/06 - reality imitating art -Delta Force revisited?

US deploys elite forces worldwide to spearhead terrorism battle

April 24, 2006

WASHINGTON: The US Defence Secretary, Donald Rumsfeld, has approved the military's most ambitious plan to fight terrorism around the world and retaliate more rapidly and decisively in the case of a terrorist attack on the US, according to defence officials.

The long-awaited campaign plan for the global war on terrorism, as well as two subordinate plans also approved within the past month by Mr Rumsfeld, are considered the Pentagon's highest priority, according to officials who spoke on the condition of anonymity.

The plans envisage a greatly expanded role for the military - and in particular a growing force of elite special operations troops - to combat terrorism outside of war zones such as Iraq and Afghanistan.

Developed over about three years by the Special Operations Command, they reflect a beefing up of the Pentagon's involvement in domains traditionally handled by the CIA and State Department.

For example, the command has dispatched small teams of special operations troops to US embassies in more than a dozen countries in the Middle East, Asia and Latin America, where they do operational planning and intelligence.

And in a subtle but important shift, the Pentagon gained the leeway to inform - rather than gain the approval of - the US ambassador before conducting operations in a foreign country.

The plans cover a wide range of military activities - from man-hunting and intelligence gathering on terrorist networks, to attacks on terrorist training camps and recruiting efforts, to partnering with foreign militaries to eliminate terrorist sanctuaries.

Together, they amount to a road map to conduct what the Pentagon now envisages as a "long war" against terrorism.

The Pentagon declined to comment on the plans.

■ The CIA has sacked a senior intelligence officer for leaking information about overseas prisons operated by the agency. A lie detector test on Mary McCarthy following a leak last November showed the possibility of deception, government officials said, and she later admitted speaking to reporters. The agency fired Ms McCarthy last Thursday.

The Washington Post

tomder55 answered on 04/24/06:

are you refering to the Delta Force disaster during the 1980 Iran hostage rescue attempt or possibly the Black Hawk Down incident (which was mostly an Army Ranger fight ) ?

They have had many more successful operations that do not get the same press. Forward deploying them is a great idea .A few examples : They were instrumental in the offensive against the Taliban in Afghanistan. They played important roles during the invasion of Iraq in 2003. [ They entered Baghdad in advance, along with SEALs from DEVGRU, building networks of informants while eavesdropping on and sabotaging Iraqi communication lines.In his book "Killing Pablo" Mark Bowden suggests that a Delta Force sniper assassinated Colombian drug lord Pablo Escobar.

paraclete rated this answer Excellent or Above Average Answer

Question/Answer
paraclete asked on 04/23/06 - The politics of Islam

Wolves in sheep's clothing on an extremist Islamic mission

By Miranda Devine
April 23, 2006

There is a new wave of sophisticated, articulate Islamic fundamentalists trying to spread the word among moderate Muslims in Sydney. Young men, wearing regular clothes, with neatly trimmed beards, broad Australian accents and fluent in Arabic, they appear to be fully assimilated, second-generation Australians.

But they belong to a political group called Hizb ut-Tahrir (Party of Liberation) that calls for the creation of a global Islamic state, or caliphate, under strict sharia law.

The message from these young men is one of division, non-assimilation and rejection of the values of the "kafir" - non-Muslims.

At a public lecture at Bankstown Town Hall earlier this month, Hizb ut-Tahrir organiser Soadad Doureihi, his brother Wassim, and Usman Badar, president of Sydney University's Muslim Student Association in 2005, outlined their utopian goal of the ultimate overthrow of Western democracies.

"Islam can never coexist one under the other or one within the other," Soadad told the crowd. "When the state is established, when people see the mercy of Islam they embrace Islam in droves."

The April 8 lecture, to about 200 men and 50 women, was titled "Should Muslims Subscribe to Australian Values?"

Banned in Britain, Germany, Holland, Russia, and much of the Muslim world, Hizb ut-Tahrir (HT) has been invited to speak at Sydney Boys High at least twice, and often addresses students at Sydney University.

Borrowing its methodology and ideology from Marxist-Leninist groups, HT calls itself a political party which works to "change the situation of the corrupt society so that it is transformed into an Islamic society", its website says.

It opposes integration and assimilation of Muslims into Australian society.

Wassim told the Bankstown crowd: "The pushing to integration and assimilation is to get us to think and believe and feel in a certain way that Islam will not condone.

"On the collective level everyone accepts you have to have one set of laws and no Muslim in this country is demanding today the implementation of sharia law.

"In this country, yes, we believe this is the best way forward but . . . our current struggle is the implementation of Islamic law in the Muslim world and that will serve as a model for the rest of humanity. [But] if governments want to interfere in the individual, personal affairs of any citizen, they are going to create the conditions of civil unrest and chaos like in France."

Soadad had a message for youth: "They must be aware of the plot of the kafir, the plot of the Western society to enforce on them a palatable Islam . . . Secularism is a clear assault on the fundamental belief of a Muslim. Democracy is a clear assault on the fundamental belief of a Muslim also."

HT says it advocates non-violence, and yet, terrorism expert Rohan Gunaratna, from Nanyang Technological University in Singapore, told a conference in 2004, "key members of the al-Qaeda organisation [such as Abu Musab al-Zarqawi] formerly belonged to the HT organisation . . .

"The upper echelons of organisations of key interest to us, operating at a violent, extremist, radical level, consist of former members of HT."

In Australia, HT's threat is its anti-integration message.

An audience member in Bankstown asked: "The reality is many of us live in Australia as citizens. We or our parents and families have accepted this citizenship with the full knowledge of Australia's social construction and her values. Can we not as Muslims hold these Australian values [while] keeping our Islam intact?"

Badar, a graduate of Malek Fahd Islamic High School in Chullora, who was such a good student he appeared on the 2002 HSC all-rounders list, answered: "It comes back to the theory that Western values, their opposition, the conflict is so clear, so stark there is no middle ground.

"How do you come to middle ground on whether sovereignty belongs to the people or to Allah? You can't.

"Yes, our parents came here. I wouldn't say they were fully aware of the Australian values and systems, way of life and so on . . . But what's more important is why did they come here? What were they running away from? Was the country in which they lived not providing for them? What was the cause of the conditions in that country?

"They were running away from the very same values . . . If you are saying they came here so we should accept or follow those values, there's a clear contradiction. The simple matter of fact is there is no middle ground."

No middle ground. Hizb ut-Tahrir is a fringe group, rejected by most Australian Muslim leaders. But its message is alluring to the disenfranchised. Is the answer to ban it? Wassim says the more the group is attacked, the more it grows. "The more we come under pressure the more we return closer to Islam."

Video of the lecture is at http://www.risala.org.

The key statement in all of this is the belief that Democracy is a clear assault on the fundamental belief of a Muslim, That being said how can Iraq become a democracy, we have already seen their inability to effectively form a government?

tomder55 answered on 04/23/06:

Is Islam compatible with democracy?
It can be. Millions of the world's 1.4 billion Muslims live in democracies
One of the worlds largest democracies is also the worlds largest Muslim nations ... Indonesia.

Bangladesh, Mali, and Senegal are also Muslim nations that are also democracies. If someone said in 1940 that Japan would be a flourishing ,prosperous democracy in less than 50 years no one would've believed it possible. Tradition and religion they would say would make that impossible.They would've been wrong. Your mistake is in assuming that Hizb ut-Tahrir represents the will of the majority of Islam. We are betting that radicals like that are not .

paraclete rated this answer Excellent or Above Average Answer

Question/Answer
Erewhon asked on 04/22/06 - Bush regrets free speech - apologises to Hu

Bush apologizes to Hu for protester
In White House meeting, leaders pledge to deepen cooperation

Friday, April 21, 2006; Posted: 2:07 a.m. EDT (06:07 GMT)

WASHINGTON (CNN) -- U.S. President George W. Bush and his Chinese counterpart Hu Jintao made little headway on trade after a White House ceremony, which was disrupted by a lone heckler and the misidentification of China's anthem.

Bush later expressed regret to Hu over the heckling, a senior Bush official said.

===

Does it see at all weird to you that Bush asks Hu to expand human rights in China and then apologises to Hu because a Chinese woman uses American human right of free speech to protest religious persecution in China?

tomder55 answered on 04/22/06:

It's a difficult thing to deal with China. Note the confusion about protocol ,wheter to have a State Dinner or a luncheon. (lol)

China's strong military and economy makes it impossible and undesirable to alienate but the supression of freedoms cannot be ignored .

Bush in his dealings with China has shown some of the nuance of policy that Kerry et.al. claimed he did not possess.He knows that trade is beneficial and has resisted the protectionist bleatings from irresponsible pols. like Chucky Shumer .At the same time he has placed sanctions on some Chinese firms that violate proliferation rules and is making headway on the issues of intellectual property . He has also done a supurb job at strengthening alliances with nations in the Pacific Rim and other regional countries like India ,and Australia .
Basically he is on the right tract .

Dr. Wang Wanyi ;according to NBC's David Gregory (no big Bush fan )provided a "fitting backdrop to a strong message the president sent on human rights in China." Gregory cited Bush's statement that "China can grow even more successful by allowing the Chinese people the freedom to assemble, to speak freely, and to worship." .

Now think about it ;Bush's critics have complained bitterly in the past that Bush and co. are very good at crowd control ,and when he is in the area protesters are shuffled off to free speech zones where presumably they won't be heard.

Assume that these critics are correct. Is it likely that a reporter for a Falun Gong backed Newspaper could've made it inside the White House compound without Bush's secret service knowing about it ...knowing it was likely that they would attempt to disrupt the proceedings ,since Falun Gong protesters had been visible at every stop during Hu's visit ? More likely ,Bush is pleased at the disruption .

Erewhon rated this answer Excellent or Above Average Answer
Mathatmacoat rated this answer Excellent or Above Average Answer
paraclete rated this answer Excellent or Above Average Answer

Question/Answer
Itsdb asked on 04/21/06 - The UN is irrelevent - and insane.

Disarmament board chooses Iranian

By NATHAN GUTTMAN
WASHINGTON

As the international community looks into ways of stopping Iran from developing nuclear weapons, an Iranian representative was elected to be the vice chair of the UN's disarmament commission.

The commission, which began its annual conference last week, was established by the UN General Assembly in the early 50's in order to promote disarmament and to review international treaties dealing with nuclear energy.

The commission does not have any authority to enforce its decisions and is not connected to the International Atomic Energy Agency, which is the international nuclear watchdog that is now investigating Iran's nuclear program and is expected to report to the UN Security Council by the end of the month.

The new Iranian vice chair, Mehdi Danesh-Yazdi, who is also the country's ambassador to the UN, said last week that Iran will cooperate with the IAEA and will "seek an acceptable venue for holding transparent talks with interested parties." The ambassador, who is one of three vice-chairpersons, said during the meeting of the commission that Israel's nuclear stockpile is among "the major sources of concern with regard to global peace and security."

The election of Iran to the senior post in the disarmament commission drew sharp criticism from US lawmakers and from Jewish organizations that are fighting against a nuclear Iran.

Representative Ileana Ros-Lehtinen (R-FL), who heads the House subcommittee on the Middle East issued a statement in which she compared the decision to "appointing a serial killer to serve as a juror in a murder trial."

Ros-Lehtinen added that choosing Iran to be the vice chair of the commission proves that the UN and the international community are "ineffective in preventing Iran from achieving nuclear capabilities."

The American Jewish Congress' chairman Jack Rosen said that the vote in the disarmament committee amounts to "a rude slap in the face of the International Atomic Energy Agency and the UN Security Council."

~~~~~~~~~~~~~~~~~~~~~~~~~~~~~~~~~~~~~~~~~~~~~~~~~~~~~~

Is that insane or what?

tomder55 answered on 04/21/06:

Hey ! Did you hear that PETA has chosen the Big Texan to promote vegetarianism .....no really .....

Glad to see that the U.N. is finally responded to the Iranian nuclear threat.It's like putting a serial killer on a murder jury ;it's like putting Michael Jackson on the staff of a Nursery School ....or as Sen. Norm Coleman put it ""Having the Iranians serve on this commission is like asking the fox to guard the hens, and will only ensure its ineffectiveness."

The election of Iran to the Vice Chair of the U.N. Disarmament Commission was immediately followed by yet more threats to destroy Israel by Iran's President Ahmadenijad.But the UN Commission will continue to claim that it is Israeli nukes that are destabilizing .

I still don't see a UN condemnation to the homicide bombing that killed 9 in Tel Aviv but they had no problem debating a motion to denounce Israel on Passover.


Itsdb rated this answer Excellent or Above Average Answer

Question/Answer
triumvirate asked on 04/20/06 - The Prez

Who is your favorite US President and why? List some examples, and compare and contrast with other Presidents.

tomder55 answered on 04/21/06:

This is my answer in my FAQs

Let him who looks for a monument to Washington look around the United States. Your freedom, your independence, your national power, your prosperity, and your prodigious growth are a monument to him. ~Louis Kossuth



I could make an equally compelling case for 3 Presidents ;Washington,Lincoln and FDR .But because of his position as first President of the country under the Constitution ,and the many precidents he set,I have to give the nod to Washington.

Washington was the most revered and influential man in the United States. A lesser person might have used his enormous power to establish a military dictatorship or to become king. Washington sternly suppressed all such attempts on his behalf by his officers .He never ceased to work for the union of the states under a strong central government during the years of the Articles of Confederation. He was a leading influence in persuading the states to participate in the Constitutional Convention, over which he presided, and he used his immense prestige to help gain ratification of the Constitution. The vast powers of the presidency, as one delegate to the Constitutional Convention wrote, would not have been made as great "had not many of the members cast their eyes towards General Washington as president; and shaped their ideas of the powers to be given to a president, by their opinions of his virtue."(Pierce Butler, letter to Weedon Butler, May 5, 1778)
Washington reluctantly accepted the Presidency of the United States. Probably no other man could have succeeded in welding the states into a lasting union. Washington fully understood the significance of the Presidency. "I walk on untrodden ground," he said. "There is scarcely any part of my conduct which may not hereafter be drawn in precedent." After eight years in office, Washington laid down the guidelines for future Presidents. As our first President, he set the precedents that define what it means to be a constitutional executive: strong and energetic, aware of the limits of authority but guarding the prerogatives of office. Washington not only rejected offers to make him king, but was one of the first leaders in world history to relinquish power voluntarily. His peaceful transfer of the presidency to John Adams in 1797 inaugurated one of America's greatest democratic traditions.




Here are some of the accomplishments during Washington's terms:

Mint Act (1792) - established a mint for the printing and coining of U.S. currency

Alexander Hamilton's Financial Program (1790) - a financial plan designed to extinguish some of the debts left for Washington by the Congress of the Articles of the Confederation.

The Bill of Rights (1791) - a set of 10 amendments to the Constitution, guaranteeing American's basic liberties, was passed.

First Bank of the U.S. chartered (1791)

Washington's Proclamation of Neutrality (1793) - stated that the U.S. shall be impartial in dealings with warring nations. Created in response to a general war of England, Spain, Austria, and Prussia versus the new French Republic.

Jay Treaty (1794) - a treaty with England that would regulate trade between America and Britain. The treaty also said that British troops would have to give up their frontier forts as of 1796

Pinkney Treaty with Spain (1795) - a treaty that would open up trade in the Mississippi River

The suppression of the Whisky Rebellion because during the course of this conflict, the government proved that it could enforce law when it needed to. The actions of the government during the Whisky Rebellion led the way for present-day policemen to enforce the laws of the U.S.

Even his farewell address
http://earlyamerica.com/earlyamerica/milestones/farewell/text.html

which ranks right up there along with the Declaration of Independence and the Constitution as one of the greates founding documents ;is best remembered for its counsel about international affairs: Washington recommended commercial relations with other nations but as few political entanglements as possible.

Often overlooked is his remider of the need to uphold the Constitution as our strongest check against tyranny and the best bulwark of our freedom. He warns us to guard against oppositions to lawful authority and those that seek to circumvent the rule of law;he warns about the excessive partisanship ; he wanted liberty to be the objective of our international relations and commerce;to encourage morality and religion.

First in war, first in peace, and first in the hearts of his countrymen, he was second to none in humble and enduring scenes of private life. Pious, just humane, temperate, and sincere; uniform dignified, and commanding; his example was as edifying to all around him as were the effects of that example lasting correct throughout, vice shuddered in his presence and virtue always felt his fostering hand. The purity of his private charter gave effulgence to his public virtues. Such was the man for whom our nation morns (John Marshall, official eulogy of George Washington, delivered by Richard Henry Lee, December 26, 1799)

Washington's is the mightiest name of earth - long since mightiest in the cause of civil liberty; still mightiest in moral reformation. On that name no eulogy is expected. It cannot be. To add brightness to the sun, or glory to the name of Washington, is alike impossible. Let none attempt it. In solemn awe pronounce the name, and in its naked deathless splendor leave it shining on. ~Abraham Lincoln

Itsdb rated this answer Excellent or Above Average Answer
purplewings rated this answer Excellent or Above Average Answer
triumvirate rated this answer Excellent or Above Average Answer

Question/Answer
arcura asked on 04/17/06 - Would you have believed this a few years ago?

Thursday, April 13, 2006 10:53 p.m. EDT
Bill Clinton Aided Iran in Quest for Nukes
Then-President Clinton, the CIA deliberately gave Iranian physicists blueprints for part of a nuclear bomb that likely helped Tehran advance its nuclear weapons development program.
The allegation, detailed recently in the book "State of War," by New York Times reporter James Risen, comes as the Iranian nuclear crisis turns white hot, with Iranian President Mahmoud Ahmadinejad boasting ominously on Wednesday that his nation has joined the world's nuclear club.
Reports Risen: "It's not clear who originally came up with the idea [to give Tehran nuclear blueprints], but the plan was first approved by Clinton."
Beginning in February 2000, the CIA recruited a Russian scientist who had defected to the US years earlier. His mission: Take the nuclear blueprints to Vienna to sell them - or simply give them - to the Iranian representatives for the International Atomic Energy Agency.
Dubbed "Operation Merlin," the plan was supposed to steer Iranian physicists off track by incorporating design flaws in the blueprints that would render the information worthless.
But in what may turn out to be one of the greatest foreign policy blunders of all time, Operation Merlin backfired when the Russian scientist spotted the design flaws immediately - and even offered to help Iran fix the problems.
Risen said the Clinton-approved plan ended up handing Tehran "one of the greatest engineering secrets in the world, providing the solution to one of a handful of problems that separated nuclear powers such as the United States and Russia from rogue countries such as Iran that were desperate to join the nuclear club but had so far fallen short."
He noted that thanks to the bizarre operation, Iran could now "leapfrog one of the last remaining engineering hurdles blocking its path to a nuclear weapon."

tomder55 answered on 04/20/06:

Iran's first started to develop nuclear power in the 1960s and 1970s and of course the US was instrumental in it's development . But it was only in it's infancy and in the construction phase before the Iran revolution .Bushehr-1 was 90% complete and 60% of its equipment had been installed, Bushehr-2 was 50% complete . The government of Prime Minister Mehdi Bazargan decided that Iran did not need nuclear energy, and the work at Bushehr was halted .

The Iraqi's bombed the site during the Iran -Iraq war which destroyed the entire core area of both reactors. But the gvt.changed it's postion on nuclear power and acquiring a bomb. During the 1980s US pressure prevented many countries from assisting them in the effort. Then in 1990 the Ruskies decided to help them restart Bushehr .China transfered uranium to them in 1991. The Clinton administration tried, unsuccessfully, to convince Russia to cancel the agreement. The Clinton administration then began charging that the plutonium that the reactors would produce would be used by Iran for making nuclear weapons. The original plan was for Russia to gather up the spent rods for proper disposal .

In 1995, it became evident that Iran may be pursuing nuclear weapons by procuring dual-use items from Western firms. The United States also learned that Iran and Russia concluded a secret protocol stipulating, among other things, construction of a gas centrifuge enrichment facility. The fear was that Iran might learn how to construct a similar clandestine facility and then produce weapons-grade uranium undetected. The United States under Clinton then imposed extensive sanctions on Iran and successfully pressured Russia and other potential suppliers, mostly in Europe, to halt exports of dual-use nuclear technology to Iran.In mid-2002, a Paris-based opposition group, the National Council of Resistance of Iran (NCRI), revealed the existence of two previously unknown facilities in Iran and thus exposed Iran's true intentions .

I am suspicious of Risen's account. For one thing he quotes Seymour Hersh ,as well as too many anonymous sources for my liking . He also in the same book got the NSA program completely wrong .

Another reason is that recent revelations show that Iran was floundering until they got aid from AQ Kahn network .Following Iran's disclosure of uranium enrichment research and subsequent inspections by the IAEA , the central role of Pakistan in Iran's nuclear programme was unearthed.Evidence uncovered by inspectors showed that Pakistan and Iran agreed around 1987 to a deal whereby a Pakistani centrifuge design was provided to Iran to resolve their unsuccessful attempts to master uranium enrichment . The transfer began in 1989, though Khan claimed to have discontinued the sale two years later. The IAEA however has evidence that Pakistani assistance continued as late as 1996.In March 2005, Pakistan acknowledged AQ Khan had provided centrifuges to Iran, though it denied having had any knowledge of the transactions. These documents were the smoking gun that proved that Iran's claim of only being interested in peaceful nuclear use was a lie.


Anyway ,we are talking about Bubba trusting a Russian double-agent scientist
....anything was possible with him .If Clinton really did screw up then don't expect front page news from the NY Slimes .





arcura rated this answer Excellent or Above Average Answer
triumvirate rated this answer Excellent or Above Average Answer

Question/Answer
fredg asked on 04/19/06 - Gas Prices in US

Hi,
Gas at one service station in Brooklyn, NY, went to $4.50 per gallon today, Wed.
Do you agree with these:
1. Iran will continue increasing it's oil prices.
2. Since the United States only uses 25% of the World oil, the United States has to "bite the bullet", watch gas prices go higher, and can't do anything about it.
3. The only time the United States will seriously start converting vehicles to non-oil use, will be when oil reaches at least $100 per barrel, causing gas prices at the pump to go to probably $8.00 a gal; with American Citizens being in a complete uproar. Now, citizens just complain, taking no action of any kind.

fredg

tomder55 answered on 04/20/06:

Fred ;I do not think $100/barrrel will have the effect .It may put a break on the economy and the Fed. will use it instead of the incremental rate hikes to cool off the economy . I think a disruption in the supply will be the thing that gets to America. Then again ;I thought that 2 disruptions in the 1970s would've waken us up . We did get a sensible energy policy back then ...one of the few things Jimmy C got right. But ,unfortunately we did not follow through . CAFE standards were eased .

The situation is far worse today .

There are emerging economies that are demanding an ever increasing share of a finite world supply . Drilling will not solve the supply or the cost problems and alternatives that we could've already exploited have been denied over environmental concerns (nuclear ...real and imagined ),and asthetics (windmills and the NIMBY syndrome .....see Cape Cod and the Kennedy's view )

The threat by Iran is real and growing . Their recent miltary exercises were designed specifically to block the flow of commerce through the Straits of Hormuz.

WE should already be running on an 85% bio-diesal mixture. European cars running on diesal today run cleaner ;and have better milage than they did when dieasal cars were introduced . American makers have taken the lead in developing flex-fuel engines but where are the stations that pump bio-diesal ? The Brazil economy was revised when they made the decision to switch to bio-diesal made totally from sugar .If we were to make the switch we ;with our agricultural capacity could be close to self-sufficient .

Erewhon rated this answer Excellent or Above Average Answer
fredg rated this answer Excellent or Above Average Answer
paraclete rated this answer Excellent or Above Average Answer

Question/Answer
Itsdb asked on 04/19/06 - What else they said

Some of what you may have missed in the 'general revolt'

Batiste on CNN:

    Number one is we've got the best military in the world, hands down, period. All Americans should be very proud of their service men and women. They're doing incredible work all over the world.

    Number two is whether we agree or not with the war in Iraq, we are where we are and we must succeed in this endeavor. Failure is frankly not an option. Success to me is setting the Iraqi people up for self reliance with their form of representative government that takes into account tribal, ethnic and religious differences that have always defined Iraqi society. Iraqis, frankly, in my experience, do not understand democracy. Nor do they understand their responsibilities for a free society.

    Number three. When my family and I returned from Germany after three years with the Big Red One, we were struck by the fact that there's a lack of sacrifice and commitment on the part of the American people. The exception of those families with soldiers committed into this fight.

    And certainly, too many of these families truly understand the meaning of sacrifice. Most Americans only confront this issue by deciding what color of magnet on the back end of their SUV. I think that our executive and legislative branches of government have a responsibility to mobilize this country for war...

    (video clip)

    GEN. PETER PACE, JOINT CHIEFS CHAIRMAN: We had then and have now every opportunity to speak our minds. And if we do not, shame on us because the opportunity is there.

    M. O'BRIEN: Have you spoken your mind internally on this?

    BATISTE: I think the world of General Pace. I respect him enormously, and I respect his words.

    M. O'BRIEN: But has that discussion gone on internally?

    BATISTE: Sure. Absolutely.

    M. O'BRIEN: Major General John Batiste, thank you for your time.


I guess they ran out of time after Batiste admitted the discussions the military supposedly didn't have the opportunity to have with their civilian bosses - had in fact happened.

Newbold in Time:

    Army General John Abizaid, head of Central Command, has been forceful in his views with appointed officials on strategy and micromanagement of the fight in Iraq--often with success. Marine Commandant General Mike Hagee steadfastly challenged plans to underfund, understaff and underequip his service as the Corps has struggled to sustain its fighting capability.

    To be sure, the Bush Administration and senior military officials are not alone in their culpability. Members of Congress--from both parties--defaulted in fulfilling their constitutional responsibility for oversight. Many in the media saw the warning signs and heard cautionary tales before the invasion from wise observers like former Central Command chiefs Joe Hoar and Tony Zinni but gave insufficient weight to their views. These are the same news organizations that now downplay both the heroic and the constructive in Iraq.


Remember how none of the generals cited military restructuring as a reason Rumsfeld should resign?

    Army Maj. Gen. John Riggs, who once headed an Army task force to transform the service's structure and weapons systems, said Rumsfeld should step down, citing an "atmosphere of arrogance."


No I'm sure the restructuring had nothing to do with it.

I bet there were no sour grapes with Riggs, who had nothing to with the Iraq war:

    Unlike the other high-ranking military critics to speak out recently, Riggs was not involved in the Iraq war. He left the military two years ago after a controversial Army decision to reduce his rank and force his retirement after an investigation found he created an "adverse command climate."


Zinni to CBS:

    We were much in line with Gen. Shinseki's view, says Zinni. We were talking about, you know, 300,000, in that neighborhood.


Does anyone recall that in February 2003 Shinseki "made clear that he was providing only his personal assessment of postwar needs, and that the final decision would be made by the commander of American forces in the region, Gen. Tommy R. Franks." Should the 'troop strength' critics aim at Franks instead?

Swannack on CNN:

    "Well specifically, Barbara, I agree with our national security objectives and the decision to remove Saddam Hussein, which was to create a stable Iraq and that subsequently will contribute to the stability of the Middle East.

    "The only other thing that I would like to add: I respect Secretary Rumsfeld. He served our nation well..."


Even though he should go...

Why did I have to search to learn what else was said? Is the call for Rummy's head the only newsworthy portion?

tomder55 answered on 04/20/06:

900 Generals serving now, about 4,000 to 5,000 retired.

At this point it is old news . The more the press brings it up the harder Bush's resolve will be . Even if he would consider replacing Rummy he would not do so uder pressure from braying hounds .

I am in the middle of reading 'War Footing : 10 Steps America Must Take to Prevail in the War for the Free World' edited by Frank Gaffney Jr. It is a call to mobilization simular to General Batiste's.

Batiste btw. was directly involved in the war planning and more than any other General mentioned could have voiced his concerns then if he had them. I still say that voicing their concern after they leave the military and making their criticism personal dangerously undermines the concept of civilian control of the military . What's next .. a junta?

"One of the things we have to understand ... is that it's bad for the military, it's bad for civil military relations and it's potentially very bad for the country, because what we're hearing and what we're seeing is not the role the military plays in our society, under our laws or, for that matter, under our Constitution," Former Chairman JCS General Richard Myers

article 94 (2)UCMJ : Any person subject to this chapter who-- with intent to cause the overthrow or destruction of lawful civil authority, creates, in concert with any other person, revolt, violence, or other disturbance against that authority is guilty of sedition .

Should these former Generals be charged ?

Itsdb rated this answer Excellent or Above Average Answer

Question/Answer
Erewhon asked on 04/17/06 - Especially for "WE" ........................................................................

As policy decisions loom, a code of silence is broken
by Richard Holbrooke

The calls by a growing number of recently retired generals for the resignation of Defense Secretary Donald Rumsfeld have created the most serious public confrontation between the military and an administration since President Harry S. Truman fired Gen. Douglas MacArthur in 1951.


In that epic drama, Truman was unquestionably correct -- MacArthur, the commanding general in Korea and a towering World War II hero, publicly challenged Truman's authority and had to be removed. Most Americans rightly revere the principle that was at stake: civilian control over the military. But this situation is quite different.

First, it is clear that the retired generals -- six so far, with more likely to come -- surely are speaking for many of their former colleagues, friends and subordinates who are still inside. In the tight world of senior active and retired generals, there is constant private dialogue. In the tight world of senior active and retired generals, there is constant private dialogue...


Recent retirees stay in close touch with old friends, who were often their subordinates; they help each other, they know what is going on and a conventional wisdom is formed. Retired Marine Lt. Gen. Greg Newbold, who was director of operations for the Joint Chiefs of Staff during the planning period for the war in Iraq, made this clear in an extraordinary, at times emotional, article in Time magazine this past week when he said he was writing "with the encouragement of some still in positions of military leadership." He went on to "challenge those still in uniform . . . to give voice to those who can't -- or don't have the opportunity to -- speak."

These generals are not newly minted doves or covert Democrats. (In fact, one of the main reasons this public explosion did not happen earlier was probably concern by the generals that they would seem to be taking sides in domestic politics.) They are career men, each with more than 30 years in service, who swore after Vietnam that, as Colin Powell wrote in his memoirs, "when our turn came to call the shots, we would not quietly acquiesce in half-hearted warfare for half-baked reasons." Yet, as Newbold admits, it happened again. In the public comments of the retired generals one can hear a faint sense of guilt that, having been taught as young officers that the Vietnam-era generals failed to stand up to Defense Secretary Robert McNamara and President Lyndon Johnson, they did the same thing.

Second, it is also clear that the target is not just Rumsfeld. Newbold hints at this; others are more explicit in private. But the only two people in the government higher than the secretary of defense are the president and vice president. They cannot be fired, of course, and the unspoken military code normally precludes direct public attacks on the commander in chief when troops are under fire. (There are exceptions to this rule, of course: In addition to MacArthur, there was Gen. George McClellan vs. Lincoln; and on a lesser note, Maj. Gen. John Singlaub, who was fired for attacking President Jimmy Carter over Korea policy. But such challenges are rare enough to be memorable, and none of these solo rebellions metastasized into a group, a movement that can fairly be described as a revolt.)

This has put President Bush and his administration in a hellish position at a time when security in Iraq and Afghanistan seems to be deteriorating. If Bush yields to the generals' revolt, he will appear to have caved in to pressure from what Rumsfeld disingenuously describes as "two or three retired generals out of thousands." But if he keeps Rumsfeld, he risks more resignations -- perhaps soon -- from generals who heed Newbold's stunning call that as officers they took an oath to the Constitution and should now speak out on behalf of the troops in harm's way and to save the institution that he feels is in danger of falling back into the disarray of the post-Vietnam era.

Facing this dilemma, Bush's first reaction was exactly what anyone who knows him would have expected: He issued strong affirmations of "full support" for Rumsfeld, even going out of his way to refer to the secretary of defense as "Don" several times in his statements. (This was in marked contrast to his tepid comments on the future of his other embattled Cabinet officer, Treasury Secretary John Snow. Washington got the point.)

In the end, the case for changing the secretary of defense seems to me to be overwhelming. I do not reach this conclusion simply because of past mistakes, simply because "someone must be held accountable." Many people besides Rumsfeld were deeply involved in the mistakes in Iraq and Afghanistan; many of them remain in power, and some are in uniform.

The major reason the nation needs a new defense secretary is far more urgent. Put simply, the failed strategies in Iraq and Afghanistan cannot be fixed as long as Rumsfeld remains at the epicenter of the chain of command. Rumsfeld's famous "long screwdriver," with which he sometimes micromanages policy, now thwarts the top-to-bottom reexamination of strategy that is absolutely essential in both war zones. Lyndon Johnson understood this in 1968 when he eased another micromanaging secretary of defense, McNamara, out of the Pentagon and replaced him with Clark M. Clifford. Within weeks, Clifford had revisited every aspect of policy and begun the long, painful process of unwinding the commitment. Today, those decisions are still the subject of intense dispute, and there are many differences between the two situations. But one thing was clear then and is clear today: Unless the secretary of defense is replaced, the policy will not and cannot change.

That first White House reaction will not be the end of the story. If more angry generals emerge -- and they will -- if some of them are on active duty, as seems probable; if the situation in Iraq and Afghanistan does not turn around (and there is little reason to think it will, alas), then this storm will continue until finally it consumes not only Donald Rumsfeld. The only question is: Will it come so late that there is no longer any hope of salvaging something in Iraq and Afghanistan?


***

Original article: http://www.washingtonpost.com/wp-dyn/content/article/2006/04/14/AR2006041401451.html

tomder55 answered on 04/19/06:

out of thousands of Generals in service and more retired their are a whole half dozen who speak out and the MSM lauds them . I recall when Clinton first decided to reform the military by easing the rules allowing openly homosexuals to serve their was a far greater outcry amongst the rank and file in the military and the same MSM chastised them because they do not make policy.

btw. the policy that is really irking the CNN Generals is the Rumsfeld reorganization plan which is heavy on integration. This is ruffling the feathers of some careerists who's personal feifdom is being shaken up.

Itsdb rated this answer Excellent or Above Average Answer
Erewhon rated this answer Excellent or Above Average Answer

Question/Answer
paraclete asked on 04/19/06 - Got a problem with Iran, We know whose fault that is, don't we?

Iraq war empowers Iran: Beazley
From: AAP

April 19, 2006


MISJUDGEMENTS over the war in Iraq have handed enormous political power to Iran, federal Opposition Leader Kim Beazley said

"The one thing that we know definitely has come out of the Iraq war so far is a massive increase in Iranian power," Mr Beazley said in Perth.

"They've got enormous political power as a result of our misjudgements in the war in Iraq.

"At the same time as that has occurred, we have got an argument with the Iranians about whether or not they should have nuclear weapons.


"Frankly they should not have nuclear weapons," he said.

tomder55 answered on 04/19/06:

I have a problem with Iran dating back to the 1980 attack on the US Embassy in Tehran and the taking of the US diplomatic corp. hostage . That was an act of war that should've been immediately taken care of .

I have a problem with Iran for the 1983 bombing of the U.S Embassy in Beirut killing 63 people ,and the U.S. Marine barracks in Beirut ,done by their surrogate thugs Hezbollah which killed 241 Americans in their sleep.That was an act of war that should've been immediately taken care of .

All of these happened well before OIF

Is it Beazley's claim that Iran started it's nuke development and miliary buuild up in 2003 as a direct response to the war in Iraq . If so,he is a dope. The Iranians abandoned their peaceful nuclear programs after the 1979 revolution and rethought their position duirng their war with Iraq.During the late 1980s and the early 1990s the US waged a campaign to stop Iran from acquiring components for their program.But Russia in 1990 agreed to help them build reactors at Bushehr ,and the Chinese provided Iran with uranium hexafluoride in 1991.

1995 President Clinton first started charging that Iran was planning to build nukes.The IAEA became aware that Iran was beginning to produce fissable material in Sept. 2003 .The governing board of the IAEA gave Iran an ultimatum to prove that her nuclear program is strictly for peaceful purposes and that is when the crisis started to unfold.

As far as Iran becoming more inluential after the Gulf war ,that is to a degree accurate . But the Gulf War I am refering to was waged in 1990 .After Saddam was defeated and pushed out of Kuwait ,he did not pose the same threat to Iran as he did before ;primarily because of US and British enforcement of the cease fire provisions and UN sanctions . Iran began their modernization and build up during the 1990s . They would be an equal threat today with or without OIF .

A stable democratically elected gvt. in Iran on the other hand poses a great threat to Iran; Greater than Saddam ever did because the Iranian people will see it working their ,and wonder why they should not have it in Iran also.





paraclete rated this answer Excellent or Above Average Answer

Question/Answer
Erewhon asked on 04/12/06 - By popular demand .. Bush back in the spotlight - up to date stuff for my fans ...

With One Filing, Prosecutor Puts Bush in Spotlight

By DAVID E. SANGER and DAVID JOHNSTON
Published: April 11, 2006

WASHINGTON, April 10 From the early days of the C.I.A. leak investigation in 2003, the Bush White House has insisted there was no effort to discredit Joseph C. Wilson IV, the man who emerged as the most damaging critic of the administration's case that Saddam Hussein was seeking to build nuclear weapons.


But now White House officials, and specifically President Bush and Vice President Dick Cheney, have been pitched back into the center of the nearly three-year controversy, this time because of a prosecutor's court filing in the case that asserts there was "a strong desire by many, including multiple people in the White House," to undermine Mr. Wilson.

The new assertions by the special prosecutor, Patrick J. Fitzgerald, have put administration officials on the spot in a way they have not been for months, as attention in the leak case seems to be shifting away from the White House to the pretrial procedural skirmishing in the perjury and obstruction charges against Mr. Cheney's former chief of staff, I. Lewis Libby Jr.

Mr. Fitzgerald's filing talks not of an effort to level with Americans but of "a plan to discredit, punish or seek revenge against Mr. Wilson." It concludes, "It is hard to conceive of what evidence there could be that would disprove the existence of White House efforts to 'punish Wilson.' "

With more filings expected from Mr. Fitzgerald, the prosecutor's work has the potential to keep the focus on Mr. Bush and Mr. Cheney at a time when the president is struggling with his lowest approval ratings since he took office.

Even on Monday, Mr. Bush found himself in an uncomfortable spot during an appearance at a Johns Hopkins University campus in Washington, when a student asked him to address Mr. Fitzgerald's assertion that the White House was seeking to retaliate against Mr. Wilson.

Mr. Bush stumbled as he began his response before settling on an answer that sidestepped the question. He said he had ordered the formal declassification of the 2002 National Intelligence Estimate on Iraq in July 2003 because "it was important for people to get a better sense for why I was saying what I was saying in my speeches" about Iraq's efforts to reconstitute its weapons program.

Mr. Bush said nothing about the earlier, informal authorization that Mr. Fitzgerald's court filing revealed. The prosecutor described testimony from Mr. Libby, who said Mr. Bush had told Mr. Cheney that it was permissible to reveal some information from the intelligence estimate, which described Mr. Hussein's efforts to acquire uranium.

But on Monday, Mr. Bush was not talking about that. "You're just going to have to let Mr. Fitzgerald complete his case, and I hope you understand that," Mr. Bush said. "It's a serious legal matter that we've got to be careful in making public statements about it."

Every prosecutor strives not just to prove a case, but also to tell a compelling story. It is now clear that Mr. Fitzgerald's account of what was happening in the White House in the summer of 2003 is very different from the Bush administration's narrative, which suggested that Mr. Wilson was seen as a minor figure whose criticisms could be answered by disclosing the underlying intelligence upon which Mr. Bush relied.

It turned out that much of the information about Mr. Hussein's search for uranium was questionable at best, and that it became the subject of dispute almost as soon as it was included in the 2002 National Intelligence Estimate on Iraq.

The answer to the question of whose recounting of events is correct Mr. Bush's or Mr. Fitzgerald's may not be known for months or years, if ever. But it seems there will be more clues, including some about the conversations between Mr. Bush and Mr. Cheney.

Mr. Fitzgerald said he was preparing to turn over to Mr. Libby 1,400 pages of handwritten notes some presumably in Mr. Libby's own hand that could shed light on two very different efforts at getting out the White House story.

One effort the July 18 declassification of the major conclusions of the intelligence estimate was taking place in public, while another, Mr. Fitzgerald argues, was happening in secret, with only Mr. Bush, Mr. Cheney and Mr. Libby involved.

Last week's court filing has already led the White House to acknowledge, over the weekend, that Mr. Bush ordered the selective disclosure of parts of the intelligence estimate sometime in late June or early July. But administration officials insist that Mr. Bush played a somewhat passive role and did so without selecting Mr. Libby, or anyone else, to tell the story piecemeal to a small number of reporters.

But in one of those odd twists in the unpredictable world of news leaks, neither of the reporters Mr. Libby met, Bob Woodward of The Washington Post or Judith Miller, then of The New York Times, reported a word of it under their own bylines. In fact, other reporters working on the story were talking to senior officials who were warning that the uranium information in the intelligence estimate was dubious at best.

Mr. Fitzgerald did not identify who took part in the White House effort to argue otherwise, but the evidence he has cited so far shows that Mr. Cheney's office was the epicenter of concern about Mr. Wilson, the former ambassador sent to Niger by the C.I.A. to determine what deal, if any, Mr. Hussein had struck there.

Throughout the spring and early summer of 2003, Mr. Fitzgerald concluded, the former ambassador had become an irritant to the administration, raising doubts about the truthfulness of assertions made publicly by Mr. Bush in his State of the Union address in January of that year that Iraq might have sought uranium in Africa to further its nuclear ambitions.

Mr. Wilson's criticisms culminated in a July 6, 2003, Op-Ed article in The Times in which he voiced the same doubts for the first time on the record. He cited as his evidence his 2002 trip to Niger, instigated, he said, because of questions raised by Mr. Cheney's office.

Mr. Wilson's article, Mr. Fitzgerald said in the filing, "was viewed in the Office of the Vice President as a direct attack on the credibility of the vice president (and the president) on a matter of signal importance: the rationale for the war in Iraq."

Mr. Fitzgerald suggested that the White House effort was a "plan" to undermine Mr. Wilson.

"Disclosing the belief that Mr. Wilson's wife sent him on the Niger trip was one way for defendant to contradict the assertion that the vice president had done so, while at the same time undercutting Mr. Wilson's credibility if Mr. Wilson were perceived to have received the assignment on account of nepotism," Mr. Fitzgerald's filing said.

====

Is this another case of "Nobody in the White House done nothing wrong!"?

Or what?



Even if you are infatuated with Bush, it ought to raise at least the shadow of a question in the mind of a thinking person, huh?

tomder55 answered on 04/12/06:

Well then ,read it and weep :
So Fitzgerald told the Judge ..........

NEVER MIND

I wonder when the tax-payer subsidized Fitzgerald fishing expedition will end ?

Erewhon rated this answer Excellent or Above Average Answer
ETWolverine rated this answer Excellent or Above Average Answer
Itsdb rated this answer Excellent or Above Average Answer

Question/Answer
Erewhon asked on 04/12/06 - All ready for someone to spin ...

Iraqi bioweapons trailers: another smoking gun goes up in smoke
By Bill Vann
12 June 2003


During his recent [2003] trip to Europe, President Bush rebuffed charges that his administration launched the war against Iraq under false pretenses. We found the weapons of mass destruction, he insisted.

The claim was based on the discovery in northern Iraqs Kurdish region of two trailers bearing laboratory equipment. On May 28, the CIA issued a white paper describing the vehicles as Iraqi Mobile Biological Warfare Agent Production Plants. The paper proclaimed that their discovery constituted the strongest evidence to date that Iraq was hiding a biological warfare program.

This assertion itself represented a damning admission. In his State of the Union address at the end of last January, Bush had warned the American public that the Saddam Hussein regime had as many as 30,000 munitions capable of delivering chemical weapons and facilities to produce over 25,000 liters of anthrax and 38,000 liters of botulinum toxin. Iraq, he continued, could be in possession of 500 tons of sarin, mustard and VX nerve agent.

Similarly, in his February 5 speech to the United Nations Security Council, Secretary of State Colin Powell spoke of an Iraqi stockpile of between 100 and 500 tons of chemical weapons agents.

Two months after the fall of Baghdad, none of the alleged chemical weapons shells nor a single ounce of the arsenal described by the administration has been found. With growing demands that the Bush administration in the US and the Blair government in Britain account for this discrepancy, Washington seized upon the two trailers as the sole evidence supposedly substantiating its allegations.

The trucks, administration officials said, matched the description given by Powell at the UN of mobile biological weapons labs that had in turn been described to US intelligence by a single Iraq defector. Information given by defectors, most of them funneled to US officials via the Iraqi National Congress, which was agitating for a US invasion, had repeatedly proven false. Moreover, UN weapons inspectors checked out some of the vehicles referred to and found that they were used for testing food or preparing chemicals used in agriculture.

Nevertheless, State Department spokesman Richard Boucher cited the two trailers discovered in the north of Iraq at a May 28 press briefing and declared, It is very important to recognize that programs that we had said existed do exist; that the kind of equipment that we had said existed does exist. In line with this political mandate, the CIA white paper set out to make a square peg fit into a round hole.

Now, a number of intelligence officials and scientists on both sides of the Atlantic have come forward to dispute Washingtons claims about the trailers and accuse the Bush administration of falsifying evidence to provide itself with political cover.

Last weekend both the New York Times and the London Observer published articles reporting challenges by US and British investigators familiar with the vehicles to the claims made about them by the Bush and Blair administrations.

The CIAs own report claiming that the vehicles were mobile bioweapons labs acknowledged that no trace of biological agents that would be used in weapons production were found in the trucks.

Moreover, it recounted that Iraqi scientists, who are presumably cooperating with American investigators, were shown pictures of the trailers and immediately identified them as equipment used to produce hydrogen for artillery weather balloons.

The balloons are sent aloft to monitor and direct artillery fire, and the equipment used to fill them must be mobile.


The CIA report dismissed the Iraqi scientists testimony on the grounds that other Iraqis have used sophisticated denial and deception methods that include the use of cover stories that are designed to work. It acknowledgedpresumably confirming that this was just such a workable storythat the equipment could be used to produce hydrogen using a chemical reaction.

The report discounted the possibility the vehicles could have been used for hydrogen production on the grounds that they would have been inefficient compared to newer and more compact hydrogen generation systems. The fact that such equipment would have been denied Iraq by United Nations sanctions apparently escaped the agencys notice.

According to scientists who are familiar with the trailers, the vehicles, if used as biological labs, would have been even more inefficientand indeed, deadlyto their operators.

As one CIA official told the New York Times, the most persuasive evidence that the trucks were bioweapons labs was the fact that they looked like the drawings Powell presented to the UN last February based upon the claims of a single defector!

The Times, which originally joined the administration in hailing the trailers discovery as a breakthrough in the hunt for Iraqi WMDs, reported on June 6 that three teams had examined the vehicles. The first two, the paper said, strongly supported the claim that they were used for producing biological weapons. However, a third team, composed of more skilled and senior experts, was sharply divided. Several of those involved charged that the CIA report was falsified to serve the political needs of the Bush White House.

Everyone has wanted to find the smoking gun so much that they may have wanted to have reached this conclusion, one intelligence expert told the newspaper, describing himself as very upset with the process.

Another WMD expert charged that the CIA white paper on the trailers was a rushed job and looks political.

A number of the experts, who spoke to the Times on the condition of anonymity, challenged the report on technical grounds, stating that the design of the equipment on the trailers made the claims of the Iraqi scientists far more credible than those of the CIA. They questioned whether a central tank found on the trailers was a fermenter used to produce large quantities of deadly germs. It is not built and designed as a standard fermenter, said one. Certainly, if you modify it enough you could use it. But thats true of any tin can.

They pointed out that the trailers lacked essential equipment for sterilizing, growing and drying bacteria, without which no weapons materials could have been produced. The CIAs response was to hypothesize a Rube Goldberg-type system in which these trailers would work in tandem with other vehicles containing the missing equipment. However, there is no evidence to support the existence of these other trailers, which presumably would have been traveling together with the ones captured by the US military.

The experts also noted that, while there was no suitable means of removing germ fluids from the vehicles processing tanks, they were equipped in a manner that would easily allow the extraction of gas, a feature consistent with the Iraqi scientists claim that they were used to produce hydrogen [a gas!] for balloons.

The Observer newspaper reported that the British military, the MI6 intelligence agency and Porton Down, Britains biochemical weapons center, have been ordered to perform their own investigation of the trailers in light of the growing skepticism among US experts.

Prime Minister Tony Blair, who is facing a growing political firestorm over charges that he backed the US war in Iraq based on phony evidence of Iraqi WMDs, had also touted the discovery of the trailers as proof of the US-British allegations.

But chemical weapons experts, engineers, chemists and military systems experts contacted by the Observer [an esteemed British Newspaper] over the past week say the layout and equipment found on the trailers is entirely inconsistent with the vehicles being mobile labs, the newspaper reported.

The Observer article noted a number of facts contradicting the claims that the vehicles had been used to make biological weapons material. These included the absence of pumps needed to create vacuum conditions essential for working with germ cultures, and the lack of steam sterilization equipment required to prevent contamination that would render bacterial weapons materials harmless. It also pointed to the canvass sides on the vehicles, which would have made them extremely dangerous to operate as bioweapons labs. Normally, such labs are airtight.

The British newspaper quoted scientists who said the failure to detect any trace of pathogens on the equipment rendered the claims of their use as bioweapons labs highly suspect. Weapons inspectors who had checked other tanks that were used in weapons production pointed out that traces were normally detectable, even if they had been scoured with chemicals.

Finally, the Observer revealed that the Iraqi military possessed precisely the kind of hydrogen-producing equipment for balloons described by the scientists who were questioned by US intelligence. A British arms manufacturer sold the system, known as Amets, or Artillery Meteorological System, to Baghdad in 1987, when both Washington and London were supporting Saddam Husseins regime.

If Washington were interested in the truth, it would invite independent experts, such as the UN weapons inspectors who worked in Iraq before the US invasion, to examine the trailers. US officials have made it clear, however, that the Bush administration has no intention of allowing the UN inspectors to conduct any such investigation. While these inspectors have the greatest knowledge of Iraqi weapons programs, they cannot be relied upon to produce the evidence that the administration demands.

The story of the bioweapons trailers follows a familiar pattern. Ever since the fall of Baghdad, the US occupation forces have repeatedly announced the discovery of smoking guns proving the existence of the alleged Iraqi WMDs, only to end up retracting the claims after a cursory investigation.

On April 7, the Pentagon announced that the 101st Airborne had discovered a major cache of missiles fitted with chemical warheads outside of Baghdad. It was also reported that buried bioweapons labs had been unearthed.

A week later, on April 13, the Washington Post disclosed that the chemical weapons found by the 101st were in reality a pesticide, probably used to control Iraqs mosquito population.

The Pentagon, meanwhile, backed off from its original announcement concerning the missiles, telling the newspaper it denies any knowledge of this alleged discovery.

Two days later CNN revealed that the bioweapons labs had proven to be nothing more than unopened crates of standard laboratory equipment, such as test tubes.

Similarly, an announcement that troops had discovered 55-gallon drums filled with a blister agent was followed by a correctionthe substance was actually rocket fuel.

Last month, the Washington Post reported that US military teams searching for weapons pursued one of their hottest leads, breaking into a locked storeroom inside the headquarters of Iraqs Special Security Organization Al Hayat, only to find vacuum cleaners.

These farcical episodes, capped by the exposure of the trailers fraud, underscore the fact that the US government plotted and carried out a war of aggression that it justified to the American public and the world through a systematic campaign of lies.

Anyone care to spin these stories to make Bush a lilywhite truth speaker?





tomder55 answered on 04/12/06:

I already discussed much of this in your other posting . So let me get this straight . The US and Brits were flying sortees to patrol the no -fly zones .....and Saddam was flying smiley faced weather balloons to direct artillery fire against our combined air assets .. What did he think we were flying ? Sopwith Camels ? Gimme a break with this already .

Erewhon rated this answer Excellent or Above Average Answer
Itsdb rated this answer Excellent or Above Average Answer

Question/Answer
Erewhon asked on 04/12/06 - Bush and Bush Senior Administration lies exposed ...


Trailers Of Mass Destruction

Secretary of State Powell stood up in front of the world, and he said,

"Iraq has got laboratories, mobile labs to build biological weapons....They're illegal. They're against the United Nations resolutions, and we've so far discovered two. And we'll find more weapons as time goes on, But for those who say we haven't found the banned manufacturing devices or banned weapons, they're wrong. We found them."
WP, "Bush: 'We Found' Banned Weapons. President Cites Trailers in Iraq as Proof, " May 31, 2003

At the time of this statement, no such weapons were found, and no such weapons have been found to this day. On this point as well as the use of the captured trailers as biolabs, the WP said this in the above article:

"U.S. authorities have to date made no claim of a confirmed finding of an actual nuclear, biological or chemical weapon. In the interview, Bush said weapons had been found, but in elaborating, he mentioned only the trailers, which the CIA has concluded were likely used for production of biological weapons."

There was no statement of fact, there was no smoking gun. The CIA's finding was advanced as an opinion based on its own particular process of elimination, and it was immediately challenged by both U.S. and U.K. intelligence analysts who had seen the trailers.
Politex, 08.09.03

Now comes this,

"Engineering experts from the Defense Intelligence Agency have come to believe that the most likely use for two mysterious trailers found in Iraq was to produce hydrogen for weather balloons rather than to make biological weapons, government officials say".

The classified findings by a majority of the engineering experts differ from the view put forward in a white paper made public on May 28 by the C.I.A. and the Defense Intelligence Agency, which said that the trailers were ["likely used"] for making biological weapons....

The State Department's intelligence branch, which was not invited to take part in the initial review, disputed the findings in a memorandum on June 2. The fact that American and British intelligence analysts with direct access to the evidence were disputing the claims included in the C.I.A. white paper was first reported in June, along with the analysts' concern that the evaluation of the mobile units had been marred by a rush to judgment.
NYT, 08.09.03

"I don't believe anyone that I know in the administration ever said that Iraq had nuclear weapons."
Defense Secretary Donald Rumsfeld, at a hearing of the Senate's appropriations subcommittee on defense, May 14, 2003

Perhaps Rummy does not know that Cheney is in the administration?

"We believe he has, in fact, reconstituted nuclear weapons."
Vice President Dick Cheney on NBC's Meet the Press, March 16, 2003

Politex writes:

Bush lies So often and in so many different ways that I've never had the patience to keep a list of them. However, when I write something and include the generalization that Bush lies, some readers will write in and say, "Oh, yeh? What did he lie about? I don't believe it."

What follows, then, is an informal listing of just some of the lies he typically tells, starting from 2/01. Now, of course, we all know that Gore lies, Lott lies, Cheney lies, etc. But the difference between those liars and Bush is the President tells us that he is telling the truth when he is lying.

Hence, he will tell us what he is going to do, like get his proposed tax cut from the surplus, then try to get his proposed tax cut from military and medicare funds, instead. Or, once he has actually begun a program, tell us lies about how or why the program has begun. Or tell a closed-door Dem meeting something and then swear up and down the next day that he didn't say it. Or saying, "Yes, Mam" and meaning "No, Mam." Or having a spinner say the opposite the next day. Or, or...you get the idea.

Some Bush backers claim he's not a liar, he's just not very bright and doesn't remember things very well. That may be true, but we're sure Bush would not allow such an excuse in his "responsibility era." We're sure Bush would agree that if he's that dumb, he shouldn't be President.

Other Bush backers claim that some of his lies are "technically correct" or "tailored to fit the audience," or some such circumlocution. What they're talking about are lies of omission rather than lies of commission. In lies of omission it's what they imply, not what they say.

For example, the other evening Bush told Congress and the American people that he was putting a "lock box" on Social Security. Now, it's very clear that Bush wanted us to feel secure in the belief that he was protecting all of our Social Security funds for the future. No question, right? Yet, the very next day when his budget book was released, we learned that Bush told a lie of omission.

What he didn't tell Congress and the American people is that he would later take from $.6 to $1 trillion out of that "lock box" to cover his tax cuts. No doubt, Bush lied. He wanted folks to believe something that he knew was not true.

Of course, politicians do this all the time. It's second nature. In sum, the thing that really bothers us about Bush's lies is that he is also a hypocrite and pretends he's above lying. As a liar, he reinforces our assumptions about politicians. As a hypocrite, he reinforces our assumptions about his character.


Condoleeza Rice - 10 Minutes, Three Lies, And No Apology!

Condi Rice, Bush's National Security Adviser, appeared on 60 Minutes Sunday evening, but, unlike Bush anti-terrorism adviser Dick Clarke at the 9/11 Probe, she did not swear on the Bible that what she would say would be the truth. While Clarke on 60 Minutes last Sunday allowed himself to be probed and turned inside and out for nearly the entire program, the edited tape of the Rice interview with Ed Bradley lasted around 10 minutes, and she said nothing new.

The short episode came across as political spin to control the bleeding, and nothing more.

Rice's Lie #1 (transcript)

DICK CLARKE (video):
I said 'Mr. President, we've done this before. We - we've been looking at this. We looked at it with an open mind, there's no connection.' He came back at me and said, 'Iraq, Saddam - find out if there's a connection.' And in a very intimidating way. I mean, that we should come back with that answer....

CONDOLEEZZA RICE:
I - I have never seen the president say an - anything to an - people in an intimidating way, to try to get a particular answer out of them. I know this president very well. And the president doesn't talk to his staff in an intimidating way to ask them to produce information - that is false.

OUR RESPONSE:
Clarke and two others were in the room with Bush. The others have gone on record as agreeing with Clarke's description of the meeting. Condi was not present.

Rice's Lie #2 (transcript)

VOICE OVER:
All week long, the White House said it had no recollection that the September 12 meeting ever took place, and that it had no record that President Bush was even in the situation room that day. But two days ago, they changed their story, saying the meeting did happen.

CONDOLEEZZA RICE:
"None of us recall the specific - conversation....

OUR RESPONSE:
Actually, two lies here. First, the White House said the meeting didn't happen, then they changed their story. Second, Condi misleads Bradley by saying "us" did not recall the specific conversation. Of course "us" didn't since it has already been established that "us" was not in the room at the time of the conversation.

Rice's Lie #3 (transcript)

ED BRADLEY:
Clarke has alleged that the Bush administration underestimated the threat from - from al Qaeda, didn't act as if terrorism was an imminent and urgent problem. Was it?

CONDOLEEZZA RICE:
Of course it was an urgent - problem....

ED BRADLEY: :
But even the former chairman of the joint chiefs of staff, General Hugh Shelton, has said that the Bush administration pushed terrorism, and I'm quoting here, "farther to the back burner."

CONDOLEEZZA RICE:
I just don't agree....

ED BRADLEY:
After 9/11, Bob Woodward wrote a book in which he had incredible access and interviewed the president of the United States. He quotes President Bush as saying that he didn't feel a sense of urgency about Osama bin Laden. Woodward wrote that bin Laden was not the president's focus or that of his nationally security team. You're saying that the administration says fighting terrorism and al-Qaeda has been a top priority since the beginning.

CONDOLEEZZA RICE:
I'm saying that the administration took seriously the threat - let's talk about what we did....

ED BRADLEY: :
You'd listed the things that you'd done. But here is the perception. The chairman of the joint chiefs of staff at that time says you pushed it to the back burner. The former Secretary of the Treasury says it was not a priority. Mr. Clarke says it was not a priority. And at least, according to Bob Woodward, who talked with the president, he is saying that for the president, it wasn't urgent. He didn't have a sense of urgency about al Qaeda. That's the perception here.

CONDOLEEZZA RICE:
Ed, I don't know what a sense of urgency - any greater than the one that we had, would have caused us to do differently.

OUR RESPONSE:
It's clear that Bradley wants to discuss the Clarke charge that the Bush administration changed terrorism from the top priority to one of secondary concern, and Rice attepts to twist the question of giving terrorism "top priority" to taking terrorism "seriously," which are two different things.

Then Bush is quoted as saying terrorism was not "urgent." Rice ignores this documented quote and goes on to disagree with Bush. As such, she is attempting to mislead by changing the terms from "top priority" to "seriously," and to simply ignore the evidence presented that Bush disagrees with her.

As such, she is on auto-pilot as she lies, spinning the implicit scenario she wants Bradley to accept.

Finally, Bradley repeatedly gave Rice the program's forum to apologize for 9/11 to the millions of viewers watching the show, like Clarke did on the show last week and previously to that under oath in front of the 9/11 Panel, but she refused each time. (transcript)

--Jerry Politex, 03.29.04

Why The Public Believes Bush's Lies

"When interviewed by Tim Russert, Vice President Cheney asserted that Iraq was "the heart of the base" for the 9/11 terrorists and went on from there with a series of half-truths and outright deceptions about almost every topic broached, including his supposed lack of current "financial interest in Halliburton." Mr. Cheney, a master of the above-reproach dead pan, just kept going, effortlessly mowing right through any objections by the host.

The vice president was banking, as Dr. Dean did on "This Week," on a cultural environment in which fiction and nonfiction have become so scrambled "and can be so easily manipulated by politicians and show-biz impresarios alike"

That credibility itself has become a devalued, if not archaic, news value. This is why the big national mystery of the moment "why do almost 70 percent of Americans believe in Mr. Cheney's fictional insinuation that Saddam Hussein had some hand in 9/11?" is not so hard to crack.

As low as the administration's credibility may be, it is still trusted more than the media trying to correct the fictions the White House plants in the national consciousness." --Frank Rich, NMYT, 09.28.03


Are these lies or are they not lies?

tomder55 answered on 04/12/06:

this is all old news . Let me ask you //Why did Saddam need a fleet of mobile trucks to fill up hydrogen balloons ? Does that make sense ? But let's assume Saddam had weather balloons floating all over the deserts of Iraq ;how does this prove what Powell said ;or what the CIA analyse is a lie ? At most it proves that they were incorrect. Or perhaps ..............Saddam had' dual use 'purposes for these as he did with much of his WMD and nuclear program ??? The real issue is :Why should it have been assumed that there were benign reasons for them ?

Here is some other information that has been disclosed :

Kurdish forces in late April 2003 took into custody a specialized tractor-trailer near Mosul and subsequently turned it over to US military control.


The US military discovered a second mobile facility equipped to produce BW agent in early May at the al-Kindi Research, Testing, Development, and Engineering facility in Mosul. Although this second trailer appears to have been looted, the remaining equipment, including the fermentor, is in a configuration similar to the first plant. The manufacturer's plates on the fermentors list production dates of 2002 and 2003suggesting Iraq continued to produce WMD . Examination of the trailers reveals that all of the equipment is permanently installed and interconnected, creating an ingeniously simple, self-contained bioprocessing system. Although the equipment on the trailer found in April 2003 was partially damaged by looters, it includes a fermentor capable of producing biological agents and support equipment such as water supply tanks, an air compressor, a water chiller, and a system for collecting exhaust gases.




(do you need a fermentor to make hydrogen for ballons ??? I don't think so )


US forces in late April also discovered a mobile laboratory truck in Baghdad. The truck is a toxicology laboratory from the 1980s that could be used to support BW or legitimate research.

(like I said ...dual use deception ...The plant's design possibly could be used to produce hydrogen using a chemical reaction, but it would be inefficient. The capacity of this trailer is larger than typical units for hydrogen production for weather balloons. Compact, transportable hydrogen generation systems are commercially available, safe, and reliable and there was no need for Iraq to make tractor trailers for hydrogen production unless it was part of Saddam's game .)

Coalition experts on fermentation and systems engineering examined the trailer found in late April and have been unable to identify any legitimate industrial usesuch as water purification, mobile medical laboratory, vaccine or pharmaceutical productionthat would justify the effort and expense of a mobile production capability. We have investigated what other industrial processes may require such equipmenta fermentor, refrigeration, and a gas capture systemand agree with the experts that BW agent production is the only consistent, logical purpose for these vehicles.

The capability of the system to capture and compress exhaust gases produced during fermentation is not required for legitimate biological processes and strongly indicates attempts to conceal production activity.


The presence of caustic in the fermentor combined with the recent painting of the plant may indicate an attempt to decontaminate and conceal the plant's purpose.


Finally, the data plate on the fermentor indicates that this system was manufactured in 2002 and yet it was not declared to the United Nations, as required by Security Council Resolutions.

It does not impress me that Colin Powell having made his testimony to the UN is now if full CYA mode about his role in the decision to go to war . I thought he was made of better stuff. The best case the critics have is that it is inconclusive wheter they were mobile WMD labs or served some other function. I have seen no evidence that they were used for the preposterous notion of filling hydrogen balloons ..Did Saddam have the balloons imprinted with smily faces ?





Itsdb rated this answer Excellent or Above Average Answer
Erewhon rated this answer Excellent or Above Average Answer
paraclete rated this answer Excellent or Above Average Answer

Question/Answer
BeelzeBUSH asked on 04/11/06 - Two Iraqi Spies



Two Iraqi spies met in a busy restaurant after they
had successfully slipped into the U.S.

The first spy starts speaking in Arabic. The second
spy shushes him quickly and whispers:

"Don't blow our cover. You're in America now. Speak Spanish."





tomder55 answered on 04/12/06:

well done !



SANTA BARBARA...Oct 12,1995..Santa Barbara County has adopted a textbook which calls for the "liberation of Aztlan" by Chicanos. The book, The Mexican American Heritage, will be used for high school "Chicano studies" throughout Santa Barbara County.

According to a review of the book by Debora L. Sutherland of Santa Barbara, the book introduces the concept of "Aztlan" in the first chapter and from that point on uses the term to mean the seven states of the Southwest which were ceded to the U.S. by Mexico in the Treaty of Guadalupe Hidalgo in 1848.

"The book consistently questions the validity of our existing border with Mexico," Sutherland wrote." It also makes it very clear that with the continuing influx of Latinos into the Southwest along with their high birthrate, these 'natives' will realize their power to control Aztlan once again."

"Latinos are now realizing that the power to control Aztlan may once again be in their hands."

"Chicana/Chicano students of California must take upon themselves the responsibilities to promote Chicanismo within the community; as well as politicize our Raza (Race) and continue the struggle for self-determination of the Chicana/Chicano people and the liberation of Aztlan" [from the MECHa (Movimiento Estudiantil Chicano de Aztlan
)1996 Constitution]

as they say : Viva la Raza

BeelzeBUSH rated this answer Excellent or Above Average Answer
excon rated this answer Excellent or Above Average Answer

Question/Answer
Erewhon asked on 04/11/06 - How could so many Americans be so wrong as to abandon Bush in millions?

Bush job rating at new low, poll finds
60 percent disapprove of presidents performance
By Richard Morin and Claudia Deane
The Washington Post
Updated: 10:01 p.m. ET April 10, 2006

Political reversals at home and continued bad news from Iraq have dragged President Bush's standing with the public to a new low, at the same time that Republican fortunes on Capitol Hill also are deteriorating, according to the latest Washington Post-ABC News poll.

The survey found that 38 percent of the public approve of the job Bush is doing, down three percentage points in the past month and his worst showing in Post-ABC polling since he became president. Sixty percent disapprove of his performance.

With less than seven months remaining before the midterm elections, Bush's political troubles already appear to be casting a long shadow over them. Barely a third of registered voters, 35 percent, approve of the way the Republican-held Congress is doing its job -- the lowest level of support in nine years.

The negative judgments about the president and the congressional majority reflect the breadth of the GOP's difficulties and suggest that problems of each may be mutually reinforcing. Although the numbers do not represent a precipitous decline over recent surveys, the fact that they have stayed at low levels over recent months indicates the GOP is confronting some fundamental obstacles with public opinion rather than a patch of bad luck.

A majority of registered voters, 55 percent, say they plan to vote for the Democratic candidate in their House district, while 40 percent support the Republican candidate. That is the largest share of the electorate favoring Democrats in Post-ABC polls since the mid-1980s.

This grim news for the GOP is offset somewhat by the finding that 59 percent of voters still say they approve of their own representative. But even these numbers are weaker than in recent off-year election cycles and identical to support of congressional incumbents in June 1994 -- five months before Democrats lost control of Congress to Republicans.

As Bush and the Republicans falter, Democrats have emerged as the party most Americans trust to deal with such issues as Iraq, the economy and health care. By 49 to 42 percent, Americans trust Democrats more than Republicans to do a better job of handling Iraq.

Democrats also hold a six-percentage-point advantage over the GOP (49 percent to 43 percent) as the party most trusted to handle the economy. Their lead swells to double digits on such as issues as immigration (12 points), prescription drug benefits for the elderly (28 points), health care (32 points) and dealing with corruption in Washington (25 points).

The public divides evenly on only one issue: terrorism, with 46 percent expressing more confidence in the Democrats and 45 percent trusting Republicans on a top voting concern that the GOP counts on dominating.


But there is plenty of time left before Election Day for Republicans to take back ground they have lost to Democrats -- or for Democrats to solidify their recent gains. In the past year, public attitudes toward Bush and the Republicans have been driven by the news. Bush's popularity rebounded at the end of last year in response to the democratic elections in Iraq and renewed optimism about the economy at home -- only to stumble as the deadly insurgency continued and scandals in Congress and the White House drove down perceptions of the president and his party.

A total of 1,027 randomly selected adults were interviewed April 6 to 9 for this survey. The margin of sampling error is plus or minus three percentage points for the overall results.

Bush's job approval rating has remained below 50 percent for nearly a year. Perhaps more ominous for the president, 47 percent in the latest poll say they "strongly" disapprove of Bush's handling of the presidency -- more than double the 20 percent who strongly approve. It marked the second straight month that the proportion of Americans intensely critical of the president was larger than his overall job approval rating. In comparison, the percentage who strongly disapproved of President Bill Clinton on that measure never exceeded 33 percent in Post-ABC News polls.

The public is even more critical of Bush's performance in specific areas. On six of seven key issues, fewer than half of the respondents approve of the job Bush is doing, while majorities express dissatisfaction with him on Iraq (62 percent), health care (62 percent) and immigration (61 percent).

Concern on gas prices
Four in 10 -- 40 percent -- say Bush is doing a good job with the economy, down eight percentage points in a month. One reason for the drop may be the recent sharp increase in fuel costs. Fewer than one in four approve of his handling of gasoline prices, virtually the same as last summer when gas prices topped $3 a gallon. Overall, 44 percent said the increases are causing "serious hardship" in their family, up significantly from August.

Half of the public now disapproves of the way Bush is handling the fight against terrorism, an issue on which majorities of Americans had typically given him high marks until last year.

The depth of public dissatisfaction with Bush and the highly partisan nature of the criticism are underscored by public attitudes toward efforts by some in Congress to censure him or impeach him for his actions as president.

Democratic and Republican congressional leaders view both scenarios as remote possibilities. Still, more than four in 10 Americans -- 45 percent -- favor censuring or formally reprimanding Bush for authorizing wiretaps of telephone calls and e-mails of terrorism suspects without court permission. Two-thirds of Democrats and half of all independents, but only one in six Republicans, support censuring Bush, the poll found.

Last month, Sen. Russell Feingold (D-Wis.) introduced a resolution in the Senate to censure Bush. A majority of Americans, 56 percent, said his move was driven more by politics than by principle.

Calls to impeach Bush are not resonating beyond Democratic partisans. One-third of Americans, including a majority of Democrats (55 percent), favor impeaching Bush and removing him from office. But more than nine in 10 Republicans and two-thirds of independents oppose impeachment.

The ongoing bloodshed and political chaos in Iraq continues to drag down support for the war, the survey found. Barely four in 10 -- 41 percent -- say the war was worth fighting, down five percentage points since December. Although more than half of Americans think troop levels in Iraq should be decreased, only 15 percent are calling for an immediate withdrawal, a figure that has not varied much over the past year.

2006 The Washington Post Company

-===-

Is this just more media lies or are Americans leaving a sinking Buship?



tomder55 answered on 04/11/06:

partly the Washington Compost is not quite as biased as the NY Slimes ...;they are correct in pointing out that most Americans think Russ Feingold is an ass for introducing the censure resolution .


for the rest of it .... well ,the media loves to regale us with low poll #s for Bush . It seems they try to find a way to remind us daily that Bush's numbers are low.

YAWN .

They do that as much as recycling old news like the release of the NIE or when they happen to come across pictures from Abu Ghraib that they missed during the first release.

Media Selectively Recycles Old News
By Jack Kelly

We journalists are environmentally friendly. We recycle. We've been recycling old news all weekend, without, of course, telling you it's old news.

"A senior administration official confirmed for the first time on Sunday that President Bush had ordered the declassification of parts of a prewar intelligence report on Iraq in an effort to rebut critics who said the administration had exaggerated the nuclear threat posed by Saddam Hussein," reported David Sanger and David Johnston in the New York Times Monday.

For the first time? Here's the AP's Tom Raum on July 20, 2003: "The White House declassified portions of an October, 2002 intelligence report to demonstrate that President Bush had ample reason to believe Iraq was reconstituting a nuclear weapons program."

"The unusual decision to declassify a major intelligence report was a bid by the White House to quiet a growing controversy over Bush's allegations about Iraq's weapons programs," wrote Dana Milbank and Dana Priest in the Washington Post the day before.

Mr. Sanger and Mr. Johnston must have slept through that month. Why the recycling? In a court filing April 5, Special Prosecutor Patrick Fitzgerald reported that I. Lewis "Scooter" Libby, former aide to Vice President Dick Cheney, told the grand jury that Mr. Cheney had authorized him to disclose portions of the National Intelligence Estimate to Judith Miller of the New York Times a couple of weeks before its general release.

The NIE was declassified to rebut charges by Ambassador Joseph C. Wilson IV that President Bush lied when he said in his 2003 State of the Union address that "the British government has learned that Saddam Hussein recently sought significant quantities of uranium in Africa."

Mr. Libby has been indicted by Mr. Fitzgerald for lying to the grand jury about whether he told Ms. Miller that Mr. Wilson's wife, Valerie Plame, worked at the CIA, and was responsible for dispatching him on his now famous trip to Niger.

Dafna Linzer and Barton Gellman of the Washington Post should be grateful no legal jeopardy is attached to lying to their readers. In their story Sunday they said: "the evidence Cheney and Libby selected to share with reporters had been disproved months before."

The opposite is true. In July of 2004, the Senate Select Committee on Intelligence concluded unanimously that it was Mr. Wilson who was lying. He had been sent to Niger by his wife, and he told the CIA officers who debriefed him that Iraqi officials had approached Nigerien officials about buying "yellowcake."

Also that month, a parliamentary panel which investigated British claims about Iraqi WMD, the Butler Commission, concluded that the statements that Saddam had tried to buy uranium in Africa were "well founded." Perhaps Ms. Linzner and Mr. Gellman slept through that month.

Most of the recycled stories this weekend described the release of portions of the NIE as a "leak," a word that was not used in July of 2003 when the NIE was made public. For good reason. A leak is an unauthorized disclosure of classified information.

"President Bush was right to approve the declassification of parts of a National Intelligence Estimate about Iraq three years ago in order to make clear why he had believed that Saddam Hussein was seeking nuclear weapons," said the Washington Post in an editorial Sunday, one which noted the holes in Mr. Wilson's story which Ms. Linzner and Mr. Gellman somehow overlooked. "Presidents are authorized to declassify sensitive material, and the public benefits when they do."

The weekend's feeding frenzy was based on the little bit of news that Judy Miller had been briefed on the NIE before its general release. Hardly earth shattering or uncommon stuff. But many journalists saw an opportunity to imply the president had done something wrong, and to repeat charges made years ago which subsequently were proven false.

We're more reluctant to reexamine old news even when there are new developments, if the new developments run counter to journalistic memes. Here's a story you didn't read on the front page: Among the captured Iraqi documents recently released to the public is a March 17, 2001 memo from an Iraqi air force brigadier general soliciting volunteers from his command for a suicide mission to "strike American interests." Gee, in what sort of suicide mission would pilots have been useful?

Another document, released Friday, has not yet been translated from Arabic, but notations on it indicate it describes the movement of chemical and biological weapons.

But Saddam had no ties to terror groups, and he had no WMD. We told you so.






Erewhon rated this answer Excellent or Above Average Answer

Question/Answer
ROLCAM asked on 04/11/06 - Facing Facts !!




Sharon era comes to symbolic end in Israel.

Ariel Sharon's tenure as Israel's prime minister came to a symbolic end at a cabinet meeting that formally designated Ehud Olmert to replace the comatose stroke victim. Under Israeli law, Sharon will be categorised as permanently incapacitated and unable to serve as prime minister on Friday, 100 days after suffering his stroke. Olmert, deputy prime minister when Sharon fell ill, was named interim prime minister at the time.

What are your views on this matter ?

Are there any contingent laws in America like
this Israeli law ?

tomder55 answered on 04/11/06:

here is the current line of succession if multiple tragedy occures :



Vice President
Speaker of the House
President Pro Tempore of the Senate
Secretary of State
Secretary of the Treasury
Secretary of Defense
Attorney General
Secretary of the Interior
Secretary of Agriculture
Secretary of Commerce
Secretary of Labor
Secretary of Health and Human Services
Secretary of Housing and Urban Development
Secretary of Transportation
Secretary of Energy
Secretary of Education
Secretary of Veterans Affairs

ROLCAM rated this answer Excellent or Above Average Answer

Question/Answer
Itsdb asked on 04/10/06 - We're doomed

Air trends 'amplifying' warming

By Richard Black
Environment Correspondent, BBC News website, in Vienna

Reduced air pollution and increased water evaporation appear to be adding to man-made global warming.

Research presented at a major European science meeting adds to other evidence that cleaner air is letting more solar energy through to the Earth's surface.

Other studies show that increased water vapour in the atmosphere is reinforcing the impact of man-made greenhouse gas emissions.

Scientists suggest both trends may push temperatures higher than believed.

But they say there is an urgent need for further research, particularly at sea.

Dimming no more

Between the 1950s and 1980s, the amount of solar energy penetrating through the atmosphere to the Earth's surface appeared to be declining, by about 2% per decade.

This trend received some publicity under the term "global dimming".

But in the 1980s, it appears to have reversed, according to two papers published last year in the journal Science.

The decline in Soviet industry and clean air laws in western countries apparently reduced concentrations of aerosols, tiny particles, in the atmosphere.

These aerosols may block solar radiation directly, or help clouds to form which in turn constitute a barrier; or both effects may occur.

The lead researcher on one of those Science papers was Martin Wild from the Institute for Atmospheric and Climate Science (IACETH) in Zurich, and this week he has been discussing the implications of those findings at the European Geosciences Union (EGU) annual meeting in Vienna.

Correlations and causality

The reversal of "global dimming" has been proposed in some circles as an alternative explanation for climatic change, removing the need to invoke human emissions of greenhouse gases.

Dr Wild dismissed this picture. His analysis suggests that "global dimming" and the man-made greenhouse effect may have cancelled each other out until the early 1980s, but now "global brightening" is adding to the impact of human greenhouse emissions.

"There is always this argument that maybe the whole temperature rise wasn't due to greenhouse warming but due to solar variations," he told the BBC News website.

"During the solar dimming we had really no temperature rise. And only when the solar dimming disappeared could we really see what is going on in terms of the greenhouse effect, and that is only starting in the 1980s."

Analyses of global temperature indicate that a sharp upward trend commenced in the early 1980s.

But, said Dr Wild, there are strong regional variations in the "solar brightening" trend.

"In Eastern Europe, we see a very strong recovery [in solar radiation] - almost back to what it was before dimming began," he said.

"But India continues with the dimming - that's very much thought to be due to increasing air pollution.

"The general position is that air pollution is still increasing in the tropics, but decreasing outside the tropics; so probably that will amplify warming a little bit outside the tropics but not inside."

Data deficit

There are, Dr Wild admitted, holes in the picture of change.

"The term 'global dimming' is a bit dangerous," he said. "I usually call it 'solar dimming' not 'global dimming' because we really only know about this where we have measurements; and we don't have measurements at many places, for example over the oceans, or land in the tropics."

More research facilities are needed, he said, in tropical regions, particularly sub-Saharan Africa, and especially the oceans.

As well as extending measurements of solar energy reaching the Earth's surface, he urged more research on aerosol concentrations in the atmosphere and on trends in cloud cover.

Rolf Philipona from the World Radiation Center in Davos, Switzerland, is attempting to improve aerosol measurements in northern Europe.

"We're trying to put a paper together which shows the aerosol depth and the amount of aerosol in the air column from about six to eight stations in Europe," he told the BBC News website.

"In Germany and Switzerland we would have stations very high up, extending all the way to the North Sea."

Last year Dr Philipona released research indicating that European warming is largely driven by increases in humidity.

The mechanism is that rising levels of what are conventionally called "greenhouse gases", such as carbon dioxide and methane, cause more evaporation of water, which in the atmosphere is itself a greenhouse gas.

He believes this is having more impact than changes to the transmission of solar energy through the atmosphere.

"From my results I believe it's the greenhouse warming and in particular the water vapour feedback," he said.

"Studies and papers are also coming now which are looking more closely at what water vapour is doing in other regions; and there are several pieces of work showing water vapour is increasing over land areas like the United States."

Satellites and ships

A further implication of "global brightening" is that the temperature difference between night and day may reduce.

The "blanket" of greenhouse gases in the atmosphere has a net heating effect during day and night, whereas changes in solar energy reaching the surface are felt only in daytime.

Disproportionately higher night-time temperatures have already been noted in many parts of the world, and research in the Philippines has linked this trend to a reduction in rice yield.

The conclusions presented here present two major challenges to the research community.

One is to find ways of extending experimental investigations into the oceans and the developing world.

The second is to integrate them into computer models of climate, something which is only just beginning to happen.

~~~~~~~~~~~~~~~~~~~~~~~~~~~~~~~~~~~~~~~~~~~~~~~~~~~~~~~

So what now? Less industry, cleaner air, reduction of nasty aerosols, the world's getting brighter - and it's only contributing to global warming - we must be doomed.

Should Bush run right out and sign Kyoto...or would that only make the problem worse?

Steve

tomder55 answered on 04/11/06:

Some solar scientists are considering whether the warming might be caused, in part, by a periodic but small increase in the Sun's energy output. An increase of just 0.2% in the solar output could have the same affect as doubling the carbon dioxide in the Earth's atmosphere. Imagine that ....the sun causes it to be hot !!!! What a concept !!!

Itsdb rated this answer Excellent or Above Average Answer

Question/Answer
paraclete asked on 04/10/06 - The winds of change blow...........ever so slowly?


Business warms to change
April 10, 2006

New research on global warming has caused a split at the top end of town, writes Deborah Snow.

WESTPAC chief executive David Morgan had an interesting story to tell at an invitation-only breakfast for a handful of journalists in Sydney last week.

The anecdote concerned a recent private conversation with the head of the giant General Electric Company in the US, Jeff Immelt.

"He said to me he was virtually certain that the first action of the next president of the United States, be it Republican or Democrat, would be to initiate urgent action on climate change. And he wasn't saying that as a casual political comment ... he is [allocating] billions of dollars worth of investment in the confidence of that development."

George Bush and John Howard have both cold-shouldered the case for more direct government intervention to combat global warming.

But last Thursday Morgan - and five other top businesss executives - put their heads above the parapet with the launch of the Australian Business Roundtable on Climate Change, a powerful new voice which wants business and government to respond more rapidly to inexorably rising world temperatures.

The move is not risk-free for the six corporate chiefs. It has flushed into the open a long-simmering row within the highest ranks of corporate Australia over global warming - the kind of dispute big business generally likes to keep behind closed doors.

In Europe and Britain, some major corporations have been in the vanguard of urging greater action on climate change for several years. In the US the experience of Hurricane Katrina which devastated New Orleans has helped turn sentiment. But in Australia the roundtable's emergence marks a significant watershed for business.

It is also an open rebuke to the Business Council of Australia, the body which represents the chief executives of Australia's top 100 companies. The council was so wracked with division the last time it debated the issue nearly four years ago that it wound up deadlocked and decided not to take a position at all.

Morgan told journalists last week that the council's debate had been "immature", and signalled that he and other members of the roundtable would now be going back into that forum to try to move it forward. "The thing that has been missing is some fact base about the economics," he said.

To fill that gap the roundtable group has commissioned detailed modelling from Allen Consulting, which demonstrates it is possible for Australia to cut its greenhouse gas emissions by 60 per cent from 2000 year levels before 2050 and still have strong economic growth.

It's an important new plank in the case for direct action, until now stymied by arguments from business opponents, and indeed from the Prime Minister, that strong greenhouse gas action by Australia would have damaging economic effects.

Insurance boss Keith Scott says an increase in extreme weather events is linked to global warming: "The year 2005 set a record in terms of the number of natural catastrophes, and there is no reason why 2006 would be any less a year. My company is committed to this [roundtable agenda] because the issue not only goes to the very heart of our business dynamic, it concerns lives, livelihoods, health, infrastructure and the ability to maintain current levels of biodiversity."

Scott is the head of Australian and New Zealand operations for Swiss Re, one of the world's largest reinsurers. With Morgan, he is a core member of the new alliance, along with Michael Hawker, the chief executive of Insurance Australia Group, Gerry Hueston, the president of BP Australasia; Grant King, the managing director of Origin Energy; and Harry Debney, the chief executive of Visy Industries. Don Henry, of the Australian Conservation Foundation, is also a member.

The six business leaders have come up with a carefully worded report aimed at demonstrating the economic costs of doing little.

For instance, 250,000 jobs could be at stake if greenhouse action is delayed. Delay will also mean the need for much more drastic and costly action later on, they argue.

The report diplomatically seeks to avoid direct criticism of the state and federal governments, or other sections of the business community which are dragging their feet.

But reading between the lines, the implied criticism is clear. "There is a widening gap between [the] advancing scientific evidence and the international response from governments," the roundtable report says. "The Australian Government has a strong long-term focus on technology which is a necessary part of the solution. However many in business perceive a near-term policy gap.

"We believe that climate change is a major business risk and we need to act now."

The real grenade the group has rolled into the ring is its call for a "national, market-based carbon pricing mechanism", which it wants the Government to sketch out a framework for next year.

What the mechanism would look like has been left deliberately vague - a concession to the sensitivities of those industries that have lobbied long and hard against such a move.

But any plain English construction of "carbon pricing mechanism" translates, for most experts, into either a direct impost on carbon (greenhouse gas) emissions, or an indirect impost through some kind of emissions trading system, like the one now operating in the European Union.

Imposing a direct cost on carbon emissions has been resisted by the biggest greenhouse gas-producing industries, which have long had the ear of the Federal Government on climate change policy. The aluminium industry and the Minerals Council of Australia have led the charge, arguing that such a move would have a disproportionate impact on them and drive some of their operations offshore.

Dr Clive Hamilton, the head of the Australia Institute, has hailed the roundtable's call for a "carbon price signal" as highly significant because "it flies in the face of current government policy and does represent a fracturing of business opinion".

But he is critical of the roundtable for sidestepping the vexed issue of the Kyoto Protocol.

Australia and the US are the only two major developed nations not to have ratified Kyoto, which binds industrialised countries to targets for reducing greenhouse gas emissions.

"They [the roundtable] are trying to avoid discussing the issue because they don't want to engage in a direct confrontation with the Federal Government - and that's playing into the Government's hands," Hamilton told the Herald.

However Origin Energy's King insists that reviving the Kyoto debate "is not constructive in my view for getting the Government to move forward in policy terms".

The CSIRO research commissioned by the roundtable reveals the frightening dimensions of the challenge ahead.

Over the past 100 years the average surface air temperature of Australia has increased by just 0.7 per cent. Yet that alone has been enough to trigger marked declines in regional rainfall along the east and west coasts.

The report says even if global emissions stabilised at current levels, the planet would be committed to additional warming of between 2 and 5.5 degrees. At the upper end of this scale, the consequences would be dramatic.

Some leading scientists now fear the massive Greenland ice sheet could melt at temperature rises of between 2 and 3 degrees.

"Destabilisation or collapse of these ice sheets would lead to centuries of irreversible sea-level rise and coastal inundation around the world," the CSIRO report warns.

The disaster movie, The Day After Tomorrow, predicated on just such scenarios, stops looking like Hollywood make-believe and more like looming reality, though on a much longer time scale.

Late last week the federal Environment Minister, Ian Campbell, reiterated the Government's line that, while it had left the door open to a carbon trading system, this could only be workable when an international system, or at least a regional system, was in place.

"It would be highly effective if you could find a market mechanism that didn't have a perverse outcome for the economy and the environment," he told the Herald. "If you set it [the price of carbon] too low it won't drive investment [into renewable energies] and if you set it too high it will drive investment offshore."

But Hamilton accuses the Government of still being hostage to a network of industry lobby groups which their opponents dub the "greenhouse mafia", representing the heavy carbon-emitting industries such as coal, oil, cement, aluminium, mining and electricity.

He cites the work of Guy Pearse, a Liberal and former government adviser, who has researched a PhD thesis on the companies and industry associations which make up this network, which he calls the greenhouse "blockers".

Pearse writes in his recently published thesis that "the self-declared 'mob' works as a pack to keep the brakes on greenhouse policy and to protect the narrow interests of a few resource-based industries ... They have demonstrated an unrivalled capacity to influence the direction of government policy ... and have drawn on the support of various captains of industry - or 'aces' - to apply pressure on the Government to get their way on greenhouse policy."

Now, with the emergence of the Roundtable on Climate Change, the forces Pearse documents may have met their match.

tomder55 answered on 04/10/06:


Kyoto is pointless, say 60 leading scientists
By Philip Sherwell


Canada's new Conservative prime minister, Stephen Harper, has been urged by more than 60 leading international climate change experts to review the global warming policies he inherited from his centre-Left predecessor.

In an open letter that includes five British scientists among the signatories, the experts praise his recent commitment to review the controversial Kyoto protocol on reducing emissions harmful to the environment.

"Much of the billions of dollars earmarked for implementation of the protocol in Canada will be squandered without a proper assessment of recent developments in climate science," they wrote in the Canadian Financial Post last week.

They emphasised that the study of global climate change is, in Mr Harper's own words, an "emerging science" and added: "If, back in the mid 1990s, we knew what we know today about climate, Kyoto would almost certainly not exist, because we would have concluded it was not necessary." Despite claims to the contrary, there is no consensus among climate scientists on the relative importance of the various causes of global climate change, they wrote.

"'Climate change is real' is a meaningless phrase used repeatedly by activists to convince the public that a climate catastrophe is looming and humanity is the cause. Neither of these fears is justified.

"Global climate changes all the time due to natural causes and the human impact still remains impossible to distinguish from this natural 'noise'."

The letter is the latest effort by climate change sceptics to counter claims that there is a consensus that human activity is causing global warming.




paraclete rated this answer Excellent or Above Average Answer

Question/Answer
ROLCAM asked on 04/09/06 -
Who to believe?


Last Sunday, a newspaper revealed that several of Britain's defence chiefs were going to meet to discuss the effects on British interests of a military strike on Iran by the United States to destroy that county's capacity to build a nuclear bomb. The story was categorically denied by Ministry of Defence officials, who told Sean Rayment, our Defence Correspondent, that there was "no truth in it whatsoever".


Yet those officials also told Rayment that by writing the story, he "had come very close to damaging national security". Asked how any story that was apparently false could possibly damage national security, the MoD officials changed tack: they admitted the story was correct in maintaining that there had been a meeting of defence chiefs - but, they insisted, an American strike on Iran had not been on the agenda.

It is, of course, no secret that the Bush administration has drawn up plans for a strike on Iran. As Seymour Hersh reports in The New Yorker tomorrow, many of the US officials opposed to a strike believe that its most immediate effect will be to generate an armed insurrection among the Shias in southern Iraq - precisely the region where British soldiers are concentrated. British soldiers are the most visible and easily accessible symbols of the American-led occupation in southern Iraq. They would be at very serious risk. Iran has also threatened to shut down its oil exports in the event of a strike, which could have a devastating effect on the world's economies.

But if you believe the MoD's press office, British defence chiefs are not talking about any of these things. They have no anxieties about what might happen to British soldiers in Iraq, and are certainly not meeting to discuss what to do in the event that the US drops "bunker-buster" bombs tipped with nuclear warheads on Iran's nuclear facilities.



Last Sunday, a newspaper revealed that several of Britain's defence chiefs were going to meet to discuss the effects on British interests of a military strike on Iran by the United States to destroy that county's capacity to build a nuclear bomb. The story was categorically denied by Ministry of Defence officials, who told Sean Rayment, our Defence Correspondent, that there was "no truth in it whatsoever".


Yet those officials also told Rayment that by writing the story, he "had come very close to damaging national security". Asked how any story that was apparently false could possibly damage national security, the MoD officials changed tack: they admitted the story was correct in maintaining that there had been a meeting of defence chiefs - but, they insisted, an American strike on Iran had not been on the agenda.

It is, of course, no secret that the Bush administration has drawn up plans for a strike on Iran. As Seymour Hersh reports in The New Yorker tomorrow, many of the US officials opposed to a strike believe that its most immediate effect will be to generate an armed insurrection among the Shias in southern Iraq - precisely the region where British soldiers are concentrated. British soldiers are the most visible and easily accessible symbols of the American-led occupation in southern Iraq. They would be at very serious risk. Iran has also threatened to shut down its oil exports in the event of a strike, which could have a devastating effect on the world's economies.

But if you believe the MoD's press office, British defence chiefs are not talking about any of these things. They have no anxieties about what might happen to British soldiers in Iraq, and are certainly not meeting to discuss what to do in the event that the US drops "bunker-buster" bombs tipped with nuclear warheads on Iran's nuclear facilities.

Why all the lies??

tomder55 answered on 04/10/06:

Let's assume that it is all lies ? Ever hear of the publishing of dis-information ? Does it not have some value ? Prior to the first Gulf War the US press scooped some war plans and published that the main thrust of the ground war would be a beach head by the Marines in Kuwait City . Saddam had to waste some of his forces to guard against that possiblity . Who knows ? I think Seymor Hersh is a rat ... perhaps he was fed all this by his unnamed sources as an elaborate deception .

ROLCAM rated this answer Excellent or Above Average Answer

Question/Answer
ROLCAM asked on 04/09/06 - OIL POLITICS ??

What is going to happen when we finally run out of oil?.

tomder55 answered on 04/10/06:

Price is already pushing us to the next generation ;and if we move too slowely then shortages will. I think the immediate future is the hybrids (although they are overrated ) and a mix of bio-deasal . In the warm climates cars can run 100 % on ethanol . Brazil has made the conversion and they are prospering as a result. They have revitalized their sugar industry because of this ,and .....sugar is renewable!!! ...and ...ethanol runs cleaner that petroleum !! It's a win ..win situation . In the colder climates ethanol has to be mixed 85% with gasoline during the colder months . But ethanol can be produced from multiple agricultural sources so countries like Australia and the US should both be self-sufficient with these resources .

Humans have survived because of our resoucefulness. Don't worry about our ability ;worry about our will to change .It could be that we squeeze the last drop of oil from shale before we finally make the conversion . We most certainly will wait until there is a major disruption of some sort . The price of a barrel of oil does not seem to be a deterent . It rose rapidly by 50% in recent years and as far as I can tell no one has significantly changed their usage patterns. But there is a growing voice in this country that alternative energy development is a national security priority .

ETWolverine rated this answer Excellent or Above Average Answer
ROLCAM rated this answer Excellent or Above Average Answer

Question/Answer
Erewhon asked on 04/09/06 - The little rift!



"It is the little rift within the lute
That by and by will make the music mute.

It is the little speck in garnered fruit,
Thaty slowly rotting inwards
Moulder all.

It is not worth the keeping,
Let it go!"




The little (?) rift (dishonesty) in Bush that I have long spoken of (and been mercilessly attacked for my honest outspokenness), is beginning now to be visible even to his 'friends.'

The slippy slope just got a whole lot slippier.

QUESTION:

Will GW "Teflon" Bush be able to lie his way out of this one?


Please keep your answers brief and to the point.



tomder55 answered on 04/10/06:

Ron ,
I addressed that in the leaky boat posting you did . There is nothing there . Even Pat Fitzgerald says that Bush did not disclose Plame.

It is prety clear that former Deputy Secretary of State Richard Armitage was the first official to name Plame. But of course that did not make the front page of the NY Slimes yet .It did make the Washington Compost about a month ago ......didn't you see it ? Didn't you hear it on NPR ;on the network news ?

Didn't think so. Armitage is one of the press darlings .

Armitage gave the name to Bob Woodward who kept it under wraps until he could finish his latest book .That is why it was Novak and not Woodward who first disclosed it . But all that is irrelevent since Plame being CIA was one of the worse kept secrets in the D.C. circuit. Joe Wilson used to brag about it at cocktail parties.

And here we have well respected news correspondant ;wife of former Fed. Chair Alan Greenspan ;Andrea Mitchell who said on CNBC's "Capital Report," Oct. 3, 2003 the following to host Alan Murray :

Murray: Do we have any idea how widely known it was in Washington that Joe Wilson's wife worked for the CIA?

Mitchell: It was widely known among those of us who cover the intelligence community and who were actively engaged in trying to track down who among the foreign service community was the envoy to Niger.

This is NON -news .


Erewhon rated this answer Excellent or Above Average Answer
Itsdb rated this answer Excellent or Above Average Answer

Question/Answer
paraclete asked on 04/08/06 - Not civil war in Iraq? the arabs see it differently

Iraq 'practically destroyed'
From: Agence France-Presse
by Sam Dagher in Dubai

April 09, 2006


EGYPTIAN President Hosni Mubarak has warned that Iraq was in the middle of a civil war that threatened the Middle East and expressed alarm about Shiite Iran's influence in the Arab world.
"There is effectively a civil war underway now (in Iraq)," Mr Mubarak said in an interview with the satellite news channel Al-Arabiya.

"If the Americans left now, it would be a catastrophe because the war will get worse and Iran and others will interfere and the country will become the theatre of an ugly civil war and terror will eat up not only Iraq but the entire region."

A suicide car bomber killed six Shiite pilgrims south of Baghdad overnight one day after 79 worshippers were killed in a triple suicide attack against one of the capital's main Shiite mosques.

"I do not know when the situation in Iraq will stabilise. I personally do not see a solution to the problem in Iraq, which is practically destroyed now," Mr Mubarak said.

"If Saddam was more just none of this would have happened," he added referring to the dictatorial rule of ousted President Saddam Hussein.

He said Iraq's situation was made worse by the mix of Shiites, Sunnis, Kurds and other groups living there.

Mr Mubarak said the Islamic Republic of Iran exerted a huge influence over Iraq's majority Shiite population and Shiites living in Arab countries.

"There are Shiites in all these countries (of the region), significant percentages, and Shiites are mostly always loyal to Iran and not the countries where they live," he said.

"Naturally Iran has an influence over Shiites who make up 65 per cent of Iraq's population."

Iraq's top Shiite political parties, the Supreme Council for the Islamic Revolution in Iraq and Dawa, were based in Iran before the fall of Saddam in 2003.

There are also significant Shiite populations in Bahrain, Kuwait, Lebanon and Saudi Arabia. In particular, Iran has close ties with the Lebanese party and militia Hezbollah.

Tehran cut diplomatic ties with Cairo after then Egyptian president Anwar Sadat made peace with Israel in 1979.

Receiving about two billion dollars a year, Egypt is the largest recipient of US foreign aid after Israel and Iraq.

Mr Mubarak is not the first Sunni Arab leader to raise concern about Iran.

Jordan's King Abdullah II warned in an interview with the Washington Post in December 2005 that Iran wanted to create "a Shiite crescent" linking it with Iraq, Lebanon and possibly Syria.

"Even Saudi Arabia is not immune from this. It would be a major problem. And then that would propel the possibility of a Shiite-Sunni conflict even more, as you're taking it out of the borders of Iraq," Abdullah said at the time.

Analysts and Western officials worry if Iran were to obtain a nuclear bomb it could spark an arms race between the Shiite and Sunni worlds. Iran denies its nuclear programme has any military purpose.


It would seem the US attacked the wrong country?

tomder55 answered on 04/09/06:

gotta love it . A Sunni dictaor wishing another Sunni dicator was still in power

paraclete rated this answer Excellent or Above Average Answer

Question/Answer
paraclete asked on 04/08/06 - Is it the US Administration or the press that has gone mad?

US 'plans nuclear strikes against Iran'
From: Agence France-Presse
From correspondents in Washington

April 08, 2006


THE administration of US President George W. Bush is planning a massive bombing campaign against Iran, including use of bunker-buster nuclear bombs to destroy a key Iranian suspected nuclear weapons facility, The New Yorker magazine reported in its April 17 issue.
The article by investigative journalist Seymour Hersh said that Mr Bush and others in the White House have come to view Iranian President Mahmoud Ahmadinejad as a potential Adolf Hitler.

"That's the name they're using," the report quoted a former senior intelligence official as saying.

A senior unnamed Pentagon adviser is quoted in the article as saying that "this White House believes that the only way to solve the problem is to change the power structure in Iran, and that means war."

The former intelligence officials depicts planning as "enormous," "hectic" and "operational," Mr Hersh writes.

One former defence official said the military planning was premised on a belief that "a sustained bombing campaign in Iran will humiliate the religious leadership and lead the public to rise up and overthrow the government," The New Yorker pointed out.


In recent weeks, the President has quietly initiated a series of talks on plans for Iran with a few key senators and members of the House of Representatives, including at least one Democrat, the report said.

One of the options under consideration involves the possible use of a bunker-buster tactical nuclear weapon, such as the B61-11, to insure the destruction of Iran's main centrifuge plant at Natanz, Mr Hersh writes.

But the former senior intelligence official said the attention given to the nuclear option has created serious misgivings inside the military, and some officers have talked about resigning after an attempt to remove the nuclear option from the evolving war plans in Iran failed, according to the report.

"There are very strong sentiments within the military against brandishing nuclear weapons against other countries," the magazine quotes the Pentagon adviser as saying.

The adviser warned that bombing Iran could provoke "a chain reaction" of attacks on American facilities and citizens throughout the world and might also reignite Hezbollah.

"If we go, the southern half of Iraq will light up like a candle," the adviser is quoted as telling The New Yorker.

tomder55 answered on 04/09/06:

drgade is right . Look at how many times in this short report unnamed Pentagon adviser ; former intelligence officials ;former defence official ;former senior intelligence official is used .It's like :How many Clintonistas can you quote in one page ? Who are Hersh's sources? Much of the time, given his massive use of unnamed individuals, it is impossible to say. Are they reputable people? Disgruntled individuals with an axe to grind? Figments of his imagination? Who knows?

I frankly do not understand why Hersh has any credibilty at all. He has a history of publishing lies ,distortions that are demonstrably false. In his 1983 anti-Kissinger book 'The Price of Power' he falsely accused India's former Prime Minister Morarji Desai of being a paid CIA informant .Hersh quoted 'anonymous intelligence officials' "recalling" Desai had been paid $20,000 yearly as a CIA informer during the Johnson administration.The elderly Desai sued ,but lost because he could not prove that there were no 'anonymous intelligence officials 'who did not tell Hersh the story .The judge ruled that Hersh need not identify his sources and Desai's attorney was prevented from questioning anyone in the CIA's employ. Hersh never even took the stand. What the case showed was that as long as he did not need to reveal his sources, an irresponsible journalist could label any public figure a CIA agent with impunity.

Hersh's 'The Samson Option '[about the building of the Israeli nuke] rests squarely on the fantasies of Ari Ben Menashe who is a known liar and teller of some of the most remarkable fantasies.Ben Menashe claimed to have been with the first George Bush in Paris in October 1980 arranging for Iran to hold the hostages until after the Presidential election to undemine the reelection bid of Jimmy Carter - this on dates when Secret Service logs show Bush in a large number of campaign appearances in the United States.) When Hersh found out he had been conned he claimed that he had "documentation" from "a private detective" confirming Ben Menashe's story. A few days later the Sunday Times revealed the "private detective" was actually Joe Flynn, a well known British hoaxer, who admitted he had deceived Hersh for money (almost 1300 English pounds delivered by Hersh's British publisher). "I am a conman," Flynn told the Times.

I could go on ,but you get the point .



As for "military plans" ;we have gone over that before here. There are people at the Pentagon and at various military academies;think tanks et.al. ,who do nothing all day but plan and play war simualations. That is there job. Hersh ;who has been rabid anti-Bush for a long time;could have simularily claimed that the Pentagon has plans to invade Cuba and he would be correct .

He shouldn't worry . The time to act on Iran's main nuke sites was 2 years ago . With every passing day Iran gets stronger and more capable militarily . The comparison with Hitler is quite correct . Two years ago the Iranian miltary resembled the German miltary 1936 . With each passing day they get closer to resembling the German Army of 1939.

Iran has just concluded 'Great Prophet' war games . The demonstrated some sophistication that suprised the West .

They demonstrated a sophisticated command and control...headquarters, subordinate headquarters, linked by secure communications, linked to a National Command Center at Tehran.

conducted successful simulation of ballistic missile launch including nuclear capable MIRVed rockets .Missile type was identified as Fajar 3, or Victory 3 which resembles the body of a Russian SS-4 .

Test fired two types of underwater warheads ,both Russian designs. The cavitation weapon is likely the KV111 which maneuvered the shallows and depths of the Straits of Hormuz.The 'Whale torpedo ' built for targeting big ships, such as carriers or supertankers was also used in the Straits.

They used 2 types of submarines ;rebuilt german electric models deployed in the Persian Gulf and Russian Kilo class ,possibly with silencing technology used in the Indian Ocean from Mozambique to Pakistan.

Command and control and communications for all this equipment was successful which possibly suggests the use of AWAC type planes and maybe Chinese satellites .

This all suggests that at some date in the not too distant future they intend to block and bottle neck the Straits and to attack shipping ,and oil facilites in the gulf .

Meanwhile ;to Seymore Hersh : this is not the first time in recent years that you have undermined the adminstration by publishing detailed information about US plans towards Iran .Jan.2005 Hersh broke in a story that :

The Administration has been conducting secret reconnaissance missions inside Iran at least since last summer. Much of the focus is on the accumulation of intelligence and targeting information on Iranian nuclear, chemical, and missile sites, both declared and suspected. The goal is to identify and isolate three dozen, and perhaps more, such targets that could be destroyed by precision strikes and short-term commando raids. "The civilians in the Pentagon want to go into Iran and destroy as much of the military infrastructure as possible," the government consultant with close ties to the Pentagon told me.

Some of the missions involve extraordinary cooperation. For example, the former high-level intelligence official told me that an American commando task force has been set up in South Asia and is now working closely with a group of Pakistani scientists and technicians who had dealt with Iranian counterparts. (In 2003, the I.A.E.A. disclosed that Iran had been secretly receiving nuclear technology from Pakistan for more than a decade, and had withheld that information from inspectors.) The American task force, aided by the information from Pakistan, has been penetrating eastern Iran from Afghanistan in a hunt for underground installations. The task-force members, or their locally recruited agents, secreted remote detection devices-known as sniffers-capable of sampling the atmosphere for radioactive emissions and other evidence of nuclear-enrichment programs.




Title 18 of the United States Code (Section 794, subsection [b] reads`Whoever, in time of war, with intent that the same shall be communicated to the enemy, collects, records, publishes, or
communicates, or attempts to elicit any information with respect to the movement, numbers, description, condition, or disposition of any of the Armed Forces, ships, aircraft, or war materials of the
United States, or with respect to the plans or conduct, or supposed plans or conduct of any naval or military operations, or with
respect to any works or measures undertaken for or connected with,or intended for the fortification or defense of any place, or any other information relating to the public defense, which might be useful to the enemy, shall be punished by death or by imprisonment for any term of years or for life.


But none dare call it treason.

ETWolverine rated this answer Excellent or Above Average Answer
paraclete rated this answer Excellent or Above Average Answer

Question/Answer
ETWolverine asked on 04/06/06 - More on Immigration.

A speech given by Harry Reid on the Senate floor in 1993. (Just read the bolded parts if this is too long for you.)

Congressional Record Senate
Monday, September 20, 1993 No. 123
139 Cong Rec S 11996
REFERENCE: Vol. 139 No. 123
TITLE: IMMIGRATION

TEXT: [*S11996] Mr. REID. Mr. President, a few years ago it would have been unlikely that the subject of immigration would have come up around the family dinner table. But today, along with such matters as the economy, health care, and crime, immigration has become one of the most serious domestic issues in the minds of the American public, perhaps because immigration has had such enormous impact in all other domestic issues. It does not matter where you go today any townhall meeting, any gathering of high school students, at a college class immigration is one of the topics that is raised very quickly.

The issue of immigration is something that was not around just a few years ago with such intensity.

I know that many of my colleagues in the Senate and the House have had the same experiences I have had.

When I speak about immigration being a subject of concern, I am not talking about to this Senator. I think I can talk for the House and the Senate.

Earlier this summer, the Republican leader, Senator Dole, convened a policy breakfast for Republican Senators on the subject of immigration because, in his words, wherever he travels around the country, he is confronted with concern about the direction of the policy of immigration as it relates to the United States.

It is clear that there is growing public dissatisfaction with our Nations immigration policies. [*S11997]

The American people are demanding reforms that will restore order to an immigration system they perceive to be out of control.

And these are not racist people who are raising this issue.
Everybody in this country of course, except for the native Americans is of immigrant stock. My father-in-law was born in Russia; my grandmother in England. If we understand our own family backgrounds, we understand the immigrant struggle. The continual flow of new people into our country is one of its strengths.

But why, Mr. President, are the American people concerned? Why are they concerned about the present state of immigration laws? Here is why:

The most recent studies show that the net costs of legal and illegal immigration to all levels of Government will be $45 billion over the next decade. And this figure takes into account the taxes paid by these immigrants.

According to the Attorney General, 26 percent of Federal prisoners are noncitizens, at a cost of $30,000 per inmate per year.

When I first heard that figure, I thought there was a mistake, Mr. President 26 percent of the people that are in our Federal institutions are noncitizens. But the fact is, that figure was not something that I heard wrong; the fact is that it is true.


The amount that it costs us to incarcerate, care for, and keep those people from the public is the same as for someone that is a citizen $30,000 a year.

In 1982, 128,000 immigrants received Social Security benefits. In 1992, 10 years later, the number has jumped to over 600,000. Mr. President, that is an increase of almost 400 percent.

In 1986, we granted amnesty and I voted against that provision in law we granted amnesty to 3.2 million illegal immigrants. After being in this country for 10 years, the average amnesty recipient had a sixth-grade education, earned less than $6 an hour, and presently qualifies for the earned-income tax credit.

Last year alone, the Immigration and Naturalization Service estimated 3.5 million illegal border crossings occurred. Of course, INS apprehended only 1 million. These aliens came from 52 different countries but, of course, most of them, Mr. President, came from Mexico.

The INS currently employs 3,800 Border Patrol agents. Keep in mind the job that these people have 3,800 Border Patrol agents. Last year, the United States had over 500 million entries. Thirty-eight hundred people cannot control 500 million. This does not take into consideration much of the other paperwork that the agents have to deal with.

INS has only 6,200 beds available for detaining 252,000 asylum applicants, and those awaiting deportation.

In 1993, in Los Angeles County, at Los Angeles County Hospital, one of the largest hospitals in the country, 67 percent of the births were to illegal alien mothers.

The State of California needs to build a school a day to keep up with the incoming immigrant children a school a day.

According to a recent study out of Dartmouth, for every seven immigrants who enter the job market, one blue-collar American worker loses a job.


The INS routinely issues work authorization without regard to the number of jobs the economy is creating.

The top five States receiving immigrants California, Florida, Texas, New York, and Illinois have higher unemployment rates than the national average. It is no wonder.

In certain professions for example, engineering we have a surplus of labor in this country. However, the Labor Department allows American companies to bring in foreign engineers.

In 1990, we took in about three times as much legal immigration not talking about illegal legal immigration as Australia, Canada, France, and Great Britain combined.

All polls show Americans want lower levels of immigration and want the borders secured. This includes a majority of Latinos, 65 percent of whom believe there are too many immigrants.

The American people, Mr. President, are upset, and I think they have a right to be upset. But they are upset for a reason. Our immigration policies, regulating all aspects of entry to the United States, have simply ceased to function in the national interest. Immigration policy and national interest are terms that are rarely heard in the context of immigration. We seem to have lost sight of the fact that it is a public policy and, like all public policies, our immigration policies should serve the public interest. But they do not.

Let us talk about legal immigration.

We now admit the equivalent of a major city each year, without having the vaguest idea of how we will educate all the new children, care for the sick, provide housing, jobs, build infrastructure, or attend to any of the human needs of the newcomers or those already here.

Mr. President, each year, we admit I repeat the equivalent of a major city. We admit more people each year than make up some of our States. We admit a new State with legal immigrants every year.

At a time of huge budget deficits and severe financial constraints, we have no idea of how these huge costs will be borne. We just do it.

We admit the equivalent of a major city without any assessment of whether these newcomers are likely to be contributing members of our society. Only a tiny fraction of those admitted each year enter because they have skills and abilities that will benefit our country. The rest come merely because they happen to be relatives of other recent immigrants. The result of this so-called policy is that there is now a backlog of almost 3 1/2 million people the population of a city the size of Los Angeles who have a claim to immigrate to the United States for no other reason than they are somebodys relative. Is this really a way to run immigration policy?

Earlier this year, Congress engaged in a bitter and protracted debate about the Presidents job stimulus program, something this Senator supported. While there was a great deal of controversy about how best to put millions of unemployed Americans back to work, no one on either side of the aisle disputed the need to create more jobs in this country.

I did not say put people back to work because some of the programs we are working on are designed to give people a job for the first time. Yet, not once during the course of that debate during 1992 and I listened to most of it did we ever stop to consider that our Government was granting more work authorizations to foreign workers than the net number of new jobs created by our economy and certainly the job stimulus program would not have kept up with the new immigrants. Can anyone fathom a logic behind this policy? I have thought about it a lot and I cannot determine the logic.

Because a relative-based immigrant system is inherently unfair, Congress has been forced to adopt even more absurd measures to deal with the demands of people who do not have relatives here. Can it be any wonder that the American people think our immigration policies are a joke when we select 40,000 new immigrants a year by lottery? Can anyone honestly tell me it is sound and rational to make public policy based on the roll of a die or the flip of a coin? Heads we admit you, tails you stay home.

But the absurdity does not stop with our legal immigration policy. When it comes to enforcing laws against illegal immigration, we have a system that will make you recoil in disbelief. We now have a permanent illegal alien population of 4 million people we think. We think that is all it is. That is more than two times larger than the State I represent, the State of Nevada 4 million people. The illegal alien population is growing by more than a quarter of a million people a year, we think, with the best statistics we have. Yet we are doing almost nothing to encourage these people to go home or even to deter them from coming here in the first place. In many parts of the country we actually make it easy to be an illegal alien.

Listen to this. New York City, always on the brink of financial insolvency, guarantees that is right, guarantees that every illegal alien who lives within the five boroughs can have the same access to every public service and benefit that an American citizen has access to. California, also teetering on the brink of bankruptcy, will spend $1 billion this year to provide health care to illegal aliens. We already talked about the schools necessary to be constructed. Not just emergency health care does Califronia provide. We can all understand providing emergency care. But routine care and elective care is something they get in California.

It is not just California. The public hospitals in Texas have become the equivalent of the family doctor for thousands of people who live on the other side of the border in Mexico.

In 48 of 50 States, illegal aliens can get a drivers license, the de facto ID document in this country, without having to prove that they are legally in this country in the first place. If you think this is no big deal, think again. Mohammed Salameh, an illegal alien and the alleged driver of the bomb-laden truck that blew up the World Trade Center, happened to live in New Jersey illegally, of course one of the two States that actually bothers to require proof of legal residence.

So he crossed the Hudson River and got himself a drivers license in New York very simple. He was not a legal resident of the United States. He was not a resident of New York legally or New Jersey or any State. But the State of New York handed him that vital document needed to live and work, not to mention rent a truck that he could place explosives on, with no questions asked and no proof required. And we wonder in this country why we have an illegal immigration problem.


If making it easy to be an illegal alien is not enough, how about offering a reward for being an illegal immigrant? No sane country would do that, right? Guess again. If you break our laws by entering this country without permission and give birth to a child, we reward that child with U.S. citizenship and guarantee full access to all public and social services this society provides. And that is a lot of services. Is it any wonder that two-thirds of the babies born at taxpayer expense in county-run hospitals in Los Angeles are born to illegal alien mothers?

Just when the American people think nothing can be more absurd than the way we deal or rather do not deal with illegal immigration, they discover we have a political asylum system that would qualify us for Senator Proxmires Golden Fleece Award 1,000 times over. I do not know why he did not make this award; he should have. Last year more than 100,000 people showed up in this country, landing at our airports, washing up on our shores in leaky boats, crossing our borders illegally, overstaying their visas, saying two magic words, political asylum, and virtually assuring themselves of being allowed into the United States forever.

Anyone, no matter how specious the claim, can utter those words and usually within a matter of hours be released on their own recognizance onto the streets of our country with a promise that they will show up for a hearing 18 months later. Can anyone really say they are surprised that the vast majority of asylum applicants never ever show up for these hearings?

I wish the statements I have been making were some kind of a nightmare or dream, an aberration. But they are true. But they do not stop. Not only do we admit more than these 100,000 people each year without knowing who they are or why they came, we actually give them all the documents they need to simply disappear into our society.

I live in a suburb of Washington. It is near the CIA. There, if you drive down Dolley Madison Boulevard, people have placed there a little memorial for the people who were gunned down earlier this year by a man by the name of Mir Amil Kanzi, a Pakistani citizen. He entered the United States illegally, lived here for a year as an illegal alien, obtained a new passport from the government he claimed to fear, and yet he was still able to show up at an INS office in Virginia and file a claim of political asylum. Was this guy given a quick hearing and sent home? No. He got work authorization, a social security card, a drivers license, and the ability to obtain an AK-47 assault rifle and gun down CIA workers.

We hear from the American people when they appear in Town Hall meetings, [*S11998] or when students at universities and high schools all over this country write to us. They are not imagining things. They are not, as some apologists for the status quo contend, trying to blame immigrants for their problems. They are simply recognizing facts. Americas immigration policies are in a shambles, and they should be fixed, and we can no longer ignore them.

It is as important and as realistic as dealing with health care, as dealing with our defense policy, any problem you want to talk about. This is right up at the top.

On August 4, I introduced the Immigration Stabilization Act, S. 1351, which calls for a comprehensive overhaul of our Nations immigration policies. For the first time in more than a quarter of a century, S. 1351 seeks to institute major and meaningful reforms to a policy that lacks purpose and direction. While other legislation has been introduced in this session of Congress to address some of the most egregious abuses of our immigration laws, this legislation is the only one that institutes comprehensive reforms of the entire process. It does not pick at the sides. It turns our immigration policy upside down and takes a real good, hard look at it.

I urge my colleagues to study this bill and join me in sponsoring this legislation, in redesigning U.S. immigration policy for the next century.

In June, Jim Hoagland, of the Washington Post, called Immigration: Topic A. I do not know Jim Hoagland. To my knowledge I have never seen him. I looked at this article and it looked like something worth reading. I agreed with him. He said that immigration is topic A throughout the developed world. It is a complex and multifaceted issue that is likely to be with us for a long time to come. I believe it is imperative that the Congress of the United States, working in conjunction with the administration, begin to treat immigration as a serious national and international issue.

Events of the past several months have begun to jolt us, I hope, out of our complacency about the state of U.S. immigration policies. Rampant and well-documented abuse of our asylum laws, the bombing of the World Trade Center, and the foiled attempt to commit other acts of terrorism in New York, fiscal crises in California, and other events will turn immigration, I hope, into a high-profile issue.

We would be mistaken as a body to deceive ourselves into believing that immigration is a flash-in-the-pan issue, that if we wait long enough it is going to go away. It is not going to go away. I have given these numbers. It is only going to get worse.

We must have the courage and the foresight and the tenacity to deal with this complex and controversial issue now before it becomes more complex and more difficult. It is clear to everyone that our current laws and regulations are unworkable. I hope it is clear because it is to those I have discussed this with. As one colleague told me, the more you learn about our immigration process, the bigger mess you see it is. It is time to begin cleaning up that mess.

In the coming weeks and months I am going to be speaking from this spot on the Senate floor about the whole range of issues covered by our immigration and refugee laws. I will explain in detail why I believe it is not enough to tinker, as I said, at the margins of U.S. immigration law, but why the United States must do comprehensive reforms that conform to the realities of the era in which we live.

Proponents of the status quo argue that any attempt to reform our Nations immigration policies is an affront to our immigration tradition. It is an argument I hear over and over each time we attempt to have meaningful debate about U.S. immigration policy. It is important, therefore, to begin by laying to rest some of the myths that are routinely promulgated to stifle debate on this important issue.

Upon close scrutiny immigration in the United States today is anything but traditional. From 1820 until 1965, a period encompassing most of the history of this Nation, the United States admitted an average of 300,000 immigrants a year. During that 145-year period, we settled the frontier, fought a civil war, created an industrial revolution, engaged in two world wars, endured a Great Depression and ultimately emerged as the worlds greatest military and economic superpower. There have been ebbs and flows of immigration over this period but, taken together, 300,000 immigrants a year is our true immigration tradition. With the exception of one relatively short period at the beginning of this century, those levels have rarely varied.

Contrast those levels of immigration with the numbers we are seeing today. Depending on estimates of illegal immigration, which we have trouble determining, we are now resettling between 1.2 million and 1.5 million newcomers every year about the size of the State of Nevada, every year. There is nothing traditional or rational, for that matter, about attempting to absorb the population of Nevada every year, year in and year out. When history is written, the decade of the eighties will be remembered as a decade of wretched excess and, true to form, immigration has been one of those excesses. As we focus on reining in many of the other excesses in the preceding decade, we must not overlook immigration.

Unless serious reforms are undertaken, 15 million newcomers will settle in the United States during the 1990s. Yet, we continue to act as though immigration exists in a vacuum. Not a whit of thought has been devoted as to how we will meet the needs of that population. There has not been a single debate in Congress about how we will provide, not a first class education, but any education to all those new children, providing quality health care to the sick, secure adequate and affordable housing, ensure that there will be sufficient jobs, improve and expand infrastructure, cope with environmental degradation, or ensure domestic tranquility. In short, we are admitting unprecedented numbers of new immigrants without even a modicum of planning. We are simply taking a leap of faith that somehow everything will work out.

Those who find comfort in the knowledge that they are not alone in dealing with the problem will be reassured to note that virtually every industrialized democracy is also dealing with this thorny issue. The problem is they are way ahead of us. In Europe, Canada, Australia, and elsewhere, this debate policy is probably 10 years ahead of us.

In most of those countries, they have concluded that immigration policies must be guided by a rational assessment of their national interests, not by emotion, not by nostalgia. In a world that is growing at the rate of nearly a billion people a decade, with the bulk of that growth occurring in underdeveloped and impoverished nations, most other developed nations have concluded that massive migration can no longer be the solution to the worlds problems. They have come to the sobering recognition that large-scale immigration holds the potential to destroy the hopes and aspirations of their own citizens without really improving the prospects for most of the world.

Let us look at what todays immigration levels will mean to the lives of our children and our grandchildren. A Census Bureau report issued last December indicated that at our current immigration pace, the population of the United States will increase to 383 million people by the year 2050. That constitutes a 50-percent population increase in less than 60 years, and the majority of that growth will be directly attributable to immigration that takes place after 1990. It is as though we will absorb the entire population of Japan within the lifetimes of todays children and young adults.

Compare it, for example, to Japan. I am not defending their immigration policy. I do not like it. They do not have any immigration basically. And you wonder why their economy is more stable than ours.

The most important question we need to ask is whether our children and grandchildren will be better off as a result of this staggering increase in population. Will our children and grandchildren enjoy the same quality of life as we have had? Will they have the same opportunities, the same freedom of movement and mobility? Can the melting pot absorb and absorb and assimilate and assimilate people arriving at this rate? Or will we become increasingly isolated and alienated from one another?[*S11999]

We must not simply leave the answers to these questions for fate and circumstance to decide. We, in Congress, who make the policies that future generations will have to live by, owe it to them to act responsibly to lead this debate. Under the legislation I have proposed, I will begin to restore immigration to its traditional and more manageable level of about 300,000 annually. Mr. President, I do not think that is really being tough on people. So it is really a lot of people. So I suggest that we should have that as a beginning point: 300,000 annually to put the excesses of our current immigration numbers in perspective. Even if we were to enact this decrease of more than 50 percent, the United States would still have the most open and generous immigration policy in the world.

More importantly, the Immigration Stabilization Act of 1993 would institute the kinds of immigration policy that the American people want. Americans are generous people who are justly proud of our tradition of welcoming newcomers to our society, but all indicators of public opinion show that the vast majority of Americans of all racial and ethnic groups want to see immigration returned to the more manageable and traditional levels.

The United States must, once and for all, adopt a comprehensive plan on immigration. The time for ad hocracy has passed, and we must begin to think in terms of integrated policy that serves the needs of the Nation and the people who make up this Nation.

In the coming weeks, I will spell out in detail the specifics of the Immigration Stabilization Act. It is designed to reform all aspects of immigration and refugee policy and will provide the United States with a workable blueprint for dealing with current world and national realities. Among the key issues I will discuss are: First, a comprehensive ceiling on immigration of 300,000 annually, and the implementation of a fair and nondiscriminatory system for selecting new immigrants to the United States;

Second, a generous refugee resettlement policy that depoliticizes the process to ensure protection is granted based on individual need, not political considerations;

Third, an overhaul of our political asylum process to eliminate the kinds of rampant abuse that has attracted so much media attention and public outrage in recent months;

Fourth, a plan to rid this country of the burden of people who are known criminals who prey on the American public and become a drain on our resources;

Fifth, assuring the people who are given the privilege of sponsoring new immigrants to our country live up to the financial commitments they make to ensure these people do not become a burden to the taxpayers;

Sixth, the effective deterrence of illegal immigration through enforcement of employer sanctions, including the adoption of a verifiable, tamperproof document to ensure that only those people who are legally in the United States work and collect benefits here. So we do not get frightened, this simply would be an enhanced Social Security card;

Seventh, we need to regain control of our borders through additional border patrol manpower and other security control measures and the adoption of a $3 border crossing fee to provide the INS the resources it needs to do the job. Every person who comes across should pay;

Eighth, an all-out crackdown against the practice of smuggling illegal aliens into the United States for profit. Our lax enforcement of immigration and asylum laws has created what can be termed as modern day slave trade;

Ninth, ensuring Federal immigration authorities receive the full cooperation of State and local governments in enforcing laws against illegal immigration;

Tenth, a clarification of the intent of the 14th amendment regarding who is entitled to U.S. citizenship.


Mr. President, we have no choice but to address all these difficult issues. We can no longer satisfy ourselves with cosmetic changes that do not address the underlying truth that America lacks an immigration policy capable of coping with the realities we are facing.

Websters defines the term policy as prudence or wisdom in the management of affairs. By that definition, it is fair to say that this country does not have a policy but, rather, an inchoate and often incomprehensible hodgepodge of statutes, regulations and procedures. In other words, what we have is a mess that is only likely to get worse unless we demonstrate some courage legislatively.

And it is time, Mr. President, we had this policy. We must be prepared to address this issue maturely, without acrimony. Just as we recognize the need to have reforms in our health care system is in no way an indictment of doctors, hospitals, and nurses but, rather, a public policy imperative, so too must we approach immigration. Reforming an immigration system that does not work is a public policy imperative and we in Congress must not shirk from these responsibilities.

I yield the floor, Mr. President.

---------------------

Quite the speech from Mr. Reid.

Not at all the policy he backs now, however. He now claims that this speech was the biggest mistake of his life, and that the Bill itself was a terrible idea. He pulled the bill just a few weeks later, claiming that he realized what a terrible mistake it was as soon as he proposed it.

Interestingly, though, he re-proposed the bill again one year later in 1994, and argued in favor of it all through the summer of 1994 in committee and on the floor. Before pulling it a second time... supposedly because he again saw the error of his ways. Or something.

So what does Reid really believe on the immigration issue? We should surely take him at his word as to what he believes... but which word? The one he espouses now, or the one he espoused back in 1993 and 1994?

Looks like we need to pull out the John Kerry flip-flops again. Harry doesn't seem to be sure what he supports anymore.

Gee, I wonder if he considers people who back the ideas in this bill to be racists. After all his defending of the position as not being a racist one (even Latinos support it, people of all backgrounds want change, etc.,) does he now believe that those who support such a stance to be racists? He seems to, but who can be sure?

Your opinions, please.

Elliot

tomder55 answered on 04/07/06:

Reid did go to the floor this week to apologize for his previous position but as you have demonstrated he has a history of flip flop and subterfuge on the issue .He's not alone .

Barbara the Boxer had a California National Guard initiative to patrol the border to keep out illegals .

Also Sen Feinstein has in the past endorsed a fence along the Mexican border . Let's see how she votes this time .

ETWolverine rated this answer Excellent or Above Average Answer

Question/Answer
Erewhon asked on 04/07/06 - Never send to know for whom the bell tolls ...



The first major step leading to the "Final Solution" was the attempt on the part of the Nazi regime to force Jews to emigrate out of Germany. Hitler's motivation seems to have been two-fold: to ensure the racial purity of Germany and to create lebensraum, "living space," for German nationals of "Aryan" blood. His obsession with the former is reflected in the Nuremburg Laws of 1935.

Throughout the 1930's Nazi domestic policy was aimed at stripping Jews of any citizenship rights, economic and political rights. The first step toward a "Final Solution of the Jewish Problem" was the complete dehumanization of the Jew.

Hitler's preoccupation with lebensraum became patently obvious with the German invasion of Poland on September 1, 1939, Yugoslavia and Greece on April 6, 1941 and the Soviet Union on June 22, 1941. On the eve of the invasion of Poland, Hitler made a speech to his generals in which he said,

"Genghis Khan had millions of women and men killed by his own will and with a gay heart. History sees in him only a great state builder... I have sent to the east...my 'Death's Head Units,' with the order to kill without mercy men, women and children of the Polish race or language. Only in such a way will we win the 'lebensraum' that we need. Who, after all, talks nowadays of the extermination of the Armenians?"
(cf. Helen Fein, Accounting for Genocide, The Free Press, 1979:4).

Prior to his rise to power in Germany, Adolph Hitler has spent several years in the extremely conservative political atmosphere of Vienna -- a city literally festering with German nationalism and antisemitism. A very strong German nationalist movement advocated union with Germany, the German domination of Europe and the expulsion of "inferior people."

Karl von Lueger, the mayor (or burgomaster) of Vienna in the late 1890's was of particular influence on young Hitler. Lueger was openly antisemitic and manipulated the antisemitic passions of Vienna into a major political victory, This was a lesson not lost on Hitler.

The publication of Darwin's Origin of the Species in the mid 1800's seems to have provided a basis for a vulgar theory of "survival of the fittest and racial theories of human behavior.

The ideas of Darwin were dramatic in their impact upon social and economic structures -- biological racism in the United States as a rationale for slavery and antisemitism in Europe as a rationale for cultural nationalism. Jews were blamed for all of the political and economic woes of Austria and Germany from the end of World War I. Hitler even blamed World War I on the "international Jewish economic conspiracy."

====

It tolls for thee?


tomder55 answered on 04/07/06:

this is not a valid comparison at all. All of us have praised the industriousness of the immigrant and have said we are more than willing to welcome them as LEGALS . Why is that so hard to comprehend ? I am tired of being labeled a racist ,a nativist ,an isolationist because I think it is reasonable for the country to know who is inside it's borders .

Erewhon rated this answer Excellent or Above Average Answer

Question/Answer
BeelzeBUSH asked on 04/07/06 - Globalization...

Question: What is the truest definition of Globalization?
Answer: Princess Diana's death.
Question: How come?
Answer: An English princess
With an Egyptian boyfriend
Crashes in a French tunnel,
Driving a German car
With a Dutch engine,
Driven by a Belgian who was drunk
On Scottish whisky, (check the bottle before you change the spelling)
Followed closely by Italian Paparazzi,
On Japanese motorcycles;
Treated by an American doctor,
Using Brazilian medicines.
This is sent to you by an American,
Using Bill Gate's technology,
And you're probably reading this on your computer,
That use Taiwanese chips,
And a Korean monitor,
Assembled by Bangladeshi workers
In a Singapore plant,
Transported by Indian lorry-drivers,
Hijacked by Indonesians,
Unloaded by Sicilian longshoremen,
And trucked to you by Mexican illegals.....
That, my friends, is Globalization



tomder55 answered on 04/07/06:

all to cover up her murder by her English husband.

BeelzeBUSH rated this answer Excellent or Above Average Answer

Question/Answer
Erewhon asked on 04/07/06 - Bush needs a mirror


"If there's a leak in my administration, I want to know who it is!"


For goodness sake put the poor fellow out of his misery.

Will you send him a mirror?

LOL

tomder55 answered on 04/07/06:

I guess it would be too simple to just brush this off as something that an indicted for perjury defendent says .So I won't go there. Besides ,the whole content of the MSM reporting on this is so rediculously and obviously wrong that it just proves my point again that they manipulate a story .

So what did Libby really say ? He said that he was authorized by President Bush, via Vice President Cheney, to leak "certain information" that was contained in the National Intelligence Estimate. That information was not about Valerie Plame.

Andrew McCarthy at National Review explains :

If you read far enough into the AP report I cite here, you will learn that the "certain information" is not otherwise specified. That is, we don't know what it was and whether it was classified.

Unfortunately, the AP reports that only after treating readers to Howard Dean's rant about how "The fact that the president was willing to reveal classified information for political gain and put interests of his political party ahead of America's security shows that he can no longer be trusted to keep America safe."

It is crucial to note here, however, that there has been no accusation -- none -- that the President or anyone else was willing to reveal, much less actually revealed, classified information. It is irresponsible to say such a thing based on the current record.

The National Intelligence Estimate has some classified sections, but it is not all classified. Indeed, it is mostly not classified and made available for public consumption. Libby was evidently allowed to talk about some parts of it before it was formally released to the public. But it is important to bear in mind that it was going to be released to the public. Classified information is not released to the public.

Almost certainly, what Libby was permitted to do was preview for certain reporters some of the highlights of what was shortly going to be made public in the NIE. That is, NOT disclose the classified information, but talk about what was going to be in the public domain. This is something that is done everyday in Washington -- probably even by Howard Dean.

It would have been an act of political insanity not to do such a thing. There were (and are) people in the government (particularly at the CIA and the State Department) who vigorously opposed the Bush foreign policy. They have leaked things left and right since the president took office, and it would be ridiculous to think they would not have put their spin on the NIE -- just the way Joseph Wilson put his misleading spin on something that, in all likelihood, was actually classified and should not have been spoken of publicly (much less made grist for a NYTimes op-ed), namely, the substance of his trip to Niger.

If administration officials like Libby did not speak to the press about what was going to be in the NIE, the American people would only have heard from people like Wilson and others opposed to the President's policies. One can only imagine how Dean would have played off that one-sided version of events ... and I have a slight suspicion -- call me crazy -- that he would not have been complaining about leaks of classified information from those sources.

In any event, when you look at these stories, it is very important to parse carefully how the press is describing Libby's testimony. The impression being (consciously) created is of recklessly irresponsible leaking of classified information. The reality is very likely far from that. Remember, leaking classified information can often be a crime, yet no one has been charged with such an offense.


Read the The National Intelligence Estimate and you will see for yourself that it was the consensus view of the national intelligence agencies that Saddam maintained a WMD program and that :

-Iraq is continuing, and in some areas expanding its chemical, biological, nuclear and missile programs contrary to UN resolutions.

-Iraq possesses proscribed chemical and biological weapons and missiles.

-Iraq could make a nuclear weapon in months to a year once it acquires sufficient weapons grade fissile material.


This was the intel. that Bush had when he made the decision to go to war .The British and the collective intelligence agencies of the world agreed with this assessment .

In the summer of 2003 the administration was besieged with leaks from opponents in the CIA and elsewhere, as well as op-eds by the likes of Joe Wilson, that misrepresented the state of the intelligence prior to the Iraq war. In order to deal with these false claims, the administration declassified the above 2002 intelligence estimate. The "leak" that you're reading about in headlines was simply the permission given to Scooter Libby to describe the contents of the intelligence estimate a few days before it was officially declassified and made public.

So in the MSM, the false leaks by Bush opponents are noble, while the administration's declassification of the report that shows them to be false is a scandal. See what I mean ???









Erewhon rated this answer Excellent or Above Average Answer
Itsdb rated this answer Excellent or Above Average Answer
purplewings rated this answer Excellent or Above Average Answer

Question/Answer
ROLCAM asked on 04/06/06 - A just peace or no peace - A first class article.


> A just peace or no peace.
>
> Israeli unilateralism is a recipe for conflict - as is the west's
> racist refusal to treat Palestinians as equals
>
> Ismail Haniyeh
> Friday March 31, 2006
> The Guardian
>
> Do policymakers in Washington and Europe ever feel ashamed of their
> scandalous double standards? Before and since the Palestinian
> elections in January, they have continually insisted that Hamas
> comply with certain demands. They want us to recognise Israel, call
> off our resistance, and commit ourselves to whatever deals Israel
> and the Palestinian leadership reached in the past.
>
>
> But we have not heard a single demand of the Israeli parties that
> took part in this week's elections, though some advocate the
> complete removal of the Palestinians from their lands. Even Ehud
> Olmert's Kadima party, whose Likud forebears frustrated every effort
> by the PLO to negotiate a peace settlement, campaigned on a
> programme that defies UN security council resolutions. His
> unilateralism is a violation of international law. Nevertheless no
> one, not even the Quartet - whose proposals for a settlement he
> continues to disregard, as his predecessor Ariel Sharon did - has
> dared ask anything of him.
>
>
> Olmert's unilateralism is a recipe for conflict. It is a plan to
> impose a permanent situation in which the Palestinians end up with a
> homeland cut into pieces made inaccessible because of massive Jewish
> settlements built in contravention of international law on land
> seized illegally from the Palestinians. No plan will ever work
> without a guarantee, in exchange for an end to hostilities by both
> sides, of a total Israeli withdrawal from all the land occupied in
> 1967, including East Jerusalem; the release of all our prisoners;
> the removal of all settlers from all settlements; and recognition of
> the right of all refugees to return.
>
>
> On this, all Palestinian factions and people agree, including the
> PLO, whose revival is essential so that it can resume its role in
> speaking for the Palestinians and presenting their case to the
> world.
>
>
> The problem is not with any particular Palestinian group but with
> the denial of our basic rights by Israel. We in Hamas are for peace
> and want to put an end to bloodshed. We have been observing a
> unilateral truce for more than a year without reciprocity from the
> Israeli side. The message from Hamas and the Palestinian Authority
> to the world powers is this: talk to us no more about recognising
> Israel's "right to exist" or ending resistance until you obtain a
> commitment from the Israelis to withdraw from our land and recognise
> our rights.
>
>
> Little will change for the Palestinians under Olmert's plan. Our
> land will still be occupied and our people enslaved and oppressed by
> the occupying power. So we will remain committed to our struggle to
> get back our lands and our freedom. Peaceful means will do if the
> world is willing to engage in a constructive and fair process in
> which we and the Israelis are treated as equals. We are sick and
> tired of the west's racist approach to the conflict, in which the
> Palestinians are regarded as inferior. Though we are the victims, we
> offer our hands in peace, but only a peace that is based on justice.
> However, if the Israelis continue to attack and kill our people and
> destroy their homes, impose sanctions, collectively punish us, and
> imprison men and women for exercising the right to self-defence, we
> have every right to respond with all available means.
>
>
> Hamas has been freely elected. Our people have given us their
> confidence and we pledge to defend their rights and do our best to
> run their affairs through good governance. If we are boycotted in
> spite of this democratic choice - as we have been by the US and some
> of its allies - we will persist, and our friends have pledged to
> fill the gap. We have confidence in the peoples of the world, record
> numbers of whom identify with our struggle. This is a good time for
> peace-making - if the world wants peace.
>
>
> Ismail Haniyeh is the new Palestinian prime minister and a Hamas
> leader. Email: ihaniyyeh@hotmail.com

#####################################################

Please read it.
Tell us what you think ?
Is Ismail correct in what he is saying?

tomder55 answered on 04/06/06:

thanks for the belly laughs.

Funny way they attempt peace . The rockets fired from Gaza are becoming more frequent ,more powerful ,with a longer range.

Israel has done alot of unilateral action . They have unilaterally withdrawn from Gaza without any return concession by the Palestinians . They have dismantled some of the West Bank settlements without reciprocal concessions . Israel attempted to comply with the agreed upon 'Road Map ' while the Palestinians did nothing to comply .


The Israelis have for the past 15 years have tried to come to an accomodation with a people who's charters declare that Israel has no right to exist .If Israel agreed to retreat to the 1967 borders the Palestinians will still make a claim that their land is occupied .

Hamas and the PLO ? What is the difference ?.....only that Hamas get's it's marching orders from Iran ? Yes they were 'democratically elected ' .That does not mean that we should be jumping for joy . That just means that the population of the Paletinians have confirmed that the murderous goals of Hamas are their goals also . Why should they be rewarded for that ?

ETWolverine rated this answer Excellent or Above Average Answer
excon rated this answer Excellent or Above Average Answer
JBodine rated this answer Excellent or Above Average Answer
ROLCAM rated this answer Bad/Wrong Answer

Question/Answer
BeelzeBUSH asked on 04/06/06 - Joe Gandelman commentary (a moderate voice)



Bush Makes It Clear: Iraq War And U.S. Role Will Last Beyond His Term

by Joe Gandelman

To those who have been pressuring or clamoring for the U.S. to end the war in Iraq or pull out a significant number of troops, President George W. Bush had this message in his press conference: not on MY watch.

He made it clear that (a) he stands by existing policy, (b) it'll be up to the next president to decide when to pull the troops out (c) he stands solidly behind Secretary of Defense Donald Rumsfeld and is brushing aside calls for Rumsfeld's resignation (d) he rejects a top Iraqi politician's characterization of Iraq as being in a state of civil war.
President Bush said Tuesday that American forces will remain in Iraq for years and it will be up to a future president to decide when to bring them all home. But defying critics and plunging polls, he declared, "I'm optimistic we'll succeed. If not, I'd pull our troops out."

The president rejected calls for the resignation of Defense Secretary Donald Rumsfeld, chief architect of wars in Iraq and Afghanistan. "Listen, every war plan looks good on paper until you meet the enemy," Bush said, acknowledging mistakes as the United States was forced to switch tactics and change a reconstruction strategy that offered targets for insurgents.

He also rejected assertions by Iraq's former interim prime minister that the country had fallen into civil war amid sectarian violence that has left more than 1,000 Iraqis dead since the bombing last month of a Shiite Muslim shrine.

"This is a moment the Iraqis had a chance to fall apart and they didn't," Bush said, crediting religious and political leaders with restraint.
Indeed, if there seemed to be policy thread in this it couldn't be called neocon-ism anymore but an official policy of "optimism."
The Guardian's lead puts it even more succinctly:
George Bush yesterday raised the vision of an American troop presence in Iraq that would extend for the next three years, an admission that a US withdrawal was unlikely during his term in the White House.

With US forces entering their fourth year in the country since the invasion, Mr Bush's comments suggest he foresees a longer military commitment in Iraq than that experienced by troops in the second world war or the Korean war.

Asked at a White House press conference whether he could foresee a complete withdrawal from Iraq, Mr Bush held out little hope. "That will be decided by future presidents and future governments of Iraq", Mr Bush told reporters.
What can you say about this? It underscores Bush's commitment to see the war through. But it also also rips away any fig leaf that Republicans who are running in 2006 could have used when confronted with rising bipartisan souring on the war, since Bush now makes it clear that there will be no big troop reduction and that he apparently sees no major flaws in existing policy, the implementation of it or the quality of the job performance of officials assigned to oversee it.

To Washington Post op-ed columnist Eugene Robinson, what's unfolding is "The Planet of Unreality":
This is not good. The people running this country sound convinced that reality is whatever they say it is. And if they've actually strayed into the realm of genuine self-delusion if they actually believe the fantasies they're spinning about the bloody mess they've made in Iraq over the past three years then things are even worse than I thought.

Here is reality: The Bush administration's handpicked interim Iraqi prime minister, Ayad Allawi, told the BBC on Sunday, "We are losing each day an average of 50 to 60 people throughout the country, if not more. If this is not civil war, then God knows what civil war is. Iraq is in the middle of a crisis. Maybe we have not reached the point of no return yet, but we are moving towards this point. . . . We are in a terrible civil conflict now."

Here is self-delusion: Dick Cheney went on "Face the Nation" a few hours later and said he disagreed with Allawi who, by the way, is a tad closer to the action than the quail-hunting veep. There's no civil war, Cheney insisted. Move along, nothing to see here, pay no attention to those suicide bombings and death-squad murders. As an aside, Cheney insisted that his earlier forays into the Twilight Zone U.S. troops would be greeted as liberators, the insurgency is in its "last throes" were "basically accurate and reflect reality."

Maybe on his home planet.

What Bush essentially did during the press conference was to reaffirm his faith in the U.S. course, say he's not going to significantly change it and that it's up to a future President to do that. This likely won't be welcome news to Republicans because now, Americans who are either militantly against the war, troubled by it, or frustrated over the execution of it may feel that, since Bush has essentially rejected any compromise with the critics who now are now dotting diverse parts of the political spectrum, the only way to get a change is....change. Perhaps starting in November....







tomder55 answered on 04/06/06:

This was published on March 21 ;2 weeks ago. We have already dissected this issue .

To refresh your memory ,here was my reply to Herr Choux

And here is my response to Ronnie on a simular posting .

Allawi ;the former designated interm PM under the reign of Viceroy Bremer is a thug who personally conducted summary executions to send a message to his opponents . He is completely discredited in my book.


My take on it has not changed in the last 2+weeks .The Sunnis certainly would be crushed in a civil war, so they have nothing to gain. The Shiites will rule the country, civil war or not, so they have no reason to seek civil war. The Kurds just want to be left alone.

No Iraqi stands to benefit from a civil war and yet there is a constant drum beat about a civil war as if some are hoping that by predicting it ,it will become a self -fullfilling prophesy .

BeelzeBUSH rated this answer Excellent or Above Average Answer

Question/Answer
paraclete asked on 04/06/06 - Now this is what I call absolutely rediculous

Grandmas 'new face' of terror
From: Agence France-Presse From correspondents in London
April 06, 2006
TWO British grandmothers are facing up to a year in prison after being arrested under new anti-terrorism legislation that outlaws protests at military bases, The Independent newspaper said today.

Helen John, 68, and Sylvia Boyes, 62, were detained at the US early warning station at Menwith Hill, in North Yorkshire, northern England, on Saturday after deliberately setting out to highlight the change in the law, it said.
Civil liberties campaigners believe the legislation, which came into force last week and covers 10 bases, including nuclear research facilities, criminalises free speech.

Anyone who breaches the legislation faces a maximum 12 months in prison and/or a fine.

The newspaper said the women, who have 10 grandchildren between them and are veteran peace protesters, will learn whether they will face prosecution on April 15.

Ms Boyes, who with Ms John was detained with a placard denouncing the new law and US military policy, was quoted as saying: "I am quite willing to break the law and prepared to be charged and go to prison.

"The government thinks it can do whatever it wants and that it has a passive public which accepts whatever it throws at it. I find it very worrying."
The British Government, which defended the new restrictions, has been keen to push through new security laws since last year's suicide attacks on London, which killed 56, including the four bombers.

But civil liberties groups were quoted as saying it had crossed the "fine line" between protecting citizens and curbing peaceful protest and free speech.

The Independent highlighted a number of other cases in which it said protesters had been turned into "terrorists".

They included that of an 82-year-old peace activist who was evicted from the annual conference of British Prime Minister Tony Blair's governing Labour Party for heckling Foreign Secretary Jack Straw during his speech on Iraq.

Have people in this world gone so utterly mad that they should so lack dissernment that they should arrest a couple of old ladies with a placard?

tomder55 answered on 04/06/06:

I am not familiar with the law . As I read it in the article it says it outlaws protests "at military bases ". Does that mean inside a facility or outside it's grounds. Certainly a security perimeter can be set up around military bases and beyond that it would be reasonable to allow protests ;but I am not a Brit. and am not familiar with all the issues.

paraclete rated this answer Excellent or Above Average Answer

Question/Answer
JBodine asked on 04/04/06 - A story to be told

I am part of a graduate study group assigned to present papers, reports, and such at the university I'm working on my Masters at. The following story is what happened Saturday morning, while I was waiting for/working with my group at a particular coffee shop near a different, state-supported university. Elliot asked if I would relay it to you fine folks, as this type of thing goes on literally all the time. Read on:


So there I am, studying and preparing with my group at that coffee shop that I told you about, proudly wearing my Sayaret Givati recon shirt.

First off, all I got was evil looks from a few folks that looked like professor-types. I acknowledged their existence and attention with a courteous and professional nod. One of them walked by on his way to get a refill and muttered, "Homo..." At that time, a member of my group (25 year old female, not at all hard to look at) arrived and greeted me with a hug and a kiss on the cheek (not unusual, and not anything sexual). I went to refill my drink and whispered to the gentleman, "Guess again." I sat back down before his order was done, as it's a whole lot faster to refill a cup of Dr. Pepper than it is to manufacture a double-whammy Fleuvian-bean latte mocha with almond chips and cinnamon sprinkles (and who's the homo again?). On the way back, he said, "You're probably racist." FIve minutes later, the other member of my small group arrived, a stunningly beautiful black lady of about 35 to 40 years. We, too, greeted with hugs and such. I looked over at the table of professor types and said loudly enough for most to hear, "You know, for all your apparent education, you sure are a judgmental dumba$$. Furthermore, I bet you go on and on about "tolerance" when you're presuming to educate all those young, impressionable minds over at that ivory tower." Two of the other professor-types laughed, but he and another one just turned beet red with obvious rage. My two companions asked me quietly what was going on. I explained, and they laughed heartily, which simply further embarrassed the goofy ones. We got down to the academic business at hand then.

Somewhere along the line, the professors left, and sure enough, a group consisting of a mixture of wannabe-hippies and a couple of Arabic looking fellows had entered. The evil looks and glares from them would have actually been funny, if they weren't so pathetic. I was simply ignoring them, as I was engrossed in the topic at hand and beginning to have some real insights. However, my companions remarked that they were beginning to get a bit uncomfortable from the attention. So I walked over to their table, introduced myself, and offered my hand for a handshake. Believe it or not Elliot, I was being sincere in trying to show goodwill and non-judgment. One of the hippies actually took my hand and shook it, which showed me that at least he was raised right. Around my parts growing up, refusing a handshake was tantamount to challenging a person to a duel to the death. As the saying goes, "Never refuse a man's handshake. Fight him to the death afterward if you must, but shake his hand first." So there's at least hope for the one guy. The others just tried to continue glaring, but couldn't maintain eye contact. Perhaps it was the proximity.... In any event, I remarked that the ladies were uncomfortable with the looks they were casting our way, and asked politely if they might consider stopping, or if perhaps we had done something to offend them. The bolder one among them said, "You walk in here wearing an Israeli t-shirt, and have all these Christian textbooks out. How can we not feel threatened?" I asked if there was something about either Isreal or Christianity that they found threatening. He launched into the typical line about Zionist America's oppression of this group and that, and so on. I remarked that I bet he knew the professor that was in there earlier, but didn't elaborate. I asked if anyone at the table had ever travelled overseas, to ANYWHERE. Negatives all around. So these are all American born and bred idiots, rather than imported idiots. I followed by stating that I had been all over the world, both on Uncle Sam's dime and my own dime, and told them about legitimate oppression and terrorism that took place EVERYWHERE BUT HERE. I told them about Christians and Jews having to publically hide their identities, not from fear of ridicule, but from FEAR OF DEATH. Simply because of their faith. I said how glad I was to be back in the States, where I could practice my faith openly, and engage in civil debate and mutual learning with other people with no fear of reprisal or judgment, then added, "Or so I thought. Tolerance, huh? You make me want to vomit. In the words of Bob Geldof in 'The Wall,' 'You fill me with the urge to defecate.'" I'm not sure, but I think they were suitably embarrassed. Don't really care either way.

The manager of the shop came up to me after a sat back down, and I thought, "Oh, man. More trouble." However, it seems that she is of Jewish decent and faith, and said that our group could stay as long as we liked, and enjoy free refills of our beverages anytime we wanted to meet there again. Apparently, she gets more than her fill of the liberal a$$holes spouting crap in there, and was glad to see some folks who valued free expression also.

Can't wait until next Saturday!

tomder55 answered on 04/05/06:

good story . now what would've happened if you had encountered Ward Churchill ?

ETWolverine rated this answer Excellent or Above Average Answer
JBodine rated this answer Excellent or Above Average Answer

Question/Answer
excon asked on 04/05/06 - Delay Dedone Redux


Hello:

Da do you remember when I suggested that Delay was dedone?? Well I da do!

How about this prognostication? The Republicrats will lose their majority in the house.

da da excon

tomder55 answered on 04/05/06:

triumvirate in a couple of ratings mentioned gerrymandering . The fact is that the party in power has always used that tool to strenghten their support and to weaken the oppositions . I think that it will play a significant role in the coming mid-terms. The Democrats need a net 15 seat change to become the majority . I think they will fall short but I could be wrong . November is 8 months away and that is more than a life-time in politics. I think they would have a much better chance of it if they would not shoot themselves in the footsies by doing things like :

having Howard Dean as a party chair. They thought they could live with him because of his alleged fund raising skills .Those skills were over-rated and he has hurt the party by being the voice of the party .

having Nancy Pelosi being the future Speaker of the House .

Lining up behind cut and run pols like Murtha or grand standing hearings on "censure" by Feinstein .

Do I think they will gain control of the gvt. eventually ? Of course I do . I have been trying to make a point in regards to the war against jihadistan that it will be waged for a long time and Congress and Presidents from both parties will be responsible to conduct it . I have no expectation that there will ever be a complete consensus on how to wage it ;but in principle we ought to start with the premise that we ought to try to win it for the sake of our continued society if nothing else.

excon rated this answer Excellent or Above Average Answer

Question/Answer
Erewhon asked on 04/04/06 - Farm Workers need your help

New amendments challenge immigration reform--send an immediate e-mail

Vote may occur as early as 2:30 today!

We need your immediate help. Late yesterday, Senator Chambliss (R-GA) submitted amendments which would render the AgJobs legislation unworkable. The Chambliss amendments would eliminate the opportunity of farm workers to earn permanent resident status, and instead would create an unworkable "report to deport" program. His proposal offers no prospect for stabilizing the work force with legal-resident workers.

For those workers who might have had an interest in becoming blue-card guestworkers under his proposal, the obstacles--including heavy fines and unrealistic work requirements--would make the program inaccessible. His amendments also would eliminate important labor protections in the H-2A temporary foreign agricultural worker program, including slashing wage protections for US and foreign guestworkers. This amendment would turn back the clock to the pre-1966 days when migrant farmworkers were excluded from the minimum wage protections. These amendments are very similar to the amendments Chambliss offered last April which lost by a vote of 77-21.

Please make a difference to farm workers on this vital bill by immediately contacting your Senators to ask them to support the Senate Judiciary Committees bipartisan compromise billwhich includes the AgJobs amendment, oppose the Chambliss amendments, and oppose any amendments not supported by AgJobs primary sponsors.
(Senators Craig, Kennedy and Feinstein)

We need you to take immediate action on this vital issue. The vote could occur as early as 2:30 this afternoon.



Take Action at: http://www.ufwaction.org/campaign/chambliss


Tell-A-Friend: Tell your friends about this

====

tomder55 answered on 04/05/06:

I do not think it unreasonable for people who came here by breaking the law ;by cutting in line in front of honest people who wait for months if not years to get their work visas;to pay a fine and to take the proper steps to be properly documented as a prerequisite for remaining in the country .


In spite of the implied charge of racism that you leveled at ET ;it is not .It is just plain wrong to accept the status quo with millions of people who's first act in the country is an illegal one .If our economy is creating millions of new jobs and we do not have the work force to fill those jobs then the answer is to stream-line the visa procedures ;to increase the number of visas issued and to prioritize the applications for the jobs being created (that also includes increasing the number of hi-tech visas) .

If the AgriJobs were well paid and had the 'normal' benefits home-based American citizens expect, then there would be no need for immigrant labour

The jobs are NOT well paid ;and in fact the wages are substandard and illegal . But no one is addressing that end of the equation . Why do American employers get away with this ? China gets slammed for slave labor conditions but what is the difference? So you go to a Walmart Superstore and get clothing made by slave labor from China and produce made by slave labor from the good ole USA . I think Elliot has a very valid point about outsourcing . The net effect is the same . Frankly ,I'd prefer the lettuce was grown in Mexico.

Erewhon rated this answer Excellent or Above Average Answer
excon rated this answer Excellent or Above Average Answer

Question/Answer
Erewhon asked on 04/04/06 - Departed DeLay Delights White House



DeLay Departs as his Deputy Gets a swift dose of honesty.

QUESTIONS:

1) Did DeLay know about the corruption and greed going on under his nose, or

1) Was DeLay kept in the dark by his underlings even as DeLay's relatives were being enriched by the corruption?


tomder55 answered on 04/05/06:

The short answer is I do not know. I am pretty convinced that the charges against him by Ron Earle in Texas are bogus and will either not go to trial;will be dismissed ;or in the unlikely event that he goes to trial ,will be exonerated.

But I do not know how far he is involved in the Abramoff ,Scanlan ,Tony Rudy web. So far it is guilt by association but if I were to guess ,Delay will be found to be somehow involved .

I take him at his word when he says he resigned so that the seat can go to a different Republican . His case is simular to a powerful Democrat from recent times ;that being Dan Rostenkowski's 1994 campaign .

Rostenkowski was at one time the powerful chair of the House Ways and Means Committee.He represented a North Side of Chicago traditionally Democrat district and it was inconceivable that a Democrat would ever lose the seat .But ,he did lose it to a nobody because he had an indictment hanging over his head .

For Delay to stay in the House at this time is nothing but negative . He has lost all of his previous power in the House but him being their undermines the current Republican leadership . It is likely that if he continued his run for reelection he would lose a traditional Republican seat in Bush country . He thinks as I do the the Earle iondictments are crap and that once it blows over he still has a political future. But if he were to lose the seat his stock falls dramatically .So he will sit this one out and hope the other scandals are contained with the fall of some of his former close aids .

I do not lament his leaving . I think his tenure as majority leader has been over-rated . I thought that Dick Armey was much more effective in delivering a conservative agenda. He lost me completely when he claimed there was no fat in the budget .He talked a good game about trimming ineffective Federal programs but while he was the man ,the Fedearl Government expanded both in entitlement programs as well as non-sense pork programs .

excon rated this answer Excellent or Above Average Answer
Erewhon rated this answer Excellent or Above Average Answer

Question/Answer
excon asked on 04/04/06 - So, when do we win????


Hello wrongwingers:

I'm curious. If we've been winning in Iraq, like you say, when does winning turn in to won?

excon

tomder55 answered on 04/04/06:

are you looking for a time line ? no can do


but let's see ;just for comparison ; we defeated Saddam in 1990 in 100 hours and then engaged his military for over a decade afterward .

All I can point out is specific indicators that lead me to conclude we are winning .

The Syrian border is essentially closed to new insurgents

Al Qaeda in Iraq has had its own internal fight and as an operational force has been reduced to gang status plundering for their very existance .There is no more safe haven for them in the Sunni triangle . Instead of waging war against "the invaders "they are doing desperate things like prison breaks and detonating car bombs against innocent civilians .The only significant infiltration now comes in from Iran.

Attacks overall are down. U.S. fatalities are down to "accident" levels. And we know that most of the attacks that are going on are more sectarian in nature.The overall violence level is down to below what you normally see daily in Rio and Brazil isn't even at war.


Now that does not mean that the war will end any time soon . Robert E. Lee once boasted that if his army were to dispurse they could continue the civil war for 20 years .But ultimately the political process will bring in the last of the Sunnis.

An open sore that needs to be settled is Mookie al-Sadr and his militia (backed by Iran) and his attempt to be a power broker and shape the new gvt. into an Iran stlye theocracy . That would be un-acceptable . The people of Iraq overwhelming voted for a parlimentary representative system and even the most influential cleric Sistani opposes a theocracy . I can only speculate that it will not be long before al Sadr tries an assasination campaign against Sistani. I am also almost certain that he will try another "uprising " which I think would be very intersting at this point .The ISF is now capable of dealing with his rag-tag militia and I do not think they will be as generous about letting him out of the noose.

In sum ;the Sunni insurgency has been defeated and the next stage of the war will be a political/military struggle against Iranian-backed Shias over the formation and policies of the new government. As I said in another post .I do not think that there will be a final victory until the issue of Iran is dealt with .



triumvirate rated this answer Excellent or Above Average Answer
excon rated this answer Excellent or Above Average Answer

Question/Answer
BeelzeBUSH asked on 04/04/06 - Runner-up Picture Of The Year


http://www.poyi.org/63/11/02.php


First place picture was the Marines loading the casket carrying the body of 2nd Lt. James Cathey's into a cargo plane. The runner-up picture shows the wife with the body all night before the burial the next day. Of course these pictures are sad. Third place was just John Kerry with an expression of exasperation waiting for GW's inaugural to start.

My heart goes out the families that lost loved ones. I know the hurt they suffer is beyond words. My hope for all our soldiers is G-dspeed for a safe trip back home soon.


George


tomder55 answered on 04/04/06:


How about this from the same web site ????

this one depicts :"A young boy prays over one of the nearly 600 coffins containing the remains of Bosniak men and boys murdered during the Srebrenica massacre ."

The Srebrenica massacre was the 1995 killing of an estimated 8,100 Bosniak males by a Serb Army.This occured even though the UN had declared in 1993 that Srebrenica was a "safe area " .Belatedly ,after Clinton waffled for 4 years ,the US decided to intervene with NATO and do the right thing...wage war against Serbia.Even though they could not get UN support of the action.

This was Clinton's most important positive contribution in the infant war against terrorism ; an action opposed by most of his political advisers. It was a classic commander-in-chief decision, made alone, without congressional support and with only reluctant backing from the Pentagon. But it worked.

Without those American troops, Bosnia would not have survived, there would be 2 million refugees wandering Europe, a criminal state would be in power in Bosnia , and we would probably have had to pursue a larger 'Operation Bosnia Freedom' in the ravines and hills of central Bosnia, where al Qaeda was putting down roots that were removed by NATO after the Dayton Accords .

American troops are still there . Was it worth it ? Hell yes !


BeelzeBUSH rated this answer Excellent or Above Average Answer
ETWolverine rated this answer Excellent or Above Average Answer

Question/Answer
BeelzeBUSH asked on 04/04/06 - Iran next?






tomder55 answered on 04/04/06:

one can only hope . I do not think there can be an end-game in Iraq until Iraninan interference is addressed . I think a nuclear Persian hegemon;or in the mind of Ahmadinejad , empire ;stretching from the Nile to the Indus is a cause for concern that like Nazi Germany is best to deal with sooner rather than later.

BeelzeBUSH rated this answer Excellent or Above Average Answer

Question/Answer
Itsdb asked on 04/03/06 - Help

Can someone please furnish the quote or quotes where Bush tied Saddam to 9/11? I'd really like to know.

Steve

tomder55 answered on 04/04/06:

I can make the link but Bush never did directly. The adm. claim was that Saddam was a sponsor of terrorism with his hand on WMD and a track record of using them. They said that the next time a terrorist attacks it could be with WMD transfered from Saddam to the terrorists . They said in a post 9-11 world we could no longer wait for that possibility .

This is not news. This morphed into the claim by the opponents of the policy that Bush linked Saddam to 9-11 .A steady drum beat of this lie by the opponents ,like Al Gore's screeds [he betrayed this country ...he played on our fears !!], has added this meme to the vernacular .

The critics point to the following quotes to make their case :

"The use of armed forces against Iraq is consistent with the United States and other countries continuing to take the necessary actions against international terrorists and terrorist organizations, including those nations, organizations or person who planned, authorized, committed, or aided the terrorist attacks that occurred on September 11, 2001." [President George W. Bush, Letter to Congress, 3/21/03]

"The liberation of Iraq is a crucial advance in the campaign against terror.We have not forgotten the victims of September the 11th -- the last phone calls, the cold murder of children, the searches in the rubble. With those attacks, the terrorists and their supporters declared war on the United States. And war is what they got." [Remarks by the President, 5/1/03]

"If we're successful in Iraq.we will have struck a major blow right at the heart of the base, if you will, the geographic base of the terrorists who have had us under assault now for many years, but most especially on 9/11." [Dick Cheney NBC's Meet the Press, 9/14/03]

There are a few others by Rumsfeld and Rice but you get the idea .Perhaps they could accuse the administration of making an emotional connection but no one ever claimed a direct link ......except me :

From the MSM :

The Herald (Glasgow, Scotland), December 28, 1999.


Iraq tempts bin Laden to attack West
Exclusive. By: Ian Bruce, Geopolitics Editor.

THE world's most wanted man, Osama bin Laden, has been offered sanctuary in Iraq if his worldwide terrorist network succeeds in carrying out a campaign of high-profile attacks on the West ...


Now we are also facing the prospect of an unholy alliance between bin Laden and Saddam. The implications are terrifying.


"We might be looking at the most wanted man on the FBI's target list gaining access to chemical, biological or even nuclear weapons courtesy of Iraq's clandestine research programmes."


The U.S. intelligence community has been squeezing bin Laden's finances steadily for several years. His personal fortune of anything up to 500m has been whittled down to single figures ...


- - - - -


U.S. Newswire, December 23, 1999.


Terrorism Expert Reveals Why Osama bin Laden has Declared War On America; Available for Comment in Light of Predicted Attacks.


... (author Yossef) Bodansky also reveals the relationship between bin Laden and Saddam Hussein and how the U.S. bombing of Iraq is "strengthening the hands of militant Islamists eager to translate their rage into violence and terrorism." ....


- - - - -


The Observer. December 19, 1999.


Sanctions reviewed in West as Saddam wields sword of Islam


The Iraqi dictator has rejected a UN deal to lift sanctions. The Western blockade, far from toppling the regime, has bolstered it. He's ditched the sunglasses and taken up the Koran to harness the fervour of fundamentalists.


By: Jason Burke, in Baghdad


... This time last year the U.S. claimed that another delegation had met Osama bin Laden, the alleged terrorist mastermind and tried to woo him to Iraq.


Senior officials claim that the Islamisation programme is an attempt to defuse the threat of Islamic militancy rather than encourage it ...


- - - - -


United Press International. November 3, 1999, Wednesday, BC cycle.


WASHINGTON -- The U.S. government has tried to prevent accused terror suspect Osama bin Laden from fleeing Afghanistan to either Iraq or Chechnya, Michael Sheehan, head of counter-terrorism at the State Department, told a Senate Foreign Relations subcommittee ...


- - - - -


Akron Beacon Journal (Ohio). October 31, 1999. Sunday 1 STAR EDITION.


BIN LADEN SPOTTED AFTER OFFER TO LEAVE
By: From Beacon Journal wire services


DATELINE: JALALABAD, AFGHANISTAN:


... The Taliban has since made it known through official channels that the likely destination is Iraq.


A Clinton administration official said bin Laden's request "falls far short" of the UN resolution that the Taliban deliver him for trial....


- - - - -


The Kansas City Star. March 2, 1999, Tuesday.


International terrorism, a conflict without boundaries


By Rich Hood


... He (bin Laden)has a private fortune ranging from $250 million to $500 million and is said to be cultivating a new alliance with Iraq's Saddam Hussein, who has biological and chemical weapons bin Laden would not hesitate to use. An alliance between bin Laden and Saddam Hussein could be deadly. Both men are united in their hatred for the United States and any country friendly to the United States....


- - - - -


Los Angeles Times. February 23, 1999, Tuesday, Home Edition.
SECTION: Metro; Part B; Page 6; Letters Desk.
HEADLINE: OSAMA BIN LADEN


Where is Osama bin Laden (Feb. 14)? That should be the U.S.'s main priority. If as rumored he and Saddam Hussein are joining forces, it could pose a threat making Hitler and Mussolini seem like a sideshow....


- - - - -


National Public Radio (NPR)
MORNING EDITION (10:00 AM on ET)
February 18, 1999.


THOUGH AFGHANISTAN HAS PROVIDED OSAMA BIN LADEN WITH SANCTUARY, IT IS UNCLEAR WHERE HE IS NOW.
ANCHORS: BOB EDWARDS
REPORTERS: MIKE SHUSTER


... There have also been reports in recent months that bin Laden might have been considering moving his operations to Iraq. Intelligence agencies in several nations are looking into that. According to Vincent Cannistraro, a former chief of CIA counterterrorism operations, a senior Iraqi intelligence official, Farouk Hijazi(ph), sought out bin Laden in December and invited him to come to Iraq.


Mr. VINCENT CANNISTRARO (Former Chief of CIA Counterterrorism Operations): Farouk Hijazi, who was the Iraqi ambassador in Turkey ... known through sources in Afghanistan, members of Osama's entourage let it be known that the meeting had taken place.


SHUSTER: Iraq's contacts with bin Laden go back some years, to at least 1994, when, according to one U.S. government source, Hijazi met him when bin Laden lived in Sudan. According to Cannistraro, Iraq invited bin Laden to live in Baghdad to be nearer to potential targets of terrorist attack in Saudi Arabia and Kuwait. There is a wide gap between bin Laden's fundamentalism and Saddam Hussein's secular dictatorship. But some experts believe bin Laden might be tempted to live in Iraq because of his reported desire to obtain chemical or biological weapons. CIA director George Tenet referred to that in recent testimony....


- - - - -


Agence France Presse. February 17, 1999.


Saddam plans to use bin Laden against Kuwait, Saudi: opposition


Iraq's President Saddam Hussein plans to use alleged terrorist Osama bin Laden's network to carry out his threats against Kuwait and Saudi Arabia, an Iraqi opposition figure charged on Wednesday.
"If the ... Jaber, a member of the Supreme Council for Islamic Revolution in Iraq (SCIRI), said Iraq had "offered to shelter bin Laden under the precondition that he carry out strikes on targets in neighbouring countries."


... Islamic fundamentalist bin Laden, who has gone missing from his base in Afghanistan, would never seek refuge in secular Iraq on ideological grounds. "I think bin Laden would keep quiet or fight to the death rather than seek asylum in Iraq," the London-based dissident, who asked not to be named, told AFP last week.....


- - - - -


Deutsche Presse-Agentur. February 17, 1999, Wednesday, BC Cycle


Opposition group says bin Laden in Iraq


DATELINE: Kuwait City


An Iraqi opposition group claimed in a published report Wednesday that Islamic militant Osama bin Laden is in Iraq from where he plans to launch a campaign of terrorism against Baghdad's Gulf neighbours.


The claim was made by Bayan Jabor, spokesman for the Teheran-based Supreme Council for the Islamic Revolution in Iraq (SCIRI).


Bin Laden "recently settled in Iraq at the invitation of Saddam Hussein in exchange for directing strikes against targets in neighbouring countries," Jabor told the Kuwaiti newspaper al-Rai al- Aam ... Taleban leaders in Afghanistan, where he had been living, said they lost track of him. Media reports have speculated he sought refuge in Chechnya, Somalia, Iraq, or with a non-Taliban group in Afghanistan.


Jabor, who was interviewed in Damascus, Syria, said Iraq began extending invitations to bin Laden six months ago, shortly after the United States bombed his suspected terrorist training camps in Afghanistan after linking him with the August 7 bombings of U.S. embassies in Nairobi, Kenya and in Dar-es-Salam, Tanzania.


The United States indicted Bin Laden for the embassy bombings and has offered a five million dollar reward for information leading to his capture. Bin Laden's disappearance has coincided with stepped up threats by Iraq against neighbours Saudi Arabia, Kuwait and Turkey for allowing the United States and Britain to use their air bases to carry out air patrols over two "no-fly" zones over northern and southern Iraq ....


- - - - -


Associated Press Worldstream. February 14, 1999.


Taliban leader says whereabouts of bin Laden unknown


... Analysts say bin Laden's options for asylum are limited.


Iraq was considered a possible destination because bin Laden had received an invitation from Iraqi President Saddam Hussein last month. And Somalia was a third possible destination because of its anarchy and violent anti-U.S. history....


- - - - -


San Jose Mercury News (California). February 14, 1999 Sunday MORNING FINAL EDITION


U.S. WORRIED ABOUT IRAQI, BIN LADEN TIES TERRORIST COULD GAIN EVEN
DEADLIER WEAPONS


U.S. intelligence officials are worried that a burgeoning alliance between terrorist leader Osama bin Laden and Iraqi President Saddam Hussein could make the fugitive Saudi's loose-knit organization much more dangerous ...


In addition, the officials said, Palestinian terrorist Abu Nidal is now in Iraq, as is arenowned Palestinian bomb designer, and both could make their expertise available to bin Laden.


"It's clear the Iraqis would like to have bin Laden in Iraq," said Vincent Cannistraro, a former head of counterterrorism operations at the Central Intelligence Agency ...


Saddam has even offered asylum to bin Laden, who has expressed support for Iraq.


... (in) late December, when bin Laden met a senior Iraqi intelligence official near Qandahar, Afghanistan, there has been increasing evidence that bin Laden and Iraq may have begun cooperating in planning attacks against American and British targets around the world.


Bin Laden, who strikes in the name of Islam, and Saddam, one of the most secular rulers in the Arab world, have little in common except their hatred of the United States ...


More worrisome, the American officials said, are indications that there may be contacts between bin Laden's organization and Iraq's Special Security Organization (SSO), run by Saddam's son Qusay. Both the SSO and the Mukhabarat were involved in a failed 1993 plot to assassinate former President George Bush ...


"The idea that the same people who are hiding Saddam's biological weapons may be meeting with Osama bin Laden is not a happy one," said one American official....


- - - - -


Associated Press Worldstream. February 13, 1999; Saturday 14:32 Eastern Time


Bin Laden said to have left Afghanistan, whereabouts unknown


... It is very unlikely bin Laden could remain in Afghanistan without Taliban officials knowing his whereabouts.


Iraqi President Saddam Hussein has offered asylum to bin Laden, who has expressed support for Iraq.


U.S. officials believe bin Laden masterminded the Aug. 7 bombings of its embassies in Kenya and Tanzania ...


Bin Laden urged devout Muslims to attack U.S. and British interests in retaliation for their joint assault on Iraq.


U.S. officials demanded that the Taliban hand over bin Laden, who has been indicted in a U.S. court on murder charges in connection with the bombings. But the Taliban had refused.


- - - - -


The Bulletin's Frontrunner. January 4, 1999, Monday.


Defiant Saddam Looks To Provoke U.S.


... Time also reported, "For now, the White House will respond to each provocation by counterattacking the offending battery."


Saddam Reaching Out To bin Laden.


Newsweek (1/11, Contreras) reported, "U.S. sources say (Saddam) is reaching out to Islamic terrorists, including some who may be linked to Osama bin Laden." ...


(Osama bin Laden was) calling for all-out war on Americans, using as his main pretext Washington's role in bombing and boycotting Iraq." In a Newsweek interview, bin Laden said that "'any American who pays taxes to his government," is a legitimate target." Newsweek reported, "The idea of an alliance between Iraq and bin Laden is alarming to the West," although "Saddam may think he's too good for such an association." However, "Now that the United States has made his removal from office a national objective....


- - - - -


The White House Bulletin. Copyright 1999. Bulletin Broadfaxing Network, Inc.


In a Newsweek interview, bin Laden said that "'any American who pays taxes to his government," is a legitimate target." Newsweek reported, "The idea of an alliance between Iraq and bin Laden is alarming to the West," although "Saddam may think he's too good for such an association." However, "Now that the United States has made his removal from office a national objective, he....


- - - - -


United Press International. January 3, 1999, Sunday, BC cycle.


UPI Focus: Bin Laden 'instigated' embassy bombings


... (The Taliban) government in Afghanistan says the Saudi does not have the money to finance projects in the country. Newsweek also reported that Iraqi leader Saddam Hussein has been making new overtures to bin Laden in an attempt to rebuild his intelligence network and to create his own terror network....




Steve Hayes has been all over the connection since 9-11 . He documented in his book 'The Connection' the direct links .

also Richard Mintner and Laurie Mylroie have been on the case in Mylroie's case ;she has made a compelling case that Saddam was also directly involved in the 2003 WTC bombing . Mylroie was adviser on Iraq to the 1992 campaign of Bill Clinton .



more links :

http://www.guardian.co.uk/alqaida/story/0,12469,798270,00.html

http://www.washtimes.com/world/20030912-012437-3992r.htm

http://www.telegraph.co.uk/news/main.jhtml?xml=%2Fnews%2F2003%2F04%2F27%2Fwalq27.xml

http://www.weeklystandard.com/Content/Public/Articles/000/000/003/033jgqyi.asp

http://www.weeklystandard.com/Content/Public/Articles/000/000/003/296fmttq.asp

http://www.forward.com/issues/2003/03.06.20/news2.html

http://www.newyorker.com/fact/content/?030210fa_fact






Itsdb rated this answer Excellent or Above Average Answer

Question/Answer
Mathatmacoat asked on 04/01/06 - The origin of April Fool's Day?

The Origin of April Fool's Day

Associated Press.

The mystery of the origin of April Fool's Day has finally been solved.

Joseph Boskin, a History professor at Boston University, has discovered that the celebration had begun during the Roman empire when a court jester had boasted to Emperor Constantine that the fools and jesters of the court could rule the kingdom better than the Emperor could.

In response, Constantine had decreed that the court fools would be given a chance to prove this boast, and he set aside one day of the year upon which a fool would rule the kingdom. The first year Constantine appointed a jester named Kugel as ruler, and Kugel immediately decreed that only the absurd would be allowed in the kingdom on that day. Therefore the tradition of April Fools was born.

So what do you know? In modern day america it's April Fool's Day every day.

tomder55 answered on 04/03/06:

yup a posting like this proves it can be April fools every day especially for those who think this little bit of history is anywhere's credible. Suggest logging onto Snopes for some clarity .

Mathatmacoat rated this answer Excellent or Above Average Answer

Question/Answer
HerrAirhorn asked on 04/02/06 - Wm F. Buckley on Bush

From Bloomberg dot com.
March 31 (Bloomberg)

"William F. Buckley Jr., the longtime conservative writer and leader, said George W. Bush's presidency will be judged entirely by the outcome of a war in Iraq that is now a *failure*.

``Mr. Bush is in the hands of a fortune that will be unremitting on the point of Iraq,'' Buckley said in an interview that will air on Bloomberg Television this weekend. ``If he'd invented the Bill of Rights it wouldn't get him out of his jam.''

Buckley said he doesn't have a formula for getting out of Iraq, though he said ``it's important that we acknowledge in the inner councils of state that it (the war) has failed, so that **we should look for opportunities to cope with that failure.**


Time to get real, isn't it?

tomder55 answered on 04/02/06:

there was a time when no true progressive would ever cite Buckley as a credible source. my how times have changed .

Buckley was never really on board with the Iraq war so this is not really a change of position for him.I read Buckley's article entitled It Didnt Work . To me it was one of the weaker papers he has ever written.He flicks from the NY Slimes a quote from an un-named Iraqi buisnessman and an un-named US soldier and an inflamatory anti-Israli quote from Mahmoud Ahmadinejad to fortify his position.

Buckley described how the businessman blames Iraqs problems on America. Since the man is described as being a member of a Sunni stronghold, it is not difficult to surmise where his loyalties originate.

The anonymous soldier is now aware of why Saddam Hussein was needed to keep the Sunnis and the Shiites from each others throats. See Saddam was a peacemaker !! Perhaps if the soldier was a little more learned he would realize that Saddam leading the Sunnis against the Shiite in a one-way contest of torture and suppression. It would be so much better for the Sunnis and Shiites and Kurds if Saddam was there to put soccer players through his plastic shredders when they lost a game and if Uday and Qusay still had their rape rooms in operation.


HerrAirhorn rated this answer Average Answer
triumvirate rated this answer Excellent or Above Average Answer

Question/Answer
Mathatmacoat asked on 04/01/06 - Will this mean war?

Last week Indonesia insulted Australia by publishing an offensice cartoon featuring Australian Prime Minister Howard. Not to be out done the Australian Murdoch newspapers have entered the fray with their own reply. Will this mean war?

Indonesian dingo cartoon 'offensive'

There is anger from both sides of federal politics over a cartoon in an Indonesian newspaper portraying Prime Minister as a dingo mounting a second dingo with the face of the Foreign Affairs Minister.

The cartoon was published in response to Australia's decision to give temporary protection visas to 42 West Papuans.

Indonesia has been highly critical of the decision and temporarily recalled its Ambassador from Australia last week.

Alexander Downer says the cartoon is tasteless.

Mr Downer says people can choose to publish tasteless and grotesque cartoons in a free society.

"I would have thought those countries in our society fell way below standards of public taste," he said.

"I think a lot of Australians would regard those kinds of publications as very offensive, but they're free to be offensive in a magazine in Indonesia if they wish to be."

Speaking on Southern Cross radio, the Federal Treasurer Peter Costello also criticised the cartoon.

"I think it's absolutely offensive," he said.

"I'm not sure which newspaper published this in Indonesia, but it doesn't do them any credit."

Labor's Kevin Rudd joined in the condemnation.

"I've seen that report this morning," he told ABC Radio.

"If the report is true, then that depiction I think is disgusting and disgraceful."

Opposition Leader Kim Beazley says the cartoon was published because of the way the Prime Minister has dealt with Indonesia.

"Because of the way the Prime Minister has handled these issues in the past, because of the way he's presented to Indonesians, which is somewhat different from what is really being done, we've got ourselves into this bind," he said.

Meanwhile, the Foreign Affairs Department has warned Australians that there have been protests outside the Australian embassy in Jakarta because of the decision to grant the temporary visas.

It says further demonstrations against the embassy or other Australian interests in Indonesia are possible and Australians should avoid them.

The Aussie relpy

http://network.news.com.au/image/0,10114,5132449,00.jpg

For those not attuned to the politics it depicts the result of indo-papuan relations with the Indonesian President perpetrating the act

tomder55 answered on 04/02/06:

and here I thought the jihadists were over reacting during the cartoon intifada .

Mathatmacoat rated this answer Excellent or Above Average Answer

Question/Answer
Erewhon asked on 04/01/06 - The fundamental divide between British Democracy and Unacceptable American Democracy vocalised by Lo



Perhaps the sharpest comment Rice heard came from former Foreign Secretary Lord Douglas Hurd CBE of Westwell., a Conservative Party stalwart (a Tory grandee!) who served under Prime Minister Margaret Thatcher and was among a panel of experts who heard Rice's speech on the need to encourage democracy around the world.

"It is quite possible to believe" that democracy is essential, Hurd said to the crowd after she spoke, but also to "believe that essentially the path must grow from the roots of its own society and that the killing of thousands of people, many of them innocent, is unacceptable whether committed by a domestic tyrant or for a good cause upon being invaded."





tomder55 answered on 04/02/06:

lets ponder that . Was the Japanese democracy grown from the roots of its own society ? A big No on that one .How about the South Korean democracy ? again ...no. What was the India experience in democracy before England instilled those principles into the sub continent ? Nada . So three of the biggest ,most vibrant democracies today did not have their democracies born from the roots of their own society . I think Hurd's hypothesis is a little flawed .

Erewhon rated this answer Excellent or Above Average Answer

Question/Answer
Erewhon asked on 04/01/06 - Bush worse than Nixon?

Bush spying called worse than Nixon's

Laurie Kellman
Associated Press
Apr. 1, 2006 12:00 AM

WASHINGTON - John Dean, Richard Nixon's White House lawyer, told senators Friday that President Bush's domestic spying exceeds the wrongdoing that toppled Dean's former boss.


Bush, Dean told the Senate Judiciary Committee, should be censured and possibly impeached.

"Had the Senate or House, or both, censured or somehow warned Richard Nixon, the tragedy of Watergate might have been prevented," Dean told the Senate Judiciary Committee. "Hopefully the Senate will not sit by while even more serious abuses unfold."

Republicans and their witnesses rejected the comparison between Watergate and Bush's wiretapping program, and attributed Sen. Russell Feingold's censure resolution to posturing in a year of midterm elections.

Sen. Lindsey Graham, R-S.C., said the comparison to Watergate is "apples and oranges" because Nixon's actions were more about saving himself and his presidency than national security.

"Quit trying to score political points," Sen. Orrin Hatch, R-Utah, shot across the aisle at committee Democrats.

Only two Democrats have co-sponsored Feingold's resolution: Sens. Tom Harkin of Iowa and Barbara Boxer of California. The rest have distanced themselves from the proposal, with many saying the resolution is premature because a Senate Intelligence Committee investigation has not concluded.

At issue is whether Bush's secretive domestic spying program violates the 1978 Foreign Intelligence Surveillance Act.

Bush has said the National Security Agency's wiretapping program is aimed at finding terrorists before they strike on American soil by tapping overseas calls. He has launched a criminal investigation to find out who leaked the program's existence to the New York Times, saying the report in December tipped off anyone who might be planning attacks.

Critics say Bush already has the ability to conduct wiretaps under the FISA law and any information gathered without a court order may be inadmissible at a trial.

Feingold's measure would condemn Bush's "unlawful authorization of wiretaps of Americans within the United States without obtaining the court orders required" by the FISA act.

"If we in the Congress don't stand up for ourselves and the American people, we become complicit in the lawbreaking," Feingold, D-Wis., told the panel. "The resolution of censure is the appropriate response."

Feingold summoned Dean to the hearing in part because the former White House counsel made his suspicions about the Bush administration clear long before the wiretapping program became public.

In his 2004 book, Worse Than Watergate: The Secret Presidency of George W. Bush, Dean wrote that the former Texas governor began to evoke Nixonian memories with his strategies against Republican John McCain's primary challenge in South Carolina in 2000.

After the New York Times revealed the NSA program in December, Dean wrote that "Bush may have outdone Nixon" and may be worthy of impeachment.

"Nixon's illegal surveillance was limited; Bush's, it is developing, may be extraordinarily broad in scope," Dean wrote in a column for FindLaw.com in December.

Nixon resigned Aug. 9, 1974.

tomder55 answered on 04/02/06:

lol ; The worse part of Watergate was the cover-up .....and who orchestrated it ?????

yup ....John Dean ..

I saw parts of the hearing .One thing I noticed was Feingold saying that there were a couple of ex-Justice dept. workers who believed that Bush's wiretapping program was illegal, and that it was a shame they couldn't be at the hearing. Didn't the Dems. call John Dean to testify, and if so, why did they decide to go with a known felon instead of individuals Feingold called credible witnesses?

Does Dean forget that activities like the break-ins at the Watergate and Ellsburg's psychiatrist were done to obtain dirt on domestic political enemies whereas the NSA authorizations are done to obtain intel. on a FOREIGN ENEMY who attacked us on mainland USA ?

Hillary keeping 900 FBI files on her political opponents in her White House office has more in common with Watergate than this .

A panel of former FISA Court judges told members of the Committee that Bush did not act illegally when he created the survaillance program .



I just hope that the Democrats continue to make this a big deal . They just do not understand how foolish and weak they look. I think tomorrow the Republicans should call in the full Senate for a vote on this resolution . There would be as many yes votes as there was for the Murtha pull out vote .But the hearings do give the Feingold campaign a needed boost among the moonbat wing of the Democrat Party .......hmmm.... could these hearings have more to do with a Feingold primary run than national security ???? HMMMM!!!


Erewhon rated this answer Excellent or Above Average Answer

Question/Answer
Erewhon asked on 03/31/06 - Bush Administration 's Mistakes ...

Condoleeza Rice has said,

"I am certain that there will be many dissertations written on the mistakes made by the Bush administration."

[Dr Condoleeza Rice in response to a question at Chatham House, Blackburn, Lancashire, England, 31 March 2006, carried on C-Span]

===

Which three mistakes of the Bush Administration do you think merit the most dissertative attention?



tomder55 answered on 04/01/06:

see ,now there is room for debate . point one was clearly not a lie . The course towards war with WMD as a causus- beli was charted by the previous administration and there was every expectation that they would be found. I know that the accepted meme now is that there were no WMD ,but that is the lie. in fact new revelations in Iraqi documents and audio tapes from Saddam himself confirm the existance of an existing WMD program and a deception campaign against the UN with the asistance of the Russian Spetsnaz .

point 2 ; yes there was no acquisition of uranium from Niger but all the President said was that Saddam attempted to obtain it .That has also been confirmed in spite of the claim by Joe Wilson .

point 3 is a matter of perception . The Administration contended thatthere was a dangerous nexus of international terrorism and rogue States that support these terrorist entities . This in a large sense dispelled the false and naive understanding of terrorism that states had become irrelevant and that even very major attacks were now the work of groups and "networks," unaided by states. I leave you with the translation of Iraqi document #ISGZ-2004-009247
recently released ,and you tell me that there was no operational relationship between Iraq and al-Qaeda .

Erewhon rated this answer Excellent or Above Average Answer

Question/Answer
Erewhon asked on 03/31/06 - Ken Rudin says, "Andy Card Leaves a Stacked Deck!" Then he explains ...



It made sense. President Bush's numbers are at their nadir. The war is, by most accounts, not going especially well. The deficit is out of control. The issue of homeland security blew up in the White House's face over the Dubai ports fiasco. And the Republican Party can't seem to come to terms about what to do about immigration.

Solution? Chief of Staff Andy Card had to go.

Actually, that's the way it works in Washington. The president has no policy disagreements with his secretaries of defense or state, and was not about to unload adviser Karl Rove, especially if no Plame-related indictment was coming. But with a growing chorus in the Republican Party demanding an Oval Office shakeup, somebody had to be thrown overboard, the somebody being Andy Card. Remember, it wasn't Yankees manager Yogi Berra's fault that Whitey Ford's arm was dead or that Roger Maris hit .200 in the 1964 World Series. But when you're supposed to beat the Cardinals and you don't, you can't dump your stars, so you dump the manager. (By the way, the official word from the White House is that Card was not fired, he resigned. And that may be what actually happened.)

Still, in fairness, the likeable and approachable Card was not entirely blameless in Bush's free fall. There was a clear political tone-deafness that came out of the White House, not only over the Dubai ports, but also of the disastrous Harriet Miers Supreme Court nomination -- two events that demoralized and angered conservatives. Not to mention the reaction to the government's response to Hurricane Katrina. If Andy Card was not responsible for the decisions, he should have anticipated the political fallout. He didn't, and now he's gone.

Bottom line: Will Bush's numbers now shoot back up? Will the war become more palatable? Will conservatives feel mollified?

The answer to all three: of course not.

Replacing Andy Card with White House Budget Director Josh Bolten is not going to make much of a difference. So while it does provide for a nice distraction, I'm not exactly sure what changes.

And as for Card, there are rumors that he will jump into the race for governor in Massachusetts, where incumbent Republican Mitt Romney is leaving after one term, and where Romney's lieutenant governor, Kerry Healey, is not doing so well as the presumptive GOP nominee to succeed him. My guess is that it doesn't happen; perhaps one reason the rumor has surfaced is that Card has previously talked of the governorship, and in fact ran once before, in 1982, finishing third in the GOP primary. (There's another scenario that has Card heading back to the Bay State to help with Romney's anticipated presidential candidacy.)

MEMORY LANE: The history of White House chiefs of staff is filled with successful and failed candidacies for office, both before and after their tenure in the administration. Here's the tally of COS candidates in the past half-century:

Sherman Adams (1953-58, under President Dwight Eisenhower)

BEFORE: Member of New Hampshire state legislature, 1941-44. Member of Congress, 1945-46. Lost 1946 Republican gubernatorial nomination. Governor, 1949-52.

Alexander Haig (1973-74, Richard Nixon)

AFTER: Sought Republican presidential nomination, 1988.

Donald Rumsfeld (1974-75, Gerald Ford)

BEFORE: Member of Congress from Illinois, 1963-69. Left Congress to become President Nixon's director of the Office of Economic Opportunity (1969-70); was U.S. ambassador to NATO (1973-74).

SINCE: Never ran for office again, but served as secretary of defense under President Ford (1975-77); returned to the cabinet as secretary of defense under the current President Bush (since 2001).

Dick Cheney (1975-77, Gerald Ford)

SINCE: Member of Congress from Wyoming, 1979-89. Left Congress to become President George H.W. Bush's secretary of defense (1989-93). Mentioned as a potential Republican presidential hopeful, 1996. Elected vice president in 2000, served in that capacity since 2001.

Hamilton Jordan (1979-80, Jimmy Carter)

SINCE: Lost 1986 Democratic Senate primary in Georgia to Wyche Fowler.

Jack Watson (1980-81, Jimmy Carter)

SINCE: Finished fourth in the 1982 Democratic gubernatorial primary in Georgia.

James Baker (1981-85, Ronald Reagan; 1992-93, George H.W. Bush)

BEFORE: Lost 1978 election for Texas state attorney general.

Howard Baker (1987-88, Ronald Reagan)

BEFORE: Lost 1964 Senate race in Tennessee to Ross Bass. Senator from Tennessee, 1967-84. Sought Republican presidential nomination, 1980.

AFTER: Never ran for office again, but served as the current President Bush's ambassador to Japan (2001-05).

John Sununu (1989-91, George H.W. Bush)

BEFORE: Lost 1980 Republican Senate primary in New Hampshire to Warren Rudman. Governor of New Hampshire, 1983-88.

Leon Panetta (1994-97, Bill Clinton)

BEFORE: Member of Congress from California, 1977-93. Left Congress to become President Clinton's budget director.

Erskine Bowles (1997-98, Bill Clinton)

AFTER: Lost 2002 Senate race in North Carolina to Elizabeth Dole. Lost 2004 Senate race to Richard Burr.

Andrew Card (2001-06, George W. Bush)

BEFORE: Massachusetts state representative, 1975-82. Lost 1982 GOP gubernatorial primary. Served the first President Bush as secretary of transportation (1992-93).

On to the questions

Q: Isn't new House majority leader John Boehner (R-OH) one of Newt Gingrich's guys? How did his election become good news for the Republicans? -- Rosemarie Bonacci, La Porte, Texas (just outside of Houston)

A: Simple. He was running against one of Tom DeLay's guys -- Roy Blunt, the Missouri Republican and Majority Whip who was the acting majority leader in DeLay's absence following the Texan's indictment. Whatever bitter residue there may be among Republicans regarding Gingrich's reign (House minority whip 1989-94, speaker 1994-98), and Boehner's role in it, it pales with how many feel about DeLay. While DeLay was most effective in lining up votes and raising money (and redrawing congressional lines in Texas), once the Democrats began their crusade over the GOP's "culture of corruption," Republican lawmakers couldn't run fast enough away from DeLay -- and that meant voting against his ally, Blunt.

Q: You were right, in your Feb. 16 column, about 1864 being the year when a winning presidential ticket (Abraham Lincoln and Andrew Johnson) was comprised of candidates from two different parties. I tend to think that 1840 was also such a year. It seems pretty clear that John Tyler was really a Democrat, not a Whig, when he was picked for vice president. -- Richard Winger, San Francisco, Calif.

A: Tyler was certainly a former Democrat when he was tapped in 1840 to be the running mate on the Whig ticket led by war hero William Henry Harrison. But he was no longer in the party, and in fact spent much of the year working on behalf of Harrison's rival for the Whig nomination, Sen. Henry Clay.

President Harrison died in office, a month into his term. From the get go, Tyler had serious difficulties with the Whig-led Congress. Clay, who was preparing another presidential bid, did everything he could to make life miserable for Tyler, his erstwhile ally. By the time of the 1844 Whig convention, delegates had long abandoned Tyler in favor of Clay. Then Tyler's allies tried to get him the Democratic nomination, but that party didn't want any part of him, either. Ultimately, Tyler gave up the idea of hanging on to the presidency and wound up backing the Democratic nominee, James K. Polk, who went on to defeat Clay.

And then there's this:

Q: I love reading your column; as a political junkie myself, I find it to be informative and fun. But I did find a mistake in your Feb. 16 discussion of the 1864 National Unity (Republican) ticket. Lincoln didn't actually choose Johnson. He was informed by telegram as to who his vice president would be. Doris Kearns Goodwin writes in her superb new book, Team of Rivals, that Lincoln, who faced some challengers for the nomination, was worried when the telegram arrived. (It said simply "Johnson nominated," or something of the sort.) He then realized that it was referring to the nomination of the VP. -- Andrew Dobbs, Austin, Texas

Q: I really enjoy your on-air and online reports. How about a podcast, too? -- Jennifer Potash, Trenton, N.J.

A: Actually, that is in the works as well. Ron Elving, NPR's senior Washington editor, and I have already done two pilot political podcasts. The first one ran 11 minutes, the second considerably longer. Two other NPR staffers, editor Beth Donovan and producer Muthoni Muturi, are working hard on making this happen. If you want my opinion, the ones we've done so far are brilliantly illuminating and hysterically funny. If you ask Ron, he will probably deny any knowledge of or involvement in this. To be continued.

THE 'LIBERAL' MEDIA:

An aside from me in the Feb. 16 column discussing Dick Cheney got me in some trouble and deserves an explanation. I wrote, "The issue, in the eyes of Democrats and many in the media (I know, they are one and the same), is Dick Cheney's modus operandi of not feeling any need to explain his actions to or make himself accountable to the American people." That brought in a slew of incredulous e-mails, some in language not suitable to print here. The Democrats and the media, one and the same? Hello??

What happened is I was trying to be cutesy and it backfired. Often, especially during the Clinton administration, whenever I would write the words "the Democrats and the media," I would invariably receive responses saying in effect, "Oh, there's a difference?" So when I wrote it in the 2/16 column, I decided to head off those emails by adding, "I know, they are one and the same." It was a lame attempt at sarcasm and it didn't transfer well in cyberspace.

====

tomder55 answered on 04/01/06:

a very long post . I'll just say that I for one wanted Card to go after the Meirs call and the White House complete mis-handling and mis-reading of the Dubai Ports World deal. He did however successfully steer the confirmation of John Roberts and Samuel Alito .

In his defense ;there is a tremendous rate of 'burn out ' related to the job . From accounts I've read ,he has consitently put in 18 hr. days as White House Chief of Staff. His 5 year tenure is exceptionally long in a job that I believe has a life span of about 2 years .Had he stayed until Sept. he would've been the longest serving Chief of Staff ever (Sherman Adams being the longest at five years and nine months ). As Leon Paneta said "You can't operate on a 24-7 clock and not lose energy ".

You see however that Bush gets critized either way . If his staff remains then he is insular and surrounds himself in a comfort zone that new ideas cannot penetrate . But when his staff moves on there is obviously a shake up and instability ,and defections in the White House even though the shuffling in and out of personel in the White House and cabinet is historically a common event .

Erewhon rated this answer Excellent or Above Average Answer

Question/Answer
Erewhon asked on 03/31/06 - When the rot set into the GOP and its failing fortunes

March 31, 2006

When people get lost, they do it believing they are going the right way. It isn't until later that they realize they have made a bad turn.

So too with political miscalculations that become historical turning points. The error only becomes apparent to all in hindsight, well after it's too late to do anything about it.

The case of Terri Schiavo is a case in point. It marks the moment when a political movement of broad and deep significance crested and began to recede.

It has been exactly one year since Terri Schiavo died. She succumbed after her feeding tube was removed and other means of sustaining her life in a vegetative state were suspended. Her death brought a sad ending to years of struggle between her husband and her parents, much of it fought out in the courts.

That's the private, personal side of a story that went public when her case became a cause celebre for social conservatives. By early 2005, years of medical opinions and legal rulings had led to a final court order allowing Schiavo's husband and doctors to remove the means of life support. But as they prepared to do so, opposition among religious activists prompted congressional leaders to call members back from Easter recess.

Aroused by appeals from Schiavo's parents and various activists, the House and Senate passed emergency legislation returning the case to the courts. President Bush returned from his ranch in Texas to sign the bill into law.

For Tom DeLay and Bill Frist, the Republican majority leaders of the House and Senate, respectively, it was a moment of triumph. Both gave emotional speeches that might have been delivered as sermons from a pulpit. Frist, a medical doctor, added his professional opinion of Schiavo's condition, based on snippets of videotape. (Autopsy results showed Frist's diagnosis to have been largely fanciful.)

Whatever the personal and political motivations of these two men, the Schiavo debacle backfired for their party and their cause.

Within days, the federal courts had turned a deaf ear to the entreaties of Congress. This included the U.S. Supreme Court, which had refused to review the case before and promptly did so again. (DeLay responded by saying some of these federal judges ought to be disciplined.) With the matter back in Florida courts, the previous rulings were reaffirmed and the removal of the feeding tube followed.

Moreover, public opinion polls immediately showed that more than 70 percent of the American public thought Congress should have stayed out of it. This astonishing level of disapproval was unlike anything registered against Congress since the Republicans won control of both chambers in 1994. It was also the first stinging rebuke for the president and congressional leadership since they were swept back into power in the elections of 2004.

The Schiavo case reached its zenith of public interest in March 2005. At the time, the president was still basking in his re-election and in the renewed strength of larger majorities in Congress. He was talking about overhauling the Social Security system and then moving on to a rewrite of the tax code. Such fundaments as pensions, retirement benefits and even progressive taxation were open for revision.

In those early weeks of 2005, President Bush spoke freely about spending his political capital to get the things he wanted. His opponents seemed everywhere on the run, or at least on the defensive.

So it is hard to believe that the past 12 months have been so different from what the president and his allies envisioned. No one seems to have seen the huge wave of public disapproval rising up in reaction to the attempted Schiavo "rescue."

That failure to discern the will of the people may have been the first sign that the ruling party was not as attuned to its constituents as it had thought. Needless to say, there have been other signs since.

But the key here is the effect on confidence. At the time of the Schiavo incident, the ruling party was going forward with an air of certitude rarely seen in recent history Washington.

It's been said that one can know truth too well. If that's true for individuals, it's truer yet for groups. And surely it's truest of all for groups with a monopoly on power.

Schiavo did not knock this sense of righteous certainty out of the party, of course, but it did erect a sign that read: Too Far. And in the months ahead, this warning would prove prescient.

The next blow came with the collapse of the president's campaign to remake Social Security with personal accounts. The news from Iraq turned bad last spring after a winter of relatively encouraging events. Casualties were continuing and the troops were complaining about a lack of proper equipment. By late summer, support for the war was eroding badly even as gasoline prices soared.

Then Hurricane Katrina hit, followed by Hurricane Rita. New Orleans became a nightmare city. The president picked a crony to be a Supreme Court justice and the outcry was so great within his own party that the nomination had to be withdrawn.

By this time, the president's poll numbers had fallen down through 50 percent and even through 40 percent. And Congress was doing still worse, bedeviled by an influence scandal revolving around a lobbyist's gifts to Republican lawmakers. Both these conditions persisted into 2006, when the nation was distracted by stories about the vice president accidentally shooting a hunting partner and about the administration signing off on a deal allowing an Arab firm to run operations at six U.S. ports.

Suddenly the topics of conversation were not pensions and taxes but the need for a shakeup at the White House and a bid by some Senate Democrats to censure the president. Neither was likely to happen, in fact, but their prominence in conversation made it clear how much the agenda had changed in a year -- and how completely the political momentum had been reversed.

And that reversal began with the Frist-DeLay decision to intervene in the case of Terri Schiavo.
==

Just more evidence that you can fool some of the people some of the time, but not all of the people all of the time.

tomder55 answered on 04/01/06:

of course all these individual events are unrelated and it is a REAL Stretch by Ron Elving to tie them to the Republican support for Terry's Schiavo's struggle to live. If his broad point ;that the the Republicans are out of touch with the electorate(and to some extent I agree ) is true ;then proof positive will come in November .

Meanwhile the opposition party wastes it's time in meaningless Censure Hearings that will go no-where ,and publishing a national security position that offers absolutely nothing new.So maybe the public will have to decide between two out of touch political parties .

Erewhon rated this answer Excellent or Above Average Answer

Question/Answer
paraclete asked on 03/31/06 - How can you trust their judgement?

Rice admits "thousands" of tactical errors. How can you trust the judgement of someone who makes one tactical error?, how then can you trust the judgement of someone who makes thousands?. You can't because it means that they haven't learned by their mistakes.


Rice admits to Iraq 'mistakes'
From: Agence France-Presse
by Peter Mackler in Blackburn

April 01, 2006


US Secretary of State Condoleezza Rice has acknowledged that the United States had made "thousands" of tactical errors in Iraq but said history would vindicate the strategic goal of ousting Saddam Hussein.
Spending the day in Foreign Secretary Jack Straw's home constituency in northwest England, Rice was dogged at every turn by small but noisy protests by dissident locals in Washington's staunchest wartime ally.

But the chief US diplomat gave new ammunition to critics who contend that the US-led forces botched the occupation and reconstruction of Iraq after invading in March 2003 and quickly capturing Baghdad.

Full coverage In-depth: Iraq - the road ahead

"I know we've made tactical errors, thousands of them, I'm sure," Ms Rice told some 200 scholars, officials and journalists at a forum organised by Britain's prestigious Chatham House foreign policy institute.

"This could have gone that way, or that could have gone that way. But when you look back in history, what will be judged is, 'Did you make the right strategic decision?"'

And toppling Saddam was the correct move, Ms Rice said, since he had been a long-time threat to the international community and "we were not going to have a different kind of Middle East with Saddam at the centre of it".

Ms Rice, one of the architects of the war that has become highly unpopular in the United States, upheld Washington's quest to introduce "liberal democracy" in a region long defined by its dictatorships.

"Who today would honestly defend Arab authoritarianism which has created a sense of despair and hopelessness so desperate that it feeds an ideology of hatred that leads people to strap bombs to their bodies and fly airplanes into buildings?" she asked.

Citing Iraq as an example, Ms Rice told the forum, "With time, with painstaking efforts, and with our steadfast support, Iraqis will build up their fragile democratic culture."

"Eventually, many decades from now, people will take it for granted," she predicted. "That democratic culture was always to be -- just as we, in America and in Britain now take for granted our democratic cultures."

Several hundred critics of US and British policy in Iraq took to the streets of Blackburn -- where one in five residents are Muslims, mainly of South Asian origin -- throughout the day to denounce Ms Rice's visit.

They forced her to use a side entrance when she went to tour a high school in the former Industrial Revolution mill town, where she also visited the Blackburn Rovers soccer team and a plant where next-generation fighter jets are built.

"Condoleezza Rice, go home!" some of the marchers yelled. Others cried out: "Hey, hey Condi Rice, how many kids have you killed today?" -- paraphrasing a chestnut from the Vietnam War protests of the 1960s.

Their placards were equally poignant: "Stop the war" and "How many lives per gallon?"

The protests had already led to the cancellation of a planned visit Friday to a mosque in Blackburn, from where Ms Rice proceeded to nearby Liverpool, the port city that gave birth to the Beatles.

Ms Rice was unfazed by the protests, saying they were a symbol of all that is good about democracy.

"It's okay," she replied when a student at the high school asked what she thought. "People have a right to protest and the right to make their views known."

"I'm not just going to visit places where people agree with me, that would be really unfortunate."

Iran's suspected efforts to develop nuclear weapons was also on Ms Rice's mind Friday, as she said threats of sanctions against the Islamic republic "have to be on the agenda" after the issue was referred to the UN Security Council.

"This is a process," she said. "Where we end up in this process in terms of the potential for sanctions ... will be in part dependent on whether the Iranian regime decides to respond to the just demands of the international system."

tomder55 answered on 04/01/06:

Just can't win with Bush critics .If mistakes aren't admitted then he is called messianic ;if mistakes are admitted then he can't be trusted .

I can't imagine how the press today would cover WWII where tactical errors were made on a daily basis. I can see the hand wringing at every reversal .Every ship sunk would require hundreds of hours of inquiry . Every decison made questioned by former WWI officers sitting in as talking head pundits . "Why are we invading the Solomon Islands ? They didn't attack us ! "

Rice is right . Regardless whether our intiative to democratize Iraq succeeds or fails ,toppling Saddam Hussien was the correct move .

Erewhon rated this answer Excellent or Above Average Answer
paraclete rated this answer Excellent or Above Average Answer

Question/Answer
Erewhon asked on 03/31/06 - Rediscovering President Bush

George Bush describes himself:

I'm a mountain bike guy.
I'm the kind of fellow, when I -- when we say something, I mean it.
I'm an optimistic fellow.
I'm a Methodist.
I'm so optimistic about the future
I'm more worried about the fellow looking for the job. That's what I'm worried about.
I'm working hard to unite the country.
It will be up to future Presidents to decide the future of Iraq
Out of the rubbles of Trent Lott's house -- the guy lost his entire house -- there's going to be a fantastic house. I look forward to sitting on the porch.
Congress effectively endorsed the program of eavesdropping without warrants under its authorization of military action against al Qaeda.
I'm mindful of your civil liberties and so I had all kinds of lawyers review the process.
The program involved a "known al Qaeda suspect, making a phone call into the United States.
I'm coming to a lot of your states" to promote allowing younger workers to divert some of their Social Security taxes into bonds and stocks.
My proposed guest worker initiative will strengthen border security and help the economy.
We would much rather have security guards chasing down terrorists or drug runners or drug smugglers than people coming to work, and so, therefore, I think a guest worker program is important.
I'm for medical liability at the federal level.

And a little peep into his mind is provided for us when he said:

No question that the enemy has tried to spread sectarian violence. They use violence as a tool to do that.
If the Iranians were to have a nuclear weapon they could proliferate.
After the bombing, most Iraqis saw what the perpetuators of this attack were trying to do." George W. Bush, on the bombing of the Golden Mosque of Samarra in Iraq, March 13, 2006, Washington, D.C.
I believe that a prosperous, democratic Pakistan will be a steadfast
partner for America, a peaceful neighbor for India, and a force for freedom and moderation in the Arab world. (Identifying Pakistan as an Arab country!)
People don't need to worry about security. This deal wouldn't go forward if we were concerned about the security for the United States of America.
And I want those who are questioning it to step up and explain why all of a sudden a Middle Eastern company is held to a different standard than a Great British company.
I think it's really important for this great state of baseball to reach out to people of all walks of life to make sure that the sport is inclusive. The best way to do it is to convince little kids how tothe beauty of playing baseball. (Ask Barry Beautiful Bonds!)
I like my buddies from west Texas. I liked them when I was young, I liked them then I was middle-age, I liked them before I was president, and I like them during president, and I like them after president.
He was a state sponsor of terror. In other words, the government had declared, you are a state sponsor of terror.
I'll be glad to talk about ranching, but I haven't seen the movie. I've heard about it. I hope you go you know I hope you go back to the ranch and the farm is what I'm about to say.
It's a heck of a place to bring your family." (on New Orleans)
You took an oath to defend our flag and our freedom, and you kept that oath underseas and under fire.
As you can possibly see, I have an injury myself not here at the hospital, but in combat with a cedar. I eventually won. The cedar gave me a little scratch. As a matter of fact, the Colonel asked if I needed first aid when she first saw me. I was able to avoid any major surgical operations here, but thanks for your compassion, Colonel. (Visiting with wounded veterans from the Amputee Care Center)
[I]t's a myth to think I don't know what's going on. It's a myth to think that I'm not aware that there's opinions that don't agree with mine, because I'm fully aware of that.
I mean, there was a serious international effort to say to Saddam Hussein, you're a threat. And the 9/11 attacks extenuated that threat, as far as I-concerned. (Tying Saddam to 9-11, again!)
I think we are welcomed. But it was not a peaceful welcome. (On Cheneys pre-war assertion that the United States would be welcomed in Iraq as liberators)
Those who enter the country illegally violate the law.
As a matter of fact, I know relations between our governments is good.
Wow! Brazil is big." (After being shown a map of Brazil by Brazilian president Luiz Inacio Lula da Silva)
Bin Laden says his own role is to tell Muslims, quote, 'what is good for them and what is not.'"
I think it's important to bring somebody from outside the system, the judicial system, somebody that hasn't been on the bench and, therefore, there's not a lot of opinions for people to look at." (On the nomination of Harriet Miers to the Supreme Court The American people disagreed with him)
We look forward to hearing your vision, so we can more better do our job. That's what I'm telling you.
If it were to rain a lot, there is concern from the Army Corps of Engineers that the levees might break. And so, therefore, we're cautious about encouraging people to return at this moment of history. (Sept. 19, 2005!!!)
Listen, I want to thank leaders of the in the faith faith-based and community-based community for being here.
So please give cash money to organizations that are directly involved in helping save lives save the life who had been affected by Hurricane Katrina.
I can't wait to join you in the joy of welcoming neighbors back into neighborhoods, and small businesses up and running, and cutting those ribbons that somebody is creating new jobs.
Brownie, you're doing a heck of a job.
We've got a lot of rebuilding to do. First, we're going to save lives and stabilize the situation. And then we're going to help these communities rebuild. The good news is -- and it's hard for some to see it now -- that out of this chaos is going to come a fantastic Gulf Coast, like it was before. Out of the rubbles of Trent Lott's house -- he's lost his entire house -- there's going to be a fantastic house. And I'm looking forward to sitting on the porch.
My thoughts are, we're going to get somebody who knows what they're talking about when it comes to rebuilding cities.
Americans should be prudent in their use of energy during the course of the next few weeks. Don't buy gas if you don't need it.
It's totally wiped out. ... It's devastating, it's got to be doubly devastating on the ground."
"The best place for the facts to be done is by somebody who's spending time investigating it.
I'm looking forward to a good night's sleep on the soil of a friend.
I was going to say he's a piece of work, but that might not translate too well. Is that all right, if I call you a 'piece of work'? (To Jean-Claude Juncker, prime minister of Luxembourg)
The relations with, uhh Europe are important relations, and they've, uhh because, we do share values. And, they're universal values, they're not American values or, you know European values, they're universal values. And those values uhh being universal, ought to be applied everywhere.
You see, not only did the attacks help accelerate a recession, the attacks reminded us that we are at war.
And the second way to defeat the terrorists is to spread freedom. You see, the best way to defeat a society that is doesn't have hope, a society where people become so angry they're willing to become suiciders, is to spread freedom, is to spread democracy.
It seemed like to me they based some of their decisions on the word of and the allegations by people who were held in detention, people who hate America, people that had been trained in some instances to disassemble that means not tell the truth. (On Amnesty International report on prisoner abuse at Guantanamo Bay)
See, in my line of work you got to keep repeating things over and over and over again for the truth to sink in, to kind of catapult the propaganda.
We discussed the way forward in Iraq, discussed the importance of a democracy in the greater Middle East in order to leave behind a peaceful tomorrow.
I think younger workers first of all, younger workers have been promised benefits the government promises that have been promised, benefits that we can't keep. That's just the way it is.
It means your own money would grow better than that which the government can make it grow. And that's important.
I can only speak to myself. (!)
It's in our country's interests to find those who would do harm to us and get them out of harm's way.
We expect the states to show us whether or not we're achieving simple objectives like literacy, literacy in math, the ability to read and write.
He understands the need for a timely write of the constitution.
Well, we've made the decision to defeat the terrorists abroad so we don't have to face them here at home. And when you engage the terrorists abroad, it causes activity and action.
But Iraq has have got people there that are willing to kill, and they're hard-nosed killers. And we will work with the Iraqis to secure their future.
I appreciate my love for Laura.
We have enough coal to last for 250 years, yet coal also prevents an environmental challenge.
Part of the facts is understanding we have a problem, and part of the facts is what you're going to do about it.
I'm going to spend a lot of time on Social Security. I enjoy it. I enjoy taking on the issue. I guess, it's the Mother in me.
We look forward to analyzing and working with legislation that will make it would hope put a free press's mind at ease that you're not being denied information you shouldn't see.

"I want to thank you for the importance that you've shown for education and literacy." George W. Bush, Washington, D.C., April 13, 2005

"I understand there's a suspicion that wewe're too security-conscience." George W. Bush, Washington D.C., April 14, 2005

"If they pre-decease or die early, there's an asset base to be able to pass on to a loved one." George W. Bush, on Social Security money held in private accounts, Cedar Rapids, Iowa, March 30, 2005

[I'm] occasionally reading, I want you to know, in the second term." George W. Bush, Washington, D.C., March 16, 2005

"In this job you've got a lot on your plate on a regular basis; you don't have much time to sit around and wander, lonely, in the Oval Office, kind of asking different portraits, 'How do you think my standing will be?'" George W. Bush, Washington, D.C., March 16, 2005

"In terms of timetables, as quickly as possible whatever that means." George W. Bush, on his time frame for shoring up Social Security, Washington D.C., March 16, 2005

"I like the idea of people running for office. There's a positive effect when you run for office. Maybe some will run for office and say, vote for me, I look forward to blowing up America. I don't know, I don't know if that will be their platform or not. But it's -- I don't think so. I think people who generally run for office say, vote for me, I'm looking forward to fixing your potholes, or making sure you got bread on the table." George W. Bush, on elections in the Middle East, Washington, D.C., March 16, 2005

"I repeat, personal accounts do not permanently fix the solution." George W. Bush, Washington, D.C., March 16, 2005

"This notion that the United States is getting ready to attack Iran is simply ridiculous. And having said that, all options are on the table." George W. Bush, Brussels, Belgium, Feb. 22, 2005

"If you're a younger person, you ought to be asking members of Congress and the United States Senate and the president what you intend to do about it. If you see a train wreck coming, you ought to be saying, what are you going to do about it, Mr. Congressman, or Madam Congressman?" George W. Bush, Detroit, Mich., Feb. 8, 2005

"Because the all which is on the table begins to address the big cost drivers. For example, how benefits are calculate, for example, is on the table; whether or not benefits rise based upon wage increases or price increases. There's a series of parts of the formula that are being considered. And when you couple that, those different cost drivers, affecting those changing those with personal accounts, the idea is to get what has been promised more likely to be or closer delivered to what has been promised. Does that make any sense to you? It's kind of muddled. Look, there's a series of things that cause the like, for example, benefits are calculated based upon the increase of wages, as opposed to the increase of prices. Some have suggested that we calculate the benefits will rise based upon inflation, as opposed to wage increases. There is a reform that would help solve the red if that were put into effect. In other words, how fast benefits grow, how fast the promised benefits grow, if those if that growth is affected, it will help on the red." George W. Bush, explaining his plan to save Social Security, Tampa, Fla., Feb. 4, 2005

"You work three jobs? Uniquely American, isn't it? I mean, that is fantastic that you're doing that." George W. Bush, to a divorced mother of three, Omaha, Nebraska, Feb. 4, 2005 (Listen to audio)

"After all, Europe is America's closest ally." George W. Bush, Mainz, Germany, Feb. 23, 2005

"Because he's hiding." George W. Bush, responding to a reporter who asked why Osama bin Laden had not been caught, aboard Air Force One, Jan. 14, 2005

"I'm also mindful that man should never try to put words in God's mouth. I mean, we should never ascribe natural disasters or anything else to God. We are in no way, shape, or form should a human being, play God." George W. Bush, ABC's 20/20, Washington D.C., Jan. 14, 2005

"I want to appreciate those of you who wear our nation's uniform for your sacrifice." George W. Bush, Jacksonville, Fla., Jan. 14, 2005

"I speak plainly sometimes, but you've got to be mindful of the consequences of the words. So put that down. I don't know if you'd call that a confession, a regret, something." George W. Bush, speaking to reporters, Washington, D.C., Jan. 14, 2005

"Who could have possibly envisioned an erection an election in Iraq at this point in history?" George W. Bush, at the white House, Washington, D.C., Jan. 10, 2005

"We need to apply 21st-century information technology to the health care field. We need to have our medical records put on the I.T." George W. Bush, Collinsville, Ill., Jan. 5, 2005

"I believe we are called to do the hard work to make our communities and quality of life a better place." George W. Bush, Collinsville, Ill., Jan. 5, 2005

Whatever else can be said about him, you have to agree that he and Yogi Berra are equally as impressive!

tomder55 answered on 04/01/06:

I have in the past made comparisons of Presidents Bush and Truman .I think the comparisons are valid especially when considering that both were in office during transitional points in our history .

Both are plain spoken men and both suffered from low approval ratings throughout their Presidencies .Truman didn't pay much attention to them. "It isn't polls or public opinion of the moment that counts,It is right and wrong and leadership." Neither does Bush."I'm not a poll-watcher."

How plain spoken was Truman ? Well one time he had a sit down meeting with Ruskie Foreign minister Molotov. He laced into him . Molotov later complained to Truman "I have never been talked to like that in my life."Truman's responded : "Carry out your agreements and you won't get talked to like that."Bush likewise lacks what Kerry called 'nuance'. He challenged the UN to enforce their mandates and when it became apparent that was going no-where Bush just said : "America will never seek a permission slip to defend the security of our people."

Truman boldly drew a line around the Soviet Union and invested heavily both in American treasure and blood so that democracies would have a chance to blossom in both Europe and the Far East . Bush likewise is investing in the proposition that democracy can be seeded and grow in the ummah .The Truman Doctrine was the model used to wage the Cold War ,even though initially it was was panned by nay-sayers .They warned that Truman would weaken the Constitution, over-inflate the presidency, militarize U.S. foreign policy. and destroy the United Nations. (Sound familiar?)Likewise, the Bush Doctrine will continue to be the model to wage the war against Jihadistan long after the Bush presidency is finished .Just look at the latest Democrat national security "plan" . All it really amounts to is a rehash of policies already in place.

You bemoan Bush's use of so called expanded executive authorities . He doesn't even come close to Truman in that regard . Truman tried to nationalize the country's steel mills in a bid to preempt a strike, citing the war in Korea as his justification. He created the National Security Agency (NSA)by executive fiat. He endorsed a "federal loyalty program that keeps Communists out of government." And he created the CIA .

Truman left the White House unpopular and history has since declared him one of the greatest US Presidents. This leads back to another of your questions yesterday . Historians will not just detail mistakes the President has made but will document his achievements which Condi Rice correctly suggested can only be viewed through the lens of history .

Erewhon rated this answer Excellent or Above Average Answer

Question/Answer
Erewhon asked on 03/31/06 - Bush Administration 's Mistakes ...

Condoleeza Rice has said,

"I am certain that there will be many dissertations written on the mistakes made by the Bush administration."

[Dr Condoleeza Rice in response to a question at Chatham House, Blackburn, Lancashire, England, 31 March 2006, carried on C-Span]

===

Which three mistakes of the Bush Administration do you think merit the most dissertative attention?



tomder55 answered on 04/01/06:

Easy :

1.His relationship with a Congress that is led by his party :

going along w/ Ted Kennedy with the No Child Left Behind Legislation

supporting and signing the AARP led Medicare Perscription Drug entitlement

generally refusing the reign in the Republican led congress that forgot they were elected because of a promise of fiscal discipline.

He should be all over them to get his judicial nominees seated

And not pushing hard enough for his legislative initiatives .

2. Allowing State Dept. decisions under Powell in Iraq to waste a valuable years time during the disasterous Viceroy Bremer CPA period .

3. His lack of seriousness regarding border security .

Erewhon rated this answer Excellent or Above Average Answer

Question/Answer
ETWolverine asked on 03/31/06 - Here's something I don't understand.

On one hand we have the vast majority of the members in Congress on both sides of the aisle against the Dubai prts deal... and correctly so. The reason that so many were against the deal was because of scurity issues that have not been answered and the threat of terrorism. And these are valid concerns.

On the other hand, we have many of those same members of Congress who are in favor of the immigration amnesty plan passed by the Senate yesterday. Many supporters of the plan support it because of the cheap labor it brings in. But the plan does nothing to secure our borders and mitigate the threat of terrorism.

Why are the same people who are so concerned with secure ports so unconcerned with secure borders? Isn't it the same risk?

Notice that I'm not naming specific names here. I think there are people ob BOTH sides of the aisle who are guilty of this... shall we say... discrepancy, and I hold the GOP members who are being so two-sided on this as responsible as the DNC members.

Does anybody have a reconcilliation of these two stances? Or am I right to wonder at the discrepancy?

Opinions, please.

tomder55 answered on 03/31/06:

Gotta agree with drgade . The swing of the Hispanic vote will decide many elections in the future . We will be called racists for being concerned about the security implications of an open border ,just like we were during the DPW discussions .

We shot it all in the Dubai Port controversy so much so that no one is making a peep about Hutchison Whampoa Ltd. ;a firm with clear known ties to the Chinese Army ; will be given no-bid contract to run sophisticated U.S. radiation detectors to scan cargo destined to US Ports .

The administration has acknowledged the deal represents the first time a foreign company will be involved in running sophisticated U.S. radiation-detection equipment at an overseas port without American customs agents present.

ETWolverine rated this answer Excellent or Above Average Answer

Question/Answer
Itsdb asked on 03/31/06 - Be afraid...

"In Africa, drought continues for the sixth consecutive year, adding terribly to the toll of famine victims.

During 1972 record rains in parts of the U.S., Pakistan and Japan caused some of the worst flooding in
centuries. In Canada's wheat belt, a particularly chilly and rainy spring has delayed planting and may
well bring a disappointingly small harvest. Rainy Britain, on the other hand, has suffered from
uncharacteristic dry spells the past few springs. A series of unusually cold winters has gripped the
American Far West, while New England and northern Europe have recently experienced the mildest
winters within anyone's recollection.

As they review the bizarre and unpredictable weather pattern of the past several years, a growing
number of scientists are beginning to suspect that many seemingly contradictory meteorological
fluctuations are actually part of a global climatic upheaval. However widely the weather varies from
place to place and time to time, when meteorologists take an average of temperatures around the globe they find that the atmosphere has been growing gradually cooler for the past three decades. The trend shows no indication of reversing. Climatological Cassandras are becoming increasingly apprehensive, for the weather aberrations they are studying may be the harbinger of another ice age.

Telltale signs are everywhere..."


From: Another Ice Age?, Time Magazine, Monday, Jun. 24, 1974

Fast forward to today...

Study finds `classic global warming' over Antarctica

Los Angeles Times
Published March 31, 2006

In the winter sky over Antarctica, scientists have detected a vast cap of steadily warming air, in the first sign that record levels of greenhouse gases in the atmosphere may be trapping heat above the ice sheets of the South Pole.

The temperature of the winter air over Antarctica has been rising at a rate three times faster than the world as a whole, the researchers reported Thursday in the journal Science.

By analyzing 30 years of high-altitude weather balloon records, meteorologists at the British Antarctic Survey in Cambridge concluded that temperatures in the polar troposphere, the dense layer of air reaching from the surface to an altitude of about 5 miles, have risen by 3.6 degrees Fahrenheit since the early 1970s.

"We have the largest regional warming on Earth at the tropospheric level," said climate specialist John Turner, who led the research team.

In their study, Turner and his colleagues drew on daily temperature records from 1971 to 2003 kept by eight international research stations that rim the continent and the U.S. station at the South Pole. It was the first time anyone had been able to collate all the high-altitude atmosphere readings.

When the researchers examined the data, they not only saw evidence of winter season warming throughout the troposphere, but a cooling in the stratosphere above, a layering effect that researchers predict as a consequence of greenhouse warming.

"We have the classic global warming signal," Turner said. "It is like the blanket on the bed: When we wrap the Earth with a blanket of greenhouse gases like carbon dioxide and methane, we trap heat under it at the expense of the atmosphere above, which then cools."

*******************************************************

Forget that in 1974 the climate had been cooling for 3 decades with no end in sight, though today's report says it's been getting warmer in Antarctica since 1971 - anyone know the temperature in Antarctica?

tomder55 answered on 03/31/06:

oh ..I get it . The solution is for us to live in poverty and for the Chinese and India to prosper and generate the green-house gasses . .......Duh .....why didn't I think of that ???

ETWolverine rated this answer Excellent or Above Average Answer
Itsdb rated this answer Excellent or Above Average Answer

Question/Answer
Itsdb asked on 03/31/06 - Be afraid...

"In Africa, drought continues for the sixth consecutive year, adding terribly to the toll of famine victims.

During 1972 record rains in parts of the U.S., Pakistan and Japan caused some of the worst flooding in
centuries. In Canada's wheat belt, a particularly chilly and rainy spring has delayed planting and may
well bring a disappointingly small harvest. Rainy Britain, on the other hand, has suffered from
uncharacteristic dry spells the past few springs. A series of unusually cold winters has gripped the
American Far West, while New England and northern Europe have recently experienced the mildest
winters within anyone's recollection.

As they review the bizarre and unpredictable weather pattern of the past several years, a growing
number of scientists are beginning to suspect that many seemingly contradictory meteorological
fluctuations are actually part of a global climatic upheaval. However widely the weather varies from
place to place and time to time, when meteorologists take an average of temperatures around the globe they find that the atmosphere has been growing gradually cooler for the past three decades. The trend shows no indication of reversing. Climatological Cassandras are becoming increasingly apprehensive, for the weather aberrations they are studying may be the harbinger of another ice age.

Telltale signs are everywhere..."


From: Another Ice Age?, Time Magazine, Monday, Jun. 24, 1974

Fast forward to today...

Study finds `classic global warming' over Antarctica

Los Angeles Times
Published March 31, 2006

In the winter sky over Antarctica, scientists have detected a vast cap of steadily warming air, in the first sign that record levels of greenhouse gases in the atmosphere may be trapping heat above the ice sheets of the South Pole.

The temperature of the winter air over Antarctica has been rising at a rate three times faster than the world as a whole, the researchers reported Thursday in the journal Science.

By analyzing 30 years of high-altitude weather balloon records, meteorologists at the British Antarctic Survey in Cambridge concluded that temperatures in the polar troposphere, the dense layer of air reaching from the surface to an altitude of about 5 miles, have risen by 3.6 degrees Fahrenheit since the early 1970s.

"We have the largest regional warming on Earth at the tropospheric level," said climate specialist John Turner, who led the research team.

In their study, Turner and his colleagues drew on daily temperature records from 1971 to 2003 kept by eight international research stations that rim the continent and the U.S. station at the South Pole. It was the first time anyone had been able to collate all the high-altitude atmosphere readings.

When the researchers examined the data, they not only saw evidence of winter season warming throughout the troposphere, but a cooling in the stratosphere above, a layering effect that researchers predict as a consequence of greenhouse warming.

"We have the classic global warming signal," Turner said. "It is like the blanket on the bed: When we wrap the Earth with a blanket of greenhouse gases like carbon dioxide and methane, we trap heat under it at the expense of the atmosphere above, which then cools."

*******************************************************

Forget that in 1974 the climate had been cooling for 3 decades with no end in sight, though today's report says it's been getting warmer in Antarctica since 1971 - anyone know the temperature in Antarctica?

tomder55 answered on 03/31/06:

I am sure that the only constant in the earth's climate is change . The buzz words are "probably", "may", and "could" .I keep on hearing about a concensus among "serious" scientists on this issue but although I am happy I only needed to burn a cord of wood instead of my ususal 2 this winter ;I'm sure the Russians would've appreciated the extra heat .A record freeze occurred in Russia that destroyed an estimated 30% of its winter crops.
In New England this winter reversed an 8 year cooling trend .

I would have to say that in the course of my life time there is no noticible significant difference in average temps. ,rain-fall ,winter cold/snow . Oh sure there are years when we go on drought alert and years when I think I should build an ark .But taken over a life-time ;nothing significant.

Perhaps a life-time is too short a time to measure .It ;probably is except that the scientists keep on telling us it is current human activity that is causing mother nature to rebel .

When you look in the course of human history there have been dramatic fluctuations in the earth's climate . You see pictures of Colonials walking across the river in the winter between Brooklyn and Manhattan . On the coldest winters that would never happen today . The scientists are concerned about melting ice in Greenland .But there was a time in recorded history that Greenland was... well ...green land .The Vikings lived there for 300 years . Then it got cold and the Vikings had to move to Minnesota .On an even longer time frame ,science tells us that we are at the end of a 12,000 year interglacial period.So warn your future clan now that they best bundle up ;we are due for another Ice Age.

These are facts not invented by the oil industry to mislead but are found in any study of geologic history .

For my 2 cents ;I think they look at trends in too short a time span and of course make an assumption that is is human activity that is the cause. What human activity in the Middle Ages caused Greenland's climate to warm sufficently to be suitable for human habitation ? Oh wait !! I know !!!! The peasants were strip cutting down the forests and growing crops ...that caused the warming . THEN ;they were culled with the black plague and their population decreased by a third ;there was a re:forestation and that caused the huge glaciers on Greenland ...yeah ..that's the ticket !!

anyway ...we just came out of a cold spell in March and the temperatures have risen a good 20 degrees . If that trend continues I should be able the fry an egg on the pavement come August. Lord have mercy !

Itsdb rated this answer Excellent or Above Average Answer

Question/Answer
JBodine asked on 03/27/06 - Just 'cause it's funny....

Monday, March 27, 2006 3:12 p.m. EST
Justice Scalia Flips Critics the Bird in Church

U.S. Supreme Court Justice Antonin Scalia startled reporters in Boston just minutes after attending a mass, by flipping a middle finger to his critics.

A Boston Herald reporter asked the 70-year-old conservative Roman Catholic if he faces much questioning over impartiality when it comes to issues separating church and state.

You know what I say to those people? Scalia replied, making the obscene gesture and explaining "That's Sicilian."

The 20-year veteran of the high court was caught making the gesture by a photographer with The Pilot, the Archdiocese of Boston's newspaper . "Don't publish that," Scalia told the photographer, the Herald said.

He was attending a special mass for lawyers and politicians at Cathedral of the Holy Cross, and afterward was the keynote speaker at the Catholic Lawyers' Guild luncheon.

Copyright 2006 United Press International, Inc. All Rights Reserved. Via NewsCom.

Can't post this on the Christianity board. Christians shouldn't find humor in this. ;)

tomder55 answered on 03/30/06:

as usual the press got it wrong . the jesture was posted today on Drudge. Being part Italian heritage I can say that it is not an obscene gesture . My religious grandparents used it all the time .




It basically means 'I could care less' or 'leave me alone' .

JBodine rated this answer Excellent or Above Average Answer

Question/Answer
Itsdb asked on 03/29/06 - Poll

You've probably heard that Sharon Stone says Hillary is too sexy to be president. She said, among other things, "A woman should be past her sexuality when she runs. Hillary still has sexual power and I don't think people will accept that. It's too threatening."

So, a poll - not limited to male responses.



Do you think Hillary is too sexy to be president?

tomder55 answered on 03/30/06:

Itsdb rated this answer Excellent or Above Average Answer
JBodine rated this answer Excellent or Above Average Answer

Question/Answer
HerrAirhorn asked on 03/28/06 - Third World War

From Dailynews dot com:

"Eric Haney, a retired command sergeant major of the U.S. Army, was a founding member of Delta Force, the military's elite covert counter-terrorist unit. He culled his experiences for "Inside Delta Force" (Delta; $14), a memoir rich with harrowing stories, though in an interview, Haney declines with a shrug to estimate the number of times he was almost killed. (Perhaps the most high-profile incident that almost claimed his life was the 1980 failed rescue of the hostages in Iran.) Today, he's doing nothing nearly as dangerous: He serves as an executive producer and technical adviser for "The Unit," CBS' new hit drama based on his book, developed by playwright David Mamet. Even up against "American Idol," "The Unit" shows muscle, drawing 18 million viewers in its first two airings.

Since he has devoted his life to protecting his country in some of the world's most dangerous hot spots, you might assume Haney is sympathetic to the Bush administration's current plight in Iraq (the laudatory cover blurb on his book comes from none other than Fox's News' Bill O'Reilly). But he's also someone with close ties to the Pentagon, so he's privy to information denied the rest of us.

We recently spoke to Haney, an amiable, soft-spoken Southern gentleman, on the set of "The Unit."

Q: What's your assessment of the war in Iraq?

A: Utter debacle. But it had to be from the very first. The reasons were wrong. The reasons of this administration for taking this nation to war were not what they stated. (Army Gen.) Tommy Franks was brow-beaten and ... pursued warfare that he knew strategically was wrong in the long term. That's why he retired immediately afterward. His own staff could tell him what was going to happen afterward.

We have *fomented civil war* in Iraq. We have probably fomented internecine war in the Muslim world between the Shias and the Sunnis, and I think *Bush may well have started the third world war*, all for their own personal policies.

Q: What is the cost to our country?

A: For the first thing, our credibility is utterly zero. So we destroyed whatever credibility we had. ... And I say "we," because the American public went along with this. They voted for a second Bush administration out of *fear, so fear is what they're going to have from now on*.

Our military is completely consumed, so were there a real threat - thankfully, there is no real threat to the U.S. in the world, but were there one, we couldn't confront it. Right now, that may not be a bad thing, because that keeps Bush from trying something with Iran or with Venezuela.

The harm that has been done is *irreparable*..."


This seems to me to be a realistic assessment of the outcome of Bush's neo-Con philosoply implemented as the War in Iraq....

tomder55 answered on 03/29/06:

I have to tell you ;I went to some sites where former Special Ops contribute and the concensus is that Haney has "gone Hollywood".

THE UNIT

What: Action-adventure about special-ops unit.
Where: CBS
When: 9 p.m. Tuesdays.

But, according to Haney, all that creative and ex-Army firepower wasn't what drew CBS chief executive Les Moonves to the drama about Special Forces operatives at work and at home, where their families cope with a life of secrecy.

"Had you come in here with only an action series, I would have passed. But when you said the wives, the sweethearts, that depth of humanity, that's when you had me," Haney recalled Moonves saying after a successful pitch meeting.

Wags have dubbed the series "Desperate Housewives Meet G.I Joe."

In "The Unit," perilous missions are mixed with domestic dangers that include an extramarital affair, adolescent angst and wives who resent the camouflage needed to protect their husbands and the national interest.
.........................................

Tommy Franks wrote a best selling book about the war after he retired . Nowhere in the book does he suggest that he was "brow-beaten " ;nor does he dispute in any way the strategy . Why should he ;he completed his phase of the war rapidly even though he did not have the 4th ID to create a Northern Front . He praised Bush and Rummy in it .

I gotta tell Eric Haney to wake up and smell jihadistan . World War III was well underway before Bush assumed office . He should know better .Haney is best known for his escape during the 1980 failed rescue of the hostages in Iran so he was in the fight against Islamo-fascism from the beginning .

I'm sure he was an outstanding Delta Force commander and we should all be tremendously grateful for his service .I intend to read his book 'Inside Delta Force' as I am sure it is fascinating but being a founding member of Delta does not make him an automatic expert on global strategy and even if he has credentials that make him an expert ;even experts have come to various conclusions . I happen to disagree with his assertion that US actions have caused the chain of events he describes .

ETWolverine rated this answer Excellent or Above Average Answer
HerrAirhorn rated this answer Excellent or Above Average Answer

Question/Answer
JBodine asked on 03/28/06 - Anywhere else put up with this?

The previous questions concerning immigration got me thinking.

Another country where a great many people are trying to get to is Israel. The influx from all around the world is astounding.

Does Israel welcome those who want to come in live in peace and make their own way? Absolutely.

Does America welcome those who want to come in and live in peace and make their own way? Absolutely.

Does Israel have a problem with illegal immigration? Judging from the caliber of the IDF troops that I've met, I'd have to give that one a big NO.

Does America have a problem with illegal immigration?
Uh-huh. Big one.

So why the difference? Does there NEED to be a difference? No, not at all. Surely there is no one (of rational mind) that has a problem with LEGAL immigration, from anywhere to anywhere. So why are those who oppose ILLEGAL immigration suddenly the villians? I suppose if I oppose murder, extortion, kidnapping, drinking and driving, etc. that I am a villian for wanted the EXISTING LAWS upheld?

Why the difference?

Sure Mexico and some other places suck, and the people there have legitimate needs. But, that is the problem of THOSE governments, NOT mine.

I ask again, where else could a person possibly get away with as much as illegal immigrants are getting away with here? Furthermore, what other country cares more about the feelings of NON-CITIZENS than of their own citizens and taxpayers?

DK

tomder55 answered on 03/28/06:

Certainly not in Mexico .The double standard in Mexico obvious. They treat Central American illegals with absolute brutality and have an Army on their southern border to intercept . Check out the comparison of the Mexican immigration laws to the US laws .

JBodine rated this answer Excellent or Above Average Answer
paraclete rated this answer Excellent or Above Average Answer
fredg rated this answer Excellent or Above Average Answer

Question/Answer
Erewhon asked on 03/28/06 - Why don't the Minutemen and the anti-immigrant worker brigade do this? .......


The solution to the alleged problem of illegal immigrant workers is in the hands of American citizens. They could put an end to it if they were seriously concerned about it enough to do somehting baout it.

Immigrant workers only come to the USA because there are an estimated 12,000,000 jobs that American citizens will not do.

If concerned American citizens would go and fill those jobs themselves, there would be no work for illegals to do and they would stop coming.

Will you be out in the fields and orchards any day soon to help solve your problem, or would you rather the crops and harvests rot on the vine, fall from the tree, or stand unharvested in the field?

You got a better solution?

tomder55 answered on 03/28/06:

nonsense

What jobs do foreigners do that Americans don't? If the wages are high enough, people will do just about anything. If you can get Americans to drive fuel trucks down Iraqi highways, you can get them to do anything else-- if the price is right.When more workers are added to the labor pool, the existing workers face more competition for jobs. Whoever is in competition with the illegal immigrants will lose ;especially when the employer often times pays cash to by-pass the payroll taxes .Suppose you had to compete for a job that the wages are sub-par because illegal workers are willing to by-pass labor laws ?Yeah ;American workers do not want to take jobs that would exploit them .

I do not understand the mind of the liberal . The same person who argues that Walmart should upgrade it's wages and benefits for their workers ,and are forever advocating an increase in the minimum wage ,would allow and encourage people to come here in work under exploited conditions . The same people who claim that ousourcing work to other countries is bad because the foreign worker is exploited would invite millions of these foreign workers to get exploited here in the States.The same people who rail against the US coporation and it's exploitation of the worker turns a blind eye when US corporations ignore and violate the labor laws when it come to hiring illegals . Liberals cannot go around complaining about economic inequality and stagnating middle class wages while at the same time supporting this basic exploitation .

This doesn't just effect your family person trying to care for their family . Employment of teenage workers is lower than it's historically been, probably because they face more competition from illegals for minimum-wage jobs. They are likely to hold a job longer than a teen seeking seasonal work, and are therefore more attractive to employers.

The system exploits them in so many ways .Do you think it is ok that they come here and get phony S.S .cards ;that when they do they pay into a retirement system knowing they will probably never see a dime of the money ? Is that fair ? How can you support it ?

Erewhon rated this answer Excellent or Above Average Answer
Itsdb rated this answer Excellent or Above Average Answer

Question/Answer
paraclete asked on 03/28/06 - When even your friends won't lie for you

Bush 'early decision' on Iraq invasion
From: Reuters
From correspondents in New York

March 27, 2006

US President George W Bush made clear to British Prime Minister Tony Blair in January 2003 that he was determined to invade Iraq without a UN resolution, The New York Times reported today.
It said he planned to go ahead even if UN arms inspectors failed to find weapons of mass destruction in the country.
Citing a confidential British memorandum, the newspaper said the president was certain that war was inevitable and made his view known during a private two-hour meeting with Blair in the Oval Office on January 31, 2003.
Information about the meeting was contained in the memo written by Mr Blair's top foreign policy adviser and reviewed by The Times.
"Our diplomatic strategy had to be arranged around the military planning," the paper quotes David Manning, Mr Blair's chief foreign policy adviser at the time, as noting in the memo.
" 'The start date for the military campaign was now pencilled in for 10 March'," Mr Manning wrote, paraphrasing the president. " 'This was when the bombing would begin'," the paper continued.

The timetable came at an important diplomatic moment, the paper said.
Five days after the Bush-Blair meeting, then US secretary of state Colin Powell was scheduled to appear before the United Nations to present evidence that Iraq posed a threat to world security by hiding unconventional weapons.
Stamped "extremely sensitive", the five-page memorandum had not been made public, according to the report. Several highlights were first published in January in the book Lawless World, which was written by British lawyer and international law professor Philippe Sands.
In early February, Channel 4 in London first broadcast excerpts from the memo.
But since then, The New York Times has been able to review the five-page memo in its entirety.
The document indicates the two leaders envisioned a quick victory and a transition to a new Iraqi government that would be complicated, but manageable, the paper said.
Mr Bush predicted that it was "unlikely there would be internecine warfare between the different religious and ethnic groups". Mr Blair agreed with that assessment.
The memo also shows that the president and the prime minister acknowledged that no unconventional weapons had been found inside Iraq, The Times noted.
Faced with the possibility of not finding any before the planned invasion, Mr Bush talked about several ways to provoke a confrontation, including a proposal to paint a US surveillance plane in the colours of the United Nations in hopes of drawing fire, or assassinating Iraqi president Saddam Hussein.

tomder55 answered on 03/28/06:

Of course they had to make plans for the war . Were they supposed to wait for all diplomatic initiatives to be exhausted before they planned it ?Bush denying it implies some kind of guilt of wrong-doing where there is none . This is humourous . The biggest critique of the war here is that there was not enough adequate planning . First ;it's no plan and now it's wrong because Bush had a plan .Do you not see that the NY Slimes is ludicrous ?

Any more of a delay ,and the invasion would have had to wait until the autumn . As it was ;the delay caused by the negotiation process gave Saddam suitable time to move his WMD program out of the country .

This is nothing ,just like the Downing Street memos which were later debunked ;authored by a hack who opposed the policy .Saddam was the one that decided not to comply with UN resolutions and ultimately only he could've prevented the war .Bush gave Saddam opportunity after opportunity to disclose his nuclear, chemical, and biological weapons programs to the UN and repeatedly Saddam met our negotiations with belligerance, lies and obfustication. Bush even gave Saddam 48 hours before the bombs started dropping to step down and select a successor for Iraq, to avoid a military conflict.

My guess ;the NY Times is trying to seize the news initiative from the Administration after the explosive content of the Saddam Papers and Tapes were disclosed . This is not new in the MSM . The released an new set of the old photos of Abu Ghraib after the story had been played out ;any thing to damage and discredit this administration . Nope ;no agenda there !

Erewhon rated this answer Excellent or Above Average Answer
Itsdb rated this answer Excellent or Above Average Answer
JBodine rated this answer Excellent or Above Average Answer
paraclete rated this answer Excellent or Above Average Answer
purplewings rated this answer Excellent or Above Average Answer

Question/Answer
paraclete asked on 03/28/06 - where is that extraordinary place?

Australia extraordinary country: Blair
From: AAP

March 27, 2006

AUSTRALIA was an extraordinary country with a sense of vibrancy and optimism, British Prime Minister Tony Blair said today.

Addressing a lunch in the Great Hall of Parliament House in his honour, Mr Blair said there were two recent events that demonstrated the Australian spirit.
"One is obviously the tragic events in the north of Queensland where the response of the federal and state governments, but most of all as I know you want to acknowledge the people there, has been quite remarkable and quite extraordinary in the face of adversity," he said.
"The second thing has been the Commonwealth Games itself. You must have felt the most extraordinary pride over the events that have unfolded in the days of the Games.
"That really sums up for me Australia. It is an extraordinary country and it is a country where I hope you always keep that sense of vibrancy and optimism.

tomder55 answered on 03/28/06:

Blairs comments about both Australia and the US were outstanding !

He did not even scratch the surface of Aussie greatness. To me the event of the last week that proved Australia was an extrordinary country was dropping a 2000 lb. bomb on that N.Korean pirate ship !!!

so ;how does your car run on that Italian wax ?

paraclete rated this answer Excellent or Above Average Answer

Question/Answer
Itsdb asked on 03/27/06 - Helping the cause?

Thousands of folks gathered for 'immigrant rallies' over the weekend as I'm sure you know, take a look at these pictures (hopefully this worked):







Looking at all those Mexican flags, does that help their cause? I mean seriously, if they're all so proud of Mexico why are they in the U.S. - fighting for U.S. rights while waving Mexican flags?

Steve

tomder55 answered on 03/27/06:

this is called reconquista . house by house ;block by block ...wake up and smell the refried beans .

this is the equivalent of hopping your neighbors fence ,pitching a tent in your neighbor's yard and then insisiting the neighbor pay your medical bills and your childs education . then protesting when your neighbor asks that your cousins are not permitted to also hop his fence .

Supposedly on April 10 there will be another spontanious orchestrated protest nationwide that is expected to draw 5 million attendees.

Like I say ;your right to own land is contingent on your ability and will to defend it . That's the bottom line.

ETWolverine rated this answer Excellent or Above Average Answer
Itsdb rated this answer Excellent or Above Average Answer
sissypants rated this answer Excellent or Above Average Answer

Question/Answer
Erewhon asked on 03/25/06 - Did Russian spies inside US Central Command tip off Saddam before the US Attack?


Moscow spies tipped Saddam on U.S. war plan

By Rowan Scarborough
THE WASHINGTON TIMES
March 25, 2006

Moscow had informants inside U.S. Central Command whose information on the March 2003 invasion of Iraq was relayed to dictator Saddam Hussein days before American troops ousted him from power, according to a Defense Department history released yesterday.


And, as U.S. troops encircled Baghdad in April, Russia's ambassador fed information from Moscow's intelligence service to Saddam's regime regarding U.S. troop movements.

The new disclosures show that Moscow was working against the Bush administration in private, as it opposed in public the U.S. desire for a United Nations Security Council resolution explicitly authorizing the invasion.

The report was produced in book form by U.S. Forces Command, which studies "lessons learned" in military operations. This document, however, focused not on American units, but on how Saddam, his regime and military prepared for the March 19, 2003, attack and tried to blunt it. Titled "Iraqi Perspective Project: A View of Operation Iraqi Freedom From Saddam's Senior Leadership," it is based largely on postwar interviews and seized documents. It is in one of those documents that the Iraqis told of spies inside U.S. Central Command, which planned and executed the invasion.

"The information that the Russians have collected from their sources inside the American Central Command in Doha [Qatar] is that the United States is convinced that occupying Iraqi cities are impossible, and that they have changed their tactic," states the Iraq intelligence report. Another part states, "Jordan had accepted the American 4th Mechanized Infantry Division."

Although U.S. forces did avoid occupying towns and cities on the march to Baghdad, they did enter Tikrit, Mosul and other large cities. But as for Jordan, the 4th Infantry Division never docked there, instead traveling by sea to Kuwait.

The Forces Command report offered no information on whether Central Command ever identified and purged the spies.

The report also tells of a seized memorandum from Iraqi's Foreign Ministry. The memo said Russia's ambassador was relaying intelligence reports to Saddam aides, including one memo that stated that allied forces would not enter Baghdad until the 4th Infantry Division arrived. That turned out to be false.

In other report findings:

The regime planned to restart production of weapons of mass destruction. It continued to hide scientists from U.N. inspectors right up to the time U.N. inspectors left and the war began.

A seized Dec. 15, 2002, memo, written by an Iraqi intelligence agent posing as a U.N. escort, states, "Inside Bader WMD inspection site, there are Russian and Turkish scientists. When we visited the site, they were forced to hide from inspectors' eyes."

And, Saddam continued to tell his commanders he still had such weapons. "For him, there were real dividends to be gained by letting his enemies believe he possessed WMD, whether it was true or not," the report said.

The quickly assembled air strike on one of Saddam's residences, Dora Farms, in pre-dawn March 19, 2003, never had a chance of succeeding. Saddam had not stayed there since 1995.

There was no evidence that Saddam or his top aides planned the insurgency, now in its fourth year; in fact, Saddam was sure the Americans would never advance on Baghdad.

"There were no national plans to transition to a guerrilla war in the event of military defeat," the report states.

This fact helps explain why commanders did not predict, nor plan for, the robust insurgency and al Qaeda terrorists now spreading violence.

Saddam's misguided belief that he would stay in power in 2003 was fed by the support he got from France and Russia, his top aide, Tariq Aziz, told U.S. investigators.

"France and Russia each secure millions of dollars worth of trade and service contracts in Iraq, with the implied understanding that their political posture with regard to sanctions on Iraq would be pro-Iraqi,"

Mr. Aziz said. "In addition, the French wanted sanctions lifted to safeguard their trade and service contracts in Iraq."
=================

Your comments invited on the likelihood and consequences of spies inside the US Central Command working for Moscow.

tomder55 answered on 03/26/06:

Ten riflemen makes a firing squad, right?I am very concerned that the Ruskies probably had a mole in CENTCOM .It is clear that the Russian ambassador to Iraq disclosed detailed US war plans to Saddam and his commanders.One of the documents details the composition, size, location and type of US military forces arrayed in the Gulf and Jordan including the numbers of tanks, armored vehicles, different types of aircraft, missiles, helicopters, aircraft carriers, and other forces and also their exact locations. It also described the positions of two Special Forces units. That is information they did not get from al-Jazzera.

These new disclosures prove that Russia is not an ally or friend of the United States. Russia is the last country we should count on to broker a deal with Iran.Bush should send some minor state department official to the G8 summit in St Petersburg and give his 'soul-mate' something to ponder. Or perhaps they forgot that we "accidently " bombed the Chinese Embassy in Belgrade after they repeatedly gave the Serbs intel.

A couple of other sources to consider however :
1. A Brit minister who was opposed to the war who had access to this info. and channeled it to the Russians.

2. A State Dept. or CIA career hack who silently opposed the mission
.

3.Some traitorous loyal Democrat U.S. Senator who had gone to the Middle East before the war and spilled the beans about the U.S. commitment to a military solution (hello Sen. Rockefeller....Rockefeller has already admitted that he gave Syria advance warning Of Iraq War intentions. He also is the vice chairman of the Senate Intelligence Committee.)

4.An old school General who Rummy pissed off and is now a talking head on cable news ;especially on Hardball with Chris Matthews.

5.Ramzaj


Erewhon rated this answer Above Average Answer

Question/Answer
purplewings asked on 03/24/06 - Blog responses to reporters of the Iraqi war.

http://news.aol.com/dailypulse/032306?id=20060323102509990001&ncid=NWS00010000000001

Do you trust the media or the Bush administration more when it comes to information about the war?
- How much does the media influence your views on the war?

Tell us, below.



dailypulseblog at 10:08:00 AM EST Link to this entry | Blog about this entry | Notify AOL

This entry has 500 comments: (Add your own)

I think the media is very much biased against anything the current administration does or will do. If there were a democrat president in office with the very same current world and national events, everything you see or read in the national news media would be slanted as very positive for him (president).
There will have to be some drastic changes in the national media before I will ever believe anything they say again.
Comment from bellrngr - 3/23/06 7:05 PM



It doesn't matter whether you are talking about the war in Iraq, Social Security, cleanliness of hotels, or the price of pickles, the media has no vested interest in presenting a fair, unbiased report. There's no ratings, and therefore no prime commercial time, in unbiased reporting. I'd like to believe that journalists enter the field with the altruistic goal of reporting and presenting the news fairly and accurately, but that is probably soon dashed by the demands of editors and producers whose salaries and bonuses depend on ratings shares. Sad commentary, if you ask me. But I truly believed in justice before I became a practicing lawyer. How dumb is that? I guess all "professionals" feel the same way about their professions.
Comment from beboesq - 3/23/06 7:05 PM



The media is interested in it's own personal gain. The media is biased. The media grandstands for money and ratings. The media is arrogant, self centered, ill informed, and sensational. they use sex as their marquee and they exagerate.
I have quit watching because the media is a scandall. May God have mercy on them.
Kathleen Shelton
Wichita, Kansas
Comment from resort1987 - 3/23/06 7:05 PM



The media is overwhelmed with itself in reporting all the negative aspects of our conflict in Iraq. We are helping the Iraqis get their feet back on the ground and get their new government set up. Our troops are doing a fabulous job! We should welcome them home as heros and true patriots. God bless them and God bless America. The media needs to bring the heartening news of boys and girls getting back to school in Iraq. We have done immense work in rebuilding the infrastructure of Iraqi's cities. The media always paints the most gruesome picture of Iraq and nothing about the great accomplishments there. What makes news? Shouldn't it be fair and equitably done?
Comment from myronp6464 - 3/23/06 7:05 PM



Quite frankly, I believe that most of the media should be charged in FEDERAL COURT for aiding the enemy! With these parasites clinging to your ass, it's no way to fight a war!
Comment from dpr36regal - 3/23/06 7:05 PM



As long as can remember the media has been one sided. If they always take the liberal (anti-war) point of view, how can ANYONE trust the to tell the truth. I know they have news to "SELL" but I think the American people are smart enough to determine the truth if the CORRECT facts are given.
Who knows how our present press would have reported WWII? By present standards the Nazi and Japanese was machines would have been "misunderstood freedom fighters".
Hey press, JUST BE HONEST
Comment from dd823 - 3/23/06 7:05 PM



I believe that the vast majority of the media would like to see us fail in Iraq. They are motivated mostly by hatred of the Bush administration. They would rather see us fail, simply because it would make the President look bad. This is more important to them than seeing our efforts succeed.

murraytur

Comment from mcdclc - 3/23/06 7:05 PM



War is never nice. I do believe the media only shows what will get attention.
When I travel I have meet many soliders returning home. They are glad to come home to their familys, but have said we are their for a good reason. And when the people in Iraq say Thank You. It makes them very happy knowing that they are helping people to someday be able to live like we do. And how the soliders appreciate what we have here that other places don't.
Comment from klr376 - 3/23/06 7:05 PM



There is an old saying among journalist, "If it bleeds, it leads!" and that was never more true than when it comes to the media's reporting of the war in Iraq. I have spoken with a large number of veterans who have returned from Iraq, who speak of all the progress that has been made, schools built, roads built or rebuilt, and you never hear a word about it in the press. You don't see stories about people having clean water and electricity for the first time.

I read reports coming in from Iraq reporting how well our service men and women are doing and how high morale is, and I see pictures of Iraqi civilians thanking soldiers and displaying signs thanking our President and America for helping them. Why don't we see that on the news? Journalists seem to excel in selling out our men on the ground and our President at home. They did it in Vietnam and they are trying to do again in this war. Thank God for the internet because I've lost all faith in the media.

John E. Touchton Sr. M.Ed., Ph.D.
Former Captain, U.S.A.R. , Handicapped Veteran
Comment from johntouchton - 3/23/06 7:04 PM



A note of sanity in what it means for the media to cover a war. A war is sanctioned violence. A war involves armies, not just the threat but the use of deadly force. A war is not giving candy to children or paying a shopkeeper $200 for destroying his means of livlihood. It's a war folks, not a picture show. Actually, the media is not being negative enough if negative means showing what happens in a real war. If they were really doing their job, the public would be totally traumatized. It is not the task of the press to sanitize a war, any war.

Peter
Comment from steager - 3/23/06 7:04 PM



The media pushes the 1st ammendment toooo far. They print what looks good to them whether it's right, wrong or in the middle. I feel the media is a big bag of wind and should report and not make up what they feel is their opinion. They should let the opinion reporting to the EXPERTS. MOST of the media people aren't anywhere close to being an expert.
One of the comments was that the media is leftist. They're just on the bandwagon against President Bush, because there are people that listen to them. The more people react to their comments the more they put out out to read or listen. That fuels the fire. That's a shame. I think the radicals that protest are not doing all their homework and are believeing everything the media puts out. (Maybe they aren't smart enough to do their own homework)
It's like the Eagles hit "Dirty Laundry", the media reports with a gleam in their eye. I check out more news sources than the American media. After Rather had 2 minutes of silence on national TV for the death of Jim Brady when Pres Reagan was shot, I lost ALL confidence in the American media. It just race to see who can get the "ratings". Ratings and polls are like sports announcer's predictions of how a game will come out. They have a 50/50 chance of getting it right or predicting the future.
Comment from osudave99 - 3/23/06 7:04 PM



The media just reports the bad stuff, why are they so negative, we need positivness spoken about too. Like when you see the soldiers with the children, those soldiers don't want to be killing peole and risk being killed too, but you don't see the compassion the soldiers have for the Iraq's you ar only told the bad and negative stuff. The media needs to wake up and smell the roses, and believe they are out ther to smell and talkl about.
Comment from ukusa65 - 3/23/06 7:04 PM



The media complains that 60 of it's personnel have been killed and many other kidnapped while reporting 'on the street.' So they have chosen to hide and show the daily 'car bombing' instead. Does anyone think that perhaps the terrorists are killing the press for exactly that reason? Isn't better for the terrorist cause to have the press muzzled and running scared? Instead of showing what is really going on throughout the country, the good and the bad, the press runs nightly footage of whatever car bomb went off and how many were killed. Looks like the big bad press ended up looking through the peep-hole of their door instead of getting out on the street. If you expect all the other agencies of the government to provide 'transparent' access to the power of the press. Then perhaps you should admit what has happened in Iraq and quit pretending the only thing that happens everday in the whole country is another car bomb or IED. Why is there nothing about the people in the new government? Why is there nothing about the schools, the repair to infa-structure and commerce? Stop being the shill of the terrorists and get out and do your job, or admit you're not.
Comment from lscbc - 3/23/06 7:04 PM



Coverage may be OK, but the actual reporting by the media is obviously filtered, slanted, and biased; that is to say, awful.

I trust Bush more than the media. He may at times have incorrect information, but he does not try to deceive, which can't be said about the media. The "major" media are laughable.

If the club - NBC,CBS,ABC,PBS, NY Times, Washington Post, LA Times, and CNN, and their leader BBC - are against what's transpiring, I know we are doing the right thing. In short, the Media don't influence me much.
Comment from jeanlaw - 3/23/06 7:04 PM



I think the media is doing as fair a job as they can, reporting on a war that never should have happened in the first place--I'm neither a Democrat, or liberal, and I agree that Saddam Hussein is evil, and should have been driven out--but I feel that over 2,000 young American lives lost, and BILLIONS spent on the war will one day show this to be one of the worst mistakes ever made by a U.S. Pres.--
Mr. Bush took some unreliable information, jumped prematurely into a war, when he should have explored other ways to rid the world of Hussein--America CANNOT be the savior of the entire world--our FIRST obligation is to the United States--I pray God will be with our troops, and bring ALL of them home soon.
Roy Parks e-mail beecherhillart@aol.com
Comment from beecherhillart - 3/23/06 7:04 PM



I concider myself a news junky. However, I have to critisize the media not only the war, but in all news in general. Part of the problem in this country is that when something big happens or down to general crime. The media always has to sugar coat the story, afraid of stepping on someones toes or insulting someones race or culture. Then we have the news talk shows. Again, a handful of professional people sitting around talking about the problem. Is that really fixing the problem?
This country is based of freedom of speech. I suggest the media get with it, and start saying things like it is.

David H.
Phila.
Comment from dwh12562 - 3/23/06 7:04 PM



The media shows us what they want, you can never fully understand what it's like over there and what the soldiers endure every day regardless of politics. I spent 10 months in Baghdad until injured. Support the troops!
Comment from harrylampus - 3/23/06 7:04 PM



Since Bush is so unpopular, the reporters take advantage by reporting only negatives. I know positives exist in Iraq because I've seen photos of the new parks, roads and school buildings, and the children hugging the soldiers while their parents smile. Otherwise I'd think America had taken it's troops into pure hell without a reason.

I'm really tired of biased, politcally based reporting that negates any good achieved by this administration, and in fact this country.

I'm glad we have a president who's shown strength to the Islam Fundamentalists. Otherwise we'd probably all be dead by now. War is always hell, but sometimes the only answer to stop a life of hell for others.
Comment from lororow921 - 3/23/06 7:03 PM



I got so sick of negative only reporting from the local news. NBC being the very worst. I now only watch the fox station, where I can get balanced reporting. It's a shame everyone doesn't have cable or satelite, so they could see fair reporting,and not be kept ignorant by the Bush hating main stream.
Comment from peacefrogzzz - 3/23/06 7:03 PM



I feel that the journalist have slanted the news. You can watch CBS, NBC, ABC, CNN and you can tell all most all are Democrats. Listeing newspeople bad mouth Bush on all channels. I never voted for him, but...... every channel sounds degradeing of our President. All I can say is God Bless America and all the other countries and bring peace. KYCRAFT

Comment from kycraft - 3/23/06 7:03 PM



Imagine if these reporters were covering the AMERICAN REVOLUTION ??

"things are spiraling out of control"
"there's no consenus"
"the british are coming"
"general washington has failed again"
"washington has no army"
"washington's troops deserting"
"congress split into factions"
"consitution not in sight"
ETC.
Comment from mrjoeamato - 3/23/06 7:03 PM



We only get fair and balanced news from internet blogs such as Lucianne.com and Townhall.com and from the FOX news channel. Radio talk shows are a mixed bag, but the network news is pathetic. They are damned and determined to find some way to help impeach President Bush because their favorite, Bill Clinton, was impeached. It is the don't get mad, get even mind set.
No good news on the networks, because if it bleeds, it leads.
Comment from lubyagain - 3/23/06 7:03 PM



Ah, blame it on the media, bombs going off, soldiers fighting insurgents, children dying and being dismissed as collateral damage, yeah, must be actors and movies scenes the media shows us.
Bush would have us believe that before the war Iraq had no commerce, no schools, no oil production, no electricity and no children playing in the streets, which was certainly not the case.
Blaming it on the media is just a last ditch to hold onto less than erudite supporters of the war and Bush.
Simply put, no matter what adancements have been made, if any, nothing matters unless civility and sanity

What would your response be?

tomder55 answered on 03/24/06:

I note that recently on ABC Nightly News they are becoming sensitive to the charge that they are not giving a complete picture of the realities in Iraq. They of course are trying to justify their reporting decisions but have admitted that the situation on the ground is more complex than they have portrayed it .

What I have never heard from the critics is this :What price was being paid by the continued containment policy ? Bottling up Saddam Hussein required parking most of the carrier fleet in and around the Persian Gulf and keeping large ground and air forces on his borders. How long was it likely to last ? We had already been playing ;along with Britain the sole enforcers of UN sanctions for over a decade .

Revelations about the extent of the Oil for Food abuses ($300 billion worth of fraud and payouts) prove that containment was and ultimately would be ineffective. Back then if everyone remembers the US was being criticized for perpetrating a genocide that systematically starved to death a million Iraqi kids - or two million, according to which "humanitarian" agency and MSM reporter you believed.The interesting thing about some of the civilain death figures attributed by the MSM to America is that they are just about the same before and after the invasion. Before we were starving them to death ;now we are responsible I guess for every terrorist blast that results in civilian deaths .

The options given that the sanction program was failing was to either invade if Saddam refused to comply with the myriad of sanctions and cease fire provisions or to leave . It is clear fron recent document releases that Saddam had a deception policy active and in reality had harbored his WMD and nuclear programs and the means to quick start them once he had defeated the sanction regime.

When America decided after September 11 that Saddam constituted an imminent danger it spent six months trying to get the UN to act under Resolution 1441. That delay imposed enormous costs, the greatest of which was that it allowed Saddam to get ready for the most announced invasion in history. Six months to prepare the "insurgency " and to act in concert with Russian Speztnaz to remove ,hide and scrub the evidence of his WMD program from the theater .According to ABC News recently released documents suggest that the Russian ambassador leaked the US war plans to Saddam Hussein.

But that is history . The history has also revealed that the press instead of being unbiased reporters made special accomodations with Saddam Hussien while he was in power in exchange for access.Peter Arnett's reports were so sympathetic to Saddam Hussein's Iraq that he could have replaced Baghdad Bob . As the US was about to enter Baghdad Arnett reported to the world that the US war plan has failed.The scandal of some Western media's silence about the atrocities of Saddam Hussein's regime, of course, is old news.CNN's news chief, Eason Jordan, so much as admitted the cover-up in the New York Times in an article entitled 'The News We Kept to Ourselves'. That alone is enough to indict not just CNN but the bulk of the MSM.






purplewings rated this answer Excellent or Above Average Answer

Question/Answer
HerrAirhorn asked on 03/24/06 - SERENITY NOW!

"Did you catch yesterday's news conference (3/21/06), the one where Little George casually passed on responsibility for getting US troops out of Iraq to "future presidents" -- the same presidents to whom he is passing on responsibility for his monumental deficit?

Of course, he never intended to get the troops out of Iraq at all.
The whole point, as I argued long ago, was to create permanent bases in the broader Middle East as a bulwark against what far-sighted US imperialists have always understood to be the real threat -- not Islamo-fascist terrorists, but an oil-hungry China which will be in a position to challenge the US for global supremacy on every front in the coming decades. That is what this war is really about, and always has been.

But the heck with facts. Let's move on to something interesting. Personality.

Did you notice Little George's mood? He seems finally to be approaching, insofar as his temperament allows, the "serenity" he was pretending to enjoy in the lead-up to the war. Of course, what passes for serenity in Little George's prosaic soul is not much more than indifference, a shrugging off of consequences -- a gesture that feels perfectly justified to him, by the way, because he is absolutely certain about the one thing he relies on: he has followed his heart.

He has prayed, and he has followed his heart.

So the rest is up to God.

That's what that news conference was really about. Little George has been praying again and his prayers have been answered again. The comfort of the Lord has descended upon him. He has been absolved. That's what accounts for his new mood. He is indifferent to his poll numbers. He is indifferent to the fate of his party in the coming elections. He is indifferent to the carnage in Iraq. He knows things will come right in the end. He has fallen back on his faith.

Since the rest is up to God, Little George is now feeling like his work is done. He has made all the decisions that matter, and he will not turn back. That would be weak. But he is strong. There's 3 years left in his presidency, but mentally he's already turned over his responsibilities. You could just sense the relief in his manner when he mentioned "future presidents." He's already outta there, heading home to Crawford, back to his pick-up trucks and mountain bikes, his special pillow, and Laura in the morning".
THOMAS DE ZENGOTITA Blogger

tomder55 answered on 03/24/06:

the first paragraph exposes the author as a conspiracy nut-job.

Bush did an excellent job in his press conference ; among other things, he vigorously and effectively defended our progress in Iraq and the accomplishments of his administration .

What, exactly, did Bush say that made his statement about troop levels in the future the most newsworthy item to come out of the press conference?

[M]y question is, one, is there a point at which having the American forces in Iraq becomes more a part of the problem than a part of the solution? Can you say that you will not keep American troops in there is they're caught in a crossfire and a civil war? And can you say to the American people -- assure them that there will come a day when there will be no more American forces in Iraq?

BUSH: The decisions about our troop levels will be made by General Casey and the commanders on the ground. They're the ones who can best judge whether or not the presence of coalition troops create more of a problem than a solution -- than be a part of the solution.

***

QUESTION: It was: Will there come a day -- and I'm not asking you when; I'm not asking for a timetable -- will there come a day when there will be no more American forces in Iraq?

BUSH: That, of course, is an objective. And that will be decided by future presidents and future governments of Iraq.

QUESTION: So it won't happen on your watch?

BUSH: You mean a complete withdrawal?

That's a timetable.

I can only tell you that I will make decisions on force levels based upon what the commanders on the ground say.



What we mainly have here is Bush's oft-stated refusal to make decisions about troop levels based on politically-inspired timetables. No news there. Beyond that, the likelihood that some American troops may be in Iraq as of 2009--the question was framed in terms of "no more American forces"--is hardly a news flash, either. We still have troops in Germany, more than 60 years after the end of World War II. We still have troops in the Balkans, as well. Neither of these deployments is controversial.

Iraq may well be seen as a desirable place to station troops for a considerable length of time. (I happen to think so based on it's strategic location next to a major adversary ...Iran)Whether such a longer-term deployment is controversial will depend on how many troops remain in Iraq and, more important, what they are doing there. If the fighting is essentially over and soldiers are not being lost, no one will care about the presence of troops in Iraq, any more than they object to the presence of troops in Germany.

In truth, this was one of the less newsworthy exchanges in the press conference. But, because it suited most journalists' yearning for a quagmire, it became the lead in newspapers and the left wing bloggers all across America.





fredg rated this answer Excellent or Above Average Answer
HerrAirhorn rated this answer Excellent or Above Average Answer
Itsdb rated this answer Excellent or Above Average Answer

Question/Answer
Mathatmacoat asked on 03/24/06 - The not so banana republic.

After being well larried in the last week, Australia can no longer be called a banana republic, since 90% of it's banana production is out of action. Before any of you enterprising yanks think it's just a case of shipping a few bananas over to help out you aussie mates, no imported bananas. We will just have do do without out favourite yellow fruit

come and see what we are missing
http://www.eco-banana.com.au/

The question must be asked.
Was Cyclone Larry a diabolic plot to detroy Australia's eco friendly banana industry?

tomder55 answered on 03/24/06:

I'm so confused about Aussie produce. Is it Australian wheat of Austrian Eggs ? Is it Iraqi oil or Italian wax ? Does BHP think that Aussie is a member of the 'League of Nations' and that the Persian Gulf war was fought between Rome and Carthage ?

Word to John Prescott ;I am not sure the transactions comply with the League of Nations provisions regarding the embargo of selling Austrian Wheat to Italy in exchange for wax. And all the tonnes of eggs sold to Italy since the Punic Wars probably violates the League's sanctions .

Gosh ;first there is no place to sell the wheat and yes; we have no bananas . Good thing Outback Steakhouse is prospering .

Seriously ; the cyclone was one of the under-reported stories of the last week .Thankfully the human loss was low .Evidently relief efforts are coming under a bit of criticism .

I wish eveyone there well and hope the Howard gvt. survives . He has been a steady friend and allie to the US .

Mathatmacoat rated this answer Excellent or Above Average Answer

Question/Answer
Erewhon asked on 03/23/06 - Will the US have to liberate Afghanistan again?


You might have heard the disturbing news that an Afghan Christian is on trial for his life for apostasy from Islam. His family turned him in (ratted him out) during a custody dispute, and the case is being handled by a religious judge who leans heavily towards Sahariaism.

The US is pressuring the Afghan goverment to maintain the right of religion that is part of their constitution, but Afghan politicians wring their hands and say it is part of the judicial process and they can't interfere.

There is a move to have the man, who converted sixteen years ago, declared insane, thus sparing him from death by the sword of justice.

Would you support an American army of liberation attacking Afghanistan so that democracy and freedom can be established?





tomder55 answered on 03/23/06:

democracy has been established there . they chose to make Sharia the guiding principle of their legal system . Bush is right to pressure them . Tell me ;do you think the Taliban was better ? Do you think perhaps that there are more freedoms now than when they were rounding up apostates and executing them in the soccer stadium ? Do you really think that when we tout democracy that we will be able to change mediaval societies overnight ?

Erewhon rated this answer Excellent or Above Average Answer

Question/Answer
ETWolverine asked on 03/23/06 - For BeelzeBUSH

George,

And now for some facts:

From a poll of military servicemen by the Military Times.

Published:
January 3, 2006

Morale
1) How satisfied or dissatisfied are you with your job?

Completely satisfied 36%
Somewhat satisfied 49%
Somewhat dissatisfied 12%
Completely dissatisfied 3%
No opinion/no answer 0%

2) How satisfied are you with your family life?

Very satisfied 55%
Somewhat satisfied 32%
Somewhat dissatisfied 7%
Very dissatisfied 4%
No opinion/no answer 2%

3) To what extent, if any, do you experience conflict between your work life and your personal life?

A great deal 7%
Quite a lot 32%
Not very much 51%
None at all 9%
Don't know/no answer 1%

4) How satisfied are you with the amount of time you spend with your family?

Very satisfied 19%
Somewhat satisfied 42%
Somewhat dissatisfied 25%
Very dissatisfied 12%
Don't know/no answer 2%

5) At this time, would you say you are worried about your family's finances, or not?

Very worried 5%
Somewhat worried 29%
Not too worried 38%
Not at all worried 26%
No opinion/no answer 1%

6) Would you recommend a military career to others?

Yes 82%
No 13%
No opinion/no answer 4%

7) If you had a son or daughter who was planning to enter the military, would you support that step or would you suggest a different occupation?

Support that step 73%
Suggest different occupation 23%
No opinion/no answer 4%

8) If you had to decide today, would you re-enlist or -- if an officer -- extend your commitment?

Yes 70%
No 19%
Don't know/no answer 11%

9) If you answered YES to No. 8, check the THREE most important reasons why.

Educational opportunities 22%
Patriotism 57%
Pay 26%
Pension 46%
Job security 43%
Tavel, adventure 17%
Health care for my family and me 40%
Career satisfaction 36%
Wars in Iraq/Afghanistan 9%
None of the above 1%

10) If you answered NO to No. 8, check the THREE most important reasons why.

Educational opportunities 11%
Patriotism 11%
Pay 30%
Pension 40%
Job security 48%
Tavel, adventure 40%
Health care for my family and me 9%
Career satisfaction 29%
Wars in Iraq/Afghanistan 47%
None of the above 6%

11) I would rate my military housing as:

Excellent 14%
Satisfactory 45%
Poor 15%
Very Poor 10%
No opinion/no answer 16%

12) I would rate military pay and allowances as:

Excellent 13%
Satisfactory 66%
Poor 17%
Very Poor 4%
No opinion/no answer 0%

13) I would rate military health care as:

Excellent 18%
Satisfactory 60%
Poor 15%
Very Poor 6%
No opinion/no answer 1%

14) Overall, officers in the military are:

All Military
respondents respondents
Excellent 23% 9%
Satisfactory 62% 66%
Poor 10% 16%
Very Poor 3% 5%
No opinion/no answer 2% 3%

15) Overall, enlisted leaders in the military are:

All Officer
respondents respondents
Excellent 31% 41%
Satisfactory 60% 54%
Poor 6% 3%
Very Poor 2% 1%
No opinion/no answer 1% 1%

16) Overall, my military quality of life is:

Excellent 22%
Satisfactory 68%
Poor 8%
Very Poor 1%
No opinion/no answer 14%

17) I am well trained for my military job.

Strongly agree 37%
Agree 54%
Disagree 7%
Strongly disagree 1%
No opinion/no answer 1%

18) People in the military today are supplied with the best possible weapons and equipment.

Strongly agree 11%
Agree 47%
Disagree 30%
Strongly disagree 9%
No opinion/no answer 3%

19) The civilian leadership of the Department of Defense has my best interests at heart.

Strongly agree 5%
Agree 35%
Disagree 33%
Strongly disagree 17%
No opinion/no answer 10%

20) President George W. Bush has my best interests at heart.

Strongly agree 19%
Agree 39%
Disagree 18%
Strongly disagree 11%
No opinion/no answer 12%

21) The senior military leadership has my best interests at heart.

Strongly agree 16%
Agree 48%
Disagree 20%
Strongly disagree 8%
No opinion/no answer 7%

22) Congress has my best interests at heart.

Strongly agree 2%
Agree 29%
Disagree 40%
Strongly disagree 17%
No opinion/no answer 11%

23) Today's service members are better than they've ever been.

Strongly agree 21%
Agree 46%
Disagree 22%
Strongly disagree 3%
No opinion/no answer 9%

24) Today's military is stretched too thin to be effective.

Strongly agree 26%
Agree 38%
Disagree 27%
Strongly disagree 3%
No opinion/no answer 7%


So... what do we have here? 57% of those who join the military do so out of PATRIOTISM, not because of the educational or training benefits. 85% are satisfied with their jobs. 90% are satisfied with military life. 91% consider themselves well-trained for their jobs. 58% feel that they are supplied with the best equipment available. 58% feel that the President has their best interests at heart. 64% feel that the military leadership has their best interests at heart. (Only 31% feel that CONGRESS has their best interests at heart... which doesn't say very many good things about Congress' support for the military.) 64% say that they are not worried about their families finances, in other words they are satisfied with their pay and benefits. 82% would recommend a military career to others, and 73% would support their children if they decided to join the military.

These are not the numbers of a military who's morale is low.

And now for the opinions of military servicement on Iraq and Afghanistan.

1) Are you on active duty?

NOTE: Only active-duty responses were counted in remaining results.

Yes 85%
No 15%

2) Service branch:

Army 48%
Navy 20%
Air Force 21%
Marine Corps. 10%
Coast Guard 1%
No response 1%

3) How many times have you deployed to Iraq?

Once 31%
Twice 11%
Three times 2%
More than three times 0%
Never/no response 53%

4) How many times have you deployed to Afghanistan?

Once 11%
Twice 3%
Three times 0%
More than three times 1%
Never/no response 85%

5) In total, I have deployed in support of the war in Afghanistan and/or Iraq for:

Less than 2 months 3%
3-6 months 19%
7-12 months 22%
13-18 months 10%
19 or more months 7%
Haven't deployed/no response 39%

6) Should the U.S. have gone to war in Iraq?

Yes 56%
No 26%
No opinion/no answer 7%
Decline to answer/no answer 11%

7) Regardless of whether you think the U.S. should have gone to war, how likely is the U.S. to succeed?

Very likely to succeed 31%
Somewhat likely to succeed 42%
Not very likely to succeed 17%
Not at all likely to succeed 3%
No opinion/no answer 6%

8) How soon do you think the Iraqi military will be ready to replace large numbers of American troops?

Less than a year 2%
1-2 years 27%
3-5 years 40%
5-10 years 17%
More than 10 years 7%
No opinion/no answer 6%

9) How long do you think the U.S. will need to stay in Iraq to reach its goals?

Less than a year 2%
1-2 years 11%
3-5 years 35%
5-10 years 30%
More than 10 years 15%
No opinion/no answer 6%

10) Do you approve or disapprove of the way George W. Bush is handling the situation with Iraq?

Approve 54%
Disapprove 25%
No opinion 9%
Decline to answer 12%

To one of your points: only 2% of respondants have been stations in Iraq or Afghanistan more than twice. This is hardly a widespread phenomenon. Next, 54% approve of Bush's handling of the war in Iraq. 73% believe that we will be successful in Iraq. 75% are of the opinion that it will take at least 3 more years before we are ready to leave Iraq, and 64% believe that it will take at least 3 years for the Iraqi military to be able to take over from US troops... and they are prepared for that eventuality.

Again, these numbers do not reflect a military who's morale is in the dumps or is flagging.

Here are a few other points from the poll that I thought might be of interest.

Some people think that by criticizing the military, news organizations weaken the country's defenses. Others think that such criticism helps keep our country militarily prepared. Which position is closest to your opinion?

Weakens defense 62%
Keeps nation prepared 22%
Don't know/no answer 16%

In general, do you think news organizations get the facts straight, or do you thnk their stories and reports are often inaccurate?

Get facts straight 11%
Stories often inaccurate 81%
Don't know/no answer 7%

How do you think each of these groups views the military?

Civilians Media Politicians
Very favorable 24% 5% 10%
Somewhat favorable 58% 33% 53%
Somewhat unfavorable 14% 38% 29%
Very unfavorable 2% 24% 6%
No answer 1% 1% 2%

Boy, those military gues really hate the mainstream media!!!

Another interesting point: virtually all military personnel are between the ages of 21 and 58. Very few, if any, are younger than 21. So your argument that the military servicement are just kids who are being folled into the military is also untrue.

The information above can be found at http://www.militarycity.com/polls/2005_main2.php

If your brother is having trouble keeping morale up, perhaps its because he's not as good at his job as he ought to be. Or perhaps he shares your views and is bringing down their morale himself. But the statistical evidence above denies the credence of your anecdotal evidence. The morale of our military remains quite strong, and they support Bush's decision to go to war and his handling of the war.

Elliot

tomder55 answered on 03/23/06:

I can only give ancedotal evidence .

My brother served proudly in the Marines. My cousin served in both Desert Storm and OIF for two tours in an Army Armor Div. When Desert Storm finished and Saddam was still in power he told me "we will go back there one day ".

Their views are reflected in the polling data sited . My cousin in particular chose a military career and he had many options on the table .My brother was disabled but would go back if he could .

ETWolverine rated this answer Excellent or Above Average Answer

Question/Answer
Erewhon asked on 03/22/06 - Right or Wrong?

Bush still sees no reason to apologise
By Rupert Cornwell in Washington
Published: 20 March 2006

If anyone was looking for even the slightest hint of second thoughts from those led the US into Iraq, they would have been sorely disappointed on the third anniversary of a war that is eating into America's soul and that may well reshape its political landscape.

More sacrifice would be required, but "our goal is nothing less than complete victory", President George Bush declared in his weekly radio address yesterday.

Ignore the doom-mongering, Dick Cheney urged his countrymen on CBS's Face the Nation programme. This was no civil war; rather the insurgents had reached "a stage of desperation". On both the security and political fronts, Iraq was showing "major progress".

Writing in The Washington Post, Donald Rumsfeld, the Defence Secretary - blamed by many for the absence of post-invasion planning - was equally unrepentant. The big picture would be determined by history, "not by daily headlines, website blogs, or the latest sensational attack", Mr Rumsfeld declared. To retreat now would be "the modern equivalent of handing post-war Germany back to the Nazis, or of asking the former Communist states of eastern Europe to return to Soviet domination because the West did not have the patience to see through the job of turning them into free countries".

The plain fact, however, is that back in March 2003, almost no Bush administration policy-maker could even imagine that yesterday the country would be in agonising debate over a conflict three years old with no end in sight - in an Iraq that even the pro-American former prime minister Iyad Allawi said was in the midst of a civil war.

When Mr Bush triumphantly proclaimed an end to the war in May 2003 from the deck of the aircraft carrier Abraham Lincoln, the Pentagon's expectation was that by the end of that year no more than 30,000 US troops would be deployed in Iraq. Today 130,000 are still there - and General George Casey, the senior US commander in the country, warned yesterday that he saw "a couple of more years of this". The war has been a drain on American blood, treasure and morale. As of yesterday, at least 2,311 US servicemen had died there, and more than 13,000 had been wounded. By the end of 2006, the conflict will have cost $320bn (183bn).

The psychological cost is unquantifiable, but enormous. For a minority the war has brought bereavement and personal sadness. Half of all Americans know someone who has served in Iraq; some 10 per cent of them had a relative or friend who had been killed or wounded there, according to a poll by USA Today.

Mr Bush's place in history will be determined by his decision to invade. Back in March 2003, his approval ratings stood at 70 per cent. Now they have dropped to less than 40 per cent. Two-thirds of the public believes the country is "on the wrong track". Iraq sweeps every other issue off the table.

This November's mid-term elections meanwhile may well turn into a referendum on Iraq, and the Republican Party may lose control of either the House of Representatives or the Senate, conceivably both.

Even among the Republican faithful, support for Mr Bush is starting to erode. "If you demand complete victory, you'll never leave," Senator Chuck Hagel, the Nebraska Republican who is mulling a 2008 White House run, said yesterday
.

The war, he declared, was helping to bankrupt the country. "And if you ask, are we better off, is the Middle East more stable than three years ago, the answer is, 'Absolutely not'."

If anyone was looking for even the slightest hint of second thoughts from those led the US into Iraq, they would have been sorely disappointed on the third anniversary of a war that is eating into America's soul and that may well reshape its political landscape.

More sacrifice would be required, but "our goal is nothing less than complete victory", President George Bush declared in his weekly radio address yesterday.

Ignore the doom-mongering, Dick Cheney urged his countrymen on CBS's Face the Nation programme. This was no civil war; rather the insurgents had reached "a stage of desperation". On both the security and political fronts, Iraq was showing "major progress".

Writing in The Washington Post, Donald Rumsfeld, the Defence Secretary - blamed by many for the absence of post-invasion planning - was equally unrepentant. The big picture would be determined by history, "not by daily headlines, website blogs, or the latest sensational attack", Mr Rumsfeld declared. To retreat now would be "the modern equivalent of handing post-war Germany back to the Nazis, or of asking the former Communist states of eastern Europe to return to Soviet domination because the West did not have the patience to see through the job of turning them into free countries".

The plain fact, however, is that back in March 2003, almost no Bush administration policy-maker could even imagine that yesterday the country would be in agonising debate over a conflict three years old with no end in sight - in an Iraq that even the pro-American former prime minister Iyad Allawi said was in the midst of a civil war.

When Mr Bush triumphantly proclaimed an end to the war in May 2003 from the deck of the aircraft carrier Abraham Lincoln, the Pentagon's expectation was that by the end of that year no more than 30,000 US troops would be deployed in Iraq. Today 130,000 are still there - and General George Casey, the senior US commander in the country, warned yesterday that he saw "a couple of more years of this". The war has been a drain on American blood, treasure and morale. As of yesterday, at least 2,311 US servicemen had died there, and more than 13,000 had been wounded. By the end of 2006, the conflict will have cost $320bn (183bn).

The psychological cost is unquantifiable, but enormous. For a minority the war has brought bereavement and personal sadness. Half of all Americans know someone who has served in Iraq; some 10 per cent of them had a relative or friend who had been killed or wounded there, according to a poll by USA Today.

Mr Bush's place in history will be determined by his decision to invade. Back in March 2003, his approval ratings stood at 70 per cent. Now they have dropped to less than 40 per cent. Two-thirds of the public believes the country is "on the wrong track". Iraq sweeps every other issue off the table.

This November's mid-term elections meanwhile may well turn into a referendum on Iraq, and the Republican Party may lose control of either the House of Representatives or the Senate, conceivably both.

Even among the Republican faithful, support for Mr Bush is starting to erode. "If you demand complete victory, you'll never leave," Senator Chuck Hagel, the Nebraska Republican who is mulling a 2008 White House run, said yesterday.

The war, he declared, was helping to bankrupt the country. "And if you ask, are we better off, is the Middle East more stable than three years ago, the answer is, 'Absolutely not'."


Right or wrong?

tomder55 answered on 03/22/06:

Michael Kelly ;editor of 'Atlantic Monthy 'was the first journalists to die in Operation Iraqi Freedom .In one of his last columns he wrote :

"Tyranny truly is a horror: an immense, endlessly bloody, endlessly painful, endlessly varied, endless crime against not humanity in the abstract but a lot of humans in the flesh. It is, as Orwell wrote, a jackboot forever stomping on a human face. I understand why some dislike the idea, and fear the ramifications of, America as a liberator. But I do not understand why they do not see that anything is better than life with your face under the boot. And that any rescue of a people under the boot (be they Afghan, Kuwaiti or Iraqi) is something to be desired. Even if the rescue is less than perfectly realized. Even if the rescuer is a great, overmuscled, bossy, selfish oaf. Or would you, for yourself, choose the boot?"

That is my comment but some point by point remarks are in order

Rummy is right ;fickle polls will not determine anything . History will be decide Bush's place .

Allawi ;the former designated interm PM under the reign of Viceroy Bremer is a thug who personally conducted summary executions to send a message to his opponents . He is completely discredited in my book.

It has been explained many times why Bush did the "mission accomplished " stunt . His declaration of major combat being complete was done at the request of Gen. Franks who had been led by some of our feckless Euro-allies that there would be non-military aid that would be forthcoming once combat operations were over.

More likely the mid term elections will be what they are almost always about ....the economy .The Republicans if they run on it should do fine.

Chuck Hagel is a Senator from Nebraska ;therein lies his constituency . Bush is President of the whole country . enough said .It is easy for Hagel to stick his finger in the wind and then heckle from the peanut gallery . God help us if he were in a position to really lead.

Erewhon rated this answer Excellent or Above Average Answer
Itsdb rated this answer Excellent or Above Average Answer

Question/Answer
JBodine asked on 03/21/06 - Sadri and the CIA

I was watching the news last night, when I should've been working on my Biblical Interpretation project. I saw that Saddam's former Foreign Ministry (?) guy, named Sadri (or something like that), may have been on CIA's payroll up until the invasion.

IF so, good move by Spook Central.

In any event, his reports on Saddams NBC weapons program differed greatly from CIA estimates. News reported that BOTH were wrong.

What are your thoughts on who's closer to the truth? Sadri? The CIA? Neither?

What does that say about SH's craftiness that not even one of his top cabinet officials knew for sure what weapons he had?

DK

tomder55 answered on 03/21/06:

The good news is that Bush and co. have decided that since leaks happen ,then they preemptively leaked.. or in other words released the documents that the coalition has captured in their raw state for anyone to interpret and analyse. This along with the revelation that Saddam... like Nixon was a tape freak... and there hours of tapes yet to vet. Already these two sources are yielding some very intersting intel . Stuff that knocks the anti-war argument out of water .

here are some choice cuts :

ISGQ-2003-M0004932 page 6

This meeting with Saddam is probably in 2001 based on the conversation about U.N. inspections. The briefer tells Saddam that three scientists arrested in Germany are a problem.
We still have two issues Sir (Saddam-RR). Very simple. What the doctor said about the experts. There held in Germany. They have detailed knowledge of our weaponry. So we should go and give the information that they gave already.

So three scientist are arrested in Germany and the briefer recomends that they come clean with what those men knew. In 2001 or close to it, Iraq was still keeping secrets about its nuclear program! After ten years of inspections they were hiding WMD information.

ISGQ-2003-M0004666 page 13 Saddam admits to inaccurate WMD report,and claims he cant admit to U.N. because they were used during WMD attack of Iran.

So in all your programs (evidence-RR) that you present in Chemical (UN chemical inspection team-RR) there still will be a gap, and whenever he (Rolf Ekeus-UN inspector-RR) wants to raise it..between the imported data (declaration of how much chemical precursors Iraq imported-RR) and the weapons produced and the destroyed, there is going to be a gap a number of weapons used in Iran you guys didnt cover.

On another tape Hussein Kamel, Saddam's son-in-law and the man who was in charge of Iraq's weapons of mass destruction efforts can be heard on the tapes, speaking openly about hiding information from the U.N.

"We did not reveal all that we have," ..."Not the type of weapons, not the volume of the materials we imported, not the volume of the production we told them about, not the volume of use. None of this was correct."

Holding anything back was a violation of all resolutions back to UNSC#687, and a valid reason to terminate the cease-fire that ended the Gulf War.


From Tape ISGC-2003-M0003997 unidentified male speaks to Saddam :

Sir, the groups of missiles whose equipment was destroyed, the warheads were removed on cattle trucks that were at the military industrial facility and at the National Communications
Also, we would like to restart our activity in reforming some of the material that was destroyed, such as the advanced technology equipment. We also removed our active warhead groups from Assad Babel [Lion of Babylon]


After the first Gulf War, Saddam apparently was destroying the bulkier missiles and keeping the warheads

ISGC-2003-M0003997
Saddam requested a briefing on the rebuilding of the Muthenna Chemical Weapons Establishment after the Gulf War. unidentified male :

.Right now, we have a very large building campaign in a number of main sites; three support sites, and the main location [indistinct] the building campaign despite the annoyance presented by the inspection teams.

ISGC-2003-M0003997

Saddam Hussein :

Before the hostility, they had information and deductions that we were working in the direction we described, a dangerous direction. Now, we can confirm for them this false direction. I mean, according to the explanation. That is, there was a group of these guys and those guys, but this all ended, and with that, the thing they will confirm is our cover story.

Unidentified Male

We must study how we can package this cover story so that we can have a program that won't cause anxiety in the future, and not work in a program that will cause anxiety again.

and the best for last ; proof that Saddam continued his Nuclear Weapons research into this decade (tape was recorded in 2000).Tape ISGQ-2003-M0007379 ;briefings by scientists from the Military Industrial Commission and the Iraqi Atomic Energy Agency about a plasma uranium enrichment nuclear power program :

Dr. Thamir Ma'aman Mawdud

Iraq's National Laboratory for plasma started in the eighties, sir. as you know. We started with the sources of plasma. In 1981 we started to create sources of plasma, which were used in the Iraqi nuclear program. The source of plasma that we created, started in . . the first start, through research and development, then simple test production, the first batches, then tests done through mathematical systems, then the production we achieved in the advanced stages at the end of the Nineties...
...Activity hasn't died in plasma because it is allowed in some of the tests which we use. Then, sir, according to what we have done in the Iraqi National Laboratory in building plasma activity, we have a very large industrial base. We have built a factory to produce plasma systems. We have built a number of factories with Military Industry, who produced specialized factories for us during this time...


The plasma enrichment program was so well-protected by the Iraqi regime that U.N. arms inspectors had never discovered it. The plasma process got a brief mention in the 2004 Duelfer Report but only as a legacy program that the Iraqis had presumably abandoned in the late 1980s.

The CIA had a good read on Iraq's WMD program .George Tenet described the case as 'slam dunk'. It is only afterwards when "stock piles " were not found that the careerists in the CIA went into CYA mode.

As more of the tapes and documents get exploited I am convincved that the Bush position will be vindicated .






















JBodine rated this answer Excellent or Above Average Answer
ETWolverine rated this answer Excellent or Above Average Answer
LTgolf rated this answer Excellent or Above Average Answer

Question/Answer
BeelzeBUSH asked on 03/21/06 - NCAA Basketball Tournament...

I tried watching basketball the other day and it was obvious that almost every commercial time out came with a full government recruiting effort. In fact it went beyond the normal Army recruiting ads. I find this disturbing and rather embarrassing that our government has to lure innocent youth. Some of the ads even focused on actors pretending to be parents that give their ok. What kind of sick love is that? George

tomder55 answered on 03/21/06:

do you want a draft ? if not then the military has to recruit . the court upheld their right to recruit on campus just like any other business can . if the demographics they are looking for is watching the NCAAs then why not place ads there . "lure innocent youth " ;no they are not . They are offering another career option ;no different than if Walmart had advertised job openings .

fredg rated this answer Excellent or Above Average Answer
BeelzeBUSH rated this answer Average Answer
purplewings rated this answer Excellent or Above Average Answer

Question/Answer
JBodine asked on 03/20/06 - How 'bout a little perspective here?

Perspective is the single most important thing that modern western media seem totally incapable of providing.

Although I don't agree with every detail of this lawyer's attempt to provide it, his work is worth your time.

If anyone doubts what our world would "look like" under a successful Wahabbi Muslim jihad, please relook at the video smuggled out of Afghanistan by a female western journalist just prior to the Allied invasion of same. Dragging innocent women out into the middle of a soccer stadium in Kabul and murdering them to the cheers and songs of the crowd looks like something out of ancient Rome, but it is not. It is what our world will look like if we don't destroy these murderous thugs before they win.

A California Lawyer's Perspective on Iraq War.


Sixty-three years ago, Nazi Germany had overrun almost all of Europe and hammered England to the verge of bankruptcy and defeat, and had sunk more than four hundred British ships in their convoys between England and America for food and war materials.

Bushido Japan had overrun most of Asia, beginning in 1928, killing millions of civilians throughout China, and impressing millions more as slave labor.

The US was in an isolationist, pacifist mood, and most Americans and Congress wanted nothing to do with the European war, or the Asian war .

Then along came Pearl Harbor on December 7, 1941, and in outrage Congress unanimously declared war on Japan, and the following day on Germany, which had not attacked us.It was a dicey thing. We had few allies.

France was not an ally, the Vichy government of France aligned with its German occupiers. Germany was not an ally, it was an enemy, and Hitler intended to set up a Thousand Year Reich in Europe. Japan was not an ally, it was intent on owning and controlling all of Asia. Japan and Germany had long-term ideas of
invading Canada and Mexico, and then the United States over the north and south borders, after they had settled control of Asia and Europe.

America's allies then were England, Ireland, Scotland, Canada, Australia, and Russia, and that was about it. There were no other countries of any size or
military significance with the will and ability to contribute much or anything to the effort to defeat Hitler's Germany and Japan, and prevent the global
dominance of Nazism. And we had to send millions of tons of arms, munitions, and war supplies to Russia, England, and the Canadians, Aussies, Irish, and Scots,
because none of them could produce all they needed for themselves.

All of Europe, from Norway to Italy, except Russia in the east, was already under the Nazi heel.

America was not prepared for war. America had stood down most of its military after WWI and throughout the depression, at the outbreak of WWII there were army units training with broomsticks over their shoulders because they didn't have guns, and cars with "tank" painted on the doors because they didn't have tanks. And a big chunk of our navy had just been sunk and damaged at Pearl Harbor.

Britain had already gone bankrupt, saved only by the donation of $600 million in gold bullion in the Bank of England that was the property of Belgium and
was given by Belgium to England to carry on the war when Belgium was overrun by Hitler - actually, Belgium surrendered one day, because it was unable to oppose the German invasion, and the Germans bombed Brussels into rubble the next day anyway just to prove they could.

Britain had been holding out for two years already in the face of staggering shipping loses and the near-decimation of its air force in the Battle of Britain, and was saved from being overrun by Germany only because Hitler made the mistake of thinking the Brits were a relatively minor threat that could be dealt with later and turning his attention to Russia, at a time when England was on the verge of collapse in the late summer of 1940.

Russia saved America's butt by putting up a desperate fight for two years until the US got geared up to begin hammering away at Germany.

Russia lost something like 24 million people in the sieges of Stalingrad and Moscow, 90% of them from cold and starvation, mostly civilians, but also more than a million soldiers. More than a million. Had Russia surrendered, then, Hitler would have been able to focus his entire campaign against the Brits, then America, and the Nazis would have won that war.

Had Hitler not made that mistake and invaded England in 1940 or 1941, instead, there would have been no England for the US and the Brits to use as a
staging ground to prepare an assault on Nazi Europe, England would not have been able to run its North African campaign to help take a little pressure off Russia while America geared up for battle, and today Europe would very probably be run by the Nazis, the Third Reich, and, isolated and without any allies (not
even the Brits), the US would very probably have had to cede Asia to the Japanese, who were basically Nazis by another name then, and the world we live in
today would be very different and much worse. I say this to illustrate that turning po ints in history are often dicey things. And we are at another one.

There is a very dangerous minority in Islam that either has, or wants and may soon have, the ability to deliver small nuclear, biological, or chemical
weapons, almost anywhere in the world, unless they are prevented from doing so.

France, Germany, and Russia, have been selling them weapons technology at least as recently as 2002, as have North Korea, Syria, and Pakistan, paid for
with billions of dollars Saddam Hussein skimmed from the "Oil For Food" program administered by the UN with the complicity of Kofi Annan and his son.

The Jihadis, the militant Muslims, are basically Nazis in Kaffiyahs - they believe that Islam, a radically conservative (definitely not liberal!) form of
Wahhabi Islam, should own and control the Middle East first, then Europe, then the world, and that all who do not bow to Allah should be killed, enslaved, or
subjugated. They want to finish the Holocaust, destroy Israel, purge the world of Jews. This is what they say.

There is also a civil war raging in the Middle East - for the most part not a hot war, but a war of ideas. Islam is having its Inquisition and its
Reformation today, but it is not yet known which will win - the Inquisition, or the Reformation.

If the Inquisition wins, then the Wahhabis, the Jihadis, will control the Middle East, and the OPEC oil, and the US, European, and Asian economies, the
techno-industrial economies, will be at the mercy of OPEC - not an OPEC dominated by the well-educated and rational Saudis of today, but an OPEC dominated by
the Jihadis.

You want gas in your car? You want heating oil next winter? You want jobs? You want the dollar to be worth anything? You better hope the Jihad, the Muslim
Inquisition, loses, and the Islamic Reformation wins.

If the Reformation movement wins, that is, the moderate Muslims who believe that Islam can respect and tolerate other religions, and live in peace with the rest of the world, and move out of the 10th century into the 21st, then the troubles in the Middle East will eventually fade away, and a moderate and
prosperous Middle East will emerge.

We have to help the Reformation win, and to do that we have to fight the Inquisition, i.e., the Wahhabi movement, the Jihad, Al Qaeda, the Islamic
terrorist movements. We have to do it somewhere. We cannot do it nowhere. And we cannot do it everywhere at once. We have created a focal point for the battle now at the time and place of our choosing, in Iraq.

Not in New York, not in London, or Paris, or Berlin, but in Iraq, where we did and are doing two very important things.

(1) We deposed Saddam Hussein. Whether Saddam Hussein was directly involved in 9/11 or not, it is undisputed that Saddam has been actively supporting the
terrorist movement for decades. Saddam is a terrorist.

Saddam is, or was, a weapon of mass destruction, who is responsible for the deaths of probably more than a million Iraqis and two million Iranians.

(2) We created a battle, a confrontation, a flash point, with Islamic terrorism in Iraq. We have focused the battle. We are killing bad guys there and the
ones we get there we won't have to get here, or anywhere else. We also have a good shot at creating a democratic, peaceful Iraq, which will be a catalyst
for democratic change in the rest of the Middle East, and an outpost for a stabilizing American military presence in the Middle East for as long as it is
needed.

The Euros could have done this, but they didn't, and they won't. We now know that rather than opposing the rise of the Jihad, the French, Germans, and
Russians were selling them arms - we have found more than a million tons of bsp;weapons and munitions in Iraq. If Iraq was not a threat to anyone, why did Saddam need a million tons of weapons?

And Iraq was paying for French, German, and Russian arms with money skimmed from the UN Oil For Food Program (supervised by UN Secretary General Kofi Annan
and his son) that was supposed to pay for food, medicine, and education , for Iraqi children.

World War II, the war with the German and Japanese Nazis, really began with a "whimper" in 1928. It did not begin with Pearl Harbor. It began with the
Japanese invasion of China. It was a war for fourteen years before America joined it. It officially ended in 1945 - a 17 year war - and was followed by another
decade of US occupation in Germany and Japan to get those countries reconstructed and running on their own again . a 27 year war.

World War II cost the United States an amount equal to approximately a full year's GDP - adjusted for inflation, equal to about $12 trillion dollars, WWII
cost America more than 400,000 killed in action, and nearly 100,000 still missing in action.
[The Iraq war has, so far, cost the US about $160 billion, which is roughly what 9/11 cost New York. It has also cost about 2,200 American lives, which is
roughly 1/2 of the 3,000 lives that the Jihad snuffed on 9/11.] But the cost of not fighting and winning WWII would have been unimaginably greater - a world
now dominated by German and Japanese Nazism.

Americans have a short attention span, now, conditioned I suppose by 60 minute TV shows and 2-hour movies in which everything comes out okay.

The real world is not like that. It is messy, uncertain,and sometimes bloody and ugly. Always has been, and probably always will be.

If we do this thing in Iraq successfully, it is probable that the Reformation will ultimately prevail. Many Muslims in the Middle East hope it will. We will be there to support it. It has begun in some countries, Libya, for instance. And Dubai. And Saudi Arabia. If we fail, the Inquisition will probably
prevail, and terrorism from Islam will be with us for all the foreseeable future, because the Inquisition, or Jihad, believes they are called by Allah to kill all the Infidels, and that death in Jihad is glorious.

The bottom line here is that we will have to deal with Islamic terrorism until we defeat it, whenever that is. It will not go away on its own. It will not
go away if we ignore it.

If the US can create a reasonably democratic and stable Iraq, then we have an "England" in the Middle East, a platform, from which we can work to help
modernize and moderate the Middle East. The history of the world is the clash between the forces of relative civility and civilization, and the barbarians
clamoring at the gates. The Iraq war is merely another battle in this ancient and never-ending war. And now, for the first time ever, the barbarians are about to
get nuclear weapons. Unless we prevent them. Or somebody does.

The Iraq war is expensive, and uncertain, yes. But the consequences of not fighting it and winning it will be horrifically greater. We have four options -

1. We can defeat the Jihad now, before it gets nuclear weapons.

2. We can fight the Jihad later, after it gets nuclear weapons (which may be as early as next year, if Iran's progress on nuclear weapons is what Iran claims it is).

3. We can surrender to the Jihad and accept its dominance in the Middle East, now, in Europe in the next few years or decades, and ultimately in America.

4. Or we can stand down now, and pick up the fight later when the Jihad is more widespread and better armed, perhaps after the Jihad has dominated France
and Germany and maybe most of the rest of Europe. It will be more dangerous, more expensive, and much bloodier then.

Yes, the Jihadis say that they look forward to an Islamic America. If you oppose this war, I hope you like the idea that your children, or grandchildren,
may live in an Islamic America under the Mullahs and the Sharia, an America that resembles Iran today.

We can be defeatist peace-activists as anti-war types seem to be, and concede, surrender, to the Jihad, or we can do whatever it takes to win this war
against them.

The history of the world is the history of civilizational clashes, cultural clashes. All wars are about ideas, ideas about what society and civilization
should be like, and the most determined always win.

Those who are willing to be the most ruthless always win. The pacifists always lose, because the anti-pacifists kill them.

In the 20th century, it was Western democracy vs. communism, and before that Western democracy vs. Nazism, and before that Western democracy vs. German
Imperialism. Western democracy won, three times, but it wasn't cheap, fun, nice, easy, or quick. Indeed, the wars against German Imperialism (WWI), Nazi
Imperialism (WWII), and communist imperialism (the 40-year Cold War that included the Vietnam Battle, commonly called the Vietnam War, but itself a major battle in a larger war) covered almost the entire century.

The first major war of the 21st Century is the war between Western Judeo/Christian Civilization and Wahhabi Islam. It may last a few more years, or most of this century. It will last until the Wahhabi branch of Islam fades away, or gives up its ambitions for regional and global dominance and Jihad, or until
Western Civilization gives in to the Jihad

Senator John Kerry, in the debates and almost daily, makes 3 scary claims:

1. We went to Iraq without enough troops.

We went with the troops the US military wanted. We went with the troop levels General Tommy Franks asked for. We deposed Saddam in 30 days with light
casualties, much lighter than we expected.

The real problem in Iraq is that we are trying to b e nice - we are trying to fight minority of the population that is Jihadi, and trying to avoid killing
the large majority that is not. We could flatten Fallujah in minutes with a flight of B52s, or seconds with one nuclear cruise missile - but we don't. We're
trying to do brain surgery, not amputate the patient's head. The Jihadis amputate heads.

2. We went to Iraq with too little planning.

This is a specious argument. It supposes that if we had just had "the right plan" the war would have been easy, cheap, quick, and clean.

That is not an option. It is a guerrilla war against a determined enemy, and no such war ever has been or ever will be easy, cheap, quick, and clean. This
is not TV.

3. We proved ourselves incapable of governing and providing security.

This too is a specious argument. It was never our intention to govern and provide security. It was our intention from the beginning to do just enough to
enable the Iraqis to develop a representative government and their own military and police forces to provide their own security, and that is happening. The US and the Brits and other countries there have trained over 100,000 Iraqi police and military, now, and will have trained more than 200,000 by the end of
next year. We are in the process of transitioning operational control for security back to Iraq.

It will take time. It will not go with no hitches. This is not TV.

Remember, perspective is everything, and America's schools teach too little history for perspective to be clear, especially in the young American mind.

The Cold war lasted from about 1947 at least until the Berlin Wall came down in 1989. Forty-two years. Europe spent the first half of the 19th century fighting Napoleon, and from 1870 to 1945 fighting Germany.

World War II began in 1928, lasted 17 years, plus a ten year occupation, and the US still has troops in Germany and Japan. World War II resulted in the
death of more than 50 million people, maybe more than 100 million people, depending on which estimates you accept.

The US has taken a little more than 2,000 KIA in Iraq. The US took more than 4,000 Killed in action on the morning of June 6, 1944, the first day of the
Normandy Invasion to rid Europe of Nazi Imperialism. In WWII the US averaged 2,000 KIA a week for four years. Most of the individual battles of WWII lost more Americans than the entire Iraq war has done so far.

But the stakes are at least as high . . a world dominated by representative governments with civil rights, human rights, and personal freedoms . or a
world dominated by a radical Islamic Wahhabi movement, by the J ihad, under the Mullahs and the Sharia (Islamic law).

I do not understand why the American Left does not grasp this. They favor human rights, civil rights, liberty and freedom, but evidently not for Iraqis. In
America, absolutely, but nowhere else.

300,000 Iraqi bodies in mass graves in Iraq are not our problem. The US population is about twelve times that of Iraq, so let's multiply 300,000 by twelve.
What would you think if there were 3,600,000 American bodies in mass graves in America because of George Bush? Would you hope for another country to help
liberate America?

"Peace Activists" always seem to demonstrate where it's safe, in America.

Why don't we see Peace Activist demonstrating in Iran, Syria, Iraq, Sudan, North Korea, in the places in the world that really need peace activism the most?

The liberal mentality is supposed to favor human rights, civil rights, democracy, multiculturalism, diversity, etc., but if the Jihad wins, wherever the
Jihad wins, it is the end of civil rights, human rights, democracy, multiculturalism, diversity, etc. Americans who oppose the liberation of Iraq are coming
down on the side of their own worst enemy.

If the Jihad wins, it is the death of Liberalism. Everywhere the Jihad wins, it is the death of Liberalism. And American Liberals just don't get it.

Raymond S. Kraft is a writer and lawyer living in Northern California.

~~~~~~~~~~~~~~~~~~~~~~~~~~~~~~~~~~~~~~~~~~~~~~~~~

Not bad for a) a lawyer; and, b) a lawyer living in the Republic of Kalifornia.

Thoughts?

tomder55 answered on 03/20/06:

Here is some more perspective :The worst month of U.S. military deaths in Vietnam was May 1968: 2,316 lives. The second worst month in Vietnam was February 1968: 2,293 lives. The total U.S. military deaths in Iraq since March 20, 2003 is 2,317 lives, one more than the worst month in Vietnam.

Sec.Def. Rumsfeld has some more to say about perspective :

Consider that in three years Iraq has gone from enduring a brutal dictatorship to electing a provisional government to ratifying a new constitution written by Iraqis to electing a permanent government last December. In each of these elections, the number of voters participating has increased significantly -- from 8.5 million in the January 2005 election to nearly 12 million in the December election -- in defiance of terrorists' threats and attacks.

One of the most important developments over the past year has been the increasing participation of Iraq's Sunni community in the political process. In the volatile Anbar province, where Sunnis are an overwhelming majority, voter turnout grew from 2 percent in January to 86 percent in December. Sunni sheiks and religious leaders who previously had been sympathetic to the insurgency are today meeting with coalition representatives, encouraging Iraqis to join the security forces and waging what violent extremists such as Abu al-Zarqawi and his al-Qaeda followers recognize as a "large-scale war" against them.

The terrorists are determined to stoke sectarian tension and are attempting to spark a civil war. But despite the many acts of violence and provocation, the vast majority of Iraqis have shown that they want their country to remain whole and free of ethnic conflict. We saw this last month after the attack on the Shiite shrine in Samarra, when leaders of Iraq's various political parties and religious groups condemned the violence and called for calm.

Another significant transformation has been in the size, capability and responsibility of Iraqi security forces. And this is vitally important, because it is Iraqis, after all, who must build and secure their own nation.

Today, some 100 Iraqi army battalions of several hundred troops each are in the fight, and 49 control their own battle space. About 75 percent of all military operations in the country include Iraqi security forces, and nearly half of those are independently Iraqi-planned, Iraqi-conducted and Iraqi-led. Iraqi security forces have a greater ability than coalition troops to detect a foreign terrorist's accent, identify local suspects and use force without increasing a feeling of occupation. It was these Iraqi forces -- not U.S. or coalition troops -- that enforced curfews and contained the violence after the attack on the Golden Dome Shrine in Samarra. To be sure, violence of various stripes continues to slow Iraq's progress. But the coalition is doing everything possible to see this effort succeed and is making adjustments as appropriate.

The rationale for a free and democratic Iraq is as compelling today as it was three years ago. A free and stable Iraq will not attack its neighbors, will not conspire with terrorists, will not pay rewards to the families of suicide bombers and will not seek to kill Americans.

Though there are those who will never be convinced that the cause in Iraq is worth the costs, anyone looking realistically at the world today -- at the terrorist threat we face -- can come to only one conclusion: Now is the time for resolve, not retreat.


Perhaps Mohammed at Iraq the Model blog offers the best expression of Iraqi hopes and fears on the anniverary of the invasion .

Life stopped and time stopped when Saddam ruled Iraq, actually that totalitarian regime was moving backwards and dragging us with it and nothing could stop the deterioration that began the moment Saddam came to power.
We had to accept the change and live with all that would come along with it whether good or bad.
The democracy we're practicing today in Iraq is the exact opposite of what we had for decades and until three years ago. This democracy carries the essence of life, the differences, the dynamics and yes, the failures but also the seed of a better future.

Before the liberation we were suffering and we had no hope, now we are also suffering but we have hope and I see this hope even in the words of those that are cynical about the outcome of the political process; who say they hope things will be better in four years or eight years
When Saddam was here we didn't have any hope and we could expect nothing good from a dead regime that cared only about its absolute existence.

Yes. We are facing enormous and dangerous challenges and this is not unexpected because the old will not easily step down and accept the loss; the old will fight back fiercely and the old here is not only Saddam and the Ba'ath, the old can be found among many of our current leaders and the mentality they carry that belong to the same generation that bred Saddam but I believe they will melt away as well because no one can go against the direction of time and the clock cannot be forced backwards.






JBodine rated this answer Excellent or Above Average Answer

Question/Answer
Erewhon asked on 03/20/06 - "US Influence In Iraq Wanes As Civil War Looms" (CSM) ...




By Scott Peterson, Staff writer of The Christian Science Monitor Mon Mar 20, 3:00 AM ET

BAGHDAD - Every day, more violence. And more uncertainty for Iraqis than they have ever known, as they mark three years since American troops invaded.
ADVERTISEMENT

The wave of optimism that once buoyed Iraqis after the fall of
Saddam Hussein is now being marked as yet another casualty of the bombs and murders that are part of daily life here.

But even as
Iraq slides toward full-scale civil war, Iraqi analysts are trying to envision a way out of a vicious insurgency, political deadlock, and sectarian bloodshed.

One factor they are considering is the changing American role. Despite the continued presence of 130,000 American troops, and arm-twisting efforts by US diplomats to forge a unity government, Iraq's democratic political process is, by definition, giving the US even less leverage to shape this broken nation's future.

"The majority of Iraqis are now against this occupation, whether they are Sunni, Shiite or Kurd," says Wamidh Omar Nadhmi, a political scientist who heads a Sunni-led group called the Foundation Conference. "But those in government positions are trying to unleash a campaign of suppression, to take advantage [of the violence], to dominate.

"Now we are told: '[The Americans] are not going to take sides,' " says Mr. Nadhmi, referring to remarks by US officials last week that Iraqi forces must handle sectarian strife on their own. "But if it comes to civil war, and the US does not try to keep order, as the controlling power, then why do they stay in Iraq?"

Tens of thousands of Iraqis have died in a maelstrom of insurgent violence. Many hundreds more are dying in sectarian killings that flared a month ago, after the destruction of the gold-domed Shiite shrine at Samarra.

The grim reality today - and the perception among so many Iraqis that the US is responsible - could not be in sharper contrast from the faith Iraqis once held, that the all-powerful Americans would solve their problems.

"It is unfortunate that we are in civil war," Iyad Allawi, Iraq's former prime minister, told BBC news Sunday. "We are losing each day an average of 50 to 60 people throughout the country, if not more. If this is not civil war, then God knows what civil war is."

That view is hotly disputed by US officials and commanders, but is on the lips of many Iraqis here. "Maybe we have not reached a point of no return yet, but we are moving towards this point," said Mr. Allawi.

But escaping that vortex will not be easy, analysts say.

"The new government is incapable of ruling the street without the American presence," says Ahmad al-Rikabi, head of the popular Radio Dijla in Baghdad. "If [US forces] left Iraq, the future of the country would be in the hands of the militias. This is the case already, but we still have some hope [the US] will keep some balance."

The stakes could not be higher, for Iraq or for the region,
President Bush said last week. "The battle lines in Iraq are clearly drawn for the world to see, and there is no middle ground," he said. "The enemy will emerge from Iraq one of two ways: Emboldened or defeated."

US Ambassador Zalmay Khalilzad has been pushing Iraqi leaders to form a unity government, that would mend rifts inside the main Shiite bloc, and ensure a significant role for the minority Sunni Arabs. At the formal opening of parliament last Thursday, he had a place in the handshaking line alongside Iraqi political leaders.

But diplomats close to the talks say not all Iraqi leaders welcome the forceful US intervention. Both Washington and Tehran have signaled that they could commence talks to solve the crisis, in what would be the first publicly acknowledged contact since
Iran's 1979 Islamic revolution.

"Saddam Hussein is still ruling Iraq - he may be behind bars, but he created the mentality we have today," says Mr. Rikabi. "Saddam has to be executed. It's not revenge, and won't be satisfying for his victims. But it will be good for Shiite and Kurds, and even Sunnis, to help them focus on their leaders."

"The symbols of the past are still in front of our eyes; we're still living in the past, and must get rid of the past," says Rikabi.

p>But getting rid of the past means getting rid of business as usual, and that means coming to terms with the increasingly pervasive Shiite militia influence in Iraq's security forces, which are accused of abuse, torture, and operating death squads that target Sunni Arabs. They also reportedly let other Shiite militias, like the Mahdi Army of anti-US cleric Moqtada al-Sadr, operate with impunity.

"The first thing is to change the minister," says a Shiite police colonel who asked not to be named, referring to Bayan Jabr, the minister of interior and a former Badr militia leader. "We need an independent one."

Ambassador Khalilzad has pushed for such a change. Shiites are targeted almost exclusively by the Sunni Arab extremist insurgents. But Sunni Arabs say they see little change on the ground.

"Nobody obliged the minister of interior to resign - he should be arrested," says Mr. Wamidh. "There have been no actual steps in favor of the Sunnis, but accumulated attacks against them."

The daily toll was again evident Sunday, as hundreds of thousands of Shiite pilgrims made their way by foot to the sacred city of Karbala, south of Baghdad, to mark Monday the death of Hussein, the grandson of the prophet Muhammad.

Pilgrims have been subject to drive-by shootings and bombings by Sunni Arab extremists that have killed four. A mortar landed near the shrine of Imam Hussein but caused no harm. An extra 700 American troops were deployed from Kuwait, to boost security during the religious event.

In Baghdad four bodies killed execution-style were found Sunday; 22 were found, by one count, the day before.

Whatever the result of the political wrangling, many Iraqis say it will not be enough to correct three years of US mistakes - from disbanding Saddam Hussein's 400,000-strong army with the stroke of a pen, to a vigorous de-Baathification plan that swept capable bureaucrats from government - that helped fuel insurgency.

Out of touch politicians
But there is a further problem, analysts say, that no amount of US influence can help: The fact that insecurity is so pervasive, that Iraqi leaders and the government meet inside the bubble of the Green Zone, among a labyrinth of 12-foot-high concrete blast walls woven together with coils of concertina wire that keep them safe, as well as isolated.

"The politicians are out of touch with the street, so it is like a group of blind people negotiating," says Rikabi. "They have nothing to do with the Republic of Iraq ... they do not feel a power cut for a second, while outside, [electricity] is off for 22 hours a day. You can't make the right decision, when the prime minister still has his family in London."

Relying on such politicians also risks the endgame for the US, which wants a unity government to take control - and control Iraqi security forces - so American forces can begin withdrawing.

"I can see their dilemma," says Nadhmi. "[President George] Bush is triumphant about democracy in Iraq, but if he tries to intervene and put in a prime minister of his own, it would be a contradiction."

Seventy-five daily insurgent attacks
Still, options are limited for the US - both military and political - which last week accused Iran of "meddling" in Iraqi affairs, and claimed that Iran had helped insurgents improve their explosive techniques.

The US military's "kinetic or muscular approach has failed to produce sustained success," says a report last month from the International Institute of Strategic Studies (IISS), noting that insurgent attacks at a tempo of 75 per day against coalition forces show "no signs of diminishing."

It notes that several key non-Al Qaeda insurgent groups secretly agreed to a 21-point "principles for dialogue" with US forces last December. Such talks have not taken place in part, the IISS says, because "the very logic of elections, bringing to power an indigenous government with a mandate, has directly reduced US influence over Iraqi politics."

This has been clear for months to many in Iraq, who look back with nostalgia on Saddam Hussein, in the way that older Russians often crave the order once instilled upon the Soviet Union by Josef Stalin.

"People were executed in Saddam's days, but it is the same today," says Rikabi, of Radio Dijla. "Then it was behind high walls, now it is by this or that militia.

"Before, people respected the traffic police, there was an organization, a state; today we have the smell, the shadow of a government," says Rikabi. "Before, people would go to restaurants until 2 a.m.; today their lives are full of fear.

"Then, we had one Saddam Hussein," concludes Rikabi. "Today, we have many Saddam Husseins."

It might be a bitter pill to swallow for those who still manage to cling on to the Bushite line that the War is going well, but even the most fierce supporters of King George should take stock of the current upsurge in violence and the almost inevitable consequences.

What must Bush do to bring peace to Iraq?

Does he have the will and the savvy to save Iraq from melt-down?

tomder55 answered on 03/20/06:

A civil war is a visible event whose indicators includes the insubordination of armed units, mass refugee flows, the rise of rival governments, etc.

One interesting indicator of how the US military sees the situation are its plans to turn over large parts of the country to Iraqi forces.

March 17 (Bloomberg ) -- The U.S. hopes to hand over 75 percent of Iraq to Iraqi Security Forces by the end of the summer, the second-ranking U.S. commander in Baghdad said. ``All indications are that we will make that,'' Lieutenant General Peter Chiarelli, commander of Multinational Corps Iraq, said from Baghdad during a briefing televised at the Pentagon today, adding that he didn't ``want to be so precise as to put myself in a box.'' ...

Since the bombing, Iraqi security forces have performed well ``without regard for their religious or tribal affiliations,'' Chiarelli said. ... He estimated that Iraqis currently control ``somewhere under 50 percent'' of the nation, adding that ``there are large areas out in al-Anbar where that is not the case.'' Insurgent violence is centered in al-Anbar, a Sunni-dominated province west of the capital. ``We are finding that Iraqi units, with our support, can be used in just about any operation we do in a counter-insurgency role,'' Chiarelli said, speaking via a video-link from Baghdad. ``They are particularly well prepared and well trained and have the ability to do that in just about any area.''


This suggests confidence, but it also admits that while Iraqi forces are coming along, they are not yet decisive without the assistance of US forces.

But no one ;not even the doom and gloom MSM is saying anything about the possibility of a Sunni insurgency succeeding anymore .In fact the whole meme has changed .No longer are the Sunnis portrayed as the aggressors ;now they are the victims . Figure that out ! If they were being honest they would admit that the campaign against the insurgency has been an unqualified success.

Erewhon rated this answer Excellent or Above Average Answer
excon rated this answer Excellent or Above Average Answer

Question/Answer
HerrAirhorn asked on 03/19/06 - Disgust in the Heartland

SOUTH BEND, Ind. -- The third anniversary of the Iraq invasion unleashed a surge of pessimism at a local farmers' market here, where stalwart Republicans, standing amid aisles of produce and miracle cures, said President Bush has messed up a war that looks more like Vietnam every day.

''It's chaos," said Roger Madaras, who voted twice for Bush. ''How many more people are going to be killed? We were going in to free the people of Iraq, but as far as I'm concerned, a lot of them are worse off today than they were under the dictatorship...."
Boston Globe dot com

How long can a "democratic" government wage a war that that the citizens don't support? They are taking to the streets now, marching and demonstrating.

tomder55 answered on 03/20/06:

"We were going in to free the people of Iraq, but as far as I'm concerned, a lot of them are worse off today than they were under the dictatorship...."


This guy has no clue about what he speaks .The only ones worse off are the remnants of the Batthist thugocracy . When people talk about "civil war " do they take into account the oppression the people suffered under the Tikriti clan ? NO ..everything was peachy and Saddam was benevolent.


How long can a "democratic" government wage a war that that the citizens don't support? They are taking to the streets now, marching and demonstrating.

a whole 200 people took time off from their busy sunny Sunday schedule to march down 5th Ave. NYC yesterday . I see the people are really up in arms !!

HerrAirhorn rated this answer Excellent or Above Average Answer

Question/Answer
HerrAirhorn asked on 03/19/06 - Asking...

more and more, we hear people asking themselves about the war going on and on and on......."Would you want to be the last soldier to die in Iraq?"

These people remember Viet Nam.


tomder55 answered on 03/20/06:

were you part of the 200 demonstrators in NYC yesterday ? By this time during Nam there were millions around the country marching ,closing down campus's ,etc.


makes me want to question the polling data .

HerrAirhorn rated this answer Excellent or Above Average Answer

Question/Answer
Erewhon asked on 03/18/06 - Bush Increasingly Focused On How Revisionist History Will See Him





March 13, 2006 | Issue 4211

WASHINGTON, DCWith many of his administration's policies facing growing public disapproval, President Bush is reportedly becoming more concerned with how he will be portrayed by future revisionist historians. "Just last summer, the president never reflected on how apologists would spin his increased lobbying for an unpopular war, or how future far-right generations would justify his failed domestic policy initiatives," presidential scholar Dr. Robert Dallek said. "He reportedly asked an aide if, decades from now, the deluded would see him as great, like Ronald Reagan, or merely as a fully redeemed elder statesman, like Richard Nixon." Margaret Meehan, a spokesman for the National Board Of Historical Revision, offered no comment on any future portrayal of "America's most beloved and accomplished president."

===

Should he be worrying about anything else?




tomder55 answered on 03/18/06:

revisionst history is already being dutifully reported by the primary sources of the day .

Erewhon rated this answer Excellent or Above Average Answer

Question/Answer
Erewhon asked on 03/17/06 - Bush's administation wastes wastes and wastes as needy people stay mired down in their need!<

GAO: Millions wasted during Katrina relief
Among other problems, $3 million spent after hurricane on beds never used

MSNBC.com

Rising from Ruin
MSNBC.com follows two towns as they rebuild after Katrina. Follow their progress through on-going stories and citizen diaries.

Rita: sea-level, refinery and political maps
Animations of potential Houston flooding
Katrina flyover: Block by block
New Orleans: Before and after
Newsweek: The view from above
NOAA's view of the devastation

GOP irritation at Bush brewing for long time
GAO: Millions wasted during Katrina relief
National Disservice
Updated: 3:22 p.m. ET March 16, 2006

WASHINGTON - The government wasted millions of dollars in its award of post-Katrina contracts for disaster relief, including at least $3 million for 4,000 beds that were never used, federal auditors said Thursday.

The Government Accountability Offices review of 13 major contracts many of them awarded with limited or no competition after the Aug. 29 storm offers the first preliminary overview of their soundness.

Waste and mismanagement were widespread due to poor planning and miscommunication, according to the five-page briefing paper released Thursday. That led to money that was paid for work never used.


The governments response to Hurricanes Katrina and Rita depended heavily on contractors to deliver ice, water and food supplies; patch rooftops; and provide housing to displaced residents, said the report by the GAO, Congress auditing arm. FEMA did not adequately anticipate needs. (Bust Bush assured the nation that "Brownie" was "Doing a good job!")

Nicol Andrews, a spokeswoman for FEMA, said the agency was working hard to improve its awarding of billions of dollars of government contracts as it prepares for the next hurricane season.

We have been working with (Homeland) Secretary (Michael) Chertoff to incorporate unprecedented levels of oversight in FEMA contracting, she said. However, in the event of a disaster when minutes count, we have the authority to do what it takes to move quickly.

By and large, the initial criticism of FEMA, from members of Congress and others, was that the agency moved too slowly to assist hurricane victims.

Contracts with politically connected companies

Of more than 700 contracts valued at $500,000 or greater, more than half were awarded without competition, often to politically connected companies such as Halliburton subsidiary Kellogg, Brown & Root, Bechtel Corp. and AshBritt Inc.

Democrats, in particular, in recent weeks have called for limits on no-bid agreements, which they say have been awarded to politically connected companies at the expense of a slow Gulf Coast rebuilding effort.

Previous reports of waste in the aftermath of Katrina have been bad, but this one is worse, said Rep. Henry Waxman, D-Calif., the top Democrat on the House Government Reform Committee.

The Bush administration has learned nothing from its disastrous contract management in Iraq, he said. The administration seems incapable of spending money in a way that actually meets the needs of Gulf Coast residents.


[TELL HIM WHERE HE IS WRING!]

The GAO report released Thursday speaks broadly and does not address the validity of no-bid contracts; those reviews are currently under way by inspector generals at Homeland Security and other agencies. But it found significant problems in its general review of the 13 contracts, most of which were limited bid.

Agencies praised for 'hard work'

According to the report, the GAO praised government agencies for their hard work in securing contracts after the disaster, but said millions could have been saved if they had adopted previous GAO recommendations to hire more personnel, prearrange contracts and better train staff.

[BUT, NOBODY LISTENS TO CRITICS!]

Among the findings:

* Non-existent communication with local officials led to misjudgments on the need for temporary housing.

They included $3 million that FEMA spent for 4,000 base camp beds that were never used and

$10 million to renovate and furnish

240 rooms in Alabama, which housed only six occupants before being closed.

* Poor coordination between FEMA and the Army Corps contributed to waste in an Americold Logistics LLCs contract for ice.

The local Corps personnel were not always aware of where ice might be delivered and did not have authority ... resulting in inefficient distribution, it said.

* Inadequate planning led to the award of a Mississippi contract for classrooms without competition.

Information in the contract files suggests the negotiated prices were inflated.

A review of that specific contract, with Akima Site Operations LLC, was continuing.

* FEMA had only 17 of the 27 monitors it deemed necessary to oversee the installation of temporary housing in four states, leading to inadequate controls.

The 13 Katrina contracts reviewed involve the following 12 companies: C. Henderson Consulting; Americold Logistics; Clearbrook LLC; CS&M Associates; Gulf Stream Coach Inc.; Morgan Building & Spas Inc.; Bechtel National; Fluor Enterprises Inc.; CH2M Hill Constructors Inc.; E.T.I. Inc.; Ceres Environmental Services Inc.; and Thompson Engineering Inc.

Some of the firms, including Gulf Stream Coach and Bechtel, have close ties to the Bush administration or have contributed significantly to the GOP.
[TELL ME THAT THE GOVERNMEN'T OWN AUDITORS ARE A BUNCH OF LEFT WING LIBERAL LOONIES AND I'LL LAUGH EVEN LOUDER!]

2006 The Associated Press.

====

Some say good old GW - others see what a scroundrel he really is.

tomder55 answered on 03/18/06:

I say this is proof positive that centralized control is a most inefficient way of operating govenment. But all the critics will be arguing for more of it . I don't get it .

Erewhon rated this answer Excellent or Above Average Answer
Itsdb rated this answer Excellent or Above Average Answer

Question/Answer
Itsdb asked on 03/16/06 - Sounds like a plan?

Movie theaters may ask to jam cell phones
Wed Mar 15, 2006 2:28 AM GMT10

LAS VEGAS (Reuters) - Movie theater owners faced with falling attendance are considering asking federal authorities for permission to jam cell phone reception in an attempt to stop annoying conversations during films, the head of the industry's trade group said on Tuesday.

Industry leaders at the ShoWest conference for theater owners want to find ways to win back crowds.

"I don't know what's going on with consumers that they have to talk on phones in the middle of theaters," John Fithian, president of the National Association of Theater Owners, told the ShoWest conference in Las Vegas.

Theaters are trying a number of ways to silence cell phones, from sweeps by ushers to funny fake movie trailers urging viewers to shut off phones.

Fithian said owners were considering other steps if that does not work.

"We will actually petition the Federal Communications (Commission) to remove the block" on jamming cell phones, he said.

That may be difficult, since federal law and FCC rules prohibit the use of cell phone jammers.

The industry is broadly trying to increase interest in the movies.

Motion Picture Association of America Chief Executive Dan Glickman told ShoWest that the industry is researching why and when people go to the movies and might consider an advertising campaign to encourage people to go out to the movies, just as the milk industry has succeeded with its Got Milk? campaign.

Reuters 2006. All Rights Reserved.

~~~~~~~~~~~~~~~~~~~~~~~~~~~~~~~~~~~~~~~~~~~~~~~~~~~~~~~

Ok, this is more thinking out loud than a question, but are these people just that stupid? They want to broadly "increase interest in the movies" and the best they can come up with is to jam cell phones? Granted, nobody wants to hear a cell phone - or the conversation - when they go to the movies so the moviegoers have some responsibility, but is this even remotely going to increase attendance?

How about making better movies?

How about making more movies that entertain instead of indoctrinating or making political statements?

How about making more movies a family can watch together as opposed to expressing the depth of the filmmaker's grasp of vocabulary?

How about treating us as customers instead of criminals? For crying out loud I can't even make a backup copy of a DVD.

How about letting us go to a show for less than an arm and a leg?

Your thoughts?

Steve

tomder55 answered on 03/17/06:

They had Steve Martin in one of those trailers ;it was a better ;more entertaining viewing than the movie I went to see .

I think it would be a bad idea if they jammed cell phones ,but patrons should know to put their ringer on vibrate and to not have conversations on the phone in the theater .One wonders sometimes how some survived all those years before cell phones. Some people should have them surgically implanted .

For my 2 cents;I shut mine down gladly . I am paying a pretty penny in the theater for a couple of hours of escapism but I can understand why others would feel it necessary to keep them on .

I know they are making a big push for people to go to the theater rather than DVD rentals but why should they care where their revenue comes from ? Perhaps the big box office is a relic of the 20th Century . They claim it is a unique experience to see a movie on the wide screen .So you go to a cineplex and what do you get ? ....tunnel-vison theater and occasionally the crowd you'd rather not hang with .The ambivance of the large theater with the big screen; chandelier;the pipe-organ and piano player days are long gone so why claim there is a special appeal that is just not there anymore ?

They should just plan on making movies and releasing them right to dvd and not obscess with box office revenue. I still go to the theater on those rare occasions when I have to see a new release .But ;there is a landmark old theater by me that runs classic movie festivals .Those I suscribe to season passes for .

ETWolverine rated this answer Excellent or Above Average Answer
Itsdb rated this answer Excellent or Above Average Answer

Question/Answer
Erewhon asked on 03/16/06 - Bush Confirms His Policy of Pre-emptive Use of Force

By DEB RIECHMANN, Associated Press Writer 6 minutes ago

WASHINGTON -
President Bush on Thursday renewed his administration's strike-first policy against terrorists and other U.S. enemies and rebuked
Iran over allegations it is secretly amassing nuclear weapons.

The White House said that by reaffirming the pre-emptive policy, the United States was not targeting Iran. Yet the national security strategy includes harsh words for the Iranian government, which Bush says may pose the greatest challenge to the U.S.

"Our preference is to act through diplomacy in conjunction with friends and allies. That is our preference. That is our preference,"

Stephen Hadley, the president's national security adviser, said about the doctrine of pre-emption.

"It simply says, that one cannot let dangers grow to the point of eminent threat to the United States without taking action, and if other measures fail, obviously we retain the right to use force."

The 49-page report also said:

"North Korea poses a serious nuclear proliferation challenge; expresses dismay at rollbacks in democratic reform in Russia; brands Syria a tyranny that harbors terrorists and sponsors terrorist activity; and warns China against denying personal and political freedoms.


"China's leaders must realize, however, that they cannot stay on this peaceful path while holding on to old ways of thinking and acting that exacerbate concerns throughout the region and the world," Bush wrote.

The report accuses Iran of meddling in Iraq and equipping the insurgency, which is threatening a fragile democracy in Baghdad. The report was released as U.S. and Iraqi forces launched the largest air assault mission against insurgents and terrorists in Iraq since the U.S.-led invasion in April 2003.

The administration is working to persuade Russia and China to support a proposed U.N. Security Council resolution demanding that Iran end its uranium enrichment program.

"This diplomatic effort must succeed if confrontation is to be avoided," Bush said. He did not elaborate on what would happen if international negotiations with Iran were to fail.

Hadley said the international effort must speak with one voice if diplomacy can succeed in getting Iran to curb this step in nuclear weapons development.

"We are, I think, beginning to get indications that the Iranians are finally beginning to listen," Hadley said. "There is beginning to be a debate within the leadership and I would hope a debate between the leadership and their people about whether the course they're on is the right course for the good of their country."

The report is an updated version of one Bush issued in 2002 that outlined the pre-emptive policy, marking an end of a deterrent military strategy that dominated the Cold War.

The latest report makes it clear Bush has not changed his mind, even though no weapons of mass destruction were found in Iraq.

"Obviously, we didn't have the intelligence we needed in that particular instance," Hadley said. "In some sense, those countries that pursue weapons of mass destruction in secret also learned an important lesson that there are risks of that kind of behavior and that kind of activity."

Susan Rice, a national security expert at the Brookings Institution, an independent policy research group, said the report echoes the 2002 version "by reaffirming the discredited doctrine of pre-emption, while shifting the presumed target of that doctrine from Iraq to Iran."

"This shift is ironic since the administration's all-encompassing, four-year preoccupation with Iraq afforded Iran the time and space to pursue its nuclear ambitions and undermine U.S. security interests in the Middle East," Rice said.

___

More of the same from Johnny-One-Note!

Are you ready for more wars?

tomder55 answered on 03/17/06:

"This shift is ironic since the administration's all-encompassing, four-year preoccupation with Iraq afforded Iran the time and space to pursue its nuclear ambitions and undermine U.S. security interests in the Middle East," Rice said.

OR

It could be credibly argued that seeing Iraq defy the "International Community's"and the UN directives for over a decade would embolden the Iranians to accelerate their nuke development even further ......with the assistance of the Libyan program which would not have been cancelled without the Iraq invasion ..... and with the full advice of the AQ Kahn network which would still be operating freely .
................


Just like rejecting the first use option regarding nuclear weapons ;it would be a terrible mistake to outright reject the preemption option . To be true to such a doctrine would compel the nation to always act on the defensive or to do the more cynical act of provoking an attack on the nation so to then claim the moral high ground .
I thought in the debates of the past that those who rejected the first strike option were unworthy of leading a nation . I still think so.

The directive reaffirms preemptive action and the spread of democracy to be the endgame of the war against terrorists and terrorist nations. That is fine by me .

Erewhon rated this answer Excellent or Above Average Answer
ETWolverine rated this answer Excellent or Above Average Answer
Itsdb rated this answer Excellent or Above Average Answer

Question/Answer
jackreade asked on 03/16/06 - Yogi-ism

"Hey, we're lost, but we are making good time"! >Yogi Berra.



Applying this quote to the War in Iraq, are we:

Lost and making good time?

Lost and making bad time?

Not lost and making good time?

Not lost and making bad time?

tomder55 answered on 03/16/06:

I'd say it is at a critical juncture . Either our experiment will succeed or not in the next few months. The bell mark to me will not be the level of violence . That could go on for some time .But ;Sectatian violence does not a civil war make. India ,the largest democracy in the world has had sectarian violence since it's forst day of freedom.

The question is if the Parliment can forge a consensus govenment . I see movement that is positive in that area . It is no longer a given that extreme Shia parties will govern .The Kurds have prudently backed away from their automatic collition with the Shia . The strong Islamist based block is disintigrating and it is no longer a given that Ibrahim Jaafari will be PM in the new gvt. (thank God ;the man is a fool)

David Ignatius at Washington Post has more on the political developments .He believes that the logjams that was delaying the political process have been broken.

But let's say that the experiment has failed . Which form of totalitarianism will you celebrate ? Realpolitk being the standard means that any brute can do anything inside the country they rule and it is nobody's business. The world watched potentlessly by while genocide occurs in Darfur ;or previously in Rhwanda ;former Yugoslavia and Cambodia . When Saddam was gassing the Kurds it was an "internal matter" in which no one had a right to meddle.Had Hitler not made the mistake of invading Poland then it would not have been anyone elses business if he had gassed up the entire Jewish population of Germany.

No matter what the outcome ;the decision to invade was the right thing to do .

Itsdb rated this answer Excellent or Above Average Answer
ETWolverine rated this answer Excellent or Above Average Answer
jackreade rated this answer Excellent or Above Average Answer

Question/Answer
Mathatmacoat asked on 03/16/06 - It gets dirty when Jesus get involved in politics?

Jesus would vote for us, says Greens
Thursday Mar 16 12:40 AEDT
Jesus Christ would vote for the Australian Greens in the South Australian election, the party says.

In a veiled shot at Christian party Family First, Greens candidate Mark Parnell said if Jesus was voting at Saturday's state election, he would not choose a party that discriminated against people on the basis of sexuality or religion.

Mr Parnell also criticised Labor's policy to publicly name and shame young offenders and said Jesus would not support "simply writing them off as worthless reprobates".


VOTE: Who would Jesus vote for?

Nor would Jesus vote for the Democrats, with their dirty campaign tactics and undermining of others to get ahead in the polls, Mr Parnell said.

"The politics of fear, division, bigotry and negative campaigning perpetrated by a number of the other parties are un-Christian and totally out of step with Jesus' emphasis on peace, love and compassion," he said.

"The teachings of Jesus Christ stress compassion, tolerance, concern for the poor, non-violence and care for the earth.

"These are the very same principles that underpin the existence and policies of the Australian Greens."

The Greens decision to ask the electorate who Jesus would vote for follows a similar tactic by the Australian Democrats, who recently claimed former SA Labor premier Don Dunstan would vote for the Democrats - and not Labor - if he was alive.

So the Question is would Jesus endorse any candidate?

tomder55 answered on 03/16/06:

He would not vote for those who support abortions .He believed in protecting the innocents .
on the Nanny State Jesus would tell them :"For even when we were with you, this we commanded you, that if any would not work, neither should he eat" (2 Thess. 3:10).

He would want the State to defend the people .Jesus spoke about being vigilant and prepared to defend one's home: "Or else how can one enter into a strong man's house, and spoil his goods, except he first bind the strong man? and then he will spoil his house" (Matt. 12:29).

Mathatmacoat rated this answer Excellent or Above Average Answer

Question/Answer
ETWolverine asked on 03/15/06 - Talk about a HUGE screwup. Or was it....

The death penalty case against Zacharias Moussaoui may be dead in the water.

This case was, in my opinion, a no-brainer. The guy deserves the death penalty, and the evidence to give him a death sentence is there. But trust a lawyer to screw the whole thing up.

From today's NY Post:

----------------------

HOW 'HELPFUL' FED LAWYER BLEW IT BIG-TIME
By ANDY GELLER

March 15, 2006 -- The lawyer whose massive bungling may have blown the death-penalty case against Zacarias Moussaoui has worked on security issues for much of her 19 years in government.

Carla Martin - the $120,000-a-year Transportation Security Administration lawyer at the center of the storm of shocking courtroom gaffes by the feds - actually thought she was helping.

But she only succeeded in getting half the government's case thrown out.


Martin, 51, who has worked for the TSA for four years and for the FAA for 15 years before that, was an FAA attorney at the 1992 Pan Am 103 trial. Colleagues say she is passionate about security issues.

The sorry spectacle began when Martin heard the prosecution's opening statement at Moussaoui's sentencing trial.

She thought the statement "created a credibility gap that the defense can drive a truck through."

So the clueless lawyer decided to act on her own.

One of Martin's whale-sized blunders was e-mailing transcripts of the opening statements and testimony by FBI Agent Michael Anticev to seven Federal Aviation Administration officials who were to appear as witnesses.

Martin, a graduate of the Washington College of Law, wanted to make sure the officials emphasized that the feds could have foiled 9/11 if Moussaoui had spilled the beans on the plot when he was arrested three weeks before the terror attacks.

But her action violated federal Judge Leonie Brinkema's order that people scheduled to testify not be given access to transcripts by prior witnesses. She barred testimony from the officials.

andy.geller@nypost.com

http://www.nypost.com/news/nationalnews/65321.htm

--------------

Hmmm, I wonder...

The Washington College of Law is not exactly known as a bastion of conservative thinking. The school offers symposia on such topics as:

~ King George or Commander in Chief? President Bushs Theory of Executive Power and the Constitution,

~ War Crimes, Military Accountability and Civilian Contractors,

~ The Washington College of Laws Feminist Roots: How Far Have We Come? A Focus on Feminist Legal Pioneers Today,


and more.

I wonder if Ms. Martin might be anti-death penalty and (ab)used her powers as an attorney involved in the case to kill the chances for the death penalty to be applied?

Any opinions?

Elliot

tomder55 answered on 03/16/06:

she violated the judges instructions .She should be slapped with a contempt citation .

But if this case is not a posterboy for the need for cases involving al-qaeda to be heard by military tribunal I don't know what is . Judge Leonie M. Brinkema ,being a Clintoon appointee most likely would not have issues a death penalty anyway .

ETWolverine rated this answer Excellent or Above Average Answer

Question/Answer
excon asked on 03/15/06 - The Oscars


Hello Politicos:

What is the enduring legacy of the second place finish of Brokeback Mountain?

It's ok to come in number two.

excon

tomder55 answered on 03/15/06:

thanks Bill Maher. Novelist Annie Proulx is not distinguishing herself with her whining about Brokeback Mountain not receiving the Best Picture Academy Award this year, especially since she didn't even write the script, only a short story on which it was based.

excon rated this answer Excellent or Above Average Answer
Itsdb rated this answer Excellent or Above Average Answer
ETWolverine rated this answer Excellent or Above Average Answer
purplewings rated this answer Excellent or Above Average Answer

Question/Answer
ETWolverine asked on 03/13/06 - For Fredg and others: Some interesting poll numbers for Bush

As per an ABC News/Washington Post poll dated 3/2/06-3/5/06:

"Please tell me whether the following statement applies to Bush or not. . . ."

"He is a strong leader"

Applies: 52%
Does Not Apply: 48%
Not Sure: 0

"He is honest and trustworthy"

Applies: 44%
Does Not Apply: 55%
Not Sure: 1%
-----------------
As per a CNN/USA Today/Gallup Poll dated Feb. 28-March 1, 2006:

"Thinking about the following characteristics and qualities, please say whether you think it applies or doesn't apply to George W. Bush. How about [see below]?"

"Is a strong and decisive leader"

Applies: 52%
Does Not Apply: 47%
Not Sure: 1%

"Is honest and trustworthy"

Applies: 47%
Does Not Apply: 52%
Not Sure: 1%

"Shares your values"

Applies: 45%
Does Not Apply: 52%
Not Sure: 3%

-------------------

From a Time Poll conducted by Schulman, Ronca & Bucuvalas (SRBI) Public Affairs. Feb. 15-16, 2006.

"Do you think that, overall, the Bush Administration has been more honest and trustworthy than most other presidential administrations, less honest and trustworthy, or about the same?"

More 19%
Less 39%
Same 40%
Unsure 2%

(Note that More and Same = 59% vs. 39% who think Bush is less honest.)
------------------------

From a ABC News/Washington Post Poll. Jan. 23-26, 2006:

Please tell me whether the following statement applies to Bush or not. . . ."

"He can be trusted in a crisis"

Applies: 53%
Does Not Apply: 47%
Not Sure: 1%

----------------------

From an CBS News/New York Times Poll. Jan. 20-25, 2006.

"Are you generally optimistic or pessimistic about the next three years with George W. Bush as president?"

Optimistic 52%
Pessimistic 45%
Unsure 3%

"Do you think George W. Bush has strong qualities of leadership, or not?"

Does 53%
Does Not 45%
Unsure 2%

------------------

These do not look like the poll numbers of a man who's approval ratings cannot improve to above 50%.

The public clearly sees Bush as a strong leader, nearly half consider him to be at least as honest and trustworthy as other presidents (outnumbering those who disblieve his honesty by about 20 percentage points), the majority think he can be trusted in a crisis, and the majority of Americans are optimistic about the next 3 years with Bush as president.

That is why I question Fredg's statement that Bush's approval ratings can never again reach 50%. These poll numbers would seem to indicate otherwise, and they happen to be fairly recent numbers as well.

Elliot

tomder55 answered on 03/14/06:

Happy Purim :May you drink until you can no longer distinguish between the phrases "Cursed is Haman" and "Blessed is Mordechai"

JBodine rated this answer Excellent or Above Average Answer
ETWolverine rated this answer Excellent or Above Average Answer
Itsdb rated this answer Excellent or Above Average Answer

Question/Answer
ETWolverine asked on 03/13/06 - Got the following from KINDJ

An interesting e-mail:

Live in the country you support

Try Doing This...

Try driving around as a Gringo in Mexico with no liability insurance and have an accident...

Enter MEXICO illegally. Never mind immigration quotas, visas,international law, or any of that nonsense...

Once there, demand that the local government provide free medical care for you and your entire family...

Demand bilingual nurses and doctors...

Demand free bilingual local government forms, bulletins, etc...

Procreate abundantly. Deflect any criticism of this allegedly irresponsible reproductive behavior with, "It is a cultural United States thing. You would not understand, pal."...

Keep your American identity strong. Fly Old Glory from your rooftop, or proudly display it in your front window or on your car bumper...

Speak only English at home and in public and insist that your children do likewise...

Insist that all products' labels, owner's manuals, instructions, etc., be written in English as well as Spanish...

Demand classes on American culture in the Mexican school system...

Demand a local Mexican drivers license. This will afford other legal rights and will go far to legitimize your unauthorized, illegal presence in Mexico...

Insist that local Mexican law enforcement teach English to all its officers.

Good luck! Because it will never happen. It will not happen in Mexico or any other country in the world... except right here in the United States... Land of the naive.

If you agree, pass it on. If you don't, go ahead and try the above in Mexico or Iran, or just about any other country in the world, for that matter.

Words of Wisdom: "Support the country you live
in...or live in the country you support"

---------------

I take no responsibility or credit for authoring the above statement. I just thought it was interesting enough to post here and offer up for discussion.

Elliot

tomder55 answered on 03/13/06:

or do this



in one of Mexico's largest cities

http://www.postchronicle.com/chronic/archives/2006/03/illegals_stick.shtml

ETWolverine rated this answer Excellent or Above Average Answer
Itsdb rated this answer Excellent or Above Average Answer

Question/Answer
BeelzeBUSH asked on 03/13/06 - Ports, roads, self-respect; it's all on sale

A friend of mine emailed me this article from an Ohio newspaper. I think it is worth a read. I think it speaks to the concerns of most Americans regardless of our political voting habits. In my opion he article is a bit over simplified, but it hits on key issues that will have to addressed by all candidiates in future campaigns. Comments welcomed. George


-------------------------------------------------------------------------------

"By Dick Feagler



Are we so inept or so shortsighted that everything here is up for grabs? Even the ground beneath our feet?

It seems as if America is running a going-out-of-business sale, cutting madcap outsourcing deals that shrink us.

The problem with the Indiana Toll Road thing is obvious. Indiana doesn't know how to run a toll road. Otherwise, it would run its own turnpike within its budget.

Advertisement





But it can't seem to do that. So a long global handshake between Spain and Australia wants to make a chunk of Indiana a kind of colony. If that's not a wet smack on the Hoosier puss, I'm dreaming.

I hope I'm dreaming, because if our ports are run by other countries, and if a 157-mile stretch of America is in control of two other countries, where does that leave us?

It's bad enough that we can't control illegal immigrants busting through our borders. It's bad enough that we are outsourcing good jobs overseas.

Where is our part in this global economy? The truth is, we haven't got a clue.

Once we thought we knew the world and our place in it. We knew our country's history. We knew our shape and size. First we had 13 states, and now we have 50. Immigrants flocked here and they all got to know America.

Often we broke our boundaries and ventured across the oceans to help the rest of the world. We left white gravestones in remote places. And then, after World War II, we helped Europe and Asia get back on track. There is no country as generous as this one.

The mistake we made was thinking we could retreat again to our fortress between the two oceans. Time caught up with us.

The global economy doesn't look so global from here. From here, it looks like we aren't reaping any profit. The profits seem to be going into the bank accounts of foreign nations.

At the moment, our big export is our Army. We are exporting kids overseas to lethally mix about in strange cultures, while here at home, the old idea of America is melting like a cake of soap.

If Indiana, a heartland state, is willing to give up a piece of its motherland, maybe that means none of us has firm ground under his feet anymore.

What scares me is that I think that's exactly what it means."



tomder55 answered on 03/13/06:

Most toll plazas in my opinion should be bull-dozed. Once the highway /bridge /tunnel is paid for then maintainance costs should be easily handled by state taxes .Tolls are the least efficient, most expensive, most polluting, and most aggravating way to pay for a road. Tolls are dangerous . Accidents are over 4 times more likely to occur near toll plazas.Countless hours and gas are wasted every year by commuters waiting on line at toll plazas. But most important; tolls do not efficiently fund highway maintainance. Instead ;tolls mostly fund patronage jobs and the jobs of the toll collectors .

Anyway ;off my soap-box . Foreign companies running Indiana toll roads is not a national security issue .


I notice Dick Feagler made reference to the Marshall Plan . But he missed an essential point . The plan was not totally an effort to help Europe and Asia get back on track. Besides looking at it in the context of an emerging Cold War with the Soviets ,it also was very beneficial for US industry .It was not a transfer of cash .It was a transfer of goods and services . Since most European currencies could not pay for it the American gvt. paid the American companies involved in the program .This helped America ease from a war-time economy to a peace-time economy. The US had a tremendous advantage in the world economy after that for many years .The years fedg is refering to were mostly those years when the US was the only game in town . I heard no American complain when American companies set up shop in Europe and Asia and ran parts of their critical infrastructure . Unfortunately it was unrealistic to expect the US to maintain that dominance .

Globalization is not new. It has been here for a long time. Europe used to think they were the center of the world until Marco Polo went to China and brought to Europe goods they had never had the opportunity to obtain before .European suddenly developed tastes for all things Oriental, such as tea, spices, herbs, gunpowder, and technologies. They benefited from new trade routes to China ;but became more effected by events in other parts of the world for the first time . Before Marco Polo would any European really care if the Muslims shut down the trade routes in Afghanistan ?But once they developed a taste for Chinese goods it became a big issue . That is why the explorers tried to find an alternate sea route to China. Thus ;the discovery of the Americas.

The net effect of globalization have generally been good for America . Is there any real remorse that shoes and clothing are not made in the US ? Americans love to go to the retail stores that sell these items at inexpensive prices ...more value for the buck. The same will be said for the next generation of the so-called outsourcing .

Yes ;there are always workers left behind. At the turn of the 19th century you could make a living shoeing horse . It wan't that many years ago that an American had a career for life at major industies as a draftsman ;now computers do in minutes what it took weeks for the draftman to accomplish .The dynamics of the work place always changes . For those who get displaced they need to take the initiative to re-educate themselves and learn a marketable skill. The government should and does provide safety nets for a transition period.

The way Dick Feagler talks you would think that unemployment is at 25-30% .Instead unemployement was at 4.7 percent. If there were no jobs here than why would all those illegals be busting through our borders ? The world market is not a fixed piece of pie to divide up ;it is expanding ,and expanding rapidly. It is up to Ameican policy makers to best take advantage of it . America first is all well and good but it should not be mistaken for America only.

BeelzeBUSH rated this answer Excellent or Above Average Answer

Question/Answer
sissypants asked on 03/10/06 - the deal is dead

With the arab company pulling out because of the political unrest in this country over the whole thing, Mr. Bush got on tv and said his administration will continue to work closely with the congress to try to educate them as to why he felt and still feels it is safe to have an arab company in charge of 6 u.s. ports. i feel he should be focusing more on convincing the people of the united states, not just the congress. then maybe even thou i still think it was a bad idea, then maybe his ratings wouldn't be so low. he still seems so arrogant to me. any comments?

tomder55 answered on 03/10/06:

some say he is arrogantand has a tin ear when people offer advice ;others say he is manipulated by those around him. some call him stupid ;others call him a Machiavellian genious . I think he believes what he says .

He probably thinks that the port deal was a good deal for the US . I wonder if his political instincts are where they should be sometimes ,but more likely he becomes convinced that the course he is taking is the correct one and has the courage of his convictions .It actually takes courage to make decisions and say to hell with the consequences . Look at the Clinton's perfomance during the port controversy and what you saw as an alternative is classic triangulation . Evita was all over the place opposing it while Bill and other Clintonistas were advising UAW as paid lobbiests . The question is :is it better to have a President who leads and stands firm ;or to have a President who leads by the result of the most recent polling #s ?

You are 100% correct however that if he wanted to get the deal done he needed a major p.r. push to convince us it is in our interest. I think he found a face saving way to 'cut bait' on this .

The deal on the surface was much more complicated then an issue of Arabs running the ports . A company owned by the Chinese military runs many US ports on the West Coast. US companies a long time ago got out of the port management business ,and many of the top 5 foreign companies will not touch US ports because the unions and their rules make the ports inefficient and too costly to run. Of the top 10 port management companies only 2 American companies are listed .

excon rated this answer Excellent or Above Average Answer
fredg rated this answer Excellent or Above Average Answer
sissypants rated this answer Excellent or Above Average Answer

Question/Answer
JBodine asked on 03/10/06 - Maybe you can help

You may know my brother in law, goes by the name of kindj.

Well, ol' DK got suspended. Anybody out there got any idea 'bout how that happened? Seems crazy to me.

Anyhow, he and I are close, almost like one person. He said if y'all wanted to talk to him, give him a shout at:

kindj@navyseals.com

JBodine, brain surgeon, OUT!

tomder55 answered on 03/10/06:

He got caught up in alot of nonsense that he was not really involved in . I do not remember the exact wording of a posting he did on the Christianity board (and it has since been deleted ) ;but management is very sensitive at this time about experts posting off topic questions or questions about other experts . There was alot of silly sniping going on and management adopted a no tolerance policy .In their zeal to clean up that board they have not done a good job in seperating the wheat from the chaff in my humble opinion. Many of the experts suspended did not deserve it . DK is definitely one of these. Say hello for me .

excon rated this answer Excellent or Above Average Answer
JBodine rated this answer Excellent or Above Average Answer

Question/Answer
Itsdb asked on 03/09/06 - DP World to Transfer Control of U.S. Ports to U.S. `Entity'

March 9 (Bloomberg) -- Dubai-owned DP World, facing congressional opposition to its operation of six U.S. ports, will transfer its ownership of those terminals to a U.S. entity, Virginia Senator John Warner said.

Warner, reading a statement from Edward Bilkey, the chief operating officer of the company, said the sale was intended to preserve ``the strong relationship between the United Arab Emirates and the United States.''

DP World, which completed its acquisition of U.K.-based Peninsular & Oriental Steam Navigation Co., said it ``had decided to transfer fully the U.S. operation of P&O Ports North America to a United States entity,'' Warner said, reading from Bilkey's statement.

The company planned to make the transfer ``in an orderly fashion'' so it ``will not suffer economic loss,'' the statement said.

The decision was announced as the congressional Republicans told President George W. Bush today that Congress was prepared to pass legislation to prevent the company from operating the U.S. port terminals, a congressional aide said.

~~~~~~~~~~~~~~~~~~~~~~~~~~~~~~~~~~~~~~~~~~~~~~~~~~~~~~~

Pending notice of what 'U.S. entity' we're talking about it looks like DPW has made a move that should please the critics...and hopefully pulled the rug out from under a whole slew of opportunistic politicians.

Steve

tomder55 answered on 03/10/06:

Bush received some temporary short term damage in the party that he will more than make up in the coming campaign season where he excels.

The Dems. drift to the right on national security will come to a screeching halt when they try to revive the move for hearings on the NSA monitoring of terrorists . But that will be turned round on them if the hearings become "who leaked sensitive security operations information to the press " .Besides it looks like the Administration and the Senate are hammering out a compromise on that issue also .

The Washington Prowler ;a blogger at American Spectator made this observation

We're already hearing rumors on Capitol Hill that one of the companies that will be interested in the Dubai Port properties once divestiture begins is ... Halliburton.

Makes sense. They are one of the few U.S. firms that could move in and handle the types of operations required.

But wouldn't that just be richest outcome? Democrats will just be spitting mad. And the MoveOn.org and Daily Kos types will be screaming that this whole thing was nothing more than a wiley Rovian plot to get Halliburton the deal.


I think Elliot called it a Harriet Meyers gambit . That bumbling Bush becomes the maniacal genious once again.

Itsdb rated this answer Excellent or Above Average Answer

Question/Answer
Erewhon asked on 03/09/06 - US Ambassador says Iraq could still descend into civil war. Disturbing events reported.





By Lutfi Abu Oun - Wed Mar 8, 4:45 PM ET

The bodies of 18 men, bound, blindfolded and strangled, were found in a Sunni Arab district of Baghdad, apparent victims of sectarian turmoil gripping Iraq and threatening the formation of a coalition government.

Iraq's Shi'ite interior minister, a hate figure for many Sunnis who accuse him of condoning death squads, escaped an apparent assassination attempt when a roadside bomb blasted his convoy. Minister Bayan Jabor, however, was not in his car.

In its annual report on human rights abuses worldwide, the U.S. State Department said reports increased in 2005 of killings by the U.S.-backed Iraqi government or its agents and members of sectarian militias dominated many police units.

"Police abuses included threats, intimidation, beatings, and suspension by the arms or legs, as well as the reported use of electric drills and cords and the application of electric shocks," the State Department said of Iraqi human rights three years after U.S. troops invaded to overthrow Saddam Hussein.

About 50 Iraqi private security guards were seized at their compound by men in police uniform on Wednesday -- but Interior Ministry officials said they were unaware of any formal arrests.

The bombing of an important Shi'ite shrine in Samarra on February 22 has pitched Iraq toward civil war, unleashing reprisal sectarian killings and deepening the mutual suspicion between the country's majority Shi'ite Muslims and minority Sunnis.

The violence has complicated faltering efforts to form a government of national unity three months after elections. Iraqi leaders, struggling to agree on who should hold the top posts, are due to meet President Jalal Talabani on Thursday to decide on a way forward. Parliament is supposed to meet by Sunday.

The dumping of bodies bearing signs of torture and killed execution-style is a feature of the violence.

The 18 bodies discovered by U.S. troops in western Baghdad late on Tuesday had all been garroted and had their hands bound with plastic ties, police and hospital officials said.

The victims, a mixture of middle-aged and young men in civilian clothes, carried no identifying papers, police said.

A policeman at the Yarmuk hospital morgue pointed to their clothing and long hair as an indication some may have been religious extremists linked to al Qaeda. Reuters reporters who saw the bodies said many appeared to be Iraqis.

Police sources said only one had so far been identified by a relative. He was a guard at an oil refinery in southern Baghdad.

The policeman at the hospital said many of the bloodied bodies appeared to have been beaten while some had small burn marks, suggesting they were tortured before being killed.

Senior officials, aware of the potential for sectarian anger if it becomes clear all are either Sunni or Shi'ite Muslims, made no formal comment on the religious identities of the dead.

Iraqi police said the bodies were dumped near the Amriya district, a stronghold of Sunni insurgent groups.

MINISTER'S CONVOY ATTACKED

Sunnis have accused the Shi'ite-led government's police and other security forces of abducting and killing Sunni civilians -- an accusation Interior Minister Jabor and the police deny.

Interior Ministry vehicles normally used to transport Jabor and his aides were attacked as they left the ministry on Wednesday. A roadside bomb destroyed one car in the convoy, killing two and wounding five, a police source told Reuters.

It follows the assassination of the top Iraqi general in Baghdad, a Sunni, by a sniper in the capital on Monday.

The U.S. commander in Iraq, General George Casey, described Major General Mubdar Hatim al-Dulaimi at his funeral as "a courageous soldier, a passionate leader and an Iraqi hero."

More than 500 people have been killed since the Samarra bombings, according to the most conservative official figures.

Despite the daily bombings and shootings there is a relative lull in the violence and officials have said the immediate crisis seems to be over -- for the time being at least.

But the U.S. ambassador conceded on Tuesday Iraq could still descend into civil war, saying Americans "opened Pandora's Box" when they toppled Saddam in 2003 and another incident like that in Samarra could push it to the brink of war again.

Eight people, including four policemen were killed in bombings in Baghdad and the western town of Falluja on Wednesday. The bodies of two people were found bound and blindfold and shot dead in eastern Baghdad, police said.

In political negotiations, Sunni and Kurdish parties refuse to accept Shi'ite Prime Minister Ibrahim al-Jaafari should stay on. His critics say he has failed to bring security or prosperity during the year in which he has been interim prime minister.

(Additional reporting by Faris al-Mehdawi, Mariam Karouny and Aseel Kami)

===
What can/should the US do to avert civil war in Iraq?

tomder55 answered on 03/09/06:

a Full civil war certainly is not in the Sunni interest .At 20 % of the population and having no territory with revenue resources in it ;the Sunnis would lose badly in short order .Do not expect the Shia to show the same restraint that coalition forces have .
The question then becomes;Why would the Sunni instigate a civil war they have no chance of winning when they are making nice strides since they decided to participate in the national politics ? The obvious answer is that the Sunnis are NOT instigating this fight .

But what if you had a civil war and nobody came ? Doesn't civil war require the participation of a large portion of the population? Yes there is violence there and indeed several hundred have been killed . It is not nearly as bad as what I thought it would be after the bombing of the gold dome. I just wounder what would happen in this country if a rival religion decided to take out St.Patrick's Cathedral ?? Given the provcations I think the Shia (except al-Sadr)have been remarkably restrained .


Here is what an Iraqi cameraman working for NBC news observed in Blogging Baghdad :The untold Story :

But you know, the direction of the anger has changed. A year ago everyone was angry at the Americans. Everyone thought they were responsible. But that is not the message I am getting on the street now. People know these attacks are being carried out by the extremists.

.In the Sunni triange the tribes are already ejecting al-Qaeda terrorists of Zarqawi from their towns .I have it from some pretty good sources that Iran may be responsible for the dome attack . Rumsfeld said Iranian Revolutionary Guard soldiers are slipping into Iraq and I believe that to be the case . I also agree with him when he says that if indeed there were a civil war that the Iraqi security forces should deal with it .

But that is not what I think is going down .I think the infiltration of the Iranians is to contine to foment ethnic strife . But more importantly ;Iran is not going to wait around to be attacked . I think they are there to disguse themselves as Shia "insurgents " . When the time comes they will attack US troops and instigate riots . This will be the "Tet Offensive "moment that the MSM will dutifully report as a great defeat for America. They are hoping for a repeat of what happened after Tet in the US . That is what I think is really happpening in Iraq today .




Erewhon rated this answer Excellent or Above Average Answer
Mathatmacoat rated this answer Excellent or Above Average Answer

Question/Answer
fredg asked on 03/09/06 - American Ports; no take-over

The American People have spoken; through emails, phone calls, letters to Congress. Foreign take-over of American Ports is dead. Republicans added an Amendment to the current Iraqi spending bill, stating NO take-over of American Ports by a Foreign Country.
Republican voting was something like 62 - 2 in favor of not letting this happen.
Democrats were already opposed, naturally.
President Bush was ready to OK the take-over; using "trust in this foreign country" and "good will" as reasons. Main unspoken reason was the hugh amounts of oil money that are involved with this foreign country.
The President can't Veto the Bill, cause he would be vetoing his own Iraqi spending Bill.
Congressional elections are this year, and Republicans in Congress don't want to lose their seats; more important than being loyal to their President.

Do you agree or disagree with this take-over being stopped?
fredg

tomder55 answered on 03/09/06:

1st ;I do not like riders and I am glad that Congress is considering a line item veto. It is a sneaky way to get earmarks and other provisions passed that would not stand up on their own weight .

2nd ; I am opposed to the deal as it currently stands but if Bush can get DPW to agree to get an American affiliate co. to do the operations like he suggested to them then I will flip on this issue.

The atmosphere has gotten ugly . I opposed it for national security reasons ;not for protectionism . I think the tone towards protectionism that this has brought is equally if not more so fraught with danger as this deal potentially is . We have to very wary of repeating the mistakes of over -reaction like the Smoot Hawley Act of 1930.

There is more than one economist who credits that act even more than the stock market crash of 1929 with causing the gereat Depression . Even worse ; As countries resorted to protectionism, the general amount of international trade radically decreased, causing the world economy to slow and the economic conflicts that resulted were one of the leading causes of WWII .So I would tread towards protectionism with caution .

I think this is a long overdue debate in this country .It is an issue for national security for sure ;but we can get carried away .The world economy is here to stay and we just have to adjust to that reality . But stifling debate by this sneaky ploy is unworthy of the so called Statesmen who we elect .


fredg rated this answer Excellent or Above Average Answer

Question/Answer
kindj asked on 03/07/06 - Oscar acceptance speech we'd like to hear:

March 7, 2006
Speech We'd Like to Hear From an Academy Award Winner
By Dennis Prager

Here's a speech we would like to hear from an Academy Award winner:

I thank you for this wonderful award. Receiving an Academy Award gives the recipient an almost unique opportunity to speak to hundreds of millions people around the world, so I would like take this once-in-a-lifetime moment to say this:

First, I want to thank my country, the United States of America. Every one of us here has this country to thank for enabling us to live lives of unprecedented freedom and unimaginable affluence. Too many of us forget that no other country in history has offered such opportunities to people in our profession or in any other profession, for that matter.

Second, I want to thank the men and women of the armed forces of the United States. While we bask in freedom and spend a good part of our lives going from party to party and award show to award show, tens of thousands of my fellow Americans are confronting a menace to our world as great as that fought by previous generations fighting Nazism and communism.

At the same time, I also want to apologize to these troops for my profession not having made even one motion picture about any of the heroic American fighters in Afghanistan and Iraq. This country is fighting a war, Hollywood. You may think this war is unwise, waged under mistaken, or even false, pretenses. And as an actor in Hollywood, you are overwhelmingly likely to hate this commander in chief. But even the men and women of Hollywood must recognize that America is fighting the worst people of our time, people who hurt every group Hollywood claims to care about -- minorities, women, gays -- people who engage in the sins Hollywood most professes to oppose -- intolerance and violence -- far more than anyone else on the planet.

In another era, when what many have labeled "the greatest generation" fought the German Nazis and the Japanese fascists, Hollywood made movie after movie depicting that great war and our great warriors. And Hollywood showed freedom's enemies as the cruel and vicious people they were. We have not produced one film yet depicting this war in positive terms or one depicting this generation's enemies of freedom as the cruel and vicious people they are.

In fact, the only nominated film about people who slaughter children at discos, blow up weddings, and bomb pizzerias and buses filled with men, women and children is one that attempts to show these murderers in God's name as complex human beings. Just imagine how the Academy would have reacted 60 years ago to a film depicting Nazi murderers as complex human beings. We have descended far.

We in Hollywood walk around thinking we are very important. That is why this year's nominated films for best picture are largely pictures with messages, pictures that relatively few people actually see. But although Hollywood was always concerned with politics, we have let ourselves be taken over by those for whom their message is more significant than the primary purposes of film -- to illuminate life and to entertain. Yes, entertain.

You know, entertainment is actually a noble pursuit. Life is difficult for almost every human being on earth. And if we can offer people an elevated way to divert their attention for a couple of hours from their troubled child, their marital tensions, their ill parent, their financial woes, we have rendered the world a greater service than by making another message-film against racism in America, the least racist country in the world.

My fellow actors, we walk around feeling that we are very important. But we do so only because we confuse fame with significance. We do have more fame than any other human beings in history. Far more people have heard of any actor here tonight than of any of the discoverers of any medication saving billions of lives, of any teacher of the disabled, of any nurse tending the aged, of almost any national leader.

But the truth is that, as noble a calling as acting can be, all we do is make-believe: We portray other people, and we speak words written by other people. Everyone knows our names, but almost no one knows us. All they know are the characters we play.

Thank you again. I hope I haven't ruined your evening.

Copyright 2005 Creators Syndicate

~~~~~~~~~~~~~~~~~~~~~~~~~~~~~~~~~~~~~~~~~~~~~~~~~

Any bets on how long it'll be before we hear that one?

DK

tomder55 answered on 03/08/06:

you could tell how out of touch they were when for the most part they did not get Jon Stewart's jokes (especially when he was panning Hollywood itself of which he said it is a a moral black hole where innocence is obliterated in an orgy of sexual gratification and greed [pauses for effect ] I dont really have a joke here. The audience was stunned ) .

He did an imitation of Howard Dean at one point "usually you have to go to a Democratic fundraiser to see this many Hollywood stars" and saying that
the Oscars are a chance to"see all your favorite stars without having to donate any money to the Democratic party."

Coming back from one break, Stewart pretended to be in mid-sentence. "And that is why I think Scientology is right, not just for this city, but for the country," clearly mocking some stars commitment to Scientology. You could count the cricket chirps after that one .

Instructing the audience to not pirate films, Stewart referred to the rich and lavishly dressed audience and said,"These are the people you're stealing from." He added that they did not even have enough money to buy gowns that cover their breasts .

The one exception of course is when he told a Cheney joke. He said that Bjork, who once wore a swan dress to the Oscars, was unable to attend because she had been shot by Dick Cheney.That brought down the house.

Stewart a progreesive to the core, but a self-deprecating one proved once again that exposing hypocracy is his forte. Too bad he will never be invited back.

Ironically ;the only awardee to thank God was the gangsta-rappers who one an Oscar for a song about pimping.

Itsdb rated this answer Excellent or Above Average Answer
kindj rated this answer Excellent or Above Average Answer

Question/Answer
ETWolverine asked on 03/06/06 - Gotta love Ralph Peters.

Another great article from Ralph Peters.

DUDE, WHERE'S MY CIVIL WAR?
By RALPH PETERS - In Iraq

BAGHDAD

I'M trying. I've been trying all week. The other day, I drove another 30 miles or so on the streets and alleys of Baghdad. I'm looking for the civil war that The New York Times declared. And I just can't find it.

Maybe actually being on the ground in Iraq prevents me from seeing it. Perhaps the view's clearer from Manhattan. It could be that my background as an intelligence officer didn't give me the right skills.

And riding around with the U.S. Army, looking at things first-hand, is certainly a technique to which The New York Times wouldn't stoop in such an hour of crisis.


Let me tell you what I saw anyway. Rolling with the "instant Infantry" gunners of the 1st Platoon of Bravo Battery, 4-320 Field Artillery, I saw children and teenagers in a Shia slum jumping up and down and cheering our troops as they drove by. Cheering our troops.

All day - and it was a long day - we drove through Shia and Sunni neighborhoods. Everywhere, the reception was warm. No violence. None.

And no hostility toward our troops. Iraqis went out of their way to tell us we were welcome.

Instead of a civil war, something very different happened because of the bombing of the Golden Mosque in Samarra. The fanatic attempt to stir up Sunni-vs.-Shia strife, and the subsequent spate of violent attacks, caused popular support for the U.S. presence to spike upward.

Think Abu Musab al-Zarqawi intended that?

In place of the civil war that elements in our media declared, I saw full streets, open shops, traffic jams, donkey carts, Muslim holiday flags - and children everywhere, waving as our Humvees passed. Even the clouds of dust we stirred up didn't deter them. And the presence of children in the streets is the best possible indicator of a low threat level.

Southeast Baghdad, at least, was happy to see our troops.

And we didn't just drive past them. First Lt. Clenn Frost, the platoon leader, took every opportunity to dismount and mingle with the people. Women brought their children out of their compound gates to say hello. A local sheik spontaneously invited us into his garden for colas and sesame biscuits.

It wasn't the Age of Aquarius. The people had serious concerns. And security was No. 1. They wanted the Americans to crack down harder on the foreign terrorists and to disarm the local militias. Iraqis don't like and don't support the militias, Shia or Sunni, which are nothing more than armed gangs.

Help's on the way, if slowly. The Iraqi Army has confounded its Western critics, performing extremely well last week. And the people trust their new army to an encouraging degree. The Iraqi police aren't all the way there yet, and the population doesn't yet have much confidence in them. But all of this takes time.

And even the police are making progress. We took a team of them with us so they could train beside our troops. We visited a Public Order Battalion - a gendarmerie outfit - that reeked of sloth and carelessness. But the regular Iraqi Police outfit down the road proved surprisingly enthusiastic and professional. It's just an uneven, difficult, frustrating process.

So what did I learn from a day in the dust and muck of Baghdad's less-desirable boroughs? As the long winter twilight faded into haze and the fires of the busy shawarma stands blazed in the fresh night, I felt that Iraq was headed, however awkwardly, in the right direction.

The country may still see a civil war one day. But not just yet, thanks. Violence continues. A roadside bomb was found in the next sector to the west. There will be more deaths, including some of our own troops. But Baghdad's vibrant life has not been killed. And the people of Iraq just might surprise us all.

So why were we told that Iraq was irreversibly in the throes of civil war when it wasn't remotely true? I think the answers are straightforward. First, of course, some parties in the West are anxious to believe the worst about Iraq. They've staked their reputations on Iraq's failure.

But there's no way we can let irresponsible journalists off the hook - or their parent organizations. Many journalists are, indeed, brave and conscientious; yet some in Baghdad - working for "prestigious" publications - aren't out on the city streets the way they pretend to be.

They're safe in their enclaves, protected by hired guns, complaining that it's too dangerous out on the streets. They're only in Baghdad for the byline, and they might as well let their Iraqi employees phone it in to the States. Whenever you see a column filed from Baghdad by a semi-celeb journalist with a "contribution" by a local Iraqi, it means this: The Iraqi went out and got the story, while the journalist stayed in his or her room.

And the Iraqi stringers have cracked the code: The Americans don't pay for good news. So they exaggerate the bad.

And some of them have agendas of their own.

A few days ago, a wild claim that the Baghdad morgue held 1,300 bodies was treated as Gospel truth. Yet Iraqis exaggerate madly and often have partisan interests. Did any Western reporter go to that morgue and count the bodies - a rough count would have done it - before telling the world the news?

I doubt it.


If reporters really care, it's easy to get out on the streets of Baghdad. The 506th Infantry Regiment - and other great military units - will take journalists on their patrols virtually anywhere in the city. Our troops are great to work with. (Of course, there's the danger of becoming infected with patriot- ism . . .)

I'm just afraid that some of our journalists don't want to know the truth anymore.

For me, though, memories of Baghdad will be the cannoneers of the 1st Platoon walking the dusty, reeking alleys of Baghdad. I'll recall 1st Lt. Frost conducting diplomacy with the locals and leading his men through a date-palm grove in a search for insurgent mortar sites.

I'll remember that lieutenant investigating the murder of a Sunni mullah during last week's disturbances, cracking down on black-marketers, checking up on sewer construction, reassuring citizens - and generally doing the job of a lieutenant-colonel in peacetime.

Oh, and I'll remember those "radical Shias" cheering our patrol as we passed by.

Ralph Peters is reporting from Forward Operating Base Loyalty, where he's been riding with the 506th Infantry Regiment, 101st Airborne Division.


http://www.nypost.com/seven/03052006/postopinion/opedcolumnists/64677.htm

Hmmm... I wonder... could it be that the information we're receiving from the various mainstream news outlets are something less than fully accurate? Nah, they wouldn't do that, would they?

Elliot

tomder55 answered on 03/06/06:

shows the difference between a reporter who gets out of the Green Zone and searches for the truth and the reports who file their leads from "Mahogeny Hill " (the bar top at the lounge of their hotel in the Green
Zone ). Victor Davis Hanson just returned from Iraq and his most recent work confirms what Peters says .

ETWolverine rated this answer Excellent or Above Average Answer
fredg rated this answer Excellent or Above Average Answer
Itsdb rated this answer Excellent or Above Average Answer

Question/Answer
excon asked on 03/05/06 - Bush


Hello experts:

Are you guy's the only ones left who support this guy? I think so.

Given that right now, nobody likes him much, do you think the Republicans will lose, gain, or stay the same in the next congressional election?

excon

tomder55 answered on 03/06/06:

Drgade makes a valid point . Lyndon Johnson had a conversation with Hubert Humphery before the 1964 election. The conversation was caught on tape and were transcribed at the U. of Va. ["The Presidential Recordings, Lyndon B. Johnson: The Kennedy Assassination and the Transfer of Power, November 1963-January 1964"]. Here is the pertinent portion:

"The St. Louis Post-Dispatch man asked me today, said, 'What Republican frightens you the most?' I said, "Well not any of them now. When one of them comes out and said, 'This is what I stand for' and gives me a foreign policy and said, 'Here's what I'm for' and then he gives me a domestic policy and said, 'Here's what I'm for. Here's what I'm for in the field of education, in the field of manpower, in the field of training, in the field of civil rights.' Then I'll get scared. Because even if it's different from mine, they'll have a choice. But now they [the voters] haven't any choice, because we're the only ones that stand for anything. They're just against things. . . . And it just makes these guys look silly."

Do today's Democrats offer voters an alternative or are they "just against things?" The later appears to be the case .

Do I think there is a chance for a Speaker of the House Nanci Pelosi ? [gag} Yes ;but there are reasons why this is a remote possibility .

First ; there is an arguement that the Presidents polls are as low as they can go and there is plenty of time before Nov. for a rebound .

Second ;the economy is sound .Third ;many of the Republicans will not hitch their wagon on the President's fate and are forming more independent positions ...note how many oppose the port deal .

Third ;for years the Democrats were in charge of redistricting ,but for the last decade the Republicans were able to redistrict many key areas .

Fourth ;the rule of incumbency still applies .

Yes the Democrats will make gains but I do not believe they will achieve majorities in either house . Where Democrats will probably claim their biggest gains will be in the State Govenorships ...and as you know putting a Govenor on the Presidential ticket is the best chance to win the Presidency .

oh yeah ...and what is the Democrats position ? Let me quote the Democrat Party Chair Howard Dean; because Dean perfectly encapsulates the state of today's Democrat Party.[from 'Meet the Press']

" "Right now it's not our job to give out specifics. We have no control in the House. We have no control in the Senate. It's our job is to stop this administration."



excon rated this answer Excellent or Above Average Answer

Question/Answer
Erewhon asked on 03/04/06 - Bush Reveals USA's New Secret Policy For Al Qaeda!


Speaking in Pakistan, Bush said in essence that his policy for dealing with Al Qaeda is to:

1. Find then
2. Charge them.

Now he knows how to deal with them their days are numbered!



What a masterful plan, eh?

tomder55 answered on 03/05/06:

say what you will ;the Bush Doctrine will be ;just like the Truman Doctrine during the Cold War ;the model that will be used for fighting the war against Jihadistan for the next 50 years .

Erewhon rated this answer Excellent or Above Average Answer
Itsdb rated this answer Excellent or Above Average Answer

Question/Answer
jackreade asked on 03/04/06 - No Exit, No Hope?

The current Iranian government, which is much more hard line than the reformists it ousted, is defiant in the face of possible U.N. sanctions aimed at stopping Iran's nuclear ambitions. Iran frightens the West. It is on the verge of forming a powerful Shiite alliance with Iraq and together cornering almost as much oil as Saudi Arabia.

It is a rich enough country to purchase an atomic weapon if the West prevents it from building one. It hates Israel to the point that a nuclear stalemate could be in the offing some time down the road.

And yet for all our apprehension, the Iranians aren't madmen. They are confronting the world with questions that were bound to come back to haunt us. As seen from the Iranian perspective, the leading questions are as follows:

Why should Pakistan have the bomb and we can't?

Why does Israel deserve nuclear power without international sanctions?

What gives foreign powers the right to make demands on a sovereign nation?

Who says that our culture must conform to Western liberal democracies?

The moderates and reformers in Iran cannot stem the tide of public opinion at home, because the regime has successfully turned the tables on the West by making the nuclear issue a matter of nationalism, pride, and sovereignty. They are also turning our own values back on us, since the U.S. would never allow the international community to determine its nuclear future and staunchly forbids international opinion from altering its policies (as witness our refusal to participate in the World Court tribunals over war crimes, our refusal to ratify the Kyoto accord on global warming, and our pre-emptive invasion of Iraq).

The chickens have come home to roost with a vengeance.


There is no way more American violence will improve the situation in the Middle-East; America must face facts. We are far worse off than we were on 910.


Jack

Debate invited.

tomder55 answered on 03/04/06:

I disagree with the first contention that Iran is forming an alliance with the Shia in Iraq. I see evidence of some ccoperation with al-Sadr but he hardly represents the majority of the Shia population . Grand Ayatolla Sistani calls the shots and he is fundamentally at odds with Khomeinism .If anything ,Iraq being free from the noose of Saddam has given Sistani an opportunity to renew contacts in Iran and influence their politics.Therefore a free Iraq poses a great threat to Iran .

And yet for all our apprehension, the Iranians aren't madmen

They are not but their luntaic leader Mahmoud Ahmadinejad is a madman . I think he like Hitler has to be taken at his word. When he says that he will use an Iranian nuke to wipe Israel off the map ;he believes it . Add to it his philosophy of preparing the way for the return of the 12th Iman and you see the makings of a messianic madman shortly aquiring the ultimate weapon.

Iran has also made great strides in medium and long range missle development . That makes them a geopolitical threat . Even France has determined that a nuke in their hands is intollerable. Russia has failed in their efforts to negotiate this crisis away . They have a great financial stake in Iranian nuclear development and yet they are on the verge of abandoning Iran on this issue. Can anyone seriously say that a nuke in the hands of Israel poses such a threat ?

The moderates and reformers have not had the support from the west that has been needed .That will possibly change as Condi Rice has a new intiative to engage with them. Akbar Ganji the jailed dissident is scheduled to be released mid-March .An indicator of Rice's seriousness on this will be if the ?US leads international pressure to secure his release . It is my opinion that Ganji is the only opposition leader in Iran that could unify the opposition .Currently it is fractured with unions staging rallys one week and students protesting another.

I'd say the US is finally becoming true to it's values .For too long we gave lip service to democracy when in truth our policy was 'democracy for me but not for thee'.Our realpolitk thinking allowed us to play footsie with some of the most brutal dicators of the 20th century in the persuit of stability . Now there is a fine legacy for you .I'm much more comfortable with the US at least making an attempt to free the oppressed people of the region .I think we are acting truer to our values now than then.I do not understand the left frankly . There was a time when I was a Democrat and they used to believe in "liberation" .What happened ? When did the left become isolationists ?

jackreade rated this answer Excellent or Above Average Answer

Question/Answer
jackreade asked on 03/04/06 - All Out War in Middle East

"A civil war in
Iraq could lead to a broader conflict in the Middle East, pitting the region's rival Islamic sects against each other, National Intelligence Director John Negroponte said in an unusually frank assessment Tuesday.


"If chaos were to descend upon Iraq or the forces of democracy were to be defeated in that country ... this would have implications for the rest of the Middle East region and, indeed, the world," Negroponte said at a hearing of the
Senate Armed Services Committee."


I'm having trouble imagining what an all out war war in the Middle East would be like, should it happen. I don't think that all that many people would be involved. What is your take on this?

Jack

tomder55 answered on 03/04/06:

I think I was making a simular point when I said that a civil war could happen if we were to withdraw too soon before the Iraqi gvt. was established and the security forces were completely trained.

I still do not see that dire scenario anyway. I find it hard to see an ideology behind the *insurgency* except for perhaps nihilism ;and opportunism from surrounding states .

The future that Zarqawi promises in his intecepted letters is that of a caliphate of repression simular to the Taliban model .The Sunnis have rejected this except for Baathist remnants who's only future they see is in a reconstituted Baathist dictatorship. I think when Saddam is seen swinging in the breeze perhaps they will see the futility of their persuit. I do not know what the future brings to Iraq ...but a reconstitued Baathist regime I definitely do not see.

Sunnis have in many cases turned on the foreign Zarqawi terrorists . There have been repeated reports of Sunni tribes battling al-Qaeda in the Sunnio triangle.

The incidents of attacks on coalition forces is considerably down . They have come to realize we can't be forced out . So their strategy is to instigate factional violence by attacking Shia .The Shia are the majority and except for clients of Iran like Mookie al-Sadr ;they are hitching their fortunes on a democratic state . The Kurds have taken all necessary steps to secure their interests either in a federal Iraq or a seperate Kurdistan.

The message out of both Sunni and Shia mosques are consistent . They do not blame each other ;but they need to blame someone evidently so they blame the US and Israel for everything . Nothing new there.

The islamic sects have for centuries acted like the Crips and Bloods ;and there have been violence throughout Muslim history .That is how I see the sectarian violence; as gangland . Is there a scenario for all out war ? Yes ;but not the way Negroponte says . The instigator of all out war will not be Iraq but the ambitions of Iran .

jackreade rated this answer Excellent or Above Average Answer

Question/Answer
HerrAirhorn asked on 03/04/06 - The Sky is Falling...

The sky is falling. No, the oceans are rising. The latest on the effect of Glabal Warming.

"The Antarctic ice sheet is losing as much as 36 cubic miles of ice a year in a trend that scientists link to global warming, according to a new paper that provides the first evidence that the sheet's total mass is shrinking significantly.

The new findings, which are being published today in the journal Science, suggest that global sea level could rise substantially over the next several centuries.

It is one of a slew of scientific papers in recent weeks that have sought to gauge the impact of climate change on the world's oceans and lakes. Just last month two researchers reported that Greenland's glaciers are melting into the sea twice as fast as previously believed, and a separate paper in Science today predicts that by the end of this century lakes and streams on one-fourth of the African continent could be drying up because of higher temperatures.

The new Antarctic measurements, using data from two NASA satellites called the Gravity Recovery and Climate Experiment (GRACE), found that the amount of water pouring annually from the ice sheet into the ocean -- equivalent to the amount of water the United States uses in three months -- is causing global sea level to rise by 0.4 millimeters a year. The continent holds 90 percent of the world's ice.


Will candidates for federal office have an anti-Global warming in their planks soon in order to gt elected?

tomder55 answered on 03/04/06:

Yes I believe some of them will ;certainly Al Gore is going to make a run for the Presidency and he will have a movie out in May about the Global Warming issue .His movie was a hit at the Sundance Festival . But you know that all of it is platitudes. I happen to think that climate is cyclical . Sure it is ok to address spewing pollutants into the air .But where they lose it is when they mask it in hysterical doom and gloom . Humanity has lived through climate changes in the past and we will adjust again . Meanwhile ...sell your beach front property while you have a chance.

fredg rated this answer Excellent or Above Average Answer
HerrAirhorn rated this answer Excellent or Above Average Answer

Question/Answer
Mathatmacoat asked on 03/03/06 - A bit of a rev up for the UN

or is it where old politicians go to die?

Senator drops UN penny

03mar06

THE worst-kept secret in Canberra may finally have been exposed -- retiring Liberal senator Robert Hill will be the new ambassador to the United Nations.


Senator Hill spent his last day in the Senate yesterday after 25 years.

Liberal senator John Watson told the chamber, "I believe he will also put his interests and skills to good use in his new role and, hopefully, in reforming and rejuvenating the United Nations."



tomder55 answered on 03/03/06:

probably a good place for a former defense minister . Hope he becomes a working compadre with US Amb. Bolton . He sounds like he has the mettle for the job.

Mathatmacoat rated this answer Excellent or Above Average Answer

Question/Answer
Mathatmacoat asked on 03/03/06 - Alien alert

According to this article the aliens have landed and it has repercussions in teh eductaion system


http://www.newswithviews.com/Wooldridge/frosty131.htm

tomder55 answered on 03/03/06:

yes . this is one of the understated unintended consequences of a defacto open border policy . It affects healthcare in this country as well.

Can you imagine such a failure rate across the country? Can you imagine the consequences of an illiterate generation leading this Republic into the 21st century? This is exactly what is happening . every honest teacher in America can vouch that this is not just local to the Denver area but a national condition .There is no amt. of tax increases that will solve it .

Mathatmacoat rated this answer Excellent or Above Average Answer

Question/Answer
Itsdb asked on 03/02/06 - This is just wrong

Teacher's anti-U.S. diatribe caught on tape
America 'probably the single most violent nation on planet Earth'

Posted: March 2, 2006
2:00 p.m. Eastern

© 2006 WorldNetDaily.com

A high school teacher caught by a student on audiotape in an anti-U.S. and anti-capitalism diatribe was placed on administrative leave after the recording was made public.

In the tape (website, or download here), Jay Bennish – teaching a 10th grade world geography class at Overland High School in Aurora, Colo. – called the U.S. "probably the single most violent nation on planet Earth" and described capitalism as a system "at odds with human rights."

Bennish told students Jan. 29 he found "eerie similarities" to Bush's statements in his State of the Union speech and things Adolf Hitler said, i.e., "We're the only ones who are right. Everyone else is backwards. And it's our job to conquer the world and make sure they live just like we want them to."

"Now, I'm not saying that Bush and Hitler are exactly the same," Bennish said. "Obviously, they are not. OK. But there are some eerie similarities to the tones that they use. Very, very ethnocentric. We're right. You're all wrong.

Twenty minutes of the class was recorded on an MP3 player by student Sean Allen, who can be heard at several points questioning Bennish's declarations.

Bennish has been with the school's social studies faculty since 2000, according to the Denver Post.

An Overland student told KUSA-TV in Denver Bennish's rant was "the usual thing in our school," the only difference being that a tape recorder was there.

"Three quarters of the teachers are anti-Bush – very much so," she said.

Cherry Creek School District spokeswoman Tustin Amole said officials are probing the incident but have taken no disciplinary action. The teacher was placed on leave "to take some of the pressure off of him" during the investigation, Amole said, according to the Post.

After listening to the tape, Superintendent Monte Moses told the Denver paper "a breach of district policy" occurred.

"Our policy calls for both sides to be present ... in the interest of intellectual discourse," Moses said.

The 16-year-old student told talk host Mike Rosen of KOA in Denver yesterday he had been disturbed by the "political rants" he heard in Bennish's class.

"If he wants to give an opinion in class, I'm perfectly OK with that," he said. "But he has to give both sides of the story."

Allen said he often has used a recorder to help take notes.

During the Jan. 29 class, the student argued that the U.S. wars in Afghanistan and Iraq were responses to unprovoked attacks on American soil.

Bennish replied that to al-Qaida, 9-11 was a response to U.S. involvement in the Holy Land and to missile attacks during the Clinton administration in Afghanistan and Sudan.

In the mind of al-Qaida, the teacher said, the World Trade Center, the White House, the Pentagon and other buildings are "military targets."

At one point, according to the tape, Bennish asked:

    Who is probably the single most violent nation on planet Earth?

    (Unidentified student responds: We are.)

    The United States of America! And we're a democracy.

    Who has the most weapons of mass destruction in the world? The United States.

    Who's continuing to develop new weapons of mass destruction as we speak? The United States.

    So, why does Mr. Bush think that other countries that are democracies won't wanna be like us? Why does he think they'll just wanna be at peace with each other? What makes him think that when the Palestinians get their own state that they won't wanna preemptively invade Israel to eliminate a potential threat to their security just like we supposedly did in Iraq?

    Do you see the dangerous precedent that we have set by illegally invading another country and violating their sovereignty in the name of protecting us against a potential future – sorry – attack?


On capitalism, Bennish said:

    If you don't understand the economic system of capitalism, you don't understand the world in which we live. OK. Economic system in which all or most of the means of production, etc., are owned privately and operated in a somewhat competitive environment for the purpose of producing profit.

    Of course, you can shorten these definitions down. Make sure you get the gist of it. Do you see how when, you know, when you're looking at this definition, where does it say anything about capitalism is an economic system that will provide everyone in the world with the basic needs that they need? Is that a part of this system? Do you see how this economic system is at odds with humanity? At odds with caring and compassion? It's at odds with human rights.

    Anytime you have a system that is designed to procure profit, when profit is the bottom motive – money – that means money is going to become more important potentially than what? Safety, human lives, etc.


According to the Rocky Mountain News, the school district first learned about Bennish's lecture Feb. 22 when someone forwarded an online column by Walter Williams. The same day, Allen's father contacted the school's principal, who forwarded the complaint to the district.

The district says its investigation will be completed this week.

As WorldNetDaily reported in November, talk host Sean Hannity urged college students to fight back against left-wing indoctrination in class by recording professors' lectures.

"All you college kids out there, check your state laws, check your campus laws," said Hannity on his national radio program.

"Get your little tape recorders if legal, and I want you to start recording these left-wingers. Bring it to this program and we'll start airing it every single time on this program. I'm sick of this indoctrination. I'm sick of this left-wing propaganda."

Hannity's call to action came in the wake of the case of Rebecca Beach, a 19-year-old freshman at Warren County Community College in Washington, N.J., who, as WorldNetDaily first reported, was rebuked sharply by an English professor for her announcement of a campus program featuring a decorated Iraq war hero.

~~~~~~~~~~~~~~~~~~~~~~~~~~~~~~~~~~~~~~~~~~~~~~~~~~~~~~~

Is this what we pay our public school teachers for? If I still had kids in school I would be visiting their classes.

tomder55 answered on 03/03/06:

Bennish is a GEOGRAPHY teacher .I'm sure that the State of Colorado has a 10th grade curriculum that does not include his diatribes ;especially as you pointed out that his soap box was paid for by the public tax rolls. Even under the guise of trying to stimulate critical thinking this was inappropriate . Had he in fact been doing that he would've balanced his presentation and to all appearances remained neutral.

Suggest Bennish use some of his geography skills to open a map and locate Dachau ;Auswitz;Treblinka and many other notorious Nazi extermination camps ;go visit them and find out what the F** he's talking about before he as the gall to make such a comparison !!

I guess it is much easier to indoctrinate students who do not have the critical thinking skills yet than to debate it in the open forum of ideas where only Michael Moore moonbats accept his exagerated premisis unquestioned .

Jay Bennish has hired an ACLU attorney.... David Lane... the lawyer Ward Churchill had defending him. He's in fine company .

Itsdb rated this answer Excellent or Above Average Answer
ETWolverine rated this answer Excellent or Above Average Answer

Question/Answer
ETWolverine asked on 03/02/06 - A great article.

This is probably the best, most honest article about the Danish Cartoon riots that I have seen, written by a Muslim. I've never heard of Nonie Darwish before, but I intend to keep an eye on her work. The article was found in the NY edition of Metro newspaper.

http://ny.metro.us/metro/blog/my_view/entry/Roots_of_the_recent_Muslim_rioting/1352.html

-----------------

Roots of the recent Muslim rioting
my view by nonie darwish

MAR 2

The ongoing controversy regarding the Danish cartoons of the Prophet Muhammad completely misses the point. Of course the cartoons are offensive to Muslims, but newspaper cartoons do not warrant the burning of buildings and the killing of innocent people. The cartoons did not cause the disease of hate that we are seeing in the Muslim world on our TV screens at night they are only a symptom of a far greater disease.

I was born and raised as a Muslim in Cairo, Egypt, and in the Gaza Strip. In the 1950s, my father was sent by Egypts President Gamal Abdel Nasser to head the Egyptian Military Intelligence in Gaza and Sinai, where he founded the Palestinian Fedayeen, or armed resistance. They made cross-border attacks into Israel, killing 400 Israelis and wounding more than 900 others. My father was killed as a result of the Fedayeen operations when I was 8 years old. He was hailed by Nasser as a national hero and was considered a shaheed, or martyr. In his famous speech announcing the nationalization of the Suez Canal, Nasser vowed that all of Egypt would take revenge for my fathers death. My siblings and I were asked by Nasser, Which one of you will avenge your fathers death by killing Jews? We looked at each other speechless, unable to answer.

In Gaza elementary schools, I learned hate, vengeance and retaliation. Peace was never an option, as it was considered a sign of defeat and weakness. At school we sang songs with verses calling Jews our dogs. Criticism and questioning were forbidden. Sadly, the way I was raised was not unique. Hundreds of millions of other Muslims also have been raised with decades of anti-West and anti-Israel blame and hate as a way to distract from the failings of their leaders.

Today, the Islamo-fascist president of Iran uses nuclear dreams, Holocaust denials and threats to wipe Israel off the map as a way to maintain control of a country where unemployment, prostitution and drug addiction are out of control. Indeed, with Denmark set to assume the rotating presidency of the U.N. Security Council, the flames of the cartoon controversy have been fanned by Iran and Syria. This is critical since the International Atomic Energy Agency is expected to refer Iran to the Security Council and demand sanctions. At the same time, Syria is under scrutiny for its actions in Lebanon. Both Iran and Syria cynically want to embarrass the Danes in order to achieve their dangerous goals.

But the rallies and riots come from a public ripe with rage. Is it any surprise that after decades of indoctrination in a culture of hate, people actually do hate? Arab society has created a system of relying on fear of a common enemy. Its a system that has brought them much-needed unity, cohesion and compliance in a land ravaged by tribal feuds, instability, violence and selfish corruption. Its time for Arabs and Muslims to stand up for their families. We must stop allowing Arab and Muslim leaders to use the West and Israel as an excuse to distract attention from their own failed leadership and their citizens lack of freedoms. What is needed is hope and not hate. Unless we recognize that the culture of hate is the true root of the riots surrounding this cartoon controversy, this violent overreaction will lead to a clash of civilizations that the world cannot bear.


Nonie Darwish is a freelance writer and public speaker who lectures to audiences worldwide about the need for change in Muslim society. She recently completed a cross-country Mothers for Peace tour for the Israel Project.


---------------

Ms. Darwish hits the nail right on the head. It's not about the cartoons, its about the culture of hate, taught to children from a very young age over decades.

Elliot

tomder55 answered on 03/02/06:

She is a regular contributor of Front Page . She also runs the web site Arabs for Israel .gotta luv that ! She should have a new bok out soon called "From Hatred to Love" .

You are right ;she hits it. I see she also recognizes the coincidence of the timing of the Cartoon Intifada's timing to coincide with the Danes assuming the Presidency of the IAEA. I see no coincidence in any of the recent events ....all events point to Iran .

ETWolverine rated this answer Excellent or Above Average Answer

Question/Answer
Itsdb asked on 03/02/06 - Catholic town

New Florida town looks to ban abortions, pornography and contraceptives

By Brian Skoloff
ASSOCIATED PRESS

10:46 a.m. March 1, 2006

NAPLES, Fla. If Domino's Pizza founder Thomas S. Monaghan has his way, a new town being built in Florida will be governed according to strict Roman Catholic principles, with no place to get an abortion, pornography or birth control.

The pizza magnate is bankrolling the project with at least $250 million and calls it God's will.

Civil libertarians say the plan is unconstitutional and are threatening to sue.

The town of Ave Maria is being constructed around Ave Maria University, the first Catholic university to be built in the United States in about 40 years. Both are set to open next year about 25 miles east of Naples in southwestern Florida.

The town and the university, developed in partnership with the Barron Collier Co., an agricultural and real estate business, will be set on 5,000 acres with a European-inspired town center, a massive church and what planners call the largest crucifix in the nation, at nearly 65 feet tall. Monaghan envisions 11,000 homes and 20,000 residents.

During a speech last year at a Catholic men's gathering in Boston, Monaghan said that in his community, stores will not sell pornographic magazines, pharmacies will not carry condoms or birth control pills, and cable television will have no X-rated channels.

Homebuyers in Ave Maria will own their property outright. But Monaghan and Barron Collier will control all commercial real estate in the town, meaning they could insert provisions in leases to restrict the sale of certain items.

I believe all of history is just one big battle between good and evil. I don't want to be on the sidelines, Monaghan, who sold Domino's Pizza in 1998 to devote himself to doing good works, said in a recent Newsweek interview.

Robert Falls, a spokesman for the project, said Tuesday that attorneys are still reviewing the legal issues and that Monaghan had no comment in the meantime.

If they attempt to do what he apparently wants to do, the people of Naples and Collier County, Florida, are in for a whole series of legal and constitutional problems and a lot of litigation indefinitely into the future, warned Howard Simon, executive director of the American Civil Liberties Union of Florida.

Florida Attorney General Charlie Crist said it will be up to the courts to decide the legalities of the plan. The community has the right to provide a wholesome environment, he said. If someone disagrees, they have the right to go to court and present facts before a judge.

Gov. Jeb Bush, at the site's groundbreaking earlier this month, lauded the development as a new kind of town where faith and freedom will merge to create a community of like-minded citizens. Bush, a convert to Catholicism, did not speak specifically to the proposed restrictions.

While the governor does not personally believe in abortion or pornography, the town, and any restrictions they may place on businesses choosing to locate there, must comply with the laws and constitution of the state and federal governments, Russell Schweiss, a spokesman for the governor, said Tuesday.

Frances Kissling, president of the liberal Washington-based Catholics for a Free Choice, likened Monaghan's concept to Islamic fundamentalism.

This is un-American, Kissling said. I don't think in a democratic society you can have a legally organized township that will seek to have any kind of public service whatsoever and try to restrict the constitutional rights of citizens.

~~~~~~~~~~~~~~~~~~~~~~~~~~~~~~~~~~~~~~~~~~~~~~~~~~~~~~~

Good idea?

Constitutionally sound or "un-American?"

Like "Islamic fundamentalism" or free exercise of religion?

Other thoughts or comments?

tomder55 answered on 03/02/06:

If Ray Nagin can build a chocolate town .....

I think the village of New Square is almost completely Hasidic .I believe that village leaders make laws that conform with their faith .

In Iowa there is Maharishi Vedic City Vedic City banned the sale of all non-organic foods. They even have their own currency.

Of course the Mormons settled Utah and still have a great deal of influence there.


Disney created a town in it's own image 1994 and ran it for a while .


All over America people live with Codes, covenants, restrictions built into their municipalities. You should see the restrictions at some of the geezer communities in Fla.
In Florida pharmacies are NOT REQUIRED to provide contraceptives, so if none are available in Ave Maria, folks who want them will have to go to another town and get them. If Ave Maria folks think that having a abortion clinic does not comport with the town's values, they are within their rights to refuse to allow one to open just like many towns now make zone prohibiting stripper bars.

Itsdb rated this answer Excellent or Above Average Answer

Question/Answer
jackreade asked on 03/01/06 - Iraq

Is the Iraq Civil War a creation of the media?


Jack

tomder55 answered on 03/02/06:

This is not even a decent ethnic cleansing (that is what Saddam was doing ). So far the Iraqi Defense forces have taken a low profile except manning road blocks ,but if necessary they are prepared to 'fill the streets with armored vehicles' in the words of Defense Minister Saadun Al Dulaimi . Admittedly there is a problem with the Interior Ministry as it is pretty clear that has been inflitrated by members of the Mahdi Army.

I agree with ET's commentary .The media refuses to place the level of violence in any sort of context. Do they compare it to oh lets say the thousands of French who were killed at the collapse of the Viche Gvt?
In the run-up to the recent elections as many as one hundred people were killed in a single suicide bombing. Also, we know that 47 Sunnis and Shia factory workers returning from a peaceful protest were pulled from a bus and executed. Batthist provocations instigated by Abu Musab al- Zarqawi (and he has made no bones about his goal of driving a wedge between the Shia and Sunni) have generally been met by a remarkable degree of restraint . In the chaos over the last year there have been two general elections and a Constituional referendum . Millions of Iraqis risked all to paint their fingers purple.

I follow Iraqi blogger Iraq the Model to get a perspective from someone who is there .

he recently posted some interesting comments :
However, it seems there are also some positive outcomes from this incident and its aftermath; the first one in my opinion was the performance of the Iraqi army which had a good role in restoring order in many places. Actually the past few days showed that our new army is more competent than we were thinking.
But the latest events have also showed the brittle structure of the interior ministry and its forces that retreated before the march of the angry mobs (if not joined them in some cases) and I think the statements that came from the meetings of our politicians pointed this out so clearly when Sunni politicians said they wanted the army to replace the police and police commandos in their regions and this indicates growing trust between the people and the army.

The other positive side is represented by the line we've seen drawn between clerics and politicians.
In spite of the attempts of clerics to look like as if they were the defenders of national unity with all their meetings, joint prayers and hugs, the political leaderships got a sense of their growing danger and the meeting at Jafari's home (which al-Hakeem didn't attend) showed that the government is keen to keep the country intact and the government systems as functional as possible to contain the crisis. This meeting indicates that politicians have realized that those clerics whether Sunni or Shia are the origin of the problem and are ready to coup on even their political allies which made the politicians more aware of the danger imposed by clerics on the project of building a state ruled by the law.

Clerics will not stop and they will carry on with their plans and I suspect they will launch the next phase of their plan soon after they received instructions from Syria (the Muslim scholars) and from Iran (the Sadrists).
The objective of the second phase will to move the conflict from one on the streets to a conflict with America. Thats not my personal opinion, but it's what clerics themselves are saying including Muqtada who returned from Qum in Iran to organize a joint Sunni-Shia demonstration against the occupation!!

Now the government has rise to the level of the challenge and proceed to take the most important and critical step and disband religious militias of all sorts and limit the influence of clerics-of any sect-in the decision-making process.
I think this is the best time for the new government to tackle this issue as the government now has all the factors that make such a move legitimate and necessary.


His comments confirm this report in the NY Post by Ralph Peters who is currently in Baghdad :

And the people here have been impressed that their government reacted effectively to last week's strife, that their soldiers and police brought order to the streets. The transition is working.


Most Iraqis want better government, better lives and democracy. It is contagious, after all. Come on over. Talk to them. Watch them risk their lives every day to work with us or with their government to build their own future.





FYI : on This posting I postulate that Iran acting through their puppet Mookie al-Sadr is responsible for the destruction of the golden dome.

Do I think that civil war is possible ? Certainly .If the US and the coalition were to leave too soon then yes it could erupt into a civil war which the Sunni would lose badly . I think that most Sunni realize that their best fate lies in a national unity govenment. That is why they participated so willingly in the last round of elections.

excon rated this answer Excellent or Above Average Answer
jackreade rated this answer Excellent or Above Average Answer
purplewings rated this answer Excellent or Above Average Answer

Question/Answer
excon asked on 03/01/06 - Ports

Hello Republicats:

Think a UAE port company is the same as a British port company, or even a Chinese company?

Neither the Brits nor the Chinese boycott our ally, Israel. But, those Arabs do.

So, you should excuse me if I sound a little bigoted, or even more than a little.

excon

tomder55 answered on 03/01/06:

better watch out . Larry Kudlow is going to call you Islamophobic. You will also be called a nativist and a protectionist .I call you right. They have been an integral part of the Israeli boycott. They have also consistently voted in the UN against our interests . Some ally !

ETWolverine rated this answer Excellent or Above Average Answer
excon rated this answer Excellent or Above Average Answer

Question/Answer
Itsdb asked on 02/27/06 - Ecoterrorism

After watching Seinfeld last night I happened upon a program called Source Code on the Free Speech Network. The episode was discussing how the Bush administration and the Patriot Act had criminalized the "free speech" of environmental activists. After all, these people aren't terrorists, they're just engaging in "civil disobedience" as their preferred method of exercising their right of "free speech." One of those interviewed described how the Patriot Act was eroding our rights, right down to "freedom of thought."

The show then went on to inform us that "Former Animal Liberation Front Warrior Rodney Coronado" was indicted for making a speech. That of course got me curious.

So who is this Coronado guy? He's quoted in the above linked article above as saying activists are "doing the only thing they know to do and that is strike a match and draw a whole lot of attention to their dissatisfaction with protecting the environment" (but destroying a human's environment is apparently ok).

Coronado was found guilty of "Conspiracy to Impede or Injure an Officer of the United
States, Interfering with a Forest Officer and Depredation of Government Property" in December of last year. Coronado served four years in prison for his part in burning down an animal-testing lab at Michigan State University in 1992. The current charge, "the felony charge of demonstrating the use of a destructive device. Though he isn't charged in the fire, he gave the speech "as the University City housing complex smoldered from an arson 2 years ago," a $50 million arson job in the San Diego area where the Earth Liberation Front left a 12 foot banner reading "If you build it, we will burn it."

Should "Teaching people how to build explosives in order to commit violent crimes" or threatening "If you build it, we will burn it" be illegal, or is it protected free speech?

Is burning down a housing complex, spiking trees, setting lab animals free, etc. acceptable" civil disobedience"?

Is ecoterrorism really terrorism?

Steve

tomder55 answered on 02/27/06:

It is not that the threaten arson ;it is that they commit arson that is troubling (at least ELF ,a close companion organiztion did ).They use firebombs, timed detonations devices ;incendiary devices, Molotov cocktails, and poison in their acts of "civil diobedience".Millions of dollars of homes, equipment, and research have been destroyed by their operatives .It is of no comfort to me that they disavow harm to humans . If someone died in their efforts would it be collateral damage ?

They are responsible for over 1200 criminal attacks in the US since 1990 .These self appointed judges would decide what I eat and wear ;and what property I should own.They would decide which if any cars you own. They would deny us the benefit of medical research .They are eco-Nazis ;the Taliban !

University of Texas Professor Dr. Steven Best is an advocate for the terrorist group and has used the university's Web site(tax payer funded of course) to direct people to ALF Web sites . Best has written a book, "Terrorist or Freedom Fighters? Reflections on the Liberation of Animals." The forward to that book was written by Ward Churchill.

I guess you could make a convoluted case that their arson is no different than the Boston tea party except I just cannot morally equate the actions .But when they threaten people with violence like they did last year in England it clearly crosses the line .

Itsdb rated this answer Excellent or Above Average Answer
purplewings rated this answer Excellent or Above Average Answer

Question/Answer
ETWolverine asked on 02/24/06 - Interesting article from James Pinkerton of Newsday.

Bush won't fight ports deal

President's 'different standard' spin doesn't fly; U.K. looks sterling, UAE looks unfriendly

James P. Pinkerton

February 23, 2006

George W. Bush is ready to fight for the Dubai-buying-U.S.-ports deal. But a growing bipartisan grouping, in regard to that fight, is saying, "Bring it on."

Defending the proposed sale, Bush said Tuesday, "I want those who are questioning it to step up and explain why all of a sudden a Middle Eastern company is held to a different standard than a great British company."

OK, I will step up. Let's begin by noting that the United Kingdom and the United Arab Emirates are different countries, with different histories.

For 400 years, England has been America's mother country and English our mother tongue. Yes, we fought a war or two against each other, but they were "cousins' wars," more akin to family feuds than wars of annihilation. And even during wartime, Americans have naturally looked to Britons for inspiration on law and culture; from William Shakespeare to the King James Bible to C.S. Lewis to J.K. Rowling, British letters have been America's letters.

And in the past century the U.S. and U.K. were shoulder to shoulder in two hot wars and one cold war. Few Americans can forget the oratory of Winston Churchill, who rallied English speakers against Nazism. (And who were the Arabs rooting for in World War II? Just asking.)

In the decades since, Washington and London have stayed close. The friendship between Ronald Reagan and Margaret Thatcher is the stuff of legend, but if anything, the bond between President Bush and Prime Minister Tony Blair is even stronger. So strong, in fact, that Blair's critics call him "Bush's Poodle." That's not nice, but it should still be a source of reassurance to Americans.

Now to the United Arab Emirates. First and most obviously, it's Arab. That's not a statement of racism; that's an observation about ethnicity and the culture that comes with it. Virtually all UAE-ers are Arab Muslims, and many probably watch Al-Jazeera TV, which serves up a steady diet of anti-American "newsaganda." That's the reality of multiculturalism on a planetary scale: People in different countries are different, see things differently, react to things differently. That's why consumers in the UAE eagerly joined in the boycott of Danish goods in the wake of the Muhammad cartoon controversy; The Associated Press reports that Denmark's exports to the UAE are down 95 percent.

Of course, it could be argued that public opinion doesn't matter much in the UAE because that country has never held an election. Freedom House, the human rights watchdog, labels the country "not free" - the lowest category. But even in dictatorial countries culture matters. The UAE was the hub of the BCCI scandal in the ྖs, which spun a web of money-laundering, embargo-evading and gun-running all the way to New York and Washington.

Later, the UAE had warm relations with the Taliban when it ruled Afghanistan and played host to the likes of Osama bin Laden. And nobody quite knows when and if all those cozy relationships were ever shut down; here's a headline in the Feb. 17, 2002, Washington Post: "Al Qaeda's Road Paved With Gold/Secret Shipments Traced Through a Lax System in United Arab Emirates." Indeed, the U.S. government is still trying to unravel UAE banks' relationships with terrorists, both Arab and Iranian.

So in challenging critics of the port deal, the president actually put the issue the wrong way. The critics aren't holding the U.K. and the UAE to a different standard; they are holding the two countries to the same standard. And according to that single standard, Britain and the UAE look different: The British look sterling, while the Arab Emirates look mottled, at best.

Bush pledges to fight to the bitter end on this issue, but I'll bet he won't. In the mordant phrase of conservative blogger Robert A. George, "'Dubai Ports World' is Arabic for Harriet Miers."

James P. Pinkerton's e-mail ad- dress is pinkerto@ix.netcom.com.
Copyright 2006 Newsday Inc.

--------------------

Hmmmm. Interesting. Another case of "watch what he does not what he says"? Possibly. And Pinkerton seems to be right, considering that the deal has suddenly been put on the back burner as the Port Authority of NY's lawsuit is reviewed and congressional "leaders" (can anyone actually name a true "leader" in congress these days?) review the facts. So it may be that Bush decides not to fight for this deal after all.

Elliot

tomder55 answered on 02/26/06:

I think he will go to the mat for this . Under pressure over the DPW deal,Bush decided to emphasize how important the UAE was to the military, allowing us to use UAE military bases and ports for our troops, Air Force, and Navy warships.Dubai does have the advantage of a strategic location in the mouth of the Gulf.

Maybe the real agenda is that the United States needs the UAE bases to attack Iran successfully. If I was going to be convinced to change my position, this reason would be it...but I won't .The national security concerns over rides any other consideration.


Consider also that he has nothing to lose by fighting it:

he's not going to be re-elected, so he loses no political capitol.

It can't really be portrayed as a "republican" thing if a Republican congress overrides his veto

If the Demas amke an issue out of it, the Republican will say "Please do. Let's focus on security as a major campaign issue, PLEASE", so the GOP loses no capitol either.

On the other hand, the UAE will know Bush went to the mats for them, and in the mindest there, they will know Bush is a friend.The Arab world might sense that there really is an 'Angry American Street', and Bush is the wise and just leader restraining them, as best he can.So he retains the alliances he's made and the port deal is defeated anyway.


ETWolverine rated this answer Excellent or Above Average Answer

Question/Answer
ROLCAM asked on 02/25/06 - Who wrote these words and in what context ??

"We should not confine ourselves within narrowly-defined frontiers because this would mean we would be going voluntarily to that form of internal exile... called the homeland".

tomder55 answered on 02/26/06:

'Of all the many elephant traps lying ahead of us, the largest and most dangerous pitfall would be the adoption of a ghetto mentality. To forget that there is a world beyond the community to which we belong, to confine ourselves within narrowly defined cultural frontiers, would be, I believe, to go voluntarily into that form of internal exile
Salman Rushdie

Mathatmacoat rated this answer Excellent or Above Average Answer
ROLCAM rated this answer Excellent or Above Average Answer

Question/Answer
HANK1 asked on 02/24/06 - GOOGLE & CHINA:


"For the first time in what some fear will signal a growing trend, Google Inc. has banned and removed a mainstream news website from all its worldwide search engines, seemingly due to the website's reports on China's geopolitical affairs and military technology."

Source: Prison Planet.com (Propaganda Matrix)

Do you think this is a good move if it's true?

HANK

tomder55 answered on 02/25/06:

Hank ; last week I responded to a simular question by Fred . My response is here

Google was one of four companies brought before a congressional committee to testify on it's role of abetting censorship in China .They claimed that they would still make their non-filtered sites available in China but a 'Beijing News 'report, citing a Google insider, suggested that the users from within China trying to access the U.S.hosted site would be redirected to Google.cn.

Companies like Google ;AOL ;Micrsoft etc. argue that their continued presence in China is beneficial not just to their shareholders, but to the development of democracy there.If Google.cn declines to filter ;its site will be blocked, and Chinese competitor Baidu will capture the market.

So they are caught between a rock and a hard place . To get a foothold in China they need to play by China's rules .However .... There will be net users in China long after this aging regime goes away . They cannot fight progress . Their attempts to block the information that internet users access is futile .

In time ;Chinese net users will remember which companies collaborated with the oppressors and they will be rewarded or punished in kind .

ETWolverine rated this answer Excellent or Above Average Answer
HANK1 rated this answer Excellent or Above Average Answer

Question/Answer
purplewings asked on 02/23/06 - Good news reported from Iraq, at last.

http://www.townhall.com/opinion/columns/calthomas/2006/02/23/187542.html

Would you care to guess who said this: "To the courageous men and women of the 3rd Armored Cavalry Regiment, who have changed the city of Tall Afar (Iraq) from a ghost town, in which terrorists spread death and destruction, to a secure city flourishing with life"?

Is this an excerpt from a Pentagon or presidential citation? Nope.

"To the lion-hearts who liberated our city from the grasp of terrorists who were beheading men, women and children in the streets for many months." Was that on a plaque from the local Kiwanis Club? Wrong again.

"To those who spread smiles on the faces of our children and gave us restored hope, through their personal sacrifice and brave fighting, and gave new life to the city after hopelessness darkened our days and stole our confidence in our ability to reestablish our city."

Give up? That is a letter from the mayor of Tall Afar in the Iraqi province of Nineveh. Mayor Najim Abdullah Abid Al-Jubouri wrote it to express his gratitude to American soldiers. Chances are you have not read this letter. I have only found it in a few small-circulation U.S. newspapers. It certainly conveys an impression opposite what much of the mainstream media and some politicians have been telling us.

Tall Afar was the main base of operations for the terrorist Abu Musab al-Zarqawi. The mayor says his city was held hostage by al-Zarqawi. "Our schools, governmental services, businesses and offices were closed. Our streets were silent, and no one dared to walk them. Our people were barricaded in their homes out of fear; death awaited them around every corner. Terrorists occupied and controlled the only hospital in the city. Their savagery reached such a level that they stuffed the corpses of children with explosives and tossed them into the streets in order to kill grieving parents attempting to retrieve the bodies of their young."

The 3rd Armored Cavalry Regiment (ACR) arrived in Iraq in 2003 and began attacking insurgents in Fallujah. Last year, they went back for a second tour, this time in Tall Afar. The mayor's letter sums up the result: "This was the situation of our city until God prepared and delivered ... the courageous soldiers of the 3rd Armored Cavalry Regiment, who liberated this city, ridding it of al-Zarqawi's followers after harsh fighting, killing many terrorists and forcing the remaining butchers to flee the city like rats to the surrounding areas, where the bravery of other 3rd ACR soldiers in Sinjar, Rabiah, Zumar and Avgani finally destroyed them."

One of the returning soldiers from this regiment is Chief Warrant Officer Roger Wood of Fort Carson, Colo. Wood, a helicopter pilot, arrived home last weekend. He tells me, "There's a lot of good going on in Iraq," adding that "negative stories" he's seen on the news "are discouraging." Wood says he's noticed a "big difference" since his first tour in 2003 in the way Iraqi soldiers and police operate. He says they are increasingly confident and able to operate independent of American forces. "Iraq will come around," Wood predicts. "Baghdad will take a little longer, but as people see change and acquire hope, we'll see a new Iraq."

The mayor of Tall Afar concludes his letter: "God bless this brave Regiment; God bless the families who dedicated these brave men and women. From the bottom of our hearts we thank the families. They have given us something we will never forget. To the families of those who have given their holy blood for our land, we all bow to you in reverence and to the souls of your loved ones. This sacrifice was not in vain. They are not dead, but alive, and their souls hovering around us every second of every minute. They will never be forgotten for giving their precious lives. ... Let America, their families and the world be proud of their sacrifice for humanity and life."

The U.S. Conference of the World Council of Churches recently condemned U.S policy in Iraq for "raining down terror" on helpless Iraqis. They should talk to the mayor of Tall Afar and tell him to his face they think al-Zarqawi's "reign of terror" should not have been ended.


So, the question is to those who have been so against our being in Iraq at all, does it ease your mind to know we really are doing some good?

tomder55 answered on 02/23/06:

hard to find that . Here is the complete letter from the mayor . I received it via email :

In the Name of God the Compassionate and Merciful

To the Courageous Men and Women of the 3d Armored Cavalry Regiment, who have changed the city of Tall Afar from a ghost town, in which terrorists spread death and destruction, to a secure city flourishing with life.

To the lion-hearts who liberated our city from the grasp of terrorists who were beheading men, women and children in the streets for many months.

To those who spread smiles on the faces of our children, and gave us restored hope, through their personal sacrifice and brave fighting, and gave new life to the city after hopelessness darkened our days, and stole our confidence in our ability to reestablish our city.

Our city was the main base of operations for Abu Mousab Al Zarqawi. The city was completely held hostage in the hands of his henchmen. Our schools, governmental services, businesses and offices were closed. Our streets were silent, and no one dared to walk them. Our people were barricaded in their homes out of fear; death awaited them around every corner. Terrorists occupied and controlled the only hospital in the city. Their savagery reached such a level that they stuffed the corpses of children with explosives and tossed them into the streets in order to kill grieving parents attempting to retrieve the bodies of their young. This was the situation of our city until God prepared and delivered unto them the courageous soldiers of the 3d Armored Cavalry Regiment, who liberated this city, ridding it of Zarqawis followers after harsh fighting, killing many terrorists, and forcing the remaining butchers to flee the city like rats to the surrounding areas, where the bravery of other 3d ACR soldiers in Sinjar, Rabiah, Zumar and Avgani finally destroyed them.

I have met many soldiers of the 3d Armored Cavalry Regiment; they are not only courageous men and women, but avenging angels sent by The God Himself to fight the evil of terrorism.

The leaders of this Regiment; COL McMaster, COL Armstrong, LTC Hickey, LTC Gibson, and LTC Reilly embody courage, strength, vision and wisdom. Officers and soldiers alike bristle with the confidence and character of knights in a bygone era. The mission they have accomplished, by means of a unique military operation, stands among the finest military feats to date in Operation Iraqi Freedom, and truly deserves to be studied in military science. This military operation was clean, with little collateral damage, despite the ferocity of the enemy. With the skill and precision of surgeons they dealt with the terrorist cancers in the city without causing unnecessary damage.

God bless this brave Regiment; God bless the families who dedicated these brave men and women. From the bottom of our hearts we thank the families. They have given us something we will never forget. To the families of those who have given their holy blood for our land, we all bow to you in reverence and to the souls of your loved ones. Their sacrifice was not in vain. They are not dead, but alive, and their souls hovering around us every second of every minute. They will never be forgotten for giving their precious lives. They have sacrificed that which is most valuable. We see them in the smile of every child, and in every flower growing in this land. Let America, their families, and the world be proud of their sacrifice for humanity and life.

Finally, no matter how much I write or speak about this brave Regiment, I havent the words to describe the courage of its officers and soldiers. I pray to God to grant happiness and health to these legendary heroes and their brave families.

NAJIM ABDULLAH ABID AL-JIBOURI
Mayor of Tall Afar, Ninewa, Iraq



[Col McMaster in Tall Afar with (from left to right) Mayor Najim, Col Khalid (Mosul Emergency Battalion) and BG Saba (Tall Afar Police Chief), among others.]



Btw :about Col. McMaster .He led the attack in the Battle of 73 Easting during the 1st Gulf War .

Reaching 70 Easting at 16:22, Eagle Troop's 2nd Squadron knocked out a screen of eight Iraqi T-72 tanks. Beyond, they could see T-72s in prepared positions at 73 Easting. This was the Iraqi Brigade Assembly Area. Fearing the loss of surprise, Eagle Troop's Captain H.R. McMaster decided not to wait for the heavier units to come forward, pass through his lines and engage the Iraqis. McMaster ordered Eagle Troop to advance and engage the Iraqi tanks.

Eagle Troop consisted of 10 M1 Abrams tanks, 13 M3 Bradleys, two M106 mortar carriers, one M577 command track and a M981 FIST-V.

Armored battles in the open desert are generally decided very quickly; 73 Easting was no exception. The 2nd ACR surprised the enemy and penetrated the Iraqi positions so quickly that they were unable to recover. Superior American night vision equipment turned the poor weather into a U.S. advantage.

McMaster's unit charged and destroyed the Iraqi tanks at 73 Easting at close range. Unlike previous engagements, the destruction of the first Iraqi tanks did not result in the wholesale surrender of Iraqi soldiers. The Iraqis stood their ground while their tanks and armored personnel carriers of the Tawakalna Division attempted to maneuver and fight. Eagle Troop destroyed more than 20 tanks and other armored vehicles, a number of trucks and bunkers, and took a large numbers prisoners with no losses to themselves. In 20 minutes, Eagle Troop had advanced in constant heavy contact with Iraqi armor from 67 Easting to 74 Easting.




purplewings rated this answer Excellent or Above Average Answer

Question/Answer
kindj asked on 02/22/06 - Please tell me!

Please, someone tell me why I should NOT be having serious doubts about Bush's concern (more properly, lack of) about our border security?

He ignored the holes in the south, even way back when he was governor, and now this port thing?

It seems to me that something just ain't adding up here. If I'm wrong, please tell me.

tomder55 answered on 02/22/06:

I don't get it . oh I know the rational .Reward a good buddy who has been helping in the GWOT .But there are other ways to award a friend. There are it appears other foreign companies running ports throughout the U.S. that I was not aware of . This practice should stop immediately . It makes no more sense to have a UAE company running the ports then it would be to have a N.Korean company running our nuclear power plants. This is like asking El Presidente Fox for help with border security.Oh wait.....he already has ...never mind.

In truth Bush got caught with his pants down . This was decided at the cabinet level .And now that the cat is out of the bag ;Bush is going to compound the error by drawing a line in the sand about it . Why 'fall on your sword' over this loser issue when he did not veto bridges to nowhere or McCain's torture bill ? It doesn't make sense .

The chief culprit is Treasury Secretary John Snow who has a direct link to Dubai Ports World.Before he became Tresury Sec. he was chairman and chief executive officer of CSX Corporation ;the company that was brought by DPW.In addition The person tapped by Bush to run the Department of Transportation's Maritime Administration, David Sanborn, was a senior executive at DPW. So there are direct ties with important adm. officials to DPW. When this becomes widespread knowlege wait until the conflict of interest charges start being flung.

I have to admit . There is a degree of satisfaction watching Chucky Shumer becoming the chief advocate for profiling .But on the other hand ;Shumer and co. are weak on national security and now the President has handed them a tailor made issue to beefup their national security credentials .

ETWolverine rated this answer Excellent or Above Average Answer
Itsdb rated this answer Excellent or Above Average Answer
kindj rated this answer Excellent or Above Average Answer

Question/Answer
excon asked on 02/22/06 - Russian Pairs
Russian Scaters


Hello:

What do you think of the Russians, Slutskya, and Pricknova?

excon

tomder55 answered on 02/22/06:

I have been so entranced by the curling that I have missed almost all the figure skating (I think they showed 3 skaters and then switched to women's bob-sled before I went to sleep ).

I hear there were some pretty good prat-falls during the ice dancing . sad when you realize you are seeing a better product on 'skating with the stars' then you do at the olympics.

Who's having a worse week Bode Miller or George Bush ?

excon rated this answer Excellent or Above Average Answer
Itsdb rated this answer Excellent or Above Average Answer

Question/Answer
arcura asked on 02/20/06 - The Click That Broke a Government's Grip.......

By Philip P. Pan
Washington Post Foreign Service
Sunday, February 19, 2006; A01

BEIJING -- The top editors of the China Youth Daily were meeting in a conference room last August when their cell phones started buzzing quietly with text messages. One after another, they discreetly read the notes. Then they traded nervous glances.

Colleagues were informing them that a senior editor in the room, Li Datong, had done something astonishing. Just before the meeting, Li had posted a blistering letter on the newspaper's computer system attacking the Communist Party's propaganda czars and a plan by the editor in chief to dock reporters' pay if their stories upset party officials.

No one told the editor in chief. For 90 minutes, he ran the meeting, oblivious to the political storm that was brewing. Then Li announced what he had done.

The chief editor stammered and rushed back to his office, witnesses recalled. But by then, Li's memo had leaked and was spreading across the Internet in countless e-mails and instant messages. Copies were posted on China's most popular Web forums, and within hours people across the country were sending Li messages of support.

The government's Internet censors scrambled, ordering one Web site after another to delete the letter. But two days later, in an embarrassing retreat, the party bowed to public outrage and scrapped the editor in chief's plan to muzzle his reporters.

The episode illustrated the profound impact of the Internet on political discourse in China, and the challenge that the Web poses to the Communist Party's ability to control news and shape public opinion, key elements to its hold on power. The incident also set the stage for last month's decision to suspend publication of Freezing Point, the pioneering weekly supplement that Li edited for the state-run China Youth Daily.

Eleven years after young Chinese returning from graduate study in the United States persuaded the party to offer Internet access to the public, China is home to one of the largest, fastest-growing and most active populations of Internet users in the world, according to several surveys. With more than 111 million people connected to the Web, China ranks second to the United States.

Although just a fraction of all Chinese go online -- and most who do play games, download music or gossip with friends -- widespread Internet use in the nation's largest cities and among the educated is changing the way Chinese learn about the world and weakening the Communist Party's monopoly on the media. Studies show China's Internet users spend more time online than they do with television and newspapers, and they are increasingly turning to the Web for news instead of traditional state outlets.

The government has sought to control what people read and write on the Web, employing a bureaucracy of censors and one of the world's most technologically sophisticated system of filters. But the success of those measures has been mixed. As a catalyst that amplifies voices and accelerates events, the Internet presents a formidable challenge to China's authoritarian political system. Again and again, ordinary Chinese have used it to challenge the government, force their opinions to be heard and alter political outcomes.

The influence of the Web has grown over the past two years, even as President Hu Jintao has pursued the country's most severe crackdown on the state media in more than a decade. The party said last week that Freezing Point would resume publishing, but Li and a colleague were fired, making them the latest in a series of editors at state publications to lose their jobs.

With newspapers, magazines and television stations coming under tighter control, journalists and their audiences have sought refuge online. The party's censors have followed, but cyberspace in China remains contested terrain, where the rules are uncertain and an eloquent argument can wield surprising power.

Dueling Views

They clashed from the start, two men named Li with conflicting ideas of what a newspaper should be.

One was the maverick editor Li Datong, 52, a tall man with a scholarly air who had spent his entire career at the China Youth Daily and helped turn the official organ of the Communist Youth League into one of the country's best papers. After the 1989 pro-democracy demonstrations in Tiananmen Square, he nearly lost his job for leading journalists in a petition drive seeking freedom of the press.

The other was the new editor in chief, Li Erliang, 50, short in stature and slick in manner, a favorite of the propaganda authorities who made his reputation running the party's official mouthpiece in Tibet. He was an outsider at the Daily, a product of the party apparatus who was sent in to get the paper's feisty staff under control.

One night soon after his arrival in December 2004, the new editor stopped the presses and tore out Li Datong's Freezing Point section because it contained an article criticizing the Chinese education system. The next morning, the chief editor went to Li Datong's office to explain, but Li was furious and refused to talk to him. He just kept writing, banging on his keyboard and ignoring his new boss, colleagues recalled.

Relations between the two men only got worse. The party's propaganda department had targeted Freezing Point in its media crackdown because it often published investigative reports that embarrassed officials, as well as essays on history, society and current events that challenged the party line. Colleagues said Li Erliang, who declined to be interviewed, tried to rein in the section to please his superiors. Li Datong, who spoke out after Freezing Point was suspended, said he fought to protect it.

"The propaganda department wanted to shut us down, and we were under a lot of pressure," he said. "They tried to get rid of our columnists and cut the size of the section and take away reporters, but we resisted."

Then, in August, Li Erliang proposed a point system for awarding bonuses to the paper's staff members. Reporters would receive 100 points if their articles were praised by provincial officials, 120 if praised by the propaganda department and 300 if praised by a member of the Politburo. Points would be deducted if officials criticized articles. Just one report that upset a party leader could mean loss of a month's salary.

The newsroom simmered with anger, reporters said. But Li Datong saw an opening to fight back. "The plan was just stupid," he said. "A newspaper can evaluate reporters that way, and many do, but it can't be so blatant about it."

Li holed up in his apartment, and two days later, emerged with a 13,000-word letter that denounced the point system, saying it would "enslave and emasculate" the paper, cause circulation to plummet and put the Daily out of business.

He also painted a damning picture of the propaganda apparatus. He described an official who measured photos of two party leaders before publication to make sure neither man would be offended. He wrote about a senior editor who resigned in protest over an obsequious column that compared President Hu's words to "a lighthouse beacon, pointing and illuminating the way for China's students." And he attacked the party's censors, questioning their legitimacy and alleging they favored publishers who showered them with gifts and banquets.

Li saved his harshest words for his new boss. But he crafted his letter carefully, citing the support of generations of party leaders for the paper's journalism and even quoting Karl Marx to make the case that editors should put readers first.

He showed the letter to a few colleagues and to the reporters on his staff. Then, on Aug. 15, at 10:09 a.m., he posted it on the newsroom's computer system. "I hoped it would have an impact," he said. "I never expected what happened next."

System of Censorship

Every Friday morning, executives from a dozen of China's most popular Internet news sites are summoned downtown by the Beijing Municipal Information Office, an agency that reports to the party's propaganda department.

The man who usually runs the meetings, Chen Hua, director of the Internet Propaganda Management Department, declined to be interviewed. But participants say he or one of his colleagues tells the executives what news they should keep off their sites and what items they should highlight in the week ahead.

These firms are private enterprises, and several, including Sina, Sohu and Yahoo! China, are listed on U.S. stock exchanges or have attracted U.S. investment. But because they need licenses to operate in China, they comply with the government's requests.

The meetings are part of a censorship system that includes a blacklist of foreign sites blocked in China and filters that can stop e-mail and make Web pages inaccessible if they contain certain keywords. Several agencies, most notably the police and propaganda authorities, assign personnel to monitor the Web.

The system is far from airtight. Software can help evade filters and provide access to blacklisted sites, and Internet companies often test the censors' limits in order to attract readers and boost profits. If an item isn't stopped by the filters and hasn't been covered in the Friday meetings, the government can be caught off guard.

That is what happened with Li Datong's letter. Minutes after he posted it, people in the newsroom began copying it and sending it to friends via e-mail and the instant messaging programs used by more than 81 million Chinese.

"We had to move quickly, before they started blocking it," recalled one senior editor, who spoke on condition of anonymity.

Pu Zhiqiang, a lawyer and advocate of journalists' rights, said he received a copy at 10:20 a.m., 11 minutes after Li posted the original. He forwarded it to 300 people by e-mail and sent it to others using Microsoft's MSN Messenger program. Then he began posting it on some of the bulletin board sites that have proliferated in China.

At 11:36 a.m., Pu put the memo on a popular forum called Yannan. Then he noticed that someone had posted a copy on another part of the site.

About the same time, the editors' meeting at the China Youth Daily ended and Li Erliang rushed back to his office. Colleagues said he contacted superiors in the propaganda department and the Communist Youth League after reading the memo.

Neither the government's censors nor the editors at the major Web sites had begun deleting the letter, yet. Some editors said they waited because it didn't challenge the party's authority or discuss subjects that were clearly off-limits, such as the Tiananmen Square massacre. At the same time, the official censors either failed to spot the memo or hesitated to act because they were worried that some senior officials might support Li Datong's views, editors said.

As they waited, the letter continued to spread.

At 12:17 p.m., it appeared on an overseas news site run by the banned Falun Gong spiritual movement, and minutes later on others managed by exiled dissidents. These sites are blocked in China, but many people access them using software that slips past the government's firewall.

By 1:30 p.m., a prominent blogger, Li Xinde, had downloaded the memo. He said he sent it using China's top instant messaging service, QQ, to more than 20 chat groups, each with 30 to 40 members. By 2 p.m., the memo had been posted on popular university Web sites.

The document was spreading so fast that many people received multiple copies. A writer in Anhui province said that when he went online to check his e-mail at 2:30 p.m., four friends immediately offered to send him the memo on MSN Messenger. But two copies were already in his inbox, including one that had been sent to 1,000 people.

Race in Cyberspace

It was midafternoon before someone in the party bureaucracy decided Li Datong's letter should be removed from Chinese cyberspace and government officials began calling executives at the major Web sites.

Some said they were contacted by the Beijing Municipal Information Office, others by its national-level counterpart, the State Council Information Office. None reported receiving a formal notice or any legal justification for the decision. As usual, they were just told to delete the offending material.

There are at least 694,000 Web sites in China, according to official statistics, and the party didn't try to contact them all. They called the most popular sites in Beijing first. Hours passed before some smaller bulletin board sites were notified. Forums with national audiences in other cities received calls only at the end of the day.

At a recent news briefing, Liu Zhengrong, a senior Internet affairs official in the State Council Information Office, declined to explain the legal basis for the orders, saying only that many comments about the China Youth Daily remained on the Web.

Even as Li's memo began disappearing from some Web sites, it went up on others the authorities had not contacted. Shortly before 10 p.m., it was posted on the popular Tianya forum. At 11 p.m., it became a featured item on Bokee, China's top blog and portal site.

Almost everywhere the letter appeared, users added hundreds of comments backing the reporters of the China Youth Daily. Inside the newsroom, spirits were buoyed. Some journalists posted notes on the internal computer system supporting Li Datong.

The next morning, officials continued calling Web sites, but readers started posting the memo on sites that had already removed it. Some Web site managers said they tried to drag their feet or leave copies on less prominent pages. One said the memo was viewed 30,000 times before he took it down.

But other Web sites added Li Datong's name to keyword filters used to block sensitive material from being posted.

At 2:15 p.m., Li Erliang distributed a rebuttal on the China Youth Daily's internal network. It was quickly leaked, too, triggering another wave of e-mails and postings.

Authorities were scrambling for a way to end the controversy. A few hours after Blog-City, an overseas blogging site, was blocked, the party announced in a rare retreat that it was ditching Li Erliang's point system.

"It was a breakthrough, and the Internet played a critical role," said Xu Zhiyong, a civil rights lawyer in Beijing. "If something is written well enough, they can't stop it from spreading. People will find a way to read it."

Freezing Point enjoyed a renaissance in the months that followed. Li Erliang appeared chastened, unwilling to risk another fight he might lose, reporters said.

But in January, propaganda officials finally shut down the section. Before doing so, they called executives from all the major Web sites to a special meeting and warned them not to allow any discussion of the action.

The news spread quickly anyway.

Researcher Jin Ling contributed to this report.

tomder55 answered on 02/20/06:

Lets say the Chinese put 40,000 staffers on the net and tried to filter it . It aint going to happen . They are fighting a losing battle .There are hundreds of thousands of net users in China and the numbers are growing and the degree of sophistication used in getting around blocked sites grows daily .Independent computer specialists are developing technologies that could reroute Internet information and put it beyond the reach of government censors. The Falun Gong are getting really good at it .

Lance Cottrell founder and president of Anonymizer is a leader in 'anonymous surfing' software, which lets people go to Internet sites without revealing the location of their computers. He's working on a plan to give away anonymous surfing services to the people of China and other countries that try to censor political and religious information.

"Anonymizer's new anti-censorship solution for Chinese citizens will be available before quarter's end," the company said on Wednesday. "The solution will provide a regularly changing URL that users can access to open the doors to unfettered access of the World Wide Web." Users' identities would also be protected from online tracking and monitoring by the Chinese government, it said.

"Anonymizer is not willing to sit idly by while the freedom of the Internet is slowly crushed," Anonymizer president Lance Cottrell said in a statement. "We take pride in the fact that our online privacy and security solutions provide access to global information for those under the thumb of repressive regimes


He is not alone in this effort .In the US the Tor Project sends spare Internet bandwidth to help others bypass the censors .In Canada there is a simular program called Psiphon .

So let Google ;AOL ;Microsoft ;and Yahoo collaborate with the Chinese gvt. They will not prevent the net users in China from getting the information they seek .Net users will outlast the current geriatric regime and will remember who helped them and who ratted on them .

arcura rated this answer Excellent or Above Average Answer
ETWolverine rated this answer Excellent or Above Average Answer
Itsdb rated this answer Excellent or Above Average Answer

Question/Answer
HerrAirhorn asked on 02/15/06 - If Cheney were Johnny Cash

"Vice President Dick "Buckshot" Cheney kept his word to the inmates at California's maximum security Folsom State Prison. He played a one hour set with his band "Dickie and The Trigger Happy Birdie Killers". The set received a luke warm reception until Cheney launched into his new, as yet unreleased, single "Go F*ck Yourself". During the guitar solo the Vice President thrilled the assembled audience by producing a rifle and opening fire. "He seems angry. Very angry" one inmate said "I mean, I always thought that the American people didn't like to vote for angry people but...Man, that dude is angry!" I managed to obtain a tape of the performance and am proud to present it here....(go to Huffington post to hear za music)


Ya I vas lookin for some jokes to lighten an othervise depressing news day ven I came across ziz on za internet. Enjoy. Herr Lufthorn

tomder55 answered on 02/16/06:

"Remember when the most embarrassing thing to happen to a vice-president was misspelling the word potato?" --Jimmy Kimmel

The White House press corps was furious. They expect to be told when the vice president shoots a 78-year-old man in the face." --Jimmy Kimmel


I am finding the situation less humorous every day ."Mr. Whittington is doing fine, but based on this development, we're going to downgrade the condition of this story from 'Incredibly Hilarious' to 'Still funny, but, mmm, now a little sad.'" --Jon Stewart,

You could say as news develops it gets more sobering. NYC famous lawyer/ investgator said last night that if Cheney admitted to one beer that means 3 in the eyes of a Grand Jury. The doctors refused to disclose the blood alcohol level of Harry Whittington and of course there was a delay between the time of the shooting and the time that the sheriff interviewed Cheney.

There also seems to be a disturbing rift that has developed between Cheney's staff and the West Wing ;if not between Bush and Cheney themselves . "If this story gets any bigger, pretty soon they're going to have to tell the president." --David Letterman



HerrAirhorn rated this answer Excellent or Above Average Answer

Question/Answer
sapphire630 asked on 02/14/06 - Katrina victims

Now they are saying that a lawyer found laws that prevent FEMA from delivering trailers to New Orleans.
No trailers are permitted to be placed in flood zones and Trailers also have to be put on concrete block.
So now they are to scrap the trailers or sell them at approximately scrap value. (If anybody hears about when and where the public can go to get them let me know)
Also this comes at a time when the government says they have paid all they are going to for Katrina 'victim' hotel bills.
Do you think the government should still pay or do you think they have had enough time to find work, other programs or sources.

I was thinking maybe they should tell any 'victims' if they can find a way and a place to get and put the trailers they can go get one.
Or
Why don't they have some type of emergency section 8 or other type program that would help them get back on their feet with housing?





tomder55 answered on 02/15/06:

This is your tax money at work :

10,770 Empty FEMA Trailers sinking in the mud at Hope Ark.

FEMA ice delivered to Maine .


Katrina displaced housed on Carnival Cruise ships at a rate of $1,275 a week

Victims abused debit cards issued to purchase $400 massages, a $450 tattoo, a $1,100 diamond engagement ring and $150 worth of products at "Condoms to Go."
GAO auditor Gregory Kutz told senators during a hearing that it was "certainly millions of dollars; it could be tens or hundreds of millions of dollars" that was wasted and abused or outright ripped off .That includes money for hotel rooms for evacuees that were paid at retail cost. Among the charges: $438 rooms in New York City and beachfront condominiums in Panama City, Fla., at $375 a night. There are even reports about MREs that were issued being sold on E-bay.

Nothing I hear about this suprises me anymore ;but it would be politically incorrect to suggest that the spiggot of Federal money should be turned off.

The more programs ;the more chances for abuse. That is the name of the game with big government nanny States . what happened to self reliance ;to community support ? This last weekend we had a large storm in NY . We made sure that everyone on our block who needed it got their driveways and walkways shoveled and plowed . Merchants did not wait for the town to shovel them out . They got their businesses up and running in hours .
NYC with the exception of some power outages came back to life in only a few hours after the last flakes fell. There were no FEMA plows and salt spreaders . The city knew the snow could come and prepared for it .

The risk of the levees breaking in NO was no secret ;everyone knew that they most likely would not withstand a cat. 5 storm . Where was the breakdown ? Why did they need the Fed. gvt. to get the people out of town ;to shelter them . Yes gvt. broke down at every level but it went further than that ;individuals failed ;families failed ;communities failed ;Parishes failed;the city gvt. of NO failed ;Lousiana failed ,and yes ultimately the Fed. gvt. failed . Now everyone is pointing fingers .




ETWolverine rated this answer Excellent or Above Average Answer
sapphire630 rated this answer Excellent or Above Average Answer

Question/Answer
excon asked on 02/14/06 - The Constitution of the United States of America


Hi Republicrats:

Everybody seems to be saying that the presidents' first duty is security for the country. I dont say that. And, just exactly where does it say that? You should excuse me, if I thought his first duty was to the Constitution.

This is his oath:

"Before he enter on the execution of his office, he shall take the following oath or affirmation:--"I do solemnly swear (or affirm) that I will faithfully execute the office of President of the United States, and will to the best of my ability, preserve, protect and defend the Constitution of the United States."

Please, educate me. Couldnt it just as easily say, protect and defend the people AND the Constitution? I think it could, and I think it doesnt. Where am I going wrong? Is some activist judge going to read something in there thats not there? Activist judges like Roberts, Scalia, Thomas, and Alito?

Nah, they said they wouldnt do that.

excon

tomder55 answered on 02/14/06:

Ex ;the concept of enumerated powers and inherent powers are derived from debates about the Constitution before the founders ratified it . That ''the executive power'' is not confined to those items expressly enumerated in Article II was asserted early in the history of the Constitution by Madison and Hamilton.

Hamilton wrote :The difficulty of a complete enumeration of all the cases of executive authority, would naturally dictate the use of general terms, and would render it improbable that a specification of certain particulars was designed as a substitute for those terms, when antecedently used. The different mode of expression employed in the constitution, in regard to the two powers, the legislative and the executive, serves to confirm this inference. In the article which gives the legislative powers of the government, the expressions are, All legislative powers herein granted shall be vested in a congress of the United States. In that which grants the executive power, the expressions are, The executive power shall be vested in a President of the United States. The enumeration ought therefore to be considered, as intended merely to specify the principal articles implied in the definition of executive power; leaving the rest to flow from the general grant of that power, interpreted in conformity with other parts of the Constitution, and with the principles of free government."

Granted ;the founders were not unanimous in some of their positions ;but in those cases there is 2 centuries of case law that affirms the inherent powers of the executive .

Just like we take for granted a "checks and balances "there is nothing written in the Constitution that says there is such .However James Madison wrote that the three branches "should not be so far separated as to have no constitutional control over each other."

The term we use to describe the type of judges we prefer is "originalists "(at least I do) .That means that they should be faithful to the original INTENT of the founders .The term "strict constructionism " is a misnomer.

excon rated this answer Excellent or Above Average Answer

Question/Answer
excon asked on 02/14/06 - Howdy


Hello Republicons:

Whadya think about the billions lost in Iraq and the millions wasted on Katrina?

If anything, I thought Repubicons were good managers, but noooooooooooo. Of course, I spose El is gonna tell me that the lost money is a secret Bush plan.....

excon

tomder55 answered on 02/14/06:

Katrina ? To compound all the money wasted on Fed. subsidized massages there is now a cash cow of unlimited Fed. $$ being set aside for the rebuilding of the City in the same major flood bank . That to me is foolish and pandering . People get displaced all the time when their homes are built where they ought not be. There is no reason or need to rebuild the city to it's former self for the sake of nostalgia. The useful areas should be salvaged ;for the most part they already have been . I can understand in the Netherlands they have no choice but to hold back the sea . This country is vast and it is unnecessary to reclaim land that mother nauture has claimed for herself. I hold that opinion wheter it be New Orleans ,or the fat cats living on dune road in the Hamptons. If they want to build there ;do it on their own dime.

As far as Iraq....See what happens when Haliburton isn't given the contract outright and is open to bidding from God knows who?When Clinton bombed Kosovo Halliburton rebuilt the infrastructure and Enron rebuilt the power plants;and for almost twice as much as the nearest competitor.(there goes Tom again bringing up Clinton)

Every year in the US there are tens of billions that cannot be accounted for. If ANY business were run with the lack of accounting like the US government, they would be prosecuted. I have to wonder how much of it is just untraceable cash used to obtain needed information . That appears to be the MO for doing business in alot of 3rd world nations ...money well spent but untraceable.

that being said . I do think we should continue to investigate these instances where it is clear that corruption occured.In the case of outright theft .They should be frog marched to Riker's Island.





ETWolverine rated this answer Excellent or Above Average Answer
excon rated this answer Excellent or Above Average Answer
Itsdb rated this answer Excellent or Above Average Answer

Question/Answer
Itsdb asked on 02/13/06 - Where Are the Adults?

From Mark Levin's NRO blog today...

"So detestable have Al Gore, Bill Clinton and Jimmy Carter become, that they are incapable of showing any restraint in the statements they make, whether here or abroad. Last week it was Carter at Mrs. King's funeral. Before that it was Clinton in Canada at another flat-earth meeting of environmental extremists in Canada. Now, Gore is at it again. AP reports, in part:

*Al Gore told a mainly Saudi audience on Sunday that the U.S. government committed "terrible abuses" against Arabs after the Sept. 11, 2001, attacks, and that most Americans did not support such treatment. *Gore said Arabs had been "indiscriminately rounded up" and held in "unforgivable" conditions. The former vice president said the Bush administration was playing into al-Qaida's hands by routinely blocking Saudi visa applications. *"The thoughtless way in which visas are now handled, that is a mistake," Gore said during the Jiddah Economic Forum. "The worst thing we can possibly do is to cut off the channels of friendship and mutual understanding between Saudi Arabia and the United States." *Gore told the largely Saudi audience, many of them educated at U.S. universities, that Arabs in the United States had been "indiscriminately rounded up, often on minor charges of overstaying a visa or not having a green card in proper order, and held in conditions that were just unforgivable." *"Unfortunately there have been terrible abuses and it's wrong," Gore said. "I do want you to know that it does not represent the desires or wishes or feelings of the majority of the citizens of my country."

More here. So, Gore goes to Saudi Arabia which has funded terrorism for decades, which is a proponent of one of the most radical forms of Islam, which was the home of Osama bin Laden and most of the 9/11 terrorist hijackers and denounces our government's response to the worst attack on U.S. soil in our history. The Democrat party is very sick right now. The hot-heads are in charge. And there are no mature voices with enough clout to temper the current party leadership. There was a time when ex-presidents and ex-vice presidents behaved as statesmen. Gore, Clinton, and Carter all of whom weakened their party are now committed to weakening our country."

~~~~~~~~~~~~~~~~~~~~~~~~~~~~~~~~~~~~~~~~~~~~~~~~~~~~~~~

Good question, where are the adults in the Democratic party today? Have I just not been paying attention or just when was it Bush "indiscriminately rounded up" Arabs in the U.S. and held them in "unforgivable" conditions? And just who the heck made Gore the spokesman and apologist for the American people?

tomder55 answered on 02/13/06:

While you are at NRO read the article about James Webb . He is the real deal and the Dems. have been salivatiing over the prospect of drafting him to run for George Allen's seat . But unless something happened in the last week ;the most recent news was that he had decided not to run .

as for Al Gore .

Instapundit has a great line :"Only Al Gore could come up with the idea of criticizing Bush for not sucking up to the Saudis enough. Sigh."


First ;he was speaking at Jiddah Economic Forum ;an organization that last week banned Danish business people from attending the forum.

Second ; He is complaining about a tightening of visas to Saudis . During the Clintoon adm. they were handing out Visas much too freely . They had streamlined the process and had created the 'Visa Express'.[submitting visa requests via travel agencies inside Saudia Arabia...most Saudi applicants could avoid contact with any U.S. offical that way ] All 19 hijackers had legal visas. Most of the visa applications were flawed and missing data according to the 9-11 commission . Three of the hijackers submitted applications that contained false statements that could have been proven to be false at the time they applied and 6 of them violated immigration laws inside the US.

Third ;the claim that we had mass roundups is completely false. To pander to the consipracy nuts in the Arab world just puts us in greater risk.

The modern Democrats have forever ended the notion that politics ends at the shoreline .


Itsdb rated this answer Excellent or Above Average Answer

Question/Answer
HANK1 asked on 02/11/06 - RATHER SHOCKING:

"The interests behind the Bush Administration, such as the CFR, the Bilderberger Group, and the Trilateral Commissionfounded by Brzezinski for David Rockefellerhave prepared for and are now moving to implement open world dictatorship within the next five years. They are not fighting against terrorists. They are fighting against citizens."

Quote by Dr. Johannes B. Koeppl, PhD, former German defense ministry official and advisor to former NATO Secretary-General Manfred Werner.

Source: Ending America

Do you know anything about this?

HANK





tomder55 answered on 02/12/06:

Hank ;please don't fall for these conspiracy theories. They have been around long before Brzezinski or Rockefeller. Before the tri-lateral commission there was the Freemasons and the Illuminati .Someone throughout history has opined of super secret cult-like organizations of elites who secretely pull the strings of wold leaders for the purpose of solidifying world power. But if they had that much influence then why have they not already achieved their goal of one world gvt. ?

The Illuminati ;founded by Adam Weishaupt ,a member of the House of Rothschild ,are accused of starting and financing every war since the French Revolution . The Rothschilds are featured prominently in all these alleged plots ;working behind the scenes with their various off-shoot organizations like supposedly the 'Council on Foreign Relations '(CFR) or their British counterpart the The Royal Institute of International Affairs .

One other thing you will find in these conspirac theories is alot of Jewish names because yes;most of those who are the loudest proponents of these theories are anti-semites .

Their first atttempt at world domination was allegedly the Napoleonic Wars in Europe. The Rothschilds financed both sides of the conflict . When the wars were over ;the Rothschilds convened the Congress in Vienna where they supposed that a war weary Europe that was indepted to them would form a "world gvt." The crowns of Europe were not buying it ....yet .

Eventually of course after WWI the 'League of Nations ' was attempted ;and after WWII the UN was formed ...all with the behind the scenes string pulling by the Illuminati in all their current manifestations .

The Bilderberger Group first met in 1954 at the Bilderberg Hotel in Arnhem, the Netherlands. The group has been depicted as an international cabal of the influental and the affluent: politicians, financiers, and media and business moguls; the elite of the elite. Some believe that they have dictated national policies, rigged (or outright stolen) national elections, caused wars, recessions, and ordered murders and ousters of world leaders such as American president John F. Kennedy and British Prime Minister Margaret Thatcher.

The original intention of the was to further the understanding between Western Europe and North America through informal meetings between powerful individuals. Each year, a "steering committee" devises a selected invitation list with a maximum of 100 names; invitations are only extended to residents of Europe and North America. The location of their annual meeting is not secret, and the agenda and list of participants are openly available to the public, but the topics of the meetings are kept secret: they are not published, and attendees pledge not to divulge what was discussed.
Donald Rumsfeld and Paul Wolfowitz are members. In general I would say there is no more conspiracy involved in these get togethers than there is in the annual World Economic meetings at Davos Switzerland.The Bilderberg group is not a secret org. it's membership is published for the world to see.

The Trilateral commission was formed in 1973 .It was formed to to foster closer cooperation among these core democratic industrialized areas of the world with shared leadership responsibilities in the wider international system. Rockefeller's idea for establishing the commission emerged after he had read a book entitled 'Between Two Ages 'written by Zbigniew Brzezinski ;National Security Advisor to President Jimmy Carter.

In his book Brzezinski proposed an alliance between North America, Western Europe and Japan.
"Resist as it might the American system is compelled gradually to accommodate itself to this emerging international context, with the U.S. government called upon to negotiate, to guarantee, and, to some extent, to protect the various arrangements that have been contrived even by private business."

Brzezinski like Kissenger is a foreign policy "realist" .Which means for the sake of stability you make deals with the devil .But that is for a different discussion . As you know ;American foreign policy is divided between the 'realists' and the 'ideologists'(that is a simplistic explanation but it would be too long to go into details )

But sorry conspiracy buffs. It is hard to run a secret organization when your memebership is published .If you request they will send you a complete listing .

So what do we have with these two groups . They are organizations that believe that if you get together and discuss problems collectively you just might find a solution to those problems. WOW !! what a conspiracy . You better watch out about people who believe the exchange of ideas can be influential !

What you need to examine is who is it who spouts these conspiracy theories the loudest . It used to be the 'John Birch 'society. Frankly ;they made some valid points in their time about the threat of Communism but they are an organization full of conspiracy -theorists . Robert Welch (founder of the John Birch Society) once publicly asserted that Republican Senator Robert Taft had died of cancer that had been passed on to him by Soviet operatives through "a radium tube planted in the upholstery of his Senate seat" .

Another biggie is the perenial Presidential candidate Lyndon LaRouche . LaRouche has developed a theory in which he says that an oligarchical faction within the financial community is the principal enemy of progress. This elite conspiracy, he says, predates and transcends both capitalism and socialism.

As far as Koeppl ;this is what he wrote about the war against Jihadistan :

"This is more than a war against terrorism. This is a war
against the citizens of all countries. The current elites are
creating so much fear that people don't know how to respond. But they
must remember. This is a move to implement a world dictatorship
within the next five years. There may not be another chance."

according to legend he was an insider in the Trilateral commission until he "realized "their goal was world takeover.In 1983, Koeppl warned, through Op-Ed pieces published in NEWSWEEK and elsewhere, that Brzezinski and the CFR were part of an effort to impose a global dictatorship.

"In 1983/4 I warned of a take-over of world governments being orchestrated by these people. There was an obvious plan to subvert true democracies and selected leaders were not being chosen based upon character but upon their loyalty to an economic system run by the elites and dedicated to preserving their power.

His big thing before the current war was his claim in 1985 that the shootdown of KA 007 was not an accident but a deliberat assassination of U.S. Congressman Larry McDonald











HANK1 rated this answer Excellent or Above Average Answer

Question/Answer
excon asked on 02/08/06 - A teaching moment missed


Hello Republicans:

President Bush hates the press even when he should be loving the press. I don't get it. Through his mouthpiece Scott McClelland, heres what he said:

"We support and respect the freedom of the press, but there are also important responsibilities that come with that freedom."

This is wrong - dangerously wrong. The guarantees of free expression (such as the First Amendment in our own country) include no "if," no "also," no "but." The guarantees do not ordain a "responsible" press, but a free press. This is what the Muslims, who often show no respect for the beliefs of others, must learn even if they learn it the hard way. Followers of the prophet are entitled to believe that a caricature of the man they revere is wrong. If they believe that, they should neither draw such a caricature nor look upon one drawn by someone else. They have no right to impose that belief on anyone else, Hindu, Christian or Hottentot. This is what Kofi Annan and the bowed heads of Europe and spokesmen for the State Department and the White House, and George W. Bush, should be telling the Muslims.

excon

tomder55 answered on 02/08/06:

In my post I provided a link to what John Batchelor basically hammered as appeasing rhetoric by the State Dept. I agree with him and I agree with you. I am appalled at how there is a separation from the Danes .When Rushdie wrote 'Satanic Verses ' and a death sentence was placed on him ;authors from around the free world rallied to him and demonstrated by reading passages of the book in public.

If we are trying to truely introduce the principles of a free society to the Islamic world then we should make it clear that if we don't agree with the press we have peaceful remedies that enhance the free exchange of ideas .Condi and company should be saying to them that if we don't agree with the NY Slimes then we point out our differences ;we can even mock and ridicule them ,but we don't burn down their offices or cut off the hands of their editorialists .We are not trying to preserve their culture .We are trying to change it . The only hope of avoiding all-out warfare in the Middle East is the spread of freedom.

But what example are we setting ? .In the Western world the trend is towards ever increasing restrictions on speech ;be it hate crime legislation ;politically correct language ;or campaign finance reform .

Many newspapers around Europe, in an act of defiance, have republished the cartoons. Blogs written about the story and published the pictures. But where is the MSM ..CNN ;NY Slimes.. those great defenders of freedom of the press ????
Have they been whipped into dhimmitude like the gvts of Eurabia appear to be ? The MSM in this country refuse for the same reason that CNN self-censored its reports during the reign of Saddam Hussein; a fear of the repercussions that would come if Saddam's atrocities were accurately reported. In this case the US press and the adminstration could learn a lesson from the European publishers who reprinted the cartoons as well ,as from the courage of Rushdie and Theo van Gogh.

excon rated this answer Excellent or Above Average Answer
Itsdb rated this answer Excellent or Above Average Answer

Question/Answer
Itsdb asked on 02/06/06 - Did you hear this?

Bolton Opens First Council Meeting - in Empty Room

By EDITH M. LEDERER, AP

UNITED NATIONS (Feb. 2) - U.S. Ambassador John Bolton presided over the U.N. Security Council for the first time on Thursday but failed to get the 14 members to show up on time or back his request for daily briefings on U.N. peacekeeping operations and global hotspots.

The United States took over the council's rotating presidency from Tanzania on Wednesday, and Bolton said he banged the gavel at 10 a.m. when members were supposed to begin meeting. "I was the only one in the room though," he lamented to reporters afterward. "We started just before 10:15 a.m."

President Bush waited until Congress adjourned to give Bolton a recess appointment as ambassador to the United Nations, bypassing the Senate after a standoff with Democrats who argued that the tough-talking conservative was unfit for the job. Since his arrival in August, in the throes of the U.N. oil-for-food scandal, Bolton has been demanding reform of the United Nations, especially its management.

Despite failing to get Thursday's council meeting to start on time, Bolton stressed his determination to improve the working methods of the U.N.'s most powerful body, including more regular, preferably daily briefings by the U.N. Secretariat on peacekeeping operations and other issues that could threaten international peace and security.

The Security Council's agenda depends on major global issues and demands in the 16 peacekeeping operations from Congo, Liberia and Sudan to Lebanon, Cyprus and Haiti.

"I think daily briefings constitute a form of intellectual discipline. Starting on time is a form of discipline," he said.

"The council is responsible for peacekeeping activities, and I think we need to do a better job of collective decision-making," he added.

Diplomats said other council ambassadors questioned the need for daily briefings and the ability of the overstretched U.N. peacekeeping department to organize them - and several demanded that any briefings be in all six official U.N. languages. The diplomats spoke on condition of anonymity because Thursday's meeting was closed.

Bolton said the council would meet again on Friday to continue its discussion about working methods.

"It's not a bad idea," Greece's U.N. Ambassador Adamantios Vassilakis said of modernizing the council's operation. "Maybe not on a daily basis, but it's not a bad idea. We have to do something to improve things."

Japan's U.N. Ambassador Kenzo Oshima welcomed new ideas for the council.

"The question is whether the frequency is right," he said. "But the idea of having some regular briefing, and some discipline in these matters is what everybody, I think, welcomes."

~~~~~~~~~~~~~~~~~~~~~~~~~~~~~~~~~~~~~~~~~~~~~~~~~~~~~~~

Now, for the spin...

John Bolton An Embarrassment on World Stage: Presides Over Empty Room!

@ 10:28 PM (3 days, 17 hours, 3 minutes ago)

John Bolton continues to show why he's such an embarrassment to the United States on the international stage.

Bolton couldn't even get confirmed to his position in the Republican-led Senate, and so to get his man, Bush resorted to a recess appointment of Bolton.

Today, in what amounted to a big "screw you" to Bolton by other diplomats, he presided over an empty U.N. Room. Bolton said "I brought the gavel down at 10. I was the only one in the room." He was trying to open his first meeting as head of the Security Council.

~~~~~~~~~~~~~~~~~~~~~~~~~~~~~~~~~~~~~~~~~~~~~~~~~~~~~~

Let's see, the UN has no teeth - or backbone - and Bolton wants the Security Council to do their job and stay on top of things. To Japan and Greece "it's not a bad idea," just don't expect us to be on time or even be there every day, it's just the Security Council for crying out loud.

To the 'progressives' it's an 'Embarrassment on World Stage' and a big 'screw you.'

If you ask me, and I know you didn't, I say go get 'em John, whip those people into shape. The embarrasment is that nobody but Bolton thought a Security Council meeting was important enough to show up.

What do you say?

tomder55 answered on 02/07/06:

punctuality is respect for the time of others .

I wonder if they will bother to show up to hear the IAEA "referal" of Iran in March or is their decision already pre-determined .

Itsdb rated this answer Excellent or Above Average Answer

Question/Answer
sissypants asked on 02/06/06 - cartoons, drama , outrage, violence, murder.......

point proven! if there isn't something to murder over, people will drum things up to be pissed off at to the point of murder!?! and this is sanity?

tomder55 answered on 02/06/06:

in this country a crucifix emersed in urine ;a sculpture of Mary mother of Jesus is created using elephant dung ...both gets Federal funding to be displayed in a museum . Yes we protest it ;yes we threaten boycott ;yes we become politically active .We do not use that as an excuse to murder . But have you seen the cartoons published in this country anywhere else except on the web? I guess we now knows what influences the MSM .

Itsdb rated this answer Excellent or Above Average Answer
sissypants rated this answer Excellent or Above Average Answer

Question/Answer
Itsdb asked on 02/03/06 - Who broke the law?

The NY Times or the president.

A fascinating article in Commentary Magazine lays out the case beautifully. Due to its length I will only offer excerpts - much of which has already been covered by experts here.

The article notes that according to section 798 of the Espionage Act, "Whoever knowingly and willfully communicates, furnishes, transmits, or otherwise makes available to an unauthorized person, or publishes, or uses in any manner prejudicial to the safety or interest of the United States or for the benefit of any foreign government to the detriment of the United States any classified information... (3) concerning the communication intelligence activities of the United States or any foreign government...Shall be fined not more than $10,000 or imprisoned not more than ten years, or both."

It points out that "With the bill narrowly tailored in this way, and with concern for public speech having thus been respected (in the words of Edgar and Schmidt), Section 798 not only passed in Congress but, perhaps astonishingly in hindsight, won the support of the American Society of Newspaper Editors. At the time, the leading editors of the New York Times were active members of that society."

As for the Whistleblower defense...

"As for whistleblowers unhappy with one or another government program, they have other avenues at their disposal than splashing secrets across the front page of the New York Times. The Intelligence Community Whistleblower Protection Act of 1998 shields employees from retribution if they wish to set out evidence of wrongdoing. When classified information is at stake, the complaints must be leveled in camera, to authorized officials, like the inspectors general of the agencies in question, or to members of congressional intelligence committees, or both. Neither the New York Times nor any other newspaper or television station is listed as an authorized channel for airing such complaints."

So much for that excuse.

"If it (the NY Times) has not inveighed directly against the war on terrorism, its editorial page has opposed almost every measure taken by the Bush administration in waging that war, from the Patriot Act to military tribunals for terrorist suspects to the CIA renditions of al-Qaeda operatives to the effort to depose Saddam Hussein. Mr. Bush and his attorney general, says the Times, have put in place a strategy for a domestic anti-terror war that [has] all the hallmarks of the administrations normal method of doing business: a Nixonian obsession with secrecy, disrespect for civil liberties, and inept management. Of the renditions, the paper has argued that they make the United States the partner of some of the worlds most repressive regimes; constitute outsourcing torture; and can be defended only on the basis of the sort of thinking that led to the horrible abuses at prisons in Iraq. The Timess opposition to the Patriot Act has been even more heated: the bill is unconstitutionally vague; a tempting bit of election-year politics; a rushed checklist of increased police powers, many of dubious value; replete with provisions that trample on civil liberties; and plain old bad law.

In pursuing its reflexive hostility toward the Bush administration, the Times, like the Chicago Tribune before it, has become an unceasing opponent of secrecy laws, editorializing against them consistently and publishing government secrets at its own discretion. So far, there has been only a single exception to this pattern. It merits a digression, both because it is revealing of the Timess priorities and because it illustrates how slender is the legal limb onto which the newspaper has climbed.

The exception has to do with Valerie Plame Wilson.

The NY Times - guilty.

Question? Will the left ever admit to their blatant hypocrisy? Will the Times be held accountable?

Steve

tomder55 answered on 02/04/06:

perhaps this time the editor of the Slimes will spend some time behind bars .Porter Goss belatedly called for the Senate to investigate the leaks ,saying they have caused "severe damage" to the agency's operations.

Speaking of the Plame/Scooter Libby investigation there is an interesting development in the case reported by Byron York in National Review .

As part od discovery ,Libby's lawyers sent a letter to Fitzgerald requesting "Any assessment done of the damage (if any) caused by the disclosure of Valerie Wilson's status as a CIA employee." They also requested "All documents, regardless of when created, relating to whether Valerie Wilson's status as a CIA employee, or any aspect of that status, was classified at any time between May 6, 2003 and July 14, 2003." (The dates during t the period when some Bush-administration officials discussed Wilson with reporters.)

Fitzgerald responded "A formal assessment has not been done of the damage caused by the disclosure of Valerie Wilson's status as a CIA employee, and thus we possess no such document" .He also told Libby's lawyers that he did not think the information was relevent .(??????)He went further to say that he had NOT sought, much less obtained, 'all documents, regardless of when created, relating to whether Valerie Wilson's status as a CIA employee, or any aspect of that status, was classified at any time between May 6, 2003 and July 14, 2003." He then went on to question the relevence to his case (perjury and obstruction of justice).


The thing is ...when Fitzgerald announced the indictment it appeared that that information was very relevent to him .He talked about Plame's status in the CIA and how serious it was that her "cover was blown"..even though the charges he was presenting appeard to have nothing to do with Libby's involvement in blowing her cover . Now he says that her status was indeed not relevent.So all the talk about national security ;blowing a CIA agents cover ;damage to recrutment was all window dressing intended to nail Libby in the court of public opinion but irrelevent evidently in a court of law.

Fitzgerald now admits that no harm was done by Libby .That his rhetoric at the press conference was just that...empty rhetoric which was more likely intended to justify the hours and expenses that he spent on his fishing expedition ...damn ! He never even SOUGHT documentary evidence to make a determination !! This case is more and more resembling the Martha Stewart rail-roading .

Itsdb rated this answer Excellent or Above Average Answer

Question/Answer
ETWolverine asked on 02/03/06 - Excon - The 4th Amendment

Ex,

You keep bringing up the 4th Amendment as your explanation of why Bush is violating the law with the NSA eavesdropping program. You have claimed that neither I nor the other conservatives on this board understand what the 4th Amendment says. And now you have gone on to accuse 4 of the most experienced judges in the entire world of not understanding the 4th Amendment either.

So in the interest of learning, I have decided to look at the 4th Amendment's text and see what it says in exacting detail.

"The right of the people to be secure in their persons, houses, papers, and effects, against unreasonable searches and seizures, shall not be violated, and no Warrants shall issue, but upon probable cause, supported by Oath or affirmation, and particularly describing the place to be searched, and the persons or things to be seized."


Lets take the first part of that:

"The right of the people to be secure in their persons, houses, papers, and effects, against unreasonable searches and seizures, shall not be violated,

That word, "unreasonable", is a key point. Is there anything "unreasonable" about search and seisure of evidence of espionage or terrorism? I don't think so, and neither do you, based on what you have said in the past. Only UNREASONABLE searches are prohibbited by the constitution. Intercepting enemy communications is not only not unreasonable, it is the very least that can be expected of our military/intelligence people. This is proven by the hoopla that arose from the fact we should have had enough information to prevent 9-11, but the information wasn't shared between agencies. Nobody argued that the information regarding 9-11 was obtained "illegally"... in fact quite the opposite. It was argued that regardless of who obtained the information and how it was obtained, it should have been shared between agencies to prevent 9-11 from occuring. And that is a position that I believe you agree with, as do I.

So, listening in on enemy communications (domestic or foreign) does not meet the criterion of "unreasonable", since it is emminently reasonable to do so. It is not reasonable to do so in criminal cases, since even criminals have the right to expect privacy in their domocile. Enemy combatants do not reasonably have that expectation.

Now, for the next part of the text:

"and no Warrants shall issue, but upon probable cause, supported by Oath or affirmation, and particularly describing the place to be searched, and the persons or things to be seized."

The text here is a bit ambiguous. It does NOT say that you cannot search a domicile without a warrant. It only states that if you DO issue a warrant, it must be based on probable cause and name the places to be searched and things to be searched for. That's an important point. If you want to be a REAL originalist (as you have claimed in the past), then you should NOT be claiming that you need a warrant for every search, since the Constitution doesn't say that it does. The courts have determined that we do need a search warrant in every criminal case where a search is performed, but that is simply a judicial interpretation, not the actual text of the Constitution. And the same courts have been consistantly making the same determination for the past 40-50 years that the 4th Amendment does NOT apply to enemy communications, especially during a time of war (but even in peacetime). So, unless you are prepared to explain, in legal terms, why you accept the court interpretation that Warrants are required in all criminal cases, but don't accept the same court's interpretation that warrants are not required cases of enemy communications, consistancy would demand that you accept both interpretations as equally legitimate. Or else explain the inconsistancy (in terms other than "it's not fair", please).

Elliot

tomder55 answered on 02/03/06:

John Batchelor talks this issue from a historical perspective. Yesterday he mentioned that in the winter of 1777-78 while Washington tried to keep the army together at Valley Forge ,the Continental Congress would meet secretly and plot to remove Washington and replace him .His point is that there has always in this country been a tension between the executive and the Congress over authority . There have been times of weak Presidencies but during times of war ;the executive has consistently won these battles. There have been frequent cases going back to the Washington Adm. that someone has accused the executive of exceeding their authority .The courts when called in to mediate has generally sided with the President's interpretation of his war powers .

So you have in this case historical precident backed by numerous court decisions ;and a genuine Congressional declarations of war where Congress gave the President authority :

to use all necessary and appropriate force against those nations, organizations, or persons he determines planned, authorized, committed, or aided the terrorist attacks that occurred on September 11, 2001, or harbored such organizations or persons, in order to prevent any future acts of international terrorism against the United States by such nations, organizations or persons.

ETWolverine rated this answer Excellent or Above Average Answer
excon rated this answer Excellent or Above Average Answer
jnlomonte rated this answer Excellent or Above Average Answer

Question/Answer
excon asked on 02/03/06 - Whip that hammer


Hello:

If you were in charge of a party that was accused of screwing the public, would you put a guy in charge by the name of boner?

excon

tomder55 answered on 02/03/06:

He wasn't my first choice ; John Shadegg was ;but he is better than Blunt who actually said that earmarks were a good thing . My problem with Boehner is that he is perceived as having ties with lobbyists . There is one story where he got caught handing out checks from tobacco lobbyists to members of Congress in 1996 . But on the positive side from a strategery point of view ;he is an Ohio Republican and they look to need the biggest help in the 2006 elections .Boehner is also one of the few who last year did not put any personal pork-barrel projects into the bloated transportation bill. In 2002, he led the fight against the pork filled farm bill. If he brings the Republicans back to the spirit of the ' Contract with America' instead of this 'spend like drunken Democrat' mode they've been in then he's the man .

excon rated this answer Excellent or Above Average Answer
Itsdb rated this answer Excellent or Above Average Answer

Question/Answer
excon asked on 02/02/06 - Methanol from Hemp


Hello:

So, we're gonna do more biofuels, huh? Ok, but corn ain't very good for that.

Hemp is. Cannabis, by far and uniquely among plant species, produces fuel-energy cheaper per BTU than fossil fuels and uranium. We could replace them both, AND the price of gasoline and electricity would DROP.

I can see it now, endless fields of marijahoochie. Only problem is, this pot doesn't get you high.

So, let's go America. Put a hippy in your tank.

excon

tomder55 answered on 02/02/06:

but we are already pumping major $$ subsidizing corn and sugar crops .Might as get something for our "investment". I cannot find anything that refutes your contention that corn is a poor source for bio-fuel . Brazil has a growing industry using sugar cane (distilling 4 billion gallons );India turned out 500 million gallons made from sugarcane. France produced over 200 million gallons from sugar beets and wheat.So our lack of foresight has put us behind the game .The question I have is at what point does converting land from food agriculture to energy farming negatively affect the food supply ? Also there apprears to be environemental and efficiency questions (energy required to produce it .I think that is where the question of corn to ethanol efficiencty arises in your post ) about bio-fuels .

There was before petro-fuels caught on a degree of success using hemp . Henry Ford actually was convinced that a renewable source of energy would be the key to the success of the automobile .He found that 30% hemp seed oil is usable as a high-grade diesel fuel and that it could also be used as a machine lubricant and an engine oil.

So grow away . I have no problem with the use of industrial Hemp in many applications . If it was good enough for George Washington ;Thomas Jefferson and Henry Ford ....it's good for the USA.

excon rated this answer Excellent or Above Average Answer
Itsdb rated this answer Excellent or Above Average Answer
sissypants rated this answer Excellent or Above Average Answer

Question/Answer
sissypants asked on 02/02/06 - the current legal drug pushing industry

i for one am sick and tired of drugs being pushed on commercials and to my doctors by all the drug companies. what is happening is the drug compamies are going in and pushing their drugs to the doctor who then wants to try it out on unsuspecting patients who all of a sudden have their meds changed in their next visit to their doctor. Then there is the commericals that make these drugs sound like wonderful things but most of us don't even know what aliment they are supposed to work on is. And now they say the biggest problem with drug abuse right now is with prescription drugs. DUH....i wonder why!

tomder55 answered on 02/02/06:

I fully agree with you and have had recent experience in my family that shows how dangerous a practice it is ;and frankly how gullible and un-informed the doctors are of the medicines they prescribe. I know doctors take years of schooling to perfect their craft but when it comes to medicines they are silly putty in the hands of Pharmaceutical 'Detail' salespersons and I frankly have to question the ethics of the system.

But ...I have not fully thought out a workable alternative .

sissypants rated this answer Excellent or Above Average Answer

Question/Answer
arcura asked on 02/01/06 - This was probably written by a Republican.

If you are a Democrate, how would you rewrite it???
<><><>
The Little Red Hen-Modern version

Once upon a time, on a farm in Texas, there was a little red hen who scratched about the barnyard until she uncovered quite a few grains of wheat.

She called all of her neighbors together and said, "If we plant this wheat, we shall have bread to eat. Who will help me plant it?"

"Not I," said the cow.

"Not I," said the duck.

"Not I," said the pig.

"Not I," said the ! goose.

"Then I will do it by myself," said the little red hen. And so she did;
The wheat grew very tall and ripened into golden grain.

"Who will help me reap my wheat?" asked the little red hen.

"Not I," said the duck.

"Out of my classification," said the pig.

"I'd lose my seniority," said the cow.

"I'd lose my unemployment compensation," said the goose.

"Then I will do it by myself," said the little red hen, and so she did.
At last it came time to bake the bread.

"Who will help me bake the bread! ?" asked the little red hen.

"That would be overtime for me," said the cow.

"I'd lose my welfare benefits," said the duck.

"I'm a dropout and never learned how," said the pig.

"If I'm to be the only helper, that's discrimination," said the goose.

"Then I will do it by myself," said the little red hen. She baked five loaves and held them up for all of her neighbors to see. They wanted some and, in fact, demanded a share. But the little red hen said, "No, I shall eat all five loaves."

"Excess profits!" cried the cow.

"Capitalist leech!" screamed the duck.

"I demand equal rights!" yelled the goose.

The pig just grunted in disdain.

And they all painted "Unfair!" picket signs and marched around and around the little red hen, shouting obscenities.

Then a government agent came, he said to the little red hen, "You must not be so greedy."

"But I earned the bread," said the little red hen.

"Exactly," said the agent. "That is what makes our free enterprise system so wonderful. Anyone in the barnyard can earn as much as he wants. But under our modern government regulations, the productive workers must divide the fruits of their labor with those who are lazy and idle,"

And they all lived happily ever after, including the little red hen, who smiled and clucked, "I am grateful, for now I truly understand,"

But her neighbors became quite disappointed in her. She never again baked bread because she joined the "party" and got her bread free.

And all the Democrats smiled. 'Fairness' had been established.
Individual initiative had died, but nobody noticed; perhaps no one cared.....as long as there was free bread that "the rich" were paying for.

Bill Clinton is getting $12 million for his memoirs.

Hillary got $8 million for hers.

That's $20 million for memories from two people, who for eight years, repeatedly testified, under oath, that they couldn't remember anything.

IS THIS A GREAT COUNTRY, OR WHAT?

tomder55 answered on 02/02/06:

todays Dems . hmmmm

there once was a farm in Texas that had a hen of privilage . Through generations that hen's family had received the best grain ;the best education and the most fertile land from the owners of the farm . None of the other farm yard animals were of such status . The hen hired them to till the land ;plant all the seeds ;harvest the wheat ; and finally bake bread . Although the wheat had produced 5 loaves of bread the greedy hen only gave the rest of the animals 1 loaf to divide amongst themselves even though it was impossible for the hen to consume the other 4 loaves herself.When she had consumed her fill some of the bread soiled and was no longer eatable .That bread was thrown into the trough (tax the rich) and dispersed amongst the rest of the starving animals (welfare ).

arcura rated this answer Excellent or Above Average Answer
CeeBee2 rated this answer Excellent or Above Average Answer

Question/Answer
congress123 asked on 02/01/06 - bills
bills

how many votes did the medicare prescription drug benifit pass committe by?

tomder55 answered on 02/02/06:

in the Senate the 54 to 44 vote was not along party lines -- 11 Democrats voted in favor and nine Republicans voted no.In the Houes of Reps it passed with 96% of Republicans supporting, 97% of Democrats opposing.Ayes: 221 (51%) ;Nays: 208 (48%)
Not Voting: 6 (1%)

Here is the roll call




congress123 rated this answer Excellent or Above Average Answer
ETWolverine rated this answer Excellent or Above Average Answer

Question/Answer
Itsdb asked on 02/01/06 - Grade the speech(es)

So what did you think of the presentations last night?

I'd give the president an A- for presentation and an B+ for substance. He deftly rubbed a thumb in the eye of a number of critics last night - I enjoyed that.

I'd give Kaine a B- with a gold star for audacity. I actually liked some of what he said, particularly the part about "moving ahead by focusing on service, competent management and results." Who can argue with that?

His opening remarks were rather disingenuous though...

"I worked as a missionary in Honduras when I was a young man and I learned to measure my life by the difference I can make in someone elses life. Coretta Scott King embodied that value and tonight, as a nation, we mourn her passing. Our faith and values teach us that there is no higher calling than serving others."

Puhleeeeeease. No knock on Kaine, but the Democratic party would be hard pressed to show they 'embody' those values. The only time "faith and values" come into play is when it's to their political advantage. How did their shameless behavior in the Alito hearings and debate show the importance of serving others? You want to serve? Sit in front of us for a few days while we run you through the shredder.

The democrats in the chamber get an A+ for chutzpah. Giving themselves a standing ovation - need I say more?

Steve

tomder55 answered on 02/01/06:

"Far from being a hopeless dream, the advance of freedom is the great story of our time. In 1945, there were about two dozen lonely democracies on Earth. Today, there are 122. And we are writing a new chapter in the story of self-government--with women lining up to vote in Afghanistan . . . and millions of Iraqis marking their liberty with purple ink . . . and men and women from Lebanon to Egypt debating the rights of individuals and the necessity of freedom. At the start of 2006, more than half the people of our world live in democratic nations. And we do not forget the other half--in places like Syria, Burma, Zimbabwe, North Korea, and Iran--because the demands of justice, and the peace of this world, require their freedom as well."

That was the highlight .the rest of it was not what I was looking for . He did not propose enough budget cuts .He offered too many feel good programs .Strong initiatives from last year have become bi-partisan study groups . After the disaster of No child left behind he offered more spending on education . At the local level it has been proven over and over again that spending does not equal results .At the Federal level could it possibly be better ? After what is quickly becoming a disaster ;the Medicare Drug program he made proposals to futher subsidize health care .We are incrementally getting Hillary care .

He did not make a strong enough case for energy independence or for the need for border security .

So for me it was a tale of 2 speeches .I generally applaud his foreign policy intitiatives but am longing for the days when Reagan came in with a huge book that represented the budget and told Congress to sharpen their pencils .

Itsdb rated this answer Excellent or Above Average Answer

Question/Answer
CeeBee2 asked on 01/31/06 - Which one?

If you are a US citizen, which do you perceive will most affect your everyday life, Bernanke as the new head of the Frederal Reserve, or Alito, the newest Supreme Court Justice? Why?

tomder55 answered on 02/01/06:

That is an excellent question .

Start with the Supreme Court . The days of Borking nominees are over ;that is a good thing . I think there is a balance in the court ..the swing vote shifted to Kennedy instead of OConner but there is still a swing vote there . The Dems. were ultimately wise not to draw the line in the sand with Alito. He may suprise them ,but the real fight for the Dems will be if one of the liberal judges or Kennedy leaves the court. Had they fillibustered Alito they would have lost and that play would not be there for them when Bush nominated someone like Janice Brown to the court .

I thought overall that Greenspan was a great Fed Chair but his time has gone . He has really cooled down an economy that could be doing much better by his incremental rate hikes .I do not know if he undersatands the global economy . The economy that thrives will have funds available . He mistook the cost of gas as an inflationary threat when in truth it was putting the very brakes on the economy that his increases were intended to do .
I think in short that Bernanke will have a much bigger impact than Alito because what he says and does on a daily basis affects the economy . He can determine how much money the homeowner has to spend and how much business has to invest .On his recommendation is alot of weight. Greenspan's luke warm to cool endorsements of Social Security reform helped shape the debate last year .

I have not heard any negative about Bernanke.His ideas of being more transparent about Feds .communication with the financial markets is welcome .Greenspan neverset targets so up to the day of the Fed. meetings no one was really sure what Greenspan would do. Bernanke will more likely set flexible target inflation rates and will not move unless those lines are crossed. But everyone will know what the targets are .I think his openess will be a welcome change .

arcura rated this answer Excellent or Above Average Answer
CeeBee2 rated this answer Excellent or Above Average Answer
excon rated this answer Excellent or Above Average Answer

Question/Answer
Itsdb asked on 01/31/06 - The real Hillary is finally emerging

Following Hillary's "plantation" remarks, she seems to be moving further away from her moderate incarnation and closer to her liberal base...

Americans are growing 'impatient" as they wait for a woman to be elected president, 2008 presidential candidate Hillary Clinton said Saturday night.

"People are saying,' Well, at least we're ready,'" Clinton told interviewer Jane Pauley, as the two held a public chat for charity in San Francisco.

"There's a feeling that it's time," she added.

Then, in quotes picked up the New York Sun, the former first lady said she detected "a certain impatience" to see a female president following the election of women to similar roles in other countries.

Despite Mrs. Clinton's claims, a Gallup poll found last week that 51 percent of Americans had already made up their minds not to vote for her.

The top Democrat offered the comments after Pauley noted that President Bush had recently said she'd make a "formidable" candidate.

Mrs. Clinton declined to return the compliment however, and instead blasted Mr. Bush for mishandling the rebuilding of New Orleans after it was destroyed by Hurricane Katrina.

"I think that basically we are now watching a deliberate policy of neglect take root," Hillary complained.

She then suggested that the White House didn't want to rebuild New Orleans because "all those Democrats might come home."

~~~~~~~~~~~~~~~~~~~~~~~~~~~~~~~~~~~~~~~~~~~~~~~~~~~~~~~

Anyone here with that feeling "it's time?"

~~~~~~~~~~~~~~~~~~~~~~~~~~~~~~~~~~~~~~~~~~~~~~~~~~~~~~~

"Mrs. Clinton said she suspected that the assignment of President Bush's top political adviser, Karl Rove, to oversee the relief effort indicated that political mischief was afoot. "Cynical minds might suggest that the destruction of the Democratic vote in Louisiana was a mixed blessing. If you rebuild New Orleans, all those Democrats might come home..."

~~~~~~~~~~~~~~~~~~~~~~~~~~~~~~~~~~~~~~~~~~~~~~~~~~~~~~~

Whose cynical mind might she be referring to, her own?

Steve

tomder55 answered on 01/31/06:

yup ;I can see a President Condi Rice.

Dick Morris has her pegged. She has to stay "tough" on foreign affairs but that does not sate the moonbats .So whenever possible she has top cover her ample left flank . It is becoming increasingly difficult to do so with the likes of Cindy Sheehad and the moveon loons pushing the party off the left cliff. Michelle Malkin calls it the being "Kos"sified or being a "Kos"sack in honor of the lefty blog Daily Kos which has increasingly put pressure on the Democrats to move to the extreme.

For the record ;Evita will emerge from her reelection campaign unscathed and with a huge war chest .(the RINOs in NY cannot even find a decent sacrifical lamb ). She will start out as the frontrunner but I do not think she has staying power . My early guess is that the Al Gore reinvention tour will propel him to the nomination.

"Mrs. Clinton said she suspected that the assignment of President Bush's top political adviser, Karl Rove, to oversee the relief effort indicated that political mischief was afoot. "Cynical minds might suggest that the destruction of the Democratic vote in Louisiana was a mixed blessing. If you rebuild New Orleans, all those Democrats might come home..."

What a dopey thing to say ! She must be sucking some of that Ray Nagin chocolate milk again ! . The thing I am perplexed about Bush is his admiration of Bill Clinton . I can assure him it is not reciprocal.

Itsdb rated this answer Excellent or Above Average Answer
labman rated this answer Excellent or Above Average Answer

Question/Answer
Itsdb asked on 01/31/06 - Conspiracy of the day

The following letter appeared in our paper today...

Skies not so friendly these days

Jan. 23 began as a sparkling clear day in Amarillo. It ended with a colorful sunset, thanks to "clouds" in the sky.

Those "clouds" came from airplanes spraying substances that don't dissipate quickly, as normal contrails do. Also, they came from many more airplanes than normally fly in and out of Amarillo. The "cloud" trails often appear in criss-cross patterns, always over populated areas.

This only could be happening with governmental approval, and perhaps sponsorship, which means our taxes are paying for it.

There are various theories about what's been going on. Type in "chemtrails" on the Internet, and you'll find a lot of information, much of it frightening.

A variety of explanations are presented. One possibility is that this is being done for our protection, albeit with harmful side-effects for some.

When I mention this "chemicalizing" of our skies to people here, most want to dismiss it. "I've got enough to worry about already" or "If the government condones it, it must be OK" seem to be the prevailing attitudes.

Why hasn't the government offered an explanation? As long as we're paying for it, it's our right to know what's going on. I invite others sharing this concern to contact me at mingls@arn.net.

Hunter Ingalls

Amarillo

~~~~~~~~~~~~~~~~~~~~~~~~~~~~~~~~~~~~~~~~~~~~~~~~~~~~~~~

Alrighty then. I guess I haven't been as on top of things as I thought as this conspiracy was new to me. So what of it? Hoax or reality? 'Chemtrails or Kooktrails? Are we now " Electro-Sensitive" thanks to "the so-called "Election" of the bush crime family?"

Steve

P.S. Hunter offered his email so I obliged him here also.

tomder55 answered on 01/31/06:

wow ;someone from the local paper has been up late at night listening to Coast to Coast with George SNORY !!

http://www.carnicom.com/coast1.htm

http://www.carnicom.com/

it is my suspicion that they are trying to seed the clouds to bring you from drought. Hope they don't go to far and create another cat. 5 like they did last fall .

Itsdb rated this answer Excellent or Above Average Answer
kindj rated this answer Excellent or Above Average Answer

Question/Answer
excon asked on 01/31/06 - Obscene profits


Hello:

I'm actually FOR unbridled capatalisim. However, even unbridled, I doubt Exxon Mobile would have made $36.13 billion all by their lonesome.

I think the fix is in.

excon

PS> Assuming it would have been unbridled, we would be energy independant. But, like I say, the fix is in.

tomder55 answered on 01/31/06:

I wonder how the oil companies would've fared if they had not had all those mergers in the 90s ? why is there Exxon -Mobile ;BP-Amaco-Arco;Chevron-Texaco;Phillips-Tosco and Marathon ? These 5 companies control 15 % of the worlds production . More than Saudia Arabia and Kuwait combined . Think about that ;who's got who over a barrel ? Unbridaled capitalism means a little more competition then what the huge companies face .(I wonder actually how much competition goes on between them frankly ). Now part of it is our fault . We do not allow for an expanding market (we discussed this during Katrina )by restricting the ability of these companies to invest in infrastructure . But I would prefer if the energy companies were subject to more competition as well as to alternatives to their product . I am on record that we need a Manhattan Project like effort with the goal to make us energy independent . The fact that the world energy supply is unstable and we will be competing with growing economies for our share makes this a compelling concern in my book.

excon rated this answer Excellent or Above Average Answer
Itsdb rated this answer Excellent or Above Average Answer

Question/Answer
excon asked on 01/31/06 - Intelligence Failure - AGAIN???


Hello Busharinos:

I know why I didn't know that Hamas was going to win, but how come our revamped intelligence agencies didn't know?

excon

tomder55 answered on 01/31/06:

I don't know ...Why did the vaunted Israeli intelligence not see it ? Why did they not see the folly of the Gaza withdrawal and how Hamas would exploit it? Why didn't you ? You called it what ? ;a strategic repositioning ? Why did they not see the phoniness behind the Hamas hudna ? The Israeli intelligence is supposed to be on top of this game with assets on the ground speaking the language . Do you think the US has that capabilility yet ;a capability that Israel has had for years ?

Now what do I think ? I think we have been living in a fantasy for years regarding the Palestinians . We thought ;hoped against hope that first Arafat and then Abbas could be bribed into being partners for peace ...So did the Israelis. Everyone said the same thing . Give Abbas a chance to crack down on terrorism and to disarm them . Let's just funnel more money into his kleptocracy and that would solve everything . Disengage as a show of faith and then the Palestinians would be more than willing to follow the 'road map'.

Meanwhile what little servces that a normal gvt. provides was being delivered by Hamas as Fatah continued to line their pockets with international aid. In retrospect it should've been a no brainer how this election would turn out .

I have no recent news to update on the status of reform within the CIA except to note that the Goss initiatives are still being met by opposition amongst the careerists inside the agency who still work to undermine policy .

Coni Rice recently made a major address about reforms she intends to initiate in State . They include removing assets from comfy European posts to where the action is .

To advance transformational diplomacy, we are and we must change our diplomatic posture. In the 21st century, emerging nations like India and China and Brazil and Egypt and Indonesia and South Africa are increasingly shaping the course of history. At the same time, the new front lines of our diplomacy are appearing more clearly, in transitional countries of Africa and of Latin America and of the Middle East. Our current global posture does not really reflect that fact. For instance, we have nearly the same number of State Department personnel in Germany, a country of 82 million people that we have in India, a country of one billion people. It is clear today that America must begin to reposition our diplomatic forces around the world, so over the next few years the United States will begin to shift several hundred of our diplomatic positions to new critical posts for the 21st century. We will begin this year with a down payment of moving 100 positions from Europe and, yes, from here in Washington, D.C., to countries like China and India and Nigeria and Lebanon, where additional staffing will make an essential difference.


She went on to say the rahter obvious ;that we need people who can speak the language :

In the Middle East, for example, as you well know, a vast majority of people get their news from a regional media network like Al Jazeera, not from a local newspaper. So our diplomats must tell America's story not just in translated op-eds, but live on TV in Arabic for a regional audience. To make this happen, we are creating a regional public diplomacy center. We are forward deploying our best Arabic-speaking diplomats and we are broadly coordinating our public diplomacy strategy both for the region and from the region. .............

More and more often, over the course of this new century, we will ask the men and women of the State Department to be active in the field. We will need them to engage with private citizens in emerging regional centers, not just with government officials in their nations' capitals. We must train record numbers of people to master difficult languages like Arabic and Chinese and Farsi and Urdu.


She also correctly pointed out that there is a better need for coordination between State and the military . Many of the SNAFUs in Iraq was due to competing visions by the Defense and State Dept.

She threw down the gauntlet to careerists at State . To advance in the agency in the future will require them to take on these assignments .

In addition, to advance in their careers, our Foreign Service Officers must now serve in what we call hardship posts. These are challenging jobs in critical countries like Iraq and Afghanistan and Sudan and Angola, countries where we are working with foreign citizens in difficult conditions to maintain security and fight poverty and make democratic reforms.

I think in the long run the reforms will succeed . But lets face the facts ;the gutting of the CIA began shortly after Nixon left office and continued throughout the Clinton Adm. The State Dept is full of old cold warriors . It will take time to bring intel and State up to where they should be .I hope we have the time .





excon rated this answer Excellent or Above Average Answer
Itsdb rated this answer Excellent or Above Average Answer

Question/Answer
excon asked on 01/31/06 - Winning the war in Iraq


Hello righty's:

I always thought of conservatives as being rather pragmatic, whereas I classify liberals as being wishful thinkers.

I know you say we are "winning" in Iraq, but you should pardon me if I am skeptical. I offer only two facts. To me, they indicate that we are NOT winning. Just who are the wishful thinkers in America?

1. Today is day 1,050 of the Iraq war. Day 912 of WW2 was D-Day. Iraq doesn't look any closer to being pacified than it did three years ago.

2. Most recent democratic elections held in the Middle East have produced victory for the terrorists. The jury is still out on the Iraqi election.

excon

tomder55 answered on 01/31/06:

as you know I view the war in Iraq not as a single event but in the context of a greater war against Jihadistan . I think you are wrong about you're assesment of the progess re :pacification (as you call it ) and about the Iraqi election in the light of the recent Palestinian election where the people were frankly offered a choice of one of 2 terrorist organizations .

First the Iraqi election . It is true that an "islamist " block took a large share of the vote .But I see no indication that the people desire a new Califate as bin Laden and Zarqawi propose or a mullocracy like the Iranian model . Right after the vote the various factions sat down and began intense negotiations about the divisions of power . If there was a danger of Civil War ,or it was the uncompromising will of the people to be led by theocrats then the Islamists would've declared victory and that would've been the end of it . I think it shows instead that the Iraqi model will not be a fanatical approach .

Zarqawi's goal is to start a civil war in order to mobilize the Sunni community.
But Sunni support of Al-Qaeda is disintegrating in Iraq.Al-Qaeda in Iraq is in disarray The Sunnis have for the most part been persuaded the join the political process. Fighting between Al-Qaeda and elements of the insurgency, and al-Qaeda and tribal groups, has been occurring in Iraq for some time. Reports have been trickling in for well over a year of incidences of open warfare between the groups. In the Qaim region of Western Iraq members of the Albu Mahal tribe rose up against Al-Qaeda .The Albu Mahal tribe is now an ally of the Iraqi government, and provides the majority of the troops for the Desert Protection Force, which provides for local security and act as scouts for Iraqi Army and U.S. Marines operating in the area. Further incidents of fighting occurred in the cities of Husaybah and Mosul. In Ramadi,Sunnis openly took up arms against Al-Qaeda for threatening their Shiite neighbors.

Sure there is a handfull of what Bush calls rejectionist Sunnis who need to be defeated (at most arond 20,000),but the vast majority of the Sunnis have brought into a political solution .If the Shia make the mistake of cutting off the Sunni provinces from the benefits of the natural resourse then sure they will fight to the death .But I think the level of negotiations that are ongoing shows that both sides are ready to forge a free republic of Iraq.

I think there will be a significant reduction in coalition troops in the next year as the Iraqi security forces in Bush's words "step up" . But I have maintained and continue to do so that in the end the US will make a deal with the new Iraqi gvt. to garrison American troops in the country in one or more military bases.Although most Iraqis would prefer if we were to leave for good they understand that some foreign military presence will be required for them to secure themselves from outside threats .They will ask us to stay because it serves their strategic interest to do so ,and we will stay because it serves our strategic interests .

excon rated this answer Excellent or Above Average Answer

Question/Answer
itaks2tano asked on 01/29/06 - kidnapping or legit jailing of women in Iraq

I read the US military was arresting the wives of suspected terrorists in Iraq.What do you think about the US military taking female hostages (including a young breastfeeding mother) in hopes of getting their terrorist husbands to trun themselves in? Doesnt that seem kind of like the common terrorist tactic of kidnapping?

tomder55 answered on 01/30/06:

Of course the scum at AP couldn't wait to release the documents they obtained even though the information is fragmented and incomplete .I believe that at least in one of the 2 cases cited the women detained turned out to be a collaborator .

But demands by kidnappers of Jill Carroll that the US military release all female Iraqi detainees have put the incidents in a new light. There is nothing wrong with it as obviously the wife can provide valuable information but I'm not sure it is worth it from a PR perspective.

ETWolverine rated this answer Excellent or Above Average Answer
itaks2tano rated this answer Excellent or Above Average Answer

Question/Answer
excon asked on 01/29/06 - Post Traumatic Bush Disorder


Hello:

Is there something I can take for that?

excon

tomder55 answered on 01/30/06:

I suggest some' hair of the dog that bit you '. Down load Bush's SOTU speech Tues. night ;don't listen to him deliver it ;just read it before the commentators have a chance to spin it for you .

I think they should frankly do away with all the pomp and rediculous applause interuptions. It's time to join the 21st century . The President would still be satisfying his congressional requirement by making a pod-cast available and perhaps e-mailing the text to all members of Congress as well as the press and post it on the White House web site. There is no need to disrupt the ABC regular scheduled broadcast of 'Commander in Chief' so that the nation can watch THE Commander in Chief .

excon rated this answer Excellent or Above Average Answer

Question/Answer
HANK1 asked on 01/28/06 - JUST WONDERING ...

... if you think it's time to reinstate Selective Service (the draft)?

HANK

tomder55 answered on 01/29/06:

Congress has to authorize a draft .The draft(HR 163) was brought up recently by idiot Rep. Charles Rangal (D NY) .He is not a legislator genuinely seeking to reinstate the draft but a Democrat seeking to make an anti-war statement, and the bill sat in committee for 21 months before Republicans brought it to the floor and overwhelmingly defeated it in October 2004(402 to 2 ).

The military think that bringing back the draft now would reduce the quality of the military, while driving up its cost.Their arguement is sound. The traditional 2 year conscription is not enough time to train the modern military while at the same time retaining them for any useful service. There would be a high cost in training personnel that would take funds the military needs for other purposes. The military has specialized needs (e.g., health care, linguistics, computer technology or engineers)that a draft would not fill. Most probably if there was a draft it would have to be restricted to those needs.

The popular excuse for the talk about the draft is the military not meeting it's recruiting goals.The Army's recruiting goal was significantly higher than in years past, because Congress authorized the Army to increase in size by 30,000 soldiers. The Army tried to accomplish the entire strength increase in one single year, and that's hard to do. The Army didn't make it but the rest of the active military did; it's not that big of a deal.The Army isn't even using all the recruiting incentives that Congress authorized.AND they are turning away thousands of volunteers each month because those recruits do not meet enlistment standards. Rangal belly aches about the poor and disadvantaged being disproportionatly represented in the military .Nothing could be further from the truth. Joining with a GED used to be the norm .Now there is a cap of 10% GEDs .All others must have a high school diploma.If you have a criminal record ;forget about it ;You are not considered .

The Army is currently undergoing a major reorganization the ultimate result of which will transfer many active duty, non-deployable "support" jobs to civilian contractors. That will allow the Army to use the active duty slots to create more combat and support units. That, in turn will allow them to decrease the length of combat deployments.

Re-enlistments rates in the Army continue to be the highest ever. While the Army isn't attracting as many new recruits as they want, they don't appear to be having any problems keeping the soldiers they've got.This with the soldiers knowing the conditions and risks they currently serve under .

The Air Force and Navy have too many . Both services are required to cut thousands of troops from their active duty roles by September . The Air Force must cut about 20,000 and the Navy about 14,000. That "reduction in force" for the Air Force and Navy has resulted in long waits (up to one year) for those wishing to join up, and the services are encouraging people to leave early, or transfer to the Army.

We have a million and a half highly trained, professional Soldiers, Sailors, Airmen, and Marines currently on active duty. Additionally, we have 1.3 million in the Active Guard and Reserves. Together, these 2.8 million all volunteer professionals can handle any foreseeable military conflict. Also, let's not forget the millions in the inactive reserves, or the millions of military retirees who, by law, can be recalled to active duty at any time.

Hank ;I see you made a comment about the possible need for a draft due to possible military conflicts with Iran. I agree that it is probable that diplomatic initiatives will not be sufficent to eliminate the threat of a Iranian nuke. A huge American military effort, involving hundreds of thousands of troops, would be needed to get boots on the ground. But I consider that to be the least likely scenario of how our military option would play out .We could do it of course ;but it will probably not be necessary.I think covert action ;a naval blockade;and surgical air strikes would be the most we would need to achieve a limitted objective (destroying their nuclear capablity ). I do not think that regime change will come unless there is a popular revolution (which could be sparked by our military action) .If that were the case we should not make the mistake we made in 1991 in Iraq. We should provide material support and air cover if needed.We should definitely target the lunatic Mahmoud Ahmadinejad as well as the key members of the mullocracy .It is the opinion of retired Army Major Gen. Paul Vallely that we could take Iran down with just our air assets. I agree if he is including air craft carrier group assets .



HANK1 rated this answer Excellent or Above Average Answer
ETWolverine rated this answer Excellent or Above Average Answer

Question/Answer
Itsdb asked on 01/27/06 - Al Gore brings down the Sundance house

PARK CITY, Utah, Jan. 26 (UPI) -- One of the biggest stars and hottest tickets at the Sundance Film Festival this week was former Vice President Al Gore and "An Inconvenient Truth."

The bigger surprise at the annual Park City, Utah, celebrity gathering is "Truth" isn't a comedy, drama or even a dramedy -- it's a documentary about global warming, the Washington Post reported Thursday.

"Al is a funny guy," Larry David of HBO's "Curb Your Enthusiasm," told the Post as the buddies attended celebrity parties and mingled with fans.

But Gore's film carries a very unfunny message -- humans may have only 10 years left to save the planet from the mass destruction of global warming.

David's wife, Laurie, convinced Gore to make the film after watching one of his global warming presentations in 2004.

The slide show "was his baby, and he felt proprietary about it and it was hard for him to let go," she said. Now they are hopeful a distributor will pick up

David says the filmmakers are in discussion with three or four distributors, hoping for a sale.

"This isn't about box office," David said. "None of us are going to make a dime." What is at stake, she told the Post, "is, you know, the planet."

~~~~~~~~~~~~~~~~~~~~~~~~~~~~~~~~~~~~~~~~~~~~~~~~~~~~~~~

Isn't great, you know, that libs and dems of all walks can come together to save the planet? Nobody's going to make a dime on Al's "documentary" - now if Laurie would just give up her private jet she could contribute to saving the planet, too.

Is Al insane or is he a first rate scientist? Has he been hanging out with Ted Danson?

tomder55 answered on 01/28/06:

Didn't Gore have a poluting zinc mine on his property that netted him $ 500,000 a year ? more on Gore later .

must be a cunundrum for the lefties this week .They can't decide if they want to hang with the Hollywood types at Sundance or jet over to Davos to hang out with the billionaires at the World Economic Forum . Gore chose Sundance and Kerry was at Davos until he was summoned back to Washington by Michael Moore;the NY Slimes and the Moveon.org crowd to muster up a belated fillibuster attempt.When the slimes calls for some guts and a spine then Kerry be the man! 'God bless John Kerry,' said Don Stewart, a spokesman for Senator John Cornyn, a Texas Republican on the Judiciary Committee. 'He just cinched this whole nomination. With Senator Kerry, it is Christmas every day.'

they couldn't even keep white sheets Byrd in the fold . Did you hear his floor speech about the Alito hearings ?

West Virginians who wrote to criticize the way in which the hearings were conducted used the same two words; they called them an outrage and a disgrace.

and this was my favorite qoute :

Mr. President ,was it really necessary to subject Mrs. Alito to the harsh glare of the television klieg lights as she fled the hearing room in tears, fighting to maintain her dignity in response to others with precious little of their own?

I can't decide if he was ripping Ted Kennedy ;Joe Biden ;or Chuck Shumer but he nailed it !

Al Gore ,hmmm ;I guess he brought the house down with lines like :

"I used to be the next president of the United States," ...... "I'm a recovering politician."

LOLOLOL what a side splitter !! Would that it was true but I suspect that Gore's recent re-emergance coincides with another attempted run at the White House . He sees himself as another Nixon ;who lost the White House as VP in 1960 ;sat out the 64 election and re-invented himself in time for the 68 run .In Al Gore's case I guess the re-invention means that he has become king of comedy central . He thinks he has found his issue ;forget terrorism ;why worry about dirty bombs when the ice caps are melting ???

If Al Gore truely wanted to cleanse his soul then he should deliver this speech.

This lenghty speech does not even deal with Gore's national security breaches when he accepted large sums of political money from China through Buddhist monks who had taken vows of poverty ? He later claimed that his April 29, 1996 visit to the Hsi Lai Buddhist Temple was no fund-raiser.

Let's put Abramoff in perspective here . Johnny Chung was a front man for funneling Chinese money to the DNC .He is now serving jail time for his efforts . He admitted to knowingly taking funds from the Chinese government;that the DNC knew the source of this money was communist China. They accepted it anyway. The money is said to have been funneled through Chinese government official Liu Chao-ying then Chung and then to the DNC.Chung visited the White House over 45 times while Clintoon was President .Chung testified under oath to Congress on his admitted relations with the Chinese Commies and the ties to the President. Charlie Trie another link to the Chi-coms entered a guilty plea for his fundraising crimes and agreed to talk with investigators .John Huang , another Clinton cronie, pled guilty to funneling Chinese funds less than a week after Trie.

So he accepted laundered money as a campaign contribution from a foreign source ,that was bad enough ;what was the quid pro-quo ?

Essentially, Clinton and Gore gave military technology to Communist China. This information included satellite guidance technology and enough information for communist China to modernize their nuclear arsenal. Chinese missile weapon technology greatly improved due to Clintons "gift." A guidance chip was also found missing from an American satellite that crashed on take off over China. The crash site was blockaded for several hours following the crash - restricted to Chinese officials. Launching our satellites on Chinese rockets from communist China? Another Clinton policy.

This all came out just as Clinton and the Chinese dismissed investigations into the "China fund-raising Scandals" as groundless. I think Johnny Chung proves otherwise. The Cox report revealed that China was actively spying on the United States and had been stealing top secret nuclear weapons plans . Clinton was informed of the espionage problem years in advance of it breaking to the public yet did absolutely nothing about it and even continued to transfer other technology and pursue close relations with China! China certainly did get something in return for all the campaigning and donations they made to Bill's reelection.

When Bill Clinton and Al Gore came to power the United States was at the apex of its power. China was then a minor player on the worlds stage militarily . The Chinese had shocked the world by ordering the Army to open fire on pro-democracy students in Tiananmen Square in June 1989 and was still under the international isolation that followed. President Bush had issued an executive order cutting off military sales to China .When Clinton and Gore came to power, the Chinese had no modern air-defense systems; it lacked a military communications network; it had no modern command-and-control systems; it had no imaging or electronic eavesdropping satellites; its strategic rockets were unreliable and its missiles were at an infant stage of development.

At the time, no other nation on earth except Japan had computer capabilities even remotely approaching those developed by the U.S. . To protect our technological advantage, and to keep militarily critical supercomputers from falling into the wrong hands, the United States had an agreement with Japan to review each export before it occurred. The Clinton administration ramped up the limits on what could be exported to China and eventually did away with the agreement with Japan. By the end of 1998, the Cox commission found that more than 600 military-grade supercomputers had been shipped to China.

By the time the Clintoons were finished China had become a world class military that is now capable of directly challenging us. Here is some of the improvements they achieved during the Clintoon years :

A new generation of road-mobile, solid-fuel strategic missiles, including the DF-31, capable of reaching the U.S. heartland;


An encrypted military-communications system virtually impervious to electronic eavesdropping, coupled to a national command-and-control system;

A new generation of ballistic-missile submarines;

AWACS command-and-control aircraft;

Sovremenny-class destroyers, equipped with SS-N-22 nuclear-capable antishipping missiles;

SA-10 air-defense missiles and S-300 antitactical ballistic-missile systems, deployed along the coast facing Taiwan;

Several hundred improved M-11 (CSS-7 Mod2) attack missiles facing Taiwan, equipped with U.S.-built GPS guidance systems as well as hundreds of Su-27 and Su-30 strike aircraft and laser-guided missiles from Russia; and

Antisatellite weapons capable of blinding the United States early-warning and intelligence-gathering capabilities.

Clinton and Gore helped create a threat to our nations security which simply did not exist before their time . That to me was the only legit reason for impeaching Clinton ,and that should forever disqualify Al Gore from becoming President .




Itsdb rated this answer Excellent or Above Average Answer

Question/Answer
excon asked on 01/26/06 - Hamas


Hello:

Well, that's a fine kettle of fish you've gotten us into, Ollie.

excon

tomder55 answered on 01/26/06:

nice to know that Iran now has a forward satellite right on Israel's border. Can we now put that road map to rest once and for all ? There was a possibility for a while that Abbas could've been that elusive partner in peace but he lost his chance when he (as he called it )refused to preside over a Palestinian civil war . He let his Altalena moment slip by and instead allowed terrorist organizations remain armed . Perhaps that civil war will now commence .


Meanwhile Hamas has proven much more successful in civil governance through their intimidation and billions of unaccounted EU and US funds are the direct responsibility of Fatah ;their credibility was shot .

I suggest that Israel should redouble the efforts to complete the wall .Like it or not ;Israel will have to deal with Hamas and so will the US . We already have active dialogue with Lebanon even though there is now a larger Hezzbollah presence in their government. Perhaps this will add clarity . No longer will Israel be attacked by a shadowy terrorist organization . They will now be taking attacks from the legitimate Palestinian government and should respond as if any other nation is attacking them .

excon rated this answer Excellent or Above Average Answer
Itsdb rated this answer Excellent or Above Average Answer
kindj rated this answer Excellent or Above Average Answer

Question/Answer
ETWolverine asked on 01/25/06 - An interesting article

Save Haleigh

By Michelle Malkin

http://www.JewishWorldReview.com

I have a question for the hordes of bleeding-heart Hollywood stars who joined the "Save Tookie" brigade, who bowed their heads in prayer with ex-Crip gangster Snoop Dogg and the Rev. Jesse Jackson and pleaded to protect convicted Death Row murderer Stanley "Tookie" Williams, and who lobbied so hard for the government to err on the side of life.

Where are you now?

In Boston, an innocent girl was sentenced to death by the state. Her name is Haleigh Poutre. Last fall, she was hospitalized after her stepfather allegedly burned her and beat her unconscious with a baseball bat. Haleigh was kept alive by a feeding tube and ventilator. Doctors said she was "virtually brain dead." They said she was in a "persistent vegetative state." The medical professionals pronounced her "hopeless."

Less than three weeks after Haleigh's hospitalization, the Massachusetts Department of Social Services was raring to remove Haleigh's feeding and breathing tubes. Even her biological mother (who had been deemed unfit to care for Haleigh and whose former boyfriend was accused of sexually abusing the child) wanted her to be put to death. The only person who wanted Haleigh alive was her stepfather, who will likely be charged with murder if Haleigh dies.

Earlier this month, the Massachusetts' Supreme Court ruled in favor of killing Haleigh, saying it was "unthinkable" to give the power to make a life-and-death decision to the man accused of putting Haleigh in a coma. Instead, the court did something just as unthinkable: It handed over that power over life and death to the same child welfare agency that had failed time and time and time again to protect Haleigh from her abusers in the first place. According to the Boston Herald, a report by her court-appointed guardian showed that the Department of Social Services had received 17 reports of abuse or neglect involving Haleigh in the three years before her adoptive mother and stepfather were charged with pummeling her into a coma.

"State can let beaten girl die," the headlines trumpeted. But there was just one small complication for all of those who, for whatever reason, were in such a rush to "let Haleigh die:"

Haleigh is fighting to live.

As state officials prepared to remove Haleigh's life support, the supposedly impossible happened. She began breathing on her own, responding to stimuli, and showing signs of emerging from what the medical establishment had deemed her hopeless condition. Everyone had given up on Haleigh except Haleigh. ''There has been a change in her condition," announced a DSS spokeswoman, Denise Monteiro. ''The vegetative state may not be a total vegetative state."

Unbelievably, the state had weaned Haleigh off her breathing tube before the state supreme court had made its ruling but the government failed to inform the court of the development. Haleigh's medical records and the social service agency's brief remain sealed.

Politicians in Massachusetts are vowing full-scale investigations of the state's incompetent child welfare bureaucrats. But where's the accountability for the medical experts whose faulty diagnosis led to Haleigh's court-approved death sentence? Will they step forward and reveal themselves? Will they explain how they erred? Will they apologize?

It was The Experts' unequivocal assessments that led the court to declare Haleigh in "an irreversible vegetative state" and to assert that "the child could not see, hear, feel, or respond." Now, they admit they were wrong. And now, Haleigh's life depends on the whims of a hopeless government agency that didn't think the court needed to know that the child was breathing on her own.

Haleigh's story is a wake-up call to "right-to-die" ideologues who recklessly put such unlimited trust in the medical profession and Nanny State. With such uncertainty surrounding persistent vegetative state diagnoses, the presumption must be in favor of life. Yet, the "right to die" lobby's mantra seems to be: When in doubt, pull it out.

While Haleigh clings to life, I've pondered how we might help persuade the plug-pullers to delay the child's state-sanctioned death sentence. I propose nominating her for a Nobel Prize. It bought Tookie Williams five extra years.

Jamie Foxx and Susan Sarandon, will you join me?


--------------

Hmmm. This opens up a whole can of worms, doesn't it?

First of all, a girl who was determined to be in a "permanent vegetative state" turns out to be improving: breathing on her own, responding to stimuli, etc. Even her doctors are admitting they were wrong. What does this case do to the argument that doctors know best when a patient will not improve and should be allowed to die? What does this do to arguments against one more check of the patient's condition? In light of this case, was Congress right to demand one more judicial and medical review of Teri Schiavo's condition before her feeding tube wa removed? Would one more review of her condition hurt anything?

What does this case do to those who argue the case against capital punishment but in favor of abortion and assisted suicide?

What are the legal ramifications of the state making the determination in favor of assisted suicide? What if the state is wrong? (I find it interesting that the same people who argue against capital punishment on the basis that the government might make a mistake and kill the wrong person suddenly argue the infallibility of the government when determining in favor of assisted suicide.)

Seems to me that the case of Haleigh Poutre raises a lot of questions on BOTH sides of the political fence: the liberals who would allow family to determine assisted suicide, and the conservatives who would allow the government to be the final arbiter. And the conservatives who would argue against assisted suicide at all need to contend with the question of what if the patient really is suffering in an intolerable manner and really does want to die.

What are your opinions?

Elliot

tomder55 answered on 01/25/06:

I opined regarding the Oregon decision that it is the right of the individual to request assisted suicide . i go no further than that . The person's intention should be clear and unambiguous or else you are opening it up to all types of potential abuse as was seen in the Schiavo case .

Here was the pertinent part of my reply previously So long as VERY strict criteria like absolute proof of patient's consent is met;that no medical professional is compelled to do so (like some would have pharmacy workers do)and you avoid slippery slope traps like the Dutch have fallen into ;I am in favor of assisted suicide as a matter of LAW . I depart from the Conservative judges on this issue But I am very concerned that loose assisted suicide laws would dangerously impact on the rights of the disabled .

In light of this case, was Congress right to demand one more judicial and medical review of Teri Schiavo's condition before her feeding tube wa removed? Would one more review of her condition hurt anything?

not at all . I thought all questions should have been answered satisfactorily before she was terminated .There was no need for the rush to remove the tube.

What are the legal ramifications of the state making the determination in favor of assisted suicide?

That is the slippery slope I am talking about . Even worse ;in Holland it is now the doctors determination .


And the conservatives who would argue against assisted suicide at all need to contend with the question of what if the patient really is suffering in an intolerable manner and really does want to die.
That is ultimately why I favor assisted suicide laws so long as it is absolutely the patients choice . No exceptions.


ETWolverine rated this answer Excellent or Above Average Answer

Question/Answer
kindj asked on 01/25/06 - Are we free?

In my previous question regarding a couple of authors, the always interesting excon made the following statement:

>>From my simplistic viewpoint, history tells me that tyranny IS and has been the status quo in the world. Although we BELIEVE in the concept of freedom, in the real world, tyranny works better (if you think an organized and controlled society is BETTER). Consequently, ALL societys, no matter what they experiment with, eventually revert to tyranny.

The US of A is no different and we're not immune. Therefore, if we don't guard our freedom, it will disappear.<<

Needless to say, I totally agree one hundred percent with the last statement. However, it was the first part of his statement that stuck in my head, and that I've been mulling over for the last twelve hours or so. And yes, I even had a dream about an "America-turned-Orwellian" last night. Of course, the ladies were hot in their Star Trek type uniforms.....but I digress.

What he said rung a bell. Freedom is HARD. It's damn sure inconvenient for all of us at one time or another. You know, those times when someone else is exercising their freedom in opposition to what we think. It's sure got to be a pain in the butt for the government, having to watch out for all those rights and stuff. Just ask Bush as he goes to the hearings on his domestic surveillance.

As I told ex, I think we're free, but not as free as we were. I further think that even RIGHT NOW our freedoms are being further eroded so slowly that we take no notice. I think that if that doesn't stop, I'll be sitting on the porch with my grandkids while they ask, "How come we can't do this or that like when you were a kid, grandpa?"

What will I say?

"Because they took those freedoms away, son, and we stood by and let them, in the name of convenience."

So what do you think? Will the Great Experiment continue to be a success, or will one day everyone just say, "I give up, this democracy thing just ain't working." Or will it be more insidious than that, with an inexorable erosion of rights and freedoms, until one day America more closely resembles the Soviet Union of the Cold War era than the republic it once was?

DK

tomder55 answered on 01/25/06:

Tocqueville observed in the early 19th century that people began to discuss the problem of poverty during the industrial revolution. At first he thought that this was strange, since the growth of the manufacturing system meant higher wages and cheaper goods. Poverty was decreasing, but at the same time it was seen as a worse problem than before.
His conclusion was that this happened not despite but because poverty was being reduced. In earlier times, poverty was seen as something given. Something that was everywhere, and something that we just had to learn to endure. But in the 18th and 19th centuries, industrialization created unprecedented wealth and millions were lifted out of poverty. The result was that the poverty that remained was perceived as so much worse.

Notwithstanding the Cassandras, the world is wealthier, healthier and happier than it has ever been. We live longer, we live more safely and we live more freely. For every successive generation, we have been able to build on the knowledge, technology and wealth of earlier generations, and add our own. We have reduced poverty, created more wealth and increased life expectancy more in the past 50 years than we did in the past 5000 years. Just 200 years ago, slavery, feudalism and tyranny ruled the world. Where was the freedom when you had to work from sun up to sundown to provide the basic existance for youre family ?Now we talk about people living fulfilling lives;not taking care of their basic sustanance. The average chance of surviving your first year was less than the chance of surviving to retirement today. Today we have more people living longer lives in freer societies than lived in all previous periods combined. Today everyone gets an education . In previous generations it was a rarity that someone made it out of high school.. Biotechnology, nanotechnology and robotics will create vast improvements in the future . We will be richer, we will live longer and we will be healthier. Continents that we thought were doomed to misery will soon have the living standards we have today.

Sure every generation is confronted with it's own unique challenges . There is an oft misquoted Franklin saying that goes : Those who would give up essential Liberty, to purchase a little temporary Safety, deserve neither Liberty nor Safety.
It is my contention that throughout our history we have given up non-essential liberty as needed to secure our greater freedoms. Was a stink made when in WWII rationing was mandatory ? Was there seriously a question about what we were sacrificing and what we were protecting ?The truth is, the government must strike a balance between privacy (not liberty )and security. We are in just as much if not more danger from unchecked terrorism than we are from unchecked intrusion.

I don't know if I answered your question or not . I just re-read it and it sounds a little disjointed . Suffice it to say that sometimes in a journey it is useful to look back and see how far you've come. We are not a perfect society and there will always be obstacles that seem insurmountable. But we have in over 200 years survived worse .Short of finding a deserted island to live your version of rugged individualism . I cannot imagine how a person living in an open society of almost 300 million could be any freer .





excon rated this answer Excellent or Above Average Answer
kindj rated this answer Excellent or Above Average Answer
labman rated this answer Excellent or Above Average Answer

Question/Answer
CeeBee2 asked on 01/22/06 - "War is the story we tell ourselves

in order to make sense out of what is otherwise organized and semiorganized violence and murder. Creating that story involves giving the violence a larger meaning and denying that the enemy has any humanity." (from author Steve Conn's review of Black Hawk: The Battle for the Heart of America by Kerry Trask)

When I read this last night, I thought, How true for this "war" we are currently waging.

tomder55 answered on 01/23/06:

Forensic psychologist ususally find the practice of empathizing trying to build up self-esteem is not the best approach in dealing with patients exhibiting serious violent tendencies.Setting clear boundaries and making judgments about their behavior works much better.The slightest hint of fear ,intimidation or sympathy is met with increased threats.Without clear boundaries, and a sense of consequences, their behavior will spirals out of control until they injure themselves and others.To humanize and to empathize is the worse possible answer to terrorism.

We make a big mistake in not naming our enemy .It is not terrorism ;but Militant Islam.Like the totalitarian ideologies of the 20th century that strove for hegemony, militant Islam must be identified, acknowledged, and defeated .

CeeBee2 rated this answer Excellent or Above Average Answer

Question/Answer
Itsdb asked on 01/20/06 - Chirac gets tough?

Chirac Nuclear Comments Draw Ire in Europe

By JAMEY KEATEN
Associated Press Writer

PARIS (AP) -- President Jacques Chirac drew scorching criticism in Europe on Friday for suggesting France would consider a nuclear response to state-sponsored terrorism.

Chirac's headline-grabbing comments in a speech Thursday sent a warning to countries like Iran and sought to nip in the bud domestic debate about whether deeply indebted France still needs its expensive nuclear deterrent in the post-Cold War world.

The French leader, with his second and probably final term nearing its end, laid out an updated doctrine for France's military might for the 21st century amid the threat of terrorism.

In the broad address, Chirac warned unspecified "leaders of states that would use terrorist means against us" that they could face "a firm and fitting response." Analysts and presidential aides said he had no specific country in mind, but newspaper editorialists widely read them to be directed at Iran, and possibly North Korea.

Chirac seemed to draw little initial support abroad for his call for the European Union to pool its deterrent forces "in the perspective of a strong Europe."

The volume of criticism could even set back that hope.

"Jacques Chirac is an idiot," chided Belgian daily De Morgen in an editorial. "He lives in a time where France is no longer a world power, but he's still acting as if prolonging a Napoleonic dynasty."

Spain's El Pais called the speech "radical and dangerous."

Many faulted the timing. France, Britain and Germany have been seeking guarantees that Iran will not develop nukes, and have taken key steps toward possible U.N. sanctions against Tehran.

"Such saber rattling in the face of the current crisis over Iran's atomic weapons program is basically a false signal," said Xanthe Hall of the International Physicians for the Prevention of Nuclear War in Berlin.

If Chirac ends his tenure as president next year, he will have left an indelible mark on France's nuclear deterrent. Shortly after winning the presidency in 1995, he drew international fury by ordering France's final nuclear tests in the South Pacific.

Conservative Milan daily Il Giornale suggested the "pacifist sympathies" for Chirac over his opposition to the U.S.-led Iraq war had worn off.

France's nuclear arsenal, which analysts estimate at some 300 warheads mostly deployed on submarines, is viewed as a deterrent tool and is not intended for a battle situation.

Observers saw a political pitch at home by Chirac: activist groups and even military circles have questioned the euro3 billion-plus ($3.62 billion) annual cost to keep up France's nuclear arsenal.

But at home, too, the speech garnered criticism.

"Another pearl in the words of Chirac - but this one is a bit dramatic and provocative," said Revolutionary Communist League party leader Alain Krivine, a longtime critic of the conservative Chirac, on French TV. "It's a completely irresponsible declaration."

~~~~~~~~~~~~~~~~~~~~~~~~~~~~~~~~~~~~~~~~~~~~~~~~~~~~~~

Was Chirac warning Iran & Syria, warning the US, just being an idiot - or all three?

tomder55 answered on 01/20/06:

getting kinda late in his life to be developing a spine . It'll be interesting to watch the MSM response to this. They'll probably say its a, "thoughtful and measured response to Iran's nuclear threat", but if Bush said those exact words, well, "irresponsible saber rattling" when a thoughtful and measured response is what's needed. I believe him .The French blew up a Green Peace Ship that got in their way .

I think this summer was a real wake up call in France. It is clearly a warning to Iran who has now developed missle technolgy sufficient to threaten parts of Europe . But if he felt this way then why did he not commit the weight and diplomatic effort to the EU initiative ?

ETWolverine rated this answer Excellent or Above Average Answer
Itsdb rated this answer Excellent or Above Average Answer

Question/Answer
purplewings asked on 01/20/06 - Bin Laden and a possible truce

WASHINGTON - Rejecting a suggestion by Osama bin Laden
of a negotiated truce in the war on terror, Vice President Dick Cheney said there was only one way to deal with terrorists. "I think you have to destroy them," Cheney said.

The vague offer of a truce coupled with a threat of another attack on the U.S. was made in an audiotape released by the Arab television network Al-Jazeera. It brought new attention to the al-Qaida leader after a yearlong lull in his public statements.

U.S. security officials said Thursday there were currently no plans to raise the nation's security threat level because of the new tape.

Counterterror officials said they have seen no specific
or credible intelligence to indicate an upcoming al-Qaida attack. Nor have they noticed an uptick in terrorist communications "chatter" although that can dramatically increase or decrease immediately before an attack.

On the tape, bin Laden warned that his fighters are preparing new attacks in the United States but offered the American people a "long-term truce" without specifying the conditions.

But Cheney, in a television interview, rejected that suggestion, saying"We don't negotiate with terrorists."

"I think you have to destroy them," he told Fox News Channel. "It's the only way to deal with them."

The tape prompted increased security at Los Angeles International Airport and other precautions at the city's port and water and power facilities.

http://news.yahoo.com/s/ap/20060120/ap_on_go_ot/us_al_qaida_tape

Does anyone think we should try talking to Osama? Would it help settle the terrorists down? Could any good come from a discussion with this man?

tomder55 answered on 01/20/06:

In the immortal words of US Grant : "No terms except an unconditional and immediate surrender can be accepted. "

He said he wanted to use the time to rebuild Iraq and Afghanistan . How magnanamous ! I can see the Bin Laden Memorial Hospital in down town Baghdad already . What he means by rebuild is to rebuild his shattered organization .

There are two reasons for this . First is that we are getting close . al-Qaeda leadership is trapped in Hindu-Kush and events from last week prove that the sanctuary is breaking down .He needs time to set up shop somewhere else .The second is that public opinion is turning against al -Qaeda in Iraq .There are more and more reports of locals turning them in or even direct battle between the Sunni insurgents and al-Qaeda invaders . But he can't walk away . His boast that he could penetrate US defenses if al Qaeda wasn't engaged fighting in Iraq is probably correct .Bush has said the same thing . ("fighting them there so we don't have to here ")

What he is proposing is what the Koran calls 'Hudna'.When you are weakend it is permitted to have a truce with the enemy for up to 10 years ."if Muslims are weak, a truce may be made for ten years if necessary, for the Prophet (Allah bless him and give him peace) made a truce with the Quraysh for that long, as is related by Abu Dawud" Muhammad in A.D. 628 made a hudna with the Arabian tribe of Quraysh.The Islamic forces used the peace to become stronger over a couple of years, then defeated the Quraysh tribe. This became known as the "Quraysh Model," which defines the meaning of a hudna. When you are militarily at a disadvantage, declare a "hudna" until you are militarily strong enough to discard it and win the war. This is a Muslim tactic that has been used over and over again throughout history with devastating success. When the times are rough you have a hudna .Then when it is convienient to you ;violate it and resume hostilities . That is what Arafat did at Oslo to get his foot back into the West Bank from his exile .And that is what OBL would do.

Itsdb rated this answer Excellent or Above Average Answer
purplewings rated this answer Excellent or Above Average Answer

Question/Answer
Choux asked on 01/19/06 - Bush's Rx Drug Plan a Fiasco

Article from Slate dot com.
Why Bush's prescription plan is such a fiasco.
By Jacob Weisberg
Posted Wednesday, Jan. 18, 2006, at 3:33 PM ET

President Bush thought that millions would welcome his intervention. But the effort has not gone as planned. Costs are spiraling out of control, and many of the people we wanted to help are protesting that the situation is worse than ever. Three years later, the entire poorly conceived enterprise is in jeopardy.

I refer, of course, to the administration's program to subsidize the cost of prescription drugs for the elderly. This plan, which went into effect on Jan. 1, offers so many baffling options that only 1 million of 21 million eligible Medicare beneficiaries have signed up for it on their own. Many of these early adopters, along with millions of impoverished Medicaid recipients transferred into the new system automatically, have been unable to obtain their prescriptions at the promised discounted price. The specter of citizens going without needed medications has provoked action by several governors, some of whom have invoked emergency powers to pay for drugs. Meanwhile, the estimated cost of this plan that no one likes has already more than doubled and is now projected at more than ***$1 trillion over the next decade***.

Any chance to get rid of this fiasco??

tomder55 answered on 01/20/06:

no ;once an entitlement is promised and woven in then it only expands .What we will have until the whole entitlment system collapses will be a 'Kick the can 'program of meaningless reform proposals . Meaningful reform will be watered down to the point of ineffective or will be mocked in Congress as doa . Don't believe it ? Just look at what they do with SS. If you want to blame Bush and the Republicans on this go right ahead .They may have well worn Donkey lapel pins on this issue . AARP sqeezed the nuts and pols in both parties coughed . No one was dutifully served except perhaps the drug industry .

Choux rated this answer Excellent or Above Average Answer
excon rated this answer Excellent or Above Average Answer

Question/Answer
ETWolverine asked on 01/19/06 - Text of excerpts from Osama bin Laden tape

Al-Qaida leader appears to be addressing the American people

The Associated Press
Updated: 2:58 p.m. ET Jan. 19, 2006


The following is the text of the excerpts aired by Al-Jazeera television from a new audiotape from al-Qaida leader Osama bin Laden. The excerpts were translated from the Arabic by The Associated Press.

Bin Laden appears to be addressing the American people:

My message to you is about the wars in Iraq and Afghanistan and how to end them. I did not intend to speak to you about this because this issue has already been decided. Only metal breaks metal, and our situation, thank God, is only getting better and better, while your situation is the opposite of that.

But I plan to speak about the repeated errors your President Bush has committed in comments on the results of your polls that show an overwhelming majority of you want the withdrawal of American troops from Iraq. But he (Bush) has opposed this wish and said that withdrawing troops sends the wrong message to opponents, that it is better to fight them (bin Ladens followers) on their land than their fighting us (Americans) on our land.

I can reply to these errors by saying that war in Iraq is raging with no let-up, and operations in Afghanistan are escalating in our favor, thank God, and Pentagon figures show the number of your dead and wounded is increasing not to mention the massive material losses.

....

And so to return to the issue, I say that results of polls please those who are sensible, and Bushs opposition to them is a mistake. The reality shows that the war against America and its allies has not been limited to Iraq as he (Bush) claims. Iraq has become a point of attraction and restorer of (our) energies. At the same time, the mujahideen (holy warriors), with Gods grace, have managed repeatedly to penetrate all security measures adopted by the unjust allied countries. The proof of that is the explosions you have seen in the capitals of the European nations who are in this aggressive coalition. The delay in similar operations happening in America has not been because of failure to break through your security measures. The operations are under preparation and you will see them in your homes the minute they are through (with preparations), with Gods permission.

Based on what has been said, this shows the errors of Bushs statement the one that slipped from him which is at the heart of polls calling for withdrawing the troops. It is better that we (Americans) dont fight Muslims on their lands and that they dont fight us on ours.

We dont mind offering you a long-term truce on fair conditions that we adhere to. We are a nation that God has forbidden to lie and cheat. So both sides can enjoy security and stability under this truce so we can build Iraq and Afghanistan, which have been destroyed in this war. There is no shame in this solution, which prevents the wasting of billions of dollars that have gone to those with influence and merchants of war in America who have supported Bushs election campaign with billions of dollars.

URL: http://www.msnbc.msn.com/id/10927325/
---------


So, what do we have here? We have Bin Laden releasing a tape just days after the Pakistan attack in which a number of his highest-level supporters were killed. In the tape he makes two interesting comments. 1) That al Qaeda is planning a new attack on the USA, and 2) that he is willing to offer a truce to the USA under his conditions. And he tries to explain that the only reason that he hasn't attacked the USA yet is because his plans weren't yet complete, but as soon as they are, he will launch an attack against us.

What does this tell you?

What it tells me is that OBL needs to project an image of strength in order to bolster the flagging morale of his followers. That he is offering a truce under any conditions to the USA--- which he has lambasted in the past as "The Great Satan" and saying that we must be destroyed at any cost--- leads me to believe that HE is the one who needs a truce, and that HE is in trouble. Which doesn't mean that he won't strike at the USA if he can. He will certainly try, and he might succeed. Which would not be a sign that Bush is ineffective, just that no security measures can possibly be 100% perfect 100% of the time.

But what is to me the most interesting of all, is the fact that we now have a CREDIBLE THREAT FROM A CREDIBLE SOURCE of an impending attack against the USA. Should Bush bother getting wire-tapping warrants which will waste time and manpower, or should he just do what he has to do to protect the nation from a national security threat as authorized by the Constitution?

Comments please.

tomder55 answered on 01/20/06:

Michelle Malkin recently had a post describing how key clues used to solve the London subway bombings came from Guantanamo Bay interrogations.

You have a simular take as mine . My first impression was of Baghdad Bob telling the world that there were no US soldiers in Baghdad. al Qaeda just watched one of their safe houses get hit ;evidently missing their big goat cheese ;Ayman al-Zawahiri.They do not know who to trust .Someone told America that there would be a meeting at that location. Enough al-Qaeda big shots were there to indicate that a major pow-wow was underway .Zawahiri can't go to far from that area either. It's not like they are riding Hummers in the mountains of Hindu Kush.

Anyway the al Qaeda bloggs are probably in a panic . Who are the leaders ? John Loftus reported on Batchelor's show that his sources are telling him the OBL died in Iran in December from his kidney illness. He said OBL most likely recorded the message around Oct. I'm not sure I buy into it . If he recorded it in Iran most likely the sound quality would be better .I think he is also somewhere in the Hindu-Kush region and pinned down .

He has used tapes in the past to send messages to cells . This may have been the one to start a low tech campaign (I do not think they are capable of the type of operation they pulled off on 9-11) against the US . He is guessing the US is about ripe for a 4-11 moment. This false impression is delivered courtesy of the MSM and the Democrat Party .

ETWolverine rated this answer Excellent or Above Average Answer

Question/Answer
Choux asked on 01/19/06 - California Spying

AP:::""An alumni group is offering students up to $100 per class to supply tapes and notes exposing University of California, Los Angeles professors who allegedly express extreme left-wing political views.

The year-old Bruin Alumni Association on its Web site says it is concerned about professors who use lecture time to press positions against President Bush, the military and multinational corporations, among other things.

The site includes a list of what the group calls the college's 30 most radical professors.

"We're just trying to get people back on a professional level of things," said the group's president and founder, Andrew Jones, a 2003 UCLA graduate and former chairman of the student Bruin Republicans.

"Having been a student myself up until 2003, and then watching what other students like myself have gone through, I'm very concerned about the level of professional teaching at UCLA." ""



Comments?

tomder55 answered on 01/19/06:

and concerned you should be but we survived the indoctrination . I consider the practice above the equivalent to 'truth in advertising 'rights and it clearly is a consumer protection issue .

My own view is that the university is a very expensive anachronism .There are alternatives like 'Univerity of Phoenix 'beginning to emerge who provide the same quality education in a different format .Once the business world begins to accept their credentials then they will attract more and more students who will not want to start out in life $100,000 in debt .

Choux rated this answer Excellent or Above Average Answer

Question/Answer
excon asked on 01/19/06 - Presidential Power


Hello rightys:

Hep me out here. The president said he has authority to stop Oregon from doing what it wants, but the Supreme Court said no. Does that mean that the president is sometimes wrong in what he can and can not do?

In relationship to the NSA spying, is it possible that the president is wrong? If the Supreme Court told him he was and ordered him to stop, would he? Should he? Is the Supreme Court the final arbiter of what he can do, or is HE? I really want to know.

I thought I learned all this in high school civics, but things aren't working out like they're supposed to. Did I miss some classes or something?

excon

tomder55 answered on 01/19/06:

Like I tried to explain to you .Equal branches means there is no final arbiter amongst the branches .It is the people who are the final arbiters.

This goes back a long time . In 2 cases in 1831 and 1832 ;the case of the Cherokee
Indians vs. The State of Georgia ;the John Marshall Court ruled in favor of the Cherokee. Andrew Jackson did not agree with the ruling .He said ;"John Marshall has made his decision, now let him enforce it."

What is happening in the NSA issue is a legitimate disagreement on Presidential authorities . In the Oregon case there is no such disagreement.

excon rated this answer Excellent or Above Average Answer

Question/Answer
sapphire630 asked on 01/18/06 - By golly Itsdb got it

White milk and chocolate...just as long as it isn't 'leche chocolata.' In October Nagin asked, "How do I ensure that New Orleans is not overrun by Mexican workers?" What a boob.

Steve

Yep, New Orleans got the whole enchilada!

tomder55 answered on 01/19/06:

ok ;we'll let in the chocolate and the enciladas ........but keep out the crackers !

ETWolverine rated this answer Excellent or Above Average Answer
Itsdb rated this answer Excellent or Above Average Answer
sapphire630 rated this answer Excellent or Above Average Answer

Question/Answer
sapphire630 asked on 01/18/06 - Lame-est loony outta toonie

cover their butt award goes to New Orleans mayor Nagin

On Martin Luther King day he said that New Orleans needs to get back to being chocolate. He doesn't care what anybody has to say New Orleans should be chocolate again.
What people had to say was that if a white person said that a devesated community has to get back to being white they would never hear the end of it.
So what does the good old mayor do but tries to cover it over by saying he meant chocolate milk
you need white milk in order to make chocolate.


tomder55 answered on 01/18/06:

mmmmmmm chocolate milk . . great for dipping oreo cookies don't ya think ?

Itsdb rated this answer Excellent or Above Average Answer
sapphire630 rated this answer Excellent or Above Average Answer

Question/Answer
Choux asked on 01/17/06 - Supreme Court Upholds Assisted Suicide Law

AP::The Supreme Court upheld Oregon's one-of-a-kind physician-assisted suicide law Tuesday, rejecting a Bush administration attempt to punish doctors who help terminally ill patients die.

Justices, on a 6-3 vote, said that a federal drug law does not override the 1997 Oregon law used to end the lives of more than 200 seriously ill people. New Chief Justice John Roberts backed the Bush administration, dissenting for the first time.

The administration improperly tried to use a drug law to punish Oregon doctors who prescribe lethal doses of prescription medicines, the court majority said.

``Congress did not have this far-reaching intent to alter the federal-state balance,'' Justice Anthony M. Kennedy wrote for himself, retiring Justice Sandra Day O'Connor and Justices John Paul Stevens, David Souter, Ruth Bader Ginsburg, and Stephen Breyer.

tomder55 answered on 01/18/06:

The administration improperly tried to use a federal drug law to prosecute Oregon doctors who prescribe overdoses.This is true and the court correctly ruled on it's narrow merit . The case was not about assisted suicide but about issues of federalism .

So let's get it straight . They over-rule California's medical marijuana law only seven months ago using the same Federal Statute ;and if the principle of 'stare decisis' is as reverent as the Democrats on the Judiciary Committee would have us believe then by all rights the Oregon law should also have been over-turned .hmmmm....

So a doctor can help a patient die with lethal drugs, but can't help a patient live with medical marijuana. Go figure.



Choux rated this answer Excellent or Above Average Answer

Question/Answer
sapphire630 asked on 01/17/06 - 15 minutes of fame


I DO NOT care what Snarlin' Arlen does in life. All his great accomplishments. All his political psycho babble...don't care 'bout none of what he has done or what he MIGHT do!
In my book his 15 minutes claim to fame was snarlin' at Kennedy @ Alito's "inquistion"

tomder55 answered on 01/18/06:

He is chair of the Judiciary Committee and has signaled that he wants to hold hearings on the NSA searches . Fine ;bring em on ...but he has to hold a tight leash on Ted ,Shumer,Biden,Durbin,Feinstein;Lehey;and Feingold ...............geeezzzzzzeeeee ....
could they pack anymore misfits on one commitee ?????

sapphire630 rated this answer Excellent or Above Average Answer

Question/Answer
Choux asked on 01/17/06 - Nine Hundred Foor Blimp

""It's the blimp industry's version of David and Goliath.

An obscure Tarzana firm run by Russian emigres is locked in competition with Lockheed Martin Corp., the world's largest defense contractor, to win a Pentagon contract to build 900-foot- long, blimp-like aircraft to move cargo and troops into combat zones.""

The following are a couple of comments about the 900 foot blimp idea fro the site it was posted on.





War blimps? Are we getting ready for the Crimean War?

BTW, what's the difference between the Hindenburg and Rush Limbaugh? Well, one is a flaming Nazi gasbag and the other is a dirigible.

Posted by: californiamike on January 17, 2006 at 03:00pm




Moving blimps into and out of combat zones?

There is a good reason why such things have not been done since World War I.

Are these guys nuts?


What do you think of the Blimp Idea??

tomder55 answered on 01/18/06:

I knew that the technlogy behind the concept would be beyond most Huff post readers.

Lockheed Martin also builds many of the nation's most advanced aircraft, including the SR-71 and U-2 spy planes.Think they would be venturing into Crimean war technolgy ? So if the Huff post crowd thinks we are talking the Good Year blimp let them .

The airships the Penatgon envisions would travel up to 138 mph, with a range of more than 10,000 miles and would be able to transport large units of personel and their equipment to the operations area .Currently, personnel and equipment travel separately; heavy weapons, such as tanks, are transported by ship, which can take more than a month. It would rapidly reduce the time needed to mobilze to a regional hot spot.


The vehicle would rise into the air thanks to nonflammable helium, much as a blimp does, but the bottom of its hull would act like a wing to give it additional lift and control. Sorta a blimp-plane hybrid .


here is one proto-type from a company called Dynalifter .

As to potential for them being hit in combat yes it's real . During WWII thousands of troops and tons of equipment were transported by sea .There was always ;still is ;a threat by submarine and yet the transports continued . They never go unescorted . I envision that something carrying such important cargo would get hi and low air cover the whole trip by helicoper ;fighter jet ;AWAC ;and drone .

Now ;let's say that you have legit reservations because of the potential of something so large and carrying so much gets shot out of the sky .Imagine the commercial potential for freight hauling or for making a simular prototype smaller eventually becoming an alternative to regional road travel .I certainly think that the concept should be explored . Geeze . The Pentagon did not award any contract on it yet ;they are only taking concept bids .

Choux rated this answer Excellent or Above Average Answer
ETWolverine rated this answer Excellent or Above Average Answer

Question/Answer
labman asked on 01/16/06 - Are these dangerous times?

Happened to notice these 2 headlines from Reuters that I keep on my MY Yahoo page. I like a varity of news sources.


Anti-terrorism official says WMD attack inevitable


Gore calls for special counsel on eavesdropping

Comments on how these 2 relate?

tomder55 answered on 01/17/06:

I wonder what Gore thought about his boss waging a war in the Balkins without Congressional resolutions authorizing the use of such force .Back then of course Congress had abdicated it's Congressional responsibility ;by "sticking it " to Nixon ;they had passed the ' War Power's Act' giving the President sixty to ninety days to wage war and then get out if the President did not report to Congress . Well the time limit did expire and no one called Clinton to the carpet for "breaking the law " . At that point it became a war undeclared by Congress. (part of the reason of course was that the majority in the House was doing their own political posturing ).For the record ; I was in favor of stopping the ethnic cleansing of Slobodan Milosevic.

I am so thankful that Bush won in 2000. What would've happened if Gore and the Clintonistas had been in the White House ? A couple of strategic tomahawk strikes ; a grand jury convened and indictments against bin Laden in abstentia . Meanwhile they would've taken advantage of the situation to do true illegal taps against political enemies. How do I know ? Well that was their track record in the past .

labman rated this answer Excellent or Above Average Answer

Question/Answer
Itsdb asked on 01/17/06 - Quotes of the day

Some of the more fascinating and revealing quotes during yesterday's observance of MLK day...

"Surely God is mad at America. He sent us hurricane after hurricane after hurricane, and it's destroyed and put stress on this country...Surely he doesn't approve of us being in Iraq under false pretenses. But surely he is upset at black America also. We're not taking care of ourselves." - New Orleans Mayor Ray Nagin

"It's time for us to come together. It's time for us to rebuild New Orleans -- the one that should be a chocolate New Orleans," the mayor said. "This city will be a majority-African American city. It's the way God wants it to be. You can't have New Orleans no other way. It wouldn't be New Orleans." - New Orleans Mayor Ray Nagin

As one guy said, "America's flavorite racist?"

"When you look at the way the House of Representatives has been run, it has been run like a plantation, and you know what I'm talking about...We have a culture of corruption, we have cronyism, we have incompetence. I predict to you that this administration will go down in history as one of the worst that has ever governed our country." - Hillary Clinton

"At present, we still have much to learn about the NSA's domestic surveillance. What we do know about this pervasive wiretapping virtually compels the conclusion that the President of the United States has been breaking the law repeatedly and persistently.

A president who breaks the law is a threat to the very structure of our government." - Al Gore

"Once violated, the rule of law is in danger. Unless stopped, lawlessness grows." - Al Gore

"That's a no-brainer. Of course it's a violation of international law, that's why it's a covert action. The guy is a terrorist. Go grab his ass." -Al Gore (sorry, that wasn't from yesterday's speech)

"If the President has commander-in-chief power to commit torture, he has the power to commit genocide, to sanction slavery, to promote apartheid, to license summary execution." - Al Gore

"The President's judicial appointments are clearly designed to ensure that the courts will not serve as an effective check on executive power...." - Al Gore

And they say Murtha was smeared...

tomder55 answered on 01/17/06:

I guess Nagan gets a pass for calling blacks 'chocolate'. I gess that is why Time saw fit to honor him in one of their year end issues.

Drudge Sunday night was talking about Al Gore and that he has been riding the global warming issue for years ,and that when news that apparently supports his position is reported Al Gore has gone off tilting at other windmills.

I will paraphrase Mark Levin : So what was Gore doing when his boss BJ Clinton was doing domestic survaillance ? He was off getting illegal campaign contributions from Buddhists monks .


I wonder how he feels about Evita having access to FBI files stored in her office.

Itsdb rated this answer Excellent or Above Average Answer

Question/Answer
excon asked on 01/15/06 - Constituency available to the highest bidder.


Hello:

Theres a constituency out there waiting to be gobbled up by some party which, in my view, could take the party over the top. This constituency is not being represented by anybody. As a matter of fact, we hate this constituency. I suggest, however, that we only hate them because weve labeled them. The label weve chosen? Illegal alien.

They are that, of course. I suggest, however, that because weve criminalized them, we allow all sorts of heinous things to rain down upon them. The gauntlet in the desert, is only the first. We let taxes be deducted from them, and are happy to keep it all knowing that they cant file for a refund. We let them pay unemployment insurance, FICA, knowing that they cant file for unemployment benefits. Plus, we keep trying to find and arrest them.

Being illegal I understand why the law and order folks among you want to crack down on them. That seems to be the knee jerk reaction on your side when faced with lawbreakers.

But, in addition to being classified as illegal aliens, they also could correctly be classified as American workers (if American means they work here as opposed to China). In any case, they do work for American companies. Of that, theres no question. And, theres between 11 million and 20 million depending on whos doing the counting.

Now, I suggest further, that if they were to rounded up, the rights main constituency, the business community, would be up in arms. As, a matter of fact, I suggest that if we were to lose, say 10 million low wage workers, the country would stop. Im not talking about your local diner either. Given those assumptions, nobody is going to let that happen. Even Bush, is supporting a guest worker program. Hes not as suicidal as some of you are, in the name of law and order.

Therefore, in the real world, they are NOT going to be rounded up and sent back. Beyond being able to stay, what WILL happen to them is unknown. If the Republicans embrace them first, I think they can keep their majority. But, if the Democrats embrace them first, they could regain power.

I could be all wet, as Im sure Elliot will tell me. He wont be able to get over the word illegal. But some of you might be able to see the reasonableness of my conclusion. No?

excon

tomder55 answered on 01/16/06:

if an employer is hiring them without legal documentation then they are more guilty then the illegals . I do not advocate a round up . I figure once they are in the country they should be legally meaningfully employed . I just want to tighten the borders to reduce the flow of unaccounted illegals in the country ;and don't tell me that increased border security would not significantly reduce the number of border crossers . I also am in favor of a guest worker program or increasing the number of legals that enter the country .

excon rated this answer Excellent or Above Average Answer

Question/Answer
excon asked on 01/15/06 - Bush's war


Hello rightys:

Let me see if I understand this correctly. The insurgency in Iraq is mainly disaffected Sunni Muslims. Plus, there are some foreign Al Quaida fighters who are fighting anybody.

So, the people who we're supporting are fighting on our side are the Shiite Muslims, who are supported by Iran, and who will build an Iranian style Theocracy once we leave. Or, if that's not who we're fighting for, who is it? I mean, I really don't know.

Or, are the Iranians really nice guys? How did George Bush get into a war supporting Iranians? Please, splain it to me.

excon

tomder55 answered on 01/16/06:

You are making a straw man argument . Those are not the only options. You assume that all Shia is aligned with Iran ,that is not the case .Only Sadr's Mahdi army is supported by Iran . The fact that most Iraqi Shia are Arabs and not Persian you disregard .The Shia of Iraq are connected by family or other ties to many surrounding countries, including Syria, Lebanon, Saudi Arabia, Bahrain In the Iran UIraq war the Iraqi troops were predominantly Shia .They fought hard against Iran.They understand that the Islamic revolution in Iran has not brought prosperity or freedom for the Iranians . They also understand that many Iranians are also unhappy with what has been happening in Iran over the last 24 years. Iran may want to export the revolution and the Iranian form of government. But Iraq is a very different place.


Historically,until the Iranian revolution the Shia have often taken an anti-governmental tone. It was unheard of that the Ayatollahs would take over a government. Some of the Ayatollahs in Iraq believe that the idea of a cleric ruling is not proper. The grand Ayatollah Sistani is one of these.That does not mean thatthe laws of the land will not have a Muslim element in them . But theocracy ;I doubt it .

While the Iraqi insurgency has shown an aptitude for dispensing violence, the question remains as to what exactly the insurgents and al Qaeda have to offer as an alternative political plan ?Terrorist violence is unifying the Iraqi people with the new, sovereign government. Iraqis are growing weary of the insurgency and taking risky action to turn in insurgents or even fight them. The Iraqis clearly do not long for a return to Baathism or a repressive Islamic state. The main Arab Sunni alliance that contested Iraq's election in December said it had been a success.The big Sunni turnout means the new government will probably consist of a broader coalition than the current one.Sunni Arab inclusion was achieved, and Sunni Arabs will be represented in the new government in numbers roughly proportional to their presence in the country.The mainstream of the insurgency;which is a minority in the Sunni community anyway will be unable to avoid the tug of the political current. As the nationalists become increasingly involved in the political sphere, the extremistsal-Qaeda types, some Iraqi jihadists, diehard Baathists should wane in influence. The political and social landscape will become more hostile for the extremists, and cases of terrorists being turned in to the security forces will increase. But the extremists will not disappear completely;not yet anyway so long as they are supported by foreign elements who have no desire to see the experiment succeed .




You completely left out the Kurds in your equation also .To me ;that is the big unknown . Will the Kurds leave ?

Choux rated this answer Excellent or Above Average Answer
excon rated this answer Excellent or Above Average Answer
Itsdb rated this answer Excellent or Above Average Answer

Question/Answer
Choux asked on 01/14/06 - BUSH ADM (and Radical Right) Smear Murtha

"""The Bush Administration first attacked Rep. Murtha for his Iraq views by associating him with the filmmaker Michael Moore and Representative Jean Schmidt likened him to a coward on the floor of the House of Representatives. When those tactics backfired, Dick Cheney called Murtha "A good man, a marine, a patriot and he's taking a clear stand in an entirely legitimate discussion."
Though the White House has backed off publicly, administration officials have nevertheless recently made calls to military leaders to condemn the congressman. So far they have refused.

Rep. Murtha spent 37 years in the Marine Corps earning a Bronze Star, two Purple Hearts and a Navy Distinguished Service Medal. His service has earned him the respect of the military, and made him a trusted adviser to both Republican and Democratic presidents and leaders of the armed forces.""


We saw a genuine smear posted just below.

Comments???

tomder55 answered on 01/15/06:

Here is one of his latest comments from the saame town hall meeting that I referenced in my response:

"I worry about a slow withdrawal which makes it look like there's a victory

Bush has decried the "defeatism" of some of his political rivals like Murtha and I agree .He is a hero from the past no doubt but he is just plain wrong now. If the Democrats seriously consider adopting his position then they are destined to continue to be a minority party.





Choux rated this answer Excellent or Above Average Answer

Question/Answer
kindj asked on 01/13/06 - Wow!

The copy of the letter which follows, to US Representative John Murtha (D - Pennsylvania) from an attorney and Army Reserve officer, was received today. Someone called Mr. Stark's office to verify this letter has been sent. It has. December 2, 2005

Law Office of Christopher Stark
1615 S. Ingram Mill Road, Building F Springfield, MO 65804

Via FAX to: Representative John Murtha P.O. Box 780 Johnstown, PA 15907


Representative Murtha:

During the dark days of the American Revolution the Commanding General, George Washington, seemed unable to win any victories. There were wholesale desertions, troops were starving, the fledgling government was sporadic with money, food and ammunition in short supply. Out of this darkness emerged a genuine American Hero. This officer brilliantly led his troops in combat and though seriously, and almost mortally wounded, won victory after victory for the desperate and beleaguered American Continental Army.

After helping to turn the tide of war in the favor of the Americans, this officers fame grew as did his prestige, but his prowess on the battlefield, his courage under fire and indeed all of his life, is forgotten because of one act. His name is now synonymous with traitor in the dictionary. General Benedict Arnold had a brilliant military career of courage, honor, and sacrifice. Like you, in my opinion, he was a traitor to his country and to his oath as an American soldier.

It is indeed fitting that you are a member of the same political party as another traitor and seditionist, former Lieutenant John Kerry USN, who betrayed his country, not only on the very floor of the House of Representatives that you now serve, but also, secretly, in the presence of our enemies in Paris, France.

Kerry is a self-proclaimed warrior, while you earned your decorations, but the pair of you forgot one important thing. The United States of America and indeed the world are at war. We are at war with an implacable enemy. An enemy of racist, bigoted fanatics whose sole goal in life is to destroy the people of the United States of America, their culture and their religion. More American civilians have died on U.S. soil in this war than died in World War I, World War II, Korea, Vietnam, Grenada, Panama, and Desert Storm put together.

We are at war, Representative Murtha, and your actions and conduct give aid and comfort to our enemies. Just in case you have forgotten the definition of treason and sedition, I have attached Websters definition as Tabs A and B to this letter.

A wise man once said, There are no former Marines, only dead Marines. He was wrong. You are not a Marine. You have lost the right to use that title.You have dishonored all of those who have fought and died up to the day you stood on the floor of the House of Representatives and demanded that we withdraw immediately. You lied to the press, when you said you did not make that statement. I watched you make that statement. Albeit your bill, submitted, which I have also read, added a caveat, as soon as practicable. That is pure horseshit and you know it.

Yes, Representative Murtha, you have given aid and comfort to our enemies in a time of war. You have given them hope, which they have fast been losing, due to all of the victories and sacrifice by our sons and daughters on the field of battle in Iraq and Afghanistan. You have been honored by our enemies on the front page of Al Jezeera.

No, Representative Murtha, you are no longer a Marine. Your soul is dead. Your honor is dead, and without a soul or honor, you are nothing.

Be advised, my son is a Marine Officer. He has commanded men in battle through two tours and he is due to return to Iraq on a third tour. If he should be harmed in any way as a result of your actions on the floor of the House this week, I will do everything in my power to see to it that you are driven from office and that you are charged and tried for treason and sedition.

The Marine Officer whose message was read on the House Floor by fellow member of Congress, Jean Schmidt, was right. You are a coward. Marines do not cut and run.

Fortunately, your obesity prevents you from wearing your Marine Uniform with even a semblance of pride, but I know your face. In future if I am in a room when you arrive, you will not enter. If you are in a room, when I arrive, you will leave.

It is as simple as that.

OUT.

Christopher J. Stark, LTC, Infantry, USAR
Robert E. Mick
Email: remick35@iowatelecom.net
Great minds discuss ideas
Average minds discuss events
Small minds discuss people

Nothing on snopes.com about this. Sounds like it could be true. If so, hats off to Mr. Stark!

tomder55 answered on 01/14/06:

I can't agree . Arnold was motivated by ambition and greed and was a genuine traitor who ended up wearing the colors of the enemy . Murtha is wrong and caught up in his Cindy Sheehad moment but is not a traitor.Not even like Kerry; who the letter correctly pointed out ;consorted and strategized with the enemy in Paris.

Read this for further details about Arnold's betrayal.

I think Sgt.Mark Seavey at a town hall meeting n Arlington puts his finger on the Murtha problem :

"Yes sir my name is Mark Seavey and I just want to thank you for coming up here. Until about a month ago I was Sgt Mark Seavey infantry squad leader, I returned from Afghanistan. My question to you, (applause)
"Like yourself I dropped out of college two years ago to volunteer to go to Afghanistan, and I went and I came back. If I didn't have a herniated disk now I would volunteer to go to Iraq in a second with my troops, three of which have already volunteered to go to Iraq. I keep hearing you say how you talk to the troops and the troops are demoralized, and I really resent that characterization. (applause) The morale of the troops that I talk to is phenomenal, which is why my troops are volunteering to go back, despite the hardships they had to endure in Afghanistan.

"And Congressman Moran, 200 of your constituents just returned from Afghanistan. We never got a letter from you; we never got a visit from you. You didn't come to our homecoming. The only thing we got from any of our elected officials was one letter from the governor of this state thanking us for our service in Iraq, when we were in Afghanistan. That's reprehensible. I don't know who you two are talking to but the morale of the troops is very high."


Michelle Malkin and The Mudville Gazette are on the case .

As bad as Murtha is ,that scumbag Rep. Jim Moran he is hanging out with is worse .In 2003 he blamed the Iraq war on the Jews ."If it were not for the strong support of the Jewish community for this war with Iraq we would not be doing this," ..... "The leaders of the Jewish community are influential enough that they could change the direction of where this is going and I think they should."


ETWolverine rated this answer Excellent or Above Average Answer
kindj rated this answer Excellent or Above Average Answer

Question/Answer
kindj asked on 01/12/06 - Why the disparity?

I'm not as politically astute as a lot of you are; rather, I simply know what I think and say it (often without thinking).

However, I've noticed something lately that intrigues me.

During the Katrina fiasco, most of the left and a bit of the right was hammering Bush for not "acting quicker" to ensure the safety of Americans.

So by that, I guess conventional wisdom (not to mention that little scrap I like to call the Constitution) says that the President has a DUTY to ensure the safety of American citizens.

Yet, these same folks are throwing a fit over this wiretapping business (which was actually authorized initially by Jimmy Carter, if I read right).

Seems to me that the President is using his lawful authority to protect American citizens again.

If, as the left claims, he had a duty to protect Americans by using federal assets within the US borders (Katrina), then they can hardly say that he doesn't have the duty to protect those same citizens within those same borders.

Unless, of course, they WANT to be protected from hurricanes, but DON'T want to be protected from terrorists.

Am I missing something here, or need to go back to college, God forbid?

DK

tomder55 answered on 01/12/06:

since schools is back in session here is your reading assignments :

Thank You for Wiretapping Why the Founders made presidents dominant on national security.

Hold the Line

This debate is as old as the Republic .The parameters of congressional and presidential power have been debated throughout American history, beginning with the famous debate between Hamilton ("Pacificus") and Madison ("Helvidius).Legal scholar Edward S. Corwin once wrote that, "The Constitution is an invitation to struggle for the privilege of directing American foreign policy." Since the beginning of the 20th century in general ;with the exceptions of the weakening of the Presidency immediately after Watergate; the president has won that struggle.

The degree that the President has exercised this power has changed from conflict to conflict based on severity .I'm sure if Roosevelt had wiretapped in WWII and someone bemoaned the loss of civil liberty they would've been scoffed at .

It is my opinion that perhaps Bush has erred in not repeatedly pointing out the severity of the conflict we are engaged in . Oh ;he did at the beginning ;and people responded . But he made an error in not asking for a level of sacrifice of the people .Frank Gaffney makes simular arguments in his new book 'War Footing : 10 Steps America Must Take to Prevail in the War for the Free World '.

Anyway ;to make a long story short ; there are many people in this country who do not take the threat seriously . Perhaps the President is a victim of his own success . There are some who argue that we will not do what is really necessary to prevail until we are hit again. I do not look forward to that "inevidibilty " .

The Katrina thing ? I'd have to go into the nature of the nanny state to explain that .

ETWolverine rated this answer Excellent or Above Average Answer
Itsdb rated this answer Excellent or Above Average Answer
kindj rated this answer Excellent or Above Average Answer

Question/Answer
Choux asked on 01/10/06 - John Ashcroft reinvents himself

"""This is the way journalism is supposed to work. The Hill breaks a story on how the former US attorney general's lobbying firm is pushing to approve the sale to South Korea of an early-warning radar system made by an Israeli company with US technology. The competing bidder, BTW, is an American company, Boeing.
Then the Chicago Tribune follows up on John Ashcroft's three-month-old career as a lobbyist, primarily "centered on companies that want to capitalize on a government demand for homeland security technology that boomed under sometimes controversial policies he promoted while in office."

In year-end filings, Ashcroft's firm, The Ashcroft Group LLC, reported collecting $269,000, including $220,000 from Oracle Corp., which won Justice Department approval of a multibillion-dollar acquisition less than a month after hiring Ashcroft in October.

That represents in some cases only initial retainers or billings, the Trib explains. Oh, and there's a touch of irony: "As attorney general, Ashcroft sued Oracle in 2004 to try to block an earlier acquisition by the company."

There's nothing illegal about what Ashcroft is doing, the Trib points out. It's just, well, tacky. "To take the kind of prestige and stature of the attorney general [and lobby] . . . . It seems a little demeaning of the office, honestly," the Trib quotes John Schmidt, an associate attorney general in the Clinton administration, now a lawyer at Mayer Brown Rowe & Maw of Chicago.

And what does the new lobbyist do in his spare time? "In addition to his lobbying work, Ashcroft is a law professor at Regent University, with campuses in Virginia Beach, Va., and Washington, D.C., run by televangelist Pat Robertson." """

tomder55 answered on 01/11/06:

That unfortunately is how the game is played . There is a revolving door between the Federal Gvt. and K street.

Don't look at scandals and who is involved . The public rightly has decided a pox on both houses (parties).It is the party that is pro-active and jumps on the Abramoff scandal to propose meaningful reform that will benefit most from the fall out .

Individual pols. will be damaged .That's ok . The Gvt. needed a good mid-term cleaning anyway. The buzz words will be pork and earmarks (and everyone knows a pig has two ears ) .

This plays very well into the Republican agenda IF (and that's a big if) they are true to their rhetoric . It is time to full-fill the pledges they made when they won the house on the wake of the Gingrich revolution .

Choux rated this answer Excellent or Above Average Answer
Itsdb rated this answer Excellent or Above Average Answer

Question/Answer
ETWolverine asked on 01/09/06 - Why Bush didn't go to the FISA Court.

The Washington Times has supplied us with the answer as to why Bush didn't go to the FISA court for warrants.

It seems that the FISA court has leaks within its ranks, as well as judges discussing their preconceived notions about cases that they may be sitting on in the future.

In an article titled "Surveillance Court Is Seeking Answers: Judges Were Unaware of Eavesdropping" dated 1/4/06, the following paragraph appears:

"Some judges who spoke on the condition of anonymity yesterday (that means "leak" --- Elliot) said they want to know whether warrants they signed were tainted by the NSA program. Depending on the answers, the judges said they could demand some proof that wiretap applications were not improperly obtained. Defense attorneys could have a valid argument to suppress evidence against their clients, some judges said, if information about them was gained through warrantless eavesdropping that was not revealed to the defense. (That would be pre-judging the case publicly before it is before them and before they have seen any evidence --- Elliot)"

Find the full article here:
http://www.washingtonpost.com/wp-dyn/content/article/2006/01/04/AR2006010401864.html

So... what do we now know? That the FISA court has judges that are speaking anonymously to the press, and that it has judges that are judging cases and rendering opinions publicly before the cases appear before them.

I seem to remember people saying that Ruth Buzzy Ginsberg was right to not answer Senate questions about cases that she might one day rule upon, lest she share a pre-conceived opinion on the case. And yet, here are at least 2 judges (possibly more) who are doing exactly that. And worse, they are members of a secret court leaking information to the press. Either of these infractions should get those judges thrown off the FISA court. With both being the case, there may actually be some criminal charges involved, such as violations regarding their security privileges.

In addition to the problems discussed above, those who claim that the FISA court regularly rubber-stamps warrants are full of crap. They claim regularly that the FISA court approved any application made for any warrant since the inception of the FISA court in 1978.

That was only true until Bush came along.

Between 1978 and 1999, there was not a single rejection of any warrant requested of the FISA court. In 2000, there was one warrant application that was changed. So between 1978 and 2000, there were 13,087 warrant requests, none of which were rejected, and only one of which was modified.

Then along came Bush. Between 2001 and 2004, there were 5,645 applications for warrants. Of that number, 179 were modified and 4 were rejected outright.

In other words the FISA court suddenly went from a .008% rejection/modification rate during its first 22 years to a 3.2% rejection rate in the subsequent four years. That's a 400% increase in the rejection/modification rate.

(Source: http://bayosphere.com/blog/rodney_graves/20051228/raw_data_year_by_year_data_for_fisa_court_activities)

So much for the "rubber stamp" argument.

I call that obstructionism. I call it a case of liberal judges who hate Bush doing anything in their power to screw him.

So now we have a FISA court with leaks, with judges who are pre-judging cases and rendering opinions publicly, and judges who are actively obstructing the president for personal or partisan reasons.

Is it any wonder that Bush would find it necessary to ignore the FISA court and authorize the wire taps under his own presidential authority?

Elliot

tomder55 answered on 01/10/06:

This is all you need to know. The FBI had Zacarias Moussaoui in it's sites prior to 9-11 .

Moussaoui was arrested in Minneapolis on Aug. 16, 2001 after an instructor at a local flight school he attended called the F.B.I. to report him.F.B.I. agents in Minnesota asked Washington to obtain a special warrant to search his laptop .But the FBI had a reason for caution about applying to NSA for special search warrants .The F.B.I had become wary after Michael Resnick,the F.B.I. supervisor in charge of coordinating terrorist surveillance operations, was disciplined by Judge Royce Lamberth of NSA because of complaints that he had submitted improper information on applications.FISA went so far as to discipline Resnick (for f****ng paperwork !!)saying they would no longer accept warrant applications from him .FISA applications are voluminous documents, containing boilerplate language as well as details specific to each circumstance.FBI officials decided to adopt a "play-it-safe" approach that meant submitting fewer applications and declining to submit any that could be questioned.The result : The FBI never brought the Mousaoui case to the court .

After 9-11 information in the laptop revealed the plot and it's links to the German al-Qaeda cell it originated from ...and a Malaysian al Qaeda boss who had met with at least two other hijackers while under surveillance by intelligence officials.

ETWolverine rated this answer Excellent or Above Average Answer

Question/Answer
paraclete asked on 01/10/06 - Australia cleans up

http://network.news.com.au/image/0,10114,5094733,00.jpg

tomder55 answered on 01/10/06:

all energy options should be on the table. Technology exists to make coal and other fossil fuels clean buring . but it is time for major investments into future energy sources .

paraclete rated this answer Excellent or Above Average Answer

Question/Answer
Itsdb asked on 01/09/06 - An admission

Finally, someone admits that it isn't about what Bush is doing that's so outrageous, it's just that it's Bush - a contention some of us have made many, many times on these boards. Leonard Pitts opened this column on the domestic eavesdropping 'conspiracy' with, "Another president, perhaps."

Perhaps Lincoln or Roosevelt (or Clinton?), but not "King George, if you will" as Pitts put it. Almost restores a little faith in the media doesn't it? Or did Pitts just have a momentary lapse and didn't realize what he was doing?

Steve

tomder55 answered on 01/09/06:

During the Civil War the Bill of Rights wasnt eroded or compromised; it was ignored completely.Lincoln suspended habeus corpus ...13,000 people were arrested under martial law including war protesters.He also shut down hundreds of newspapers that disagreed with him. He also used the war to push through the "American System," a program of de facto nationalization of the transportation industry via massive subsidies to corporations that would agree to build "internal improvements" railroads, waterways, and canals.

Roosevelt interned 120,000 Japanese Americans . Roosevelt also ignored the 10th Amendment during the great depression . He railroaded through massive socialist legislations that the Supreme Court ruled unconstitutional . He threated to stack the court . So the Supreme's behaved and did not challenge any more of his New Deal.With all of that legislation, the country still went into a freefall in 1937-1938 . However, events of World War II from 1941-1945 and war mobilization changed the American economy .

How about Truman ? He seized the nation's steel mills in 1952 with an executive order .With the order the government took possession of over 80 percent of the industrys capacity.

There are many other examples of what manty would consider outright un-Constitutional Presidential actions that history has brushed aside knowing the extaordinary times in which these decisions were made.

My only conclusion is that people like Leonard Pitts do not take the war on Jihadistan seriously . Thankfully President Bush does.


Itsdb rated this answer Excellent or Above Average Answer

Question/Answer
excon asked on 01/08/06 - Supporting the Troops?


Hello wrongwingers:

Yeah, I support the troops too. But, instead of putting a bumper sticker on my car (so my neighbors will think highly of me), I wrote to my congressmen and demanded they give the Marines the body armor they needed to save lives.

Well, they didn't. I dunno why. Maybe they needed the money for a submarine or something. And, your sons died.....

So, what are you GOING TO DO, about your governments laze fare interest in your sons lives? Put a bumper sticker on your car?

excon

tomder55 answered on 01/09/06:

well then perhaps the Pentagon should ask the troops on the ground if they think extra armor is necessary . It is far from conclusive that is the case . Many of the troops are releaved when they can peel it off when they return from patrol . They say they feel encumbered by the weight and restricted by fabric that does not move as they do.It affects their mobility .They feel it increases their chance of being killed .

The debate between protection versus mobility has dominated military doctrine since the Middle Ages, when knights wrapped themselves in metal suits for battle, said Capt. Jamey Turner, 35, of Baton Rouge, La., a commander in the 1st Squadron, 33rd Cavalry Regiment.

The issue comes up daily on the battlefield in Iraq, and soldiers need to realize there is no such thing as 100 percent protection, he said.

"You've got to sacrifice some protection for mobility," he added. "If you cover your entire body in ceramic plates, you're just not going to be able to move."

Others in the regiment said the issue of protecting soldiers with more body armor is of greater concern at home than among soldiers in Iraq, who have seen firsthand how life and death hang on a sliver of luck when an improvised explosive device hits a Humvee.

"These guys over here are husbands, sons and daughters. It's understandable people at home would want all the protection in the world for us. But realistically, it just don't work," said Sgt. Paul Hare, 40, of Tucumcari, N.M.


Pressure on the infantry soldier to carry more and more equipment in order to improve capability is mounting.Soldiers need to carry ammunition, water, food and protective equipment. They use Personal Load Carrying Equipment (PLCE), a modular system of camoflaged belt, yoke and pouches. To this can be added two small rucksacks and a large rucksack for additional carrying capacity, when required.

Every soldier has a combat helmet which allows the soldier to still wear a respirator, ear defenders, goggles and a radio set, as necessary.On average a 160 lb soldier carries an additional 70 lbs of weight. Common injuries associated with the extra load include foot blisters, stress fractures, back strains, and knee pain. This in a desert environment where in the summer days frequently reach 130 degrees. How effective would they really be if you added ceramic enhanced armor to their load ?


excon rated this answer Excellent or Above Average Answer

Question/Answer
Choux asked on 01/07/06 - US Government Opening your Snail Mail

WASHINGTON - ""In the 50 years that Grant Goodman has known and corresponded with a colleague in the Philippines he never had any reason to suspect that their friendship was anything but spectacularly ordinary.

But now he believes that the relationship has somehow sparked the interest of the Department of Homeland Security and led the agency to place him under surveillance.

Last month Goodman, an 81-year-old retired University of Kansas history professor, received a letter from his friend in the Philippines that had been opened and resealed with a strip of dark green tape bearing the words by Border Protection and carrying the official Homeland Security seal.""



Any Comments?

tomder55 answered on 01/08/06:

The only conclusion I can come to from these various postings about survaillance is that the American people do not believe that we are at war. The Bush Adm. is a victim of it's own success. Perhaps if jihadistan did a couple more mass murders of Americans then the perception would be closer to the truth.Part of it is Bush's fault. We were told to go about our daily business. In general we have not been asked to sacrifice too much for the war effort .We don't even do sloganeering like .Loose lips sink ships.Hmm that's a concept eh ?

This article makes it sound unusual for items coming into the country to be subject to inspection ...it is not . I made this point in conncetion with the whole NSA brooohaaahaa ; When you physically go through customs returning from abroad you are subject to all types of insection of you're property and sometimes you're body ;and no falsely perceived claim to privacy can change that .

From 'Customs and Border Protection 'website :

All mail originating outside the United States Customs territory that is to be delivered inside the U.S. Customs territory is subject to Customs examination,

This isnt some secret program, they are telling you ahead of time that this will be done.

The Homeland Security Dept. has a whole list of what should be considered 'suspicious' published on their site. I'm sure a postal inspector doing the job they were hired to do alerted HSD about the letter because it fell within those guidelines. But this is not recent . Mail coming into this country for years was inspected for; as an example ;possible drug importing . During wars in the past mail was subject to larger degrees of inspection . Why is it an outrage now ?

Choux rated this answer Excellent or Above Average Answer
labman rated this answer Excellent or Above Average Answer
LTgolf rated this answer Excellent or Above Average Answer

Question/Answer
excon asked on 01/05/06 - Delay DeScussion


Hello:

I hear you loud and clear about trials. Am I to gather from that, that (1) you don't believe Delay is a crook, (2) you don't WANT to believe Delay is a crook, (3) irrespective of your belief, you think Asst. Attorney General Fisher (who got her job through a recess appointment which Bush used to bypass the Senate), and her boss, Attorney General Alberto Gonzales, will, or (4) will not play politics and go easy on the Republicans.

Heres my real question, is it ok with you that the guy YOU support is a crook, as long as he is supporting your adgenda? Is corruption ok with you? If a cop was about to give you ticket, but would take a hunskie instead, would you do it?

excon

tomder55 answered on 01/05/06:

My postion on Delay has not changed re:the Texas indictments . But the Abramoff connections are disturbing . I think for the best of the Republican Congressional delegation he should give up his attempt to regain the leadership.

I think that the House and Senate Republicans should be pro-active and to immediately introduce lobbyist influence peddling reforms .It is one thing to exercise free speech and another to bribe .The largest part of any reform has to have the onus on the elected official to control themselves . I would also beg them to eliminate the practice of 'earmarking ' pork riders onto appropriation legislation . Lobbyists make hay when they 'convince 'a lawmaker to do so . They should also tighten up severely the junket rules.

excon rated this answer Excellent or Above Average Answer

Question/Answer
ETWolverine asked on 01/05/06 - Things can change on a dime, can't they.

With Ariel Sharon's stroke/cerebral hemorrhage yesterday, it has become pretty clear that he can no longer serve as Prime Minister of Israel. His powers have been transferred to his deputy, Ehud Ohlmert, pending the upcoming election in March. I am sure that all our prayers are with him, regardless of our political views of Sharon. May G-d bless him with a speedy and complete recovery. No matter what you can say about the guy, pro or con, he is a man of conviction who put his life on the line for his country and for the Jewish people repeatedly, and he deserves to be honored for that alone, if nothing else. And I personally think there's plenty more to honor him for as well.

Until yesterday, it was believed that Sharon's new Kadima party would win the upcoming election pretty handily, relegating Likud and Labor to secondary positions. Now, though, the political landscape has changed completely. Without Sharon at the head, the new party will pretty much whither and die. Names of possible PMs are already being floated, including Peres (ugghhh!), Ohlmert (a decent choice in my opinion), Netanyahu (I'm only lukewarm on this posibility) and Justice Minister Tzipy Livni (don't know enough about her to have an opinion). The future Israeli politics has just become very unclear. Sharon had no obvious successor, and the political fighting is going to be pretty fierce. Buth then, politics has always been a bloodsport in Israel, so that is nothing new.

On the international political front, Sharon was one of Bush's staunchest supporters, and certainly his strongest supporter in the Middle East. The loss of Sharon as an ally will certainly hurt (though probably not cripple) Bush's peace plans for the Israel/PA region. In my opinion, the chances of a lasting peace that also provides security for Israel just declined significantly, and that cannot be good for Bush, for the USA, or for Israel. One thing that Bush should learn from this, however, is the need to choose an heir apparent that the rest of the party can get behind. Right now, there is no obvious Bush-backed candidate for 2008. Bush needs to change that state of affairs.

Things have become very confused very quickly, and I have no idea how they are going to play out. I haven't been this clueless about what will happen next in Israeli politics in years.

Interesting times ahead... I just hope that it isn't in the Chinese sense of the word.

Elliot

tomder55 answered on 01/05/06:

great observations although you never did explain to me why Bebe is a bad choice in you're view.

It is interesting how suddenly with Sharon's life in peril how the MSM has finally found words of praise for him. the major theme of their meme was that he had been a hawk who realized that his untennable positions could not bring lasting peace ;but that finally understood that a unilateral disengagement was the path to peace.

I pray for his recovery . He has been a good leader and a staunch ally .Although I doubt that he will be able to resume a political career I think he still has a valuable contribution yet to make (if nothing else the viability of his party) .
.......................................
Honestly ; I think Bush has been paying off some quid pro quo with McCain (there was no one who worked harder for Bush 2004 then McCain ) and has been doing everything in his power to make him shine.

ETWolverine rated this answer Excellent or Above Average Answer
excon rated this answer Excellent or Above Average Answer
labman rated this answer Excellent or Above Average Answer

Question/Answer
excon asked on 01/03/06 - DeLay is DeDone


Hello Experts:

Have I ever asked that question before? I think I dedid. First Scanlon, now Abramoff, and about 15 congressmen are next. I think DeLay is desecond on delist.

DeExcon

tomder55 answered on 01/04/06:

I have heard as many as 20 Reps in Congress and Senators and their staffers from both parties.The politically amoral Abramoff wasn't too partisan in his bribery and influence peddling.The news is obsessed with Delay but hardly a mention of Harry Reid's connections are mentioned (***see below the roster of Democrat beneficiaries compiled by FEC records).If they are implicated and they are guilty then screw em ...Let the chips fall where they may .

Abramoff has nothing on Geroge Soros in the influence peddling and buying game. Abramoff spent about $5 million on political influence operations.Soros reportedly spent $400 million in 2004 on his network of foundations and non-profit groups just trying to oust President Bush .

I guess John McCain of Keating 5 Fame was not able to change the political culture in Washington just by making a bogus campaign reform law.It did nothing to deter the K Street fat cats. The Indian tribes have shelled out big wampum to both parties to keep their casinos tax free.
........................................

*** Abramoff Lobbying & Political Contributions to Democrats,
per FEC Records
# # # # #



Democratic Senatorial Campaign Cmte $423,480
Democratic Congressional Campaign Cmte $354,700
Democratic National Cmte $65,720
Patrick J. Kennedy (D-RI) $42,500
Patty Murray (D-Wash) $40,980
Charles B. Rangel (D-NY) $36,000
Harry Reid (D-Nev) $30,500
Byron L. Dorgan (D-ND) $28,000
Tom Daschle (D-SD) $26,500
Democratic Party of Michigan $23,000
Brad R. Carson (D-Okla) $20,600
Dale E. Kildee (D-Mich) $19,000
Steny H. Hoyer (D-Md) $17,500
Tom Harkin (D-Iowa) $15,500
Democratic Party of Oklahoma $15,000
Chris John (D-La) $15,000
John Breaux (D-La) $13,750
Frank Pallone, Jr (D-NJ) $13,600
Richard A. Gephardt (D-Mo) $12,000
Mary L. Landrieu (D-La) $11,500
Barney Frank (D-Mass) $11,100
Max Baucus (D-Mont) $11,000
Maria Cantwell (D-Wash) $10,000
Democratic Party of North Dakota $10,000
Nick Rahall (D-WVa) $10,000
Democratic Party of South Dakota $9,500
Democratic Party of Minnesota $9,000
Ron Kind (D-Wis) $9,000
Peter Deutsch (D-Fla) $8,500
Joe Baca (D-Calif) $8,000
Dick Durbin (D-Ill) $8,000
Xavier Becerra (D-Calif) $7,523
Tim Johnson (D-SD) $7,250
Democratic Party of New Mexico $6,250
Daniel K. Inouye (D-Hawaii) $6,000
David E. Bonior (D-Mich) $5,000
Jon S. Corzine (D-NJ) $5,000
Democratic Party of Montana $5,000
Fritz Hollings (D-SC) $5,000
Jay Inslee (D-Wash) $5,000
Thomas P. Keefe Jr. (D-Wash) $5,000
Barbara A. Mikulski (D-Md) $5,000
Deborah Ann Stabenow (D-Mich) $5,000
Earl Pomeroy (D-ND) $4,500
Tom Carper (D-Del) $4,000
Kent Conrad (D-ND) $4,000
Jerry Kleczka (D-Wis) $4,000
Sander Levin (D-Mich) $4,000
Robert T. Matsui (D-Calif) $4,000
George Miller (D-Calif) $4,000
Kalyn Cherie Free (D-Okla) $3,500
James L. Oberstar (D-Minn) $3,500
Charles J. Melancon (D-La) $3,100
Jeff Bingaman (D-NM) $3,000
Cal Dooley (D-Calif) $3,000
John B. Larson (D-Conn) $3,000
David R. Obey (D-Wis) $3,000
Ed Pastor (D-Ariz) $3,000
Nancy Pelosi (D-Calif) $3,000
Richard M. Romero (D-NM) $3,000
Brad Sherman (D-Calif) $3,000
Bennie G. Thompson (D-Miss) $3,000
Max Cleland (D-Ga) $2,500
Grace Napolitano (D-Calif) $2,500
Henry A. Waxman (D-Calif) $2,500
Bill Luther (D-Minn) $2,250
Gene Taylor (D-Miss) $2,250
Neil Abercrombie (D-Hawaii) $2,000
Ken Bentsen (D-Texas) $2,000
Dan Boren (D-Okla) $2,000
Rosa L. DeLauro (D-Conn) $2,000
John D. Dingell (D-Mich) $2,000
Doug Dodd (D-Okla) $2,000
Ned Doucet (D-La) $2,000
Lane Evans (D-Ill) $2,000
Sam Farr (D-Calif) $2,000
John Neely Kennedy (D-La) $2,000
Carl Levin (D-Mich) $2,000
Blanche Lincoln (D-Ark) $2,000
Nita M. Lowey (D-NY) $2,000
Robert Menendez (D-NJ) $2,000
Adam Schiff (D-Calif) $2,000
Ronnie Shows (D-Miss) $2,000
Adam Smith (D-Wash) $2,000
Ellen O. Tauscher (D-Calif) $2,000
Mike Thompson (D-Calif) $2,000
Maxine Waters (D-Calif) $2,000
Peter DeFazio (D-Ore) $1,500
Norm Dicks (D-Wash) $1,500
John Kerry (D-Mass) $1,400
Barbara Boxer (D-Calif) $1,000
Dennis Cardoza (D-Calif) $1,000
Hillary Rodham Clinton (D-NY) $1,000
Jim Costa (D-Calif) $1,000
Susan A. Davis (D-Calif) $1,000
Eliot L. Engel (D-NY) $1,000
Dianne Feinstein (D-Calif) $1,000
Tim Holden (D-Pa) $1,000
Patrick Leahy (D-Vt) $1,000
Joe Lieberman (D-Conn) $1,000
Jim Maloney (D-Conn) $1,000
David Phelps (D-Ill) $1,000
Charles S. Robb (D-Va) $1,000
Brian David Schweitzer (D-Mont) $1,000
Pete Stark (D-Calif) $1,000
Gloria Tristani (D-NM) $1,000
Derrick B. Watchman (D-Ariz) $1,000
Rick Weiland (D-SD) $1,000
Paul Wellstone (D-Minn) $1,000
Ron Wyden (D-Ore) $1,000
Bob Borski (D-Pa) $720
Shelley Berkley (D-Nev) $500
Howard L. Berman (D-Calif) $500
Henry Cuellar (D-Texas) $500
Democratic Party of Washington $500
Barbara Lee (D-Calif) $500
Loretta Sanchez (D-Calif) $500

The question becomes how much of his activities were illegal. Cunningham was taking fists full of dollars ...it gets murkier when you are talking about golf junkets .



excon rated this answer Excellent or Above Average Answer
purplewings rated this answer Excellent or Above Average Answer

Question/Answer
excon asked on 01/03/06 - What if they gave a war and nobody came?


Hello experts:

In the upside-down topsy turvy world of George Bush, this war might just be that one.

If we just leave Iraq, the war would end, wouldn't it? If it won't, who's gonna be fighting who? And, is staying there gonna stop that?

Help me out. I'm a little confused, just as are 60% of us. We still don't quite know what we're doing in Iraq.

Certainly a war worth fighting is a war worth understanding. Isn't it?

excon


tomder55 answered on 01/03/06:

ok so when we finished wars against Germany and Japan we up and left .....didn't we ? When military operations were finished in the Balkans we just left ....no we didn't ;same in Afghanistan . But lets see now ..we did punch out and go home in 1991 in Iraq . Alot of good that did us.

perhaps we should bring our troops home from guarding Munich beer halls first .

the plan is for a freed Iraq to inspire and stabilize the entire Middle East . First it is our responsibilty to help them stabilize their infant democracy .


There is some justice in one charge that is frequently leveled against the United States, and more generally against the West: Middle Easterners frequently complain that the West judges them by different and lower standards than it does Europeans and Americans, both in what is expected of them and what they may expect, in terms of their economic well-being and their political freedom. They assert that Western spokesmen repeatedly overlook or even defend actions and support rulers that they would not tolerate in their own countries.

...there is nevertheless a widespread [Western] perception that there are significant differences between the advanced Western world and the rest, notably the peoples of Islam, and that these latter are in some ways different, with the tacit assumption that they are inferior. The most flagrant violations of civil rights, political freedom, and even human decency are disregarded or glossed over, and crimes against humanity, which in a European or American country would evoke a storm of outrage, are seen as normal and even acceptable.

...The underlying assumption in all this is that these people are incapable of running a democratic society and have neither concern nor capacity for human decency."


'The Crisis of Islam', Bernard Lewis.

The Bush doctine is inviting the Muslim world to join the civilized world. Sure it is a gamble but the alternative is worse . It will not be easy to wage war with a billion Muslims ;primitive as they may be ;given their increasing ability to gain weapons that can even the playing field .

Choux rated this answer Excellent or Above Average Answer
excon rated this answer Excellent or Above Average Answer

Question/Answer
Choux asked on 12/31/05 - Political Parties

This afternoon, I read a book review about the beginning of the Republic, the failures of the founding fathers...one of the points made was that the founding fathers did not anticipate the formation of political parties.

The vision was that a virtuous man would vote his conscience and the Electoral College was in place to correct any unhealthy excess. Finally, the electoral college makes sense. That was the filter for "scoundrels" that is supposed to be the job of the political parties now(I guess :)).

Anyway, weren't there political parties in England at that time? Is anyone familiar with this subject; I'm not planning on reading the book, and I would like to hear some discussion on this failure to take into account that political parties would be formed.

Thanks!

tomder55 answered on 01/01/06:

Truth be told ;the political parties that emerged were natural extentions of the debates that were made at the founding of the Republic(the Federalists and Anti-Federalists) . There was at the beginning sectional and philisophical differences that can be summed up in the question of the strength of the Federal Government. Yes ;many of the Founding Fathers had a negative view of political parties. Despite their objections, most of these men found themselves affiliated with a political party during their careers in government

Northern founders led by Alexander Hamilton, believed in a strong national government with centralized authority. Although George Washington never embraced a party,but he leaned toward these ideals also. Thomas Jefferson and James Madison,and the Southern contingent in general believed in a modest central government, limited commercial activity, and strong farming communities . By Washington's second term these paries had developed and in his second term he had to deal with the conflict .Being as charismatic as he was he was able for the most part to stay above the fray.Washington despised the idea of political parties, formed in such a way as to pit one group of citizens against another. In his farewell address in 1796 he said:

They serve to organize faction, to give it an artificial and extraordinary force; to put, in the place of the delegated will of the nation, the will of a party, often a small but artful and enterprising minority of the community; and, according to the alternate triumphs of different parties, to make the public administration the mirror of the ill-concerted and incongruous projects of faction, rather than the organ of consistent and wholesome plans digested by common counsels, and modified by mutual interests.

"However combinations or associations of the above description may now and then answer popular ends, they are likely, in the course of time and things, to become potent engines, by which cunning, ambitious, and unprincipled men will be enabled to subvert the power of the people, and to usurp for themselves the reins of government; destroying afterwards the very engines, which have lifted them to unjust dominion."


The 1796 election between John Adams and Jefferson became the first battle ground of these parties. Adams representing the Federalist Party and Jefferson the Republican Party (which would eventually become the Democrats).

James Madison was the last Federalist elected President in 1816 .The Republican Party also disolved rather quickly . As economic growth and territorial expansion came the agrarian ideals of Jefferson became less relevent . The party split in 1828.

The Democratic-Republican Party, led by Andrew Jackson, was formed. Supporters favored a limited national government and were opposed to an economic aristocracy. Eventually, this party changed its name to the Democrat Party.

The National Republican Party, led by John Quincy Adams, was also formed. Supporters favored strong economic nationalism, much like the Federalists. 1834 they disolved and the Whig Party emerged.They later joined with the 3rd Party The Free Soil Party ,(a party formed to oppose exansion of slavery into new territores ),to form the Republicans.

Of all the founders perhaps only Franklin could be catagorized as a centrist but he was more a philosopher than a politician.

.......................................

English political parties can be dated to 1662 after the English Civil War, with the creation of the Court Party and the Country Party, soon becoming known as the Tories (now the Conservative party,although they are still commonly referred to as 'the Tories') and the Whigs (now the Liberal Democrats,or Liberals). The two remained the main political parties until well past the founding of the United States.The term 'Tory' originates from the Exclusion Bill crisis of 1678-1681... the Whigs were those who supported the exclusion of the Roman Catholic Duke of York from the thrones of England, Ireland and Scotland, and the Tories were those who opposed it. Both names were originally insults: a "whiggamor" was a cattle driver, and a "tory" was an Irish term for an outlaw.Generally, the Tories were associated with lesser gentry and the Church of England, while Whigs were more associated with trade, money, "land magnates", expansion and tolerance.





Choux rated this answer Excellent or Above Average Answer
powderpuff rated this answer Excellent or Above Average Answer

Question/Answer
Choux asked on 12/29/05 - Square One
Laugh or Cry or Whatever


Free Saddam to end woes in Iraq, lawyer tells Bush


AMMAN (Reuters) - The United States should free Saddam Hussein if it wants to end its problems in Iraq and earn the friendship of Arabs, the former Iraqi president's lawyer wrote in a letter to U.S. President George W. Bush.

The chief lawyer for Saddam at his trial for crimes against humanity in Baghdad told Bush that Iraqis who supported their former leader were waiting for a bold decision from the world's most powerful statesman to free him.

"I call on you (President Bush) to release Mr. President (Saddam) immediately to allow the Iraqis to decide his fate. Only then will you get out of your predicament in Iraq and truly become an advocate of justice," Khalil Dulaimi wrote in a letter obtained by Reuters.



Are you laughing or crying??

Comments??

tomder55 answered on 12/30/05:

I agree ;free the bastard and watch the stones start flying !!! Here is what I think would be punishment fitting the crime .

Choux rated this answer Excellent or Above Average Answer
ETWolverine rated this answer Excellent or Above Average Answer
Itsdb rated this answer Excellent or Above Average Answer
labman rated this answer Excellent or Above Average Answer
purplewings rated this answer Excellent or Above Average Answer

Question/Answer
Choux asked on 12/29/05 - English Only Speakers?

USA Today(Dec. 29) -- A state judge could rule soon on whether Alabama must give driver's license exams only in English or can test potential motorists in 12 other languages as it has since 1998.

A long-running legal battle pits English-only advocates here against the state and civil rights groups. The driver's license issue, which states have grappled with for decades, is part of a debate over immigration that has reached Congress and state legislatures.

Six states still require residents to take the written exams in English, says K.C. McAlpin, executive director of ProEnglish, an Arlington, Va.-based organization that supports laws or constitutional amendments declaring English the USA's official language.""



Is it possible to legislate English speaking only for particiption in American life, and sepaately, is it a good idea?

tomder55 answered on 12/30/05:

as an alternative lets say that we print road tests in any language the applicant wants and then pass on the additional expenses to the licensing fees of the applicants who request it .

I am in favor of recognition that English is the official language in the United States for government purposes.

Choux rated this answer Excellent or Above Average Answer
Itsdb rated this answer Excellent or Above Average Answer
purplewings rated this answer Excellent or Above Average Answer

Question/Answer
Choux asked on 12/29/05 - Soldiers Blogging

""ANYONE wanting to hear daily insights into what it is like to be in a convoy hit by an explosion or ordered to pick up the body parts of comrades dismembered by a suicide bomber does not have to be there in person any more.

Instead they just need to log on to the internet from the safety of their home or office.

In a development that is worrying US military commanders in Iraq, a growing number of US soldiers - 200 at the last count - have set up their own blogs, or internet diaries, and are updating them from the battlefield.

The phenomenon, helped by internet cafes at almost all US camps to permit soldiers regular contact with home, has for the first time allowed personal reports of the reality of combat to be read as they happen.""



Do you thing the Military should prohibit soldiews from blogging????

tomder55 answered on 12/30/05:

no so long as they do not disclose operation sensitive material that might put the rest of their unit at risk . They should know that if they are blogging then the world is reading it . The military does have the right to sensor them however .In previous wars the soldiers letters were simularily sensored for the same reasons I cite above .

I log on to the bloggers sites not to get the morbid picture that the author cites as their perverted reason to do so ;but from these blogs one gets a different picture of what is really happening in Iraq divorced from the one -sided accounts of reporters who never stray outside the green-zone.

Choux rated this answer Excellent or Above Average Answer

Question/Answer
Choux asked on 12/26/05 - Coalition Provisional Authority Scandal

Last Friday I was on the panel of Bill Mahers season finale of his HBO show Real Time. Bills guest by satellite was Senator Norm Coleman from my home state of Minnesota. Because Coleman serves on the homeland security committee, Maher asked him to comment on former homeland security director Tom Ridges recent revelation that the Bush administration would often issue terror alerts that he didnt think were warranted.
Could it be that they were using terror alerts politically?

Coleman answered that it was always good to err on the side of caution. Maher followed by asking Coleman if it struck him as odd that there havent been any terror alerts since the election?

After a long laugh from the audience, Coleman answered with some stuff about there still being a high level alert, but then reassured everybody with: "If in fact people used these things for political purposes, Im sure Congress will look into that."

Wow! If people used these things for political purposes, hes sure the Republican Congress will look into it? How is it that this guy cant get through a five minute interview on a political comedy show without having to resort to total b.s.?

Right now Coleman is looking into the Oil-for-Food program, which was administered by the Security Council in the U.N., mostly by the U.S. and Britain. That didnt stop Coleman from demanding Kofi Annans resignation without any proof of any wrongdoing on his part. There appears to be anywhere from one to two billion dollars stolen through the program with most of that going to Saddam. Primarily the U.S. and Britain took it upon themselves to make sure that none of this money went toward making W.M.D.s. They seemed to have done a pretty good job.

Meanwhile, ***the Coalition Provisional Authority, which we ran, has lost 8.8 billion dollars***. By lost, I mean its totally unaccounted for. Not only has Congress not "looked into" this $8.8 billion and who might have it now, but it seems that some members are completely unaware that this staggering sum, which was supposed to go toward rebuilding Iraq, is missing. The Sunday morning after the White House Correspondents dinner, I ran into Senator George Allen at a brunch thrown by John McLaughlin and his wife. Allen had never heard of the missing $8.8 billion, or at least that's what he told me. And he's on the Senate Foreign Relations Committee.

Stunned, I went up to Susan Page of USA Today and her husband Carl Lubsdorf of the Dallas Morning News, two veteran Washington political reporters, and told them about Allens ignorance of this huge scandal, which has no doubt contributed to hatred for America and the deaths of our troops. Theres less electricity in Iraq now than there was before we invaded Iraq.

Turns out that Page and Lubsdorf had also never heard of the unaccounted-for $8.8 billion. For a moment I thought that maybe I had been imagining things.

Then I spotted my friend Norm Ornstein, scholar from the American Enterprise Institute. "Would you believe it if Norm Ornstein told you about the $8.8 billion?" I asked Susan and Carl.

"Sure."

I brought Norm over, and indeed I had not been imagining things. "It was a huge story," Norm told them.

"Was it in the New York Times?" Carl asked Norm.

"Yes," Norm assured him.

What in Gods name is going on?""-Al Franken


What *is* going on????

tomder55 answered on 12/27/05:

As you know I was very critical of the CPA and how they administered Iraq during Bremer's tenure .I am stunned that Allen had not heard of the missing money if this story is accurate .(I do not know if the amt. is accurate ;but that is why investigations are needed .Perhaps Allen was just brushing off this author because he did not want to deal with it at a brunch );the story was news in 2003 .

The claim that Iraq has less electricity is an outright lie. Sure ;if you measure Baghdad exclusively then perhaps the claim is true since the insurgency loves to attack the electic grid there ;but overall there are many more places on the grid in Iraq then ever were under Saddam. During October,the powering-up of two generators in al-Qudas, a rural village 30 miles west of Baghdad,added an additional 192 megawatts of electricity to the national grid .To put it in context, about one megawatt of electricity can power 3,000 homes in Iraq. Don't forget ...there were many areas of the country that he could care less about except to beat them down and conduct reigns of terror on. Octobers production in the country regularly exceeded 5,000 megawatts, compared to the pre-war level of 4,400. Since arriving last year, the U.S. Army Corps of Engineershas strung 8,600 kilometers of transmission line, built over 1,200 towers and added over 1,800 megawatts to the grid.


This author loses all credibilty when he/she gives Koffi Annan a pass on the 'oil-for food' program. Had the sanction program worked then possibly there would've been no need to take out Saddam militarily . The author severly underestimates the amt of money plundered by the Annan's and their accomplices ;the numbers in the 'oil for food 'plunder reaches into the area of $21 billion .

Choux rated this answer Excellent or Above Average Answer

Question/Answer
excon asked on 12/24/05 - Hi Ho, Hi Ho, the wicked witch is dead


Hello wingers:

Bush will be impeached. It doesn't make me happy, because I don't like Cheney either.

You can't spy on people without a warrant in this country. The Fourth Amendment is a bedrock of our democracy.

By the way, are you going to give me the tired old excuses that other presidents have done the same thing? Well, if they did, and weren't prosecuted, then shame on you. But, of course, they didn't.

Believe me, if you wingers could have gotten Clinton on a charge like this, instead of just enjoying a blow job, you would have.

excon

tomder55 answered on 12/26/05:

every President since Lincoln has used the inherent executive authority to survaille against enemies of the country both foreign and domestic in times of war. That is undeniable. In WWII Roosevelt got around his liberal sensitivities by spying on Brits and having the Brits spy on our nationals ,and then just exchanging the information .

as to you're point about Nixon in your'e response to Ceebee; Nixon was using the law enforcement agencies to spy on domestic POLITICAL ENEMIES . That is where the abuse of power is ;not in the survaillance of names in the lap top computer of an al-
Qaeda chief captured in Afghanistan.(I could also add Hillary's use of FBI files as a violation equally as damning as Nixon's but that is a different discussion .No;I do not think they impeached Clinton for the right reason ;there were other more compelling reasons to do so )

Bush will only be impeached if the Democrats capture the Congress in 2006 ;not very likely in my book. I have no doubt that there will be hearings on the survaillance issue .Bring them on ! I'd love to know what scoundrel leaked this information to the NY Slimes ;that is where the real crime lay.

excon rated this answer Excellent or Above Average Answer

Question/Answer
excon asked on 12/24/05 - The Constitution is not a suicide pact.


Hello wingers:

Just who amongst you believes that stupid crap? Ive heard some real Fascist stuff recently, but this takes the cake. Hell, if I were you, I wouldn't admit it either.

Inherent in the statement above, is the belief that the Constitution is really foo foo document. Thats its really girly girly stuff. But, that when it comes time to REALLY protect us, you need to shitcan the Constitution.

If I believed that, then living here, being a staunch American, and even voting doesnt mean anything. Its all a house of cards. Our history and who we are, are meaningless.

Well, not to me. I like the Constitution. I think it means something. Oh, oh, by the way, in the oath of office, the president swore to protect the Constitution. He didnt swear to protect you.

excon

tomder55 answered on 12/25/05:

Here is the complete quote of Justice Robert H. Jackson in his dissenting opinion in Terminiello v. Chicago :This Court has gone far toward accepting the doctrine that civil liberty means the removal of all restraints from these crowds and that all local attempts to maintain order are impairments of the liberty of the citizen. The choice is not between order and liberty. It is between liberty with order and anarchy without either. There is danger that, if the Court does not temper its doctrinaire logic with a little practical wisdom, it will convert the constitutional Bill of Rights into a suicide pact."The phrase can be hyperbolic but under certain circumstances, war for instance, it is appropriate in emphasizing how consitutional interpretations under peacetime conditions would be inappropriate under wartime conditions.


Federalist 23, Alexander Hamilton wrote that those responsible for the nation's defense must be granted all of the powers necessary to achieve that end.

The answer to you're comment lies in the very oath you quote."I do solemnly swear (or affirm) that I will faithfully execute the Office of President of the United States, and will to the best of my Ability, preserve, protect and defend the Constitution of the United States." The President can override statutes if in doing so he believes that the result will be the preservation of the constitution.But even so ;in this case he did not override the specific satute involved .

The FISA Act does allow for activities to bypass normal Fourth Amendment protections against unreasonable searches.The condition for this is that any criminal activity discovered outside the scope of national security cannot be given to a domestic prosecutor because that would constitute evidence garnered without the conventional warrant normally required which would be infringement of civil liberties.
The only privilege denied is the right to secretly plot the demise of fellow Americans.

The federal courts have long recognized that when it comes to waging war, the President, not Congress or the courts, is the supreme authority. The military is sworn to"support and defend the Constitution of the United States against all enemies, foreign and domestic." Given that during time of war the military can perform searches and surveillance operations without warrants, why is it that their commander in chief can't do the same when the enemy fluctuates between foreign and domestic? These are the very two enemy categories that our military is sworn to fight and so is the Commander in Chief .

The Fourth Amendment, which states:

The right of the people to be secure in their persons, houses, papers, and effects, against unreasonable searches and seizures, shall not be violated, and no warrants shall issue, but upon probable cause, supported by oath or affirmation, and particularly describing the place to be searched, and the persons or things to be seized.

So all searches and seizures of Americans or their property (including interceptions of telephonic and electronic communications) must be reasonable. The Fourth Amendment includes requirements for the issuance of search warrants, and many critics of the NSA program seem to assume that this means that all searches must be executed pursuant to a warrant. This assumption is wrong. There are dozens of situations where warrantless searches have been approved by the courts. The overriding principle is that searches of Americans must be reasonable. The interception of communications between al-Qaeda and their domestic contacts can and should be interpreted as reasonable.



In my other replies to this issue I have backed up my comments with court decisions dating back to the Civil War.

excon rated this answer Excellent or Above Average Answer

Question/Answer
Choux asked on 12/23/05 - Unilateral Use of Presidential Power

"President Bush is taking on an issue of presidential powers over which presidents have stumbled before.

At a news conference, the president said that warrantless eavesdropping targeted at foreign citizens by the National Security Agency will continue as long as the enemy threat continues. He cited the Congressional resolution of 2001 authorizing the use of force against Iraq.
The president also denounced the news media for breaking the story, calling it harmful to national security.

Mr. Bush chose not to avail himself of the tool that Congress has provided for the purpose of eavesdropping - The Foreign Intelligence Surveillance Act, passed in 1978 as a reaction to President Nixon's domestic spying. That Act permits the president to apply in secret to a special court for a warrant. The administration is authorized to begin surveillance for 72 hours while waiting for the warrant, which is almost always granted.

In his defiance, there may be peril for the president, as President Nixon discovered when the House Judiciary Committee voted three articles of impeachment against him, one of them for abusing the power of three agencies - the FBI, CIA, and IRS. Nixon took the position that he was using inherent presidential powers granted by the Constitution.

The Constitution says that the president shall exercise the "executive power" and shall be commander in chief of the armed forces, but it doesn't spell out what those powers are. Some presidents have come up with what they call the "inherent power" of the presidency, which tends to be what they make it.

Historians have said that President Lincoln freed the slaves, blockaded Southern ports, and instituted a draft all without constitutional authority. President Reagan invoked "inherent powers" to justify the illegal sale of missiles to Iran and the illegal financing of the civil war in Nicaragua. Short of impeachment, the Congress has no way of stopping a willful president except to deny him funds. That, of course, is unlikely, especially with a Republican-controlled Congress.

Senate Judiciary Committee Chairman Arlen Specter promises open hearings on eavesdropping early in the new year. This is clearly an issue which is not likely to go away."= Daiel Schorr



www.csmonitor.com | Copyright 2005 The Christian Science Monitor. All rights reserved.



Are we at a point in history where we need a President that can operate at will???

tomder55 answered on 12/24/05:

we are not even close to the point where the President is operating "at will".
These comparisons to Nixon are rediculous. For the record Nixon was using the various agencies to spy on domestic political opponents . That was abuse.

President Bush is exercising war powers as Commander in Chief granted by the Constitution to gather intelligence on a foreign enemy we are at war with . Please not the difference.

Warrantless surveillance, is nothing new historically, legally or constitutionally, particularly in wartime. Never mind the authority that Congress gave him to fight al-Qaida in the September 2001 Authorization for Use of Military Force Act. He has "inherent authority" under the Constitution, as commander in chief, to conduct specific kinds of warrantless surveillance. And this "inherent authority" isn't something Bush cooked up.

"The Department of Justice believes, and the case law supports(see below for just some court decisions on this issue ), that the president has inherent authority to conduct warrantless physical searches for foreign intelligence purposes." Nope that is not ALberto Gonzalez speaking ;that is former President Clinton's deputy attorney general,Jamie Gorelick's testimony before the Senate Intelligence Committee from a 1994 hearing on the president's "inherent authority" to order physical searches of homes of U.S. citizens without a warrant.

In Sealed Case No. 02-001, FISA said in 2002: "The Truong court, as did all the other courts to have decided the issue, held that the President did have inherent authority to conduct warrantless searches to obtain foreign intelligence information. . . . We take for granted that the President does have that authority and, assuming that is so, FISA could not encroach on the President's constitutional power."
That Clinton and every president since Carter inclusive have used the NSA this way means Bush certainly has a reasonable stance in asserting this power.

What the author of this article is really saying is that if the NSA picks up a communication between al Qaida operatives and Achmed in the USA and they tell him when and where to set off the dirty bomb;the author doesn't want the NSA to listen in on that conversation because Achmed's rights are more important than thousands of Americans'lives. To that I just have to say that I disagree.



...............................

1928. The Supreme Court case of Olmstead v. United States (217 U.S. 438), a criminal case, resulted with the Court stating the use of warrantless wiretaps was not unconstitutional, because the Fourth Amendment's warrant requirement did not include conversations. The Court ruled, "The reasonable view is that one who installs in his house a telephone instrument with connecting wires intends to project his voice to those quite outside, and that the wires beyond his house, and messages while passing over them, are not within the protection of the Fourth Amendment."

1974 Third Circuit Courts opinion in U.S. v. Butenko was, "foreign intelligence gathering activitymay be conducted through warrantless electronic surveillance." The Court added "[a]s Commander-in-Chief, the President must guard the country from foreign aggression, sabotage, and espionage."

1974 The Fifth Circuit Court case of Ivanov v. United States (419 U.S. 881), stated that "warrantless electronic surveillance (is) permitted so long as the primary purpose was to obtain foreign intelligence."








Choux rated this answer Excellent or Above Average Answer
excon rated this answer Excellent or Above Average Answer

Question/Answer
Choux asked on 12/23/05 - 10 Christmas Wishes

Dear Santa: Here are ten political wishes that you can grant and make my Christmas. by **Bob Burnett**

൒. Come out of hiding, Al Gore. The Democratic Party needs you as their environmental spokesperson. After Hurricane Katrina, and the other disastrous storms of 2005, the American people dont have to be convinced that theres a problem with global climate change.
They need to know what to do. You understand this. Your speech, When there is no vision, the people perish, was a superb first step. Al, you can provide the leadership that the Party and the nation need to deal with this peril. Come out, come out, wherever you are.

9. Let go, John Kerry. Youre acting out the lyrics from Lucinda Williams song, Well it's over, I know it, but I can't let go. You lost your bid for the presidency. You couldnt defeat an inept President who made one disastrous mistake after another. Its over. Quit acting as if youre the leader of the Democratic Party. You arent even the leader of the Senate Minority. You had your chance. Please dont run again. Let go, John.

8. Get a job, Bill Clinton. Charging thousands of dollars for inspirational speeches isnt a real job. Its celebrity fluff. How about taking on a challenge? Rebuild your reputation by making a contribution that will strengthen America. The aftermath of Hurricane Katrina left a mess that the Bush Administration is unwilling to clean up. America needs the kind of leadership that you can provide. Move to New Orleans. Become our champion for the reconstruction. Bring us together. Take on a real job, Bill. Its way too early for you to retire to a life of Oprah appearances and golf.

7. Get out, Joe Lieberman. The Democratic Party has a big tent but no more room for you. Either quit backing President Bush and blasting your fellow Dems, or leave the Party. Youre worse than an irritant. You are an embarrassment. Your statements seem scripted by Karl Rove. Youre not a centrist Dem. You are a Republican. Get out of the Party, Joe.

6. Get a grip, Hillary Clinton. Its time for you to quit being coy about your position on Iraq. The Democratic Party needs leadership in 2006 and you can provide it. You have an easy re-election campaign for the Senate. No harm will come to you by taking a strong position on Iraq. Support John Murthas position. America needs a leader not another fundraiser-in-chief. Get a grip, Hillary.

5. Leave town, Howard Dean. Youve heard it said, they can take the boy out of the country, but they cant take the country out of the boy. Thats the problem with your new job as DNC chair. It keeps you in Washington when you really should be working full-time with grass-roots activists. The heart of the Party isnt in DC. Its in the rest of the country the sane part. Theres a lot of anger out here in the boonies. It needs to be harnessed. You can provide this leadership. Get out of DC town, Howard.

4. Get an agenda, Nancy Pelosi and Harry Reid. A laundry list of scintillating ideas isnt an agenda its detritus from brainstorming sessions conducted by over-paid Washington consultants. The country needs a few clear notions about who Democrats are. Help us out. What do the Dems stand for besides not being Bushites? Get a real agenda, Nancy and Harry.

3. Convert to Christianity, George Bush. Abandon the fundamentalist pabulum youve been spouting the Weekly Reader summary of the Old Testament and the Book of Revelations. Read the Gospels. Pay attention to the teachings of Jesus: The admonition to tell the truth. Not to murder. To care for the poor. Step into the light, George. Before its too late for you to save your soul.

2. Pay attention, Congress. While youre obsessed with garnering the next big contribution, the country is going down the drain. Theres this guy Osama bin Laden and his band of thugs, Al Qaeda. While youre wasting our money on a pointless war in Iraq, theyre gaining momentum. By the way, the people down in New Orleans who got left behind in the flight from Hurricane Katrina are still left behind. One more thing, theres this country named China. Theyre about to eat our lunch. Do your job, Congress. Pay attention.

1. Wake up, America. Democracy isnt a spectator sport. Eternal vigilance isnt a video game. Everything you really care about is slipping away. Its not just about you. Its about your family and friends everyone you love. Wake up, America, the bad guys are winning."



Comments....

tomder55 answered on 12/24/05:

I fully support 9,8,5,4

Al Gore occasionally comes out to foam at the mouth and howl at the moon.

The Democrats need more Truman Democrats like Lieberman not less.

Hilary is doing classic Clintoon triangulation .But it really doesn't matter unless she is making a Presidential run. Her seat in NY is a lock. Notice her conspicuous absence during the recent transit strike (same with Shumer).When she doesn't need to take a position she will not .When she does she gets her thrills by straddling the fence.

let Bob Burnett worry about his own soul .He is clueless if he thinks that OBL and al-qaeda is "gaining ground'. But he does have a point about China.

America had one of it's largest voter turnouts in 2004 .Guess what Burnett!!!! Bush is still President !!!

Choux rated this answer Excellent or Above Average Answer
ETWolverine rated this answer Excellent or Above Average Answer

Question/Answer
Itsdb asked on 12/22/05 - SDI - STAR DREAMS INITIATIVE

PEACE-ON-EARTH: SEND A LETTER TO THE U.N. GENERAL ASSEMBLY

H.E. Mr. Jan Eliasson
President of United Nations General Assembly
Office of the President of the General Assembly
United Nations, New York, NY, 10017
Tel: (212) 963 7555, fax (212) 963 3301

UNGA WEBPAGE: http://www.un.org/ga/president/60/office/index.html

re: U.N. TO ADOPT A PEACE-ON-EARTH PLATFORM

* DISCLOSURE about the Extraterrestrial presence.

* DECADE OF CONTACT Funding for public education
about Contact.

* DISARMAMENT Banning space-based weapons and
warfare in space.

* DIPLOMACY with ethical Off-Planet Cultures now
visiting Earth.

TO: U.N. SECRETARY GENERAL & PRESIDENT OF THE GENERAL ASSEMBLY OF THE UNITED NATIONS AND ALL U.N. MEMBER NATIONS

WHEREAS, United Nations General Assembly decision 33/426, 1978, approved 18 December 1978, provides: Ŗ. The General Assembly invites interested Member States to take appropriate steps to coordinate on a national level scientific research and investigation into extraterrestrial life, including unidentified flying objects, and to inform the Secretary-General of the observations, research and evaluation of such activities.

WHEREAS, Hon. Paul Hellyer, former Minister of Defence and Deputy Prime Minister of Canada has stated:

From 1963-1967, Hon. Paul Hellyer was Minister of National Defence of Canada in the cabinet of Prime Minister Lester B. Pearson, who won a Nobel Peace Prize. On September 25, 2005, at the Toronto Exopolitics Symposium held at Convocation Hall, University of Toronto, former Minister of National Defence Hellyer stated: "UFOs, are as real as the airplanes that fly over your head," and "I'm so concerned about what the consequences might be of starting an intergalactic war, that I just think I had to say something."

WHEREAS, Mr. Hellyer stated the people of Earth may be threatened with the consequences of war in outer space: "The United States military are preparing weapons which could be used against the aliens, and they could get us into an intergalactic war without us ever having any warning."

WHEREAS, Former Minister of Defence Hellyer went on to state: "The time has come to lift the veil of secrecy, and let the truth emerge, so there can be a real and informed debate, about one of the most important problems facing our planet today."

NOW THEREFORE, WE the undersigned individuals, citizens of U.N. Member Nations, hereby Petition SECRETARY GENERAL & PRESIDENT OF THE GENERAL ASSEMBLY OF THE UNITED NATIONS AND ALL U.N. MEMBER NATIONS to take forthwith all Diplomatic, Executive, Parliamentary & Legislative, Moral, and Public Communications actions necessary to adopt and implement A PEACE-ON-EARTH PLATFORM:

PEACE-ON-EARTH PLATFORM - Ř-Ds": Disclosure - Decade of Contact - Disarmament - Diplomacy

(1) DISCLOSURE - Open, transparent, official governmental disclosure and declassification of all past and present programs related to Extraterrestrial Presence.

(2) DECADE OF CONTACT - Public funding of a 10 year process of formal public education, scientific research, educational curricula development, strategic planning, community activity, and public outreach about the Extraterrestrial Presence and our future in a populated Universe.

(3) DISARMAMENT - A permanent ban on all space-based weapons and warfare through a Space Preservation Treaty.

(4) DIPLOMACY Public Interest Diplomacy with ethical Off-Planet Cultures now visiting Earth.


INFORMATION:
http://www.peaceinspace.net

Click here to send your letter to the Parliament of Canada requesting public "ET" Hearings

Could that be what happened to Iraq's WMD's, the US is deploying them in space to battle the aliens?

tomder55 answered on 12/23/05:

At last, after two thousand years of research, the illudium Q-38 explosive space modulator. At last...


The earth? Oh the earth will be gone in a few seconds...I'm going to blow it up..... It's obstructing my view of Venus. [ Marvin the Martian]

ETWolverine rated this answer Excellent or Above Average Answer
Itsdb rated this answer Excellent or Above Average Answer

Question/Answer
sapphire630 asked on 12/21/05 - we need a good pyschic ASAP

http://www.predictions.tv/
with George Noory
premieres Jan 12, 2006 9 pm est
predict what the Dow will be at the close of Jan 17, 2006 and you could win 5,000.

tomder55 answered on 12/22/05:

I call him George Snory .Whenever I get a bout of insomnia I turn him on . In 2 minutes it's lights out!

sapphire630 rated this answer Excellent or Above Average Answer

Question/Answer
ETWolverine asked on 12/22/05 - Munich Buzz

I'm hearing a lot about the new Spielberg movie Munich. Some are saying it is a fabulous movie with great and complex characters that doubt themselves and their mission.

Others are saying that in reality the Israeli assasins never had any such ambiguous feelings, and that Speilberg is merely putting his own political views about the Israeli Palestinian conflict into the film. As such, they argue that the film portrays Israel as the bad-guy for going after the Munich terrorists when even their own people felt that it was wrong, and when it led to an escalation of the violence. This point of view would say that the film is anti-Israel.

Yet a third group argues that the film is awful because it portrays Arab characters as one-dimentional, whereas the Israeli characters are fleshed out. As such, it is biased in favor of Israel and against the Arabs.

What have you heard about the film? Do you have an oppinion? If not an opinion of the movie itself, do you have an opinion about the buzz the movie is generating?

I'm not quite sure which way to lean on this one yet. Jury is still out for me. I will say that I know one of the people that Spielberg consulted with regarding the film, a former Israeli CT expert named Jubal Aviv who was a member of one of the assasination teams. He now runs a global security firm called Interfor, based in New York. I believe that he gave Spielberg the information as it happened. The question is whether Speilberg is portraying what Aviv told him as he told it to him, or whether he decided to imbellish it with his own political statement. I don't know yet.

Elliot

tomder55 answered on 12/22/05:

I will have to reserve opinion until I see the flix. The sites I have read seem to suggest that Spielbeg makes the mistake of making a moral equivalence of the hit team with the PLO terrorists.

The Guardian said that he claimed that the biggest threat to the Middle East was neither the Palestinians nor the Israelis but intransigence on both sides. That of course is nonsense.

Yale Kramer at The American Spectator suggests the flaw in the film (if there is one) is that :Between the two stories -- the massacre story and the revenge story -- there is little to choose. Both are gripping, human, dramatic, full of twists, suspense, and irony. But if you emphasize the massacre story, the sympathy is bound to be for the Israeli athletes and their wives and families. If you emphasize the revenge story the sympathy could easily be with the Arab victims and their families.(Kramer also gives a good summary of the facts which is worth reading).

I thought he did a decent job portraying the holocost in 'Schindler's list'.I am suprised (if what I read is true )that he would portray people with simular aims as the Nazis in a more sympathetic light. It may well be that he is wrestling with the idea of peace above all else. If that is the case, then the movie is appropriatly named.

ETWolverine rated this answer Excellent or Above Average Answer

Question/Answer
purplewings asked on 12/20/05 - Sorry about the pics not showing up.

Maybe the site owners decided not to accept any more pics from me because of all the ones I posted on the forum last month....

I apologize for wasting your time looking at a red x.

tomder55 answered on 12/20/05:

I am not supposed to say anything about her .As this photo clearly shows ;she is still in deep despair .

ETWolverine rated this answer Excellent or Above Average Answer
purplewings rated this answer Excellent or Above Average Answer

Question/Answer
nyyfan81481 asked on 12/15/05 - constitution

the governor of illinois may veto individual items in appropriation bills. true of false???

tomder55 answered on 12/16/05:

Although he has the authority I found no recent headlines indicating that Govenor Rod Blagojevich will use the line item veto.

Question/Answer
rivers asked on 12/16/05 - history and current events

I barely passed history class so if you could answer a few of my questions maybe it would help me understand what is going on right now.

1) Where did all the Jewish people of the holocaust come from?
2) After the holocaust, whose idea was it to make a Jewish state in Israel out there in the middle of hostile territories, or is that where all the jewish people came from to begin with?
3) Was it a religious or political decision?
4) Do you have a problem with President Ahmadinejad's suggestion that the Jewish people of Israel be moved to Europe or another area where those responsible for the crimes of the holocaust should pay the price by giving them their own land?
5) Is there any acceptable place in Europe that the Jewish people could have their own state and peaceful existance?
6) Who is responsible for the holocaust?
7) How many Jewish people were murdered?
8) Why don't other ethnic groups who have suffered genocide get their own states?

tomder55 answered on 12/16/05:

1. they were primarily Jews from conquered Europe.

2. Zionism was a movement long before WWII .But in fact it has been an ancient desire of the Jewish people to return to their homeland and live in peace in a modern Jewish nation. Their historic homeland is Israel.

"a people which weeps and mourns for the loss of its homeland 1800 year ago and does not forget - such a people will never be destroyed. Such a people can rest assured that its homeland will be returned to it." Napolean Bonaparte

American Presidents as early as John Quincy Adams expressed support for a Jewish State .

After WWI Arthur J. Balfour, British Foreign Secretary in a letter to Lord Lionel Rothschild which became known as the Balfour Declaration wrote . "His Majesty's government views with favour the establishment in Palestine of a national home for the Jewish people." That was when the principle of the State of Israel became policy . President Woodrow Wilson fully supported the concept.

3. both

4. Ahmadinejad is a lunatic .Yes I have problems with his lies. The truth is that the Jews were driven from their homeland and now have returned . He cannot change that fact or the history .

5. no ;there is no historical justification to establish a Jewish State in Europe.

6.Adolph Hitler ;it was his policy .

7.estimates are around 6 million.

8. There may or may not be justification for that . Many ethnic groups never established States but were ethnic minorities in other states. I would certainly support states for Kurds ;Armenians and ethnic minorities now being slaughtered in Darfur among others . The pertinent point about Israel is that it has already been established and is a functioning state for over 50 years. The bottom line reality in the world however is that you're right to land is contingent on you're ability to defend it .Always has been ;always will be. That applies to individuals and nations equally .


ETWolverine rated this answer Excellent or Above Average Answer
rivers rated this answer Excellent or Above Average Answer

Question/Answer
ETWolverine asked on 12/16/05 - Bushs approval ratings trends since 9/30/05.

Here is some information from major pollsters regarding Bush's approval ratings. I'm trying to track the trends in his popularity. I'm not sure if this will come out properly... I have it set up in a grid-format, but I don't know if it will take on Answerway's system.

Fox News/Opinion Dynamics.

Approve Disapprove Unsure
% % %
12/13-14/05 42 51 7
11/29-30/05 42 48 10
11/8-9/05 36 53 11
10/25-26/05 41 51 8
10/11-12/05 40 51 9


Diageo/Hotline Poll

Approve Disapprove Unsure
% % %
12/12-13/05 50 47 3
11/11-15/05 39 59 2
10/12-16/05 40 57 2


NBC News/Wall Street Journal Poll

Approve Disapprove Unsure
% % %
12/9-12/05 39 55 6
11/4-7/05 38 57 5
10/8-10/05 39 54 7


Gallup Poll and CNN/USA Today/Gallup Poll

Approve Disapprove Unsure
% % %
12/9-11/05 42 55 3
12/5-8/05 43 52 5
11/17-20/05 38 57 5
11/11-13/05 37 60 3
11/7-10/05 40 55 5
10/28-30/05 41 56 3
10/24-26/05 41 56 3
10/21-23/05 42 55 3
10/13-16/05 39 58 3


Cook Political Report/RT Strategies Poll

Approve Disapprove Neither/
Mixed Unsure
% % % %
12/8-11/05 42 55 1 1
11/17-20/05 41 52 5 2



Pew Research Center for the People & the Press survey

Approve Disapprove Unsure
% % %
12/7-11/05 38 54 8
11/3-6/05 36 55 9
10/12-24/05 40 52 8
10/6-10/05 38 56 6


Zogby America Poll

Excellent/Good Fair/Poor
% %
12/6-8/05 38 62
10/29 - 11/2/05 39 61
10/19-21/05 45 55


Associated Press-Ipsos poll

Approve Disapprove Mixed Unsure
% % % %
12/5-7/05 42 57 1 1
11/7-9/05 37 61 1 1
10/31 - 11/2/05 37 59 3 1
10/3-5/05 39 58 3 -


CBS News/New York Times Poll

Approve Disapprove Unsure
12/2 - 6/05 40 53 7
10/30 - 11/1/05 35 57 8
10/3-5/05 37 58 5


The basic overall trend that I am seeing from these polls is that Bush hit his popularity low point in late October and early November. However, over the past couple of weeks his numbers have been creeping back up. The highest poll number shown is from Diageo/Hotline, which currently has Bush at 50%... an 11 point jump in a one month period. This is the ONLY poll to show such a huge jump, and so I would take it with a grain of salt. The Cook/RT poll has only been in existence long enough for 2 polls to take place, so no trend can really be gleaned from them. Furthermore, until they have a proven track record, I would look askance at anything they say. Only Zogby has Bush still trending downward. All other polls show Bushs numbers going up over the past month or so. Most of those polls put his approval rating at 42%, and so I think that is a reasonable assumption.

Interesting information. Bush is not out of the game yet

Elliot

tomder55 answered on 12/16/05:

of course they are . I do not consider polls all that important and it doesn't appear that Bush does either (although for the life of me I cannot figure why he caved in on the McCaine Amendment ).Harry Truman's poll #s were so low that they were a factor in his decision not to run again (a 24 percent low and like Bush ,Truman won reelection despite having a Gallup poll approval rating below 50 percent just prior to the election.) He is reveared today as a man with firm convictions who did what he thought best for the country and damn the politics .Truman's reputation among historians is consistently high.

VDH has a great editorial in yesterday's papers . He points out how often American war Presidents have been trashed in the heat of the war and how the critiques is generally forgotten when the smoke clears .

I love this line :

But as the conditions in Iraq improve, and comparisons to our sole loss in Vietnam ring hollow, expect critics to grow silent. And savvy fence-sitters like Hillary Clinton will begin to preen, rather than express ambivalence, over past votes to remove Saddam.
The blame game is not unusual on the impatient home front during American wars -- and is soon mostly forgotten after we finally win. Iraq is, and will be, no exception.



ETWolverine rated this answer Excellent or Above Average Answer

Question/Answer
Choux asked on 12/16/05 - 700 Mile Long Fence

From Yahoo News:::

The House called for construction of a fence along parts of the U.S. border from the Pacific Coast to the Gulf of Mexico as a bill aimed at shutting down illegal immigration moved forward Thursday.

The two-layered fence, about 700 miles long, would be built in parts of California, Arizona, New Mexico and Texas. The provisions, passed 260-159, put priority on construction near Laredo, Texas. The city is across the border from Nuevo Laredo, Mexico, where warring drug cartels have been blamed for more than 140 murders this year.

Supporters said the fence would cut down on crime and drug smuggling, but Rep. Sheila Jackson Lee (news, bio, voting record), D-Texas, said it would create "the largest gated community in the Western hemisphere."

The House voted 220-206 to approve a parliamentary measure needed to move ahead on the bill, but only after GOP leaders appealed in a private meeting for party unity. Some members were threatening to vote against the bill if it did not include a guest worker program, while others opposed adding such a program.

Late Thursday, Republicans were still gauging support among their ranks for the bill.

A few were unhappy that a provision denying citizenship to children born to illegal immigrants in the United States was not among the first 15 amendments getting a vote. The contentious measure could be offered Friday.

"Those people who are against this bill don't want any changes in the existing system except perhaps amnesty, or, excuse me, `earned legalization,' or perhaps citizenship for those who have broken the law," said Rep. James Sensenbrenner, a chief sponsor and chairman of the
House Judiciary Committee.

Earlier on a voice vote, House members approved an amendment requiring Border Patrol uniforms to be made in the United States. Rep. Rick Renzi (news, bio, voting record), R-Ariz., said they are now made in Mexico, possibly posing security problems.

Supporters of the overall bill defended their decision to cut off the flow of illegal entrants before turning to the tougher issues of a guest worker program or other means to fill the jobs that now attract millions of undocumented workers.

Almost all Democrats, and several border-state Republicans such as Rep. Jeff Flake (news, bio, voting record) and fellow Arizonan Jim Kolbe, pushed for a more comprehensive measure that would deal with the estimated 11 million illegal immigrants already in the United States.

The GOP bill "does nothing to solve the real problems of illegal immigration," Kolbe said. "In fact, it's worse than nothing."

The White House said in a statement that it strongly supported the House bill, while adding that it "remains committed to comprehensive immigration reform, including a temporary worker program that avoids amnesty."

Senate Majority Leader Bill Frist, R-Tenn., says the guest worker issue will be on the table when the Senate takes up immigration overhaul in February. The main dispute is over whether the estimated 6 million illegal workers should have to leave the country before applying for a temporary worker program.

The border security aspects, King said, include requiring the
Homeland Security Department to employ the personnel and technology needed to secure the border; ending the "catch-and-release" policy for non-Mexicans; and requiring the
Pentagon and Homeland Security to come up with a common plan on the use of military technology to stop illegal crossings.

The bill also outlines increased penalties for smugglers and those re-entering illegally; authorizes police along the border to enforce immigration law; and makes illegal presence in the United States, now a civil offense, a misdemeanor crime.

The bill originally made illegal presence a felon. Sensenbrenner's spokesman, Jeff Lungren, said that was being changed because felonies require jury trials and consume too many resources.

Most significantly, the bill requires all employers in the country, more than 7 million, to check the legal status of workers.

___

Associated Press writer Jim Abrams contributed to this report.

___

bill, H.R. 4437, can be found at http://thomas.loc.gov/



Do you agree with the provisions of this bill??
Why???
Shy not???

tomder55 answered on 12/16/05:

of course I am in favor of it . It doesn't matter that a comprehensive plan has not been agreed to yet ;nor should anyone conclude that building the fence is the only measure that is needed to be taken. But;this is an important step to control the flow of illegals into the country and those who opposed it may have given any number of rationals for their nay vote;but in truth they are not really interested in border security and that is how they should be viewed.

Choux rated this answer Excellent or Above Average Answer
Itsdb rated this answer Excellent or Above Average Answer

Question/Answer
Choux asked on 12/16/05 - Eavesdropping

WASHINGTON, Dec. 15 - Months after the Sept. 11 attacks, President Bush secretly authorized the National Security Agency to eavesdrop on Americans and others inside the United States to search for evidence of terrorist activity without the court-approved warrants ordinarily required for domestic spying, according to government officials.

Under a presidential order signed in 2002, the intelligence agency has monitored the international telephone calls and international e-mail messages of hundreds, perhaps thousands, of people inside the United States without warrants over the past three years in an effort to track possible "dirty numbers" linked to Al Qaeda, the officials said. The agency, they said, still seeks warrants to monitor entirely domestic communications."


Do you think that this is OK????

tomder55 answered on 12/16/05:

In one sentence it says they were monitoring without warrants ,and in the next sentence it says they continue to seek warrants . Which is it ? The answer : they are seeking warrants to expand the monitoring of DOMESTIC Calls. The administration had briefed Congressional leaders about the program and notified the judge in charge of the Foreign Intelligence Surveillance Court, the Washington court that deals with national security issues so it isn't true that the program was a secret one ;both Congress and the Judiciary were informed.


The eavesdropping program helped uncover a plot by Iyman Faris, an Ohio trucker and naturalized citizen who pleaded guilty in 2003 to supporting Al Qaeda by planning to bring down the Brooklyn Bridge with blowtorches. What appeared to be another Qaeda plot, involving fertilizer bomb attacks on British pubs and train stations, was exposed last year in part through the program.
The eavesdropping program grew out of concerns after the Sept. 11 attacks that the nation's intelligence agencies were not poised to deal effectively with the new threat of Al Qaeda and that they were handcuffed by legal and bureaucratic restrictions better suited to peacetime than war. In response, President Bush eased limits on American intelligence and law enforcement agencies and the military.
Bush's executive order is based on legal opinions that assert that the president has broad powers to order such searches, derived in part from the September 2001 Congressional resolution authorizing him to wage war on Al Qaeda and other terrorist groups.
Until the new program began, the N.S.A. typically limited its domestic surveillance to foreign embassies and missions in Washington, New York and other cities, and obtained court orders to do so.Warrants are still required for eavesdropping on entirely domestic-to-domestic communications.


I think it is telling that the NY Slimes ;the paper that broke the story ; also wrote how the NSA came under criticism by the 9-11 Commission for being risk-adverse and for "adhering to self-imposed rules that were stricter than those set by federal law."

Damned if they do ;Damned if they don't . Intelligence agencies have come under fire for not being able to prevent 9-11. But at the same time, an intelligence agency takes action, to protect U.S. citizens from terrorism and it is painted as a violator of civil liberties. Someone better show me where the constitution was violated in this case .I don't see it. The President was authorized by Congress with broad powers to wage war against al-Qaeda.Other Presidents given simular powers have done much worse and they are considered national heros.

Choux rated this answer Excellent or Above Average Answer

Question/Answer
HANK1 asked on 12/14/05 - GOOD LUCK, IRAQ!



Thanks to God, our United States Constitution will continue to allow us freedom and justice. Transversely, any new and bonafide Societal Constitution will allow you to live MORALLY and ETHICALLY in your NEIGHBORHOODS without any necessity to amend any FOUNDATION based on happiness for all. Perhaps political entrapment will be non-existent. Your new CHARTER OF DECENCY will see to it. Let this goodness help you abide by all laws that need to be obeyed. The BY-LAWS of your new REPUBLIC will be secondary in importance only when intervention deals with fundamental DEMOCRATIC law. A new SOCIETAL CONSTITUTION will afford you God-given rights to monitor your own progression in your NEIGHBORHOODS, TOWNS and CITIES. This new manner of community indulgence will help you gratify willingness so you can SURVIVE morally and ethically.

"Men by their constitutions are naturally divided into two p[artries: (1) Those who fear and distrust the people, and wish to draw all powers from them into the hands of the higher classes. (2) Those who identify themselves with the people, have confidence in them, cherish and consider them as the most honest and safe, although not the mose wise depository of the public interests. In every country these two parties exist; and in every one where they are free to think, speak and qwrite, they will declare themselves ." - Thomas Jefferson (in a letter to Henry Lee dated August 10, 1824)

Let us and all people enlighten those obedient and brave souls who go to the polls tomorrow. Let the tyranny and oppressions of body and mind vanish like evil spirits at the dawn of day. Let these forces of duty not only effect obligations and faithfulness to protect the affections and personalities of the burdensome, but effectuate the accomplishments as well. Allow this power of vigor to intensify action that must exist within your minds as a life-giving force. Without it, you're meek and weak. ENJOY YOUR FREEDOM! I'm sure ALLAH wants you to ... forever!

HANK


tomder55 answered on 12/15/05:

Iraqi Blogger Iraq The Model has some initial reactions and they will updatre throughout the day :


We got our purple fingers!


We have noticed on our way to the polling station that the streets are much quieter this time than they were back in January.
Of course we did hear a few explosions, probably RPG or mortar fire but nothing compared to the attacks we had last time in number or size.so far at least.

The deployment of Iraqi security forces on the streets was heavy with a noticeable absence of American forces except for their presence in the skies; there are many Apache helicopters and jet fighters as well as small surveillance planes al over Baghdad.

The number of voting officials, independent observers and political bodies representatives is higher than what we saw in January.
From what we saw, people feel safer walking to the voting centers this time; many of them were carrying Iraqi flags.



You should read it all and see some of the pictures that they have posted . I enjoyed reading this report from Baghdad :

Last night was really tough and long because we spent it under the pressure of the Arabic MSM terrifying the people and spreading Zarqawi threats that it would be a bloody day. On the other hand a rumor spread out in the middle of the night telling that the water is poisoned, well I guess the "antihuman" wanted to poison our election day.

Early in the morning the Iraqis flocked toward the polling centers not caring about some mortars that went down here and there, the kids kept on playing soccer in the empty streets, the mosques-Sunni and Shii-were calling and urging people to vote.

There was a little difference in the two calls; while the Sunni were calling the people to vote for the sake of Arab Nationalism the Shia mosques were calling to vote under the Fatwa of Sistani whom being fiercely attacked by al-Jazeera news channel and this made his Fatwa gain more voters.

Al-Haj Abo Mohammed al-Furaiji took all his family members that were eligible to vote and soon they all walked out from the polling center smiling and saying we chose what we wanted.

Shandookh Alwan Ibrahim, a disabled man on wheelchair and a father of three young men who were executed by Saddam said:Our future is in our hands, today is the celebration day for the poor and I'm one of them and we need someone in charge who can shoulder the responsibility



Muhaisin Bidairy Abdullah who was born in 1900 and I think he is the oldest amongst the voters came leaning on his grandsons and could hardly breathe with tears visible in his eyesmaybe because he wont be able to attend the next elections.

Asla Hussein a 70 year old woman, who has 5 members of her family executed by Saddam, said we want the remaining members of our family to live in peace.

The encouraging thing I noticed was that security forces treated the voters well and didnt interfere with the voting process.





HANK1 rated this answer Excellent or Above Average Answer

Question/Answer
kindj asked on 12/14/05 - Proof that fame does NOT equal intelligence

>>Tuesday, Dec. 13, 2005 11:45 p.m. EST
Jane Fonda: U.S. Troops Are 'Killing Machines'


"Hanoi Jane" Fonda is claiming that ever since Vietnam, U.S. troops have been trained to commit atrocities against innocent civilians as a matter of military policy.

"Starting with the Vietnam War we began training soldiers differently," the anti-American actress says in an email to the Washington Post.

Fonda claims she learned of the policy switch in "secret meetings" she had with military psychologists "who were really worried about what was happening to our combat personnel."

One doctor, she insists, told her U.S. troops had been deliberately trained to be "killing machines."

"This began," Fonda maintained, "because the military discovered that in World War II and Korea, [U.S.] soldiers weren't killing enough."
"So they changed training procedures" to teach troops how to commit atrocities.

Still, the anti-war gadfly cautions, it's important not to blame the soldiers themselves for carrying out war crimes.

Recalling the "Winter Soldier" hearings that she and John Kerry staged in 1971, Fonda lamented: "When you put young people into an atrocity-producing situation where enemy and civilian are commingled, where the 'other side' is dehumanized, we cannot be surprised."

Anti-war vets now returning from Iraq, Fonda cautioned, should be listened to instead of being dismissed as "unpatriotic."

"We have not learned the lessons of Vietnam," she declared.<<


I gotta say, it takes a fair bit to offend me, but she's managed to pull it off.

I guess the first question is where did she get this "information?" How many boot camps did she attend? How many FIELDEX's? How many service schools?

In all of my military training and schools (and there's been a sh!tload), NEVER had we been taught/trained/encouraged to commit atrocities on ENEMY SOLDIERS, much less civilians! Furthermore, most of my instructors were from the Vietnam era (with the occasional Korean War vet), and none of THEM had anything to do with such things!

Quite the opposite, in fact. We were trained on how to AVOID civilian casualties at almost all costs, and trained on handling dead and captured enemy combatants, and "atrocities" were NEVER part of the curriculum. Rather, RESPECT (while maintaining strength) was the watchword. In many cases, this training and the subsequent obeying of procedures related to handling of enemy combatants and civilians almost got a few of us killed!

Why won't this lady just shut up and talk about something she may actually KNOW something about, like how to marry and divorce a egomaniac millionaire or how to make bad B movies...

DK

tomder55 answered on 12/15/05:

If Barbarella says it is so then it must be .Through no fault of her own there is a kernel of truth in what she says . Training has changed since WWII. Reports from then suggested that many of the infantry men never fired their weapons in combat . The efficiency also increased when the Army went professional . In the minds of anyone but a moonbat that is a good thing. This disgraceful person truely believes that the purpose of an army is to commit atrocities .

kindj rated this answer Excellent or Above Average Answer

Question/Answer
excon asked on 12/14/05 - Medicare


Hello experts:

If you're on medicare and your hand doesn't work too well, your drugs are covered. However, if your penis doesn't work too well, those drugs are not.

What is the difference between your hand and your penis?

excon

tomder55 answered on 12/14/05:

there's a joke in there somewhere.

the last I heard Drugs for sexual performance were to be covered under the new plan . But of course you knew that Pfizer's interests would supercede the taxpayers .

For my 2 cents the doctor should determine if erectile failure is the result of a condition besides the normal aging process.If it is a matter of a healthy man just trying to get a couple more years out of it then that becomes recreational rather than a medical necessity .

Sorry ...choices like this will become more prevelent...not less as medical care becomes socialized .It's the price you have to pay to live in a nanny state.

excon rated this answer Excellent or Above Average Answer

Question/Answer
ETWolverine asked on 12/13/05 - New ABC/Times poll. Released 12/12/05

ABC and Time Magazine conducted a poll in Iraq. The results are rather surprising at least to some.

Here are some of the highlights:

1) 71% of Iraqis feel that things are going well in their personal lives. On the downside, only 44% feel that things are going well for the country overall. However, 69% of Iraqis expect things for the country overall to improve in the next year. (Among Sunnis, that number is only 35%.)

2) Average salaries are $263 per month, up from an average of $164 per month in February 2004.

3) 57% of Iraqis support the idea of a Democracy as their form of government. In mixed population areas 75% support Democracy. In Kurdish areas 63% support Democracy. In Shiite areas, support for Democracy is 45%. In Sunni areas, only 38% support Democracy.

Interestingly, 64% support Democracy in Iraq for 5 years from now including 55% of Sunnis.

4) Here is a breakdown of Shiite vs. Sunni opinions:

Own life good: Shiite: 86%, Sunni 43%
Things going well in Iraq: Shiite 53%, Sunni 9%
The USA was right to invade: Shiite 59%, Sunni 7%
Feel very safe: Shiite: 80%, Sunni 11%
Approve of the new Constitution: Shiite 82%, Sunni 27%
Confidence in the Iraqi army: Shiite 87%, Sunni 37%.

The contrast between the two groups is pretty stark, and it is clear that the Sunnis, who lost the most in the post-Saddam environment, are dragging down the poll numbers from other groups.

5) Local conditions in Iraq - % saying good. (Numbers in parenthesis are from a February 2004 poll)
Schools: 74% (72%)
Crime protection: 66% (53%)
Medical Care: 62% (51%)
Security: 61% (49%)
Clean water: 58% (50%)
Electricity: 45% (35%)
Jobs: 38% (26%)

6) Womens rights: % in favor.
Voting: 99%
As doctors: 99%
Driving: 84%
In national assembly: 80% (among Sunnis: 62%)
As governors: 51% (among Sunnis: 22%)
As president: 46% (among Sunnis: 21%)
As mukhtar (religious leader): 38% (among Sunnis: 6%)

7) Roughly half of Iraqis feel that US troops should stay in Iraq for some time. 31% say that they should remain until security is restored, 16% say they should stay until the Iraqi security forces can operate independently, and 5% say that they should stay longer. Only 26% say that US forces should pull out immediately, and another 19% say that they should pull out after the new government being elected this week takes office.

The full results of the poll can be found at http://abcnews.go.com/images/Politics/1000a1IraqWhereThingsStand.pdf

The facts on the ground seem to be somewhat different than what John Murtha, Ted Kennedy, Joe Biden and others on the left would have us believe. Contrary to what Murtha has claimed, 80% of Iraqis do not want the USA to pull out immediately. Contrary to what Kennedy and others have said, the Iraqis see improvement in their situations, and expect further improvements in the coming year. The majority see the advantages that Democracy has to offer and want a piece of that pie. Only a quarter of the population (mostly Sunnis) wants an immediate pullout of US troops.

Dont believe the rhetoric from the left. There is no quagmire, there has been significant progress, and there is optimism about the future in Iraq. Overall, an exceptional set of poll results. And especially exceptional considering who put the poll out in the first place. ABC and Time Magazine are not exactly known for their strong pro-war stances.

Elliot

tomder55 answered on 12/14/05:

The MSM is starting to hedge their bets . David Ignatius in the Washington Compost wrote this little bit of praise in his hit piece :

The shame for America isn't that we have tried to topple the rule of the assassins but that we have so far been unsuccessful. ... it's still there, in the shadows of the shadows. George W. Bush gets a lot of things wrong, but he knows that he's fighting the assassins. On days like these, I'm glad that he is such a stubborn man. ... Amid the Bush administration's mistakes and lies about Iraq over the past three years, it's easy to lose sight of what is at stake in this battle. But this week brings it back to square one: It's about breaking the power of the assassins. ... People like the Tuenis who refuse to be intimidated should inspire the rest of us. So should the millions of Iraqis who will vote tomorrow. They are trying to break the culture of intimidation and death. Americans should feel proud to be on their side.
................................
I'm going to try to get some purple ink for my finger for tomorrow. I think it would be a powerful signal if alot of Americans showed their solidarity with the Iraqi's by dipping their index finger in purple ink .

ETWolverine rated this answer Excellent or Above Average Answer

Question/Answer
ETWolverine asked on 12/13/05 - New ABC/Times poll. Released 12/12/05

ABC and Time Magazine conducted a poll in Iraq. The results are rather surprising at least to some.

Here are some of the highlights:

1) 71% of Iraqis feel that things are going well in their personal lives. On the downside, only 44% feel that things are going well for the country overall. However, 69% of Iraqis expect things for the country overall to improve in the next year. (Among Sunnis, that number is only 35%.)

2) Average salaries are $263 per month, up from an average of $164 per month in February 2004.

3) 57% of Iraqis support the idea of a Democracy as their form of government. In mixed population areas 75% support Democracy. In Kurdish areas 63% support Democracy. In Shiite areas, support for Democracy is 45%. In Sunni areas, only 38% support Democracy.

Interestingly, 64% support Democracy in Iraq for 5 years from now including 55% of Sunnis.

4) Here is a breakdown of Shiite vs. Sunni opinions:

Own life good: Shiite: 86%, Sunni 43%
Things going well in Iraq: Shiite 53%, Sunni 9%
The USA was right to invade: Shiite 59%, Sunni 7%
Feel very safe: Shiite: 80%, Sunni 11%
Approve of the new Constitution: Shiite 82%, Sunni 27%
Confidence in the Iraqi army: Shiite 87%, Sunni 37%.

The contrast between the two groups is pretty stark, and it is clear that the Sunnis, who lost the most in the post-Saddam environment, are dragging down the poll numbers from other groups.

5) Local conditions in Iraq - % saying good. (Numbers in parenthesis are from a February 2004 poll)
Schools: 74% (72%)
Crime protection: 66% (53%)
Medical Care: 62% (51%)
Security: 61% (49%)
Clean water: 58% (50%)
Electricity: 45% (35%)
Jobs: 38% (26%)

6) Womens rights: % in favor.
Voting: 99%
As doctors: 99%
Driving: 84%
In national assembly: 80% (among Sunnis: 62%)
As governors: 51% (among Sunnis: 22%)
As president: 46% (among Sunnis: 21%)
As mukhtar (religious leader): 38% (among Sunnis: 6%)

7) Roughly half of Iraqis feel that US troops should stay in Iraq for some time. 31% say that they should remain until security is restored, 16% say they should stay until the Iraqi security forces can operate independently, and 5% say that they should stay longer. Only 26% say that US forces should pull out immediately, and another 19% say that they should pull out after the new government being elected this week takes office.

The full results of the poll can be found at http://abcnews.go.com/images/Politics/1000a1IraqWhereThingsStand.pdf

The facts on the ground seem to be somewhat different than what John Murtha, Ted Kennedy, Joe Biden and others on the left would have us believe. Contrary to what Murtha has claimed, 80% of Iraqis do not want the USA to pull out immediately. Contrary to what Kennedy and others have said, the Iraqis see improvement in their situations, and expect further improvements in the coming year. The majority see the advantages that Democracy has to offer and want a piece of that pie. Only a quarter of the population (mostly Sunnis) wants an immediate pullout of US troops.

Dont believe the rhetoric from the left. There is no quagmire, there has been significant progress, and there is optimism about the future in Iraq. Overall, an exceptional set of poll results. And especially exceptional considering who put the poll out in the first place. ABC and Time Magazine are not exactly known for their strong pro-war stances.

Elliot

tomder55 answered on 12/13/05:

I think it was mentioned briefly on the ABC Sunday morning 'This Week' show.

Turns out that Murtha's biggest gripe is that W didn't invite him to dinner to schmooze about the war like his daddy did .

To quote Newsweek :"None. None. Zero. Not one call," a baffled Murtha told NEWSWEEK. "I don't know who the hell they're talking to. If they talked to people, they wouldn't get these outbursts. If they'd talked to me, it wouldn't have happened."

Ok so there you have it . The real reason Murtha has become the latest poster-person for the anti-war crowd


Now there's a man of conviction for you .

ETWolverine rated this answer Excellent or Above Average Answer

Question/Answer
arcura asked on 12/10/05 - Is the Bush administration dismantling the VA?

Please take a thoughtful look at this and answer the question form your experience.
Thanks
http://www.military.com/opinion/0,,77901,00.html

tomder55 answered on 12/11/05:

I do not see streamlining as a bad thing so long as Veterans do not lose benefits .The truth is that Bush signed a bill last week that increased health care budget for Vets.The funding bill included up to $22.5 billion for veterans medical services in the current fiscal year. Thats a 16 percent increase from the $19.3 billion in the last fiscal year.Now I do not know if that funding is sufficient but it seems that only in Washington can a 16% increase be called a cut.

this chart shows that Bush has increased the budget for Vets every year in office. If you were to argue that it is not enough ,then you are probably correct but I take Chuck's position that waste in the bugetary process is the biggest culprit.

arcura rated this answer Excellent or Above Average Answer

Question/Answer
kindj asked on 12/09/05 - From an "eco-traitor"

Former Greenpeace Co-Founder Praises US for Rejecting Kyoto
Marc Morano
Senior Staff Writer

Montreal (CNSNews.com) - A founding member of Greenpeace, who left the organization because he viewed it as too radical, praised the United States for refusing to ratify the Kyoto Protocol.

"At least the [United] States is honest. [The U.S.] said, 'No we are not going to sign that thing (Kyoto) because we can't do that,'" said Patrick Moore, who is attending the United Nations Climate Change Conference in Montreal.

Moore noted that many of the industrialized nations that ratified the treaty limiting greenhouse gas emissions are now failing to comply with those emission limits. Moore, who currently heads the Canadian-based environmental advocacy group Greenspirit Strategies helped found both Greenpeace in 1971 and Greenpeace International in 1979.



"Canada signed [Kyoto] and said, 'Oh yeah, we can do that,' and then it merrily goes on its way to increase CO2 (carbon dioxide) emissions by even more than the U.S.," Moore told Cybercast News Service.

Other industrialized nations -- including Japan and at least 11 of the 15 European Union nations that ratified Kyoto -- are struggling to meet their emission targets.

As Cybercast News Service previously reported, many organizations attending the Climate Change Conference have declared the Kyoto Protocol "dead" because of the signatories' lack of compliance. The treaty establishes a 2012 goal of having top industrialized nations cut their industrial emissions 5.2 percent below the level that was produced in 1990.

"I think this whole Kyoto process is a colossal waste of time and money," said Moore, who rejects alarmist predictions of human-caused 'global warming."

The U.N.'s 11th Annual Climate Change Conference in Montreal failed to impress Moore, who is there to promote nuclear energy.

"There is nothing concrete going on here. There is nothing good happening here as far as I can see. [The participants at the U.N. conference are] just spending a whole pile of money and auguring and talking," he added.

Moore also slammed the movement he helped found, accusing today's environmental groups of being co-opted by the political Left.

"The Left figures it owns the environmental movement and that has corrupted the movement greatly," Moore said. "The [left-wing] influence has brought great dysfunction into the environmental movement. [It's turned it into] an elitist movement."

Moore said he decided to leave Greenpeace in 1986 after the group became too radical and he could "no longer agree with the policies that were being espoused."

The final straw, according to Moore, came when he failed to persuade Greenpeace to abandon its campaign to ban chlorine worldwide.

"I pointed out that chlorine was the main element used in our medicine and adding it to drinking water was the biggest advance in public health in human history," Moore said. "[My argument] just fell on deaf ears. [Greenpeace] didn't care about any of that because a global chlorine ban was a good campaign [for them]."

Even though he was a pioneer of the movement, liberal environmentalists spare no criticism of Moore, frequently referring to him as a "traitor" and an "Eco-Judas."

Moore dismissed the criticism and asserted that the green movement has steered off course from its original mission.

"I think it's in a dismal state -- I think almost across the board, whether it's in energy policy or agriculture policy regarding their zero tolerance on GM [genetically modified foods] or in forestry policy," Moore said.

tomder55 answered on 12/09/05:

that is the point . the treaty is all show . none of the countries will attempt to seriously comply . The US is actually doing a good job of meeting the artifical targets .It is refreshing to hear from an environmentalist who is objective. His last statement is great . GMO with proper controls will feed the world.

kindj rated this answer Excellent or Above Average Answer

Question/Answer
kindj asked on 12/09/05 - Category 5 blizzared

(For those of you who are not aware, North Dakota and southeastern Montana got hit with their first blizzard of the season a couple of weeks ago)

This text is from county emergency manager out in the western part of North Dakota after the storm.

Amusing...


WEATHER BULLETIN


Up here in the Northern Plains we just recovered from a Historic event --- may I even say a "Weather Event" of "Biblical Proportions" --- with a historic blizzard of up to 24" inches of snow and winds to 50 MPH that broke trees in half, stranded hundreds of motorists in lethal snow banks, closed all roads, isolated scores of communities and cut power to 10's of thousands.

George Bush did not come....
FEMA staged nothing....
No one howled for the government...
No one even uttered an expletive on TV...
Nobody demanded $2,000 debit cards.....
No one asked for a FEMA Trailer House....
No news anchors moved in.

No looting
No killing
Jessie did not show up
We just melted snow for water, sent out caravans to pluck people out of snow engulfed cars, fired up wood stoves, broke out coal oil lanterns or Aladdin lamps and put on an extra layer of clothes.


Even though a Category " blizzard of this scale has never fallen this early...we know it can happen and how to deal with it ourselves.


Everybody is fine.

tomder55 answered on 12/09/05:

We got about 6" of snow this morning . I was just about to go out and see if I could muster up one of those 'frozen'MRE's that FEMA is handing out.

kindj rated this answer Excellent or Above Average Answer

Question/Answer
kindj asked on 12/08/05 - Ops in Iran

I can't seem to post the follow up. Go figure. Anyway, I was smart enough to save my text, so here it is:

Easy? "Easy" is a relative term, as we all know. After all, the only easy day was yesterday...

I've been doing a quick search for any imagery of suspected sites in Iran, but with no luck.

What would I do? Remember that I'm only a past-my-prime, former NCO mudbreather, so ops planning (on this scale) are well outside any arena of expertise I might've ever had.

That said...

First, I would instruct Mossad to conduct an intelligence review, as well as establish (or ramp up) existing operations (info only) in the area to get up-to-the-hour intel that's as reliable as possible. They are second to none at this, so I would also tell the CIA to butt the hell out (which would also help relations, by giving the US plausible deniability of any knowledge or assistance, to satisfy the terrorist sympathizers that make up half our country). I have to assume that this step is already well in motion, as Mossad is not stupid, and would have such ops going with or without Ministry knowledge or approval.

Second, I would probably use Sayerot Mat'kal for this (off the top of my head), since they are awesome at being in places theyre not supposed to be in. They would be supported by elements from the Golani brigade, since (if I remember correctly) theyre a little better schooled in guerrilla tactics and operations than the Givati is. They also can do more with less, much like the U.S. Marines.

Extraction after the fact is always the tricky part, and thats where geography would be crucial, hence my desire for maps. After the splosions, these guys will be hunted, and hunted fiercely. Several good E and E (evasion and escape) routes would be critical, and they should travel on foot in relatively small groups if possible. Again, geography and terrain dictate. If deniability is NOT an issue, however, then they have an awesome air commando unit that could pick them up no sweat.

The key, I think, would be infiltration and precision destruction of critical elements, rather than simply blowing the hell out of whatever looks important. Again, intelligence is absolutely critical.

But thats just the opinion of one ex-groundpounder, for whatever its worth.

tomder55 answered on 12/08/05:

you mean you wouldn't try to fly RH-53D Sea Stallions across 700 miles of desert with EC-130 Hercules refueling them in the attempt ?

kindj rated this answer Excellent or Above Average Answer

Question/Answer
excon asked on 12/08/05 - Death Penalty


Hello experts:

You know what I don't get? I don't get a society that can keep a guy in prison for 24 years (which is long enough for most lifers to serve their time and be released), and then kill him.

excon

tomder55 answered on 12/08/05:

agreed

the appeals process should be streamlined. Tookie should've been shut down years ago.

His legacy ? nahh it isn't children's books ;it's his son who is on the lam after he raped a 13 yr old girl at gun point ;and the other crips who have comitted thousands of murders .

ETWolverine rated this answer Excellent or Above Average Answer
excon rated this answer Excellent or Above Average Answer
sapphire630 rated this answer Excellent or Above Average Answer

Question/Answer
kindj asked on 12/08/05 - Someone may have it right

Netanyahu's Strong Words
Cal Thomas
Syndicated Columnist


December 7, 2005

We know several things about Iran. We know their new president has said publicly he believes Israel should be wiped off the map. We also know that Iran is developing nuclear devices that many Westerners strongly believe will be turned into nuclear weapons. Put the two together and you have a prescription for disaster in the Middle East.

While many world leaders and diplomats are rubbing the equivalent of worry beads, former Israeli Prime Minister Benjamin Netanyahu has spoken the unspeakable and thought the unthinkable. While campaigning for the office he once held in Israels March 28 election, Netanyahu has said he would support a pre-emptive strike against Irans nuclear program. It is the first time an Israeli leader has openly called for military action against Iran.



Netanyahu recalled Menachem Begins air strike against Iraqs nuclear reactor in 1981 and how it gave the region something resembling tranquility for twenty years.

Its funny that when Iran and other nations threaten mass destruction against Israel and the U.S., its no big deal. But when Israel calls for self defense, the world becomes upset. Netanyahu, like Begin before him, may be right.

Cal Thomas is a nationally syndicated columnist based in Washington, D.C.

Is BN right, in your opinion?

DK

tomder55 answered on 12/08/05:

I have long though of Bibi as one of the best Israeli leaders. The assumption that Israel could disable Iran's program with their airforce is a false one . To incapacitate their program would require a sustained bombing campaign and possilby a follow up ground assault.Iran is about 1,000 kilometers from Israel.According to the World Tribune Israel's deep-strike air capability is their F-15I and F-16C/D aircraft;range of no more than 600 kilometers Only the US has the capability to take them out militarily because of our carrier fleet. I am sure that spec-ops have plans for sabotage but I do not think that alone could stop the Iranians ;only delay them.

The nobel peace prize winning Mohamed ElBaradei and the IAEA now informs us that Iran is months away from a bomb. So much for that 10 year window .Thanks again CIA for that solid piece of intel. ! Think less about undermining the Bush Administration and a little more about supporting anti-regime forces in Iran .That would be a pleasant change!!! Maybe they can give us at least some solid intel. about where the nuke sites are ??? probably not .

I'm afraid the best Israel can do is threaten M A D ...but ....when you have someone like Ahmadinejad running Iran ,you can't count on that bluff working .

(he is as crazy as they come .today he said that Israel should establish its state on the German or Austrian soil since the two European countries feel responsible for the slaughter of millions of Jews during World War II ...He claims to be preparing a place for Shiite Muslims' "hidden" 12th Imam.
According to Shiite Muslim teaching, Abul-Qassem Mohammad, the 12th leader whom Shiites consider descended from Mohammed, disappeared in 941 but will return at the end of time to lead an era of Islamic justice.) He is currently consolidating his control on the country .

It would take fewer bombs to effectively eliminate Israel ,and if the crazies think that an Israeli response would just create more martyrs seeking virgins then all bets are off. Perhaps Mosad can send a special delivery right between his eyes .

ETWolverine rated this answer Excellent or Above Average Answer
excon rated this answer Excellent or Above Average Answer
kindj rated this answer Excellent or Above Average Answer

Question/Answer
excon asked on 12/08/05 - Re-cap


Hello experts:

We've been arguing politics here for years. How're we doin? Has any argument made by anyone here changed your mind or had an impact upon you in ways you did not contemplate?

Although it might not seem like it, I have been swayed by arguments from kindj, tom, Its and yes, the Wolverine dude (don't tell him, though). Swayed - not convinced! I have not been swayed by anything gade or the labdude says.

Has any argument I've ever made swayed you? HANK, CeeBee, Choux, Pdub - any of those?

excon

tomder55 answered on 12/08/05:

yup ;you've had an impact on my attitude about law and order and civil society in general. Just yesterday I mentioned you're response about the 'war on Christmas ' issue .

ETWolverine rated this answer Excellent or Above Average Answer
excon rated this answer Excellent or Above Average Answer
Itsdb rated this answer Excellent or Above Average Answer
purplewings rated this answer Excellent or Above Average Answer

Question/Answer
HANK1 asked on 12/07/05 - JUST WONDERING ...


... how many Japanese planes attacked Pearl Harbor on this date in 1941?

HANK

tomder55 answered on 12/08/05:

at least 350 were used

http://www.friesian.com/pearl.htm

Here is the strength of the attack force

6 aircraft carriers,
2 battleships, 3 cruisers,
9 destroyers,
8 tankers,
23 fleet submarines,
5 midget submarines,
441 planes

Here is the Japanese losses:

29 planes destroyed,
55 airmen killed,
5 midget submarines sunk,
9 submariners killed, 1 captured.

Here is the American losses:

3,581 military, 103 civilians killed or injured;
4 battleships sunk;
4 battleships damaged;
3 cruisers damaged;
3 destroyers sunk;
2 other ships sunk,
188 planes destroyed,
155 planes damaged

http://en.wikipedia.org/wiki/Attack_on_Pearl_Harbor

HANK1 rated this answer Excellent or Above Average Answer

Question/Answer
sapphire630 asked on 12/07/05 - How delirious is the radical left

Maureen Dowd Dec 5, 2005

W's head in the sand
first two sentences.....
In the Christmas spirit, the time has come for the reality based community to reach out to the White House. The Bush warriors are so deluded, they're even faking their fakery.


Pray tell, who are the reality based community???.....
certainly not Kerry, Dean, Boxer etc....they wouldn't know reality if it bit them in the butt and growled in their face!!!!

Anybody who would like to post a list of all the double talk all the loony left does I sure would appreciate it...and thanks ahead of time....


tomder55 answered on 12/08/05:

this is the pot calling the kettle black . To prove that she is delusional all one has to do is read her current book Are Men Necessary;a postmodern lament to love; where she proves to be a liberal women envious of the life she sees traditional conservative women have.For this she is bitter and she takes it out on men. She prominently diplays the old Norman Mailer quote "...all women needed were about a hundred semen slaves that they could milk every day...and they could keep the race going. So they don't need us."But whereas Mailer had a little tongue in cheek ;Dowd believes it with conviction.

"reality based community " is a term that lefty bloggers started using in the last election cycle to counter the "faith-based "community concept.

The source of the term is a quotation in an October 17, 2004, New York Times Magazine article by writer Ron Suskind , quoting an unnamed aide to George W. Bush:

The aide said that guys like me were "in what we call the reality-based community," which he defined as people who "believe that solutions emerge from your judicious study of discernible reality." ... "That's not the way the world really works anymore," he continued. "We're an empire now, and when we act, we create our own reality. And while you're studying that realityjudiciously, as you willwe'll act again, creating other new realities, which you can study too, and that's how things will sort out. We're history's actors . . . and you, all of you, will be left to just study what we do."

An anonymous source....a quote that makes the White House look bad.... the NYT. You do the math.

"reality based " is a reinvention of another idea that liberals ;especially in the MSM cling to; "conventional wisdom".Since they are primarily the source then it is understandable that they would cling to the concept. It is the key to their power hold over the acceptable ideas .

To find out if you are part of the reality based community you need to answer the following questions correctly :

The "Reality-Based" Community Yes/No Quiz



1) Do you think a significant percentage of prominent Republicans would secretly like to see the US become a theocracy?

2) Do you believe it was a mistake to go to war in Afghanistan?

3) In your opinion, is it a myth that American soldiers were spit on when they returned from Vietnam?

4) Michael Moore's distribution group, Front Row Entertainment, received help marketing "Fahrenheit 9/11" in Lebanon from the terrorist group Hezbollah. Do you believe that was appropriate?

5) Do you think you can be a patriotic American and support Iraq's anti-occupation resistance?

6) Do you think there is a significant chance that the capture of Saddam Hussein was timed to help George Bush politically?

7) In your opinion, is there a significant chance that Diebold is rigging elections in order to help the GOP?

8) Is George Bush more "evil" than Saddam Hussein?

9) In your opinion, is there a significant chance that Republicans rigged some of the Senate races in 2002?

10) Was Ingrid Newkirk right when she said, "There is no rational basis for saying that a human being has special rights. A rat is a pig is a dog is a boy. They're all mammals"?

11) Is there any nation in the world that's more of a force for good than the United States?

12) In your opinion, is the US a "stingy" country?

13) Is there a significant chance that America will become a fascist state in let's say the next 10 years?

14) Do you think there's a significant possibility that liberals will be rounded up and put into some sort of camps in let's say the next 10 years?

15) Is America an imperialist nation in your opinion?

16) Do you think "losing" in Vietnam was good for America?

17) Are you sometimes ashamed to be an American?

18) Do you think it's wrong for the President to put the welfare of Americans ahead of the welfare of people in other countries?

19) Do you see significant, noteworthy, parallels between America and Nazi Germany?

20) In your opinion, was Iraq primarily a "war for oil"?

21) What about Afghanistan? Was that primarily a "war for oil" as well?

22) Do you think it's likely a draft will be declared by the end of George Bush's term?

23) Do you think Iraq was preordained and planned before 9/11 ever took place?

24) In your opinion, is sleep deprivation a form of torture?

25) Would you prefer that we lose in Iraq?

26) Do you believe anyone who goes to Afghanistan or Iraq as a soldier is fighting for an evil cause under an evil commander in chief?

27) Was Michael Moore correct when he said, "There is no terrorist threat in this country. This is a lie?"

28) Is there in your opinion a significant chance that the Bush administration either was behind 9/11 or knew it was coming and allowed it to happened?

29) Do you think there is a significant possibility that the Bush administration had a hand in Paul Wellstone's death?

30) Do you believe that somebody rigged the vote in Ohio during the 2004 Presidential election?

31) In your opinion, do you think there is a significant chance that the Bush administration was behind the anthrax letters that were mailed out to some members of the media and US Senate?

32) Had George Bush lost the election, do you believe there was a significant chance Republicans would have thrown a coup?

33) Do you believe there's a significant chance that Karl Rove or someone else in the Bush administration had something to do with the last minute appearance of the Bin Laden tape right before the Nov. 2nd election?

34) Do you believe comparisons of George Bush to Hitler are appropriate?

35) Do you think Communism could work if the right people were running it?

36) Do you believe that black Americans who support and vote Republican are betraying their race?

37) Do you think people who say Al-Qaeda doesn't exist are right?

38) Are the insurgents in Iraq roughly comparable to Americans who fought against the British in your opinion?

39) Do you believe Congresswoman Marcy Kaptur was correct when she said, "One could say that Osama bin Laden and these non-nation-state fighters with religious purpose are very similar to those kind of atypical revolutionaries that helped to cast off the British crown"?

40) Do you believe there's a significant chance that the US Government knows where Bin Laden is and is deliberately allowing him to remain free?

Scoring

1-5 Yes Answers: Everybody has a few oddball beliefs, so this is nothing to worry about.
6-10 Yes Answers: OK, now you should be starting to get worried...well, maybe not if you're a contrarian....hmmm...uh....
11-15 Yes Answers: If you scored this high, you should be comfortable at the Democratic Underground.
16-20 Yes Answers: Hello "Reality Based Community", good-bye reality!
21-40 Yes Answers: Hello Noam Chomsky, Ted Rall, & Norman Mailer. When people talk about the "loony left," they mean you! I guess the list should also include Kerry, Dean,and Boxer .


Itsdb rated this answer Excellent or Above Average Answer
sapphire630 rated this answer Excellent or Above Average Answer

Question/Answer
Itsdb asked on 12/07/05 - CIA Leak investigation?

With all the brouhaha over Plamegate, you know 'leaking' the name of a non-covert agent that apparently everyone in the media knew about, why are there no investigations over potentially devastating leaks from CIA insiders?

"ABC News reported Monday night that two secret CIA prisons in eastern Europe were closed last month and 11 al-Qaida suspects were transferred to a facility in North Africa. The report, which ABC attributed to current and former CIA officers who spoke on condition of anonymity, said the prisons were shut down after Human Rights Watch said it had evidence suggesting such facilities existed in Romania and Poland."

~~~~~~~~~~~~~~~~~~~~~~~~~~~~~~~~~~~~~~~~~~~~~~~~~~~~~~

Personally, it doesn't really concern me if the CIA has "black site" prisons, I'm sick of the left's insistence we play footsie with terrorists instead of dealing with them like the scum they are. I am concerned that we're spending so much time and effort trying to nail Libbey and Rove when the CIA can't keep its own mouth shut. Just Google CIA OFFICER CONDITION OF ANONYMITY and see how many hits are returned.

More and more I am of the belief there really is a CIA war against the White House as tom has previously discussed.

What should be done? Should we praise these CIA insiders for exposing the horrors of the Bush administration? Should we investigate and prosecute the leakers for breaches of national security? Is the CIA a rogue agency or are there too many rogue CIA officials that don't give a hoot about national security? What?

Steve

tomder55 answered on 12/07/05:

The question remains : If the CIA was hot to get the leaker in the Plame case then why are they equally not demanding an investigation into thie leak of the black sites ? Why did classified documents make it to the press and the Kerry campaign during the election ? Why did the CIA permit an agent to publish a hit book on Bush called 'Imperial Hubris'as "Anonymous"(later identified as Michael Scheuer )?

The CIA ran the Wilson op. ;probably was the agency involved in forging the documents about Niger(they are now attempting a misdirection and focusing the source towards France while at the same time Sen Rockefeller is trying to pin the forgeries on Bush...the irony in all this is that Bush NEVER cited the forged docs in the SOTUS ) ;made sure that the administration would get their hands on the docs. knowing that the Administration would use them as evidence about Saddam's desire to reconstitue his nuclear program (which in itself was not a falsehood.)Sent Joe Wilson to Niger without any agreement of confidentiality where he ran a sloppy investigation and came home with the news that the claim about Iraq was bogus;and proceded to spill it out to any reporter willing to listen. It has since been revealed that Niger was deeply involved in providing AQ Khan yellow cake for an Islamic bomb (http://www.americanthinker.com/comments.php?comments_id=3782) Why would Niger be providing the Khan network with illegal transfers on uranium but not Saddam ?

Why was the information that Wilson obtained in a CIA op not confidential and classified ?
Was the Bush adm. supposed to sit back and take it ? There was nothing illegal in identifying Plame because of her connection to this coup attempt(Fitzgerald would've nailed someone by now if it were) and from a legal matter she had not been covert in years (her cover had been blown twice before ) . What happened however is that when Novak wrote about Plame he also mentioned her cover employer Brewster Jennings which wasa well-established CIA proprietary company, linked to ARAMCO. ARAMCO operates, manages, and maintains virtually all Saudi oil fields ...25 % of the worlds oil supply is run by ARMACO .They used Brewster Jennings to do ,among other things ,spy on Saudia Arabia. This was an asset that the CIA did not want to lose especially when the speculation is that Saudi oil production has already peaked and is on an irreversable decline.That is when their political opposition to the Bush Adm . became an all out bloodless coup attempt.

The latest attempt is of course the leaking of the black op sites . It is telling that again they are willing to risk one of their operations to GET the President. They went so far as to identify front airlines ;and flight plans .Amazing ! The NY Times reported in May that its sources included "interviews with former C.I.A. officers and pilots." It seems difficult to believe that the information conveyed in those interviews was unclassified. But if the agency made any objection to the NY Times's disclosure, it has not been publicly recorded. But the Washington Compost got the coup-de-gras when they identified European destinations (although the withheld the names of the Eastern European countries involved in the covert program, at the request of senior U.S. officials.)

To the charge that the CIA was exposing operations that the Bush Adm "forced them into "I can only get belly laugh at that . The rendition program began long before bush became President . I am looking for the CIA to "refer" the leak to the CIA and demand a special prosecutor ....but I'm not holding my breath .

Itsdb rated this answer Excellent or Above Average Answer

Question/Answer
HANK1 asked on 12/06/05 - KERRY'S BIG MOUTH AGAIN:



According to Paul Harvey this noon, John Kerry said in a CBS interview, "US troops are terrorizing women and children in Iraq."

WOW!

HANK

tomder55 answered on 12/06/05:

he sure did ;he had a swift boat flashback .not only that but then he said that he thought we should let the Iraqis do the terrorizing instead .

See Steve's response to my question yesterday .He links to the interview that Kerry had on the CBS Sunday talk show.

HANK1 rated this answer Excellent or Above Average Answer
sapphire630 rated this answer Excellent or Above Average Answer

Question/Answer
Itsdb asked on 12/06/05 - Worst DNC head ever?

No need to poll historians, just read or listen to what Howard Dean says...

"Saying the "idea that we're going to win the war in Iraq is an idea which is just plain wrong," Democratic National Chairman Howard Dean predicted today that the Democratic Party will come together on a proposal to withdraw National Guard and Reserve troops immediately, and all US forces within two years.

Dean made his comments in an interview on WOAI Radio in San Antonio.

"I've seen this before in my life. This is the same situation we had in Vietnam. Everybody then kept saying, 'just another year, just stay the course, we'll have a victory.' Well, we didn't have a victory, and this policy cost the lives of an additional 25,000 troops because we were too stubborn to recognize what was happening."

Yeah, that's the ticket, admit defeat now. Does anyone actually believe Dean and the rest of the anti-war left gives a damn about the lives of American troops?

Steve

tomder55 answered on 12/06/05:

YYEEAAARRGGHH!!!! Must be off my Meds again!!!!
One would think, as much abuse as he suffers at the hands of all those who would parody him that he would turn to making rational statements.

"In predicting that America will lose the war in Iraq, Howard Dean is the latest national Democrat leader to embrace retreat and defeat in the central front in the War on Terror. His outrageous prediction sends the wrong message to our troops, the enemy, and the Iraqi people just 10 days before historic elections. Democrats across the nation should stand up and reject the pessimism of their chairman and strategy of defeat by their Congressional leaders."

-- Ken Mehlman, RNC Chairman


Dean said the Democrat proposal is not a 'withdrawal,' but a 'strategic redeployment',much like the nonsense that Murtha is espousing and Pelosi is parroting . A Democrat 'cut and run in the face of victory 'strategy is emerging .Redeployment" by disengagement with no intent to return to the battlefield by anyones definition is a retreat.

The good Doctor said that after the national guard came home the remaining US troops could be garrisioned in a friendly M.E. country so that we could continue to battle Zarqwai ;but outside of his range to do us any harm.

Which country ??? He did not say. Turkey did not let us launch the 4th ID from bases there so they are out of the question . Kuwait and Qatar are on the opposite side of Iraq from the Northwestern border trouble spots in Iraq. Saudia Arabia ? Well our troops stationed there is what supposedly pissed off OBL in the first place.

ETWolverine rated this answer Excellent or Above Average Answer
Itsdb rated this answer Excellent or Above Average Answer

Question/Answer
Choux asked on 12/05/05 - Worst President Ever

Donnie Fowler blogging:::

""Well, the historians have gone and done it. Risking the creation of a White House Commission on Historical Quality to refute their findings with real science, an overwhelming 338 of 415 historians polled by George Mason University said Friday that George W. Bush is failing as a president. And fifty of them rated Bush as the worst president ever, ranking him above (below?) any other past president -- even those you've never heard of who were also really awful.

Why do these misguided, obviously-socialist, ivy-smoking, and (of course) American-hating intellectuals feel that Bush isn't doing his best?

Well, they look at the record ...

# He has taken the country into an unwinnable war and alienated friend and foe alike in the process;

# He is bankrupting the country with a combination of aggressive military spending and reduced taxation of the rich;

# He has deliberately and dangerously attacked separation of church and state;

# He has repeatedly "misled," to use a kind word, the American people on affairs domestic and foreign;

# He has proved to be incompetent in affairs domestic (New Orleans) and foreign (
Iraq and the battle against al-Qaida);

# He has sacrificed American employment (including the toleration of pension and benefit elimination) to increase overall productivity;

# He is ignorantly hostile to science and technological progress;

# He has tolerated or ignored one of the republic's oldest problems, corporate cheating in supplying the military in wartime.

Quite an indictment. It is, of course, too early to evaluate a president."



Very convincing case he made, don't you think?



tomder55 answered on 12/06/05:

this was originally from a Richard Reeves hit piece . It is safe to assume that academia is still dominated by lefty loonies and the History Depts are no exception .Until historians have gained access to the archival records, they are no better at evaluating his Presidency ;which still has 3 years left; (a lifetime in politics) than any other voter. Lincoln was judged a complete failure just 10 months before his asassination and Truman would never have survived re-election.

Compared to some recent Democrat Presidents Bush deserves a place on Mt. Rushmore.The only reason I do not consider him one of the best is because he has blown it on some domestic issues. I wish he'd buy himself a veto pen and use it once and a while .The deficit is troublesome to a degree but when does it become a factor and at what level? The only meaningful way to measure the deficit is as a percentage of GDP. Measured that way, it is not close to all time highs and is declining because the tax cuts brought in more money to the treasury, not less.

The US economy seems to be doing very well .His tax cuts revived the economy after twin hits of the tech melt down and the economic impact of 9-11 .


Reeves is completely wrong about Iraq and the over-all War against Islamo-fascists ;but most great war Presidents are not recognized for many years after their terms are up . Reeves if he were being honests would admit that he prefered it when the jack-boot of Saddam and his thug sons ruled Iraq. I say that just the removal of Saddam from power was a worthy enough reason for war ;and the vision of transforming the greater middle east is a measure of greatness.

Saad Eddin Ibrahim, who's spent years as a democracy advocate in Egypt, told the Washington Post's Jim Hoagland that although he'd opposed the invasion of Iraq, he had to admit it had "unfrozen the Middle East, just as Napoleon's 1798 expedition did." Elections in Iraq force the theocrats and autocrats to put democracy on the agenda.It has had a positive ripple effect in Egypt ,Lebanon (and Syria) ;Libya among other nations (even to a lesser extent Saudi Arabia).

What Reeves and his ilk seem to forget ;and history proves this true ;the choice is NEVER between action and status quo.US patrolling and enforcing endless UN Sanctions forever was never a realistic option. The sanction regime was already collapsing due to the Oil for Food corruption .

It is likely that Bush will be remembered for his willingness to take action against anti-demoracy forces, bring democracy to Afghanistan and Iraq , as well as destroying Al-Qaeda as a functional terrortist organisation.



Choux rated this answer Excellent or Above Average Answer
ETWolverine rated this answer Excellent or Above Average Answer
sapphire630 rated this answer Excellent or Above Average Answer

Question/Answer
Choux asked on 12/03/05 - Political Ethics

"According to a recent article in the Advocate, Ford Motor company, failing in its efforts to increase plummeting sales, has apparently decided that the best way to regain the hearts of Americans is to embrace homophobia.

The virulently anti-gay American Family Association (AFA) recently approached the foundering automobile company and informed them that, because Ford was advertising its Jaguar and Land Rover brands in gay publications, the AFA would launch a boycott of the company.

Apparently, Ford immediately caved. On December 2nd, Ford spokesman Mike Moran told the Advocate.com that indeed, the company will stop advertising its Jaguar and Land Rover brands in all gay publications.

For a short time Ford actually had been supportive of equal rights for all Americans, supporting various minority causes. No longer. Donald Wildmon, the AFAs chairman, was quoted as saying [Ford has] heard our concerns; they are acting on our concerns. We are pleased with where we are.

Moran didnt provide many details about its agreement with the AFA, although the assumption has to be that in upcoming Ku Klux Klan parades, the participants will all be driving brand new white Ford Tauruses.

Homophobia is an interesting policy for a company started by Henry Ford, one of Hitlers strongest supporters in America, and one the countrys most outspoken anti-Semites. Ford was the author of such books as: "The International Jew, the World's Foremost Problem," which were distributed through Ford's car dealerships.

This bit of unhappy news comes at a time when Ford is in deep financial trouble, closing its plants and desperately trying to restructure the company. Interesting that the company seems to have decided that gays dont buy Fords, or that the gay market, far bigger than the AFA market, isnt worth holding on to. Also interesting that the company has decided its future lies in its past, in its gone-but-not-forgotten policies of discrimination and bias.

Most Americans are not familiar with the companys shameful history.....""


Is it ethical to pressure Corporations for religious agendas? Is it ethical for a Corporation to cave-in to pressure?

Do any ethical considerations cover product boycots?

tomder55 answered on 12/04/05:

Good question. The practice began in Ireland and targeted a ruthless landlord named Boycott. All of his tenants were so upset that they refused to have anything to do with Boycott and his family. The practice came to the U.S. in support of labor movements. And in the 1960's it gained popularity as a political tool.Over recent years you've seen more boycotts. That's because they appear to be working

When South Africa was run by the white gvt. thre were many instituions public and private who were pressured to divest of their South African holdings . I imagine that it must have had an impact.

When talk hostess Dr.Laura Schlessinger made comments that were "anti-gay" the Gay and Lesbian Alliance Against Defamation decided to take her on .Proctor & Gamble under pressure from the group pulled ads joined by Xerox, AT&T, United Airlines, Toys 'R' Us and American Express.
Eventually her show was pulled from many markets.

Proctor and Gamble it appears cannot catch a break. Last year the AFA launched a boycott against P&G for the company's financial support of a campaign to repeal a Cincinnati city-charter amendment approved in 1993 that banned same sex marriage.

Does an ideologically based boycott of those who espouse unpopular opinions remind us of the Joseph McCarthy era? If it does when the AFA applies the pressure it equally does when GLAAD does . I will not speak against the use of boycotts .It is simply a matter of free speech in my view. It's fair to vote with your money by financially supporting businesses that you admire. Or to withhold your business from companies you disapprove of.
Boycotts work apparently for economic reasons .The ethics involved appears to be a factor only dependent on which side of the issue of a dispute you take.

Ford calculated the economic impact and decided to pull the ads . I do not read anything else into their motivation. It is rediculous to suggest that their decision has anything to do with Henry Ford's beliefs . For my part;I have never owned a Ford. My boycott has nothing to do with Henry Ford (he was a bastard) but because I am suspect about their quality. Find On Road Dead ...Fix Or Repair Daily is how I would catagorize a Ford car.

Choux rated this answer Excellent or Above Average Answer
Itsdb rated this answer Excellent or Above Average Answer

Question/Answer
Choux asked on 12/01/05 - Eyewitness Charges Bush Lied

"Yesterday, President Bush claimed that Iraqi security forces primarily led the assault on the city of Tal Afar. Bush highlighted it as an especially clear sign of the progress Iraq security forces were making in Iraq.

The progress of the Iraqi forces is especially clear when the recent anti-terrorist operations in Tal Afar are compared with last years assault in Fallujah. In Fallujah, the assault was led by nine coalition battalions made up primarily of United States Marines and Army with six Iraqi battalions supporting themThis year in Tal Afar, it was a very different story. The assault was primarily led by Iraqi security forces 11 Iraqi battalions, backed by five coalition battalions providing support.

TIME Magazine reporter Michael Ware, who is embedded with the U.S. troops in Iraq who participated in the Tal Afar battle, appeared on Anderson Cooper yesterday. He said Bushs description was completely untrue:

I was in that battle from the very beginning to the very end. I was with Iraqi units right there on the front line as they were battling with al Qaeda. They were not leading. They were being led by the U.S. green beret special forces with them."


Comments?

tomder55 answered on 12/02/05:

The official report from Centcom says that Iraqi units led the operation. Other reports suggest it was a joint operation. What is important in my mind is that once Tal Afar was cleaned up then Iraqi security was left in place to patrol and prevent what happened in Fallujah (we had to take the city twice). What was important and underreported in Bush's speech has been a different strategy to combat an insurgency . Previously in places like Vietnam we played a game of 'hit the mole'[that old arcade game when you hit a mole and it pops up someplace else]. Now the strategy is to take a city /town and with the increase of Iraqi security forces we do not have to abandon it to be reoccupied by Zarqwai forces again.

Joe Lieberman in his Wall Street Journal op-ed this week stated the strategy as well as anyone I've heard .This after visiting Iraq for the 4th time (more than I would venture Rep. Murtha has traveled)

The administration's recent use of the banner "clear, hold and build" accurately describes the strategy as I saw it being implemented last week.
We are now embedding a core of coalition forces in every Iraqi fighting unit, which makes each unit more effective and acts as a multiplier of our forces. Progress in "clearing" and "holding" is being made. The Sixth Infantry Division of the Iraqi Security Forces now controls and polices more than one-third of Baghdad on its own. Coalition and Iraqi forces have together cleared the previously terrorist-controlled cities of Fallujah, Mosul and Tal Afar, and most of the border with Syria. Those areas are now being "held" secure by the Iraqi military themselves. Iraqi and coalition forces are jointly carrying out a mission to clear Ramadi, now the most dangerous city in Al-Anbar province at the west end of the Sunni Triangle.

Nationwide, American military leaders estimate that about one-third of the approximately 100,000 members of the Iraqi military are able to "lead the fight" themselves with logistical support from the U.S., and that that number should double by next year. If that happens, American military forces could begin a drawdown in numbers proportional to the increasing self-sufficiency of the Iraqi forces in 2006.

Choux rated this answer Excellent or Above Average Answer
ETWolverine rated this answer Excellent or Above Average Answer

Question/Answer
arcura asked on 12/01/05 - What do you make of this? Note the legal opinions.

HEIL HITLER! IT HAS BEGUN
By: Devvy Kidd
December 1, 2005
NewsWithViews.com
"If we wish to be free, if we mean to preserve inviolate those inestimable privileges for which we have been so long contending, if we mean not basely to abandon the noble struggle in which we have been so long engaged, and which we have pledged ourselves never to abandon until the glorious object of our contest shall be obtained - we must fight!" Patrick Henry
Anyone who doesn't recognize that a police state is being erected right in front of their eyes is either in a state of denial or welcomes a repeat of Nazi Germany under Adolph Hitler. Two months ago a woman named Deborah Davis was reading a book while riding to work on a public bus. When her bus stopped outside the gates of the Denver Federal Center in Lakewood, Colorado (only three miles from my former home in Lakewood), a guard climbed aboard the bus and demanded that all the passengers produce identification. Mrs. Davis didn't bite:
"I told him that I did have identification, but I wasn't going to show it to him," Davis explains. "I knew that I wasn't required by law to show ID and that's why I decided I wasn't going to. The whole thing seemed to be more about compliance than security."
This guard who obviously has no understanding of the U.S. Constitution and the Bill of Rights, called the federal dragoons who proceeded to drag Mrs. Davis off this public bus, handcuffed her like some criminal, shoved her into the back seat of their Barney Rubble guard car and transported her to a police station within the Federal Center. For all this guard knew, Mrs. Davis could have been going anywhere outside the fencing of the Denver Federal Center (there is a post office just outside the gated entrance on the south side) once she got off the bus. His apprehension of her in my opinion isn't just unlawful detention, but kidnapping. Mrs. Davis' son is in Iraq fighting Bush's war to control the oil in the middle East while his mother is being subjected to the same treatment as those who live in communist countries and did under Hitler's regime.
Mrs. Davis will be arraigned on December 9, 2005, and faces up to 60 days in jail on federal criminal misdemeanor charges. These charges would be that "citizens must, when requested, display Government or other identifying credentials to Federal police officers or other authorized individuals." The second would be that citizens must comply with "the lawful direction of Federal police officers and other authorized individuals."
Open your eyes America. First, Mrs. Davis was on a public bus, she was not on federal property. This guard had no right to demand any American produce papers of any kind whether they are riding a bus or walking.
Second, under the U.S. Constitution, Congress has the power to make criminal only four types of conduct: treason, piracies and felonies on the high seas, counterfeiting, and offenses against the laws of nations. Just because Congress has been getting away with passing a zillion laws with all kinds of "offenses" does NOT make it legal. They have only gotten away with it all this time because the American people have refused to hold their elected public servants accountable out of blind loyalty to their damn party. What you are seeing right now in this country is the result of foolish voters. Despite the refusal by the sheeple to boot out rotten politicians, the law is still law:
"The highest law of the land is the Constitution of the United States." Stephen K. Huber, Professor of Law, University of Houston
"The general misconception is that any statute passed by legislators bearing the appearance of law constitutes the law of the land. The United States Constitution is the supreme law of the land, and any statue must be in agreement with it to be valid. It is impossible for both the Constitution and a law violating it to be valid; one must prevail over the other. The Sixteenth American Jurisprudence, (2nd ed., Section 256), states:
"The general rule is that an unconstitutional statue, though having the form and name of law, is in reality no law, but is wholly void and ineffective for any purpose; since unconstitutionality dates from the time of its enactment and not merely from the date of the decision so branding it. A void act cannot be legally consistent with a valid one. An unconstitutional law cannot operate to supersede any existing valid law. Indeed, insofar as a statute runs counter to the fundamental law of the land, it is superseded thereby." Dr. Jacques S. Jaikaran, author of Debt Virus
The same nonsense is about to be unleashed upon innocent, law abiding citizens in Miami, Florida. The announcement came November 28, 2005: local coppers will "...stage random shows of force at hotels, banks and other public places to keep terrorists guessing and remind people to be vigilant."
"Deputy Police Chief Frank Fernandez said officers might, for example, surround a bank building, check the IDs of everyone going in and out and hand out leaflets about terror threats.<> "This is an in-your-face type of strategy. It's letting the terrorists know we are out there," Fernandez said. The operations will keep terrorists off guard, Fernandez said. He said al-Qaida and other terrorist groups plot attacks by putting places under surveillance and watching for flaws and patterns in security. < style="font-family: times new roman;">
"People are definitely going to notice it," Fernandez said. "We want that shock. We want that awe. But at the same time, we don't want people to feel their rights are being threatened. We need them to be our eyes and ears."
Deputy Police Chief Frank Fernandez is a fool who should be removed from his job as well as the Mayor and any member of the Miami City Council who approved this BS. First: There is no legal authority for coppers to demand anyone entering or leaving a bank, hotel or other public places "show their papers." The courts have consistently upheld the absolute right of Americans to travel freely without interference or harassment and walking is traveling, just like riding a horse or driving a car.
Second, the CIA's creation called 'al-Qaida' doesn't give a hoot about a bunch of local coppers acting like testosterone pumped goons harassing little old ladies. You just wait until one of these little old ladies is so "shocked and awed" by a sudden show of force, they drop dead from a heart attack or massive stroke. Then the City of Miami will pay dearly.
Walking into or exiting a hotel, bank or "other public places" is a fundamental right and an action freely chosen by an individual; it is not a mandated activity by any federal, state or local law, ordinance or statute. Free Americans have a constitutional right to travel which is protected by the U.S. Constitution; see Crandall v. Nevada, 73 U.S. (6 Wall.) 35, 49 (1868)("We are all citizens of the United States, and as members of the same community must have the right to pass and repass through every part of it without interruption, as freely as in our own states"); Kent v. Dulles, 357 U.S. 116, 125, 78 S.Ct. 1113, 1118 (1958)("The right to travel is a part of the 'liberty' of which the citizen cannot be deprived without the due process of law under the Fifth Amendment"); United States v. Guest, 383 U.S. 745, 757, 86 S.Ct. 1170, 1178 (1966)("The constitutional right to travel from one State to another, and necessarily to use the highways and other instrumentalities of interstate commerce in doing so, occupies a position fundamental to the concept of our Federal Union");
Shapiro v. Thompson, 394 U.S. 618, 629, 89 S.Ct. 1322, 1329 (1969)("This Court long ago recognized that the nature of our Federal Union and our constitutional concepts of personal liberty unite to require that all citizens be free to travel throughout the length and breadth of our land uninhibited by statutes, rules or regulations which unreasonably burden or restrict this movement") Dunn v. Blumstein, 405 U.S. 330, 339, 92 S.Ct. 995, 1001 (1972)("....since the right to travel was a constitutionally protected right, 'any classification which serves to penalize the exercise of that right unless shown to be necessary to promote a compelling governmental interest, is unconstitutional'");(The Court in Dunn also declared that "The right to travel is an 'unconditional personal right, ' a right whose exercise may not be conditioned.'" Id, at 341); and Memorial Hospital v. Maricopa County, 415 U.S. 250, 254, 94 S.Ct. 1076, 1080(1974)("The right of interstate travel has repeatedly been recognized as a basic constitutional freedom").
See also Schachtman v. Dulles 225 F2d. 938, 941 (D.C.Cir. 1955)("The right to travel, to go from place to place as the means of transportation permit, is a natural right subject to the rights of others and to reasonable regulation under law"); Worthy v. Herter, 270 F.2d 905, 908 (D.C.Cir. 1959)("The right to travel is a part of the right to liberty"); Cole v. Housing Authority of City of Newport, 435 F2.d 807, 809 (1st Cir.1970)("...the right to travel is a fundamental personal right that can be impinged only if to do so is necessary to promote a compelling governmental interest"); King v. New Rochelle Municipal Housing Authority, 442 F.2d 646, 648 (2nd Cir. 1971)("It would be meaningless to describe the right to travel between states as a fundamental precept of personal liberty and not to acknowledge a correlative constitutional right to travel within a state"); and
Demiragh v. DeVos, 476 F.2d 403, 405 (2nd Cir. 1973)("...the fight to travel....[is] a 'fundamental' one, requiring the showing of a 'compelling state or local interest to warrant its limitation"); United States v. Davis, 482 F.2d 893, 912 (9th Cir. 1973)("....it is firmly settled that freedom to travel at home and abroad without unreasonable governmental restriction is a fundamental constitutional right of every American citizen....At the minimum, governmental restrictions upon freedom to travel are to be weighed against the necessity advanced to justify them, and a restriction that burdens the right to travel 'too broadly and indiscriminately' cannot be sustained"); and McLellan v. Miss. Power & Light Co., 545 F.2d 919, 923 n. 8 (5th Cir. 1977)("The Constitutional right to travel is 'among the rights and privileges of National citizenship");
Costa v. Bluegrass Turf Service, Inc., 406 F.Supp. 1003, 1007 (E.D.Ken. 1975)("...pure administrative convenience, standing alone, is an insufficient basis for an enactment which...restricts the right to travel"); Coolman v. Robinson, 452 F.Supp. 1324, 1326 (N.D.Ind. 1978)("The right to travel is a very old and well established constitutional right"); Tetalman v. Holiday Inn, 500 F.Supp. 217, 218 (N.D.Ga. 1980)("the constitutionally protected right to travel...is basically the right to travel unrestricted by unreasonable government interference or regulation"); Bergman v. United States, 565 F.Supp. 1353, 1397 (W.D. Mich. 1983)("The right to travel interstate is a basic, fundamental right under the Constitution, its origins premised upon a variety of constitutional provisions").
This right to travel is also a constitutional right under our state constitution, embodied within its "liberty" provisions; People v. Olivas (1976) 17 Cal.3d 235, 131 Cal. Rptr. 55, 551 P.2d 375, 381 (right to travel is a fundamental liberty interested protected by the 14th Amendment to the U.S. Constitution; further "We conclude that personal liberty is a fundamental interest, second only to life itself, as an interest protected under both the California and United States Constitutions," 551 P.2d at 384); People v. Horton (1971) 14 CalApp.3d 930, 92 Ca.Rptr. 666, 668 ("...the right of the citizen to drive on a public street with freedom from police interference...is a fundamental constitutional right").
The Bill of Rights is just that: rights, not privileges and no federal or state agency can violate our rights as reaffirmed by the Bill of Rights. These are rights we are all born with, no government gave them to us nor can they take them away. These precious tenets are the very foundation of our Republic. In Miller v. U.S., 230 F., 2nd 286, 489 the court said: "The claim and exercise of a Constitutional Right cannot be converted into a crime."
The City of Miami has no compelling reason to demand it's citizenry show identification papers while they practice a "show of force" to impress some would be terrorists. If the 80 million gun owners of this country would get off their hands and demand their state legislatures reconstitute the lawful, constitutionally authorized state militias, there would be no need for this ridiculous horse and pony "show of force" in any city in America. As for the question of terrorists, I cannot stress how important it is for the American people to get all the known facts at this time regarding 9/11 - the justification for the continued assault on our God given rights by Congress and state and local municipalities.
Jefferson said it perfectly and city fathers, local law enforcement and the federal dragoons need to pay attention:
"Under the law of nature, all men are born free, every one comes into the world with a right to his own person, which includes the liberty of moving and using it at his own will. This is what is called personal liberty, and is given him by the Author of nature, because necessary for his own sustenance." --Thomas Jefferson: Legal Argument, 1770. FE 1:376
We the people are not going to lay down and take this flavor of tyranny. Oh, there will be those who quiver at the very thought of standing up for their rights, but there will always be cowards who want others to fight for their freedom. Mrs. Davis drew her line in the sand and so should the tens of millions of Americans in this country who claim they will fight for their rights. We must let our local, state and federal elected servants hear the roar of NO from coast to coast, border to border.
Will you fight by standing your ground or will you surrender yourself to slavery?
I will leave you with these words from real warriors:
"As long as a hundred of us remain alive we will never be subject to tyrannical dominion because it is not for glory or riches or honours that we fight, but for freedom alone which no worthy man loses except with his life." The Declaration of Arbroath 1320

tomder55 answered on 12/01/05:

First observation . I am very suspicious of web sites who's lead article is about the Tri-Lateral Commission ;or other world wide global conspiracy groups . (1st article entitled :Who Are the Global Elites )

Then I'd like to see who Devvy Kidd is ?

From what I can determine the checks occure in front of the Fed Pen. only on days when the site is on "heightened alert" ; BEFORE THE BUS PASSES THROUGH THE FACILITY AND DROPS OFF SOME OF THE 7-8000 PEOPLE WHO WORK THERE DAILY. This has been SOP since the Oklahoma City Bombing .

Unfortunately the general public is not always aware of the alert. But in this case it is false to say that she did not know ;or that this was a sudden intrusion .Davis said she showed her ID when a Federal Center guard asked to see it for the first couple of days she rode the RTD bus through the center . But it bothered her.For a few subsequent days, she told the guards she wasn't getting off in the Federal Center and didn't have an ID. They let her stay on the bus.Finally, on a Friday, a guard told Davis she had to have an ID the next time. Then she decided to make her stand.Before she pulled her stunt she consulted with Bill Scannell, an activist who has helped publicize other challenges to government requirements that people show identification. Scannell created a Web site during the Thanksgiving weekend about Davis' case: papersplease.org/Davis.

Sorry ;I do not see her as a 21st Century Rosa Parks.


In NY we are subject to random id checks at bridge and tunnel crossings when the alert level is heightened. I have no problem with it and do not see us sinking into a NAZI state as a result.

arcura rated this answer Excellent or Above Average Answer
Itsdb rated this answer Excellent or Above Average Answer

Question/Answer
labman asked on 11/30/05 - Time table

So who remembers ''Over There''? ''We are going over there, and we won't be back until it is over, over there.''

tomder55 answered on 12/01/05:

George M Cohen a great American wrote it .

Is it over ;not quite yet ;but it has been won ;we are in mop up mode. We have to stay a while longer at present levels to ensure that the Iraqis can maintain security themselves . The only difference in American positions regarding withdrawal is the reason behind it . On one side you have those who call for withdrawal because we have been defeated (Murtha);and those who think we should withdraw after the Iraqis can maintain stability on their own .[one caveat : security means internal .if they were ever threatened by their neighbors they would have to rely on an alliance with us to prevent being overwhelmed (much like our 'friends 'in NATO still do). We are training counter-insurgency forces not mobile armored units even though they still have about 400 T-72 tanks they will eventually fire up again . But the reason I believe we will garrison there for some time yet is the threat from Iran.] .

BTW ;did you hear Kerry's flip-flop yesterday ? He basically said that the Democrats are not calling for a time table for leaving Iraq, but were instead calling for a time table for success in Iraq which would allow for the U.S. military to leave.huh?What is his or his party's definition of success ? Got me.

They will hold their elections in two weeks, over 65% of those eligible will vote.The Sunni's for the most part will participate in the federal system and without wholesale Sunni support the Zarqwai's and the "rejectionists " will have no place to hide.

ETWolverine rated this answer Excellent or Above Average Answer
labman rated this answer Excellent or Above Average Answer

Question/Answer
excon asked on 11/30/05 - Lobbyists, money and corruption



Hello experts:

You do know that the congress has been sold to the highest bidder. Lobbyists even write legislation. Lobbyists suck. I hope they ban 'em - ALL OF 'EM.

You?

excon

tomder55 answered on 11/30/05:

They sure strain their relationship with free speech advocates but I am more concerned with politicians with their hands in the honey pot . Duke Cunningham could've quit Congress ;got a job on K Street and then all the stuff he got would've been legal but he was sworn to the public trust .

On the tenth of May, 1972 Lieutenant Randy Cunningham and his partner LT(JG) Willie Driscoll would shoot down two MIGs, making them the first American aces of the Vietnam War.He went on to be a Top Gun trainer .

His is a real sad story ;the fall of a genuine hero to hubris. The making of a Greek Tragedy . The Repubicans came into Washington in 1994 as the reformers to the system ;they apparently got caught up in it instead.

Ethics rules have grown to the size of the tax code and yet the nature of the beast hasn't changed .The temptation grows as the size of the government does . I hope Cunningham enjoyed his commode . I have a feeling the crapper he sits on in jail will not be as comfortable.

excon rated this answer Excellent or Above Average Answer
Itsdb rated this answer Excellent or Above Average Answer

Question/Answer
sapphire630 asked on 11/29/05 - what it takes to find things out

This morning I was with the Moon detectives and asked them why the channel 4 news was there at my neighbors house. They said, "there was an incident" and left it at that. I just looked up one of the sites that Tomder has posted to see what all they had for news today and here is my neighbor and the incident.

http://www.wsoctv.com/news/5426229/detail.html

Mom Allegedly Spikes Mac & Cheese With Bleach

POSTED: 10:59 am EST November 29, 2005

CORAOPOLIS, Pa. -- A mother has been charged with trying to poison her adult daughter and her daughter's family after allegedly pouring bleach into their macaroni and cheese.
Nancy O'Donnell, 56, of Moon Township, Pa., was charged with four counts each of aggravated assault and recklessly endangering another person after the reported incident Saturday night, police said.
O'Donnell's adult daughter, Victoria Lynn O'Donnell, 24, was preparing dinner around 6:30 p.m. for her live-in boyfriend, Jamal Scott, 30, and their 6-year-old son and 2-year-old daughter. The four share a home with Nancy O'Donnell.
Victoria O'Donnell went upstairs while the food was cooking, and police said that when she came back downstairs, she could smell bleach in the food. She tried the macaroni and cheese and immediately spit it out after tasting the bleach, police said.
No one else tasted the food.
Victoria O'Donnell then confronted her mother about the allegedly tainted dinner. According to court records, Nancy O'Donnell said she wanted to sicken her daughter because "you don't deserve those children."
Police reported a strong odor of bleach when they arrived to investigate. Nancy O'Donnell later denied pouring bleach into the pasta, police said.
Nancy O'Donnell is being held in the Allegheny County Jail on $25,000 bond. A judge ordered that she receive a mental health evaluation.

And to think, a couple weeks ago, one of my other neighbors told the mother who put the bleach in the food "why don't you move in with Linda" when she
said she that she didn't want to live with her daughter any more!

tomder55 answered on 11/30/05:

hope you don't plan on adding it to the recipe book.

Let's kick it up another notch....BAM !

wow ,a very exciting neighborhood you live in !

ETWolverine rated this answer Excellent or Above Average Answer
sapphire630 rated this answer Excellent or Above Average Answer

Question/Answer
sapphire630 asked on 11/29/05 - Bush vows crack down on illegals

Is this the same type vow O.J. made to track down Nicoles *real* killers?

tomder55 answered on 11/30/05:

I believe so ;looking for all the illegals at the golf clubs (I think they can be found at the maintainance shed ).

I think Bush took a much needed first baby step .Perhaps he is proposing what he thinks is viable ;but his proposal is a far cry from fixing the border to my satisfaction.

excon rated this answer Excellent or Above Average Answer
sapphire630 rated this answer Excellent or Above Average Answer

Question/Answer
kindj asked on 11/29/05 - Iraq weapons

Not really political, but interesting nonetheless.

This came from an uncle of mine. An old service buddy of his has a son in the Corps in Iraq right now, and this comes from his conversations with his boy.

Funny, though, how what they're saying now is the same thing we said 15 years ago, the first time around.

I thought about editing some of the language, then decided not to. Soldiers may be a lot of things, but spotless in their language is not one of them.

All that follows is his:

Weapons in Iraq - what works and what doesn't



>>Thought the group would like to see the following from an individual with their boots on the ground. I still marvel how "management" in all organizations ignores what their people doing the work tell them and insist on doing things their way even if it is failing.
>
> =======================================

> The information below is from a close friend of mine. Jordan is his son. Feel free to pass it along. Tom

Hello to all my fellow gunners, military buffs, veterans and interested guys. A couple of weekends ago I got to spend time with my son Jordan, who was on his first leave since returning from Iraq. He is well (a little thin), and already bored. He will be returning to Iraq for a second tour in early ག and has already re-enlisted early for 4 more years. He loves the Marine Corps and is actually looking forward to returning to Iraq.
>
Jordan spent 7 months at "Camp Blue Diamond" in Ramadi. Aka: Fort Apache. He saw and did a lot and the following is what he told me about weapons,
equipment, tactics and other miscellaneous info which may be of interest to you. Nothing is by any means classified. No politics here, just a Marine with a bird's eye view's opinions:

1) The M-16 rifle : Thumbs down. Chronic jamming problems with the talcum powder like sand over there. The sand is everywhere. Jordan says you feel
filthy 2 minutes after coming out of the shower. The M-4 carbine version is more popular because it's lighter and shorter, but it has jamming problems
also. They like the ability to mount the various optical gunsights and weapons lights on the picattiny rails, but the weapon itself is not great in
a desert environment. They all hate the 5.56mm (.223) round. Poor penetration on the cinderblock structure common over there and even torso hits cant be reliably counted on to put the enemy down. Fun
fact: Random autopsies on dead insurgents shows a high level of opiate use.

2) The M243 SAW (squad assault weapon): .223 cal. Drum fed light machine gun. Big thumbs down. Universally considered a piece of shit. Chronic
jamming problems, most of which require partial disassembly. (that's fun in the middle of a firefight).

3) The M9 Beretta 9mm: Mixed bag. Good gun, performs well in desert environment; but they all hate the 9mm cartridge. The use of handguns for self-defense is actually fairly common. Same old story on the 9mm: Bad guys hit multiple times and still in the fight.

4) Mossberg 12ga. Military shotgun: Works well, used frequently for clearing houses to good effect.

5) The M240 Machine Gun: 7.62 Nato (.308) cal. belt fed machine gun, developed to replace the old M-60 (what a beautiful weapon that was!!). Thumbs up. Accurate, reliable, and the 7.62 round puts 'em down.
Originally developed as a vehicle mounted weapon, more and more are being dismounted and taken into the field by infantry. The 7.62 round chews up the structure over there.

6) The M2 .50 cal heavy machine gun: Thumbs way, way up. "Ma deuce" is still worth her considerable weight in gold. The ultimate fight stopper, puts their dicks in the dirt every time. The most coveted weapon
in-theater.

7) The .45 pistol: Thumbs up. Still the best pistol round out there. Everybody authorized to carry a sidearm is trying to get their hands on one. With few exceptions, can reliably be expected to put 'em down
with a torso hit. The special ops guys (who are doing most of the pistol work) use the HK military model and supposedly love it. The old government model .45's
are being re-issued en masse.

8) The M-14: Thumbs up. They are being re-issued in bulk, mostly in a modified version to special ops guys. Modifications include lightweight Kevlar stocks and low power red dot or ACOG sights. Very reliable in the sandy environment, and they love the 7.62 round.

9) The Barrett .50 cal sniper rifle: Thumbs way up.
> Spectacular range and accuracy and hits like a freight train. Used frequently to take out vehicle
suicide bombers ( we actually stop a lot of them) and barricaded enemy. Definitely here to stay.

10) The M24 sniper rifle: Thumbs up. Mostly in .308 but some in 300 win mag. Heavily modified Remington 700's. Great performance. Snipers have been used heavily to great effect. Rumor has it that a Marine
sniper on his third tour in Anbar province has actually exceeded Carlos Hathcock's record
for confirmed kills with OVER 100.

11) The new body armor: Thumbs up. Relatively light at approx. 6 lbs. and can reliably be expected to soak up small shrapnel and even will stop an AK-47 round. The bad news: Hot as shit to wear, almost unbearable in the summer heat (which averages over 120 degrees). Also, the enemy now goes for head shots whenever possible. All the bullshit about the "old" body armor
making our guys vulnerable to the IED's was a non-starter. The IED explosions are enormous and body armor doesn't make any difference at all in most cases.

12) Night Vision and Infrared Equipment: Thumbs way up. Spectacular performance. Our guys see in the dark and own the night, period. Very little enemy action after evening prayers. More and more enemy being
whacked at night during movement by our hunter-killer teams. We've all seen the videos.

13) Lights: Thumbs up. Most of the weapon mounted and personal lights are Surefire's, and the troops love 'em. Invaluable for night urban operations. Jordan carried a $34 Surefire G2 on a neck lanyard and loved it.

I cant help but notice that most of the good fighting weapons and ordnance are 50 or more years old!!!!!!!!! With all our technology, it's the WWII
and Vietnam era weapons that everybody wants!!!! The infantry fighting is frequent, up close and brutal. No quarter is given or shown.

Bad guy weapons:

1) Mostly AK47's The entire country is an arsenal. Works better in the desert than the M16 and the .308 Russian round kills reliably.

PKM belt fed light machine guns are also common and effective. Luckily, the enemy mostly shoots like shit. Undisciplined "spray and pray" type fire.
However, they are seeing more and more precision weapons, especially sniper rifles. (Iran, again) Fun fact: Captured enemy have apparently marveled at the
marksmanship of our guys and how hard they fight. They are apparently told in Jihad school that the Americans rely solely on technology, and can be
easily beaten in close quarters combat for their lack of toughness. Let's just say they know better now.

2) The RPG: Probably the infantry weapon most feared by our guys. Simple, reliable and as common as dogshit. The enemy responded to our up-armored
humvees by aiming at the windshields, often at point blank range. Still killing a lot of our guys.

3) The IED: The biggest killer of all. Can be anything from old Soviet anti-armor mines to jury rigged artillery shells. A lot found in Jordan's
area were in abandoned cars. The enemy would take 2 or 3 155mm artillery shells and wire them together. Most were detonated by cell phone, and the
explosions are enormous. You're not safe in any vehicle, even an M1 tank. Driving is by far the most dangerous thing our guys do over there. Lately,
they are much more sophisticated "shape charges" (Iranian) specifically designed to penetrate armor. Fact: Most of the ready made IED's are supplied by Iran, who is also providing terrorists (Hezbollah
types) to train the insurgents in their use and tactics. That's why the attacks have been so deadly lately. Their concealment methods are ingenious,
the latest being shape charges in Styrofoam containers spray painted to look like the cinderblocks that litter all Iraqi roads. We find about 40%
before they detonate, and the bomb disposal guys are unsung heroes of this war.

4) Mortars and rockets: Very prevalent. The soviet era 122mm rockets (with an 18km range) are becoming more prevalent. One of Jordan's NCO's lost a leg to one. These weapons cause a lot of damage "inside
the wire". Jordan's base was hit almost daily his entire time there by mortar and rocket fire, often at night to disrupt sleep patterns and cause fatigue (It
did). More of a psychological weapon than anything else. The enemy mortar teams would jump out of vehicles, fire a few rounds, and then haul ass in a
matter of seconds.

5) Bad guy technology: Simple yet effective. Most
> communication is by cell and satellite phones, and also by email on laptops. They use handheld
GPS units for navigation and "Google earth" for overhead views of our positions. Their weapons are good, if not fancy, and prevalent. Their explosives and bomb technology is TOP OF THE LINE. Night vision
is rare. They are very careless with their equipment and the captured GPS units and laptops are treasure troves of Intel when captured.

Who are the bad guys?:
Most of the carnage is caused by the Zarqawi Al Qaeda group. They operate mostly in Anbar province (Fallujah and Ramadi). These are mostly "foreigners", non-Iraqi Sunni Arab Jihadists from all over the Muslim world
(and Europe). Most enter Iraq through Syria (with, of course, the knowledge and complicity of the Syrian govt.) , and then travel down the "rat line"
which is the trail of towns along the Euphrates River that we've been hitting hard for the last few months. Some are virtually untrained young Jihadists that often end up as suicide bombers or in "sacrifice
squads". Most, however, are hard core terrorists from all the usual suspects (Al Qaeda, Hezbollah, Hamas etc.) These are the guys running around murdering
civilians en masse and cutting heads off. The Chechens (many of whom are Caucasian), are supposedly the most ruthless and the best fighters. (they
have been fighting the Russians for years). In the Baghdad area and south, most of the insurgents are Iranian inspired (and led) Iraqi Shiites. The Iranian Shiia have been very adept at infiltrating the Iraqi
local govt.'s, the police forces and the Army. The have had a massive spy and agitator network there since the Iran-Iraq war in the early 80's. Most
of the Saddam loyalists were killed, captured or gave up long ago.

Bad Guy Tactics:
When they are engaged on an infantry level they get their asses kicked every time. Brave, but stupid. Suicidal Banzai-type charges were very common
earlier in the war and still occur. They will literally sacrifice 8-10 man teams in suicide squads by sending them screaming and firing Ak's and RPG's
directly at our bases just to probe the defenses. They get mowed down like grass every time. ( see the M2 and M240 above). Jordan's base was hit like
this often. When engaged, they have a tendency to flee to the same building, probably for what they think will be a glorious last stand.
Instead, we call in air and that's the end of that more often than not.
These hole-ups are referred to as Alpha Whiskey Romeo's (Allah's Waiting Room). We have the laser guided ground-air thing down to a science. The
fast mover's, mostly Navy/Marine F-18's, are taking an ever increasing toll on the enemy. When caught out in the open, the helicopter gunships and AC-130 Spectre gunships cut them to ribbons with cannon and rocket fire, especially at night. Interestingly, artillery is hardly used at all. Fun fact: The enemy death toll is supposedly between 45-50 thousand. That is
why we're seeing less and less infantry attacks and more IED, suicide bomber shit. The new strategy is simple: attrition.

The insurgent tactic most frustrating is their use of civilian non-combatants as cover. They know we do all we can to avoid civilian casualties and therefore schools, hospitals and (especially) Mosques are
locations where they meet, stage for attacks, cache weapons and ammo and flee to when engaged. They have absolutely no regard whatsoever for civilian casualties. They will terrorize locals and murder
without hesitation anyone believed to be sympathetic to the Americans or the new Iraqi govt. Kidnapping of family members (especially children) is common to
influence people they are trying to influence but cant reach, such as local govt. officials, clerics, tribal leaders, etc.).

The first thing our guys are told is "don't get captured". They know that if captured they will be tortured and beheaded on the internet. Zarqawi
openly offers bounties for anyone who brings him a live American serviceman. This motivates the criminal element who otherwise don't give a shit about the war. A lot of the beheading victims were actually kidnapped by common criminals and sold to Zarqawi. As such, for our guys, every fight is to the death. Surrender is not an option.

The Iraqi's are a mixed bag. Some fight well, others aren't worth a shit. Most do okay with American support. Finding leaders is hard, but they are
getting better. It is widely viewed that Zarqawi's use of suicide bombers, en masse, against the civilian population was a serious tactical mistake.
Many Iraqi's were galvanized and the caliber of recruits in the Army and the police forces went up, along with their motivation. It also led to an
exponential increase in good intel because the Iraqi's are sick of the insurgent attacks against civilians. The Kurds are solidly pro-American and fearless fighters.

According to Jordan, morale among our guys is very high. They not only believe they are winning, but that they are winning decisively. They are stunned and dismayed by what they see in the American press, whom
they almost universally view as against them. The embedded reporters are despised and distrusted. They are inflicting casualties at a rate of 20-1 and then see shit like "Are we losing in Iraq" on TV and the
print media.
For the most part, they are satisfied with their equipment, food and leadership. Bottom line though, and they all say this, there are not enough guys there to drive the final stake through the heart of the
insurgency, primarily because there aren't enough troops in-theater to shut down the borders with Iran and Syria. The Iranians and the Syrians just cant stand the thought of Iraq being an American ally (with, of course, permanent US bases there).
>
> Anyway guys, that's it, hope you found it interesting, I sure did.<<

tomder55 answered on 11/29/05:

That is simular to reports I've seen and heard .Interesting to note is that some Iraqi officials are complaining that recruits are using AK-47s in training .They think that they should be getting American armaments but everything I've read says that the AKs are quite capable in that environment.

This information from last months "Report on Measuring Stability and Security in Iraq."

Here are the key parts about Iraqi security force readiness :

-- A continued increase in the number of Iraqi units able to take the lead in combat operations against the insurgency. There are now 88 Iraqi army and special operations battalions conducting combat operations against the enemy -- an increase of nine since the July report. Of the 88 operational units, 36 are assessed as being "in the lead" or fully independent -- a 50 percent increase over units at these levels of readiness in the July report. There are 28 Special Police Force battalions capable of combat operations -- an increase of 13 since the last report.

-- Progress of Iraqi units in assuming responsibility for the battle space. Since the last report, Iraqi forces have taken responsibility for security in several areas of Iraq and now have the lead in one Iraqi province, roughly 87 square miles of Baghdad and over 450 square miles in other provinces.

-- A continued increase in the number of units and individuals trained, equipped, and formed into operational status. More than 87,000 soldiers, sailors, and airmen have now been trained and equipped -- an increase of 10,000 since the last report. A total of 68,800 police have been trained and equipped, an increase of 5,500 since the last reporting period. These work alongside 35,500 other Ministry of Interior forces. Overall, this represents a 12 percent increase in Ministry of Defense and Ministry of Interior forces trained and equipped for counter-insurgency operations since July 2005.

As you can see there is some real progress being made in many areas that have not received proper political and media attention. More astute and/or opportunistic Democrats like Barak Obama and Joe Biden have called for gradual troop reductions next year knowing very well that is the intention of the Administration anyway. They will claim in the fall elections next year that it was the pressure they brought to bear on the White House that achieved that aim .But progress is coming from the planning after the disasterous Paul Bremer CPA was removed from the scene and is being done by the troops every day .

Itsdb rated this answer Excellent or Above Average Answer
kindj rated this answer Excellent or Above Average Answer

Question/Answer
excon asked on 11/28/05 - Iraq - redux


Hello experts:

Im worried. More worried than I have been for a long long time.

Iraq is NOT Vietnam. We lost there - nothing happened. If we lose in Iraq, a lot will happen, and ALL of it bad. Because Bush has not outlined what VICTORY will look like (other than the democratization of Iraq - which aint gonna happen), ANY outcome other than that will be viewed as (and will be) a loss.

Because he wouldnt win (and it was a choice), a quagmire has, indeed, developed. We cant leave - and we cant stay.

Any of you wishful thinkers get a grip yet??? Probably not.

excon

tomder55 answered on 11/28/05:

All I can do is continue to report the progress that gererally gets no coverage .I would say this in itself is an exact opposite of Vietnam where the official military news tried to distort progress ;in Iraq progress gets downplayed .

The Administration reversed the disasterous Paul Bremer CPA decision to disband the Iraqi Army ;especially the junior officers corp. (officers ranking from lieutenant to major are being screened and vetted). So far 2,662 officers have joined .So far about 200,000 Iraqi security forces have been trained .The military is training and deploying soldiers and police at a rate of nearly 1,000 per week.On Oct. 28,transfere of security responsibility for the Diyala province was handed over to the Iraq Army.17 bases have been turned over to the Iraq Army .They control and patrol most of Baghdad.

The real failure in Vietnam was that we did not truly invest in a democratic South Vietnam and that we abandoned our commitement to them when they were invaded from the North after Watergate. That cannot be said in Iraq we have actively supported self determination .

In Iraq the so called insurgency (which is nothing more than former Baathists desperately trying to cling to power )is no more potent than the Vietcong would've been without the North Vietnam regulars and regular steady material support from 2 rival superpowers .

What the Democrats know is that there will be some draw down of coalition troops sometime in the next year ,and they are positioning themselves to take credit for it .

excon rated this answer Excellent or Above Average Answer

Question/Answer
powderpuff asked on 11/25/05 - TV license?

TV Licensing inspectors have caught almost 350,000 people watching TV without a licence so far this year.

People who fail to get a TV licence are pointing the finger of blame at pets in outlandish bids to avoid punishment

TV Licensing spokeswoman Jessica Ray said: "Claiming the TV is only ever watched by the family dog is not an excuse for not being properly covered by a TV licence.

"Being caught red-handed by TV Licensing isn't most people's idea of a funny situation......

...
but I think it is ;D

You can read the full article here:

http://news.bbc.co.uk/1/hi/uk/4469422.stm

.... and all along I thought a requirement for a Fishing License was over the top.

tomder55 answered on 11/26/05:

"Using a television without an appropriate licence is a criminal offence. Every day we catch an average of 1,200 people using a TV without a licence. There is no valid excuse for using a television and not having a TV Licence, but some people still try - sometimes with the most ridiculous stories ever heard. Our detection equipment will track down your TV. The fact that our enquiry officers are now so well equipped with the latest technology means that there is virtually no way to avoid detection."
-- from the official website of the British Television Licensing Authority



The Brits have no choice but to pay it whether they actually watch it or not .(when they are 75 years old the fee is waved ....whoooop whooop !!) One cannot simply refuse this without appearing to be dishonest .To the TVLA there are licence-payers and licence-dodgers and the non-viewer is treated as a suspected licence-dodger. Doesn't everybody watch and own a television?

Given the poor quality and content of their programing I would be tempted to be a non-viewer also.They only get 5 channels and only the BBC channels are totally commercial free .Owning a TV set for watching VHS and DVDs does not require a TV licence but the gvt. argues that those sets have the potential of receiving a signal, so a tax is levied.

But the licensing fee,about $200 /year( The cost goes up each year ) ,is certainly cheaper than the monthly cable bill most people in the US pays for 300 channels of garbage .

In the US they already tax us to death .There is federal income tax, state income tax, state sales tax, city wage tax, home owners tax, a monthly tax on using a telephone line, special alcohol and cigarette taxes,gasoline tax, outrageous hotel taxes and the list goes on and on.(yet there are some who claim we are undertaxed ).I'm suprised that the Feds. haven't tapped into this revenue pool also. The US has 280 million people and 219 million television sets.Perhaps its' is our 2nd Amendment protections that deters them .

paraclete rated this answer Excellent or Above Average Answer
powderpuff rated this answer Excellent or Above Average Answer
sapphire630 rated this answer Excellent or Above Average Answer

Question/Answer
Choux asked on 11/24/05 - Saddam Anti AlQuaeda?

By Murray Waas, special to National Journal
National Journal Group Inc.
Tuesday, Nov. 22, 2005

""Ten days after the September 11, 2001, terrorist attacks on the World Trade Center and the Pentagon, President Bush was told in a highly classified briefing that the U.S. intelligence community had no evidence linking the Iraqi regime of Saddam Hussein to the attacks and that there was scant credible evidence that Iraq had any significant collaborative ties with Al Qaeda, according to government records and current and former officials with firsthand knowledge of the matter.


The administration has refused to provide the Sept. 21 President's Daily Brief, even on a classified basis, and won't say anything more about it other than to acknowledge that it exists.




The information was provided to Bush on September 21, 2001 during the "President's Daily Brief," a 30- to 45-minute early-morning national security briefing. Information for PDBs has routinely been derived from electronic intercepts, human agents, and reports from foreign intelligence services, as well as more mundane sources such as news reports and public statements by foreign leaders.

One of the more intriguing things that Bush was told during the briefing was that the few credible reports of contacts between Iraq and Al Qaeda involved attempts by Saddam Hussein to monitor the terrorist group. Saddam viewed Al Qaeda as well as other theocratic radical Islamist organizations as a potential threat to his secular regime. At one point, analysts believed, Saddam considered infiltrating the ranks of Al Qaeda with Iraqi nationals or even Iraqi intelligence operatives to learn more about its inner workings, according to records and sources.""


Comments?

tomder55 answered on 11/25/05:

I do not know what was in the daily brief but it just is not true . I find it funny that the critics of the administration point to a number of daily briefs he received that concluded Saddam still had an active WMD program as bogus but evidently there is one gem of a PDB that is God's own truth to them .But ;this one apparently is the most incorrect PDB of the group .

The Senate Intelligence Report of July 7,2004 notes the following about the intelligence that was presented to the administration :

Despite four decades of intelligence reporting on Iraq, there was little useful intelligence collected that helped analysts determine the Iraqi regime's possible links to al Qaeda. . . . The Central Intelligence Agency (CIA) did not have a focused human intelligence (HUMINT) collection strategy targeting Iraq's links to terrorism until 2002. The CIA had no [redacted] sources on the ground in Iraq reporting specifically on terrorism.

It was not just reporting on Iraq that was inadequate. "The CIA had no [redacted] credible reporting on the leadership of either the Iraqi regime or al Qaeda, which would have enabled it to better define a cooperative relationship, if any did in fact exist."


From what I've read ,and I've researched this alot ; the Bush administration has underplayed the connections between Saddam Hussein and al Qaeda.[for now I will deal with the links to al Qaeda and not the direct links to 9-11 ;the first WTC bombing ,or the possible connection to the Oklahoma City bombing] .

Feb.23,1998 ;just days after President Clinton addressed the nation about Iraq's ties to terrorism by daying ;: "...meeting the threat posed by Saddam Hussein is important to our security in the new era we are entering." He warned about the threats from the "predators of the 21st century," rogue states working with terrorist groups. "There is no more clear example of this threat than Saddam Hussein's Iraq." ;
Osama bin Laden and Ayman al Zawahiri announced the formation of al Qaeda. In the fatwa they decried the presence of U.S. troops on the Arabian Peninsula. They protested the "great devastation inflicted on the Iraqi people by the crusader-Zionist alliance." They cited American support for Israel and surmised that the United States sought to distract world attention from the killing of Muslims in Jerusalem. To support this claim, the fatwa turned once again to Iraq: "The best proof of this is their eagerness to destroy Iraq, the strongest neighboring Arab state."

The CIA has confirmed in interviews with detainees and informants that Ayman al-Zawahiri, met with Iraqi intelligence in Baghdad in 1992 and 1998. The Agency has "irrefutable evidence" that the Iraqi regime paid Zawahiri $300,000 in 1998, around the time his Islamic Jihad was merging with al Qaeda.

Oct. 7 ,2002 ,George Tenet sent a lettet to the Senate Intelligence Committee. Here are the key points he made about the connection :

Our understanding of the relationship between Iraq and al-Qa'ida is evolving and is based on sources of varying reliability. Some of the information we have received comes from detainees, including some of high rank.

We have solid reporting of senior level contacts between Iraq and al-Qa'ida going back a decade.

Credible information indicates that Iraq and al-Qa'ida have discussed safe haven and reciprocal nonaggression.

Since Operation Enduring Freedom, we have solid evidence of the presence in Iraq of al-Qa'ida members, including some that have been in Baghdad.

We have credible reporting that al-Qa'ida leaders sought contacts in Iraq who could help them acquire W.M.D. capabilities. The reporting also stated that Iraq has provided training to al-Qa'ida members in the areas of poisons and gases and making conventional bombs.

Iraq's increasing support to extremist Palestinians coupled with growing indications of a relationship with al-Qa'ida, suggest that Baghdad's links to terrorists will increase, even absent U.S. military action.


This came from the CIA with its history of institutional skepticism about the links.They thought like Waas and the new conventional wisdom that Saddam Hussein's secular regime would not collaborate with Islamic fundamentalists like bin Laden.


I would also note that the Clinton Administration learned of the pact forged between al-Qaeda and Saddam and made it a key part of the 1998 indictment against bin-Laden .

"Al Qaeda reached an understanding with the government of Iraq that al Qaeda would not work against that government and that on particular projects, specifically including weapons development, al Qaeda would work cooperatively with the Government of Iraq."

As the war progressed we've learned even more .

Abu Abdullah al-Iraqi, an al Qaeda WMD specialist was sent by bin Laden to Iraq several times to seek WMD training, and possibly weapons, from the Iraqi regime.

There was a meetings in Baghdad between al Qaeda leaders and Uday Hussein in April 1998, at a birthday celebration for Saddam.

I fault the Bush Administration for not making the links more apparent to the public. To me there was always a more compelling justification to the war with the connection than the case for WMD ,even though the President made the important link between rogue states with their hands on WMD and their ability to transfer them to terrorists. The Bush Doctrine of premption correctly argues that it is too late to wait for the transferto happen .

Some of the obvious evidence that Bush did not use was the Salam Pak camp for training terrorists how to hijack airplanes. U.N. inspectors confirmed the camp's existence, including the presence of a Boeing 707. Sabah Khodada, a captain in the Iraqi Army, worked at Salman Pak. In October 2001, he told PBS's "Frontline" about what went on there. "Training is majorly on terrorism. They would be trained on assassinations, kidnapping, hijacking of airplanes, hijacking of buses, public buses, hijacking of trains and all other kinds of operations related to terrorism. . . . All this training is directly toward attacking American targets, and American interests."

The administration has not done a good enough job 'connection the dots' about the Mohammed Atta meeting with Iraqi Intelligence officers in Prague between April 4 and April 11, 2001. The Czech Republic has not changed the story ,at least 5 different high-ranking Czech officials have publicly confirmed that Mohammed Atta met with Ahmed Khalil Ibrahim al-Ani, an Iraqi intelligence officer working at the Iraqi embassy.

Murray Waas needs to do a better job of research . The information is available to anyone willing to read beyond the NY Slimes.The truth is that after he was repelled from Kuwait Saddam began to make the journey to religious fanatacism himself .Much of the oil for food money was used to build lavish mosques .His later speeches were peppered with references to Muslim scripture.He even had a koran written in his own blood. Perhaps he was playing to the majority ,but more likely he saw Islam as his best hope of keeping power. He was bordered by a Shia Fundamentalist Iran on one side and a Wahabist Saudia Arabia on his other flank.His ties to Palestinian terrorists were well known. He just determined that it would be convienient to cooperate with al Qaeda rather than to have bin Laden as an enemy also.





Choux rated this answer Excellent or Above Average Answer

Question/Answer
Choux asked on 11/23/05 - Bush Spins Facts

Harris Poll as viewed on the on line Wall Street Journal site::

""A majority of U.S. adults believe the Bush administration generally misleads the public on current issues, while fewer than a third of Americans believe the information provided by the administration is generally accurate, the latest Harris Interactive poll finds.

While the telephone survey of 1,011 U.S. adults indicates about 64% of Americans believe the Bush administration "generally misleads the American public on current issues to achieve its own ends,"...""



Is there any way for Bush to overcome the public perception that he is a spinner and liar???

tomder55 answered on 11/24/05:

Despite acrimony at home, the politics of two national elections and a third on the horizon, and the slander of war crimes and incompetence, those on the battlefield of Iraq have almost pulled off the unthinkable the restructuring of the politics of the Middle East in less than three years.

read the rest of this Victor Davis Hanson article here

btw ;since we are using meaningless polling numbers which are reflections of brief moments in time .I will add one .A new survey found that 56 percent of the public thinks that efforts to establish a stable democracy in Iraq will succeed according to the latest Pew Research Center survey.

Choux rated this answer Excellent or Above Average Answer
purplewings rated this answer Excellent or Above Average Answer

Question/Answer
excon asked on 11/22/05 - My favorite activist judge


Hello experts:

How many times have you heard the phrase, well, if you dont break the law, then you have nothing to worry about? When I discuss with right wingers, the inroads law enforcement has made on the Constitution, this is their retort - their only retort.

So, what are they saying? The meaning is clear. Im a good American. Good Americans dont need the Constitution, only bad Americans do. Why should I fight to help bad Americans?

Unfortunately, Judge Alito thinks the same thing.

These people kinda miss what our country stands for. If, in the year 2005, they dont know why they should support the rights of the accused in this country, there is nothing - absolutely NOTHING you or I could say that would change their mind.

Why is that? Because one side is ignorant of history.

excon

tomder55 answered on 11/23/05:

Prisoner and accused rights have expanded consistently since the founders framed the Constitution . The founders ran the country after the ratification through the first 6 Presidencies. During that time the founders passed such acts as sedition laws and used the military to quell domestic disturbances.
Due process procedures and laws have consistently expanded in the last 200 years so that Americans who are accused enjoy greater freedoms than were practiced by the very founders who forged the Constitution.Example ;the Bill of Rights applied only to the federal government until the 1920s but most criminal cases were State matters .In most states, there were few procedural rights, and even the ones that existed were not stringently enforced. Today the courts have nationalized procedures. They are already expansive beyond the founders intent .

Beyond that I would need to see the specific decision of Alito's that you find objectionable .There have been one or two that have been reversed so I guess his record is not spotless ;at least in the eyes of the higher court but in general he shares my philosophy of judicial restraint.

excon rated this answer Excellent or Above Average Answer

Question/Answer
paraclete asked on 11/22/05 - surely he isn't this dumb?

Bush had al-Jazeera attack in mind, says paper


November 23, 2005

The US President, George Bush, planned to bomb the pan-Arab television broadcaster al-Jazeera, according to a British newspaper that cited a Downing Street memo marked "Top secret".

The five-page transcript of a conversation between Mr Bush and the British Prime Minister, Tony Blair, published in the Daily Mirror yesterday, reveals that Mr Blair talked Mr Bush out of launching a military strike on the station, unnamed sources told the newspaper.

The transcript of the pair's talks during Mr Blair's visit to Washington on April 16 last year allegedly shows Mr Bush wanted to attack the satellite channel's headquarters.

Mr Blair allegedly feared such a strike, in the business district of Doha, the capital of Qatar, a key western ally in the Persian Gulf, would spark revenge attacks.

The Mirror quoted an unnamed British government official as saying Mr Bush's threat was "humorous, not serious".

Al-Jazeera's perspectives on the war in Iraq have drawn criticism from Washington since the US-led invasion in March 2003.

The station has broadcast messages by Osama bin Laden and the beheadings of Western hostages by insurgents in Iraq, as well as footage of dead coalition servicemen and Iraqi civilians killed in fighting.

A source told the Mirror: "The memo is explosive and hugely damaging to Bush.

"He made clear he wanted to bomb al-Jazeera in Qatar and elsewhere. Blair replied that would cause a big problem.

"There's no doubt what Bush wanted to do - and no doubt Blair didn't want him to do it."

Another source said: "Bush was deadly serious, as was Blair. That much is absolutely clear from the language used by both men."

A spokesman for Mr Blair's Downing Street office said: "We have got nothing to say about this story. We don't comment on leaked documents."

The Mirror said the memo turned up in the office of the then British MP Tony Clarke, a member of Mr Blair's Labour Party, in May last year. A civil servant, David Keogh, is accused under the Official Secrets Act of handing it to Mr Clarke's former researcher Leo O'Connor. Both are to appear at Bow Street Magistrates Court in central London next week.

Mr Clarke returned the memo to Downing Street. He said O'Connor had behaved "perfectly correctly". He told Britain's domestic Press Association news agency that O'Connor had done "exactly the right thing" in bringing it to his attention.

The Mirror said such a strike would have been "the most spectacular foreign policy disaster since the Iraq war itself".

The newspaper said that the memo "casts fresh doubt on claims that other attacks on al-Jazeera were accidents". It cited the 2001 direct hit on the channel's Kabul office.

Agence France-Presse

tomder55 answered on 11/23/05:

I probably would've just jammed the transmission but I can see why Bush wanted to smoke them . I wanted them taken out after they shamefully played and replayed Dan Berg's beheading.

excon rated this answer Excellent or Above Average Answer
paraclete rated this answer Excellent or Above Average Answer

Question/Answer
Choux asked on 11/23/05 - Shock and Awe? or Fear and Loathing.

""Back in the year 2000, I believed almost without thinking about it that the US was a "superpower", the only "superpower" in the world. Maybe it was true and maybe it wasn't, but there was a lot of money around, Americans were pretty prosperous, and most people around the world had a benign view of the US.
Maybe the clearest sign of our "superpower" status was that the right wing and the press could beat up on Bill Clinton with absolutely no effect on US power or the perception of US power. Beating up on Bill Clinton was a kind of parlor game that the participants cared about, but was in the end of no international import. The most surprising thing, then, about the last five years is how quickly and absolutely the US has ceased to be a superpower.

We are a country that can no longer pay our bills, no longer wage an effective military action, and no longer protect our citizens from disaster. And it doesn't matter what fiscal responsibility individuals show, what bravery individual soldiers show, or what generosity individual Americans show. As a nation-as a geopolitical entity-we have been stripped of all of our superpowers and many of our powers, and it has been done quickly and efficiently, in the name of blind patriotism, by Bush, Cheney, Rumsfeld, Rice, and their neocon advisors. The very powers that these people thought they were going to enjoy exercising have slipped out of their grasp. It's laughable now to remember the name of the campaign against Baghdad, "Shock and Awe". No one in Iraq feels any "shock and awe" toward the US presence there any longer. "Fear and Loathing" is more like it.

Whether or not the administration and the press know that the powers are gone doesn't matter. They are. And they aren't coming back, because the last twenty-five years of Republican free market political devolution have resulted in a completely different country from the one that in the course of the 20th century became a superpower. Bush and Cheney have provided the final, telling blows to American power, but actions of the corporatocracy laid the ground work. There is nothing left in the US of real substance. The only thing that remains is ego, bullying, and public relations. The question thoughtful Americans are going to have to answer eventually is one they should be thinking about
now-when our superpowers are gone, what are we and what do we want to be?

As everyone knows, the multinational corporation has jealously preserved its right to pay its workers as little as possible-to put its factories wherever wages were lowest, and to exploit the natural resources of every corner of the globe while paying as little to the locals who ostensibly "owned" them as they could, preferably nothing This is such a precious idea for capitalists that when a company, like Costco, for example, operates on a different, more humane model, they become resentful and vengeful-it is implied that the power of Wall Street will be brought to bear on such a renegade business model-customers and workers must never come before shareholders. Nor must the public safety be considered. All regulations that protect the environment or even those who purchase some item, are to be as much as possible prohibited, or at least flouted with impunity. To these corporate types, the public safety of one's own fellow citizens is as much a matter of indifference as the public safety of people ten thousand miles away.

What most Americans, indeed, most people, normally think of as desirable, such as stable communities with histories, jobs, and a middle class, is not what the corporations have shown themselves to care about. They do not care about the actual substance of the US, a set of geographical areas with a varied population of human beings. The taxpayers present themselves to the corporation much as consumers do-a bunch of suckers to be fooled and robbed for the sake of shareholder profit. The way you rob customers is by dressing up something cheap and worthless to look desirable. The way you rob taxpayers is by constantly challenging them to defend their patriotism and their religion. The average American has a long history of being reflexively xenophobic, so getting him worked up about enemies from abroad, especially dark-skinned ones, has always been an especially effective way of distracting him while you pick his pocket. But I say, let me be exactly as patriotic as some corporate executive who has outsourced his American workforce to India, bought homes around the world, made sure his children don't have to fight in American wars, and banked his money offshore so that he can avoid paying taxes.

In exchange for the towns that Big Ag has depopulated, the cities that Big Manufacturing has hollowed out, the healthcare that Big Pharma has helped destroy, the environment that Big Chemical has contaminated, and the public school system that the corporate tax giveaways have hobbled,****** what has the average American gotten? Only the sense of grandiosity and self-righteousness that come from thinking of oneself as part of a "superpower."****** by==Jane Smiley


Comments...

tomder55 answered on 11/23/05:

I can't tell if Smiley is hand wringing or celebrating America's decline.

America never had the heart or the will to be the lone superpower. We are both militarily and even more so economically a powerhouse and we do not use that to our advantage . We have a large segment of our population like Jane Smiley ,and former President Bill Clinton who are instintivly uncomfortable with the idea of America being a lone super power ,and in the former President's case took active steps to equalize the playing field. [refer back to the reasons I say he should've been impeached.Clinton sold the Chinese all the hardware and software they need to develop first-strike nuclear ICBM capability
].

Did the United States keep it's military strength to pre-Cold War levels ? No ;we demilitarized big time and still have what is considered the premiere force in the world . Do we use it to our advantage say like former empires like Rome ? Hardly.

In WWII we smashed 2 large industrial nations but lost more than 400,000 KIA in the process. Today ;we pussy foot in a low level conflict ;give back territory captured only to fight for it again ;and cry quagmire when the 2000 casualty threshold is reached .

Do you think in a conflict with China they would care how many of their troops were killed ? No they wouldn't . Already our South East Asian allies are re-evaluating their relationship with us based on the new found belief ;and justifiably in my view ;that the US will not do what is necessary to guarantee their security .

This from the latest Insight Magazine (subscription required):

Tokyo Governor Shintaro Ishihara has gone public, warning that the United States would lose any war with China.

"In any case, if tension between the United States and China heightens, if each side pulls the trigger, though it may not be stretched to nuclear weapons, and the wider hostilities expand, I believe America cannot win as it has a civic society that must adhere to the value of respecting lives," The governor said the U.S. military could not counter a wave of millions of Chinese soldiers prepared to die in any onslaught against U.S. forces. After 2,000 casualties, he said, the U.S. military would be forced to withdraw.

"Therefore, we need to consider other means to counter China," he said. "The step we should be taking against China, I believe, is economic containment."


To continue to excel as a major economic force securing our energy supply is important . Have we taken any real measures to do so ? How many new refineries ,nuclear plants have been built in the last 30 years ? How much exploratory drilling has been done ? How much have we invested in new technolgies ? I guarantee the Chinese are . Just their attempt to purchase Unocol proves that they understand the relationship of energy to a robust economy .

The U.S. has spent heavily to bring the world forward and past a world recession in the last 5 years .Yes its a debt.Will Smiley give us credit for that or the economic expansion we are currently experiencing ? I doubt it . I would not write off the US economy yet. In the 80s it was the Japanese that were going to overtake us ...remember ?

And also consider how and why the current Chinese economic expansion is occuring . Yes ;they've adopted those evil exploitive capitalists market strategies. But I also predict it will implode on itself if the government does not open up and allow the people freedoms they are not inclined to give them now. Talk about an exploitive culture ! The rich party members drive the cars while the rest of the poor folks sweep the roads clean .

Large segments of our population like Smiley do not believe in the superiority of our culture . This week Chris Matthews as an example made a speech where he said the following amazing statement [refering to America post 9-11] : "If we stop trying to figure out the other side, we've given up. The person on the other side is not evil -- they just have a different perspective." Think the Romans ever cared about the Hun's perspective ?



Choux rated this answer Excellent or Above Average Answer
ETWolverine rated this answer Excellent or Above Average Answer

Question/Answer
Choux asked on 11/23/05 - Shock and Awe? or Fear and Loathing.

""Back in the year 2000, I believed almost without thinking about it that the US was a "superpower", the only "superpower" in the world. Maybe it was true and maybe it wasn't, but there was a lot of money around, Americans were pretty prosperous, and most people around the world had a benign view of the US.
Maybe the clearest sign of our "superpower" status was that the right wing and the press could beat up on Bill Clinton with absolutely no effect on US power or the perception of US power. Beating up on Bill Clinton was a kind of parlor game that the participants cared about, but was in the end of no international import. The most surprising thing, then, about the last five years is how quickly and absolutely the US has ceased to be a superpower.

We are a country that can no longer pay our bills, no longer wage an effective military action, and no longer protect our citizens from disaster. And it doesn't matter what fiscal responsibility individuals show, what bravery individual soldiers show, or what generosity individual Americans show. As a nation-as a geopolitical entity-we have been stripped of all of our superpowers and many of our powers, and it has been done quickly and efficiently, in the name of blind patriotism, by Bush, Cheney, Rumsfeld, Rice, and their neocon advisors. The very powers that these people thought they were going to enjoy exercising have slipped out of their grasp. It's laughable now to remember the name of the campaign against Baghdad, "Shock and Awe". No one in Iraq feels any "shock and awe" toward the US presence there any longer. "Fear and Loathing" is more like it.

Whether or not the administration and the press know that the powers are gone doesn't matter. They are. And they aren't coming back, because the last twenty-five years of Republican free market political devolution have resulted in a completely different country from the one that in the course of the 20th century became a superpower. Bush and Cheney have provided the final, telling blows to American power, but actions of the corporatocracy laid the ground work. There is nothing left in the US of real substance. The only thing that remains is ego, bullying, and public relations. The question thoughtful Americans are going to have to answer eventually is one they should be thinking about
now-when our superpowers are gone, what are we and what do we want to be?

As everyone knows, the multinational corporation has jealously preserved its right to pay its workers as little as possible-to put its factories wherever wages were lowest, and to exploit the natural resources of every corner of the globe while paying as little to the locals who ostensibly "owned" them as they could, preferably nothing This is such a precious idea for capitalists that when a company, like Costco, for example, operates on a different, more humane model, they become resentful and vengeful-it is implied that the power of Wall Street will be brought to bear on such a renegade business model-customers and workers must never come before shareholders. Nor must the public safety be considered. All regulations that protect the environment or even those who purchase some item, are to be as much as possible prohibited, or at least flouted with impunity. To these corporate types, the public safety of one's own fellow citizens is as much a matter of indifference as the public safety of people ten thousand miles away.

What most Americans, indeed, most people, normally think of as desirable, such as stable communities with histories, jobs, and a middle class, is not what the corporations have shown themselves to care about. They do not care about the actual substance of the US, a set of geographical areas with a varied population of human beings. The taxpayers present themselves to the corporation much as consumers do-a bunch of suckers to be fooled and robbed for the sake of shareholder profit. The way you rob customers is by dressing up something cheap and worthless to look desirable. The way you rob taxpayers is by constantly challenging them to defend their patriotism and their religion. The average American has a long history of being reflexively xenophobic, so getting him worked up about enemies from abroad, especially dark-skinned ones, has always been an especially effective way of distracting him while you pick his pocket. But I say, let me be exactly as patriotic as some corporate executive who has outsourced his American workforce to India, bought homes around the world, made sure his children don't have to fight in American wars, and banked his money offshore so that he can avoid paying taxes.

In exchange for the towns that Big Ag has depopulated, the cities that Big Manufacturing has hollowed out, the healthcare that Big Pharma has helped destroy, the environment that Big Chemical has contaminated, and the public school system that the corporate tax giveaways have hobbled,****** what has the average American gotten? Only the sense of grandiosity and self-righteousness that come from thinking of oneself as part of a "superpower."****** by==Jane Smiley


Comments...

tomder55 answered on 11/23/05:

I can't tell if Smiley is hand wringing or celebrating America's decline.

America never had the heart or the will to be the lone superpower. We are both militarily and even more so economically a powerhouse and we do not use that to our advantage . We have a large segment of our population like Jane Smiley ,and former President Bill Clinton who are instintivly uncomfortable with the idea of America being a lone super power ,and in the former President's case took active steps to equalize the playing field. [refer back to the reasons I say he should've been impeached.Clinton sold the Chinese all the hardware and software they need to develop first-strike nuclear ICBM capability
].

Did the United States keep it's military strength to pre-Cold War levels ? No ;we demilitarized big time and still have what is considered the premiere force in the world . Do we use it to our advantage say like former empires like Rome ? Hardly.

In WWII we smashed 2 large industrial nations but lost more than 400,000 KIA in the process. Today ;we pussy foot in a low level conflict ;give back territory captured only to fight for it again ;and cry quagmire when the 2000 casualty threshold is reached .

Do you think in a conflict with China they would care how many of their troops were killed ? No they wouldn't . Already our South East Asian allies are re-evaluating their relationship with us based on the new found belief ;and justifiably in my view ;that the US will not do what is necessary to guarantee their security .

This from the latest Insight Magazine (subscription required):

Tokyo Governor Shintaro Ishihara has gone public, warning that the United States would lose any war with China.

"In any case, if tension between the United States and China heightens, if each side pulls the trigger, though it may not be stretched to nuclear weapons, and the wider hostilities expand, I believe America cannot win as it has a civic society that must adhere to the value of respecting lives," The governor said the U.S. military could not counter a wave of millions of Chinese soldiers prepared to die in any onslaught against U.S. forces. After 2,000 casualties, he said, the U.S. military would be forced to withdraw.

"Therefore, we need to consider other means to counter China," he said. "The step we should be taking against China, I believe, is economic containment."


To continue to excel as a major economic force securing our energy supply is important . Have we taken any real measures to do so ? How many new refineries ,nuclear plants have been built in the last 30 years ? How much exploratory drilling has been done ? How much have we invested in new technolgies ? I guarantee the Chinese are . Just their attempt to purchase Unocol proves that they understand the relationship of energy to a robust economy .

The U.S. has spent heavily to bring the world forward and past a world recession in the last 5 years .Yes its a debt.Will Smiley give us credit for that or the economic expansion we are currently experiencing ? I doubt it . I would not write off the US economy yet. In the 80s it was the Japanese that were going to overtake us ...remember ?

And also consider how and why the current Chinese economic expansion is occuring . Yes ;they've adopted those evil exploitive capitalists market strategies. But I also predict it will implode on itself if the government does not open up and allow the people freedoms they are not inclined to give them now. Talk about an exploitive culture ! The rich party members drive the cars while the rest of the poor folks sweep the roads clean .

Large segments of our population like Smiley do not believe in the superiority of our culture . This week Chris Matthews as an example made a speech where he said the following amazing statement [refering to America post 9-11] : "If we stop trying to figure out the other side, we've given up. The person on the other side is not evil -- they just have a different perspective." Think the Romans ever cared about the Hun's perspective ?



Choux rated this answer Excellent or Above Average Answer
ETWolverine rated this answer Excellent or Above Average Answer
purplewings rated this answer Excellent or Above Average Answer

Question/Answer
excon asked on 11/21/05 - Fences??? On the border??? You've got to be kidding!!


Hello experts:

Boy, I just dont see how we can keep making the same mistakes over and over again. Now, Im just an exconvict, but it seems to me that changing the immigration law would be easier, cheaper and infinitely more effective than building a fence.

Consider for a minute, that if our immigration policy worked, there would be more legal immigrants applying for jobs than illegal ones. Now, we don't have ANY legals applying for "those" kinds of jobs. So, instead of building a fence to keep them out, we should open our doors, and let in those who we want. Im not too bright, but we do want dishwashers, dont we?

Were we to do that, the dishwasher at your favorite restaurant wouldnt have to worry about being busted by the INS. He wouldnt have to worry about not paying taxes. He wouldnt have to worry about armed vigilantes at the border. He wouldnt have to worry about the people smugglers that hed have to pay $1,000s for the privilege of smothering in a trailer with 100s of other people. He wouldnt have to worry about dying in the dessert. He wouldnt have to worry about unethical employers who run sweatshops because illegals cant snitch.

The other advantage of a policy like that would be that the guys who are THEN illegally crossing the border are most likely terrorists - not a leafblowers.

excon

tomder55 answered on 11/21/05:

Why is Israel building a fence ? Egypt, which criticized Israel's decision to build a security fence, is building one of its own around the tourist hot-spot of Sharm el-Sheik.Why is India building a 2,500-mile-long fence along its border with Bangladesh ? Russia is determined to build a security fence along its border with Chechnya . Brunei is building a security fence along its 20-kilometer border with Limbang. Big countries ;little countries are building fences ;in fact it is the rule rather than the exception . But the richest country in the world cannot .

Crime has sky rocketed among illegals but still we do not build .Social services like schools and hospitals are stretched to the breaking point but still we do not build . A fence will cost at most $8 billion and would pay for itself in no time many times over and will probably save American lives . Americans want it ;It should be built.

Choux rated this answer Excellent or Above Average Answer
excon rated this answer Excellent or Above Average Answer

Question/Answer
excon asked on 11/21/05 - Arabs


Hello experts:

I know some of you will take offense at my premise, but thats never stopped me before.

Im an uneducated exconvict. Bush is an educated president. Nonetheless, I put myself on the same intellectual plain with Bush in terms of world affairs. Pretty arrogant of me, huh?

Heres why. Well, having no education, I never knew how the world WAS. But I read, so I know how the world IS. Bush, being educated, knows how the world WAS. But he doesnt read, so he doesnt know how the world IS.

Given all that, Id say that makes us even. You may say something different, and Im sure you will.

Heres what I know: The Arabs of today cant get along with anybody. Thats the way it is, and I believe thats the way its been. Do you think they can?

Heres the predicament: Our very future, maybe even our survival, is based on whether the Iraqis can keep their country together after we leave, WHENEVER that happens to be. And that date will come. Bush bet OUR future, the ENTIRE ballgame, on the ability of Arabs to get along.

I would never make a bet like that. I dont think that was very good. You would have?

excon

tomder55 answered on 11/21/05:

I think we will have a permanent presence in Iraq simular to our continued presence in Japan ;Germany ;South Korea ,and the Balkans . I think both Democrat and Republicans in Congress instictively know this reality .

The purpose of wars change .The Civil War went from the war to preserve the union to a war to end slavery . WWI the US entered to retaliate against nations who were violating free passage on the oceans to the war keep the world safe for democracy . Whatever the reason prewar ;the end now is to change the status quo in the middle east from pre -9/11 conditions ( I believe it always was ). That will require a long term commitment .

I agree with you that the US has not yet realized what our commitment to defeat Islamo-fascism will require .If you wish to pin the blame on the President for that go right ahead. America was lured by this 'Powell Doctrine' that said that an exit strategy had to be in place before you commit your forces ;that you should not commit your forces unless the task can be completed in 100 hrs.

The new Iraqi gvt. will be our ally .Iraq will demonstrate to the rest of the Arab world that there is alternatives to their historical reality .It already has to some extent ;but all this effort will be wasted if we withdraw. I expect Americans will be grarisoned in Iraq for many years

excon rated this answer Excellent or Above Average Answer

Question/Answer
excon asked on 11/21/05 - Patriot or Traitor


Hello Bushies:

Oh, boy are we in trouble. This guy is leading us down the golden path.

So, what is it? Are you traitor if you disagree with Bush like you were last week, or are you a patriot like you were yesterday?

I'm worried - very worried... You're not??? So, going from hot to cold in world affairs is good?????

excon

tomder55 answered on 11/21/05:

no ;I will not fall into the trap of name calling with ad hominems like "traitor" ;or the counter slander "chicken hawk".

But I will say that during the Civil War the 2nd guessing of the Union effort damn near defeated the Union. Had the Democrat candidate McClellan had won the 1864 election then the Union would've no doubt settled for a negotiated peace that would've meant some kind of partition of the country .Most likely Lincoln won re-election due to the capture of Atlanta . Once Lincoln was re:elected it was just a matter of months before the war was finished.

Throughout the war Lincoln was subject to the 2nd guessing of the press ,and more importantly by Congressional committees who dissected every decision ;every strategy ,and every battle .They turned up and down the heat with the ebb and flow of public opinion . The South of course strategized to take maximum advantage of this ;as our enemy does today .

I wish I had the stats to confirm this but I am certain that incidents of attacks against our troops have risen as the internal debate has raged in this country .This was the playbook of General Lee and Gen. Vo Nguyen Giap .I am sure it is the same for Zarqwai (if he is still alive ).

excon rated this answer Excellent or Above Average Answer

Question/Answer
excon asked on 11/20/05 - The linking of Iraq to 9/11


Hello wrongwingers:

Bush has continually linked the two wars. So do you. So, when we are forced to withdraw from Iraq, which we should do (and since the people cant tell the difference), we'll also be forced to withdraw from the war on terror, which we absolutely should not do.

The problem with this lie, is Americans believe it. So, when we lose one war, we'll lose the other.

I'll bet you wingers will now be yelling that they are, indeed, separate wars. Well dudes, it's too late. Hopefully, you've not doomed us.

Me? I'm moving to a deserted island.

excon

tomder55 answered on 11/21/05:

first ;a definition change . it is no longer the war against terror ,that is too vague . We have a specific enemy ;the Islamo-fascists ;jihadistan etc. Terrorism is their asymetrical tactic to fight this war. One of the big mistakes the adm. made is not defining the enemy ;Bush has recently started using the term Islamo-fascists but I'm afraid too late.

If we leave Iraq before the job is done then I predict we will lose our will in the war against jihadistan and isolationist tendencies will rear their ugly head. The administration continuously warns against this very scenario.

The Senate I'm afraid made a terrible blunder by passing the Warner amendment. It calls for �� to be a year of significant transition" from American to Iraqi forces irrespective of the conditions on the ground in Iraq.The 79-19 vote for the Warner amendment is evidence of the onset of defeatism.(John McCaine knew that the amendment was folly and was one of the 19 nay votes ). It can only encourage our enemies in Iraq and elsewhere in their conviction that the United States lacks the will to fight a determined totalitarian ideology like Islamofascism, and that it is just a matter of time until we are defeated . I mentioned in previous postings that the enemy long ago determined that America was unwilling to fight a war of attrition. I thought they were wrong but now I have my doubts . If political pressure forces the adm. to retreat then I think I will join you on that desert island.

excon rated this answer Excellent or Above Average Answer

Question/Answer
excon asked on 11/19/05 - Freedom - gone!


Hello experts:

Do you think you're free? Do you think you can publish what you want on THIS website FREE from government interference?

I don't think I can. They've got this "carnivore" program that looks for words in the internet. I don't know what words, but I'm afraid to use words like bomb, attack, World Trade Center, revolution, building, kill Bush. I've decided to use them in this question just to see who's watching.

Do you think about that at all? Do you hesitate to write something for fear of what some authority might think about it? Am I nuts? Am I paranoid? Should we, in America, have a fear like that?

I know what your typical right wing response would be: Well Son, just dont use those words and youll have nothing to fear.

excon

tomder55 answered on 11/20/05:

parania ,the destroyer (Kinks)

I have regularly been openly critical of the CIA on these pages and sleep well at night.

btw ; the FBI has essentially abandoned the use of Carnivore in favor of commercially available software since January .It had been in use since 1999.


The FBI has been involved in wiretapping for decades .Why is this different ? It is subject to exactly the same restrictions.By existing U.S.law, USG personnel are required to get a warrant or court order naming specific people or email addresses that may be monitored.

Open source research however is an invaluable investigative tool .Had the Pentagon taken the open source research of Lt. Col. Anthony Shaffer who was working on their 'Able Danger ' data mining mission then possibly 9-11 could've been averted.He had identified Mohammed Atta and three other 9/11 hijackers as members of an al Qaeda cell operating in the United States in 1999. Shaffer claims that he alerted the FBI in September 2000 about the information uncovered by 'Able Danger' but he alleges three meetings he set up with bureau officials were blocked by military lawyers.

excon rated this answer Excellent or Above Average Answer

Question/Answer
Choux asked on 11/19/05 - NeoCon Movement Imploding...

Taking Right Wing "Journalists" with it.

""We experienced a similar political Tsunami in the U.K. when Thatchers Conservative party imploded amid scandal, nepotism and plain old incompetence. Those events and the resulting sting inflicted upon the British people have kept that party from power for three consecutive terms.

Ive been back and forth to the USA for many years now and I always made time for Bill OReilly. I may not agree with most of his politics, but I enjoyed his sincerity and now and again found myself nodding. Besides, a mans got to satiate that
Reptilian part of the brain one way or another, and thats what Fox news is for, right?

Anyway, Ive noticed since the onset of the war how old Bill has quickly deteriorated. Returning to his No Spin Zone over the past three years has been akin to visiting a distant relative in a nursing home who is suffering the onset of Alzheimer's. He has trouble recollecting important facts, and bursts into rants that make no logical sense. And just as with Alzheimer's, its really sad to watch an individual lose what little dignity they possessed.

The problem for Bill and many other journalists, is that hes part of a Neocon movement thats quite simply begun to implode. Individuals are already jockeying for position in the hope of surviving in the changing political landscape. However some like Bill - are so deeply embedded in the Neocon mythos, theres really nowhere for them to go and theyre scared.

For when you zealously advocate sending young men and women to their deaths in a misconceived war, its an all or nothing bet. And they have lost. Most Republicans know it, few will say it, and some like Bill OReilly will never, ever admit it. Why should he? If President Bush is incapable of admitting mistakes, why should his devoted followers?

Bills rants concerning San Francisco come from fear and frustration. Hes angry, but not really with the Bay. Hes angry because the whole Neocon house of cards is tumbling down and theres not a damned thing he can do about it. They control both houses and its all going to hell. The usual methods of disinformation, Orwellian double talk (Clean Air Act/No Child Left Behind) and downright lies no longer work. This is a very frustrating time for Bill.

What is really getting Bill down above all else is the Bloggers. The Mainstream media is dying. More and more people are turning away from their TVs and searching for the truth online. Ideas are being exchanged, people are becoming aware, and all the time Big Brother whines in the corner.

And as this phenomenon grows, people like Bill OReilly find themselves held accountable for their rantings. Lets face it, its really annoying when someone reminds you of some stupid thing you said. Smart thing to do is to admit it, apologise and move on. But dont accuse them of smearing you, not when theyre smearing you with you own words. And Bill, whatever you do, dont try to pass off your venom as a satirical riff.

Because as we say in Glasgow, A kick in the balls, is a kick in the balls. Dont try to pass it off as a kiss, not when my balls are still ringing.

Posted by: ScottishScript on November 18, 2005 at 01:10am


Your comments.....

tomder55 answered on 11/19/05:

this author is ill-informed about OReilley's positions . I'd hardly call him a neo-con and the only way he could be would be if this author generalized all people who were in favor of the Iraq invasion as neo-cons. The author shows a willing ignorance of the the neo-con movement which is understandable if he listens to the left talking points. The neo-con movement predates William Kristol and his PNAC .Actually it was his father Irving Kristol who is credited with founding the movement. He wrote an essay in The Weekly Standard attempting to explain the movement .That is a good start but much more research would have to be devoted to truely understand the neocons.The biggest difference between neocons and traditional conservatives is the role of the US in the world .Neocons think it a noble enterprise to use American influence to promote the spread of freedom in the world .Traditional conservatives tend to be isolationist.(which btw the Democrats foreign policy traditionally was closer to the neocons ;now they appear to be the new isolationsist and even more disturbing ;the appeaser party). The term "neocon," is somewhat controversial and is rejected by many to whom the label is applied. Others say it lacks any coherent definition, especially since many so called neoconservatives disagree with one another on major issues.

He is correct about OReilly's distrust of the blogsphere. This is a postion where I disagree with OReilly. OReilly is the product of CBS ;he later broke away to exploit cablecast. I think he realizes he has taken that ride as far as he can .Now I suspect he would like to get back to the MSM .He has modified his view to a centralist
and takes pride that both the left and right attack his positions .The blogshere to me has been the greatest movement that has come from the internet revolution . No longer can the Dan Rather's of the world produce lies without it being filtered through the prism of scrutiny .

His comments about SF may have been a little over the top but I share the sentiment .From all appearances SF does not desire to be defended either by the military and evidently they chose not to defend themselves (at least based on the personal handgun ban .) Since the author of this hit piece is so free at throwing generalities around then I think it fair to say that the good people of SF perfectly define what weak-kneed liberalism is all about .

Choux rated this answer Excellent or Above Average Answer

Question/Answer
excon asked on 11/18/05 - Greenies


Hello DEAer's:

I knew MLB was juiced. But I didn't know they were speeding too. Did you?

We're not talking about drugs that build up your bodies and make you stronger, yet leave you with a clear head. No, were talking about speed, greenies, amphetamine, dexies. These are simply illegal street drugs that get you high. It's not any different than cocaine.

So, where was the DEA? In the gettho busting the poor? They didn't know about the "greenies" in the locker room? Somehow, I think they did.

More BS from your friendly federal government.

excon

tomder55 answered on 11/18/05:

of course they knew .


Steve Bechler ;a pitcher with the Orioles died 2 years ago from the effects of taking a legal supplement (at the time);ephedra ,and only then did baseball and Congress make it illegal to take .Ephedra has simular properties as speed and is usually taken for the same reasons (weight loss and to get high due to an overall stimulated body ).


Jim Bouton documented ancedotes of players popping greens in his book 'Ball Four ' back in 1970 . The book was groundbreaking and shocking at the time but put in the light of the antics of today's players ;it is down-right tame by comparison .

Baseball only institued the new policy grudgingly after Congress gave them an ultimatum .Truth be told ;the fans liked the sport better in the mid 90s when the juiced up ubermen were hitting 500 ft. homeruns.

but you want to talk about hypocracy ;ok . How about the fact that the Congress was jumping all over themselves in rightous sanctimony about the need to clean up 'America's past-time' (which is favored with unprecidented anti-trust exemptions ) ;while they for years have refused to do anything about the most corrupt sport that has ever had professional status in the country...Boxing ...which exploits the poor in society as much as any illegal activity .Just this week the House voted 233-190 against forming a U.S. Boxing Commission .

Choux rated this answer Excellent or Above Average Answer
excon rated this answer Excellent or Above Average Answer

Question/Answer
excon asked on 11/16/05 - Internet poker


Hello rollbacktheclockers:

Internet gambling is hot. It's everywhere. It's in your home town. I'll bet (pun intended) your kids do it.

It's also illegal. What should we do about it? The prisons are already filled with non-violent drug offenders. Should we look the other way? Should we change the law? Should gambling on your home computer be legal? After all, how bad can gambling be if the king of virtue, Bill Bennet gambles his ass off?

excon

tomder55 answered on 11/16/05:

the nature of the net makes any ban on internet gambling difficult if not impossible to enforce .The hypocracy is in the fact that almost all States run numbers games. I predict they will try to enforce various laws and will eventually settle for the tax revenue they can get from it.

I do not see gambling as immoral so long as the person is dealing with disposable assets ;which Bennet clearly was .

Gambling addiction is serious business but the addict can find many means to legally indulge if they choose to so the prohibition of internet gambling seems arbitrary .

The greater concern is with as you point out ,underage gambling . My daughter is over 18 now so I have no control over that ;but as a minor I am sure she never used home base as a casino . It is easy to snoop on what activites are done on a computer ;and in that regard I did not believe in her right to privacy.

ETWolverine rated this answer Excellent or Above Average Answer
excon rated this answer Excellent or Above Average Answer

Question/Answer
Choux asked on 11/15/05 - McCarthyism Alive and Well

"Last Tuesday, Bill OReilly encouraged terrorists to target San Francisco because he was upset that the city voted to ban military recruiters from high-school and college campuses. OReilly said if Al Qaeda comes in here and blows you up, were not going to do anything about it. Were going to say, look, every other place in America is off limits to you, except San Francisco.

At first, OReilly defended his comments as not controversial. That didnt seem to work, so tonight he claimed the whole thing was a satirical riff.

In OReillys view the only real problem is the internet smear sites drawing attention to his comments:

Some far left internet smear sites have launched a campaign to get me fired over my point of view. I believe they do this on a daily basis. This time the theme is OReilly is encouraging terrorist attacks. Unbelievably stupid. Not unusual with these guttersnipes.

Fairly typical comments from OReilly. But he added an unusual twist. OReilly promised to publish the names of everyone who supported these internet smear sites on his website:

Im glad the smear sites made a big deal out of it. Now we can all know who was with the anti-military internet crowd. Well post the names of all who support the smear merchants on billoreilly.com. So check with us.

Its unclear where OReilly would find such a list. But since he has labeled everyone who supports websites like MediaMatters.org and ThinkProgress.org as anti-military it seems to be an effort to intimidate and shame our readers.

At the time this post was published, the promised list doesnt appear on billoreilly.com. Stay tuned for updates."--


So, are you liking what you read??

tomder55 answered on 11/16/05:

I think in OReilly's defense ;unlike Pat Robertson ,he was speaking tongue in cheek.

He does point out that the city did pass the resolution that was clearly anti-military .Mayor Gavin Newsom calls it 'predatory recruiting'.SF also drove the Navy fleet out of town but still pretends to have Fleet Week each year.

In the mid 1990s, SF was floating a resolution to secede from California. This got a lot of support until someone at the State level said: "Fine! Secede, but take in NO Tax dollars from the rest of California". OReilly mentioned that the same conditions should apply with Homeland Security dollars ;after all ,the people of SF have demonstrated time and again that they are not concerned with their defense.

If the Sec. Def. decided in light of the resolution to close down every military recruiting station and installation in the greater San Francisco area I wonder if the good folks of SF would reconsider .

Choux rated this answer Excellent or Above Average Answer

Question/Answer
Choux asked on 11/15/05 - W 2001 v. W 2005

Cut and Paste article by Byron Williams on Huffington Post Blog::::


"Veterans Day is the day set aside to commemorate former members of the armed services. Traditionally, the president goes to Arlington National Cemetery to lay a wreath at the tomb of the Unknown Soldier

This year, it was the vice president, and not he commander-in-chief, that participated at the Veterans Day ceremonies.

It seems the president was preoccupied with other matters.

Mr. Bush was in full campaign mode, complete with matching game face. Only this time the president was not campaigning for reelection, but for something more formidable than the challenges presented by Al Gore or John Kerry: his credibility.

The president finds himself locked in an epic struggle against himself; or perhaps more precise the version of himself, who 48 months ago had an 87 percent approval rating.

In this W 2005 v. W 2001 one discovers some very stark differences. W 2001 was trusted, W 2005 is not. W 2001 had the support of the world, W 2005 does not. W 2001 could use conjecture to trump facts, W 2005 cannot.

In a democratic society, loss of credibility is a difficult hurdle to overcome, W 2001 had it, and W 2005 cant find it.

It is an insult to the American people for the president to simply state: We do not torture when the facts clearly suggest something very different.

If indeed imitation is the sincerest form of flattery, then W 2001 should be bowled over by the platitudes of W 2005---for they come right out of the Karl Rove 2001 playbook.

In his customary prefabricated production, complete with a prescreened audience, W 2005 ***accused Senate Democrats*** [LOLOLOLOL!!!] of misleading the nation about the threat from Iraqs weapons programs.

According to the president, These baseless attacks send the wrong signal to our troops and to an enemy that is questioning Americas will. As our troops fight a ruthless enemy determined to destroy our way of life, they deserve to know that their elected leaders who voted to send them to war continue to stand behind them.

Those of you old enough to remember W 2001 no doubt recognize the aroma of this argument. It is the American people who have grave misgivings about the presidents leadership, and yet, he blames Senate Democrats.

Moreover, did I not detect what my conservative colleagues would classify as classic Clintonian spin in the presidents words?

The senate did not vote to send men and women to war, the 77-23 vote authorize the use of United States armed forces against Iraq, the decision to go to war was the presidents and his alone.

This strategy worked masterfully for W 2001, the nation was rocked by 9/11, so desperately needed to trust its leadership, gave the president the benefit of the doubt as he morphed preemptive war against Iraq into the overall war on terror.

These tactics are much more difficult to achieve when 65 percent of the American people disapprove of the Presidents handling of Iraq and 53 percent now believe the war was not worth fighting, according the latest NBC/Wall Street Journal poll.

The recent piece in the New York Times magnifies the difference between the credibility of W 2001 and W 2005. According to the article, a laptop computer captured last year that shows that Iran is actively trying to figure out how to build a nuclear warhead. The Bush administration, however, is having trouble convincing our allies that the laptop isn't a fake.

Contrary to presidents assertions, Senate Democrats are not the reason for his troubles, it is the cumulative weight of the facts.

While the Administration headed for the stonewalling bunkerthere were no weapons of mass destruction, there was the Downing Street memo, a key White House staff member has been indicted, and another former key member of State Department recently used the adjective cabal to describe the war planning between the vice presidents office and the Defense Department.

So far, the W 2001 approach is not working for W 2005. I wonder what Swift Boat Veterans for Truth are doing these days?""




Question::: LOLOLOLOL..... Liars Liars, pants on fire! Right guys?

tomder55 answered on 11/16/05:

The lie that Bush lied us into war threatens the Bush presidency in a way no ordinary political charge does. Bush needs to refute it,and to keep on refuting it.

Bush has the facts on his side and should continue to use them. I have written too many rebuttals to the assertion and do not care to detail them here ;but Norman Podhoretz at Commentary Magazine does a decent summary and rebuttal of the Big Democrat LIE !

The unanswered assault by Bush's enemies have had an effect so the President has no choice but to aggressively counter them if he wants a successful 2nd term. He needs to keep doing so, and also to continue making the positive case for why the war was right and necessary.

Choux rated this answer Excellent or Above Average Answer

Question/Answer
ETWolverine asked on 11/16/05 - Some Presidential Job-Approval-Rating facts

People keep looking at Bush's low approval ratings and see that as the downfall of the Republican party. Here are some facts about just how low (and high) presidential approval ratings have gone in the past fourty years or so.

Bush (G.W.) Highest: 92% 10/08-09/01, ABC Poll
Lowest: 35% 10/30-11/1/05, CBS Poll

Clinton Highest: 73% 1/28/98 CBS/NYTimes Poll and 12/19-20/98, USA Today/CNN/ Gallup Poll
Lowest: 36% 5/26-27/93, Yank/Time/CNN Poll

Bush (G.H.W.) Highest: 89% 2/28/-3/3/91, Gallup Poll
Lowest: 29% 7/31-8/2/92, Gallup Poll

Reagan Highest:68 5/8-11/81, Gallup Poll and
5/16-19/86, Gallup Poll
Lowest: 35% 1/28-31/83, Gallup Poll

Carter Highest 75% 3/18-21/77, Gallup Poll
Lowest: 28% 6/29-7/2/79, Gallup Poll

Ford Highest: 74% 8/16-19/74, Gallup Poll
Lowest: 37% 1/10-13/75, Gallup Poll and
3/28-31/75, Gallup Poll

Nixon Highest: 67% 11/12-17/69, Gallup Poll and
1/26-29/73, Gallup Poll
Lowest: 23% 1/4-7/74, Gallup Poll

Johnson Highest: 80% 2/28-3/5/64, Gallup Poll
Lowest: 35% 8/7-12/68, Gallup Poll

Kennedy Highest: 80% 3/8-13/62, Gallup Poll
Lowest: 56% 9/12-17/63, Gallup Poll

Eisenhower Highest: 79% 12/14-19/56, Gallup Poll
Lowest: 48% 3/27-4/1/58, Gallup Poll

Truman Highest: 87% 6/1-5/45, Gallup Poll
Lowest: 22% 2/9-14/52, Gallup Poll

Roosevelt Highest: 84% 1/8-13/42, Gallup Poll
Lowest: 48% 8/18-24/39, Gallup Poll

Source: The Roper Center

Basically, the only thing we can learn from presidential approval ratings is that they are NOT an indicator of how either party or the President will do in the following elections.

Reagan had job approval ratings as low as W's (just 35%) one year before he was re-elected for his second term in the largest landslide in US presidential history.

Clinton had a 36% approval rating just one year before he was re-elected for his second term.

Low approval ratings don't say anything about either the party or the results of the next election. Neither do high approval ratings. So don't start writing Bush or the GOP off just yet. There's 3 more years to come, and a lot can happen in 3 years.

Comments, please.

Elliot

tomder55 answered on 11/16/05:

Harry Truman was plagued by low numbers ,but I consider him the finest Democrat President of the last century .History has looked on his Presidency alot kinder than his contemporaries did . There is a danger in reading too much into the poll tea leaves .They are portraits of a very small time line .

Bush went through a brutal quarter .What is suprising is that will all the natural disasters ;the controversies ;other indicators ,especially economic were very favorable. There was another quarter of growth which is really suprising in light of the series of blows due to the weather . Unemployment remains virtually non-existant (5%)and job growth is up again .except for the ripple effect of the price of the fuel disruption ,prices have remained stable.

The good news in Iraq continues to be ignored or under reported . The passing of the constitution was an important development ;and elections for the 1st democratically elected gvt. will be held mid-December . More Iraqis are involved in the security of their country ;and politicians like Chalabi are having success in bringing Sunnis into the fold. Yesterday's sense of the Senate resolution (Warner Amdt. No. 2518)only mirrors the over-all plan that has been in the works since viceroy Bremer departed the scene.The President approved of the Senate action even as the MSM called it a rebuttal. The truth is that the President already complies with the provisions ;he and his staff have updated frequently .The problem is that few want to hear it .

ETWolverine rated this answer Excellent or Above Average Answer

Question/Answer
z298418 asked on 11/15/05 - poverty and politics

Hello,

Do you think Political have to do with or cause Poverty?
what do you think about riot in French that is cause Poverty right?

Thank you for your time

tomder55 answered on 11/16/05:

poverty has a little to do with the French rioting . The French nanny social system is a failure . 10% unemployment is the norm.The French state owns 30% of all housing. These people are dependents of the French state. They live where they are told. A French elite mentality exists so they are permanently segregated . All this is true .

But the root cause of the rioting and you will see it more frequently throughout Eurasia is the increase in the population of Muslims .The intifada raging in France has been characterized by overwhelmingly Muslim rioters engaged in acts of wanton destruction, punctuated by claims of territorial control over sections of various French cities.What you are seeing is right out of Mohammed's playbook ;A peaceful coexistance is tolerated until the Muslim population becomes statistically significant ;and then they force the country to accommodate changes in the culture and the laws of the land .It was the fact that the police had treated these areas as "no-go zones," left to be run by the Muslims themselves.I'm betting they will eventually demand that the areas they live in become semi-autonomous zones; like the West Bank and Gaza. Eventually they become the majority in the land ;then People of other cultures are to be forcibly converted to Islam except for a few whose existence may be tolerated so long as they accept the status of "dhimmi," second-class citizens who dutifully accept a whole series of restrictions lest they offend the sensibilities of Muslims.

excon rated this answer Excellent or Above Average Answer
purplewings rated this answer Excellent or Above Average Answer
z298418 rated this answer Excellent or Above Average Answer

Question/Answer
excon asked on 11/15/05 - A taste of their own medicine perhaps???


Hello politicos:

Jordanians Shocked! For the first time in history, an Arab population took to the streets to protest their shock at suicide bombings - especially a wedding.

And why are they shocked? Because the terrorists blew up Jordanians. As long as Islamic terrorists blew up men, women and children who are Jewish, Christian, Hindu, American, Australian and black Sudanese, the Arab and larger Muslim worlds were not particularly disturbed. In fact, Palestinians celebrated when Jews were blown up at Passover seders and at weddings.

But guess who the Jordanians are mostly made up of? Thats right, Palestinians. Bummer, dudes.

excon

tomder55 answered on 11/15/05:

there is already spin going down on "the Arab Street " that this bombing was a deliberate attack by guess who ? yup ;Israel.

Maj. Gen. Bashir Nafeh, head of Palestinian military intelligence, and Jihad Fattouh, the brother of a Palestinian parliament speaker was killed in the blast as well as 3 40 years old Chinese "students " . It was reported that these Chinese students were meeting with the Palestinian officials when they all were killed by the bombing.

The motives it is alleged is that the Israeli's were sending a signal to the Chinese to stop their cooperation with the Palestinians .

Tack this one up there with the claim that Jews were warned to stay out of the twin towers on 9-11 . Yup ;the theory goes that the Israelis were warned to stay out of the Amman hotels also.

http://www.scoop.co.nz/stories/HL0511/S00214.htm

Choux rated this answer Excellent or Above Average Answer
ETWolverine rated this answer Excellent or Above Average Answer
excon rated this answer Excellent or Above Average Answer

Question/Answer
Itsdb asked on 11/15/05 - Just thinking...

I thought it curious that today our front page featured two stories by the AP. One spoke of Bush fighting back against Dems critical of his "misleading" the country into war, and one about the VA's failure to spend all they promised on research into the "thousands" of Gulf War vets suffering the effects of oil fires, nerve gas and other toxic substances.

Let me get this straight, thousands of gulf war vets still suffer from exposure to nerve gas and other toxic substances, but it was misleading to go after the man responsible?

Steve

tomder55 answered on 11/15/05:

From David Kay's testimony to the Senate Armed Services Committee :


We have discovered hundreds of cases, based on both documents, physical evidence and the testimony of Iraqis, of activities that were prohibited under the initial U.N. Resolution 687 and that should have been reported under 1441, with Iraqi testimony that not only did they not tell the U.N. about this, they were instructed not to do it and they hid material.

And let me take one of the explanations most commonly given: Analysts were pressured to reach conclusions that would fit the political agenda of one or another administration. I deeply think that is a wrong explanation.

As leader of the effort of the Iraqi Survey Group, I spent most of my days not out in the field leading inspections. It's typically what you do at that level. I was trying to motivate, direct, find strategies.

In the course of doing that, I had innumerable analysts who came to me in apology that the world that we were finding was not the world that they had thought existed and that they had estimated. Reality on the ground differed in advance.


And never -- not in a single case -- was the explanation, "I was pressured to do this." The explanation was very often, "The limited data we had led one to reasonably conclude this. I now see that there's another explanation for it."

And each case was different, but the conversations were sufficiently in depth and our relationship was sufficiently frank that I'm convinced that, at least to the analysts I dealt with, I did not come across a single one that felt it had been, in the military term, "inappropriate command influence" that led them to take that position.


The Dems are walking into a trap by taking this stance that the intel was manipulated . Thousands of documents that the Iraqi's did not have time to destroy are now centrally located in a large warehouse in Doha ;and are being uploaded into a database called 'Harmony'. Many of these have already been declassified and the information ;just judging from the titles they are being given is revealing . Steven Hayes at the Weekly Standard has compiled a list of some of them including many of them that suggest a link between Iraq and al-qaeda via the Taliban (# 6 is truely fascinating when one considers what the Salman Pak site was used to train for );the continued clandestine WMD program ;and French ,German ,and Russian cooperation with Iraq in pre-war planning :

1. Iraqi Intelligence Service (IIS) Correspondence to Iraq Embassy in the Philippines and Iraq MFA (Ministry of Foreign Affairs)
2. Possible al Qaeda Terror Members in Iraq
3. IIS report on Taliban-Iraq Connections Claims
4. Money Transfers from Iraq to Afghanistan
5. IIS Agent in Bulgaria
6. Iraqi Intel report on Kurdish Activities: Mention of Kurdish Report on al Qaeda--reference to al Qaeda presence in Salman Pak
7. IIS report about the relationship between IIS and the Kurdish Group Jalal Talibani [sic]
8. Iraqi Mukhabarat Structure
9. Locations of Weapons/Ammunition Storage (with map)
10. Iraqi Effort to Cooperate with Saudi Opposition Groups and Individuals
11. Order from Saddam to present $25,000 to Palestinian Suicide Bombers Families
12. IIS reports from Embassy in Paris: Plan to Influence French Stance on U.N. Security Council
13. IIS Importing and Hiding High Tech Computers in Violation of UN
14. IIS request to move persons, documents to private residences
15. Formulas and information about Iraq's Chemical Weapons Agents
16. Denial and Deception of WMD and Killing of POWs
17. 1987 orders by Hussein to use chemical weapons in the Ealisan Basin
18. Ricin research and improvement
19. Personnel file of Saad Mohammad Abd Hammadi al Deliemi
20. Memo from the Arab Liaison Committee: With a list of personnel in need of official documents
21. Fedayeen Saddam Responds to IIS regarding rumors of citizens aiding Afghanistan
22. Document from Uday Hussein regarding Taliban activity
23. Improvised Explosive Devices Plan
24. IIS reports on How French Campaigns are Financed
25. French and German relationships with Iraq
26. IIS reports about Russian Companies--News articles and potential IIS agents
27. IIS plan for 2000 of Europe's Influence of Iraq Strategy
28. IIS plans to infiltrate countries and collect information to help remove sanctions
29. Correspondence from IIS and the stations in Europe
30. Contract for satellite pictures between Russia, France and Iraq: Pictures of Neighboring Countries (Dec. 2002)
31. Chemical Gear for Fedayeen Saddam
32. Memo from the IIS to Hide Information from a U.N. Inspection team (1997)
33. Chemical Agent Purchase Orders (Dec. 2001)
34. Iraq Ministry of Defense Calls for Investigation into why documents related to WMD were found by UN inspection team
35. Correspondence between various Iraq organizations giving instructions to hide chemicals and equipment
36. Correspondence from IIS to MIC regarding information gathered by foreign intelligence satellites on WMD (Dec. 2002)
37. Correspondence from IIS to Iraqi Embassy in Malaysia
38. Cleaning chemical suits and how to hide chemicals
39. IIS plan of what to do during UNSCOM inspections (1996)
40. Secret Meeting with Taliban Group Member and Iraqi Government (Nov. 2000)

As far as I am concerned ;if the Dems. want hearings by all means have them.

To Your other point. I think the gvt. should take Gulf War syndrome a little more seriously .This is beginning to resemble the shameful way that agent Orange exposure was treated.



ETWolverine rated this answer Excellent or Above Average Answer
excon rated this answer Excellent or Above Average Answer
Itsdb rated this answer Excellent or Above Average Answer
purplewings rated this answer Excellent or Above Average Answer

Question/Answer
Choux asked on 11/14/05 - South of the Border

MEXICO CITY, Mexico (AP) --" Venezuela called its ambassador to Mexico home Monday rather than apologize after President Hugo Chavez warned Mexican leader Vicente Fox: "Don't mess with me." Mexico responded by recalling its own diplomat.

In an interview with CNN en Espaol, Fox said he would meet with Foreign Secretary Luis Ernesto Derbez to decide what to do next.

Tensions between Fox and Chavez spilled over after this month's Summit of the Americas in Argentina, where Fox defended a U.S.-backed proposal for a Free Trade Area of the Americas while Chavez proclaimed the idea dead."


What on earth??
It kinda makes me nervous when two Latin Presidents act all hot-headed.

tomder55 answered on 11/15/05:

Chavez calls Fox a Bush puppy ..from my perspective I think he's got it a little backwards .

No I do not think war will erupt . What I think Chavez is doing is trying to influence the Mexican elections as he is doing in Equador and Bolivia.

The field is set for next years elections in Mexico. Roberto Madrazo will run for the Institutional Revolutionary Party, or PRI, Felipe Calderon from the National Action Party, or PAN (Fox's party ) ,and Mexico City Mayor Andres Manuel Lopez Obrador,for the Democratic Revolutionary Party, or PRD. Lopez Obrador ,a leftist ,is expected to win and it is likely that Chavez is trying to position himself with him .

Whereas Fox is seen as aligned with the US economic interests ;like free trade;Lopez Obrador would most likely shun U.S. policies.

Roberto Madrazo said he sees simularities between Chavez and Lopez Obrador .
"I see an authoritarianism in both. I see irreducible positions as holders of the absolute truth ... social-assistance policies instead of a battle for equality, and I see a lack of respect for the law."

Choux rated this answer Excellent or Above Average Answer
excon rated this answer Excellent or Above Average Answer

Question/Answer
jnlomonte asked on 11/14/05 - Poster

Poster seen at a political rally with a picture of Dubya.

"Will somebody please give this guy a blowjob so we can impeach him."

tomder55 answered on 11/14/05:

to impeach you need to find a crime. In Clinton's case ;although I thought they impeached for the wrong reason ;the fact is that there was a clear case of perjury .It was in deposition in the Paula Jones sexual harrassment lawsuit that he not only perjured himself ;but he also suborned Monica to give false testimony. The case itself was dismissed but Congress proceded with the impeachment anyway . I happen to agree with the Democrats that the most the President's conduct merited was censure .

there were other more compelling reasons to impeach Clinton that were national security breeches but they were not persued. No one asked me.

jnlomonte rated this answer Excellent or Above Average Answer

Question/Answer
paraclete asked on 11/13/05 - let's tell it like it is?

Sick of counter-terrorism experts?
Sunday, November 13, 2005 - 09:28 PM

That terribly well-educated alleged terrorism and security dude Aldo Borgu from Canberra was on TV the other night waffling on about Australia's alleged terrorism suspects. He was blathering about the lack of employment opportunities for such people.

A bit of a problem for his argument that almost all of them were gainfully employed.

We really are at the buttock end of intelligent debate in Australia.

Everyone's talking loud, but saying nothing.

We're stuck in clowntown.

I think it's partly the price of travel. But hell, there are clowns the world over.

Nevertheless, the babies we have thumping their chests to be seen as national security journalists or experts really ought to get out more.

If they don't get beheaded by unemployed people, they might learn something.

Take this recent hullabaloooooo about "CIA black sites" - quiet little detention centres around the world where bad counter-terrorism goons abuse Muslims.

Old news.

Here's a bit about the capture of alleged Sep 11 organiser and former Manila resident, Khalid Sheik Moohamed, from Mark Bowden's The Dark Art of Interrogation in The Atlantic Monthly of October 2003:

Once more hooded, Sheikh Mohammed was driven to Chaklala Air Force base, in Rawalpindi, and turned over to U.S. forces. From there he was flown to the CIA interrogation center in Bagram, Afghanistan, and from there, some days later, to an "undisclosed location" (a place the CIA calls "Hotel California")presumably a facility in another cooperative nation, or perhaps a specially designed prison aboard an aircraft carrier. It doesn't much matter where, because the place would not have been familiar or identifiable to him. Place and time, the anchors of sanity, were about to come unmoored. He might as well have been entering a new dimension, a strange new world where his every word, move, and sensation would be monitored and measured; where things might be as they seemed but might not; where there would be no such thing as day or night, or normal patterns of eating and drinking, wakefulness and sleep; where hot and cold, wet and dry, clean and dirty, truth and lies, would all be tangled and distorted.

And dig this:

There is no clear count of suspected terrorists now in U.S. custody. About 680 were detained at Camp X-Ray, the specially constructed prison at Guantnamo, on the southeastern tip of Cuba. Most of these are now considered mere foot soldiers in the Islamist movement, swept up in Afghanistan during the swift rout of the Taliban. They come from forty-two different nations. Scores of other detainees, considered leaders, have been or are being held at various locations around the world: in Pakistan, Saudi Arabia, Egypt, Sudan, Syria, Jordan, Morocco, Yemen, Singapore, the Philippines, Thailand, and Iraq, where U.S. forces now hold the top echelon of Saddam Hussein's dismembered regime. Some detainees are in disclosed prisons, such as the facility at Bagram and a camp on the island of Diego Garcia. Othersupper-tier figures such as Sheikh Mohammed, Abu Zubaydah, Abd al-Rashim al-Nashiri, Ramzi bin al-Shibh, and Tawfiq bin Attashare being held at undisclosed locations.

And quite frankly, good.

Shame there's not more of it.

I've long been in favour of the war against jihadist swine being conducted largely in quiet mode. Abductions, killings, abuse.

Sure, you can jump up and down about police states, etc, and that's entertaining, but who does it help?

General Patton slapped shell-shocked men across the head, called them cowards in public. Yes he was a bastard, but he was still one of the few enemies the Nazi military personally feared.

Thank the stars it was Patton who was unleashed on the Nazi hordes instead of the waffle brigade who might have argued the Nazis were driven to anti-social behaviour by unemployment and cultural marginalisation.

The jihadist swine need a good smack.

The ease with whichy one can make bombs from household products means we can expect troubles for some time, and some casualties, but let's not lose our nerve.

The World Trade Center was first attacked in 1993, by jihadist swine based in Manila with help from more swine out of a Brooklyn mosque.

Not too long after the unholy Soviets had been driven out of holy Afghanistan with the help of the evil US.

So let's drop the fixation on the current war to democratise Iraq. After all, when anyone defends the "insurgency", they're defending stinking little losers who love nothing better than to chuck a nasty surprise and blow up in public with the express aim of killing civilians.

Wow. Well done. Moral legitmacy. Muslims are against killing.

So this jihad caper might be a problem we have for a while.

And the goofs raving on about Iraq might like to answer bin Loser's remarks about Australia and East Timor.

I'm glad the goofs do little but sit on the sidelines and whine.

Thank goodness there are a few Pattons out throwing some punches few people even know about.

Two years later it might make the news here.

By which time a lot of people will still be living who might otherwise be dead.

Clowntown.

Mind you there are exceptions like Kit Collier of Canberra and the International Crisis Group. He puts his arse in the grass, so to speak.

But as for most others - like I said, clowntown.

tomder55 answered on 11/14/05:

love it . I would like to add some commentary by Wrechard at Belmont Club .He is refering to the torture amendment that McCain is trying to attach to the military appropriations bill .I have only 1/2 tongue-in -cheek suggested that the logical remedy is to take no prisoners. As Wrechard points out ;if there is no reasonable expectation to get actionable intelligence from a detainee then why bother capturing them ?

It is normally more complicated and dangerous to take an enemy combatant a prisoner rather than to kill him. This is widely acknowledged and patrols that capture the enemy are normally decorated for the extra risk they take. Why do soldiers take the additional risk? Because there is the expectation of obtaining life-saving or battle-winning intelligence. Everyone, even film-makers, know the stock situation in which a person saves his life by saying "I know too much for you to kill me". But if prisoners can no longer be interrogated effectively, short of torture, what is the incentive to capture them? If there is none, what man will take extra risks to take a prisoner, lead him back through hostile territory, keep him alive though wounded and if need be carry him to friendly lines?

paraclete rated this answer Average Answer

Question/Answer
Bishop_Chuck asked on 11/14/05 - Lets call in Walmart

I see where Walmart has anounced new record earnings.
Even during this period of bad economy and oil shortages. I do hope Congress is going to call them in also and try to find out how they did this at the expence of the American Public. Perhpas they too should be looked at to turn over thier excess profits back. How can congress let our free enterprise system go wild and these major companies be making money.

If it was right to call in the oil companies lets call in other companies making more money than the oil companies are.

tomder55 answered on 11/14/05:

the premise of course is that it was the right thing to do to haul in the oil execs. ;It was not .. Oh ;it plays well in populist politics but it was only grandstanding .

Gasoline is expensive for two reasons: Demand for oil is exceeding our ability to pump it; and there's a shortage of American refineries to turn oil into gasoline. Refineries were running at 96 percent capacity before 2 hurricanes temporarily knocked some of them off-line .

For many years, oil companies performed at a low end of profitability. From the early 1980s until now , profits for oil companies lagged behind profits in other industries.Did anyone offer these companies any sympathy when the price of a barrel of oil was $15 ? Already the price of gas is falling because the supply is stabilizing .Profits will not repeat this past quarter ;especially if as the indistry indicates ;they will be adding more money into infrastructure improvement.But Oil firms will only invest in those things if and they perceive future profits to be made from them.

The next sticker shock item will be natural gas this winter . Again ;Congress will demand to know why and will finger -wag and accuse the execs of windfall profits . But in reality ;they already know that a growing demand outpaces supply and exploration for new sources .

as for Wal-mart ?They posted their smallest gain in 4 years due to higher energy prices and the hurricanes ;and most of that was due to food sale and price slashing .Hurricanes Katrina, Rita and Wilma cost them $40 million in the quarter and closed 10 stores.They are lagging behind Target in profits 16% to 36% over the last 3 years and now have to deal with documentary hit pieces like "The High Cost of Low Price" .

Bishop_Chuck rated this answer Excellent or Above Average Answer
ETWolverine rated this answer Excellent or Above Average Answer
excon rated this answer Excellent or Above Average Answer

Question/Answer
excon asked on 11/11/05 - Veterans


Thanks, dudes.

excon

tomder55 answered on 11/12/05:

"War is an ugly thing, but not the ugliest of things; the decayed and degraded state of moral and patriotic feeling which thinks nothing worth a war, is worse. A man who has nothing which he cares more about than he does about his personal safety is a miserable creature who has no chance at being free, unless made and kept so by the exertions of better men than himself." ( John Stuart Mill )

Better men, indeed.

Choux rated this answer Excellent or Above Average Answer
excon rated this answer Excellent or Above Average Answer

Question/Answer
excon asked on 11/10/05 - Prisoner Abuse


Hello experts:

Without going back to read your previous responses, I would bet that some of you (and you know who you are), said that the abuses at Abu Grahib were the result of just a few rouge lower level soldiers.

In light of the recent exposure of a chain of CIA prisons in Asia, and Cheney's attempt to exempt parts of the goverment from torturing, while Bush says that WE don't torture......

It is rather confusing, I know. I guess it is simpler just to deny the whole thing is happening and go out for a burger.

excon

tomder55 answered on 11/10/05:

I don't consider General Karpinski as low level .

excon rated this answer Excellent or Above Average Answer

Question/Answer
excon asked on 11/10/05 - Ethics???


Hello Bush dudes:

If ethics AREN'T a problem at the white house, why would classes be undertaken now? Shouldn't senior government officials already KNOW about ethics in government?

I'll be interested in your spin on this one.

excon

tomder55 answered on 11/10/05:

no spin ;I think it is a terrible over-reaction by the West Wing to publically announce these ethics classes. Bad politics...real bad.

But ;I have worked in the same industry for years and we periodically get refresher courses in a number of issues that we have vast experiences in ;including but not limited to standard procedures ;regulatory issues ;saftey etc. so it is not unusual for classes to be regularily conducted .

excon rated this answer Excellent or Above Average Answer

Question/Answer
ETWolverine asked on 11/10/05 - An interesting quote from Joe Wilson

"I was convinced before the war that the threat of weapons of mass destruction in the hands of Saddam Hussein required a vigorous and sustained international response to disarm him. Iraq possessed and had used chemical weapons; it had an active biological weapons program and quite possibly a nuclear research program all of which were in violation of United Nations resolutions. Having encountered Mr. Hussein and his thugs in the run-up to the Persian Gulf war of 1991, I was only too aware of the dangers he posed."

Published on Sunday, July 6, 2003 by the New York Times, "What I Didn't Find in Africa", by Joseph C. Wilson 4th

Interesting, hmmm? I think this is the only truth Joe Wilson has uttererd in the past four years.

tomder55 answered on 11/10/05:

and that wasn't the only time he said something like it . Wilson, in a speech he delivered three months after the invasion at the Education for Peace in Iraq Center, made the following remark: I remain of the view that we will find biological and chemical weapons and we may well find something that indicates that Saddams regime maintained an interest in nuclear weapons.

ETWolverine rated this answer Excellent or Above Average Answer
excon rated this answer Excellent or Above Average Answer

Question/Answer
Choux asked on 11/09/05 - Overturn Roe V. Wade

Cut and Paste from Huffingto Blog follows:

"In this quiet time as the left and the right polish their swords, huddle with their strategists, and burnish their armor, in these weeks before Judge Samuel Alito goes before the Senate Judiciary Committe in January (a veritable lifetime away in the ADD world of Beltway affairs), the lines are being drawn, and Roe v. Wade is once again the battleground.
The presumption is that Alito is primed to rule against Roe. Whether he would or wouldnt, the Democrats are prepared to oppose him.

But what if they did something utterly unpredictable? What if the Democrats simply decided to walk away from this particular battle, a tactical retreat that no Republican in their right mind (pun intended) expects? What if, yes, the Democrats decided that to let those proverbial chips fall where they may, and allow for the possibility that the Supreme Court just might overturn Roe and declare that there is no constitutional right to abortion.

The Democrats would be far better off.

To begin with, lets once again lay to rest a popular canard: overturning Roe would not, repeat would not, make abortion illegal. That simple truth aint so simple. In fact, if you stopped ten people on the streets of New York and Los Angeles, where its fair to say support for Roe runs high, high, high, seven, eight, or even nine would say that if Supremes overturn Roe, were back to the days of dark alleys and wire hangers.

Not true.

If Roe disappears, very little changes - at first. Roe enshrines a federal, constitutional right to privacy, which in turn bars state legislatures from passing laws making abortion illegal. Before Roe, nothing stood in the way of states making abortion legal. Post Roe, nothing would change in the Blue States.

But in many Red states, abortion is already de facto restricted. Try finding an abortion clinic in rural Alabama, Mississippi or Georgia. Abortion is already socially illegal in many parts of the country; mores often matter more than laws.

But if Roe is overturned, suddenly, every state would be forced to discuss and debate, and that would propel the Democrats from defense to offense. Imagine: in Red states where Democrats have been portraying themselves as kinder, gentler Republicans, there would suddenly be an opportunity to debate choice, privacy, state power versus individual freedoms, morality, life, death, and science. Would they win everywhere? Hardly. But they would be central and relevant about issues that engender passion and heated disagreements.

Unfortunately, this issue is a third rail for the left, a cow so sacred that even this mere mention of retreating on Roe triggers, shall we say, sharp reactions. Come Alito time early next year, it looks like abortion will take center stage, and the Democrats once again will be forced on the defensive. The agenda will be set for them, and they will, if form holds, respond by trying to out-shrill their opponents. Alito will be confirmed, and the Democrats will have solidified their base and moved the needle not at all."-Zachary


Comments?
"

tomder55 answered on 11/10/05:

that has been my basic position for years. the reason that abortion is not a hot button issue in Europe is because it was decided legislatively . the reason it is here is because it was decided by judicial fiat .Roe hijacked the normal democratic process .

Edward Lazarus is a former clerk to Harry Blackmun.Here is his take as a pro-choice opponent to Roe:

As a matter of constitutional interpretation and judicial method, Roe borders on the indefensible. I say this as someone utterly committed to the right to choose, as someone who believes such a right has grounding elsewhere in the Constitution instead of where Roe placed it, and as someone who loved Roes author like a grandfather.

What, exactly, is the problem with Roe? The problem, I believe, is that it has little connection to the Constitutional right it purportedly interpreted. A constitutional right to privacy broad enough to include abortion has no meaningful foundation in constitutional text, history, or precedent - at least, it does not if those sources are fairly described and reasonably faithfully followed.


[The Lingering Problems with Roe v. Wade, and Why the Recent Senate Hearings on Michael McConnells Nomination Only Underlined Them, FindLaw Legal Commentary, Oct. 3, 2002]


Even former ACLU lawyer Justice Ruth Bader Ginsburg agrees :

Heavy-handed judicial intervention was difficult to justify and appears to have provoked, not resolved, conflict.[(North Carolina Law Review, 1985)]

Now the decision is subject to stare decisis even though it is a flawed ruling .

Choux rated this answer Excellent or Above Average Answer
ETWolverine rated this answer Excellent or Above Average Answer

Question/Answer
Choux asked on 11/09/05 - French Riots

"Projecte Quel Dommage is a spontaneous outpouring of blogger concern for our effete neighbors across the Atlantic, who are currently experiencing a greater rate of spontaneous Citroen fires than normal. Were going to create real change by holding up pieces of paper. Please show your support by joining this important cause."



Write the following on a piece of paper and hold it up to your computer screen.

So Long and
thanks for
the
Statue of Liberty



HeeHee

tomder55 answered on 11/09/05:

A Chirac adviser and Arabist calls for new Andalusia in which Muslims and Chrsitians would create a new culture. Andalusia was ruled by Muslims;Christians lived in second class status as dhimmis. Not a great way to deal with a supremacist ideology.... but certainly the French way.

Choux rated this answer Excellent or Above Average Answer

Question/Answer
Itsdb asked on 11/09/05 - Texas election

The voters in Texas overwhelmingly approved a constitutional amendment defining marriage as only between one man and one woman, roughly 76% to 24%. The residents of White Settlement voted 9 to 1 against changing the name of the town to West Settlement.

What does that say about Texans? Are we just a bunch of racist homophobes?

One local resident addressed the marriage amendment results this way, "My concern is that it will legitimize hate," said Bekki McQuay, president of Outstanding Amarillo, a gay and lesbian advocacy group. "And for the people who already hate, now they're backed up by the government and supposedly by the people. It's pretty disturbing."

Is Bekki right, or do the results say something entirely different?

Steve

tomder55 answered on 11/09/05:

that ballot initiative would pass in almost every State if it were on the ballot ;most of them by simular numbers.

Choux rated this answer Excellent or Above Average Answer
Itsdb rated this answer Excellent or Above Average Answer

Question/Answer
ETWolverine asked on 11/09/05 - Evidence of WMDs.

For all those who claim we never found WMDs in Iraq...

From http://www.strategypage.com/strategypolitics/articles/20031004.asp

What Weapons of Mass Destruction Evidence Have We Found In Iraq? Excerpts from David Kay's Report

by Dan Masterson
October 4, 2003

If your where to listen to the talking heads on the various mainstream media outlets, or read the commentary and reporting in the mainstream newspaper, it would seem that Saddam had repented of his previous weapons of mass destruction sins and had begun to emulate the life of Mahatma Ghandi.

The following are excerpts from David Kays testimony to the House and Senate Committees. They may not have found a Fat Man sitting in Saddams kitchen (neither Saddam or a nuclear bomb) but they have found some fairly scary things. Read on and decide yourself. Also, at the bottom of the page there are photographs of some of the evidence that has been found or destroyed.

...

Why are we having such difficulty in finding weapons or in reaching a confident conclusion that they do not exist or that they once existed but have been removed? Our search efforts are being hindered by six principal factors:

From birth all of Iraq's WMD activities were highly compartmentalized within a regime that ruled and kept its secrets through fear and terror and with deception and denial built into each program;

Deliberate dispersal and destruction of material and documentation related to weapons programs began pre-conflict and ran trans-to-post conflict;


Post-OIF looting destroyed or dispersed important and easily collectable material and forensic evidence concerning Iraq's WMD program. As the report covers in detail, significant elements of this looting were carried out in a systematic and deliberate manner, with the clear aim of concealing pre-OIF activities of Saddam's regime;


Some WMD personnel crossed borders in the pre/trans conflict period and may have taken evidence and even weapons-related materials with them;


Any actual WMD weapons or material is likely to be small in relation to the total conventional armaments footprint and difficult to near impossible to identify with normal search procedures. It is important to keep in mind that even the bulkiest materials we are searching for, in the quantities we would expect to find, can be concealed in spaces not much larger than a two car garage;


The environment in Iraq remains far from permissive for our activities, with many Iraqis that we talk to reporting threats and overt acts of intimidation and our own personnel being the subject of threats and attacks. In September alone we have had three attacks on ISG facilities or teams: The ISG base in Irbil was bombed and four staff injured, two very seriously; a two person team had their vehicle blocked by gunmen and only escaped by firing back through their own windshield; and on Wednesday, 24 September, the ISG Headquarters in Baghdad again was subject to mortar attack.

... We have discovered dozens of WMD-related program activities and significant amounts of equipment that Iraq concealed from the United Nations during the inspections that began in late 2002. The discovery of these deliberate concealment efforts have come about both through the admissions of Iraqi scientists and officials concerning information they deliberately withheld and through physical evidence of equipment and activities that ISG has discovered that should have been declared to the UN. Let me just give you a few examples of these concealment efforts, some of which I will elaborate on later:

A clandestine network of laboratories and safehouses within the Iraqi Intelligence Service that contained equipment subject to UN monitoring and suitable for continuing CBW research.

A prison laboratory complex, possibly used in human testing of BW agents, that Iraqi officials working to prepare for UN inspections were explicitly ordered not to declare to the UN.

Reference strains of biological organisms concealed in a scientist's home, one of which can be used to produce biological weapons.

New research on BW-applicable agents, Brucella and Congo Crimean Hemorrhagic Fever (CCHF), and continuing work on ricin and aflatoxin were not declared to the UN.

Documents and equipment, hidden in scientists' homes, that would have been useful in resuming uranium enrichment by centrifuge and electromagnetic isotope separation (EMIS).

A line of UAVs not fully declared at an undeclared production facility and an admission that they had tested one of their declared UAVs out to a range of 500 km, 350 km beyond the permissible limit.

Continuing covert capability to manufacture fuel propellant useful only for prohibited SCUD variant missiles, a capability that was maintained at least until the end of 2001 and that cooperating Iraqi scientists have said they were told to conceal from the UN.

Plans and advanced design work for new long-range missiles with ranges up to at least 1000 km - well beyond the 150 km range limit imposed by the UN. Missiles of a 1000 km range would have allowed Iraq to threaten targets through out the Middle East, including Ankara, Cairo, and Abu Dhabi.

Clandestine attempts between late-1999 and 2002 to obtain from North Korea technology related to 1,300 km range ballistic missiles --probably the No Dong -- 300 km range anti-ship cruise missiles, and other prohibited military equipment.

In addition to the discovery of extensive concealment efforts, we have been faced with a systematic sanitization of documentary and computer evidence in a wide range of offices, laboratories, and companies suspected of WMD work. The pattern of these efforts to erase evidence - hard drives destroyed, specific files burned, equipment cleaned of all traces of use - are ones of deliberate, rather than random, acts. For example,

On 10 July 2003 an ISG team exploited the Revolutionary Command Council (RCC) Headquarters in Baghdad. The basement of the main building contained an archive of documents situated on well-organized rows of metal shelving. The basement suffered no fire damage despite the total destruction of the upper floors from coalition air strikes. Upon arrival the exploitation team encountered small piles of ash where individual documents or binders of documents were intentionally destroyed. Computer hard drives had been deliberately destroyed. Computers would have had financial value to a random looter; their destruction, rather than removal for resale or reuse, indicates a targeted effort to prevent Coalition forces from gaining access to their contents.

All IIS laboratories visited by IIS exploitation teams have been clearly sanitized, including removal of much equipment, shredding and burning of documents, and even the removal of nameplates from office doors.

Although much of the deliberate destruction and sanitization of documents and records probably occurred during the height of OIF combat operations, indications of significant continuing destruction efforts have been found after the end of major combat operations, including entry in May 2003 of the locked gated vaults of the Ba'ath party intelligence building in Baghdad and highly selective destruction of computer hard drives and data storage equipment along with the burning of a small number of specific binders that appear to have contained financial and intelligence records, and in July 2003 a site exploitation team at the Abu Ghurayb Prison found one pile of the smoldering ashes from documents that was still warm to the touch.
...

Following are photographs of some of the goodies that have been found. (Sorry, I couldn't post the pictures themselves. You can see them at the website I listed above. --- Elliot)

Vials: A total of 97 vials-including those with labels consistent with the al Hakam cover stories of single-cell protein and biopesticides, as well as strains that could be used to produce BW agents-were recovered from a scientist's residence.


Lab Equipment From Mosque.


Burned Documents Found at SAAD Center: An exploitation team on a recent mission to the SAAD Center, part of the Baghdad New Nuclear Design Center, found massive looting and the remnants of deliberately destroyed documents. Other documents were left untouched, however, and recovered by the team


Storage room in basement of Revolutionary Command Council Headquarters. Burned frames of PC workstations visible on shelves. All rooms sharing walls with this storage room were untouched from fire or battle damage.

---------------

I have trouble with the claims that we didn't find WMDs in Iraq as a justification for the claim thaqt "Bush lied". Here is evidence of WMDs. All the components needed to make the bio and chemical weapons are there: the missiles, the fuel, the bio and chemical materials, the labs and the sanitized documentation.

OK. I grant you all that we didn't find any large stockpiles of read-to-launch missiles. But how hard is it to understand that with just a few day's or week's work, Saddam could have had an entire bio- and chemical arsenal at his disposal? And that he probably DID have one that he moved elsewhere (ie: Iran or Syria).

C'mon guys. Don't you think that its time to put this "Bush lied" stuff to bed? It's not true, we all know its not true, and we have the evidence to back it up. Furthermore, Bush never said anything that wasn't also said by the very critics who are claiming that he lied. And everything he and they said is true.

Enough with the "Bush lied" crap already.

Elliot

tomder55 answered on 11/09/05:

If he had large stock piles even Hans Blix would've been able to find them .

Ironically 500 tons of yellow cake that the UN had sealed was found at the Tuwaitha site, along with 300 tons of radioisotopes including Cesium-137 and Cobalt-60 . ( 500 tons of uranium is enough to create 142 nuclear weapons....eat that Joe Wilson!)Heck ;the UN complained about 1.8 tons of enriched uranium the US removed from Iraq.

ETWolverine rated this answer Excellent or Above Average Answer

Question/Answer
Choux asked on 11/08/05 - Shoots Foot

cut and paste from Huffington Post Blog::

"Too funny. Hastert and Frist make a big show of calling for an investigation into a leak allegedly affecting national security -- the locations of secret "black site" torture prisons. And then -- BOOM!!! Lott just said, Tuesday afternoon, that he thinks it was a GOP Senator who leaked the info to the Washington Post last week. He says the details had been discussed at a GOP Senators-only meeting last week, and that many of those details made it into the WaPo story.

Money quote from Lott; "We can not remain silent. We have met the enemy, and it is us."

All just reported on CNN. We are, folks, witnessing the full-on implosion of the national Republican Party. And not a second too soon."


Hmmmmm....I wonder?

tomder55 answered on 11/09/05:

whoever leaked it I don't care . It does much more damage that the outing of a desk jockey at the CIA because it puts our allies involved in a spot . . Let the investigation begin.

Choux rated this answer Excellent or Above Average Answer
excon rated this answer Excellent or Above Average Answer

Question/Answer
Choux asked on 11/08/05 - Democrat probable winner in NewJersey Race

"Exit polls for the New Jersey governors' race show democratic Senator Jon Corzine is the decisive winner over Republican businessman Doug Forrester.

The Huffington Post has learned that Senator Corzine wants to appoint acting New Jersey Governor Richard Codey to fill his Senate seat.

Corzine must resign from the Senate in January to assume his position as governor. His Senate term, however, does not end until 2007. State law gives the governor the power to appoint a replacement for the seat. Gov. Richard Codey has been acting governor since Jim McGreevey's resignation took effect on November 15, 2004."


I have no idea of the qualifications of either candidate, one or both could be hatchet murderers, but here we go, first sign that America wants to sweep out the Radical Republicans!

tomder55 answered on 11/09/05:

In 1993 both the Whitman win in NJ and the Allen win in Va. foretold of the 1994 Republican win in Congress in 1994 . It would be foolish if the Republicans were not concerned about the Corzine and the Kaine Democrat wins in NJ and Va. . In NJ the democrat machine has been staggered by cooruption . It took dowm both Gov. McGrevey and the torch Sen. Torricelli in recent years . Still Corizine won a billionaire's slug out . True he outspent Forrester but I do not buy into the theory that the most money wins. Forrester had plenty of opportunity to get his message out. Although the campaign was not run as a national referendum ;that clearly was the deciding factor.

The message to Republicans ;and they better listen . When the people are forced to choose between a Democrat and a party that acts like the Democrats the people will opt for the real thing.

Choux rated this answer Excellent or Above Average Answer

Question/Answer
Choux asked on 11/07/05 - OK, So We Got Even for 9-11

NOW WHAT?????

WE IN AMERICA ARE WORSE OFF THAN WE COULD EVER HAVE IMAGINED.

TONY BLAIR SHOULD HAVE KNOWN BETTER, SHOULDN'T HE???

tomder55 answered on 11/08/05:

so long as we continue to treat the enemies of the US with political correctness and don't sell this war against jihadistan for what it is ;a world war;a clash of civilizations that cannot easily coexist then we will continue to live under the illusion that the Afghanistan campaign was the end of the war instead of the first of many offensives we need to make before the war is over .I thought you understood that . Tony Blair is a visionary ;but even he does not spell out what is at stake.

So long as we treat captured enemy as people who deserve the same due process as a free citizen then yes we are in trouble .

Here is an example of what to expect when they are left in even the most secure prisons. Louis Pepe, a federal security guard, was assigned to a high-security wing at the Metropolitan Correction Center (MCC) reserved for some of the most dangerous international terrorists being held in federal custody including several alleged al Qaeda operatives. On November 1, 2000, Pepe was brutally attacked by Mamdouh Mahmud Salim, a former top aide to Osama bin Laden who was awaiting trial on charges that included attempting to purchase nuclear weapons components.

Salim used hot sauces purchased in the prison to temporarily blind Mr. Pepe and then thrust a sharpened plastic comb three inches into Pepes left eye, resulting in severe brain damage, partial paralysis, the loss of his left eye and the loss of 60 percent of the vision in his right eye. Notes later found in Salims cell indicate that the attack was part of a plan to seize hostages in an effort to break out of jail.

Salim still faces trial and a possible life sentence on conspiracy charges in the 1998 bombings of two U.S. embassies in Africa. Salim has also said he attacked Mr. Pepe to try to get the prison keys and stab his two court-appointed lawyers waiting in a nearby conference room, so that the judge in the terrorism case would be forced to appoint him new lawyers.

Choux rated this answer Excellent or Above Average Answer
ETWolverine rated this answer Excellent or Above Average Answer
Itsdb rated this answer Excellent or Above Average Answer
sapphire630 rated this answer Excellent or Above Average Answer

Question/Answer
CeeBee2 asked on 11/06/05 - Avian flu and Rummy

Front Page
Nov 4, 2005

SPEAKING FREELY
Rummy's bird flu bonanza
By F William Engdahl

(Speaking Freely is an Asia Times Online feature that allows guest writers to have their say.)

No sooner are indictments being handed down to I Lewis "Scooter" Libby, chief of staff of the vice president of the United States for lies and coverup regarding information used deliberately to suppress the fact the Bush administration had no "smoking gun" to prove Saddam Hussein was building a nuclear arsenal, but a new scandal is surfacing, every bit as outrageous and ultimately, likely also criminal.

The world population is being whipped into a fear frenzy by irresponsible public health officials from the US administration to the World Health Organization (WHO) to the United States Centers for Disease Control. They all warn about the imminent danger that the bird flu virus might mutate into a malicious strain that is transmissible between humans, contaminating the human species in pandemic proportions. Often the flu pandemic of 1918, which is said to have killed 18 million worldwide, is cited as an example of what "might" lie in store for us.

On November 1, appropriately enough the day after Halloween, President George W Bush visited the National Institutes of Health in Bethesda, Maryland, to announce his administration's strategy for preparing for the next flu epidemic, whether from bird flu or some other strain. The plan has been a year in the making. It was no small presidential photo op. The secretaries of state, homeland security, agriculture, health and human services, transportation, and veteran affairs, as well as the director general of the World Health Organization, who flew in from Geneva for the event, were at the president's side.

Bush began his remarks with the now-obligatory scare-story from 1918: "At this moment, there is no pandemic influenza in the United States or the world. But if history is our guide, there is reason to be concerned. In the last century, our country and the world have been hit by three influenza pandemics - and viruses from birds contributed to all of them. The first, which struck in 1918, killed over half-a-million Americans and more than 20 million people across the globe ..."

He was remarkably candid about the imminent danger to the American people: "Scientists and doctors cannot tell us where or when the next pandemic will strike, or how severe it will be, but most agree: at some point, we are likely to face another pandemic. And the scientific community is increasingly concerned by a new influenza virus known as H5N1 - or avian flu ..."

He went on to stress, "At this point, we do not have evidence that a pandemic is imminent. Most of the people in Southeast Asia who got sick were handling infected birds. And while the avian flu virus has spread from Asia to Europe, there are no reports of infected birds, animals, or people in the United States. Even if the virus does eventually appear on our shores in birds, that does not mean people in our country will be infected. Avian flu is still primarily an animal disease. And as of now, unless people come into direct, sustained contact with infected birds, it is unlikely they will come down with avian flu."

Despite all this, the president called on Congress to immediately pass a new US$7.1 billion in emergency funding to prepare for that not-imminent not-pandemic danger. Now that's precaution. Prominent among his list of emergency measures was a call for Congress to appropriate another $1 billion for Tamiflu.

On October 28, the Senate passed an $8 billion emergency funding bill to address the growing avian flu panic. Health and Human Services Secretary Mike Leavitt, in a moment of candor during the debate on the Senate bill, told the press, "If it isn't the current H5N1 virus that leads to an influenza pandemic, at some point in our nation's future, another virus will."

If a meteorite doesn't hit Washington, DC, in the next days, someday it might ... In the meantime, taxpayer billions will have gone to a handful of pharmaceutical giants positioned to profit. None stands to reap more lucre than the Swiss-US firm, Roche Holdings of Basle.

The only medicine, we are told, which reduces the symptoms of avian flu is a drug called Tamiflu. Today Roche holds the sole license to manufacture Tamiflu. Due to the panic, the order books at Roche are filled to overflowing.

However, the real point of interest is the company in California which developed Tamiflu and gave the marketing rights for its patented discovery to Roche.

Rummy flu
Tamiflu was developed and patented in 1996 by a California biotech firm, Gilead Sciences Inc. Gilead is a NASDAQ-listed stock company which prefers to maintain a low profile in the current rush to Tamiflu. That might be because of who is tied to Gilead. In 1997, before he became Pentagon chief, Donald Rumsfeld was named chairman of the board of Gilead Sciences, where he remained until early 2001 when he became defense secretary. Rumsfeld had been on the board of Gilead since 1988, according to a 1997 company press release.

Rumsfeld holds a Gilead stake valued at between $5 million and $25 million, according to his federal financial disclosures. In the past six months, the global rush to buy Tamiflu has sent Gilead's stock from $35 to $47 - amounting to a windfall of at least $1 million for Rumsfeld. And now, with Gilead collecting royalties averaging 10% from Roche's sales of Tamiflu, he is poised to reap more gains for a flu panic his administration has done everything it can to promote.

Gilead Sciences is no small-time biotech startup. Its board today includes Bechtel Corp director and former secretary of state George Shultz (Bechtel is right up there with Halliburton in contracting to rebuild Iraq), Gordon Moore of Intel, and Viscount Etiene Davignon, a Belgian who seems to be involved in everything big and Atlanticist, whether it be Bilderberg meetings or trilateral commissions.

The Gilead model suggests a parallel to the brazen corruption of Halliburton, whose former CEO is Vice President Dick Cheney. Cheney's company has so far gotten billions worth of US construction contracts in Iraq and elsewhere. And Cheney's closest political friend is Don Rumsfeld.

It's another example of what someone has called the principle of modern US corrupt special interest politics: '"Concentrate the benefits; diffuse the costs." Bush had ordered the US government to buy $2 billion worth of Tamilflu - that was before his November 1 speech calling for another $1 billion worth.

Small pox, big bucks
It appears that the defense secretary is quite an accomplished hand at getting the government to buy vaccines from companies in which he has a direct financial interest. Recall the scare just after September 11, 2001 when the Bush administration was talking loudly about the "possible" danger of Osama bin Laden (for those of you who may have forgotten who he was, he was the man who was cited as the reason the Bush administration launched its "war on terror") releasing a deadly smallpox attack that would devastate the American population.

Fortunately, the administration was equally vigilant then as it is today against bird flu pandemics. Rumsfeld at that time ordered members of the armed forces to be inoculated against smallpox, an inoculation with horrendous side effects. The package also included injection with a drug named Vistide, to treat side-effects of smallpox infection should it occur.

Vistide was also a product of Gilead Sciences, and Rumsfeld was the person who signed off on the decision to give US troops Vistide. We can be sure that the men and women of the US armed forces will be among the first this time, as well, to get Tamiflu from their ever-vigilant chief. Curious that the Washington Post doesn't investigate this prima facie conflict of interest involving the defense secretary, at a time the media seem to have discovered administration lies about Iraqi weapons of mass destruction are finally worth reporting. Perhaps they think readers can only handle one scandal at a time.

GMO chickens come home to roost
But Tamiflu conflicts of interest are perhaps just the tip of the iceberg of the avian flu story. There is high-level biological research under way in Britain and presumably also the United States to develop a genetic engineering method to make chickens and other birds resistant to avian flu viruses.

Laurence Tiley of Cambridge University and Helen Sang of the Roslin Institute in Scotland are involved in developing "transgenic chickens" that would involve genetic material inserted into eggs to allegedly make the chickens H5N1-resistant.

Tiley told the Times of London on October 29, "Once we have regulatory approval, we believe it will only take between four and five years to breed enough chickens to replace the entire world [chicken] population." The real question in this dubious undertaking is which GMO (genetically modified organisms) giants are underwriting the research and development of GMO chickens and who will control their products. It is increasingly clear that the entire saga of avian flu is one whose dimensions are only slowly coming to light. What we can see so far is not at all pretty.

******************
F William Engdahl is author of A Century of War: Anglo-American Oil Politics and the New World Order, Pluto Press, and the soon-to-be released book, Seeds of Destruction: The Geopolitics of Gene-ocide. He can be contacted through his website.


tomder55 answered on 11/07/05:

last year the adm came under fire for a shortage of flu injections . either we are not prepared or we are fanning the flames. gimee a break .

CeeBee2 rated this answer Excellent or Above Average Answer

Question/Answer
CeeBee2 asked on 11/05/05 - Ending the Fraudulence about the Reverse Midas Touch

Ending the Fraudulence
By Paul Krugman
The New York Times

Monday 31 October 2005

Let me be frank: it has been a long political nightmare. For some of us, daily life has remained safe and comfortable, so the nightmare has merely been intellectual: we realized early on that this administration was cynical, dishonest and incompetent, but spent a long time unable to get others to see the obvious. For others - above all, of course, those Americans risking their lives in a war whose real rationale has never been explained - the nightmare has been all too concrete.

So is the nightmare finally coming to an end? Yes, I think so. I have no idea whether Patrick Fitzgerald, the special prosecutor, will bring more indictments in the Plame affair. In any case, I don't share fantasies that Dick Cheney will be forced to resign; even Karl Rove may keep his post. One way or another, the Bush administration will stagger on for three more years. But its essential fraudulence stands exposed, and it's hard to see how that exposure can be undone.

What do I mean by essential fraudulence? Basically, I mean the way an administration with an almost unbroken record of policy failure has nonetheless achieved political dominance through a carefully cultivated set of myths.

The record of policy failure is truly remarkable. It sometimes seems as if President Bush and Mr. Cheney are Midases in reverse: everything they touch - from Iraq reconstruction to hurricane relief, from prescription drug coverage to the pursuit of Osama - turns to crud. Even the few apparent successes turn out to contain failures at their core: for example, real G.D.P. may be up, but real wages are down.

The point is that this administration's political triumphs have never been based on its real-world achievements, which are few and far between. The administration has, instead, built its power on myths: the myth of presidential leadership, the ugly myth that the administration is patriotic while its critics are not. Take away those myths, and the administration has nothing left.

Well, Katrina ended the leadership myth, which was already fading as the war dragged on. There was a time when a photo of Mr. Bush looking out the window of Air Force One on 9/11 became an iconic image of leadership. Now, a similar image of Mr. Bush looking out at a flooded New Orleans has become an iconic image of his lack of connection. Pundits may try to resurrect Mr. Bush's reputation, but his cult of personality is dead - and the inscription on the tombstone reads, "Brownie, you're doing a heck of a job."

Meanwhile, the Plame inquiry, however it winds up, has ended the myth of the administration's monopoly on patriotism, which was also fading in the face of the war.

Apologists can shout all they like that no laws were broken, that hardball politics is nothing new, or whatever. The fact remains that officials close to both Mr. Cheney and Mr. Bush leaked the identity of an undercover operative for political reasons. Whether or not that act was illegal, it was clearly unpatriotic.

And the Plame affair has also solidified the public's growing doubts about the administration's morals. By a three-to-one margin, according to a Washington Post poll, the public now believes that the level of ethics and honesty in the government has declined rather than risen under Mr. Bush.

So the Bush administration has lost the myths that sustained its mojo, and with them much of its power to do harm. But the nightmare won't be fully over until two things happen.

First, politicians will have to admit that they were misled. Second, the news media will have to face up to their role in allowing incompetents to pose as leaders and political apparatchiks to pose as patriots.

It's a sad commentary on the timidity of most Democrats that even now, with Lawrence Wilkerson, Colin Powell's former chief of staff, telling us how policy was "hijacked" by the Cheney-Rumsfeld "cabal," it's hard to get leading figures to admit that they were misled into supporting the Iraq war. Kudos to John Kerry for finally saying just that last week.

And as for the media: these days, there is much harsh, justified criticism of the failure of major news organizations, this one included, to exert due diligence on rationales for the war. But the failures that made the long nightmare possible began much earlier, during the weeks after 9/11, when the media eagerly helped our political leaders build up a completely false picture of who they were.

So the long nightmare won't really be over until journalists ask themselves: what did we know, when did we know it, and why didn't we tell the public?

***********************************

Will journalists start asking each other those questions?

tomder55 answered on 11/07/05:

lol ;the media has been on a crusade against this administration from day one. now they think they have given Bush a mulligan? Haaaa!

CeeBee2 rated this answer Excellent or Above Average Answer

Question/Answer
Choux asked on 11/04/05 - IMPEACHMENT

New Poll: Majority of Americans Support Impeachment
Submitted by davidswanson on Fri, 2005-11-04 04:24. Activism

For Immediate Release: November 4, 2005

New Poll Shows Majority of Americans Support Impeachment;
ImpeachPAC is Launched to Support Pro-Impeachment Candidates

"By a margin of 53% to 42%, Americans want Congress to impeach President Bush if he lied about the war in Iraq, according to a new poll commissioned by AfterDowningStreet.org, a grassroots coalition that supports a Congressional investigation of President Bush's decision to invade Iraq in 2003.

The poll was conducted by Zogby International, the highly-regarded non-partisan polling company. The poll interviewed 1,200 U.S. adults from October 29 through November 2.

The poll found that 53% agreed with the statement:

"If President Bush did not tell the truth about his reasons for going to war with Iraq, Congress should consider holding him accountable through impeachment."

42% disagreed, and 5% said they didn't know or declined to answer. The poll has a +/- 2.9% margin of error.

"These results are stunning," said AfterDowningStreet.org co-founder Bob Fertik. "A clear majority of Americans now supports President Bush's impeachment."

tomder55 answered on 11/05/05:

Media coverage of polling results often neglects to mention the self-interestedness of the sponsor, and John Zogby is a leading enabler.His is hardly an 'independent ' polling organization despite the general acclaim . John Zogby polled for a Democratic opponent to Rep. Tom DeLay paying out of his own pocket because he wanted to provide a "fresh challenge" to the congressman. Zogby describes his personal political history as "very left Democrat." His brother James, heads the Arab American Institute,and was an adviser to Al Gore's presidential campaign.

In addition to being a spokesman for the Wahabists, Zogby is regarded as a con man for hire , who manipulates results to suit his clients and further their stated agendas .In the past half-decade numerous Zogby Polls for various special interests have relied on creative phrasing to give the impression of wide public support for the view that the given client is promoting.Back in 1996,Zogby hit the bullseye in predicting the results of the Presidential election;and that was his polling zenith . In 2002, Zogby appeared to show a lean in favor of the Democrats, and he was way off in his mid-term
election predictions.Last year , at the end of the spring, Zogby actually came out and predicted John Kerry would win the election, which showed that his bias had
reached the point of full-blown partisanship against the President.Zogby's refusal to show his work, only magnifies the apparent distortion of his results.

CeeBee2 rated this answer Excellent or Above Average Answer
Choux rated this answer Excellent or Above Average Answer

Question/Answer
Choux asked on 11/04/05 - How Ironic

I remember when Senator Durbin (D ILL) made a speech that compared how America treated Islamic prisoners to a Soviet-style gulag.

Wellll, now we learn that the situation wasn't just a metaphorical gulag, BUT REAL CHAINS OF DETENTIONS CAMPS IN EASTERN EUROPE....LITERAL GULAGS!!!

tomder55 answered on 11/04/05:

Natan Saransky was a resident of the gulags .I think he would see the comparison as absurd .

Choux rated this answer Excellent or Above Average Answer
excon rated this answer Excellent or Above Average Answer
jnlomonte rated this answer Poor or Incomplete Answer

Question/Answer
Choux asked on 11/03/05 - Rove's Dying Dream

Karl Rove's Dying Dream
So much for the permanent Republican majority.
By Jacob Weisberg
Posted Wednesday, Nov. 2, 2005, at 5:18 PM ET



"Karl Rove's dream is dying. This is happening for reasons that have nothing to do with Valerie Plame.

Rove's dream was to reshape American politics by creating a durable Republican majority. In the old days, Rove told anyone who would listen that his role model in this project was the legendary political boss Mark Hanna.


The key to McKinley's political success was the alliance Hanna forged between industrialists like himself, who provided the cash, and workers, who provided the votes. In Rove's alliance, the rich provide the cash, and religious conservatives provide the votes. Refuting the conventional wisdom that successful presidential candidates must lay claim to the political center, Bush has governed from the right and won re-election in 2004 with a "base-in," rather than a "center-out," strategy.


When Bush was re-elected, everyone hailed Rove's strategy as a masterstroke. But would Rove's protg have eked out victories in 2000 and 2004 absent special circumstances, lame opponents, and good luck? Less than a year into Bush's second term, the president's approval rating is down around 40 percent. Many things have gone wrong for Bush, but the underlying problem is his relationship to the constituency that elected him. Bush's debt to his big donors and to religious conservatives has boxed him in and pitted him against the national consensus on various issues. His extremism is undermining Rove's realignment.

The problem has become clear with Bush's difficulties in filling Sandra Day O'Connor's slot on the Supreme Court. The Harriet Miers nomination was an attempt to satisfy both the militant conservative base and the eternally moderate American electorate. With the Alito nomination, Bush has acknowledged that splitting this difference is impossible. Faced with a choice, he has chosen, once again, to dance with the ones who brought him. But by appointing a superconservative, Bush risks propelling his increasingly beleaguered administration even further toward the right-hand margina place where his party cannot win future national elections.

Bush aims to be the Second Coming of Ronald Reagan. But he has never understood the genius of Reagan's method, which was to placate the religious right without giving in where it mattered. Reagan could proclaim his undying support for a constitutional amendment to ban abortion without doing anything to endanger Roe v. Wade. (He was the one who nominated O'Connor, remember?) In the same way, Bill Clinton managed to keep liberal interest groups onboard without advancing their politically untenable wish list. But whether because he is less adroit or because he truly believes, Bush seems able to appease his base only by surrendering to its wishes. He has caved to conservatives on Terri Schiavo, on stem-cell research, on Social Security privatization, and on "intelligent design." Now, most important, he is caving by at least creating the appearance that he is trying to get enough votes on the Supreme Court to reverse Roe.

Bush's failure at base-pacification is not entirely his fault. The evangelicals, who were pragmatically willing to settle for half a loaf during the Reagan and Bush 41 years, now feel empowered, emboldened, and owed. James Dobson and Pat Robertson don't understand that they would do their cause the most good by keeping their mouths shut and not scaring everyone witless. Conservatives of all kinds are in a militant mood heightened by their success in muscling Bush on Miers. They do not realize how their militancy alienates not just the left, but the swingers in the center whom Republicans need to win.

Rove is actually the second Republican realigner to stumble in this way in recent years. After the 1994 election, Newt Gingrich had his own visions of political sugarplums. Gingrich's unsuccessful revolution was more libertarian and less moralistic. He thought the new Republican majority would coalesce around shrinking government (a theme Bush has soft-pedaled, preferring to undermine government through neglect and incompetence). Gingrich was also, frankly, a little nuts. But he failed because he made the same basic mistake that Rove did. Gingrich thought he'd won a mandate for radical change and enshrined a new governing majority. He forgot about the country's nonideological majority, which likes Medicare, Social Security, national parks, and student loans. Republicans have retained control of Congress since Gingrich's downfall, but only by reversing his austerity program and spending like a bunch of drunks.


Like McKinley, Bush has a potential successor who would like to change his party's direction. John McCain spouts reform and idolizes Teddy Roosevelt. And oh yeshe and Karl Rove loathe each other."

Question:: Any comments about this inciteful article from Slate Magazine???

tomder55 answered on 11/04/05:

I don't get it . His model is some guy who worked for a 3 year Presidency ;a President who got whacked by an anarchist ?

Now if his goal was to create a Republican majority like FDR forged for the democrats then I would be impressed .

Choux rated this answer Excellent or Above Average Answer

Question/Answer
excon asked on 11/04/05 - Torture


Hello:

My last favorite president, indeed my only favorite, was JFK. All the rest suck. He was my Commander in Chief. When he said go to war, I did.

What I remember most about my youth and my coming of age under his presidency, is the stark picture that was painted of the Soviet gulags. The reasons I went to war for my country, were particularly, those prisons and the torture heaped upon those prisoners, I didnt know much more about them, but that was enough. They were SO different than us.

Now, we are them.

My feelings, however, about a country that would do that have not changed, even though that country is my own. Do you think I should change my views - or my country?

excon

tomder55 answered on 11/04/05:

you really think that no "torture " was sanctioned in 250 years ;or even in the short time of Kennedy's term ? Was there no CIA then ? The difference is that there wasn't small personal video recording devices back then. There is really nothing new here except perhaps that the Bush administration is actually trying to define the parameters of what is acceptable. The other stuff ;secret prisons ;renditions ;outsourcing interrogations ,yup ;they were all in practice . Back in Kennedy's day we had these dictators that were our dictators to handle the messy details .

I heard White House Council Dan Bartlett inteview on Hardball this week . He basically says that the Bush Adm. is acting within the law ,and that any evidence of the contrary will be "aggressively investigated".

You have it backwards . Specific methods of permissible interrogation techniques need to be defined in clear language ;and all personnel need to be trained not to deviate beyond the SOP .It is not good enough to say that torture is like the definition of pornography.That is why drgade and myself continue to ask it . When Pfc. England is in court she should have been able to point to the section of her manual that shows these techinques are acceptable ;or she should point to the person who instructed her to do so.




excon rated this answer Excellent or Above Average Answer

Question/Answer
Choux asked on 11/02/05 - GOP Unity Stability Shaken

"Former House majority leader Tom DeLay's efforts to retain power despite his indictment have angered some rank-and-file Republicans, many of whom say his ethical problems and uncertain status are staining them and destabilizing GOP unity.

Although he was forced to relinquish his leadership post Sept. 28, after the first of two indictments for alleged involvement in money laundering related to the 2002 Texas election, DeLay continues to use an office in the leadership suite, occasionally presides over private meetings with committee chairmen and lobbies members during key floor votes.

Also, the Texas Republican's staff continues to maintain the House schedule and dash off memos to lawmakers, ostensibly as employees of a majority leader's office without a full-fledged majority leader. And on his trips to the sheriff's office for an Oct. 20 booking in Houston and a court appearance in Austin on Oct. 21, DeLay was accompanied by three bodyguards from the Capitol Hill police force, just as he was when he was majority leader.

"My issue is having an indicted former leader hanging around the leadership offices," said one House Republican, who spoke on the condition of anonymity because of DeLay's remaining authority. "This guy did so much good work getting us into the majority. Why does he want to stick around? He's not helping us."

WashingtonPost on line


Comments...

tomder55 answered on 11/03/05:

it doesn't suprise me . there are always people jockeying for position even in unified organizations. There was a simular story going around when Frist replaced Lott in the Senate . Lott is still angry over the White House lack of support ;and he took it out this week on his 'lack of confidence' statement about Rove.

Choux rated this answer Excellent or Above Average Answer

Question/Answer
Choux asked on 11/02/05 - Chalabi to Visit Washington

"Chalabi's visit is the political version of getting the band back together.

And, having orchestrated the greatest career makeover since Paris Hilton went from Internet porn curiosity to Vanity Fair cover girl, Chalabi has now set his sights on becoming Iraq's new prime minister following the next round of voting on Dec. 15.

Not bad for a guy who, less than two years ago, was being accused by the Bush administration of passing intelligence to Iran that could 'get people killed.'

But, apparently, now that Chalabi is a power player in Iraq, all appears forgiven. At least around the White House. The rest of us, on the other hand, would do well to remember that this is still the guy who:

* was a prime source of trumped up claims about Saddam's WMD
* bamboozled the Bushies while pocketing $340,000 a month from the US government
* tried to sabotage the UN's efforts to put in place an interim government in Iraq
* helped the White House Iraq Group sell the war by regularly passing faulty intel to Judy Miller
* introduced Curveball, another bogus source on WMD, to the intelligence community
* was accused of spying for the Iranians
* controlled a group of thugs accused of fraud, torture, kidnapping, and misuse of U.S. funds
* was convicted in abstentia of embezzling millions of dollars in Jordan in the 1980ies.



Comments?

tomder55 answered on 11/03/05:

I have addressed many of these charges on previous postings .

*On the issue of the INC feeding false intel. to the US ;the Robb-Silverman WMD Report (page 108 ) says the following :

"In fact, over all, CIA's post-war investigations revealed that INC-related sources had a minimal impact on pre-war assessments."........."Despite speculation that Curveball was encouraged to lie by the Iraqi National Congress (INC), the CIA's post-war investigations were unable to uncover any evidence that the INC or any other organization was directing Curveball to feed misleading information to the Intelligence Community. Instead, the post-war investigations concluded that Curveball's reporting was not influenced by, controlled by, or connected to, the INC."
(*curveball* being the human source that the CIA and State Dept. relied on heavily for pre-war assessments of Saddam's WMD program ).
The CIA's tried to scapegoat Chalabi for its own failures.


*The Jordanian charges are bogus. Saddam pressured Jordan to charge him .Chalabi and his bank were judged, in 1989, by a specially convened Jordanian military tribunal. This is not the way that banking regulation normally takes place. But it was notorious that the Jordanian military at that time was dependent on its Iraqi big brother and neighbor. And Chalabi had asserted in public that the Jordanian regime had helped Iraq violate various arms embargos by purchasing weaponry allegedly for Jordanian use and then trans-shipping it to Baghdad. A lawsuit was filed by Ahmad Chalabi in the District of Columbia Superior Court.It charges various high officials in the Jordanian government, and the Jordanian monarchy in general, with having sought to loot the Chalabi family's Bank of Petra. The suit further alleges that several named Jordanians used their influence to try to have Chalabi forcibly returned to his native Iraq in 1989, there to be tortured and killed by Saddam Hussein.


*Based on the allegation about Chalabi feeding bogus info. and then the phoney charges that he was an Iranian agent.The CIA broke an Iranian code;a drunken CIA field agent supposedly blabbed it to Chalabi ;and Chalabi then let Iran know the code was broken by using those very same broken codes to inform them ; do you believe that is probable ????? A grad of the U. Chicago would not be that stupid. He fell out of favor with the Bush Adm. (he was already persona non grata in State and CIA before the war.)Shrills like Maureen Dowd were quick to gloat : "Ahmad Chalabi conned his neocon pals, thinking he could run Iraq if he gave the Bush administration the smoking gun it needed to sell the war."

Well on the basis of the Robb-Silverman WMD Report he is exonerated .

He risked his life and his fortune to overthrow one of the worst tyrants of the 20th century. The US would have done better in Iraq if it had been listening to Ahmad Chalabi more . As an example ;

* Chalabi argued before the war that there was a danger of turning the liberation into an occupation.

*It was Chalabi's idea to set up an interum Gvt. He was ignored on the lame excuse that the exiles had no standing in Iraq.Which was ...ahem.... obviously so since Chalabi is now playing a leading role in the new Iraqi elected government.He is a leading candidate for prime minister .

Instead the CPA was set up under the leadership of Viceroy Bremer and we lost 1 valuable year of momentum .After the CPA failure we ended up setting up an interm government anyway.

*Long before the war Chalabi said his dream for Iraq was to create a broad-based popular movement committed to representative government and to the protection of the human rights of all Iraqis (as offering) moral and practical power,.... despite the ethnic and religious complexity of Iraqi society, parties representing all facets are united in this democratic goal. (JINSA Report #481 )


Choux rated this answer Excellent or Above Average Answer

Question/Answer
Choux asked on 11/01/05 - Design Flaws in NO Levees

First of all, I don't think that the New Orleans disaster is in any way Bush's fault.

Design Flaws

By BRETT MARTEL

NEW ORLEANS (AP) - The engineers who designed the floodwalls that collapsed during Hurricane Katrina did not fully consider the porousness of the Louisiana soil or make other calculations that would have pointed to the need for stronger levees with deeper pilings and wider bases, researchers say.

At least one key scenario was ignored in the design, say the researchers, who are scheduled to report their findings at a congressional hearing Wednesday: the possibility that canal water might seep into the dirt on the dry side of the levees, thereby weakening the embankment holding up the floodwalls.

``I'd call it a design omission,'' said Robert Bea, a University of California at Berkeley civil engineering professor who took part in the study for the National Science Foundation.

The research team found other problems in the city's flood-control system, including evidence of poor maintenance and confusion over jurisdiction.

Bea also questioned the margin for error engineers used in their designs, saying the standards - which call for structures to be 30 percent stronger than the force they are meant to stop - date to the ****first half of the 1900s, when most levees were built to protect farmland, not major cities****.

``The center of New Orleans is certainly not protection of farmland, so the factor of safety was incredibly low,'' Bea said. ``We're talking about thousands of families without homes and shutting down a commercial infrastructure that's pretty darn important to the United States.''

While surging waters from the Gulf of Mexico flowed up and over levees east of the city, flooding in central New Orleans and parts of downtown was caused by breaches at barriers along the 17th Street and London Avenue canals, both of which have been built since the late 1980s.

Floodwaters eventually inundated 80 percent of New Orleans and had to be pumped out over weeks because of the city's saucer-like topography.

The UC team is one of three independent teams working with the U.S. Army Corps of Engineers, the agency responsible for the levees' design and construction, to determine why the barriers failed and make recommendations to repair them.

Corps officials have said the barriers were never intended to withstand a storm as powerful as Katrina. Congress instructed them to build a network of levees and floodwalls that could withstand a Category 3 storm similar to Hurricane Betsy, which flooded New Orleans in 1965. Katrina was a Category 4 hurricane when it came ashore Aug. 29.

But since Katrina's center passed to the east of New Orleans, there is debate as to whether the city experienced more than the equivalent of a Category 3 storm.

Bea said the NSF team believes the Corps has suffered from a lack of funding and technical resources over the years.

Paul Mlakar, an Army Corps of Engineers senior research director, said the Corps shares Bea's concerns.

``He raises an interesting question that needs to be looked at,'' Mlakar said. ``If something wasn't done right, we want to be the first to change and make it right.''

Steel-sheet pilings driven into the ground are meant to stop seepage from the wet side of the levee to the dry side and serve as an anchor for the levees' protective, concrete walls. But a number of engineers have said the pilings apparently were not driven deeply enough into the relatively loose, porous soil endemic to southern Louisiana.

The result: Water seeped deep into the ground and destabilized the soil, causing the walls to collapse.

Bea also said that the flood-control system has many jurisdictions involved, and the resulting confusion leaves ``no one minding the store.''

While the Corps is responsible for levee construction, local levee boards take care of most maintenance. In some cases, the state highway department or railroad companies handle maintenance of floodwalls when their rights of way cross the levee system.

A flood gate near the Industrial Canal, which helped inundate parts of east New Orleans, was missing because of damage caused by a train, Bea said. Railroad workers had removed the gate for repairs, and it dispatched employees to fill the gap with sandbags as Katrina approached.

``It didn't hold,'' Bea said. ``There isn't a door, and they've got measly sandbags they're putting in to compensate.''

At another canal, the UC group found a levee built to five different elevations by five different agencies.


11/01/05 17:29

Copyright The Associated Press. All rights reserved. The information contained In this news report may not be published, broadcast or otherwise distributed without the prior written authority of The Associated Press.

Asterisks mine.


Question: Should New Orleans be rebuilt back to the large population that lived there prior to Hurricane Katrina? Is it worth the huge amount of money needed and the cost of landfill if used???




tomder55 answered on 11/02/05:

Design flaw? I thought they were doing okay until the government detonated the explosives!If there was a design foaw then there is no doubt that the US Army Corps of Engineers was involved .The whole agency is a natural disaster far worse than global warming .



Choux rated this answer Excellent or Above Average Answer

Question/Answer
ETWolverine asked on 11/01/05 - This is what they call dirty politics.

GOP furious at closed Senate session
Minority leader holds unusual secret meeting about prewar intelligence

MSNBC staff and news service reports
Updated: 4:42 p.m. ET Nov. 1, 2005


WASHINGTON - Democrats forced the Republican-controlled Senate into an unusual closed session for just more than two hours Tuesday, questioning intelligence that President Bush used in the run-up to the war in Iraq and accusing Republicans of ignoring the issue.

They have repeatedly chosen to protect the Republican administration rather than get to the bottom of what happened and why, Democratic leader Harry Reid said.

Taken by surprise, Republicans derided the move as a political stunt.

The United States Senate has been hijacked by the Democratic leadership, said Majority Leader Bill Frist. They have no convictions, they have no principles, they have no ideas, the Republican leader said.

In a speech on the Senate floor, Reid said the American people and U.S. troops deserved to know the details of how the United States became engaged in the war, particularly in light of the indictment of I. Lewis Scooter Libby, Vice President Dick Cheneys former chief of staff.

Reids spokesperson said the purpose of the meeting was to persuade Republicans to reopen the investigation into the prewar intelligence, MSNBC.coms Tom Curry reported. The Senate Intelligence Committee is chaired by Pat Roberts, R-Kan.

Reid demanded the Senate go into closed session. With a second by Sen. Dick Durbin, D-Ill., the public was ordered out of the chamber, the lights were dimmed, senators filed to their seats on the floor and the doors were closed. No vote is required in such circumstances.

Reids move shone a spotlight on the continuing controversy over intelligence that President Bush cited in the run-up to the war in Iraq. Despite prewar claims, no weapons of mass destruction have been found in Iraq, and some Democrats have accused the administration of manipulating the information that was in their possession.

Provoked by Libby indictment
Vice President Dick Cheneys chief of staff, I. Lewis Scooter Libby, was indicted last Friday in an investigation that touched on the war, the leak of the identity of a CIA official married to a critic of the administrations Iraq policy.

The Libby indictment provides a window into what this is really all about, how this administration manufactured and manipulated intelligence in order to sell the war in Iraq and attempted to destroy those who dared to challenge its actions, Reid said before the doors were closed.

Libby resigned Friday after being indicted on charges of obstruction of justice, making false statements and perjury in an investigation by a special prosecutor into the unauthorized leak of a CIA agents identity.

Reid accused Republicans of playing upon post-9/11 fears as grounds for going to war.

Obviously we know now their nuclear claims were wholly inaccurate, Reid said. But more troubling is the fact that a lot of intelligence experts were telling the Administration then that its claims about Saddam's nuclear capabilities were false.

Reids spokesperson, Jim Manley, said the purpose of this closed session is to persuade Republicans to reopen the intelligence committee investigation into prewar intelligence

Democrats challenging war justification
Democrats contend that the unmasking of Valerie Plame was retribution for her husband, Joseph Wilson, publicly challenging the Bush administrations contention that Iraq was seeking to purchase uranium from Africa. That claim was part of the White Houses justification for going to war.

Sen. Trent Lott, R-Miss., said Reid was making some sort of stink about Scooter Libby and the CIA leak.

A former majority leader, Lott said a closed session is appropriate for such overarching matters as impeachment and chemical weapons the two topics that last sent the senators into such sessions.

In addition, Lott said, Reids move violated the Senates tradition of courtesy and consent. But there was nothing in Senate rules enabling Republicans to thwart Reids effort.

As Reid spoke, Majority Leader Bill Frist met in the back of the chamber with a half-dozen senior GOP senators, including Intelligence Committee Chairman Pat Roberts, who bore the brunt of Reids criticism. Reid said Roberts reneged on a promise to fully investigate whether the administration exaggerated and manipulated intelligence leading up to the war.

The Associated Press contributed to this report.
2005 MSNBC.com

URL: http://www.msnbc.msn.com/id/9886959/

-----------

Excon,

Can you find any example where the Republicans did anything quite as dirty as this? Ever?

That is why I call what the Dems do "dirty politics".

Elliot

tomder55 answered on 11/02/05:

My bigger problem is that this is a war of communications ;and the White House and the Republican leadership have not waged it for some time now . I think this hunker down strategy is a glaring mistake ;and I hold the leadership in the White House responsible for not getting the message out . The Cindy Sheehads and the Joe Wilson's are defining the debate .

ETWolverine rated this answer Excellent or Above Average Answer

Question/Answer
Choux asked on 11/01/05 - Rare Senate Closed Session


Democrats Force Senate Into Iraq Meeting

By LIZ SIDOTI
WASHINGTON (AP) - Democrats forced the Republican-controlled Senate into an unusual closed session Tuesday, questioning intelligence that President Bush used in the run-up to the war in Iraq and accusing Republicans of ignoring the issue.

``They have repeatedly chosen to protect the Republican administration rather than get to the bottom of what happened and why,'' Democratic leader Harry Reid said....



In a speech on the Senate floor, Reid demanded the Senate go into closed session. The public was ordered out of the chamber, the lights were dimmed, and the doors were closed. No vote is required in such circumstances.

Reid's move shone a spotlight on the continuing controversy over intelligence that President Bush cited in the run-up to the war in Iraq. Despite prewar claims, no weapons of mass destruction have been found in Iraq, and some Democrats have accused the administration of manipulating the information that was in their possession.

Vice President Dick Cheney's chief of staff, I. Lewis ``Scooter'' Libby, was indicted last Friday in an investigation that touched on the war, the leak of the identity of a CIA official married to a critic of the administration's Iraq policy.

``The Libby indictment provides a window into what this is really all about, how this administration manufactured and manipulated intelligence in order to sell the war in Iraq and attempted to destroy those who dared to challenge its actions,'' Reid said before invoking Senate rules that led to the closed session."



The more the Republicans try to stonewall and cover-up the facts surrounding the question of lies about WMD's and the rush to war.....the more decent citizens and decent leaders have to push and push to expose the true facts.

tomder55 answered on 11/02/05:

It was a brilliantly executed sucker punch ;that may have some tactical success in the short run. Bush had been able to get the attention away from Libby with the Alito nomination and the bird flu announcement. Reid instantly got the attention back on the indictments. His stunt made the morning papers .I fault Frist in acting foolish with his over reaction .He as unprepared for this which is inexcusable in light of the fact that the Dems published their playbook back in 2003 ["Prepare to launch an independent investigation when it becomes clear we have exhausted the opportunity to usefully collaborate with the majority." ]
.I was embarrassed for Frist ,he was owned by that pathetic joker, Reid.

Frist should've gone into closed session and immediately moved to end the closed session . The Republicans should bring back Lott as the majority leader .

What I do not think it was is good strategic politics by Reid. This was an act of desperation . They banked on going into the mid term elections with indictments ;resignations ;possibly convictions about national security crimes out of the Fitzgerald investigation ;and all they got was that Libby lied . The Democrats appear almost neutered . They are flying air cover for Joe Wilson as he takes the point on the ground .Even some of the biggest pundits realized that what Reid did was weak . Last night on Hardball Andrea Mitchell said that the Senate Democrats were being "disingenuous" because Senate Democrats had plenty of time ; before, during, and since; to have raised these questions. And they didn't have the guts do it. Newsweek's Howard Fineman agreed with her. The truth is that the Senate Intelligence Committee is methodically investigating the WMD issue. Phase one is complete ;and based on that they made recomendations to overhaul the intelligence community . Phase II ;well I can't wait for it!!![even though I believe that chairman Pat Roberts is about ready to finish up Phase II;from what he said yesterday ;it is the Democrats that have been stalling the process ] 1/2 the Democrat leadership will have to justify the comments they made supporting the over-throw of Saddam based on reading the very same intelligence reports that was given to the Administration by the various intelligence agencies ****see below a short list of some of the things democrats said ***** .

Do the Democrats want to solidify themselves as an anti-war party again ;especially with the war against jihadistan likely to last through multiple administrations ? Hey ;let em . It worked so well for them before.













One way or the other, we are determined to deny Iraq the capacity to develop weapons of mass destruction and the missiles to deliver them. That is our bottom line." President Clinton, Feb. 4, 1998

"If Saddam rejects peace and we have to use force, our purpose is clear. We want to seriously diminish the threat posed by Iraq's weapons of mass destruction program." President Clinton, Feb. 17, 1998

"Iraq is a long way from [here], but what happens there matters a great deal here. For the risks that the leaders of a rogue state will use nuclear, chemical or biological weapons against us or our allies is the greatest security threat we face." Madeline Albright, Feb 18, 1998

"He will use those weapons of mass destruction again, as he has ten times since 1983." Sandy Berger, Clinton National Security Adviser, Feb, 18, 1998

"[W]e urge you, after consulting with Congress, and consistent with the U.S. Constitution and laws, to take necessary actions (including, if appropriate, air and missile strikes on suspect Iraqi sites) to respond effectively to the threat posed by Iraq's refusal to end its weapons of mass destruction programs." Letter to President Clinton, signed by Sens. Carl Levin, Tom Daschle, John Kerry, and others Oct. 9, 1998

"Saddam Hussein has been engaged in the development of weapons of mass destruction technology which is a threat to countries in the region and he has made a mockery of the weapons inspection process." Rep. Nancy Pelosi (D, CA), Dec. 16, 1998

"Hussein has chosen to spend his money on building weapons of mass destruction and palaces for his cronies." Madeline Albright, Clinton Secretary of State, Nov. 10, 1999

"There is no doubt that Saddam Hussein has reinvigorated his weapons programs. Reports indicate that biological, chemical and nuclear programs continue apace and may be back to pre-Gulf War status. In addition, Saddam continues to redefine delivery systems and is doubtless using the cover of a licit missile program to develop longer-range missiles that will threaten the United States and our allies." Letter to President Bush, Signed by Sen. Bob Graham (D, FL,) and others, Dec, 5, 2001

"We begin with the common belief that Saddam Hussein is a tyrant and a threat to the peace and stability of the region. He has ignored the mandated of the United Nations and is building weapons of mass destruction and the means of delivering them." Sen. Carl Levin (D, MI), Sept. 19, 2002

"We know that he has stored secret supplies of biological and chemical weapons throughout his country." Al Gore, Sept. 23, 2002

"Iraq's search for weapons of mass destruction has proven impossible to deter and we should assume that it will continue for as long as Saddam is in power." Al Gore, Sept. 23, 2002

"We have known for many years that Saddam Hussein is seeking and developing weapons of mass destruction." Sen. Ted Kennedy (D, MA), Sept. 27, 2002

"The last UN weapons inspectors left Iraq in October of 1998. We are confident that Saddam Hussein retains some stockpiles of chemical and biological weapons, and that he has since embarked on a crash course to build up his chemical and biological warfare capabilities. Intelligence reports indicate that he is seeking nuclear weapons..." Sen. Robert Byrd (D, WV), Oct. 3, 2002

"I will be voting to give the President of the United States the authority to use force-- if necessary-- to disarm Saddam Hussein because I believe that a deadly arsenal of weapons of mass destruction in his hands is a real and grave threat to our security." Sen. John F. Kerry (D, MA), Oct. 9, 2002

"There is unmistakable evidence that Saddam Hussein is working aggressively to develop nuclear weapons and will likely have nuclear weapons within the next five years . We also should remember we have always underestimated the progress Saddam has made in development of weapons of mass destruction." Sen. Jay Rockefeller (D, WV), Oct 10, 2002

"He has systematically violated, over the course of the past 11 years, every significant UN resolution that has demanded that he disarm and destroy his chemical and biological weapons, and any nuclear capacity. This he has refused to do" Rep. Henry Waxman (D, CA), Oct. 10, 2002

"In the four years since the inspectors left, intelligence reports show that Saddam Hussein has worked to rebuild his chemical and biological weapons stock, his missile delivery capability, and his nuclear program. He has also given aid, comfort, and sanctuary to terrorists, including al Qaeda members ... It is clear, however, that if left unchecked, Saddam Hussein will continue to increase his capacity to wage biological and chemical warfare, and will keep trying to develop nuclear weapons." Sen. Hillary Clinton (D, NY), Oct 10, 2002

"We are in possession of what I think to be compelling evidence that Saddam Hussein has, and has had for a number of years, a developing capacity for the production and storage of weapons of mass destruction." Sen. Bob Graham (D, FL), Dec. 8, 2002

"Without question, we need to disarm Saddam Hussein. He is a brutal, murderous dictator, leading an oppressive regime . He presents a particularly grievous threat because he is so consistently prone to miscalculation ... And now he is miscalculating America's response to his continued deceit and his consistent grasp for weapons of mass destruction . So the threat of Saddam Hussein with weapons of mass destruction is real ..." Sen. John F. Kerry (D, MA), Jan. 23. 2003

Choux rated this answer Excellent or Above Average Answer

Question/Answer
Choux asked on 11/01/05 - What's Wrong with Bush?

Today, I watched Bush give a speech about America's preparation for the possibility of Avian Flu. He mispronounced words and stumbled in each sentence at the beginning of the speech. Just now, I ran into this essay by Nora Ephron; I think it brings up some interesting thoughts.

"What's wrong with the president? Is he fighting depression? Is he being medicated in some way that isn't quite working? What's up? I even bought a copy of one of the supermarket tabloids that alleged he'd started drinking again, but the article (like all articles in supermarket tabloids) was extremely disappointing; even the over-exciting picture of the President on the front page, holding a glass of wine, turned out to be an old irrelevant photograph of him making a toast at some banquet; there was no real evidence in the article that he was back on the sauce.

But I've been wondering about what's going on with W ever since he emerged from his bizarre groundhog-like vacation and responded to Hurricane Katrina as if he were under water. He had no affect at all. He was almost robotic. His meager vocabulary seemed to have shrunk even further. He conveyed no feeling for the victims -- and this was early on, way before anyone realized how many poor people were involved. It was strange. What's so hard about cranking yourself up for hurricane victims, especially when you think they're mostly white people who have lost their second homes on the Gulf Coast?

At the time I wondered if Bush was on Paxil or Lexapro, drugs that several of my friends are taking and that seem to have turned them into strangely muted versions of themselves. I asked my friend Rita, who's a shrink, but Rita is very careful about committing on subjects of this sort. She did point out, though, that sometimes, when the President talks, his mouth has a strange sideways twitch, which is apparently common in people who are on antidepressants. Actually it might have been my husband who said this, I can't remember.

But I started thinking about all this again on Sunday. On the Chris Matthews Show, there was some old footage of the president from last year's presidential campaign. He was outdoors, talking to a group of people in hard hats; he was energetic, focused, confident, on top of the world. Now you could easily counter: of course he was, it was a lovely day, he was surrounded by supporters, things were going well. But the President we're seeing these days is a completely different man.

He has, of course, a lot of reasons to be depressed -- no point in enumerating them, you know what they are. But most of all, I think he's depressed because the job has turned out to be so much more onerous than he expected -- he said as much to a friend of mine in September. "You have no idea," he said, "how hard these five years have been." This is a fairly breathtaking remark given the number of people who, thanks to this president, are now dead as a result of his five years in the Oval Office, but never mind.

The point is that it seems possible to me that when George Bush gave up alcohol in 1986, he dealt with the depression that often accompanies sobriety by becoming an obsessive exerciser. And that's what he's essentially done ever since. He's never held anything that could be confused with a job. Owning a football team is not a job. Even being governor of Texas takes only a couple of months a year, it turns out. So he was free to exercise.

But at some point this year, something happened and the exercise regimen stopped working. Bush started becoming depressed. My theory is that a certain amount of panic ensued, and more exercise was prescribed: hence, the afternoon on the bicycle in Maryland, and the reluctance to disturb an already disturbed, irritable man. (Interestingly, the incident happened just after the President returned from a four-day trip to Europe, which had not only required him to work several hours each day but undoubtedly interrupted his exercise routine.) Then came the vacation in August, the odd, sequestered vacation, a perfect time for the President's doctor to try medication, or change medication, or adjust medication. Then Katrina and the emergence in the fall of an unenergetic, irritable, muted, unfocussed President, the man you see today.



I think she is on to the truth. Bush exhibits symptoms of deep depression.....


Comments????

tomder55 answered on 11/02/05:

just another smear ;really not worth responding to . The attacks on him rival the vile that was spewed at Lincoln during his Presidency . Guess he is in good company .

Choux rated this answer Excellent or Above Average Answer

Question/Answer
excon asked on 11/01/05 - Alito - just another liberal.


Hello verywrongwingers:

Im a Constitutionalist, like you pretend to be. I would like a SC that would interpret the Constitution as it was written. If only we could get people like that.......... But, we dont.

The Fourth Amendment says that we should be protected from unreasonable searches The authority for any anticipated search, is a judge who issues the warrant, specifying the places and the person to be searched.

In Doe v. Groody (2004), Alito wrote that reasonable police officers could interpret the warrant to mean that they could search something different.

Gee, the Constitution doesnt give the cops any authority to do that. It looks like a NEW law to me. Looks like legislating from the bench, to me. Looks like cops have new authorities never written down by our founders. If the founders thought a judge should make that decision, then thats good enough for me.

But no. An activist judge is only ACTIVE if he makes law the right wing DISAGREES with.

Alito is as liberal as you get, if changing the Constitution to mean what you want it to mean is a liberal idea.

excon

tomder55 answered on 11/01/05:


Strict construction is not results oriented .Sometimes his opinions will go specifically against the conservative meme.
I exect that I will disagree with some of his decisions just like I did with Scalia recently .I have not studied the issue of "all person" warrants to determine if they fall within the 4th
Amendment "unreasonable " restrictions. Illegal drugs can be hid in some small orifices .

excon rated this answer Excellent or Above Average Answer

Question/Answer
excon asked on 11/01/05 - Values??? Bull!!!


Hello experts:

Bush likes to talk about values. Let's see, didn't he say that he would fire anyone involved in leaking Valerie Plames name? I think he did. Well, now we know that Karl Rove was in it up to his ears. Let's see him uphold honesty as a virtue by firing Karl Rove.

I understand Rove has "value" to Bush, but I didn't think that's what you folks meant when you talk about values.

Honesty? Integrity? In this White House? You've got to be kidding.

excon

tomder55 answered on 11/01/05:

well not exactly in that wording . here is what he actually said 9/30/2003 :

"If there is a leak out of my administration, I want to know who it is," "And if the person has violated law, the person will be taken care of."

he later restated it in July of this year :

"If someone committed a crime, they will no longer work in my administration."

However during a Q/A session June 2004 ;a reporter distorted his comment and had the following exchange with Presidentr Bush

REPORTER: Given recent developments in the CIA leak case, particularly Vice President Cheney's discussions with the investigators, do you still stand by what you said several months ago, suggesting that it might be difficult to identify anybody who leak the agent's name? And do you stand by your pledge to fire anyone found to have done so?

BUSH: Yes. And that's up to the U.S. attorney to find the facts.
His answer was that yes he would stand by his original statement ;not an affirimitive to the reporters distortion (do you stand by your pledge to fire anyone found to have done so?).Bush had never said that . He clearly was referring to someone who has committed a crime.

Fitzgerald has investigated the Plam leak for 2 years now and has not found that anyone committed a crime in leaking her name . I do not see why the President should fire Rove and he certainly shouldn't based on media distortions. While the media dances with glee over the fact that one high ranking White House official got nailed they should look in the mirror . If it was a crime to out Plame then the national news media colluded in the crime . But I still hold my contention that Libby was a whistle blower .Aren't whistleblowers those who expose people perpetrating a fraud,and aren't they supposed to be protected for speaking up and exposing things like Wilson's lies?




excon rated this answer Excellent or Above Average Answer

Question/Answer
Choux asked on 10/31/05 - Hurricand Katrina funding...for Alaska

Oct 31 2005 12:20 PM

By TimChapman

"Capitol Report has learned about a provision tucked away in the Senate Budget Reconciliation Bill that would direct Medicaid money intended for Katrina affected states (Mississippi, Alabama, Louisiana) to Alaska.

The Budget Reconciliation package (PDF) contains $71.4 billion in new savings but it also spends $32.4 billion. Portions of that new spending were intended to be Katrina relief funds, but it seems Alaskan interests have once again succeeded in redirecting funds (PDF) to the state which has become famous for its "Bridge to Nowhere."

In addition to providing money for Katrina states, the provision also changes the way Alaska receives federal assistance for its Medicaid services. By changing the federal funding matching percentage for Medicaid in Alaska, the provision will provide an additional $130 million in federal Medicaid funding for Alaska. This additional $130 million is a direct result of tampering with federal matching percentages that results in Alaska being relieved of Medicaid related fiscal burdens that all the other 50 states face."


Comments?

tomder55 answered on 11/01/05:

Until recently I had no idea how much clout the reps of one of the least populus states had . I have to give them credit ;they have shown a consistent ability to bring home the pork. Per dollar of Federal tax collected in Alaska citizens receive approximately $1.91 in the way of federal spending. This ranks the state 3rd highest nationally and represents a large rise from 1992 when Alaska received $1.26 per dollar of taxes in federal spending . I just wish that NY reps would do more than just pay lip service to the need for a return to the State . As it stands now there is a great wealth transfer out of the State that I have to assume the population of NY is more than willing to pay .....of course they are ! the re-elect these people time after time after time........!!

By contrast the people of Alaska enjoy the earliest 'tax freedom day' in the nation (March 30th compared to April 17th nationally ).Alaska levies no individual income taxes .Alaska levies no general sales or use tax on consumers . Alaskas gasoline tax stands is the 2nd lowest nationally .During the past two decades, Alaskas state and local tax burden has been consistently ranked as one of the nations lowest. Estimated now at 6.4% of income, Alaskas tax burden percentage remains the lowest nationally, well below the national average of 10.1%.

Hell ! I'm moving !! I can stand the cold !;and if I get sick ??? well there is all that Medicaid money available .I'm packin my bags gonna cross that bridge to no-where to get to the land of milk and honey !!!!!!!!

Choux rated this answer Excellent or Above Average Answer

Question/Answer
ETWolverine asked on 10/31/05 - Now this is more like it.

Found at MSLSD:

Bush nominates Alito for Supreme Court
Conservative federal judge picked to replace retiring O'Connor


BREAKING NEWS
The Associated Press
Updated: 8:40 a.m. ET Oct. 31, 2005


WASHINGTON - President Bush, stung by the rejection of his first choice, nominated conservative judge Samuel Alito on Monday to replace moderate Justice Sandra Day OConnor in a bid to reshape the Supreme Court and mollify his political base.

Judge Alito is one of the most accomplished and respected judges in America, the president said in announcing Alitos selection. Hes got a mastery of the law and a deep commitment to justice. Bush exhorted the Senate to confirm his choice by the end of the year.

The choice was likely to spark a political brawl. Unlike the nomination of Harriet Miers, which was derailed Thursday by Bushs conservative allies, Alito faces opposition from Democrats.

The Senate needs to find out if the man replacing Miers is too radical for the American people, said Senate Minority Leader Harry Reid, D-Nevada.

In contrast to Miers, Alito has more prior judicial experience than any Supreme Court nominee in 70 years, the president said.

Consistent conservative
So consistently conservative, Alito has been dubbed Scalito or Scalia-lite by some lawyers because his judicial philosophy invites comparisons to conservative Supreme Court Justice Antonin Scalia. But while Scalia is outspoken and is known to badger lawyers, Alito is polite, reserved and even-tempered.

Wasting no time, the White House arranged for Alito to go to the Capitol after the announcement. The schedule called for Senate Majority Leader Bill First to greet him and accompany the nominee to the Capitol Rotunda to go to the coffin of the late civil rights pioneer Rosa Parks.

The Supreme Court is an institution I have long held in reverence, said the bespectacled judge, a former prosecutor and government attorney who has argued 12 cases before the Supreme Court. During my 29 years as a public servant, Ive had an opportunity to view the Supreme Court from a variety of perspectives.

From the bench, Alito has staked out positions supporting restrictions on abortion, such as parental and spousal notification.


Republican rift (This part is utter partisan bull$h!t--- Elliot)
The White House hopes the choice mends a rift in the Republican Party caused by the failed nomination of Miers, a Bush loyalist, and puts his embattled presidency on a path to political recovery.

With the rebuke of Miers, the rising death toll in Iraq, his slow-footed response to Katrina and last Fridays indictment of top vice presidential aide I. Lewis Scooter Libby, Bushs approval ratings are at the lowest ebb of his presidency.

Polls show Democrats and most independents dont approve of his job performance, leaving the conservative wing of his party the only thing keeping Bush afloat politically.

Miers bowed out last Thursday after three weeks of bruising criticism from members of Bushs own party who argued that the Texas lawyer and loyal Bush confidant had thin credentials on constitutional law and no proven record as a judicial conservative.

If he is confirmed by the Senate, Alito would join another Bush pick on the bench, Chief Justice John Roberts. OConnor, who is retiring, has been a decisive swing vote in a host of affirmative action, abortion, campaign finance, discrimination and death penalty cases.

Officials, who spoke on on condition of anonymity before Bush's announcement, said Alito was virtually certain from the start to get the nod from the moment Miers backed out. The 55-year-old jurist was Bushs favorite choice of the judges in the last set of deliberations but he settled instead on someone outside what he calls the judicial monastery, the officials said.


Bush believes that Alito has not only the right experience and conservative ideology for the job, but also has a temperament suited to building consensus on the court. A former prosecutor, Alito has experience off the bench that factored into Bushs thinking, the officials said.

The president has made an excellent choice today which reflects his commitment to appoint judges in the mold of Scalia and Thomas, said Kay Daly, president of the conservative Coalition for a Fair Judiciary.

Its a pretty predictable move from a politically crippled president, said Democratic consultant Jim Jordan. Toss out a judicial extremist to pacify his base and provoke a fight that he hopes changes the subject away from indictments and Iraq and Katrina and a soft economy.

Democrats warn of partisan brawl
While Alito is expected to win praise from Bushs allies on the right, Democrats have served notice they will fight it. Reid had warned Sunday that it would create a lot of problems.

Unlike Miers, who has never been a judge, Alito, a jurist from New Jersey, has been a strong conservative voice on the 3rd U.S. Circuit Court of Appeals since Bushs father, former President George H.W. Bush, seated him there in 1990.

Judicial conservatives praise Alitos 15 years on the Philadelphia-based court, a tenure that gives him more appellate experience than almost any previous Supreme Court nominee. They say his record shows a commitment to a strict interpretation of the Constitution, ensuring that the separation of powers and checks and balances are respected and enforced. They also contend that Alito has been a powerful voice for the First Amendments guarantees of free speech and the free exercise of religion.

Liberal groups, on the other hand, note Alitos moniker and say his nomination raises troubling concerns, especially when it comes to his record on civil rights and reproductive rights. Alito is a frequent dissenter on the 3rd Circuit, one of the most liberal federal appellate benches in the nation.

In the early 1990s, Alito was the lone dissenter in Planned Parenthood v. Casey, a case in which the 3rd Circuit struck down a Pennsylvania law that included a provision requiring women seeking abortions to notify their spouses.

The Pennsylvania legislature could have rationally believed that some married women are initially inclined to obtain an abortion without their husbands knowledge because of perceived problems such as economic constraints, future plans or the husbands previously expressed opposition that may be obviated by discussion prior to the abortion, Alito wrote.

He has not been a down-the-line abortion foe. In 2000, Alito joined the majority that found a New Jersey law banning late-term abortions unconstitutional. In his concurring opinion, Alito said the Supreme Court required such a ban to include an exception if the mothers health was endangered.

The case ended up at the Supreme Court where the justices, in a 6-3 decision struck down the spousal notification provision of the law. The late Chief Justice William H. Rehnquist cited Alitos reasoning in his own dissent.

Alito, an Italian-American who grew up in Trenton, N.J., has a resume filled with stepping stones to the high court. He was educated at Princeton University and earned a law degree from Yale University, the presidents alma mater.

2005 The Associated Press. All rights reserved. This material may not be published, broadcast, rewritten or redistributed.
2005 MSNBC.com

URL: http://www.msnbc.msn.com/id/9874588/page/2/


-----------

Opinions please.

tomder55 answered on 10/31/05:

why should his nomination provoke a bawl ? He was confirmed by unanimous consent by the Senate for the Appellate court .W eshould be getting a slew of soundbites from Shumer any minute now .

re rift. The genious Karl Rove proved this last election that there is no 'middle America ' . He knew that whoever energized the base would win in 04 . What the President tried to do ,and any failure that can be attributed to him since he became President ,is to play 'consensus politics'to reach common ground. Where it has been attemped it has resulted in weakened or failed policy .Once bit twice shy ;he should at least finally realize that there is no middle ground . The Democrats see that as policy from a weak position.

I think this selection makes the President stronger . His supporters will fight for him and the Democrats will whimper .MY bet is that they will not carry through with fillibuster bluffs. But if they do then it will be time for the Republicans to assert their majority .

ETWolverine rated this answer Excellent or Above Average Answer
Itsdb rated this answer Excellent or Above Average Answer

Question/Answer
Choux asked on 10/30/05 - Tomorrow, and tomorrow, and tomorrow

More and more, Bush looks like a man who has reached the point of no return. Watching his robotic speech on Iraq this week, you got the sense his heart is no longer really in it. He seems defeated. Resigned. Running on empty.

It got me thinking of the way this happens in every great tragedy -- Greek or Shakespearean. The moment arrives when we know that all is lost. For some reason, I keep thinking of Macbeth.

Macbeth's fatal deed -- from which there was no return -- was, of course, killing King Duncan ("Had I but died an hour before this chance, I had lived a blessed time"). Bush's fatal deed was invading Iraq. It led directly to Plamegate -- an attempt to cover up the lies and deceptions used to sell an unnecessary war to the American people. It derailed the war on terror, increased anti-American feeling around the world, contributed to the soaring budget deficit, made us less safe here at home, and set the table for the disastrous mishandling of Hurricane Katrina.

In the Scottish play, Shakespeare perfectly captures the infinite weariness that sets in when you've reached the end of the road:

"Tomorrow, and tomorrow, and tomorrow, creeps in this petty pace from day to day, to the last syllable of recorded time; and all our yesterdays have lighted fools the way to dusty death."

Can you think of a better summation of the position Bush now finds himself in? There will be no legacy of endless Republican power. No grand remaking of the Middle East. No privatization of, well, everything. No shrinking the government. No superseding his father.-Ariana Huffington


RADICAL RIGHT WING DUDES

ALL IS LOST

tomder55 answered on 10/31/05:

I think he has reached the nadir of his Presidency but there is still 3 1/2 years left to it so the FLuffington's will have to deal with it . I see his presidency more like Henry V 's victory at Agincourt.


Men of few words are the best men

In peace there s nothing so becomes a man
As modest stillness and humility;
But when the blast of war blows in our ears,
Then imitate the action of the tiger:
Stiffen the sinews, summon up the blood.


Choux rated this answer Excellent or Above Average Answer

Question/Answer
Choux asked on 10/28/05 - Cheney's Goose is Cooked

"The indictment today of Scooter Libby marks the beginning of the end of one of the most bold and cutthroat rises to power in American history. Dick Cheney was a former congressman from Wyoming, former Secretary of Defense and finally CEO of Halliburton. Not bad for a guy who flunked out of Yale.
Twice. Then one night he gets a call that he had been waiting for all his life. His old boss George senior asks him to pick a vice-presidential candidate to run behind his inexperienced son. We know that Dick wanted to be President himself because he actually ran for that office in 1996 but dropped out before the first primary when he realized that someone of his dull looks and anti-charisma didn't stand a chance. This second time around he must have understood that by hiding behind a glib, telegenic heir to an important political family he could have all the power of the Presidency without that pesky problem of people actually having to like him. So instead of suggesting a VP who might actually help carry an important region or state, Cheney convinced the Bushes to go with him, a guy from a state with a whopping three electoral votes. (Read Dick, by John Nichols to learn more.)

It was a match made in some sort of heaven. One guy burned to have the power of the Presidency, while the other guy just wanted to play one on TV.

This isn't (just) liberal ravings here. Former Treasury Secretary Paul O'Neil and Colin Powell's former Chief of Staff Larry Wilkerson both have gone public with how hands-off Bush governs. In fact when he did try to actually be the President and ***for once make a decision for himself what did he do? He chose Harriet Miers***.

So once in office Cheney looks around the world and what does he see? Does he want to beat the Democrats into insignificance like Karl Rove and Tom Delay? Not particularly. Does he have some utopian ideal of stabilizing and democratizing the globe like Paul Wolfowitz? Are you kidding? As CEO of Halliburton he lobbied Congress to ease sanctions against Lybia, Syria and even Iran. No, once handed the keys to the kingdom this guy looked around the world and saw money.

Remember his first significant act in office? He presided over a ***secretive, closed-door energy task force***. Bush's biggest donor Ken Lay was there. And what was on the table? Talk of conservation to buy us more time before oil reserves dry up? Aggressive investment in new technologies to free us of our dependence on foreign oil? Nope. According to Judicial Watch what were on the table were maps. Maps of Saudi and UAE oil fields and also a map of a country over which we, at that time, had no control -- Iraq. That was in March of 2001. We all know what happened next. In the days after 9/11 people like former counterterrorism czar Richard Clarke and Prime Minister Tony Blair were shocked to hear Bush, Cheney and Rumsfeld talking about Iraq when none of the hijackers came from that country. Just as Cheney saw a golden opportunity for himself in being picked to pick Bush's VP, he saw 9/11 as opportunity dripping with oil. Hiding behind a national tragedy he could mobilize the strongest army in the history of the world to do ***his corporate bidding***.

We know this to be true because of the rush to war. No one, not the CIA, the State Department, or any foreign intelligence outfit thought that Iraq's threat was imminent. They all thought there was a threat because Saddam kept saying there was one, but no one thought that it was less than years away. The United States with the help of our little buddy Great Britain, rushed into war without the backing of any other major nations because Dick Cheney didn't want to have to share the spoils of war with them. And you can't spell "spoil" without "oil."

With Libby's indictment caring Americans can now press for the whole truth to come out. Cheney and his White House Iraq Group lied us into war then viciously went after anyone like Ambassador Wilson who dared get in their way. They almost got away with it. Almost.

Republicans keep saying that Democrats looked at the same intelligence briefings they did when they overwhelmingly voted to go to war. That is true. They all saw the same books, but what this indictment should bring to light next, is that those books had been cooked". -Trey Ellis

from Huffington Post Blog site today.

tomder55 answered on 10/29/05:

if Tony Blair was shocked he got over it quick enough as he made one of the most compelling arguments for the invasion.

I do not quite undersand all that happened yesterday yet. Why did Libby lie ?He could've easily said that Cheney was his source and still no crime would've been committed because it was not illegal for them to be discussing it even if it was high classified material. Fitzgerald will be hard pressed I think to come up with a motive.Perjury prosecutions are extremely difficult to prove. I have to wonder, what evidence does he have that Libby consciously misled or lied to the Grand Jury, as opposed to simply making a mistake? Hell ;at least you could pin financial reward on Martha Stewart as a motive.By the time he testified to the grand jury he must'veknown that espionage act did not apply. He is too smart a lawyer himself to fall into this trap ,and even if he were not ;his lawyer should've informed him the dangers of lying to a grand jury.
There are still other questions .Who leaked the story to Robert Novak? What, precisely, was Valerie Wilson's status at the CIA at the time Novak's column revealed her identity?The indictment charges the mere fact that Plame worked at the CIA was 'classified information'. Fitzgerald presumably knows the answers to those questions. But, at least so far, he isn't saying.How can Tim Russert keep reporting on a story in which he is a part?

But as you point out ;the Huffington crowd is not really concerned about the facts of this case as much as their desire that it damages the Bush Presidency .politics as usual .

Choux rated this answer Excellent or Above Average Answer

Question/Answer
ETWolverine asked on 10/28/05 - What Republicans Can Learn From Democrats.

Hello all,

You all know that I am no supporter of the Democratic Party or the liberal left. But I do believe that there is something we can learn from them.

Party discipline.

The fact is that Democrats all tow the party line. That line may be dead wrong, but they tow it anyway. A good example was the 2004 election where Dems towed the "wrong war, wrong time" party line despite clear indications from polls that the vast majority of the country supported Bush on the war in Iraq. They were wrong on the issue, and towing that particular line was a disaster for the Dems in the election, but they still maintained party discipline. Even if it led them off a cliff like a bunch of dodos.

Compare that to the lack of party discipline within the Republican party. You have Arlan Specter threatening to rake any Bush nominee he doesn't approve of over the coals. You have John McCain publicly and loudly parting with the President on so many issues and threatening to kill cloture votes and nuclear option votes. You have the Gang of 14 making back-room deals without the approval of the party leadership (yes, I know, half of them are Dems, but the deals are ones that favored the Dems and push the Dem agenda forward. Its the Reps who have broken with their party). There's no party discipline.

Where's the party Whip (Roy Blunt)? Where's the party leadership? Why aren't they slapping these guys down and bringing them into line? This would not be tolerated on the other side of the isle, and we need to start playing hardball politics insteead of trying to find "common ground" or "moderation" or other such nonsense.

Your opinions, please.

tomder55 answered on 10/28/05:

Of course we are in NY where the Republican party is where democrats go when they can't get on the ballot otherwise

I have been no fan of the Republican party ;especially in the halls of the Senate .

I have my theory so here it is . The Republican Party is now the majority party . To accomplish this task they had to grow the party ;to become a 'big tent'party( Phil Gramm would say that a great party is a big magnet, not a big tent. A big tent party is a party without a compelling political agenda, a party having the sole purpose of acquiring and holding power.)It is up to the elected leaders to keep this coalition together .Ron Reagan's 11th commandment was sensible as the party grew.

But too often today they do not follow the 'party line' .I am not sure that is a good or a bad thing. The charge that dems are koolaid drinkers has some merit but to hold to you're principles means less give and take ;and when there is a convergence of principles in an alliance of convienience then sometimes that relationship gets strained.To look for the lowest common denominator means to settle for an agenda void of principles . It appears that party leaders are now jockying for postion for like after Bush. But they should take a page out of Bush's playbook .He succeeds when he appeals to the base. When he tries to be 'inclusive ' he loses support .

ETWolverine rated this answer Excellent or Above Average Answer

Question/Answer
excon asked on 10/27/05 - What happened?



Hello Wishersthenextonewilloverturnroe:

The Miers debacle was foreseeable. It does NOT have the smell and feel of a Karl Rove tactic. I can't believe that Rove didn't see it coming? Is he out of the loop? Without his stringpulling, George is a loose cannon. Who is running the show? Andy Card? Laura? Jeb? 41?

I'm worried. Tom, you seem to know about this stuff.

excon

tomder55 answered on 10/28/05:

First ;the Dems are bleating that it was radical right wing christians etc that killed this nomination . But ;It was Bush who made a big blunder by bringing religion into the discussion in the first place ;then Rove made it worse by assuring the evangelicals that "she is one of us" . Given that information how can anyone think it was the christian wing that killed the nomination ? The White House did not shine during the whole episode . People who were opposed to Miers were called elitists and sexists . None of these issues were pertinant . Had Bush nominated Janice Rogers Brown or Priscilla Owens there would've been unanamous support by conservatives to the selection .If he wants a women on the court by all means make those choices.If a Democrat filibuster is inevitable, then so be it. The President's party has a majority in the Senate ;they should start acting like a majority .

Bush was stubborn in his sticking by the Miers pick even as it became increasingly clear that she would not survive the inquisition .In this case his loyalty probably did permanent damage to Miers' future career . Who knows ? over time she may have served as a district and appellate judge ;I doubt it now .Loyalty is a fine thing but there is a fine line between loyalty and cronyism. In the next few days his loyalties are likely to be put under the test . He should do what is good for the country and if indictments should come down he should publicly demand resignations and not wait for them .It is not a question of guilt or not ;it is about having an effective executive .

There is talk that Bush may go with Alberto Gonzales as his next pick .That would be a monumental mistake for the reasons I've already mentioned.[if he wants a hispanic then he should consider Miguel Estrada :He is an immigrant from Honduras, who came here with his mother at the age of 17. Five years later, he graduated Phi Beta Kappa, and Magna Cum Laude from Columbia University. He went on to Harvard Law School, where he graduated near the top of his class, and was an editor of the Harvard Law Review. He spent time clerking for Supreme Court Justice Anthony Kennedy,and has had a distinguished legal career.Let the Dum~rats fillibuster him! ]

If the President wants to accomplish anything in his second term he needs his base behind him .The Miers withdrawal is a move in that direction. It should be obvious to him now what we want him to do ...nominate brilliant candidates who believe in orginalism ; attack the Federal Budget /control spending (damn hypocrites in Congress...it is time for Bush to get aquainted with his never used veto pen );come to terms that we want a sensible solution to the border security issue and continue to take the offensive in the war against jihadistan ;the last being the most important .

excon rated this answer Excellent or Above Average Answer

Question/Answer
sapphire630 asked on 10/26/05 - Bury Cindy shehan

Cindy Shehan is symbolically dying on the White House Lawn today.....can we hurry up and get a bus and drive there so we can *symbolically* bury her???
Maybe Louis Farakan can do her eulogy and beam her up to the big mothership.

tomder55 answered on 10/27/05:

sorry ;;leaving litter on the lawn is a misdemeanor. she should stick to plan 'A' ...chaining herself to the fence and hoping someone notices

Itsdb rated this answer Excellent or Above Average Answer
sapphire630 rated this answer Excellent or Above Average Answer

Question/Answer
sapphire630 asked on 10/26/05 - Terrorists have their list

of things they find offensive that they want us to do away with. They find pigs offensive and England is to do away with piggy banks, Hollywood is offensive, women's rights are offensive, and creamation is offensive to name a few. What about 9/11 and the beheadings and all the things offensive to America. Why do Liberals concern themselves with pleasing the terrorists and not looking at the reality. Do they seriously believe terrorists can be *reasoned*** with?

When I was in High School (eons ago)a DJ played a record about a woman that took a dying snake home and warmed it up by her fireplace and nursed it back to health. Once the snake was healthy it turned and bit her and said you stupid woman.
The stupid woman sounds like the liberals logic to me.
I doubt the DJ remembers this record.
This DJ is now a local radio talk show host.
A little too right-wringer for me, but I'd rather listen to a right-wringer than a liberal-lefty-luie

http://www.warroom.com

tomder55 answered on 10/27/05:

http://www.rangerjarhead.com/911jumpers

sapphire630 rated this answer Excellent or Above Average Answer

Question/Answer
Itsdb asked on 10/27/05 - No Miers, what now?

Below is the text of the letter from Harriet Miers to President Bush, withdrawing her name from consideration as a justice of the United States Supreme Court.

Dear Mr. President:

I write to withdraw as a nominee to serve as an Associate Justice on the Supreme Court of the United States. I have been greatly honored and humbled by the confidence that you have shown in me, and have appreciated immensely your support and the support of many others. However, I am concerned that the confirmation process presents a burden for the White House and our staff that is not in the best interest of the country.

As you know, members of the Senate have indicated their intention to seek documents about my service in the White House in order to judge whether to support me. I have been informed repeatedly that in lieu of records, I would be expected to testify about my service in the White House to demonstrate my experience and judicial philosophy. While I believe that my lengthy career provides sufficient evidence for consideration of my nomination, I am convinced the efforts to obtain Executive Branch materials and information will continue.

As I stated in my acceptance remarks in the Oval Office, the strength and independence of our three branches of government are critical to the continued success of this great Nation. Repeatedly in the course of the process of confirmation for nominees for other positions, I have steadfastly maintained that the independence of the Executive Branch be preserved and its confidential documents and information not be released to further a confirmation process. I feel compelled to adhere to this position, especially related to my own nomination. Protection of the prerogatives of the Executive Branch and continued pursuit of my confirmation are in tension. I have decided that seeking my confirmation should yield.

I share your commitment to appointing judges with a conservative judicial philosophy, and I look forward to continuing to support your efforts to provide the American people judges who will interpret the law, not make it. I am most grateful for the opportunity to have served your Administration and this country.

Most respectfully,

Harriet Ellan Miers

The President
The White House
Washington, D.C. 20502

tomder55 answered on 10/27/05:

That was the wise and correct move to make .Her relationship with the President was ultimately going to derail the nomination . She mentioned the document issue ;and I fully agree with the President's position about the appropriateness of surrendering the docs. There was also lawyer /client conflicts of interest which could possibly mean that Meirs would have to recuse herself from many of the upcoming court decisions .

What now ? I hope Bush gives his base what they want . When people talk about declining support for Bush they are ultimately talking about an eroding of support from the base. There was no "swing voter " last election as Rove proved . He has to pick an originalist and if necessary slug it out in the Senate .He has to sharpen his veto pen and start doing some real budget reductions if the Congress refuses to do .He has to get out of his self pity funk and man the bully pulpit and push his agenda . He has to constantly hawk the good news out of Iraq and update the nation about the war against Jihadistan .

ETWolverine rated this answer Excellent or Above Average Answer
Itsdb rated this answer Excellent or Above Average Answer

Question/Answer
Itsdb asked on 10/26/05 - What to do?

Lack of black players troubles baseball execs. Astros are first World Series team since ཱ to have no black players; fewer than 10 percent in MLB are black

THE ASSOCIATED PRESS

Joe Morgan worries about the face of baseball. Watching the World Series, Morgan, a Hall of Famer, is troubled by what he sees.

His old team, the Houston Astros, was down 2-0 to the Chicago White Sox heading into last night's game, but it's not their lineup that concerns Morgan. It's their makeup.

The Astros are the first World Series team in more than a half century with a roster that doesn't include a single black player.

"Of course I noticed it. How could you not?" Morgan said while the Astros took batting practice before the opener in Chicago. "But they're not the only ones. There are two or three teams that didn't have any African-American players this year."

Morgan said that it's a predicament and a challenge for Major League Baseball. While more players from around the world are making it to the majors - Japan, Korea - the number of blacks is declining.

"It's a daunting task to get African-American kids into baseball, and I don't see the trend changing," he said.

The last World Series team without a black player was the 1953 New York Yankees. It wasn't until 1955 - eight years after Jackie Robinson broke the color barrier in 1947 - that Elston Howard became the first black in Yankees pinstripes.

Black players accounted for just about 9 percent of big-league rosters this season.

"We know that we have to work to do," Commissioner Bud Selig said yesterday. "We'll continue to intensify our efforts.

"I'm very aware, I'm extremely sensitive about it, and I feel badly about it. But we need to get to work to change things."

General Manager Tim Purpura of the Astros agrees.

"I think it's a huge, huge problem for baseball," he said. "The pool of African-American players just isn't there. And as baseball becomes more college oriented in its draft, there aren't a lot of players to pick.

"The African-American athletes are going into other sports," he said.

The most recent survey by the NCAA, taken during the 2003-04 season, showed that only 6 percent of Division I baseball players were black. Half of the men's-basketball players were black, as were 44 percent of football players.

Houston has six Hispanic players - it was the first team to open a baseball academy in Venezuela. Bench coach Cecil Cooper is black.

Outfielders Charles Gipson and Charlton Jimerson, both black, played for the Astros during the regular season.

The White Sox have three black players on their Series roster: Jermaine Dye, Carl Everett and Willie Harris, along with coaches Tim Raines and Harold Baines. They also have eight Hispanic players and Tadahito Iguchi, a Japanese second baseman.

"We're diverse because we're looking for the best in talent and character," General Manager Ken Williams said before the Series started. "It just happened that way. I could care less what the makeup of the club is as long as it works as a whole."

Williams is the only black general manager in the majors. Williams, a former big-league outfielder, joined the White Sox in 1992 as a scout, confident he could find players in the inner cities. After a year of trying, Williams felt as if he'd failed.

Morgan is disturbed by what he's found, too.

Morgan, a two-time NL MVP, helped Cincinnati win two straight championships. In 1976, along with fellow black teammates Ken Griffey, George Foster and Dan Dries-sen, the Big Red Machine swept a Yankees team that had 10 black players on its roster.

Just 10 years ago, Atlanta and Cleveland each had five black players when they met in the World Series.

In 2003, Derek Jeter and the Yankees lost to Florida. Jeter's father is black and his mother is white; Jeter, an All-Star shortstop, has said he considers himself both black and white.

"There's a perception among African-American kids that they're not welcome here, that baseball is not for inner-city kids," Morgan said. "It's not true, and I hate that the perception is out there."

~~~~~~~~~~~~~~~~~~~~~~~~~~~~~~~~~~~~~~~~~~~~~~~~~~~~~~~

Is diversity not diverse as long as there aren't enough blacks? What percentage would be enough?

I have no baseball loyalties but my favorite sports team, the Dallas Cowboys, are currently 61% black - 41 out of 67 active and inactive players. There is also I believe one Hispanic and one Vietnamese Cowboys player. Should I be concerned there aren't enough whites on my Cowboys, or should I think "I could care less what the makeup of the club is as long as it works as a whole"?

Steve

tomder55 answered on 10/27/05:

I love MLB but the executives have no idea at all what they are doing to the sport. The feature games of their sport ;the World Series and most of the play off games are played 8:30 pm starts and last an average of 3 to 4 hours. That puts viewership of the games beyond the endurance of most working people and almost all kids . NO KIDS ARE WATCHING THE SPORT !! HELLOOOOOO !! see anything wrong with that ???

Watching the game must be like watching paint dry to all but the most devotee . In the same time it takes a batter to adjust his gloves ;scratch his ass;spit his chew ;dig into the batters box ,the pitcher shakes off 3 signs by the catcher then throws the ball to 1st base ;and the process starts over ;eventually the pitch is thrown .....etc. finally a single play occures ...... In football you have seen 3 plays and the replay ;in basketball possibly 10 points have been scored .

Go to a high school . The football games in many places are events ;the stands are packed ;the same with basket ball. Kids all over America are kicking soccer balls or playing hoops. Go ot a high school Baseball game and you may be the only person beyond parents who is watching . Unless you are an infielder or a pitcher ,the game is boring to a kid. The ones with marginal skills ends up picking daisies in the outfield and perhaps getting lucky to field 2 ground balls that the infielders misplayed .

The better players of basketball and football stand a chance of getting a decent scholarship for their efforts because college football and basketball attract attendance and therefore revenue for the schools .Some baseball players get them but for the most part it doesn't fund their education .College baseball does not attract a national television audience and only die-hards watch the college world series.

So where will the black athelete go ? Basketball requires almost no expense to hone skills ;a ball and a couple of friends and you can play all day . Baseball you pretty much need to play in organized leagues to become skilled sufficently to become a pro. When a college basketball player and football player is good they move on from school to the pros and make big bucks right away. A baseball player may still needs years of minor leagues before they have a chance.

Wonder why blacks don't play baseball ?;there it is

Choux rated this answer Excellent or Above Average Answer
Itsdb rated this answer Excellent or Above Average Answer

Question/Answer
Choux asked on 10/26/05 - Call Me Ishmael.

"I was a crewmember aboard the Pequod when Captain Ahab went hunting for the white whale known as Moby Dick.

For Ahab, the white whale became the embodiment of all that was evil in the world. He assembled a crew of 30 from various backgrounds and origins, not revealing his primary cause until they are well out to sea.

As the lone survivor, Herman Melville placed the burden on me to tell the tragic events of how a crew was destroyed because it went along with a mission of megalomania that offered little chance for survival.

I fear that 21st century America may be suffering from the effects of what I call the Ahab Complex.

One of the great lessons from that tragic voyage, and there are many, was that the crew was never honest about Ahabs quest. Likewise, the crew was never honest with themselves".

-Byron Williams-essay cut and pasted from
Huffington Post Blog Site


What say you?

tomder55 answered on 10/26/05:

I made the same comparison recently to Ronnie Earle's quest to indict Tom Delay . The apparent strategy of the Dems now is that if you can't beat em at the polls ..indict em .

I could not find the link in Huffington so I cannot comment on the specific context.

Choux rated this answer Excellent or Above Average Answer
Itsdb rated this answer Excellent or Above Average Answer

Question/Answer
excon asked on 10/25/05 - Iraq - North Korea


Hello warmongers:

If a strategy of pre-emptive war, based upon WMD's, hate for the US, the spread of terrorisim, and a dreadfully oppressed people, is valid, why don't we attack North Korea?

If the strategy is wrong, then why won't you warmongers say it? If it's right, why aren't you raising hell about North Korea?

If the strategy is just a subterfuge for protecting one's supply of oil, I expect the politicians to deny it, but why would you? And, if that's the reason, why can't we say it? It's certainly no worse than the reasons we gave.

excon

tomder55 answered on 10/26/05:

I suspect that if the reverse had happened and N Korea was militarily addressed that he critics would be asking :"why not Iraq ? "It really is a straw man argument.

From 1950 - 1991, the U.S. and the Soviet Union maintained a 'detente' since each had nuclear weapons .There aren't very many people in hindsight today who think we should have initiated a military campaign against the Soviet Union ;although it may have been logical to do so before they got the capability as Patton had suggested . The decision to go to war with a nuclear power must be examined with much more caution than a decision to attack an easily assailable nation that does not have nuclear weapons. The time to address the N Koreans was in the 90s when their program was in it's infancy.

It was universally agreed by the International community that Saddam had WMD ;used WMD and covetted to continue his WMD program . The only real question remaining is the available stock pile he had at the time of the invasion . The only debate is not if ;but when . Was Saddam an imminent threat ? In retrospect probably not ..but the time to deal with him just like in the case of N Korea was not when the threat was imminent but before . In N Korea that threshold passed in the mid 90s ;the threshold was rapidly passing in the case of Iraq .

The sanction regime was collapsing..Oil for food was undermining it ;we could not maitain a large presence in the gulf forever (remember OBLs supposed big beef with the US ...our presence there ).Our troops were under fire while patrolling the no fly zones ;and cut and run was no option ,Saddam was just too dangerous with WMD .

Saddam was host and patron to many terrorists (don't deny it ;you know it is true . Colin Powell in his presentations accurately pointed out Zarqwai as being an Iraqi patron ).The idea of terrorists getting Iraq WMD was a real possibilty .Say what you want about Kim Jong Il ;there is no idication that he supports terrorists . Also Il has on many occasions voiced a desire to a thawing of relations with the US .

N.Korea is in a better situation to reach a diplomatic solution . Their patron to the north is uncomfortable with their nuclear program . South Korea is trying to reach a solution to the divided Koreas .Japan and other nations have an interest in a negotiated solution . There are more cards to play at the diplomatic end.

excon rated this answer Excellent or Above Average Answer

Question/Answer
Choux asked on 10/23/05 - Strong Words

***Do not read on*** if you are fragile or do not like strong talk about religion::


"We live in a twisted world, where right is wrong and wrong reigns supreme. It is a chilling fact that most of the world's leaders believe in nonsensical fairytales about the nature of reality. They believe in Gods that do not exist, and religions that could not possibly be true. We are driven to war after war, violence on top of violence to appease madmen who believe in gory mythologies.
These men are called Christians, Muslims and Jews.

Osama bin Laden is insane. He believes God whispered in the ear of Mohammed 1,400 years ago about how he should conquer Arabia. Mohammed was a pure charlatan -- and a good one at that. He makes present religious frauds like Pat Robertson look like amateurs.

He said God told him to have sex with as many of the women he met as possible. I'm sorry, I meant to say "take them as wives." God told him to kill all other tribes that stood in his way or that would not placate him with assurances of loyalty or bribes. God told him, conveniently, that everyone should follow him and never question a word he said.

He sold this bag of goods to the blithering idiots who lived in the Arabian Peninsula at the time. If that weren't shockingly stupid enough, over a billion people continue to believe the convenient lies that Mohammed told all that time ago -- to this very day.

We live in a world full of insane people. Sanity is an island battered in an ocean of frothing delusion. The people who believe in science are the minority. The people who believe in bloody fairytales are the overwhelming majority.

George W. Bush is the most powerful man alive. He is a class A imbecile. He is far less intelligent than the average Christian. But like most of the others, he believes Jesus died for his sins. That idea is so perverse and devoid of logic it should shock the conscience. Instead, it gets him elected, and earns him the reverence of a great percentage of America. America! The most advanced country in the world -- run by a bunch of villagers who still believe Santa Claus is going to save them.

There is no fucking Easter Bunny. There is no Jesus waiting to return. Moses never even existed. These were all convenient lies from the men of those times to gain power. Their actions were rational -- they wanted to deceive their brethren so that they could amass power. I get their motivations. But I cannot, for the life of me, understand our motivations, thousands of years later, still following the conmen of yesteryear into our gory, bloody, violent end.

Jesus is said to have said on the cross, "My God, My God, why have you forsaken me?" Because Jesus was insane and the God he thought would rescue him did not exist. And he died on that cross like a fool. He fancied himself the son of God and he could barely convince twelve men to follow him at a time when the world was full of superstition.

Excellent marketing by some of his followers would later rescue his botched effort. How many people saw his miracles? One? Twelve? Eighty? Why didn't he show the whole world? Not because this is some giant pop quiz by God to test us -- but because he did not perform any miracles!

Even his apostles can't agree on what miracles he supposedly carried out or when he carried them out. Or whether he returned after death or he didn't. Whether they saw him in person or just as a vision. Rational human beings shouldn't believe this kind of nonsense. Yet most of the world does.

If a man today killed his only son to show how much he loved other people, he would be considered a madman, locked in jail and earn society's contempt. Yet we think this is some sort of noble act by our Father in Heaven.

In Heaven? What, with the harps and the winged angels and the 72 virgins? My God, how stupid do you have to be to believe that?

I know most of you don't actually read your religious texts, and when you do, you assiduously try to avoid the parts that make no sense whatsoever or hide underneath the comforting grasp of your religious leaders who have concocted a bunch of circular logic (a crime to even use that word in regards to Christianity, Islam or Judaism) to shield you from the obvious folly of the written text.

So, I'm not calling you stupid if you haven't really read the material. And I know how powerful brainwashing is. We all received it when we were young and it is exceedingly difficult to break its grasp. But people dance around the issue out of politeness because they don't want to call you what you are -- ignorant.

There are a lot of people I love dearly and respect wholeheartedly who believe in religion. I hate to do this to them. But we have killed far too many people, wasted far too much time on this nonsense for us to keep going in this direction for fear of offense.

Jesus was a lunatic. God is not coming to your rescue. He hasn't come to anyone's rescue in thousands of years, including Jesus. Mohammed was a power hungry, scam artist and ruthless conqueror. Moses and Abraham were figments of the imagination some long dead rabbi. He would probably laugh his ass off at all of you who still believe the fairytales he made up thousands of years ago. He probably wouldn't even believe it if you told him.

Did I mention Judaism? The chosen people? Come on, get off it. People walk around in clothes from 18th century Russia, thinking they have been chosen by God when they look like a bunch of jackasses. I'm tired of all the deaths because we did not want to give offense. Orthodox Jews are wrong and ridiculous.

As are the orthodox and fundamentalists of all of the religions. It says in the Bible that it is an abomination to wear clothes made of two different cloths or to eat shellfish. If you think God will hate you because you mixed wool and linen or because you ate some shrimp, you are insane.

How long are we going to dance around the 800-pound gorilla in the room? The world is run by madmen. It's not just Bush and bin Laden. It is the leader of all of the countries in the Middle East, almost all of the Americas and most of the rest of the world.

Have I offended you? That's too bad. Stop killing each other in the name of false and ridiculous Gods and I will stop ridiculing you. Trust me, your offense is much worse than mine.

Right now as you read this, there are ignorant, hateful Muslims teaching other ignorant Muslims how to put on a suicide belt. There are orthodox Jews telling other Jews how they must never leave their "holy land" no matter what the consequences are to other human beings. They assure their followers -- remember, they are not the chose ones, we are. If we crush and oppress them, don't worry, God will excuse it, and even desires it, because He is on our side.

There are maniacal Christians who are praying for the end of time. Who are hoping that most of the world's population is wiped off the face of the Earth by their vengeful and murderous God. Whom they believe is, ironically, a loving God. Unless, of course, you make the fatal mistake of not kissing his ass and appeasing him, in which case he will slaughter you and condemn you to eternal torture. What kind of sick people believe this?

The kind who live next to you. The kind who voted for George Bush. The kind who send their religious leaders to the White House to argue against even-handedness in the Middle East because it would prevent their sick prophecy. The kind who have undue influence over how we use the greatest and most lethal army ever built by man.

If you don't want to be called ignorant or misinformed, then get informed. Learn the real nature of our universe and put aside old wives tales about resurrected Gods, omniscient prophets and a guy who could split the Red Sea but couldn't find where he's going in the desert for forty years.

It's the year 2005.
Let's start acting like it."-Cenk Uygur

Strong words. Do you have any comments??

tomder55 answered on 10/24/05:

christians have to suffer the fools among us .

Choux rated this answer Bad/Wrong Answer
ETWolverine rated this answer Excellent or Above Average Answer
Itsdb rated this answer Excellent or Above Average Answer

Question/Answer
excon asked on 10/23/05 - Iraq - or maybe it's about Harriet Miers


Hello experts:

Overheard at the White House:

President Bush: "So, I told the Iraqis that they can't have a fair JUDICIAL SYSTEM if they make FUNDAMENTALIST RELIGIOUS beliefs the main criteria for picking JUDGES. Heh-heh."

Bwahahahah.

excon

tomder55 answered on 10/24/05:

there is no getting around it ;the Miers pick is a mistake. The White House digs in deeper the more they try to justify the selection. Rove telling Dobson in a conference call that Miers was an evangelical was bad politics and a point irrelevent to her qualifications . I want an originalist ;not and activist for single issues politics .

Maybe I'm wrong about Miers ;maybe she has broad knowlege of constitutional law and all my apprehensions will be erased during confirmation . But ;there is no need to waith that long . Hundreds of alternatives were there for the taking .

excon rated this answer Excellent or Above Average Answer

Question/Answer
Choux asked on 10/21/05 - Tom DeLay's Mugshot

I have been seeing Tom DeLay's mugshot a few places, and it is so funny, I really get to laughing at it. There he is all posed like he won the JayCee's Man of the Year award! LOL!! Just thinking about it..lol.

His mugshot is a pleasant change from Nick Nolte...Michael Jackson....James Brown....

tomder55 answered on 10/22/05:

it was brilliant politics for him to smile like he won the lottery ;the Dems are going nuts because they had planned to plaster his mug shot on every campaign spot in the next election rounds.

His challenger next year is expected to be former Rep. Nick Lampson, who lost his seat in 2004 after he was forced to run in a new district because of a redistricting plan pushed by DeLay.I understand his anger .

My congressman the great Ben Gilman retired after his district was redrawn out of existance.The people didn't vote him out. The Democrats couldn't lay a hand on him .The things that mattered to him mattered to us. No event was too small for Ben Gilman. He marched with Boy Scouts ; He square-danced with kids ; He never missed a County Fair."You can't forget your people,'' said Gilman "Then you become a statesman and then you're gone.''

When I visited Washington I brought my family unannounced to his office . He did not know me from any other of his constituents but greated me as if he knew me for years. It was my intention to just get a pass to enter the Capitol building and wait on line like the rest of the other tourists .Instead Gilman set me up with his staff. They escorted my family to the subway linking the Capitol with the congressional offices. There they asked the tour guides to allow us into the next tour (they had to ask for a personal favor to the congressman ;a favor that I'm sure was repaid).We were then given passes to sit in the galley of both the Senate and House deliberations.

Now ;the congressional district I live in is primarily in the Bronx;but it snakes up the Hudson river and slices off enough of Gilman's old district that if he ever tried to run in it ,he would not stand a chance against the Democrat Bronx vote. My new congressman ,Eliot Engel is not a bad man ;I just do not feel he represents my interests . He set up an office in my county and occasionally visits it. Actually the part that makes me angriest is that the Republicans ;from the White House ;to Govenor Pataki did absolutely nothing to attempt to fight the carving out of the district.

Anyway ;I understand why Delay has such a big target on him. I just wonder if he did anything criminal. It doesn't appear to be so. This has more and more the appearance of a rail-road job ;from Ronny Earl ,the prosecutor who more and more resembles Captain Queeg (and I predict will also ultimately be destroyed by his mad persuit);and the DNC and moveon.org contributing judge Bob Perkins who in all fairness should be recused from the case.I do not know about Texas ;but in NY a judge is barred from playing politics once the judge is elected ;that includes participation in party politics or donating to public campaigns . The appearance of impartiality must be maintained.


I noticed how some of the left web sites super imposed a picture of Delay sans the smile and super imposed it on a mug shot like backround but I prefer the original .

Choux rated this answer Excellent or Above Average Answer
excon rated this answer Excellent or Above Average Answer
Itsdb rated this answer Excellent or Above Average Answer

Question/Answer
HANK1 asked on 10/21/05 - ABSOLUTELY BRILLIANT:


Major General Dr. Vernon Chong, USAF, Ret. wrote this:

"Our World problem"

"To get out of a difficulty, one usually must go through it. Our country
is now facing the most serious threat to its existence, as we know it,
that we have faced in your lifetime and mine (which includes WWII! ).

The deadly seriousness is greatly compounded by the fact that there
are very few of us who think we can possibly lose this war and even fewer
who realize what losing really means.

First, let's examine a few basics:

1. When did the threat to us start?

Many will say September 11th, 2001. The answer as far as the United
States is concerned is 1979, 22 years prior to September 2001, with the
following attacks on us:
Iran Embassy Hostages, 1979;
Beirut, Lebanon Embassy 1983;
Beirut, Lebanon Marine Barracks 1983;
Lockerbie, Scotland Pan-Am flight to New York 1988;
Dhahran, Saudi Arabia Kh! obar Towers Military complex 1996;
Nairobi, Kenya US Embassy 19 98;
Dares Salaam, Tanzania US Embassy 1998;
Pentagon 2001.

(Note that during the period from 1981 to 2001 there were 7,581
terrorist attacks worldwide).

2. Why were we attacked?

Envy of our position, our success, and our freedoms. The attacks
happened during the administrations of Presidents Carter, Reagan, Bush 1, Clinton
and Bush 2. We cannot fault either the Republicans or Democrats as there
were no provocations by any of the presidents or their immediate predecessors,
Presidents Ford or Carter.

3. Who were the attackers?

In each case, the attacks on the US were carried out by Muslims.

4. What is the Muslim population of the World? 25%

5. Isn't the Muslim Religion peaceful?

Hopefully, but that is really not material. There is no doubt that the
predominately Christian population of Germany was peaceful, but under
the dictatorial leadership of Hitler (who was also Christian), that made no
difference. You either went along with the administration or you were
eliminated. There were 5 to 6 million Christians killed by the Nazis for
political reasons (including 7,000 Polish priests). (see
http://www.nazis.testimony.co.uk/7-ahtm).

Thus, almost the same number of Christians were killed by the Nazis
as the 6 million holocaust Jews who were killed by them, and we seldom
heard of anything other than the Jewish atrocities. Although Hitler kept the
world focused on the Jews, he had no hesitancy about killing anyone who got in
his way of exterminating the Jews or of taking over the world - German,
Christian or any others.

Same with the Muslim terrorists. They focus the world on the US, but kill
all in the way -- their own people or the Spanish, French or anyone else.
The point here is: that just like the peaceful Germans were of no
protection to anyone from the Nazis, no matter how many peaceful Muslims
there may be, they are no protection from the terrorist Muslim leaders and
what they are fanatically bent on doing -- by their own pronouncements--
killing all of us "infidels." I don't blame the peaceful Muslims. What
would you do if the choice was shut up or die?

6. So who are we at war with?

There is no way we can honestly respond that it is anyone other than
the Muslim terrorists. Trying to be politically correct and avoid
verbalizing this conclusion can well be fatal. There is no way to win if you don't
clearly recognize and articulate who you are fighting.

So with that background, now to the two major questions:

1. Can we lose this war?

2. What does losing really mean?

If we are to win, we must clearly answer these two pivotal questions.

We can definitely lose this war, and as anomalous as it may sound,
the major reason we can lose is that so many of us simply do not fathom the
answer to the second question: What does losing mean?

It would appear that a great many of us think that losing the war means
hanging our heads, bringing the troops home and going on about our
business, like post Vietnam. This is as far from the truth as one can get. What
losing really means is:

We would no longer be the premier country in the world. The attacks will
not subside, but rather will steadily increase. Remember, they want us
dead, not just quiet. If they had just wanted us quiet, they would not have
produced an increasing series of attacks against us over the past 18 years.
The plan was clearly for terrorists to attack us, until we were neutered
and submissive to them.

We would of course have no future support from other nations, for fear of
reprisals and for the reason that they would see that we are impotent,
and cannot help them.

They will pick off the other non-Muslim nations, one at a time. It will
be increasingly easier for them. They already hold Spain hostage. It
doesn't matter whether it was right or wrong for Spain to withdraw its troops
from Iraq. Spain did it because the Muslim terrorists bombed their train and
told them to withdraw the troops. Anything else they want Spain to do will be
done. Spain is finished.

The next will probably be France. Our one hope on France is that they
might see the light and realize that if we don't win, they are finished too,
in that they can't resist the Muslim terrorists without us. However, it may
already be too late for France. France is already 20% Muslim and fading
fast!

If we lose the war, our production, income, exports and way of life will
all vanish as we know it. After losing, who would trade or deal with us,
if they were threatened by the Muslims.

If we can't stop the Muslims, how could anyone else?

The Muslims fully know what is riding on this war, and therefore are
completely committed to winning, at any cost.

We'd better know it too, and be likewise committed to winning at any
cost.

Why do I go on at such lengths about the results of losing? Simple. Until
we recognize the costs of losing, we cannot unite and really put 100% of
our thoughts and efforts into winning. And it is going to take that 100%
effort to win.

So, how can we lose the war?

Again, the answer is simple. We can lose the war by "imploding." That
is, defeating ourselves by refusing to recognize the enemy and their
purpose, and really digging in and lending full support to the war effort. If we
are united, there is no way that we can lose. If we continue to be divided,
there is no way that we can win!

Let me give you a few examples of how we simply don't comprehend the
life and death seriousness of this situation.

President Bush selects Norman Mineta as Secretary of Transportation.

Although all of the terrorist attacks were committed by Muslim men
between 17 and 40 years of age, Secretary Mineta refuses to allow
profiling. Does that sound like we are taking this thing seriously? This is war!
For the duration, we are going to have to give up some of the civil rights
we have become accustomed to. We had better be prepared to lose some of our
civil rights temporarily, or we will most certainly lose all of them permanently.

And don't worry that it is a slippery slope. We gave up plenty of civil
rights during WWII, and immediately restored them after the victory and
in fact added many more since then.

Do I blame President Bush or President Clinton before him?

No, I blame us for blithely assuming we can maintain all of our Political
Correctness, and all of our civil rights during this conflict and have a
clean, lawful, honorable war. None of those words apply to war. Get them
out of your head.

Some have gone so far in their criticism of the war and/or the
Administration that it almost seems they would literally like to see us
lose. I hasten to add that this isn't because they are disloyal. It is
because they just don't recognize what losing means. Nevertheless, that
conduct gives the impression to the enemy that we are divided and
weakening. It concerns our friends, and it does great damage to our cause.

Of more recent vintage, the uproar fueled by the politicians and media
regarding the treatment of some prisoners of war, perhaps exemplifies
best what I am saying.

We have recently had an issue, involving the treatment of a few
Muslim prisoners of war, by a small group of our military police.

These are the type of prisoners who just a few months ago were
throwing their own people off buildings, cutting off their hands, cutting out
their tongues and otherwise murdering their own people just for disagreeing
with Saddam Hussein.

And just a few years ago these same type of prisoners chemically killed
400,000 of their own people for the same reason. They are also the same
type of enemy fighters who recently were burning Americans, and dragging
their charred corpses through the streets of Iraq.

And still more recently, the same type of enemy that was and is providing
videos to all news sources internationally, showing the beheading of
American prisoners that they held.

Compare this with some of our press and politicians, who for several days
have thought and talked about nothing else but the "humiliating" of some
Muslim prisoners -- not burning them, not dragging their charred corpses
through the streets, not beheading them, but "humiliating" them.

Can this be for real?

The politicians and pundits have even talked of impeachment of the
Secretary of Defense.

If this doesn't show the complete lack of comprehension and understanding
of the seriousness of the enemy we are fighting, the life and death
struggle we are in and the disastrous results of losing this war, nothing can.

To bring our country to a virtual political standstill over this prisoner
issue makes us look like Nero playing his fiddle as Rome burned --
totally oblivious to what is going on in the real world.

Neither we, nor any other country, can survive this internal strife.

Again I say, this does not mean that some of our politicians or media
people are disloyal It simply means that they are absolutely oblivious
to the magnitude of the situation we are in, and into which the Muslim
terrorists have been pushing us, for many years.

Remember, the Muslim terrorists' stated goal is to kill all infidels!

That translates into all non-Muslims -- not just in the United
States, but throughout the world.

We are the last bastion of defense.

We have been criticized for many years as being 'arrogant.' That
charge is valid in at least one respect. We are arrogant in that we believe
that we are so good, powerful and smart, that we can win the hearts and minds of
all those who attack us, and that with both hands tied behind our back, we
can defeat anything bad in the world!

We can't!

If we don't recognize this, our nation as we know it will not survive,
and no other free country in the World will survive if we are defeated.

And finally, name any Muslim countries throughout the world that
allow freedom of speech, freedom of thought, freedom of religion, freedom of
the press, equal rights for anyone (let alone everyone), equal status or any
status for women---or that have been productive in one single way that
contributes to the good of the world.

This has been a long way of saying that we must be united on this war
or we will be equated in the history books to the self-inflicted fall of
the Roman Empire. If, that is, the Muslim leaders will allow history books
to be written or read.

If we don't win this war right now, keep a close eye on how the
Muslims take over France in the next 5 years or less. They will continue to
increase the Muslim population of France and continue to encroach little by
little on the established French traditions. The French will be fighting among
themselves over what should or should not be done, which will continue
to weaken them and keep them from any united resolve. Doesn't that sound
eerily familiar?

Democracies don't have their freedoms taken away from them by some
external military force. Instead, they give their freedoms away,
politically correct piece by politically correct piece.

And they are giving those freedoms away to those who have shown,
worldwide, that they abhor freedom and will not apply it to you or even
to themselves, once they are in power.

They have universally shown that when they have taken over, they then
start brutally killing each other over who will be the few who control
the masses. Will we ever stop hearing from the politically correct, about
the "peaceful Muslims"?

I close on a hopeful note, by repeating what I said above. If we are
united, there is no way that we can lose. I hope now after the election,
the factions in our country will begin to focus on the critical situation we
are in, and will unite to save our country. It is your future we are talking
about! Do whatever you can to preserve it.

After reading the above, we all must do this not only for ourselves,
but our children, our grandchildren, our country and the world.

Whether Democrat or Republican, conservative or liberal --- and that
includes the politicians and media of our country and the free
world---Please forward this to any you feel may want, or NEED to read
it. Our "leaders" in Congress ought to read it, too.

There are those that find fault with our country, but it is obvious
to anyone who truly thinks through this, that we must UNITE."

HANK

tomder55 answered on 10/21/05:

When did the threat to us start?

before our nation was conceived . sometime after Mohammed became an adult he had a messianic vision that he used to formulate a model for world conquest.

Why were we attacked?

because we do not accept the tenents of Islam and everything we stand for is anathema to the vision of Islamism as described by Wahhabbism


Isn't the Muslim Religion peaceful?

not really relevent . in fact ,most of the Muslims are ;however many of the holy texts read like Mein Kampf.

My take on the rest ;well ;I think surrender and retreat are unacceptable options . We played the realpolitik option for years and that did nothing to make us any safer .It comes down to two options really . The least palatable one is all out war with 25 % of the world . The other option is the one we now persue. By planting the seeds of democracy in the heart of the Islamic world we are hoping to bear fruit . WE are betting that once the people taste it that eventually their world will reform .The early indicators are positive .

ETWolverine rated this answer Excellent or Above Average Answer
HANK1 rated this answer Excellent or Above Average Answer
labman rated this answer Excellent or Above Average Answer

Question/Answer
sapphire630 asked on 10/19/05 - God bless our Politically correct system

Don't ya just love it

Saddam refuses to cooperate with his court hearing because he doesn't recognise it and he gets them to cater to his ways. If I went into court acting the exact same way all I would get would be fined for contempt of court and whatever goes along with that
30 days I would suppose.

tomder55 answered on 10/20/05:

We haven't quite reached the pinnacle yet . The moral equivalence argument has not been advanced by anyone except the moonbats at Air America . They have taken the play book of Noam Chomsky,who argued about the Nuremberg trials after WWII that any Allied attempt to prosecute the Nazis was itself unjust because it was presided over by worse war criminals than Hitler. What really should be asked is this ;how did such a butcher carry on for so long in the face of an international system that pretends to speak for civilization? Why are they not looking in the mirror and asking themselves why Saddam was not stopped earlier ?

sapphire630 rated this answer Excellent or Above Average Answer

Question/Answer
Choux asked on 10/18/05 - Most Corrupt Nations


Last Updated: Tuesday, 18 October 2005, 08:32 GMT 09:32 UK


Corruption leads to much-needed resources leaking overseas
Corruption is on the rise in some rich countries as well as poorer ones, research by anti-corruption watchdog Transparency International suggests.

The group's Corruption Perceptions Index labels Bangladesh and Chad as the most corrupt places on the planet.

The situation worsened in countries such as Costa Rica, Russia and Sri Lanka - as well as Canada and Ireland.

But nations where perceptions of corruption are declining include Hong Kong, Turkey and even Nigeria.

TI's survey asks businesspeople, academics and public officials about how countries they live in or do business with are perceived.


The two scourges (of corruption and poverty) feed off each other, locking their populations into a cycle of misery


The results are used to gauge how corrupt public officials are. The CPI does not deal directly with private-sector corruption.

Usual suspects?

At both the top of the list and the bottom, the index shows little change from 2004.

Topping the list, the cleanest countries are Iceland, Finland and New Zealand, with Switzerland not far behind.

The UK is equal 11th with the Netherlands, with the US back at 17.

Bangladesh and Chad - joint 158th - share the bottom end of the table with the likes of Haiti and Turkmenistan.

Several other parts of the former Soviet Union also fare badly. Russia itself is joint 126th, while Georgia, Kyrgyzstan, Azerbaijan, Uzbekistan and Tajikistan rank even lower.

Resource-rich African states are seen as particularly corrupt, the CPI says. Nigeria, Angola, Equatorial Guinea, Sudan and the Democratic Republic of Congo are all in the bottom 20.

But Nigeria has managed to move up the rankings, from being ranked third-bottom last year.

TI said the survey demonstrated that the corruption continued to threaten development by hampering growth and putting off investors.


THE BEST AND THE WORST
5 least corrupt states:
Iceland
Finland
New Zealand
Denmark
Singapore

5 most corrupt states:
Chad
Bangladesh
Turkmenistan
Burma
Haiti
Source: Transparency International

"Corruption is a major cause of poverty as well as a barrier to overcoming it," said TI chairman Peter Eigen.

"The two scourges feed off each other, locking their populations into a cycle of misery."

But although poor countries stood to gain the most from fighting corruption, TI said richer countries needed to take responsibility too, by investigating and prosecuting companies suspected of corrupt practices abroad and barring them from public contracts.

Action

Corruption has been high on the official development agenda for some years, but campaigners have often argued that governments only pay lip-service to it.

Recently, however, attitudes appear to be changing.

The United Nations Convention against Corruption comes into force in December 2005, enshrining in international law rights to pursue looted resources sent overseas.

In the UK, the government's Commission for Africa has called for a much tougher line on the proceeds of corruption and their repatriation.

Similarly, London's Metropolitan Police is working on an initiative to strengthen economic crime prevention, including anti-corruption activities, across the Commonwealth.


Any commets? I must say that I was surprised about Canada....My perception had been that Canada was not too corrupt....

tomder55 answered on 10/19/05:

I think I posted about this a while ago .;that bloggers in Canada (actually it started just across the border with Michigan bloggers) were reporting information the Canadian gvt was desperate to cover-up. It involves the Public Works Ministry and overall corruption in the ruling Liberal Party .If you google sponsorship scandal or "AdScam"you'll find more information than you need . The scandal involved the misuse and misdirection of funds that were intended to go to government advertising in Quebec over the preceding decade. The funds were apparently allocated to advertising firms that were allies of the Quebec branch of the federal Liberal Party, and evidence suggests that in some cases few or no services were rendered in return.


The Gomery Commission was set up by Paul Martin to investigate .Most of the testimony heard by the Commission has been public, but Judge Gomery decided to create a publication ban on the testimony of three key witnesses: Jean Brault, president of the ad agency Groupaction, Charles Guite, an officer of the Public Works ministry who worked on the Sponsorship Program, and Paul Coffin, president of the ad agency Coffin Communications. The potential damage of their testimony has unnerved the Liberal Party .

Choux rated this answer Excellent or Above Average Answer

Question/Answer
Itsdb asked on 10/18/05 - Today's AP (Anti-American Press)

Besides the report that Zimbabwe and Venezuela's leaders blame the US for "famine, war, and pollution" with Mugabe calling Blair and Bush "two unholy men of our millenium," this Associate Press blurb was in my paper this morning:

United Nations - "A Human Security Report issued Monday, and financed by five governments, paints a surprising picture of war and peace in the 21st century: Armed conflicts have declined by more than 40 percent since 1992, and genocide and human rights abuses have plummeted around the world.

The only form of political violence that appears to be getting worse is international terrorism - a serious threat that kills extraordinarily few people compared to wars, it said."

~~~~~~~~~~~~~~~~~~~~~~~~~~~~~~~~~~~~~~~~~~~~~~~~~~~~~~~

So what exactly are we supposed to take from this analysis?

Steve

tomder55 answered on 10/18/05:

certainly not the idea that the UN had anything to do with the reduction in armed conflicts (although I'm sure Kofi Annan is willing to take credit ) . logic says that the collapse of the Soviet Union and it's state sponsor of insurgencies around the world is the biggest factor .I guess for kudos you could throw in that the only remaining superpower is not expansionist. Take the example of Afghanistan The commission admits that "more than 3.5 million Afghans have returned to their homeland since the end of 2001" one of the most remarkable reversals of refugee flows in history;and then gives the credit to the United Nations"when the Bonn Agreement set Afghanistan on the long and bumpy road to political stability and socio-economic development." I guess the liberation of the country had nothing to do with it . There has also been a simular return of refugees to Iraq but I'm sure Koffi will take simular credit not only for that reversal ;but also for the vote this week .

Itsdb rated this answer Excellent or Above Average Answer

Question/Answer
sapphire630 asked on 10/17/05 - Louis Farakan's strange statement

He claimed when he was somewhere in South America he was climbing a mountain and "they" signaled to him and took him to a smaller spaceship where he was taken to a bigger ship where he talked with Eli Mohammed.

If Pat Robertson made a statement that God took him up into the clouds and talked to him people would definitely be all over him for such a statement.

P.S. One of the actors (from star trek)ashes have been sent up to orbit the earth for something like a hundred years.


Sapph
stuck to gravity (or is that reality?)

tomder55 answered on 10/18/05:

yup screwy Looueee believes he was taken up to the mother wheel to meet personally with Elijah Muhammed . Farrakhan's followers have long held that Elijah Muhammad did not die, but escaped a death plot, was restored to health, and is aboard "that huge wheel-like plane that is even now flying over our heads." Among Muhammad's passengers on the so-called "Mother Wheel" is the mysterious figure named W.D. Fard, a light-skinned man who Muhammad said came from the Middle East and told him he was Allah. Farrakhan's followers believe that Muhammad is "the Last Messenger of Allah" and will soon return and lead them to redemption.

If you care for a good chuckle the speech he made in 1996 is here

That Mother Wheel is a dreadful looking thing. White folks are making movies now to make these planes look like fiction, but it is based on something real. The Honorable Elijah Muhammad said that Mother Plane is so powerful that with sound reverberating in he atmosphere, just with a sound, she can crumble buildings. And the final act of destruction will be that Allah will make a wall out of the atmosphere over and around North America. You will see it, but you won't be able to penetrate it. He said Allah (God) will cut a shortage in gravity and a fire will start from 13-layers up and burn down, burning the atmosphere. When it gets to the earth, it will burn everything. It will burn for 310 years and take 690 years to cool off.



sapphire630 rated this answer Excellent or Above Average Answer

Question/Answer
excon asked on 10/17/05 - Mark Furhman


Hello:

The reason OJ was acquitted is because Mark Furhman lied. He has been, in my view, totally discredited.

Since then, he has been interviewed many times on TV, and appeared as a pundit.

My question is, why would anybody consider him an expert on anything, or worthy of exhibiting an opinion? What does it say about us?

To me, a lying cop is the scum of the earth. But, that's just me.

excon

tomder55 answered on 10/17/05:

He worked 20 years in the force and up to the OJ case was one of the top investigators . He found the bloody glove that later "didn't fit " and put it back at the crime scene a shame that he "framed "a guilty man because that guilty man is now free and Furhman's 20 year career was ruined It was a lack of judgement in not following strict procedures ,he also pled no contest to perjury charges so you are correct in saying that he lied in testimony about "racist comments " he had made not related to the case.His book on the subject is as good a reference as any of the other 60 books on the OJ murders if you discount his white-washing of the mistakes he made .But facing the truth ;there were plenty of seedy characters and incompetent people in that drama;including the OJ defense team ;judge Ito ;the prosecutor team and the media circus that the press produced .Many of them are also subsequently published .

Since the case he used his experience as an investigator to almost single-handedly solve the murder of Martha Moxley (Michael Skakel, a relative of the Kennedy family was the murderer . Skakel was convicted for the murder in June 2002.)He has also written a book that spells out the case pretty well about the key suspicions and inconsistencies involving Michael Schrivo .

What can I say ;punditry doesn't require a clean past .Anyone can determine if what the person says has any veracity based on the persons past or the experience they bring to the table .As far as investigative journalism goes he brings more to the table than most of the pundits do .

excon rated this answer Excellent or Above Average Answer
sissypants rated this answer Excellent or Above Average Answer

Question/Answer
excon asked on 10/15/05 - For Pdub, and the rest


Hi again, Pdub:

Referring to my previous question,

>>>Is it good for whom?<<<

It should be noted, that when I speak of what's good for "us", the "US" I am referring to is the "us", who our founding fathers called the "people".

Never, never, in my wildest dreams, would I propose something that would be good for the cops in their never ending quest to lock us all up.

Let me ask you a question? We've got 2 million people behind bars, AND we have the Bill of Rights. How many people do you think would be locked up if we didn't have a Bill of Rights?

(a) No more. (b) Lot's more. (c) All of us.

excon

tomder55 answered on 10/15/05:

without the bill of rights the constituton would be toilet paper . You will be happy to discover that the first decision of the Roberts court has been rendered and that it affirms defendents habeus corpus protection . (Dye v. Hofbauer.The Supreme Court reverses a Sixth Circuit decision to deny habeas relief and reinstates the federal habeas petition of a Michigan man convicted of murder in state court.)

excon rated this answer Excellent or Above Average Answer

Question/Answer
excon asked on 10/15/05 - Philosophically speaking


Hello:

Is it good to have laws that can't be, or aren't being enforced?

excon

tomder55 answered on 10/15/05:

you must be talking about speed limits !! as arbitrary an enforcement schedule as ever there was one .

I am interested in the enforcement of wet land laws in my area . Our local community passed ordinances that I helped champion to strenghten them .They are good for water retention and drainage which can be a huge issue by us when we have weeks in the N.E. like we've had this week. But the town has neither the will or the resources (I guess ) to enforce them so every real estate project has some component of land fill .

I agree with you're oveall point that laws that are unenforced are essentially non-laws .But in many cases the answer is to bring in executives who care to enforce the laws .Some laws need to be revised or eliminated ;some need to be strenghtened . The mayor has been served notice .He has one more year to get his act together or I will have a busy autumn next year .

excon rated this answer Excellent or Above Average Answer

Question/Answer
Itsdb asked on 10/12/05 - Did anyone notice?

besides the fact the Cowboys thrashed the Eagles on Sunday, but "Iraqi Politicians Reach Deal on Amendments"

By THOMAS WAGNER
Associated Press Writer

BAGHDAD, Iraq (AP) -- President Jalal Talabani and other top politicians on Wednesday praised as "historic" the last-minute compromises that negotiators reached on the draft constitution and urged Iraqis to vote "yes" in this weekend's referendum.

"I have good news for the Iraqi people on this historic day. An agreement has been reached on amendments to the draft constitution," Talabani said during a nationally televised news conference. "There is no excuse for Arab Sunnis to boycott the vote now that we have responded to all their demands and suggestions."

He was followed at the microphone by several other Iraqi politicians who also praised the compromises reached on Tuesday night by Sunni, Shiite and Kurdish powerbrokers on the charter ahead of Saturday's referendum.

They included National Assembly Speaker Hajim Al-Hassani, a Sunni; Vice President Ghazi al-Yawer, a Sunni; former Prime Minister Ayad Allawi, a Shiite; and Abdul Aziz Al-Hakim, a Shiite who heads the Iraqi United Alliance, the largest coalition in parliament.

The draft constitution already has been printed by the United Nations and millions of copies are being distributed to the public for the vote. New additions cannot be included now.

Therefore, the only way that they can be announced to potential voters is through the media, and Talabani began that campaign Wednesday with his televised news conference.

The breakthrough deal reached Tuesday night by powerbrokers from Iraq's Shiite majority and Sunni and Kurdish minorities attempts to address concerns among Sunnis that have prompted many of them to say they will vote "no" in the referendum.

Most Shiites and Kurds plan to vote "yes," but Sunnis fear the draft document being distributed to voters will fragment Iraq, allowing Shiites and Kurds to create mini-states in the oil-rich north and south, leaving Sunnis in a poor central zone.

The deal the negotiators reached Tuesday night agrees on a mechanism to consider amending the constitution after it is approved in Saturday's nationwide vote.

The next parliament, to be formed in December, will set up a commission to consider amendments, which would later have to be approved by parliament and submitted to another referendum. That would give Sunnis the ability to try later to introduce major changes they want, aimed at reducing the autonomous powers that Shiites and Kurds would have under the federal system created by the charter, negotiators said.

The agreement, which U.S. officials apparently promoted behind the scenes, boosts the chances that the draft constitution will be passed in Saturday's referendum..."

~~~~~~~~~~~~~~~~~~~~~~~~~~~~~~~~~~~~~~~~~~~~~~~~~~~~~~~

And btw, the special session of Iraq's National Assembly did approve today. So will it pass now?

Steve
P.S. ex, remember my suggestion on the negative publicity? On this article, the AP has links to the "Proposed Iraqi Constitution" directly followed by "Key Findings on WMD" and also, "Prison Abuse Scandal," ,500 U.S. Deaths," "Street Fighting" and "Coalition Casualties." Why?

tomder55 answered on 10/13/05:

It was going to pass even without the additional concessions to the Sunni. However ;what they did was "punt" on some of the controversial issues .That was prudence at it's best and simular to our founders deferring to decide on the issue of slavery as per Article 1 sec.9 clause 1 of the US Constitution until 1808

Itsdb rated this answer Excellent or Above Average Answer

Question/Answer
excon asked on 10/12/05 - How could anyone not be against torture?


Hello experts:

And, while were at it.....

What does it say about this president, when he threatened to use his veto, for the FIRST time in his presidency, to defeat a law that says we would prohibit the cruel, inhuman or degrading treatment of prisoners in the custody of the US military?

The Republican controlled senate passed the measure 90-9.

Some of you will argue, as you have in the past, that our actions are not as horrifying as al_Qaidas, so we should not be concerned. Since when did al Qaida become the standard by which we measure the morality of our country?

excon

PS. To Its: Yes, I copied some words from the NY Times.

tomder55 answered on 10/12/05:

He should've used his veto sooner ;but it was sheer cowardice of the Republicans in the Senate to attach this as a rider to the defense appropriations bill . If they feel that strongly about it then do their job and make the law and debate it on it's own merit . Now you have a scenario that with up coming elections anyone who votes against the bill because of this rider will be accused of not supporting the troops .That is why they voted the way they did .It had NOTHING to do with their position about the merit of the rider . It was cheap and sleezy and McCaine should be ashamed to move such important legislation ;that would curtail the ability of the Chief Executive to execute the war ,in such a way . He becomes more Democrat by the day .

I ask again ...and will continue to ask; what methods of interogation are acceptable ? The rider does nothing to spell out the specifics .

Erewhon rated this answer Poor or Incomplete Answer
excon rated this answer Excellent or Above Average Answer
LTgolf rated this answer Excellent or Above Average Answer
purplewings rated this answer Excellent or Above Average Answer

Question/Answer
CeeBee2 asked on 10/12/05 - Gas hog

What's with this new character, an energy hog, being promoted by the Bush administration? It looks creepy.

tomder55 answered on 10/12/05:

ya got Smokey the Bear to prevent forest fires ;now ya got Energy Hog !

here is the logo .

I think it accurately depicts wasteful use of fuel

CeeBee2 rated this answer Excellent or Above Average Answer
Choux rated this answer Excellent or Above Average Answer

Question/Answer
Choux asked on 10/12/05 - al-Zawahri Speaks

al-Qaida No. 2: U.S. 'Ran' From Vietnam

By KATHERINE SHRADER
WASHINGTON (AP) - In a letter to his top deputy in Iraq, al-Qaida's No. 2 leader said the United States ``ran and left their agents'' in Vietnam and the jihadists must have a plan ready to fill the void if the Americans suddenly leave Iraq.

``Things may develop faster than we imagine,'' Ayman al-Zawahri wrote in a letter to his top deputy in Iraq, Abu Musab al-Zarqawi. ``The aftermath of the collapse of American power in Vietnam - and how they ran and left their agents - is noteworthy. ... We must be ready starting now.''

Senior U.S. military commanders have said that Iraqi security forces are improving significantly and some U.S. forces could return home early next year. Yet skeptics have raised concerns about whether such statements simply let the insurgency know how long they must wait for the U.S. to leave.

In a letter taking up 13 typed pages in its English translation, al-Zawahri also recommended a four-stage expansion of the war that would take the fighting to neighboring Muslim countries.


Comments?

tomder55 answered on 10/12/05:

I printed it out this morning and have finished reading it .(full text here

Reading Zawahiri's letter is almost enough to make you feel sorry for him. Zawahiri was once a doctor, Zarqawi was a Jordanian street thug and is now a mass murderer. You can almost hear Zawahiri's remorse for unleashing the thug. He protests against al Qaeda's descent into nihilism and sadism ;the ultimate destination of all totalitarian creeds.
Zawahiri's letter is replete with evidence of al Qaeda's last grasps of survival.

My dear brother, we are following your news, despite the difficulty and hardship... I made sure in my last speech-that Aljazeera broadcast Saturday, 18 June 2005-to mention you, send you greetings, and show support and thanks for the heroic acts you are performing in defense of Islam and the Muslims, but I do not know what Aljazeera broadcast. Did this part appear or not?
Likewise, I showed my support for your noble initiative to join with your brothers, during a prior speech I sent to the brothers a number of months ago, but the brothers' circumstances prevented its publication.

I want to reassure you about our situation. The summer started hot with operations escalating in Afghanistan. The enemy struck a blow against us with the arrest of Abu al-Faraj, may God break his bonds. However, no Arab brother was arrested because of him. The brothers tried-and were successful to a great degree-to contain the fall of Abu al-Faraj as much as they could.

However, the real danger comes from the agent Pakistani army that is carrying out operations in the tribal areas looking for mujahedeen.

I have a definite desire to travel to you but I do not know whether that is possible from the standpoint of traveling and getting settled, so please let me know.

Please take every caution in the meetings, especially when someone claims to carry an important letter or contributions. It was in this way that they arrested Khalid Sheikh.

Likewise, please, if you want to meet one of your assistants, I hope that you don't meet him in a public place or in a place that is not known to you. I hope that you would meet him in a secure place, not the place of your residence. Because Abu al-Faraj - may God set him free and release him from his torment - was lured by one of his brothers, who had been taken into custody, to meet him at a public location where a trap had been set.

The brothers informed me that you suggested to them sending some assistance. Our situation since Abu al-Faraj is good by the grace of God, but many of the lines have been cut off. Because of this, we need a payment while new lines are being opened. So, if you're capable of sending a payment of approximately one hundred thousand, we'll be very grateful to you.

I don't know if you all have contact with Abu Rasmi? Even if it is via the Internet, because I gave him a copy of my book (A Knight under the banner of the Prophet) so he could attempt to publish it, and I lost the original. Al-Sharq al-Awsat newspaper published it truncated and jumbled. I think that the American intelligence services provided the aforementioned newspaper with it from my computer which they acquired, because the publication of the book coincided with a publication of messages from my computer in the same newspaper. So if you can contact him and get the original of the book, if that is possible for you all, then you can publish it on your blessed website and then send a copy to us, if that is possible.



This is not the picture of a successful organization. Zawahiri does lays out their strategy :

The first stage: Expel the Americans from Iraq.

The second stage : Establish an Islamic authority or amirate, then develop it and support it until it achieves the level of a caliphate- over as much territory as you can to spread its power in Iraq, i.e., in Sunni areas

The third stage: Extend jihad to the secular countries neighboring Iraq.

The fourth stage: the clash with Israel


The US will not be driven out of Iraq but He states clearly he is hoping for a Vietnam like outcome .

You might ask an important question: What drives me to broach these matters while we are in the din of war and the challenges of killing and combat?
My answer is, firstly: Things may develop faster than we imagine. The aftermath of the collapse of American power in Vietnam-and how they ran and left their agents-is noteworthy.


Of course this comparison is all the rage with the anti-war crowd;the MSM and many of the Democrat opposition ;No wonder Zawahiri finds the Vietnam precedent encouraging.

Zawahiri makes it clear that al Qaeda is a Sunni organization and is at war with Shia .

The collision between any state based on the model of prophecy with the Shia is a matter that will happen sooner or later. This is the judgment of history, and these are the fruits to be expected from the rejectionist Shia sect and their opinion of the Sunnis. These are clear, well-known matters to anyone with a knowledge of history, the ideologies, and the politics of states....
And do the brothers forget that we have more than one hundred prisoners - many of whom are from the leadership who are wanted in their countries - in the custody of the Iranians? And even if we attack the Shia out of necessity, then why do you announce this matter and make it public, which compels the Iranians to take counter measures ?


now that's what I call winning hearts and minds ! It's a losers strategy .

Choux rated this answer Excellent or Above Average Answer

Question/Answer
ETWolverine asked on 10/12/05 - What happened in New Orleans?

Hey guys,

Ive been thinking about everything that went wrong in New Orleans for the past few weeks. Theres no question that things went wrong in the aftermath of Hurricane Katerina. But what caused the situation to be so screwed up? I believe that the problems began long before Katerina ever hit.

In order to determine where the system failed, I compared New Orleans to the response in New York in the aftermath of 9-11. I know that there are many differences between these two tragedies in terms of scope, area of effect, logistics and so forth. But there are a number of similarities too, in terms of number of lives affected, the fear involved, etc. So there is a fairly good basis of comparison.

As I said, I believe the problems began long before Katerina. If we compare New Orleans to New York, we find some important trend differences in terms of crime and law enforcement: to wit, New York has experienced a decade-long decrease in crime rates. The decrease in the NYC crime rate has been so extreme that it has both led the country in rate of decrease, and increased the national average rate of decrease. By contrast, New Orleans has experienced a decades-long increase in crime, especially violent crime. New Orleans is (or was until Katerina hit) the murder capital of the USA. Most muggings in New Orleans end in murder, because the criminals believe that it is easier to kill the witness than risk being caught. And where there are no witnesses, the NOPD doesnt seem to be able to solve cases.

Another trend difference is with regard to nuisance crimes and simple cleanliness of the streets. New York has been cleaning up its act (literally) for over a decade now. Times Square, once the toilet of the USA, is now the exemplification of big-city family oriented tourism. The bums are (for the most part) gone, the pot-holes are being fixed, the sex trade is relegated to limited areas, and nuisance crimes like littering, loud music and graffiti are aggressively handled by the cops. By contrast, Anew Orleans is a pig sty. Its dirty, its broken down, its a slum, and bums sleep and piss wherever they feel like. The streets are broken down, and nuisance crimes are simply something that residents and visitors have to deal with.

And then theres the way the cops are treated by the local government. In NYC, the cops, under Giuliani and Bloomberg, have been given the authority the freedom and the tools to do their jobs. And because of that, the cops are usually seen as a) doing their jobs effectively, and b) real authority figures, even by those who dislike them. As a result, the cops see themselves that way, and are proud of who and what they are. By contrast, the policies in New Orleans have stressed criminal rights over the ability of cops to fight crime. The cops have their hands handcuffed in dealing with crime, because they are more worried about covering their butts than fighting crime and risking being sued or jailed for doing their jobs. And as a result, the cops are viewed as ineffective and lacking in authority and that is how they see themselves as well.

What does all of this have to do with Hurricane Katerina?

Simple: if cops see themselves as ineffective, useless and lacking authority for long enough, and if they are faced with a big enough problem, theyll stop doing their jobs and just look out for their own families instead, just as 1/3 of them did when Katerina hit. On the other hand, if the cops see themselves as worthwhile, effective, valued and good at their jobs, theyll run into a burning building to try to help save a life because its their job and they are proud of their job.

If the people see that their streets are a pig sty for long enough, and the local government doesnt care enough about them to do something about it, then they will stop listening to and relying on the government when they need to. If they are so lacking in self worth for such a long period of time, they stop valuing everyone else too, and it becomes a situation of every man for himself. On the other hand, if the people see that the government cares enough to clean up their streets and get rid of the garbage and the potholes, it makes them feel valued, and that in turn allows them to value others, and thus help others in a time of crisis.

If crime is a way of life, then looting and shooting will occur, because that is how people survive. If crime is looked upon with disdain, then people will not (for the most part) become criminals during a time of crisis.

What Im saying is that the policies in place for years before the various tragedies occurred set the stage for the peoples and the authorities reactions in the aftermaths of the crises.

Most people think that Rudy Giuliani was a great leader for how he reacted in the wake of 9-11. Hogwash. Rudy became a great leader long before 9-11 by implementing policies that created a leadership environment. He created the situation in which cops would be listened too, firemen would be listened to, and he himself would be listened to. His reaction during 9-11 was simply the culmination of the effectiveness of those policies. Rudy became a leader when he started leading after his election, long before 9-11 hit.

By contrast, Ray Nagin has been anything but a leader during his administration. He has kept the old policies of his predecessors. He hasnt led the way on a single issue of note. He certainly hasnt curtailed crime or given cops the power to do their jobs. Ray Nagins failures as a leader took place long before Katerina hit. And the same for Kathleen Blanco. If you lack leadership policies during normal times, then dont expect to be able to lead in a crisis.

But what about the failures of FEMA and the Bush Administration?

Lets compare and contrast again: FEMAs reaction to 9-11 was just fine. FEMAs reaction to Katerina and Rita in Mississippi and Alabama was just fine as well. It was only in Louisiana that large-scale problems occurred. One could infer from this that Louisiana was different because of how the people reacted to them which was a direct result of the policies discussed above. The people didnt listen to the authority figures of FEMA because they distrust and lack respect for all authority figures. Their basis of experience is that authority figures are ineffective. They shot at FEMA rescue workers because in their experience if you want something you shoot in order to get it, and that includes wanting to be rescued.

Any failures on FEMAs part were a direct result of the policies of the policies in place in New Orleans. Just as the successes of FEMA and the rescue workers after 9-11 were the result of the policies in place in New York. Its all about the leadership.

Elliot

tomder55 answered on 10/12/05:

too bad you weren't around during the height of the debate .My body of work defending the Federal Response can be found on both the Christianity and this board . I shudder to think that people would abandon the principles of limited govenment and federalism over something that demands strong and decisive response at the local level ;starting with individual and family responsibilites to take care of you're own .

I linked and article from the Atlantic Monthly to a response to Claude about the American stoic attitude about natual disasters . This was published before the hurricanes so America may be in the process of reevaluation but it says alot.

Bernard-Henri Lvy has been traveling the US for the Atlantic Monthly as a sort of latter-day Tocqueville observing and reporting about our country . He wrote in the August addition about America's relationship with the weather :

For a European, one of the most enigmatic characteristics of the American ethos is its relationship with nature.

There's the wildness of nature here, first of all. The closeness of this wild nature that we tend to think has been domesticated by technologywhen in fact it's just been pushed back a little, moved farther away. Here, for instance, in the Everglades, in this national park scarcely thirty miles from Miami, it's been contained within an immense reservation right at the edge of populated areas. In Europe, I believe, they would have exterminated the wildlife that continues here to paddle around in the swamp's deep waters. I am convinced that these boa constrictors, these lizards, these cottonmouths with their deadly poison, these powerful blue herons that feed on baby alligatorsand the alligators themselves, presented to us as the "guardians of the Everglades" and carefully observed by the old nature buffs in the countywould doubtless have been victims of the great prophylactic cleanup demanded by European civilization, whose dream ever since Descartes has been to turn us into masters and possessors of nature. Not here. Here there is no real mastery. No possession. The Floridians don't tame nature; they push it back. Instead of subjugating it, they drive it away. Florida is so vast, and space is of such lesser importance than it is in Europe, that there's room for both city and nature. And the same goes for California, where, my friend Charlie Lyons tells me, some nights he hears coyotes howling in the hills behind his house.

There is the violence of nature. There is the extreme brutality, also unimaginable in Europe, not just of certain animals but of the elements, especially hurricanes and tornadoes. I heard them talked about at every stage of my journey, and I ended up realizing that they are more numerous and, in a sense, more devastating in the United States than anywhere else among so-called developed countries. "Florida under attack!" a disheveled, livid journalist shouted on CNN the other day, live from some coastal town buffeted by a storm in this paradise for retirees. Attack by what? I wondered. Who was attacking? Which Osama bin Laden or Saddam Hussein emulator? But it was just Jeanne. It was the nascent Hurricane Jeanne, coming from the Bahamas, fast approaching the southeastern American coast. It would be easy in this case to wax ironic. One could detect in this journalist's anxiety an additional manifestation of the American taste for grand spectacles and exaggeration. But you might snigger a little less if you tried to imagine behind Jeanneand also, in recent months, behind the familiar names Alex, Frances, Ivan, Charley, Karl, and Lisa; or, last year, Kate, Larry, Isabel, Erika, Ana, and Claudettethe torrential floods, the walls of furious water beating down on the beaches, the houses with their roofs blown off, the rain of frogs and lizards, the trees uprooted; in short, the landscapes of desolation that we have no actual concept of in France and that three weeks ago in Punta Gorda, for instance, resulted in sixteen dead People in the United States don't need to imagine; they know. (And this knowledge feeds their extreme sensitivity to this kind of cataclysm when it takes the form of a tsunami and devastates a destitute country.)

Finally, the most striking aspect for a European when faced with this implacable recurrence of natural catastrophes, some of which (Hurricane Andrew; the Mississippi flood of 1927) have gone down in history and have shaped the construction of the American landscapethe most incomprehensible thingis the relatively passive roles of politicians and citizens. Oh, I'm well aware of how television carries on about the weather. I know that Florida has the most effective meteorological-forecasting stations in the world. But let me tell you about Homestead. I'll take the example of this town on the road to the Everglades, in a landscape of fake trees painted yellow, orange, blue, and red as if to liven things up, this town devastated a dozen years ago by Hurricane Andrew, and also hit by many of the ensuing hurricanes. What takes you by surprise in Homestead is the vulnerability of the houses. What bewilders and stuns you is that everything has been rebuilt just as it was before, with the same prefab kits and the same kinds of trailers, which look as if they've been set down ready-made, patched together, a little rickety. You wonder what will keep them from flying apart in the same way when the next Andrew, Mitch, or Allison comes along.

America has the means to protect Homestead. The America that hasn't ceased to dream of the Star Wars missile-defense shield has the most effective warning and prevention systems in the world. But, strangely enough, it doesn't use even a tenth of its capacity to keep the inhabitants of Homestead out of danger by strengthening building and insurance codes. Just as I've never seen a European airport as profoundly paralyzed as the major American airports can be by a snowstorm, for instance, so I can't imagine the principle of precaution so poorly applied in my country as it is here in Homestead. Why is it so neglected?

There's the culture of risk, stronger than the culture of security and the inclination to self-protection.

There are the remains of a pioneering spirit that for decades, or rather for centuries, has accommodated itself to a sense of temporary habitat, perched as it were on the side of the road, pressing forward with the frontier, and by definition precarious.

But there is also, anchored deep in the mentality of the country, a magical, semi-superstitious relationship to what Americans, even the secular ones, are prone to call Mother Nature. As if their omnipotence found its limits there, reached its rational confines there. As if the Promethean will to get the better of all things and all people imposed on itself a limit of principle and wisdom in this relationship to the elements. No pity for our enemies, the American of the twenty-first century seems to be saying; no mercy for terrorists, certainly, or even for opponents of the country's economic supremacy. But we'll let nature take her best shot.




ETWolverine rated this answer Excellent or Above Average Answer
ladybugca rated this answer Excellent or Above Average Answer

Question/Answer
Itsdb asked on 10/11/05 - Republican vs. Democrat women

I'm a little weary of disasters, body counts, and death tolls so today I decided I needed a change of pace.

Mike S. Adams, one of my favorite columnists, revealed this week why he became a Republican. The reason? "Republican women are simply more attractive than Democratic women."

Since someone finally came out and said it I'll admit I've often thought the same thing. And you? Ladies, feel free to offer your take on Republican vs. Democrat men - or not. :)

Steve

tomder55 answered on 10/11/05:

I don't know ......is Jessica Biel a Republican ?

Itsdb rated this answer Excellent or Above Average Answer

Question/Answer
excon asked on 10/10/05 -
Hypocrisy


Hello experts:

Today, the FBI announced that it would no longer ban previous pot smokers from working at the agency.

So, when they busted me for pot, instead of sending me to jail, they could have recruited me? If that wasn't so sad, I'd be laughing.

How does it feel to live in, and support a country, that is so hypocritical???

excon

tomder55 answered on 10/10/05:

that's really not the issue you know . I think that if you could convince them that you were once an occasional toker ;maybe say "I didn't inhale" ;and worked in other government agencies like the CIA or the State Dept.where you were doing top secret work or was an intelligence analysts, linguists, computer specialists ,but now wanted to transfer to the FBI ....Then maybe just maybe the FBI will be willing to give up their otherwise straight-laced image .But if you wanted to be a special agents or an investigators in the field ;you're out of luck .

My problem with the FBI currently has nothing to do with who they hire. They are not disclosing the true facts behind the attempted homicide bombing by Joel Hinrichs at OU which the press is being silent about also.

excon rated this answer Excellent or Above Average Answer
Itsdb rated this answer Excellent or Above Average Answer
sissypants rated this answer Excellent or Above Average Answer

Question/Answer
Choux asked on 10/06/05 - From Bush's Speech Today

WASHINGTON (AP) - President Bush sought Thursday to revive waning public support for the war in Iraq, accusing militants of seeking to establish a ``radical Islamic empire that spans from Spain to Indonesia'' with Iraq serving as the main front.

Islamic radicals are being sheltered by ``allies of convenience like Syria and Iran,'' Bush declared in a speech before the National Endowment for Democracy.

He said the United States and its allies had foiled at least 10 plots by the al-Qaida terror network in the four years since the Sept. 11 terror attacks - three of them in the United States - and he warned other nations not to support or harbor groups with al-Qaida ties."

Comments about these specific remarks??
Thanks

tomder55 answered on 10/10/05:

This is what I think . The slow inevidible march of Arabs and to a greater extent Islam towards democratic principles has been given more than a little nudge by the US since 9-11. All Arab states now have adopted the rhetoric of democracy whether their leadership believes it or not .Even if they don't believe it ;they are finding that they have to make accomodations to democratic principles because the wave of expectations ios beginning to overwhelm them .All other attempts at state governance has failed there be it socialism, Arab nationalism, Islamism, Baathism, monarchism.It has not been a guarantor of either rights or security .Pressure for reforms both politically and economically have converged with activists in these countries who can no longer be silenced .They recognize that they will be left behind if they do not adopt more liberal economic policies in this global marketplace.

"Iraqis are tired of their history being written by violent minorities, by men who ride to power on tanks. On January 30, the silent majority of Iraqis, the long-suffering millions for whom no regional leader ever spoke up, the disenfranchised whose lives were blighted by war and persecution, will take power through the ballot box. What could possibly be wrong with that?"
- Maha Alattar( Assistant Professor at the University of North Carolina and a member of the Women's Alliance for a Democratic Iraq)


We are on the verge of a major breakthrough there . When one arab state like Iraq or Turkey shows that the democratic model can succeed in the region then the flood gates will be open and there will be no stopping progress.

Bush came the closest yet to identifing the enemy in his speech . It is well that he make simular speeches more often lest we forget who the enemy is . In politics the big lie eventually becomes the truth .The counter to that is if the truth is told often enough and we remain committed to leading this effort we will demonstrate the power of our ideals.

Choux rated this answer Excellent or Above Average Answer

Question/Answer
kindj asked on 10/06/05 - Miers nomination

This is a bit long, but an interesting read. It offers a bit more insight into the politics surrounding all this Supreme Court hoopla. All that follows is written by another person:


Don't misunderestimate Miers
October 4th, 2005


President Bush is a politician trained in strategic thinking at Harvard Business School, and schooled in tactics by experience and advice, including the experience and advice of his father, whose most lasting political mistake was the nomination of David Souter. The nomination of Harriet Miers to the Supreme Court shows that he has learned his lessons well. Regrettably, a large contingent of conservative commentators does not yet grasp the strategy and tactics at work in this excellent nomination.

There is a doom-and-gloom element on the Right which is just waiting to be betrayed, convinced that their hardy band of true believers will lose by treachery those victories to which justice entitles them. They are stuck in the decades-long tragic phase of conservative politics, when country club Republicans inevitably sold out the faith in order to gain acceptability in the Beltway media and social circuit. Many on the right already are upset with the President already over his deficit spending, and his continued attempts to elevate the tone of politics in Washington in the face of ongoing verbal abuse by Democrats and their media allies. They misinterpret his missing verbal combativeness as weakness.

There is also a palpable hunger for a struggle to the death with hated and verbally facile liberals like Senator Chuck Schumer. Having seen that a brilliant conservative legal thinker with impeccable elite credentials can humble the most officious voices of the Judiciary Committee, they demand a replay. Thus we hear conservatives sniffing that a Southern Methodist University legal education is just too non-Ivy League, adopting a characteristic trope of blue state elitists. We hear conservatives bemoaning a lack of judicial experience, and not a single law review article in the last decade as evidence of a second rate mind.

These critics are playing the Democrats game. The GOP is not the party which idolizes Ivy League acceptability as the criterion of intellectual and mental fitness. Nor does the Supreme Court ideally consist of the nine greatest legal scholars of an era. Like any small group, it is better off being able to draw on abilities of more than one type of personality. The Houston lawyer who blogs under the name of Beldar wisely points out that practicing high level law in the real world and rising to co-managing partner of a major law firm not only demonstrates a proficient mind, it provides a necessary and valuable perspective for a Supreme Court Justice, one which has sorely been lacking.

Ms. Miers has actually managed a business, a substantial one with hundreds of employees, and has had to meet a payroll and conform to tax, affirmative acttion, and other regulatory demands of the state. She has also been highly active in a White House during wartime, when national security considerations have been a matter of life and death. When the Supreme Court deliberates in private, I think most conservatives would agree that having such a perspective at hand is a good thing, not a bad thing.

Other conservatives are dismayed that the President is playing politics (!), rather than simply choosing the best candidate. But the President understands that confirmation is nothing but a political game, ever since Robert Bork, truly one of the finest legal minds of his era, was demonized and defeated.

The Presidents smashing victory in obtaining 78 votes for the confirmation of John Roberts did not confirm these conservative critics in their understanding of the Presidents formidable abilities as a nominator of Justices. Au contraire, this taste of Democrat defeat whetted their blood lust for confirmation hearing combat between the likes of a Michael Luttig or a Janice Rogers Brown and the Judiciary Committee Democrats. Possibly their own experience of debating emotive liberals over-identifies them with verbal combat as political effectiveness.

In part, I think these conservatives have unwittingly adopted the Democrats playbook, seeing bombast and gotcha verbal games as the essence of political combat. Victory for them is seeing the enemy bloodied and humiliated. They mistake the momentary thrill of triumph in combate, however evanescent, for lasting victory where it counts: a Supreme Court comprised of Justices who will assemble majorities for decisions reflecting the original intent of the Founders.

Rather than extend any benefit of the doubt to the Presidents White House lawyer and counselor, some take her lack of a paper trail and a history of vocal judicial conservatism as a sign that she may be an incipient Souter. They implicitly believe that the President is not adhering to his promise of nominating Justices in the mold of Scalia and Thomas. The obvious differences between Souter, a man personally unknown to Bush 41, and Miers, a woman who has known Bush 43 for decades, and who has served as his close daily advisor for years, are so striking as to make this level of distrust rather startling. Having seen the Souter debacle unfold before his very eyes, the President is the last man on earth to recapitulate it.

He anticipates and is defusing the extremely well-financed opposition which Democrat interest groups will use against any nominee. Yes, he is playing politics by nominating a female. A defeated nominee does him and the future of American jurisprudence no favors. By presenting a female nominee, he kicks a leg out from under the stool on which the feminist left sits. Not just a female, but a career woman, one who has not raised children, not married a male, and has a number of firsts to her credit as a pioneer of women's achievement in Texas law. Let the feminists try to demonize her.

If they do so, almost inevitably, they will seize on her religious beliefs and practice. Some on the left will not be able to restrain their scorn for an evangelical Christian Sunday school teacher from Dallas, and this will hurt them. They will impose a religious test against a member of a group accounting of a third of the voting base. Speculation on her being a lesbian has already started. "She sure seems like a big ol' Texas lesbian to me," as one of the Kos Kidz put it.

They are going to make themselves look very ugly.

The President must also prepare himself for a possible third nominee to the Court. With the oldest Justice 85 years old, and the vagaries of mortality for all of us being what they are, it is quite possible that a third (or even fourth) opportunity to staff the Court might come into play. Defusing, demoralizing and discrediting the reflexive opposition groups in the Democrats base is an important goal for the President, and for his possible Republican successors in office.

Then there is the small matter of actually influencing Supreme Court decision-making.

This president understands small group dynamics in a way that fewif any of his predecessors ever have. Perhaps this is because he was educated at Harvard Business School in a legendary course then-called Human Behavior in Organizations. The Olympian Cass Gilbert-designed temple/courtroom/offices of the Supreme Court obscure the fact that it is a small group, subject to very human considerations in its operations. Switching two out of nine members in a small group has the potential to entirely alter the way it operates. Because so much of managerial work consists of getting groups of people to work effectively, Harvard Business School lavishes an extraordinary amount of attention on the subject.

One of the lessons the President learned at Harvard was the way in which members of small groups assume different roles in their operation, each of which separate roles can influence the overall function. The new Chief Justice is a man of unquestioned brilliance, as well as cordial disposition. He will be able to lead the other Justices through his intellect and knowledge of the law. Having ensured that the Courts formal leader meets the traditional and obvious qualities of a Justice, and is a man who indeed embodies the norms all Justices feel they must follow, there is room for attending to other important roles in group process.

According to a source in her Dallas church quoted by Marvin Olasky, Harriet Miers is someone who

taught children in Sunday School, made coffee, brought donuts: "Nothing she's asked to do in church is beneath her."

As the courts new junior member, the 60 year old lady Harriet Miers will finally give a break to Stephen Breyer, who has been relegated to closing and opening the door of the conference room, and fetching beverages for his more senior Justices. Her ability to do this type of work with no resentment, no discomfort, and no regrets will at the least endear her to the others. It will also confirm her as the person who cheerfully keeps the group on an even keel, more comfortable than otherwise might be the case with a level of emotional solidarity.

But there is much more to it than group solidarity, important though that ineffable spiritual qualty may be. Ms. Miers embodies the work ethic as few married people ever could. She reportedly often shows up for work at the White House at 5 AM, and doesnt leave until 9 or 10 PM. I have no doubt that she will continue her extraordinary dedication to work once confirmed to the Court. She will not only win the admiration of those Justices who work shorter hours, she will undoubtedly be appreciated by the law clerks who endure similar hours, working on the research and writing for the Justices. These same law clerks interact with their bosses in private, and their influence intellectual and emotional may be more profound than some Justices might like to admit.

The members of the Supreme Court all see themselves as serving the public and the law to the best of their abilities. Their self-regard depends on their belief in the righteousness and fairness of their deliberations. They must listen to the arguments of the other Justices. But their susceptibility to viewpoints they had not yet considered is matter of both an intellectual and emotional character. Open-mindedness uusally requires an unfreezing of deeply and emotionally-held convictions.

Having proven herself capable of charming the likes of Harry Reid, leader of the Senate Democrats, is there much room for doubt that Harriet Miers is capable of opening up opponents emotionally to hear and actually consider as potentially worthwhile the views of those they might presume to be their enemies?

George Bush has already succeeded in having confirmed a spectacularly-qualified intellectual leader of the Court in Chief Justice Roberts. If conservatives dont sabotage his choice, Harriet Miers could make an enormous contribution toward building Court majorities for interpretations of the Constitution faithful to the actual wording of the document.

Thomas Lifson is the editor and publisher of The American Thinker.

tomder55 answered on 10/07/05:

I don't know enough about her yet for me to endorse her selection. I do not believe I will either. This was a weak pick from the President who is trying to avoid another fight. Yesterday's disgraceful vote by the Senate ;attaching a rider to the defense approriations bill that limits the way the commander in chief can conduct the war against jihadistan ;proves that Bush cannot necessarily depend on his Republican Senate for support .Therefore he elected not to have an ideological battle over his court selection . He picked instead someone without a paper trail . That may be safe politics but it aint good governanace . I do not understand why the ideological debate can be waged on boards like this but not in the halls of our representatives .

kindj rated this answer Excellent or Above Average Answer

Question/Answer
excon asked on 10/06/05 - The Constitution strictly interpreted


Hello righties:

>>>Amendment IX

The enumeration in the Constitution, of certain rights, shall not be construed to deny or disparage others retained by the people. <<<


I too, am an originalist. The Bill of Rights is short, simple, clearly written and to the point. I believe the words mean what they say and not something else. I would like a justice to be able to read them as they are written and uphold laws that conform to them.

Therefore, having read and understood the Ninth Amendment, alliterating those non-enumerated rights, such as privacy, the rights of homosexuals to wed, and those yet to be revealed, are consistent with the meaning of this amendment.

Clearly, as it becomes apparent in the future that other citizens rights, which also have not yet been enumerated and are being violated, those too must be added to the growing list of enumerated rights.

This amendment appears to trump the idea that a citizen is not entitled to rights that arent specifically listed in the Constitution. If it doesnt mean that, what does it mean? Were they just kidding when they wrote it?

If you believe what the Ninth Amendment says, and of course, an originalist must (you cant pick and choose which amendments you like), then by what procedure would these non-enumerated rights be revealed to us, if ever?

excon

tomder55 answered on 10/07/05:

I interpret it as a little differently . Clearly the meaning of "retained by the people " means that legislative and not judical action should be the criteria for rights expansions .Until the 19th Amendment was ratified, the issue of whether or not women could vote was left to the states. A state could grant women the right to vote if it chose to do so, and the federal government couldn't stop them. But if a state chose not to, the federal courts couldn't step in and order them to give women the vote by claiming that women had a "right to vote" under the 9th Amendment. That's why it took a constitutional amendment to secure nationwide female suffrage.Substitute that with any other right ;like privacy ,and you have the gist of the debate .Did the courts step in and say that women had a right to vote ? no ;it took an Constitutional Amendment to grant them the right nationally .
But today the Supreme Court,usurps the legislatures authority to set social policy. It has seized from the people the power to make such determinations. Five justices is all it takes to substitute their personal judgments for those of Congress and every state government. Instead of focusing on individual controversial issues look at the larger picture . This intrusion by the courts ;this power grab was never intended by the founders . None of the Federalist Papers mention the courts determining which rights you have . The Supreme Court should refuse to be drawn into making public policy, and it should strike down legislation only when a clear constitutional violation exists.

excon rated this answer Excellent or Above Average Answer
kindj rated this answer Excellent or Above Average Answer

Question/Answer
excon asked on 10/06/05 - No TV. No Newspapers. No Internet.


Hello experts:

If you don't watch TV, and don't read newspapers, and don't go online, how would you know what's going on?

Is it good that a leader of a country does that?

excon

tomder55 answered on 10/06/05:

probably not ;but his time is so tight that the information he needs gets filtered by his staff before he sees it anyway .That is pretty much SOP for the Presidency .

Frankly the executive dept has become so unwieldy that it has got to be a difficult task to manage. When he let's his Dept Heads run it he is accused of having a detached style ;Others like Carter got buried in minutia . The real question is if he is effective . This year he is in a slump but his first term I thought he was effective.

CeeBee2 rated this answer Excellent or Above Average Answer
Choux rated this answer Excellent or Above Average Answer
excon rated this answer Excellent or Above Average Answer
labman rated this answer Excellent or Above Average Answer

Question/Answer
Choux asked on 10/05/05 - Shocking Revelation

A foreign spy worked in the White House in the office of the Vice President....Al Gore and Dick Cheney's Vice Presidencies.

See the following article: ABC NEWS Oct. 5, 2005 "Both the FBI and CIA are calling it the first case of espionage in the White House in modern history.

Officials tell ABC News the alleged spy worked undetected at the White House for almost three years. Leandro Aragoncillo, 46, was a U.S. Marine most recently assigned to the staff of Vice President Dick Cheney.



"I don't know of a case where the vetting broke down before and resulted in a spy being in the White House," said Richard Clarke, a former White House advisor who is now an ABC News consultant.

Federal investigators say Aragoncillo, a naturalized citizen from the Philippines, used his top secret clearance to steal classified intelligence documents from White House computers.

In 2000, Aragoncillo worked on the staff of then-Vice President Al Gore. When interviewed by Philippine television, he remarked how valued Philippine employees were at the White House.

"I think what they like most is our integrity and loyalty," Aragoncillo said.

Classified Material Transferred by E-Mail

Officials say the classified material, which Aragoncillo stole from the vice president's office, included damaging dossiers on the president of the Philippines. He then passed those on to opposition politicians planning a coup in the Pacific nation.

"Even though it's not for the Russians or some other government, the fact that it occurred at the White House is a matter of great concern," said John Martin, who was the government's lead espionage prosecutor for 26 years.

Last year, after leaving the Marines, Aragoncillo was caught by the FBI while he worked for the Bureau at an intelligence center at Fort Monmouth, N.J.

According to a criminal complaint, Aragoncillo was arrested last month and accused of downloading more than 100 classified documents from FBI computers.

"The information was transferred mostly by e-mails," said U.S. Attorney Christopher J. Christie at the time of Aragoncillo's arrest.

Since that arrest, officials say Aragoncillo has started to cooperate. He has admitted to spying while working on the staff of Vice President Cheney's office.

Comments?

tomder55 answered on 10/06/05:

The White House should investigate all the staffers they inherited to see if any other 'vetting' issues comes up . The administrations change hands at the top but the executive operatives and bureocrats work from administration to administration .

Richard Clarke was national security advisor in the White House at the time so he should shut up about the "vetting "that went on . He's also full of crap about security breaches that happened under his watch .In fact the whole 'Chinagate'fiasco ;the espionage of Wen Ho Lee at Los Alamos happened under his watch while the rest of the country obsessed about stains on Monica's dress.

This Leandro Aragoncillo should be tried and then shot as far as I am concerned ;but his gathering of intel. about the Phillipine leaders pales in comparison to selling our national security out to the Chinese for campaign contributions (of which Al Gore was the big player ).Even worse ;the justice dept.s handling of the coverup is related to the security failures that resulted in the attacks on 9-11.

Choux rated this answer Excellent or Above Average Answer

Question/Answer
Choux asked on 10/03/05 - Frist Biggest Tenn. Liar Since Davy Crocket

Christpopher Byron of the NewYork Post::

"The news that both the Securities and Exchange Commission and the Department of Justice have opened probes of Senate Majority Leader Bill Frist's recent skulking about on Wall Street offers some welcome illumination concerning the current state of law enforcement on capitalism's street of dreams: Basically, there ain't none . . . and what little that exists is driven mainly by the press.

As a result (and as has been argued many times in this space over the past couple of years), it may not yet be possible to get away with murder on Wall Street. But if you keep your felonious behavior to a level that doesn't breach the frontiers of grand larceny, you can make a nice living on Wall Street as a white-collar lawbreaker.

Given the facts of what Frist actually did, and his ***preposterous*** after-the-fact explanations for them, the senior senator from Tennessee may go down in history as the Volunteer State's biggest liar since Davy Crockett."



tomder55 answered on 10/04/05:

He is as guilty as Martha was .and you know what I thought of the Martha coviction.

Chris Byron;s assertion that Wall Street is lawless fails the test of credibility .NY Att. General Elliot Spitzer is not content unless he is making headlines daily with his personal crusade against the Wall Street crowd. As Slate gushingly declared "He may be America's most powerful politician outside Washington. He has transformed a sleepy office into the nation's dominant regulator and re-engineer of the financial services industryall in the name of protecting consumers......Spitzer is just as relentless as Giuliani was when he was a prosecutor. " His crusade btw has cost me and every other taxpayer in NY a pretty penny in taxes ;in cases that are mostly not justified or overblown . He is the NY equivelent of Ronnie Earle .

The gist of the Byron article is that the SEC and U.S. Attorney's office in Manhattan failed to act until the press reported the story . That may well be true ;but they cannot act on hearsay like the press can. They actually need evidence that something illegal went down and that is not an easy task in the financial world.

It doesn't seem likely that Frist did not have 'insider information ' .There was a series of sales of the stock from insiders of HCA while at the same time they were reporting rising earnings ; there was no indication that the company was going to report a loss . So the suspicions are real.

But it is also possible that he was just divesting himself of the stock because he was contemplating a Presidential run. I do not think he should be thrown under the bus like the Republicans did to Delay until all the facts are known . If Frist can be shown to have had access to information about HCA's upcoming earnings news before it became public, he is politically toast no matter what the legal outcome.If it can't ;then Frist will survive .



Choux rated this answer Excellent or Above Average Answer
Itsdb rated this answer Excellent or Above Average Answer

Question/Answer
Choux asked on 10/03/05 - Serious Fraud Afghani Elections

KABUL, Afghanistan, Oct. 2 - Election officials and observers said Sunday that with 80 percent of the ballots counted in Afghanistan's national and provincial elections, they had found significant incidents of fraud.

Whole districts have come under suspicion for ballot box stuffing and proxy voting, said Peter Erben, the chief of the United Nations-assisted Joint Election Management Board. He said ballot boxes from 4 percent of the 26,000 polling places - about 1,000 stations - had been set aside for investigation on suspicion of fraud and other irregularities.

The European Union observer mission said the reports of fraud and possible voter intimidation in places were "worrying."

Comments?

tomder55 answered on 10/04/05:

significant incidents of fraud....hmmmm....
what were they ....hanging chads ? or maybe reports of police barring minorities from voting ; or maybe registering dead people ? or maybe the election fiasco in Washington State ;or Ohio ? or maybe they have a situation like in NJ where the 2000 court decision (Bush v Gore )allowed the party in power to appoint a US Senator who is now running for Govenor and when he wins that he will resign the Senate and appoint his replacement......on and on I could go about allegations and actual fraud in the American system .

This is their 1st parlimentary ballot in many years . I did not expect it to go perfect. Maybe they should've invited Jimmy Carter there to certify it .

Choux rated this answer Average Answer
Itsdb rated this answer Excellent or Above Average Answer
purplewings rated this answer Excellent or Above Average Answer

Question/Answer
excon asked on 09/30/05 - Bush has no clue!


Hello experts:

Heres a good example of why we are in deep, deep doo doo in Iraq. The president's confidante, Karen Hughes, has been on a "listening tour" of the Middle East. After meeting with Sheikh Muhammad Sayyed Tantawi, the Grand Imam of Al-Azhar University, the academic center of Sunni Islam, she said it was a "wonderful meeting," because the two of them were able to talk "about the common language of the heart."

Hearing Hughes talk about Sheikh Tantawi, you could almost forget what he said in 2002, as translated from a report by the Middle East Media Research Institute (MEMRI), when he called on Palestinian Muslims to "intensify the martyrdom operations against the Zionist enemy" men, women and children and described the barbarous slaughter as "the highest form of Jihad operations" and "a legitimate act according to (Islamic) law." Maybe that's the "spirit of love" Hughes was gushing about.

Then there was what Sheikh Tantawi said in 2003, also reported by MEMRI, when he called for jihad against U.S. forces in Iraq. "Jihad is an obligation for every Muslim when Muslim countries are subject to aggression," he explained. "The gates of Jihad are open until the Day of Judgment, and he who denies this is an infidel or one who abandons his religion."

Amazingly, even after everything we've been through, Bush, Hughes, and her advisors are still unaware of who is a terrorist and who is not.

excon

tomder55 answered on 09/30/05:

Hughes is there to improve America's image . I say our image gets improved when they see Iraqis and ;Lebanese ;and Afghanis voting in free elections . When they see reforms that are in motion gaining momentum they will thank us for the part we play. We should not concern ourselves with our image with mullahs ;iman and clerics .We should care what the arab on the street thinks .

U.S.-made goods (cars, for example) that don't sell anywhere else still enjoy robust markets in Arab countries. America has been the No. 1 foreign tourist destination for Arabs since the 1980s, and has remained so despite restrictions imposed on Arab visitors after 9/11. Arabs from all walks of life and of all political sensibilities also love to send their children to study in America. American hotel chains ,restaurants thrive in the arab world . Hollywood movies are still a big source of entertainment there;80 percent of the films shown in Arab cinemas are made in Hollywood.American style clothing is worn in every place but in the most restrictive area ;especially in Iran. More than half of all major articles in the two main pan-Arab daily newspapers come from The New York Times, The Washington Post, USA Today, the Los Angeles Times, Newsweek and Time magazines and other U.S. publications. Some American columnists have become household names in most Arab countries.(no wonder the negative impression then!)

I would think then her task is easy . Continue speaking the truth about the nature of our common enemy (islamo-fascism ),and pressuring 'the power' to speed up reform .

excon rated this answer Excellent or Above Average Answer

Question/Answer
excon asked on 09/30/05 - Freedom Center


Hello experts:

What "anti-American" activites would/could take place in the proposed/defeated Freedom Center? What would be "American" activities that could have taken place there?

Would it be that right wing conservative activities would be considered "American"? Whereas left of center activites would be considered "anti-American"?

Pretty arrogant, if you ask me.

excon

tomder55 answered on 09/30/05:

perhaps it is arrogant. but the cast of characters who are the driving force behind the IFC are a who's who of what I call America haters ,and many of their exhibits were going to be anti-American .Can't spin it any other way . They would take a site like the WTC grounds ;hallowed grounds to many Americans and certainly to the vicitims families and create a museum that will be given a high-tech, multimedia tutorial about Native American genocide to the lynchings and cross-burnings of the Jim Crow South. I do not argue that such a museum should not be built . I say that to put the museum there is an insult. Any museum there should be a tribute to the police ;and firefighters who are the heroes of 9-11 ;and to the victims who's only fault was to go to work that day .

Tom Bernstein is the driving force behind the project . He is an activist lawyer who put his name to an amicus brief on behalf of "dirty bomber" Jose Padilla.Others include Michael Posner;Anthony Romero;and George Soros .

But my favorite America hater in the group is Eric Foner, history professor at Columbia University who, even as the bodies were being pulled out Ground Zero, wrote, "I'm not sure which is more frightening: the horror that engulfed New York City or the apocalyptic rhetoric emanating daily from the White House." This is the professor at a "teach-in" at Columbia who said "The only true heroes are those who find ways to defeat the U.S. military," and called for "a million Mogadishus." Saying something like that is anti-American to me.

excon rated this answer Excellent or Above Average Answer
kindj rated this answer Excellent or Above Average Answer

Question/Answer
Choux asked on 09/29/05 - Air America Radio Healthy

Cut and Paste

"Recently, Air America Radio came under attack from the same cast of right wing media characters who have attacked the Network for ideological reasons from day one.

A recent piece in the New York Post by John Mainelli states that Air America is in...bad financial shape. On September 20th Bill OReilly on Fox News which, like the New York Post is owned by Rupert Murdochs News Corporation said that Air America could be on its last legs.

This is untrue.

Air America is in strong financial shape. Last week we started broadcasting from our new multi-million dollar studios.
Several weeks earlier the Board of Directors of Air Americas parent company accelerated re-payment of a loan from the Gloria Wise Boys and Girls Club of $875,000 two years in advance of a previously agreed upon re-payment plan. In the last several months, Air America has expanded its executive team to augment our efforts on the internet and in affiliate relations.

The pretext for the latest smears is an initiative I launched last week called Air America Associates, in which I asked our listeners to support our programming financially and at various levels offer bumper stickers, tote bags, etc. as a way of thanking them. (We received thousands of responses, far beyond what we projected for the first few days).

Many of our listeners also listen to NPR stations and Pacifica and are used to supporting radio programming they like. I got the idea from the Nation Magazines program The Nation Associates which helps them fund investigative journalism. Like Air America Radio, The Nation is a for-profit company.

But the conservative propagandists have tried to make it seem like there is something unseemly because Air America Radio is both commercialand a radio network, as OReilly said last night, I have never seen a commercial enterprise ask their listeners for moneyever This is also false. The modern model of the broadcasting business involves numerous revenue streams. If anything, Air America has been late in fully building such an infrastructure which the Associates is a part of.

For example, Rush Limbaughs Web-site offers his fans the Limbaugh Letter for $34.95 a year and a totally separate service called Rush 24/7 which includes access to archived programs at the cost of $49.95 a year. The Limbaugh site also features the EIB Store which sells such items as $19.95 polo shirt which amusingly says, My Mullah went to Gitmo and all I got was this lousy t-shirt.

The Sean Hannity Web-site features a subscription to something called, The Hannity Insider for $5.95 a month.

But no one tops the self proclaimed non-spinner Bill OReilly. Bill OReilly.com offers a premium membership for either $4.95 a month or $49.95 a year. He also offers a Gift certificate for $14.95. Products for sale on the Web site include:

-- Radio Factor diner coffee mug available in white or navy blue for $14.95
- OReilly Factor keychain for $7.95 while supplies last.
--Three different No Spin tote bags at $14.95 apiece
--Ten different hats at a cost of $16.95 each
--The no spin jacket for $79.95
--The Unisex Black Fleece embroidered with The Spin Stops Here for $39.95
--Several bumper stickers including one that reads Boycott France for $2.50
--License plate frame for $18.95
--Three different No Spin tote bags at $14.95 each
--An OReilly Factor Gear Bag at $64.95
-- Mens Garment Bag for $64.95 (sorry ladies!)
--a Spin Stops Here organizer briefcase
--A Spin Stops Here pen and pad bundle for $19.95
--Two different designs of Spin Stops Here doormats for $49.95 and
--Two different Rain Stops Here umbrellas at $24.95(Show everyone who protects you from the rain)

Mainellis article also repeated another falsehood about Air America saying More recently the 70 station left network has been suffering lower ratings. His corporate cousin OReilly wishfully stated on August 17th said Air Americanobody is listening to it, On Aug 3rd OReilly claimed that Air America cannot support itself because of low ratings and on July 26th OReilly said The Air America radio network continues to fail with catastrophic ratings here in New York City.

In fact, the ratings for the Bill OReilly radio show in New York were worse than those on Air America that he described as catastrophic In the key 25-54 year demographic which talk radio offers to advertisers, the Spring, 2005 Arbitron ratings showed that Monday-Friday from 2-4 PM when OReilly is on WOR-AM and which at Air Americas 1190 WLIB-AM contains the last hour of The Al Franken Show and the first hour of The Randi Rhodes Show, that OReilly had a .3 share and Air America a .4 share. OReilly had a cumulative audience of 75,400 and Air America had a cumulative audience of 89,300.

Inevitably ratings go up and down and vary from time slot to time slot and from market to market. Right wing bloggers have had fun cherry picking isolated pieces of ratings reports to distort the enormous enthusiasm Air Americas growing audience has demonstrated. At the vast majority of our affiliates Air America ratings are up. On a nationwide basis the most recent Arbitron ratings Spring 2005 book showed that our affiliates reach over three million people per week each of whom listens for an average of several hours a week. This is more than triple the amount of people who were listening when measured one year earlier in the Spring, 2004 book.

I do not intend to write something every time something like this happens. In the almost six months during which I have been CEO of Air America Radio, I have refrained, for the most part, from responding to the litany of attacks, lies, half-truths and smears from various members of the right-wing media. In general, it seems to me that paying too much attention to these people only encourages them and that we, at Air America, need to get used to the fact that the spirited progressive opinions of our on-air talent and of our audience will attract the kind of mean-spirited smears that are endemic to contemporary political conversation.

After having a near monopoly on talk radio for so many years, some conservative media types are literally freaked out at confronting robust, persistent and passionate opposition. On Sept 26th, OReilly desperately claimed that Air Americas basic flaw is that Americans do not want to hear that their country sucks 24 hours a day. Of course the talent and management of Air America have a love of our country which is what animates all passionate debate on political issues form the left, right and center.
It is an obsession with stifling debateeven at the cost of using lies and distortions, which is un-American."

To those who spread lies, comments?

tomder55 answered on 09/30/05:

lol ;the theft of money is now a loan?

In response to the OReilley segment about the stations 'fund drive' ,Randi Rhodes said :Last night yet another story on Fox that wasnt true.........OReillyyoure a neutered Bush lap dog. Your show should be on Animal Planet.

Their point is well taken about the merchandising that OReilley and Rush does . The difference to me was the way they are marketting it . Air America is using the PBS model of asking for donations for the goods like they are some kind of charitable enterprise. That is deceiving . You have to admit that asking donations from the listeners of commercial radio is an unususal move. If they want to merchandise air-head bobble dolls then they should just do so. Most stations give away their bumper stickers ,and you can get your name on many of them for free on days like your birthday . Air America wants $250 for the privilidge . Their appeals drive sounds too much like begging and begging doesn't sound like growth to me .


They are being deceptive about the condition of their network . I take their word for it about their financial situation because I do not feel like reseaching it . I dont have access to the networks financial reports so I dont know how much money they have or what their debts are. But, it doesnt take a genius to at least wonder if something isnt wrong . Their ratings should be a huge source of concern to them . Yesterday it was announced that they were getting the heave-ho in the Philly market (WHAT-AM).Ratings were abysmal.In the New York market ;one of the bluest of the blue state ;their ratings are low and declining (about a 1.2 % share ). Recently WLIB ;which had been a caribeen format before Air America went back to it in the late night period. Even during the election cycle their #s were in decline .

Check out the ratings from the last period :

In Washington, D.C., WWRC fell to a 0.3 rating and is in last place in the market.

In Boston, two stations take up the last two spots in the ratings chart with ratings of 0.4 and 0.2.

In New York, the flagship WLIB has just a flat 1.0 rating and is in 24th place.

In Detroit, WDTW was a flat 0.4 which is tied for last in the city.

In Chicago, WCPT is stuck at 0.4 and just out of the cellar in the market.

In San Francisco, KQKE 1.4 in the most liberal city in the county and is in 23rd place. The station is down 0.5 from last summer.

In San Jose, KQKE 1.1, but is down from 1.5 .

In Los Angeles, KLTK 0.9, 30th place in the market.

In Monterey-Santa Cruz Air America Radio station didn't even make the ratings list.

In San Diego, KLSD, which was once considered one of the AAR stations that was succeeding, is flat at a 1.7 rating. That is down nearly a full point from last Summer.

In Riverside-San Bernardino, CA KCAA doesn't even make the ratings list.

In Philadelphia, WHAT turned in a 0.5 rating

In Akron, Ohio, WARF is 0.4 last place

In Al Franken's hometown of Minneapolis, KTNF is down to just a 0.9 rating.

In Cincinnati, the hometown of Jerry Springer, WCKY got a 0.9 rating

In Denver, KKZN actually would be considered a "success" 2.1 rating which is up slightly but is still just 19th place in the market.

In Dallas, KXEB is 0.5

In Columbus, Ohio, WTPG 0.7.

In Seattle, another "success" ; KPTK is 1.8 and in 24th place in the market.

In Miami, WINZ is 2.0

In Atlanta, WWAA 0.6 rating is near the bottom of the market.

In Honolulu, KUMU doesn't even make the ratings list.

Phoenix KXXT is up to a 1.2 rating.

I cannot gage what their long term viability is in new markets like XM radio and they claim success in un-monitored markets like Internet streaming but why would the numbers be any different ?

Choux rated this answer Excellent or Above Average Answer

Question/Answer
excon asked on 09/29/05 - 600 Inmates left to die!


Hello:

Officers in New Orleans simply walked away from a locked and flooding jail building that housed 600 inmates. They were left to fend for themselves (how, I don't know) for 3 days.

Did some die? I dunno. Some are missing. Stay tuned.

The animals in the zoo weren't left like that. No wonder that bastard quit. To those of you who are surprised at the behavior of these sonofabitches - shame on you for not paying better attention. It doesn't, however, surprise me in the slightest.

I'm sure I'll hear from some of you that the prisoners themselves are to blame. Shame on you too.

excon

tomder55 answered on 09/30/05:

I have to be consistent . I thought the efforts of the locals were dismal across the board; That includes now the correctional system also . Prisoners abandoned in their cells ;nursing house patients left to drown; general population herded into the Super Dome without provisions . They certainly had plenty of time to draw up adequate plans to deal with all these contingencies .It had to be a forewarning that the Red Cross, well before the storm ,refused to set up a permanent shelter systems inside the city limits .

Bottom line.....it's a dumb place to build a city; but human history is replete with examples of folly.

Choux rated this answer Excellent or Above Average Answer
excon rated this answer Excellent or Above Average Answer
sissypants rated this answer Excellent or Above Average Answer

Question/Answer
Choux asked on 09/29/05 - Clinton gives Death Blow to IFC

"Hillary Rodham Clinton yesterday dealt a crushing blow to the International Freedom Center planned for Ground Zero, saying she wants the project canned for failing to listen to the 9/11 families.

"I cannot support the IFC," Clinton declared last night in a strongly worded statement in response to an inquiry from The Post.

Her tough comments are Clinton's first significant remarks about the controversy raging at Ground Zero over the Freedom Center, which 9/11 families and other critics fear will become a center of anti-Americanism.

"While I want to ensure that development and rebuilding in lower Manhattan move forward expeditiously, I am troubled by the serious concerns family members and first responders have expressed to me," Clinton said.

"The LMDC [Lower Manhattan Development Corp.] has authority over the site and I do not believe we can move forward until it heeds and addresses their concerns."

The family members of victims, as well as unions representing the city's cops and firefighters, want nothing less than the Freedom Center being booted from Ground Zero.

Given her influence, Clinton's hard line could spell doom for the Freedom Center's hopes of remaining at the World Trade Center site.

Clinton spoke out the day after the IFC released a plan intended to save its spot at the site, but it was met with immediate opposition from 9/11 families.

Clinton won't support any plan unless the families and first responders back it, said her spokesman, Philippe Reines.

Many relatives of 9/11 victims denounced the Freedom Center plan as an insult to the 2,749 people who diedat the Twin Towers because it would paint them as a little more than a footnote to the world's march toward freedom.

The families, cops and firefighters say the IFC's plan to use hallowed land at Ground Zero to highlight poverty as a barrier to freedom diminishes the tragedy of 9/11.

Sen. Chuck Schumer (D-N.Y.) also voiced concern yesterday and called for a compromise although he didn't state flat-out opposition to the Freedom Center.

"There's got to be a way to meet the families' sincere and real needs and build a center that honors the freedom that the victims died for. We hope that the LMDC will find some common ground quickly," Schumer said.

Gov. Pataki who wields strong influence over the LMDC, which will soon decide the Freedom Center's fate is traveling abroad and has yet to take a stand on the Freedom Center's latest proposal. Pataki has said thathe won't support any plan that offers a forum for anti-Americanism.

Clinton's opposition means that the anti-IFC push is now a bipartisan cause. Three New York Republicans Reps. John Sweeney (Saratoga), Peter King (L.I.) and Vito Fossella (S.I.) are already challenging it as a "blame America first" project.

Yesterday, the trio of Republicans formally requested a congressional oversight hearing as a step toward blocking the IFC from getting any of the $2.7 billion in federal funds allocated for Ground Zero.

"The whole thing was hijacked. If you asked people on the street what they wanted at Ground Zero, this would be the last thing that they wanted," Sweeney said."


I read up on the IFC.

We have to give Hillary her props!

tomder55 answered on 09/30/05:

yes we do ;I give her all the credit in the world for sticking her finger in the air and seeing which way the wind blows . but it was not her but Govenor Pataki who made the call.where was she at the beginning of this issue months ago ? there was a growing grass roots movement that would not tolerate the anti-American IFC to be built there which am proud to have done my part . That stupid crescent in Shanksville is next to go .

Choux rated this answer Excellent or Above Average Answer

Question/Answer
labman asked on 09/29/05 - Majority rule?

Remember these names. None of them seem to care that the majority of the voters voted for a man likely to appoint supreme court judges that have read the constitution and feel obligated to follow it in their rulings.

Akaka (D-HI)
Bayh (D-IN)
Biden (D-DE)
Boxer (D-CA)
Cantwell (D-WA)
Clinton (D-NY)
Corzine (D-NJ)
Dayton (D-MN)
Durbin (D-IL)
Feinstein (D-CA)
Harkin (D-IA)
Inouye (D-HI)
Kennedy (D-MA)
Kerry (D-MA)
Lautenberg (D-NJ)
Mikulski (D-MD)

Obama (D-IL)
Reed (D-RI)
Reid (D-NV)
Sarbanes (D-MD)
Schumer (D-NY)
Stabenow (D-MI)

tomder55 answered on 09/29/05:

the Obama vote disappoints me a little. the rest are the ususal suspects . the good news is that this group would not have the clout to maintain a fillibuster on the next nominee. I see my Senator, Hillary ,is straddling the lefty fence today . last week she rode the center rails when she voiced opposition to the IFC at the WTC site . You wouldn't think her legs were that long .

labman rated this answer Excellent or Above Average Answer

Question/Answer
excon asked on 09/28/05 - DeLay Indicted but Frist was First


Hello wrongwingers:

The Republicans is toast. You got a "Burning Bush", "Dr. Frist the First", and "I'm Delighted He's Indicted", Delay.

Now, If da Dems can only figure out what they're about, I wager they'll retake both houses of congress plus the White House.

excon

tomder55 answered on 09/29/05:

me thinks a can of worms has been opened that the Democrats really did not want opened . Everyone in Congress will be looking over their shoulders . They already are doctoring their travel records ,and now they have to make sure that their campaign contribution records are pristine . I do have to confess that I do not understand all the various state and federal campaign finance laws .The more they tinker with them ;the more they become counter -productive . In NJ they have a millionaire vs. a billionaire running for Govenor. So much for finance reforms making elections more accessible to the comman man. Every time I read provisions of campaign finance reform I scratch my head and wonder how it can possibly be constitutional .


I also think that Ronnie Earle ;who does not have clean hands in this; will regret his Inspector Javert obsession at getting Tom Delay. (you know and I know that not all indictments are clean). Probably he will be exonerated, Ronnie Earl's record is of indicting big name pols like Kay Bailey Hutchison and Jim Mattox;only to see to the cases fall apart .


Delay on the other hand appears to be paying the price for daring to gerrymander in Texas . You should see my congressional district . It looks like a cobra as it snakes through democrat neighborhoods.

I am happy Delay stepped down. He is obviouly feeling the heat and has lost touch when he says sh*t like the fat has been cut from the budget. It is a pity that the Republican's who control the ethics committee has not acted against him instead of trying to change the ethic committee rules .There may have been a time when Delay went to Congress to be a reformer .But he got caught up in all the corruption that evidently is inherent in the position .

My only hope is that it doesn't end with him. The slime ozzes out of most of Congress's orifices .More likely however this will be simular to 1987 when House Speaker Jim Wright and House Ways and Means Committee chair Dan Rostenkowski went down and the Dems. had to weather all the corruption charges. Just like then ;both parties swim in the swamp.

excon rated this answer Excellent or Above Average Answer

Question/Answer
drgade asked on 09/28/05 - Ground Zero
Ground Zero

Just got the word from "Take Back the Memorial" that Gov. Pataki has announced the removal of the International Freedom Center (IFC) from Ground Zero.

This the group that had taken control of the 9/11 Memorial and was trying to make it into a "peace" symbol where all would know that "America was at fault and caused the attack".

To keep in touch with the progress now, make this a Favorite: www.takebackthememorial.org

A question? Well, isn't this good news?

tomder55 answered on 09/29/05:

received this email this morning :

Dear Tom,

We are very pleased to announce that Governor Pataki has announced the removal of the International Freedom Center (IFC) from Ground Zero. See http://www.washingtonpost.com/wp-dyn/content/article/2005/09/28/AR2005092801849.html for details.

Every since June 8, 2005 when Debra Bulingame's op-ed, The Great Ground Zero Heist, appeared in the Wall Street Journal, we have fought together for the preservation of the dignity of Ground Zero. With your help, we have achieved a major victory toward that goal.

We will continue to monitor the plans for Ground Zero to ensure that a fitting and proper memorial is built; one that is respectful of the victims murdered that day, their families, the first responders, and the American people.

A press release on the removal of the IFC from the 15 family member groups is expected in the next 24 hours and we will post it @ www.takebackthememorial.org as soon as it becomes available.

Thank you again for your support, prayers, and dedication. We simply could not have done this without you.

Sincerely,

Robert D. Shurbet
Founder/Web Master
TakeBackTheMemorial.org

.................................
ground roots action sometimes works . I am thrilled . Now if only they could get rid of that rediculous Freedom tower and just make the site a memorial park ....guess I'm asking too much in the heart of the NY financial district .

drgade rated this answer Excellent or Above Average Answer

Question/Answer
Choux asked on 09/27/05 - Ten Years Ago Today

It has been ten years ago today since Johnny Cochran said the infamous words, "If it doesn't fit, you must acquit."

Time does race by, doesn't it?

Comments?

tomder55 answered on 09/28/05:

yes time does "race " by . I hear OJ is still searching every sandtrap in Fla. looking for his wife's killer. Has he looked in the mirror ?

Choux rated this answer Excellent or Above Average Answer
labman rated this answer Excellent or Above Average Answer

Question/Answer
Itsdb asked on 09/27/05 - Are they forgetting something?

I read another "bring our troops home now" opinion today and it made me wonder, are these people forgetting something...like the Iraqis? On the one hand they cite the number of Iraqi casualties (usually over 100,000) while on the other they scream "bring our troops home now."

Which do they care about? How do they reconcile the two positions? How can they care about Iraqis dying while demanding we leave them to the slaughter?

Steve

tomder55 answered on 09/27/05:

screwed up the forst one


Christopher Hitchens has the answer of who they are .

I think even though the fickle polling #s show the majority of Americans oppose the way the war is being conducted ,about 2/3 oppose 'bringing the troops home now' . There I did it ;I broke one of my cardinal rules about not citing polling data . damn !

Choux rated this answer Excellent or Above Average Answer
excon rated this answer Excellent or Above Average Answer
Itsdb rated this answer Excellent or Above Average Answer

Question/Answer
Itsdb asked on 09/27/05 - Are they forgetting something?

I read another "bring our troops home now" opinion today and it made me wonder, are these people forgetting something...like the Iraqis? On the one hand they cite the number of Iraqi casualties (usually over 100,000) while on the other they scream "bring our troops home now."

Which do they care about? How do they reconcile the two positions? How can they care about Iraqis dying while demanding we leave them to the slaughter?

Steve

tomder55 answered on 09/27/05:

Christopher Hitchens has the answer of who they are .

I think even though the fickle polling #s show the majority of Americans oppose the way the war is being conducted ,about 2/3 oppose 'bringing the troops home now' . There I did it ;I broke one of my cardinal rules about not citing polling data . damn !

Itsdb rated this answer Excellent or Above Average Answer

Question/Answer
Choux asked on 09/24/05 - Get Out of Iraq TV Ad Campaign

Anti-war and get out of Iraq peace movements have gone to national television media. At the march today in Washington DC, people were calling for the impeachment of Bush. Welcome to the Sixties, dudes.

Comments..

tomder55 answered on 09/25/05:

I watched some of that impressive rally on C Span . Had they recruited a couple of frizzbee players and the people playing a pick- up soft ball game they could've doubled the attendance. geeze... Washington gets more Cub Scouts on tour on a typical day. During the 60s there were hundreds of thousands jamming into the national mall.

But for all the hippy geezers who want to reminisce I offer you this little bit of nostalgia :

Though your brother's bound and gagged
And they've chained him to a chair
Won't you please come to Chicago
Just to sing

In a land that's known as freedom
How can such a thing be fair
Won't you please come to Chicago
For the help we can bring

We can change the world -
Rearrange the world
It's dying - to get better

Politicians sit yourself down,
There's nothing for you here
Won't you please come to Chicago
For a ride

Don't ask Jack to help you
Cause he'll turn the other ear
Won't you please come to Chicago
Or else join the other side

We can change the world -
Rearrange the world

It's dying - if you believe in justice
It's dying - and if you believe in freedom
It's dying - let a man live it's own life
It's dying - rules and regulations, who needs them
Open up the door

Somehow people must be free
I hope the day comes soon
Won't you please come to Chicago
Show your face

From the bottom to the ocean
To the mountains of the moon
Won't you please come to Chicago
No one else can take your place

We can change the world -
Rearrange the world

It's dying - if you believe in justice
It's dying - and if you believe in freedom
It's dying - let a man live it's own life
It's dying - rules and regulations, who needs them

Open up the door
We can change the world

Graham Nash and CSN







Choux rated this answer Excellent or Above Average Answer
voiceguy2000 rated this answer Excellent or Above Average Answer

Question/Answer
Itsdb asked on 09/24/05 - Who wants to be president?

Katrina:

"Last Thursday I was in south Florida and sat outside while the eye of Hurricane Katrina passed over my head. It was only a Category 1 then but it was pretty nasty. Eleven people died and, as of today, there were still homes without power. That night the weatherman said this storm was on its way to New Orleans. That was Thursday! Did anybody tell you? I know you didn't want to interrupt your vacation and I know how you don't like to get bad news. Plus, you had fundraisers to go to and mothers of dead soldiers to ignore and smear. You sure showed her!

I especially like how, the day after the hurricane, instead of flying to Louisiana, you flew to San Diego to party with your business peeps. Don't let people criticize you for this -- after all, the hurricane was over and what the heck could you do, put your finger in the dike?"

Rita:

"Before leaving Washington, Bush bristled when asked if his original plans for San Antonio were made merely so he could be photographed there, and whether his presence might hinder rescue operations. He said he planned an unobtrusive visit to check on preparations.

"We will make sure that my entourage does not get in the way of people doing their job," Bush said."

tomder55 answered on 09/25/05:

that was NBC White House correspondent David Gregory .. He was Stuck on stupid at the time .Bush eventually decided not to go ;but had he gone ;he would've been sitting secure and out of the way ,monitoring events from the Northern Command.

from the transcripts :

Bush ......... What I am going to do is observe the relationship between the state and local government, particularly out in Colorado Springs. That's what I want to see. See, NorthCom is the main entity that interfaces that, uses federal assets, federal troops to interface with federal and state government. I want to watch that relationship. It's an important relationship and I need to understand how it works better.

I saw the exchange on one of the cable newscasts . I thought Bush almost broke into a laughing fit over the rediculous tone of the questioning .



Itsdb rated this answer Excellent or Above Average Answer

Question/Answer
Choux asked on 09/23/05 - Saudis "Iraq Falling Apart"

Cut and Paste

"Iraq is moving toward disintegration, and war there could spread to its neighbors, Saudi Foreign Minister Saud al-Faisal said Thursday.


In part because of a new constitution that would give more power to various regions in Iraq, "there seems to be no dynamic that is pulling the country together," Saud said. Iraqis are to vote on the constitution next month. Sunni Arab leaders are urging a "no" vote, while majority Shiites urge approval.

"All the dynamics there are pushing people away from each other," said Saud, whose nation is predominantly Sunni.

The main problem, Saud told a small group of reporters here, is the split between Sunnis and Shiites in central and southern Iraq. Continued autonomy for non-Arab Kurds in northern Iraq is less of a concern, he said.

"If things go the way they are ... there will be a struggle among the three for natural resources," Saud said, and Iraq's neighbors will be drawn into a wider war.

He said
Iran, a predominantly Shiite but non-Arab nation, would intervene on the side of Iraqi Shiites. Turkey, which has a big Kurdish minority, has repeatedly threatened to enter northern Iraq if Kurds there declare independence. If Iraq's Sunni Arab minority appears to lose out, "I don't see how the Arab countries will be left out of the conflict in one way or another."

The State Department had no comment on Saud's remarks.

Saud, who met later with Secretary of State
Condoleezza Rice, faulted the Bush administration for adding to sectarian tensions by treating all members of
Saddam Hussein's mainly Sunni Baath Party as "criminals" after ousting Saddam. He urged the United States to work harder to persuade Shiites to reach out to Sunni Arabs to assure them of their safety and equality and of Iraq's territorial integrity.

Although Saudi Arabia provided limited help to the United States in the initial phases of the war, Saud had recommended a coup to oust Saddam - not the dismantling of the Iraqi government. "It's no secret that Saudi Arabia does not believe military action in Iraq will achieve the objective it is aimed at," he said in a March 2002 interview with USA TODAY."



What say you?

tomder55 answered on 09/24/05:

in case Saud al-Faisal hadn't noticed ;we want the dynamics happening in Iraq to spread throughout the region . duh ...has he not heard anything the President has said in the last 4 years ? Elections in Lebanon;Egypt having it's first contested election ;even in Saudia Arabia they have had to give concessions to democratic forces.

What he is really saying is that the oppressed majorities in Iraq are finally getting a say in how political affairs in Iraq evolve.

I would remind Saud that Iraqi tanks were once a couple of hours drive from Rijhad with only the 82nd Airborn in their way .

Choux rated this answer Excellent or Above Average Answer

Question/Answer
Choux asked on 09/23/05 - Bush's Nemesis Alcohol

The National Inquirer reports are that Laura Bush caught President Bush drinking again. He has been dry since he was forty years old, a self-confessed alcoholic. Under promise of anonymity, a family member fears that Bush is falling apart.

tomder55 answered on 09/24/05:

too bad we don't have libel laws. did they put that article next to one about Britany Spears alien spawn ?

Choux rated this answer Excellent or Above Average Answer
powderpuff rated this answer Excellent or Above Average Answer

Question/Answer
excon asked on 09/23/05 - Iraq


Hello war dudes:

Do you think the Iraqis are better off since we invaded? Forget, for a moment, that you desperately want them to be (so do I), but are they? Is their future better now than it was under Saddam?

Me? I think they were screwed under Saddam, but I think they're more screwed now.

You can change your mind, you know. Even O'Reilly is starting to criticize Bush on Iraq.

excon

tomder55 answered on 09/23/05:

lets see . before the war .... Iraq was ruled by auniversally recognized Satalinist dictator . The prospect was there for living under his rule for decades to come and after him under his heir's who's behavior if anything was even crueler and more sadistic than the old man .

lets see . before the war Saddam had led his country in wars of aggression twice in a decade with loss of life amounting in the millions . He brutally put down both Kurdish and Shite revolts against his rule . He used chemical and biological weapons against enemies both foreign ad domestic .Human Rights Watch estimates that Saddam's 1987-1988 campaign of terror against the Kurds killed at least 50,000 and possibly as many as 100,000 Kurds .He displaced Kurds living in and near Mosul; 100,000 Kurds, Assyrians and Turkomans were expelled from Kirkuk and surrounding districts .Between 4,500 and 5,000 Kurdish villages in northern Iraq were destroyed and the population placed in "resettlement camps."

He drained the marshes in Southern Iraq because he thought they were rebel sanctuaries destroying the lives and livelyhoods of thousands of marsh Arabs .Refugees International reported that the "Oppressive government policies have led to the internal displacement of 900,000 Iraqis"

In 2001 Amnesty International reported that Iraq under Saddam had the worse record for number of persons who were reporteded disappearances and unaccounted for ;over 1600 according to the UN ;overall an estimated 300,000 Iraqi citizens vanished without a trace . In a 10 year period between 1992 and 2002 he refused to allow UN human rights monitors into the country . Former UN Human Rights Special Rapporteur Max Van der Stoels report in April 1998 stated that Iraq had executed at least 1,500 people during the previous year for political reasons.
Tens of thousands of political opponents and ordinary citizens were subjected to arbitrary arrest and imprisonment, summary execution, and torture by beating and burning, electric shock, starvation, mutilation, and rape. Wives were tortured in front of their husbands, children in the presence of their parents. Saddam Hussein's regime carried out frequent summary executions, including: 4,000 prisoners at Abu Ghraib prison in 1984 3,000 prisoners at the Mahjar prison from 1993-1998 2,500 prisoners were executed between 1997-1999 in a "prison cleansing campaign" 122 political prisoners were executed at Abu Ghraib prison in February/March 2000 23 political prisoners were executed at Abu Ghraib prison in October 2001 At least 130 Iraqi women were beheaded between June 2000 and April 2001 .1,600 death row prisoners were transferred in 1995 to a special unit for chemical and biological weapon experiments.

Under the oil-for-food program, the international community sought to make available to the Iraqi people adequate supplies of food and medicine, but the regime blocked sufficient access for international workers to ensure proper distribution of these supplies. Saddam blamed the suffering of Iraq's people on the U.N., even as he uses his oil wealth to build lavish palaces for himself, and buy arms for his country.There were widespread reports that food and medicine that could have been made available to the general public was stockpiled in warehouses or diverted for the personal use of some government officials. He plundered and helped facilitate the plunder of money recieved under the oil for food program . An estimated 400,000 Iraqi children under the age of five died of malnutrition and disease in the 5 year period before the war .

there is plenty more information available ;including this and this

need I say more ? oh ;ok I admit it . The Baathists who ruled Iraq under Saddam are having a rougher go of it .


excon rated this answer Excellent or Above Average Answer

Question/Answer
sapphire630 asked on 09/22/05 - latest news...global warming..........


.......................................................on MARS!!!!

I guess 'em aliens are making the same stupid mistakes we are!

tomder55 answered on 09/23/05:

the story goes that they (we ) screwed up Mars first and had to come to Earth to escape .

sapphire630 rated this answer Excellent or Above Average Answer

Question/Answer
sapphire630 asked on 09/22/05 - great combo for next election

I think Howard Dean would be great for Pres. as long as Tom Cruise can be his vice Pres.
Imagine it! They can go on Leno and Tom can jump on the furniture while dean does his famous scream....maybe it will take the stress out of politics and we can live happily ever after....and who would need prozac ever again!!?

tomder55 answered on 09/23/05:

I wonder how Deam would fare in a Cruise run Scientology conversion tent ?

sapphire630 rated this answer Excellent or Above Average Answer

Question/Answer
kindj asked on 09/22/05 - Things I have learned:

Things I have learned from watching the news on TV during the last few weeks:

The hurricane only hit black families' property.

New Orleans was devastated and no other city was affected by the hurricane.

Mississippi is reported to have a tree blown down.

New Orleans has no white people.

The hurricane blew a limb off a tree in the yard of an Alabama resident.

When you are hungry after a hurricane, steal a big screen TV.

The hurricane did 23 billion dollars in improvements to New Orleans: now the city is welfare, looters and gang free and they are in your city.

White folks don't make good news stories.

Don't give thanks to the thousands that came to help rescue you, instead, bitch because the government hasn't given you a debit card yet.

Only black family members got separated in the hurricane rescue efforts.

Ignore warnings to evacuate and the white folks will come get you and give you money for being stupid.

tomder55 answered on 09/23/05:

Bishop Chuck is right . Screwy Lois Louis Farrakhan is preaching that the levee was blown up . But then again 'minister ' Farrakhan once said that there's a mother wheel hovering above the earth and that he was transported to the mother wheel, where he had a discussion with Elijah Mohammed.

The economic situation in New Orleans comes 70 years after President Roosevelt converted the federal government into a welfare provider for the poor and needy. The primary purpose of the federal government became to relieve poverty through confiscation of wealth from the rich in order to redistribute the money to the poor. Thirty years after the New Deal came Lyndon Johnsons war on poverty all across America. The federal government became a massive tax-and-spend engine to help the poor. New Orleans is the predictable result. Tens of thousands of people lacking sufficient money to enable them to escape ,dependent on the federal government ,desperately waiting for federal officials to deliver food and water to them and to pick them up and deliver them to government run refugee centers ; A fitting legacy of modern day liberalism.

kindj rated this answer Excellent or Above Average Answer
powderpuff rated this answer Excellent or Above Average Answer

Question/Answer
Itsdb asked on 09/21/05 - Bill for first lady

Hoping to generate 1 million emails urging Hillary to run for president, a group of her supporters jave launched the Bill-for-First-Lady.com website. There, you can watch two videos, "Boxers, Briefs or Thong" and "Men's Room," buy a Mr. First Lady Talking Doll, or buy some "Republicans Suck T-Shirt Hall of Fame" merchandise by politicalsmartass.com.

My favorites, "Mommy says Republican is another word for f**ker," "Hello, my name is Mr. sh*tferbrains" and "Karl Rove killed my kittens." You too can "laugh at Republicans...not with 'em" with your politicalsmartass.com gear.

Or you can just "time travel back to the good 'ol days."

Alrighty then. Is this what politics has come to?

Steve

tomder55 answered on 09/21/05:

see where a chinese firm is going to market Clinton Condoms ?

The Clinton legacy ....the gift that keeps on giving !

Itsdb rated this answer Excellent or Above Average Answer

Question/Answer
Itsdb asked on 09/21/05 - Howard Dean Saved the Democratic Party?

Tuesday, Sept. 20, 2005 10:20 a.m. EDT

DNC Chairman Howard Dean is now boasting that he's the savior of the Democratic Party, in a none-too-subtle slap at former party chief Terry McAuliffe, not to mention the last Democratic standard bearer, Sen. John Kerry.

Asked why he wanted to run the DNC, Dean told ABC's "The View" last week: "Somebody had to save the party."

He insisted that Democrats were heading in the wrong direction before he took over, telling "View" gabber Joy Behar, "We thought we were going to win by becoming Republicans."

The ex-Vermont governor suggested that Sen. Kerry didn't have the backbone to defeat President Bush in last year's election, saying, "If you want to win, it's not so much what you believe ... it's whether you're willing to fight for what you believe. And the Democrats had given up. We had simply not been willing to stand up and fight."

Dean's bizarre attack on his fellow Democrats went unnoticed by the mainstream press. But talk radio host Steve Malzberg told NewsMax he had a field day playing the clips while filling in on Atlanta's WGST.

After criticizing his predecessors for being too lame, Dean turned his fire on the GOP.

"The truth is, they are a white Christian party," he insisted. "They don't welcome and embrace diversity."

Dean also blasted the Bush administration for what he charged was a bid to deflect blame over Hurricane Katrina, saying, "That really was a [Karl] Rove inspired thing - to go attack the local people."

But when it came to New Orleans Mayor Ray Nagin, Dean turned defensive, saying it wasn't Nagin's fault that the city's school buses weren't used to evacuate his trapped constituents.

"The school buses were controlled by the school board, not the mayor," Dean insisted. "You can't blame the mayor for that."

~~~~~~~~~~~~~~~~~~~~~~~~~~~~~~~~~~~~~~~~~~~~~~~~~~~~~~~

Yeah, but can he save himself from his mouth?

Steve

tomder55 answered on 09/21/05:

I think he perfectly represents the modern democrat party . Don't believe me ?;ask Zell Miller . I think he should talk and be as outspoken as he is as often as he likes . This blends perfectly into my theory that the Clinton's are having second thoughts about their move to the center strategy .

Itsdb rated this answer Excellent or Above Average Answer

Question/Answer
excon asked on 09/21/05 - Pardon my while I gag!


Hello fiscal righties:

Whats this BS from Tom Delay? No fat in the budget??? He sounds absolutely, positively, liberal. How can you stomach that crap?

excon

tomder55 answered on 09/21/05:

I can't . I only hope he was talking tongue in cheek sarcasm .By that I mean he is challenging others in both parties to come up with things to cut from the budget .But; he probably meant it .Fiscal discipline is sorely missing from both parties in Washington:

Stevie Wonder could see the waste ooozing out of the federal budget . I just did a lenghty response to Bradd on the Christianity Board on this very subject so I won't repeat some of the details . There is twice the money needed for Katrina relief than anyone is proposing just by trimming the fat .

excon rated this answer Excellent or Above Average Answer

Question/Answer
ROLCAM asked on 09/21/05 - Help with Interpretation ??

"I'm not too sure if his heart is as big as his head."
HARRY REID the Senate Democratic leader, on Judge John G. Roberts Jr.

I need some help with interpreting this quotation:-

What does the Senator mean ?

1) Has John G. Roberts a big head ?
2) Has John G. Roberts a little heart?
3) Is Harry Reid being complimentary ?
4) Is Harry Reid being disrespectful ?
5) What is Harry Reid's status ?
6) Why is Harry Reid so important in the scheme of things?

rolcam.

tomder55 answered on 09/21/05:

Reid is being disrespectful;

Reid is the top ranking Democrat in the Senate . He is Senator from Nevada . He sets the marching orders for the Democrats in the Senate. He knows that Roberts will be confirmed. He wants to prove that he can muster 41 opposition votes, not to derail Roberts ;but to set the stage for the next Court nominee who he thinks will be someone who could disrupt the balance of the court .

Before the OcConner retirement and the death of Chief Justice Rhenquist the court was composed of 4 judges who generally decided with a liberal slant and 4 who decided conservative. Usually OcConner was a "swing vote ,and decisions were decided by a 5-4 majority . What Reid fears is that Bush will fill the two positions with judges who will give the conservatives a solid 5-4 majority .
Then they fear that the judical abortion decision that made it legal nationwide would be reversed. His fear is unwarrented . Roberts has said he considers the decision 'established law' .

If Reid gets 41 opposition votes then the next nominee will be fillibustered ( a delying tactic ). The Republicans will not be able to break it with out changing the Senate rules.

Itsdb rated this answer Excellent or Above Average Answer
ROLCAM rated this answer Excellent or Above Average Answer

Question/Answer
Itsdb asked on 09/19/05 - It had to happen

Since the president accepted responsibility last week for the poor Katrina response the next logical thing would be for the left to call on him to start accepting blame for every other problem in this country, beginning with Iraq. Here we go...

John Kerry sent an email inviting us to preview the speech he plans on giving at Brown University today, in which he plans on saying:

"I know the President went on national television last week and accepted responsibility for Washington's poor response to Katrina. That's admirable. And it's a first. As they say, the first step towards recovery is to get out of denial. But don't hold your breath hoping acceptance of responsibility will become a habit for this administration. On the other hand, if they are up to another "accountability moment" they ought to start by admitting one or two of the countless mistakes in conceiving, "selling", planning and executing their war of choice in Iraq."

I say just like the sharks they are, the left smells blood in the water and they're going in for the kill...just as some of us predicted would happen if Bush ever admitted to a mistake. And you?

Steve

tomder55 answered on 09/20/05:

that is why I argued recently on the Expert Forum that it would be a mistake for the President to ever "admit mistakes" .At least he didn't say because I could.

I will still debate anyone who thinks the war was a conceptual mistake. I will not defend all of the execution because I thought they drifted from the correct course when they delayed the handover to Iraqi control in favor of the Viceroyship of Bremer.

doesn't Kerry know yet that he is irrelevent ? Cindy Sheehad has more of an impact on the party for that matter ;so does Michael Moore.

Notice that after Bush attempted to rehab Bill Clinton's legacy again Clinton couldn't wait to get on national t.v. to blast Bush . What is hapening is that the Clinton's are realizing that Hillary's drift to the middle is not resonating with the increasingly radicalized base of the Democrats. Anyone who comes out in favor of the war will not win in the primaries . Hillary needs to get away from her stay the course strategy . Bill is just clearing the way for her to re:reinvent herself.

I can hear Kerry now :

"I was for the war before I was against the war; before she was for the war ,before she was against the war. "

Itsdb rated this answer Excellent or Above Average Answer

Question/Answer
Itsdb asked on 09/19/05 - Right to privacy?

Legalized abortion is justified as a right to privacy issue, so what gives anyone the right to give, sell or publish my personal information without my consent?

Steve

tomder55 answered on 09/19/05:

It seems that the right to privacy protecting personal information is not considered as important as the right of privacy of personal autonomy .There are "financial " privacy laws ;but they do not sufficiently protect consumers.The FTC did a study in 2000 that found most sites falling outside of the jurisdiction of the right of privacy laws do not adequately inform consumers about collection practices, nor do the majority of sites adequately protect the privacy of visitors personal information. The penumbras get all the attention. It was a huge leap to go from 'Griswold v. Connecticut' ;and the right to use condoms to 'Roe v Wade' ;and the right to kill your baby .Now it is 'established law' according to our future Chief Justice . I wonder if he thought 'Bowers v. Hardwick' was established law before the court overturned it in 'Lawrence v. Texas' ?

Itsdb rated this answer Excellent or Above Average Answer

Question/Answer
Choux asked on 09/17/05 - Ten President Bushes

I would rather have ten President Bushes than one leader like Islam produces.

Saudi King
General Mushariff
Osama bin Laden
Hosni Mubarak
Colonel Kadafi
Mohamed Atta
Cleric who heads Iran Govt(forget his name)
Etc.....

Comments?

tomder55 answered on 09/19/05:

that stupid cowboy pulled another one out of his hat yesterday . Imagine that ;another residual outcome of the Bush doctrine !

Choux rated this answer Excellent or Above Average Answer

Question/Answer
paraclete asked on 09/18/05 - what do you think of this quote

"I detest war and the meatheads who volunteer to kill other human beings. The US alliance is a funnel that draws us into unnecessary wars; first Vietnam and then Iraq."

tomder55 answered on 09/19/05:

Aussies were wise to reject Latham . Now he publishes a sour grapes vitriolic diary ? So what else is new .I hear he even attacked Gough Whitlam in the book.

I thank God for the strong leadership of peole like John Howard .

paraclete rated this answer Excellent or Above Average Answer
sissypants rated this answer Excellent or Above Average Answer

Question/Answer
paraclete asked on 09/18/05 - what do you think of this quote

"I detest war and the meatheads who volunteer to kill other human beings. The US alliance is a funnel that draws us into unnecessary wars; first Vietnam and then Iraq."

tomder55 answered on 09/19/05:

Aussies were wise to reject Latham . Now he publishes a sour grapes vitriolic diary ? So what else is new .I hear he even attacked Gough Whitlam in the book.

I thank God for the strong leadership of peole like John Howard .

paraclete rated this answer Average Answer
sissypants rated this answer Excellent or Above Average Answer

Question/Answer
Choux asked on 09/16/05 - Talk is Cheap

On Thursday, September 8, The Hill quoted Senator Bill Frist saying Americans deserve answers. We must do all we can to learn from this tragedy, improve the system and protect all of our citizens.

On Friday, September 9, Senator Rick Santorum issued a press release saying It will be important to study the response to this disaster and to better understand what went right and what went wrong.

On Tuesday, September 13, The Washington Post reported that 76 percent of the public favors an investigation of federal storm response efforts by an independent commission similar to the one that probed the Sept. 11, 2001, terrorist attacks, and that the proposed commission drew strong bipartisan support: 64 percent of all Republicans and 83 percent of Democrats favored creating the independent panel."

On Wednesday, both Senators Frist and Santorum voted to block the creation of and independent, bipartisan panel modeled on the 9/11 Commission to investigate the federal, state and local governments response to Katrina.


In a recent poll, Santorum is behind in the polls(14%) in his attempt to get reelected Senator from Pennsylvania in 2006.

Comments?

tomder55 answered on 09/17/05:

It is the duty of Congress to have oversight . Let them get about their business and stop these showboat calls for 'independent commissions ' . My God how they find ways to shirk their duty ! Bush of arm chair quarterbacks ! When the 9-11 commission recommended an 'independent ' oversight to congressional compliance to the commissions recomendations did congress add it to the legislation ? hell no ! . They bitch and moan about executive overreach and have done nothing creative to help in the war on terror. They don't like GITMO .What is their alternative ? nothing .

Choux rated this answer Average Answer

Question/Answer
CeeBee2 asked on 09/15/05 - Posse Comitatus....................

tomder55 had responded to my leader question by saying:

...Imagine the perception of a Southern State being occupied by union troops again without State approval ? The law is clear on this . Bush would've had to invoke the Insurrection Act and the Posse Comitatus Act of 1878. If he makes the decision to go in without local acceptance, he's guilty of violating states rights...

I say to tomder55, A Southern state wouldn't have been "occupied."

The Posse Comitatus Act of 1878 says "we can't arrest you." It doesn't say "sorry, we can't save you." (quote on Nightline, 09/15/05)

The Posse Comitatus Act of 1878

20 Stat. L., 145
June 18, 1878

CHAP. 263 - An act making appropriations for the support of the Army for the fiscal year ending June thirtieth, eighteen hundred and seventy-nine, and for other purposes.

SEC. 15. From and after the passage of this act it shall not be lawful to employ any part of the Army of the United States, as a posse comitatus, or otherwise, for the purpose of executing the laws, except in such cases and under such circumstances as such employment of said force may be expressly authorized by the Constitution or by act of Congress; and no money appropriated by this act shall be used to pay any of the expenses incurred in the employment of any troops in violation of this section And any person willfully violating the provisions of this section shall be deemed guilty of a misdemeanor and on conviction thereof shall be punished by fine not exceeding ten thousand dollars or imprisonment not exceeding two years or by both such fine and imprisonment.

10 U.S.C. (United States Code) 375

Sec. 375. Restriction on direct participation by military personnel:

The Secretary of Defense shall prescribe such regulations as may be necessary to ensure that any activity (including the provision of any equipment or facility or the assignment or detail of any personnel) under this chapter does not include or permit direct participation by a member of the Army, Navy, Air Force, or Marine Corps in a search, seizure, arrest, or other similar activity unless participation in such activity by such member is otherwise authorized by law.

18 U.S.C. 1385

Sec. 1385. Use of Army and Air Force as posse comitatus

Whoever, except in cases and under circumstances expressly authorized by the Constitution or Act of
Congress, willfully uses any part of the Army or the Air Force as a posse comitatus or otherwise to
execute the laws
shall be fined under this title or imprisoned not more than two years, or both.

Thus, President Bush should have immediately sent aid via the military into the Southern states ravaged by Katrina.

tomder55 answered on 09/16/05:

The question of what the real meaning of the Posse Comitatus Act was has been the subject of some dispute ever since its passage. Rigid interpretations of the law slowed down deployment of active-duty troops after the storm. Others say there are exceptions to the law, and it can be waived by the president.But many legal scholars worry that minor exceptions to the PCA can quickly expand to become major exceptions .

Gen. Peter Pace, designated as the next chairman of the military's Joint Chiefs of Staff, on Thursday called for re-examination of the law;so has Senator John Warner.

Clearly the law needs to be revisited .Congress (the silent voice in the war on terror) should probably repeal it outright and then define the terms that the military should be used in domestic operations .It probably should've been redone after the Waco fiasco ;but certainly in light of both homeland security measures and the Katrina response ,Congress should take immediate steps to bring the law into the 21st century.


see :
http://www.freep.com/news/nw/kask16e_20050916.htm

CeeBee2 rated this answer Excellent or Above Average Answer

Question/Answer
Choux asked on 09/14/05 - Wake up Call

LA Times staff writers:

WASHINGTON "It was conceived as the solution to confusion and bureaucratic logjams that hampered responses to the Sept. 11 terrorist attacks a 426-page master plan to coordinate government agencies in a disaster.

When it was unveiled amid fanfare in January, the Department of Homeland Security's National Response Plan promised "vastly improved coordination among federal, state, local and tribal organizations to help save lives" from storms, floods, earthquakes or terrorist assaults.


Hurricane Katrina turned out to be its first real-world test but the plan broke down soon after the monster winds blew in.

Its failures raise unsettling questions about the federal government's readiness to deal with future crippling disasters. An examination of how the plan was administered during the crucial early hours of this natural disaster reveal more confusion than coordination and repeated failures of leadership.

The plan on paper was not always apparent on the ground. Cooperation among government agencies faltered at almost every level, right up to the White House.

For example:

The Federal Emergency Management Agency, responsible for supervising relief and rescue operations, failed to position adequate equipment to carry out the dual assignments. FEMA was especially short of helicopters from the outset. It was forced to concentrate on rescue missions and gave short shrift to ferrying supplies to trapped evacuees.

Coordination with private relief agencies broke down and led to maddening delays. Water, food, clothing and medical supplies backed up in distant warehouses.

More than 50 civilian aircraft responding to separate requests for evacuations from hospitals and other agencies swarmed to the area a day after Katrina hit, but FEMA blocked their efforts. Aircraft operators complained that FEMA waved off a number of evacuation attempts, saying the rescuers were not authorized. "Many planes and helicopters simply sat idle," said Thomas Judge, president of the Assn. of Air Medical Services.

Military cooperation was stymied. In advance of the storm, New Mexico Gov. Bill Richardson offered the governor of Louisiana hundreds of National Guard troops. They were poised to fly into Louisiana on Monday, Aug. 29, just as the levees were about to give way. Instead, red tape and paperwork at National Guard headquarters in Washington delayed their arrival until Friday. Deployment orders had not been not properly filled out, the New Mexico National Guard was told.

Telephones and radios failed everywhere, complicating efforts to monitor field conditions and coordinate response. FEMA officials were caught by surprise. Better communications was supposed to be a highlight of the plan, but it took up to six days to get working telephones to some FEMA employees on the ground.

In the face of rising criticism, FEMA officials pointed to bright spots. "There's the perception that we didn't do anything. But we had a life-saving mission, which we met, and we had a life-sustaining mission, which we met," said Marty Bahamonde, who helped coordinate a FEMA emergency response team.

Before the Storm

In the calm before the storm, preparations got off to a promising start. Homeland Security Secretary Michael Chertoff convened interagency meetings, created an operations center in Baton Rouge, La., and dispatched FEMA Director Michael D. Brown as his representative on the ground.

Food, water, blankets and personnel were pre-positioned on the fringes of the expected severe-impact zone.

President Bush activated the National Response Plan on Saturday, Aug. 27, two days before the hurricane struck, when he declared a federal emergency in Louisiana. Under the plan, this made the Department of Homeland Security "responsible for coordinating federal resources utilized in response to major disasters."

Then, on Monday, 140-mph winds slammed into New Orleans, a storm so fierce that no amount of planning was likely to prevent flooding, deaths and substantial destruction.

That day, Bush declared the region a federal disaster area, releasing more federal funds and resources.

And on Tuesday, more than 24 hours after surging waters breeched levees in New Orleans, Chertoff declared Katrina the nation's first "incident of national significance" as outlined in the response plan. This committed the federal government to a major and long-term relief effort.

Survivors were already waving for help from rooftops and increasingly restless residents displaced without food or water were demanding help outside the Superdome, where they had sought safety before Katrina struck. As the emergency response floundered on television screens around the world, some White House aides suggested state and local officials were to blame. By then, however, it had become a federal problem."


We are "lucky" that we had this wake-up call. What can we do going forward to improve response to national disasters, both natural and whatever???

tomder55 answered on 09/15/05:

agreed ,we have a long way to go ,and I do not think that FEMA should be the point agency . There also has to be seperate response plans for natural and terror or attacks ;they are not the same situations . The UStaxpayers have spent a fortune in the last 4 years to fund local responders . I still contend that the forst line of defense has to be at the local level . There were 4 States involved in the Katrina attack and 3 of them were prepared .

Choux rated this answer Excellent or Above Average Answer
kindj rated this answer Excellent or Above Average Answer

Question/Answer
CeeBee2 asked on 09/14/05 - What is a leader? Should a leader always obey the

rules?

This was in this morning's paper. The bold sections were my addition:

A way out of the chaos
3 heroes bypass the brutal banality of getting bogged down by bureaucracy

Kathleen Parker, Chicago Tribune Media Services
Published September 14, 2005

Katrina's detritus will be months in the sifting, but what best reveals what went wrong may be found in the contrast between bureaucrats ensnared in red tape and three individuals who sprang into action as circumstances required.

Their names are Deamonte Love, Jabbar Gibson and Sheriff Warren C. Evans.

Deamonte Love is probably the most familiar. He is the 6-year-old who led a troupe of children to safety after rescuers separated them from their parents. Deamonte was the oldest of the group, which included his 5-month-old brother, three toddlers in the 2-year-old range, a 3-year-old and her 14-month-old brother.

All held hands as Deamonte led the group along Causeway Boulevard in New Orleans, where he identified himself and his associates to authorities. In a sea of helpless victims, while heartier adults dithered or complained, Deamonte found the guts and fortitude to take care of himself, his family and friends.

Another victim of the storm, Gibson is perhaps better known as the 20-year-old who commandeered a school bus and drove 70 homeless people from New Orleans to the Houston Astrodome, beating the other 25,000 or so awaiting evacuation from the Superdome by officials still trying to figure out who was in charge.

When no one is in charge, as seems to have been the case for too long in New Orleans, a leader eschews the clipboard and takes action. While city officials couldn't find their way to use hundreds of available school buses to evacuate some 100,000 residents without transportation, Gibson "stole" a bus and rescued 70 strangers.

A photo of the abandoned and eventually submerged school buses has become an iconographic image in Hurricane Katrina's record--a kaleidoscopic history that would qualify as comedy if the results had not been so tragic. At times like this, bureaucracy isn't just a frustrating boondoggle; it is a faceless accomplice to negligent homicide.

"No one is to blame because, sir, we were just following the rules."

Not Warren C. Evans. The sheriff of Wayne County, Mich., which includes Detroit, ignored his own governor's pleas to wait for "formal requests" and put his leadership instincts to better use. While other law enforcement volunteers were held up for two to three days dealing with paperwork, Evans led a convoy of six tractor-trailers, three rental trucks and 33 deputies to Louisiana.

Explaining his pre-emptive action to the New York Times, Evans said: "I could look at CNN and see people dying, and I couldn't in good conscience wait for a coordinated response."

Meanwhile, other more obedient citizens and potential rescuers, as well as evacuation vehicles, medical and food supplies, even a floating hospital, were stalled or unused as officials and politicians bickered over territory and protocol and--in an indictment that speaks for itself--gender sensitivity concerns.

I wish I were kidding. Hundreds of firefighters who volunteered to help with the Katrina relief were held up for days in Atlanta while they took classes on sexual harassment and community relations, all courtesy of FEMA, the Federal Emergency Management Agency in charge of coordinating federal relief. At the White House, concerns about overriding the female governor of Louisiana reportedly contributed to the decision not to take control of a national disaster that clearly had overwhelmed state and local officials.

tomder55 answered on 09/14/05:

Unfortunately the concerns were real. Imagine the perception of a Southern State being occupied by union troops again without State approval ? The law is clear on this . Bush would've had to invoke the Insurrection Act and the Posse Comitatus Act of 1878. If he makes the decision to go in without local acceptance, he's guilty of violating states rights. If he decides to not to go in he is guilty of hating blacks.

Rumsfeld used an executive order to put Adm. Timothy J. Keating in charge of operations concerning Katrina, before it ever became a hurricane. Keating then approved the use of bases in Meridien, Miss., and Barksdale, La., to position emergency meals and medical equipment a week before Katrina ever hit the Gulf coast. They also placed ships in the Gulf.All that was required was for the Govenor to initiate the repponse .She did not . The governors of Alabama and Mississippi had no problem relinquishing control of their recovery efforts to the federal government but evidently she balked at giving up control of the National Guard as would have been required by the Insurrection Act .

My opinion about pc has not changed . I'm sure there is some mandated protocol for teaching gender sensitivity to first responders .

CeeBee2 rated this answer Excellent or Above Average Answer

Question/Answer
Itsdb asked on 09/13/05 - I'm baaack!

Like it or not :) Guess what, there is still another world out there besides Katrina. Those poor Californians, cutting the wrong cables in LA, dog ear cropping bill failed, Huntington Beach and Santa Cruz battling for the right to be called Surf City. But more importantly, LSU and the Saints won, and the Cowboys defeated the Chargers as a nice closer to our brief San Diego vacation.

Oh yeah, a question...Friday in USA Today Jon Saraceno said the "NFL should have delayed games in wake of Katrina."

"A city lies in watery ruins. Bloated corpses float in contaminated floodwaters. Army tanks roll down streets. Police, fire and medical personnel tend to the sick, suffering and frightened. Thousands of evacuees across our nation fear the worst for missing family and friends.

Are you ready for some football?

The games must go on as scheduled ... right?

No they don't..."

Should the NFL have delayed the games?

Steve

tomder55 answered on 09/13/05:

I do not understand why anyone thinks it right and proper to tell someone else shut down their business . We had the same situation in NYC on Sunday .They were having a memorial service at the WTC site ,and some were complaining that local businesses in the area were operating ;that tourist busses still passed the site . What happens on Memorial day in this country ? Retail sales ! that is the American way .

I'm sure the good aints fans were happy that the team was playing and thrilled with the win . I got to watch the exciting end (well almost ;they cut to the Giants game before the final field goal ).

Itsdb rated this answer Excellent or Above Average Answer

Question/Answer
jocase asked on 09/12/05 - Power Outage InLos Angeles

There was a power outage in Los Angeles, today. It lasted about an hour and a half.

Is George Bush to blame?

tomder55 answered on 09/13/05:

it was Enron

jocase rated this answer Excellent or Above Average Answer

Question/Answer
Choux asked on 09/12/05 - Opinion about Power and Bush Administration

The following is a cut and paste from Nora Ephron's blog:

"For some time I've been wondering whether anyone is going to explain the true mystery of what happened after Hurricane Katrina struck. I read thousands of words on the subject in this mornings New York Times, and I still dont get it. Where was the President? And more to the point, where was the Vice President?
And dont tell me Crawford Texas and on a ranch in Wyoming. For days there was an absolute vacuum at the top. Why? What was going on?

Youll be happy to hear that I have a theory. Is it possible that the President and the Vice President have fallen out? I mean, Im just asking. But if you remember September 11, 2001 -- and Im sure you do -- the President had no idea what to do, but the Vice President did. The Vice President took over. He didnt even consult with the President. He put the President on Air Force One and the President spent the day flying from one airport to another, which was something that even the President eventually understood made him look as if he wasnt in charge.

The relationship between Cheney and Bush has always reminded me of a moment I witnessed in the movie business many years ago. I had written a script for an actress, and she had decided she wanted to direct it. This was a terrible idea, because she was famous for dithering, but there was no question that the studio would make the movie if she directed it. Dont worry about it, the producer of the movie said to me when I asked if she was remotely capable of directing a movie. We can walk her through it.

Its always been clear to me that five years ago, when all those Republican guys got together and realized that George Bush could be elected president and that he wasnt remotely capable they came to an understanding: they would walk him through it. Im sure it seemed like a swell idea, especially because it meant that theyd be in a perfect position to convince him to do all sorts of exciting things they had always wanted to do.

Cheney was the point man. Cheney was the guy they put on Meet the Press. Cheney was the person who seemed always to be the first responder. Cheney was the official they put into the bunker last May when a plane flew too close to the White House; Bush, who was bicycling in Maryland, wasnt even told about the episode until forty minutes after it was over. Even Laura Bush, who was in the bunker with Cheney, publicly questioned the decision to keep the President in the dark.

But if you look at the chart in Sunday's New York Times, which tells you who was where when Katrina struck, Cheney doesnt even get a listing. Its Bush, Chertoff, Brown. Bush I and Bill Clinton were summoned to help. But Cheney didnt even turn up back in Washington until last week, when he was sent off for a day of spouting platitudes while touring the flood zone.

Like the curious incident of the dog that didnt bark in the famous Sherlock Holmes story, Cheneys the missing person in this event, and one has to wonder why. If he were a woman, I would guess hed been busy recovering from a face-lift, but hes not. So I can only suppose that something has gone wrong. Could the President be irritated that Cheney helped con him into Iraq? Oh, all right, probably not. Could Cheney and not just his aides -- possibly be involved in the Valerie Plame episode? Is Cheney not speaking to Karl Rove? Does the airplane/bicycle incident figure into this in any way? And how is it possible that the President is off on vacation and the Vice President is too? Not that it matters that much if the President is on vacation; on some level, the President is always on vacation. But where was Cheney?"


What do you think of the points she makes??

tomder55 answered on 09/13/05:

It is a tribute to the role that Cheney has played in this administration that anyone even noticed he was missing . Most VPs sit around waiting for the boss to expire . Ever hear some of the quotes by former VPs ?

"[The Vice Presidency] is the most insignificant office that ever the invention of man contrived or his imagination conceived." - John Adams, 1st Vice President

"The vice presidency isn't worth a pitcher of warm spit." - John Nance Garner

"If you give me a week, I might think of one." - Dwight Eisenhower, in response to a reporters question about a major policy contributed by then vice president Richard Nixon.

"A little over a week ago, I took a rather unusual step for a vice presidentI said something." - Spiro T. Agnew

"The second office of this government is honorable and easy, the first is but a splendid misery." - Thomas Jefferson



"It just is not possible in politics for a vice president to 'chart out his own course.'" - Richard Nixon



"Look at all the Vice Presidents in history. Where are they? They were about as useful as a cow's fifth teat."

- Harry S. Truman


"I would a great deal rather be anything, say professor of history, than vice president."- Theodore Roosevelt

"This is a hell of a job. I can only do two things: one is to sit up here and listen to you birds talk....The other is to look at newspapers every morning to see how the president's health is." - Charles Dawes


"If the tide of defamation and abuse shall turn and my administration come to be praised, future vice presidents who may succeed to the presidency may feel some slight encouragement to pursue an independent course." John Tyler

"I should hate to think that the Senate was as tired of me at the beginning of my service as I am of the Senate at the end."- Charles G. Dawes

"Keep your mouth shut, your head down, and don't act like you want it." - Jack Kemp


"I do not propose to be buried until I am really dead." - Daniel Webster, on not accepting the Vice Presidency

"The President has only 190 million bosses. The Vice President has 190 million and one."- Hubert Humphrey

Choux rated this answer Excellent or Above Average Answer

Question/Answer
sapphire630 asked on 09/12/05 - First Celine

NOw Cher

Celine said let the poor touch the T.V.'s maybe it is the only chance they get....
Let them touch hers and see if she still says the same

Now Cher says "we don't need Ipod's"... and video games, etc, she says she didn't need her tax break...etc...
they talk a good talk but what are they actually doing...
...Then again when they have Farm Aid, Tsumai relief, etc none of the money actually ends up helping where it was meant to
.....food is left sitting on docks rotting while bureacratic red tape never ends, money filters through the rulers of the countries and trickles down to nothing because they spend it on fountains, Rolex's and limo's for their personal use as the children continue to starve.

So Sleep well at night Cher, Celine, & all

cause I can't think of a better solution..............



tomder55 answered on 09/13/05:

actually I have nothing against celebs having something to say . I think they first should know a little about what they are talking about . Bab Streisand has views I completely disagree with but posts them on her site in a clear sober rational way .They all should take a lesson from her .

sapphire630 rated this answer Excellent or Above Average Answer

Question/Answer
sapphire630 asked on 09/12/05 - Memorial?

I heard they are making a memorial at the 9/11 crash site with a crescent of red windchimes and a crescent of trees. Then I heard it is the same symbol the terroists use. Can you find a news article about all this for me?
If so why this type of smack in the face?

tomder55 answered on 09/13/05:

Belmont Club blog went so far as to do the math calculations and determined that the crescent is oriented towards Mecca . If that is the case I doubt if the design is unintentional .

sapphire630 rated this answer Excellent or Above Average Answer

Question/Answer
Choux asked on 09/10/05 - HELP WANTED-FEMA EXECUTIVES

Cut and Paste follows

"Immediate opening. Executive Director for top public sector disaster management agency of a well-known North American superpower. No previous experience necessary but candidate should be skilled in talking in circles and capable of sitting on his/her hands for extended periods of time.
Poltical sycophancy is a plus and candidate must be willing to relocate to historical trash bin when expedient. Employer is willing to work with the right candidate to generate an appropriate resume. No references required (if youre not already on donors list your application will be discarded). Competitive salary, full benefits, short hours. Call: When you get around to it"

Comments?

tomder55 answered on 09/12/05:

It is not always a violation of the principles of good government for politicians to appoint their friends and allies to government jobs. However, when those friends and allies lack any reasonable qualifications, it certainly is. As a constituent of the Daley's I'm sure you are aware of that .

Patronage cronyism and nepotism has been a plague since the beginning of civilization . Is Bush the only President in recent years to have done so ? Not if one remembers that Bill Clinton fired all U.S. attorneys after he became President, and then appointed Paula Casey, a former campaign worker, as U.S. Attorney in Arkansas, where, by the way, the U.S. Attorney's Office happened to be investigating the Whitewater scandal.


Choux rated this answer Excellent or Above Average Answer

Question/Answer
Bradd asked on 09/10/05 - Sluggish Kitchen Sink Drain

(This isn't about politics, but plumbing doesn't appear on the home page, and if I leave it there, it will die a slow death).


Double sink in kitchen - each sink has a drain that U's into a common drain. When I run the water (either sink) it rises above the drain opening and slowly fills the bowl. Then, water off, the drainage is slow.

Over the last few days, I've tried two servings of Liquid Plummer (2 different times), and one of Drano. Neither has worked.

Thanks.

tomder55 answered on 09/11/05:

cover one hole and use a plunger on the other . repeat on the other hole. Don't use draino . there are better enzyme treatments that you should use on a regular basis whether the sink backs up or not . Probably you are getting too many food particles down the drain . I tell my daughter all the time the sink is not a garbage disposal. On the type of sink you have there are sharp 90 degree angles close to the drain on one side so that is likely to back up quicker . Use that side for rinsing and the other side for washing dishes ect.

Bradd rated this answer Excellent or Above Average Answer

Question/Answer
sissypants asked on 09/10/05 - voting and elections

with elections just around the corner and much unrest in this country, i would like to know how many of you out there plan to vote and how many always vote or if you have just decided to vote. darlene

tomder55 answered on 09/11/05:

I vote every year .

sissypants rated this answer Excellent or Above Average Answer

Question/Answer
sapphire630 asked on 09/08/05 - From what I can understand

1. Clinton cut much of the budget for the levee way before Bush cut it some more....and much of what WAS meant for the levee money THE MAYOR used toward culture activities, water fountains and other landscaping and architechial type things.
2. It is the local governments (THE MAYORS) job to evacuate its people; NOT Washington's.
3. New Orleans DID have an evacuation plan; but THE MAYOR and governor did not think (or maybe even didn't care) how serious the damage would be.
4. FEMA and the Red Cross could not do anything to help the situation until THE MAYOR or governor signed for them to have the go ahead....which didn't happen til way too late.
5. While us outsiders are trying to find ways in to help THE MAYOR is forcing the residents out...instead of getting their help as well.
6. Instead of coming together and focusing on dealing with the problem the focus is diverted to the blame game; AND THE MAYOR is more concerned about sending his police force to Las Vegas since they are so stressed...(shouldn't that come after the immediate problems are resolved)
7. 'They' are *better off now* because they were impoverished anyway....(Barbara Bush)
and now the government can go in there and build expensive condo's without worrying what to do with these *impoverished* people....
...So it is a win-win situation since many of these *impoverished* walked off with T.V.'s, jewelry and other items they otherwise couldn't afford to even touch...(Celine Dion)
and the local government THE MAYOR has Eminent Domain due to nature...no fault of their own...they can now build million dollar mansion,etc...and attract the rich....while *the impoverished* are distributed throughout the U.S.A. no longer the mayors concern....
I could go on about all this mess...

but bottom line:
seems to me THE MAYOR has the brunt of the blame
...talk about making my head spin.

Well if my *impoverished* truck miraculously heals
I hope to go to somewhere between the Mississippi mud and the ragin' cajun's to help whatever way I can....so please pray for my luck to change enough that I can manage it.

tomder55 answered on 09/08/05:

not only that ;but there is now combined pledges between the US government(60+billion to this point with more coming );private donations (over 200 million to Red Cross alone ) . Why not just pool all the money together and divy it up and then tell the victims this is a resettlement fund . (probably adds up to a couple a hundred grand by now per )We will not waste another dime rebuilding a city in a sink-hole.

1. in all fairness .the funding was not really the issue ;and short-funding of an impossible project started in the 60s . Yes it was an impossible project . As they reclaimed wet lands to build the city and the suburbs ;they forgot that the Mississipi river has a big thing to do with the soil . They built the city on silt. They diverted the river and built all these levee thingys .That dried up the silt ,and compacted it .The city began to sink .It was 10 ft. below sea level and sinking more every year. The only part of the city that didn't was the 'crescent' that the original city was built on . That is how big the city should've grown . Does anyone realize that the high they built the levees ;the higher the water level that would've eventually filled in the "bowl" ?An even bigger disaster was waiting in the future.
2.3. now in fairness to the mayor or maybe in spite of him ; More people evacuated the city and its burbs then anyone anticipated in model scenarios . Up to 40% of respondents had indicated that they would not evacuate for alot of varied reasons . The reason so many did evacuate was because Bush insisted that the Governor get off her ass and start the process .
4. is absolutely correct and has been the biggest focus of my responses here .
5.As I said ;the city should be abandoned except the crescent area .If you can make a small port town there fine ;if not move the port facilities up river.
6. correct ,but in spite of that I ask anyone to show me an example where something the magnitiude of this relief and recovery effort has been handled better .Since this event is unprecedented I doubt anyone can.
7. her words were indefensible. I know that she was trying to shine a light on Texas' contribution but they sounded insensitive.

purplewings rated this answer Excellent or Above Average Answer
sapphire630 rated this answer Excellent or Above Average Answer

Question/Answer
Yiddishkeit asked on 09/07/05 - Vacation...

You are the President of the US! It's August 28th and Louisiana has been declared a State of Emergency, you now?


A) Put a halt to your vacation plans and focus on the disastrous effect of Hurricane Katrina.

OR

B) Give a speech about the Iraqi war campaign, keep strumming your guitar, then end the vacation days later on August 31st.




*Today my local news (Las Vegas) was reporting that Sen. Harry Reid of Nevada is considering opening an investigation as to our President's chosen vacation itinerancy.





Bobby

tomder55 answered on 09/08/05:

Exactly what do you think would have happened if Bush would have mobilized troops, declared martial law, sent in the seebee', and taken control of the situation away from local and state authorites as soon as the storm turned into a cat 5 and headed for N.O.?

Shortly before midnight Friday (before the storm hit ) the Bush administration sent Louisiana Gov. Blanco a proposed legal memorandum asking her to request a federal takeover of the evacuation of New Orleans.
The administration sought unified control over all local police and state National Guard units reporting to the governor. Louisiana officials rejected the request after talks throughout the night, concerned that such a move would be comparable to a federal declaration of martial law. Some officials in the state suspected a political motive behind the request.

Louisiana did not reach out to a multi-state mutual aid compact for assistance until Wednesday. As of Saturday, Blanco still had not declared a state of emergency.

Blanco made two moves Saturday that protected her independence from the federal government: She created a philanthropic fund for the states victims and hired James Lee Witt, Federal Emergency Management Agency director in the Clinton administration, to advise her on the relief effort.

The Red Cross stood ready with trucks of fod and water to deliver to the Superdome. They were blocked by Louisiana's Homeland security dept. under the justification that they did not want to create a magnet for more people to come to the Superdome or Convention Center .They wanted them to evacuate instead .

During the crucial early stages of the disaster it was Govenor Blanco ;not President Bush who was asleep at the wheel .

Itsdb rated this answer Excellent or Above Average Answer
powderpuff rated this answer Excellent or Above Average Answer
Yiddishkeit rated this answer Excellent or Above Average Answer

Question/Answer
excon asked on 09/07/05 - Pretty quiet over here


Hello: excon to republi-cons:

Of course, I'M embarrassed, but I've always known Bush was a dufus. I think his remarks and actions over the last few days confirms that. This time, I think he's done for.

However, I've thought that before, and he's proved me wrong. What do you, the Bush supporter, think? Please don't spin it, like he can't prevent hurricanes... That's just dumb.

excon

tomder55 answered on 09/07/05:

I sympathize with your frustration. The city and the state were always aware of the perilous position of New Orleans below sea level. But the lack of preparation for such an event, was not blamed on them, but directed at Bush. This is tantamount to an admission that individual states have no responsibility for their own destiny ,and I reject that view of our country

I did not mind the legitimate questions about FEMA ;preparedness etc. but it gets tiring doing a line by line rebutal against charges like "the national guard is in Iraq ,that is why they were not in N.O. That was nonsense and it covered for Govenor clueless Blanco who had complete authority under Louisiana law for a swift mobilization .The fact that it was Bush who had to remind her of her duty is buried ,but none-the-less true .The federal government can not, legally, send in combat troops unless requested by a State under Posse Comitatus law .i do not think you would want that changed ..do ya ?

excon rated this answer Excellent or Above Average Answer
kindj rated this answer Excellent or Above Average Answer
nikki6 rated this answer Excellent or Above Average Answer
powderpuff rated this answer Excellent or Above Average Answer

Question/Answer
kindj asked on 09/07/05 - Another perspective

A commentary by James Hirsen, who is decidedly to the right. He has his own perspective on this hurricane thing, and throws in a few facts as well. All that follows is his, and yes, it's a C and P:

by James Hirsen

Hollywood's elite didn't miss a beat in using Hurricane Katrina to launch a political attack on the Bush administration - or to seek publicity for themselves. While many of the stars' actions have been well-intentioned, some sought to turn the hurricane tragedy to their own political advantage and to further an agenda that seemingly blames Bush and the GOP for all of the world's ills - even a natural disaster. According to some on Hollywood's political fringe, whether it's global warming, the wetlands, the funding for levees, racism, the tax cut or the president's vacation, Hurricane Katrina and the devastation that ensued is somehow the fault of George W. Bush. The opportunistic villains in the Katrina tragedy include: Sean Penn, a vociferous critic of President Bush, traveled to New Orleans and criticized authorities for what he perceived as a lack of support for the victims. "There are people dying and (the U.S. government is) not putting the boats in the water. I think that's criminal negligence. I don't think anybody ever anticipated the criminal negligence of the Bush administration in this situation." Penn's personal crusade to rescue stranded victims hit a snag when his small boat - which also carried his personal photographer - sprang a leak and began taking on water. No wonder he complained on one interview that authorities have yet to provide a place for people to rinse off the sludge from their bodies after wading through flood waters. Pierce Brosnan took advantage of the spotlight at the Deauville film festival in France to say: "This man called President Bush has a lot to answer for. I don't know if this man is really taking care of America. This government has been shameful." On his weekly TV show, Bill Maher told his audience that the natural disaster was caused by global warming. Bush-basher Michael Moore, in a posting on his Web site, wrote that "those pesky scientists" had "predicted this would happen because the water in the Gulf of Mexico is getting hotter and hotter, making a storm like this inevitable." He further mocked the president with the comment, "Ignore them and all their global warming Chicken Littles." According to Moore's theory, hurricanes didn't exist before man-induced global warming happened. And in left-looped logic, Bush is to blame for the hurricane because he failed to submit Kyoto to another losing vote. (The Senate disposed of the treaty 1995 to nothing in the 1999 vote.) Regarding efforts to improve the levees protecting New Orleans, Moore maintained that Bush "specifically reduced the Army Corps of Engineers' budget for New Orleans this summer for the third year in a row," adding that there was a "much more important construction job for them - BUILDING DEMOCRACY IN IRAQ!" Moore is apparently still having problems with his fact checking. Sadly, even with full funding, none of the flood-control projects would have been completed in time to prevent the surge of water that rushed over the city. And on another woeful note, a concrete wall that was breached had already been completely upgraded in accordance with plans that spanned several administrations. During NBC's live broadcast of "Concert for Hurricane Relief," rapper Kanye West told viewers that National Guardsmen were given orders to shoot African-Americans in New Orleans. West said when African-Americans were caught stealing in New Orleans "they were called looters," but when whites were caught they were "just feeding their families." He then suggested that "George Bush doesn't care about black people." NBC producers promptly cut away before West could go any further. Angelina Jolie took a dim view of the massive relief efforts in the stricken region, saying, "It is wonderful to hear of the relief efforts that are finally coming to New Orleans and the rest of the region, but as we all know, it is simply not going to be enough." She sent letters to members of Congress and the White House asking them to increase aid efforts. Media celebrities also fell prey to the urge to use the hurricane to attack Bush. Nancy Giles of CBS claimed that the war on poverty is being lost because of the Bush tax cuts, and that since he visited Iraq but not the New Orleans Superdome, Bush doesn't "give a damn" about black people. The idea that the president would intentionally withhold assistance to any of our people who are in need in such desperate times is so out of line with the character of the man, statements like these are self-refuting. On Location at the Devastation: New Orleans Other celebrities refrained from sharply criticizing Bush or the relief efforts, but jumped at the chance to showboat and garner some of the media spotlight. Oprah Winfrey visited New Orleans, then traveled to the Astrodome in Houston, where she spoke of the "inhumane" and "embarrassing" conditions displaced people were forced to live in. She also said she would be presenting her show live from the area to ensure that their "stories would be told." Dr. Phil McGraw also hopped on the bandwagon, paying a visit to the Astrodome that was taped for airing on the future show. John Travolta flew to Louisiana on his private jet to deliver food to hurricane victims. Crooner Harry Connick Jr., who is from New Orleans, visited the stricken city soon after the hurricane struck and questioned why authorities couldn't get to the people holed up in the convention center. For sure, Hollywooders have a right to voice their opinion - just like everyone else. But in a time of emergency and crisis, should celebrities like Connick be floating through New Orleans in a row boat looking for a photo op? Is this the future of newsfotainment? Good Celebrity Stuff Celebrities who have been out front trying to use their people power to help Americans in need have shown the "right stuff" and deserve to be lauded. Last week, Matt Lauer hosted an NBC fund-raising telethon featuring Connick, Tim McGraw, Faith Hill, Mike Myers, Aaron Neville, Kanye West, Hilary Swank, Lindsay Lohan, Glenn Close, Richard Gere, John Goodman and Leonardo DiCaprio. Jerry Lewis added hurricane victims to the recipients of his Labor Day telethon. Alan Jackson agreed to headline a concert at the Grand Ole Opry. BET made plans for a benefit telecast starring Stevie Wonder, Chris Rock, Brandy, Diddy, Usher, Alicia Keys and Wynton Marsalis. ABC, CBS and Fox are currently collaborating on an Ellen DeGeneres-hosted one-hour live broadcast, which will reportedly be similar to the post-9/11 "A Tribute to Heroes" telethon. MTV, VH1 and CMT announced they would hold a joint benefit featuring Ludacris, Green Day, Gretchen Wilson, Usher, Alicia Keys, John Mellencamp and Dave Matthews Band. "Hollywood Nation" The intrusion of celebrities into national events and politics exemplifies how Hollywood is trying to set the news agenda. As my book "Hollywood Nation" details, the entertainment world has a subtle but powerful influence in shaping public opinion. Media moguls, with their politically charged films, distorted documentaries and skewed docudramas, are trying to set the agenda with little regard for the truth.Even worse, some so-called journalists are mixing information and entertainment in an attempt to ratchet up ratings - and inject their own views into the news. The Knock Iraq/Blame Bush Game Often those views are at direct odds with the facts. Some folks who are opposed to the war are trying to tie our military presence in Iraq to the Katrina response, claiming that if those members of the National Guard who were in Iraq had been available, they would have stopped the looting and shooting and been able to rescue everyone. This talking point with a Democratic scent is making the media rounds. Jesse Jackson recently took to the airwaves and made reference to the "five-billion-dollar-a-month war in Iraq." The numbers here don't really work out. Only 12 percent of our military forces are in Iraq, Afghanistan and Kuwait. Louisiana actually had plenty of guardsmen available. The tragic truth is the number of troops wasn't the problem. Getting to the area was. Interstate 10 and other highways had collapsed and the roads were flooded. Media Missteps While much of the media coverage deserves praise, some demands criticism. Just prior to the disaster, we remember how the media seemed to be immersed in a 24-hour Aruba-oriented news cycle. Initially some appeared to take this same type of approach to the hurricane coverage. The result was a sense of a stretch for the sensational without the sensitivity needed to correspond with the unfolding crisis. Most striking in contrast are the set of media standards that were in place following 9/11 and the moving of the marker that seems to have occurred with some of the hurricane disaster coverage. Following 9/11, in deference to those who had lost life and in consideration of viewers' sensibilities, the media generally avoided showing pictures of the bodies of the deceased. With the hurricane catastrophe, though, many of the networks repeatedly showed images of people who had died, sometimes with makeshift coverings strewn over their lifeless forms. When we think about mainstream media's frequent left-of-center take on events and issues, we realize that the potential for selective coverage exists even in the face of a national disaster. Unfortunately, in this regard, some media networks remained in bias mode. Less than stellar leadership on the part of local and state officials escaped scrutiny while negative remarks about President Bush garnered ample airtime. Party affiliation appeared to be the "newsworthy" deciding factor. On one occasion, CNN's Jack Cafferty asked, "Where's President Bush? Is he still on vacation? "Based on his approval rating in the latest polls, my guess is getting back to work might not be a terrible idea." As if fresh from a Cindy Sheehan rally, MSNBC's Keith Olbermann reported that Ŝ,000 Guardsmen from Mississippi and Louisiana who might have helped, might have been deployed in the relief efforts are, in fact, in Iraq and not in Mississippi and Louisiana." And MSNBC anchor Lisa Daniels dourly characterized trucks arriving with emergency supplies as "too little too late." While discussion is warranted to determine whether 72 hours is a reasonable time for a federal response to a crisis of this proportion, for some opportunists appropriate analysis gave way to political sniping. Truth Be Told A few facts are in order: President Bush declared Louisiana a disaster area two days before the hurricane struck the New Orleans area.

President Bush urged New Orleans Mayor C. Ray Nagin and Louisiana Gov. Kathleen Babineaux Blanco to order the mandatory evacuation that was issued on Sunday, August 28. First responders to a disaster are always state and local emergency agencies. FEMA is there to supplement the state and local activities.

The hurricane threatened an area as large as 90,000 square miles covering three states. Immediate relief could not possibly have been delivered to all the places that required attention. An AP photo showed a large fleet of New Orleans buses soaking in six feet of water. The mayor apparently had the means to evacuate many of the folks who ended up stranded at the Superdome and the convention center. FEMA began its activities immediately, not expecting the magnitude of the flooding, the non-response at the city and state level, and the anarchy that resulted.

The local and state governments had rehearsed for a different scenario. Disaster drills in New Orleans had taken place, but with a false assumption that the levees would hold. Both the law and protocol prohibit the president from ordering military troops into a state without a formal request to do so from the governor of the affected state.

A Final Note

On August 29 Hurricane Katrina roared into the Gulf region, crushing cities, severing families and destroying lives. Here on the Left Coast our arms stretch out to surround those who grieve. And though our hearts ache at the sight of the devastation and the toll this disaster has taken, our lips join in prayer and our spirit in hope that God will wipe away every tear, work out all things for good and heal our land.

tomder55 answered on 09/07/05:

A good picture of Sean Penn in action is here

It is not just Hollywoodheads but also the music industry . Tomorrow is the NFL kick off concert .It is scheduled to include an all star Bush haters lineup :

The smartest man in pop music ;"George Bush doesn't care about black people." Kanye West

The Strolling Bones and their new hit 'Sweet Neo Con'

Green Day

and performers at the DNC Moron Maroon 5


kindj rated this answer Excellent or Above Average Answer

Question/Answer
Choux asked on 09/06/05 - Natonal Weather Service Warning

--The following is a cut and paste--

On Sunday, August 28, the National Weather Service posted the following message on its website.

Hurricane Katrina hit New Orleans at daybreak on Monday morning, over 12 hours later.

""URGENT - WEATHER MESSAGE
NATIONAL WEATHER SERVICE NEW ORLEANS LA
413 PM CDT SUN AUG 28 2005

EXTREMELY DANGEROUS HURRICANE KATRINA CONTINUES TO APPROACH THE
MISSISSIPPI RIVER DELTA

DEVASTATING DAMAGE EXPECTED

MOST OF THE AREA WILL BE UNINHABITABLE FOR WEEKS...PERHAPS LONGER. AT
LEAST ONE HALF OF WELL CONSTRUCTED HOMES WILL HAVE ROOF AND WALL
FAILURE. ALL GABLED ROOFS WILL FAIL...LEAVING THOSE HOMES SEVERELY
DAMAGED OR DESTROYED.

THE MAJORITY OF INDUSTRIAL BUILDINGS WILL BECOME NON FUNCTIONAL.
PARTIAL TO COMPLETE WALL AND ROOF FAILURE IS EXPECTED. ALL WOOD
FRAMED LOW RISING APARTMENT BUILDINGS WILL BE DESTROYED. CONCRETE
BLOCK LOW RISE APARTMENTS WILL SUSTAIN MAJOR DAMAGE...INCLUDING SOME
WALL AND ROOF FAILURE.

HIGH RISE OFFICE AND APARTMENT BUILDINGS WILL SWAY DANGEROUSLY...A
FEW TO THE POINT OF TOTAL COLLAPSE. ALL WINDOWS WILL BLOW OUT.

AIRBORNE DEBRIS WILL BE WIDESPREAD...AND MAY INCLUDE HEAVY ITEMS SUCH
AS HOUSEHOLD APPLIANCES AND EVEN LIGHT VEHICLES. SPORT UTILITY
VEHICLES AND LIGHT TRUCKS WILL BE MOVED. THE BLOWN DEBRIS WILL CREATE
ADDITIONAL DESTRUCTION. PERSONS...PETS...AND LIVESTOCK EXPOSED TO THE
WINDS WILL FACE CERTAIN DEATH IF STRUCK.

POWER OUTAGES WILL LAST FOR WEEKS...AS MOST POWER POLES WILL BE DOWN
AND TRANSFORMERS DESTROYED. WATER SHORTAGES WILL MAKE HUMAN SUFFERING
INCREDIBLE BY MODERN STANDARDS.

THE VAST MAJORITY OF NATIVE TREES WILL BE SNAPPED OR UPROOTED. ONLY
THE HEARTIEST WILL REMAIN STANDING...BUT BE TOTALLY DEFOLIATED. FEW
CROPS WILL REMAIN. LIVESTOCK LEFT EXPOSED TO THE WINDS WILL BE
KILLED.""


Isn't this chilling?
Comments?

tomder55 answered on 09/07/05:

How about this one

TIMES-PICAYUNE of New Orleans published a story on July 24, 2005 stating:,i> City, state and federal emergency officials are preparing to give a historically blunt message: "In the event of a major hurricane, you're on your own."

Staff writer Bruce Nolan reported some 7 weeks before Katrina: "In scripted appearances being recorded now, officials such as Mayor Ray Nagin, local Red Cross Executive Director Kay Wilkins and City Council President Oliver Thomas drive home the word that the city does not have the resources to move out of harm's way an estimated 134,000 people without transportation."

"In the video, made by the anti-poverty agency Total Community Action, they urge those people to make arrangements now by finding their own ways to leave the city in the event of an evacuation.

"You're responsible for your safety, and you should be responsible for the person next to you," Wilkins said in an interview. "If you have some room to get that person out of town, the Red Cross will have a space for that person outside the area. We can help you."

see my rating of drgade for the link .I made a mistake and posted it there first .

Choux rated this answer Excellent or Above Average Answer

Question/Answer
CeeBee2 asked on 09/07/05 - Guess who's going to head the inquiry

into what happened during and after Katrina?

Katrina Inquiry

tomder55 answered on 09/07/05:

I think we should get an "independent" inquiry just like the 9-11 commission (choke).

Maybe Govenor Blanco will be invited to sit on it like Jamie Gorelick was on the 9-11 dog and pony show .

Perhaps they can fabricate a concerned victims group who will find the time to attend every meeting and make themselves available to Chris Matthews every night so they can complain about the racisim of Bush .

They can have New Orleans Mayor Ray Nagin testify and hurl invectives to their cheering approval .

Then the commission can come up with a bunch of flawed and faulty conclusions ;and they will enable themselves to pressure Congress to add layers of beurocracy to an already unwieldly system.

CeeBee2 rated this answer Excellent or Above Average Answer

Question/Answer
Choux asked on 09/06/05 - Fuel Saving Plan

Cut and Paste

On The Lighter Side

Newswire story Minot, Dakota

""Jundt was so determined to rein in his spending on gasoline that he got out of bed early and rode his 14-year-old quarterhorse mare to work.


Jundt lives 15 miles south of Minot and works as a mechanic at Goodyear Tire & Auto Service in the city.

He said he and his co-workers had been talking about rising fuel prices, and he joked that he would ride his horse to work if gasoline ever hit $3 a gallon.

His co-workers laughed, but when the price at the pump soared to $3.20 last week, Jundt headed for the barn.

He said he was only five minutes late riding his mare, Patty, to work.

While he worked, Patty waited patiently, eating hay out of the back of a truck.""



What are your plans to save on fuel prices? :=D

tomder55 answered on 09/07/05:

I was concerned about heating prices for a while now. I rebuilt my fireplace and cut wood. I converted from a 1930s oil fired system to natural gas ;and I installed efficient windows wher my old single pane /no storm sash windows were .I finished them this summer . The whole project has been almost a decade . I do not drive gas guzzlers ,and already as a rule walk into my village for local shopping . I mapped out a route to go home where I can pick up almost anything I can't get locally .I try to combine trips as much as possible .

Choux rated this answer Excellent or Above Average Answer

Question/Answer
excon asked on 09/05/05 - Potholes


Hello experts:

In my view, the first job of government is to protect me from attack, and the second is to fix the potholes. This government has failed on the first two, and the rest doesn't count.

Please elect someone who get's that. The Republicans are too busy checking out my activities in the bedroom, and the Democrats are too busy making sure nobody gets rich.

Screwing who I want (as long as it's not a kid), and getting as rich as I want are none of the governments' business.

excon

tomder55 answered on 09/06/05:

Who do you trust more ;your'local gvt. or the federal gvt.? Federalism cannot work if the locals do not pull their weight.Alabama and Mississippi are doing better in their response and the only difference I see is that the locals took the bull by the horn . I suppose that is the inherent weakness in our system but The alternative everyone is suggesting just chills me . Everyone said you could set your watch to the time of the German trains during Hitler's reign .

Louisiana Senator Mary Landrieu cited the 30% increase in transportation funding that she secured for her state in the transportation pork bill . I don't think that would've helped hold back the waters.

excon rated this answer Excellent or Above Average Answer
purplewings rated this answer Excellent or Above Average Answer

Question/Answer
Choux asked on 09/03/05 - Renquist Dead from Cancer

Are there any frontrunners for the new vacancy?

tomder55 answered on 09/04/05:

Thank You Chief Justice Rehnquist for your service to our Country. Condolences to Chief Justice Rehnquist family. May he rest in peace.He carried on the fight to the end;he gave everything he had for this Nation and the Rule of Law when he could have retired earlier.

If Bush asked me ;I would say he should elevate Judge Thomas to the Chief Justice slot ;and then nominate either Emilion Garza ;Janice Rogers Brown(my choice) ;or Edith Clement for the vacant seat.

Garza would be the first Hispanic to the court. Clement would be a fantastic choice (from Louisiana as an added bonus).

The libs would go nuts . Would they dare oppose 2 minorities just because they are conservatives ? Schumer is in convulsions over the possibility .Teddy Kennedy knows what's cooking . He is already calling for a postponement of the Roberts hearings under the pretext of the hurricane .(I'm suprised he is not in N.O. himself ;utilizing his unique underwater life saving skills)

My guess is that he will try to elevate Gonzalez to the Chief Justice position . But he may have been convinced by the open opposition in the ranks to Gonzalez to reconsider the choice.

Choux rated this answer Excellent or Above Average Answer
purplewings rated this answer Excellent or Above Average Answer

Question/Answer
QueenChoux asked on 09/02/05 - October 17th

Bob Cesca's Blog, 9-2-2005::: "It hasn't been widely discussed yet, but another disaster will strike the victims of Hurricane Katrina on October 17, 2005. And Bush can't say that he didn't anticipate it. He orchestrated it.

The president's beloved Bankruptcy Abuse Prevention and Consumer Protection Act goes into effect on that day.
Interesting that while the bill was passing though the House Judiciary Committee early this year, Democrats attempted to amend the bill to include measures to protect victims of natural disasters such as hurricanes.

The amendment to the bill, proposed by Rep. Sheila Jackson Lee (D-TX) was voted down without debate. Along party lines.

Seems the Republicans would rather see hurricane victims slammed head-first into poverty instead of continuing to be protected by Chapter 7 bankruptcy. Seems the profit margins of MBNA are more important."

INteresting...

tomder55 answered on 09/03/05:

Thomas Jefferson was a frequent user of the debt. When he left office in 1809, his wine bill alone exceeded $10,000. Add that to his 40-year project, Monticello, a lavish house that boasts 43 rooms and 13 skylights, and you'll understand why Jefferson ended up more than $107,000 in debt." According to the inflation calculator, a $10,000 wine debt from 1809 is equal to $115,259.89 today. His total debt would be $1,233,280.77 in todays dollars.He manipulated friends to help repay some of it but lived in debt his whole life.


I was against the bill because it did not make the credit card companies who make the terms of credit card use deceptive,charge loan shark rates to the people with the worse credit ratings ,preapprove with out proper creit checks,manipulates the uninformed into getting in over their heads,liable for their actions .Many card companies change the terms (in their favor, of course) over the course of a loan.
I think the expansion of consumer credit is a good thing ,but their marketing practices are dishonest, and their complaints that their loans to poor risks sometimes are bad loans doesn't impress me.When I was going to school there was zero chance that I would get credit for anything except possibly purchasing a car (collateral).Even then my rating would've been scrutinzed. Now my daughter almost daily receives solicitations extending up to $8000 in credit even though she only works part time at the library while she goes to school. I have of course STRONGLY discouraged her from accepting .

The Republican and Conservatives who favor the new law have not thought it through. Bankruptcy protection has played an integral part in our capitalist economy in that it rewards risk taking, and also works as a market mechanism that counters excessive lendingIt was also bad politics as you point out because it further paints a picture of the Republicans as being slaves to big business.

There are these commercials that show people who have done some identity theft living luxury lives .I have imagined simular commericals where someone gets hounded because they ran into unexpected medical bills, lost jobs, and suddenly missed a payment. Then your best friend the credit card company is about as friendly as Michael Moore trying to get the the front of the line at a pig roast.

We forgive debt to Africa .We certainly should do so for people struck by disaster .The new law should've only applied to credit abusers.Jerry Nadler ;NY Congressman; will introduce legislation in the house to make it easier for Katrina victims to file . The bill should be passed and Bush should sign it before Oct.17. I will write to my Reps and Bush to express that thought.



excon rated this answer Excellent or Above Average Answer
Yiddishkeit rated this answer Excellent or Above Average Answer

Question/Answer
CeeBee2 asked on 09/02/05 - "Nightline"

On ABC-TV Thursday evening (09/01/05) Ted Koppel interviewed Michael Brown, head of FEMA. Mr. Koppel had no interest in the spin, and began at least five questions with "With all due respect Mr Brown, but..." Koppel is leading the growing chorus of speaking truth to power.

Scroll down at Koppel and FEMA's Brown to see a video of the interview.

tomder55 answered on 09/02/05:

It's up to local and state officials to tell the feds what they need and to run the emergency command centers, not just throw their arms up in the air and start blaming everyone else.As soon as the flood water came, New Orleans govt simply melted down.

In other areas when a mandatory evacuation is declared the local police /fire dept. will go neighborhood to neighborhood informing residents and if necessary assist those who cannot evacuate themselves . Then before the storm ;the officials move their equipment to a safe place and wait til the storm is over .High winds and flooding could destroy the rescue equipment and turn first responders into victims .They then bring their equipment back in to start recovery operations .But in the early days of the news footage what did we see ? Police cars ,ambulances ,buses ,fire equipment all floating in the flood waters . If city officials told people to go to the convention center and to the Super Dome for days, how did they just forget about all the people they walked away from ? Did anyone from the city or state think about something so obvious as evacuating hospitals?

There was NO working chain of command. There was no plan to evacuate people who did not have cars. There was no provision for food, water, sanitary items, etc at either the Superdome or Convention Center. Compare N.O. govt in the flood to N.Y.C govt on 9/11. compare N.O. to San Francisco when they have the wholly expected earthquakes.

Ted Koppel can slam Michael Brown all he wants to ;Shep Smith is doing the same frustrated 'why isn't someone doing something 'plea now even as the feds are acting and starting to do the work that local officials should've already initiated. Everyone can point fingers at FEMA and not have the slightest idea what it is that FEMA has done wrong.To what extent the federal government has failed anyone remains to be seen, but it ought to be clear to the world that the City of New Orleans and the state of Lousiana was not in any way prepared for Katrina.

CeeBee2 rated this answer Excellent or Above Average Answer
excon rated this answer Excellent or Above Average Answer
voiceguy2000 rated this answer Excellent or Above Average Answer

Question/Answer
QueenChoux asked on 09/02/05 - FLIP FLOP

Associated Press:: CORONADO, Calif. -- President Bush answered growing antiwar protests yesterday with a fresh reason for US troops to continue fighting in Iraq: protection of the country's vast oil fields, which he said would otherwise fall under the control of terrorist extremists.


The president, standing against a backdrop of the USS Ronald Reagan, the newest aircraft carrier in the Navy's fleet, said terrorists would be denied their goal of making Iraq a base from which to recruit followers, train them, and finance attacks.

''We will defeat the terrorists," Bush said. ''We will build a free Iraq that will fight terrorists instead of giving them aid and sanctuary."

We are so disgraced in the eyes of the world. This and NewOrleans. If ever, this is a President who must be impeached.

Comments?

tomder55 answered on 09/02/05:

you don't think the oil fields should be protected ? yeah there were a few news orgs that latched onto this misleading hack job by Jennifer Loven of AP . Bush did indeed add in a comment that if we were to leave Iraq now we would risk having Zarqwai using Iraqi oil revenue to finance terrorism ,but the main focus of his speech in Coranado was to commemorate VJ Day . There has been no change in policy ...no flip flop. I do not know why everyone is still reaching for straws ;the election is over ...Bush won . Moveon.

purplewings rated this answer Excellent or Above Average Answer
QueenChoux rated this answer Average Answer

Question/Answer
CeeBee2 asked on 09/01/05 - Apples and oranges.

Today President Bush said, "New Orleans is more devastated than New York was."

Hmmmmmmm. Please tell me why I'm having trouble digesting that comment.

tomder55 answered on 09/02/05:

When terrorists struck on September 11, the carnage was huge and the loss of life staggering, but an entire community was not wiped out. With this disaster, America confronts for the first time the reconstruction of large urban and industrial centers as well as complex social systems and political organizations not just in N.O. but in the greater Gulf Coast.

Why do you nitpick about a single sentence ?As you know ;that was part of a larger interview with Diane Sawyer yesterday on 'Good Morning America'

http://abclocal.go.com/kgo/story?section=nation_world&id=3403364

Surely you understand the complexity of the rebuilding effort ,even after the human recovery is complete and the structural damage is assessed ? He is right. The WTC attack was contained in a couple of city blocks . This requires a rebuilding effort not seen in America since the reconstruction of the South after the Civil War. There are some who would say that that effort is still on-going .

CeeBee2 rated this answer Excellent or Above Average Answer

Question/Answer
QueenChoux asked on 08/31/05 - Cost of Iraqi War in $$

The following is a cut and paste from a newseire story::

WASHINGTON (Reuters) - The U.S. war in Iraq now costs more per month than the average monthly cost of military operations in Vietnam in the 1960s and 1970s, according to a report issued on Wednesday.

The report, entitled "The Iraq Quagmire" from the Institute for Policy Studies and Foreign Policy in Focus, both liberal, anti-war organizations, put the cost of current operations in Iraq at $5.6 billion per month. This breaks down to almost $186 million a day.

"By comparison, the average cost of U.S. operations in Vietnam over the eight-year war was $5.1 billion per month, adjusting for inflation," it said.

As a proportion of gross domestic product, the Vietnam War was more significant, costing 12 percent of annual GDP, compared to 2 percent for the Iraq War. However, economists said the Iraq war is being financed with deficit spending and may nearly double the projected federal budget deficit over the next 10 years.

What are your comments?

tomder55 answered on 09/01/05:

inflation ;different war;I do not recall any outcry that Nam jeeps weren't up-armored .Reuters has no shame.

QueenChoux rated this answer Average Answer

Question/Answer
Bradd asked on 08/31/05 - George Stops Funding For New Orleans Levee

Tax cuts for his wealthy pals and huge budgets for his war in Iraq resulted in the Bush White House stopping funding for critically needed levee work in Lake Ponchatrain - you know, the lake that is now inundating New Orleans?

The elimination of previously agreed upon federal dollars for the work stopped in 2004 amid warnings of possible disaster by those who were there. Specifically, work at the now broached 17th St. levee.

Interesting story....

http://www.editorandpublisher.com/eandp/news/article_display.jsp?vnu_content_id=1001051313

tomder55 answered on 09/01/05:

It was a real brain storm to build a major city in a sink hole. Can you guarantee that all the additional funding for levy building would've prevented this ? of course you can't.

The Army Corp of engineers has a history of work that ultimately proves to be folly . Most of it involves creating livable space on wetlands . I have seen a small fortune wasted building jettys for the purpose of beach retention .Why ? so fat cats can build mansions on barrier beaches . They build break water barriers that disappear during high tide and become boating hazzards .

Perhaps we should come to the realization that not all the land mass on the planet is inhabitable.

Can you please wait until all the survivors are rescuded and the bodies recovered before you start making cheap political hay from this ?

Bradd rated this answer Excellent or Above Average Answer
Itsdb rated this answer Excellent or Above Average Answer
kindj rated this answer Excellent or Above Average Answer
QueenChoux rated this answer Poor or Incomplete Answer

Question/Answer
QueenChoux asked on 08/30/05 - The Blame Game

"As his poll numbers sink, Bush is getting desperate. From his address today in San Diego:

'They looked at our response after the hostage crisis in Iran, the bombings of the Marine barracks in Lebanon, the first World Trade Center attack, the killing of American soldiers in Somalia, the destruction of two U.S. embassies in Africa, and the attack on the USS Cole. They concluded that free societies lacked the courage and character to defend themselves against a determined enemy After September the 11th, 2001, weve taught the terrorists a very different lesson: America will not run in defeat and we will not forget our responsibilities.'

****(Conveniently, Bush doesnt mention any terrorist attack that occurred during his fathers administration.)*** Asterisks mine, Mary Sue

Once upon a time, the President didnt believe in playing the blame game:

Well, the President is not one that focuses on blame or finger pointing. The President focuses on what we need to do to address challenges.

It appears that statement is inoperative.

Comments?

tomder55 answered on 08/31/05:

sorry you got your facts wrong . October 3, 1993 was the date of the battle . It was Clinton who correctly determined that the peace-keeping mission could not succeed unless Mohamed Farrah Aidid was captured ,killed or marginalized. I have not problem with that .

My problem is that he cut and ran when things got tough....btw the casualties of the day were 19 Americans killed between 500 -1000 militia terrorists killed . The battle was a victory ;but Clinton treated it like a defeat.

So yes ;Bush Sr. did send American troops on a humanitarian mission in Dec. of 1992 ;but it was Clinton who retreated .

purplewings rated this answer Excellent or Above Average Answer
QueenChoux rated this answer Excellent or Above Average Answer

Question/Answer
excon asked on 08/31/05 - Trouble looming (or at our doorstep??)


Hello experts:

Bush, as you know, sucks.

However, the Dem's suck worse. When the NY Times blares the following in an op-ed editorial, you know we're in trouble:

>>>Hillary Rodham Clinton is betting that there's no need to do anything rash now, like leading.<<<

Now I read that two thirds of Americans think creationism should be taught along side evolution in science class.....

The next world war is going to be between our theocracy and their theocracy. Me?? I'm leaving.

excon

tomder55 answered on 08/31/05:

Evita lead ?


The debate is about ID . For my 2 cents it should be taught but not as science except to point out that there are alot of fallacies in the evolution hypothesis. But since ID is itself not falsifiable ,it should not be taught as science .

Imagine someone portraying a scientific hypotheisis as a fact ...can we think of another example of that ?? oh yeah ;global warming .

excon rated this answer Excellent or Above Average Answer
ladybugca rated this answer Excellent or Above Average Answer

Question/Answer
QueenChoux asked on 08/30/05 - Crisis on Southern BorderPat Buchanan's Take

Time to Impeach Bush per Pat Buchanan: "It is a mark of the cowardice of our leaders that they are so terrified of being called "bigots" they tolerate this criminality(of illegal aliens). The moral rot of political correctness runs deep today in both national parties.

A president like Teddy Roosevelt would have led the Army to the border years ago. And if Fox did not cooperate, T.R. would have gone on to Mexico City. Nor would Ike, who deported all illegal aliens in 1953, have stood still for this being done to the country he had defended in war.

The question of whether America is going to remain one nation, or whether our Southwest will wind up as a giant Kosovo separated by language and loyalty from the rest of America is on the table.

Where is Bush? All wrapped up in the issue of whether women in Najaf will have the same rights in divorce and custody cases as women in Nebraska. His legislative agenda for the fall includes a blanket amnesty for illegals, so they can be exploited by businesses who want to hold wages down as they dump the social costs for their employees health care, schools, courts, cops, prisons onto taxpayers.

Not only have Richardson(Gov of New Mexico) and Napolitano(Gov of Ariz) awakened they are on the front lines so, too, has Hillary Clinton, who has spoken out against illegal immigration with a forthrightness that makes Bush sound like a talking head for La Raza.

Why is a Republican Congress permitting this president to persist in the dereliction of his sworn duty?

George Bush is chief executive of the United States. It is his duty to enforce the laws. Can anyone fairly say he is enforcing the immigration laws? Those laws are clear. People who break in are to be sent back. Yet, more than 10 million have broken in with impunity. Another million attempt to break in every year. Half a million succeed. Border security is homeland security. How, then, can the Department of Homeland Security say America is secure?

Who can guarantee that, of the untold millions of illegals here, and the scores of thousands ordered deported for crimes who have disappeared into our midst, none is a terrorist waiting for orders to blow up a subway or mall and massacre American citizens?

Most of these illegals come to work to send money back to their families. They are not bad people. But because they are predominantly young and male, they commit a disproportionate share of violent crimes.

Why should U.S. citizens be assaulted, robbed, raped and murdered, and have their children molested, because their government will not enforce its own laws?

Is this not an indictment of democracy itself? What dictatorial regime would put up with this?

The Republican Party claims to be a conservative party. But what kind of conservative is it who, to cut a few costs or make a few bucks, will turn his family's home into a neighborhood flop house?

Twice, George Bush has taken an oath to "preserve, protect and defend the Constitution of the United States." Article IV, Section 4 of that Constitution reads, "The United States shall guarantee to every State in this Union a Republican Form of Government, and shall protect each of them against invasion."

Well, we are being invaded, and the president of the United States is not doing his duty to protect the states against that invasion. Some courageous Republican, to get the attention of this White House, should drop into the hopper a bill of impeachment, charging George W. Bush with a conscious refusal to uphold his oath and defend the states of the Union against "invasion."

It may be the only way left to get his attention, before the border vanishes and our beloved country dissolves into MexAmerica, what T.R. called a "polyglot boarding house for the world."

I'd like to get your comments on this incendary column. Thanks, sorry it is too long. Will avoid this in the future.



tomder55 answered on 08/31/05:

The question of whether America is going to remain one nation, or whether our Southwest will wind up as a giant Kosovo separated by language and loyalty from the rest of America is on the table.

I would say that is the case not just in the Southwest but throughout the country .Here are pictures of a typical day at a Long Island NY corner where illegal day laborors gather .

My take is that Buchanan is an A**H***; Does he really think Teddy Roosevelt personally lead the troops anywhere when he was president ?

That doesn't change the fact that Bush's feet should be held to the fire on illegal immigration. Illegal immigration is way up since Bush took office in 2000.

QueenChoux rated this answer Excellent or Above Average Answer

Question/Answer
QueenChoux asked on 08/30/05 - Worldwide Weapons Sales

Published: August 30, 2005

WASHINGTON, Aug. 29 - "The value of military weapons sales worldwide jumped in 2004 to the highest level since 2000, driven by arms deals with developing nations, especially India, Saudi Arabia and China, according to a new Congressional study.

The total of arms sales and weapons transfer agreements to both industrialized and developing nations was nearly $37 billion in 2004, according to the study.

That total was the largest since 2000, when global arms sales reached $42.1 billion, and was far above the 2003 figure of $28.5 billion.

The United States once again dominated global weapons sales, signing deals worth $12.4 billion in 2004, or 33.5 percent of all contracts worldwide."

Mary Sue: America is the largest weapons seller to China. What do you make of that?

tomder55 answered on 08/31/05:

The US is the largest weapons exporter. I'll take your word for it that we also are the largest supplier to China. My preference would be for us to stop it . That old axiom by Lenin is as true today as when he stated it;(paraphrase)" "The capitalists will sell us the rope with which to hang them."

purplewings rated this answer Excellent or Above Average Answer
QueenChoux rated this answer Average Answer

Question/Answer
QueenChoux asked on 08/30/05 - The Blame Game

"As his poll numbers sink, Bush is getting desperate. From his address today in San Diego:

'They looked at our response after the hostage crisis in Iran, the bombings of the Marine barracks in Lebanon, the first World Trade Center attack, the killing of American soldiers in Somalia, the destruction of two U.S. embassies in Africa, and the attack on the USS Cole. They concluded that free societies lacked the courage and character to defend themselves against a determined enemy After September the 11th, 2001, weve taught the terrorists a very different lesson: America will not run in defeat and we will not forget our responsibilities.'

****(Conveniently, Bush doesnt mention any terrorist attack that occurred during his fathers administration.)*** Asterisks mine, Mary Sue

Once upon a time, the President didnt believe in playing the blame game:

Well, the President is not one that focuses on blame or finger pointing. The President focuses on what we need to do to address challenges.

It appears that statement is inoperative.

Comments?

tomder55 answered on 08/31/05:

ok I'll mention one of the attacks during his father's adm. ......Saddam invaded ;looted and raped Kuwait .He then massed his troops on the border of Saudi Arabia. It was the swift action of Bush and Thatcher that prevented an advance into Saudi Arabia .They then proceded to evict Iraqi forces from Kuwait.

Bush is correct .Our respionse to terrorism was weak. But if you don't believe Bush when he speaks about the preception that left with the terrorists then how about the words right out of the horses mouth (OBL)

But your most disgraceful case was in Somalia; where -- after vigorous propaganda about the power of the USA and its post cold war leadership of the new world order -- you moved tens of thousands of international force, including twenty eight thousands American solders into Somalia. However, when tens of your solders were killed in minor battles and one American Pilot was dragged in the streets of Mogadishu you left the area carrying disappointment, humiliation, defeat and your dead with you. Clinton appeared in front of the whole world threatening and promising revenge , but these threats were merely a preparation for withdrawal. You have been disgraced by Allah and you withdrew; the extent of your impotence and weaknesses became very clear. It was a pleasure for the "heart" of every Muslim and a remedy to the "chests" of believing nations to see you defeated in the three Islamic cities of Beirut , Aden and Mogadishu.

Itsdb rated this answer Excellent or Above Average Answer
kindj rated this answer Excellent or Above Average Answer
QueenChoux rated this answer Excellent or Above Average Answer

Question/Answer
Itsdb asked on 08/30/05 - Between Iraq and a hard place

On page 3 of our paper this morning the following two articles appeared, one under the other:

Sunnis face dilemma on Iraq constitution
ROBERT H. REID
Associated Press

BAGHDAD, Iraq - Rebuffed in the constitution deliberations, Sunni Arabs now face a dilemma: boycott the Oct. 15 referendum on a new charter and hand the Shiites a landslide victory, or take part in a vote that demographics suggest they'll lose.

But the Shiite community itself is divided over the constitution, and interviews on Baghdad streets indicate the key federalism proposal may be a hard sell to many on both sides.

About 2,000 people, mostly Sunnis, marched Monday against the constitution in Saddam Hussein's hometown of Tikrit. Some carried pictures of Saddam and repeated chants heard in countless stage-managed protests during his regime: "We sacrifice our souls and blood for you, Saddam."

Others carried pictures of radical Shiite clerics Muqtada al-Sadr and Jawad al-Khalisi, who have joined the Sunnis in opposing the constitutional draft because of federalism - which critics fear will lead to the disintegration of Iraq...

~~~~~~~~~~~~~~~~~~~~~~~~~~~~~~~~~~~~~~~~~~~~~~~~~~~~~~~

Newsview: New Mideast Violence May Wane
By STEVEN GUTKIN, Associated Press Writer
Monday, August 29, 2005

(08-29) 14:56 PDT JERUSALEM, Israel (AP) --

A lethal arrest raid, a suicide bombing, fresh land expropriations, a threatening Hamas video: So far, that's the follow-up to Israel's historic Gaza pullout.

Rather than seize the moment to jump-start negotiations, Israelis and Palestinians appear to be falling into a familiar pattern of violence and rhetoric. Still, the withdrawal from Jewish settlements in Gaza is of such significance that even the latest spasms are unlikely to torpedo all momentum for peace.

Ariel Sharon won praise for fulfilling his pledge in a dramatic, breakneck sweep that ended last week to evacuate 8,500 settlers from the Gaza Strip and another 500 from the northern West Bank. Now the Israeli leader wants to send a very clear message that terrorism won't be tolerated and that major West Bank settlement blocs will remain Israeli...

Sharon's critics say now is the time to capitalize on the goodwill created by the Gaza evacuation, not to flex muscles. Many fear the two sides already have begun to squander a unique historical opportunity. And the recent friction has brought home the pitfalls of trying to get Israelis and Palestinians together after five years of trust-destroying violence.

Both sides say they're still prepared to talk, however. Officials said a meeting is possible between Sharon and Palestinian leader Mahmoud Abbas when the two are in New York next month to address the United Nations.

And Sharon's withdrawal, ending 38 years of Israeli presence in the Gaza Strip, is likely to have long-lasting ripple effects on Mideast peacemaking that could weather some setbacks. With the settlers gone, the army is expected to complete its own pullout in the coming days...

~~~~~~~~~~~~~~~~~~~~~~~~~~~~~~~~~~~~~~~~~~~~~~~~~~~~~~~

Play along with me here, is there anything wrong with these portrayals? Why is the Iraq story portrayed as more or less a disaster, while the Israeli/Palestinian story is portrayed as hopeful?

Steve

tomder55 answered on 08/30/05:

2000 protesters in Saddam's stronghold ? . That sounds about right. No clearer indication about the real state of the 'insurgency' is necessary. It has failed ,and the Sunnis who were sitting on the fence are betting their lot on politicians who are trying to make the best deal possible . All this talk of defeating the Constitution at the polls is bluster .

I think it was a blunder to not finish off al-Sadr while we had a chance. He is still primarily an agent of Iran .We(primarily the Brits) have also allowed his minions to gain too much of a stronghold in Basra .

However ;together the combined influences of Saddam's Baathists ;al-Sadr ,and Zarqwai are not enough to derail the process (even if they were capable or inclined to combine their efforts).

Our press is myopic ,and have resorted to the platitudes of comparing Iraq to Vietnam even as they complain that Bush uses tired platitudes to rally the faithful.

Truth be told ;there is more room for optimism in Iraq than in Palestine. The Gaza withdrawal followed the old play book of Israeli concessions seen as military victories to the Palestinians ;and no reciprocal concessions by the Palestinians . The dust from the last bulldozed settlement house had not settled yet before the next homicide bomber had made an attempt at murdering innocent Israelis. The pattern since Oslo continues . The road map ...a one way street. I hope Condi is happy .

Sharon's critics say now is the time to capitalize on the goodwill created by the Gaza evacuation, not to flex muscles. Many fear the two sides already have begun to squander a unique historical opportunity.

I have to ask STEVEN GUTKIN how he thinks Sharon squandered anything ?...except possibly the good will of some Israeli citizens who he forced at gun point to leave their homes.

Itsdb rated this answer Excellent or Above Average Answer
kindj rated this answer Excellent or Above Average Answer

Question/Answer
excon asked on 08/28/05 - Oil


Hello experts:

With supplies stretched to the limit, what will happen if Katrina destroys some (all?) of the rigs just off shore from New Orleans?

Time to buy oil futures?

excon

tomder55 answered on 08/29/05:

under the circumstances ;given the nature of a natural disaster ,I think it would be appropriate to release oil from the strategic reserve to supplement the loss of supply from the rigs.

That will not take care of the biggest pressing problem ;the lack of refining capablility that we still refuse to address. Every market analysis is saying that the oil supply is sufficient . But unfortunately our energy use is not in raw crude but in processed fuel .

excon rated this answer Excellent or Above Average Answer

Question/Answer
paraclete asked on 08/26/05 - Regard the past, arrogant youth

By Mike Carlton
August 27, 2005


THE new political correctness of the ratbag right decrees that nobody must compare the unhappy result of the Vietnam war to the wonderful march of democracy in Iraq.

Anyone who mentions the word quagmire can only be a pathetic baby boomer, dissolute and decrepit, pining for the bad old days of moratorium demos, Whitlamism, bell-bottom pants, Jane Fonda, etc.

This view is trumpeted most loudly by the thirtysomething know-alls of the right-wing blogosphere, whose ferocious enthusiasm for the Iraq war is matched only by their reluctance to take part in it. (Perhaps they have other priorities, as Dick Cheney once explained his decision not to enlist for Vietnam.)

But I was surprised to see the Herald's Washington correspondent, Michael Gawenda, hop into a spot of baby-boomer bashing last Monday.

"There are no lessons from Vietnam that apply to Iraq. And the Cold War, which was the context and the pretext for the American intervention in Vietnam, shares no similarities with the war on terrorism," he wrote.

"It's time all those old baby boomers, for whom the Vietnam War and those halcyon days of protest and love were the most intense time of their lives, got on with planning their retirements." Phew. I felt like curling up with a Greatest Hits of the ླྀs CD "and another girl to take my pain away", as the Rolling Stones used to sing.

And how nice to see them back at it again, too. Why, the daft old geezers have even written a song called Sweet Neo-Con, which is rude about George W.Bush and Condoleezza Rice.

Anyway. Groping about in the fog of senility, I did think of one or two lessons to draw from ancient history, a few parallels that might be seen.

Firstly, it is not a good idea to go to war on a lie or, worse, on CIA intelligence fantasies. President Lyndon Johnson did this with the fabricated Gulf of Tonkin incident of 1964, exactly as Dubya did with Saddam's weapons of mass destruction, his nuclear program, his links to Osama bin Laden, his involvement in 9/11, etc. Discovery of the lie down the track makes public support for the war so much harder to sustain.

Secondly, do not believe that overwhelming technological superiority and firepower lead to a quick military and political victory. It did not happen against the North Vietnamese or the Viet Cong, and it is not happening with the suicide bombers of Iraq, despite the airy promise that the US marines would be garlanded with flowers in the streets of Baghdad.

Thirdly, do not think you can rush headlong to impose a one-size-fits-all system of liberal democracy on people who have not known such a thing in all their history. Each time Dubya hails the new constitution of Iraq, I hear LBJ making the same sort of gurgling noises over the American sponsored constitution of the Republic of South Vietnam. I wonder where that proud document is now.

Finally and most importantly, have war aims and an exit strategy. Johnson just escalated. Richard Nixon's ploy was to declare that America's job was done, that South Vietnam could now defend itself, thank you and goodnight Saigon. We know how that ended.

Dubya - or his successor - will have to do much the same thing. The American people have begun to tire of young soldiers coming home horizontally for no visible result. Eventually, whoever is in the White House will announce that Baghdad is ready to command its own destiny, blah blah. Iraq will then either collapse into civil war or become a hardline Islamist theocracy in league with Iran. It will be the fault of Dubya and the neo-cons if the US is humiliated, with the war on terrorism no nearer an end. But by then we baby boomers will be safely tucked away in the sunset home.

tomder55 answered on 08/27/05:

the former hippy geezers were wrong about Nam and they are wrong about Iraq. Doesn't Mick Jagger and the Strolling Bones realize how pathetic they look ? They are all converging on Crawford Texas now doing a reenactment of the summer of love ..pathetic !



excon rated this answer Excellent or Above Average Answer
paraclete rated this answer Excellent or Above Average Answer
purplewings rated this answer Excellent or Above Average Answer
QueenChoux rated this answer Excellent or Above Average Answer

Question/Answer
CeeBee2 asked on 08/26/05 - Iraq is no longer a state

Bush Must Face Facts: Iraq is No Longer a State

Everything that held Iraq together has disintegrated or is morally unsupportable. In this op-ed from the Jerusalem Post, the author argues that the sooner the Bush Administration accepts the fact that Iraq is no more, the more quickly a long-term solution for the people in the area will be found.

By Shlomo Avineri

August 22, 2005

Original Article (English)

Despite all the recent frantic attempts at constitution-making, Iraq is not a state anymore. It is difficult for the U.S. government, as well as for the international community, to realize this, but the earlier it sinks in the better the chances for a realistic approach which could give the people in Iraq a chance for a more peaceful future.

Even since Iraq was cobbled together by British imperial dreamers in the 1920s from three very disparate provinces of the old Ottoman Empire the only way to hold it together was by brute force. The British vested power in the Baghdad-based Sunni Arab minority. The Kurdish minority in the north, as well as the Shiite majority in the south, were virtually excluded from power. Consequently, all Iraqi governments were faced with recurring mutinies: by the Kurds, by the Shiites, even by the small Christian Assyrian community.

Saddam Hussein's regime was the most brutal of all of Iraq?s Sunni Arab minority regimes, and this is why Iraq has always long been the most repressive Arab regime.

The end of Saddam Hussein also toppled Sunni Arab minority rule; the current mayhem in Iraq is mainly the work of Sunni Arabs trying to abort any alternative government. The sophistication, logistic precision and overall planning of the terrorist attacks, as well as the apparent availability of hundreds of suicide bombers, cars and explosives all point to a well-prepared campaign based on the human and material resources of Saddam's old regime.

It is obvious that the Kurds, who have enjoyed de facto autonomy since the early 1990s under the protection of the Allied "No Fly Zone," are not going to accept being subjected to Sunni Arab rule. The Kurdish regional government runs a more or less successful system of political authority. For a decade now, schools in the area have taught in Kurdish and not in Arabic, and a de facto arrangement allows the Kurdish authorities to use oil revenue in the area to pay for impressive development projects.

Given their terrible experiences in the past, the Kurds will accept only the kind of federal structure that guarantees them effective control over their own affairs, including maintaining their own armed forces.

Similarly, the Shiites are not going to accept Sunni hegemony any longer, and the brutal terrorist attacks of the Sunni insurgents against Shiite shrines only strengthens their resolve to insist on a Shiite autonomous region in the south, similar to the Kurdish area in the north.

The Sunnis rightly realize that unless they succeed in re-imposing their power by brute force, they are doomed to minority status - something which is alien to the Sunni Arab tradition. Hence the Sunni boycott of the elections and the attempt by Sunni terrorists to frighten any moderate Sunni ready to cooperate in setting up a democratic Iraq.

Constitutional phraseology is not a remedy for these conundrums.

When the Soviet Union and Yugoslavia were on the verge of collapse along ethnic lines, the administration of Bush Senior urged the maintenance of the existing structures: It failed dismally. Iraq may now be going the way of Yugoslavia, yet the U.S. government does not wish to recognize this obvious fact. What is failing in Iraq is not only the attempt to build democracy, but the very attempt to keep the country together.

'Operation Spear and Dagger' by U.S.-Led Iraqi Forces [From Ad Dustour, Jordan]

There is no way of putting Humpty-Dumpty together again. The Kurds and the Shiites will go their separate ways, and both entities have the paramilitary capability to do so. There is no Iraqi army capable of maintaining the unity of the country. And, just as in the former Yugoslavia, the separate countries - Slovenia, Croatia, Serbia - have a better chance of creating coherent and democratic systems than the old coercive Yugoslavia, the same may apply to Iraq.

The U.S. will obviously have to change its policy over Iraq - maybe this is what President George W. Bush is devoting his vacation to. It would be advisable to think outside the box and realize that Iraq is not a country anymore.

This is not the end of the world, but it calls for courageous and creative thinking about alternatives.

**The writer is professor of political science at the Hebrew University of Jerusalem.**


What think you?

tomder55 answered on 08/27/05:

the separate countries - Slovenia, Croatia, Serbia - have a better chance of creating coherent and democratic systems than the old coercive Yugoslavia, the same may apply to Iraq.

if that is the case then why is there still a US presence in the Balkans(where was the exit strategy there btw ? )? Why was there are a UN presence in Timor ? Why not let the Basques in Spain break away ;Quebec in Canada. Why did we not let the South break away ?

Do you really believe that breaking down nations to ethnic and tribal units is a better solution ? I cannot believe what I read here sometimes. The war critics alternate between a Baathists solution (strong dictatorship)and a Zarqwai solution (civil war). There are 3 groups who reject this constitutional solution... The Baathists who want a return to a Stalin-like regime;the Sunni jihadists who want all nations in the region disolved and consolidated into a greater Caliphate;and the al-Sadr Shia who dream of a Iran like theocracy. All three groups are capable of mischief and violent disruption .But their appeal is limited and since their agendas are so disparate they are incapable of cooperating towards a common end .

The ELECTED Iraqi leaders continue to struggle ;compromise to try to forge a free nation. The Iraqis expressed a desire to do so in January ,and the turnout in the next referendum election will even be greater ;even though the threat from Baathists and jihadists to murder them for taking the courageous step of voting still exists.

The Sunnis are now regestering in historic numbers.They now realize that their boycott was a blunder .Yet;even though they did boycott, the majority have allowed Sunni representatives into the process.If the goal was dissolution then they would not be making compromises and accomodations .

The bottom line however is 80% of Iraq's population wants this constitution. Why 20% should be allowed to hold it up is a mystery to me .

CeeBee2 rated this answer Excellent or Above Average Answer
Itsdb rated this answer Excellent or Above Average Answer

Question/Answer
CeeBee2 asked on 08/26/05 - Constitutions for dummies?

Lack of detail bedevils Iraq's new constitution
August 26, 2005
BY DEBRA PICKETT CHICAGO SUN-TIMES

"The constitution, to be successful, has to take into account the legitimate interests and fashion a balance in the federalism aspect of it and in the other key things that they're worried about so that they'll all nod and say, 'Well, I really don't like it, it's not perfect, but it's good enough, and by golly, if we have to amend it, lots of other countries have amended their constitution.'" -- Donald Rumsfeld

There's something darned inspiring about watching democracy in action. It's hard not to get a little misty at the thought of a young nation, born of struggle, deciding that, by golly, it's time to take a stand on the kind of timeless and immutable principles that are, well, you know, pretty much good enough for now.

Somewhere, Mrs. Vogel is freaking out. My eighth-grade civics teacher -- a woman whose idea of classroom management was to tell an unruly kid he was "ill bred" -- must be thrilled that the week's news revolves around the relative merits of federalism. But it also must be making her nuts that a lot of the people involved in the whole Iraqi constitution-writing process seem to be taking it about as seriously as the average 14-year-old takes a term paper assignment.

"The constitution," she used to tell us, "embodies our highest principles."

So I bet it really bothers her that, despite the celebration of its sort-of completion, almost on time, the newly drafted Iraqi constitution manages to say nothing and to mean even less. Full of the broad generalizations and hedge-your-bets contradictions found in the middle of triple-spaced book reports with extra-wide margins, the document fails to answer even the most basic questions about how the "new" Iraq will be governed.

As an exercise in taking the exam without having read any of the books, it's impressive, even artful. But as the foundation for a new democracy -- one established at the cost of thousands of lives -- it leaves a lot to be desired.

No contradictions allowed

At least one of the delegates must have picked up a copy of Constitutions for Dummies because the proposed document starts off strong, with a familiar-sounding preamble that begins "We the people . . ."

But things quickly go south from there and, by article two, the delegates' lofty rhetoric descends into something that sounds a little like the rules for Fight Club. The document reads:

First, Islam is the official religion of the state and is a basic source of legislation:

a) No law can be passed that contradicts the undisputed rules of Islam.

b) No law can be passed that contradicts the principles of democracy.

c) No law can be passed that contradicts the rights and basic freedoms outlined in this constitution.


There's no mention of what's supposed to happen if the rules of Islam and the principles of democracy happen to contradict each other, as seems pretty much inevitable given the pronouncements of some of Iraq's ruling clerics.

Guaranteed, unless it isn't

Things just get more complicated from there, as the constitution goes on to list some of the rights and freedoms that are "guaranteed" to Iraqi citizens.

Article 17 says that each person has the right to privacy "as long as it does not violate the rights of others or general morality."

And, similarly, Article 36 guarantees freedom of expression, assembly, protest and the press, "as long as [this freedom] does not violate public order and morality."

There's no mention of how morality should be defined, but, if you check out the writings of the senior Shia cleric in Iraq, Grand Ayatollah Ali Sistani, it becomes clear that, for many of our, ahem, democratic allies in Iraq, morality has a certain know-it-when-I-see-it quality. According to Sistani, chess and backgammon are unquestionably sinful, as is playing the lottery and maintaining a friendship with a member of the opposite sex, while birth control, betting on horses and drinking nonalcoholic beer are all totally OK, as is plastic surgery.

Open, unless they're secret

Article 19 says that Iraqi court sessions will be open "unless the court decides to make them secret." Which, when you get down to it, is not exactly ironclad protection against a tyrannical regime of secret laws and secret courts.

In article after article, the proposed constitution makes passing reference to fundamental rights and freedoms and then stops short of protecting them in any meaningful way. Maybe that sort of thing is not supposed to matter.

Maybe this constitution -- these governing principles that are supposed to be the foundation of a new republic -- is just a symbol. And maybe what it says doesn't really matter because, as Rumsfeld points out, it can always be changed later.

But if that's what this great new democracy is all about -- a list of freedoms that come with a built-in mechanism for taking them away -- you have to wonder if it's really worth fighting and dying for.

***************************

Do we agree with Rumsfeld, or what?

tomder55 answered on 08/26/05:

are you in favor of self determination or "imposing your way of life "on them ?


Our founders took the most serious issue of the day ;slavery ;and agreed to disagree and punted the issue to future generations . I imagine the nature of Islam in a democratic society would be just a perplexing a problem for them.

This experiment in democracy is the only hope that Islam has to avoid their equivelent of the 30 Years War.

CeeBee2 rated this answer Excellent or Above Average Answer

Question/Answer
CeeBee2 asked on 08/26/05 - Venue selection

Have you noticed where Bush is giving his speeches lately? - before the VFW in Utah, at National Guard rallies in Idaho. Careful venue selection was a hallmark of Richard Nixon's last days. Nixon was reduced to delivering his speeches on military bases and at Billy Graham revivals because he didn't dare go out in public. When will George Bush speak in a public venue, and face the inevitable howl of protest? Isn't that something we expect of leaders in an open society? (paraphrase of columnist Neil Steinberg, Chicago Sun-Times)

tomder55 answered on 08/26/05:

This is new ? He has been selective since he became President . No I do not expect the President to speak where protesters can drown out his message with insults directed at him ; but I would prefer that he would give addresses and press conferences to the public more often .




fyi Nixon went right up to the protesters who were camped out at the Lincoln Memorial to engage in a dialogue with them .

CeeBee2 rated this answer Excellent or Above Average Answer
QueenChoux rated this answer Excellent or Above Average Answer

Question/Answer
excon asked on 08/26/05 - Sunni's


Hello experts:

Well, we don't need no stinkin Sunni's involved in Iraq, do we? Don't you just wish they would go away so that the country could write a constitution and split in two?

Well, they won't go away, and they WILL make a civil war. Who are the bunch of dead people they found with their hands handcuffed behind them? The (payback) killing has started, and our boys (and girls) are stuck right smack in the middle.

Come on, pretend that it isn't happening. Come on, pretend that we did a GOOD thing there. Come on, tell me how the women of Iraq are better off since we instilled Democracy. Come on, I haven't had a good laugh today.

And, the reason you righty's haven't said anything about it lately, is because you're afraid that I'm right.

excon

tomder55 answered on 08/26/05:

This week 3,500 members of the Iraqi security force graduated from training programs across the country.This brings the combined strength of the Iraqi army and police forces to almost 184,000 individuals.
One hundred percent of brigade-level operations this week were combined coalition and Iraqi security force operations, and Iraqi forces continue to show improved combat capabilities.

Iraqi leadership continues to work with all the political factions of the country in an effort to build a constitution that represents all of Iraq . When the constitution is submitted to the people of Iraq , the choice of democracy and freedom will rest in their hands.The press continuosely misrepresents the process by saying there is failure at each artifical deadline. There is a continuation of a process ;of negotiations ;of discussions . I shudder to think what it would've been like if our constitution was negotiated under the light of 24 hr. broadcasts. Madison destroyed his notes of our Constitional convention lest history learn how contentious it truely was.

The issues of federalism in Iraq are complex but not unresolvable or insurmountable .It is ultimately in the Sunni's interests to resolve their issues politically because they cannot succeed in a civil war.

excon rated this answer Excellent or Above Average Answer
Itsdb rated this answer Excellent or Above Average Answer

Question/Answer
QueenChoux asked on 08/24/05 - Americans Disgusted with Politicians

HE WALL STREET JOURNAL ONLINE
August 24, 2005

President Bush's job approval ratings are at their lowest point of his presidency as only 40% of U.S. adults have a favorable opinion of his job performance and 58% have a negative opinion, according to a Harris Interactive poll.

This is a decline from two months ago, when the president's ratings were 45% positive and 55% negative. The war in Iraq and the economy climbed to the top of a list of issues Americans say are most important for the U.S. to address. Social Security declined sharply.

At the same time, Vice President Dick Cheney's approval ratings slipped to 35% from 38% in June, while Secretary of Defense Donald Rumsfeld's approval ratings dropped to 40% from 42%. Secretary of State Condoleezza Rice is the only cabinet member whose approval ratings rose, to 57% from 52% in June.


Both Republicans and Democrats saw declines in their approval ratings. About one-third of adults gave a positive rating to Democrats in Congress, while 65% gave Democrats a negative rating. Republicans fared about as badly, with a 32% positive rating, down from a 37% positive rating in June.

Americans were also asked in the poll to name the two most important issues that the U.S. government needs to address. When considering the most important issues, 41% of those polled say the war is most important, sharply higher than 24% in June. The second most important issue is the economy, the poll showed.

Here are the results of the latest poll:
"How would you rate the job ______ are/is doing (excellent, pretty good, only fair, or poor)"

Comments.....

tomder55 answered on 08/25/05:

I rarely pay attention to polls ...too fickle. I also do not like pols. who stick their finger in the air to see how the wind is blowing . They tell us what they will do during campaigns and then I make a choice . If they make a 180 degree once they are in based on polling data I feel betrayed .

As I have pointed out ;some of our greatest politicans had what would've been pretty miserable polling numbers ;including 2 of my favorites ;Truman and Lincoln.

QueenChoux rated this answer Excellent or Above Average Answer

Question/Answer
QueenChoux asked on 08/24/05 - FCC and TV Media, Etal.

I am wondering, is if against FCC Regulations to call for the murder of an individual on a television broadcast?

If a person calls for the murder of an individual in America, isn't that assault? Who files a complaint...the person whose murder is called for. Does it matter if the person is not an American?

Is calling for murder "protected speech"??? Thanks in advance.

tomder55 answered on 08/25/05:

The 700 Club" is syndicated which makes it subject to Federal Communications Commission rules.It is also carried by the cable ABC Family Channel and satellite television, neither of which fall under FCC jurisdiction.The FCC has received numerous complaints about his stupid comments ;and therehave been some 'personalities ' who have called for them to fine Robertson.

Executive Order 12333 makes it against US policy to use assassination.

I do not think that the FCC will deny him license any more than they block the hate filled rants of screwy Loooy Farrakhan when he appears on local cable Muslim broadcasts .

QueenChoux rated this answer Excellent or Above Average Answer

Question/Answer
powderpuff asked on 08/24/05 - Mulsim leaders held to different standard than Xian leaders?

If an Islamic Imam suggested that his state or country assassinate the US President, he would (at least) be given the label of terrorist and dealt with as such. So why is it that a Christian leader can suggest that the US assassinate Chavez and his statements are brushed off with: "Private citizens say all kinds of things all the time"?

Is it because in the US we have free speech? Our speech isn't completely free, there are hate speech laws.

What is the difference between a Muslim leader calling for the assassination of a foreign leader and a Christian leader calling for the assassination of a foreign leader?

tomder55 answered on 08/24/05:

Hate speech laws are in my opinion unconstitutional .However there are laws designed to prevent incitement.

A Muslim leader is rarely if ever officially critized by our government for making these type of declarations even thhough they often do so on a routine basis.

What is the difference between a Muslim leader calling for the assassination of a foreign leader and a Christian leader calling for the assassination of a foreign leader ? None really ;both are inappropriate .

XIAN is a famous city in China . Did you mean Christian ?

Itsdb rated this answer Excellent or Above Average Answer
powderpuff rated this answer Excellent or Above Average Answer

Question/Answer
excon asked on 08/23/05 - Iraq - DISASTER


Hello wingnuts:

So, how's it gonna feel to know that we fought a war to create an Islamofacist state?

Our friends, Turkey are now gonna have to fight a war in the North with the Kurds, and our boys are stuck in the middle of a civil war between two factions of Arabs who despise the other.

What the hell are we doin???? And, is your great leader goin on another bike ride today???? Puleese!!

excon

tomder55 answered on 08/23/05:

Coming up with a Constitution is a tremendous step forward for Iraq . It's a funny thing ; Everyone said "you are imposing your way of life on them ".But when they draft a Constitution that is not Jeffersonian then the same critics are seemingly opposed to self determination . So be it . I think this puts to rest once and for all that Iraq is America's puppet,or colony .They are free ,and they've got to be allowed to make their own decisions. That's one of the reason we stayed on in Iraq, to help them become a democracy. Thay are making progress .

Here is the draft Constitution text (some highlights of note ):

Article Two

The political system is republican, parliamentary, democratic and federal.

1. Islam is a main source for legislation.

-- a. No law may contradict Islamic standards.

-- b. No law may contradict democratic standards.

-- c. No law may contradict the essential rights and freedoms mentioned in this constitution.



Article Seven

1.Any organization that follow a racist, terrorist, extremist, sectarian-cleaning ideology or circulates or justifies such beliefs is banned, especially Saddam's Baath Party in Iraq and its symbols under any name. And this should not be part of the political pluralism in Iraq.

2. The government is committed to fighting terrorism in all its forms, and works to protect Iraqi soil from being a center or passage for terrorist activities.



Article 36

The State guarantees:

1. Freedom of expression by all means.

2. Freedom of the press, printing, advertising and publishing.





Article 39

Iraqis are free to abide in their personal lives according to their religion, sects, beliefs or choice. This should be organized by law.




Article 135

This constitution guarantees the administrative, political, cultural and educational rights of different ethnic groups such as Turkomen, Chaldean, Assyrians and other groups.




Article 151

No less than 25 percent of Council of Deputies seats go to women

(the US House and Senate should be so represented )

A quick look at the doc shows me that they weaken the central gvt. in favor of a looser federalist system .The drafting committee left it up to the National Assembly to sort out issues including specifics on regional rights.

The text calls for liberties such as freedom of expression and the press. It gives Islam a role in national affairs, while offering Iraqis the option of following civil code in areas such as marriage, divorce and inheritance.On the role of religion in legislation, the constitution calls Islam "a main source" of legislation instead of "the main source". This is to be expected and it does not go nearly as far as a theocracy would.

The constitution is a set of compromises. It won't make extreme Islamists happy and it won't please secular democrats. But it acknowledges the beliefs and aspirations of Iraqis have to be pursued through a democratic system of government. In the Arab world, that is truly revolutionary. No wonder Al Qaeda and the Baath thugs fear the constitution. People speak of Islam and democracy being incompatible. The constitution shows they may have a chance to coexist.


We are a Western nation that has over 200 years of self rule under our belt . We have yet to form that elusive "more perfect union ". Before the Constitution there was The Articles of Confederation in effect, the first constitution of the United States. Drafted in 1777; operative on March 1, 1781 it was a flawed document that lasted until the new Constitution was drafted September 17, 1787 and took effect in 1789. Our founders who we now revere in almost super human status gave us a flawed government intially that lasted over 8 years and when they forged a 2nd Constitution they evaded the most divisive issues of the day .

Are there areas that could be improved in the draft ? Sure there are . Are there areas of disagreement amongst the Iraqis over the details ;of course . Just like Americans today cannot agree on the need for amendments and the meaning of our Constitution.

Is the role of religion simular to the debate the founders had over slavery ? I think so . Our founders solution was to punt the ball for other generations to decide . I think the debate about the role of religion in the law and society will be as lively in Iraq as it is here .

Where you see disaster ;I see accomplishment.






excon rated this answer Excellent or Above Average Answer
Itsdb rated this answer Excellent or Above Average Answer
paraclete rated this answer Excellent or Above Average Answer
QueenChoux rated this answer Excellent or Above Average Answer

Question/Answer
excon asked on 08/22/05 - Protest 'emboldens' our enemy.


Hello experts:

I read, today, the following typical stuff: "The enemy loves the fact that folks are marching in the same way the Viet Cong loved the tie-dyed peace flags. It fuels the fire with the enemy and tears down support for the country and destroys the troops morale."

Where does that come from? Has anyone ever interviewed an enemy to find out if this is so? Why would exhibiting one of the basic tenets of freedom "the right to protest", embolden our enemy? Certainly, you don't believe that everyone from their side supports them - or do you? If you saw that they didn't, would it make you believe in your side even MORE?

I was a 'troop' once. Frankly, what the people did back home had nothing whatsoever to do with my job. Can any veteran tell me any different? How does an "emboldened" enemy act differently than a non emboldened enemy does?

So, if that stuff isn't true (and I don't believe it is), where did it come from?

excon

tomder55 answered on 08/22/05:

Where does it come from ? Right from the mouths of our enemies .

Gen. Vo Nguyen Giap of N.Vietnam wrote in his 1985 account of the warHow we won the war

;"What we still don't understand is why you Americans stopped the bombing of Hanoi. You had us on the ropes. If you had pressed us a little harder, just for another day or two, we were ready to surrender! It was the same at the battles of TET. You defeated us! We knew it, and we thought you knew it. But, we were elated to notice the media were definitely helping us. They were causing more disruption in America than we could in the battlefields. Yes, we were ready to surrender. You had won!"

North Vietnamese Col. Bui Tin, who served under Gen. Giap on the general staff of the North Vietnamese army, received South Vietnam's unconditional surrender on April 30, 1975.

In an interview he credited leaders of the U.S. anti-war movement, saying they were "essential to our strategy."


Q: Was the American antiwar movement important to Hanoi's victory?


A: It was essential to our strategy. Support of the war from our rear was completely secure while the American rear was vulnerable. Every day our leadership would listen to world news over the radio at 9 a.m. to follow the growth of the American antiwar movement. Visits to Hanoi by people like Jane Fonda, and former Attorney General Ramsey Clark and ministers gave us confidence that we should hold on in the face of battlefield reverses. We were elated when Jane Fonda, wearing a red Vietnamese dress, said at a press conference that she was ashamed of American actions in the war and that she would struggle along with us.

Q: Did the Politburo pay attention to these visits?


A: Keenly.




Q: Why?


A: Those people represented the conscience of America. The conscience of America was part of its war-making capability, and we were turning that power in our favor. America lost because of its democracy; through dissent and protest it lost the ability to mobilize a will to win .






In fact ;John Kerry's picture hangs in the Vietnamese communist's war museum in Saigon in which he is immortalized in tribute to aiding the communists in winning the war.His anti-war protests were critical .


excon rated this answer Excellent or Above Average Answer
LTgolf rated this answer Excellent or Above Average Answer
QueenChoux rated this answer Excellent or Above Average Answer

Question/Answer
excon asked on 08/18/05 - Iraq


Hello:

Those who support the war in Iraq firmly believe that it is a continuation of the War on Terror. Those against it don't see any connection between the two.

This isn't a minor difference of opinion. How can each side be so far from the other?

excon

tomder55 answered on 08/22/05:

I just would like to point out this article in a Middle East publication dated March 16 2001 (before 9-11) :

Iraqi Spies Reportedly Arrested in Germany
16 March 2001

Al-Watan al-Arabi (Paris) reports that two Iraqis were arrested in Germany, charged with spying for Baghdad. The arrests came in the wake of reports that Iraq was reorganizing the external branches of its intelligence service and that it had drawn up a plan to strike at US interests around the world through a network of alliances with extremist fundamentalist parties.

The most serious report contained information that Iraq and Osama bin Ladin were working together.
German authorities were surprised by the arrest of the two Iraqi agents and the discovery of Iraqi intelligence activities in several German cities. German authorities, acting on CIA recommendations, had been focused on monitoring the activities of Islamic groups linked to bin Ladin. They discovered the two Iraqi agents by chance and uncovered what they considered to be serious indications of cooperation between Iraq and bin Ladin. The matter was considered so important that a special team of CIA and FBI agents was sent to Germany to interrogate the two Iraqi spies.
Somehow the 9-11 Commission forgot or decided to ignore this rather pertinent bit of information when they published their report .

To answer your question :why ? It appears to me that it takes an allfull lot of digging to discover these nuggets of information ;and when it does come out ;authors like Stephen Hayes and Laurie Mylroie are subject to a MSM hatchet jobs.





excon rated this answer Excellent or Above Average Answer

Question/Answer
QueenChoux asked on 08/17/05 - Cheater's Proof

"President Bush's campaign against what he once termed the "axis of evil" has suffered reverses on all three fronts in recent days that underscore the profound challenges confronting him 3 1/2 years after he vowed to take action.

First, multilateral talks orchestrated by the United States to pressure North Korea to give up nuclear weapons adjourned last week after 13 days without agreement. Then Iran restarted its program to convert uranium, in defiance of the United States and Europe. Finally, negotiators in Iraq failed to draft a new constitution by Monday's deadline amid an unrelenting guerrilla war against U.S. forces."

All the spin and lies and propaganda that the Republicans have pumped out over the last years have come back to haunt them by way of the old Sport's Adage, "Cheater's Proof". If you lie to win, you will eventually be exposed and humiliated.

Ah, the good old days when Bill Clinton was President. Compared to now, that was Paradise!!

tomder55 answered on 08/22/05:

always easier to appease and surrender.

QueenChoux rated this answer Poor or Incomplete Answer

Question/Answer
QueenChoux asked on 08/18/05 - Facing Reality

ANCHORAGE, Alaska (AP) - Anyone doubting the effects of human activity on global climate change should talk to the people it affects in Alaska and the Yukon, U.S. Sen. John McCain said Wednesday.

Fresh from a trip to Barrow, America's northernmost city, McCain said anecdotes from Alaskans and residents of the Yukon Territory confirm scientific evidence of global warming.

"We are convinced that the overwhelming scientific evidence indicated that climate change is taking place and human activities play a very large role," McCain said.

McCain, accompanied by Sens. Hillary Rodham Clinton, D-N.Y., Susan Collins, R-Maine, and Lindsey Graham, R-S.C., spoke to villagers in Canada whose spruce trees are being attacked by the northward spread of spruce beetles. On Alaska's northern coast, they met Native Alaskans dealing with melting permafrost and coastal erosion.

"I don't think there is any doubt left for anyone who actually looks at the science," Clinton said. "There are still some holdouts, but they are fighting a losing battle. The science is overwhelming, but what is deeply concerning is that climate change is accelerating."

Isn't it time that all American politicians face reality like Senator McCain and Senator Clinton? It is the future of our grandchildren that is on the line.

Comments?

tomder55 answered on 08/22/05:

and Australia had record snowfalls this month . What does it mean ? Climate change ALWAYS takes place . The only question is ;how much of it is human induced ?

QueenChoux rated this answer Poor or Incomplete Answer

Question/Answer
QueenChoux asked on 08/18/05 - Senators Rated

Rick Santorum has the lowest net approval rating of any U.S. Senator, according to a poll by Survey USA. The survey reveals 42% approve of the job Santorum is doing, while 46% disapprove. The difference gives Santorum a 4% approval rating.

Survey USA collected results of fifty separate public opinion polls for Senators in all fifty states. The average approval rating was 56% and the disapproval rating was 32%.

The two highest rated Senators were the two Senators from Maine.

Barak Obama of Illinois rated third with an approval rating of 71%.

Looks like Santorum's senate seat is totally in jeapardy. A swing to the Democratic column.

Comments?

tomder55 answered on 08/22/05:

I think Santorum survives ;but I would not place a bet on the two Ohio Republican Senators ;Dewine and Voinovich . The Ohio Republicans are in trouble statewide .Dewine was one of the group of 14 Senators that sold out on Bush's judicial nominees .He will face a primary challenge at least . Voinovich's seat is not up yet but I for one will not forget his teary eyed opposition to Bolton .

The news in NY is that Hillary will face a serious challenge for her seat. If she runs for reelection she will still win but it will cost her big bucks;more than what she wants to spend just to defend her seat given that her war-chest is targetted for a Presidential run.

Santorum will have a primary challenge from John Featherman ;a nobody who is a north-east Republican (RINO) who Santorum will easily defeat . The big race will be against Democrat state Treasurer Robert P. Casey Jr. He is choosing to make the race a referendum on Iraq which will be the game plan for most Democrats in 2006 . I think if he tried to challenge Santorum on his social conservatism he would lose big time.

QueenChoux rated this answer Excellent or Above Average Answer

Question/Answer
QueenChoux asked on 08/18/05 - Future Oil Prices

Goldman Sachs Group Inc., the third- biggest U.S. securities firm by market value, raised its oil forecast for next year to $68 a barrel and said crude will stay at about $60 for years to come.

Goldman increased its projection for New York prices for 2006 from $55 a barrel. The forecast for $60 oil for the ``long term'' was raised from $45 because companies aren't investing enough in new supply, according to the Aug. 17 report from the firm's commodity research analysts, including Steve Strongin in New York and Jeffrey Currie in London.


MISSION ACCOMPLISHED!!!!


Comments?

tomder55 answered on 08/22/05:

sorta blows the theory that we went to war to grab Iraqi oil out of the water ;in fact I find no relevence between Iraq war and oil prices . Demand is up ;supply is shrinking and refining capacity stalled due to a combination of environmental restraints and NIMBY . All told however we are still not quite where oil prices should be with inflation adjusted bucks from the early 80s ;but it should be clear that the days of cheap gas is over . Have we reached crisis levels so our elected pols can get off their asses yet ?

QueenChoux rated this answer Excellent or Above Average Answer

Question/Answer
QueenChoux asked on 08/20/05 - General Schoomaker Speaks

WASHINGTON - "The Army is planning for the possibility of keeping the current number of soldiers in
Iraq well over 100,000 for four more years, the Army's top general said Saturday.


In an Associated Press interview, Gen. Peter Schoomaker said the Army is prepared for the "worst case" in terms of the required level of troops in Iraq. He said the number could be adjusted lower if called for by slowing the force rotation or by shortening tours for soldiers.

Schoomaker said commanders in Iraq and others who are in the chain of command will decide how many troops will be needed next year and beyond. His responsibility is to provide them, trained and equipped.

About 138,000 U.S. troops, including about 25,000 Marines, are now in Iraq."

Any comments or additiona?

tomder55 answered on 08/22/05:

this appears to be nothing new.they plan for many contingencies and more often than not history shows that the unexpected determines the ultimate course of action.there is some logic in keeping the troop level as it is and then supplementing them with additional Iraqi forces as they deploy .But ;politics in this country as it is ;support wavering ;I suspect there will be troop reductions before the goal is realized . I guess OBL and Saddam learned the lesson of Mogadishu well.

QueenChoux rated this answer Excellent or Above Average Answer

Question/Answer
purplewings asked on 08/16/05 - Where is Tomder? Does anyone know?

I haven't seen him around lately and I miss his input. I Hope he isn't sick?

PW

tomder55 answered on 08/21/05:

I'm back from vacation . Will become more involved in the next couple of days after I catch up at home/work.

purplewings rated this answer Excellent or Above Average Answer

Question/Answer
excon asked on 08/11/05 - Iraq - Quagmire


Hello experts:

Do you know what our plan is to win the war? I don't either. "Staying the course" when the course isn't working, is not a plan.

Here's what John McCain said about Vietnam: "It was a shameful thing to ask men to suffer and die, to persevere through god-awful afflictions and heartache, to endure the dehumanizing experiences that are unavoidable in combat, for a cause that the country wouldn't support over time and that our leaders so wrongly believed could be achieved at a smaller cost than our enemy was prepared to make us pay."

Kinda sounds like this war, doesn't it?

excon

tomder55 answered on 08/12/05:

Mao said that an insurgency is like fish in the water . To an outside observer all the fish look alike. The idea is to get the people to actively support the insurgency ;or to strike enough fear into the people to not oppose the insurgency .Fear is inflicted by assassinations, bombings and other terrorist acts.

Mao also said the way to defeat an insurgency is to dry up that lake .If the enemy (the coalition and the Iraqi government) can convince the people to overcome their fears and support them then the insurgents freedom to move and operate is steadily reduced . The way to do this is to provide services that the people lacked under the murderous regime of Saddam . Civil services under Saddam were not much better than they are now and in fact services in many parts of Iraq now are enormously better than they ever were under Saddam. Thats drying up the lake in which the Iraqi insurgencies swim. That takes care of the domestic insurgency . In time with a firm commitment the counter-insurgecy against Saddamits cannot fail .

The other insurgency is more of an invasion force of jihadists who are infiltrating the border . The evidence suggests that the people are turned off of their tactic of blowing up civilians in an attempt to foster a civil war.Although I think the US forces have put a big hurt on the ones foolish enough to cross the border I do not think that will be resolved until enough Iraqis are trained to patrol inside the country .That would free up our troops to strat draining the swamp in Syria and Iran. On the otherhand ..our Air force is not too busy right now .......

Re :Vietnam . The Vietnamese lost their independence by an invasion from the North.Part of the fault clearly lies with the people not willing to defend their freedoms but a large part of the blame lies in the fackless US Congress and their defunding Vietnamization which was clearly working .

Re.Afghanistan .One of OBL's biggest complaints is that the Gulf States have all this wealth ;but that it does not trickle down to the people . But what did he and his cronies the Taliban do when they had Afghanistan to run ? They combined normal gvt. inefficiency with domestic fascism and made the lives of ordinary Afghani a living hell. Islamo-fascism both religious and secular is weak . The people of both Iraq and Afghanistan are beginning to get the sniff of freedom and they like what they smell.....

Our armed forces will fight for peace in Iraq, a peace built on more secure foundations than are found today in the Middle East. Even more important, they will fight for two human conditions of even greater value than peace: liberty and justice.
John McCain

excon rated this answer Excellent or Above Average Answer
LTgolf rated this answer Excellent or Above Average Answer

Question/Answer
powderpuff asked on 08/11/05 - doing a good job?

Headlines News today

Serious about deportations:

British authorities said Thursday they plan to deport 10 foreigners who've been detained on suspicion of posing a threat to national security.

~~~~~~~~~~~~~~~~~~~~~~~~~~~~~~~
PAPERWORK MIX-UP
Deportation sought for couple in West Toledo
U.S. says pair here illegally since 1996....

In the predawn semidarkness of a small West Toledo apartment, Dae and Yung Jung stumbled toward the thumping at their front door.

Seconds later, officers in dark jackets emblazoned with Homeland Security crammed into the couple's living room demanding passports and drivers' licenses. Mrs. Jung was escorted to jail. Upstairs, the couple's son, Andrew, hid, stunned and baffled.

Dae Jung is a sushi chef; Yung Jung is a longtime school volunteer. Andrew, 14, is a U.S. citizen because he was born in Toledo while Mr. Jung was studying at the University of Toledo on a student visa.

Mrs. Jung has been behind bars in a string of Michigan prisons since U.S. Homeland Security officers raided the couple's home in February. Acting on deportation orders signed in 1996, they arrested Mrs. Jung but released Mr. Jung to care for Andrew.

Most frustrating for the Jungs' supporters is that the couple was never allowed court hearings into the matter.

At about 2:35 p.m. today, the Jungs will board Northwest Airlines Flight No. 25 for a 7,157-mile flight to Incheon, South Korea. Andrew, who plays violin with the Toledo Youth Orchestra and yesterday tried out for the golf team at Emmanuel Baptist, will remain in Toledo.

You can read more on the story behind the Jungs deportation at ---> http://toledoblade.com/apps/pbcs.dll/article?AID=/20050810/NEWS02/508100399&SearchID=73216928770131

~~~~~~~~~~~~~~~~~~~~~~~~~~~~~~~~

I'm pretty sure that there are bigger threats to our national security here, than the Jungs. Is our country doing its best to protect us, or is PC preventing us from finding the real threats? Are the British doing a better job of finding and deporting national security threats?




tomder55 answered on 08/11/05:

Hard to find info. about this. I kept on getting Kim Dae Jung President of South Korea .

I risk sounding heartless but here it goes :

On the surface they appear to be hard working and honest people .For everyone of them there are also probably millions who would also like to come here .Their place in line gets cut into by illegals who do not respect our laws .Were they here illegally, and did they know it? I cannot tell for sure. It seems that they should've been able resolved issues of a SNAFU with the INS by now from 1996.

INS since the homeland security rearrangement does not exist anymore . It has been split up to Immigration and Customs Enforcement (ICE ) and Customs & Border Protection (CBP).The ICE probably did the raid .

ICE is focusing its enforcement efforts on illegal immigrants that fall into two categories: criminals and absconders;or in other words immigrants that have skipped court dates which results in a deportation orders or gone to court and been ordered to be deported but have failed to leave.They missed the date in 1996 ;true and they had 9 years to fix it ;but also he lied on the visa by saying that he intended to go to school. That was the very same thing that the 9-11 hijackers did.

My gut feeling is that these people are good folks who have just been caught in a bad situation. The INS was notoriously slow ,and hopefully the process has been changed to expedite . In this case it is easy to say that we overlook so many cases and that should be done here also . I on the other hand am in favor of closing the loop-holes that allow illegals into the country .

powderpuff rated this answer Excellent or Above Average Answer

Question/Answer
excon asked on 08/10/05 - W


Hello experts:

I think W is going to go down as the worst president in history, even worse than Nixon. I think most of the people in the country agree with me, now.

Do you, even his ardent supporters here, have anything good to say about him? Come on, it's ok to change your mind.

excon

PS The problem is that the Democrats don't have anybody better. I like John McCain.

tomder55 answered on 08/11/05:

believe it or not ;with 3 1/2 years left in his 2nd term it is still too early to make a determination .

My early comparison is with Harry Truman . He barely survived re-election in 1948 ;would not have won in 1952 ;made a ton of unpopular decisions ;was very plain spoken and not very eloquent ;shaped and made the blue print for a prolonged struggle against an ideological enemy. History has been very kind to him even though many thought unfavorably of him while he was in office .

excon rated this answer Excellent or Above Average Answer

Question/Answer
Itsdb asked on 08/09/05 - NCAA finally goes off deep end

By BEN COOK
Scripps Howard News Service
August 09, 2005

- A month or so ago I wrote a column warning that the NCAA was about to attack nicknames again, and sure enough it has come to pass.

I had hoped that enough people would be up in arms after reading my column and descend on the NCAA with placards and other implements of discord telling the lame brains that run the NCAA to leave well enough alone. But it didn't happen and now teams like the Florida State Seminoles, Illinois Fighting Illini and Utah Utes have been branded by the NCAA as "hostile and abusive" to American Indians.

Except for a handful of people, most native American Indians did not realize these teams were being hostile and abusive; most felt a team carrying a Native American nickname was doing so as a term of honor and respect.

Silly people.

It is a perfect example of a small group of dissenters stirring a pot that's being watched over by a group of people who believe since they have achieved various levels of education that they are therefore smart. But as the NCAA has demonstrated over and over again, one does not necessarily go along with the other.

The directive is aimed at the 18 schools that have Indian nicknames, Florida State and Illinois being two of the most visible. The schools have to either change their name or remove all references to the nickname (such as the spear on the FSU helmets) and keep mascots off the field. The ban also applies to cheerleaders, marching bands and dance teams.

If these schools do not fall in line, they will not be invited to participate in any NCAA-sponsored championship events.

Do you happen to remember who won the most lucrative event in the NCAA last season? It was almost the Illinois Fighting Illini, who lost to North Carolina in the NCAA basketball title game back in April. This decision goes into effect in February. Can you imagine the firestorm that will erupt if the Fighting Illini are in position to win their first NCAA title in men's basketball but the NCAA doesn't invite them to the tournament?

More to the point, can you imagine the Illinois lawyers who will get rich tearing into the NCAA in court like hungry wolves on a wounded animal?

The NCAA also says no team with any Indian name will ever be allowed to host any NCAA championship event, although anything that is already scheduled will be permitted as long as the schools cover anything that refers to its abusive and hostile name.

Presumably, any native American students who attend participating schools will be properly covered and not allowed to show their faces during the championship event.

The Seminole tribe in Florida is as upset over the ruling as Florida State. The two groups have worked together over the years to most accurately show the FSU mascot in a manner that best exemplifies the "spirit of the Seminole Tribe of Florida," according to FSU president T.K. Wetherell.

Over a dozen schools have changed nicknames and logos over the past 15 or 20 years, bowing to pressure from small groups of protesters. That's fine. If any of the remaining 18 schools want to roll over and fly in the face of their own traditions, that's their business.

What I don't want to see happen, though, is for all 18 to change against their will just because Myles Brand and his uptight band of stuffed shirts held them hostage, which is exactly what they are doing with the threat to withhold the right to participate in NCAA championships.

Universities need to draw the line in sand. It's time to end Myles Brand's era of ineptitude and get the NCAA out of the hands of academicians and put athletic people in charge of college athletics. Let the academicians go back to their classrooms. The business of college athletics is beyond their capabilities.

~~~~~~~~~~~~~~~~~~~~~~~~~~~~~~~~~~~~~~~~~~~~~~~~~~~~~~~

Florida State didn't take kindly to this:

"That the NCAA would now label our close bond with the Seminole people as culturally 'hostile and abusive' is both outrageous and insulting," Florida State president T.K. Wetherell said in a statement.

"I intend to pursue all legal avenues to ensure that this unacceptable decision is overturned, and that this university will forever be associated with the 'unconquered' spirit of the Seminole Tribe of Florida," he added.

~~~~~~~~~~~~~~~~~~~~~~~~~~~~~~~~~~~~~~~~~~~~~~~~~~~~~~~

PC police run amok or good decision by the NCAA?

tomder55 answered on 08/10/05:

Choux ....fighting Irish ...no offense taken as none is intended .

ex. ..in this case FSU has for years had a relationship with the Seminole Indians .Clearly there is no intent to disparage. When the Cleveland Indians had a mascot 'Chief Knockahoma' that went too far as he was a charictature ,the St.Johns 'Red men 'changed their name because it was insulting .But this is not the case with FSU where the Seminols Tribal Council has passed a resolution supporting FSU's use of the "Seminole" name .

Itsdb rated this answer Excellent or Above Average Answer
QueenChoux rated this answer Excellent or Above Average Answer

Question/Answer
excon asked on 08/09/05 - Security????? Pure, Unadulterated BS


Hello:

For some time, here, Ive been mentioning that ALL the security weve instituted, and ALL the billions weve spent on it, are wasted. Having been in the security industry, I know that people buy what makes them feel safer - not what actually makes them safer.

It's a pretty stupid thing to do. I dunno why we do it. Lets take the subway searches now going on in NY. In the highly unlikely event that a terrorist with a bomb is selected for a search, he can simply say no and exit the system with no questions asked. It has to be that way if the city is going to argue in court that the searches are voluntary (a dubious claim, given how important the subway is to the average New Yorker).

Upon leaving the subway, a terrorist unlucky enough to be picked for a bag check can try again at another station, hand his bag off to an accomplice, or detonate his bomb at a crowded location above the ground. It should not be hard to find one in New York City.

Im telling you, America: Stop being stupid!

excon

tomder55 answered on 08/09/05:

you're right;random voluntary searches are useless . It should not be voluntary . Anyone who tries to use public transportation should be subject to the possibility of being searched . There should also be profiling to limit the scope but since the ACLU would be screaming if that was instituted then random searches is the only option left .

It is not unreasonable to protect oneself from death at the hands of another . The whole 2nd amendment is predicated on that fact .

If searches have already been deemed legal and reasonable at airports, where is the legal argument against them for subway use ? Is subway system to be exempt from the rules while public air transportation is not?

To those who walk around saying they choose not to be searched I say fine ,find some other way to get to work. I chose not to be blown up due to the lack of an adequate prevention.

If a bomb goes off and these precautions were not instituted I can guarantee that demands for explanations will happen ;followed closely by the law suits .

excon rated this answer Excellent or Above Average Answer
Itsdb rated this answer Excellent or Above Average Answer
LTgolf rated this answer Excellent or Above Average Answer

Question/Answer
QueenChoux asked on 08/08/05 - New Market for American Banks

The banking industry is opening its doors to a controversial new market: illegal immigrants.

Despite heated political debate in Washington over illegal immigration in the United States, an increasing number of banks are seeing an untapped resource for growing their own revenue stream and contend that providing undocumented residents with mortgages will help revitalize local communities.

What are your thoughts on this bold move by some American Banks?

tomder55 answered on 08/09/05:

I wonder how they plan to collect when an illegal defauts ? It illustrates the contradiction in US policy . If they have no right to be here ;it should be illegal to extend them credit ;they should not be catered to in any way . All this does is to encourage futher infiltration .

What's worse is that the FDIC and the the Mortgage Guaranty Insurance Corp. is now formally encouraging U.S. banks to enter this largely untapped market,.For you and I to qualify for a mortage we need a solid credit and income history .These normal requirements are being bypassed. In many cases all they need is a Individual Tax Identification Number (ITIN)to qualify for the loan.

U.S. Criminal Code 274: It is a crime punishable by 10 years in jail for aiding and abetting someone in this country illegally for commercial gain. The Bank Secrecy Act of 1972 :Banks must know their customers and any illegal activity must be reported to the government." The quisling banks involved must therefore be complicit in illegal activity and they have added new meaning to 'mortaging our future. '

purplewings rated this answer Excellent or Above Average Answer
QueenChoux rated this answer Excellent or Above Average Answer

Question/Answer
excon asked on 08/08/05 - Lobbying


Hello sperts:

Lobbying should be outlawed. Is it true that congress wouldn't know what to do without 'em?

excon

tomder55 answered on 08/08/05:

Lobbying is the process of educating lawmakers to help them understand a specific cause and to influence them when they are making decisions on specific legislation .


Lobbying should be outlawed

What hapened to fidelity to the 1st amendment ? ;or maybe it is only non-profit lobbies like NORML that should be permitted ? Would a single pot smoker have any influence without joining together with outher like minded stoners pooling their resources and voices to try to influence legislation ?
It is true that in many countries lobbying is a regulated activity ,and this country is drifting in that direction with laws like McCain -Feingold .In the US lobbyists need to register since 1995 except governments groups like the National Govenor's Assc. (yes they are a lobby group also)


Is it true that congress wouldn't know what to do without 'em?


probably true . they would certainly miss all those wanderers of K Street offering free lunches.




excon rated this answer Excellent or Above Average Answer

Question/Answer
jnlomonte asked on 08/07/05 - Rove Rage

From The Dallas Morning News, 8/7/2005

Ex-GOP big Tom Pauken wonders why conservatives aren't demanding answers.

Why have so many conservatives rushed to the defense of White operatives Karl Rove and and Lewis "Scooter" Libby, who are under investigation for their roles in the possibly illegal leaking of CIA operative Valerie Plame's name?

This scandal began in 2002, when the CIA sent former Ambassador Joseph Wilson to Niger to investigate claims that Saddam Hussein was trying to acquire WMD materials there. He concluded the charges were untrue. President Bush subsequently cited Niger as a "casus belli" in his 2003 State of the Union address, stating that "the British government has learned that Saddam Hussein recently sought significant quantities of uranium from Africa." After this, Mr. Wilson went public disputing the administration's claims.

Unnamed Bush officials then launched a counterattack against Mr. Wilson, challenging his credibility. Part of that counterattack possibly included the "outing" of Ms. Plame as a CIA operative in stories about the controversy. We now know that the president's top political advisor, Mr. Rove, and Mr. Libby, chief of staff to Vice President Cheney, talked to at least one reporter, Time magazine's Matthew cooper, about Ms. Plame's work as a CIA analyst.

Most of the attention to date has been focused on Mr Rove's participation in the campaign to discredit Mr. Wilson because Mr. Rove has a long history of using "slash and burn" tactics to go after political foes. But theinvolvement of Mr. Libby--national security advisor to Mr. Cheney and a close associate of former Defense official Paul Wolfowitz, widely credited as the architect of the Iraq war--may be more significant.

That's because one of the main charges against Mr. Wolfowitz and other administration neoconservative hawks is that they used faulty intelligence, information they knew to be of dubious credibility, to sell the president and Congress on the war. It is a perennial problem in the intelligence business when policymakers have reached a conclusion already and only want "intelligence" that will bolster their position. Any intelligence that throws into doubt the conclusions they already have reached has to be discarded or discredited. That's why Mr. Wilson's credibility had to be undermined.

Predictably, the controversy has taken a partisan tone, with Democrats and liberals using the issue to go after Mr. Rove and the president. Meanwhile, a number of conservatives publications, talk show hosts and spokesmen like GOP chairman Ken Mehlman have adopted a "party line" that goes like this: "Ambassador Wilson and his wife are partisan Democrats, and Ms. Plame wasn't on covert status at the time her identity was revealed."

But a recent Washington Post report revealed taht as he was boarding Air Force One, Secretary of State Colin Powell was given a classified document after Mr. Wilson's op-ed ran that pointed out his wife's role at the CIA. That document was marked "S" for secret, indicating that her status was still covert. Seven days after Mr. Powell received it, conservative columnist Robert Novak outed Ms. Plame as a CIA operative.

Was there a connection? Conservatives should be clamoring for answers.

Lest we forget, it was conservatives who pushed the hardest for passage of the 1982 statute designed to protect U. S. spies. Known as the "Philip Agee" law, it was put in place to discourage people like Mr. Agee (a disaffected leftist who had been in the CIA) from outing clandestine CIA operatives and jeopardizing operations. Whatever Ms. Plame's status, administration officials had no business playing fast and loose with her identity as a CIA agent. Ex-CIA analyst Larry Johnson has a point when he wonders why no Republican lawmaker has had the courage to stand up and "speak out against them."

National security is too important for partisan gamesmanship. This principle is even more important the observe when it comes to waging war, the gravest act any nation can undertake. Patriotic conservatives ought to be concerned abour whether Mr. Bush, Congress and the American people may have been misled into the Iraq war by influential persons manipulating intelligence data to justify that action. One conservative Republican congressman, Rep. Walter Jones of North Carolina, believes he was duped and is now calling for an Iraq exit strategy.

Isn't it in our national interest to fine out who was responsible for that misinformation rather than simply take the Republican party line on the Plame affair? Conservatives must decide which is more important: the good of our country or the good of the Republican Party.

==========
Tom Pauken, a Dallas businessman, served in the Reagan administration and was a military intelligence officer in Vietnam. He can be emailed at twpauken@sbcglobal.net

**********

Isn't it interesting that all rightwing conservatives aren't kneejerk Rove apologists.

John

tomder55 answered on 08/08/05:

in another case of the CIA leaking information that could run against Bush policy ,it was reported last week that Iran was 10 years away from nukes. these leaks by the CIA are meant to soley undermine policy . Why are they not worse then the naming of a CIA desk jockey ?

The fact that the leak that the Washington Compost reported is just plain bogus is besides the point .It doesn't take a genious to figure out that after we got nukes ;all our allies and Russia acquired them in a few years ;that AQ Kahn was giving nuke secrets to anyone with cash ;that Pakistan ;the next door neighbor of Iran has them .The Post dutifully reported it as written on tablet ;this classified intelligence report that was not meant for the public to see. I guess they got off their high horse of indignant outrage of the alleged leaking of a name long enough to wallow in the information from 'un-named sources ' . I just wonder if Joe Wilson is the source . Did he go to Tehran and sip mint tea with the Mullahs ?

Itsdb rated this answer Excellent or Above Average Answer
jnlomonte rated this answer Excellent or Above Average Answer

Question/Answer
excon asked on 08/05/05 - Roberts - Supreme Court Dude


Hello politicos:

Rush Limprod doesn't like Roberts. That's good enough for me. Put him on the court.

excon

tomder55 answered on 08/06/05:

I think Roberts has some splainin to do . Usually pro-bono work is of little consequence;but he gave ammo to the wrong side of one of the worse recent Supreme Court decisions;Romer vs. Evans. His work was crucial .Rush's actual concern was for the underlying legal point that Roberts' work undermined in that he fought to judicially overturn a ballot initiative voted on and approved by the citizens. That is exactly the kind of judicial activism true conservatives don't want to see on any court, much less the Supreme Court, whether or not it has anything to do with homosexuality.

Roberts did not mention the case in his 67-page response to a Senate Judiciary Committee questionnaire that was released Tuesday even though the committee had asked for specific instances in which he had performed pro bono work.
Perhaps he forgot his role in the decision ?Roberts was not the chief litigator so it is possible that he indeed forgot his contribution.

I dont expect to see any significant fall out from it except from possibly the Ann Coulter/Michale Savage Conservatives. He was the lead appellate attorney in a firm of 1000 lawyers and this was part of his job.He has done over 50,000 hours of legal work ;this case took up about 6 hrs. of it. I prefer to have everyone calm down and examine the total body of his work before passing judgement .

excon rated this answer Excellent or Above Average Answer

Question/Answer
excon asked on 08/05/05 - Robert Novak - dufus

Hello right wingers:

Soooooo, what do you think of your wonderful spokesperson, Novak now?

Saying bullshit on national TV, and during the daytime too, when young children could be watching. He's got a filthy mind.

excon

tomder55 answered on 08/06/05:

I never thought much of Novak to begin with . His columns have an underlying anti-semitism to them . On CNN he has acted much like Alan Colmes on Fox ;the "conservative" foil. Usually he is sitting on a panel with 3 libs on 'Crossfire' and 'The Capital Gang 'who make personal comments about him and he sits back with a smile and takes it .

I heard the exchange that prompted his tantrum . James Carvelle was interupting him and challenged him ;questioning his "backbone". I probably would've hammered Carvelle's cue-ball head first before I walked out.

CNN has suspended him . I do not recall CBS suspending Dan Rather when he had a hissy-fit because a tennis tournament went long ,and into his evening news . He was a no -show that day.

And of course lets not forget Lawrence O'Donnell's melt down on MSNBCs 'Scarborough Country 'when he screamed obscenitites at John O'Neill because of the Swift Boat book. I did a quick search on that incident and only found it in blogs ;no major news coverage. Not that it deserved it ;but neither does the Novak meltdown.

excon rated this answer Excellent or Above Average Answer

Question/Answer
Bradd asked on 08/02/05 - On-base Percentage

(Didn't come up to home page in baseball category - so I put it in politics)

What does it include? Hits, walks, hit by pitch - anything else? Errors? WP? Passed ball? How about a fielder's choice? Anything else?

tomder55 answered on 08/03/05:

Add up all plate appearances. This is every time you come to bat.

Subtract sacrifice bunts. The number you get is your total at bats. Since 1984, Major League Baseball has included sacrifice flies in total at bats but has not included sacrifice bunts.

Add up all the times you reached base safely, which include hits, walks and the number of times you reached base by a hit by pitch. This total does not include the times you reached base because of an error or a fielder's choice.

Divide the times you reached base safely by your total at bats. Round to the third decimal place. [ie. 41051 is .411.]

OBP=H+BB+HBP/AB+BB+HBP+SF

Ted Williams is the all time leader with a .482 OBP ; Babe Ruth is second with a .474 OBP


Bradd rated this answer Excellent or Above Average Answer

Question/Answer
QuixoticChoux asked on 07/29/05 - Bush Coalition Falling Apart?

There are real cracks in the Bush Coalition....regular Republicans, Religious Right, and Angry White Guys.

Frist, who has eyes on the Presidential nomination in 2008 has broken with Bush's stand with the Religious Right and is moving toward Stem Cell research.

Bush lost the War on Terrorism per his change in rhetoric renaming the Iraq debaucle The Global Struggle with Extremism. The American public has abandoned Bush according to public opinion polls.

He is so frustrated now with his crumbling popularity due primarily to the Iraqi War, he has reduced himself to giving a backward "flipping the bird" to the press as he walked down a corridor.

Three days ago on Daytime with Linda Vestor(on vacation) on Fox the official media mouthpiece of the Bush Administration, a guest declared that there will be a muclear attack on America *very soon* as Osama wants to kill about 4 million people. Huh? Fear tactics to unite the populace??? Sorry, won't work, as we *all remember* Bush set out in September 2001 to "hunt down Osama and bring him to justice". Yet, I guess he is still out there because we were side-tracked by the neo-Cons pet delusion that an invasion of Iraq on trumped-up reasons would be a good thing.

So, now, four years later, Osama is going to nuke us???

tomder55 answered on 07/30/05:

I posted about the al-Qaeda nuclear threat last week . But hey if you don't believe us Bush cheer -leaders then perhaps the words of frequent Bush critic ;former CIA operative ;and author of the formerly anonamous book 'Imperial Hubris',Michael Scheurer will convince you that the threat is real .

But if bin Laden is much stronger than he was, why haven't there been more attacks on the United States?

"One of the great intellectual failures of the American intelligence community, and especially the counterterrorism community, is to assume if someone hasn't attacked us, it's because he can't or because we've defeated him," says Scheuer. "Bin Laden has consistently shown himself to be immune to outside pressure. When he wants to do something, he does it on his own schedule."

"You've written no one should be surprised when Osama bin Laden and al Qaeda detonate a weapon of mass destruction in the United States," says Kroft. "You believe that's going to happen?"

"I don't believe in inevitability. But I think it's pretty close to being inevitable," says Scheuer.

A nuclear weapon?

"A nuclear weapon of some dimension, whether it's actually a nuclear weapon, or a dirty bomb, or some kind of radiological device," says Scheuer. "Yes, I think it's probably a near thing."

What evidence is there that bin Laden's actually working to do this?

"He's told us it. Bin Laden is remarkably eager for Americans to know why he doesn't like us, what he intends to do about it and then following up and doing something about it in terms of military actions," says Scheuer. "He's told us that, 'We are going to acquire a weapon of mass destruction, and if we acquire it, we will use it.'"

.....................................
As I mentioned in the expert forum ;since I never liked the terminology GWOT ;I am not upset to see a re-naming but The 'Struggle with Extremism'doesn't work for me either .'War against jihadist scum'would be better . But he may have a point . Does struggle against extremism include Moveon.org ,Ted Kennedy et-al. ?

.........................................

most 2nd term Presidents coalitions split . The big players stake out their owm positions for runs at the Presidency .Since campaigns begin shortly after the last one ends it is not really suprising then that people like Bill Frist would split from Bush ;or that more liberal Republicans like George Pataki would stake independent positions ,or that Bush himself would split from some of his core supporters . That's politics baby !

QuixoticChoux rated this answer Excellent or Above Average Answer

Question/Answer
excon asked on 07/28/05 - Coughenour's da man


Hello righty's (wrongy's):

Here is what the judge said when he sentenced a terrorist yesterday:

"... our system works. We did not need to use a secret military tribunal, or detain the defendant indefinitely as an enemy combatant, or deny him the right to counsel, or invoke any proceedings beyond those guaranteed by the Constitution. I would suggest the message to the world from today's sentencing is that our courts have not abandoned our commitment to the ideals that set our nation apart....

"Unfortunately, some believe that this threat renders our Constitution obsolete. This is a Constitution for which men and women have died and continue to die and which has made us a model among nations. If that view is allowed to prevail, the terrorists will have won."

Reagan appointed him. But, he sounds a lot like me.

excon

tomder55 answered on 07/28/05:

He doesn't see the difference does he ? The US caught this clown Ahmed Ressam on American soil . The Guantanamo detainees were captured outside the US .As we have gone over and over ;they were not entitled to status of civilian prisoners or to POW status for that matter .But lets say they were ; still under Geneva provisions they would not be entitled ;nor are they even allowed to be tried in civilian court .

Further ;any intelligence gained from operation or interrogation would have to be divulged inside the court no matter how it compromises intelligence gathering .

When we tried Shiek Omar after the 1993 WTC bombing he used his attorney to pass on messages to his cell . We now have Zacarias Moussaoui in the same situation and he is demanding his right as a defendent to see classified documents and to question other al-Qaeda terrorists on the stand .

Give me a break . This judge is a blowhard and a disgrace ! Military Tribunals have AGAIN been held Constitutional ;and every detainee has already been through a review .

excon rated this answer Excellent or Above Average Answer
Itsdb rated this answer Excellent or Above Average Answer

Question/Answer
excon asked on 07/27/05 - Oil and Terrorisim


Hello Bushies:

There is an energy bill about to be passed that does substantially nothing to end our addiction to Saudi oil.

To me, this virtually guarantees the war on terror will continue for years, resulting in untold innocent American lives lost. Further, it shows that our leaders (both left and right) dont yet have clue how to win it.

But, what do you expect when you put an oilman in the White House and red states rule? What? You don't see a connection?

excon


tomder55 answered on 07/27/05:

not really . Arabian oil has been an issue since 1973 . Only Jimmy Carter made a serious issue of it .

You are right that both parties suck on this issue ,and that the Energy Bill that is going though Congress is a joke . Bush should muster the courage to veto it ;but he won't [it allows exploration in ANWR] . The Senate had some provisions for mandating utilities to produce 10% of their power from solar, wind and other renewable sources by 2020 and some Co2 caps ,but they have been weeded out in conference committee negotiations .Instead it is full of pork projects and does little to address our energy problems. It also contains a mandate to use ethanol[There are benefits to being the first state in the union to hold a primary. All that Iowa corn gets to be turned in to ethanol].


btw . The war against Islamic terrorism cannot be won without cutting off the flow of oil money to the Middle East. This is true ,but becoming energy independent does not guarantee victory either . We cannot withdraw from the Middle East and say ok now stop attacking us. This issue was never our presence in the Middle East .The issue was our presence.

excon rated this answer Excellent or Above Average Answer
labman rated this answer Excellent or Above Average Answer

Question/Answer
excon asked on 07/27/05 - Punishment


Hello:

Should convicted prisoners be sentenced for what they did, or should they be given more time because they didn't cooperate (snitch)?

Is refusal to cooperate with the cops a crime? If it isn't, why should someone do time for it?

excon

tomder55 answered on 07/27/05:

It depends I guess . If the police is behind you with siren going and you refuse to pull over it is a crime because you are obstructing their ability to do their job. If you are a reporter and a judge orders you to disclose your source who may also be the subject of a prosecutors investigation and you refuse to do so it can be considered a contempt of court for obstruction. If it involves 5th Amendment protections (self incrimination) then it should not be a crime.

excon rated this answer Excellent or Above Average Answer

Question/Answer
Itsdb asked on 07/26/05 - Jane is "coming out"

Fonda on warpath - Plans U.S. tour to protest Iraq policy

BY ADAM NICHOLS
DAILY NEWS STAFF WRITER

"Hanoi Jane" is doing it again.

Actress Jane Fonda has launched an anti-Iraq war protest - infuriating veterans still seething from her Vietnam antics.

"I've decided I'm coming out," she said, announcing plans to cruise a vegetable oil-fueled bus across the U.S., stopping for high-profile protests.

It's her first active campaigning since being photographed laughing as she sat on a gun used by North Vietnamese troops to blow U.S. aircraft from the sky. The wounds from that stunt are still far from healed.

"She should keep her mouth shut," said Iraq veteran Lee Hadziyianis, 33, of New Hyde Park, Queens. "She did enough damage in Vietnam. We don't need that again." Hadziyianis, a military police officer and weapons specialist, went into Baghdad in May 2003. He returned to New York last year.

"What does she expect to achieve?" he said. "The decision makers aren't going to be influenced by her. Even if they are, pulling the troops out early would mean all those guys who died died in vain. We need to finish the job, and having somebody with her reputation making a noise from home helps nobody."

"It's like putting gasoline on a bonfire for so many veterans who will never forget what she did in Vietnam," said American Legion spokeswoman Ramona Joyce. "We don't want to see a Baghdad Jane
."

The 67-year-old Fonda's plans include signing up families of Iraq war veterans to speak out against the U.S. presence, she said at a signing of her autobiography, "My Life So Far," over the weekend.

Fonda, whose popularity was shattered by the Vietnam visit, has reinvented herself this year with a starring role in the blockbuster movie "Monster-in-Law," and the publishing of her successful autobiography.

Announcing the bus tour, she said, "I have not taken a stand on any war since Vietnam. I carry a lot of baggage from that." But she has spoken against the Iraq war, claiming it was based on lies - a claim she also made about Vietnam.

"I think it was a mistake," she told the Daily News this year. "It's another example of the government lying to the American people in order to get us into war." She has said she believes American troops should be supported.

Fonda's bus tour is being planned to begin in March, the third anniversary of the start of the Iraq conflict.

For some, her return to anti-war protest is welcomed. Sue Niederer's son, Army 1st Lt. Seth Dvorin, 24, of Hopewell, N.J., was killed in February last year trying to disarm a bomb in Iraq. Said his mother, "She speaks from her heart, and it's because she's controversial that people admire her. She stands up for what she believes.

"If I'm asked, it would be my pleasure to stand with her and speak with her."

~~~~~~~~~~~~~~~~~~~~~~~~~~~~~~~~~~~~~~~~~~~~~~~~~~~~~~~

Sigh...

Question? Hmmm...perhaps you'd prefer this version of Jane's "coming out" party?

Steve


tomder55 answered on 07/26/05:

it was inevidible and predictable that the 60s foggies would try one last stab at reliving their glory days .

What I don't get is the new generation . Ricky Martin is a good example . "I promise I will become a spokesperson, if you allow me to, a spokesperson on your behalf. I will defend you and try to get rid of any stereotypes," the 33-year-old singer told youngsters from 16 mainly Arab countries at a youth conference on Monday.

Anyone else see a problem here ? Of course all the mullahs will love having a Latin pop artist influencing their youth with songs like "She Bangs," "Shake Your Bon-Bon" and "Livin' La Vida Loca,"

Martin posed for photos with fans at the conference, at one point draping over his shoulders a traditional Palestinian kaffiyeha scarf with the slogan "Jerusalem is ours" written in Arabic on it. I hope he isn't too concerned about alienating his Jewish fans.

Itsdb rated this answer Excellent or Above Average Answer
excon rated this answer Excellent or Above Average Answer

Question/Answer
Itsdb asked on 07/25/05 - Iran Acknowledges Widespread Prison Abuses

Monday July 25, 2005 12:31 AM

By ALI AKBAR DAREINI

Associated Press Writer

TEHRAN, Iran (AP) - In an unprecedented report, Iran's hard-line judiciary acknowledged widespread human rights violations in prisons, including the use of torture, state-run media reported Sunday.

The report said prison guards and officials in detention centers have ignored a legal order banning torture. It also said police have made several arrests without sufficient evidence and held suspects in undeclared detention centers.

The report, which was broadcast on state-run radio and appeared on the front page of several newspapers, said a judicial investigation had discovered human right violations including the ``blindfolding and beating'' of defendants, a 13-year-old boy jailed for stealing a hen, a woman who was imprisoned because her husband was a fugitive and a man who has been in prison since 1988 without a verdict in his case.

The report has been handed over to the head of judiciary Ayatollah Mahmoud Hashemi Shahroudi.

Abbas Ali Alizadeh, head of the Tehran Justice Administration, who drafted the report, said some detention centers run by the hard-line elite Revolutionary Guards had refused to admit inspectors or investigate whether prisoners' human rights were being respected.

Last year, Shahroudi ordered a ban on the use of torture for obtaining confessions - a move seen as Iran's first public acknowledgment of the practice.

Iran's constitution specifically outlaws torture, but human rights groups say the Islamic Republic's security forces routinely use it to extract confessions.

Iranian hard-liners have jailed several dozen reformist journalists and political activists and closed about 100 pro-democracy publications in the past five years for criticizing the rule of the country's unelected clerics.

In 2003, a special U.N. envoy visited Iran, during which he said he received ``many complaints'' regarding human rights violations, including torture, from pro-reform dissidents, writers and activists.

The bleak situation in Iranian prisons was highlighted by the case of Zahra Kazemi, the Iranian-Canadian photojournalist who died in jail July 2003 about three weeks after being detained for taking photographs outside a Tehran prison during anti-government protests. Reformers said she was beaten to death.

Hard-line officials have long denied the use of torture in Iranian detention centers, despite complaints by intellectuals and student leaders of intolerable physical and psychological torture while being incarcerated.

Several journalists and political dissidents have said they made false confessions under duress.

~~~~~~~~~~~~~~~~~~~~~~~~~~~~~~~~~~~~~~~~~~~~~~~~~~~~~~~

Wow. Perhaps they'll release Ganji next?

tomder55 answered on 07/25/05:

When picture like this are published it sorta makes it hard for them to deny . A far cry from wearing panties and doing a naked human pyramid .

The case of Zahra Kazemi has put a strain on Iran's relations with Canada .She died from a blow to the head during interrogations .
Allegations from an Iranian doctor exiled in Canada said Kazemi was also raped and tortured. Iran's Nobel Peace Prize winner, Shirin Ebadi has criticized the court that heard the case.

I guess with their proven record of treating journalists rough it took some guts for Sean Penn to go to Iran to report on the elections .

I wonder how this report will sit with President-"elect" Mahmoud Ahmadinejad who was himself a torturer and chief executioner at Evin Prison, one of the most brutal detention facilities run by the Iranian regime .

The Iran blogs published a letter from Gangi today dated 7/22

In my opinion, our [intellectual] elite are familiar, more than any other time, with the culture of democracy. The pioneers of democracy and those who developed the democratic process in undemocratic societies never consisted of as many philosophers and theoreticians as do our elite today. Our problem is not "lack of knowledge on democracy", but not being ready to pay the price. Democracy needs men of action, women of courage, and resilient youth. Self-sacrifice and selflessness open the way to freedom and human rights, not just a knowledge of modern culture. We should get to know modernity and modern society, and we should build a bridge between our knowledge and our actions in order to set up a democracy.

Itsdb rated this answer Excellent or Above Average Answer

Question/Answer
oncloud9 asked on 07/24/05 - Newspaper article

Here it is:
Innocent man was shot

By SUN ONLINE REPORTER

SCOTLAND Yard Commissioner Sir Ian Blair has apologised for the death of Jean Charles de Menezes.

Mr de Menezes was shot at Stockwell Tube station on Friday after armed police pursued him in the belief he was a suicide bomber.

But the Metropolitan Police have admitted the 27-year-old Brazilian was innocent and had nothing to do with the recent London bombings.

Sir Ian said: "This is a tragedy. The Metropolitan Police accepts full responsibility for this.

"To the family I can only express my deep regrets."

Sir Ian also insisted the death of Mr de Menezes would not change the "shoot-to-kill policy" for tackling suicide bombers.

He added: "Somebody else could be shot. But everything is done to make it right.

"This is a terrifying set of circumstances for individuals to make decisions."

The link is:

http://www.thesun.co.uk/article/0,,2004600000-2005340009,00.html

tomder55 answered on 07/25/05:

It is an awful, awful tragedy--and one which the usual suspects will exploit to further their anti-cop, anti-gun, anti-War on Terror agenda. But London Mayor Ken Livingstone (someone who I usually have issues with) hits the nail on the head :

"The police acted to do what they believed necessary to protect the lives of the public.
"This tragedy has added another victim to the toll of deaths for which the terrorists bear responsibility."



Confronted with a furtive individual who reportedly refused to obey orders, dressed suspiciously, and fled over a turnstile towards the subway train, London authorities made a split-second decision in a moment of great danger.

oncloud9 rated this answer Excellent or Above Average Answer

Question/Answer
powderpuff asked on 07/24/05 - How are wars classified?

I read in the news yesterday and today that more Americans than Japanese think we will have another world war in our lifetime.

What exactly is a "world war"? It seems to me that what we are currently experiencing with terror attacks being committed in many different countries all over the world, THIS is a world war.

How does a war get classified as a world war? Sure the US is in a war in Iraq, and Afghanistan.... but in reality, the war seems to be without borders, and involves the whole world. It is not just the US and coalition forces against the Taliban in Afghanistan and "insurgents" in Iraq.

Is this war with Islamic extemist terrorists a world war or not? If not, what has to happen before it will be classified as a world war?

tomder55 answered on 07/25/05:

this is world war IV world war III being the so called cold war (over 100,000 American soldiers died in this one )

powderpuff rated this answer Excellent or Above Average Answer

Question/Answer
powderpuff asked on 07/23/05 - ideas

There is no getting away from it, we are at war. And we are in it for the long haul.

If you were in charge, how would you fight (and win) the war on terror?

tomder55 answered on 07/25/05:

First I'd like to say that I agree with excon about our dependence on foreign oil. I agree that it is an imperitive to break the cycle of dependency on petroleum . We should've been doing so for the last 6 administrations dating back to the 1973 embargo. To that end I endorse the goals of Americans for Energy Independence .

I also agree with Drgade that Bush over-all has done a good job in the fight in the GWOT or as I call it the war against Jihadistan .

Why Jihadistan ? Because it gives a name to the enemy ;something the West has been very reluctant to mention. Local battles of Tamil Tigers in Sri Lanka ;Basque seperatists in Spain ;the IRA in Ireland ;have not been and are not our concern . All these movements have used the terrorist tactict and yet overall are none of our business. Our enemies are specific ;and should be named .That would help the population of the US realize that yes we are actually in a war for our survival . It is not helpful when the BBC within hours of the London attacks changed the word "terrorists " to "bombers".

The West hampers it's efforts with political correctness. In WWII posters defining the enemy were all over the place .But today Islamo-fascism can't be condemned because that would imply superiority in our core western values.We can't have that as we've known since the 1960's that America is the root of all evil in the world.To fail to understand and sympathize with Arab rage smacks of colonialism. Jihadistan gives a name to the Muslims who's goal is the creation of a Global Caliphate .

Once this necessary step is done only then can a coherent plan be implemented . Bush stumbled on the right course of action with the promotion of democracy in the Middle East .I fully support that initiative as the ONLY alternative to total war.

powderpuff rated this answer Excellent or Above Average Answer

Question/Answer
oncloud9 asked on 07/23/05 - Sadness

A Brazilian man was killed by mistake in London subway. Now the police has to explain, he has white skin and no Asiatic features. They recognized their mistake and say they feel sorry for it. He has NO relation with terrorist attacks, as it was proved.Isn't it sad?

tomder55 answered on 07/24/05:

He was warned to stop ;he did not stop .His mistake.

It wasn't a racial profiling .He was wearing a heavy coat in the middle of the summer and carrying a back pack . That is the profile of a suicide bomber no matter what his color is.

Turns out he was also an illegal alien . I will never understand why people think their first act in a country they wish to live in should be an illegal act.

Suppose he was carrying a bomb and the cops did not stop him over some crazy sensistivity to political correctness. It's a shame he was killed ,but I do not blame the cops for doing so.

oncloud9 rated this answer Above Average Answer
powderpuff rated this answer Excellent or Above Average Answer

Question/Answer
excon asked on 07/22/05 - Yes, I have another question


Hello experts:

So, let's say that we ALL will have to be searched when we board public transportation, when we enter public buildings and whereever the authorities think we need to be searched.

Of course, all these searches are for the purposes of stopping terrorisim. Ok, so what will happen to someone who tries to board a subway, and has, not a bomb, but pot in his backpack?

Too bad for him?

excon

tomder55 answered on 07/23/05:

Yeah, it is getting harder and harder to carry a joint anywhere.

The right of the people to be secure in their persons, houses, papers, and effects, against unreasonable searches and seizures, shall not be violated, and no Warrants shall issue, but upon probable cause, supported by Oath or affirmation, and particularly describing the place to be searched, and the persons or things to be seized.

Depends on how one reads the 4th Amendment. If probable cause and a warrant are required to carry out unreasonable searches, then it is a fair interpretation that they are not required to carry out reasonable ones. The applicability of the Amendment hinges on what is or is not unreasonable, and not on the search itself. I'm sure, especially after the Madrid train bombings, the London bombings ,many people consider them quite reasonable.

What happens now at random roadside check points ;or at the gate of sporting events ;or at Disneyland for that matter ?But the first person to get arrested for a joint will go to court and win. That person will successfully argue that the intent of the search was specific to terrorist related activity and was not intended to be a drug search. Sobriety check points are pretty specific ,and so are these random searches .


Why is it prudent for the Israelis to take these steps but not us?

As to your point about OBL ;no he is not trying to change us in the ways we are changing ;he isn't concerned about us having too much freedom . The Islamo-nazis want to kill us ,not change us.They want us in the state of Dhimmitude .It is not a question of our freedoms to them. The Russians have no freedoms and they wage jihad equally against them.

excon rated this answer Excellent or Above Average Answer

Question/Answer
excon asked on 07/22/05 - For tomder and others


Hello tom, and everybody else:

Yes, I believe in open DEFENSIBLE borders. I do not believe that we will ever secure 6,000 miles of land that borders both our northern and southern flanks. In addition, I dont see a need to do so. What? Are the Mexicans going to attack us with leaf blowers? Are the Canadians going to attack us with BC bud (I hope so)? I think we have bigger fish to fry.

Being an old Navy man, I believe that we can defend our shores including the Mexican and Canadian coasts. Lets open both the Northern and Southern frontiers and move our efforts to sea in conjunction with both neighbors.

Yes, its radical. But less so, than changing the face of America. Besides, changing our face, short of creating a police state, isnt going to stop anything.

Question? Do I need a question?

excon

tomder55 answered on 07/22/05:

No ;it is not the Mexicans and Canadians I fear . If you will guarantee that only those would cross the border then I could see your point a little better . Do you deny that jihadists have crossed over from both borders already ?

Here's one that we got lucky with .

American border patrol agents arrested 158 illegal aliens in Cochise County, Arizona, on 6/13/04. One agent, who speaks Farsi and Arabic, overheard dozens of detainees speaking Arabic in the back of the detention vehicle. He counted 53 of Middle Eastern rather than Hispanic origin. His superiors told him to keep that information to himself; but he reported it to the local newspaper, the Tombstone Tumbleweed.(World Tribune
8-4-04)The agent involved said the men were dressed like ordinary border-crossers - baseball caps, jeans, sneakers and t-shirts. What distinguished the Middle Easterners was the fact that their clothes were as new and fresh as their identical haircuts and mustaches.

If that doesn't concern you ;know that al jazerra is showing interest in the porus nature of our "flanks " .



excon rated this answer Excellent or Above Average Answer

Question/Answer
Itsdb asked on 07/22/05 - Koehler orders German parliament dissolved, sets new election

DAVID McHUGH
Associated Press Writer

BERLIN Germany's president agreed Thursday to dissolve parliament and hold early elections Sept. 18 that could give the country its first woman chancellor.

Chancellor Gerhard Schroeder had sought the early balloting, saying he had lost the mandate to govern after his Social Democrats badly lost a key regional contest May 22.

President Horst Koehler agreed, and set the date for the national vote.

Schroeder's chief challenger will be Angela Merkel of the Christian Democratic Union, who is leading in the polls by some 17 or 18 points, giving her a chance to become Germany's first woman chancellor.

Analysts say she would likely get along better with President Bush's administration than did Schroeder, who forcefully opposed the U.S.-led invasion of Iraq.

Koehler stressed the election will be critical, saying Germany faced "giant challenges" in attacking its high unemployment rate now 11.3 percent and swollen budget deficits.

"Our future and the future of our children is at stake," he said. "In this serious situation, our country needs a government that can pursue its goals with steadiness and vigor."

Since winning re-election in 2002, Schroeder has struggled to push through limited cuts in jobless benefits and worker protections to get the sluggish economy going.

But his reforms, which included cutting long-term jobless pay in an attempt to prod people to accept jobs, have provoked opposition from within his own Social Democratic Party.

Schroeder, who became chancellor in 1998 and narrowly won a second term in 2002, welcomed Koehler's decision and said that he would seek re-election as chancellor to get new support for his reforms, which he called "correct and necessary."

"The majority of people in our land want new elections. This way they can decide themselves the direction in which our country will go," he said.

Merkel has promised to create a more business-friendly environment by cutting the payroll tax for unemployment insurance, making up for it by increasing the value-added tax. The idea is to cut non-wage labor costs that discourage businesses from hiring.

But she has said she intends to keep Germany's system of expensive welfare-state benefits, while vowing to make the country more competitive and efficient.

She differs from Schroeder by opposing European Union membership for Turkey, saying it would stress the union's political and economic resources.

To get the new ballot, Schroeder deliberately lost a no-confidence vote July 1 by asking his supporters to abstain a tactic he was forced to use because the German constitution does not permit parliament to dissolve itself.

By law, only the president, Koehler, could make the decision to hold new elections. That decision may face a court challenge from deputies who have said the government does not really lack a working majority.

Original here.

~~~~~~~~~~~~~~~~~~~~~~~~~~~~~~~~~~~~~~~~~~~~~~~~~~~~~~~

I don't quite get European politics, but I would shed no tears over the shredding of Schroeder by this woman - especially if she is more supportive of Bush and the U.S. - don't you know the left is heaving another collective groan today?

tomder55 answered on 07/22/05:

Shroder has left the bar pretty low. Like Chirac in France ;he has been a disaster for his country ;to Europe ;and to the Western Alliance.Auf Wiedersehen Gerhard!

The Washington Times calls Angela Merkel, " a bland East German who is unloved by voters , but the chancellor's ratings have slipped so low that there is every likelihood she will be the country's new leader by the end of the year." I for one hope she is the reincarnation of Maggie Thatcher .

She most likely would not change direction of German policy regarding Iraq .She has been vocally critical of what she calls US unilateral action .But at the same time she also understands better what the stakes are in the GWOT .She has said "It is wrong to separate the issues of terrorism and Saddam Hussein. We need to see things from the perspective of the United States."

She has also been critical of Shroder's support for lifting the arms sales ban to China.

What will she expect from the US in return to a warmer relationship ? Probably US support for a German Security Council seat. Something I'm not sure Bush will agree to .

She has been vocal in her opposition to Turkish entry into the EU .She will bump heads with Bush on that issue . Bush sees Turkey's entry as symbolic ;a potential role model for other countries in the Middle East due to Turkey's embrace of secular democracy.
Germany took the last decade to get their economic house in order after reunification and is not thrilled with seeing more countries into the economic community that would siphen resources out of Germany.


She is more market oriented (what Shroder derides as "neo-liberal market policy and international capitalism. )She has only laid out a general plan to revitalize the German economy .The Economist calls her reform plans quite radical in the German context: simplifying tax, overhauling pensions, reducing job protection and curbing the trade unions.

Itsdb rated this answer Excellent or Above Average Answer

Question/Answer
excon asked on 07/22/05 - So Long - America


Hello experts:

You do know that we enjoy rights, not because we want to protect bad guys, but because we want to protect the good guys - us.

Now, I have no love for terrorists, and I'd like to see 'em all in the slam forever. But my question is, how do you weed 'em out, and STILL protect the good guys? Trust the government? No. Governments aren't trustworthy.

If the Patriot Act is made permanent, this country is toast.

excon

tomder55 answered on 07/22/05:

People were quick to react yesterday to the so called outrage of random searches of bags in the NY City subway system yesterday . An enterprising person was quick to design tee shirts with the slogan I do not consent to be searched . They of course have a right under the 4th Amendment to subject to unreasonable searches and seizures. The question is ;Is it not reasonable to do random searches for the greater good of security ? My solution to the whiners is not to ride the subway system if you object the the inconvienience . I prefer these tee shirts .

[I object to it for a different reason that has more to do with the stupidity of political correctness and profiling but that is not the point of this posting ].

I have argued here that at times it is necessary to surrender some personal liberties for security . As you know in times of war there is historical precident .Those who say that Lincoln went to far ;or Roosevelt I just fundamentally disagree with .I think the Patriot Act that was passed after 9-11 has been a very valuable tool in the GWOT ,and it has proven to have had a minimal impact on the general liberites in this country .

It appears that Congress (especially the Senate ) is scrutinizing the Patriot Act much closer than they did when it was first passed .There is significant differences between the House and Senate versions that will have to be reconciled sometime in the Fall I guess. Most of the debate has been about a few provisions :

-allowing the FBI to obtain records from financial companies, libraries, doctors' offices and other businesses in terrorism investigations (section 215 )

-allowing "roving wiretaps" to apply to a person rather than a particular telephone.

Section 215 currently needs a court order .I think there may be an additional provision of needing the FBI Director's approval before it is utilized being debated .Grand Juries have long had the power to obtain these types of records so I do not really understand the objection. The grand jury investigating Eric Rudolph's 1996 bombing of Atlanta's Olympic Park subpoenaed bookseller records that showed that Rudolph had purchased a book on bomb detonation. That was important information to have .9/11 hijackers Nawaf al-Hazmi and Khalid al-Midhar reviewed airline reservations for their 9/11 flight at a New Jersey state college library computer; three more 9/11 hijackers used Florida's Delray Beach Public Library in July 2001 in preparation for their crime.

I will agree with you that these provision should not be made permanent . Instead I would recommend a regular review of them to determine need and effectiveness.

Your follow up comment is only partly correct . There is a primary obligation of a government of free people to 'provide for the common defense'.There is no more important a role for the Federal Government.Policy makers have neither the moral nor the constitutional prerogative to neglect our security .In dangerous times we have always managed to summon America's greatest strength ; our openness to change and our confidence that a free people will choose wisely.

excon rated this answer Excellent or Above Average Answer
Itsdb rated this answer Excellent or Above Average Answer
ladybugca rated this answer Excellent or Above Average Answer

Question/Answer
excon asked on 07/21/05 - Tell me, wadd I say??


Hello experts:

"I will fire anyone involved in the leak." "I will fire anyone who committed a crime." "I will fire anyone who doesn't win his appeal after he's convicted. "I will fire anyone who is denied parole."

"Hell, I aint gonna fire any of my friends, and yes, I lied."

Are you proud of him? Or are you gonna spin it and tell me how wonderful he is?

excon

excon

tomder55 answered on 07/21/05:

For the record :

George W. Bush: September 30th, 2003

"If there is a leak out of my administration I want to know who it is, and if that person has violated law, that person will be taken care of. "


Here is a follow up press confrence after the G8 Summit:

Q Given -- given recent developments in the CIA leak case, particularly Vice President Cheney's discussions with the investigators, do you still stand by what you said several months ago, a suggestion that it might be difficult to identify anybody who leaked the agent's name?

THE PRESIDENT: That's up to --

Q And, and, do you stand by your pledge to fire anyone found to have done so?

THE PRESIDENT: Yes. And that's up to the U.S. Attorney to find the facts.

Q My final point would be -- or question would be, has Vice President Cheney assured you --

THE PRESIDENT: It's up to the --

Q -- subsequent to his conversations with them, that nobody --

THE PRESIDENT: I haven't talked to the Vice President about this matter, and I suggest -- recently -- and I suggest you talk to the U.S. Attorney about that.


It was dependent on the investigators findings in other words ..but you knew that .If you review the complete questions and answers for this subject from September 30, 2003, June 10th, 2004 and July 18th, 2005, it is obvious that his position has not change.



The media wants Bush to fire people for setting the record straight. Plame conspired with her husband to send him on an African junket so he could drink "sweet mint tea" and come back to write "Bush lied" in the New York Times. I though you liked whistle blowers. Yes ;a whistle blower who exposed a cabal in the CIA who was doing ops. to undermine policy and we can throw nepotism in their as a bonus .That Joe Wilson is discreditied is beyond dispute . The only WMD definitely found in Iraq was yellow cake .



Sadly our President is not very articulate ;and he does not make up for it by hiring competent spokespeople .Scott McLellan is an incompetent in my view .He is the one who put the suggestion that Bush would fire any leaker in the mix .Somehow I do not think Ari Fleischer would've made that mistake . McLellan came into the job and the press first set him up with talk of a kinder and gentler relationship between the White House and the press and then they have subsequently chewed him up and spit him out. His performance has been abysmal .I conceed that point .

But is that what the issue has been reduced to ? What I am curious to is why the press ,an institution that relies so much on the leaked information ,would be so hot to try to reveal and punish a leaker who DID NOT BREAK THE LAW .Yes ;very curious indeed .

excon rated this answer Excellent or Above Average Answer
ladybugca rated this answer Excellent or Above Average Answer

Question/Answer
Itsdb asked on 07/18/05 - Shame on the media

Did you know that 36 news organizations filed a friend of the court brief in the Miller/Cooper/Plame case in which they made the following claims?

"the circumstances necessary to prove that crime seem not to be present here"

"At the threshold, an agent whose identity has been revealed must truly be "covert" for there to be a violation of the act. To the average observer, much less to the professional intelligence operative, Plame was not given the "deep cover" required of a covert agent." She worked at a desk job at CIA headquarters, where she could be seen traveling to and from, and active at, Langley. She had been residing in Washington - not stationed abroad - for a number of years. As discussed below, the CIA failed to take even its usual steps to prevent publication of her name.

Moreover, the government may have "publicly acknowledged or revealed" her intelligence relationship prior to publication of Novak's July 2003 column...An article in the Washington Times indicated that Plame's identity was compromised twice prior to Novak's publication (7)."

(7) Bille Gertz, "CIA officer named prior to column," The Washington Times, July 23, 2004, at A4 (noting that a Moscow spy first disclosed Plame's identity to Russia in the mid-1990's, and a more recent "inadvertent disclosure" stemming from references to Plame in "confidential documents sent by the CIA to the U.S. Interests Section of the Swiss Embassy in Havana," which was read by the Cubans."

"There are sufficient facts on the public record that cast considerable doubt as to whether the CIA took the necessary "affirmative measures" to conceal Plames identity. Indeed these facts esatblish such sloppy tradecraft that, at minimum, the CIA was indifferent to the compromise of her identity."

"If a serious request not to publicize Plame's identity had been made, Novak would have obliged...Novak's column can be viewed as critical of CIA ineptitude."

So, the media believed there wasn't sufficient evidence of a crime, Plame worked a desk job in Washington with no "deep cover," the CIA outed Plame to Castro, Novak's column was critical of CIA ineptitude, and Karl Rove is to blame? Who's lying now?

Steve

P.S. Thanks to Andrew McCarthy at NRO for pointing this out.

tomder55 answered on 07/19/05:

I'm so tired of this case . I think Aldrich Ames himself disclosed Plame to the Russians and the CIA shut down any operation she was involved in BEFORE she met Wilson.


Where was all the press frenzy when Senator John Kerry outed a CIA operative ?

Where was it when former Senator Robert Torricelli outed a paid CIA informant, Guatemalan Col. Julio Roberto Alpirez ?

Itsdb rated this answer Excellent or Above Average Answer

Question/Answer
abx12 asked on 07/16/05 - Trying again - Worldcom

I didnt get answers I was look for. ebbers dint steal anything but yes investrs lost. That's not same thing. Experts answeered fraud, but I meant what exactly was the fraud? Failing to file forms was just a add-on to charges.
Like - Joe was convicted of murder - ok, WHO did Joe murder? Etc. thank you.

tomder55 answered on 07/17/05:

you got answers to your question which was

Worldcom .....anyone tell why, in simple langiage, this ceo was given 25 =yeras?

I answered accurately and in very simple language the reason .When you asked if he 'miscounted'I replied that he INTENTIONALLY cooked the books. The type of 'fraud 'he was convicted of is theft. Yes fraud = theft .Why ? Because billions of dollars were invested in Woldcom based on the faulty figures that were given to the public. If the true nature of Worldcom's financial situation were known then no one but a fool would've put their personal money into the company ,and the financial institutions in charge of managing multi billions dollars of pension;IRA ;401 K funds safely would not have risked putting their funds into Worldcom stock.

abx12 rated this answer Average Answer
CeeBee2 rated this answer Excellent or Above Average Answer
excon rated this answer Excellent or Above Average Answer

Question/Answer
Itsdb asked on 07/13/05 - Lawyer: Cooper Burned Karl Rove

Roves attorney talks to NRO.

The lawyer for top White House adviser Karl Rove says that Time reporter Matthew Cooper "burned" Rove after a conversation between the two men concerning former ambassador Joseph Wilson's fact-finding mission to Niger and the role Wilson's wife, CIA employee Valerie Plame, played in arranging that trip. Nevertheless, attorney Robert Luskin says Rove long ago gave his permission for all reporters, including Cooper, to tell prosecutor Patrick Fitzgerald about their conversations with Rove.

In an interview with National Review Online, Luskin compared the contents of a July 11, 2003, internal Time e-mail written by Cooper with the wording of a story Cooper co-wrote a few days later. "By any definition, he burned Karl Rove," Luskin said of Cooper. "If you read what Karl said to him and read how Cooper characterizes it in the article, he really spins it in a pretty ugly fashion to make it seem like people in the White House were affirmatively reaching out to reporters to try to get them to them to report negative information about Plame."

First the e-mail. According to a report in Newsweek, Cooper's e-mail to Time Washington bureau chief Michael Duffy said, "Spoke to Rove on double super secret background for about two mins before he went on vacation..." Cooper said that Rove had warned him away from getting "too far out on Wilson," and then passed on Rove's statement that neither Vice President Dick Cheney nor CIA Director George Tenet had picked Wilson for the trip; "it was, KR said, wilson's wife, who apparently works at the agency on wmd issues who authorized the trip." Finally all of this is according to the Newsweek report Cooper's e-mail said that "not only the genesis of the trip is flawed an[d] suspect but so is the report. he [Rove] implied strongly that there's still plenty to implicate iraqi interest in acquiring uranium fro[m] Niger..."

A few days after sending the e-mail, Cooper co-wrote an article headlined "A War on Wilson?" that appeared on Time's website. The story began, "Has the Bush administration declared war on a former ambassador who conducted a fact-finding mission to probe possible Iraqi interest in African uranium? Perhaps."

The story continued:

Some government officials have noted to Time in interviews (as well as to syndicated columnist Robert Novak) that Wilson's wife, Valerie Plame, is a CIA official who monitors the proliferation of weapons of mass destruction. These officials have suggested that she was involved in her husband's being dispatched to Niger to investigate reports that Saddam Hussein's government had sought to purchase large quantities of uranium ore, sometimes referred to as yellow cake, which is used to build nuclear devices.

Plame's role in Wilson's assignment was later confirmed by a Senate Intelligence Committee investigation.

Luskin told NRO that the circumstances of Rove's conversation with Cooper undercut Time's suggestion of a White House "war on Wilson." According to Luskin, Cooper originally called Rove not the other way around and said he was working on a story on welfare reform. After some conversation about that issue, Luskin said, Cooper changed the subject to the weapons of mass destruction issue, and that was when the two had the brief talk that became the subject of so much legal wrangling. According to Luskin, the fact that Rove did not call Cooper; that the original purpose of the call, as Cooper told Rove, was welfare reform; that only after Cooper brought the WMD issue up did Rove discuss Wilson all are "indications that this was not a calculated effort by the White House to get this story out."

"Look at the Cooper e-mail," Luskin continues. "Karl speaks to him on double super secret background...I don't think that you can read that e-mail and conclude that what Karl was trying to do was to get Cooper to publish the name of Wilson's wife."

Nor, says Luskin, was Rove trying to "out" a covert CIA agent or "smear" her husband. "What Karl was trying to do, in a very short conversation initiated by Cooper on another subject, was to warn Time away from publishing things that were going to be established as false." Luskin points out that on the evening of July 11, 2003, just hours after the Rove-Cooper conversation, then-CIA Director George Tenet released a statement that undermined some of Wilson's public assertions about his report. "Karl knew that that [Tenet] statement was in gestation," says Luskin. "I think a fair reading of the e-mail was that he was trying to warn Cooper off from going out on a limb on [Wilson's] allegations."

Luskin also shed light on the waiver that Rove signed releasing Cooper from any confidentiality agreement about the conversation. Luskin says Rove originally signed a waiver in December 2003 or in January 2004 (Luskin did not remember the exact date). The waiver, Luskin continues, was written by the office of special prosecutor Fitzgerald, and Rove signed it without making any changes with the understanding that it applied to anyone with whom he had discussed the Wilson/Plame matter. "It was everyone's expectation that the waiver would be as broad as it could be," Luskin says.

Cooper and New York Times reporter Judith Miller have expressed concerns that such waivers (top Cheney aide Lewis Libby also signed one) might have been coerced and thus might not have represented Rove's true feelings. Yet from the end of 2003 or beginning of 2004, until last Wednesday, Luskin says, Rove had no idea that there might be any problem with the waiver.

It was not until that Wednesday, the day Cooper was to appear in court, that that changed. "Cooper's lawyer called us and said, "Can you confirm that the waiver encompasses Cooper?" Luskin recalls. "I was amazed. He's a lawyer. It's not rocket science. [The waiver] says 'any person.' It's that broad. So I said, 'Look, I understand that you want reassurances. If Fitzgerald would like Karl to provide you with some other assurances, we will.'" Luskin says he got in touch with the prosecutor "Rule number one is cooperate with Fitzgerald, and there is no rule number two," Luskin says and asked what to do. According to Luskin, Fitzgerald said to go ahead, and Luskin called Cooper's lawyer back. "I said that I can reaffirm that the waiver that Karl signed applied to any conversations that Karl and Cooper had," Luskin says. After that which represented no change from the situation that had existed for 18 months Cooper made a dramatic public announcement and agreed to testify.

A few other notes: Luskin declined to say how Rove knew that Plame "apparently" (to use Cooper's word) worked at the CIA. But Luskin told NRO that Rove is not hiding behind the defense that he did not identify Wilson's wife because he did not specifically use her name. Asked if that argument was too legalistic, Luskin said, "I agree with you. I think it's a detail."

Luskin also addressed the question of whether Rove is a "subject" of the investigation. Luskin says Fitzgerald has told Rove he is not a "target" of the investigation, but, according to Luskin, Fitzgerald has also made it clear that virtually anyone whose conduct falls within the scope of the investigation, including Rove, is considered a "subject" of the probe. "'Target' is something we all understand, a very alarming term," Luskin says. On the other hand, Fitzgerald "has indicated to us that he takes a very broad view of what a subject is."

Finally, Luskin conceded that Rove is legally free to publicly discuss his actions, including his grand-jury testimony. Rove has not spoken publicly, Luskin says, because Fitzgerald specifically asked him not to."

~~~~~~~~~~~~~~~~~~~~~~~~~~~~~~~~~~~~~~~~~~~~~~~~~~~~~~~

Someone tell me where the inconsistency is here.

tomder55 answered on 07/14/05:

I've said my say on this subject for now. The only thing I have to add is there is a curious silent lack of outrage when the NY Slimes outed the names of CIA operatives;front companies ;plane types and id #s and airports used for renditions. Why was that not a breech of national security ? Why did that not put CIA operatives in danger ?

Itsdb rated this answer Excellent or Above Average Answer

Question/Answer
abx12 asked on 07/13/05 - Worldcom

anyone tell why, in simple langiage, this ceo was given 25 =yeras?

tomder55 answered on 07/14/05:

WorldCom was the United States' second largest long distance phone company . WorldCom grew largely by acquiring other telecommunications companies, most notably MCI. June 2002, an internal audit discovered that $3.8 billion had been 'miscounted.' The US Securities and Exchange Commission launched an investigation .WorldCom filed for Chapter 11 bankruptcy protection.By the end of 2003, it was estimated that the company's assets had been inflated by around $12 billion . The company emerged from Chapter 11 bankruptcy in 2004 with a new name, MCI, and about $5.7 billion in debt and $6 billion in cash. About half of the cash was intended to pay various claims and settlements. Previous bondholders ended up being paid 35.7 cents on the dollar, in bonds and stock in the new MCI company. The previous stockholders' stock was valueless. .The company paid $750 million to the SEC in cash and stock in the new MCI, which was intended to be paid to wronged investors.

On March 15, 2005 Ebbers was found guilty of all charges and convicted on fraud, conspiracy and filing false documents with regulators ;all related to the $12 billion accounting scandal.There are other former company execs. who also either face charges or have been convicted . He may potentially be let out of prison at the age of 83 on terms of good behaviour but more likely given his health he will be essentially serving a life sentence .Ebbers and his wife, Kristie, will be stripped of almost all their assets in a seperate civil case ;but his assets come way short of paying back all the investors and pensioneers he defrauded .If anyone thinks the sentence is too harsh ;then they sould ask the former employees of Worldcom who's pensions are lost forever ;who's careers were destroyed by the fraud that Ebbers committed .



abx12 rated this answer Above Average Answer
excon rated this answer Excellent or Above Average Answer

Question/Answer
excon asked on 07/12/05 - A title for my question


Hello politicos:

"I did not have sex with that woman" - "I did not leak her name".

Same thing - or are you going to dither?

excon

tomder55 answered on 07/12/05:

not me . I never thought CLintoon deserved to be impeached because he couldn't keep his pants up. I thought he should've been impeached because he undermined US national security by allowing the sale of MIRV missle technology and advanced submarine technology to China in return for re-election campaign contributions . I also though more effort should've been paid to the Juanita Broderick allegations of rape .

and since I do not dither . I do not think that anything was done that was illegal in the Plame case .If Judith Miller decides she should go to jail over some misguided loyalty to her insider sources ...too bad . I compare her's to that idiot Susan McDougle who sat in jail rather than to cooperate in the Whitewater investigation and implicate the CLintoon's .

If any good comes from this story it is that it further exposes the sordid sleezy way that the press fabricates covers the news .

excon rated this answer Excellent or Above Average Answer
Itsdb rated this answer Excellent or Above Average Answer

Question/Answer
paraclete asked on 07/10/05 - what a bunch of wimps

Reaction polarized as G8 concludes
Blair: 'It is a beginning, not an end'

Saturday, July 9, 2005 Posted: 0610 GMT (1410 HKT)






leaders concluded an economic summit shaken by terrorism, offering an "alternative to the hatred" -- aid packages for Africa and the Palestinian Authority and a pledge to address global climate change.

But reaction to the outcome was polarized: Rockers Bob Geldof and Bono, two of the world's best known Africa fund-raisers, declared victory. But aid groups said the pledges didn't go nearly far enough.

"We speak today in the shadow of terrorism, but it will not obscure what we came here to achieve," British Prime Minister Tony Blair, the summit host, said Friday as he closed the three-day gathering, referring to Thursday's bus and train bombings in London.

"It is in the nature of politics that we do not achieve absolutely everything we hope to achieve, but nonetheless I believe we have made very substantial progress indeed," Blair said.

With a last-minute pledge from Japan, Blair won a key victory, announcing that aid to Africa would rise from the current US$25 billion to US$50 billion by 2010.

Geldof praised the leaders for pledging to double aid to Africa, saying it will save the lives of hundreds of thousands of people. "The world spoke and the politicians listened," Bono added.

But aid groups didn't see it that way.

"What Africa needed from the G-8 was a giant leap forward. All it got was tiny steps," said Caroline Sande Mukulira of ActionAid. "The deal that has been announced falls way short of our demands. We have some aid, but not enough, some debt relief but not enough and virtually nothing on trade. Once again, Africa's people have been shortchanged."

Peter Hardstaff, head of policy for the World Development Movement, described the summit's final declarations as "a disaster for the world's poor."

"The agreements on trade, debt, aid and climate change are nowhere near sufficient to tackle the global poverty and environmental crisis we face," he said. "We are furious, but not surprised."

World Bank President Paul Wolfowitz praised the G-8's plan on Africa, but insisted success depended on rich nations and African leaders delivering on their commitments. Wolfowitz also said further progress was needed on fair trade to ensure developing countries could compete.

"It is very important to stress that it is a partnership," he told AP. "It is not just about spending more money. It is about having African leaders who understand their responsibility to make sure that that money is spent wisely."

In a separate joint statement on terrorism, the leaders pledged new joint efforts against terrorism in light of the London bombings, including cooperation to improve the safety of rail and subway travel.

Blair lost his push to get all summit countries to commit to boosting foreign aid to an amount equal to 0.7 percent of national income by 2015. Instead, a summit document said the European Union had agreed to that support but did not mention the United States.

U.S. President George W. Bush had refused to be bound by the 0.7 percent target. The United States is giving 0.16 percent of national income, the smallest percentage of any of the G-8 countries.

Blair ticked off a list of accomplishments from a meeting that nonetheless produced less than he hoped. The major failure was in global warming, where staunch opposition from Bush thwarted Blair's efforts to get a U.S. commitment to firm targets for reducing greenhouse gas emissions blamed for global warming.

But Blair noted that all the G-8 leaders took the unusual step of signing the final summit communiques as a way of demonstrating their determination to meet the new goals.

"If we implement this, we will make poverty history," Blair told reporters.

French President Jacques Chirac said finding new ways to finance the support that rich country's give Africa was crucial. He put in a plug for his idea to have all countries levy a tax on international airline tickets to support poverty relief.

Russian President Vladimir Putin, who has been accused of rolling back democracy in Russia, said it was important to protect democratic freedoms in the battle against terrorism. He said they "want to use the instruments of democratic society to destroy democracy."

German Chancellor Gerhard Schroeder said the summit had been "overall successful," with leaders able to deal with the summit agenda in spite of the London bombings.

Aside from the increase in aid for Africa, the leaders pledged to set a date for ending subsidies on farm exports, which Blair said he believed would be done at a meeting of the World Trade Organization in December in Hong Kong.

The leaders also endorsed a deal reached by their finance ministers last month to cancel the debt of 18 of the world's poorest nations, pledged universal access to AIDS treatment, renewed their commitment to a peacekeeping force in Africa and heard African leaders promise to move toward democracies that follow the rule of law.

"All of this does not change the world tomorrow -- it is a beginning, not an end," Blair said, with leaders of the G-8 and five African nations standing behind him. "And none of it today will match the same ghastly impact as the cruelty of terror. But it has a pride and a hope and humanity at its heart that can lift the shadow of terrorism and light the way to a better future."

Nigerian President Olusegun Obasanjo thanked the leaders for focusing on Africa and for "their resolve not to be diverted by these terrorist acts."

The G-8 also agreed on an aid deal of up to US$3 billion per year for the Palestinian Authority over the next three years, their joint communique said. Blair said the deal would allow "two states, Israel and Palestine, two peoples and two religions (to) live side by side in peace."

The leaders, struggling to keep to their mission in the aftermath of the London bombings, shortened the summit's final day to allow Blair to rush back to lead a government panel dealing with the blasts. Bush also left Gleneagles earlier than scheduled Friday.

Blair left the summit for several hours Thursday to confer with officials at Scotland Yard and calm a nation shocked by the worst attacks on the capital since World War II.

Within hours of the London bombings, Bush and the other leaders issued a special joint statement that condemned "these barbaric acts."

"We are united in our resolve to confront and defeat this terrorism that is not an attack on one nation, but on all nations and on civilized people everywhere," the leaders said.

Copyright 2005 The Associated Press.

This is nothing more than business as usual thinly disguised under a veneer of aid to the underprevledged. It doesn't address the underlying problem of loaning poor nations money without creating the mechanism to pay it pack, which means removing the protection from the "fat cat" american and european farmers, nor does it progress the debate on the environment which is now of lesser value than it was under George WH Bush at least he was big enough to admit a problem.

What it really says is, it's all too hard, let's throw some money at it and it might go away. comments

tomder55 answered on 07/11/05:

It is ludicrous that the G8 need to bow to pressure from BONO . There was much more important things for them to do in Scotland then to bolster tinpot dictators with more giveaways of our tax money . Forget loan forgiveness . The best way to help Africa is to reform their governments and to open free trade with their nastions.

Itsdb rated this answer Excellent or Above Average Answer
paraclete rated this answer Excellent or Above Average Answer

Question/Answer
excon asked on 07/11/05 - Crime and Punishment


Hello experts:

I dont understand something. Clear it up for me. There is a NY Times reporter in jail for refusing to name Karl Rove as her source. We KNOW Karl Rove was her source, so whats the point of keeping her in jail? Is the point of her jailing to find out who her sources are, or to punish her for her arrogance?

Karl Rove is the prick who should be in the slam. Robert Novak is a prick who should be in the slam. But, no, Judith Miller is in the slam, for what I dont know.

But, Im sure youll tell me. By the way, I didnt know arrogance was a crime. If it is, Dubya qualifies.

excon

tomder55 answered on 07/11/05:

She should learn when to pick her fights . You don't believe she is in jail for protecting Rover ,do you ? Matt Cooper's notes mentioned Rove but never suggests he used Plame's name .I do not believe he would risk jail time to protect Rove either . They are protecting someone else'perhaps another reporter ? Andrea Mitchell was asked, on MSNBC, whether it was generally known to news people, before the incident that Plame worked for the CIA. She answered that it was.

If Andrea Mitchell knew this, I bet Robert Novak knew it, too. So, he didn't need a leak to tell him that Plame worked for the CIA. He was just interested in whether or not she had anything to do with Wilson's mission to Niger.He also has this tendency to embellish his reporting a little too much for my liking .He always has this cache of beltway insider ananomous sources that he quotes .


At best Rove is one of the sources .I don't think that reporters should be able to shield sources who are suspected of committing crimes.Do I think there should be some form of a Federal Shield law ;yes ,but not without restricions .

Both Novak and Rove testified before the Grand Jury .One would think that if Novak and Rove both went before the Grand Jury and said what seems to be the content of the Emails and notes that Time surrendered to the prosecutor; why was he not indicted for outing Plame before now? If Novak says Rove is his source and Rove says he leaked her to the press, then that would be enough to secure potential indictments right there. Unless the leak had nothing to do with Ms. Plame's status as a covert agent.

What do we know about the case

The illusion that Joe Wilson was an innocent State Department official who is being persecuted for "being honest", and Valerie Plame was a noble CIA field agent whose career and personal safety is now in jeopardy has been disproved by revelations, accepted by a bipartisan Senate committee.

Wilson was dispatched by the CIA in February 2002 to investigate reports that Iraq sought to reconstitute its nuclear weapons program with uranium from Africa.He was specifically recommended for the mission by his wife, a CIA employee, contrary to what he has said publicly.Wilson's assertions about what he found in Niger and what the Bush administration did with the informationwere undermined by the bipartisan Senate intelligence committee report.

The Senate Intelligence committee panel found that Wilson's report, rather than debunking intelligence about purported uranium sales to Iraq, as he has said, bolstered the case . And contrary to Wilson's assertions the CIA did not tell the White House it had qualms about the reliability of the Africa intelligence that made its way into President Bush's January 2003 State of the Union address.

The report said Plame told committee staffers that she relayed the CIA's request to her husband, saying, "there's this crazy report" about a purported deal for Niger to sell uranium to Iraq. Plame had already made up her mind about the truthfulness of the report, and dispatched her husband to Niger not to investigate, but specifically to come back with debunking evidence.

The information Wilson returned with actually strengthened the administration's case, so he lied about what its conclusions were to the press.

The report also said Wilson provided misleading information to The Washington Post . He said then that he concluded the Niger intelligence was based on documents that had clearly been forged because the dates were wrong and the names were wrong.Committee staff asked how he could have come to the conclusion that the 'dates were wrong and the names were wrong' when he had never seen the CIA reports and had no knowledge of what names and dates were in the reports. Wilson told the panel he may have been confused and may have, "misspoken" to reporters.

The documents ;sales agreements between Niger and Iraq ; were not in U.S. hands until eight months after Wilson made his trip to Niger.

Wilson lied about how he got to Niger, he lied about seeing a report that didn't even exist at the time, he lied about the conclusions of his own report; he lied about what the administration had been told, and his wife, Valerie Plame, specifically sent him on a mission to intentionally debunk a claim, not to find facts or perform inspections.

Rove ,attempting to discredit the ludicrously false claims,told the press that Wilson was sent to Niger on dubious premises in the first place at the recommendation of his wife(he never disclosed her name ;Nothing in the Cooper e-mail suggests that Rove used Plame's name or knew she was a covert operative. Not that it matters since she is the darling of the cover of Vanity Fair) . It was Judith Miller's own NY Slimes that beat the drum loudest for an aggressive prosecutor for the case(December 31, 2003 editorial)




excon rated this answer Excellent or Above Average Answer
Itsdb rated this answer Excellent or Above Average Answer

Question/Answer
excon asked on 07/08/05 - Level 3 Sex Offender


Hello experts:

I don't understand something. But, I'm really dumb, so maybe you can explain it to me.

Here we have a level 3 sex offender (many of 'em) who shouldn't be walking the streets because they might offend again. So, we track 'em - or try to. And, they offend again - Duh!!!!!

So, if they shouldn't be on the streets, why the hell are they? The government doesn't mind giving people long sentences. Oh, oh, that's right - only pot smokers get long sentences.

Do you feel better protected when a pot smoker goes in, but to make room for him, a sexual deviant get's out. Something is screwed up here - and I know what it is - do you?

excon

tomder55 answered on 07/09/05:

I have proposed the 'mandatory castration for level 3 sex offenders act ',but no one has taken up my idea in the legislature. 5th Amendment violation ? I don't think so. If I am not mistaken ,the long sentences for pot smokers comes because some States that have 3 strikes and your out guidelines. I do not see the logic in them . The punishment should fit the crime in all sentencing guidelines.

I could not find definitive #s .There is a relatively low # of prisoners serving in the Fed. system but State and local #s were difficult to get. It was also not clear if pot possession was the only crime being punished. Stats say about 87% of arrests were for possession ,and only around 13 %for sale .Those #s seem scewed . The emphasis should be in interdiction and sales of the drug.
Would we be better served by keeping violent criminals in prison longer by freeing up beds now used by marijuana offenders? Yes;but that does not change my opinion on the legality of the drug .

excon rated this answer Excellent or Above Average Answer
sapphire630 rated this answer Excellent or Above Average Answer

Question/Answer
excon asked on 07/07/05 - A Patriot - jailed


Hello experts:

Today, a patriot is going to jail. She's doing it for you - and me. Without her sacrifice, your news would come in the form of press releases.

I'm glad somebody has the balls (whatever, dude) to stand up for what they believe in. More importantly, for standing up for the founding fathers, the Constitution and our beloved country.

Go ahead - argue with me.

excon

tomder55 answered on 07/08/05:

There is no Fed. Shield law . If they had one I would not object to it ,but they don't and as I explained before ;I do not consider the reporter /source confidentiality concept as absolute. It is not priest -penetant ;doctor- patient (see Rush Limbugh);or lawyer -client ;none which are absolute either in the eye of the law.

Yes ;they have a first Amendment right to publish . That does not mean that they have the right to act above the law . Everyone when supoenaed has to give testimony to a Grand Jury ;including the President and his staff (all of whom have testified in this case) .

Does my right to publish here or on a blog preclude me from having to testify when called to do so ?

excon rated this answer Excellent or Above Average Answer
Itsdb rated this answer Excellent or Above Average Answer

Question/Answer
Itsdb asked on 07/07/05 - Series of Explosions Rock London

Thursday, July 07, 2005

LONDON Multiple explosions shook London on Thursday morning, as a series of seven or eight blasts were reported on the city's subways and buses.

At least two people were killed and nine injured in the nearly simultaneous blasts, and officials shut down the entire underground transport network. Media reports said the number of casualties was about 90 people.

"There have been a number of dreadful incidents across London today," said Home Secretary Charles Clarke (search), Britain's top law enforcement officer. He said there were "terrible injuries."

The near simultaneous explosions came a day after London was awarded the 2012 Olympics and as the G-8 summit was getting underway in Scotland. Initial reports blamed a power surge but speculation that it was a terrorist attack is strong.

"We are concerned this is a coordinated attack," London Police Chief Ian Blair told Sky News. He cautioned, however, against reaching premature conclusions. "We really need to understand what we're dealing with."

An explosion destroyed a double-decker bus near Russell Square (search) not long after several blasts were reported on London subways, police said. A witness said the entire top deck of the bus was destroyed.

"I was on the bus in front and heard an incredible bang, I turned round and half the double decker bus was in the air," Belinda Seabrook told Press Association (search), the British news agency.

She said the bus was packed with people.

"It was a massive explosion and there were papers and half a bus flying through the air," she said.

Officials shut down the entire underground network after the explosions. Initial reports blamed a power surge, but officials were not ruling out an intentional attack.

The attacks came a day after London was awarded the 2012 Olympics and as the G-8 (search) summit was getting underway in Scotland.

One witness, Darren Hall, said some passengers emerging from an evacuated subway station and that some passengers had soot and blood on their faces. He told BBC TV that he was evacuated along with others near the major King's Cross station and only afterward heard a blast.

British police reported that there were still no confirmed deaths, but that serious injuries were reported.

There was no immediate official comment from British Prime Minister Tony Blair, who was hosting the world's most powerful leaders at Gleneagles, Scotland. It was not clear if the G-8 gathering focusing on climate change and aid for Africa -- but from which Iraq has largely been left off the agenda -- would have to be postponed.

Police said incidents were reported at the Aldgate station near the Liverpool Street railway terminal, Edgware Road and King's Cross in north London, Old Street in the financial district and Russell Square in central London, near the British Museum.

London Ambulance Service said several vehicles had been dispatched to the area near Liverpool Street station.

"We believe there was some sort of explosion. There are some walking wounded at Aldgate," said a spokesman for City of London police, speaking on condition of anonymity.

"We are not sure of the scale of the incident. Reports are still coming in."

Bradley Anderson, a subway passenger, told Sky News that "there was some kind of explosion or something" as his train reached the Edgware Road station in northeast London.

"Everything went black and we collided into some kind of oncoming train," Anderson said.

Simon Corvett, 26, who was on an eastbound train from Edgware Road station, said: "All of sudden there was this massive huge bang."

"It was absolutely deafening and all the windows shattered," he said. "There were just loads of people screaming and the carriages filled with smoke.

"You could see the carriage opposite was completely gutted," he said. "There were some people in real trouble."

~~~~~~~~~~~~~~~~~~~~~~~~~~~~~~~~~~~~~~~~~~~~~~~~~~~~~~

First, this makes me sad - my prayers are with the British.

Second, this is just absolutely barbaric. If I hear one more Senator talk about how America's image is being tarnished over Gitmo I think I'm going to throw up. This is what the outrage is, this is what we're fighting for, this is why we're holding terrorists, and I think it time for the Bush bashers and leftists everywhere to drop the sensitivity and theocracy crap in America and take a stand against this Islamic extremist terrorist barbarism.

May God bless the British people, give healing and peace to the victims and protection from this evil.

Steve

tomder55 answered on 07/07/05:

England is a nation who's character was forged by events like the defeat of the Spanish Armada ;Trafalgar ;Wellington at Waterloo ;Dunkirk ;and the London Blitz. I hope the Brits of today have the character of their forebearers .....then again ...they have put up with the rants of Abu Hamza al-Masri (Captain Hook )for years.

Itsdb rated this answer Excellent or Above Average Answer

Question/Answer
excon asked on 07/04/05 - Poker


Hello experts:

On-line poker is illegal. Why don't we do anything about it? People go on television and are proud of the fact that they gambled on-line - right here in America.

Is gambling ok now, since the king of virtue, Bill Somethingorother, gambles his ass off? What if he smoked pot? Would that be ok now too?

Doesn't disregarding that law weaken our respect for the other laws that we're serious about? For the record, I gamble - I smoke pot - and I voted.

Eeegads, a felon voting? Guess, I got no respect.

excon

tomder55 answered on 07/05/05:

Doesn't disregarding that law weaken our respect for the other laws that we're serious about? not necessarily some laws are just plain wrong and violating them is an appropriate form of protest ,so long as you realize the inherent risks in breaking the law.Many a patriot served time in jail.

That Bill would be Bill Bennett. Although he could be open to charges of hypocracy ;he did nothing illegal . I do not have an opinion about on-line gambling .The gvt. bans it but runs number games with odds any Vegas regular would laugh at .

excon rated this answer Excellent or Above Average Answer
ladybugca rated this answer Excellent or Above Average Answer

Question/Answer
excon asked on 07/04/05 - Happy Birthday, America!


Boy, I love this country.

excon

tomder55 answered on 07/05/05:

On July 4 1826, Adams and Jeffersonbitter foes early in their lives died within hours of each other. Ironically, the date of their deaths marked the 50th anniversary of the signing of the Declaration of Independence.

Jefferson, 83, and Adams, 89, struggled to hang on until the 4th.

By running second to George Washington in the Electoral College balloting of 1788, Adams became the nations first vice president.

And by the same methoda 1796 second place finish in the Electoral College vote Jefferson was elected vice president to serve under Adams, the elections winner and second U.S. president.

Adams eight years as Washingtons VP provided him with an excellent foundation to serve the country. But replacing the beloved Washington was too tough an act for Adams to follow. The people wanted Washington to serve forever.

By the end of Adams first term, Jefferson openly criticized Adams, calling him a leader who was "distrustful, obstinate, excessively vain" and who "takes no counsel from anyone."

In the contentious 1800 election, said to be the birthplace of negative campaigning, Jefferson prevailed over the incumbent Adams.


According to accounts Jefferson, who had been drifting in and out of consciousness for weeks, awoke on the 4th, asked only, "Has July 4th arrived?" then died at his Monticello home at 12:50 P.M.

Five hundred miles to the northeast in Quincy, Massachusetts, Adams uttered his last words just moments before Jefferson passed away: "Jefferson survives!"

In their final years the two former presidents, once intense political rivals, mellowed toward each other and carried on an extendsive letter writing friendship.
Adams and Jefferson recognized that they were both dedicated to the same causes;independence and a grand and glorious America.







It is said Patriotism is the last refuge of a scoundrel(Samuel Johnson );

From one scoundrel to another ;I hope you had a great day .

excon rated this answer Excellent or Above Average Answer
ladybugca rated this answer Excellent or Above Average Answer

Question/Answer
Itsdb asked on 07/01/05 - Foreigners blamed for suicide attacks

U.S., Iraqi officials say bombers recruited in gulf region, N. Africa

Associated Press. Published July 1, 2005

BAGHDAD -- The vast majority of suicide attackers in Iraq are thought to be foreigners--mostly Saudis and other Persian Gulf Arabs--and the trend has become more pronounced this year with North Africans also streaming in to carry out deadly missions, U.S. and Iraqi officials say.

The bombers are recruited from Sunni communities, smuggled into Iraq from Syria after receiving religious indoctrination and then quickly bundled into cars or strapped with explosive vests and sent to their deaths, the officials said. The young men are not so much fighters as human bombs, a small but deadly component of the Iraqi insurgency.

"The foreign fighters are the ones that most often are behind the wheel of suicide car bombs, or most often behind any suicide situation," said U.S. Air Force Brig. Gen. Don Alston, spokesman for the Multinational Force in Iraq.

Officials have long believed that non-Iraqis infiltrating the country through its porous borders with Syria, Iran and Saudi Arabia were behind most suicide missions, and the wave of bloody strikes in recent months has confirmed that thinking...

...Progress against infiltration is crucial in reducing the number of suicide bombings. Authorities have found little evidence that Iraqis have been behind the near-daily stream of such attacks over the past six months, U.S. and Iraqi intelligence officials said, speaking on condition of anonymity.

Since 2003, less than 10 percent of more than 500 suicide attacks have been carried out by Iraqis, according to one defense official. So far this year, there have been at least 213 suicide attacks--172 by vehicle and 41 by bombers on foot--according to an Associated Press count.

Another U.S. official said American authorities believe Iraqis are beginning to look at suicide bombers as a liability. "Just as there is no shortage of people willing to do this, nor is there any shortage of targets, and they tend to be police," the official said.

The trend does not mean Iraqis are not part of the bloody insurgency: On the contrary, Iraqi insurgents are thought to be responsible for much of the violence and fighting in the country, although most of those are non-suicide attacks.

"I still think 80 percent of the insurgency, the day-to-day activity, is Iraqi--the roadside bombings, mortars, direct weapons fire, rifle fire, automatic-weapons fire," said Kenneth Katzman, a Middle East expert with the Congressional Research Service, which advises U.S. lawmakers.

But he added: "The foreign fighters attract the headlines with the suicide bombings, no question."

The key role of foreign fighters in suicide attacks is one reason many senior military officials, including the top U.S. general in the Middle East, tend to view the war in Iraq as slowly developing into an international struggle against militant Islam.

The military brass say Islamic extremists like Abu Musab al-Zarqawi and his Al Qaeda in Iraq organization are determined to start a civil war by attacking Iraqi security forces and members of the country's Shiite majority.

"It's not about one man. It's about his network," the top general in the region, U.S. Gen. John Abizaid, said recently. "His network exists inside Iraq. It's connected to Al Qaeda. It's got facilitation nodes in Syria. It brings foreign fighters in from Saudi Arabia and from North Africa..."

...Overall, the number of foreign fighters coming into the country seems to be increasing from what it was six months ago, Abizaid said. "There's probably about 1,000 foreign fighters and about somewhere less than 10,000 committed insurgents in the field," he said...

...In interviews while visiting prisons, militant groups and government officials, he was told that there were so many suicide bombers coming out of the Persian Gulf states that the loose networks that deploy them--many run through mosques--had to turn away potential attackers.

~~~~~~~~~~~~~~~~~~~~~~~~~~~~~~~~~~~~~~~~~~~~~~~~~~~~~~~

In another version of the story...

BAGHDAD, Iraq -- The majority of suicide attackers in Iraq are thought to be foreigners -- mostly Saudis and other Gulf Arabs -- and the trend has become more pronounced, U.S. and Iraqi officials say.

"The foreign fighters are the ones that most often are behind the wheel of suicide car bombs, or most often behind any suicide situation," said U.S. Air Force Brig. Gen. Don Alston, spokesman for the Multinational Force in Iraq.

Since 2003, less than 10 percent of more than 500 suicide attacks have been carried out by Iraqis, according to one defense official.

So far this year, there have been at least 213 suicide attacks -- 172 by vehicle and 41 by bombers on foot -- according to an Associated Press count.

Authorities have found little evidence that Iraqis have been behind the near-daily stream of suicide attacks during the past six months, U.S. and Iraqi intelligence officials said, speaking on condition of anonymity because of the subject's sensitivity.

There have been a few exceptions.

On Jan. 30, Election Day in Iraq, an Iraqi boy, wearing a suicide vest, attacked a polling station. An attack on a U.S. military mess hall in the northern city of Mosul in December that killed 22 also was believed to have been carried out by an Iraqi, as was a June 11 attack on the Baghdad headquarters of the Interior Ministry's Wolf Brigade..."

~~~~~~~~~~~~~~~~~~~~~~~~~~~~~~~~~~~~~~~~~~~~~~~~~~~~~~~

Just a note here, the version in my paper this morning read, "On election day Jan. 30, a mentally handicapped Iraqi boy, wearing a suicide vest, attacked a polling station."

I guess the AP has had a partial recovery from their coma and realized what many of us have been saying for some time now.

Oh yes, a question...isn't this why we're fighting the global war on terror? Add to this the new Iranian president's "wave of the Islamic revolution will soon reach the entire world" comment, is the picture getting any clearer? Perhaps some day we won't have to read about mentally ill children being turned into bombs?

Steve

tomder55 answered on 07/03/05:

I don't know what happened to my response yesterday . Try again :

U.S.: Insurgents no longer capable of combat operations



SPECIAL TO WORLD TRIBUNE.COM
Friday, July 1, 2005
BAGHDAD The U.S. military reported that Sunni insurgents have lost the capability to sustain combat operations against coalition forces in Iraq.

Officials said the insurgents, most of them commanded by Al Qaida, have resorted to suicide bombings and roadside explosions rather than confront Iraqi and U.S. forces. They said the reduction in insurgency assaults has taken place over the last eight months.

"I think that the ability of the enemy to sustain high-volume attacks is just something that we haven't seen," Brig. Gen. Donald Alston, a Multinational Force Iraq spokesman, said in a briefing on Thursday.



Alston said the core of the insurgency amounted to several hundred people, Middle East Newsline reported. He said the total number of insurgents could reach 20,000, with about five percent of them being foreigners.
On Thursday, more than 1,000 Iraqi and U.S. troops continued Operation Sword in the Sunni Triangle city of Hit. So far, 13 suspected insurgents were said to have been captured.

"Resistance is being reported by commanders in the city as light," a U.S. military statement said. "No foreign fighter presence has been detected within the city."

Maj. Gen. James Conway, commander of the 1st Marine Expeditionary Unit, said Operation Sword and similar missions were meant to deny insurgency sanctuaries in the Anbar province near the Syrian border. Conway said the U.S.-led coalition was also trying to encourage Iraqis not to cooperate with the insurgents.

"It tells the insurgent that there will be none of those types of sanctuary in Al Anbar," Conway told a Pentagon briefing on Thursday. "It passes a message to the local civilians that we're going to try to ensure that you are not intimidated by the presence of these forces and that it kills terrorists that are coming in from the Syrian border."

Officials said the last major insurgency confrontation took place in Faluja in November 2004. They said U.S. forces drove thousands of insurgents a combination of Al Qaida operatives and former Saddam Hussein officers from the city.

In the spring of 2005, Sunni insurgents began to increasingly employ suicide bombers and roadside explosives, officials said. They said many car bombers were thwarted because of a steadily improving intelligence network and poorly-assembled explosives that failed to detonate.

"We see increased use of suicide and VBIEDs [vehicle borne IEDs] the thing that gives you the big blast and possibly causes more casualties, seems to be more media newsworthy and believe me, they're very sensitive to that," Conway said. "We see that in everything that they're doing. At the same time, those types of blasts are having less effect on our troops."

Itsdb rated this answer Excellent or Above Average Answer

Question/Answer
Itsdb asked on 07/01/05 - A note from John Kerry

Dear Steve,

The Fourth of July is a time for family, fun and fireworks.

But something happened today that ought to remind everyone what this holiday really symbolizes -- the freedom that makes America great.

That's exactly what hangs in the balance now that Sandra Day O'Connor has resigned from the Supreme Court.

This is no small deal. Over and over, she was the Justice who cast the critical vote in 5-4 cases deciding the most important issues in our nation.

Here's our bottom line for the johnkerry.com community heading into the holiday weekend: we can never let her be replaced by a Justice who does not respect the right to privacy and Roe v. Wade, and who doesn't understand the freedoms protected in our Constitution.

So, this weekend, as you enjoy the Fourth -- take a minute to think about what it means, and come back on Tuesday morning ready to fight for our freedom. It's all at stake now, and we need to come together more than ever.

Get ready,

John Kerry

~~~~~~~~~~~~~~~~~~~~~~~~~~~~~~~~~~~~~~~~~~~~~~~~~~~~~~~

I guess this is another one of those "extraordinary circumstances."

I read just this morning how everyone on both sides has been geared up for this for months. People for the American Way in particular, putting vacation plans on hold, preparing commercials and lists of every conceivable nominee - ready to destroy the lucky winner. Funny that Kerry's letter confirms that...

So what are the chances of a Bush man/woman taking her place? Who's going to be the nominee?

Steve

tomder55 answered on 07/02/05:

Bush should appoint Justice Janice Rogers Brown to the Supreme court. But he will not do so. I hope he doesn't listen to those who are saying that O'Conner was a swing vote so therefore he needs to appoint another . Let them try to fillibuster. At least 2 of the so called moderate Republicans said they would go nuclear if the Democrats dod not live up to their end of the bargain. Also ;one different procedure . The nomination cannot be held up in committee. Even a negative vote in the committee has to go to the floor .

excon rated this answer Excellent or Above Average Answer
Itsdb rated this answer Excellent or Above Average Answer

Question/Answer
Itsdb asked on 07/01/05 - Foreigners blamed for suicide attacks

U.S., Iraqi officials say bombers recruited in gulf region, N. Africa

Associated Press. Published July 1, 2005

BAGHDAD -- The vast majority of suicide attackers in Iraq are thought to be foreigners--mostly Saudis and other Persian Gulf Arabs--and the trend has become more pronounced this year with North Africans also streaming in to carry out deadly missions, U.S. and Iraqi officials say.

The bombers are recruited from Sunni communities, smuggled into Iraq from Syria after receiving religious indoctrination and then quickly bundled into cars or strapped with explosive vests and sent to their deaths, the officials said. The young men are not so much fighters as human bombs, a small but deadly component of the Iraqi insurgency.

"The foreign fighters are the ones that most often are behind the wheel of suicide car bombs, or most often behind any suicide situation," said U.S. Air Force Brig. Gen. Don Alston, spokesman for the Multinational Force in Iraq.

Officials have long believed that non-Iraqis infiltrating the country through its porous borders with Syria, Iran and Saudi Arabia were behind most suicide missions, and the wave of bloody strikes in recent months has confirmed that thinking...

...Progress against infiltration is crucial in reducing the number of suicide bombings. Authorities have found little evidence that Iraqis have been behind the near-daily stream of such attacks over the past six months, U.S. and Iraqi intelligence officials said, speaking on condition of anonymity.

Since 2003, less than 10 percent of more than 500 suicide attacks have been carried out by Iraqis, according to one defense official. So far this year, there have been at least 213 suicide attacks--172 by vehicle and 41 by bombers on foot--according to an Associated Press count.

Another U.S. official said American authorities believe Iraqis are beginning to look at suicide bombers as a liability. "Just as there is no shortage of people willing to do this, nor is there any shortage of targets, and they tend to be police," the official said.

The trend does not mean Iraqis are not part of the bloody insurgency: On the contrary, Iraqi insurgents are thought to be responsible for much of the violence and fighting in the country, although most of those are non-suicide attacks.

"I still think 80 percent of the insurgency, the day-to-day activity, is Iraqi--the roadside bombings, mortars, direct weapons fire, rifle fire, automatic-weapons fire," said Kenneth Katzman, a Middle East expert with the Congressional Research Service, which advises U.S. lawmakers.

But he added: "The foreign fighters attract the headlines with the suicide bombings, no question."

The key role of foreign fighters in suicide attacks is one reason many senior military officials, including the top U.S. general in the Middle East, tend to view the war in Iraq as slowly developing into an international struggle against militant Islam.

The military brass say Islamic extremists like Abu Musab al-Zarqawi and his Al Qaeda in Iraq organization are determined to start a civil war by attacking Iraqi security forces and members of the country's Shiite majority.

"It's not about one man. It's about his network," the top general in the region, U.S. Gen. John Abizaid, said recently. "His network exists inside Iraq. It's connected to Al Qaeda. It's got facilitation nodes in Syria. It brings foreign fighters in from Saudi Arabia and from North Africa..."

...Overall, the number of foreign fighters coming into the country seems to be increasing from what it was six months ago, Abizaid said. "There's probably about 1,000 foreign fighters and about somewhere less than 10,000 committed insurgents in the field," he said...

...In interviews while visiting prisons, militant groups and government officials, he was told that there were so many suicide bombers coming out of the Persian Gulf states that the loose networks that deploy them--many run through mosques--had to turn away potential attackers.

~~~~~~~~~~~~~~~~~~~~~~~~~~~~~~~~~~~~~~~~~~~~~~~~~~~~~~~

In another version of the story...

BAGHDAD, Iraq -- The majority of suicide attackers in Iraq are thought to be foreigners -- mostly Saudis and other Gulf Arabs -- and the trend has become more pronounced, U.S. and Iraqi officials say.

"The foreign fighters are the ones that most often are behind the wheel of suicide car bombs, or most often behind any suicide situation," said U.S. Air Force Brig. Gen. Don Alston, spokesman for the Multinational Force in Iraq.

Since 2003, less than 10 percent of more than 500 suicide attacks have been carried out by Iraqis, according to one defense official.

So far this year, there have been at least 213 suicide attacks -- 172 by vehicle and 41 by bombers on foot -- according to an Associated Press count.

Authorities have found little evidence that Iraqis have been behind the near-daily stream of suicide attacks during the past six months, U.S. and Iraqi intelligence officials said, speaking on condition of anonymity because of the subject's sensitivity.

There have been a few exceptions.

On Jan. 30, Election Day in Iraq, an Iraqi boy, wearing a suicide vest, attacked a polling station. An attack on a U.S. military mess hall in the northern city of Mosul in December that killed 22 also was believed to have been carried out by an Iraqi, as was a June 11 attack on the Baghdad headquarters of the Interior Ministry's Wolf Brigade..."

~~~~~~~~~~~~~~~~~~~~~~~~~~~~~~~~~~~~~~~~~~~~~~~~~~~~~~~

Just a note here, the version in my paper this morning read, "On election day Jan. 30, a mentally handicapped Iraqi boy, wearing a suicide vest, attacked a polling station."

I guess the AP has had a partial recovery from their coma and realized what many of us have been saying for some time now.

Oh yes, a question...isn't this why we're fighting the global war on terror? Add to this the new Iranian president's "wave of the Islamic revolution will soon reach the entire world" comment, is the picture getting any clearer? Perhaps some day we won't have to read about mentally ill children being turned into bombs?

Steve

tomder55 answered on 07/02/05:

The U.S. military reported that Sunni insurgents have lost the capability to sustain combat operations against coalition forces in Iraq.

Officials said the insurgents, most of them commanded by Al Qaida, have resorted to suicide bombings and roadside explosions rather than confront Iraqi and U.S. forces. They said the reduction in insurgency assaults has taken place over the last eight months.

"I think that the ability of the enemy to sustain high-volume attacks is just something that we haven't seen," Brig. Gen. Donald Alston, a Multinational Force Iraq spokesman, said in a briefing on Thursday.



Alston said the core of the insurgency amounted to several hundred people, Middle East Newsline reported. He said the total number of insurgents could reach 20,000, with about five percent of them being foreigners.
On Thursday, more than 1,000 Iraqi and U.S. troops continued Operation Sword in the Sunni Triangle city of Hit. So far, 13 suspected insurgents were said to have been captured.

"Resistance is being reported by commanders in the city as light," a U.S. military statement said. "No foreign fighter presence has been detected within the city."

Maj. Gen. James Conway, commander of the 1st Marine Expeditionary Unit, said Operation Sword and similar missions were meant to deny insurgency sanctuaries in the Anbar province near the Syrian border. Conway said the U.S.-led coalition was also trying to encourage Iraqis not to cooperate with the insurgents.

"It tells the insurgent that there will be none of those types of sanctuary in Al Anbar," Conway told a Pentagon briefing on Thursday. "It passes a message to the local civilians that we're going to try to ensure that you are not intimidated by the presence of these forces and that it kills terrorists that are coming in from the Syrian border."

Officials said the last major insurgency confrontation took place in Faluja in November 2004. They said U.S. forces drove thousands of insurgents a combination of Al Qaida operatives and former Saddam Hussein officers from the city.

In the spring of 2005, Sunni insurgents began to increasingly employ suicide bombers and roadside explosives, officials said. They said many car bombers were thwarted because of a steadily improving intelligence network and poorly-assembled explosives that failed to detonate.

"We see increased use of suicide and VBIEDs [vehicle borne IEDs] the thing that gives you the big blast and possibly causes more casualties, seems to be more media newsworthy and believe me, they're very sensitive to that," Conway said. "We see that in everything that they're doing. At the same time, those types of blasts are having less effect on our troops."


Itsdb rated this answer Excellent or Above Average Answer

Question/Answer
sapphire630 asked on 06/30/05 - agenda 21



http://www.sovereignty.net/p/sd/a21/


this is unbelievable, worse than communism and just some of the globalization we are headed for.

tomder55 answered on 07/01/05:

Signed by the U.S. during the Earth Summit at Rio de Janeiro in 1992, Agenda 21 is a 40-chapter manifesto to reorganize the world using socialist and pantheistic principles to protect Earth .

Agenda 21 represents a major fundamental change in the role of government in social and land-use policy. Under its concept of sustainability, the primary purpose of government will no longer be to serve the people. Rather, the focus of Agenda 21 is to protect nature from people. Governance will be by consensus among "stakeholders and partnerships." The concept of elected representation that holds the government accountable to the citizens will be eliminated.

Agenda 21 requires that by 2000 "All States...have designed and initiated costed and targeted national action programmes, and to have put in place appropriate institutional structures and legal instruments to implement Agenda 21. The Clinton Administration responded creating the President's Council on Sustainable Development which published its report entitled Sustainable America in 1996 .

President Bush refused to attend a follow up meeting in Johannesburg in 2002 which did not make the environmentalists happy ;Colin Powell went instead.


sapphire630 rated this answer Excellent or Above Average Answer

Question/Answer
sapphire630 asked on 06/30/05 - why did Texas get away with religious display

3. Supposedly heritage/culture display as opposed to 'religious' display

2. If the Supreme Court bans ALL religious displays
in political places they will have to paint over their own painting of Moses with the 10 commandments.

1. YOU DON'T MESS WITH TEXAS!!!

tomder55 answered on 07/01/05:

you see there was this Cecil B. DeMille film that had a bunch of left over props. ;they were distributed around the country for promo purposes and Texas got their hands on one of em. They did not know what to do with em...too big for a paperweight in the office so they leave it outside hoping that someone will pick it up as curbside treasure. Nobody did so it hung around there as a picnic bench all these years.This homeless guy who caint read worth a lick passes it every day on his way to collect his free guvernmet cheese somehow takes offense to them ;or more likely some ACLU guy tells him he can makes a ton of money ifn he sues the gvnment for discriminatin cause he dont understand all them shalls and shall nots. He argues that it would be much simple if they read like this :

(1) Just one God.
(2) Honor yer Ma & Pa.
(3) No telling tales or gossipin'.
(4) Git yourself to Sunday meeting.
(5) Put nothin' before God..
(6) No foolin' around with another fellow's gal.
(7) No killin'.
(8) Watch yer mouth.
(9) Don't take what ain't yers.
(10) Don't be hankerin' for yer buddy's stuff

the Supreme court decides that if you leave em as a picnic bench thar aint anything wrong with them so long as you leave other picnic benches with the code of Hammurabi; the 10 planks of the Communist manifeso and maybe some other movie props and rusted pickup trucks in your yard to balance em.

Itsdb rated this answer Excellent or Above Average Answer
sapphire630 rated this answer Excellent or Above Average Answer

Question/Answer
excon asked on 06/30/05 - More for you Righty's


Helloooo:

Wasssup?? Freedom of the press doesn't interest you? The silence is deafening over there. This is as basic a freedom as we have. And the loss of it is an equal defeat for freedom as was the eminent domain shaft.

Do you think that crook Nixon would have gotten away with hijacking the country if this ruling would have been in play? Wed probably all be required to give the Nixon Heil!

What? Because the reporters facing jail are from liberal rags? The slime who should be going to jail is that Novak jerkoff, and the asshole in the White House who called him.

Hey - if you don't speak up now, next time it will be your friend Rush Limprod facing the slamola.

excon

tomder55 answered on 06/30/05:

well I did post about the Valeri Plame situation when it first came up . Right off the bat I hope Judith Miller of the New York Times and Matthew Cooper of Time magazine stay out of jail.I do not dispute the rational that the judge uses on giving them a possible contempt of court ,but I think the whole case just aint worth all the trouble . Novak was the one who outed her and he is off because of some deal he made to cooperate in the investigation . U.S. Attorney Patrick Fitzgerald apparently has information that the two journalists talked to someone who offered to leak to them. Fitzgerald wants to know who that someone was.

Remember ;this is an investigation that all the left (and some hot heads in the CIA who were sore at Potter Goss medling in their sanctuary )was howling for .They were out for blood because they believe there was some crime in disclosing Plame's name (which there isn't really because Plame is not a field agent and eveyone and their mother new she worked for the CIA).So now a prosecutor is doing an investigation ;what did they expect? They were the ones who put meat into the investigation by overplaying their hand .The Adminstration had no choice but to put an aggressive investigation together .

I am sympathetic to their claim that they have a right under the First Amendment to protect their confidential sources. The problem is, every court that has heard the claim has rejected it including the fools in black .Does the First Amendment protect reporters from testifying ? In a 1972 case the Supreme Court said reporters have no such privilege.

What the Dems.should've done was what they did during Iran -Contra. ;run Congressional hearings ;give everyone immunity and let them spill the beans . That would've had the effect they desire ... to embarass the Administration.

It appears that Time Mag. gave Cooper a face saving way out . They decided to give his notes to the prosecution .If the NY Times does not follow that example it could very well mean that Miller will spend some time behind bars .Myron Farber ,another NY times reporter spent 40 days in jail in 1978 in a simular case ,so it is not without precident (precident .....isn't that a wonderful word? )

The press wants absolute privilege to refuse to reveal the source of any story under any circumstance .Only the the clergy-penitent relationship should have that privilage .

excon rated this answer Excellent or Above Average Answer
Itsdb rated this answer Excellent or Above Average Answer
jnlomonte rated this answer Excellent or Above Average Answer

Question/Answer
excon asked on 06/29/05 - Question for tomder - and everybody else too


Hello tom:

I read with interest your referrals. I also read with interest many publications that deny, absolutely, that there was any connection between Saddam and 9/11.

How come everybody is entrenched in their beliefs? It would seem to me that proof is available, but both sides seem to have proof.

Saddam did or didn't have anything to do with 9/11. How come we (all of us) don't KNOW, absolutely, whether he did or not? I think the truth is pretty important here.

It's like history. We seem to know what the generals did and what they were thinking during the Revolutionary War. Why dont we KNOW what went on there? Are we so blinded by partisanship that the truth evades us? And, if we are, there's no hope for us.

Because it makes all the difference to me. If Saddam attacked us, then he deserved to be attacked back.

excon

tomder55 answered on 06/30/05:

For clarification ;I posted some that connected Saddam to 9-11 but I can understand that the evidence is not overwhelming alhtough I consider the visit of Atta with Iraqi intelligence in Prague as a smoking gun.

I have other connections that fuel my suspicions ;including an Iraqi connection to the first WTC bombing .Saddam did harbor Abdul Yasin who is charged with helping mix chemicals for the bomb .A 60 Minutes news crew revealed that when they traveled to Baghdad in 2002 to interview him. Saddam had even pressured Jordan to release Yasin when Jordan had briefly detained the bomber.In addition ;Laurie Mylroie, formerly of Harvard and the U.S. Naval War College, details Ramzi Yousef,the mastermind of the first WTC bombing, connection to Saddam in "The World Trade Center Bomb: Who is Ramzi Yousef? And Why it Matters" .This essay was published in 1995 and is fully sourced .

There are some real evidence linking saddam with the Oklahoma City Bombing but one thing at a time.(for more on this read :The Third Terrorist : The Middle Eastern Connection to the Oklahoma City Bombing by Jayna Davis )

The evidence of Saddam's connections and cooperation with al-Qaeda is so convincing as to make them in my book indesputable .I did not even come close to documenting all of them yesterday . Here is another example from the Guardian October 19, 2000 ;before Bush was elected :
Investigators in Yemen yesterday uncovered evidence suggesting the bomb attack on the warship USS Cole had been a meticulously organised conspiracy, which a leading US terrorism expert said may have been the first joint operation between Osama bin Laden and Saddam Hussein....

Bin Laden, a Saudi national based in Afghanistan, has Yemeni family roots and close links with some of the local tribal warlords. A few weeks before the attack, he distributed a video in which he issued familiar calls for a holy war against the "forces of evil". He was wearing Yemeni tribal costume and a Yemeni dagger.

"He's puckish like that. On one hand he does not want to give out his address, but on the other hand, he likes to let his followers know he is leading the fight," Mr Cannistraro said.

He argued that the sophistication of the bomb - an estimated 272kg of high explosive shaped and placed within a metal container to channel the blast and penetrate the armoured hull of the USS Cole - suggested the involvement of a state.

"The Iraqis have wanted to be able to carry out terrorism for some time now," Mr Cannistraro said. "Their military people have had liaison with al-Qaeda in Afghanistan, and could well have supplied the training."


The question :What if the extensive state resources needed to infiltrate the Aden port operation and gather intelligence on the Cole's movements were mobilized by a state friendly enough to Yemen and hostile enough to the United States to achieve the bombing of the Cole?;was never adequately answered by the Clinton Adminstration. The Cole was in the region for one reason and one reason only ;to enforce the UN sanctions against Iraq.

Much of the evidence is detailed, conclusive, and corroborated by multiple sources. Some of it is new information obtained in interviews with high-level al Qaeda terrorists and Iraqi officials that have been captured and detained and some of it is more than a decade old. The picture that emerges is one of a history of collaboration .Iraq-al Qaeda contacts began in 1990 and continued through mid-March 2003, days before OIF began. A good source to look at is the book by Stephen Hayes The Connection : How al Qaeda's Collaboration with Saddam Hussein Has Endangered America .He writes in The Weekly Standard :
At some unspecified point in 1991, according to a CIA analysis, "Iraq sought Sudan's assistance to establish links to al Qaeda." The outreach went in both directions. According to 1993 CIA reporting cited in the memo, "bin Laden wanted to expand his organization's capabilities through ties with Iraq." The primary go-between throughout these early stages was Sudanese strongman Hassan al-Turabi, a leader of the al Qaeda-affiliated National Islamic Front. Numerous sources have confirmed this. One defector reported that "al-Turabi was instrumental in arranging the Iraqi-al Qaeda relationship. The defector said Iraq sought al Qaeda influence through its connections with Afghanistan, to facilitate the transshipment of proscribed weapons and equipment to Iraq. In return, Iraq provided al Qaeda with training and instructors."

One such confirmation came in a postwar interview with one of Saddam Hussein's henchmen. As the memo details: . According to a May 2003 debriefing of a senior Iraqi intelligence officer, Iraqi intelligence established a highly secretive relationship with Egyptian Islamic Jihad, and later with al Qaeda. The first meeting in 1992 between the Iraqi Intelligence Service (IIS) and al Qaeda was brokered by al-Turabi. Former IIS deputy director Faruq Hijazi and senior al Qaeda leader [Ayman al] Zawahiri were at the meeting the first of several between 1992 and 1995 in Sudan. Additional meetings between Iraqi intelligence and al Qaeda were held in Pakistan. Members of al Qaeda would sometimes visit Baghdad where they would meet the Iraqi intelligence chief in a safe house. The report claimed that Saddam insisted the relationship with al Qaeda be kept secret. After 9-11, the source said Saddam made a personnel change in the IIS for fear the relationship would come under scrutiny from foreign probes." Just these lines alone are damning for those who deny the Iraq-Al Quaeda collaboration. But this is just the tip of the iceberg.




There is alot more too. What confirming evidence does the "other side " bring to the table ? Frequently they cite Richard Clarke

In The Age of Sacred Terror
Daniel Benjamin, the National Security Council's director for counterterrorism during the Clinton administration, and Steven Simon, its first senior director of counterterrorism, argue that OBL is merely a branch of a larger network of terrorism .In it they defend Clinton's decision to attack the aspirin factory in the Sudan with cruise missles.They clearly in the book point out that Clarke agreed with them about an Iraqi connection to the al-qaeda operation there . 'The Age of Sacred Terror 'justifies the Clinton attack on the grounds that "Iraqi weapons-scientists" were linked to Bin Laden's factory and that the chemical EMPTA, detected at the site, was used only by Iraq to make VX nerve gas. At the time, Clarke defended the bombing telling the press that he was "sure" that "intelligence existed linking bin Laden to Al Shifa's current and past operators, the Iraqi nerve gas experts and the National Islamic Front in Sudan." The U.N. arms inspector who corroborated the evidence implicating Saddam btw was David Kay.Clarke basically told the Washington Post in a Jan. 23, 1999 the same story . Now he flatly denies any connection .

For the record ;Clinton prempted in this attack ;the Sudanese regime had diplomatic relations with Washington and it had previously agreed to deport Bin Laden to Afghanistan (which was in retrospect, a mistake).Clinton went in without Congressional approval ;he did not seek Security Council approval ;there was no demand for inspections by the Hans Blix UN teams ........but it was the right thing to do .

Later that week ;Clinton sent in missles into Afghanistan in an attempt to assasinate bin-Laden. Richard Clarke joked "he probably escaped to Baghdad".

If you can come up with other credible rebutals I'd love to hear them . I hear alot of blanket parroting by people saying there was no connection but the evidence looks different to me .








excon rated this answer Excellent or Above Average Answer

Question/Answer
Itsdb asked on 06/29/05 - Speaking of Saddam and bin Laden...

It seems the main thing the left heard in Bush's speech were his references to 9/11:

"The president's frequent references to the terrorist attacks of September 11 show the weakness of his arguments. He is willing to exploit the sacred ground of 9/11, knowing that there is no connection between 9/11 and the war in Iraq" Nancy Pelosi

"I said this was an elective war ... Saddam wasn't a part of 9/11, Saddam didn't have WMD to threaten America, but now that we're there, we have to succeed" Wesley Clark

"The president's numerous references to September 11 did not provide a way forward in Iraq" Harry Reid

"We did not expect Mr. Bush would apologize for the misinformation that helped lead us into this war, or for the catastrophic mistakes his team made in running the military operation. But we had hoped he would resist the temptation to raise the bloody flag of 9/11 over and over again to justify a war in a country that had nothing whatsoever to do with the terrorist attacks...The president, who is going to be in office for another three and a half years, cannot continue to obsess about self-justification and the need to color Iraq with the memory of 9/11. The nation does not want it and cannot afford it." NY Times (also quoted by Al Jazeera)

"The president tried to link the Iraq campaign to the terrorist attacks of 9/11, even though there is no credible evidence that Saddam Hussein was involved in them. Nor was Iraq a terrorist haven before the U.S. invasion, though it has become one since." USA Today (also quoted by al Jazeera)

"President Bush's pep talk to the nation Tuesday night was a major disappointment. He again rewrote history by lumping together the terrorist attacks of Sept. 11, 2001, and the need for war in Iraq, when, in fact, Saddam Hussein's Iraq had no connection to Al Qaeda." LA Times

My analysis? It wouldn't have mattered what Bush said. Since I was up at 4:00 AM for some reason I watched a Democrat strategist - I don't recall her name - on Fox this morning let the cat out of the bag. She said something to the effect of "Bush didn't say he was wrong..." That's the whole thing right there. Bush didn't say he was wrong. USA Today backs me up on that:

"There was no acknowledgement of the misjudgments that many Americans now see plainly, but Bush seems unable or unwilling to recognize."

That's all they want, they just want Bush to say he was wrong - is that so hard? Just say you're wrong Mr. Bush and we can move on...to impeachment proceedings.

Am I wrong?

Steve

tomder55 answered on 06/29/05:

it would be nice if Bush could say "this is where we made mistakes ;misjudgements ;errors in planning "etc. and that he could be confident that the opposition wouldn't turn it into fodder.What war goes exactly as planned ? unfortunately that is not the case the headlines would be Bush admits mistake ;Dems. demand hearings and firings . Kennedy says :lie after lie after lie after lie after lie after lie after lie (hiccup).

CeeBee2 rated this answer Excellent or Above Average Answer
excon rated this answer Excellent or Above Average Answer
Itsdb rated this answer Excellent or Above Average Answer
labman rated this answer Excellent or Above Average Answer

Question/Answer
excon asked on 06/28/05 - The Pres


Hello experts:

I heard nothing new. But I did see a very human man, in the last 10 seconds. For the first time, I liked him.

You?

excon

tomder55 answered on 06/29/05:

The content of the speech was a restatement of the argument that the counterinsurgency in Iraq is part of the War on Terror, and that we must stay the course.

Many terrorists who kill innocent men, women and children on the streets of Baghdad are followers of the same murderous ideology that took the lives of our citizens in New York and Washington and Pennsylvania...

Some wonder whether Iraq is a central front in the war on terror. Among the terrorists, there is no debate.

Here are the words of Osama bin Laden: This third world war is raging in Iraq. The whole world is watching this war. He says it will end in victory and glory or misery and humiliation.

The terrorists know that the outcome will leave them emboldened or defeated. So they are waging a campaign of murder and destruction. And there is no limit to the innocent lives they are willing to take.



This is the sort of argument that Bush should be making every two or three months. It's the job of a president to rally the nation. These speeches need to be more frequent . He did well in stating what the Terrorists have failed to achieve :

The terrorists -- both foreign and Iraqi -- failed to stop the transfer of sovereignty.

They failed to break our Coalition and force a mass withdrawal by our allies.

They failed to incite an Iraqi civil war.

They failed to prevent free elections.

They failed to stop the formation of a democratic Iraqi government that represents all of Iraq's diverse population.

And they failed to stop Iraqis from signing up in large number with the police forces and the army to defend their new democracy.


The lesson of this experience is clear: The terrorists can kill the innocent, but they cannot stop the advance of freedom. The only way our enemies can succeed is if we forget the lessons of September the 11th, if we abandon the Iraqi people to men like Zarqawi, and if we yield the future of the Middle East to men like Bin Laden. For the sake of our nation's security, this will not happen on my watch.


My only complaint is that he did not hammer home the many successes that have been achieved . Since the press refuses to report it ,then it is up to the President to hammer these points home.




excon rated this answer Excellent or Above Average Answer
Itsdb rated this answer Excellent or Above Average Answer

Question/Answer
Choux asked on 06/27/05 - Is This Legal??

"Bush administration is planning new measures that would target the U.S. assets of anyone conducting business with a handful of Iranian, North Korean and Syrian companies believed by Washington to be involved in weapons programs, U.S. officials said yesterday.

"The latest action is outlined in a draft executive order administration officials are hoping President Bush will sign before attending the Group of Eight summit in Scotland on July 2. Officials who agreed to discuss the details only on the condition of anonymity said the order's success would rely heavily on U.S. intelligence and that it is modeled in part on measures the government took against al Qaeda's finances shortly after the terrorist strikes of Sept. 11, 2001". The Washington Post

tomder55 answered on 06/28/05:

American businesses and business people are convicted and fined, and American business people do go to jail for export resriction violations .There are both civil and criminal penalties.

In addition to fines and prison terms under criminal and civil sanctions, there are administrative sanctions (including denial of export privileges and exclusion from practice), statutory sanctions, seizure and forfeiture , cross-debarment, denial of licenses or approvals, and suspensions of the right to contract with the United States Government.

Choux rated this answer Excellent or Above Average Answer

Question/Answer
Choux asked on 06/27/05 - Forbidding Prayer at the Naval Academy??

Fahrenthold
Washington Post Staff Writer
Saturday, June 25, 2005; Page B05

*From the Jewish Anti-Defamation web site*

The Anti-Defamation League has asked the U.S. Naval Academy to stop holding prayers before midshipmen eat lunch, saying the practice is an unconstitutional endorsement of religion.

The request was made in a June 17 letter from Abraham H. Foxman, the league's national director, to the academy's superintendent, Vice Adm. Rodney P. Rempt.


In the letter, Foxman says the constitutional separation of church and state is violated "when 4,000 midshipmen of many different faiths are brought together for compulsory prayer."

As precedent, the letter cites a recent ruling by a federal appeals court that organized mealtime prayers at the Virginia Military Institute were unconstitutional.

Foxman was not available for comment yesterday, a spokeswoman for the league said. His letter was first reported yesterday by the Capital newspaper in Annapolis.

Foxman's request comes at a time when the U.S. Air Force Academy in Colorado Springs has been accused of permitting an inappropriately religious atmosphere. A military report issued Wednesday found that some professors, coaches and cadets at the Air Force Academy had been proselytizing for evangelical Christianity.

The report concluded, however, that there was no overt discrimination against cadets of different faiths.

In Annapolis, a Naval Academy spokesman, Cmdr. Rod Gibbons, said all midshipmen stand to hear announcements before lunch, which are followed by a prayer, a moment of silence or "devotional thoughts."

These observances are led by one of the academy's Catholic, Protestant or Jewish chaplains, Gibbons said.

"The prayer is not mandatory or compulsory," Gibbons said, and midshipmen may decline to participate as long as they do not disturb others.

Gibbons said the Naval Academy had not yet received the letter from the Anti-Defamation League and would not comment on it. He said there were no plans to alter the noontime religious routine.

Neither the U.S. Military Academy at West Point, N.Y., nor the Air Force Academy has organized prayers before meals. Air Force cadets observe a 20-second period of silence, a spokeswoman for the academy said.

What say you?

tomder55 answered on 06/28/05:

I do not think that it is a necessary practice ;nor do I think it is a big problem . Whatever the academy ultimately does is ok with me .

Choux rated this answer Excellent or Above Average Answer
excon rated this answer Excellent or Above Average Answer

Question/Answer
Choux asked on 06/27/05 - New Short Bio of Jefferson

"In the brief-biography arena, essayist Hitchens' Jefferson vies with historian Joyce Appleby's Thomas Jefferson (2003) for the loyalty of the time-challenged reader. Hitchens brings his inimitable style of coruscating precision to Jefferson's ambiguities, which have long provoked disparate opinions. Hitchens gives no quarter on the way Jefferson contradicted himself on slavery, but eschewing easy condemnation, wishes Jefferson were less diffident against it: in any case, he ***extols Jefferson's bolder achievements (e.g., against state-sanctioned religion) that ensured that the American Revolution "was to be truly a change of system and not a change of master***."

This admiration girds Hitchens' assertion that Jefferson was the author of America, citing as proof the Declaration of Independence and Jefferson's presidency. Imaginative without being inventive, Hitchens is the rare author who can prune a complicated life such as Jefferson's to reveal its salient characteristics." Gilbert Taylor
Copyright

Comments?

tomder55 answered on 06/28/05:

I have read Christopher Hitchens ' works for years .He is a contributor to the 'Atlantic Monthly '.I am sure he does a great job as usual. What can I say ? Jefferson is far from my favorite founder ; I think he is given too much credit and he get hammered too hard in contemporary rewrites. He is everything both good and bad that is written about him and more .He was a great founder ;a great President ,who had too many contradictions to describe in a brief reply .

I think the bottom line on him is that he was Southern Aristocracy which made him well educated .As he grew older he became increasingly parochial like most of the Southerners and surrendered to a slave culture that he was at least luke warm about .In Hitchens' words he "he gave himself over to "a slave power that he half-abominated."This surrender, by a man of the Enlightenment and a man of truly revolutionary and democratic temperament," writes Hitchens, "is another reminder that history is a tragedy and not a morality tale."

Choux rated this answer Excellent or Above Average Answer

Question/Answer
Choux asked on 06/24/05 - Torturing Captured Terrorists

Washington has for the first time acknowledged to the UN that prisoners have been tortured at US detention centers in Guantanamo Bay as well as Afghanistan and Iraq, a UN source said.

The acknowledgement was made in a report submitted to the UN Comittee Against Torture said a member of the ten person panel, speaking on a condition of anonymity.

An unrelated story, VP Dick Cheney was driven to a hospital after landing at the Vail, Colorado airport, and released later.

Comments? Additions?

tomder55 answered on 06/26/05:

my only comment is that I visted Huffington's blog and saw some of these tidbits of compassion about Cheney in the comment section of the posting :

Hasn't he had his "last throes" of chest pains yet?
Posted by: Anonymous at June 24, 2005 10:28 PM

***

I wonder how many regular readers to this pathetic, self-important blog are hoping that Cheney comes out with a sheet over his head...?

Posted by: Eric at June 24, 2005 10:32 PM

***
His heart is listening to all the lies coming out of his mouth. It can't take much more of this drivel.

Posted by: Fred Colton at June 24, 2005 10:33 PM

***

I wish the evil zombie would stop leaving his underground bunker. Surely, there's a medical ward where he lurks below the surface, near Washington, D.C.

Posted by: Citizen Milenko at June 24, 2005 10:37 PM

***

You bet we losers want him dead. And I'm glad to be a loser. If I were a winner, I'd have to be around the kind of a**holes who like Bush and Cheney.

Posted by: Medina at June 24, 2005 10:37 PM

***

"Last throes," does one suppose? Here's hoping.

Posted by: HopingAgainst"Hoffman" at June 24, 2005 10:37 PM

***

You're all assuming he actually still has a heart. I don't think so.

I think it was replaced by a teflon pump a long time ago. Therefore he can no longer feel any pain.

Posted by: Don P at June 24, 2005 10:42 PM

***

"I wonder how many regular readers to this pathetic, self-important blog are hoping that Cheney comes out with a sheet over his head...?"

Actually, Eric, I was thinking more along the lines of a stake through his heart.

Posted by: Chuck Feney at June 24, 2005
11:08 PM

...............................

I may be wrong,but I do not remember seeing this type of crap when Clinton went in for his bypass surgury.

I checked in later in the day ,and to Ariana's credit she deleted the offensive comments


Choux rated this answer Excellent or Above Average Answer
Itsdb rated this answer Excellent or Above Average Answer
kindj rated this answer Excellent or Above Average Answer

Question/Answer
Choux asked on 06/24/05 - Fair and Balanced

Fox News, the "official" telvision outlet for Republican propaganda has been broadcasting ARUBA 24/7 for about a week now!

Fair and balanced? LOL

Apparently, "News" is what the Republican Government wants the public to know. If the ews iin Washington is all bad news for the Government, then there is no news.

Kinda like the Soviet newspapter "PRAVDA" (truth in Russian) which only published stories the government wanted the public to see.

tomder55 answered on 06/24/05:

the Aruba story bores me . but so did the media obsession with Michael Jackson . I guess those were slow war days also.

Choux rated this answer Excellent or Above Average Answer

Question/Answer
powderpuff asked on 06/24/05 - "At some point ...?

"At some point the Americans will begin to think they can't trust the Italians,"

ROME - An Italian judge on Friday ordered the arrests of 13 CIA officers for secretly transporting a Muslim preacher from Italy to Egypt as part of U.S. anti-terrorism efforts a rare public objection to the practice by a close American ally.

The arrest warrants were announced Friday by the Milan prosecutor's office, which has called the disappearance a kidnapping and a blow to a terrorism investigation in Italy.

The full new article can be found here:
http://news.yahoo.com/s/ap/20050624/ap_on_re_eu/italy_cia

At what point will the US understand that WE might not be trusted if we are willing to interfere with other countries justices? And I wonder, if we managed to capture this suspected terrorist who is believed to have fought in Afghanistan, why was he delivered to Egypt instead of Guantanamo Bay or instead of the authorities in Italy where he was captured (kidnapped)?

tomder55 answered on 06/24/05:

I do not know how intelligence agencies will be able to function if all their black ops become public knowlege. The only real thing that is news is that the justice system of Italy disclosed it and that there is an apparent strain in relations between the US and Italy ;a strain that happens occasionally and is usually smoothed over with some behind the scene negotiations .

Does anyone suppose that simular types of ops did not happen on a regular basis during the cold war ? Every once in a while they became public like the Israeli kidnapping of Adolf Eichmann in Argentina;but turst me ;there were others .

It cracks me up . The CIA was brutalized for relying on electronic espionage and not having humint.But when they use agents the way agents have always been used there is an out cry of indignation.How can a civilized nations permit it ? Because that is how the game has always been played. If they are handcuffed and not permitted to use operations procedures then they cease to be effective. Does anyone really believe that Italian ops perform any differently ?

"Good people sleep peaceably in their beds at night only because rough men stand ready to do violence on their behalf."

Choux rated this answer Excellent or Above Average Answer
powderpuff rated this answer Excellent or Above Average Answer

Question/Answer
CeeBee2 asked on 06/24/05 - Flag amendment.......................................

Do you for or against it?

tomder55 answered on 06/24/05:

In this topsy turvy world the governemnt can ban incitements to violence, but not violent pornography; the burning of crosses, but not the burning of flags. The government must not fund promotion of religion; but it must fund artists' denigration of religion. The voting public is rarely if ever consulted . The Congress ,which is supposed to be representing the people has just passed the Amendment ;one of the steps to make it the Constitutional law of the land.

As much as I sympathise with the sentiment involved ,I am not in favor of changing the Constitution over it .The anti-flag burning amendment would be a pointless, symbolic gesture whose only utility would be to make some people feel good. However ....the Supreme Court was way off base overturning State laws(Texas V. Johnson 491 U.S. 397 (1989)) prohibiting flag burning ,and declaring it free speech.We had flag-burning bans from at least 1968 (when Congress passed the Flag Protection Act) until the Court struck them down . As far as I can tell, the country didn't experience any dire consequences as a result.

The irony of the whole debate is that the only way to respectfully dispose of a flag is by buring it.

But what really gets me is the same folks who would defend to death the right of someone to burn the flag do everything in their power to deny someone else the right of free speech by flying a Confederate flag.

But my real reason for opposing the ban is that on the 4th of July I like buying those American flag paper napkins. Frankly;thje biggest desecration of the flag I saw was after the temporary patriotic fervor of the post 9-11 days had lapsed there were scores of flags that fell off of cars discarded on curbs . I was disgusted .

CeeBee2 rated this answer Excellent or Above Average Answer
Choux rated this answer Excellent or Above Average Answer
excon rated this answer Excellent or Above Average Answer
labman rated this answer Excellent or Above Average Answer

Question/Answer
arcura asked on 06/23/05 - Other than via impeachment............................

How can a USA Federal Supreme Court Judge be removed from office?
It's time to get rid of a few.
Thanks,
arcura (Fred)

tomder55 answered on 06/24/05:

Fred ;that in itself would be a novelty as it has never been tried . There is little else that can be done. Unlike the rest of the Federal Court System where the Senate can make and remove whole Federal Districts at will ;once a Supreme gets in they can stay until they drop (except impeachment).

It has been rumored that some justices would have retired if a Democrat were President ;but they are gutting it out hoping for a change in 08 . I think it is likely that Sandra Day OConner will leave this term ;and possibly Chief Justice Rhenquist due to poor health.

I'd love to see Kennedy ;Ginsberg ;Souter ;Stevens and Bryer just go.

arcura rated this answer Excellent or Above Average Answer

Question/Answer
Choux asked on 06/21/05 - Senator Dick Durbin (D Ill)

Dick Durbin has lost the good offices of Mayor Daley of Chicago who today issued an angry denunciation of his remarks about Gitmo.

If he has lost Mayor Daley, he has lost the "Common Man". Durbin is now requited to go a lot farther in his apologies for his offensive remarks. Maybe he has even lost his "job". That is how thing work in Illinois Democratic Politics. :)

Comments?

tomder55 answered on 06/22/05:

I think he is in trouble but only if he gets a strong primary challenge . Upstate Ill. will vote in whatever Dem. is put up for the position .

His weepy "apology" did not impress me because it dod not contain a retraction.

Choux rated this answer Excellent or Above Average Answer
powderpuff rated this answer Excellent or Above Average Answer

Question/Answer
powderpuff asked on 06/21/05 - All things considered,

Would it have been a better idea for our military to stay focused on Afghanistan and capturing Osama Bin Laden rather than dividing our interests and war efforts on Iraq and Saddam Hussein which, as it turns out, did not pose an immediate threat to the US?

Do you think we would have made more progress in Afghanistan if we had put the effort there instead of dividing it with Iraq?

Personally, I understand Iraq was/IS a problem, but wasn't/isn't Afghanistan with its terror training camps, heroin production, home of Al Qaeda and Osama Bin Laden the immediate threat? Is Pakistan hiding Bin Laden?

tomder55 answered on 06/21/05:

I think our military can function on many fronts .We still have personel in the Balkan States . We have troops engaged in Phillipines and in Columbia. We have troops guarding potential hot spots all over the world .I think the talk of us being overstreached is a little over done. The fact is that the Marines have met and /or exceeded their recruitment goals recently .

Afghanistan was a done deal before OIF . If Bin Laden escaped ,he did so before OIF.(thanks to the likes of Dick Durbin who disclosed to the world that we were monitoring Bin Laden's sat. phone) Afghanistan is run by Afghanis now . We have a small force assisting them in weeding out remaining al Qaeda ,and Taliban but the internal affairs of the country are theirs to decide.

My own supposition is that if Bin Laden is not buried in a cave at Tora Bora then he is being given sanctuary in Iran. It is possible he is in Pakistan but operations have been pretty intense there and it is not likely that he would stick around if he is in danger. For all his bravado about martyrdom ,he has proven to be a coward .

Choux rated this answer Excellent or Above Average Answer
powderpuff rated this answer Excellent or Above Average Answer

Question/Answer
excon asked on 06/20/05 - 2008 - Frist or McCain?


Hello Dudes:

Boy, I sure like McCain. Who'd win - Hillory vs McCain, Frist, Jeb Bush - who else?

excon

tomder55 answered on 06/20/05:

never too early to speculate . The Washington Post's E. J. Dionne Jr last week wrote that basically Bush and co. will owe his presidency to McCain and will ultimately support a McCain bid .

Bush has been battling, with Rove's help, for a long-term political realignment in favor of the Republicans. The president could well come to see McCain as the only Republican with a chance to push a Republican era forward. McCain, in turn, knows that his only way around the Republican right is to run with Bush's open blessing, if not his outright endorsement.

the quid pro quo being that Jeb Bush is picked as a VP candidate

And here is where Florida Gov. Jeb Bush, the president's brother, could be the deal-closer. Jeb Bush has said he will not run in 2008. But that does not rule him out as a vice presidential candidate. If McCain won, Jeb would be the No. 2 to a president who will turn 72 on Aug. 29, 2008, and might well serve only a single term. If McCain lost, Jeb would have enhanced national recognition for a run in 2012. If picking Jeb is the price of winning over George W., McCain will pay it.

McCain could be a strong candidate against Hillary Clinton, were he able to secure the nomination. Concern over the Republican base staying home would be lessened by the probable turnout to defeat Hillary.

Frist is DOA . He cannot manage the Senate let alone the Presidency .Guilliani is damaged goods plus I think it would be difficult for a former mayor to win the Presidency . If he became Govenor of NY or defeated Hillary in the Senate then he could think about the Presidency . Condi needs more seasoning .

Both parties primaries should be wild contests . I do not presume Hillary as the nominee but she would be formidible until the electorate realized that she has done nothing . This Sentate post if her first elected position . She has to date no major legislative initiatives .She has correctly stayed away from some of the more controversial frays but how long can that last with the likes of George Soros calling the shots for the Dems? The Moveon/Michael Moore types complained that Kerry was too moderate after the 2004 elections. I do not foresee them endorsing a candidate who is attempting to distance herself from the fringe.

She has been banking on the idea that the Republicans would pick a far right winger .If McCain were the likely GOP nominee, then the Democrats might be forced to consider an alternative candidate, someone like Virginia Governor Mark Warner or Indiana Senator Evan Bayh.

Choux rated this answer Excellent or Above Average Answer
excon rated this answer Excellent or Above Average Answer
Itsdb rated this answer Excellent or Above Average Answer

Question/Answer
Itsdb asked on 06/20/05 - Downing Street Memo Originals Destroyed

Monday, June 20, 2005 10:46 a.m. EDT

The so called Downing Street Memo - which was presumed to be authentic when Bush administration critics began touting it last month as evidence the president committed impeachable crimes - is actually a manually recreated copy - with the source of the memo now admitting he retyped the document before destroying the originals.

British reporter Michael Smith, who broke the memo story in the London Times on May 1, revealed to the Associated Press over the weekend that he "he protected the identity of the source he had obtained the documents from by typing copies of them on plain paper and destroying the originals."

Smith's admission means there's now no independent way to determine the accuracy of the Downing Street Memo, i.e., whether he made any typos or transcription errors that could have changed the memo's meaning.
The revelation has conjured up memories of the CBS News forged document scandal last year, where anchorman Dan Rather argued that damaging records he obtained from President Bush's National Guard file were essentially accurate, even though they had been faked by his source.

While British officials hadn't disputed the authenticity of the Downing Street Memo, a senior member of the Blair government who reviewed the Downing Street Memo in light of reporter Smith's admission could say only that its contents "appeared authentic."

That official, however, requested anonymity, refusing to make an on-the-record endorsement of the memo's accuracy.

New questions about the authenticity Downing Street Memo come at a particularly awkward time for Democrats in America. Only last week, House Democrats staged a mock impeachment hearing based on the re-created document.

Former presidential candidate John Kerry announced on June 2 that he intended to confront Congress with the Downing Street Memo, believing at the time that the document's authenticity was beyond reproach.

"I think it's a stunning, unbelievably simple and understandable statement of the truth and a profoundly important document that raises stunning issues here at home," he told a reporter.

Last week, a Kerry aide said his boss was sending a letter to President Bush demanding that he answer questions about the fake memo."

~~~~~~~~~~~~~~~~~~~~~~~~~~~~~~~~~~~~~~~~~~~~~~~~~~~~~~~

Comments?

tomder55 answered on 06/20/05:

Did he copy them at Kinkos? The Boston Herald says that Kerry has quitely backed down from his earlier bravado.

By summer of 2002 the US was ramping up in preparation for war. Was it really a suprise to some Brit pencil pusher that some officials saw war as inevedible? As for the other smoking gun ;that the WMD intel. was " fixed "well that does not concure with any number of post war commissions findings .The fact is ;whether the WMD existed or not (and I have made it no secret where I think they are )the world thought he had them . All the major intelligence agencies of the world thought so .Hans Blix reported to the UN days before the invasion about which arsenal was unaccounted for . The Downing Street Memos will make good fodder for Huffington blog but if it is a smoking gun ,the gun shot blanks .

excon rated this answer Excellent or Above Average Answer
Itsdb rated this answer Excellent or Above Average Answer

Question/Answer
voiceguy2000 asked on 06/19/05 - More on Turban Durbin

The invaluable Mark Steyn, in the Chicago Sun-Times:

Durbin slanders his own country

June 19, 2005

BY MARK STEYN
SUN-TIMES COLUMNIST

Throughout the last campaign season, senior Democrats had a standard line in their speeches, usually delivered with righteous anger, about how "nobody has a right to question my patriotism!" Given that nobody was questioning their patriotism, it seemed an odd thing to harp on about. But, aware of their touchiness on the subject, I hasten to add that in what follows I am not questioning Dick Durbin's patriotism, at least not for the first couple of paragraphs. Instead, I'll begin by questioning his sanity.

Last Tuesday, Senator Durbin, Democrat of Illinois, quoted a report of U.S. "atrocities" at Guantanamo and then added:

"If I read this to you and did not tell you that it was an FBI agent describing what Americans had done to prisoners in their control, you would most certainly believe this must have been done by Nazis, Soviets in their gulags, or some mad regime -- Pol Pot or others -- that had no concern for human beings."

Er, well, your average low-wattage senator might. But I wouldn't. The "atrocities" he enumerated -- "Not only was the temperature unbearably hot, but extremely loud rap music was being played in the room" -- are not characteristic of the Nazis, the Soviets or Pol Pot, and, at the end, the body count in Gitmo was a lot lower. That's to say, it was zero, which would have been counted a poor day's work in Auschwitz or Siberia or the killing fields of Cambodia.

But give Durbin credit. Every third-rate hack on every European newspaper can do the Americans-are-Nazis schtick. Amnesty International has already declared Guantanamo the "gulag of our times." But I do believe the senator is the first to compare the U.S. armed forces with the blood-drenched thugs of Pol Pot's Khmer Rouge. Way to go, senator! If you had a dime for every crackpot Web site that takes up your thoughtful historical comparison, you'd be able to retire to the Caribbean and spend the rest of your days torturing yourself with hot weather and loud music, as well as inappropriately provocative women and insufficient choice of hors d'oeuvres and all the other shameful atrocities committed at Guantanamo.

Just for the record, some 15 million to 30 million Soviets died in the gulag; some 6 million Jews died in the Nazi camps; some 2 million Cambodians -- one third of the population -- died in the killing fields. Nobody's died in Gitmo, not even from having Christina Aguilera played to them excessively loudly. The comparison is deranged, and deeply insulting not just to the U.S. military but to the millions of relatives of those dead Russians, Jews and Cambodians, who, unlike Durbin, know what real atrocities are. Had Durbin said, "Why, these atrocities are so terrible you would almost believe it was an account of the activities of my distinguished colleague Robert C. Byrd's fellow Klansmen," that would have been a little closer to the ballpark but still way out.

One measure of a civilized society is that words mean something: "Soviet" and "Nazi" and "Pol Pot" cannot equate to Guantanamo unless you've become utterly unmoored from reality. Spot the odd one out: 1) mass starvation; 2) gas chambers; 3) mountains of skulls; 4) lousy infidel pop music turned up to full volume. One of these is not the same as the others, and Durbin doesn't have the excuse that he's some airhead celeb or an Ivy League professor. He's the second-ranking Democrat on the Senate Judiciary Committee. Don't they have an insanity clause?

Now let us turn to the ranking Democrat, the big cheese on the committee, Patrick Leahy of Vermont. Leahy thinks Gitmo needs to be closed down and argues as follows:

"America was once very rightly viewed as a leader in human rights and the rule of law, but Guantanamo has drained our leadership, our credibility, and the world's good will for America at alarming rates."

So, until Guantanamo, America was "viewed as a leader in human rights"? Not in 2004, when Abu Ghraib was the atrocity du jour. Not in 2003, when every humanitarian organization on the planet was predicting the deaths of millions of Iraqis from cholera, dysentery and other diseases caused by America's "war for oil." Not in 2002, when the "human rights" lobby filled the streets of Vancouver and London and Rome and Sydney to protest the Bushitler's plans to end the benign reign of good King Saddam. Not the weekend before 9/11 when the human rights grandees of the U.N. "anti-racism" conference met in South Africa to demand America pay reparations for the Rwandan genocide and to cheer Robert Mugabe to the rafters for calling on Britain and America to "apologize unreservedly for their crimes against humanity." If you close Gitmo tomorrow, the world's anti-Americans will look around and within 48 hours alight on something else for Gulag of the Week.

And this is where it's time to question Durbin's patriotism. As Leahy implicitly acknowledges, Guantanamo is about "image" and "perception" -- about how others see America. If this one small camp of a few hundred people has "drained the world's good will," whose fault is that?

The senator from Illinois' comparisons are as tired as they're grotesque. They add nothing useful to the debate. But around the planet, folks naturally figure that, if only 100 people out of nearly 300 million get to be senators, the position must be a big deal. Hence, headlines in the Arab world like "U.S. Senator Stands By Nazi Remark." That's al-Jazeera, where the senator from al-Inois is now a big hero -- for slandering his own country, for confirming the lurid propaganda of his country's enemies. Yes, folks, American soldiers are Nazis and American prison camps are gulags: don't take our word for it, Senator Bigshot says so.

This isn't a Republican vs Democrat thing; it's about senior Democrats who are so over-invested in their hatred of a passing administration that they've signed on to the nuttiest slurs of the lunatic fringe. It would be heartening to think that Durbin will himself now be subjected to some serious torture. Not real torture, of course; I don't mean using Pol Pot techniques and playing the Celine Dion Christmas album really loud to him. But he should at least be made a little uncomfortable over what he's done -- in a time of war, make an inflammatory libel against his country's military that has no value whatsoever except to America's enemies. Shame on him, and shame on those fellow senators and Democrats who by their refusal to condemn him endorse his slander.
Original here.

According to Power Line, former House Speaker Newt Gingrich sent the following letter to all U.S. Senators yesterday:
Dear Senator _______________:

By his statements equating American treatment of suspected terrorists at Guantanamo Bay with the behavior of the evil regimes of Nazi Germany, the Soviet Union, and Pol Pots Cambodia, Senator Richard Durbin has dishonored the United States and the entire U.S. Senate. Only by a vote to censure Senator Durbin for his conduct can the U.S. Senate restore its dignity and defend American honor.

Senator Durbins comparison, sadly, is despicable.

U.S. Senators should be clear about the gravity of Senator Durbins comparison. Nine million innocent human beings were murdered in Hitlers death camps, nearly three million perished in the gulags under Stalin, and more than one and a half million were slaughtered in the killing fields of Cambodia at the hand of Pol Pot. And while not a single terrorist has died in detention at Guantanamo, Senator Durbin sees fit to liken our American service men and women to the terrifying murderers of three evil despotic regimes.

Moreover, Senator Durbin equates the terrorist detainees at Guantanamo with the millions of innocent men, women, and children exterminated by the order of evil dictators. The fact that he did so as a high ranking member of the Senate on the Senate floor makes his comparison all the more shocking.

This moral equivalence isnt just utterly false; it endangers the lives of our young men and women in the military because it arms every radical Islamist with the official-record words of a Senate leader to justify their war of terror against civilized people everywhere.

Senator Durbins statement of regret on Friday has only compounded the need for the Senate to act. In it, Senator Durbin said that I have learned from my statement that historical parallels can be misused and misunderstood. I sincerely regret if what I said caused anyone to misunderstand my true feelings Incredibly, Senator Durbin is sticking to his original assertion that there is indeed, in his own words, an historic parallel between U.S. soldiers at Guantanamo Bay and the killers under Hitler, Stalin and Pol Pot. In other words, his only regret is that Americans dont understand his misreading of history and that he has caused us to misunderstand him. Offering no apology for the original slanderous statement itself, Senator Durbin has chosen instead to actually defend his comparisons. This defense makes his original speech all the more revolting.

Its one thing for one Senator to endanger young Americans and defame America; it would be the shame of the Senate if the other 99 senators did not stand up to defend America and to defend the reputation of our young men and women in uniform.

A Senate censure of Senator Durbin is justified and would reaffirm a standard for healthy, rational debate. By voting for or against the censure, the rest of the members of the U.S. Senate can go on record and make clear how they judge Senator Durbins characterization of American soldiers. It will also send a clear message to terrorists who will use the words of a Senate leader against us that the Senate stands in support of America and our military and against those who seek to destroy the free people of the United States.

There is historic precedent for censuring Senators whose words bring dishonor and disrepute on the Senate and impair its dignity; Senator Durbins words fit that precedent.

In this case, expressing outrage is not enough. It is time for the Senate to act. Senator Durbin must be censured now.

Sincerely,

Newt Gingrich
Original here.

As John Hinderaker comments, "the American people deserve to know who, if anyone, agrees with Durbin's slander of our armed forces, so that when those Senators run for re-election, they can be defeated. Senators should not be able to hide behind a discreet 'no comment,' as Hillary Clinton has done. This is not a time for our elected officials to be neutral as between the terrorists and the armed forces of the United States."

Seriously, with friends like Durbin, who needs enemies?

tomder55 answered on 06/20/05:

Wasn't it Durbin who was the subject of a "criminal referral"by the CIA over the revelations of us monitoring the OBL satellite phone ? Dick must know that al-Jazzera picked up his remarks and aired them before even the MSM got a hold of them. But don't call him un-patriotic . Bill Kristol goes even further than Newt . Instead of demanding for censure ;he wants the Dems. to remove Durbin from their leadership (he brings up the Trent Lott example I used last week ).

In many cases the Vietnam war defeatists have taken positions of authority in government ,and certainly in academia and the MSM .It is getting bad. The Downing St. Memos are so lame that even the MSM have decided there is not much there .That did not stop Rep. Conyers from holding a charade last week . He held mock (Senate)Judiciary hearings in the DNC Wasserman Conference Room .He invited every extreme fringe kook group like Moveon.org to participate . CSPAN did a live feed of it until even they became bored with it.

Hugh Hewitt says ;

The election of 2004 might have been the occasion when the Democratic leadership took account of where American public opinion stands on this war. That leadership rejected the results of November because those results rejected them. In response they have upped the rhetoric, intent on a replay of the anti-war movement and rhetoric of the late 60s and early 70s, hopeful of converting Bush to Nixon, and of driving American power back to its own shores. The tactic of demonizing the American military worked then, so it is being replayed now. If this rhetoric is not checked, it is only a matter of time until we have a new John Kerry discussing the "Genghis Khan" tactics of the American military operating in the Middle East.

Christopher Chantrill at The American Thinker calls Durbin "post-patriotic".

Maybe Dick Durbin and Co. are doing us a favor. Our modern elites seem incapable of building anything but top-down bureaucracies like the welfare state and the EU. So it is better for them to spout their offensive similes. Their mischievous post-patriotism is just the thing to keep them out of jointand out of power.


I think he gives Durbin way to much credit. When academia talks about post-modern it usually means that they have thoughtfully examined the art, literature,history of the modern era. Given Durbin's original and subsequent statements one can conclude that he has not thought this through at all ;and that is giving a huge benefit of the doubt but I do not want to fall into exagerations lest I compare Durbin to Vidkun Quisling or Benedict Arnold .



voiceguy2000 rated this answer Excellent or Above Average Answer

Question/Answer
voiceguy2000 asked on 06/16/05 - Al Jazeera' Favorite Senator

Every time you think you've seen everything, something comes along to prove you wrong.

What on earth could possess Dick Durbin to swan-dive so far off the deep end?

See:

Power Line and the articles linked from here.

Hugh Hewitt and linked items.

And as background for all of this, read the extended essay here. This puts the whole Gitmo discussion into vivid perspective.

Where are the voices of sanity in the Democratic party? Or, better still, the voices of outrage? Has Bush fever so overtaken the Democrats that they are willing to destroy the United States in order to "prove" themselves correct?

tomder55 answered on 06/17/05:

Steve posted about Durbin on the Christianity Board and it sparked a firestorm .


I clearly remember reading about the horror of not having a/c in the Nazi Concentration camps .

This represents a new low . Forget a Senator ;I would think his comments would be over the top for Michael Moore or the idiots at Moveon.org. I want the Republican leadership to smack him down as hard as Democrats did Trent Lott when he made "inappropriate comments " at Strom Thurmond's birthday party .


None Dare Call It Treason

voiceguy2000 rated this answer Excellent or Above Average Answer

Question/Answer
sapphire630 asked on 06/09/05 - how can you find out where, how and what

you can send to military people in Abu graib or Abu dobi or where ever. My daughter didn't come to Pittsburgh so I didn't get to see her friend (in the air force) before he went to Abu whatever. I want to send a card of support and find out what all you are allowed to send.
WHere would I find out? would a state representative have that information?

tomder55 answered on 06/10/05:

she needs to find out the unit her friend is from . Then their state-side rep will be able to get a mailing address (perhaps a family member can assist).You will need to use the soldier's full name (without rank), unit and APO address

sapphire630 rated this answer Excellent or Above Average Answer

Question/Answer
excon asked on 06/09/05 - Bush and his Buddies


Hello Bush guys:

Our Justice Department decided to recommend only a $10 billion fine against the tobbaco companies who are again on trial. The cost of the non smoking program they are suing for is going to be $130 billion and they WERE asking for that much.

But, they caved and dropped it to $10 bil. Do you think it has anything to do with his business buddies? When is the administration going to come down on the side of the "people"?

excon

tomder55 answered on 06/09/05:

since 1997, the prevalence of smoking in the U.S. has fallen from 24.7% of the population to 21.6% in 2003, according to the U.S. Centers for Disease Control In 1979, 33.3% of Americans were smokers. Seems to me that Americans are already getting the message ;the education .Perhaps a $130 billion, 25-year program proposed is not really necessary .10 billion is not chump change ;especially when you tack onto it all the individual State class action suits that big tobacco is paying out already since 1998 (principally designed to be a cash cow for the lawyers ,and to pad state budgets ,and not to benefit the people who were actually harmed by smoking ).

But here is a scoop ,back in Feb. an appeals court denied a request by the gvt. to seek $280 billion and told the gvt. that any remedies in the case must be "forward-looking" to prevent and restrain future wrongful acts, rather than to address even lingering consequences of past acts.That is why the education program was designed . If court-appointed monitors find that the tobacco companies continue to commit acts of fraud, the court can extend and expand the program to exceed the $10 billion up to and beyond the 25 years and above the $130 bil .

excon rated this answer Excellent or Above Average Answer

Question/Answer
Itsdb asked on 06/08/05 - From a white, Christian conservative...

Howard Dean continues unhinged...and I love it.

"You know, the Republicans are not very friendly to different kinds of people. They're a pretty monolithic party. Pretty much, they all behave the same, and they all look the same. ... It's pretty much a white Christian party,'' the former Vermont governor told a San Francisco roundtable Monday in reaction to a question about the lack of outreach to minority communities by political parties.

"Our folks have got to spend time in the communities,'' he said. "We want a very diverse group of people running for office -- African Americans, Asian Americans, Latinos.''

He defended his comments with, "This is a diversion from the issues that really matter: Social Security, and adequate job opportunity, strong public schools, a strong defense."

GOP Chairman Ken Mehlman joked, "a lot of folks who attended my Bar Mitzvah would be surprised" he heads a Christian party.

"We gotta get ourselves beyond this point where when we disagree about politics, we call the other guy names."

~~~~~~~~~~~~~~~~~~~~~~~~~~~~~~~~~~~~~~~~~~~~~~~~~~~~~~~

While I love to hammer away at people like Dean and the insulting, idiotic, intolerant things they say about me, there's more to my concerns here.

Diversion is a tactic used by both sides, but how does one justify such obvious hypocritical tactics? Dean is going around the country saying he "hates Republicans," they're "pretty much a white, Christian party," have "never made an honest living in their lives," but holding him accountable for these insults is "a diversion from the issues"?

Speaking of those issues, hasn't Bush been traveling the country trying to address Social Security? Isn't a "strong military" a cornerstone of the GOP?

Liberals are constantly ranting that the "Republican controlled congress" is avoiding "the issues that matter" while focusing on abortion, gay marriage and conservative judges.

Let me ask, are these issues that Republicans raised or were they forced on us?

Is it conceivable that the left has forced these issues to divert attention from "the issues that matter" that they don't have an answer for, or just stalling for the day they regain power?

How can Dean say "We want a very diverse group of people running for office -- African Americans, Asian Americans, Latinos" while the Bush administration has held one of, if not the most diverse cabinet in American history?

How can he say that while his party has whined about and obstructed the nominations of a number of blacks and hispanics?

How can he say that while his own are warning a black judicial nominee wants to take us back to the days of the civil war?

Can someone help this white, Christian conservative understand the liberal mindset?

Steve

tomder55 answered on 06/08/05:

and people thought that the scream was an aberation . clearly he is beholden to the George Soros ;Michael Moore wing of the party .I guess gone forever is those brief days when he claimed that he was going to expand the Democrat base into the red states. Time to turn one of the democrat cliches back at them :I wonder why they keep on doing things contrary to their best interests ?

Just like Hillary who has been carefully reinventing herself as a middle of the roader. Then she gets in front of a womens group in NY and crash ......speaking about her husband former President Bill Clinton the Bush Administration she said ,"It's very hard to stop people who have no shame about what they're doing. It's very hard to stop people who have never been acquainted with the truth."

She has no legislative accomplishments to speak of ;you never see her name as a primary sponsor of any bill .This is her first elective position ,and still she is the presumptive nominee. I don't get it .

Itsdb rated this answer Excellent or Above Average Answer
kindj rated this answer Excellent or Above Average Answer

Question/Answer
Itsdb asked on 06/07/05 - Access To Legal Pharmaceuticals Act

What do you experts think of the proposed ALPhA act, which is designed basically to guarantee women can get the contraceptives they're prescribed?

The first thing I notice is these findings:

The Congress finds as follows:

(1) An individual's right to religious belief and worship is a protected, fundamental right in the United States.

(2) An individual's right to access legal contraception is a protected, fundamental right in the United States.

(3) An individual's right to religious belief and worship cannot impede an individual's access to legal prescriptions, including contraception.

~~~~~~~~~~~~~~~~~~~~~~~~~~~~~~~~~~~~~~~~~~~~~~~~~~~~~~~

Doesn't number 3 pretty much trump number 1? How can one's "right to religious belief and worship" be protected if it "cannot impede an individual's access to legal prescriptions?"

Example, we still have a few home-owned pharmacies in town, and say the owner is the only pharmacist but objects to selling the morning after pill, will he/she be forced to provide it or face fines?

Is this government telling pharmacies they must sell certain items?

And for you ex, isn't this a bit hypocritical in light of yesterday's medical marijuana ruling? The Feds won't allow medical marijuana even in states that legalized it, but want to force pharmacies to sell "emergency contraception" to a woman that sleeps around?

tomder55 answered on 06/08/05:

"Conscience clauses" are nothing new to medicine.Is a doctor forced to perform an abortion ? The American Medical Association allows physicians, hospitals, and hospital staff to opt out of any act that violates "personally held moral principles."The American Pharmacists Association permits pharmacists to decline to fill prescriptions if they provide some other avenue for patients to get their prescriptions.Kerry and Rick Santorum, have introduced the Workplace Religious Freedom Act . This bill would allow pharmacists to refuse to dispense certain drugs as long as another pharmacist who would is available.The ACLU is of course opposed to the bill .


Choux rated this answer Excellent or Above Average Answer
Itsdb rated this answer Excellent or Above Average Answer

Question/Answer
Itsdb asked on 06/07/05 - Kerry updates

Yale grades portray Kerry as a lackluster student
His 4-year average on par with Bush's

By Michael Kranish, Globe Staff | June 7, 2005

WASHINGTON -- During last year's presidential campaign, John F. Kerry was the candidate often portrayed as intellectual and complex, while George W. Bush was the populist who mangled his sentences.

But newly released records show that Bush and Kerry had a virtually identical grade average at Yale University four decades ago.

In 1999, The New Yorker published a transcript indicating that Bush had received a cumulative score of 77 for his first three years at Yale and a roughly similar average under a non-numerical rating system during his senior year.

Kerry, who graduated two years before Bush, got a cumulative 76 for his four years, according to a transcript that Kerry sent to the Navy when he was applying for officer training school. He received four D's in his freshman year out of 10 courses, but improved his average in later years.

The grade transcript, which Kerry has always declined to release, was included in his Navy record. During the campaign the Globe sought Kerry's naval records, but he refused to waive privacy restrictions for the full file. Late last month, Kerry gave the Navy permission to send the documents to the Globe.

Kerry appeared to be responding to critics who suspected that there might be damaging information in the file about his activities in Vietnam. The military and medical records, however, appear identical to what Kerry has already released. This marks the first time Kerry's grades have been publicly reported.

The transcript shows that Kerry's freshman-year average was 71. He scored a 61 in geology, a 63 and 68 in two history classes, and a 69 in political science. His top score was a 79, in another political science course. Another of his strongest efforts, a 77, came in French class.

Under Yale's grading system in effect at the time, grades between 90 and 100 equaled an A, 80-89 a B, 70-79 a C, 60 to 69 a D, and anything below that was a failing grade. In addition to Kerry's four D's in his freshman year, he received one D in his sophomore year. He did not fail any courses.

''I always told my Dad that D stood for distinction," Kerry said yesterday in a written response to questions, noting that he has previously acknowledged that he spent a lot of time learning to fly instead of focusing on his studies.

Kerry's weak grades came despite years of education at some of the world's most elite prep schools, ranging from Fessenden School in Massachusetts to St. Paul's School in New Hampshire.

It is noteworthy, however, that Kerry received a high honor at Yale despite his mediocre grades: He was chosen to deliver his senior class oration, a testament to his reputation as a public speaker. He delivered a speech questioning the wisdom of the Vietnam War, in which he would soon see combat.

Kerry gradually improved his grades, averaging 81 in his senior year. His highest single grade was an 89, for a political science class in his senior year. Despite his slow start, he went on to be a top student at Naval Candidate School, command a patrol boat in Vietnam, graduate from law school, and become a prosecutor, lieutenant governor, US senator, and presidential candidate.

In his Navy application, Kerry made clear that he spent much of his college time on extracurricular activities, including the Yale Political Union, the Debating Association, soccer, hockey, fencing, and membership in the elite Skull and Bones Society. Asked to describe nonschool training that qualified him for the Navy, Kerry wrote: ''A great deal of sailing -- ocean and otherwise, including some navigation. Scuba diving. Rifle. Beginning of life saving." He said his special interests were ''filming," writing, and politics, noting that the latter subject occupied 15 hours per week.

Gaddis Smith, a retired Yale history professor who taught both Kerry and Bush, said in a telephone interview that he vividly remembers Kerry as a student during the 1964-1965 school year, when Kerry would have been a junior. However, Smith said he doesn't have a specific memory about Bush.

Based on what Smith recalls teaching that year, Kerry scored a 71 and 79 in two of Smith's courses. When Smith was told those scores, he responded: ''Uh, oh. I thought he was good student. Those aren't very good grades." To put the grades in perspective, Smith said that he had a well-earned reputation for being tough, and noted that such grades would probably be about 10 points higher in a similar class today because of the impact of what he called ''grade inflation."

Bush went to Yale from 1964 to 1968; his highest grades were 88s in anthropology, history, and philosophy, according to The New Yorker article. He received one D in his four years, a 69 in astronomy. Bush has said he was a C student.

Like Kerry, Bush reportedly suffered through a difficult freshman year and then pulled his grades up.

~~~~~~~~~~~~~~~~~~~~~~~~~~~~~~~~~~~~~~~~~~~~~~~~~~~~~~~

Now if we can just explode the myth of Kerry supporters being smarter than Bush supporters.

~~~~~~~~~~~~~~~~~~~~~~~~~~~~~~~~~~~~~~~~~~~~~~~~~~~~~~~

Kerry allows Navy release of military, medical records

"The records, which the Navy Personnel Command provided to the Globe, are mostly a duplication of what Kerry released during his 2004 campaign for president, including numerous commendations from commanding officers who later criticized Kerry's Vietnam service."

~~~~~~~~~~~~~~~~~~~~~~~~~~~~~~~~~~~~~~~~~~~~~~~~~~~~~~~

If it was so glowing, why not just release them and answer the critics last year?

~~~~~~~~~~~~~~~~~~~~~~~~~~~~~~~~~~~~~~~~~~~~~~~~~~~~~~~

So did anyone notice if Kerry kept his promise to push for Bush's impeachment yesterday?




tomder55 answered on 06/07/05:

Kerry was doing what everyone else was doing in college .John Kerry was just a college kid looking for adventure ; ;sucking down brew at the skull and bones frat parties ;traveling during spring break to Vale Colorado chalets ;summer breaks in Europe hitch-hiking driving a new Austin Healy as is due a Boston Brahmin. .....yup... just like every other college kid ;preparing to be a populist ;a man of the people ;he would grab a quick Kobe beef with truffled cheese lunch at ཙ Newbury'and the stop off for a quick desert at the Ritz-Carlton (gourmet chocolate-chip cookies (Lindt chocolate, not Nestle). Who had time to study ? He needed the down time to fine tune his skills at thinking through complex issues three-dimensionally (or in the common vernacular ...flip-flopping)

Itsdb rated this answer Excellent or Above Average Answer
jocase rated this answer Above Average Answer

Question/Answer
Choux asked on 06/04/05 - Impeachment

I'm a news junky, and I haven't heard one word about any so-called Kerry Impeachment initative on cable news or anywhere.

How did this rumor get started? Can anyone provide a link where I can read about the pendiing impeachment initative?

tomder55 answered on 06/05/05:

It started with a Ralph Nader op-ed last week .

Then Kerry picked up on it. He will bring it up tomorrow in the Senate ,It is related to the' Downing Street' memos. The major news in this country is ignoring it ,but it was in NewsMax.com ;and al-Jazerra . It will become major news probably today on the morning talk shows ;or tomorrow after Kerry raises the issue in the Senate. Timing is everything.

Kerry of course is being a complete hypocrite on this . Here is a quote he made before he flip-flopped on the issue.

"The threat of Saddam Hussein with weapons of mass destruction is real, but as I said, it is not new. It has been with us since the end of that war, and particularly in the last 4 years we know after Operation Desert Fox failed to force him to reaccept them, that he has continued to build those weapons. He has had a free hand for 4 years to reconstitute these weapons, allowing the world, during the interval, to lose the focus we had on weapons of mass destruction and the issue of proliferation." (John Kerry, October 9, 2002 )

The "smoking gun "that the left wing blogs are touting is the Downing Street memo .It "claims" that TOny Blair admitted in a meeting that Bush was fixing the evidence on WMD to support justification for OIF. The memo doesn't quote either Bush or Blair directly and lacks any sort of evidence supporting a cover up. But that doesn't matter;facts are never a problem for the left. Kerry said :"I think it's a stunning, unbelievably simple and understandable statement of the truth and a profoundly important document that raises stunning issues here at home."

How about that????Kerry is calling a leaked British memo featuring secondhand information "a profoundly important document."

Choux rated this answer Excellent or Above Average Answer

Question/Answer
excon asked on 06/04/05 - A Nation in Decline



Hello:

I've been wondering when this country will get back on the right track. However, I don't think "when" is the right question. I think "if" is more appropriate.

Yes, we are a world power - frankly, the only world power. But we are a nation in decline. The good jobs are gone forever, and so are our morals. Pensions don't mean anything, and SS is going broke. Decent health care is out of reach for many of us with more joining the roles of the un-insured each day. I used to have hope, but now I don't.

In a nutshell, the problem is that billions of people from all over the world are willing to work harder and for less money than we are. The Chinese (billions of them), the Indians, Koreans and Pakistanis. Hell, you don't have to look further than right here. We have illegal Mexicans (who work very hard).

I think we're toast. Europe too. Buy gold.

excon


tomder55 answered on 06/05/05:


My pension is solid ,and will be for a long time because I was given the choice to manage it myself.

In the 80s it was the Japanese that were going to overtake us . Our economy still out performs all the nations that you mentioned. Don't think that the Chinese and the Indian economies will not experience growing pains. Don't think that forever their populations will toil for less. With growing prosperity they will demand more . When the Chinese went from a planned economy to a market based one they opened a pandora's box they will not be able to close. No longer will they be able to maintain control on their population. The communist system there is experiencing a slow death (a little too slow for my taste ;but death nonetheless)

We have always adapted to changing realities .I see no difference today. The good jobs are not gone ; The old jobs are .

excon rated this answer Excellent or Above Average Answer

Question/Answer
Itsdb asked on 06/03/05 - Kerry's smoking gun already extinguished?

U.N.: Weapons Equipment Missing in Iraq
By EDITH M. LEDERER, Associated Press Writer

UNITED NATIONS -- U.N. satellite imagery experts have determined that material that could be used to make biological or chemical weapons and banned long-range missiles has been removed from 109 sites in Iraq, U.N. weapons inspectors said in a report obtained Thursday.

U.N. inspectors have been blocked from returning to Iraq since the U.S.-led war in 2003 so they have been using satellite photos to see what happened to the sites that were subject to U.N. monitoring because their equipment had both civilian and military uses.

In the report to the U.N. Security Council, acting chief weapons inspector Demetrius Perricos said he's reached no conclusions about who removed the items or where they went. He said it could have been moved elsewhere in Iraq, sold as scrap, melted down or purchased.

He said the missing material can be used for legitimate purposes. "However, they can also be utilized for prohibited purposes if in a good state of repair."

He said imagery analysts have identified 109 sites that have been emptied of equipment to varying degrees, up from 90 reported in March.

The report also provided much more detail about the percentage of items no longer at the places where U.N. inspectors monitored them.

From the imagery analysis, Perricos said analysts at the U.N. Monitoring, Verification and Inspection Commission which he heads have concluded that biological sites were less damaged than chemical and missile sites.

The commission, known as UNMOVIC, previously reported the discovery of some equipment and material from the sites in scrapyards in Jordan and the Dutch port of Rotterdam.

Perricos said analysts found, for example, that 53 of the 98 vessels that could be used for a wide range of chemical reactions had disappeared. "Due to its characteristics, this equipment can be used for the production of both commercial chemicals and chemical warfare agents," he said.

The report said 3,380 valves, 107 pumps, and more than 7.8 miles of pipes were known to have been located at the 39 chemical sites.

A third of the chemical items removed came from the Qaa Qaa industrial complex south of Baghdad which the report said "was among the sites possessing the highest number of dual-use production equipment," whose fate is now unknown." Significant quantities of missing material were also located at the Fallujah II and Fallujah III facilities north of the city, which was besieged last year.

Before the first Gulf War in 1991, those facilities played a major part in the production of precursors for Iraq's chemical warfare program.

The percentages of missing biological equipment from 12 sites were much smaller -- no higher than 10 percent.

The report said 37 of 405 fermenters ranging in size from 2 gallons to 1,250 gallons had been removed. Those could be used to produce pharmaceuticals and vaccines as well as biological warfare agents such as anthrax.

The largest percentages of missing items were at the 58 missile facilities, which include some of the key production sites for both solid and liquid propellant missiles, the report said.

For example, 289 of the 340 pieces of equipment to produce missiles -- about 85 percent -- had been removed, it said.

At the Kadhimiyah and Al Samoud factory sites in suburban Baghdad, where the report said airframes and engines for liquid propellant missiles were manufactured and final assembly was carried out, "all equipment and missile components have been removed."

UNMOVIC is the outgrowth of a U.N. inspections process created after the 1991 Gulf War in which invading Iraqi forces were ousted from Kuwait. Its staff are considered the only multinational weapons experts specifically trained in biological weapons and missile disarmament.

The report noted that the commissioners who advise UNMOVIC again raised questions about its future. Iraq has called for its Security Council mandate to be terminated because UNMOVIC is funded from past Iraqi oil sales and it wants to be treated like other countries, but the council has not taken up the issue.

France's U.N. Ambassador Jean-Marc de La Sabliere said Thursday the commission's expertise "should not be lost for the international community."

~~~~~~~~~~~~~~~~~~~~~~~~~~~~~~~~~~~~~~~~~~~~~~~~~~~~~~~

Just when Kerry announces his Downing Street Memo impeachment initiative, more of those items that "did not exist" are now missing?

Steve

tomder55 answered on 06/04/05:

well the UN is a little slow

this is confirmation and is ominous in light of the additional underreported news yesterday about Syria launching a test of 3 scud missles last week . One of the missles broke up while overflying Turkey raining debris in two villages .

They developed the scud D rocket withthe help of N Korea .It has a range around 450 miles. They are believed to possess chemical warheads for these .Not only does Syria have the WMD that was trasfered from Iraq ,but it has an active WMD program.It has a stockpile of the nerve agent sarin that can be delivered by aircraft or ballistic missiles, and has engaged in the research and development of more toxic and persistent nerve agents such as VX.

Itsdb rated this answer Excellent or Above Average Answer

Question/Answer
Itsdb asked on 06/03/05 - Kerry to push for Bush impeachment

By Sher Zieve

"John Kerry announced Thursday that he intends to present Congress with The Downing Street Memo, reported by the London Times 1 May 2005. As reported by NewsMax, the memo purports to include minutes from a July 2002 meeting with Tony Blair, in which Blair ostensibly said that President Bushs Administration fixed intelligence on Iraq in order to justify the Iraqi war. In an interview with the Standard Times, Kerry said: "It's amazing to me the way it escaped major media discussion. It's not being missed on the Internet, I can tell you that."

The Boston Globe published an article by Ralph Nader, Tuesday, in which Nader also called for President Bushs impeachment. The story is being carried on Michael Moores website and the Democratic Underground. Failed presidential candidate Kerry advised that he will begin the presentation of his case for President Bushs impeachment to Congress, on Monday."

So Kerry is going to push for impeachment based on a memo from a meeting which Bush did not attend, that offers one possibly incriminating statement by the British intelligence chief? "But the intelligence and facts were being fixed around the policy."

This should be interesting...

Steve

tomder55 answered on 06/03/05:

I see nothing incriminating in the Downing Street memo. The rat pack journalists in this country would've been all over it if there was any evidence of impeachable actions. Even if everything is true the memo does not demonstrate misconduct.In Britain where the story broke there was a quick reaction and then it blew over .

OIF opponents continue to misread the war aims . They were to deny the potential that Saddam would develop or acquire WMD [that he had WMD was not disputed by anyone], and also to deny to terrorists Iraq as a haven. The first aim is achieved. We are close to achieving the second.

The best case against the Iraq war would be that we should have dealt with North Korea or Iran first.But that was an after thought reason for opposing the war .Had we addressed those nations first the opposition would've said :Why Iran .....Why not Iraq?

This is just another case of the Democrat obstructionist throwing spagetti on the wall and seeing if it sticks.In another example of living in the past they think these are the 21st century 'Pentagon Papers'. But there is nothing there except for the field day that the liberal blogs like Michael Moore and the airhead America ,Randi Rhodes groupies are having over them.

Itsdb rated this answer Excellent or Above Average Answer

Question/Answer
excon asked on 06/03/05 - Marijahootchi

Hello anti-drug dudes:

Marijuana has never killed anyone. That's NEVER with a capital N. Everybody who smokes pot knows that. First question: Do you know that?

There's an ad put out by the Partnership for a Drug Free America. Personally, I support a drug free America, but I think the present efforts are having the opposite effect.

As an example, the ad says that there is more cancer causing tar in one joint than in a whole carton of cigarettes. That's nice and scary - but just not true. You don't get cancer from pot. If it was true, don't you think we (as a Nation) would KNOW it? We certainly KNOW that cigaretts kill people. Do you believe it? If it's true, please direct me to the scientific study that declares it to be so.

Do you think that it is the right tactic to take? Do you think pot smokers believe it? Don't you think, that if our goal was, indeed, a drug free America, that telling the truth about drugs would be a better tactic?

What is the truth about drugs? That we let Afghanistan grow poppies because it suits our political needs? That people who use crack cocaine need to be in jail a lot longer than people who use powdered cocaine? That if you get caught smoking pot, you'll never get a student loan?

I don't think those messages help us to achieve our goal. Do you?

excon

tomder55 answered on 06/03/05:


THC is a neuro-toxin .Besides that marijuana smoke contains toxins that are known to be hazardous to the respiratory system. Among them are carcinogenic polynuclear aromatic hydrocarbons, a prime suspect in cigarette-related cancers. Marijuana users experience the same health problems as tobacco smokers, such as bronchitis, emphysema and bronchial asthma.Smoking marijuana increases the work of the heart. The changes in heart rate and blood pressure are the same as those found in a person under extreme stress. The increased workload raises the heart rate and often the blood pressure, though the blood pressure may drop to an abnormally low level as well.

It increases the carbon monoxide in the blood and decreases the amount of oxygen. Some of the toxins are essentially a byproduct of combustion.Many of the toxins get stored in the human fat cells so they are present long after someone stops smoking . When you burn marijuana it produces quite a lot of tar(I don't know if it is 10x a cigarett but it is a higher concentration because pipes and joints generally do not have proper filters. Bongs are not the answer, because you inhale concentrated smoke. Some argue that a vaporized eliminates alot of the risk but I have no solid evidence of it.)

Marijuana contains many microorganisms. Many of these organisms may pose a threat to immunosuppressed individuals.(note to medical marijuana users who already in many cases have depressed immune systems).Both mold and bacteria are present in most samples of marijuana tested .To eliminate it would require sterilization and or irradiation of the herb .Alot of the toxins identified survive combustion ;that includes marijuana grown in hydroponic enviroment;let's not even discuss what fertiliers or other pesticides may be present in the marijuana on the street today;or the sanitary conditions of the person doing the harvesting .I remeber in my hay day of use how many times herbacides [paraquat]were used to try to eradicate the crops.

excon rated this answer Excellent or Above Average Answer

Question/Answer
sapphire630 asked on 06/01/05 - I need a laugh

would also like some stupid stories about
stupid criminals, stupid things people have sued over....and won and anything else of non-sense.

tomder55 answered on 06/02/05:

I go to the Darwin awards * for things like this :


(13 July 2004, West Virginia) An unidentified man in Blacksville, while relaxing in a portable outhouse, decided that it would be even more relaxing to light up a cigarette, inadvertently demonstrating another reason to give up the life-shortening habit. According to a spokeswoman for the ambulance, the methane in the porta-potty didnt take too kindly to the lit cigarette and exploded in a fireball. The man, whose identity was withheld due to privacy policies, was able to drive himself to the Clay-Battelle Health Center on the Mason-Dixon Highway. At that point, an ambulance was called in to transfer this difficult case to Ruby Memorial Hospital. The mans life was not in danger, but health care workers declined to reveal whether the mans injuries were serious enough to remove his contribution to the gene pool.

* The Darwin Awards salute the improvement of
the human genome by honoring those who accidentally
kill themselves in really stupid ways.

sapphire630 rated this answer Excellent or Above Average Answer

Question/Answer
sapphire630 asked on 06/01/05 - why are liberals...

so il~logical!?


....and while you are at it; give me some simple, nutty examples....

Thanx!

tomder55 answered on 06/02/05:

I am confused by the liberals . Until the revelation that Mark Felt was Deep Throat, I was sure that liberals disapproved when a top official of the FBI gathered information and used that information to accomplish a personal agenda, by threatening to use it to discredit top politicians, or even using it to bring down someone.

Itsdb rated this answer Excellent or Above Average Answer
sapphire630 rated this answer Excellent or Above Average Answer

Question/Answer
Choux asked on 06/01/05 - Correction

That would be turd-blossom. Internet anecdotes are notoriously inaccurate. :) Who called who a turd-blossom?

tomder55 answered on 06/02/05:

actually that is Bush's pet name for Karl Rove.

speaking of Putin ;

is Putin Paris Hilton in drag ?

or was he seperated at birth with Harry Potter character Dobby ?

Choux rated this answer Excellent or Above Average Answer
sapphire630 rated this answer Excellent or Above Average Answer

Question/Answer
excon asked on 06/01/05 - Nuclear (now "Constitutional") Option


Hello:

Yes, I think the filibuster will end. I think the radical right will kill it. I think it should stay, though. I agree with the moderates (although it kills me to say so).

It really has nothing to do with the present fight over judicial nominations, where admittedly, I do have an interest. It's nothing more than my wish to preserve the nature of the senate. Certainly, the Republicans will again be in the minority, and it should be preserved as a tool for them too (so they can defeat really good nominees).

Now, my question has nothing to do with what I just said. I wonder what you think it would be like today, if Deep Throat would have kept his mouth shut.

excon

tomder55 answered on 06/01/05:

Nixon and his administration were scum and deserved what they got . The press is still missing alot of the story. What was the connection between Felt and Woodward.(hint. Woodward worked for National Security Agency specifically for Al Haig and Kissinger before he became a "reporter" for the Washington Compost) . Did the 'echelon system 'bring down Nixon ?Since the CIA was banned from domestic spying did the Brits share intecepts with the NSA and the FBI ? Was there a Howard Hughes connection to Watergate ?Was Watergate a coup ?


This is a story where there are no good guys . Felt had a beef with the Nixonians because he got passed over for head of the FBI.He was in his mind the successor to Hoover but Nixon put L. Patrick Gray in the position. Felt was in charge of an investigation and handed over grand jury material to Woodward which is in itself a criminal act . The same people who are cheering him on today are the same ones who not so long ago was looking for someone to hang because of the outing of Valerie Plame.

The FBI was disfunctional under Hoover but if Hoover had ever disclosed to the press what he knew of let's say JFK and his sleeping with spies and Mafia princess how would the country have reacted ? Would they have blamed the president or would they have said that the domestic intelligence agency had gone rogue ?

None of this excuses Nixon . He was a disaster waiting to happen his whole political career . But those who think that Felt is the final piece to the puzzle are mistaken .

as to your question . Felt should've gone to the grand jury with his information . His actions are what makes me question his motives. There was a right way to do it . He instead took the criminal way to do it . Had he devoted his efforts to building a solid criminal case and a well run investigation instead of feeding his facts (some of them which were blatantly false) to Woodward I think the nation would've been better served .

excon rated this answer Excellent or Above Average Answer
Itsdb rated this answer Excellent or Above Average Answer

Question/Answer
Itsdb asked on 05/31/05 - Is Lott behind the "nuclear disarmament"?

The American Spectator reports:

"It hasn't been the best week," says a Republican leadership aide. "But Senator Frist isn't the one conservatives ought to be attacking."

If not Bill Frist, then who?

Well, for starters, Sen. Trent Lott, who, true to his reputation as true Senate Operator, was pulling the strings on the Gang of 14 nuclear disarmament team.

Knight Ridder last Friday reported on a secret meeting between Lott and Sen. John McCain in the hours leading up to last week's compromise. Lott apparently made a production of entering McCain's office space through a side door, but then, later spoke to several reporters about his meeting.

According to several Lott staffers involved with his management of the Rules Committee, Lott actually handed off his negotiations -- as well as the various proposals he had been working on with Sen. Ben Nelson -- after it was reported that Lott was trying to cut the legs out from under his Republican colleagues.

In fact, Lott was in almost constant contact with McCain and several other allies, including Sen. Lindsey Graham, who joined the coalition later in the negotiations. Throughout, however, Lott refused to sit in on further meetings with Democrats, leaving that to McCain.

"McCain was already on board, and Lott knew that he more than anyone would be willing to work the media in a way that Lott could not," says a Rules Committee aide. "But we were all working on this. When you have an opportunity to move into leadership, you don't pass it up."

Leadership, you say?

As previously reported by the Prowler, Lott has his eye on the majority leadership once again, after mismanaging the politics and the policies of the GOP Senate for several years.

"Lott knows how to work a caucus for a vote, but he just has lousy political instincts," says a Senate colleague. "The Strom Thurmond mess was just the capper."

Along the way, Lott was singularly responsible for the 50-50 split Republicans had to deal with after the 2000 election, when he allowed Sen. Connie Mack of Florida to retire from his safe seat with no clear favorite to replace him in 2000.

Lott has been looking for ways to undercut both President Bush and Sen. Frist, as he blames both -- though Bush more -- for his political purgatory out of leadership.

But Lott intends to challenge Sen. Mitch McConnell for Senate leader after Frist's retirement in January 2007. "This coalition is more than just about judges," says the Rules Committee staffer. "It's a Republican group that the Senator believes will give him his leadership slot back. These independents won't support McConnell. At least that's what we think now."

Lott, apparently, isn't stopping at the judge deal. According to a Senate Democratic leadership aide, the man from Mississippi has been speaking with Sen. Joe Biden about brokering a Bolton vote, again pulling an end run around Frist and his leadership team.

"He isn't trying to help the President," says the leadership aide. "He's working the caucus in a way that would damage Bolton's chances for confirmation. Every conservative should be worried about this.
"

Frist's lousy week has little to do with his leadership abilities, which should not be in doubt given his track record of forcing the issues with Democrats. It has more to do, Senate insiders say, with the unstable and often precarious situation that takes hold in the cloakrooms before Senators reach the floor for votes. It is there where the real arm-twisting takes place, not among staffers and their bosses, but the bosses themselves.

"I don't think a lot of these guys know which they will vote before they hit the floor on some issues," says a Senate staffer. "I don't know what my boss is going to do half the time. It might depend on which Republican he talks to before he walks out on the floor. Sometimes it's that fluid. Frist and his people can only do so much before it is out of his hands. The judges votes is a great example of that."

~~~~~~~~~~~~~~~~~~~~~~~~~~~~~~~~~~~~~~~~~~~~~~~~~~~~~~~

Comments?

Steve

tomder55 answered on 06/01/05:

It would not suprise me . Disgruntled Lott teamed up with disgruntaled McCain to undermine the majority leadership and the President. It doesn't matter . We know what they are dealing with . The Democrats proved with the Bolton cloture vote that they cannot even honor deals they make for more than a couple of hours . These so called moderates and RINOs shot themselves in the foot and all they accomplished was strenghtening the Democrats hand.Eventually a fillibuster will be tried ,and the nuclear option will have to be invoked. It is unfortunate however that the Senate will be a 'do nothing 'Congress for the 2nd Bush term . Unfortunate because there is a lot to get done .

Itsdb rated this answer Excellent or Above Average Answer

Question/Answer
powderpuff asked on 05/30/05 - the arrest of Mohsen Abdul-Hamid in Baghdad

I'm confused about the arrest of Mohsen Abdul-Hamid.

I read that the President of Iraq condemned the arrest and demands Hamid's immediate release. "The Iraqi president said that no one gave prior notice to the Presidential Council about the arrest of Dr. Mohsen Abdul-Hamid. This way of dealing with such a distinguished political figure is unacceptable," the statement added.

Do you think Mohsen Abdul-Hamid will be released immdediately or will he be held until he is investigated for possibly assisting and organizing attacks by enemies of Iraq?

If he is helping the insurgency, what difference does it make that he is a political figure in Iraq? He should still be held for crimes against his country shouldn't he?

tomder55 answered on 05/31/05:

It was a screw up ;no other way to describe it . Miscalculations and mistakes are all a part of war ,but it is not good enough to chalk it down as 'we blundered ....oops'.
Iraqi officials are right in saying that such actions undermining efforts to include Sunnis in the nation's political process.Actions of this nature should be filtered through a consultation of Iraqi leaders . We want them to build their own government ;we should start trusting their judgement a little more. Also I am looking for a demand of accountability from the Pentagon.The field commander either blundered or took orders from someone higher.

powderpuff rated this answer Excellent or Above Average Answer

Question/Answer
Itsdb asked on 05/26/05 - Cahoots?

I wonder if Howard Dean and Ronnie Earle have been hanging out together. Our paper pointed out something interesting in an editorial this morning:

"One would think Ronnie Earle, the veteran Travis County district attorney, would be smarter than what he showed the other day.

Earle is up to his eyeballs in an investigation into whether Republican House Majority Leader Tom DeLay violated state campaign finance laws.

Republicans contend Earle is a partisan hack, a Democrat "out to get" DeLay.

Democrats defend the prosecutor, saying he has gone after Democratic legislators in the past as well.

So, what does Earle do? He stands on May 12 before a Democratic crowd in Dallas, raises about $102,000 for potential Democratic candidates and then talks directly about the man - DeLay - he is investigating for alleged criminal wrongdoing.

Earle's performance in Big D - offering up the political red meat on which partisans in both parties thrive - gives the GOP all the ammo it needs to undercut his efforts at finding the truth about DeLay and whether he violated state campaign spending laws.

Earle has been in public life a long time. He has been at the center of these kinds of investigations before. Earle clearly should have known better than to speak so openly and with such contempt about DeLay at a partisan event.

Then again, maybe all that political power Earle has acquired as district attorney in Travis County has gone to his head."

The Houston Chronicle quoted Earle as saying "This case is not just about Tom DeLay. If it isn't this Tom DeLay, it'll be another one, just like one bully replaces the one before..."

"This is a structural problem involving the combination of money and power," he added. "Money brings power and power corrupts."

~~~~~~~~~~~~~~~~~~~~~~~~~~~~~~~~~~~~~~~~~~~~~~~~~~~~~~~

Funny he should say that, helping raise $102,000 for a democratic organization...

Steve

tomder55 answered on 05/26/05:

alot of the prosecutors are publicity hounds. Look at the lengths that Gwinnett County district attorney Danny Porter went to keep the 'Run-away bride 'story going . What a piece of work ;what a waste of resources ! He says that she cost the county money . How much is his charade costing ?

We got this hot shot State AG Eliot Spitzer going after people like Martha Stewart and parlaying his renown into a run for the Govenorship.

You can find this type of stuff all over the country .If they get a big name in their sites forget-about-it . There isn't a camera or a mike they won't gravitate to .There isn't a talk show they refuse to appear on .

If Ronnie Earle had anything on Delay you would've already seen him do the perp walk.

Itsdb rated this answer Excellent or Above Average Answer

Question/Answer
ROLCAM asked on 05/25/05 - Noise Pollution ??

What rights do residents living next to main roads have, to reduce or if possible eliminate noise pollution caused by light and heavy vehicles? Is the government obliged to install noise absorbers along the roads?

tomder55 answered on 05/25/05:

no they aren't unless there is ordinances prohibiting excessive noise.

The resident can certainly petition the local gvt. to get corrective action (the more residents involved the more likely they will respond ). The best thing for him to do is to get someone to monitor decible readings and to report the results to the locality .Most townships are reasonable about it and will at least restrict the noise during the late night and early morning times.

Itsdb rated this answer Excellent or Above Average Answer
ROLCAM rated this answer Excellent or Above Average Answer

Question/Answer
ROLCAM asked on 05/25/05 - Who uttered these words ?

Who coined the catchphrase "nabobs of negativism" ?

tomder55 answered on 05/25/05:

Spiro T Agnew ; Nixon's Vice President before he had to resign due to corruption. He was talking about the press and it's coverage of the Vietnam war .

ROLCAM rated this answer Excellent or Above Average Answer

Question/Answer
purplewings asked on 05/23/05 - Does the government fund your cancer prescriptions?

Probably not since the money is going here instead:

ALBANY, N.Y. (May 22) - Scores of convicted rapists and other high-risk sex offenders in New York have been getting Viagra paid by Medicaid for the last five years, the state's comptroller said Sunday.

Audits by Comptroller Alan Hevesi's office showed that between January 2000 and March 2005, 198 sex offenders in New York received Medicaid-reimbursed Viagra after their convictions. Those included crimes against children as young as 2 years old, he said.

Hevesi asked Michael Leavitt, secretary of the U.S. Department of Health and Human Services, in a letter Sunday to "take immediate action to ensure that sex offenders do not receive erectile dysfunction medication paid for by taxpayers."

A call to Leavitt's office was not immediately returned Sunday.

According to Hevesi, the problem is an unintended consequence of a 1998 directive from federal officials telling states that Medicaid prescription programs must include Viagra. His office discovered that the state was helping sex offenders pay for Viagra by checking Medicaid pharmacy expenditures against the state's sex offender registry.

New York's two senators said Sunday the problem should be corrected.

Sen. Hillary Rodham Clinton said in a statement that it was ''deeply disturbing and runs contrary to the purpose of Medicaid, which is to provide health care coverage for uninsured, low-income individuals.'' Clinton, a Democrat, urged Leavitt to look into the matter, and said she would explore legislative options.

New York's other senator, Democrat Chuck Schumer, said at a press conference in New York City that he hoped the issue could be resolved without a bill, but he's prepared to offer one if needed.

''While I believe that HHS did not do this intentionally, when the government pays for Viagra for sex offenders, it could well hurt many innocent people,'' he said.

New York auditors are reviewing whether other prescription drugs for sexual dysfunction are being reimbursed by Medicaid for convicted sex offenders, Hevesi spokesman David Neustadt said.

While the auditors didn't review the situation on Viagra reimbursement by Medicaid in other states, he said they have no indication that the policies are different elsewhere.


tomder55 answered on 05/24/05:

Leave it to Chuck the Schmuck to notice this .I guess he got tired of standing under gas station price signs on Sunday mornings begging the President to open up the strategic reserves so SUV drivers could get a price break.
Hillary and Schumer ought to be careful what they say . These sex offenders if they have their way will be their largest most faithful future voting block if they get their way .
It is outrageous of course ,and the real question is ;why should drugs like Viagra and other lifestyle drugs be included in any Gvt. giveaway other peoples money benefit plan?

purplewings rated this answer Excellent or Above Average Answer

Question/Answer
Itsdb asked on 05/24/05 - Praise be the Senate...

Robert C. Byrd--"the republic is saved"

So the Dems have come to the rescue. The Senate has avoided the "nuclear" (constitutional) option" and will hold a vote on 3 judical nominees while still sacrificing others. How long has this been the plan, for the left to hold out until the last minute and "save" the republic? Will that be the rallying cry in the next election, if it weren't for the democrats our republic would have been doomed?

Steve

tomder55 answered on 05/24/05:

I noticed yesterday how White Sheets Byrd looks an alot like Star Wars Emperor Palpatine . We can call this chapter the 'Revenge of the RINOs '.

The seven Republican signers were Sens. Mike DeWine of Ohio, Susan Collins of Maine, Lindsey Graham of South Carolina, Lincoln Chaffee of Rhode Island, John McCain of Arizona, John W. Warner of Virginia and Olympia J. Snowe of Maine. Click to Visit The seven Democratic signers were Sens. Ben Nelson of Nebraska, Joe Lieberman of Connecticut, Mark Pryor of Arkansas, Robert C. Byrd of West Virginia, Mary L. Landrieu of Louisiana, Ken Salazar of Colorado and Daniel K. Inouye of Hawaii.

Now instead of having a majority approval for justice nominations we will have a cabal of 14 "moderates " making the decision . It almost made me ill watching the love fest on the stage. The only saving grace from the deal they forged is it is only bound by the "trust they have for each other " . The deal should last about a week then. I will be interested to see how many of these Republicans jump ship and vote no on the three nominees that are now up for a vote .

It appears that the Republicans cannot handle being in the majority .It may be that they had been a minority party for so long that they just don't know how to handle success.

It will be interesting to watch the Democrats filibuster Bush's Supreme Court nominees. Are there 'extraordinary circumstances 'related to Scalia ? Probably .If I were the President I would nominate Scalia for Chief Justice and fill his position by nominating Edwin Meese ;and if they gave me a hard time about it ....Robert Bork (but Bush will probably pick Alberto Gonzalez for the vacant seat ) .

With this deal it has now become as much a Republican power grab as a Democrat one . We're going to start talking about who would be a good judge and who wouldn't," ..."And the White House is going to get more involved and they are going to listen to us more." Lindsey Graham (R-S.C.).

There is a difference between advising, consenting & using a Senate rule to override the US Constitution. Some Senators from both sides of the isle seem unable to understand that. They are more concerned about the "tone" in their workplace than in their Constitutional responsibilities.Just to remind them .It is the Presidents Constitutional responsibility to select judges .


excon rated this answer Excellent or Above Average Answer
Itsdb rated this answer Excellent or Above Average Answer

Question/Answer
sapphire630 asked on 05/23/05 - whoSane?

Now I guess we are going to spend millions of dollars on trying to get to the bottom of who took the pix of Saddam because some PC sensitive idiots are offended and worry his Geneva convention rights have been violated.

I think I would be too busy being grossed out to be caring about offended.

I know this is old by now but with the public library computers getting pickier with the parental control thing many computers will not let me on this site.

tomder55 answered on 05/24/05:

I don't have my shorts in a knot over this . If we had pix of Hitler in briefs during WWII and they were published would anyone been upset ? I don't think the Germans would've fought any more determined either .

I do think we need to find and deal with the leaker .

sapphire630 rated this answer Excellent or Above Average Answer

Question/Answer
ken123 asked on 05/22/05 - Ideologies

I just finished a first year political studies class and I have this question that has been bugging me for a while...

When a government is in power and they make a big mistake, the opposition parties try to capitalize on this by gaining more support. For example, in Canada, the Conservatives are trying to overthrow by gaining the support lost by the Liberal party.

Maybe it is just because I am young and have not fully involved myself in politics yet but I do not understand how a mistake in government makes people switch their political ideology support... Someone who supported the Liberal Party obviously supports liberalism. Why would they just switch to conservatism because of a mistake? In my opinion, I do not think a mistake can really influence your political beliefs. Is there no way for the angry Liberal supporters to seek changes in the party itself?

I also understand that this is supposed to be one of the advantages of democracy. If a party does not fulfill its expectations, it may not return to power when the next election comes around.

If there were a socialist government and they made an error, I find it really absurd that supporters of social programs, etc. would all of sudden support a conservative government (for example).

All feedback is appreciated! Again, please remember I am young so I you feel like I am missing information on an important part, let me know! Thanks

tomder55 answered on 05/23/05:

the fault lies in the presupposition that every one has a solid ideological belief in a political system one way or the other . The political parties in Canada hold stronger ideological positions and try to infuence people to vote for them ,and then build coalitions inside the gvt to obtain a majority . In the US the coalition building happens inside the political parties so there is less of an ideological identification between the parties.(current polarizations being the exception)

ken123 rated this answer Excellent or Above Average Answer

Question/Answer
Itsdb asked on 05/21/05 - It's not just Newsweek

Sometimes, someone else just says what you're thinking better...

Michelle Malkin

May 18, 2005

If you want to hear an earful, ask an American soldier how he feels about our news media. You will invariably hear an outpouring of dismay and outrage over antagonistic and reckless reporting. I have stacks of letters and e-mails from soldiers and their families sharing those frustrations. During the Vietnam War, those sentiments would get packed away -- private hurts to be silently borne for decades.

But today the Internet has allowed soldiers on the front to disseminate their views -- breaking through the media's entrenched, anti-military bias -- in unprecedented ways. In the wake of Newsweek's publication of its unsourced, mayhem-inducing and now-retracted item about Koran desecration by U.S. military interrogators at Guantanamo Bay, a sergeant in Saudi Arabia immediately responded on a blog called The Anchoress (theanchoressonline.com):

I have placed my life and the life of my fellow soldiers in danger in order to achieve a measure of the freedoms we enjoy at home for the Iraqi and Afghani people. As soldiers, we all understand that we may be asked to participate in wars (actions) that we (or our countrymen) don't agree with. The irresponsible journalism being practiced by organizations such as Newsweek, however, [is] just inexcusable. At this point, because of their actions and failure to follow up on a claim of that magnitude, they've set the process back in Afghanistan immensely . . .

I don't regret serving my country, not one bit, but to have everything I'm doing here undermined by irresponsible journalists leaves me disgusted and disappointed.


Military bloggers across the Web this week echoed the sergeant's disgust with American journalism. And it's not just Newsweek.

It's the New York Times and CBS News and the overkill over abuses at Abu Ghraib prison. It's the Boston Globe publishing porn photos passed off by an anti-war city councilor as proof that American GIs were raping Iraqi women.

It's the constant editorial drumbeat of "quagmire, quagmire, quagmire."

It's the mainstream media's bogus reporting on the military's failure to stop purported "massive" looting of Iraqi antiquities.

It's the hyping of stories like the military's purported failure to stop looting of explosives at al Qa Qaa right before the 2004 presidential election -- stories that have since dropped off the face of the earth.

It's the persistent use of euphemisms -- "insurgents," "hostage-takers," "activists," "militants," "fighters" -- to describe the terrorist head-choppers and suicide bombers trying to kill American soldiers and civilians alike. It's the knee-jerk caricature of American generals as intolerant anachronisms. It's the portrayal of honest mistakes in battle as premeditated murders.

It's the propagandistic rumor-mongering spread by sympathizers of Italy's Giuliana Sgrena and former CNN executive Eason Jordan about American soldiers targeting and/or murdering journalists.

It's the glorification of military deserters, who bask in the glow of unquestioning -- and largely uncorroborated -- print and broadcast profiles.

And it's the lesser-known insults, too, such as the fraudulent manipulation of Marine recruits by Harper's magazine. In March, the liberal publication plastered a photo of seven recruits at Parris Island, S.C., under the headline, "AWOL in America: When Desertion Is the Only Option." None of the recruits is a deserter. When some expressed outrage over the deception, the magazine initially shrugged.

"We are decorating pages," sniffed Giulia Melucci, the magazine's vice president for public relations, to the St. Petersburg Times.

As Ralph Hansen, associate professor of journalism at West Virginia University and a rare member of academia with his head screwed on straight, observed: "Portraying honorable soldiers as deserters is clearly inappropriate. And I don't see any way Harper's could claim that they weren't portraying the young Marines as deserters. A cover is more than just art. I think that someone had a great idea for a cover illustration and forgot that he or she was dealing with images of real people."

The members of our military are more than just an expedient means to a titillating magazine cover or juicy scoop or Peabody Award. Too often since the "War on Terror" was declared, eager Bush-bashing journalists have forgotten that the troops are real people who face real threats and real bloodshed as a consequence of loose lips and keyboards.

It's not just Newsweek that needs to learn that lesson.

~~~~~~~~~~~~~~~~~~~~~~~~~~~~~~~~~~~~~~~~~~~~~~~~~~~~~~

Comments?

tomder55 answered on 05/22/05:

Ann Coulter on the Newsweek story :

When ace reporter Michael Isikoff had the scoop of the decade, a thoroughly sourced story about the president of the United States having an affair with an intern and then pressuring her to lie about it under oath, Newsweek decided not to run the story. Matt Drudge scooped Newsweek, followed by The Washington Post.

When Isikoff had a detailed account of Kathleen Willey's nasty sexual encounter with the president in the Oval Office, backed up with eyewitness and documentary evidence, Newsweek decided not to run it. Again, Matt Drudge got the story.

When Isikoff was the first with detailed reporting on Paula Jones' accusations against a sitting president, Isikoff's then-employer The Washington Post which owns Newsweek decided not to run it. The American Spectator got the story, followed by the Los Angeles Times.

Assistant Managing Editor Evan Thomas justified Newsweek's decision to run the incendiary anti-U.S. story about the Quran, saying that "similar reports from released detainees" had already run in the foreign press "and in the Arab news agency al-Jazeera."



The legacy media's reaction to the outing of Newsweek has been to go on the aggressive. We've already mentioned the press conferense where they verbally assaulted Scott McClellan and challenged the Administrations right to complain about their coverage or suggest corrective action.
The NY Times has begun extensive reporting on prisoner abuse in Afghanistan.Sort of if we got that one wrong we'll give you simular stories ;throw spaghetti on the wall ;some of it will stick.But as the saying goes :If the Koran doesn't fit, you must acquit.

It must be demoralizing to the troops as Michele Malkin is trying to illustrate ,and walking on egg shells is a dangerous way to conduct a war. The protocol established before this recent flap was to handle the Koran wearing protective gloves and to treat it in an almost ceremonious way .By the time the media is finished with this we will have our prison guards bowing to Mecca 6 tmes a day .

Itsdb rated this answer Excellent or Above Average Answer

Question/Answer
excon asked on 05/21/05 - Sexual Offender Registry


Hello experts:

Bal got me to thinking.

I too, am a citizen who wishes to be left alone. As such, why would I only be interested in the sex offenders who lived by? I dont have children at home, but I have a car. They should post where all the car thieves live too, shouldnt they? Why just sex offenders? Because theyre the worst? Theyre the sickest, for sure, but not the worst. I have a house, so Im very interested in where all the burglars live. Wouldnt you? I walk the streets, so Id definitely want to know where all the muggers live. Why not? And yes, sometimes I find myself inside a bank lobby. How come I dont know where all the bank robbers live - so I can protect myself? Drunk drivers??? Id certainly like to know where they live. Wouldnt you?

And, why stop there? Why not tell us where every single bad guy is and what theyre capable of? Dont you want to know?

But then, where would we live? At the rate people have been locked up, Ill bet theres at least one or two bad guys living on your block. And, where would they live? On the streets? Thats where MOST sex offenders live, because theyve been driven out of our neighborhoods. That doesnt make us any safer - maybe even less safe. Maybe we should just round them up and put them in camps.

excon

tomder55 answered on 05/22/05:

I support the identification of some of the sex crimes comitters but think that often the laws are written too broadly . The term sex offender brings to mind the worst child molester(the ones I think should be identified).But many sex offender laws include persons who had underage consensual sex or who's only crime was downloading the wrong dirty pictures of the internet. Laws without boundaries are bad laws. It is a complex problem .You cannot lock them away forever ;but once they are out they need a place of residence. It is reasonable for a parent to want to know if a child molester ;who are high risks for reoffending;is living in their neighborhood.By limiting the publicized list to the people who have committed sexual assault ,or that have demonstrated repeat criminal behavior the list becomes more useful.

excon rated this answer Excellent or Above Average Answer

Question/Answer
Itsdb asked on 05/20/05 - WTC Survey

OK folks, check out the following images and give us your vote:

Twin Towers II

Freedom Tower

Please also explain the reasoning behind your vote...and I consider your opinion of the Donald to be irrelevent for the purpose of this survey.

Steve

tomder55 answered on 05/21/05:

I do not like either .

People forget that when the Twin Towers was forst constructed they were considered monstrosities . Why build it when reting them would be so difficult ? No one would rent above the 50th floor .

The Freedom tower is a 70 story building with a 50 story shell structure on top of it . Trump is right to call it a 'disgusting pile of junk '.

Trump got it right later when he made this comment. "If for some reason, it can't be built, because there is a possibility that people do not want to be in any of the buildings on the site, then what we should do is ... build a great memorial park ".

There is enough room there to build reasonable sized office space and to make a memorial park. Today the WTC site attracts tourism and all it is today is a hole in the ground . Build a park there and people will go to it just like they go to the USS Arizona Memorial at Pearl Harbor .

Itsdb rated this answer Excellent or Above Average Answer

Question/Answer
excon asked on 05/20/05 - Rapist Released Early!


Hello experts:

I think I know the answer to my question, but I want to hear it firsthand. Im referring to cases that have been reversed on appeal.

When you hear or read headlines like that, what is your first reaction? That some smart criminal lawyer found a technicality? That the bad guy found a liberal judge? That this is just more evidence of a broken justice system that coddles criminals?

Or, do you think that this is just another example of the Constitution at work, protecting all of us?

Do you know that every conviction that is reversed is reversed because a defendants rights are violated? Do you think the Constitution is just a technicality? Do you believe a defendant should have rights? Do you believe that a prosecutor has to obey the rules? And, do you believe that if he doesnt, it shouldnt matter in terms of the conviction? Do you believe that when one of these things happen, you and your family are more or less safe? Do you believe that your rights are protected when these things happen, or do you believe you dont need these rights (and the bad guys shouldnt have them either), because you dont break the law and dont need them because the cops will never come after you? If you think the Constitution is a technicality, would you think the same thing if the cops arrested you? Or do you think that the cops just will NEVER arrest you?

Do you believe that OJ was found not guilty because, (a) the jury was racist, (b) he had a good attorney, (c) the prosecutor blew it, (d) he wasnt guilty?

I vote for c. Why? I think he did it, but the prosecution put a cop on the stand who lied his ass off and got caught by the defense. If I were on the jury, that - and that alone - would have been reason for my vote to acquit.

Finally, no, I dont believe bad guys should be let out of prison. I do believe, however, that they should be put in there according to the rules, and that if they arent, its the prosecutors fault. Plus, I think the term bad guys needs to be refined. From my standpoint, I dont think anybody who is in jail for marijuana offenses are bad guys, and should all be released forthwith.

excon

tomder55 answered on 05/20/05:

I do not know the specifics of the case but yes ;usually a release on appeal is not a matter of guilt as much as a technical screw up by a prosection or a judge.

Or, do you think that this is just another example of the Constitution at work, protecting all of us?

how could it be ? if a rapist is released on a technical violation of procedures and is free to commit the crime again ;how is that protecting all of us ? I'm not saying that he shouldn't be freed . There are rules to the game ,and the law enforcement should also follow the rules also .

With OJ I think it was A/B/C and not D .

I do not think marijuana possession merits jail time either unless it contributed to another crime or accident. But I do think deals should do time .


excon rated this answer Excellent or Above Average Answer
purplewings rated this answer Excellent or Above Average Answer

Question/Answer
Itsdb asked on 05/19/05 - Dean unhinged...

Tom posted a great column by Robert Novak telling how Howard Dean's rants have been largely ignored by the MSM. I'd have to say he's right.

A Google news search of the following quote returned one hit, the Arizona Republic. They quote Dean as accusing the president of having "stole a trillion dollars out of the Social Security trust fund so he could give it away in tax cuts."

Dean, also quoted in the article as "tired of being lectured to about moral values by a corrupt party," has some interesting values himself. As has been pointed out earlier, he's tried and convicted Tom DeLay who he thinks "ought to go back to Houston where he can serve his jail sentence."

He's also quoted as saying "there's corruption at the highest level of the Republican Party, and they're going to have to face up to that one of these days, because the law is closing in on Tom DeLay," and "I think he's guilty ... of taking trips paid for by lobbyists, and of campaign-finance violations during his manipulation of the Texas election process."

Now contrast that with this quote by Dean:

"I've resisted pronouncing a sentence before guilt is found. I will have this old-fashioned notion that even with people like Osama, who is very likely to be found guilty, we should do our best not to, in positions of executive power, not to prejudge jury trials."

So the DNC head is ready for a Republican to go to jail without a trial, but doesn't want to prejudge Osama? The only hit returned on this Osama quote in a Google news search was WorldNetDaily.

Why so much media silence on Howard Dean? Why isn't the media holding his feet to the fire for some of the outrageous things he's doing and saying?

Steve

tomder55 answered on 05/19/05:

I am a big fan ;a Deaniac .yeeeeeeeeeeeeeehhhhhhhh!!! I want him to keep on talking ;yucking it up .Let him the man who's party is above racism continue to say : You think the Republicans could get this many people of color in a single room?".... Only if they had the hotel staff in here. He is the perfect leader for people like Senator Robert 'white sheets' Byrd who once belonged to the KKK and used the term white nigger on national television. Or the leader of NAACP President Kweisi Mfume now a US Senate Candidate in Maryland who called Condi Rice, Colin Powell, and Roderick R. Paige hustlers and yes men. The party of inclusiveness that is now fillibustering a highly qualified African American Court of Appeals nominee ;Janice Rogers Brown ; who was raised in Democrat segregationist South and became a self made success who was slammed by another great Democrat ;Harry Reid with this inane comment "She is a woman who wants to take us back to the Civil War days," .

yep Dean is a perfect spokesman for this group of missfits .


Itsdb rated this answer Excellent or Above Average Answer

Question/Answer
Itsdb asked on 05/18/05 - Media with impunity?

From yesterday's White House press briefing...

"Scott, you said that the retraction by Newsweek magazine of its story is a good first step. What else does the President want this American magazine to do?

MR. McCLELLAN: Well, it's what I talked about yesterday. This report, which Newsweek has now retracted and said was wrong, has had serious consequences. People did lose their lives. The image of the United States abroad has been damaged; there is lasting damage to our image because of this report. And we would encourage Newsweek to do all that they can to help repair the damage that has been done, particularly in the region.

And I think Newsweek can do that by talking about the way they got this wrong, and pointing out what the policies and practices of the United States military are when it comes to the handling of the Holy Koran. The military put in place policies and procedures to make sure that the Koran was handled -- or is handled with the utmost care and respect. And I think it would help to point that out, because some have taken this report -- those that are opposed to the United States -- some have taken this report and exploited it and used it to incite violence.

Q With respect, who made you the editor of Newsweek? Do you think it's appropriate for you, at that podium, speaking with the authority of the President of the United States, to tell an American magazine what they should print?

MR. McCLELLAN: I'm not telling them. I'm saying that we would encourage them to help --

Q You're pressuring them.

MR. McCLELLAN: No, I'm saying that we would encourage them --

Q It's not pressure?

MR. McCLELLAN: Look, this report caused serious damage to the image of the United States abroad. And Newsweek has said that they got it wrong. I think Newsweek recognizes the responsibility they have. We appreciate the step that they took by retracting the story. Now we would encourage them to move forward and do all that they can to help repair the damage that has been done by this report. And that's all I'm saying. But, no, you're absolutely right, it's not my position to get into telling people what they can and cannot report."

~~~~~~~~~~~~~~~~~~~~~~~~~~~~~~~~~~~~~~~~~~~~~~~~~~~~~~~

Newsweek publishes an unsubstantiated story that damages the credibility of the country and is used by Islamic fundamentalists to incite riots leading to deaths and threats of Jihad on the US...and the president gets chastised for it. How insane is that?

Steve

tomder55 answered on 05/19/05:

here is Bob Novak's commentary on Dean :

May 19, 2005
Howard Dean, Unmuzzled
By Robert Novak

WASHINGTON -- After Howard Dean last weekend declared Tom DeLay ought to be in jail, a longtime Democratic operative told me the party's national chairman had momentarily ripped off his muzzle but that it soon would be restored. My source erred, however, in believing that Dean ever had been muzzled. It's just that nobody has paid much attention to his rants.

Since his election as chairman of the Democratic National Committee (DNC) on Feb. 12, Dean has studiously avoided most national television exposure. But he has been talking constantly to party gatherings across the country, and his intemperate language at these outings contradicts the notion that he has been kept under control. That he will leap onto the national stage Sunday by accepting a long-pending request to appear on NBC's "Meet the Press" with Tim Russert raises concern among the Democratic political players whether he will contain himself.

Dean's election by the DNC membership was a case of the inmates seizing control of the asylum. After the 2004 election, party leaders spent more than three months in a fruitless effort to find an alternative to Dean. Their fears of money drying up under Dean have largely been realized, but they have deluded themselves into thinking the former Vermont governor who screamed his way out of any hope for the 2004 presidential nomination was under firm restraint.

The party's congressional leaders, Sen. Harry Reid and Rep. Nancy Pelosi, sat down with the newly elected Chairman Dean for a heart-to-heart talk. They politely urged him to restrain his rhetoric, to organize rather than inflame. Dean thereupon buried himself seeking Democratic converts in the "red" states of Republican America, giving the impression that he was heeding the pleas of the congressional leadership.

He was not. He has described the Republican leadership, in various venues, as "evil," "corrupt" and "brain-dead." He has called Sen. Rick Santorum, chairman of the Senate Republican Conference, a "liar." He has referred to conservative radio host Rush Limbaugh as "drug-snorting."

What he said last weekend differed from this invective only in that it was presented to an urban forum and so became public knowledge. Addressing the Massachusetts Democratic convention in Lowell, Dean declared: "I think DeLay ought to go back to Houston where he can serve his jail sentence down there courtesy of the Texas taxpayers." Dean would jail DeLay without trial, without indictment and without accusation of any crime.

National chairmen are supposed to fire up the troops, but Dean's rhetoric crosses a line. What he said was too much even for so tough a partisan Democrat as Rep. Barney Frank, who attended his state's convention in Lowell and was appalled by Dean's language.

Dean's deficiencies as face and voice of the Democratic Party were supposed to be overcome by his legendary prowess, evident by his run for president, raising funds in small packages. That so far has proved a grievous disappointment. First quarter figures show the DNC received only $13 million from inviduals, compared to $32 million raised by the Republican National Committtee (RNC). Overall figures were $34.2 million by the RNC, $16.7 million by the DNC.

Dean has not always kept himself faithful to the Democratic message. On Feb. 23 at Cornell University, he blurted out that Social Security benefits -- if the system is left unchanged -- 30 years from now will be 80 percent of what they are now. That was a shocking departure from the party line that nothing has to be done.

But the only place that Dean's Social Security departure appeared was in the Cornell Daily Sun, the student newspaper. His limited exposure generally means that little of what he says is communicated to the public. He has been convinced that he has nothing to gain from face-to-face debates on television with his cool, well-organized Republican counterpart, Ken Mehlman.

Accordingly, anticipation of Howard Dean, cut loose and unmuzzled, on "Meet the Press" Sunday is unsettling for the party's faithful. This will be his first exposure as chairman on a major network interview, and Russert predictably will be well prepared with a rap sheet of the chairman's verbal assaults. The prospect that Dean will make juicy additions to that collection unnerves Democrats.



Itsdb rated this answer Excellent or Above Average Answer

Question/Answer
sapphire630 asked on 05/18/05 - I'm opposed to it!

I don't know what they have to say,
it makes no difference anyway -
whatever it is, I'm against it!
No matter what it is or who commenced it,
I'm against it!

Your proposition may be good,
but let's have one thing understood -
whatever it is, I'm against it!
And even when you've changed it or condensed it,
I'm against it!

I'm opposed to it.
On general principles I'm opposed to it.

For months before my son was born,
I used to yell from night to morn -
"Whatever it is, I'm against it!"
And I've kept yelling since I first commenced it,
"I'm against it!"

Grouch Marx

tomder55 answered on 05/19/05:

did you say Harry Reid ?

sapphire630 rated this answer Excellent or Above Average Answer

Question/Answer
sapphire630 asked on 05/18/05 - Everybody gave her the finger.....

Now the story is that the Wendy's chili finger
was given to her husband by a friend who lost it in an accident. He owed her husband 50.00 so he gave him his finger instead.

Can I get away with giving the gas company my finger instead of fifty bucks?

tomder55 answered on 05/18/05:

but they did find a finger in pudding . so we're 1 of 2 in the finger dept. That gives you a 50-50 chance I figure .

Itsdb rated this answer Excellent or Above Average Answer
sapphire630 rated this answer Excellent or Above Average Answer

Question/Answer
excon asked on 05/18/05 - Civil war - Iraq


Hello pundits:

Is Iraq in the midst of a civil war? How would we know? Does somebody have to declare it as such? If somebody does (I doubt Bush would), what do we do? What is the point of our continued occupation? Can the insurgency outlast us? Really, I'm not sure what we're doing there anymore. WMD's?? Ok, that's settled. Regime change?? Ok, that's settled. Security for their country?? That'll never happen. We can't even secure our own country and we have friends on all sides.

By the way, I suggested round about 2 years ago that Iraq would devolve into civil war, and I think it did. History shows that the average insurgency lasts 9 years. We've been there for 2. Are we going to stay for another 7?

I don't think we will. Should we leave early, what do you think will happen, both here and there. Of course, if you think we'll leave with everything being hunky dory, I'm accepting bids for the Space Needle.

excon

tomder55 answered on 05/18/05:

The US will have a presence there at least until 2007 . Whether the central gvt, survives remains to be seen . What is happening now is not civil war ;but an attempt by the jihadists to create one (as has been referenced countless times from Zarqwai's letter). Behind the scene the Bush Adm. has been thinking about the possibility that a strong central gvt. in Iraq migh fail (the only example of that succeeding is the brutal dictatorship model which we clearly do not endorse). The Council on Foreign Relations has recommended as a possibility the restructuring of Iraq into six states under a single national government. David Phillips of the council says ;
"It is about the distribution of political power through institutions and laws that guarantee accountable rule," ..."In the new Iraq, federal Iraqi states should control all affairs not explicitly assigned to the federal government"....Phillips, proposed the establishment of two or three states dominated by Shi'ites. Another state would be comprised of mostly Sunnis and a third state would be Kurdish. Baghdad would be a separate state.

Phillips authored a report called Power Sharing in Iraq. It says ;"Consistent with the principle of decentralisation, federal Iraqi state and local authorities should have the ability to adopt laws that conform to local custom." A far cry from Western Democracy for sure but still it retains the essence of self determination /democratization. It is clear that a Federal system is the model for the new Iraqi Constitution .

Recent successes in (for lack of a better word)Iraqization has allowed the coalition to hand over the security of much of the country over to primarily the Shia and Kurds. It will now be up to the Sunni to decide that they want to participate in the political and security arrangement .The impact of the Iraqi elections has been to accelerate the transformation of a potential nationalist resistance to U.S. occupation into a more narrowly based Sunni resistance to Shiite domination. Attacks on US troops are down ,and attacks on civilians have unfortunately risen. There is no doubt that a Sunni-based insurgency will ultimately be defeated ;if not by the US then by the combined weight of the Shia and Kurd populations . I guess that it could be argued that makes the situation a pseudo-civil war ;one that the Sunnis have not hope of success in ;and one that could not possibly devolve into a full scale conventional war (unless there was a future falling out of the Shia Kurdish alliance.)

The progress made in the forming of a central gvt. will prevent a full scale civil war .The Sunnis have badly suffered from their internal divisions. Sunnis are divided into three groups. One group has thrown its lot in with the insurgency in the hope of restoring the remnants of the Baath Party to power . Another group, led by people like Ghazi Al-Yawar and Adnan Pachachi, has shown genuine attachment to the democratic project and has remained in the game. A third group has hedged its bets .The Sunnis will join the political process ;everything indicates they will .Note how long it took to form a gvt.Theoretically, the Shiite-Kurdish coalition could have ignored the Sunnis since they boycotted the elections . But this did not happen . Both Shiite and Kurdish leaders went out of their ways to bring Sunnis back into the game.

The Baathists have shot their load so to speak. They are left to a pathetic campaign of homicide bombing/kidnapping /murder .This is not civil war and cannot become one so long as there is a committment to a free Iraq by the majority populations.

excon rated this answer Excellent or Above Average Answer

Question/Answer
excon asked on 05/18/05 - Jacket


Hello experts:

If I wore a jacket with the word police printed on the back, can I be arrested? No badge - no Id - just the jacket.

excon

tomder55 answered on 05/18/05:

I don't think so ;you can buy them on the net . Hats ;Tee shirts ;patches from different Police units are worn all the time. In NY the NYPD baseball caps are big sellers. I don't even think it is illegal to have a shield .

If it is used in a crime ;that's a different story .

excon rated this answer Excellent or Above Average Answer

Question/Answer
Itsdb asked on 05/18/05 - Media with impunity?

From yesterday's White House press briefing...

"Scott, you said that the retraction by Newsweek magazine of its story is a good first step. What else does the President want this American magazine to do?

MR. McCLELLAN: Well, it's what I talked about yesterday. This report, which Newsweek has now retracted and said was wrong, has had serious consequences. People did lose their lives. The image of the United States abroad has been damaged; there is lasting damage to our image because of this report. And we would encourage Newsweek to do all that they can to help repair the damage that has been done, particularly in the region.

And I think Newsweek can do that by talking about the way they got this wrong, and pointing out what the policies and practices of the United States military are when it comes to the handling of the Holy Koran. The military put in place policies and procedures to make sure that the Koran was handled -- or is handled with the utmost care and respect. And I think it would help to point that out, because some have taken this report -- those that are opposed to the United States -- some have taken this report and exploited it and used it to incite violence.

Q With respect, who made you the editor of Newsweek? Do you think it's appropriate for you, at that podium, speaking with the authority of the President of the United States, to tell an American magazine what they should print?

MR. McCLELLAN: I'm not telling them. I'm saying that we would encourage them to help --

Q You're pressuring them.

MR. McCLELLAN: No, I'm saying that we would encourage them --

Q It's not pressure?

MR. McCLELLAN: Look, this report caused serious damage to the image of the United States abroad. And Newsweek has said that they got it wrong. I think Newsweek recognizes the responsibility they have. We appreciate the step that they took by retracting the story. Now we would encourage them to move forward and do all that they can to help repair the damage that has been done by this report. And that's all I'm saying. But, no, you're absolutely right, it's not my position to get into telling people what they can and cannot report."

~~~~~~~~~~~~~~~~~~~~~~~~~~~~~~~~~~~~~~~~~~~~~~~~~~~~~~~

Newsweek publishes an unsubstantiated story that damages the credibility of the country and is used by Islamic fundamentalists to incite riots leading to deaths and threats of Jihad on the US...and the president gets chastised for it. How insane is that?

Steve

tomder55 answered on 05/18/05:

It's as if they have the same reverence to the written word of the legacy media that the fundamentalist jihadist have to the Q'ran. Drudge ran a longer segment of the Q&A session . The reporter (Liz ;I think from the NY Times) interupted him throughout ;as if he was in the hot seat,under investigation by a detective . If Newsweek wrote it ;it must be true was the assumption behind the line of questioning ;as evidenced by this Question .

Q. As far as the Newsweek article is concerned, first, how and where the story came from? And do you think somebody can investigate if it really happened at the base, and who told Newsweek? Because somebody wrote a story.

Meanwhile ;more proof that the legacy media when they are not satisfied with the content of the news has no problem with making it up on the fly .....as told by Barcepundit ..ready for this ?

Shortly after March 11, ABC had a world exclusive: a photograph of the only backpack that hadn't exploded and which had 'm[y]steriously' appeared the night after the blasts in a police station, in a room where all personal belongings that had been recovered were carried to. The problem: according to Madrid's newspaper El Mundo, it's a staged photo op:
The American TV network ABC's news division showed a false image of the backpack that did not explode on March 11, 2004, according to reporter Fernando Mgica in the Spanish daily El Mundo . According to Mgica, at a Madrid police station "the officers wanted to help the ABC reporters, but when the camera crew came, they didn't have the backpack that had contained the bomb there, so one of the officers showed them a similar backpack which was the property of another officer." Said Mugica, "I don't know whether the network knew this or simply accepted that the bag they were shown was the real one."




Itsdb rated this answer Excellent or Above Average Answer

Question/Answer
sapphire630 asked on 05/17/05 - Good or not?

I have been spending approximately 60 hours a month making over 800. scrapping metal and thinking I am doin' a good thing. Then I read that the scrap yards are sending the metal over to Asia where they could be using the metal to make weapons to use against us.
So should I no longer 'give a crap about your scrap' or keep on scrapping?

tomder55 answered on 05/18/05:

you should not presume that . the emerging nations of Asia have a growing industrial base. perhaps the metal you scraped is providing someone with a auto where in the past they needed a rickshaw.

sapphire630 rated this answer Excellent or Above Average Answer

Question/Answer
ROLCAM asked on 05/18/05 - POLITICIANS !!

Do most politicians prate about democracy?

tomder55 answered on 05/18/05:

Yes ;as do most people . Most talk of democracy and have not a clue what democracy really is . It comes as a suprise to most people that both the American system and the Parlimentary systems are forms of Representative governments and not democracy.Not all issues are put forward to us in referendum . We select representatives who we trust will act on our behalf.

ROLCAM rated this answer Excellent or Above Average Answer

Question/Answer
labman asked on 05/17/05 - Aiding the enemy in a time of war.

Why is America taking the Newsweek outrage so calmly? They couldn't wait to verify a story that badly damaged America and got people killed. Americans too will eventually die over this. They were forced to admit it wasn't true, but aren't acting like it is a big deal. Like many other lies, this one will never die. False accusations are never completely disproved. Not even Jane Fonda damaged America like this. Nobody that loves America will ever buy Newsweek again.

tomder55 answered on 05/18/05:

Watching Newsweek fabricate a story and getting caught is like seeing Mike Tyson resorting to ear-biting on his inevitable decline from greatness . The reason America is taking the Newsweek outrage so calmly is two fold . One ;much of the message is still controlled by the legacy media. Did you hear the White House Press conference about this ? You would think that it was the White House that floated the false story (as reported by Drudge).

REVENGE OF THE SITH PRESS: REPORTERS RIP BUSH SPOKESMAN OVER NEWSWEEK MESS

Mainline reporters moved into battle mode today during a White House press briefing -- as one of their own outlets continued to hemorrhage over a now retracted story!

Afghanistan's government said Tuesday that NEWSWEEK should be held responsible for damages caused by deadly anti-American demonstrations after the magazine alleged U.S. desecration of the Quran.

The White House said the United States' image abroad had suffered irreparable damage by the story.

But it was the press's turn to fight back as Bush spokesman Scott McClellan opened his briefing to questions.

[Joined in progess]

Q With respect, who made you the editor of Newsweek? Do you think it's appropriate for you, at that podium, speaking with the authority of the President of the United States, to tell an American magazine what they should print?

MR. McCLELLAN: I'm not telling them. I'm saying that we would encourage them to help --

Q You're pressuring them.

MR. McCLELLAN: No, I'm saying that we would encourage them --

Q It's not pressure?

MR. McCLELLAN: Look, this report caused serious damage to the image of the United States abroad. And Newsweek has said that they got it wrong. I think Newsweek recognizes the responsibility they have. We appreciate the step that they took by retracting the story. Now we would encourage them to move forward and do all that they can to help repair the damage that has been done by this report. And that's all I'm saying. But, no, you're absolutely right, it's not my position to get into telling people what they can and cannot report....

Q Are you asking them to write a story about how great the American military is; is that what you're saying here?

MR. McCLELLAN: Elisabeth, let me finish my sentence. Our military --

Q You've already said what you're -- I know what -- how it ends.

MR. McCLELLAN: No, I'm coming to your question, and you're not letting me have a chance to respond. But our military goes out of their way to handle the Koran with care and respect. There are policies and practices that are in place. This report was wrong. Newsweek, itself, stated that it was wrong. And so now I think it's incumbent and -- incumbent upon Newsweek to do their part to help repair the damage. And they can do that through ways that they see best, but one way that would be good would be to point out what the policies and practices are in that part of the world, because it's in that region where this report has been exploited and used to cause lasting damage to the image of the United States of America. It has had serious consequences. And so that's all I'm saying, is that we would encourage them to take steps to help repair the damage. And I think that they recognize the importance of doing that. That's all I'm saying.

Q As far as the Newsweek article is concerned, first, how and where the story came from? And do you think somebody can investigate if it really happened at the base, and who told Newsweek? Because somebody wrote a story.

MR. McCLELLAN: I think Newsweek has talked about it. They took it --



Note the last question.Obviously since Newsweek wrote it ;it must be true. What arrogance!!!

The second issue is one that I have not heard much about from the press or anyone else. In this country of great tolerance it is permissable to put a crucifix in urine ;to smear feces on an image of the Virgin Mary ,to put swastika graffitti on Jewish synagogues and cemetaries ;and all other types of outrages.Perhaps we have just become too complacent about these acts and think it natural for the rest of the world to not consider desecration a big deal. I do not recall a high degree of outrage when the Taliban used mortars to destroy the statues of Buddah in Afghanistan .There were no riots by outraged Buddhists .In fact the biggest reaction was a shaking of heads and saying 'what a shame'.

Remember when Arafat's thugs occupied the Church of the Nativity in Bethlehem ? Here was the Washington Times report from that incident :

Palestinians In Nativity Church
Used Bibles As Toilet Paper
ISpecial To Washington Times
5-15-2


BETHLEHEM, West Bank - The Palestinian gunmen holed up in the Church of the Nativity and later deported by Israel seized church stockpiles of food and "ate like greedy monsters" until the food ran out, while more than 150 civilians went hungry.

They also guzzled beer, wine and Johnnie Walker scotch that they found in priests' quarters, undeterred by the Islamic ban on drinking alcohol.

The indulgence lasted for about two weeks into the 39-day siege, when the food and drink ran out, according to an account by four Greek Orthodox priests who were trapped inside for the entire ordeal that ended Friday.

About 30 priests, monks and nuns, and more than 150 Palestinian civilians, who ran inside on April 2 to escape a gunbattle between Israelis and Palestinians, remained inside the church with the armed militants for more than five weeks.

A church helper, who gave his name as Milad, said the quantity of food consumed by the gunmen in the first 15 days should have lasted for six months.

As they feasted and boozed, Palestinian civilians subsisted on a meager diet, with barely enough for a single meal a day.

Conditions improved somewhat for the civilian refuge seekers when the governor of Bethlehem intervened and partially succeeded in reallocating food supplies, priests and others witnesses said.

Angry Orthodox priests yesterday showed two reporters about 20 empty bottles of whiskey, champagne, vodka, cognac and French wine on a kitchen shelf and on the floor of two rooms.

"They should be ashamed of themselves. They acted like animals, like greedy monsters. Come, I will show you more," said one priest, who declined to give his name.

He gestured toward empty bottles of Israeli-brewed Maccabi beer and hundreds of cigarette butts strewn on the floor.

The priest then took the reporters to see computers taken apart and a television set dismantled for use as a hiding place for weapons.

"You can see what repayment we got for 'hosting' these so-called guests," said Archbishop Ironius, another cleric, as he showed reporters the main reception hall of the Greek Orthodox Monastery.

"All the media concentrated on the Franciscan [Catholic] quarter, where little damage was done," the archbishop said. "Why? The Franciscans actually let the gunmen in, then guided the gunmen to our rooms."

Archbishop Ironius showed onlookers where the militants had broken in to the monks' quarters by smashing locked doors while, he said, the monks were praying downstairs.

"The Franciscans then blocked their own rooms' doors with iron bars," Archbishop Ironius said.

The 39-day siege ended Friday under a deal in which 13 Palestinian fighters were sent to exile in Europe and another 26 were sent to Gaza, where they were received as heroes.

While in the church, the top Palestinian gunmen slept on comfortable beds in the elegant apartment of Father Parathaious, while others rested on mattresses there and elsewhere under high-quality woolen blankets.

The civilians slept on cold tile floors in the main church downstairs.

"It's a shame, the mess they made and the way they kept the food. I'm not proud of what they did, even though as a Palestinian I sympathize with their cause of a liberated homeland," said Milad, 15.

The gunmen belong to the Al Aqsa Martyrs Brigade, part of Palestinian leader Yasser Arafat's Fatah organization.

Before Mr. Arafat visited the church yesterday, Palestinian security officials ordered a thorough removal of all debris and a total cleanup.

But prior to Mr. Arafat's arrival, the reporters were shown the room belonging to Father Vasareillon, where a key Al Aqsa leader, Nabil Abayat, was fatally shot.

While inviting the reporters inside, the priests refused to allow three young members of Mr. Abayat's extended family into the room, fearing they would insist it became a Muslim shrine to their fallen colleague.

The scene in the room was grim. Seven gunmen had sheltered there, said another young church helper, and Mr. Abayat had died when a bullet ricocheted off a radiator.

Pools of his blood had stained the bed on which he had collapsed.

Priests pointed to numerous bullet holes that appeared to have been fired from inside the room.

Wine bottles and a Johnny Walker Red Label whisky bottle lay empty on a bedside table with cooked rice splattered on the floor.

Another top gunman from the militant Abayat clan was fatally shot 10 days ago close to a well that the Palestinians had used to hide their weapons.

Another four gunmen died during the exchanges of fire and sniping, as well as a mentally retarded bell ringer who, the Israelis say, ran toward soldiers, ignoring orders to stop.

Several Israeli soldiers were seriously wounded in the exchanges of gunfire during the five-week siege.

The Orthodox priests and a number of civilians have said the gunmen created a regime of fear.

"Their word was law," said one civilian, "and they told us civilians who left the church would either be shot by the Israelis or dealt with later by the gunmen's comrades."

Even in the Roman Catholic areas of the complex there was evidence of disregard for religious norms.

Catholic priests said that some Bibles were torn up for toilet paper, and many valuable sacramental objects were removed.

"Palestinians took candelabra, icons and anything that looked like gold," said a Franciscan, the Rev. Nicholas Marquez from Mexico.

"We were told later that they gave them back."

The gunmen and civilians who emerged on Friday went through metal detectors, revealing no stolen objects.


You did not see rioting over this incident either . Perhaps if our Press was truely being honest with us they would point out this small discrepency ;and deal more accurately with the true nature of Jihadistan ;our enemies. Perhaps the press thinks that they will have such freedoms of speech when they live in Dhimmitude






labman rated this answer Excellent or Above Average Answer

Question/Answer
denberg asked on 05/17/05 - So what do you in America think of George Galloway?

A bit different to American politicians, isn't he? We've had to put up with his shit for years.

Paul,
London

tomder55 answered on 05/18/05:

His was a grandstand performance before the Coleman committee. Galloway understood his weakness on substantive issues and turned his testimony into a screed, attempting to change the ground of the debate. The Coleman hearings are about Oil For Food ;not about his views on Operation Iraqi Freedom or the neo-cons.Galloway was a loose cannon under oath and that was his biggest mistake .

The dots are going to stay connected long after Galloway's fifteen minutes of fame have passed. I think George Galloway will see his performance replayed more often than he would like. The Coleman committee has the evidence on him . He purjured himself yesterday ; His hurling insults to the Committee does not change that fact.

Just because he was able to beat the rap in the UK does not make him innocent .He's a disgusting creature .He laundered bribe money in a childrens cancer charity ...It doesn't get any lower than that !! He is toast at the end of this . I wonder what the sentence will be for this Brit MP .... to serve hard time in a US jail ?

denberg rated this answer Excellent or Above Average Answer

Question/Answer
ROLCAM asked on 05/17/05 - Who wrote this ? And about who ?

"Authority forgets a dying king,"

tomder55 answered on 05/18/05:

Lord Tennyson wrote it in ' Morte D'Arthur'.

"Ah, miserable and unkind, untrue,
Unknightly, traitor-hearted! Woe is me!
Authority forgets a dying king,
Laid widowed of the power in his eye
That bowed the will. I see thee what thou art,
For thou, the latest-left of all my knights,
In whom should meet the offices of all,
Thou wouldst betray me for the precious hilt;
Either from lust of gold, or like a girl
Valuing the giddy pleasure of the eyes.
Yet, for a man may fail in duty twice,
And the third time may prosper, get thee hence:
But, if thou spare to fling Excalibur,
I will arise and slay thee with my hands."


Arthur said that to Sir Bedivere. Why do you ask ?

ROLCAM rated this answer Excellent or Above Average Answer

Question/Answer
Itsdb asked on 05/16/05 - Hypocrisy revisited

Dean: DeLay Belongs in Jail

In comments that offended even members of his own party, Democratic National Committee Chairman Howard Dean blasted House Majority Leader Tom DeLay on Saturday as a criminal, saying that the top Republican belongs in jail.

"I think Tom DeLay ought to go back to Houston, where he can serve his jail sentence down there," Dean told Massachusetts Democrats at their state convention.

Incredibly, in the next breath the top Democrat complained about "this ugly, nasty dialogue that is coming from the right wing of the American Republican Party."

Dean's comments drew a sharp rebuke from Rep. Barney Frank, who told the Boston Globe, "That's just wrong."

"I think Howard Dean was out of line talking about DeLay," the House Democrat complained. "The man has not been indicted. I don't like him, I disagree with some of what he does, but I don't think you, in a political speech, talk about a man as a criminal or his jail sentence."

~~~~~~~~~~~~~~~~~~~~~~~~~~~~~~~~~~~~~~~~~~~~~~~~~~~~~~~

This, just after Harry Reid likely violated Senate ethics rules by smearing judicial nominee Henry Saad:

"Henry Saad would have been filibustered anyway," Mr. Reid said on the floor yesterday, about the Michigan Appeals Court judge who is nominated to the U.S. Court of Appeals for the 6th Circuit.

"All you need to do is have a member go upstairs and look at his confidential report from the FBI, and I think we would all agree that there is a problem there," Mr. Reid continued.

Republican staff members and supporters of Mr. Bush's nominees were outraged.

"Can you think of a better way to trash someone's reputation?" Sean Rushton of the conservative Committee for Justice asked after seeing a transcript of the remarks. "Say that there is bad stuff from an FBI investigation in a file somewhere and leave that hanging. This is character assassination of the lowest order and completely improper."

Republicans on Capitol Hill weren't saying much publicly, but several denounced the action privately as an "underhanded smear" or worse.

Sen. Orrin G. Hatch, Utah Republican who was chairman of the Judiciary Committee when Judge Saad faced his latest background investigation, declined to discuss the matter.

"As important as Senator Hatch thinks these comments are, he never comments on FBI reports or anything pertaining to them, and he doesn't believe anybody else should either," Hatch spokesman Peter Carr said.

Republican aides pointed to Standing Rule of the Senate 29, Section 5: "Any Senator, officer, or employee of the Senate who shall disclose the secret or confidential business or proceedings of the Senate, including the business and proceedings of the committees, subcommittees, and offices of the Senate, shall be liable, if a Senator, to suffer expulsion from the body; and if an officer or employee, to dismissal from the service of the Senate, and to punishment for contempt."

Furthermore, a "Memorandum of Understanding" covering the use of FBI background reports limits access to committee members and the nominee's home-state senators. Mr. Reid would fall into neither category."

~~~~~~~~~~~~~~~~~~~~~~~~~~~~~~~~~~~~~~~~~~~~~~~~~~~~~~~

This, from a former ethics committee member that has more connections and family members benefiting from his position (pdf file) than DeLay could ever hope to have and told school kids that Bush was "a loser."

Do these people even pay attention to what comes out of their own mouths? Why doesn't the MSM holding the feet of idiots like this to the fire like they do for say, Tom DeLay?

Steve

tomder55 answered on 05/17/05:

Robert Novak wrote in the Chicago Sun Times :On May 5, the U.S. Judicial Conference in Washington received a request from a Mike Rice of Oakland, Calif., for the financial disclosure records of U.S. Appeals Court Judge Edith Jones (5th Circuit) of Houston...
...Jones was not alone as a target, and Rice is not just a nosy citizen. He and Craig Varoga, a former aide to Senate Minority Leader Harry Reid, are partners in a California political consulting firm. Their May 5 petition requested financial information on 30 appellate judges in all but one of the country's judicial circuits, including nine widely mentioned Supreme Court possibilities. Varoga & Rice's client: NARAL Pro-Choice America.

Nobody can recall any previous mass request for such disclosures by federal judges.


It is a fishing expedition in which the Democrats hope to find something, anything to derail these nominees. This is a strong indication that the Democrats view their fight to implement the judicial filibuster as a lost cause, and are now looking for other means to keep conservative judges off the bench. It shows us that once the filibuster fight is over and won, we'll still have to keep fighting it out; the Democrats seem determined to fight us tooth and nail on the issue of judges, and that makes sense because only through the courts can the Democrats have a ghost of a chance of winning their fights.


Looks like Reid and the Dems are taking a page out of the J Edgar Hoover playbook .Since Reid got such a tame reaction ;I'm sure the next step will be to leak the contents of the FBI investigation to Newsweek. I'm sure they would be willing to put out a blurb about Saad attrbuted to an un-named senior official from the swamp. If the Republicans used confidential files and secret investigators to dig up dirt on public servants like this, we'd be hearing about the end of civil liberties and the coming police state.

Itsdb rated this answer Excellent or Above Average Answer

Question/Answer
sapphire630 asked on 05/16/05 - What sense is this....

I live by the 911th air lift base, an armory, and some other military & reserve bases that are to be closed.
Not even 2 years ago the township police were given thousands and thousands of dollars because we were considered high risk to terrorism. Now these bases are being closed down.

tomder55 answered on 05/17/05:

Since the proposed base closures generate the most news and concern, its easy to miss the broader issues and intent of Base Realignment and Closure Commission (BRAC) recommendations. The Pentagon certainly intends to save money;a projected $49 billion to $64 billion over two decades. But today reshaping the military is as important as reducing waste. With brigades of tanks stationed in Europe, overlapping domestic research facilities, underused naval stations, and duplicate training centers, Rumsfeld believes America's bases are still arrayed for the Cold War and not today's reality . "Current arrangements pretty much designed for the Cold War must give way to the new demands of war against extremists and other evolving 21st-century challenges," .

He has been in favor of an integrated miltary where the forces work together and train together . Bases that were just Navy ,or just Airforce were hit harder than others. So although Fort Monmouth's base will close ; McGuire Air Force Base in NJ will expand and combine their operations with Fort Dix .

The list contained more reshuffling than outright closure.Just like weapon systems that he has nixed ,he will meet with opposition regarding the base closures because it does have an economic impact on the areas around the bases (and concurrently there are alot of old school brass who have been fighting turf protection wars since Rummy came in and made his realignment plans known. That is why you see alot of ex-Generals on MSNBC pointing out how poorly they think the military is being run)). The BRAC, appointed by President Bush, will now review the Pentagon recommendations and has until September to make changes, but major revisions are unlikely.

There is a logic to the reshuffling that will be more evident when there is also future reshuffling of American troops overseas. Our basing heavy forces in Germany to guard beer halls makes little sense . I think a major return of troops home will occure shortly ;with lighter mobile smaller detachments (the Modular Army Brigades )being stationed overseas . We have already demonstrated that a rapid deployment can take place from inside the U.S. so long as troops are stationed by major transportation routes. Some of the bases that have been considered for closure in earlier rounds, like Fort Riley, Kan., and Fort Carson, Colo., were designated by the Pentagon last week for expansion because of large training areas and newly renovated railroad connections that allow rapid deployment. Fort Bliss in Texas will also receive troops coming home from Germany.

Just as important is the need to restructure the Reserves away from the model developed after Vietnam. Were relying on our Reserve units to be ready and able to step into combat,and other responsibilities. They need to be trained appropriately. Reserve center closings are being balanced with the creation of 125 Armed Forces Reserve Centers.I do not know how this will affects the role that guard units will play in future State emergencies .

sapphire630 rated this answer Excellent or Above Average Answer

Question/Answer
ROLCAM asked on 05/16/05 - BIG TALK !!

"Action would have to be taken."
STEPHEN HADLEY, the national security adviser, on reports that North Korea might perform a nuclear test.

Do you envisage any imminent new dangers ?

ROLCAM.

tomder55 answered on 05/16/05:

yes ;these are perilous times.during the elections the only thing that Kerry and Bush seemingly agreed on is the danger of nukes in the hands of N.Korea Iran and the terrorists they could pass them on to.

So much for nuianced foreign policy.It has been over a year since N.Korea walked away from negotiations ; Iran is making fools of the EU-3 and recently cut off their talks . Of the nations involved in the 6 nation negotiations with N.Korea there is far from a united front . Japan agrees with the US to add more pressure to get Kim Jong mentally IL back to the table . S.Korea looks like they want to go it alone . China (I can't believe they are comfortable with the idea of a loon like IL having nukes and the means to deliver them )want us to back off because they think the added pressure will push N.Korea closer to a test . But if they are that close to a test ;that means they are already past the point of no return . N.Korea we should assume is already a nuclear power .

If North Korea becomes a recognized nuclear power, other countries like Japan and South Korea would have to consider doing likewise. You can kiss non-proliferation goodby.

excon rated this answer Excellent or Above Average Answer
ROLCAM rated this answer Excellent or Above Average Answer

Question/Answer
powderpuff asked on 05/14/05 - Military base closures

Did any of you lose military bases with the latest round of closures? I live about 10 miles from an Air Guard base that I was kind of hoping would be closed. I like military bases, its just that this one is an Air base and when their fighter jets roar overhead, they are so loud you cannot even hear yourself think, truly deafening. Its a small base, with only 16 fighter jets. But they fly over 2 by 2 or 4 minimum. Some days are worst than others. I think it would have been fine to close it :)

People in the area were really hoping it would stay open, they say we could lose a lot of money if it closed. The news is, it is marked to stay open and expand. They are going to go from 16 fighter jets to 41. I wonder if everyone will be so happy once all those jets take to the air with their practice maneuvers? Yes, they even do their flights at night. And I wonder, with all the wide open spaces we have in this country, why do Air bases get stationed near communities?

tomder55 answered on 05/15/05:

We lost 2 significant ones ;one expected and one a suprise that I think they should re-examine.Fort Monmouth air base in NJ I expected ;but the biggie here is the Submarine Base in New London Connecticut. The base is homeport to 18 attack submarines and also home of the Naval Submarine School.

Rhode Island is gaining from the closure ,but I have been a fan of the home port concept. Bases like Norfolk Va. and Jacksonville Fla. are large and I do not like putting all the eggs in one basket ;too vulnerable when they are so large.

powderpuff rated this answer Excellent or Above Average Answer

Question/Answer
excon asked on 05/12/05 - Who is running the show?? The Secret Service, that's who!


Hello experts:

While Washington DC was under attack the president was on a bike ride and the Secret Service didn't tell him until he was done riding - a full 15 minutes into the attack. I say attack because, for all intents and purposes, we WERE under attack. It was only determined afterwards to have been an errant private pilot.

But, for those 15 minutes, somebody other than the elected leader, was making decisions for the country. Who was that? Were they doing it under Bush's orders? Isn't this kinda like his reading a story to some kids after he was informed about 9/11? Am I missing something here?

excon

tomder55 answered on 05/13/05:

Hereis the pertinent part of Scott McClellan's news conference abou the issue :

Q Scott, yesterday the White House was on red alert, was evacuated. The First Lady and Nancy Reagan were taken to a secure location. The Vice President was evacuated from the grounds. The Capitol building was evacuated. The continuity of government plan was initiated. And yet, the President wasn't told of yesterday's events until after he finished his bike ride, about 36 minutes after the all-clear had been sent. Is he satisfied with the fact that he wasn't notified about this?

MR. McCLELLAN: Yes. I think you just brought up a very good point -- the protocols that were in place after September 11th were followed. The President was never considered to be in danger because he was at an off-site location. The President has a tremendous amount of trust in his Secret Service detail.

The Secret Service detail that was traveling with the President was being kept apprised of the situation as it was developing. They were in close contact with officials back here at the White House. And the President appreciates the job that they do.

Q The fact that the President wasn't in danger is one aspect of this. But he's also the Commander-in-Chief. There was a military operation underway. Other people were in contact with the White House. Shouldn't the Commander-in-Chief have been notified of what was going on?

MR. McCLELLAN: John, the protocols that we put in place after September 11th were being followed. They did not require presidential authority for this situation. I think you have to look at each situation and the circumstances surrounding the situation. And that's what officials here at the White House were doing. That's what officials were doing that were with the President at the off-site location, and this was a matter of minutes when all this was happening, when the alert level was going from yellow to orange to red, and then it went back down to yellow when the plane turned away.

The plane was -- as described yesterday -- lost and accidently in the restricted airspace around the Capital region. And we appreciate the job that was done by all those who worked to make sure that the protocols that were in place were followed. That was one of the President's priorities after September 11th, was making sure that we were prepared for a situation like this. And the fact is that the protocols were followed.

Q I take it that it's not the Secret Service's duty to inform the President of national security circumstances, that that would come from somebody here at the White House. Even on a personal level, did nobody here at the White House think that calling the President to say, by the way, your wife has been evacuated from the White House, we just want to let you know everything is okay.

MR. McCLELLAN: Actually, all the protocols were followed and people were -- officials that you point out were taken to secure locations or evacuated, in some cases. I think, again, you have to look at the circumstances surrounding the situation, and it depends on the situation and the circumstance. But the Secret Service detail that was with the President was being kept apprised throughout while the situation was developing. There is always a military aide that is right with the President. That military aide was in close contact with the Situation Room here at the White House, which is overseen by the National Security Council. And --

Q Nobody thought to say, by the way, this is going on, but it's all under control?

MR. McCLELLAN: And I think it depends on each situation and the circumstances surrounding the situation when you're making those decisions.

Q Scott.

MR. McCLELLAN: Go ahead, Kelly.

Q Thank you. Isn't there --

MR. McCLELLAN: And welcome.

Q Thank you, I appreciate that. Isn't there a bit of an appearance problem, notwithstanding the President's safety was not in question, protocols were followed, that today, looking at it, he was enjoying a bike ride, and that recreation time was not considered expendable to inform him of this. Isn't there just an appearance problem?

MR. McCLELLAN: Well, I mean, John mentioned 36 minutes after the all-clear. Remember, this was a matter of minutes when all this was happening. The all-clear was given at 12:14 p.m., and it had gone down to yellow a few minutes before that, as well. So again, you have to take into account the circumstances; you have to take into account where the President is. ,b>The President was never considered to be in danger. The protocols that we put in place after September 11th, I think, worked.

This plane was warned by flares and it turned and then was escorted to an airport in the area. And the pilots were questioned. It was determined that this was an accident, that they should not have been in the area and they did not realize where they were at the time.

Q Scott, I think there's a disconnect here --

MR. McCLELLAN: Let me keep going through here and then I'll come back to the -- I'll come back to your --

Q It's a follow-up.

MR. McCLELLAN: I know, I'll come back to you.

Q But has the President even indicated that even if everything was followed that he would prefer to be notified, that if the choice is: tell the Commander-in-Chief or let him continue to exercise, that he would prefer to be informed?

MR. McCLELLAN: Again, it depends on the situation and the circumstances. And you have to take all that into account, and I think that's what people were doing here at the White House, as well as those people that were with the President.

And let me just step back from this specific question because any time you have a situation like this, we will review the procedures and protocols that were followed. And if there are any steps that need to be made to make improvements, those steps will be made. But that's not pointing to anything specific at this point. The President appreciates the job that those that were with him did in the situation like this. They were constantly being kept apprised of the situation as it was developing.

Q Is he ever -- is he on any protocol to be informed at any point? Is there a protocol that involved when he's informed?

MR. McCLELLAN: Of course, and he has been in the past, Helen.

Q Scott --

MR. McCLELLAN: Go ahead, Steve.

Q New subject, or do you want --

MR. McCLELLAN: Well, let's stay on this subject since there seems to be interest in this subject.

Now, go ahead.

Q I think there's a disconnect here because, I mean, yesterday you had more than 30,000 people who were evacuated, you had millions of people who were watching this on television, and there was a sense at some point -- it was a short window, a 15-minute window, but there was a sense of confusion among some on the streets. There was a sense of fear. And people are wondering was this not a moment for the President to exercise some leadership, some guidance during that period of time? Was this not a missed opportunity for the President to speak out and at least clarify what -- that he was informed, and what was taking place at that time? If not even during the 15-minute window, why not later in the day?

MR. McCLELLAN: The President did lead, and the President did that after September the 11th when we put the protocols in place to make sure that situations like this were addressed before it was too late. And that was the case -- that was the case in this situation. And in terms of during this time, this was a matter of minutes when this was occurring. And all the appropriate security personal and Homeland Security officials and others were acting to implement those protocols. And we commend all those that worked to follow those protocols and make sure that this situation was addressed. And it worked, in terms of the protocols.

Q Beyond the protocols here, I mean we're talking about just simply demonstrating to the American people, I understand what's taking place, we're in control of the situation, and I've been apprised of what is happening here -- because there were thousands of people involved in what was a very scary moment.

MR. McCLELLAN: Yes, and we briefed you about the circumstances of the event shortly after that. But during that time period, it's important in those minutes when this is occurring, that everybody is focused on making sure the people in the area of the threat are protected, and there are protocols in place to make sure that the people in the area of the threat are protected. Those protocols were followed. You all covered this on the coverage last night and pointed out how those protocols were followed and how jets were scrambled. This was an instance where presidential authority was not required because we had put these protocols in place after September 11th.

Q Any consideration of reexamining these protocols in light of yesterday?

MR. McCLELLAN: Like I said, I'm not pointing to anything specifically, but in any situation of this nature, there's always going to be a review to look at how things transpired. And if there are any improvements that need to be made, they will be made.
.................................
sounds like a non-issue to me .People on my staff work independent of my direct supervision all the time .

excon rated this answer Excellent or Above Average Answer
Itsdb rated this answer Excellent or Above Average Answer

Question/Answer
excon asked on 05/10/05 - Hypocricy


Hello experts:

Don't you just love it when a right winger who staunchly opposed any civil rights legislation for gays, turns out to be a fag and a cheat?

Well, you might not, but I sure do.

excon

tomder55 answered on 05/10/05:

well the hypocracy is a new angle I guess . Otherwise I see no difference between this case and Jim McGreevey;former Guv of NJ who put his boy toy up for the position of homeland security tzar of NJ ;or Barney Frank ;who's main squeeze ran a male prostitution ring out of Franks DC apartment;or any of the clergy who have been caught as child abusers . They all suck .

elgin_republicans rated this answer Excellent or Above Average Answer
excon rated this answer Excellent or Above Average Answer
Itsdb rated this answer Excellent or Above Average Answer

Question/Answer
Itsdb asked on 05/09/05 - Democrat logic...an oxymoron?

From Mario Cuomo's radio address of April 30th:

"Now, the Republicans in the Senate, instead of dealing with his litany of failures, are threatening to claim ownership of the Supreme Court and other federal courts, hoping to achieve political results on subjects like abortion, stem cells, the environment and civil rights that they can not get from the proper political bodies: the Congress and the presidency.

How will they do this? By destroying the so-called filibuster, a vital part of the 200-year-old system of checks and balances in the Senate that allows the fullest possible debate before one of the president's choices for the Supreme Court or other federal courts is allowed to take his or her place on the bench. That would be a change so undesirably destructive that it has been called the nuclear option.

The Republicans say it would assure dominance by the majority in the Senate. That sounds democratic until you remember that the Bill of Rights was adopted, as James Madison pointed out, in order to protect all Americans from what he called, the tyranny of the majority. And it sounds nearly absurd when you learn that the minority Democrats in the Senate actually represent more Americans than the majority Republicans do.

The public surveys reveal that most Americans believe there appears to be no good reason to change the Senate process after all these years. In fact, there is none."

~~~~~~~~~~~~~~~~~~~~~~~~~~~~~~~~~~~~~~~~~~~~~~~~~~~~~~~

Let me get this straight...Democrats believe "the Congress and the presidency" are the proper bodies through which to achieve political results?

More Americans are represented by Democrat senators than Republican Senators, so the Republicans don't really have a majority in the Senate?

tomder55 answered on 05/10/05:

He is right about the Senate . It owes it's existance to the Connecticut Compromise / ;but is wrong when you consider that the House of Representatives ;which is picked by population proportionality is also a Republican Majority ;and in fact has a higher majority than the Senate does.

He is also wrong about the fillibuster . As has been pointed out before on this board ;it is unprecedented (with the rare exception of the Abe Fortes nomination by Johnson which was opposed by a large bipartisan majority ;Fortes withdrew his name from consideration before a floor vote could take place) to utilize the fillibuster to block Presidential judiciary appointments.

He is also wrong in that the whole point is that the judiciary makes court decisions that are super-legislative in nature .If the laws indeed came from the Legislative Branchs of the States and the Federal Gvt. then there would be no issue.

Itsdb rated this answer Excellent or Above Average Answer

Question/Answer
Itsdb asked on 05/05/05 - Marine exonerated in mosque killing

SIGNONSANDIEGO NEWS SERVICES
9:54 a.m. May 5, 2005

CAMP PENDLETON A Camp Pendleton Marine corporal caught on videotape fatally wounding an unarmed Iraqi insurgent in a Fallouja mosque will not face charges, military officials said today.

Maj. Gen Richard F. Natonski said a review of the tape showed that the corporal, whose name has not been released, broke no rules and will not be punished for the Nov. 13, 2004 shooting.

"The actions of the Marine in question were consistent with the established rules of engagement, the law of armed conflict and the Marine's inherent right of self-defense," according to a statement released by Natonski, the commanding general of the 1st Marine Division, 1st Marine Expeditionary Force at Camp Pendleton.

Natonski ordered an investigation shortly after the shooting, which was caught on tape by embedded NBC correspondent Kevin Sites.

The tape showed the Iraqi hiding his left arm behind his head, but insurgents commonly feigned surrender or death before they attacked Marines, Natonski said.

The corporal admitted to killing three Iraqis in the mosque, "believing they posed a threat to him and his fellow Marines," according to a Marine Corps statement.

Natonski said autopsy reports, ballistics tests and interviews with 22 Marines with the 3rd Battalion, 1st Regiment who helped storm the mosque in Fallouja were part of the investigation.

~~~~~~~~~~~~~~~~~~~~~~~~~~~~~~~~~~~~~~~~~~~~~~~~~~~~~~

Ooh-Rah!

tomder55 answered on 05/06/05:

good thing they did . the armed forces can't meet their recruiting goals as it is . Imagine telling new recruits that you have to be aware that videos will be pointed at you in the heat of battle and you should not shoot at the enemy unless fired upon . The terrorists made the battle a free for all. By all rights the city of Fallujah should not be anything today but a glass desert.

Itsdb rated this answer Excellent or Above Average Answer

Question/Answer
kindj asked on 05/05/05 - How to destroy the USA

We all know Dick Lamm as the former Governor of Colorado. In that context his thoughts are particularly poignant. Last week there was an immigration-overpopulation conference in Washington, DC, filled to capacity by many of American's finest minds and leaders. A brilliant college professor named Victor Hansen Davis talked about his latest book, "Mexifornia," explaining how immigration - both legal and illegal - was destroying the entire state of California. He said it would march across the country until it destroyed all vestiges of The American Dream.

Moments later, former Colorado Governor Richard D. Lamm stood up and gave a stunning speech on how to destroy America. The audience sat spellbound as he described eight methods for the destruction of the United States. He said, "If you believe that America is too smug, too self-satisfied, too rich, then let's destroy America. It is not that hard to do. No nation in history has survived the ravages of time. Arnold Toynbee observed that all great civilizations rise and fall and that 'An autopsy of history would show that all great nations commit suicide.'"

"Here is how they do it," Lamm said:

First to destroy America, "Turn America into a bilingual or multi-lingual and bicultural country. History shows that no nation can survive the tension, conflict, and antagonism of two or more competing languages and cultures. It is a blessing for an individual to be bilingual; however, it is a curse for a society to be bilingual.

"The historical scholar Seymour Lipset put it this way: 'The histories of bilingual and bi-cultural societies that do not assimilate are histories of turmoil, tension, and tragedy.' Canada, Belgium, Malaysia, Lebanon all face crises of national existence in which minorities press for autonomy, if not independence. Pakistan and Cyprus have divided. Nigeria suppressed an ethnic rebellion. France faces difficulties with Basques, Bretons, and Corsicans."

Lamm went on:

Second, to destroy America, "Invent 'multiculturalism' and encourage immigrants to maintain their culture. I would make it an article of belief that all cultures are equal. That there are no cultural differences. I would make it an article of faith that the Black and Hispanic dropout rates are due to prejudice and discrimination by the majority. Every other explanation is out of bounds.

Third, "We could make the United States an 'Hispanic Quebec' without much effort. The key is to celebrate diversity rather than unity. As Benjamin Schwarz said in the Atlantic Monthly recently: 'The apparent success of our own multiethnic and multicultural experiment might have been achieved not by tolerance but by hegemony. Without the dominance that once dictated ethnocentrically and what it meant to be an American, we are left with only tolerance and pluralism to hold us together.'"

Lamm said, "I would encourage all immigrants to keep their own language and culture. I would replace the melting pot metaphor with the salad bowl metaphor. It is important to ensure that we have various cultural subgroups living in America reinforcing their differences rather than as Americans, emphasizing their similarities."

"Fourth, I would make our fastest growing demographic group the least educated. I would add a second underclass, unassimilated, undereducated, and antagonistic to our population. I would have this second underclass have a 50% dropout rate from high school."

"My fifth point for destroying America would be to get big foundations and business to give these efforts lots of money. I would invest in ethnic identity, and I would establish the cult of 'Victimology.' I would get all minorities to think their lack of success was the fault of the majority. I would start a grievance industry blaming all minority failure on the majority population."

"My sixth plan for America's downfall would include dual citizenship and promote divided loyalties. I would celebrate diversity over unity. I would stress differences rather than similarities. Diverse people worldwide are mostly engaged in hating each other - that is, when they are not killing each other. A diverse, peaceful, or stable society is against most historical precedent. People undervalue the unity it takes to keep a nation together. Look at the ancient Greeks. The Greeks believed that they belonged to the same race; they possessed a common language and literature; and they worshipped the same gods. All Greece took part in the Olympic games. A common enemy Persia threatened their liberty. Yet all these bonds were not strong enough to over come two factors: local patriotism and geographical conditions that nurtured political divisions. Greece fell. "E. Pluribus Unum" -- From many, one. In that historical reality, if we put the emphasis on the 'pluribus' instead of the 'Unum,' we can balkanize America as surely as Kosovo."

"Next to last, I would place all subjects off limits ~ make it taboo to talk about anything against the cult of 'diversity.' I would find a word similar to 'heretic' in the 16th century - that stopped discussion and paralyzed thinking. Words like 'racist' or 'xenophobe' halt discussion and debate."

"Having made America a bilingual/bicultural country, having established multi-culturism, having the large foundations fund the doctrine of 'Victimology,' I would next make it impossible to enforce our immigration laws. I would develop a mantra: That because immigration has been good for America, it must always be good. I would make every individual immigrant symmetric and ignore the cumulative impact of millions of them."

In the last minute of his speech, Governor Lamm wiped his brow. Profound silence followed. Finally he said, "Lastly, I would censor Victor Hanson Davis's book Mexifornia. His book is dangerous. It exposes the plan to destroy America. If you feel America deserves to be destroyed, don't read that book."

There was no applause. A chilling fear quietly rose like an ominous cloud above every attendee at the conference. Every American in that room knew that everything Lamm enumerated was proceeding methodically, quietly, darkly, yet pervasively across the United States today. Every discussion is being suppressed. Over 100 languages are ripping the foundation of our educational system and national cohesiveness. Barbaric cultures that practice female genital mutilation are growing as we celebrate 'diversity.' American jobs are vanishing into the Third World as corporations create a Third World in America - take note of California and other states - to date, ten million illegal aliens and growing fast. It is reminiscent of George Orwell's book ��." In that story, three slogans are engraved in the Ministry of Truth building: "War is peace," "Freedom is slavery," and "Ignorance is strength."

Governor Lamm walked back to his seat. It dawned on everyone at the conference that our nation and the future of this great democracy is deeply in trouble and worsening fast. If we don't get this immigration monster stopped within three years, it will rage like a California wildfire and destroy everything in its path, especially The American Dream.

tomder55 answered on 05/05/05:

Here is an opinion piece VDH wrote in January on illegal immegration

January 24, 2005
Illegal Immigration Is a Moral Issue
by Victor Davis Hanson
Tribune News Services

As President Bush's guest worker proposals slog through Congress, new reports suggest that there may be not 8 million, but almost 20 million illegal aliens in the United States, a population larger than most entire states. $400 billion in taxesalmost the current annual budget deficit are not collected due to a growing underground cash economy.

Mexico brazenly issued a survival guide for its intrepid citizens on how to cross illegally into the United States. A 2,000 mile border is porous at a time when stealthy terrorists count on such laxity to enter the United States.

The hallowed assimilationist formula has too few overt defenders these days even though measured, legal immigration, English emersion, multiracialism instead of multiculturalism, and integration have ensured that past legal immigrants from Mexico are among America's finest citizens.

The laissez-faire right still lectures on open borders as if it were a matter of robust lawful immigrationemphasizing global competitiveness that accrues from cheap labor. The minimum wage, not illegality, supposedly is its only problem: if only the self-correcting market could be set free to adjudicate wages, $2 an hour might not tempt any more from rural Mexico.

The therapeutic left will not even talk of "illegal immigration"taboo nomenclature that supposedly denotes racism. "Undocumented workers" is the politically correct terminology, even though not all aliens are working or simply misplaced their certification.

If employers count on inexpensive industrious laborers in the shadows, chauvinists envision a revolving, but still permanent unassimilated constituency to enhance their own agendas. In response to the tired rhetoric, perhaps it is better to envision illegal immigration from Mexico not as a question of divisive politics, but of collective morality. Is it ethical for the Mexican government to export annually 1 million to 2 million of its unwanted citizens to avoid long-overdue reform hoping to free itself of dissidents and earn $12 billion in subsidies from its poorest abroad? No wonder Mexico talks of the problem in terms of U.S. imperialism in lieu of its own cynicism.

Is it moral for employers to count on illegal industrious workers, usually without English or education, to undercut the wages of American citizensas if a laborer remains youthful and hale in perpetuity with no need of social entitlements when disabled or impoverished years later? No wonder employers claim that they are only providing a service to Mexico's poor.

Is it so liberal that governments must pay for those who ignore the law while citizens go without? In California, the money to incarcerate more than 14,000 felonious illegal aliens from Mexicowell over $400 millionwould fund the start-up costs of 20 university campuses like the new University of California at Merced, at a time when Americans (including many first-generation Mexican-American citizens) who are eligible for higher education cannot find access or financial support.

Is it so fair to assume that the unemployed in our midstover 10 percent of the work force in many counties of the American Southwest that are most affected by illegal immigrationcannot find entry-level work? No wonder we insist that no one can discover a citizen to mow the lawn or cook his foodas if 30 years ago our yards were weedy and we did not eat out, as if states without illegal aliens have poor landscaping and empty restaurants. Picking an illegal worker up at the local lumber yard, paying him in cash for a day of digging, and then dumping him on the curb at twilight "out of sight, out of mind"is neither liberal nor humane even if done in Santa Cruz or Carmel.

And is it equitable that laws must be sacred for most, but not for some? Do we really want a bureaucratic system near collapse from fraudulent Social Security numbers, off-the-books wages, false names, cars without registration and insurance, even as millions abroad queue up to enter our shores lawfully? Are we to tell waiting Punjabis or Filipinos to certify their education, skills and method of supporteven as we ask far less of those who break the law to cross the border from Mexico?

Who, then, is the real moralist? Is it the police officer who stops an illegal alien but cannot call immigration authorities? The contractor who knowingly accepts falsified identification and pays untaxed cash wages? The La Raza ("The Race") activist who promotes ethnic chauvinism for those to whom it will prove most deleterious? Perhaps the grandstanding Mexican consul who faults the United States for his own country's callousness?

Or is it the rest of us, who in fear of being slurred as "racists" or "nativists" often keep silentjust when candor and honesty on all sides are needed now if we are to avoid becoming an amoral apartheid society with a permanent underclass in the shadows?




...................................

In his meeting with Mexican President Vicente Fox and Canadian Prime Minister Paul Martin, President Bush said he opposes American citizens who are monitoring the border for illegal aliens.

Characterizing them as "vigilantes" President Bush said, "Im for enforcing the law in a rational way."

Huh?? The Minute Men did a hell of a job last month ! Where they sat ;no infiltration occured .Imagine if the national guard rotated deployments on the border to beef up border patrol and interjection .Not all illegals could be caught ;not all drug smuggling could be stopped ;but it would put a dent into both .

I do not mind responsible immigration . It is in the national interest .But along time ago the quit refering to a melting pot and opted for the rhetoric of a mosaic . I am not in favor of balkinization . We have seen where that leads .

Itsdb rated this answer Excellent or Above Average Answer
kindj rated this answer Excellent or Above Average Answer
purplewings rated this answer Excellent or Above Average Answer

Question/Answer
Itsdb asked on 05/02/05 - Dominionist Domination

The Left runs with a wild theory.
Stanley Kurtz

May 02, 2005, 9:44 a.m

What is the real agenda of the religious far Right? Ill tell you what it is. These nuts want to take over the federal government and suppress other religions through genocide and mass murder, rather than through proselytizing. They want to reestablish slavery. They want to reduce women to near-slavery by making them property, first of their fathers, and then of their husbands. They want to execute anyone found guilty of pre-martial, extramaritial, or homosexual sex. They want to bring back the death penalty for witchcraft.

But arent extremists like this far from political power? On the contrary, the political and religious movement called "Dominionism" has gained control of the Republican party, and taken over Congress and the White House as well. Once they take over the judiciary, the conversion of America to a theocracy will be sealed. The Dominionists are very close to achieving their goal. Once they have the courts in their hands, a willing Dominionist Republican-controlled Congress can simply extend the death penalty to witchcraft, adultery, homosexuality, and heresy. The courts will uphold all this once conservatives are in control, since Scalia himself appears to be a Dominionist.

Shocking as it seems, Dominionists have gained extensive control of the Republican party, and the apparatus of government throughout the United States. Yet Dominionists continue to operate in secrecy. It is estimated that 35 million Americans who call themselves Christian adhere to Dominionism, although most of them are unaware of the true nature of their own beliefs and goals. Dominionism has met its timetable for the complete takeover of the American government. It would be a mistake, by the way, to think of Dominionists as fundamentalist Protestants alone. Dominionism has stealthily swept over America, incorporating conservative Roman Catholics and Episcopalians within its ranks. And of course, Dominionists are allied with the neoconservative followers of the political philosopher, Leo Strauss. The quest of these neoconservatives for power and world domination is a self-conscious program of pure, unmitigated evil.

You dont believe me? Well, consider the fact that on December 24, 2001, Pat Robertson resigned his position as president of the Christian Coalition. Religious conservatives understood very well that Robertson had stepped aside to allow the new president of the United States to take his rightful place as the head of the true American Holy Christian Church. Robertson openly revealed at least a portion of his Dominionist plans on The 700 Club on May 13, 1986, when he clearly stated: We can change the government, we can change the court systems, we can change the poverty problem, we can change education...We can make a difference.

For Dominionists, possibly the single-most-important event of the last half of the 20th century occurred when Jim Jones proved that religious people would follow a leader, even to their deaths. Lest we all end up like the followers of Jim Jones, its time for Americans to take a leaf from those rare, brave souls, like George Soros. Following Soros, weve got to stand up to the Dominionist menace. There is an infection, a religious and political pathology that has corrupted our churches. Those we have trusted have embraced evil. Let us pray that Americans will go to the voting booth and finally free this country from the Republican Dominionist menace.



But They're Serious
O.K., its me again. Im back from the fever swamps of the Left, which Ive been exploring ever since I discovered a wild conspiracy theory about conservative Christians in the latest cover story of Harpers Magazine. You want political paranoia? You want guilt by association? You want flat-out looniness? Well, Joe McCarthys got nothing on the good liberal folks who are warning us about a takeover by Dominionist Christians. What you've just read is a composite I've created (often word for word) by drawing on a couple of web-sites I'll link you to in a moment. The disturbing thing is that this sort of conspiratorial nonsense is being taken seriously by real media and political players.

There is, in fact, a fringe Christian group of Dominionists or Reconstructionists, who really would like to see an American theocracy, and a return to the death penalty for blasphemy, adultery, sodomy, and witchcraft. The dystopian political program of this utterly marginal, extremist sect has absolutely no traction with anyone of significance. But that hasnt stopped conspiracy mongers on the Left from imagining a murderous Christian plot to destroy America. Ive found a number of Lefty sites that link to the following description of Dominionism at religioustolerance.org. This description includes the claim that Dominionists advocate genocide for followers of minority groups and non-conforming members of their own religion. Im not sure this is accurate, even for the minuscule number of actual Dominionists. But the disturbing thing is the way this and other Left-leaning sites use logical sleight-of-hand to tar ordinary evangelicals with the madcap musings of a few fevered Dominionists.

You can see the basic technique of the conspiracy mongers in this 1994 report on the Dominionists for Public Eye Magazine. All you have to do is quote a fringe Dominionist desperate to prove that his radical ideas are catching on. Dominionists have a long-term political strategy to establish a full-blown American theocracy based on Old Testament law. And look! Some other Christians want to participate in the political process, too. They even believe in developing a long-term political strategy! Ah ha! That must mean that, even though they are unaware of the original source of their ideas, conservative Christians are in fact under the influence of authentic Dominionists. Voila. By quoting a pathetic Dominionist extremists desperate efforts to prove his own influence, clever liberals can now argue that the ultimate goal of all conservative Christians is the re-institution of slavery, and execution for blasphemers and witches.

This theory reminds me of the poor kid who thought hed caused the great New York City blackout of 1965 because he happened to throw a rock at a transformer the moment the lights went out. Conservative Christians didnt turn to politics because they were egged on by wild-eyed Dominionists. They were goaded into defensive action by the post-sixties secularist challenge to their way of life. Christians would have taken up politics whether a silly Dominionist fringe existed or not. In fact, Dominionism itself is nothing but a hapless and hopeless response to the secular social changes of the past forty years. But the Left has decided that its in their interest to buy into the Dominionists own bogus and pathetic claims of influence and to exaggerate even those bogus claims beyond recognition.

The champion of this approach appears to be Kathryn Yurica, whose piece, The Despoiling of America, was the source for much of the account at the beginning of this piece. (Unlike religoustolerance.org, Yurica does not use the word "genocide" and does not talk about re-instituting slavery. She speaks only of extending the death penalty to things like adultery, rebelliousness, homosexuality, witchcraft, effeminateness, and heresy.) Yuricas article is so wild-eyed and strange that it would barely be worth mentioning, were Yurica not a featured speaker at a recent conference called, Examining the Real Agenda of the Religious Far Right. That conference, held this past weekend, was supported by the National Council of Churches, People for the American Way, The Nation, The Village Voice, and United Americans for Separation of Church and State. (You can read a Washington Times report on the conference here.)

I noted last week that Dominionist conspiracy theory broke into the mainstream with the latest cover story of Harpers Magazine. (Yurica herself now supplements her own account of the Dominionist conspiracy with a link to one of those Harpers articles.)

The notion that conservative Christians want to reinstitute slavery and rule by genocide is not just crazy, its downright dangerous. The most disturbing part of the Harpers cover story (the one by Chris Hedges) was the attempt to link Christian conservatives with Hitler and fascism. Once we acknowledge the similarity between conservative Christians and fascists, Hedges appears to suggest, we can confront Christian evil by setting aside the old polite rules of democracy. So wild conspiracy theories and visions of genocide are really excuses for the Left to disregard the rules of democracy and defeat conservative Christians by any means necessary.

In the wake of their big New York City conference, well see what, if anything, The Nation, The Village Voice, and People for the American Way actually do with this newly fashionable Dominionist conspiracy theory. I hope a little sunlight suffices to put a stop to these ill-advised attack on conservative Christians. I guess well soon enough learn what the real agenda of the irreligious far Left actually is.

~~~~~~~~~~~~~~~~~~~~~~~~~~~~~~~~~~~~~~~~~~~~~~~~~~~~~~~

So who is the real threat here, the left or conservative Christians?

Steve

tomder55 answered on 05/03/05:

I guess that makes me a proud member of the right wing conspiracy dominionist. They held their conference over the weekend here in NY. It was titled :Examining the Real Agenda of the Religious Far Right .

The conference description :

Most Americans outside the Bible Belt have little idea of the beliefs held by millions of fundamentalist churchgoers. We have an almost total lack of awareness of the rise of Dominionism and Christian Reconstructionism, forms of theology that advocate a biblical vision of Gods kingdom on earth. Some fundamentalists also foresee events such as The Rapture, the Times of Tribulation, Armageddon, and the Second Coming of Christ as we enter The End Days.
This conference will give rigorous attention to the worldview of Dominionism, its influence in contemporary political culture and its agenda for America. While not all Christian fundamentalists are Dominionists, Dominionisms influence is powerful and growing. Its adherents play a significant role in secretive organizations such as The Council on National Policy, which exerts a strong influence on the strategy of the religious right.


Here was the agenda of presentations :



Fundamentalism: The Fear and the Rage -- Karen Armstrong

The Rise of Dominionism in the U.S. Government -- Joan Bokaer

Millennialist and Apocalyptic Influences on Dominionism -- Chip Berlet

Learning about the Christian Right, and What in the World to Do? -- Frederick Clarkson

The Real Hidden Religious Agenda: The Theocratic States of America -- John Sugg

Is an Unholy American Theocracy Here? -- Katherine Yurica

On the Psychology and Theocracy of George W. Bush: Reflections in a Culture of Fear -- Charles Strozier

Christian Jihad -- Skipp Porteous

Jesus Plus Nothing: Elite Fundamentalism, Pragmatic Dominionism -- Jeff Sharlet

Religion and Secrecy in the Bush Administration -- Hugh Urban




The Washington Times reported on it yesterday . Speakers outlined such concepts -- others would say conspiracy theories -- as Christian reconstructionism and dominionism to a crowd that Mr. White said does "not understand the further reaches of religion."....Dominionism is the theory that the account in Genesis in which God gave man dominion over the earth has become a political teaching advocating that Christians gain and hold power. Christian reconstructionism is the theory that Christian conservatives intend to impose Old Testament law in America.

And what are these "Old Testament" laws that we want to impose on America ? Joan Bokaer, founder of TheocracyWatch.org, said :..."Tax cuts combined with increased funding for faith-based social programs and decreases in welfare spending, Ms. Bokaer said, were examples of "the theological right ... zealously setting up to establish their beliefs in all aspects of our society." .

HMMM marginal tax rates ..yeah ...I read about them in the 2nd book of Kings.

She continued her rant by comparing the FCC toughening it's decency monitoring with the Taliban. In Afghanistan under the Taliban, girls weren't permitted to attend elementary school, and large numbers of people were executed before crowds in soccer stadiums.Women were routinely beaten in the streets if any skin was exposed. In America we are asking not to let a women's breast be exposed on commercial network television during the hours our children are watching .Same thing .

Justice Janice Rogers Brown (one of the fillibustered court nominees )recently commented that There seems to have been no time since the Civil War that this country was so bitterly divided. Its not a shooting war, but it is a war, ...These are perilous times for people of faith, ...not in the sense that we are going to lose our lives, but in the sense that it will cost you something if you are a person of faith who stands up for what you believe in and say those things out loud. Her implication that faith has become a litmus test for political appointments practically dares her opposition to use it against her politically over the next two weeks.The Dems.of course are salivating waiting to bring it up during her confirmation fillibuster debate .Hostility to those of traditional faith is behind the judicial nomination fight . Kurtzs article provides indirect evidence that is the case.








Itsdb rated this answer Excellent or Above Average Answer

Question/Answer
CeeBee2 asked on 04/29/05 - During the energy portion of last night's press

conference, I was so hoping President Bush would say that he plans to go after the auto makers to produce more fuel-efficient cars (yes, they can...). I was also hoping to hear him encourage the American public to really conserve fuel this summer by finding close-to-home vacation/get-away places and to limit driving only to what needs to done (i.e., walk or bike otherwise if possible). (I believe we would be healthier and thinner too if we used our cars less often.)

What are your thoughts on oil/fuel conservation?

tomder55 answered on 04/29/05:

Bush dropped the ball on Energy last night . There are many alternatives he did not bring up. Most of his major points are valid ,but he came way short of announcing a comprehensive energy policy .Although conservation alone is not sufficient by itself to fix the problem it cannot be ignored . Energy is a national security priority and that is how it should be presented .

I direct all to Set America Free's Blue Print for U.S. Energy Security .There are practical solutions to the energy problem with technologies that exist today.

CeeBee2 rated this answer Excellent or Above Average Answer
Itsdb rated this answer Excellent or Above Average Answer
purplewings rated this answer Excellent or Above Average Answer

Question/Answer
sapphire630 asked on 04/28/05 - Just rewards

Wendy's finger story
The latest is that the woman that TRIED to sue Wendy's had a history of suing. She allegedly got the finger from someone who died. She was arrested for two counts of grand theft. One for falsely attempting to sue Wendy's; and the other, I think, was for someone else she sued.
I said a couple weeks ago that Wendy's needed to sue her.

tomder55 answered on 04/29/05:

I wonder what would happen if I ordered a spicy cajun breast ?

sapphire630 rated this answer Excellent or Above Average Answer

Question/Answer
excon asked on 04/28/05 - GOP meltdown


Hello experts:

So, what happened? They caved on the house ethics rules. Bush is going to cave on SS tonight. And, the Bolton nomination is toast.

The train looks like it derailed.

excon

tomder55 answered on 04/29/05:

I said this week that the Republicans would cave compromise on the house rules. If the Democrats want to proceed with this then I say bring it on . Let's see who else is dirty. Letting the charges levelled against Delay hang around would've been the worse thing the Republicans could do .There are important things the House needs to do ;and this phony drama is a distraction .

Bolton will be confirmed ;but if he goes down if Bolton goes down, itll be because he has not had nice things to say about the U.N.;and he will have been found guilty of being too nasty....oooooooooohhhhhh dear me !Democrat Sen. Daniel Patrick Moynihan , who was ambassador 1975-76, upon leaving the U.N. called it "a theater of the absurd, a decomposing corpse, and an insane asylum." He also said "There is no United Nations. There is an international community that occasionally can be led by the only real power left in the world -- and that's the United States -- when it suits our interests, and when we can get others to go along...."
He wasn't particularly diplomatic either but was a very effective Ambassador.

The major media is trying to portray opposition to the Bolton nomination as a rising groundswell emanating from the general public.Most of the public could not name Bolton ,and could care less .Instead, we find a variety of well-funded radical groups and the usual assortment of perpetual whiners.Here are some examples "
Citizens for Global Solutions ;the U.S. arm of theDemocratic
World Federalists
an association which advocates one-world government and global taxes - which would, of course, fall disproportionately on Americans. Hopelessly corrupt Kofi Annan and the cabal of anti-American, anti-Semitic thugocracies which make up the United Nations heartily concur with this position.

The Friends Committee on National Legislation (FCNL) is a pacifist group which has been opposed to all actions in the War on Terrorism. According to links on their website, their solution to 9/11 would have been to "stand our ground and reach out with love." FCNL even gave an award to Democrat Barbara Lee, due to her sole "no" vote on the authorization for the U.S. to go to war in Afghanistan.


Two of the groups opposing Mr. Bolton are linked directly to wealthy Bush hater George Soros. Morton Halperin , executive director of the Soros-funded Open Society Policy Institute , spoke at an anti-Bolton press conference. Halperin also serves as senior vice-president for the Center for American Progress , whose affiliate the American Progress Fund is signed on to the bash Bolton campaign. Halperin is also a former associate of CIA defector and Castro collaborator Philip Agee[Today, Agee runs a website from his home in Havana (http://www.cubalinda.com/), which uses loopholes to arrange illegal holidays to Cuba for American citizens.Agee is a strong supporter of Fidel Castro and of the Cuban Revolution.]

The "Peace Action" group opposed any action in Afghanistan and Iraq, and responded to the attacks of September 11th by preaching that Americans should seek "justice not war". Dennis Kucinich, who favored the establishment of a federal "Department of Peace" received a 100 percent rating from them.
The "Win Without War" coalition, and Moveon.org are other opposition groups (another Soros site).

It appears that those so virulently opposed to the Bolton nomination are nothing more than the usual suspects. They have been fighting against American power and American interests since the 1960s. The bottom line is it doesn't matter how much Democrats "Bork" Bolton in the next couple of weeks; the only way his nomination dies is if Republicans abandon him and defy the president.

Likewise ;the Democrats in lieu of having an agenda or any new ideas are taking on Bush's nominees for the courts ,all his appointments ;and his agenda by using obstruction .I'm not sure what I think about means testing S.S.like Bush floated last night ,but I am convinced that some kind of privatization of the system is needed.Roosevelts original Social Security proposal had voluntary contributory annuities. also .Such a system has a proven track record of success in places like Galveston Texas ;and other nations that have simular plans. But Barbara Boxer opposes it so who am I to say ?



The Democrats are putting party politics and special interests ahead of the needs of the American people.While Barbara Boxer alleges that the successful privatized retirement system of Galveston, Texas has failed , where are the ideas from the Democratic Party? Is the only solution they have to the problems we face higher taxes? If you have nothing to offer your only chance to beat the other guy is to obstruct. It is a losing strategy but it is the only one they have right now.

excon rated this answer Excellent or Above Average Answer

Question/Answer
kindj asked on 04/28/05 - Great quotes

1) Suppose you were an idiot.
And suppose you were a member of Congress.
But then I repeat myself
............Mark Twain

2) I contend that for a nation to try to tax itself into prosperity is like a man standing in a bucket and trying to lift himself up by the handle.
...........Winston Churchill

3) A government which robs Peter to pay Paul can always
depend on the support of Paul.
...........George Bernard Shaw

4) A liberal is someone who feels a great debt to his fellow man, which debt he proposes to pay off with your money.
............G Gordon Liddy

5) Democracy must be something more than two wolves
and a sheep voting on what to have for dinner.
..........James Bovard, Civil Libertarian (1994)

6) Foreign aid might be defined as a transfer of money from poor people in rich countries to rich people in poor countries.
............Douglas Casey, Classmate of Bill Clinton at Georgetown Univ.

7) Giving money and power to government is like giving
whiskey and car keys to teenage boys
.............P.J. O'Rourke, Civil Libertarian

8) Government is the great fiction, through which everybody endeavors to live at the expense of everybody else.
.........Frederic Bastiat, French Economist (1801-1850)

9) Government's view of the economy could be summed up
in a few short phrases:
If it moves, tax it If it keeps moving, regulate it.
And if it stops moving, subsidize it.
......Ronald Reagan (1986)

10) I don't make jokes. I just watch the government and
report the facts.
.............Will Rogers

11) If you think health care is expensive now, wait until you see what it costs when it's free.
.......P.J. O'Rourke

12) In general, the art of government consists of taking as much money as possible from one party of the citizens to give to the other.
.........Voltaire (1764)

13) Just because you do not take an interest in politics doesn't mean politics won't take an interest in you.
..........Pericles (430 B.C.)

14) No man's life, liberty, or property is safe while the legislature is in session.
.........Mark Twain (1866)

15) Talk is cheap .. except when Congress does it.
.........Unknown

16) The government is like a baby's alimentary canal,
with a happy appetite at one end and no responsibility
at the other.

17) The inherent vice of capitalism is the unequal sharing of the blessings. The inherent blessing of socialism is the equal sharing of misery.
........Winston Churchill

18) The only difference between a tax man and a taxidermist is that the taxidermist leaves the skin.
........Mark Twain

19) The ultimate result of shielding men from the effects of folly is to fill the world with fools.
.........Herbert Spencer, English Philosopher (1820-1903)

20) There is no distinctly native American criminal class...save Congress.
..........Mark Twain

21) What this country needs are more unemployed politicians.
Edward Langley, Artist (1928 - 1995)

22) A government big enough to give you everything you
want, is strong enough to take everything you have.
..........Thomas Jefferson


#'s 19 and 22 are my personal favorites.

DK

tomder55 answered on 04/28/05:

glad to see a Will Rogers quote in there . here are some more from him :

And the thing about my jokes is, they don't hurt anybody. You can take 'em or leave 'em - you can say they're funny or they're terrible or they're good, or whatever, but you can just pass 'em by. But with Congress, every time they make a joke, it's a law! And every time they make a law, it's a joke!


Alexander Hamilton started the U.S. Treasury with nothing, and that was the closest our country has ever been to being even.

All I know is just what I read in the papers, and that's an alibi for my ignorance.


Itsdb rated this answer Excellent or Above Average Answer
kindj rated this answer Excellent or Above Average Answer

Question/Answer
Itsdb asked on 04/26/05 - It Didn't Start With Tom DeLay

CBSNews.com

WASHINGTON, April 26, 2005

A new study shows that members of Congress have taken more than $16 million in privately financed trips over the past five years, with many of the trips sponsored by non-profit groups that are not obligated to disclose who paid the bills.

The results of the study by PoliticalMoneyLine, an Internet site that compiles campaign finance information, were first reported by USA Today.

The problems of House Majority Leader Tom DeLay, R-Texas, have placed a spotlight on congressional travel. DeLay has been accused of a spate of alleged ethical lapses, including travel that may have been paid for by a lobbyist.

Congressional rules permit privately financed travel, provided the money doesn't come from a lobbyist or the representative of a foreign interest.

But the study shows that more than half the private money spent on congressional travel since 2000 - $8.8 million - came from non-profit organizations who are not obligated to identify who may be actually paying the bills.

Widespread interest in DeLay's woes have spread bipartisan jitters through the halls of Congress. The Washington Post reports that members are racing to put their travel and campaign finance records in order in case their own activities come under scrutiny.

The newspaper also said that some members are restricting privately financed travel or even halting it altogether because of affaire DeLay.

The PoliticalMoneyLine study reviewed 5,410 trips taken by 605 members of the House and Senate. Democratic lawmakers had the edge, taking 3,025 trips, to 2,375 trips for GOP members.

The No. 1 trip-taker in dollar terms was Rep. James Sensenbrenner, R-Wis., chairman of the House Judiciary Committee. Sensenbrenner took 19 trips valued at $168,000.

In contrast, by taking 14 trips valued at $94,568.

Rep. Harold Ford Jr., D-Tenn., took the most trips - 63. But Ford's less expensive domestic jaunts only totaled $61,000.

Top travel destinations, besides the U.S., were Mexico and Israel.

~~~~~~~~~~~~~~~~~~~~~~~~~~~~~~~~~~~~~~~~~~~~~~~~~~~~~~~

Delay doesn't have anything on the Dems here, maybe that's why they're scrambling to get all their records in order. Interestingly enough, besides the Dems having the edge in numbers of trips, 14 of the top 20 dollar wise are Democrats...including our good buddy Joe Biden 10 spots ahead of Delay.

Add Bush's display of support for Delay today to the scramble by congressmen to get their travel records straight, does Delay's demise still sound so imminent?

tomder55 answered on 04/27/05:

I see that all lawmakers in Congress are scrambling to rewrite their disclosure forms to cover their asses . Attention paid to disclosure filings by Rep. Tom DeLay, R-Texas, has led congressional staffs to go over their own reports and update them or make changes, the Washington Post reported Tuesday........
The office where disclosure reports are filed is busy, the Post said, with journalists looking for discrepancies and legislative aides trying to correct anything questionable.
Jason Roe, chief of staff to Rep. Tom Feeney, R-Fla., told the Post, "Every time we go down there to check on something or refile something, we have noticed someone is going through our file."
.[Washington Times ]


Between Delay and Bolton I am reminded of the Salem Witch hunts . You got playboys and fillanderers making judgements about how Bolton treated women . When the slick one is up for UN Sec.General (God forbid) will it matter that he sexually assaulted Kathleen Willey ? That was evidently not as bad as shouting during a disagreement over policy.

The Post predicts that the "controversy over House Majority Leader Tom DeLay will trigger an ethics war" in Washington ;And to what purpose ? The more ethics rules they pass the more they are likely to break .The only benefit I see to this whole sorry affair is that while these clowns are pointing fingers and slinging mud ;they are not sitting down deciding to spend more of my money .

But there are important matters that they need to deal with . I do not think that Delay should step down ;but I think they should do what is necessary to get him a hearing with the Ethics Committee as he has requested .I think that the Republicans will capitulate compromise to reinstate some of the Committee rules just to get over this impass.

Itsdb rated this answer Excellent or Above Average Answer

Question/Answer
purplewings asked on 04/23/05 - Moussaoui tells all. Will he survive?

Hardened and only 36 years old. What punishment to do you think befits his acts? Will the French government
support him? Will he be assassinated before revealing more of what he knows?

....or should America retrain and rechannel him into our own CIA or other secret government activities?

tomder55 answered on 04/24/05:

is there a death penalty for conspiracy to commit a crime that would've merited the death penalty ? I don't know. I think all the information he gives is bogus and should be discounted. He has little intelligence value.

purplewings rated this answer Excellent or Above Average Answer

Question/Answer
ken123 asked on 04/22/05 - DeLay Info

Hi,
I was just wondering if anyone could give me a description about Tom DeLay and his role in government. I am Canadian and I am not sure exactly what the significance of being the majority leader in the House of Representatives. How important is this position in the US? Of course please follow up with why he has been caught in controversey!

Thanks so much

tomder55 answered on 04/22/05:

The Majority Leader is elected in a party caucus of the majority ruling party. His primary function, aside from being the spokesman of the majority party, is to direct the deliberations on the floor.

In the present set-up of the House, the Majority Leader is also the Chairman of the Committee on Rules. All matters relevant to the Rules of the House, specifically the calendar of bills, preparation of Order of Business and Calendar of Business are within his responsibilities.

That is where Delay's problems come from . He is accused of ethics violations .In a more reasonable time the charges against him would be investigated by an Ethics committee ,and they would if they found him in violation of House rules would sanction him .As Chair of the Rules committee he introduced some new rules governing the Ethics Committee . The Democrats did not like that so they have boycotted the Committee meetings . The committee cannot function without their participation so any hearing on Delay is stalled. That leaves the Democrats the opportunity to ;like a pack of hyiennas ;circle what they perceive as wounded prey and pile on unfounded /univestigated charges .Delay has asked that he get a chance to go in front of the Ethics Commitee to answer the charges but unless agreememt about Committee rules can be bargained then nothing will get done.
This unfortunately weakens his effectiveness (which is all that the Dems. want anyway) at ushering in the President's agenda . The strategy of the Democrats for the next 2 years will be to stall by any means necessary Republican agendas ;and to hope they can regain the Congress or Senate in 2006 .

excon rated this answer Excellent or Above Average Answer
ken123 rated this answer Excellent or Above Average Answer

Question/Answer
excon asked on 04/22/05 - Drug Dealers


Hello experts:

Some time ago, I posted several question about the pharmaceutical industry. I called their television adds and their marketing strategy nothing more than drug dealing. - in the worse sense of that word. The right wingers among you (Elliot) suggested that it was nothing more than ordinary good ol American advertising and selling.

That is true. However, I maintain that we have certain professions in this country, that don't benifit us when they "sell" their services to us. Your personal physician comes to mind. Hes not like the used car dealer who you know is looking out for his own pocketbook. Most of us assume that our doctors are looking out for OUR health - rather than THEIR own retirement. Yes, its prudent to seek a second MEDICAL opinion, but we dont shop around for doctors based upon price. And I dont want to begin.

The pharmaceutical companies encourage doctors to prescribe certain medicines. How? Gifts, thats how. TV advertising encourages US to ask for certain medicines - even though we might not know what they treat.

Nope! From my point of view, I would rather my doctor prescribe a medicine solely because I need it - not because I asked for it or the pharmaceutical companies need it to be sold.

Anything else is drug dealing - pure and simple.

Go ahead - argue with me.

excon

tomder55 answered on 04/22/05:

nothing to argue ;you said it all . Your personal physician is a drug dealer who is not looking out for you but instead using you as a chemistry experiment while they pocket gratis from the Pharmaceutical Companies. That is what you said... isn't it ?

I'm no big fan of the Pharmaceutical Companies or the FDA but.... lets face the facts . A decade ago everyone was accusing them of being so slow in the approval process that beneficial life saving drugs were not making it onto the market . The mantra then was :Those greedy Pharmaceutical Companies are withholding those needed drugs because they can't make a profit .Legislation was passed to sreamline the approval process. The FDA responding to the public pressure came up with new procedures and did just that ; new drugs in the last decade have flooded the market.

Now ; well we find out that some of the drugs that were approved did not get the safety studies that were required . Now there is a big a scandal :How could they approve unsafe drugs ??? They are beholden to the greedy Pharmaceutical Companies that want to poison us with dangerous drugs for profit is the new mantra .

Meanwhile the shark trip and fall lawyers are circling around ready to feed off both the doctor and the Pharmaceutical companies . So here we have the doctors ;they pharmaceutical companies and the lawyers all looking out for the patient. Whata f***ed up system !

excon rated this answer Excellent or Above Average Answer
purplewings rated this answer Excellent or Above Average Answer

Question/Answer
tarot10 asked on 04/19/05 - Prediction of new Pope

Does anyone understand how some people were able to predict who was going to get elected pope and even what name he was going to choose to go by?

The television station I was watching mentioned that authors of books guessed both, but I would like to understand how they came about that conclusion.

Does anyone have any clues?

tomder55 answered on 04/20/05:

people gotta get a life .Betting on the selection of the Pope like it is some horse race is lame .

It was predictable that Cardinal Ratzinger would be selected because of his relationship with Pope JPII and the fact that he was Dean of the College of Cardinals .If the balloting stalemated ,and went on for more days the "odds" would decrease because that meant that he would not have the solid support of the Cardinals needed . If that happened it would've been more likely that a Pope would be selected from the third world . Pope JPII had many years to install Cardinals who shared his vision of the church . He must've made it clear that he favored Ratinger as his replacement.

ETWolverine rated this answer Excellent or Above Average Answer
tarot10 rated this answer Excellent or Above Average Answer

Question/Answer
ETWolverine asked on 04/19/05 - Some Background Info on Pope Benedict XVI

From Wikipedea:

Joseph Cardinal Ratzinger (b. April 16, 1927) is a Cardinal Bishop of the Roman Catholic Church. In 1981 Cardinal Ratzinger was appointed prefect of the Congregation for the Doctrine of the Faith by Pope John Paul II, made a Cardinal Bishop of the see of Velletri-Segni in 1993, and was elected Dean of the College of Cardinals in 2002, becoming titular bishop of Ostia.

Born in Marktl am Inn, in Bavaria, Germany, Ratzinger entered a preparatory seminary in 1939. In 1943, at the age of 16 he was, along with the rest of his class, drafted into the Flak or anti-aircraft corps. He went into basic training for the Wehrmacht infantry in November of 1944. In 1945 he was interned in a POW camp as a German soldier. By June he was released, and he and his brother (Georg) reentered the seminary. On June 29, 1951, he and his brother were ordained by Cardinal Faulhaber of Munich. His dissertation (1953) was on Saint Augustine, his Habilitationsschrift (second dissertation) on Saint Bonaventure.

Ratzinger was a professor at the University of Bonn from 1959 until 1963, when he moved to the University of Muenster. In 1966, he took a chair in dogmatic theology at the University of Tbingen, where he was a colleague of Hans Kng but was confirmed in his traditionalist views by the liberal atmosphere of Tbingen and the Marixist leanings of the student movement of the 1960s. In 1969 he returned to Bavaria, to the University of Regensburg.

At the Second Vatican Council (1962 1965), Ratzinger served as a peritus or chief theological expert, to Cardinal Joseph Frings of Cologne, Germany.

In 1972, he founded the theological journal Communio with Hans Urs von Balthasar, Henri de Lubac and others. Communio, now published in German, English, and Spanish editions, has become one of the most important journals of Catholic thought. In March 1977 Ratzinger was named archbishop of Munich and Freising and in the consistory that June was named a Cardinal by Pope Paul VI.

On November 25, 1981 Pope John Paul II named Ratzinger prefect of the Congregation for the Doctrine of the Faith, formerly known as the Holy Office of the Inquisition, which was renamed in 1908 by Pope Pius X. He resigned the Munich archdiocese in early 1982, became cardinal-bishop of Velletri-Segni in 1993, vice-dean of the College of Cardinals in 1998, and was elected Dean in 2002. In office, Ratzinger usually takes very conservative views on topics such as birth control, inter-religious dialogue, and ecumenism.

On September 30, 2003, Ratzinger's statement, "We should pray for the Pope", was published by the German weekly Bunte, and subsequently, the quote made headlines worldwide, raising questions about the Pope's health and fueling speculation that the Pope was nearing death.

On January 2, 2005, Time quoted unnamed Vatican sources as saying that Ratzinger was a frontrunner to succeed John Paul II should the Pope die or become too ill to continue as Pontiff. His see, Cardinal-Bishop of Istia, has traditionally been an antechamber to the Papal seat.

-------------

And a snippet from WHO2:

In the U.S. election of 2004 Ratzinger caused a stir by writing a memo to Cardinal Theodore McCarrick of Washington suggesting that clergy deny communion to supporters of abortion rights. The memo was made public and was widely perceived as a veiled attack on Catholic candidate John Kerry.

----------

I found that last paragraph most interesting. I like him already.

I'm not sure what blessing is said for the election of a new Pope, but all of my Catholic collegues have my congradulations, and my hopes that Pope Benedict XVI is as successesful and influential for good causes and beliefs as Pope John Paul II was.

Habeus Papum

Comments are appreciated.

Elliot

tomder55 answered on 04/19/05:

The words broke like a thunderclap inside St. Peter's Basilica on Monday. Cardinal Joseph Ratzinger, addressing the world's cardinals just hours before they sequestered themselves to choose the next leader of the world's 1 billion Catholics, decided to define this conclave.

"We are moving toward a dictatorship of relativism which does not recognize anything as for certain and which has as its highest goal one's own ego and one's own desires."

The modern world, Ratzinger insisted, jumped "from one extreme to the other: from Marxism to liberalism, even to libertinism; from collectivism to radical individualism; from atheism to a vague religious mysticism; from agnosticism to syncretism."

Those are fighting words. They guaranteed that Ratzinger, who was Pope John Paul II's enforcer of orthodoxy, will set the church's course . Decades from now, many conservative Catholics will see the war against the "dictatorship of relativism" as their central mission. It's not a line you forget.

http://www.chron.com/cs/CDA/printstory.mpl/editorial/outlook/3140951


The test for me will be what he does about Cardinal Law .I think it was a disgrace that Law celebrated one of the Masses at the Vatican last week . If Pope Benedict beleives what he says then he will impose a no tollerance policy on predatory clerics .

ETWolverine rated this answer Excellent or Above Average Answer

Question/Answer
excon asked on 04/15/05 - Time for TRUTH - and it's on my side


Hello experts:

OK, its time to stop all the bull and speak the truth. Here is some truth the right wing would rather not hear.

In the past three years, the US Senate has confirmed 205 of Bushs nominations to the federal bench. Just 10 nominees have been held up by dems.

Ninety-five percent of federal court seats are filled, making for the lowest vacancy rate in 13 years.

In contrast, when the Republicans controlled the Senate, as many as 50 Clinton nominations were held up.

Lets look at just one of these Christian nominees being held up by the dems. William G Meyers, a 9th Circuit Court nominee has never worked as a judge before or participated in a jury trial. Hes a lobbyist for the mining industry and known mainly for his strident private opinions.

In Myers words, the Supreme Courts Griswold v. Connecticut and Roe v. Wade decisions were motivated by the personal moral values of the justices... He once likened the federal land regulations to King Georges tyrannical rule and called the California Desert Protection Act an example of legislative hubris. He compared the Interior Depart introduction of wolves in Yellowstone to Englands requirement that colonists in America shelter its soldiers..

Is this a man qualified to rule fairly and impartially?

excon

tomder55 answered on 04/16/05:

Questions about his qualifications are legit debate; as well as questions about his lobbiest activities and his positions on mining and land use ,and all the other concerns that the environmentalists have . Debate them and have an up or down vote like all nominees are entitled to.

In Myer's case ,he was approved by the Senate Judiciary Commitee on March 7 of this year .In other words he has already been vetted by the proper commitee Only the phony filibuster prevents his seating.
I also note that you only mention the most controversial candidate. What is the reason for the others ? Pure politics ;not qualifications ;but a so called litmus test on positions; or as that idiot Sen. Biden keeps saying in these hearings ..."It's your judgeeemeeennt".

Elliot is right;almost all the confirmations are lower court and it is the appellate judges being blocked. That line that the Dems. and their press supporters are using about conformations intentionally misses the point.

excon rated this answer Excellent or Above Average Answer

Question/Answer
excon asked on 04/14/05 - Justice DeLayed - but not denied


Hello experts:

Blows me away. DeLay doesn't get it. Since he's so connected to God, you'd think he's get some divine inspiration. But nahhh - he's toast.

I'll give you 2 to 1 that he loses his majority post by June 1. Do you think he'll make it through the summer? I know, Elliot, you wish he would, but fortunately, the rest of us have him figured out.

excon

tomder55 answered on 04/15/05:

I examined more of the details yesterday .
The charges against him appear to be like I said almost Standard Operating Procedures in Congress. He took junket trips ;and he employed his wife and daughter are the two biggies .Am I missing something here ? If they were doing legitimate work then Congressional rules allows it. The only other thing I have heard is that a prosecutor in Texas indicted some of his friends. Is that guilt by association ?
By all means ;if he did something wrong then he should step down .But he has asked for an Ethics Committee investigation and he should be granted one .Until then he has the right of presumptive innocence :just like Harry Reid ;Nancy Pelosi ;Hillary Clinton ;and Chuckie Shumer receive .

ETWolverine rated this answer Excellent or Above Average Answer
excon rated this answer Excellent or Above Average Answer
Itsdb rated this answer Excellent or Above Average Answer

Question/Answer
excon asked on 04/14/05 - Justice DeLayed - but not denied


Hello experts:

Blows me away. DeLay doesn't get it. Since he's so connected to God, you'd think he's get some divine inspiration. But nahhh - he's toast.

I'll give you 2 to 1 that he loses his majority post by June 1. Do you think he'll make it through the summer? I know, Elliot, you wish he would, but fortunately, the rest of us have him figured out.

excon

tomder55 answered on 04/14/05:

Last week I thought for sure he was gone ;but it looks like he might just hang in there .His case reminds me of the Ethics Committee investigation that brought down Dem. Speaker of the House Jim Wright(also from Texas )in 1989 ;and simular circumstances (he got his wife a job and perks to get around contribution limits).I do not really understand the travelling charges . To me a Congress person taking junkets is SOP .

The AP has come out with a list today of all of the esteeemed members of our Representative Branch who have put family members on the books .They are besides Delay :


Sen. Joe Lieberman.Dem.-Conn
Rep. Dave Reichert, R-Was
Rep. Fortney "Pete" Stark D-Calif
Rep. Jerry Lewis, R-Calif
Rep. Bart Stupak, D-Mich
Rep. Bob Ney, R-Ohio
Rep. Jim Costa, D-Calif.
Rep. Chris Cannon, R-Utah
Rep. Lincoln Davis, D-Tenn
Rep. Louie Gohmert, R-Texas
Rep. Tim Bishop D -N.Y.
Rep. Dana Rohrabacher R- Calif.

I guess they forgot to add Socialist Rep. Bernard Sanders from Vermont who also put his family to work

I will not hold my breath waiting for the MSM to call for their removal .


I have no problem with his comments about judges ,but the ethics charges are troubling .He has offered to go in front of the Ethics Committee to clear his name . So far that has been denied .I think he should be given the chance to defend himself .But the Committee on Standards of Official Conduct reprimanded DeLay on three different counts last fall No matter what he is weakened ;damaged goods .

This is not a good situation .It can potentially split the Republicans . Some have come out in strong defense of him ,and some like Sen. Santorum have called for him to come clean .As majority leader it is his job to push through the President's agenda. I question his effectiveness as damaged goods. It may be better if he were to at least step down from his Majority leadership while he defends himself .



excon rated this answer Excellent or Above Average Answer
Itsdb rated this answer Excellent or Above Average Answer

Question/Answer
Choux asked on 04/09/05 - Vicious Attacks on the Judicial Branch

I'm still alarmed by the vicious attacks on the Judicial Branch of the Federal Government by the likes of Tom Delay.....

Comments like they are not answerable to the people for their decisions. OF COURSE THEY AREN'T ANSWERABLE TO THE PEOPLE!!!!!! Checks and balances on the Congress and Executive Branch who are elected by the people.

The redical religious right is a threat to the Constitution of the United States of America. There is no doubt in my mind that they want to "overthrow" the government. We the decent people must protect our Constitution.

Not only do they want to take away the function of the court system which is TO **IMPARTIALLY** JUDGE THE LAWS THAT CONGRESS PASSES AND INTERPRET THE CONSTITUTIONALITY OF JUDICIAL DECISIONS AND COURT CASES they want to make America a Christian theocracy.

tomder55 answered on 04/10/05:

If we are to be honest about it ;this stuff started with Roe v Wade. Why ? In 'The Right Nation',John Micklethwait and Adrian Wooldridge say "European countries liberalized abortion through legislation and, occasionally, referenda. This gave legalization the legitimacy of majority support, and allowed countries to hedge the practice with all sorts of qualifications." But here in the U.S. the Supreme Court took the matter out of politicians hands and imposed it on us . "By going down the legislative road...the Europeans managed to neutralize the debate; by relying on the hammer blow of a Supreme Court decision, the Americans institutionalized it."

When the Supreme Court did this ,it not only bypassed the necessaty political debate needed to make such a radical change but it also continued a trend that is arguably as long as the 'Marbury v. Madison' decision;judicial usurpation of power; whether judges should be deciding such questions in the first place.Before that ;no pro-life movement;no *Religious Right* that you so fear.

Had abortion been left to the normal legislative course there still would probably be some form of it availables in the States without the 32 years of acrimony .Because of this we now have *conservatives* fearing *radical judges * who invent rights on a whim;and *liberals * fearing *radical judges* who would take those invested rights away.It's amazing that any judge gets approved.Defining the popular will on abortion, same-sex marriage, and the death penalty was properly the duty of the U.S. Congress and state legislators. But for several decades now, American politicians have shirked that duty. Congress has also ducked its constitutional obligation to reign in a renegade judiciary. The result: an unchecked court system

The Courts have continued on this path in the meantime.So it should not be a suprise that a showdown is approaching .Both sides of the spectrum think they are engaged in a cultural war(lets be honest about that). Since the courts now reign supreme in this country both sides feel they need to control the courts. But the bigger issue is the threat to American self-government is not "right-wing" or "left-wing" judges but the imperial judiciary itself. This imperial judiciary issue, cant be conceded, for when it is, self-governence is lost.If you call it 'over throwing the govenment ' fine . I call it restoring the Proper balance to a system now out of balance.

Choux rated this answer Excellent or Above Average Answer
ETWolverine rated this answer Excellent or Above Average Answer
excon rated this answer Excellent or Above Average Answer
Itsdb rated this answer Excellent or Above Average Answer

Question/Answer
tarot10 asked on 04/09/05 - In a culture where so much emphasis is placed on beauty

I would like your comments as to why a man would prefer a woman, Camilla, who looks so unattractive over Princess Diana who was so well liked by people all over the world and more attractive in more ways than one. In a world where so much emphasis is put on looks.

I'm would be interested in knowing what women who think that attractive women have it made in life.

tomder55 answered on 04/09/05:

Dianas open and informal style was in conflict with the more rigid traditions of the British royal family. And gnarly Charlie is about as stiff as they come. There was a conflict between Dianas desire for freedom and a more relaxed approach to life with her husbands sense of what wassuitable for a princess.
Camille on the other hand has been a concubine ;and she will hold the title of Consort very well.

tarot10 rated this answer Excellent or Above Average Answer

Question/Answer
powderpuff asked on 04/09/05 - Protesting in Iraq

What do you think of the tens of thousands of protesters in Iraq that are demanding that the US armed forces leave?

Do they think they can succeed at building their country and controlling the insurgency better without US involvement? Why do you think they want the US to leave?

tomder55 answered on 04/09/05:

just some good old nationalism at work . well that and the cleric Moqtada al-Sadr called for his followers to join the protest .al-Sadr was the cleric who's Mahdi Army twice fought coalition forces ;and got a good spanking each time.

The reports are saying 10s of thousands ;but how does that compare to the 8 million + who voted in January ? American's are just as anxious to get to the day when Iraqis can self govern. But it would be irresponsible to leave now . They have shown a desire to govern their own affairs ;and enough nationalism ;and their newly elected legislature have acted sensibly to demonstrate that the predicitons of civil war were probably unfounded.

Looking at this demonstration (with the burning of Saddam in effigy btw)and making any conclusions from it would be like looking a the street demonstrations during the Republican Convention in September and concluding that Kerry would win.

powderpuff rated this answer Excellent or Above Average Answer

Question/Answer
powderpuff asked on 04/08/05 - Second term failure?

What do you think? Will President Bush do whatever he wants, regardless of consequenses, since he is in his second term?

Of the 15 previous presidents who have been elected and then re-elected, not one had a more successful second term than his first; and for almost half of those (7), their second terms were mired in corruption and controversy.

Will President Bush complete a successful second term, or will he fail?

tomder55 answered on 04/08/05:


Both Wilson and Roosevelt successfully executed major wars in their later terms (Wilson 2nd ;Roosevelt 3rd. ) Wilson however was not able to get approval of the League of Nations. Roosevelt accomplished most of his New Deal in his first term. It is generally agreed that his 2nd was his weakest .Eisenhowers second term was not marred by major mistakes but no real initiative was achieved either . Eisenhowers record of peace and prosperity in his 2nd term is enviable to his successors.Nixon's of course was a truncated disaster .Reagan achieved major legislative successes of tax reform, increased defense spending, and a more confrontational approach to the Cold War in his 2nd term .Reagans challenge to Gorbachev to tear down the Berlin Wall, the INS arms treaty, Reagans trip to the Kremlin all happened in his 2nd term. Only the Iran -Contra scandal marred his 2nd term. Was that worse than the Marine barracks bombing and the subsequent retreat from Lebanon in his first term? Clinton; God knows what his agenda was anyway. I think his 2nd term a disaster ;but his 1st term was not that great either. He did manage to work with Congress on needed Welfare Reform however and I did agree with his troop deployment to Kosovo .

It appears that of all the 2nd termers in the 20th Century Ike had the smoothest run ,but he also took the safest path .I think Bush understands more than most the dangers because he was witness to his father's weaknesses. I do not see as yet any major scandal on the horizon .If he took a safe course I'm sure that he could sail through his 2nd term .

But ;Bush is a risk taker and actually believes the things he says . He thinks by his election he has been given a mandate to advance his agenda ;and that agenda is ambitious .

On his side he has a majority in Congress to work with ;an advantage that Dwight Eisenhower, Richard Nixon, Ronald Reagan, and Bill Clinton did not have. He has replaced old Cabinet dead wood with those willing to advance his agenda(most of the other 2nd termers were not able to fill open positions with devote loyalists ;Bush had a bigger problem with some of his first term cabinet appointees who came form the ranks of career buroecrats ).

His promotion od democracy abroad is off and running and has had some early successes. His domestic agenda is a little more problematic .If he doesn't get meaningful S.S. reform in the books by his 2nd year then it will not happen .He also has to be very careful to monitor the economy . He led us out of recession but he must continue to work for economic expansion ;and for God's sake get a control on Federal Spending !!! I also wish he would pay a little more attention to the border problem. It will not hurt him ;but to me the whole success the greater WOT depends on it .I also think he has to get his judicial appointments passed at all cost. On a personal preference I wish he would initiate a comprehensive energy policy . That is just as much a national security issue as the WOT .

Working against him is the most stubborn minority party I have ever seen . They have gone from being a classic minority party to an outright obstructionist party .But Bush can be a pit-bull .His history shows that if he wants to get something done ;it gets done.

ETWolverine rated this answer Excellent or Above Average Answer
powderpuff rated this answer Excellent or Above Average Answer

Question/Answer
excon asked on 04/07/05 - He really is stupid!


Hello experts:

I didn't graduate from Yale. Anybody who did is usually held in high esteem by me. And, I have tried to give him every benefit of the doubt. But, I have determined, after watching him intensely, that he really is stupid.

What did he do this time? Repudiated our debt!

It's a good thing the rest of the world also knows he's stupid. Otherwise our economy would be in the crapper today.

No? Tell me why.

excon

tomder55 answered on 04/07/05:

I take it you are talking about Bush demonstrating that the trust fund is sitting on a bunch of IOUs in the form of T bills ? He is right and the NY Times is being more than a little disengeuous in saying those bonds are backed by the full faith and credit of the U.S. Government. You know and I know that is a house of straw .

The point is this: that is ALL they are backed up by. No assets to be sold. No shares in productive enterprises to pay dividends. No nothing except a promise that future generations can be taxed to make good on the IOUs .

The SSA has always backed them up. It was easy to do so when the worker to retiree ratio was 6:1 . That aint the fact anymore. Neither is it true that you will get the payback you think .Congress has raised taxes ;increased dates of recovery of benefits .True bond holders get their money back ,but you know that by stuffing the Trust fund with TBills to pay general revenues they are plundering it .

Unlike the past several Presidents, George W. Bush is attempting to address this issue he inherited. All those who trash GWB for taking this issue on, is forgetting that this problem has been ignored since the 1950's

ETWolverine rated this answer Excellent or Above Average Answer
excon rated this answer Excellent or Above Average Answer
labman rated this answer Excellent or Above Average Answer
purplewings rated this answer Excellent or Above Average Answer

Question/Answer
Choux asked on 04/05/05 - Soldiers Returning from Iraq

Sunday, someone on TV stated the one out of four soldiers returning from Iraq combat had symptoms of mental illness. What is this about? What kind of symptoms of mental illness?? What percentage of soldiers in previous say 20th Century wars, had "battle fatigue".

tomder55 answered on 04/06/05:

From George Carlin :

I don't like words that hide the truth. I don't words that conceal reality. I don't like euphemisms, or euphemistic language. And American English is loaded with euphemisms. Cause Americans have a lot of trouble dealing with reality. Americans have trouble facing the truth, so they invent the kind of a soft language to protest themselves from it, and it gets worse with every generation. For some reason, it just keeps getting worse. I'll give you an example of that. There's a condition in combat. Most people know about it. It's when a fighting person's nervous system has been stressed to it's absolute peak and maximum. Can't take anymore input. The nervous system has either snapped or is about to snap. In the first world war, that condition was called shell shock. Simple, honest, direct language. Two syllables, shell shock. Almost sounds like the guns themselves. That was seventy years ago. Then a whole generation went by and the second world war came along and very same combat condition was called battle fatigue. Four syllables now. Takes a little longer to say. Doesn't seem to hurt as much. Fatigue is a nicer word than shock. Shell shock! Battle fatigue. Then we had the war in Korea, 1950. Madison avenue was riding high by that time, and the very same combat condition was called operational exhaustion. Hey, were up to eight syllables now! And the humanity has been squeezed completely out of the phrase. It's totally sterile now. Operational exhaustion. Sounds like something that might happen to your car. Then of course, came the war in Viet Nam, which has only been over for about sixteen or seventeen years, and thanks to the lies and deceits surrounding that war, I guess it's no surprise that the very same condition was called post-traumatic stress disorder. Still eight syllables, but we've added a hyphen! And the pain is completely buried under jargon. Post-traumatic stress disorder. I'll bet you if we'd of still been calling it shell shock, some of those Viet Nam veterans might have gotten the attention they needed at the time. I'll betcha. I'll betcha.

Choux rated this answer Excellent or Above Average Answer
excon rated this answer Excellent or Above Average Answer

Question/Answer
powderpuff asked on 04/01/05 - patterns

Is President Bush creating a pattern for making decisions based on inadequate, incomplete, and false information? First he led the country into war with Iraq based on false and incomplete information. Since then he signed law passed by his congress based on false and incomplete information (the Terri Schiavo case).

Are there other decisions being made with equally inadequate knowledge of facts?

I'm not trying to bash Bush. I voted for him. I just need to be careful as a voter. The decisions our President is making have profound and lasting effects on all of us, including future generations.

Do you think this could be a problem?

tomder55 answered on 04/01/05:

I am still undecided about the WMD . Something just doesn't ring true when I am told that the whole world's intelligence networks all got it wrong. Nobody ;I repeat NOBODY ;not one gvt.s intelligence agency stepped up and said at the time that the basic facts were wrong. Germany warned Bush that Saddam was prepared to use them at our advancing troops. President Mubarak of Egypt personally told General Franks that Saddam had the weapons ready to deploy on the front lines.

We are told in the report that there was no fresh information since 1998 . Well in 1998 ;the UN was reporting that Saddam had massive stockpiles still not destroyed ;and we had all the Democrats from Clinton to Kerry warning us of the dangers of Saddam's stockpile .


Now we are told that the most reliable of the agents in Iraq was a drunk who made up info. about mobile labs ,and incredibly Saddam had this fleet of trucks like Powell accurately described moving around the country so as not to be detected ;and the reason for them was ....now get this .....to fill up weather ballons !

Where did all his weapons go ???? The new conventional wisdom says that he destroyed them without telling anyone. uhh huuhh . More likely they were moved out of Iraq prior to the invasion .Investigations have proven that he retained the capability to produce more stockpiles in short order .

Anyway it is clear that we are uncertain as to what actually happened ;and that in itself constitutes a failure of intelligence ;but why then did George Tenet get a medal???? Sorry to say a piece of the puzzle is still missing in my book .

Itsdb rated this answer Excellent or Above Average Answer
powderpuff rated this answer Excellent or Above Average Answer
purplewings rated this answer Excellent or Above Average Answer

Question/Answer
Itsdb asked on 03/31/05 - Another Wolfowitz followup

Wolfowitz set for World Bank nod
Thursday, March 31, 2005 Posted: 1011 GMT (1811 HKT)

Bush says Wolfowitz has "good experience" to lead the World Bank.

BRUSSELS, Belgium -- The World Bank board is expected to approve Paul Wolfowitz as its leader one day after the deputy defense secretary received the support of the European Union.

Wolfowitz met with EU officials Wednesday in Brussels, where they decided to back him as president despite reservations about the unillaterist goals they associate with the White House.

"There are no objections of EU countries" to Wolfowitz, Belgian Development Aid minister Armand De Decker told The Associated Press after the meeting.

In an unusual move, EU officials had asked one of the Bush administration's most hawkish figures to come to Europe to deliver his views on development issues after his nomination sparked controversy around the world's capitals.

Many leaders said they had trouble reconciling Wolfowitz's role in the Iraq war with the needs of the developing world, and described him as having a reputation for unilateralism and lack of development experience.

They also questioned his credentials and management style.

But after hearing him speak, European Commissioner Olli Rehn said he "was satisfied with everything he heard from Mr Wolfowitz concerning free trade and also on poverty reduction and development policy," a Commission spokeswoman told a news conference.

The 184-nation World Bank board is due to choose a successor to veteran president James Wolfensohn on Thursday in Washington, after U.S. President George W. Bush nominated him for the top post.

The 61-year-old Wolfowitz, who was the main architect of the war in Iraq, was keen to appease the EU after meeting with development and finance ministers in Brussels.

Talking to reporters after the meeting, Wolfowitz defended himself as a man committed to helping the world's poor, saying he believed deeply in the work of the Washington-based bank.

"Helping people lift themselves out of poverty is truly a noble mission," the former U.S. ambassador to Indonesia said.

Before Wednesday, only Germany, Britain and Italy had publicly backed him.

Wolfowitz appeared at Wednesday's news conference with Luxembourg Prime Minister Jean-Claude Juncker, whose country holds the EU presidency.

Asked whether he understood that his reputation as a neo-conservative worried some members of the EU, Wolfowitz replied: "Of course I do."

"I hope as people get to know me better that I really do believe deeply in the mission of the bank," he said.

"It's important to emphasize that President Bush believes deeply in the mission of the bank ... he believes deeply in poverty reduction," he added.

The World Bank's mission is to fight poverty and boost the living standards of people living in poor nations.

*******************************************************

So what's going on here? Why does Europe keep having these changes of heart toward the Bush administration? It was only last week that Le Monde called his nomination a "new manifestation of America's arrogance."

tomder55 answered on 04/01/05:

they must be thinking about the Robert S. McNamara 's play book .1st a member of the Def. Dept. during an *unpopular war* ;then redeeming oneself by being the director of the World Bank and being hailed in developing countries as a great Bank president who increases aid to stratospheric levels ;and finally having an epiphany and pronouncing your mea culpas to the world.
There may be a grain of truth in this ;he is still officially a registered Democrat.

Itsdb rated this answer Excellent or Above Average Answer

Question/Answer
Choux asked on 03/30/05 - March 4, 1865

I ran aross this speech when I was looking up a quote for an answer on another Board.

A question or so ago, we talked about threats to democracy. Here are some quotes from Abraham Lincoln's Second Inaugural Address.

Fellow-Countrymen:-".....Now at the expiration of four years, during which public declarations have been constantly called forth in every point and phase of the great contest which still absorbs the attention and engrosses the energies of the nation, little that is new could be presented. The progress of our arms, upon whch all else cheifly depends, is as well known to the public and to myself, and it is, I turst, reasonably satisfactory and encouraging to all. With high hope for the future, no prediction in regard to it is ventured. .......parties deprecated war, but one of them would make war rather than let the nation survive, and the other would accept war rather than let it perish, and the war came.

One eighth of the whole population was colored slaves, not distributed generally over the Union, but localized in the southern part of it. ...All knew that this interest was somehow the cause of the war. ...Both read the same Bible and pray to the same But, and each invokes His aid against the other. It may seem strange that any men should dare to ask a just God's assistance in wringing their bread from the seeat of other men's faces, but let us judge not, that we be ot judged. The prayers of both could not be answered....With malice toward none, with charity for all, with firmness in the right as God gives up to see the right, let us strive on to finish the work we are in, to bind up the nation's wounds, to care for him who shall have borne the battle and for his widow and his orphan, to do all which may achieve and cherish a just and lasting peace among ourselves and with all nations."

Any comments??

tomder55 answered on 03/31/05:

The issue of slavery was a festering problem that vexed the founders. They created elaborate compromises to patch together a nation with divergent interests ,and throught the early history the combined efforts of Congressional compromises kept the fragile union together without addressing the core issue that kept the country from truely being united. The issue was an intractable one ,and although it can be argued that if they had hung together a little longer it is possible that the South would've eventually abandoned the slave trade as England had already done. What really set the country to the point of no return was 2 things . 1 ;the fear from both sides that the other would get the upper hand in Congress when the new territories became states ;and 2. The Dred Scott decision (typical court excess even then). The pro-slavery judges wanted their intentions to be obvious. The court held that the Missouri Compromise of 1820, which restricted slavery in the territories, was unconstitutional. In other words, the federal government did not have the power to restrict slavery in its territories. So dont bother us anymore.It effectively brought many abolitionists and anti-slavery proponents, particularly in the North, over the edge.

Lincoln said Fourscore and seven years ago our fathers brought forth on this continent a new nation, conceived in liberty and dedicated to the proposition that all men are created equal. The whole institution of slavery was founded on bankrupt principles that some of the thinking slave holders like Jefferson realized that a nation built upon it was on shaky ground.[The whole commerce between a master and slave is a perpetual exercise of the more boisterous passions, the unremitting despotism on the one part, and degrading submission on the other... indeed, I tremble for my country when I reflect that God is just, that his justice cannot sleep forever.]Yet as great a man as Jefferson was ;he could not find an answer to the problem . So they wrote a Constitution with references to slavery without actually naming it in the document.[The 3/5 th clause in Article 1 ;the banning of importation of slaves as of 1808 but not the interstate trade of slavery in Article 1 ;and in Article 4 the clause that states "no Person held to Service or Labour in one State ... escaping to another .. shall be discharged from such Labor, but shall be delivered up on Claim of the Party to whom such Service or Labour may be due." which was the bases for the Supreme Courts ruling in the Dred Scott Case.]

The founders hoped that slavery could be constrained as a state issue and that the federal government could remain neutral. As often happens in this country that was an unrealistic impossible goal .It is just a very difficult request to ask people to compromise on issues they feel are moral.

Lincoln's call for unity in his 2nd inaugural set the tone for the nation that now calls itself The United States instead of THESE United States . But as you know we are constantly faced with the challenges of disunity over moral issues.Now like then it seems that compromise is more like a band-aid than a solution.

Choux rated this answer Excellent or Above Average Answer

Question/Answer
powderpuff asked on 03/28/05 - Is our country's democracy deteriorating ?

As the Bush White House desperately maneuvers in Iraq to prevent the new government from being run according to the dictates of religious fundamentalists, it desperately maneuvers in America to pander to religious fundamentalists who want to dictate how the government should be run.

What do you think?

To read this news item, visit: International Herald Tribune at http://www.q59.com/script/headline_newsmanager.php?id=395852&pagecontent=opinionprogressive&feed_id=276

tomder55 answered on 03/29/05:

sorry ;I can't take the rants of Maureen Dowd seriously .

As Penny notes there is no serious concern about religious fundamnetalists "dictating " to the government. Even the combined efforts of the US Congress ;the President ;the Govenor of Florida;and much of the Florida legislature could not prevent the killing of Terri Schaivo . If anything the religious are desperately trying to preserve their last vestige of influence in this increasingly humanist State.

As for Iraq; the U.S. has not tried to prevent any group ;except for the Batthist rebels who would return Iraq to pre-Saddam days ; from assuming power in the country .Today the newly elected Iraqi legislature will meet for the 2nd time where they will most likely continue the debate on minister positions in the new government .The fact that it has taken them this long proves that a theocratic dictatorship was not elected in Jan. elections. Chrenkoff has a section in his blog called Good News From Iraq that is worth linking to from time to time to bring some balance to the Main Stream Media's pessimism.

As for Tom Delay ;his ethics are in question (the House Ethics Committee, last year rebuked him on three occasions);his ability to lead is thus compromised .He should resign for the good of his party.

Itsdb rated this answer Excellent or Above Average Answer
powderpuff rated this answer Excellent or Above Average Answer

Question/Answer
paraclete asked on 03/28/05 - The United States of dispair

A recent poll has shown concern for the direction the US is headed, where once it was "all the way with LBJ", it's now where they go they go alone. What has cuase America's close ally to stepback and reevaluate?


Our new nightmare: the United States of America
By Tom Allard and Louise Williams
March 29, 2005


Australians are as just as concerned about United States foreign policy as Islamic extremism and regard the US as more dangerous than a rising China, according to a new poll.

The Australians Speak: 2005 survey, commissioned by the Lowy Institute for International Policy, found 57 per cent of Australians were "very worried" or "fairly worried" about the external threat posed by both US foreign policy and Islamic extremism.

"We asked about a series of threats from the outside," said the institute's executive director, Allan Gyngell. "Most startling of all was the precise equivalence of Islamic fundamentalism and US foreign policy as a source of concern.

"The question is whether this is a response to a particular administration or a broader cultural drifting apart."

More than two-thirds - 68 per cent - said Australia took too much notice of the US in its foreign policy deliberations.

The findings would not be welcomed by the Howard Government, which has railed against perceived anti-Americanism and emphasised the importance of the alliance as the US takes a more unilateralist and activist posture in world affairs.

The Lowy Institute found that 72 per cent regarded the US alliance as very important or fairly important. But in another finding, the survey of 1000 people found respondents were strongly opposed to siding with the US over Taiwan should conflict flare between Taipei and Beijing.

Mr Gyngell said he was also very surprised that China rated so positively. Only 35 per cent of respondents had concerns about China's growing power.

"It's not that I thought Australians had a particularly bellicose view on China, but people see opportunities in China, both economically and strategically."

Taiwan is an issue of acute sensitivity at the moment, with tensions escalating with the mainland after China's parliament rubber-stamped a decree warning the province of dire consequences if it declares independence. Asked if they had positive or negative feelings about a list of 15 different countries, institutions and regions, respondents rated the US only 11th, six percentage points ahead of Indonesia, which has traditionally been viewed as a threat by many Australians.

Fifty-eight per cent of those surveyed viewed the US positively, compared with 94 per cent for New Zealand, 86 per cent for Britain, 84 per cent for Japan, and 69 per cent for China.

Fifty-one per cent thought a free-trade agreement with China was a good idea, compared with only 34 per cent for the US deal.

Unfriendly nations acquiring nuclear weapons and global warming were considered more worrying than international terrorism.

Asked about Australian foreign policy goals, most support came for "improving the global environment", with 75 per cent judging it to be "very important".

Protecting jobs and strengthening the economy rated as high, followed by combating terrorism and preventing nuclear proliferation. Promoting democracy rated bottom.

The poll had a margin of error of 3.1 per cent.

tomder55 answered on 03/29/05:

will Australia honor their ANZUS obligations and come to the defense of Taiwan if it is attacked ?The sheer audacity of the anti-secession law should draw attention to Chinas broader set of military policies, which Beijing in turn will be pressured to explain and defend. With China announcing their Anti-Secession Law ,even Europe is revisting the logic of the sale of military goods to China.


The Australian has a different take on the poll cited .

PUBLIC opinion is a wonderfully plastic commodity. In the hands of academic interpreters it can be bent and shaped to prove almost anything.

The Lowy Institute poll on Australians' attitudes to international issues shows how the narrow sets of views held by foreign policy academics in Australia will inevitably replicate themselves in answers to questions designed by such folk.

In other words, this poll tells us little about public opinion but a great deal about think-tank opinion.

The Lowy Institute, devoted as it is to Australian foreign policy, is a good thing. It's full of conscientious folk doing useful work. Unfortunately, it does not look like it's going to inject any fresh thinking into Australian foreign policy or generate any new voices. Rather it will reinforce the sadly quite narrow range of opinions held among professional academic and quasi-academic foreign policy commentators.

Two conclusions of this survey demonstrate the point. They are that Australians have a deep commitment to international law, and would never support the US militarily to protect Taiwan from China.

You could not find two more perfect expressions of foreign policy orthodoxy. How did the poll produce such results? The answer lies in the questions.

On international law, respondents were asked to choose between these alternatives: "Australia should rely on international law even though decisions may go against us OR Australia should do whatever benefits us the most in any given situation regardless of what international law says."

Not surprisingly, the first alternative gets the majority vote. But what would the answer be to a question phrased: If a group of officials from non-democratic countries with appalling human rights records operating in a UN committee directed Australia to do something the majority of its people thought was wrong, should Australia follow international law even though it involves doing wrong or should it do what it believes is right?

In reality, that is much more how questions of breaching international law would present themselves to Australian opinion.

Even to ask a question about so loaded a concept as international law without giving some idea of what you mean is inherently dishonest.

The pollsters' question on Taiwan is even more loaded. Respondents were asked to agree or disagree with the proposition: "Australia should act in accordance with our security alliance with the US even if it means following them to war with China over the independence of Taiwan."

Not surprisingly, a majority would not sign a blank cheque for a hypothetical war.

A more realistic question would have been: Do you think China is justified in mounting a military invasion of Taiwan, even if it causes tens of thousands dead, in order to reunify it with mainland China?

But the pollsters' question perfectly reflects the orthodox assumptions the only possible reason Australia would lend support to Taiwan against a Chinese military move would be because of the US alliance. The merits of the case would have nothing to do with it.

In the real world, in all our history, no decision to participate in a military operation has ever involved a debate purely in those terms.

This poll, sadly, is very nearly meaningless. Politicians and other policy makers would pay attention to it at their peril.





If I were Australia I'd be very concerned about China's growing military strength and sophistication . Thanks in large part to the previous American Administration they were able to upgrade their naval capability significatly .They have already made aggressive jestures in the South China Sea by militarily occupying disputed atolls that may have significant oil reserves under them .

I have no problem with Australia and China finish negotiations towards a free trade zone .I always favor free trade . But be advised that just because they are moving from Maoist Communism economically they still harbor hegemony ambitions .Sooner or later Australia's interests and China's will clash. I suspect it will be over the Chinese "string of pearls "strategy to militarily control the flow of natural resources through the naval choke points in the region.

Prime Minister Howards government doubled its defense budget for the next decade.That should be proof of how serious Australia is about matching its military means to its strategic ends. Howard realizes that Australian security ultimately is linked to a shared defense with the United States. Both nations need to strenghten this important alliance.

Itsdb rated this answer Excellent or Above Average Answer
paraclete rated this answer Excellent or Above Average Answer

Question/Answer
Choux asked on 03/27/05 - Ironic

It is ironic, isn't it. Conservatives and radical religious have been defeated in their efforts to save Terri by their own "principles".

All the judges(save one) who voted to uphold the judge's decision to allow Terri's feeding tube removed were conservative judges appointed by Republicans. Including members of the Supreme Court who turned down hearing her perents case. NO ACTIVIST JUDGES HERE. Do I hear the clapping of the Radical Right Propagandists? A victory for Conservatism.

Wait, save one? a lone Clinton appointee who voted to hear the case.... a "liberal" who cares about the rights of individuals such as "life, liberty, and the pursuit of happiness". A judicial activist??

Ironic, isn't it.

tomder55 answered on 03/28/05:

I would like to know where Alberto Gonzales is ? Why has the Justice Dept. not enforced either the supoenas or the law passed last weekend?

It was a Clintoon appointee at the 11th Circuit court who refused to do what Congress specifically directed him to do. It was Justice Kennedy who made the decision for the Supremes (he oversees the 11th District courts);the same Justice Kennedy who recently stated that 'evolving international standards ' were the basis of our laws.

"The courts of the United States can find a right for the abortion industry to take a fully formed, totally healthy baby at nine months' term, out of his mother's womb and murder it by putting scissors through it's brain and grinding them about.

They do this without one single word of support from any Congressional act of any kind ever.

They can find a right of savage murderers of innocent women who drown them for a lark to avoid the death penalty because they are old enough to drive and to kill but supposedly too young to be executed. Again, there is not one syllable in any Congressional act that sanctions this protection of the guilty.

But with the Congress and the President of the United States pleading for the life of a woman who is not brain dead, who responds to words and to touch, who is not on life support, whose parents beg for her to be kept alive, whose nurses give affidavits that she can be rehabilitated, with a specific law commanding the courts to review the case to keep this poor soul alive, the courts instead find no rights for her.

This is a court system totally out of control, obviously committed to death, obviously bound by nothing beyond its morbid obsession with its own omnipotence and its fascination with the letting the innocent die. This is simply terrifying. The Falange followers of Francisco Franco had an evil cry: Long live death. Obviously, Justice Kennedy was listening."(Ben Stein)

Choux rated this answer Excellent or Above Average Answer
ETWolverine rated this answer Excellent or Above Average Answer
sapphire630 rated this answer Excellent or Above Average Answer

Question/Answer
Choux asked on 03/25/05 - Bush's Poll Numbers Sliding

President Bush's approval numbers are sliding downward over the last week when the Terri Schiavo political circus began *again* after its hiatus since 2003.

Is this the issue the Radical Republicans will kill their Party over? I think it is.

Comments?

tomder55 answered on 03/26/05:

poll numbers fluctuate . I think they are currently within an acceptable margin of error. If this case proves anything it is that Bush does not act at the whim of popular opinion polls like his predacessor did . He is bucking the tide in the Social security debate ,and with this case if you believe the polls about it.

The coalition that the Republicans have forged is bound to splinter ;all coalitions do. There are the libertines in the party who could care less about taking a moral stand on anything ;and are only concerned with the boogyman they call gvt. intrusion. Where are they going to go ;to the Democrats??? Some of the hard core libertarians have formed their own party and they usually skim 1-2 % off the vote. They are the fringe ;the radical.

Choux rated this answer Excellent or Above Average Answer

Question/Answer
sapphire630 asked on 03/23/05 - old news with my twist added

A bum was [loitering] daily at a public library and stunk up the library so they told him to leave. He sued for thousands of dollars and won. Then he was hanging out & stinking up some port authority transit place. They told him to leave he is suing for twice what he sued the library for.
My twist to what should happen next:
He should go hang out at the court where he is suing these places. Then when he gets kicked out of there sue them for twice as much as he is suing the public transit for.

tomder55 answered on 03/24/05:

maybe he should hang out in the shower

purplewings rated this answer Excellent or Above Average Answer
sapphire630 rated this answer Excellent or Above Average Answer

Question/Answer
Choux asked on 03/22/05 - Backfired?

Did Bush's and assorted Republican politician's grandstanding about poor Terri Schiavo backfire. Polls revealed on "Hardball" with Chris Matthews showed that 62% of Americans agree that Terri should be allowed to die in peace. Seventy percent think that this matter is none of the Federal Government's business.

tomder55 answered on 03/23/05:

I don't care .It's not about who is going to win politically . I'd rather suffer political loss and be right on this issue. Michael Schiavo's fight to remove his wife's feeding tube has raised suspicions about his motives ,and that is reason enough to delay this decision.

PVS has now been dilluted by this case from the Karen Quinlan case where she was removed from respirator to now where someone who only needs a feeding tube is declared unfit to live. Florida law defines a vegetative state as "the absence of voluntary action or cognitive behavior" and "an inability to communicate or interact purposefully with the environment."

Terri seems to respond to people and events. Although some experts have dismissed the significance of her actions, others disagree. For instance, neuropsychologist Dr. Alexander Gimon argued that her responses "are completely inconsistent with a diagnosis of vegetative state."

Contended neurologist Dr. William Hammesfahr, Terri "is alert and responsive to her environment. She responds to specific people best." Her one-time court-appointed guardian ad litem, Jay Wolfson, concluded that Terri had a "distinct presence" and responded to family visitors.

Finally, Dr. William Hammesfahr, who has aided people with chronic brain injuries, said: "There are many approaches that would help Terri Schiavo. I know, because I had the opportunity to personally examine her, her medical records, and her X-rays."
[http://www.spectator.org/dsp_article.asp?art_id=7928]


Abortion went from the need to save the mother's health, to protecting rape victims ,to abortion on demand. It went from 1st trimester to bashing in the brains of babies at the momement of birth. In Holland the law now gives doctors the right to decide life and death of babies .See where this slippery slope leads ? You are always harping on the cost of medical care .What happens when they dillute the standards to where Alzheimer patients are offed?Who else is unfit to live ?

If Schiavo was pronounced brain dead, there would be no controversy. She would be legally dead, and the hospital would allow her to be taken off life support even if a relative objected. Had she left written instructions ,a living will ,or a health proxy, someone to make the decision ,there would also be no dispute.Schiavo's case is unclear and President Bush and the Republicans are correct in saying we should err on the side of life .

From a Constitutional side there are still things that the Federal Gvt. can do . bush can simply nullify the Courts decisions[Federalist #78 says the judiciary is "dependent on the executive even for the efficacy of its judgements".] and make an executive decision to have her taken into protective custody (as a witness supoenaed by Congress );and have her feeding tubes reinserted until the Courts reopen the case as Congress ordered them to do .That would fulfill the balance of powers .The courts have blatantly ignore caretakers and witnesses who have sworn affidavits as to Terri's true condition and have also witnessed this man say "when is that b*tch going to die" and ignore all the rest of the very pertinent medical facts in this case.

The court system in this country now is an uber-judiciary . I do not recall them ever being given the power to order someones death as the Florida judge did last week.

ETWolverine rated this answer Excellent or Above Average Answer
Choux rated this answer Excellent or Above Average Answer

Question/Answer
Choux asked on 03/21/05 - DIRTY TRICKS 2005

TODAY, A REPUBLICAN AD APPEARED ON TV. THE UBIQUITOUS SERIOUS MALE VOICE ASKS THAT VIEWER, WHAT PROPOSALS HAVE THE DEMOCRATS PUT ON THE TABLE TO REFORM SOCIAL SECURITY?? CAN YOU NAME ONE??

THEN, A STOPWATCH STARTS TICKING, CLOSE0UP OF STOPWATCH.

DON'T WE HAVE A REPUBLICAN PRESIDENT, A REPUBLICAN SENATE, A REPUBLICAN HOUSE OF REPRESENTATIVES?????
AREN'T THE REPUBLICANS THE PARTY THAT WANTS TO REFORM SOCIAL SECURITY???
ISN'T MEDICARE AND MEDICAID THE REAL PRESSING PROBLEM????

WHAT ARE YOUR PROPOSALS, GEORGE????
QUIT PLAYING GAMES WITH THE FUTURE!!!!

COMMENTS?

tomder55 answered on 03/21/05:

If this is true then it is demagoging the issue. The Republicans in Congress cannot get together with the President on a bill they agree on .Why should the Democrats propose an alternative ? Also I think the debate is only just beginning . Bush has spent some effort to get on the bully pulpit just to try to convince people that there is some urgency to the SS problem. Americans live for the now and don't see that the system is a fraud ;that the non-existant trust fund will be in the red now;not in 2018 ;not in 2042 .

Congress has been embezzling from the fund ,and there should be the same outrage about that that there was was Enron and Worldcom scams were revealed .

The Republicans are waiting for Bush to be able to prove there is public support for change ;but too many people listen to idiot special interest groups like AARP and the teacher's unions . Why AARP has any concern about it at all is beyond me. No senior will be affected by the reforms. Only people like me in the age group below the 55 ers really get screwed by the reforms as Bush imagines them but guess what:? I think the personal accounts are an excellent idea. I just wish someone had thought of them 25 years ago..

I would say that both SS and Medicare are equally in trouble the only question is when they become a crisis because that is the only time people get their heads out of their.....um ...out of the sand. . The silly Senior's prescription plan only worsened the situation ,and that should be reversed before it becomes set in stone.

Choux rated this answer Excellent or Above Average Answer
ETWolverine rated this answer Excellent or Above Average Answer

Question/Answer
Choux asked on 03/19/05 - OIL(HENCE GASOLINE) PROBLEMS

WILL WE EVER GET ANY LEADERSHIP OUR OF OUR GOVERNMENT ON INTERNAL CRISES????? FOR ONE, SINCE THE OIL CRISES IN THE 1970'S WHEN OPEC CUT EXPORTS, WE HAVE HAD NO LEADERSHIP IN GOVERNMENT TO LESSEN OUR DEPENDENCE ON IMPORTED MIDDLE-EASTERN OIL. TODAY, FOLKS ARE STILL DRIVING GAS GUZZLERS, IMPROVEMENT IN MASS TRANSIT-- IGNORED.

THE PRICE OF OIL-GASOLINE IS ONLY GOING TO GO UP AND UP. WE HAVE GROWING COMPETETION FROM INDIA AND CHINA FOR OIL AS THEY INDUSTRIALIZE. MORE AND MORE DEMAND, HIGHER AND HIGHER PRICES. THAT IS A GIVEN.

ONE OTHER MATTER, THE CRISES IN HEALTHCARE. WILL THE REPUBLICANS AND DEMOCRATS COOPERATE IN DELING WITH THE MEDICARE AND MEDICAID FIASCO COMING IN A FEW YEARS OR WILL DECISIONS BE MADE ONLY WHEN THE WHOLE SYSTEM IS UNDER SEVERE STRESS??

REALLY, HOW DO THESE FACTS FIT INTO THE SCENARIO FOR WHAT LIFE WILL BE LIKE IN AMERICA IN 25 YEARS??



tomder55 answered on 03/20/05:

not enough leadership ;certainly I'd like to see a comprehensive strategy ;but some is evident.

The US Senate Environment & Public Works Committee passed legislation to increase the US's Renewable Fuels Standard (RFS) to mandate the production of 6 billion gallons of ethanol by 2012.http://www.renewableenergyaccess.com/rea/news/story?id=23873

The US Department of Agriculture (USDA) is working to disseminate techniques for bio-fuel production from forest bio-mass, a practice which has become popular in Scandinavian countries. http://agnews.tamu.edu/dailynews/stories/FRSC/Mar1405a.htm

They formed a partnership with the Environmental Protection Agency to review and promote systems for methane gas recovery via anaerobic digestion of animal waste for use in rural agriculture.

The EPA is extending the Energy Star program to provide guidelines for more efficient AC adapters.

The Energy Dept. has launched a new Hydrogen Program website. The new site links the four DOE Offices ( Energy Efficiency and Renewable Energy , Fossil Energy , Nuclear Energy, Science and Technology , and Science) that participate in the Hydrogen Fuel Initiative (HFI) and serves as an integrated portal for information on DOEs HFI efforts. http://www.hydrogen.energy.gov/

The US has signed a multilateral agreement that will develop next-generation nuclear energy technologies that will include substantial research into designs for electricity/hydrogen producing reactors. http://www.energy.gov/engine/content.do?PUBLIC_ID=17543&BT_CODE=PR_PRESSRELEASES&TT_CODE=PRESSRELEASE
(that is of course if activist don't throw monkey wrenches into new nuclear development)

Government funded labs are working hard to develop the next generation hydrogen catalysts for much more efficient use of that technology http://www.eurekalert.org/pub_releases/2005-03/dnl-cnc030705.php

And plug in hybrids are on the way http://www.iags.org/pih.htm

In some areas geo-thermal electricity is an option. Construction has begun on the first geothermal plant to be built in Nevada in over a decade. The Galena Geothermal Project will join three other geothermal plants operated by Ormat Nevada in the Steamboat Hills south of Reno, and will generate about 20 megawatts of power.

Wind and solar can and should be encouraged in localized regions of the country ,but they are in themselves no enough to fix the problem.

Oil from shale is a potential source and now that the price is right it might be worth developing the technology to extract it.Oil shale is plentiful and could someday be processed economically.
http://www.worldenergy.org/wec-geis/publications/reports/ser/shale/shale.asp

...and of course the Senate approved drilling in ANWR which means that in 10 years a 6 month supply of oil will be available to the world market.

Conservation is a laudable goal but it is not a policy .We cannot drill our way to energy independence either (although that is part of the near term solution and many areas that we are not drilling should be opened up .We have not built new refineries in years because of NIMBY . We have built no new nuclear reactors for the same reason. ThThese would help in the short term.

But only the aggressive funding of research by industry in cooperation with the gvt. can provide the means to fuel our economy in the future.






Choux rated this answer Excellent or Above Average Answer

Question/Answer
elgin_republicans asked on 03/17/05 - March Madness Presidential Tourney

Who will be the Republican Candidate? Pick the next Republican candidate for President out of this field of 64 (just like the the NCAA). Go to this website:

http://www.surveysaintlouis.com/marchmadness/Bracket.php

Its a hoot. (hoot?)I still don't know how to make a direct link.

tomder55 answered on 03/18/05:

Buchanan beat Arnold ? don't think so. bracket ;McCain losing to Tancredo ?

I got Santorum beating Rudy 1st bracket ;McCain winning 2nd bracket ;Jeb Bush beating Bill Frist in a close contest for the 3rd ;and JC Watts going far but ultimately losing to Condi in the last bracket .

Question/Answer
Yiddishkeit asked on 03/16/05 - Hello my friends...

Just in case anyone should ask why my activity on the boards has slowed down. I was once married for eight and half years, had a divorce, and now I'm dedicating more time with my soon to be wife. I have been blessed with somebody extremely faithful and loving. We are planning on being married later on this year and I hope everyone understands the importance of this time in my life. I will try and check in every once in awhile.

Take Care,
Bobby

tomder55 answered on 03/17/05:

that is great news ;best of happiness to you both !!!

check in when you can and if she is so inclined perhaps she'd like to join in also .

all the best

Tom

Yiddishkeit rated this answer Excellent or Above Average Answer

Question/Answer
Choux asked on 03/14/05 - Briefly on Cable

This morning just after I got up, there was a story on cable news about how Israel will take out Iran's nuclear weapons plants if Iran continues to pursue nuclear weapons. Then, added, that diplomacy will continue with Iran..ie America paying them not to produce weapons.

Did I hear this correctly??

tomder55 answered on 03/16/05:

Ephraim Sneh, a member of the Israeli parliamentary defence and foreign affairs committee states that Israel will only attack Iran "as a last resort".The NY Times reported that Israel has plans for a combined attack on the Natanz nuclear facility, including raids by the Kingfisher commando unit and bunker-busters dropped from F-15s. Ariel Sharon and his inner cabinet gave preliminary approval for a plan to strike last month in a meeting at his ranch.




http://www.guardian.co.uk/israel/Story/0,2763,1436990,00.html

http://www.timesonline.co.uk/article/0,,2089-1522978,00.html

Choux rated this answer Excellent or Above Average Answer

Question/Answer
curious98 asked on 03/14/05 - Press complaints


Im reading today in Le Journal de Genve that more than 50 USA press organizations, (amongst them the Associated Press, The Miami Herald, The Sun Sentinel, etc.) have gathered in what seems to be called the Week of Light, to protest for the changes carried out by the Bush Administration regarding freedom of information that, according to these entities, attempt against the 1st Amendment of the US Constitution.
Is it true?
Curious98

tomder55 answered on 03/14/05:

Yeah I saw their editorial spin in every newspaper I read Sunday. It is called "Sunshine Week: Your Right To Know .It is a week long campaign that is supposed to push for more government openness.The coalition claims they will lobby for legislation and seek to educate the public about First Amendment issues.They have also sponsored a bill (S. 394) called the Open Government Act of 2005', that is intended to get results quicker under the Freedom of Information Act

What they really want however is the power to decide which information gets to the public . What they (the press ) thinks you need to know). Most Americans understand that there is security sensitive information that we just dont need to know about. All other information should be public knowledge .

It is laughable that these major news sources who have made a concerted effort this year to attack 'bloggers' those citizen journalists who have attempted to be a watch dog to the media watchdogs should take a postition like this . This reeks of hypocrisy in the face of numerous scandals and an ingrained mindset that they answer to no one.The bloggers are ironically opening up the media's biases to sunshine .

These are the clowns who have protected us from decapitation photos of victims of terrorism while at the same time have no hesitation showing the 'innocent victims ' of 'American aggression' . They suppressed photos of victims of 9-11 making the decision to leap from the top of the World Trade Centers lest they needlessly offend Muslims .These are organizations that just happen to be on the street corner ;camera's ready when a car bomb explodes in Baghdad. Jack Stokes, the Associated Press director of media relations explained
"Insurgents want their stories told as much as other people " They of course sware that their sources need to be protected .

Since Vietnam, the press's self-described mission of reporting the news without fear or favor has been corrupted. Journalists now enter the business to change society or save the world. Long gone are the days (if they ever existed ) when the media was unbiased. It is foolish for them to remind us of their vital role while they blatantly missuse it to advance a political agenda.






Choux rated this answer Excellent or Above Average Answer
curious98 rated this answer Excellent or Above Average Answer
ETWolverine rated this answer Excellent or Above Average Answer
kindj rated this answer Excellent or Above Average Answer

Question/Answer
HANK1 asked on 03/11/05 - DO YOU AGREE?



I think what we had better do is get the citizens in Iraq safely settled and then bring our troops home PDQ. I'm getting very tired of America being a 'savior nation' when it effects our own citizens. There's plenty of 'work' to be done here. Let the United Nations and N.A.T.O. take over our responsibilities. Neither have done a damn thing to help us since 9/11. We need peace at home and to hell with non-functioning continents such as Europe and others -- north, south, east and west. It's just too bad politicians have to run our country. It's a cinch that we no longer have representation. All we have is polls.

Do you agree?

HANK

tomder55 answered on 03/12/05:

I thought that the US made a mistake by going to the UN asking permission to go to war was a mistake to begin with. The justifications for a declaration of war already existed with Iraqis firing on US planes patrolling the UN mandated no fly zones.Add on Saddam's other violations of the 1991 cease fire and the cause for war was solid.

I understand the frustration . Even with the declaration of war against the West by Jihadistan ;the mass killings ,beheadings ,suicide bombings ;the persecutions of women ,the forced conversions; nations that we should be counting on as allies are more fearful of American intents than the murderous allies of terrorism.

It is tempting to say ;let the French with their one leaky aircraft carrier defend their own interests[the sailors are suing because the radiation levels are several times the allowed levels.] .In fact ;let the Germans re-militarize and see how well the French sleep at night.Let the Italians and other European nations who have a greater stake in the Balkans put their young women soldiers at risk there in that unsettled part of their continent. Let the combined forces of the EU defend the choke points that their source of energy must pass through. Let the South Koreans fend for themselves against the mad-man in the North.Let the Taiwanese ,and Japanese ,and the Australians ;all staunch allies of the US fend for themselves against the growing threat of China.Even the Canadians have during successive liberal gvt.s gutted their military capability ,and diverted funds to a paternalistic welfare state to the point where they cannot defend ice islands in the Arctic Sea from Norwegian incursions.They have recently bowed out of missle defense compacts with us.

The long French goal of driving America from continental Europe is close .Perhaps it would be in our best interest to abandon cold war alliances ;leave the European's ;who finance their welfare states while America picked up the tab for their security; to their own devices .But we still have to do what is right.Turning over responsibility to those who would shirk is irresponsible.Remember ;America's pull back after WWI it can be argues helped lead to the eventual rise of fascism.The consequences to us was severe . Most of Asia appreciates and supports the need for American presence .

Bush is handling it correctly .He is re-aligning America away from Western Europe.Let's see if the Germans fear jihadistan as much as they fear American withdrawal from bases used by our troops to defend Germany's beer halls. Our European alliances are increasingly becoming ones with Eastern European nations that lived under the yoke of Soviet tyranny nad understand that free nations should stick together.The Poles would appreciate us relocating our bases there. I suggest we reward them for their continued support by moving out of Germany and into Poland.

You are wrong about NATO . NATO has a presence in Afghanistan ;and although there was obstruction by some of our allies regarding Iraq ;there has been a large degree of cooperation ,and sharing of intelligence in the GWOT. France now is allied with us ,and was key in helping us get a resolution about Lebanon through the UN. They along with Germany and Britain are negotiating with Iran over their nuclear program. I do not know (I doubtit in fact) that they will be successful;but for now Condi Rice has indicated that the U.S. supports their continued effort. We should continue to push for a strong NATO ;especially when you look at the proposed alternative(a EU force independent of American concerns).

It appears that at the dawn of the new century security alliances will be temporary alliances of convienience;shifting from player to player depending on national concerns.

As far as Iraq ;it would be foolinsh and dangerous to leave there without leaving a secure nation. We are closer than you realize. just having an election there has created ripples across the Arab region that may wake them out of a 13th century slumber. Jihadistan must be defeated . We will not be secure otherwise. The battle has been taken there. The alternative to that we saw on 9-11-01 .

Choux rated this answer Excellent or Above Average Answer
ETWolverine rated this answer Excellent or Above Average Answer
HANK1 rated this answer Excellent or Above Average Answer

Question/Answer
sapphire630 asked on 03/09/05 - Pissr

Did I hear this backwards or what?
People in search of SAFE restrooms is a 6 month organization that is gay, lesbian and transvestites that are in search of restrooms 'they feel safe in'. They have websites where they post restrooms 'they feel safe in'. Sounds kinda backwards to me. Don't they mean so others feel safe by not running into something?

tomder55 answered on 03/10/05:

any rest stop on the NJ Turnpikes is a safe rest room for them ;especially the 'Vince Lombardi Rest Stop'

Itsdb rated this answer Excellent or Above Average Answer
sapphire630 rated this answer Excellent or Above Average Answer

Question/Answer
purplewings asked on 03/09/05 - A concept to make peace worldwide.

http://www.beliefnet.com/story/161/story_16163_1.html

I posted an interview from Beliefnet.com with Marianne Williamson, on the Spirituality Board today. It indicates an effort to legislate a Department of Peace. The suggestion being that terrorism is a disease that cannot be cut out by war or threats just as a doctor cannot cut out cancer once it has metabolized, so we must learn a better way to end it before it can get to that stage.

Who would be part of the Department of Peace?
The legislation as it has been offered by Congressman Dennis Kucinich calls for a peace academy as a complement to the military academy. At the military academy, we teach and learn the most advanced ways to wage war. At the peace academy, we would learn the most advanced ways to wage peace. Iraq is a tragic example of American effectiveness at waging war, i.e. destroying what we didnt want in the first days of the war, but our inability in the days and months following to wage peace.

Do you find this to have merit as far as Militarily or Politically? What about taxes or religious issues? I'd also like to have your answer on the Spirituality board, based on the Spiritual premise if you would.

Thanks so much.

PW

tomder55 answered on 03/09/05:

oh my God ! it's true !! I can't believe this one slipped by my radar ! its HR1673 sponsored by the big K and has 52 other sponsors . I especially like Sec. 109 creating an" Office of Peaceful Coexistence and Nonviolent Conflict Resolution". Wonder if Jihadistan will go along with this ?

Forget all the domestic Orwellian powers this bill would allow the Dept. of Peace ;it would also interfere ;and directly control certain military matters;namely any mission undertaken by the militarty that is deemed 'peace keeping '.

The bill also overlaps the powers of this new super agency into the Dept. of Education ;the Dept. of State ; and the Dept. of Tresury .Indeed I'll wager that this gives the Dept. enough power that war could not be waged without the Dept.Heads approval ! [The Secretary of Defense and the Secretary of State shall consult with the Secretary (of Peace) concerning nonviolent means of conflict resolution...prior to the initiation of any armed confilct between the United State and any other nation, Section 112 ]

Leave it o Denis K to come up with this . Mind you ;I think the military or more likely the Dept. of State should develop post war specialists to deal with reconstruction ,and I think U.S. NGOs like the Peace Corp does some outstanding work ;but lets not kid ourselves about intentions here. This bill is an attempt to take the decision making away from the Constituionally appointed means determining when to go to war ;and handing it over to an intractable buerocracy. We have something simular to that now .....it's called the U.N.

Choux rated this answer Excellent or Above Average Answer
ETWolverine rated this answer Excellent or Above Average Answer
purplewings rated this answer Excellent or Above Average Answer

Question/Answer
ROLCAM asked on 03/09/05 - Very concerned !!

What are you views about China's new attitude
towards TAIWAN ?

ROLCAM.

tomder55 answered on 03/09/05:

Not that their views are new ; they have always considered Taiwan a renegade province of China and never have considered them independent . China has always talked of invasion should Taiwan make overt moves of independence ;now it appears that China is preparing to be in a position to do so.

Taiwan was a part of the Chinese Empire until it was ceded to Japan at the end of the Sino-Japanese War in 1895.after WWII the United States forced Japan to relinquish control to the Nationalist Chinese government in 1945 .In 1949, the Nationalist government moved to Taiwan and set up shop as a sort of government in exile after the Mao Communist take over . For years we had the position that the government in Taiwan was the legitimate government of China but after US recognition of the communist regime in China, this was dropped. Over the years the people of the island transformed themselves into a vibrant, free-market democracy which remained Chinese in culture, ceased for all practical purposes to be part of China .The Chinese communists have never accepted this reality and demand that Taiwan surrender itself to China. The United States opposes any change in the status of Taiwan by force and has pledged military support for Taiwan if China attempts to invade.Recently Japan has joined the US in a mutual agreement over security concerns in the region .This includes cooperation on the Taiwan issue.Japan has the world's third largest Navy and in actual effective combat units probably the second largest.They could effectively prevent an invasion force from crossing by themselves.


China this week introduced legislation ;a so called anti-secession law ;that would only really put a legal face on the status quo. They have been pointing a greater # of missles at the island and have been spending alot of the US dollars they get from Walmart items on building up their navy .

Make no mistake;China is an emerging force in the world. I have posted here about China using provisions of the "Law of the Sea " treaty (LOST ) to claim islands in the South China Sea [including Taiwan](which they feel is in their sphere of influence ;and we are intruders ).They are undertaking to extend their naval influence to the Persian Gulf through basing rights (the so called String of Pearls stategy ). The Chinese through a shipping company closely linked to their military controls operations in the Panama Canal , and they are negotiating simular economic and military deals throughout South America ;and the Carribbean .

To make things more complicated ;our good buddies in the EU (especially France ) want to lift the ban on selling to China advanced weapons [but why shouldn't they ? Clinton practically single handedly moderninzed their submarine fleet and their ICBM MIRVs]. Also ;France has already conducted joint Naval Exercises with the Chinese.

What does all this mean ? The US has always maintained a "strategic ambiguity" regarding our intentions . We have always left the Chinese guessing what we would do ;even though our actions have always indicated that we would help in the defense of Taiwan.So long as some ambiguity is preserved, China is more able to tolerate what would otherwise become an intolerable U.S. position.They can save face by maintaining the fiction od a 'one China'and they were less likely to test our resolve on the issue.It also left some doubt with Taiwan ,and they were less likely to declare independence without assurances of U.S. backing.

I do not know how much longer this status quo can last .I do not think anything will happen before the 2008 Olympics that will be held in Beijing ,but am uncertain how long the status quo can be maintained afterwards .China is increasingly worried that Taiwan, under the leadership of President Chen Shui-bian, is inching towards a formal breakaway ;and this new law in China may provoke US lawmakers into recognizing Taiwan again and resume diplomatic ties that were cut when we officially recognized China. I suspect that Condi Rice will have alot to do in her term as Sec.State.

One thing to keep in mind ;President Bush (that unilateralist) has spent alot of effort in the last 4 years to strengthen ties with our allies in the Pacific and Indian Ocean regions. Japan;Australia have been a big help in the GWOT ;so has other nations like Indonesia; and Pakistan .Condi will visit India this month to strenghten ties with them.Even our large, visible tsunami response fit well into the President's plan.It demonstrated to the region just who can put resources on the ground quickly.

What we are doing is ringing China with a deterence.And who can China count on ? ; North Korea ? (I'm assuming that France will be as dependable to China as it is to the U.S. ;anyway in the region France would not be a factor).

ETWolverine rated this answer Excellent or Above Average Answer
ROLCAM rated this answer Excellent or Above Average Answer

Question/Answer
sapphire630 asked on 03/07/05 - Professor Churchill

I was listening to some recordings on a talk show of things that Prof. Churchill has to say to his college students. One that really caught my attention was...
When {he} is in another state and gets a ticket he looks to see how much the ticket is for, then makes sure he does that much property damage before he leaves that state.
What a thing for students to look up to as a fine rule model!
It's one thing to pass yourself off as N.A. Indian when you only have a token gimmick card but gee!

?????

tomder55 answered on 03/08/05:

He reminds me of Christpher Lloyds character Rev. Jim from the TV series Taxi .

sapphire630 rated this answer Excellent or Above Average Answer

Question/Answer
ETWolverine asked on 03/07/05 - For all you NYers out there.

From the New York Post, 3/7/05.

JERSEY EXPLOITING N.Y.'S STADIUM BATTLE, MIKE SAYS
By STEPHANIE GASKELL

New Jersey is taking advantage of the fight over where to build a new stadium for the Jets by proposing a plan to build an arena there, Mayor Bloomberg charged yesterday.

"Gov. [Richard] Codey is working very hard to keep the Jets there," Bloomberg said during a press conference in Staten Island.

The New Jersey Sports and Exposition Authority did a study showing that if the Jets stayed in Jersey and built a stadium that the Jets and Giants could share, it would cost each team about $450 million.

That's much cheaper than the $800 million the Jets are willing to spend to come to Manhattan.

"We should realize we have plenty of competition from New Jersey, and if it gets too tough or too expensive for the Jets, you can see them doing that," Bloomberg said. "I hope they don't, and that's why we're working very hard."

A Jets spokesman insisted the team still wants to come to New York.

"We're ready to invest over $1 billion to bring our team back home to New York," said Matt Higgins, the Jets' vice president of strategic planning.

A source close to negotiations told The Post that Jets owner Woody Johnson "is determined to see this through to the end."

The stadium flap came on a day packed with several events for the mayor including a St. Patrick's Day Parade in West Brighton, Staten Island, where the mayor got cheers for trying to get the stadium and some boos for not giving raises to teachers.

"People were screaming, 'Get the stadium! Get the stadium!' because people want jobs, and the stadium means jobs," Bloomberg said. "There were some people that screamed they want contracts . . . but I think the crowd could not have been friendlier."

While some paradegoers screamed things like, "Don't listen to 'em, Mike Get the stadium!" others were peeved that the mayor wasn't listening to their needs. Dozens of teachers lined up along the route and chanted, "Don't turn your back on Staten Island schools!"

Earlier in Queens, Bloomberg defended another issue, gay marriage.

During the "all-inclusive" St. Patrick's Day Parade in Sunnyside, Bloomberg told thousands of paradegoers that "The city is doing what it can to make sure that it opens itself to everyone."

Additional reporting by Justin Terranova

-------------

I happen to be in favor of the Stadium. But Bloomberg's hypocracy really pisses me off, even though I agree with the issue. Bloomberg claims to be worried that NYC will lose out to NJ if the Jets don't come to NY.

Where was his concern with loosing out to NJ when he raised NYC taxes, eliminated the tax exemption for clothing, and regulated the hell out of NYC... all of which has forced countless businesses out of NYC to NJ.

Bloomy has GOT to go.

Elliot

tomder55 answered on 03/07/05:

It is really funny going to the Jersey Malls and seeing NYC taxmen taking down the license plates on NY cars that are there to take advantage of the tax free clothing in NJ. Today as I went to work the gas stations on the NY side of my commute were well over the $2.00 /gal. mark. NJ gas was going for around $1.80. but .... there is a special place on my tax form where I'm supposed to calculate the difference in tax savings from shopping in NJ ;and give it to NY. I guess I get to pay more for the privilege of saying I'm a NYer .

Jersey is right about the stadium also . They share a stadium now ,and get 16 home games a season instead of the 8 that a single home stadium would bring .They also utilize Giants Stadium now with college games. Personally I think they should renovate the existing stadium and try to bring rail into it . The site is perfect otherwise . Plenty of parking . Tailgaters love the spaciousness. If you show up early enough you can play a touch football game in the lot .If they put a retractable dome ;and added some more luxury suites then it would be a natural for a SuperBowl. They could still take advantage of the NY City amenities ,and on game day just cross the Hudson at the Lincoln tunnel and they are right there .

The Jets have belly ached about not having a Jets identity there for years .(ok maybe they should change the name of the stadium so the Jets don't play at the 'Giants'Stadium).But they sell out every game . The crowd is boisterous (much more so than the Giants crowd).the only thing they have not done there is win the big one .The idea that the fan base is from Long Island is a myth.That may have been true in 68 ;but not now.

Bloomberg could care less for the Jets . He wants the stadium for an Olympic bid.He is obsessed with it . Then he thinks the stadium would handle the spill over from the Convention business. It would be a nighmare to attend games there ,and unless he could find other ways to utilize it ;it would be vacant for all but 8 weeks out of the year.(the fans that tail gate now you would never hear the end of their belly aching ).

I will say 2 thing in his defense .I think his handling of the school system is bold and imaginative and the Gates Foundation has heaped praise upon.Under the mayor New York City already has a network of high-performing small schools and alternative schools and may well be a national model for the inner city .The reforms are worth trying until it is proven ineffective.

I also like the fact that he does not get beat by the unions .

ETWolverine rated this answer Excellent or Above Average Answer

Question/Answer
Choux asked on 03/05/05 - USA Winning War

On a talk show this morning on CNN, the usual suspects were interviewing a mamber of a "think tank" about the situation in the middle east. Very interesting. Anyway, he said that the *consensus* was that the USA is winning the War in the Middle East.

That the ploy of Middle Eastern leaders to blame America and Jews for all the discontents of the people they govern in order that the folks don't turn on them as the real cause of all their problems is *over*.

Comments?

tomder55 answered on 03/07/05:

BRATISLAVA, Slovakia, March 7 (UPI) -- We could well be living through the second great wave of democratization in living memory. And this time it may be going global.

Last week's display of "people power" in Lebanon, which brought down the country's pro-Syrian government, comes just weeks after mass street protests in Ukraine brought pro-democracy campaigners to power against seemingly overwhelming odds. That revolution, in its turn, self-consciously modeled itself on previous peaceful revolutions in Georgia, Serbia and Slovakia.

Globalized images through 24-hour-a-day television news channels like CNN and BBC World are transmitting democratic best practice across the world. Those images are being accompanied by the relentless rhetoric of a U.S. president who has made the spread of democracy the centerpiece of his second term agenda. Triumphalism on the Right is back. But can the administration really take credit for what is now going on? And is freedom really on the march, as the White House says it is?

Personally, I always thought the Bush administration was far ahead of the pack in its understanding of political dynamics in the Middle East. I also thought a U.S.-led agenda for global democracy was the best overarching strategy to combat the root causes of the "grievances" that led to 9/11. I even thought it might succeed.

But Lebanon's Druze opposition leader Walid Jumblatt, a man whose hatred of the Bush administration once extended to lamenting the fact that Deputy Defense Secretary Paul Wolfowitz had not been murdered in a terrorist attack, is perhaps better placed than I to comment. On Feb. 23, he told David Ignatius in The Washington Post: "This process of change has started because of the American invasion of Iraq. ... I was cynical about Iraq. But when I saw the Iraqi people voting three weeks ago, 8 million of them, it was the start of a new Arab world. ... The Berlin Wall has fallen. We can see it."

And then there was another equally unlikely convert to the neocon foreign policy agenda -- Nasser al-Kidwa, the Palestinian foreign minister. Speaking on CNN last week, he said, "I would say the situation in the region, the U.S. agenda in the whole region to ensure all the democratization ... strongly suggests that we might be heading into a different situation."

Enter Hosni Mubarak, once president for all eternity of Egypt, promising multi-candidate presidential elections by the end of the year. The list goes on. As Jordanian Foreign Minister Hani al-Mulki put it succinctly, "I think the train has just left the station."

It seems well nigh incontrovertible that the two key events in the region sparking such changes -- the Iraqi elections and the Palestinian elections -- were indeed the direct result of a policy agenda that is specific to the Bush administration. Elections in Iraq, of course, could not have taken place without the invasion. That much is beyond all doubt. What is less obvious is that the kind of elections that took place in the Palestinian territories were also made possible by the Bush-led policy of isolating Yasser Arafat and publicly condemning all that he stood for. The White House could not, of course, have foretold his death last year. But once Arafat was gone, U.S. policy had so narrowed the range of options available to any future leadership that a shift toward genuine democracy and genuine engagement in the peace process inevitably became the most compelling direction to go in. In London last week, Palestinian President Mahmoud Abbas was booed by a crowd of Islamic extremists concerned that, in their terms, he was selling out. In other words, they're worried that he's for real. They never worried for one moment that Yasser Arafat was for real. He wasn't, and George Bush knew it.

Now, I would be the first to say that there is a long, long way to go. If the point of reference is the first wave of democratization, which came after the fall of communism in 1989, then events in that region over the last decade and a half should alert us to just how bumpy the road is going to be. There were, of course, tremendous successes: the Czech Republic, Poland, Hungary and the five other former communist countries that joined the European Union last year. But there were also some disastrous failures: Yugoslavia, the whole of Soviet Central Asia, Chechnya and so on. There is no immutable law that says that casting off one form of tyranny will not lead to another, or even that it won't lead to war.

For optimists about democracy, however, recent events in the region now offer renewed hope. The move to democracy in Ukraine may yet inaugurate a wave of new revolutions, this time against post-communism, which could sweep away nasty little dictatorships like that of Alexander Lukashenko's Belarus and even reverse the trend toward authoritarianism in Vladimir Putin's Russia. Nonetheless, no honest observer could deny that the complexities of democratization in the former communist world have been vast or that there is still much to be done. It will not be easier in the Middle East.

Then again, to quote Shakespeare's King Lear, "Nothing will come of nothing." It has to start somewhere. For a president dubbed by his opponents as the stupidest man ever to have sat in the White House, Bush has shown a remarkable ability to push things in the right direction. This should serve as a warning to those whose response to everything related to George Bush is reflex hostility and visceral hatred. Many sections of the European and American Left made the same mistake with Ronald Reagan. They were not especially welcome at parties following the fall of the Berlin Wall in Eastern Europe, having opposed the policies of a man who helped make it all happen. If freedom really is on the march in the Arab world today -- and the least we can say is that the first steps have now been taken -- the risk is that this will be the second great democracy party at which such people will be left moping on the sidelines, wondering, once again, how they got it all so wrong.

Robin Shepherd is an adjunct fellow of the Center for Strategic and International Studies.

Choux rated this answer Excellent or Above Average Answer
purplewings rated this answer Excellent or Above Average Answer

Question/Answer
Choux asked on 03/05/05 - Ward Churchill on Bill Mayer's Show

Last night I was surprised when Bill Mayer introduced Ward Churchill at the beginning of his show! Out came Professor Churchill and the two sat side beside in front of the live audience. Mayer asked hiim if he was part Native American, and he said yes.

Bill reassured him that he was in friendly territory although he said that he didn't agree with everything Chruchill said. Then, he asked him to explain the thought process that went behind his "Little Eichmanns" comment.

I was shocked to see that Chruchill was tongue-tied. Bill tried another question, and Churchill had difficulty giving an answer to that!

Fiinally, Bill said that HE would explain Churchill's comments and background!!! (trying to save the show!)So, Bill attempted to explain some background. Finally, hurchill kinda tried to explain "techno- something" is why the people in the Twin Towers died and deserved to.

Bill then brought out the brother of a Twin Tower's victim...the whole thing was a disaster.

I was wondering....why couldn't Churchill get a word out???????

For those interested, the show eill be rerun on HBO several times during the coming week. Tune in and see the fiasco that was Churchill.

Puzzled...he couldn't get a sensible two sentences out together, WOW.

tomder55 answered on 03/06/05:

sorry for the clarification ;

Some other choice tidbits from the tongue-tied professah .Of course Maher also got into the America bash-fest ;even as he asked Churchill *tough questions* while at the same time sympathising with him for his being *Dixie-Chicked* When Churchill obviously could not make his case in an articulate manner Maher tried to do it for him : "Let me do it for you... There was the bringing over of the slaves... Then we're talking about the Indians in America, your people, you're part Indian" [Churchill nods approvingly ..someone finally believes that]..... "And they add up numbers." ..."So we have a lot of blood on our hands... So then you talk about the first Iraq war. How many died there?.... And then the sanctions... I know we don't want to hear this but the country of America has blood on its hands.... Not to mention in Germany and Japan when we were close to winning the war we obliterated Dresden...."Maher asks him whether fighting Hitler was wrong. Churchill said, "I wouldn't say that opposing Hitler was wrong but some of the ways he was opposed were wrong." Then Churchill says it is not the gvt. but the American people who are wrong. Maher: "Even if America committed these crimes I don't understand what you're saying. I don't understand how the average American can be as guilty as you say they should be."Churchill then started to babble about the "technological elite" in the WTC. Churchill says that if you were "performing a technical function... you were not innocent. You were performing a function" like Eichmann.

Then the disgraceful Maher brings out a family member of a 9-11 victim who SUPPORTS what Churchill has been saying
Maher continues : "I don't understand how you can compare the passive aggressive... We're lazy and arrogant and greedy and myopic, and all those things cause some misery around the world. But Eichmann was proactively killing people."Churchill responded with some mubo-jumbo about America displacing profits and"moving labor to sweatshops in [Malaysia] you're doing things comparable to what Eichmann did."Churchill then says that the relatives brother was not a little Eichmann [sort of ] but then he bizzarly asks that if he wasn't guilty ,there must've been an "imaculate genocide"..HUH ???????????????? this guy is teaching our kids !!!!!!!!????????.

Maher the A.H. closes by suggesting that they build a "Why they hate us "pavillion at the WTC site . Over all a virtuo performance by two clowns.

Choux rated this answer Excellent or Above Average Answer
ETWolverine rated this answer Excellent or Above Average Answer
purplewings rated this answer Excellent or Above Average Answer

Question/Answer
HANK1 asked on 03/04/05 - Power:



Should our government give much more power to the States?

HANK

tomder55 answered on 03/04/05:

The Fed. powers are spelled out ,the States presumably have the remaining powers .As Excon said it is not for the Feds to grant to the States. I am a little wary of the term 'States Rights' however as that terminology have been used for years by racists and for thje sake of discrimination ;but that is a human failing . The theory behind decentralized power is sound. Tension between local and national power is another healthy check and balance .

HANK1 rated this answer Excellent or Above Average Answer

Question/Answer
ETWolverine asked on 03/04/05 - MAN WHO GOT EICHMANN DIES

From the NY Post, 3/3/05

Peter Malkin, the Mossad agent who nabbed top Nazi official Adolf Eichmann on a Buenos Aires street in 1960, has died, Israeli media reported yesterday. He died in New York at 77.

The Mossad security agency tracked Eichmann to Argentina, and Malkin stopped him in the street. According to his memoirs, "Eichmann in My Hands," Malkin said to him simply, "Un momentito, seor" (just a moment, sir), before kidnapping him.

Those were the only words Maklin knew in Spanish, according to a Web site of the World Zionist Organization. He grabbed Eichmann's arm and wrestled him to the ground as another agent grabbed his legs, and they stuffed him into a car.

Eichmann was interrogated for 10 days in a safe house before being spirited to Israel on a plane that carried an unwitting diplomat, Abba Eban, later Israel's foreign minister, for a meeting with Argentine officials as a cover.

Eichmann headed the "final solution," the plan to exterminate Jews. AP


--------------

And now, for an Op Ed piece on Malkin, also from the Post, 3/3/05.

BETTER THAN BOND
BY ERIC FETTMANN

ZVI Malchin was not only the single greatest secret agent the state of Israel was lucky enough to produce, he was one of the most extraordinary people one could ever hope to meet.

The world knew him as Peter Z. Malkin, the man who on a cold night in 1960 kidnapped a factory worker named Riccardo Klement outside his ramshackle Buenos Aires home and brought him to Israel where he stood public trial as Adolf Eichmann, the Nazi official who saw to it that 6 million Jews were murdered efficiently.

Yet that was just one of hundreds of exploits undertaken by Malchin who died here Tuesday night at age 75 during more than a quarter-century with Israeli intelligence, first as an agent and, ultimately, as chief of operations.

Even today, nearly 30 years after his retirement, many of Malchin's most spectacular achievements remain hidden behind the veil of official Israeli censorship. But enough of what he did is publicly known to leave you in awe that one man could have accomplished so much.

Malchin unmasked Israel Be'er, one of the top aides to then-Prime Minister David Ben-Gurion, as a Soviet spy. He bugged a meeting of Arab League heads of states. He uncovered former Nazi scientists who had gone to work for Egypt in the 1950s. He battled Palestinian terrorism in Beirut.

Even in retirement, he outdid active agents.

During the 70s, he went to Brazil in search of Josef Mengele, the infamous Nazi doctor of Auschwitz. While there, he uncovered a Soviet agent who was bribing Brazilian army officers and buying U.S. Army materiel.

"He asked me to notify the CIA," recalled Manhattan DA Robert Morgenthau, who frequently used Malchin as a freelance investigator. "I said he didn't have enough evidence. So he went back and did a black-bag job on the guy came up with his passport and his visa.

"I called Stanley Sporkin, then general counsel of the CIA. Within hours, the agency had sent people to New York to take the evidence. 'We know all about this guy,' Sporkin told me. 'We just had no idea where the hell he was.' "

How did Malchin know? "After all these years," he told Morgenthau, "I can smell them."

His career alone was exceptional enough. But Zvika, as he was known to his friends, was not some Hollywood stereotype of a secret agent.

Malchin was an artist, whose stunning paintings including a series of sketches done while he was interrogating Eichmann in Argentina in recent years have been exhibited in leading museums around the world. (He divided his time between Israel, Florida and New York he kept a studio here on the Lower East Side and lived in the East 30s.)

"Was being a painter my cover story for the Mossad, or was being in the Mossad my cover story for being a painter? Sometimes, I'm not sure, he joked." (His art can be seen as peterzmalkin.com.)

Malchin was a poet an irrepressibly funny and always eloquent observer of the human condition. He enthralled audiences around the world as a lecturer; they all wanted to hear how he'd captured Eichmann, but he had much more to say to them.

It's wrong, he would say, to call Eichmann a monster. After all, "a monster can be excused for his behavior. A human being, though how does a human being become a beast who can kill children, women, the elderly? The problem is not how a monster could do it, but how a human being did it."

"He was a brilliant analyst," said Morgenthau. "For him, information was always much more important than action." No doubt, that's what attracted early Israeli leaders to Malchin by age 13, he'd been recruited into the pre-statehood Haganah underground.

And though he saw more than his share of genuine dramatic action, Zvika had little regard for popular fiction's idea of a spy. "In 28 years, I never killed anyone," he said. "My most important weapon wasn't a gun it was my brain."

That, and his engaging, larger than life, personality which allowed him to talk his way out of some 40 arrests or detentions during his career.

Though he loved being acclaimed for his achievements, he didn't go seeking glory. He was a curious mixture of humility and pride who never demanded recognition, but was always happy to receive it.

You couldn't help being hopelessly charmed by such a man; everyone who met him wanted to spend more time with him. His friends, and I was lucky to count myself as one for more than 25 years, were fiercely loyal.

Last night, many of those friends gathered at the Park East Synagogue to say goodbye to a man who was literally a legend in his lifetime.

Today, he is headed on his final journey back to Israel, the country and people he served so well.

---------

I had the opportunity and privledge to meat Zvi Malchin at one of his lectures a few years back. Aside from being genuinely funny, and an eloquent speaker (in 3 languages that I heard), he was also a truly nice person.

We lost one of the good ones on Wednesday.

Baruch dayan ha'emes (Blessed is the True Judge--- this a a Jewish prayer/saying after learning of a death).

Elliot

tomder55 answered on 03/04/05:

he did great service for the world .his kind is rare ;they leave the world a better place than they inherit.

ETWolverine rated this answer Excellent or Above Average Answer
Choux rated this answer Excellent or Above Average Answer

Question/Answer
HANK1 asked on 03/04/05 - The Supreme Court:



Does the United States of America need a Supreme Court?

HANK

tomder55 answered on 03/04/05:

yes ;it definitely needs one to act as a co-equal branch of gvt. as the framers intended it to do . Suggested reading ; Men In Black by Mark Levin. "The biggest myth about judges is that they're somehow imbued with greater insight, wisdom, and vision than the rest of us, that for some reason God Almighty has endowed them with superior judgment about justice and fairness,"but they are an important cog in the balance of power .

ETWolverine rated this answer Excellent or Above Average Answer
HANK1 rated this answer Excellent or Above Average Answer

Question/Answer
kindj asked on 03/03/05 - Rumor

This is a rumor that just won't seem to go away. What I post below is only one source that says the same thing as many others.

You folks have any thoughts on it?

Has U.S. threatened
to vaporize Mecca?
Intelligence expert says nuke option is reason bin Laden has been quiet

--------------------------------------------------------------------------------
Posted: January 7, 2005
1:00 a.m. Eastern



2005 WorldNetDaily.com

Why hasn't Osama bin Laden's terror network executed an attack on U.S. soil since 9-11?

Simple, says Dr. Jack Wheeler, creator of an acclaimed intelligence website dubbed "the oasis for rational conservatives": The U.S. has threatened to nuke the Muslim holy city of Mecca should the terror leader strike America again.

On his website, To the Point, Wheeler explains how the Bush administration has identified the potential of wiping Mecca off the map as bin Laden's ultimate point of vulnerability the Damoclean Sword hanging over his head.


"Israel recognizes that the Aswan Dam is Egypt's Damoclean Sword," writes Wheeler. "There is no possibility whatever of Egypt's winning a war with Israel, for if Aswan is blown, all of inhabited Egypt is under 20 feet of water. Once the Israelis made this clear to the Egyptians, the possibility of any future Egyptian attack on Israel like that of 1948, 1967, and 1972 is gone."

Wheeler says talk of bin Laden's Damoclean Sword has infiltrated the Beltway.

Writes Wheeler in his members-only column: "There has been a rumor floating in the Washington ether for some time now that George Bush has figured out what Sword of Damocles is suspended over Osama bin Laden's head. It's whispered among Capitol Hill staffers on the intel and armed services committees; White House NSC (National Security Council) members clam up tight if you begin to hint at it; and State Department neo-cons love to give their liberal counterparts cardiac arrhythmia by elliptically conversing about it in their presence.

"The whispers and hints and ellipses are getting louder now because the rumor explains the inexplicable: Why hasn't there been a repeat of 9-11? How can it be that after this unimaginable tragedy and Osama's constant threats of another, we have gone over three years without a single terrorist attack on American soil?"

Available only to subscribers of To the Point, Wheeler ends his column by explaining the effectiveness of the Mecca threat.

"Completely obliterating the terrorists' holiest of holies, rendering what is for them the world's most sacred spot a radioactive hole in the ground is retribution of biblical proportions and those are the only proportions that will do the job.

"Osama would have laughed off such a threat, given his view that Americans are wussies who cut and run after a few losses, such as Lebanon in 1983 and Somalia in 1993. Part of Bush's rationale for invading Afghanistan and Iraq obviously never expressed publicly was to convince Osama that his threat to nuke Mecca was real. Osama hates America just as much as ever, but he is laughing no more."

Wheeler says bin Laden is "playing poker with a Texas cowboy holding the nuclear aces," so there's nothing al-Qaida could do that could come remotely close to risking obliterating Mecca.

Writes Wheeler: "So far, Osama has decided not to see if GW is bluffing. Smart move."

tomder55 answered on 03/03/05:

I don't think the terrorists give a rats ass about the holy places . It doesn't stop them from waging jihad from minarets . I would think in OBLs warped mind that an attack on Mecca would be the spark he is looking for for his global jihad movement to galvanize. I'm suprised he hasn't bombed it himself.

Choux rated this answer Excellent or Above Average Answer
ETWolverine rated this answer Excellent or Above Average Answer
excon rated this answer Excellent or Above Average Answer
kindj rated this answer Excellent or Above Average Answer

Question/Answer
ETWolverine asked on 03/01/05 - Death Penalty for Minors

The US Supreme Court just came down with a decision in Roper v. Simmons, in which they have decided that it is illegal to apply the death penalty to those under the age of 18 at the time their crimes were committed.

Kennedy wrote the Court's opinion, with Stevens, Souter, Ginsberg, and Breyer concuring. O'Connor wrote a lone dissenting opinion, and Scalia wrote a separate dissenting opinion to which Rehnquist and Thomas joined.

The basis of the Court's decision was twofold: 1) there is a consensus in the USA that the death penalty for minors is wrong in all cases, and 2) there is an international consensus that the death penalty is wrong for minors. The Court applied these consesnsi to the 8th Amendment against Cruel and Unusual Punishment. They said that the definition of Cruel and Unusual Punishment is subject to change as the consensus of opinion regarding appropriate behavior changes. Since, the Court claimed, there was a national and international consensus against death penalty for minors, they ruled against such a death penalty for minors. The Court further cited several studies that said that minors as a general rule are too immature to be deterred by a death penalty or to understand the consequences of their actions as support for their decision.

O'Connor dissented. She argued that there is no clear consensus, and therefore there is no way to determine that the definition of Cruel and Unusual Punishment has changed to include a death penalty for minors. She further claims that while it is true that many teens are too immature to be deterred by a death penalty or understand the consequences of their actions, there are those minors who ARE sufficiently mature, and judgements should be made on a case-by-case basis.

Scalia, Thomas and Rehnquist also dissented, with Scalia writting. Instead of summarizing Scalia, I want to cite his opinion. I think it gives the flavor of his words much better than I ever could. (I have eliminated the citations within his opinion to shorten this post.)

Hang on to your hats folks. This is going to be a long one...

JUSTICE SCALIA,with whom THE CHIEF JUSTICE and
JUSTICE THOMAS join,dissenting.

In urging approval of a constitution that gave life-
tenured judges the power to nullify laws enacted by the peoples representatives,Alexander Hamilton assured the citizens of New York that there was little risk in this, since [t ]he judiciary ...ha [s ] neither FORCE nor WILL but merely judgment. But Hamilton had in mind a traditional judiciary,bound down by strict rules and precedents which serve to define and point out their duty in every particular case that comes before them. Bound down, indeed. What a mockery today s opinion makes of Hamilton s expectation,announcing the Courts conclusion that the meaning of our Constitution has changed over the past 15 years not,mind you,that this Courts decision 15 years ago was wrong ,but that the Constitution has changed .

The Court reaches this implausible result by purporting to advert,not to the original meaning of the Eighth Amendment,but to the evolving standards of decency,(internal quotation marks omitted),of our national society.It then finds,on the flimsiest of grounds, that a national consensus which could not be perceived in our people s laws barely 15 years ago now solidly exists.Worse still,the Court says in so many words that what our people s laws say about the issue does not,in the last analysis,matter:[I ]n the end our own judgment will be brought to bear on the question of the acceptability of the death penalty under the Eighth Amendment. (internal quotation marks omitted). The Court thus proclaims itself sole arbiter of our Nation s moral standards and in the course of discharging that awesome responsibility purports to take guidance from the views of foreign courts and legislatures. Because I do not believe that the meaning of our Eighth Amendment, any more than the meaning of other provisions of our Constitution, should be determined by the subjective views of five Members of this Court and like-minded foreigners, I dissent.

I
In determining that capital punishment of offenders
who committed murder before age 18 is cruel and un-
usual under the Eighth Amendment,,the Court first
considers,in accordance with our modern (though in my
view mistaken)jurisprudence,whether there is a na-
tional consensus,ibid.(internal quotation marks omit-
ted),that laws allowing such executions contravene our
modern standards of decency,1. We have held that this determination should be based on objective indicia that reflect the public attitude toward a given sanction namely,statutes passed by society s elected representatives. (internal quotation marks omitted). As in Atkins v.Virginia, the Court dutifully recites this test and claims halfheartedly that a national consensus has emerged since our decision in Stanford ,because 18 States or 47% of States that permit capital punishment now have legislation prohibiting the execution of offenders under 18,and because all of four States have adopted such legislation since Stanford. Words have no meaning if the views of less than 50% of death penalty States can constitute a national consensus.

Our previous cases have required overwhelming opposi-
tion to a challenged practice,generally over a long period of time.In Coker v.Georgia, a plurality concluded the Eighth Amendment prohibited capital punishment for rape of an adult woman where only one jurisdiction authorized such punishment.The plurality also observed that [a]t no time in the last 50 years ha[d] a majority of States authorized death as a punishment for rape. In Ford v.Wainwright, we held execution of the insane unconstitutional, tracing the roots of this prohibition to the common law and noting that no State in the union permits the execution of the insane. In Enmund v.Florida, we invalidated capital punishment imposed for participation in a robbery in which an accomplice committed murder, because 78% of all death penalty States prohibited this punishment.Even there we expressed some hesitation,because the legislative judgment was neither wholly unanimous among state legislatures,...nor as compelling as the legislative judgments considered in Coker . By contrast,agreement among 42% of death penalty States in Stanford ,which the Court appears to believe was correctly decided at the time, was insufficient to show a national consensus.

In an attempt to keep afloat its implausible assertion of national consensus,the Court throws overboard a proposition well established in our Eighth Amendment jurisprudence.It should be observed, the Court says,,that the Stanford Court should have considered those States that had abandoned the death penalty altogether as part of the consensus against the juvenile death penalty ...;a State s decision to bar the death penalty altogether of necessity demonstrates a judgment that the death penalty is inappropriate for all offenders,including juveniles. The insinuation that the Court s new method of counting contradicts only the Stanford Court is misleading. None of our cases dealing with an alleged constitutional limitation upon the death penalty has counted,as States supporting a consensus in favor of that limitation,States that have eliminated the death penalty entirely. And with good reason. Consulting States that bar the death penalty concerning the necessity of making an exception to the penalty for offenders under 18 is rather like including old-order Amishmen in a consumer-preference poll on the electric car.Of course they don t like it,but that sheds no light whatever on the point at issue. That 12 States favor no executions says something about consensus against the death penalty, but nothing absolutely nothing about consensus that offenders under 18 deserve special immunity from such a penalty.In repealing the death penalty,those 12 States considered none of the factors that the Court puts forth as determinative of the issue before us today lower culpability of the young, inherent recklessness, lack of capacity for considered judgment, etc. What might be relevant,perhaps,is how many of those States permit 16-and 17-year-old offenders to be treated as adults with respect to non-capital offenses.(They all do;2 indeed, some even require that juveniles as young as 14 be tried as adults if they are charged with murder.3 ) The attempt by the Court to turn its remarkable minority consensus into a faux majority by
counting Amishmen is an act of nomological desperation.

Recognizing that its national-consensus argument was
weak compared with our earlier cases,the Atkins Court
found additional support in the fact that 16 States had
prohibited execution of mentally retarded individuals
since Penry v.Lynaugh. Indeed,the Atkins Court distinguished Stanford on that very ground,explaining that [a]lthough we decided Stanford on the same day as Penry ,apparently only two state legislatures have raised the threshold age for imposition of the death penalty. (emphasis added). Now, the Court says a legislative change in four States is significant enough to trigger a constitutional prohibition.4 It is amazing to think that this subtle shift in numbers can take the issue entirely off the table for legislative debate.

I also doubt whether many of the legislators who voted
to change the laws in those four States would have done so if they had known their decision would (by the pronouncement of this Court) be rendered irreversible. After all,legislative support for capital punishment, in any form,has surged and ebbed throughout our Nations history.

***Snip***

II
Of course,the real force driving today s decision is not the actions of four state legislatures,but the Courts own judgment that murderers younger than 18 can never be as morally culpable as older counterparts. The Court claims that this usurpation of the role of moral arbiter is simply a retur [n ] to the rul [e ]established in decisions predating Stanford, That supposed rule which is reflected solely in dicta and never once in a holding that purports to supplant the consensus of the American people with the Justices views 7 was repudiated in Stanford for the very good reason that it has no foundation in law or logic. If the Eighth Amendment set forth an ordinary rule of law,it would indeed be the role of this Court to say what the law is.But the Court having pronounced that the Eighth Amendment is an ever-changing reflection of the evolving standards of decency of our society, it makes no sense for the Justices then to prescribe those standards rather than discern them from the practices of our people.

On the evolving-standards hypothesis, the only legitimate function of this Court is to identify a
moral consensus of the American people. By what conceivable warrant can nine lawyers presume to be the
authoritative conscience of the Nation?8 The reason for insistence on legislative primacy is obvious and fundamental:[I ]n a democratic society legislatures, not courts,are constituted to respond to the will and
consequently the moral values of the people. For a similar reason we have,in our determination of society s moral standards,consulted the practices of sentencing juries: Juries maintain a link between contemporary community values and the penal system that this Court cannot claim for itself. Today s opinion provides a perfect example of why judges are ill equipped to make the type of legislative judgments the Court insists on making here. To support its opinion that States should be prohibited from imposing
the death penalty on anyone who committed murder before age 18,the Court looks to scientific and sociological studies,picking and choosing those that support its position. It never explains why those particular studies are methodologically sound;none was ever entered into evidence or tested in an adversarial proceeding. As THE CHIEF JUSTICE has explained:
[M ]ethodological and other errors can affect the reli
ability and validity of estimates about the opinions
and attitudes of a population derived from various
sampling techniques.Everything from variations in the survey methodology,such as the choice of the target population, the sampling design used,the questions asked, and the statistical analyses used to interpret the data can skew the results.

In other words,all the Court has done today,to borrow
from another context,is to look over the heads of the
crowd and pick out its friends.

We need not look far to find studies contradicting the
Court s conclusions.

***Snip***

Given the nuances of scientific methodology and conflicting views,courts which can only consider the
limited evidence on the record before them are ill
equipped to determine which view of science is the right one. Legislatures are better qualified to weigh and evaluate the results of statistical studies in terms of their own local conditions and with a flexibility of approach that is not available to the courts. Even putting aside questions of methodology,the studies cited by the Court offer scant support for a categorical prohibition of the death penalty for murderers under 18.

At most,these studies conclude that,on average ,or in
most cases, persons under 18 are unable to take moralresponsibility for their actions. Not one of the cited studies opines that all individuals under 18 are unable to appreciate the nature of their crimes. Moreover,the cited studies describe only adolescents
who engage in risky or antisocial behavior,as many young people do. Murder,however,is more than just risky or antisocial behavior.It is entirely consistent to believe that young people often act impetuously and lack judgment, but,at the same time,to believe that those who commit premeditated murder are at least sometimes just as culpable as adults. Christopher Simmons,who was only seven months shy of his 18th birthday when he murdered Shirley Crook,described to his friends beforehand [i ]n chilling,callous terms, as the Court puts it,,ante ,at 1 the murder he planned to commit.He then broke into the home of an innocent woman, bound her with duct tape and electrical wire,and threw her off a bridge alive and conscious. In their amici brief,the States of Alabama,Delaware,Oklahoma,Texas,Utah,and Virginia
offer additional examples of murders committed by individuals under 18 that involve truly monstrous acts. In Alabama,two 17-year-olds,one 16-year-old,and one 19-year-old picked up a female hitchhiker,threw bottles at her, and kicked and stomped her for approximately 30 minutes until she died. They then sexually assaulted her lifeless body and, when they were finished,threw her body off a cliff. They later returned to the crime scene to mutilate her corpse. Other examples in the brief are equally shocking. Though these cases are assuredly the exception rather than the rule, the studies the Court cites in no way justify a constitutional imperative that prevents legislatures and juries from treating exceptional cases in an exceptional way by determining that some murders are not just the acts of happy-go-lucky teenagers,but heinous crimes deserving of death.

***Snip***

The Court concludes,however,ante ,at 18,that juries cannot be trusted with the delicate task of weighing a defendants youth along with the other mitigating and
aggravating factors of his crime. This startling conclusion undermines the very foundations of our capital sentencing system,which entrusts juries with mak [ing ] the difficult and uniquely human judgments that defy codification and that buil[d ] discretion,,equity,and flexibility into a legal system. The Court says, that juries will be unable to appreciate the significance of a defendant s youth when faced with details of a brutal crime. This assertion is based on no evidence; to the contrary,the Court itself acknowledges that the execution of under-18 offenders is infrequent even in the States without a formal prohibition on executing juveniles, suggesting that juries take seriously their responsibility to weigh youth as a mitigating factor.

Nor does the Court suggest a stopping point for its
reasoning. If juries cannot make appropriate determinations in cases involving murderers under 18, in what other kinds of cases will the Court find jurors deficient? We have already held that no jury may consider whether a mentally deficient defendant can receive the death penalty, irrespective of his crime. Why not take other mitigating factors,such as considerations of childhood abuse or poverty, away from juries as well? Surely jurors overpower[ed ] by the brutality or cold-blooded nature of a crime could not adequately weigh these mitigating factors either.

The Court s contention that the goals of retribution and deterrence are not served by executing murderers under 18 is also transparently false. The argument that [r ]etribution is not proportional if the law s most severe penalty is imposed on one whose culpability or blameworthiness is diminished, is simply an extension of the earlier, false generalization that youth always defeats culpability. The Court claims that juveniles will be less susceptible to deterrence, because [t ]he likelihood that the teenage offender has made the kind of cost-benefit analysis that attaches any weight to the possibility of execution is so remote as to be virtually nonexistent. The Court unsurprisingly finds no support for this astounding proposition, save its own case law. The facts of this very case show the proposition to be false. Before committing the crime, Simmons encouraged his friends to join him byassuring them that they could get away with it because they were minors. This fact may have influenced the jury s decision to impose capital punishment despite Simmons age. Because the Court refuses to entertain the possibility that its own unsubstantiated generalization about juveniles could be
wrong,it ignores this evidence entirely.

III
Though the views of our own citizens are essentially irrelevant to the Court s decision today,the views of other countries and the so-called international community take center stage.

The Court begins by noting that Article 37 of the
United Nations Convention on the Rights of the Child, which every country in the world has ratified save for the United States and Somalia, contains an express prohibition on capital punishment for crimes committed by juveniles under 18. at 22 (emphasis added). The Court also discusses the International Covenant on Civil and Political Rights (ICCPR), December 19,1966 which the Senate ratified only subject to a reservation that reads:
The United States reserves the right,subject to its
Constitutional restraints,to impose capital punishment on any person (other than a pregnant woman) duly convicted under existing or future laws permitting the imposition of capital punishment, including such punishment for crime committed by persons below eighteen years of age.

Unless the Court has added to its arsenal the power to join and ratify treaties on behalf of the United States, I cannot see how this evidence favors,rather than refutes,its position. That the Senate and the President those actors our Constitution empowers to enter into treaties, -have declined to join and ratify treaties prohibiting execution of under-18 offenders can only suggest that our country has either not reached a national consensus on the question, or has reached a consensus contrary to what the Court announces. That the reservation to the ICCPR was made in 1992 does not suggest otherwise, since the reservation still remains in place today. It is also worth noting that,in addition to barring the execution of under-18 offenders,the United Nations Convention on the Rights of the Child prohibits punishing them with life in prison without the possibility of release. If we are truly going to get in line with the international community, then the Court s reassurance that the death penalty is really not needed, since the punishment of life imprisonment without the
possibility of parole is itself a severe sanction gives little comfort.

It is interesting that whereas the Court is not content to accept what the States of our Federal Union say ,but insists on inquiring into what they do (specifically, whether they in fact apply the juvenile death penalty that their laws allow),the Court is quite willing to believe that every foreign nation of whatever tyrannical political makeup and with however subservient or incompetent a court system in fact adheres to a rule of no death penalty for offenders under 18. Nor does the Court inquire into how many of the countries that have the death penalty, but have forsworn (on paper at least) imposing that penalty on offenders under 18, have what no State of this country can constitutionally have: a mandatory death penalty for certain crimes, with no possibility of mitigation by the sentencing authority,for youth or any other reason. I suspect it is most of them. To forbid the death penalty for juveniles under such a system may be a good idea, but it says nothing about our system, in which the sentencing authority, typically a jury, always can, and almost always does, withhold the death penalty from an under-18 offender except, after considering all the circumstances, in the rare cases where it is warranted. The foreign authorities, in other words,do not even speak to the issue before us here.

More fundamentally,however,the basic premise of the
Court s argument that American law should conform to
the laws of the rest of the world ought to be rejected out of hand. In fact the Court itself does not believe it. In many significant respects the laws of most other countries differ from our law including not only such explicit provisions of our Constitution as the right to jury trial and grand jury indictment, but even many interpretations of the Constitution prescribed by this Court itself. The Court-pronounced exclusionary rule, for example,is distinctively American. When we adopted that rule in Mapp v. Ohio, it was unique to American Jurisprudence. Since then a categorical exclusionary rule has been universally rejected by other countries, including those with rules prohibiting illegal searches and police misconduct, despite the fact that none of these countries appears to have any alternative form of discipline for police that is effective in preventing search violations. England, for example, rarely excludes evidence found during an illegal search or seizure and has only recently begun excluding evidence from illegally obtained confessions. Canada rarely excludes evidence and will only do so if admission will bring the administration of justice into disrepute. The European Court of Human Rights has held that introduction of illegally seized evidence does not violate the fair trial requirement in Article 6,1,of the European Convention on Human Rights.

The Court has been oblivious to the views of other countries when deciding how to interpret our Constitution s requirement that Congress shall make no law respecting an establishment of religion.... Amdt.1. Most other countries including those committed to religious neutrality do not insist on the degree of separation between church and state that this Court requires. For example, whereas we have recognized special Establishment Clause dangers where the government makes direct money payments to sectarian institutions, countries such as the Netherlands, Germany,and Australia allow direct government funding of religious schools on the ground that the state can only be truly neutral between secular and religious perspectives if it does not dominate the provision of so key a service as education,and makes it possible for people to exercise their right of religious expression within the context of public funding. England permits the teaching of religion in state schools. Even in France,which is considered America s only rival in strictness of church-state separation, [t ]he practice of contracting for educational services provided by Catholic schools is very widespread.

And let us not forget the Court s abortion jurisprudence, which makes us one of only six countries that allow abortion on demand until the point of viability. Though the Government and
amici in cases following Roe v.Wade,urged the Court to follow the international community s lead,these arguments fell on deaf ears.

The Court s special reliance on the laws of the United
Kingdom is perhaps the most indefensible part of its
opinion. It is of course true that we share a common
history with the United Kingdom,and that we often consult English sources when asked to discern the meaning of a constitutional text written against the backdrop of 18th- century English law and legal thought. If we applied that approach today,our task would be an easy one. As we explained in Harmelin v.Michigan, the Cruell and Unusuall Punishments provision of the English Declaration of Rights was originally meant to describe those punishments out of [the Judges ] Power that is,those punishments that were not authorized by common law or statute, but that were nonetheless administered by the Crown or the Crown s judges. Under that reasoning,the death penalty for under-18 offenders would easily survive this challenge. The Court has,however I think wrongly long rejected a purely originalist approach to our Eighth Amendment, and that is certainly not the approach the Court takes today. Instead, the Court undertakes the majestic task of determining (and thereby prescribing) our Nation s current standards of decency. It is beyond comprehension why we should look, for that purpose,to a country that has developed,in the centuries since the Revolutionary War and with increasing speed since the United Kingdom s recent submission to the jurisprudence of European courts dominated by continental jurists a legal, political,and social culture quite different from our own. If we took the Court s directive seriously, we would also consider relaxing our double jeopardy prohibition, since the British Law Commission recently published a report that would significantly extend the rights of the prosecution to appeal cases where an acquittal was the result of a judge s ruling that was legally incorrect. We would also
curtail our right to jury trial in criminal cases since, despite the jury system s deep roots in our shared common law, England now permits all but the most serious offenders to be tried by magistrates without a jury.

The Court should either profess its willingness to reconsider all these matters in light of the views of foreigners, or else it should cease putting forth foreigners views as part of the reasoned basis of its decisions. To invoke alien law when it agrees with one s own thinking, and ignore it otherwise,is not reasoned decisionmaking, but sophistry.

The Court responds that [i]t does not lessen our fidelity to the Constitution or our pride in its origins to acknowledge that the express affirmation of certain fundamental rights by other nations and peoples simply underscores the centrality of those same rights within our own heritage of freedom. To begin with, I do not believe that approval by other nations and peoples should buttress our commitment to American principles any more than (what should logically follow) disapproval by other nations and peoples should weaken that commitment. More importantly,however,the Court s statement flatly misdescribes what is going on here. Foreign sources are cited today,not to underscore our fidelity to the Constitution, our pride in its origins, and our own [American ] heritage.. To the contrary, they are cited to set aside the centuries-old American practice a practice still engaged in by a large majority of the relevant States of letting a jury of 12 citizens decide whether,in the particular case, youth should be the basis for withholding the
death penalty. What these foreign sources affirm, rather than repudiate, is the Justices own notion of how the world ought to be, and their diktat that it shall be so henceforth in America. The Court s parting attempt to downplay the significance of its extensive discussion of foreign law is unconvincing. Acknowledgment of foreign approval has no place in the legal opinion of this Court unless it is part of the basis for the Court s judgment
which is surely what it parades as today.

IV
To add insult to injury,the Court affirms the Missouri
Supreme Court without even admonishing that court for
its flagrant disregard of our precedent in Stanford. Until today,we have always held that it is this Courts prerogative alone to overrule one of its precedents. That has been true even where changes in judicial doctrine ha[ve ] significantly undermined our prior holding, and even where our
prior holding appears to rest on reasons rejected in some other line of decisions,. Today, however,the Court silently approves a state-court decision that blatantly rejected controlling precedent.

One must admit that the Missouri Supreme Court s
action,and this Court s indulgent reaction,are,in a way, understandable. In a system based upon constitutional and statutory text democratically adopted, the concept of law ordinarily signifies that particular words have a fixed meaning.Such law does not change,and this Court s pronouncement of it therefore remains authoritative until (confessing our prior error) we overrule. The Court has purported to make of the Eighth Amendment, however, a mirror of the passing and changing sentiment of American society regarding penology. The lower courts can look into
that mirror as well as we can; and what we saw 15 years
ago bears no necessary relationship to what they see
today. Since they are not looking at the same text, but at a different scene, why should our earlier decision control their judgment?

However sound philosophically, this is no way to run a
legal system. We must disregard the new reality that, to the extent our Eighth Amendment decisions constitute
something more than a show of hands on the current Justices current personal views about penology, they
purport to be nothing more than a snapshot of American
public opinion at a particular point in time (with the
timeframes now shortened to a mere 15 years). We must
treat these decisions just as though they represented real law,real prescriptions democratically adopted by the American people,as conclusively (rather than equentially) construed by this Court. Allowing lower courts to reinterpret the Eighth Amendment whenever they decide enough time has passed for a new snapshot leaves this Court s decisions without any force especially since the evolution of our Eighth Amendment is no longer determined by objective criteria. To allow lower courts to behave as we do, updating the Eighth Amendment as needed, destroys stability and makes our case law an unreliable basis for the designing of laws by citizens and their representatives,and for action by public officials. The result will be to crown arbitrariness with chaos.

-------------

I just love the snideness of Scalia's comments. It fits my own opinions of the ruling perfectly.

What are your opinions of the ruling and the dissenting opinions? All can be found here:

Roper v. Simmons

Elliot

tomder55 answered on 03/02/05:

I spent much of last night researching this case and did intend to post this morning on the same topic. I think full appreciation of the case cannot be had until one reads the details of the crime committed.

Simmons ;a 17 year old puke (9 months before his 18th birthday .He premeditively planned to commit burglary and murder with 2 others. He assured his friends that they could get away with it because they were minors. Simmons and his friend Charles Benjamin entered the home of Shirley Crook about 2 AM .

They bound her mouth and eyes with duct tape ;put her in a minivan and drove to a State Park .They bound her hands and feet with wire ;covered the rest of her head with duct tape ,and threw her over a railroad trestle into the Meramec river to die .

Simmons went on to brag about the killing to his friends telling them that he killed her because "the bitch seen my face" .When the police arrested him he waived Miranda rights and confessed ;giving the police a video reenactment. He was quite proud of himself .


The hypocrisy in Kennedy's decision is mind boggling. 15 years ago Kennedy voted to confirm death penalties for under 18 years. What changed ?;certainly not the Constitution.The New York Sun in an editorial nailed it when they said :His opinion in yesterday's case relied heavily on what he termed "the laws of other countries" and "international authorities." That is a perilous path, for, as Justice Scalia pointed out in his dissent, most of those other countries also don't apply the exclusionary rule barring illegally obtained evidence, and most of them don't have as full rights to abortion as America does. The justices seem only to use international standards when it fits their purposes.(and speaking about abortion ; if teens are not held to adult standards then why do lawmakers balk at parental notification ?They don't have the mental ability to be fully responsible for a crime yet they are able to make a choice on their own about having an abortion.)

When the new principle of 'evolving standards' are applied to the justice system ,are the same standards to be expected by the criminal? Some of the most disgusting; sickening; heinous crimes are committed by gang members; the majority of whom are under the age of 18. Are we to give them carte blanc to do the crime; knowing the worse they will get is a truncated time in jail? One of the criminals that got spared yesterday was on death row in Texas . He committed murder 1 day shy of his 18 birthday.

Where does this so called 'national consensus ' come from? The Wall Street Journal says : His evidence for this "consensus" is that of the 38 states that permit capital punishment, 18 have laws prohibiting the execution of murderers under the age of 18. As we do the math, that's a minority of 47% of those states. The dozen states that have no death penalty offer no views about special immunity for juveniles -- and all 12 permit 16- and 17-year-olds to be treated as adults when charged with non-capital offenses.

This idea of invoking state laws to define a "consensus" also runs up against any number of notable Supreme Court precedents, including Roe v. Wade. When Roe was decided in 1973, all 50 states had some prohibition against abortion on the books. But never mind.
(http://online.wsj.com/article/1,,SB110972710932567833,00.html?mod=TOPIC)

having computer problems so I stop this now ,but you get the point.

ETWolverine rated this answer Excellent or Above Average Answer
excon rated this answer Excellent or Above Average Answer
Itsdb rated this answer Excellent or Above Average Answer

Question/Answer
ETWolverine asked on 02/28/05 - An interesting point.

Hello everyone.

I'm not much of a fan of Andrew Wilkow's talk-radio show, but I heard him yesterday (Sun 2/27/04), and he made an intersting point about Social Security reform and Personal Social Security Accounts that I wanted to pass on.

Why is it that the same Liberals who argue "Keep your laws off my body, keep your laws out of my bedroom, choice choice choice," reject the Conservative argument for social security reform of "Keep your laws off my money, keep your hands out of my wallet, choice, choice, choice"? Isn't it essentially the same argument? Why do they argue in favor of free choice for abortions and gay marriage, but reject free choice for how people plan for their retirements?

Its a good point, and I'm wondering if anyone on the liberal side of the spectrum has a response.

Elliot

tomder55 answered on 02/28/05:

I tend to agree with excon on this . Both sides say things that are diametric to what they do . The best example is probably this notion of Conservatives being champions of fiscal responsibility and small government. The Republicans of the 90s were proposing sweeping reforms that would eliminate whole departments of the Federal Government(including Dept. of Education ) . Now when they are in a position of majority you rarely hear anything as "radical" as that .(by contrast today less emphasis is placed on privatizing education and more on NCLB which is if anything places a bigger emphasis on Federal mandates) .

As far as S.S. is concerned ;it did not take long for the Republicans to start back tracking on it . Serious negotiations have not even begun yet and the President has signalled a willingness to raise taxes(increasing the payroll tax cap) ,and Congressional Republicans are already signaling a willingness to reduce the percentage of income set aside for personal accounts.I have not heard of any reciprocal concessions by the Democrats . They have stated that their playbook is to dig their heals in the sand and obstruct .[http://www.washingtonpost.com/wp-dyn/articles/A56464-2005Feb26.html?sub=AR ]

ETWolverine rated this answer Excellent or Above Average Answer
excon rated this answer Excellent or Above Average Answer

Question/Answer
Choux asked on 02/26/05 - Mubarak calls for Elections

Today, on my ISP, there was a news story about Mubarak calling for limited elections in Egypt. Mybarak has ruled Egypt since 1981 after the assassination of Awar Sadat.

The success of the democratic process in Iraq has not escaped all the dictators in the Middle East. The average muslim is NOT a child who needs a firm dictator; he/she is a person who wants freedom/democracy and THEY KNOW IT.

All the world knows it, specially anti-American European appeasers to terrorism.

Comments?

tomder55 answered on 02/27/05:

Interesting ;this comes at the same time that Condi Rice abruptly canceled a trip to Egypt to protest the arrest of political opponent to Mubarak; Ayman Nour, a member of thebparliament and head of an opposition party called Al-Ghad (tomorrow).When Egyptian Foreign Minister Ahmed Aboul Gheit visited Washington last week, Rice made her displeasure clear.[He must've been shakin in his boots when he saw Condi in her boots]

The United States however called the announcement of a constitutional change as welcoming news ,and opposition groups have called this historic .Ther is little doubt that pressure from America is the cause of this change.

``This concession is made to the United States of America. It is better for him (Mubarak) if this decision came as a result of the national dialogue with the opposition parties and in response to the protests against the law,'' she said. ``Let us wait and see, because a free campaign of more than one candidate requires more than a statement from the president.'' said
Activist Aida Seif el-Dawla


I have my suspicions .Any candidate planning on running has to belong to an official political party.They also need approval from parliament as well. Mubarak's party has a solid majority in the Parliament.Any major challenger could simply be denied outright while less threatening opponents would still be allowed to run.

The proof will be in the process. If viable candidates are allowed to run against Mubarak then the reforms will be real. As it stands now,the U.S. should continue to pressure Egypt to open up it's process and not be fooled by this apparent p.r. move .I will be more impressed if he relaeses Nour and allows him to run.

Mubarak should not be afraid to open the process. If he offeres real democratic reforms ;perhaps expanding his idea to open up the electoral process for Parlament;he could actuall legitmately win ,and go down in Egptian history as the Father of Democracy.

Choux rated this answer Excellent or Above Average Answer

Question/Answer
kindj asked on 02/23/05 - Rush goes to Afghanistan

As you may or may not know, Rush Limbaugh took a trip over to Afghanistan to visit the troops and such. Here's a bit from his report:

>>Limbaugh said he was humbled by the experience of meeting the troops on the front lines. "I can talk about them on the radio program as you do and we salute them and so forth," he told Hedgecock. "But I just felt so small compared to these people standing before them."

Afghanistan, he said, was still a "hellhole" after years of Taliban rule and tribal fighting. "I've never seen a place like this. I guess bombed-out Dresden in World War II might look somewhat like it."

Still, said Limbaugh, the Afghan people seemed to realize that their future was brighter thanks to American intervention.

"I met a bunch of native Afghanistan people today who their big fear is that the U.S. will leave," he explained. "They're so excited about the changes in the elections that took place that they're really jazzed."

In a town hall meeting with the troops, the top talker said he got only had a couple policy questions about Afghanistan. "One of them was, 'Do you think that the US system should be what they should adopt or do you think we should allow them to adopt their own system?'"

Limbaugh replied that he wouldn't try to impose an American-style government on the country. "I trust freedom. I trust free people. Let free people make up their own minds about things and you can trust the results."<<

What do you think about that last paragraph? Do you think the same thing would be true in Iraq?

DK

tomder55 answered on 02/24/05:

I think that was the whole point of the Bush doctrine from the beggining . No one in the administration ever suggested we would impose our type of gvt. on them. The leap of faith is that a free people will chose to remain free.

ETWolverine rated this answer Excellent or Above Average Answer
excon rated this answer Excellent or Above Average Answer
kindj rated this answer Excellent or Above Average Answer

Question/Answer
sapphire630 asked on 02/23/05 - changing the subject for a minute

The things I previously said to take for cancer:
The combination of shark cartiledge and CoQ10 blocks cancer cells from being fed. Also good--Essaic, astragulus, periwinkle, antioxidents and greentea.


High blood pressure:
garlic
quercetin (found in onions)
green tea
antioxidents (blueberries, tomatoes, red wine)
omega fatty acids
Vitamins E and B complex

iron poor blood:
Vitamin K -(blood thickener)
vitamin E
liver and molasses


Now to tie in somewhat:
The medical field knows herbal supplements are better than drugs, but they make better money with drugs. Economy is more important than humans lives. They keep it hushed and deny the problems until they are at an undeniable rate. Like one kid killing when he is on Zoloft is nothing until it becomes an undeniable epidemic. Just like Prozac in the early 90's they denied any link to the side effects until hundredS had killed themselves.
Drs. have even admitted to me they do not know. It is like a guessing game or Russian roulette. They can not even figure out a lot of things and only give out medicine to mask the symptoms. Morgellons disease is something they say they have no idea what is causing it. Some alternative drs. say it is a parasite type thing found mostly in bottled water.

Many people are tired of it and going to alternative med. drs. who get to the root of the ailment and cure it.

Doctors are good for check ups, checking on the progression of symptoms or checking healing of problem, and injuries.



tomder55 answered on 02/24/05:

The green tea should be decaffinated . I also think the claims about shark cartilige are dubious. They have not passed the test of real clinical research .

The field of dietary supplements is an important one that is sometimes overlooked .I do think your claim about profits has some merit ;although there is plenty of money to be made in the field .In the U.S., the dietary supplement industry is a $20 billion industry. The pharmaceutical industry has tended to concentrate on cures .A drug will correct a symptom ,but does not address preventive care generally(yes there are some exceptions like taking aspirin as a heart attack preventative).

Scientific research on the associations between supplements and health is accumulating rapidly.The theory behind supplements is that the standard diet of the modern world is lacking in basic preventive nutrients ,or that if they are present they are dilluted by food processing . Everyone has heard the quote 'an apple a day keeps the doctor away '. Supplement research trys to find out why; isolate the benifical compounds ;put it in a pill ,and sell it to you so that you get the effects without necessarily eating the apple. In my view this is a 2nd best option ,but it is better than nothing .A healthy diet should be premium to good health.

Other items like herbs and amino acids are also part of the indusrty . There is a growing understanding that there was something behind traditional remedies . Chamomile was used for ages as a calmative and as a cold remedy . No one knew why it worked ;it just did . Now science is able to research ,and extract the useful components from this herb ;concentrate it ,and produce a natural non-chemical alternative to cold relief .

The pharmaceutical companies have seen the writing on the wall. In many cases they are buying supplement companies to get into the market. They are also introducing new products that have supplements in them . Unfortunately I also think they are for a large extent behind the move to regulate the industry as tight as the pharmaceutical industy is .I think additional regulations are necessary ,and the responsible players in the industry agree;however there is no reason at all to regulate a vitamin C tablet the same way you would production of digitalis. All they would accoplish is make a relatively inexpensive alternative health care approach prohibitivly expensive .(disclaimer :I have ben employed in both pharmacutical and the supplement industry)

sapphire630 rated this answer Excellent or Above Average Answer

Question/Answer
Itsdb asked on 02/23/05 - What's wrong with this picture?

NYC Kids Send Anti-War Letters to Soldier
Tuesday, February 22, 2005
By David Andreatta

The New York City Department of Education, red-faced over Brooklyn sixth-graders who slammed a GI with demoralizing anti-Iraq-war letters as part of a school assignment, will send the 20-year-old private a letter of apology Tuesday.

Deputy Schools Chancellor Carmen Farina, who has a nephew serving in Iraq, plans to personally contact Pfc. Rob Jacobs and his family, said department spokeswoman Michele McManus Higgins.

"She knows how difficult it is to have a loved one in a war zone," Higgins said.

Jacobs is stationed 10 miles from the North Korean border and who has been told he may be headed to Iraq in the near future.

The GI got the ranting missives last month from pint-sized pen pals at JHS 51 in Park Slope.

Filled with political diatribes, the letters predict GIs will die by the tens of thousands, accuse soldiers of killing Iraqi civilians and bash President Bush.

Teacher Alex Kunhardt had his students write Jacobs as part of a social-studies assignment.

He declined to comment Monday on whether he read the rants before passing them along, but said he planned to contact Jacobs soon to explain the situation.

In an accompanying letter to Jacobs, Kunhardt had written that the students "come from a variety of backgrounds and political beliefs, but unanimously support the bravery and sacrifice of American soldiers around the world."

"Support" was not the word that came to Jacobs' mind when he read the letters.

One girl wrote that she believes Jacobs is "being forced to kill innocent people" and challenged him to name an Iraqi terrorist, concluding, "I know I can't."

Another girl wrote, "I strongly feel this war is pointless," while a classmate predicted that because Bush was re-elected, "only 50 or 100 [soldiers] will survive."

A boy accused soldiers of "destroying holy places like mosques."

Even one kid smitten with soldiers couldn't keep politics out of the picture, writing, "I find that many extreme liberals are disrespectful to you."

Uplifting letters from children are dear to soldiers, Jacobs said. He looks at a batch he got from a Girl Scout troop from his hometown of Middletown, N.J., whenever he feels lonely.

At the time the 21 JHS 51 letters were penned, Jacobs, who has been stationed in Korea for nearly a year, was told that he may be headed to Iraq. But no official order for deployment was given.

"If I were in Iraq and read that the youth of our nation doesn't want me to be there and doesn't believe in what I'm doing, it would mess up my head," Jacobs said.

Jacobs said he would welcome a letter from the Department of Education and the teacher.

"I want to think these letters were coached by the teacher or the parents of these children," Jacobs said in an interview from Camp Casey, Korea.

"It boggles my mind that children could think this stuff."

*******************************************************

I don't want to implicate the teacher...yet, but shouldn't he at least have told the kids hey, this guys is in Korea, not Iraq, that he is only doing his duty, and perhaps that the letters should be encouraging?

Do you think they were coached by parents, or that maybe the media had a role in influencing these kids, or that the mayor is right, who said something like we don't censor our kids? Can you imagine how this guy felt, expecting encouragement from home and being told by a sixth grader he is "being forced to kill innocent people"? What is wrong with this world???

tomder55 answered on 02/23/05:

I discussed this last night with my wife who is also a teacher. She said that the children should be instructed at that age to 'say something nice or not at all'.If the teacher did not preview the letters and vett them than shame on him . He is not doing his job.

The mayor is wrong .The children do not understand how hurtful and discouraging these types of letters could be . The children should be made aware that it is not the role of the soldier to make policy. If the children have an issue with the policy then they should be encouraged to write to politicians and leaders . That would help nurture a sense of civic participation .

These children did not form these views on their own . Perhaps they are parroting their parents ;or perhaps their teacher ;or even perhaps the propensity of the MSM to never show positive news from Iraq .

If I were the soldier I would write letters to the children thanking them for their correspondence ,and try to convey to them ;especially to the young boy who apparently is Muslim ;that what they are trying to do is to protect them and to try to help the people of the countries where the wars are being waged to achieve freedom .

ETWolverine rated this answer Excellent or Above Average Answer
Itsdb rated this answer Excellent or Above Average Answer

Question/Answer
excon asked on 02/21/05 - Goodbye Gonzo


Hello experts:

Hunter S. Thompson said something like "I don't want to encourage drugs, alcohol or insanity, but they've always worked for me".

That's a man after my own heart.

excon

tomder55 answered on 02/22/05:

What madness possessed him to take his own life ? perhaps this little bit from an interview with Mathew Hahn of 'The Atlantic Monthly gives a little clue :

MH: How is your health? How are you feeling now?
HST: I haven't started any savings accounts.... I tell you, you'd act differently if you thought you were going to die at noon tomorrow. You probably wouldn't be here doing this. I just figured, "Bye, bye, Miss American Pie, good old boys drinkin' whiskey and rye, singin' this'll be the day that I die." Yeah, I just felt that all along.

MH: Live every day like your last, because you don't know what tomorrow's going to be like?

HST: Well, there's no plan for it. It's like going into the 27th inning in a baseball game. You're like, what the f**k am I doing here, man?





I read one of his books ;it was about the 72 election . He was making alot of fun of Hubert H Humphrey pandering to everyone. Thompson ended up endorsing McGovern . He claimed to be an opponent of big Gvt. but ususally endorsed or supported big gvt. candidates in the U.S. and around the world like Fidel Castro few countries hoave as mush total control by the State as Cuba).

The news is touting him as one of the leading journalists of the counter culture. I have frankly read much more from Ken Kesey and Jack Kerouac ,and think they will be the standard bearers of that generation if anything from then is lasting .

excon rated this answer Excellent or Above Average Answer

Question/Answer
excon asked on 02/20/05 - Hypocracy!


Hello experts:

Why is it, that the last two American Presidents both have both smoked pot, but still think you should go to jail for it?

Isn't there something, unbelievably screwed up about this scenario? (I know, Elliot, not to you).

excon

tomder55 answered on 02/20/05:

One sort of admitted it .The other had a conversation with a skunk who recorded the conversation violating his trust .He also in the tape did not admit to it ,but I think both ,and Al Gore ;and a whole generation of people ;or a significant portion of them ,did indeed use pot. I never thought usage was worthy of jail time unless it was a contibuting factor in say an automobile related crime (intoxication behind a wheel ,which most pot smokers who drive eventually are guilty of ).But I do think it should be illegal .I agree with States that have some kind of decriminalization laws or conditional release laws . When I used it ,I accepted the risks.I assume they did too.

excon rated this answer Excellent or Above Average Answer

Question/Answer
Choux asked on 02/20/05 - Unlimited REsources

Many people, including myself, think of America having unlimited resourses. Now, I think that is *not true* although I have no statistics, no results of projections.

Primarily, what about the healthcare industry. What a disaster!

One point. How long can we spend huge amounts of money on saving fetus' and newborn at the cost of millions of dollars EACH?

How long can we continue pouring millions of dollars into 'useless care' for people over say 75? Example:: replacing two knees on an 80 year old man. At lease one nation has guidelines in place; they have limited resources.

Just to get me thinking.

Does America have limitless resources going forward????

How can we think about the morality of how we spend our money??

tomder55 answered on 02/20/05:

Does it make sense to abort and then invite guest workers into the country because there are not enough workers to do the jobs needed ?

I do not think it is moral to deny health care .What you should be asking ..is it moral for an elder person to use the resourses of the nation for frivoulous care.( I do not think your knee replacement example qualifies but Medicare for Viagra certainly does .)I remember that Colorado Guv. Richard Lamm once stirred up controversy by saying that it is a moral responsibility to make room for the young. As leaves fall from the trees in the fall, so old people have a duty to die. I would not go that far ,but I have already made a living will that says I do not desire to get extrordinary care. As I get older I do not wish to be a burder ;financial or otherwise on my daughter . I like to believe I would take a walk in the woods .

Choux rated this answer Excellent or Above Average Answer

Question/Answer
excon asked on 02/20/05 - Social Security

Hello,

I agree with Bush. Ok, you can get up now, Elliot. People should be responsible for themselves. My question is this, what are we going to do for those who arent, and those who are, but lose their money?

Nothing? They had their chance and blew it? Homelessness isnt so bad. If were going to offer a safety net (and we are, Elliot, no matter what you think), then why dont we just leave the safety net called SS in place?

excon

tomder55 answered on 02/20/05:

The system is obscenely broken .I have no problem with the safety net idea ,and if the government was running it by sound economic principles then I would say leave it alone. But they are not ;they are plundering the money and giving you alot of b.s. about the number of workers it takes to fund your retirement ;now at 3 workers for one retiree .The truth is that for the average ;and even the slightly below average paid worker ;the system should be self sustaining 1:1 .

Some math : an 18 year old worker averaging $30,0000 per year over the course of their employment puts away just 4% (the max.amt. the president's plan allows as opposed to the 6.2% of the workers and another 6.2 % of that worker's employer contribution into the current system)of income and invests it into the same gvt. securities available to the gvt. now (T bill and gvt security funds average about 6% return over the 47 years between 18 and 65 ).That worker at retirement will has a nest egg of $318,000 .That worker could continue to leave it in this safe investment and still draw a monthly income from that at around $800 /month(more than S.S. promises that worker currently even though that worker will have invested less money than the S.S. witholdings currently steal) . When that worker passes on ;it goes to that workers inheritance ;not to the gvt.

Now let's say this same worker was to invest it into a common mutual fund portfolio (which in any given 40 year period in history has netted about 10%)
That worker can expect to have a nest egg over $1,800,000 to draw retirement from. That's security .

The alternative plan being floated by the opponents calls for the cap cut off of contributions to be raised from its current $90,000 . Will that fix it ? Not .Why ? Because the gvt. will not be doing anything different with the money .They will still put it into a fictional trust fund. Why is it fictional ? Because they plunder from it and purchase gvt. securities with it ,and use the revenue from the securities to finance the operations of government. What are those securities backed by ?? The full faith of the US government . All raising the ceiling would accomplish would be allowing the gvt. to throw more I.O.U.s into the "trust fund" .(They already made this mistake in Medicare ...has the system been any better as a result ?)Recently the president has floated this idea ;and I say to him ."I read your lips ". The one thing Im not open-minded about is raising the payroll tax rate. And all the other issues go on the table. All that increase in tax promises is a worse deal for young workers. They are promised a system where they pay more ,and get less (if the system isn't completely collapsed by the time they retire eventually the gender wars in this country would boil over. )

I fully expect reform to continue to provide that safety net. But the calculations that were used for the $30,000 avearge worker work the same no matter what income you plug in . The fact is that it is a better deal for workers to invest their money than to have big incompetent brother doing it for them .





ETWolverine rated this answer Excellent or Above Average Answer
excon rated this answer Excellent or Above Average Answer

Question/Answer
CeeBee2 asked on 02/18/05 - Mexico, our 51st state.

The other day at the public library where I work, one of my volunteers, who is from Brownsville, TX suggested that adding Mexico as the 51st state (or make several states out of Mexico) would solve problems not only for Mexico but also for our country. What do you think?

tomder55 answered on 02/18/05:

as opposed to the current reality ;that being Mexico trying to reconquista the American South West by infiltration ?

CeeBee2 rated this answer Excellent or Above Average Answer
Choux rated this answer Excellent or Above Average Answer
excon rated this answer Excellent or Above Average Answer
kindj rated this answer Excellent or Above Average Answer

Question/Answer
excon asked on 02/18/05 - Robert Novak


Hello:

Why is he not being pressured to reveal his sources? He revealed the same information as others who are in jail.

Do you believe that reporters should be able to conceal their sources, as the Constitution says? Do you believe the president should plant ringers in the White House Press Corps? Are any of these activities of your dude linked? Do they indicate any sort of cohesive policy regarding the relationship between Bush and the press?

Is it wrong for him to have 1) jailed reporters for not revealing their sources, 2) planted fake reporters IN the White House, and 3) PAID reporters for their services?

excon

tomder55 answered on 02/18/05:

too much for a single post . for time sake I will deal with the Novak issue :

I do not think that Novak should have been required to reveal his source ;I don't think that Judith Miller and Matthew Cooper should either ...why ? ...because I do not think it was wrong to mention that Plame was a low level CIA employee who's conflict of interest was the real story .

The Intelligence Identities Protection Act wasn't violated .To violate it the CIA person has to be an undercover operative which Plame wasn't and the leaker had to know that disclosure would hurt national security ..which it didn't .The "special prosecutor "which the stupid Dems. called for; is over reaching .there is no case here .

But ;if there was ;then I do not think the press is exempted from the rule of law. Thirty-two years ago, the Supreme Court held that the First Amendment does not create a journalists privilege: like anyone else, journalists must testify when ordered to do so.(UCLA School of Law professor Eugene Volokh )The case he was talking about is Branzburg v. Hayes, 408 U.S. 665 (1972)


As for Novak ;I'd be willing to wager that he has already sung his tune to the investigators ;which is why he has noty been called to the carpet .

ETWolverine rated this answer Excellent or Above Average Answer
excon rated this answer Excellent or Above Average Answer
Itsdb rated this answer Excellent or Above Average Answer

Question/Answer
excon asked on 02/17/05 - Proposition


Hello experts:

I suggest that the Pharmaceutical Industry of today is equivalent to the Tobacco Industry of years past.

They are bald faced liars. They have foisted drugs upon us that are killing us, that they knew would kill us, and then they lied about knowing it. They are comprised of despicable human beings.

Will their friend in the White House protect them?

rexcon


tomder55 answered on 02/17/05:

The FDA is certainly in their pockets. David Graham a scientist at the FDA was going to present the as-yet-unpublished information about the heart risk of pain relievers to a panel of health experts at a panel discussion about safety issues with pain relievers like Merck's Vioxx. But Graham said that an e-mail from his supervisor said that if he continued to press for inclusion of the new data he would be doing so at his own risk.

I do not know all the answers .The system sometimes allows dangerous drugs on the market ,and other times prevents valuable ones that deserve a streamlining to the market from approval.My guess is that if aspirin was a new drug it would take years for approval and then would be classified prescription .

I am frankly suprised that a big scandal hasn't already erupted about the cozy relationship between the indusry and the Agency. I have seen company execs. get nailed by the 3rd world like systems of drug approvals /bribes /and short-cut taking that was allowed by FDA corrupted FDA inspectors .






full disclosure :
I ,and my wife have worked in the industry. There is alot of good that they do ;but they are too prone to dismiss natural remedies ,and tend to concentrate on finding cures to existing illness rather than looking for preventatives . I have since been employed in the natural alternative/nutraceutical industry .

excon rated this answer Excellent or Above Average Answer

Question/Answer
sapphire630 asked on 02/16/05 - How can they win

when in a catch 22?
I forget the soliders name but they are trying him for murder because he killed 2 terrorists. It wasn't the incident in the mosque but a different one. They said he continued to fight for 3 months until a 'disgruntled fellow solider' complained.

What is the use of anybody enlisting and going to fight where they can end up charged with murder! It is stupid! If these people that cry murder are the same ones that cry to 'talk it out and make peace' why don't they go over there and do just that if it is that simple!?

tomder55 answered on 02/17/05:

The case involving Lt. Ilario Pantano is a complex one . On appearance it looks like a travesty . His mother set up a web site about the incident involving the charges (Defend the defenders .org ) to raise money for his defense .The Washington Times has a good summary of the charges .

I really do not know what to think about the case .On the surface it sounds like a bum rap ;but I'm confident that the truth will come out in testimony . There were plenty of witnesses.

Choux rated this answer Excellent or Above Average Answer
sapphire630 rated this answer Excellent or Above Average Answer

Question/Answer
excon asked on 02/12/05 - Medicare


Hello experts:

Ok, I'll admit it. I'm dumb. I don't understand the Medicare prescription drug bills and controversy. Hep me out.

$720 billion is a lot of scratch. The bill is either a boon to seniors or a boon to the drug companies. Which one is it?

Don't tell me both, I won't believe it. And, you should forgive me if my pre-disposition in the matter is to suggest that the beneficiary of this bill is the pharmaceutical industry - not poor old folks.

I'd be happy to find that I'm wrong, and that Bush is really a good guy after all.

excon

tomder55 answered on 02/13/05:

It's great politics ;all you have to do is say over and over ;"prescription drug benefit for seniors" and you lock in the senior vote.If AARP wants it they get it. Unfortunately, it's not responsible . The plan is over-costly, but nobody cares because it's the young taxpayers;with families and young children to raise who have to pay for it.As much as people characterize our older generation as people struggling to get by, the fact is they are by far the wealthiest segment of our population. Why we need a subsidy program for a group of people who largely don't need it is beyond me. I am sure there are plenty that it will mean a lot to.But three out of four seniors , had some form of drug coverage from a variety of sources before this plan.The "access" problem for prescription drug coverage affected only a minority of seniors who were poor and could not afford or did not have access to private, employer-based, or Medicaid coverage.

But instead of solving that problem , Congress and Bush insisted on creating a costly and unnecessary universal drug entitlement that will displace existing spending with new federal spending and disrupt most seniors existing coverage.
I would make the whole program needs to "means tested" .

Oh and ;life style drugs like Viagra ....no way !

You are correct that it will be a boon for the Pharmaceutical Industry. It will block most efforts to obtain drugs from Canada.

I would repeal the plan and work on making perscrptions available for poor elderly .

Choux rated this answer Excellent or Above Average Answer
excon rated this answer Excellent or Above Average Answer
purplewings rated this answer Excellent or Above Average Answer

Question/Answer
excon asked on 02/11/05 - Outsourcing Torture


Hello fellow Americans:

According to a compelling and deeply disturbing article in the current issue of The New Yorker, Jane Mayer tells the following story:

Aher Arar is a 34-year-old native of Syria who immigrated to Canada as a teenager. On Sept. 26, 2002, as he was returning from a family vacation in Tunisia, he was seized by American authorities at Kennedy Airport in New York.

Mr. Arar, a Canadian citizen, was not charged with a crime, but was placed in handcuffs and leg irons by plainclothes officials and transferred to an executive jet. He was being taken, on the orders of the U.S. government, to Syria, where he would be tortured.

He did confess (who wouldn't?), but the confession under torture was worthless. Syrian officials reported back to the United States that they could find no links between Mr. Arar and terrorism. He was released in October 2003 without ever being charged and is now back in Canada.

The following are MY comments:

Any government that commits, condones, promotes or fosters torture is a malignant force in the world. And those who refuse to raise their voices against something as clearly evil as torture are enablers, if not collaborators.

How in the world did we become a country in which gays' getting married is considered an abomination, but torture is O.K.?

excon

tomder55 answered on 02/11/05:

"Extraordinary Rendition" was a Clinton creation, and was approved by Clinton's lawyers, with no objection from either the Justice Department or from anyone in Congressional oversight committees.

I have also read .Maher Arar's statement
of events .It does not sound pleasant.

The reason that the US acted on Maher Arar is that Canadas CSIS (domestic intel service ) were survailing Arar for quite some time. They had serious concerns about him. CSIS made the recommendation that the US deport Arar. CSIS still believed that Arar was a terrorist and made recommendations to the Canadian government not to intervene even after he ended up in Syria.

I have asked repeatedly what interogation techniques are acceptable ,and have never received a response. Clearly this is not right ,and the practice should be stopped. (He will probably win his law suit against the U.S. also.)If we are going to use aggressive techniques then they should be administered by American personnel under American law that is defined and agreed upon . If we are too squeemish to do it ourselves ,it probably crosses the line.

Choux rated this answer Excellent or Above Average Answer
drgade rated this answer Excellent or Above Average Answer
excon rated this answer Excellent or Above Average Answer

Question/Answer
ETWolverine asked on 02/10/05 - The statements of Lt. General Jim Mattis

Hi everyone.

I was wondering what your feelings were regarding the statements made recently by Lt. Gen. Jim Mattis.

At a recent military conference in San Diego, he gave a speach in which he was discussing (among other things) the fact that the enemy s not yet broken, and their will to fight is still strong.

He said: "Don't patronize this enemy. They mean business. They mean every word they say. Don't imagine an enemy somewhere in the future and you're going to transform so you can fight him. They're killing us now. Their will is not broken."

He followed that with the following comments:

"Actually, it's quite fun to fight'em, you know. It's a hell of a hoot. It's fun to shoot some people. I'll be right up front with you. I like brawling."

The general then made clear who "some people" are. "You go into Afghanistan. You've got guys who slapped women around for five years because they didn't wear a veil. You know guys like that ain't got no manhood left anyway. So it's a hell of a lot of fun to shoot them."

Naturally there are now certain elements calling for Gen. Mattis' head.

---------------

Now... here's my take on it.

He's 100% right. War is terrible, but it can be fun too. Or perhaps not fun, but certainly satisfying.

I have never been in combat. But I have spent quite a bit of time learning how to fight. I have nearly 30 years of martial arts experience. Fighting IS fun. There's no time that you feel quite as alive as when someone is trying to hit you, and you are doing the same to them. There's the enjoyment of the technique itself, the block-and-counter properly executed, the kick that connects where it was supposed to. And there's the fear. There's nothing quite as exhilerating as when your opponent's attack ALMOST takes your head off. The adrenaline rush is AMAZING, and it feels GOOD. And of course, if you win, that's pretty damn cool too.

I've only been in two real fights in my life outside the dojo. Both times were much faster than any spar session in the dojo. They started and ended much quicker. But the adrenaline high was bigger too. Nothing focusses one's mind quite as well as death... or the possibility of it. (I think Ben Franklin said that, but I'm not sure. Whoever said it was right.)

So I can understand the idea of combat being fun and enjoyable, despite the horror. As big as that enjoyment is with hand-to-hand combat, I suspect that it's even more true in all-out war. And its all the more enjoyable if the sonofabitch you are killing actually DESERVES it.

Sherman said "War is hell". Years later, Patton said "War is hell, but G-d help me, I love it." He was right about that.

Perhaps the best saying ever on the subject was from Robert E. Lee. He said "It is good that war is so terrible, lest we grow too fond of it."

These military men understood that war is enjoyable. It is eerie to think that, but it's true. Being more powerful than your enemy is thrilling. Knowing that you hold the power of life and death in your hands is thrilling. A close miss by enemy fire is thrilling. And killing a man who is trying to kill you is thrilling.

Civilized people don't like to believe that war is fun. It's not PC to say it. Only savages get their rocks off on war. But it's true, and Mattis, a Marine's Marine understands that. Mattis is more than a soldier. He's a fighter. He's a LEADER among fighters. And right now we need more than just soldiers. We need fighters, and Mattis is one of the best.

Back in the Civil War, there was a particular general who was running up particularly high casualty rates. Someone demanded that Lincoln remove that general and replace him with another. Lincoln's response was "I can't spare this man; he fights."

That's my response to the PC crowd that want's Mattis' head: We can't afford to lose Jim Mattis. He fights. He's blunt, he's not PC, and he's not the type of guy to bring home to momma. But he understands war, and by G-d, the man can fight. We need him.

That's my opinion. What's yours.

Elliot

tomder55 answered on 02/10/05:

can't say it any better than Patton :

" want you to remember that no bastard ever won a war by dying for his country. He won it by making the other poor, dumb bastard die for his country.

Men, all this stuff you've heard about America not wanting to fight, wanting to stay out of the war is a lot of horse dung. Americans, traditionally, love to fight. All real Americans love the sting of battle.

When you were kids you all admired the champion marble shooter, the fastest runner, big league ball player, toughest boxer. Americans love a winner and will not tolerate a loser. Americans play to win all the time. I wouldn't give a hoot in hell for a man who lost and laughed. That's why Americans have never lost and will never lose a war, because the very thought of losing is hateful to Americans.

Now, an army is a team. It lives, eats, sleeps, fights as a team. This individuality stuff is a bunch of crap. The bilious bastards who wrote that stuff about individuality for the Saturday Evening Post don't know anything more about real battle than they do about fornicating.

Now we have the finest food, equipment, the best spirit, and the best men in the world. You know, by god I, I actually pity those poor bastards we're going up against, by god, I do. We're not just going to shoot the bastards; we're going to cut out their living guts and use them to grease the treads of our tanks. We're going to murder those lousy Hun bastards by the bushel.

Now, some of you boys, I know are wondering whether or not you'll chicken out under fire. Don't worry about that. I can assure you that you will all do your duty.

The Nazis are the enemy. Wade into them. Spill their blood. Shoot them in the belly. When you put your hand into a bunch a goo that a moment before was your best friend's face, you'll know what to do.

Now there's another thing I want you to remember: I don't want to get any messages that we are holding our position. We're not holding anything. Let the Hun do that. We are advancing constantly and we're not interested into holding onto anything except the enemy. We're going to hold onto him by the nose and we're going to kick him in the ass. We're going to kick the hell out of him all the time and we're going to go through him like crap through a goose.

Now, there's one thing that you men will be able to say when you get back home. And you may thank God for it. Thirty years from now when youre sitting around your fireside with your grandson on your knee, and he asks you what did you do in the great World War II? You won't have to say, "Well, I shoveled shit in Louisiana."

Alright, now you sons-a-bitches, you know how I feel. I will be proud to lead you wonderful guys into battle anytime, anywhere.

That's all."

btw . Lincoln refused to dismiss Grant after Shiloh when he made that comment.

ETWolverine rated this answer Excellent or Above Average Answer

Question/Answer
Choux asked on 02/09/05 - Wade Chruchill

Are you familiar with the situation in Colorado and Professor Wade Churchill? Do you think that this Professor should be fired for his views and stances?

What about free speech and tenure and the world of ideas?

tomder55 answered on 02/10/05:

Ward Churchill is an employee of the school ,and the school is State funded . So his 'free speech argument doesn't fly. He is an example of an epedimic . As a parent who is about to spend a small fortune in the next few years to send my daughter to an institution of higher indoctrination I now know that U.of Colorado is one of the institutes that she will not be attending on my dime(she is not planning on majoring in 'Ethnic Studies'{whatever that means}anyway).For what it's worth ;if they send the usual propaganda that fills my mail box from recruting offices I will certainly inform them why they are not in our consideration .

I think it ironic that all types of "hate speech "is tolerated in many campuses around the country 'but in many other cases " free speech "is not . So this is how the system works ;a buffoon professor under the guise of "academic freedom "can spew all types of lies ;distortions;and hate ;some students ;the ones who have the correct opinions ;also can sponsor forums of hate-fest (see recent events at Columbia University In the course of the past 20 or 30 years, Columbia has shed its most eminent scholars in the humanities and social sciences by deaths and retirements. To replace them they have hired an incredible number of politically correct Multiculturalists, Marxists, ;terrorist sympathizers and revolutionary theorists . )

But on most of these same campuses students are denied the right of "free expression ".
Many of these institutes have adopted codes or policies prohibiting speech that offends any group based on "race, gender, ethnicity, religion or sexual orientation." (Last year Roger Williams College froze publishing funds for a conservative newsletter after it published articles deemed by some to be "hostile" to others.)

Churchill's business is ideas and the expression and transmission of those ideas to the students . The fact that I disagree with Mr. Churchill is not itself justification for his termination. But that his ideas are garbage is . Using the argument that we cannot censure him for expressing his views is nonsense.Those ideas and expressions, are all that he brings to the table and that is what he must measure him against.I find this the eqivalent of a medical professor;a math teacher or any other of the hard fact disciplines giving a student garbage information. The Constitution doesn't require the government to subsidize stupidity . This is not a matter of whether he has the right to say what he says;it is only an issue if the State is required to pay for his pablum.

ETWolverine rated this answer Excellent or Above Average Answer
Choux rated this answer Excellent or Above Average Answer

Question/Answer
excon asked on 02/07/05 - Ethics???? Talk about twisted!!!


Hello experts:

When asked about a pilot program in Canada to distribute free heroine to long term addicts, David Murry, an aid to drug czar John Walters, said it is "medically unethical".

I suppose it's more medically ethical to let them catch AIDS or Hepatitis and die on the street. Its medically ethical to ignore hordes of women selling their bodies for drugs. Or better yet, it's medically ethical to throw them into a hell hole where they'll be corn holed and beaten up a lot.

Yeah, that's much more medically ethical......

Are you unable to see that the emperor has no clothes?

excon

tomder55 answered on 02/07/05:

Would giving a free pass to a Casino to a gambling addict be ethical ? ;How about a bottle of scotch to an alcoholic? Obviously a better proposal would have the addicts get rehab ,and if that means getting a dose of heroin ;then it could be structured for them to get decreasing levels of the drug until they are drug free. I have heard that this is a better alternative than methadone therapy which is currently being administered in the U.S .

The Canadians are also planning on putting tatoo parlors in their prisons at a cost of $3.7-million to the tax payers because they are afraid the inmates will use their dirty heroin needles to tatoo temselves .

excon rated this answer Excellent or Above Average Answer
Itsdb rated this answer Excellent or Above Average Answer
powderpuff rated this answer Excellent or Above Average Answer

Question/Answer
Choux asked on 02/05/05 - Iraqi Govt Takes Initiative

On Fox News today, it was reported that the Iraqi Govt has taken a bold initiative in fighting terrorism.

They have begun televising before and after videos of captured terrorists. That is, one terrorist who was videoed in one of the decapitating videos was captured and during his interrogation, he was videoed crying and begging for his life.

I did not see the videos, but this psy op plan is expected to buck up the Iraqi people seeing....the violent death impulse terrorists as sniveling crybabies.

Let us hope that the Iraqi people feel empowered aganst these violent cowards.

tomder55 answered on 02/05/05:

I heard the report also . Give them courage .Nice to see pix of terrorists crawling out of spider holes !


other news ;

Sunni groups are coming in from the cold and are offering to help map out a new constitution.

Condi Rice got Germany's Chancellor Gerhard Schroeder to commit to helping rebuild Iraq.

Iyad Allawi's party is not doing well in election results and he will probably not be the PM in the new gvt.The leading candidate to replace him may be Adel Abdel Mahdi, the current Finance Minister .

Another suprise in the eledtion is how well more secular Shia parties are faring . It doesn't appear that any single party can dominate the new gvt. without building a coalition.

Choux rated this answer Excellent or Above Average Answer
ETWolverine rated this answer Excellent or Above Average Answer

Question/Answer
excon asked on 02/04/05 - Yeah, it's me again...


Hello experts:

I think we can be divided into two camps, here on the political board. There are those who support the Constitution unequivocally (and basic human rights for everyone), (thats me), and there are those who think that our present situation requires a suspension (if only temporarily) of basic human rights (thats you).

I believe what I do, not because I sympathize with how our government treats bad guys, but because I worry that, if we let them do it to the bad guys, then theyll do it to the good guys. (Or continue to do it, but thats another argument.)

My position stems from the belief that the present assault on the Constitution is not new. In spite of those assaults, that document has withstood the test of time and still stands erect (with minor changes) today. Im proud of that record and who we are (were).

Today, however, Im in fear for our cherished traditions of due process and equal rights. A government with the power to spirit people away and declare that's the end of the matter is exactly the kind of government that we always claimed to oppose. For us to become that kind of government is unbelievably scary.

Your president, along with his new Attorney General, is fighting for nothing less than the death of due process for anyone it rounds up, no matter how arbitrarily it did so, in its enemy combatant sweeps. Such tyrannical powers should offend anyone who cares about such old-fashioned notions as the rule of law, checks and balances, and constitutional guarantees.

I know, youre gonna give me a list of reasons why these people shouldnt be entitled to the same rights and protections you enjoy. Just l ike every time the Constitution was assaulted. And some of you will deny that what we are doing goes against anything. Thats bothersome.

But, what bothers me the most, is that I seem to be the only voice for the Constitution here on these boards.

excon

tomder55 answered on 02/04/05:

Zacarias Moussaoui (the "twentieth hijacker" ) has succeeded in turning his federal criminal trial into a due process circus. Do you really think that in WWII German and Japanese prisoners were subject to 4th 5th and 6th Amendment rights? Should they have receive Miranda warnings ? Conversely when we capture terrorists on the battle field are they entitled to only give their name ;rank and serial number ?

I do not think that the measure that were taken in the aftermath of 9-11 have been severe .Certainly there is historical precidence for much worse . The Constitution survived mass detentions ;suspension of habeus corpus;supression of dissent ( both Adams and Woodrow Wilson prosecuted those who argued against policy ) or the shutting down of the press(Lincoln had newspapers seized).Excesses of the past were not repeated.

Although I think that military commissions and tribunals should commence and happen at a more timely basis ,I find nothing unconstitutional in the concept . Roosevelt had simular ones set up for Nazi saboteurs caught in the U.S.

I agree that the Administration should not be the only branch of the gvt. involved in the decision about how to handle what is a unique situation ;this so called war on terrorism where we are dealing not with nation states but nationless entities that wage war against us. The Bush Adm. acted out of necessity of an immediate response . The courts are only now playing their role(and poorly at that ) ,and the Congress is MIA .Even though they could easily establish civilian 'special courts ' that would handle cases where issues of national security confict with an open system of judicial review.

Michael Chertoff has argued that what the system needs is structural reform .Rules governing the detention of terrorism suspects must be explicit and comprehensible. A definition of what is an "enemy combatant" is needed and what that special status means . What are the criteria by which an individual may be determined to be an enemy combatant? Who is authorized to review challenges ? What rights does an enemy combatant have? It is past time for the administration out of necessity going it alone on these matters. It is past time for fingerpointing about the deficiencies of a system that no one is offering a better alternative to .Congress and the Courts all have a role to play in our system . It is about time they do.





excon rated this answer Excellent or Above Average Answer

Question/Answer
darkstar asked on 02/03/05 - 12,000 for death compensation to our soldiers

When I heard that the usgov wants to change the death benifet for dieing while in service to our country is only $12,000 I was horrified. I couldn't believe that that was all our gov was willing to pay out to the families of these couragous men and women who risk their lives for our countries safty and place in the world....I mean come on, $12,000. That is such a slap in the face. I was glad to hear that a proposed increase to 100,000 was being considered. I think that is way more in line and maybe should even be higher than that. What are your feelings concerning this?

tomder55 answered on 02/03/05:

Certainly should be increased . . Truth be told ;both Clinton and Bush have tried to trim budgets on the back of military benefits and that was just not right .Recent bills have been rejected on this very issue . The Penatagon supports an increase and not even considering right or wrong; it seems that they need to increase incentives if the military has any chance of meeting recruiting goals.

The public is becoming more aware of the sacrifices military families have ,and will support an increase.When the nation was working together after 9-11 we raised about $1 million per family of the vicitims .

A bipartisan bill in the Senate cosponsored by Sen. Jeff Sessions (R) of Alabama, and Sen. Joseph Lieberman (D) of Connecticut,would increase the death benefits to $100,000. The proposed change would be retroactive to when the US entered Afghanistan. In addition, the government will pay for a $150,000 increase in life insurance coverage to $400,000. Let the gvt. trim the budget somewhere else. The military ;especially in a time of war;should not be treated as an after thought.

Choux rated this answer Excellent or Above Average Answer
darkstar rated this answer Excellent or Above Average Answer
ETWolverine rated this answer Excellent or Above Average Answer
Itsdb rated this answer Excellent or Above Average Answer
powderpuff rated this answer Excellent or Above Average Answer
purplewings rated this answer Excellent or Above Average Answer

Question/Answer
powderpuff asked on 02/02/05 - TSA

Could you improve on the wacky methods of the Transportation Security Administration?

Being a frequent flyer I recently found that it is no longer a coincidence or random happening, I am always flagged for 'additional screening' procedures when I go through security check points at our airports. The first few times, I thought it could be just a random thing. So this last time I asked, why is it always me? One of the guards gave me a nice little form letter with the greeting "Dear Traveler" and the explanation that my name is on their 'list'. So far it has not matched any name on the 'no fly' list, but it does match up with the 'selectee' list of names to be selected for additional screening procedures. My name matches a name on the 'watch list' "for persons who are known to pose, or are suspected of posing, a threat to civil aviation or national security." The result is that at every airport and every airline company, I am flagged for the complete work up with security screening. Each time I go through this procedure it is basically the same, though recent changes have made it a little less embarrassing for women. (they still check the breast area with their hands, but don't put their hands down between the breasts)

I have noticed that there seems to be way less people being screened with this 'additional screening' procedure than what is possible. At each security check point I've had to go through, there are at least six chairs and many many more security officers/guards. There are so many officers/guards in fact, that when they get me, they use at least 3 guards at the same time to screen me while most of the other available chairs in the 'additional screening' area sit empty and other security guards stand around looking like they don't have anything to do. While going through the intimidating military style process of 'additional screening', I watch as the other passengers sail through security without even getting a second look. Yes, even the ones who look scary don't seem to draw any attention what so ever.

I think someone should suggest a better way to find suspicious persons to screen. I'm not sure how or why my name matches up with a name that is, in fact, on any such list..... and I am working on trying to find out and clear up any confusion, but even so... The 'additional screening' procedure is not being used to the best of its ability, IMO. Because of the amount of unused chairs and unused security guards, I am led to believe that how they get people into the additional screening area is not by using a thought process. I think it must be some unintelligent coding technique that cannot use reason in its processing. This leaves me with a feeling of insecurity where the presence of security should give me the opposite feeling. It also causes me to believe that our airports are not nearly as secure as they should be or could be.

I have no criminal history of any kind including never even having had a speeding ticket. I'm not in the typical age group of terrorists, never learned how to fly a plane, and I am not in the typical gender class of a terrorist, (though I like to tell myself that I am very strong, in reality, I am not a physically strong person either). Yet because of a secret technique, my name matches a name that is on a 'watch list', so no matter how ridiculous it might seem, I am automatically flagged for the 'works'.

If you were in charge of finding ways to profile passengers for additional screening, what techniques would you use?

tomder55 answered on 02/03/05:

at least we know that you are not carrying a shoe bomb onto the plane. If you are a frequent flyer then you should get a gold or platinum flyer card so you can get on a VIP line to reach X-ray security checkpoints quicker. I had heard that they were going to offer biometric screening for frequent flyers. In fact it is in a pilot test program at various airports like Logan and Reagan airports .You would provide finger prints and/or iris scans ,and then go past the rest of the screening .A passenger usually becomes a part of the program by invitation. Airlines identify frequent travelers through frequent flyer programs, and invite passengers to join because it will help them get through security faster. Once a passenger agrees to become a part of the program, TSA runs a background check on them including credit history and criminal history. If there are no problems with the background check, the passenger is approved for the program and they are fingerprinted and the irises of their eyes are scanned. When they go to the airport, they go through the same initial security measures -- taking off coats and shoes and placing bags on the belt -- as other passengers, but at a separate biometric kiosk that verifies their identity. They will not be selected for further screening unless they set off the metal detector.

Of course the ACLU is opposed to this .

Itsdb rated this answer Excellent or Above Average Answer
powderpuff rated this answer Excellent or Above Average Answer

Question/Answer
ETWolverine asked on 02/02/05 - Desperation?

Are the terrorists getting desperate, or what?

NY POST
G.I. 'HOSTAGE' IS A REAL DOLL
By NILES LATHEM

WASHINGTON Master terrorist Abu Musab al-Zarqawi, meet your new hostage "Cody" the action figure.

In the wake of historic elections and top-level terror arrests, Zarqawi's minions yesterday claimed they had kidnapped an American soldier and posted his picture on a Web site.

"Our mujahedeen have managed to capture the American soldier, John Adam, after killing a number of his colleagues," said a Zarqawi-linked group called the Mujahedeen Squadrons on the terrorist group Ansar al-Islam's Web site.

A photo on the site appeared to show an African-American soldier sitting on a floor with a gun pointed at his head in front of a banner with the words "There is no God but God and Muhammad is his messenger" a standard Muslim profession of faith.

But the tied-up GI turned out to be a nothing more than a toy soldier.

Dragon Models USA, a California-based toy company, recognized the supposed American "hostage" as being one of their popular action figures called "Cody." It's sold on U.S. military bases in Kuwait.

"It is our doll. To me, it definitely looks like it is," said Dragon Models spokesman Liam Cusak.

"Everything the guy is wearing is exactly what comes with our figures," he added.

The "soldier" had no markings or insignia on his uniform and looked stiff and expressionless, sort of like, well, a doll.

U.S. military spokesmen initially took the posting seriously.

The Mujahedeen Squadrons has claimed responsibility for at least two other kidnappings. In this instance, they threatened to cut off the soldier's head in 72 hours if male and female prisoners were not released from U.S. prisons.

But military officials quickly realized that no soldiers were missing.

In the end, the taking of the toy soldier appeared to be an act of desperation by the terrorists or just mischievous hackers.

The members-only, password-protected Ansar al-Islam site is the place where Zarqawi first posts his communiqus and where videos of his group's grisly beheadings first surfaced, according to terrorism expert Rita Katz, who studies Islamic extremist Web sites.

Meanwhile, Iraqi President Ghazi al-Yawer said yesterday that it's too soon to start talking about the withdrawal of real American soldiers from Iraq, despite the success of the elections.

"It's only complete nonsense to ask troops to leave in this chaos and this vacuum of power," al-Yawer, a Sunni, told reporters.

He said he believes that by the end of the year once more Iraqi forces are trained there can be a reduction in U.S. and coalition troops.

http://www.nypost.com/news/worldnews/39295.htm

----------------

Better watch out. Someone might try to kidnap your Barbie and Ken dolls to use as hostages.

Pathetic.

Elliot

tomder55 answered on 02/02/05:

They already Captured Barbie ;and Major Matt Mason too !!! Oh NO !!

But on the positive side ;here is proof positive that OBL's been captured .



Here is G.I. Joe's P.O.W. diary :
19 January 2005: The Slinky betrayed us. I should have known. I never trusted him. He was an unstable character, always going back and forth, back and forth, never showing a shred of backbone. "Come, senor, I know the way to the insurgents' headquarters," he rasped. The fact that he was an Arab toy speaking with a stereotypical Spanish accent should have tipped me off. But hindsight is always 20/20. Literally. I can turn my head 360 degrees.


I only knew my men by their code names, but even in that short space of time we shared a bond that only six-inch plastic combatants can truly understand. They were my family, my brothers in petroleum-based products. One night we all melted the tips of our fingers and became plastic brothers.



And I led those brave action figures into the trap.



"My spider-sense is tingling," muttered "Peter Parker," as he flexed his fingers on his M16. We were all on edge, and our quirks were coming to the fore. "Prince Adam" kept waving his weapon in the air, hollering "By the power of Grayskull!" Damn Wiccans. "Hugh Jackman" had huddled deeply into his trenchcoat, whispering "Am I Wolverine or Van Helsing?" to anyone who made the mistake of standing next to him. And "Elmo" kept singing his goofy song. "Elmo loves his rifle/His bullets, too"



The insurgents caught us by surprise in that deserted Iraqi backyard. BBs perforated the sullen quiet of the hot Iraqi afternoon. Firecrackers sizzled and roared around us in a symphony of extremity-disintegrating horror. Mean little kids stomped us with the hard soles of their brand-new Keds -- weapons of mass destruction. And the gentlest one of us all lost it completely. "Elmo is thinking about genocide!" he screamed, as he unleashed a hail of foam darts upon our adversaries. "Elmo is Death, destroyer of worlds!" War does awful things to toys.



I tried to remember my training. My old drill sergeant, G.I. Joe, had put me through worse than this. "Are you gonna MOR yet, maggot?" he would scream, as he tied me to the wheel of a 10-speed Schwinn. (MOR: Melted On Request.) 'Sir, no, sir!" I would scream, even as the gravel scraped the paint off my face. He pushed me and prodded me, but he made me the action figure I am today. Just before Water Survival training, he gave me a piece of advice I'll always remember: "Son, when you get right down to it, you have no nerve endings." Then he flushed me down the toilet.



A repulsive splatting sound above my head brought me back to the present. "Gas! GAS!" We scrambled in vain for our gas masks as a haze of vaporous death descended upon us. Mustard gas? Try beans and broccoli. The last thing I remember was the leering visage of our hated enemy, the puppet master of al Qaeda, peering down on us.



The CIA lied. The bombs in Bora Bora hadn't killed him after all.



Evil Bert. The legends were true.



24 January 2005: The interrogators were relentless. But I gave them only my name, rank, and UPC code.



They mocked my fear. "It better here than American prison, yes? We read all about atrocities performed on Iraqi action figure POWs."



"What happened at the Island of Misfit Toys," I hissed, "was not policy. That was just some crazy rogue reindeer, screwing around unsupervised. Santa Claus will still be confirmed by 75-80 votes in the Senate."



As I huddle in the shoebox that will soon define the four corners of my world, my thoughts turn to my wife, Barbie; my brother, Fireman Rescue Hero; and my son, Lego Luke Skywalker. I must be strong for them.



I've had to be strong all my life. It's hard to be a poor plastic kid in a video-game world, and even harder when you're an immigrant -- I was made in China. My mother was a Chinese novelty factory and my father was a petroleum by-products distributor who just played around with my mother and then disappeared. Nobody wanted a soldier toy in Clinton's nineties, so I made my way playing minimum-wage gigs like "Thug #3" in the Hudson Hawk action figure line. But after a shameful night of drinking nail polish remover and driving a Mattel remote-control car full of underage Jem sidekicks into a telephone pole, a judge gave me a choice: an Army enlistment, or a Goodwill box. I chose the former.



The elite Action Figure corps took me for my menacing glower, sculpted abs, and gift for languages. After taking several crash language courses at the Army facility in Monterrey, I could speak all the major tongues. Monchichi. Teddy Bear. Cabbage Patch. Smurf.



The rubber bands chafe my wrists, and I haven't had a decent meal from an Easy-Bake oven in days. My Eastern-European-looking guard is clear proof that the Russians are helping the insurgents. He's always shrieking "One! One captured American soldier! Ha ha ha!" Then he counts my grenades, over and over again.



I'll get you for this, Evil Bert.



31 January 2005: Today my captors took my picture outside, in front of a special banner that was deliberately repetitive and misspelled in order to honor the stuttering illiterates of Iraq.



"Is good," said Evil Bert, sounding like a cross between Andy Kaufman and Dr. Nick Riviera. "Now decadent American press will see picture on our website and report that live American soldier held captive. Momentum from election blunted. Boxer-Kennedy win in 2008!"



"No chance, you unibrowed monster," I growled. "There's no way that America's mainstream media would ever fall for such a ruse. The second you post that picture on the Internet, crack investigative teams from the Minneapolis Star-Tribune and The New York Times and, above all, CBS News will be on hand to check facts, verify data, and offer uncompromising insights into the validity of your photograph, even if doing so will force them to lose a potential scoop while indirectly aiding the Bush administration."



"No, no," replied Evil Bert, "American soldier not use humor to build bond between himself and captors. You funny guy, soldier boy, but we still gonna blend you in Cuisinart."



"It doesn't matter what you do, because the validity of those elections still stands. You think all of those blue fingers are manufacturing defects? Iraq has embraced democracy, Mr. What's-Your-Thing-With-Ernie, and the fate of one action figure won't change anything."



Evil Bert grabbed his turban from his head and threw it to the ground. "Screw you, action figure! There was no real election the TV footage is all fake! Blue ink is easy to distribute! And election invalid anyway because not enough Sunnis voted. And Supreme Court may call for recount. And New York Times still not convinced. AndandJews! All their fault! Everything their fault! Jews! And Ernie only Platonic friend! Backrubs and handholding not any big deal! Oooohstupid American!" He stormed off.



1 February 2005: I have bribed a guard to fax this document. (The guard seems to be a hairy Mediterranean fellow with big buggy eyes and a passion for cookies. Strange.) I am sending this fax to the only person I can trust: Lucy Ramirez, somewhere in Texas. If this document appears elsewhere, you'll know that the lying irresponsible blogosphere is to blame.



I've slipped a sharpened staple into my boot. Soon I'll break out of here. I'll get new, better accessories, the kind that aren't legal in the US. Maybe a plastic missile that shoots out of my butt. Yeah, that's the ticket.



I will put out the eyes of Iraqi insurgents with my unsafe features. I will carry on the fight for freedom, one poorly-balanced step at a time. And I will fight for freedom wherever there's trouble.



I am John "Cody" Adam. Soon-to-be-former hostage. American action figure. And damned proud of it.

http://techcentralstation.com/020205F.html



ETWolverine rated this answer Excellent or Above Average Answer
excon rated this answer Excellent or Above Average Answer
Itsdb rated this answer Excellent or Above Average Answer
powderpuff rated this answer Excellent or Above Average Answer
purplewings rated this answer Excellent or Above Average Answer
sapphire630 rated this answer Excellent or Above Average Answer

Question/Answer
Itsdb asked on 02/02/05 - Smoking ordinances

Yesterday, our little West Texas city commission decided, based on a petition, to put a smoking ban ordinance up for vote in May. The ordinance would ban smoking in virtually all public places...restaurants, bars, offices, you name it, and 25' from entrances. Violation would cost you a $2000...yes, a $2000 fine.

I know smoking ordinances are all the rage now, but I have issues with them. I see both sides, non-smokers don't want to be 'victims' of second-hand smoke, smokers think it is taking away one of their freedoms to engage in a legal activity.

My contention is first, many, if not most restaurants in our city have already gone smoke-free. Most offices are smoke-free, as are all government buildings, hospitals, shopping malls, etc.

One supporter of the ordinance complained after opponents spoke, that basically non-smokers apparently have no rights. I contend that no, probably 85% of public places are already smoke-free, and non-smokers have the right to choose whether or not to patronize a restaurant that has a smoking section (and restaurants are basically the primary issue here). However, this ordinance would take away both the smoker's right to have ANY public place to smoke AND the property owner's right to determine what legal activities to allow on his property.

Whether or not one is a smoker is irrelevent, I fear what happens next. What is the next public ban on private property because someone is offended? Background music? Perfume? Televisions in sports bars? Cell phones? Crosses? Prayers? Bibles?

Are these bans even constitutional? Is it a problem the market will take care of? Is there a compromise available? Will this lead to further erosion of property rights? Your unbiased thoughts?

Steve

tomder55 answered on 02/02/05:

"those who torment us for our own good will torment us without end for they do so with the approval of their own conscience.
- C.S. Lewis. "

Itsdb rated this answer Excellent or Above Average Answer
sapphire630 rated this answer Excellent or Above Average Answer

Question/Answer
excon asked on 02/01/05 - Iraq -


Hello peacniks:

If the result is positive in Iraq (and I hope it is), does that mean that we shouldnt discuss the fact that we went there for reasons OTHER than to establish democracy? Is Bush excused? Does that mean hell do it again? Iran? Syria? N. Korea? Should he do it again?

Do you even still think we went there for the RIGHT reasons (Elliot does)? Please dont argue that even the Dems thought there were WMDs. Of course they did. So did I. Why did I think so? Because my government (who is supposed to KNOW) told me so. They were wrong. It does matter.

excon

tomder55 answered on 02/01/05:

I stand by my answer I gave to Bobbie about WMD.

The UN said he had WMDs.

Russia said he had WMDs.

Germany and France agreed he had WMDs.

Egypt said he had WMDs.

King Abdullah of Jordan said he had WMDs.

The Iranians said he had WMDs.

Israel said he had WMDs.

Somehow, I don't think they ALL were wrong.

But now we are supposed to believe that Saddam had a fleet of trucks designed as mobile labs to inflate weather balloons(or maybe inflate balloons for Saddam's grandchildrens' birthday )!!!

Former weapons inspector David Kay said ... "But we know from some of the interrogations of former Iraqi officials that a lot of material went to Syria before the war, including some components of Saddam's WMD programme. Precisely what went to Syria, and what has happened to it, is a major issue that needs to be resolved."


Possibly the first noticeable person to warn that Iraqi weapons of mass destruction could go to Syria was former UN inspector Richard Butler. He warned that when he worked in Iraq between 1997 and 1999, he saw intelligence indicating that suspicious containers were routinely moved in and out of Iraq from Syria, and that there was evidence they contained banned materials.

On January 17th ,2002, according to Italian sources, Saddam Hussein signed a secret agreement with Syria. Iraq would send three CDs of formulas and technical information about weapons including nuclear explosions; 3 test-tubes full of anthrax and botulinum spores; and detailed analysis of tests carried out with these weapons on people to Syria, in exchange for Syria harboring Iraqi scientists, technicians and their information.

By the end of February, three Iraqi microbiologists and a small group of technicians would be at safety in Syria, and a top nuclear physicist and his team soon arrived in early March 2002.(March 22, 2003, Michael Ledeen citing Italian paper, Il Fogio.)The World Net Daily( June 23, 2003.)reported that the major transfer of WMD goods from Iraq to Syria and Lebanon occurred between January and March of 2003. However, the first shipments occurred even before then.

In December of 2002, Israel claimed that Syria was hiding Iraqi mobile biological weapons labs, chemical and biological components and munitions.Israeli intelligence says that between January 10th and March 10th, the transfer of Iraqi chemical (and possibly biological) weapons to Syria and Lebanons Bekaa Valley was completed. They were placed in giant tankers and taken to Syria, and then to the Bekaa Valley under the protection of Syrian special forces and air-force intelligence units. Syrian army engineers then took over supervising the tankers at the poppy fields and used special drilling equipment to dig holes approximately 20-26 feet across and 82-115 feet deep. The weapons were buried under the poppy fields grown for heroin and under the rows of cotton plants in two of the most fertile regions of Lebanon. .Israeli satellite photos showed it occurred at night and the crews wore protective suits. The local farmers were reportedly bribed into not saying anything, but we can also not rule out threats. Intelligence sources indicate that satellite photos prove the whole transfer here took place.(Debkafile, May 1, 2003).

I could go on ,but you get the point .This information has been confirmed by both Syrian and Iraqi defectors ,and by Sattelite photos. I do not know why or if this information does not get the airing it deserves. My best guess is that the Adm. is waiting for the right time to do so.



excon rated this answer Excellent or Above Average Answer
Itsdb rated this answer Excellent or Above Average Answer
paraclete rated this answer Excellent or Above Average Answer

Question/Answer
powderpuff asked on 01/30/05 - Iraq election log

Here is a link to an ongoing log at a BBC news site.

Very interesting

You can bookmark this page and come
back to read the latest posts each day


http://news.bbc.co.uk/2/hi/in_depth/middle_east/2004/iraq_log/default.stm

tomder55 answered on 01/31/05:

Mohammed and Omar wrote their feelings about the elections yesterday .

We would love to share what we did this morning with the whole world, we can't describe the feelings we've been through but we'll try to share as much as we can with you.
We woke up this morning one hour before the alarm clock was supposed to ring. As a matter of fact, we barely slept at all last night out of excitement and anxiety.

The first thing we saw this morning on our way to the voting center was a convoy of the Iraqi army vehicles patrolling the street, the soldiers were cheering the people marching towards their voting centers then one of the soldiers chanted "vote for Allawi" less than a hundred meters, the convoy stopped and the captain in charge yelled at the soldier who did that and said:
"You're a member of the military institution and you have absolutely no right to support any political entity or interfere with the people's choice. This is Iraq's army, not Allawi's".
This was a good sign indeed and the young officer's statement was met by applause from the people on the street.
The streets were completely empty except for the Iraqi and the coalition forces ' patrols, and of course kids seizing the chance to play soccer!


We had all kinds of feelings in our minds while we were on our way to the ballot box except one feeling that never came to us, that was fear.
We could smell pride in the atmosphere this morning; everyone we saw was holding up his blue tipped finger with broad smiles on the faces while walking out of the center.


I couldn't think of a scene more beautiful than that.
From the early hours of the morning, People filled the street to the voting center in my neighborhood; youths, elders, women and men. Women's turn out was higher by the way. And by 11 am the boxes where I live were almost full!
Anyone watching that scene cannot but have tears of happiness, hope, pride and triumph.

The sounds of explosions and gunfire were clearly heard, some were far away but some were close enough to make the windows of the center shake but no one seemed to care about them as if the people weren't hearing these sounds at all.
I saw an old woman that I thought would get startled by the loud sound of a close explosion but she didn't seem to care, instead she was busy verifying her voting station's location as she found out that her name wasn't listed in this center.

How can I describe it!? Take my eyes and look through them my friends, you have supported the day of Iraq's freedom and today, Iraqis have proven that they're not going to disappoint their country or their friends.

Is there a bigger victory than this? I believe not.

I still recall the first group of comments that came to this blog 14 months ago when many of the readers asked "The Model?" "Model for what?"
Take a look today to meet the model of courage and human desire to achieve freedom; people walking across the fire to cast their votes.

Could any model match this one!? Could any bravery match the Iraqis'!?
Let the remaining tyrants of the world learn the lesson from this day.

The media is reporting only explosions and suicide attacks that killed and injured many Iraqis s far but this hasn't stopped the Iraqis from marching towards their voting stations with more determination. Iraqis have truly raced the sun.

I walked forward to my station, cast my vote and then headed to the box, where I wanted to stand as long as I could, then I moved to mark my finger with ink, I dipped it deep as if I was poking the eyes of all the world's tyrants.
I put the paper in the box and with it, there were tears that I couldn't hold; I was trembling with joy and I felt like I wanted to hug the box but the supervisor smiled at me and said "brother, would you please move ahead, the people are waiting for their turn".

Yes brothers, proceed and fill the box!
These are stories that will be written on the brightest pages of history.

It was hard for us to leave the center but we were happy because we were sure that we will stand here in front of the box again and again and again.
Today, there's no voice louder than that of freedom.

No more confusion about what the people want, they have said their word and they said it loud and the world has got to respct and support the people's will.

God bless your brave steps sons of Iraq and God bless the defenders of freedom.

Aasha Al-Iraq.Aasha Al-Iraq.Aasha Al-Iraq.

Mohammed and Omar.

powderpuff rated this answer Excellent or Above Average Answer

Question/Answer
powderpuff asked on 01/29/05 - Polling stations in Iraq

I was wondering why the Iraqi election voting stations are not at mosques instead of schools and if the voting was to be done at mosques, do you think the terrorists would be bombing and blowing up the mosques instead of the schools?

tomder55 answered on 01/30/05:

so far (this is realtime news so things could change ) thre have been no reports of a shool being hit while polling is taking place. The terrorist attacks have not penetrated the security zone around the voting stations.

Consider that the most violent attacks have occured in bombings at mosques I think the schools are the safer choice.

http://abcnews.go.com/International/wireStory?id=432003


http://www.pbs.org/newshour/bb/middle_east/july-dec03/iraq_8-29.html

http://english.people.com.cn/200401/09/eng20040109_132255.shtml

Mohammed at Iraq the Model blog wrote on Friday views that I think the vast majority of Iraqis share "

It's a moment of pure freedom but still surrounded by lots of dangers just like any beautiful rose surrounded by spikes.
There is fear from the enemies of freedom who have their weapons already prepared to intimidate us and stop us from choosing our future.
But at the same time we're full of hope as we know that we've put our feet on the right track and even if we make a bad choice once, we know that we will have the chance to reevaluate the situation again.
No more tyrants ruling the country for decades.

We're standing before a historic moment and I won't be exaggerating if I said that it's an important moment for the whole world; we're standing before a crossroads and everyone should watch and learn from the rebirth of Iraq.

Regardless of the winners in the se elections, those who opposed the elections and resisted the change will have to deal with the new reality.

In 48 hours from now, the dying dictatorships and their filthy tools, the terrorists, will find themselves facing an elected legitimate government in Iraq.

The tyrants nightmare is becoming reality, now they will have to deal with the scariest word in their dictionaries; THE PEOPLE'S CHOICE.
The terrorists have challenged the bravery of the Iraqi people but they messed with the wrong people. The people have accepted the challenge; democracy and elections are not a luxury for Iraqis, it's an issue of life or death. And the terror brutal campaign has only made the people more determined to go on with the change.

powderpuff rated this answer Excellent or Above Average Answer
purplewings rated this answer Excellent or Above Average Answer

Question/Answer
Choux asked on 01/26/05 - Scandal Brewing in Bush Admin.

There is a scandal brewing in the Bush Administration that will soil Bush badly. A second example of Bush's Secretary of Education in his first administration ordering money paid to media types for pushing his programs, ie No Child Left Behind and The Marriage Initiative(can't remember the actual title, sorry).

Today, it was revealed that a female media type was paid $21,000.00 for publicizing Bush'w Marriage Initiative in the media. Two weeks or so ago, a media- type man from Chicago was paid $248,000.00 or so for pushing Bush's o Child Left Behind initiative.

This buying the Media goes to the highest level of Government! Since when can the President of the United States *buy* what is put on the public airways *without disclosure*?? As if it is the opinion of the "supposedly neutral" media type?????

tomder55 answered on 01/27/05:

I have to be consistant . I condemned the practice earlier this month when I posted about the Armstrong Williams payoff ,and I condemn it when Maggie Gallagher ,a columnist I routinely read ,does it .Not only is it unethical ,but it is just stupid for the Administration to risk it's credibility in this manner. If a policty is sound then it stands on it's own merit . It gives the appearance that money is the only way to make new initiatives and policies look good.
Also there are plenty of supporters out there who are more than willing to hawk the administration for no fee.

These and others (whoever else has taken payolla for their views ) have done a GREAT DISSERVICE to the cause of getting a Conservative voice in the main stream media. Now everyone who espouses a conservative view will come under the cloud of suspicion .It's this simple ;If you want to be a columnist, be a columnist. If you want to be in public relations, be in public relations. You cant have both.

STUPID ;STUPID ;STUPID ;STUPID ;STUPID ;STUPID ;STUPID !!!!!!!!!!!!!!

Choux rated this answer Excellent or Above Average Answer
ETWolverine rated this answer Excellent or Above Average Answer
excon rated this answer Excellent or Above Average Answer
Yiddishkeit rated this answer Excellent or Above Average Answer

Question/Answer
kindj asked on 01/26/05 - Commentary from Europe

Matthias Dapfner, Chief Executive of the huge German publisher
Axel Springer AG, has written a blistering attack in DIE WELT,
Germany's largest daily newspaper, against the timid reaction of Europe
in the face of the Islamic threat.

EUROPE - THY NAME IS COWARDICE

(Commentary by Mathias Dapfner CEO, Axel Springer, AG)

A few days ago Henry Broder wrote in Welt am Sonntag, "Europe -your family name is appeasement." It's a phrase you can't get out of your head because it's so terribly true.
Appeasement cost millions of Jews and non-Jews their lives as England and France, allies at the time, negotiated and hesitated too long before they noticed that Hitler had to be fought, not bound to toothless
agreements.
Appeasement legitimized and stabilized Communism in the Soviet Union, then East Germany, then all the rest of Eastern Europe where for decades, inhuman, suppressive, murderous governments were glorified as
the ideologically correct alternative to all other possibilities.
Appeasement crippled Europe when genocide ran rampant in Kosovo, and even though we had absolute proof of ongoing mass-murder, we Europeans debated and debated and debated, and were still debating when finally the Americans had to come from halfway around the world, into Europe yet again, and do our work for us.
Rather than protecting democracy in the Middle East, European appeasement, camouflaged behind the fuzzy word "equidistance," now countenances suicide bombings in Israel by fundamentalist Palestinians.
Appeasement generates a mentality that allows Europe to ignore nearly 500,000 victims of Saddam's torture and murder machinery and, motivated by the self-righteousness of the peace-movement, has the gall
to issue bad grades to George Bush... Even as it is uncovered that the loudest critics of the American action in Iraq made illicit billions, no, TENS of
billions, in the corrupt U.N. Oil-for-Food program. And now we are faced with a particularly grotesque form of appeasement...
How is Germany reacting to the escalating violence by Islamic fundamentalists in Holland and elsewhere? By suggesting that we really should have a "Muslim
Holiday" in Germany.
I wish I were joking, but I am not. A substantial fraction of our (German) Government, and if the polls are to be believed, the German people, actually
believe that creating an Official State "Muslim Holiday" will somehow spare us from the wrath of the fanatical Islamists. One cannot help but recall
Britain's Neville Chamberlain waving the laughable treaty signed by Adolf Hitler, and declaring European "Peace in our time".
What else has to happen before the European public and its Political leadership get it? There is a sort of crusade underway, an especially perfidious crusade consisting of systematic attacks by fanatic Muslims, focused on civilians, directed against our free, open Western societies, and intent upon Western
Civilization's utter destruction.
It is a conflict that will most likely last longer than any of the great military conflicts of the last century - a conflict conducted by an enemy that cannot be tamed by "tolerance" and "accommodation" but is actually spurred on by such gestures, which have proven to be, and will always be taken by the Islamists for signs of weakness.
Only two recent American Presidents had the courage needed for anti-appeasement: Reagan and Bush.
His American critics may quibble over the details, but we Europeans know the truth.
We saw it first hand: Ronald Reagan ended the Cold War, freeing half of the German people from nearly 50 years of terror and virtual slavery. And Bush,
supported only by the Social Democrat Blair, acting on moral conviction, recognized the danger in the Islamic War against democracy. His place in history will
have to be evaluated after a number of years have passed. In the meantime, Europe sits back with charismatic self-confidence in the multicultural
corner, instead of defending liberal society's values and being an attractive center of power on the same playing field as the true great powers, America and China.
On the contrary - we Europeans present ourselves, in contrast to those "arrogant Americans", as the World Champions of "tolerance", which even (Germany's Interior Minister) Otto Schily justifiably criticizes. Why?
Because we're so moral?
I fear it's more because we're so materialistic,
so devoid of a moral compass.
For his policies, Bush risks the fall of the dollar, huge amounts of additional national debt, and a massive and persistent burden on the American
economy - because unlike almost all of Europe, Bush realizes what is at stake -literally everything.
While we criticize the "capitalistic robber barons" of America because they seem too sure of their priorities, we timidly defend our Social Welfare
systems.
Stay out of it! It could get expensive! We'd rather discuss reducing our 35-hour workweek or our dental coverage, or our 4 weeks of paid vacation... Or
listen to TV pastors preach about the need to "reach out to terrorists. To understand and forgive".
These days, Europe reminds me of an old woman who, with shaking hands, frantically hides her last pieces of jewelry when she notices a robber breaking into a neighbor's house.
Appeasement?

Europe, thy name is Cowardice.

tomder55 answered on 01/27/05:

Dgade made my point . Hitler and the National Socialist Party made no secret about what they planned to do. They were discounted as crazy ,but the did what they said they would do. bin Laden never hid his intentions ,and boasted throughout the 90s about his successful attacks that were not responded to. Today Zarqwai has twice in letter and a recent audio tape mapped out his plan of attack .

In both bin Laden's December "fatwah" and the recent Zarqwai tape they have laid their cards on the table. They are not waging war against American Imperialism /occupation .Their war is plainly against democracy .bin Laden said :"Anyone who participates in these elections has committed apostasy against Allah.". Zarqwai said : democracy was based on un-Islamic beliefs and behaviors such as freedom of religion, rule of the people, freedom of expression, separation of religion and state, forming political parties and majority rule. He said that ?"freedom of expression is allowed even cursing God.This means that there is nothing sacred in democracy." He said Islam requires the rule of God and not the rule of "the majority or the people."

A close examination of Islamic text reveals countless examples of jihad in the sense that the word is recognized by non-Muslims .Europe has been at war with Islam since it's infancy . They have in the past beaten back the threat ,and briefly had the upper hand .But somewhere along the way ;(probably during it's early 20th Century fratricide) ;Europe lost it's will to defend it's culture.

ETWolverine rated this answer Excellent or Above Average Answer
kindj rated this answer Excellent or Above Average Answer

Question/Answer
Choux asked on 01/25/05 - Iraqi Elections

Looks like the Iraqi Elections are going to be a huge success! *Phew* Many of the people have gotten behind the idea of voting and having a say in their government. Actually, loving the idea. (Per polls)I hope that this potential success will prove America right for being pro active in going after the causes of Islamofascism.

What are your thoughts about the upcoming elections?

tomder55 answered on 01/26/05:

Most successful democracies today where elections were held for the first time in the past 100 yrs (India ;Philipines; Taiwan ; Brazil to name a few ) have had enormous ethnic and religious divisions in the beginning but stayed to the constitution and eventually embraced the concept of a nationalism that includes all major groups and ethnicities.
Iraqis have developed some national consciousness since they became a country 80 years ago. In the war against Iran, hundreds of thousands of Iraqi Shiite soldiers fought bravely, even though Iran is Shiite.

The Shiite leaders like Sistani today have decided to try to treat the Sunnis fairly ;even with the provocation from history and the recent Sunni terrorism; and decided to go for national unity. I think they have a good chance.

The Opinion Journal said today :

Behold these elections: they are not a prelude to civil war, as some of our sages continually warn. They are the substitute for a civil war. Indeed, the remarkable thing about the Shiites has been their restraint in the face of the terror that the remnants of the old regime and the jihadists have thrown at them. It is their leaders and their mosques and their weddings and their religious gatherings that have been the steady targets of the terror. It is their faith that Abu Musab al-Zarqawi and his band of killers continue to dismiss as a heresy at odds with Islam's "purity." Men are not angels. The Shiite restraint has rested on the hope that redemption shall come at the ballot box.
We needn't be afraid of a Shiite electoral victory. The scarecrow that stayed America's hand in the first Gulf War ought to be seen for what it is. There is no "sister republic" of the Iranian theocracy in Iraq's future. The religious scholars in Najaf know that theirs is a country that differs from Iran; it is a checkered country of multiple communities. The Shiite secularists know this as well. Besides, the Iranian state next door offers no panacea today, only terrible economic and cultural sterility. It has been Iraq's luck that Ayatollah Sistani was there when most needed. A jurist of deeply quietist bent who embodies Shiism's historical aversion to political redemptionism, he has reined in the passions of his community. He has held out the hope that history could be changed without large-scale violence, and without millenarianism. Grant the old man his due.



My best guess is that a Shiite slate of candidates that are supported by Sistani ; the United Iraqi List; will emerge as the leaders of the new government. This will bring Ahmed Chalabi back to position of prominance (smart move by State Dept. and CIA to alienate him ;or maybe their intention to discredit him was a ruse ? :> nyuk ).

I think the days of Allawi being relevent are over or at least diminished . . He has done some good ,but he has not done as good as he could've ;particularily in the security area .He also may have been caught with his fingers in the cookie jar.I think that the Bush Adm. should embrace Chalabi again if he gains prominance and accept the outcome of the elections.





Meanwhile this report shows that the overthrow of Saddam has SAVED Iraqi lives ;disputing the claim of 100,000 excess deaths of civilians .Even with all the violence today it was worse for the people under Saddam's rule.

From this it is reasonable to dismiss the claim of 100,000 "excess deaths" since March, 2003 as entirely unreliable.
The next anti-war source of information on Iraqi deaths is Iraqbodycount.net (IBC), which keeps a running count of what it calls Iraqi civilian casualties. In the first 16 months -- March 21, 2003 through July 21, 2004 -- IBC says between 11600 and 13574 died. But no effort is made to distinguish those killed by US action from those killed by the Iraqi army, or by the various Baathist, al-Qaeda, Ansar al-Islam, or Mehdi Army groups.
Using Iraqbodycount's high figure of 13,574 civilian dead killed by all sides in Iraq, the toll is 3093 fewer than the number killed by Saddam's regime in a comparable period of time. Using the low figure of 11600 killed, 5067 fewer have been killed. Based on the war protesters own numbers, 3000 to 5000 more Iraqis are alive today because of the overthrow of Saddam.


Consider where this country was before the coalition invasion . Mass murders by the government. Saddam and his cronies stealing great sums that were targetted to feed the people .Saddam in defiance of the world making a mockery of the UN.;his WMD not accounted for (it still isn't but at least we know they are not in his hands );supporting terrorism (definitely in
Palestine;definitely harboring wanted terrorists ;definitely providing training areas for terrorist groups like the one Zarqwai leads ;most probably in some kind of an agreement with bin Laden).Saddam a threat to his neighbors ;having used chemical and biological weapons on both his external and internal enemies ;having destroyed much of the environment in the swamp areas of the country ;having blown up oil wells an/or releasing countless thousands of gallons of oil into the sea .No hope for a majority of the people who had before his Batthist reign of terror been leaders in Iraqi society .

No longer is he that threat. His despotic days are over.He is being replaced with hope. A new political paradigm is in it's infancy there that will rock the Arab world .Zarqwai did not say last week he was waging war against occupation /imperialism . He declared war against 'democracy'.It is not sectarian .I think that many more Sunni's will opt to participate in the process than the press is willing to admit. Why ? Because to them the choice is not the status quo antibellum .Those days are gone forever. The choices being offered them today are Zarqwai's unique version of fundamentalism ,and liberty . I think the vast majority will choose liberty.


Choux rated this answer Excellent or Above Average Answer

Question/Answer
excon asked on 01/25/05 - Prison Stuff


Hello experts:

In the main, do we imprison people because we're afraid of them, or do we lock them up because we're mad at them? Are both sentiments proper?

What is the proper role of prisons, in your opinion? Should felons be allowed vote? Why or why not? What does voting have to do felonious activity? If driving is a privilege instead of a right, why don't they take that away instead? Wouldn't bother me to see felons take busses, but not being allowed to vote just seems kinda un-American.

On the other hand, why should either be taken away?

excon

tomder55 answered on 01/25/05:

There is more of a case for losing the privilege of driving if someones actions are abusive and have been or are potentially harmful to others .

Since most prisoners will return to the community after serving , society has an interest in cultivating their sense of social obligation and encouraging civic participation .While they are in jail they are still American citizens .

The laws ;because of the Federal System ;are a hodge-podge of laws that are all over the place .Maine, Massachusetts and Vermont allow prisoners to vote .The other 47 do not. 32 States bar excons to vote while on probation or parole. 15 State deny voting rights to excons for life. Voting rights are a matter of Federal Constitution .There should be a uniform standard for all States regarding this issue.

American citizens of voting age should not be disenfranchised..period.

excon rated this answer Excellent or Above Average Answer

Question/Answer
kindj asked on 01/24/05 - I'm just curious

How long until someone claims that GW orchestrated the NFL playoffs to pre-ordain the Super Bowl contestents?

I mean, the Patriots and the Eagles?

C'mon!

DK

tomder55 answered on 01/25/05:

I would think that if it was a GW conspiracy it would've been the Cowboys v. Texans.

I'm begging to think that the real genious behind Bill Parcells was Bill Bellichik. He's won 2 and is going to his third Bowl without Parcells ,but Parcells has never won a Bowl without Bellichik by his side.

ETWolverine rated this answer Excellent or Above Average Answer
kindj rated this answer Excellent or Above Average Answer

Question/Answer
Choux asked on 01/22/05 - Inaugural Address

To be honest, I'm still upset about the Inaugration Address. I wonder if I'm the only person upset, or say concerned? I need some clarification. I felt that he spoke for the world, as if he had the right to, as he was "king" of sorts.

I'm uneasy. Is hubris the work for Bush? A little help please, experts.

tomder55 answered on 01/23/05:

We live in uneasy times. You are certainly not alone . There has been commentary from both the left(no suprise there )and the right critical of the Address.Former Reagan speech writer Peggy Noonan said :

The inaugural address itself was startling. It left me with a bad feeling, and reluctant dislike. Rhetorically, it veered from high-class boilerplate to strong and simple sentences, but it was not pedestrian. George W. Bush's second inaugural will no doubt prove historic because it carried a punch, asserting an agenda so sweeping that an observer quipped that by the end he would not have been surprised if the president had announced we were going to colonize Mars.

Perhaps these words from the Address have been ovelooked and should be emphasised a little more .:

This is not primarily the task of arms, though we will defend ourselves and our friends by force of arms when necessary. Freedom, by its nature, must be chosen, and defended by citizens, and sustained by the rule of law and the protection of minorities. And when the soul of a nation finally speaks, the institutions that arise may reflect customs and traditions very different from our own. America will not impose our own style of government on the unwilling. Our goal instead is to help others find their own voice, attain their own freedom, and make their own way.

This was not a call to arms just a recognition of what American policy has historcally been .

I find it almost hysterical that Noonan who at one time wrote some of the bluntest defenses of liberty for her boss now finds that Bush lacks 'nuance'.Of course a careful reading of the Address finds it sophisticated and nuanced .He acknowledges free nations and asks for their help . He recognizes nations like Russia and China who have potentially started on the path of freedom;("journey of progress and justice, and America will walk at your side" )while also letting them know that we will hold them accountable for oppression(no different than Carter's Human Rights emphasis).

The genious of the speech is the shattering of the dichotomy of the false choice between idealism ,and the practice of realpolitik.To Bush, there is no fundamental conflict between American ideals and American interests; by promoting the former, we secure the latter. Maybe he'll turn out to be wrong, but for now the burden ought to be on those who, in the wake of Sept. 11, hold to a pre-9/11 view of what is "realistic."

Lincoln once said that a nation half slave and half free could not endure(paraphrase).There was WWII poster that proclaimed This World Cannot Exist Half Slave and Half Free.These words hold as much truth today as they did in the 1940s.
~~~~~~~~~~~~~~~~~~~~~~~~~~~~~~~~~~~~~~~~~
All the world over, so easy to see
People everywhere just wanna be free
Listen, please listen, that's the way it should be
Peace in the valley, people got to be free

You should see, what a lovely, lovely world this would be
If everyone learned to live together
It seems to me such an easy, easy thing this would be
Why can't you and me learn to love one another

All the world over, so easy to see
People everywhere just wanna be free
I can't understand it, so simple to me
People everywhere just got to be free
Ah, ah, yeah . . . ah, ah, yeah

If there's a man who is down and needs a helping hand
All it takes is you to understand and to see him through
Seems to me, we got to solve it individually
And I'll do unto you what you do to me

There'll be shoutin' from the mountains on out to sea
(out to the sea)
No two ways about it, people have to be free
(they got to be free)
Ask me my opinion, my opinion will be
(ah-ha)
It's a natural situation for a man to be free

Oh, what a feelin's just come over me
Enough to move a mountain, make a blind man see
Everybody's dancin', come on, let's go see
Peace in the valley, now they want to be free (Young Rascals)







powderpuff rated this answer Excellent or Above Average Answer
purplewings rated this answer Excellent or Above Average Answer
Choux rated this answer Excellent or Above Average Answer
excon rated this answer Excellent or Above Average Answer

Question/Answer
Choux asked on 01/20/05 - Inaugural Address

I watched the inauguration and the inaugural address a couple of hours ago, and I have to say that I was a little stunned.

My underatanding of one of the points was that America will support all revolutionary leaders of a country who overthrow dictatorships and replace them with Liberty. That America is going to shine the light of Liberty into all corners of the earth.

Did I get that point right??

tomder55 answered on 01/21/05:

One has to be a little cautious here ;not all revolutionary leaders are promoters of liberty .History is replete with examples of revolution replacing tyranny with another tyranny. I watched this week 'The History Channel's program on the French Revolution . There was one great example of a failure of liberty. A more recent example is the apparent hijacking of democracy in Venezuela by Hugo Chavez.

I was suprised by the magnitude of the address. I immediately downloaded it from the White House web site ,and read it a few times yesterday trying to take it all in .

I have never been a big fan of 'realpolitik '.In my view we too often backed dictators in the pursuit of stability .This to me was ultimately self defeating .Did our propping up of Saddam in the 80s or the Palavi reign in Iran ultimately serve our interests ? The stability we achieve is short term but the ramifications last generations.

Yesterday Bush took a shot across the bow of 'realist'thinking while at the same time invited them into an overall American doctrine . I think he made a compelling case that this short term thinking of 'realism'is flawed and that ultimate security lies in human liberty .Thus he argued that "We are led, by events and common sense, to one conclusion: The survival of liberty in our land increasingly depends on the success of liberty in other lands. The best hope for peace in our world is the expansion of freedom in all the world.

The spread of freedom is a noble goal and security is a necessary goal . Bush argued they are one and the same. He said his policy ,as idealistic as it is will ultimately achieve the goal of the 'realists'.Not since Kennedy said "We shall pay any price, bear any burden, meet any hardship, support any friend, oppose any foe, in order to assure the survival and the success of liberty" has a bolder foreign policy agenda been presented in an Inaugural Address.

recommended reading :"The Case for Democracy," by Natan Sharansky . Bush has said that this book has had an impact on his thinking . Although it was suprising that he would hit on that theme so forcefully ;one should not be suprised that Bush believes this . He said this 4 years ago in his First Inaugural Speech :"Through much of the last century, America's faith in freedom and democracy was a rock in a raging sea. Now it is a seed upon the wind, taking root in many nations."



Choux rated this answer Excellent or Above Average Answer
ETWolverine rated this answer Excellent or Above Average Answer

Question/Answer
Choux asked on 01/20/05 - US Military / Tsunami Ravaged Areas

The basic work of delivering emergency food and water to tsunami devastated areas in South East Asia is over. The rebuilding phase has begun and will be handled by respective governments and the UN.

tomder55 answered on 01/21/05:

Given the fact that Indonesia is in the middle of a radical Muslim insurection in the Aceh region ;and that eventually our troops would've gotten caught up in the cross fire ;it is just as well that we phase out our military contribution.

Muslim extremists have been waging war there for almost 30 years .They had an informal truce there after the tsunami hit ;but I do not trust that it can be sustained. As it is ,120 rebels who were trying to disrupt relief efforts have been killed by gvt. troops.

Paul Wolfowitz visited the relief area and called for a political solution to the conflict .Although both Vice President Yusuf Kalla ,and the rebel leaders have also called for a sit down to resolve it ;I do not think anything will come of it soon.

Given all the potential hot spots in the region (Aceh;Sri Lanka etc.)and the fact that immediate needs have been provided ;it is prudent for the military to scale back it's presence and to 'civilianize' the effort. The fleet can still provide assistance off shore on a need by need basis.

Hopefully they will also quickly dispatch the leeches from the U.N. ;those uninvited "guests " of the USS Lincoln .As one of the Officers of the Lincoln reports :

My warship had been transformed into a floating hotel for a bunch of trifling do-gooders overnight.

As I went through the breakfast line, I overheard one of the U.N. strap-hangers, a longhaired guy with a beard, make a sarcastic comment to one of our food servers. He said something along the lines of Nice china, really makes me feel special, in reference to the fact that we were eating off of paper plates that day. It was all I could do to keep from jerking him off his feet and choking him, because I knew that the reason we were eating off paper plates was to save dishwashing water so that we would have more water to send ashore and save lives. That plus the fact that he had no business being there in the first place.

My attitude towards these unwanted no-loads grew steadily worse that day as I learned more from one of our junior officers who was assigned to escort a group of them. It turns out that they had come to Indonesia to assess the damage from the Dec. 26 tsunami.

Well, they could have turned on any TV in the world and seen that the damage was total devastation. When they got to Sumatra with no plan, no logistics support and no five-star hotels to stay in, they threw themselves on the mercy of the U.S. Navy, which, unfortunately, took them in. I guess our senior brass was hoping for some good PR since this was about the time that the U.N. was calling the United States stingy with our relief donations.

As a result of having to host these people, our severely over-tasked SH-60 Seahawk helos, which were carrying tons of food and water every day to the most inaccessible places in and around Banda Aceh, are now used in great part to ferry these relief workers from place to place every day and bring them back to their guest bedrooms on the Lincoln at night. Despite their avowed dedication to helping the victims, these relief workers will not spend the night in-country, and have made us their guardians by default.

When our wardroom treasurer approached the leader of the relief group and asked him who was paying the mess bill for all the meals they ate, the fellow replied, We arent paying, you can try to bill the U.N. if you want to.


of note;The U.S. taxpayer doesn't pay for officers food. They pay a monthly wardroom bill from their own paychecks. Enlisted personnel get their chow but officers do not.

He also comments on the 'contributions ' of the Indonesian military :

As for the Indonesian officers, while their job is apparently to encourage our leaving as soon as possible, all they seem to do in the meantime is smoke cigarettes. They want our money and our help but they dont want their population to see that Americans are doing far more for them in two weeks than their own government has ever done or will ever do for them.

To add a kick in the face to the USA and the Lincoln, the Indonesian government announced it would not allow us to use their airspace for routine training and flight proficiency operations while we are saving the lives of their people, some of whom are wearing Osama bin Ladin T-shirts as they grab at our food and water. The ship has to steam out into international waters to launch and recover jets, which makes our helos have to fly longer distances and burn more fuel.


Yes ;it's time to end the militaries contribution to the effort. Hopefully (and I fear this will happen soon)relief workers will not be kidnapped and beheaded by the ingrates .


Choux rated this answer Excellent or Above Average Answer
ETWolverine rated this answer Excellent or Above Average Answer

Question/Answer
ETWolverine asked on 01/18/05 - An Alternative Inaugural Address

This is both hilarious and containing a hint of truth.


From the January 24, 2005 issue of The Weekly Standard:

What if George W. Bush weren't a compassionate conservative . . .
by P.J. O'Rourke
01/24/2005, Volume 010, Issue 18

MY FELLOW AMERICANS, I had intended to reach out to all of you and bring a divided nation together. But I changed my mind. America isn't divided by political ethos or ethnic origin. America isn't divided by region or religion. America is divided by jerks. Who wants to bring a bunch of jerks together with the rest of us? Let them stew in Berkeley, Boston, and Ann Arbor.

The media say that I won the election on the strength of moral values. If the other fellow had become president, would the media have said that he won the election on the strength of immoral values? For once the media would have been right.

We are all sinners. But jerks revel in their sins. You can tell by their reaction to the Ten Commandments. Post those Ten Commandments in a courthouse or a statehouse, in a public school or a public park, and the jerks go crazy. Why is that? Christians believe in the Ten Commandments. So do Muslims. Jews, too, obviously. Show the Ten Commandments to Hindus, Buddhists, Confucians, or to people with just good will and common sense and nobody says, "Whoa! That's all wrong!"

But jerks take issue with every one of the Ten Commandments. Jerks are particularly offended by the first two Commandments. Of course people of faith, decent people, differ on interpretations of the first two Commandments. For example, we don't all agree about the meaning of "Thou shalt not make unto thee any graven image." However, we do all agree about "Thou shalt not bow down thyself to them" when them is Freud, Marx, and Dan Rather.

"Thou shalt not take the name of the Lord thy God in vain." How many times, over the last few months, have we heard, "Ohmigod, ohmigod, ohmigod, I can't believe George Bush won"?

"Remember the Sabbath day, to keep it holy." Let's be fair about this. We did see a lot of white, non-Hispanic Democrats in churches in 2004. But they were all running for president. And the churches were inner-city black churches. I happen to know that there are churches in the white, non-Hispanic suburbs where these Democrats live. Apparently jerks can't find them.

"Honor thy father and thy mother." Are telling lies about a bankrupt Social Security system and trying to block its privatization reform ways to do this?

"Thou shalt not kill." Why, in the opinion of jerks, is it wrong to kill a baby but all right to kill a baby that's so little he hasn't been born yet? And why do the same jerks who favor abortion oppose the death penalty? We can imagine people so full of loving kindness that they can accept neither the abortionist nor the executioner. We can even imagine people so cold-hearted that they embrace them both. But it takes a real jerk to argue in favor of killing perfect innocents and letting Terry Nichols live.

"Thou shalt not commit adultery." The jerks have begun praising marriage lately. But only if the bride and groom each have a beard.

"Thou shalt not steal." In 2004 the United States government spent $2,318,800,000,000. Thus every American benefited from $7,919.37 worth of federal services. Let me ask the jerks something. Say you're average jerks, a "blended family" of four. Did you pay $31,677.48 in taxes last year? If you didn't, you took things from other Americans. What did you give in return?

"Thou shalt not bear false witness against thy neighbor." Especially not in return for vast wealth, abundant prizes, and lavish praise from fellow jerks. I'm talking to you, Michael Moore.

And then there is the Tenth Commandment. "Thou shalt not covet thy neighbor's house, thou shalt not covet thy neighbor's wife, nor his manservant, nor his maidservant, nor his ox, nor his ass, nor anything that is thy neighbor's." The Ten Commandments are God's basic rules about how we should live--a brief list of sacred obligations and solemn moral precepts. The first nine Commandments concern theological principles and social law. But then, right at the end, is "Don't envy your buddy's cow." How did that make the top ten? What's it doing there? Why would God, with just ten things to tell Moses, choose as one of those things jealousy about the starter mansion with in-ground pool next door?

Yet think how important the Tenth Commandment is to a community, to a nation, indeed to a presidential election. If you want a mule, if you want a pot roast, if you want a cleaning lady, don't be a jerk and whine about what the people across the street have--go get your own.

The Tenth Commandment sends a message to all the jerks who want redistribution of wealth, higher taxes, more government programs, more government regulation, more government, less free enterprise, and less freedom. And the message is clear and concise: Go to hell.



P.J. O'Rourke is a contributing editor to The Weekly Standard and author, most recently, of Peace Kills (Atlantic Monthly Press).


Copyright 2005, News Corporation, Weekly Standard, All Rights Reserved.

tomder55 answered on 01/19/05:

PJ is the Will Rodgers of this era. He is a regular contributor to Atlantic Monthly which I suscribe to so I get a regular fix of his stuff.

I have not read 'Peace Kills' yet but a few years ago I read what is one of the best satirical dissections of our system ever written . 'Parliament Of Whores' by O'Rourke is a must read .

some quotes by PJ:

Every government is a parliament of whores. The trouble is, in a democracy, the whores are us.


Sloths move at the speed of congressional debate but with greater deliberation and less noise.


Neither conservatives nor humorists believe man is good. But left-wingers do.

War will exist as long as there's a food chain.

And biotechnology is a worry. What if they take genetic material from wet noodles and blowfish and splice it into politician chromosomes and create a Clinton administration?

The Soviet constitution guarantees everyone a job. A pretty scary idea, I'd say.

Social Security is a government program with a constituency made up of the old, the near old and those who hope or fear to grow old. After 215 years of trying, we have finally discovered a special interest that includes 100 percent of the population. Now we can vote ourselves rich.





ETWolverine rated this answer Excellent or Above Average Answer

Question/Answer
Choux asked on 01/18/05 - Condileeza Rice before the Senate

I have been watching the Senate hearings regarding approval for Condi being appointed Secretary of State.

What is with Barbara Boxer(D Calif)? She is making a very bad representation of herself. I'm wondering if there is a California "thing", that is, dumb or dumber.

tomder55 answered on 01/19/05:

I listened and /or watched much of the testimony yesterday . Sorry ;I could not take Boxer seriously .She was like comic relief .She and Lincoln Chafee, the RINO from R.I.could not be taken seriously (you should've heard his defense and sucking up to Hugo Chavez of Venezuela ) .

I enjoyed the exchanges between Condi and Kerry ,and wish a serious foreign policy debate like that had occured during the elections. I could've listened to the give and take between Barak Obama and Condi all night. Rarely is such intelligence displayed in the halls of Congress ! He was tough but coherent ;a relief from the oft ramblings of Joe Biden.

Obama spent much of his first round of questioning just trying to clarify Biden's line of questioning about nuclear proliferation . In his second round Obama and Rice had a great exchange and an extended questioning about where the Bush Doctrine is applicable .

Sen.Obama:We are unanimous in wanting to root out terror. It appears that even within the administration there's ambiguity with respect to our views on tyranny. Tyranny is problematic, but if engaged in by an ally of ours or a country that's sufficiently powerful that we don't think we can do anything about it doesn't prompt military action. In other cases it does. Part of the, I think, debate and divisiveness of Iraq had to do with the fact that it appeared that the administration sold military action in Iraq on the basis of concern about terror and then the rationale shifted or at least got muddied into an acknowledged desire to get rid of a tyrant.

And I guess what I'm trying to figure out here -- and this is particular to military action and military incursions -- do we have a well thought through doctrine that we can present to the world that explains when we feel that military action is justified and when it is not? Apparently, it's not justified in Sudan, where there's a genocide taking place. It wasn't justified in Rwanda, despite, I think, a unanimity that that was one of the greatest tragedies that occurred in my lifetime. There are a number of circumstances in which we have felt that such incursions or nation-building are not appropriate, despite the evidence of great tyranny, and yet in Iraq and perhaps in Iran and perhaps in other circumstances we think it is.

And so what I'm looking for is some clearly articulated statement as to when you think, as secretary of State, military action is appropriate. Or do you think, alternatively, that we should just be -- the administration should be able to engage in sort of ad hoc judgments as it goes along, as to whether, well, you know, "let's take these folks out, and let's not take these folks out"?

MS. RICE: Well, it's a very interesting question, Senator. It's one that actually is debated in academy (sic) around the world, when -- how can you think about a standard for the use of military force. In fact --

SEN. OBAMA: Right, although -- not to interrupt, but of course this is not academic --

MS. RICE: No, of course. Of course, but it's --

SEN. OBAMA: -- because we have 150,000 troops over there right now.

MS. RICE: -- but it's exactly my point, that when you're not debating it in the academy, it's a bit more difficult to have a hard and true definition of when one would use military force and when one would not, because circumstances differ, and one has to, when choosing a policy course, look at the mix of tools available to you.

Military force should really be a last resort, certainly not a resort that is early on in the process, because so much at stakes and -- so much is at stake, and lives are at stake, and war is an unpredictable blunt instrument. And so it is indeed, outside the confines of the academy, very difficult to have a specific definition of when you use force and when you do not.

I think that when one looks at Iraq, you look at a circumstance in which an awful lot of factors came together to make the case of Saddam Hussein approachable, really, ultimately only through the use of military force; that it was, in that sense, a last resort, because you had had 12 years of failed diplomacy after a war in which he'd fought a war of aggression, in which he had then signed on to certain obligations, not kept those obligations. He signed on to the obligations, by the way, in order to end the 1991 conflict.

He then didn't live up to those obligations, flaunted them before the international community, continued to threaten his neighbors, continued to threaten our pilots trying to enforce the no-fly zones.

We did have someone with a history and a present and a shadow of the future concerning the world's most dangerous weapons. And we had someone who was an ally of terror and was in the world's most dangerous region. I think he had the whole package.

SEN. OBAMA: Dr. Rice, I don't mean to interrupt you, but I know that I'm going to be running out of time soon. I see that yellow light going off.

I guess my point is is not to relitigate the Iraq issue; I think it's to move forward. The concern that many of my constituents in Illinois express is that we went into Iraq, at least in their minds, because of a very specific threat of terror -- not tyranny but terror. Had the administration sold the plan to go into Iraq based on this complex mix then it's not clear it would have generated public support. That's past.

What now as we move forward and we look at Iran or we look at North Korea or these other circumstances? I think it's important for us to be clear that the American people have to have an honest accounting of why we're going in because once we're in we're stuck.

MS. RICE: Yes.

SEN. OBAMA: And we're now going to be spending at least $200 billion in Iraq, and we've lost over 1,300 lives and it's counting.

And so part of the public diplomacy, both internationally as well as domestically, requires this administration to at least be able to articulate these reasons in a way that are coherent and somewhat consistent. I understand that the world is complicated and it's not always going to be fitting into the neat boxes of the academy. But right now at least it seems like it's moving target, both for the American people and for the international community.

MS. RICE: Senator, I appreciate that, but if I could just speak to the moving target notion, because I don't think it's been a moving target. And the fact is tyranny and terror are linked. They are linked.

We know that if we deal with Osama bin Laden and al Qaeda and the organization that did 9/11, we're still going to be dealing with its spawn, and we're still going to be dealing with the ideology of hatred that it's been perpetrating. And we know that the ground in which that ideology of hatred has grown and matured and prospered is the ground of places in the world where there's a freedom deficit, and where the anger and hopelessness has been channeled into these very malignant forces.

SEN. OBAMA: Absolutely. But again -- and I -- again, I know I'm out of time here, but, that's true in Sudan. There's a lot of anger in Sudan. There's a lot of anger all through sub-Saharan Africa, and yet we don't make these decisions.

So I'm not disputing that if you have a vibrant democracy and a healthy, functioning free market system, there's less likelihood of terrorism. And I think all of us recognize that connection. But we're making very specific calculations on the basis of flawed information, or flawed intelligence, and finite resources. And so, we've got to make the best judgments we can in these circumstances. And so, the fact that there's a link somewhere between terror and tyranny is not sufficient for us to be making decisions about spending $200 billion to $300 billion or sacrificing the lives of American servicemen and -women.

MS. RICE: Senator, I appreciate that, but I have to say, I don't think it's a vague link. When you talk about the Middle East, it's a pretty clear link. You're talking about the rise of Islamic extremism, you're talking about jihadism, you're talking about the ground in which it grew up. And you're talking about a very narrow definition of terrorism if you only talk about trying to take down the al Qaeda organization.

SEN. OBAMA: I think that's fair. And if that's the case in the -- again, I don't want to belabor this, but -- I'm just trying to give you a sense of where I think our public diplomacy fails. There is certainly a link between tyranny in Saudi Arabia and terrorism. And yet we make a whole series of strategic decisions about accommodating the Saudi regime. And I'm not saying that's a bad decision. But what I am saying is, is that the degree to which you as the spokesperson for U.S. foreign policy is (sic) able to articulate greater consistency in our foreign policy, and where those links exist between tyranny and terror you are able to apply those not just in one or two areas but more broadly, then I think your public diplomacy is going to be more successful.


Choux rated this answer Excellent or Above Average Answer

Question/Answer
tsunami asked on 01/16/05 - Does a President have the right to destroy America?

No one voted on Election Day to destroy the Arctic National Wildlife Refuge.But President Bush is now claiming a mandate to do exactly that.

Congressional leaders are pushing for a quick vote that would turn America'sgreatest sanctuary for Arctic wildlife into a vast, polluted oil field.
Even worse, they are planning to avoid public debate on this devastating measure by hiding it in a must-pass budget bill.

As responsible Americans can you please do what you can to stop this?

Please go to: http://www.savebiogems.org/arctic/takeaction.asp?ms=RR0501 right now and send a message telling your U.S. senators and representative to reject this sneak attack on the Arctic Refuge.

Thank you, good people.




tomder55 answered on 01/17/05:

that would turn America'sgreatest sanctuary for Arctic wildlife into a vast, polluted oil field.

That statement is hyperbole and does not help your cause. Drilling (especially exploratory which is all that would happen initially )would leave "footprints "but would hardly destroy the region.The technology has improved greatly since the Prudhoe Bay find and it impacts a much smaller area .Also consider that of the vast region ,only 2,000 acres was targetted for it's oil potential .(But I have heard up to 8% or 1.5 million acre which still leaves 17.5 million acres in the Preserve untouched ) .The impact on wildlife claim was used in during the Prudhoe Bay debate . In fact since the pipeline was built the Caribou Herd which migrates through Prudhoe Bay has grown from 3000 to 32,000 animals. The arctic oil fields have very healthy brown bear, fox and bird populations equal to their surrounding areas.


Even worse, they are planning to avoid public debate on this devastating measure by hiding it in a must-pass budget bill.

That is possible since the procedure of adding riders onto general spending bills has been practiced in the past ;is an outrage that should either be stopped by Congressional procedure changes or by giving the President a line item veto.
```````````````````````````````````````````
ConocoPhillips, the largest oil producer on Alaskas North Slope, pulled out of a lobbying group pushing for opening up the refuge. The company said it has other priorities, which include developing fields in established areas already open to oil companies and getting approval for a natural gas pipeline from Alaska to the lower 48 states.BP pulled out of the group two years ago. BP has scaled back its exploration for new fields in Alaska, which ranks is one of the most expensive places for oil production.The economic feasability is obviously something even the big oil companies question.

The amt. of oil that could be extracted from ANWR is insignificant compared to the problem ;which is dependency on foreign sources. The best estimates suggest that if ANWR were in full production by 2025, the United States would import about 66 percent of its oil if ANWR production were permitted, compared with an estimated 70 percent if production were not allowed.Middle East countries would still determine the price per barrel (oil markets are global and Persian Gulf states control most of the world's oil supply and decide on the quantity they pump .)

I do not believe that we can drill our way to energy independence. But oil exploration should be part of an over-all energy policy.
It is hard for me to find areas that environmentalists would approve of exploration. They do not want it in ANWR;they oppose off shore;they oppose it in other Federal Lands .
Further ;it seems that most other enery sources are also out of bounds. Nuclear? no way. Coal ?furgetaboutit. Even clean sources like windmill get opposition. Either they kill birds (not true),or they are unsightly and NIMBY becomes a bigger concern than the fact that clean energy is produced. Please tell me where you would approve of oil exploration ?




tsunami rated this answer Excellent or Above Average Answer
powderpuff rated this answer Excellent or Above Average Answer

Question/Answer
excon asked on 01/16/05 - The War on Terror - Why we should be afraid!


Hello:

What kind of government seeks the power to lock up, forever, those suspected of being its enemies? What kind of government hides its prisoners from the world? What kind of government ships out its prisoners to other countries for torture?

Ashamedly, mine does.

The contemplation of secret lifetime prisons have increased my feelings that I am living in a rogue nation on the brink on totalitarianism.

The right of an accused person to know the charges against him/her and to defend themselves in a public trial is the bedrock of what we, in this country, call "freedom" From the Magna Carta (1215) to the Declaration of the Rights of Man and of the Citizen (1789) to the Bill of Rights (1791), people seeking freedom from tyranny have forbidden their governments to imprison anyone without just cause or without a trial.

I find it terribly frightening that Bush seems to have convinced a huge number of people that this important right can be ignored in the case of "terrorism suspects". If we allow the government to place itself above the law, then we truly open the door to the rule of terror. Because, if they can do it to them, they'll get around to you.

If we don't stop it, we will soon find ourselves afraid to speak out.

excon

tomder55 answered on 01/17/05:

You do love posing Arthurian dilemmas.
Last June the US Supreme Court decided that the U.S. government has the power to hold American citizens and foreign nationals without charges or trial, but that detainees can challenge their treatment in U.S. courts(Rasul v. Bush and Hamdi v. Rumsfeld). The rulings were 6-3 so it was a clear majority . This made it clear that the Bush Adm. did not have a blank check on this issue. Of the apx 550 detainees in Gitmo that got their hearing ;only 2 were recommended for release so the military must have done a good job of filtering them . Over 200 were released prior to the ruling.

I throw the question back to you ;how can a democracy reasonably deal with those committed to its destruction ? Indefinite detention is a bad idea ;and releasing enemies who are sworn to our destruction is a bad idea too.We already have had released jihadists recaptured or killed in combat with our troops inspite of them signing pledges that renounce violence.And these were the prisoners the military thought didn't pose a threat anymore.
The hardcore that are still in custody are more dangerous. Are your values a suicide pact ?

excon rated this answer Excellent or Above Average Answer

Question/Answer
VisionsInBlue asked on 01/15/05 - LOL!!!! How hysterical is this?

Moore Gets No Love From Old High School

Sat Jan 15, 9:32 AM ET Entertainment - AP

DAVISON, Mich. - Oscar on the shelf or not, Michael Moore is not getting much respect at his old high school. Despite his fame and many honors, the filmmaker has been rejected all four times that he has been nominated for Davison High School's Hall of Fame.

"Would you want him as a role model? Would you want your son or daughter to be like him?" asked Don Hammond, a member of the Hall of Fame selection committee. "I haven't talked to anybody yet who's for him. The word to describe Michael Moore is embarrassing. He embarrasses everybody."

Ryan Eashoo disagrees. The 1997 Davison High graduate has spent 80 hours the last two weeks and $600 of his own money trying to get Moore elected.

"We've been blacklisted," Eashoo, 25, told the Detroit Free Press. "I'm a huge Michael Moore fan. He's a great producer, great filmmaker, always sticking up for minorities. He's kind of an underdog."

So far, Eashoo has 300 signed nominations of Moore. His goal is 2,000 by Feb. 1. The committee meets Feb. 11 to choose its inductees.

Moore's film "Fahrenheit 9/11" attacked President Bush's rationale for the war in Iraq and accused him and his administration of fostering fear for political gain. Moore spent the weeks before the election traveling across the country to urge Americans to vote Bush out of office.

His "Bowling for Columbine" won the Oscar for best documentary in 2003.

http://story.news.yahoo.com/news?tmpl=story&cid=529&ncid=529&e=2&u=/ap/20050115/ap_en_mo/people_michael_moore

Poor Mooron. Are they rejecting him because he never seems to shower? :(

tomder55 answered on 01/16/05:

From The Daily News
Clint Eastwood squinted like Dirty Harry Tuesday night as he took aim at Michael Moore.
"Michael Moore and I actually have a lot in common - we both appreciate living in a country where there's free expression," Eastwood told the star-dotted crowd attending the National Board of Review awards dinner at Tavern on the Green, where Eastwood picked up a Special Filmmaking Achievement prize for "Million Dollar Baby."

Then, the Republican-leaning actor/director advised the lefty filmmaker:"But, Michael, if you ever show up at my front door with a camera - I'll kill you."

The audience erupted in laughter, and Eastwood grinned dangerously.

"I mean it," he added, provoking more guffaws.


VisionsInBlue rated this answer Excellent or Above Average Answer

Question/Answer
Choux asked on 01/14/05 - Poverty Pimp

Bill Cosby has taken up the cause of lack of positive values in the poor Black community. Culture of gangster-violence, non-achievenemt, rap-music and abuse of women....he talked of Jesus and taking up your cross and carrying it.

In addition, he mentioned the term "Poverty Pimp" which was a slap at Jesse Jackson who he claims lives off the victimhood of poor blacks.

Comments?

tomder55 answered on 01/15/05:

Cosby says that thuggery, street lunacy, gangsta rappers, teen-age pregnancy, educational underachievement and antisocial behavior are not OK .

If Martin Luther King Jr. was alive today, he would be appalled by today's State of Black America .Cosby needs help because he can't wage a one-man crusade against black apathy and attitude by himself . He knows that he won't get that support from the twin "Povert Pimps" Jesse Jackson and Al Sharpton.(what a great description!)

Perhaps Cosby can find help and hope in Barack Obama.Newsweek wrote on Obama: "Some party constituencies might be in for a surprise. Obama may be the only African-American in the Senate, but this is not a man who wants to be seen as the leader of black America. When he spoke at a Congressional Black Caucus reception recently, Obama graciously thanked several Caucus leaders by name, then concluded with a short but telling statement: I'm looking forward to working with you on behalf of ALL Americans."

Perhaps the bullies who beat up 16-year-old Lucas Hardeman of Los Angeles because he answered too many questions right in a Los Angeles classroom one day should listen to Obama.

Those are the same kind of bullies who, as Cosby writes , ridiculed Hardeman for buying CDs by Britney Spears, 'N Sync and the Backstreet Boys, then smashed them at a school function as they told Hardeman, "Dude, you're whiter than they are."

That's the kind of lunacy to which Cosby is referring. It's also a major example why Cosby needs assistance from a potential difference-makers like Obama.

VisionsInBlue rated this answer Excellent or Above Average Answer
Choux rated this answer Excellent or Above Average Answer
ETWolverine rated this answer Excellent or Above Average Answer

Question/Answer
excon asked on 01/13/05 - Trial Lawyers - Good Guys


Hello all you very bright and wonderful people: ((((very bright and wonderful people)))

Think frivolous lawsuits are the cause of skyrocketing medical costs???? Think again. A friend had an ingrown toenail clipped. It took 30 seconds. Her insurance company was billed $1,451.00.

I think the insurance companies are the bad guys, not the doctors or lawyers. Besides, I hate insurance. I bought plenty but never enjoyed any of it.

excon

tomder55 answered on 01/14/05:

I doubt that I could open the insurance companies books and see much red ink. But frivolous law suits cannot be discounted as a major problem .

Look at Vioxx and Celebrex as examples of the problem . How many ads have inundated the air waves by lawyers looking for clients who took the products ?They claim that there was some kind of negligence by the drug companies. But how can that be so? They spend years and millions of dollars getting their products tested for safety and effectiveness by federal regulators. How could there be negligence if they in good faith followed the approval guidelines and obeyed all Federal approval regulations ?

It is almost impossible to put a drug product on the market that is completely safe and effective; there are always side effects. Look at the statin drugs used for cholesterol busting ;wonderful life saving products . Turns out that statin use depletes Coenzyme Q10 which is highly concentrated in the heart ;which causes myopathy or neuromuscular degeneration .In layman's terms it weakens the very muscle it is being taken to cure . I'm suprised an army of lawyers have not already capitalized on that.

Most people would not deny an injured person to get fair compensation .Many suggest that an advisory board of experts review the merits of the claims before they proceed .It is the $$$$$$$$$$ amt. of the settlements that attract the trial lawyers. The same group that spent multimillions lobbying lawmakers to pass laws favorable to the very practices they employ. In fact ;it is mostly lawyers that are in legislatures around the country making the laws that benefit the lawyers. What an unholy alliance!

excon rated this answer Excellent or Above Average Answer

Question/Answer
kindj asked on 01/13/05 - Inauguration costs

I'm seeing a lot of people complaining about the cost of the Bush inauguration. Something like 40 mil. OK, sure, that seems a bit extravagent to me, especially with all that's going on. If it were me, I'd just as soon take the oath, smile for the camera, make a short speech, and get back to the job I was being paid for.

But that's just me, and I'm no politician.

However, SURELY this is not the first inauguration that has come with a hefty price tag. Anyone have any figures for other Presidential inaugurations?

DK

tomder55 answered on 01/13/05:

It is big; but it has proportionally grown over the years .Clinton's first inaugural cost $33 million .His second was more modest ;costing $29.6 million .Both were paid with private funding .Bush's first also cost arounf $40 million all paid by private funding .


The big issue is that Bush wants some of the cost to come from Homeland Security Dept. funds targetted for DC.(about $12 million ) The price of security has never been included in the cost of the event.Generally the Fed. Gvt.pays for the security .

I think it is a tempest in a tea pot. If DC has to shell out some of it's federal funds it will be more than offset by the economic value to the city of hosting the events .According to The Washington Times:

The Presidential Inaugural Committee hopes to raise as much as $50 million, which will be dumped into the District to pay for the inaugural festivities.
The Washington DC Convention & Tourism Corp. (WCTC) estimates overnight visitors during the inauguration will generate about $44.6 million for the local economy. That doesn't include the money spent by people visiting for just the day on Thursday who will ride the Metro, use the parking lots, take cabs and buy breakfast, lunch and souvenirs.
Thousands of people are expected to attend the festivities, including the swearing-in ceremony and the parade. Day visitors usually spend about $60 each during their visit, according to the WCTC.



NYC shelled out big bucks to host the Republican Convention but it was an economic plus for the city to the tune of $265 million economic gain .

kindj rated this answer Excellent or Above Average Answer
purplewings rated this answer Excellent or Above Average Answer

Question/Answer
Yiddishkeit asked on 01/12/05 - Iraq: thoughts and questions...

According to Sec. of State Colin Powell, he believes that during 2005 our troops will gradually start reterning home. I hopeful that he is correct, but suspicious (more on this in the third paragraph)...what do you believe?


I think the upcoming Iraqi election is as big an oportunity for the insurgent terrorists to give a blood bath as is for the law abiding civilians to cast votes. I hope the election day does not have any violence so perhaps it will hasten our troops return back to the States, if possible. We need to do a very good job securing perimeters around areas of ballot casting. Your thoughts?


Now that the search for WMD's in Iraq has fairly concluded, does Pres. Bush need to explain to the America public why he was wrong...that is according to House Minority Leader Nancy Pelosi (D-Cal)? Should we start looking elsewhere? If so, would that mean that the troops currently in Iraq that Powell thinks could be coming home in 2005 would actually be re-deployed for another hot spot role?


Here are some non-political orientated questions:

According to the news we are going to blast away at a comet of ice. I forgot where it's location is in conjunction to the earth, but the blast is supposed to leave a crater the size of the Roman Coliseum for research purposes. The impact is scheduled to occur on the Fourth of July. But more importantly, and this is just my opinion, that our government is interested in knowing if we can deter any comet that may be headed our direction in the future (of course if needed on a larger scale using nuclear explosives). I believe they are checking for accuracy and impact analysis in such an event. I know that in the very recent past the planet Saturn was hit by comets and it was devestating. Your thoughts?


Beltran has left my beloved Astros for the Mets. I was really hoping that we could re-sign him. Do the Astros have much of a chance of getting back in the playoffs next season?

Now...on to my beloved Cowboys. Oh forget it! I'm just disappointed in them. I have the draft to look forward to. Here's a better subject: Superbowl! My guess is that if Vick stays healthy I think the Falcons (the NFC) and either the Colts or Steelers (the AFC) will be in the Super Bowl...what teams do you think will play in the Super Bowl and why?





Bobby





tomder55 answered on 01/13/05:

What is more probable . The whole body of the Worlds intelligence agencies getting it wrong ....or Saddam transporting the WMD to Syria and Lebanon ? Occam's Razor tells me that it is more likely the WMD was moved.

But lets look at another possibility being floated and is gaining acceptance . This theory is as follows :
Saddam did in fact get rid of his weapons of mass destruction but deliberately kept the world guessing about it an effort to divide the international community .(what a genious ;he fooled the whole world including the Israeli's ;the Iranians ;the Egyptians ;the UN ;the US ;Germany ;Russia .You would think someone that ingenious would not make such a miscalculation)The strategy, was designed to make him look strong in the eyes of the Arab world. At the same time, Saddam retained the technical know-how and brain power to restart the programs at any time. The indictment on the President then is that since therewas no "imminent"threat (which Bush never claimed anyway)then there was no justification for preemption.

Lets play this out to it's conclusion . Bush doesn't preempt .No weapons would have been found, and the Blix inspections would have ended. There would no longer be any justification for US military pressure on Iraq and the UN sanctions would also have to end. With the inspectors gone and Saddam's freedom of operation restored with the technical capability and know-how still in place, Iraq would be able to reconstitute its weapons. And who would listen to American concerns about Iraqi weapons at that point? Eventually Iraq has fully operational weapons and really does pose an imminent threat to the US and our allies.It would be all that much harder for us to defeat Saddam.

Those who squawk "Bush lied" like Nancy Pelosi (but never squawk "Saddam lied") might be satisfied with the above scenario, but I'm glad we dismantled Saddam's regime when we had the chance.


~~~~~~~~~~~~~~~~~~~~~~~~~~~~~~~~~~~~~~~~~~

I think Powell is correct that this year will see a draw down of troops there ,but it may just be a hike and a hop skip across the border .

~~~~~~~~~~~~~~~~~~~~~~~~~~~~~~~
I do not think the election will be violence free . The Zarqwai playbook calls for increased levels of violence right up to election day. He knows what is at stake if the elections take place. The 150,000 troops we have there I believe represents the maximum that we can deploy with our current active and reserve force .The security of the elections has to fall on the Iraqi security forces ;which have been expanded greatly in recent weeks but still are inexperienced.

~~~~~~~~~~~~~~~~~~~~~~~~~~~~~~~~~~~~~~~~~

The reason that you gave for the comet inteception is sound. It would also help us in missle intecept technology and there could be some biological reasons also . One of the new (secular) theories going around is that since there is nothing in the earths geology that could've caused the chain reaction that led to life on earth ;that organic matter was 'transported ' to earth on a comet that 'seeded 'the earth .

~~~~~~~~~~~~~~~~~~~~~~~~~~~~~~~~~~~~~~~
The Mets have been very active in the free agent market this year .They also grabbed Pedro Martinez. I wonder what infuence former Yank Willie Randoph has in these decisions ? More about the 'stros.. How is my man ASndy Pettitte doing ? Will he be back this year ?

~~~~~~~~~~~~~~~~~~~~~~~~~~~~~~~~~~~~~

One thing that doesn't get much play about Parcells is that when he has a bad year ;it's a real stinker . He had a couple of 3 win seasons with the Giants . Think he may be too loyal to Vinnie Green-nuts ?

I think it's the Colts year . The NFC ..it would not suprise me to see St.Louis make it in . The Eagles lost Terrell Owen so they look alittle more beatable .I do not trust Vick to lead a team all the way yet .

excon rated this answer Excellent or Above Average Answer
Yiddishkeit rated this answer Excellent or Above Average Answer

Question/Answer
Itsdb asked on 01/12/05 - How will you mark the inauguration?

There are many alternative planned activities to mark the inauguration...

"The DC Anti-War Network (DAWN) has called for a permitted anti-war rally and march, and it has called separately for nonviolent civil disobedience die-ins to draw attention to the dead at the hands of the Bush Administration. These actions have been called for January 20, 2005. We invite the entire world to participate in these events, which have been endorsed by United for Peace and Justice."

Student Walkouts!

"College and High School students will be performing walkouts against the inauguration on both the 19th and 20th. These are likely to merge into larger actions. Get involved, organize with your friends!"

Anarchist Mobilization at the Inauguration

"All out for an anarchist mobilization against centralized power...at the 2005 Presidential Inauguration: Washington, DC - January 15-20th."

Washington DC counter-inaugural critical mass bike ride

"Plans are being drawn up right now to have the largest critical mass bike ride DC has ever seen."

Women's March and Funeral Procession

"Women's March and Funeral Procession, in the style of the New Orleans jazz funeral, to take place on January 20, 2005, in Washington, DC, to coincide with "Inauguration Day."

Billionaires for Bush auction off Social Security

"The always fun and photogenic Billionaires in tuxedos and ball gowns will carry out our president's mandate by auctioning off Social Security to the highest bidder, as well as the Arctic National Wildlife Refuge."

Turn Your Back on Bush

"This is a non-violent, mass action that will powerfully demonstrate the broad opposition that exists in this country to the presidency of George Walker Bush. We call on people to dress neutrally and line the parade route. As the presidential motorcade approaches many of us will simply turn our backs on him. We invite you to join us!"

Noise Against Fascism

"Noise Against Fascism is intended as a positive (albeit brutally loud) response to the inaugural activities otherwise fouling the air on January 20."

Billionaires for Bush Re-Coronation Inaugural Ball

"All the Excess, Twice the Greed

Bigger, Larger, More Unchecked!

The Billionaires for Bush will host a black tie ball to celebrate the re-coronation of the president we paid for.

Afro-funk Big Band Chopteeth!
Billionaires Follies Cabaret!
Swank Swing and Hip House DJ!
Three Floors of Dancing!

Tickets from $20, including advance internet purchases at $10 (restrictions apply). Top hats and tiaras available, but do dress to opress! Our website has tips on how to look wealthy."

Deliver a Spine to DNC Headquarters

"Call for Participation:
Deliver a Spine to DNC Headquarters Join the Backbone Procession to DNC Headquarters to Demand Progressives Not Be Taken For Granted."

No New Wars! March on the Neocons

"We will start at American Enterprise Institute and march on 11 total Neocon Institutes, all within a few blocks of each other. Bring your signs, props, puppets, noisemakers! Stop their next war: Syria, Iran, No. Korea??"

I will, as always, be celebrating my wife's birthday more than anything...but I would love to watch the "Deliver a Spine to DNC Headquarters." How about you? How will you mark the occasion?

tomder55 answered on 01/12/05:

I heard about the "boycott " yesterday . Bogus is all I can say . Who does it hurt? ;certainly not the President. I intend to gas up my car whether it is needed or not . Maybe go out to a steak house dinner .I'll try to coordinate so that groceries need to be picked up in the 20th .Maybe I'll go see 'Phanton of the Opera' at the theater or at least I'll try to order some pay per view movie. I wouldn't go to Target but maybe I'll really piss em off by going on an impluse shopping spree in Walmart !!! yeah baby !!! time to prime that economic pump!!

Itsdb rated this answer Excellent or Above Average Answer
ETWolverine rated this answer Excellent or Above Average Answer
kindj rated this answer Excellent or Above Average Answer

Question/Answer
sapphire630 asked on 01/11/05 - Don't know much about SS reform

I keep hearing people oppose it because they are afraid their money would be lost in bad stock market investments (other than that I don't think they have any idea better of an idea of the reform than I do)
I am for it if it is done in a way that will work and guarantees to ensure the government can't mess it up? Like why can't the money be invested like the 401k's? Why can't they put limits on the changes they are allowed to make down the road (so they can't end up taking money away from what started as a good system)?
I heard that Chile or somewhere has personalized system where they get way more than the chump change we get when we retire. Why couldn't we use them as an example?

tomder55 answered on 01/12/05:

It is not called the third rail of American Politics for nothing . There is no real consensus that they system is in trouble ;or what time frame is there to fix it .The Congressional Budget Office estimates that by 2052, Social Security will only be able to pay out 81 percent of its obligations. So I should just take the attitude that I'll get mine .......but that would be irresponsible ,and S.S. has survived by 'band-aid' solutions for far too long .Bush claims that by 2018 it will start running deficits due to demographics.

The transition cost to set up the privitized plan ;as dgade mentioned; is one of the one big issue .It is estimated that a 1-2 trillion dollar borrow by the gvt. would be needed. But I think if it is doable then the idea behind personal accounts is sound. The gvt. should be doing more to encourage personal savings anyway.

It appears that most of the Democrats would prefer to fix it the old fashion way ...increasing SS taxes . A memo leaked last week indicated that Bush may want to creatively cut benifits (increase age of retirement ;tie it to means testing )

But it is hard to form an opinion to date because what has been lacking from the debate to this point is specifics .Bush says he has a mandate to deal with the problem but I don't think Reagan had a big enough one to do it. It will require true bipartisanship and I do not think that will or the perception of urgency is there.

sapphire630 rated this answer Excellent or Above Average Answer

Question/Answer
ROLCAM asked on 01/12/05 - Who and When said these words ??

" Taxation without representation is tyranny."

tomder55 answered on 01/12/05:

James Otis ;but that was a sentiment shared by quite a few patriots in the Revolution era .

He would've been a better known patriot if he had not been clubbed in the head with a cane for a newspaper article he wrote about a local customs house official in Boston .He then suffered bouts of mental instability . He quickly became the village idiot of Boston .

Do I pass the quiz ?

ROLCAM rated this answer Excellent or Above Average Answer
Choux rated this answer Excellent or Above Average Answer

Question/Answer
ROLCAM asked on 01/12/05 - Who uttered these words ??

"The world is governed by people far different from those imagined by the public."

tomder55 answered on 01/12/05:

Benjamin Disraeli

ROLCAM rated this answer Excellent or Above Average Answer
Choux rated this answer Excellent or Above Average Answer

Question/Answer
excon asked on 01/11/05 - Abbas: Peacemaker???


Hello all you fine people, (((((fine people)))):


One moment Abbas demands that Palestinian terrorists stop their attacks on Israel and the next he (literally) embraces them, calling them "heroes fighting for freedom." Also, he talks of both stopping the violence and of the "right of return for over 4 million Palestinians to Israel, a well-known way of calling indirectly for the elimination of Israel.

What Gives?

Actually, there is no contradiction. By insisting on a "right of return," Abbas signals that he, like Yasir Arafat and most Palestinians, intends to undo the events of 1948; that he rejects the very legitimacy of a Jewish state and will strive for its elimination. He differs from Arafat, only in being able to imagine more than one way of achieving this goal.

excon


tomder55 answered on 01/11/05:

This is all you need to know . Right after the victory Abbas proclaimed: "The small jihad is over and the big jihad has begun."

excon rated this answer Excellent or Above Average Answer

Question/Answer
excon asked on 01/10/05 - Voting vs Banking


Hello very well informed and especially smart people:

Here's one thing I don't understand. Oh, I don't understand plenty, but this one nears the top of my confusions:

If I'm able (along with a jillion other people) to deposit and withdraw money, either in person or over the internet, from my bank, and not lose a penny, and have an audit trail if I did, then I'm lost about why we can't count votes.

Somebody has a vested interest in keeping the present Swiss cheese system in play. I don't know who that would be, but somebody does. Do you know?

excon

tomder55 answered on 01/11/05:

if someone can document a flawless election please do so. I do not think it has happened . I do not suscribe to a conspiracy theory either .What is being asked for here is Federal Standards ,and I do not beleive there are constitutional provisions for Federal standards.

excon rated this answer Excellent or Above Average Answer
ETWolverine rated this answer Excellent or Above Average Answer

Question/Answer
excon asked on 01/10/05 - Voting vs Banking


Hello very well informed and especially smart people:

Here's one thing I don't understand. Oh, I don't understand plenty, but this one nears the top of my confusions:

If I'm able (along with a jillion other people) to deposit and withdraw money, either in person or over the internet, from my bank, and not lose a penny, and have an audit trail if I did, then I'm lost about why we can't count votes.

Somebody has a vested interest in keeping the present Swiss cheese system in play. I don't know who that would be, but somebody does. Do you know?

excon

tomder55 answered on 01/10/05:


That would assume of course that there has never been transaction problems with your banking . Would that it were true. I am not exactly a high roller Donald Trump type ,but there have been quite a few errors over the years that I have had to reconcile .

This idea that electronic voting is a panacea is just wrong .Around the country there were reports of machine malfunctions that affected precincts that voted for both candidates. In Cuyahoga County, Ohio, where Kerry beat Bush by 220,000 votes ,the county's election result Web site reported that the number of actual voters exceeded the number of registered voters by huge margins in some precincts.In Sarpy County, Neb. a single voting machine recorded 10,000 extra votes;in Carteret County, N.C., 4,500 votes were lost.In Florida supposedly state of the art touch screen balloting was done. Now everyone is crying that they need a paper ballot backup. For years, prominent computer security experts have been arguing that paperless machines present major security problems, including buggy software and the risk of malicious code affecting the outcome of an election.Purely electronic systems will always be open to massive voting results manipulation with no ability to perform any kind of meaningful recount to verify the results.

Hillary last week claimed that India went to the polls ,and 550 million people using electronic machines cast votes without any glitches. I do not beleive it . It would be the equivalent of turning the ignition on 550 million cars and expecting all of them to start .

As for national standards . The Constitution doesn't provide for that .

excon rated this answer Excellent or Above Average Answer

Question/Answer
Choux asked on 01/08/05 - Newt Gingrich

Newt Gingrich is planning a run for the Presidency as a Republican in 2008. He sure is one intelligent guy, I'll say that for him.
Will he make a good candidate?

tomder55 answered on 01/09/05:

He is on tour to promote his new book 'Winning the Future: A 21st Century Contract with America',and his book tour convieniently brings him to Iowa and New Hampshire.I'd say he is at least exploring the possibility of a run at the White House.

In the book he takes shots at the Iraq policy simular to the ones I mention.He writes that the Bush administration erred by creating a U.S.led provisional authority instead of an interim government as it did in Afghanistan."While the Iraq war was just and the military campaign brilliant, the process of creating a democratic Iraq has been difficult. The decision to have an American administration in Baghdad was a mistake." He also liberally takes shots at Rumsfled.

I do not think he will be a major player in the next election and will be eliminated early in the process. He has been a Congressman in the House of Representatives and Congressmen do not become President without having held some other position (ususally VP).Also ther is some baggage about morals and ethics.I would love to camapign against a guy who touts family values who pressed his wife for divorce while she was recovering from cancer surgury still in the hospital .After the divorce he had to be taken to court as a dead beat dad.He was the subject of ethics investigations while in Congress.

He is brilliant and has a place in policy debate ,but no;I do not think he is a good leader.


Choux rated this answer Excellent or Above Average Answer

Question/Answer
Choux asked on 01/08/05 - Iraq Elections

Yesterday on cable news, I saw a story that in some parts of Iraq it is too dangerous to have elections.

We have been in Iraq for, how long now, and now we have to face the fact that the War was a terrible idea for all the reasons we see on the news every day. But, mostly because the leaders(Neo-Cons) engaged in "wishful thinking" on what would happen in an occupied Iraq.

Do you think that we will be there another twenty years, or so?

tomder55 answered on 01/09/05:

We held elections in 1864 when 1/2 of the nation was too dangerous.In his debate with Edwards Cheney cited that El Salvador held elections even while a guerilla insurgency held 1/3 of the terrain.Elections should proceed on schedule.

Probably they will set aside seats in the government(which will only be a temporary one designed to design a permanent constitution ;with further elections scheduled for later this year)for the Sunni segments.Still they represent a small minority ,and most likely the slate of candidates endorsed by Sistani will seat the largest contingent (they include btw. Ahmend Chalibi .Like it or not the U.S. will have to deal with him ;word to the State Dept.

I contend that the biggest problem has been that the Neo-Con plan was not followed . That was (as I documented before ) a quick in ,to remove Saddam , and a hand over of power to the INC and other exile groups .

That is in the past .It was a mistake to
remain occupiers ;and to delay handover by appointing Viceroy Bremer et al. ;We move on .

In spite of everyone's fears od Shia majority rule ,there is no plans of setting up a theocracy. Sistani doesn't want it and he has surrounded himself with people who do not want it . Chalabi said this week ; "First, we do not want any interference in the Iraqi elections; second, the alliance is not about an Islamic republic or a theocratic state, it is about democracy and pluralism, and third, we will need American forces to be in Iraq for the foreseeable future." The Iranian leaders accepted these points, he said. "They understand that the situation in Iraq is very different from Iran," he said.

I believe we will be there for a long time with Iraqi approval in bases outside of urban areas . We need to be there ;not for the long term security of Iraq (they will assume security when they have a legimate government);but as a forward base to deal with other long term security issues (post WWII comes to mind with forward basing in West Europe ;S.Korea;the Fillipines Subic Bay and bases in Okinawa ). Those who do not think we need to be in the region ignore the past reality that we have had boots on the ground since 1990 ,and that our fleet has kept the free flow of commerce open for many years before .

Choux rated this answer Excellent or Above Average Answer

Question/Answer
kindj asked on 01/08/05 - Sort of an update, I guess...

Got this today, thought I would share:

January 04, 2005
Action Vs. Talk In Indonesia

For a realistic, on the ground account of what's happening in Sumatra, check out The Diplomad, a blog written by American foreign service officers. Their comments on the U.N.'s contributions are amusing:

Well, dear friends, we're now into the tenth day of the tsunami crisis and in this battered corner of Asia, the UN is nowhere to be seen -- unless you count at meetings, in five-star hotels, and holding press conferences.
Aussies and Yanks continue to carry the overwhelming bulk of the burden, but some other fine folks also have jumped in: e.g., the New Zealanders have provided C-130 lift and an excellent and much-needed potable water distribution system; the Singaporeans have provided great helo support; the Indians have a hospital ship taking position off Sumatra. Spain and Netherlands have sent aircraft with supplies.

The UN continues to send its best product, bureaucrats.


Most interesting to me was this memo written by Dutch diplomats and circulated at an EU meeting in Indonesia:

The US military has arrived and is clearly establishing its presence everywhere in Banda Aceh. They completely have taken over the military hospital, which was a mess until yesterday but is now completely up and running. They brought big stocks of medicines, materials for the operation room, teams of doctors, water and food. Most of the patients who were lying in the hospital untreated for a week have undergone medical treatment by the US teams by this afternoon. US military have unloaded lots of heavy vehicles and organize the logistics with Indonesian military near the airport. A big camp is being set up at a major square in the town. Huge generators are ready to provide electricity. US helicopters fly to places which haven't been reached for the whole week and drop food. The impression it makes on the people is also highly positive; finally something happens in the city of Banda Aceh and finally it seems some people are in control and are doing something. No talking but action. European countries are until now invisible on the ground. IOM staff (note: this is a USAID-funded organization) is very busy briefing the incoming Americans and Australians about the situation.

tomder55 answered on 01/09/05:

I check the Diplomad every day for updates.

Let's see what happens now that the Base Group has bee disolved and the UN now has control of the coordination . I do not expect to see improvements.

kindj rated this answer Excellent or Above Average Answer

Question/Answer
Choux asked on 01/07/05 - Barak Obama

This week, the new Senators and Representatives took up shop in Washington, DC. The Senator Elect from Illinois was caught for a photo op by Jesse Jackson in a sorta hug, Obama apparently sucking a lemon. Obama was ambushed by Jackson who is now an outdated relic of the old way.

Do you think that Obama will have to seriously deal with Jackson? The Black Caucus?

tomder55 answered on 01/08/05:

He has already been tainted by Jackson and the fringe of the Democrats. He publicly supported Boxer's challenge to the Presidential Elections 2004.Obama succeeded because he was able to convince moderates in Ill. that he can represent them.

He was recently on the cover of Newsweek and admits that he may have difficulty living up to his hype.He joked at Washington's Gridiron Club dinner in early December: "I figure there's nowhere to go from here but down, so tonight I'm announcing my retirement from the United States Senate."
"This is not a man who wants to be seen as the leader of black America. When he spoke at a Congressional Black Caucus reception recently, Obama graciously thanked several caucus leaders by name, then concluded with a short but telling statement: "I'm looking forward to working with you on behalf of all Americans."

Jackson was an early supporter of Obama so I guess he owes some allegience to him .I suspect eventually he will need a
Sister Souljah moment with him if he want to move beyond the fringe.

Choux rated this answer Excellent or Above Average Answer

Question/Answer
Choux asked on 01/06/05 - Kofi Demands

Today Kofi Annan demanded that all the countries who coillected money for the victims of the tsunami disaster in South East Asia turn that money over to the United Nations.

Comments?

tomder55 answered on 01/07/05:

This of course has been a blessing in disquise for Kofi since it distracts attention from the other scandals on the disfunctional organization he runs.

The Diplomad again hits the nail on the head today.(if all State Dept. apparatiks were like the Diplomad I'd probably think much better of State ).

This Embassy has been running 24/7 since the December 26 earthquake and tsunami. Along with my colleagues, I've spent the past several days dealing non-stop with various aspects of the relief effort in this tsunami-affected country. That work, unfortunately, has brought ever-increasing contact with the growing UN presence in this capital; in fact, we've found that to avoid running into the UN, we must go out to where the quake and tsunami actually hit.

As we come up on two weeks since the disaster struck, the UN is still not to be seen where it counts -- except when holding well-staged press events. Ah, yes, but the luxury hotels are full of UN assessment teams and visiting big shots from New York, Geneva, and Vienna. The city sees a steady procession of UN Mercedes sedans and top-of-the-line SUV's -- a fully decked out Toyota Landcruiser is the UN vehicle of choice; it doesn't seem that concerns about "global warming" and preserving your tax dollars run too deep among the UNocrats.

Sitting VERY late for two consecutive nights in interminable meetings with UN reps, hearing them go on about "taking the lead coordination role," pledges, and the impending arrival of this or that UN big shot or assessment/coordination team, for the millionth time I realized that if not for Australia and America almost nobody in the tsunami-affected areas would have survived more than a few days. If we had waited for the UNocrats to get their act coordinated, the already massive death toll would have become astronomical. But, fortunately, thanks to "retrograde racist war-mongers " such as John Howard and George W. Bush, as we sat in air conditioned meeting rooms with these UNocrats, young Australians and Americans were at that moment "coordinating" without the UN and saving the lives of tens-of-thousands of people.

Seeing these UNocrats perched at the table, whispering to each other, back-slapping, shaking hands, they seemed like a periodic reunion of old cynical Mafia chieftains or mercenaries who run into each other in different hot spots, as they move from one slaughter to another, "How are you? Haven't seen you since Bosnia . . .." As the hours wore on, however, and I nervously doodled in my note pad, shifted in my chair, looked at my watch, and thought about all the real work I had to do that evening, I decided that, no, labeling them mafiosos or mercenaries was much too kind. They seemed more to be the progeny resulting from a mating between a mad oracle and a giant carrion-eater. They were akin to some sort of ancient mythical Greco-Roman-Aztec-Wes Craven-Egyptian-bird-god that demands constant sacrifice and feeding, and speaks in riddles which only it can solve. Yes, I decided, the UNocrats are great hideous vultures, roused from their caves in the European Alps and in the cement canyons and peaks of Manhattan by the stench of death in the Turd World. They leisurely take flight toward the smell of death; circle, and then swoop down, screeching UNintelligble nonsense. They arrive and immediately force others, e.g., the American tax payer, to build them new exclusive nests in the midst of poverty, and make themselves fat on the flesh of the dead. My friends, allow The Diplomad to present to you The High Priest Vulture Elite (HPVE).


At an emergency summit in Jakarta Annan said that about 1 billion dollars was required now to implement specific programs to deal with the catastrophe.By agreeement the U.N. has disolved the core group of 4 nations that was leading the relief effort(whatever that means ) and yesterday assumed the role of coordinating relief work .That is the bad news .The good news is that donar nations will control where their funds are directed .The U.N. has come under criticism in the region for it's lack of ability to do anything.

"If we wait for the UN to tell us what to do, we wouldn't do anything," said Abdul Hadi bin e Rashid, first admiral of the Malaysian navy at the country's operations tent at Banda Aceh airport."There are people who are hungry and angry. Why wait? So we just do it."

Representatives of aid groups said the lack of co-ordination by the UN has led to some aid missions being duplicated. Many drew attention to the fact that the UN has no staff at Banda Aceh airport, which is the focal point for the relief response.

Michael Elmquist, head of the United Nations Office for the Co-ordination of Humanitarian Assistance in Indonesia , admitted the UN's response had encountered difficulties, but added it was poised to accelerate rapidly."Mr Elmquist attributed the delays to five key factors: the sheer destruction and number of people killed, including three of the UN's nine local staff; lack of telecommunications in the early days; a lack of truck drivers and fuel; the obliteration of the local government; and Aceh's troubled history."But he failed to mention the biggest reason; sheer U.N. incompetence !










Yesterday Congress approved a measure that would allow private donations for tsunami relief made this month to be tax deductible in 2004 .I think Congress should authorize that a significant part ;if not all of our commitment to the releif effort be diverted from direct UN distribution to organizations like World Vision which has a long track record of getting things done in the region .World Vision carefully monitors and reviews programs and costs, uses donations and grants for their intended purposes, and looks for ways to leverage funds. World Vision strives to keep its overhead rate low. Last year, 87 percent of the total revenue directly benefited children and families in need.

Uggghh .The faxes at the Swiss banks must be running out of paper by now.

Choux rated this answer Excellent or Above Average Answer
ETWolverine rated this answer Excellent or Above Average Answer

Question/Answer
excon asked on 01/06/05 - Done??? Not by a long shot


Hello very well behaved right wingers:

You have a Constitutional right to sue for grievances. If your lawsuit is frivolous, the courts will throw it out. If it is not, the courts will let the suit go forward. If a jury of your peers agrees that you have been damaged and awards you compensation, then the jury of your peers has decided that the damages are just.

That's the way it is. I trust my peers. If it was YOUR child that was maimed by a bad doctor, you certainly would not like your award capped.

Bush is taking away another fundamental Constitutional right. Are you gonna jump in lock step with him and ETWolverine, or are you gonna stand up for your rights?

excon

tomder55 answered on 01/06/05:

If you don't believe that the frivolous law suits are a drag on the economy ;the health care system;the auto insurance system . I can't convince you .

My wife was sued for multi-million dollars in what was essentially a fender bender . The lady faked injuries . We documented all the facts and gave the info.to the insurance co. claiming the fraud that it was . The lawyer for the plaintiff was speechless during the discovery phase when my wife presented the information about the case. Her deposition was unimpeachable .Our insurance co. lawyer in contrast poked holes all over the ladies story .

The lawyers got together afterwords for a couple of drinks at the bar and hashed out a settlement where the lady was awarded $30,000 ! Why ? for nothing ! Because it would cost both of the crooked insurance companies more to litigate than to settle .No charges of frivolous lawsuit; no compensation even to the tax payers for wasting the courts time .

Doctors leave the profession daily because they can't afford the liability insurance .Prices rise because liability insurance has to be added onto the cost of doing business . I have to waste my time filling out wavers if I want to get on a horse for an hours hide. What a joke ! The system is a disaster and in need of an overhaul. I do not think I should become a millionaire becasue I fouind a bug in my Birds Eye frozen vegetables.

excon rated this answer Excellent or Above Average Answer
purplewings rated this answer Excellent or Above Average Answer

Question/Answer
sapphire630 asked on 01/05/05 - Bush slow?

Who beat Bush to send aid to the Tsunami victims?
Where was Koffi? Kerry? I think scary terry kerry still hasn't found her checkbook since she lost it before the Pittsburgh flood from Ivan. Is she going to make an appearance there to and say her famous 'I wish there were something I could do' and leave?
I bet the answers are as good as 9/11 (when Bush was accused of spending 7 minutes in the grade school) Clinton's excuse was he was in a paralized shock for 45 minutes.

tomder55 answered on 01/06/05:

let them go naked !

see my post about the rapid fleet response. that could not have happened without immediate decisions being made

sapphire630 rated this answer Excellent or Above Average Answer
Choux rated this answer Excellent or Above Average Answer
paraclete rated this answer Excellent or Above Average Answer

Question/Answer
jocase asked on 01/06/05 - Not a question...an answer

It is actually a football game played by women in lingerie. Not mud wrestling. I saw it on the news. Another example of american decadence.

tomder55 answered on 01/06/05:

yeah ;I heard some Mullahs saying that the tsunamis were God's punishment for allowing decadent Westerners to vacation on the beaches . Those holy jihadi hijackers frequently patronized strip bars also .What's my point ? got me What's yours ?

Choux rated this answer Excellent or Above Average Answer
purplewings rated this answer Excellent or Above Average Answer
jocase rated this answer Excellent or Above Average Answer

Question/Answer
Itsdb asked on 01/06/05 - It ain't over...

I received an interesting message this week and thought I'd share it with you...

Tuesday, January 4th, 2005

Dear Members of the U.S. Senate,

Welcome back! The 109th session of Congress has just begun. I'm watching you on C-SPAN right now and you all look so snap-happy and clean-faced. It's like the first day of school all over again, isn't it?

I have a favor to ask of you. Something isn't right with the vote from Ohio. Seems a lot of people didn't get to vote. And those who did, thousands of theirs weren't counted.

Does that seem right to you? I'm just asking. Forget about partisan politics for a moment and ask yourself if there is a more basic right, in a democracy, than the right of the people to vote AND have ALL their votes counted.

Now, I know a lot of you wish this little problem of Ohio would just go away. And many of you who wish this are Democrats. You just want to move on (no pun intended!). I can't say I blame you. It's rough to lose two elections in a row when the first one you actually won and the second one you should have won. And it seems this time around, about 3 million more Americans preferred to continue the war in Iraq and give the rich more tax breaks than those who didn't. No sense living in denial about that.

But something isn't right in Ohio and more than a dozen members of the House of Representatives believe it is worth investigating.

So on Thursday at 1:00pm, Rep. John Conyers of Detroit will rise and object to the vote count in Ohio. According to the laws of this land, he will not be allowed to speak unless at least one of you -- one member of the United States Senate -- agrees to let him have the floor.

A very embarrassing moment during the last session of Congress occurred in the first week when none of you would allow the members of Congress who were black to have the floor to object to the Florida vote count. Remember that? You thought no one would ever notice, didn't you? You certainly lucked out that night when the networks decided not to show how you shut down every single member of the Congressional Black Caucus.

No such luck this year. Everyone now knows about that moment of shame. Thank you? Youre welcome.

But this Thursday, at 1:00pm, you will have a chance to redeem yourself.

Congressman Conyers and a dozen other members of Congress have some serious questions about how the Republican secretary of state in Ohio (who was also the states co-chair of Bushs reelection campaign) conducted the election on November 2. The list of possible offenses of how voters were denied access to the polls and how over a hundred thousand of their votes have yet to be counted is more than worthy of your consideration. It may not change the outcome, but you have a supreme responsibility to make sure that EVERY vote is counted. Who amongst you would disagree with that?

If you would like to read more about the specific charges, I ask that you read these two links: Senators Should Object to Ohio Vote by Jesse Jackson and Ten Preliminary Reasons Why the Bush Vote Does Not Compute, and Why Congress Must Investigate Rather Than Certify the Electoral College. I am asking everyone on my mailing list to send you a letter joining me in this call to you to do your job and investigate what happened before you certify the vote.

It only takes one member of the House and one member of the Senate to stop the acceptance of the Electoral College vote and force a legitimate debate and investigation. Do you know why this provision is set in stone in our nations laws? I mean, why would we allow just two officials in a body of 535 members to throw a wrench into the works? The law exists because nothing is more sacred than the integrity of the ballot box and if there is ANY possibility of fraud or incompetence, then it MUST be addressed. Because if we don't have the vote, what are we left with?

C'mon Senators! Especially you Democrats. Here is your one shining moment of courage. Will you allow the gavel to come down on our black members of Congress once again? Or will you stand up for their right to object?

We will all be watching.

Yours,

Michael Moore
www.michaelmoore.com
mmflint@aol.com

P.S. My whereabouts this week: I will be on the Today Show Thursday morning, Jay Leno on Friday night. And... the People's Choice Awards are this Sunday night, live on CBS at 9pm! Can we defeat the superheroes Spiderman, Incredibles and Shrek for best picture? A documentary??? Whoa... tune in...

~~~~~~~~~~~~~~~~~~~~~~~~~~~~~~~~~~~~~~~~~~~~~~~~~~~~~~~

John Kerry has also asked, much more graciously, for us to call our congressman and demand every vote be counted, although he won't be taking part in the protest of the Ohio vote...and also plans on introducing election reforms. But come on, why are democrats still whining about Florida 2000? And how many like Moore are still under the illusion that Bush lost...and even believe Kerry "should have won" this election?

Kerry said yesterday, "our legal teams on the ground have found no evidence that would change the outcome of the election". If, as the freepress.org says, the vote doesn't compute according to "mathematicians, election experts and independent research teams...experts who studied the voting machines, tabulators and other electronic equipment...and from a team of attorneys", wouldn't Kerry be fighting a bit harder?

In Molly Ivins' latest, she implies the losers should do what the Ukrainians do, "The Ukrainians showed us all what people who really care about democracy do when there's cheating at the polls."

Is there any hope that the left will ever "get over it" and find something to do besides whine? And by the way, Bush has increased his record tally to over 62,000,000 votes...

Steve

tomder55 answered on 01/06/05:

I heard Jesse shouting to the masses :Keep the myth of the stolen election alive !!!!!!!!!!!


Congressman Conyers is from Michigan . The vote count in Michigan was much closer than the Ohio vote ;as were a few other States that went to Kerry. Ohio has already certified the electors ;there is no basis for objection . I have not researched the Congressional rules and procedures regarding "acceptance" and do not plan to do so . It is a waste of time ;recounts have already been done in Ohio and nothing changed . The vote was not even close in Ohio ! Pennsylvania was much closer and there were some real issues there .

I will never forget the picture of Moore being elevated to having the distinction of sitting next to a former President at the Dem. convention . The Democrats really have to purge folks like Moore or they risk a further slip/slide to irrelevence.

purplewings rated this answer Excellent or Above Average Answer
Itsdb rated this answer Excellent or Above Average Answer

Question/Answer
kindj asked on 01/05/05 - Saddam's Dream

Saddam Hussein phoned President Bush and said, 'George, I called you because I had this incredible dream last night. I could see all of America, and it was beautiful and on top of every building, there was a beautiful banner.'
Bush asked, 'What was on the banner?'
Saddam responded, 'It said Allah is God, and God is Allah.'
Bush said, 'You know, Saddam, I'm really glad you called, because last night I had a dream too. I could see all of Baghdad, and it was even more beautiful than before the war. It had been completely rebuilt, and on every building there was also a beautiful banner.'
Saddam said, 'What was on the banner?'
Bush replied, 'I really don't know. I don't read Hebrew.'

tomder55 answered on 01/05/05:

"Alas, alas that great city Babylon, that mighty city! for in one hour is thy judgment come"(revelations 18:10)

kindj rated this answer Excellent or Above Average Answer

Question/Answer
excon asked on 01/05/05 - Values????? - I say horse pucky!!!


Hello right wing uncivilized people:

Leaders of the nation that would impose democracy on Iraq at the point of a gun are plotting ways to deny fundamental human rights to detainees suspected of terrorism.

Your dude and his gang want to hold some of the detainees FOREVER without the inconvenience of a trial, let alone a conviction.

The Defense Department is asking congress to fund a 200 bed prison dubbed Camp 6 to hold detainees who would never go through a military tribunal because there is not enough evidence to convict them, even in a venue where convictions are easy.

Like I said earlier, if those are your "values", you can keep 'em!

excon

tomder55 answered on 01/05/05:

I have offered a solution to this on another board ;play the Deguello before battle .

I am in favor of releasing all illegal combatants and prisoners of war at the conclusion of the war ;whenever that will be .

excon rated this answer Excellent or Above Average Answer
ETWolverine rated this answer Excellent or Above Average Answer

Question/Answer
Choux asked on 01/04/05 - Rudy Guiliani

Former New York Mayor Rudy Guiliani is leading Republican polls as the favorite to run for President as the Republican candidate in 2008.

Some people are scratching their heads. Guiliani positions on the hot issues are all the same as Liberal Democrats, except one. He *favors* abortion(he is ProChoice), he is for gun control, gay unions, etc, HOWEVER he is for lower taxes for the well to do. The majority of folks who support the Republicans only care about money; face it folks. Don't give me any values crap. :):):)

tomder55 answered on 01/05/05:

He lost alot of capital over the Kerik snafu ;but not so much that 'Bloomy'is actively seeking his endorsement. Rudy is following his instincts now and laying low until the heat is turned down a little.But I think it has leveled the playing field (besides ;it is way too early to speculate).

I think this has eroded the relationship between Bush and Guiliani. When you think about it ;if Rudy had not so highly recommended Kerik then I'm sure the White House would've been more diligent with the vetting process and the President would not have been embarrassed . I do not know what plans the administration had for Rudy before ,but it doesn't seem likely they will call on him to do anything significant in the next 4 years.

Here in NY he is like teflon ,but the rest of the country has now seen his weakness.It would not be long before the questions about the relationship of Rudy and Kerik would be ,"what did he know ;and when did he know it" ?Kerik has since "resigned " from Giuliani Partners.

Rudy ,will forever be in my eyes a hero . He tore into the crime rate in NYC and sent it in a seemingly irreversable downward spiral. Bloomy eagerly took the mantle from Rudy and now there were only 570 homicides in NYC last year ;the second lowest in 50 years.(I do not share VIB's dislike of Mayor Bloomberg
he has done a credible job under tough financial times .He has been willing to take on the unions that would otherwise bankrupt the city ;but that is for a different post)

Speculation differs about Rudy's future . Many are calling for him to run against Hillary for Senator. That would be an interesting battle.I am not sure that in NY (bluer than blue State )that she can be defeated .

But there is another possible scenario that has recently been served up. Gov. Pataki is almost definitely going to try to take the mantle of moderate Republicans into the campaign season. If he doesn't run again for Guv. then there is no real credible Republican in the state to take on NY Att. Gen. Elliot Spitzer except Rudy.

Either way ;Rudy is a survivor.



Choux rated this answer Excellent or Above Average Answer
ETWolverine rated this answer Excellent or Above Average Answer

Question/Answer
Choux asked on 01/04/05 - Muslims Condemn Saudi Arabia, Kuwait

Saudi Arabia and oil Emarits(sp)were soundly condemned by Muslims because they are TOO STINGY in giving to fellow Muslims in Indonesia who are suffering greatly. Saudi Arabia originally pledged only $10 mil in aid. They have now upped their contribution to a "whopping" $30 mil. I addition, they are going to hold a "telethon" for the victims. Kuwait who has had $9 BIL in oil prices this year also gave a paultry amount.

Setting aside our *duty* as a rich country to give extensive humitarian aide, we are going to have a huge public relations bonanza. Go figure.

tomder55 answered on 01/05/05:

We measure ourselves by how we live . They measure themselves by how they die.

Choux rated this answer Excellent or Above Average Answer
paraclete rated this answer Excellent or Above Average Answer

Question/Answer
ETWolverine asked on 12/31/04 - More on "American Stinginess"

Here's some additional information about the 'stingy' donations being made toward disaster relief in Asia.

(I will be using "banker's notation" in which "M" means "thousand" and "MM" means "million". It's just faster for me to type this way.)

US Government - $35MM to start, with more promissed.
American Red Cross - $28MM so far
Catholic Relief Services (Baltimore) - unknown... their web server was knocked down because they couldn't handle the volume of hits by donors.
Amazon.com - $4.8MM so far (donations are expected to surpass the $6.8MM donated for 9-11 relief).
Save the Children - $5MM so far, more expected
Bill & Melinda Gates Foundation - $3MM
Microsoft - unlimited matching funds donation program for employees. Whatever employees donate, the company will match.
America Online - $3MM raised so far from 53M donors. Additional $200M donated by the company. Will also match employee donations up to $50M.
eBay - setting up program to allow proceeds of sales to be donated to disaster relief.
Cisco Systems - $2.5MM
JP Morgan Chase - $3MM + matching of employee contributions.
CitiGroup - $3MM + matching of employee contributions
PepsoCo - $1MM
Pfizer - $10MM in cash + $25MM in medical supplies + matching of employee contributions
Johnson & Johnson - $2MM + medicine and medical supplies
Abbott Laboratories - $2MM + medicine and medical supplies
Merck & Co. - $250M + medicine & medical supplies
Bristol-Meyers Squibb - $100M + medicine & medical supplies.
Starbucks - $100M + $2 for every pound of Sumatra coffee sold during January.

And that's just to start... nor is this a very complete list. There are a lot of individual and corporate donors that are not on this list. These are just some of the big names.

I hope all people and corporations are this stingy with relief aid.

Elliot

tomder55 answered on 01/01/05:

I have decided to up my contributions to 'Habitat for Humanity'.They had ongoing efforts in all the countries affected before the disaster ;including many local Habitiat offices in the immediate areas.Habitat will work with the Consortium of Humanitarian Agencies (CHA) a group of more than 60 international, national and local nongovernmental aid organizations, with the national director of Habitat for Humanity Sri Lanka serving on a Disaster Management Team set up by the CHA to plan the best possible options. There are an estimated 5 million new homeless people in Sri Lanka alone. I know that there are more pressing needs than finding shelter ;but it looks like there is going to be plenty of short term resources if they are handled effiently (meaning UN keeps their grubby paws out of the pie).Also many of the damaged homes can be restored at a relativiely low cost assuming that a group like Habitat moves quickly.

I blew a casket last week when I started reading and hearing the US bashing.In all fairness to the UN official he was Not pointing to US "stinginess" but to Western countries .He was wrong of course .Any FAIR analysis of the situation would've pointed out that initial pledges were just that initial pledges and that more would be forthcoming as the magnitude of the situation and assessing the needs became evident. I was angry at how the press in the country used it as fodder to make cheap talking points about the president. The US is leading the relief effort whiel the UN is only now making plans to discuss options.

ETWolverine rated this answer Excellent or Above Average Answer

Question/Answer
Choux asked on 12/28/04 - Bloggers

I would like a recommendation for a couple of *good* political blog sites you can recommend. I want to check them out. Thanks!

tomder55 answered on 12/29/04:

In addition to the sites VG gave I also visit routinely

The Belmont Club which provides a keen analysis to the news .

There are some intersting blogs coming out of Iraq . My favorite is Iraq Now

Like VG said .Many blog sites have a roll call of links to other bloggers they visit . From these you can pick and choose the ones you find most valuable . From my perspective ;they provide links to news sources and commentary that I would never know about on my own. It saves alot of surfing . You can also if you choose refine your list to the political views you subscribe to . I read the broadest range that time permits and almost never read the daily press anymore. My Yahoo page has the links to about 50 web sites where I gather most of my info. from.

ETWolverine rated this answer Excellent or Above Average Answer
Choux rated this answer Excellent or Above Average Answer

Question/Answer
Choux asked on 12/28/04 - Stingy?

A prominent UN official called the USA "Stingy" because he didn't think that the *initial* humanitarian aid to Tsunami devastated countries of $15 million was enough! How quick he was to cast aspersions on America's generosity. Is there such a gulf between the UN and the USA that differences can't be resolved?

tomder55 answered on 12/28/04:

yes and it is not just them . Here is some of the crap in the world press.

From Australia :
A pity our army is busy fighting America's immoral war when they should be providing assistance to the affected areas. - Shane Arnold

These divine winds show that the Gods are displeased with the world's state of affairs. - Tomoyuki Yamashita

An opportunity for western governments to divert some funds to aid assistance projects rather than their billion dollar war obsessions. - Mother Nature strikes

This latest tragic disaster should open all our eyes to the fact that the world seems to already have its "hands full" coping with seemingly ongoing natural disasters rather than creating such man made disasters as we have contributed to in Iraq. - wayne gregory

Dont expect a genuinely compassionate response from the U.S. Government, as a "war on earthquakes" will not be as profitable as good ol' terrorism - Nick Loveday

And here's one where they try to link the tsunami to global warming.

The $15 million pledged by the US was just an initial pledge . Powell has already said there was more. On top of that already millions have been collected by US private organizations ;even from Florida residents who as you know were devistated this year by their own natural disasters.

Choux rated this answer Excellent or Above Average Answer
purplewings rated this answer Excellent or Above Average Answer
ETWolverine rated this answer Excellent or Above Average Answer
Itsdb rated this answer Excellent or Above Average Answer
fredg rated this answer Excellent or Above Average Answer

Question/Answer
Itsdb asked on 12/28/04 - The press and 'insurgents'

If you haven't heard about the IRAQ THE MODEL bloggers, perhaps you should take a look. It seems they and other bloggers are onto something concerning the press and Iraqi 'insurgents'.

Today's entry leads to some fascinating entries on the execution of an Iraqi election worker, photographed by an 'anonymous' AP photographer.

Additionally, Reuters apparently reported the AP furnished video of the murder of an Italian hostage to Al-Jazeera. "The channel said it obtained the videotape from the Associated Press."

Has the AP joined in the Arab media war against Iraq as GM proposed?

As Omar asked, "Who's going to get killed next? And who's going to cover it live?"

Or, are these bloggers just full of feces?

Steve

tomder55 answered on 12/28/04:

I read those and also Belmont Club who wrote a series of blogs on the issue. No they are not full of it . AP even admits without saying in so many words that they have reporters embedded with the terrorists .

Jack Stokes, the Associated Press director of media relations explained how that photographer was recruited.

Insurgents want their stories told as much as other people and some are willing to let Iraqi photographers take their pictures. It's important to note, though, that the photographers are not "embedded" with the insurgents. They do not have to swear allegiance or otherwise join up philosophically with them just to take their pictures.
Well aint that a relief????They do not have to swear allegiance to the "insuregent" but they can be tipped off that a murder is going to take place ;happen to be there ahead of time to provide al-Jazerra with propaganda footage but we are supposed to believe they are unbiased.

If AP doesn't get why what they did was f*d up then perhaps Egyptian blogger Big Pharaoh can educate them about their ethics :

The case at hand has to do with the brutal killing of 2 Iraqi heroes whose only mistake was trying to organize an election in their country. This is a moral case and we, the friends of Iraq and of the troops serving there, should not let this incident pass unnoticed.


There is no doubt about it in my mind. Without the cameras ;those murders do not occure .

Yiddishkeit rated this answer Excellent or Above Average Answer
Itsdb rated this answer Excellent or Above Average Answer
purplewings rated this answer Excellent or Above Average Answer
ETWolverine rated this answer Excellent or Above Average Answer

Question/Answer
excon asked on 12/27/04 - The Politics of the Death Penalty


Hello,

The death penalty is cruel and unusual punishment and thereby, unconstitutional. Here's one reason why:

The people who are done in are punished twice for the same crime. They serve the equivalent of what would be a life sentence in most states and THEN they're put to death.

What's up with that?

excon

tomder55 answered on 12/27/04:

The 5th Amendment guarantees that no one shall be deprived of "life, liberty, or property, without due process of law." The clear implication is that depriving someone of his or her life is permissible under the Constitution. As far as the 8th amendment goes .It used to be acceptable to whip and now it is not so maybe the definition of cruel has evolved. Unusual I do not think applies .It would be unusual to sentence a coomon thief with the death penalty but if all convicted pre-meditated murderers were subject to the death penalty that would be the ususal sentencing .The courts have generally used the application to interpret inequities in percentage of minorities sentenced without deciding outright that the death penalty itself was a violation of the 8th.

ETWolverine rated this answer Excellent or Above Average Answer
excon rated this answer Excellent or Above Average Answer
Itsdb rated this answer Excellent or Above Average Answer
powderpuff rated this answer Excellent or Above Average Answer

Question/Answer
Choux asked on 12/23/04 - Bowling for Palestine

Just when you think you have heard every possible absurdity, there comes on the news a story that Yasser Arafat invested one and one-half million dollars in a NYC bowling alley.

All I want for Christmas is some sort of leader for the Palestinean people.

Please God!

Choux

tomder55 answered on 12/24/04:

yesterday the first of 3 stages of the Palestinian elections were held (municipal).Reports are that turnout was very heavy (turnout could exceed 90 percent of the 150,000 Palestinians eligible to vote)and that the slated Hamas candidates did not do well;The people tended to vote along tribal line.

Back to Arafat : Of note is that these are only investments that Arafat reported ;some $799 million of investments;$30 million of which went to US investments . WE may never know where the rest of his ill gotten gain was squirelled away or what nefarious uses it went to. Supposedly Arafat invested in the bowling alley through a middleman named Zeid Masri who did not disclose to the owner that the silent partner was Arafat.He also invested $285 million in an Egyptian mobile phone company ;$3.2 million was placed with Virginia-based Simplexity Inc, which makes electronic-commerce software.Simplexity is located in Herndon Va. Herndon is also the location of 'International Institute of Islamic Thought'(IIIT); one of a group of Muslim organizations raided by federal authorities in an antiterrorism investigation. IIIT promotes the Wahhabi sect of Islam.It has made large financial contributions to the World Islamic Studies Enterprise in Tampa. The Justice Department called the Tampa group a "front organization that raised funds for militant Islamic-Palestinian groups such as the Palestinian Islamic Jihad and Hamas. Herndon is also the home of the Muslim World League ;also raided for suspected ties to al-Qaeda .HMMMMMMMMM coincidence? I think not.

ETWolverine rated this answer Excellent or Above Average Answer
Choux rated this answer Excellent or Above Average Answer

Question/Answer
excon asked on 12/23/04 - Query for Mr. ETWolverine


Hello Elliot:

Since this fiasco in Iraq started you have consistantly said that this war could be won "if".....

Your plan is sound. However, you don't seem to understand, that your beloved Bush didn't take your advice and we're losing our ass.

I know we can win. I know we're not winning. Go ahead, tell me we are. I thought you conservatives didn't have your heads in the clouds. It's us who are supposed to be in la la land. Guess what?

excon

tomder55 answered on 12/23/04:

excellent point made in Powerline "If one were to watch the first half of the local news one would see flashing blue lights, flashing red lights, a building on fire, a small plane crash, wrecked vehicles on the side of the road and the grieving family of a shooting victim. With only this input one would think that their city was in utter chaos. But we get to see the weather report, the sports news, the opening of the new hospital and the ribbon cutting ceremony for the new bridge. More importantly, we can get up off the couch, open the front door, step outside and see that chaos does not reign. We can not open the door and see what is going on in Iraq. There are people whose job it is to let us know what is going on but they seem to gravitate to the spectacular rather than the mundane. They are not telling the whole story.


Increasingly, Iraqi newspapers are denouncing the terrorists who try to disrupt next month's elections. Haider Ajina translated for us an editorial that appeared in the Iraqi newspaper Al-Sabah yesterday. Here are some excerpts:

Those who commit military operations under whatsoever titles, claims or pretexts are clear in their target. This clarity is proved by acts and not words as it is clear enough that slogans and nominations cannot withhold it.
The message is clear by targeting political process at its core; the elections, and then attacking people's drive of rebuilding the state and authority on democratic bases that embody their will and interest following decades of dictatorship that oppressed peoples rights of expressing options and decisions.

True and honest Iraqis stand with their project of national fate to found their state based on elections and peaceful rotation of power.

The compass of victory points to the right side as it works between those [terrorists] and true Iraqis.


Haider adds:

Editorials such as these are becoming commonplace in Iraqi newspapers, across the political spectrum. Iraqis understand what is at stake and will not let terrorism win. Iraq will be a strong ally against terrorism, and will help make the world safer from terrorists as Iraqs democracy matures and security improves. Is this not what president Bush wanted and promised to pursue? We are taking the fight to the terrorist and we are winning. Our president has promised the Iraqis democracy and means to keep his promise. The Iraqis understand this and are acting accordingly.

Choux rated this answer Excellent or Above Average Answer
ETWolverine rated this answer Excellent or Above Average Answer
excon rated this answer Excellent or Above Average Answer

Question/Answer
Itsdb asked on 12/22/04 - A Defeat For an Empire

by Robert Jensen

The United States has lost the war in Iraq, and that's a good thing.

I don't mean that the loss of American and Iraqi lives is to be celebrated. The death and destruction are numbingly tragic, and the suffering in Iraq is hard for most of us in the United States to comprehend.

The tragedy is compounded because these deaths haven't protected Americans or brought freedom to Iraqis. They have come in the quest to extend the American empire in this "new American century."

So, as a U.S. citizen, I welcome the U.S. defeat for a simple reason: It isn't the defeat of the United States -- its people or their ideals -- but of that empire. And it's essential that the American empire be defeated and dismantled.

The fact that the Bush administration says we are fighting for freedom and democracy (having long ago abandoned fictions about weapons of mass destruction and terrorist ties) does not make it so.

We must look at the reality, no matter how painful. The people of Iraq are better off without Saddam Hussein's despised regime, but that does not prove our benevolent intentions or guarantee that the United States will work to bring meaningful democracy to Iraq.

In Iraq, the Bush administration invaded not to liberate but to extend and deepen U.S. domination. When Bush said, "We have no territorial ambitions; we don't seek an empire," on Nov. 11, 2002, he told a half-truth.

The United States doesn't want to absorb Iraq or take direct possession of its oil. That's not the way of empire today; it's about control over the flow of oil and oil profits, not ownership.

In a world that runs on oil, the nation that controls the flow of oil has great strategic power. U.S. policy-makers want leverage over the economies of competitors -- Western Europe, Japan and China -- that are more dependent on Middle Eastern oil.

The Bush administration has invested money and lives in making Iraq a platform from which the United States can project power.

That requires not the liberation of Iraq but its subordination. But most Iraqis don't want to be subordinated, which is why the United States in some sense lost the war on the day it invaded. One lesson of contemporary history is that occupying armies generate resistance that, inevitably, prevails over imperial power.

When we admit defeat and pull out -- not if, but when -- the fate of Iraqis will depend in part on whether the United States makes good on legal and moral obligations to pay reparations and allows international institutions to aid in creating a truly sovereign Iraq.

We shouldn't expect politicians to do either without pressure. An anti-empire movement -- the joining of anti-war forces with the movement to reject corporate globalization -- must create that pressure.

We should all carry a profound sense of sadness at where decisions made by U.S. policy-makers -- not just the gang in power today but a string of Republican and Democratic administrations -- have left us and the Iraqis. But that sadness should not keep us from pursuing the most courageous act of citizenship in the United States today: pledging to dismantle the American empire.

The planet's resources do not belong to the United States. The century is not America's. We own neither the world nor time. And if we don't give up the quest -- if we don't find our place in the world instead of on top of the world -- there is little hope for a safe, sane and sustainable future.

Robert Jensen is a journalism professor at the University of Texas at Austin and the author of "Citizens of the Empire: The Struggle to Claim Our Humanity." He can be reached at rjensen@uts.cc.utexas.edu.

2004 The Star-Telegram

~~~~~~~~~~~~~~~~~~~~~~~~~~~~~~~~~~~~~~~~~~~~~~~~~~~~~~~

Comments?

tomder55 answered on 12/23/04:

UT is a public and not a private university ? Nice to know that his existance is taxpayer supported .No doubt this clown is tenured so he is immune from consequences.But he should not be immune from the criticism of the public :


Robert Jensen
School of Journalism
1 University Station A1000
University of Texas
Austin, TX 78712-0113
work: (512) 471-1990
fax: (512) 471-7979
email : rjensen@uts.cc.utexas.edu

His message borders on sedition. Good for him that is legal but one has to seriously wonder where his allegiance is .I have to point out right off the bat that the liberty he has to spew such venom (freedom of expression/academic freedom /assembly etc)comes from the sacrifices of the soldiers he roots against .They I suppose have an obligation to defend to the death his right to say this .

But does he respect others rights ? Jensen was a speaker at an open meeting of the Palestinian Solidarity Committee on campus . One of the speakers refused to continue until two members of Texans for Israel who showed up to listen were ejected from the meeting.Jensen defended the expulsion .If it had been a classroom, I would have stepped in. The students run events, it is not the role of faculty to step in and dictate. More proof that Jensen believes in the right of free speech and assembly but it is someone elses problem to guarantee it .

To say his message is flawed is to state the obvious. If the US wanted to" subordinate" them we would've done so . It would've cost us alot fewer lives. Our troops are dying to set up elections . The terrorists that he praises as "resistance" are killing election workers in cold blood conspicuosly and coincidently close to the MSM cameras .One lesson of contemporary history is that occupying armies generate resistance that, inevitably, prevails over imperial power. Which explains why the Japanese and the Germans drove us out of their countries.

In his warped mind it is not radical Islamo-fascists that are the enemy ;it is the US military that is the puppet of multi-national corporations . I have a vision of him and Michael Moore sitting in a living room sucking on payotee and drinking cool-aid ;except that he has managed to find a way to trash Michael Moore also .

I am amazed that in light of all the evidence about the UN Oil for Food scandal that someone still thinks it was the US and not those nations opposed to the war that were motivated by oil but in his mind America is always to blame
.
Perhaps we can all contribute to the Robert Jensen Deportation Fund

ETWolverine rated this answer Excellent or Above Average Answer
Itsdb rated this answer Excellent or Above Average Answer
Choux rated this answer Excellent or Above Average Answer

Question/Answer
Yiddishkeit asked on 12/21/04 - The Sunni's claim responsibility for the latest...

In todays news...the lives of another 22 soldiers was taken when a mess hall tent was hit near Mosul. This is becoming a common tactic and it's an easy target. My brother's life would had been jeopardy when this very same thing happened in his part of Iraq (his base camp) about a month ago, except that he was fortunate enough to had been given a temporary assignment and dodged that direct hit.

My concern is that apparently no amount of perimeter securing is enough with multiple insurgent culprits pouring in from various sources. I would appreciate the boards views on how we can adjust to accomplish a finality of this ongoing war in Iraq.




Bobby

tomder55 answered on 12/22/04:

I personally think that an Iraqi worker in the compound was an insider in the attack . They knew exactly when to strike. Also it was reported that this base was subject to alot of attacks but nothing was done about it . According to a Chaplain at Mosul the terrorists began to lob mortar rounds into the compound in the area of the base hospital after the explosion . This was an attack where again they are able to hide inside the civilian areas to avoid a response , but I do not see why that should be a deterent . One of the unfinished jobs of this war is taking the will to resist out of the Sunni areas ;areas as I noted before were spared the bulk of the initial push because the Turks did not allow the 4th ID to enter the offensive from the North.

I agree with Elliot. By any definition our enemies there are unlawful combatants and need to be treated as such . The Sunnis need to know that there are consequences for backing the wrong horse.Their will to fight needs to be broken. I am not advocating targetting civilians ;but relocating them and creating a safe perimiter around our bases would be a good start .

We recently started to go on the offensive aimed at safe havens in Iraq . That policy needs to be expanded .Safe havens on the other side of the Syrian border need to be attacked if Syria refuses to cooperate .

Meanwhile ,I have yet to see anything to disuade me from my belief that the silent majority in Iraq are willing to make a clean break from the past. There is nothing that will prevent the elections ;not Saddamites and terrorists murders of election workers done suspiciously close to where the Main Stream Media's cameras are at ;or the bleatings of Kofi Annan. In spite of what Brad says improvements in the economy, reconstruction, infrastructure, health and education, cultural life and security are happening there every day{see Winds of Change blog for some of the news you did not hear from the press}.

The Afghani's held elections and now we hear reports that Talibanis are tired of the fight and want to return to a normal life . Why would Iraqis be different?

Today in 1944 the American troops were in a desperate battle in the Ardennes taking horrific casualties. Within months the Germans were defeated .Perhaps a plaque recognizing the sacrifices of the American troops to secure Iraqi freedom will be on display in Mosul like there is today in Bastogne :"They fought for this land as if it was their own."

powderpuff rated this answer Excellent or Above Average Answer
Yiddishkeit rated this answer Excellent or Above Average Answer

Question/Answer
purplewings asked on 12/21/04 - FROM WAR SPRINGS HEROS, Or ANGELS.

BELIEFNET'S MOST INSPIRING PERSON, FINALIST:
Cpl. Jason Dunham
Marine who sacrificed his life to save his unit

Though thousands of soldiers serving in Iraq exhibit bravery in combat every day, Cpl. Jason Dunhams selfless act of courage made him a candidate for the first Medal of Honor to be awarded since 1993.

On April 14, Dunham, 22, was on a mission with his Marine unit in the Iraqi town of Karabiliah when reports came in of an insurgent attack against another group of Marines nearby. Dunhams team went in search of the perpetrators in an attempt to stop the attack. When they came upon a line of Iraqi vehicles, the team checked each one. One vehicles driver, an Iraqi, lunged out of the drivers side, and he and Dunham wrestled to the ground. Other Marines at the scene rushed to help, but one heard Dunham yell, No, no, nowatch his hand!
The Iraqi was holding a hand grenade, which was on a hair trigger. When the insurgent released the grenade, Dunham threw his helmet and his body over the weapon, taking the brunt of the explosion. Eight days later, with his parents at his side, Dunham died from his injuries.

Dunham, a native of Scio, New York who has been nominated for the militarys highest honor, re-enlisted in the Marines last July so that he could remain in Iraq for his battalions entire tour. The Wall Street Journal reports that a colleague of Dunhams asked him why he was extending his tour.

I want to make sure everyone makes it home alive. I want to be sure you go home to your wife alive, Dunham told his friend.

Because so many like this young man have re-enlisted, it tells me they believe in what they are doing. Is it possible that the USA being a Superpower, has a moral duty to help people who have been under Dictators rule forever, to start them on the road to freedom? Just your opinion please. And a prayer for the families, if you're so inclined.

PW

tomder55 answered on 12/21/04:

I really wish we had the resources because I do believe there is a moral imperitive to help people gain freedom from oppression. Since we do not have unlimited resources then it should be limited to where our national interests coincide .

On the morning of November 15, 2004, the men of 1st Battalion, 3rd Marines awoke before sunrise and continued what they had been doing for seven days previously - cleansing the city of Fallujah of terrorists house by house.

At the fourth house they encountered that morning the Marines kicked in the door and "cleared" the front rooms, but then noticed a locked door off to the side that required inspection. Sgt. Rafael Peralta threw open the closed door, but behind it were three terrorists with AK-47s. Peralta was hit in the head and chest with multiple shots at close range.

Peralta's fellow Marines had to step over his body to continue the shootout with the terrorists. As the firefight raged on, a "yellow, foreign-made, oval-shaped grenade," as Lance Corporal Travis Kaemmerer described it, rolled into the room where they were all standing and came to a stop near Peralta's body.

But Sgt. Rafael Peralta wasn't dead - yet. This young immigrant of 25 years, who enlisted in the Marines when he received his green card, who volunteered for the front line duty in Fallujah, had one last act of heroism in him.

Peralta was proud to serve his adopted country. In his parent's home, on his bedroom walls hung only three items - a copy of the United States Constitution, the Bill of Rights and his boot camp graduation certificate. Before he set out for Fallujah, he wrote to his 14-year old brother, "be proud of me, bro...and be proud of being an American."

Not only can Rafael's family be proud of him, but his fellow Marines are alive because of him. As Sgt. Rafael Peralta lay near death on the floor of a Fallujah terrorist hideout, he spotted the yellow grenade that had rolled next to his near-lifeless body. Once detonated, it would take out the rest of Peralta's squad. To save his fellow Marines, Peralta reached out, grabbed the grenade, and tucked it under his abdomen where it exploded.

"Most of the Marines in the house were in the immediate area of the grenade," Cpl. Kaemmerer said. "We will never forget the second chance at life that Sgt. Peralta gave us." http://www.humaneventsonline.com/article.php?id=6062

ETWolverine rated this answer Excellent or Above Average Answer
purplewings rated this answer Excellent or Above Average Answer
powderpuff rated this answer Excellent or Above Average Answer

Question/Answer
excon asked on 12/19/04 - Ask, and you shall receive


Hello rightwing dudes:

We're in for a rough 4 years. In the recent few weeks, Bush has packed his cabinet with insider yes men. Plus it's interesting to note that nobody lost their job over Iraq. Not only does your dude not read, he doesn't want ANY outside opinion to interrupt his agenda.

Bush, insolated and isolated with false sense of mandate, is gonna cause bigger problems than he already has.

Amongst his supporters here, only Tomder will understand how scary that is. The rest of you will tell me to "trust" him.

Right!

excon

tomder55 answered on 12/20/04:

The Bush administration faces the "grave and growing threat of a bickering deficit" which will jeopardize U.S. foreign policy during the president's second term, according to an unnamed future former senior administration official.

"We've gone from feast to famine when it comes to the intra-Cabinet squabbling which is essential to formulating a globally-respected foreign policy," said the official. "It is crucial that the president appoint department secretaries who not only disagree with his philosophy and strategy, but also personally dislike each other, envy the president and don't mind talking about it with reporters."

Many long-time employees in the departments of state and defense have also privately expressed alarm at the president's nomination of Condoleezza Rice as secretary of state.

"I hope the Senate will do something to head off this train wreck at Foggy Bottom," said the anonymous source. "Otherwise, America will be represented abroad by a competent, hard working woman who understands the president's objectives, shares his philosophy and has earned his trust. How can anything good come from that?"

Shockwaves have also rippled through news organizations, as editors and producers scramble to retool their foreign policy coverage strategies in the face of what one called "the leak drought which could result from a cabinet focused on accomplishment rather than individual self-aggrandizement."
{http://www.scrappleface.com/ 11/17/04}





Bush is courting Joe Lieberman for Homeland Security . Would he also be one of those yes men ? Of course I do not see that happening because the Democrats would never go for it .It would destroy this myth of 'yes men'(the Republican bloggers are not happy of this speculation;they would consider this a betrayal ) and more importantly it would probably cost them a Senate seat(Conn. has a Republican Senator).If he comes in then he would be the 2nd Democrat in the Bush White House.(Norman Maneta at Transportation being the 1st;and frankly has been a burr .He is the one most responsible for old women being strip searched at the airports instead of young Muslim men. )

I guess it wouldn't persuade you that every 2nd term President reshuffles his cabinet with people who are most likely to implement his agenda. Clinton did it . People like Leon Panetta his Chief of Staff were removed (maybe because he told Clinton that sex in the Oval office was a bad idea ).Suffice it to say that No President appoints staffers who had different goals and philosophies .It wouldn't make sense to do so. Policy disagreements also should not be aired in public as people like Colin Powell ,and Christie Whitman earlier ,have been known to do.

Bush has an ambitious agenda for his second term and really a small window of opportunity to set it into motion .By 2006 he will be considered lame duck ,and will have a harder time enacting it . Already the moderate Republicans are carving out positions that are seperate from the Administration .

This is a rare situation in American politics .Normally a 2nd term President has a VP that will run in the next election cycle who is handed off the mantle of the administration . The VP's assention to Pres. is seen as a validiation of the former Presidents policies. This time there is no VP to be considered the front runner as Cheney has already decided not to run. The Republicans who are going to run will be trying to grab their constituencies earlier then ever. [ yet another comparison to Truman's Presidency .Truman ended his Presidency and his VP did not run in 1952 ].

The example that best illustrates your point is Bush asking John Snow to stay on at Treasury . Many felt that Bush would appoint someone else like Steve Forbes or Phil Gramm to the position. Snow is seen by his critics as a 'cheerleader ' especially when compared to Paul O'Neill who left the adminstation to write a hit piece about Bush's economic policies. Snow is a disciple of Jack Kemp ;and Kemp is best known to advocate simplifying the tax code. It is well known that Bush would love to rewrite the tax codes and Snow will be his front man in that effort.

I have been reluctant to support the removal of Rumsfeld but I am beginning to see an inevitability to it . I think he will hang on until after the Jan. 30 Iraqi elections and until the Quadrennial Defense Review is completed some time in the Spring. Then he will offer his resignation .

I might suggest that Powell lost his job over Iraq.It was in reality his and the State Dept.obstruction that has caused alot of the problems we face today in Iraq as I detailed in other postings .What will make Rice effective at State is that she does agree with Bush .When negotiating with foreign leaders they will not have to doubt that what Rice says really does reflect policy ;that confusion has been eliminated.

We elected one president and like it or not we did it because we approve of his policy and agenda. It is not hubris on the Presidents part to think he was given a mandate to persue his agenda.

excon rated this answer Excellent or Above Average Answer
ETWolverine rated this answer Excellent or Above Average Answer
purplewings rated this answer Excellent or Above Average Answer

Question/Answer
sapphire630 asked on 12/16/04 - Leftists now terrorists

Memebers of Move On took hostages at the Seattle Starbucks. They made them eat sausages, drink Stop 'n Go coffee and wear something politically incorrect from WalMarts. Their demands had something to do with wanting to go to Canada.

tomder55 answered on 12/17/04:

American Liberals Sneaking
Across Border Into Canada

Author Unknown
11-24-4

The flood of American liberals sneaking across the border into Canada Has intensified in the past week, sparking calls for increased patrols to stop the illegal immigration.

The re-election of President Bush is prompting the exodus among Left leaning citizens who fear they'll soon be required to hunt, pray and agree with Bill O' Reilly.

Canadian border farmers say its not uncommon to see dozens of sociology professors, animal rights activists and Unitarians crossing their fields at night.

I went out to milk the cows the other day, and there was a Hollywood producer huddled in the barn," said Manitoba farmer Red Greenfield, whose acreage borders North Dakota. The producer was cold, exhausted and hungry. He asked me if I could spare a latte and some free-range chicken. When I said I didn't have any, he left. Didn't even get a chance to show him my screenplay, eh?"

In an effort to stop the illegal aliens, Greenfield erected higher fences but the liberals scaled them. So he tried installing speakers that blare Rush Limbaugh across the fields.

Not real effective," he said. The liberals still got through, and Rush annoyed the cows so much they wouldn't give milk."

Officials are particularly concerned about smugglers who meet liberals Near the Canadian border, pack them into Volvo station wagons, drive them across the border and leave them to fend for themselves.

A lot of these people are not prepared for rugged conditions," an Ontario border patrolman said. I found one carload without a drop of drinking water. They did have a nice little Napa Valley cabernet, though."

When liberals are caught, they're sent back across the border, often Wailing loudly that they fear retribution from conservatives. Rumors have been circulating about the Bush administration establishing re-education camps in which liberals will be forced to drink domestic beer and watch NASCAR.

In the days since the election, liberals have turned to sometimes ingenious ways of crossing the border.

Some have taken to posing as senior citizens on bus trips to buy cheap Canadian prescription drugs. After catching a half-dozen young vegans disguised in powdered wigs, Canadian immigration authorities began stopping buses and quizzing the supposed senior-citizen passengers.

If they can't identify the accordion player on The Lawrence Welk Show, We get suspicious about their age," an official said.

Canadian citizens have complained that the illegal immigrants are Creating an organic-broccoli shortage and renting all the good Susan Sarandon movies.

I feel sorry for American liberals, but the Canadian economy just cant support them," an Ottawa resident said. How many art-history majors does one country need?"

In an effort to ease tensions between the United States and Canada, Vice President Dick Cheney met with the Canadian ambassador and pledged that the administration would take steps to reassure liberals, a source close to Cheney said.

We're going to have some Peter, Paul & Mary concerts. And we might put some endangered species on postage stamps. The president is determined to reach out."

ETWolverine rated this answer Excellent or Above Average Answer
Itsdb rated this answer Excellent or Above Average Answer
purplewings rated this answer Excellent or Above Average Answer
sapphire630 rated this answer Excellent or Above Average Answer

Question/Answer
Ccl471 asked on 12/16/04 - Rudy Giuliani for President?

I just read on a Yahoo! News article that Rudy Giuliani is seen by many to be a White House contender in 2008.

But won't Dick Cheney be the Republican candidate in 2008? Or is Giuliani going to run as a Democratic candidate?


Many thanks,

C.L.

tomder55 answered on 12/17/04:

Cheney will not run .

Rudy might make a run but I'd prefer he challenge Hillary in 06 for the Senator from NY or perhaps make a run for NY Govenor.

Some NYC mayors have made tries at running for President ;the last being John Lindsay in 1972 ;but they have not had success. In a blessing in disguise scenario Teddy Roosevelt made a run at NYC mayor and lost. Had he became mayor he probably never would've been President.

To succeed as mayor in NYC takes a unique person. There are many strange politica alliances that need to be forged . It just doesn't translate well nationally . Rudy was very successful running the city but his views are socially liberal and the national press that for the most part did not get to view the melodrama of his marital difficulties would have a field day rehashing it.

Look at what is going on with Kerik . For the local news here in NY ,Kerik's lifestyle did not even merit page six of the tabloids but put him in an national stage and look out !

This must be the earliest a Presidential election season has ever begun .Bush has not even been sworn in for his second term yet. But since we are into early speculation let me throw out a name that I have not heard yet(not that I am endoursing him ,I have already sung the praises of Condi Rice) . Former Govenor of NJ and former co-Chair of the 9-11 Commission Tom Kean . He has run a State ;run a commission where he had unprecidented access to national security information ,and has been President of a University.

The moderates in the GOP are already positioning themselves for a run at the White House (note all the sniping at Rumsfeld who has replaced John Ashcroft on the dart board).There will be a weeding out process and Kean could well emerge as the top moderate challenger to John McCain for the moderate mantle.

ETWolverine rated this answer Excellent or Above Average Answer
Itsdb rated this answer Excellent or Above Average Answer
Choux rated this answer Excellent or Above Average Answer
Ccl471 rated this answer Excellent or Above Average Answer
powderpuff rated this answer Excellent or Above Average Answer

Question/Answer
Choux asked on 12/15/04 - Bernie Kerik

I think that it is embarrassing to President Bush that he backed Kerik for Director of Homeland Security based on the recommendation of Guiliani. Guiliani looks bad now. He recommended a guy who had about ten red flags aganst him! Kerik is one sneaky and dishonest operator. Birds of a feather?

tomder55 answered on 12/16/04:

It is an interesting situation. Most of the pile -on seems to involve sex ,and for 8 years we were told that sex doesn't matter.

Kerik in NYC was supurb in every task he was assigned. I do not know how well he did training Iraqi police and I do not think he was there long enough to make any real impact. Had everything worked out he would've been a refreshing change from these tired old retreads that the Washington establishment uses.I think a fresh approach is needed in Homeland Security and he would've provided it.He is probably the right type of person for the job ;tough and street smart ;but the wrong person for it .

He was right to withdraw;the White House deserves some hit for not properly vetting him ;but the pile on by the press is a little overboard .Certainly Kerik is lacking in political skills and he was wrong not to be upfront about the potential problems during the selection process.He is certainly not the first person to have skeltons in the closet ;some have overcome and some have fallen .

Rudy ,like Kerik has been known to kick in a few doors in his time.The rough edge is what makes crime fighters effective.

Rudy's life has already been dragged through the mud ;especially by the NY tabloids . His divorce and affair were public red meat for a long time(at least Kerik tried to be discreate ;Rudy was very public about it.Donna Hanover went to court to prevent Rudy from having his mistress go to public functions.In his defense he never tried to run as a moralist.).

I do not think these types of revelations will hurt Rudy too much . If he has been damaged it is with the trust that Bush has in him. I think all the political capital Rudy picked up by actively campaigning for Bush may have been blown in this Kerik appointment.Personally I never thought Rudy could survive a Presidential run.(most Republicans see him as too moderate and his "hero of 9-11 "persona would've only gone so far in a prolonged primary battle)I still would either like to see him in a cabinet position or as a Senator from N.Y.

Choux rated this answer Excellent or Above Average Answer
ETWolverine rated this answer Excellent or Above Average Answer

Question/Answer
Itsdb asked on 12/15/04 - A letter from Michael Moore

12/13/04

Dear Friends,

It is no surprise that the Republicans are sore winners. They have spent the better part of the past month beating their chests, threatening to send to Siberia any Republican who doesnt toe the line (poor Arlen Specter), and promising everything short of martial law if the Democrats dont do what they are told.

Whats worse is to watch the pathetic sight of the DLC (the conservative, pro-corporate group of Democrats) apologizing for being Democrats and promising to purge the party of the likes of, well, all of US! Their comments are so hilarious and really not even worth recognizing but the media is paying so much attention to them, I thought it might be worth doing a little reality check.

The most people the DLC is able to get out to an event of theirs is about 200 at their annual dinner (where you have to pay thousands of dollars to get in).

Contrast this with the following:

* Total Members of Move On: More than 2,000,000
* Total Attendance at Vote for Change Concerts: An estimated 280,000
* Total Union Members in U.S.: Around 16,000,000
* Total Number of People Who Have Seen Fahrenheit 9/11: Over 50 million
* Total Number of You Reading This: Perhaps 10 million or more

The days of trying to move the Democratic Party to the right are over. We lost a very close election (a one-state difference) by running the #1 liberal in the Senate. Not bad. The country is shifting in our direction, not to the right. But the country was attacked and people were scared. They were manipulated with fear. And America has never thrown a sitting president out during wartime. Thats the facts. Oh, and our candidate could have run a better campaign (but well have that discussion another day).

In the meantime, while we reflect on what went wrong, I would like to pass on to you an essay that a friend who works with abuse victims sent to me. It was written by a woman who has spent years working as an advocate for victims of domestic abuse and she sees many parallels between her work and the reaction of many Democrats to last months election. Her name is Mel Giles and here is what she had to say


Watch Dan Rather apologize for not getting his facts straight, humiliated before the eyes of America, voluntarily undermining his credibility and career of over thirty years. Observe Donna Brazille squirm as she is ridiculed by Bay Buchanan, and pronounced irrelevant and nearly non-existent. Listen as Donna and Nancy Pelosi and Senator Charles Schumer take to the airwaves saying that they have to go back to the drawing board and learn from their mistakes and try to be better, more likable, more appealing, have a stronger message, speak to morality. Watch them awkwardly quote the bible, trying to speak the new language of America. Surf the blogs, and read the comments of dismayed, discombobulated, confused individuals trying to figure out what they did wrong. Hear the cacophony of voices, crying out, "Why did they beat me?"

And then ask anyone who has ever worked in a domestic violence shelter if they have heard this before.

They will tell you: Every single day.

The answer is quite simple. They beat us because they are abusers. We can call it hate. We can call it fear. We can say it is unfair. But we are looped into the cycle of violence, and we need to start calling the dominating side what they are: abusive. And we need to recognize that we are the victims of verbal, mental, and even, in the case of Iraq, physical violence.

As victims we can't stop asking ourselves what we did wrong. We can't seem to grasp that they will keep hitting us and beating us as long as we keep sticking around and asking ourselves what we are doing to deserve the beating.

Listen to George Bush say that the will of God excuses his behavior. Listen, as he refuses to take responsibility, or express remorse, or even once, admit a mistake. Watch him strut, and tell us that he will only work with those who agree with him, and that each of us is only allowed one question (soon, it will be none at all; abusers hit hard when questioned; the press corps can tell you that). See him surround himself with only those who pledge oaths of allegiance. Hear him tell us that if we will only listen and do as he says and agree with his every utterance, all will go well for us (it won't; we will never be worthy).

And watch the Democratic Party leadership walk on eggshells, try to meet him, please him, wash the windows better, get out that spot, distance themselves from gays and civil rights. See the Democrats cry for the attention and affection and approval of the President and his followers. Watch us squirm. Watch us descend into a world of crazy-making, where logic does not work and the other side tells us we are nuts when we rely on facts. A world where, worst of all, we begin to believe we are crazy.

How to break free? Again, the answer is quite simple.

First, you must admit you are a victim. Then, you must declare the state of affairs unacceptable. Next, you must promise to protect yourself and everyone around you that is being victimized. You don't do this by responding to their demands, or becoming more like them, or engaging in logical conversation, or trying to persuade them that you are right. You also don't do this by going catatonic and resigned, by closing up your ears and eyes and covering your head and submitting to the blows, figuring its over faster and hurts less if you don't resist and fight back.

Instead, you walk away. You find other folks like yourself, 57 million of them, who are hurting, broken, and beating themselves up. You tell them what you've learned, and that you aren't going to take it anymore. You stand tall, with 57 million people at your side and behind you, and you look right into the eyes of the abuser and you tell him to go to hell. Then you walk out the door, taking the kids and gays and minorities with you, and you start a new life. The new life is hard. But it's better than the abuse.

We have a mandate to be as radical and liberal and steadfast as we need to be. The progressive beliefs and social justice we stand for, our core, must not be altered. We are 57 million strong. We are building from the bottom up. We are meeting, on the net, in church basements, at work, in small groups, and right now, we are crying, because we are trying to break free and we don't know how.

Any battered woman in America, any oppressed person around the globe who has defied her oppressor will tell you this: There is nothing wrong with you. You are in good company. You are safe. You are not alone. You are strong. You must change only one thing: Stop responding to the abuser.

Don't let him dictate the terms or frame the debate (he'll win, not because he's right, but because force works). Sure, we can build a better grassroots campaign, cultivate and raise up better leaders, reform the election system to make it fail-proof, stick to our message, learn from the strategy of the other side. But we absolutely must dispense with the notion that we are weak, godless, cowardly, disorganized, crazy, too liberal, naive, amoral, "loose, irrelevant, outmoded, stupid and soon to be extinct. We have the mandate of the world to back us, and the legacy of oppressed people throughout history.

Even if you do everything right, they'll hit you anyway. Look at the poor souls who voted for this nonsense. They are working for six dollars an hour if they are working at all, their children are dying overseas and suffering from lack of health care and a depleted environment and a shoddy education.

And they don't even know they are being hit.


How true. And that is our challenge over the next couple of years; to hold out our hand to those being hit the hardest and help them leave behind a party that only seeks to keep beating them, their children, and the kid next door whos on his way to Iraq.

Yours,

Michael Moore
www.michaelmoore.com
MMFlint@aol.com

~~~~~~~~~~~~~~~~~~~~~~~~~~~~~~~~~~~~~~~~~~~~~~~~~~~~~~~

Is the country shifting to the left?

Are these poor, downtrodden liberals, like battered women, the victims of abuse?

Does the left have "the mandate of the world", "a mandate to be as radical and liberal and steadfast as we need to be"?

Will the left just "walk away" or will they work toward a more unified America, respecting the beliefs of opinions of those they disagree with?

tomder55 answered on 12/15/04:

I'm suprised he took time off from campaigning for the "People's Choice Award" for his movie to write this.He certainly has been pandering to the Hollywood elitists lately ;maybe he wants the Capitol moved to L.A.

I would like to remind him that the last two Democrats to win the Whitehouse were centrist Southerners .The only real question I have is ;can the Democrats survive fools like him much longer ? Simple math ;if all the unionists had voted Dem. then Kerry would be Prez. I know a few union members .They do not toe the party line as well as they used to . Besides 4 States that the Dems won were real close(PA, MI, WI & MN.
much closer than Ohio ). Had they gone to Bush then it would have been an electoral landslide.And as a side note:the number of folks who have seen his movie or read his books are not an indicator of their political bend. I for one have read all his books and have seen all his movies except 'Farenheit 9-11'.I intend to see it when I can see it for free as I have never paid for the privilege of experiencing his wisdom before .He is amusing and I like to know what people like him are thinking .Maybe I'm strange that way ,but I tune in to Air America and watch Chris Matthews also.

There seems to be a differing of opinion in the ranks of the Dems. about where to go from here . I first heard Ron Reagan Jr.,in a show the day after the election, speculate that it is a good time for the Dems. to creat a "real progressive party" .

This is also the position that Moveon .Org took when they said to their membership ;"Now it's our Party: we bought it, we own it, and we're going to take it back."They were particularily critical of Terry McAuliffe .The DNC claims that he made the party financially "stronger then ever before ",but Moveon .org contends with some merit that they are better fund raisers in the McCain-Feingold era. Funny how they and Michael Moore are less critical of their candidate ;John Kerry who did not spend all the money he raised,and did Not donate it to other Democrat candidates who with some extra funding may have won their races.They are supporting Howard Dean for Chairmanship of the DNC.

No ,I do not think they are abused or down trodden ;yada yada . So long as they think they are then they are likely to continue to make the same mistakes. In a sober evaluation of what went wrong James Carville has been making much more sense.

Itsdb rated this answer Excellent or Above Average Answer

Question/Answer
Choux asked on 12/13/04 - McCain calls for Rummy's Resignation

Senator John McCain called for Sect of Defense Rumsfeld's resignation today citing his failures in conducting the War in Iraq. It seems to me that McCain wouldn't have spoken so openly without the approval of President Bush. Do you think I'm wrong in thinking this??

tomder55 answered on 12/14/04:

McCain publicly and actively supported Bush in the elections .It is no secret that he covets SecDef position.Anything he says has to be viewed through that prism.McCain in a way is right . Specialized personel are required ,as well as some civilians linguists, and civil affairs to help in reconstruction efforts .But I would be hesitant to add combat personel unless the military specifically asked for them.

General Abizaid has not called for more troops .Is McCain also questioning him? The Military leaders in theater are the experts on how many troops are needed. And every time they've asked for more, they've gotten them. I do not necessarily agree that more troops are needed soley on the basis of that is how the press is reporting the situation in Iraq.What is different from Vietnam as far as news goes is that the troops have unparrelled access to the homefront and their accounts seem much more positive then the press's. Also ;leading social and economic indicators coming from Iraq are positive .I could spend the rest of this post detailing specifics but that is not what this is about.

U.S. troops are an irritant in the Sunni triangle do you really think sending a lot more would make things better? I don't. The Iraqis will have to fight for their future, that's what we're trying to do.

I keep on going back to this point .Rumsfeld was charged with winning the war . The occupation as it turned out was a State Dept. effort . Rumsfeld wanted to put an Iraqi face on everything at the outset .He wanted some 10,000 expatriates trained to fight alongside the coalition(also included would've been the many Kurdish forces that did participate ). An Iraqi force would've been there at the fall of Baghdad .It would've been perceived as an Iraqi victory .

He then with Jay Garner wanted to hand over power to an Iraqi interm government asap with the leadership of those groups in control. Instead the State Dept. and the CIA opposed his plan .Instead of thousands of exiles ,a couple of hundred were flown in .He wanted to aggressivly persue cross border raiders from Syria and Iraq ;that was also nixed. They then replaced Garner with Viceroy Bremer who held onto power and further delayed the handover of the real government until this summer even though Ayatollah Sistani was calling for early elections.. The rest is history . There is a certain resentment to the American's being there because of the shame they feel for not being able to overthrow Saddam themselves . That combined with the Sunni's trying to hold onto the privilaged positions they had under Saddam ;and the influx of jihadis and you have the making of the current "insurection".

The US has already announced an increase in troop strength for the election period.Beyond that an accelerated pace of training Iraqi security forces would probably be advisable. It is my hope that next year this time the U.S. troop levels will be reduced significantly ;to a level simular to what we have in Afghanistan now.


Choux rated this answer Excellent or Above Average Answer
ETWolverine rated this answer Excellent or Above Average Answer
purplewings rated this answer Excellent or Above Average Answer

Question/Answer
sapphire630 asked on 12/11/04 - Hillarious Hillary

I heard (on one of those Political radio talk shows) a tape of Hillary saying not to worry about social security not being around when they reach retirement age. Social security will still be available at least for another 40 years.
The talk show host said she was talking to people in their early 20's. Doing the math that would be around when they are approaching their 60's. I wonder how much comfort that really gave them.

tomder55 answered on 12/12/04:

This week she gained a seat on the important Senate Aging Committee which oversees important issue related to the elderly like Social Security.She was also re-appointed to the Health, Education Labor and Pensions Committee, the Environment and Public Works Committee, and the Armed Services Committee.A full plate of important assignments that will make her very influential in the Senate.

The Republican's in Congress(and certain Administration officials) are already getting weak-kneed on this issue ;trying to down play the extent of the problem and diluting the expectations to real reform of the system.They are falling into the rhetorical trap of using language like ;increased retirement age/benefit cuts/limiting the potential size of personal accounts/incresed taxes. This only emboldens the Democrats to resist change .

The President has to take the bull by the horn ,and publicly tell the American people his plan for an ownership society ,and make it clear to the Republicans in Congress that he will not accept a watered down version of his plan . If not then he risks his plan suffering the same defeat as Hillary's health care plan.

As for Hillary :she should be politically astute enough to reqalize that the country is willing to try something new. It is no secret that the entitlement system in this country cannot sustain itself in the future. If she has plans for a run at the Presidency she has got to break away from the Roosevelt model that the Democrats seem to have a problem letting go of .She could be a leader on this issue if she chose to do so and the party would have little choice but to go along with her.Right now all she has to say is "I'm running" and the party would hand her the nomination on a silver platter. But unless she takes a lesson from the hubby,and positions herself as a centrist/reformist(Clinton did well by running on Welfare Reform)'then she would lose to virtually anyone the Republicans run.

sapphire630 rated this answer Excellent or Above Average Answer

Question/Answer
Choux asked on 12/11/04 - Dioxin Poisioning?

I guess Putin is stooping to his old KGB tricks in order to grab power. Is he trying to resurrect the old USSR to oppose America on the world stage along with allies France and ?....It was confirmed today that Yevshenko was poisioned by Dioxin(toxic chemical) at some point during the political campaign for president of Ukrane. Desperate measure, I would say. Too Medieval for these times...as if it would not be found out????? What are the behind the scenes forces that are causing this sort of move by Putin???

tomder55 answered on 12/12/04:

Sorta blows that bad Sushi theory out of the water ! If someone wanted him dead, they picked a very poor poison to do it. Dioxin is not fatal there has never been a recorded death from dioxin.He will have chloracne the rest of his life .He will have this disfigurement the rest of his life ;a painful reminder of his vulnerability ,which is the probable message being sent to him .

If this was Putin's doing (which it probably was)then it should be a reminder to the world of the clumsy brutish tactics of the KGB which he seems hell bent to reconstitute. Putin risked his reputation, influence and good graces with the Bush administration in trying to win the election for Yanukovych.Desperate to maintain their sphere of influence in the nations that border them the Russians may now have to settle for some face saving result.The people of Ukraine are speaking through this "Orange Revolution". Yushchenko will be thier leader and it may be difficult for Russia to try to have cordial relations with someone they tried to poison.Maybe next time Russia won't back a crimial.


Meanwhile the Orange Revolution rolls along . I think if you check into this site on a regular basis you will get a good idea of what is happening in Ukraine. As you see;right now there seems to be a deadlock in negotiations on election reforms that are needed before they hold new elections on Dec.26 . Without those reforms the results could again be suspect.

Choux rated this answer Excellent or Above Average Answer

Question/Answer
Choux asked on 12/10/04 - Bush Supports Kofi Annan

A huge scandal, and Bush is supporting Kofi Annan?? What if Kerry were president and Kerry supported Kofi???? Is this just a good ole boys club mentality? I'm disgusted. What's going on?

tomder55 answered on 12/11/04:

Lets see now :

-Oil for Food Corruption:
Calls for his resignation from Norm Coleman["As long as Mr. Annan remains in charge, the world will never be able to learn the full extent of the bribes, kickbacks and under-the-table payments that took place under the U.N.'s collective nose." ];
as well as leading journals like Wall Street Journal and that bastion of liberalism the NY Times.Coleman's Senate Subcommittee on Investigations is one of at least 5 Congressional Investigations into the sordid scandal.

-Rebellion within the ranks over sexual harrassment scandals involving senior UN staffers which prompted the staff union to give him a no confidence vote .

-He recently accepted responsibility for human rights abuses (including murder ) by UN personnel in Congo that are far worse than anything revealed at Abu Graib .

-The credibility of the organization completely discredited.A lesser man would've hung em up a long time ago.

The UN has done virtually nothing to deal with the genocide in Sudan ;reminicent of their lame response to the Rwanda genocides (Annan was head of UN peacekeeping forces).He now admits that he could and should have done more to stop the genocide in Rwanda but he has done nothing about Sudan either .A report prepared for Annan recently admitted that countries like Sudan gain seats on the Human Rights Commission to deflect criticism of their own dismal human rights records.

He has proven to be at best a mediocre Sec.Gen. ( a complete failure in my opinion )and an appeaser of the worse dictators in the world.His belated commitee to look into reforming the U.N. includes among the ranks Yevgeny Primakov ,an ex-KGB spy master.The committee recommends increasing the size of the permanent Security Council (temporarily without veto power)designed to further dilute the U.S. influence.

But still he lingers on with the backing of Gerhard Schroeder,Jacques Chirac and.....George Bush .

What's wrong with this picture ???????

The United States is the biggest contributor to the UN's budget and putting financial pressure to bear could be what is needed to begin the process of reform (if it is even worth the effort). While we call for transparency as a key feature in promoting democracy we have allowed this organization to operate with no accountability for years on our soil.Given that the real long term goal of the UN is a one world government that would severely curtail U.S.sovereignty ,I see no need to further support the organization provided that some of the useful NGO work it does gets retained within another global organization of democratic nations.

The question however is why would Bush voice his "confidence" in Annan. This is speculation on my part.Bush twice refused to support the secretary-general last week, stressing that he wanted a "full and fair and open accounting of the oil-for-food program" so U.S. taxpayers "feel comfortable about supporting the United Nations."Then exiting Ambassador John Dansforth supposedly speaking for the Administration gave full US support to Annan.

Annan is to meet this week with Condi Rice and Colin Powell.Powell is a friend of Annan and wants the UN to take a greater role in the upcoming Jan. elections in Iraq. I believe there is some horse trading going on ;increased UN support for our efforts in the Middle East for reducing the pressure on the UN from the U.S. I also think this is in a way a political payback to England .Tony Blair is cashing in on good will between the US and Great Britain. He would love to see a greater role by the US and the UN in the Palistinian/Israeli dispute. England also has given it's support to Annan.

Dansford said that the call for a complete investigation had been misinterpreted as a lack of confidence in Annan and a desire for his resignation.But like Coleman I do not think a thorough investigation can take place as long as Annan remains SecGen.







Choux rated this answer Excellent or Above Average Answer
powderpuff rated this answer Excellent or Above Average Answer

Question/Answer
Choux asked on 12/09/04 - Humvie-gate

Yesterday, a brave soldier took Secretary of Defense Donald Rumsfeld to task over the lack of war-worthy humvies(sp). He said that the soldiers have to scavange in fields,etc. to get metal in order to reinforce their vehicles.

After all these months in Iraq, isn't Rumsfeld culpable for not insisting these brave men get proper vehicles? Fact: 400 vehicles are manufactured a month according to Fox News(or CNN).

tomder55 answered on 12/10/04:

The military is armed and equipped the way it is today because of allocation decisions made some years ago, and our forces of tomorrow will be equipped the way that we are deciding today. We cannot avoid war simply because all may not be ideal, and we cannot pull out of war because deficiencies are identified. The Pentagon and military contractors are reacting as fast as they can.

This issue concerning armor is not as clear-cut as it appears at first glance.The Hummer was introduced as a replacement for the Jeep, which wasn't armoured at all.The Army initially thought it would need 235 armored humvees .The plan at the outset of the war was for a mobile light force that would fight a standing army in the desert. The vehicles were not made for urban combat. The fact that they can be armoured at all is testimony to the versatility of the vehicle.

In WWII the American tank was inferior at the outset to the German tanks . Patton jury rigged all types of armour onto his tanks including trees tied to them by chains .Innovative troops put plows on their vehicles to get through the hedgerows of France when the Germans were using them to ambush the troops.

Agreed;retooling could be taking place at a faster pace(Production was radically ramped up, going from 15 a month in May 2003 to 450 a month today.) ;but that is more of an indictment of the American manufacturing base ;and a procurement system that seems to need reform .Rumsfeld since he became Sec.Def. has been initiating reforms that were long overdue . In the process he ruffled some feathers but that has not detered him. The administrations is in charge of forming policy & the forces are in charge of implementing it.This should ruffle a few more feathers in the top brass.

Edward Lee Pitts of the Chattanooga Times Free Press is embedded with the 278th Regimental Combat Team in Iraq.He emailed this to his editor :

"I just had one of my best days as a journalist today. As luck would have it, our journey North was delayed just long enough see I could attend a visit today here by Defense Secretary Rumsfeld. I was told yesterday that only soldiers could ask questions so I brought two of them along with me as my escorts. Before hand we worked on questions to ask Rumsfeld about the appalling lack of armor their vehicles going into combat have. While waiting for the VIP, I went and found the Sgt. in charge of the microphone for the question and answer session and made sure he knew to get my guys out of the crowd.


So during the Q&A session, one of my guys was the second person called on. When he asked Rumsfeld why after two years here soldiers are still having to dig through trash bins to find rusted scrap metal and cracked ballistic windows for their Humvees, the place erupted in cheers so loud that Rumsfeld had to ask the guy to repeat his question. Then Rumsfeld answered something about it being "not a lack of desire or money but a logistics/physics problem." He said he recently saw about 8 of the special up-armored Humvees guarding Washington, DC, and he promised that they would no longer be used for that and that he would send them over here. Then he asked a three star general standing behind him, the commander of all ground forces here, to also answer the question. The general said it was a problem he is working on. "




Here we have a reporter bragging about getting away with not only planting a question, but reporting on it without telling his readers that he had planted it. That's almost as fraudulent as Dan Rather's faked memos.One of the intents of the embed program was to build mutual trust between the military and the press, and an incident like this goes in the opposite direction.Ironically, the soldier's question was a good one. Too bad it was a setup.



Choux rated this answer Excellent or Above Average Answer
ETWolverine rated this answer Excellent or Above Average Answer
excon rated this answer Excellent or Above Average Answer
Yiddishkeit rated this answer Excellent or Above Average Answer

Question/Answer
excon asked on 12/07/04 - Usury laws???


Hello sperts:

When I was in the Navy, you would get into big trouble if you lent money for interest. Typically, a lender would loan $5 for $6 back. It was expensive, but still less so than the legal money lenders that line the sidewalks outside all military bases and are ripping off our boys.

What happened to usury laws?

excon

tomder55 answered on 12/07/04:

loan sharks are pond scum legal or not

excon rated this answer Excellent or Above Average Answer

Question/Answer
excon asked on 12/07/04 - Question? I don't need no stinkin question!

H
ello political gurus:

In my most recent political incarnation, I adopted the economic viewpoint espoused by the likes of Milton Friedman, of the University of Chicago. That philosophy, in general, said that business would flourish in an atmosphere of non regulation. As a businessman, I saw most regulation as having nothing to with promoting the goals the legislation, but as a means of employment for the unqualified.

Today, I see that if business is left unbridled, they will rip us off. They (or their lobbyist) write much of the legislation that passes. They have a very friendly congress and president. Insurance companies, the pharmaceutical manufactures and the bankers (credit card issuers) are the worst thieves around. And, I know something about thievery.

excon

P.S. Oh, no question. Im just angry as hell.

tomder55 answered on 12/07/04:

if I'm not mistaken you are angry about a system of business regulations and not about a system of unfettered business. it is Congressional involvement in business that creates the abuses you mention . Not that I am laisse-fare by any means. I think gvt. has an important role to play especially regarding public safety but the more they get involved the more abuses like this you find.

excon rated this answer Excellent or Above Average Answer

Question/Answer
paraclete asked on 12/05/04 - Here is a horny political problem which needs a sage solution -

It is the problem of the continuing poor relationship between the "white" Australian community and the "indigenous" Australian community

Here are two examples of the seething unrest

Island riot: court bails accused
December 6, 2004 - 1:55PM


A man accused of being involved in last month's Palm Island riot has been released on bail.

Lex Patrick Wotton, 37, was the first of 17 men accused of rioting on the island off north Queensland on November 26 to face a bail hearing in Townsville's District Court before Queensland Chief Magistrate Marshall Irwin.

The riot followed a community meeting over the death in custody of 36-year-old Cameron Doomadgee.

Wotton, who's charged with serious assault, arson and damaging a building, was released on nine bail conditions including one that he not attend Mr Doomadgee's pending funeral.

The packed courtroom erupted in applause as he was bailed to face court again in Townsville on March 10 next year.

Bail applications for the other 16 accused rioters are continuing.

Judge Irwin was today told 19 police stationed on the island barricaded themselves by putting a pool table against the garage door of the police barracks and some armed themselves with pool cues when 300 islanders converged on the police station.

Police prosecutor Julia Cook told the court police were given one hour to leave the island after rioters threatened to kill them, pelting them with rocks and brandishing weapons such as monkey wrenches.

She also told the court a dog housed at the police station had to be put down due to injuries and distress arising from the riot.

The court was told the island's water mains were turned off stopping the fire brigade putting out a blaze at the police station barracks and court house.

Rocks were also placed across the road leading to the airport where islanders were stopping aircraft from arriving.

The hearing continues.

AAP

Anger at secret plan to bulldoze Redfern tower
December 6, 2004 - 3:43PM


The Redfern area and surrounds looking north towards Sydney's CBD.


Revealed: how Redfern will be reborn



Fixing the Block:

$27m development planned Towers demolished as aid to social levelling Residents of one of Sydney's most controversial buildings took to the streets today to protest over secret NSW government plans to bulldoze their home.

A week after leaked documents showed the government planned to sell off chunks of Redfern and Waterloo to private developers, hundreds of locals gathered to protest outside the area's public housing towers.

The distinctive Redfern and Waterloo towers could be pulled down under a $540 million plan that may allow private developers to profit from government land.

Sydney Lord Mayor Clover Moore, also the local MP, headlined a procession of speeches and accused the government of keeping residents uninformed and powerless.

"This is about development," said Cr Moore, standing on a makeshift stage on the back of a ute.

"It's not about urban renewal. It's not about addressing the social issues of our area."

Cr Moore said the government already had "all the power it needs" to address the neighbourhood's social problems - which earlier this year erupted in a race riot after the death of Aboriginal teenager Thomas "TJ" Hickey.

Demanding residents be consulted "every step of the way", she also objected to the unusual move of establishing a Redfern-Waterloo Authority to oversee development, a body that would have powers to override Sydney City Council planning regulations.

Cr Moore, a Redfern resident, would be on the board of the new authority, according to the proposal which goes before the NSW upper house tomorrow.

"This legislation is not democratic. It doesn't involve proper process and it overrides the Heritage Act," she said.

The building's neighbourhood is not far from the troubled Aboriginal area, known as The Block.

Several police attended today's rally, set where every building within eyesight would be redeveloped under the plan, but there were no reports of violence.

Instead, locals debated the matter vigorously among themselves, with some defending their home and others calling for a complete makeover.

"The people of Sydney think that Redfern and Waterloo are some sort of a low-down stinking slum," said Frank Kingston from his motorised stroller.

"If you don't think it's architecturally perfect then you must be blind."

Another man, who walked off before giving his name, commented: "I'd blow them all up. What about the stabbings, what about the burglars, what about the rape that's going on?"

AAP

The whole thing is an inability of government to conduct meaningful dialogue with aboriginal communites who believe they should be consulted about anything and everything even if they are squatters in the middle of a major city..

Please don't say relocate them, that doesn't work as they have no affinity with the new area and it's local population. An urgent political solution neeeded here?



tomder55 answered on 12/06/04:

Interesting ...... (wasn't Doomadgee found dead in a cell with broken ribs and died from a punctured lung ;apparently the victim of brutality ?).

During a less civilized time relocation was an effective way to deal with the native population here in the U.S. People now with the hind sight of history look upon it as a genocide of sorts .Andrew Jackson made a decision in 1838 that the only way to keep the peace between natives and the emerging country was to remove them from the territories east of the Mississippi river.[Prior to that from 1814 to 1824, Jackson was instrumental in negotiating nine out of eleven treaties which divested the southern tribes of their eastern lands in exchange for lands in the west. ] It just wouldn't do to have pioneer developments subject to continued attacks. It didn't matter much to him that the Cherokee was one of the tribes that seemed to be moving towards a type of assimilation [they were part of the 5 "civilized tribes" because they adopted Western practices like large scale farming ;western style clothing etc.].They were rounded up and marched 1000 miles to the Oklahoma territory.
Jackson truly believed he was doing what was best for all involved . No one at that time believed that the U.S. would extend beyond the Mississippi River . He thought that by relocation the natives and the United States would be free to conduct their affairs without conflict .

Of course now we know diffently . The U.S. continued it's expansion . The natives were settled into reservations . Some assimilated ;many remain there.

Today the US government recognizes over 500 Indian tribes,
most of which have reservations west of the Mississippi River . Many tribes in the East Coast have applications in for government recognition. Having Trust land gives them a sense and degree of autonomy but it is difficult for them to attract commerce except gambling casinos which began to flourish on reservations around 1989.The ones that have casinos saw unemployment drop dramatically .Tribes use their profits for the betterment of the reservation and its people. They are building schools & colleges and community centers, setting up education trust funds & scholarships, investing in alcohol and drug treatment programs,financing new business enterprises , and putting in water and sewer systems on the reservations .It is not much ,but a start(there is some legitimate moral opposition to this solution within and outside the native populations but economic considerations seem to outweigh them) .


I hear that a football star who is aboriginal walked across the continent to have a meeting with Howard to discuss the plight of the natives. I think Howard is a good leader and will at least attempt to deal with their grieviences.

ETWolverine rated this answer Excellent or Above Average Answer
paraclete rated this answer Excellent or Above Average Answer

Question/Answer
Choux asked on 12/03/04 - "The Plot Against America"

is the new book by Philip Roth. I'm about to start it, and I was wondering if anyone would like to be in a sorta "book club" so we could have discussion of the book on the Politics Forum??

It is "what if" history. The premise is that Charles Lindberg wins the 1940 Presidential election. (He was a Nazi supporter)

Anyone interested?

tomder55 answered on 12/05/04:

I will put the book on my list .

I like alternative history fictions.I recently read a 2 books written by Newt Gingrich about :What if Gen.Lee had not fought the battle of Gettysberg and had slipped around the union flank as Gen. Longstreet suggested ?

I have only read 2 Roth books ;'Goodbye Columbus'and 'Our Gang'.

Choux rated this answer Excellent or Above Average Answer

Question/Answer
Itsdb asked on 12/03/04 - Spampoetry

Currently, 85% of my email is spam. I received this bit of 'poetry' in a recent spam message...

This algol is declarative
Is ringside enterprise of therefore glassine
The bromley is jocose
This embalm furniture of pageant of dignity
Patriarchy of multiplet is passband
The uniroyal pawn
This vitae signpost is puncture
This tickle
The dispel of wrestle is edna

~~~~~~~~~~~~~~~~~~~~~~~~~~~~~~~~~~~~~~~~~~~~~~~~~~~~~~~

Sigh...

Got any good spampoetry of your own to share?

tomder55 answered on 12/04/04:

never know what I might find,
on any day I go online.
I used to get in quite a huff,
while wading through unwanted stuff.
But then I changed the man I am,
the day I answered all my spam.

Now every time I check my box,
I load up on fantastic stocks.
I'll gladly say I felt no loss,
when, with a smile, I fired my boss.
With just one click, the best thing yet,
I freed myself of all my debt.

I have, paying a few small fees,
ten university degrees.
Now that I'm losing all this weight,
I'm sure, someday, I'll get a date.
Instead of going to a show,
I spy on everyone I know.
(That's easy, since I have in hand,
this nifty wireless video cam.)

I spend my evenings viewing screens,
of barely legal horny teens.
And with a little credit charge,
Whoopee! My penis was enlarged!
Meanwhile these shots of Britney Spears
should be enough to last for years.

And so I lead this online life,
my monitor is now my wife.
It has become my greatest dream,
to launch my own get-rich-quick scheme.
And if you think you might get missed,
relax, you're on my e-mail list. (by Alex Silbajoris)


Itsdb rated this answer Excellent or Above Average Answer

Question/Answer
Itsdb asked on 11/29/04 - Mandatory retirement for Supremes?

WASHINGTON (AP) Six in 10 Americans say there should be a mandatory retirement age for Supreme Court justices, according to an Associated Press poll.

The survey found public support for an idea that has arisen periodically in Congress without ever making headway.

Only one of the nine current justices, Clarence Thomas, is younger than 65. Chief Justice William H. Rehnquist, 80, appointed to the court by President Nixon, has thyroid cancer. In the survey, people were asked if they could identify what job Rehnquist held, and 59 percent did not know.

The appointment of justices without term limits or a mandatory retirement age historically has helped to insulate the court from politics, said Dennis Hutchinson, a Supreme Court expert from the University of Chicago Law School. At the same time, that can have the unintended consequence of letting some justices serve beyond their most effective years.

People over 65 were among those most likely to favor mandatory retirement, according to the poll conducted for the AP by Ipsos-Public Affairs.

The question on retirement mentioned no specific retirement age. Lifetime appointment of Supreme Court justices is dictated by the Constitution and could be changed only by an amendment.

''The justices hold office year after year,'' said Opal Bristow, an 84-year-old Democrat and retired teacher who lives near San Antonio. ''Some of them are old codgers who need to get out of the way and let the younger folks with fresh ideas come in.''

Over the years, justices frequently have served into their 70s and 80s and often have died in office. In the past few decades, it has become more common for justices to step down when they face serious illnesses.

If President Bush has to nominate a replacement for any of the nine justices, the landmark 1973 Roe v. Wade decision that allowed legal abortions in the first three months of pregnancy is certain to be a central issue.

The survey found that 59 percent of respondents said they favor choosing a nominee who would uphold Roe v. Wade, while 31 percent wanted a nominee who would overturn the ruling.

While the public is generally divided on the abortion issue, polling consistently has found a clear majority of people who think abortion should be legal in at least some cases.

The preference for Supreme Court nominees who would uphold Roe v. Wade could be found among both men and women, most age groups, most income groups and people living in urban, suburban and rural areas. Fewer than half of Republicans, evangelicals and those over 65 said they favored a nominee who would uphold the abortion ruling.

''While I don't have a strong feeling about abortions personally, I wouldn't want the law overturned and return to the days of backdoor abortions,'' said Colleen Dunn, 40, a Republican and community college teacher who lives outside Philadelphia.

The survey found that 61 percent of respondents said Supreme Court nominees should state their position on abortion before being approved for the job.

Most of those who have taken a position on whether a nominee should uphold or overturn Roe v. Wade say they wanted a nominee to state his position on abortion before confirmation. Almost two-thirds of each group said they would want to know.

''In a perfect world they wouldn't have to talk about it,'' said Kenneth Cole, 39, a consultant from Columbus, Ohio, and a Republican who leans toward wanting Roe v. Wade overturned. ''But whoever President Bush nominates, people will know where they stand. They won't be able to avoid the issue.''

Another issue the Supreme Court will have to deal with at some point is homosexual marriage.

By 61 percent to 35 percent, people opposed gay marriage, with young adults between 18 and 29 about evenly split. Recent polls have indicated people are about evenly divided on the question of civil unions, which would provide many of the same legal protections as gay marriage.

The AP-Ipsos poll of 1,000 adults was taken Nov. 19-21 and has a margin of sampling error of plus or minus 3 percentage points.

By Will Lester, Associated Press, 11/28/2004 20:03

~~~~~~~~~~~~~~~~~~~~~~~~~~~~~~~~~~~~~~~~~~~~~~~~~~~~~~~

Should there be a mandatory retirement age for Supremes? Should they, as 84-year-old Democrat Opal Bristow believes, "get out of the way and let the younger folks with fresh ideas come in?"

Steve - who believes Supreme Court justices should "support and defend the Constitution of the United States against all enemies, foreign and domestic...bear true faith and allegiance to the same...take this obligation freely, without any mental reservation or purpose of evasion; and...faithfully discharge the duties of the office," not find 'fresh ideas' to subvert it.

tomder55 answered on 11/29/04:

gotta agree with excon , the Constitution says lifetime. it was meant to keep the court above politics .it has not worked as well as it could ,but putting terms or making it an elected position would be a move in the wrong direction.

ETWolverine rated this answer Excellent or Above Average Answer
Itsdb rated this answer Excellent or Above Average Answer

Question/Answer
Choux asked on 11/29/04 - Supreme Ct Refuses to Hear

the constitutionality of Massacheusetts gay marriage law.

Can you say politics???

Bush does not want this case in front of the American people or adjudicated for that matter.

tomder55 answered on 11/29/04:

sorta politics but not what you think . Rhenquist is not likely to ever return to the bench . it is possible that the court would decide 4-4 about the case which would affirm the law.rather than risk a tie the court will steer away from any controversial decisions if it cannot reach concensus or at least a majority one way or the other .

Choux rated this answer Excellent or Above Average Answer
excon rated this answer Excellent or Above Average Answer
ETWolverine rated this answer Excellent or Above Average Answer
Itsdb rated this answer Excellent or Above Average Answer
purplewings rated this answer Excellent or Above Average Answer
powderpuff rated this answer Excellent or Above Average Answer

Question/Answer
excon asked on 11/28/04 - Morals??? Really now!!


Hello wrong wingers:

Let me see if Ive got this straight. Republicans won on the issue of morals. Morals huh? Well, if what you have is called morals, then Ill stick to being amoral, thank you very much.

Columbia, South America, George Bush, your president, in his zeal to end the drug war there has struck a deal with the devil himself. Diego Fernando Murillo, one of the most feared commanders of Columbias right wing death squads is wanted for cocaine trafficking in the US and mass murder in his own country. His army turned thousands of civilians into targets sometimes killing them with chainsaws or claw hammers. His group is suspected of killing 40,000 innocent people.

While he imprisons pot and cocaine dealers here, Bush is going to make a deal with this killer that will allow him to retain his freedom, wealth and power. That means cocaine trafficking will continue unabated, and I will continue to call a spade a spade.

excon

tomder55 answered on 11/29/04:

Here is a link

The Prez of Colombia ;Uribe is trying to disarm the right wing militias . He is making political deals with them. Some of the drug cartels have set up paramilitary groups so they can try to qualify for these deals which includes in some cases a forgiving of past criminal activities which in the case of Murillo meant that he was a major player in the Medellin drug cartel.

The drug war has had some success recently in Colombia. Crops of coca were reduced in Colombia by 16 percent in 2003;That's a 47 percent decline over three years.Production of poppies, is down by 33 percent in the past two years.Colombia in recent months has seized record amounts of cocaine headed to US ports, approved the extradition drug barons like Cali cartel chief Gilberto Rodrguez Orejuela, and confiscated scores of luxurious properties belonging to notorious narcotraffickers.

Colombia has launched a military drive called Plan Patriot against the leftist rebels 'Revolutionary Armed Forces of Colombia' (FARC). Plan Patriot has dispatched 17,000 Colombian troops into remote southern areas to capture territory and top leaders.


So here we have the Colombians fighting leftist insurgencies;disarming right wing para-militaries AND waging effective war against the drug cartels (the big cartels have been destroyed and have splintered into smaller groups).

Bush at tremendous personal risk went to Bogota to show support for Uribe's efforts. Good ;I support them too.

excon rated this answer Excellent or Above Average Answer
ETWolverine rated this answer Excellent or Above Average Answer
powderpuff rated this answer Excellent or Above Average Answer

Question/Answer
ETWolverine asked on 11/24/04 - For the Soldiers

A message of thanks to our soldiers on Thanksgiving, and a reminder to those who benefit from their sacrafice.


Tommy
By Rudyard Kipling

I went into a public-'ouse to get a pint o' beer,
The publican 'e up an' sez, "We serve no red-coats here."
The girls be'ind the bar they laughed an' giggled fit to die,
I outs into the street again an' to myself sez I:
O it's Tommy this, an' Tommy that, an' "Tommy, go away"
But it's "Thank you, Mister Atkins", when the band begins to play,
The band begins to play, my boys, the band begins to play,
O it's "Thank you, Mister Atkins", when the band begins to play.

I went into a theatre as sober as could be,
They gave a drunk civilian room, but 'adn't none for me;
They sent me to the gallery or round the music-'alls,
But when it comes to fightin', Lord! they'll shove me in the stalls!
For it's Tommy this, an' Tommy that, an' "Tommy, wait outside"
But it's "Special train for Atkins" when the trooper's on the tide,
The troopship's on the tide, my boys, the troopship's on the tide,
O it's "Special train for Atkins" when the trooper's on the tide.

Yes, makin' mock o' uniforms that guard you while you sleep
Is cheaper than them uniforms, an' they're starvation cheap;
An' hustlin' drunken soldiers when they're goin' large a bit
Is five times better business than paradin' in full kit.
Then it's Tommy this, an' Tommy that, an' "Tommy, 'ow's yer soul?"
But it's "Thin red line of 'eroes" when the drums begin to roll,
The drums begin to roll, my boys, the drums begin to roll,
O it's "Thin red line of 'eroes" when the drums begin to roll.

We aren't no thin red 'eroes, nor we aren't no blackguards too,
But single men in barricks, most remarkable like you;
An' if sometimes our conduck isn't all your fancy paints,
Why, single men in barricks don't grow into plaster saints;
While it's Tommy this, an' Tommy that, an' "Tommy, fall be'ind",
But it's "Please to walk in front, sir", when there's trouble in the wind,
There's trouble in the wind, my boys, there's trouble in the wind,
O it's "Please to walk in front, sir", when there's trouble in the wind.

You talk o' better food for us, an' schools, an' fires, an' all:
We'll wait for extry rations if you treat us rational.
Don't mess about the cook-room slops, but prove it to our face
The Widow's Uniform is not the soldier-man's disgrace.
For it's Tommy this, an' Tommy that, an' "Chuck him out, the brute!"
But it's "Saviour of 'is country" when the guns begin to shoot;
An' it's Tommy this, an' Tommy that, an' anything you please;
An' Tommy ain't a bloomin' fool -- you bet that Tommy sees!


Thanks to all of our "Tommys". We will NOT forget your sacrafices this time.

Elliot

tomder55 answered on 11/26/04:

thanks for posting this I think Kipling did some timeless stuff

here are some sites to go to to donate to the troops or to just say hi and thanks

USO
A biggie for the troops is just sending pre-paid phone cards so they can call home.

Operation Gratitude

US Troop Care package

Adopt a platoon

and there are many other simular sites at
Defend America

ETWolverine rated this answer Excellent or Above Average Answer

Question/Answer
ETWolverine asked on 11/24/04 - The Budget Deficit - Putting our money where our mouths are.

Hello all,

During the electioon, we heard a lot of talk about balancing the budget, and how quickly it should be done. I heard quite a few members of this board sound off about how terrible it is that Bush hasn't managed to balance the budget.

Well, here's your chance to help.

According to government sources (the White House, the Budget Office, etc.) the budget shortfall for 2005 is slated to be approximately 363.6 billion dollars.

For 2003, there were 131 million personal tax returns filed, almost 6 million corporate tax returns filed, 2.2 million partnership tax returns filed and 296 thousand non-for-profit returns filed (form 990's). That is a total of 139,158,247 returns of all types filed in 2003.

If we divide the budget deficit by the number of tax returns filed, the budget deficit is $2,612.85 per tax return.

Who here volunteers to donate $2,612.85 of their hard earned money to the high-priority cause of balancing the budget?

Or if you would prefer, we can look at it from a population standpoint.

The current US population is 294,829,528 (as of 12:01 pm EST today). If we substitute that number for the number of individual tax returns filed, the number of entities in the USA (individuals, corporations, partnerships and not-for-profit organizations) is 303,259,415. Dividing the budget deficit by this number gives you a deficit of $1,198.97 per entity. that is certainly a better number to live with than the $2,612.85 that I got up above.

So... who's willing to donate $1,198.97 for each member of their family to balance the budget?

Don't all raise your hands too quickly now... you might strain your arms.

The point that I'm making is that people talk about the budget problems, but there are VERY FEW who are either willing or able to do anything about it.

Bush is doing what he can to balance the budget. He has made a promise to halve the gap within either 4 or 5 years (I don't remember which). So far, he has kept his promisses for the most part. I say that we should give him a chance to keep this promiss too. He might fail, but the one thing we know about Bush is that he ALWAYS tries. You might not like his decisions or the promisses he makes, but you have to give him credit for plugging away and at least attempting to keep those promises.

Just thought I would share that.

Elliot

tomder55 answered on 11/24/04:

There is little he can do about it short of asking Congress to raise taxes .I doubt if tax increase legislation would pass this Congress nor should it .

He can recommend spending reductions ,but that is even less likely to get through Congress. A spending Bill of $388 billion was ushered through Congress last weekend with bipartisan support . It was full of pork ,and a disgrace but the House Republicans were proud of it .They claim they made tough choices;held the line ,and called it an austere budget . The press lamented the loss of funding from everything from education (which was increased by about 2%)to the environment(which was increased by 3.5%). They even trimmed spending that Bush requested.Overall this spending bill was 2 percent larger than last year's versions. (the pork in the bill is a disgrace .The President needs a line item veto badly like Reagan used to say )

So taxes will not be increased ,and spending will not be reduced. That leaves one possibility to reduce the deficit. The theory that if the economy expands more revenues will be collected. I like the theory but have yet to see it work in itself. The deficit reduction of the 90s was a combination of an economic boom (especially in the tech sector where everyone was spending like crazy trying to fix the Y2K issues) AND the largest tax increase in history.

One other thought on the subject . Is it necessarily bad to be running deficits ? I look at it like my own financing . The only reason I have a house today is that I was willing to borrow someone elses money to pay for it . I will pay it off over time because I am mortal and do not want to pass on the debt to my daughter ,but I have gained an asset on someone elses dime. Any other borrowing I do be it credit card purchases or buying a card etc. is made based on my assets .They know they can lean my house to collect unpaid debt. I cannot borrow more than my assets can cover .The credit companies would not extend me above it .


The Gvt. by borrowing is basically doing the same thing . The assets of the nation clearly cover the debt .Heck the gvt. owns more than 1/2 the real estate in the West ! I do not even think the deficit approaches the percentage of the GDP that it did in the early 90s . So while I am concerned about deficit spending from a discipline point of view .I do not see them as tragic. They have not affected anything ;not even interest rates or the rates of bonds.Further ;the deficit is financing the economic recovery from the effects of 9-11 .

Choux rated this answer Excellent or Above Average Answer
ETWolverine rated this answer Excellent or Above Average Answer
powderpuff rated this answer Excellent or Above Average Answer
Yiddishkeit rated this answer Excellent or Above Average Answer

Question/Answer
sapphire630 asked on 11/23/04 - backwards and more backwards

Sunday, Nov. 21, 2004 9:19 p.m. EST
Kerry: Bin Laden Cost Me Election

Failed presidential candidate John Kerry is blaming Osama bin Laden for "scaring" voters into re-electing President Bush.
Fox News Channel's Geraldo Rivera caught up to Kerry on Thursday in Little Rock, Ark., where he was attending the opening of the Clinton Presidential Library.
(seems to me he was trying to scare people into voting for Kerry and it backfired?)

Also I heard that Saddam wants to sue the U.S. for war crimes and going against the Geneva Convention.


tomder55 answered on 11/23/04:

Actually, he's right. OBL did indirectly cost him the election, because Kerry wasn't seen as being strong or serious about fighting terror.Bin Laden's tape message was basically a proposal for a non-aggression pact .I wonder if Kerry would've signed on.

Gotta give him credit he has not fallen into an Al Gore funk .He is not growing his beard and taking extra portions at Taco Bell. But on the other hand his pledge to be a positive feel -good reach across the aisle and be a uniter rhetoric ended rather abruptly .

Over the weekend he sent an e-mail to his supporters on his web site . Kerry accused Bush of moving to eliminate debate and dissent from the State Department and CIA. He also charged that Bush's Cabinet was being remade "to rubber stamp policies that will undermine Social Security, balloon the deficit, avoid real reforms in health care and education, weaken homeland security and walk away from critical allies around the world."


sapphire630 rated this answer Excellent or Above Average Answer
labman rated this answer Excellent or Above Average Answer
Choux rated this answer Excellent or Above Average Answer

Question/Answer
Choux asked on 11/23/04 - Dan Rather stepping down

as anchor of the CBS Evening News. Just in.

tomder55 answered on 11/23/04:

just heard it also . But he is not stepping down from 60 Minutes or 60 Minutes II ;the show where the controversy surrounding Rather was aired . I have no problem with him continuing to be on the show so long as he ;like Andy Rooney has ;makes it plain that he has an ax to grind and an agenda.

Choux rated this answer Excellent or Above Average Answer
labman rated this answer Excellent or Above Average Answer

Question/Answer
excon asked on 11/22/04 - Logic will prevail (well, maybe not with you guy's)


Hello Conservatives:

So, you say that the obvious message to emerge from the election is that liberal democrats are simply out of touch with mainstream America.

WRONG! Its you who are out of touch, a bit lazy and not paying attention.

I have absolutely no doubt that gay marriage will become a reality in this country. Why? Like a lot of historys controversial issues (evolution, womens vote, civil rights) conservatives fought tooth and nail what eventually became mainstream thinking. Gay marriage, as an issue, is no different.

Conservatives may be happy about the shift to the right in this country , but I doubt it will last. History has shown us time and time again that when it comes to social issues, conservatives invariably get it wrong.

Yes, the earth is round, man did evolve, and black people have the right to vote. Dont you view as silly those past fights? Why cant you see the gay marriage issue as the same thing?

excon

tomder55 answered on 11/22/04:

The left has gotten lazy . They rely so much on the juducial process that they have forgotten the art of persuasion.If gay marriage is ever going to be accepted by the majority in this country, proponents are actually going to have to get their hands dirty in democratic debate. It will certainly not happen simply because activist judges wants to shove it down our throats.

excon rated this answer Excellent or Above Average Answer

Question/Answer
CeeBee2 asked on 11/22/04 - Condoleezza Rice, Part 2

Dawn Turner Trice
Impact of Rice's 'first' fizzles among blacks
Chicago Tribune
Published November 22, 2004

A good friend was at an airport in Nashville last week when she learned that President Bush had asked National Security Adviser Condoleezza Rice to become our country's next secretary of state.

Like Rice, my friend is a 50-year-old African-American woman who grew up in the segregated South, was very well educated and had opportunities that exceeded even her parents' grand dreams.

While still in the airport, my friend promptly typed an e-mail message to me on her Blackberry.

In short, it said: As a black woman, why am I not excited about Rice's historic achievement? If confirmed, she will be the first African-American woman to hold that post. Why am I not jumping up and down in the aisles?

Well, part of the reason a number of us aren't giddy with excitement is because Rice, however brilliant, however accomplished, is in lockstep with a president whose policies many African Americans--if you consider the 89 percent who voted against him--don't support.

So it's no wonder why some blacks haven't felt the spirit to jump up and down as a result of the nomination. What I think is probably more interesting--and something I sensed within myself and in the e-mail--is that we still want to jump.

We still want to experience that giddiness, that sense of pride we've felt in the past when people who look like us achieve some momentous first or some other measure of greatness. And we're disappointed when we don't.

Why do we want to jump? Well, why does any group want to jump?

Why do we as Americans find ourselves cheering for our countrymen in the Olympics when all we know we have in common with a particular athlete is that we live under the same flag?

The answer is connection. When a person with whom we feel a real connection accomplishes something, their achievement makes those common cells of our DNA tingle. We feel exuberant and uplifted. And while there are these inspirational veins that run throughout the body of the larger community, there are others that pump this feeling of excitement in a much more contained and localized way.

Catholics felt a great sense of pride when one of their own became president. The Latino community, I am sure, feels it when one of their own favorite sons or daughters takes their place in positions of leadership and power.

This connection doesn't come without a price, though. As we tie our hopes and dreams to those on the rise, it's hard to disentangle ourselves whenever they fall.

The African-American community long has been lifted by tying itself to its champions, leaders and heroes.

In a multimedia world, which often shows the black community in its worst light, we have stars and rising stars, from Oprah to Obama, that supply their own light and shine brightly.

And although we have learned the hard way that pinning our hopes on these bold but breakable human beings can be demoralizing when they fall, we are more willing than ever to do it again.

So why aren't we jumping up and down for our sister Condoleezza?

Despite what many may think, it's not simply because she's a Republican. While most blacks didn't announce it, the black community tied its heart to Rice's predecessor, Colin Powell.

He is a fiscal conservative who, despite his party's platform, supported gun control, abortion rights, affirmative action and separation of church and state. As a diplomat, he worked hard to put Africa on the map.

As Powell soldiered on in various Republican administrations, he may have marched with a different political party than most blacks are associated with, but we never felt he was walking away from us.

In the black community, we expect something in return from our leaders. And I don't mean in the way that, say, a Halliburton expects a little something-something from having a friendly face in the vice president's chair.

We expect our leaders to be role models for our sons and daughters. We want to be able to say, "You, too, can reach great heights and break through with hard work and perseverance." But if your children became all they wanted to be but forgot where they came from, would you be jumping up and down?

----------

dtrice@tribune.com.

tomder55 answered on 11/22/04:

Any uppity black person who refuses to toe the lefts line is frequently subject to high-tech lynchings. They can be done any time, anywhere, by just about anyone--and no one has the least bit of fear of being called to task for it. Indeed, other than a few conservatives, very few people ever bother to say anything at all.

Booker T. Washington's comments from nearly a century ago still hold true:

"There is a class of colored people who make a business of keeping the troubles, the wrongs, and the hardships of the negro race before the public. Some of these people do not want the negro to lose his grievances, because they do not want to lose their jobs. There is a certain class of race-problem solvers who don't want the patient to get well."

Clarence Page was amazed that pre election polls indicated blacks were increasingly willing to vote for Bush. Page, groping for more reasons, decides that blacks are turning to Bush because he appointed Powell and Rice to high profile positions. "A little symbolism goes a long way," he said demeaning Powell and Rice. Bush names talented, highly qualified individuals to two of the most important jobs in government who are black but all Page sees are symbols. Tokens.

He correctly concludes that the Democratic Party shouldn't take the African-American vote for granted. Why then doesn't he extend this by adding that blacks, like other groups, should vote for whomever they think would help them the most. Competition would benefit all.


ETWolverine rated this answer Excellent or Above Average Answer
CeeBee2 rated this answer Excellent or Above Average Answer
kindj rated this answer Excellent or Above Average Answer
purplewings rated this answer Excellent or Above Average Answer

Question/Answer
Choux asked on 11/21/04 - Bob Brinker

Bob Brinker is a well-respected author and talk radio host of "Money Talk". He is *not* a partisan. I listened to his show today for the first time in a long time, and what he was talking about really scared me. He said that the spending the Federal Govt is doing, the proposition to make tax cuts permanent...he likened to a locomotive out of control...that there is no one in Washington who will attempt to control all this excess spending in every direction. How do you feel about the ever-increasing national debt with no checks on it in sight? Isn't this reckless? (Kerry was going to increase taxes)Concerned.

tomder55 answered on 11/22/04:

I have listened to Brinker many times. His concern is the out of control spending .He is right to be concerned.Saturday Congress passed a $388 billion spending bill that was described as "lean "and" austere".The projects funded in the bill included: $335,000 to protect sunflowers in North Dakota from blackbird damage. $225,000 to study catfish genomes at Alabama's Auburn University. $2 million for the government to try buying back the former presidential yacht Sequoia, which was sold three decades ago.

His position on the tax cuts ,at least during the election was that he doesn't buy the-tax-cut-for-the-rich rhetoric, but he did make a good case that the tax cut could have been better directed toward those with a higher propensity to spend.His point on Kerry's proposal to roll back the tax cuts was that by increasing taxes at the upper tax brackets less money is available for investment and thus the economy grows at a slower rate.

Choux rated this answer Excellent or Above Average Answer
excon rated this answer Excellent or Above Average Answer

Question/Answer
ETWolverine asked on 11/19/04 - Is worldwide anti-semitism growing?

Palestinian 'Art' Exhibit

A New York public facility hosts a fundraiser for an art exhibit that vilifies Israelis and extols suicide bombers.

This Saturday (Nov. 20), a fundraiser will be held at the Westchester County Center in White Plains, New York, raising money to bring a Palestinian art exhibit to the New York metro area. Here's one of the paintings from the proposed exhibit (previously shown in Houston, TX), portraying Israeli Prime Minister Ariel Sharon collecting and boiling a young Palestinian's blood:

The painting rehashes the historic anti-Semitic blood libel, with Sharon as sadistic torturer and the United States his accomplice.

Suicide bombers are also praised in the 'art' exhibit the official introduction explains that

[t]o several of the artists, the subject of the martyrs is an all-important topic. A true martyr is anyone who gives his life in service of his people, including... suicide bombers that attack Israeli civilians.

One of the works 'pay[s] homage to the first 13 martyrs in the current uprising.'

Though two state legislators and local Jewish groups urged Westchester County Executive Andrew Spano to cancel the fundraiser which will be held in a public building Spano has decided to allow Saturday's event to take place.

In the (NY) Journal-News, the protesting legislators clarify that

this isn't about free speech - it's about whether to allow [the fundraiser] on county property. Spano has banned gun shows from the County Center because he opposes such a use of the space, they argue, and he should exert the same discretion here.

Comments to Westchester County Executive Andrew Spano: click here (Please remember that polite criticism is far more effective.)

2) Earlier this week, municipal signs of the town of Oleiros in northern Spain flashed the public message 'Let's stop the animal!!! Sharon the assassin, stop the neo-Nazis':



In response to strong protest from the Israeli Foreign Minister among others, the mayor of Oleiros, who at first defended the signs, agreed to remove them.

This episode may indicate that anti-Israel hysteria, exacerbated by the European media, has reached a new fever pitch on the Continent.

HonestReporting

--------

For the full story with pictures go to http://www.honestreporting.com/articles/45884734/critiques/Palestinian_Art_Exhibit.asp

Is anti-semitism growing? Is it bcoming more prolific? Has it gotten to such a point that elected and appointed officials both in the USA and abroad will allow it to happen?

Elliot

tomder55 answered on 11/19/04:

Well .....as a Catholic who has seen the most vile expressions of art against Catholics praised ;displayed ,and publically funded I can certainly understand the objections to the display . One of my locals Rep. Ryan Karben has been one of the most vocal opponents of this display but for some reason it appears that Spano is going to allow it even though the display has been rejected across the country by others in Spano's position. Among some of the 'art' that will be displayed is :

An image of an Arab headdress trapped in a Star of David made of barbed wire.

A reference to the creation of Israel in 1948 as a "catastrophe."

works by an artist described as "a former general in the Palestinian Liberation Organization."

A tent entitled "Memorial to 418 Palestinian Villages Destroyed, Depopulated and Occupied by Israel in 1948." The names of the villages are embroidered onto the tent.

A piece paying homage to "the first 13 Palestinian martyrs in the anti-Israel uprising that began in 2000."

"This exhibit does not promote art - it promotes terrorism and violence," Karben said, "These artists' muses are murder and martyrdom and their work is distasteful and divisive." ...."This exhibit is a propaganda show for assassins," .... "The blood spilled by the PLO would fill countless canvases."
(http://www.midhudsonnews.com/News/MadeinPal_Karben-13Nov04.htm)


When Karben said that it wasn't art but trash ;The American-Arab Anti-Discrimination Committee condemned Karben's "racist comments." They say his objections are in violation of existing 'hate crime 'laws.



ETWolverine rated this answer Excellent or Above Average Answer

Question/Answer
ETWolverine asked on 11/18/04 - CIA: 'Dysfunctional' and 'rogue'

townhall.com

CIA: 'Dysfunctional' and 'rogue'
Robert Novak

November 18, 2004

WASHINGTON -- After President Bush nominated him to be Director of Central Intelligence (DCI), Rep. Porter Goss walked across the Capitol to meet with a senator he hardly knew and who had criticized him: John McCain. There he received advice confirming his determination to take a course that soon became the talk of Washington.

McCain told Goss the Central Intelligence Agency (CIA) is "a dysfunctional organization. It has to be cleaned out." That is, the CIA does not perform its missions. McCain told Goss that as DCI, he must get rid of the old boys and bring in a new team at Langley. Moreover, McCain told me this week, "with CIA leaks intended to harm the re-election campaign of the president of the United States, it is not only dysfunctional but a rogue organization."

Following a mandate from the president for what McCain advised, Goss is cleaning house. The reaction from the old boys confirms those harsh adjectives of "dysfunctional" and "rogue." The nation's capital has become an echo chamber of anti-Goss invective with CIA officials painting a picture for selected reporters of a lightweight House member from Florida, a mere case officer at the CIA long ago, provoking high-level resignations and dismantling a great intelligence service.

Veteran CIA-watchers such as McCain regard the Agency as anything but great and commend Goss for taking courageous steps that previous DCIs avoided. George Friedman, head of the Stratfor private intelligence service, refers to Goss's housecleaning as "long overdue."

That cleansing process has been inhibited by the CIA's fear factor as an extraordinary leak machine. Its efficiency was attested to when Goss appointed Michael V. Kostiw, recently staff director of the House Intelligence Subcommittee on Terrorism, as the CIA's executive director. Before Kostiw could check in at Langley, the old boys leaked information that Kostiw was caught shoplifting in 1981 after 10 years as a CIA case officer.

Kostiw then resigned the Agency's third-ranking post, though Goss retained him as a special assistant. Kostiw's treatment has enraged people who have known him during a long, successful career in Washington -- including John McCain. The senator called Kostiw "one of the finest, most decent men I have ever met."

The story fed by Goss's enemies in the Agency is that dedicated career intelligence officers have been replaced by Capitol Hill hacks. Their real fear is that Goss will put an end to the CIA running its own national security policy, which in the last campaign resulted in an overt attempt to defeat Bush for re-election (intensifying after George Tenet left as DCI ).

I reported on Sept. 27 that Paul R. Pillar, the CIA's national intelligence officer for the Near East and South Asia, told a private dinner on the West Coast of secret, unheeded warnings to Bush about going to war. I learned of this because of leaks from people who attended, but many other senior Agency officials were covertly but effectively campaigning for Sen. John Kerry.

That effort seemed to include "Imperial Hubris," an anonymously published attack on Iraq War policy by CIA analyst Michael Scheuer. He has since left the Agency, but he was still on the payroll when the CIA allowed the book to be published. The Washington Post on Election Day quoted Scheuer as saying CIA officials muzzled him in July only after they realized that he was really criticizing them, not the president. "As long as the book was being used to bash the president," he said, "they gave me carte blanche to talk to the media."

Traditional bipartisanship in intelligence has been the victim, with Democrats cheering the CIA Bush-bashing. Rep. Jane Harman, ranking Democrat on the House Intelligence Committee, abandoned pretense of bipartisanship, and Sen. Jay Rockefeller, the Senate committee's vice chairman, never pretended. Both are attacking their former colleague who is now DCI.

McCain's use of the word "rogue" carries historical implications. A long, debilitating time of troubles began for the CIA in 1975 after Sen. Frank Church called it "a rogue elephant" that is out of control causing trouble around the world. The current use of the word refers to the intelligence agency playing domestic politics, which is an even more disturbing aberration.

-----------------


Now for my questions:

Has the CIA really become the intelligence wing of the Bush-bashing DNC or is this an exageration?

If it is NOT an exageration, can Porter Goss make a difference? Can the CIA's philosophy be changed by an outsider who is not part of the old boy's club?

Elliot

tomder55 answered on 11/19/04:



The question of the Valerie Plame outing is a good place to start to answer this .
It was common knowledge in Washington that Plame was a CIA analyst (not operative).So when senior Adm. Officials told him that it wasnt really news nad it was not illegal for them to say so . What was revealed was that she was neither an agent in the field nor was any sensitive national security detail disclosed.Novaks use of the word operative was to embellish his article .He later admitted that he had misstated her position. This says more about his journalistic style then anything else.

What has to be questioned is why was Wilson ,who was clearly by association opposed to Adm. Policy ,sent to Niger in the first place?. Wilson was affiliated with the pro-Saudi Middle East Institute and he had recently been the keynote speaker for the Education for Peace in Iraq Center, a far-Left group that opposed not only the war but also the sanctions and the no-fly zones . Why would the CIA send Wilson , a retired, Bush-bashing diplomat with no investigative experience on a sensitive fact finding mission ? Wilson claims that Cheney made the request for him to go , but we now know that Cheney made a request to the CIA to send someone and that he went there on his wifes recommendation. Isn't there something seriously wrong with a federal agency striving to undermine the White House? Think about it . Cheney has concerns about Saddam getting uranium from Niger for his . He asks the CIA to get intel. on it. Who do they send ;James Bond? No ;they send a retired diplomat who opposes the Adm. to do the fact finding.

While Wilson was in Niger he wined and dined for eight days by his own admission(eight days sipping sweet mint tea and meeting with dozens of people) . Based on those conversations, he concluded that "it was highly doubtful that any sale of uranium from Niger to Iraq had ever taken place."(N.Y.Times July 6,2003). Lets surmise that one of the officials that he was sucking tea with had taken a bribe from Saddam .Would that official then have been likely to reveal if either Saddam had obtained Nuke material or had attempted to do so (which was the only thing Bush actually said in the SOTU message.)?It was the CIA that later asked for the DOJ investigation into this . Another analyst ; Larry C. Johnson, sent a letter to Dennis Hastert demanding that Congress hold the White House accountable for deliberately revealing the identity of Plame.


I do not know how far up the ladder the CIA revolt against the Adm. goes. Tenet ,according to testimony ,had initiated some reforms inside the agency before he resigned but it is not clear if he was part of the problem or not (according to Woodward Bush had doubts about the intelligence about Iraq WMD and Tenet told him the case was slam dunk) .We will get a better handle on that question in the days ahead if he becomes a network analyst or writes a tell-all book .Clearly though it is just plain wrong for CIA operatives to write policy books under the anon. pseudonym while they are still employed at the agency ,and the leak by the agency of the January 2003 NSE right before the election reveals that elements in the CIA are active political hacks ,and they need to be purged no matter which party they support. Theirs is to gather information and to present their analysis of the info. to the President . It is not their job to make or to undermine policy .





ETWolverine rated this answer Excellent or Above Average Answer
Itsdb rated this answer Excellent or Above Average Answer
labman rated this answer Excellent or Above Average Answer
excon rated this answer Excellent or Above Average Answer

Question/Answer
labman asked on 11/17/04 - Your chance to support accused Marine

As somebody said on another thread, what is posted here is seen all over the world. Send them a message about real Americans. http://www.petitiononline.com/as123/petition.html

tomder55 answered on 11/18/04:

Marine Lance Cpl. Jeramy Ailes, 22, of Gilroy was killed Monday in Al-Fallujah by small arms fire. "They had finished mopping up in Fallujah and they went back to double-check on some insurgents. From what we gathered, somebody playing possum jumped up and shot him,'' said his father, Joel Ailes, who learned of his death Monday evening. "It's extremely hard."

... His first time in Iraq, Jeramy Ailes gave $10 to each child he came across because he knew it would feed their families for 30 days. This time, he asked his family to mail as many soccer balls as they could. His family sent 300 balls, and Jeramy Ailes' platoon handed them out to children.

Joel Ailes warmly remembered the last conversation he had with his son last month, in which Jeramy Ailes recounted how he had come across a large man walking with a 12-year-old girl carrying a huge bale of straw on her back. His son, who spoke and read Arabic, exchanged words with the man. And, for the next seven miles, his son carried the girl on his back and the man carried the bales of straw. "That was my son," Joel Ailes said.

WWII saw the Japanese booby-trap bodies(Out of the 22,000 Japanese soldiers on Iwo Jima , only 212 were taken prisoners); same in VietNam . A soldier has only moments to decide, and its a life-and-death decision;it was possible the wounded man was concealing a firearm or grenade.

The enemy deliberately breaks all the rules of war: fighting from mosques, feigning surrender and fighting under a white flag, by bobby-trapping bodies, fighting from ambulances, using civilians as human shields and to deliver suicide bombs.

Perhaps the press could show some objectivity too. If they are going to show this video every 15 minutes they should at least play the execution of the blindfolded Margaret Hassan video at the same frequency .(I have yet to see it on the U.S. networks ).Her crime was a lifetime of helping the Iraqi people .

labman rated this answer Excellent or Above Average Answer
Itsdb rated this answer Excellent or Above Average Answer

Question/Answer
hafj1 asked on 11/17/04 - hi elliot please help again!

thanks for your help before. got another question. If natonal identity is a unique thing for each nation, then can people with more than one citizenship share both national identities? I mean, say an asian person whose ethnic background is in India and their family and history etc is Indian, but they were born and bought up in Britain or America speaking English,with western education etc. Holly

tomder55 answered on 11/17/04:

My guess is that first or 2nd generations may feel a dual national identity but with each passing generation the ties that bind become weaker . I am an American who;s ancestors are Irish and Italian .I have no connection to the 'homeland' of my ancestors and would not feel any special loyalty to those nations . I simularily have close association with Indians who have moved to America. I see the same trends in their children . They are well on their way to adopting an American identity.

ETWolverine rated this answer Excellent or Above Average Answer

Question/Answer
excon asked on 11/17/04 - Hello, excon to Earth


Dear Experts:

In the world that I live in, we ARE losing the war in Iraq. We HAVE lost the war in Iraq, and we are never going to WIN the war in Iraq.

Do you remember the Russians in Afghanistan?

No, I suppose you don't!

excon

tomder55 answered on 11/17/04:

The pacification of the Sunni triangle is long overdue. Political solutions to the problems in the Sunni Triangle were sought by the Iraqi Governing council and the Interim government, but these attempts have failed.So a military solution is in order .Remember the 4th Infantry Division would've done so had they been allowed to proceed with plan A . The regime collapsed before the Sunni Triangle came under the sword and like the Germans in WWI ,they did not feel like they had lost . That will change in the coming months .The delay in the process made the insurgency temporarily emboldened but our patients also demonstrated that we were willing to let the Iraqis come to a political accomodation.Now Iraqi security forces have been trained and are participating in the action that is crushing the insurgency .Fallujah will never be permitted to be a central staging area for a large insurgency again . Mosul ;Tikrit ;Ramadi and every other village will all be dealt with simularily . Then the elections will be held .The Iraqis are committing to restoring order to their nation and have a stake in the outcome. While there is little doubt more American troops would have helped with restoring order sooner, it is beneficial in the long run with having the Iraqis actively participating in the restoration of order and their own liberation from the brutal coalition of terrorists and Saddam loyalists.

excon rated this answer Excellent or Above Average Answer
LTgolf rated this answer Excellent or Above Average Answer
powderpuff rated this answer Excellent or Above Average Answer

Question/Answer
Choux asked on 11/17/04 - Putin Announcement

Putin announced this morning that Russia will soon have a "new" kind of nuclear missle. At first, I thought of the old cold war, but now, I'm thinking that the "new" weapons are short range nuclear devices of some kind that can reach targets in the Middle East. Isn't Islam their emeny now, not America?

tomder55 answered on 11/17/04:

alot of bluster . He is thinking that they can develop a missle system that could make course changes mid flight thus making the U.S. plans of a missle defense more difficult. He should concentrate on paying his soldiers a decent wage ;but so should we .

ETWolverine rated this answer Excellent or Above Average Answer
Choux rated this answer Excellent or Above Average Answer

Question/Answer
Choux asked on 11/15/04 - Colin Powell for President

in 2008 as a Democrat? Any comments about Powell's resignation today and related topics?

tomder55 answered on 11/16/04:

Generally speaking Generals do not make good Presidents (Washington and Jackson being the noted exceptions ).Ike spent alot of time on the golf course ;Andrew Johnson 'Taylor, Pierce Hayes, Arthur,were undistinguished and Grant is best known for the corruption of his terms .(Garfield and Harrison died too soon to have a real record to judge ).Of the generals who became president, all but four did so after holding some lower elective office (Powell has served as NSA and Sec.State but not in an elected position).

Powell will have to play the role that past Generals have played .They cannot wear their ambitions on their sleeves Wesley Clark did not admit he was a Dem. for a long time ,and let a draft -Clark movement emerge. He has to play the role of 'Cincinnatus',the Roman General who "reluctantly" assumed power in a national crisis and relinquished it when the crisis was over .He cannot appear to want it . He has to appear to be bi-partisan .Both parties would try to draft him (Clinton considered Powell as a possible 2nd term appointment).

My own opinion .I do not think that he will run. Presidents portray themselves as movers and shakers;not satisfied with the status quo and impatient with the beltway mentality . If anything the last 4 years have proved that Powell is first and foremost a bureaucrat entrenched and comfortable with the status quo mentality at State Dept..In the military he rose up the ranks basically in bureaucratic functions (except for a short stint in Vietnam he was not a warrior). As head of the Joint Chiefs he developed the 'Powell Doctrine 'which was praised for it's skilled use of a large Cold War military but was overly cautious . He was one of the prime advocates of not removing Saddam from power in 1991 ,and that decision was a terrible mistake.He has proved to be equally cautious in running DOS . What we needed in these times was a George Marshall (another former General)but Powell was not a strong advocate of the Bush Doctrine .

If the Dems. draft him then they will be willing to overlook alot of potential baggage in Powell's past related to the Mai-Lai investigation;Iran Contra ;and some of Reagan's wars .I do not know if the Michael Moore wing would be willing to do so ;although you never know. Moore seemed pleased to forget the Kosovo campaign with Clark .He has in the past praised Affirmative Action so that may be a plus when the Dems .make their consideration .Hey you neve know!

Choux rated this answer Excellent or Above Average Answer
ETWolverine rated this answer Excellent or Above Average Answer

Question/Answer
Itsdb asked on 11/12/04 - The myth of the Bigoted Christian Redneck

Charles Krauthammer

November 12, 2004

WASHINGTON -- In 1994, when the Gingrich revolution swept Republicans into power, ending 40 years of Democratic hegemony, the mainstream press needed to account for this inversion of the Perfect Order of Things. A myth was born. Explained the USA Today headline: ``Angry White Men: Their votes turned the tide for the GOP.''

Overnight, the revolution of the Angry White Male became conventional wisdom. In the 10 years before the 1994 election, there were 53 Nexis mentions of angry white men in the media. In the next seven months there were more than 1,400.

At the time, I looked into this story line -- and found not a scintilla of evidence to support the claim. Nonetheless, it was a necessary invention, a way for the liberal elite to delegitimize a conservative victory. And even better, a way to assuage their moral vanity: You never lose because your ideas are sclerotic or your positions retrograde, but because your opponent appealed to the baser instincts of mankind.

Plus ca change ... Ten years and another stunning Democratic defeat later, and liberals are at it again. The Angry White Male has been transmuted into the Bigoted Christian Redneck.

In the post-election analyses, the liberal elite, led by the holy trinity of The New York Times -- Krugman, Friedman, and Dowd -- just about lost its mind denouncing the return of medieval primitivism. As usual, Maureen Dowd achieved the highest level of hysteria, cursing the Republicans for pandering to ``isolationism, nativism, chauvinism, puritanism and religious fanaticism'' in their unfailing drive to ``summon our nasty devils.''

Whence comes this fable? With President Bush increasing his share of the vote among Hispanics, Jews, women (especially married women), Catholics, seniors and even African-Americans, on what does this victory-of-the-homophobic-evangelical rest?

Its origins lie in a single question in the Election Day exit poll. The urban myth grew around the fact that ``moral values'' ranked highest in the answer to Question J: ``Which ONE issue mattered most in deciding how you voted for president?''

It is a thin reed upon which to base a General Theory of the Election. In fact, it is no reed at all. The way the question was set up, moral values was sure to be ranked disproportionately high. Why? Because it was a multiple-choice question and moral values cover a group of issues, while all the other choices were individual issues. Chop up the alternatives finely enough, and moral values is sure to get a bare plurality over the others.

Look at the choices:

-- Education, 4 percent
-- Taxes, 5 percent
-- Health Care, 8 percent
-- Iraq, 15 percent
-- Terrorism, 19 percent
-- Economy and Jobs, 20 percent
-- Moral Values, 22 percent

``Moral values'' encompasses abortion, gay marriage, Hollywood's influence, the general coarsening of the culture, and, for some, the morality of pre-emptive war. The way to logically pit this class of issues against the others would be to pit it against other classes: ``war issues'' or ``foreign policy issues'' (Iraq plus terrorism) and ``economic issues'' (jobs, taxes, health care, etc).

If you pit group against group, moral values comes in dead last: war issues at 34 percent, economic issues variously described at 33 percent, and moral values at 22 percent -- i.e., they are at least a third less salient than the others.

And we know that this is the real ranking. After all, the exit poll is just a single poll. We had dozens of polls in the run-up to the election that showed that the chief concerns were the war on terror, the war in Iraq and the economy.

Ah, yes. But the fallback is then to attribute Bush's victory to the gay marriage referendums that pushed Bush over the top, particularly in Ohio.

This is more nonsense. George Bush increased his vote in 2004 over 2000 by an average of 3.1 percent nationwide. In Ohio the increase was 1 percent -- less than a third of the national average. In the 11 states in which the gay marriage referendums were held, Bush increased his vote by less than he did in the 39 states that did not have the referendum. The great anti-gay surge was pure fiction.

This does not deter the myth of the Bigoted Christian Redneck from dominating the thinking of liberals, and from infecting the blue-state media. They need their moral superiority like oxygen, and cannot have it cut off by mere facts. And so once again they angrily claim the moral high ground, while standing in the ruins of yet another humiliating electoral defeat.

2004 Washington Post Writers Group

~~~~~~~~~~~~~~~~~~~~~~~~~~~~~~~~~~~~~~~~~~~~~~~~~~~~~~~

Are 'Bigoted Christian Rednecks' responsible for the Bush victory?

Are 'Bigoted Christian Rednecks' a threat to civil rights?

Are 'Bigoted Christian Rednecks' homophobes?

Are 'Bigoted Christian Rednecks' working toward a theocracy?

What is the threat, if any, of these mythical 'Bigoted Christian Rednecks'?

Comments?

tomder55 answered on 11/15/04:

"It is not true that the Democrats didn't show sympathy toward fundamentalists during the campaign. They did -- just to the wrong ones. Islamic fundamentalists received a great deal of understanding and tolerance from the Democrats. John Kerry made a point of showing sensitivity to the Islamic community and for it earned numerous endorsements from Muslim Imams. Perhaps herein lies a new strategy for the Democrats: What if they treated Christians as respectfully as they treated Yasser Arafat? What if they extended to Christianity the tolerant understanding they extend to Islam? Maybe from time to time the Democrats could refer to Christianity as a religion of peace. One would think a party that can canonize a de facto terrorist and jihadist like Arafat could tolerate a Southern preacher or two. Jerry Falwell has never blown up an airplane like Arafat, but Democrats wouldn't be caught dead in his company".

http://www.spectator.org/dsp_article.asp?art_id=7383

Itsdb rated this answer Excellent or Above Average Answer

Question/Answer
voiceguy2000 asked on 11/12/04 - Why Don't the Media Report The True Yasser Arafat?

Here is a more realistic assessment of Arafat. Why is it that world media aren't willing to face the truth about this depraved sociopath?

Andrew C. McCarthy
NRO Contributor

November 12, 2004, 8:27 a.m.

The Father of Modern Terrorism


The true legacy of Yasser Arafat.



For the last week of his life, the scuttlebutt about the Palestinian movement's centrifugal force concerned whether his impending demise was driven by AIDS, likely contracted, according to leaked foreign-intelligence reports, by his omnivorous, orgiastic sexual appetite. This as if, after three quarters of a century's worth of megalo-sadism, additional indicia of Yasser Arafat's throbbing depravity were somehow necessary. And so, evidently, they were. Thus is reflection on his life, a signal emblem of the late 20th century's triumph of terror and fraud over security and reason, as instructive about our times as it is about him.

A Thug's Life

About him, while there is much to say, there is little to glean. He was a thug. One of the most cunning of all time for sure, but quite simply a ruthless, thoroughly corrupt, will-to-power thug.

As is often the case in the modern information age, just about everything in his life is known and almost nothing in his proffered legend is true. The man airbrushed in Thursday-morning encomiums from Kofi Annan and Jacques Chirac (among others) as the courageous symbol of Palestinian nationalism was not really named Yasser Arafat, was not a native Palestinian, and tended to sit out warfare with Israel whenever conventional fighting was involved.

Although he occasionally claimed to have hailed from what are now the Palestinian territories, Muhammad Abdel Rahman Abdel Rauf al-Qudwa al-Husseini was actually born in Egypt in 1929, the fifth child of a well-to-do merchant. He was educated in Cairo, although, after his mother's death when he was four, he lived at least part of the time with an uncle in Jerusalem.

Jerusalem was then the heart of the territory known as Mandatory Palestine, which chafed under British rule as a result of a 1918 League of Nations mandate. The era, to put it kindly, was not the Crown's finest hour. Sowing seeds for recriminations that persist to this day, the Brits appeared during WWI to promise some or all of the territory alternatively to Arabs and to Jews, only to exacerbate matters by keeping Palestine themselves for three decades.

Arafat's formative years were thus spent in a milieu of sectarian violence, annealed in a hatred for Jews that, far from ever subsiding, propelled him. As an engineering student in Cairo during World War II, he was powerfully influenced by Haj Amin el-Husseini, the Islamic mufti of Jerusalem who was closely aligned with Hitler and schemed from Berlin to import the Fuhrer's genocidal program to Palestine. Indeed, as the New York Sun observed in an editorial last week, one of el-Husseini's biographers relates that Arafat was a blood relative of the mufti, who preferred him to another up-and-comer, George Habash (al-Hakim), among the fiercest of Israel's Nasserite enemies who eventually founded the Popular Front for the Liberation of Palestine (PFLP), a frequent Arafat ally.

Nevertheless, though he may have been a local gun-runner, the 19-year-old Arafat refrained from combat in 1948, when, upon Israel's declaration of independence, it was attacked by the Arab League (Egypt, Syria, Transjordan, Lebanon, and Iraq), which was defeated in the war still regarded by Palestinians and other Arabs as "al-Nabka" (the Catastrophe). Nor did he partake in the 1956 Suez War, although, as recounted last week by the Wall Street Journal's Bret Stephens, he later claimed to have done so.

Raising Terror

While Arafat's mantel as the "Father of Palestine" is dubious given that he is singularly responsible for the failure of a Palestinian nation to emerge, his credentials as the "Father of Modern Terrorism" are solid. In the late 1950's, he co-founded Fatah, the "Movement for the National Liberation of Palestine." His mtier, and thus Fatah's, was the sneak attack on soft Israeli targets, the better to maximize carnage and fear. The first efforts were ham-handed: failed attempts in 1965 to bomb the national water carrier and the railroad. But the organization soon hit its stride, successfully attacking villages and civilian infrastructure. By 1969, Arafat was the chairman of the Palestine Liberation Organization, the umbrella group he never ceased to dominate after merging Fatah into it a year earlier. The PLO had a single purpose: the destruction of Israel.

Actually, make that two purposes. The PLO was also a fabulously profitable criminal enterprise. Though Arafat purported to have made it big in the engineering business in Kuwait, British investigators, as Stephens reported, concluded after a searching probe that his wealth stemmed from sidelines his organization maintained in "extortion, payoffs, illegal arms-dealing, drug trafficking, money laundering and fraud" that yielded billions. Throughout his career, moreover, Arafat proved a master at culling funds whether from levies on strapped Palestinian workers or gushing subsidies from starry-eyed European and American governments. From these, he skimmed millions and stashed them throughout the world including in Israeli banks keeping his wife on a lavish $100,000-per-month allowance in Paris while his people starved, and, of course, blamed Israel for their troubles.

By the late 1960s, the PLO had set up shop in Jordan, wreaking havoc in the kingdom. Arafat and his affiliates soon became innovators in a tactic later refined by al Qaeda: the civilian airliner as terror weapon. On February 21, 1970, the PFLP by then also under the PLO arch bombed SwissAir Flight 330 enroute to Tel Aviv, murdering 47 passengers and crew. Eight months later, on September 6, they attempted a spectacular atrocity: a quadruple hijack, which now appears an eerie harbinger of the tectonic bin Laden operation on another September day 31 years later.

As recalled in the riveting account of "Black September" by hostage David Raab, all the hijacked flights were bound from Europe to the United States. One, a Pan-Am 747, was taken to Cairo, where it was blown up on the tarmac just after the passengers were allowed to exit. A second, targeting an El-Al aircraft, was foiled in flight by Israeli sky marshals. But a TWA 707 and a SwissAir DC-8, with a combined 310 passengers and crew, were hijacked to a Jordanian dessert. The terrorists segregated Israeli, American, Swiss, and West German passengers for captivity releasing the others and threatened to kill the hostages and blow up the planes unless jailed militants were released. Under international pressure, King Hussein resolved to reassert control. War broke out on September 13. By the time it ended two weeks later, the hostages had been released, but over 2,000 people had been killed as Arafat and his terrorist band were driven out of the country.

In the first of his many rises from the ashes, Arafat relocated to Lebanon. Staging from there, the PLO embarked, almost exactly a year to the day later, on another of the late 20th century's most infamous murder sprees. On September 5, in the midst of the Munich Summer Olympic Games of 1972, eight PLO operatives (a wing of Arafat's Fatah group known as the "Black September" brigade) carried out a plan that enabled five of them to steal into the Olympic village, quickly murder two members of the Israeli team (the wrestling coach and a weightlifter), and take nine other Israeli athletes hostage. The terrorists demanded the release of 200 Arab prisoners and safe passage back to the Middle East. German authorities lured them, with their captives, to the airport, but a rescue attempt was badly botched. In the resulting battle, the Palestinians killed all nine Israeli athletes by grenade and gunfire, as well as murdering a German policeman. Five of the terrorists were killed in the struggle, but German authorities managed to capture the remaining three. True to form, Arafat's organization responded the following month by hijacking a Lufthansa jet and taking the passengers hostage. The Germans capitulated, releasing the killers.

Arafat, meanwhile, also kept Israel's support network, the U.S., in his sights. On March 1, 1973, another eight-member Black September cell raided the Saudi embassy in Khartoum, Sudan, taking as captives two American government officials, Ambassador Cleo Noel and the Charge d'Affaires George Curtis Moore, as well as a Belgian diplomat named Guy Eid. The terrorists demanded the release of Sirhan Sirhan in California (jailed for the 1968 slaying of Robert F. Kennedy), of Palestinians imprisoned in Jordan (including Black September's own Abu Daoud, who later claimed to be the master-planner of the Munich Olympics massacre), and of Palestinian women jailed in Israel. When they were rebuffed, the terrorists murdered Noel, Moore, and Eid, and then anxiously surrendered to the Sudanese authorities.

These murders, theoretically an act of war against the U.S., were never "solved" in the sense of convicting the man ultimately responsible. The FBI was reported to have reopened an investigation of them earlier this year, and at least one State Department spokesman has strangely claimed the link between Arafat and Black September was never conclusively established even as he acknowledged Black September's membership in Arafat's own Fatah faction.

Nonetheless, a number of Israeli and American intelligence officials have long maintained that Arafat personally ordered the killings by issuing a radio message, to wit: "Why are you waiting? The people's blood in the Cold River cries for vengeance" Cold River reportedly being a predetermined code directing the executions. Furthermore, in the kangaroo court that passed for a Sudanese prosecution, one of the terrorists, Salim Rizak, testified: "We carried out this operation on the orders of the Palestine Liberation Organization" while another witness, the Sudanese official who conducted interrogations, reported that the killers had taken their cues from radio messages emanating from Fatah headquarters in Beirut. Thus abound dark suspicions, not to mention an explicit allegation by former NSA official James J. Welsh, that Arafat's complicity was shunted aside for what was perversely perceived as the greater good of diplomatically cultivating him. Meanwhile, of the eight surrendering Black September terrorists, two were released immediately by the Sudanese due to purportedly insufficient evidence, while the remaining six were convicted, sentenced to life-imprisonment, and...released the very next day to the open arms of the PLO.

From his Lebanese perch, Arafat's rampage of Israel continued apace. On April 11, 1974, the PLO slaughtered eighteen residents of Kiryat Shmona in their apartment building. A month later, on May 15, Palestinian terrorists attacked a school in Ma'alot, murdering 26 Israelis, including several children. Then, in June, the PLO through the "Palestinian National Council" endorsed what it called a "phased plan" to obliterate Israel.

Weak-Kneed Appeasement

Seven years earlier, of course, Egypt, joined by Syria and Jordan, had foolishly launched yet another war of aggression against Israel. They were routed in the Six Day War of June 1967, at the end of which Israel's territorial holdings had drastically swelled to include the West Bank and East Jerusalem (taken from Jordan), the Suez and Gaza (from Egypt), and the Golan Heights (from Syria). It was understood that this expansion would not be permanent in accordance with U.N. Security Council Resolution 242, Israel agreed eventually to withdraw from some undetermined portion of these territories in exchange for peace treaties that settled borders and acknowledged Israel's right to exist. In Arafat's 1974 phased plan, however, the PLO reaffirmed its rejection of Resolution 242 and committed itself to establish, in any ceded territory, a Palestinian state that would work toward Israel's destruction.

Adumbrating the global strategy for dealing with terror that would reign supreme through the quarter century leading up to the 9/11 attacks, the world reacted to Arafat's contemptuous belligerence with weak-kneed appeasement. The PLO was rewarded with observer status in the U.N., and on November 13, 1974, a triumphant and utterly unrepentant Arafat, holster strapped to his hip, addressed the General Assembly in New York City. By 1980, the European Economic Community recognized him as the "sole legitimate representative" of the Palestinian people.

Not that there weren't setbacks. In 1979, Israel had struck a historic peace deal with Egypt in which it agreed to a phased pull-out from the Sinai (completed in 1982) and acknowledged that there should eventually be some form of autonomy for the Palestinian enclaves of the West Bank and Gaza. With its southern flank calmed, Israel wearied of continuing missile attacks and other sorties launched against its northern communities from the PLO's Lebanese stronghold. Israel invaded in 1982, inducing Arafat to flee to Tunis.

From Killing Klinghoffer to "Nobel" Star

The PLO's bloodlust did not abate. In 1985, a cell identifying itself as the Palestine Liberation Front, led by Mohammed Abu al-Abbas, hijacked the Italian cruise ship, Achille Lauro. As his horrified wife looked on, the terrorists viciously shot a 69-year-old, wheelchair-bound Jew named Leon Klinghoffer, then tossed him overboard to die in the sea. Despite indications that the PLF was acting on instructions from PLO headquarters in Tunis, a State Department spokesman incredibly contended as late as 2002 that the PLF had been a renegade group broken off from the PFLP, and that Arafat was probably blameless in the Achille Lauro operation. But, aside from the fact that the PLO's website (for its U.N. mission) listed the PLF as one of its constituents, Abbas had actually been a member of Arafat's own PLO Executive Committee. More to the point, when Abbas died last year in Iraq (where he had been harbored by Arafat's staunch ally, Saddam Hussein), Arafat issued an official statement lavishly praising him as a "martyr leader" and "a distinguished fighter and a national leader who devoted his life to serve his own people and his homeland."

Not long after Achille Lauro, Arafat began in 1987 to blaze the path that, by the mid-1990's, sickeningly transformed him into a regular White House guest and a Nobel Laureate. As was his Orwellian wont, he started on the road to faux respectability with a terrorist barrage that became known as the First Intifada. (With Arafat, it had to be the First Intifada because there would, of course, be a Second.)

The siege was ignited by two unconnected events in the powder keg of Gaza: the December 6 murder of an Israeli, followed quickly by the tragic December 10 death of four Palestinians in a car accident which was falsely, but unrelentingly, hyped as a revenge killing. Skirmishes quickly broke out in Gaza, and careened through the West Bank and East Jerusalem. The violence, a roller-coaster of lulls and explosions, lasted over six years. In the first four years that is, the period before the ebb that marked the onset of the 1991 Gulf War Israeli defense forces responded to more than 3,600 Molotov cocktail attacks, 100 hand grenade attacks, and 600 assaults with guns or explosives, all of which killed 27 and wounded over 3000. Although the PLO was rivaled in the operation by militant Islamic groups such as Hamas and Islamic Jihad, Arafat's group dominated the so-called "Unified Leadership of the Intifada," using leaflets to direct the days and targets of attacks.

Israelis were not alone among the terror casualties. Arafat unleashed PLO death squads to kill numerous Arabs who were deemed to be collaborating with the enemy. In 1990, the Arabic publication Al-Mussawar reported Arafat's defense of the tactic: "We have studied the files of those who were executed, and found that only two of the 118 who were executed were innocent." As for those putative innocents, Arafat sloughed them off as "martyrs of the Palestinian revolution."

Even as the violence hummed, Arafat assumed his statesman's face for the West, to great effect. As the body count mounted in 1988, the U.N. granted the PLO's observer mission the right to participate, though not vote, in General Assembly sessions. In addition, the administration of George H. W. Bush held open the possibility of direct dialogue if Arafat would renounce terrorism and agree to be bound by Resolution 242. This he purported to do on December 16, 1988, claiming to acknowledge "the right of all parties concerned in the Middle East conflict to exist in peace and security...including the state of Palestine and Israel and other neighbors according to the Resolutions 242 and 338" and asserting: "As for terrorism...I repeat for the record that we totally and absolutely renounce all forms of terrorism, including individual, group and state terrorism." Like the Europeans, the U.S. officially recognized Arafat as the legitimate leader of the Palestinians.

The bankruptcy of these claims was revealed as the Intifada ensued and Arafat blundered by publicly aligning with Saddam both after the invasion of Kuwait and throughout Iraq's scud missile attacks on Israel. But just as it seemed he might finally fade away, the strongman caught a lifeline when Gulf War victory failed to carry the first President Bush to re-election. Bush's successor, President Bill Clinton, saw in the intractable Israeli/Palestinian conflict the chance for an enduring legacy, and saw in Arafat a viable "peace partner."

With Clinton as determined midwife, Arafat and the government of Prime Minister Yitzhak Rabin signed the ballyhooed Oslo Accords of 1993. The Palestinian Authority was created, Arafat was appointed its chief executive, and a plan for eventual self-government by Palestinians in the West Bank and Gaza was set in motion. But euphoria over this seeming breakthrough blurred appreciation of both Arafat's innate mendacity and Oslo's patent failure to resolve key contentious issues, including final borders, the status of East Jerusalem, and the rights of Israeli settlers and Palestinian refugees under the delusion that Arafat would work in good faith toward a peaceful, comprehensive settlement with Israel over a five-year period.

The mega-murderer was suddenly statesman, star, and, in 1994, winner of the Nobel Peace Prize a once-coveted honor now, by his attainment of it, reduced to a joke best listed among his countless victims. Thanks to this peace partner, it soon became clear that Oslo was a charade, a case of a credulous American president choosing his honey over his lying eyes.

The Palestinian Authority reneged on its promises of democratic reform and establishment of the rule of law holding elections exactly once and never again after Arafat was overwhelmingly elected. Arafat also failed to honor, despite incessant pleading by Clinton administration figures, a commitment that the Palestinian National Charter would be amended to remove clauses calling for the destruction of Israel. The PA made a show of appearing to comply, disingenuously noting the provisions purportedly slated for nullification and calling for a new draft of the Charter to be produced. No revised Charter, however, was ever forthcoming. Meanwhile, what education system existed in the territories, much like Arafat's public statements in Arabic (always far more menacing than the English he spoke to the Western world), continued to instill hatred for Jews and calls for the demise of their state. Naturally, the terrorist activity also proceeded, with the PA ineffectual in halting it when not encouraging it outright.

There should have been surprise in none of this. As Stephens reports, in 1996, Arafat brayed to an Arab audience in Stockholm, "We plan to eliminate the State of Israel and establish a purely Palestinian state. We will make life unbearable for Jews by psychological warfare and population explosion.... We Palestinians will take over everything, including all of Jerusalem." Asked about his plans on Egyptian television in 1998, Arafat explained that strategic pause was a venerable Islamic strategy, referring specifically to the "Khudaibiya agreement" in which the Prophet Mohammed made a ten-year treaty with the Arabian tribe of Koreish, but broke it after two years during which his forces used the security of the pact to marshal their strength and then conquered the Koreish tribe.

Such machinations were certainly no secret to the governments and media in the U.S., Europe and Israel itself. They knew precisely who Yasser Arafat was. But politically and culturally, hopeful hearts and good intentions were for them more essential than results on the ground the "process" always took precedence over the "peace." Thus, in the Wye River Accords of 1998, the Clinton administration and Israel, now led by Prime Minister Benjamin Netanyahu, took the terrorist at his word when he promised, yet again, to crack down on terror, this time in exchange for a pull back of Israeli forces (which had entered the territories in response to terror attacks), the ceding of additional territory to PA control, and even the release hundreds of Palestinian prisoners many of whom had been incarcerated for terrorism offenses.

14 9/11s

The violence never stopped. Yet, with his presidency winding down in 2000 and desperate for an accomplishment that might balance a record besmirched by scandal, President Clinton boldly sought a final time to forge a comprehensive settlement. He brought Arafat and yet another new Israeli Prime Minister, Ehud Barak, to Camp David. Under intense U.S. pressure, Israel offered the creation of a Palestinian state over 90 percent of the West Bank and all of Gaza, with its capital to be in East Jerusalem. In a move comprehensible only if one accepts that Arafat was incorrigibly devoted to Israel's extermination in which case, it was entirely comprehensible Arafat rejected this stunning offer, with poison-pill insistence that millions of Palestinians be accorded a right of return to Israel.

The breakdown of negotiations resulted, like night followed day with Arafat, in a new round of terror: the Second Intifada, which continues to this day. This program has been pursued mostly by suicide bombings often including explosives strapped to children encouraged by the culture of shahada, or martyrdom, which thrived under Arafat's corrupt and dysfunctional leadership. In the main, attacks have willfully targeted civilians in busses, restaurants, shopping centers, synagogues, hotels and other public centers. Since 2000, approximately 900 Israelis, three quarters of whom were civilians, have been murdered. To extrapolate to American proportions, for a country the size of Israel this is the rough equivalent of over 40,000 dead or, as the Hudson Institute's Anne Bayefsky has calculated, about 14 9/11s.

Arafat's world, like everyone else's, radically changed on September 11, 2001. The Bush Doctrine, announcing a commitment to eradicate terrorists and terror supporting governments, did not immediately spell the end for the Palestinian strongman. He was, however, gradually marginalized and reduced to pariah status but for the markedly less frequent, and ineffectual, paeans from Europe, the Islamic world and the U.N.

The magic began to fail even his most trusted old tricks. For example, on December 16, 2001, with American forces suppressing terrorists in Afghanistan, an ostensibly chastened Arafat appeared on PA-controlled Palestinian television to warn Hamas and Islamic Jihad against "all military activities" against Israel, and to purportedly "renew" his "call to completely halt any activities, especially suicide attacks, which we have condemned and always condemned." This time, the ploy fell flat undercut, no doubt, after the Nobel laureate characteristically followed it up only two days later with a speech at a Ramallah rally the kind of red meat always conveniently ignored in the halcyon pre-9/11 days. "With God's help," he boasted:
next time we will meet in Jerusalem, because we are fighting to bring victory to our prophets, every baby, every kid, every man, every woman and every old person and all the young people, we will all sacrifice ourselves for our holy places and we will strengthen our hold of them and we are willing to give 70 of our martyrs for every one of theirs in this campaign, because this is our holy land. We will continue to fight for this blessed land and I call on you to stand strong.
The jig was up. Arafat's celebrity might be a product of the "international community" but his relevance was strictly made-in-the-USA, and America was no longer buying. The administration of President George W. Bush let it be known that Arafat would no longer be dealt with. When the president eventually proposed his "roadmap" to resume negotiations toward an eventual Palestinian state, he snubbed Arafat and made unconditional cessation of all Palestinian terrorism a nonnegotiable prerequisite. Critically, the administration also eased the restraints that had for decades compelled Israel to accord its sworn enemy so wide a berth.

Now under Prime Minister Ariel Sharon, Israel responded forcefully to the terror onslaught, including through high-profile "targeted assassinations" of Hamas leaders. Its forces tightened the noose around Arafat. Unable to leave his squalid Ramallah compound with any assurance that he'd either survive or be permitted to return, the "president" of what was more a racket than a government and decidedly not a nation remained holed up there for over two years until his evacuation to Paris, in extremis, in late October. There he died on Wednesday, one of history's most repulsive conmen and killers.

"The power of bad men," Burke famously observed, "is no indifferent thing." The power of this evil man informed an age the age of terrorism. The Israelis and Palestinians may never coexist peacefully, but as long as Yasser Arafat lived they didn't even have a chance.

Andrew C. McCarthy, who led the 1995 terrorism prosecution against Sheik Omar Abdel Rahman and eleven others, is a senior fellow at the Foundation for the Defense of Democracies.

Link to article.


tomder55 answered on 11/13/04:

The State Dept. to this day denies their knowledge of the link between the murderers of Noel and Arafat.Arafat publicly denied any complicity ,but during a private dinner with Romanian dictator Nicolae Ceausescu in May 1973 he bragged about it.
American Administrations since the Nixon Adm. were reluctant to pursue murder charges against Arafat due to political considerations related to the Cold War and the Middle East.

James J. Welsh was an NSA employee whos job was to analyze communications of the PLO for potential threats to the United States . He received a call from Mike Hargreaves working out of Cyprus alerting him to a highly unusual radio message intercepted between Beirut and Khartoum. He sent a high priority warning to the U.S. embassy in Khartoum .Intercepts from Cyprus later revealed that Arafat's deputy ,Abu Iyad had personally ordered the executions .Also Arafat personally got on the radio to praise the murderers. Welsh believes that recordings of those intercepts still exist and that Kissinger himself was responsible for the cover up.

In 1986, 47 U.S. senators signed a letter to attorney general Edwin Meese demanding that the Justice Department indict Arafat for murder.(including Al Gore who has praised Arafat as a champion of peace).


February 12, 1986

The Honorable Edwin Meese III
Department of Justice
Washington, D.C. 20530

Dear Mr. Attorney General:

We understand that the Department of Justice has received information linking PLO leader Yasser Ararat to the brutal 1973 slaying of Ambassador Cleo Noel and Charge d'Affaires C.Curtis Moore in Khartoum, Sudan.

The material is reported to include various State Department cables that may confirm Ararat's role in the murders. It is also reported to include an assertion that the U.S. government has a tape recording of an intercepted message in which Arafat allegedly ordered the assassination of Ambassador Noel and Charge d'Affaires Moore, who were taken hostage when Palestinian terrorists seized the Saudi Arabian Embassy in Khartoum on March 2, 1973.

As you know, press reports indicate that the eight terrorists involved in the incident identified themselves as members of Black September. They demanded the release from prison of Sirhan Sirhan, the Baader-Meinhof gang, and a group of Fatah members being held In Jordan.

Press reports indicate that when their demands were not met, the terrorists selected the three Westerners among the hostages--U.S. Ambassador Cleo Noel, Charge d'Affaires C.Curtis Moore, and Belgian diplomat Guy Eid--and machine-gunned them after first allowing them to write farewell notes to their families and then beating them. A day later, the terrorists surrendered to Sudanese authorities after a lengthy round of transoceanic communications involving, among others, Arafat and the Vice President of Sudan.

Press reports indicate that Sudanese President Gaafar Mohammed Nimeiri went public at once with evidence showing that the operation had been run out of the Khartoum office of Fatah. One month after the slayings, the Washington Post reported that according to Western intelligence sources, Arafat was in the Black September radio command center in Beirut when the message to execute three Western diplomats was sent out. The Post also reported that Arafat's voice was monitored and recorded. Although according to the Post's sources, it was unclear if Arafat himself or his deputy gave the order to carry out the executions, Arafat reportedly was present in the Operations center when the message was sent and personally congratulated the guerrillas after the execution.

These allegations, if substantiated, leave little doubt that a warrant for Arafat's arrest should be issued, and a criminal indictment filed against him. To allow other factors to enter into this decision is to make a mockery of our laws and our stated commitment to eradicate terrorism. As President Reagan told an American Bar Association convention this July, "we will seek to indict, apprehend, and prosecute" terrorists.

We understand that this matter is presently under review at the Justice Department. We urge the Justice Department to assign the highest priority to completing this review, and to issue an indictment of Yasser Arafat if the evidence so warrants. We would also ask that you keep us advised of the progress of your investigation.


It appears that Welsh has been obtaining proof of his allegations through the 'Freedom of Information 'Act.The truth is finally being revealed .Nothing can be done about Arafat now,but the State Dept.'s complicity and their sympathetic attitudes toward Palestinian terrorists is something the public should be aware of ,and a clean sweeping of the dept. should be initiated.







voiceguy2000 rated this answer Excellent or Above Average Answer

Question/Answer
sapphire630 asked on 11/10/04 - Post Election Selection Sydrome or


Bush Derangement Syndrome: the acute onset of paranoia in otherwise normal people in reaction to the policies, the presidency -- nay -- the very existence of George W. Bush.
Mental Health pyshicans have come up with these post-tramatic titles and are now trying to make a living on the 'anybody but Bush' extreme-leftists who
want do everything and anything from to flee the country to kill theirself.

Maybe it is one more thing to blame on Bush?

tomder55 answered on 11/10/04:

When I'm stuck a day
That's gray,
And lonely,
I just stick out my chin
And Grin,
And Say,
Oh!

The sun'll come out
Tomorrow
So ya gotta hang on
'Til tomorrow
Come what may
Tomorrow! Tomorrow!
I love ya Tomorrow!
You're always
A day
A way!

(Annie)

sapphire630 rated this answer Excellent or Above Average Answer

Question/Answer
Choux asked on 11/09/04 - Day 600-War in Iraq-15,567 DEAD

Today is November 9, 2004, the assault on Fallujah is a couple of days old. Here are some statistics collected since March 19, 2003, the day Bush declared War on Iraq::::

Killed USA 1,137-----UK 74------OTHER 72

WOUNDED USA 4,438

WOUNDED RTD USA 3,849 (Returned To Duty)

IRAQI CITIZENS KILLED MINIMUM 14,284
MAX 16,419

CONSERVATIVE BODY COUNT EFF 11-04

----15,567 men and women-----

I got these statistics from searching the web with the following and viewing the first or first three sites. ::Iraqi citizens killed::::and casualities in Iraq war.

tomder55 answered on 11/10/04:

you forgot some :

Russian children dead at Beslan - 200+

American's dead 9-11-01 - 3000 +

Bali Bombing dead -202

Tunisia bombing dead -19

Madrid bombing -190

Australian embassy bombing - 11

people beheaded by terrorist - 15

U.N. Headquarter bombing in Iraq - 17+

Countless Iraqi civilians mudered by terrorists using car bombs .

The battle of Fallujah has been joined .It was a strategic defeat in april to attack and then withdraw at the cusp of victory .I was critical of it then because we left the terrorists with that 'Mogodishu 'impression that we could be driven out and our political will was not what it should be .Since then it was a mistake to allow safe areas for terrorists where their power could be consolidated and their cells expanded .Our timidity throughout our election cycle was a step back in progress,as was some aspects of the Bremer Viceroyship but those days are over . By the weekend the terrorists will be pinned down against the banks of the Euphrates river. The marines have perfected their urban combat capability ;and 21st century weapon systems give them a tremendous advantage .Individual terrorists may escape their fury ,but never again will terrorists be able to concentrate in Iraq ,and never again will a part of the country be considered a sanctuary to those who would undermine the Iraqi gvt. Zarqwai has utterly failed in his attempt to foment a civil war or a popular revolt jut like al-Sadr before him. Elections will procede on schedule ,and the long process of creating a new Iraqi society will begin.

ETWolverine rated this answer Excellent or Above Average Answer
Choux rated this answer Excellent or Above Average Answer

Question/Answer
bobill asked on 11/09/04 - Can You Believe This?

"... securing the safety of Americans from crime and terror has been achieved" - John Ashcroft in his letter of resignation today.

I'm not making this up, folks, He really said it. What could be more Orwellian?

Maybe Bennet's statement some years ago that the "drug war is over" - this was before he wrote his sanctimonious book on morality, and before he was found to be a degenerate gambler.

Or how about the darling of the right wing - Rush Limbaugh? And his propensity for drugs - all the while declaiming the evils of drug use?

No, you couldn't make this stuff up. Nobody would believe it.

Hey, let's bomb Fiji - or Iceland. They could use some American "values"!

tomder55 answered on 11/10/04:

Ashcroft was one of the finest AG's America ever had. His is a record of achievement. Crime is down nationwide;and far more important ;there has not been another terrorist attack in the U.S.since the days after 9-11.(the D.C. sniper and the anthrax attacks occured before anti-terrorism measures were enacted).

He has worked tirelessly to bridge the gap that former Justice Dept. hacks like Jamie Gorelic left us ;where a coordinated effort with other intelligence agencies was a prohibition(the wall ).The country is indeed safer because of his service.

Opposition has used all kinds of ridicule against him because he is a modest man ,but even left wingers should applaud his tenure.If they are opposed to things like the Patriot Act they should at least acknowledge that he has not excessively applied it 's provisions. The ACLU has been begging for information on alleged abuse of the act and has found none. Even the controversial library provision has NEVER been used as a tool of the Justice Dept. . All the anti-gun laws that have been passed have been vigorously enforced under his tenureship. Fed. gun charges are up 75 % in 4 years. Gun use in crimes has dropped throughout his term.

My one beef with him ,and that goes for the whole Administration is it's lackadaisical enforcement of border security .Actually there are mixed results here . The U.S. and Canadians have had success,but there is still too much illegal alien border crossings and not enough interdiction of drugs .

He should be proud of his achievements and the nation should be thankful for his service.

Speculation is that White House counsel Alberto Gonzalez will replace him . Let's see the Democrats in the Congress block another Latino Bush appointment. My advice :it would be a bad idea to do so.

bobill rated this answer Excellent or Above Average Answer

Question/Answer
Choux asked on 11/08/04 - Responsibility

Maybe now the Conservative Right Wing supporters of President Bush will be willing to accept responsibility for the actions of his administration since 2000 and going forward to 2008.

Will we be seeing bellyaching and namecalling in the form of blaming past presidents for Bush's problems, but congratulating Bush for Bush's success'??? Every President in the past has taken responsibility for the actions on his watch.

WE are going to have a lot of posts about what the Republican government and how it is governing domestically and in foreign policy. Let's see if the *haters* can participate on the board without hating and spinning and lying.

tomder55 answered on 11/09/04:

Post election analysis is normal and typical,and a necessary part of politics . I think the initial reactions are overstated ,and certainly the emotions in the analysis has led to many false conclusions and alot of stuff I chose not to respond to.

N.Y. which voted for Kerry was one of the States with the lowest turnout of registerd voters (the weather was great here too)Suppose the Democrats had done a better job than the GOP at registering and getting their supporters out to the polls ? Bush may have lost in Ohio;which probably would have swung the election to Kerry . The press would be running columns about the liberal shift in public opinion, the defeat of the fundamentalists, and the importance of antiwar sentiments.

(btw . in spite of conventional wisdom the gay marriage bill in Ohio did not swing the State . A bigger factor may have been the 'Guardian' of England trying to influence American politics in Ohio. They set up a letter writing campaign in Clark Co. Ohio that backfired on them badly. 11000 readers wrote to the residents telling them how bad Bush was . The folks reaction? ;well in 2000 Gore won the county . This time Bush won making it the only county in Ohio that switched from Dem. to Republican.Clark County swung Ohio to Bush)

Moral issues have been and always will be an integral part of the American debate .It was when slavery was the issue ;it was during the civil rights days ;it was during the temperance days ;and it is today . But I do not think that they were the determining factor . Stuff like abortion (late term or otherwise),and gay marriage were barely mentioned . Bush improved his voting support among people who attend church frequently and who describe themselves as Catholics, Protestants and Jews, but Sen. Kerry won nearly half of all Catholic votes and over three-fourths of all Jewish ones.


The undeniable overwhelming consideration was all the aspects of foreign policy/terrorism/Iraq.It was the most debated issue and most of the stump speeches I listened to had that topic as it's theme. The majority of those who voted gave an endorsement to Bush's handling of it .


I do not think that Bush can dismiss the views of 48 million Americans easily .That being said ;the first thing that was mentioned immediately after the election was the need for Bush to reach out across the aisle without a simular call from the opposition to do so. Agree or disagree with him ;he layed his cards out on the table . We know how he will conduct foreign policy in the next 4 years .As a Supporter of his policy I intend to support him so long as he is true to his policy.

Choux rated this answer Excellent or Above Average Answer

Question/Answer
Choux asked on 11/06/04 - 26 Saudi Scholars and Preachers

issued a statement over the internet today. Due to extraordinary circumstances, they call for unity and are urging Iraqi citizens to join the militants and Insurgents in waging a holy war agains the US led co-alition forces.

I ask those in favor of waging war in order to subdue the Middle East crazies....can the war end in any way that Civil War in Iraq among the Shi'a Suni and Kurds after the US troups are driven out?

tomder55 answered on 11/08/04:

Target them like they are enemy combatants .

Could be that in a week or two they are going to lose a large part of their support in a place called Fallujah ? There must be thousands of clerics in Saudia Arabia so a handful is not necessarily representative . We need to find out how many of them hold this view and then demand that the Saudis deal with it.

Choux rated this answer Excellent or Above Average Answer

Question/Answer
purplewings asked on 11/07/04 - What do you think about France starting a mini war with the Ivory Coast?

It seems the tiny military, consisting of 2 planes and 3 heliocopters accidentally dropped a bomb.

The Associated Press
Sunday, November 7, 2004; 8:05 AM


ABIDJAN, Ivory Coast - Ivory Coast forces will immediately pull back from front lines, the presidential spokesman said Sunday, after a day of clashes with French peacekeepers that left the West African nation's air force in shreds.

"We are going immediately to pull back to the confidence zone, spokesman Desire Tagro said, referring to a buffer zone manned by French and U.N. peacekeepers between rebel north and loyalist south. Ivory Coast warplanes crossed the buffer zone Saturday, bombing a French peacekeeping position and killing nine French and an American.

Ivory Coast would ask the Security Council for action against France, Tagro said, saying, "We are faced with aggression by one country against another country. We are going to inform the entire world ... that France has come to attack us."

tomder55 answered on 11/08/04:

The rebels are Muslims . Guess which side the French have sided with even though they are there to be 'peace keepers' ?

ETWolverine rated this answer Excellent or Above Average Answer
purplewings rated this answer Excellent or Above Average Answer

Question/Answer
Itsdb asked on 11/05/04 - Michael Moore finally speaks

11/5/04

Dear Friends,

Ok, it sucks. Really sucks. But before you go and cash it all in, let's, in the words of Monty Python, 'always look on the bright side of life!' There IS some good news from Tuesday's election.

Here are 17 reasons not to slit your wrists:

1. It is against the law for George W. Bush to run for president again.

2. Bush's victory was the NARROWEST win for a sitting president since Woodrow Wilson in 1916.

3. The only age group in which the majority voted for Kerry was young adults (Kerry: 54%, Bush: 44%), proving once again that your parents are always wrong and you should never listen to them.

4. In spite of Bush's win, the majority of Americans still think the country is headed in the wrong direction (56%), think the war wasn't worth fighting (51%), and don't approve of the job George W. Bush is doing (52%). (Note to foreigners: Don't try to figure this one out. It's an American thing, like Pop Tarts.)

5. The Republicans will not have a filibuster-proof 60-seat majority in the Senate. If the Democrats do their job, Bush won't be able to pack the Supreme Court with right-wing ideologues. Did I say "if the Democrats do their job?" Um, maybe better to scratch this one.

6. Michigan voted for Kerry! So did the entire Northeast, the birthplace of our democracy. So did 6 of the 8 Great Lakes States. And the whole West Coast! Plus Hawaii. Ok, that's a start. We've got most of the fresh water, all of Broadway, and Mt. St. Helens. We can dehydrate them or bury them in lava. And no more show tunes!

7. Once again we are reminded that the buckeye is a nut, and not just any old nut -- a poisonous nut. A great nation was felled by a poisonous nut. May Ohio State pay dearly this Saturday when it faces Michigan.

8. 88% of Bush's support came from white voters. In 50 years, America will no longer have a white majority. Hey, 50 years isn't such a long time! If you're ten years old and reading this, your golden years will be truly golden and you will be well cared for in your old age.

9. Gays, thanks to the ballot measures passed on Tuesday, cannot get married in 11 new states. Thank God. Just think of all those wedding gifts we won't have to buy now.

10. Five more African Americans were elected as members of Congress, including the return of Cynthia McKinney of Georgia. It's always good to have more blacks in there fighting for us and doing the job our candidates can't.

11. The CEO of Coors was defeated for Senate in Colorado. Drink up!

12. Admit it: We like the Bush twins and we don't want them to go away.

13. At the state legislative level, Democrats picked up a net of at least 3 chambers in Tuesday's elections. Of the 98 partisan-controlled state legislative chambers (house/assembly and senate), Democrats went into the 2004 elections in control of 44 chambers, Republicans controlled 53 chambers, and 1 chamber was tied. After Tuesday, Democrats now control 47 chambers, Republicans control 49 chambers, 1 chamber is tied and 1 chamber (Montana House) is still undecided.

14. Bush is now a lame duck president. He will have no greater moment than the one he's having this week. It's all downhill for him from here on out -- and, more significantly, he's just not going to want to do all the hard work that will be expected of him. It'll be like everyone's last month in 12th grade -- you've already made it, so it's party time! Perhaps he'll treat the next four years like a permanent Friday, spending even more time at the ranch or in Kennebunkport. And why shouldn't he? He's already proved his point, avenged his father and kicked our ass.

15. Should Bush decide to show up to work and take this country down a very dark road, it is also just as likely that either of the following two scenarios will happen: a) Now that he doesn't ever need to pander to the Christian conservatives again to get elected, someone may whisper in his ear that he should spend these last four years building "a legacy" so that history will render a kinder verdict on him and thus he will not push for too aggressive a right-wing agenda; or b) He will become so cocky and arrogant -- and thus, reckless -- that he will commit a blunder of such major proportions that even his own party will have to remove him from office.

16. There are nearly 300 million Americans -- 200 million of them of voting age. We only lost by three and a half million! That's not a landslide -- it means we're almost there. Imagine losing by 20 million. If you had 58 yards to go before you reached the goal line and then you barreled down 55 of those yards, would you stop on the three yard line, pick up the ball and go home crying -- especially when you get to start the next down on the three yard line? Of course not! Buck up! Have hope! More sports analogies are coming!!!

17. Finally and most importantly, over 55 million Americans voted for the candidate dubbed "The #1 Liberal in the Senate." That's more than the total number of voters who voted for either Reagan, Bush I, Clinton or Gore. Again, more people voted for Kerry than Reagan. If the media are looking for a trend it should be this -- that so many Americans were, for the first time since Kennedy, willing to vote for an out-and-out liberal. The country has always been filled with evangelicals -- that is not news. What IS news is that so many people have shifted toward a Massachusetts liberal. In fact, that's BIG news. Which means, don't expect the mainstream media, the ones who brought you the Iraq War, to ever report the real truth about November 2, 2004. In fact, it's better that they don't. We'll need the element of surprise in 2008.

Feeling better? I hope so. As my friend Mort wrote me yesterday, "My Romanian grandfather used to say to me, 'Remember, Morton, this is such a wonderful country -- it doesn't even need a president!'"

But it needs us. Rest up, I'll write you again tomorrow.

Yours,

Michael Moore
MMFlint@aol.com

tomder55 answered on 11/07/04:

I enjoy reading Michael Moore ,but his films seem amateurish to me ;Certainly not the quality that should get cerious consideration for awards . In the end he is catering to a marginal part of society ,and the Dems. would be wise to marginalize his influence .His track record is he made a mockery of the Nader campaign in 2000;he undermined first Gen. Clark's primary run ;and did nothing positive for Kerry.

His new 'investigative'book will be on the Pharmaceutical industry .Remind me to purchase stock in Merck,Sandoz,Novartis;and J&J when the movie comes out.

Itsdb rated this answer Excellent or Above Average Answer

Question/Answer
excon asked on 11/05/04 - Libertarian Country's


Hello Experts:

Anybody know of any country that is operated on a libertarian philosophy?

excon

tomder55 answered on 11/06/04:

trouble is that libertarianism is such a personal philosophy that it would almost be impossible to form a government of consent.Libertarians regard society as just a mass of individuals in the same place under the same government, ignoring common traditions, culture, religion, etc.That is not how the world works. I have read many of Ayn Rands novels and know how libertarians view society .It appears to me to be as Social Darwinistic a plan as I have seen and as much a utopian vision as Communism.

excon rated this answer Excellent or Above Average Answer

Question/Answer
Choux asked on 11/04/04 - Howard Dean

It was just on TV that Howard Dean is in the running for the Chairman of the Democratic National Committee replacing McAuliffe who is out.

tomder55 answered on 11/06/04:

I heard this last night ,and then I said why Howard Dean for Chairman DNC when right in Chicago there is a Dem. who is sure to get out the vote by any means necessary;Richard Daley:

Mayor Daley said Thursday his beloved Democratic Party took a beating in Tuesday's election because they were "outfoxed": Republicans remembered that all politics is local, while Democrats became the "party of Washington" insiders and big money.

"We always thought the Republican Party was Washington, D.C. The Democrats are Washington, D.C., politicians. They don't reach out to a mayor, a governor, or the state chairman. There's no local anymore," Daley said.

"If you watch the Republican Party, they're to the people. . . . They're more grass-roots than Democrats. We think we are. The Republicans outfoxed the Democrats. They became the party of precincts, a county, a city. Their strategy was to go to the people and not to the money people. . . . We're supposed to be the party of the people. We're the party of the money. . . . We've become the party of the insider."


The Daley family;notorious in their ability to get out the vote ;living or dead;(no taxation without resperation)has had a history of delivering Ill.;with the exception of Cook County an otherwise red State ;to the Dems.(as noted by this CNN map if you open Ill.only Cook County is solid blue).
Why not give him a chance to work his magic in a national scale ?

Hillary may indeed be the candidate in 08,but first she will do a reality T.V. style extreme make over. First she will emphasis that in her heart she in not an elitist N.Y.er but is instead from the heartland ;Chicago ,Arkansas;God's country. She will be photo oped at every opportunity going to religious services.She will try to be seen whenever possible in the presence of Bill playing the part of the loving wife ;trying to convince the country that her marriage resembles George and Laura and not John and Theresa. The Dem. theme that she will gladly now adopt is one of how the Dem. platform (that will have another extreme makeover ,plank by plank is the one that really reflects traditional values(well at least those of the Roosevelt Dems.)
She will show that she can out hawk the hawks of both parties and try to convince us that the security of the nation will be safe in her hands.It should be very interesting to watch.

Her problem? Well just like John Edwards she will be a one term Senator without much accomplishment in spite of her high profile. Expect her to be busy in the next 2 years intiating a barrage of legislation.No longer will she defer to Shumer.He will have to share the platform when he makes his weekend chicken- little press conferences.Edwards will make a play of it in the primaries,but he is damaged goods.It should be evident to the Democrats by now that had they picked Gephardt they had a better shot of winning this election.It will be interesting to see if Gephardt is willing to invest the next 4 years to continue to be a player on the national scene.I kind of doubt it.




Choux rated this answer Excellent or Above Average Answer

Question/Answer
ETWolverine asked on 11/04/04 - Found this at USA Today

This is a really cool map of how the country voted, county by county.

http://www.usatoday.com/news/politicselections/vote2004/countymap.htm

Look at all that red. I find this surprising. It seems to me that even within the blue states, the counties voted mostly red.

Elliot

tomder55 answered on 11/05/04:

If Kerry had won the same bundle of states that gave him 252 electoral votes in this election, but the states were still valued according to the Congressional apportionment based on the Census of 1960, he would have won the election, 270 electoral votes to 268. The trend since then:

1960 census (1964, 68 elections) - Kerry 270, Bush 268

1970 census (1972, 76, 80 elections) - Kerry 270, Bush 268

1980 census (1984, 88 elections) - Bush 276, Kerry 262 1990 census (1992, 96, 2000 elections) - Bush 279, Kerry 259

2000 census (2004, 08 elections) - Bush 286, Kerry 252

This is indicative of a potential long-term problem for the Democrats: they are strongest in the parts of the country that aren't growing anymore. Even since the 2000 election (which was still based on the 1990 Census) the states Kerry won this time around are worth seven fewer electoral votes

(http://www.samizdata.net/blog/archives/006890.html)

They have tried to replace their former core constituency ;the FDR Democrats(a shrinking core for sure ) ,with pandering to minority coalitions including the largest one ,the newly arrived Americans from Latin America ;but suprise ,suprise ,suprise; they never expected them to not be monolithic and to have a significant part of the the Latino community bolt to the Republican ranks .That they have in general the same basic core beliefs as middle class America has.

ETWolverine rated this answer Excellent or Above Average Answer

Question/Answer
Choux asked on 11/04/04 - Howard Dean

It was just on TV that Howard Dean is in the running for the Chairman of the Democratic National Committee replacing McAuliffe who is out.

tomder55 answered on 11/05/04:

he showed them how to fund raise and organize in the 21st Century. If they are insistant about shooting themselves in the foot by picking a N.E. Dem. then a better choice would be Chris Dodd who right now has moved beyond the disappointment of the last election and is opening the debate in the party about it's future in a clear honest manner .(he won so it was not as devastating to him)

We Democrats better think long and hard about what happened ... and how our party is going to connect with the hopes and aspirations of the people,"...."We have lost the ability to connect with people's value systems, and we're going to have to work to get that back."

Green Party leader Jo Chamberlain said that leftists should now abandon the Democratic Party :

"We stepped aside and told the Democrats, 'Go get the White House,' and they failed," she grumbled. "So now, the progressive left is suffering because of their poor planning and leadership."

If the Dems.were smart ;they would kick out the progressive wing of the party and let them go their own way.

Choux rated this answer Excellent or Above Average Answer
elgin_republicans rated this answer Excellent or Above Average Answer

Question/Answer
Itsdb asked on 11/04/04 - Indications?

As many of you expressed, I hope for (but wouldn't bet on) a more unified country. In just checking again on the Democratic Underground today, the forums are back up...and they are a hoot. (see, we like to laugh)

coloradodem2004 tells us, "I am seriously thinking about cutting Republicans I know out of my life.

I am so resentful right now and many of them are heartless whiners anyway. They are so full of themselves."

Some replies, or at least ones I could post:

BUSHOUT (1000+ posts) Wed Nov-03-04 11:46 PM

4. Do it. They need to realize this is about more than gay marriage.

This is about the health and safety of America and the world.

It's personal.

all_hail_gwb (892 posts) Wed Nov-03-04 11:48 PM

7. I already have!

Old friends (husband and stepford wife) of about 15 years. Rabid 'pukes. I will never speak to them again.

My in-laws. Haven't spoken with them in at least 2 years and now they get a 4 year extension.

They'll be missed!

The Great Escape (336 posts) Wed Nov-03-04 11:48 PM

8. I Agree...

I can't cut my Mom and Dad out. I just can't. But the other aunts and uncles and cousins and the occasional co worker I'd share a beer with. Well, I won't make a big production out of it but I'll just let them slip away. Why should I waste my beautiful mind worrying about something like that!"

reeree (38 posts) Wed Nov-03-04 11:53 PM

11. I take votes for W personally.

I get really resentful of my Republican friends. Especially my female Republican friends. I just want to yell at them, "How stupid can you be? Don't you realize your party hates you because you're a woman?" I only have about two Republican friends I care to continue talking to... we just can't talk about politics ever.

Thank god my dad hated W enough not to vote for him this year. (He abstained.) My mom voted for Kerry, specifically to avoid my wrath. Otherwise I would have had difficulty continuing to talk to my parents. It would have been like a personal blow, like they just hated me so much they decided to destroy the future for me.

Gloria (1000+ posts) Wed Nov-03-04 11:56 PM

14. I already don't talk to my brother or his fundie wife....and I'm now

finding it almost impossible to talk to my freeper mother. I am to the point that I've lost all respect for her."

Jen6 (1000+ posts) Thu Nov-04-04 12:22 AM

34. I am. Every last one

they are toxic.

But hey, at least Jackie97 still wants to communicate...

42. I don't think that's a good idea.

I have mixed feelings between wanting to give them hell and wanting to stay friends with them.

Here's the thing. Not communicating with them has gotten us nowhere all these years. Marching down the street and screaming at counter protesters hasn't made the situation better between us.

I think one of the ways to change this country for the better is to keep talking to them when we have it in us to again. We've got to try to change their hearts and minds. All conservatives are not alike. Some are just fiscal, some are just social conservatives. I think there might be a way to get through to some of them that voting for somebody like Bush in the future is wrong.

When Bush's hell comes down on us, they'll be right beside us. They'll need a shoulder to lean on, and it won't help us to push them away.

I'm resentful too, but I think we need to keep talking to them if we can.

Let's not let them chase us out of the country either. The last thing I want is a world superpower being controlled solely by conservatives.

*******************************************************

That's from the "Help and Support" forum...I haven't even checked out the "Fighting and Acrimony" forum - and they think we're full of ourselves?

Do we just write this off as venting a little, or a lot of frustration, or is this an indication of things to come?

Is this how the radical left really feels, willing to sacrifice friends and family over their agenda, unwilling to compromise but willing to tolerate anything but opposing points of view, unwilling to consider the notion they just might be wrong about a few things themselves?

I for one will never, by the grace of God, cut people out of my life because we don't think the same.

Steve

tomder55 answered on 11/04/04:

I've checked into some left wing blog sites also ;it is indeed a hoot. Some are in denial ;some spew venum ,some are genuinely reflective about the future of the progressive movement. I checked into George Soros's web page .Evidently he now has too much time on his hands. Some have been saying that the ballot boxes were rigged and did not count the vote accurately . Over at Air America they are saying that they are still relevent ,and are not going anywhere. But if they were established for the sole purpose of defeating Bush maybe their backers will pull the plug.

One word of advice to the progressives. They may be doomed to being a perpetual minority in this country .Anyway you slice or dice it ;abortions disproportionately cull their ranks .

Itsdb rated this answer Excellent or Above Average Answer
ETWolverine rated this answer Excellent or Above Average Answer

Question/Answer
Choux asked on 11/04/04 - Ashcroft Leaving

sooner rather than later. Just on CNN.

tomder55 answered on 11/04/04:

I expect Bush will reshuffle his cabinet .If Ashcroft goes it will not suprise me . Don't expect to see a libertarian replace him however. Bush is free to choose people who reflect his views ,and it is doubtful that he will face serious challenges to his nominees ;as you know ;Bush was tough about law enforcement in Tx. as gov. I expect he will pick someone simularily tough. It is my hope that whoever he picks gets tougher with border integrity.

I'm more interested in seeing if Powell ;Rumsfeld ,and Rice remain . Rice in particular has indicated that she'd like to return to the privare sector .Bush should do everything he can to convince her to remain .

Choux rated this answer Excellent or Above Average Answer
excon rated this answer Excellent or Above Average Answer

Question/Answer
darkstar asked on 11/03/04 - Surreality TV.

Surreality TV.


In the end (as Bill O'Reilly would have been forced to reply in a deposition), there is one thing that almost hurt our righteous efforts in the 2004 election more than anything: our old nemesis, something secular people refer to as "reality." Seemingly billions of dollars spent to paint Mr. Kerry out to be a bumbling, incompetent scaredy-cat, were turpentined down the toilet by Mr. Kerry being sneaky enough to have stellar performances in all three debates. Mr. Bush, on the other hand, is better suited to scripted comedy than reality TV. He is better at "approving" a taped message rather than venturing into the more treacherous terrain of trying to articulate one live. At the debates, stripped of a script and relentless adulation, our handsome President showed a snarling impatience more commonly expected on the mugs of the hourly help at Gucci than the Leader of the Free World. As we've learned from Victoria Gotti (that dreadful cross-pollination of Nancy Sinatra, John Belushi, Donatella Versace and a longshoreman), if the goal is to be respected, some people are simply not meant to have a reality show. And our handsome President was never meant to let reality show. Fortunately for us, as the growth of the evangelical movement has proved, Americans have never had much taste for that "reality" nonsense.

betty bowers

tomder55 answered on 11/04/04:

The plain-spoken moralist for whom religion matters greatly, the common man who seems too small for the presidency but is confronted in office by a cataclysm that re-creates him; who rises to the challenge and transcends it; who faces a tough re-election battle and wins it; who redefines the nation's mission in the world and emerges a hero--that is a traditional American story. It is Lincoln's story. (In summer 1864, prominent Republicans wanted to find a
stronger candidate.) No president matches Lincoln's greatness, but in modern times this was Harry Truman's story; and today it is George W. Bush's also(http://www.weeklystandard.com/Content/Public/Articles/000/000/004/884gumsm.asp)

darkstar rated this answer Excellent or Above Average Answer

Question/Answer
sapphire630 asked on 11/03/04 - First in 4

...Just think if'n four more years the United States says a non-natural citizen can run for President it can be Schwarzenegger vs. Hillary. So we would have either our first alien Pres. or our first women Pres. and the added bonus would be ole Bill would be first man.
So things could be worse....I suppose...

tomder55 answered on 11/04/04:

I stand on record now of opposing any amending of the requirement that a President be U.S. born . I will not support an Arnold run at the presidency regardless ;even if it means voting for Hillary(God forbid).

darkstar rated this answer Excellent or Above Average Answer
sapphire630 rated this answer Excellent or Above Average Answer
Itsdb rated this answer Excellent or Above Average Answer
Choux rated this answer Excellent or Above Average Answer

Question/Answer
Choux asked on 11/03/04 - Congratulations

to the members of the Board who supported Pres. Bush on a clear cut victory in yesterday's election. All our hopes are for a successful second term. Cordially, Choux

tomder55 answered on 11/03/04:

thanks

I also appreciate that Kerry will be gracious in defeat.His conduct is sending a strong message of unity and resolve. This should be a shining example to the world how free people conduct business of governance . Already world leaders opposed to Bush's policies are calling for a new relationship with us.

Choux rated this answer Excellent or Above Average Answer
ETWolverine rated this answer Excellent or Above Average Answer

Question/Answer
Itsdb asked on 11/03/04 - Election coverage

What's your take? Who did you watch? For the most part I watched Fox News (I know, no surprise there), but I gave a fair amount of attention to CNN and scanned everyone else from time to time. Besides hearing various forms of the word 'caution' a gazillion times, it was quite entertaining.

Fox's team and graphics were by far the best, except for Brit Hume, who was positively silly later on from being up past his bed time. They win this morning, too...but I'd give them a huge vote for any show featuring Kieran Chetry :)

I liked CNN's graphics concept, but combined with their roving Blitzer show it was really ugly. Larry King looked and sounded positively archaic.

Didn't spend much time on MSNBC or the big three, although I'd give the biggest kudos to Jennings and ABC among them. What I saw of CBS almost had me rolling in the floor. Rather was panicked and desperate, and aside from Weld and Sharpton on The Daily Show, gave me the biggest laugh. When I first turned on the TV, Dan looked like he was about to cry, and in his best Cosmo Kramer impersonation told us to "get off your duffs and go vote" and then come back and watch him.

At 8:00 when I left for work, only Fox had given Ohio to Bush while the rest were still desperately trying to spin out the possibilities for Kerry. But I never did figure out the differing electoral counts they were giving. With only 1% of the vote out in New Mexico and Bush up by 12,000, Fox had Bush at 269 for hours. I think the others showed Bush at 254 without calling Ohio, so somewhere I missed what 5 electoral votes Fox hadn't given but the others had...but I know the majority of the networks were holding out every last ounce of hope for Kerry...no surprise there.

So, unless I've missed it while writing this, will any of them call it before a concession by Kerry, or how long will we wait? It's just a matter of time :):)

Steve

tomder55 answered on 11/03/04:

not an answer ;just an update

KERRY TO CONCEDED AT 1:00 EST

(after Bush threatened to go first and claim victory .)

Itsdb rated this answer Excellent or Above Average Answer
purplewings rated this answer Excellent or Above Average Answer

Question/Answer
ETWolverine asked on 11/03/04 - Jumping the Gun

It seems to me that there were several cases of "jumping the gun" in yesterday's election coverage.

First there were the early exit polls at around 4:30pm EST. Matt Drudge and several blogers jumped the gun and posted the polls which showed Kerry leading in most of the swing states early on. This scared the bejezus out of a lot of Reps, and people began scrambling for a better review of the data. And they were wrong.

This marks the THIRD election in a row that the early exit polls were dead wrong, and in which they may have influenced the election process.

I think it's high time that exit poll results were BANNED from being publicly displayed until the polls close. They are inaccurate, and they have a negative effect on the elections themselves. That is an election law reform that I would back.

The second case is a little more iffy. I'm not sure it really was a case of jumping the gun or not. I'd like your opinions.

As a general rule, most of the networks were very careful not to project winners without VERY solid data to back up those projections. In a few cases, the closing data reached 99% before a winne was projected. There were those who commented that maybe the newsies were waiting a little too long to make projections, but the general concensus was better safe than sorry. I happen to agree... I don't mind waiting another hour to get the info, as long as it is correct.

The sole possible exception was FOX who called Ohio for Bush. They were the only network to do so. The data on which they were basing their projection was solid... the numbers clearly showed Bush up by over 100,000 with something like 98% of the districts in. What they didn't consider was the challenge of the provisional ballots by the Kerry people. And even later, they were able to convincingly argue that with a 140,000 lead, Bush was statistically unbeatable.

So the question is, did they jump the gun? Should they have held off on their projection as the other networks did?

Elliot

tomder55 answered on 11/03/04:

ABC radio was saying at 3:00 that although it would be almost impossible for Kerry to win Ohio ,the network had not yet called it . I think all the neworks were gun shy about calling a state . They remember Fla. 2000 very well. Peter Jennings spent 5 minutes trying to explain the procedures they Generally I was not thrilled by Fox 's coverage. They were suckered in early by the exit polling data ,and were ready to start dusting off the excuses why Bush failed. I found that ABS and Peter Jennings did a fine job . I turned on Rather for comic relief and Brokaw occasionally to see his ashen look of despair when realization of Bush victory was becoming evident.

The exit polls have been an unreliable indicator for a long time now.(at least 3 election cycles). I think either the voters are intentionally lying to the pollsters or they are doing a poor job of sampling. My theory: folks likely to vote Republican have things to do and will not stop and take the time to be polled ;so they end up with an unrepresentitve sampling.

ETWolverine rated this answer Excellent or Above Average Answer

Question/Answer
Choux asked on 11/02/04 - Election Coverage

What networks(public and cable)do you think will have the most comprehensive election coverage tonight? I"m looking for rapid coverage in the field, good graphics, pleasant anchors, I guess that's it.
Thanks, Choux

tomder55 answered on 11/02/04:

I don' stick with one . I like C Span ;ABC (especially when George Will commentates ) and Fox's Brit Hume primarily (although Chris Matthews and Tim Russett can be good also on MSNBC) . Before I go to bed I will log onto the blogsphere .Some of them may have sources in the networks and unlike the networks will not be shy about reporting results .None of them will make a prediction before 11 EST. because the polls in theWest Coast will not close before then . But my gut says it will not be as close as the polls indicate .

Choux rated this answer Excellent or Above Average Answer

Question/Answer
darkstar asked on 11/02/04 - whats your voting weather today?

election officials in ohio have stated that the weather affects the voter turn out....Whats your weather today? mine is rainy and foggy(nw ohio), infact it has remained that way here now for 3 weeks. And have you been out to vote today?

tomder55 answered on 11/02/04:

the weather in N.Y. is irrelevent. turnout will be high but Kerry will take the State. I am in N.J. now ,and am hoping that Bush can take the State . I think however that Bush's popular vote will have to exceed 55% for him to win here . Nice weather overall ;mild ;cloudy ;no excuse not to go to the polls. My best guess is that Bush will garner about 280+ electoral votes and will be announced the winner sometime about midnight EST by the networks (the bloggers will know earlier) . There is a real chance that he may even get a majority (50%+ of the votes .)I just logged onto tradesports.com .There investors bet on the outcome of the election.Right now Bush is leading there by 56% .That doesn't mean much except they are putting their money up on the outcome.

I have to drive home to N.Y. later this evening and vote before the polls close(there won't be a long line in my little town ). Thankfully there was little for me to do locally this year as far as campaigning .All my candidates ran unopposed.

Choux rated this answer Excellent or Above Average Answer
darkstar rated this answer Excellent or Above Average Answer
excon rated this answer Excellent or Above Average Answer
Itsdb rated this answer Excellent or Above Average Answer

Question/Answer
excon asked on 11/01/04 - The Supremes


Hello Sperts:

The next pres will probably appoint 4 Supreme Court Justices. That legacy will last far longer than his administration, whoever wins.

I fear for my freedoms if Bush appoints the Ashcrofts of the world. If GW wins, do you think Roe v Wade is safe? If not, what will happen if it is reversed? Riot? Civil war? Nothing? What about civil rights? Any reversals there, and/or consequences?

And what would a liberal court do? Gay marriage? Big deal!!! Yeah, I think it should be so, but what other great liberal ideas are there? None that I can think of.

What do you think will change with a new court?

excon

tomder55 answered on 11/01/04:

Hard to tell what the impact of new appointees will be . Right now there is a balance with OConner being the swing vote in most cases. Replacing Rhenquist with another consevative will not change the balance.OConner has indicated she'd like to retire .I do not know of any other who is considering it so I doubt that 4 Supremes will be appointed.

Right now Bush can't get district court judges through the confirmation process let alone a Supreme. The Dems . have tied Bush's hands even though there is little to object about his appointments.I doubt if an 'extreme' court appointee would be confirmed no matter who the Pres. is .

Both sides are using the issue to energize their bases but I think the issue is way overblown. I don't like judges making law ;and both liberal and conservative courts have been guilty of that in the past.

In this months 'The Atlantic Monthly', writer Benjamin Wittes demonstrates that liberals and conservatives have been predicting apocalypse for the court for more than 20 years.(sorry I can't provide a link because it is by subscription only ) Partly because of that, the U.S. Senate has become much more aggressive in vetting nominees. It is extremely unlikely that, a brilliant but ideological nominee like Antonin Scalia could make it through Senate confirmation today.

excon rated this answer Excellent or Above Average Answer

Question/Answer
Saladin asked on 10/30/04 - More of the same ...


The Bush administration continues with its tired old plicies of "Lies, Deny, and Cover Up"

When will the American people be told the truth about 9-11?

Consider this:

=======================================================


Part of 9/11 Report Remains Unreleased; An Inquiry Is Begun
By JIM DWYER

Published: October 30, 2004


One last chapter of the investigation by the Sept. 11 commission, a supplement completed more than two months ago, has not yet been made public by the Justice Department, and officials say it is unlikely to be released before the presidential election [no prize for guessing why not!], even though that had been a major goal of deadlines set for the panel.

Drawing from this unpublished part of the inquiry, the commission quietly asked the inspectors general at the Departments of Defense and Transportation to review what it had determined were broadly inaccurate accounts provided by several civil and military officials about efforts to track and chase the hijacked aircraft on Sept. 11. [Broadly inaccurate = ???]

David Barnes, a spokesman with the Department of Transportation, said yesterday that if the reviews found wrongdoing, the inspector general could recommend administrative penalties or ask federal prosecutors to begin a criminal investigation.

"The investigation is ongoing,'' Mr. Barnes said, "and we don't know when it will be done."

In testimony before the commission, officials had described a quick response to the hijackings that narrowly missed intercepting some of the planes, but the commission's investigators later determined from documentary evidence that none of the military planes were anywhere near the four airliners.

In addition, officials at the Federal Aviation Administration testified that they had notified the military within a few minutes of each hijacking, but the investigation found that tape recordings contradicted that assertion.

The commission, in its final report, said that the true picture "did not reflect discredit" on individuals, but that unreliable testimony about the events had made it harder to understand the problems.

Besides the pursuit of the hijacked planes, the supplement, a monograph 60 to 70 pages long, revisits other subjects in the commission's final report of July - telephone calls made from the hijacked airplanes, airline security and orders issued that morning by President Bush and Vice President Dick Cheney - and provides additional detail or context, former commission members said.

The monograph also finds shortcomings in the Transportation Security Administration, the agency formed to buttress airline security after the hijackings, said Bob Kerrey, the former Democratic senator from Nebraska and a commission member.

Mr. Kerrey suggested that presidential politics were behind the delay in the report's release, but a spokesman for the Justice Department, Mark Corallo, said that an ordinary review of the material for national security clearance was complicated when the commission shut down in August.

"It's unlikely in the next few weeks," Mr. Corallo said of when the supplement would be released. "It was a real legal quandary."

The monograph was submitted to the Justice Department just as the commission's term expired on Aug. 21, a date selected by Congress after long negotiations to avoid bringing out the commission's report at the height of the presidential campaign.

It arrived not only as the commission became legally defunct, but also as many commission members and the staff lost their security clearances, Mr. Corallo said.

That meant no one from the commission could discuss with the Justice Department lawyers how to edit material that needed to be changed for security reasons, he said.

"Had the commission gotten it to them two or three days before the deadline, they could have resolved any issue in minutes, as they usually do," Mr. Corallo said.

As a result of these complications, the supplement is the first of the commission's documents to be completely controlled by the Bush administration.

While the commission was still in business, it was able to exert pressure on the White House when all 10 members, 5 Democrats and 5 Republicans, simply issued a public request for cooperation.

"I am surprised that the process has dragged on this long, and I think it's inappropriate," Richard Ben-Veniste, a Democrat on the commission, said. "It is longer than any other review of written material."

Discussions on the monograph's fate are being held between the Office of Legal Counsel at the Justice Department and Daniel Marcus, the commission's former general counsel.

"I think I've convinced them that even though we don't exist anymore, it ought to be viewed as a public document," Mr. Marcus said.

The monograph has two sections, he said. One concerns airline security, discussing the Federal Aviation Administration and the Transportation Security Administration.

The other section, he said, provides a detailed timeline of the movements of the hijacked planes the morning of Sept. 11 and the response by the civil and military aviation officials.

On July 29, Mr. Marcus wrote to the inspectors general of the Transportation and Defense Departments requesting reviews of the testimony of those officials.

He would not comment this week on the request or the letters, but representatives for both departments confirmed that investigations were under way.

=======================================================

"Lie, Deny, Cover Up"

No thank you.

Sal

tomder55 answered on 11/01/04:

besides the obvious partisan swipes that will be used;what will any of this prove except that the U.S. was not prepared for the 9-11 attacks ;and would not have been prepared for them no matter who the President was ?I'm tired of the whole thing .the 9-11 commission did some good ;but they could've been much more effective if the partisanship grandstanding that was done could've been avoided . The members you state; Richard Ben-Veniste ;and Bob Kerry were two of the worst along with the dramatics by witness Richard Clarke. Bi-paritisan indeed ;those stooges did everything in their power to point an accusing finger at the Bush Adm. while ignoring the deficiencies of the previous Adm!

Saladin rated this answer Excellent or Above Average Answer

Question/Answer
LTgolf asked on 10/29/04 - UBL tape

Will the Osama Bin Laden tape, just hours old , have any impact on the election.

It is obvious to me that UBL does not want Bush reelected.

Leon

tomder55 answered on 10/30/04:

A State where it may make a difference is N.J. That former Blue State is now a swing State. There are many former 'soccer moms' there who are now 'security moms'. They watched the towers fall.They had friends or family die on 9-11. They saw their school blueprints on al-Qaeda hard drives.They know that in Spain al-qaeda bombed a train;that in the Aussie election there was bombing at the Australian embassy in Jakarta before the vote ;that in both the Afghan election and now in the run up to the Iraqi elections that terrorists are trying to disrupt the polling by intimidation .In the U.S.;we get a couple of video tapes.

ETWolverine rated this answer Excellent or Above Average Answer
LTgolf rated this answer Excellent or Above Average Answer

Question/Answer
CeeBee2 asked on 10/29/04 - Just so the other side doesn't feel left out --

This was in today's Chicago Sun-Times (and of course those lovable Republicans will refute everything):

For God's sake, vote him out
October 29, 2004
BY ANDREW GREELEY

There are two proportionate reasons for rejecting President Bush's bid for re-election. Both the United States and the world are a mess. Mr. Bush is responsible for both messes. The first president ever to claim de facto infallibility, Mr. Bush tells us that he follows his instincts in decision-making after praying over the decision and talking to God. He admits no mistakes -- how could anyone who has a direct link to God make a mistake! In his next administration he will receive more divine inspirations which will make both the country and the world even more messy.

Consider the American economy. He has turned the biggest budget surplus in history into the biggest deficit because he wanted to give more money to the "haves and have mores" as he called them. He has presided over the largest job losses since the Great Depression. He has stood idly by while hundreds of thousands of American jobs have been flown overseas. His reform of health care has made it more expensive and more difficult for the elderly. He declines to rein in the greed of the drug companies and thus drives many Americans to Canada -- of all places -- to buy the medicines needed to stay alive. He has cast doubt on the future of Social Security. He has been on the bridge during the current absurd panic over flu vaccine; the deaths of those elderly and children who are not able to obtain flu shots are on his hands. What if one of those who die is your parent or spouse or child? He has not lifted a finger to help the many Americans whose pensions are being eaten up by greedy employers. Oil prices are climbing rapidly and the stock market is tanking.

We want four more years of this stuff?

Fecklessly he started the ill-advised and ill-prepared war in Iraq in which some Americans have to come close to mutiny to protect them from orders to bring contaminated fuel in badly equipped trucks to units that won't accept it. He misled the American people about the weapons in Iraq and the involvement of Iraq in the World Trade Center attack. He is disgusted, he tells us, by the kidnappings and the beheadings, the car bombs and roadside bombs, the ambushes and murder of civilians, but the bad decisions he and his cabinet made were mandated by God and could not have been mistakes. Pat Robertson tells us, however, that Mr. Bush told him that God had disclosed that the casualties in Iraq would be light. Maybe that was God's mistake!

Do we want him to continue with these god-driven policies for four more years? Eleven hundred dead Americans already. How many more thousands will have to die before God will tell Mr. Bush to get out of Iraq? How many tens of thousands more Iraqis will have to die?

The world is a mess because the United States is the natural leader of the free world and the American president the natural president of the free world. He blew the capital of support and sympathy that flowed to the United States after the World Trade Center attack by his "Bush Doctrine" that turned him into the bully of the free world. Next year the Poles will leave Iraq because the Polish people don't like the war. The Poles -- our strongest allies in Mr. Rumsfeld's "New Europe" -- are fed up with us! Four more years of divine inspiration and what will be left of America's power and prestige? We will still be a giant but like Gulliver a tattered giant chained to the ground by our president's madcap inspirations.

The pope is infallible only in certain limited circumstances and on specific matters. Unlike the pope, Mr. Bush apparently sets no limits on the policy decisions that will be made by conversations with God. We want four more years of those decisions?

The president, like every human, is entitled to his own relationship with God. He is entitled to use that relationship to make decisions, to justify them later, and to stick to them no matter what happens. Many Americans will accept such decisions because they believe he is a "godly" man. Not everyone else has to tolerate four more years of his divine right to govern.

Even if the election is close, Mr. Bush will win it. His lawyers are ready to go back into court and the supine Supreme Court will give the country four more years of divine right rule.

Do we really want that?

tomder55 answered on 10/30/04:

yesterday it was reported the U.S. economy posted a 3.7% growth. That was declared disapointing by the MSM. By contrast the German economy by far the most robust in the EU grew at 1.8% and they were expecting more declines. Kerry keeps saying that this is the worse economy since Hoover .It is a lie.


I don't have time to address all the misconceptions in this article.Suffice it to say I respect Father Greely when he speaks on matters of the church but his socialist leanings are well known and that context should be understood in every political piece he pens.

CeeBee2 rated this answer Excellent or Above Average Answer
ETWolverine rated this answer Excellent or Above Average Answer
powderpuff rated this answer Excellent or Above Average Answer
purplewings rated this answer Excellent or Above Average Answer

Question/Answer
Itsdb asked on 10/29/04 - Kerry's Afghan Amnesia

By Charles Krauthammer

Friday, October 29, 2004; Page A23

In the 1990s, Afghanistan was allowed to fall to the Taliban and become the global center for the training, indoctrination and seeding of jihadists around the world -- including the mass murderers of Sept. 11, 2001. This week, just three years after a two-month war that destroyed the Taliban, Afghanistan completed its first free election, choosing as president a pro-American democrat enjoying legitimacy and wide popular support.

This represents the single most astonishing geopolitical transformation of the past four years. (Deposing Saddam Hussein ranks second. The global jihad against America was no transformation at all: It existed long before the Bush administration. We'd simply ignored al Qaeda's declaration of war.) But perhaps even more astonishing is how this singular American victory has disappeared from public consciousness.

Americans have a deserved reputation for historical amnesia. Three years -- an eon -- have made us imagine that the Afghan war was easy and foreordained.

Easy? In 2001, we had nothing there. What had the Clinton administration left in place? No plausible military plan. Virtually no intelligence. No local infrastructure. No neighboring bases. The Afghan Northern Alliance was fractured and weak. And Pakistan was actively supporting the bad guys.

Within days of Sept. 11, the clueless airhead president that inhabits Michael Moore's films and Tina Brown's dinner parties had done this: forced Pakistan into alliance with us, isolated the Taliban, secured military cooperation from Afghanistan's northern neighbors, and authorized a radical war plan involving just a handful of Americans on the ground, using high technology and local militias to utterly rout the Taliban.

President Bush put in place a military campaign that did in two months what everyone had said was impossible: defeat an entrenched, fanatical, ruthless regime in a territory that had forced the great British and Soviet empires into ignominious retreat. Bush followed that by creating in less than three years a fledgling pro-American democracy in a land that had no history of democratic culture and was just emerging from 25 years of civil war.

This is all barely remembered and barely noted. Most amazing of all, John Kerry has managed to transform our Afghan venture into a failure -- a botched operation in which Bush let Osama bin Laden get away because he "outsourced" bin Laden's capture to "warlords" in the battle of Tora Bora.

Outsourced? The entire Afghan war was outsourced. How does Kerry think we won it? How did Mazar-e Sharif, Kabul and Kandahar fall? Stormed by thousands of American GIs? They fell to the "warlords" we had enlisted, supported and directed. It was their militias that overran the Taliban.

"Outsourcing" is a demagogue's way of saying "using allies." (Isn't Kerry's Iraq solution to "outsource" the problem to the "allies" and the United Nations?) And in Afghanistan it meant the very best allies: locals who had a far better chance of knowing which cave to storm without getting blown up. As Kerry himself said on national television at the time of Tora Bora (Dec. 14, 2001): "What we are doing, I think, is having its impact and it is the best way to protect our troops and sort of minimalize the proximity, if you will" -- i.e., not throwing American lives away in tunnels and caves in alien territory. "I think we have been doing this pretty effectively and we should continue to do it that way."

Now, as always, the retroactive military genius says he would have done it differently. Yet in the same interview, when asked about how things were going overall in Afghanistan, he said, "I think we have been smart; I think the administration leadership has done it well and we are on the right track."

Once again, the senator's position has evolved, to borrow the New York Times' delicate term for Kerry's many about-faces.

This election comes down to a choice between one man's evolution and the other man's resolution. With his endlessly repeated Tora Bora charges, Kerry has made Afghanistan a major campaign issue. So be it. Whom do you want as president? The man who conceived the Afghan campaign, carried it through without flinching when it was being called a "quagmire" during its second week and has seen it through to Afghanistan's transition to democracy? Or the retroactive genius, who always knows what needs to be done after it has already happened -- who would have done "everything" differently in Iraq, yet in Afghanistan would have replicated Bush's every correct, courageous, radical and risky decision -- except one. Which, of course, he would have done differently. He says. Now.

*******************************************************

Shouldn't Kerry see a doctor about all his amnesia episodes?

tomder55 answered on 10/30/04:

huge armies for centuries have tried and failed to conquer Afghanistan. The Russians poured hundreds of thousand troops their and died by the thousands in Tora Bora and other parts of the country. Kerry has no clue how to be a commmander . Most of our troops wouldn't let him lead them up an ant hill

Itsdb rated this answer Excellent or Above Average Answer
ETWolverine rated this answer Excellent or Above Average Answer

Question/Answer
Choux asked on 10/29/04 - Close Election

It appears that the election on Tuesday could be decided by good or bad weather, or other variables. What a battle!

I voted absentee/disabled on October 7th for Kerry/Edwards and mailed in my ballot. In addition, I voted for Barak Obama for the Senate because the Republican Party in Illinois, which is in shambles(despite my sorority sister being the leader :D), nominated what *many* call a "goofball" for their Senate candidate. He is currently polling 19%. :D

Choux

tomder55 answered on 10/30/04:

Dr Keyes is brilliant and has solid moral principles.But if I was in Illinois I would vote for Obama also. I can't say on the one hand Hillary is a carpetbagger without saying the same about Keyes. He never stood a chance there,but I think that he is trying to show the black community there that there are alternatives to the one party loyalty mindset.It will ultimately serve the black community better if no party can take their vote for granted.

Choux rated this answer Excellent or Above Average Answer
elgin_republicans rated this answer Excellent or Above Average Answer

Question/Answer
ETWolverine asked on 10/28/04 - State level elections

Hi all,

We've been paying so much attention to the Presidential race, that I haven't been watching the Senate, House and Gubanatorial races (except NY). Has anyone been watching these? Which way is the wind blowing in your area?

Elliot

tomder55 answered on 10/29/04:

N.Y. you know ;incumbent Democrats are going to sweep. Mills a local politician is pretty good in the assembly but he is clearly a sacrificial lamb.He is doing the pathetic N.Y. Republican party a favor in return for I do not know what. Shumer (being the only legit Senator from N.Y.)is going to romp ;and continue to gain national recognition with his 'chicken little' Sunday press confrence gig.

ETWolverine rated this answer Excellent or Above Average Answer

Question/Answer
ETWolverine asked on 10/28/04 - Arafat's Health

Most of the people that I have been talking to have assumed that Arafat's death would be a good thin for Israel.

I wonder if that is true.

John Gambling and Malcolm Honeline were speculating about this issue yesterday. One of their speculations caught me off guard.

The Sharon plan for unilateral pullout and closing off the West Bank is predicated on the fact that as long as Yasser Arafat is in power, Israel has no peace partner.

If Arafat is out of the way, that may not be seen as the case anymore.

If Arafat dies, the EU may pressure Israel and say, "Arafat is dead, now go and find a peace partner." And Israel may have no political choice but to comply. This could put the whole pullout plan on hold, throwing Sharon's plans into complete disarray.

I can't find any specific reason to disagree with this possibility, and I know Malcolm Honeline well enough to know that he know's what he's talking about. I find that just a bit scary.

Your comments are appreciated.

Elliot

tomder55 answered on 10/28/04:

I just heard that they are sending him to Paris ;effectively ending his isolation. [Interesting ;I wonder if they will insist he remove that dirty diaper he wraps around his head.] anyway ,consider the possibility that he survives .He may not return and try to build up his base of support in the European capitals. he could bring more pressure upon Israel alive then dead.

ETWolverine rated this answer Excellent or Above Average Answer

Question/Answer
ETWolverine asked on 10/28/04 - Arafat's Health

Most of the people that I have been talking to have assumed that Arafat's death would be a good thin for Israel.

I wonder if that is true.

John Gambling and Malcolm Honeline were speculating about this issue yesterday. One of their speculations caught me off guard.

The Sharon plan for unilateral pullout and closing off the West Bank is predicated on the fact that as long as Yasser Arafat is in power, Israel has no peace partner.

If Arafat is out of the way, that may not be seen as the case anymore.

If Arafat dies, the EU may pressure Israel and say, "Arafat is dead, now go and find a peace partner." And Israel may have no political choice but to comply. This could put the whole pullout plan on hold, throwing Sharon's plans into complete disarray.

I can't find any specific reason to disagree with this possibility, and I know Malcolm Honeline well enough to know that he know's what he's talking about. I find that just a bit scary.

Your comments are appreciated.

Elliot

tomder55 answered on 10/28/04:

Europeans invested heavily in Arafat for years .Where do they even presume to tell Israel what to do ? Had Arafat gone along with any number of peace proposals then the wall would not have been necessary .Arafat will be leaving behind a disfunctional society instead of a homeland ;quite a legacy! Without him I imagine ,Palestine may well have been it's own nation already .

If the Palestinians start implementing the 'Road Map' faithfully then Israel may indeed be confronted with the possibility of actually having a partner in peace .
But I see no hope for that ,and Israel should not place false hopes on that happening .I do not think that is what will happen . I do think it likely that a period of civil war will erupt in Palestine (hmmmmmmm----sit back and watch terrorist kill each other ).Fatah will die with him and hopefully a new Palestinian leadership will emerge to make a fresh start instead of Hamas.

Israel needs to build the wall ,and pray for U.S. success in Iraq and in the overall success of the Bush doctrine.

Itsdb rated this answer Excellent or Above Average Answer
ETWolverine rated this answer Excellent or Above Average Answer
excon rated this answer Excellent or Above Average Answer

Question/Answer
ETWolverine asked on 10/28/04 - This has been a VERY weird 24 hours.

In the past 24 hours, we have seen some very weird things happen.

- A full eclipse of he moon
- The Red Sox winning the Sorld Series
- Yasser Arafat rushed to the hospital and on the "brink of death"
- Yet another CBS/Dan Rather/NY Times moment as the claim of Bush being responsible for the missing 380 tons of high explosives is disproven. (The Washington Times is running an article claiming that Russian Spetznaz Special forces troops moved them to Syria in the weeks before the war... and maybe the WMDs too.)

What are we to make of all this happening all at once?

We are livin in "interesting times" as the old Chinese curse says.

And speaking of curses: it seems that the Curse of the Bambino is dead.

Or is it?

There's an old saying: if you are bad, you go to Hell. But if you are really bad, they give you a glimpse of Heaven first.

Could this have been the glimpse of heaven that makes the next 86 years so agonizing for the Boston Fans?

Only time will tell.

Elliot

tomder55 answered on 10/28/04:

that is funny . I will use it on my uncle who is a Sox fan.

Hopefully Arafat will soon be getting that glimpse of heaven before he fully joins the ranks of the 'spiritual leaders'.

ETWolverine rated this answer Excellent or Above Average Answer

Question/Answer
Itsdb asked on 10/27/04 - And Bush is using 'scare tactics'?

In Florida, Cher just informed us, "All the gay guys, all my friends, all my gay friends, you guys you have got to vote, alright? Because it would only be a matter of time before you guys would be so screwed, I cannot tell you. Because, you know, the people, like, in the very right wing of this party, of these Republicans, the very, very right wing, the Jerry Falwell element, if they get any more power, you guys are going to be living in some state by yourselves. So, I hate scare tactics, but I really believe that that's true."

Really? And will members of say, the Vice President's family get an exemption?

tomder55 answered on 10/28/04:

was this the rally that under 1000 people showed up for ;or maybe it was Rosie ODonnell's 68 person rally.

Itsdb rated this answer Excellent or Above Average Answer

Question/Answer
Yiddishkeit asked on 10/28/04 - Congrats to the Red Sox...

I would trade in another 86 years without a world series victory if it meant not having Bush in office for four more years. I suspect Bush will win and America will lose. Anyway I'm happy for Boston...starting with the comeback against the Yankees thru the sweeping of the Cardinals in the WS that was the greatest play-off performance in Baseball history. Red Sox are the only team in Baseball to ever comeback from an 0-3 game defict in the playoffs. They also set a record of eight straight victories in a single post-season...wow.


Bobby

tomder55 answered on 10/28/04:

fitting that the favorite team of Stephen King should break a curse on a night when there is an eclipse of a full moon.

time for this Yankee fan to tip his cap.8 straight is hard enough to do in the regular season. to do it in the playoffs ;after they were a couple of innings from elimination;using an ace pitcher who's ankle tendon was stitched to his skin just so he could suit up ;is mind boggling.

Yiddishkeit rated this answer Excellent or Above Average Answer

Question/Answer
elgin_republicans asked on 10/27/04 - Who will win?

Time to see who (or whom) the REAL experts are on this Board.

PREDICTIONS PLEASE (and why). I will donate a .50-off coupon for a can of Heinz beans to the nearest food pantry in the winner's name.

Rules: You must name the next President of the U.S., and THEN you must give the percent of his popular vote total. Please be aware that the next President need not have the highest percentage of the vote (just in case you didn't know that). Then tell us why you think such a thing.

I shall go first. Next President will be G.W.Bush. His percentage of the popular vote will be 54.8%.

Why: Because all the polls are trending in his direction, even the ones that are being skewed by the MSM in order to pretend this is really a close race.

Bush leads in all the swing states that Kerry MUST win, (MN, WI, IA, NM), and is even leading in states (or within 1%)that are supposed to be locked for Kerry (NJ, MI, and Hawaii for crying out loud).

The Bush swing states (except for NH) are pretty much done deals for the President (AR, MO, WV, FL, NV).

The winner will be the person with the closest percentage, AND the correct choice of the next President. Stars will be awarded on November 3rd (as will the coupon for the beans). GOOD LUCK.

tomder55 answered on 10/28/04:

The number of new registrants is such a big wild card in this race that I do not trust the polling #s . Just look at N.J. for an example . Gore won it in 2000 handily ;but yesterday the Quinnipiac University poll showed it a dead even race. I do not however know if the poll reflects the attitudes of the newly registered in N.J. ;apx. 463,000 of them. I do not know the demographics of the new voters;or if they will even vote in a large proportion .Three quarters of these new voters signed up in the 14 New Jersey counties carried by Al Gore four years ago.

While I think that 9-11 had arguably a bigger impact on N.J.than any State in the union ;which should help Bush's #s in the State ;this new group of voters may swing the State to Kerry. It is possible that new voters nationwide could swing this vote towards Kerry with a comfortable plurality (shudder at the thought).

On the other hand ,I am encouraged by the weeks campaigning sites. It appears that Bush has been making an attempt to reach out to the former Reagan Democrats in his stump appearances while Kerry has been touring States he should've locked up along time ago.
His stump message has been asking Dems. to recall the great leaders of the Democrat past like FDR and Truman and asking them if Kerry passes that muster. Kerry and his accomplices in the major news networks in have been lobbing one grenade after another at Bush's leadership ability ;but none of them have hit.

I am also encouraged by polling #s that show Bush chipping away at traditional Democrat constituencies. Black support for Bush has doubled to almost 20 percent . Although that # may sound small;in a close election that could be decisive.Kerry has not been able to crack Bush's base support to that degree.

I think the American people in their heart realize what is at stake in the election . I concede that Bush has not been the best preformer . Mistakes have been made ;as happens in all wars .The Americans understand that reality .

The war against Islamo-fascism will take a number of years to execute and more than one Democrat will be elected to lead it;but please not this one. Had the Democrats picked Lieberman ;or even Gephart I would be more comfortable about a Democrat President this election ;but they didn't .They culled the pack ,and Kerry was the best they could come up with? Why ? what does he bring to the table?

When he said that he would put American policy to a 'global test 'he confirmed my worst fears. He had said in the past that he was more comfortable with U.S. soldiers fighting with blue helmets on .In a world where freedom depends on American muscle he would have our soldiers and decision makers subject to the judical disgression of an International tribune that is not of our control .When he said he wants to bring us back to the good old days when terrorism was a "nuisance"and comparable to victimless crimes;then he clearly signaled to me he would not take the war seriously .

I do not believe his pledge to continue in Iraq ,and neither do his supporters.Can youi imagine the howling of the likes of Michael Moore in the summer of next year if Kerry has not withdrawn from Iraq? But if he stays the course ;then his " wrong war in the wrong place at the wrong time "rhetoric will undermine his own efforts . Why would the allies he has promised to bring in even consider it if he has signaled that he is not committed to the effort? Why would our existing allies hang around? The only conclusion I can make from Kerry is that he would cut and run .

ok to predictions : No war time President has been ousted in American history . In the 20th Century the only incumbents who have been ousted are during elections where there was a STRONG 3rd party challenge. I think the winner of this election will win by a significant plurality ,and with enough of an electoral college lead to avoid a rediculous challenge in the Court. Speaking of the Courts :I think the cancer of Renhquist will remind the base of Republican party that they cannot sit this one out(as many of them did in 2000).They will be energized. The winner of this contest will likely pick 3-4 Supreme Court justices ,and an untold # of Fed. judges Many will feel that the foundations of American culture are in the balance .

Expect a huge turnout ;expect a Bush win (your 4-5 point percentage is about right).

elgin_republicans rated this answer Above Average Answer

Question/Answer
powderpuff asked on 10/27/04 - civil war

Does anyone think there might be civil war coming to the US? With our country so deeply divided over the Presidential race, if we get some terrorist attacks on our soil post election, what do you think the chances are of a civil war happening here? How could our government handle a situation like that and at the same time continue to fight the global war on terror?

A little worried......

tomder55 answered on 10/27/04:

One comment regarding Elliot's response.Communications technology is replacing regional separations as a way for people to isolate themselves.I can choose my news sources like never before .This may be leading to a time of entrenched opinions that we have not seen since the civil war .I would like to think that we are all looking to a common goal but there are cultural splits that are evident.

That being said ;I have tried on a few responses to demonstrate that the divisions in the country are not that unusual ,and in fact are healthy to the political process. What is wrong with passionate debate ? We have weathered even greater divide in recent history than we have today . The 1960s were far more turbulant .The current campaign is no more rough and tumble than the 1968 campaign. Reagan in the 80s was a polarizing force also . The 1960 election between Nixon and Kennedy was as close as this one is projected to be and certainly as close as the 2000 election was. Of course the big difference then was that Nixon had the good sense not to challenge the results in court . Much of the garbage that is happening this cycle is a result of the candidates;instead of acting like statesmen dragged the courts into the process.

Although much of the political animosity that is usually confined to the the Washington beltway has seeped into the general populace this time;I still think it is isolated to the diehards who are desperatly trying to exercise the masses to their cause.My experience is that most people have strong opinions but are not firebrand and do not let it affect their relationships .

excon rated this answer Excellent or Above Average Answer
ETWolverine rated this answer Excellent or Above Average Answer
powderpuff rated this answer Excellent or Above Average Answer

Question/Answer
Yiddishkeit asked on 10/26/04 - 50 more volunteers needed...

Allawi blamed US and coalition forces for the recents deaths of the 50 Iraqi soldiers that were killed heading home after graduating from a US run training course. Personally I don't think it's fair that Allawi to have us shoulder complete blame for the unfortunate tragedy. However I can understand his frustration. Also top Iraqi officials are worried concerning the inside information infiltrated by individuals clearly part of the jihad. Btw on a sidenote between June and September 92 car bombs in Iraq, killing 569 people and wounding 1318. As we would say in the Marines one big clusterf*ck.



Bobby

tomder55 answered on 10/27/04:

Allawi was not our first choice ;nor was he a U.N. favorite;he was picked by the Interm Government in other words ;no puppet he .The executions were apparently an inside job. Terrorists have infiltrated some of the new Iraqi security apparatus . He would never admit that so the next best thing to do is for him to blame the coalition. In his shoes I would do the same. There are elections coming up and I assume he is a candidate for some position. It would not do for him to admit a weakness at this point .

Truth is that 50 recruits being transported should've either been armed or provided an escort. There seems to be negligence here ;but who is to blame?

He has to prove he is a strong leader ,AND to show the Iraqis he is not our puppet. Our side should understand where his comments are coming from and cut him some slack. If he is indeed an independent ruler of Iraq ;and I think he is ;then we can certainly expect to see more policy differences between us and him in the future.

As to the increase in violence before the U.S. and Iraqi elections ;it was predicted . We knew that would happen . The terrorists like Zarqwai told us so. ["Know your enemy and know yourself; in a hundred battles, you will never be defeated.("Sun Tzu:
The Art of War)]



It is our hope to accelerate the pace of work and that companies and battalions with expertise, experience, and endurance will be formed to await the zero hour when we will begin to appear in the open, gain control the land at night, and extend it into daylight, the One and Conquering God willing. We hope that this matter, I mean the zero hour, will [come] four months or so before the promised government is formed. As you can see, we are racing against time. If we are able, as we hope, to turn the tables on them and thwart their plan, this will be good. If the other [scenario] [happens] and we seek refuge in God and the government extends its control over the country, we will have to pack our bags and break camp for another land in which we can resume carrying the banner or in which God will choose us as martyrs for his sake.

ETWolverine rated this answer Excellent or Above Average Answer
excon rated this answer Excellent or Above Average Answer
Yiddishkeit rated this answer Excellent or Above Average Answer
Itsdb rated this answer Excellent or Above Average Answer

Question/Answer
Yiddishkeit asked on 10/26/04 - Any Red Sox fans?

You guys are looking good up 3-0. In my opinon this Red Sox club is amazing and with the Cards backs against the wall it should make for an interesting game four.



Bobby

tomder55 answered on 10/27/04:

I don't think that the Cards can duplicate the Red Sox comeback against the Yanks. oh well a good curse can't last forever. Dennis Miller yesterday said. The Red Sox nick name is Bosox ;which reminds him of Botox ;which reminds him of Kerry.

ETWolverine rated this answer Excellent or Above Average Answer
Yiddishkeit rated this answer Excellent or Above Average Answer

Question/Answer
Itsdb asked on 10/26/04 - What can we take seriously?

The Times is apparently devoting this week's headlines to boost the Kerry campaign. Yesterday, they led with the story of missing explosives in Iraq. Today they continue with the 'story', in which they quote Kerry as saying, "Now we know that our country and our troops are less safe because this president failed to do the basics...This is one of the great blunders of Iraq, one of the great blunders of this administration. The incredible incompetence of this president and his administration has put our troops at risk and put our country at greater risk than we ought to be."

The Times also quoted Edwards' attack on the President, "It is reckless and irresponsible to fail to protect and safeguard one of the largest weapons sites in the country. And by either ignoring these mistakes or being clueless about them, George Bush has failed. He has failed as our commander in chief; he has failed as president."

However, last night NBC reported the weapons were missing when the US arrived at Al Qaqaa, citing their embedded reporter that was with the troops when they arrived.

NBC News Said Explosives Were Gone When US Troops Arrived

By Susan Jones

CNSNews.com Morning Editor
October 26, 2004

(CNSNews.com) - NBC News reported Monday night that 380 tons of missing explosives were already gone when U.S. troops arrived at the Al-Qaqaa weapons installation in April 2003 - one day after Saddam's government was toppled.

NBC should know. It had a reporter embedded with the U.S. troops when they arrived at Al-Qaqaa in April 2003.

While the Kerry campaign blasted the Bush administration for "stunning incompetence" on Monday, many Bush supporters questioned the timing of Monday's New York Times report about the missing explosives -- coming as it did just eight days before the presidential election.

NBC News Correspondent Jim Miklaszewski suggested a political motive as well: In his report on the missing explosives Monday night, he quoted one official as saying, "Recent disagreements between the administration and the head of the International Atomic Energy Agency makes this announcement appear highly political."

According to the New York Times, the IAEA said it had warned the Bush administration about the need to secure the Al-Qaqaa facility both before and after the war.

In a follow-up report on Tuesday, the New York Times did not mention the fact that NBC had an embedded reporter on the scene when the missing explosives were discovered -- the day after Baghdad fell.

Tuesday's New York Times report -- entitled "Iraq Explosives Become Issue in Campaign" -- covers how the Bush administration "sought to explain the disappearance of 380 tons of high explosives in Iraq that American forces were supposed to secure."

Bush's aides, the Tuesday article said, "tried to explain why American forces had ignored warnings from the International Atomic Energy Agency about the vulnerability of the huge stockpile of high explosives, whose disappearance was first reported on Monday by CBS and The New York Times."

The New York Times report portrayed the Bush administration as being on the defensive -- trying to "minimize the importance of the loss" of the military explosives.

The report noted that President Bush "never mentioned the disappearance of the high explosives during a long campaign speech in Greeley, Colo., about battling terrorism."

"There are certainly some questions about when the explosives were missing," Kerry campaign adviser Howard Wolfson admitted on Fox & Friends early Tuesday morning. But the Kerry campaign is not expected to let the matter drop.

In a press release late Monday night, the Kerry campaign accused the Bush campaign of trying to cover up its "failure" to secure the explosives.

"Instead of distorting John Kerry's words, the Bush campaign is now falsely and deliberately twisting the reports of journalists. It is the latest pathetic excuse from an administration that never admits a mistake, no matter how disastrous," Kerry-Edwards senior advisor Joe Lockhart said."

The NY Times failed to mention there is another side to this story. The NY Times also today published a story portraying Bush as a hypocrite regarding his faith, and 2 of their 3 editorials are anti-Bush. Has the media ever worked so hard at electing one candidate while destroying another as they have in this campaign? Will Kerry's headline and distortion driven campaign succeed, or will it backfire?

Steve
P.S. Bobby, if you need more context follow the links.

tomder55 answered on 10/26/04:

I have been answering this question on the Christianity board for 2 days . Here are fragments of my reply ;most of which you have already covered in your post .You really have to admire Kerry's ability to seize both sides of every issue. First there are no threatening weapons ;now the weapons that did not exist are missing .First we rushed to war ;now we did not wage the war soon enough.

Miklaszewski was imbedded with the 101 Airborne when they arrived . He is an eye witness that the weapons were already gone.

Why is the U.N. nuclear agency suddenly warning NOW that insurgents in Iraq may have obtained missing explosives in early 2003?

How do looters carry-off 380 tons of weapons materials? In backpacks? It would about 40 truckloads to remove that much .

Isn't it more likely that it was hauled off by Sadamn just prior to the war?


Who is this Iraq official that wrote this confidential letter to the UN? And, if it was confidential, why was it released by the UN?Isn't it more likely that Mohammed el Baradai is orchestrating this because Bush opposes his re-appointment to the IAEA ?

This report is old news being dredged up by the main stream media in the service of the desperate Kerry campaign .

If your looking for blame ;blame our 'allies' in the U.N. whose delay tactics made it possible for Saddam to disperse his stockpile of weapons and to ship his WMD to Syria.Does anyone really belive that Saddam sat around for almost a year without making preparations ?


I think the Slimes will get egg on their face as bad as the Jayson Blair scandal ,but like Dan Rather they will survive to continue to preach to the faithful. The damage may be done nonetheless .This was an attempt to duplicate the last week revelation in 2000 that Bush was a drinker . That turned his comfortable lead into the nail biter the election turned out to be .

Itsdb rated this answer Excellent or Above Average Answer
ETWolverine rated this answer Excellent or Above Average Answer
purplewings rated this answer Excellent or Above Average Answer
sapphire630 rated this answer Excellent or Above Average Answer

Question/Answer
ETWolverine asked on 10/25/04 - Political dilema

Guys,

I've got a real problem, and it's weighing heavily on me. I don't know what to do about it, and I'm looking for your advice.

You all know my political leanings. I'm a huge Bush partisan, and very anti-Kerry. I have agreed with Bush on most of the issues, with only a few notable exceptions.

But I am faced with a dilema of having to turn away from the Bush name on an issue that is important to me.

You see, I enjoy Heinz Baked Beans more than Bush's Baked Beans. I can't seem to get past this, and I'm afraid I have to cross party lines on this issue. I know... its terrible, but I can't help myself.

Is there any hope for me?

Elliot

tomder55 answered on 10/25/04:

I like Bush beans better . They taste more like barbeque beans but as is true in most of the country we are split 50-50 . My daughter prefers the Heinz brand . My wife is undecided but usually will take the time to soak and prepare dried beans and cook them up . She will make them in a large pot and portion them in containers in the freezer .So although it takes alot of time to prepare .There are plenty of meals out of a batch.

There is a company in Tennesee ;Vietti Foods Inc ,that is marketting baked beans called :"Conservative Republican Texas Chili Beans" ;and "Liberal Democrat Boston Baked Beans. " The company is doing an informal poll of the election based on sales . Bush was leading as of Sept. 27 but there is still time to pick up a case if you prefer chili beans. Then you could wait until next Wed. and probably pick up the Boston Baked beans at a huge discounted close out price.

ETWolverine rated this answer Excellent or Above Average Answer
Itsdb rated this answer Excellent or Above Average Answer

Question/Answer
purplewings asked on 10/24/04 - About voter registration.

Do we have to register yearly or is our card good for as long as live at the same address?

I can't remember.
PW

tomder55 answered on 10/25/04:

I see from another post you live in Michigan :

From the Jackson County Michigan Clerk's office

Q: Do I have to register for each election?
A: NO. Registration is permanent as long as you continue to live in the city or township where you are registered.

purplewings rated this answer Excellent or Above Average Answer

Question/Answer
purplewings asked on 10/23/04 - A proposal from the state of Texas.



Future of Texas
Please note that Texas is the only state with a legal right to secede from the Union (please refer to the Texas-American Annexation Treaty of 1848). We Texans love y'all, but we'll have to take action if Kerry wins president over Bush. We'll miss you too.

Texas has given all those complainers plenty of time to get used to the results. After seeing the whiners along the campaign route, the folks from Texas are considering taking matters into our their hands.

Here is our solution:

#1: Let John Kerry become President of the United States. (all 49 states.)

#2: George W. Bush becomes the President of the Republic of Texas. So what does Texas have to do to survive as a Republic?


1. NASA is just south of Houston, Texas. (We will control the space industry.)

2. We refine over 85% of the gasoline in the United States.

3. Defense Industry. (We have over 65% of it.) The term "Don't mess with Texas" will take on a whole new meaning.

4. Oil - we can supply all the oil that the Republic of Texas will need for the next 300 years. Yankee states? Sorry about that.

5. Natural Gas - Again we have all we need and it's too bad about those northern states. John Kerry will figure a way to keep them warm....

6. Computer Industry - We currently lead the nation in producing computer chips and communications: Small places like Texas Instruments, Dell Computer, EDS, Raytheon, National Semiconductor, Motorola, Intel, AMD, Atmel, Applied Materials, Ball Semiconductor, Dallas Semiconductor, Delphi, Nortel, Alcatel, Etc, Etc. The list goes on and on.

7. Health Centers - We have the largest research centers for cancer research, the best burn centers and the top trauma units in the world and other large health planning centers.

8. We have enough colleges to keep us going: U.T., Texas A&M, Texas Tech, Rice, SMU, University of Houston, Baylor, UNT, Texas Women's University, etc. Ivy grows better in the south anyway.

9. We have a ready supply of workers. (Just open the border when we need some more.)

10. We have control of the paper industry, plastics, insurance, etc.

11. In case of a foreign invasion, we have the Texas National Guard and the Texas Air National Guard. We don't have an army, but since everybody down here has at least six rifles and a pile of ammo, we can raise an army in 24 hours if we need it. If the situation really gets bad, we can always call Department of Public Safety and ask them to send over a couple Texas Rangers.

12. We are totally self sufficient in beef, poultry, hogs and several types of grain, fruit and vegetables and let's not forget seafood from the gulf. And everybody down here knows how to cook them so that they taste good. Don't need any food.


This just names a few of the items that will keep the Republic of Texas in good shape. There isn't a thing out there that we need and don't have.

Now to the rest of the United States under President Kerry:

Since you won't have the refineries to get gas for your cars, only President Kerry will be able to drive around in his 9 mile per gallon SUV. The rest of the United States will have to walk or ride bikes.

You won't have any TV as the Space Center in Houston will cut off your communications. You won't have any natural gas to heat your homes, but since Mr. Kerry has predicted global warming, you will not need the gas.

Signed,

The People in Texas



tomder55 answered on 10/24/04:

unrelated ;supposeddly the Washington Times will run a front page story Monday about foreign policy that should put the nail in the Kerry campaign coffin (at least that is the buzz on the net)

powderpuff rated this answer Excellent or Above Average Answer
purplewings rated this answer Excellent or Above Average Answer
Yiddishkeit rated this answer Excellent or Above Average Answer

Question/Answer
sapphire630 asked on 10/20/04 - Would like words to

The Bush Kerry song that goes with the Dueling Banjos I think.

tomder55 answered on 10/21/04:

can't find em ;but if I do I'll sing em with you in a Kerry~okie duet .

Question/Answer
sapphire630 asked on 10/20/04 - I learn something new everyday!

I didn't know deer hunters 'crawled on their bellies'
I thought only Little Egypt's 7 Kids (song by The Coasters) and reptiles 'crawled on their bellies'....but Kerry is looking forward to yet another photo ...oops deer hunting 'crawling on his belly' in Mahony County, Ohio.

tomder55 answered on 10/21/04:

I heard that he will be hunting goose today . Around here that is not exactly showing your skills or prowess. The geese here are overpopulated and people love feeding them wonder bread .Which turns them into domestic instead of migratory birds.

I always said that if I was ever homeless I'd never go hungry ;just snare me a goose start up a trash can barbeque and hopefully someone would donate the ingredients for a good orange sauce.

Back to Kerry .He should stop trying to prove he is a gun supporter .He already lost the NRA endorsement. What is his PETA constituency going to say about him pluggin Bambi?

labman rated this answer Excellent or Above Average Answer
sapphire630 rated this answer Excellent or Above Average Answer

Question/Answer
Yiddishkeit asked on 10/20/04 - Baseball...

I watched history tonight!

Sorry Yankee fans. I know how it feels my Astros lost earlier in the evening in extra innings and I think it's going to be very difficult to beat the Cards in a game seven especially since they have home field advantage.


Anyway the Red Soxs looked amazing in their come back and I believe they have the momentum and will finally win the World Series. I know it pains NY fans to hear this, but I'm afraid that whomever wins between the Cards and Astros are going to have the frustration of the Red Sox (1918) taken out on them. Of course I'm just guessing and anything could happen...my fellow baseball fans how do y'all think it will play out?


Bobby

tomder55 answered on 10/21/04:

the curse is not over til it's over . I'm sure as a 'stros fan you remember 1986 very well(Mets winning every game not involving Mike Scott). Boston v.Mets;all they need is Buckner to field a lazy grounder to 1st and they win;but the ball went though his legs. The good part of last nights game is that I was able to turn it off early since the Yanks were out of it by the third inning. Kevin Brown had a 'deer in the headlights 'look to him from the beginning. Then Ortiz comes up and I'm screaming at the t.v."WALK HIM!!!" Bam!! could've turned the t.v. off then .

I give Boston all the credit . They could've laid down ;but didn't . They have got to get rid of the grundge look however .Too many of them look like they just finished dumpster diving.

I can't root for the 'stros to win until Clemens returns the Humvee he got from the Yanks as a retirement gift.

Heard Curt Shilling on an interview. They were asking him about a play when A-Rod tried to knock the ball out of his hand .They called it a 'Bush League 'play. He quickly corrected the interviewer ;"it was a 'Kerry League' play he said .

Yiddishkeit rated this answer Excellent or Above Average Answer

Question/Answer
excon asked on 10/19/04 - Messages for the children


Hello experts:

You know, whenever I hear that blankety blank sends the wrong message to our children, I know that they are about to take away a legitimate adult activity.

Should children be kept entirely in the dark about adult activities? Do you think that if you don't expose them to certain things that they won't learn about them?

Do you, like George Bush, not know whether homosexuality is a choice? Do you like the taste of sand with your head in the hole?

excon

tomder55 answered on 10/19/04:

Show me the scientific evidence. He at least is honest about not knowing . I would make some declaritive opinion that it is not a choice but a compulsion.Again ;I do not have facts to back that up either .Kerry was not asked to answer the question ,but there is no clear cut proof that someone is born a homosexual.

Raising my daughter I prefered to have her learn about "adult activities "on my time table. I know that was not always realistic however . I'm not sure where you are going with the question ;decency standards perhaps ?

excon rated this answer Excellent or Above Average Answer
labman rated this answer Excellent or Above Average Answer

Question/Answer
Yiddishkeit asked on 10/16/04 - More elections held in Iraq...

To begin Ramadan with a bang more elections were held this past Friday. Insurgents elected to kill five more US marines and blast away at a five churches in the Iraqi capital, Baghdad.

Christian "Republican" voters what's your view on the quagmire in Iraq?



Bobby


tomder55 answered on 10/17/04:

only Christian Republicans? ...I'll answer anyway.

Hopefully we will not make the same mistake in Ramadan as we did last year when we decided to be culturally sensitive. Some of the worse violence happened during their 'religious holiday'.U.S. casualties actually dropped because we were sending out fewer patrols ,but we let the insurgency get the initiative ;especially al-Sadr. We ended up with higher casualties later due to that decision.

The jihadists are trying to dublicate the Madrid scenario It worked in Spain ,but had no effect in Australia;and they were not able to prevent the Afghan elections. Zarqwai's playbook in the form of a letter was found and published in the press a few months ago. In it he wrote"

"There is no doubt that the space in which we can move has begun to shrink and that the grip around the throats of the mujahidin has begun to tighten. With the deployment of soldiers and police, the future has become frightening."

"The army and police have begun to deploy in those areas and are growing stronger day by day. They have put chiefs [drawn] from among Sunni agents and the people of the land in charge. In other words, this army and police may be linked to the inhabitants of this area by kinship, blood, and honor. In truth, this area is the base from which we set out and to which we return. When the Americans disappear from these areas and they have begun to do so and these agents, who are linked by destiny to the people of the land, take their place, what will our situation be?

If we fight them {and we must fight them}, we will confront one of two things. Either:

1 We fight them, and this is difficult because of the gap that will emerge between us and the people of the land. How can we fight their cousins and their sons and under what pretext after the Americans, who hold the reins of power from their rear bases, pull back? The real sons of this land will decide the matter through experience.
Democracy is coming, and there will be no excuse thereafter.

2 We pack our bags and search for another land, as is the sad, recurrent story in the arenas of jihad, because our enemy is growing stronger and his intelligence data are increasing day by day."
.............................................
As you see the situation is getting desperate for the jihadists and they have reacted by increasing the level of violence .They know time is not on their side . Once democracy comes to Iraq their 'justifcation' for incursion ceases .

Don't let the level of violence discourage you . The Summer of 1864 as Grant campaigned his way towards Richmond many thought the effort was hopeless .While casualties mounted ,and the war dragged on ,McClellan (a decorated veteran)was running on a peace platform (I have a plan)that was appealing to the war weary nation. The decisive victory at Atlanta by Sherman changed the equation and although a lenghty siege of Petersburg still needed to be completed victory was assured.

In WWII the last months of the conflict in both the European and Japanese theater were some of the bloodiest as the enemy became increasingly desperate. 50,000 casualties on Okinawa ;apx 81,000 during the Battle of the Bulge.People thought the wars would drag on for years ;but a couple of months later the combat ended.

Before any country can enjoy freedom, it must defeat the enemies of freedom. The enemies of freedom have suffered serious defeats in Afghanistan and Iraq, even though they are fighting to retain a hold in both countries. The solution is not to withdraw in disillusion, but to maintain the commitment.

I do not think it is a quagmire.What is a quagmire anyway? The dictionary defines it as a difficult situation .Ok ;if that is the definition then yes in a way it is . Is our troop commitment in the Balkans a quagmire?We have been there for a decade. How about Korean peninsula ;we have held a hostile front since the mid- 1950s?The question really is :is it the right thing to do .I think it is .

The rebellion of al-Sadr has been neutralized and Iraqi Kurdistan is essentially pacified. The terrorists in Falluja have already begun to alienate their base by their attempt to transform the city into a fundamentalist redoubt.Tensions between local residents and the foreign jihadists are beginning to boil over with the Washington Post reporting that some residents have taken up arms against them.Operations in Ramadi and last month is Sumara has shown a new paradigm where no longer will they permit mosques to be treated as sanctuary and bases of operation against the coalition. This week P.M Allawi gave an ultimatum to Falluja ;hand over Zarqwai or offensive operations would be renewed.As of this morning the battle for Fallujah has been rejoined. A spokesman for the Fallujah city delegation and the Falluja police commander were arrested for cooperating with the jihadists.

Elections will proceed ,and then we will maintain a presence their so the new Iraqi gvt. can build up the security forces that are playing an ever larger role in the defense of their country. Neither candidate will change that .


labman rated this answer Excellent or Above Average Answer
purplewings rated this answer Excellent or Above Average Answer
Yiddishkeit rated this answer Excellent or Above Average Answer

Question/Answer
Bishop_Chuck asked on 10/14/04 - Forgot my government 101

Ok, the early voting has started today.

Now, what happens, if one of the canidates would die
( not that I want any of them to)

Does the VP canidate automaticly become the Presidential canidate, or does the party have to re-nominate someone?

tomder55 answered on 10/15/04:

There is no Federal standard to deal with this nightmare scenario.There are a hodge-podge of State statutes but it is a troubling scenario. No ,the VP does not become the candidate.The problem is that election day is set in stone by the Constitution. Perhaps a new amendment is needed to address this.

If a winning candidate dies after the election but before the Electoral College meets, some state laws would apparently require electors to vote for him (with his running mate taking office in January); but Congress, might refuse to count these votes. After losing to Ulysses Grant in November 1872, presidential candidate Horace Greeley died, but some electors from states that he carried nevertheless voted for him; Congress refused to treat these votes as valid.


Another issue : Electors are not committed to vote as the State voted .They have been called 'faithless Electors' and rarely do they not vote as instructed to do.

"Since the founding of the Electoral College, there have been 156 faithless Electors. 71 of these votes were changed because the original candidate died before the day on which the Electoral College cast their votes. Three of the votes were not cast at all as three Electors chose to abstain from casting their Electoral vote for any candidate. The other 82 Electoral votes were changed on the personal initiative of the Elector.

Sometimes Electors change their votes in large groups, such as when 23 Virginia Electors acted together in 1836. Many times, however, these Electors stood alone in their decision."

In 2000 one elector;Barbara Lett-Simmons (Democrat, District of Columbia)abstained in protest because of lack of representation in Congress for the District of Columbia.

(http://www.fairvote.org/e_college/faithless.htm)

Bishop_Chuck rated this answer Excellent or Above Average Answer
ETWolverine rated this answer Excellent or Above Average Answer
labman rated this answer Excellent or Above Average Answer
sapphire630 rated this answer Excellent or Above Average Answer

Question/Answer
sapphire630 asked on 10/14/04 - Grand Finale

I could not believe it and never expected it.
At the end of the debates they asked about 'the women in their lives'. As Bush was reminiscing about how he meet Laura at a backyard Texan Bar-B-que; I was anticipating a reply from Kerry of some redeeming quality to Teresa's character and his love & affection for her. I was stunned when he made a joke about marrying UP and quickly changed the subject to his late, beloved mother.
I was listening on the radio, but heard that
Teresa was not at all happy sitting there listening to this.

minus some points for Kerry!

tomder55 answered on 10/14/04:

it was a Dukakis "take my wife.......please" moment
It wasn't even a good joke (I did not learn it was until that idiot Bob Schieffer started laughing hysterically .

It would've worked better if Kerry had said :

KERRY: Well, I guess the president and you and I are three examples of lucky people who married up.

(LAUGHTER)

And some would say maybe me moreso than others.

(LAUGHTER)

But I can take it.

(LAUGHTER)

I married up......then I got divorced and REALLY MARRIED UP !!!
(rimshot)

The station I was watching did not pan over to see her reaction . I can imagine her reaction . Kerry even managed to say a nice thing about Laura Bush but not Theresa :

And I'm blessed, as I think the president is blessed, as I said last time. I've watched him with the first lady, who I admire a great deal.......

Yup he's got some serious time comin to him in the dog house. But on a positive note ;how bad could the dog house of a multi-billionaire be ?

ETWolverine rated this answer Excellent or Above Average Answer
powderpuff rated this answer Excellent or Above Average Answer
purplewings rated this answer Excellent or Above Average Answer
sapphire630 rated this answer Excellent or Above Average Answer

Question/Answer
ETWolverine asked on 10/14/04 - An Article I saw...

Hello all,

I first saw this article in the Houston Chronicle on Friday Oct. 8, 2004. I found it reprinted here.

I will first print the article here.

Then I'll rip it apart.

Your comments are appreciated.

-------------

Israel Wins the Debates
More of the Same
By TARIF ABBOUSHI

John Kerry and John Edwards used their debates with George W. Bush and Dick Cheney to tell America that if the incumbents are re-elected, as far as Iraq goes we can expect 'More of the same.' Those four words exemplify the essence of the Democrats' argument for regime change in the U.S., a Kerry-Edwards mantra, if you will, that defines the Bush administration's plan for dealing with the debacle they got us into in Iraq. But there's another mantra that bodes equally ill for our efforts to win the peace in Iraq, one that is used by Democrats and Republicans alike to encapsulate their approach to dealing with the mother of all issues in the region: 'Israel has a right to defend itself.'

When John Edwards was pointedly asked to explain his party's plan for dealing with the Israel-Palestine conflict, he ignored the question to wax indelible about Israel's right to self-defense. Offered his opportunity to articulate the incumbency's position, Cheney could only agree with his opponent. It would be closer to the truth for both parties to acknowledge that their plans for Israel-Palestine are one and the same: whatever Israeli prime minister Ariel Sharon's foot-soldiers inside the beltway tell them it's going to be. At its heart, more of the same twisted logic that ascribes the right to self-defense to the occupier, but not the occupied.

A point of contention during the vice-presidential debate was John Edwards' barb that the Bush administration has outsourced the capture of Usama Bin Laden to Afghan warlords. But there is no arguing that it has outsourced the Israel-Palestine conflict resolution to Israel. After repeatedly asserting that he would do whatever it takes to ensure that Israelis and Palestinians comply with his peace plan, the much-vaunted but ill-fated 'Road Map', President Bush has shown he was only talking the talk. When it came time to walk, he toed Sharon's line--all the way around the illegal Israeli colonies on the West Bank.

Our leaders on both sides of the aisle trip over themselves to trumpet another of Sharon's hypnotic mantras: 'Israel has no partner for peace.' The truth is Sharon can find no Palestinian who will accept Israel's definition of peace: a Palestinian entity the contours of which are defined by Israeli settlements, the borders, airspace and aquifers of which are controlled by Israel, and the disjointed non-contiguity of which more resembles Apartheid South Africa's reviled Bantustans than any viable state in the world today. For all its despicable corruption and ineptitude, the Palestinian Authority would ink a peace deal tomorrow if Israel would withdraw from the territories it occupied in 1967, but that is a step Israel has never countenanced--not in Oslo, not at Camp David, not even as a theoretical response to the cessation of Palestinian violence. It is truly the Palestinians who have no partner for peace.

While it is refreshing to hear the Democratic team of Kerry and Edwards join Republican Senators John McCain, Chuck Hagel and Richard Luger in articulating self-evident truths about the quagmire in Iraq (and in doing so highlight the Bush administration's dastardly delusional denial) the truth is neither party has a hope of turning things around for America in the Middle East without first confronting the Israeli-Palestinian conflict in a manner that puts America's interests ahead of Israel's. Focusing on Iraq as the core issue of this election misses the point, for we cannot win the region's hearts and minds through a war conceived and championed by avowed pro-Israel zealots like Paul Wolfowitz, Douglas Feith, David Wurmser and Richard Perle. The folly of their approach continues to unfold before us on a daily basis. In the bigger picture, it is perhaps best captured by President Bush's pre-war claim that the peace train to Jerusalem goes through Baghdad. How Jerusalemites must be praying to their sacred heavens that Bush's train never reaches their holy city.

During his debate with John Edwards, Dick Cheney cited Saddam Hussein's support for Abu Nidal and for the Palestinian suicide bombers as evidence of the deposed Iraqi dictator's links with terror. But Abu Nidal and the suicide bombers are enemies of Israel, not of the U.S. It is the Bush administration's neoconservatives that have succeeded in defining our enemies as Israel's enemies, and not, as is painfully clear, to our benefit. Unfortunately, both the Democratic challengers and the Republican incumbents define their future approach to Israel--and, unwittingly, its consequences for our nation--with the same dreaded four words: More of the same.

Tarif Abboushi lives in Houston. He can be reached at: TAbboushi@aol.com

-----------

Let's see... where to begin....

I know... let's start with some of the basic assumptions made by Mr. Abboushi.

1) That Palestinian terrorism (o terrorism by any other group of terrorists) can ever be justified.
2) That Israel (or any other country) should be forced to negotiate while under the gun of terrorism.

These two assumptions go hand in hand. Without them, Abboushi doesn't have a leg to stand on.

So let's be clear about this: Terrorism is NEVER justified. Deliberate attacks against non-combatants are unjustifiable without exeption. Furthermore, no country should negotiate with terrorists while under the influence of terrorism. To do so is a failure of the government to protect the security of the nation it governs. It doesn't matter whether we are talking about Israel or Africa. The rules and the responsibilities are the same.

3) Israel does not have the right to defend itself.

This assumption is made clear in the fourth paragraph when he lambasts Edwards for saying that Israel DOES have the right to defend itself, and lambasts Cheney for agreeing. Clearly Mr. Abboushi does NOT believe that Israel has the right to defend itself.

In this belief, Mr. Abboushi denies Israel the right of every soveriegn government in the world... the right to secure and safe borders, and the right to defend against attack from its enemies. Not only does Israel (and every other country) have the right to defend itself... it has the RESPONSIBILITY to do so. Anything less is a shirking of that responsibility to its citizens and a monumental failure of the government.

4) That the USA is under duress from Israel, and that Israel is forcing the USA to take actions that are counter to its own best interests.

Mr. Abboushi makes it very clear that he believes that Paul Wolfowitz, Gouglas Feith, David Wurmser and Richard Perle and other prominent no-cons are puppets of Ariel Sharon and are influencing the US government in directions that are not good for the USA.

First of all, if Israel has any influence over the USA, it is as an ally and a peace partner, and a partner in the war on terrorism. It is NOT as a master over a puppet state. It never has been, and it never will be.

Second, supporting Israel in its battle against terrorism is good for America for quite a few reasons.

a) Israel is the USA's only completely reliable and safe port of call in the entire region.
b) Israel is a partner in the war on terror. Anything that supports the elimination of terrorists is good for America and Israel.
c) Israel is an economic ally as well. Anything that can bring stability in the region and eliminate the terrorist threat is good for American business.

Plus, let us not forget that the "support" that Bush has offered Israel until now is simply a hands off policy rather than any affirmative support of any kind. That is the best type of support to give Israel... just stay out of their way, and let them do what they need to do. So the idea that Bush is "supporting" Israel against the Palestinian terrorists is ludicrous. What Mr. Abboushi REALLY means is that the Bush government has refused to oppose Israel, and that is what makes him so angry.

Then there are the factual problems with Mr. Abboushi's article.

I - Mr. Abboushi claims that "The truth is Sharon can find no Palestinian who will accept Israel's definition of peace: a Palestinian entity the contours of which are defined by Israeli settlements, the borders, airspace and aquifers of which are controlled by Israel, and the disjointed non-contiguity of which more resembles Apartheid South Africa's reviled Bantustans than any viable state in the world today."

Clearly, he has not seen the Sharon plan if he can say that. While it is true that some settelements will remain, most of the smaller ones are being disbanded... especially where it effects the congruity of Palestinian territories. Furthermore, the plan will open up Palestinain borders and airspace, and clarify Palestinian water rights. To say otherwise is a clear lie.

II - Then there is the claim that "For all its despicable corruption and ineptitude, the Palestinian Authority would ink a peace deal tomorrow if Israel would withdraw from the territories it occupied in 1967, but that is a step Israel has never countenanced--not in Oslo, not at Camp David, not even as a theoretical response to the cessation of Palestinian violence. It is truly the Palestinians who have no partner for peace. "

Actually, in September 2000, Ehud Barak offered Yassir Arafat 98% of the territories represented by the 1967 borders. The last 2% was withheld for security reasons, but other lands were offered in lieu of that 2%.

Arafat rejected the offer and pulled out of the Camp David talks. He didn't make a counter offer. He didn't debate the issue. He just rejected it and pulled out, claiming that Israel was being unreasonable in its negotiations.

Now I don't know about you, but if someone offers me 98% of what I'm looking for at the negotiating table in any business deal, I jump at it. I certainly DON'T try to pull out of the deal. Yet that is what Arafat did.

So for Mr. Abboushi to claim that Palestinians would "jump at the chance" to take a deal that brought them back to the 1967 borders, but that Israel has "never countenanced" such a deal is another clear lie. Even Clinton, who was at the time an unabashed Arafat supporter, said that Arafat pulling out of the negotiations after being offered the moon was insane and completely unreasonable. Sorry, but the facts are NOT what Abboushi claims they are.

Your comments are appreciated.

Elliot

tomder55 answered on 10/14/04:

The Palestinian Charter and The Hamas Covenant both state clearly that Israel's existance is illegal.Both organizations went into alliance in 1998 But ...didn't the Palestinians revoke the Charter in 1998? yeah right !Perhaps they didn't need the charter anymore since the covenant provides them with their slogan: "Allah is its goal, The Prophet its model, the Qur'an its Charter, jihad its path, and death for the cause of Allah its most sublime belief" (Article 8)If Arafat truley wanted peace with Israel he would not have walked away from Camp David . Why did he ? Because he would've had to waive the rediculous 'Right of Return'."There is no solution for the Palestinian question except through jihad. All initiatives, proposals, and international conferences are a waste of time and vain endeavors" (Article 13).

Hitler understood that doctrine needed to be preserved in text if it is meant to be binding . The 'Palestinian Charter' and 'The Hamas Covenant' serves as worthy successors to 'The Palestinian Carter' which in turn was a worthy succesor to 'Mein Kampf'.

ETWolverine rated this answer Excellent or Above Average Answer

Question/Answer
Itsdb asked on 10/13/04 - Unbelievable

Ok all you leftists out there, please justify this. At Friendship Missionary Baptist Church in Miami over the weekend, besides his gay marriage diversions, Kerry told his audience "Never again will a million African-Americans be denied their right to exercise their vote in the United States of America."

Come again? Will everyone who was denied their right to vote in 2000 please stand up?

Media Fund has an ad out which states "Republicans want you to sit out this election and simply stay home..."Bush is a rich man in the White House who is sending black men and women to be slaughtered in Iraq while Cheney and Halliburton boys get rich on oil. Don't keep getting played."

Anyone know the casualty figures for Blacks in Iraq? Do liberals have no shame???

Steve


tomder55 answered on 10/14/04:

there are always voting irregularities ,always will be .when they are identified usually they atre addressed. I saw no evidence of disenfranchisement based on race. In fact the only charge I have seen that merits disenfranchisement is when the absentee ballots of American troops were challenged.

ETWolverine rated this answer Excellent or Above Average Answer
Itsdb rated this answer Excellent or Above Average Answer
Yiddishkeit rated this answer Excellent or Above Average Answer

Question/Answer
Yiddishkeit asked on 10/13/04 - A complete sweep as Bush lost again in the debates...

I think most Bush supporters are loyal short of an impeachment or death. Are there any republicans with a ounce of objectivity, scratch that, how about just any of our board republicans wavering in your decision to vote for Mr. Bush?

Bobby

tomder55 answered on 10/14/04:

I am not Republican or Democrat ,I am not registered with any party ,but I work for candidates from either party at the local level depending on the campaign season .This year all my candidates are running unopposed so my time has been freed up . So although I have been a avid Bush supporter (that wasn't always the case)I feel that I approach this campaign with objectivity.

The domestic debate was supposed to be played in Kerry's court .It was not supposed to be Bush's strength. But I think he at least came away with a draw in this round.

Kerry has good debating skill no doubt about it . To a fault he backs up his positions with a litany of "facts" .But on most of the issues ,on sincerity and connecting to the people I think Bush did a much better job. As I said on another posting ;Bush speaks and the question is do you agree with his position and vision or not. When Kerry speaks more often than not ;after he adds his qualifiers and nuances the question frequently becomes :What exactly did he say;where does he really stand ??

In a nut shell his position was .Bush does it wrong ;I can do it better .Any problem I can solve by making the government responsible for it . To pay for it ;I'll sock it to 2 %of the population.bush did an excellent job of making Kerry's voting record the focus of the domestic debate. He could not adequately respond to charge because high taxes and higher government spending is at the core of Kerry's philosophy .He be better if he acknowleged that and gave a good rational to his reasoning .

Bush had trouble answering the economic questions . This is where his record makes him vulnerable. The skirted the jobs question a couple of times. Although I think Kerry's ideas on health care would be a disaster ,Bush did not offer any option that will solve the issues of health care . In my view a combination of goverment subsidies on essential items like immunizations and other preventitive measures; combined with choice of private health care ;along with addressing the liability issues that Kerry dismisses but really are a big deal is the path the country should be heading in .Bush did a little better in at leat answering Social Security's future . Kerry has no idea on what to do .

Bush was clearly better in addressing social and security issues.

On security ;Bush finally got around to calling Kerry out on not voting for the Desert Storm resolution even though it passed every measure of Kerry's international test .("Apparently you can't pass any test under his vision of the world." )

Kerry makes a big deal about container inspections ,but the reality is that 100% inspection would kill the economy .(I do not know what a reasonable % would be required to be perfectly safe) My industry relies on imports and we already feel the effects of delays due to inspections . Neither candidate addresses the border security adequately . So one has to wonder how either can claim to be concerned about security at home. I agree with Bush that when possible it is better to take the fight 'over there'.

Kerry was clearly uncomfortable in answering questions that morality comes into play.He flip-floped from the debate a week ago addressing abortion ,when he said that he would not have a litmus test on judicial apointments .Last night he said that he he won't appoint a judge to the Court who's going to "undermine a constitutional right." .Bush's position whether you agree or not on the 'right to abortion'was that we all can agree that we should be working to reduce the total # of abortions in the country .He listed ways to promote life.

When asked about faith Bush said he prays alot but made it clear that God wants humans to be free. Kerry treaded lightly on the issue saying that we need to follow the principles of love God and love your neighbor. Call that a draw.

Kerry ;who is supposed to be solid with the women's vote made a big blunder when they were both asked about their wives .First he implied that he had "married up",which brings the whole issue of his divorce'the controversy over his 'annulment',and the issue of his marrying for opporunistic reasons.When Bush spoke of Laura you could see the love. By contrast Kerry barely mentioned Theresa.He failed to speak highly of her at all[in fact he spoke more highly of Laura Bush than Theresa].Not one heartwarming ancedote at all . He focused on his mom and later briefly about his daugters. This was a Dukakis moment that may or not get exploited in the final weeks .

He also repeated Edwards mistake of bringing Cheney's daughter's homosexuality into the debate. Can you imagine the outrage that the homosexual lobby would've made if Bush had done something simular ?

All told I think Kerry was on the defensive for most of the debate .bush seemed more relaxed ;optimistic,and confident. Bush wins a close debate.

ETWolverine rated this answer Excellent or Above Average Answer
Itsdb rated this answer Excellent or Above Average Answer
labman rated this answer Excellent or Above Average Answer
powderpuff rated this answer Excellent or Above Average Answer
purplewings rated this answer Excellent or Above Average Answer
sapphire630 rated this answer Excellent or Above Average Answer
Yiddishkeit rated this answer Excellent or Above Average Answer

Question/Answer
excon asked on 10/12/04 - Uhhh, dumb question!



Hello fiscal conservatives, I know you must be out there:

Uhhh, uhhh, we have a deficit, huh? A big one, huh? So, why are the Republicans giving business another 1.3 billion dollar tax break? Is this la la land? Did I wake up to the Mad Hatter being in charge?

excon

tomder55 answered on 10/12/04:

the bill that was loaded with pork was signed overwhelmingly by as bi-parisan Senate 69-17 . It is a gross example of how special interests dominate both parties politics . Good ideas like providing disaster relief for Fla. and helping U.S. manufactures deal with foreign tarrifs are mixed in with crap spending .

according to Chicago Sun-Times :

"The final 633-page product pared taxes for interests ranging from major manufacturers to native Alaskan whalers and ethanol producers. Other winners included fishing tackle box makers, NASCAR track owners, Chinese ceiling fan importers, and foreigners winning bets at U.S. horse and dog racing tracks.

"http://www.suntimes.com/output/news/cst-nws-cong12.html

Other provisions would benefit oil companies, timber producers, movie studios, accounting firms, cruise ship operators, tobacco farmers and dozens of other groups.

Republican John McCain "It is the worst example of the influence of special-interest groups I have ever seen,"

Bush has said that he is concerned that it is overloaded with special-interest provisions but will reluctantly sign it anyway.How would it look if he veteoed a disater relief bill ? Until there is a reform of the system that includes executive line item vetoe then this stuff will continue to happen . Fiscal conservatism in such a system is a fantasy.


http://www.theledger.com/apps/pbcs.dll/article?AID=/20041012/ZNYT01/410120437/1001/BUSINESS

excon rated this answer Excellent or Above Average Answer
kindj rated this answer Excellent or Above Average Answer
ETWolverine rated this answer Excellent or Above Average Answer

Question/Answer
Itsdb asked on 10/09/04 - Voting rights, voting wrongs

UNION-TRIBUNE
October 1, 2004

"We intend to enforce the fundamental constitutional right of every American to vote to ensure that the Constitution's promise is realized and that, in disputed elections, every vote is counted fully and fairly."

John Kerry
Democratic presidential nominee


"In 2004, Democrats will win the White House back the old-fashioned way by counting every vote."

John Edwards
Democratic vice presidential nominee

"Every American must be able to exercise his or her basic, non-negotiable right to vote. This year, Americans deserve an error-free, intimidation-free, voter-disenfranchisement-free, chad-free, butterfly-free election."

Terry McAuliffe
Democratic National Committee chairman

The party of Kerry and Edwards and McAuliffe continues to insinuate that the last presidential election was stolen. Democrats continue to suggest that black and senior voters were disenfranchised.

They continue to promote the fiction that, if each and every vote had been counted four years ago, George W. Bush would not be sitting in the Oval Office.

So this time around, Democrats say, they are going to dispatch observers and lawyers to polling places throughout the country "to monitor elections and enforce the law." In this presidential election, they declare, every ballot cast will be counted.

Well, it's nice to know that Mssrs. Kerry and Edwards and McAuliffe have dedicated themselves this year to protecting the voting rights of poor little minorities, like yours truly, as well as seniors.

But I, for one, am not worried about my voting rights being abrogated. I'm far more concerned that the ballot I cast on Election Day will be neutralized by voting wrongs.

For example, a coalition of groups supporting Kerry's candidacy has set up a Web site that urges supporters of independent presidential candidate Ralph Nader to vote for Kerry in battleground states in exchange for votes by Kerry supporters for Nader in states where the outcome is not expected to be close.

Groups engaged in this unlawful vote-swapping claimed to have "helped 36,000 voters exchange pledges" in 2000. Those votes could have, conceivably, changed the election outcome in Iowa, New Mexico, Oregon and Wisconsin states that Bush lost that by fewer than 17,000 votes total.

Then there's the recent revelation that some 46,000 New York City residents are unlawfully registered to vote not only in the Empire State but also in Florida. Of those registered in both states, nearly 70 percent are Democrats.

Nearly 1,700 of those registered in both states requested that absentee ballots be sent to their home in the other state. It appears that many, if not most, voted twice, making the much-contested 2000 Florida presidential election closer than it otherwise would have been.

A recent article in the Journal of the American Medical Association identifies yet another area of voting abuse the large numbers of Americans suffering from Alzheimer's disease and other forms of dementia who are allowed to cast ballots, no matter how mentally incapacitated.

Workers for a party or a candidate who show up at a nursing home to "assist" with voting can commit "wholesale fraud," said Stanford University law professor Pamela S. Karlan, an author of the article.

That is borne out by two other studies, one in Pennsylvania, the other in Rhode Island, which found that patients in dementia clinics actually turned out in higher numbers in proportion to the general population than did voters overall in the last presidential election.

And Democrats continue to troll for votes in nursing homes and other long-term-care facilities. For example, in Santa Clara County, Kerry campaign operatives have sued the local Veteran's Affairs medical center for preventing them from signing up dementia patients to vote.

Finally, there's the issue of noncitizen voting. The 11-year-old National Voter Registration Act declared that state driver's license applications "shall serve as an application for voter registration with respect to elections for federal office."

The so-called "Motor Voter Law" has made it all too easy for noncitizens to add their names to voter rolls in states that do not require proof of U.S. citizenship as a condition of obtaining a driver's permit.

Indeed, in his just published book, "Stealing Elections," a former colleague of mine, Wall Street Journal columnist John Fund, notes that at least eight of the 19 hijackers who carried out the Sept. 11, 2001, terror attacks were actually able to register to vote in either Virginia or Florida.

Yet, whenever the suggestion is made that, in a nation where some 32 million residents are foreign-born (where as many as one-quarter of the foreign-born are here illegally), people ought to provide proof of citizenship when they register to vote, Democrats cry "intimidation."

Whenever the proposal is made that people who show up at polls to cast their votes ought to present some form of photo identification, the Democrats shout "racism."

The party of Kerry and Edwards and McAuliffe hasn't had nearly as much to say about voting wrongs as it has about voting rights.

Maybe that's because the Democrats know they stand to benefit most from vote-swapping, double voting, registration of dementia patients, and noncitizen voting."

*************************************************

Defeat George Bush
Support the critical voice of third parties
Build a progressive majority

You can make your vote for president count by joining the national VotePair campaign and using your vote to elect Kerry-Edwards while supporting the role of progressive third-party candidates. As we learned in 2000, a few hundred votes in the right states can make all the difference in the world--and the whole world is watching now.

In vote-pairing, swing-state progressives whose first instinct might have been to vote for Nader, Cobb or Badnarik are paired with Democrats (and others whose first choice for President is Kerry) in 'safe' states where either Bush or Kerry has a decisive lead. Paired voters can communicate with each other and decide to vote strategically: swing-state participants for Kerry and safe-state participants for Nader, Cobb or Badnarik. As a result, the paired voters' support for progressive third parties is recorded in the popular vote and their preference for Kerry over Bush finds voice in the Electoral College.

Defeating Bush means redirecting the current Administration's aggressive unilateral foreign policy, protecting the environment and women's right to choose, and keeping Bush from stacking the Supreme Court and federal courts with right-wing judges like Justices Scalia and Thomas.

Together, we can vote strategically to build a progressive majority and prevent another disastrous Bush presidency.

Isn't that special?

tomder55 answered on 10/10/04:

Florida is in the news again . The Dems. are setting up voter fraud as an issue before the elections begin.There is a flurry of law suits in the works . One demands that touch screen voting machines produce a hard copy.

Sec. of State Glenda Hood will soon be as recognizable a name as Katherine Harris. In one case she has made controversial rules about provisional ballots which are used when elections officials cannot find a voter's name on voting rolls. Hood had told county supervisors that provisional ballots cast by voters in the wrong precincts cannot be counted. The Florida Democratic Party challenged that in a lawsuit Thursday.

In yet another case 67 elections supervisors that they should reject incomplete voter registration forms.
Hood's office told the counties they should disqualify voters who failed to check a box confirming they are U.S. citizens.Imagine that ; preventing non-citizens from voting!!

That is just a few examples .There is another one involving the NAACP and another where registration forms have bogus addresses, including some that match a public park, a parking lot and a Jacksonville utilities building.

That is just in Florida .Other States are reporting problems also.The lawyers will
have a busy time next month . Around the country simular reports are coming in .Registration fraud is being reported in Pennsylvania in Ohio in Minnesota and many other locals. Democrats are taking Teresa Heinz Kerry's advice to"Vote often and vote well."seriously .



This amid reports that election related violence is on the increase.A a gunman took a few shots into the Knoxville, TN Bush-Cheney campaign headquarters. The AFL-CIO stormed Republican campaign headquarters in Orlando and Miami. Adding insult to injury, they also filed criminal charges against one of the people they attacked.Vandals burned a swastika into the lawn of a Madison, WI man who displayed a Bush-Cheney 2004 sign. From later reports, it appears more than one home was vandalized in this fashion."It was not an act of hate," said Dustin "Dusty" Dzuck, 17, a senior at Denfeld High School."My mom called me a terrorist. It wasn't terrorism; it was activism. It was for a cause.... The whole thing is, basically, I just wanted to get the word out there that in my opinion Bush isn't doing this country any good."And they say the Afghanistan elections are in turmoil;they say you can't hold elections amid violence in Iraq. We are living proof that it can be done.







Yiddishkeit rated this answer Excellent or Above Average Answer
Itsdb rated this answer Excellent or Above Average Answer

Question/Answer
Yiddishkeit asked on 10/09/04 - Debate again...

Ok I'm not going to hammer home a point on the debates. I thought Bush finally articulated a few issues better than in his first debacle. Sen. Kerry had another wonderful night. Personally I would hate to have to debate Kerry, but this post is not about being republican or democrat, however it is about strategy.

I'm concerned about the small numbers we have in he service abroad...also to mention the comparably small numbers in help we recieve from our allied forces. Pres Bush has to know that our numbers are inadequate and John Kerry must address this if he becomes President, at least in the short term. As many know my brother is full-time Army now has gone back into Iraq again. My brother is an engineer and doesn't have a need to understand all the intricacy of being a grunt (infantry). But for the last few months that's exactly what they now have been training him for..infantry. I'm really getting tired of Bush and I wish he would either sh*t or get off the pot. This has become a quagmire folks, like it or not.



Bobby

tomder55 answered on 10/09/04:

What was a quagmire was keeping a large force of troops bottled up in Saudia Arabia for over a decade .What was a quagmire was flying hundreds and thousands of sortees to protect the Kurds and Shia.What was a quagmire was enforcing a sanctions regime that our allies were actively working to undermine.

Both candidates have essentially the same game plan . 'Iraqification' and withdrawal. Kerry speaks of a timeline. That is unrealistic .The situation on the ground will dictate the timeline .It is so much rubbish for him to keep on harping this 'win the peace' rhetoric. What does that mean? You fight a war to win a war;then there is peace.

Honestly ,the tone of the political opposition at home has slowed the process in Iraq in my view. Lets say that the President had a free unfettered hand ,and did not have to constantly look over his shoulder at domestic reaction . Perhaps then he would let the dogs out and finally crush the 'insurection'. I think he will do so after the elections .for that I am critical . I do not think that domestic politics should stop progress but I understand why he would .

When Iraq is finished as the primary front in the war then there will be other hot spots that Americans will be deployed in .You and your brother should realize that reality.I agree that troop levels should be increased. I was one of those who was concerned about the #s pre-war.But I do not agree that there should be a draft.

There is one aspect of that that Kerry fails to mention when he speaks of the issue.According to the plan ,the 4th ID was supposed to enter Iraq from Turkey and create a northern front to the battle. Negotiations ;trying to get the world community on board ;all that rubbish that delayed the battle;is what put pressure on the Turks to not allow the deployment.So one full infantry division was left floating on boats off Turkey .They remained there primarily as a decoy to prevent Iraqi northern Armies from reinforcing it's southern flank.

The 4th ID eventually was shipped almost 1/2 way around the world and had to enter Iraq through crowded ports in the Persian Gulf.They missed the major combat.

The net result of this decision was that most of the Sunni triangle was spared combat .They were not made to realize they were defeated. The first aim of war is to defeat your enemy and let them know they are defeated.

Bush said that he has undertaken a reorganization of the existing American forces. In Europe we have too many troops defending beer halls in nations that have no need to be defended and their political decisions have suggested that they do not care to remain in a joint defense arrangement. When this war is over there will be a big draw down of troops from the theater unless Syria and Iraq become hot spots. Otherwise perhaps 50,000 troops in the region would be more than adequate to defend our interests there . But your brother is a specialist and unfortunately specialists are also required in hot spots.

voiceguy2000 rated this answer Excellent or Above Average Answer
excon rated this answer Excellent or Above Average Answer
Yiddishkeit rated this answer Excellent or Above Average Answer

Question/Answer
purplewings asked on 10/08/04 - If John Kerry should become President of the US,

What do you think, or just hope - he will do about the Iraqi situation, and about the Islamic terrorist who are so focused on the United States?

PW

tomder55 answered on 10/09/04:

The 'war on terrorism'(the enemy with no name) will be fought for years by a number of administrations of both parties. It is my sincere hope that no matter who is in the White House they will recognize the seriousness of the endoever and what is at stake.

My concern is that the population itself is falling into some romantic notion about the 'good old days' of the 90s when terrorism is something that happened over there. That America could sit back ,trust that the intelligence and police agencies would prevent the worse while the people remain happy-go lucky and not burdened by the task. Kerry it appears trys to tap into that sentiment which he masks in tough guy rhetoric .But I could be wrong. If elected ;it will be in everyones interest to hope he succeeds .

purplewings rated this answer Excellent or Above Average Answer

Question/Answer
hafj1 asked on 10/08/04 - political theory

sorry i meant 'is it possible to have a political theory which is NOT at the same time ideological?'
thanks holly

tomder55 answered on 10/08/04:

populist movements tend to be non-ideological

Question/Answer
excon asked on 10/08/04 - Finally!!!


Hello Experts:

I changed my mind. Bush is going down - big. I don't know how he squandered his lead, but he did. Maybe it's because he's a liar and a dufus???? Nahh.

excon

tomder55 answered on 10/08/04:

it certainly is possible that he will be defeated ,but neither candidate will win or go down in a big way. I'm suprised that the October suprise turned out to be a report that says the WMD were not there at the time of the invasion (could they be in Syria....Nahh ),and since Bush had control of the date of release he could've opted for the safer route of not releasing the report until after the elections . I know that the MSM has ignored some of the report that clearly demonstrates in my view that Saddam needed to be removed ;and they have only glossed over the fact that the nations that Kerry things should be the arbiters of our policies were neck deep in theft of money that was targetted for Iraqi children .It is way beyond a stretch for anyone to now say that Bush lied about the WMD.

CeeBee2 rated this answer Excellent or Above Average Answer
excon rated this answer Excellent or Above Average Answer
labman rated this answer Excellent or Above Average Answer

Question/Answer
voiceguy2000 asked on 10/06/04 - This Really Puts the Issue in Sharp Focus

I was struck by this essay I ran across today:

But how do we deter people who want to die? How do we deter people who need only the skill and the means to push a button on a briefcase, or open a box cutter and be prepared to do bloody work with it? How do we deter the assassin lost in the crowd at the Superbowl? How do we deter enemies who are so dispersed, so ethereal and fragmentary, that hostile governments can arm and shelter them knowing full well that we will not retaliate with a nuclear attack against millions of genuine innocents in Cairo, or Tehran, or Riyadh?

If a suitcase nuke detonates in Times Square, or Long Beach harbor, or outside the Capitol building, what do we do? Nuke Mecca? Incinerate Damascus? Because so help me God, I tremble to say it that is exactly the response our enemies would hope for. They care not a whit about their own people because they have no allegiance to anyone but themselves and their vision of a vengeful and bloodthirsty Allah. A million, ten million innocents under American mushroom clouds are just that many more martyrs gone to paradise. It is they, not we, who dream of a clash of civilizations, with its promised sweeping away of the decadent and godless by the blood and faith of the Believer.

We might yet be able to stop this on the cheap. If we do not, I fear the day will come when 3000 civilians and 1000 American soldiers will look like a very, very small bill to pay.

What we learned on 9/11 is that there are people out there who are not deterrable. Given the chance given the weapons these people will strike without any regard to consequences. The ultimate horror of a world enveloped in nuclear fire is just peachy keen with them if it will bring about the New Caliphate. We love death the way you Americans love life, they say. They are not kidding. They are serious. You can pretend otherwise, but that will not make it change. There are people who are determined to kill us for who we are and what we believe. They can not be deterred.
The entire essay appears here.

He adds:
President Bush warned that this was going to be a different war something unlike anything we had ever seen. The front line now, at this critical time, is in the hearts and minds of our own people. Thats where the real battle is now. That is our weakest point, our breach, our point of failure. We have not made the case to enough people and time is running out.
This really captures the thoughts I have had over the last year. It seemed to me that in the summer of 2003 we had a lot of momentum that was causing countries like Pakistan and later Libya to change their approaches. Had this country stood firm in its resolve, I think George Bush would have had a lot more clout to encourage other significant change. But he had the rug cut out from under him by State Department and CIA types who disagreed with his policies, and then the Presidential campaign produced demagogues like Howard Dean who sought to win votes by tearing down everything this country was trying to accomplish.

Are we, as a country, in a perpetual state of denial?

tomder55 answered on 10/07/04:

It is easy to live with blinders on ;to think that the world is indeed a good as we'd like it to be . If 9-11 did not bring such illusions back to reality then perhaps those folks who claim that we need to get 'hit again' to wake us up have a valid point.Whittle write in the essay :

I used to be a carrot man. Like most larval liberals, I grew up in a life that would be unrecognizable to all but the thinnest sliver of humans that ever lived on this great rock in space that thin, thin sliver being everyone and everything you and I know and take for granted.

Reality meaning the wolves have never been so far from the door as they are today. So believing in the power of goodwill and friendship, of handshakes and agreement and compromise, of trusting to the good and noble in mankind was easy for me, for the consequences of being wrong in that belief cost me nothing at all. Id never been robbed, raped, beaten or victimized in any way. That belief in goodwill, compromise, concession and trust grew as a result of being surrounded by decent people in a well-ordered, lawful society, with a long history of compromise and cooperation.


All you need to do is to look at Darfur to see what the intentions of our enemies are .
What incentives do you need to offer them to abandon their rigid ideology of hate? What global test do you need to pass to defend yourself against them ?

The Duelfer Report is clear that Saddam maintained his capability to produce WMD both CW and BW ,and that he experimented his research on humans .When we found out that the Nazis had conducted simular atrocities the question asked was 'how was that permitted to happen ? 'Indeed. We are watching holocausts occure daily in the parts of the world where Isamo-fascism is on the march . Offer them a carrot and they will ask for your blood instead .Bill Whittle is right . Deterence only works if the threat is plausable ;and it only works if your enemy is rational enough to be detered.

labman rated this answer Excellent or Above Average Answer
purplewings rated this answer Excellent or Above Average Answer
voiceguy2000 rated this answer Excellent or Above Average Answer

Question/Answer
sapphire630 asked on 10/05/04 - 911 hype

Dick Morris' reply to Mike Moore's Farenheit 911

tomder55 answered on 10/06/04:

'Celsius 41.11 -- The Temperature at Which the Brain Begins to Die '

sapphire630 rated this answer Excellent or Above Average Answer

Question/Answer
Yiddishkeit asked on 10/05/04 - The VP debate...

Without all the hype of the Presidential debates the VP candidates re-hashed the war issues, mixed in a little economy and health care issues. I was happy to see that Cheney (according to the VP himself) finally made the aqaintance of Sen. Edwards for the first time. But what impressed me most was Cheney's (the human Mt. St. Helen) control to not use explicits after being grilled about Haliburton...though it was evident that his blood pressure was rising as his eye-brows went just above the third wrinkle on his forehead. Edwards trying to stir up an academy award speech about his father watching tv to learn math...I suspect will not be scripted for a Hollywood production anytime soon. Anyway in this debate both candidates appeared to serve their parties well and the analysts saw it as even. The Historians remarked that the only thing unusual about this VP debate was that it touched more on war. My personal opinion, for what it's worth, is that Cheney looks exhausted without any fresh direction. Edwards surprised me with his support for Israel to defend herself. I also commend Edwards for his compassion in defending Cheney's homosexual daughter, that left Cheney speechless.



Bobby

tomder55 answered on 10/06/04:

Cheney sorta blew that line about meeting Edwards for the first time. He was introduced to Edwards during Libby Dole's swearing in the Senate ,and he also thanked Edwards in February 2001, at a Senate prayer breakfast.

Edwards also distorted the whole Halliburton thing .The bulk of Halliburton's contracts were awarded in competitive bids. There are some no-bid contracts they have ;specifically in the repair of the Iraqi oil infrastructure;something that they are uniquely competent at. No bid contracts are not unusual. The Clinton Adm. frequently awarded Halliburton simular bids ;especially in Kosovo. Halliburton employees frequently take on high risk tasks. It was Kellogg, Brown & Root( a subsidiary of Halliburton) employees that were hanging from the bridge in Fallujah .I do not think Cheney answered the charges that Edwards made good enough but it may be because ongoing investigations prohibitted him from addressing some of the charges.

I thought Cheney did a good job answering some of the charges that Kerry made last week ,and Edwards restated. Cheney also did something else that Bush failed to do. He highlighted the Senate record of both Kerry and Edwards.

Cheney had a difficult time explaining his difference with Bush policy relating to gay marriage ,and to Edwards credit he did not hammer home that contradiction . But Edwards was also dead wrong when he stated that no state has to recognize the marriages of another State. That shows a complete lack of understanding the Constitution.It also is deceptive because he knows damn well that if one State refused to recognize a marriage from another State the Federal Courts would be on it faster than an athiest on a Nativity Scene displayed in San Francisco.

For the record ,both are wrong. The reason why it is not a State issue is because of the 'full faith and credit clause ' ;Section One of Article four of the United States Constitution .

Edwards also repeated the Kerry distortion on the stem cell issue . The Adm. has NOT banned stem cell research . All they did was stop Fed. Funding on NEW embryonic stem cells . Stem cells already made can be used. Private research can still proceed ,and the gvt. still funds research on adult stem cells. There is no evidence that embryonic stem cells are better than adult stem cells ,and there is no evidence that the research will make any impact on diseases like Parkinson or Alzheimers.

Cheney was best when he pointed out the change in position of both Edwards and Kerry about Iraq when they felt pressure from the Dean candidacy .Edwards was best in driving home the job situation in the country . Something that Cheny countered with recent stats,but otherwise he had a hard time defending the Bush record .


One thing I do not understand. Edwards is in his 50s . That means when he was a kid it was in the early 1950s . He said his father would watch TV to learn math . What show was he watching ? Sesame Street was not around yet.

CeeBee2 rated this answer Excellent or Above Average Answer
purplewings rated this answer Excellent or Above Average Answer
Yiddishkeit rated this answer Excellent or Above Average Answer

Question/Answer
sapphire630 asked on 10/05/04 - Scary Terry Kerry at it again

Now she came back to her Foxchapel home in Pittsburgh to visit nearby Sharpsburg after our flood from Ivan and she said, "I wish I were in a position to help"
I guess she spent all her millions on the Boston bash and couldn't even help even with a case of ketchup and a keg of Iron (Pittsburghs beer). I guess she doesn't own a pair of work pants or work boots either.
Help me with excuses for her, please!

tomder55 answered on 10/06/04:

let them go naked !

sapphire630 rated this answer Excellent or Above Average Answer

Question/Answer
Itsdb asked on 10/05/04 - Are you insane?

John Edwards: "I'd say If you live in the United States of America and you vote for George Bush, you've lost your mind." (ABCs "Nightline," 10/4/04)

Who will join me at the funny farm?

tomder55 answered on 10/05/04:

If I wasn't already completly loony I will be after tonight. Just for fun ;count how many times 'Halliburton 'gets said by Mr Smiles. Then if that doesn't make you hurl you will certainly gag when he tells his life story and how his father was a working man and he was the first person in his family to go to college .

Lets see the moderator challenge him on this tonight. When Bush called Clintoon and Gore a bunch of Bozos the press was all over him . Here we have a talking hair-doo calling at least 1/2 the country crazy .

Itsdb rated this answer Excellent or Above Average Answer
labman rated this answer Excellent or Above Average Answer
purplewings rated this answer Excellent or Above Average Answer
sapphire630 rated this answer Excellent or Above Average Answer

Question/Answer
denberg asked on 10/02/04 - Question on Bush's draft dodging from a Brit

Ok experts here's a good one for you all - and I want plenty of answers on this!

I live in the UK and I don't have much knowledge of US politics. However I do know that Bush Jr is accused of draft dodging and of using "influence" to avoid going to Vietnam in 1968. Now what I want to know is this:

How exactly could his father have "arranged" for the military to place his son in a national guard unit?

Bush Sr didn't run the army, navy or air force. They didn't work for him. And they didn't work for any of the senators or politicians whose names crop up in these discussions either. So even allowing for the fact that someone is incredibly rich, how do you explain it?

Perhaps I'm showing my naivete here, but I've never heard of anyone from my own country being able to arrange appointments in the army, navy or air force. Correction. It was generally know that Prince Charles would be given a ship to command when he joined up. But I have never even heard of anyone else even being ACCUSED of using influence to secure a military appointment in this country since...oh, I don't know, World War One. And I've never heard of any rich man's son being able to avoid conscription during national service days. So...how could it have been done?

This isn't a "partisan" question. I just want to know how it could have been done (if it was). What happens...does Sr pick up the phone and say, Hey there, I want my boy in the Guard? Or what?

British MPs can't arrange things like this for their constituents, so how can US Senators?

Come on experts,

Enlighten me

Paul ( I even passed an exam called Advanced Level Government and Political Studies in 1980, aged 18)!

London UK

tomder55 answered on 10/03/04:

The draft in America has always been a contentious issue. Even when our nation teetered on the brink of collapse in the Civil War there were draft riots in N.Y. and other places.It is best that it not be contemplated again unless some real emergency erupts.Not that I'm opposed to some kind of national service being mandatory with military service being an option.But even the military will tell you today that they in no way favor the introduction of a draft for military conscription.

The issue of Bush's service record has been raised and beaten to death here on the politics board.The definitive account of his record was posted by Voiceguy2000.,Ceebee,and myself in the last couple of weeks .This artice by Byron York details his service record .

The notable thing about the Air National Guard during the Vietnam era was that unlike many of the other Guard branches there was a higher probability that a unit of the Air Guard would be called to active duty.Flying a jet is a specialized skill.This article by By Retired Chief Warrant Officer 2 John W. Listman, Jr. details some of their service .
As with the mobilized Army Guardsmen who served during the Vietnam War, the members of the Air Guard who served on active duty, many of whom spent nearly a year in tours overseas, have received very little lasting recognition for their service. At least seven died in combat while performing their duty.

Their missions were often difficult and dangerous but always carried out with the professional determination for which the Air Guard is known. Above all, it was a job well done, and largely forgotten.


Bush did not serve in a combat role in Vietnam ,but it is an insult to other guard members to say that going into the guard is a dodge.The facts of his service and honorable discharge dispell any notion that he dodged responsibilities or that he was a 'fortunate son'.



denberg rated this answer Excellent or Above Average Answer

Question/Answer
Bishop_Chuck asked on 10/02/04 - Vice Presidents

Ok, we all have views on the election. But have many of us, stopped to consider what the leadership would be like if either party was elected and shortly afterwards the President ( which ever one) would die ( not that we would want either one to).

Many Americans can't even tell you who the two men who could be president even is. Would we want either of these men ( the vice presidents) running our country.

tomder55 answered on 10/02/04:

yes ;Cheney has a long history of public service . I would have no problem with him running the country .

Edwards on the other hand is an ambulance chaser.The trial lawyers who finance the Democrats would be thrilled to have him in the Oval Office. Talk about a puppet.

Bishop_Chuck rated this answer Excellent or Above Average Answer

Question/Answer
Itsdb asked on 10/01/04 - A theocracy?

Pardon the diversion from the debates, but in discussing his new book, 'Cruel and Unusual: Bush/Cheney's New World Order' (which I have not read), Mark Crispin Miller states concerning Bush, "the regime's goal is to abort democracy," and impose a theocracy on America...and the world, which "means replacing the Constitution with the Pentateuch."

"Mark Crispin Miller: I wrote Cruel and Unusual to make the case that Bush & Co. is fundamentally un-American -- an order wholly alien to the spirit of our founding documents. Certainly the regime represents some dark old strains in U.S. history: nativism, white supremacism, theocratic tyranny. But as far as our mainstream political traditions are concerned, Bush & Co. have simply junked them. They've hijacked the U.S. ship of state, and have it on a suicidal course.

I argue that Bush & Co. is the anti-Jefferson. This regime is not conservative, but represents a radical subversive movement -- one now largely in control of all three branches of the government, and also dominant throughout the press. What ultimately drives them is irrational. Sure, they're in it for the money and the oil; but that's not all that's going on here. They're neo-Calvinists, quite clearly working toward the imposition of theocracy on the United States, and then on the whole world. (Although mostly atheists and Jews, the Straussian types around Rumsfeld and Cheney are fine with that agenda, as they believe that theocratic government is best, because it makes the populace compliant.)"

------------------------------------------------------

Anyone out there believe Miller is right? Why or why not?

Steve

tomder55 answered on 10/01/04:

this is obviously too absurd to be taken seriously . if didn't know better I'd have thought he was talking about the Islamo-fascists .But Miller is a classic Bush hater .Bush seems to be his obsession. This is not his first book on Bush . He also wrote a hack job called : 'The Bush Dyslexicon'.Get this;he actually thinks that there is a pro-Bush bias in the press . In the introduction to the book he writes :
"The Dyslexicon attempts to give the lie to that enormous wave of propaganda -- a joint production of the GOP and the major media -- whereby George W. Bush was forced on us as President, then, after his inauguration, hailed nearly universally for his amazing charm, his democratic ease, his rare ability to be all things to all Americans, and so on. Our experience of this transparent coup has been disorienting from the start."


These Bush bashers should get their stories straight. This clown thinks that the Bush adm. is some cabal of Christian Reconstructionists and others think he is influenced by a cabal of 'Jewish neo-cons'.

Itsdb rated this answer Excellent or Above Average Answer
Ccl471 rated this answer Excellent or Above Average Answer

Question/Answer
Yiddishkeit asked on 09/30/04 - The first debate is historically the most watched...

I'd never seen Sen. Kerry debate before tonight and I was a bit surpised by how well he did. He controlled the room and for those who viewed the split screen shots of both candidates, witnesssed Pres. Bush losing his composure while Sen. Kerry was focused and steady.

Most of the spin doctor experts from both the Democratic and Republican camps agree that Sen. Kerry faired better. Personally I don't know if it will be enough to give Sen. Kerry an edge in this election, but I do believe the race is tightening. When the issues over economy come up in the next debate let's see how that plays out.


Bobby

tomder55 answered on 10/01/04:

ok ,I'll take the bait. I have not seen a worse debate performance except when Admiral James Stockdale in 1992 said "Who am I? Why am I here?". Bush remained focused on a couple of talking points that should've taken about 15 minutes of the debate and hammered them over and over . Kerry was the one who appeared Presidential and on top of the issue.

Not that Kerry did not make mistakes; it was just that Bush did not challenge him on them well. It was Kerry who brought up both Vietnam and more important his conduct in the protest and that went unchallenged . His 20 years of opposing needed weapon systems went unchallenged ;not even when he gave Bush a wide open shot at him by saying that amazingly he was opposed to the development of bunker busters ! I couldn't believe it!

He spewed a litany of distorted charges against Bush and Bush did not defend himself on them .But the worst thing was that Bush had a difficult time articulating why the conduct of the war against the jihadists was important to the security at home.

There were some positives for Bush . He did a good job when Kerry blundered by asserting that he thought we needed to pass a global test to act preemptivly . He also was correct in countering the silly notion of having bilateral negotiations with N.Korea. What is Kerry thinking ? On one hand he says we are acting unilaterally ,and then he criticises bilateral action for the N.Korean issue.

There is speculation out there that Bush's decision to visit with hurricane victims before the debate was both physically and emotionally tiring for the President . That may be ;but the point is that foreign policy is supposed to be the Presidents strength,and by last nights performance it appeared that Kerry was the one on top of the issue. I agree with Bush on the substance but Kerry was the one who was able to articulate his position.

Yiddishkeit rated this answer Excellent or Above Average Answer

Question/Answer
labman asked on 09/30/04 - Rather/CBS poll

Are these people for real? A USA Today/CNN/Gallup poll in my local paper, 56% say the story was an honest mistake, 64% wouldn't fire Rather, the majority trust CBS News, and overwhelmingly believe the report was not part of any political agenda. Surely USA Today/CNN/Gallup wouldn't have managed to conduct a biased poll. This is the most upsetting poll I have seen since the one saying the majority of Americans admire Bill Clinton for his ability to lie out of things.

tomder55 answered on 09/30/04:

it doesn't suprise me . Rather and company probably feel vindicated in their decision to dig in /hunker down and weather this storm . notice how the mainstream media has taken to attacking the 'pajama wearing'hacks (bloggers).

CBS has not drawn any lessons from this either . They have recently been in the forefront and elbow deep in the spreading the lie that Bush is planning to institue the draft((based on hoax emails and an interview with another dubious source( the testimony of a woman who is a chapter president of 'People Against the Draft' without identifying her as an advocate)

Karas: The truth of the e-mails were absolutely irrelevant to the piece, because all the story said was that people were worried. Its a story about human beings that are afraid of the draft. We did not say that this (e-mail) was true, its just circulating. We are not verifying the e-mail.



The last time I checked the Senate bill Senate Bill 89,and the House bill HR 163,were both introduced by Democrats (Fritz Hollings and Charles Rangal respectivly )

labman rated this answer Excellent or Above Average Answer
sapphire630 rated this answer Excellent or Above Average Answer

Question/Answer
sapphire630 asked on 09/28/04 - ...uneducated? (stupid me)

Scary Terry Kerry says that Pittsburgh women are uneducated. She likes to go on and on about how she as a woman stands up for her right to be heard. I guess she lived in Pittsburgh less years than I have that she is intellectual enough to have her say and I having lived here for the most part of my 50 years makes me uneducated and should shut up.
If she is so educated and I am so uneducated I guess I should count my blessing for being stuck in stupid!

tomder55 answered on 09/29/04:

(sung to :'How do you solve a problem like Maria')
How do you solve a problem like Teresa?
How do you shut her mouth and shout her down?
How do you solve a problem like Teresa?
A flibberty-gibbit, a shrew, a nag, a clown!

Many a time you know he'd try to kiss her
For politics she ought to understand
But how can he force her face
From wincing all over the place
He can't even get the wench to hold his hand!

Oh, how do you solve a problem like Teresa?
? How do you hold some ketchup in your hand?

When I'm with her I'm confused
Out of focus and bemused
And I hardly know exactly how she stands

Unpredictable as weather
She's as looney as a lefter
She's a Dem! She's a demon! She's a sham!

She'd outpester any pest
Drive a hornet from its nest
She could throw a whirling dervish out of whirl
She is senile! She is wild!
She's a riddle! She's a child!
She's a headache! She's a nightmare!
She's just weird!




sapphire630 rated this answer Excellent or Above Average Answer

Question/Answer
excon asked on 09/28/04 - Hep me - hep me; I'm conflicted


Hello all you people who have it all figured out, because I sure as hell don't:

George Bush will go down in history as the worst president (by far), that we've ever had. He's gotten us into some really big trouble that I'm not sure we're going to recover from.

But, he (along with his Christian right followers), are staunch supporters of Israel.

On the other hand, irrespective of whatever Kerry happens to believe at the moment, his party supports the Palestinians.

I'm a Jew and a democrat (no - not Democrat - democrat), and a defender of the only democracy in the Middle East.

Should I punt?

excon

tomder55 answered on 09/28/04:

I still haven't convinced you that Bush compares favorably to Harry Truman yet ?

excon rated this answer Excellent or Above Average Answer

Question/Answer
voiceguy2000 asked on 09/26/04 - Another Jimmy Carter?

I hate to sound like such a broken record, but I still think that Kerry sounds like another Jimmy Carter in the making. From todays New York Times:

For 15 minutes in Milwaukee theother day, Senator John Kerry pummeled his staff with questions about an attack on President Bush, planned for later that morning, that accused the White House of hiding a huge Medicare premium increase.

Talking into a speakerphone in his hotel suite, sitting at a table scattered with the morning newspapers, Mr. Kerry instructed aides in Washington to track down the information he said he needed before he could appear on camera. What could have slowed down the premium increase? How much of it was caused by the addition of a prescription drug benefit? What would the increase cost the average Medicare recipient?

Mr. Kerry got the answers after aides said they spent the morning on the telephone and the Internet, but few of those facts found their way into his blistering attack.

The morning Medicare call was typical of the way Mr. Kerry, a four-term senator with comparatively little management experience, has run his campaign. And, his associates say, it offered a glimpse of an executive style he would almost surely bring to the White House.

Mr. Kerry is a meticulous, deliberative decision maker, always demanding more information, calling around for advice, reading another document - acting, in short, as if he were still the Massachusetts prosecutor boning up for a case.

. . . .

In interviews, associates repeatedly described Mr. Kerry as uncommonly bright, informed and curious.

But the downside to his deliberative executive style, they said, is a campaign that has often moved slowly against a swift opponent, and a candidate who has struggled to synthesize the information he sweeps up into a clear, concise case against Mr. Bush.

Even his aides concede that Mr. Kerry can be slow in taking action, bogged down in the very details he is so intent on collecting, as suggested by the fact that he never even used the Medicare information he sent his staff chasing.

. . . .

Unlike Mr. Bush, who was a governor and a business executive before he ran for president, Mr. Kerry - who has spent the past 20 years as a legislator, with a staff of perhaps 60 - has little experience in managing any kind of large operation. Several Democrats suggested that this presidential campaign was in many ways a learning experience for him.
Link to source.

The article also speaks of Kerrys penchant for soliciting opinions on anything from an ever-widening circle of people, commenting that he tends to go with the view he heard last.

Obviously there are many actual differences between Kerry and Carter, but in this regard they cure sound similar to me. Am I missing something?

tomder55 answered on 09/27/04:

I agree that the plodding styles of both Carter and to a degree Clinton did not serve them well. Carter from a grand world stategy had Zbigniew Brzezinski to advise him so at least for the time of his Presidency he had a clear view of the U.S. role in the world. Kerry it seems to me would be more comfortable sitting in that great debate society ;The House of Lords .

voiceguy2000 rated this answer Excellent or Above Average Answer

Question/Answer
ROLCAM asked on 09/26/04 - Time for a change in strategy on Iraq ??

The situation in Iraq seems to be getting worse by the day. Violence is on the increase; law and order has not been established; many parts of the country are 'no go' areas for both the coalition forces and the interim government's security forces; and terrorist attacks are becoming more sophisticated and better co-ordinated.

What is the solution to this problem?

ROLCAM.

tomder55 answered on 09/27/04:

There is no doubt that mistakes were made in the execution of the war. I have been openly critical of certain aspects ,as a perusal of my postings and responses will confirm .I do not concure that the missteps are as Kerry cynically calls them ""colossal failures of judgment" .That stinks of demagoguery to me.

It can be argued that some of our greatest military victories were full of bad decisions and folly that have been disected not only by historians ,but also from contemporary critics.

Lincoln won the Civil War but not without numerous setbacks. There were out right defeats twice at
Manassas ;Fredricksburg;Chancellorville;and Chickamauga(that resulted in the Army being surrounded and seiged at Chattanoga for months).There were victories that were not followed up at Antietam and Gettysburg (with decisive action after these battles Lee's army could've been destroyed).The press and the political opposition called Lincoln every invective they could think of. If not for the fall of Atlanta in Sept. 1864 'Lincoln might very well have lost his reelection bid ,and his opponent (General McClellan) would've most likely ended the war prematurely and we would be looking at two separate nations here today.

WWII was also full of setbacks .Roosevelt for one certainly had much more warning of an impending attack on the U.S. by the Japanese than Bush had of 9-11. Our troops fought early battles in Tunisia; and Guadalcanal where we suffered horrendous casualties. McArthur(actually Gen.Wainwright as McArthur had already withdrawn) lost to surrender a whole army at Corregidor.There were failed assaults ,and we almost faced disaster during the desperate counter offensive by the Germans during the 'Battle of the Bulge' . Roosevelt did a poor job at preparing a post war period (at least by the standards of today's critics). Much of Eastern Europe was left to the occupation of a despotic Soviet regime. We failed to account for a post war Korea and Southeast Asia .

So much for historical content. So what to do in Iraq? I think in regard to the 'no go zones'it is again time to go in. It probably will not happen until after the U.S. elections but the U.S. recent bombing runs at Fallujah suggests that the enemy is being softened up for an assault. The Israeli example at Jenin shows that if terrorists are denied sanctuary then they have no base to stage their operations .In 2002, 228 Israelis died in 42 suicide bombings ;This year there have been 10 suicide bombings and 53 Israeli deaths. A dramatic decrease. Palestinian deaths have declined also so it can be argued that agressive action in Jenin saved Palestinian lives . Certainly Israeli response to suicide bombers families(bulldozing of homes) has had an impact also . Families are begining to turn in their sons who are planning to cash in on the 72 virgin plan.

Iraqi's are being trained to take over much of the fighting as this Washington Post article points out.

The highlights :

.Approximately 164,000 Iraqi police and soldiers (of which about 100,000 are trained and equipped) and an additional 74,000 facility protection forces are performing a wide variety of security missions.

Training is on track and increasing in capacity.

Iraqi security forces are in the fight and have suffered substantial casualties as they take on more and more of the burden to achieve security in their country. Since Jan. 1 more than 700 Iraqi security force members have been killed, and hundreds of Iraqis seeking to volunteer for the police and military have been killed as well.

Six battalions of the Iraqi regular army and the Iraqi Intervention Force are now conducting operations.

Two of the regular army battalions, along with the Iraqi commando battalion, the counterterrorist force, two Iraqi National Guard battalions and thousands of policemen recently contributed to successful operations in Najaf.

In the next 60 days, six more regular army and six additional Intervention Force battalions will become operational. Nine more regular army battalions will complete training in January, in time to help with security missions during the Iraqi elections.



I am planning to watch developments in Iraq ,and wait to see how the elctions are run . Until then I have seen no alternative course suggested that is palatable.Iraqi P.M. Allawi who was shamefully mocked by John Kerry spoke elegently to the world last week . "Allawi's talk was exalting. He spoke of a world divided between the forces of hope and the forces of fear. He laid out a sharp dichotomy between a government that expresses the aspirations of a free people and one that engorges itself on the power extorted by thuggery. He spoke about a people who only now are finding the voice of constructive citizenship. And he offered thanks to the heart and muscle of America, who had dreamed for his people before they had even dared." (http://www.spectator.org/dsp_article.asp?art_id=7160)

excon rated this answer Excellent or Above Average Answer
ROLCAM rated this answer Excellent or Above Average Answer
SanchoPanza rated this answer Excellent or Above Average Answer
voiceguy2000 rated this answer Excellent or Above Average Answer

Question/Answer
PLUTUS1947 asked on 09/26/04 - The Illuminati

The Illuminati, fact or fiction?

Your comments please. (Comments from Blair, Bush, Healey, Kerry or any other Illuminati Member welcomed (if it exists)).

tomder55 answered on 09/26/04:

good thing we have watch dog groups like "The Pilgrims of Saint Michael" to warn us of this immenent global take over. But you are a little confused. The group of men above are not 'Illuminatis' .They are in fact members of the 'Tilateral Commission'(or The Bilderberg Group I forget which) . Also 2 of them are rumored to be members of 'Skull and Bones'.

PLUTUS1947 rated this answer Above Average Answer
Saladin rated this answer Excellent or Above Average Answer

Question/Answer
SanchoPanza asked on 09/25/04 - Are the knives out for Blair?

Next week brings the Labour Party Conference, not the democratic, policy making, forum that it used to be but more of an American style showcase convention, since Blair became the Labour Party Leader. Yet there are factions among the old school who want to debate and vote on policy and who want to review the events of the past two years.
Thursday is scheduled for an inquest into the events in Iraq and tonight there where a couple of interesting programs on the BBC (remember we have only 5 regular TV channels here). The first programs looked at the relationship between Blair and the media personalities that he surrounded himself with during his first election campaign and how they view his Premiership, illustrating a disillusionment with his style of leadership and his increasing tendency to follow his own path rather than any party policy. The second program focused on the relationship between Blair and the man everyone sees as his natural successor Chancellor Gordon Brown. This was followed by a documentary about the Zulu Wars which showed how Victorian spin turned the events at Roarke's Drift into a heroic encounter, to divert attention away from the disaster the same day at Ishandawala.
The narration to this documentary seemed to have a contemporary undertone. It spoke of a meaningless war which the British public couldn't understand how we had become involved in and in which 10,000 Zulus were killed for no purpose and how it ultimately ended the Premiership of Disraeli in a resounding electoral defeat. Once I picked up the thread of the commentary the parallels between then and now were glaringly obvious. The message seems to be that Blair may just have taken too much on his own back. This will be the last conference before he has to call an election and he would expect to receive either the full backing of the three factions of the party (Trade Unions Local Labour Groups and the Parliamentary Members) or a challenge to his leadership if he fails to do so.
Coming so soon before the US election, a post mortum on Iraq could be telling.

tomder55 answered on 09/26/04:

I'm sure the Bigley kidnapping is weighing heavily on Blair's mind and conscious. His term has been of a man who has been willing to make the tough choices and to make things happen . Now he must appear to be powerless to influence the events . If the coalition caves and releases Dr.Germ

I am sure that Blair would like to make the conference a referendum on his domestic agendas but the events in the war are taking center stage.Again ;like in Spain ; the terrorists are calling the shots .It is up th the Labor party ,and the British people to show the resolve that they have become known for .

SanchoPanza rated this answer Excellent or Above Average Answer

Question/Answer
Itsdb asked on 09/24/04 - U.S. Air Cleanest Ever Since 1970

2003 Status Report Shows U.S. Air Cleanest Ever Since 1970

Cynthia Bergman 202-564-9828 / bergman.cynthia@epa.gov

(Washington, D.C.-September 22, 2004) Total emissions of the six principal pollutants identified in the Clean Air Act dropped again in 2003, signaling that America's air is the cleanest ever in three decades, the U.S. Environmental Protection Agency (EPA) reported today. Annual emissions statistics for the six pollutants are considered major indicators of the quality of the nation's air because of their importance for human health and the existence of their long-standing national standards.

Emissions have continued to decrease even as our economy has increased more than 150 percent. Since 1970, the aggregate total emissions for the six pollutants [Carbon Monoxide (CO), Nitrogen Oxides (NOx), Sulfur Dioxide (SO2), Particulate Matter (PM), Volatile Organic Compounds (VOCs) and Lead (Pb)] have been cut from 301.5 million tons per year to 147.8 million tons per year, a decrease of 51 percent. Total 2003 emissions were down 12 million tons since 2000, a 7.8 percent reduction. (See summary table at: http://www.epa.gov/airtrends/econ-emissions.html ).

"Thanks to this progress, today's air is the cleanest most Americans have ever breathed," said Administrator Mike Leavitt. "Now, EPA is taking up the challenge to accelerate the pace of that progress into the future."

The Agency recently issued regulations that will cut diesel pollution by 90 percent, and later this year will finalize regulations cutting power plant pollution by approximately 70 percent.

A major reason for the nation's progress is the innovative, market-based acid rain cap-and-trade program enacted in 1990. The Acid Rain Progress Report, also released today, shows annual SO2 and NOx emissions have declined 5.1 million tons (32 percent) and 2.5 million tons (37 percent), respectively, since 1990. The program generated double-digit cuts at its inception and is now maturing, with small fluctuations up and down as emissions gradually near their respective end goal caps.

"Cleaning the air gets more difficult as the maximum benefits from existing rules are achieved and the low-hanging fruit is gone," said Leavitt. "The sharp cuts of the early years of the Acid Rain Program are behind us now, and it's time to take the next step to protect people's health the next step is the Clean Air Interstate Rule."

The Bush Administration's Clean Air Interstate Rule (CAIR) stands to be the acid rain program of this decade, enabling the country to once again enjoy sharp cuts in harmful pollutant levels. It will use the same proven cap-and-trade approach as the Acid Rain program, creating financial incentives for electricity generators to look for new and low-cost ways to reduce emissions early.

CAIR will use cap-and-trade to address power plant emissions in 29 eastern states plus the District of Columbia. The program would cut SO2 by more than 40 percent from today's levels by 2010, and 70 percent when fully implemented. NOx emissions would be cut by 50 percent from today's levels by 2010, and 60 percent when fully implemented. The Administration plans to finalize CAIR this fall.

"The Acid Rain Program is a national success story because we achieved early reductions, cost-effectively and with near-perfect compliance," said Leavitt. "CAIR will provide similar benefits, ensuring that our nation's air continues to get cleaner well into the next decade."

John Kerry on the environment:

Protect Our Health By Reducing Dangerous Air Emissions

"As president, John Kerry will reverse the Bush-Cheney rollbacks to our Clean Air Act, plug loopholes in the law, take aggressive action to stop acid rain, and use innovative, job-creating programs to reduce mercury emissions and other emissions that contribute to global warming."

Does this guy always run on implenting policies that are already in place...and working?

Steve


tomder55 answered on 09/24/04:

I guess it depends who you are going to believe . Here is a report today from my local paper The Journal News

It says that ...Air quality in Westchester County and the New York City area ranks among the worst in the nation and is deteriorating,.This coming in a year in which the amt. of rainfall should have naturally cleaned the air.

Earlier this month ;Ben Lieberman of the New York Post wrote that new regulations to be phased in will make emission standards on vehicles like SUV's and trucks more stingent. Air quality should then improve with the normal turnover from old to new vehicles.

I think it is important to continuously look for ways to eliminate as much as possible particulate emissions into the air. I do not subscribe to any set way of doing it ,but more times than not I think industry falls into bad habits when there are not strong regulations to back up initiatives like the Bush Clear Skies Initiative.

Itsdb rated this answer Excellent or Above Average Answer

Question/Answer
CeeBee asked on 09/24/04 - (to be fair) Bush Resume, Part 2

(c/p from the same Voy chatboard)

Education: Yale University, B.A. 1968;
Harvard University, M.B.A. _________________

Bush's National Guard years

What do you really know about George W. Bush's time in the Air National Guard? That he didn't show up for duty in Alabama? That he missed a physical? That his daddy got him in?

News coverage of the president's years in the Guard has tended to focus on one brief portion of that time - to the exclusion of virtually everything else. So just for the record, here, in full, is what Bush did:

The future president joined the Guard in May 1968. Almost immediately, he began an extended period of training. Six weeks of basic training. Fifty-three weeks of flight training. Twenty-one weeks of fighter-interceptor training.

That was 80 weeks to begin with, and there were other training periods thrown in as well. It was full-time work. By the time it was over, Bush had served nearly two years.

Not two years of weekends. Two years.

After training, Bush kept flying, racking up hundreds of hours in F-102 jets. As he did, he accumulated points toward his National Guard service requirements. At the time, guardsmen were required to accumulate a minimum of 50 points to meet their yearly obligation.

According to records released earlier this year, Bush earned 253 points in his first year, May 1968 to May 1969 (since he joined in May 1968, his service thereafter was measured on a May-to-May basis).

Bush earned 340 points in 1969-1970. He earned 137 points in 1970-1971. And he earned 112 points in 1971-1972. The numbers indicate that in his first four years, Bush not only showed up, he showed up a lot. Did you know that?

That brings the story to May 1972 - the time that has been the focus of so many news reports - when Bush "deserted" (according to anti-Bush filmmaker Michael Moore) or went "AWOL" (according to Terry McAuliffe, chairman of the Democratic National Committee).

Bush asked for permission to go to Alabama to work on a Senate campaign. His superior officers said OK. Requests like that weren't unusual, says retired Col. William Campenni, who flew with Bush in 1970 and 1971.

"In 1972, there was an enormous glut of pilots," Campenni says. "The Vietnam War was winding down, and the Air Force was putting pilots in desk jobs. In 1972 or 1973, if you were a pilot, active or Guard, and you had an obligation and wanted to get out, no problem. In fact, you were helping them solve their problem."

So Bush stopped flying. From May 1972 to May 1973, he earned just 56 points - not much, but enough to meet his requirement.

Then, in 1973, as Bush made plans to leave the Guard and go to Harvard Business School, he again started showing up frequently.

In June and July of 1973, he accumulated 56 points, enough to meet the minimum requirement for the 1973-1974 year.

Then, at his request, he was given permission to go. Bush received an honorable discharge after serving five years, four months and five days of his original six-year commitment. By that time, however, he had accumulated enough points in each year to cover six years of service.

During his service, Bush received high marks as a pilot.

A 1970 evaluation said Bush "clearly stands out as a top notch fighter interceptor pilot" and was "a natural leader whom his contemporaries look to for leadership."

A 1971 evaluation called Bush "an exceptionally fine young officer and pilot" who "continually flies intercept missions with the unit to increase his proficiency even further." And a 1972 evaluation called Bush "an exceptional fighter interceptor pilot and officer."

Now, it is only natural that news reports questioning Bush's service - in The Boston Globe and The New York Times, on CBS and in other outlets - would come out now. Democrats are spitting mad over attacks on John Kerry?s record by the group Swift Boat Veterans for Truth.

And, as it is with Kerry, it's reasonable to look at a candidate's entire record, including his military service - or lack of it. Voters are perfectly able to decide whether it's important or not in November.

The Kerry camp blames Bush for the Swift boat veterans? attack, but anyone who has spent much time talking to the Swifties gets the sense that they are doing it entirely for their own reasons.

And it should be noted in passing that Kerry has personally questioned Bush's service, while Bush has not personally questioned Kerry's.

In April - before the Swift boat veterans had said a word - Kerry said Bush "has yet to explain to America whether or not, and tell the truth, about whether he showed up for duty." Earlier, Kerry said, "Just because you get an honorable discharge does not, in fact, answer that question."

Now, after the Swift boat episode, the spotlight has returned to Bush.

That's fine. We should know as much as we can.

And perhaps someday Kerry will release more of his military records as well.

Byron York is a White House correspondent for National Review. His column appears in The Hill each week. E-mail: byork@thehill.com


Is any of this true?

tomder55 answered on 09/24/04:

Bush's Honorable Air National Guard Service
By Col. John H. Wambough, Jr. USAF (Ret.)
September 20, 2004

George Bush and I were fighter pilots. Lt. Bush flew F-102s in the Air National Guard (ANG) -- 111th Fighter Interceptor Squadron (FIS); I flew F-105s in combat -- 34th Tactical Fighter Squadron (TFS). Both our flying assignments were inherently dangerous -- Lt. Bush's because of the high performance nature of the fighter interceptor aircraft he was flying, the training required to fly the F-102, and the high risks that come with all weather (night and day) intercept missions.

Sen. John Kerry, the Democratic National Committee's Mr. Terry McAuliffe and the anti-war (weak on National Security) left wing of the Democrat Party have relentlessly attacked the service of Lt. Bush and by inference other pilots and service members in the ANG and Reserve forces as cowards and shirkers of responsibility for not being in Vietnam. Their flippant slandering of our Guard and Reserve forces in an effort to discredit President Bush and win an election is beyond the pale. They have no decency left.

Lt. Bush's opportunity to fly jets and serve his country came through the Air National Guard when he was 22 years old. Just like Lt. Bush, my goal as a young man was to fly high performance jet fighter aircraft -- both of us realized our dream. I might have been just a dumb fighter pilot but I don't remember looking ahead (and I'm sure Lt. Bush didn't either) to what missions we could be assigned -- peacetime or wartime. All we wanted to do as young men was to fly these magnificent flying machines (jets) and enjoy the opportunity to serve our country. (Contributing to the Air National Guard's Air Defense mission, Lt. Bush flew hundreds of hours in the F-102 -- the world's first supersonic all-weather jet interceptor aircraft; he served his country protecting the United States.)

Neither Lt. Bush nor I had control over mission assignment, where we would be deployed or how the service would task the units we were assigned. Lt. Bush would have gladly gone to Vietnam or anywhere else his unit was deployed -- but the reality was that young Lt. Bush had no say as to how his unit would be utilized to support our country's National Security interests. I guess you could say such decisions were above our pay grade. Lt. Bush's mission, as a squadron fighter interceptor pilot, was to intercept and destroy enemy aircraft inbound to the United States; for example, Soviet Nuclear Bombers. Remember, we were still in the Cold War in the 1970s with Air Defense a high priority mission. Today our Air Defense forces protect us against aircraft with terrorists onboard.

I can say from my experience that flying operational fighter jets is highly dangerous. People don't strap fighter jets to their backside if they are overly concerned for their future. While in F-105 training at McConnell AFB in early 1968, we lost five aircraft in six weeks (one aircraft crashed in air-to-air combat training; one aircraft crashed on the air-to-ground gunnery range; one crashed on take off; one crashed on final approach at a nearby airfield; and one crashed coming back from a cross-country mission). My nephew was killed while flying a Marine Corp EA-6B Prowler during a low level state-side training mission. I was in a flight where an F-105 pilot was killed while we were training on an air-ground gunnery range. Also, I've been in F -105 and F-111 operational units where a number of pilots were killed while training for their war time mission. We got really good at flying "Missing Man Formations" and doing memorial services for our fallen comrades and their families. I can assure you that Lt. Bush was continuously exposed to similar dangers during all weather scrambles and during training exercises as evidenced by the F-102 pilots killed in his unit.

Cowards (or people who lack courage) don't take on the risks that Lt. Bush did in flying Fighter Interceptor Aircraft. Flying jets in wing formation in the weather and carrying explosive ordnance on board is dangerous work. The pilots in these squadrons (including Lt. Bush) did what their country asked them to do. They performed their assigned mission and did it well. In November 1970, the Commander of the Texas Air National Guard, Lt. Col. Jerry B. Killian, called Mr. Bush, then 24, "a dynamic outstanding young officer" who stood out as "a top-notch fighter interceptor pilot." "Lt. Bush's skills far exceed his contemporaries," Colonel Killian wrote: "He is a natural leader whom his contemporaries look to for leadership. Lt. Bush is also a good follower with outstanding disciplinary traits and an impeccable military bearing."

As a Fighter Squadron Commander in the Air Force (F - 111E aircraft, 55 TFS, Royal Air Force, Upper Heyford, United Kingdom) and having been in fighter squadrons during my career in the Air Force, all I can say is that the young people who make up these squadrons (like Lt. Bush) are the cream of the crop, top performers, talented, courageous and willing to take on any mission challenge presented to them, anytime, anyplace and anywhere. Everyone in a unit realizes that they serve to protect the National Security Interests of the United States and that they can be mobilized -- with short notice -- to deploy anywhere in the world.

During the Vietnam conflict, military pilot training was greatly expanded to accommodate the increased need for pilots. Thousands of pilots were trained during this conflict, primarily to support mission and pilot rotation requirements. F-105, F 4 fighter pilots, and the pilots of other combat aircraft were routinely rotated out of the combat theatre after completing their 100 combat missions. That meant that other pilots needed to be trained to take their place. As the Vietnam conflict began to phase down around 1971, there was a surplus of hundreds of pilots in the U.S. Military, for which there were relatively few flying jobs. Thus, the active duty force as well as ANG and Reserve forces could be very accommodating to those who wanted to pursue alternative career paths (such as Lt. Bush going to Harvard Business School). In fact, these sorts of administrative actions (early releases) helped alleviate the challenges facing the services of a pilot surplus. Also, commanders were lenient in allowing individuals to fulfill their service obligations in ways not involving flying duty. Such arrangements were coordinated at the unit level.

Just as Lt. Bush's supervisor released him from the ANG to go to Harvard, I released a pilot from the Air Force months early (when I was Commander of the 4442nd Tactical Control Group) so he could participate in the pilot hiring cycle of Delta Airlines. I could have held this pilot to the end of his service commitment but chose not to -- since letting him go early created no hardship to our unit. Rather, it gave a pilot (who had served his country well) an immediate opportunity for a future career. I point this fact out so that the public knows that Commanders have the prerogative to make decisions that take into consideration the needs of the Unit and the needs of an individual ready to make a career transition out of the service. Having been a Squadron Commander, I can tell you this -- we know the status of our assigned personnel all the time -- everyone is accounted for. We reported the status of all our squadron personnel daily to a higher level in our organization. Likewise, Lt. Bush's Squadron Commander of 30 plus years ago (Lt. Col. Killian, now deceased), would have known where Lt. Bush was or, at the very least, how to contact him should that have been necessary. The bottom line: Lt. Bush's documented Air National Guard service exceeded the requirements set forth in his Guard contract and Lt. Bush received an Honorable Discharge.

As Lt. Bush completed his flying assignment in April 1972, F-102s were being phased out of the Air National Guard. What we know is that he served honorably; he flew fighter jets; he embraced the inherent high risk of flying an F -102; he served our country; he met his Air National Guard requirements and he received an Honorable Discharge. The attacks on Lt. Bush are designed to diminish Lt. Bush's service to our country in the eyes of our citizens and soldiers some thirty years after Lt. Bush received an Honorable Discharge. This is truly reprehensible and driven by political adversaries like Sen. Kerry, Mr. Terry McAuliffe and their left-wing media surrogates. (Go to cspan.org for the White House Press Briefing with Scott McClellan on 02/10/04 -- the impetus for me to write this letter.)

There is a much bigger story to be told than anything related to Lt. Bush's honorable service to our country. It is the story of a fringe element of the Democrat Party (as represented by Sen. Kerry from Massachusetts and Mr. Terry McAuliffe of the DNC) that will stoop to any depth to obtain political power to include: attacking the motives of our service members (ANG and Reserve); it is a fringe element that is willing to undermine the confidence of our fighting forces in their commander-in-chief in the midst of our global war on terrorism; it is a fringe element that places political power higher on their priority list than U.S. National Security; otherwise, they wouldn't be denigrating and diminishing ANG and Reserve Service in an effort to undermine the credibility of the commander-in-chief of our Armed Forces.

Political leadership should be looking ahead not behind. But since we are looking 30 years behind I will make several comments: (1) I served with true heroes, although none of the guys I served with ever considered themselves heroes -- they just put their butts on the line every day whether flying in operational, combat or training units; (2) Lt. Bush put his butt on the line every time he scrambled on an Air Defense mission. He is a true hero that our soldiers and citizens should be rightfully proud of; and (3) the service of our Guard and Reserve soldiers should never be denigrated or diminished for political purposes or to win an election -- as is being done by Sen. Kerry and Mr. Terry McAuliffe.

Like many veterans of Vietnam, I returned to a country that was, for the most part, unappreciative of the service rendered by our fighting forces. It is sad that this attack on our commander-in-chief results in reopening the feelings and wounds of bygone years, and brings back thoughts of many comrades in arms that never returned to United States -- having given their lives for their country.

Although it is fair to recognize Sen. Kerry's four month war record and medals, it is what he did after leaving the military that deserves the greatest scrutiny. He became a turn coat by misrepresenting to the American public what our soldiers were doing in Southeast Asia. As part of the anti-war movement with Jane Fonda, he maligned, mocked and discredited our soldiers while they were still engaged in battle. He lied about what our soldiers were doing in combat. He defamed our brave fighting men. The ultimate insult our citizens could inflict on the Armed Forces of the United States would be to vote into office (as commander-in-chief) the person who betrayed his comrades in arms while they were still fighting and dying on the battlefield and in air combat.

Further, military people understand that Sen. Kerry has voted against the major weapon systems needed by the military to carry out their mission. Additionally, he has voted against CIA funding of human intelligence needed to preclude attacks on our country (such as 9/11) and protect our citizens and soldiers overseas. Senator Kerry voted for the war in Iraq and then voted against funding the war after our troops were placed in harm's way.

As we all know, since Sept. 11, 2001 our country has been at war with international terrorism. Instead of supporting our national leadership (in taking the battle to our terrorist enemies), a cabal of power-hungry politicians, supported by the liberal media elites and their vitriolic followers, have done everything they can to undermine our war on terror, our troops and the commander-in-chief of our armed forces.

Today we look with pride at our service people who risk their lives everyday for us. I'm grateful that we now have a population that, for the most part, appreciates our armed forces and their efforts to defend us. We need to fully support our soldiers and their president. Our country's success in fighting the war on terror depends on our (1) supporting those that are on the front line protecting us and (2) standing by the president who is taking the battle to the enemy.

We have not been attacked in the United States after 9/11 because we have a president and an administration that have been proactive in going after the national security threats to United States. Just think about how our national security would likely have been handled by the anti-war left of the Democrat party and Sen. Kerry. We would probably still be debating what to do in the United Nations; Afghanistan would likely still be under control of the Taliban; Iraq would still be under the control of Saddam Hussein, and cities in the United States would have come under attack on multiple occasions as terrorist organizations were further emboldened by our meek responses. And it wouldn't be surprising if we were negotiating with al Qaeda and other terrorist organizations to preclude further attacks on our cities.

The question everyone needs to ask themselves before voting in November is: Who do you trust to handle our national security? I trust President Bush, Vice President Cheney, Secretary of State Colin Powell, Secretary of Defense Rumsfeld, National Security Advisor Dr. Condoleezza Rice and Homeland Security Secretary Tom Ridge. Who do you trust?

Addendum

DNC - Kerry Campaign - CBS 60 Minutes
Left-Wing Allegations and Fraudulent Documents
Response to DNC Video "Fortunate Son

The Democrat National Committee (DNC), the Kerry Campaign, CBS 60 Minutes and their liberal media supporters have launched the fifth major attack on President Bush's service in the Air National Guard (ANG). The purpose of this attack is to enhance Kerry's election prospects by dishonoring President Bush's service in the Air National Guard (ANG) and undermining his credibility as Commander-In-Chief of our Armed Forces.

Concurrent with the airing of Dan Rather's (CBS' 60 Minutes program on 09/08/2004) hit piece on President Bush, DNC's Chairman Terry McAuliffe and Senator Tom Harkin piled on with vitriolic attacks on President Bush's service record. CBS' program (using fraudulent documents to launch their attack on President Bush) served as the kick-off call for a coordinated nationwide attack on Bush's ANG record. "Operation Fortunate Son" is the code name that Terry McAuliffe and the DNC have given for their attacks on George W. Bush's service in the Guard.

Lying Allegation:


The DNC alleges that President Bush falsely claimed he served in the United States Air Force and therefore lied about his military service record. This is the DNC's specific allegation: "George W. Bush's campaign literature claimed that he 'served in the U.S. Air Force.' The only problem? He didn't," "George Bush has a clear pattern of lying about his military service,"

Response to Allegation:

When I went through flight training in the United States Air Force (USAF), we had Air National Guard (ANG) Officers in my pilot training squadron. For economic reasons, the ANG turns to the Air Force to do its undergraduate pilot training. President Bush served in the Air Force for more than a year (as an ANG officer) while going through USAF flight training.

Also, the United States has exchange officers from other countries such as the United Kingdom (UK) that come to the U.S. to fly in our Air Force flying squadrons. Upon return to the UK, these Royal Air Force (RAF) pilots would say they served in the USAF (as an RAF Exchange officer) and that's what would be reflected in their personnel records. Similarly, our flying officers are assigned to squadron flying positions in the RAF. Their personnel records would reflect that they served in the RAF (as an USAF Exchange Officer).

Additionally, as part of Lt. Bush's training (prior to attending USAF Flight Training), Lt. Bush was placed on active duty with the Air Force for 120 days. This would be another basis for saying he served in the Air Force. To deny that fact would mean that all service people who are in training status are not serving their country. This would be absurd but nothing surprises me any more with the DNC/Kerry Campaign and what they will do and say in their quest for power.

Conclusion: The DNC doesn't know what it's talking about. You don't have to be an active-duty Air Force officer to serve in the Air Force as either an ANG pilot or an exchange pilot. George W. Bush's description in his campaign literature as having served in the USAF and ANG was correct; he did not lie about his service record as alleged by the DNC.


Influence Allegation: Another allegation is that Lt. Bush used political influence to get in the ANG.


Response to Allegation:

President Bush's father (President George H.W. Bush) has publicly stated that the Bush family did not pull any strings to get George W. Bush into the ANG. George W. Bush got a pilot slot in the ANG because he took the steps necessary to make himself a worthy applicant.

Left-Wing Democrats have alleged that there were 150 applicants trying to get in the ANG; that Bush was a "Fortunate Son" who was given preferential treatment; and that he was leapfrogged over all these applicants based on political pressure and special influence. This is a false charge. Here are the facts.

Most applicants were applying for ANG enlisted positions not for pilot training. The highest number of pilot applicants that the Texas ANG Group had (at any one time) during the Vietnam War was around ten. The reason for this very low number was there were stringent educational, security clearance and physical requirements that had to be met for pilot training in the ANG; also, there was a high "danger factor" in flying the F-102 aircraft. For every ten pilot applicants, usually only two were selected by the ANG to attend USAF pilot training school.

The question everyone needs to honestly ask themselves is: Why wouldn't the ANG want George W. Bush in their organization? How many Yale graduates do you think the ANG had to choose from (probably only one) his name was George W. Bush? So when George W. Bush went for his interview with Col. Walter "Buck" Staudt (Texas ANG Group Commander at Ellington Air Force Base in Houston), it would seem to me that Col. Staudt's decision to select George W. Bush for pilot training was a good one for the ANG - separate from any other consideration. Given that George W. Bush ended up being President of the United States, one might conclude that Colonel Staudt was an excellent judge of character; that he made the right decision.

Ben Barnes, another one of CBS' 60 Minutes helpers in orchestrating the attacks against President Bush's ANG record, states in his interview with Dan Rather that he helped George W. Bush get into the ANG. The only problem is that Ben Barnes previously swore under oath that he had nothing to do with getting George W. Bush into the Guard. Additionally, this question came up in the 2000 election and Ben Barnes' daughter asked her father if he had anything to do with helping George W. Bush get into the ANG. He told his daughter no (that he didn't help Bush get in the Guard). So what has changed? Ben Barnes is now a big fund raiser and supporter of John Kerry and may have a job with Kerry - should Kerry be elected President. This is the "reliable source" Dan Rather used to attack President Bush and help validate the content of CBS' fraudulent memos.

Conclusion: I believe that the liberal media - agenda driven - propaganda machine has not presented a fair and balanced perspective about George W. Bush's ANG service, instead, their many articles have left readers with the impression that George W. Bush was given preferential treatment over 150 ANG applicants and that the only way he would have been selected for a pilot slot would be if someone helped him politically; nothing could have been further from the truth.


Default Allegation: George W. Bush defaulted on meeting his ANG Service Requirements


Response to Allegation:


Lt. Bush entered the ANG in May 1968 and took his last (F-102) flight in the Guard four years later in April 1972. His flying tour included pilot training and than operational flying in the F-102 (111th Tactical interceptor Squadron). During Lt. Bush's time in the Guard he accumulated hundreds of hours of flying time; he served his nation honorably; he flew close to 4 years straight and performed Guard duties in 1972 and 1973 satisfactory to his Squadron Commander (Lt. Col Killian) and satisfactory to the ANG; he was given an honorable discharge in October 1973.

It was not uncommon for pilots to depart the ANG earlier than their established service commitment back in the early seventies. Allowing pilots to transition into civilian life served the best interests of the ANG in managing its pilot force. Why? The answer is simple: there were too many pilots versus the number of cockpits required to keep pilots current in their assigned aircraft. The glut of pilots was the result of a phase down of flying operations in Southeast Asia and Vietnam.

Also, it was common for ANG pilots (who were going to leave the service) to work out with their commanders, assignments not involving flying duty. The benefit to the individual was that it helped work the transition from ANG service to civilian life. The benefit to the Guard was that they could replace less experienced pilots with "high flight time" combat seasoned pilots.

An issue that keeps popping up is: why Lt. Bush didn't take an annual flight physical? The answer is simple. Lt. Bush was not going to continue flying F-102's in the 111th Fighter Interceptor Squadron. His squadron was scheduled to convert from an operational to a training squadron. Since Lt. Col Killian is deceased, it is impossible to know the dialogue that went on between Lt. Bush and Lt. Col Killian. What we do know is that Lt. Bush received an honorable discharge.

Like all Guard members, Lt. Bush was required to accrue a minimum of 50 points (annually) to meet Guard service requirements (a minimum of 300 points in six years). What the liberal media may not have covered in their many articles about Lt. Bush's ANG service is that Lt. Bush accumulated 954 points - exceeding the six-year Air National Guard requirement for service - threefold. Of course, everyone knows this, right? All those investigative reporters must have brought this fact out a dozen times. I just must have missed it.

Byron York in his September 9, 2004 article provided the numbers I use in the below chart.

All I did was add up the points. NOTE: Any objective reporter could have totaled the numbers to give people a more balanced perspective on Lt. Bush's ANG record; they chose not to. Now let's look at the ANG point system and how Lt. Bush measured up against it.

Accumulating Points toward Air National Guard Service Requirements

As you serve in the Air National Guard, you accumulate points toward meeting your ANG service requirements. When Lt. Bush was serving, the minimum points required (annually) to meet ANG requirements was fifty (50 points). The DNC, Kerry Campaign and Liberal Media are obsessed with validating that Lt. Bush earned all the ANG points necessary to qualify for his honorable discharge. They are particularly concerned with points accumulated after April 1972.

I've constructed the chart below to show that Lt. Bush not only met his annual requirements but exceeded them threefold - 954 earned versus a 300 point minimum requirement. Lt. Bush's strong record of Guard service supported his early release from a six-year service obligation to attend Harvard Business School. He received an Honorable Discharge in October 1973 having served 5 years, 4 months and 5 days.

BUSH EXCEEDS AIR NATIONAL GUARD ANNUAL SERVICE REQUIREMENTS (THREEFOLD FOR A SIX YEAR PERIOD)

Air National Guard Service Period
Minimum Annual Requirement
ANG Points Earned by Lt. Bush

May-68 to May-69
Minimum Annual Requirement - 50
ANG Points Earned by Lt. Bush - 253

May-69 to May-70
Minimum Annual Requirement - 50
ANG Points Earned by Lt. Bush - 340

May-70 to May-71
Minimum Annual Requirement - 50
ANG Points Earned by Lt. Bush - 137

May-71 to May-72
Minimum Annual Requirement - 50
ANG Points Earned by Lt. Bush -112

May-72 to May-73
Minimum Annual Requirement - 50
ANG Points Earned by Lt. Bush - 56

Jun -73 to Jul-73
Minimum Annual Requirement - 50
ANG Points Earned by Lt. Bush - 56

Ref:
http://www.hillnews.com/york/090904.aspx
Minimum Annual Requirement - 300
ANG Points Earned by Lt. Bush - 954

Fraudulent Documents Used to Attack Bush

Multiple salvos are being fired at Bush by the DNC, Kerry Campaign and their liberal media surrogates - to dishonor President Bush's service in the Air National Guard and undermine George W. Bush's credibility as Commander-In-Chief. The preponderance of evidence is that the Kerry Campaign was the planned benefactor of the CBS 60 Minutes hit piece on President Bush. But something went terribly wrong; we have a fraudulent document scandal and the mask has been pulled off CBS' extraordinary effort to undermine the President of the United States; help the DNC and get Kerry elected President.

It appears that CBS' 60 Minutes attack on Bush helped to get (in one fell swoop) thousands of liberal writers to dust off their last article (bashing President Bush's ANG service), touch it up and re launch it. This massive left-wing driven effort hopes to get the media focus off of Kerry and back onto President Bush.

Now look at the Fraudulent Memo to File (below) produced by CBS.

As of 09/15/2004, CBS' 60 Minutes stands by its story that the memos were copies of original Air National Guard documents.

Observations regarding the 18 August 1973 memo:


Typewriters used by the Air National Guard in 1973 (at the 111th Fighter Interceptor Squadron) lacked the technology to produce the 18 August 1973 Memo for File -- to the specifications of today's Microsoft Word technology. The technology lacking was proportional typeface, superscript, curly apostrophes, Times New Roman font and vertical spacing.

Should you have doubts that the memo is a fraud (after viewing my re-type), then validate the memo yourself. Instructions: Take the 18 August 1973 memo that CBS says is a copy of the original document produced in 1973 and copy it into Microsoft Word. Then set your font to the default setting - "Times New Roman Font 9". Don't change anything from Microsoft Word default settings; i.e., the type size (font 9), tab stops, and margins. Now type the memo that CBS says Lt. Col Killian typed over 30 years ago. Walla, you get the 1973 message exactly. The spacing is the same, letters line up (look above and below each sentence) and line breaks are the same.

Since anyone can quickly ascertain that this document is fraudulent merely by going through the above exercise (you don't need a documents expert, handwriting expert or typewriting expert), then why weren't the DNC, Kerry's Campaign and CBS able to determine that these documents were fraudulent (using their experts)?


Other Points about the 18 August 2004 Memo for File:


Filing a memo with "SUBJECT: CYA" is unlikely; files are subject to inspection.

During the years Lt. Bush was in the Air National Guard (ANG) the abbreviation for an Officer's Efficiency Report would be O.E.R. not OETR.

Col. Staudt retired from the Air National Guard on 1 March 1972 almost 1 years before the date of the fraudulently constructed memo of 18 August 1973.

Staudt would not be in a position to exert pressure on Hodges or Killian (to sugar coat Lt. Bush's O.E.R.) because he was not in the command line of either; he was retired.

Lt. Bush would not be getting an O.E.R. from Lt. Col Killian because he wasn't under Lt. Col Killian's observation for the number of days required to rate him. Also, Lt. Bush was leaving the Air National Guard and such a report would be of no advantage to the Guard and no advantage to Lt. Bush for career advancement in the Guard. The import of an O.E.R. (being rendered on Lt. Bush) would be minimal since he was on his way to Harvard Business School and leaving the military.


I'm not going to analyze the other CBS memos but offer this comment. Most of these memos would be a lot about nothing even if they weren't fraudulent. It's fascinating that the content of these memos just happen to coincide with the attack points used by the Kerry Campaign/DNC.

The DNC, Kerry Campaign, CBS' 60 Minutes and the liberal media establishment know nothing about ANG flying units; know nothing about the conversations Lt. Bush had with Lt. Col Killian 30 years ago; know nothing about what Lt. Col Killian authorized Lt. Bush do (regarding meeting his service requirements); know nothing about what they talked about concerning Lt. Bush's flight physical and know nothing about the views of the officers holding command positions at the time Lt. Bush served in the ANG.

But what all of us now know (based on the recent comments of ANG Colonels and Generals) is that George W. Bush was seen as an excellent officer, a top-notch pilot and highly respected by the officers and commanders who flew with him in the ANG. We know, also, that he was greatly respected by his Fighter Interceptor Squadron Commander - Lt. Col Jerry Killian -- based on comments of Killian's wife and son. We know that Lt. Bush received an Honorable Discharge from the Air National Guard.

Despite all the positive comments of the commanders in Lt. Bush's ANG chain of command (as well as the pilots that flew with him), the DNC/Kerry campaign and the mass liberal media continue belittling the service of our Commander-In-Chief.

Kerry's efforts to malign Bush's military service in the Air National Guard and undermine President Bush's credibility with the American people (while promoting himself as a war hero) will not be well received by the American people. See my article: Kerry's Strategy to Undermine Bush CLICK HERE Zell Miller best discribes John Kerry's qualifications for Commander-In-Chief (Click Here). President Bush has served as Commander-In-Chief for three and a half years. We can judge his performance. I'm voting for the re-election of President Bush.

What Should Happen Now?

CBS must divulge who gave them the fraudulent documents. It's okay for news organizations to protect information sources. It is not okay for them to use their journalistic cover to protect people who have played a part in a hoax on the American public. The use of fraudulent documents (in an effort to influence the outcome of a national election and take down a sitting President) is serious criminal activity and needs to be fully investigated and prosecuted.

-----------

John Wambough is a retired Air Force colonel with 28 years of service. During his career in the Air Force, Colonel Wambough flew F-105 and F-111 tactical fighter aircraft. His combat tour in Southeast Asia was in the F-105s with the 34th Tactical Fighter Squadron. Also, he served in Current Operations at 7th Air Force Headquarters, TanSon Nhut Air Base, RVN. He was a Fighter Squadron Commander (F - 111E aircraft, 55 TFS, Royal Air Force, Upper Heyford, United Kingdom) and later Group Commander at the same base. He served on the Air Staff and Joint Staff in the Pentagon -- and attended the National War College. He was head of the Joint Studies Group at Tactical Command Headquarters, Langley AFB, Virginia and completed his service in the Air Force as Commander of the 4442nd Tactical Control Group and Commandant of the United States Air Force Air Ground Operations School, Hurlburt Field, Florida. He retired in March 1990 - having served 28 years in the Air Force.

--------------------

Note -- The opinions expressed in this column are those of the author and do not necessarily reflect the opinions, views, and/or philosophy of GOPUSA.

http://www.gopusa.com/commentary/guest/2004/jw_0920p.shtml

labman rated this answer Excellent or Above Average Answer
CeeBee rated this answer Excellent or Above Average Answer
purplewings rated this answer Excellent or Above Average Answer
powderpuff rated this answer Excellent or Above Average Answer

Question/Answer
sapphire630 asked on 09/21/04 - Photo-ops

From what I hear Kerry loves his photo ops. He tried to do one at a fast food place with some military guys and they turned him down. He tried to do them at a couple shooting ranges and they turned him down. The one he did do one at said he held the gun all wrong. There was the one were Terry and Kerry waved corn out the train. Then, last time he was in Pittsburgh he and Terry Kerry did one at a lawyers and they were rather rude to him. He felt used for the shoot and decided he will not vote for Kerry.

I thought it was my imagination, but I heard someone say what I was thinking, so I guess it isn't just me.
They said that the Kerry's strut around like they are royalty and everybody else the little people.

tomder55 answered on 09/22/04:

Here is another example of his disconnect with the common person .Whether wind-surfing in his dapper wetsuit or running into Secret Service men on snow-covered slopes, John Kerry just doesn't seem to get the sports of the commoners Kerry's list of athletic malaprops includes:

Kerry identifying Eddie Yost as his favorite Boston Red Sox player of all time when, in fact, Yost never played for the Red Sox.

Kerry professing to being a big fan of Red Sox star Manny Ortez. The player does not exist. Kerry apparently merged the names of Sox outfielder Manny Ramirez and first baseman David Ortiz.

Kerry discussing the Buckeyes ( Ohio State University's team )while in the territory of the archival Michigan Wolverines.

Kerry misidentifying Lambeau Field in Wisconsin as Lambert Field. He apparently believed the home of the Green Bay Packers was named after Steelers hall of fame linebacker Jack Lambert.

This provided Dick Cheney with some fresh material. Cheney said during a recent Wisconsin campaign stop that "next Kerry will think that former Packers coach Vince Lombardi is a foreign leader."

Two Washington lawyers supporting Bushs re-election have registered Football Fans for Truth ,as a 527 organization . They plan to publicize Kerrys recent sports misstatements .

Jeff Larroca and Dino Panagopoulos, both members of the law firm Ballard Spahr Andrews & Ingersoll, said they have no plans to raise large amounts of cash.Instead, they hope to get free publicity from sports talk radio and other media.




sapphire630 rated this answer Excellent or Above Average Answer

Question/Answer
excon asked on 09/19/04 - The lesser of two evils.


Hello sperts:

Out next president will be either the man whose chief virtue is that he is not George Bush or the man whose chief virtue is that he is not John Kerry.

Thats just plain wrong. Are we missing anything? If we're not missing anything, and if what we see is indeed what we get, what does that tell us about the kind of people we have let ourselves become?

excon

tomder55 answered on 09/20/04:

My guess is that Ron Paul will not be running any time soon . There are few politicians that I can find without flaws. I influence what I can at the local level and hope it trickles up .That is the best I can do . I do not think our system is flawed. But our populace needs to become more informed and participate . They used to teach basic civics in the schools. But that doesn't pass muster with the pc. crowd .

excon rated this answer Excellent or Above Average Answer
purplewings rated this answer Excellent or Above Average Answer

Question/Answer
Itsdb asked on 09/17/04 - Anybody got a muzzle?

WASHINGTON - Teresa Heinz Kerry has done it again, chirping off a one-liner that landed like a thud on the campaign trail.

Visiting volunteers in New York packing relief supplies for Hurricane Ivan victims, the Democratic presidential nominee's wife said that food and water are more important than clothes in the shipments to the Caribbean islands.

``Clothing is wonderful, but let them go naked for a while, at least the kids,'' Heinz Kerry said, raising some eyebrows. ``Water is necessary, and then generators, and then food, and then clothes.''

``Let's just say that remark probably wasn't in her talking points,'' said one Democratic source close to the Kerry campaign.

During the Democratic convention in July, Heinz Kerry told a journalist to ``shove it'' - an episode that was caught by a nearby TV camera.

More recently she touted her husband's health care plan, saying: ``Only an idiot wouldn't like this. Of course, there are idiots.''

Heinz Kerry, one of the world's most wealthy philanthropists, also said: ``The common man doesn't look at me as some rich witch.''

When pro-George Bush hecklers interrupted a Kerry rally and chanted ``Four more years'' she shot back: ``They want four more years of hell.''

They really should let her speak freely more often.

tomder55 answered on 09/17/04:

she should also practice what she preaches and get naked.
I wonder if she broke a fingernail helping pack supplies ?.... I doubt it ;....she probably utilized Kerry's band of butlers .

Chouxxx rated this answer Excellent or Above Average Answer
Itsdb rated this answer Excellent or Above Average Answer
kindj rated this answer Excellent or Above Average Answer

Question/Answer
CeeBee asked on 09/16/04 - Attacks grow more sophisticated, ruthless

from the Chicago Sun-Times
September 16, 2004

BY KIM HOUSEGO

BAGHDAD, Iraq -- The scale and sophistication of militant attacks in Iraq are steadily increasing, with coordinated strikes and complicated ambushes that increasingly hit their targets, officials and analysts said Wednesday.

The spike in bloodshed -- more than 200 dead in four days -- has stifled American hopes that the transfer of sovereignty and the prospect of a democratic vote in four months could take the steam out of the uprising and pave the way for a reduction in U.S. troops.

Instead, there are signs the Americans and their Iraqi allies are facing an enemy more determined than ever. Insurgents have learned from mistakes and shifted strategy, cooperating more closely with each other and devising new ways to put their relatively simple arsenal to treacherous use.

"More thought is going into the execution of the attacks," said Lt. Col. Paul Hastings of Task Force Olympia, which is trying to bring stability to a swath of northeastern Iraq.

Sustained onslaughts

Militants now follow up roadside bomb attacks with a deluge of rocket-propelled grenades instead of fleeing, or fire off mortar rounds to lure soldiers out of their base and into freshly laid mine fields, military commanders say.

In a July attack in Samarra, for example, militants detonated a car bomb and then hammered a military headquarters with a mortar barrage as troops fled the building. Five American soldiers died.

At least 47 people were killed in a car bombing in Baghdad on Tuesday targeting would-be police recruits, the deadliest single strike in the capital in six months.

"The enemy has been able to construct IEDs [Improvised Explosive Devices] that are more complex, include more rounds in the form of a 'daisy chain,' and tend to have a higher lethality," said Maj. Neal O'Brien of the Army's 1st Infantry Division.

O'Brien also said that an increase in the use of car bombs in the last two months coincided with an influx of foreign fighters with the bomb-making know-how in July.

"They graduated to more coordinated attacks," he said.

Analysts say the plethora of armed groups behind the insurgency are increasingly working together.

"As time goes on, various gangs get together and it does become more coordinated," said Judith Kipper, a Middle East expert at the Council on Foreign Relations. "Groups start small, get know-how and become more lethal over time."

Acts of desperation?

American commanders, however, insist the stepped-up attacks and the possibility of increased cooperation among militant groups are signs that the insurgents have realized time is running out for them with the onset of elections in January.

"There is a level of desperation associated with the anti-Iraqi forces. They absolutely don't want to see free elections and reconstruction projects work," Hastings said.

But the attacks have fueled a growing backlash against the United States and interim Iraqi Prime Minster Ayad Allawi.

"The situation is getting worse day after day and the Americans are still in the streets," said Kawakib Butris, 40, a supermarket worker in Baghdad.

AP


Will there ever be peace in this country? How? When?

tomder55 answered on 09/16/04:

The press thought the march on Baghdad was a quagmire right up until the statue fell.I think some who say that the US is losing are letting their opposition to the war cloud the facts on the ground. Iraqi terrorists are killing American forces at the rate of about 3 a day. This compares to the Vietnam rate of about 2 dozen a day.In Vietnam, they also killed South Vietnamese troops at the rate of about 100 a day. This is not happening in Iraq.

Where are all the angry jihadis? Note that jihadi terrorists have a population base of 1 billion Muslims from which they can recruit.Where is the general uprising ? With the exception of the failed Najaf adventure by al-Sadr almost all the attacks on the coallition forces are happening in the Sunni triangle.

The terrorists are getting desperate but their sophistication is not improving .Does the author think that exploding a IED followed by a couple of mortar rounds sophisiticated ? Yes attacks will increase before the Jan. scheduled elections. Zarquawi was very specific in that detail .I hate to be the one to break the bad news to the fine residents of Fallujah but the truth is that if they do not wrest control of their city from the terrorists ,and if they choose not to participate in the elections then Allawi will be more than happy to isolate the city and hold elections without them.

btw. now that I think about it? Where do you think these"insurgents" are getting all this firepower? Not from old rusty Saddam armaments I assure you .They are being infiltrated in through Syria and Iran. Both nations that will have to be dealt with before this conflict is over and peace comes to Iraq.

In your other posting you bemoan the lack of progress of reconstruction . I was critical of the CPA and their handling of the matter also but bottom line ;security comes before sewers .

CeeBee rated this answer Excellent or Above Average Answer
excon rated this answer Excellent or Above Average Answer

Question/Answer
ETWolverine asked on 09/15/04 - Has Kerry blown the cheesehead vote?

Has Kerry blownthe cheesehead vote?
Candidate tries to regainfooting after Lambeau flub

By Jim VandeHei
The Washington Post
Updated: 11:50 p.m. ET Sept. 14, 2004


MILWAUKEE, Sept. 14 - Forget soccer moms and NASCAR dads. The most important demographic in these parts transcends gender and geography -- it's Green Bay Packers fans.

Both candidates are targeting them with the ferocity of a Brett Favre bullet, but only John F. Kerry has fumbled the name of the hallowed grounds on which the Packers play, the frozen tundra of Curly Lambeau Field.

At a campaign event last month, the Democratic presidential nominee called it Lambert Field -- a slip of the tongue carried on television, in papers throughout the state and on ESPN's Web site.

That's akin to call the Yankees the Yankers or the Chicago Bulls the Bells. This is a place where Packers jackets often outnumber sports coats in church and thousands of fans wear a big chunk of yellow foam cheese atop their head with the pride of a new parent. President Bush's warning to terrorists is apropos to the passions of Packers fans -- you are either with 'em or against 'em.

"I got some advice for him," Bush told Wisconsinites a few days after the Lambert gaffe. "If someone offers you a cheesehead, don't say you want some wine, just put it on your head and take a seat at Lambeau Field." Vice President Cheney made the obligatory pilgrimage to Green Bay last week to pile on. "I thought after John Kerry's visit here I'd visit Lambert Field," Cheney told a crowd at a Republican fundraising dinner Thursday night. Then he went in for the kill. "The next thing is he'll be convinced Vince Lombardi is a foreign leader."

Perhaps Lombardi, the Hall of Fame coach who put Titletown on the map in the 1960s, is working from the beyond the grave to trip up the Massachusetts senator. After all, Richard M. Nixon considered Lombardi as his running mate in 1968. There's one problem with this: Lombardi was a Kennedy Democrat. In fact, the Kennedy's connection to the Green and Gold runs even deeper. In 1955, Packers Coach Lisle Blackbourn flirted with a talented young pro prospect in Massachusetts: Ted Kennedy, who now plays offensive line for the Kerry campaign.

'Stick to windsurfing analogies'
Still, it might take more than the Kennedy mystique to put the "Lambert" moment behind Kerry -- a moment Kerry aides call a slip of the tongue. The Bush campaign is planning to rehash the comment until Election Day as way of portraying Kerry as detached from the beer-drinking, bratwurst-eating folks of Wisconsin. College Republicans in Madison, where Kerry will visit Wednesday, are planning to greet him with a new sign: "It's Lambeau, Stupid!"

"He tries [too] hard to fit in with Wisconsinites, and he fumbles every time," said Jennifer Millerwise, a Bush campaign spokeswoman. "He should stick to windsurfing analogies, only problem for Senator Kerry is that most people watch the Packers on Sunday."

This strategy is not confined to Cheeseland either. Republicans poked fun of Kerry for talking about the Buckeyes (of Ohio State University) while visiting arch rival Michigan (home of the Wolverines). These seemingly innocuous digs fit into a larger Bush-Cheney plan of fashioning the president as a common man and Kerry as a pandering patrician.

Kerry's slip is rookie stuff compared with Bush's verbal blunders, including his famous creation of the word "misunderestimated."

And David Wade, a Kerry spokesman, said Packers fans will see the failed "playbook" of the Republicans. "Any Packers fan knows . . . Bush has fumbled on Iraq, did a double reverse on the assault weapons ban and dropped the ball on health care." Then Wade went personal. "I don't think we need any lectures in sports from a former cheerleader," referring to one of Bush's activities while at prep school.

So, could a candidate lose the state by tripping over the name of a stadium? Probably not, though Al Gore won Wisconsin by only few thousand votes in 2000 and small shifts can make a big difference in this battleground state this year, state political observers say. "It sort of plays into the perception, right or wrong, that people think John Kerry is an opportunist who when he is not out windsurfing comes in to try to be a regular guy," said Ken Goldstein, a professor at the University of Wisconsin. The latest polls shows Kerry down eight points; he was tied in most polls before the gaffe.

Regain his footing
Cheney, by comparison, hit all the right notes when he visited Green Bay last week, according to local papers. Not only did he speak to the biggest issue in the state -- the Packers -- he did so with Bart Starr, the Hall of Fame quarterback by his side. "I've never been around someone I was more impressed with," Starr said of Cheney. The QB and VP also visited the Packer Hall of Fame, footsteps from the stadium

In a campaign stop here Tuesday, Kerry looked to regain his footing. Speaking to a small group of seniors, nine hours after the Packers defeated NFC Champion Carolina Panthers, 24-14, Kerry said he "got in town last night in time to watch the Pack," though he landed well into the third quarter. It was among his biggest applause lines. "So I know you would all be in a good mood today."

In the end, it's the Packers' score -- more than the Packers vote -- that could determine Kerry's fate. In the past 18 presidential elections, if the Redskins lost or tied the last game before the election, the party in the White House lost, too. The Redskins' opponent Oct. 31: the Packers, but not at Lambert, er, Lambeau Field.

2004 The Washington Post Company
URL: http://www.msnbc.msn.com/id/6004366/
-----------------

Sorry I just felt like posting this... I thought i was pretty funny. Feel free to comment.

Elliot

tomder55 answered on 09/16/04:

notice how Kerry has stuck to the rust belt strategy ? rarely does he venture from Ohio;Michigan;Wisconsin;or West Va. When he does ;he usually makes brief appearances in the North West.

These ancedotes are amusing .They happen alittle too frequently for him to deny a disconnection with the folks he is trying to reach. Of course wheter he hunts deer with a shot gun or does an imitation of Charleston Heston's "out of my dead fingers" moment while holding a rifle with a pistol grip that he thinks should be banned; these moments will not be the deciding factor in the race.

What I see in Kerry is a candidate who's message did not work .At this late stage of the campaign he is running to the left to shore up his base and piecing his positions together on the fly. He has coopted the Dean platform ;especially Iraq .On May 3, 2003, Kerry said that "it was the right decision to disarm Saddam Hussein. And when the president made the decision, I supported him, and I support the fact that we did disarm him."Last month on August 9 he said that knowing what we know now he still would've supported the Iraq war . Then yesterday on Imus he was asked if there were any circumstances under which "we should have gone to war in Iraq," . Kerry said no. Which prompted Imus to bemoan ;"I was just back in my office banging my head on the jukebox," "This is my candidate, and ... I don't know what he's talking about."

He is talking all the left's talking points.He recently raged against letting the assault rifle ban slide ;he played the race card at the Black Congrssional Caucus meeting .He is going from State to State in the rust belt beating the economy while the economy steadily improves ,and polling numbers show that only about 10% of the electorate consider the economy the most important factor in the election.

Today he is going to address the National Guard at the same time that he has been indirectly dispariging those who serve in it as doing an elaborate dodge.That should be fun!


ETWolverine rated this answer Excellent or Above Average Answer
sapphire630 rated this answer Excellent or Above Average Answer

Question/Answer
ETWolverine asked on 09/15/04 - To all conservatives...

I'll be out of touch for the next few days due to the Jewish holiday.

Hold down the fort while I'm gone.

Keep the faith...

And a Happy (Jewish) New Year to all of you. May G-d inscribe you all in His book for goodness.

Elliot

tomder55 answered on 09/16/04:

L'Shanah Tovah ; have a Happy New Year !

ETWolverine rated this answer Excellent or Above Average Answer

Question/Answer
Chouxxx asked on 09/15/04 - CBS to "Apologize"

shortly today for the forged documents presented on the nightly news re: Bush's National Guard Service. The appropriate response is for Dan Rather to resign(in disgrace).

Do you agree?

tomder55 answered on 09/15/04:

yes ;but he won't . They have no choice but to apologize . Document experts that CBS say vetted the program are coming out and claiming that they warned CBS about the docs.Emily Will, a document examiner from told ABC News she the there were problems with the docs.
"I found five significant differences in the questioned handwriting, and I found problems with the printing itself as to whether it could have been produced by a typewriter," she said.

Will says she sent the CBS producer an e-mail message about her concerns and strongly urged the network the night before the broadcast not to use the documents.

"I told them that all the questions I was asking them on Tuesday night, they were going to be asked by hundreds of other document examiners on Thursday if they ran that story," Will said.



Rather, according to The American Spectator (who along with blogger Little Green Footbals discovered the truth about the documents ),is hunkering down and trying to ride out the storm. "Rather and his supporters now believe the controversy surrounding the four discredited Texas National Guard memos has been engineered by the Bush campaign"...."Rather's guys are now going after the president of the United States in a way probably no news organization has ever done before. This is now deeply personal to them...."In the end, it probably doesn't matter," says the CBS News producer. "We're sunk. Our reputations have been impugned, and if we didn't look like we were shilling for Kerry a week ago, we look like we're trying to at least protect a source who gave us these documents who might be supportive of Kerry or at least the Democratic Party."
"



Chouxxx rated this answer Excellent or Above Average Answer
ETWolverine rated this answer Excellent or Above Average Answer

Question/Answer
ETWolverine asked on 09/14/04 - Dangerous ground.

I just read this article at Slate. I'm not sure what to think of it. Is torture ever an acceptable action by America for gathering intelligence information? In certain cases I might argue in favor of it, but as a POLICY, I don't think so.

What do you think? Your opinions are appreciated.

Does Torture Work?
Seymour Hersh evades the question.
By Fred Kaplan
Posted Tuesday, Sept. 14, 2004, at 2:01 PM PT

Seymour Hersh's new book, Chain of Command: The Road From 9/11 to Abu Ghraib, reveals our most intrepid investigative reporter working near the top of his game. Basically a compilation of the pieces that Hersh wrote for The New Yorker over the past few yearsexpanded, updated, and re-edited, in some cases significantly sothe book holds up as a cohesive tale and a searing indictment of the Bush administration: its chicanery with intelligence in the months leading up to the Iraq war, its inadequate planning for the war's aftermath, and its muffing of all the warsin Iraq, Afghanistan, and the broader war against terrorismever since.

There is, however, one gnawing equivocation in Hersh's otherwise forthright account. It comes in the first section, called "Torture at Abu Ghraib," which takes up over 70 pages of this 370-page book. Hersh blew the lid off the Abu Ghraib scandal last springthe photographs, the Taguba report, the cover-ups, the links up the chain of command (which, in his book, he extends all the way up to the Oval Office). But he has always skirted a vital question: Does torture work?

Hersh is not alone in his evasiveness. Liberals have a tendency to accept, all too eagerly, the argument that torture is ineffective, that it doesn't yield useful information, that a tortured detainee will tell his inquisitors whatever they want to hear. This is an appealing argument. If it's true, we don't have to wrestle with any moral or legal dilemmas. If torture simply doesn't work, all those difficult questions are moot.

But it is, in fact, very likely that, under some circumstances, with some detainees, torture does produce, in the parlance of the trade, "actionable intelligence." Torture to produce a confession ("Yes, I am a terrorist") almost certainly is useless; at some point of pain, many people would confess to anything. But torture to elicit specific information (Who told you to do this? Where did the meeting take place? Who else is in your cell? What are they planning to blow up tomorrow?) sometimes will doclearly, has donethe job. If it hasn't, many times over the centuries, then why do so many regimes engage in it? Some no doubt do it for the kicks, but they're not all purely sadists.

I do not mean to advocate torture. I mean only to suggest that it's time to start wrestling with those moral and legal dilemmas, to face them straightforwardly. If al-Qaida strikes the United States again, our leaderswhoever they arewill be tempted to resort to torture as a method of getting vital intelligence quickly, and we or they or someone should have mapped out crucial distinctions ahead of time: What is acceptable, what isn't; who should engage in it, who shouldn't; for what purposes is it legitimate, for what purposes isn't it; or whether we should decide, after an honest appraisal of its costs and benefits, that the whole business of torturehowever you define itis irredeemably beyond the pale.

It should be noted that the torture at Abu Ghraib appears to be utterly unjustified by any standards. Hersh clearly showsand the Schlesinger report has confirmedthat the vast majority of the inmates at Abu Ghraib were common criminals or total innocents rounded up in random sweeps who were subjected to no screening before their horrendous ordeals began.

But what about the inmates elsewhere, many of whom really were, and are, al-Qaida operatives? Hersh refers to a highly classified "special-access program"approved by President Bush and carried out by Secretary of Defense Donald Rumsfeldthat involved, as he puts it, "snatching or strong-arming suspected terrorists and questioning them in secret prison facilities in Singapore, Thailand, and Pakistan, among other sites." What about the torturepresumably there's torture of one sort or anotherthat went on there? For the moment, forget about whether such techniques are proper. That's a separate though no less important matter, to be dealt with after this question is answered: Did they produce useful intelligence?

At one point, Hersh suggests that they did. He writes that, early on in the Iraqi insurgency, detainees weren't giving their American interrogators any substantive information. Hersh quotes a "former intelligence official" on what Stephen Cambone, the assistant secretary of defense in charge of the operation, did in response in mid-2003:

Cambone says, I've got to crack this thing and I'm tired of working through the normal chain of command. I've got this apparatus set upthe black special-access programand I'm going in hot. So he pulls the switch, and the electricity begins flowing last summer. And it's working. We're getting a picture of the insurgency in Iraq and the intelligence is flowing into the white world. We're getting good stuff.

Things went awry, Hersh's source goes on, because, when the order went out, too few soldiers were trained in what to do, and too many of their commanders looked the other way. But notice what the source said about the initial results: "[I]t's working. We're getting good stuff." So, is the problem Cambone's orders or the fact that the U.S. military didn't have enough people with the brains or training to carry them out with restraint?

The "former senior intelligence official" seems to suggest the latter. Hersh, summarizing his words, writes:

The SAP [special-access program] was useful as long as it was under the control of "good, well-trained guys. But politics got involved, and decisions were based on speed, and not patience."

Similarly, Hersh quotes a "Pentagon consultant" as saying of the Abu Ghraib torturers:

We don't raise kids to do things like that. When you go after Mullah Omar, that's one thing. But when you give the authority to kids who don't know the rules, that's another.

Again, the question is tacitly raised: What about when "you go after Mullah Omar"? Then is it all right to use extreme measures, if necessary?

In a later chapter, dealing with the failure of U.S. intelligence and especially the collapse of the CIA's clandestine service through the 1980s and ྖs, Hersh tells a tantalizing story about the Jordanian security service. In the mid-1980s, Abu Nidal's terrorist organization threatened the life of Jordan's King Hussein. The king told the service, "Go get them." In response, security agents seized close family members. Hersh continues:

The Abu Nidal suspect would be approached, given a telephone, and told to call his mother, who would say, according to one CIA man, "Son, they'll take care of me if you don't do what they ask." (To his knowledge, the official carefully added, all the suspects agreed to talk before any family members were actually harmed.) By the early 1990s, the group was crippled by internal dissent and was no longer a significant terrorist organization. "Jordan is the one nation that totally succeeded in penetrating a group," the official added. "You have to get their families under control."

Hersh doesn't explicitly endorse this method. But does he implicitly? Should he? Should we? He adds, "Such tactics defy the American rule of law, of course, and the CIA's procedures, and many experts doubt that they are even effective." Who are these doubtful experts, and what's their reasoning? Hersh's CIA source seems to think the tactics were effective. As for law and procedures, should they stand in the way of taking apart al-Qaida? It's a radical proposition for the U.S. government to start acting like the Mafia. Again, I don't have the answers. But it's time that we all began to ask the questions.

Fred Kaplan writes the "War Stories" column for Slate.

http://slate.msn.com/id/2106702/

Elliot

tomder55 answered on 09/15/04:

I am inclined to agree with Alan Dershowitz's premise that a 'ticking bomb scenario ' is ample justification for the use of "torture" techniques.

ETWolverine rated this answer Excellent or Above Average Answer

Question/Answer
Chouxxx asked on 09/14/04 - USA Losing Iraqi War

When are we going to hear *anything* about our current wars, realistic appraisals and statistics and projections based on something other than spin-speak?Anything meaningful from our President and his Opponent? Anywhere? I'd like to know.

tomder55 answered on 09/15/04:

This doesn't directly answer your question but it should add some perspective .

A car bombTuesday ripped through a busy market near a Baghdad police headquarters where Iraqis were waiting to apply for jobs on the force, and gunmen opened fire on a van carrying police home from work in Baquoba, killing at least 59 people total and wounding at least 114. Many of the victims were waiting outside a police station to apply for work. A second attack was carried out in Baquoba, when gunmen sprayed a van filled with Iraqi police with gunfire, killing twelve. The terrorists are attempting to discourage Iraqis from joining the police force; at least 710 police officers have been killed; but that has not discouraged recruitment. Since April 2003, the Iraqi police force grew to over 130,000 officers on patrol and the Iraq National Guard has grown to 45,000 . This growing force of the Iraqi gvt. is the greatest threat to the insurgency ;not the U.S. presence. The U.S. and our allies militaries will probably eventually leave Iraq and these Iraqi forces will be the difference between stability and chaos. Lately Iraqi forces have conducted joint operations with coalition forces in cities like Mosul .

Basically the resistance is composed of Shia led by al-Sadr , and former Saddamites who have aligned themselves with jihadists in Falluja and Ramadi ;cities that have become havens for fighters loyal to al-Zarqawi . Yes things are tense and it could get ugly in the next few months because the terrorist will try to duplicate the Madrid model . In Iraq they can potentially influence 2 elections . The Iraqi elections and the U.S. elections . This is their Tet moment and they of course can count on their accomplices in the press to spin it the way they want it as illustrated brilliantly by The Investors Business Daily Tuesday:Zarqawi, of course, is trying to take over Iraq and crush its emerging democracy. What matters to him aren't the reports' raw facts, such as damage and body counts, but reporters' opinions. And most reporters, including those in wire services that once prided themselves on objectivity, are looking for signs of American defeat.
So they opine on the basis of a day's violence that the security situation is spiraling "out of control." Or they judge the insurgency to be on the rise, implying that the American effort to bring peace and democracy to Iraq is on the skids.
If the events in Iraq were just a matter of perception, the Sunday of car bombs and shelling would have to be chalked up as a huge victory for the insurgents. And we expect to see more such carnage as the presidential election nears. If there were ever a time to strike at the U.S. war effort with maximum political impact, this is it.




ETWolverine rated this answer Excellent or Above Average Answer
Yiddishkeit rated this answer Excellent or Above Average Answer
Chouxxx rated this answer Excellent or Above Average Answer

Question/Answer
excon asked on 09/14/04 - Our Hero

The average age of the military man is 19 years. He is a short haired,
tight-muscled kid who, under normal circumstances is considered by
society as half man, half boy. Not yet dry behind the ears, not old
enough to buy a beer, but old enough to die for his country. He never
really cared much for work and he would rather wax his own car than wash
his father's; but he has never collected unemployment either.

He's a recent High School graduate; he was probably an average student,
pursued some form of sport activities, drives a ten year old jalopy, and
has a steady girlfriend that either broke up with him when he left, or
swears to be waiting when he returns from half a world away. He listens to
rock and roll or hip-hop or rap or jazz or swing and 155mm howitzer. He
is 10 or 15 pounds lighter now than when he was at home because he is working
or fighting from before dawn to well after dusk.

He has trouble spelling, thus letter writing is a pain for him, but he can
field-strip a rifle in 30 seconds and reassemble it in less time in the dark.
He can recite to you the nomenclature of a machine gun or grenade launcher and
use either one effectively if he must. He digs foxholes and latrines and can
apply first aid like a professional. He can march until he is told to
stop or stop until he is told to march.

He obeys orders instantly and without hesitation, but he is not without spirit
or individual dignity. He is self-sufficient. He has two sets of fatigues: he
washes one and wears the other. He keeps his canteens full and his feet dry.
He sometimes forgets to brush his teeth, but never forgets to clean his rifle.
He can cook his own meals, mend his own clothes, and fix his own hurts. If
you're thirsty, he'll share his water with you; if you are hungry, his food.
He'll even split his ammunition with you in the midst of battle when you run low.

He has learned to use his hands like weapons and weapons like they were his hands.
He can save your life - or take it, because that is his job. He will often do twice
the work of a civilian, draw half the pay and still find ironic humor in it all.
He has seen more suffering and death then he should have in his short lifetime.

He has stood atop mountains of dead bodies, and helped to create them. He has wept
in public and in private, for friends who have fallen in combat and is unashamed.
He feels every note of the National Anthem vibrate through his body while at rigid
attention, while tempering the burning desire to 'square-away' those around him who
haven't bothered to stand, remove their hat, or even stop talking. In an odd twist,
day in and day out, far from home, he defends their right to be disrespectful.

Just as did his Father, Grandfather, and Great-grandfather, he is paying the price
for our freedom. Beardless or not, he is not a boy. He is the American Fighting
Man that has kept this country free for over 200 years. And, he has asked nothing
in return, except our friendship and understanding.

tomder55 answered on 09/14/04:

Lt. Sam Weinberg-"Why do you like them so much?"


Lt. Comdr. Joeanne Gallaway-"Because they stand on a wall and say nothing's going to hurt you, not on my watch!"
[A Few Good Men 1992]



ETWolverine rated this answer Excellent or Above Average Answer
kindj rated this answer Excellent or Above Average Answer
excon rated this answer Excellent or Above Average Answer

Question/Answer
kindj asked on 09/13/04 - THIS is problem solving?

From NewsMax.com:

Kerry: I'll 'Take on the Terrorists' With Gun Control

Presidential candidate John Kerry promised over the weekend that he would "take on the terrorists" who attacked the U.S. on 9/11 by forcing them to obey America's gun control laws.

Kerry said laws like the Assault Weapons Ban, which expires today, were valuable "not just to fight ordinary crime but to take on terrorists."


And he complained that President Bush was "making the job of the terrorists easier" by not pushing for a renewal of the sweeping gun control law.

"The 9/11 commission and other reports have shown that al-Qaida wanted to come into America, and in the al-Qaida manual of terror, they were telling people to go out and buy assault weapons," Kerry told a crowd in Missouri.

The top Democrat did not explain why the Assault Weapons Ban failed to protect America against the 9/11 attacks, which were executed by al-Qaida operatives armed with small knives.

Nor did Kerry say how many terrorists had been arrested and charged with violations under the Assault Weapons Ban.

tomder55 answered on 09/14/04:

Do I have to give up my NRA membership if I say that there are certain weapons that I do ot think the public should own ? I am open to arguments about assault rifles either way and am undecided if the ban should've remained. The Democrats have a valid argument when they say that Bush's rhetoric on the issue did not match his actions. Is he for the ban or not ? He said he would sign the bill if it came to his desk ,but he did not ask for an extention .I guarantee Kerry will try to pin him down for a clarification if they debate the issue.

ETWolverine rated this answer Excellent or Above Average Answer
kindj rated this answer Excellent or Above Average Answer
Yiddishkeit rated this answer Excellent or Above Average Answer

Question/Answer
purplewings asked on 09/14/04 - Things yet to be said.

WOULDN'T IT BE GREAT TO TURN ON THE TV AND HEAR GEORGE W. BUSH OR JOHN KERRY GIVE THE FOLLOWING SPEECH?

My Fellow Americans:
As you all know, the defeat of Iraq regime has been completed. Since congress does not want to spend any more money on this war, our mission in Iraq is complete.

This morning I gave the order for a complete removal of all American Forces from Iraq. This action will be complete within 30 days. It is now to begin the reckoning.

Before me, I have two lists. One list contains the names of countries which have stood by our side during the Iraq conflict. This list is short. The United Kingdom, Spain, Bulgaria, Australi a, and Poland are some of the countries listed there.

The other list contains everyone not on the first list. Most of the worlds nations are on that list. My press secretary will be distributing copies of both lists later this evening.

Let me start by saying that effective immediately, foreign aid to those nations on List 2 ceases immediately and indefinitely. The money saved during the first year alone will pretty much pay for the costs of the Iraqi war.

The American people are no longer going to pour money into third world Hell-holes and watch those government leaders grow fat on corruption.

Need help with a famine? Wrestling with an epidemic? Call France.

In the future, together with Congress, I will work to redirect this money toward solving the vexing social problems we still have at home. On that note, a word to terrorist organizations. Screw with us and we will hunt you down and eliminate you and all your friends from the face of the earth. Thirsting for a gutsy country to terrorize? Try France, or maybe China.

To Israel and the Palestinian Authority. Yo, boys. Work out a peace deal now. Just note that Camp David is closed. Maybe all of you can go to Russia for negotiations. They have some great palaces there. Big tables, too. I am ordering the immediate severing of diplomatic relations with France, Germany, and Russia. Thanks for all your help, comrades. We are retiring from NATO as well. Bon chance, mes amis. I have instructed the Mayor of New York City to begin towing the many UN diplomatic vehicles located in Manhattan with more than two unpaid parking tickets to sites where those vehicles will be stripped, shredded and crushed. I don't care about whatever treaty pertains to this. You creeps have tens of thousands of unpaid tickets. Pay those tickets tomorrow or watch your precious Benzes, Beamers, and limos be turned over to some of the finest chop shops in the world. I love New York.

A special note to our neighbors. Canada is on List 2. Since we are likely to be seeing a lot more of each other, you folks might want to try not pissing us off for a change. Mexico is also on List 2. President Fox and his entire corrupt government really need an attitude adjustment. I will have a couple extra tank and infantry divisions sitting around. Guess where I am going to put em? Yep, border security. So start doing something with your oil.
Oh, by the way, the United States is abrogating the NAFTA treaty --- starting now. We are tired of the one-way highway.

It is time for America to focus on its own welfare and its own citizens. Some will accuse us of isolationism. I answer them be saying, "darn tootin."

Nearly a century of trying to help folks live a decent life around the world has only earned us the undying enmity of just about everyone on the planet. It is time to eliminate hunger in America. It is time to
eliminate homelessness in America. It is time to eliminate World Cup Soccer from America. To the nations on List 1, a final thought. Thanks guys. We owe you and we won't forget. To the nations on List 2, a final thought. Drop dead.

God bless America.
Thank you and good night.

Any comments?
PW

tomder55 answered on 09/14/04:

There is no point in the 'war on terror'(uggh I hate that phrase. I mean the war against jihadists )where we have 'gone it alone 'or acted unilaterally . It is a distortion to say so and the repeated reciting of this mantra does a big disservice and dishonor to the allies we assembled who have sacrificed their people in this cause .How will Kerry be able to bring in more allies when he ridicules them with lines like 'coallition of the bribed'.

Colin Powell said :

There are still 31 nations in the coalition because we believed it was a danger to us and to the region and to the world. But, in other places throughout the world, with respect to the Iranian nuclear weapons program, what are we doing? We're working with the international community. What are we doing in Korea? Invading Korea? No not at all. We're working with Korea's neighbors to persuade Korea, North Korea, that nuclear weapons are not in its interests, and that a de-nuclearized peninsula is in its interest. The United States has worked to expand NATO. We've supported the expansion of the European Union. We are passing programs in our Congress that provide huge amounts of assistance to developing nations. We are taking the lead in trying to relieve the suffering in Sudan. We're taking the lead in fighting HIV/AIDS. So, notwithstanding some common perceptions that all the United States wants to do is find some place to conduct unilateral military operations, it's the contrary that's true. The United States believes in development. It believes in alliances. It believes in multilateral approaches. But where multilateral approaches won't solve the problem, we have to have freedom of action to get a willing coalition that will solve the problem.

NATO along with France have aided in the efforts in Afghanistan (although I am more than willing to conceed that France has acted as a trip wire every step of the way.)
We have gotten a high degree of coopoeration even with allies who oppose us in intelligence gathering against terror cells around the world.

The world is too small for the U.S. not to take an active role .

Chouxxx rated this answer Excellent or Above Average Answer
ETWolverine rated this answer Excellent or Above Average Answer
kindj rated this answer Excellent or Above Average Answer
purplewings rated this answer Excellent or Above Average Answer

Question/Answer
Chouxxx asked on 09/11/04 - Why Trash Newspapers?

Americans get their information from television.

Proof::: How many millions and millions of dollars do political candidates spend on television time to get their "message" out??? And, compared to newspaper ads.

Newspapers are owned privately, not by the government. They can say what they want!!!!

And, for the information of all the crypto-Nazi-s on the Board, there are Liberal publications, like Mother Jones or Utne Reader. Big city newspapers are Moderate.

When you trash newspapers, you trash democracy.

Cordially, Choux

tomder55 answered on 09/12/04:

they should be honest about their content. Renown papers like the N.Y. Times used to confine opinion to the editorial pages .Now the content of their news sections is loaded with opinion and slant. That in itself is not wrong.But they should be upfront about it .

This is their claim :

Here is the living disproof of the old adage that nothing is as dead as yesterday's newspaper... This is what really happened, reported by a free press to a free people. It is the raw material of history; it is the story of our own times. -[Henry Steel Commager, preface to a history of the New York Times, 1951]

They consider themselves an unbiased reference for the ages. Would that they lived up to that lofty goal.

But truth be told early American publications made no bones about where they stand .It was a rough and tumble ;take no prisoners style that many would find libelous by todays standards. Many major political figures owned and operated their own journals ;some wrote in pseudonymity ;some didn't bother to try to hide it. This continued into the early 20th century with publishers like Hearst not only 'reporting'the news but setting in motion the events of the times. Today the internet provides plenty of sites to filter the news for accuracy so it is easier to 'seperate the wheat from the chaff'.

You are also correct to point out that newspapers have been in a large extent supplanted as the nations primary source of news and information.



Chouxxx rated this answer Excellent or Above Average Answer
ETWolverine rated this answer Excellent or Above Average Answer
powderpuff rated this answer Excellent or Above Average Answer

Question/Answer
excon asked on 09/11/04 - It's Time For Cooperative Effort!


Hello experts:

There are fewer than two months until the election, an election that will decide the next President of the United States. The man elected will be the President of all Americans, not just the Democrats or the Republicans.

To show our solidarity as Americans, let's all get together now and show each other our support for the candidate of our choice. It's time that we all come together, Democrats and Republicans alike.

If you support John Kerry, please drive with your headlights on during the day.

If you support the policies and character of George Bush, please drive with your headlights off at night.

Thank you for your support of this important cooperative effort!

excon

tomder55 answered on 09/12/04:

Like dgade ;my car denys me that option so I have an alternative idea that Mark Levin first proposed . Since we are expecting an unusually high turn out and we do not want to overy inconvenience voters by making them stay in long lines. I propose that all Bush supporters vote on Tues. Nov 2 ;and Kerry supporters vote on Wed. Nov.3

excon rated this answer Excellent or Above Average Answer
purplewings rated this answer Excellent or Above Average Answer

Question/Answer
Chouxxx asked on 09/10/04 - September **10** 2001

The following is a post from Askme made on the above date by a user whose last name was an anagram including the letters a-r-a-b. After the events of 9-11, I was drawn back to this post, and well....see what you think...

" 'End of Story'....

Greetings friends,

Death...do you ffear looking into his face?

"...a man once said t me, that death looks at us all (blurred) can do is smile back..."

How does it feel taking the ultimate last step into the unknown"

My theory of life and death is this:"

[The author's long discourse follows, I'll not post it now]

I felt that terrorists were using Askme as a message service and more.

What do you think of this internet post on the day before 911??

Cordially, Choux

tomder55 answered on 09/11/04:

I never considered that the 'expert' was sending messages but in hindsight considering the cryptic nature of some of the rantings that the expert posted it is possible.

Chouxxx rated this answer Excellent or Above Average Answer

Question/Answer
voiceguy2000 asked on 09/10/04 - Tomorrow is 9/11

Tomorrow is 9/11/2004 -- the third anniversary of the unprovoked attack by Islamic terrorists on unarmed civilians in this country, designed for no other purpose than mass destruction and loss of life.

Daniel Henninger of The Wall Street Journal has written one of his better columns (and that is saying something) today, available at this link.

What I find myself wondering is this: Will media sources such as The Washington Post or The New York Times devote even a fraction of the space to commemorating 9/11/2001 that they devoted for days on end to photographs from Abu Ghraib prison? There seems to be a bizarre, self-imposed blackout on actually showing pictures of the 9/11 destruction in U.S. media -- you have to go to overseas sources for much of it. The same is true, for that matter, for photos of the destruction of that school in Beslan.

We are likely to see more coverage of the [almost certainly forged] documents concerning George Bush's 1972 National Guard service than we are of the historic attack on the United States that took place 3 years ago tomorrow.

There is something massively wrong with this, isn't there?

tomder55 answered on 09/10/04:

Indeed there is ;and yes there is a self imposed boycott on showing pictures of 9-11 among other things .If it were about sensitivity then the endless photos of human pyramids would not have been the lead story night after night .

It has been a while since I relied on the major media sources as my sole supply of news . They ;especially CBS with Dan Rather as their news editor have long ago ceased being a source of unbiased news (if they ever were in the first place .remember ,Truman used to complain about a one party press in the 40s ). CBS will have a tough time spinning the story behind them being presumably duped by the Democrats over the forged Bush National Guard Docs. The American Spectator had a great summary of what happened called : "Anatomy of a Forgery".The relevent part of it was that ABC news believes that CBS was suckered into airing it ,but typical of the left ,they also floated a trial balloon that Karl Rove may have pulled a dirty trick on both CBS and the Kerry campaign.

Anyway ,I spend alot of time surfing the net .I find that bloggers act as a great filtering tool. It was blogger One Hand Clapping who actually typed up copies of the documents on his computer to prove that the font used could not have been used in typewriters available in the early 70s.

Will CBS and Dan Rather take a simular hit that the Times took over the Jason Blair fiasco ? I doubt it .We will be lucky to get a retraction.

Yup ,the web is my source . That is where pictures of 9-11 are available ;and where with enough digging the truth can be found.

voiceguy2000 rated this answer Excellent or Above Average Answer

Question/Answer
Itsdb asked on 09/09/04 - And they think Cheney is mean?

How about these tidbits from the Kerry side...

James Carville, speaking of Zell Miller to Don Imus:

"They got that poor man in the twilight of his career and just used him...They said, 'Look, go up there and say this,' and they handed him a bunch of documents...They probably shot him up with something, you know. He just likes screaming at people."

Carville on Bush:

8/27/04 "Now, all this debate about whether President Bush is stupid or not or whether we should call the president of the United States stupid isn't for me. I'll stay away from the S-word, but I'll go right to the I-word. He's just ignorant...Now, Mr. President, I don't know if you're stupid or not, but I sure know this. You're ignorant big time."

Paul Begala on Bush:

8/18/04 "I think that's the kind of intelligence we ought to be talking about, the president's lack of intelligence."

8/20/04 "Why is he so gutless?"

8/25/04 "Bush was gutless..."

Bush has been called "cheap thug," "killer" and a "liar" at Kerry campaign events. He's been labeled by liberal columnists, celebrities and strategists, etc. a draft dodger, bigot, racist, dictator, moron, evil, terrorist threat, stupid, lazy, and my favorite by Margaret Cho, "one monkey head used Lipton tea bag."

Will the party of peace and tolerance please stand up?








tomder55 answered on 09/10/04:

Begala is an idiologue. Carville .....well... he is the best argument against inbreeding.

Itsdb rated this answer Excellent or Above Average Answer

Question/Answer
Chouxxx asked on 09/09/04 - Fox News Presidential Poll

reported today on Fox showed a statistical tie between Bush and Kerry(error margin + - 3%)with 8% undecided. Bush's service record in the National Guard is besmearched by previously unearthed and undisclosed records showing he was suspended for missing required medical exam.

Looks like a real knuckle bitter down to Nov 2.

tomder55 answered on 09/09/04:

No one from the White House or the campaign has questioned the veracity of these documents.I have heard rumors of forgeries proven by a different font used in the type writers but unless the campaign addresses the issue then we have to assume they are authentic. The question then becomes what do they mean ? [Bobbie did a good job pointing out that Ben Barnes is a liar and that 60 Minutes II is lame in not checking out the facts before airing a feature spot based on his lie.]

Kerry and company have been all over the place claiming the Bush was a coward for joining the Guard ;or that he used going into the Guard as an elaborate dodge. Now Kerry is scheduled to speak next week to the National Guard in Las Vegas . I wonder how he will address this issue to them?

All this stuff kills me. For 8 years in the 90s the military record of the President didn't matter one bit even though he waged campaigns against two genuine war heros. Now this year .Two candidates with questionable records is all that is talked about. [FYI Byron York of The Hillhas a good summary of what is known of the Bush National Guard record.]

Bush has a 4 year Presidential record and Kerry a 20 year Senate record to dissect . Time is wasting.

Chouxxx rated this answer Excellent or Above Average Answer
ETWolverine rated this answer Excellent or Above Average Answer

Question/Answer
Itsdb asked on 09/08/04 - Kerry on Iraq

Just for grins, would anyone care to take a look at the Kerry On Iraq video, compare it to his current position, and then try and explain what his position on Iraq might be?

tomder55 answered on 09/09/04:

He has held the same position on Iraq for a couple of days now .As Regis said ;Is that your final answer? Of course the devil is in the details . He leaves more questions unanswered then he answers. To sum up;Kerry has basically adopted the Howard Dean take on the war . The whole thing is and was wrong.He has seemingly used the word wrong 100 times in the last 2 or 3 days .He criticizes Bush for not having a plan but sorta fails to unveil one of his own lest he tie his hands. He has indicated that he would begin U.S. withdrawal in as soon as 6 months after taking office ,and he hopes it would be complete in the first 4 years. No longer is victory a precondition.

Taking a page from the Clinton book ;he is trying to view the war in terms of it's economic cost rather than how it relates to the over-all war on the Islamo-fascists and the despots who support them.This was his take on it December of last year:

"Iraq may not be the war on terror itself, but it is critical to the outcome of the war on terror, and therefore any advance in Iraq is an advance forward in that"


He offers up a bunch of domestic spending priorities that are being neglected ,and could've been funded with the money spent. That is pure hog-wash demogogery; which in itself was a hallmark of the Dean candidacy.

Here is William Kristol's take on Kerry in yesterdays 'Standard' :

He has decided he's against the war in Iraq. It was "the wrong war in the wrong place at the wrong time."

Fine. Now we have a clear choice in the presidential election. Bush went to war to remove Saddam. Kerry, it now appears, would not have. This means the choice is between the world we have now, and a world with Saddam still in power. For the meaning of saying we fought the wrong war at the wrong time, is that we would have been better off leaving Saddam in power. If John Kerry were president, Saddam would still be in power.

So Kerry has to answer this question: Would we be safer with Saddam still in power? Would the world? What would such a world look like? Surely we couldn't have left 150,000 troops in the nations bordering Iraq for two years. Surely, then, the inspectors would once again have been expelled. And the sanctions regime was collapsing. Does Kerry then believe Saddam would not
have moved to reconstitute his weapons of mass destruction? Would that have been acceptable? Does Kerry believe pro-American, anti-terror forces in the Middle East, to say nothing of the forces of reform in that region, would be stronger or weaker if Saddam were still in power? What would have been the global effect on American credibility if we had authorized the president to use force, as Kerry voted to do, and then backed off? And what would a Kerry administration do now? How could a President Kerry ask any young American to be the last one to die for a mistake?


Itsdb rated this answer Excellent or Above Average Answer

Question/Answer
Chouxxx asked on 09/08/04 - The Hardball Starts

Cheney made remarks that if Kerry was elected, we would have another terror attack in America(paraphrase). Bush tries to withdraw from one of three debates because there is no moderator to control questions.

tomder55 answered on 09/08/04:

I was not happy w/ Cheney's comments .If we get attacked before the elections what does that mean ?

I can't blame Bush for asking out of the debate. It was to be set up as a town hall format with Gallup picking 'undecided' voters,and it was to be held in a battle ground State;Missouri. Given the fact that trolls were able to penetrate security at the GOP convention ,the Bush camp is concerned that Kerry supporters masquerading as undecided voters could become part of the audience and become uncivil. A legit concern. I'm sure both sides will meet and agree on an acceptable format.

Chouxxx rated this answer Excellent or Above Average Answer
elgin_republicans rated this answer Excellent or Above Average Answer
ETWolverine rated this answer Excellent or Above Average Answer

Question/Answer
popeye asked on 09/07/04 - Did The Protesters Help Kerry?

After the RNC last week here in the city last week do you think the angry mobs of protesters getting themselves arrested to make their point helped the Kerry/Edwards campagine? My feeling is after all the B.S. that these people pulled it just makes the democrats look bad.

I am not speaking of those who demonstrated peacefully and orderly. I am mainly spaking of those who felt it was necessary to take off their clothes, climb buildings, block traffic, beat up police officers and the like.
Not to mention disrupt a youth organization inside the garden probably terrorifying the kids who were supposed to be there. I am kind of upset that they would use such a terrible tactic to make a point.

tomder55 answered on 09/08/04:

re :the 9-11 referense ;we need to be reminded of this every day.

popeye rated this answer Excellent or Above Average Answer

Question/Answer
Chouxxx asked on 09/07/04 - Putin Speech Today

Putin condemned "The West" for the anemic response to Islamoterrorism and barbarity in a sarcastic speech today. I would like to read the contents of the speech. Thanks, Choux

tomder55 answered on 09/08/04:

The events in Russia has me so angry that I have trouble thinking clearly about it .Many thoughts go rapidly through my mind. Here are a couple of them :

1. The increase in Islamic terrorist attacks against Russia, including the Beslan massacre means that WMDs in the hands of Jihadis pose as serious a threat to Russia as anyone else. Nuclear armed Iranian missiles would certainly be. Russia has been one of the principal suppliers of nuclear technology to Iran. The scum who slaughtered Ossetian schoolchildren may have bribed their way past checkpoints. Putin may wish to reconsider whether it is wise to do business with them. The Beslan siege clearly illustrates that the Russian response to terrorism has been inadequate. Putin was 100% correct to slam critics around the world (including those in the U.S. Sate Dept.)who dare suggest that Russia should open up talks with them ,and today Russia's top general threatened to attack "terrorist bases" anywhere in the world. What the critics are looking for is a bunch of hand wringing self examining mea- culpas like they saw in the 9-11 and Abu Ghraib hearings .


2.RALPH PETERS of the N.Y. Post has an excellent article about the cowardly attacks on children and the apparent jihadist goal of preventing children from getting a secular education. He writes not only of the Chechen attack on the innocent Russian children ,but also of attacks in Afghanistan and Iraq against schools that dare to stray from the Koranic verses.

He points out that the world of Islam was once the protectors of higher learning but that was many years ago and ;If anything could eventually triumph over the passionate ignorance crippling the Middle East, it's education real education.Since that is probably as big a threat to the Islamo-fascists as American military forces we can probably expect this to be incorporated into their m.o. First ;the kidnapping and beheading of innocent adults ,and now the shooting of starving, half-naked children in the back ; Are there any cowardly tactics they will not adopt? Which brings me to a final point.

3. We have now watched them ;behead people while praising Allah;slaughter innocent children; murdered women and daughters over perceived offenses(including the barbaric use of stoning) ;burn down temples and churches of other faiths ,many times while worshipers prayed inside them. We have witnessed them subject women to all types of deprivation ;treating them worse than their farm animals. We have seen the reintroduction of slavery ;the mutilation and forced conversions of the conquered . Their clerics fill them full of visions of a Paradise full of sexual rewards if they die in the cause .Young men and women who strap bombs on themselves and use them are idol worshipped in their communities . With that in mind ;isnt it about time that we identify this enemy without a name? Calling this world war a 'war on terrorism is clearly inadequate . Why are we so afraid of it ? Have they not demonstrated what is at stake? Bush may have gotten it right when he proclaimed that a war on terrorism is not winnable. He displayed a fleeting moment of candor and reality . A war on terrorism does not fire up the imagination because it is far too abstract. Terrorism is not the enemy . The enemy is Islam as defined by the fascists that control it.




Chouxxx rated this answer Excellent or Above Average Answer
ETWolverine rated this answer Excellent or Above Average Answer

Question/Answer
Chouxxx asked on 09/07/04 - Saladin

Is he another identity for JeffreyBryson? I really don't follow various identities around here very meticulously so I'm curious. [Any other help re: identities appreciated.] Choux

tomder55 answered on 09/08/04:

He is the reincarnate Saladin the scourge of the Crusades seeking out his old nemisis Richard the Lionheart.

ETWolverine rated this answer Excellent or Above Average Answer
Chouxxx rated this answer Excellent or Above Average Answer

Question/Answer
Gguru asked on 09/05/04 - 21st century warcrime scandal!

21st century warcrime scandal!

Ammunition factories are using depleted uranium in their products.
Anti-tank shells and alike have tips with depleted uranium,which make it possible and easier to burn thru
tank armour,but leaving behind extremely toxic dust,that lasts for a hundred years!
This toxic radio-active dust will be touched and inhaled by anyone getting near the area where it is used and far , far beyond.It causes deadly disease and genetical deformations by the victims as well as by their children,their children's children etc,etc.
All kinds of deformations do occur.
Like children with no eyes or no nose or other deformed body-parts or illnesses!
Its a crime against humanity and LIFE as such to produce these weapons!
Hundreds of deformed and sick children are being born in Iraq.
Hundreds of English and American soldiers are dying from this and having genetically deformed children,who will pass this thru to their next generations!
Its a crime against humanity and LIFE to use this kind of weapons!
Why isnt this being discussed thru the U.N.?!
Why isnt this scandalic issue part of the election campaigns?!
There is plenty of evidence around!
Its a crime against humanity and LIFE not to address this issue!
It should be posted on every available board on the internet!
People should know!

tomder55 answered on 09/07/04:

Extensive studies have concluded that no radiological health hazard should be expected from exposure to depleted uranium.

Scientists say depleted uranium has about 40 percent of the radiation of natural uranium, which itself is not a health hazard. But since Italy started studying the illnesses of 30 Balkans veterans ;seven of whom died of cancer;including five cases of leukemia ; more than a dozen nations have been testing soldiers serving or having returned from the Balkans. None have found any traces of depleted uranium.

A number of other nations have sent teams to the Balkans to analyze the environment for health risks. To date, there has been no indication of increased level of radiactivity at any of the sites tested.

http://www.ifrance.com/avigolfe/balkans.htm

http://news.bbc.co.uk/1/hi/world/europe/1205632.stm

ETWolverine rated this answer Excellent or Above Average Answer
Gguru rated this answer Excellent or Above Average Answer

Question/Answer
excon asked on 09/05/04 - Don't you feel safer???


Hello experts:

My son in moving to Seattle. He packed up a 26' U-haul truck to gills. The SUV he was towing was also packed to the top.

Last night, as he was about to cross the Hoover dam in NEV. your homeland security cops made him unload every last box, right on the side of the road so they could make sure he's not going to blow up the dam.

He's still there, on the side of the f***ing road right now, after 18 hours.

Does this make you feel safer?

excon

tomder55 answered on 09/06/04:

The U.S. is 3,000 miles from one coast to another, and 1,400 miles between Mexico and Canada . 93 percent of the U.S. border is free of surveillance.There is an official list of terrorist targets the FBI and CIA share with the media. There are additional targets these agencies don't make public.Hoover Dam is on the list.

Lets say Hoover Dam was destroyed. The devastation would be much greater than the hurricanes that hit Fla.this month and the economic impact would be far reaching .Just for starters it would cripple Los Angeles, Phoenix, and Las Vegas for years ;and of course an incalculable number of lives would be lost. The resulting floods would destroy many river towns along the Colorado .The environmental impact would be devastating .Hoover Dam contains the largest reservoir in the United States, supplying potable water to 30 million people.It would take years and apx 250 million to replace such an asset.

The obvious answer to the security would to build a traffic by-pass around all the major dams in the country. Until such time as that is done however Hoover Dam remains under tightened security. It's web sight is very clear on the restrictions :

Noncommercial vehicles that may cross the dam include:

Passenger cars
Mini-vans and full-size passenger vans
SUV's
Pickup trucks
Trailers (RV, boat, horse, etc.) **Will be inspected**
Motorhomes and other recreational vehicles **Will be inspected**
Rental trucks (e.g. U-haul, Ryder) **Will be inspected**
Note for vehicles that "Will be inspected". If your vehicle is packed so full as to make it impossible to perform a visual inspection of all items in the vehicle, then you will have to take the detour. In other words, if you have a U-haul or an RV that is packed all the way to the top, or close to it, you will not be allowed to cross the dam.

He should consider himself lucky that they asked him to off load the u-haul instead of making him take a detour.




VisionsInBlue rated this answer Excellent or Above Average Answer
excon rated this answer Excellent or Above Average Answer
Chouxxx rated this answer Excellent or Above Average Answer
powderpuff rated this answer Excellent or Above Average Answer
ETWolverine rated this answer Excellent or Above Average Answer
Yiddishkeit rated this answer Excellent or Above Average Answer

Question/Answer
powderpuff asked on 09/05/04 - Muslims: new U.S. vote bloc

From : deseretnews.com

Muslims: new U.S. vote bloc

ROSEMONT, Ill. American Muslims opened their largest annual gathering Friday, emphasizing the importance of voting as the Muslim community struggles for political recognition.

More than 30,000 people were expected to attend the three-day meeting organized by the Islamic Society of North America, an umbrella association representing Muslim groups and mosques nationwide.

Voter registration booths will be set up, and American Muslims who have been elected to public office will discuss their campaigns. Two sessions have been dedicated to getting out the vote.

"We are this nation," said Kareem Irfan, chairman of the Council of Islamic Organizations of Greater Chicago.

The conference is the third for the Islamic Society since the Sept. 11, 2001, terrorist attacks, and organizers planned to address many of the problems that have plagued the community since then.

Presentations will be made on defending Islam against prejudice, creating ties with leaders of other faiths and preserving Muslims' civil rights during the domestic war on terrorism.

Sheik Muhammad Nur Abdullah, president of the Islamic Society, opened the session with a prayer asking God to "heal the wound from 9/11," and urged U.S. Muslims to build relations with other faiths as a model of tolerance for fellow Muslims overseas.

"As Muslim Americans, we can set that example," he said.
However, the presidential race will be a top issue.

As recently as the 2000 election, some Muslim immigrants debated whether their religion even allowed them to participate in democratic elections. American Muslim leaders say those questions are no longer being raised, and they are working for high voter turnout in their communities.

Muslims have sizable populations in swing states such as Ohio, Michigan and Florida and hope this will help them gain visibility in the tight presidential race.

Already, they have been pleased with their representation at the Republican and Democratic conventions, according to Sayyid Syeed, secretary general of the Islamic Society. Muslims gave invocations or benedictions at both events. An Islamic Society spokesman said as of Friday, there was no indication either party would send a representative to the conference.

In 2000, major Muslim organizations made their first collective endorsement of a presidential candidate, backing George W. Bush. However, many Muslim leaders said they came to regret that decision after Sept. 11.

They said the broad new powers the federal government gained through the USA Patriot Act have made all Muslims suspects. The Bush administration has defended the law as critical for national security.

Community leaders say they have not yet decided whether to make an endorsement in this year's presidential campaign. Polls indicate that most U.S. Muslims plan to vote for Democrat John Kerry.

On interfaith relations, Syeed said his organization has made gains, working with The Church of Jesus Christ of Latter-day Saints, the U.S. Conference of Catholic Bishops and the National Council of Churches, which represents mainline Protestant and Orthodox churches.

Syeed noted many mosques have been so successful in reaching out to those of other faiths that non-Muslims sometimes outnumber Muslims at Friday prayers.

"We are proud that in this society, in spite of the fact there were tremendous provocations . . . the dominant spirit has been the spirit of understanding," he said.
------------

My question: Do you have any concerns about the political influence of Muslim Americans during this time of world terror led by Arab Muslims? And do you believe there is a difference between American Muslims and other Muslims, or not?

tomder55 answered on 09/06/04:

I know CAIR reports that Fifty four percent of eligible Muslim voters said they would vote for Kerry and 26 percent for Nader, an American of Lebanese descent, while 14 percent remain undecided,;but you could hardly call them an unbiased source.

I tend to believe this as I think that the silent majority in the Muslim world is sympathetic to the bin-Ladens of the world.

Kerry should start a new 527 ;'Terrorist sympathizers and European appeasers for Kerry'.

powderpuff rated this answer Excellent or Above Average Answer

Question/Answer
ETWolverine asked on 09/03/04 - The Bounce

The first post RNC poll has come out. Time Magazine. Its available on their website.

http://www.time.com/time/press_releases/article/0,8599,692562,00.html

Friday, Sep. 03, 2004
Campaign 2004: Bush Opens Double-Digit Lead
TIME Poll: Among likely voters, 52% would vote for President George Bush, while 41% would vote for John Kerry and 3% would vote for Ralph Nader

New York: For the first time since the Presidential race became a two person contest last spring, there is a clear leader, the latest TIME poll shows. If the 2004 election for President were held today, 52% of likely voters surveyed would vote for President George W. Bush, 41% would vote for Democratic nominee John Kerry, and 3% would vote for Ralph Nader, according to a new TIME poll conducted from Aug. 31 to Sept. 2. Poll results are available on TIME.com and will appear in the upcoming issue of TIME magazine, on newsstands Monday, Sept. 6.

Most important issues: When asked what they consider are the most important issues, 25% of registered voters cited the economy as the top issue, followed by 24% who cited the war on terrorism as the top issue. The situation in Iraq was rated the top issue by 17% of registered voters, moral values issues such as gay marriage and abortion were the top issue for 16% of respondents, and health care was the most important issue for 11% of respondents.

Bush vs. Kerry:
The economy: 47% trust President Bush more to handle the economy, while 45% trust Kerry.
Health care: 48% trust Senator Kerry to handle health care issues, while 42% trust Bush.
Iraq: 53% trust Bush to handle the situation in Iraq, while 41% trust Kerry.
Terrorism: 57% trust Bush to handle the war on terrorism, while 36% trust Kerry.
Understanding the needs of people: 47% said they trust Kerry to understand the needs of people like themselves, while 44% trusted Bush to understand their needs.
Providing strong leadership: 56% said they trust Bush to provide strong leadership in difficult times, while 37% said they trust Kerry to provide leadership in difficult times.
Tax policy: 49% trust Bush to handle tax policy, while 40% trust Kerry.
Commanding the Armed Forces: 54% said they trust Bush to be commander-in-chief of the armed forces, while 39% said they trust Kerry.

Bush on the Issues:
Iraq: Half (50%) of those surveyed approve of the way President Bush is handling the situation in Iraq, while 46% disapprove. In last weeks TIME poll, 48% approved of the way Bush was handling the situation in Iraq and 48% disapproved.
Terrorism: Almost two thirds (59%) said they approve of how President Bush is handling the war on terrorism, while 38% disapprove. Last weeks TIME poll found 55% approved of Bushs handling of the war on terrorism, while 40% disapproved.
The Economy: Survey respondents were split on the Presidents handling of the economy. Almost half (48%) said the approved of Bushs handling of the economy, while 48% said the disapproved.

Other results include:
Was U.S. Right Going to War with Iraq? Over half of those surveyed (52%) think the U.S. was right in going to war with Iraq, while 41% think the U.S. was wrong to go to war.

Have the United States actions in Iraq made the world safer? Almost half (45%) think the United States actions in Iraq have made the world safer, while 45% think the world is more dangerous. In a similar TIME poll taken Aug. 3 5, over half (52%) said the world was more dangerous, and 38% said the world was safer.

# # #

Methodology: The TIME Poll was conducted August 31 September 2 by telephone among a random sample of 1,316 adults, including 1,128 reported registered voters and 926 likely voters. The margin of error for registered voters is +/- 3% points, and +/- 4% points for likely voters. Schulman, Ronca, & Bucuvalas (SRBI) Public Affairs conducted the poll, and more complete results are attached.

Contacts:
Ty Trippet, 212-522-3640
Jennifer Zawadzinski, 212-522-9046

-----------

Guys, this isn't a bounce. It's a friggin' ROCKET LAUNCH.

Your comments, please.

Elliot

tomder55 answered on 09/05/04:

In interviews, leading Democrats ; governors, senators, fund-raisers and veteran strategists said they had urged Kerry's campaign aides to concentrate almost exclusively on challenging President Bush on domestic issues from here on out, saying he had spent too much of the summer on national security.

The first part of Bush's speech detailed an unbelievably ambitious domestic agenda. Bush is planning his second term as if he will be coming in with a mandate . He proposes broad changes in the way entitlements are funded and distributed ,and nothing less than a sweeping overhaul of the tax system.

Kerry has spoke little of his own agenda ;spending most of his time attacking Bush's performance instead. "If you give me a hundred dollars, I couldn't tell you a single policy thing they talked about,'' Ed Gillespie, the national Republican chairman, said. "They gave us a huge opening, and we jumped on it.''Kerry answered this as he did in the primaries. He reshuffled his staff.

But there is a big contrast between them on taxation that will be an interesting debate .

If re-elected,Bush will make permanent the tax cuts he introduced and will reform the tax structure so as to lighten taxes on dividends and earnings. That means that taxes on consumption will have to go up.
Kerry would repeal the tax cuts for families earning more than $200,000, and return the rate to 39.6 percent . He has also promised to raise dividend and capital gains taxes on high earners.

This is nothing less than a debate on what should the government role on an individuals economic life be ? Bush wants to return control of money to the earners ,Kerry thinks the gvt. can do a better job.That is the standard by which any debate on the domestic agenda should be viewed.

I think Kerry will attempt to join this debate since a focus of him as a leader who we can trust with the security of the nation has backfired badly.

ETWolverine rated this answer Excellent or Above Average Answer

Question/Answer
Chouxxx asked on 09/04/04 - War and the Election

I don't believe that Peace for a country's population can be achieved through war. I'm speaking of Iraq and other Middle Eastern Countries(not the general War on Terrorism-Hammas, Al Quaeda, etal).

There is no way to "win" through violence. I look back at the Korean War and the Second World War. Peace was won through helping Japan, Korea, and Germany become countries where the common people had a say in chosing their government, America build and/or rebuilt their economies and called the countries our friends going forward.

Neither candidate for president discussed the future of Iraq in any manner that was meaningful to me. This War is Bush's catastrophe. We as citizens have the duty to vote him out of office on November 2 for this incredible blunder.

I am reminded of how Lyndon Johnson declined to run for reelection because of his failure to end the VietNam War, so then, the New President Nixon was able to "negotiate" the end.

I think we have learned from Bush's and the New-Con's blunder that we cannot pursue the policy of preemptive war in the Middle East to any satisfactory peaceful conclusion. The religious crazies outnumber us; they always will because of their outlandish birth rates and fear of change in their social and cultural fabric.

I'm going to vote for Kerry. I have to.

tomder55 answered on 09/05/04:

I have in a number of responses tried to explain that war in Iraq as a battle in the wider war was inevideble. I do not have time to rehash it as it looks that my moms home took the eye od Francis yesterday and my computer time is limited today. Suffice it to say that there is ample room to criticise the timing of the invasion as excon and now James Fallows does in the most recent Atlantic monthly edition;and there is certainly room to debate the execution of the post war period ,especially the management of the CPA by Paul Bremer ;but there is no doubt in my mind that the war was necessary to fight.

Your point about Japan and Germany are just wrong. The common people had no say . We installed democracies there and stayed around long enough for them to take root. First we kicked the crap out of them and took the will to fight away. Perhaps that is the biggest difference. But as I mentioned above ,I think the post war period was mismanaged to an extent,and monies that are allocated already to reconstruction are only now beginning to find its way to where it can do some good.


The pre-emptive strategy is necessary .Will we now wait until Iran has nukes to deal with it? Do we continue to allow Syria to attack us as they have through surrogates since the early 80s? Do we wait for another 9-11 before we respond as Kerry has suggested he would ? It took 3 bloody years to defeat a single nation that had indoctrinated one generation into fascism. How long should it take us to change the hearts and minds of multiple nations who's populace has learned jihad for over a millenium? Bush has said that it will be a long war that will take multiple administrations to complete.

The choice is clear .Kerry wants to execute this war as persuing terrorist groups in a law enforcement manner;not holding accountable nations that support,finance,and harbor these organizations. He has said nothing that would indicate otherwise.Bush has laid out a long term strategy that if successful would end the threat of Islamo-fascism to future generations .

powderpuff rated this answer Excellent or Above Average Answer
Chouxxx rated this answer Excellent or Above Average Answer

Question/Answer
Chouxxx asked on 09/02/04 - McCain on Letterman 9-1-04

Last night on Letterman, Senator McCain said that(I paraphrase)Senator Kerry would make a fine President. I really felt good about his remarks since all the right wing media is bashing Kerry relentlessly.

I didn't watch the keynote address because I don't care for aged Southern Racist bastards no matter what party they are in.

Cheney...his speech was refreshing. Cold calculating and unemotional. Really, I loved it!

How about you?

tomder55 answered on 09/03/04:

In 1993 then Gov. Zell Miller led the unsuccessful fight to change the Georgia flag from the Confederate symbol. It almost ended his career.So if indeed he was a racist in the past(his remark about Johnson selling his soul n 1964 which he has since disawowed) he has evidently had a change of heart ,as many Southern Democrats did (note:Jimmy Carter in his early years played the race card from time to time ,and Robert Byrd of course was once a KKK and is now admired by civil rights groups).In office ,Miller's treatment of minorities was impecable. As Govenor he initiated universal prekindergarten education and college scholarships for every student with a B average( the HOPE Scholarship is sending an entire generation to college for free .As a result Georgia, mainly Atlanta, has become the economic engine of the South ,and the largest Black middle class communities are located there).

Anyway ,his speech was good ole fashion politics ;the type of speech that television coverage is making a lost art of . He was if nothing else honestly outraged at the direction the Democrat Party has drifted . There are many former Democrats (like me) who agree.

Cheney did not suprise me as I have always felt the demonization of him was unwarrented. He has never been a hack .He has always been a loyal administrator for any administration he served in .He has never to my knowledge shown ambitions to become the President although it is probably true that he is the most influential VP in history.

Senator McCain has walked the fence by wooing favor on both sides of the aisle.It didn't suprise me that he would make a comment like on Letterman At the RNC he said :

"All of us, despite the differences that enliven our politics, are united in the one big idea that freedom is our birthright and its defense is always our first responsibility.

All other responsibilities come second.

We must not lose sight of that as we debate who among us should bear the greatest responsibility for keeping us safe and free.

We must, whatever our disagreements, stick together in this great challenge of our time.

My friends in the Democratic Party - and I'm fortunate to call many of them my friends - assure us they share the conviction that winning the war against terrorism is our government's most important obligation.

I don't doubt their sincerity.

They emphasize that military action alone won't protect us, that this war has many fronts: in courts, financial institutions, in the shadowy world of intelligence, and in diplomacy.

They stress that America needs the help of her friends to combat an evil that threatens us all, that our alliances are as important to victory as are our armies.

We agree.

And, as we've been a good friend to other countries in moments of shared perils, so we have good reason to expect their solidarity with us in this struggle.

That is what the President believes.

And, thanks to his efforts we have received valuable assistance from many good friends around the globe, even if we have, at times, been disappointed with the reactions of some.

I don't doubt the sincerity of my Democratic friends. And they should not doubt ours."



It is not a bad strategy on his part . No matter who wins this election the nation will be pretty much fed up with the partisanism that the nation has gone to .He is banking on the fact that he can win as a fusion candidate. He will not be alone in that strategy. Guilliani will run as a moderate Republican and I suspect that Lieberman may make another go of it.

Chouxxx rated this answer Excellent or Above Average Answer
johnh1234 rated this answer Excellent or Above Average Answer
powderpuff rated this answer Excellent or Above Average Answer

Question/Answer
purplewings asked on 09/02/04 - Do Bush Sr. and Bush Jr. ever talk politics? LOL

As you watch the Republican Convention, listening to what the party has to say about domestic and foreign policy, present and future, thought you'd find the following perspective on Iraq interesting.

In his memoirs, A World Transformed, published in September 1998, George Bush Sr. wrote the following to explain why he didn't go after Saddam Hussein at the end of the first Gulf War:

"Trying to eliminate Saddam would have incurred incalculable human and political costs. Apprehending him was probably impossible ... We would have been forced to occupy Baghdad and, in effect, rule Iraq ... there was no viable "exit strategy" we could see, violating another of our principles. Furthermore, we had been self-consciously trying to set a pattern for handling aggression in the post-Cold War world. Going in and occupying Iraq, thus unilaterally exceeding the United Nations'mandate, would have destroyed the precedent of international response to aggression that we hoped to establish. Had we gone the invasion route, the United States could conceivably still be an occupying power in a bitterly hostile land."

If only his son could read.

tomder55 answered on 09/02/04:

I would prefer to read the text of John McCain's address to the GOP . His take on it better reflects the post 9-11 reality that we faced regarding Iraq.

"After years of failed diplomacy and limited military pressure to restrain Saddam Hussein, President Bush made the difficult decision to liberate Iraq.

"Those who criticize that decision would have us believe that the choice was between a status quo that was well enough left alone and war. But there was no status quo to be left alone.

The years of keeping Saddam in a box were coming to a close. The international consensus that he be kept isolated and unarmed had eroded to the point that many critics of military action had decided the time had come again to do business with Saddam, despite his near daily attacks on our pilots, and his refusal, until his last day in power, to allow the unrestricted inspection of his arsenal.

Our choice wasn't between a benign status quo and the bloodshed of war.

It was between war and a graver threat. Don't let anyone tell you otherwise. Not our critics abroad. Not our political opponents."

http://www.usatoday.com/news/politicselections/nation/president/2004-08-30-mccainfulltext_x.htm

But old man Bush was wrong then also . The fact remains that he incited the Shia to revolt against Saddam under the implied support for an overthrow of his regime. When Bush failed to support them ,Saddam brutally massacred many of them under the watchful eyes of the U.S. and allied militaries that were in a position to do something about it. How much of the current problems we have there are a direct result of the lack of trust the people have over that folly ? Why should they trust us ? We already proved back then that we could not keep our word . It is my sincere delight that Bush jr. is his own man and not his father.


ETWolverine rated this answer Excellent or Above Average Answer
purplewings rated this answer Excellent or Above Average Answer

Question/Answer
kindj asked on 09/01/04 - Now THIS is interesting,

and gets to the REAL point:

KERRY SERVICE RECORD

Wednesday, August 25, 2004 7:06 PM

John Kerry does not want his service record questioned. This is the reason why.

Subject: Hanoi John's Military Service

On 18 Feb. 1966 John Kerry signed a 6 year enlistment contract with theNavy (plus a 6-month extension during wartime).

On 18 Feb. 1966 John Kerry also signed an Officer Candidate contract for 6 years -- 5 years of ACTIVE duty & ACTIVE Naval Reserves, and 1 year of
inactive standby reserves (See items #4 & #5).

Because John Kerry was discharged from TOTAL ACTIVE DUTY of only 3 years and 18 days on 3 Jan. 1970, he was then required to attend 48 drills per year, and not more than 17 days active duty for training. Kerry was also subject to the Uniform Code of Military Justice. Additionally, Kerry, as a commissioned officer, was prohibited from making adverse statements
against his chain of command or statements against his country, especially during time of war. It is also interesting to note that Kerry did not obtain an
honorable discharge until Mar. 12, 2001 even though his service obligation should have ended July 1, 1972.

Lt. John Kerry's letter of 21 Nov. 1969 asking for an early release from active US Navy duty falsely states "My current regular period of obligated
service would be completed in December of this year."

On Jan. 3, 1970 Lt. John Kerry was transferred to the Naval Reserve Manpower Center in Bainridge, Maryland.

Where are Kerry's Performance Records for 2 years of obligated Ready Reserve, the 48 drills per year required and his 17 days of active duty per year training while Kerry was in the Ready Reserves? Have these records been released?

Has anyone ever talked to Kerry's Commanding Officer at the Naval Reserve Center where Kerry drilled?

On 1 July 1972 Lt. John Kerry was transferred to Standby Reserve -Inactive.

On 16 February 1978 Lt. John Kerry was discharged from US Naval Reserve.

Below are some of the crimes Lt. Kerry USNR committed as a ReadyReservist, while he was acting as a leader of Vietnam Veterans Against the War:

1. Lt. Kerry attended many rallies where the Vietcong flag was displayed while our flag was desecrated, defiled, and mocked, thereby giving aid and comfort to the enemy.

2. Lt. Kerry was involved in a meeting that voted on assassinating members of the US Senate.

3. Lt. Kerry lied under oath against fellow soldiers before the US Senate about crimes committed in Vietnam.

4. Lt. Kerry professed to being a war criminal on national television, and condemned the military and the USA.

5. Lt. Kerry met with NVA and Vietcong communist leaders in Paris, in direct violation of the UCMJ and the U.S. Constitution.

Lt. Kerry by his own words & actions violated the UCMJ and the U.S. Code while serving as a Navy officer. Lt. Kerry stands in violation of Article 3, Section 3 of the U.S. Constitution. Lt. Kerry's 1970 meeting with NVA Communists in Paris is in direct violation of the UCMJ's Article 104 part 904, and U.S. Code 18 U.S.C. 953. That meeting, and Kerry's subsequent support of the communists while leading mass protests against our military in the year that followed, also place him in direct violation of our Constitution's Article 3, Section 3, which defines treason as "giving aid
and comfort" to the enemy in time of warfare.

The Constitution's Fourteenth Amendment, Section 3, states, "No person shall be a Senator or Representative in Congress, or elector of President and
Vice-President ... having previously taken an oath . to support the Constitution of the United States, [who has] engaged in insurrection or rebellion against the same, or given aid or comfort to the enemies
thereof."

A. L. "Steve" Nash, MAC Ret, UDT/SEAL SEAL Authentication Team -Director AuthentiSEAL Phone 707 438 0120 "The only service where all investigators
are US

tomder55 answered on 09/02/04:

In 1996, a left-wing news service raised questions about two small "V" clips that the chief of Naval operations wore over two of the medals on his chest full of them. The clips are awarded for valor under fire, and there was some doubt about whether Boorda's two tours in Vietnam aboard combat ships qualified him for the awards, although the Washington Post reported that a 1965 Navy manual appeared to support Boorda's right to wear the clips. Unlike Kerry, the awards did not provide grounds for Boorda to shorten his tours of duty. Hours before he was scheduled to meet with Newsweek reporters to discuss the controversy, the admiral went to his home at the Navy Yard and shot himself in the chest. The CNO had been in command of the Navy during a troubled period and his leadership was being criticized by its senior officers. Still, among the notes he left was one to "the sailors" expressing his fear that the controversy over his decorations might harm the Navy. Boorda had lied about his age to join the Navy and was the first CNO to rise through the enlisted ranks.

What did John Kerry have to say at the time about the matter? Is it wrong? Yes, it is very wrong. Sufficient to question his leadership position? The answer is yes, which he clearly understood, said Sen. John Kerry.Citing uncertainty of whether Boorda deliberately wore the pins improperly, Kerry added: If he made a mistake, in my judgment it wasn't worth his life, so I'm very sad about it.

The Boston Globe for the same day:

The military is a rigorous culture that places a high premium on battlefield accomplishment, said Sen. John F. Kerry, who received numerous decorations, including a Bronze Star with a "V" pin, as a Navy lieutenant in Vietnam.

In a sense, there's nothing that says more about your career than when you fought, where you fought and how you fought, Kerry said.

If you wind up being less than what youre pretending to be, there is a major confrontation with value and self-esteem and your sense of how others view you.

Of Boorda and his apparent violation, Kerry said: When you are the chief of them all, it has to weigh even more heavily.(http://www.620wtmj.com/620programs/charliesykes/weblog.asp?id=8&entry=4008)




Toronto Blue Jays manager Tim Johnson fired up his baseball teams with bloody tales of his days as a U.S. Marine in Vietnam. The truth was that he had been in the Marine Reserves. An exemption for baseball players had kept him out of combat. The Blue Jays fired Johnson.

Brian Dennehy said he served five years in Vietnam. He'd been hit by shrapnel. Combat, he told Playboy magazine, was "absolute f---ing chaos." Dennehy had been a Marine, but his only overseas assignment had been as a football player on a service team in Okinawa.

"Stolen Valor
How the Vietnam Generation Was Robbed of Its Heroes and Its History"
B.G. Burkett and Glenna Whitley

Over the past 15 years, Burkett has investigated perhaps 2,000 claims of military service; at least 1,500 of them were bogus in one way or another.People should be skeptical, he said, about those who wear heroism on their sleeves, tell wild tales of wartime mayhem or claim to have led clandestine missions that the government won't acknowledge.






kindj rated this answer Excellent or Above Average Answer

Question/Answer
VisionsInBlue asked on 08/30/04 - Donations for Michael Moore... HERE:

He looks like he is in desperate need of:

a) soap (both to shower and wash his mouth)
b) Crest Whitening Strips and toothpaste
c) laundry detergent and clothes (other than the usual garbage can find)
d) razor blades (for shaving... I would never suggest anything else... Would I?)
e) lots and lots and lots of hamburgers, he looks starved
f) new contact lenses, hearing aids and Gingko

Please make check payable to Fattenheit 350 pounds, Second Cardboard Box, Third row under the B'klyn Bridge, New York. Don't use dollars. The man doesn't like anything American. French or Iraqi currency best.

How much are you sending? Thank you. ;)

tomder55 answered on 08/31/04:

"[The choice of going to war in Iraq] was between war and a graver threat. Don't let anyone tell you otherwise. Not our critics abroad. Not our political opponents.

And certainly not a disingenuous film maker who would have us believe that Saddam's Iraq was an oasis of peace when in fact it was a place of indescribable cruelty "
(John McCain last night)

kindj rated this answer Excellent or Above Average Answer
VisionsInBlue rated this answer Excellent or Above Average Answer

Question/Answer
Chouxxx asked on 08/30/04 - Guerrilla War revisited!

Since I was clear as mud on my last question regarding guerrilla war, let me restate.

We, Americans, are occuping Afghanistan and Iraq, and "GUERRILLAS" (non-traditional fighting forces) are fighting against us. They want us out fighting non-traditionally as Castro fought Batista in Cuba in the fifties.

Do our US Army soldiers have much of a chance fighting against these guerrills forces in the long run? Isn't it better going forward to use intelligence and undercover agents(spys, assassins, etc) to make headway against terrorist groups? (Hopefully the questions is better stated!)

Thaks,

tomder55 answered on 08/31/04:


What Donald Rumsfeld has been trying to prepare the country for since before he became head of DOD was the realignment of our capabilities to deal with
asymmetric warfare(defined as adversaries of inferior strength find a way to exploit vulnerabilities and achieve a horrifically disproportionate impact).
He said in a speech at Andrews Air Base ;"This is a global war," "It's unlike any other that America has ever fought. In Afghanistan and elsewhere around the world, U.S. fighting forces, together with our coalition partners, are breaking new ground. They're using new technologies in entirely new ways and proving once again that the United States can and will adapt to meet any challenge to peace and to freedom."

In the weeks prior to 9-11-01 he had prepared a Quadrennial Defense Review .It was published in Nov. 2001 asummary is here. The attacks on America forced the U.S. to move rapidly towards some of his recommendations.He set out to radically transform the military.Of course he met some opposition in the higher ranks as he shuffled the established structure but he has moved steadfastly forward in his quest to streamline the military and make it more mobile. His critics who call for his dismissal have yet to propose an alternate policy for running the military . Many of them entrenched in the cold war mentality or even in the Powell doctrine are not looking at the reality .

American forces are being trained to fight n an urban environment and have shown themselves capable. They have developed methods to detect the roadside IEDs and the incidents of casualties by them have decreased. At any case it is easier to fight a gorilla fight in thistype of environment than in a triple canopied jungle.We are training an Iraqi force that began to show some capability in the recent Najaf fight. Eventually they will depend less and less on American fire-power to perform the task of securing their country.

Human intelligence on the ground is vital and our assets on the ground grow daily . We are actively recruiting and training Iraqis and Afghanis to act as our eyes and ears. Our intelligence capabilities took a hit in the 90s with the Torricelli Principle(named after that moron Sen. from N.J.)which prohibited the use of intelligence sources with criminal records. It was inspired by the discovery that human rights violators were working as paid CIA informants . The effect was a decrease in both the quantity and quality of intelligence information gathered from many foreign countries. I believe this idiocy has been reversed ;possibly by executive order.

So to sum it up; spys are for intelligence gathering ,and soldiers are for fighting .Asymmetric warfare is a tactic that trained troops can counter .


Chouxxx rated this answer Excellent or Above Average Answer
purplewings rated this answer Excellent or Above Average Answer

Question/Answer
kindj asked on 08/30/04 - For the Tolkein fans...

MINAS TIRITH (Gondor News Network) - Thousands of peace activists took to the streets of Minas Tirith and other cities of Middle Earth today to protest what they termed a rush to war with Mordor.

"We need more time for diplomacy," said a key member of the Middle-Earth Security Council, Saruman the White. "I am not convinced by the evidence presented by my esteemed colleague, Gandalf the Grey, or that the Dark Lord Sauron presents an imminent danger to the peoples of the West."

Many of the people protesting war in Mordor agreed with Saruman's remarks. "Sauron says he's destroyed his Rings of Mass Destruction (RMD) and that's good enough for me," said one fellow carrying a sign that said "Elrond is a Balrog." Another demonstrator urged, "Give the RMD inspectors more time. There's no reason to rush to any judgment just because Mount Doom is belching lava, the Dark Tower is rebuilt, and Osgiliath has been decimated." A third protester piped up, "I haven't heard a single bit of convincing evidence connecting the Nazgul with Sauron. I think they destroyed Osgiliath on their own initiative without any support from Sauron. Besides, it's understandable they're angry with Gondor. We haven't done nearly as much for the Orcs and Goblins and Easterlings as the Nazgul and Sauron have. It's understandable they throw their support to them. It's our own fault really."

As the protesters continued their march through the city, they chanted, "No blood for Mount Doom," voicing a common sentiment that the leaders of the Western peoples are really seeking to get their hands on the powerful Mount Doom, where the One Ring of Power was allegedly forged.

Gandalf the Grey was unavailable for comment. A spokesman said he was in an undisclosed underground location, which sources have revealed is codenamed "Moria."

tomder55 answered on 08/30/04:

Lets not forget that the entire war is a fabrication by Aragorn to make up for his familys earlier mistake! No War for Isildurs Folly!

ETWolverine rated this answer Excellent or Above Average Answer
Itsdb rated this answer Excellent or Above Average Answer
kindj rated this answer Excellent or Above Average Answer

Question/Answer
kindj asked on 08/30/04 - Another request for thoughts

It was put to me that if we REALLY wanted to combat terrorism, we should've invaded Saudi Arabia instead of Iraq.

Of course, at this point we'd been joined by a couple of other people, "armchair quarterbacks" at their finest. They whined and screeched that the Saudis were our ALLIES, and our only friend in the region. Afer the Colonel and I quit laughing, we informed them that the only nation that comes close to being our friend in the region is Israel, and they're busy looking out for themselves right now, and righfully so. Both of us having been to Saudi and having stayed there for some time, we told the other guests that the Saudis are NOT our "allies," and never have been. A mutual enemy does not an ally make. They will shake your hand with their right hand while using their left to put a knife in your back.

Do you agree with this gentleman who said we should've invaded SA if we were really serious about fighing terror?

DK

tomder55 answered on 08/30/04:

I do not think so ;not at this time. we were fully aware of the Saudi connection to terrorist funding ,and their responsibility of coddling of Wahabbiism.Their seeming lack of cooperation with investigations of the Kobar Towers led me to believed that we would eventually have to confront the House of Saud.

Recently with the Saudis coming under direct attack from Al-Qaeda they have begun to realize the Frankenstein they created. The Washington Post had an article in June called: "For Saudi Arabia, Al Qaeda Threat Is Now Hitting Home Kingdom Itself Seen as a Target "(June,8).In it they say that "The brewing conflict between the government and the militants has forced many people here to reassess where they stand. In a nation where large segments of society support native son Osama bin Laden's efforts to destroy the United States and its Western allies, mainstream Saudis who cheered him are starting to realize that the government bin Laden and his followers really wanted to topple all along was their own."The article goes on to say that the House of Saud now seems very committed to fighting terrorism inside Saudi Arabia.

It remains to be seen how committed they are . Will they turn off the spiget of "charity"funding that flows to the terrorists ? Will they reform the madrasses educational system that is at the core of the recruitment drive for new radicals? Will they initiate internal reforms to bring their society out of the dark ages ? Will they hold onto power ,even as these reforms are designed to eventually replace their decrepit middle ages Monarchy?

For the moment the Saudis do not represent an immediate threat to us . Their cooperation leads me to hope that like Libya ;they have apparently seen the light.

ETWolverine rated this answer Excellent or Above Average Answer
excon rated this answer Excellent or Above Average Answer
kindj rated this answer Excellent or Above Average Answer

Question/Answer
Chouxxx asked on 08/27/04 - Malfeasance or Miscalculation

President Bush confessed today that his administration made a miscalculation in handling the War in Iraq. When the US went into Baghdad and overthrew Saddam's government, the soldiers just went home and otherwise blended into the landscape along with their weapons. Only to reappear later as a guerrilla army along with foreign religious fighters. And so it is now.

HOWEVER; did the administration IGNORE the War in Afghanistan? How the fighters just disappeared into the hills and hovels as did their weapons when the American soldiers came close to victory in minor fights?? THAT EVIDENCE WAS ONLY A FEW MONTHS PRIOR TO THE INVASION OF IRAQ.

Did the administration ignore the evidence right in front of their faces?? They didn't consider this as a likely alternative??? I want some answers and so do the American people.

tomder55 answered on 08/30/04:

These were not the same at all. In Afghanistan the lesson probably that should've been learned was that the U.S. could not necessarily count on surrogate forces like the Northern Alliance and other war-lords to do the critical fighting . If an objective like the surrender of the al-Qaeda forces in places like Tora Bora was to be achieved then it became clear that the U.S. forces needed to be used to complete the objective. The lessons were to know when to work with allied forces and when to go it alone. All it took was bin Laden handing out some cash to some tribal leaders to assure that the issue at Tora Bora would not be pressed and that his escape routes to Pakistan would remain open.

As with the Afghan campaign ;the military has performed well .But also as in the case of Afghanistan when we have counted on surrogate forces we have fallen short of our objectives. This is clearly the case in Fallujah ,where we stopped short of total victory ,and handed over the battle to a Sunni former Saddam commander . Fallujah as a result still remains a hot bed of insurrection . To a lesser extent the recent battles against the al-Sadr forces have reaped the same results .

Chouxxx rated this answer Excellent or Above Average Answer

Question/Answer
Chouxxx asked on 08/28/04 - Moderate Americans

The vast number of Americans and American voters call themselves Moderates. When questioned in a recent survey, they labeled their positions as Moderate the policies enacted by the Liberals years ago. Such issues as the government's responsibility to protect citizens and others from pollution of all kinds, for example. The Liberal heritage is now the mainstay of the Moderate Americans, fully accepted and desired.

tomder55 answered on 08/29/04:

I don't know if it is fully accepted ar desired ,but it is true that provisions of the liberal agenda once considered radical ,like Social Security,are now considered almost untouchable; Even if reform is badly needed.It seems the only reform accepted is adding on to it with stuff like the disasterous Senior Drug program.Friday Alan Greenspan again warned Congress that they have promised more payback than the nation can deliver.He Said "As a nation, we owe it to our retirees to promise only the benefits that can be delivered."

But as usual his wise warnings were met with a barrage of criticsm.They will never make meaningful S.S. reform .Why? Because it would deny them the cash from the reserve fund they regularly plunder to fund other liberal agendas and pet pork projects.

But I have to agree with you ;once irresponsible programs become codified as entitlements it is virtually impossible to even reform them let alone eliminate them.

Chouxxx rated this answer Excellent or Above Average Answer

Question/Answer
excon asked on 08/28/04 - Patriot Act Hypothetical


Hello again, Wingers:

Let me pose this outrageously hypothetical question. Let's say that a very liberal contingent was running the country. Let's say that this liberal contingent perceived that there were certain "undesirable" people on the far right. Let's say that rightly or wrongly, some group you belong to, or some magazine you subscribe to, got you put on a government watch list. Let's say the FBI visited you because it "heard" you were going to the Democratic Convention, and wanted to innocently ask you about it.

Let's say that during this administration, we got attacked. Let's say that this administration asked for the power to investigate and deal with "wrong" wherever they find it. Would you be opposed to giving that government the Patriot Act?

excon

tomder55 answered on 08/29/04:

I'm in deep then because my net and library records show that I access info. from all types of sources .My web page links with al-jazzera and Ihave read both 'Mein Kampf' and 'The Communist Manifesto'(and 'The Palistinian Charter ').I have traveled to Iran .Am I concerned about being rounded up? No. I will keep on saying this;The Patriot Act simply codified the authority law enforcement had already had for decades. this talk of being secretly rounded up just is not happening.Survaillance still requires a judicial approval.

excon rated this answer Excellent or Above Average Answer

Question/Answer
excon asked on 08/27/04 - Commie, Pincko crap from the ACLU


Hello Sperts:

Think the Patriot Act is just for terrorists?

http://www.aclu.org/pizza/images/screen.swf

Right.....

excon

tomder55 answered on 08/27/04:

for a minute there I thought you were talking about the ACLU.

looks like they've gone off the deep end.

Still waiting for them to identify one person who's rights have been violated by the provisions of the Patriot Act .Sen. Dianne Feinstein (D) of California said: "I have never had a single abuse of the Patriot Act reported to me. My staff asked the ACLU for instances of actual abuses. They said they had none." But that doesn't stop them from showing extreme alarmist video like this . The only thing that suprised me is that there was not a solicitation for money at the end of the presentation.

excon rated this answer Excellent or Above Average Answer
kindj rated this answer Excellent or Above Average Answer

Question/Answer
ETWolverine asked on 08/27/04 - Kerry the hypocrit

Once again, Kerry has shown his massive hypocricy.

Yesterday, George Bush, in response to his critics, joined John McCain to say that he intended to sue the 527 organizations to get them to stop their advertizing. This included ALL 527s, including those pro-Bush and those anti-Bush. He called on John Kerry to join the suit.

John Kerry refused.

After all his bluster about the Swift Vets, and all his demands for Bush to condemn the Swift Vets as an outsider 527, he doesn't have the cojones to step up with Bush to get rid of the 527.

Wanna know why?

It seems that the pro-Kerry 527s are raising about 5 times as much money as the pro-Bush 527s are, and so they benefit Kerry to a greater degree than they benefit Bush.

So out goes Kerry's call for an end to 527s from earlier this week, and in comes the hypocracy. In fact, Kerry never really wanted an end to the 527s... he just wanted an end to the PRO-BUSH 527s like the Swift Vets.

Hypocrit.

Elliot

tomder55 answered on 08/27/04:


The facts support this reasoning .

According to the Center for Responsive Politics[A non-partisan, non-profit research group based in Washington, D.C. that tracks money in politics and its effect on elections and public policy. ] 21 of the 22 individuals who have contributed at least $500,000 to 527s have donated a cumulative $56.7 million to Democrat groups . Peter Lewis ($14 million), George Soros ($12.6 million) and Steven Bing ($8.1 million).


Only one of the major donors has supported 527s with a conservative message -Carl Lindner ($1.02 million).

http://www.opensecrets.org/527s/527indivs.asp?cycle=2004

ETWolverine rated this answer Excellent or Above Average Answer

Question/Answer
kindj asked on 08/27/04 - Alice Cooper and his faith

Regarding what we discussed on the board, here's one of the stories, and I've put a link to another one at the end:

Alice Cooper goes with God
'Shock-rock' pioneer now 'dedicated to follow Christ'

--------------------------------------------------------------------------------
Posted: March 1, 2002
1:00 a.m. Eastern



2002 Charisma News Service


Alice Cooper says he loves God but doesn't want to become a 'celebrity' believer.

The father of shock rock whose music and stage antics have outraged parents for more than 30 years told how the fear of hell turned him to God.

Alice Cooper, who sang about necrophilia and chopped up baby dolls during his concerts, said that although he continues to record and tour with a theatrical horror-style show, "My life is dedicated to follow Christ."

Although he became a Christian in the 1980s, apart from brief comments in some interviews the 54-year-old singer has always been guarded about his faith until now. But in a frank interview with a Christian music magazine, he spoke at length publicly for the first time about his love for God and reluctance to become a "Christian celebrity."

A chart-topper with the teen anthem "School's Out," Cooper who legally changed his name from Vince Furnier has been credited with paving the way for the likes of today's outrageous performers such as Marilyn Manson.


Cover of Cooper's latest album, DragonTown

But he maintains that his act was never political or religious and always had "a sense of humor." He told HM: The Hard Music Magazine that he was always insulted whenever he was accused of being satanic. Raised in a Christian home, he still believed in God, although he was not committed.

That changed when alcoholism threatened his marriage. He and his wife, Sheryl, attended a church with a "hellfire pastor." Cooper said he became a Christian "initially more out of the fear of God, rather than the love of God ... I did not want to go to hell." Interviewed for HM's March/April issue, Cooper views his faith as "an ongoing thing."

"Being a Christian is something you just progress in. You learn. You go to your Bible studies. You pray," he said.

He has avoided "celebrity Christianity," because "it's really easy to focus on Alice Cooper and not on Christ. I'm a rock singer. I'm nothing more than that. I'm not a philosopher. I consider myself low on the totem pole of knowledgeable Christians. So, don't look for answers from me."

Yet he has been able to speak to others in the music scene about his faith.

"I've had a couple of people that were friends of mine that I've talked to that have vocally said they have [accepted Christ]. I have talked to some big stars about this, some really horrific characters ... and you'd be surprised. The ones that you would think are the furthest gone are the ones that are more apt to listen."

Songs on Cooper's more recent recordings have hinted at his change of heart. He sees his stage persona now as "the prophet of doom," telling people: "'Be careful! Satan is not a myth. Don't sit around pretending like Satan is just a joke.' I think my job is to warn about Satan."

He no longer performs some of his older repertoire. Any song promoting promiscuous sex and drinking "gets the axe," he said. "I'm very careful about what the lyrics are. I tried to write songs that were equally as good, only with a better message."

Cooper told HM he answers his critics: "'I was one thing at one time, and I'm something new. I'm a new creature now. Don't judge Alice by what he used to be. Praise God for what I am now.'"

Copies of the Cooper interview can be ordered through HM Magazine's website.



Another related story:

http://www.jesusjournal.com/jj_testimony/cooper.html


DK

tomder55 answered on 08/27/04:

thanks for the links . I did not know that.

I always recognized his act as something not to be taken seriously .There was as much evil in it as an Edgar Allen Poe novel.

I am always happy to hear of someone being able to kick a dependency habit .

I have not listened to any of his recent recordings ;maybe I should check em out .

kindj rated this answer Excellent or Above Average Answer

Question/Answer
ETWolverine asked on 08/26/04 - Presidential Debates

It seems that Kerry has challenged Bush to a debate every week from now until the election.

Which makes sense, since that is how often Kerry changes his positions on every issue.

Elliot

tomder55 answered on 08/26/04:

if it could be arranged it would be a good thing. I wonder if the networks would suspend their precious reality shows long enough to air them .

I would love to see a national debate on the war against Islamo-fascists . In fact I would love to see at least one debate just on 'World War IV: How It Started, What It Means, and Why We Have to Win 'by Norman Podhoretz? in this months Commentary Magazine. (It should be required reading by everone on this board).

The question for the debate is found near the end of the essay:

"what will happen if the Democrats behind John Kerry defeat George W. Bush in November? Will they follow through on their violent denunciations of Bushs policy, or will they, like the Republicans of 1952 with respect to Korea, quietly forget their campaign promises of reliance on the UN and the Europeans, and continue on much the same course as Bush has followed in Iraq? And looking beyond Iraq itself, will they do unto the Bush Doctrine as the Republicans of 1952 did unto the Truman Doctrine?{continuity} Will they treat Iraq as only one battle in the larger warWorld War IVinto which 9/11 plunged us? Will they resolve to go on fighting that war with the strategy adumbrated by the Bush Doctrine, and for as long as it may take to win it?

From the way the Democrats have been acting and speaking, I fear that the answer is no. Nor was I reassured by the flamboyant display of hawkishness they put on at their national convention in July. Yet as a passionate supporter of the Bush Doctrine I pray that I am wrong about this. If John Kerry should become our next President, and he may, it would be a great calamity if he were to abandon the Bush Doctrine in favor of the law-enforcement approach through which we dealt so ineffectually with terrorism before 9/11, while leaving the rest to those weakest of reeds, the UN and the Europeans. No matter how he might dress up such a shift, it wouldrightlybe interpreted by our enemies as a craven retreat, and dire consequences would ensue. Once again the despotisms of the Middle East would feel free to offer sanctuary and launching pads to Islamic terrorists; once again these terrorists would have the confidence to attack usand this time on an infinitely greater scale than before."



darkstar rated this answer Excellent or Above Average Answer
ETWolverine rated this answer Excellent or Above Average Answer
Itsdb rated this answer Excellent or Above Average Answer
purplewings rated this answer Excellent or Above Average Answer

Question/Answer
Itsdb asked on 08/25/04 - Love that media spin...

The Associated Press put some nice spin on the 527 ad wars on the front page of my local paper.

They report the top groups in this presidential battle as:

Pro-Democratic:

1. Media Fund: $28.1 million.

2. ACT NOW PAC, also known as America Coming Together: $26.9 million.

Pro-Republican:

1. Republican Governors Association: $32.5 million.

2. The Club for Growth: $8.5 million.

Well, they got the first part right...but the top pro-Republican 527 in this election is The Club for Growth. The Republican Governors Association's primary mission is electing Republican governors, and according to Publicintegrity.org has expenditures of $24,474,202 since the year 2000, with a whopping $2,938,033 spent this year, none of which went to smearing Kerry in this election cycle.

The AP implies the RGA is the top soft money spender in the anti-Kerry game, yet omits any reference to the Democratic Governor's Association. The press hammers on Bob Perry's $200,000 in contributions to the Swift Boat Veterans but ignores the $26,630,000 contributed by Peter Lewis and George Soros alone to defeat Bush. The true top pro-Republican player in the presidential election, Club for Growth is number 7 on the list of top 50 committees, $62,086,165 behind pro-Democrat Media Fund, ACT, and MoveOn.org in spending according to opensecrets.org.

Will Kerry join in condemning all soft money ads, or will he continue to try to silence only those that aren't favorable to him? What should Bush tell Cleland when he tries to present his letter to Bush calling on him to stop the Swifvet ads? Will Kerry stop the 15 upcoming MoveOn ads? Who's spinning these ad wars, me or the left and a friendly media? Does the truth matter any more?

tomder55 answered on 08/25/04:

I have done a flip flop on the whole campaign finance thing. I now see McCain -Feingold as a disaster.

Both campaigns now call for the end of ads by PACs ,special interests, and outside groups .The media;those bastions of 1st amendment rights have no problem with the concept of suppression of political speech but God forbid someone try to suggest that obscenity be curbed. In their view it is the issue ads that are the obscenity.

To their credit ,the Swifites have told Bush that they have no intention of pulling their ads .

Robert J. Samuelson of the Washington Post got it right today when he wrote :"We've arrived at this juncture because it's logically impossible both to honor the First Amendment and to regulate campaign finance effectively. We can do one or the other -- but not both. Unfortunately, Congress and the Supreme Court won't admit the choice. The result is the worst of both worlds. We gut the First Amendment and don't effectively regulate campaign finance.("http://www.washingtonpost.com/wp-dyn/articles/A30280-2004Aug24.html)

That is the bottom line. I thought the growth of money and it's influence was corrupting the political process .Now I see that there are bigger dangers and the cure is worse than the illness.


ETWolverine rated this answer Excellent or Above Average Answer
Itsdb rated this answer Excellent or Above Average Answer

Question/Answer
Chouxxx asked on 08/24/04 - Republican Convention

I have heard interviews particularly on Hannity radio show with an anti-war activist, that there will be a lot of "action" in the streets in New York next week while the Republican convention is in town. No matter if the protestors are cordoned(sp)-off to special areas by the police. Aslo, many thousands of protestors representing all kinds of interest groups such as anti-war and environmental I guess, will join forces.

How do you think mayhem in the streets will effect the average voters this year, if it is televised. Do you think the American media will cover the protesters?

Thanks,

tomder55 answered on 08/25/04:

An anarchist group calledA-31 has discussed forming blockades and disrupting order outside the offices of Citigroup, the Carlyle Group, the Rand Corporation, and Hummer of Manhattan, as well as a meeting of Bank of America executives at Tavern on the Green. Other radicals have refused to rule out attacks on property.

The police have been conducting preparations with power saws and bolt cutters to separate protesters who may chain themselves together.

Imagine the reaction in the media, and among Boston government officials, if groups of conservative protesters had announced their intention to "disrupt" the business of the city during the Democratic convention last month.

Mayor Bloomberg(no Rudy Guilani) has come up with a different response: he is giving out discount peaceful protesterbuttons for use at local restaurants, stores, and Broadway shows, to all protesters who promise not to be violent. He said : It's no fun to protest on an empty stomach." Yeah .thats the ticket ! Send them to the Carnagie Deli ,and for a show . If tickets for the Producers are not available perhaps theyd prefer to see "I'm Gonna Kill the President!" A Federal Offense.

But the protesters do not want discounts ,they want Central Park .The city has offered a march past Madison Square Garden and large part of West Side Highway to hold their protest rallies ,but that does not satisfy them .
United for Peace and Justice the largest group of protesters has been in court to demand that they be given access to the park.
Leslie Cagan
; leader of the group says that if the courts rule in favor of the city ,then they probably would do the march and then not hold a rally but instead would migrate towards Central Park as private citizens .(more on Cagan) The courts have denies other groups access but there may be problems in denying this group . Central Park can accommodate the numbers of people that are being suggested ,and it would be easy for the police to contain a crowd there ,;but there are other factors that come into the decision. The City has spent millions ,and continues to do so for the upkeep of the park . Since the 1980s the lawn and its surroundings have become magnificent !

This seems to be at the heart of Bloombergs objection. He is not someone who naturally stifles protest. He has been followed by protesters from the N.Y.C police dept. for weeks now over a salary dispute.They have protested around the clock ,chanting late into the night in front of his residence. But he feels in the case of the protesters there will be an easier time providing security if the rally was staged at a location other than the park.

No problem for the DNC to put protestors behind 10 ft.,razor wire topped, cages in Boston. Protesters at the Democratic National Convention in Boston were confined to a "free-speech zone" that resembled an exercise yard inside a prison. Somehow they managed to have peaceful protests there in spite of the fact that few in the press seemed to notice them. I assure you ;whatever protest occurs ,and where it happens ,a full contingent of the press will report it and will pray for a reenactment of the 1968 Chicago Democrat convention.

Chouxxx rated this answer Excellent or Above Average Answer
ETWolverine rated this answer Excellent or Above Average Answer
Yiddishkeit rated this answer Excellent or Above Average Answer

Question/Answer
HANK1 asked on 08/24/04 - Muslims!



The 19 men who US officials say hijacked four American passenger jets and flew them on suicide missions that left more than 7,000 people dead or missing were all from the Middle East. Most of the hijackers have been identified as Muslims.

The vast majority of Muslims in the Middle East were as shocked and horrified as any American by what they saw happening on their TV screens. And they are frightened of being lumped together in the popular American imagination with the perpetrators of the attack.

But from Jakarta to Cairo, Muslims and Arabs say that on reflection, they are not surprised by it. And they do not share Mr. Bush's view that the perpetrators did what they did because "they hate our freedoms."

Is this the real reason?

HANK

tomder55 answered on 08/25/04:

Hank ,read this from the Salafi Society of NORTH AMERICA.It explains alot.

That is the primary reason. The secondary reason why bin Laden turned on the U.S. is because he had gone to the Saudi gvt. with a proposal to send his Mujahadin to Saudi to defend it from the Iraqis in 1990 . The Saudis opted instead to invite the U.S. military into the country . He hated the fac t that the U.S. defeated Iraq so easy ,and that the Saudis invited the U.S. to maintain bases on Saudi soil .Bin Laden became angered and planned moves against the Saudis . They found out and expelled him to Somalia . The rest is history .

The truth is that while many in the Islam world expressed shock at the attack they secretely approved of it. The problem is not that OBL believes that this is a religious war against America. It's that millions of people across the Islamic world agree.

HANK1 rated this answer Excellent or Above Average Answer

Question/Answer
ETWolverine asked on 08/24/04 - Alright, Democrats. You wanted it, you got it.

Yesterday, Bush denounced the Swift Vet ads.

"I'm denouncing all the stuff being on TV of the 527s. That means that ad and every other ad," Bush said yesterday. "I think Sen. Kerry served admirably, and he ought to be - he ought to be proud of his record. But the question is, who is best to lead the country in the war on terror"

The Swift Vets, however, are vowing to continue to air the ads. Adm. Roy Hoffman said, "We have no intention of pulling our ads. We feel that we earned the right to be heard by our combat. It would make no difference if Kerry were a Republican, Democrat or Independent." When asked why the 264 Swift Vets dislike Kerry so much, Hoffman said: "When he came back, he told people that we were murderers who committed untold atrocities."

The more Kerry complains about the Swifties, the more chances Hoffman, O'Neil and others get to say things like that to the press.

Meanwhile Kerry has been using McCain's name without his permission in an attack ad against the Swift Vets, and that too is getting press time. McCain is NOT happy with Kerry, and is making that well-known to the press.

Now... all you Democrats that have called for Bush to denounce the Swift ads... you have your wish.

And it changes nothing at all.

The questions about Kerry's service are out there. People are questioning Kerry's voracity, his truthfulness, not just on the issue of his service but on EVERY issue. The Swift Ads are still being aired, and new ones are on the way. And now that Bush has given Kerry what he asked for in terms of denouncing the ads, Bush has taken away Kerry's biggest piece of short-term ammunition; the accusation that Bush supported the Swift ads. So the Swift ads are still airing, Kerry is still suffering from them, his truthfulness is in question, and Kerry has nothing to hit Bush back with on this issue. So what is Kerry going to do about it that won't make him look either desparate to bury the truth or a rabid attack dog?

Still think Bush is stupid? I think he's a brilliant political thinker.

Elliot

tomder55 answered on 08/24/04:

Most likely the major media would've ignored the Swifities had Kerry himself not over reacted to the ads. Alison Mitchell, of The New York Times said :"I'm not sure that in an era of no-cable television we would even have looked into it" .Washington Post Executive Editor Leonard Downie Jr. said "There is much more media, but we still judge for ourselves which facts we report in The Washington Post." They would prefer to underplay the adds . But as Kerry made his 4 month tour a centerpiece of his campaign ;and invited te scrutiny(remember "Bring it on") they have had little choice but to keep the story alive.

The real problem that the major media faces today is that there are many other choices for an informed public to get information . Like the Lewinsky scandal in the Clinton years ; The information about the discrepencies in Kerry's record were well known by folks who follow the news on the net .This means that the 4th estate cannot control the information as they once did .

I am one to think that Kerry probably served his time well enough ,but since more often than not wrote his own after action reports was inclined to embellish his record a tad.
Mickey Kaus in MSNBC observes :

"The problem is that Kerry is running for president on this official hype of a more-than-honorable record (one reason he's constantly referring reporters to his official medal citations). He's not only running on the hype but pushing it to the limit, milking it for all it's worth. That's dangerous in, yes, the Internet era! Obsessive fact-checkers can smoke out the exaggerations and get them past the ex-gatekeepers.** Unfortunately, it's more or less all Kerry's got. It wouldn't be so important if Kerry had a) a discernable ideology; b) a political message; c) a record of achievement; or d) an appealing personality! "

The real problem for Kerry is not that the Swifties are calling him on whether a sliver of shrapnel qualifies for a purple heart ;or if fishing a shipmate out of a river qualifies for a silver star(LBJ once got one for being an observer during a non-combat plane ride).The big problems for Kerry will have come with the latest Swiftie ad.;the one that questions his actions after his time in Nam.Can Kerry seriously object to quoting the speech he made to launched his political career? As Mark Steyn of the Daily Telegraph observes :"What sort of idiot would make the centrepiece of his presidential campaign four months of proud service in a war he's best known for opposing?........How cocooned from reality do you have to be to think you can transform one of the most divisive periods in American history in which you were largely responsible for much of the divisiveness into a sappy, happy-clappy, soft-focus patriotic blur without anybody objecting? Most Vietnam veterans of my acquaintance loathe John Kerry, and, if he wasn't aware of that, he's too out of it to be President."



ETWolverine rated this answer Excellent or Above Average Answer

Question/Answer
kindj asked on 08/20/04 - Judicial Watch Calls for Investigation

Aug 18, 2004 Contact: Press Office
202-646-5172


Judicial Watch Calls For Investigation Into Kerrys Medals, Anti-War Actions

Formal Complaint Filed Over Senators Vietnam Awards, Post-Service Activities




(Washington, D.C.) Judicial Watch, the public interest group that investigates and prosecutes government corruption, today filed a request with the U.S. Navy and the Defense Department for an investigation into the awards granted to Sen. John Kerry during his service with the U.S. Navy in Vietnam. Judicial Watch also requested that military authorities investigate Kerrys anti-war activities, including his meeting with North Vietnamese and Viet Cong delegations in Paris, while a member of the Naval Reserve.



Basing its requests on a recently published book, Unfit for Command, by former Navy officer John E. ONeill and Jerome R. Corsi, Ph.D, and on news media interviews of other officers and sailors who served with Kerry, Judicial Watch notes that unresolved allegations against Kerry include: false official reports and statements; dishonorable conduct; aiding the enemy; dereliction of duty; misuse and abuse of U.S. government equipment and property; war crimes; and multiple violations of U.S. Navy regulations and directives, the Uniform Code of Military Justice and the U.S. Code.



Kerry was awarded three Purple Hearts, a Silver Star and a Bronze Star for wounds received and actions in Vietnam, but eyewitnesses refute his version of a number of the events that were the basis for receiving the commendations. Judicial Watch is asking the Department of Defense and the U.S. Navy, including its Department Board of Decorations and Medals, to look into the circumstances surrounding Kerrys awards.



Judicial Watch also is requesting an investigation of Kerrys anti-war activities. After he was released from active duty but while he was a commissioned officer in the inactive Naval Reserve, Kerry joined the anti-war group Vietnam Veterans Against the War and traveled to Paris to meet with delegations from North Vietnam and the Communist Viet Cong. He held a press conference in Washington, D.C., following the meeting and advocated the peace proposal, which included war damage reparations, put forth by the North Vietnamese and Viet Cong.



The allegations concerning Kerrys conduct during the Vietnam War are credible, serious and shocking, said Judicial Watch President Tom Fitton. The sooner an investigation begins, the better.

To see the text of the complaint, go to
http://www.judicialwatch.org/archive/2004/kerryawards.htm



tomder55 answered on 08/23/04:

the whole thing would be solved if Kerry were to exercise his release of his records by signing a Standard Form 180 . Then the public would see his complete record and decide for ourselves .

kindj rated this answer Excellent or Above Average Answer
Yiddishkeit rated this answer Excellent or Above Average Answer

Question/Answer
Chouxxx asked on 08/22/04 - My Decision 2004

Please help. I am a one issue voter in this election, and I have narrowed down my assessment of who would be the best President going forward, best proactive for carrying out what I consider the single most important issue.

Bush. I think his administration would be the most steadfast in the War on Terrorism despite the collapse of the CIA on his watch which I don't consider is fault, but a combination of stuff....despite all the hysterical supporters he caters to, like the Useful Idiots of the far right and religious right.

Kerry::: Have to have a leap of faith that he will continue the War on Terrorism even more stringently than Bush.

PS Every single President since 1980 has BETRAYED America and I'm sick over the office being used to pay off crony-s instead of providing leadership. SPECIFICALLY, I waited in gas lines in the late 1970"s when Saudi Arabia raised oil prices which was an attack of sorts on America. We pledged outselves to smaller engines, less gas consumptions and weening ourselves off Saudi and Middle East oil.

No worthwhile leadership from Reagan, Bush, Clinton, Bush 2. A COMPLETE TRAGEDY FOR AMERICA. AND NEEDLESS.

Still on the fence. What to do?

tomder55 answered on 08/23/04:

You bring up 2 issues ,so I will address them . The first being the war on terrorism. As you know I have previously made my case why Bush would be the best leader in the war ((my reply) .

Kerry recently made one of many blunders in his campaign by 1st campaigning as an anti-war candidate even though he voted for the war in Iraq ,and then saying that even if he knew what he knows now that would not have changed his vote. I think he alienated a lot of Deaniacks and they may be no shows or vote for Nader as a result .

He has also sent confusing signals recently about troop deployment. . He originally suggested that he was in favor a post cold war realignment from Europe and S.Korea. Then after Bush made the suggestion last week to do so Kerry criticized the idea as being hasty.

His biggest blunder recently was his suggesting a time line for the withdrawal of Americans from Iraq. He said that he hoped to begin reducing the number of U.S. forces in Iraq within six months of taking office . This was a terrible signal to send to our enemies . They know that if he is elected all they have to do is wait him out .



A look back at Kerrys Senate years would be a good barometer of his leadership qualities ( I do not want to talk about the Vietnam stuff ;he has damaged himself too much with that already).

The 1st WTC attack was in Feb. 1993 . Terrorists tried to topple the towers with a car bomb . They were not successful but there were 8 deaths and over 1000 other injuries . Kerry ;a prominent member of the Senate Intelligence Committee proposed in the aftermath of the attack in 1994 and 1995 to cut the budgets of the U.S. intelligence agencies by a combined 6 Billion dollars .His measures were soundly defeated .Most prominent members of the Senate Democrats recognized the folly of his proposals . Dennis DeConcini, chairman of the Senate's Select Committee on Intelligence, said in response ; We no longer seem immune from acts of terrorism in the United States." He said that it made "no sense for us to close our eyes and ears to developments around the world which could ultimately save U.S. lives and resources." DeConcini belived that the Intelligence Committee had already taken a "long hard look at what we are spending on intelligence."and determined that enough post cold war cuts had already been made(if Kerry werent so often absent maybe he wouldve known that ). Sen. Daniel Inoue, of Hawaii ;also a member of the Committee was highly critical of his proposals . In fact ;even Ted Kennedy voted against it.

Kerry often complains about the lack of human intelligence . Hey ;he is right . What did he do about it as a member of the Senate? Ummmmmm.. He proposed at least 3 budget cuts ,and no increases for the funding of extra intelligence assets . In 1997 ;when the threat of al-qaeda should have been on his mind he said : Why it is that our vast intelligence apparatus, built to sustain America in the long twilight struggle of the Cold War, continues to grow at an exponential rate? Now that that struggle is over, why is it that our vast intelligence apparatus continues to grow even as government resources for new and essential priorities fall far short of what is necessary?"


You make a good point of linking the war on terror with an overall comprehensive energy policy . On this issue I am not sure who would be better able to deal with the problems we face . Kerry has outlined a pretty decent plan on paper which calls for the development of alternatives . I can buy that ,but I think he and other members of his party short change the use of domestic resources we already have.

Bush would be wise to announce a comprehensive plan of his own in the next week ,and to take some immediate actions . He should announce that effective immediately he is going to stop the purchase of oil for the SOR at these inflated prices . Then he should release some of the SOR into the market 'as Bush 1 and Clinton did previously ,to stabilize prices that are the result of terrorism ,and market speculation .
He should in his policy announcement propose stricter compliance with CAFE and ask the Congress to reconsider exploratory drilling. This is a decent compromise ,and a good first start . I think these measures would address the immediate concern of higher crude pricing stalling the economic recovery . But one has to be blind not to see the long term trends. Forgetting the toll that terrorism is taking ,the reality is that emerging economic powerhouses in Asia will now be competitors with the U.S. on a dwindling supply of the resource. It is time to expidite the recovery of existing un-tapped supplies ;to maximize efficiency ;and to begin to develop with a greater sense of urgency the future energy supplies.If it were me ;everything would be on the table for discussion.

kindj rated this answer Excellent or Above Average Answer
Chouxxx rated this answer Excellent or Above Average Answer

Question/Answer
Chouxxx asked on 08/12/04 - All Out Siege?

A revered Islamic cleric who never left his house in the last 50 years just went to London for tests. Is this a sign that to the US that an all out siege on, what's his name, sorry, and his fighters, is ok with Islam? A sign they don't want them around?

tomder55 answered on 08/13/04:

I certainly think that Sistani has given his defacto approval of the assault. Even if the cover story of his needing medical treatment is true ,he has had plenty of opportunity since Sunday to weigh in on the seige. All he has done is make a plea to protect the Imam Ali Shrine .This is the equivalence as if the Nazi's had occupied St Peters in Rome during WWII and the Pope remained silent as the allies and the Nazis battled it out .He knows as well as everyone else that any damage to the Mosque will be from the hands of the Mookie Army.

It is telling that there are boundries that civilized people are reluctant to cross ;the Imam Ali Shrine ,the Church of the Nativity in Bethlahem ;where savages have no reluctance to exploit by creating killing zones inside sanctuaries.
During the 1991 uprising the Shia fled to the shrine and Saddam rolled up his tanks and blasted away.

The fighting in Najaf is different from April .Iraqi forces have shown no reluctance about reporting for duty and are reporedly fighting well. Sadr's efforts do not seem to be coordinated with Sunni insurgent forces at Fallujah either .

But although the dogs seemed yesterday to be unleashed . I am seeing reports today that US has again suspended offensive operations in Najaf short of the objective as this AP Report suggests .


This is disturbing to me but I have to look at this in the context of the new political situation in Iraq. Like it or not the final call is not in the hands of the U.S. forces but in the hands of P.M. Allawi . Here's hoping that he will order an assault on the shrine by Iraqi regulars or permits the US to fill it so full of tear gas that Sadr's thugs will be forced to flee . Allawi has already tried to hand a carrot to Mookie to participate in the process but al-Sadr has thumbed his nose at the offer instead urging his troops to fight to the death. Rumor has it that he was wounded a number of times yesterday .Maybe this convinced him that we are not fooling around.

Chouxxx rated this answer Excellent or Above Average Answer
ETWolverine rated this answer Excellent or Above Average Answer

Question/Answer
Itsdb asked on 08/11/04 - Kerry campaign news...

Here's the latest daily dispatch from the Kerry campaign:

"I'm interrupting John Kerry's dispatches from the road to tell you about something that will make you angry, but which is important for you to hear.

We knew it was coming: the Bush campaign and several allied right-wing groups are using August to launch a vicious smear attack against John Kerry. They think we're vulnerable this month because while George Bush is still able to raise money for attack ads, Kerry had to stop fundraising at the Democratic convention last week. They're taking this opportunity to go for the jugular.

We need you to join us in this fight today by supporting the Democratic Party.

https://www.democrats.org/support/kerry.html

The worst of the current wave of attacks is coming from a Republican-funded group called "Swift Boat Veterans for Truth." (I'll call them by a more accurate name: Swift Boat Veterans for Bush.) The group is led by a longtime Republican operative and financed by GOP contributors with strong ties to George Bush. Its function in the overall Republican strategy is this: tear down John Kerry since Bush has no record or vision to run on.

The swift boat ad is full of lies. Thirteen men who never served with John Kerry lie about knowing him and viciously attack his record. It is a new low for the Republicans.

Every time they have attacked us like this, it has only made us stronger. When they ran dishonest attack ads (like the unbelievable new one on the air right now), thousands of new supporters flocked to our campaign to give us the resources we need to fight back on the airwaves.

Now that the general election has begun, they're hoping things are different. They're going to be sorely disappointed because we are not alone. The Democratic National Committee is well prepared to take on this fight.

And therefore I'm asking you: everything you did for our campaign, please do now for our Democratic Party. Give the party the resources it needs to fight against these right-wing attacks:

https://www.democrats.org/support/kerry.html

The Democratic Party is responsible for putting organizers and volunteers into the field and turning out the millions of votes we need to win. We literally cannot win this election without the party.

https://www.democrats.org/support/kerry.html

You and I know that if we so much as yield an inch to George Bush and the Republicans this month, we'll live to regret it. In fact, we may live to regret it for four long years. Join with me in supporting the Democratic Party today.

Thank you,

Mary Beth Cahill
Campaign Manager"

How truthful is this email? What's true and what's not? Is this a part of the "positive campaign" that Kerry is allegedly running?

Steve

tomder55 answered on 08/12/04:

The Kerry Campaign has now admitted that Kerry's oft-repeated stories about being in Cambodia on Christmas Day, 1968 aren't true.

He is the one who brought up his service in Nam as a qualifier .I am suprised that he did so .He has been fighting off these charges for along time. Bush at least has the good sense not to try to emphasis his service years. But since he has made it a key part of his campaign .He should expect his record to be scrutinized.

He doesn't need it .All he had to do to prove he is a hero is to tell this story about how he saved U.S. Sen. Chic Hecht's life.

elgin_republicans rated this answer Excellent or Above Average Answer
ETWolverine rated this answer Excellent or Above Average Answer
Itsdb rated this answer Excellent or Above Average Answer

Question/Answer
SanchoPanza asked on 08/12/04 - Rapist serving life wins 7 million on lottery

While on temporary release from a bail hostel (a very open prison)a rapist who has already served 15 years won the British National Lottery. A row has broken out and the Home Secretary wants to introduce legislation that might ban prisoners on temporary release from participating or enforce them to make contributions from their winnings to victim compensation funds.

Critics are calling this a knee jerk reaction.

http://www.guardian.co.uk/lottery/story/0,7369,1280703,00.html

With his new found wealth he is considered more of a security risk and has been transfered to a more closed prison.

A case of locking the stable door?

tomder55 answered on 08/12/04:

Some of it should be contributed to victim funds and to the increased security that will now have to be provided .Other than that he should keep the balance.

Son of Sam ( David Berkowitz )was a mass murderer. While in prison there were rumors he would write a book about his murders .The law said that the state would confiscate any income that a person earned from accounts of their illegal deeds, and would put it into an escrow account. Victims of the crime could then claim the money by bringing a civil action.

In 1991, the U.S. Supreme Court found that New York's law violated the First Amendment because it singled out income from speech over other kinds of crime-related earnings.

In your case ,the prisoner got lucky . There was nothing prohibiting him from playing the lotteries. But if he merited temporary release and he committed no crime . He should be allowed to keep his prize.

excon rated this answer Excellent or Above Average Answer
SanchoPanza rated this answer Excellent or Above Average Answer

Question/Answer
ETWolverine asked on 08/11/04 - The flip-flops of John Kerry.

John Kerry's Flip Flops

Flip Flopped On Trade With China

In 1991, Kerry Supported Most-Favored Trade Status For China. Sen. John Kerry said yesterday that he is breaking party ranks to support most-favored-nation trade status for China I think the president has some strong arguments about some of the assets of most-favored-nation status for China, Kerry said. (John Aloysius Farrell, Kerry Breaks Party Ranks To Back China Trade Status, The Boston Globe, 6/15/91)

In 2000, Kerry Voted In Favor Of Permanent Normal Trade Relations With China. (H.R. 4444, CQ Vote #251: Passed 83-15: R 46-8; D 37-7, 9/19/00, Kerry Voted Yea)

Now Kerry Criticizes The Bush Administration For Trading With China. Democratic presidential candidate John Kerry said on Monday Americans workers were paying the price for President Bush's weak stance on trade with China and other countries. On the bus tour, Kerry singled out the Bush administration's handling of trade with China and said that country was manipulating its currency. (Caren Bohan, "Kerry Pledges Aggressive Trade Stance," Reuters, 4/26/04)


Flip-Flopped On Iraq War

Kerry Voted For Authorization To Use Force In Iraq. (H.J. Res. 114, CQ Vote #237: Passed 77-23: R 48-1; D 29-21; I 0-1, 10/11/02, Kerry Voted Yea.)

In First Dem Debate, Kerry Strongly Supported Presidents Action In Iraq. KERRY: George, I said at the time I would have preferred if we had given diplomacy a greater opportunity, but I think it was the right decision to disarm Saddam Hussein, and when the President made the decision, I supported him, and I support the fact that we did disarm him. (ABC News, Democrat Presidential Candidate Debate, Columbia, SC, 5/4/03)

Kerry Later Claimed He Voted To Threaten Use Of Force In Iraq. I voted to threaten the use of force to make Saddam Hussein comply with the resolutions of the United Nations. (Sen. John Kerry, Remarks At Announcement Of Presidential Candidacy, Mount Pleasant, SC, 9/2/03)

Now, Kerry Says He Is Anti-War Candidate. CHRIS MATTHEWS: Do you think you belong to that category of candidates who more or less are unhappy with this war, the way its been fought, along with General Clark, along with Howard Dean and not necessarily in companionship politically on the issue of the war with people like Lieberman, Edwards and Gephardt? Are you one of the anti-war candidates? KERRY: I am -- Yes, in the sense that I dont believe the president took us to war as he should have, yes, absolutely. (MSNBCs Hardball, 1/6/04)

And yesterday, he said he would have done things exactly the same as Bush did them, even knowing what we know now. ""Almost two years after he voted for the war in Iraq, and almost 220 days after switching positions to declare himself the anti-war candidate, my opponent has found a new nuance," Bush said as he stumped in Florida, a swing state.

"After months of questioning my motives and even my credibility, Senator Kerry now agrees with me that even though we have not found the stockpile of weapons we all believe were there, knowing everything we know today, he would have voted to go into Iraq and remove Saddam Hussein from power," the president added.

"I want to thank Sen. Kerry for clearing that up," Bush said sarcastically, adding that Kerry still had time to change his position: "There are still 84 days left in the campaign."

Bush was referring to comments on the Iraq war that Kerry made on Monday, when the Democratic nominee said he would still have voted to authorize the war knowing what he knows now.

"I believe it was the right authority for a president to have," Kerry said, adding he would have used the authority better than Bush did." (Vincent Morris, "CAMPAIGN WAR ZONE" New York Post, 8/11/04)


Flip-Flopped On Eliminating Marriage Penalty For Middle Class

Kerry Said He Will Fight To Keep Tax Relief For Married Couples. Howard Dean and Gephardt are going to put the marriage penalty back in place. So if you get married in America, were going to charge you more taxes. I do not want to do that. (Fox News Special Report, 10/23/03)

Said Democrats Fought To End Marriage Penalty Tax. We fought hard to get rid of the marriage penalty. (MSNBCs News Live, 7/31/03)

But, In 1998, Kerry Voted Against Eliminating Marriage Penalty Relief For Married Taxpayers With Combined Incomes Less Than $50,000 Per Year, Saving Taxpayers $46 Billion Over 10 Years. (S. 1415, CQ Vote #154: Rejected 48-50: R 5-49; D 43-1, 6/10/98, Kerry Voted Yea)


Flip-Flopped On Patriot Act

Kerry Voted For Patriot Act. The Patriot Act was passed nearly unanimously by the Senate 98-1, and 357-66 in the House. (H.R. 3162, CQ Vote #313: Passed 98-1: R 49-0; D 48-1; I 1-0, 10/25/01, Kerry Voted Yea)

Kerry Used To Defend His Vote. Most of [The Patriot Act] has to do with improving the transfer of information between CIA and FBI, and it has to do with things that really were quite necessary in the wake of what happened on September 11th. (Sen. John Kerry, Remarks At Town Hall Meeting, Manchester, NH, 8/6/03)

Now, Kerry Attacks Patriot Act. We are a nation of laws and liberties, not of a knock in the night. So it is time to end the era of John Ashcroft. That starts with replacing the Patriot Act with a new law that protects our people and our liberties at the same time. Ive been a District Attorney and I know that what law enforcement needs are real tools not restrictions on Americans basic rights. (Sen. John Kerry, Remarks At Iowa State University, 12/1/03)


Kerry Took BOTH Sides On First Gulf War

Kerry Took BOTH Sides In First Gulf War In Separate Letters To Same Constituent. Rather than take a side--albeit the one he thought was most expedient--Kerry actually stood on both sides of the first Gulf war, much like he did this time around. Consider this Notebook item from TNRs March 25, 1991 issue, which ran under the headline Same Senator, Same Constituent: Thank you for contacting me to express your opposition ... to the early use of military force by the US against Iraq. I share your concerns. On January 11, I voted in favor of a resolution that would have insisted that economic sanctions be given more time to work and against a resolution giving the president the immediate authority to go to war. --letter from Senator John Kerry to Wallace Carter of Newton Centre, Massachusetts, dated January 22 [1991] Thank you very much for contacting me to express your support for the actions of President Bush in response to the Iraqi invasion of Kuwait. From the outset of the invasion, I have strongly and unequivocally supported President Bushs response to the crisis and the policy goals he has established with our military deployment in the Persian Gulf. --Senator Kerry to Wallace Carter, January 31 [1991] (Noam Scheiber, Noam Scheibers Daily Journal of Politics, The New Republic Online, 1/28/04)


Flip-Flopped On Gay Marriage Amendment

In 2002, Kerry Signed Letter Urging MA Legislature To Reject Constitutional Amendment Banning Gay Marriage. We rarely comment on issues that are wholly within the jurisdiction of the General Court, but there are occasions when matters pending before you are of such significance to all residents of the Commonwealth that we think it appropriate for us to express our opinion. One such matter is the proposed Constitutional amendment that would prohibit or seriously inhibit any legal recognition whatsoever of same-sex relationships. We believe it would be a grave error for Massachusetts to enshrine in our Constitution a provision which would have such a negative effect on so many of our fellow residents. We are therefore united in urging you to reject this Constitutional amendment and avoid stigmatizing so many of our fellow citizens who do not deserve to be treated in such a manner. (Sen. John Kerry, et al, Letter To Members Of The Massachusetts Legislature, 7/12/02)

Now, In 2004, Kerry Wont Rule Out Supporting Similar Amendment. Asked if he would support a state constitutional amendment barring gay and lesbian marriages, Kerry didnt rule out the possibility. Ill have to see what language there is, he said. (Susan Milligan, Kerry Says GOP May Target Him On Wedge Issue, The Boston Globe, 2/6/04)


Flip-Flopped On Attacking President During Time Of War

In March 2003, Kerry Promised Not To Attack President When War Began. Senator John F. Kerry of Massachusetts said he will cease his complaints once the shooting starts. Its what you owe the troops, said a statement from Kerry, a Navy veteran of the Vietnam War. I remember being one of those guys and reading news reports from home. If America is at war, I wont speak a word without measuring how itll sound to the guys doing the fighting when theyre listening to their radios in the desert. (Glen Johnson, Democrats On The Stump Plot Their War Rhetoric, The Boston Globe, 3/11/03)

But Weeks Later, With Troops Just Miles From Baghdad, Kerry Broke His Pledge. What we need now is not just a regime change in Saddam Hussein and Iraq, but we need a regime change in the United States, Kerry said in a speech at the Peterborough Town Library. Despite pledging two weeks ago to cool his criticism of the administration once war began, Kerry unleashed a barrage of criticism as US troops fought within 25 miles of Baghdad. (Glen Johnson, Kerry Says Us Needs Its Own Regime Change, The Boston Globe, 4/3/03)


Flip-Flopped On Death Penalty For Terrorists

In 1996, Kerry Attacked Governor Bill Weld For Supporting Death Penalty For Terrorists. KERRY: Your policy would amount to a terrorist protection policy. Mine would put them in jail. (1996 Massachusetts Senate Debate, 9/16/96)

In 1996, Kerry Said, You Can Change Your Mind On Things, But Not On Life-And-Death Issues. (Timothy J. Connolly, The Snoozer Had Some Life, [Worcester, MA] Telegram & Gazette, 7/3/96)

But, In 2002, Kerry Said He Supported Death Penalty For Terrorists. KERRY: The law of the land is the law of the land, but I have also said that I am for the death penalty for terrorists because terrorists have declared war on your country. (NBCs Meet The Press, 12/1/02)


Flip-Flopped On No Child Left Behind

Kerry Voted For No Child Left Behind Act. (H.R. 1, CQ Vote #371: Adopted 87-10: R 44-3; D 43-6; I 0-1, 12/18/01, Kerry Voted Yea)

But Now Kerry Is Attacking No Child Left Behind As Mockery. Between now and the time Im sworn in January 2005, Im going to use every day to make this president accountable for making a mockery of the words No Child Left Behind. (Holly Ramer, Kerry Wants To Make Environmental Justice A Priority, The Associated Press, 4/22/03)

Kerry Trashed NCLB As Unfunded Mandate With Laudable Goals. Kerry referred to [No Child Left Behind] as an unfunded mandate with laudable goals. Without the resources, education reform is a sham, Kerry said. I cant wait to crisscross this country and hold this president accountable for making a mockery of the words no child left behind. (Matt Leon, Sen. Kerry In Tune With Educators, The [Quincy, MA] Patriot Ledger, 7/11/03)


Flip-Flopped On Affirmative Action

In 1992, Kerry Called Affirmative Action Inherently Limited And Divisive. [W]hile praising affirmative action as one kind of progress that grew out of civil rights court battles, Kerry said the focus on a rights-based agenda has inadvertently driven most of our focus in this country not to the issue of what is happening to the kids who do not get touched by affirmative action, but toward an inherently limited and divisive program which is called affirmative action. That agenda is limited, he said, because it benefits segments of black and minority populations, but not all. And it is divisive because it creates a perception and a reality of reverse discrimination that has actually engendered racism. (Lynne Duke, Senators Seek Serious Dialogue On Race, The Washington Post, 4/8/92)

In 2004, Kerry Denied Ever Having Called Affirmative Action Divisive. CNNs KELLY WALLACE: We caught up with the Senator, who said he never called affirmative action divisive, and accused Clark of playing politics. SEN. KERRY: Thats not what I said. I said there are people who believe that. And I said mend it, dont end it. Hes trying to change what I said, but you can go read the quote. I said very clearly I have always voted for it. Ive always supported it. Ive never, ever condemned it. I did what Jim Clyburn did and what Bill Clinton did, which is mend it. And Jim Clyburn wouldnt be supporting it if it were otherwise. So lets not have any politics here. Lets keep the truth. (CNNs Inside Politics, 1/30/04)


Flip-Flopped On Ethanol

Kerry Twice Voted Against Tax Breaks For Ethanol. (S. Con. Res. 18, CQ Vote #44: Rejected 48-52: R 11-32; D 37-20, 3/23/93, Kerry Voted Nay; S. Con. Res. 18, CQ Vote #68: Motion Agreed To 55-43: R 2-40; D 53-3, 3/24/93, Kerry Voted Yea)

Kerry Voted Against Ethanol Mandates. (H.R. 4624, CQ Vote #255: Motion Agreed To 51-50: R 19-25; D 31-25, 8/3/94, Kerry Voted Nay)

Kerry Voted Twice To Increase Liability On Ethanol, Making It Equal To Regular Gasoline. (S. 517, CQ Vote #87: Motion Agreed To 57-42: R 38-10; D 18-32; I 1-0, 4/25/02 Kerry Voted Nay; S. 14, CQ Vote #208: Rejected 38-57: R 9-40; D 28-17; I 1-0, 6/5/03, Kerry Voted Yea)

On The Campaign Trail, Though, Kerry Is For Ethanol. KERRY: Im for ethanol, and I think its a very important partial ingredient of the overall mix of alternative and renewable fuels we ought to commit to. (MSNBC/DNC, Democrat Presidential Candidate Debate, Des Moines, IA, 11/24/03)


Flip-Flopped On Cuba Sanctions

Senator Kerry Has Long Voted Against Stronger Cuba Sanctions. (H.R. 927, CQ Vote #489, Motion Rejected 59-36: R 50-2; D 9-34, 10/17/95, Kerry Voted Nay; S. 955, CQ Vote #183: Rejected 38-61: R 5-49; D 33-12, 7/17/97, Kerry Voted Yea; S. 1234, CQ Vote #189, Motion Agreed To 55-43: R 43-10; D 12-33, 6/30/99, Kerry Voted Nay; S. 2549, CQ Vote #137: Motion Agreed To 59-41: R 52-3; D 7-38, 6/20/00, Kerry Voted Nay)

In 2000, Kerry Said Florida Politics Is Only Reason Cuba Sanctions Still In Place. Senator John F. Kerry, the Massachusetts Democrat and member of the Foreign Relations Committee, said in an interview that a reevaluation of relations with Cuba was way overdue. We have a frozen, stalemated, counterproductive policy that is not in humanitarian interests nor in our larger credibility interest in the region, Kerry said. It speaks volumes about the problems in the current American electoral process. The only reason we dont reevaluate the policy is the politics of Florida. (John Donnelly, Policy Review Likely On Cuba, The Boston Globe, 4/9/00)

Now Kerry Panders To Cuban Vote, Saying He Would Not Lift Embargo Against Cuba. TIM RUSSERT: Would you consider lifting sanctions, lifting the embargo against Cuba? SEN. KERRY: Not unilaterally, not now, no. (NBCs Meet The Press, 8/31/03)

Kerry Does Not Support Opening Up The Embargo Wily Nilly. Kerry said he believes in engagement with the communist island nation but that does not mean, Open up the dialogue. He believes it means travel and perhaps even remittances or cultural exchanges but he does not support opening up the embargo wily nilly. (Daniel A. Ricker, Kerry Says Bush Did Not Build A Legitimate Coalition In Iraq, The Miami Herald, 11/25/03)


Flip-Flopped On NAFTA

Kerry Voted For NAFTA. (H.R. 3450, CQ Vote #395: Passed 61-38: R 34-10; D 27-28, 11/20/93, Kerry Voted Yea)

Kerry Recognized NAFTA Is Our Future. NAFTA recognizes the reality of todays economy - globalization and technology, Kerry said. Our future is not in competing at the low-level wage job; it is in creating high-wage, new technology jobs based on our skills and our productivity. (John Aloysius Farrell, Senates OK Finalizes NAFTA Pact, The Boston Globe, 11/21/93)

Now, Kerry Expresses Doubt About NAFTA. Kerry, who voted for NAFTA in 1993, expressed some doubt about the strength of free-trade agreements. If it were before me today, I would vote against it because it doesnt have environmental or labor standards in it, he said. (David Lightman, Democrats Battle For Labors Backing, Hartford Courant, 8/6/03)


Flip-Flopped On Double Taxation Of Dividends

December 2002: Kerry Favored Ending Double Taxation Of Dividends. [T]o encourage investments in the jobs of the future - I think we should eliminate the tax on capital gains for investments in critical technology companies - zero capital gains on $100 million issuance of stock if its held for 5 years and has created real jobs. And we should attempt to end the double taxation of dividends. (Sen. John Kerry, Remarks At The City Club Of Cleveland, 12/3/02)

May 2003: Kerry Said He Opposed Ending Double Taxation Of Dividends. Kerry also reiterated his opposition to the Republican plan to cut taxes on stock dividends. This is not the time for a dividends tax cut that goes to individuals, he said. (Kerry Says Time Is On Dems Side, The Associated Press, 5/8/03)


Flip-Flopped On Raising Taxes During Economic Downturn

September 2001: Said Should Not Raise Taxes In Economic Downturn. The first priority is the economy of our nation. And when you have a downturn in the economy, the last thing you do is raise taxes or cut spending. We shouldnt do either. We need to maintain a course that hopefully will stimulate the economy. . . . No, we should not raise taxes, but we have to put everything on the table to take a look at why we have this structural problem today. . . .[Y]ou dont want to raise taxes. (NBCs Meet The Press, 9/2/01)

We Should Absolutely Not Raise Taxes. Well, I think its very clear what I favor because we voted for it early in the spring, which was the Democratic budget alternative that had triggers in it where you didnt wind up spending money you dont have. It had a smaller tax cut but more tax cut for a stimulus, which is what we need. So you ask me, what do we need now? Yes, we need additional stimulus. We should absolutely not raise taxes. We should not cut spending. What we need to do is drive the economy of this country. The economy is the number one issue. It is the most important thing we should focus on. (CNNs Evans, Novak, Hunt & Shields, 9/8/01)

April 2002: Said He Wanted Larger Tax Cut And Was Not In Favor Of Repeal. CNNs TUCKER CARLSON: Senator Kerry . . . [many Democrats] [g]et a lot of political mileage out of criticizing [President Bushs tax cut], but nobody has the courage to say repeal it. Are you for repealing it? KERRY: Its not a question of courage. . . . And its not an issue right now. We passed appropriately a tax cut as a stimulus, some $40 billion. Many of us thought it should have even maybe been a little bit larger this last year [T]he next tax cut doesnt take effect until 2004. If we can grow the economy enough between now and then, if we have sensible policies in place and make good choices, who knows what our choices will be. So its simply not a ripe issue right now. And Im not in favor of turning around today and repealing it. (CNNs Crossfire, 4/16/02)

December 2002: Flip-Flopped, Would Keep Tax Cuts From Taking Effect. NBCs TIM RUSSERT: Senator . . . should we freeze or roll back the Bush tax cut? KERRY: Well, I wouldnt take away from people whove already been given their tax cut What I would not do is give any new Bush tax cuts. RUSSERT: So the tax cut thats scheduled to be implemented in the coming years KERRY: No new tax cut under the Bush plan. . . . It doesnt make economic sense. RUSSERT: Now, this is a change (NBCs Meet The Press, 12/1/02)


Called For Freeze Of Bush Tax Cuts In Favor Of Year-Long Suspension Of Payroll Taxes On First $10,000 Of Personal Income. Kerry said Bushs tax cuts have mainly benefited the rich while doing little for the economy. Kerry is proposing to halt Bushs additional tax cuts and instead impose a yearlong suspension of payroll taxes on the first $10,000 of income to help the poor and middle class. (Tyler Bridges, Kerry Visits Miami To Start Raising Funds, The Miami Herald, 12/7/02)


Flip-Flopped On Small Business Income Taxes

Kerry Voted Against Exempting Small Businesses And Family Farms From Clinton Income Tax Increase. (S. Con. Res. 18, CQ Vote #79: Motion Agreed To 54-45: R 0-43; D 54-2, 3/25/93, Kerry Voted Yea)

Three Months Later, Kerry Voted In Favor Of Proposal To Exclude Small Businesses From The Increased Income Tax. (S. 1134, CQ Vote #171: Motion Rejected 56-42: R 43-0; D 13-42, 6/24/93, Kerry Voted Yea)

Kerry Claimed He Fought To Exempt Small Businesses From Income Tax Increases. I worked to amend the reconciliation bill so that it would exempt small businesses who are classified as subchapter S corporations from the increased individual income tax. (Sen. John Kerry, Congressional Record, 6/29/93, p. S 8268)


Kerry Flip-Flopped On 50-Cent Gas Tax Increase

In 1994, Kerry Backed Half-Dollar Increase In Gas Tax. Kerry said [the Concord Coalitions scorecard] did not accurately reflect individual lawmakers efforts to cut the deficit. It doesnt reflect my $43 billion package of cuts or my support for a 50-cent increase in the gas tax, Kerry said. (Jill Zuckman, Deficit-Watch Group Gives High Marks To 7 N.E. Lawmakers, The Boston Globe, 3/1/94)

Two Years Later, Kerry Flip-Flopped. Kerry no longer supports the 50-cent [gas tax] hike, nor the 25-cent hike proposed by the [Concord] coalition. (Michael Grunwald, Kerry Gets Low Mark On Budgeting, The Boston Globe, 4/30/96)


Flip-Flopped On Leaving Abortion Up To States

Kerry Used To Say Abortion Should Be Left Up To States. I think the question of abortion is one that should be left for the states to decide, Kerry said during his failed 1972 Congressional bid. (John Kerry On The Issues, The [Lowell, MA] Sun, 10/11/72)

Now Kerry Says Abortion Is Law Of Entire Nation. The right to choose is the law of the United States. No person has the right to infringe on that freedom. Those of us who are in government have a special responsibility to see to it that the United States continues to protect this right, as it must protect all rights secured by the constitution. (Sen. John Kerry [D-MA], Congressional Record, 1/22/85)


Flip-Flopped On Litmus Tests For Judicial Nominees

Kerry Used To Oppose Litmus Tests For Judicial Nominees. Throughout two centuries, our federal judiciary has been a model institution, one which has insisted on the highest standards of conduct by our public servants and officials, and which has survived with undiminished respect. Today, I fear that this institution is threatened in a way that we have not seen before. This threat is that of the appointment of a judiciary which is not independent, but narrowly ideological, through the systematic targeting of any judicial nominee who does not meet the rigid requirements of litmus tests imposed (Sen. John Kerry, Congressional Record, 2/3/86, p. S864)

But Now Kerry Says He Would Only Support Supreme Court Nominees Who Pledge To Uphold Roe v. Wade. The potential retirement of Supreme Court justices makes the 2004 presidential election especially important for women, Senator John F. Kerry told a group of female Democrats yesterday, and he pledged that if elected president he would nominate to the high court only supporters of abortion rights under its Roe v. Wade decision. Any president ought to appoint people to the Supreme Court who understand the Constitution and its interpretation by the Supreme Court. In my judgment, it is and has been settled law that women, Americans, have a defined right of privacy and that the government does not make the decision with respect to choice. Individuals do. (Glen Johnson, Kerry Vows Court Picks To Be Abortion-Rights Supporters, The Boston Globe, 4/9/03)


Flip-Flopped On Federal Health Benefits

In 1993, Kerry Expressed Doubts That Federal Employees Health Benefits System Worked Well. Hillary Rodham Clinton today offered a fresh description of one of the most confusing elements of the Administration health care plan, the health insurance purchasing alliances, saying they would let all Americans choose coverage in the way members of Congress do. Senator John Kerry, Democrat of Massachusetts, said he was not sure that the Federal program worked all that well. (Adam Clymer, Hillary Clinton Says Health Plan Will Be Familiar, The New York Times, 12/8/93)

Kerry Expressed Personal Dissatisfaction With His Coverage Through Federal Program. Earlier this month, when Hillary Rodham Clinton came to Boston and vowed that average Americans would get as good coverage as that enjoyed by their senators and representatives, Sen. John F. Kerry told Clinton that he thought the country could do better. The Massachusetts Democrat said he was thinking, among other recent disasters, of his $500 dental bill for treatment of an abscessed tooth. Because it was done in the dentists office, rather than the hospital, they didnt cover it. So they were urging me to go spend twice as much in a hospital, said Kerry, who is covered by BACE, the Beneficial Association of Capitol Employees. (Ana Puga, Lawmakers Talk Health Care, The Boston Globe, 12/19/93)

Now, On Campaign Trail, Kerry Is Enthusiastic About Health Care He Receives As Senator. As a U.S. Senator, I could get the best health care in the world. Most people arent so lucky, and we need to change that. Thats why my plan gives every American access to the same kind of health care that members of Congress give themselves. Because your familys health care is just as important as any politicians in Washington. (Sen. John Kerry, Affordable Health Care For All Americans, Remarks At Mercy Medical, Cedar Rapids, IA, 12/14/03)

Kerry: Im Going To Make Available To Every American The Same Health Care Plan That Senators And Congressmen Give Themselves (Sen. John Kerry, AARP Democrat Candidate Debate, Bedford, NH, 11/18/03)


Flip-Flopped On Tax Credits For Small Business Health

In 2001, Kerry Voted Against Amendment Providing $70 Billion For Tax Credits For Small Business To Purchase Health Insurance. (H. Con. Res. 83, CQ Vote #83: Rejected 49-51: R 48-2; D 1-49, 4/5/01, Kerry Voted Nay)

Now, Kerry Promises Refundable Tax Credits To Small Businesses For Health Coverage. Refundable tax credits for up to 50 percent of the cost of coverage will be offered to small businesses and their employees to make health care more affordable. (John Kerrys Plan To Make Health Care Affordable To Every American, John Kerry For President Website, www.johnkerry.com, Accessed 1/21/04)


Flip-Flopped On Health Coverage

In 1994, Kerry Said Democrats Push Health Care Too Much. [Kerry] said Kennedy and Clintons insistence on pushing health care reform was a major cause of the Democratic Partys problems at the polls. (Joe Battenfeld, Jenny Craig Hit With Sex Harassment Complaint - By Men, Boston Herald, 11/30/94)

But Now Kerry Calls Health Care His Passion. Sen. John Kerry says expanding coverage is my passion. (Susan Page, Health Specifics Could Backfire On Candidates, USA Today, 6/2/03)


Flip-Flopped On Welfare Reform

In 1993, Kerry Voted To Kill Bipartisan Welfare Work Requirement. In 1993, Kerry and Kennedy voted against a welfare-to-work requirement that was supported by many Democrats, including Sens. Dianne Feinstein (D-CA) and Harry Reid (D-NV):

Fiscal 1993 Supplemental Appropriations - Welfare Work Requirement. Moynihan, D-N.Y., motion to table (kill) the DAmato, R-N.Y., amendment to sharply cut federal welfare administration aid to states that do not, within a year, require at least 10 percent of their able-bodied welfare recipients without dependents to work. The required workfare participation rate would be increased by 2 percent a year until 50 percent were working. (H.R. 2118, CQ Vote #163: Rejected 34-64: R 1-42; D 33-22, 6/22/93, Kerry Voted Yea)

But In 1996, Kerry Voted For Welfare Reform. (H.R. 3734, CQ Vote #262: Adopted 78-21: R 53-0; D 25-21, 8/1/96, Kerry Voted Yea)


Flip-Flops On Stock Options Expensing

Kerry Used To Oppose Expensing Stock Options. Democratic Senator John F. Kerry was among those fighting expensing of stock options. (Sue Kirchhoff, Senate Blocks Options, The Boston Globe, 7/16/02)

Kerry Said Expensing Options Would Not Benefit The Investing Public. KERRY: Mr. President, the Financial Accounting Standards Board has proposed a rule that will require companies to amortize the value of stock options and deduct them off of their earnings statements I simply cannot see how the FASB rule, as proposed, will benefit the investing public. (Sen. John Kerry, Congressional Record, 3/10/94, p. S2772)

But Now Kerry Says He Supports Carrying Of Stock Options As Corporate Expense. On an issue related to corporate scandals, Kerry for the first time endorsed the carrying of stock options as a corporate expense. The use of stock options was abused by some companies and contributed to overly optimistic balance sheets. Kerry applauded steps by Microsoft Corp. to eliminate stock options for employees and said all publicly traded companies should be required to expense such options. (Dan Balz, Kerry Raps Bush Policy On Postwar Iraq, The Washington Post, 7/11/03)


Flip-Flopped On Medical Marijuana

Kerry Said His Personal Disposition Is Open To The Issue Of Medical Marijuana. Aaron Houston of the Granite Staters for Medical Marijuana said that just a month ago Mr. Kerry seemed to endorse medical marijuana use, and when asked about the content of his mysterious study, said, I am trying to find out. I dont know. Mr. Kerry did say his personal disposition is open to the issue of medical marijuana and that hed stop Drug Enforcement Administration raids on patients using the stuff under Californias medical marijuana law. (Jennifer Harper, Inside Politics, The Washington Times, 8/8/03)

But Now Kerry Says He Wants To Wait For Study Analyzing Issue Before Making Final Decision. The Massachusetts Democrat said Wednesday hed put off any final decision on medical marijuana because theres a study under way analyzing what the science is. (Jennifer Harper, Inside Politics, The Washington Times, 8/8/03)


Flip-Flopped On Burma Sanctions

In 1995, Kerry Was Against Burma Sanctions. I question whether isolation is a successful means of promoting political change, Kerry told a constituent in a 1995 letter justifying his opposition to a Burma sanction bill. (Geeta Anand, et al., Menino Gets Ahead Of Himself, Starts Contemplating Third Term, The Boston Globe, 5/18/97)

But Now Kerry Supports Burma Sanctions. In his 1996 reelection campaign, Kerry, after Governor William F. Weld took up the cause, was badgered by advisers into shifting his position. But as he eyes a presidential campaign and the Burma sanction movement gains credibility, Kerry describes the Burma regime as a semi-criminalized dictatorship which should not be treated with respect by other nations, but should be instead subject to limitations on travel, investment, and access to the most developed nations. (Geeta Anand, et al., Menino Gets Ahead Of Himself, Starts Contemplating Third Term, The Boston Globe, 5/18/97)


Flip-Flopped On Military Experience As Credential For Public Office

Kerry: Service Should Not Be Litmus Test For Leadership. Mr. President, you and I know that if support or opposition to the war were to become a litmus test for leadership, America would never have leaders or recover from the divisions created by that war. You and I know that if service or nonservice in the war is to become a test of qualification for high office, you would not have a Vice President, nor would you have a Secretary of Defense and our Nation would never recover from the divisions created by that war. (Sen. John Kerry, Congressional Record, 10/08/92, p. S17709)

But Now Kerry Constantly Challenges The Stature Of His Democratic Opponents Over Their Lack Of Military Service. And more than ever, Mr. Kerry is invoking his stature as a Vietnam veteran as he challenges the stature of his Democratic opponents -- none of whom, he frequently points out, have worn the uniform of our country -- to withstand a debate with Mr. Bush on national security. (Adam Nagourney, As Campaign Tightens, Kerry Sharpens Message, The New York Times, 8/10/03)


Flip-Flopped On PACs

Kerry Used To Decry Special Interests And Their PAC Money. Im frequently told by cynics in Washington that refusing PAC money is naive, Kerry told his supporters in 1985. Do you agree that it is nave to turn down special interests and their PAC money? (Glen Johnson, In A Switch, Kerry Is Launching A PAC, The Boston Globe, 12/15/01)

But Now, Kerry Has Established His Own PAC. A week after repeating that he has refused to accept donations from political action committees, Senator John F. Kerry announced yesterday that he was forming a committee that would accept PAC money for him to distribute to other Democratic candidates. Kerrys stance on soft money, unregulated donations funneled through political parties, puts him in the position of raising the type of money that he, McCain, and others in the campaign-finance reform movement are trying to eliminate. (Glen Johnson, In A Switch, Kerry Is Launching A PAC, The Boston Globe, 12/15/01)


Flip-Flopped On $10,000 Donation Limit To His PAC

When Kerry Established His PAC In 2001, He Instituted A $10,000 Limit On Donations. A week after repeating that he has refused to accept donations from political action committees, Senator John F. Kerry announced yesterday that he was forming a committee that would accept PAC money for him to distribute to other Democratic candidates The statement also declared that the new PAC would voluntarily limit donations of so-called soft money to $10,000 per donor per year and disclose the source and amount of all such donations. (Glen Johnson, In A Switch, Kerry Is Launching A Pac, The Boston Globe, 12/15/01)

One Year Later, Kerry Started Accepting Unlimited Contributions. Senator John F. Kerry, who broke with personal precedent last year when he established his first political action committee, has changed his fund-raising guidelines again, dropping a $10,000 limit on contributions from individuals, a cap he had touted when establishing the PAC. The Massachusetts Democrat said yesterday he decided to accept unlimited contributions, which has already allowed him to take in soft money donations as large as $25,000, because of the unprecedented fund-raising demands confronting him as a leader in the Senate Democratic caucus. (Glen Johnson, Kerry Shifts Fund-Raising Credo For His Own PAC, The Boston Globe, 10/4/02)


Flip-Flopped On Using Personal Funds In 1996 Race

In 1996, Kerry And Weld Established $500,000 Limit Of Personal Wealth To Be Used In Senate Campaign. In 1996, Kerry and Weld gave their already noteworthy Senate race added significance by establishing a spending cap. The candidates agreed to spend no more than $6.9 million from July 1 through the election. Weld ended up spending $6.6 million and Kerry $6.3 million. One key element of the agreement limited the candidates to spending $500,000 in personal wealth, a clause Weld favored because Kerry is married to a millionaire, Teresa Heinz. (Glen Johnson, In Kerrys Plan For A Pac, The Resolution Of Opposites, The Boston Globe, 12/18/01)

Kerry Broke Agreement By Spending $1.2 Million Over Limit. [P]ost-election reports showed a last-minute infusion of $1.7 million from Kerrys wife, heiress Teresa Heinz. [K]erry denied that his campaign violated its agreement. The money had been loaned--not contributed--by his wife, he explained. There was nothing in the agreement that restricted us from taking a loan and we paid it back in $1,000 and $2,000 chunks. (Global Ecology Lobby Rocked By Defection, Political Finance, The Newsletter, 1/02)


Flip-Flopped On Israel Security Fence

October 2003: Kerry Calls Fence Barrier To Peace. And I know how disheartened Palestinians are by the Israeli governments decision to build a barrier off the green line, cutting deeply into Palestinian areas. We do not need another barrier to peace. Provocative and counterproductive measures only harm Israels security over the long- term, they increase hardships to the Palestinian people, and they make the process of negotiating an eventual settlement that much harder. (Sen. John Kerry, Remarks Before Arab American Institute National Leadership Conference, Dearborn, MI, 10/17/03)

February 2004: Kerry Calls Fence Legitimate Act Of Self-Defense. US Senator John Kerry of Massachusetts, the frontrunner in the race for the Democratic presidential nomination, described Israels construction of a security barrier as a legitimate act of self defense after Sundays suicide bombing in Jerusalem, clarifying a position he took in October when he told an Arab American audience, We dont need another barrier to peace. (Janine Zacharia, Kerry Defends Security Fence, The Jerusalem Post, 2/25/04)


Flip-Flop-Flipped On Ballistic Missile Defense

Kerry Called For Cancellation Of Missile Defense Systems In 1984 And Has Voted Against Funding For Missile Defense At Least 53 Times Between 1985 And 2000. (John Kerry On The Defense Budget, Campaign Position Paper, John Kerry For U.S. Senate, 1984; S. 1160, CQ Vote #99: Rejected 21-78: R 2-50; D 19-28, 6/4/85, Kerry Voted Yea; S. 1160, CQ Vote #100: Rejected 38-57: R 6-45; D 32-12, 6/4/85, Kerry Voted Yea; S. 1160, CQ Vote #101: Rejected 36-59: R 1-49; D 35-10, 6/4/85, Kerry Voted Yea; S. 1160, CQ Vote #103: Rejected 33-62: R 28-22; D 5-40, 6/4/85, Kerry Voted Nay; H.J. Res. 465, CQ Vote #365: Motion Agreed To 64-32: R 49-2; D 15-30, 12/10/85, Kerry Voted Nay; H.R. 4515, CQ Vote #122: Ruled Non-Germane 45-47: R 7-42; D 38-5, 6/6/86, Kerry Voted Yea; S. 2638, CQ Vote #176: Motion Agreed To 50-49: R 41-11; D 9-38, 8/5/86, Kerry Voted Nay; S. 2638, CQ Vote #177: Rejected 49-50: R 10-42; D 39-8, 8/5/86, Kerry Voted Yea; S. 1174, CQ Vote #248: Motion Agreed To 58-38: R 8-37; D 50-1, 9/17/87, Kerry Voted Yea; S. 1174, CQ Vote #259: Motion Agreed To 51-50: R 37-9; D 13-41, With Vice President Bush Casting An Yea Vote, 9/22/87, Kerry Voted Nay; S. 2355, CQ Vote #124: Motion Agreed To 66-29: R 38-6; D 28-23, 5/11/88, Kerry Voted Nay; S. 2355, CQ Vote #125: Motion Agreed To 50-46: R 38-7; D 12-39, 5/11/88, Kerry Voted Nay; S. 2355, CQ Vote #126: Motion Rejected 47-50: R 38-6; D 9-44, 5/11/88, Kerry Voted Nay; S. 2355, CQ Vote #128: Motion Rejected 48-50: R 6-39; D 42-11, 5/11/88, Kerry Voted Yea; S. 2355, CQ Vote #136: Motion Agreed To 56-37: R 9-34; D 47-3, 5/13/88, Kerry Voted Yea; S. 2355, CQ Vote #137: Motion Agreed To 51-43: R 38-5; D 13-38, 5/13/88, Kerry Voted Nay; H.R. 4264, CQ Vote #251: Motion Rejected 35-58: R 35-9; D 0-49, 7/14/88, Kerry Voted Nay; H.R. 4781, CQ Vote #296: Motion Agreed To 50-44: R 5-39; D 45-5, 8/5/88, Kerry Voted Yea; S. 1352, CQ Vote #148: Motion Agreed To 50-47: R 37-6; D 13-41, 7/27/89, Kerry Voted Nay; H.R. 3072, CQ Vote #202: Rejected 34-66: R 27-18; D 7-48, 9/26/89, Kerry Voted Nay; H.R. 3072, CQ Vote #213: Adopted 53-47: R 39-6; D 14-41, 9/28/89, Kerry Voted Nay; S. 2884, CQ Vote #223: Adopted 54-44: R 2-42; D 52-2, 8/4/90, Kerry Voted Yea; S. 2884, CQ Vote #225: Motion Agreed To 56-41: R 39-4; D 17-37, 8/4/90, Kerry Voted Nay; S. 2884, CQ Vote #226: Motion Agreed To 54-43: R 37-6; D 17-37, 8/4/90, Kerry Voted Nay; S. 3189, CQ Vote #273: Passed 79-16: R 37-5; D 42-11, 10/15/90, Kerry Voted Nay; H.R. 5803, CQ Vote #319: Adopted 80-17: R 37-6; D 43-11, 10/26/90, Kerry Voted Nay; H.R. 4739, CQ Vote #320: Adopted 80-17: R 37-6; D 43-11, 10/26/90, Kerry Voted Nay; S. 1507, CQ Vote #168: Rejected 39-60: R 4-39; D 35-21, 7/31/91, Kerry Voted Yea; S. 1507, CQ Vote #171: Motion Agreed To 60-38: R 40-3; D 20-35, 8/1/91, Kerry Voted Nay; S. 1507, CQ Vote #172: Motion Agreed To 64-34: R 39-4; D 25-30, 8/1/91, Kerry Voted Nay; S. 1507, CQ Vote #173: Rejected 46-52: R 5-38; D 41-14, 8/1/91, Kerry Voted Yea; H.R. 2521, CQ Vote #207: Motion Agreed To 50-49: R 38-5; D 12-44, 9/25/91, Kerry Voted Nay; S. 2403, CQ Vote #85: Adopted 61-38: R 7-36; D 54-2, 5/6/92, Kerry Voted Yea; H.R. 4990, CQ Vote #108: Adopted 90-9: R 34-9; D 56-0, 5/21/92, Kerry Voted Yea; S. 3114, CQ Vote #182: Motion Rejected 43-49: R 34-5; D 9-44, 8/7/92, Kerry Voted Nay; S. 3114, CQ Vote #214: Rejected 48-50: R 5-38; D 43-12, 9/17/92, Kerry Voted Yea; S. 3114, CQ Vote #215: Adopted 52-46: R 39-4; D 13-42, 9/17/92, Kerry Voted Nay; H.R. 5504, CQ Vote #228: Adopted 89-4: R 36-4; D 53-0, 9/22/92, Kerry Voted Yea; S. 1298, CQ Vote #251: Adopted 50-48: R 6-36; D 44-12, 9/9/93, Kerry Voted Yea; S. Con. Res. 63, CQ Vote #64: Rejected 40-59: R 2-42; D 38-17, 3/22/94, Kerry Voted Yea; S. 1026, CQ Vote #354: Motion Agreed To 51-48: R 47-6; D 4-42, 8/3/95, Kerry Voted Nay; S. 1087, CQ Vote #384: Rejected 45-54: R 5-49; D 40-5, 8/10/95, Kerry Voted Yea; S. 1087, CQ Vote #397: Passed 62-35: R 48-4; D 14-31, 9/5/95, Kerry Voted Nay; H.R. 1530, CQ Vote #399: Passed 64-34: R 50-3; D 14-31, 9/6/95, Kerry Voted Nay; H.R. 2126, CQ Vote #579: Adopted 59-39: R 48-5; D 11-34, 11/16/95, Kerry Voted Nay; H.R. 1530, CQ Vote #608: Adopted 51-43: R 47-2; D 4-41, 12/19/95, Kerry Voted Nay; S. 1635, CQ Vote #157: Rejected 53-46: R 52-0; D 1-46, 6/4/96, Kerry Voted Nay; S. 1745, CQ Vote #160: Rejected 44-53: R 4-49; D 40-4, 6/19/96, Kerry Voted Yea; S. 1745, CQ Vote #187: Passed 68-31: R 50-2; D 18-29, 7/10/96, Kerry Voted Nay; S. 936, CQ Vote #171: Rejected 43-56: R 2-53; D 41-3, 7/11/97, Kerry Voted Yea; S. 1873, CQ Vote #131: Motion Rejected 59-41: R 55-0; D 4-41, 5/13/98, Kerry Voted Nay; S. 1873, CQ Vote #262: Motion Rejected 59-41: R 55-0; D 4-41, 9/9/98, Kerry Voted Nay; S. 2549, CQ Vote #178: Motion Agreed To 52-48: R 52-3; D 0-45, 7/13/00, Kerry Voted Nay)

Kerry Then Claimed To Support Missile Defense. I support the development of an effective defense against ballistic missiles that is deployed with maximum transparency and consultation with U.S. allies and other major powers. If there is a real potential of a rogue nation firing missiles at any city in the United States, responsible leadership requires that we make our best, most thoughtful efforts to defend against that threat. The same is true of accidental launch. If it were to happen, no leader could ever explain not having chosen to defend against the disaster when doing so made sense. (Peace Action Website, Where Do The Candidates Stand On Foreign Policy? http://www.peace-action.org/2004/Kerry.html, Accessed 3/10/04)

Now Kerry Campaign Says He Will Defund Missile Defense. FOX NEWS MAJOR GARRETT: Kerry would not say how much all of this would cost. A top military adviser said the Massachusetts Senator would pay for some of it by stopping all funds to deploy a national ballistic missile defense system, one that Kerry doesnt believe will work. KERRY ADVISOR RAND BEERS: He would not go forward at this time because there is not a proof of concept. (Fox News Special Report, 3/17/03)


Flip-Flopped On 1991 Iraq War Coalition

At The Time, Kerry Questioned Strength Of 1991 Coalition. I keep hearing from people, Well, the coalition is fragile, it wont stay together, and my response to that is, if the coalition is so fragile, then what are the vital interests and what is it that compels us to risk our young Americans lives if the others arent willing to stay the course of peace? I voted against the president, Im convinced were doing this the wrong way (CBS This Morning, 1/16/91)

Now Kerry Has Nothing But Praise For 1991 Coalition. SEN. JOHN KERRY: In my speech on the floor of the Senate I made it clear, you are strongest when you act with other nations. All presidents, historically, his father, George Herbert Walker Bush, did a brilliant job of building a legitimate coalition and even got other people to help pay for the war. (NBCs Meet The Press, 1/11/04)


Flip-Flopped On View Of War On Terror

Kerry Said War On Terror Is Basically A Manhunt. Kerry was asked about Bushs weekend appearance on Meet the Press when he called himself a war president. The senator, who watched the session, remarked: The war on terrorism is a very different war from the way the president is trying to sell it to us. Its a serious challenge, and it is a war of sorts, but it is not the kind of war theyre trying to market to America. Kerry characterized the war on terror as predominantly an intelligence-gathering and law enforcement operation. Its basically a manhunt, he said. You gotta know who they are, where they are, what theyre planning, and you gotta be able to go get em before they get us. (Katherine M. Skiba, Bush, Kerry Turn Focus To Each Other, Milwaukee Journal Sentinel, 2/13/04)

Two Weeks Later, Kerry Flip-Flopped, Saying War On Terror Is More Than A Manhunt. This war isnt just a manhunt a checklist of names from a deck of cards. In it, we do not face just one man or one terrorist group. We face a global jihadist movement of many groups, from different sources, with separate agendas, but all committed to assaulting the United States and free and open societies around the globe. (Sen. John Kerry, Remarks At University Of California At Los Angeles, Los Angeles, CA, 2/27/04)


Flip-Flopped On Funding For Our Troops In Iraq

Kerry Pledged To Fund Reconstruction With Whatever Number Of Dollars It Took. NBCS TIM RUSSERT: Do you believe that we should reduce funding that we are now providing for the operation in Iraq? SEN. JOHN KERRY: No. I think we should increase it. RUSSERT: Increase funding? KERRY: Yes. RUSSERT: By how much? KERRY: By whatever number of billions of dollars it takes to win. It is critical that the United States of America be successful in Iraq, Tim. (NBCs Meet The Press, 8/31/03)

Then Kerry Voted Against Senate Passage Of Iraq/Afghanistan Reconstruction Package. Passage of the bill that would appropriate $86.5 billion in fiscal 2004 supplemental spending for military operations and reconstruction in Iraq and Afghanistan. The bill would provide $10.3 billion as a grant to rebuild Iraq, including $5.1 billion for security and $5.2 billion for reconstruction costs. It also would provide $10 billion as a loan that would be converted to a grant if 90 percent of all bilateral debt incurred by the former Iraqi regime of Saddam Hussein has been forgiven by other countries. Separate provisions limit reconstruction aid to $18.4 billion. It also would provide approximately $65.6 billion for military operations and maintenance and $1.3 billion for veterans medical care. (S. 1689, CQ Vote #400: Passed 87-12: R 50-0; D 37-11; I 0-1, 10/17/03, Kerry Voted Nay)

Kerry Later Claimed: I Actually Did Vote For The $87 Billion Before I Voted Against It. (Glen Johnson, Kerry Blasts Bush On Protecting Troops, The Boston Globe, 3/17/04)


Flip-Flopped On Tapping Strategic Petroleum Reserve

In February 2000, Kerry Said Release Of Oil From Strategic Petroleum Reserve Would Not Be Relevant. Without being specific, Kerry, a key member of the Senate Foreign Relations Committee, suggested the US could retaliate economically in other trade areas. He also said he does not want a release of oil from the Strategic Petroleum Reserve. A release is not relevant. It would take months for the oil to get to the market, he said. (Cathy Landry, US Energy Chief Warns Of Gasoline Crisis, Platts Oilgram News, 2/17/00)

Now, In March 2004, Kerry Called For Stop In Filling Strategic Petroleum Reserve To Reduce Prices. Kerry would pressure oil-producing nations to increase production and temporarily suspend filling the U.S. Strategic Petroleum Reserve, according to campaign documents. ... The Bush administration has put the SPR fill program on automatic pilot without regard to the short-term effect on the US market, the campaign documents said. The program needs better management ... Kerry would temporarily suspend filling SPR until oil prices return to normal levels. (Patricia Wilson, Kerry To Offer Plan To Reduce Record Gasoline Prices, Reuters, 3/29/04)


Flip Flopped On Internet Taxation

In 1998, Kerry Voted To Allow States To Continue Taxing Internet Access After Moratorium Took Effect. Kerry voted against tabling an amendment that would extend the moratorium from two years to three years and allow states that currently impose taxes on Internet access to continue doing so after the moratorium takes effect. (S. 442, CQ Vote #306: Motion Rejected 28-69: R 27-27; D 1-42, 10/7/98, Kerry Voted Nay)

In 2001, Kerry Voted To Extend Internet Tax Moratorium Until 2005 And Allow States To Form Uniform Internet Tax System With Approval Of Congress. (H.R. 1552, CQ Vote #341: Motion Agreed To 57-43: R 35-14; D 22-28; I 0-1, 11/15/01, Kerry Voted Nay)

Kerry Said We Do Not Support Any Tax On The Internet Itself. We do not support any tax on the Internet itself. We dont support access taxes. We dont support content taxes. We dont support discriminatory taxes. Many of us would like to see a permanent moratorium on all of those kinds of taxes. At the same time, a lot of us were caught in a place where we thought it important to send the message that we have to get back to the table in order to come to a consensus as to how we equalize the economic playing field in the United States in a way that is fair. (Sen. John Kerry, Congressional Record, 11/15/01, p. S11902)



My question: How can ANYONE possibly justify voting for John Kerry, when even KERRY doesn't know what he stands for?

tomder55 answered on 08/11/04:

Kerry vs. Kerry

ETWolverine rated this answer Excellent or Above Average Answer

Question/Answer
Chouxxx asked on 08/10/04 - Republican National Convention

The Republican Convention is coming up in New York City in a couple of weeks. I want your opinions. If Bush doesn't address the War on Terrorism and Iraq, GOING FORWARD, I will be forced to vote for Kerry. [I am a one issue voter this time around]How can the Republicans score a home run? Is it possible?


Thanks,

tomder55 answered on 08/11/04:

Home run ? Im pretty sure Bush is not capable of that kind of rhetoric. I compare his situation to Harry Trumans at the onset of the Cold War . He needed to address immediate security concerns and make long term plans to contain and eventually defeat the enemy. Truman forged NEW alliances with nations of common interest. He made new security arrangements at home and abroad. He set up a long term plan to financially stabilize nations that were under the threat of Soviet expansion ;and he and subsequent administrations made plans to promote democracy ;especially in the nations that were already under the yoke of Soviet control.
He correctly defined the conflict in terms of freedom vs. slavery .

Truman was not considered to be smart . He was a yokel the "Senator from Pendergast"a derisive reference to the party machine in Missouri he came from.As you recall he also won his election in a close call.

Bush ;like Truman is charged with confronting an enemy that we were slow to recognize. Unlike Truman Bush realizes that a policy of containment has been tried already with Islamo-fascism and it is time to roll back their progress. He said in 2002;

"We seek to create a balance of power that favors human freedom. We will defend the peace by fighting terrorists and tyrants. ... We will extend the peace by encouraging free and open societies on every continent." [U.S. National Security Strategy" speech September 20, 2002(www.whitehouse.gov/nsc/nss.pdf ) ]. Bush recognized that the U.S. in a unipolar world did not have the luxury of trying to maintain the status quo. Bin Ladanism would replace national sovereignty as the organizing structure of the world with subservience to his interpretation of Islam . Bin Laden and the Islamo-nazis did not care how or whom they attacked to achieve their goal.They are for now the biggest threat to American sovereignty and global security .

Bush calls it the way he sees it . He has identified the struggle in what many call simplistic terms ;good and evil .He is mocked as being evangelical when he does so. He is not a new comer to shaping our conflicts in those terms. FDR said of the struggle against the Nazis: "We are fighting to cleanse the world of ancient evils, ancient ills," "There never has been--there never can be--successful compromise between good and evil."( Jan. 6, 1942). Eisenhower said in 1953 : "We sense with all our faculties that forces of good and evil are massed and armed and opposed as rarely before in history," "Destiny has laid upon our country the responsibility of the free world's leadership."Neither man would ever be mistaken for religious zealotry .

He does not believe as some would argue that democracy is beyond the reach of the world dominated by Islam.He quoted Islamic scholars who lament the lag of democratic development in the Middle East : "This freedom deficit undermines human development and is one of the most painful manifestations of lagging political development."( November 07, 2003).He went on to say : The freedom deficit they describe has terrible consequences, of the people of the Middle East and for the world. In many Middle Eastern countries, poverty is deep and it is spreading, women lack rights and are denied schooling. Whole societies remain stagnant while the world moves ahead.( Text of Bush speech to the National Endowment for Democracy). He concluded the speech by saying :
Therefore, the United States has adopted a new policy, a forward strategy of freedom in the Middle East. This strategy requires the same persistence and energy and idealism we have shown before. And it will yield the same results. As in Europe, as in Asia, as in every region of the world, the advance of freedom leads to peace.
His policy has already been laid out . Roll back the advance of the Islamo-nazis wherever they are . Secure the domestic front to prevent attacks when possible. Promote freedom as an alternative to the hopelessness that Bin Laden and his ilk exploit. Bush will argue that he proposes to stay this course . It is then only an issue to debate if this is the right course ,and how has he done so far .

As you know ,I have consistently supported this policy as the only alternative .Making accommodations and inaction delayed us by 20 years in our taking the fight to the enemy. Those nations like the Philippines and France who are deluded into thinking that they can bargain with the enemy are facing internal pressures they may not survive from . So ,if he says that he will stay the course with this policy ,then I support him for it .

How has it gone so far ? The main focus of the war right now is in Iraq . In a short time we have defeated Saddam Hussein . Have handed over the governing of the country to an interim government that seems committed to democratic principles ,and we have battled the internal and external opposition to this experiment to a point that they are on the verge of failure . We have surrounded the thugs of the Iranian backed al-Sadr in Najaf .Hopefully we will not succumb to political correctness and this time finish him off. Speaking of Iran ;we are building a solid case for intervention in that nation at some future date (dont be surprised if the first salvos are fired before November .The clerics are counting on us being paralyzed because of our national elections and are trying to exploit it with this latest insurgency ,and the accelerated pace of their nuclear weapon development). Bush has to tie the war in Iraq in the context of the wider war on terror. It is a phony argument that has gained some acceptance that Iraq was a distraction to the war on terror as I will now address.

1.Recent arrests of Al Qaeda leadership have hurt it and I believe may have disrupted its planned attack on America before the election (just a guess ;I think they could still do a low level attack but I think we have thwarted a major 9-11 type attack ). The intelligence leads are developing quickly.

2. Afghanistan : democracy is slowly taking root there . National elections are coming in spite of repeated attempts to stop them. U.S. forces have continued their dogged pursuit of al Qaeda and the remnants of the Taliban . It would be useful if Bin Laden and Mullah Omar were killed or captured but they are but 2 heads of the hydra at this point. Al-Qaeda was never a centralized entity as the 9-11 Report explained . Zarqawi in Iraq ,and other members have assumed operational control and are a greater threat to the U.S. then bin Laden is . His capture would be a symbolic blow,but the fight would not end there. It is doubtful that progress would have gone faster had the U.S. not put Iraq in the cross hairs.

3.Al qaeda is on the run around the world ;in the Middle East , North Africa, Southwest Asia, Central Asia, Southeast Asia, Europe and North America .We have mustered the forces of homeland security ,diplomatic pressure ,alliance building , covert operations, security force training , special operations and conventional combat against the enemy . Anyone who says that we are going it alone is not looking at the total picture . After WWII new alliances needed to be formed to deal with new threats . For reasons of their own national interests (I think) ,some of the nations who allied themselves with us against the Soviet threat have chosen to not do so in certain areas of this war.In other areas of the war they have been very helpful (.Intelligence sharing did not suffer as a result of the Iraq war . ) We have forged new alliances with nations who share the threat.
-Pakistan was once a nation that harbored and supported terror . Now they actively pursue terrorist and battle them on their Afghan border . They are making for the first time gestures of peace with their fellow nuclear neighbor India . They were once proliferators of nuclear technology ;now they are fighting to stop its spread.
- Saudia Arabia has been forced to deal for the first time with the extremist inside their country . Perhaps they now realize that the monster they have created will not hesistate to turn on them .
-Libya is another example of a nation that has changed sides . For how long? I dont know,but useful information about the proliferation of WMD has come from Kadaffis recent conversion (including I might add Saddams role in the development of the so called Islamic Bomb)

In all these cases Bush can point to real accomplishments :In fighting international terrorism ,and preventing nations from giving them safe haven ; In the prevention of WMD proliferation ;In planting the seeds of democracy in a region where it has not flourished .

I have been waiting for Kerry to suggest a better alternative but all he proposes is to do it in a gentler way. It is too bad . A national debate on the best way to combat the threat would be a good thing. This war will last well past the President and the next President. . Im not sure it can happen with the present political climate in America but it would really be useful if we started off with a national consensus on how to deal with possibly the greatest threat of our age .

Chouxxx rated this answer Excellent or Above Average Answer
ETWolverine rated this answer Excellent or Above Average Answer

Question/Answer
Yiddishkeit asked on 08/09/04 - Oui Oui...

CNN is reporting that Lance Armstrong may be stripped of his 6th Tour de France title.

In a random check for the banned substances, 3 were found in Armstrong's hotel room.

The 3 substances banned by the French, that were found in his hotel room were as follows:

(1) Toothpaste
(2) Deodorant
(3) Soap




Mother sent me this joke...Bobby

tomder55 answered on 08/09/04:

my wife and family are French . let's just say that her grand dad was in the resistance and here family is not from the Vichey/Chirac block of French society.

Chouxxx rated this answer Excellent or Above Average Answer
Yiddishkeit rated this answer Excellent or Above Average Answer

Question/Answer
Chouxxx asked on 08/08/04 - Laws against Gay Marriage/Unintended Results

The religious right is so angry about the fact that it is almost certain that the Amendment to The Constitution banning gay marriage most certainly fail, that they tried a new tactic. If we(Religious Right) can't amend the Constitution, we can make it irrelevant!
The US HOuse of REpresentatives recently approved a bizarre bill callen the Marriage Protection Act which would insulate state laws banning gay marriage from any review by federal courts, including the Supreme Court. If the bill becomes law, it will change the way our government has operated for more than two centuries.

Stripping the Supreme Court of their power is a drastic step and would most certainly come back to haunt conservatives.

Liberals could pass stringent laws in several states banning gun ownership--and then prohibit federal courts from deciding whether those laws violated the Second Amendment.

Comments?

tomder55 answered on 08/09/04:

I still maintain that the issue is a Federal and not a State one .Article Four Section one of the Constitution says :

Full faith and credit shall be given in each state to the public acts, records, and judicial proceedings of every other state. And the Congress may by general laws prescribe the manner in which such acts, records, and proceedings shall be proved, and the effect thereof.

The 10th Amendment doesn't supercede this . This means that if one State were to recognize a gay marriage all States must.

There is precident for Federal law to supercede State law on the marriage issue . Utah was not admitted into the union until bigamy was outlawed in the State .

The Marriage Protection Act is designed to prevent the courts from finding DOMA unconstitutional not State laws .But because there are provisions in DOMA that are unconstitutional in my view because of the same 'full faith and credit 'clause (DOMA says that no State need give recognition to a same-sex marriage from another State)there is a need for a Federal definition of marriage . I think that an amendement is the only alternative .

Chouxxx rated this answer Excellent or Above Average Answer

Question/Answer
Chouxxx asked on 08/08/04 - Oil, the Euro, Trade Balance

I can think of few things more complicated than Ecomonics, World Econmics. Today I read another take on one reason for a War on Saddam(Iraq).

Oil is priced in American dollars. Iran had somewhat recently decided to price oil in Euro-s, and Saddam of Iraq was just beginning the process to change pricing its oil in Euro-s. Pricing oil in Euro's would be a potental economic disaster for America. America would be subject to the fluxuations in a foreign currency value and the potential for huge trade deficits could ruin America's economy.

So now? Is America at war with Islamic terrorists and their hidden sympathizers like France and Germany in order to save our economy? With those who want to bring America down by ruining our economy? Thus.....?????

[Great Britain is not on the Euro; they are still on the pound sterling.]

tomder55 answered on 08/09/04:

I am not a monetary expert but this is my take on this . I first heard this theory on Air America ,and did some reading on it . There are a number of web-sites that support this but it doesn't ring true because of the following:

Iraqi Liberation Act- 1998
Saddam switches to Euro- 1999.The US policy of regime change happened before Saddam decided to convert to Euro's .In fact the neo-cons at PNAC were talking regime change even before 1998. If you consider that the U.S. was at war in Iraq for 12 years running (including all the times Saddam violated the cease fire agreement)then our conflict with Saddam began before the euro existed .


This Guardian article explains better than I can why this conspiracy theory is not valid.

http://www.guardian.co.uk/business/story/0,3604,949435,00.html




It is not apparent to me that the U.S.wants to maintain a strong dollar.The Treasury Department and the Federal Reserve is allowing the dollar to drop.Bush pays lip service to a strong dollar but has not favored intervention. He has 'let the market work'.Besides ;it is the strong dollar that is causing trade deficits . A weak dollar would mean that Americans could purchase less imports for the buck.


In the US, oil prices are way up . Oil is up only slightly up in Europe.It is because the dollar has dropped against the euro.If it was Bush's plan to shore up the dollar by the invasion it didn't work very well.

As to the euro; I do not expect it continue to be a force for long. It rises and falls as markets dictate but it is the currency of a very weak confederacy. I do not expect the EU to become a strong Republic anytime soon ,and it's individual members are still primarily interested in their own national interests (France being one of the biggest violators http://msnbc.msn.com/id/5197826/site/newsweek/.)

ETWolverine rated this answer Excellent or Above Average Answer
purplewings rated this answer Excellent or Above Average Answer
Chouxxx rated this answer Excellent or Above Average Answer

Question/Answer
HANK1 asked on 08/08/04 - SOCIAL SECURITY:



(This must be an issue in ��")

SOCIAL SECURITY:

(This is worth reading. It is short and to the point.)

Perhaps we are asking the wrong questions during election years. !

Our Senators and Congresswomen do not pay into Social Security and, of course, they do not collect from it.

You see, Social Security benefits were not suitable for persons of their rare elevation in society. They felt they should have a special plan for themselves. So, many years ago they voted in their own benefit plan.

In more recent years, no congressperson has felt the need to change it. After all, it is a great plan.

For all practical purposes their plan works like this:

When they retire, they continue to draw the same pay until they die. Except it may increase from time to time for cost of living adjustments.

For example, former Senator Byrd and Congressman White and their wives may expect to draw $7,800,000.00 (that's Seven Million, Eight-Hundred Thousand Dollars), with their wives drawing $275,000.00 during the last years of their lives.

(This is calculated on an average life span for each of those two Dignitaries)

Younger Dignitaries who retire at an early age, will receive much more during the rest of their lives. Their cost for this excellent plan is $0.00. NADA....ZILCH....

This little perk they voted for themselves is free to them. You and I pick up the tab for this plan. The funds for this fine retirement plan come directly from the General Funds;

"OUR TAX DOLLARS AT WORK"!

From our own Social Security Plan, which you and I pay (or have paid) into, -every payday until we retire (which amount is matched by our employer)- we can expect to get an average of $1,000 per month after retirement.

Or, in other words, we would have to collect our average of $1,000 monthly benefits for 68 years and one (1) month to equal Senator! Bill Bradley's benefits!

Social Security could be very good if only one small change were made.

That change would be to jerk the Golden Fleece Retirement Plan from under the Senators and Congressmen. Put them into the Social Security plan with the rest of us ... then sit back and watch how fast they would fix it.

Spread the word, guys!

HANK

tomder55 answered on 08/09/04:

you make a valid point. There has been some ideas floated ,and they quickly get buried in procedures ;they never get serious debate.

In May, 2001, the President appointed a bipartisan Commission to Strengthen Social Security, co-chaired by the late Senator Daniel Patrick Moynihan and by Richard Parsons. The sixteen-member Commission, composed of eight Republicans and eight Democrats, issued a unanimous report of recommendations to strengthen Social Security. These recommendations, if implemented, would establish voluntary personal accounts for workers, and permanently fix Social Security without changing benefits for current seniors or raising payroll taxes. [from GeorgeWBush.com]Basically Bush's plan would not change the benefit status of current retirees or near-retirees(I wonder who is considered a 'near-retiree'?).

He wants to semi-privatize it for the rest by setting up 'voluntary personal accounts 'within Social Security. These accounts would allow you to invest part of it as you choose.Not a terrible idea by any means .SS now gives todays workers a much lower return than market rates .

He would NOT raise payroll taxes to cover the SS shortfalls .

Kerry during the convention said :``I will not privatize Social Security,'' he declared in his acceptance speech to the Democratic National Convention. ``I will not cut benefits.'' Ok ;then what will he do? There are really only three options for Social Security reform: raise taxes, cut benefits or invest privately. He has ruled out 2 of the options ;privatizing ,and cutting benefits .All that is left is raising taxes to cover it (his rolling back of tax cuts to the wealthy will not come close to cover all the government spending he has already promised) .

Bush hasn't moved on his plan since he proposed it ,and Kerry has a non-plan.Looks like there will continue to be a dearth of leadership on this issue.

HANK1 rated this answer Excellent or Above Average Answer

Question/Answer
Chouxxx asked on 08/07/04 - Illinois Republicans select

a Maryland man to run for the US Senate after Jack Ryan quits race over sex scandal. [Many may not know that the Republican party in Illinois which has been in power for the better part of 30 years is in disarray due to the usual political excesses and corruption.]

Keyes has been a regular Republican/Conservative gadfly over the years; kinda like a really hyper version of William F. Buckley.

My question? How can a guy from Maryland run for Senate in Illinois. Isn't there some sort of residency requirement. (Those who want to talk about Hilary, save it! lol)

tomder55 answered on 08/08/04:



Residency requirements have been in issue in past elections in U.S. history.Since we have a mobile nation it has not been unusual for ambitious people to move to areas of opportunity. Lincoln was resident in Kentucky to Indiana and Illinois at stages of his career.Illinois famous pol. James Semplar moved from Kentucky to Illinois, then to Missouri, back to Kentucky, got a law degree and returned to Illinois, where he served as state attorney general, state representative and House speaker; then to Bogota, Colombia, as charge d'affaires, back to Illinois for election as state Supreme Court justice and then to the Senate according to the Chicago Sun Times.

I was not able to find the specific residency requirement for the State but I believe all he has to do is be a resident before election day.If it is as lax as N.Y. is then Bobby Kennedy setting up residence in a N.Y.City hotel room was sufficient for him to get elected. Hillary at least purchased a home that to my knowledge she has never been seen in since she won .But she fullfilled the requirements .

If Keyes decides to run you will probably be treated to one of the greatest debates or series of debates in the history of the country.Keyes stands on the issues sharply contrast to those of Obama.Keyes is a good man with strong moral positions. I agree with many of the positions he stands for ,but he may be too inflexible for a Senator.

It shows how desperate the GOP is to get a candidate.First Ryan and then Barthwell's sexcapades have already made this Senate run very interesting.With Obama being propelled to national status with the keynote address I guess the GOP feels compelled to field a nationa figure to challange him.But I do not think it will work. I think Obama wins in a landslide no matter who the GOP can muster.

Keyes is currently a radio talk show host .So there are alot of public statements out there fo an opponent to challenge him on ;including this :

"I deeply resent the destruction of federalism represented by Hillary Clinton's willingness go into a state she doesn't even live in and pretend to represent people there, so I certainly wouldn't imitate it." That is from Alan Keyes three years ago.

Chouxxx rated this answer Excellent or Above Average Answer
purplewings rated this answer Excellent or Above Average Answer

Question/Answer
Chouxxx asked on 08/05/04 - Bill O'Reilly Upset

Fox television personality Bill O'Reilly was upset at an ad[obviously funded by Republican money] that showed vietnam veterans bashing Kerry for his service in VietNam. They didn't even serve with Kerry. Kerry saved the life of a man, his friend, who served with him on a Swift boat.

O'Reilly is upset that this ad can and will probably will alienate many voters who know the truth.

In addition, any talk of service reminds voters how Bush spent most of the time during the VietNam war drunk, didn't report to his Nationl Guard unit as required, and ws in a drunk driving accident which he caused!!

Partisans and crypto-fascists, do you think that this kind of non-sense will win Bush any votes?

DON'T YOU THINK PEOPLE WILL VOTE *FOR* BUSH BECAUSE OF HIS POLICIES[WAR AGAINST ISLAM-O-TERRORISM] AND *IN SPITE* OF HIS CHARACTER?????

I do.

tomder55 answered on 08/06/04:

I have stated that I wish they would avoid the war years . It is true that some of the swift boat people who are in the add were not on Kerry's boat but all were in his unit .No boat goes out on a mission alone . There are many eye witnesses to Kerry's tour who dispute the accounts of his exploits. This is not new .I have known about questions about the 'facts ' surrounding 2 of his purple hearts ,his bronze star and other aspects of his service.

I can go into details ,but I would like instead to talk about the merits of his proposals and his 20 years of inaction in the Senate instead of his 4 months of service. It is Kerry who paraded out his 'band of brothers' during the convention. In my view there has been way too much emphasis on his service. I think it is poor strategy on his part. It only rehashes the whole antiwar stuff he did after his time in Nam.

I would take his record at his word . He said :"Yes, I commited the same kind of atrocities as thousands of other soldiers had committed, in that, I took part in shootings in free fire zones, I conducted harassment and interdiction fire, I used 50 caliber machine guns which we were granted and ordered to use, which were our only weapon against people, ahh, I took part in search and destroy missions, in the burning of villages. All of this is contrary to the Laws of Warfare, all of this is contrary to the Geneva Conventions." (John Kerry, April 18th, 1971).

How are the hundeds of thousands of vets of Nam supposed to feel about that ? Why is it that a few vets who support him are allowed to campaign for him but 'The Swift Boat Veterans for Truth'are supposed to be silent? Did they not also earn the right to be heard ?

O'Reilley prides himself in conducting a'no spin zone'. Does he not see the fairness in having all sides of the debate aired?

I agree with you that it would be better for both candidates to downplay their service time .It is telling that both have been extremely reluctant to release their repsective service records. I do not completely agree with your characterization of Bush's Guard duty . He trained as a jet fighter .He logged in many hours of flight time.This proves that his training was not the piece of cake. The controversy about a couple of months has already been aired by the same people who now object to Kerry's service time being scrutinzed.

Now everyone knows the questions .I would leave it at that myself ,but if Kerry keeps trumpetting his service record as his proof of leadership then he should expect to have his service record looked over in detail. John McCain or Bill O'Reilley have no business calling it out of bounds under those conditions.

Chouxxx rated this answer Excellent or Above Average Answer
excon rated this answer Excellent or Above Average Answer
Itsdb rated this answer Excellent or Above Average Answer

Question/Answer
kindj asked on 08/03/04 - This is kinda weird...

Someone at the Christianity board posted this, so please DON'T JUMP ON ME!!!!

I just want to get some thoughts on it, as it seems sort of Orwellian to me...

<><><><><><><><><><><><><><><><><><><><><><

Equal time ...... Uni-Agdistis 08/03/04

Bush or Kerry ....Kerry or Bush.

.

Bush Planned Iraq 'Regime Change' Before Becoming President


By Neil Mackay

15 September 2002: A SECRET blueprint for US global domination reveals that President Bush and his cabinet were planning a premeditated attack on Iraq to secure 'regime change' even before he took power in January 2001.

The blueprint, uncovered by the Sunday Herald, for the creation of a 'global Pax Americana' was drawn up for Dick Cheney (now vice- president), Donald Rumsfeld (defence secretary), Paul Wolfowitz (Rumsfeld's deputy), George W Bush's younger brother Jeb and Lewis Libby (Cheney's chief of staff). The document, entitled Rebuilding America's Defences: Strategies, Forces And Resources For A New Century, was written in September 2000 by the neo-conservative think-tank Project for the New American Century (PNAC).

The plan shows Bush's cabinet intended to take military control of the Gulf region whether or not Saddam Hussein was in power. It says: 'The United States has for decades sought to play a more permanent role in Gulf regional security. While the unresolved conflict with Iraq provides the immediate justification, the need for a substantial American force presence in the Gulf transcends the issue of the regime of Saddam Hussein.'

Much more at:

http://www.informationclearinghouse.info/article1221.htm

><><><><><><><><><><><><><><><><><><><><><><><>
It's back to me again. Anyone know anything about this "information clearing house" site? Reliable place? No? Depend on your viewpoint?

DK

tomder55 answered on 08/04/04:

I don't know about the'information clearing house 'and do not care their conclusion is wrong . The PNAC is real and has existed for a number of years ;and yes some prominent members of the Administration are in it;including dare I say the name,Wolfowitz.

They argued for regime change in Iraq for years ;long before it became the official policy of the U.S. by Congrssional decree in 1998. ("Iraq Liberation Act of 1998" Public Law 105-338)

Before his State of the Union Address in 1998 they sent Clinton a letter concerning Iraq.It stated:

The policy of containment of Saddam Hussein has been steadily eroding over the past several months. As recent events have demonstrated, we can no longer depend on our partners in the Gulf War coalition to continue to uphold the sanctions or to punish Saddam when he blocks or evades UN inspections. Our ability to ensure that Saddam Hussein is not producing weapons of mass destruction, therefore, has substantially diminished. Even if full inspections were eventually to resume, which now seems highly unlikely, experience has shown that it is difficult if not impossible to monitor Iraqs chemical and biological weapons production. The lengthy period during which the inspectors will have been unable to enter many Iraqi facilities has made it even less likely that they will be able to uncover all of Saddams secrets. As a result, in the not-too-distant future we will be unable to determine with any reasonable level of confidence whether Iraq does or does not possess such weapons.

The policy of containment of Saddam Hussein has been steadily eroding over the past several months. As recent events have demonstrated, we can no longer depend on our partners in the Gulf War coalition to continue to uphold the sanctions or to punish Saddam when he blocks or evades UN inspections. Our ability to ensure that Saddam Hussein is not producing weapons of mass destruction, therefore, has substantially diminished. Even if full inspections were eventually to resume, which now seems highly unlikely, experience has shown that it is difficult if not impossible to monitor Iraqs chemical and biological weapons production. The lengthy period during which the inspectors will have been unable to enter many Iraqi facilities has made it even less likely that they will be able to uncover all of Saddams secrets. As a result, in the not-too-distant future we will be unable to determine with any reasonable level of confidence whether Iraq does or does not possess such weapons.

Such uncertainty will, by itself, have a seriously destabilizing effect on the entire Middle East. It hardly needs to be added that if Saddam does acquire the capability to deliver weapons of mass destruction, as he is almost certain to do if we continue along the present course, the safety of American troops in the region, of our friends and allies like Israel and the moderate Arab states, and a significant portion of the worlds supply of oil will all be put at hazard. As you have rightly declared, Mr. President, the security of the world in the first part of the 21st century will be determined largely by how we handle this threat.source PNAC website

The letter went on to say :
In any case, American policy cannot continue to be crippled by a misguided insistence on unanimity in the UN Security Council.

Clinton understood that the policy of containment was a failure. The choices were to abandon the region ,and let Saddam free to reconstitute his WMD build up,to continue a failing policy or to start a new policy of regime change.They urged Clinton to act ;and of course he did in his ususal half measured way. He signed off on the new American policy of regime change;launched a barrage of cruise missles almost a year after he got the letter from PNAC ,and called it a day.His attempt was half-assed ,but his policy was certainly the correct one . This is waht he said back then:

"Just consider the facts,"
"Iraq repeatedly made false declarations about the weapons that it had left in its possession after the Gulf War. When UNSCOM would then uncover evidence that gave the lie to those declarations, Iraq would simply amend the reports. For example, Iraq revised its nuclear declarations four times within just 14 months and it has submitted six different biological warfare declarations, each of which has been rejected by UNSCOM. In 1995, Hussein Kamal, Saddam's son-in-law, and chief organizer of Iraq's weapons-of-mass-destruction program, defected to Jordan. He revealed that Iraq was continuing to conceal weapons and missiles and the capacity to build many more. Then and only then did Iraq admit to developing numbers of weapons in significant quantities and weapon stocks. Previously, it had vehemently denied the very thing it just simply admitted once Saddam Hussein's son-in-law defected to Jordan and told the truth."

"Now listen to this: What did it admit? It admitted, among other things, an offensive biological warfare capability--notably 5,000 gallons of botulinum, which causes botulism; 2,000 gallons of anthrax; 25 biological-filled Scud warheads; and 157 aerial bombs. And might I say, UNSCOM inspectors believe that Iraq has actually greatly understated its production.

Next, throughout this entire process, Iraqi agents have undermined and undercut UNSCOM. They've harassed the inspectors, lied to them, disabled monitoring cameras, literally spirited evidence out of the back doors of suspect facilities as inspectors walked through the front door. And our people were there observing it and had the pictures to prove it. "
"We have to defend our future from these predators of the 21st century," he argued."They will be all the more lethal if we allow them to build arsenals of nuclear, chemical, and biological weapons and the missiles to deliver them. We simply cannot allow that to happen. There is no more clear example of this threat than Saddam Hussein."

The PNAC from the outset was clear in it's goals ."Of course, the United States must be prudent in how it exercises its power. But we cannot safely avoid the responsibilities of global leadership or the costs that are associated with its exercise. America has a vital role in maintaining peace and security in Europe, Asia, and the Middle East. If we shirk our responsibilities, we invite challenges to our fundamental interests. The history of the 20th century should have taught us that it is important to shape circumstances before crises emerge, and to meet threats before they become dire. "




purplewings rated this answer Excellent or Above Average Answer
kindj rated this answer Excellent or Above Average Answer
ETWolverine rated this answer Excellent or Above Average Answer
Yiddishkeit rated this answer Excellent or Above Average Answer

Question/Answer
Chouxxx asked on 08/02/04 - Irrational Power Brokers

Until 1979, the average resident of the Western world had cause to believe that the rational pursuit of kowledge was the shared vocation of all humanity. At the time, the strict market economics of Britain's Margaret Thatcher and the fundamentalist Islam of Iran's Ayatollah Khomeini did not appear to have much in common. But, their almost simultaeouos ascents were harbingers of a revolutionary retreat from the Enlightenment faith in reason. Ever since, the citizenry of the civilized world has been frightenly susceptible to all mnner of quack spiritualism, moral obsfucation, economic snake oil, and emotional pandering.*

Not my vision for the future of America or the world. And, I'm a warrior!!

Comments or not...

Choux

*New Book, "Idiot Proof: Deluded Celebrities, Irrational Power brokers, Media Morons and the Erosion of Common Sense" by Francis Wheen. OH, Francis is a man for all you deluded right wing nazis! lol

tomder55 answered on 08/03/04:

A quick review of his work shows that he mocks both the leftist new age ideas as well as fundamentalist notions.
There is enough for all folks to be entertained and offended;to agree and to disagree. He spares no one but offers no solution except to"just think a little more and believe a little less." real deep analysis!

His previous books have shown preferences to socialism/Marxism so it doesn't suprise me that he lumps capitalism into what he defines as idiocy .

I like the cover of the book it puts Hillary right next to the Ayatolla .

Chouxxx rated this answer Excellent or Above Average Answer
excon rated this answer Excellent or Above Average Answer

Question/Answer
SCOOBY asked on 08/01/04 - candidate

Hello excon,
Did I understand correctly that one of the presidential candidates is a veteran of war who actually was on the battle fielding killing?

tomder55 answered on 08/02/04:

If the Democrats did nothing else last week ,they proved beyond a doubt that Kerry served in Vietnam. The unanswered question remains ;Why should we entrust the security of the nation to him? It was clear to me from his speach that he intends if elected to keep America on the defensive. No preemption;no wars of 'choice'. Wait until we are attacked,or an attack is 'imminent'whatever that means.

excon rated this answer Excellent or Above Average Answer
purplewings rated this answer Excellent or Above Average Answer
SCOOBY rated this answer Excellent or Above Average Answer

Question/Answer
demie asked on 07/30/04 - Kerry's acceptance Speech

Did Kerry's speech last night make anyone on this board change their mind as to who to vote for, who would be the better man? If so, why?
Demie

tomder55 answered on 07/30/04:

It did not really change my mind;only reinforced my misgivings about him .I have been waiting for him to detail an effective policy for leading the nation in the war on terrorism ,and he instead resorted to language that bolsters my belief that he is planning on running a Presidency as if it was Sept.10,2001 .

He claims that he will rebuild "our alliances." He said "we need a president who has the credibility to bring our allies to our side". Besides the obvious insult to the British,Italians ,Poles ,S.Koreans ,Australians and the many other nations that have helped us in Iraq and Afghanistan ;which nations is he talking about? France and Germany no doubt.Those nations had their own motives for opposing the war ,as the 'oli for food 'scandal has revealed . In other aspects of the war the Bush Adm. has done a great job of forging alliances not only with those allies that Kerry refers to ,but with nations that have NEVER cooperated with us before .In no way has the war been conducted in a unilateral way.

Kerry last night confirmed to me that he is inclined to defer the U.S. interests to the wishes of the 'international community '. That is unacceptable to me .

demie rated this answer Excellent or Above Average Answer
Itsdb rated this answer Excellent or Above Average Answer
Chouxxx rated this answer Excellent or Above Average Answer

Question/Answer
Chouxxx asked on 07/30/04 - Crack, Zoom

That is the sound of John Kerry hitting a home run with his acceptace speech at the Democratic National Convention last night. He reached the people he intended to reach, and the Convention was extremely well run, and the solidarity! didn't even look like Democrats! We are off to the races, citizens!

tomder55 answered on 07/30/04:

Kerry said he wouldn't go to war "because we want to, we only go to war because we have to."When Kerry combines that line with another that he said ;a tough sounding "I will never hesitate to use force when it is required. Any attack will be met with a swift and certain response" leads me to believe that he is inclined to wait until we are attacked . That is not the President want to entrust the safety of the nation to .

Last night Joe Lieberman tried in futility once again to try to make his party understand the nature of the threat.Besides giving him the courtesy of limited polite applause ,his message fell on deaf ears .Joe tried to tie his message to the Kerry candidacy but Kerry quickly dashed that by tying the war in Iraq to the persuit of oil ("no young American in uniform will ever be held hostage to our dependence on oil from the Middle East.").He just as well should've had Michael Moore as his spokesman. Kerry says we should trust him to guide the fortunes of an America at war because "I defended this country as a young man and I will defend it as president." i wil not argue on the various aspects of his truncated tour of duty but will say ; Sorry ;there are plenty of brave veterans who have served honorably who would not make good Presidents . The nation has turned them down in the past . They will do so again.


CeeBee rated this answer Excellent or Above Average Answer
Chouxxx rated this answer Excellent or Above Average Answer
Itsdb rated this answer Excellent or Above Average Answer

Question/Answer
_JacquelineA asked on 07/30/04 - Question on Bush again

My boyfriend thinks that George Bush is a racist. He told me this after I told him I liked him. I disagree with him. What do you think?
He said the government put all those liquor stores and drugs in the inner cities to keep the black people from making it in life.

tomder55 answered on 07/30/04:

Liquor stores were in the cities long before Bush was alive. If I go to a rural liquor store ;is that also there to prevent me from making it ?or is it just that your boy friend does not think that 'black people' are capable of making lifes decisions on their own ? Sounds like he is the racist to me. Tell your boy friend that 2 of Bush's highest advisers Codi Rice National Security Advisor and Colin Powell Secretary of state are black.

ETWolverine rated this answer Excellent or Above Average Answer
excon rated this answer Excellent or Above Average Answer
labman rated this answer Excellent or Above Average Answer
purplewings rated this answer Excellent or Above Average Answer
_JacquelineA rated this answer Excellent or Above Average Answer

Question/Answer
Meezan asked on 07/29/04 - Liquer was once banned in USA! When? How? Why?

Hi,

Is it true that at some time in history of America, Liquer was completely banned. If yes, when was that. Was it a step from government? How did it effect America's economy? Was that the result of any movement or if a single person was behind this? Feel free to send in the details.

Regards.

Ali.

tomder55 answered on 07/29/04:

I used this clarification on another reply but it should add useful information to what the other experts have already supplied :

Before prohibition ,a substantial number of people in the U.S. drank booze. Historians believe that in the beginning of the era, and at least until the middle of the 1920s, most Americans respected the prohibition laws, hoped that it would endure, and regarded its passage as directly responsible for the reduced incidence of public drunkenness and of alcohol-related crime, imprisonments, and hospitalizations. Statistics show that Prohibition reduced the annual per capita consumption from 2.6 gallons of absolute alcohol to 0.97 gallons .Moreover, there was no real evidence of a crime wave during the 1920s due to prohibition , although the crime rate did in fact rise. The average death rate from cirrhosis of the liver was 7.3 per 100,000 in the years 1920-1933; the average rate for the rest of the 20th century was 11.5. Admissions to state mental hospitals for alcohol psychosis also fell from 10.1 per 100,000 in 1919 to 4.7 per 100,000 in 1928.

In the late 1920s, however, more and more Americans found the idea of repeal increasingly attractive. The reasons for this were numerous, the governments failure to enforce the law being only one of them. Most Americans were happy that the old time saloon had been abolished, but they felt that a new society was emerging in the 1920s an urban and industrial society of great geographic and social mobility and great ethnic and religious diversities, in which the protection of the family from alcohol was perhaps less socially urgent than the expansion and protection of individual freedom. A major shift in public opinion occurred during the Great Depression, when opponents could argue that Prohibition deprived people of jobs and the governments of tax revenue. The annual per capita consumption of absolute alcohol in the country rose after the repeal, but most states still retain restrictions on the sale and consumption of alcohol. Today there is a national age limit. No one under 21 may purchase alcohol . All states have licensing restrictions on the merchants ,and may prohibit the sale on certain days like Sunday,or restrict the sale to stores that specialize in the sale of alcohol.

ETWolverine rated this answer Excellent or Above Average Answer
Meezan rated this answer Excellent or Above Average Answer
purplewings rated this answer Excellent or Above Average Answer
Yiddishkeit rated this answer Excellent or Above Average Answer

Question/Answer
chekhovToo asked on 07/28/04 - How the Americans and British got it all so wrong

The Economist July 17, 2004.

How the Americans and British got it all so wrong

ON THE evening of July 11th, George Tenet lit a fat cigar and loitered in the compound of the Central Intelligence Agency in Langley, Virginia. His last hours as America's spy chief were passing painfully. According to the Senate Intelligence Committee, in a report released two days earlier, America invaded Iraq on the strength of intelligence that was out-of-date, inaccurate, badly analysed and, in short, did not justify the nub of George Bush's case for war.

America did not, as Mr Bush had claimed, have conclusive evidence that Iraq had retained and replenished its chemical and biological weapons, or that it was reactivating its nuclear programme. Nor, from any angle, could the agency find an alliance between Saddam and al-Qaeda. Mr Tenet was seen to pause by a garden feature: a hefty slab of the Berlin Wall. Perhaps the CIA's failure to predict the Soviet Union's collapse seemed suddenly less burdensome. On that, the agency was merely short-sighted. On Iraq, it appears to have been hallucinating.

Mr Tenet's decision to resign "for personal reasons" last month now looks wise. His farewell party was held on the eve of the report's release. But for John Scarlett, the best known British spy, the price of failure seems to be promotion. On July 14th, a British inquiry into intelligence on Iraq's putative weapons of mass destruction (WMD), headed by Lord Butler, a former top civil servant, delivered a broadly similar verdict to the Senate committee's, but in a kindlier tone. Where the senators' report barely contained their dismay at the CIA's ineptitude, his lordship's criticism was more of a finger-wagging, with much talk of collective responsibility.

Mr Scarlett, the newly appointed head of Britain's Secret Intelligence Service (known as MI6) and former head of the Joint Intelligence Committee (JIC), was one of the few officials named in the report. In his old role, Mr Scarlett was responsible for the government's "dossier" on Iraq's WMD, published in September 2002, which has proved to be misleading and substantially false. There have been calls for Mr Scarlett's head, which Lord Butler urged the government to ignore.

The British agency's record on Iraq does withstand scrutiny a bit better than that of its American counterparts. The Senate found that the flotilla of intelligence agencies under Mr Tenet, the director of central intelligence, were gullible and incompetent, reflecting a "broken corporate culture and poor management". They had no agents in Iraq since 1998, and even before then relied heavily on information supplied by UN weapons inspectors (some of whom were moonlighting as spies). By September 2002, when the agencies were told to draft a National Intelligence Estimate (NIE) of Iraq's weapons of mass destruction, which were a key exhibit in Mr Bush's case for war, most of what they "knew" about weapons programmes pre-dated the first Gulf war in 1991.

MI6 did have five agents inside Iraq, but they seem hardly to have improved its intelligence. According to Lord Butler, only two of those agents were wholly reliable; their assessments of Iraq's WMD capabilities were "less worrying than the rest". Intriguingly, Lord Butler notes that intelligence obtained from another agency on Iraq's biological weapons was withdrawn after it was found to be seriously flawed, owing to "misunderstandings".

Making the most of a hard case

In the autumn of 2002, British and American intelligence services found themselves in a similar fix. For years, they, like every other western intelligence service, had advised their governments that Iraq's WMD programmes were a persistent menace. Now, they were being instructed to make that case fully and publicly, and suddenly it didn't look quite such a "slam-dunk," as Mr Tenet is said to have described it.

This was for two reasons. First, spies rarely produce the sort of clinching evidence beloved of investigative journalists and fictional detectives. Intelligence tends to be fragmentary and accretive. It does not lend itself to Powerpoint presentations. Second, due to the difficulties of operating in Iraq and their own deficiencies, both services were pitifully wanting in good current intelligence. In addition, the CIA's job was made especially difficult by Mr Bush and his more warlike followers. They were already making fiery claims about Saddam Hussein and his alleged alliance with al-Qaeda which the agency was unable to substantiate.

Nonetheless, the CIA tried not to disappoint. Into the NIE, according to the Senate report, went a shower of dodgily-sourced and overstretched intelligence. In the estimate's declassified version, possibles became probables, and caveats were cut, as scraps of potentially alarming intelligence emerged as terrifying facts.

Numerous qualifiers--"we judge that", "most analysts believe that"--were excised. Thus were reasonable, though wrong, intelligence suppositions reborn as erroneous facts. To the (now discredited) claim that Iraq was developing its ability to deliver biological weapons was attached the dread phrase, "including potentially against the US homeland."

Much of the intelligence used in the British dossier was also denuded of important qualifiers. This was "a serious weakness", according to Lord Butler, which put more weight "on the intelligence than it could bear". In a perverse example of this, an original statement that the JIC had little information about Iraq's WMD programmes since 1998, was rewritten to suggest that the resulting uncertainty was itself a cause for suspicion. Lord Butler found that the dossier's judgments "went to (although not beyond) the outer limits of the intelligence available."

Mr Powell's smoking guns

In February 2003, Colin Powell, the secretary of state, delivered a presentation to the UN Security Council that has been credited with convincing many sceptics of the case for war. Mr Powell outlined "many smoking guns" in Iraq, including a fleet of mobile biological weapons laboratories. All of these, he said, had been identified with the use of solid intelligence, corroborated by multiple sources. But the Senate committee found otherwise.

Shortly before Mr Powell delivered his speech, a CIA agent read a draft version of it, and reacted with horror. According to e-mails provided to the committee, the agent identified himself as being the only American agent to have interviewed the main source behind the mobile-lab intelligence. He considered the source, codenamed Curve Ball, an unreliable drunkard whose identity was not yet established. Three additional sources who were supposed to have corroborated Curve Ball's claims were either known "fabricators" or had not, in fact, corroborated the claims at all. In reply to these concerns, the agent was told by a senior CIA official: "Let's keep in mind the fact that this war's going to happen regardless of what Curve Ball said or didn't say, and that the Powers That Be probably aren't terribly interested in whether Curve Ball knows what he's talking about."

Similarly slapdash, the British dossier contained the hair-raising claim that Iraq could deploy WMD in a mere 45 minutes. This triggered apocalyptic newspaper headlines across the country. "Brits 45 mins from Doom," screamed the Sun, a popular British tabloid.

The dubiousness of the 45-minute claim (which, it later emerged, applied only to battlefield weapons) has dominated several earlier inquiries into Britain's pre-war intelligence. Hours before Lord Butler's report was released, Iraq's new prime minister, Ayad Allawi, sheepishly told the BBC that his emigre group was behind the claim. Speculation lingers that Mr Allawi's supporters lifted it from the user-guide for a Soviet missile. Lord Butler said that it should not have appeared in the form it did in the dossier.

Both the American and British reports put the agencies' mistakes down to incompetence, not intent. Though consistently wrong on almost every count where Iraq's WMD were concerned, the spies apparently thought they were right. Lord Butler attributes this partly to the fact that Mr Hussein had made fools of the spies often before. There was, he said, "a tendency for assessments to be coloured by over-reaction to previous errors," ensuring that the worst-case scenario ruled.

Faith-based intelligence

The Senate committee explained this phenomen in a colder light. It diagnosed a severe case of "group-think": that is, that the spies were failing to test the general assumption that Iraq had a growing WMD programme. To have done so would have been considered heresy; which may be why Hans Blix, the chief UN weapons inspector, accused America of positing "faith-based intelligence."

Thus programmed, America's spies tended to reject any intelligence that didn't support the thesis. Whatever did corroborate it, they embraced, with little regard to the credibility of the source. Accordingly, defectors who claimed that Iraq had abandoned its pursuit of nuclear weapons by the mid-1990s, were dismissed as untrustworthy. Exiled opposition politicians and their relatives--including the dubious Ahmed Chalabi who had not visited Iraq in decades--were considered more reliable. And egged on by men with a keen interest in demonising Saddam, the CIA (and to a lesser degree the British agencies) were compromised at every stage of their intelligence collection and analysis.

When, in 2002, Iraq was found to be importing aluminium tubes that could theoretically be used to make uranium centrifuges, both the CIA and the JIC decided that this was probably the case. Investigators from the International Atomic Energy Agency then pointed out that the tubes could not be used in centrifuges without modification, and that they were in fact being used to make artillery rockets. The CIA dismissed them as naive.

In the face of such conviction, any evasiveness from Iraq--one of the world's most corrupt and incompetent regimes--was taken as an indication of guilt. In particular, where Iraq tried to hide deliveries of dual-purpose material, western spooks always assumed the worst. In fact, Iraqis typically used front companies to evade UN sanctions, and maximise the potential for corruption, on even their most harmless imports.

In response to the report's criticisms, both Mr Bush and Mr Blair reiterated their opinions that the war remained right. They would hardly have said otherwise. But both had argued a case for war based on the threat of Iraq's WMD. Mr Blair persuaded the British parliament to vote for war on the basis of Iraq's weapons--which the prime minister termed a "serious and current" threat--and the connected argument that Saddam had consistently breached United Nations resolutions. Had MI6's intelligence been a little more accurate, he could not have claimed that Britain was under threat.

Mr Bush did not make quite such a specific argument for war. In the slipstream of the outrages on September 11th 2001, he also suggested that Saddam was too bad a man, and too full of hate for America, to be left in power. But, contrary to what some Bushites now claim, the weapons remained the cornerstone of his case.

This was why he ordered Mr Tenet hurriedly to prepare the NIE in time for Congress to read before voting on whether to authorise the war. Similarly, when the CIA failed to find a link between al-Qaeda and Saddam Hussein, the administration relied on the threat of WMD to keep fears of such a link alive. Saddam "could decide secretly to provide weapons of mass destruction to terrorists for use against us...it would take just one vial, one canister, one crate to bring a day of horror to our nation," said the vice-president, Dick Cheney, a few weeks before the invasion of Iraq began.

Bruised, not mauled

On the face of it, neither Mr Blair nor Mr Bush was mauled by the inquiries. Lord Butler found no evidence that the prime minister's staff "sexed up" the dossier, though he does show that the intelligence was hardened for public consumption as it went through the system. He blames no individual and does not find any sign that Mr Blair doubted the intelligence he was being fed by the JIC.

In signing off, Lord Butler recommended that greater care be taken to maintain the distinction between intelligence and political persuasion. He also pondered the thought that WMD could still be found in Iraq, a good-sized country, with "lots of sand".

For Mr Bush's supporters, the Senate report's best sentence was the one that appeared to absolve his administration of the charge that it coerced and co-opted the nation's spies. The Republican-led committee found no evidence that the "mischaracterisation or exaggeration of the intelligence on Iraq's weapons of mass destruction capabilities was the result of political pressure." But the president is not off the hook yet.

The report is bad news for Mr Bush because it has given Americans yet another reason to regret supporting him over Iraq. The committee's lead Republican and lead Democrat both said that they would not have voted for the war knowing what they know now. However Senator Pat Roberts, the committee's chairman, nuanced this, saying that he would have considered voting for a conflict more "like Bosnia and Kosovo", which did not involve American ground troops.

Small wonder that an Economist/YouGov poll released this week suggests that 61% of Americans believe they were given false information about WMD in Iraq; 70% blame the CIA for misleading them; and 59% blame the White House. Short of the under-the-sand miracle that Lord Butler mooted, Mr Bush's distrust rating seems destined to rise.

For the Senate committee will now embark on the "second phase" of its inquiry, to consider how the administration used the intelligence agencies and their information to make its case for war. It will not complete this task until after Americans have voted in November. But there is already a good deal to suggest that the administration employed strategies to mould the intelligence to its purpose.

Certain statements, by Mr Cheney in particular, tempt the thought that sometimes the administration could not even be bothered with this formality. And Donald Rumsfeld, the secretary of defence, had little use for qualifying evidence when he announced in September 2002 "We know they have WMD. We know they have active programmes. There isn't any debate about it."

The report contains comments by the CIA's ombudsman, a figure appointed to investigate allegations of political interference in the agency. After interviewing intelligence analysts involved in the drafting of a June 2002 agency paper, which examined possible links between Iraq and al-Qaeda, the ombudsman found that about half a dozen spooks complained of coming under intense pressure from officials. Mostly, this involved being asked the same questions again and again. After conducting a July 2003 investigation into the pre-war intelligence processes, a former deputy chief of the CIA concluded that such "repetitive tasking" was partly aimed at eliciting the desired response.

A more insidious example of the administration's whip-cracking ways was the intelligence-scanning cell established by Douglas Feith, an under-secretary of defence. In mid-2002, Mr Feith detailed two researchers to double-check the CIA's files for links between Iraq and al-Qaeda. Mr Feith has denied that his men had any influence over the gathering or analysis of intelligence. But, in an appendix to the report, John Rockefeller, the committee's vice-chairman, claims that the Pentagon team offered a hawkish alternative to the CIA's analysis, firmly linking al-Qaeda to Iraq. When Mr Tenet rejected the team's presentation, they took it, says Mr Rockefeller, directly to the White House.

Chasing after fragments

Perhaps most tellingly, Mr Rockefeller accuses the administration of putting pressure on its spies to conform to the certainty contained in its own strident pronouncements. The CIA was faced with the risk of failing to support its government, or of failing to anchor its conclusions in the evidence to hand. In the absence of any substantial intelligence linking Iraq and al-Qaeda, for example, the CIA's agents declined to parrot Mr Bush's characterisation of Saddam in October 2002 as a man "who would like to use al-Qaeda as a forward army." But the president would not have thanked them for their restraint.

By the time the Senate judges Mr Bush in the next phase, the verdict will be in from a bigger jury, America's voters. Mr Blair, too, is preparing to face voters next year. The Senate report concludes thus: "While analysts cannot dismiss a threat because at first glance it seems unreasonable or it cannot be corroborated by other credible reporting, policymakers have the ultimate responsibility for making decisions based on this same fragmentary, inconclusive reporting."

tomder55 answered on 07/28/04:

When Hans Blix reported to the U.N.about the various unaccounted for weapons that they had documented Iraq had was he also a victim of this 'group-think'.What do you think he meant when he said this to the U.N.:

"Unlike South Africa, which decided on its own to eliminate its nuclear weapons and welcomed the inspection as a means of creating confidence in its disarmament, Iraq appears not to have come to a genuine acceptance, not even today, of the disarmament which was demanded of it and which it needs to carry out to win the confidence of the world and to live in peace. "

"It is not enough to open doors. Inspection is not a game of catch as catch can. Rather, it is a process of verification for the purpose of building confidence."

This is what Blix said about chemical bombs :

"The document indicates that 13,000 chemical bombs were dropped by the Iraqi air force between 1983 and 1998, while Iraq has declared that 19,500 bombs were consumed during this period. Thus, there is a discrepancy of 6,500 bombs. The amount of chemical agent in these bombs would be in the order of about 1,000 tons. In the absence of evidence to the contrary, we must assume that these quantities are now unaccounted for."

I don't see any qualifiers in this statement.

This is what he reported about biological weapons :

"There are strong indications that Iraq produced more anthrax than it declared and that at least some of this was retained over the declared destruction date. It might still exist. "

This was the U.N. chief inspector . I guess he was wrong . So the U.S. and Britain ,and the U.N. ;oh lets not leave out Vladdy Putin who said that Russia believed that Iraq still possessed the weapons . The Germans believed that Iraq had WMD. The French believed that Iraq had WMD.Clinton believed that Iraq had WMD Everyone believed that Iraq had WMD!!!

So now the new spin is that everyone was wrong and was duped by some drunkard source in the Iraqi resistance. That Chalabi was some kind of genious manipulator that fooled the Intelligence agencies of the world ,and the U.S. into invading Iraq;for whom? ;well of course for his friends the Iranians. Makes perfect sense to me! Oh ,and we are also now to beleive that these mobile labs that we have found ;a fleet of them; were there for the purpose of making helium for Saddam's weather ballons .That is what is considered credible.





chekhovToo rated this answer Excellent or Above Average Answer
Chouxxx rated this answer Excellent or Above Average Answer
ETWolverine rated this answer Excellent or Above Average Answer
labman rated this answer Excellent or Above Average Answer

Question/Answer
Chouxxx asked on 07/27/04 - Polls, Schmolls

The Republicans should be very afraid; they have a lot of work to do before the November election. The only statistic that matters is this:

Clinton left office with a 60% + approval rating by the Americam public. George W. Bush is currently running under a 50% approval rating. There are lots of people who are disatisfied with Bush. It looks like he will be overcome by the desire of the American people to change our leadership.

Comments?

tomder55 answered on 07/27/04:

Polls, Schmolls .A Kerry supporter would likely say that today after getting no bump in the polls from yesterdays opening round .With network ratings of the convention hovering around cable talk show numbers .

Chris Mathews made an interesting point yesterday (actually 2 points ).1. The delegates are overwhelmingly opposed to the Iraq war. Why did the platform say 'honest people can disagree',and not make a difinitive anti-war statement?
2.The platform of the 'party of inclusion' only had a brief 1/5th of a page statement regarding minority issues .


This is ,and will be through the election ,too close to call.

my question remains ;how would Kerry conduct the war on Islamo-fascists differently .He says he has a plan . I'd like to hear it .

Chouxxx rated this answer Excellent or Above Average Answer

Question/Answer
Itsdb asked on 07/27/04 - Ah, Democrat values...

And so the DNC is all about strength, security and values...did someone forget to tell Teresa?

"Speaking Sunday night before the delegation from her home state of Pennsylvania Heinz Kerry criticized uncivil "and sometimes un-American traits" that she said were creeping into politics. Afterward, she was asked about the comment by the editorial page editor of a conservative Pittsburgh newspaper that opposes her senator husband.

Their exchange concluded with Heinz Kerry saying, "You said something I didn't say. Now shove it!"

"If someone is really attacking your honor, or trying really to be dishonest, really to try to get you, I think most Americans, most people, would say, you know, defend yourself. And that's what I did," she said today on CBS-TV's "The Early Show."

http://www.thepittsburghchannel.com/video/3579456/detail.html

As Hillary said, Good for you! You go girl!

So what are we to get from this, that it's an American value to attack others for asking what you meant by what you said, while denying you said it...even though it's on tape?

tomder55 answered on 07/27/04:

"I always say what I think.I don't go out and say it willy-nilly for its own sake, but if called upon I do."
T Heinz on "The Morning Show" today .

Lets hope she continues to do so tonight when she addresses the convention . I'm still looking for that next Howard Dean moment. Of course when she was married to a Republican she said "the Democratic machine in this country is putrid." I wonder what she said about Kerry back then?

darkstar rated this answer Excellent or Above Average Answer
ETWolverine rated this answer Excellent or Above Average Answer
Itsdb rated this answer Excellent or Above Average Answer
powderpuff rated this answer Excellent or Above Average Answer

Question/Answer
excon asked on 07/25/04 - The War President Buzz! Beep! Wrong! Failure!!!

Hello Experts:

Osama lives! Al Qaeda IS operational! Iraq is a bust and were bogged down. We are more vulnerable at home, not less.

I know youre gonna spin this a jillion ways, but heres the undeniable facts. Our real enemy is, was and remains, Osama Bin Laden. We defeated a paper tiger and got stuck while our real enemy skipped town. Our failure to get him, and our foolishness in Iraq has spawned a terror network of wanna bes all around the world.

We are NOT safer at home. So far, weve LOST the War on Terror. I dont hate Bush, but, he is undeniably, a miserable failure. Would Kerry have done better? I dunno, but Bush has proven his inadequacy beyond your ability to spin it otherwise.

excon

tomder55 answered on 07/27/04:

John Kerry's slogan to 'restore respect for America abroad' and to 'make it safer' are accusations that we have created the dangers we face now.Even though the weaknesses of the multilateral approach has been detailed and exposed in the last few years ,the Democrats under Kerry seem to be saying diplomacy and concession, is the best way to settle accounts with Islamo-fascists .Does he really think that we could ever have talked Chirac or Putin to go along with the policy ?

Both candidates have to understand that there are only a few things that can come from the war against the Islamo-fascists. Democratization eventually takes root ,but before that we can settle on reforms in Islamic societies that moves them away from their self destructive ,victimized pathos .The Islamo-fascists could successfully launch a WMD attack on the U.S. and unleash an all out war between the U.S. and the Islamic world .The Islamo-fascists could win ;radicalize the whole Middle East ,push the U.S. out of the region and move the world steadily toward dhimmitude .

The process of liberalizing Arab societies will take many years,but by our actions in Afghanistan and Iraq we have planted the first seeds . Hopefully we succeed .I do not like the alternatives .

excon rated this answer Excellent or Above Average Answer

Question/Answer
excon asked on 07/20/04 - This Land



Hello Experts,

http://atomfilms.shockwave.com/af/content/this_land

excon

tomder55 answered on 07/26/04:

finally got to see it . What a blast ! Wonder if Kerry is still planning on using the song at the convetion ? I know he has been playing it on the campaign trail. But if he did ,would the Republicans connect him to Woodie Guthrie that famous American socialist? (probably not because old man Bush used it also )

Martin Van Buren had a campaign song that he used to make fun of William Henry Harrison (Tippecanoe and Tyler too). It went to the tune of 'Rock-a by baby) :

"Rock-a-bye Baby, when you awake

You will discover Tip is a fake

Far from the battle, war cry and drum

He sits in his cabin a'drinking bad rum."

Just shows that bashing during a campaign is as American as Apple pie .

excon rated this answer Excellent or Above Average Answer

Question/Answer
Itsdb asked on 07/21/04 - Could it be?

Nuclear arms reportedly found in Iraq

BAGHDAD, Iraq, July 21 (UPI) -- Iraqi security reportedly discovered three missiles carrying nuclear heads concealed in a concrete trench northwest of Baghdad, official sources said Wednesday.

The official daily al-Sabah quoted the sources as saying the missiles were discovered in trenches near the city of Tikrit, the hometown of ousted Iraqi President Saddam Hussein.

"The three missiles were discovered by chance when the Iraqi security forces captured former Baath party official Khoder al-Douri who revealed during interrogation the location of the missiles saying they carried nuclear heads," the sources said.

They pointed out that the missiles were actually discovered in the trenches lying under six meters of concrete and designed in a way to unable sophisticated sensors from discovering nuclear radiation.

The sources said al-Douri, who is related to former Vice Chairman of the Iraq Revolution Council and Saddam's right-hand man Izzat Ibrahim al-Douri, was captured after Iraqi police intercepted an e-mail message in which he set a meeting with another former Baath official.

The report could not be authenticated by the interior ministry or the national security department, but the paper noted Iraqi Foreign Minister Hoshiar Zibari made a surprise request recently to Mohammed el-Baradei, head of the International Atomic Energy Agency, to resume inspections for weapons of mass destruction in Iraq.

http://washingtontimes.com/upi-breaking/20040721-081010-1808r.htm

tomder55 answered on 07/22/04:

It brings to mind one of my biggest concerns there. i have always thought that the insurgents want the U.S. forces out of the population centers and redeployed concentrated out in the open. This is how we deployed in Saudia Arabia . This is also how we were deployed in Beirut in 1983 when a car bomb was able to take out 241 Marines .Once that happens then if they had WMD of any kind ,that would be the place to use them .

I am sure that ultimately once the handover is completed we will reposition our troops .Something to keep an eye on .

All the information I have looked at seems to indicate that Saddam was part of a network to create an "Muslim Bomb". The net work included money from the oil sheiks and Iran ;Pakistan (Abdul Qadeer Khan provided the plans to Libya ,Iran and N.Korea)led the effort .Saddam may have kept a nuclear program going on his own (the trying to obtain yellow cake from Niger is confirmed )but it was too risky to continue the program in Iraq .It has been alleged that his nuclear experts were sent to Libya where the Muslim Bomb program was being conducted. It is noteworthy that both Iran and Libya decided to reveal their activities in the aftermath of the invasion of Iraq. The overthrow of Saddam's regime has had a profound impact on the thinking of governments in the Middle East.

It would not suprise me if nukes were discovered there, but I kinda doubt it .

Itsdb rated this answer Excellent or Above Average Answer

Question/Answer
Bradd asked on 07/20/04 - The Final Arbiter

9 black-robed justices or the democratic will of the 280 million? Tom posed this question (paraphrased) and I think it's an excellent one.

Currently, the final say lies with the Supreme Court. There is no appeal from that Court. Even legislation restricting the power of the Court would ultimately be ruled on by that Court.

Who do you trust (more)?

9 very smart people or the masses who we know from history can easily be swayed? The justices of course can be ideologically driven, but they have been known to drop or reverse ideology (Earl Warren).

Neither method is perfect, but I prefer the nine to the millions.

tomder55 answered on 07/21/04:

Alexis de Tocqueville noted the unique position of the Supreme Court in the history of nations . The representative system of government has been adopted in several states of Europe, he re-marked, but I am unaware that any nation of the globe has hitherto organized a judicial power in the same manner as the Americans. . . . A more imposing judicial power was never constituted by any people.

This quote described a feared tendency rather than a reality at the time he wrote it but his fear were evidently justified .

Marbury v Madison supposedly established the Supreme Court as the Constitutions final arbiter ;just like the pretender to the throne establishes himself by declaring, "Im king!" As long as the other branches of government stand around and shuffle their feet and do not exercise their defined role and concede the supremacy of the courts ;the only un-elected branch of the government; okay then,they are supreme.

The Supreme Court declared itself the final arbiter; not the Constitution. The genius of the "rule of law" in our system is founded on the checks and balances established by the founding fathers in our tripartite system of government. (btw. Marbury established only that the judiciary would play an important role in constitutional interpretation, not that it would play the ultimate role.)

This was not always the case . Before the Civil War all three branches of the government interpreted the Constitution. The Congressional record of the time is full of debates over the meaning of important constitutional provisions. Presidential vetoes of congressional acts were almost solely on constitutional grounds.

But since 1958 Cooper v. Aaron when the court declared themselves "supreme in the exposition of the law of the Constitution,"they have been equating their interpretations with the Constitution itself.

The courts are now managing school districts, levying taxes, operating prison systems, managing the federal lands (and the state lands in many western states), deciding who lives or dies in abortion clinics and under state death penalties, establishing standards of mental competence, drawing election districts, and making the final decision of general elections .

But the system is designed for the people ;yes all 280 million of them to be the final arbiter .The Supreme Court confirmed in the Dred Scott decision the legitimacy of slavery .The people over-ruled the court with the 13th amendment. We are assaulted by the sensibilities of a minority view of moral positions that are being imposed on us. It has degenerated into a partisan justifier of current social thinking The court asserts that toleration for offensive and evil conduct is essential to freedom, that pornography is protected speech but prayer is not.They can change the language or definitions of words . They are the enablers of the Mordreds among us ;trying to undermine the society by using its legal system .

Look at court decisions throughout our history . For every decision that is praised there are at least one that is disgraceful . It is not a partisan gripe as these poor decisions have transcended political ideology and both sides today complain about them. Besides the Dred Scott decision there was Plessy v Ferguson that affirmed Jim Crow laws . There were decisions in the early industrial era that trampled workers rights . I would prefer that if we are to surrender our democratic republican system to a tyranny of a judiciary that they not so often be wrong in their decisions .

Bradd rated this answer Excellent or Above Average Answer

Question/Answer
ETWolverine asked on 07/20/04 - Medal of Honor Recipients Support President Bush --- I received this today from a friend

[Open letter to veterans signed by twenty-one Medal of Honor recipients]

Dear fellow veterans:

We have listened to Senator Kerry falsely attack President Bush for months over funding for veterans, and it's time to set the record straight (www.GeorgeWBush.com/Veterans/). The truth is President Bush has led the way on improving veterans' benefits, supporting our troops and restoring honor and dignity to the White House.

Since 2001, President Bush has increased veterans funding by over $20 billion, and funding for veterans' health care has increased by 40 percent since he took office. Funding for veterans has gone up twice as fast under President Bush as it did under President Clinton, and those who accuse the President of cutting funding are simply not being honest with veterans.

Meanwhile, John Kerry voted against a $1.3 billion increase in veterans health care, skipped votes on concurrent receipt and voted against funding for our troops in Afghanistan and Iraq. And while he talks a good game, Kerry's record shows he is out of the mainstream. The American Flag symbolizes our ideals, our history and our values. President Bush shares this belief and supports a Constitutional Amendment banning desecration of the Flag. John Kerry believes this is "an attack on free speech."

We are disturbed that John Kerry would try to scare veterans with his false accusations, and we are disappointed in his lack of support for today's troops. Please join us in setting the record straight (www.GeorgeWBush.com/Veterans/) and showing your support for President Bush -- a leader who has proven his support for those who have served, backs our troops defending our nation and shares our values.

Sincerely,

John Baker, Jr. Medal of Honor Recipient

General Pat Brady, USA (Ret.) Medal of Honor Recipient

Robert Bush Medal of Honor Recipient

David C. Dolby Medal of Honor Recipient

Walter Ehlers Medal of Honor Recipient

James Fleming Medal of Honor Recipient

Rodolfo "Rudy" Hernandez Medal of Honor Recipient

Thomas J. Hudner Medal of Honor Recipient

Robert R. Ingram Medal of Honor Recipient

Arthur Jackson Medal of Honor Recipient

George Charles Lang Medal of Honor Recipient

Major General James Livingston, USMC (Ret.) Medal of Honor Recipient

David McNerney Medal of Honor Recipient

Ola "Lee" Mize Medal of Honor Recipient

Robert Modrzejewski Medal of Honor Recipient

Robert O'Malley Medal of Honor Recipient

Richard A. Pittman Medal of Honor Recipient

James Taylor Medal of Honor Recipient

Michael Thornton Medal of Honor Recipient

Gary Wetzel Medal of Honor Recipient

Paul Wiedorfer Medal of Honor Recipient


*The signatures on this letter of Medal of Honor recipients does not denote the support or endorsement of the Congressional Medal of Honor Society

-----------

Your comments, please...

Elliot

tomder55 answered on 07/20/04:

I don't know . Rhetorical support for veterans has been a staple of politics throughout the country's history, but veterans have long struggled to get promised benefits. One thing Bush could do as a show of faith is to reverse a 1995 Clinton era decision to rescind a promise of free lifetime health care benefits for soldiers, who from 1941 to 1956 had been told that if they signed up and served 20 years they and their dependents would get free care. The government stopped honoring that pledge and many veterans 65 and older have been forced to pay for benefits through Medicare, which now costs about $60 a month and pays for 80 percent of medical care after a $100 deductible has been paid. Now, that free care has been ended. Veterans can still be cared for at VA hospitals , but the wait for non-emergency treatment is long.

The case is up for review soon with the Supreme Court and it could become an issue this campaign. It is the type of issue that Kerry could take from Bush if he is not careful .

ETWolverine rated this answer Excellent or Above Average Answer
Yiddishkeit rated this answer Excellent or Above Average Answer

Question/Answer
kindj asked on 07/20/04 - Gee, wonder why THIS didn't make the nightly news...

Vegas Casino Boots Linda Ronstadt
Jul 19, 2:17 PM EST

The Associated Press

LAS VEGAS -- Singer Linda Ronstadt not only got booed, she got the boot after lauding filmmaker Michael Moore and his new movie, "Fahrenheit 9/11," during a performance at the Aladdin hotel-casino.

Before singing "Desperado" for an encore Saturday night, the 58-year-old rocker called Moore a "great American patriot" and "someone who is spreading the truth." She also encouraged everybody to see the documentary about President Bush.

Ronstadt's comments drew loud boos and some of the 4,500 people in attendance stormed out of the theater.

People also tore down concert posters and tossed cocktails into the air.

"It was a very ugly scene," Aladdin President Bill Timmins told The Associated Press. "She praised him and all of a sudden all bedlam broke loose."

Timmins, who is British and was watching the show, decided Ronstadt had to go for good. Timmins said he didn't allow Ronstadt back in her luxury suite and she was escorted off the property.

Ronstadt's antics "spoiled a wonderful evening for our guests and we had to do something about it," Timmins said.

Timmins said it was the first time he sent a performer packing.

"As long as I'm here, she's not going to play," Timmins said.

Ronstadt had been booked to play the Aladdin for only one show.

Calls to Ronstadt's manager were not immediately returned.

In an interview with the Las Vegas Review-Journal before the show, Ronstadt said "I keep hoping that if I'm annoying enough to them, they won't hire me back."

Looks like she got her wish.

tomder55 answered on 07/20/04:

I don't know ,she probably meant it to honor Moore but a closer examination of the song shows that it could be a lesson Moore could use :

"Desperado, why don't you come to your senses?
You been out ridin' fences for so long now
Oh, you're a hard one
I know that you got your reasons
These things that are pleasin' you
Can hurt you somehow

Don't you draw the queen of diamonds, boy
She'll beat you if she's able
You know the queen of hearts is always your best bet

Now it seems to me, some fine things
Have been laid upon your table
But you only want the ones that you can't get

Desperado, oh, you ain't gettin' no younger
Your pain and your hunger, they're drivin' you home
And freedom, oh freedom well, that's just some people talkin'
Your prison is walking through this world all alone

Don't your feet get cold in the winter time?
The sky won't snow and the sun won't shine
It's hard to tell the night time from the day
You're loosin' all your highs and lows
Ain't it funny how the feeling goes away?

Desperado, why don't you come to your senses?
Come down from your fences, open the gate
It may be rainin', but there's a rainbow above you
You better let somebody love you, before it's too late "
(Written by Don Henley & Glenn Frye Originally performed in 1973 by
The Eagles Remade in 1977 by Johnny Rodriguez Remade again in 1983 by Linda Ronstadt)

I have always enjoyed Linda's music .One of my favorite shows was seeing her perform in 'The Pirates of Penzance' with Kevin Klein in Central Park I expect that if I were at the show I would've been annoyed ,but more so at missing a fine concert. I just wish all these entertainers would heed Laura Ingram's advice;Shut up and Sing.

ETWolverine rated this answer Excellent or Above Average Answer
kindj rated this answer Excellent or Above Average Answer

Question/Answer
drgade asked on 07/20/04 - A Democrat's Watergate?

Is one of John Kerry's advisors going to turn out to be the start of a watergatesque investigation?

Sandy Berger was seen stealing materials from the National Archives. This is illegal. He is now under investigation about the material of former President Clinton's information on Al Quaida:
"some drafts of a sensitive after-action report on the Clinton administration's handling of al-Qaida terror threats during the December 1999 millennium celebration are still missing, officials and lawyers told The Associated Press."

He gave some of them back, but "inadvertently" threw others away.

Want to bet that this doesn't get as much run in the media as Watergate?

tomder55 answered on 07/20/04:

Berger said he knowingly removed handwritten notes he had made while reading the documents at the archives by sticking them in his jacket and pants. He also 'inadvertently' took copies of actual classified documents in a leather portfolio . Maybe we can call this one 'trouser-gate'.

This did not get much media play yesterday ,but maybe today it will be a bigger deal. What would happen if Bush's National Security Adviser(Rice) was the target of an investigation relating to stealing documents related to terrorism? That is the benchmark of how the press should cover this . Then again ;they haven't made a big deal about Joe Wilson lying to discredit the Bush administrations claim that Saddam was seeking uranium yellow cake in Africa.But maybe this is too big to ignore .

drgade rated this answer Excellent or Above Average Answer
excon rated this answer Excellent or Above Average Answer

Question/Answer
Chouxxx asked on 07/19/04 - Kenny Lay and the Presidential Election

Bush has reason to sweat because his friend Kenny-boy may talk.

The Justice Department is now prosecuting a man who knows where a lot of the bodies are buried. Lay could probably explain, for example, why Enron gave Christian Coalition head Ralph Reed a $300,000 job at the White House's urging, or why a fleet of Enron corporate jets was put at the disposal of the Bush campaign during the frantic 2000 Florida recount. As a Cheney advisor, he recommended against capping electricity proces shortly before Enron began manipulating the market and fleecing California consumers our of billions.

OR, he could tell us what hapened at those secret meetings of Dick Cheney's energy task force. After all, he doesn't have to worry about being invited back the the White House anymore.

Comments?



tomder55 answered on 07/19/04:

President Bush may know indicted ex-Enron boss Ken Lay as ``Kenny Boy,'' but John Kerry and his wife also have past financial and personal ties to Lay .Kerry and Teresa reported more than $250,000 in Enron stock ownership before the firm's 2003 collapse.The majority of Enron and Ken Lay's donations went to the Democrats and not to the Republicans as many would have you believe. Besides Ken Lay and Enron donating millions to various Democratic causes he also contributed tons of soft money during the 1990's to various PAC's and foundations.

Kerry has repeatedly tried to distance himself from Ken Lay.Enron and Ken Lay donated a half a million dollars to Republican candidates over the past 10 years and only $5000 went directly to Bush. However more than $12 million dollars went to Democratic candidates during the same time period. Untold thousands were indirectly channeled to Kerry via back door "Soft Money" contributions. This may be under investigation by the DOJ in connection with Lay's other problems .

It is interesting to note that Bill Clinton acted on Enrons behalf after receiving a hefty political contribution. Enron also contributed to Bill Clintons presidential campaigns( Clinton helped Ken Lay get a $3 billion power plant project in India for Enron. Four days before the deal went through, Enron gave $100,000 to the Democratic party).But contrary to the myth ;Lay did not sleep in the Lincoln bedroom .But the corruption related to Enron and the Clinton Adm . was so well documented that it would be too long to post it all in this reply .

The Bush White House did not respond to Enron pleas to bail out the company. So perhaps you are right .Maybe Ken Lay has an ax to grind with Bush . But the Democrats have picked the wrong battle on this one . The best that can be said is that the whole body politic has blood on their hands .

Chouxxx rated this answer Excellent or Above Average Answer
ETWolverine rated this answer Excellent or Above Average Answer
Itsdb rated this answer Excellent or Above Average Answer

Question/Answer
Yiddishkeit asked on 07/18/04 - "WORLD LARGELY HAS IGNORED SUDANESE MASSACRES"

That was the headlines by James Traub, an opinion columnist for The New York Times Magazine. What should or can be done about this atrocity that has been compared to the genocide in Rwanda?



Bobby

tomder55 answered on 07/19/04:

If the American people thought that they could, they would stop the slaughter of civilians in Darfur. If the situation in Darfur were a disaster created by nature, aid would already have arrived. Americans are generous when acts of nature cause human suffering. We help first and ask questions later.

Darfur is not a disaster created by nature. It was created by politics.Even if immediate aid fixes the situation today, the political circumstances that created the problem will still be there tomorrow. Somehow, the international system has fallen to a point where threatening to remove a savage government is considered (unthinkable;or tack on any of the other slanders thrown at Bush and Blair) while allowing hundreds of thousands to starve to death massacred is considered business-as-usual. The measures that get public support; UN resolutions and "diplomatic pressure" cannot be effective unless backed by a credible threat of force. The result is inaction . Dictators expand their power by violent means knowing that the outside world is fearful of intervening in situations where violence is too extreme or the commitment would be too long or costly .

Yiddishkeit rated this answer Excellent or Above Average Answer

Question/Answer
Chouxxx asked on 07/17/04 - Hilary Rodham Clinton to speak!

As the dust settles, Hilary Rodham Clinton will speak at the Democratic Convention beginning July 26th. She will introduce Bill Clinton. This will be a minor role as Kerry wishes.

And, after all, if BUSH wins, Senator Clinton will run for the office of the President of the United Stated in 2008 and she will win.

tomder55 answered on 07/18/04:

I guarantee that she will turn this 'minor role' into the address of the ages .Hope she blows a fuse and puts out one of her classic rants . Like when she sqawked "I'm sick and tired of people who say that if you debate and disagree with this administration, somehow you're not patriotic. We need to stand up and say we're Americans, and we have the right to debate and disagree with any administration." Of course that charge is a complete invention by the Democrats .The Administration has never questioned that ,only flame throwers like Rush .

Eventually she will run for Prez. of that I'm certain .Kerry's cave has assured it .There was a chance that he could've wrestled the Democrat Party away from the Clinton junta had he stuck to his guns.It is telling that going into the convention he still has to contend with the likes of Terry McAuliffe.Back in March I was sure that one of Kerry's first prioities would be to oust him ,but he is still calling the shots in the DNC.If Kerry wants to prove he's the top Democrat now he needs to get rid of McAuliffe.

Hillary may or may not win ,but I see no scenario in the next few elections that will change the basic 50-50 split that the country is currently in .

Ccl471 rated this answer Excellent or Above Average Answer
Chouxxx rated this answer Excellent or Above Average Answer

Question/Answer
Itsdb asked on 07/16/04 - A new trend?

Passengers' good will turned soldiers' trip home into a flight of fancy

10:42 PM CDT on Wednesday, July 14, 2004

By MICHAEL E. YOUNG / The Dallas Morning News

It all began with a chance encounter at an airport, a glance, an offer, a quiet chat.

What's your seat number, soldier?

It's 23-B, sir, the soldier told the businessman.

No, son, that's my seat. Yours is in first class.

As more soldiers boarded, similar offers quickly came from the other first-class passengers.

And eight soldiers heading home from Iraq for two weeks of R&R found themselves with their officers in the big seats up front instead of the center seats in coach.

U.S soldiers fresh from Iraq were surprised but grateful for the first-class seats on Flight 866 from Atlanta to Chicago. That spontaneous act of good will transformed American Airlines Flight 866, from Atlanta to Chicago, on June 29.

"The soldiers were very, very happy, and the whole aircraft had a different feeling," said Lorrie Gammon, one of the Dallas-based flight attendants working the trip.

"There were 14 seats in first class, and there were 12 soldiers there. The other two first-class passengers wanted to give up their seats, too, but they couldn't find any more soldiers."

Flight attendant Candi Spradlin of Conway, Ark., said she was impressed with how passengers treated the soldiers.

"If nothing else, those soldiers got a great homecoming," she said.

The soldiers were so surprised they barely knew what to do, said Ms. Gammon, who lives in Frisco.

"They were so humble and thankful they spent the whole flight saying thank you," she said.

"But we should have been saying thank you to them for what they're doing for us."

http://www.dallasnews.com/sharedcontent/dws/news/dmn/stories/071504dnmetfirstclass.20618.html

Amen.

tomder55 answered on 07/17/04:

glad the hear it I was upset earlier this week when I read this story from The Seatle Post about a Iraqi vet who was booed in the nastiest way at a 4th of July Parade on Bainbridge Island .

As the report says :To understand why the reaction of strangers hurt so much, you must read what the young man had written in a letter from Iraq before he was disabled in an ambush:

"I really miss being in the states. Some of the American public have no idea how much freedom costs and who the people are that pay that awful price. I think sometimes people just see us as nameless and faceless and not really as humans. ... A good portion of us are actually scared that when we come home, for those of us who make it back, that there will be protesters waiting for us and that is scary."


His worse nightmare came true.

ETWolverine rated this answer Excellent or Above Average Answer
Itsdb rated this answer Excellent or Above Average Answer

Question/Answer
excon asked on 07/15/04 - Constitutional Rights - Who has 'em?


Hello sperts:

Please tell me what rights the majority has and where can I find them listed?

excon

tomder55 answered on 07/16/04:

Need I remind you that the Constitution starts :WE the People .The most important decisions of our time and our country are not being made by the people or their elected representatives. The Courts have restructured our political community without the consent of the people.
Perhaps it is an unforseen consequence that the founders overlooked ,or perhaps it is an out right usurpation but the net effect is the same; a few dissenters can, and often do, use the courts to thwart the legislative intentions of those elected by majority vote.This tyranny by a few, using the courts to contravene law, is a continuing threat to our freedoms. The Constitution set up a system of government in which laws are made by representatives who are accountable to the voters, not by judges who are protected from the voters by lifetime appointments.


Time for me to quote some dead guy again.

George Washington warned us about this in his Farewell Address to the nation.

"If in the opinion of the people the distribution or modification of the constitutional powers be in any particular wrong, let it be corrected by an amendment in the way which the Constitution designates. But let there be no change by usurpation; for though this in one instance may be the instrument of good, it is the customary weapon by which free governments are destroyed. "

ETWolverine rated this answer Excellent or Above Average Answer
excon rated this answer Excellent or Above Average Answer

Question/Answer
Itsdb asked on 07/14/04 - Don't you love it?

Algore:

"What a disgrace that their families have to hold bake sales to buy discarded Kevlar vests to stuff into the floorboards of the Humvees! Bake sales for body armor."

John Kerry:

"As president, I will see to it that we don't have to have bake sales and bargain-basement sell-offs, yard sales by parents and buy on the Internet to supply the troops of the United States of America."

USA Today:

"Kerry also gave Republicans another remark to chew on, telling the women's lunch he was "proud" that he and Edwards voted against $87 billion for troops and reconstruction in Iraq. "We knew the policy had to be changed. We needed to get it right," Kerry said."

"Bake sales for body armor" is a myth, and our troops in Iraq have all had the newest armor since January. But if Kerry had his way with the vote, wouldn't that have made bake sales MORE likely?

tomder55 answered on 07/15/04:

Some New Hampshire families held a bake sale in early May to raise money for more care package items and to fund a coming-home party next year. In Tennessee, a chapter of the Veterans of Foreign Wars held a dance and a bake sale to support a Marine who had returned to their community from Afghanistan missing a leg. And police officers around the country have collected discarded bulletproof vests and shipped them to Iraq for the troops to use on the doors and floors of their Humvees, making them lightly armored vehicles.

The military has been gradually replacing the older Kevlar vests with Interceptor vests ("Interceptor" body armor, is about ten pounds lighter and offers protection against machine guns fired at point-blank range ). Initially about 40,000 troops lacked the new Interceptor armor, although every soldier on the ground possessed the older armor.The Manufacturers have been working overtime to make up the short-fall .If the families held bake sales they would still not be able to purchase the new vests .By mid-January 2004, all troops in Iraq possessed the new vests. By mid-summer, sufficient numbers of armored Humvees will have arrived.

But ;lets not let the facts get in the way of a good political slogan.

ETWolverine rated this answer Excellent or Above Average Answer
Itsdb rated this answer Excellent or Above Average Answer

Question/Answer
ETWolverine asked on 07/14/04 - The REAL righteous Dis - with apologies to ITSDB

Have you all heard the latest from the Democratic National Convention? The list of speakers for the DNC has been released.

And Hillary Clinton's name isn't on it.

There are two schools of thought floating around as to why it happened.

1) Kerry and Edwards are afraid of being upstaged by Hillary.

2) Hillary didn't ask to be on the program.

But no matter how you look at it, its a major dis to Hillary. If Kerry was behind it, it is a dis from Kerry to Hillary... just one more in a long line of disses between these two power families in the Democratic party.

If she simply wasn't included because she didn't ask, that too is a major dis. Hillary is seen by many as the Democratic Party's hope for 2008... the heir apparent. If that is the case, and given her position as a Senator from NY (in the first post 9-11 national convention), the former First Lady, and the party's hope for 2008 (at least by some people's reckoning), then she should have been INVITED by the party. Not being invited to speak is a HUGE dis.

The DNC has said that the list of speakers is still subject to change, including additions to the roster. So I think in the end, Hillary will end up speaking at the convention. But it will have the feel of "sloppy seconds"... kind of like being invited in the second round of invitations to a wedding, rather than the first.

So what is up with that? Why did the party dis her like that?

tomder55 answered on 07/14/04:

sounds like the 2008 campaign is starting early . lets assume that Kerry loses . 2008 is likely to be a primary race between Edwards and Evita . The Kerry/Edwards camp is vying for power with the Clintons. It represents the final takeover of the party by the far left . Zell Miller and Leiberman should leave before the boat completely sinks .

Hillary not speaking doesn't suprise me as much as Sharpy Sharpton not getting a spot.
Why ? Because Bill Clinton will be a speaker of prominence sometime during the convention and will naturally upstage her . I think they should get Gore ,Teddy Kennedy,Dean ,and Hillary to do a joint primal scream. That would be a highlight .
...........................................
The lineup
Scheduled speakers for the Democratic National Convention:

-- July 26: Former Vice President Al Gore, former Presidents Bill Clinton and Jimmy Carter, Rep. Stephanie Tubbs Jones of Ohio, Rep. Tammy Baldwin of Wisconsin, Rep. Bob Menendez of New Jersey.

-- July 27: Sen. Ted Kennedy of Massachusetts, Christie Vilsack, wife of Iowa Gov. Tom Vilsack, Gov. Janet Napolitano of Arizona, Teresa Heinz Kerry, wife of Sen. John Kerry.

-- July 28: Gov. Bill Richardson of New Mexico, Mayor Martin O'Malley of Baltimore, retired Marine Lt. Col. Steve Brozak of New Jersey, presumptive vice presidential nominee Sen. John Edwards, who will be introduced by his wife, Elizabeth Edwards.

-- July 29: Presumptive presidential nominee Sen. John Kerry, who will be introduced by former Sen. Max Cleland of Georgia, Kerry's daughters, Alex and Vanessa Kerry, and Kerry's stepsons, Chris and Andre Heinz.

Source: Associated Press

Itsdb rated this answer Excellent or Above Average Answer
ETWolverine rated this answer Excellent or Above Average Answer
purplewings rated this answer Excellent or Above Average Answer

Question/Answer
Itsdb asked on 07/13/04 - The John-John love fest

Only in politics...taken from gop.com

Kerry on Edwards prior to becoming running mates:

*** When I Came Back From VietnamI Dont Know If John Edwards Was Out Of Diapers Then. In the Senate four years - and that is the full extent of public life - no international experience, no military experience, you can imagine what the advertising is going to be next year, Mr. Kerry said. With a grin, he added: When I came back from Vietnam in 1969 I dont know if John Edwards was out of diapers then. Well, Im sure he was out of diapers. (Adam Nagourney and Jim Rutenberg, With Hopes Up And Elbows Out, Democrats Give Iowa Their All, The New York Times, 1/19/04)

*** I think the American people want an experienced hand at the helm of state, said Kerry, who has spent 19 years in the Senate compared with Edwards five. This is not the time for on-the-job training in the White House on national security issues. (Mark Z. Barabak, Diverse States May Reshape The Democratic Race Today, Los Angeles Times, 2/3/04)

*** Kerry took aim at Edwardss lack of military and foreign policy experience while responding to Edwardss comment that both candidates shared similar plans to rebuild Iraq. Well, I think he would like it to be that way, said Kerry, a decorated veteran, but I think I have 35 years of experience in international security, foreign policy, and military affairs, and I think that makes an enormous difference here. I think that the world is looking for leadership that is tested and sure. (Raja Mishra, Edwards And Kerry Emphasize Contrasts, The Boston Globe, 2/23/04)

*** The veteran senator also questioned the former trial lawyers pursuit of the presidency after less than one term in elective office. And people call me ambitious? a Globe reporter once overheard Kerry asking an aide. (Glen Johnson, Once a Rival, Edwards Staying Close to Kerry, The Boston Globe, 6/10/04)

*** "If Mr. Kerry himself unswervingly took the high road, his spokeswoman, Stephanie Cutter, did not hesitate to rebut the criticisms by Dr. Dean and Mr. Edwards in the sharpest tones. Of Mr. Edwards, Ms. Cutter said: If his intent is to remove special interests from Washington, why has he, as a member of the Judiciary Committee, taken more than $11 million from lawyers and law firms? (Todd S. Purdum, With End Near In New Hampshire, A Day For Final Jabs And Last-Minute Pleas, The New York Times, 1/27/04)

*** Mr. Kerry made clear that if Mr. Edwards tries to tar him as a tool of special interests, he will respond in kind. It seems to me theres a huge amount of money thats been compiled in his campaign from one particular area of our economy, Mr. Kerry said, referring to Mr. Edwardss support from trial lawyers. People who live in glass houses shouldnt throw stones, Mr. Kerry told MSNBC. (Josh Gerstein, Kerry Advances, As Edwards Hangs On, The New York Sun, 2/4/04)

*** A moment later, after the interview was over, in an aside that his aides insisted had not been intended for public consumption, Mr. Kerry mocked Mr. Edwardss claim that he was the strongest Democrat the party could put up against Mr. Bush because he was from the South. Edwards says hes the only one who can win states in the South, Mr. Kerry said to a senior aide, David Wade, in a remark that was picked up by a reporters microphone. He cant win his own state. (Adam Nagourney, For Kerry And Edwards, Sharp Exchanges Reflect A Crucial Day Of Voting, The New York Times, 2/3/04)

*** An e-mail from a Kerry spokesperson reacting to Edwards trade/economic speech yesterday jibes, Below is John Edwards MAJOR economic address, where he promises to reward work and to create opportunity. Theres one thing missing from this speech: TRADE. Apparently, cracking down on unfair trade practices and promoting fair and balanced trade was not a priority to John Edwards just eight months ago. Mr. Johnny Come Lately on Trade! Another Kerry e-mail actually picked apart Edwards proposals. (NBCs First Read, 2/20/04)

*** KERRY: On trade, there is no difference between what John Edwards would do today and what I would do today. And to listen to John try to carve out this - what I think is sort of a protectionist point of view in the past, actually is not documented by the record. John Edwards has been in the Senate for five years. Hes talked more in the last five weeks about trade than he has in the entire five years. The fact is that he didnt vote in the 1994 election when he had a chance to vote about trade. He didnt talk about it, against it, in his election in 1998 when he ran for the Senate. And he went to The New York Times last week and said that he thought that NAFTA, in fact, was good for the prosperity of our country. (CBS/New York Times Democrat Presidential Candidates Debate, New York, NY, 2/29/04)

*** Edwards promised to run a positive campaign, one Kerry campaign statement read, noting that Mr. Edwards had voted for the China trade agreement. But now Edwards attacks John Kerry and runs from his own record. (Adam Nagourney and David M. Halbfinger, Kerry And Edwards Square Off As Dean Abandons Campaign, The New York Times, 2/19/04)

*** Of the Senator [Edwards] nipping at Kerrys heels, a senior Kerry advisor poses a counter query, The question is how many times you can come in second before you realize you didnt come in first? (ABCs The Note, 2/18/04)

*** [Kerry] also took a rare jab at Edwards for suggesting that the two men hold a debate in this state before its primary Sounds like a little grandstanding to me, Kerry said. (Patrick Healy, Even In Victory, Kerrys Caution Is Seen As Risky, The Boston Globe, 2/25/04)

*** Kerry Accused Edwards Of Looking For Differences That Didnt Exist. I know hes looking for some differences because you need them, Kerry chided Edwards at one point. (Tom Raum, Kerry, Edwards Find More Common Ground Than Differences In Debate, The Associated Press, 2/27/04)

Edwards on Kerry before becoming running mates:

*** [S]enator Kerry has consistently said that he can pay for all the things that hes proposing and substantially reduce the deficit, I think Ive heard him say cut it in half, in his first term. Well, The Washington Post just analyzed his proposals, and its the same old thing. Here we go again. In fact, in fact, he overspends, in terms of being able to pay for all of his proposals, he overspends by $165 billion in his first term, which means he would drive us deeper and deeper into deficit. My point is very simple about all this. This is the same old Washington talk that people have been listening to for decades. They want something different (Sen. John Edwards, CBS News/The New York Times Democratic Presidential Candidates Debate, New York, NY, 2/29/04)

*** There are multiple differences on economics. One, the difference in our personal stories. Two, Im the person whos focused much more on what has to be done for the middle class. Three, I have not heard him talk about poverty. I doubt if it would be a priority. (Jonathan Alter and Arian Campo-Flores, I Talk About Things People Care About, Newsweek, 3/1/04)

*** Edwards, on ABCs This Week, was asked about Kerrys explanations of his vote for the resolution authorizing President Bush to go to war in Iraq. Hes not been clear to me, Edwards said. I think hes said some different things at different points in time. So I think theres been some inconsistency. (Dan Balz and Paul Schwartzman, Reinforcements Rally For Frantic Final Push, The Washington Post, 1/26/04)

*** After Kerry delivered a lengthy response to a question about whether, having voted for the resolution authorizing war in Iraq, he felt responsibility for U.S. casualties, Edwards said, That is the longest answer I have ever heard to a yes-or-no question. (Dan Balz and David S. Broder, Edwards Challenges Kerry In Debate, The Washington Post, 2/16/04)

*** First of all, if I can get the truth-o-meter out here again for just a minute. John Kerry, you are not the only one who has a plan to bring down the cost of health care. I have a very clear plan about how to do that. (Sen. John Edwards, The Des Moines Register Democrat Presidential Candidates Debate, Des Moines, IA, 1/4/04)

*** Sen. John Edwards of North Carolina says that Kerry, after being in Washington so long, cant bring about real change. And in a dig at Kerrys background, Edwards says the concerns of poor Americans are not academic for me. Ive lived it. (Matt Stearns, Kerrys Big Link To Big Money, The Kansas City Star, 2/8/04)

*** Ive been looking forward to the time when this is a two-person race and people will focus on Senator Kerry and myself. It now appears that were very close to that place and maybe already there. And so this is the moment Ive been looking for. I believe if that happens, the choice that I give voters, which is different than Senator Kerry, is somebody who comes from them, who understands the problems that they have in their life, who understands what the loss of a job means, who has both trade policy and tax policy that will work for all Americans and not just a privileged few. And I think there are real differences between us. (CNNs Larry King Live, 2/17/04)

*** The question is whether that change can be brought by somebody who spent most of their life in politics and in Washington, said Edwards, a first-term North Carolina senator who made his fortune as a trial lawyer. (Ron Fournier, Democrats Spar In N.H. After Iowa Jolt, The Associated Press, 1/20/04)

*** The [Edwards] document, which aides confirmed was part of a 50-page packet they distributed earlier this month, instructs captains to describe Howard Dean as an elitist from Park Avenue in New York City. It says John Kerry cant claim to change America because he has been part of the failed Washington politics for too long. (CNNs The Morning Grind, 1/22/04)

*** In order to beat [President] Bush, we need a nominee who can win Southern states, not another New Englander or Washington insider who loses every Southern state, the [Edwards] memo reads, referring to both Dean and Kerry. (CNNs The Morning Grind, 1/22/04)

*** [Edwards] said: The question is, who on the top of the Democratic ticket can go every place in America and campaign with the candidates and strengthen their ability to get elected? He added, In Georgia, do you want John Edwards campaigning with you? Do you want Howard Dean campaigning with you? Do you want John Kerry campaigning with you? (David M. Halbfinger and Randal C. Archibold, Ignoring Other Candidates, Kerry Turns Focus On Bush, The New York Times, 1/22/04)

*** "Its one thing to talk about special interests, [Edwards] said. Its something else to do something about it. He emphasized he was not attacking Kerry, a Massachusetts senator. Its a difference between Senator Kerry and me. (David Espo, New Hampshire Democrats Cast First Votes, The Associated Press, 1/27/04

*** We want someone who hasnt been there for 15 to 20 years, if youre going to bring change, Mr. Edwards told reporters here, referring to Mr. Kerrys service in the Senate. He added: I dont take contributions from lobbyists. He obviously does. (Adam Nagourney, For Kerry And Edwards, Sharp Exchanges Reflect A Crucial Day Of Voting, The New York Times, 2/3/04)

*** I voted against the Chilean trade agreement. I voted against the Singapore trade agreement. I voted against final fast track authority for this president. I voted against the African trade agreement. I voted against the Caribbean trade agreement. Senator Kerry voted for all those. (ABCs This Week, 2/22/04)

*** Kerry and Edwards also sparred over international trade, with Edwards pointing out that he has voted against a series of trade agreements supported by his rival. There is a difference here, he said. There is a difference between Senator Kerry and myself. What hes saying now is different than what he did in the past. (Dan Balz, In Debate, Kerry Touts Experience And Edwards Stresses Electability, The Washington Post, 2/27/04)

And today?

Kerry: "This is the first time I've appeared anywhere without John Edwards in the last four days. I'm feeling this withdrawal"

Have you noticed they can't keep their hands off of each other now?




tomder55 answered on 07/14/04:

it has happened before . Reaganomics was coined by Bush Sr. during a nasty primary battle . Later Bush Sr. became a very loyal VP for 8 years

ETWolverine rated this answer Excellent or Above Average Answer
Itsdb rated this answer Excellent or Above Average Answer

Question/Answer
Itsdb asked on 07/13/04 - One Righteous Dis

July 13, 2004, 9:56 a.m.

One Righteous Dis

The president was right to turn down the NAACP.

Liberal bellies are aching these days over President George W. Bush's absence from this week's Philadelphia convention of the National Association for the Advancement of Colored People. Citing scheduling conflicts, the White House recently sent the president's regrets. As journalists have explained in grave and slightly damning tones, Bush is the first president since Warren Harding not to address the NAACP. The insinuation is that Bush's no-show before America's oldest and largest civil-rights group reflects his neglect of, if not disdain for, black Americans.

No one should be surprised, however, to see Bush toss the NAACP's invitation into the trash. That's exactly where the Baltimore-based organization has relegated him since 2000. NAACP chairman Julian Bond and president Kweisi Mfume have played tag team in bashing Bush and the GOP.

"So, we've got...a president that's prepared to take us back to the days of Jim Crow segregation and dominance," Mfume told Washington journalist Hazel Trice Edney just last week. Mfume either is lying through his teeth or is clinically delusional if he believes Bush hopes to reintroduce segregated water fountains and "colored only" waiting rooms. Mfume should try the truth, or see a psychiatrist.

Bond's rhetoric is equally reckless.

Bush and the GOP "preach racial equality but practice racial division," Bond said June 23 in Indianapolis. "Their idea of equal rights is the American flag and Confederate swastika flying side by side."

Bond told the NAACP's July 2003 Miami Beach conference: "Republicans appeal to the dark underside of American culture, to that minority of Americans who reject democracy and equality."

President Bush "has selected nominees from the Taliban wing of American politics," Bond informed the NAACP's New Orleans confab on July 8, 2001, as the September 11 hijackers learned to fly. "He has appeased the wretched appetites of the extreme right wing. And he has chosen Cabinet officials whose devotion to the Confederacy is nearly canine in its uncritical affection."

No wonder Bush found a better use of his valuable time than to associate with these racial bomb throwers.

Far from dissing black Americans, Bush has met with them throughout his presidency. He attended the National Urban League's 2001 and 2003 conventions. He hosted a White House celebration of the 1964 Civil Rights Act's 40th anniversary. Urban League president Marc Morial was there, as was civil-rights veteran Dorothy Hite. He has spoken to black churchgoers about his faith-based initiative.

Mfume also whined that "the president has refused to meet with the Congressional Black Caucus."

There he goes again.

Bush, in fact, invited the all-Democratic CBC to the Cabinet Room on January 31, 2001. "They had a warm meeting," White House Assistant Press Secretary Anne Womack told me then. "It was scheduled for 30 minutes and actually lasted nearly an hour."

President Bush even has addressed...the NAACP. The day after Bond's "Taliban" outburst, Bush offered its 2001 convention a video greeting. "I believe that even when disagreements arise," Bush said, "we should treat each other with civility and with respect."

Bush appeared personally before the NAACP as a 2000 presidential contender. In thanks, it telecast an infamous ad that fall which virtually implicated Bush in the 1998 truck-dragging murder of James Byrd in Jasper, Texas. Never mind that two of this black man's three white killers were sentenced to death on Governor Bush's watch.

Yes, Bush should campaign before black Americans, but he should not bother to plead with black leftists who hate his guts. Instead, he should meet with moderate to conservative blacks who are open to and even supportive of his policies. The Congress of Racial Equality, the National Center for Neighborhood Enterprise, and Project 21 as well as black business and religious groups would treat Bush respectfully.

Julian Bond, Kweisi Mfume, and their NAACP cronies should stop screaming like infants and learn this simple lesson: Don't expect grown adults to treat hand grenades like engraved invitations.

New York commentator Deroy Murdock is a columnist with the Scripps Howard News Service and a national-advisory-board member of Project 21, a Washington-based organization of black free-marketeers.

http://www.nationalreview.com/murdock/murdock200407130956.asp

Delaware County Democratic Party Chairman Clifford E. Wilson said, "Its one thing to not speak to a group of non-supporters in a local election, but theres a difference when youre the president...Its a no lose situation for him. If hes treated poorly it would be to his benefit because he is the president and it would seem disrespectful. But when speaking to a group like that you never leave the room with more opponents. You might actually end up with a few supporters."

Was Bush right to decline?

During this all out assault on Bush from the left, does anyone really believe like Wilson that if the President was treated poorly it would have benefited him?





tomder55 answered on 07/14/04:

If I were Bush I would not attend . I would inform them that when they get mature leadership then I would consider it .Instead I would have a different outreach plan for the 'African-American' community . I would emphasis that I am the only candidate that cares enough to give them a choice at which school their children attend. I would talk about law enforcement that has made their communities safer . I would talk about the faith based initiatives that are better at funnelling services to the needy than a big gvt. agency is .I would address their concerns about job opportunities ,and the education needed to obtain the jobs of the future. I would point out that no other administration has had as many minorities appointed in the key roles of the National Gvt.

But Bush is a terrible communicator so his message is often lost .

ETWolverine rated this answer Excellent or Above Average Answer
Itsdb rated this answer Excellent or Above Average Answer
purplewings rated this answer Excellent or Above Average Answer
Yiddishkeit rated this answer Excellent or Above Average Answer

Question/Answer
excon asked on 07/10/04 - Bush = Toast


Hello politicos,

They're neck and neck now, but how can a president who launched a bogus war that we're still stuck in the middle of, be re-elected?

I think it's a bigger mistake than say, breaking into the Watergate and then lying about it, driving drunk and killing Mary Jo Kopeckni, getting your schlong gobbled in the white house and then lying about it, screwing Marilyn Monroe and then lying about it, or being a nice guy from Plains with a drunken brother.

LBJ made a bigger one, but he recognized it.

excon

PS. Youre not truly going to argue that the war is a good thing even though we were mistaken, are you?
Or, are you going to argue that the investigative committees were wrong in their assessments?

The fearsome foursome (Elg, gade, Elliot and Tom) will probably take that route.

tomder55 answered on 07/11/04:

you predicted my response accurately .tHe very fact that they claim the whole world got it wrong is just amazing to me . Friends of Saddam;enemies of Saddam;the Russians,French,Germans ;Britain,the U.N.,the Israelis ;everyone for a decade got it wrong. Does that sound even remotely plausable ?

Well just this week it was revealed that the Poles found over a dozen Sarin filled warheads.The U.S. shipped over 2 tons of uranium found in Iraq .Terrorists tried to use a sarin containing shell as a IED .Another tried to attack the U.S. embassy in Jordan with WMD that originated in Iraq but were mysteriously obtained in Syria. A British inquiry found that indeed they were correct to suggest that Saddam was trying to obtain nuclear'yellow cake' from Niger.(Joe Wilson's 15 minutes of fame are over).

But the whole world got it wrong.

Hans Blix in his report to the U.N. documented tons of un-accounted for WMD. Now he says that WMD is not a threat at all;that Global warming is a bigger threat.

Isn't it more likely that Saddam Hussein used the long months between the time when the threat of invasion was debated at the United Nations and the time when it actually occurred to dismantle his weapons facilities and disperse them, perhaps to some neighboring country?There is already photographic evidence of a massive dismantling of a facility of some sort before last year's invasion. These photos were published on the front page of the New York Times. Now the Times says that Bush pressured the CIA to lie.

Nations do not wait for iron-clad proof when there are lethal threats. The massive Manhattan Project that produced the first atomic bomb was begun when the United States was at peace because of reports that Hitler's scientists were working on such a weapon.We had no proof but after Germany surrendered, it turned out that Hitler's atomic bomb project was nowhere near the stage that we feared. But we couldn't take that chance.


ETWolverine rated this answer Excellent or Above Average Answer
excon rated this answer Excellent or Above Average Answer
powderpuff rated this answer Excellent or Above Average Answer
Yiddishkeit rated this answer Excellent or Above Average Answer

Question/Answer
Chouxxx asked on 07/09/04 - BUSH AND CHENEY

Cough...I was thinking that Bush should get a new running mate and have Cheney as his official "side-kick" of some kind. It looks to me like he is going to lose by a landslide in November.

Cheney bears an uncanny resemblance to Dr. Strangelove at this point in his life.....this is really off-putting, to say the least!

How about Bush and Rice?

Or Bush and [name your choice]

Regards, Choux"xx" :D

How about Bush and

tomder55 answered on 07/10/04:

McCain scoffs at the idea. "I think the day that President Bush drops Vice President Cheney will be a cold day in Gila Bend, Arizona,I see no scenario in which the president would replace Dick Cheney." "One of Bush's strengths is that he sticks to his guns. He would appear both weak and political to dump Dick Cheney, and he's not going to do that," said Charles Black, a Republican adviser who is close to the White House.

If he was going to do that he should've announced it already .I would support a VP candidate other than Cheney because it would promote a viable Presidential candidate in 4 years .It was probably a good move ,but it is too late now unless Cheney decides to remove himself from consideration .What if Cheney comes to believe that he's become a burden on the ticket and withdraws perhaps citing health reasons?Few would question his motives under such circumstances. Rice would be the first serious Black candidate and being a woman and one brilliant person would be a natural ,and of course I would support her candidacy.If your watched her grilling on the 9-11 committee she can chew up and spit out anyone with out ever falling to the Democrat attack Lawyers tricks.

Not that I am concerned about Cheney being on the ticket .Cheney's negatives stack up well against Edwards .Most of the Haliburton thing is a non-issue in light of the Kerry family connection to Enron ,and the Clinton indiscriminate use of Halliburton during the Balkins conflicts.Perhaps both Administrations awarded no bid contracts to them because they are actually good at what they do. Had Cheney been such a factor ,then the announcing of Edwards as VP candidate would have given Kerry a bigger bounce in the polls than it did .

Alot of this calling for Cheney's removal is that they fear a debate between a seasoned politico with years of national security ;business ,and administration management ,and their empty suit hair piece.Apparently, so does John Kerry. When he was asked on TV about how he must be looking forward to a Cheney/Edwards debate, Kerry replied that everyone blows the debates out of proportion and they really aren't important.


Should Bush decide to toss Cheney now , the Dems will be the first to label him a flip flopper. Cheney is solid with the conservative base.Bush will be free to move to the middle and seize the territory that the Hairy~Kerry team has abandoned (the most liberal teaming since Mondale/Ferraro ,and possibly the most populist since William Jennings Bryant).Perhaps during the depression era a populist candidate could forge an alliance of the down-trodden ,and muster a majority ;but today? They try to bring the relatively confortable middle -class into the ranks of the downtrodden by (do I dare use a Gore-ism?)playing on their fear.It just doesn't resonate. A millionaire ambulance chaser telling us how to reform health care ? Cheney will have a field day with that one.

could go on ,but it is a sunny /low humidity perfect morning in N.Y.Time to pick some zucchinis. Glad you're back

Chouxxx rated this answer Excellent or Above Average Answer
excon rated this answer Excellent or Above Average Answer
Yiddishkeit rated this answer Excellent or Above Average Answer

Question/Answer
ETWolverine asked on 07/09/04 - World Court rules against Israel on barrier

(See my comments below.)

Calls 'de facto annexation' illegal, urges U.N. to take action

MSNBC News Services

Updated: 11:52 a.m. ET July 09, 2004

THE HAGUE, Netherlands - The International Court of Justice ruled Friday the barrier Israel is building in the West Bank violates international law, and urged the United Nations to take action to stop its construction.

The court dismissed Israel's arguments that the barrier was essential for its security and said the system of walls and fences went too far in infringing on the freedom of the Palestinians.

The decision by the world court was part of a lengthy and complex ruling read by court president Shi Jiuyong of China.

"The court accordingly finds that the construction of the wall, and its associated regime, are contrary to international laws," the court said.

The court also ordered Israel to pay reparations to Palestinians harmed by the barrier and return land seized to construct it.

"The court is of the view that the United Nations, and especially the General Assembly and the Security Council, should consider what further action is required to bring to an end the illegal situation resulting from the construction of the wall," the judgment said.

The judges were unexpectedly united in backing the decisions, by a vote of 14-1 for most paragraphs of the decision, with only the American judge dissenting.

'De facto annexation'
The court said the barrier was routed in a way that would encompass 80 percent of the Israeli settlers in the West Bank, while cutting off more than 230,000 Palestinians from their surrounding areas.

Despite Israel's protests that the barrier was temporary and not designed as a political boundary, the court said it could amount to "de facto annexation" by creating new facts on the ground.
It said the building of the barrier "severely impedes the exercise of the Palestinian people of its right to self-determination, and therefore is a breach of Israel's obligation to the respect of that right."

The 15-member court's advisory opinions are nonbinding, but bear moral and historic weight.
Court dismisses Israeli objections
The court dismissed Israel's objections that the U.N. General Assembly acted irregularly in asking the court for an advisory opinion.

It also rebuffed Israel's argument -- supported by the United States and several European countries -- the court should refrain from interfering because the issue was political, not legal, and could disrupt Mideast peace efforts.

"A legal question also has political aspects," said the ruling.

"The court accordingly has jurisdiction to give the advisory opinion" requested by the General Assembly, it said.

The court said it was aware of the political negotiations called the "road map," but said it was not clear its legal opinion would influence those efforts.

"The court has no compelling reason to use its discretionary power not to give that opinion," the ruling said.

The court dealt in passing with issues long at dispute between Israel and the Arab states.

It determined that the lands captured by Israel in the 1967 Middle East war are occupied territory, including East Jerusalem. Israel has refused to recognize Jerusalem as occupied since it was formally annexed by the Israeli parliament shortly after the war.

While the General Assembly and Security Council had never recognized Israel's claims, it was the first time Israel's status in the West Bank was the subject of an international legal judgment.

The court said Israel was obliged by all international treaties and conventions of international law, including the Geneva Conventions and common humanitarian law.

The Associated Press and Reuters contributed to this report.

-------------

Despite everything that I know and have learned in 35 years, I still find this kind of international condemnation of Israel surprising.

What I am about to say now is going to be VERY controvercial. I know it will, because I will be repeating the words of Rabbi Meir Kahane (zt'l), and he was assasinated for saying things like this. Nevertheless, I will say it anyway, and I will accept your criticisms, and rebutt them as I see fit.

When Israel shoots back at terrorists, it is condemned for violence. When it targets terrorist leaders, the international community condemns them for instigating and elevating the violence. When it tries to get rid of the violence by building a friggin' wall to sepparate themselves from their enemies in a NONVIOLENT MANNER, they are also condemned.

Why is it that Europe is so quick to condemn Israel for simply defending itself? Because Europe cannot understand or abide a Jew who defends himself. In their eyes, such a Jew should not exist.

If you ever wander through Europe, you will find that the towns that gave up Jews to Hitler and his Nazis are the towns with the most beautiful Holocaust memorials. Why is that?

If you look into the houses of Europeans, you will find in most cases a prominantly-displayed crucifix with Jesus or a picture of Jesus on the cross. Simply put, Europeans worship dead Jews. They celebrate them. They honor them. They see them as gods. Therefore, they create the most beautiful memorials you could possibly imagine to them.

But a LIVE Jew confounds Europeans. They just don't get us. Somehow, we Jews have managed to stay alive for thousands of years, with literally everyone trying to kill us. The Romans, the Greeks, the Babylonians... they're all gone, but we're still here. The Crusades, the Inquisitions, the progroms... we survived them too. The Holocaust SHOULD have finnished us off, but we outlived that as well... not just outlived it, but have grown, and have returned to our historical homeland.

All of this is a physical imposibility. It simply should not be possible for ANY people to survive, much less thrive, after such a history of persecution. There is only one possible explanation for our survival.

It's a MIRACLE FROM G-D.

But if we Jews are still receiving miracles like that, it must mean that we are still G-d's Chosen People. Which in turn calls into question all of Christiandoms religious beliefs... for instance the idea that we WERE the Chose People, but have lost that status to the Christians for whom Jesus supposedly allowed himself to be killed.

Our existance calls into question the fundamental religious and spiritual beliefs of most of Europe. And THAT is why a living Jew is an anthema to Europeans. A dead Jew confirms their beliefs, but a living Jew brings those fundamental beliefs into question.

That is why they refuse to allow us ANY method of self-defense... because we should not be allowed to defend ourselves.

I don't think that this works on a conscious level, by the way. I think that a lot of it takes place subconciously. But it is there nonetheless. Jews confound Europe so much that they would rather side with terrorists who want them dead too than side with the Jews who survive to call their beliefs into question.

I await your repsonses.

Elliot

tomder55 answered on 07/10/04:

the facts speak for themselves.terrorism in the areas where the wall is up is down close to 90%. All other methods of dealing with the Palestinians has been tried short of Meir Kahane's 'final solution'.Israel has a right and obigation to self defense.

There is no defending the European attitude about Jews.I am not sure that it's attitude can be blamed on it's Christian roots Anti-semitism has increased as Europe has become increasingly god-less. I do not defend the actions of the Catholics throughout history.The record speaks for itself.But the mass murders of the 20th Century were by the Hitler Nazis and the Stalinist of Russia;both god-less dogma.

America by contrast is a nation that still trys to retain some of it's Judeo-Christian ethic .It has been the greatest friend and defender of Israel.The cynical will say that it is because the Israeli lobby is so powerful,but I think it goes beyond that .There is a shared value system that transends politics.Both nations recognize God as the source of their legitimacy .William Penn, the founder of the state of Pennsylvania, said, "Those people who are not governed by God's Law will be ruled by tyrants."Without foundations like the Ten Commandments, liberty is left to the whims of the times , to the benevolence of the elite and the disgression of the powerful.

Chouxxx rated this answer Excellent or Above Average Answer
ETWolverine rated this answer Excellent or Above Average Answer
excon rated this answer Excellent or Above Average Answer

Question/Answer
Itsdb asked on 07/06/04 - Now THIS is an outrage

July 06, 2004, 9:03 a.m.
Can the U.N. Save Florida?

A group of congressmen go outside the U.S. for the presidential election.

America is doomed to suffer ever-escalating acts of political inanity until the November election. The assault on our collective intelligence by Hollywood and the media should end then, but some Dems are planning to continue it on into 2005. To do this, they are turning to the self-appointed international arbiter of honesty and democracy, the United Nations.

The liberals' bitterness in their defeat in 2000 hasn't abated one little bit. It wasn't cured by Howard Dean's primal scream therapy or by Michael Moore's movie. This bitterness has festered and metastasized into an anti-Bush obsession that will only end with a Kerry victory. (If Mr. Bush wins again, let us pray, we should all buy stock in Prozac-maker Eli Lilly on November 3. There will be nervous breakdowns all over Hollywood, New York, and DC.) Having failed to win the 2000 election first by working desperately to disenfranchise the many military members who vote by absentee ballot in Florida, and then in the Supreme Court the Dems have stooped to a new low. Seeking to group the United States among the third-world kakistocracies, a number of House Dems last week tried to set the stage for the U.N. to be the final arbiter in 2004.

Rep. Eddie Bernice Johnson (Tex.) with fellow Dems Joseph Crowley (N.Y.), Raul Grijalva (Ariz.), Danny Davis (Ill.), Corrine Brown (Fla.), Carolyn Maloney (N.Y.), Jerrold Nadler (N.Y.), Michael Honda (Calif.), Elijah Cummings (Md.), Julia Carson (Ind.) and Edolphus Towns (N.Y.) last week wrote to U.N. Secretary General Kofi Annan requesting U.N. monitoring and intervention in the presidential election. Their request cited the scurrilous report by the U.S. Commission on Civil Rights that determined contrary to the evidence that "...the disenfranchisement of Florida's voters fell most harshly on the shoulders of black voters and in poor counties...."

Set aside the fact that American elections are a model of democracy and fairness that almost none of the world's nations come close to equaling. And set aside the fact that this election is very unlikely to be close. Historically, America's close elections are few and far between. In 2004, the odds are miniscule that we will see a replay of the Florida Follies of 2000. But even if we did, we cannot tolerate any U.N. intervention, for to so do would be to surrender our independence and most basic freedom.

America in our Constitution and in every state has laws that protect the voters from interference with their right to vote. Any U.N. monitoring would violate those laws, and more. Neither the U.N. nor any other foreign organization has any legal or moral authority to enter American voting halls or to be present when votes are tabulated. If any U.N. representatives tried to do so, it would be the duty of state and local officials to show them the door peacefully or otherwise. Any discussion between U.N. "election monitors" and, say, members of the Texas Rangers or Virginia state police would be very brief and quite conclusive.


These congressional calumniators apparently believe the Dems would get a better break from the U.N. than from American voters or our own courts. In that, at least, they are right. The U.N.'s membership gaggle of 191 states includes fewer than 50 which can even arguably be said to be free. Some among those few such as the Russian Federation don't really allow voters self-determination. How would the U.N. membership the majority of which are dictatorships, despotisms and terrorist regimes judge an American election? They would certainly do as well as the U.N. has in administering the Iraq Oil-for-Food program.

If John Kerry were elected there would be little folderol and the matter would dissolve into some congratulatory messages to our new president. He would then be able to as he has promised go to the U.N. within two weeks of his inauguration to "rejoin" the international body. Heartiest congratulations would come from those, such as France, which are desperate to return to the Clinton era when the American power was used more in pursuit of the U.N.'s interests than in America's. But what if President Bush were reelected in a very close race?

The U.N. debate would be a riotous anti-American orgy. There would be U.N. commissions hearing testimony about election abuses from the U.N. observers and their supporters among the Democrats. Any U.S. Supreme Court ruling upholding the election would be ridiculed. Every dictatorship and despotism and every faux ally such as France would be breaking its pick to delegitimize the election and heap scorn on America. The EU would scold us for not living up to European values, North Korea and Sudan would lecture us on human rights, and few would bother to speak in our behalf. And, once again, we would be diverted from pursuing the war against terrorists and the nations that support them by those self same nations.

Britain, Israel, Australia, and Poland would support our right to self-determination (as might some of the other former Warsaw Pact nations whose freedom is newer, and more appreciated than that of Old Europe). Taiwan wouldn't, for the simple reason that its democratic government isn't allowed a seat in the U.N. There would be condemnation from all of the usual suspects, including the secretary general (to the extent he can take time away from covering up the Oil-for-Food scam). It might go so far that we would be subjected to the indignity of vetoing a Security Council resolution attempting to set aside the result, unless the Brits did us the honor of vetoing it before we had to.

It is hard to imagine a greater libel against American democracy than would inevitably come out of any U.N. debate. American freedom all 228 years of it is an affront to the members of the United Nations, and to the U.N. itself. Of that, among many other things, we should be very proud.

NRO contributor Jed Babbin is the author of Inside the Asylum: Why the UN and Old Europe are Worse than You Think.

http://www.nationalreview.com/babbin/babbin200407060903.asp


tomder55 answered on 07/06/04:

very nice . Here we are touting our system oversees ,and we have elected officials who came from the very system they are criticizing trying to undermine it and delegitimize it in the world opinion.

Sorry to burst their bubble .The U.N. is the only unelected group being considered here . I do not recognize their authority .The States ,the courts ,and the Congress (the body that these clowns are elected to )have the Constitutional authority and responsibility to monitor the elections .

Itsdb rated this answer Excellent or Above Average Answer

Question/Answer
excon asked on 07/06/04 - What the US is all about!


Hello Wingers:

I have quoted the following political philosophy in many of my questions. What surprises me, is that many of you disavow the sentiment in its entirety. Frankly, that blows me away. In the first instance because most of you dont understand it (you think it means anarchy), and in addition to not understanding it, you dont even realize that it forms the basis of OUR legal system in THIS country the country in which you live.

In a free society, my freedom ends where your nose begins. That means that I have the right to act in any manner I choose as long as my action does not interfere with anybody elses right to do the same.

I didnt write it. I didnt think it up. I learned it in high school civics. Was my teacher a commie? Or do I just not understand how this country works?

excon

tomder55 answered on 07/06/04:

That was Justice Oliver Wendell Holmes who's quote you borrow .It sounds plausible. Of course, the real challenge comes in accurately knowing how a particular action affects everything else. For some actions, that might be easy. But the world is a very complex place in which people do not live in isolation and where a seemingly insignificant decision on my part may have a totally unconsidered impact on your part. Like a stone thrown into the center of a pond, a ripple effect may be produced with unintended results.Why can't I marry my sister? Why can't I walk down the street naked if I want to? Why can't I get a beer in a bar if I'm only 16?

Jefferson wrote man is free from all but the moral law. This is the simple meaning of the proposition that all men are created equal.The problem with unbridled freedom is that while everyone possesses the same rights and liberties by nature, those rights are insecure without government.

But what is government itself but the greatest of all reflections on human nature? If men were angels, no government would be necessary. [Federalist 51, James Madison]

excon rated this answer Excellent or Above Average Answer

Question/Answer
HANK1 asked on 07/05/04 - NO POLITICS?


What kind of a Nation would the United States be WITHOUT politics?

HANK

tomder55 answered on 07/06/04:

I can't tell you how many times I attend town and village meetings from places outside the jurisdiction of my local community .Why? Because the decisions they are considering will have an impact on my life . Should I not have a say even though I do not live within the boundries of the locality ? Can my town dump untreated sewage upsream from some other towns drinking supply ? Will the Interstate connection that gets completed in some other State not affect the traffic flow through my town ,causing unsafe levels of ozone in the air I breath ? There are too many examples of this lack of regional planning for me to mention ,but constantly live with the impact of short sighted decsions from places I have no representation in .

HANK1 rated this answer Excellent or Above Average Answer

Question/Answer
excon asked on 07/03/04 - Values


Gmorning Mavens,

Kerry used the V-word the other day. Hes taking it back from the right wingers. I think he has a point. As we approach our day in the sun, Im curious to hear your take on what, indeed, are this countrys values?

To be EXCLUSIVE - as in our policy to insure that homos and combatants (among others) dont get rights?

Or to be INCLUSIVE - as our founding documents require us to be?

Am I old fashioned, nuts, leftwing, or just plain wrong, when I take the words written by Thomas Jefferson to mean what they say, when he wrote our Declaration of Independence? He uses the term ALL, and says that these rights are inalienable. I believe him. Doesnt that mean that everybody automatically has these rights period? And, isnt it our job to make that happen?

Please dont dwell on the specific examples I used. They are not the point.

Do I misread who we are? Or who we have become?

excon

tomder55 answered on 07/06/04:

I would be remiss in pointing out that Kerry invoking the use of values after this weekends announcement that although he is personally pro-life ,and believes that life begins at conception ,and then stating that although he is morally opposed to abortion ;he still supports abortion rights is the absolute height of hypocracy .He has no business talking values .

I guess he is in good company . Jefferson in his time talked out of both sides of his mouth . The contradictions of his prose to the way he conducted his life are well recorded in history so there is no need for me to recount them.

A reading of history makes the purpose of marriage very clear . Marriage is not meant solely, or even mainly, for husbands and wives. Marriage exists as a public institution because children need mothers and fathers. Once marriage is treated as a mere celebration of the love of two adults, there is no reason for it ; (the legal benefits of which based on the current rate of divorce is something that our lawmakers should review anyway).I have in the past said that homosexuals should be granted the legal benefits that straight couples have ,without the term marriage being used to describe the relationship . Ultimately, the marriage issue is going to be resolved nationally. Either the Supreme Court will nationalize gay marriage, or we shall have a Federal Marriage Amendment. The states have always had the power to regulate the specifics of marriage. But the l definition of marriage has always been a matter for the nation: That's why Utah's admission to the union was conditioned on its abandoning the legality of polygamy.

The right "to pursue happiness," is key to understanding the Declaration's basic principle. Happiness is an individual, subjective notion. What makes you happy is not necessarily what makes me happy. In principle, the more we ask government to do for us, the more we undermine that vision. But collectively; instead of individually we have voluntarily accepted these restrictions on our liberty. Go and ask most Americans if they would like to lose the entitlement of Social Security. Overwhelmingly they want it. The price of not choosing the time of your retirement is restriction accepted as part of the program.

I am sure that the founders did not see the concept of individual freedom as unlimited .By forging a Republican Government they conceded the need of suborning liberty to a degree for the common security. To maintain as much freedom as possible they supported the concept of a limited government, with safeguards built into the system .The most important one being the review process; and the separation of powers.

The government has in the past restricted liberties to protect security in times of emergency. But the restrictions were reversed when the emergency passed. The end to the war on terrorism will not be an identifiable so there is no point at which it is apparent that the threat will end.. This is a new situation for this country to debate .. Most Americans support the government's actions but are naturally suspicious of granting powers of detention, surveillance and secret collection of information.

Our commitment to liberty and due process is as strong as our demand of national security .To safeguard our liberty we must insist that no one or agency, not even the president, should be permitted to restrict liberties without the review process that is written in the Constitution. That is why the Supreme Court was correct last week in its Guantanamo decisions .

Congress passed the Patriot Act with dates for periodic review .Many powers granted in the Patriot Act are necessary, for instance, surveillance of certain individuals who have not yet committed a crime. But oversight, safeguards and review are necessary to prevent abuses. Our system is flexible enough to both provide security and to protect rights.


excon rated this answer Excellent or Above Average Answer
Yiddishkeit rated this answer Excellent or Above Average Answer

Question/Answer
chekhovToo asked on 06/30/04 - Shattered illusions

Francis Fukuyama: Shattered illusions - The Age.

29jun04

OF all of the different views that have now come to be associated with neo-conservatives, the strangest one to me was the confidence that the US could transform Iraq into a Western-style democracy and go on from there to democratise the broader Middle East.

It struck me as strange precisely because these same neo-conservatives had spent much of the past generation warning about the dangers of ambitious social engineering and how social planners could never control behaviour or deal with unanticipated consequences.
If the US cannot eliminate poverty or raise test scores in Washington, DC, how in the world does it expect to bring democracy to a part of the world that has stubbornly resisted it and is virulently anti-American to boot?

Several neo-conservatives, such as Pulitzer prize-winning columnist Charles Krauthammer, have noted how wrong people were after World War II in asserting that Japan could not democratise. Krauthammer asks: "Where is it written that Arabs are incapable of democracy?" He is echoing an argument made most forthrightly by the eminent Middle East scholar Bernard Lewis, who has at several junctures suggested that pessimism about the prospects for a democratic Iraq betrays lack of respect for Arabs.

It is, of course, nowhere written that Arabs are incapable of democracy, and it is certainly foolish for cynical Europeans to assert with great confidence that democracy is impossible in the Middle East. We have, indeed, been fooled before, not just in Japan but in Eastern Europe after the collapse of communism.

But possibility is not probability, and good policy is not made by staking everything on a throw of the dice. Culture is not destiny, but culture plays an important role in making possible certain kinds of institutions something that is usually taken to be a conservative insight.

Though I, more than most people, am associated with the idea that history's arrow points to democracy, I have never believed that democracies can be created anywhere and everywhere through simple political will.

Prior to the Iraq war, there were many reasons for thinking that building a democratic Iraq was a task of a complexity that would be nearly unmanageable. Some reasons had to do with the nature of Iraqi society: the fact that it would be decompressing rapidly from totalitarianism, its ethnic divisions, the role of politicised religion, its tribal structure and the dominance of extended kin and patronage networks, and its susceptibility to influence from other parts of the Middle East that were passionately anti-American.

But other reasons had to do with America. The US has been involved in approximately 18 nation-building projects between its conquest of the Philippines in 1899 and the current occupations of Afghanistan and Iraq, and the overall record is not a pretty one. The cases of unambiguous success Germany, Japan and South Korea were all cases where US forces came and then stayed indefinitely.

In the first two cases, we weren't nation-building at all, but only re-legitimating societies that had very powerful states. In all of the other cases, the US either left nothing behind in terms of self-sustaining institutions, or else made things worse by creating, as in the case of Nicaragua, a modern army and police but no lasting rule of law.

This gets to a fundamental point about unipolarity. True, there is vast disparity of power between the US and the rest of the world, vaster even than Rome's dominance at the height of its empire. But that dominance is clear-cut only along two dimensions of national power, the cultural realm and the ability to fight and win intensive conventional wars. Americans have no particular taste or facility for nation-building; we want exit strategies rather than empires.

So where does the domestic basis of support come for this unbelievably ambitious effort to politically transform one of the world's most troubled and hostile regions? And if the nation is really a commercial republic uncomfortable with empire, why should Americans be so eager to expand its domain? In Iraq, since the US invasion, we Americans have been our usual inept and disorganised selves in planning for and carrying out the reconstruction, something that should not have surprised anyone familiar with American history.

As it happened, many Europeans raised some of these doubts in the lead-up to war in March 2003. Many Europeans did not particularly trust the US to handle the post-war situation well, much less the more ambitious agenda of democratising the Middle East. They also tended not to be persuaded that Iraq was as dangerous as the Bush administration claimed.

They argued that Baathist Iraq had little to do with al-Qa'ida, and that attacking Iraq would be a distraction from the larger war on terrorism. And they believed that the ongoing Palestinian-Israeli conflict was a more dangerous source of instability and terrorism than Iraq, and that the Bush administration was undercutting its own credibility by appearing to side so strongly with the policies of Ariel Sharon.

All of these were and are, of course, debatable propositions. On the question of the manageability of post-war Iraq, the more sceptical European position was almost certainly right.

The Bush administration went into Iraq with enormous illusions about how easy the post-war situation would be: it thought the reconstruction would be self-financing, that Americans could draw on a lasting well of gratitude for liberating Iraq, and that we could occupy the country with a small force structure and even draw US forces down significantly within a few months.

On the question of the threat posed by Iraq, everyone Europeans and Americans were evidently fooled into thinking that it possessed significant stockpiles of chemical and biological weapons. But on this issue, the European bottom line proved to be closer to the truth than the administration's far more alarmist position.

The question of pre-war Iraq-al-Qa'ida links has become intensely politicised in the US since the war. My reading of the evidence is that these linkages existed but that their significance was limited. We have learned since September 11 that al-Qa'ida did not need the support of a state such as Iraq to do a tremendous amount of damage to the US, and that attacking Iraq was not the most direct way to get at al-Qa'ida.

On the question of Palestine, the Europeans are likely wrong, or at least wrong in their belief that we could move to a durable settlement of the conflict if only the US decided to use its influence with Israel.

The point here is not who is right, but rather that the prudential case was not nearly as open-and-shut as many neo-conservatives believed. They talk as if their (that is, the Bush administration's) judgment had been vindicated at every turn, and that any questioning of their judgment could only be the result of base or dishonest motives. If only this were true. The fact that Washington's judgment was flawed has created an enormous legitimacy problem for the US, one that will hurt American interests for a long time to come.

The lesson of Iraq is that the US needs to be more prudent and subtle in exercising power in pursuit of both its interests and values. The world's sole superpower needs to remember that its margin of power is viewed with great suspicion around the world and will set off countervailing reactions if that power is not exercised judiciously.

This means, in the first instance, doing the simple work of diplomacy and coalition-building that the Bush administration seemed so reluctant to undertake prior to the Iraq War, and to not gratuitously insult the "common opinions of mankind".

The US does not need to embrace the UN or multilateralism for its own sake, or because we somehow believe that such institutions are inherently more legitimate than nation states.

On the other hand, the US needs like-minded allies to accomplish both the realist and idealist portions of our agenda, and should spend much more time and energy cultivating them.

Democracy promotion, through all of the available tools at America's disposal, should remain high on the agenda, particularly with regard to the Middle East. But the US needs to be more realistic about its nation-building abilities, and cautious in taking on large social engineering projects in parts of the world it doesn't understand very well.

Francis Fukuyama, professor of international political economy at the Paul H. Nitze School of Advanced International Studies at Johns Hopkins University in Washington, is author of The End of History and the Last Man (Penguin, 1992). A longer version of this article appears in the upcoming edition of The National Interest in Washington.


tomder55 answered on 06/30/04:

I for one was never under any illusion that this would be easy ,or that we could go in ,liberate ,and get out. Still the gamble that was taken was worth it ,and although it has not gone as smooth as I would've liked,the job is getting done .

Fukuyama says we've undertaken 18 nation building projects in roughly 100 years ,and mentions only 4 by name. I would be interested to see which nations he is talking about . Hard to dispute if there are no further details . He is wrong about Germany and Japan of course. First it did not take an indefinite stay in either case. Although American troops are still in these nations it is by invitation and treaty . How can it be occupation if we would leave whenever we were requested to do so?(Like is Subic Bay ;Phillipines) The Germans after 2 years of occupation were not in as good a shape to run their affairs than Iraq is today. He says that :we were 're-legitimating societies that had very powerful states.'.That is non-sense. Both were totalitarian .In the case of Germany ,they had a very limited exposure to self rule during the disasterous Weimar era .They were a 'tribal' nation that was united twice by military autocracy and fascism. The Japanese never before 1945 experienced democracy in their long history .They were ruled by an emperor who's authority was greater than the european divine rights of kings . South Korea has been an economic powerhouse as a free nation .Is it his contention that only a nation that can defend it's own freedom without external help is deserving of freedom? If that were the case ,then only the U.S. is deserving of freedom. the rest of the world has relied on alliances for their mutual protection .

What can happen in Iraq? Look at the Kurdistan region of Iraq to explore the possibilities . Seven million people live under a humane government with less than 300 American troops. How did that happen? The people of Kurdistan are Islamic, often quarrelsome , in the heart of the Middle East .They now residing in relative safety and autonomy, and express good will toward the United States. They accept that we don't want Kurdish oil any more than we want to take over the sands and slums of the Sunni Triangle. The problem in the rest of Iraq is not us, but unlike Kurdistan which had a decade of transition toward consensual society ;thanks to American pilots ,the country is reeling from 30 years of autocracy, in which State fascism offered the only alternative to Arabian styled sheikdoms or to the new Islamo-fascism. None of these alternatives offer any hope to the people . We have always had a "plan" in Iraq .It was to leave the country something like its northern third in Kurdistan.

Neocon" is now a slur for "Jew." General Zinni is now endlessly on television promoting his new book,and criticizing the war talks about "Perle, Wolfowitz, and Feith" and all those who purportedly got us into Iraq. Al Gore Recently screamed out the names of those who must walk his plank, and went into an exorcist-like trance when spit out the name "Woolfwoootizzzzz."
"Cabal" and "Nazi-like" are also used by others and with increasing frequency to promote the old idea of crafty, sneaky people pulling the wool over our eyes .I will take the neo-con idealism over the real-politik maneovers we have done in the past .To me ;they were the devil bargains.

I could go on and on about this posting ;his argument is so full of holes . The Europeans ? It is pretty clear in light of the oil for food scandal where their interest lied . All the charges of 'blood for oil',and they are the ones caught with their fingers in the cookie jar. The U.S. on the other hand has liberated 2 nations in a little over a decade where 1/3 of the worlds oil supply is ,and have returned both nations . If the Europeans are so concerned about 'occupation ' (the plight of the Palestinians) then why do they continue to
do business with China ?Why not demand a free Tibet as a condition of doing business with them?

This may be the most absurd comment in this piece:[We have learned since September 11 that al-Qa'ida did not need the support of a state such as Iraq to do a tremendous amount of damage to the US, and that attacking Iraq was not the most direct way to get at al-Qa'ida.].Is he joking? al Qaeda could never have existed with out State sponsoring.

I have to cut this off now ,but am willing to persue this debate in more detail at another time .


chekhovToo rated this answer Excellent or Above Average Answer
elgin_republicans rated this answer Excellent or Above Average Answer
ETWolverine rated this answer Excellent or Above Average Answer

Question/Answer
HANK1 asked on 06/26/04 - JUNE 30TH ...



That's the date the United States is going to give Iraq back to the Iraqis. Your thoughts re: a smooth or rough landing!

HANK

tomder55 answered on 06/27/04:

it is a great day coming for te Iraqis ,another milestone in the road to self rule.

From the American standpoint we have to remember that Iraq is just one of the battles in the war against the Islamo-fascists.Either we are waging the war in Afghanistan,Iraq,Syria,Iran or the streets of Manhattan.June 30 will not change that reality.As Elgin correctly points out after June 30 when a car bomb explodes,or a pipe line or an electric plant is sabotaged,the terrorists will be waging war against the Iraqi people ,and not 'the occupation'.PM Allawi has vowed to take a tough stand against the terrorists ,I I believe he can deliver.

All the efforts to undermine the effort has failed. First the U.N. road block ;then the smuggling of French and Russian equipment prior to the war were dealt with by our brave forces.Abu Musab al-Zarqawi's attempts to create civil war have failed,as his two desperate intercepted letters to bin Laden have illustrated.One might remark on the extremity of the Jihadi effort in Iraq. They are sending their best team, the team that harried the IDF out of Lebanon . US forces have quietly become very efficient, with chemical test kits to screen suspects for explosive residue, aircraft which electronically detonate IEDs, a steady drumbeat of raids on explosives factories and other operational advances. The enemy is still able to kill Americans, but not in any decisive numbers.

The Iranian interventions including the failed attempt by al Sadr to create a Shite uprising have been checked .The Iranians will have to be dealt with in the future as any real assessment of the situation there shows that we are in a shooting war with both nations already.Teheran threatens to become a nuclear power in the very near future and is, despite Sunni pretensions to the contrary, still the central star in the Jihadi . For the present, it is actually in the Coalition's interest and probably no one else's to build up a truly independent Iraq. Iraq would become another player to the game to balance off Syria, Saudi Arabia and Iran. A strong Iraq, especially an independent Shi'ite Iraq would be a deadly threat to the Mullahs in Tehran.

The week leading up to the formal transfer of power to the Iraqi interim government will be punctuated by heavy yet pointless violence. The transfer is as unstoppable as the Overlord invasion, Zarqawi or no. The enemy had better prepare his fallback position and prepare for the next phase of the campaign.





HANK1 rated this answer Excellent or Above Average Answer
purplewings rated this answer Excellent or Above Average Answer
Yiddishkeit rated this answer Excellent or Above Average Answer

Question/Answer
Itsdb asked on 06/25/04 - In honor of Al Gore

Comfortably Numb (Gilmour, Waters)

Hello?
Is there anybody in there?
Just nod if you can hear me.
Is there anyone at home?
Come on, now,
I hear you're feeling down.
Well I can ease your pain
Get you on your feet again.
Relax.
I'll need some information first.
Just the basic facts.
Can you show me where it hurts?

There is no pain you are receding
A distant ship, smoke on the horizon.
You are only coming through in waves.
Your lips move but I can't hear what you're saying.
When I was a child I had a fever
My hands felt just like two balloons.
Now I've got that feeling once again
I can't explain you would not understand
This is not how I am.
I have become comfortably numb.

O.K.
Just a little pinprick.
There'll be no more aaaaaaaaah!
But you may feel a little sick.
Can you stand up?
I do believe it's working, good.
That'll keep you going through the show
Come on it's time to go.

There is no pain you are receding
A distant ship, smoke on the horizon.
You are only coming through in waves.
Your lips move but I can't hear what you're saying.
When I was a child
I caught a fleeting glimpse
Out of the corner of my eye.
I turned to look but it was gone
I cannot put my finger on it now
The child is grown,
The dream is gone.
I have become comfortably numb.

tomder55 answered on 06/26/04:

too funny .that is from one of my all time favorite albums 'The Wall' by Pink Floyd.
One of my favorite renditions of it was performed by Van Morrison in July 1990 when
Roger Waters invited a number of guest stars to perform 'The Wall 'at the Brandenburg Gate.

Itsdb rated this answer Excellent or Above Average Answer

Question/Answer
ETWolverine asked on 06/22/04 - AIR AMERICA HITS SOUR RATINGS NOTE By JOHN MAINELLI



(Also from today's NY Post. The liberals just don't seem to be having a good week. --- Elliot)



June 22, 2004 -- ENCOURAGING preliminary ratings for all-liberal Air America in New York have collapsed along with the fledgling radio network's finances.
An unofficial "extrapolation" of Arbitron data released last Friday which Air America's hosts crowed about last month but virtually ignored yesterday showed WLIB's ratings dropping back to their lowly levels before the net's April launch.

Arbitron cautions stations and advertisers not to read too much into this interim monthly data but that didn't stop Air America star Al Franken from boasting last month that he'd beaten WABC's Rush Limbaugh among the 25- to 54-year-old listeners chased by radio advertisers.

Instead of ratings yesterday, Franken discussed a Wall Street Journal article that provided frightening details about alleged phantom finances including purported promises of major backing from liberal moneybags Norman Lear and Larry David that resulted in a near-complete shakeup of Air America's executive suite last month.

According to the article, many Air America investors thought the network had raised $30 million when, in fact, only $6 million had been raised before the network launched.

"We have a new influx of cash coming up," said Franken, whose contract promises more than $1 million a year, according to the Journal.

"I am being paid now," he told listeners yesterday. "I've been paid for weeks."

Franken took issue with Fox News Channel's Bill O'Reilly for interviewing a reporter last week who said Air America investors have been throwing money "down a rat hole."

"Fox News Channel lost $150 million in its first two years," Franken said. "That was a much bigger rathole than our rathole."

Air America president Jon Sinton, who declined comment through a spokesman yesterday, told The Post in April that it would be unfair to make too many judgments too soon.

"It takes a long time to develop a [talk radio] audience," Sinton said, echoing the sentiments of many talk radio experts. "This is a long-term project."

The lefty network, which hopes to counter the election-year influence of high-rated conservative talk radio, is still without affiliates in Los Angeles and Chicago after noisy disputes over station lease payments.

Except for New York's WLIB (1190 AM), its 15 affiliates are in medium to small markets.

Fox News Channel is owned by News Corp., which also owns The Post.


---------------

Looks like they've lost that second viewer...

Elliot

tomder55 answered on 06/23/04:

The original owners appear to have . . . er . . . creatively misdescribed the state of their finances when they were raising money and starting operations. The $6 million they actually raised, they blew threw by the first couple of weeks on the air.

Now the guy with the original idea, Sheldon Drobny, is apparently trying to stage an asset sale to a new entity in order to salvage the operation. This is tricky, because it leaves the new entity vulnerable to charges of setting up a "sham transaction" in order to bilk Air America's creditors. Maybe they saw 'The Producers' one to many times .

ETWolverine rated this answer Excellent or Above Average Answer

Question/Answer
ETWolverine asked on 06/22/04 - MOORE IS LESS By LOU LUMENICK

From today's NY Post Entertainment section ---


June 22, 2004 --

Fahrenheit 9/11

Dude, where's my expos? Running time: 116 minutes.

Not rated (disturbing images). Tomorrow at the Lincoln Plaza and Loews Village VIII (additional theaters on Friday).

'FAHRENHEIT 9/11," Michael Moore's much-hyped and very heavy-handed polemic against George W. Bush, is basically a two-hour argument for regime change that isn't half as incendiary or persuasive as its maker would have you believe.

Far from uncovering the smoking gun that he promised before the movie's premiere at the Cannes Film Festival last month, the leftist filmmaker basically rehashes a litany of familiar gripes against Dubya, many of them recycled from Moore's best-selling "Dude, Where's My Country?"

For a liberal who awarded four stars to Moore's "Bowling for Columbine," this wet firecracker of a film is especially disappointing.

Though the version that hits theaters tomorrow is slightly more effective than the edition that won the top prize in Cannes - thanks to additional footage of Dubya and administration officials spouting on increasingly dubious links between Iraq and Al Qaeda, as well as those as-yet-to-be-discovered weapons of mass destruction in Iraq - Moore is still basically preaching to the converted and is unlikely to win over all that many hearts and minds.

Liberals may chortle at the sight of Deputy Defense Secretary Paul Wolfowitz slicking down his hair with his own spit to prepare for a TV appearance, or Attorney General John Ashcroft belting out an excruciating self-composed ditty called "Let the Eagle Soar," but these attempts at humor end up undercutting what Moore apparently intends as a mostly serious attack on the president's response to the 9/11 attacks.

Moore comes closer to drawing blood with damning footage of a dazed Bush continuing to read with a group of Florida schoolchildren at a photo op for seven agonizing minutes after learning of the attack on the second World Trade Center tower.

But the filmmaker can't resist going over the top with his sarcastic narration: "What's he thinking, 'I've been hanging out with the wrong crowd? Which of them screwed me?'"



This leads into a long and eyeball-glazing section on the longtime association between the Bush family and the Saudi royal family, including members of Osama bin Laden's large extended clan - not exactly the scandal Moore makes it to be, considering that the Bushes have long been in the oil business, as have the Saudis.

Moore's critics on the right have long complained he plays fast and loose with the facts, and he gives them ammunition by implying that Bush allowed a large contingent of bin Ladens and other Saudis, mostly students, to leave the United States immediately after 9/11, while commercial flights were still grounded.

In fact, the bipartisan Sept. 11 commission has found that most, if not all, of these evacuations took place after Sept. 13, when the ban on commercial flights was lifted - and the evidence weighs against Dubya even being aware of these special flights.

The sheer scope of the material that Moore is trying to cover in a two-hour documentary - the Sept. 11 attacks themselves rate maybe five minutes, less time than is devoted to the Patriot Act and supposed war profiteering by the politically connected Halliburton Corp. - leading to an incredibly superficial and misleading treatment at times.

Sure, there's some graphic footage - apparently shot by one of the camera crews that Moore claims to have smuggled in with embedded troops - showing Iraqis maimed during U.S. air strikes.

We also see American soldiers laughing as they place hoods over Iraqi prisoners, and one GI touching a sleeping detainee's genitals through a blanket.

But the latter scene actually conflicts with one of Moore's main arguments - that GIs have been victimized by being forced to participate in what he considers to be the unnecessary and immoral invasion of Iraq.

He is also shocked, shocked to learn that Army recruiters target areas with heavy minority populations - something that's been true since at least the Vietnam era (though most of the soldiers in Iraq Moore depicts are white).

Moore's big stop-the-presses revelation is that the name of James Bath, an old pal of the president who works as a money manager for the bin Laden family (which has long renounced Osama), was excised from 1972 National Guard records released by the White House. Yawn.

In "Bowling for Columbine," Moore had something new to add to the gun-control debate and did so in a refreshingly entertaining manner.

ŝ/11" does not lend itself to such a glib approach, and while Moore may get laughs by presenting Bush and his staff in a brief "Bonanza" spoof titled "Afghanistan," the humor often seems more forced here.

There is much less footage of Moore himself than in his previous features, possibly because his trademark ambush stunts - like trying to get congressmen to volunteer their children to serve with the military in Iraq - fall so flat.

By far the best sequence features Lila Lipscomb, a patriotic woman from Moore's hometown of Flint, Mich., who lost her Marine son in Iraq.

But when she tries to go to the White House to express her newly anti-war feelings, Moore ends up with a pallid echo of the high point of "Columbine," where victims of that high school massacre descend on Kmart headquarters to demand that the chain stop selling ammunition.

The weakest section is the opening, in which Moore revisits the now wearisome accusations that Dubya "stole" the election from Al Gore with help from a right-leaning Supreme Court.

Moore conveniently doesn't mention that in the last election, he was backing Ralph Nader - whose candidacy many believe effectively handed the White House to Bush - or that in "Dude, Where's My Country?" Moore states the next president should be ... Oprah Winfrey!

-----------

Gee, I don't suppose that THIS could be the reason that Disney didn't want to distribute the film could it?

Elliot

tomder55 answered on 06/22/04:

Rule # 3:

The strengthening of the commitment, intellectual performance, and morale of those already on your side is an essential task, both in order to bind them more securely to the cause and to make them more effective exponents of it.

A PRIMER FOR POLEMICISTS by Irving Kristol

Does anyone really believe that the FBI compounded it's mistake of not tracking terrorists in the country by allowing anyone to leave the country immediately after 9-11 without being properly vetted ? Apparently so. It abosolutely boggles the mind!

Disney it seems drew the line between fantasy ,which they have been purveyors for years ,and docu-fantasy.

ETWolverine rated this answer Excellent or Above Average Answer

Question/Answer
Itsdb asked on 06/19/04 - Did anyone notice?

Putin says Russia gave U.S. information about potential Iraqi attacks in United States

By Bagila Bukharbayeva
The Associated Press

ASTANA, Kazakhstan - "Russian President Vladimir Putin said Friday his government warned Washington that Saddam Hussein's regime was preparing attacks in the United States and its interests abroad - an assertion that appears to bolster President Bush's contention that Iraq was a threat.

Putin emphasized that the intelligence didn't cause Russia to waver from its firm opposition to the U.S.-led war last year, but his statement was the second this month in which he has offered at least some support for Bush on Iraq.

"After Sept. 11, 2001, and before the start of the military operation in Iraq, the Russian special services ... received information that officials from Saddam's regime were preparing terrorist attacks in the United States and outside it against the U.S. military and other interests," Putin said.

"Despite that information ... Russia's position on Iraq remains unchanged," he said in the Kazakh capital, Astana, after regional economic and security summits. He said Russia didn't have any information that Saddam's regime had actually been behind any terrorist acts.

"It's one thing to have information that Saddam's regime is preparing terrorist attacks, (but) we didn't have information that it was involved in any known terrorist attacks," he said.

Putin didn't elaborate on any details of the alleged plots or mention whether they were tied to al-Qaida. He said Bush had personally thanked one of the leaders of Russia's intelligence agencies for the information but that he couldn't comment on how critical it was in the U.S. decision to invade Iraq.

In Washington, a U.S. official said Putin's information did not add to what the United States already knew about Saddam's intentions.

The official, who spoke on condition of anonymity, said Putin's tip didn't give a time or place for a possible attack.

Bush alleged Thursday that Saddam had "numerous contacts" with al-Qaida and said Iraqi agents had met with the terror network's leader, Osama bin Laden, in Sudan.

Saddam "was a threat because he had terrorist connections - not only al-Qaida connections, but other connections to terrorist organizations," Bush said."

Why is it the press only begrudgingly acknowledged these facts?

tomder55 answered on 06/20/04:

I agree with the other experts in mistrusting Putin's motives. I think the more he supports the war on terror ,the more he can justify his consolidation of power in Russia. However,you are correct in that the press as usual downplays anything that would support the administrations efforts while at the same time harps ad nausium things that undermines the effort.

Itsdb rated this answer Excellent or Above Average Answer

Question/Answer
Itsdb asked on 06/18/04 - The bigger 9-11 picture

On the collective orgasm the left is having over the sentence "We have no credible evidence that Iraq and al Qaeda cooperated on attacks against the United States," it seems everyone has missed the bigger picture here. As a public service, here is the last page of the report from which the notorious sentence was extracted.

"Bin Ladin continues to inspire many of the operatives he trained and dispersed, as well as smaller Islamic extremist groups and individual fighters who share his ideology. As a result, al Qaeda today is more a loose collection of regional networks with a greatly weakened central organization. It pushes these networks to carry out attacks, and assists them by providing guidance, funding, and training in skills such as bomb-making or urban combat.

Al Qaeda remains extremely interested in conducting chemical, biological, radiological, or nuclear attacks. In 1994, al Qaeda operatives attempted to purchase uranium for $1.5 million; the uranium proved to be fake. Though this attempt failed, al Qaeda continues to pursue its strategic objective of obtaining a nuclear weapon. Likewise, it remains interested in using a radiological dispersal device or dirty bomb, a conventional explosive designed to spread radioactive material. Documents found in al Qaeda facilities contain accurate information on the usage and impact of such weapons.

Al Qaeda had an ambitious biological weapons program and was making advances in its ability to produce anthrax prior to September 11. According to Director of Central Intelligence George Tenet, al Qaedas ability to conduct an anthrax attack is one of the most immediate threats the United States is likely to face. Similarly, al Qaeda may seek to conduct a chemical attack by using widely-available industrial chemicals, or by attacking a chemical plant or a shipment of hazardous materials.

The Intelligence Community expects that the trend toward attacks intended to cause everhigher casualties will continue. Al Qaeda and other extremist groups will likely continue to exploit leaks of national security information in the media, open-source information on techniques such as mixing explosives, and advances in electronics. It may modify traditional tactics in order to prevent detection or interdiction by counterterrorist forces. Regardless of the tactic, al Qaeda is actively striving to attack the United States and inflict mass casualties.

Does anyone even realize this report is not about Iraq, but is an "Overview of the Enemy" that still seeks to destroy us today?

Steve

tomder55 answered on 06/18/04:

Well, the Commission did say there was "no credible evidence" and that's what's being put out there for the puplic. It doesn't matter if there was more to the report because that one blurb is the direction the media wants to play it.

Yesterday both Tom Kean and Lee Hamilton were interviewed by Maargret Warner.When asked about the 'staff statement'(it is not a final conclusion by the Commission )Kean said:

But the staff in their investigation has found that, yes, there were contacts between Iraq and al-Qaida, a number of them, some of them a little shadowy. They were definitely there. But as far as any evidence that Saddam Hussein was in any way involved in the attack on 9/11, it just isn't there.

Lee Hamilton said :

I don't think there's any doubt but that there were some contacts between Saddam Hussein's government and al-Qaida, Osama bin Laden's people.

In other words ,the Press reporting that there was no leak according to the staff statement is a distortion.But I do not blame the press for the initial reporting of this .The commission leaked the paragraph to the media. This started the ball rolling before anyone had a chance to put the report in context.Since then there has been plenty of opportunity for the press to accurately report the truth ,and not much has been done in the way of retraction or clarification .

I concure that not only the threat to us from al-qaeda is being intentionally understated ;but also the nature of the enemy .

Itsdb rated this answer Excellent or Above Average Answer

Question/Answer
excon asked on 06/17/04 - 9/11 Commission


Hello war dudes whore doin the wrong war:

It's hard to imagine how the bi-partisan commission investigating the 2001 terrorist attacks could have put it more clearly yesterday: there was never any evidence of a link between Iraq and Al Qaeda, between Saddam Hussein and Sept. 11.

I guess the information you had somehow didnt get to the committee, or maybe its a cover-up by (what would gade call it?) the hate Bush crowd.

This is not just a matter of the president's diminishing credibility, although that's disturbing enough. The real war on terror has actually suffered as the conflict in Iraq has diverted military and intelligence resources from places like Afghanistan, where there could really be Qaeda forces, including bin Laden.

Did he lie, or was he fooled? I dunno and I dont care. In either case, his incompetence is evident. If theres anything I expect from a Republican administration, it's competence. My only fear is that Kerry may be equally incompetent.

excon

tomder55 answered on 06/17/04:

I have about 100 links that says otherwise. Here is the relevent part of the statement by the commission:

Bin Laden also explored possible cooperation with Iraq during his time in Sudan, despite his opposition to Hussein's secular regime. Bin Laden had in fact at one time sponsored anti-Saddam Islamists in Iraqi Kurdistan. The Sudanese, to protect their own ties with Iraq, reportedly persuaded Bin Laden to cease this support and arranged for contacts between Iraq and al Qaeda. A senior Iraqi intelligence officer reportedly made three visits to Sudan, finally meeting Bin Laden in 1994. Bin Laden is said to have requested space to establish training camps, as well as assistance in procuring weapons, but Iraq apparently never responded. There have been reports that contacts between Iraq and al Qaeda also occurred after Bin Laden returned to Afghanistan, but they do not appear to have resulted in a collaborative relationship. Two senior Bin Laden associates have adamantly denied that any ties existed between al Qaeda and Iraq. We have no credible evidence that Iraq and al Qaeda cooperated on attacks against the United States.

Ok ,now they had access to all types of files;including CIA . But here is what George Tenet wrote to Congress 10/7/02 .

Our understanding of the relationship between Iraq and Al Qaeda is evolving and is based on sources of varying reliability. Some of the information we have received comes from detainees, including some of high rank. We have solid reporting of senior level contacts between Iraq and Al Qaeda going back a decade. Credible information indicates that Iraq and Al Qaeda have discussed safe haven and reciprocal nonaggression. Since Operation Enduring Freedom, we have solid evidence of the presence in Iraq of Al Qaeda members, including some that have been in Baghdad. We have credible reporting that Al Qaeda leaders sought contacts in Iraq who could help them acquire W.M.D. capabilities. The reporting also stated that Iraq has provided training to Al Qaeda members in the areas of poisons and gases and making conventional bombs. Iraq's increasing support to extremist Palestinians coupled with growing indications of relationship with Al Qaeda suggest that Baghdad's links to terrorists will increase, even absent U.S. military action.

He said the same thing to the Senate Armed Services Committee March 9, 2004.

According to the staff report, bin Laden originally pressed Khalid Sheik Mohammed "that the attacks occur as early as mid-2000," even though bin Laden "recognized that Atta and the other pilots had only just arrived in the United States to begin their flight training .Then suddenly Atta makes two trips to Prague (to meet with Iraqi Intelligence Agents according to Chech Intelligence)immediately before heading to the United States to begin that flight training. Coincidence?

The 9-11 Commission is not the last word on this . Because they did not find proof positive that Saddam wasn't involved in operational planning of 9-11 doesn't mean that there existed no cooperation between Saddam and al Qaeda. One of the first missions of the current war was a bombing raid of an al qaeda camp operating inside of Iraq. How is it that an al-qaeda camp was there if Saddam did not permit it?

ETWolverine rated this answer Excellent or Above Average Answer
Itsdb rated this answer Excellent or Above Average Answer
excon rated this answer Excellent or Above Average Answer
purplewings rated this answer Excellent or Above Average Answer

Question/Answer
SCOOBY asked on 06/10/04 - 911

some people are of the belief that the government intentionally allowed terrorists to attack the twin towers in NY. Do you agree? Does anyone have any idea what might be leading these people to believe that?

tomder55 answered on 06/11/04:

take any major event ,and you can find a conspiracy theory as a parallel explanation to the event. Immediatlely after 9-11 ,the story that the Mossad was involved was circulated .others claimed that Bin Laden is on the Bush family payroll .Michael Moore (Jabba the Nut) took that as a premise ,did a cut and paste hack job from news footage ,ripped off the title of a Ray Bradbury story(FAHRENHEIT 451 ;Moore changed to FAHRENHEIT 911.Bradbury is not a happy camper about that ),and turned it into a political propaganda piece that is intended to undermine the Bush re:election campaign. Moore would make Hitler's Master propagandist Paul Joseph Goebbles proud ;but then again Moore resembles Goebbles in Hermann Goering's body.(actually when you think about it ;Moore has surpassed Goering ,and now more closely resembles the Hindenburg Blimp).






Note -
Bradbury told a reporter for the Swedish daily Dagens Nyheter:"Michael Moore is a screwed a**hole, that is what I think about that case. He stole my title and changed the numbers without ever asking me for permission."" He is a horrible human being. Horrible human!"

ETWolverine rated this answer Excellent or Above Average Answer
Itsdb rated this answer Excellent or Above Average Answer

Question/Answer
Itsdb asked on 06/10/04 - More from an idiot

Pardon the uncensored bs from...

Ted Rall, "America's BS detector"

Thursday, June 10, 2004
Rush Declares Jihad On Your Humble Narrator!

I don't understand why so few of my fellow Democrats are afraid to say in public what they say in private, over beers with their friends. And I can't imagine how anyone, even hard-right Reaganite Christianists, can find it surprising or confusing that not everybody is convulsing with grief now that the guy has finally admitted, 20 years late, that he's dead.

After all, millions of Americans became homeless because of his budget cuts of psychiatric facilities. Millions more lost their jobs because he bankrupted the federal treasury. And half a million people died of AIDS because he refused to fund research into the disease. Millons of college students were forced to drop out of school because of him, and millions of other Americans earned, and still earn, less because of his union busting and corporate welfare. Surely conservatives must recognize that those people--most people--might still hold a grudge, what with the death and maiming and poverty and all.

Alas not. For we must all kneel at the casket of a man who didn't give a shit about his own children, much less the citizenry at large.

Well, not me, anyway. And for that unpardonable sin, right-wing talk radio king Rush Limbaugh lambasted me on his nationally syndicated radio show yesterday. It should be pointed out, by the way, that this cowardly drug-addled blowhard didn't dare go toe to toe with me on his program to discuss Reagan.

http://www.rall.com/rants.html

As I commented to an answer on my previous Rall post, if Rall doesn't want to respect the man or the office, he could at least respect the fact that many, including the family are grieving the loss of a man they loved. It's hypocritical, and typical of those of this mindset, to detest a perceived insensitivity by Reagan toward certain groups, and be so utterly insensitive and insulting at this time of loss...and have the gall to call himself "Your Humble Narrator."

Steve

tomder55 answered on 06/10/04:

I tuned into Air America during lunch ,and by coincidence Ted Rall was being interviewed. Thety asked him what he thought of the network ,and he said that they weren't mean enough ,and that is why the ratings are suffering. Just gives you an idea about his M.O.

Itsdb rated this answer Excellent or Above Average Answer
ETWolverine rated this answer Excellent or Above Average Answer
voiceguy2000 rated this answer Excellent or Above Average Answer
stevehaddock rated this answer Average Answer

Question/Answer
ETWolverine asked on 06/09/04 - My e-mail to Ted Rall regarding his June 6 blog

All right, I know I probably shouldn't have done it, but I wrote the dumb schmuck. Here's what I wrote:

To Ted Rall,

I missed you on "Hannity and Colmes". Sorry, but I'm going to have to limit my comments to what I read in your blog.

First of all, I have a friend who was a student in the Granada Medical school that was under siege. And it was indeed under siege. The fact that there were no enemy soldiers inside the building doesn't mean that the students or teachers were free to leave. They were being held hostage. Of course, if we had done nothing and those students had died, you would be ranting about how ineffective Reagan was in protecting Americans abroad.

Second, the soldiers (if you can call them that) that were faced by the US troops in Granada were CUBAN REGULARS!!! In other words, Cuba invaded Granada. That, in military parlance, is what they call an ACT OF WAR. Of course, if they were invited in by the Granadan government, that would make holding those students under siege an act of war by Granada, wouldn't it? Either way, Reagan was right to send in the US Military to respond to an act of war in which US citizens' lives were in danger.

Third, Reagan bankrupted the Soviet Union single-handedly. It was HIS policies that forced the Soviet Union to become active in multiple theatres (Asia, the Middle East, Africa, South America). He forced them to spend money trying to find a defense against an SDI system that didn't even exist. In short, he bankrupted the Soviets.

"But wait, that wasn't all Reagan's fault, it was the fault of the Soviets and of the policies of the Presidents that came before him," is what you will argue. Or maybe not--- I'm not sure if you're smart enough to have thought of it on your own.

The Soviet Union had been tottering on the edge of bankruptcy for decades. So what. If they were having cash flow problems, they jus stopped feeding a million people or so. They had done it before, and there was no reason to believe that they wouldn't have kept on doing so, if they were able. They sure as hell didn't care if a few more people starved to death for the benefit of the Motherland. But Reagan had the Soviets spending so much money in so many different direction that even starving its own people to death couldn't improve their cash flow enough to pull them out of bankruptcy. Reagan won the Cold War because he understood that the Cold War had turned into a war of attrition, and that our economic system was better suited to survive a war of attrition... even if we ran up deficits to do it. Nobody in the USA ever starved to death because of a US budget deficit. And guess what: Reagan was right.

Fourth, he cut taxes at ALL LEVELS of income. His supply-side economics created more jobs than any of the previous 3 presidents did. He paved the way for the economic boom that we saw in the late 80s and the 90s... a decade that saw more millionaires created from the middle class than any other in history, and a decade in which employment was extremely high, and employees ruled the job market. According to William Niskanen and Stephen Moore of the CATO Institute, 8 out of 10 economic factors measured performed better during the Reagan years than in the Ford/Carter years or the Bush/Clinton Years.

- Economic Growth. The average annual growth rate of real gross domestic product (GDP) from 1981 to 1989 was 3.2 percent per year, compared with 2.8 percent from 1974 to 1981 and 2.1 percent from 1989 to 1995. The 3.2 percent growth rate for the Reagan years includes the recession of the early 1980s, which was a side effect of reversing Carter's high-inflation policies, and the seven expansion years, 1983-89. During the economic expansion alone, the economy grew by a robust annual rate of 3.8 percent. By the end of the Reagan years, the American economy was almost one-third larger than it was when they began. Figure 1 (go to the website---Elliot) shows the economic growth rate by president since World War II. That rate was higher in the 1980s than in the 1950s and 1970s but was substantially lower than the rapid economic growth rate of more than 4 percent per year in the 1960s. The Kennedy income tax rate cuts of 30 percent that were enacted in 1964 generated several years of 5 percent annual real growth.

- Economic Growth per Working-Age Adult. When we adjust the economic growth rates to take account of demographic changes, we find that the expansion in the Reagan years looks even better and that the 1970s' performance looks worse. GDP growth per adult aged 20-64 in the Reagan years grew twice as rapidly, on average, as it did in the pre- and post-Reagan years.

- Median Household Incomes. Real median household income rose by $4,000 in the Reagan years--from $37,868 in 1981 to $42,049 in 1989... This improvement was a stark reversal of the income trends in the late 1970s and the 1990s: median family income was unchanged in the eight pre-Reagan years, and incomes have fallen by $1,438 in the anti-supply-side 1990s, following the 1990 and 1993 tax hikes. Most of the declines in take-home pay occurred on George Bush's watch. Under Bill Clinton's tenure, there has been zero income growth in median household income.

- Employment. From 1981 through 1989 the U.S. economy produced 17 million new jobs, or roughly 2 million new jobs each year. Contrary to the Clinton administration's claims of vast job gains in the 1990s, the United States has averaged only 1.3 million new jobs per year in the post-Reagan years. The labor force United States has averaged only 1.3 million new jobs expanded by 1.7 percent per year between 1981 and 1989, but by just 1.2 percent per year between 1990 and 1995.

- Hours Worked. Table 1 confirms that hours worked per adult aged 20-64 grew much faster in the 1980s than in the pre -or post-Reagan years.

- Unemployment Rate. When Reagan took office in 1981, the unemployment rate was 7.6 percent. In the recession of 1981-82, that rate peaked at 9.7 percent, but it fell continuously for the next seven years. When Reagan left office, the unemployment rate was 5.5 percent. This reduction in joblessness was a clear triumph of the Reagan program. Figure 3 shows that in the pre-Reagan years, the unemployment rate trended upward; in the Reagan years, the unemployment rate trended downward; and in the post-Reagan years, the unemployment rate has fluctuated up and down but today remains virtually unchanged from the 1989 rate.

- Productivity. For real wages to rise, productivity must rise. Over the past 30 years there has been a secular downward trend in U.S. productivity growth. Under Reagan, productivity grew at a 1.5 percent annual rate, as shown in Figure 4. This was lower than in the 1950s, 1960s, and 1970s but much higher than in the post-Reagan years. Under Clinton, productivity has increased at an annual rate of just 0.3 percent per year--the worst presidential performance since that of Herbert Hoover.

- Inflation. The central economic evil that Ronald Reagan inherited in 1981 from Jimmy Carter was three years of double-digit inflation. In 1980 the consumer price index (CPI) rose to 13.5 percent. By Reagan's second year in office, the inflation rate fell by more than half to 6.2 percent. In 1988, Reagan's last year in office, the CPI had fallen to 4.1 percent. Figure 5 shows the inflation and interest rate trend.

- Interest Rates. In 1980 the interest rate on a 30-year mortgage was 15 percent; this rate rose to its all-time peak of 18.9 percent in 1981. The prime rate steadily fell over the subsequent six years to a low of 8.2 percent in 1987 as the inflationary expectation component of interest rates fell sharply. The prime rate hit its 20-year low in 1993 at 6.0 percent. The Treasury Bill rate also fell dramatically in the 1980s--from 14 percent in 1981 to 7 percent in 1988. In the 1990s, interest rates have continued to migrate gradually downward...

- Savings. The savings rate did not rise in the 1980s, as supply-side advocates had predicted. In fact, in the 1980s the personal savings rate fell from 8 percent to 6.5 percent. In the 1990s the average savings rate has fallen even further to an average of 4.9 percent --although the rate of decline has slowed.

---from Supply Tax Cuts and the Truth About the Reagan Economic Record, by William Niskanen and Stephen Moore, October 22, 1996. http://www.cato.org/pubs/pas/pa-261.html

Seems that Reaganomics isn't quite the unmitigated disaster you would have everyone believe it is.

Fifth, Reagan also built the military into a cohesive and power group that was capable of winning the 'illegal' war in Granada in 48 hours, the first Gulf War in just a few weeks and the current Gulf war in a couple of months. We may not be liked around the world right now, but we are NOT a laughingstock. In fact, we are actually FEARED around the world, which is more effective, politically speaking, than being liked. And it is ALL BECAUSE OF REAGAN!!!

Quoting from Ralph Peters... who was there at the time:

IN 1976, I joined the U.S. Army as a private. Our military was broken. My first unit, in Germany, had trucks built in the 1940s, inadequate winter clothing, inept medical care and an atmosphere of pessimism. We were not "combat ready."
Had "the balloon gone up," our Infantry would have entered battle in death-trap M113s that were no match for Soviet infantry combat vehicles. Our tanks couldn't rival the firepower of the new Russian models. Our radios were unreliable and the antique encryption devices rarely worked.
Our war games weren't about winning but about losing as slowly as possible. We always had to resort to nukes in the end.
Then came Ronald Reagan.
Yes, he raised Defense budgets dramatically. And the money mattered. But the increased funding and higher pay wouldn't have made a decisive difference without the sense that we had a real leader in the White House again. The man in the Oval Office genuinely admired the men and women who served. When he saluted his Marine guards, he meant it. The troops could tell.
I attended Officer Candidate School in Georgia during the 1980 presidential election. When I returned to Germany in late 1981, the change in the quality and morale of the "dirty boots" Army was already unmistakable. Even before the new equipment began arriving, the Army was regaining its fighting spirit.
We still had some bad apples but fewer with every infusion of new, better-educated recruits. Officers were held to ever-higher standards. The young sergeants coming up had an energy and optimism that had been missing for years, while the senior NCOs who lasted were the toughest and best of them all. And our new generals, men who had commanded battalions and brigades in Vietnam, had learned the right lessons.
New gear began to arrive. Training budgets increased. We even replaced our janitor-style uniforms with camouflage fatigues. We looked like soldiers again.
We had a president who cared about us, a man who was proud of us and proud of the country we were pledged to defend. He even understood the power of uniforms and would not enter the Oval Office himself unless wearing a tie.
Ronald Reagan brought down the Soviet Union. After all the academic arguments about the USSR's internal weakness and the inevitability of its ultimate failure, the truth is that none of those who speak so knowingly now had the strategic insight of an aging former actor when it mattered.
Which brings me to my confession. Having grown up in the late ླྀs and early ྂs, I carried some of my generation's prejudices along with me into the Army. While I realized that Jimmy Carter had been an inept president (if a good man), I didn't support Ronald Reagan in 1980. I believed that Carter remained the safer of two mediocrities. I bought into the bigotry of those who mocked Reagan as lacking the intelligence to be president.
And it's doubtless true that he didn't possess the highest IQ ever to enter the White House. That goes directly to what Reagan taught me: As we recently saw with another president, the greatest intelligence isn't a substitute for vision, courage and leadership. Above all, a president needs good instincts, guts and sound values. The world's overstocked with brilliant people who never get anything done.
Reagan got things done.
He gave us the military that serves the cause of freedom so well today. He gave us back our pride. And he gave us back our country.
If that wasn't clear from the campus, it was obvious to those of us in the mud on the frontiers of freedom.

--- Excerpts from the New York Post, Reagan to the Rescue, By Ralph Peters, Opinion Column, June 9, 2004.

I believe that I'll take the word of a soldier, and officer who was THERE as it was happening over the word of a guy who's greatest sacrifice was that his best friends lost their scholarships and he didn't have anyone to play with anymore... oh, and just missing that Pulitzer must have been tough too. Boo hoo hoo. Sorry, that doesn't qualify you as more of an expert on the effects of Ronald Reagan's policies on military affairs than someone who was in the military as it was happening.

Finally, with regard to AIDS, Reagan spent $5.7 BILLION on AIDS research... more than any president before or since, with the exception of George Bush. No disease has ever received that much for research from s single president. Not cancer, not kidney disease, not MD. So when you claim that Reagan let hundreds of thousands of AIDS victims die without bating an eyelash, you're not being very truthful on the matter, are you? What pisses you and every other liberal off about Reagan is that he didn't let his $5.7 BILLION support for AIDS research translate into a support for gay rights. If Reagan had backed gay rights, while spending the exact same amount of money on AIDS, liberals would have loved him for 'supporting AIDS sufferers'. But he didn't support gay rights, and THAT is what upsets you so much.

I don't expect you to place this on your blog page. Frankly, I don't think you have the guts to post an e-mail with well-reasoned arguments and historical fact to back it up. You would prefer to dredge up the lowest-comon-denominator e-mails you can find... the ones with vitriol and hate and no logic to them, because that is where you feel most comfortable, and you can argue with them on their level. But I'm cheating... I'm using logic and fact to back up my political point of view. No fair. So no, I don't expect you to post this message online.

Now, go back to drawing pictures to make yourself understood, little boy, and leave the political discussions to the big boys who aren't too lazy to do 10 minutes of research before jumping to an opinion. If you're America's BS detector, do you start beeping every time you walk into a room?

Oh, and by the way, Maggie Thatcher, who has suffered several strokes recently and can barely move, can still kick your pansy ass. Just so you didn't think I had gotten too far from my Right-Wing, gun-totin' roots.

Elliot

tomder55 answered on 06/10/04:

I think Ted Rall pay a visit to the Arizona Cardinals .After his horrid comments about Pat Tillman I'm sure he'd get a warm welcome. He is really the scum of the earth.

ETWolverine rated this answer Excellent or Above Average Answer

Question/Answer
elgin_republicans asked on 06/09/04 - Ahem, please note article posted below, the Giuliani for VP article - you know, the one I predicted


Bush camp considers switch leading to NYC convention


Joseph Farah
WorldNetDaily.Com
June 9, 2004
1:00 a.m. Eastern


EXCERPT:


WASHINGTON There are whispers among high-level political advisers to President Bush suggesting the possibility of replacing Dick Cheney with former New York Mayor Rudolph Giuliani as the vice presidential running mate prior to the Republican National Convention in New York beginning Aug. 30.

Cheney, who has no aspirations to run for president in 2008 and has had well-publicized heart problems, has been involved in the discussions and is open to the idea if it strengthens the ticket and helps position a viable Republican candidate to succeed Bush, sources tell WND.

Originally, Cheney was asked by Bush to lead the effort to find the best running mate in 2000. After months of interviews and offering advice, Bush surprised many by picking Cheney.

Giuliani, as well as New York Gov. George Pataki, has been expected to play a starring role at the convention. Both are also considered possible presidential candidates in 2008. Securing the vice presidential nomination, however, would instantly make Giuliani the front-runner among all potential Republican candidates.

"There is some thinking at the very highest political levels that this move could add some late sizzle to the campaign, steal any thunder generated by the Democrats in Boston and even potentially put the state of New York in play for the president," said one source close to both Giuliani and the White House.

No one is talking on the record, and the plan is not yet set in stone.

So far, the only people who have speculated publicly about such a move are Democrats (and a seer of the future named Elign_Republicans).

Any comments (or desires to wager multiple chocolate doughnuts? - I'll really pay up this time if I'm wrong, honest, really).

tomder55 answered on 06/10/04:

I wouldn't mind it at all . Cheney is alot of baggage on the ticket. Charges that he is secretely running foreign affairs,his relations with the energy industry ,appearances of conflicts of interest due to his relationship with Halliburton are all things that the Dems. will try to exploit.

Still ,there is equal speculation that Rudy may be headed to the CIA (or FBI would be a better fit )http://www.nypost.com/news/nationalnews/22330.htm .

Heck Rudy to A.G would work too if Ashcroft gets too bloodied over the Interegation memos .But maybe Rudy will replace Tom Ridge at Homeland Security .
My preference would be for Rudy to stay local and replace either Chucky Shumer or Senator Evita .For VP ;I'd love to see it offered to Dr.Rice but I heard rumors that Powell is out ,and Rice will move to State .

For CIA, I'd like to see Frank Gaffney get the job ,but most likely Bush will go for one of 2 former CIA directors ;either R. James Woolsey,or Bush 41.

elgin_republicans rated this answer Excellent or Above Average Answer
ETWolverine rated this answer Excellent or Above Average Answer
stevehaddock rated this answer Excellent or Above Average Answer

Question/Answer
Itsdb asked on 06/09/04 - In case you missed it...

Sunday, June 06, 2004
"How Sad...

...that Ronald Reagan didn't die in prison, where he belonged for starting an illegal, laughably unjustifiable war against Grenada under false pretenses (the "besieged" medical students later said they were nothing of the sort) and funneling arms to hostages during Iran-Contra.

Oh, and 9/11? That was his. Osama bin Laden and his fellow Afghan "freedom fighters" got their funding, and nasty weapons, from Reagan.

A real piece of work, Reagan ruined the federal budget, trashed education, alienated our friends and allies and made us a laughing stock around the world.

Hmmmm...sounds familiar.

Anyway, I'm sure he's turning crispy brown right about now."

http://www.rall.com/rants.html

He's one to talk...

tomder55 answered on 06/09/04:

Ted Rall is an A**HOLE !

Itsdb rated this answer Excellent or Above Average Answer

Question/Answer
kindj asked on 06/09/04 - Ted Kennedy

I didn't write this, but I tend to agree with it. I figure the political name-calling and mud-slinging will be gearing up to full force pretty soon, so I figured I'd post this here and just deal with the heat.


"When Sen. Ted Kennedy was merely just another Democrat bloviating on Capitol Hill on behalf of liberal causes, it was perhaps excusable to ignore his deplorable past.

But now that he's become Sen. John Kerry's leading campaign attack dog, positioning himself as Washington's leading arbiter of truth and integrity, the days for such indulgence are now over.

It's time for the GOP to stand up and remind America why Sen. Kerry's chief spokesman had to abandon his own presidential bid in 1980 - time to say the words Mary Jo Kopechne out loud.

As is often the case, Republicans have deluded themselves into thinking that most Americans already know the story of how this "Conscience of the Democratic Party" left Miss Kopechne behind to die in the waters underneath the Edgartown Bridge in July 1969, after a night of drinking and partying with the young blonde campaign worker.

But most Americans under 40 have never heard that story, or details of how Kennedy swam to safety, then tried to get his cousin Joe Garghan to say he was behind the wheel.

Those young voters don't know how Miss Kopechne, trapped inside Kennedy's Oldsmobile, gasped for air until she finally died, while the Democrats' leading Iraq war critic rushed back to his compound to formulate the best alibi he could think of.

Neither does Generation X know how Kennedy was thrown out of Harvard on his ear 15 years earlier - for paying a fellow student to take his Spanish final.

As they listen to the Democrats' "Liberal Lion" accuse President Bush of "telling lie after lie after lie" to get America to go to war in Iraq, young voters don't know about that notorious 1991 Easter weekend in Palm Beach, when Uncle Teddy rounded up his nephews for a night on the town, an evening
that ended with one of them credibly accused of rape.

It's time for Republicans to state unabashedly that they will no longer "go along with the gag" when it comes to Uncle Ted's rants about deception and moral turpitude inside the Bush White House. And if the Republicans don't, let's do it ourselves by passing this forgotten disgrace around the Internet to wake
up memories of what a fraud and fake Teddy really is.

The Democratic Party, not to mention Sen. John Kerry, should be ashamed to have the national disgrace from Massachusetts as their spokesman.

And the GOP needs to say so out loud."

tomder55 answered on 06/09/04:

no one really takes the swimmer seriously anymore (except Kerry,and even there I think he is doing it out of courtesy).

ETWolverine rated this answer Excellent or Above Average Answer
excon rated this answer Excellent or Above Average Answer
kindj rated this answer Excellent or Above Average Answer

Question/Answer
Itsdb asked on 06/08/04 - Was it for oil?

Iraq controls oil industry
By Katarina Kratovac in Baghdad

June 09, 2004

IRAQI officials today said the interim government had assumed full control of the country's oil industry ahead of the June 30 handover of sovereignty from the US-led occupation administration.

"Today the most important natural resource has been returned to Iraqis to serve all Iraqis," Prime Minister Iyad Allawi said.

"I'm pleased to announce that full sovereignty and full control on oil industry has been handed over to the oil ministry today and to the new Iraqi government as of today."

The announcement came as Mr Allawi and Oil Minister Thamir Ghadbhan toured the al-Doura oil refinery in southern Baghdad.

After meeting and shaking hands with the refinery workers, the two ministers expressed their gratitude to oil sector workers.

"We are totally now in control, there are no more advisers," Mr Ghadbhan said.

"We are running the show, the oil policies will be implemented 100 percent by Iraqis."

Mr Allawi said the handover of the oil ministry before June 30 reflects "our full confidence in the oil minister. It's evidence that the oil ministry has worked perfectly".

Referring to the former regime of Saddam Hussein, Mr Allawi said that "in the past, Iraqi oil was used in building palaces, buying weapons to achieve one person's goals".

The new government will have full control of the oil revenues, Mr Allawi said, adding that Iraq is the "second largest country in oil reserves, it has more than 110 billions barrels of crude oil and about 100,000 trillion cubic meters of natural gas."

"We have to protect these precious natural resources," Mr Allawi said. A force has been established solely for the protection of the oil infrastructure, made up of about 14,000 guards.

"This will help protect the oil pipelines and eliminate oil smuggling, with the help of coalition forces and Iraqi allies," Mr Allawi also said. "By this, we shall serve all the Iraqis, instead of a small select group of criminals."

http://www.theaustralian.news.com.au/common/story_page/0,5744,9790322%255E1702,00.html

Was the war still about oil? I guess the bad news now is things are starting to look up for Iraq.

Steve

tomder55 answered on 06/09/04:

yes it was for oil . the opposition by France ,and Germany was because they were trying to protect their sweetheart contracts with the dictator.they also wanted the oil for corruption program to continue.that is also the reason for the opposition by the U.N. Blood for oil-is on the hands of the nations that opposed the invasion.

ETWolverine rated this answer Excellent or Above Average Answer
Itsdb rated this answer Excellent or Above Average Answer
purplewings rated this answer Excellent or Above Average Answer

Question/Answer
excon asked on 06/08/04 - Gade said it first Hes right on!


Hello pundits,

>>>maybe there is more about religion in this war on terror than we dare speak<<<

Gade said it. Hes right on.

The problem I have with Bush, is Im afraid he believes it too. You don't?

excon

tomder55 answered on 06/09/04:

It is the great unspoken reason . Bush went so far as to remove a whole sentence from Ikes address to the troops on D-Day when he quoted it last week because it used the word 'crusade'.

Western society, in spite of all the protestations to the contrary, is based on a Judeo-Christian worldview. An important aspect of this worldview is the use of persuasion, or argument, rather than force. Individual freedom is an accepted fact. Those instances we think of where force was used ie. Salem witch trials, inquisition are aberrations, which is why we condemn them.

Islamic society is based on force, rather than persuasion. Violence and force are the historical norm, not an aberration. The Islamic nations are almost exclusively dictatorships. Islamic attempts at reasoned argument often seem childish or irrational to us. Their history is one of centuries of warfare and forced conversions.

The crusades, which are often brought up , are another case in point. The crusades were a response to the violent spread of Islam across the known world. The fact that people in the West condemn them is just more evidence of the Judeo-Christian ethic that pervades our society. At the same time, no one in Islamic society condemns the violent Islamic conquests that led to the crusades. The idea of violent conquest is an integral part of the Islamic worldview.

The Judeo-Christian worldview lends itself to Democracy. The Islamic worldview does not. The choices are 1. Reject Democracy. 2. Reject Islam 3. Reform Islam - which Muslims see as equal to #2. No one needed to drop their Jewish heritage,or Christianity to accept Democracy.

What we are attempting there is a gamble that is well worth taking.It is impossible to keep America safe by just playing defense. The Middle East is the epicenter of the terror network. Despotic governments bring out the worst in religion.
Change the political environment in the Middle East and hopefully we change the associated religious pathology.If not ,then the only alternative left will be a clash of civilizations to the end.


excon rated this answer Excellent or Above Average Answer

Question/Answer
Itsdb asked on 06/08/04 - Any takers?

Anyone care to wager that Kerry either takes credit for or criticizes the UN Security Council resolution on Iraq that passed unanimously today?

tomder55 answered on 06/09/04:

it was a unanamous vote ,but Kerry will claim he could've done it better.

ETWolverine rated this answer Excellent or Above Average Answer
Itsdb rated this answer Excellent or Above Average Answer
purplewings rated this answer Excellent or Above Average Answer

Question/Answer
STONY asked on 06/08/04 - IT WAS SUGGESTED TO ME THAT I POST THIS UNDER POLITICS

STONY 06/07/04
SINCE MOST EVERYONE HERE REVEALS THEIR FEELINGS IN ONE WAY OR ANOTHER I THOUGHT THIS MAN'S WERE APPROPRIATE.................



APOLOGY




I heard some Arabs are asking for an apology for Iraqi prisoner abuse at Abu Gahreb prison. I humbly offer mine here:

-- I am sorry the last seven times Americans took up arms and sacrificed the blood of our youth it was in defense of Muslims (Bosnia, Kosovo, Sudan, Persian Gulf War, Kuwait, etc.).

-- I am sorry there was no such call for an apology from the Muslim extremists after 9/11.

-- I am sorry all of the murderers on 9/11 were Arabs.

-- I am sorry Arabs have to live in squalor under savage dictatorships.

-- I am sorry Arab leaders squander their oil wealth.

-- I am sorry Arab governments breed hate for the United States in their religious schools.

-- I am sorry Yasir Arafat was kicked out of every Arab country and
hijacked the Palestinian "cause."

-- I am sorry no other Arab country will take in the Palestinians or offer more than a token amount of financial help to those same Palestinians.

-- I am sorry the United States has to step in and be the biggest financial
supporter of poverty stricken Arabs while the insanely wealthy Arabs blame the United States for their plight.

-- I am sorry our own left wing elite and news media can't understand
any of this.

-- I am sorry the United Nations scammed the poor people of Iraq out of the "food for oil" money so they could get rich while the common folk
suffered.

-- I am sorry some Arab governments pay the families of homicide
bombers upon their deaths after they kill hundreds of innocent men, women and children. I am also sorry that those same bombers are overwhelmed with the thought of sex -- seeking to be rewarded with 72
virgins.

-- I am sorry homicide bombers think babies are a legitimate
target.

-- I am sorry our troops closed the rape rooms and the filling of
mass graves of Iraqi dissidents by Saddam Hussein.

-- I am sorry Muslim extremists have killed more Arabs than any other group.

--I am sorry foreign-trained terrorists are trying to seize control of Iraq and turn it into a terrorist state.

-- I am sorry we don't drop a few dozen Daisy-Cutters on Fallujah.

-- I am sorry when each time I see terrorists hide, they find a convenient "holy site."

-- I am sorry Muslim Extremists didn't apologize for driving jets into the World Trade Center towers that collapsed and destroyed St. Nicholas Greek Orthodox Church -- one of OUR holy sites.

-- I am sorry they didn't apologize for flight 93 and 175, the USS Cole,
the embassy bombings, etc.

-- I am sorry Michael Moore is American.

-- I am sorry the French dont remember who liberated them during World War II.

America will get past this latest absurdity. We will punish those
responsible because that's what we do. We hang out our dirty laundry for all the world to see and then move on. We don't hide this stuff like all countries that are now demanding an apology. Remember, thought that, until they were captured, we were trying to kill these same prisoners. Our compassion is tempered with the vivid memories of our own people killed, mutilated and burnt amongst a joyous crowd of celebrating Fallujans and the beheading of our citizens in the name of God.

NOW, LETS NOT BE SHY HERE EXPRES YOURSELVES!! JUST REMEMBER OPINIONS ARE LIKE EARS, EVERYBODY HAS THEM.

tomder55 answered on 06/08/04:

Earlier this spring in Phoenix, the American Islamic Forum for Democracy (AIFD) held a "Rally Against Terror" that gave moderate Muslims a platform on which to condemn terrorism and pledge support for the United States.

Identified by the Arizona Republic as "the nation's FIRST Muslim rally against terrorism,"(note this is 2004 ,almost 3 years since 9-11) the 50-minute event drew, between 250 and 400 people, most of them NON-MUSLIMS. Considering that the event was actively promoted within Phoenix's 50,000-strong Muslim community, that number is a bit disappointing. I am sorry that more of the so called moderate Muslims did not show up.

I am sorry that Bill Clinton is angry that he was not invited to speak at the Reagan funeral.

I am sorry that the members of the new Iraqi interm government hink that al qaeda and Saddam were linked .

I'm sorry that the ACLU is offended by the Calf. State Seal.

I'm sorry that Ontario is allowing Muslim clerics to administer Sharia law in what used to be a free country .

STONY rated this answer Excellent or Above Average Answer

Question/Answer
ETWolverine asked on 06/07/04 - Ronald Reagan's death and the 2004 Elections

I think that it is terrible in this time of national grief for the loss of one of our greatest presidents for politics to become linked to Reagan's name. However, I have heard at least one person say that they believe that Reagan's death will give Bush a bump in the polls that he's been looking for. They say that:

1) Inevitable comparison's between Reagan and Bush vs Reagan and Kerry will work in Bush's favor,

2) Republican's and moderate Democrats (the so-called Reagan Democrats) will come together, despite ideological differences, to remember Bush, and that connection may stay in place until after the election.

What is your opinion of these arguments?

Elliot

tomder55 answered on 06/07/04:

Sen. John Kerry's former presidential campaign manager said that he'd been "dreading" the prospect of Ronald Reagan's death because it was likely to give President Bush a major political boost.
"I've been dreading this every election year for three cycles," said Jim Jordan, who resigned from the Kerry campaign last year, in comments to the New York Times. "Bush has totally attached himself to Ronald Reagan. He's going to turn Reagan into his own verifier." Kerry campaign aides said they hoped the political impact of Reagan's death would fade by the summer

But the Times also pointed out that Some Republicans said the images of a forceful Mr. Reagan giving dramatic speeches on television provided a less-than-welcome contrast with Mr. Bush's own appearances these days, and that it was not in Mr. Bush's interest to encourage such comparisons. That concern was illustrated on Sunday, one Republican said, by televised images of Mr. Reagan's riveting speech in Normandy commemorating D-Day in 1984, followed by Mr. Bush's address at a similar ceremony on Sunday.
"Reagan showed what high stature that a president can have and my fear is that Bush will look diminished by comparison," said one Republican sympathetic to Mr. Bush, who did not want to be quoted by name criticizing the president.
Another senior Republican expressed concern that by identifying too closely with Mr. Reagan, Mr. Bush risked running a campaign that looked to the past, which this adviser described as a recipe for a loss.
Kerry announced that he would suspend campaigning this week to honor Reagan . However some of the more cynical among us might suggest that he could care less about honoring Reagan. He knew that between Reagan's death and his funeral, the upcoming G8 summit he would be no where in the picture anyway.

While watching some of the tributes yesterday ,my wife pointed out that it was too bad that we do not have a President now who could communicate and define what is at stake in the war on terror the way Reagan was able to. She had a good point .Frequently I have to go to a text download of Bush's speeches,and interviews to be able to determine if indeed he was on message. Most people would not go to such lengths, and as a result I believe most folks do not realize what is at stake . I am reminded that during his term Reagan faced some of the same criticism about forward deployment of Pershing II Cruise Missles to counter the Soviet SS-20s. He was a cowboy,he was plunging the world into WWIII .He risked would not merely destroy the Atlantic Alliance but risked Armageddon . He committed to his strategy ,and did not waver . While doing this he used the 'bully pulpit ' to accurately define the threat that the Soviets were. He called them the Evil Empire to their face . He refused to tolerate a notion of a moral equivalence between them and us.At home and abroad ,he used the power of the media to get his message across ,not only to the 'choir',but to his opponents ,and more importantly to the people of the world that the Soviets enslaved .His war of ideas gave hope and inspiration to subject peoples the world over and demoralized their Communist oppressors.

If only Bush would follow Reagan's example .Islamofacism is as much a threat to the world as international Stalinism was .Too frequently Bush succumbs to what is politically correct . Last week at the Air Force academy he dropped an entire sentence from Eisenhower's address to the troops before D-Day ,because it had the word 'crusade' in it .
with the sentence it was this :

"Soldiers, Sailors and Airmen of the Allied Expeditionary Force! You are about to embark upon the Great Crusade, toward which we have striven these many months. The eyes of the world are upon you. The hope and prayers of liberty-loving people everywhere march with you. In company with our brave Allies and brothers-in-arms on other Fronts, you will bring about the destruction of the German war machine, the elimination of Nazi tyranny over the oppressed people of Europe, and security for ourselves in a free world."

Possibly ,his speech writers editied it ,but he get's no pass on that .Reagan's speech writers begged him to bag "Mr. Gorbachev, tear down that wall!". His omission illustrates his weakness ;the ability to communicate with clarity; his vision ;his convictions .


ETWolverine rated this answer Excellent or Above Average Answer
purplewings rated this answer Excellent or Above Average Answer
excon rated this answer Excellent or Above Average Answer

Question/Answer
Itsdb asked on 06/05/04 - Another UN followup

BY MICHELLE MALKIN

RELEASE: WEDNESDAY, JUNE 2, 2004

THE AMBULANCES-FOR-TERRORISTS SCANDAL

The United Nations and Red Cross have been providing cover for terrorists -- literally. And American taxpayers are footing some of the bill.

Last week, an Israeli television station aired footage of armed Arab terrorists in southern Gaza using an ambulance owned and operated by the United Nations Relief and Works Agency for Palestine Refugees (UNRWA). Palestinian gunmen used the UNRWA emergency vehicle as getaway transportation after murdering six Israeli soldiers in Gaza City on May 11. The footage shows two ambulances with flashing lights pull onto a street. Shots and shouts ring out during the nighttime raid. A gang of militants piles into one of the supposedly neutral ambulances, clearly marked "U.N." with the agency's blue flag flying from the roof, which then speeds away from the scene.

AccessMiddleEast.org, a nonprofit global news monitoring service, posted the video (shot by a Reuters TV cameraman) on its Web site last week. (It's available at e.tln0.com/ame/archives/ reuters_UN_amblulances_11_may_04.wmv). To date, Access Middle East managing director Richard Bardenstein in Israel informs me, not a single U.S. television news station has expressed interest in showing the footage to American viewers.

Why should we care? Because since 1950, the U.S. has provided UNRWA with $2.5 billion in taxpayer subsidies -- about one-third of the relief agency's total budget. And because instead of investigating this latest black eye-inducing scandal, the U.N. is blasting American troops for defending themselves against such outrageous tactics -- now being emulated by Iraqi guerrilla warriors sniping at our men and women from ambulances in Fallujah.

International relief officials are in stubborn denial about the abuse of their emergency vehicles and hospital credentials by terrorists. They claim the videotaped May 11 ambulance-assisted attack was an isolated incident and that the driver was forced to transport the gunmen. But this ambulances-for-terrorists program has been going on for years. And "humanitarian" workers have been willing collaborators.

According to the Intelligence and Terrorism Information Center at the Center for Special Studies (CSS), senior UNRWA employee Nahed Rashid Ahmed Attalah confessed to using his official U.N. vehicle to bypass security and smuggle arms, explosives, and terrorists to and from attacks. He was in charge of distributing food supplies to Palestinian refugees. Nidal 'Abd al-Fataah 'Abdallah Nizal, a Hamas activist, worked as an UNRWA ambulance driver and admitted he had used an emergency vehicle to transport munitions to terrorists.

U.N. vehicles aren't the only ones being used by terrorists. An intensive care ambulance carrying the acronym of the Palestine Red Crescent Society (PRCS) was used to deliver an explosive belt found underneath a stretcher on which a sick child was lying in spring 2002. Female suicide bomber Wafa Idris, who blew herself up in a January 2002 attack in Jerusalem, was a medical secretary for the PRCS. Her recruiter was an ambulance driver for the same organization. PRCS receives financial support from governments and organizations around the world, including the American Red Cross and International Committee of the Red Cross.

The UNRWA has long been suspected of providing aid and comfort to terrorists. Rep. Eric Cantor, R-Va., chairman of the Congressional Task Force on Terrorism and Unconventional Warfare, recently documented how "buildings and warehouses under UNRWA supervision are allegedly being used as storage areas for Palestinian ammunition and counterfeit currency factories." Cantor's 2002 report also noted that UNRWA hosts summer camps in martyrdom for young terrorists-in-training. Rep. Chris Smith, R-N.J., has also lobbied for increased scrutiny of UNRWA funding, which has been used to publish anti-Semitic textbooks and posters in schools that "glorify homicide bombers and the slaughter of innocents."

Moreover, according to Rep. Smith, a UNRWA school hosted a Hamas rally by a key Hamas leader in July 2001 and another UNRWA employee praised homicide bombers, proclaiming: "The road to Palestine passes through the blood of the fallen, and these fallen have written history with parts of their flesh and their bodies."

While jihadists gain shelter in its emergency vehicles, the U.N. continues to lambaste the U.S. for assorted wartime "atrocities." Not one more American dime should go to fund the bloody self-righteousness of the world's most generous terrorist relief organization.

Michelle Malkin is author of "Invasion: How America Still Welcomes Terrorists, Criminals, and Other Foreign Menaces to Our Shores" (Regnery). Michelle Malkin's e-mail address is malkin@comcast.net.

COPYRIGHT 2004 CREATORS SYNDICATE, INC.

http://www.creators.com/opinion_show.cfm?columnsName=mim

tomder55 answered on 06/06/04:

Last month it was reported that they were using UN ambulances to steal and transport parts of the bodies of five IDF soldiers killed in an ambush along the Gaza-Egypt border .But the world body has for the most part turned a blind eye to such abuses in the past.
Israel has over the past decade produced evidence that UNWRA facilities and vehicles throughout Judea, Samaria and the Gaza Strip are regularly used by and provide haven to terrorist killers.

A similar trend exists along Israel's northern border, where the southern-Lebanon based UNIFIL is supposed to prevent cross-border attacks.

Hezbollah attacks on Israel are carried out with impunity, and have even been launched from inside UNIFIL compounds.

When three IDF soldiers were attacked and kidnapped in October 2000, "peacekeepers" stood by and watched.

Itsdb rated this answer Excellent or Above Average Answer

Question/Answer
Itsdb asked on 06/04/04 - Air America ratings double

Written by Charlie Van Horn

(BURBANK, California)--Air America, the liberal leaning radio network has announced that the ratings have doubled since its inception earlier this year.

Ned Willkins, 31, of Bridgeport, Connecticut became the second listener of the "O'Franken Factor'.

"I just turned it on when I was in my garage working on my lawnmower, and it really felt like Al was talking just to me" Willkins said Sunday.

According to Arbitron, the company responsible for radio ratings, Ned was correct. He was the only person tuned in to the broadcast.

Air America's first listener, Janine Garofalos' niece, Ashley Teegan Reynolds, says she only listens when grandpa is on with her aunt. Otherwise, she says, "she just bitches too much"

Al Franken, who was in one scene of the 1983 movie "Trading Spaces", was rather upset by this notion that nobody listens to his show.

"This is just another right-wing fascist conspiracy put out by the radical Christian right in an effort to target me for the ovens that Dubya is constructing to conceal his hate crimes against this and many other planets" Franken said in a statement released by his publicist, who also frankly admitted to not listening to his show either.

Air America is currently heard on almost thirty radio stations, mostly small teeny tiny AM ones, and in markets where they get their ass handed to them every quarter. You can also listen to them on the internet, but who really wants to do that.

((UPDATE--Al Franken to tackle me in a public place in 5, 4, 3......))

The story as represented above is written as a satire or parody. It is fictitious.

http://www.thespoof.com/news/spoof.cfm?headline=s8i4162


tomder55 answered on 06/05/04:

Amazing ,but not unexpected;when Air America went on ,it was trumpeted in the major press as the antidote to conservative talk radio .Now as it tanks you barely hear a peep about it.

Already short Al is working without pay .Chicago Tribune quoted an inside source as saying that Chicago staffers were never enrolled in a health insurance plan, even though Air America promised them coverage and deducted health insurance premiums from their paychecks.

No one in the establishment media picked up on the story , even though it happened in the same week that government lobbyists were promoting something called "Cover the Uninsured Week."

But with George Soros's deep pockets ,I am sure this venture will last at least until election day.

BTW ,don't ask the heads at Air America to take the blame for their troubles . They found a scape goat;the vast Right wing conspiracy. They fired CEO Evan Cohen for 'mismanagement' ,but as Reuters reported ""Papantonio and others attributed the company's troubles thus far to mismanagement by Cohen, a former Republican political operative in Guam."


Itsdb rated this answer Excellent or Above Average Answer
XCHOUX rated this answer Excellent or Above Average Answer

Question/Answer
XCHOUX asked on 06/03/04 - Kofi Under Attack Re: Sudan

I think we may see Kofi Anan(sp)forced out of his cushy job at the United Nations over the scandal of the Sudan. On television last night, O'Reilly revealed in an interview with an African American congressperson that the reason Kofi has not used the United Nations to stop the genocide in the Sudan is that he is supported by Islamic countries who want the extermination to continue for religious reasons. As a black man he has sold out these African people for the pleasure of having his JOB!!!

Please add to the information.

tomder55 answered on 06/03/04:

If youre black in Darfur Province in Sudanan and you werent murdered, you may have simply had your village burned to the ground. If you escaped the murderous Muslim Khartoum regime and their militias death sentence, you may have escaped into Chad. Or worse, you may have ended up in what many are calling concentration camps .According to the African Society of International and Comparative Law, which testified to the UN Commission on Human Rights on April 19 in Geneva; Two such camps were the Al-Intifada Camp in South Nyala, with approximately 9,000 prisoners, and the Belail Camp in East Nyala, with approximately 11,000 prisoners. There are said to be hundreds of thousands of other prisoners in camps throughout Darfur. The National Islamic Government of Sudan denies this, but as they are either denying or limiting access into Darfur to humanitarian and monitoring groups, it is impossible to verify the situation in many locations. There is a great fear that these camps could be become extermination sites to prevent the inmates from being witnesses at a later date to what they have seen and heard. There are also reports of a lack of food, water, and adequate hygienic facilities.


Kofi Annan has had his plate full of course, in trying to cover up the UN Oil-for-Food scandal And with the current ten-year anniversary of the Rwandan Genocide, he has been obliged to address and make speeches about genocide.

Before a Human Rights Commission Memorial marking the anniversary remembering the 800,000 dead in Rwanda (when he was head of the United Nations Peacekeeping Agency), in which the UN famously stood idly by, he said , referring to the Sudan, the International Community cannot stand idle, even as he does just that.

First, he said that military forces might be needed in Darfur Province to stop ethnic cleansing. He then explained further: by action in such situations, I mean a continuum of steps which may include military action. And then he added, The risk of genocide remains frighteningly real He will not, against all evidence, actually admit genocide is taking place, because even he cannot stand idle in the face of such atrocity .Article 2, Clause (c) of the United Nations Convention of the Prevention and Punishment of the Crime of Genocide, 1948, states that genocide includes not only killing members of a racial or ethnic group, in whole or in part, but deliberately inflicting on the group conditions of life calculated to bring about its physical destruction in whole or in part. It takes an enormous level of denial to ignore that that this is what is taking place. .So he contents himself with talking generalities about this reality , as if it were merely an idea and not a terrible actuality affecting the lives of hundreds of thousands of people in Sudan.

Under his stewardship, the UN can talk of ethnic cleansing as being problematic, but not bad enough to demand serious action or the risk of genocide, even more problematic, but still not bad enough to demand the most serious response. Of course, they form all sorts of committees, make all sorts of studies, issue all sorts of declarations, but whatever he says, Annan cannot use the term genocide , such as Genocide is taking place. That simple statement will either force the UN to admit its impotence, or force it to act with military force.

It is estimated that 1000 blacks are being murdered each week there. In ten years time no doubt, Annan will be living on a large UN pensions, tucked away at some bureaucratic well-known peace institute, having by then worked out a few answers absolving himself of any responsibility for the first genocide of the Twenty First Century.



purplewings rated this answer Excellent or Above Average Answer
XCHOUX rated this answer Excellent or Above Average Answer

Question/Answer
XCHOUX asked on 05/31/04 - Holiday Fantasy Question

If a spaceship landed on earth and aliens assessed the blow-'em-up violence everywhere, Africa MiddleEast what would the space travelers propose to the countries of earth so they could end the horror? What would each country contribute to the peace?

tomder55 answered on 06/02/04:

"Gort, Klaatu barada nikto" ;Already been done(sorta) : The Day the Earth Stood Still,(1951)


`disarm or we'll remove your threat by destroying you' hmmmmm maybe Bush is Gort

ETWolverine rated this answer Excellent or Above Average Answer
XCHOUX rated this answer Excellent or Above Average Answer

Question/Answer
Petesharky asked on 06/01/04 - Iraqi Government

Ok Bush Announces The new Iraqi Government so which members will be first assassinated?

Place your bets here.

tomder55 answered on 06/02/04:

If I was to play this game ,I would put my money on Barham Saleh, deputy prime minister for national security affairs;a Kurd.
This is the most important appointment. It legitimizes the Kurds who already have a functioning military branch to come out of the North and help restore order in the rest of the country. This will help free up US troops to seal off the borders (and to prepare for the next phase of military operations ?);The Kurds are doing it already to some extent on the Iranian Border,[agents of the Iranians will probably try to off him ].Yesterday a car bomb blew up outside the offices of a Kurdish political party in central Baghdad, killing at least three people and wounding 20.

Petesharky rated this answer Excellent or Above Average Answer

Question/Answer
Petesharky asked on 06/01/04 - Iraqi Government

Ok Bush Announces The new Iraqi Government so which members will be first assassinated?

Place your bets here.

tomder55 answered on 06/01/04:

Terrorists are going to do their utmost to sabotage Iraq's new government. Especially since the US election hinges on it as well. All Iraqi leaders should fear assassination by the death cultists. Assassination and explosives are the best friend the Jihadist psychos ever found(next to CNN and the New York Times). Can you imagine what these assassins would do with a few nukes at hand? Islamic war doctrine is all about cowardly killings via assassination. Explosives magnify the impact of assassinations that were once done with just a dagger and could kill only one at a time.



First some facts :

Here is the list of Iraqi ministries and the Cabinet members who will lead them, announced Tuesday by the prime minister-designate, Iyad Allawi(Americans favored former Foreign Minister Adnan Pachachi as head of state)Pachachi inspires respect, but [al-Yawer] represents the future, said a Kurdish council representative, Mahmud Osmn

Allawi, a former dissident member of Saddam's Baath Party was appointed on Friday by the IGC ,bypassing Brahimi.Brahimi was brought in by the United States to mediate between Washington and Iraq's various ethnic, religious and political groupings. But the Governing Council had sought to push its own candidates on the U.N. envoy.

U.S. and U.N. officials have said in the past that the Governing Council did not have the right to make appointments on its own. It caught Brahimi off guard on Friday by one of their number, Allawi, to the top job of prime minister. Many of the other nominations from the Council have come from among their own ranks.

(Brahimi seems to be increasingly irrelevent to the events ;which is fine with me). Allawi thanked the U.S for freeing Iraq ;(that must make him another U.S. puppet)something that was reported on Fox News ,and no network source .

Here is the cabinet announced today
(I hope Bush didn't attempt to read these names at his speech this morning.):


Deputy Prime Minister for National Security Barham Saleh


Oil Thamir Ghadbhan


Defense Hazem Shalan al-Khuzaei


Interior Falah Hassan al-Naqib


Justice Malik Dohan al-Hassan


Human Rights Bakhityar Amin


Electricity Ayham al-Samarie


Finance Adil Abdel-Mahdi


Health Alaa Alwan


Communication Mohammed Ali Hakim


Housing Omar Farouk


Public Works Nesreen Mustafa Berwari


Science and Technology Rashad Mandan Omar


Planning Mahdi al-Hafidh


Trade Mohammed al-Joubri


Sport and Youth Ali Faik Alghaban


Transportation Louei Hatim Sultan al-Aris


Provincial Affairs Waeil Abdel-Latif


Women's Affairs Nermin Othman


Immigration and Refugees_ Bascal Essue


Irrigation Abdul-Latif Rasheed


Labor Leila Abdul-Latif


Education Sami Mudahfar


Higher Education Tahir al-Bakaa


Agriculture Sawsan Sherif


Culture Mufeed al-Jazaeri


Industry Hajim al-Hassani


Three ministers of state Qassim Dawoud, Mahmoud Farhad Othman, Adnan al-Janabi

The IGC in office since July, has voted to dissolve effective immediately to allow the new government to begin taking over its responsibilities.


A Look at Iraq's New Interim Government





Details on key members of Iraq's new interim government.


_Prime Minister Iyad Allawi: Shiite Muslim with military connections. His power base, the Iraqi National Accord, made up largely of former members of Saddam Hussein's Baath party and former military men, stresses secularism and counts Sunnis and Shiites among its members.


_President Ghazi Mashal Ajil al-Yawer: Prominent Sunni member of the Shammar tribe, which includes Shiite clans and is one of the largest tribes in the Gulf region. The presidency is a largely ceremonial post.


_Vice President Ibrahim al-Jaafari: A leader of the Shiite Muslim Dawa Islamic Party.


_Vice President Rowsch Shaways: Parliament speaker in the Kurdish autonomous region in Irbil and member of Kurdistan Democratic Party, one of two rival parties running northern Iraq.


_Foreign Minister Hoshyar Zebari: Since his appointment in July as foreign minister of Iraq's interim government, the 51-year-old Zebari has traveled worldwide to canvass support for what he calls a new, united and democratic Iraq. Iraq's first Kurdish foreign minister was a guerrilla fighter during Kurdish rebellions against Saddam.


_Deputy Prime Minister for National Security Affairs Barham Saleh: A leader of one of the main Iraqi Kurdish groups, the Patriotic Union of Kurdistan.


_Interior Minister Falah Hassan: Provincial official in Tikrit, Saddam's home region. Son of Gen. Hassan al-Naqib, a former deputy chief of staff under Saddam who defected in the late 1970s and became active in the exiled opposition.


_Finance Minister Adil Abdel-Mahdi: French-educated son of a respected Shiite cleric who was a minister in Iraq's monarchy. Official of the powerful Shiite Supreme Council for the Islamic Revolution in Iraq.



_Oil Minister Thamir Ghadbhan: Has been directing oil matters for months.


_Justice Minister Malik Dohan al-Hassan: As elected president of Iraq's Lawyers League after Saddam's fall, had lodged early protests about the conditions under which the U.S.-led occupation administration was holding prisoners and about the prisoners' lack of legal defense. Had been a political prisoner under Saddam. Was culture minister in the mid-1960s.

This should shut up the 'Who are we going to hand over power to on July 1 ' parrots of the left .

If it were me ,my first act as PM would be to offer a bounty on the head of all insurgents in Iraq both foreign and domestic ,to be paid for from oil revenues. Then I would announce that as long as I am in power every Iraqi will get a share in the oil revenues of the nation ;simular to what Alaska does .

elgin_republicans rated this answer Excellent or Above Average Answer
Petesharky rated this answer Excellent or Above Average Answer
XCHOUX rated this answer Excellent or Above Average Answer

Question/Answer
XCHOUX asked on 05/29/04 - Uprising in Saudi Arabia

There is a terrorist uprising starting today in Saudi Arabia. This is the kind of *face* of terrorosts and their activities that Americans and the world needs to see. The reality of the methods and true goals of evil Islamic death cults. Guarantees Bush's reelection, imo.

Comments.....

tomder55 answered on 05/30/04:

The militants killed 16 people when they attacked the compound on Saturday.
Saudi commandos today rescued most of some 50 foreigners held hostage by the militants.
This is a good strategy by the insurgency.If expatriots are driven out of Saudi ,then the oil industry ,and thus Saudi economy collapses;and it is also an attack on'American Interests ' .The good news is that Saudi justice system knows how to deal with them without a whole lot of introspective hand wringing.

ETWolverine rated this answer Excellent or Above Average Answer
purplewings rated this answer Excellent or Above Average Answer
XCHOUX rated this answer Excellent or Above Average Answer

Question/Answer
XCHOUX asked on 05/29/04 - Dedication of WWII Memorial Today

The dedication of the WWII Memorial is on television today, this Memorial Day Weekend.

Anyone care to make a tribute to the Greatest Generation who fought the war?

tomder55 answered on 05/30/04:

In the late 1800's and early 1920's millions of immigrants gazed in awe at Lady Liberty and were processed through Ellis Island. Since many of these immigrants were not educated in academics or the basic civics in a free democracy, two coping mechanisms shifted into motion by far-thinking men. They heeded Teddy Roosevelt's wisdom: "The one absolutely certain way of bringing this nation to ruin, or preventing all possibility of its continuing as a nation at all, would be to permit it to become a tangle of squabbling nationalities."

First, our nation's leaders took a pragmatic assessment of how many such newcomers we could absorb without damaging our own economic and civic culture. They placed a cap on the number of immigrants. After 1930, the numbers steadily decreased to a low of 100,000 to an average of 178,000 . This allowed assimilation into the American Dream. The English language was taught as the cohesive glue to hold all immigrants together.

Second, leading educators of the day, fully understood the delicate language/cultural/civic balance essential to a Democracy. They devised a curriculum to "Americanize" the newcomers and their children. Those children went on to become known as "The Greatest Generation."

In his essential book published in 2000, "Bowling Alone: The Collapse and Revival of American Community," Harvard political scientist Robert Putnam gives the Greatest Generation another name. He calls it "the long civic generation." Its members were a remarkable collection of democratic citizens.

Members of the generation born in the 1920s, Putnam writes, "belong to almost twice as many civic associations as do members of the generation of their grandchildren born in the late 1960s." The grandparents, Putnam continues, "are more than twice as likely to trust other people as the grandchildren are."

They "vote at nearly double the rate of the most recent cohorts." They "are nearly twice as interested in politics" and "nearly twice as likely to attend church regularly." They are "twice as likely to work on a community project" and "are the last rabid newshounds: they are almost three times as likely to read a daily newspaper."

Of course they were civic, because they lived in a time when public action was seen to have worked to overcome the Depression and win the war. There was a draft, yet much was accomplished during World War II not by compulsion but by the voluntary efforts of communities that saw their own interests linked to a larger interest.

During the war ,America faced a severe rubber shortage .President Roosevelt asked the public to turn in "old tires, old rubber raincoats, old garden hose,rubber shoes, bathing caps, gloves whatever you have that is made of rubber." The result? "Literally millions of Americans responded to the president's appeal," Putnam writes, "and in less than four weeks roughly 400,000 tons of scrap rubber six pounds for every man, woman and child in the country (or at the front) were collected."

The leaders of todays so-called establishment did not have to listen in a classroom lecture or make a field trip to learn about poverty, We lived it in the depths of the Great Depression. The horrors of war are not just a subject for a term paper to a generation that sent its finest young men to fight at Omaha Beach. Ronald Reagan

ETWolverine rated this answer Excellent or Above Average Answer
XCHOUX rated this answer Excellent or Above Average Answer

Question/Answer
XCHOUX asked on 05/28/04 - Reality in America

I saw the statistics about incarceration in the United States. One in Seventy-Five males are in prison. This problem must be dealt with immediately!

It's unbelieveable isn't it?

tomder55 answered on 05/29/04:

Whether thats good or bad depends on whom is asked.Attorney General John Ashcroft said the report shows the success of efforts
to take hard-core criminals off the streets.
"It is no accident that violent crime is at a 30-year low while prison population is
up," Ashcroft said. "Violent and recidivist criminals are getting tough sentences
while law-abiding Americans are enjoying unprecedented safety".
Unfortunately I detect a trend in the United States of moving away from stricter punishment . Much of this is budget driven: the budgetary crises in various states have demanded substantial budget cuts and Corrections Departments are a politically easy places to cut. Many of us seem to think cutting prison budgets means we coddle prisoners less. But many of the expenses of incarceration are required by federal laws and regulations. The real effect is not less comfortable prisons but fewer prisoners.The result will be more crime. That,is inescapable.

ETWolverine rated this answer Excellent or Above Average Answer
excon rated this answer Excellent or Above Average Answer
XCHOUX rated this answer Excellent or Above Average Answer

Question/Answer
excon asked on 05/28/04 - Good cop - Bad cop


Hello sperts:

Please pardon me my jailhouse analogies, but hell, it's how I relate.

Al Gore is saying things the Democrats wish John Kerry would say. Is that by design? Are the dems playing good cop - bad cop?

Why isn't Kerry saying that stuff? Is it that he doesn't believe it, or is he pandering to the Republican wing of the Democratic Party?

excon

tomder55 answered on 05/28/04:

trust me ,Kerry is being very smart keeping above the fray ,and not stooping to the rediculous levels of Gore ,and Teddy Kennedy .Actually Gore is playing what has been the traditional role of a VP ala Agnew,attack ,attack ,attack.He looks as one pundit said like he is off his lithium again.

From The American Spectator today :
"On Wednesday, senior campaign staffers for Kerry were laughing at the draft text of former vice president Al Gore's remarks when he sent them to Kerry as a courtesy.

Gore had shared them, perhaps believing his rant against the Bush Administration would be viewed favorably.

"It was like "Oh, my God, what medication is he on," says a political consultant who has been doing work for Kerry and formerly did work for Gore. "Then the Kerry folks realized the speech would actually help them. After Gore, anything Kerry said would seem downright substantive and moderate."

http://www.spectator.org/dsp_article.asp?art_id=6621

Meanwhile ;while the attack dogs bark non-sense Kerry is free to make policy speeches like he did yesterday ;outlining a 4 point plan on National Security .

1.strenthen alliances (old news ,we've heard that before ). it is doubtful that a Kerry Adm. would be successful in changing the world view that America is the arrogant super power . Bottom line is that even under Kerry ,America would have to make 'unilateral ' decisions . It would be just like Bush ;'this is what we will do;hop aboard the train if your inclined to;maybe we'll make it worth your while'.

2.revamp the military to meet new challenges (which is a good idea ,but not new .Rummy has already initiated that )

3.more reliance on public diplomacy :"beyond military power, we must deploy all the power in America's arsenal. A strategy that invokes our non-military strength early enough and effectively enough so military force doesn't become our only option." Kerry is personalizing anti-American attitudes and lack of cooperation from other countries as anti-Bush.Perhaps the force of his personality can turn nations like France ?

4.end American dependence on foreign oil.I am in agreement with this goal also ,but that is a long term policy that neither party has done enough about .


text

ETWolverine rated this answer Excellent or Above Average Answer
excon rated this answer Excellent or Above Average Answer
XCHOUX rated this answer Excellent or Above Average Answer

Question/Answer
excon asked on 05/25/04 - Un-hypocritize yourself.

Hello drug warriors:

I dont believe pot should be illegal, but as long as it is, people should be prosecuted.

If youve ever uttered a phrase similar to the one above, I call upon you to free yourself from the yolk of hypocrisy.

Stand up for what you believe. Do something about it. Write, speak and most importantly, dont vote for those whose opinion goes against yours. You are right they are wrong. This country wasnt forged by people who went along. It was forged by men who walked the walk. Dont lay down for them.

Imagine where we would be if we never changed laws that were wrong, or are you waiting for somebody else to fix it?

Of course, if you think people who deal and use marijuana should be prosecuted, then never mind.

excon

tomder55 answered on 05/26/04:

having survived my years of tokin .I have to say no way should marijuana be legal unless it is prescribed for specific medical purposes. Had pot been legal. I most certainly would've been completely out of control .Lets be real here. How many smokers can honestly say that they NEVER drove a vehicle while impaired by the drug?

excon rated this answer Excellent or Above Average Answer
Itsdb rated this answer Excellent or Above Average Answer
purplewings rated this answer Excellent or Above Average Answer

Question/Answer
XCHOUX asked on 05/23/04 - Peace (Pt 2)

The Arab leaders stated that they want to keep their culture and their values intact; meaning, no more Western influence in their sphere. However, I believe it is too late for the Islamic cultures and countries. Women have access to the internet and will not stand forever being second class humans virtual servants in their cultures. Young women will realize that a young lady who dies young has to look forward to this in "paradise": being a sexual slave to an arrogant man for eternity.

Finally, I an encouraged because the second to last time I was in my local library, I saw a young Muslim woman(country or origin unknown-she may have been Arab, Persian, Afghani, Indones, Pakistani, etc)come over to the computer section and sit across from me. She looked around and then signed on and had a merry old time!

tomder55 answered on 05/24/04:

We in the Western often praise moderate Islam. But in practice, moderate Islam often turns out to be moderate in its actions only. As decent human beings, moderate Muslims will of course refrain from committing acts of oppression, cruelty, and terrorism. But intellectually, moderate Muslims have a difficult time explaining why those acts are un-Islamic. Take for example the 'moderate' cleric, Sheikh Yousef Al-Qaradhawi.He said this after last years terrorist attacks in Saudi and Morrocco:


Therefore, I say that there is no use in operations of this type, because they kill peace-loving, innocent people. Not everyone who was killed in Riyadh was American and not everyone who was killed in Casablanca was American or foreign. Not every foreigner deserves to be killed. Killing has specific conditions. There are people whom we call under Muslim protection who have entered our country. They must not be harmed and their blood must not be spilled. The brothers harmed, among others, a Belgian club, even though Belgium's opinion was good it opposed the war on Iraq and wanted to try Sharon and some American officers (http://www.memri.de/uebersetzungen_analysen/themen/islamistische_ideologie/isl_qaradhawi_30_06_03.html)

He rejected the actions because he was concerned for the death of innocent Muslims ,but not the reason behind the action.What Westerners really want is a liberal /secular Islam ; one that accepts peace and tolerance on principle .A liberal Islam, would pose a very serious challenge to the structure of Islamic thought and practice.A liberal Islam would have to challenge the system. It would have to begin by examining the Koran itself . Where did this book truly come from? How was it assembled? What do we genuinely know historically about Muhammad ?
Such questioning advanced the West beyond the dark ages.

XCHOUX rated this answer Excellent or Above Average Answer

Question/Answer
XCHOUX asked on 05/23/04 - Bush Adm. Wanted Chalabi Out

I watched part of Meet the Press this mornng in which William Safire gave his viewpoint on why our man Chalabi is being forced out by lies and innuendo. The Bush administration wants Iraq to be settled down before the election in November. He has made behind the scenes deals so that Arabs will be the ruling government of Iraq, no Kurds represented. The Administration, The State Department and the CIA have been in serious disagreement about how we should proceed with the War.

So, Bush is settling the matter so he will have a chance to win the election in November.

Do you think this is a far assessment?

tomder55 answered on 05/24/04:

He was convicted in absentia in a kangaroo court. No proof has ever been presented in public. Chalabi is a member of the Iraqi Governing Council ;the quasi-ruling body of the State.It doesnt surprise me that Chalabi ,and other IGC members kept up a regular dialogue with the Iranians .It only makes sense to have talks and meetings with a powerful neighbor on your border .Colin Powell has a dialogue on-going with them .Does that make him a spy also ?

But Chalabi's arrest has also been linked to the Oil for Food scandal, which is centered around Kofi Annan, the very man Lakhdar Brahimi represents, and who is "examining the quality of individuals"(see below)being put forward to lead the Iraqi interim government.

There are NO Shia Arab Governments (The Iranians are Persian). The political system in the Arab world since the Ottoman Empire has been: Sunnis on top, Shias on the bottom getting crapped on. An Iraq run by Shias is a threat to the stability to neighboring Sunni regimes. So ask yourself: Why is Brahimi a former spokesman for the Arab league, and Pat Lang, the man who's the source for virtually every Chalabi hit piece out in the press leading the charge to knock down a secular Shia???

The UN's man in Iraq is Brahimi ,and he and Chalabi have had verbal squabbles in the public for quite some time. Brahimi is a Sunni and will soon be related to the Jordanian Royal family. Brahimi's daughter is engaged to a Jordanian Prince, and the Jordanian government HATES Chalabi. Brahimi is a longtime spokesman for the Arab League.

Pat Lang, is a former DIA Middle East analyst. In citing Lang as an expert on Iraq, few media sources(lately CBS and the Washington Post) ever has mentioned that Lang has registered with the Justice Department as a foreign agent for an Arab government. How can somebody working for an Arab government parade about as a neutral analyst?

Ahmad Chalabi is in possession of tons of documents with the potential to expose politicians, corporations and the United Nations as having taken bribes ,kickbacks and false pricing of oil worth billions. That may have been enough to provoke the raid. So explosive are the contents of the files that their publication would cause serious problems for US allies and states around the world who we are counting on to provide a U.N. based assistance to the post war period.

Late last year and several months before the CPA became involved, Chalabi had amassed enough information concerning corruption in the oil-for-food scandal to realize that he was sitting on explosive material. It was information that would lead to the publication in a Baghdad newspaper of a list of 270 businessmen, politicians and corporations, that were alleged to have received kickbacks from Saddam's regime.
The Boston Globe hinted at the kind of high level horse trading already taking place:

WASHINGTON -- In a sign that a plan for the new Iraqi interim government is beginning to take shape, Italian Prime Minister Silvio Berlusconi told the Italian parliament yesterday that a candidate has been offered a top position in the new Iraqi interim government, but has yet to accept it. ...
With the deadline for a transition to a new Iraqi interim government six weeks away, US officials and Lakhdar Brahimi, the United Nations envoy responsible for brokering the new government, have been under pressure to come up with a list of interim leaders who are acceptable to a majority of Iraqis.
''We have a lot of work to do now in the next 42 days, roughly," Secretary of State Colin L. Powell told a meeting yesterday of representatives from countries that have contributed to the US-led effort in Iraq. ''We have been in constant consultations with Lakhdar Brahimi all through his current stay in Iraq. We think he is getting closer to the designation of individuals who will be in the interim Iraqi government."
Powell also said that the ''slate of officers" Brahimi will designate will be brought to the UN Security Council and to Secretary-General Kofi Annan so that they could ''examine the quality of these individuals."
( from Boston Globe http://www.boston.com/news/nation/washington/articles/2004/05/21/top_job_in_iraq_said_to_be_offered/ )

I fear you are right ,and that expediency will result in yet another Kurd sell-out .

Iraqi Kurds Optimistic

excon rated this answer Excellent or Above Average Answer
XCHOUX rated this answer Excellent or Above Average Answer

Question/Answer
chekhovToo asked on 05/21/04 - America's Greatest President was?

Who, in your view, was America's greatest president and why?

tomder55 answered on 05/23/04:

Let him who looks for a monument to Washington look around the United States. Your freedom, your independence, your national power, your prosperity, and your prodigious growth are a monument to him. ~Louis Kossuth



I could make an equally compelling case for 3 Presidents ;Washington,Lincoln and FDR .But because of his position as first President of the country under the Constitution ,and the many precidents he set,I have to give the nod to Washington.

Washington was the most revered and influential man in the United States. A lesser person might have used his enormous power to establish a military dictatorship or to become king. Washington sternly suppressed all such attempts on his behalf by his officers .He never ceased to work for the union of the states under a strong central government during the years of the Articles of Confederation. He was a leading influence in persuading the states to participate in the Constitutional Convention, over which he presided, and he used his immense prestige to help gain ratification of the Constitution. The vast powers of the presidency, as one delegate to the Constitutional Convention wrote, would not have been made as great "had not many of the members cast their eyes towards General Washington as president; and shaped their ideas of the powers to be given to a president, by their opinions of his virtue."(Pierce Butler, letter to Weedon Butler, May 5, 1778)
Washington reluctantly accepted the Presidency of the United States. Probably no other man could have succeeded in welding the states into a lasting union. Washington fully understood the significance of the Presidency. "I walk on untrodden ground," he said. "There is scarcely any part of my conduct which may not hereafter be drawn in precedent." After eight years in office, Washington laid down the guidelines for future Presidents. As our first President, he set the precedents that define what it means to be a constitutional executive: strong and energetic, aware of the limits of authority but guarding the prerogatives of office. Washington not only rejected offers to make him king, but was one of the first leaders in world history to relinquish power voluntarily. His peaceful transfer of the presidency to John Adams in 1797 inaugurated one of America's greatest democratic traditions.




Here are some of the accomplishments during Washington's terms:

Mint Act (1792) - established a mint for the printing and coining of U.S. currency

Alexander Hamilton's Financial Program (1790) - a financial plan designed to extinguish some of the debts left for Washington by the Congress of the Articles of the Confederation.

The Bill of Rights (1791) - a set of 10 amendments to the Constitution, guaranteeing American's basic liberties, was passed.

First Bank of the U.S. chartered (1791)

Washington's Proclamation of Neutrality (1793) - stated that the U.S. shall be impartial in dealings with warring nations. Created in response to a general war of England, Spain, Austria, and Prussia versus the new French Republic.

Jay Treaty (1794) - a treaty with England that would regulate trade between America and Britain. The treaty also said that British troops would have to give up their frontier forts as of 1796

Pinkney Treaty with Spain (1795) - a treaty that would open up trade in the Mississippi River

The suppression of the Whisky Rebellion because during the course of this conflict, the government proved that it could enforce law when it needed to. The actions of the government during the Whisky Rebellion led the way for present-day policemen to enforce the laws of the U.S.

Even his farewell address
http://earlyamerica.com/earlyamerica/milestones/farewell/text.html

which ranks right up there along with the Declaration of Independence and the Constitution as one of the greates founding documents ;is best remembered for its counsel about international affairs: Washington recommended commercial relations with other nations but as few political entanglements as possible.

Often overlooked is his remider of the need to uphold the Constitution as our strongest check against tyranny and the best bulwark of our freedom. He warns us to guard against oppositions to lawful authority and those that seek to circumvent the rule of law;he warns about the excessive partisanship ; he wanted liberty to be the objective of our international relations and commerce;to encourage morality and religion.

First in war, first in peace, and first in the hearts of his countrymen, he was second to none in humble and enduring scenes of private life. Pious, just humane, temperate, and sincere; uniform dignified, and commanding; his example was as edifying to all around him as were the effects of that example lasting correct throughout, vice shuddered in his presence and virtue always felt his fostering hand. The purity of his private charter gave effulgence to his public virtues. Such was the man for whom our nation morns (John Marshall, official eulogy of George Washington, delivered by Richard Henry Lee, December 26, 1799)

Washington's is the mightiest name of earth - long since mightiest in the cause of civil liberty; still mightiest in moral reformation. On that name no eulogy is expected. It cannot be. To add brightness to the sun, or glory to the name of Washington, is alike impossible. Let none attempt it. In solemn awe pronounce the name, and in its naked deathless splendor leave it shining on. ~Abraham Lincoln

chekhovToo rated this answer Excellent or Above Average Answer
XCHOUX rated this answer Excellent or Above Average Answer
ETWolverine rated this answer Excellent or Above Average Answer

Question/Answer
SCOOBY asked on 05/21/04 - s.h.

well I have been trying to get rid of s.h. I have told him not to contact me further. I dont want anything more to do with him...is he going to continue to look me up?

tomder55 answered on 05/21/04:

possibly
t.d.

purplewings rated this answer Excellent or Above Average Answer
SCOOBY rated this answer Bad/Wrong Answer

Question/Answer
XCHOUX asked on 05/21/04 - Kerry/Nader Ticket

What are the chances of that? Could that ticked defeat Bush?

Comments please...

tomder55 answered on 05/21/04:

I don't think so . I think they met and will continue to do so in the coming weeks to iron out a deal for Nader to drop out .

Deep down when he peels away his enormous ego ,Nader realizes that he kills Kerry's chances . He cannot be happy with the fact that even the hard core liberals on 'Air America ' are turning on him because of his campaign .

Meanwhile Kerry is in a tough situation . He is the candidate of the anti-war party,but is trying to run a 'I can run the war better ' strategy .Expect up to 5-10% of the Dems. to bolt to a Nader candidacy soley on the war issue.

Look for Kerry to maybe offer a bone to Nader like a cabinet position ,or some lesser position like 'trade tzar'.Then they will hold a joint press announcement where Nader will say that they have met ,and have reached common ground ,and that the most important task is not to focus on the few minor differences they may have but to defeat Bush in the fall.

You were probably right earlier when you predicted a Kerry /Edwards ticket (even though Kerry said during the primaries that Edwards would not carry his own State).

XCHOUX rated this answer Excellent or Above Average Answer
ETWolverine rated this answer Excellent or Above Average Answer
stevehaddock rated this answer Above Average Answer

Question/Answer
Doc05 asked on 05/20/04 - Was this attack justified?

BAGHDAD, Iraq - A U.S. helicopter fired on a wedding party in the remote desert near the border with Syria, killing more than 40 people, most of them women and children, Iraqi officials said. The U.S. military said it attacked a safehouse for foreign fighters near Syria.

Associated Press Television News footage showed a truck containing bloodied bodies, many wrapped in blankets, piled one atop the other. Several were children, one of whom had been decapitated.

The attack on the wedding party occurred about 2:45 a.m. in a desert region near the border with Syria and Jordan, according to Lt. Col. Ziyad al-Jbouri, deputy police chief of Ramadi, the provincial capital about 250 miles to the east. He said between 42 and 45 people died, including 15 children and 10 women. Dr. Salah al-Ani, who works at a hospital in Ramadi, put the death toll at 45.

The area, a desolate region populated only by shepherds, is popular with smugglers, including weapons smugglers, and the U.S. military suspects militants use it as a route to slip in from Syria to fight the Americans. It is under constant surveillance by American forces.

Iraqis interviewed on the videotape said revelers had fired volleys of gunfire into the air in a traditional wedding celebration before the attack took place. American troops have sometimes mistaken celebratory gunfire for hostile fire.

Lt. Col. Dan Williams, a U.S. military spokesman, said the military was investigating.

"I cannot comment on this because we have not received any reports from our units that this has happened nor that any were involved in such a tragedy," Williams wrote in an e-mail in response to a question from The Associated Press.

"We take all these requests seriously and we have forwarded this inquiry to the Joint Operations Center for further review and any other information that may be available," Williams said.

One military officer at the Pentagon said a U.S. military operation was carried out at about 3 a.m. Wednesday against a "foreign fighter safehouse" in the desert about 15 miles from the Syrian border.

The U.S. troops came under hostile fire and called in close-air support. The officer said he did not know what type of aircraft was called in. He refused to say who was killed by the aircraft.

The U.S. troops recovered weapons, Iraqi and Syrian currency, some passports and some satellite communications gear, the officer said.

APTN video footage showed mourners with shovels digging graves over a wide dusty area in Ramadi, the provincial capital where bodies of the dead had been taken to obtain death certificates. A group of men crouched and wept around one coffin.

Al-Ani, the doctor, said people at the wedding fired weapons in the air, and that American troops came to investigate and left. However, al-Ani said, helicopters later arrived and attacked the area. Two houses were destroyed, he said.

"This was a wedding and the (U.S.) planes came and attacked the people at a house. Is this the democracy and freedom that (President) Bush has brought us?" said a man on the videotape, Dahham Harraj. "There was no reason."

Another man shown on the tape, who refused to give his name, said the victims were at a wedding party "and the U.S. military planes came... and started killing everyone in the house." (AP)

Comments please?

tomder55 answered on 05/20/04:

Al-Qaim, has been in the news before. It was the scene of intense fighting between the US Marines and Syrian infiltrators all of last year.Why was a wedding party in full swing at 2:45 AM in the middle of the desert? The area in which the wedding took place was 250 kilometers from Dr. Salah al-Ani, who works at a hospital in Ramadi.A long way to go for medical treatment or burial when Qusabayah is much closer. Under normal circumstances, there are two wounded for every dead. By the normal ratios there should have been at least 90 injured. There was a videotape of showing a truck containing bodies of people who were allegedly killed in the incident. Most of the bodies were wrapped in blankets and other cloths, but the footage showed at least eight uncovered, bloody bodies, several of them children. One of the children was headless. A video of the dead, but where were the wounded?

chekhovToo rated this answer Excellent or Above Average Answer
Doc05 rated this answer Excellent or Above Average Answer
stevehaddock rated this answer Bad/Wrong Answer

Question/Answer
Petesharky asked on 05/20/04 - Gas Prices

I am positive that I am posting this in the wrong place but couldn't find a business board that I thought was appropriate. So here goes. I received this email the other day about Gas prices here it is. Any suggestions would be appreciated.

YES it is copied and pasted so now that we have that out of the way please continue to read. There is a question at the bottom I promise.


IT HAS BEEN CALCULATED THAT IF EVERYONE IN THE UNITED STATES DID NOT PURCHASE A DROP OF GASOLINE FOR ONE DAY AND ALL AT THE SAME TIME, THE OIL COMPANIES WOULD CHOKE ON THEIR STOCKPILES. AT THE SAME TIME IT WOULD HIT THE ENTIRE INDUSTRY WITH A NET LOSS OF OVER 4.6 BILLION DOLLARS WHICH AFFECTS THE BOTTOM LINES OF THE OIL COMPANIES. THEREFORE MAY 19TH HAS BEEN FORMALLY DECLARED "STICK IT UP THEIR behind DAY AND THE PEOPLE OF THIS NATION SHOULD NOT BUY A SINGLE DROP OF GASOLINE THAT DAY. THE ONLY WAY THIS CAN BE DONE IS IF YOU FORWARD THIS E-MAIL TO AS MANY PEOPLE AS YOU CAN AND AS QUICKLY AS YOU CAN TO GET THE WORD OUT. WAITING ON THIS ADMINISTRATION TO STEP IN AND CONTROL THE PRICES IS NOT GOING TO HAPPEN. WHAT HAPPENED TO THE REDUCTION AND CONTROL IN PRICES THAT THE ARAB NATIONS PROMISED TWO WEEKS AGO? REMEMBER ONE THING, NOT ONLY IS THE PRICE OF GASOLINE GOING UP BUT AT THE SAME TIME AIRLINES ARE FORCED TO RAISE THEIR PRICES, TRUCKING COMPANIES ARE FORCED TO RAISE THEIR PRICES WHICH EFFECTS PRICES ON EVERYTHING THAT IS SHIPPED. THINGS LIKE FOOD, CLOTHING, BUILDING MATERIALS, MEDICAL SUPPLIES ETC. WHO PAYS IN THE END? WE DO! WE CAN MAKE A DIFFERENCE. IF THEY DON'T GET THE MESSAGE AFTER ONE DAY, WE WILL DO IT AGAIN AND AGAIN. SO DO YOUR PART AND SPREAD THE WORD. FORWARD THIS EMAIL TO EVERYONE YOU KNOW. MARK YOUR CALENDARS AND MAKE MAY 19TH A DAY THAT THE CITIZENS OF THE UNITED STATES SAY "ENOUGH IS ENOUGH"


To me all this will do is make May 20th pay day for the oil companies. This really won't solve anything except the oil companies will have to wait 1 day before they are raking the cash again. I think a more intelligent idea would be instead of filling up every time you get gas only go half way. If it takes $20 to fill up the tank at $2.00 a gallon only buy $10.00 worth. Yes I know eventually over time this leads to more frequent trips to the gas station and the company gets their money anyway. Just slows down the pace of how they get it, decreases their daily sales by half considering everyone complied.

If a gas station on a regular day takes in $5,000 (relax it's just a number) in Gas sales, and all of a sudden it's taking in $2,500 That still means it's weekly GAS sales would be cut from $35,000 to $17,500 assuming it's open 7 days a week.

Not to mention it leaves their underground tanks half full which means that the gas trucks will only be able to refill only half the amount of gas on a daily basis. Which to me is a more direct hit on oil stock piles.

Assuming the average person only drives 5 to 10 miles to work daily this could be done. My truck goes 2 weeks before I buy gas if it's a full tank.



Not only does it effect the amount of money made in 1 day. Now the oil companies are only distributing half the amount of gas on a daily basis. Meaning that if a truck delivers to 1 station every other day now it will only have to deliver every 4th day.

Station owners will take a direct hit and not be able to meet pay roll based on gas sales and not on donut and coffee sales, repairs etc.


Oil companies stock piles will stack up. Leaving them holding more oil in their tanks and not able to move it as quickly. Although eventually it will sell they have to hold it longer therefore decreasing their daily weekly and monthly profit. Half the amount of delivery trucks will be needed to deliver which means half the company fleet sits in the yard or each truck goes out half full and makes half the money.


The amount of oil bought by these companies would also decrease meaning oil distributing nations would also suffer a daily weekly and monthly loss. Their stock piles will also stack up.


Question (finally): Although in the long run the oil companies are going to sell the gas and make the money wouldn't this be a better way to get the point across? Keeping in mind that it would take everyone to do this and meaning if the price drops we keep on doing it.

tomder55 answered on 05/20/04:

I am not familiar on the economic impact of various boycott techniques ,but think that they are probably useless exercises in the long run. When the email was sent to me ,I looked it up on urbanlegends and they made a compelling case that the boycott was a waste of time ,but it offered the consumer a 'painless way to express their frustrations ;a 'feel good 'statement.


We have not built any new refinery capacity in the last 20 years . If OPEC flooded the market with crude oil ,the price would still only drop marginally .American refineries operate at 95 % capacity. Each region of the country has different requirements in the blends of gasoline due to environmental concerns. So regional refineries could create a greater shortage of supply based on the usage in the region .

There is more competion than ever for the world oil supply . Emerging industrial powerhouses like China and India are using a greater percentage than ever before . Still the U.S. with 5 % of the worlds population consumes over 20%of the worlds oil production .

The average price of gasoline in Europe was $3.50/gal. before the recent spike in prices. The biggest difference is the level of taxation .American's continue to purchase gas guzzling SUVs .CAFE standards have been relaxed ,or are being circumvented by loopholes in the law.America made a foolish choice in 1980 of scrapping a perfectly good energy policy that Jimmy Carter initiated . We have not had a consistant policy since .Nobody even has a clue what Bush's is since Cheney deliberated his Energy Task Force in private ,and has yet to disclose what the recommendations of the Task Force are .I doubt if conservation is a hallmark of his proposal. At least Kerry ,for all his other faults, seems to have a clear idea of what his energy policy would be.

I am strongly against Kerry's idea of tapping into the strategic oil reserve and equally opposed to Bush continuing to stock pile it at these higher prices .The SPR was created for national emergencies and higher gas prices do not qualify as one. Besides we only have 700 million barrels in reserve .We import 10 million A DAY . If we then used it up in short order . What would happen during a REAL emergency ,especially during a time of war ?

I also doubt that new discoveries of domentic crude would make a dent in the price of oil .I do not buy into the idea that there is a huge vast reserve in the Alaskan wilderness ,and if there was ;it would only be profitable to pump it if the price of crude remained high. It would not solve the issue of the price of gas. The facts are pretty obvious to me . The demand of oil ,and natural gas will continue to increase worldwide ,and the supplies will continue to diminish . Oil is NOT a renewable resource.

We need to make smarter choices .We need to develop technolgies that burn coal cleaner;without undesirable emissions .(the U.S. has the largest coal reserve in the world). For all the focus on oil ,the natural gas supply is a much larger concern .Consumption has exceeded our domestic production for 10 years now.
Even without the increase of our population from immigration ;our hunger for computers ,big cars ,a/c home entertainment systems ,sophisticated mendical devices and all the other things that Americans have come to expect in the quality of our lives ,our consuption of energy resources across the board is more likely to increase instead of decline.

Without a effective ,consistant ,longterm energy policy it does little good to point fingers at 'big oil' ,and OPEC .We knew of this problem in 1973. We have had 30 years to deal with it ,and have done nothing. We can blame no one but ourselves. Have oil prices now gone high enough to justify new investments in alternative renewable sources ? Why should we have even waited for new technolgies to emerge? Why not just sponsor projects to test them ? Anyway you slice it or dice it ;part of the reason that we have our troops fighting and dying in the Middle East for the last 20 years has been to secure the source of the fuel that powers our economy.We should be developing new technologies with the same urgency as the Manhattan Project developed the nuclear bomb! Oil supplies have not reached crisis levels yet ,but the price should finally be our last wake up call.



ETWolverine rated this answer Excellent or Above Average Answer
Petesharky rated this answer Excellent or Above Average Answer
stevehaddock rated this answer Excellent or Above Average Answer

Question/Answer
XCHOUX asked on 05/19/04 - Symbolism for Terrorists

In the centuries past, the Middle East was in the crossroads of trade between the East and the West. With the advent of modern transportation such as trains and airplanes, the need to pass through the Middle East has diminished to almost nothing.

Isn't the compulsion to blow up Airplanes and Trains symbolic of the terrorists unhappiness at the reality of the fall of the Middle East as a prosperous cultural and economic region, and its failure to adapt to the real world?

Comments please...

tomder55 answered on 05/19/04:

perhaps in this regard we owe the Muslim world a little bit of thanks . after the fall of Constantinople in 1453 to Ottoman Turks
the West was compelled to find alternate travel routes to the far East to obtain the luxuries that could be traded for there . The
rise of the West ,and the decline of the Muslim world started shortly thereafter .

ETWolverine rated this answer Excellent or Above Average Answer
stevehaddock rated this answer Bad/Wrong Answer
XCHOUX rated this answer Excellent or Above Average Answer

Question/Answer
excon asked on 05/19/04 - Prison Abuse - Orders or License


Hello,

For the first time in the history the Texas Board of Pardons and Paroles, it recommended that Gov. Rick Perry commute the death sentence of a mentally ill man. The Guv snuffed him instead.

This is the State that George Bush ran before we didnt elect him.

If you dont see a connection between Iraq prison abuse and Texas, youre not looking too hard, or you dont want to see the connection. If the soldiers didnt have orders to do what they did, they certainly had license.

Whats my question? How come I see the real world and you dont?

excon

tomder55 answered on 05/19/04:

in America we give a wink and a nod ,and a joke about soap -on- a -rope and do nothing about the fact that an estimated 10% of the prison male population gets raped in our prisons . Yes ; if we then send the guards who stand by and let this stuff happen to Iraq in a less supervised setting ,with according to Gen. Taguba no operational guidelines ;then sure ,I can see where you are heading.
I do not suscribe to this notion that it is uniquely a Texas problem. I know a guard in upstate N.Y. who was a whistle blower ,and took alot of flax for it ;so it is quite obviously a national issue ,and not a 'Texas/Bubba issue.

excon rated this answer Excellent or Above Average Answer
stevehaddock rated this answer Above Average Answer

Question/Answer
voiceguy2000 asked on 05/19/04 - Sarin? What Sarin?

May 19, 2004
OP-ED COLUMNIST

Sarin? What Sarin?

By WILLIAM SAFIRE

You probably missed the news because it didnt get much play, but a small, crude weapon of mass destruction may have been used by Saddams terrorists in Iraq this week.

The apparent weapon was sarin gas, a highly toxic nerve agent that causes victims to choke to death. Developed by the Nazis, it has been used in the past by terrorists in Japan to kill a dozen subway riders and panic thousands, and by Saddam Hussein, who produced tons of it to kill Iraqi Kurds.

Rigged as an improvised explosive device, or roadside bomb, the 155-millimeter howitzer shell was accidentally detonated by a U.S. ordnance team. Two men were treated for what an Army spokesman called minor exposure to the nerve gas.

You never saw such a rush to dismiss this as not news. U.N. weapons inspectors whose reputations rest on denial of Saddams W.M.D. pooh-poohed the report. It doesnt strike me as a big deal, said David Kay.

Sarin Bomb Is Likely a Leftover From the 80s was USA Todays Page 10 brushoff; maybe the terrorists didnt know their shell was loaded with sarin. Besides, say our lionized apostles of defeat, a poison-gas bomb does not a stockpile make. Even the Defense Department, on the defensive, strained not to appear alarmist, saying confirmation was needed for the field tests.

In this rush to misjudgment, we can see an example of the Four Noes that have become the defeatists platform.

The first no is no stockpiles of W.M.D., used to justify the war, were found. With the qualifier so far left out, the absence of evidence is taken to be evidence of absence. In weeks or years to come when the pendulum has swung, and it becomes newsworthy to show how cut-and-runners in 2004 were mistaken logic suggests we will see a rash of articles and blockbuster books to that end.

These may well reveal the successful concealment of W.M.D., as well as prewar shipments thereof to Syria and plans for production and missile delivery, by Saddams Special Republican Guard and fedayeen, as part of his planned guerrilla war the grandmother of all battles. The present story line of Saddam was stupid, fooled by his generals would then be replaced by Saddam was shrewder than we thought.

This will be especially true for bacteriological weapons, which are small and easier to hide. In a sovereign and free Iraq, when germ-warfare scientists are fearful of being tried as prewar criminals, their impetus will be to sing and point to caches of anthrax and other mass killers.

Defeatisms second no is no connection was made between Saddam and Al Qaeda or any of its terrorist affiliates. This is asserted as revealed truth with great fervor, despite an extensive listing of communications and meetings between Iraqi officials and terrorists submitted to Congress months ago.

Most damning is the rise to terrors top rank of Abu Musab al-Zarqawi, who escaped Afghanistan to receive medical treatment in Baghdad. He joined Ansar al-Islam, a Qaeda offshoot whose presence in Iraq to murder Kurds at Saddams behest was noted in this space in the weeks after 9/11. His activity in Iraq was cited by President Bush six months before our invasion. Osamas disciple Zarqawi is now thought to be the televised beheader of a captive American.

The third no is no human-rights high ground can be claimed by us regarding Saddams torture chambers because we mistreated Iraqi prisoners. This equates sleep deprivation with life deprivation, illegal individual humiliation with official mass murder. We flagellate ourselves for mistreatment by a few of our guards, who will be punished; he delightedly oversaw the shoveling of 300,000 innocent Iraqis into unmarked graves. Iraqis know the difference.

The fourth no is no Arab nation is culturally ready for political freedom and our attempt to impose democracy in Iraq is arrogant Wilsonian idealism.

In coming years, this will be blasted by revisionist reportage as an ignoble ethnic-racist slur. Iraqis will gain the power, with our help, to put down the terrorists and find their own brand of political equilibrium.

Will todays defeatists then admit they were wrong? Thats a fifth no.

http://www.nytimes.com/2004/05/19/opinion/19SAFI.html

tomder55 answered on 05/19/04:

today the Washington Post declared :
Fear of Failure Growing

The terrorists must be thrilled. The whole terrorist game plan in the Middle East and in Iraq is based on creating an atmosphere of fear, gloom and doom in the American citizenry. It seems that some media outlets actually have signed on to help the Islamo-fascists achieve that very goal. Downplay the good, hammer the bad. Run the Abu Ghraib story on the front page for three weeks. Bury the Nick Berg story after one day. Ignore the Sarin and mustard gas finds. If the story will help Bush, bury it. If it will hurt Bush, run it day after day. No bias there!



Fear of Failure Growing (http://www.washingtonpost.com/wp-dyn/articles/A37745-2004May18.html)

ETWolverine rated this answer Excellent or Above Average Answer
voiceguy2000 rated this answer Excellent or Above Average Answer
XCHOUX rated this answer Excellent or Above Average Answer

Question/Answer
chekhovToo asked on 05/17/04 - War Crimes

What is a war crime and should the likes of Kissinger, Rumsfeld, and those other alleged criminals that "tortured" Iraqis in Abu Ghraib, be put on trial by an independent international court?

tomder55 answered on 05/18/04:

Somewhere in the line of command at the Abu Ghraib prison stands an unjust and extremely foolish official who condoned or encouraged the abuse of prisoners. The American investigators will find that person, and he will not be Donald Rumsfeld. But even if he directly ordered the abuses(not torture).I would not subject him to any justice system except the American system. We are not signees to the Hague International Court for a good reason .The State Dept. lays out a credible case here

chekhovToo rated this answer Excellent or Above Average Answer
ETWolverine rated this answer Excellent or Above Average Answer
Itsdb rated this answer Excellent or Above Average Answer

Question/Answer
XCHOUX asked on 05/17/04 - WMD's Found in Iraq

An artillery shell containing sarin gas was found in Iraq today. Sarin gas is what the Japanese cult used to kill people in Tokyo's subway system years ago.

Comments?

tomder55 answered on 05/18/04:

They also found a mustard gas shell this week. However we will not find large stockpiles there since Saddam's WMD were transported to Syria in the months that the U.N. stalled the conflict .The most likely possibility is that it was old overlooked arsenal.Saddam had twice the arsenal of explosives (non-nuclear) that the U.S. had . He was adept at hiding them . There are 1200 troops going around the country looking for hidden arsenal .The Iraq Survey Group, ISG, managed by Charles Duelfer, a former State Department official and deputy chief of the U.N.arms-inspection teams, has found "hundreds of cases of activities that were prohibited" under U.N. Security Council resolutions.It is amusing how many times
they come across drums labeled 'fertilizer' at depots;like the find in Karbala last year.

http://www.chron.com/cs/CDA/ssistory.mpl/special/iraq/1855674

At Taji ,an Iraqi weapons complex as large as the District of Columbia , U.S. combat units discovered "pesticides" stockpiled in specially built containers, smaller in diameter but much longer than the standard 55-gallon drum. The 4th Infantry Division found 55-gallon drums containing a substance identified through mass spectrometry analysis as cyclosarin -- a nerve agent.
Nearby were surface-to-surface and surface-to-air missiles, gas masks and a mobile laboratory that could have been used to mix chemicals at the site. Later tests by the "experts" revealed that these were only pesticides.It seems Iraqi soldiers were obsessed with keeping ammo dumps insect-free.

Everytime the U.S has found some "small evidence" that Saddam had WMD, this evidence has been refuted by administration critics as well as some members of the press, as being too small of a reason to have invaded Iraq. Due to these past reactions by many Americans and the U.S press, instead of slowly releasing more information that corroborates what the U.S administration knew all along, they are waiting to release all the proof and information at once. Even so ,the bits and pieces that we know of is telling .

But while the president's critics and the media might plausibly hide behind ambiguity and a lack of sensational-looking finds for not reporting some discoveries, in the case of Saddam's ballistic-missile programs they have no excuse for their silence. Saddam Hussein's prohibited missile programs are as close to a slam dunk as you will ever find for violating United Nations resolutions.

What are "stockpiles" of Chemical Weapon agents supposed to look like? Was anyone really expecting Saddam to have freshly painted warehouses packed with chemical munitions, all neatly laid out in rows, with labels written in English?
Or did they think that Saddam would guide U.S. troops to smoking vats full of nerve gas in an abandoned factory after he was captured?

When former weapons inspector David Kay reported to Congress in January that the United States had found "no stockpiles" of forbidden weapons in Iraq, his conclusions made front-page news. But when he detailed what the ISG had found in testimony before the House Permanent Select Committee on Intelligence last October, few took notice.

Among Kay's revelations, :


A prison laboratory complex that may have been used for human testing of Biological Warfare agents and "that Iraqi officials working to prepare the U.N. inspections were explicitly ordered not to declare to the U.N." Why was Saddam interested in testing biological-warfare agents on humans if he didn't have a biological-weapons program?

"Reference strains" of a wide variety of biological-weapons agents were found beneath the sink in the home of a prominent Iraqi BW scientist. "We thought it was a big deal," a senior administration official said. "But it has been written off [by the press] as a sort of 'starter set.'"

New research on BW-applicable agents, brucella and Congo-Crimean hemorrhagic fever, and continuing work on ricin and aflatoxin that were not declared to the United Nations.

A line of unmanned aerial vehicles, UAVs, or drones, "not fully declared at an undeclared production facility and an admission that they had tested one of their declared UAVs out to a range of 500 kilometers [311 miles], 350 kilometers [217 miles] beyond the permissible limit."

"Continuing covert capability to manufacture fuel propellant useful only for prohibited Scud-variant missiles, a capability that was maintained at least until the end of 2001 and that cooperating Iraqi scientists have said they were told to conceal from the U.N."

"Plans and advanced design work for new long-range missiles with ranges up to at least 1,000 kilometers [621 miles] -- well beyond the 150-kilometer-range limit [93 miles] imposed by the U.N. Missiles of a 1,000-kilometer range would have allowed Iraq to threaten targets throughout the Middle East, including Ankara [Turkey], Cairo [Egypt] and Abu Dhabi [United Arab Emirates]."
In addition, through interviews with Iraqi scientists, seized documents and other evidence, the ISG learned the Iraqi government had made "clandestine attempts between late 1999 and 2002 to obtain from North Korea technology related to 1,300-kilometer-range [807 miles] ballistic missiles -- probably the No Dong -- 300-kilometer-range [186 miles] antiship cruise missiles and other prohibited military equipment," .

Look for the Bush Adm. to contine to compile all the evidence ,and release it either before the summer conventions or shortly after .






XCHOUX rated this answer Excellent or Above Average Answer

Question/Answer
XCHOUX asked on 05/17/04 - Kerry and McCain

The talk today on the radio shows is a ticket of Kerry/McCain. I think that the rationale is that the ticket will be promoted as bridging partisanship for the best interests of American.

Really would like some comments...

tomder55 answered on 05/18/04:

I think it telling that the Dems.are so desperate for him to name a VP . Many times the VP is not selected until the convention or just before . There is certainly time. I do not see the rush except the Dems. think that Kerry needs another charge in his campaign . A sad commentary considering how much material Bush has left him to work with.

McCain does not want to play 2nd fiddle,especially since unlike Cheney he would not be given a bow. Kerry meanwhile will not pick someone more charismatic than he is ;which eliminates most other possibilities. He may pick Bob Kerry because the 9-11 hearings proves he doesn't mind being an attack dog. Dean would also do well in that role ,and would wear his anti-war credentials like Kerry wears his purple hearts . But since they are from the same region I do not think it likely .

I still think Kerry is going to do a rust belt strategy ,and when considering a VP as boring as he is ;someone who would not demand the spotlight ;then Gephardt still seems to be the best bet.

XCHOUX rated this answer Excellent or Above Average Answer

Question/Answer
excon asked on 05/11/04 - Prison Abuse


Hello Citizens:

In their zeal to confirm America's honor in the face of widespread Iraqi prison abuse, the Bush administration points out innumerable examples why we really are a good and decent people. Conspicuously absent from the list of examples, is our very own prison system. Dont you wonder why?

If you want to know why your soldiers did what they did, look no further than your own back yard. Bush was the governor of Texas, for crying out loud. What the hell did you people expect?

excon

tomder55 answered on 05/17/04:

Have to agree with you on this one .

The Other Prison Abuse by Nick Lowery

excon rated this answer Excellent or Above Average Answer

Question/Answer
chekhovToo asked on 05/15/04 - Only the UN can save us

Only the UN can save us

There is one last chance to avert total conflagration in Iraq - Blair must lay it on the line to Bush

Polly Toynbee
Friday May 7, 2004
The Guardian

Just when things could not get any worse in Iraq, they do. The Washington Post's disgusting new pictures yesterday presage as many more horror stories as there are civilians randomly killed and people imprisoned or disappeared without explanation. Desperate families outside jails, waving bits of paper with names and begging for news, have had their pleas ignored for a year by the powers that invaded on a promise to bring the rule of law and human rights.
The systematic torture of prisoners in Abu Ghraib and elsewhere is so poisonous in its symbolism that not even America's mortal enemies could have devised such a PR coup. Sexual abuse and humiliation of naked Muslim prisoners, urinated on and sodomised, and orders from US intelligence to "soften up" victims in Saddam's old torture chamber almost defies belief.

Except it doesn't. Atrocities are entirely predictable wherever absolute power holds the utterly helpless in secret: that is a universal law of human nature. In peace, that is as true of old people, the mentally ill and children in institutions hidden from view. In war, degraded captives bring out an instinctive disgust, contempt and violence in the captors who degrade them. That is why habeas corpus was the founding principle of British justice, even before Magna Carta, banning the holding of people uncharged, unseen without trial.

"Their treatment does not reflect the nature of the American people," said President Bush plaintively. Indeed, but it is in the nature of the circumstances that Bush has authorised for holding 10,000 prisoners without trial, many in unknown, secret prisons. "That's not the way we do things in America," he says. Indeed, it is only the way America does things when it goes abroad; the American constitution protects its own citizens. The self-blinding American myth is that a "freedom-loving nation" built on the ideals of Thomas Paine, Benjamin Franklin and Thomas Jefferson could never do such things.

But it was Tony Blair's foreign policy adviser, Robert Cooper, who elevated double standards into a doctrine, declaring human rights are only for the civilised: "Among ourselves we keep the law but when operating in the jungle, we must also use the laws of the jungle." That is a self-defeating way to bring civilised values to those whose hearts and minds are the real battleground of "the war on terror". So, although the British foreign terror suspects held without trial have lawyers and are reviewed regularly by a court of law, the need for transparent justice is why they must all be brought to trial or let go. Vera Baird, the MP and QC, finds no reason why the procedures she has used during gangster trials for protecting secret sources and disguising witnesses would not hold good for protecting any risky security sources in these cases.

Invading armies always commit atrocities - often not revealed until years later. Where they can no longer discriminate friend from foe in a sea of alien faces, they are bound to kill indiscriminately. Warning bells rang when, even after the regime fell, UK and US forces still refused to count civilian deaths.

Few of us who argued against this war imagined things would be this bad. As the president begs Congress for another $25bn in the shadow of this chaos, the rubble of neo-conservative strategy lies all about him. The dream that this expedition would herald a new era of democracy across the Middle East is dead on his lips. Essential contractors have quit as insurance brokers declare Iraq the most dangerous place in the world to business. A Gallup poll in Baghdad, taken just before the torture pictures appeared, showed only 10% of Iraqis had a favourable opinion of the US.

In a majestic lecture last night at the LSE, Professor Fred Halliday, a foremost Arab expert, described the full seriousness of what he called this "crisis of our times". What gives special power to Halliday's analysis is this: with Iraqi connections dating back to before Saddam, and having seen at close quarters the horror of that regime, he did not oppose intervention. WMD was always a dubious excuse, he argues, but Saddam's defiance of UN resolutions was justification, especially his refusal to implement human rights and democracy contained in resolution 688 in 1991.

But now, he says,"the USA has destroyed the goodwill it initially enjoyed" in the days after Saddam fell. Now "the situation is quite literally out of control", with no coherent policy. Paul Bremer in his bunker leaves the initiative to military commanders who have no sense of politics or diplomacy. The US has alienated its allies across the region with its reckless endorsement Sharon and helped to awaken a transnational army of jihadis. Its traditional allies, "the corrupt, weak dictatorships of the region", have been left angry and vulnerable. The "shock and awe" of American global dominance turning into a daily spectacle of ineptitude and failure. Halliday puts what shreds of hope he retains in the UN's Lakhdar Brahimi, as the last chance to avoid total conflagration and the triumph of extremists. But he doubts Washington now is capable of listening to Brahimi. The US now needs the UN, but still refuses to bow sufficiently to the only hope left of rescue.

The neo-conservative dream of total American hegemony without need of allies or international law has been exposed as impossible as well as undesirable. All this causes much smirking satisfaction in the more rabid anti-American camp. But the effect of an Iraq meltdown could have ominous global repercussions. A US retreat into isolationism is no answer. The US is only superpower: the UN and the world have as much need of it as ever for humanitarian interventions.

For example: Human Rights Watch today publishes a damning report on Sudan, where government-backed militias in Darfur are accused of ethnically cleansing an entire district - the Arabs expelling, burning, killing and raping hundreds of thousands of Africans from fertile lands. Watching the TV pictures, the impulse is to cry out: "Do something, someone!" But who? The UN? On Monday Sudan was elected to the UN Commission on Human Rights, put there the African regional group. Cuba and Zimbabwe have also just been elected, and Libya chaired the commission last year. No wonder the US walked out.

But for all its need of reform, the UN is all there is, and Brahimi is Iraq's last best hope. If Tony Blair wants to save what is left of his fearful Iraq error, now is the time for him to put loud pressure on Bush to guarantee the UN a central role after the June 30 handover, with command over the military, and drawing in Turkey and Arab nations under a UN banner. All prisoners must be handed over to UN authority to be dealt with transparently under international law. Otherwise Blair should warn that Britain will follow Spain, Bulgaria and Poland in ordering a withdrawal of troops. Demanding a UN handover is his last chance to do the right thing.

tomder55 answered on 05/17/04:

In 1967, Egypt ordered UN forces out of the Sinai to clear the way for an attack on Israel. Then Secretary General of the UN,
U-Thant, agreed. The Six day
war was the result.



From 1975 to 1979, Pol Pot killed more than 2 million of his own Cambodian countrymen. The UN did nothing.



In 1994, the UN withdrew its forces from Rwanda, facilitating the genocidal
killing of over 800,000 people.



In 1995, UN forces stood by as 7,000 Bosnians were massacred in
Srebrenica, in the heart of a UN-declared safe haven.



In 2000, three Israeli soldiers were kidnapped while on patrol on their side of
the Lebanese border, by Hizbollah guerillas. UN soldiers took video footage of the attack, as if it was some kind of tourist
show. Kofi Anan at first denied the existence of the tapes, then acknowledged them,but allowed Israel, and the victims' families, to see only a highly edited version.



Of the 189 member nations, only 82 have anything resembling a stable democracy. That
is, they have free elections, a free press, freedom of assembly and they protect their
citizens' right to free expression. A substantial majority of the UN's
membership is made up of dictatorship regimes .For the US, and other free nations, to subject herself to the UN on any issue
that bears on our defense or economic health is a betrayal of our constitution.



chekhovToo rated this answer Excellent or Above Average Answer
XCHOUX rated this answer Excellent or Above Average Answer

Question/Answer
XCHOUX asked on 05/16/04 - Finally Become Obvious?

Has it finally become obvious to all that we are at the beginning of WWIII(or WWIV, if you count the defeat of the USSR in the "cold war")?

tomder55 answered on 05/17/04:

In an address to a group of college students, former CIA director James Woolsey described the Cold War as the third world war and said "This fourth world war, I think, will last considerably longer than either World Wars I or II did for us. Hopefully not the full four-plus decades of the Cold War." He said the new war is actually against three enemies: the religious rulers of Iran, the "fascists" of Iraq and Syria, and Islamic extremists like al Qaeda. Woolsey said that all three enemies have waged war against the United States for several years but the United States has just "finally noticed." "As we move toward a new Middle East over the years and, I think, over the decades to come ... we will make a lot of people very nervous." It will be America's backing of democratic movements throughout the Middle East that will bring about this sense of unease, he said. "Our response should be, 'good . Singling out Egyptian President Hosni Mubarak and the leaders of Saudi Arabia, he said, "We want you nervous. We want you to realize now, for the fourth time in a hundred years, this country and its allies are on the march and that we are on the side of those whom you -- the Mubaraks, the Saudi Royal family -- most fear: We're on the side of your own people."

"Does my country really understand that this is World War III? And if this attack was the Pearl Harbor of World War III, it means there is a long, long war ahead."
Thomas Friedman, New York Times, Sept. 14, 2001

The Wall Street Journal identified the conflict with Islamo-fascists correctly as WWIV back in Nov. 2001 .
The enemy in this war is not "terrorism"--a distilled essence of evil, conducted by the real-world equivalents of J. K. Rowling's Lord Voldemort, Tolkien's Sauron or C. S. Lewis's White Witch--but militant Islam. The enemy has an ideology, and an hour spent surfing the Web will give the average citizen at least the kind of insights that he might have found during World Wars II and III by reading "Mein Kampf" or the writings of Lenin, Stalin or Mao. Those insights, of course, eluded those in the West who preferred--understandably, but dangerously--to define the problem as something more manageable, such as German resentment about the Versailles Treaty, an exaggerated form of Russian national interest, or peasant resentment of landlords taken a bit too far. In the reported words of one survivor of the Holocaust, when asked what lesson he had taken from his experience of the 1940s, "If someone tells you that he intends to kill you, believe him."






ETWolverine rated this answer Excellent or Above Average Answer
excon rated this answer Excellent or Above Average Answer
purplewings rated this answer Excellent or Above Average Answer
voiceguy2000 rated this answer Excellent or Above Average Answer
XCHOUX rated this answer Excellent or Above Average Answer

Question/Answer
elgin_republicans asked on 05/14/04 - Rudy Giuliani Survey?

I was contacted by the Zogby Polling company yesterday, and informed that they were doing a survey for the Italian-American Anti-Defamation League.

They then asked me a bunch of questions about how I view Italian-American men and women, what stereotypes do I have, what does the media tell me, what do I believe, would I marry one, would I let my sister marry one, would I join a letter writing campaign to stop Italian-American bashing, AND right in the middle of the survey, "would you vote for an Italian-American candidate?" There were various possible answers, yes, no, not sure, depends.

My question - are the Repubs really behind this, and is this the first step to see if Rudy is viable, therefore leading to his replacing Cheney on the ticket, OR am I grasping at straws and should go lie down for a while?

tomder55 answered on 05/16/04:

your instincts are correct .I think it is part to gage public perceptions on Italian Americans in general ,and of Rudy in particular. The I-A ADF probably would commmission such a poll. i just don't trust Zogby all that much .
Cheney is a liability on the ticket so long as the perception of a large # of people is that he pullls the strings in the White House. His connections to Halliburton;his stupid secret energy meetings without any public accounting ;his heading of the anti-terror
task force that allegedly never met prior to 9-11;and his health concerns are all fodder to the Dem.attack dogs like move on.org ,and Kerry himself.Rudy would make a fine replacement,but for selfish reasons ,I hope he concentrates on runnin gfor N.Y. Senator .
Taking out either Hillary or Shumer is fine with me. Actually Shumer would be my first choice to go.Hillary has been mild compared to his 'chicken little' act

elgin_republicans rated this answer Excellent or Above Average Answer
XCHOUX rated this answer Excellent or Above Average Answer

Question/Answer
chekhovToo asked on 05/12/04 - International Relations and World Politics - An Introduction.

Dear Choux, Purplewings and ETWolverine,

I feel that a lot of your responses to questions on the Politics Board lack an understanding of International Relations/World Politics. So, I am going to help you and offer some advice on some good books to read on IR and Politics. That way, after reading them, you will be better informed and thus your responses will reflect your learning in this area.

The books I recommend are as follows.

The Globalization of World Politics: An Introduction to International Relations by Baylis and Smith.

Understanding International Conflicts and The Paradox of American Power by Joesph Nye.

And a subscription to Foreign Affairs Magazine would be good as well.

Age of Extremes by Eric Hobsbawm.

Best wishes,

A.

tomder55 answered on 05/13/04:

How about :

The Clash of Civilizations by Samual P Huntington.

Islamic Fundamentalism The New Global Threat by Mohamma Mohaddessin


The Lexus & The Olive Tree by Tom Friedman

Special Providence: American Foreign Policy and How It Changed the World by Walter Russell

Of Paradise and Power by Robert Kagan

chekhovToo rated this answer Excellent or Above Average Answer

Question/Answer
chekhovToo asked on 05/12/04 - Legitimising Terrorism

Legitimising Terrorism
By Endorsing Ariel Sharon's Plan George Bush Has Legitimised Terrorism

by Robert Fisk; The Independent; April 17, 2004

So President George Bush tears up the Israeli-Palestinian peace plan and that's okay. Israeli settlements for Jews and Jews only on the West Bank. That's okay. Taking land from Palestinians who have owned that land for generations, that's okay. UN Security Council Resolution 242 says that land cannot be acquired by war. Forget it. That's okay.



Does President George Bush actually work for al-Qa'ida? What does this mean? That George Bush cares more about his re-election than he does about the Middle East? Or that George Bush is more frightened of the Israeli lobby than he is of his own electorate. Fear not, it is the latter.



His language, his narrative, his discourse on history, has been such a lie these past three weeks that I wonder why we bother to listen to his boring press conferences. Ariel Sharon, the perpetrator of the Sabra and Shatila massacre (1,700 Palestinian civilians dead) is a "man of peace" - even though the official 1993 Israeli report on the massacre said he was "personally responsible" for it. Now, Mr Bush is praising Mr Sharon's plan to steal yet more Palestinian land as a "historic and courageous act".



Heaven spare us all. Give up the puny illegal Jewish settlements in Gaza and everything's okay: the theft of land by colonial settlers, the denial of any right of return to Israel by those Palestinians who lived there, that's okay. Mr Bush, who claimed he changed the Middle East by invading Iraq, says he is now changing the world by invading Iraq! Okay! Is there no one to cry "Stop! Enough!"?



Two nights ago, this most dangerous man, George Bush, talked about "freedom in Iraq". Not "democracy" in Iraq. No, "democracy" was no longer mentioned. "Democracy" was simply left out of the equation. Now it was just "freedom" - freedom from Saddam rather than freedom to have elections. And what is this "freedom" supposed to involve? One group of American-appointed Iraqis will cede power to another group of American-appointed Iraqis. That will be the "historic handover" of Iraqi "sovereignty". Yes, I can well see why George Bush wants to witness a "handover" of sovereignty. "Our boys" must be out of the firing line - let the Iraqis be the sandbags.



Iraqi history is already being written. In revenge for the brutal killing of four American mercenaries - for that is what they were - US Marines carried out a massacre of hundreds of women and children and guerillas in the Sunni Muslim city of Fallujah. The US military says that the vast majority of the dead were militants. Untrue, say the doctors. But the hundreds of dead, many of whom were indeed civilians, were a shameful reflection on the rabble of American soldiery who conducted these undisciplined attacks on Fallujah. Many Baghdadi Sunnis say that in the "New Iraq" - the Iraqi version, not the Paul Bremer version - Fallujah should be given the status of a new Iraqi capital.



Vast areas of the Palestinian West Bank will now become Israel, courtesy of President Bush. Land which belongs to people other than Israelis must now be stolen by Israelis because it is "unrealistic" to accept otherwise. Is Mr Bush a thief? Is he a criminal? Can he be charged with abetting a criminal act? Can Iraq now claim to Kuwait that it is "unrealistic" that the Ottoman borders can be changed? Palestinian land once included all of what is now Israel. It is not, apparently, "realistic" to change this, even to two per cent?



Is Saddam Hussein to be re-bottled and put back in charge of Iraq on the basis that his 1990 invasion of Kuwait was "realistic"? Or that his invasion of Iran - when we helped him try to destroy Ayatollah Khomeini's revolution - was "realistic" because he initially attacked only the Arabic- speaking (and thus "Iraqi") parts of Iran?



Or, since President Bush now seems to be a history buff, are the Germans to be given back Danzig or the Sudetenland? Or Austria? Or should we perhaps recreate the colonial possessions of the past 100 years? Is it not "realistic" that the French should retake Algeria - or part of Algeria - on the basis that the people all speak French, on the basis that this was once part of the French nation? Or should the British retake Cyprus? Or Aden? Or Egypt? Shouldn't the French be allowed to take back Lebanon and Syria? Why shouldn't the British re-take America and boot out those pesky "terrorists" who oppose the rule of King George's democracy well over 200 years ago?



Because this is what George Bush's lunacy and weakness can lead to. We all have lands that "God" gave us. Didn't Queen Mary die with "Calais" engraved on her heart? Doesn't Spain have a legitimate right to the Netherlands? Or Sweden the right to Norway and Denmark? Every colonial power, including Israel can put forward these preposterous demands.



What Bush has actually done is give way to the crazed world of Christian Zionism. The fundamentalist Christians who support Israel's theft of the West Bank on the grounds that the state of Israel must exist there according to God's law until the second coming, believe that Jesus will return to earth and the Israelis - for this is the Bush "Christian Sundie" belief - will then have to convert to Christianity or die in the battle of Amargeddon.



I kid thee not. This is the Christian fundamentalist belief, which even the Israeli embassy in Washington go along with - without comment, of course - in their weekly Christian Zionist prayer meetings. Every claim by Osama bin Laden, every statement that the United States represents Zionism and supports the theft of Arab lands will now have been proved true to millions of Arabs, even those who had no time for Bin Laden. What better recruiting sergeant could Bin Laden have than George Bush. Doesn't he realise what this means for young American soldiers in Iraq or are Israelis more important than American lives in Mesopotamia?



Everything the US government has done to preserve its name as a "middle- man" in the Middle East has now been thrown away by this gutless, cowardly US President, George W Bush. That it will place his soldiers at greater risk doesn't worry him - anyway, he doesn't do funerals. That it goes against natural justice doesn't worry him. That his statements are against international law is of no consequence.



And still we have to cow-tow to this man. If we are struck by al-Qa'ida it is our fault. And if 90 per cent of the population of Spain point out that they opposed the war, then they are pro-terrorists to complain that 200 of their civilians were killed by al-Qa'ida. First the Spanish complain about the war, then they are made to suffer for it - and then they are condemned as "appeasers" by the Bush regime and its craven journalists when they complain that their husbands and wives and sons did not deserve to die.



If this is to be their fate, excuse me, but I would like to have a Spanish passport so that I can share the Spanish people's "cowardice"! If Mr Sharon is "historic" and "courageous", then the murderers of Hamas and Islamic Jihad will be able to claim the same. Mr Bush legitimised "terrorism" this week - and everyone who loses a limb or a life can thank him for his yellow streak. And, I fear, they can thank Mr Blair for his cowardice too.

tomder55 answered on 05/13/04:

ownership of land is certainly dependent on ones ability to defend it. now lets see ,a unilateral withdrawal from Gaza without any reciprocal concessions by the Palestinians is now in Fisk's strange world considered a land grab? I would also add that the Palestinians according to the flawed "peace plan" that Fisk bemoans is being ignored called on the Palestinians to dismantle the terrorists networks in Palestine .That of course did not happen because no one holds the Palestinians accountable for anything .A pregnant Jewish woman and her children get attacked and executed ,and that barely makes the news . The Palestinians have stubbornly resisted any attempt to forge a 2 State solution to the problem. Instead they hold on to the fantasy of 'right of return' ;even though everyone knows that it is part of a grand strategy designed by Arafat in 74 for the complete destruction of the State of Israel. At least Fisk should be honest enough to admit that he favors that goal .

That is some great spin he puts on the Fallujah action . reminds me of the phony outrage over Jenin . He brushes aside the brutal murder and desecration of Americans who were there to protect food convoys so that Iraqis might eat.The fact that the cowardly insurgents hid behind a civilian population as they took pot shots at American Marines must be acceptable to him. The fact that the U.S. did not finish the action in Fallujah due to concern of creating further civilian casualties is ignored completely .Funny ,I do not recall a simular outrage when the Russians were shelling Grozny. Nor did we hear such outrage when Clinton was lobbing cuise missiles into Iraq . I guess the civilian collateral damage was ok back then to Fisk.

Let him get his Spanish passport and move out .The sooner the better .

chekhovToo rated this answer Excellent or Above Average Answer
ETWolverine rated this answer Excellent or Above Average Answer
purplewings rated this answer Excellent or Above Average Answer

Question/Answer
denberg asked on 05/11/04 - Will Saddam be executed?


Will Saddam be executed?

Not should he be, but WILL he be executed? In my opinion, the longer it takes to put him on trial, the more difficult it will be to execute him afterwards.

I know many people think it is pointless to predict, but what the hell - predictions please, Experts!

tomder55 answered on 05/12/04:

From my understanding the IGC has set up a tribunal under the leadership of Salem Chalabi, a U.S.-educated lawyer and nephew of the head of the Iraqi National Congress. He was named as general director of the tribunal and he has appointed a panel of seven judges and four prosecutors.(Amnesty International of course objected to the formation of the tribunal).He said the tribunal will be an Iraqi court, not an international body, and will rely on a mix of Iraqi criminal law, international regulations such as the Geneva Convention and experiences of bodies such as the Rwanda war crimes tribunal. Documents seized by American forces since the war are likely to form part of the evidence against the former Iraqi leadership. Since the fall of Saddam, 300,000 bodies have been found buried in mass graves.

Salem Chalabi told reporters in Kuwait that trials would begin early next year and that judges would receive ``files'' on the suspects at the end of this year.

``We will put 100 people ... including Saddam Hussein, on trial,'' he said. The suspects, he added, ``will be delivered to us by the coalition before the transfer of power,'' which is scheduled for June 30.

However ; A U.S. defense department spokesman, said it has not been decided when to turn Saddam over to Iraqi authorities. Nor has it been determined when and where Saddam will be tried, but the intention remains to have him tried by the Iraqi people, the spokseman said. I suggest the sooner the better . The Iraqis know how to deal with the scum.

denberg rated this answer Excellent or Above Average Answer

Question/Answer
XCHOUX asked on 05/08/04 - More Horrific Photos To Come

What can Bush do to try to rescue his Presidency? His War on Iraq.

Will it be up to Kerry to straighten out this worst case scenario? A barbaric, antediluvian Middle-East up for grabs. Got oil?

Chou

tomder55 answered on 05/10/04:


Iraq could be lost if the Bush administration continues in the course it is taking.The plan in Iraq is to wait for Lakhdar Brahimi to name an interim Iraqi government by the end of May that will take power on July 1, and prepare for elections in January 2005. The biggest mistake made by the Bush administration has been the failure to move Iraq more rapidly toward elections. Ayatollah Ali Sistani called for early elections back in January . He was right then ,and now .We have been asking and training them to do more of the fighting and dying ,but we still seem to have a reluctance to let them govern themselves. It is not real sovereignty when a U.N. official ;especially Lakhdar Brahimi who doesnt give a damn about democracy ,and would prefer that they live under some tyrant,tells Iraqis who their next prime minister will be.

Moving up the Iraqi elections for sometime in the Summer would have some real positive results . It would take Iraqi minds off of anger over the abuse scandal. It would make it clear to Iraqis that insurgents are not only attacking the U.S. occupation,but also the democratic process in Iraq.Sunnis would have to organize politically if they hope to get any say in the new Iraq. American military actions could be seen not just as an effort to suppress rebellious Iraqis but as a support for the elections process, and for democracy. Americans would be fighting to give Iraqis a chance to vote. Elections in Iraq within a few months might give Americans here at home greater confidence that things can be turned around in Iraq.

Bush and Rumsfeld should stop being stubborn about troop strength. We need more there ;of that I have no doubt.The military needs more money and more troops. After scraping up soldiers from around the world and from behind desks, the Defense Department has left us vulnerable to the lunatic Kim Jon Il in North Korea, and to any other regime that, sensing our lack of resolve in Iraq, decides that now is the time to strike against American interests. That includes China, should it decide to make a move against Taiwan while it believes our military is too thinly spread around the world to intervene. The army, which had 18 active-duty divisions in 1991, now has only 10 divisions (thanks to Clinton and Congressional cut backs ). This puts an unfair burden on the National Guard, consisting of dedicated but less well-trained men and women, many of whom never expected to do prolonged service overseas,and frankly it is unfair to continue to ask them to do so . This prisoner scandal is clearly the risk you take when weekend warriors are asked to do the job of soldiers .

There is no cheap route to victory. The president correctly is trying to persuading us that we are in a war for the survival of our values and way of life.But at the same time he doles out tax refunds to keep us happily in the malls,and resists measures to reduce our reliance on Middle Eastern oil(which in itself is another national scandal in the making for 30 years ,and encompassing administrations from both parties). No pain, lots of gain.

Congress authorized the spending of $18.4 billion on reconstructing Iraq. But for some reason his State Dept. bureaucrats in Iraq are not spending that money. Less than 5 percent of that money has been spent. Bush should get into Powells face to force him to get his hacks to do their job ;which is to implement the policies of the president . If they dont like it ,they should quit and stop being monkey wrenches in the machine.

Bush remains our best bet to lead us to victory in the war,. John Kerry would call it quits in Iraq .He would have our troops hand over their responsibilities to blue-helmet U.N. peacekeepers(it is still not clear how he would convince the U.N. to participate ) while Bush would hand off power to some version of a sovereign Iraqi government in the vision of U.N.'s Israel-hating Brahimi, and at least stay long enough to restore some semblance of order to the country. Kerry's history of opposing increases in military spending, and his commitment to step up spending on domestic entitlement programs, suggests that he is even more likely than the president to look for a cheap victory.




excon rated this answer Excellent or Above Average Answer
XCHOUX rated this answer Excellent or Above Average Answer

Question/Answer
stevehaddock asked on 05/08/04 - Republicans out of the loop

I was watching CNN last week (not a common event for me as it turns out) and Wolf Blitzer was talking to a couple of California representatives (one D, one R) about the prisoner-of-war abuse scandal.

The Republican was very well spoken and very supportive of the administration, calling the abuse an aboration by a small minority of soldiers when the vast majority were hard working people. However, the Democrat and Blitzer pointed out that it could turn out that there had been more abuses that hadn't been disclosed yet. The Republican was steadfast it wasn't a problem.

A few days later, the other shoe dropped. The Red Cross went public saying the problems were endemic and had been going on for almost a year. Finally, Rumsfeld came out and said he knew about widespread abuse in January. Yesterday, he said the stuff we've seen is tame compared to some other video and pictures that he saw.

After my initial "shock and awe", my mind returned to the poor Republican representative who was so supportive of the administration and the military earlier this week. He was obviously earnest and didn't know about the scope of the abuse - no person who had would have dared been as supportive as he was. The message was clear - the White House didn't even let people in its own party know how bad the situation is. This one representative is probably not the only poor Republican who is going to be roasted over the coals by his or her constitutents over this issue.

So here's the question - the Republicans in Congress may be facing an uphill battle in November. Is the administration so unconcerned about the prospects of their own party that they will continue to keep their own people in the dark about events that will certainly go public? If so, we could be looking at the post-Watergate election of 1974 when the Republicans suffered big losses.

tomder55 answered on 05/10/04:

Would it matter ? Bush has a majority in both houses now ,and his legislative agenda has pretty much stalled,as well as his appointments . It is clear that the Defense Dept. knew of the problem by Jan. ,and the Tagubda report came out in Feb. Apparently ,Rumsfeld had the information at that time ,and did not disclose the full extent of the problem to Bush .

Rumsfeld announced that four citizens Dr. James Schlesinger,a Defense Secretary from 1973 to 1975, Ms. Tillie Fowler,who has headed the Air Force Academy's oversight board into sexual assaults' ,retired Air Force General Charles Horner and Dr. Harold Brown ,Secretary of the Air Force from Oct. 1965 to Feb. 1969 ,have agreed to review Department of Defense detention operations. They will provide independent, professional advice on issues related to the mistreatment of detainees. They will address issues such as force structure, training of regular and reserve personnel, organization, detention policy and procedures, interrogation policy and procedures, command relationships and operational practices.They will be asked to report their findings within 45 days.

Sen. Chuck Hagel, R-Neb., told Face the Nation on Sunday. I think they have made major mistakes. And we will see how far this goes and where it goes. Yes, I think its still in question whether Secretary of Defense Rumsfeld and, quite frankly, Gen. Myers can command the respect and the trust and the confidence of the military and the American people to lead this country.

Sen. Lindsey Graham, R-S.C., said on Meet the Press.If theres more to come, lets get it out, "The American public needs to understand we're talking about rape and murder here. we're not just talking about giving people a humiliating experience." For Gods sake, lets talk about it because (U.S. military) men and womens lives are at stake given how we handle this.

John Warner ;chair of the Senate Armed Services Committee was angry that they were not informed .Still, key Republicans closed ranks behind Rumsfeld after his testimony. Senate Majority Leader Bill Frist said he wanted him to stay, as did Warner.
Warner is going to chair another meeting tomorrow where the Committee will see more photos that are still classified .

My gut feeling is that if there is full disclosure then the impact of this will not reverberate into the fall elections . I think more important will be the July transfer of power ,and how the war is conducted from here on out . If this war brings down the Administration then yes I think that Congress could also go Democrat,but not to the same degree as Watergate ,when the president was forced to resign over illegalities.

Right now it is very apparent that the War in iraq will be the over riding issue in the coming elections . Forget talk of economy ,job loss (now defined by Kerry as quality of new jobs created),or any of the periferal issues. The events in Iraq will determine the next President.






stevehaddock rated this answer Excellent or Above Average Answer

Question/Answer
XCHOUX asked on 05/09/04 - Rush Limbaugh Disgraces Himself and Conservatives

That gasbag of right wing "Conservatism" Rush(heroin(synthetic)) Limbaugh has disgraced everyone connected to the Bush Administration and Radical Radio by his comments about the treatment of Iraqi prisioners of war.

A station discussiog the current situation played a clip from his show in which he described the torture of Iraqi prisioners as frat pranks, laughed maniac-lly(sp), I can't remember others, but were disgusting.

Can we get America back from the partisan and irrational media, the likes of this guy, O'Reilly, Ann Coulter, a professional liar, and the lot? They are really harming us.

Is the possibility of rational debate gone forever?

tomder55 answered on 05/10/04:

I have to plea guilty also. My initial reaction based on the released photos was that it was more of a college hazing type activity .As more details are coming known ,it is apparent that the problem is much deeper than that .

As far as rational debate. There is good solid debate from both sides of the political spectrum ,and there are entertainers catering to the extremes like Limbaugh and "O'Frankin". Niether could be convinced of the merit of the others argument no matter how much compelling evidence is presented. O'Reilly does not fit into that catagory ,but since there is annamus between him and Frankin ,you'll never hear that from Frankin ,and Garafalo .If I were Bush and Kerry I would distance myself from the whole lot of them .

ETWolverine rated this answer Excellent or Above Average Answer
excon rated this answer Excellent or Above Average Answer
XCHOUX rated this answer Excellent or Above Average Answer

Question/Answer
rebeccajr asked on 05/08/04 - Ronald Takaki and Louis Hartz

How would you compare and critque Iron Cages race and culture in 19th century America, by Ronald Takaki and The LiberalTradition in America by Louis Hartz? What is the name of the political theories of both these books?

tomder55 answered on 05/09/04:

Ronald Takaki debates diversity in Iron Cages He disputes the theme of American history that this country was settled by European immigrants and that Americans are white. He covers the period from post revolution to the 20th century.during a time when the country grew in size, population, and importance. Takaki looks at these issues showing American philosophy of moving westard and expanding trade. He focuses on the race related issues of the period such as the roles of Native Ameiricans, Blacks and Chinese in this country.He says that even thogh the founders tried to forge different society form Europe ,the old European attitudes about race permeated throughout white America,not just in the slavery issue,but also in how all othr races were treated. The theme of this book and I assume Hartz's book is multiculturalism.



Doc05 rated this answer Excellent or Above Average Answer

Question/Answer
chekhovToo asked on 05/06/04 - Restoring Our Honor

Restoring Our Honor
By THOMAS L. FRIEDMAN

We are in danger of losing something much more important than just the war in Iraq. We are in danger of losing America as an instrument of moral authority and inspiration in the world. I have never known a time in my life when America and its president were more hated around the world than today. I was just in Japan, and even young Japanese dislike us. It's no wonder that so many Americans are obsessed with the finale of the sitcom "Friends" right now. They're the only friends we have, and even they're leaving.

This administration needs to undertake a total overhaul of its Iraq policy; otherwise, it is courting a total disaster for us all.

That overhaul needs to begin with President Bush firing Secretary of Defense Donald Rumsfeld today, not tomorrow or next month, today. What happened in Abu Ghraib prison was, at best, a fundamental breakdown in the chain of command under Mr. Rumsfeld's authority, or, at worst, part of a deliberate policy somewhere in the military-intelligence command of sexually humiliating prisoners to soften them up for interrogation, a policy that ran amok.

Either way, the secretary of defense is ultimately responsible, and if we are going to rebuild our credibility as instruments of humanitarian values, the rule of law and democratization, in Iraq or elsewhere, Mr. Bush must hold his own defense secretary accountable. Words matter, but deeds matter more. If the Pentagon leadership ran any U.S. company with the kind of abysmal planning in this war, it would have been fired by shareholders months ago.

I know that tough interrogations are vital in a war against a merciless enemy, but outright torture, or this sexual-humiliation-for-entertainment, is abhorrent. I also know the sort of abuse that went on in Abu Ghraib prison goes on in prisons all over the Arab world every day, as it did under Saddam without the Arab League or Al Jazeera ever saying a word about it. I know they are shameful hypocrites, but I want my country to behave better not only because it is America, but also because the war on terrorism is a war of ideas, and to have any chance of winning we must maintain the credibility of our ideas.

We were hit on 9/11 by people who believed hateful ideas ideas too often endorsed by some of their own spiritual leaders and educators back home. We cannot win a war of ideas against such people by ourselves. Only Arabs and Muslims can. What we could do and this was the only legitimate rationale for this war was try to help Iraqis create a progressive context in the heart of the Arab-Muslim world where that war of ideas could be fought out.

But it is hard to partner with someone when you become so radioactive no one wants to stand next to you. We have to restore some sense of partnership with the world if we are going to successfully partner with Iraqis.

Mr. Bush needs to invite to Camp David the five permanent members of the U.N. Security Council, the heads of both NATO and the U.N., and the leaders of Egypt, Jordan, Saudi Arabia and Syria. There, he needs to eat crow, apologize for his mistakes and make clear that he is turning a new page. Second, he needs to explain that we are losing in Iraq, and if we continue to lose the U.S. public will eventually demand that we quit Iraq, and it will then become Afghanistan-on-steroids, which will threaten everyone. Third, he needs to say he will be guided by the U.N. in forming the new caretaker government in Baghdad. And fourth, he needs to explain that he is ready to listen to everyone's ideas about how to expand our force in Iraq, and have it work under a new U.N. mandate, so it will have the legitimacy it needs to crush any uprisings against the interim Iraqi government and oversee elections and then leave when appropriate. And he needs to urge them all to join in.

Let's not lose sight of something as bad as things look in Iraq, it is not yet lost, for one big reason: America's aspirations for Iraq and those of the Iraqi silent majority, particularly Shiites and Kurds, are still aligned. We both want Iraqi self-rule and then free elections. That overlap of interests, however clouded, can still salvage something decent from this war if the Bush team can finally screw up the courage to admit its failures and dramatically change course.

Yes, the hour is late, but as long as there's a glimmer of hope that this Bush team will do the right thing, we must insist on it, because America's role in the world is too precious to America and to the rest of the world to be squandered like this.

Your comments please...

tomder55 answered on 05/06/04:


Mostly this is alarmist nonsense .America is doing way too much hand ringing and self flagellation for my taste. We are in danger of paralysis by analysis. So much good has been accomplished in Iraq that only wavering on our commitment can cause our defeat.

I do Not think we need to grovel to the Security Council or any U.N. function at all . All our critics in the Security Council stole fortunes from the Oil for Food program. I would further add that if the U.N. was serious about the sanctions, there would be no Iraq war to debate today. We are the friends of the Iraqi people trying to free them from a tyrant; not the U.N.

As to the role of the UN in forming a new Iraq gvt. : We have already unfortunately brought in Lakhtar Brahimi who personally delivered Lebanon to Syrian annexation and tyranny in 1989 and said Israel was "poisoning" the Middle East to mediate a transfer. Perhaps he will also carve up Iraq also ;1/3 to Iran ,1/3 to Syria, and the Kurds to Turkey. That is his type of solution. He certainly will not work to bring democracy there.

Sit down and eat crow to Syria? Is he joking? If my war plans were in effect from the beginning we would've invaded from Lebanon through Syria in the North and Iraq from the South. Many of the insurgents holed up in Fallujah are Syrian nationals.

I may or may not agree that Rummy has to go. I thought that the post war planning was poor, and this scandal may very well tip the scale for him. There are others in the Administration I would like to see go first. Start with, and primarily George Tenet. He has been wrong on almost every important intelligence estimate for the last decade. I would also like to see many of the State Dept. cold war mentality hacks retired. The way they see the world; playing a Kissinger realpolitk game may have been ok or even necessary in the past (I am undecided if it didnt cause more harm than good. Probably confronting the Soviet Union directly after WWII would have been less painful), but there is no justification for playing those games anymore .Bushs basic Wilsonian tendencies are correct. If democracy can be established, then the Middle East will be a better place, and the world a safer place.

chekhovToo rated this answer Excellent or Above Average Answer
excon rated this answer Excellent or Above Average Answer

Question/Answer
chekhovToo asked on 05/05/04 - Censorship


The Walt Disney Company is blocking its Miramax Films division from distributing Michael Moore's documentary Fahrenheit 9/11, which criticises President George W Bush's handling of September 11 and connects the Bush family with that of Osama bin Laden.

Moore attributes Disney's decision to concerns that the documentary will endanger tax breaks the company receives from Florida, where Bush's brother Jeb is governor.

"I would have hoped by now that I would be able to put my work out to the public without having to experience the profound censorship obstacles I often seem to encounter," Moore wrote today in a statement on his website.

The filmmaker did not immediately respond to a request for comment from The Associated Press.

Miramax confirmed that Disney told the company it can't release the film.

"We hope to amicably resolve the situation in the near future," Miramax spokesman Matthew Hiltzik said.

Disney representatives Zenia Mucha and John Spelich did not return calls early today.

Disney has a contractual agreement with Miramax principals Bob and Harvey Weinstein allowing it to prevent the company from distributing films under certain circumstances, such as an NC-17 rating which bars anyone under age 17 from being admitted to the film, or a budget of more than $US30-35 million ($A41-48 million).

"Some people may be afraid of this movie because of what it will show," Moore wrote. "But there's nothing they can do about it now because it's done, it's awesome, and if I have anything to say about it, you'll see it this summer - because, after all, it is a free country."

According to The New York Times, which first reported the story, Fahrenheit 9/11 describes decades-old financial links between the Bush family and prominent Saudi Arabian families.

The film says the government helped members of bin Laden's family leave the United States after the terrorist attacks of September 11, 2001.

Moore won an Oscar for his 2002 documentary Bowling for Columbine, about the Columbine High School shooting and US gun control policy.

The film earned $US21.6 million at the box office, making it the highest-grossing documentary ever. He is also known for the 1989 film Roger & Me, which explored the effects of General Motors on his hometown of Flint, Michigan.

Fahrenheit 9/11 will have a high-profile screening as one of 18 films in competition at the Cannes Film Festival, which begins May 12.

Moore ran into similar interference with his book Stupid White Men, which almost never made it to print. Publication was postponed after September 11, and publisher HarperCollins considered cancelling the book or editing its criticisms.

After lengthy discussions, Stupid White Men came out uncensored. It almost immediately sold out a first printing of 50,000 and went on to top the New York Times nonfiction best seller list.

AP

Comments?

tomder55 answered on 05/06/04:

It is not likely that Moore will have a problem marketting this film . In fact I think the decision was made months ago by Disney(back in May 2003) ,and Moore has waited until just before the Canne film festival to stir this pot. Maybe he thinks he can simulate Mel Gibson's coup of going independent; or maybe he is desperate to get it released before the elections so he can impact them ? This really cracks me up . He has a whole film that will essentially say that the Bush family has been friendly with the Saudis for years . Duuuhhhhh ya think? Then he probably (this is only speculation) makes a leap from that to conclude that Bush and OBL were in cahoots over 9-11 ;or maybe OBL is on the secret Bushie payroll. He covered most of this topic in 'Dude ,Where's my Country?'

chekhovToo rated this answer Excellent or Above Average Answer
elgin_republicans rated this answer Excellent or Above Average Answer

Question/Answer
Doc05 asked on 05/05/04 - The Children's Peace Committee

In the late 70's or early 80's, there was a group of children, led by a ten year old girl, who formed a committee and called it The Children's Peace Committe. Did anyone on this board ever hear of the young girl or the Committee? I recall that they raised funds for the poor all over the world and were always on TV. I just wondered whatever happened to them. I understood at that time that they were also very vocal at the UN and in politics.

tomder55 answered on 05/05/04:

This is the only referencce I found about the organization :

In 1979, Monique Grodzki was a nine-year-old growing up in suburban New York when she happened to see a television documentary on the plight of Cambodian refugees. While moved by the tragedy of it all, Monique was more powerfully struck by the peoples undaunted spirit. " I found it amazing that they were still singing and had the hope and strength to go on. After I saw that, I actually cried. I really had to do something about it."

Monique was also moved by the discovery that children suffer most from hunger and from war. In her view, children should have a say about wars, about government: "They should have a say about things in general, about things that affect them." Monique resolved to give children a say in establishing the priorities of the world. "Were putting millions and billions of dollars into nuclear armaments. Wouldnt it be better to have a stop world hunger race than a nuclear armament race?"

The Children's Peace Committee was the result of Moniques resolve: its purpose to "help abolish world hunger and promote world peace." By 1982, a growing membership of two hundred children aged ten to thirteen were actively engaged in a variety of activities. Theyve addressed the general public at the Dag Hammarskjold Plaza of the United Nations, circulated petitions, collected money and sent it to organizations fighting hunger in the field. Looking closer to home, they raised money to buy bullet-proof vests for police officers in New York, and contributed to the family of an officer killed in the line of duty. More important than the specific activities of Monique and her friends is the simple fact of their willingness to take responsibility for something much bigger than anyone would expect a to comprehend. If nothing else, it makes it that much harder for adults to feel too small to make a difference.

Wadsworth Sociology Resource Center

Doc05 rated this answer Excellent or Above Average Answer
stevehaddock rated this answer Excellent or Above Average Answer

Question/Answer
XCHOUX asked on 05/04/04 - Bush Outed a CIA Operative

What possible rationalization can the Bush Administration make for outing a CIA Operative? His own father declared that type of behavior is unacceptable!

tomder55 answered on 05/05/04:

The premise of the question is flawed . It is probable that the information was leaked ,and there should be an investigation about it . Outing an intelligence person ,even if they only do clerical work(which is not the case Mrs.Wislon was an analyst of some degree or another) invites a compromise in the Agency .But I am more inclined to believe that 'factions ' in the White House who are opposed to the Bush foreign policy ;especially Iraq, and are more than willing to take steps to undermine the policy and are the culprits .(see below for more detail)

Bob Novak opposed the Iraq war from the git-go ,and is a snake anyway. Wilson is a State Dept. hack who made no bones about his opposition . I posted during the 'yellow cake ' controversy in Breaking News Board that Wilson spent much of his time sitting in hotel rooms in Niger sucking mint tea instead of determining if they had supplied Saddam with nuclear material .

Here is part of my posting :

Wilson was an against U.S. military intervention in Iraq from the get-go . He's a member at the Middle East Institute http://www.mideasti.org/ which advocates for Saudi interests. The March 2002 issue of the Saudi government weekly Ain-Al Yaqeen lists the MEI as an "Islamic research institutes supported by the Kingdom."He wrote in the March edition of the Nation magazine, that Bush has "imperial ambitions." http://www.thenation.com/doc.mhtml?i=20030303&s=wilson .According to Wilson , the world worries that "America has entered one of it periods of historical madness."

Ironically prior to the U.S. invasion of Iraq, Wilson did believe that Saddam had biological weapons of mass destruction. He raised that possibility to argue against toppling Saddam, warning ABC's Dave Marash that if American troops were sent into Iraq, Saddam might "use a biological weapon in a battle that we might have. " He added that Saddam also might attempt to take revenge by unleashing "some sort of a biological assault on an American city, not unlike the anthrax, attacks that we had last year." He also admitted to Sean Hannity in October that
Saddam had a WMD development program that included nuclear weapon development . Here is the relevant part of the interview :
HANNITY: if he's going after weapons of mass destruction, with his willingness to use them, and your belief, have they having told you they'd use them, isn't it important we get him before he got these other weapons? Wouldn't that be ...
WILSON: Well ...
HANNITY: Wouldn't that make good sense in your mind?
WILSON: Well, first of all, Sean just a second. Just let me be very clear. He has chemical weapons.
HANNITY: Right.
WILSON: He has biological weapon capability ...
HANNITY: Right.
WILSON: ... and may have biological weapons. He doesn't have nuclear weapons, to the best of anybody's knowledge, even though he has an aggressive program to try and get them.





Self-Fulfilling Prophecy
State vs. Iraq planning.By Michael Rubin

XCHOUX rated this answer Excellent or Above Average Answer
stevehaddock rated this answer Excellent or Above Average Answer

Question/Answer
XCHOUX asked on 05/04/04 - Accusations of Brutality Exaggerated

So, the Iraqi prisoners were stacked up naked like cordwood with hoods on their heads and photographed? Big deal. Our soldiers are being killed over their. What better way to get intelligence from the other prisoners than to show them a picture of a plie of anonymous naked guys especially with a woman making fun of them? They weren't hurt.

It's a war!

tomder55 answered on 05/05/04:

No actually it is deeper than that ;U.S. officials are investigating the deaths of at least 10 prisoners in Iraq and Afghanistan and are weighing a possible criminal investigation of a Central Intelligence Agency case officer in the death of one prisoner.

These actions were reprehensible and must be stopped immediately, and the perpetrators severly punished. There is a significant lack of training and supervision that must be corrected quickly .The reservists were men and women who for the most part had recently been converted from other military jobs to Military Police .The Iraq operations required a lot of MPs, and one of the traditional MP jobs is guarding prisoners. It was always feared that the newly trained MPs, lacking many experienced MP NCOs and officers to supervise them, would get into trouble. But unless they can prove that they were directly ordered to do this ,then I can't give them a pass . In fact ,the Military Code of Conduct demands that soldiers refuse to commit illegal acts if ordered to do so.

The soldiers involved in the prison scandal will be dealt with by the military. It's a sad testimony to see Brig. General Janis Karpinski, who was in charge of these prisons, attempt to pass the buck rather than take some responsibility for those under her command.

A number of American soldiers began to complain to their superiors that the methods used on the Iraqi prisoners seemed to be excessive, if not illegal. Investigations were being conducted and those found violating the army regulations were being identified and prosecuted under military law well before the al-Jazzera affiliated CBS aired the photos. The uproar over the excessive interrogation methods will lead to numerous restrictions on interrogators and much more scrutiny of how interrogations are conducted. The interrogations will be less productive for a while. As a result of this, attacks on coalition troops will be more frequent and successful. In war, information is a matter of life and death. So more Americans will die as a result of all this. Few will raise the issue in these terms. Instead, much will be made about how unnecessary and counterproductive the brutality was.

What is truly unfortunate is that those responsible for real atrocities like those committed in Fallujah continue to roam free and most likely will never face justice.The Arab media made much of the brutality of American interrogators, but said little about Saddams decades of interrogations in the Arab style. Apparently if Arabs are brutal to Arabs, it isnt news. But if non-Arabs do it, its a war crime.
Let's not allow the media to make more of this than there really is. We already know many in the Arab world hate us. If we allow this to turn into a "PC" war, they will no longer fear us. They need to learn that bad things will happen to them if they mess with the United States. War is hell.

ETWolverine rated this answer Excellent or Above Average Answer
XCHOUX rated this answer Excellent or Above Average Answer
chekhovToo rated this answer Excellent or Above Average Answer
stevehaddock rated this answer Excellent or Above Average Answer

Question/Answer
excon asked on 05/03/04 - Ok, let's put it on the table - Arabs suck


Hello again you right and left wingers:

Attitude!! Bigotry!!!

I dont suggest that what happened at the prison is the result of written policy. However, I am suggesting that our collective racism has been allowed to undermine that policy. Plus, I further suggest, that the Bush Administration is at the root of this racism.

Its not overt racism either. Were too smart for that its covert (the worst kind). We still speak democratic platitudes out of the right side of our mouth, while the left side think Arabs are scum. And, I personally, understand it. Im mad as hell at the Arabs and have been for years. Im a Jew.

But, as a nation, we need to rise above our collective racism. If we dont actively, positively, and purposefully train our prison guards in appropriate behavior, then we encourage inappropriate behavior. If we dont consider our prisoners (all of them) to be worthy of the rights spelled out in our Constitution, then we foster that racism (and I dont care where a particular person was born).

We should not have looked for reasons make up a new classification (enemy combatant) of prisoner specifically to ensure that they have no rights. Instead we should have looked for reasons to include them. We dont have to look far The Geneva Convention, The Constitution, The Trials at Nuremburg. We should have shown the world what democracy really means - that even our most hated enemy is treated fairly.

But we didnt. So, Im not surprised that soldiers, who were given no guidance, acted the way they did. But they were carrying out your and my policy, werent they?

excon

tomder55 answered on 05/04/04:

Sudan won an uncontested election on Tuesday to the United Nations' main human rights watchdog (Human Rights Commission), prompting the United States to walk out due to Sudans ethinc cleansing in the Darfur region.

This from Human Rights Watch World Report 2001 :

The government of Sudan remained a gross human rights abuser, while rebel groups committed their share of violations. In the seemingly endless seventeen-year civil war, the government stepped up its brutal expulsions of southern villagers from the oil production areas and trumpeted its resolve to use the oil income for more weapons. Under the leadership of President (Lt. Gen.) Omar El Bashir, the government intensified its bombing of civilian targets in the war, denied relief food to needy civilians, and abused children's rights, particularly through its military and logistical support for the Ugandan rebel Lord's Resistance Army (LRA), which held an estimated 6,000 Ugandan children captive on government-controlled Sudanese territory. As for the Sudan People's Liberation Movement/Army (SPLM/A), the principal armed movement of the south and of all Sudan, its forces continued to loot food (including relief provisions) from the population, sometimes with civilian casualties, recruit child soldiers, and commit rape. On both sides, impunity was the rule.
Sudan's human rights record of gross abuses was one factor in the General Assembly vote in October that denied a Security Council to Sudan, nominated by the Organization of African Unity, and instead granted the African seat to Mauritius.

Or how about this from Amnesty International last week (April 30,2004)?

Most villages in Darfur have now been destroyed and the population hardly dares to leave the displacement camps. The Janjawid (government-supported militia) block the roads and even invade the camps. In Ardamata camp for displaced people near al-Jeneina town, Janjawid are reported to enter openly and choose women to rape.

Furthermore, the conflict is in danger of spreading. On 28 April Sudanese planes bombed Kolbus village in Chad and the Janjawid attacked refugees and Chadian civilians across the border.

"This is not an unavoidable ethnic conflict. It is a tragedy deliberately created by the government's support for the Janjawid and fuelled by total impunity for grave violations of human rights", said Amnesty International.

Sudan's delegate immediately shot back that the U.S. delegation was "shedding crocodile tears" and turning a blind eye to atrocities committed by U.S. soldiers in Iraq against civilians as well as against prisoners.

Ha ! what a comparison ! To date over 2 million have been killed in Sudan .Not quite a comparison to U.S. troops posing naked prisoners in a pyramid ;and yet it is exactly this type of comparison that type of juvenile college initiation like prank invites .

ETWolverine rated this answer Excellent or Above Average Answer
excon rated this answer Excellent or Above Average Answer

Question/Answer
XCHOUX asked on 05/03/04 - Improving Middle Eastern Problems

with the Iraq War. There is a school of thought that the war with Iraq will have a quicker resolution of George W. Bush is not reelected.

Bush is hated in the Middle-East. The terrorists are fighting agaist Bush for his war on Afghanistan and Iraq.

Thoughts?

tomder55 answered on 05/03/04:

I don't know how that ended up as a clarification ,but consider it my answer also.

stevehaddock rated this answer Excellent or Above Average Answer
XCHOUX rated this answer Excellent or Above Average Answer

Question/Answer
excon asked on 05/02/04 - Iraq aka Viet Nam

Hello experts:

I served five and half years in the military. I was supposed to be released on my 21st birthday in 1964, but with two weeks to go, I got a telegram from the Defense Department. They told me they needed me in a place called Viet Nam. Now, where the hell was that? Well, I found out.

After my return home, my support for the war was unfailing. In the summer of 68 I went to a peace march in LA to see what the fuss was about. I didnt go to march. I wore my uniform (w/medals) and stood on the sidelines. The cops still beat me up.

Shortly thereafter, I was watching the evening news. They gave the obligatory daily body count 47 American dead, 748 Viet Cong killed. Then they went on to the Paris peace talks. They were not talking about peace, however. No, the topic at hand was the shape of the table they were going to be seated at. Not how may people were killed that day or how to stop it, but they were discussing the shape of ****ing table.

47 of my brothers - dead on the battlefield, and all they could talk about was the shape of the table. Hmmm. This is insane. I didnt want to be on their side anymore, and I havent been.

The War in Iraq has all the components of Viet Nam, only compressed in time. It even has its own brand of insanity "the re-Baathification of the de-Baathified.."

We have lost our way.

excon

tomder55 answered on 05/03/04:

Here is where the 2 wars are similar . The overall makeup of these guerrillas is unclear. They include Baathists, Al Qaeda operatives, and foreign fighters from Iran, Syria, and Saudi Arabia. They have found common cause in seeking to disrupt and end the American presence. Given the numbers and sophistication of US forces in Iraq, it seems improbable that they can ever achieve that through conventional warfare . But the philosophy behind guerrilla warfare is that military action against superior forces is an means to achieve political goals.In Iraq, as in Vietnam, the aim of the guerrillas is to undermine US resolve to the point where the US withdraws its forces, awarding the guerrillas the victory they could not otherwise win. The forces attacking U.S. troops appear to be well trained and they're able to find cover in a civilian population. They are counting on American popular support of the war to waver . They want the Democrat Party opposition in the House to deny funding like they did at the end of the Vietnam war ,insuring that the Paris Peace treaty would fail . And yes ;some of the political decisions being forced on the military seem wrong ,and not in the interest of our goals.
But there are more real differences than similarities .The real enemy in the Vietnam War was the Chinese/Soviet alliance.. There is no superpower that is backing these minority of Shias and Sunnis who are seeking to gain political power through violence . We aren't fighting proxies in the Middle East. Iraq is the primary battle ground for now against our primary enemies ; Islamofascist terrorists, whether religious or secular.

There is no comparison as far as the sanctuary which this enemy has. Throughout the entire Vietnam War the North Vietnamese Army held its territory under the cover of a nuclear-armed Soviet Union. And the Vietcong used the North as a base for insurgency in the South. There is no Iraqi equivalent. Coalition forces hold almost 100 percent of the ground. That ground was taken easily. Jihadists may find limited sanctuary in neighboring states like Syria and Iran . But Syria is not North Vietnam. It is not a client state of a superpower. If Syria pushes too hard, regime-change in Damascus is viable.

Iraq is a lot less violent. More than 58,000 Americans were killed in Vietnam. Most of Vietnam is covered in triple-canopy jungle. Iraq is flat desert. In Iraq, the opposition is confined to a relatively small geographic area. Much of the country is free of attacks against US and British forces. We can track people and movement from outer space. Our satellites can read license plates. We can see in the dark and strike bunkers from hundreds of miles away.
Guerillas and terrorists can hide in cities, but they are widely despised almost everywhere..

In Vietnam, the US fielded a conscript Army. In Iraq, the volunteer US forces are better trained and experienced.

Saddam Hussein invaded Iran and Kuwait. He threatened to attack Saudi Arabia. He promised to "burn" Israel. He was the patron of international terrorists. He was a sworn enemy of America. And he is unique among dictators for producing WMD and then deploying them for genocidal reasons . The reasons for being involved in Vietnam were less clear . The U.S. could afford to run away from Vietnam because the Vietcong were only a threat to Americans so long as we stayed. Many of the terrorists in Iraq are a part of the same murky network that murdered thousands of Americans in the last 20 years . There is new evidence (see below case closed)that Saddam Hussein was connected with Al Qaeda as far back as the early 1990s.

Both Baathists and Islamists are gunning for us now in Iraq . And if we let them drive us out, it will be one of the greatest propaganda coups for terrorists ever(greater than the Soviet withdrawal from Afghanistan). The Iraq War was not one of choice, though its opponents present it as one. Having driven the Taliban out of Afghanistan after 9-11, regime change in Iraq was absolutely necessary if the United States was to continue the transformation of the entire Middle East from being a constant threat to Western civilization. The Middle East, if this policy is continued, will be composed of modern nation states. The despots will have been driven from power. That will be our real security.Waiting to be attacked again is not an option.

case closed

excon rated this answer Excellent or Above Average Answer
voiceguy2000 rated this answer Excellent or Above Average Answer
elgin_republicans rated this answer Excellent or Above Average Answer

Question/Answer
ETWolverine asked on 04/26/04 - To Chou, a question about environmentalism:

Chou,

Do you believe that it is the job of government to make laws that protect birds and their habitats? Is it the government's job to protect the nests and eggs of, say, bald eagles? And if so, how should the government enforce those laws. What penalties would you recommend for those who break those laws?

Elliot

tomder55 answered on 04/26/04:

The U.S. Fish and Wildlife Service was proposing that the bald eagle be declared fully recovered by July 2000, but the decision was delayed until they figure out how they will manage the species once it is taken off the list. Even if the U.S. Fish and Wildlife Service removes the bald eagle from the "threatened" species list, it will still be protected .

State governments traditionally have been the chief stewards of wildlife within their borders. The federal government generally has intervened only to protect imperiled species, manage federal lands and enforce international agreements. One way to encourage effective federal conservation strategies is to encourage states to assume greater responsibility for protecting threatened and endangered species.However ,wildlife migrations are not confined to State boundries. .Regional cooperation should be encouraged ,or when necessary Federal regulations mandated .

This is not a liberal issue ,but a conservative one . The President with the greatest understanding of the environment was Teddy Roosevelt . One thing to consider ;With the exception of overhunting ,or population sprawl ,anything else that is killing off animals is probably not good for people either .In our area a dead crow is a sure sign of West Nile Virus.

Many provisions of Federal law actually harm wild life . These should be looked at closely ,and where applicable they should be eliminated . Examples of this can be found in subsidies for livestock grazing , and mining , on Federal lands .270 million acres of rangeland managed by the Bureau of Land Management and the , U.S. Forest Service are set aside for livestock feeding For feeding on public lands, ranchers pay only a fraction of what they would if their animals grazed on private property. Poor grazing management on federal land contributes directly to the decline of roughly a fourth of all species listed under the Endangered Species Act.

United States mining policy is also at odds with both the nation's economic and conservation goals. The General Mining Law of 1872, signed by President Ulysses S. Grant ,and still in effect, gives mining companies first claim on federal lands and then allows them to extract minerals without paying the government any royalty whatsoever. Not only is the environmental damage caused to public lands by mining techniques shocking, but the law also passes the cleanup tab right back to the taxpayer. Many of the designated Superfund sites, including some of the largest in the nation, are either abandoned or active mines.

Intead of subsidizing bad behavior ,incentives to promote good land use by private landowners are needed.Tax breaks for wetland preservation makes much more sense to me than tax breaks for those who pollute or who's activities ultimately threaten wildlife .

ETWolverine rated this answer Excellent or Above Average Answer
stevehaddock rated this answer Excellent or Above Average Answer

Question/Answer
XCHOUX asked on 04/24/04 - Free Radio Airways

I have been listening to the new AirAmerica radio station, most specifically, but not exclusively, "The O'Frankin Factor", and I realize how thoroughly and negatively the right wing radio shows are impacting America. Now, I have the chance to hear individuals like Bob Woodward interviewed at LENGTH. I get to hear facts and opinions that I can evaluate myself. In addition, natural interactions among intelligent and funny people, not shouting lies and generalities ad nauseum.

I highly recommend listening to this show for people who are interested in issues in depth, and enjoy a good back-and-forth discussion instead of lying, yelling, and spinning on the radio.

Of course, comments welcome...

tomder55 answered on 04/25/04:

I too listen to him . I do not see the improvement from the first week . He is a bomb thrwer from the left ,like Rush is a bomb thrower from the right.
Neither interview people who have opposing views of theirs ,so all you get is cheer-leading by both.
Air-America should model their shows after their successful counterparts on cable t.v.;the John Stewart'Daily Show' ,and
the Bill Maher ,'Politically Incorrect'. Both shows entertain ,and in the process advocate a liberal agenda.
They should also listen to Alan Colmes,who's success is attributed to a more moderate ,rational ,liberalism.

XCHOUX rated this answer Excellent or Above Average Answer

Question/Answer
ETWolverine asked on 04/23/04 - G-d, Mom and Apple Pie

Hello, everyone.

I happened to be contemplating the saying "G-d, Mom and Apple Pie" last night and something occured to me.

There is NOTHING in that saying that true about the USA anymore.

G-d: the 'sepparation of Church and State' crowd is actively trying to eliminate G-d from any public venue in the USA. In fact, if they could get rid of Him altogether, they would.

Mom: given the recent issues being raised regarding gay marriage in recent months, a child is just as likely to grow up with 2 dads and no mother at all.

Apple pie: sorry, but the low-fat, low-cholesteral, low-carb, sugar-free, dietetic version just doesn't do it for me.

So what are our current values in the USA, now that the old ones are no longer considered 'in vogue' anymore?

Any opinions?

Elliot

tomder55 answered on 04/23/04:

Do not lose faith there are many people in the country who still hold onto those values (btw ;you forgot baseball). Today it was reported that former NFL star Pat Tillman died in action in Afghanistan . He willingly gave his life to serve .He believed in America. Tillman's story had gotten little press, but it's not the media's fault. Tillman had not granted a single interview since his enlistment. Apparently determined that his endeavor not be construed as self-aggrandizing or insincere, he had simply left behind the fantasy world of sports to serve his country. What could be more important than riches and fame? Why sacrifice when our culture so often portrays sacrifice as the preserve of misfits and losers? Friends said that the 9/11 terrorist attacks had affected him deeply. Cardinals defensive coordinator Larry Marmie, after a conversation with his former player, said Tillman felt he needed to "pay something back" for the comfortable life he had been afforded. Whatever his rationale, he clearly was serious about his pursuit. He and his brother Kevin completed basic training in July and advanced through individual training in October. They graduated from parachute school in November, and completed the Ranger Indoctrination Program in December .This is what Peggy Noonan wrote when he turned down a multi-million dollar contract with the Arizona Cardinals to enlist in the Army .

We are making a lot of Tillmans in America, and one wonders if this has been sufficiently noted. The other day friends, a conservative intellectual and his activist wife, sent a picture of their son Gabe, a proud and newly minted Marine. And there is Abe, son of a former high aide to Al Gore, who is a lieutenant junior grade in the Navy, flying SH-60 Seahawk helicopters. A network journalist and his wife, also friends, speak with anguished pride of their son, in harm's way as a full corporal in the Marines. The son of a noted historian has joined up; the son of a conservative columnist has just finished his hitch in the Marines; and the son of a bureau chief of a famous magazine was commissioned a second lieutenant in the Army last month, on the day he graduated from Princeton.
As the Vietnam-era song said, "Something's happening here." And what it is may be exactly clear. Some very talented young men, and women, are joining the armed forces in order to help their country because, apparently, they love it. After what our society and culture have been through and become the past 30 years or so, you wouldn't be sure that we would still be making their kind, but we are. As for their spirit, Abe's mother reports, "Last New Year's, Abe and his roommate [another young officer] were home and the topic came up about how little they are paid [compared with] the kids who graduated from college at the same time they did and went into business.
"Without missing a beat the two of them said, 'Yeah--but we get to get shot at!' and raised their beer bottles. No resentment. No anger. Just pure . . . testosterone-laden bravado."

Hebrews 10:34-39 "For ye had compassion of me in my bonds, and took joyfully the spoiling of your goods, knowing in yourselves that ye have in heaven a better and an enduring substance. Cast not away therefore your confidence, which hath great recompence of reward. For ye have need of patience, that, after ye have done the will of God, ye might receive the promise. For yet a little while, and he that shall come will come, and will not tarry. Now the just shall live by faith: but if any man draw back, my soul shall have no pleasure in him. But we are not of them who draw back unto perdition; but of them that believe to the saving of the soul. "

ETWolverine rated this answer Excellent or Above Average Answer
stevehaddock rated this answer Above Average Answer
excon rated this answer Excellent or Above Average Answer

Question/Answer
XCHOUX asked on 04/22/04 - Horrible Train Wreck

There was a horrible train wreck in North Korea near the border. A train with Kim Il had just passed through the area when a train loaded with oil crashed into another train with another ingredient(I forget).

Here's to you brave men of the CIA?

tomder55 answered on 04/23/04:

Typical CIA to blow it by 9 hrs .
From April 10-16, Cheney traveled through China, South Korea and Japan, discussing issues of common concern, including the "war on terror", Seoul's troop dispatch to Iraq, and, of course, North Korea's nuclear-weapons program. During his stay in China, the smiles at photo sessions and declarations of solid relations may have masked tougher talk behind closed doors.

Shortly after Cheney left China on April 16, the Chinese invited Kim Jong-il to visit Beijing for high-level talks with Hu Jintao. The "secret" meeting was one of the most heavily monitored events ever involving Kim. South Korea's national broadcast KBS broke the story of Kim's train passing into China late Sunday. Pyongyang's official Korean Central News Agency reported that Kim had indeed visited China, shortly after his train had returned to North Korea, and shortly before the explosion.

US is seeking more Chinese pressure on N Korea .Cheney made it clear that the United States is pursuing all options in an effort to end the nuclear impasse, and China needs to be far more proactive in ending the problem than it has so far demonstrated. China's role in North Korea's stability and the regime's survival is well known . China supplies an estimated million tons of oil a year. China has restricted energy exports to North Korea in the past. Last spring, in response to North Korean missile tests in the Sea of Japan ,China closed its last remaining pipeline into North Korea. China said the pipeline was closed for maintenance, but it was reopened three days later. The message was clear: Energy supplies can and will be used to force North Korea's compliance and accepted standards of international behavior. North Korea agreed soon afterward to six-party summit talks held in Beijing. The talks involved both Koreas, the US, China, Japan and Russia.

The removal of Kim would usher in the possibility of change and peace on the peninsula. Of course, with a system such as that of North Korea, a viable opposition is simply impossible, and there are no political groups waiting in the wings to take the reins and move the country forward . But there is a military structure which could accept responsibility for managing the nation in a post-Kim era. If we are going to speculate on the endless possibilities ,why not include :1.S.Korea 2.China 3. coup-de-tat as possibilities ?




stevehaddock rated this answer Excellent or Above Average Answer
XCHOUX rated this answer Excellent or Above Average Answer

Question/Answer
XCHOUX asked on 04/22/04 - The Big Lie

Politics is a dirty business. I know this is not a news flash to anyone here. I want to share with you what is the big lie in Americal Politics. It cannot be refuted.

Liberals overwhelmingly outnumber conservatives in the American populace. When asked in pubic opinion surveys, liberals support clean water, air, pollution control of all kinds. This issue is on the liberal agenda. Americans support helping the elderly, disabled and children who are slipping through the safety net of dignity and income. And HEALTHCARE care, every American deserves healthcare.

And, Bush's Compasionate Conservatives....those people are "liberals" He adopted that catchphrase in the last election to solildify his liberal supporters.

NO wonder Americans are so confused; lies thrown at them all the time...A rose is a rose is a rose......

Cordially, Chou

tomder55 answered on 04/22/04:

Yes ,it was a catch phrase to shore up the center in the 2000 election .It was always an idiotic expression that was all spin. To his opponents ,he simply has lied ;or as Al Franken says :

HE LIIIIIIIIIIIIEEEEEEEEEDDDD!!

But does anyone really know what he meant by Compasionate Conservatism? I don't .
Was it a campaign slogan ? Was it a governing style ? I recall that he had Dem. leaders into the White House early in his Presidency .He's dine with fat Teddy,and Tom Daschel. That didn't last too long.Did liberals get upset when he wouldn't compromise on core social beliefs? Was it just that he steadfastly stands by right to life issues even though he has not actively used the bully pulpit to champion them ? The idea that Bush would retreat from basic positions was largely an invention of the national media. Because most journalists assume that most Americans share their views on social issues, and that the Republican positions are therefore wrong , they think "compassionate conservatism" is the opposite of moral conservatism.

Bush advisers say loss of standing on the compassion measures is a byproduct of the emphasis on terrorism and foreign policy. In
all fairness ,and to the chagrin of many conservatives he has delivered some policy that is not traditional conservatism. education, Medicare prescriptions, help for homeownership and AIDS treatment and education reform have all been passed and advocated by him . Still he has been steadfast in issues that divide;taxes,abortion rights,gay marriage,gun rights,death penalty etc.

An area where he has fallen flat is in environmemtal issues. It would take too large a posting to list how he has screwed that up .Suffice it to say that years of progress have stalled needlessly .With the exception of the Clean Water Act which the courts have interfered in ,while to his credit he has continued to enforce ,and his correctly opposing Kyoto ,his policies have not reflected someone who understands the larger issues at stake. Enforcing existing CAFE standards instead of letting them roll back for SUV lovers ,would be more beneficial to our energy independence and the environment than all the oil he could suck out of ANWR .

However ,the real issue on compassionate conservatism is that he has not met the expectation of the media pundits . It is a question to be decided in Nov. wheter he has not met the expectation of the" overwhelming outnumbers "of Americans.


XCHOUX rated this answer Excellent or Above Average Answer
stevehaddock rated this answer Excellent or Above Average Answer

Question/Answer
CeeBee2 asked on 04/21/04 - Wearing the flag

from Bill Moyers' new book, Moyers On America --

I wore my flag on air tonight. Until now I hadn't thought it necessary to display a little metallic icon of patriotism for everyone to see. It was enough to vote, pay my taxes, perform my civic duties, speak my mind, and do my part to raise our kids to be good Americans, as they are. Sometimes I would offer a small prayer of gratitude that I had been born in a country whose institutions sustained me, whose armed forces protected me, and whose ideals inspired me; I offered my heart's affections in return. It no more occurred to me to flaunt the flag on my chest than it did to pin my mother's picture on my lapel to prove her son's love. Mother knew where I stood; so does my country. I even tuck a valentine in with my tax returns on April 15.

So why wear it? Well, I put it on to take it back. The flag's been hijacked and turned into a logo: the trademark of a monopoly on patriotism. On those Sunday morning talk shows, official chests appear adorned with the flag as if it were the Good Housekeeping Seal. During the State of the Union address, did you notice Bush and Cheney wearing the flag? How come? No administration's patriotism is ever in doubt, only its policies, and the flag bestows no immunity from error.

Most galling are all those moralistic ideologues in Washington sporting the flag in their lapels while writing books, running Web sites, and publishing magazines attacking dissenters as unAmerican. They are people whose ardor for war grows disproportionately to their distance from the fighting. They belong to the same league as those swarms of corporate lobbyists prowling Capitol Hill with flags in their lapels, trolling for tax breaks even as they call for more spending on war.

So I put on the flag as a modest ripost to men with flags on their lapels who shoot missiles from the safety of Washington think tanks, or argue that sacrifice is good as long as they don't have to make it, or approve of bribing governments to join the coalition of the willing. I put it on to remind myself that the flag belongs to the country, not to the government; that one is not unAmerican to see war--except in self-defense--as a failure of moral imagination, political nerve, and diplomacy. Come to think of it, standing up to your government can mean standing up for your country. (February 2003)

What do you think?

tomder55 answered on 04/22/04:

"True believers in the god of the market would leave us to the ruthless cruelty of unfettered monopolistic capital where even the law of the jungle breaks down."

-- Bill Moyers, Keynote speech to the Environmental Grantmakers Association, October 16, 2001


Maybe he s comfortable wearing an American flag ,but Im sure he would be more so if he was wearing a hammer and sickle,or at the very least a blue U.N. lapel. Moyers is dedicated to promoting the views of most extreme elements of the far left in America. Moyers is a member of that narrow, unrepresentative elite group that has spent hundreds of millions of dollars over the last decade trying to ram leftist propaganda down the throats of Americans trying to advance an anti-capitalist, anti-American agenda; All while having his commentary ,and documentaries funded by public financing .

Moyers used "Now",his PBS show that will air until after the Nov.Elections to provide a platform for the very activists he funds but rarely acknowledged the financial support he provides his guests . Bill, foments disharmony and political tension. If he worked for a privately funded media outlet I would defend his right to opinions different from mine. But PBS receives public funds. That means that taxpayers who span the political spectrum pay his wages. We do not exist simply to cast money toward PBS and its obvious left-wing bias while he piously slams our beliefs, our values and our way of life.

In a speech a year ago, Moyers bashed the Heritage Foundation for pressuring the Bush administration, on behalf of wealthy bankers, to halt the global campaign to freeze terrorists financial assets. Bill Moyers has learned well from his comrade George Soros prime mover and shaker of Move On .org . In addition to sitting on the board of Soros Open Society Institute, Moyers is president of the Florence and John Schumann Foundation from which position he funds the radical Left. Bill Moyers, and the Schumann brothers, hate the free market . They spend millions inherited from an IBM founder and a president of the General Motors Acceptance Corporation to tear down America's free market economy.

Moyers history of fully disclosing the existence of these sort of relationships is spotty.Sometimes this is disclosed in the credits at the end of a program, though not always. During a one-hour special on campaign finance reform, three of the people Moyers interviewed were from organizations that received funding from the Schumann Foundation Moyers lists the Center for Responsive Politics as a source for one of his campaign finance stories on the "Now" series, and gives the group "special thanks" for their help on "Trade Secrets." But Moyers neglected to mention the millions the center has gotten from his foundation over the past decade, or that the center received a two-year grant for $1 million from Schumann.

Moyers is anti-corporate, complaining that "for years now the ruling ideology, the ruling religion of America, has been free markets. Its god is profit. Its heaven is the corporate boardroom. Its hell is regulation. Its Bible is the Wall Street Journal." Despite Moyers' antagonism for corporate boardrooms, his foundation maintains a stock portfolio of American businesses including companies Moyers has attacked on PBS. The foundation owns 3,400 shares of Dupont and 18,300 of Nova Chemicals.

This man is a Christian-basher and a Jew-basher. If a Christian or Jew takes his or her religion seriously, he or she is seen as a threat to Bill Moyers. He equates the Christian Right with Osama bin Laden, the Taliban .Moyers then adds some slurs against Jews. "Within all of the great Abrahamic traditions there is a violent tendency, a violent strain," . At the same time, he sees Muslims caught up in a "possibly quite hopeful inner debate.

After the mid term elections in 2002 Moyers ranted on the PBS web site .Among his charges he claimed :

Bush has acquired "the power of the state to force pregnant women to give up control over their own lives."

Would use the taxing power to transfer wealth from working people to the rich."

Would select "judges with a political agenda appointed for life. If you liked the Supreme Court that put George W. Bush in the White House, you will swoon over what's coming."(as if both parties arent guilty of that he also failed to mention the vicious campaign waged against Bush court nominee Miguel Estrada, first Hispanic nominated for the D.C. Circuit Court of Appeals by the left )

Complained that they dont mind the GOP being refered to as the party of God
Again all of this while being subsidized by taxpayer money.

I do not mind this type of politicking on network or cable tv . He certainly has a right to his opinions .I think he should be upfront about it ,and stop trying to hide behind journalistic integrity ,or impartiality .Rush Limbaugh and Sean Hannity do not hide their beliefs ,but at the same time they clearly do a commentary show ,and not a investigative journalism show ;and neither get a dime of my money .Moyers in ideed an American ,and has the right to wear the flag. I guess you can love your country ,and still despise what it stands for . That seems to fit where Moyers stands .




purplewings rated this answer Excellent or Above Average Answer
CeeBee2 rated this answer Excellent or Above Average Answer

Question/Answer
XCHOUX asked on 04/20/04 - Second Term Presidents

Since President Eisenhower, second term Presidents have been largely ineffective because they have spent their second terms defending scandals and being a lame duck from day one of their second term. Nixon, Reagan, Clinton...

Do you agree that it is probably better to vote for Kerry based on the obvious situation that is developing for Bush over the Iraq War?

tomder55 answered on 04/20/04:

Not when you consider that single term Presidents are usually disasters. Nixon had Watergate ,but Reagan and Clinton were pretty effective ;even with stuff like Iran Contra ,and Clinton's impeachment occupying a significant part of their time .Obviously in hind site it appears that Clinton did not do enough to combat the growing threat of terrorism,but neither did Carter ,Reagan,and Bush I .During Reagan's 2nd term the Soviet Union collapsed. Clinton's policies kept the economic recovery going ,and erased the deficit .

With as much as I disagree with Bush on many issues ,I cannot see us electing Kerry ,and the way he sees national security .When push comes to shove ,he'd lead us into the spider hole.

ETWolverine rated this answer Excellent or Above Average Answer
stevehaddock rated this answer Above Average Answer
XCHOUX rated this answer Excellent or Above Average Answer

Question/Answer
XCHOUX asked on 04/18/04 - Buckle Your Seatbelts,

It's going to be a bumpy ride to election day. INformation gathered and released today by Condi Rice has AlQuaeda, all plumped up from their success in controlling the cowardly Spanish people's election, are going to do the same to America in order to get Kerry elected. I guess the expectation being that Kerry will withdraw from Iraq and go soft on the War on Terror. I'm so disgusted at the thought that these arrogant male Arabs think that they can control the civilized world representing a squalid, pathetic, sewer-bound, underachieving culture

When the heck are we going to get tough on these degenerates??

tomder55 answered on 04/19/04:

Why wouldnt they attempt something similar here ? Spains new P.M.Zapatero announced within hrs. of being sworn in that he was withdrawing Spanish troops . Kerry seems to be molded from similar metal as Zapatero. Zapatero has managed to put the forces of the coalition countries with smaller numbers of troops in grave danger. al Sadr has asked his followers to stop their attacks on Spanish soldiers. "We are appealing to them to guarantee the safety of the Spanish troops until their departure, as long as they don't commit acts of force on the Iraqi citizenry", said Kai of El Chazaali, a close friend of Sadr,"The other countries which have sent soldiers to Iraq under the auspices of the coalition are called upon to follow Spain's example and to withdraw their troops to protect the life of their soldiers."

Zapatero said that he wouldve left the troops there under U.N. auspices . Kerry on Meet the Press yesterday told Tim Russett: ``I will immediately reach out to other nations in a very different way from this administration. Within weeks of being inaugurated I will return to the U.N. and I will rejoin the community of nations.''
Last week Kerry said :The goal here in my judgment is a stable Iraq," he said, "not whether or not that's a full democracy.I can't tell what it's going to be;but a stable Iraq. And that stability could take several different forms." President Bush's Iraq policy is revolutionary because it overcomes the idea that Arabs do not desire democracy and aren't capable of running a government based on popular elections. What Mr. Kerry is embracing here is the idea of a strongman;the notion that it's acceptable and possibly even preferable for a strongman to emerge and keep a lid on the chaos. This is what led the U.S. to back Saddam Hussein in the first place . Instability was a great threat because it offered the Soviet Union an opportunity to use insurgents to topple an unstable regime and set up client states. It was always a bargain with the devil as stability came with a price. In Iraq that meant a strongman who oppressed his people and ultimately turned on the United States.

As far as I am concerned, any further blood shed by Americans in Iraq from terrorists and thugs is on the hands of John Kerry. He is undermining the President every step of the way and now the Spanish are echoing Kerry with the reason they are withdrawing from Iraq is because the UN is not involved. Speaks volumes about Kerry when a known Socialist (Communist) who now heads Spain echoes the words of Kerry.

The Islamo-fascists know what they want ; an Islamic world of the 8th century, parasitic on the resources and technology of the 21st, by which all the better to destroy a soft and divided West. And if the present chaos continues, they are apparently on the right track. Osama bin Laden is making the US and European peace movement an instrument of his strategy. Bin Laden offered of a truce to Europe from attacks on April 15.Initially European leaders rejected it ,but the attack on the Spanish rail system ,and the subsequent election results have proven that Western politcs can be influence with acts of terrorism.

ETWolverine rated this answer Excellent or Above Average Answer
purplewings rated this answer Excellent or Above Average Answer
XCHOUX rated this answer Excellent or Above Average Answer

Question/Answer
XCHOUX asked on 04/10/04 - AIRAMERICA

Follow-Up. I have been listening to Al Franken a little everyday because I love his sense of humor; satire and sarcasm are my specialty, too. I like to bring down the pompous and priviliged whenever I can!

Anyway, I accidently heard about 15 minutes of the show that led into his, and all I have to say is PATHETIC with a capital P. The two women hosts were illogical and god! just plain stupid; what a mess. No brain cells working there.

Anyone else horrified by these two?

tomder55 answered on 04/11/04:

I've listened to him on and off for the last week.Can't say I agree with much of what he says
Anyway ,to have some fun with this Liberal Radio experiment(which I do not believe will last past the
election unless they continue to get funding from groups like Moveon.org)
Here are some useful slogans for Air America:


Rush Limbaugh is a big, fat idiot, and other meaningful, thought-provoking discussions.

Radio Lib -- You thought you could hide this 'radio' thing from us forever, huh?

Liberal Radio -- It's not what you know, it's what you can say about other people.

Left Radio: If you don't like the message, we'll get a court somewhere to make you listen.

It's like NPR with more commercials.

Left Radio -- Everyone else is a liar.

Saving radio from mindless blowhards with nothing of substance to say by bringing out-of-work, liberal comedians to the airwaves.

Liberal Radio -- Soon to be taxpayer-supported!

Don't confuse me with stuff like facts...Left Radio.

Have we mentioned Rush Limbaugh is a tubby, drug-addicted liar? No, really.

Left Wing Radio. If this doesn't work, we're trying the wireless telegraph next!

Left Radio: can you hear me now?

Air America: now that's true air pollution.

Al Franken: talent on loan from Gollum.

Garafalo: talent on loan from Al Franken.

Left Radio: Masturbation for the mind
Air America: Your Terrorist loving airline

Liberal Radio: Capitalism Sucks, advertise with us!

Al Franken: I have Rush Limbaugh Envy

Air America: Usurping all the small minority-voice stations and turning them into one liberal voice

Listen to Air America...it beats working

Left Radio: Why bother thinking when you can hear our feelings?

Air America, we give you one promise: We will be the first to blame America first.

Liberal Radio - Fact or Fiction? You decide

Liberal Radio - Hot Airwaves.




Meet our new Director of Public Policy...Howard Stern.


XCHOUX rated this answer Excellent or Above Average Answer

Question/Answer
excon asked on 04/10/04 - Rush is not the only hypocrite.


Hello sperts:

Billions spent on the Drug War!!! Millions of lives ruined!!! Yet, when it comes to GWs political agenda, Zalmay Khalilzad, the U.S. Ambassador to Afghanistan, said politics require [us] not to go to hard with [poppy opium] eradication.

This doesnt embarrass me - Im against the drug war. But, you must be mortified. Unless, of course gade is right, were all hypocrites.

excon

tomder55 answered on 04/11/04:

I don't think this is hypocracy . We have urged Hamed Karzai to address the problem ,and he started taking steps last week to do so.

Kabul - Afghan President Hamid Karzai on Tuesday called for a 'jihad', or holy war, against the country's growing narcotics trade, saying the drug industry threatened the stability of the government.

In his first press conference since returning to Kabul following an aid meeting in Berlin, at which donors pledged $8.2bn to help rebuild the war-shattered nation, Karzai called on tribal elders to struggle against the narcotics trade.

"Narcotics is one of the things which threatens our dignity, our economy, our agriculture. It threatens our government and our roots - and it is against our religion," he said.

"Narcotics are very bad for Afghanistan's name, and the profits go to the mafia," Karzai said, adding that his administration would attempt to combat the problem, whether it received international assistance or not.

"This is a widespread jihad which covers the entire country."

Heroin

Afghanistan is the world's biggest producer of opium, used to make heroin, and according to UN reports, this year's poppy crop is expected to be the largest ever.

Karzai admitted that Afghanistan had made mistakes in combatting drugs in the past and now had to increase its efforts to remove the scourge.

"We have made some mistakes," he said, saying that a plan to pay farmers, whose poppy crops were destroyed by government officials, had only encouraged people to plant the lucrative poppy.

"That cash encouraged farmers to cultivate poppy this year," he said. "The farmers said: 'If the government destroys our land, it will pay money and if it doesn't destroy the land you have your poppy field'."

Karzai praised the Berlin meeting, at which donors pledged more than $4bn for this financial year and $8.2bn over the next three as a "great success".

"We thank the world for their kind donations for Afghanistan," he said.

However, in accepting this money, Afghanistan also accepted responsibilities - these included eradicating the narcotics industry, removing weapons from irresponsible groups or warlords, and using the world's donations responsibly and transparently, he said.

"I don't see any other problem in Afghanistan but misuse of guns by irresponsible groups and bad opium cultivation. For the future of Afghanistan ... struggle against these two phenomenon is very necessary."

http://www.news24.com/News24/World/News/0,,2-10-1462_1508980,00.html

excon rated this answer Excellent or Above Average Answer
stevehaddock rated this answer Excellent or Above Average Answer

Question/Answer
excon asked on 04/08/04 - Rush Limprod is goin to jail


Hello Experts:

Rush Limbaugh loves the government at the very same time the Justice Department is trying to stick it up way inside him. How can he possibly do that? How can he believe his own schtick anymore? More importantly, how can anybody else still believe him?

Frankly, I have to smile when I hear of his troubles - not because of his legal troubles (I hate the DEA, and Im proud to say it), but because I love it when hypocrites get their due.

excon

tomder55 answered on 04/08/04:

The seizing of his medical records was no different than the break in of Daniel Ellsburg's Psychiatrists office. Florida prosecutors admitted in February that without Limbaugh's medical records, they have no other prosecutable evidence against him. Other charges they floated and had to retract are of his trafficing ,and money laundering .

I thought that being involoved in this case would serve to temper some of his more unsympathetic rhetoric against drug addicts ,but from the times I've heard his show,it evidently is not the case.

excon rated this answer Excellent or Above Average Answer
stevehaddock rated this answer Excellent or Above Average Answer

Question/Answer
excon asked on 04/06/04 - Yours and Bush's war, Mr. ETWolverine


Hello Elliot (and the rest of you miscreants):

Its time to apply the Q and the B word to Iraq. I listened to you, Elliot, and in my heart, I hoped you were right. But in my brain, I knew you were wrong. Quagmire and bogged down are the two words Im thinking of.

Our differences: 1) I distinguish between Iraq and Afghanistan. To you, its all the same war. 2) You see us as liberators, but the Iraqis (and I) see us as invaders. They did not attack us, nor did they plan to. Plus, they had no capacity to attack even if they wanted to. Hell, even the Shiite, whom we expected would throw flowers in our paths, hate our guts. 3) Your friend Bush, wants them to have elections before they write a constitution. Really? 4) Your friend Bush wants to turn government over to the un-governable in 3 months time. Surly, hes kidding. 5) Now that were there, well be there for generations. If we leave anytime soon, the killing in the Balkans will appear tame. Civil war will erupt and will engulf the entire region. 6) Your friend Bush maintains that he has enough troops to finish the job. Poppycock!

Were vulnerable all over the place and were getting our asses handed to us. In my view, I think were on the verge of a world war. The entire region will become embroiled and were stuck in the middle. Iraq IS a job that we would have eventually had to undertake in any case, but it should have been on OUR terms, and it isnt.

Nobodys going to join the Reserves anymore, and our regulars are stretched too thin. Korea can tell us to go to hell, and they did just that.

Were in deep, deep doo-doo and your guy put us there.

excon

tomder55 answered on 04/07/04:

Problems with our conduct post war .

1 we failed to shoot looters.
2 we failed to have trials and executions of those who still oppose us .
3 we did not make enough of an effort to disarm the old Iraqi Army,
4 we allowed private Armies to be formed.
5 we did not stop border incursions .We still do not hold bordering countries like Iran ,Syria,and Saudia Arabia accountable for the prevention of jihaddists crossing their borders .
6 we set a deadline for transfer of power .(possibly the biggest mistake ) The president unwisely reaffirmed his commitment to the June 30 date. Of the many lessons Vietnam taught us , or should have ,one of the most important is that if you establish a schedule, it's not just yours: It's the enemy's as well.
7 we shouldve hand picked an interim executive president (I wouldve picked the biggest meanest Kurd)
8 from the beginning ,there shouldve been a heavy visible price to pay for attacks against the coalition .I favor the Israeli bulldozer method.
9 We shouldve taken out the Iran nuclear facility along time ago.
10 We shouldve taken out al Sadr when his supporters killed the Ayatollah Muhammad Baqer al-Hakim(with the support of Iran ,and Hezballah)and more importantly when his supporters hacked to death the Cleric Abdul Majid al-Khoei . He of all the clerics could've convinced the people that our goals are good for them .
11 We went in with too few troops . Yes I know that no armored invasion was ever done faster ,but
we did not have enough second wave troops to clean up urban centers ,and to act as the new police /security force.
12 Proper equipment was not provided for . It is a disgrace that our troops have to provide for their own body armor
13 why isnt the $18 billion in reconstruction money appropriated being spent? The reason: The politicians don't want another "Halliburton" campaign issue, and are trying to award the contracts as they would in peacetime, and it's taking far too long. Thousands of Iraqis who could be employed building their nation now are unemployed. They have nothing better to do than sit around and listen to the mullahs preaching violence against Americans.Haliburton knows how to do this , They were the contractor of choice for the Clinton Adm. As well as Bush .

Our policy toward the radical mullahs has to be consistent and firm. Those who plan and organize terrorism, like Sadr, must be arrested and imprisoned. Because you call yourself a cleric doesn't give you immunity from capture and punishment if you're organizing violence. Just as the Israelis were right in killing Yassin, the "spiritual leader" of Hamas, we have not only the right but the obligation to arrest and silence those Iraqi "spiritual leaders" who are organizing and inciting murder.

ETWolverine rated this answer Excellent or Above Average Answer
excon rated this answer Excellent or Above Average Answer
XCHOUX rated this answer Excellent or Above Average Answer

Question/Answer
XCHOUX asked on 04/04/04 - John Dean

Like me, I bet there are a lot of people here who remember John Dean while watching those Watergate hearings daily on television in the Seventies. He was quite the witness stating that he told Nixon that there was a cancer on the Presidency. Of course, he went down with Nixon.

Now, he is on television promoting his new book. He states that Bush and his administration have committed impeachable offenses regarding lying about the reasons for invading Iraq. That these reasons are more serious than the reasons that brought down Nixon.

Is the issue the right of an administration to secrecy regarding its actions and decisions. JUst how far can a President go before he crosses the line?

Will Bush be "impeached"in the election in NOvember?

tomder55 answered on 04/05/04:

There was an effort to impeach Nixon for his bombing of Cambodia . Nixon was charged with "false and misleading statements to the Congress" concerning that bombing. But the House Judiciary Committee's impeachment inquiry did not address the question of the president's lying, rather whether he had conducted an unlawful war. By a vote of 26 to 12 the committee decided Nixon had not committed an impeachable offense

ETWolverine rated this answer Excellent or Above Average Answer
XCHOUX rated this answer Excellent or Above Average Answer

Question/Answer
ETWolverine asked on 04/02/04 - Excon, XChouX, SteveHaddock: read this.

Friday, April 02, 2004

WASHINGTON U.S. businesses created jobs at the fastest pace in nearly four years as hiring increased across a wide array of industries in March, the government said on Friday in a report that stunned financial markets and provided long-awaited evidence of a job market recovery.





The report offers comfort to President Bush as the jobs market -- a hot political issue in the U.S. presidential campaign -- finally made a decisive break out of a long slump. Nevertheless, U.S. jobs lost since Bush took office still number a hefty 1.8 million.

Non-farm payrolls climbed 308,000 in March, helped a bit by the return of workers after a labor dispute at California grocery stores ended, the Labor Department (search) said. This was the biggest gain since April 2000 and well above the 103,000 rise expected on Wall Street.

The civilian unemployment rate, however, ticked up 0.1 percentage point from 5.6 percent in February. That occurred because more job seekers renewed their searches last month, but were unsuccessful.

"All in all, this is a very strong report," said Kurt Karl, head of research at Swiss Re in New York. "This is a number that everyone has been waiting for."

"It bodes well for the economy going forward," he said.

Economists said the report suggested the Federal Reserve (search) could raise overnight interest rates from the current 1958 low of 1 percent sooner than had been expected.

"I've been saying August for the first Fed hike, but maybe now it could even be in June," said Stephen Stanley, chief economist at RBS Greenwich Capital.

The department said the end to the California grocery store dispute, which had idled 72,000 workers, boosted March payrolls by 10,000 to 20,000. The impact was muted because many of the returning employees were displacing temporary hires.

January and February payrolls were revised upward a combined 87,000, contributing to the report's positive tone.

Job gains were widespread across industries.

Construction payrolls shot up by 71,000, a bounceback from a 21,000 decline in February many economists had pinned on bad weather.

Retailers added 47,000 workers, in part a reflection of the return of the idled grocery store employees.

While a long-hoped for rise in manufacturing employment did not appear, the department said factory payrolls were unchanged last month, finally breaking a string of 43 consecutive monthly declines.

Reuters and the Associated Press contributed to this report.

---------

Have you been convinced yet that Bush's tax cuts were right for the economy? Or do you need another brick to slam you on the head?

tomder55 answered on 04/04/04:

Many people do vote their pocketbooks and if the economy continues to picks up steam, the President's approval ratings will go up in most of the battleground states.


Twelve of the battleground states, according to the Wasghington Times, that likely will decide the outcome of the 2004 presidential campaign have unemployment rates below the national average..


The states with unemployment rates in the 4 percent range include: Florida, 4.6 percent; Iowa, 4.1 percent; Minnesota, 4.7 percent; Nevada, 4.4 percent; and New Hampshire, 4.2 percent.

States within the 5 percent range include: Arizona, 5.3 percent; Arkansas, 5.5 percent; Maine, 5 percent; Missouri, 5.1 percent; New Mexico, 5.6 percent; Ohio, 5.9 percent; Pennsylvania, 5.1 percent; West Virginia, 5.4 percent; and Wisconsin, 5.2 percent.

Only three big battleground states exceeded the current 5.7 percent national average: Michigan, 6.6 percent; Oregon, 7.1 percent; and Washington 6.1 percent.


ETWolverine rated this answer Excellent or Above Average Answer

Question/Answer
XCHOUX asked on 03/31/04 - airamericaradio.com

Hi,

I'm listenng to the first "The O'Franken Factor"(Al Franken)on 95 AM in Chicago, it is also on the above internet connection. This show runs opposite "The Rush Limbaugh" conservative radio show and bills itself as a liberal radio show. A little rocky at first, getting it together, and soon will take phone calls.

It is certianly refreshing to get different information and opinions via the radio. Senator Kerry was just on discussion the hearings about terror. Al Franken did a fab impression of Rummy. Asking rhetorical questions and then answering them.

Have a great day,
Chou

tomder55 answered on 03/31/04:

it airs here in N.Y. on WLIB (I kid you not).

I heard the opening salvo during lunch but the station has crappy reception . I was waiting patiently for him to say:"I'm smart enough; I'm funny enough; and doggone it, people like me" . How was Kerry . He's quite a bomb thrower on the 9-11 commision . I can't wait for Condi to go toe to toe with him .

ETWolverine rated this answer Excellent or Above Average Answer
XCHOUX rated this answer Excellent or Above Average Answer

Question/Answer
excon asked on 03/27/04 - The Pledge


Hello experts:

Before the words under God were injected into the Pledge (1954), it read quite smoothly and coherently from one thought to the next. It made sense. There were no contradictions. You didnt have to figure it out, because it meant what it said. It used to flow from one nation [to], indivisible, [to] liberty and justice for all. The last word is the most important.

When a sentiment, like under God is inserted, those who dont believe in God are not included in the word all anymore. But that cant be, because all means all - doesnt it? And if all doesnt mean all anymore, instead of removing the words under God we should change the word all to most. And, of course, well have to remove the word indivisible, because religion divides us (as it should we are diverse).

The Pledge of Allegiance, however, should be available for ALL of us to say, and NONE of us should have to mumble over ANY of its words.

Or am I nuts?

excon

tomder55 answered on 03/29/04:

Before the words went into the pledge, Jefferson, Washington & Lincoln used them.

On July 2, 1776, George Washington rallied his troops on Long Island to prepare for battle against the British, who had assembled on Staten Island. In his general orders to the troops Washington said: "The fate of unborn millions will now depend, under God, on the courage and conduct of this army." After the Declaration of Independence was signed he said to the troops: This important event will serve as a fresh incentive to every officer and soldier to act with fidelity and courage, as knowing that now the peace and safety of the country depends, under God, solely on the success of our arms."

Jefferson said :"God who gave us life, gave us liberty. And can the liberties of a nation be thought secure when we have removed their only firm basis, a conviction in the minds of the people that these liberties are a gift from God?

Lincoln resolved in his Gettysburg Address "that this nation, under God, shall have a new birth of freedom and that government of the people, by the people, for the people shall not perish from the Earth."

I would much rather that children learn American history than to learn to recite the pledge.

excon rated this answer Excellent or Above Average Answer

Question/Answer
XCHOUX asked on 03/25/04 - Homeland Security-Just Business as Usual??

"Time Magazine" this week:

Where are the highest risks for terorist attacks? According to AIR Worldwide Corp, New York is number one, followed by Illinois(Chicago-land) then California. They calculated the risk based on the potential insured losses by state.

And the MONEY goes to: North and South Dakota, Wyoming, Montana, Vermont. DC, and Alaska!! Distributing homeland-security money gives a disproportionate amount to less populated places!

Wyoming: the least populous state in the US..493,000 people is a wide open place and the capital does not even have an airport that can service jets.

Meanwhile, I practically SEE 500,000 people a day when I go out, and just south of where I live is a suburb with a Mosque that has had two terrorists-either sent to jail(he confessed!) or deported for funding terrorists!

Come on now!

Comments?........

tomder55 answered on 03/26/04:

but the cattle ,don't forget the cattle ! mad cow disease was terrorizing us for weeks last year !

when the cookie jar is full everyone wants to make sure they get theirs . I guarantee
that my Senators ;Chucky Shumer and
Hillary(Evita)Clinton will whine about it until N.Y. gets what it is due,and not just NYC mind you ;but the little village of Chappaqua (where the Clinton's nest)with it's Median household income: $163,201 and Median house value: $562,400 needs protection also.

XCHOUX rated this answer Excellent or Above Average Answer

Question/Answer
XCHOUX asked on 03/23/04 - Bush-Kerry Neck and Neck

A survey whose results were published today indicate that the race for president is in a virtual dead heat(Bush ahead by a couple of % points). Kerry is favored by voters who are concerned most about the issue of jobs in America. Bush by those who are most concerned about Foreign Policy.

Can Bush more realistically appeal to people who are VERY interested in the issue of jobs?

tomder55 answered on 03/24/04:

I don't know . I think he is very vulnerable in some important swing States in the rust belt . The Rust Belt will turnout for Democrats. Labor unions rallied around Gore in 2000 and will do the same for Kerry . Ohio has hemorrhaged jobs and was narrowly lost by Gore. Ohio, Michigan, Illinois and Wisconsin may go to Kerry also (maybe even Pennsylvania).Bush's attempt at protectionism(a campaign promise he had to renege on) with steel import tarrifs badly back fired .

He has little room to move on the economy. The deficit and financing the war on terror(he has no choice but to contine post war expenditures) prevents further tax cuts ,and the Fed. already has interest rates at historical lows . The best he can do is wait it out and hope that the recovery starts producing jobs. He has not moved to curb public spending ,and even a diehard Conservative has to admit that Bush has added to entitlement spending as much as any Liberal could possibly dream of . Social Security is collapsing before our eyes ,and all he has done is make weak statements about privatizing it .

stevehaddock rated this answer Excellent or Above Average Answer
XCHOUX rated this answer Excellent or Above Average Answer

Question/Answer
ETWolverine asked on 03/22/04 - The Myths of Unemployment in the Bush Presidency

Hello everyone.

There have been a number of statements put out by Democrats against Bush that have been used to bash Bushs economic policies. The vast majority of these statements are related to employment or job creation. The media has been repeating these claims and pundits have been playing them up with some success. But are these claims true?

I have done some limited research, and have found that there are a number of popular myths that are being bandied about. But the truth tends to be far different from what has been stated by the liberal media and the Democratic nominees and their supporters. Here are a few examples:

Myth #1:
There are fewer people employed today than there were in January 2001, when Bush took office.

The truth:
The number of people age 16 and over who were employed in January 2001 was 137,790,000 (adjusted for seasonality). In February 2004, that number was 138,301,000 (adjusted). This represents an increase in the number of employed people of 510,000, on an adjusted basis. On an unadjusted basis (raw numbers), the number of employed in January 2003 was 136,181,000, and the number employed today is 137,384,000. This represents an increased number of employed people of 1,203,000. Thus, in fact, there are 1.2 million more people employed today than there were in January 2001.

Myth #2:
The Unemployment Rate is up, which means that jobs have been lost.

The truth:
As seen in the answer to the first myth, this is simply not true. While it is true that the Unemployment Rate is currently 5.6% (as of February 2004) as compared to 4.2% in January 2001, unemployment is actually down from its high of 6.2% in July 2003.

Additionally, the unemployment rate does not reflect the number of people unemployed. It reflects the number of people unemployed as a PERCENTAGE OF THE TOTAL LABOR FORCE. The Labor Force is the number of people over the age of 16 who are available and willing to work. The Labor Force in January 2001 on a seasonally adjusted basis was 143,787,000. That number has grown over the past 3 years to 146,471,000, an increase of 2,684,000 people in the labor force. This mostly represents the number of people who have finished school and are now available for work, or who were not part of the labor force, but are now entering the labor force (for instance, former stay at home mothers who are now re-entering the work force). Thus 2.7 million more people became available to work in the past 3 years.

The number of unemployed at January 2001 (adjusted) was 5,997,000. When compared to the Labor Force at that time, the Unemployment Rate was 4.2%.

5,997,000 / 143,787,000 * 100 = 4.2%

The number of unemployed at February 2004 (adjusted) was 8,170,000. When compared to the Labor Force at that time, the Unemployment Rate was 5.6%.

8,170,000 / 146,471,000 * 100 = 5.6%

But here is the key point. Notice the difference between the numbers of unemployed at January 2001 vs. February 2004.

8,170,000 5,997,000 = 2,173,000.

A 2.2 million increase in unemployment. But the increase in the Labor Force was 2.7 million. Thus, despite a growth of people looking for jobs of 2.7 million, there was only an increase in the actual number of unemployed people of 2.2 million. Which means that 500,000 of those people entering the job market found jobs. And it means that the number of available jobs is NOT decreasing, but rather has increased by 500,000. Its just that the labor force increased at a greater rate. (This jives with the increased number of employed of 510,000, adjusted, discussed in the response to Myth #1.)

The bottom line is that the country is not loosing jobs, as some would like to claim. The problem is not a loss of available jobs. The problem is that the number of jobs is not increasing AS FAST AS the number of people entering the job market. So the idea that Bush has somehow caused a decrease in the number of available jobs in the USA is patently false. There has been significant job growth... just not at significant as the number of people looking for jobs.

Myth #3: Bushs policies are driving jobs overseas.

The truth:
As seen above, the number of employed people during the Bush Presidency has actually increased by 1.2 million unadjusted, 510,000 adjusted. While there are indeed a number of jobs going overseas, particularly to India, the fact is that there has still been a net growth in the number of employed people in this country. Bushs policies have done nothing to cause jobs to leave the country. And while we would like to see a greater increase in the number of available jobs, the idea that Bush is causing jobs to disappear is absurd.

Myth #4: Unemployment is at record levels.

The truth:
We are not at record high unemployment levels. In fact, we have not reached any kind of record unemployment during the Bush Presidency. Even in the months following 9/11 when unemployment rose dramatically, we did not reach such record levels. The highest level of unemployment came in 1982, during the Reagan era, when unemployment soared to 12.1 million. And in fact, unemployment during the Clinton era (supposedly the good years) reached a high of 10.0 million. So far, during the Bush Presidency, the highest level of unemployment we have seen was 9.2 million, in July 2003. And we have seen an improvement of 1.1 million jobs since then. So we have not been anywhere near the highest levels of unemployment in history, and we have seen tremendous improvement in just seven months. And our current unemployment rate of 5.6% is the same rate that existed in February 1996, the year that Clinton was re-elected.

Myth #5: The Bush Administration is only creating 29,000 per month, when they should be creating 250,000 per month.

The truth:
Who says we should be creating 250,000 jobs per month? And in fact, job creation has actually been much higher than that over the past few months.

Month/Year: Unemployed (000s): Job Growth (000s)
June 2003---------9,245----------------N/A
July 2003---------9,048----------------197
Aug 2003----------8,929----------------119
Sep 2003----------8,966---------------(-37)
Oct 2003----------8,797----------------169
Nov 2003----------8,653----------------144
Dec 2003----------8,398----------------255
Jan 2004----------8,297----------------101
Feb 2004----------8,170----------------127

Total Job Growth (8 months)-----------1,075
Average Job Growth (8 months)----------134.4

As seen above, job growth has averaged 134,400 per month over the past 8 months, and has totaled 1,075,000 in that time period. Thus, job growth has not been 29,000 per month as some have argued. It has been much higher than that for most months, and continues set a steady pace of improvement. And if job growth continues along the same pattern until the election (and there is no reason to believe that it wont), and if the Labor Force remains relatively level (and I dont foresee any huge shifts between now and then), unemployment will decrease by another 1.2 million by October, and the Unemployment rate will drop to 4.8%... the lowest level since July 2001.

Myth #6: No president has ever been re-elected with a net loss of jobs during his first term.

The truth: Reagan was reelected after just such a situation. Unemployment was 6.7 million when Reagan took office in January 1980. At October 1984 (the most recent information available to voters in the general election in November), unemployment was 8.4 million, a net jobs loss of 1.7 million. Yet Reagan was re-elected with the greatest landslide victory in history... he took 49 of 50 states.

Basically, what I have found is that the statements made in the media are pretty much false. The economy IS creating new jobs at an average rate of 134,400 per month. The number of employed people in the USA is up by 1.2 million (510,000 if adjusted for seasonality), and we are not at historic levels of unemployment. The rhetoric of the Democrats turns out to be, as usual, more myth than fact.

tomder55 answered on 03/23/04:

I am not an economics expert ,so I have to defer to what they are saying .Alan Greenspan last week said in testimony to Congress: "New hires and recalls from layoffs... are far below what historical experience indicates," "To a surprising degree, firms seem to be able to continue identifying and implementing new efficiencies in their production processes and thus have found it possible so far to meet increasing orders without stepping up hiring."He seems to be putting the blame on increased productivity due to business investing in their equipment .Which in itself is a good thing. Productivity typically leads to job growth and higher living standards in the long run.

Economists don't yet know why employment growth is slow but there seems to be a consensus that it is slow . Everything they know about the relationship between economic growth and employment suggests that employment should be stronger than it is. Economists continue to believe that the jobs will come if growth continues at its current pace.

Some of the discrepancy could be in the numbers being reported for self-employment . Self-employment is recorded, but only in the household survey, which asks a sample of people whether they are employed or not. Economists prefer data from the payroll survey, which comes from business employment records.

Other factors that could be contributing to a slow job growth are that businesses may not believe the recovery is permanent and are in a wait and see mode . The Tempo. Industry and outsourcing has allowed employers to increase output without hiring permanent employees . The growth in tempo jobs 32,000 overshadowed the national job gain total in Feb. The tempo job sector has grown more rapidly than any other, by 10.2 percent, since the official end of the recession in November 2001.

Many employers complain about the cost of benefits ; especially healthcare . Some employers are now arguing that they are at a competitive disadvantage compared with businesses in countries with national health insurance. In the United States, businesses mostly pay for health insurance out of sales, whereas in many foreign countries health coverage is paid for by through the tax system.

The reality is that job growth has not kept up with the Bush Administrations forecasts .They had predicted with alot of fanfare that there would be 150,000 new jobs in December . The reality is that only 1000 were reported . The Labor Department said that businesses created 21,000 jobs in February, far below the 130,000 economists had predicted .The Bush Administration had set the bar that they are being measured against .To meet the Bush administration promise to create 2.6 million jobs this year, 300,000 jobs a month would have to be added.Job growth has not come neat this expectation yet .

With a continued recovery the job situation could turn around . But the lack of new jobs could negatively affect consumer confidence ,and put the brakes on the recovery .


ETWolverine rated this answer Excellent or Above Average Answer
stevehaddock rated this answer Above Average Answer

Question/Answer
XCHOUX asked on 03/19/04 - Assasination Attempt!

We were discussing the political situation in Taiwan yesterday, and today's news, "assasination attempt on Chen and his VP candidate". Does this have CIA written all over it?

tomder55 answered on 03/19/04:

Doubt it ,they would not have missed.

Reportedly the bullet was fired from a crowd during a campaign motorcade . It was one of those magic bullets .It hit his VP in the leg first ,and then hit him in the stomach . Both injurues are not life threatening .

It is more likely that it came from a supporter of Lien. Chen is from the group of Taiwan 'natives' .His opponent Lien is from the "mainlander" minority who fled to Taiwan in 1949 after the Nationalist government lost China's civil war to Mao.Chen advocates independence from China while Lien favors a conciliatory approach .

ETWolverine rated this answer Excellent or Above Average Answer
stevehaddock rated this answer Excellent or Above Average Answer
XCHOUX rated this answer Excellent or Above Average Answer

Question/Answer
Fr_Chuck asked on 03/18/04 - Fishing license

I had the fun pleasure of getting my TN fishing license today, plus registering my boat and motor, buying flares, first aid kit and some other items.

I also got a copy of the boating and fishing rules and regs for the state, plus an attached pamplet with some of the federal boating rules in them.

Now I was just thinking, if George Washington, or Patrick Henry, was to come to visit today, and we explained to them, that no, they could not just walk down to the stream and go fishing. They first had to know what type of fish they were going to catch, to see if any special fish stamps were needed, then they had to get a permit from the government to fish, Next if they were going to use a boat, it had to have a license for it.

Would they beleive us, that the rules to fish and boat take up more space than the founding documents of our entire nation did.

Would they think it was all worth it, or would they throw all the work they had done out the window, and write up a much more speicific document granting and not granting the rights the way they intended.

Somehow I always beleived, that good ole George would have beleived, we could always go down and fish, without the government being able to control it.

tomder55 answered on 03/19/04:

no I think George would realize that back then was a simpler time ,and today it is much more complicated . Most rivers and lakes in this country would be lifeless were it not for laws and regs designed to control activities like fishing ,boating ,polluting .
I am not sure ,but I would bet that the lake /river you plan on fishing probably has been restocked ,with the fish you like to catch,from some hatchery ;and that it was probably the State that paid for the restocking. As far as boating goes; back in Washington's day ,you got a dingy and a couple of oars . Now on some lakes you could go across them without getting wet just by hopping from one power boat to another . You have the same right to pilot a boat as you do to drive a car . No one thinks an un-licensed ,uninsured motorist is being denied rights .

Back in 1794,a group of men from Western Pennsylvania farmers thought it was their right to drink tax free whiskey . Washington had a different opinion .He raised an army of 1200 militia ;personally donned his old General's uniform ,and led them to crush what became known as the Whiskey Rebellion .
As President ,Washinton was completly willing to use all the power necessary to enforce the tax laws ,and to tax and control activities deemed normal and natural for the greater public good .

ETWolverine rated this answer Excellent or Above Average Answer
Fr_Chuck rated this answer Excellent or Above Average Answer
stevehaddock rated this answer Excellent or Above Average Answer

Question/Answer
ETWolverine asked on 03/16/04 - Another interesting case of media bias

Here's another case of media bias. To be fair, this morning's NY Post did indeed pick up the story on its front page. But I doubt to many others did.

---------------

Hebron/Arutz7-Israel National News Commentary

The Little Shahid
by David Wilder
The Jewish Community of Hebron
March 16, 2004

Shalom.

What does the name Abdallah Koran mean to you? Anything? If not, it should. Abdallah Koran should be a banner-sized headline in all news publications around the world, hard-copy and internet.

Why?

Abdallah Koran is about 10 years old. His exact age is not 100% clear. Yesterday afternoon Abdallah, before making his way through an IDF checkpoint near Shechem, in Samaria, was made a tantalizing offer. Approached by some older people, Abdallah was promised a great sum of money if he would do them a small favor. They requested that Abdallah act as a messenger and take with him, along with his school books, a backpack for some people waiting on the other side of the checkpoint.

Of course, little Abdallah agreed. Why pass up a chance to make some good money so easily.

At the checkpoint a border policewoman, examining all those crossing over, became very suspicious. The backpack was big and heavy, much heavier than would normally be used by a ten year old. The youngster was quickly questioned What is in this backpace?

Never having been questioned by uniformed Israelis before, Abdaallah was frightened. Its not mine. Someone gave it to me to take across the checkpoint, as a messenger.

The backpack was carefully removed to an isolated area, where an army sapper examined it and proceeded to explode it. Inside the backpack was a 6 10 kilo (about 15 lbs) bomb belt, hooked up to a cellular telephone.

It seems that the terrorists who provided the bomb to little Abdallah had planned on blowing him up, together with the Israeli soldiers at the checkpoint, at the time of its examination. But the plan failed. The bomb didnt go off.

Had he managed to get it through the checkpoint undetected, it is most likely, according to Israeli intelligence-security forces, that the bomb would have been detonated on a bus packed with people, murdering Abdallah and the others on the bus.

As I wrote, you should know this already because this story should be headlining all news programs, radio, t.v., newspapers and internet. But, its not. As of this writing, this story does not appear on the homepage of CNN, Fox, or MSNBC.

Lets, for a minute, play one of my favorite games: Make Believe. Make believe, for a moment, that a ten year old from, lets say, Hebron, was found carrying a bomb on his back, attempting to kill some Arabs in Hebron. Or, maybe a little boy or girl from Yitzhar, who was trying to kill some Arabs somewhere in Samaria. Or, or or

Can you imagine the headlines. I mean, after all, one of the pictures of the year, a few years ago, was a photograph of a Hebron child pulling off an Arab womans headscarf. That was a major international crime which made blazing headlines. But, a 10 year old with a bomb, who was not only to be the killer, but also the target?! Why should anyone pick up the story? A 10 year old Arab with a bomb - that's everyday stuff. Whats it worth?

Only a couple of days ago 10 Israelis were killed in cold blood by two terrorists who managed to bypass the infamous wall or fence or call it whatever you wish the magical barrier which would prevent any such infiltrations from Gaza into Israel proper, in this case, the Ashdod port. That terror attack was billed as an almost Mega-attack, due to the proximity of the terrorists to poisonous bromide tanks, which, upon explosion, could cause the deaths of thousands of people. But, here again, the terrorist detonated himself too fast, for one reason or another, and the big tragedy was averted. This time. But, according to most news broadcasters, commentators and everyone else, it will (G-d forbid) happen. Its just a question of time. The Spanish attacks will seem like childs play in comparison.

In my opinion, mega-terror cannot be strictly measure by the number of people killed. Of course, numbers do mean a lot. But each and every individual person is just that, a person. And every loss is just that, a loss. To the families of victims, it is little consolation that their loved one was one of many. When Mom or Dad, Brother or Sister, Son or Daughter is gone theyre gone and it makes no difference how many went with them.

In my opinion, the attempted use of Abdallah Koran, an Arab 10 year old child, is Mega-terror. The vile, immoral, depraved use of a child to kill others, this is mega terror. Perhaps not in the quantitative sense, but certainly qualitatively.

Only the brilliant alertness of a young border policewoman saved many lives lives of Israelis and the life of little Abdullah. Otherwise Abdullah Koran would have been labeled, for the rest of eternity, the little Shahid the little martyr, who killed and died for his people, without his even knowing it, against his will.

With blessings from Hebron.

-------------

Here is the NY Post's coverage of the story:

TERROR FIENDS TRY TO TURN BOY INTO BOMB

By URI DAN

March 16, 2004 -- Palestinian terrorists yesterday stooped to a sickening new low - trying to turn this baby-faced 10-year-old boy into a human bomb by having him unknowingly smuggle an explosives-laden backpack past a West Bank checkpoint.
Impoverished little Abdallah Kora'an - working as a pint-sized porter at the Huwwara security checkpoint near Nablus - had been offered money by two men to lug what they hoped would pass as a schoolbag through the heavily guarded spot and into Israel, government officials said.

The lethal package - laden with nuts and bolts to make it even deadlier - was then to be handed to cohorts on the other side and used for a future homicide bomber in Israel, officials said.

But the terrorists had a chilling backup plan: If the boy was stopped and searched, they would use a cell phone to immediately detonate the 15 to 22 pounds of explosives, murdering nearby soldiers - and the innocent boy, officials said.

That's the scenario that the thugs were left with when an alert guard grew suspicious and stopped Abdallah for a search at around 6 a.m.

The terrorists feverishly tried to detonate the device while watching the scene unfold from afar. But they were apparently foiled by a glitch in their wiring, Israeli military chiefs said.

The officials said the rag-tag boy, who lives with his family in the Balata refugee camp, was simply an innocent player in the horrific scheme and had no idea he was carrying a bomb.

"[Someone] asked him to carry through a bag . . . and left. He just wanted to make money. He's just a poor kid," one official said.

Israeli officials said the boy told them he had been promised "a large sum of money" to deliver the bag to a Palestinian woman on the other side.

The boy regularly helps tote baggage for Palestinians not allowed to drive through the security check, officials said.

He also had his own bag filled with schoolbooks at the time.

As Abdallah began trudging through the security check, an eagle-eyed female soldier noticed that the backpack seemed unusually heavy for such a small boy.

When she told the increasingly nervous child she wanted to search it, he immediately confessed, "The bag is not mine. I was asked to carry it across."

The woman started to look in the bag and spotted a cell phone. That's when explosives experts were called in. They eventually safely detonated the package away from the scene.

Several of the terrorists who dispatched the boy have already been captured, Israeli sources said.

The government has blamed the Tanzim - Palestinian militants linked to the Fatah group under leader Yasser Arafat - for the plot.

But a senior leader with the terror group denied involvement.

"We do not use children," he said. "We do not lack in [adult] fighters. The aim of the Israeli army is to belittle the Martyrs Brigades. They are lying."

News of the plot came only a day after two 17-year-old Palestinian boys - juniors in high school - blew themselves up in Israel's main port of Ashdod, killing 11 people and injuring 18.

The killings prompted Israeli Prime Minister Ariel Sharon yesterday to nix any potential peace talks with the Palestinians in the near future. He charged that it's obvious they can't control the terrorists in their camp.

The Bush administration did not immediately respond to Sharon's announcement, but acknowledged that any peace efforts would be difficult with the ongoing terror campaign.

With Kate Sheehy in New York and Post Wire Services

------------

Elliot

tomder55 answered on 03/17/04:

It was also in the Guardian UK .; about 2 paragraphs worth . I forget the # s but I know that for every "successful"terrorist attack in Israel there are many more that
the IDF stops . Perhaps it is deemed unnews worthy . Words like despicable are understatements .I cannot find comparable words to describe the disgust I have at hearing how low these scum can go.

ETWolverine rated this answer Excellent or Above Average Answer

Question/Answer
ETWolverine asked on 03/16/04 - Your Opinions please---

I received this e-mail this morning from a website called Honestreporting.com. This is an organization that battles media bias against Israel. I had to reformat the letter because we can't post tables here. But the information is unchanged from what they sent me. Please let me know what you think.

----------

REAL PROGRESS ON THE 'T-WORD'

Dear HonestReporting Subscriber,

For over three years, in continual alerts and through TerrorPetition.com, HonestReporting has led the campaign to insist that news outlets call Palestinian terror "terror." (See our extensive webpage devoted to this issue.) Now, as the scourge of Islamic terrorism continues to spread throughout the globe, it is more important than ever that Israel's struggle against terrorism be properly identified as part of the larger battle to preserve civil, democratic society against militant Islam.

The past week saw the horrific bombing of commuter trains in Madrid, and the Palestinian terror attack at the Israeli seaport at Ashdod. While the bombings in Madrid were of greater magnitude in terms of human loss, in essence the two were very similar terror attacks targeting sensitive areas of national infrastructure with the goal of destroying the opposing society.

This time, while some news agencies continued to show a double standard vis-a-vis Israel, we're pleased to report that others are beginning to heed HonestReporting's insistent call to refer to Palestinian terror as "terror." Here's a review, starting with the duplicitous offenders:

News Agency: Associated Press
Coverage of Madrid bombings: Headlined 'Terror Blasts Kill at Least 198 in Spain'
Coverage of Ashdod bombings: Headlined 'Eight Die in Israeli Port Suicide Attack'

News Agency: Washington Post
Coverage of Madrid bombings: "Millions of Spaniards united... to denounce the terrorist attacks that killed nearly 200 people in the capital a day earlier."
Coverage of Ashdod bombings: "Two Palestinian suicide bombers blew themselves up at one of Israel's largest industrial seaports late Sunday afternoon..."

News Agency: BBC
Coverage of Madrid bombings: Interviewed politicians regarding the "Madrid terror attack"
Coverage of Ashdod bombings: Reported "the suicide blasts in the southern Israeli port of Ashdod."

News Agency: Agence France-Presse (AFP)
Coverage of Madrid bombings: "..investigators probed a claim that the Al-Qaeda network was behind the deadliest terror attacks in Spain's history."
Coverage of Ashdod bombings: "Two explosions in ... Ashdod were carried out in a joint operation by the hardline Palestinian groups Hamas and the Al-Aqsa Martyrs Brigades"

News Agency: LA Times
Coverage of Madrid bombings: "The body bags outside Madrid's Atocha train station and the commuters sitting stunned on the tracks were graphic reminders of terrorism's evil."
Coverage of Ashdod bombings: "Two Palestinian militant organizations, Hamas and the Al Aqsa Martyrs Brigade, claimed joint responsibility for the attack"

Meanwhile, the New York Times, CNN, The Christian Science Monitor and even London's The Guardian are to be commended for breaking from past policies and calling both attacks "terrorism":

News Agency: New York Times
Coverage of Madrid bombings: "when terrorists blew up commuter trains packed with run-of-the-mill people...suddenly the equation changed."
Coverage of Ashdod bombings: " If the terrorists did come from Gaza, south of here, it would be the first time in more than three years of conflict ..."

News Agency: CNN
Coverage of Madrid bombings: "One of the five men identified Sunday as suspects in last week's terrorist attacks in Madrid..."
Coverage of Ashdod bombings: " If the terrorists did come from Gaza, south of here, it would be the first time in more than three years of conflict ..."

News Agency: Christian Science Monitor
Coverage of Madrid bombings: "Terrorist Bombings Jolt Spain"
Coverage of Ashdod bombings: "It was not the first time terrorism exerted its veto power over attempts to lure Israelis and Palestinians back to discussions"

News Agency: The Guardian
Coverage of Madrid bombings: "...the terrorists behind the March 11 attack have ties to a radical Islamist group..."
Coverage of Ashdod bombings: "It was also the first time that militants from Gaza have staged a terrorist attack"

[And of course, the "news" agency Reuters held by their absurd editorial standard to refer to no attack as "terrorism" they called the Spanish bombs a "guerilla attack."]

We reiterate that this is not merely an academic, semantic issue. As the West unites against barbaric Islamic terrorism that now also haunts continental Europe, it is essential that Israel's struggle against Palestinian terror be properly identified as part of the larger battle. When news outlets differentiate between a port attack in Israel and a train attack in Madrid, they expose an editorial decision that the Palestinian attack is somehow more justified. That's wrong, dangerous, and far from "neutral reporting."

It is encouraging indeed that four major news outlets have responded to the hundreds of emails sent by HonestReporting subscribers, and have finally begun calling Palestinian terror "terror." Now is the time to write to the other news agencies above, encouraging them to join their colleagues in rectifying this longstanding anti-Israel double standard.

Comments to Associated Press: feedback@ap.org
Comments to Washington Post: letters@washpost.com
Comments to BBC: newsonline@bbc.co.uk
Comments to AFP: contact@afp.com
Comments to LA Times: letters@latimes.com


STUDY: SAN JOSE MERCURY NEWS

Kudos to HonestReporting subscribers Andrew and Michelle Gross, whose newly-released 5-month study of the San Jose Mercury News (available on the HR site here) focuses on precisely this problem of biased terror reportage when it comes to Israel.

The study found that during the period of August 1 ? December 31, 2003, 108 headlines and 125 lead paragraphs in the Mercury News applied the 'terror' term to specific acts of violence, aggressors, countermeasures, and legal procedures in China India, Indonesia, Iraq, Russia, Turkey, the United States, and elsewhere. But in reporting on nine major terrorist attacks in Israel during this time, the Mercury News never used 'terrorism' in a headline, and only once used the term in an actual article. The conclusion:

Mercury News terminology gives readers the impression that the global scourge of terrorism does not affect Israel. While the newspaper routinely identifies acts of terror and anti-terror actions around the globe, similar acts within Israel are not labeled as such. Moreover, while Al Qaida and allied groups are consistently identified as terrorist organizations, Palestinian groups with parallel goals, targets, and methods are not. This double standard should be unacceptable to a newspaper that declares as its mission, "reporting and writing accurately and fairly."

If you have observed a similar pattern in your local paper, use the Grosses' fine study as model to produce your own study. For more information, see the HonestReporting guide: 'Become a Media Patroller,' as well as HonestReporting's expose of the media's use of the "T-word."

Thank you for your ongoing involvement in the battle against media bias.

----------

Elliot

tomder55 answered on 03/17/04:

Instead of suicide bombers they should be called homicide bombers ,but I agree that there is a conscious decision in the press to separate International Terrorism with Palestinian terrorism (or as Pat Bucchannan calls it :freedom fighters).

The attack at Ashdod was a failed Chemical weapons attack . The two terrorists blew themselves up near tanks of bromide and other dangerous chemicals. They used high-grade explosives. If either of the men had gotten any closer to chemical tanks in the area, the blast would have released a cloud of deadly chemical gas.


How does the Gannet papers stack up ? My town has a large Orthodox population ,but I think Gannet is a national newspaper and information company.

ETWolverine rated this answer Excellent or Above Average Answer
purplewings rated this answer Excellent or Above Average Answer

Question/Answer
XCHOUX asked on 03/16/04 - Four Baptist Missionaries....

killed in Iraq. What on earth are missionaries doing there...yeah sure, working to help reconstruction, but proseletizers in Iraq at this time????? Allowing then to go there is a disgrace. Who allowed this. Of course, they are going to be assasinated.

Please, comments.

tomder55 answered on 03/17/04:

They were on a humanitarian mission -- a water purification project . The reality is that there are alot of civilian NGOs represented in Iraq .Why exclude ones affiliated with a religious organization only when many of them have an agenda that may go beyond pure humanitarianism?

XCHOUX rated this answer Excellent or Above Average Answer

Question/Answer
XCHOUX asked on 03/15/04 - Spain Pulling out Troups from Iraq

So, did the barbarians win one?

Disgusted, Chou

tomder55 answered on 03/15/04:

Spain's new leader Jose Zapetero said that Spanish troops would stay if the U.N. had a role . He wants 'cordial relations with the U.S. but 'fantastic' relations with France.AHEM maybe Zapetero and Chirac can share the same spiderhole.

ETWolverine rated this answer Excellent or Above Average Answer
XCHOUX rated this answer Excellent or Above Average Answer

Question/Answer
purplewings asked on 03/09/04 - What do you think about this?

At about the time our original 13 states adopted their new constitution,
in the year 1787, Alexander Tyler (a Scottish history professor at The
University of Edinburgh) had this to say about "The Fall of The Athenian
Republic" some 2,000 years prior.

"A democracy is always temporary in nature; it simply cannot exist as a permanent form of government. A democracy will continue to exist up until the time that voters discover that they can vote themselves generous gifts from the public treasury.

From that moment on, the majority always votes for the candidates who promise the most benefits from the public treasury, with the result that every democracy will finally collapse due to loose fiscal policy, (which is) always followed by a dictatorship."

"The average age of the worlds greatest civilizations from the beginning of history, has been about 200 years. During those 200 years, these nations always progressed through the following sequence:

From Bondage to spiritual faith;
From spiritual faith to great courage;
From courage to liberty;
From liberty to abundance;
From abundance to complacency;
From complacency to apathy;
From apathy to dependence;
From dependence back into bondage."

Professor Joseph Olson of Hamline University School of Law, St. Paul, Minnesota, points out some interesting facts concerning the most recent Presidential election:

Population of counties won by:
Gore=127 million
Bush=143 million
Square miles of land won by:
Gore=580,000
Bush=22,427,000
States won by:
Gore=19;
Bush=29
Murder rate per 100,000 residents in counties won by :
Gore=13.2
Bush=2.1

Professor Olson adds: "In aggregate, the map of the territory Bush won was mostly the land owned by the tax-paying citizens of this great country. Gore's territory encompassed those citizens living in government-owned tenements and living off government welfare.."

Olson believes the U.S is now somewhere between the "apathy" and "complacency" phase of Professor Tyler's definition of democracy; with some 40 percent of the nation's population already having reached the "governmental dependency" phase.

What can we do to change this prediction?
PW


tomder55 answered on 03/09/04:

(the majority always votes for the candidates who promise the most benefits from the public treasury)

If Tyler's theory was correct about the voters ,then Reagan would not have won 2 terms in the 80s . Bush I would not have won in 1988,and Bush II would not now be President. All of them ran on reducing the size of the government. Even Clinton tried to tie welfare benefits in such a way as to not make it a permanent entitlement.

purplewings rated this answer Excellent or Above Average Answer
stevehaddock rated this answer Average Answer

Question/Answer
XCHOUX asked on 03/07/04 - Immigration

Victor Hanson makes strong points about the failuare to control immigration, from our southern border. Immgration quotas were originated(for one) to control the number of groups from different countries so that they could assimilate into American culture.

California now has separate graduation ceremonies for Spansh speaking and Anglo students. (Some Universities) Young people are not prepared for life and working in America, so he says. That, coupled with the fact tha tere may be as many as 40% illegals in California creates a serious problem.

HOwever, Americans are rich and do not want to do jobs like cleaning a baby's iapers, cut their grass on and on.....

Whose political fault is this, Reps or Dems? or both?

Coments....

tomder55 answered on 03/08/04:

The fault lies in whichever Party is in power at the time . Last week Bush and Fox agreed to not using finger print or photo id. for Mexicans who routinely cross the border .I am all for legal immegration . A growing economy needs a labor force . For security reasons ,we must get control of the border . We also need a better extradition agreement with Mexico .The killer of Deputy David March should face trial for the murder in the United States .(see below )



------------------------------------------
Some Mexicans and Mexican-Americans want to see California, New Mexico and other parts of the United States given to Mexico. They call it the reconquista, Spanish for reconquest, and they view the millions of Mexican illegal aliens entering this country as their army of invaders to achieve that takeover.
A Latino student group(MEChA) that drew attention during the California gubernatorial campaign of Lt. Gov. Cruz Bustamante says it will not disavow a founding document outlining the aim of recapturing the southwestern United States for Mexico. Bustamante was criticized for his involvement with the group while a student at Fresno State University.


Ironically it was Mexico allowing immigrant settlers from the United States to go to Texas that eventually resulted in Mexico losing the American South West.


Here is the document El Plan Espiritual de Aztln


David March

XCHOUX rated this answer Excellent or Above Average Answer

Question/Answer
XCHOUX asked on 03/06/04 - Took The Bait??

There is talk in Chicago-land that Bush "took the bait" regarding the issue of marriage for homosexuals by proposing an Amendment to the Constitution to describe the specifics of marriage.

That this issue will lose him his second term in the White House.

My opinion, he is gong to lose on this issue if he pushes it.

Comments....

tomder55 answered on 03/06/04:

I don't think it will be a major issue. After 1 week it seems to be losing significant steam here in N.Y.
Jason West ,the mayor of the hamlet of New Paultz was performing gay marriages (solemnizing them [is that a word?])
Now he faces charges for violating N.Y. law. The State Att.General Elliot Spitzer has ruled that it is a violation of N.Y. Law to perform them .The mayor of another
village was going to do them ,but has backed off. Mayor Blumberg paid little atttention
to gay partners who went to City Hall to apply for marriage licenses. He has said he will enforce the law.However he is not opposed to
civil partnerships with equal rights .

I don't think the Amendment will get anywhere
(maybe as far as the ERA went)
I feel the ball is in the wrong court .
Gays should try to change the constitution
if they feel current laws are discriminatory.
There should not be an amendment supporting laws
already in effect in most if not all of the states.
The Constitution says in Article IV

Section 1. Full faith and credit shall be
given in each state to the public acts,
records, and judicial proceedings of every
other state. And the Congress may by general
laws prescribe the manner in which such acts,
records, and proceedings shall be proved,
and the effect thereof.

This takes the issue away from the States rights issue ,
and certainly there is no provision I know of that gives
municipalities the right to make laws that are counter to
State or Federal law.

XCHOUX rated this answer Excellent or Above Average Answer

Question/Answer
excon asked on 03/06/04 - Gay Marriage

Hello experts:

Is gay marriage a legal or a religious issue? Certainly, its a political issue. Im asking this same question on the law board as well as on the political board to see if the responses differ depending on ones particular expertise. If youre on those boards (as I know some of you are), feel free to answer from a different perspective if you have one. Personally, I have many.

However, I think its a religious issue. If it werent, there wouldnt be any fuss about it. Or would there be? Is the gene out of the bottle? What happens to those licenses already granted? Is it here to stay? Or has Armageddon arrived?

excon

tomder55 answered on 03/06/04:

I see it as a legal battle . There are no religious grounds for the gays . It is almost condemned by most major religions .

I have answered this question a few times on religion boards and the Spirituality board. If you don't mind;I'll cut and paste

If words can be redefined on a whim ,then words have no meaning . A marriage is the union between a man and a women . However ,I
am a believer of equal rights under the law.
My solution to this would be for civil unions ,both marriage and other be recognized by the State ,let marriage be a matter of church recognition .

I see no reason why a stable relationship between 2 consenting adults should not have the same legal status ,privilages and responsiblities regardless of the gender makeup of the coupling .We who speak of marriage being a stabilizing factor in society are living in the past . Over 50 % of marriages are failed leaving a terrible toll on children that is not healthy for society. Also ,there are many children orphaned for many reasons.Many gay couples have assumed their care and the responsibility for their well-being .The values that such gay couples exhibit in their daily lives are often indistinguishable from those of their straight neighbors. They're loyal to their mates, are monogamous (at least to the same degree as heterosexuals are) , devoted partners. They value and participate in family life, are committed to making their neighborhoods and communities safer and better places to live, and honor and abide by the law. Many make valuable contributions to their communities, serving on school boards, volunteering in community charities, and trying to be good citizens.Far be it from me to condemn them .Instead ,we should applaud their assuming the responsibility. Are we to then tell the children their 'parents' are bad ;or should not get the same financial benifits the tax codes provide for married couples? Should they not have visitation privaleges that any other spouse would have if one became infirmed ? Should they not be automatically given equal survivor rights,or health care(I put my wife on my insurance ;can they ) ?

As a Catholic ,I am very uncomfortable dealing with the issue . I am taught that it is wrong morally ,and I tend to agree that it is .Reality shows me something else. Scientific studies have shown that the outcomes of the children raised in the homes of gay and lesbian couples are just as good as those of straight couples. The differences have been shown again and again to be insignificant. Psychologists tell us that what makes the difference is the love of the parents, not their gender. The studies are very clear about that. And gay people are as capable of loving children as fully as anyone else.

I refuse to condemn someone who is acting more responsible then some of my brethren do ,and I certainly will not deny anyone equal rights just because I am uncomfortable with their life style.

excon rated this answer Excellent or Above Average Answer

Question/Answer
excon asked on 02/26/04 - co-incidence


Hello again,

Would you believe that I posted my question to you at the same moment you posted you question about the movie?

Interesting.

Personally, I think the world is just looking for a reason to come down on the Jews again.

But, I'm ready for 'em. Bring it on. I'm a soldier for David's army.

excon

tomder55 answered on 03/01/04:

It is my sincere hope ,and belief that this movie will not inspire the hatred you suggest .If it ever came to that ,I'm with you .

ETWolverine rated this answer Excellent or Above Average Answer
excon rated this answer Excellent or Above Average Answer

Question/Answer
XCHOUX asked on 02/23/04 - Bin Laden Boxed In

According to a London Daily paper(sorry, I fogot the name),Bin Laden has been boxed into small area in the lawless region in south west Pakistan/Afghanistan. He has about 50 supporters who will die and fight for him as his protection.

Comments.....

tomder55 answered on 02/23/04:

If he is still alive ,he is boxed into a 10 square mile area(still rather large area) of some of the most rugged terrain in the world . He has guides who can show him every goat trail in the area ,and he is on the Pakistan border(we will still need the cooperation of our "allies " in Pakistan. Supposedly he has been using a satellite phone (duh).The U.S. I fear made a big mistake disclosing the information . They also reported that a Sec.Ops group that was working in Iraq has now been dispatched to Afghanistan to track him down . Mullah Omar is also in the net . Get us a two-fer out of the deal.

ETWolverine rated this answer Excellent or Above Average Answer
purplewings rated this answer Excellent or Above Average Answer
XCHOUX rated this answer Excellent or Above Average Answer

Question/Answer
ETWolverine asked on 02/12/04 - Back to the question of intelligence---

A few weeks ago, Itsdb asked a question regarding a statement on the Democratic Underground that Democrats (or at least those at Democratic Underground) are more intelligent than Republicans.

At the time, I answered by pointing out the intellectual accomplishments of various members of the Republican leadership.

I have since found out something new about President Bush that is of interest.

We all knew that President Bush received his BA from Yale. Democrats have argued that he only got into Yale because his father was a legacy, and that he was too stupid to get in on his own. This is, of course, pure poppycock, and the fact that Bush was a fighter pilot, and therefore had to be well-schooled in physics and aerodynamics (neither of which are easy subjects) and probably some mechanical and electrical engineering as well, ought to prove that Bush is not a stupid man.

But there is another point that I was unaware of.

Did anyone here know that Bush also got his MBA from Harvard. And he definitely didn't get into one of the toughest business schools in the country because his father was a legacy at Harvard, because he wasn't.

Furthermore, did any of you know that President Bush is the first President with an MBA? Other Preidents have had advanced degrees, of course. Lincoln, for instance, was an attorney. But Bush is the first with an MBA.

Stupid? With a BA from Yale, an MBA from Harvard, and a license to fly fighter planes? I don't friggin' think so.

The MBA explains a lot, though. It explains his strong understanding of economic and business concepts that escape so many otherwise brilliant leaders. And it explains why Bush seems to run the country like a business. He doesn't look to the past, and he doesn't point fingers. Like a good executive, he tries to solve problems, and he looks at the big picture and the long-term view. He may make mistakes, but he is decisive, and when he discovers a mistake, he generally acts to correct it.

In general, he acts the way I would expect a business leader to act. And that clearly stems both from his past political experience and his MBA training. (And probably a little bit from the 'fighter-jock' that he was in the past. Fighter-jocks are among the most self-assured people around, and so are successful businessmen.)

Elliot

tomder55 answered on 02/12/04:

Mark Rozell, a political scientist at Catholic University in Washington, D.C., says that if Democrats think making fun of Bush's intellect is the key to unseating him, they had better think again. He notes that three of the best-rated presidents of the 20th century Franklin Roosevelt, Harry Truman and Ronald Reagan were not known for their intellect. Two who were Herbert Hoover and Jimmy Carter were generally viewed as flops.

"Americans don't look for deep-thinking intellectuals in their presidents. They look for strong leadership and trust," Rozell said. "Bush comes across as an amazingly self-confident man who knows who he is and what he believes. Democrats better learn to deal with it." A case can be made that President Bush's strength as president derives from his lack of sophistication. There are no shaded pixels in his worldview, only solid colors--particularly black and white.

ETWolverine rated this answer Excellent or Above Average Answer

Question/Answer
ETWolverine asked on 02/12/04 - Back to the question of intelligence---

A few weeks ago, Itsdb asked a question regarding a statement on the Democratic Underground that Democrats (or at least those at Democratic Underground) are more intelligent than Republicans.

At the time, I answered by pointing out the intellectual accomplishments of various members of the Republican leadership.

I have since found out something new about President Bush that is of interest.

We all knew that President Bush received his BA from Yale. Democrats have argued that he only got into Yale because his father was a legacy, and that he was too stupid to get in on his own. This is, of course, pure poppycock, and the fact that Bush was a fighter pilot, and therefore had to be well-schooled in physics and aerodynamics (neither of which are easy subjects) and probably some mechanical and electrical engineering as well, ought to prove that Bush is not a stupid man.

But there is another point that I was unaware of.

Did anyone here know that Bush also got his MBA from Harvard. And he definitely didn't get into one of the toughest business schools in the country because his father was a legacy at Harvard, because he wasn't.

Furthermore, did any of you know that President Bush is the first President with an MBA? Other Preidents have had advanced degrees, of course. Lincoln, for instance, was an attorney. But Bush is the first with an MBA.

Stupid? With a BA from Yale, an MBA from Harvard, and a license to fly fighter planes? I don't friggin' think so.

The MBA explains a lot, though. It explains his strong understanding of economic and business concepts that escape so many otherwise brilliant leaders. And it explains why Bush seems to run the country like a business. He doesn't look to the past, and he doesn't point fingers. Like a good executive, he tries to solve problems, and he looks at the big picture and the long-term view. He may make mistakes, but he is decisive, and when he discovers a mistake, he generally acts to correct it.

In general, he acts the way I would expect a business leader to act. And that clearly stems both from his past political experience and his MBA training. (And probably a little bit from the 'fighter-jock' that he was in the past. Fighter-jocks are among the most self-assured people around, and so are successful businessmen.)

Elliot

tomder55 answered on 02/12/04:

Mark Rozell, a political scientist at Catholic University in Washington, D.C., says that if Democrats think making fun of Bush's intellect is the key to unseating him, they had better think again. He notes that three of the best-rated presidents of the 20th century Franklin Roosevelt, Harry Truman and Ronald Reagan were not known for their intellect. Two who were Herbert Hoover and Jimmy Carter were generally viewed as flops.

"Americans don't look for deep-thinking intellectuals in their presidents. They look for strong leadership and trust," Rozell said. "Bush comes across as an amazingly self-confident man who knows who he is and what he believes. Democrats better learn to deal with it." A case can be made that President Bush's strength as president derives from his lack of sophistication. There are no shaded pixels in his worldview, only solid colors--particularly black and white.

ETWolverine rated this answer Excellent or Above Average Answer

Question/Answer
XCHOUX asked on 02/08/04 - Meet The Press This Morning

President Bush made a rare Q&A appearance---on 'Meet The Press' this morning with Tim Russert. Tim asked some hard questions and Bush looked terrible. He was all puffy faced and his answers were canned and terrible. HE was a mess.

What the heck is going on? How could the powers that be let him out there like that. He looked and acted like a drunk on the morning after.

Bush is going to have to get it together or he doesn't stand a chance in NOvember.

Comments?

tomder55 answered on 02/09/04:

just a little add on to my comments this morning .I have now re:read the text again.In spite of the appearance ,this is where Bush succeeded in the interview (unfortunately you have to read the text to do the dissemination .By appearance he looked unsure. )

He left a clear, impression that he based his decision to invade Iraq on the best intelligence available, the same intelligence and conclusions held by the prior administration. Russert repeatedly asked questions presupposing our prewar intelligence was in err, like "knowing what you know now," or "since Saddam apparently had no weapons," or "Don't we need iron-clad, 100% certain intelligence."What Bush needed to do better is establish the absurdity of such thinking. A president cannot Monday morning quarterback national policy. He's got to make decisions based on the information he has available to him not on hypothetical and what ifs .

He deflected the AWOL charge. Before Russert could follow-up Bush had already made it a conversation about whether National Guardsmen were true servicemen. Bush essentially charged his opponents with questioning the legitimacy of NG service. He also should have pointed out that N.Y.Times has officially cleared Bush of the charge that the Boston Globe made ,and that his former commander of the Guard now recants what he told the Globe .

"I'm a wartime President". The more he reminds the electorate that we're at war, the less comfortable voters are going to be changing horses midstream. Bush perched himself above "politics" on numerous occasions .He also said that he will continue to tell the American people what he intends to do ,and follow thru on it .this was a shot at Kerry who seems to change positions in a Clintonesque way.

On the economy ,he correctly pointed out that he inherited a lot of the problems ,and that the attack of 9-11 ,and the subsequent wars have hurt the economy also . He was unapologetic in the course he has taken to correct the economy .Time will tell if he is correct.

ETWolverine rated this answer Excellent or Above Average Answer
XCHOUX rated this answer Excellent or Above Average Answer

Question/Answer
XCHOUX asked on 02/08/04 - Meet The Press This Morning

President Bush made a rare Q&A appearance---on 'Meet The Press' this morning with Tim Russert. Tim asked some hard questions and Bush looked terrible. He was all puffy faced and his answers were canned and terrible. HE was a mess.

What the heck is going on? How could the powers that be let him out there like that. He looked and acted like a drunk on the morning after.

Bush is going to have to get it together or he doesn't stand a chance in NOvember.

Comments?

tomder55 answered on 02/09/04:

I read the text this morning .It comes off alot better than the inteview looked . It is obviously not his strong suit and should limit live interviews and press conferences. The problem is that in doing so,the administration appears stealth and secretative.This is a time I think where they would be better served ,and better serve the nation by being up front .

XCHOUX rated this answer Excellent or Above Average Answer

Question/Answer
XCHOUX asked on 02/05/04 - I have been thinking,

and that is always dangerous, but anyway, I want to know what you think of my idea.

I have been thinking about different ways to lower the rate of crime in some catagories..robbery, battery, whatever.

Anyway, I was thinking how Rudi Guiliani lowered the rate of crime in NYC by picking up the vagrants washing car windows, etc.

OK, so, I was thinking that federal and state and local governments not charge tax on cigarettes and alcohol. If the cost of these items were quite low, marginal people wouldn't have trouble(financially) purchasing them.

I really think this would help lower the crime rate.

BUT, billlions in lost revenue. What to tax to replace the revenue?

Think about it, makes sense, doesn't it?

Chou

tomder55 answered on 02/06/04:

He did deal with quality of life issues ,but he had no problem taking on the big crooks also .As a Fed.Prosecutor he did a good job taking on both the Mafia ,and Wall Street corruption . He made N.Y.C. a better place to be .

There is so much hypocracy in the legality of cigarettes and booze that it is not worth talking about.Suffice it to say that I would sell both tax free if the consumer would sign a waver saying that my money doesn't have to pay for the health costs and care of the user.

excon rated this answer Excellent or Above Average Answer
XCHOUX rated this answer Excellent or Above Average Answer

Question/Answer
ETWolverine asked on 01/29/04 - Is Dean done?

I have heard people saying that if Dean didn't come within 10 percentage points of first place in New Hampshire after his poor showing in Iowa, his campaign is finished. Given that all the results I have seen show Dean finishing 13 points behind Kerry, does this mean that Dean is, for all intents and purposes out of the race? What's your opinion?

Elliot

tomder55 answered on 01/29/04:

no I don't think so ,

His anti-Washington populism is attractive to many of the voters in the mid-West and the rust belt regions . He will now begin to attack the inconsistancies of Kerry that have been flossed over because Dean was the front-runner;so will the other candidates .

Exit polls showed that the Dems were looking for a candidate who is electable. Hello ! Bush can't wait to slap the ole Mass. Liberal label on Kerry ,and bring up both his association with Dukakis ,and the fact that Kennedy has been at every Kerry rally. (did you happen to catch the irony of Kennedy introducing to the crowd someone who Kerry SAVED from drowning ?). Historically only 3 sitting members of Congress (2 Senators and one member of the House) have ever been elected as President. Govenors have a much better track record ;so do former Generals.



He spent alot of money ,but his campaign still gets plenty via internet contributions .Kerry will be in trouble financially unless there is a big change in contributions .He cannot rely forever on his wife's pocket change .


Finally ,I am rooting for a brokered convention . I want to see these guys doing these candidate debates for a couple of more months ,and hopefully see a few more melt-downs .

elgin_republicans rated this answer Excellent or Above Average Answer
ETWolverine rated this answer Excellent or Above Average Answer

Question/Answer
excon asked on 01/25/04 - Borders? We don't need no stinkin borders.


Ok, Mr. Conservative, you want to talk about borders? Bring it on (I like that phrase). We dont need no stinking borders!

Borders are obsolete. Theyre relics from the past. Weve outgrown our need for them, as they no longer provide the security that they once did. Our northern and southern boundaries are nothing more than imaginary lines anyway. The problem is, they reside in our imaginations much more so than in the rest of the worlds. So, we continue to pretend that our borders are secure.

But theyre not, really. Get a map out and take a look. Ask yourself, do we look safe from intrusion? Your own sensibilities tell you that we are not. Im constantly blown away by people who think we stop penetration - any penetration, on any border. And, I do mean that literally. I appear to be saying it over and over again, but the message is not getting through. We cant stop it no matter what.

Now, I know that youve got the entire federal government telling you that our borders are secure. GW said so the other night. So, I dont blame you if you believe it. But, security is only a state of mind. How you feel about your security has nothing whatsoever to do with how secure you really are. For your own safety, I suggest that you not buy that bill of goods.

In one of my many incarnations, I was in the security business. Some of my customers bought top of the line, state of the art, security systems. And some just bought a sign and planted it in their yard. Is a sign good enough? I dunno. I suggest that all our efforts at securing ourselves, are tantamount to planting a sign in our yard.

Well then, you say, lets really lock it up. Lets put troops along the border. Are you still looking at the map? Thats 3,000 miles of unguarded border to the north. Aside from the desert in the southwest (a pretty formative guard, in and of itself - that doesnt stop anybody), weve got 1,500 miles of unguarded border there. And, what of our coastlines? Are you still looking at the map?

If youre still not convinced, tell me why the Israelis cant guard their borders. What, theyre not tough enough? You think walls, razor wire and armed guards offer security? Tell me why our prisons are full of drugs? If borders work, why did we invade Iraq? If borders work, why are we afraid of the N. Koreans?

Some of you diehards will scoff and say, well if we really, really, really get tough, we can protect ourselves. But, we cant. Thats the reality. Take another look at the map, and lets do something different.

Besides, I dont want to live in a country thats locked up. Been there done that.

Im reminded of a story emanating from my old prison days. I never could understand the administrations response to bad behavior. When one convict broke the rules, they punished everybody. When somebody got caught in the broom closet with his girlfriend on visiting day (a perfectly natural thing for a con to do), they would stop contact visits for everybody. If there was a fight, wed all be locked down for a few days. If they discovered a batch of hooch, or some contraband, theyd lock us down again. So, instead of punishing the guys who did it, we all would get punished. I never got that.

And thats what happened to us after 9/11. We know who did it, but who got punished? Us! They gave us The Patriot Act a law akin to, and the beginning of locking us down for real. Well I, for one, dont like living in a locked down country. I like the freedoms we have (I mean had). And, if its freedom that Bin Laden hates, then we played right into his hands, because the Patriot Act restricts freedom. It tells the world what the USA really stands for. Less freedom - not more. Bush responded exactly like Bin Laden would have liked.

Furthermore, instead of hiding what were doing to the people in Gitmo, we should be holding them up as a shining example of how the US Justice system works even (no especially) for those we hate. They should be charged and tried in public with all the rights we cherish so much. I know, these people arent Americans. So what? We cant covet these rights for ourselves. Theyre in-alienable, after all meaning everybody has them, automatically, whether we like it or not. If our system is good then its good no matter who the bad guy is, or how bad he is. From my viewpoint, it should be even better the worse the bad guy is.

Ok, back to borders. Lets see, weve discovered borders dont stop job seekers from coming in. Since we cant stop job seekers from coming in, then presumably, we cant stop terrorists either. Plus, borders dont stop our jobs from going overseas. Indians write very good software for half the cost, and theyre very, very busy.

Now, I know what borders used to do, but they dont do it anymore. Globalization is a fact. The US of A is no longer predominantly white. It will get browner even still. Christianity will no longer be the major religion (Ill bet that scares the pants off some of you). The melding of cultures is happening. The distinctions between people will most likely disappear in the next century (if we make it). Now, you can argue whether these things are good or bad, but they're happening nonetheless.

So, what do borders do thats good? I cant think of a thing. Theyre obsolete. As a matter of fact they hold us back. Borders are a relics from the past that prevent us from moving no leading into the future.

Well, there is one thing you can say about borders - they cause good wars.

excon

tomder55 answered on 01/27/04:

just like your answer to the drug problem ;don't confront it ;surrender

ETWolverine rated this answer Excellent or Above Average Answer
excon rated this answer Excellent or Above Average Answer

Question/Answer
ETWolverine asked on 01/16/04 - Bush's Immigration Plan

Hello everyone.

I'm intersted in hearing your opinions on Bush's plan to give citizenship to 'undocumented' immigrants who are otherwise law-abiding, productive members of society? What do you think?

Here's my opinion, for what it's worth:

I think he's dead wrong on this issue. I believe that Bush's plan rewards illegal behavior and denigrates all those naturalized citizens who obeyed the rules. I think that the security risks associated with his plan are high. What it says is that if an illegal immigrant can manage to hide for long enough in this country, get and hold a job under false pretenses and with false application information, and not get arrested, he can become a citizen. I am against the plan.

On the other hand, I want to point out 2 facts.

1) This is not a case of Bush playing up to a particular constituancy in an election year. Bush has held this beliefe since his early days as Governor of Texas. This is not a policy switch for him. I may not agree with it, but Bush has been consistant in his policies and political beliefs.

2) While I think it's a bad policy, I believe that it is a BRILLIANT political move. It steals a plank right out from under the platforms of several of the Democratic candidates, including Dean, who has said that he believes in 'earned citizenship' for illegal immigrants. And the Democrats can hardly disagree with this plan, since they have been agitating for just such a plan for years now. It gives Bush another hook for the moderate Demacrats, a constituency the Democratic party seems to be losing.

Elliot

tomder55 answered on 01/16/04:

The amnesty in 1986 was supposed to fox the illegal problem . Since then an estimated 8-12 million more have snuck across the border . This new version of amnesty will open the flood gates further ,and bring what his amigo Vincente Fox wants ;an open unguarded border . If he had any intention of securing the border, than I would feel a little better about the plan ;but he wont . Further, it just is not fair to the many folks who in an effort to play by the rules have been denied a timely access to the American Dream while at the same time rewarding those who broke the law. One more thing. This plan ,a guest worker plan ,similar to the one that the Saudis have will drive the final nail into the melting pot theory. There will be a permanent sub class dejur in the United States ,workers that will be living and working in this country in limbo under some kind of temporary worker status,not citizens and not guaranteed the basic rights of a citizen . I have no problem with an amnesty that documents them ,and requires them to apply for resident status ,with a reasonable time frame to either apply for citizenship or to leave .At this point it would be almost impossible to round them all up and to export them ,and since they are in the country anyway ,I just as soon have them working.

Under Bush's plan they become beholden to a business sponsor. Does this make them indentured ? What happens when they go for another job;do they have to reapply ? When they are now guest workers will they now get the full rights of a worker in this country ? What is to prevent employers from hiring the next wave of illegals that cross the border ?

You point out that Bush has always courted the Hispanic vote .That is correct ,but it will not help him . Hispanics for the most part vote Dem.(except Cuban nationals in Fla.) ,and will continue to do so . Look at California .Once a Repubican fortress it is now a permanent Dem. State (forget Arnold ,that is an exception due to a particularily incompetent Dem.).In 2000 N.J. was a swing State . The demographics have changed there so much that I doubt Bush will win it (even if he does I don't see a Republican Govenor in its future). Also you are right about him undermining a Dem. plank,much like he did with the medicare entitlement. This was a bad one to take. The big issues will be the economy ,and national security . He has just delivered a message that the U.S. is not serious about security . What good is checking fingerprints of travelers into the country ,when anyone wishing us harm could be smuggled in via Mexico? This is a bad plan and I hope it fails in the deabates in Congress.

ETWolverine rated this answer Excellent or Above Average Answer
purplewings rated this answer Excellent or Above Average Answer

Question/Answer
excon asked on 01/10/04 - New?


Ok, Mr. Conservative, lets talk. But first, lets get our terminology straight. I am conservative. You could label me, as you have, an anarchist or libertarian, however I am neither. Were I to label myself, and thats quite hard to do, I would call myself a Constitutionalist. The Libertarians are a political party of which I am not a member, however, I share some political philosophy with them.

I am a conservative in the sense that it is my wish to conserve the Republic as it was Constitutionally created. Every change in the Constitution, beyond the Bill of Rights, since then has been changed due to a liberal reading of the document. If being conservative means that I believe in small government and no redistribution of the wealth, then Im a conservative. In my view, both the Republican and Democratic parties are liberal, and I stand way to the right of them.

The Democrats want to take your money and spend it on social programs. The Republicans want to take your money and spend it on, as weve discussed, police programs and un-needed wars. Both of them take my money and spend it on programs that I dont agree with. So, theyre both equally liberal in my view.

I like the term Robert Ringer came up; Demopublican.

If the Demopublicans didnt open the cash windows to their friends in the insurance industry, medical care would still be affordable for all. But since that didnt happen, and the insurance companies have their hooks so far into us that itll never change, YES, If theyre going to take my money, then spend it for something that I now cant afford medical care!

The original Constitution called for the creation of only one (1) bureaucracy, and thats the post office. For the first 100 years of this country, thats the way it was No federal power whatsoever. Every other creation beyond that is some liberals idea, of a way to get at the cash. And during the first 100 years this countrys industrial engine got lit and it hasnt gone out - yet.

Then in the 1850s the Supreme Court opened the door when it allowed the formation of the Interstate Commerce Commission, and the walls came tumbling down. What we have today is not what the framers intended.

So, if you support the status quo then you are a liberal, because liberals rule the roost.

excon

tomder55 answered on 01/14/04:

interesting discussion . I would like to point out that it doesn't seem to me to be a question of ideology . I find that traditional Conservatives abandon the States Rights mantle as often as Liberals take it up . I first noticed this in the 2000 election where the Conservatives in the Supreme Court ruled against a States Rights argument during Bush v.Gore.

Now when social issues like Gay Marriages are raised ,it is the Conservatives who have been endorsing a Federal solution to the issue ,while Democrats like Dean are declaring their support to these State decisions . Conservatives now "control the levers of power in Washington, and have a clear policy agenda as well as the tools to implement it," said Timothy Conlan, a George Mason University professor who studies the relationship between states and the federal government.Liberal justices appear sympathetic to the argument that states should have the right to enact stricter levels of state-church separation than mandated by the federal Constitution.
Liberals in the past looked to the federal government to protect the civil rights of blacks, conservatives have asked the court to protect them from violations of their religious liberties by state courts and legislatures.


"It would take a great deal of self-discipline for them to say: 'This is the policy we seek, but the way to implement it goes against our philosophy, so therefore we won't do it,'" Conlan said. "I call it political pragmatism."



Excon , do you think that a limited Gvt. would be successful in the industrial age ?
Seems to me that people here in the States enjoy unprecedented freedoms and the social programs as you call them are pretty much asked for by the people .The people want improvements in education ,and in health care . They want the gvt. to provide for the general welfare ,to make and enforce laws that protect their wealth ,health ,and security .Even as Bush has expanded things like Medicare ,and the Federal role in education ,his critics for the most part have said that he did not go far enough . The Buchanan's and the voices of the Lbertarians have been heard barely above a wisper.

ETWolverine rated this answer Excellent or Above Average Answer
excon rated this answer Excellent or Above Average Answer

Question/Answer
ETWolverine asked on 12/12/03 - TOMDER55 --- McCain-Feingold and modern campaign finance.

Hey Tom.

My take on McCain-Feingold is that most candidates in the future will do what Dean has done. That is, they'll skirt the whole issue by not taking matching government funds.

The only time hard- and soft-money issues come into play with McCain-Feingold is if you want to receive matching funds from the government. The government matches up to a certain level of campaign contributions (2% I think but I could be off), but only if you play by their rules.

In the old days, it made sense for a politician to play by those rules, because it meant free money. But with the limitations of McCain-Feingold in terms of limtations on campaing financing from a single source as well as limitations on soft money, it no longer makes sense. It costs more in terms of campaign dollars for a campaing to comply with the fund matching rules than he is getting in matching funds. What good is getting $1,000,000 in matching funds if you have to turn down $5,000,000 to get it? You're at a net loss of $4 million.

To get around this, Dean (and most of the other Dems in the presidential race have followed suit) has not filed for matching funds. That way he can get money from lots of different sources, co-mingle hard and soft money, and not worry about airing soft-money ads within 60 days of the election.

We have entered a new era of campaign finance... one in which the rules only apply to those who need to follow them and are most hurt by them, and in in which the rules do not apply to those who simply wish to ignore them and can afford to do so. If you've got money, you don't need to follow McCain Feingold. If you don't have money and need matching funds, McCain Feingold limits your ability to raise additional funds.

McCain Feingold may have "reformed" something, but it wasn't the level playing field of campaign finance. Now the field is even more skewed than ever toward the rich. Another example of political tinkering with established law at its best... or worst.

Elliot

tomder55 answered on 12/13/03:

It was a nice try ,but it fails on one basic premise ;every American has a right to be heard ,and money in politics is the way to be heard ,that is the reality . The right to influence the outcome of elections is a very basic part of Free Speech. Even restricting how much a candidate can spend hurts the 1st Amendment. I understand inherent dangers of corruption that big money can cause ,but that could be minimized by a reform bill that would simply demand FULL and INSTANT DISCLOSURE of where the money comes from .

ETWolverine rated this answer Excellent or Above Average Answer

Question/Answer
ETWolverine asked on 12/10/03 - Howard Dean --- Pro-terrorist and Anti-Israel

Howard Dean promised that if he is elected president, the United States will no longer support Israel the way it has in the past under both democratic and Republican presidents. In his own words he will insist that the United States be "even handed". This is a term regularly employed by Arafat and his coterie of adherents that means to be anti-Israel!!

Governor Dean made these comments on CNN on September 10,2003 on the Wolf Blitzer show. He has repeated those words since.

If this were not enough, Governor Dean on that same show characterized the Hamas terrorists as "soldiers". For the first time since 9/11 we have someone running for the office of President of the United States calling terrorists soldiers.

I urge you that if you have any love for America and Israel you should not and cannot vote for Howard Dean for the office of President.

This coming election may very well be one of the most important in decades. Please pass this message on to as many of your family and friends as you can.

tomder55 answered on 12/11/03:

What people seem to be missing is that Gore did exactly what Clinton wants him to do. If a Dem runs and (on the off chance) wins against Bush, Hillary can't challenge in 2008, and will have to wait till 2012. But if the Dems lose in 2004, she can challenge a non-incumbant Republican in 2008,and have a good shot at winning.
------------------------------------------

I don't think that is the issue . I think Gore fully expects Dean to lose the general election ,but is hoping to wrestle the party chairmanship away from the Clintons control ( Terry McAuliffe). Gore ,with a Dean nomination will appoint a party chair ,and be in a better position to take on Hillary in 08.

ETWolverine rated this answer Excellent or Above Average Answer

Question/Answer
ETWolverine asked on 12/10/03 - Howard Dean --- Pro-terrorist and Anti-Israel

Howard Dean promised that if he is elected president, the United States will no longer support Israel the way it has in the past under both democratic and Republican presidents. In his own words he will insist that the United States be "even handed". This is a term regularly employed by Arafat and his coterie of adherents that means to be anti-Israel!!

Governor Dean made these comments on CNN on September 10,2003 on the Wolf Blitzer show. He has repeated those words since.

If this were not enough, Governor Dean on that same show characterized the Hamas terrorists as "soldiers". For the first time since 9/11 we have someone running for the office of President of the United States calling terrorists soldiers.

I urge you that if you have any love for America and Israel you should not and cannot vote for Howard Dean for the office of President.

This coming election may very well be one of the most important in decades. Please pass this message on to as many of your family and friends as you can.

tomder55 answered on 12/10/03:

I dropped out of the Democrat party this year ,mostly due to the positions they took on the Iraq war . The only one I thought was salvagable was Joe Lieberman ;now it looks like Gore stabbed him in the back.(Ironic in that Gore always said that he wanted to wait out the recall in Florida to have all the voters counted because only voters should decide an election ;now it appears he is content to annoint a nominee before a single vote is cast ......but I digress).

I have noted in the last few days that columists like Bill Krystol have been sounding the warning that Dean has a good chance to win the election . Maybe he is just trying to sound a wakeup call to Bush ,but I doubt we will see any less than a Goldwater type defeat this year for him ,as he continues to out "Left" his opponents . Things would need to go sour in a hurry for him to have a chance . The economy needs to have a major reversal in the recovery ,and unmentionable disasters would need to happen in foreign affairs .

Lately Dean has resorted to trying to out bizzare John Kerry . While Kerry is riding motorcycles ,and swearing up a storm ,Dean has been making like a caller to the Art Bell radio show ,and promoting a conspiracy theory that Bush knew in advance about 9-11 . This is irresponsible ,as well as calling terrorists (who blow up pizza parlors in Israel)soldiers is .It is my firm belief that both men have taken leave of their senses ,and would be a danger to the country in the oval office .

AliMcJ rated this answer Excellent or Above Average Answer
ETWolverine rated this answer Excellent or Above Average Answer
stevehaddock rated this answer Above Average Answer

Question/Answer
excon asked on 11/08/03 - Drug War

Hello experts:

There is a video airing on the news showing a high school under siege by local narcs guns drawn and all. Its frightening. No drugs were found.

To me, this video shows the horrors of a war declared upon its own people. I see a resemblance to Kent State. I suppose others of you (perhaps my friend, the Padre) will see something else, entirely.

Personally, Im ashamed and outraged at the actions of my government. Arent you?

excon

tomder55 answered on 11/09/03:

Well ,I can say that South Carolina is not where I live ,and I don't think that police in my district would act in such a disgraceful manner.It was wrong;end of story. I do not care if as the principle claimed that drug activity was increasing in the school. The point is;I would not want my daughter to be treated in such manner.Therefore ,no student should be treated as as suspect unless there is compelling evidence of their involvment.

Certainly a drug sweep could've been conducted without the strong -arm tactics,especially the handcuffing of some of the students.

excon rated this answer Excellent or Above Average Answer

Question/Answer
chekhovToo asked on 10/30/03 - North Korea

Do you think that North Korea a threat to stability?

tomder55 answered on 11/02/03:

Here are some of the sources regarding N.Korean hardware sales to Middle Eastern countries

From BBC August 29,2003
Last week the White
House was reported to
be imposing sanctions
on a North Korean company for selling missile
parts to Yemen, although the sanctions are
largely symbolic as there is no trade
relationship between the US and Pyongyang.
http://news.bbc.co.uk/2/hi/asia-pacific/2222661.stm

From Word Tribune August 26,2003
On
Saturday, Yemeni President Ali Abdullah Saleh confirmed that Sanaa acquired
Scud missiles from North Korea. Saleh said the United States did not impose a
military ban on Yemen.

"We have bought those missiles and this is a legitimate right of Yemen," Saleh
said.

U.S. officials said North Korea was also believed to have shipped missile
components to several Middle East states over the last two years. They included
such countries as Egypt, Iran, Libya and Syria.

http://216.26.163.62/2002/ss_nkorea_08_26.html


also from World Tribune May 16,2003

Western intelligence sources said Pyongyang increased its exports in 2001 and
2002 with missile sales to such countries as Egypt, Iran, Libya, Pakistan Syria and
Yemen. They said the exports included components, technology as well as entire
Scud-based systems.

The report of the increase in North Korean missile exports comes as the Bush
administration has launched an effort to halt Pyongyang's missile sales. U.S.
officials said the effort would focus on an international embargo imposed on North
Korea with the help of the United Nations Security Council.

The increase in missile exports was first seen in 2001, the sources said, when
North Korea completed several deals with Middle East clients, the sources said.
Missile sales were said to have further increased in 2002.

The sources said North Korean missile exports averaged fewer than $500 million in
the 1990s. They said the exports began to increase at the end of the 1990s with
Pyongyang's aid to Iran's Shihab-3 intermediate-missile program.

http://216.26.163.62/2003/ea_nkorea_05_16.html

The Isaac and Mildred Brochstein Fellow in Peace and
Security
James A. Baker III, Institute for Public Policy
Rice University

In recent years, North Korea has contributed to the
destabilization of the Middle East, more so, perhaps, than any
other country. It has been supplying ground-to-ground missiles
to a number of countries in the region. It has started setting up
missile production lines in a number of countries such as Syria
and Libya. It has extended the range of the missiles, improved
their precision and increased the carrying capacity of their
warheads. North Korea signed an agreement with Egypt for
the delivery of 50 No Dong missile engines (medium range),
but this agreement was canceled under pressure from
Washington. At the height of the Iraqi crisis, North Korea sold
Scud Missiles to Yemen. That consignment was intercepted
on its way from North Korea by Spanish and American naval
forces. Various intelligence services are convinced that
Yemen has procured these missiles in N. Korea for Iraq.

http://www.rice.edu/projects/baker/Pubs/workingpapers/israel/israel_4.html

chekhovToo rated this answer Excellent or Above Average Answer

Question/Answer
denberg asked on 10/31/03 - American Quangos

In Britain we have QUANGOs. I can't help wondering:

1/ What is your equivalent, and how do people get onto them?
2/ What are the most notorious ones in the USA?

Many thanks,

Paul,
London.

tomder55 answered on 11/01/03:

quango (quasi~nongovernmental~organiztion)
Chartered by the government to fulfill a governmental purpose more
entrepreneurially than government can. Government guaranteed mortgage
companies like Ginnie Mae(Government National Mortgage Association)
Fannie Mae and Freddie Mac are the closest the US has to the quango,
In the U.S. most of the Quangos functions are fulfilled by private agencies.

denberg rated this answer Excellent or Above Average Answer
HANK1 rated this answer Excellent or Above Average Answer

Question/Answer
ETWolverine asked on 10/31/03 - Bush and the Economy

The Democrats and Liberals have been calling Bush the worst president on the Economy since Herbert Hoover. (Dick Gephardt said it first, but every Democrat has agreed with it since then.) They have accused him of giving tax breaks that favor the rich. And they have argued that his 'reckless' tax cuts will bankrupt the country and create a deficit that our grandkids will be paying for. They call Bush's economic plan a disaster.

The US Commerce Department seems to disagree with them. Yesterday they reported that the economy grew a shocking 7.2% in the 3rd quarter of 2003. That is the largest single quarter growth since 1984.

Interestingly enough, the growth in 1984 and the growth in 3rd quarter 2003 were caused by the same thing... large tax cuts that put more cash back in hands of individuals and businesses, which in turn primed the economic pump, increasing the demand for goods and increasing the number of people employed to create those goods. 1984 was just 2 years after Reagan's historic tax cuts of 1982. 2003 is just 2 years after the Bush tax cuts of 2001. 2 years seems to be the magic number... it takes 2 years for the effect of tax cuts to be seen in the economy.

Bottom line, the economy is improving. Jobless claims are down, capital expenditure is up, and the demand for durable goods has increased. As the capital expenditures and durable goods expenditures continue, new jobs will be created. All because people had more money to spend. And this is because Bush infused about $29 Billion into the economy with his child tax credit increase ($400 per minor child x 72 million children = $28.8 billion). Similarly, by refunding $300 per family in 2001, he infused the economy with approximately $15 billion. So over 2 years, Bush put about $45 billion of spendable cash back into the hands of the people. Which has induced $45 billion worth of spending. Which will shortly result in $45 billion worth of job creation. Which will in turn result in even MORE spending by people who now have jobs that didn't before. Which will increase wealth all around. Which means that there will be more collected by the government in taxes. Which means that the "Bush deficit" will be wiped away.

Tax cuts work. They are not reckless. They stop recessions, and start growth.

I suggest we all stop calling Bush "the worst President on the economy since Hoover". That is clearly not the case. Hoover's economy contracted more than 25% during his presidency. The "Bush recession" lasted only 8 months (March to November 2001) and began only six weeks after he took office, which means that it was clearly not his fault in the first place. AND the recession ended only 2 months after 9-11. Since then, the economy has been stable through the 2nd quarter of 2003. And in the third quarter, a 7.2% growth rate, far exceeding even the most optimistic predictions. Even if growth drops to only 4% in the coming year, there will still be enough growth to create hundreds of thousands of new jobs.

Start giving Bush his due. The guy's a lot smarter than anyone (myself included) gave him credit for. Strong on military and security issues, and effective on economic issues.

Elliot

tomder55 answered on 10/31/03:

Of course, demand-side pessimists think these tax-cut effects will wear off in another quarter or two. But they'll be proven wrong because they do not understand the incentive model that rewards additional work, risk, and investment when all three get paid more after-tax. This inflation-free economic boom, bolstered by continued technology advances in broadband and Internet services, could last seven to ten years just like the prior two booms of the 1980s and 1990s. Just think of it.
Larry Kudlow


just so as we do not make the same mistakes of the 60s and decide to not pay for the wars we wage .

ETWolverine rated this answer Excellent or Above Average Answer

Question/Answer
chekhovToo asked on 10/30/03 - North Korea

Do you think that North Korea a threat to stability?

tomder55 answered on 10/30/03:

I don't think there is any doubt that N.Korea is a threat to it's neighbors ,and to American and global stability . In 2002 alone North Korea sold $60 million worth of Scud missiles and missile parts to Iraq, Iran, Syria and Yemen . It is estimated that they have sold 400 ballistic missles to the Middle East since the mid 80s .Does anyone doubt that when they finish developing a nuclear capability that they would shy away from selling it to the highest bidder ?

Kim Jong-il cannot be trusted His nuclear weapons program exists for one purpose: to threaten the international community and blackmail it into providing aid.Aid that never manages to make it to the N.Korean people btw.He operates a system of slave labour camps that can be compared only with the worst nightmares of Stalin's Gulag .Aid in the past has been used to further bolster his military capability . Negotiations will only go so far . Eventually ,they will fail ,and he will have to be dealt with .




chekhovToo rated this answer Excellent or Above Average Answer
excon rated this answer Excellent or Above Average Answer
stevehaddock rated this answer Above Average Answer

Question/Answer
ETWolverine asked on 10/27/03 - New York City Ballot Proposals --- Question #3

On 11/4/03, New York State will be voting in the general election. The big issue in New York City this election day is the proposal to establish a new electoral system for Mayor, Public Advocate, Comptroller, Borough President, and Council members. The current system calls for Party Primaries, wherein only members of a party can vote for members of that party. The front-runners of each party then run against each other in a general election. The new system would open up primaries to any registered voters, regardless of party affiliation. The idea is to create a non-partisan primary system.

Here is the text (from the NYC Campaign Finance Board's voters guide).

-----------------------------------
Official Text

This proposal would amend the City Charter to establish a new system of city elections for the offices of Mayor, Public Advocate, Comptroller, Borough President, and Council member. The September primary election would be open to all voters and all candidates, regardless of party membership or independent status. The top two vote getters would compete in the November general election. In both elections, candidates could indicate their party membership or independent status on the ballot. Candidates participating in the voluntary campaign finance program, which provides public campaign funding, could not accept contributions from political parties or party committees. The new system would replace the current system of political party nominations through primary elections in which only party members may vote. The changes would take effect after the 2005 Citywide election. Shall this proposal be adopted?
Official Summary (prepared by the Charter Revision Commission)

This proposal would make changes to the way the City conducts its elections for the offices of Mayor, Public Advocate, Comptroller, Borough President, and City Council Member. The proposal would provide for a single September primary election open to all eligible voters and candidates, and the top two vote getters in that election would compete in the November general election. The changes would take effect after the 2005 citywide election and apply to all elections, including special elections to fill vacancies in office.

How it Works Currently, candidates who are running for elected office compete against each other in party primary elections (followed in certain instances by runoff primaries). In these elections, which are held in September, each partys members cast ballots to decide the general election nominee of that party. Candidates may also be nominated for the general election through an independent nominating petition process. Under the proposed system, all candidates for an office would run against each other in the September primary election. Voters would be eligible to vote for any candidate, including a candidate who belongs to a different political party or is registered as an independent. Only the top two finishers in the primary election, regardless of their party or independent registration status, would compete in the November general election.

Who is Eligible to Vote Currently, only registered voters who are enrolled in political parties may vote in primary elections. Voters registered as independents cannot participate until the general election. Under the proposed system, all voters, including independent voters not enrolled in any party, would be eligible to participate in the primary election.

Ballot Access Currently, registered voters who carry or sign a petition to place a candidate on the ballot generally must be enrolled in the same party as the candidate petitioning to get on the ballot. Under the proposed system, all eligible registered voters, regardless of their party enrollment or independent status, would be permitted to carry and sign a petition to place a candidate on the ballot. In addition, the maximum number of petition signatures required would generally be the same maximum number that is currently provided for candidates to be placed on the primary election ballot. The new signature requirements would also apply to candidates in special elections to fill vacancies.

The Ballot Currently, the ballot lists a candidates party affiliation. Under the proposed system, candidates would be permitted, but not required, to list their party registration or independent status on the ballot.

Political Party Campaign Contributions Currently, all candidates, including those who participate in the Citys voluntary campaign finance program, which provides money from the City treasury to help fund candidates campaigns, are permitted to accept campaign contributions from political parties, up to specified limits, but candidates may not accept contributions from corporations or unregistered political committees. Under the proposed system, candidates who participate in the campaign finance program would also be prohibited from accepting campaign contributions from political parties. In addition, the Campaign Finance Board would have the authority to address party expenditures in relation to candidates participating in the Citys campaign finance program in order to protect the contribution and spending limits of the program under the new system of elections.

Voter Guide Currently, all candidates may place biographical summaries, campaign information, and a picture of themselves in the printed Voter Guide. This proposal would create a video Voter Guide that broadcasts candidates making brief statements on a municipal cable television channel.

Vacancies in Nominations Currently, if a candidate dies or a nomination otherwise becomes vacant, the candidates committee to fill vacancies generally chooses a successor to run in his or her place, if time allows. Under the new system, the next eligible top vote getter in the primary election could fill the vacancy and advance to the general election accordingly. If the vacancy could not be filled in that manner, then a committee to fill vacancies would fill it.

-------------------------------------

My question: what is your opinion of this proposal?

Elliot

tomder55 answered on 10/28/03:

This proposal is supported by Republicans who otherwise do not have a snowballs chance in hell of winning a local NYC election . The Democratic party has pretty much a dictatorial control over New York City ;notwithstanding the fact that the last 2 mayors have been elected from the Republican ticket [one was elected after a buffoon was running the city ,and the other needed to spend a rather large personal fortune to counter the Democrat machine]. The fact is that in general only Democrats under the current set up have a realisitic chance of winning an election there.Many consider the Democrat Primary as the general election .That system by it's nature disqualifies many talented and qualified candidates from presenting themselves to the electorate ; Not only Republicans ,but 3rd party and independent candidates as well . This is not a healthy situation ,and I favor trying any means possible to open up the process.

ETWolverine rated this answer Excellent or Above Average Answer
stevehaddock rated this answer Excellent or Above Average Answer

Question/Answer
Raiden936 asked on 10/23/03 - Senator running for President question.

Everybody keeps saying the Senator Clinton is going to run for the Presidency. However, she is in the middle of her 6 year term as a senator. How could she run for president if she won't be able to finish her 6 year term when she becomes the president?

tomder55 answered on 10/23/03:

Politicians resign from their positions frequently to run for ,or get appointed to another office . Then what happens is some executive ,like in this case the Govenor of N.Y. would appoint a successor .Sometimes special elections are held .

What Hillary would do if she considered a Presidential run would be to make her case to the N.Y. electorate ;[a sorta mea culpa for lying about finishing her term ,but she could serve N.Y. better by being Prez.yada-yada -yada]. Then she would run while she was still a Senator (fall back in case she loses so she could still be on the public payroll).She would only actually resign if she won .

Hey what do you expect from a carpet-bagger ?(at least she purchased a home in N.Y. Robby Kennedy took residence in a hotel, nad still became a N.Y. Senator)

Raiden936 rated this answer Excellent or Above Average Answer

Question/Answer
ETWolverine asked on 10/14/03 - This is one of the funniest political OP ED pieces I've seen in years.

Your comments are welcome.

Elliot
----------------
What Democrats believe
Rich Lowry

September 25, 2003

A presidential primary is a way for a political party to make up its mind. Through the process of nominating a candidate, a party figures out its stances on the new issues and what adjustments, if any, it will make in its positions on the old. So with that, through their collective rhetoric and actions, the 10 Democratic candidates have arrived at the outlines of a rough philosophy -- the credo of the Democrats of ཀ.

This credo is often nonsensical and hypocritical, but it is clearly discernible. The Democrats of ཀ believe:

That wars should be authorized, but never fought.

That the United Nations is the world's last, best hope, and every jot of its writ should always be respected, unless it inconveniences Saddam Hussein.

That nation-building is always a humanitarian and just cause, unless it is undertaken in Iraq.

That anyone who said Saddam had weapons of mass destruction prior to the war was lying, unless his or her name is Bill Clinton, Al Gore, Madeleine Albright, Bill Cohen, John Kerry or Joe Lieberman, or the person ever served in the Clinton cabinet or as a Democratic senator.

That French Foreign Minister Dominique de Villepin is always right.

That President Bush isn't devoting enough resources to the reconstruction of Iraq, and that -- in light of his $87 billion aid proposal -- he is devoting far too many resources to the reconstruction of Iraq.

That George Bush maneuvering the United States into war is an act of manipulative genius, and also is very stupid.

That (fill in blank with latest conflict here) is another Vietnam.

That the U.S. military is overextended -- and should be smaller.

That unilateral U.S. diplomatic pressure is always wrong, unless it is brought to bear on Israel.

That it is absolutely necessary for the cause of clean government for candidates to abide by the limits set by the presidential public-financing system, unless they -- like Kerry and Howard Dean -- have enough money not to.

That big money corrupts politics, unless it is big money raised by California Gov. Gray Davis.

That punch-card ballots are a travesty of justice, unless they elect a Democrat (as they did in California just one year ago).

That Bush is bankrupting the federal government, but is a tightfisted ogre for countenancing only a $400 billion new prescription-drug benefit.

That Bush is fiscally profligate, but isn't spending enough on education, "first responders," health care or anything else not called "defense."

That the nation cannot afford the pending retirement of the baby boomers, but the baby boomers should get more benefits for their pending retirements.

That Bush is responsible for an economic downturn that began before he was elected and that Clinton is responsible for an economic recovery that began before he was elected (here at last -- a kind of consistency!).

That small-business owners are the heart of the economy unless they succeed, at which point they become "the rich."

That it is evil to be rich, unless you got that way by marrying Teresa Heinz.

That it is wrong to be a millionaire, unless you got that way by suing people.

That the sons of the upper-crust Northeastern elite are always and everywhere out-of-touch, unless they are named Howard Dean.

That it is unseemly to mix military matters with politics, but you should vote for FORMER GENERAL Wesley Clark, and salute when you do so.

That a deranged candidate should not be elected president, unless he is named Bob Graham.

That no child should be left behind, unless it is in an urban public-school system.

That no child should be left behind, unless it is in the womb.

That the Patriot Act is denying Americans their liberties, and John Kerry, Joe Lieberman, John Edwards or Bob Graham should be elected president after having voted for it.

That deposing John Ashcroft would be preferable to deposing Mullah Omar.

That library records are sacred, but the Constitution -- a "living document" subject to manipulation by judges -- is not.

tomder55 answered on 10/14/03:

excellent ! on a posting in Spirituality I put some of the quotes of some Democrats mentioned above ,and their statements visa-vie WMD prior to the war . It is amusing to see the hypocracy.
............................................
............................................

That nation-building is always a humanitarian and just cause, unless it is undertaken in Iraq.

ie. Haiti ?
.............................................

'That Bush is bankrupting the federal government, but is a tightfisted ogre for countenancing only a $400 billion new prescription-drug benefit.'

or a paltry $15 Billion to combat AIDS in Africa


.............................................
'That wars should be authorized, but never fought.'

'That the Patriot Act is denying Americans their liberties, and John Kerry, Joe Lieberman, John Edwards or Bob Graham should be elected president after having voted for it.



Ironically Graham on top of voting for the Patriot Act also said after Congress authorized the use of force in Iraq in an overwhelmingly bipartisan vote that the war resolution did not go far enough,and was "too timid "'arguing that Congress should have broadened the resolution to give Bush even greater authority to attack international terrorists.


ETWolverine rated this answer Excellent or Above Average Answer
stevehaddock rated this answer Above Average Answer

Question/Answer
ETWolverine asked on 09/23/03 - Decision by the 9th Circuit Court of Appeals!!!

New information has appeared on the 9CCs website.

The en banc court has ruled that the district court was not wrong in its decision.

What does this mean?

A little background:

The case is called Soutwest Voter Registration v Shelly. The case was brought by several minority voter interest groups. The complaint (that punch-card ballots are a violaton of equal protection rules) was originally heard by a district court, which ruled that the case, while having some merit, did not have a liklihood of success. The district court, therefore, dnied the sought-after injuncton that would have postponed the recall election (as well as two initiatives on the ballot).

The plaintifs appealed the case to the 9th Circuit Court of Appeals, which ruled last week (by a 3 member panel) that there was indeed merit to the case, and that there was a possibility of success by the plaintifs in their complaint. They therefore ordered the postponement of the recall vote. However, they ordered a stay on their own ruling in order for higher authority to hear the case.

Today, the en banc court, after a 'limited review', reveresed the panel's decision, saying that the District Court was correct in its decision not to order the injunction or postpone the lection. They based their decision on three points:

1) The plaintiff, while showing a possibility of success in their case, did not show a high probability of success.

2) The plaintiff failed to show based on the merits of the case that there was a perponderance of hardship for the plainiffs

3) The election is already underway by dint of the fact that absentee voters have already cast their ballots, money has already been spent, funds have already been raised, campaigns have already begun getting out their messages, all with the expectation of an October 7th election date. Thus, postponing the election would cause undue hardship to the public. While they acknowledged that the punch-card ballots might also cause undue hardhip, that case has not been proven, and therefore, they agreed that the perponderance of the hardship would be caused by a postponement.

The Court noted that it was reluctant to enjoin an impending election in any case, and that enjoining an election that has already begun is unprecedented.

What this all means is that the election is still on, and last week's decision by the 3 judge panel no longer stands.

For more details, check out the 9th Circuit Court's website.

http://www.ce9.uscourts.gov/

Elliot

tomder55 answered on 09/24/03:

there is no guarantee that hi-tech balloting is the solution to this "equal rights" issue . Hi-tech balloting was as flawed in Florida last year as punch card ballots were. The answer, if there is an Equal Protection issue, is to mandate proven electoral methods. Placing a simple, mark beside the candidate of your choice on a simple paper ballot and then placing the ballot yourself into a sealed ballot box remains the simplest and safest method of voting.Optical-scanning technology, if necessary, can ensure the swift counting of the ballots if you do not trust hand counting . There would be alot fewer disputed ballots that way .

It was nonsense for them to claim that minorities were being discriminated by using puch card ballots ,and the 9th was correct in reversing the panels decision .

ETWolverine rated this answer Excellent or Above Average Answer

Question/Answer
ETWolverine asked on 09/15/03 - California Recall & the 9th Circuit Court

What the HECK!!!

This is from The New York Post's website.

-----
LOS ANGELES (AP) -- A federal appeals court postponed the Oct. 7 recall election Monday in a decision that threw what has already been a chaotic campaign into utter turmoil.

The 9th U.S. Circuit Court of Appeals ruled that the historic vote cannot proceed as scheduled because some votes would be cast using outmoded punch-card ballot machines. The decision applies to all the recall questions on the ballot, as well as two propositions.

The court withheld ordering the immediate implementation of its decision by one week to allow time for an appeal to the U.S. Supreme Court.

"Give us 24 hours," said Ted Costa, head of the Sacramento-based Peoples' Advocate, one of the groups that put the recall on the ballot, who said an appeal is certain.

A spokesman for Gov. Gray Davis said the governor will continue his campaign "until the issue is resolved in the courts," but supported the appeals court's ruling.

"Anything that leads to greater enfranchisement in California is something we support," said spokesman Peter Ragone.

Neither Davis nor Arnold Schwarzenegger, the leading Republican among the 135 replacement candidates, had an immediate reaction to the three-judge panel's ruling, which could force the U.S. Supreme Court to rule on another highly partisan political issue - one Democrats have said echoes the 2000 election in which the high court declared Republican George Bush the winner.

The ruling is likely to benefit Davis if the election is delayed to the next regularly scheduled primary, March 2.

The March presidential primary is expected to draw large numbers of Democratic voters, and the months until then would give Davis more time to address the state's problems and force Schwarzenegger into a longer campaign.

The decision came as the race's top names were enlisting big national stars in their campaigns.

Trying to soften his image with women voters, Schwarzenegger assured talk show host Oprah Winfrey on Monday that reports of a salacious, party-hard past were more tall tale than truth and do not bear on his run for California governor.

Davis was in Southern California with former President Clinton to dedicate the William Jefferson Clinton elementary school in the impoverished suburb of Compton, and the two had planned to later attend a fund raiser.

In the ruling Monday, the judges of the 9th Circuit, the nation's largest and most liberal federal appeals court, agreed with the American Civil Liberties Union that the punch-card voting machines still used in six California counties are prone to error.

The counties - Los Angeles, in addition to Mendocino, Sacramento, San Diego, Santa Clara and Solano - were already under a separate court order to replace them by the March primary, but the machines wouldn't be replaced in time for an Oct. 7 special election.

"In sum, in assessing the public interest, the balance falls heavily in favor of postponing the election for a few months," the court said.

Schwarzenegger and his wife, television journalist Maria Shriver, were in Chicago on Monday morning taping the season premiere of "The Oprah Winfrey Show." Polls had showed the Republican actor struggling to win over women in the recall race, and women are the show's primary audience.

Schwarzenegger, speaking before the court ruled, told Winfrey he was excited about the campaign. He also talked about old magazine articles that had resurfaced in which he described a sexually salacious, party-hard lifestyle and said they reflected a 1970s strategy to pump up interest in body building, the sport that made him famous.

"We really were out there doing crazy things. We were trying to get the attention," he said. "At that time I didn't think I was going to run for governor."

Before Schwarzenegger took the stage, Winfrey asked Shriver - a member of the Kennedy family - whether she had been brought up to look the other way if her husband was a womanizer.

"You know that ticks me off," Shriver said. "I am my own woman, I have not been bred to look the other way. I accept him with all his strengths and all his weaknesses, as he does me."

Davis, meanwhile, was scheduled to make a second day of campaign stops with Clinton, following a joint appearance Sunday at a predominantly black church in Los Angeles.

Clinton had spoken passionately against the recall during the Sunday service, mixing Scripture with politics at the First African Methodist Episcopal Church.

He repeated his party's theme that the recall election is part of a right-wing power grab, and said removing Davis could scare future officeholders away from making difficult choices.

"This is way bigger than him," Clinton said. "It's you I'm worried about. It's California I worry about. I don't want you to become a laughing stock, or a carnival, or the beginning of a circus in America where we throw people out for making tough decisions."

"Don't do this. Don't do this," he said. The congregation erupted in applause.

Davis was also scheduled to campaign this week with other prominent Democratic figures, including former Vice President Al Gore, the Rev. Jesse Jackson and several of the 2004 Democratic presidential candidates. It wasn't immediately clear Monday if those plans would change.

State Democrats on Saturday, in an emergency meeting, said they opposed the recall but endorsed Lt. Gov. Cruz Bustamante as a back up candidate if voters decided to recall Davis.



Associated Press Writer Erica Werner in Los Angeles and David Kravets in San Francisco contributed to this report.

---------------------------

Just so you are all aware, the specific counties that still use punchcard ballots in California are predominantly low-middle class minority neighborhoods.

By making this decision, the 9th Circuit Court (viewed as a predominantly Liberal court) has effectively said that people in those neighborhoods would be too easily confused by the punchcard system that has been used effectively for nearly a century.

So exactly which party is it that marginalizes minorities and says they're too stupid to vote? I don't see this coming from the Republican side of the floor.

Elliot

tomder55 answered on 09/16/03:

It is a fitting end to a non-sense election . I predicted this last month .

http://www.answerway.com/myquesv.php?category=194&msection=0&quesid=14142

I agree with voiceguy2000 . The courts have been legislating by judicial fiat for years . The only ones who complain are the ones whose ox is being Gore[d].

ETWolverine rated this answer Above Average Answer
stevehaddock rated this answer Average Answer

Question/Answer
Bobbye asked on 09/02/03 - Governor of CA?

Do you think Arnold will make it? Will he sweep it? Or do you feel that the Democrats will retain the Governorship? Just curious. Thanks. bobbye

tomder55 answered on 09/02/03:

he is certainly helped by a post-Clinton morality in politics .

I think that Bustamante and McClintock will emerge as the front-runners after the people of California come out of their fog .

ETWolverine rated this answer Excellent or Above Average Answer
stevehaddock rated this answer Above Average Answer
Bobbye rated this answer Excellent or Above Average Answer

exper   © Copyright 2002-2008 Answerway.org. All rights reserved. User Guidelines. Expert Guidelines.
Privacy Policy. Terms of Use.   Make Us Your Homepage
. Bookmark Answerway.